{"id": "18437", "image": [], "answer": "A", "solution": "null", "level": "高二", "question": "在 $\\triangle A B C$ 中, 已知 $a=1, b=2, C=60^{\\circ}$, 则 $c$ 等于 $(\\quad)$", "options": "A. $\\sqrt{3}$\nB. 3\nC. $\\sqrt{5}$\nD. 5", "subject": "解析几何", "analysis": ""} {"id": "18444", "image": [], "answer": "C", "solution": "null", "level": "高二", "question": "在 $\\triangle A B C$ 中, 已知 $a=2$, 则 $b \\cos C+c \\cos B$ 等于( )", "options": "A. 1\nB. $\\sqrt{2}$\nC. 2\nD. 4", "subject": "解析几何", "analysis": "解析 $b \\cos C+c \\cos B=b \\cdot \\frac{a^{2}+b^{2}-c^{2}}{2 a b}+c \\cdot \\frac{c^{2}+a^{2}-b^{2}}{2 a c}=\\frac{2 a^{2}}{2 a}=a=2$."} {"id": "18445", "image": [], "answer": "B", "solution": "null", "level": "高二", "question": "在 $\\triangle A B C$ 中, 已知 $b^{2}=a c$ 且 $c=2 a$, 则 $\\cos B$ 等于 $(\\quad)$", "options": "A. $\\frac{1}{4}$\nB. $\\frac{3}{4}$\nC. $\\frac{\\sqrt{2}}{4}$\nD. $\\frac{\\sqrt{2}}{3}$", "subject": "解析几何", "analysis": "解析 $\\because b^{2}=a c, c=2 a, \\therefore b^{2}=2 a^{2}, b=\\sqrt{2} a$,\n\n$\\therefore \\cos B=\\frac{a^{2}+c^{2}-b^{2}}{2 a c}=\\frac{a^{2}+4 a^{2}-2 a^{2}}{2 a \\cdot 2 a}=\\frac{3}{4}$."} {"id": "18446", "image": [], "answer": "B", "solution": "null", "level": "高二", "question": "在 $\\triangle A B C$ 中, $\\sin ^{2} \\frac{A}{2}=\\frac{c-b}{2 c}$ ( $a, b, c$ 分别为角 $A, B, C$ 的对应边), 则 $\\triangle A B C$ 的形状为( )", "options": "A. 正三角形\nB. 直角三角形\nC. 等腰直角三角形\nD. 等腰三角形", "subject": "解析几何", "analysis": "解析 $\\because \\sin ^{2} \\frac{A}{2}=\\frac{1-\\cos A}{2}=\\frac{c-b}{2 c}$,\n\n$\\therefore \\cos A=\\frac{b}{c}=\\frac{b^{2}+c^{2}-a^{2}}{2 b c} \\Rightarrow a^{2}+b^{2}=c^{2}$, 符合勾股定理.\n\n故 $\\triangle A B C$ 为直角三角形."} {"id": "18447", "image": [], "answer": "B", "solution": "null", "level": "高二", "question": "在 $\\triangle A B C$ 中, 已知面积 $S=\\frac{1}{4}\\left(a^{2}+b^{2}-c^{2}\\right)$, 则角 $C$ 的度数为 ( )", "options": "A. $135^{\\circ}$\nB. $45^{\\circ}$\nC. $60^{\\circ}$\nD. $120^{\\circ}$", "subject": "解析几何", "analysis": "$\\because S=\\frac{1}{4}\\left(a^{2}+b^{2}-c^{2}\\right)=\\frac{1}{2} a b \\sin C$,\n\n$\\therefore a^{2}+b^{2}-c^{2}=2 a b \\sin C, \\quad \\therefore c^{2}=a^{2}+b^{2}-2 a b \\sin C$.\n\n由余弦定理得: $c^{2}=a^{2}+b^{2}-2 a b \\cos C$,\n\n$\\therefore \\sin C=\\cos C$,\n\n$\\therefore C=45^{\\circ}$."} {"id": "18451", "image": [], "answer": "C", "solution": "null", "level": "高二", "question": "已知 $a 、 b 、 c$ 为 $\\triangle A B C$ 的三边长, 若满足 $(a+b-c)(a+b+c)=a b$, 则 $\\angle C$ 的大小为", "options": "A. $60^{\\circ}$\nB. $90^{\\circ}$\nC. $120^{\\circ}$\nD. $150^{\\circ}$", "subject": "解析几何", "analysis": "解析 $\\because(a+b-c)(a+b+c)=a b$,\n\n$\\therefore a^{2}+b^{2}-c^{2}=-a b$,\n\n即 $\\frac{a^{2}+b^{2}-c^{2}}{2 a b}=-\\frac{1}{2}$,\n\n$\\therefore \\cos C=-\\frac{1}{2}, \\therefore \\angle C=120^{\\circ}$."} {"id": "18457", "image": [], "answer": "C", "solution": "null", "level": "高二", "question": "在 $\\triangle A B C$ 中, 若 $2 \\cos B \\sin A=\\sin C$, 则 $\\triangle A B C$ 的形状一定是", "options": "A. 等腰直角三角形\nB. 直角三角形\nC. 等腰三角形\nD. 等边三角形", "subject": "解析几何", "analysis": "解析 $\\because 2 \\cos B \\sin A=\\sin C=\\sin (A+B)$,\n\n$\\therefore \\sin A \\cos B-\\cos A \\sin B=0$,\n\n即 $\\sin (A-B)=0, \\therefore A=B$."} {"id": "18459", "image": [], "answer": "D", "solution": "null", "level": "高二", "question": "$\\triangle A B C$ 的三边分别为 $a, b, c$ 且满足 $b^{2}=a c, 2 b=a+c$, 则此三角形是 ( )", "options": "A. 等腰三角形\nB. 直角三角形\nC. 等腰直角三角形\nD. 等边三角形", "subject": "解析几何", "analysis": "解析 $\\because 2 b=a+c, \\therefore 4 b^{2}=(a+c)^{2}$, 即 $(a-c)^{2}=0$,\n\n$\\therefore a=c . \\therefore 2 b=a+c=2 a . \\therefore b=a$, 即 $a=b=c$."} {"id": "18460", "image": [], "answer": "A", "solution": "null", "level": "高二", "question": "在 $\\triangle A B C$ 中, 角 $A, B, C$ 所对的边长分别为 $a, b, c$, 若 $C=120^{\\circ}$, $c=\\sqrt{2} a$, 则 ( )", "options": "A. $a>b$\nB. $a0, \\quad \\therefore a^{2}>b^{2}, \\quad \\therefore a>b$."} {"id": "19637", "image": [], "answer": "C", "solution": "null", "level": "高二", "question": "$\\triangle A B C$ 的内角 $A, B, C$ 的对边分别为 $a, b, c$, 若 $\\cos C=\\frac{2 \\sqrt{2}}{3}, b \\cos A+a \\cos B=2$,则 $\\triangle A B C$ 的外接圆的面积为 ( )", "options": "A. $4 \\pi$\nB. $8 \\pi$\nC. $9 \\pi$\nD. $36 \\pi$", "subject": "解析几何", "analysis": "解析: $: \\mathrm{b} \\cos \\mathrm{A}+\\mathrm{a} \\cos \\mathrm{B}=2$,\n\n$\\therefore$ 由余弦定理可得: $\\mathrm{b} \\times \\frac{b^{2}+c^{2}-a^{2}}{2 b c}+\\mathrm{a} \\times \\frac{a^{2}+c^{2}-b^{2}}{2 a c}=2$, 整理解得: $\\mathrm{c}=2$,\n\n又 $\\because \\cos C=\\frac{2 \\sqrt{2}}{3}$, 可得: $\\sin C=\\sqrt{1-\\cos ^{2} C}=\\frac{1}{3}, \\therefore$ 设三角形的外接圆的半径为 $\\mathrm{R}$, 则 $2 \\mathrm{R}=$ $\\frac{c}{\\sin C}=\\frac{2}{\\frac{1}{3}}=6$, 可得: $\\mathrm{R}=3, \\therefore \\triangle \\mathrm{ABC}$ 的外接圆的面积 $\\mathrm{S}=\\pi \\mathrm{R}^{2}=9 \\pi$.\n\n$2 . B$\n\n解析: $\\triangle \\mathrm{ABC}$ 中, $\\mathrm{A}=60^{\\circ}, \\mathrm{b}=1, S_{\\triangle A B C}=\\sqrt{3}, \\therefore \\frac{1}{2} \\mathrm{bcsinA}=\\frac{1}{2} \\times 1 \\times \\mathrm{c} \\times \\sin 60^{\\circ}=\\sqrt{3}$, 解得 $\\mathrm{c}=4$;\n\n$\\therefore \\mathrm{a}^{2}=\\mathrm{b}^{2}+\\mathrm{c}^{2}-2 \\mathrm{bc} \\cos \\mathrm{A}=1^{2}+4^{2}-2 \\times 1 \\times 4 \\times \\cos 60^{\\circ}=13, \\therefore \\mathrm{a}=\\sqrt{13} ; \\therefore \\frac{c}{\\sin C}=\\frac{a}{\\sin A}=\\frac{\\sqrt{13}}{\\frac{\\sqrt{3}}{2}}=\\frac{2 \\sqrt{39}}{3}$."} {"id": "19640", "image": [], "answer": "B", "solution": "null", "level": "高二", "question": "在 $\\triangle A B C$ 中, $A=60^{\\circ}, b=1, S_{\\triangle A B C}=\\sqrt{3}$, 则 $\\frac{c}{\\sin C}=(\\quad)$", "options": "A. $\\frac{8 \\sqrt{3}}{81}$\nB. $\\frac{2 \\sqrt{39}}{3}$\nC. $\\frac{26 \\sqrt{3}}{3}$\nD. $2 \\sqrt{7}$", "subject": "解析几何", "analysis": "$\\triangle \\mathrm{ABC}$ 中, $\\mathrm{A}=60^{\\circ}, \\mathrm{b}=1, S_{\\triangle A B C}=\\sqrt{3}, \\therefore \\frac{1}{2} \\mathrm{bcsinA}=\\frac{1}{2} \\times 1 \\times \\mathrm{c} \\times \\sin 60^{\\circ}=\\sqrt{3}$, 解得 $\\mathrm{c}=4$;$\\therefore \\mathrm{a}^{2}=\\mathrm{b}^{2}+\\mathrm{c}^{2}-2 \\mathrm{bc} \\cos \\mathrm{A}=1^{2}+4^{2}-2 \\times 1 \\times 4 \\times \\cos 60^{\\circ}=13, \\therefore \\mathrm{a}=\\sqrt{13} ; \\therefore \\frac{c}{\\sin C}=\\frac{a}{\\sin A}=\\frac{\\sqrt{13}}{\\frac{\\sqrt{3}}{2}}=\\frac{2 \\sqrt{39}}{3}$."} {"id": "19641", "image": [], "answer": "C", "solution": "null", "level": "高二", "question": "在 $\\triangle A B C$ 中, $A, B, C$ 的对边分别为 $a, b, c$, 若 $2\\left(a^{2}+c^{2}\\right)-a c=2 b^{2}$,则 $\\sin B=(\\quad)$", "options": "A. $\\frac{1}{4}$\nB. $\\frac{1}{2}$\nC. $\\frac{\\sqrt{15}}{4}$\nD. $D$", "subject": "解析几何", "analysis": "C 解析: 在 $\\triangle \\mathrm{ABC}$ 中, 由余弦定理得: $\\mathrm{a}^{2}+\\mathrm{c}^{2}-\\mathrm{b}^{2}=2 \\mathrm{accos} \\mathrm{B}$, 代入已知等式得: $2 \\mathrm{accos} \\mathrm{B}=\\frac{1}{2} \\mathrm{ac}$, 即 $\\cos \\mathrm{B}=\\frac{1}{4}, \\therefore \\sin \\mathrm{B}=\\sqrt{1-\\left(\\frac{1}{4}\\right)^{2}}=\\sqrt{1-\\frac{1}{16}}=\\frac{\\sqrt{15}}{4}$,"} {"id": "19642", "image": [], "answer": "C", "solution": "null", "level": "高二", "question": "$\\triangle A B C$ 中, 角 $A, B, C$ 的对边分别是 $a, b, c$, 已知 $b=c, a^{2}=2 b^{2}(1-\\sin A)$,则 $A=(\\quad)$", "options": "A. $\\frac{3 \\pi}{4}$\nB. $\\frac{\\pi}{3}$\nC. $\\frac{\\pi}{4}$\nD. $\\frac{\\pi}{6}$", "subject": "解析几何", "analysis": ":$\\because b=c, \\therefore a^{2}=b^{2}+c^{2}-2 b \\cos A=2 b^{2}-2 b^{2} \\cos A=2 b^{2}(1-\\cos A), \\because a^{2}=2 b^{2}(1-\\sin A)$, $\\therefore 1-\\cos \\mathrm{A}=1-\\sin \\mathrm{A}$, 则 $\\sin \\mathrm{A}=\\cos \\mathrm{A}$, 即 $\\tan \\mathrm{A}=1$, 即 $\\mathrm{A}=\\frac{\\pi}{4}$,"} {"id": "19643", "image": [], "answer": "C", "solution": "null", "level": "高二", "question": "已知 $\\triangle A B C$ 中, 内角 $A, B, C$ 的对边分别为 $a, b, c$, 若 $a^{2}=b^{2}+c^{2}-b c$,则 $\\angle A=(\\quad)$", "options": "A. $30^{\\circ}$\nB. $45^{\\circ}$\nC. $60^{\\circ}$\nD. $75^{\\circ}$", "subject": "解析几何", "analysis": "$\\because \\mathrm{a}^{2}=\\mathrm{b}^{2}+\\mathrm{c}^{2}-\\mathrm{bc}$, 可得: $\\mathrm{bc}=\\mathrm{b}^{2}+\\mathrm{c}^{2}-\\mathrm{a}^{2}, \\therefore \\cos \\mathrm{A}=\\frac{b^{2}+c^{2}-a^{2}}{2 b c}=\\frac{b c}{2 b c}=\\frac{1}{2} , \\because \\mathrm{A} \\in\\left(0,180^{\\circ}\\right)$, $\\therefore \\mathrm{A}=60^{\\circ}$."} {"id": "19644", "image": [], "answer": "D", "solution": "null", "level": "高二", "question": "在 $\\triangle A B C$ 中, 内角 $A, B, C$ 所对的边分别是 $a, b, c$, 若 $3 a=2 b$, 则 $\\frac{2 \\sin ^{2} B-\\sin ^{2} A}{\\sin ^{2} A}$ 的值为 ( )", "options": "A. $-\\frac{1}{9}$\nB. $\\frac{1}{3}$\nC. 1\nD. $\\frac{7}{2}$", "subject": "解析几何", "analysis": "$\\because 3 \\mathrm{a}=2 \\mathrm{~b}, \\therefore \\mathrm{b}=\\frac{3}{2} a$, 根据正弦定理可得 $\\frac{2 \\sin ^{2} B-\\sin ^{2} A}{\\sin ^{2} A}=\\frac{2 b^{2}-a^{2}}{a^{2}}=\\frac{2 \\times \\frac{94^{2}}{4}-a^{2}}{a^{2}}=\\frac{9}{2}-1=\\frac{7}{2}$,"} {"id": "19645", "image": [], "answer": "A", "solution": "null", "level": "高二", "question": "在 $\\triangle A B C$ 中, 内角 $A, B, C$ 所对的边分别是 $a, b, c$, 若 $c^{2}=(a-b)$ ${ }^{2}+6, C=\\frac{\\pi}{3}$, 则 $\\triangle A B C$ 的面积是 $(\\quad)$", "options": "A. $\\frac{3 \\sqrt{3}}{2}$\nB. $\\frac{9 \\sqrt{3}}{2}$\nC. $\\sqrt{3}$\nD. $3 \\sqrt{3}$", "subject": "解析几何", "analysis": "由 $c^{2}=(a-b)^{2}+6$, 可得 $c^{2}=a^{2}+b^{2}-2 a b+6$, 由余弦定理: $c^{2}=a^{2}+b^{2}-2 a b \\cos C=a^{2}+b^{2}-a b$,所以: $\\mathrm{a}^{2}+\\mathrm{b}^{2}-2 \\mathrm{ab}+6=\\mathrm{a}^{2}+\\mathrm{b}^{2}-\\mathrm{ab}$, 所以 $\\mathrm{ab}=6$; 则 $\\mathrm{S}_{\\triangle \\mathrm{ABC}}=\\frac{1}{2} \\mathrm{absinC}=\\frac{3 \\sqrt{3}}{2}$;"} {"id": "19646", "image": [], "answer": "A", "solution": "null", "level": "高二", "question": "在 $\\triangle A B C$ 中, 已知 $b=2, a=3, \\cos A=-\\frac{5}{13}$, 则 $\\sin B$ 等于()", "options": "A. $\\frac{8}{13}$\nB. $\\frac{9}{13}$\nC. $\\frac{10}{13}$\nD. $\\frac{11}{13}$", "subject": "解析几何", "analysis": "$\\because \\cos \\mathrm{A}=-\\frac{5}{13}, \\therefore \\sin \\mathrm{A}=\\sqrt{1-\\cos ^{2} A}=\\frac{12}{13}, \\because \\mathrm{b}=2, \\mathrm{a}=3$, 由正弦定理可得 $\\sin \\mathrm{B}=\\frac{b \\sin A}{a}=$ $\\frac{2}{3} \\times \\frac{12}{13}=\\frac{8}{13}$,"} {"id": "17736", "image": [], "answer": "D", "solution": "null", "level": "高二", "question": "已知 $\\sin \\alpha>\\sin \\beta$, 那么下列命题成立的是", "options": "A. 若 $\\alpha, \\beta$ 是第一象限角, 则 $\\cos \\alpha>\\cos \\beta$\n\nB. 若 $\\alpha, \\beta$ 是第二象限角, 则 $\\tan \\alpha>\\tan \\beta$\n\nC. 若 $\\alpha, \\beta$ 是第三象限角, 则 $\\cos \\alpha>\\cos \\beta$\n\nD. 若 $\\alpha, \\beta$ 是第四象限角, 则 $\\tan \\alpha>\\tan \\beta$", "subject": "解析几何", "analysis": "解析: 逐个象限判断, 只有第四象限适合.\n\n答案: D"} {"id": "17742", "image": [], "answer": "A", "solution": "null", "level": "高二", "question": "使 $\\sin x \\leqslant \\cos x$ 成立的 $x$ 的一个变化区间是", "options": "A. $\\left[-\\frac{3 \\pi}{4}, \\frac{\\pi}{4}\\right]$\nB. $\\left[-\\frac{\\pi}{2}, \\frac{\\pi}{2}\\right]$\nC. $\\left[-\\frac{\\pi}{4}, \\frac{3 \\pi}{4}\\right]$\nD. $[0, \\pi]$", "subject": "解析几何", "analysis": ""} {"id": "17744", "image": [], "answer": "D", "solution": "null", "level": "高二", "question": "如果 $M P, O M$ 分别是角 $\\frac{3 \\pi}{16}$ 的正弦线和余弦线, 那么下列结论正确的是 ( )", "options": "A. $M PO M>0$\nD. $O M>M P>0$", "subject": "解析几何", "analysis": ""} {"id": "17750", "image": [], "answer": "D", "solution": "null", "level": "高二", "question": "已知 $\\alpha$ 是第四象限角, $\\tan \\alpha=-\\frac{5}{12}$, 则 $\\sin \\alpha=$", "options": "A. $\\frac{1}{5}$\nB. $-\\frac{1}{5}$\nC. $\\frac{5}{13}$\nD. $-\\frac{5}{13}$", "subject": "解析几何", "analysis": "解析: 因为 $\\tan \\alpha=-\\frac{5}{12}$ ,所以 $\\frac{\\sin \\alpha}{\\cos \\alpha}=-\\frac{5}{12}$ ,\n\n所以 $\\cos \\alpha=-\\frac{12}{5} \\sin \\alpha$,\n\n代入 $\\sin ^{2} \\alpha+\\cos ^{2} \\alpha=1$ ,解得 $\\sin \\alpha= \\pm \\frac{5}{13}$ ,\n\n又 $\\alpha$ 是第四象限角,所以 $\\sin \\alpha=-\\frac{5}{13}$.\n\n答案: D"} {"id": "17755", "image": [], "answer": "A", "solution": "null", "level": "高二", "question": "已知 $3 \\cos ^{2} \\theta=\\tan \\theta+3$, 且 $\\theta \\neq k \\pi(k \\in \\mathbf{Z})$, 则 $\\sin \\theta \\cos \\theta=$", "options": "A. $-\\frac{1}{3} \\quad$ B. $\\frac{1}{3} \\quad$ C. $\\frac{2}{3}$\nD. $-\\frac{2}{3}$", "subject": "解析几何", "analysis": "解析:由题意可得 $3 \\cos ^{2} \\theta-3=\\tan \\theta$ ,即 $-3 \\sin ^{2} \\theta=\\frac{\\sin \\theta}{\\cos \\theta}$ ,由于 $\\theta \\neq k \\pi(k \\in \\mathbf{Z})$ ,所以 $\\sin \\theta \\cos$ $\\theta=-\\frac{1}{3}$.\n\n答案: A"} {"id": "17758", "image": [], "answer": "A", "solution": "null", "level": "高二", "question": "已知 $\\sin \\alpha-\\cos \\alpha=\\frac{4}{3}$, 则 $\\sin \\alpha \\cos \\alpha=$", "options": "A. $-\\frac{7}{18}$\nB. $-\\frac{1}{9}$\nC. $\\frac{1}{9}$\nD. $\\frac{7}{18}$", "subject": "解析几何", "analysis": "解析: $\\because \\sin \\alpha-\\cos \\alpha=\\frac{4}{3} , \\therefore(\\sin \\alpha-\\cos \\alpha)^{2}=\\frac{16}{9}$ ,\n\n即 $1-2 \\sin \\alpha \\cos \\alpha=\\frac{16}{9} , \\therefore \\sin \\alpha \\cos \\alpha=-\\frac{7}{18}$.\n\n答案: A"} {"id": "17759", "image": [], "answer": "B", "solution": "null", "level": "高二", "question": "若 $\\sin \\theta, \\cos \\theta$ 是方程 $4 x^{2}+2 m x+m=0$ 的两根, 则 $m$ 的值为", "options": "A. $1+\\sqrt{5}$\nB. $1-\\sqrt{5}$\nC. $1 \\pm \\sqrt{5}$\nD. $-1-\\sqrt{5}$", "subject": "解析几何", "analysis": "解析: 由题意知 $\\sin \\theta+\\cos \\theta=-\\frac{m}{2} , \\sin \\theta \\cdot \\cos \\theta=\\frac{m}{4}$.\n\n又 $(\\sin \\theta+\\cos \\theta)^{2}=1+2 \\sin \\theta \\cos \\theta$ ,\n\n$\\therefore \\frac{m^{2}}{4}=1+\\frac{m}{2}$ ,解得 $m=1 \\pm \\sqrt{5}$.\n\n又 $\\Delta=4 m^{2}-16 m \\geqslant 0 , \\therefore m \\leqslant 0$ 或 $m \\geqslant 4 ,$\n\n$\\therefore m=1-\\sqrt{5}$.\n\n答案: B"} {"id": "17765", "image": [], "answer": "C", "solution": "null", "level": "高二", "question": "若 $\\sin (\\pi-\\alpha)-\\cos (-\\alpha)=\\frac{1}{2}$, 则 $\\sin ^{3}(\\pi+\\alpha)+\\cos ^{3}(2 \\pi+\\alpha)$ 的值是", "options": "A. $-\\frac{3}{16} \\quad$ B. $\\frac{11}{16}$\n C. $-\\frac{11}{16}$\n D. $-\\frac{5}{16}$", "subject": "解析几何", "analysis": "解析 : 由已知, $\\sin (\\pi-\\alpha)-\\cos (-\\alpha)=\\sin \\alpha-\\cos \\alpha=\\frac{1}{2}$,所以 $(\\sin \\alpha-\\cos \\alpha)^{2}=\\left(\\frac{1}{2}\\right)^{2}$, 解得 $\\cos \\alpha \\sin \\alpha=\\frac{3}{8}$.\n\n故 $\\sin ^{3}(\\pi+\\alpha)+\\cos ^{3}(2 \\pi+\\alpha)=\\cos ^{3} \\alpha-\\sin ^{3} \\alpha=(\\cos \\alpha-\\sin \\alpha)\\left(\\cos ^{2} \\alpha+\\cos \\alpha \\sin \\alpha+\\sin ^{2} \\alpha\\right)=-\\frac{1}{2}$ $\\times\\left(1+\\frac{3}{8}\\right)=-\\frac{11}{16}$, 故选 C.\n答案: C"} {"id": "17766", "image": [], "answer": "C", "solution": "null", "level": "高二", "question": "已知函数 $f(x)=a \\sin (\\pi x+\\alpha)+b \\cos (\\pi x+\\beta)$, 且 $f(2015)=3$, 则 $f(2016)$ 的值是", "options": "A. -1\n B. -2\n C. -3\n D. 1", "subject": "解析几何", "analysis": "解析: $\\because f(2015)=a \\sin (2015 \\pi+\\alpha)+b \\cos (2015 \\pi+\\beta)=a \\sin (\\pi+\\alpha)+b \\cos (\\pi+\\beta)=-a \\sin \\alpha-$ $b \\cos \\beta=3$,\n\n$\\therefore a \\sin \\alpha+b \\cos \\beta=-3$.\n\n$\\therefore f(2016)=a \\sin (2016 \\pi+\\alpha)+b \\cos (2016 \\pi+\\beta)$\n\n$=a \\sin \\alpha+b \\cos \\beta=-3$.\n\n答案: C"} {"id": "17771", "image": [], "answer": "A", "solution": "null", "level": "高二", "question": "已知 $\\cos \\alpha=k, k \\in \\mathbf{R}, \\alpha \\in\\left(\\frac{\\pi}{2}, \\pi\\right)$, 则 $\\sin (\\pi+\\alpha)=$", "options": "A. $-\\sqrt{1-k^{2}}$\nB. $\\sqrt{1-k^{2}}$\nC. $-k$\nD. $\\pm \\sqrt{1-k^{2}}$", "subject": "解析几何", "analysis": "解析: 因为 $\\alpha \\in\\left(\\frac{\\pi}{2}, \\pi\\right)$, 所以 $\\sin \\alpha>0$, 则 $\\sin (\\pi+\\alpha)=-\\sin \\alpha=-\\sqrt{1-\\cos ^{2} \\alpha}=-\\sqrt{1-k^{2}}$,故选 A.\n\n答案: A"} {"id": "17772", "image": [], "answer": "D", "solution": "null", "level": "高二", "question": "已知 $\\cos (\\pi-\\alpha)=\\frac{\\sqrt{3}}{2}\\left(\\frac{\\pi}{2}<\\alpha<\\pi\\right)$, 则 $\\tan (\\pi+\\alpha)=$", "options": "A. $\\frac{1}{2}$\nB. $\\frac{\\sqrt{3}}{3}$\nC. $-\\sqrt{3}$\nD. $-\\frac{\\sqrt{3}}{3}$", "subject": "解析几何", "analysis": "解析: 法一: $\\cos (\\pi-\\alpha)=-\\cos \\alpha=\\frac{\\sqrt{3}}{2}$,\n\n$\\therefore \\cos \\alpha=-\\frac{\\sqrt{3}}{2}$.\n\n$\\because \\frac{\\pi}{2}<\\alpha<\\pi, \\quad \\therefore \\sin \\alpha>0, \\quad \\therefore \\sin \\alpha=\\sqrt{1-\\cos ^{2} \\alpha}=\\sqrt{1-\\frac{3}{4}}=\\frac{1}{2}$,\n\n$\\therefore \\tan (\\pi+\\alpha)=\\tan \\alpha=\\frac{\\sin \\alpha}{\\cos \\alpha}=-\\frac{\\sqrt{3}}{3}$.\n\n法二: 由 $\\cos \\alpha=-\\frac{\\sqrt{3}}{2}, \\frac{\\pi}{2}<\\alpha<\\pi$, 得 $\\alpha=\\frac{5}{6} \\pi$,\n\n$\\therefore \\tan \\alpha=-\\frac{\\sqrt{3}}{3}, \\therefore \\tan (\\pi+\\alpha)=\\tan \\alpha=-\\frac{\\sqrt{3}}{3}$.\n\n答案: D"} {"id": "17773", "image": [], "answer": "A", "solution": "null", "level": "高二", "question": "若 $\\alpha, \\beta$ 的终边关于 $y$ 轴对称, 则下列等式成立的是", "options": "A. $\\sin \\alpha=\\sin \\beta$\nB. $\\cos \\alpha=\\cos \\beta$\nC. $\\tan \\alpha=\\tan \\beta$\nD. $\\sin \\alpha=-\\sin \\beta$", "subject": "解析几何", "analysis": "解析: 法一: $\\because \\alpha, \\beta$ 的终边关于 $y$ 轴对称,\n$\\therefore \\alpha+\\beta=\\pi+2 k \\pi$ 或 $\\alpha+\\beta=-\\pi+2 k \\pi, k \\in \\mathbf{Z}$,\n\n$\\therefore \\alpha=2 k \\pi+\\pi-\\beta$ 或 $\\alpha=2 k \\pi-\\pi-\\beta, k \\in \\mathbf{Z}$,\n\n$\\therefore \\sin \\alpha=\\sin \\beta$.\n\n法二: 设角 $\\alpha$ 终边上一点 $P(x, y)$, 则点 $P$ 关于 $y$ 轴对称的点为 $P^{\\prime}(-x, y)$, 且点 $P$ 与点 $P^{\\prime}$到原点的距离相等, 设为 $r$, 则 $\\sin \\alpha=\\sin \\beta=\\frac{y}{r}$.\n\n答案: A"} {"id": "17774", "image": [], "answer": "D", "solution": "null", "level": "高二", "question": "已知 $\\sin (\\pi+\\theta)=-\\sqrt{3} \\cos (2 \\pi-\\theta),|\\theta|<\\frac{\\pi}{2}$, 则 $\\theta$ 等于", "options": "A. $-\\frac{\\pi}{6}$\nB. $-\\frac{\\pi}{3}$\nC. $\\frac{\\pi}{6}$\nD. $\\frac{\\pi}{3}$", "subject": "解析几何", "analysis": "解析: $\\because \\sin (\\pi+\\theta)=-\\sqrt{3} \\cos (2 \\pi-\\theta)$,\n\n$\\therefore-\\sin \\theta=-\\sqrt{3} \\cos \\theta$,\n\n$\\therefore \\tan \\theta=\\sqrt{3}, \\because|\\theta|<\\frac{\\pi}{2}, \\quad \\therefore \\theta=\\frac{\\pi}{3}$.\n\n答案: D"} {"id": "17776", "image": [], "answer": "A", "solution": "null", "level": "高二", "question": "若角 $\\alpha$ 的终边过点 $A(2,1)$, 则 $\\sin \\left(\\frac{3}{2} \\pi-\\alpha\\right)=$", "options": "A. $-\\frac{2 \\sqrt{5}}{5}$\nB. $-\\frac{\\sqrt{5}}{5}$\nC. $\\frac{\\sqrt{5}}{5}$\nD. $\\frac{2 \\sqrt{5}}{5}$", "subject": "解析几何", "analysis": "解析: 根据三角函数的定义可知 $\\cos \\alpha=\\frac{2}{\\sqrt{5}}=\\frac{2 \\sqrt{5}}{5}$,\n\n则 $\\sin \\left(\\frac{3}{2} \\pi-\\alpha\\right)=-\\cos \\alpha=-\\frac{2 \\sqrt{5}}{5}$, 故选 A.\n\n答案: A"} {"id": "17791", "image": [], "answer": "C", "solution": "null", "level": "高二", "question": "已知 $\\alpha$ 为锐角, 且 $2 \\tan (\\pi-\\alpha)-3 \\cos \\left(\\frac{\\pi}{2}+\\beta\\right)+5=0, \\tan (\\pi+\\alpha)+6 \\sin (\\pi+\\beta)-1=0$, 则 $\\sin \\alpha$的值是\\n\\n$", "options": "A. \\frac{3 \\sqrt{5}}{5} & \\text { B. } \\frac{3 \\sqrt{7}}{7} & \\text { C. } \\frac{3 \\sqrt{10}}{10} & \\text { D. } \\frac{1}{3}\\end{array}$", "subject": "解析几何", "analysis": "解析:由已知可得 $-2 \\tan \\alpha+3 \\sin \\beta+5=0$,\\n\\n$\\tan \\alpha-6 \\sin \\beta-1=0$, 可解得 $\\tan \\alpha=3$,\\n\\n又 $\\alpha$ 为锐角, 故 $\\sin \\alpha=\\frac{3 \\sqrt{10}}{10}$."} {"id": "17804", "image": [], "answer": "B", "solution": "null", "level": "高二", "question": "函数 $y=\\cos (\\sin x)$ 的最小正周期是", "options": "A. $\\frac{\\pi}{2}$\n B. $\\pi$\n C. $2 \\pi$\n D. $4 \\pi$", "subject": "解析几何", "analysis": "解析 : $\\cos [\\sin (x+\\pi)]=\\cos (-\\sin x)=\\cos (\\sin x)$,\n\n$\\therefore T=\\pi$, 故选 B.\n\n答案: B"} {"id": "17819", "image": [], "answer": "D", "solution": "null", "level": "高二", "question": "已知函数 $f(x)=f(\\pi-x)$, 且当 $x \\in\\left(-\\frac{\\pi}{2}, \\frac{\\pi}{2}\\right)$ 时, $f(x)=x+\\sin x$, 设 $a=f(1), b=f(2), c=f(3)$,则", "options": "A. $a0)$, 将 $y=f(x)$ 的图象向右平移 $\\frac{\\pi}{3}$ 个单位长度后, 所得的图象与原图象重合,则 $\\omega$ 的最小值等于 ()", "options": "A. $\\frac{1}{3}$\n B. 3\n C. 6\n D. 9", "subject": "解析几何", "analysis": "解析: 将 $y=f(x)$ 的图象向右平移 $\\frac{\\pi}{3}$ 个单位长度后得到 $y=\\cos \\left[\\omega\\left(x-\\frac{\\pi}{3}\\right)\\right]$, 所得图象与原图象重合,所以 $\\cos \\left(\\omega x-\\frac{\\pi}{3} \\omega\\right)=\\cos \\omega x$ ,则 $-\\frac{\\pi}{3} \\omega=2 k \\pi(k \\in \\mathbf{Z})$ ,得 $\\omega=-6 k(k \\in \\mathbf{Z})$. 又因为 $\\omega>0$ ,所以 $\\omega$ 的最小值为 6 ,故选 C.\n\n答案: C"} {"id": "17859", "image": [], "answer": "B", "solution": "null", "level": "高二", "question": "函数 $y=2 \\sin \\left(\\frac{x}{2}+\\frac{\\pi}{5}\\right)$ 的周期, 振幅依次是", "options": "A. $4 \\pi,-2$\nB. $4 \\pi, 2$\nC. $\\pi, 2$\nD. $\\pi,-2$", "subject": "解析几何", "analysis": "解析:周期 $T=\\frac{2 \\pi}{\\frac{1}{2}}=4 \\pi$ ,振幅为 2 ,故选 B.\n\n答案: B"} {"id": "17868", "image": [], "answer": "B", "solution": "null", "level": "高二", "question": "设函数 $f(x)=2 \\sin \\left(\\frac{\\pi}{2} x+\\frac{\\pi}{5}\\right)$. 若对任意 $x \\in \\mathbf{R}$, 都有 $f\\left(x_{1}\\right) \\leqslant f(x) \\leqslant f\\left(x_{2}\\right)$ 成立, 则 $\\left|x_{1}-x_{2}\\right|$ 的最小值为", "options": "A. 4\nB. 2\nC. 1\nD. $\\frac{1}{2}$", "subject": "解析几何", "analysis": "解析 : 函数 $f(x)$ 的周期 $T=4$. 因为对任意 $x \\in \\mathbf{R}$ ,都有 $f\\left(x_{1}\\right) \\leqslant f(x) \\leqslant f\\left(x_{2}\\right)$ 成立,所以 $\\left|x_{1}-x_{2}\\right| \\min$ $=\\frac{T}{2}=2$.\n\n答案: B"} {"id": "17873", "image": [], "answer": "D", "solution": "null", "level": "高二", "question": "动点 $A(x, y)$ 在圆 $x^{2}+y^{2}=1$ 上绕坐标原点沿逆时针方向匀速旋转, 12 秒旋转一周. 已知当时间 $t=0$ 时, 点 $A$ 的坐标是 $\\left(\\frac{1}{2}, \\frac{\\sqrt{3}}{2}\\right)$, 则当 $0 \\leqslant t \\leqslant 12$ 时, 动点 $A$ 的纵坐标 $y$ 关于 $t$ (单位:秒)的函数的单调递增区间是( )", "options": "A. $[0,1]$\n B. $[1,7]$\n C. $[7,12]$\n D. $[0,1]$ 和 $[7,12]$", "subject": "解析几何", "analysis": "解析:由已知可得该函数的周期 $T=12 , \\therefore \\omega=\\frac{2 \\pi}{T}=\\frac{\\pi}{6}$. 又 $\\because$ 当 $t=0$ 时, $A\\left(\\frac{1}{2} , \\frac{\\sqrt{3}}{2}\\right) , \\therefore y=$ $\\sin \\left(\\frac{\\pi}{6} t+\\frac{\\pi}{3}\\right) , t \\in[0 , 12]$. 可解得函数的单调递增区间是 $[0 , 1]$ 和[7,12].\n\n答案: D"} {"id": "17880", "image": [], "answer": "A", "solution": "null", "level": "高二", "question": "设 $y=f(t)$ 是某港口水的深度 $y$ (米)关于时间 $t$ (时)的函数, 其中 $0 \\leqslant t \\leqslant 24$. 下表是该港口某一天从 0 时至 24 时记录的时间 $t$ 与水深 $y$ 的关系:\n\n经长期观察, 函数 $y=f(t)$ 的图象可以近似地看成函数 $y=k+A \\sin (\\omega t+\\varphi)$ 的图象. 下面的函数中,最能近似表示表中数据间对应关系的函数是", "options": "A. $y=12+3 \\sin \\frac{\\pi}{6} t, t \\in[0,24]$\nB. $y=12+3 \\sin \\left(\\frac{\\pi}{6} t+\\pi\\right), t \\in[0,24]$\nC. $y=12+3 \\sin \\frac{\\pi}{12} t, t \\in\\left[\\begin{array}{ll}0,24\\end{array}\\right]$\nD. $y=12+3 \\sin \\left(\\frac{\\pi}{12} t+\\frac{\\pi}{2}\\right), t \\in[0,24]$", "subject": "解析几何", "analysis": "解析: 在给定的四个选项中,我们不妨代入 $t=0$ 及 $t=3$ ,容易看出最能近似表示表中数据间对应关系的函数是选项 A,故选 A.\n\n答案: A"} {"id": "18465", "image": [], "answer": "D", "solution": "null", "level": "高二", "question": "在 $\\triangle A B C$ 中, $\\sin A=\\sin B$, 则 $\\triangle A B C$ 是( )", "options": "A. 直角三角形\nC. 针角三角形\nB. 锐角三角形\nD. 等腰三角形", "subject": "解析几何", "analysis": ""} {"id": "18471", "image": [], "answer": "B", "solution": "null", "level": "高二", "question": "在 $\\triangle A B C$ 中, 若 $\\frac{a}{\\cos A}=\\frac{b}{\\cos B}=\\frac{c}{\\cos C}$, 则 $\\triangle A B C$ 是( )", "options": "A. 直角三角形\nB. 等边三角形\nC. 针角三角形\nD. 等腰直角三角形", "subject": "解析几何", "analysis": "解析 由正弦定理知: $\\frac{\\sin A}{\\cos A}=\\frac{\\sin B}{\\cos B}=\\frac{\\sin C}{\\cos C}$,\n\n$\\therefore \\tan A=\\tan B=\\tan C, \\therefore A=B=C$."} {"id": "18472", "image": [], "answer": "D", "solution": "null", "level": "高二", "question": "在 $\\triangle A B C$ 中, $\\sin A=\\frac{3}{4}, a=10$, 则边长 $c$ 的取值范围是( )", "options": "A. $\\left(\\frac{15}{2},+\\infty\\right)$\nB. $(10,+\\infty)$\nC. $(0,10)$\n\n$$\n\\text { D. }\\left(0, \\frac{40}{3}\\right]\n$$", "subject": "解析几何", "analysis": "解析 $\\because \\frac{c}{\\sin C}=\\frac{a}{\\sin A}=\\frac{40}{3}, \\therefore c=\\frac{40}{3} \\sin C$.\n\n$\\therefore 0<_{C} \\leqslant \\frac{40}{3}$."} {"id": "18473", "image": [], "answer": "A", "solution": "null", "level": "高二", "question": "在 $\\triangle A B C$ 中, $a=2 b \\cos C$, 则这个三角形一定是 ( )", "options": "A. 等腰三角形\nB. 直角三角形\nC. 等腰直角三角形\nD. 等腰或直角三角形", "subject": "解析几何", "analysis": "解析 由 $a=2 b \\cos C$ 得, $\\sin A=2 \\sin B \\cos C$, $\\therefore \\sin (B+C)=2 \\sin B \\cos C$,\n\n$\\therefore \\sin B \\cos C+\\cos B \\sin C=2 \\sin B \\cos C$,\n\n$\\therefore \\sin (B-C)=0, \\quad \\therefore B=C$."} {"id": "18474", "image": [], "answer": "B", "solution": "null", "level": "高二", "question": "在 $\\triangle A B C$ 中, 已知 $(b+c):(c+a):(a+b)=4: 5: 6$, 则 $\\sin A: \\sin B: \\sin C$ 等于 $(\\quad)$", "options": "A. $6: 5: 4$\nB. $7: 5: 3$\nC. $3: 5: 7$\nD. $4: 5: 6$", "subject": "解析几何", "analysis": "解析 $\\because(b+c):(c+a):(a+b)=4: 5: 6$,\n\n$\\therefore \\frac{b+c}{4}=\\frac{c+a}{5}=\\frac{a+b}{6}$.\n\n令 $\\frac{b+c}{4}=\\frac{c+a}{5}=\\frac{a+b}{6}=k(k>0)$,\n\n则 $\\left\\{\\begin{array}{l}b+c=4 k \\\\ c+a=5 k \\\\ a+b=6 k\\end{array}\\right.$, 解得 $\\left\\{\\begin{array}{l}a=\\frac{7}{2} k \\\\ b=\\frac{5}{2} k \\\\ c=\\frac{3}{2} k\\end{array}\\right.$\n\n$\\therefore \\sin A: \\sin B: \\sin C=a: b: c=7: 5: 3$."} {"id": "17888", "image": [], "answer": "D", "solution": "null", "level": "高二", "question": "一段圆弧的长度等于其所在圆内接正方形的边长, 则其圆心角的弧度数为", "options": "A. $\\frac{\\pi}{2}$\n B. $\\frac{\\pi}{3}$\n C. $\\sqrt{3}$\n D. $\\sqrt{2}$", "subject": "解析几何", "analysis": "解析:设圆内接正方形的边长为 $a$, 则该圆的直径为 $\\sqrt{2} a$,\n\n$\\therefore$ 弧长等于 $a$ 的圆弧所对的圆心角 $\\alpha=\\frac{l}{r}=\\frac{a}{\\frac{\\sqrt{2}}{2} a}=\\sqrt{2}$故选 $\\mathrm{D}$.\n\n答案: D"} {"id": "17891", "image": [], "answer": "B", "solution": "null", "level": "高二", "question": "已知弧度数为 2 的圆心角所对的弦长也是 2 , 则这个圆心角所对的弧长为 (", "options": "A. 2 B. $\\frac{2}{\\sin 1}$\nC. $2 \\sin 1$\nD. $\\sin 2$", "subject": "解析几何", "analysis": "解析: 扇形的半径 $r=\\frac{1}{\\sin 1}$, 因此弧长 $l=|\\alpha| \\cdot r=\\frac{2}{\\sin 1}$\n\n答案:B"} {"id": "17906", "image": [], "answer": "D", "solution": "null", "level": "高二", "question": "已知角 $\\alpha$ 的终边与单位圆交于点 $\\left(-\\frac{4}{5}, \\frac{3}{5}\\right)$, 则 $\\tan \\alpha$ 等于", "options": "A. $-\\frac{4}{3}$\nB. $-\\frac{4}{5}$\nC. $-\\frac{3}{5}$\nD. $-\\frac{3}{4}$", "subject": "解析几何", "analysis": ""} {"id": "17907", "image": [], "answer": "B", "solution": "null", "level": "高二", "question": "若角 $\\alpha$ 的终边过点 $P(2, \\sqrt{5})$, 点 $Q(-4 \\sqrt{5}, 10)$ 在角 $\\beta$ 的终边上, 则有 $(\\quad)$", "options": "A. $\\sin \\alpha<\\sin \\beta$\nB. $\\sin \\alpha=\\sin \\beta$\nC. $\\sin \\alpha>\\sin \\beta$\nD. 不能确定", "subject": "解析几何", "analysis": "解析: $\\because$ 角 $\\alpha$ 终边上的点 $P$ 到原点的距离 $r_{1}=\\sqrt{2^{2}+(\\sqrt{5})^{2}}=3, \\therefore \\sin \\alpha=\\frac{\\sqrt{5}}{3}$.\n\n$\\because$ 角 $\\beta$ 终边上的点 $Q$ 到原点的距离 $r_{2}=\\sqrt{(-4 \\sqrt{5})^{2}+10^{2}}=6 \\sqrt{5}, \\therefore \\sin \\beta=\\frac{10}{6 \\sqrt{5}}=\\frac{\\sqrt{5}}{3} . \\therefore \\sin$ $\\alpha=\\sin \\beta$.\n\n答案: B"} {"id": "17908", "image": [], "answer": "B", "solution": "null", "level": "高二", "question": "若 $\\alpha$ 是第三象限角, 则下列各式中不成立的是", "options": "A. $\\sin \\alpha+\\cos \\alpha<0$\nB. $\\tan \\alpha-\\sin \\alpha<0$\nC. $\\cos \\alpha-\\tan \\alpha<0$\nD. $\\tan \\alpha \\sin \\alpha<0$", "subject": "解析几何", "analysis": "解析: 在第三象限, $\\sin \\alpha<0, \\cos \\alpha<0, \\tan \\alpha>0$, 由此可知选 B.\n\n答案: B"} {"id": "17912", "image": [], "answer": "D", "solution": "null", "level": "高二", "question": "化简 $\\overrightarrow{A B}+\\overrightarrow{B D}-\\overrightarrow{A C}-\\overrightarrow{C D}=$", "options": "A. $\\overrightarrow{A D}$\nB. $\\overrightarrow{D A}$\nC. $\\overrightarrow{B C}$\nD. 0", "subject": "解析几何", "analysis": "解析: $\\overrightarrow{A B}+\\overrightarrow{B D}-\\overrightarrow{A C}-\\overrightarrow{C D}=(\\overrightarrow{A B}+\\overrightarrow{B D})-(\\overrightarrow{A C}+\\overrightarrow{C D})=\\overrightarrow{A D}-\\overrightarrow{A D}=0$.\n\n答案: D"} {"id": "17913", "image": [], "answer": "B", "solution": "null", "level": "高二", "question": "平面内有四边形 $A B C D$ 和点 $O$, 若 $\\overrightarrow{O A}+\\overrightarrow{O C}=\\overrightarrow{O B}+\\overrightarrow{O D}$, 则四边形 $A B C D$ 的形状是", "options": "A. 梯形\n B. 平行四边形\n C. 矩形\n D. 菱形", "subject": "解析几何", "analysis": ""} {"id": "17920", "image": [], "answer": "C", "solution": "null", "level": "高二", "question": "若 $|\\overrightarrow{B B}|=5,|\\overrightarrow{A C}|=8$, 则 $|\\overrightarrow{B C}|$ 的取值范围是", "options": "A. $[3,8]$\\n B. $(3,8)$\\n C. $[3,13]$\\n D. $(3,13)$\\n\\n", "subject": "解析几何", "analysis": "C"} {"id": "17915", "image": ["9017.jpg"], "answer": "A", "solution": "null", "level": "高二", "question": "如图, 在四边形 $A B C D$ 中, 设 $\\overrightarrow{A B}=\\boldsymbol{a}, \\overrightarrow{A D}=\\boldsymbol{b}, \\overrightarrow{B C}=\\boldsymbol{c}$, 则 $\\overrightarrow{D C}$ 等于\n\n", "options": "A. $\\boldsymbol{a}-\\boldsymbol{b}+\\boldsymbol{c}$\nB. $\\boldsymbol{b}-(\\boldsymbol{a}+\\boldsymbol{c})$\nC. $\\boldsymbol{a}+\\boldsymbol{b}+\\boldsymbol{c}$\nD. $\\boldsymbol{b}-\\boldsymbol{a}+\\boldsymbol{c}$", "subject": "解析几何", "analysis": "解析: $\\overrightarrow{A B}+\\overrightarrow{B C}=\\overrightarrow{A C} , \\therefore \\overrightarrow{D C}=\\overrightarrow{A C}-\\overrightarrow{A D}=\\boldsymbol{a}+\\boldsymbol{c}-\\boldsymbol{b}$.\n\n答案: A"} {"id": "17916", "image": [], "answer": "B", "solution": "null", "level": "高二", "question": "$\\overrightarrow{O P}-\\overrightarrow{Q P}+\\overrightarrow{P S}+\\overrightarrow{S P}$ 等于", "options": "A. $\\overrightarrow{Q P}$ B. $\\overrightarrow{O Q}$\nC. $\\overrightarrow{S P}$\nD. $\\overrightarrow{S Q}$", "subject": "解析几何", "analysis": "解析: $\\overrightarrow{O P}-\\overrightarrow{Q P}+\\overrightarrow{P S}+\\overrightarrow{S P}=\\overrightarrow{O P}+\\overrightarrow{P Q}=\\overrightarrow{O Q}$.\n\n答案: B"} {"id": "17922", "image": [], "answer": "D", "solution": "null", "level": "高二", "question": "若 $\\overrightarrow{O P_{1}}=\\boldsymbol{a}, \\overrightarrow{O P_{2}}=\\boldsymbol{b}, \\overrightarrow{P_{1} P}=\\lambda \\overrightarrow{P P}_{2}(\\lambda \\neq-1)$, 则 $\\overrightarrow{O P}=$", "options": "A. $\\boldsymbol{a}+\\lambda \\boldsymbol{b}$\n B. $\\lambda \\boldsymbol{a}+\\boldsymbol{b}$\n C. $\\lambda \\boldsymbol{a}+(1+\\lambda) b$\n D. $\\frac{\\boldsymbol{a}+\\lambda \\boldsymbol{b}}{1+\\lambda}$", "subject": "解析几何", "analysis": "解析: $\\because \\overrightarrow{P_{1} P}=\\lambda \\overrightarrow{P P}{ }_{2} , \\therefore \\overrightarrow{O P}-\\overrightarrow{O P}_{1}=\\lambda\\left(\\overrightarrow{O P}_{2}-\\overrightarrow{O P}\\right),(1+\\lambda) \\overrightarrow{O P}=\\lambda \\overrightarrow{O P}_{2}+\\overrightarrow{O P}_{1}, \\quad \\therefore \\overrightarrow{O P}=\\frac{\\boldsymbol{a}+\\lambda \\boldsymbol{b}}{1+\\lambda}$.\n\n答案: D"} {"id": "17923", "image": ["9018.jpg"], "answer": "C", "solution": "null", "level": "高二", "question": "如图, 在 $\\triangle A B C$ 中, $D, E$ 分别为 $A B, A C$ 的中点, $C D$ 与 $B E$ 交于点 $F$, 设 $\\overrightarrow{A B}=\\boldsymbol{a}, \\overrightarrow{A C}=\\boldsymbol{b}$, $\\overrightarrow{A F}=m \\boldsymbol{a}+n \\boldsymbol{b}$, 则 $m+n=(\\quad)$\n\n", "options": "A. 1\nB. $\\frac{4}{3}$\nC. $\\frac{2}{3}$\nD. $\\frac{5}{6}$", "subject": "解析几何", "analysis": "解析: $\\overrightarrow{A F}=m \\overrightarrow{A B}+n \\overrightarrow{A C}=m \\overrightarrow{A B}+2 n \\overrightarrow{A E}$ ,\n\n由 $B , F , E$ 三点共线,得 $m+2 n=1$ ,(1)\n\n$\\overrightarrow{A F}=m \\overrightarrow{A B}+n \\overrightarrow{A C}=2 m \\overrightarrow{A D}+n \\overrightarrow{A C}$\n\n由 $C, F, D$ 三点共线,得 $2 m+n=1$ ,\n\n(1) +(2)得 $3(m+n)=2, m+n=\\frac{2}{3}$.\n\n答案: C"} {"id": "17929", "image": [], "answer": "B", "solution": "null", "level": "高二", "question": "如果 $\\boldsymbol{e}_{1}, \\boldsymbol{e}_{2}$ 是平面 $\\alpha$ 内两个不共线的向量, 那么在下列各命题中不正确的有\n\n(1) $\\lambda e_{1}+\\mu e_{2}(\\lambda, \\mu \\in \\mathbf{R})$ 可以表示平面 $\\alpha$ 内的所有向量; (2)对于平面 $\\alpha$ 内的任一向量 $\\boldsymbol{a}$, 使 $\\boldsymbol{a}$ $=\\lambda e_{1}+\\mu e_{2}$ 的实数 $\\lambda, \\mu$ 有无数多对; (3)若向量 $\\lambda_{1} e_{1}+\\mu_{1} e_{2}$ 与 $\\lambda_{2} e_{1}+\\mu_{2} e_{2}$ 共线, 则有且只有一个实数 $\\lambda$, 使 $\\lambda_{1} e_{1}+\\mu_{1} e_{2}=\\lambda\\left(\\lambda_{2} e_{1}+\\mu_{2} e_{2}\\right)$; (4)若实数 $\\lambda, \\mu$ 使 $\\lambda e_{1}+\\mu e_{2}=0$, 则 $\\lambda=\\mu=0$.", "options": "A. (1)(2)\nB. (2)(3)\nC. (3)(4)\nD. (2)", "subject": "解析几何", "analysis": "解析: 由平面向量基本定理可知,(1)(4)是正确的;对于(2),由平面向量基本定理可知,一旦一个平面的基底确定,那么任意一个向量在此基底下的实数对是唯一的;对于(3),当两向量的系数均为零,即 $\\lambda_{1}=\\lambda_{2}=\\mu_{1}=\\mu_{2}=0$ 时,这样的 $\\lambda$ 有无数个. 故选 B.\n\n答案: B"} {"id": "17930", "image": [], "answer": "C", "solution": "null", "level": "高二", "question": "在 $\\triangle A B C$ 中, 点 $D$ 在 $B C$ 边上, 且 $\\overrightarrow{B D}=2 \\overrightarrow{D C}$, 设 $\\overrightarrow{A B}=\\boldsymbol{a}, \\overrightarrow{A C}=\\boldsymbol{b}$, 则 $\\overrightarrow{A D}$ 可用基底 $\\boldsymbol{a}, \\boldsymbol{b}$ 表示为", "options": "A. $\\frac{1}{2}(\\boldsymbol{a}+\\boldsymbol{b})$ B. $\\frac{2}{3} a+\\frac{1}{3} b$\nC. $\\cdot \\frac{1}{3} a+\\frac{2}{3} b$\nD. $\\frac{1}{3}(\\boldsymbol{a}+\\boldsymbol{b})$", "subject": "解析几何", "analysis": "解析: $\\because \\overrightarrow{B D}=2 \\overrightarrow{D C}, \\therefore \\overrightarrow{B D}=\\frac{2}{3} \\overrightarrow{B C}$.\n\n$\\therefore \\overrightarrow{A D}=\\overrightarrow{A B}+\\overrightarrow{B D}=\\overrightarrow{A B}+\\frac{2}{3} \\overrightarrow{B C}=\\overrightarrow{A B}+\\frac{2}{3}(\\overrightarrow{A C}-\\overrightarrow{A B})=\\frac{1}{3} \\overrightarrow{A B}+\\frac{2}{3} \\overrightarrow{A C}=\\frac{1}{3} \\boldsymbol{a}+\\frac{2}{3} \\boldsymbol{b}$.\n\n答案: C"} {"id": "17936", "image": [], "answer": "D", "solution": "null", "level": "高二", "question": "已知四边形 $A B C D$ 为平行四边形, 其中 $A(5,-1), B(-1,7), C(1,2)$, 则顶点 $D$ 的坐标为", "options": "A. $(-7,6)$\nB. $(7,6)$\nC. $(6,7)$\nD. $(7,-6)$", "subject": "解析几何", "analysis": "解析: 设 $D(x, y)$ ,由 $\\overrightarrow{A D}=\\overrightarrow{B C}$ ,得 $(x-5, y+1)=(2 ,-5) , \\therefore x=7 , y=-6 , \\therefore D(7 ,-$ 6).\n\n答案: D"} {"id": "17937", "image": [], "answer": "C", "solution": "null", "level": "高二", "question": "已知向量 $\\boldsymbol{a}=(x, 1)$, 向量 $\\boldsymbol{b}=\\left(-x, x^{2}\\right)$, 则向量 $\\boldsymbol{a}+\\boldsymbol{b}$", "options": "A. 平行于 $x$ 轴\n\nB. 平行于第一、三象限的角平分线\n\nC. 平行于 $y$ 轴\n\nD. 平行于第二、四象限的角平分线", "subject": "解析几何", "analysis": "解析 : $\\boldsymbol{a}+\\boldsymbol{b}=\\left(0 , 1+x^{2}\\right)$ 对应的点在 $y$ 轴上,所以此向量必平行于 $y$ 轴. 故选 C.\n\n答案: C"} {"id": "17941", "image": [], "answer": "B", "solution": "null", "level": "高二", "question": "在平行四边形 $A B C D$ 中, $A C$ 为一条对角线. 若 $\\overrightarrow{A B}=(2,4), \\overrightarrow{A C}=(1,3)$, 则 $\\overrightarrow{B D}=$", "options": "A. $(-2,-4)$\nB. $(-3,-5)$\nC. $(3,5)$\nD. $(2,4)$", "subject": "解析几何", "analysis": "解析: $\\because \\overrightarrow{A C}=\\overrightarrow{A B}+\\overrightarrow{A D}, \\therefore \\overrightarrow{A D}=\\overrightarrow{A C}-\\overrightarrow{A B}=(-1,-1), \\therefore \\overrightarrow{B D}=\\overrightarrow{A D}-\\overrightarrow{A B}=(-3 ,-5)$, 故选 B.\n\n答案: B"} {"id": "17942", "image": [], "answer": "D", "solution": "null", "level": "高二", "question": "设向量 $\\boldsymbol{a}=(1,-3), \\boldsymbol{b}=(-2,4), \\boldsymbol{c}=(-1,-2)$. 若表示向量 $4 \\boldsymbol{a}, 4 \\boldsymbol{b}-2 \\boldsymbol{c}, 2(\\boldsymbol{a}-\\boldsymbol{c}), \\boldsymbol{d}$的有向线段首尾相接能构成四边形, 则向量 $\\boldsymbol{d}$ 为", "options": "A. $(2,6)$\nB. $(-2,6)$\nC. $(2,-6)$\nD. $(-2,-6)$", "subject": "解析几何", "analysis": "解析: $\\because \\boldsymbol{a}=(1,-3), \\boldsymbol{b}=(-2,4), \\boldsymbol{c}=(-1,-2), \\therefore 4 \\boldsymbol{a}=(4,-12), 4 \\boldsymbol{b}-2 \\boldsymbol{c}=(-6$, 20),2(a-c)=(4,-2). 又 $\\because$ 表示 $4 \\boldsymbol{a} , 4 \\boldsymbol{b}-2 \\boldsymbol{c} , 2(\\boldsymbol{a}-\\boldsymbol{c}) , \\boldsymbol{d}$ 的有向线段首尾相接能构成四边形, $\\therefore 4 \\boldsymbol{a}+(4 \\boldsymbol{b}-2 \\boldsymbol{c})+2(\\boldsymbol{a}-\\boldsymbol{c})+\\boldsymbol{d}=0$. 解得 $\\boldsymbol{d}=(-2,-6)$. 故选 D.\n\n答案: D"} {"id": "17943", "image": [], "answer": "B", "solution": "null", "level": "高二", "question": "在 $\\triangle A B C$ 中, 点 $P$ 在 $B C$ 上, 且 $\\overrightarrow{B P}=2 \\overrightarrow{P C}$, 点 $Q$ 是 $A C$ 的中点. 若 $\\overrightarrow{P A}=(4,3), \\overrightarrow{P Q}=(1$, 5), 则 $\\overrightarrow{B C}=$", "options": "A. $(-2,7)$\nB. $(-6,21)$\nC. $(2,-7)$\nD. $(6,-21)$", "subject": "解析几何", "analysis": "解析: 如图, $\\because \\overrightarrow{Q C}=\\overrightarrow{A Q}=\\overrightarrow{P Q}-\\overrightarrow{P A}=(1,5)-(4,3)=(-3,2), \\therefore \\overrightarrow{P C}=\\overrightarrow{P Q}+\\overrightarrow{Q C}=(1,5)$ $+(-3,2)=(-2,7), \\therefore \\overrightarrow{B C}=3 \\overrightarrow{P C}=(-6,21)$.\n\n![](https://cdn.mathpix.com/cropped/2024_04_19_f934fd3200dbf4af6dd7g-154,.jpg?height=280&width=343&top_left_y=525&top_left_x=891)\n\n答案: $\\mathrm{B}$"} {"id": "17949", "image": [], "answer": "D", "solution": "null", "level": "高二", "question": "已知向量 $\\boldsymbol{a}=(-1,2), \\boldsymbol{b}=(1,-2 y)$. 若 $\\boldsymbol{a} / / \\boldsymbol{b}$, 则 $y$ 的值是", "options": "A. 2\nB. -2\nC. -1\nD. 1", "subject": "解析几何", "analysis": "解析:因为 $\\boldsymbol{a} / / \\boldsymbol{b}$ ,所以 $(-1) \\times(-2 y)=2 \\times 1$ ,解得 $y=1$.\n\n答案: D"} {"id": "17953", "image": [], "answer": "B", "solution": "null", "level": "高二", "question": "已知向量 $\\boldsymbol{a}=(1,2), \\boldsymbol{b}=(1,0), \\boldsymbol{c}=(3,4)$. 若 $\\lambda$ 为实数, $(\\boldsymbol{a}+\\lambda \\boldsymbol{b}) / / \\boldsymbol{c}$, 则 $\\lambda=$", "options": "A. $\\frac{1}{4}$\nB. $\\frac{1}{2}$\nC. 1\nD. 2", "subject": "解析几何", "analysis": "解析: 由题意可得 $\\boldsymbol{a}+\\lambda \\boldsymbol{b}=(1+\\lambda , 2)$. 由 $(\\boldsymbol{a}+\\lambda \\boldsymbol{b}) / / \\boldsymbol{c}$ ,得 $(1+\\lambda) \\times 4-3 \\times 2=0$ ,解得 $\\lambda=\\frac{1}{2}$.\n\n答案: B"} {"id": "17954", "image": [], "answer": "D", "solution": "null", "level": "高二", "question": "已知向量 $\\boldsymbol{a}=(1,-2),|\\boldsymbol{b}|=4|\\boldsymbol{a}|, \\boldsymbol{a} / / \\boldsymbol{b}$, 则 $\\boldsymbol{b}$ 可能是", "options": "A. $(4,8)$\nB. $(8,4)$\nC. $(-4,-8)$\nD. $(-4,8)$", "subject": "解析几何", "analysis": "解析: $\\because \\boldsymbol{a}=(1,-2)=-\\frac{1}{4}(-4,8),|\\boldsymbol{b}|=4|\\boldsymbol{a}|$,\n\n$\\therefore \\boldsymbol{b}$ 可能是 $(-4,8)$.\n\n答案: D"} {"id": "17964", "image": [], "answer": "B", "solution": "null", "level": "高二", "question": "若向量 $\\boldsymbol{a}, \\boldsymbol{b}$ 满足 $|\\boldsymbol{a}|=1,(\\boldsymbol{a}+\\boldsymbol{b}) \\perp \\boldsymbol{a},(2 \\boldsymbol{a}+\\boldsymbol{b}) \\perp \\boldsymbol{b}$, 则 $|\\boldsymbol{b}|=$", "options": "A. 2 B. $\\sqrt{2}$\n C. 1\n D. $\\frac{\\sqrt{2}}{2}$", "subject": "解析几何", "analysis": "解析: $\\because(\\boldsymbol{a}+\\boldsymbol{b}) \\perp \\boldsymbol{a},(2 \\boldsymbol{a}+\\boldsymbol{b}) \\perp \\boldsymbol{b},|\\boldsymbol{a}|=1$ ,\n\n$\\therefore\\left\\{\\begin{array}{l}(a+b) \\cdot a=0, \\\\ (2 a+b) \\cdot b=0,\\end{array} \\therefore\\left\\{\\begin{array}{l}\\boldsymbol{a} \\cdot \\boldsymbol{b}=-\\boldsymbol{a}^{2}=-1, \\\\ 2 \\boldsymbol{a} \\cdot \\boldsymbol{b}+\\boldsymbol{b}^{2}=0, \\text { (2) }\\end{array}\\right.\\right.$\n\n把(1)代入(2),得 $-2+\\boldsymbol{b}^{2}=0 . \\therefore \\boldsymbol{b}^{2}=2 , \\therefore|\\boldsymbol{b}|=\\sqrt{2}$. 故选 B.\n\n答案: B"} {"id": "17968", "image": [], "answer": "A", "solution": "null", "level": "高二", "question": "向量 $\\boldsymbol{a}$ 的模为 10, 它与 $x$ 轴正方向的夹角为 $150^{\\circ}$, 则它在 $x$ 轴正方向上的投影为", "options": "A. $-5 \\sqrt{3}$\nB. 5\nC. -5\nD. $5 \\sqrt{3}$", "subject": "解析几何", "analysis": "解析: $\\boldsymbol{a}$ 在 $x$ 轴正方向上的投影为 $|\\boldsymbol{a}| \\cdot \\cos 150^{\\circ}=-5 \\sqrt{3}$.\n\n答案: A"} {"id": "17969", "image": [], "answer": "B", "solution": "null", "level": "高二", "question": "如图所示, 在 Rt $\\triangle A B C$ 中, $\\angle A=90^{\\circ}, A B=1$, 则 $\\overrightarrow{A B} \\cdot \\overrightarrow{B C}$ 的值为 ( )\n\n![](https://cdn.mathpix.com/cropped/2024_04_19_f934fd3200dbf4af6dd7g-163,.jpg?height=291&width=257&top_left_y=1468&top_left_x=937)", "options": "A. 1\nB. -1\nC. 2\nD. -2", "subject": "解析几何", "analysis": "解析: $\\overrightarrow{A B} \\cdot \\overrightarrow{B C}=\\overrightarrow{A B} \\cdot(\\overrightarrow{A C}-\\overrightarrow{A B})=\\overrightarrow{A B} \\cdot \\overrightarrow{A C}-\\overrightarrow{A B^{2}}=-|\\overrightarrow{A B}|^{2}=-1$.\n\n答案: B"} {"id": "17971", "image": [], "answer": "A", "solution": "null", "level": "高二", "question": "已知 $\\boldsymbol{a}, \\boldsymbol{b}$ 均为单位向量, $(2 \\boldsymbol{a}+\\boldsymbol{b}) \\cdot(\\boldsymbol{a}-2 \\boldsymbol{b})=-\\frac{3 \\sqrt{3}}{2}$, 则 $\\boldsymbol{a}$ 与 $\\boldsymbol{b}$ 的夹角为 $(\\quad)$", "options": "A. $30^{\\circ}$\nB. $45^{\\circ}$\nC. $135^{\\circ}$\nD. $150^{\\circ}$", "subject": "解析几何", "analysis": "解析: $\\because(2 a+b) \\cdot(\\boldsymbol{a}-2 \\boldsymbol{b})=2 \\boldsymbol{a}^{2}-4 \\boldsymbol{a} \\cdot \\boldsymbol{b}+\\boldsymbol{a} \\cdot \\boldsymbol{b}-2 \\boldsymbol{b}^{2}=-3 \\boldsymbol{a} \\cdot \\boldsymbol{b}=-\\frac{3 \\sqrt{3}}{2} , \\therefore \\boldsymbol{a} \\cdot \\boldsymbol{b}=\\frac{\\sqrt{3}}{2}$. 设 $\\boldsymbol{a}$ 与 $\\boldsymbol{b}$ 的夹角为 $\\theta$ ,则 $\\cos \\theta=\\frac{\\boldsymbol{a} \\cdot \\boldsymbol{b}}{|\\boldsymbol{a}| \\cdot|\\boldsymbol{b}|}=\\frac{\\sqrt{3}}{2}$. 又 $\\because \\theta \\in\\left[0^{\\circ} , 180^{\\circ}\\right] , \\therefore \\theta=30^{\\circ}$.\n\n答案: A"} {"id": "17975", "image": [], "answer": "C", "solution": "null", "level": "高二", "question": "设向量 $\\boldsymbol{a}=(1,0), \\boldsymbol{b}=\\left(\\frac{1}{2}, \\frac{1}{2}\\right)$, 则下列结论中正确的是( )", "options": "A. $|\\boldsymbol{a}|=|\\boldsymbol{b}|$\nB. $\\boldsymbol{a} \\cdot \\boldsymbol{b}=\\frac{\\sqrt{2}}{2}$\nC. $(\\boldsymbol{a}-\\boldsymbol{b}) \\perp \\boldsymbol{b}$\nD. $\\boldsymbol{a} / / \\boldsymbol{b}$", "subject": "解析几何", "analysis": "解析: $\\because|\\boldsymbol{a}|=1 ,|\\boldsymbol{b}|=\\frac{\\sqrt{2}}{2} , \\therefore|\\boldsymbol{a}| \\neq|\\boldsymbol{b}|$ ,故 A 错误; $\\boldsymbol{a} \\cdot \\boldsymbol{b}=(1 , 0) \\cdot\\left(\\frac{1}{2}, \\frac{1}{2}\\right)=\\frac{1}{2} \\neq \\frac{\\sqrt{2}}{2}$ ,故 B 错误; $\\because \\boldsymbol{a}-\\boldsymbol{b}=\\left(\\frac{1}{2} ,-\\frac{1}{2}\\right), \\therefore(\\boldsymbol{a}-\\boldsymbol{b}) \\cdot \\boldsymbol{b}=\\left(\\frac{1}{2} ,-\\frac{1}{2}\\right) \\cdot\\left(\\frac{1}{2}, \\frac{1}{2}\\right)=\\frac{1}{4}-\\frac{1}{4}=0 , \\therefore(\\boldsymbol{a}-\\boldsymbol{b}) \\perp \\boldsymbol{b}$, 故 C 正确; $\\because 1 \\times \\frac{1}{2}-0 \\times \\frac{1}{2}=\\frac{1}{2} \\neq 0, \\therefore \\boldsymbol{a}$ 与 $\\boldsymbol{b}$ 不平行,故 D 错误.\n\n答案: C"} {"id": "17976", "image": [], "answer": "A", "solution": "null", "level": "高二", "question": "设向量 $\\boldsymbol{a}=(\\cos \\alpha, \\sin \\alpha), \\boldsymbol{b}=(\\cos \\beta, \\sin \\beta)$, 其中 $0<\\alpha<\\beta<\\pi$, 若 $|2 \\boldsymbol{a}+\\boldsymbol{b}|=|\\boldsymbol{a}-2 \\boldsymbol{b}|$, 则 $\\beta-\\alpha$等于 $(\\quad)$", "options": "A. $\\frac{\\pi}{2}$\n B. $-\\frac{\\pi}{2}$\n C. $\\frac{\\pi}{4}$\n D. $-\\frac{\\pi}{4}$", "subject": "解析几何", "analysis": "解析 : 由向量 $\\boldsymbol{a}=(\\cos \\alpha , \\sin \\alpha) , \\boldsymbol{b}=(\\cos \\beta , \\sin \\beta)$ ,且 $|2 \\boldsymbol{a}+\\boldsymbol{b}|=|\\boldsymbol{a}-2 \\boldsymbol{b}|$ ,两边平方可得 4 $+1+4 \\cos (\\alpha-\\beta)=1+4-4 \\cos (\\alpha-\\beta)$ ,解得 $\\cos (\\alpha-\\beta)=0$ ,又 $0<\\alpha<\\beta<\\pi$ ,所以 $0<\\beta-\\alpha<\\pi$ ,则 $\\beta-\\alpha=\\frac{\\pi}{2}$ ,故选 A.\n\n答案: A"} {"id": "17977", "image": ["9032.jpg"], "answer": "A", "solution": "null", "level": "高二", "question": "设 $A(a, 1), B(2, b), C(4,5)$ 为坐标平面上的三点, $O$ 为坐标原点. 若 $\\overrightarrow{O A}$ 与 $\\overrightarrow{O B}$ 在 $\\overrightarrow{O C}$ 方向上的投影相同, 则 $a$ 与 $b$ 满足的关系式为 $($ )", "options": "A. $4 a-5 b=3$\n B. $5 a-4 b=3$\n C. $4 a+5 b=14$\n D. $5 a+4 b=14$", "subject": "解析几何", "analysis": "解析 : 由图知,要使 $\\overrightarrow{O A}$ 与 $\\overrightarrow{O B}$ 在 $\\overrightarrow{O C}$ 方向上的投影相同,只需使 $\\overrightarrow{A B} \\perp \\overrightarrow{O C}$ ,即 $(2-a, b-1) \\cdot(4$ , $5)=0$ ,得 $4 a-5 b-3=0$ ,即 $4 a-5 b=3$.\n\n\n\n答案: A"} {"id": "17981", "image": [], "answer": "B", "solution": "null", "level": "高二", "question": "若向量 $\\overrightarrow{A B}=(3,-1), \\boldsymbol{n}=(2,1)$, 且 $\\boldsymbol{n} \\cdot \\overrightarrow{A C}=7$, 则 $\\boldsymbol{n} \\cdot \\overrightarrow{B C}=$", "options": "A. -2\nB. 2\nC. -2 或 2\nD. 0", "subject": "解析几何", "analysis": "解析: $\\because \\overrightarrow{A B}+\\overrightarrow{B C}=\\overrightarrow{A C}, \\therefore \\boldsymbol{n} \\cdot(\\overrightarrow{A B}+\\overrightarrow{B C})=\\boldsymbol{n} \\cdot \\overrightarrow{A C}$, 即 $\\boldsymbol{n} \\cdot \\overrightarrow{A B}+\\boldsymbol{n} \\cdot \\overrightarrow{B C}=\\boldsymbol{n} \\cdot \\overrightarrow{A C} \\cdot \\therefore \\boldsymbol{n} \\cdot \\overrightarrow{B C}=\\boldsymbol{n} \\cdot \\overrightarrow{A C}-\\boldsymbol{n} \\cdot \\overrightarrow{A B}$ $=7-5=2$.\n\n答案: B"} {"id": "17982", "image": [], "answer": "B", "solution": "null", "level": "高二", "question": "设 $x, y \\in \\mathbf{R}$, 向量 $\\boldsymbol{a}=(x, 1), \\boldsymbol{b}=(1, y), \\boldsymbol{c}=(2,-4)$, 且 $\\boldsymbol{a} \\perp \\boldsymbol{c}, \\boldsymbol{b} / / \\boldsymbol{c}$, 则 $|\\boldsymbol{a}+\\boldsymbol{b}|=(\\quad)$", "options": "A. $\\sqrt{5}$\nB. $\\sqrt{10}$\nC. $2 \\sqrt{5}$\nD. 10", "subject": "解析几何", "analysis": "解析: 由 $\\boldsymbol{a} \\perp \\boldsymbol{c}$ ,得 $2 x-4=0$ ,则 $x=2$. 由 $\\boldsymbol{b} / / \\boldsymbol{c}$ ,得 $-4=2 y$ ,则 $y=-2$ ,故 $|\\boldsymbol{a}+\\boldsymbol{b}|=$ $\\sqrt{(2+1)^{2}+(1-2)^{2}}=\\sqrt{10}$.\n\n答案: B"} {"id": "17983", "image": [], "answer": "C", "solution": "null", "level": "高二", "question": "若向量 $\\boldsymbol{a}=(1,2), \\boldsymbol{b}=(1,-1)$, 则 $2 \\boldsymbol{a}+\\boldsymbol{b}$ 与 $\\boldsymbol{a}-\\boldsymbol{b}$ 的夹角等于", "options": "A. $-\\frac{\\pi}{4}$\nB. $\\frac{\\pi}{6}$\nC. $\\frac{\\pi}{4}$\nD. $\\frac{3 \\pi}{4}$", "subject": "解析几何", "analysis": "解析: $\\because \\boldsymbol{a}=(1,2), \\boldsymbol{b}=(1,-1), \\therefore 2 \\boldsymbol{a}+\\boldsymbol{b}=(3,3), \\boldsymbol{a}-\\boldsymbol{b}=(0 , 3)$ ,则 $\\cos \\langle 2 \\boldsymbol{a}+\\boldsymbol{b}, \\boldsymbol{a}-\\boldsymbol{b}\\rangle$\n$=\\frac{3 \\times 0+9}{3 \\sqrt{2} \\times 3}=\\frac{\\sqrt{2}}{2}, \\therefore\\langle 2 \\boldsymbol{a}+\\boldsymbol{b}, \\boldsymbol{a}-\\boldsymbol{b}\\rangle=\\frac{\\pi}{4}$.\n\n答案: C"} {"id": "17984", "image": [], "answer": "A", "solution": "null", "level": "高二", "question": "若 $\\boldsymbol{a}=(2,3), \\boldsymbol{b}=(-4,7)$, 则 $\\boldsymbol{a}$ 在 $\\boldsymbol{b}$ 方向上的投影为", "options": "A. $\\frac{\\sqrt{65}}{5}$\nB. $\\sqrt{65}$\nC. $\\frac{\\sqrt{13}}{5}$\nD. $\\sqrt{13}$", "subject": "解析几何", "analysis": "解析: 设 $\\boldsymbol{a}$ 与 $\\boldsymbol{b}$ 的夹角为 $\\theta$ ,则 $\\cos \\theta=\\frac{\\boldsymbol{a} \\cdot \\boldsymbol{b}}{|\\boldsymbol{a}| \\boldsymbol{b} \\mid}=\\frac{2 \\times(-4)+3 \\times 7}{\\sqrt{4+9} \\times \\sqrt{16+49}}=\\frac{\\sqrt{5}}{5} , \\therefore \\boldsymbol{a}$ 在 $\\boldsymbol{b}$ 方向上的投影为 $|\\boldsymbol{a}| \\cos \\theta=\\sqrt{13} \\times \\frac{\\sqrt{5}}{5}=\\frac{\\sqrt{65}}{5}$.\n\n答案: A"} {"id": "17989", "image": [], "answer": "D", "solution": "null", "level": "高二", "question": "在四边形 $A B C D$ 中, 若 $\\overrightarrow{A B}+\\overrightarrow{C D}=0, \\overrightarrow{A C} \\cdot \\overrightarrow{B D}=0$, 则四边形为", "options": "A. 平行四边形\nB. 矩形\nC. 等腰梯形\nD. 菱形", "subject": "解析几何", "analysis": "解析: $\\because \\overrightarrow{A B} / / \\overrightarrow{C D},|\\overrightarrow{A B}|=|\\overrightarrow{C D}|$ ,且 $\\overrightarrow{A C} \\perp \\overrightarrow{B D} , \\therefore$ 四边形 $A B C D$ 为菱形.\n\n答案: D"} {"id": "17991", "image": ["9035.jpg"], "answer": "A", "solution": "null", "level": "高二", "question": "已知点 $A(7,1), B(1,4)$, 直线 $y=\\frac{1}{2} a x$ 与线段 $A B$ 交于点 $C$, 且 $\\overrightarrow{A C}=2 \\overrightarrow{C B}$, 则 $a$ 等于", "options": "A. 2\n B. 1\n C. $\\frac{4}{5}$\n D. $\\frac{5}{3}$", "subject": "解析几何", "analysis": "解析: 设 $C(x, y)$ ,则 $(x-7, y-1)=(2-2 x , 8-2 y)$ ,\n\n\n\n$\\because$ 点 $C$ 在直线 $y=\\frac{1}{2} a x$ 上, $\\therefore 3=\\frac{1}{2} a \\times 3 , \\therefore a=2$.\n\n答案: A"} {"id": "17992", "image": [], "answer": "D", "solution": "null", "level": "高二", "question": "在直角三角形 $A B C$ 中, 点 $D$ 是斜边 $A B$ 的中点, 点 $P$ 为线段 $C D$ 的中点, 则 $\\frac{|P A|^{2}+|P B|^{2}}{|P C|^{2}}=$", "options": "A. 2\n B. 4\n C. 5\n D. 10", "subject": "解析几何", "analysis": "解析: 将 $\\triangle A B C$ 各边及 $P A, P B, P C$ 均用向量表示,\n\n则 $\\frac{|P A|^{2}+|P B|^{2}}{|P C|^{2}}=\\frac{\\overrightarrow{P A}^{2}+\\overrightarrow{P B^{2}}}{\\overrightarrow{P C}^{2}}=\\frac{(\\overrightarrow{P C}+\\overrightarrow{C A})^{2}+(\\overrightarrow{P C}+\\overrightarrow{C B})^{2}}{\\overrightarrow{P C}^{2}}$\n$=\\frac{2|\\overrightarrow{P C}|^{2}+2 \\overrightarrow{P C} \\cdot(\\overrightarrow{C A}+\\overrightarrow{C B})+\\overrightarrow{A B}{ }^{2}}{|\\overrightarrow{P C}|^{2}}=\\frac{|\\overrightarrow{A B}|^{2}}{|\\overrightarrow{P C}|^{2}}-6=4^{2}-6=10$.\n\n答案: D"} {"id": "17998", "image": [], "answer": "D", "solution": "null", "level": "高二", "question": "若物体在共点力 $\\boldsymbol{F}_{1}=(\\lg 2, \\lg 2), \\boldsymbol{F}_{2}=(\\lg 5, \\lg 2)$ 的作用下产生位移 $\\boldsymbol{s}=(2 \\lg 5,1)$, 则共点力对物体所做的功 $W$ 为", "options": "A. $\\lg 2$\nB. $\\lg 5$\nC. 1\nD. 2", "subject": "解析几何", "analysis": "解析: $W=\\left(\\boldsymbol{F}_{1}+\\boldsymbol{F}_{2}\\right) \\cdot s=(\\lg 2+\\lg 5 , 2 \\lg 2) \\cdot(2 \\lg 5 , 1)=(1,2 \\lg 2) \\cdot(2 \\lg 5 , 1)=2 \\lg 5+2 \\lg 2$ $=2$ ,故选 D.\n\n答案: D"} {"id": "17999", "image": [], "answer": "C", "solution": "null", "level": "高二", "question": "已知 $A, B, C$ 是坐标平面上的三点, 其坐标分别为 $A(1,2), B(4,1), C(0,-1)$, 则 $\\triangle A B C$的形状为", "options": "A. 直角(非等腰)三角形\n\nB. 等腰(非等边)三角形\n\nC. 等腰直角三角形\n\nD. 以上均不正确", "subject": "解析几何", "analysis": "解析: $\\because \\overrightarrow{A B}=(3,-1), \\overrightarrow{A C}=(-1,-3), \\overrightarrow{A B} \\cdot \\overrightarrow{A C}=3 \\times(-1)+(-1) \\times(-3)=0$ ,且 $|\\overrightarrow{A B}|=$\n$|\\overrightarrow{A C}|=\\sqrt{10}, \\therefore \\triangle A B C$ 为等腰直角三角形.\n\n答案: C"} {"id": "18000", "image": [], "answer": "D", "solution": "null", "level": "高二", "question": "在 $\\triangle A B C$ 中, 若 $\\overrightarrow{B A} \\cdot(2 \\overrightarrow{B C}-\\overrightarrow{B A})=0$, 则 $\\triangle A B C$ 一定是", "options": "A. 直角三角形\nB. 等腰直角三角形\nC. 正三角形\nD. 等腰三角形", "subject": "解析几何", "analysis": "解析 : $\\overrightarrow{B A} \\cdot(2 \\overrightarrow{B C}-\\overrightarrow{B A})=\\overrightarrow{B A} \\cdot(\\overrightarrow{B C}+\\overrightarrow{B C}-\\overrightarrow{B A})=\\overrightarrow{B A} \\cdot(\\overrightarrow{B C}+\\overrightarrow{B C}+\\overrightarrow{A B})=\\overrightarrow{B A} \\cdot(\\overrightarrow{B C}+\\overrightarrow{A C})=-\\overrightarrow{B A} \\cdot(\\overrightarrow{C B}$ $+\\overrightarrow{C A})=0$. 由向量加法的平行四边形法则,知以 $C A , C B$ 为邻边的平行四边形的对角线互相垂直,所以 $\\triangle A B C$ 一定是等腰三角形.\n\n答案: D"} {"id": "18017", "image": [], "answer": "C", "solution": "null", "level": "高二", "question": "设 $\\boldsymbol{a}$ 是非零向量, $\\lambda$ 是非零实数, 则下列结论中正确的是", "options": "A. $\\boldsymbol{a}$ 与 $\\lambda \\boldsymbol{a}$ 的方向相同\nB. $\\boldsymbol{a}$ 与 $-\\lambda \\boldsymbol{a}$ 的方向相反\nC. $\\boldsymbol{a}$ 与 $\\lambda^{2} \\boldsymbol{a}$ 的方向相同\nD. $|\\lambda \\boldsymbol{a}|=\\lambda|\\boldsymbol{a}|$", "subject": "解析几何", "analysis": "解析: 只有当 $\\lambda>0$ 时, 才有 $\\boldsymbol{a}$ 与 $\\lambda \\boldsymbol{a}$ 的方向相同, $\\boldsymbol{a}$ 与 $-\\lambda \\boldsymbol{a}$ 的方向相反,且 $|\\lambda \\boldsymbol{a}|=\\lambda|\\boldsymbol{a}|$ : 因为 $\\lambda^{2}>0$ ,所以 $\\boldsymbol{a}$ 与 $\\lambda^{2} \\boldsymbol{a}$ 的方向相同.\n\n答案: C"} {"id": "18019", "image": [], "answer": "A", "solution": "null", "level": "高二", "question": "已知四边形 $A B C D$ 是菱形, 点 $P$ 在对角线 $A C$ 上(不包括端点 $A, C$ ), 则 $\\overrightarrow{A P}=$", "options": "A. $\\lambda(\\overrightarrow{A B}+\\overrightarrow{A D}), \\lambda \\in(0,1)$\n B. $\\lambda(\\overrightarrow{A B}+\\overrightarrow{B C}), \\quad \\lambda \\in\\left(0, \\frac{\\sqrt{2}}{2}\\right)$\n C. $\\lambda(\\overrightarrow{A B}-\\overrightarrow{A D}), \\lambda \\in(0,1)$\n D. $\\lambda(\\overrightarrow{A B}-\\overrightarrow{B C}), \\lambda \\in\\left(0, \\frac{\\sqrt{2}}{2}\\right)$", "subject": "解析几何", "analysis": "解析: 由向量加法运算法则可知, $\\overrightarrow{A C}=\\overrightarrow{A B}+\\overrightarrow{A D}$ ,又点 $P$ 在线段 $A C$ 上,故 $\\overrightarrow{A P}$ 与 $\\overrightarrow{A C}$ 同向,且 $|\\overrightarrow{A P}|<|\\overrightarrow{A C}| \\cdot$ 故 $\\overrightarrow{A P}=\\lambda(\\overrightarrow{A B}+\\overrightarrow{A D}), \\lambda \\in(0,1)$.\n\n答案: A"} {"id": "18024", "image": [], "answer": "C", "solution": "null", "level": "高二", "question": "点 $C$ 在线段 $A B$ 上, 且 $\\overrightarrow{A C}=\\frac{3}{5} \\overrightarrow{A B}$, 则 $\\overrightarrow{A C}=$", "options": "A. $\\frac{3}{2} \\overrightarrow{B C}$\nB. $\\frac{2}{3} \\overrightarrow{B C}$\nC. $-\\frac{3}{2} \\overrightarrow{B C}$\nD. $-\\frac{2}{3} \\overrightarrow{B C}$", "subject": "解析几何", "analysis": "解析 : 依题意,可得 $A C=\\frac{3}{2} B C , 又 \\overrightarrow{A C}$ 和 $\\overrightarrow{B C}$ 方向相反,所以 $\\overrightarrow{A C}=-\\frac{3}{2} \\overrightarrow{B C}$.\n\n答案: C"} {"id": "18026", "image": [], "answer": "A", "solution": "null", "level": "高二", "question": "已知 $\\boldsymbol{e}_{1}, \\boldsymbol{e}_{2}$ 是不共线向量, 则下列各组向量中, 是共线向量的有\n\n(1) $a=5 \\boldsymbol{e}_{1}, \\quad \\boldsymbol{b}=7 \\boldsymbol{e}_{1} ; \\quad$ (2) $a=\\frac{1}{2} \\boldsymbol{e}_{1}-\\frac{1}{3} \\boldsymbol{e}_{2}, \\quad \\boldsymbol{b}=3 \\boldsymbol{e}_{1}-2 \\boldsymbol{e}_{2}$;\n\n(3) $a=e_{1}+e_{2}, b=3 e_{1}-3 e_{2}$.", "options": "A. (1)(2)\nB. (1)(3)\nC. (2)(3)\nD. (1)(2)(3)", "subject": "解析几何", "analysis": "解析: (1)中, $\\boldsymbol{a}$ 与 $\\boldsymbol{b}$ 显然共线; (2),因为 $\\boldsymbol{b}=3 \\boldsymbol{e}_{1}-2 \\boldsymbol{e}_{2}=6\\left(\\frac{1}{2} \\boldsymbol{e}_{1}-\\frac{1}{3} e_{2}\\right)=6 a$, 故 $\\boldsymbol{a}$ 与 $\\boldsymbol{b}$ 共线;\n\n(3)中,设 $\\boldsymbol{b}=3 \\boldsymbol{e}_{1}-3 \\boldsymbol{e}_{2}=k\\left(\\boldsymbol{e}_{1}+\\boldsymbol{e}_{2}\\right)$ ,无解,故 $\\boldsymbol{a}$ 与 $\\boldsymbol{b}$ 不共线,故选 A.\n\n答案: A"} {"id": "19770", "image": [], "answer": "C", "solution": "null", "level": "高二", "question": "在 $\\triangle A B C$ 中, 若 $b, a, c$ 成等差数列, 且 $\\sin ^{2} A=\\sin B \\sin C$, 则 $\\triangle A B C$ 的形状为 ( )", "options": "A. 等腰三角形\nB. 直角三角形\nC. 等边三角形\nD. 等腰直角三角形", "subject": "解析几何", "analysis": "$\\because \\triangle \\mathrm{ABC}$ 的三边 $\\mathrm{b}, \\mathrm{a}, \\mathrm{c}$ 成等差数列, $\\therefore \\mathrm{b}+\\mathrm{c}=2 \\mathrm{a}(1)$, 又 $\\sin ^{2} \\mathrm{~A}=\\sin B \\sin \\mathrm{C}$,\n\n根据正弦定理化简得: $\\mathrm{a}^{2}=\\mathrm{bc}(2)$, 由(1)得: $\\mathrm{a}=\\frac{b+c}{2}$, 代入(2)得:\n\n$\\frac{(b+c)^{2}}{4}=\\mathrm{bc}$, 即 $(\\mathrm{b}-\\mathrm{c})^{2}=0, \\therefore \\mathrm{b}=\\mathrm{c}$, 故 $\\mathrm{a}=\\mathrm{b}=\\mathrm{c}$, 则三角形为等边三角形. 故选: $\\mathrm{C}$."} {"id": "19780", "image": [], "answer": "A", "solution": "null", "level": "高二", "question": "在 $\\triangle A B C$ 中, 角 $A 、 B 、 C$ 的对边分别为 $a 、 b 、 c$, 如果 $a 、 b 、 c$ 成等差数列, $B=30^{\\circ}$, $\\triangle \\mathrm{ABC}$ 的面积为 $\\frac{3}{2}$, 则 $\\mathrm{b}$ 等于 $(\\quad)$", "options": "A. $1+\\sqrt{3}$\nB. $2+\\sqrt{3}$\nC. $\\frac{1+\\sqrt{3}}{2}$\nD. $\\frac{2+\\sqrt{3}}{2}$", "subject": "解析几何", "analysis": "由余弦定理得 $b^{2}=a^{2}+c^{2}-2 a c \\cos B=(a+c)^{2}-2 a c-2 a c \\cos B(1)$,\n\n又 $\\mathrm{S}_{\\triangle \\mathrm{ABC}}=\\frac{1}{2} \\mathrm{acsin} \\mathrm{B}=\\frac{1}{4} \\mathrm{ac}=\\frac{3}{2}, \\therefore \\mathrm{ac}=6,(2) \\because \\mathrm{a} 、 \\mathrm{~b} 、 \\mathrm{c}$ 成等差数列, $\\therefore \\mathrm{a}+\\mathrm{c}=2 \\mathrm{~b}$, (3), 将(2)(3)代入\n\n(1)得 $b^{2}=4 b^{2}-12-6 \\sqrt{3}$, 化简整理得 $b^{2}=4+2 \\sqrt{3}$, 解得 $b=1+\\sqrt{3}$. 故选 A."} {"id": "18032", "image": [], "answer": "A", "solution": "null", "level": "高二", "question": "$\\cos 27^{\\circ} \\cos 57^{\\circ}-\\sin 27^{\\circ} \\cos 147^{\\circ}$ 等于", "options": "A. $\\frac{\\sqrt{3}}{2}$\nB. $-\\frac{\\sqrt{3}}{2}$\nC. $\\frac{1}{2}$\nD. $-\\frac{1}{2}$", "subject": "解析几何", "analysis": "解析: 原式 $=\\cos 27^{\\circ} \\cos 57^{\\circ}-\\sin 27^{\\circ} \\cos \\left(180^{\\circ}-33^{\\circ}\\right)=\\cos 27^{\\circ} \\cdot \\cos 57^{\\circ}+\\sin 27^{\\circ} \\cos 33^{\\circ}=\\cos$ $27^{\\circ} \\cos 57^{\\circ}+\\sin 27^{\\circ} \\sin 57^{\\circ}=\\cos \\left(57^{\\circ}-27^{\\circ}\\right)=\\cos 30^{\\circ}=\\frac{\\sqrt{3}}{2}$. 故选 A.\n\n答案: A"} {"id": "18033", "image": [], "answer": "A", "solution": "null", "level": "高二", "question": "若 $\\sin \\alpha-\\sin \\beta=\\frac{\\sqrt{3}}{2}, \\cos \\alpha-\\cos \\beta=\\frac{1}{2}$, 则 $\\cos (\\alpha-\\beta)$ 的值为( )", "options": "A. $\\frac{1}{2}$\n B. $\\frac{\\sqrt{3}}{2}$\n C. $\\frac{\\sqrt{3}}{4}$\n D. 1", "subject": "解析几何", "analysis": "解析: 由 $\\sin \\alpha-\\sin \\beta=\\frac{\\sqrt{3}}{2}, \\cos \\alpha-\\cos \\beta=\\frac{1}{2}$ ,\n\n得 $\\sin ^{2} \\alpha+\\sin ^{2} \\beta-2 \\sin \\alpha \\sin \\beta=\\frac{3}{4}$ ,\n\n$\\cos ^{2} \\alpha+\\cos ^{2} \\beta-2 \\cos \\alpha \\cos \\beta=\\frac{1}{4}$\n\n(1) + (2) 得 $2-2(\\sin \\alpha \\sin \\beta+\\cos \\alpha \\cos \\beta)=1$.\n\n$\\therefore \\sin \\alpha \\sin \\beta+\\cos \\alpha \\cos \\beta=\\frac{1}{2}$.\n\n$\\therefore \\cos (\\alpha-\\beta)=\\frac{1}{2}$.\n\n答案: A"} {"id": "18034", "image": [], "answer": "C", "solution": "null", "level": "高二", "question": "若 $\\cos (\\alpha-\\beta)=\\frac{\\sqrt{5}}{5}, \\cos 2 \\alpha=\\frac{\\sqrt{10}}{10}$, 并且 $\\alpha, \\beta$ 均为锐角, 且 $a<\\beta$, 则 $\\alpha+\\beta$ 的值为", "options": "A. $\\frac{\\pi}{6}$\n B. $\\frac{\\pi}{4}$\n C. $\\frac{3 \\pi}{4}$\n D. $\\frac{5 \\pi}{6}$", "subject": "解析几何", "analysis": "解析: $\\because 0<\\alpha<\\beta<\\frac{\\pi}{2} , \\therefore-\\frac{\\pi}{2}<\\alpha-\\beta<0 , 0<2 \\alpha<\\pi$.\n\n由 $\\cos (\\alpha-\\beta)=\\frac{\\sqrt{5}}{5}$ ,得 $\\sin (\\alpha-\\beta)=-\\frac{2 \\sqrt{5}}{5}$.\n\n由 $\\cos 2 \\alpha=\\frac{\\sqrt{10}}{10}$ ,得 $\\sin 2 \\alpha=\\frac{3 \\sqrt{10}}{10}$.\n\n$\\therefore \\cos (\\alpha+\\beta)=\\cos [2 \\alpha-(\\alpha-\\beta)]=\\cos 2 \\alpha \\cos (\\alpha-\\beta)+\\sin 2 \\alpha \\sin (\\alpha-\\beta)$\n\n$=\\frac{\\sqrt{10}}{10} \\times \\frac{\\sqrt{5}}{5}+\\frac{3 \\sqrt{10}}{10} \\times\\left(-\\frac{2 \\sqrt{5}}{5}\\right)=-\\frac{\\sqrt{2}}{2}$.\n\n又 $\\because \\alpha+\\beta \\in(0, \\pi) , \\therefore \\alpha+\\beta=\\frac{3 \\pi}{4}$.\n\n答案: C"} {"id": "18040", "image": [], "answer": "B", "solution": "null", "level": "高二", "question": "若 $\\cos \\alpha=\\frac{1}{17}, \\cos (\\alpha+\\beta)=-\\frac{47}{51}$, 且 $\\alpha, \\beta$ 都是锐角, 则 $\\cos \\beta$ 的值为 ( )", "options": "A. $-\\frac{1}{7}$\nB. $\\frac{1}{3}$\nC. $\\frac{403}{867}$\nD. $-\\frac{403}{867}$", "subject": "解析几何", "analysis": "解析: $\\because \\beta=(\\alpha+\\beta)-\\alpha$ ,\n又 $\\because \\cos \\alpha=\\frac{1}{17} , \\cos (\\alpha+\\beta)=-\\frac{47}{51} , \\alpha , \\beta$ 都是锐角,\n\n$\\therefore \\alpha+\\beta$ 是钝角, $\\therefore \\sin \\alpha=\\frac{12 \\sqrt{2}}{17}, \\sin (\\alpha+\\beta)=\\frac{14 \\sqrt{2}}{51}$.\n\n$\\because \\cos \\beta=\\cos [(\\alpha+\\beta)-\\alpha]=\\cos (\\alpha+\\beta) \\cos \\alpha+\\sin (\\alpha+\\beta) \\sin \\alpha$,\n\n$\\therefore \\cos \\beta=-\\frac{47}{51} \\times \\frac{1}{17}+\\frac{14 \\sqrt{2}}{51} \\times \\frac{12 \\sqrt{2}}{17}=\\frac{-47+336}{51 \\times 17}=\\frac{289}{51 \\times 17}=\\frac{1}{3}$.\n\n答案: B"} {"id": "18041", "image": [], "answer": "A", "solution": "null", "level": "高二", "question": "已知 $\\sin \\left(\\frac{\\pi}{6}+\\alpha\\right)=\\frac{3}{5}, \\frac{\\pi}{3}<\\alpha<\\frac{5 \\pi}{6}$, 则 $\\cos \\alpha$ 的值是", "options": "A. $\\frac{3-4 \\sqrt{3}}{10}$\nB. $\\frac{4-3 \\sqrt{3}}{10}$\nC. $\\frac{2 \\sqrt{3}-3}{5}$\nD. $\\frac{3-2 \\sqrt{3}}{5}$", "subject": "解析几何", "analysis": "解析: $\\because \\frac{\\pi}{3}<\\alpha<\\frac{5 \\pi}{6} , \\therefore \\frac{\\pi}{2}<\\frac{\\pi}{6}+\\alpha<\\pi$.\n\n$\\therefore \\cos \\left(\\frac{\\pi}{6}+\\alpha\\right)=-\\sqrt{1-\\sin ^{2}\\left(\\frac{\\pi}{6}+\\alpha\\right)}=-\\frac{4}{5}$.\n\n$\\therefore \\cos \\alpha=\\cos \\left[\\left(\\frac{\\pi}{6}+\\alpha\\right)-\\frac{\\pi}{6}\\right]=\\cos \\left(\\frac{\\pi}{6}+\\alpha\\right) \\cos \\frac{\\pi}{6}+\\sin \\left(\\frac{\\pi}{6}+\\alpha\\right) \\cdot \\sin \\frac{\\pi}{6}=-\\frac{4}{5} \\times \\frac{\\sqrt{3}}{2}+\\frac{3}{5} \\times \\frac{1}{2}=\\frac{3-4 \\sqrt{3}}{10}$.\n\n答案: A"} {"id": "18046", "image": [], "answer": "D", "solution": "null", "level": "高二", "question": "$\\sin 165^{\\circ}$ 等于", "options": "A. $\\frac{1}{2}$\nB. $\\frac{\\sqrt{3}}{2}$\nC. $\\frac{\\sqrt{6}+\\sqrt{2}}{4}$\nD. $\\frac{\\sqrt{6}-\\sqrt{2}}{4}$", "subject": "解析几何", "analysis": "解析: $\\sin 165^{\\circ}=\\sin 15^{\\circ}=\\sin \\left(45^{\\circ}-30^{\\circ}\\right)=\\frac{\\sqrt{6}-\\sqrt{2}}{4}$.\n\n答案: D"} {"id": "18049", "image": [], "answer": "B", "solution": "null", "level": "高二", "question": "已知 $\\sin (\\alpha+\\beta)=\\frac{1}{2}, \\sin (\\alpha-\\beta)=\\frac{1}{3}$, 则 $\\log \\sqrt{5}\\left(\\frac{\\tan \\alpha}{\\tan \\beta}\\right)^{2}=$", "options": "A. 5\n B. 4\n C. 3\n D. 2", "subject": "解析几何", "analysis": "解析: $\\because \\sin (\\alpha+\\beta)=\\frac{1}{2}, \\sin (\\alpha-\\beta)=\\frac{1}{3}$ ,\n\n$\\therefore \\sin \\alpha \\cos \\beta+\\cos \\alpha \\sin \\beta=\\frac{1}{2}, \\quad \\sin \\alpha \\cos \\beta-\\cos \\alpha \\sin \\beta=\\frac{1}{3}$,\n\n$\\therefore \\sin \\alpha \\cos \\beta=\\frac{5}{12}, \\quad \\cos \\alpha \\sin \\beta=\\frac{1}{12}$,\n\n$\\therefore \\frac{\\tan \\alpha}{\\tan \\beta}=5$,\n\n$\\therefore \\log \\sqrt{5}\\left(\\frac{\\tan \\alpha}{\\tan \\beta}\\right)^{2}=\\log \\sqrt{5} 5^{2}=4$. 故选 B.\n\n答案: B"} {"id": "18061", "image": [], "answer": "C", "solution": "null", "level": "高二", "question": "求值: $\\frac{1-\\tan 15^{\\circ}}{1+\\tan 15^{\\circ}}=$", "options": "A. $\\frac{1}{2}$\nB. $\\sqrt{3}$\nC. $\\frac{\\sqrt{3}}{3}$\nD. $\\frac{\\sqrt{3}}{2}$", "subject": "解析几何", "analysis": "解析: $\\frac{1-\\tan 15^{\\circ}}{1+\\tan 15^{\\circ}}=\\frac{\\tan 45^{\\circ}-\\tan 15^{\\circ}}{1+\\tan 45^{\\circ} \\tan 15^{\\circ}}=\\tan \\left(45^{\\circ}-15^{\\circ}\\right)=\\tan 30^{\\circ}=\\frac{\\sqrt{3}}{3}$. 故选 C.\n\n答案: C"} {"id": "18063", "image": [], "answer": "D", "solution": "null", "level": "高二", "question": "已知 $\\alpha+\\beta=\\frac{\\pi}{6}$, 且 $\\alpha, \\beta$ 满足 $\\sqrt{3}(\\tan \\alpha \\tan \\beta+2)+2 \\tan \\alpha+3 \\tan \\beta=0$, 则 $\\tan \\alpha$ 等于", "options": "A. $-\\frac{\\sqrt{3}}{3}$\n B. $\\sqrt{3}$\n C. $-\\sqrt{3}$\n D. $3 \\sqrt{3}$", "subject": "解析几何", "analysis": "解析: $\\because \\sqrt{3}(\\tan \\alpha \\tan \\beta+2)+2 \\tan \\alpha+3 \\tan \\beta=0$ ,\n\n$\\therefore \\sqrt{3} \\tan \\alpha \\tan \\beta+3(\\tan \\alpha+\\tan \\beta)=\\tan \\alpha-2 \\sqrt{3}$. (1)\n\n$\\because \\tan (\\alpha+\\beta)=\\frac{\\tan \\alpha+\\tan \\beta}{1-\\tan \\alpha \\tan \\beta}=\\frac{\\sqrt{3}}{3}$,\n\n$\\therefore 3(\\tan \\alpha+\\tan \\beta)=\\sqrt{3}(1-\\tan \\alpha \\tan \\beta)$.\n\n将(2)代入(1),得 $\\sqrt{3}=\\tan \\alpha-2 \\sqrt{3} , \\therefore \\tan \\alpha=\\sqrt{3}+2 \\sqrt{3}=3 \\sqrt{3}$.\n\n答案: D"} {"id": "18064", "image": [], "answer": "B", "solution": "null", "level": "高二", "question": "已知 $\\tan \\alpha, \\tan \\beta$ 是方程 $x^{2}+3 \\sqrt{3} x+4=0$ 的两根, 且 $-\\frac{\\pi}{2}<\\alpha<\\frac{\\pi}{2},-\\frac{\\pi}{2}<\\beta<\\frac{\\pi}{2}$, 则 $\\alpha+\\beta$ 的值为", "options": "A. $\\frac{\\pi}{3}$\n B. $-\\frac{2 \\pi}{3}$\n C. $\\frac{\\pi}{3}$ 或 $-\\frac{2 \\pi}{3}$\n D. $-\\frac{\\pi}{3}$ 或 $\\frac{2 \\pi}{3}$", "subject": "解析几何", "analysis": "解析 : 由一元二次方程根与系数的关系得 $\\tan \\alpha+\\tan \\beta=-3 \\sqrt{3}, \\tan \\alpha \\cdot \\tan \\beta=4 , \\therefore \\tan \\alpha<0$ , $\\tan \\beta<0$.\n\n$\\therefore \\tan (\\alpha+\\beta)=\\frac{\\tan \\alpha+\\tan \\beta}{1-\\tan \\alpha \\tan \\beta}=\\frac{-3 \\sqrt{3}}{1-4}=\\sqrt{3}$.\n\n又 $\\because-\\frac{\\pi}{2}<\\alpha<\\frac{\\pi}{2}, \\quad-\\frac{\\pi}{2}<\\beta<\\frac{\\pi}{2}$,\n\n且 $\\tan \\alpha<0 , \\tan \\beta<0 ,$\n\n$\\therefore-\\pi<\\alpha+\\beta<0, \\quad \\therefore \\alpha+\\beta=-\\frac{2 \\pi}{3}$.\n\n答案: B"} {"id": "18068", "image": [], "answer": "A", "solution": "null", "level": "高二", "question": "$\\frac{1-\\tan 27^{\\circ} \\tan 33^{\\circ}}{\\tan 27^{\\circ}+\\tan 33^{\\circ}}$ 等于", "options": "A. $\\frac{\\sqrt{3}}{3}$\nB. $\\sqrt{3}$\nC. $\\tan 6^{\\circ}$\nD. $\\frac{1}{\\tan 6^{\\circ}}$", "subject": "解析几何", "analysis": "解析: $\\because \\frac{\\tan 27^{\\circ}+\\tan 33^{\\circ}}{1-\\tan 27^{\\circ} \\tan 33^{\\circ}}=\\tan \\left(27^{\\circ}+33^{\\circ}\\right)=\\tan 60^{\\circ} , \\therefore$ 原式 $=\\frac{1}{\\tan 60^{\\circ}}=\\frac{\\sqrt{3}}{3}$.\n\n答案: A"} {"id": "18071", "image": [], "answer": "A", "solution": "null", "level": "高二", "question": "若 $\\alpha+\\beta=\\frac{\\pi}{4}$, 则 $\\frac{1-\\tan \\beta}{1+\\tan \\beta}=$", "options": "A. $\\tan \\alpha$\nB. $\\tan \\beta$\nC. $\\tan \\left(\\frac{\\pi}{4}-\\alpha\\right)$\nD. $\\tan \\left(\\frac{\\pi}{4}-\\beta\\right)$", "subject": "解析几何", "analysis": "解析 : $\\tan (\\alpha+\\beta)=\\frac{\\tan \\alpha+\\tan \\beta}{1-\\tan \\alpha \\tan \\beta}$ ,即 $\\tan \\alpha+\\tan \\beta=1-\\tan \\alpha \\tan \\beta$.\n\n$\\therefore \\tan \\alpha(1+\\tan \\beta)=1-\\tan \\beta$\n\n$\\therefore \\frac{1-\\tan \\beta}{1+\\tan \\beta}=\\tan \\alpha$.\n\n答案: A"} {"id": "18076", "image": [], "answer": "D", "solution": "null", "level": "高二", "question": "$\\frac{1}{2}-\\sin ^{2} 15^{\\circ}=$", "options": "A. $\\frac{\\sqrt{6}}{4}$\nB. $\\frac{\\sqrt{6}-\\sqrt{2}}{4}$\nC. $\\frac{\\sqrt{3}}{2}$\nD. $\\frac{\\sqrt{3}}{4}$", "subject": "解析几何", "analysis": "解析 : 原式 $=\\frac{1}{2}-\\frac{1-\\cos \\left(2 \\times 15^{\\circ}\\right)}{2}=\\frac{\\cos 30^{\\circ}}{2}=\\frac{\\sqrt{3}}{4}$.\n\n答案: D"} {"id": "18077", "image": [], "answer": "C", "solution": "null", "level": "高二", "question": "已知 $\\cos \\left(\\frac{\\pi}{4}-x\\right)=\\frac{3}{5}$, 则 $\\sin 2 x=$", "options": "A. $\\frac{18}{25}$\n B. $\\frac{7}{25}$\n C. $-\\frac{7}{25}$\n D. $-\\frac{16}{25}$", "subject": "解析几何", "analysis": "解析: 因为 $\\sin 2 x=\\cos \\left(\\frac{\\pi}{2}-2 x\\right)=\\cos \\left[2\\left(\\frac{\\pi}{4}-x\\right)\\right]=2 \\cos ^{2}\\left(\\frac{\\pi}{4}-x\\right)-1$ ,所以 $\\sin 2 x=2 \\times\\left(\\frac{3}{5}\\right)^{2}-1$ $=\\frac{18}{25}-1=-\\frac{7}{25}$.\n\n答案: C"} {"id": "18082", "image": [], "answer": "A", "solution": "null", "level": "高二", "question": "$\\frac{1-\\tan ^{2} 15^{\\circ}}{2 \\tan 15^{\\circ}}=$", "options": "A. $\\sqrt{3}$ B. $\\frac{\\sqrt{3}}{3}$\nC. 1\nD. -1", "subject": "解析几何", "analysis": "解析: 原式 $=\\frac{1}{\\frac{2 \\tan 15^{\\circ}}{1-\\tan ^{2} 15^{\\circ}}}=\\frac{1}{\\tan 30^{\\circ}}=\\sqrt{3}$.\n\n答案: A"} {"id": "18084", "image": [], "answer": "D", "solution": "null", "level": "高二", "question": "已知 $\\sin 2 \\alpha=\\frac{2}{3}$, 则 $\\sin ^{2}\\left(\\alpha+\\frac{\\pi}{4}\\right)=$", "options": "A.$ \\frac{1}{6} B. \\frac{1}{2} C. \\frac{2}{3} D. \\frac{5}{6}$", "subject": "解析几何", "analysis": "解析: $\\sin ^{2}\\left(\\alpha+\\frac{\\pi}{4}\\right)=\\frac{1-\\cos \\left[2\\left(\\alpha+\\frac{\\pi}{4}\\right)\\right]}{2}=\\frac{1+\\sin 2 \\alpha}{2}=\\frac{5}{6}$.\n\n答案: D"} {"id": "18085", "image": [], "answer": "C", "solution": "null", "level": "高二", "question": "$\\sqrt{1+\\cos 100^{\\circ}}-\\sqrt{1-\\cos 100^{\\circ}}=(\\quad)$", "options": "A. $-2 \\cos 5^{\\circ}$\nB. $2 \\cos 5^{\\circ}$\nC. $-2 \\sin 5^{\\circ}$\nD. $2 \\sin 5^{\\circ}$", "subject": "解析几何", "analysis": "解析: 原式 $=\\sqrt{2 \\cos ^{2} 50^{\\circ}}-\\sqrt{2 \\sin ^{2} 50^{\\circ}}=\\sqrt{2}\\left(\\cos 50^{\\circ}-\\sin 50^{\\circ}\\right)=2\\left(\\frac{\\sqrt{2}}{2} \\cos 50^{\\circ}-\\frac{\\sqrt{2}}{2} \\sin 50^{\\circ}\\right)=$\n$2 \\sin \\left(45^{\\circ}-50^{\\circ}\\right)=-2 \\sin 5^{\\circ}$.\n\n答案: C"} {"id": "19829", "image": [], "answer": "B", "solution": "null", "level": "高二", "question": "设常数 $a \\in R$, 集合 $A=\\{x \\mid(x-1) \\quad(x-a) \\geq 0\\}, B=\\{x \\mid x \\geq a-1\\}$, 若 $A \\cup B=R$,则 $a$ 的取值范围为", "options": "A. $(-\\infty, 2)$\nB. $(-\\infty, 2]$\nC. $(2,+\\infty)$\nD. $[2,+\\infty)$", "subject": "解析几何", "analysis": "当 $a>1$ 时, $A=(-\\infty, 1] \\cup[a,+\\infty), B=[a-1,+\\infty)$, 若 $A \\cup B=R$, 则 $a-1 \\leq 1, \\therefore 10$ 恒成立, 则 $k$ 的取值范围是 ( )", "options": "A. $(0,+\\infty)$\nB. $[0,+\\infty)$\nC. $[0,4)$\nD. $(0,4)$", "subject": "解析几何", "analysis": "当 $\\mathrm{k}=0$ 时, 不等式 $k x^{2}-k x+1>0$ 可化为 $1>0$, 显然恒成立;\n\n当 $\\mathrm{k} \\neq 0$ 时, 若不等式 $k x^{2}-k x+1>0$ 恒成立, 则对应函数的图象开口朝上且与 $\\mathrm{x}$ 轴无交点,\n\n则 $\\left\\{\\begin{array}{l}k>0 \\\\ \\Delta=k^{2}-4 k<0\\end{array}\\right.$, 解得: $0<\\mathrm{k}<4$, 综上 $\\mathrm{k}$ 的取值范围是 $[0,4)$. 故选 C."} {"id": "19836", "image": [], "answer": "C", "solution": "null", "level": "高二", "question": "已知关于 $x$ 的不等式 $x^{2}-a x-b<0$ 的解集是 $(2,3)$, 则 $a+b$ 的值是 ( )", "options": "A. -11\nB. 11\nC. -1\nD. 1", "subject": "解析几何", "analysis": "若关于 $\\mathrm{x}$ 的不等式 $\\mathrm{x}^{2}-\\mathrm{ax}-\\mathrm{b}<0$ 的解集是 (2, 3),\n\n则 2,3 是方程 $x^{2}-a x-b=0$ 的根, 故 $a=5, b=-6$ 故 $a+b=-1$, 故选 C."} {"id": "19839", "image": [], "answer": "C", "solution": "null", "level": "高二", "question": "设集合 $A=\\left\\{x \\mid x^{2}-3 x+2<0\\right\\}, B=\\{x \\mid 1", "options": "A. -3\n B. 3\n C. -1\n D. 1", "subject": "解析几何", "analysis": "当 $a=0$ 时, $z=x$. 仅在直线 $x=z$ 过点 $A(1,1)$ 时,\n\n$z$ 有最小值 1 , 与题意不符.\n\n当 $a>0$ 时, $y=-\\frac{1}{a} x+\\frac{z}{a}$.\n\n斜率 $k=-\\frac{1}{a}<0$,\n\n仅在直线 $z=x+a y$ 过点 $A(1,1)$ 时,\n\n直线在 $y$ 轴的截距最小, 此时 $z$ 也最小,\n\n与目标函数取得最小值的最优解有无数个矛盾.\n\n当 $a<0$ 时, $y=-\\frac{1}{a} x+\\frac{Z}{a}$, 斜率 $k=-\\frac{1}{a}>0$,\n\n为使目标函数 $\\mathrm{z}$ 取得最小值的最优解有无数个, 当且仅当斜率 $-\\frac{1}{a}=k_{A C}$. 即 $-\\frac{1}{a}=\\frac{1}{3}, \\therefore a$ $=-3$."} {"id": "18617", "image": [], "answer": "B", "solution": "null", "level": "高二", "question": "如图所示的坐标平面的可行域内(阴影部分且包括边界), 若使目标函数 $\\mathrm{z}=a x+y(a>0)$取得最大值的最优解有无穷多个, 则 $a$ 的值为 ( )\n\n![](https://cdn.mathpix.com/cropped/2024_04_19_3540a5c85e1968e614e9g-095\\.jpg?height=237&width=320&top_left_y=1886&top_left_x=911)", "options": "A. $\\frac{1}{4}$\nB. $\\frac{3}{5}$\nC. 4\nD. $\\frac{5}{3}$", "subject": "解析几何", "analysis": "解析 由 $y=-a x+z$ 知当 $-a=k_{A C}$ 时, 最优解有无穷多个. $\\because k_{A C}=-\\frac{3}{5}, \\therefore a=\\frac{3}{5}$."} {"id": "18625", "image": ["9132.jpg"], "answer": "A", "solution": "null", "level": "高二", "question": "若实数 $x, \\quad y$ 满足不等式组 $\\left\\{\\begin{array}{l}x+3 y-3 \\geqslant 0, \\\\ 2 x-y-3 \\leqslant 0, \\\\ x-y+1 \\geqslant 0,\\end{array} \\quad\\right.$ 则 $x+y$ 的最大值为 $(\\quad)$", "options": "A. 9\nB. $\\frac{15}{7}$\nC. 1\nD. $\\frac{7}{15}$", "subject": "解析几何", "analysis": "解析 画出可行域如图:\n\n \n\n当直线 $y=-x+z$ 过点 $A$ 时, $z$ 最大.\n\n由 $\\left\\{\\begin{array}{l}2 x-y-3=0, \\\\ x-y+1=0\\end{array}\\right.$ 得 $A(4,5), \\therefore z_{\\max }=4+5=9$."} {"id": "18629", "image": ["9136.jpg"], "answer": "D", "solution": "null", "level": "高二", "question": "已知点 $P(x, y)$ 的坐标满足条件 $\\left\\{\\begin{array}{l}x+y \\leqslant 4, \\\\ y \\geqslant x, \\\\ x \\geqslant 1,\\end{array}\\right.$ 则 $x^{2}+y^{2}$ 的最大值为 $($ )", "options": "A. $\\sqrt{10}$\nB. 8\nC. 16\nD. 10", "subject": "解析几何", "analysis": "解析 画出不等式组对应的可行域如下图所示\n\n\n\n易得 $A(1,1),|O A|=\\sqrt{2}, B(2,2)$,\n\n$|O B|=2 \\sqrt{2}$,\n\n$C(1,3),|O C|=\\sqrt{10}$.\n\n$\\therefore\\left(x^{2}+y^{2}\\right)_{\\text {max }}=|O C|^{2}=(\\sqrt{10})^{2}=10$."} {"id": "18631", "image": ["9138.jpg"], "answer": "A", "solution": "null", "level": "高二", "question": "设变量 $x, y$ 满足约束条件 $\\left\\{\\begin{array}{l}x-y+2 \\geqslant 0, \\\\ x-5 y+10 \\leqslant 0, \\\\ x+y-8 \\leqslant 0,\\end{array}\\right.$ 则目标函数 $z=3 x-4 y$ 的最大值和最小值分别为 $(\\quad)$", "options": "A. $3,-11$\nB. $-3,-11$\nC. $11,-3$\nD. 11,3", "subject": "解析几何", "analysis": "解析 作出可行域如图阴影部分所示, 由图可知 $z=3 x-4 y$ 经过点 $A$ 时 $z$ 有最小值, 经过点 $B$ 时 $z$ 有最大值. 易求 $A(3,5), B(5,3) . \\therefore z$ 最大 $=3 \\times 5-4 \\times 3=3, \\quad Z$ 最小 $=3 \\times 3-4 \\times 5=$ -11 .\n\n"} {"id": "18632", "image": [], "answer": "B", "solution": "null", "level": "高二", "question": "设不等式组 $\\left\\{\\begin{array}{l}x \\geqslant 1, \\\\ x-2 y+3 \\geqslant 0 \\\\ y \\geqslant x\\end{array}\\right.$, 所表示的平面区域是 $\\Omega_{1}$, 平面区域 $\\Omega_{2}$ 与 $\\Omega_{1}$ 关于直线 $3 x-4 y-9=0$ 对称. 对于 $\\Omega_{1}$ 中的任意点 $A$ 与 $\\Omega_{2}$ 中的任意点 $B$, 则 $|A B|$ 的最小值为 $(\\quad)$", "options": "A. $\\frac{28}{5}$\nB. 4\nC. $\\frac{12}{5}$\nD. 2", "subject": "解析几何", "analysis": "如图所示。由约束条件作出可行域, 得 $D(1,1), E(1,2), C(3,3)$.\n\n要求 $|A B|_{\\min }$, 可通过求 $D 、 E 、 C$ 三点到直线 $3 x-4 y-9=0$ 距离最小值的 2 倍来求.\n\n经分析, $D(1,1)$ 到直线 $3 x-4 y-9=0$ 的距离 $d=\\frac{|3 \\times 1-4 \\times 1-9|}{5}=2$ 最小, $\\therefore|A B|_{\\min }=$"} {"id": "19855", "image": [], "answer": "C", "solution": "null", "level": "高二", "question": "设 $a, b \\in R_{+}, A=\\sqrt{a}+\\sqrt{b}, B=\\sqrt{a+b}$, 则 $A, B$ 的大小关系是( )", "options": "A. $A \\geq B$\nB. $A \\leq B$\nC. $A>B$\nD. $A0$.\n\n又 $A>0, B>0, \\therefore A>B$. 选 C."} {"id": "19859", "image": [], "answer": "D", "solution": "null", "level": "高二", "question": "已知 $m=x^{2}+2 x, n=3 x+2$, 则 ( )", "options": "A. $m>n$\nB. $\\mathrm{m}<\\mathrm{n}$\nC. $m=n$\nD. $\\mathrm{m}$ 与 $\\mathrm{n}$ 的大小不能确定", "subject": "解析几何", "analysis": "$m-n=x^{2}+2 x-(3 x+2)=x^{2}-x-2=\\left(x-\\frac{1}{2}\\right)^{2}-\\frac{9}{4} \\geq-\\frac{9}{4}, \\because m-n$ 无法判断与 0 的大小,\n\n$\\therefore m$ 与 $n$ 的大小不能确定. 故答案为:D"} {"id": "19849", "image": [], "answer": "A", "solution": "null", "level": "高二", "question": "若 $x \\neq-2$ 且 $y \\neq 1$, 则 $M=x^{2}+y^{2}+4 x-2 y$ 的值与 -5 的大小关系是 ( )", "options": "A. $M>-5$\nB. $M-5$\nC. $M \\geqslant-5$\nD. $M \\leqslant-5$", "subject": "解析几何", "analysis": "$\\mathrm{A} M-(-5)=x^{2}+y^{2}+4 x-2 y+5=(x+2)^{2}+(y-1)^{2}$, 解析$\\because x \\neq-2, y \\neq 1, \\therefore(x+2)^{2}>0,(y-1)^{2}>0$, 因此 $(x+2)^{2}+(y-1)^{2}>0$, 故 $M>-5$. 选 A"} {"id": "18443", "image": [], "answer": "B", "solution": "null", "level": "高二", "question": "在 $\\triangle A B C$ 中, $a=7, b=4 \\sqrt{3}, c=\\sqrt{13}$, 则 $\\triangle A B C$ 的最小角为 $(\\quad)$", "options": "A. $\\frac{\\pi}{3}$\nB. $\\frac{\\pi}{6}$\nC. $\\frac{\\pi}{4}$\nD. $\\frac{\\pi}{12}$", "subject": "度量几何学", "analysis": "解析 $\\because a>b>c, \\therefore C$ 为最小角,\n\n由余弦定理 $\\cos C=\\frac{a^{2}+b^{2}-c^{2}}{2 a b}$\n\n$=\\frac{7^{2}+(4 \\sqrt{3})^{2}-(\\sqrt{13})^{2}}{2 \\times 7 \\times 4 \\sqrt{3}}=\\frac{\\sqrt{3}}{2} . \\therefore C=\\frac{\\pi}{6}$."} {"id": "18455", "image": ["9105.jpg"], "answer": "A", "solution": "null", "level": "高二", "question": "已知 $\\triangle A B C$ 中, $A B=1, B C=2$, 则角 $C$ 的取值范围是 $(\\quad)$", "options": "A. $0\n\n方法二 (应用数形结合)\n\n如图所示, 以 $B$ 为圆心,以 1 为半径画圆,\n\n则圆上除了直线 $B C$ 上的点外, 都可作为 $A$ 点. 从点 $C$ 向圆 $B$ 作切线, 设切点为 $A_{1}$ 和 $A_{2}$, 当 $A$ 与 $A_{1} 、 A_{2}$ 重合时, 角 $C$ 最大, 易知此时: $B C=2, A B=1, A C \\perp A B, \\therefore C=\\frac{\\pi}{6}$,\n\n$\\therefore 00$,\n\n$\\therefore c+x$​ 所对的最大角变为锐角."} {"id": "17749", "image": [], "answer": "D", "solution": "null", "level": "高二", "question": "下列说法不正确的是", "options": "A. 当角 $\\alpha$ 的终边在 $x$ 轴上时, 角 $\\alpha$ 的正切线是一个点\\n\\nB. 当角 $\\alpha$ 的终边在 $y$ 轴上时, 角 $\\alpha$ 的正切线不存在\\n\\nC. 正弦线的始点随角的终边位置的变化而变化\\n\\nD. 余弦线和正切线的始点都是原点\\n\\n", "subject": "度量几何学", "analysis": "D"} {"id": "17735", "image": ["8958.jpg"], "answer": "D", "solution": "null", "level": "高二", "question": "若 $0<\\alpha<2 \\pi$, 且 $\\sin \\alpha<\\frac{\\sqrt{3}}{2}, \\cos \\alpha>\\frac{1}{2}$, 则角 $\\alpha$ 的取值范围是", "options": "A. $\\left(-\\frac{\\pi}{3}, \\frac{\\pi}{3}\\right)$\n B. $\\left(0, \\frac{\\pi}{3}\\right)$\n C. $\\left(\\frac{5 \\pi}{3}, 2 \\pi\\right)$\n D. $\\left(0, \\frac{\\pi}{3}\\right) \\cup\\left(\\frac{5 \\pi}{3}, 2 \\pi\\right)$", "subject": "度量几何学", "analysis": "解析: 由图知, 公共区域为 D.\n\n答案: D"} {"id": "17743", "image": [], "answer": "A", "solution": "null", "level": "高二", "question": "函数 $f(x)=\\tan \\left(2 x-\\frac{\\pi}{4}\\right)$ 的定义域为", "options": "A. $\\left\\{x \\left\\lvert\\, x \\neq \\frac{3 \\pi}{8}+\\frac{1}{2} k \\pi\\right., \\quad k \\in \\mathbf{Z}\\right\\}$\nB. $\\left\\{x \\left\\lvert\\, x \\neq \\frac{3 \\pi}{8}+k \\pi\\right., \\quad k \\in \\mathbf{Z}\\right\\}$\nC. $\\left\\{x \\left\\lvert\\, x \\neq \\frac{3 \\pi}{8}+2 k \\pi\\right., \\quad k \\in \\mathbf{Z}\\right\\}$\nD. $\\left\\{x \\left\\lvert\\, x \\neq \\frac{5 \\pi}{8}+\\frac{1}{2} k \\pi\\right., \\quad k \\in \\mathbf{Z}\\right\\}$", "subject": "度量几何学", "analysis": ""} {"id": "17754", "image": [], "answer": "A", "solution": "null", "level": "高二", "question": "若 $\\tan \\alpha=2$, 则 $\\frac{\\sin \\alpha+\\cos \\alpha}{\\sin \\alpha-\\cos \\alpha}+\\cos ^{2} \\alpha=$", "options": "A. $\\frac{16}{5}$\nB. $-\\frac{16}{5}$\nC. $\\frac{8}{5}$\nD. $-\\frac{8}{5}$", "subject": "度量几何学", "analysis": "解析: $\\frac{\\sin \\alpha+\\cos \\alpha}{\\sin \\alpha-\\cos \\alpha}+\\cos ^{2} \\alpha=\\frac{\\sin \\alpha+\\cos \\alpha}{\\sin \\alpha-\\cos \\alpha}+\\frac{\\cos ^{2} \\alpha}{\\sin ^{2} \\alpha+\\cos ^{2} \\alpha}=\\frac{\\tan \\alpha+1}{\\tan \\alpha-1}+\\frac{1}{\\tan ^{2} \\alpha+1}=\\frac{16}{5}$. 故选 A.\n\n答案: A"} {"id": "17756", "image": [], "answer": "B", "solution": "null", "level": "高二", "question": "已知 $\\sin x=2 \\cos x$, 则 $\\frac{\\sin x-\\cos x}{\\sin x+\\cos x}=$", "options": "A. $\\frac{1}{2}$ B. $\\frac{1}{3} $ C. $\\frac{1}{4} $ D. $ \\frac{1}{5}$", "subject": "度量几何学", "analysis": "解析: $\\because \\sin x=2 \\cos x , \\therefore \\tan x=2$ ,\n\n$\\therefore$ 原式 $=\\frac{\\tan x-1}{\\tan x+1}=\\frac{2-1}{2+1}=\\frac{1}{3}$.\n\n答案: B"} {"id": "17757", "image": [], "answer": "A", "solution": "null", "level": "高二", "question": "若 $\\alpha$ 是三角形的内角, 且 $\\sin \\alpha+\\cos \\alpha=\\frac{2}{3}$, 则三角形是", "options": "A. 针角三角形\nB. 锐角三角形\nC. 直角三角形\nD. 等腰三角形", "subject": "度量几何学", "analysis": "解析: 将 $\\sin \\alpha+\\cos \\alpha=\\frac{2}{3}$ 两边平方,得 $1+2 \\sin \\alpha \\cos \\alpha=\\frac{4}{9}$ ,即 $2 \\sin \\alpha \\cos \\alpha=-\\frac{5}{9}$. 又 $\\alpha$ 是三角形的内角,\n\n$\\therefore \\sin \\alpha>0 , \\cos \\alpha<0 , \\therefore \\alpha$ 为钝角. 故三角形为钝角三角形.\n\n答案: A"} {"id": "17778", "image": [], "answer": "B", "solution": "null", "level": "高二", "question": "已知 $\\cos 31^{\\circ}=m$, 则 $\\sin 239^{\\circ} \\tan 149^{\\circ}$ 的值是", "options": "A. $\\frac{1-m^{2}}{m}$\n B. $\\sqrt{1-m^{2}}$\n C. $-\\frac{1-m^{2}}{m}$\n D. $-\\sqrt{1-m^{2}}$", "subject": "度量几何学", "analysis": "解析 : $\\sin 239^{\\circ} \\tan 149^{\\circ}$\n\n$=\\sin \\left(180^{\\circ}+59^{\\circ}\\right) \\tan \\left(180^{\\circ}-31^{\\circ}\\right)$\n\n$=-\\sin 59^{\\circ}\\left(-\\tan 31^{\\circ}\\right)$\n\n$=-\\sin \\left(90^{\\circ}-31^{\\circ}\\right)\\left(-\\tan 31^{\\circ}\\right)$\n\n$=-\\cos 31^{\\circ} \\cdot\\left(-\\tan 31^{\\circ}\\right)$\n\n$=\\sin 31^{\\circ}=\\sqrt{1-\\cos ^{2} 31^{\\circ}}$\n\n$=\\sqrt{1-m^{2}}$."} {"id": "17786", "image": [], "answer": "C", "solution": "null", "level": "高二", "question": "已知 $\\alpha \\in(0, \\pi)$, 且 $\\cos \\alpha=-\\frac{5}{13}$, 则 $\\sin \\left(\\frac{\\pi}{2}-\\alpha\\right) \\cdot \\tan \\alpha=$", "options": "A. $-\\frac{12}{13}$\nB. $-\\frac{5}{13}$\nC. $\\frac{12}{13}$\nD. $\\frac{5}{13}$", "subject": "度量几何学", "analysis": "解析: $\\because \\alpha \\in(0, \\pi)$, 且 $\\cos \\alpha=-\\frac{5}{13}, \\therefore \\sin \\alpha=\\frac{12}{13}$, 由诱导公式及同角三角函数的商数关系知 $\\sin \\left(\\frac{\\pi}{2}-\\alpha\\right) \\cdot \\tan \\alpha=\\cos \\alpha \\cdot \\frac{\\sin \\alpha}{\\cos \\alpha}=\\sin \\alpha=\\frac{12}{13}$.故选 C.\n\n答案: C"} {"id": "17817", "image": [], "answer": "B", "solution": "null", "level": "高二", "question": "函数 $y=\\sin \\left(2 x+\\frac{5}{2} \\pi\\right)$ 的图象的一个对称中心是", "options": "A. $\\left(\\frac{\\pi}{8}, 0\\right)$\nB. $\\left(\\frac{\\pi}{4}, 0\\right)$\nC. $\\left(-\\frac{\\pi}{3}, 0\\right)$\nD. $\\left(\\frac{3 \\pi}{8}, 0\\right)$", "subject": "度量几何学", "analysis": "解析: 正弦曲线的对称中心为曲线与 $x$ 轴的交点, 将四个点代入验证, 只有 $\\left(\\frac{\\pi}{4}, 0\\right)$ 符合要求, 故选 B.\n\n答案: B"} {"id": "17820", "image": ["8979.jpg"], "answer": "D", "solution": "null", "level": "高二", "question": "若函数 $y=2 \\cos x(0 \\leqslant x \\leqslant 2 \\pi)$ 的图象和直线 $y=2$ 围成一个封闭的平面图形, 则这个封闭图形的面积是", "options": "A. 4\n B. 8\n C. $2 \\pi$\n D. $4 \\pi$", "subject": "度量几何学", "analysis": "解析: 由图可知, 图形 $S_{1}$ 与 $S_{2}, S_{3}$ 与 $S_{4}$ 都是对称图形, 有 $S_{1}=S_{2}, S_{3}=S_{4}$, 因此函数 $y=$ $2 \\cos x$ 的图象与直线 $y=2$ 所围成的图形面积等于矩形 $O A B C$ 的面积.\n\n\n\n$\\because|O A|=2,|O C|=2 \\pi, \\quad \\therefore S_{\\text {䂑形 }}=2 \\times 2 \\pi=4 \\pi$.\n\n答案: D"} {"id": "17836", "image": [], "answer": "D", "solution": "null", "level": "高二", "question": "函数 $y=\\tan \\left(x+\\frac{\\pi}{4}\\right)$ 的定义域是 ( )", "options": "A. $\\left\\{x \\left\\lvert\\, x \\neq-\\frac{\\pi}{4}\\right.\\right\\}$\nB. $\\left\\{x \\left\\lvert\\, x \\neq \\frac{\\pi}{4}\\right.\\right\\}$\nC. $\\left\\{\\left.x\\right|_{x \\neq k \\pi-\\frac{\\pi}{4},}, \\quad k \\in \\mathbf{Z}\\right\\}$\nD. $\\left\\{\\left.x\\right|_{x \\neq k \\pi+\\frac{\\pi}{4},} \\quad k \\in \\mathbf{Z}\\right\\}$", "subject": "度量几何学", "analysis": "解析: $x+\\frac{\\pi}{4} \\neq k \\pi+\\frac{\\pi}{2}, k \\in \\mathbf{Z}, \\quad \\therefore x \\neq k \\pi+\\frac{\\pi}{4}, \\quad k \\in \\mathbf{Z}$.\n\n答案: D"} {"id": "18469", "image": [], "answer": "C", "solution": "null", "level": "高二", "question": "在 $\\triangle A B C$ 中, $B=60^{\\circ}$, 最大边与最小边之比为 $(\\sqrt{3}+1): 2$, 则最大角为 $($ )", "options": "A. $45^{\\circ}$\n B. $60^{\\circ}$\n C. $75^{\\circ}$\n D. $90^{\\circ}$", "subject": "度量几何学", "analysis": "设 $C$ 为最大角, 则 $A$ 为最小角, 则 $A+C=120^{\\circ}$,\n\n$\\therefore \\frac{\\sin C}{\\sin A}=\\frac{\\sin \\left(120^{\\circ}-A\\right)}{\\sin A}$\n\n$=\\frac{\\sin 120^{\\circ} \\cos A-\\cos 120^{\\circ} \\sin A}{\\sin A}$\n\n$=\\frac{\\sqrt{3}}{2 \\tan A}+\\frac{1}{2}=\\frac{\\sqrt{3}+1}{2}=\\frac{\\sqrt{3}}{2}+\\frac{1}{2}$,\n\n$\\therefore \\tan A=1, A=45^{\\circ}, C=75^{\\circ}$."} {"id": "17887", "image": [], "answer": "D", "solution": "null", "level": "高二", "question": "$1920^{\\circ}$ 的角化为弧度制为", "options": "A. $\\frac{16}{3}$\nB. $\\frac{32}{3}$\nC. $\\frac{16}{3} \\pi$\nD. $\\frac{32}{3} \\pi$", "subject": "度量几何学", "analysis": "解析: $\\because 1^{\\circ}=\\frac{\\pi}{180} \\mathrm{rad}$,\n$\\therefore 1920^{\\circ}=1920 \\times \\frac{\\pi}{180}=\\frac{32}{3} \\pi$.\n\n## 答案: D"} {"id": "17890", "image": [], "answer": "B", "solution": "null", "level": "高二", "question": "已知 $\\alpha=\\frac{6}{7} \\pi$, 则角 $\\alpha$ 的终边在 ()", "options": "A. 第一象限\nB. 第二象限\nC. 第三象限\nD. 第四象限", "subject": "度量几何学", "analysis": "解析: $\\frac{\\pi}{2}<\\frac{6}{7} \\pi<\\pi$, 所以角 $\\alpha$ 的终边在第二象限, 选 B.\n\n## 答案: B"} {"id": "17892", "image": [], "answer": "A", "solution": "null", "level": "高二", "question": "把 $-\\frac{11}{4} \\pi$ 表示成 $\\theta+2 k \\pi(k \\in \\mathbf{Z})$ 的形式, 使 $|\\theta|$ 最小的 $\\theta$ 的值是", "options": "A. $-\\frac{3 \\pi}{4}$\nB. $-\\frac{\\pi}{4}$\nC. $\\frac{\\pi}{4}$\nD. $\\frac{3 \\pi}{4}$", "subject": "度量几何学", "analysis": "解析:由 $-\\frac{11}{4} \\pi=\\theta+2 k \\pi(k \\in \\mathbf{Z})$,\n\n得 $\\theta=-\\frac{11}{4} \\pi-2 k \\pi(k \\in \\mathbf{Z})$,\n\n显然 $k \\leqslant 0$ 时, $|\\theta|$ 取最小值.\n\n$k=-1$ 时, $\\theta=-\\frac{3}{4} \\pi,|\\theta|=\\frac{3}{4} \\pi$;\n\n$k=-2$ 时, $\\theta=\\frac{5}{4} \\pi,|\\theta|=\\frac{5}{4} \\pi>\\frac{3}{4} \\pi$;\n\n$k=0$ 时, $\\theta=-\\frac{11}{4} \\pi,|\\theta|=\\frac{11}{4} \\pi>\\frac{3}{4} \\pi$.\n\n故满足题意的是 $\\theta=-\\frac{3}{4} \\pi$.\n\n答案: A"} {"id": "17898", "image": [], "answer": "D", "solution": "null", "level": "高二", "question": "已知角 $\\alpha$ 的终边经过点 $(-4,3)$, 则 $\\cos \\alpha$ 等于", "options": "A. $\\frac{4}{5}$\nB. $\\frac{3}{5}$\nC. $-\\frac{3}{5}$\nD. $-\\frac{4}{5}$", "subject": "度量几何学", "analysis": ""} {"id": "17899", "image": [], "answer": "A", "solution": "null", "level": "高二", "question": "若角 $\\alpha$ 的终边与直线 $y=3 x$ 重合且 $\\sin \\alpha<0$, 又 $P(m, n)$ 是 $\\alpha$ 终边上一点, 且 $|O P|=\\sqrt{10}$,则 $m-n$ 等于", "options": "A. 2\n B. -2\n C. 4\n D. -4", "subject": "度量几何学", "analysis": "解析: 分析题意知 $\\alpha$ 的终边位于第三象限, 故 $m<0, n<0, n=3 m$ 且 $\\sqrt{m^{2}+n^{2}}=\\sqrt{10}, \\therefore m$ $=-1, n=-3, m-n=2$.\n\n答案: A"} {"id": "17900", "image": [], "answer": "C", "solution": "null", "level": "高二", "question": "若角 $\\alpha$ 的终边过点 $P\\left(2 \\sin 30^{\\circ},-2 \\cos 30^{\\circ}\\right)$, 则 $\\sin \\alpha$ 的值等于( )", "options": "A. $\\frac{1}{2}$\n B. $-\\frac{1}{2}$\n C. $-\\frac{\\sqrt{3}}{2}$\n D. $-\\frac{\\sqrt{3}}{3}$", "subject": "度量几何学", "analysis": "解析 : $\\because 2 \\sin 30^{\\circ}=2 \\times \\frac{1}{2}=1,-2 \\cos 30^{\\circ}=-2 \\times \\frac{\\sqrt{3}}{2}=-\\sqrt{3}, \\therefore P(1,-\\sqrt{3})$,\n\n$\\therefore$ 点 $P$ 到原点的距离为 $\\sqrt{1^{2}+(-\\sqrt{3})^{2}}=2$,\n\n$\\therefore \\sin \\alpha=-\\frac{\\sqrt{3}}{2}$.\n\n答案: C"} {"id": "18006", "image": [], "answer": "D", "solution": "null", "level": "高二", "question": "已知在边长为 2 的菱形 $A B C D$ 中, $\\angle A B C=60^{\\circ}$, 则 $|\\overrightarrow{B D}|=(\\quad)$", "options": "A. 1\n B. $\\sqrt{3}$\n C. 2\n D. $2 \\sqrt{3}$", "subject": "度量几何学", "analysis": "解析: 易知 $A C \\perp B D$, 且 $\\angle A B D=30^{\\circ}$, 设 $A C$ 与 $B D$ 交于点 $O$, 则 $A O=\\frac{1}{2} A B=1$. 在 Rt $\\triangle A B O$中, 易得 $|\\overrightarrow{B O}|=\\sqrt{3}$, 则 $|\\overrightarrow{B D}|=2|\\overrightarrow{B O}|=2 \\sqrt{3}$. 故选 D.\n\n答案: D"} {"id": "18011", "image": [], "answer": "C", "solution": "null", "level": "高二", "question": "汽车以 $120 \\mathrm{~km} / \\mathrm{h}$ 的速度向西走了 $2 \\mathrm{~h}$, 摩托车以 $45 \\mathrm{~km} / \\mathrm{h}$ 的速度向东北方向走了 $2 \\mathrm{~h}$, 则下列命题中正确的是", "options": "A. 汽车的速度大于摩托车的速度\n\nB. 汽车的位移大于摩托车的位移\n\nC. 汽车走的路程大于摩托车走的路程\n\nD. 以上都不对", "subject": "度量几何学", "analysis": "解析:由向量不能比较大小,可知选 C.\n\n答案: C"} {"id": "18013", "image": [], "answer": "C", "solution": "null", "level": "高二", "question": "设 $O$ 为坐标原点, 且 $|\\vec{O} M|=1$, 则动点 $M$ 的集合是", "options": "A. 一条线段\nB. 一个圆面\nC. 一个圆\nD. 一段圆弧", "subject": "度量几何学", "analysis": "解析: 动点 $M$ 到原点 $O$ 的距离等于定长 1 , 故动点 $M$ 的轨迹是以 $O$ 为圆心, 1 为半径的圆.\n\n答案: C"} {"id": "18038", "image": [], "answer": "A", "solution": "null", "level": "高二", "question": "$\\cos \\left(45^{\\circ}-\\alpha\\right) \\cos \\left(\\alpha+15^{\\circ}\\right)-\\sin \\left(45^{\\circ}-\\alpha\\right) \\sin \\left(\\alpha+15^{\\circ}\\right)$ 等于", "options": "A. $\\frac{1}{2}$\nB. $-\\frac{1}{2}$\nC. $\\frac{\\sqrt{3}}{2}$\nD. $-\\frac{\\sqrt{3}}{2}$", "subject": "度量几何学", "analysis": "解析: 原式 $=\\cos \\left(\\alpha-45^{\\circ}\\right) \\cos \\left(\\alpha+15^{\\circ}\\right)+\\sin \\left(\\alpha-45^{\\circ}\\right) \\sin \\left(\\alpha+15^{\\circ}\\right)=\\cos \\left[\\left(\\alpha-45^{\\circ}\\right)-\\left(\\alpha+15^{\\circ}\\right)\\right]=$ $\\cos \\left(-60^{\\circ}\\right)=\\frac{1}{2}$.\n\n答案: A"} {"id": "18053", "image": [], "answer": "A", "solution": "null", "level": "高二", "question": "计算 $\\sin 44^{\\circ} \\cos 14^{\\circ}-\\cos 44^{\\circ} \\cos 76^{\\circ}$ 的值为", "options": "A. $\\frac{1}{2}$\nB. $\\frac{\\sqrt{3}}{3}$\nC. $\\frac{\\sqrt{2}}{2}$\nD. $\\frac{\\sqrt{3}}{2}$", "subject": "度量几何学", "analysis": "解析: 原式 $=\\sin \\left(44^{\\circ}-14^{\\circ}\\right)=\\sin 30^{\\circ}=\\frac{1}{2}$.\n\n答案: A"} {"id": "18055", "image": [], "answer": "C", "solution": "null", "level": "高二", "question": "设 $\\alpha \\in\\left(0, \\frac{\\pi}{2}\\right), \\beta \\in\\left(\\frac{\\pi}{2}, \\pi\\right)$, 若 $\\cos \\beta=-\\frac{1}{3}, \\sin (\\alpha+\\beta)=\\frac{7}{9}$, 则 $\\sin \\alpha$ 的值为( )", "options": "A. $\\frac{1}{27}$\nB. $\\frac{5}{27}$", "subject": "度量几何学", "analysis": "解析: 由 $\\cos \\beta=-\\frac{1}{3} , \\sin (\\alpha+\\beta)=\\frac{7}{9} \mathrm{~ , 得 ~} \\sin \\beta=\\frac{2 \\sqrt{2}}{3} , \\cos (\\alpha+\\beta)=-\\frac{4 \\sqrt{2}}{9}$ ,所以 $\\sin \\alpha=\\sin [(\\alpha$ $+\\beta)-\\beta]=\\frac{7}{9} \\times\\left(-\\frac{1}{3}\\right)-\\left(-\\frac{4 \\sqrt{2}}{9}\\right) \\times \\frac{2 \\sqrt{2}}{3}=\\frac{1}{3}$.\n\n答案: C"} {"id": "18069", "image": [], "answer": "C", "solution": "null", "level": "高二", "question": "已知 $\\alpha \\in\\left(\\frac{\\pi}{2}, \\pi\\right), \\sin \\alpha=\\frac{4}{5}$, 则 $\\tan \\left(\\alpha+\\frac{\\pi}{4}\\right)=$", "options": "A. $\\frac{1}{7}$\nB. 7\nC. $-\\frac{1}{7}$\nD. -7", "subject": "度量几何学", "analysis": "解析: $\\alpha \\in\\left(\\frac{\\pi}{2}, \\pi\\right), \\sin \\alpha=\\frac{4}{5} , \\therefore \\cos \\alpha=-\\frac{3}{5} , \\tan \\alpha=\\frac{\\sin \\alpha}{\\cos \\alpha}=-\\frac{4}{3}$\n\n$\\therefore \\tan \\left(\\alpha+\\frac{\\pi}{4}\\right)=\\frac{\\tan \\alpha+1}{1-\\tan \\alpha}=\\frac{-\\frac{4}{3}+1}{1+\\frac{4}{3}}=-\\frac{1}{7}$.\n\n答案: C"} {"id": "18070", "image": [], "answer": "C", "solution": "null", "level": "高二", "question": "已知 $\\tan \\alpha=\\frac{1}{4}, \\tan (\\alpha-\\beta)=\\frac{1}{3}$, 则 $\\tan \\beta$ 等于", "options": "A. $\\frac{7}{11}$\nB. $-\\frac{11}{7}$\nC. $-\\frac{1}{13}$\nD. $\\frac{1}{13}$", "subject": "度量几何学", "analysis": "解析 : $\\tan \\beta=\\tan [\\alpha-(\\alpha-\\beta)]=\\frac{\\tan \\alpha-\\tan (\\alpha-\\beta)}{1+\\tan \\alpha \\cdot \\tan (\\alpha-\\beta)}=-\\frac{1}{13}$.\n\n答案: C"} {"id": "18078", "image": [], "answer": "D", "solution": "null", "level": "高二", "question": "若 $\\tan \\theta+\\frac{1}{\\tan \\theta}=4$, 则 $\\sin 2 \\theta=$", "options": "A. $\\frac{1}{5}$ B. $ \\frac{1}{4} $ C. $\\frac{1}{3}$ D. $\\frac{1}{2}$", "subject": "度量几何学", "analysis": "解析: 法一: $\\because \\tan \\theta+\\frac{1}{\\tan \\theta}=\\frac{1+\\tan ^{2} \\theta}{\\tan \\theta}=4 , \\therefore 4 \\tan \\theta=1+\\tan ^{2} \\theta, \\therefore \\sin 2 \\theta=2 \\sin \\theta \\cos \\theta=$ $\\frac{2 \\sin \\theta \\cos \\theta}{\\sin ^{2} \\theta+\\cos ^{2} \\theta}=\\frac{2 \\tan \\theta}{1+\\tan ^{2} \\theta}=\\frac{2 \\tan \\theta}{4 \\tan \\theta}=\\frac{1}{2}$.\n\n法二: $\\because \\tan \\theta+\\frac{1}{\\tan \\theta}=\\frac{\\sin \\theta}{\\cos \\theta}+\\frac{\\cos \\theta}{\\sin \\theta}=\\frac{1}{\\cos \\theta \\sin \\theta}=\\frac{2}{\\sin 2 \\theta}, \\therefore 4=\\frac{2}{\\sin 2 \\theta}, \\quad \\therefore \\sin 2 \\theta=\\frac{1}{2}$.\n\n答案: D"} {"id": "19860", "image": [], "answer": "D", "solution": "null", "level": "高二", "question": "已知 $a 、 b$ 分别对应数轴上的 $A 、 B$ 两点, 且 $A$ 在原点右侧, $B$ 在原点左侧,则下列不等式成立的是 ( )", "options": "A. $a-b \\leqslant 0$\nB. $a+b<0$\nC. $|a|>|b|$\nD. $a^{2}+b^{2} \\geqslant-2 a b$", "subject": "度量几何学", "analysis": "则 $a-b>0$, 而 $a+b$ 的符号不确定, $|a|$ 与 $|b|$ 的大小也不确定. 故排除 $\\mathrm{ABC}$对于 $D, a^{2}+b^{2}+2 a b=(a+b)^{2} \\geq 0$, 进而得到 $a^{2}+b^{2} \\geq-2 a b$."} {"id": "19851", "image": [], "answer": "A", "solution": "null", "level": "高二", "question": "若 $-1<\\alpha<\\beta<1$, 则下面各式中恒成立的是().", "options": "A. $-2<\\alpha-\\beta<0$\nB. $-2<\\alpha-\\beta<-1$\nC. $-1<\\alpha-\\beta<0$\nD. $-1<\\alpha-\\beta<1$", "subject": "度量几何学", "analysis": "$\\because-1<\\alpha<\\beta<1, \\therefore-1<\\alpha<1,-1<-\\beta<1, \\alpha-\\beta<0, \\therefore-2<\\alpha-\\beta<0$. 选: A."} {"id": "18904", "image": [], "answer": "A", "solution": "null", "level": "高二", "question": "在锐角 $\\triangle A B C$ 中, $a, b, c$ 分别是角 $A, B, C$ 所对的边, $\\triangle A B C$ 的面积 $S=2$, 且满足 $a \\cos B=b(1+\\cos A)$, 则 $(c+a-b)(c+b-a)$ 的取值范围是()", "options": "A. $(8 \\sqrt{2}-8,8)$\nB. $(0,8)$\nC. $\\left(\\frac{8 \\sqrt{3}-8}{3}, 8 \\sqrt{3}\\right)$\nD. $\\left(\\frac{8 \\sqrt{3}-8}{3}, 8\\right)$", "subject": "度量几何学", "analysis": "由题意, 在锐角 $\\triangle A B C$ 中, 满足 $a \\cos B=b(1+\\cos A)$, 由正弦定理可得\n\n$\\sin A \\cos B=\\sin B+\\sin B \\cos A$ , 即 $\\sin A \\cos B-\\sin B \\cos A=\\sin B ,$\n\n可得 $\\sin (A-B)=\\sin B$, 所以 $A-B=B$, 即 $A=2 B<\\frac{\\pi}{2}$,\n\n所以 $B \\in\\left(0, \\frac{\\pi}{4}\\right)$, 所以 $A+B=3 B \\in\\left(\\frac{\\pi}{2}, \\frac{3 \\pi}{4}\\right)$, 则 $C=\\pi-3 B \\in\\left(\\frac{\\pi}{4}, \\frac{\\pi}{2}\\right)$,\n\n所以 $\\tan C=\\frac{2 \\tan \\frac{C}{2}}{1-\\tan ^{2} \\frac{C}{2}}>1$, 可得 $1>\\tan \\frac{C}{2}>-1+\\sqrt{2}$,\n\n又由 $\\triangle A B C$ 的面积 $S=\\frac{1}{2} a b \\sin C=2$, 所以 $a b=\\frac{4}{\\sin C}$,\n\n则 $(c+a-b)(c+b-a)=c^{2}-a^{2}-b^{2}+2 a b=-2 a b \\cos C+2 a b=2 a b(1-\\cos C)$\n\n$=\\frac{8}{\\sin C}(1-\\cos C)=8 \\times \\frac{1-\\left(1-2 \\sin ^{2} \\frac{C}{2}\\right)}{2 \\sin \\frac{C}{2} \\cos \\frac{C}{2}}=8 \\tan \\frac{C}{2} \\in(8 \\sqrt{2}-8,8)$"} {"id": "18949", "image": [], "answer": "B", "solution": "null", "level": "高二", "question": "在 $\\triangle A B C$ 中, “ $A>60^{\\circ}$ ” 是 “ $\\sin A>\\frac{\\sqrt{3}}{2}$ ” 的( )", "options": "A. 充分不必要条件\nB.必要不充分条件\nC. 充要条件\nD.既不充分也不必要条件", "subject": "度量几何学", "analysis": "由 “ $A>60^{\\circ}$ ” 不一定推出 “ $\\sin A>\\frac{\\sqrt{3}}{2}$ ” ,如 $A=135^{\\circ}>60^{\\circ}, \\sin 135^{\\circ}$\n\n$<\\sin 120^{\\circ}=\\frac{\\sqrt{3}}{3}$. 反之若 “ $\\sin A>\\frac{\\sqrt{3}}{2}$ ”, 则有 “ $A>60^{\\circ}$ ” . 故选 B."} {"id": "18977", "image": [], "answer": "A", "solution": "null", "level": "高二", "question": "在 $\\triangle A B C$ 中, 角 $A, B, C$ 的对边分别为 $a, b, c$. 若 $\\triangle A B C$ 为锐角三角形,且满足 $\\sin B(1+2 \\cos C)=2 \\sin A \\cos C+\\cos A \\sin C$ ,则下列等式成立的是()", "options": "A. $a=2 b$\n B. $b=2 a$\n C. $A=2 B$\n D. $B=2 A$", "subject": "度量几何学", "analysis": "由 $\\sin B(1+2 \\cos C)=2 \\sin A \\cos C+\\cos A \\sin C$,\n得 $\\sin B+2 \\sin B \\cos C=\\sin A \\cos C+\\sin B$,\n\n即 $2 \\sin B \\cos C=\\sin A \\cos C$ ,所以 $2 \\sin B=\\sin A$ ,即 $2 b=a$​ ,选 A."} {"id": "18997", "image": [], "answer": "A", "solution": "null", "level": "高二", "question": "在 $\\triangle A B C$ 中, $a, b, c$ 分别为内角 $A, B, C$ 的对边, $\\angle A C B=\\frac{\\pi}{3} . D 、 E$ 分别是边 $B C$ 和 $A B$ 的中点, $A D$ 与 $C E$ 交于 $G$ 点, $C G=2$, 则 $\\triangle A B C$ 面积的最大值是 ( )", "options": "A. $3 \\sqrt{3}$\n B. $2 \\sqrt{3}$\n C. $\\frac{5 \\sqrt{3}}{3}$\n D. $\\frac{4 \\sqrt{3}}{3}$", "subject": "度量几何学", "analysis": "依题意, $G$ 为 $\\triangle A B C$ 的重心, 因为 $C G=2$, 所以 $G E=1, C E=3$. 设 $\\angle A E C=\\alpha$, 则 $\\angle B E C=\\pi-\\alpha$. 在 $\\triangle A E C$ 中, 有 $b^{2}=3^{2}+\\left(\\frac{c}{2}\\right)^{2}-2 \\times 3 \\times \\frac{c}{2} \\cos \\angle A E C$, 在 $\\triangle B E C$ 中, 有 $a^{2}=3^{2}+\\left(\\frac{c}{2}\\right)^{2}-2 \\times 3 \\times \\frac{c}{2} \\cos \\angle B E C$, 两式相加, 则 $a^{2}+b^{2}=18+\\frac{c^{2}}{2}$.\n\n又 $c^{2}=a^{2}+b^{2}-2 a b \\cos \\frac{\\pi}{3}=a^{2}+b^{2}-a b$, 消去 $c^{2}$, 得 $a^{2}+b^{2}=36-a b$, 因为 $a^{2}+b^{2} \\geq 2 a b$, 所以所以 $a b \\leq 12$, 所以 $S_{\\triangle A B C}=\\frac{1}{2} a b \\sin \\frac{\\pi}{3}=\\frac{\\sqrt{3}}{4} a b \\leq \\frac{\\sqrt{3}}{4} \\times 12=3 \\sqrt{3}$.\n\n当且仅当 $a=b=2 \\sqrt{3}$ 时, $\\triangle A B C$ 的面积取得最大值, 最大值为 $3 \\sqrt{3}$. 故选 A."} {"id": "17741", "image": ["8962.jpg"], "answer": "A", "solution": "null", "level": "高二", "question": "设 $a=\\sin 46^{\\circ}, b=\\cos 46^{\\circ}, c=\\tan 46^{\\circ}$, 则", "options": "A. $c>a>b$\nB. $a>b>c$\nC. $b>c>a$\nD. $c>b>a$", "subject": "算术", "analysis": "解析: 如图, 结合三角函数线知 $A T>M P>O M$, 所以有 $\\tan 46^{\\circ}>\\sin 46^{\\circ}>\\cos 46^{\\circ}$, 即 $c>a>b$,故选 A.\n\n答案: A"} {"id": "17764", "image": [], "answer": "D", "solution": "null", "level": "高二", "question": "$\\sin 240^{\\circ}$ 的值为", "options": "A. $\\frac{\\sqrt{3}}{2}$\nB. $\\frac{1}{2}$\nC. $-\\frac{1}{2}$\nD. $-\\frac{\\sqrt{3}}{2}$", "subject": "算术", "analysis": "解析:由诱导公式二得 $\\sin 240^{\\circ}=\\sin \\left(180^{\\circ}+60^{\\circ}\\right)=-\\sin 60^{\\circ}=-\\frac{\\sqrt{3}}{2}$, 故选 D.\n\n答案: D"} {"id": "17783", "image": [], "answer": "B", "solution": "null", "level": "高二", "question": "若 $\\sin \\alpha=\\frac{1}{3}$, 则 $\\cos \\left(\\frac{\\pi}{2}+\\alpha\\right)=$", "options": "A. $\\frac{1}{3}$\nB. $-\\frac{1}{3}$\nC. $\\frac{2 \\sqrt{2}}{3}$\nD. $-\\frac{2 \\sqrt{2}}{3}$", "subject": "算术", "analysis": "解析: $\\cos \\left(\\frac{\\pi}{2}+\\alpha\\right)=-\\sin \\alpha=-\\frac{1}{3}$.\n\n答案: B"} {"id": "17810", "image": [], "answer": "A", "solution": "null", "level": "高二", "question": "下列函数中, 周期为 $\\frac{\\pi}{2}$ 的是", "options": "A. $y=\\cos 4 x$\nB. $y=\\sin 2 x$\nC. $y=\\cos \\frac{x}{4}$\nD. $y=\\sin \\frac{x}{2}$", "subject": "算术", "analysis": "解析: 对于 A, $\\because \\cos 4\\left(x+\\frac{\\pi}{2}\\right)=\\cos (2 \\pi+4 x)=\\cos 4 x, \\therefore T=\\frac{\\pi}{2}$; 对于 B, $\\because \\sin 2\\left(x+\\frac{\\pi}{2}\\right)=\\sin (\\pi$ $+2 x)=-\\sin 2 x, \\therefore T \\neq \\frac{\\pi}{2}$. 同理可知 $\\mathrm{C}, \\mathrm{D}$ 的周期均不是 $\\frac{\\pi}{2}$.\n\n答案: A"} {"id": "17811", "image": [], "answer": "C", "solution": "null", "level": "高二", "question": "函数 $f(x)=2|\\sin x|$ 的最小正周期为", "options": "A. $2 \\pi$\nB. $\\frac{3 \\pi}{2}$\nC. $\\pi$\nD. $\\frac{\\pi}{2}$", "subject": "算术", "analysis": "解析: $\\because \\sin (x+\\pi)=-\\sin x,|\\sin x|=|-\\sin x|$,\n\n$\\therefore f(x+\\pi)=f(x), \\therefore$ 函数 $f(x)=2|\\sin x|$ 的最小正周期为 $\\pi$. 故选 C.\n\n答案: C"} {"id": "17839", "image": [], "answer": "C", "solution": "null", "level": "高二", "question": "若 $f(x)=\\tan \\left(x+\\frac{\\pi}{4}\\right)$, 则", "options": "A. $f(1)>f(0)>f(-1)$\nB. $f(0)>f(1)>f(-1)$\nC. $f(0)>f(-1)>f(1)$\nD. $f(-1)>f(0)>f(1)$", "subject": "算术", "analysis": "解析: $f(0)=\\tan \\frac{\\pi}{4}, f(-1)=\\tan \\left(\\frac{\\pi}{4}-1\\right), f(1)=\\tan \\left(\\frac{\\pi}{4}+1\\right)=\\tan \\left(\\frac{\\pi}{4}+1-\\pi\\right)=\\tan \\left(1-\\frac{3}{4} \\pi\\right)$.\n\n$\\because-\\frac{\\pi}{2}<1-\\frac{3}{4} \\pi<\\frac{\\pi}{4}-1<\\frac{\\pi}{4}<\\frac{\\pi}{2}$\n\n又 $y=\\tan t$ 在 $t \\in\\left(-\\frac{\\pi}{2} , \\frac{\\pi}{2}\\right)$ 上是增函数,\n\n$\\therefore \\tan \\frac{\\pi}{4}>\\tan \\left(\\frac{\\pi}{4}-1\\right)>\\tan \\left(1-\\frac{3}{4} \\pi\\right)$\n\n$\\therefore f(0)>f(-1)>f(1)$.\n\n答案: C"} {"id": "17905", "image": [], "answer": "B", "solution": "null", "level": "高二", "question": "$\\sin \\frac{13 \\pi}{6}$ 的值是", "options": "A. $-\\frac{1}{2}$\nB. $\\frac{1}{2}$\nC. $-\\frac{\\sqrt{3}}{2}$\nD. $\\frac{\\sqrt{3}}{2}$", "subject": "算术", "analysis": ""} {"id": "19705", "image": [], "answer": "B", "solution": "null", "level": "高二", "question": "已知等比数列 $\\left\\{a_{n}\\right\\}$ 满足 $a_{1}+a_{3}=10, a_{2}+a_{4}=5$, 则 $a_{5}=(\\quad)$", "options": "A. 1\nB. $\\frac{1}{2}$\nC. $\\frac{1}{4}$\nD. 4", "subject": "算术", "analysis": "$\because$ 等比数列 $\\left\\{\\mathrm{a}_{\\mathrm{n}}\\right\\}$ 满足 $\\mathrm{a}_{1}+\\mathrm{a}_{3}=10, \\mathrm{a}_{2}+\\mathrm{a}_{4}=5, \\therefore\\left\\{\\begin{array}{c}a_{1}+a_{1} q^{2}=10\\ \\ a_{1} q+a_{1} q^{3}=5\\end{array}\\right.$,解得 $a_{1}=8, q=\\frac{1}{2}, a_{5}=a_{1} q^{4}=8 \\times \\frac{1}{16}=\\frac{1}{2}$. 故选 $B$."} {"id": "19708", "image": [], "answer": "D", "solution": "null", "level": "高二", "question": "数列 $\\left\\{a_{n}\\right\\}$ 为等比数列, 若 $a_{3}=-3, a_{4}=6$, 则 $a_{6}=(\\quad)$", "options": "A. -24\n B. 12\n C. 18\n D. 24", "subject": "算术", "analysis": "设等比数列 $\\left\\{\\mathrm{a}_{\\mathrm{n}}\\right\\}$ 的公比为 $\\mathrm{q}, \\because\\ \\mathrm{a}_{3}=-3, \\mathrm{a}_{4}=6, \\therefore \\mathrm{q}=\\frac{a_{4}}{a_{3}}=-2$,则 $\\mathrm{a}_{6}=a_{4} \\times q^{2}=6 \\times(-2)^{2}=24$. 故选: D."} {"id": "19710", "image": [], "answer": "A", "solution": "null", "level": "高二", "question": "等比数列中 $\\left\\{a_{n}\\right\\}, a_{1}, a_{5}$ 为方程 $x^{2}-10 x+16=0$ 的两根, 则 $a_{3}=c$", "options": "A. 4\n B. 5\n C. $\\pm 4$\n D. $\\pm 5$", "subject": "算术", "analysis": "$ \\because a_{1}, a_{5}$ 为方程 $x^{2}-10 x+16=0$ 的两根, $\\therefore a_{1}+a_{5}=10, a_{1} a_{5}=16$, 则 $a_{1}, a_{5}$ 为正数, 在等比数列中 $\\left\\{a_{n}\\right\\}$ 中, $a_{3}{ }^{2}=a_{1} a_{5}=16$, 则 $a_{3}= \\pm 4, \\because a_{1}, a_{5}$ 为正数, $\\therefore a_{3}=4$, 故选: A."} {"id": "19711", "image": [], "answer": "C", "solution": "null", "level": "高二", "question": "正项等比数列 $\\left\\{a_{n}\\right\\}$ 中, $a_{3}=2, a_{4} \\cdot a_{6}=64$, 则 $a_{5}+a_{1}+a_{2}$ 的值是", "options": "A. 4\n B. 8\n C. 16\n D. 64", "subject": "算术", "analysis": "设正项等比数列 $\\left\\{\\mathrm{a}_{\\mathrm{n}}\\right\\}$ 的公比为 $\\mathrm{q}, \\because \\mathrm{a}_{3}=2, \\mathrm{a}_{4} \\mathrm{a}_{6}=64$, $\\therefore a_{1} q^{2}=2, a_{1}^{2} q^{8}=64$, $\\therefore$ 解得 $\\mathrm{q}^{2}=4, \\therefore \\frac{a_{5}+a_{6}}{a_{1}+a_{2}}=\\frac{q^{4}\\left(a_{1}+a_{2}\\right)}{a_{1}+a_{2}}=q^{4}=16$. 故选 C"} {"id": "19712", "image": [], "answer": "D", "solution": "null", "level": "高二", "question": "已知等比数列 $\\left\\{a_{n}\\right\\}$ 为递增数列, $S_{n}$ 是其前 $n$ 项和. 若 $a_{1}+a_{5}=\\frac{17}{2}, a_{2} a_{4}=4$, 则 $S_{6}=$", "options": "A. $\\frac{27}{16}$\n B. $\\frac{27}{8}$\n C. $\\frac{63}{4}$\n D. $\\frac{63}{2}$", "subject": "算术", "analysis": "设递增的等比数列 $\\left\\{\\mathrm{a}_{\\mathrm{n}}\\right\\}$ 的公比为 $\\mathrm{q}$,$\\because \\mathrm{a}_{1}+\\mathrm{a}_{5}=\\frac{17}{2}, \\mathrm{a}_{2} \\mathrm{a}_{4}=4=\\mathrm{a}_{1} \\mathrm{a}_{5}, \\therefore$ 解得 $\\mathrm{a}_{1}=\\frac{1}{2}, \\mathrm{a}_{5}=8, \\therefore \\frac{a_{5}}{a_{1}}=q^{4}=16$, $\\because$ 等比数列 $\\{\\mathrm{an}\\}$ 为递增数列, $\\therefore$ 解得 $\\mathrm{q}=2, \\therefore \\mathrm{S}_{6}=\\frac{\\frac{1}{2}\\left(2^{6}-1\\right)}{2-1}=\\frac{63}{2}$. 故选 D."} {"id": "19713", "image": [], "answer": "B", "solution": "null", "level": "高二", "question": "等比数列 $\\left\\{a_{n}\\right\\}$ 的各项均为正数, 且 $a_{5} a_{6}+a_{4} a_{7}=18$, 则 $\\log _{3} a_{1}+\\log _{3} a_{2}+\\cdots \\log _{3} a_{10}=(\\quad)$", "options": "A. 12\n B. 10\n C. 8\n D. $2+\\log _{3} 5$", "subject": "算术", "analysis": "$\\because a_{5} a_{6}=a_{4} a_{7}, \\therefore a_{5} a_{6}+a_{4} a_{7}=2 a_{5} a_{6}=18, \\therefore a_{5} a_{6}=9$$\\therefore \\log _{3} \\mathrm{a}_{1}+\\log _{3} \\mathrm{a}_{2}+\\ldots \\log _{3} \\mathrm{a}_{10}=\\log _{3}\\left(\\mathrm{a}_{5} \\mathrm{a}_{6}\\right)^{5}=5 \\log _{3} 9=10$, 故选 $\\mathrm{B}$."} {"id": "19714", "image": [], "answer": "B", "solution": "null", "level": "高二", "question": "如果 -1, , , b, c, -9 成等比数列, 那么 ( )", "options": "A. $\\mathrm{b}=3, \\mathrm{ac}=9$\n B. $\\mathrm{b}=-3, \\mathrm{ac}=9$\n C. $\\mathrm{b}=3, \\mathrm{ac}=-9$\n D. $\\mathrm{b}=-3, \\mathrm{ac}=-9$", "subject": "算术", "analysis": "由等比数列的性质可得 $\\mathrm{ac}=(-1) \\times(-9)=9$,$\\mathrm{b} \\times \\mathrm{b}=9$ 且 $\\mathrm{b}$ 与奇数项的符号相同, $\\therefore \\mathrm{b}=-3$, 故选 $: \\mathrm{B}$"} {"id": "17997", "image": [], "answer": "B", "solution": "null", "level": "高二", "question": "人骑自行车的速度是 $v_{1}$, 风速为 $v_{2}$, 则逆风行驶的速度为", "options": "A. $v_{1}-v_{2}$\nB. $v_{1}+v_{2}$\nC. $\\left|v_{1}\\right|-\\left|v_{2}\\right|$\nD. $\\left|\\frac{v_{1}}{v_{2}}\\right|$", "subject": "算术", "analysis": "解析 : 由向量的加法法则可得逆风行驶的速度为 $v_{1}+v_{2}$. 注意速度是有方向和大小的,是一个向量.\n\n答案: B"} {"id": "19739", "image": [], "answer": "A", "solution": "null", "level": "高二", "question": "已知 $\\left\\{a_{n}\\right\\}$ 为等差数列, 若 $a_{1}+a_{5}+a_{9}=4 \\pi$, 则 $\\cos a_{5}$ 的值为 ( )", "options": "A. $-\\frac{1}{2}$\nB. $-\\frac{\\sqrt{3}}{2}$\nC. $\\frac{\\sqrt{3}}{2}$\nD. $\\frac{1}{2}$", "subject": "算术", "analysis": "$: \\because\\left\\{a_{n}\\right\\}$ 为等差数列, $a_{1}+a_{5}+a_{9}=4 \\pi, \\therefore 3 a_{5}=4 \\pi$, 解得 $a_{5}=\\frac{4 \\pi}{3}$.\n\n$\\therefore \\operatorname{cosa}_{5}=\\cos \\frac{4 \\pi}{3}=-\\frac{1}{2}$. 故选: A."} {"id": "19743", "image": [], "answer": "D", "solution": "null", "level": "高二", "question": "已知等差数列 $\\left\\{a_{n}\\right\\}$ 中, $a_{7}+a_{9}=16, a_{4}=1$, 则 $a_{12}$ 的值是 ( )", "options": "A. 64\nB. 31\nC. 30\nD. 15", "subject": "算术", "analysis": "设等差数列 $\\left\\{a_{n}\\right\\}$ 的公差为 $d, \\because a_{7}+a_{9}=16, a_{4}=1, \\therefore\\left\\{\\begin{array}{l}2 a_{1}+1 \\mu u=16 \\\\ a_{1}+3 d=1\\end{array}\\right.$,解得 $\\mathrm{a}_{1}=-\\frac{17}{4}, \\mathrm{~d}=\\frac{7}{4}$ 则 $\\mathrm{a}_{12}=-\\frac{17}{4}+\\frac{7}{4} \\times 11=15$. 故选: D."} {"id": "19744", "image": [], "answer": "C", "solution": "null", "level": "高二", "question": "已知等差数列 $\\left\\{a_{n}\\right\\}$, 且 $a_{10}=\\frac{1}{2} a_{12}+6, a_{2}=4$, 则 $a_{5}=(\\quad)$", "options": "A. 2\nB. 4\nC. 8\nD. 16", "subject": "算术", "analysis": "根据题意, 设等差数列 $\\left\\{\\mathrm{a}_{n}\\right\\}$ 的公差为 $\\mathrm{d}$,\n\n又由 $a_{10}=\\frac{1}{2} a_{12}+6, a_{2}=4$, 则 $(4+8 \\mathrm{~d})=\\frac{1}{2}(4+10 \\mathrm{~d})+6$,\n\n解可得: $\\mathrm{d}=\\frac{4}{3}$, 则 $\\mathrm{a}_{5}=\\mathrm{a}_{2}+3 \\mathrm{~d}=8$; 故选: $\\mathrm{C}$."} {"id": "19745", "image": [], "answer": "D", "solution": "null", "level": "高二", "question": "在等差数列 $\\left\\{a_{n}\\right\\}$ 中, $a_{3}, a_{15}$ 是方程 $x^{2}-6 x+8=0$ 的两个根, 则 $a_{7}+a_{8}+a_{9}+a_{10}+a_{11}$为 $(\\quad)$", "options": "A. 12\nB. 13\nC. 14\nD. 15", "subject": "算术", "analysis": "等差数列 $\\left\\{\\mathrm{a}_{n}\\right\\}$ 中, $\\mathrm{a}_{3}, \\mathrm{a}_{15}$ 是方程 $\\mathrm{x}^{2}-6 \\mathrm{x}+8=0$ 的两个根,\n\n$\\therefore \\mathrm{a}_{3}+\\mathrm{a}_{15}=2 \\mathrm{a}_{9}=6, \\quad \\therefore \\mathrm{a}_{9}=3 ; \\quad \\therefore \\mathrm{a}_{7}+\\mathrm{a}_{8}+\\mathrm{a}_{9}+\\mathrm{a}_{10}+\\mathrm{a}_{11}=\\left(\\mathrm{a}_{7}+\\mathrm{a}_{11}\\right)+\\left(\\mathrm{a}_{8}+\\mathrm{a}_{10}\\right)$\n\n$+a_{9}=5 a_{9}=5 \\times 3=15$."} {"id": "19746", "image": [], "answer": "C", "solution": "null", "level": "高二", "question": "等差数列 $\\left\\{a_{n}\\right\\}$ 中, $a_{2}=12, a_{n}=-20$, 公差 $d=-2$, 则项数 $n=(\\quad)$", "options": "A. 20\nB. 19\nC. 18\nD. 17", "subject": "算术", "analysis": "等差数列 $\\left\\{\\mathrm{a}_{\\mathrm{n}}\\right\\}$ 中, $\\mathrm{a}_{2}=12, \\mathrm{a}_{\\mathrm{n}}=-20$, 公差 $\\mathrm{d}=-2$,\n\n$\\therefore \\mathrm{a}_{\\mathrm{n}}=\\mathrm{a}_{2}+(\\mathrm{n}-2) \\mathrm{d}, \\therefore-20=12-2(\\mathrm{n}-2)$, 解得 $\\mathrm{n}=18$, 故选: $\\mathrm{C}$."} {"id": "19747", "image": [], "answer": "D", "solution": "null", "level": "高二", "question": "设 $x>0, y>0$, 若 $x \\lg 2, \\lg \\sqrt{2}, y \\lg 2$ 成等差数列, 则 $\\frac{1}{x}+\\frac{9}{y}$ 的最小值为", "options": "A. 8\nB. 9\nC. 12\nD. 16", "subject": "算术", "analysis": "$\\mathrm{x}>0, \\mathrm{y}>0$, 若 $x l g 2, l g \\sqrt{2}, y l g 2$ 成等差数列,\n\n可得 $2 \\lg \\sqrt{2}=x \\lg 2+y \\lg 2$, 即为 $x+y=1$,\n\n则 $\\frac{1}{x}+\\frac{9}{y}=(\\mathrm{x}+\\mathrm{y})\\left(\\frac{1}{x}+\\frac{9}{y}\\right)=10+\\frac{y}{x}+\\frac{9 x}{y} \\geq 10+2 \\sqrt{\\frac{y}{x} \\cdot \\frac{9 x}{y}}=16$, 当且仅当 $\\mathrm{y}=3 \\mathrm{x}=\\frac{3}{4}$ 时,上式取得等号, 则 $\\frac{1}{x}+\\frac{9}{y}$ 的最小值为 16 , 故选:D."} {"id": "19748", "image": [], "answer": "C", "solution": "null", "level": "高二", "question": "已知等差数列 $\\left\\{a_{n}\\right\\}$ 满足 $a_{1}+a_{2}+a_{3}+\\cdots+a_{101}=0$, 则有 $(\\quad)$", "options": "A. $a_{1}+a_{101}>0$\nB. $a_{2}+a_{100}<0$\nC. $a_{3}+a_{99}=0$\nD. $a_{51}=57$", "subject": "算术", "analysis": "数列列 $\\left\\{\\mathrm{a}_{\\mathrm{n}}\\right\\}$ 是等差数列,\n\n则: 当 $m+n=p+q$ 时, 则: $a_{m}+a_{n}=a_{p}+a_{q}$,\n\n由于等差数列 $\\left\\{a_{n}\\right\\}$ 满足 $a_{1}+a_{2}+a_{3}+\\ldots+a_{101}=0$,\n\n则: $a_{1}+a_{101}=a_{2}+a_{100}=a_{3}+a_{99}=0$. 故选 C."} {"id": "19749", "image": [], "answer": "D", "solution": "null", "level": "高二", "question": "《九章算术》卷第六《均输》中, 提到如下问题: “今有竹九节, 下三节容量四升上四节容量三升. 问中间二节欲均容, 各多少? ”其中 “欲均容” 的意思是: 使容量变化均匀, 即每节的容量成等差数列. 在这个问题中的中间两节容量分别是()", "options": "A. $\\frac{67}{66}$ 升、 $\\frac{41}{33}$ 升\nB. 2 升、 3 升\nC. $\\frac{3}{22}$ 升、 $\\frac{37}{33}$ 升\nD. $\\frac{67}{66}$ 升、 $\\frac{37}{33}$ 升", "subject": "算术", "analysis": "由题意知九节竹的容量成等差数列, 至下而上各节的容量分别为 $a_{1}, a_{2}, \\ldots, a_{9}$, 公差为 $d$,\n\n即 $\\left\\{\\begin{array}{c}a_{1}+a_{2}+a_{1}=4 \\\\ a_{6}+a_{7}+a_{8}+a_{9}=3\\end{array}, \\therefore\\left\\{\\begin{array}{c}3 a_{1}+31=4 \\\\ 4 a_{1}+26 d=3\\end{array}\\right.\\right.$, 解得 $\\mathrm{a}_{1}=\\frac{95}{66}, \\mathrm{~d}=-\\frac{7}{66}$,\n\n$\\therefore$ 中间两节的容量 $\\mathrm{a}_{5}=\\mathrm{a}_{1}+4 \\mathrm{~d}=\\frac{95}{66}-\\frac{28}{66}=\\frac{67}{66}, \\mathrm{a}_{4}=\\frac{37}{33}$, 故选 $: D$."} {"id": "19762", "image": [], "answer": "C", "solution": "null", "level": "高二", "question": "记 $S_{n}$ 为等差数列 $\\left\\{a_{n}\\right\\}$ 的前 $n$ 项和. 若 $a_{4}+a_{5}=24, S_{6}=48$, 则 $\\left\\{a_{n}\\right\\}$ 的公差为", "options": "A. 1\nB. 2\nC. 4\nD. 8", "subject": "算术", "analysis": "$\\because \\mathrm{S}_{\\mathrm{n}}$ 为等差数列 $\\left\\{\\mathrm{a}_{\\mathrm{n}}\\right\\}$ 的前 $\\mathrm{n}$ 项和, $\\mathrm{a}_{4}+\\mathrm{a}_{5}=24, \\mathrm{~S}_{6}=48$,\n\n$\\therefore\\left\\{\\begin{array}{l}a_{1}+3 d+a_{1}+4 d=24 \\\\ 6 a_{1}+\\frac{6 \\times 5}{2} d=48\\end{array}\\right.$, 解得 $\\mathrm{a}_{1}=-2, \\mathrm{~d}=4, \\therefore\\left\\{\\mathrm{a}_{\\mathrm{n}}\\right\\}$ 的公差为"} {"id": "19765", "image": [], "answer": "B", "solution": "null", "level": "高二", "question": "在等差数列 $\\left\\{a_{n}\\right\\}$ 中, $a_{1}=2, a_{3}+a_{5}=10$, 则 $a_{7}=(\\quad)$​", "options": "A. 5\nB. 8\nC. 10\nD. 14", "subject": "算术", "analysis": "$\\because$ 在等差数列 $\\left\\{a_{n}\\right\\}$ 中 $a_{1}=2, a_{3}+a_{5}=10, \\therefore 2 a_{4}=a_{3}+a_{5}=10$, 解得 $a_{4}=5$,$\\therefore$ 公差 $\\mathrm{d}=\\frac{a_{4}-a_{1}}{4-1}=1, \\therefore \\mathrm{a}_{7}=\\mathrm{a}_{1}+6 \\mathrm{~d}=2+6=8$, 故选 $\\mathrm{B}$."} {"id": "19766", "image": [], "answer": "D", "solution": "null", "level": "高二", "question": "已知等差数列 $\\left\\{a_{n}\\right\\}$ 的前 $n$ 项和为 $S_{n}, a_{5}+a_{7}=14$, 则 $S_{11}=(\\quad)$", "options": "A. 140\nB. 70\nC. 154\nD. 77", "subject": "算术", "analysis": "等差数列 $\\left\\{\\mathrm{a}_{n}\\right\\}$ 的前 $\\mathrm{n}$ 项和为 $\\mathrm{S}_{\\mathrm{n}}=\\frac{n\\left(a_{1}+a_{n}\\right)}{2}$, 又 $\\mathrm{a}_{5}+\\mathrm{a}_{7}=14$,\n\n$\\therefore \\mathrm{S}_{11}=\\frac{11}{2}\\left(\\mathrm{a}_{1}+\\mathrm{a}_{11}\\right)=\\frac{11}{2}\\left(a_{5}+a_{7}\\right)=\\frac{11}{2} \\times 14=77$. 故选 D."} {"id": "19767", "image": [], "answer": "D", "solution": "null", "level": "高二", "question": "在等差数列 $\\left\\{a_{n}\\right\\}$ 中, 若 $a_{5}, a_{7}$ 是方程 $x^{2}-2 x-6=0$ 的两根, 则 $\\left\\{a_{n}\\right\\}$ 的前 11 项的和为", "options": "A. 22\nB. -33\nC. -11\nD. 11", "subject": "算术", "analysis": "等差数列 $\\left\\{a_{n}\\right\\}$ 中, 若 $a_{5}, a_{7}$ 是方程 $x^{2}-2 x-6=0$ 的两根, 则 $a_{5}+a_{7}=2, \\therefore a_{6}=\\frac{1}{2}\\left(a_{5}+a_{7}\\right)$ $=1, \\therefore\\left\\{\\mathrm{a}_{n}\\right\\}$ 的前 11 项的和为 $\\mathrm{S}_{11}=\\frac{11 \\times\\left(a_{1}+a_{11}\\right)}{2}=11 \\mathrm{a}_{6}=11 \\times 1=11$. 故选 D"} {"id": "19768", "image": [], "answer": "C", "solution": "null", "level": "高二", "question": "等差数列 $\\left\\{a_{n}\\right\\}$ 和 $\\left\\{b_{n}\\right\\},\\left\\{a_{n}\\right\\}$ 和 $\\left\\{b_{n}\\right\\}$ 的前 $n$ 项和分别为 $S_{n}$ 与 $T_{n}$, 对一切自然数 $n$, 都有 $\\frac{\\mathrm{S}_{n}}{T_{n}}=$ $\\frac{\\mathrm{n}}{\\mathrm{n}+1}$ ,则 $\\mathrm{a}_{5} \\mathrm{a}_{5}$ 等于 $(\\quad)$", "options": "A. $\\frac{3}{4}$\nB. $\\frac{5}{6}$\nC. $\\frac{9}{10}$\nD. $\\frac{10}{11}$", "subject": "算术", "analysis": "$\\because \\mathrm{S}_{9}=\\frac{9\\left(a_{1}+a_{9}\\right)}{2}=9 \\mathrm{a}_{5}, \\mathrm{~T}_{9}=\\frac{9\\left(b_{1}+b_{9}\\right)}{2}=9 b_{5}, \\therefore \\mathrm{a}_{5}=\\frac{1}{9} \\mathrm{~S}_{9}, \\mathrm{~b}_{5}=\\frac{1}{9} \\mathrm{~T}_{9}$, 又 $\\because$ 当 $\\mathrm{n}=9$ 时, $\\frac{S_{9}}{T_{9}}$\n\n$=\\frac{9}{10}, \\quad \\therefore \\frac{a_{5}}{b_{5}}=\\frac{S_{9}}{T_{9}}=\\frac{9}{10}$, 故选 C."} {"id": "19769", "image": [], "answer": "A", "solution": "null", "level": "高二", "question": "在数列 $\\left\\{a_{n}\\right\\}$ 中, $a_{1}=2,2 a_{n+1}-2 a_{n}=1$, 则 $a_{101}$ 的值为", "options": "A. 52\nB. 51\nC. 50\nD. 49", "subject": "算术", "analysis": "在数列 $\\left\\{a_{n}\\right\\}$ 中, $a_{1}=2,2 a_{n+1}-2 a_{n}=1, \\therefore a_{n+1}-a_{n}=\\frac{1}{2}$, 故数列 $\\left\\{a_{n}\\right\\}$ 为首项为 2 公差为 $\\frac{1}{2}$ 的等差数列, $\\therefore \\mathrm{a}_{101}=2+100 \\times \\frac{1}{2}=52$. 故选 A."} {"id": "19773", "image": [], "answer": "B", "solution": "null", "level": "高二", "question": "等差数列 $\\left\\{a_{n}\\right\\}$ 中, $a_{7}+a_{9}=16, a_{4}=2$, 则 $a_{12}=(\\quad)$", "options": "A. 10\nB. 14\nC. 15\nD. 30", "subject": "算术", "analysis": "设等差数列 $\\left\\{a_{n}\\right\\}$ 的公差为 $d, \\because a_{7}+a_{9}=16, a_{4}=2, \\therefore 2 a_{1}+14 d=16, a_{1}+3 d=2$. 解得 $\\mathrm{a}_{1}=-\\frac{5}{2}, \\mathrm{~d}=\\frac{3}{2}$. 则 $\\mathrm{a}_{12}=-\\frac{5}{2}+11 \\times \\frac{3}{2}=14$. 故选 $: \\mathrm{B}$."} {"id": "19776", "image": [], "answer": "A", "solution": "null", "level": "高二", "question": "一已知等差数列 $\\left\\{a_{n}\\right\\}$ 中, 其前 $n$ 项和为 $S_{n}$, 若 $a_{3}+a_{4}+a_{5}=42$, 则 $S_{7}=(\\quad)$", "options": "A. 98\nB. 49\nC. 14\nD. 147", "subject": "算术", "analysis": "等差数列 $\\left\\{a_{n}\\right\\}$ 中, 因为 $a_{3}+a_{4}+a_{5}=42$, 所以 $a_{4}=42$, 解得 $a_{4}=14$,\n\n所以 $\\mathrm{S}_{7}=\\frac{7\\left(a_{1}+a_{7}\\right)}{2}=7 \\mathrm{a}_{4}=7 \\times 14=98$, 故选 A."} {"id": "19777", "image": [], "answer": "D", "solution": "null", "level": "高二", "question": "等差数列 $\\left\\{a_{n}\\right\\}$ 的前 $n$ 项和为 $S_{n}$, 若 $a_{1}=\\frac{1}{2}, S_{4}=20$, 则 $S_{6}=(\\quad)$", "options": "A. 16\nB. 24\nC. 36\nD. 48", "subject": "算术", "analysis": "$\\because a_{1}=\\frac{1}{2}, \\mathrm{~S}_{4}=20, \\therefore \\mathrm{S}_{4}=2+6 \\mathrm{~d}=20, \\quad \\therefore \\mathrm{d}=3, \\therefore \\mathrm{S}_{6}=3+15 \\mathrm{~d}=48$. 故选: $\\mathrm{D}$."} {"id": "19778", "image": [], "answer": "C", "solution": "null", "level": "高二", "question": "如果等差数列 $\\left\\{a_{n}\\right\\}$ 中, $a_{3}+a_{4}+a_{5}=12$, 那么 $a_{1}+a_{2}+\\cdots+a_{7}=(\\quad)$", "options": "A. 14\nB. 21\nC. 28\nD. 35", "subject": "算术", "analysis": "$\\mathrm{a}_{3}+\\mathrm{a}_{4}+\\mathrm{a}_{5}=3 \\mathrm{a}_{4}=12, \\mathrm{a}_{4}=4, \\therefore \\mathrm{a}_{1}+\\mathrm{a}_{2}+\\ldots+\\mathrm{a}_{7}=\\frac{7\\left(a_{1}+a_{7}\\right)}{2}=7 \\mathrm{a}_{4}=28$ ,故选 C"} {"id": "19779", "image": [], "answer": "C", "solution": "null", "level": "高二", "question": "已知等差数列 $\\left\\{a_{n}\\right\\}$ 中, $a_{1}+a_{2}+a_{3}+\\cdots a_{100}=0$, 则 ( )", "options": "A. $\\mathrm{a}_{1}+\\mathrm{a}_{101}>0$\nB. $a_{2}+a_{100}<0$\nC. $a_{3}+a_{98}=0$\nD. $a_{5}=51$", "subject": "算术", "analysis": "等差数列 $\\left\\{a_{n}\\right\\}$ 中, $a_{1}+a_{2}+a_{3}+\\ldots+a_{100}=50\\left(a_{1}+a_{100}\\right)=50\\left(a_{3}+a_{98}\\right)=0$. 故选:C."} {"id": "19781", "image": [], "answer": "A", "solution": "null", "level": "高二", "question": "在等差数列 $\\left\\{a_{n}\\right\\}$ 中, $a_{1}=-2015$, 其前 $n$ 项和为 $S_{n}$, 若 $\\frac{S_{12}}{12}-\\frac{S_{10}}{10}=2$, 则 $S_{2018}$ 的值等于 ( )", "options": "A. 4036\nB. 2018\nC. 2017\nD. $-2 \\quad 017$", "subject": "算术", "analysis": "设等差数列 $\\left\\{\\mathrm{a}_{\\mathrm{n}}\\right\\}$ 的公差为 $\\mathrm{d}$, 由 $\\frac{S_{12}}{12}-\\frac{S_{10}}{10}=2$,\n\n得 $\\frac{\\left(a_{1}+a_{12}\\right) \\times 12}{2}-\\frac{\\left(a_{1}+a_{12}\\right) \\times 10}{2}=2$, 即 $\\frac{a_{1}+a_{12}}{2}-\\frac{a_{1}+a_{10}}{2}=2, \\therefore \\mathrm{a}_{12}-\\mathrm{a}_{10}=4$, 则 $2 \\mathrm{~d}=4, \\mathrm{~d}=2$.\n\n$\\therefore \\mathrm{S}_{2018}=2018 \\times(-2015)+\\frac{2018 \\times 2017}{2} \\times 2=4036$. 故选 $: \\mathrm{A}$."} {"id": "19782", "image": [], "answer": "A", "solution": "null", "level": "高二", "question": "若一个等差数列前 3 项的和为 34 , 最后 3 项的和为 146 , 且所有项的和为 390 , 则这个数列有", "options": "A. 13 项\nB. 12 项\nC. 11 项\nD. 10 项", "subject": "算术", "analysis": "依题意 $a_{1}+a_{2}+a_{3}=34, a_{n}+a_{n-1}+a_{n-2}=146$\n\n$\\therefore a_{1}+a_{2}+a_{3}+a_{n}+a_{n-1}+a_{n-2}=34+146=180$, 又 $\\because a_{1}+a_{n}=a_{2}+a_{n-1}=a_{3}+a_{n-2} \\quad \\therefore a_{1}+a_{n}=\\frac{180}{3}=60, \\quad \\therefore S_{n}$ $=\\frac{\\left(a_{1}+a_{n}\\right) n}{2}=\\frac{60 n}{2}=390 \\quad \\therefore \\mathrm{n}=13$, 故选 A"} {"id": "19784", "image": [], "answer": "D", "solution": "null", "level": "高二", "question": "对任意等比数列 $\\left\\{a_{n}\\right\\}$, 下列说法一定正确的是()", "options": "A. $a_{1}, a_{3}, a_{9}$ 成等比数列\nB. $a_{2}, a_{3}, a_{6}$ 成等比数列\nC. $a_{2}, a_{4}, a_{8}$ 成等比数列\nD. $a_{3}, a_{6}, a_{9}$ 成等比数列", "subject": "算术", "analysis": "A 项中 $\\mathrm{a}_{3}=\\mathrm{a}_{1} \\cdot \\mathrm{q}^{2}, \\mathrm{a}_{1} \\cdot \\mathrm{a}_{9}=a_{1}^{2} \\cdot \\mathrm{q}^{8},\\left(\\mathrm{a}_{3}\\right)^{2} \\neq \\mathrm{a}_{1} \\cdot \\mathrm{a}_{9}$, 故 A 项说法错误,\n\nB 项中 $\\left(\\mathrm{a}_{3}\\right)^{2}=\\left(\\mathrm{a}_{1} \\cdot \\mathrm{q}^{2}\\right)^{2} \\neq \\mathrm{a}_{2} \\cdot \\mathrm{a}_{6}=a_{1}^{2} \\cdot \\mathrm{q}^{6}$, 故 B 项说法错误,\n\nC 项中 $\\left(\\mathrm{a}_{4}\\right)^{2}=\\left(\\mathrm{a}_{1} \\cdot \\mathrm{q}^{3}\\right)^{2} \\neq \\mathrm{a}_{2} \\cdot \\mathrm{a}_{8}=a_{1}^{2} \\cdot \\mathrm{q}^{8}$, 故 C 项说法错误,\n\nD 项中 $\\left(\\mathrm{a}_{6}\\right)^{2}=\\left(\\mathrm{a}_{1} \\cdot \\mathrm{q}^{5}\\right)^{2}=\\mathrm{a}_{3} \\cdot \\mathrm{a}_{9}=a_{1}^{2} \\cdot \\mathrm{q}^{10}$, 故 D 项说法正确, 故选: D."} {"id": "19787", "image": [], "answer": "B", "solution": "null", "level": "高二", "question": "如果-1, $a, b, c,-9$ 成等比数列, 那么()", "options": "A. $b=3, a c=9$\nB. $b=-3, a c=9$\nC. $b=3, a c=-9$\nD. $b=-3, a c=-9$", "subject": "算术", "analysis": "由等比数列的性质可得 $a c=(-1) \\times(-9)=9, b \\times b=9$ 且 $b$ 与奇数项的符号相同, $\\therefore \\mathrm{b}=-3$, 故选: B."} {"id": "19788", "image": [], "answer": "A", "solution": "null", "level": "高二", "question": "等比数列 $\\left\\{a_{n}\\right\\}$ 中, $a_{1}=\\frac{1}{8}, q=2$, 则 $a_{4}$ 与 $a_{8}$ 的等比中项是 ()", "options": "A. $\\pm 4$\nB. 4\nC. $\\pm \\frac{1}{4}$\nD. $\\frac{1}{4}$", "subject": "算术", "analysis": "等比数列 $\\left\\{\\mathrm{a}_{n}\\right\\}$ 中, $\\mathrm{a}_{1}=\\frac{1}{8}, \\mathrm{q}=2, \\quad \\therefore a_{n}=a_{1} q^{n-1}=\\frac{1}{8} \\times 2^{n-1}=2^{n-4}$, $\\therefore \\mathrm{a}_{4}=1, \\mathrm{a}_{8}=16,1,16$ 的等比中项有 $\\pm 4$. 故选 A."} {"id": "19789", "image": [], "answer": "B", "solution": "null", "level": "高二", "question": "等比数列 $\\left\\{a_{n}\\right\\}$ 的各项均为正数, 且 $a_{5} a_{6}+a_{4} a_{7}=18$, 则 $\\log _{3} a_{1}+\\log _{3} a_{2}+\\cdots \\log _{3} a_{10}=(\\quad)$", "options": "A. 12\nB. 10\nC. 8\nD. $2+\\log _{3} 5$", "subject": "算术", "analysis": "$\\because \\mathrm{a}_{5} \\mathrm{a}_{6}=\\mathrm{a}_{4} \\mathrm{a}_{7}, \\quad \\therefore \\mathrm{a}_{5} \\mathrm{a}_{6}+\\mathrm{a}_{4} \\mathrm{a}_{7}=2 \\mathrm{a}_{5} \\mathrm{a}_{6}=18 \\quad \\therefore \\mathrm{a}_{5} \\mathrm{a}_{6}=9$\n\n$\\therefore \\log _{3} \\mathrm{a}_{1}+\\log _{3} \\mathrm{a}_{2}+\\ldots \\log _{3} \\mathrm{a}_{10}=\\log _{3}\\left(\\mathrm{a}_{5} \\mathrm{a}_{6}\\right)^{5}=5 \\log _{3} 9=10 \\quad$ 故选 B."} {"id": "19790", "image": [], "answer": "D", "solution": "null", "level": "高二", "question": "已知等比数列 $\\left\\{a_{n}\\right\\}$ 为递增数列, $S_{n}$ 是其前 $n$ 项和. 若 $a_{1}+a_{5}=\\frac{17}{2}, a_{2} a_{4}=4$,则 $S_{6}=(\\quad)$", "options": "A. $\\frac{27}{16}$\nB. $\\frac{27}{8}$\nC. $\\frac{63}{4}$\nD. $\\frac{63}{2}$", "subject": "算术", "analysis": "设递增的等比数列 $\\left\\{\\mathrm{a}_{n}\\right\\}$ 的公比为 $\\mathrm{q}, \\because \\mathrm{a}_{1}+\\mathrm{a}_{5}=\\frac{17}{2}, \\mathrm{a}_{2} \\mathrm{a}_{4}=4=\\mathrm{a}_{1} \\mathrm{a}_{5}$,\n\n$\\therefore$ 解得 $\\mathrm{a}_{1}=\\frac{1}{2}, \\mathrm{a}_{5}=8, \\quad \\therefore \\frac{a_{5}}{a_{1}}=q^{4}=16, \\therefore$ 等比数列 $\\left\\{\\mathrm{a}_{\\mathrm{n}}\\right\\}$ 为递增数列, $\\therefore$ 解得 $\\mathrm{q}=2$, $\\therefore \\mathrm{S}_{6}=\\frac{\\frac{1}{2}\\left(2^{6}-1\\right)}{2-1}=\\frac{63}{2}$. 故选 D."} {"id": "19791", "image": [], "answer": "C", "solution": "null", "level": "高二", "question": "已知等比数列 $\\left\\{a_{n}\\right\\}$ 中, $a_{1}+a_{2}=3, a_{3}+a_{4}=12$, 则 $a_{5}+a_{6}=(\\quad)$", "options": "A. 3\nB. 15\nC. 48\nD. 63", "subject": "算术", "analysis": "$\\because \\mathrm{a}_{1}+\\mathrm{a}_{2}=3, \\mathrm{a}_{3}+\\mathrm{a}_{4}=12, \\therefore\\left(\\mathrm{a}_{1}+\\mathrm{a}_{2}\\right) \\mathrm{q}^{2}=\\mathrm{a}_{3}+\\mathrm{a}_{4}$, 即 $\\mathrm{q}^{2}=4$,则 $a_{5}+a_{6}=\\left(a_{3}+a_{4}\\right) q^{2}=12 \\times 4=48$, 故选 : C."} {"id": "19792", "image": [], "answer": "C", "solution": "null", "level": "高二", "question": "正项等比数列 $\\left\\{a_{n}\\right\\}$ 中, $a_{3}=2, a_{4} \\cdot a_{6}=64$, 则 $\\frac{a_{5}+a_{6}}{a_{1}+a_{2}}$ 的值是 ( )", "options": "A. 4\nB. 8\nC. 16\nD. 64", "subject": "算术", "analysis": "设正项等比数列 $\\left\\{a_{n}\\right\\}$ 的公比为 q, $\\because \\mathrm{a}_{3}=2, a_{4} \\cdot a_{6}=64$,\n\n$\\therefore a_{1} q^{2}=2, a_{1}^{2} q^{8}=64, \\therefore$ 解得 $\\mathrm{q}^{2}=4, \\therefore \\frac{a_{5}+a_{6}}{a_{1}+a_{2}}=\\frac{q^{4}\\left(a_{1}+a_{2}\\right)}{a_{1}+a_{2}}=q^{4}=16$. 故选 C."} {"id": "19793", "image": [], "answer": "D", "solution": "null", "level": "高二", "question": "设等比数列 $\\left\\{a_{n}\\right\\}$ 的前 $n$ 项和为 $S_{n}$, 且满足 $a_{6}=8 a_{3}$, 则 $\\frac{S_{6}}{S_{3}}=(\\quad)$", "options": "A. 4\nB. 5\nC. 8\nD. 9", "subject": "算术", "analysis": "等比数列 $\\left\\{\\mathrm{a}_{\\mathrm{n}}\\right\\}$ 的前 $\\mathrm{n}$ 项和为 $\\mathrm{S}_{\\mathrm{n}}$, 且满足 $\\mathrm{a}_{6}=8 \\mathrm{a}_{3}, \\quad \\therefore \\frac{a_{6}}{a_{3}}=\\mathrm{q}^{3}=8$, 解得 $\\mathrm{q}=2, \\quad \\therefore \\frac{S_{6}}{S_{3}}$ $=\\frac{1-q^{6}}{1-q^{3}}=1+\\mathrm{q}^{3}=9$. 故选: D."} {"id": "18039", "image": [], "answer": "B", "solution": "null", "level": "高二", "question": "$\\cos 555^{\\circ}$ 的值是", "options": "A. $\\frac{\\sqrt{6}}{4}+\\frac{\\sqrt{2}}{4}$\nB. $-\\frac{\\sqrt{6}}{4}-\\frac{\\sqrt{2}}{4}$\nC. $\\frac{\\sqrt{6}}{2}-\\frac{\\sqrt{2}}{2}$\nD. $\\frac{\\sqrt{2}}{2}-\\frac{\\sqrt{6}}{2}$", "subject": "算术", "analysis": "解析: $\\because \\cos 555^{\\circ}=\\cos 195^{\\circ}=-\\cos 15^{\\circ}=-\\cos \\left(45^{\\circ}-30^{\\circ}\\right)=-\\frac{\\sqrt{2}}{2} \\times \\frac{\\sqrt{3}}{2}-\\frac{\\sqrt{2}}{2} \\times \\frac{1}{2}=-$ $\\frac{\\sqrt{6}+\\sqrt{2}}{4}$.故选 B.\n\n答案: B"} {"id": "18048", "image": [], "answer": "D", "solution": "null", "level": "高二", "question": "已知 $\\cos \\left(x-\\frac{\\pi}{6}\\right)=\\frac{1}{3}$, 则 $\\cos x+\\cos \\left(x-\\frac{\\pi}{3}\\right)=$", "options": "A. $\\frac{\\sqrt{3}}{2}$\n B. $\\sqrt{3}$\n C. $\\frac{1}{2}$\n D. $\\frac{\\sqrt{3}}{3}$", "subject": "算术", "analysis": "解析: $\\cos x+\\cos \\left(x-\\frac{\\pi}{3}\\right)=\\cos \\left[\\left(x-\\frac{\\pi}{6}\\right)+\\frac{\\pi}{6}\\right]+$ $\\cos \\left[\\left(x-\\frac{\\pi}{6}\\right)-\\frac{\\pi}{6}\\right]=2 \\cos \\left(x-\\frac{\\pi}{6}\\right) \\cos \\frac{\\pi}{6}=\\frac{\\sqrt{3}}{3}$ ,故选 D.\n\n答案: D"} {"id": "18054", "image": [], "answer": "D", "solution": "null", "level": "高二", "question": "下列各式中, 不正确的是", "options": "A. $\\sin \\left(\\frac{\\pi}{4}+\\frac{\\pi}{3}\\right)=\\sin \\frac{\\pi}{4} \\cos \\frac{\\pi}{3}+\\frac{\\sqrt{3}}{2} \\cos \\frac{\\pi}{4}$\nB. $\\cos \\frac{5 \\pi}{12}=\\frac{\\sqrt{2}}{2} \\sin \\frac{\\pi}{3}-\\cos \\frac{\\pi}{4} \\cos \\frac{\\pi}{3}$\nC. $\\cos \\left(-\\frac{\\pi}{12}\\right)=\\cos \\frac{\\pi}{4} \\cos \\frac{\\pi}{3}+\\frac{\\sqrt{6}}{4}$\nD. $\\cos \\frac{\\pi}{12}=\\cos \\frac{\\pi}{3}-\\cos \\frac{\\pi}{4}$", "subject": "算术", "analysis": "解析: $\\because \\sin \\frac{\\pi}{3}=\\frac{\\sqrt{3}}{2} , \\therefore \\mathrm{A}$ 正确; $\\because \\cos \\frac{5 \\pi}{12}=-\\cos \\frac{7 \\pi}{12}=-\\cos \\left(\\frac{\\pi}{3}+\\frac{\\pi}{4}\\right), \\therefore$ B 正确; $\\because \\cos \\left(-\\frac{\\pi}{12}\\right)=$ $\\cos \\left(\\frac{\\pi}{4}-\\frac{\\pi}{3}\\right) , \\therefore$ C 正确; $\\because \\cos \\frac{\\pi}{12}=\\cos \\left(\\frac{\\pi}{3}-\\frac{\\pi}{4}\\right) \\neq \\cos \\frac{\\pi}{3}-\\cos \\frac{\\pi}{4} , \\therefore$ D 不正确.\n\n答案: D"} {"id": "18056", "image": [], "answer": "A", "solution": "null", "level": "高二", "question": "若 $\\sqrt{3} \\sin x+\\cos x=4-m$, 则实数 $m$ 的取值范围是", "options": "A. $[2,6]$\nB. $[-6,6]$\nC. $(2,6)$\nD. $[2,4]$", "subject": "算术", "analysis": "解析: $\\because \\sqrt{3} \\sin x+\\cos x=4-m$,\n\n$\\therefore \\frac{\\sqrt{3}}{2} \\sin x+\\frac{1}{2} \\cos x=\\frac{4-m}{2}$,\n\n$\\therefore \\sin \\frac{\\pi}{3} \\sin x+\\cos \\frac{\\pi}{3} \\cos x=\\frac{4-m}{2}$,\n\n$\\therefore \\cos \\left(x-\\frac{\\pi}{3}\\right)=\\frac{4-m}{2}$.\n\n$\\because\\left|\\cos \\left(x-\\frac{\\pi}{3}\\right)\\right| \\leqslant 1, \\quad \\therefore\\left|\\frac{4-m}{2}\\right| \\leqslant 1, \\quad \\therefore 2 \\leqslant m \\leqslant 6$.\n\n答案: A"} {"id": "18083", "image": [], "answer": "C", "solution": "null", "level": "高二", "question": "$\\cos ^{2} 75^{\\circ}+\\cos ^{2} 15^{\\circ}+\\cos 75^{\\circ} \\cos 15^{\\circ}=$", "options": "A.$\\frac{\\sqrt{6}}{2} $$ B. \\frac{3}{2} $ C. $\\frac{5}{4}$\nD. $1+\\frac{\\sqrt{3}}{4}$", "subject": "算术", "analysis": "解析: 原式 $=\\sin ^{2} 15^{\\circ}+\\cos ^{2} 15^{\\circ}+\\sin 15^{\\circ} \\cos 15^{\\circ}=1+\\frac{1}{2} \\sin 30^{\\circ}=1+\\frac{1}{4}=\\frac{5}{4}$.\n\n答案: C"} {"id": "18098", "image": [], "answer": "D", "solution": "null", "level": "高二", "question": "函数 $f(x)=2 \\sin \\frac{x}{2} \\sin \\left(\\frac{\\pi}{3}-\\frac{x}{2}\\right)$ 的最小值是", "options": "A. $\\frac{1}{2}$\nB. $\\frac{3}{2}$\nC. $-\\frac{1}{2}$\nD. $-\\frac{3}{2}$", "subject": "算术", "analysis": "解析: $\\because f(x)=2 \\sin \\frac{x}{2} \\cdot \\sin \\left(\\frac{\\pi}{3}-\\frac{x}{2}\\right)=$\n\n$-\\left[\\cos \\left(\\frac{x}{2}+\\frac{\\pi}{3}-\\frac{x}{2}\\right)-\\cos \\left(\\frac{x}{2}-\\frac{\\pi}{3}+\\frac{x}{2}\\right)\\right]=$\n\n$\\cos \\left(x-\\frac{\\pi}{3}\\right)-\\frac{1}{2}, \\therefore y_{\\min }=-1-\\frac{1}{2}=-\\frac{3}{2}$.\n\n答案: D"} {"id": "19862", "image": [], "answer": "C", "solution": "null", "level": "高二", "question": "如果 $a, b, c$ 满足 $c\\mathrm{ac}$\nB. $b c>a c$\nC. $\\mathrm{cb}^{2}<\\mathrm{ab}^{2}$\nD. $\\mathrm{ac}(\\mathrm{a}-\\mathrm{c})<0$", "subject": "算术", "analysis": "$ \\because c0, c<0, \\quad \\therefore a b-a c=a(b-c)>0, \\quad b c-a c=(b-a) c>0, \\quad a c(a-c)<0$,\n\n$\\therefore \\mathrm{A}, \\mathrm{B}, \\mathrm{D}$ 中的不等式均恒成立, $\\because \\mathrm{b}$ 可能等于 0 , 也可能不等于 $0, \\therefore \\mathrm{cb}^{2}<\\mathrm{ab}^{2}$ 不一定成立, 故选 $\\mathrm{C}$."} {"id": "19863", "image": [], "answer": "B", "solution": "null", "level": "高二", "question": "若 $a\\frac{1}{b}$\nB. $2^{\\mathrm{a}}>2^{\\mathrm{b}}$\nC. $|a|>|b|$\nD. $\\left(\\frac{1}{2}\\right)^{a}>\\left(\\frac{1}{2}\\right)^{b}$", "subject": "算术", "analysis": "对于A, 由 $a0$, 因此 $a \\cdot \\frac{1}{a b}\\frac{1}{b}$, 即 $a$ 成立, 故 $A^{\\text {正确; }}$\n\n对于C, 由 $a-b>0$, 因此 $|a|>|b|>0$ 成立, 故C正确;\n\n对于D, 因为 $\\left.\\mathrm{a}<\\mathrm{b}, \\mathrm{y}=\\left(\\frac{1}{2}\\right)^{\\mathrm{x}}\\right)^{\\mathrm{x}}$ 是减函数, 所以 $\\left(\\frac{1}{2}\\right)^{\\mathrm{a}}>\\left(\\frac{1}{2}\\right)^{\\mathrm{b}}$ 成立, 故D正确,\n\n对于 $\\mathrm{B}$, 因为 $\\mathrm{a}<\\mathrm{b}, \\mathrm{y}=2^{\\mathrm{x}}$ 是增函数, 所以 $\\mathrm{2}^{\\mathrm{a}}<2^{\\mathrm{b}}$, 选项 B不成立, 故选 B."} {"id": "19866", "image": [], "answer": "C", "solution": "null", "level": "高二", "question": "设 $a, b, c \\in R$, 且 $a>b$, 则下列不等式成立的是", "options": "A. $a^{2}>b^{2}$\nB. $a c^{2}>b c^{2}$\nC. $a+c>b+c$\nD. ${ }^{\\frac{1}{a}<\\frac{1}{b}}$", "subject": "算术", "analysis": "$ \\because 1>-2$, 但是 $\\frac{1}{1}<\\frac{1}{-2}$ 不成立, 故D不正确; $\\because 1>-2$, 但是 $(-1)^{2}>(-2)^{2}$ 不成立, 故A不正确;\n\n$\\because a>b, \\therefore a+c>b+c, c_{\\text {正确; }} c=0$ 时, $0=a c^{2}>b c^{2}=0$, 不成立, 故选 $B$."} {"id": "19841", "image": [], "answer": "D", "solution": "null", "level": "高二", "question": "对于实数 $a, b, c$, 下列结论中正确的是 ( )", "options": "A. 若 $a>b$, 则 $a c^{2}>b c^{2}$\nB. 若 $a>b>0$, 则 $\\frac{1}{a}>\\frac{1}{b}$\nC. 若 $ab, \\frac{1}{a}>\\frac{1}{b}$, 则 $a b<0$", "subject": "算术", "analysis": "对于 $\\mathrm{A}, a>b$, 若 $c=0$, 则 $a c^{2}=b c^{2}$, 故 $\\mathrm{A}$ 错;\n\n对于 B, $a>b>0$, 取 $a=2, b=1, \\frac{1}{2}<1$, 即 $\\frac{1}{a}<\\frac{1}{b}$, 故 B错;\n\n对于 C, $a\\frac{1}{2}$, 即 $\\frac{a}{b}>\\frac{b}{a}$, 故 C 错;\n\n对于 $\\mathrm{D}$, 若 $a>b$, 则 $a-b>0$, 又 $\\frac{1}{a}>\\frac{1}{b}$, 所以 $\\frac{1}{a}-\\frac{1}{b}>0$,\n\n所以 $\\frac{b-a}{a b}>0$, 又 $a-b>0$, 所以 $a b<0$, 故 D 正确."} {"id": "19847", "image": [], "answer": "B", "solution": "null", "level": "高二", "question": "若 $a>b, c>d$, 则不等式一定成立的是 ( )", "options": "A. $a-c>b-d$\nB. $a+c>b+d$\nC. $a c>b d$\nD. $|a|>|b|$", "subject": "算术", "analysis": "由同向不等式的可加性可知 $a>b, c>d \\Rightarrow a+c>b+d$ 成立. 故选 B.\n\n考点: 不等式的基本性质."} {"id": "19848", "image": [], "answer": "D", "solution": "null", "level": "高二", "question": "设 $a=2^{0.1}, b=\\lg \\frac{3}{2}, c=\\ln \\frac{9}{10}$, 则 $a, b, c$ 的大小关系是 ( )", "options": "A. $b>c>a$\nB. $a>c>b$\nC. $b>a>c$\nD. $a>b>c$", "subject": "算术", "analysis": "D"} {"id": "19852", "image": ["9260.jpg"], "answer": "B", "solution": "null", "level": "高二", "question": "已知 $a, b, c, d \\in R$, 则下列说法中一定正确的是 ( )", "options": "A. 若 $a>b, c>b$, 则 $a>c$\nB. 若 $a>-b$, 则 $c-ab, cb^{2}$, 则 $-a<-b$", "subject": "算术", "analysis": "B 解析 对于 $A$, 若 $a=4, b=2, c=5$, 显然 $a>c$ 不成立; 对于B, 若 $a>-b$, 则 $-a\n不成立, 故选 B."} {"id": "19874", "image": [], "answer": "A", "solution": "null", "level": "高二", "question": "已知 $-1 \\leq a \\leq 3,2 \\leq b \\leq 4$, 则 $2 a-b$ 的取值范围是 ( )", "options": "A. $[-6,4]$\nB. $[0,10]$\nC. $[-4,2]$\nD. $\\left[\\begin{array}{ll}-5, & 1\\end{array}\\right]$", "subject": "算术", "analysis": "$\\because-1 \\leq a \\leq 3, \\therefore-2 \\leq 2 a \\leq 6$, 又 $\\because 2 \\leq b \\leq 4, \\therefore-4 \\leq-b \\leq-2, \\therefore-6=-2-4 \\leq 2 a-b \\leq 6-2=4$,\n\n即 $-6 \\leq 2 \\mathrm{a}-\\mathrm{b} \\leq 4, \\therefore 2 \\mathrm{a}-\\mathrm{b}$ 的取值范围是 $[-6,4]$; 故选 A."} {"id": "19877", "image": [], "answer": "C", "solution": "null", "level": "高二", "question": "已知集合 $A=\\left\\{x \\mid-x^{2}+4 x \\geq 0\\right\\}, B=\\left\\{x \\left\\lvert\\, \\frac{1}{81}<3^{x}<27\\right.\\right\\}, C=\\{x \\mid x=2 n, n \\in N\\}$, 则 $(A \\cup B)$ $\\cap C=(\\quad)$", "options": "A. $\\{2,4\\}$\nB. $\\{0,2\\}$\nC. $\\{0,2,4\\}$\nD. $\\{x \\mid x=2 n, n \\in N\\}$", "subject": "算术", "analysis": "$\\mathrm{A}=\\left\\{\\mathrm{x} \\mid-\\mathrm{x}^{2}+4 \\mathrm{x} \\geq 0\\right\\}=\\{\\mathrm{x} \\mid 0 \\leq \\mathrm{x} \\leq 4\\}$,\n\n$B=\\left\\{x \\left\\lvert\\, \\frac{1}{81}<3^{x}<27\\right.\\right\\}=\\left\\{x \\mid 3^{-4}<3^{x}<3^{3}\\right\\}=\\{x \\mid-40$, 且 $\\left|a_{10}\\right|0$,\n\n$\\therefore a_{1} a_{2} a_{3}=(5-d) \\cdot 5 \\cdot(5+d)=80$,\n\n$\\therefore d=3, a_{1}=2$.\n\n$\\therefore a_{11}+a_{12}+a_{13}=3 a_{12}=3\\left(a_{1}+11 d\\right)$\n\n$=3 a_{1}+33 d=3 \\times 2+33 \\times 3=105$."} {"id": "18715", "image": [], "answer": "A", "solution": "null", "level": "高二", "question": "若 $\\left\\{a_{n}\\right\\}$ 为等差数列, $S_{n}$ 为其前 $n$ 项和, 若 $a_{1}>0, d<0, S_{4}=S_{8}$, 则 $S_{n}>0$ 成立的最大自然数 $n$ 为 ()", "options": "A. 11\nB. 12\nC. 13\nD. 14", "subject": "算术", "analysis": "解析 $S_{4}=S_{8} \\Rightarrow a_{5}+a_{6}+a_{7}+a_{8}=0 \\Rightarrow a_{6}+a_{7}=0$, 又 $a_{1}>0, d<0, S_{12}=\\frac{\\left(a_{1}+a_{12}\\right) \\cdot 12}{2}=0$, $n<12$ 时,\n\n$S_{n}>0$."} {"id": "18716", "image": [], "answer": "D", "solution": "null", "level": "高二", "question": "在等差数列 $\\left\\{a_{n}\\right\\}$ 中, $a_{1}=-2008$, 其前 $n$ 项和为 $S_{n}$, 若 $\\frac{S_{2} 008}{2008}-\\frac{S_{2} 006}{2006}=2$, 则 $S_{2} 012$等于( )", "options": "A. -2012\nB. 2012\nC. 6033\nD. 6036", "subject": "算术", "analysis": "解析 $\\frac{S_{n}}{n}=a_{1}+\\frac{(n-1) d}{2}$,\n\n$\\therefore \\frac{S_{2} 008}{2008}-\\frac{S_{2006}}{2006}=a_{1}+\\frac{2008-1}{2} d-a_{1}-\\frac{2006-1}{2} d$\n\n$=d=2$.\n\n$\\therefore S_{2012}=2012 \\times(-2008)+\\frac{2012 \\times 2011}{2} \\times 2$\n\n$=2012 \\times 3=6036$​."} {"id": "18720", "image": [], "answer": "B", "solution": "null", "level": "高二", "question": "数列 $\\left\\{a_{n}\\right\\}$ 的前 $n$ 项和为 $S_{n}$, 若 $a_{n}=\\frac{1}{n(n+1)}$, 则 $S_{5}$ 等于 $(\\quad)$", "options": "A. 1\nB. $\\frac{5}{6}$\nc. $\\frac{1}{6}$\nD. $\\frac{1}{30}$", "subject": "算术", "analysis": "解析 $\\because a_{n}=\\frac{1}{n(n+1)}=\\frac{1}{n}-\\frac{1}{n+1}$,\n\n$\\therefore S_{5}=\\left(1-\\frac{1}{2}\\right)+\\left(\\frac{1}{2}-\\frac{1}{3}\\right)+\\cdots+\\left(\\frac{1}{5}-\\frac{1}{6}\\right)$\n\n$=1-\\frac{1}{6}=\\frac{5}{6}$."} {"id": "18726", "image": [], "answer": "C", "solution": "null", "level": "高二", "question": "数列 $\\left\\{a_{n}\\right\\}$ 的通项公式 $a_{n}=\\frac{1}{\\sqrt{n}+\\sqrt{n+1}}$, 若前 $n$ 项的和为 10 , 则项数为 ()", "options": "A. 11\nB. 99\nC. 120\nD. 121", "subject": "算术", "analysis": "解析 $\\because a_{n}=\\frac{1}{\\sqrt{n}+\\sqrt{n+1}}=\\sqrt{n+1}-\\sqrt{n}$,\n\n$\\therefore S_{n}=\\sqrt{n+1}-1=10, \\quad \\therefore n=120$."} {"id": "18727", "image": [], "answer": "A", "solution": "null", "level": "高二", "question": "数列 $1 \\frac{1}{2}, 2 \\frac{1}{4}, 3 \\frac{1}{8}, 4 \\frac{1}{16}, \\cdots$ 的前 $n$ 项和为 $(\\quad)$", "options": "A. $\\frac{1}{2}\\left(n^{2}+n+2\\right)-\\frac{1}{2^{n}}$\nB. $\\frac{1}{2} n(n+1)+1-\\frac{1}{2^{n-1}}$\nC. $\\frac{1}{2}\\left(n^{2}-n+2\\right)-\\frac{1}{2^{n}}$\nD. $\\frac{1}{2} n(n+1)+2\\left(1-\\frac{1}{2^{n}}\\right)$", "subject": "算术", "analysis": "解析 $1 \\frac{1}{2}+2 \\frac{1}{4}+3 \\frac{1}{8}+\\cdots+\\left(n+\\frac{1}{2^{n}}\\right)$\n\n$=(1+2+\\cdots+n)+\\left(\\frac{1}{2}+\\frac{1}{4}+\\cdots+\\frac{1}{2^{n}}\\right)$\n\n$=\\frac{n(n+1)}{2}+\\frac{\\frac{1}{2}\\left(1-\\frac{1}{2^{n}}\\right)}{1-\\frac{1}{2}}$\n\n$=\\frac{1}{2}\\left(n^{2}+n\\right)+1-\\frac{1}{2^{n}}$\n$=\\frac{1}{2}\\left(n^{2}+n+2\\right)-\\frac{1}{2^{n}}$."} {"id": "18728", "image": [], "answer": "C", "solution": "null", "level": "高二", "question": "已知数列 $\\left\\{a_{n}\\right\\}$ 的通项 $a_{n}=2 n+1$, 由 $b_{n}=\\frac{a_{1}+a_{2}+a_{3}+\\cdots+a_{n}}{n}$ 所确定的数列 $\\left\\{b_{n}\\right\\}$ 的前 $n$项之和是( )", "options": "A. $n(n+2)$\nB. $\\frac{1}{2} n(n+4)$\nC. $\\frac{1}{2} n(n+5)$\nD. $\\frac{1}{2} n(n+7)$", "subject": "算术", "analysis": "解析 $a_{1}+a_{2}+\\cdots+a_{n}=\\frac{n}{2}(2 n+4)=n^{2}+2 n$.\n\n$\\therefore b_{n}=n+2, \\quad \\therefore b_{n}$ 的前 $n$ 项和 $S_{n}=\\frac{n(n+5)}{2}$."} {"id": "18729", "image": [], "answer": "B", "solution": "null", "level": "高二", "question": "已知 $S_{n}=1-2+3-4+\\cdots+(-1)^{n-1} n$, 则 $S_{17}+S_{33}+S_{50}$ 等于 ( )", "options": "A. 0\nB. 1\nC. -1\nD. 2", "subject": "算术", "analysis": "解析 $S_{17}=(1-2)+(3-4)+\\cdots+(15-16)+17=9$,\n\n$S_{33}=(1-2)+(3-4)+\\cdots+(31-32)+33=17$,\n\n$S_{50}=(1-2)+(3-4)+\\cdots+(49-50)=-25$,\n\n所以 $S_{17}+S_{33}+S_{50}=1$."} {"id": "18730", "image": [], "answer": "A", "solution": "null", "level": "高二", "question": "数列 $\\left\\{a_{n}\\right\\}$ 满足 $a_{1}, a_{2}-a_{1}, a_{3}-a_{2}, \\cdots, a_{n}-a_{n-1}$ 是首项为 1 , 公比为 2 的等比数列,那么 $a_{n}$ 等于 ( )", "options": "A. $2^{n}-1$\nB. $2^{n-1}-1$\nC. $2^{n}+1$\nD. $4^{n}-1$", "subject": "算术", "analysis": "解析 由于 $a_{n}-a_{n-1}=1 \\times 2^{n^{-1}}=2^{n^{-1}}$,\n\n那么 $a_{n}=a_{1}+\\left(a_{2}-a_{1}\\right)+\\cdots+\\left(a_{n}-a_{n-1}\\right)$\n\n$=1+2+\\cdots+2^{n-1}=2^{n}-1$​."} {"id": "19918", "image": [], "answer": "B", "solution": "null", "level": "高二", "question": "已知 $x>0, y>0, x+2 y+2 x y=8$, 则 $x+2 y$ 的最小值是", "options": "A. 3\nB. 4\nC. $\\frac{9}{2}$\nD. $\\frac{11}{2}$", "subject": "算术", "analysis": "考察均值不等式 $x+2 y=8-x \\cdot(2 y) \\geq 8-\\left(\\frac{x+2 y}{2}\\right)^{2}$, 整理得$(x+2 y)^{2}+4(x+2 y)-32 \\geq 0$ 即 $(x+2 y-4)(x+2 y+8) \\geq 0$, 又 $x+2 y>0, \\therefore x+2 y \\geq 4$"} {"id": "19919", "image": [], "answer": "C", "solution": "null", "level": "高二", "question": "已知 $x>1$, 则 $x+\\frac{4}{x-1}$ 的最小值为", "options": "A. 3\nB. 4\nC. 5\nD. 6", "subject": "算术", "analysis": "由题意, 因为 $x>1$, 则 $x-1>0$,\n\n所以 $x+\\frac{4}{x-1}=x-1+\\frac{4}{x-1}+1 \\geq 2 \\sqrt{(x-1) \\cdot\\left(\\frac{4}{x-1}\\right)}+1=5$,\n\n当且仅当 $x-1=\\frac{4}{x-1}$ 时, 即 $x=3$ 时取等号,\n\n所以 $x+\\frac{4}{x-1}$ 的最小值为 5 , 故选 C."} {"id": "19920", "image": [], "answer": "B", "solution": "null", "level": "高二", "question": "下列函数中, 最小值为 2 的是 ( )", "options": "A. $y=x+\\frac{1}{x}$\nB. $y=3^{x}+3^{-x}$\nC. $y=\\lg x+\\frac{1}{\\lg x}(00,3^{-x}>0$, 由基本不等式可知 $3^{x}+3^{-x} \\geq 2 \\sqrt{3^{x} \\cdot 3^{-x}}=2$, 当且仅当 $x=0$ 时取等号, 故正确; $C$项中, $0\n\n则 $|\\sin x|=\\frac{1}{2}$,\n\n$\\therefore \\sin x=\\frac{1}{2}$ 或 $\\sin x=-\\frac{1}{2}$,\n\n$\\because x \\in[-2 \\pi, 2 \\pi]$,\n\n$\\therefore$ 方程 $\\sin x=\\frac{1}{2}$ 的 4 个根关于 $x=-\\frac{\\pi}{2}$ 对称,\n\n则对称的 2 个根之和为 $-\\pi$,\n\n则 4 个根之和为 $-2 \\pi$,\n\n同理, 由对称性可得 $\\sin x=-\\frac{1}{2}$ 的四个根之和为 $2 \\pi$, 故选 A.\n\n答案:"} {"id": "17816", "image": [], "answer": "C", "solution": "null", "level": "高二", "question": "函数(1) $y=x^{2} \\sin x$; (2) $y=\\sin x, x \\in[0,2 \\pi]$; (3) $y=\\sin x, x \\in[-\\pi, \\pi]$; (4) $y=x \\cos x$ 中, 奇函数的个数为", "options": "A. 1\\nB. 2\\nC. 3\\nD. 4\\n\\n", "subject": "代数", "analysis": "C"} {"id": "17822", "image": ["8980.jpg"], "answer": "D", "solution": "null", "level": "高二", "question": "函数 $y=\\cos x\\left(\\frac{\\pi}{6} \\leqslant x \\leqslant \\frac{7 \\pi}{6}\\right)$ 的最小值, 最大值分别为", "options": "A. 0,1\nB. $-1,1$\nC. $-\\frac{\\sqrt{3}}{2}, 1$\nD. $-1, \\frac{\\sqrt{3}}{2}$", "subject": "代数", "analysis": "解析: 由 $y=\\cos x\\left(\\frac{\\pi}{6} \\leqslant x \\leqslant \\frac{7 \\pi}{6}\\right)$ 的图象(如图)可知, 当 $x=\\frac{\\pi}{6}$ 时, $y=\\cos x$ 有最大值 $\\frac{\\sqrt{3}}{2}$; 当 $x$ $=\\pi$ 时, $y=\\cos x$ 有最小值 -1 . 故选 D.\n\n\n\n答案: D"} {"id": "17823", "image": ["8981.jpg", "8982.jpg", "8983.jpg", "8984.jpg"], "answer": "D", "solution": "null", "level": "高二", "question": "函数 $y=-x \\cos x$ 的部分图象是 ( )\n\n", "options": "A.\n\n\n\nB\n\n\n\nC\n\n\n\nD", "subject": "代数", "analysis": "解析: $\\because y=-x \\cos x$ 是奇函数, 它的图象关于原点对称, $\\therefore$ 排除 A, C 项; 当 $x \\in\\left(0, \\frac{\\pi}{2}\\right)$ 时, $y=-x \\cos x<0, \\therefore$ 排除 B 项, 故选 D.\n\n答案: D"} {"id": "17830", "image": [], "answer": "B", "solution": "null", "level": "高二", "question": "关于正切函数 $y=\\tan x$, 下列判断不正确的是", "options": "A. 是奇函数\n\nB. 在整个定义域上是增函数\n\nC. 在定义域内无最大值和最小值\n\nD. 平行于 $x$ 轴的直线被正切曲线各支所截线段相等", "subject": "代数", "analysis": "解析: 正切函数在整个定义域上不具有单调性,正切函数在每个单调区间内是增函数.\n\n答案: B"} {"id": "17831", "image": ["8985.jpg"], "answer": "B", "solution": "null", "level": "高二", "question": "已知 $a, b$ 是不等于 1 的正数, $\\theta \\in\\left(\\frac{3 \\pi}{2}, 2 \\pi\\right)$, 若 $a^{\\tan \\theta}>b^{\\tan \\theta}>1$, 则下列关系式成立的是", "options": "A. $a>b>1$\n B. $aa>1$", "subject": "代数", "analysis": "\n\n$\\therefore a", "options": "A.\n\n\n\nC\n\n\n\nB\n\n\n\nD", "subject": "代数", "analysis": "解析: 当 $x \\in\\left[0 , \\frac{\\pi}{2}\\right) \\cup\\left[\\pi , \\frac{3 \\pi}{2}\\right)$ 时, $\\tan x \\geqslant 0$; 当 $x \\in\\left(\\frac{\\pi}{2} , \\pi\\right)$ 时, $\\tan x<0$.当 $x=\\frac{\\pi}{2}$ 或 $\\frac{3 \\pi}{2}$ 时, $\\tan x$无意义.从而当 $x \\in\\left[0 , \\frac{\\pi}{2}\\right) \\cup\\left[\\pi , \\frac{3 \\pi}{2}\\right)$ 时, $y=|\\tan x| \\cos x$ 与 $y=\\sin x$ 的图象相同; 当 $x \\in\\left(\\frac{\\pi}{2} , \\pi\\right)$时, $y=|\\tan x| \\cos x$ 与 $y=\\sin x$ 的图象关于 $x$ 轴对称,故选 C.\n\n答案: C"} {"id": "17861", "image": ["8997.jpg"], "answer": "B", "solution": "null", "level": "高二", "question": "已知函数 $f(x)=A \\cos (\\omega x+\\varphi)(A>0, \\omega>0)$ 的部分图象如图所示, $f\\left(\\frac{\\pi}{2}\\right)=-\\frac{2}{3}$, 则 $f(0)=$\n\n", "options": "A. $-\\frac{2}{3}$\nB. $\\frac{2}{3}$\nC. $-\\frac{1}{2}$\nD. $\\frac{1}{2}$", "subject": "代数", "analysis": "解析 : 由图象可知所求函数的周期为 $T=2\\left(\\frac{11 \\pi}{12}-\\frac{7 \\pi}{12}\\right)=\\frac{2 \\pi}{3}$ ,故 $\\omega=\\frac{2 \\pi}{\\frac{2 \\pi}{3}}=3$. 将 $\\left(\\frac{11 \\pi}{12} , 0\\right)$ 代入解析式,得 $A \\cos \\left(3 \\times \\frac{11 \\pi}{12}+\\varphi\\right)=0$ ,即 $\\cos \\left(\\frac{11 \\pi}{4}+\\varphi\\right)=0$ ,\n\n$\\therefore \\frac{11 \\pi}{4}+\\varphi=\\frac{\\pi}{2}+2 k \\pi , k \\in \\mathbf{Z} , \\therefore \\varphi=-\\frac{9 \\pi}{4}+2 k \\pi , k \\in \\mathbf{Z}$. 令 $\\varphi=-\\frac{\\pi}{4}$, 代入解析式得 $f(x)=$ $\\operatorname{Acos}\\left(3 x-\\frac{\\pi}{4}\\right)$.\n\n又 $\\because\\left(\\frac{\\pi}{2}\\right)=-\\frac{2}{3}, \\therefore\\left(\\frac{\\pi}{2}\\right)=-A \\sin \\frac{\\pi}{4}=-\\frac{\\sqrt{2}}{2} A=-\\frac{2}{3}, \\therefore A=\\frac{2}{3} \\sqrt{2}$.\n\n$\\therefore f(0)=\\frac{2}{3} \\sqrt{2} \\cos \\left(-\\frac{\\pi}{4}\\right)=\\frac{2}{3} \\sqrt{2} \\cos \\frac{\\pi}{4}=\\frac{2}{3}$.\n\n答案: B"} {"id": "17871", "image": ["9003.jpg"], "answer": "D", "solution": "null", "level": "高二", "question": "如图, 单摆从某点开始来回摆动, 离开平衡位置 $O$ 的距离 $s(\\mathrm{~cm})$ 和时间 $t(s)$ 的函数关系式为 $s$ $=6 \\sin \\left(2 \\pi t+\\frac{\\pi}{6}\\right)$, 那么单摆来回摆动一次所需的时间为\n\n", "options": "A. $2 \\pi \\mathrm{s}$\nB. $\\pi \\mathrm{s}$\nC. $0.5 \\mathrm{~s}$\nD. $1 \\mathrm{~s}$", "subject": "代数", "analysis": "解析: 单摆来回摆动一次所需的时间为函数 $s=6 \\sin \\left(2 \\pi t+\\frac{\\pi}{6}\\right)$ 的周期. 又因为 $T=\\frac{2 \\pi}{2 \\pi}=1$ ,所以单摆来回摆动一次所需的时间为 $1 \\mathrm{~s}$ ,故选 D.\n\n答案: D"} {"id": "17879", "image": [], "answer": "C", "solution": "null", "level": "高二", "question": "商场人流量被定义为每分钟通过入口的人数, “五一” 期间某一天商场的人流量满足函数 $F(t)=50+4 \\sin \\frac{t}{2}(t \\geqslant 0)$, 则人流量增加的时间段是", "options": "A. $[0,5]$\nB. $[5,10]$\nC. $[10,15]$\nD. $[15,20]$", "subject": "代数", "analysis": "解析: 由 $2 k \\pi-\\frac{\\pi}{2} \\leqslant \\frac{t}{2} \\leqslant 2 k \\pi+\\frac{\\pi}{2}, k \\in \\mathbf{Z}$ ,知函数 $F(t)$ 的单调递增区间为 $[4 k \\pi-\\pi, 4 k \\pi+\\pi], k \\in \\mathbf{Z}$.当 $k=1$ 时, $t \\in[3 \\pi , 5 \\pi]$. 因为 $[10 , 15] \\subseteq[3 \\pi , 5 \\pi]$ ,故选 C.\n\n答案: C"} {"id": "17882", "image": ["9015.jpg"], "answer": "B", "solution": "null", "level": "高二", "question": "如图所示, 一个单摆以 $O A$ 为始边, $O B$ 为终边的角 $\\theta(-\\pi<\\theta<\\pi)$ 与时间 $t(\\mathrm{~s})$ 满足函数关系式 $\\theta$ $=\\frac{1}{2} \\sin \\left(2 t+\\frac{\\pi}{2}\\right), t \\in[0,+\\infty)$, 则当 $t=0$ 时, 角 $\\theta$ 的大小及单摆频率是\n\n", "options": "A. $2, \\frac{1}{\\pi}$\nB. $\\frac{1}{2}, \\frac{1}{\\pi}$\n$\\mathrm{C} \\cdot \\frac{1}{2}, \\pi$\nD. $2, \\pi$", "subject": "代数", "analysis": "解析: 当 $t=0$ 时, $\\theta=\\frac{1}{2} \\sin \\frac{\\pi}{2}=\\frac{1}{2}$ ,由函数解析式易知单摆周期为 $\\frac{2 \\pi}{2}=\\pi$ ,故单摆频率为 $\\frac{1}{\\pi}$​​.答案: B"} {"id": "17889", "image": [], "answer": "B", "solution": "null", "level": "高二", "question": "设集合 $M=\\left\\{\\alpha \\left\\lvert\\, \\alpha=\\frac{k \\pi}{2}\\right., k \\in \\mathbf{Z}\\right\\} \\cup\\left\\{\\alpha \\left\\lvert\\, \\alpha=k \\pi+\\frac{\\pi}{4}\\right., k \\in \\mathbf{Z}\\right\\}, N=\\left\\{\\beta \\left\\lvert\\, \\beta=\\frac{k \\pi}{4}\\right., k \\in \\mathbf{Z}\\right\\}$, 则集合 $M$与 $N$ 的关系是", "options": "A. $M N$\n B. $M N$\n C. $M=N$\n D. $M \\cap N=\\varnothing$", "subject": "代数", "analysis": "解析: 集合 $M$ 中, $\\alpha=\\frac{k \\pi}{2}(k \\in \\mathbf{Z})$ 是 $\\frac{\\pi}{2}$ 的整数倍的角, 其终边在坐标轴上, $\\alpha=k \\pi+\\frac{\\pi}{4}(k \\in \\mathbf{Z})$ 的终边在直线 $y=x$ 上; 集合 $N$ 中, $\\beta=\\frac{k \\pi}{4}(k \\in \\mathbf{Z})$ 是 $\\frac{\\pi}{4}$ 的整数倍角, 其终边在直线 $y=x$ 上, 或 $y$ 轴上, 或直线 $y=-x$ 上, 或 $x$ 轴上, 故 $M N$.\n\n答案: B"} {"id": "17914", "image": [], "answer": "C", "solution": "null", "level": "高二", "question": "下列等式中, 正确的个数为 (\n(1) $0-\\boldsymbol{a}=-\\boldsymbol{a}$;\n(2) $-(-a)=a$;\n(3) $\\boldsymbol{a}+(-\\boldsymbol{a})=0$; (4) $\\boldsymbol{a}+0=\\boldsymbol{a}$; (5) $\\boldsymbol{a}-\\boldsymbol{b}=\\boldsymbol{a}+(-\\boldsymbol{b})$; (6) $\\boldsymbol{a}-(-$ a) $=0$.", "options": "A. 3\nB. 4\nC. 5\nD. 6", "subject": "代数", "analysis": "解析: (1)(2)(3)(4)(5)正确.\n\n答案: C"} {"id": "17921", "image": [], "answer": "B", "solution": "null", "level": "高二", "question": "若 $k_{1} \\boldsymbol{a}+k_{2} \\boldsymbol{b}=0$, 则 $k_{1}=k_{2}=0$, 那么下面关于向量 $\\boldsymbol{a}, \\boldsymbol{b}$ 的判断正确的是 ( )", "options": "A. $\\boldsymbol{a}$ 与 $\\boldsymbol{b}$ 一定共线\nB. $\\boldsymbol{a}$ 与 $\\boldsymbol{b}$ 一定不共线\nC. $\\boldsymbol{a}$ 与 $\\boldsymbol{b}$ 垂直\nD. $\\boldsymbol{a}$ 与 $\\boldsymbol{b}$ 中至少有一个为 0", "subject": "代数", "analysis": "解析:由平面向量基本定理可知,当 $\\boldsymbol{a} , \\boldsymbol{b}$ 不共线时, $k_{1}=k_{2}=0$\n\n答案: B"} {"id": "17948", "image": [], "answer": "D", "solution": "null", "level": "高二", "question": "设向量 $\\boldsymbol{a}=(m, n), \\boldsymbol{b}=(s, t)$, 定义两个向量 $\\boldsymbol{a}, \\boldsymbol{b}$ 之间的运算 “ $\\otimes$ ” 为 $\\boldsymbol{a} \\otimes \\boldsymbol{b}=(m s, n t)$. 若向量 $\\boldsymbol{p}=(1,2), \\boldsymbol{p}^{\\otimes} \\boldsymbol{q}=(-3,-4)$, 则向量 $\\boldsymbol{q}=$", "options": "A. $(-3,2)$\n B. $(3,-2)$\n C. $(-2,-3)$\n D. $(-3,-2)$", "subject": "代数", "analysis": "解析: 设向量 $\\boldsymbol{q}=(x, y)$ ,根据题意可得 $x=-3 , 2 y=-4$ ,解得 $x=-3 , y=-2$ ,即向量 $\\boldsymbol{q}=(-3 ,-2) ,$ 故选 D.\n\n答案: D"} {"id": "17960", "image": ["9030.jpg"], "answer": "A", "solution": "null", "level": "高二", "question": "", "options": "A. $-\\frac{1}{2}$\nB. $\\frac{1}{2}$\nC. -2\nD. 2", "subject": "代数", "analysis": "解析: $m \\boldsymbol{a}+n \\boldsymbol{b}=(2 m-n, 3 m+2 n), \\boldsymbol{a}-2 \\boldsymbol{b}=(4,-1)$.\n\n$\\because(m \\boldsymbol{a}+n \\boldsymbol{b}) / /(\\boldsymbol{a}-2 \\boldsymbol{b}), \\therefore(2 m-n) \\times(-1)-(3 m+2 n) \\times 4=0$,\n\n$\\therefore 2 m=-n$ ,即 $\\frac{m}{n}=-\\frac{1}{2}$.\n\n答案: A"} {"id": "17961", "image": [], "answer": "C", "solution": "null", "level": "高二", "question": "下面给出的关系式中正确的个数是 ( )\n\n(1) $0 \\cdot \\boldsymbol{a}=0$; (2) $\\boldsymbol{a} \\cdot \\boldsymbol{b}=\\boldsymbol{b} \\cdot \\boldsymbol{a} ;$ (3) $\\boldsymbol{a}^{2}=|\\boldsymbol{a}|^{2}$; (4) $|\\boldsymbol{a} \\cdot \\boldsymbol{b}| \\leqslant \\boldsymbol{a} \\cdot \\boldsymbol{b} ;$ (5) $(\\boldsymbol{a} \\cdot \\boldsymbol{b})^{2}=\\boldsymbol{a}^{2} \\cdot \\boldsymbol{b}^{2}$.", "options": "A. 1\nB. 2\nC. 3\nD. 4", "subject": "代数", "analysis": "解析: (1)(2)(3)正确,(4)(5)错误, $(\\boldsymbol{a} \\cdot \\boldsymbol{b})^{2}=(|\\boldsymbol{a}| \\cdot|\\boldsymbol{b}| \\cos \\theta)^{2}=\\boldsymbol{a}^{2} \\cdot \\boldsymbol{b}^{2} \\cos ^{2} \\theta \\neq \\boldsymbol{a}^{2} \\cdot \\boldsymbol{b}^{2}$.\n\n答案: C"} {"id": "18004", "image": [], "answer": "C", "solution": "null", "level": "高二", "question": "下列量不是向量的是", "options": "A. 力\nB. 速度\nC. 质量\nD. 加速度", "subject": "代数", "analysis": "解析: 质量只有大小, 没有方向, 不是向量.\n\n答案: C"} {"id": "18010", "image": [], "answer": "D", "solution": "null", "level": "高二", "question": "下列说法正确的是", "options": "A. 数量可以比较大小, 向量也可以比较大小\n\nB. 方向不同的向量不能比较大小, 但同向的可以比较大小\n\nC. 向量的大小与方向有关\n\nD. 向量的模可以比较大小", "subject": "代数", "analysis": "解析: 向量不能比较大小, 但是向量的模是实数, 可以比较大小.\n\n答案: D"} {"id": "18012", "image": [], "answer": "C", "solution": "null", "level": "高二", "question": "下列说法正确的是", "options": "A. 若 $|a|>|b|$, 则 $a>b$\n\nB. 若 $|\\boldsymbol{a}|=|\\boldsymbol{b}|$, 则 $a=b$\n\nC. 若 $a=b$, 则 $a / / b$\n\nD. 若 $a \\neq b$, 则 $a, b$ 不是共线向量", "subject": "代数", "analysis": "解析: 向量不能比较大小, 所以 $\\mathrm{A}$ 不正确; $a=b$ 需满足两个条件: $a, b$ 同向且 $|a|=|b|$,所以 $\\mathrm{B}$ 不正确; $\\mathrm{C}$ 正确; 若 $a, b$ 是共线向量, 则 $a, b$ 方向相同或相反, $\\mathrm{D}$ 不正确.\n\n答案: $C$"} {"id": "19736", "image": [], "answer": "B", "solution": "null", "level": "高二", "question": "给出下列结论:\n\n(1)若用图象表示数列, 从图象上看都是一系列孤立的点;\n\n(2)数列的项数都是无限的;\n\n(3)数列可以看成是以正整数集或其有限子集 $\\{1,2,3, \\cdots, n\\}$ 为定义域的函数.\n\n其中正确的结论是( )", "options": "A. (1)(2).\nB. (1)(3).\nC. (2) (3).\nD. (1)(2)(3).", "subject": "代数", "analysis": "解析 对于(1), 因为数列可以看成是以正整数集或其有限子集 $\\{1,2,3 , \\ldots$, $\\mathrm{n}\\}$ 为定义域的函数,所以数列的图象都是一群孤立的点,正确;对于(2), 无穷数列的项数是无限的, 但有穷数列的项数是有限的, 故错误.对于(3), 数列可以看成是以正整数集或其有限子集 $\\{1,2 , 3 , \\ldots$,$\\} 为定义域$的函数, 故正确; 故选 B."} {"id": "19737", "image": ["9258.jpg", "9259.jpg"], "answer": "A", "solution": "null", "level": "高二", "question": "\n(4) $a_{n}=\\left|\\sin \\frac{n \\pi}{2}\\right|$, 其中能作为数列: $0,1,0,1,0,1,0,1, \\cdots$ 的通项公式的是 ( )", "options": "A. (1)(2) (3)\nB. (1)(2) (4)\nC. (2)(3)(4)\nD. (1)(3)(4)", "subject": "代数", "analysis": "(2) $\\mathrm{a}_{\\mathrm{n}}=\\frac{1+(-1)^{n}}{2}$, 满足条件; (3) $\\mathrm{a}_{\\mathrm{n}}=\\frac{1+\\cos n \\pi}{2}$, 满足条件;(4) $\\mathrm{a}_{\\mathrm{n}}=\\left|\\sin \\frac{n \\pi}{2}\\right|, \\mathrm{n}=1$ 时, $\\mathrm{a}_{1}=1 ; \\mathrm{n}=2$ 时, $\\mathrm{a}_{2}=0$, 以此类推, 不满足条件; 故选:A."} {"id": "18025", "image": [], "answer": "A", "solution": "null", "level": "高二", "question": "已知向量 $\\boldsymbol{a}, \\boldsymbol{b}$ 是两个非零向量, 在下列四个条件中, 一定能使 $\\boldsymbol{a}, \\boldsymbol{b}$ 共线的是\n\n(1) $2 \\boldsymbol{a}-3 \\boldsymbol{b}=4 \\boldsymbol{e}$ 且 $\\boldsymbol{a}+2 \\boldsymbol{b}=-2 \\boldsymbol{e}$;\n\n(2)存在相异实数 $\\lambda, \\mu$, 使 $\\lambda \\boldsymbol{a}-\\mu \\boldsymbol{b}=0$;\n\n(3) $x \\boldsymbol{a}+y \\boldsymbol{b}=0$ (其中实数 $x, y$ 满足 $x+y=0$ );\n\n(4)已知梯形 $A B C D$, 其中 $\\overrightarrow{A B}=\\boldsymbol{a}, \\overrightarrow{C D}=\\boldsymbol{b}$.", "options": "A. (1)(2)\nB. (1)(3)\nC. (2)\nD. (3)(4)", "subject": "代数", "analysis": "解析: 由 $2 \\boldsymbol{a}-3 \\boldsymbol{b}=-2(\\boldsymbol{a}+2 \\boldsymbol{b})$ 得 $\\boldsymbol{b}=-4 \\boldsymbol{a}$ ,故(1)正确; 由 $\\lambda \\boldsymbol{a}-\\mu \\boldsymbol{b}=0$ ,得 $\\lambda \\boldsymbol{a}=\\mu \\boldsymbol{b}$ ,故(2)正确; 若 $x=y=0 , x \\boldsymbol{a}+y \\boldsymbol{b}=0$ ,但 $\\boldsymbol{b}$ 与 $\\boldsymbol{a}$ 不一定共线,故(3)错误; 梯形 $A B C D$ 中,没有说明哪组对边平行,故(4)错误.\n\n答案: A"} {"id": "19771", "image": [], "answer": "C", "solution": "null", "level": "高二", "question": "已知数列 $\\left\\{a_{n}\\right\\}$ 满足递推关系: $a_{n+1}=\\frac{a_{n}}{a_{n}+1}, a_{1}=\\frac{1}{2}$, 则 $a_{2017}=(\\quad)$", "options": "A. $\\frac{1}{2016}$\nB. $\\frac{1}{2017}$\nC. $\\frac{1}{2018}$\nD. $\\frac{1}{2019}$", "subject": "代数", "analysis": "$: a_{n+1}=\\frac{a_{n}}{a_{n}+1}, \\mathrm{a}_{1}=\\frac{1}{2}, \\therefore \\frac{1}{a_{n+1}}-\\frac{1}{a_{n}}=1, \\therefore$ 数列 $\\left\\{\\frac{1}{a_{n}}\\right\\}$ 是等差数列, 首项为 2 , 公差为 $1, \\therefore$ $\\frac{1}{a_{2017}}=2+2016=2018$, 则 $a_{2017}=\\frac{1}{2018}$. 故选 C."} {"id": "19828", "image": [], "answer": "D", "solution": "null", "level": "高二", "question": "实数集 $R$, 设集合 $P=\\left\\{x \\mid x^{2}-4 x+3 \\leq 0\\right\\}, Q=\\left\\{x \\mid x^{2}-4<0\\right\\}$, 则 $P \\cup\\left(C_{R} Q\\right)=(\\quad)$", "options": "A. $[2,3]$\nB. $(1,3)$\nC. $(2,3]$\nD. $(-\\infty,-2] \\cup[1,+\\infty)$", "subject": "代数", "analysis": "实数集 $R$, 集合 $P=\\left\\{x \\mid x^{2}-4 x+3 \\leq 0\\right\\}=\\{x \\mid 1 \\leq x \\leq 3\\} , Q=\\left\\{x \\mid x^{2}-4<0\\right\\}=\\{x \\mid-21\\right\\}$, 则 $C_{B} A=( )$", "options": "A. $[3,+\\infty)$\nB. $(3,+\\infty)$\nC. $(-\\infty,-1] \\cup[3,+\\infty)$\nD. $(-\\infty,-1) \\cup(3,+\\infty)$", "subject": "代数", "analysis": "$A=\\left\\{x \\mid x^{2}-2 x-3<0\\right\\}=\\{x \\mid-11\\right\\}=\\{x \\mid x>-1\\}$,\n\n$\\mathrm{C}_{\\mathrm{B}} \\mathrm{A}=[3,+\\infty)$,故选 A."} {"id": "19835", "image": [], "answer": "D", "solution": "null", "level": "高二", "question": "函数 $f(x)=\\log _{2}\\left(x^{2}+2 x-3\\right)$ 的定义域是 ( () $)$", "options": "A. $[-3,1]$\nB. $(-3,1)$\nC. $(-\\infty,-3] \\cup[1,+\\infty)$\nD. $(-\\infty,-3) \\cup(1,+\\infty)$", "subject": "代数", "analysis": "由题意得: $x^{2}+2 x-3>0$, 即 $(x-1)(x+3)>0$\n\n解得 $x>1$ 或 $x<-3$ 所以定义域为 $(-\\infty,-3) \\cup(1,+\\infty)$ 故选:D."} {"id": "19837", "image": [], "answer": "A", "solution": "null", "level": "高二", "question": "不等式 $x^{2}-4 x+3<0$ 的解集为 ( )", "options": "A. $(1,3)$\nB. $(-3,-1)$\nC. $(-\\infty,-3) \\cup(-1,+\\infty)$\nD. $(-\\infty, 1) \\cup(3,+\\infty)$", "subject": "代数", "analysis": "不等式 x^{2}-4 x+3<0$ 可化为 $(x-1)(x-3)<0$, 解得 $10$ 的解集为 ( )", "options": "A. $\\left\\{x \\left\\lvert\\, \\frac{1}{3}\\frac{1}{2}\\right.\\right\\}$\nC. $\\left\\{x \\left\\lvert\\, x<\\frac{1}{3}\\right.\\right\\}$\nD. $\\left\\{x \\left\\lvert\\, x<\\frac{1}{3}\\right.\\right.$ 或 $\\left.x>\\frac{1}{2}\\right\\}$", "subject": "代数", "analysis": "不等式 $\\left(\\frac{1}{2}-x\\right)\\left(x-\\frac{1}{3}\\right)>0$ 可化为 $\\left(x-\\frac{1}{2}\\right)\\left(x-\\frac{1}{3}\\right)<0$; 解得 $\\frac{1}{3}1$ 时, $A=(-\\infty, 1] \\cup[a,+\\infty), B=[a-1,+\\infty)$, 若 $A \\cup B=R$, 则 $a-1 \\leq 1, \\therefore 1\n\n可获得利润为 $z$ 万元, 则 $\\left\\{\\begin{array}{l}x+y \\leqslant 60, \\\\ x \\geqslant \\frac{2}{3} y, \\\\ x \\geqslant 5, \\\\ y \\geqslant 5,\\end{array}\\right.$\n\n$z=0\\.4 x+0\\.6 y$.\n\n由图象知,\n\n目标函数 $z=0\\.4 x+0\\.6 y$ 在 $A$ 点取得最大值.\n\n$\\therefore y_{\\max }=0\\.4 \\times 24+0\\.6 \\times 36=31\\.2$ (万元)."} {"id": "18619", "image": ["9128.jpg", "9129.jpg"], "answer": "B", "solution": "null", "level": "高二", "question": "某加工厂用某原料由甲车间加工出 $A$ 产品, 由乙车间加工出 $B$ 产品, 甲车间加工一箱原料需耗费工时 10 小时, 可加工出 7 千克 $A$ 产品, 每千克 $A$ 产品获利 40 元, 乙车间加工一箱原料耗费工时 6 小时, 可加工出 4 千克 $B$ 产品, 每千克 $B$ 产品获利 50 元. 甲、乙两车间每天共能完成至多 70 箱原料的加工, 每天甲、乙两车间耗费工时总和不得超过 480 小时, 甲、乙两车间每天总获利最大的生产计划为( )", "options": "A. 甲车间加工原料 10 箱, 乙车间加工原料 60 箱\n\nB. 甲车间加工原料 15 箱, 乙车间加工原料 55 箱\n\nC. 甲车间加工原料 18 箱, 乙车间加工原料 50 箱\n\nD. 甲车间加工原料 40 箱, 乙车间加工原料 30 箱\n\n", "subject": "代数", "analysis": "解析 设甲车间加工原料 $x$ 箱, 乙车间加工原料 $y$ 箱, 由题意可知\n\n\n\n甲、乙两车间每天总获利为 $z=280 x+200 y$.\n\n画出可行域如图所示.\n\n点 $M(15,55)$ 为直线 $x+y=70$ 和直线 $10 x+6 y=480$ 的交点, 由图象知在点 $M(15,55)$ 处 $z$取得最大值."} {"id": "18620", "image": ["9130.jpg"], "answer": "C", "solution": "null", "level": "高二", "question": "如图所示, 目标函数 $z=k x-y$ 的可行域为四边形 $O A B C$, 点 $B(3,2)$ 是目标函数的最优解, 则 $k$ 的取值范围为 ( )", "options": "A. $\\left(\\frac{2}{3}, 2\\right)$\nB. $\\left(1, \\frac{5}{3}\\right)$\nC. $\\left(-2,-\\frac{2}{3}\\right)$\nD. $\\left(-3,-\\frac{4}{3}\\right)$\n\n", "subject": "代数", "analysis": "解析 $y=k x-z$. 若 $k>0$, 则目标函数的最优解是点 $A(4,0)$ 或点 $C(0,4)$, 不符合题意.\n\n$\\therefore k<0, \\because$ 点 $(3,2)$ 是目标函数的最优解.\n\n$\\therefore k_{A B} \\leqslant k \\leqslant k_{B C}$, 即 $-2 \\leqslant k \\leqslant-\\frac{2}{3}$."} {"id": "19854", "image": [], "answer": "D", "solution": "null", "level": "高二", "question": "设 $a, b \\in R$, 若 $a-|b|>0$, 则下列不等式正确的是 ( )", "options": "A. $\\mathrm{b}-\\mathrm{a}>0$\nB. $a^{3}+b^{3}<0$\nC. $a^{2}-b^{2}<0$\nD. $\\mathrm{b}+\\mathrm{a}>0$", "subject": "代数", "analysis": "由a- $|b|>0$ 得 $a>|b| \\therefore a>|b|>0 \\therefore a> \\pm b \\therefore a+b>0$"} {"id": "19861", "image": [], "answer": "C", "solution": "null", "level": "高二", "question": "设 $\\frac{1}{3}<\\left(\\frac{1}{3}\\right)^{\\mathrm{b}}<\\left(\\frac{1}{3}\\right)^{\\mathrm{a}}<1$, 则 ( )", "options": "A. $\\mathrm{a}^{\\mathrm{a}}<\\mathrm{a}^{\\mathrm{b}}<\\mathrm{b}^{\\mathrm{a}}$\nB. $\\mathrm{a}^{\\mathrm{a}}<\\mathrm{b}^{\\mathrm{a}}<\\mathrm{a}^{\\mathrm{b}}$\nC. $\\mathrm{a}^{\\mathrm{b}}<\\mathrm{a}^{\\mathrm{a}}<\\mathrm{b}^{\\mathrm{a}}$\nD. $\\mathrm{a}^{\\mathrm{b}}<\\mathrm{b}^{\\mathrm{a}}<\\mathrm{a}^{\\mathrm{a}}$", "subject": "代数", "analysis": "$\\because \\frac{1}{3}<\\left(\\frac{1}{3}\\right)^{\\mathrm{b}}<\\left(\\frac{1}{3}\\right)^{\\mathrm{a}}<1, \\quad \\therefore 0<\\mathrm{a}<\\mathrm{b}<1 . \\therefore \\mathrm{b}^{\\frac{\\mathrm{a}^{\\mathrm{b}}}{}}=\\mathrm{a}^{\\mathrm{a}-\\mathrm{b}}>1 . \\therefore \\mathrm{a}^{\\mathrm{b}}<\\mathrm{a}^{\\mathrm{a}}$.\n\n$\\because \\mathrm{a}^{\\mathrm{a}}=\\left(\\frac{\\mathrm{a}}{\\mathrm{b}}\\right)^{\\mathrm{a}}, \\quad, \\quad \\frac{\\mathrm{a}}{\\mathrm{b}}<1, \\mathrm{a}>0, \\quad \\therefore\\left(\\frac{\\mathrm{a}}{\\mathrm{b}}\\right)^{\\mathrm{a}}<1 . \\therefore \\mathrm{a}^{\\mathrm{a}}<\\mathrm{b}^{\\mathrm{a}} . \\therefore \\mathrm{a}^{\\mathrm{b}}<\\mathrm{a}^{\\mathrm{a}}<\\mathrm{b}^{\\mathrm{a}}$. 故答案为: $\\quad \\mathrm{C}$"} {"id": "19864", "image": [], "answer": "A", "solution": "null", "level": "高二", "question": "已知 $\\mathrm{x} \\in \\mathrm{R}$, 设 $\\mathrm{M}=\\mathrm{x}^{2}-\\mathrm{x}, \\mathrm{N}=\\mathrm{x}-2$, 则 $\\mathrm{M}, \\mathrm{N}$ 的大小关系为 ( )", "options": "A. $\\mathrm{M}>\\mathrm{N}$\nB. $\\mathrm{M}<\\mathrm{N}$\nC. $\\mathrm{M}=\\mathrm{N}$\nD. 无法确定", "subject": "代数", "analysis": "因为 $M-N=x^{2}-x-(x-2)=(x-1)^{2}+1 \\geq 1>0$, 所以 $M>N$, 故选 $A$."} {"id": "19865", "image": [], "answer": "B", "solution": "null", "level": "高二", "question": "对于任意实数 $\\mathrm{a}, \\mathrm{b}, \\mathrm{c}, \\mathrm{d}$, 给出下列叙述:\n\n(1)若 $a>b, c \\neq 0$, 则 $a c>b c$;(2)若 $a>b$, 则 $a c^{2}>b c^{2}$;(3)若 $a c^{2}>b c^{2}$, 则 $a>b$.\n其中, 正确叙述的个数是 ( )", "options": "A. 0\nB. 1\nC. 2\nD. 3", "subject": "代数", "analysis": "对(1), $c<0 \\Rightarrow a c>b c$, (1)不正确; 对(2), $c=0 \\Rightarrow a c^{2}=b c^{2}$, (2)不正确;\n\n若 $a c^{2}>b c^{2}$, 则 $c^{2}>0$, 不等式两边同乘以 $\\frac{1}{2}$ 可得 $a>b$, (3)正确, 故选 B."} {"id": "19842", "image": [], "answer": "D", "solution": "null", "level": "高二", "question": "设 $x=\\sqrt{2}, y=\\sqrt{7}-\\sqrt{3}, z=\\sqrt{6}-\\sqrt{2}$, 则 $x, y, z$ 的大小关系是 ( )", "options": "A. $x>y>z$\nB. $\\mathrm{z}>\\mathrm{x}>\\mathrm{y}$\nC. $y>z>x$\nD. $\\mathrm{x}>\\mathrm{z}>\\mathrm{y}$", "subject": "代数", "analysis": "$\\mathrm{y}=\\sqrt{7}-\\sqrt{3}=\\frac{4}{\\sqrt{7}+\\sqrt{3}}, \\quad \\mathrm{z}=\\sqrt{6}-\\sqrt{2}=\\frac{4}{\\sqrt{6}+\\sqrt{2}}, \\quad \\because \\sqrt{7}+\\sqrt{3}>\\sqrt{6}+\\sqrt{2}>0, \\quad \\therefore \\mathrm{z}>\\mathrm{y}$.\n\n$\\because x-z=\\sqrt{2}-\\frac{4}{\\sqrt{6}+\\sqrt{2}}=\\frac{2 \\sqrt{3}+2-4}{\\sqrt{6}+\\sqrt{2}}=\\frac{2 \\sqrt{3}-2}{\\sqrt{6}+\\sqrt{2}}>0, \\quad \\therefore x>z . \\therefore x>z>y$. 故答案为: $D$"} {"id": "19846", "image": [], "answer": "D", "solution": "null", "level": "高二", "question": "若 $\\frac{1}{a}<\\frac{1}{b}<0$, 则下列结论不正确的是 ( )", "options": "A. $a^{2}|a+b|$", "subject": "代数", "analysis": "因为 $\\frac{1}{a}<\\frac{1}{b}<0$, 所以 $\\left\\{\\begin{array}{l}a<0, \\\\ b<0, \\text { 则 } a^{2}b,\\end{array}\\right.$ 故选项 A、B、C 正确, 而 $|a|+|b|=|a+b|$, 故 D 错误."} {"id": "19867", "image": [], "answer": "B", "solution": "null", "level": "高二", "question": "不等式 $\\frac{3 x-1}{2-x} \\geq 1$ 的解集是 ( )", "options": "A. $\\left\\{\\left.x\\right|_{4} ^{3} \\leq x \\leq 2\\right\\}$\nB. $\\left\\{\\left.x\\right|_{4} ^{3} \\leq x<2\\right\\}$\nC. $\\left\\{x \\mid x>2\\right.$ 或 $\\left.x \\leq \\frac{3}{4}\\right\\}$\nD. $\\left\\{x \\left\\lvert\\, x \\geq \\frac{3}{4}\\right.\\right\\}$", "subject": "代数", "analysis": "不等式 $\\frac{3 x-1}{2-x} \\geq 1$, 移项得: $\\frac{3 x-1}{2-x}-1 \\geq 0$, 即 $\\frac{x-\\frac{3}{4}}{x-2} \\leq 0$,\n\n可化为 $\\left\\{\\begin{array}{l}\\left(x-\\frac{3}{4}\\right)(x-2) \\leq 0 \\\\ x \\neq 2\\end{array}\\right.$ 解得: $\\frac{3}{4} \\leq \\mathrm{x}<2$, 则原不等式的解集为 $\\left\\{x \\left\\lvert\\, \\frac{3}{4} \\leq x<2\\right.\\right\\}$,"} {"id": "19871", "image": [], "answer": "A", "solution": "null", "level": "高二", "question": "不等式 $\\frac{x+2}{3-x}>2$ 的解集是 ( )", "options": "A. $\\left\\{\\left.x\\right|_{3} ^{4}3\\right\\}$\nC. $\\left\\{x \\left\\lvert\\, x>\\frac{4}{3}\\right.\\right\\}$\nD. $\\left\\{x \\left\\lvert\\, x<\\frac{4}{3}\\right.\\right\\}$", "subject": "代数", "analysis": "由于不等式 $\\frac{x+2}{3-x}>2$, 转化为 $\\frac{x+2}{3-x}-2>0$,\n\n即 $\\frac{x+2-2(3-x)}{3-x}=\\frac{3 x-4}{3-x}>0$, 亦即 $(\\mathrm{x}-3)(3 \\mathrm{x}-4)<0$, 解得 $\\frac{4}{3}<\\mathrm{x}<3$.\n\n故不等式的解集是 $\\frac{4}{3}1$ 的解集是 ( )", "options": "A. $\\{x \\mid x>1\\}$\nB. $\\{x \\mid x<1\\}$\nC. $\\{x \\mid 01$ 或 $x<-1\\}$", "subject": "代数", "analysis": "不等式 $\\frac{1}{x}>1$ 可知 $\\mathrm{x}>0$,\n\n不等式化为 $x<1$, 所以不等式的解集为: $\\{x \\mid 0"} {"id": "19895", "image": ["9281.jpg"], "answer": "D", "solution": "null", "level": "高二", "question": "设变量 $x, y$ 满足约束条件 $\\left\\{\\begin{array}{l}2 x+y \\geq 0, \\\\ x+2 y-2 \\geq 0, \\\\ x \\leq 0, \\\\ y \\leq 3,\\end{array}\\right.$ 则目标函数 $z=x+y$ 的最大值为 $(\\quad)$", "options": "A. $\\frac{2}{3}$\nB. 1\nC. $\\frac{3}{2}$\nD. 3", "subject": "代数", "analysis": "\n\n目标函数为四边形 $\\mathrm{ABCD}$ 及其内部, 其中 $A(0,1), B(0,3), C\\left(-\\frac{3}{2}, 3\\right), D\\left(-\\frac{2}{3}, \\frac{4}{3}\\right)$, 所以直线 $z=x+y$ 过点 $\\mathrm{B}$ 时取最大值 3 , 选 D."} {"id": "19898", "image": ["9283.jpg"], "answer": "B", "solution": "null", "level": "高二", "question": "设 $x, y$ 满足约束条件 $\\left\\{\\begin{array}{l}x+y \\geq a \\\\ x-y \\leq-1\\end{array}\\right.$, 且 $z=x+a y$ 的最小值为 7 , 则 $a=(\\quad)$", "options": "A. -5\nB. 3\nC. -5 或 3\nD. 5 或 -3", "subject": "代数", "analysis": "\n\n根据题中约束条件 $\\left\\{\\begin{array}{c}x+y \\geq a \\\\ x-y \\leq-1\\end{array}\\right.$ 可画出可行域如图所示,\n\n两直线交点坐标为: $A\\left(\\frac{a-1}{2}, \\frac{a+1}{2}\\right)$,\n\n当 $a=0$ 时, $z=x+a y$ 无最小值;\n\n当 $a<0$ 时, $z=x+a y$ 在 $A\\left(\\frac{a-1}{2}, \\frac{a+1}{2}\\right)$ 处取最大值, 无最小值.\n\n当 $a>0$ 时, $z=x+a y$ 在 $A\\left(\\frac{a-1}{2}, \\frac{a+1}{2}\\right)$ 处有最小值:\n\n$z=\\frac{a-1}{2}+a \\times \\frac{a+1}{2}=\\frac{a^{2}+2 a-1}{2}$, 则 $\\frac{a^{2}+2 a-1}{2}=7$, 解得 $a=3$, 故选 B."} {"id": "19901", "image": [], "answer": "C", "solution": "null", "level": "高二", "question": "已知 $x>0, y>0$, 且 $x+y=2 x y$, 则 $x+4 y$ 的最小值为", "options": "A. 4\nB. $\\frac{7}{2}$\nC. $\\frac{9}{2}$\nD. 5", "subject": "代数", "analysis": "由 $x+y=2 x y$ 得 $\\frac{1}{x}+\\frac{1}{y}=2$, 由 $x>0, y>0$, $x+4 y=\\frac{1}{2}(x+4 y)\\left(\\frac{1}{x}+\\frac{1}{y}\\right)=\\frac{1}{2}\\left(5+\\frac{4 y}{x}+\\frac{x}{y}\\right) \\geq \\frac{1}{2}(5+4)=\\frac{9}{2}$,\n\n当且仅当 $\\frac{4 y}{x}=\\frac{x}{y}$ 时, 等号成立, 即 $x+4 y$ 的最小值为 $\\frac{9}{2}$, 故选 C."} {"id": "19902", "image": [], "answer": "C", "solution": "null", "level": "高二", "question": "设正实数 $\\mathrm{x}, \\mathrm{y}, \\mathrm{z}$ 满足 $\\mathrm{x}^{2}-3 \\mathrm{xy}+4 \\mathrm{y}^{2}-\\mathrm{z}=0$, 则当 $\\frac{z}{x y}$ 取得最小值时, $\\mathrm{x}+2 \\mathrm{y}-\\mathrm{z}$ 的最大值为 ( )", "options": "A. 0\nB. $\\frac{9}{8}$\nC. 2\nD. $\\frac{9}{4}$", "subject": "代数", "analysis": "由题得 $z=x^{2}+4 y^{2}-3 x y \\geq 4 x y-3 x y=x y(x, y, z>0)$,\n\n即 $\\mathrm{z} \\geq \\mathrm{xy}, \\frac{z}{x y} \\geq 1$. 当且仅当 $\\mathrm{x}=2 \\mathrm{y}$ 时等号成立,\n\n则 $x+2 y-z=2 y+2 y-\\left(4 y^{2}-6 y^{2}+4 y^{2}\\right)$\n\n$=4 \\mathrm{y}-2 \\mathrm{y}^{2}=-2\\left(\\mathrm{y}^{2}-2 \\mathrm{y}\\right)$\n\n$=-2\\left[(\\mathrm{y}-1)^{2}-1\\right]=-2(\\mathrm{y}-1)^{2}+2$.\n\n当 $y=1$ 时, $x+2 y-z$ 有最大值 2故选 C."} {"id": "19903", "image": [], "answer": "A", "solution": "null", "level": "高二", "question": "若 $x>0, y>0$, 且 $\\frac{2}{x}+\\frac{1}{y}=1, x+2 y>m^{2}+7 m$ 恒成立, 则实数 $m$ 的取值范围是 ( )", "options": "A. $(-8,1)$\nB. $(-\\infty,-8) \\cup(1,+\\infty)$\nC. $(-\\infty,-1) \\cup(8,+\\infty)$\nD. $(-1,8)$", "subject": "代数", "analysis": "由基本不等式得 $x+2 y=\\left(\\frac{2}{x}+\\frac{1}{y}\\right)(x+2 y)=\\frac{4 y}{x}+\\frac{x}{y}+4 \\geq 2 \\sqrt{\\frac{4 y}{x} \\cdot \\frac{x}{y}}+4=8$,\n当且仅当 $\\frac{4 y}{x}=\\frac{x}{y}(x, y>0)$, 即当 $x=2 y$ 时, 等号成立, 所以, $x+2 y$ 的最小值为 8 .\n\n由题意可得 $m^{2}+7 m<(x+2 y)_{\\text {min }}=8$, 即 $m^{2}+7 m-8<0$, 解得 $-80)$ 的解集为 $\\left(x_{1}, x_{2}\\right)$, 则 $x_{1}+x_{2}+\\frac{a}{x_{1} x_{2}}$ 的最小值是( )", "options": "A. $\\frac{\\sqrt{6}}{3}$\nB. $\\frac{2}{3} \\sqrt{3}$\nC. $\\frac{4}{3} \\sqrt{3}$\nD. $\\frac{2}{3} \\sqrt{6}$", "subject": "代数", "analysis": "不等式 $\\left.x^{2}-4 a x+3 a^{2}<0 ( a>0\\right)$ 的解集为 $\\left(x_{1}, x_{2}\\right)$,\n\n$\\therefore \\mathrm{x}_{1}+\\mathrm{x}_{2}=4 \\mathrm{a}$, 且 $\\mathrm{x}_{1} \\mathrm{x}_{2}=3 \\mathrm{a}^{2}$;\n\n$\\therefore x_{1}+x_{2}+\\frac{a}{x_{1} x_{2}}=4 \\mathrm{a}+\\frac{1}{3 a} \\geq 2 \\sqrt{4 a \\times \\frac{1}{3 a}}=\\frac{4 \\sqrt{3}}{3}$,\n\n当且仅当 $4 \\mathrm{a}=\\frac{1}{3 a}$, 即 $\\mathrm{a}=\\frac{\\sqrt{3}}{6}$ 时“=”成立;\n\n故所求的最小值是 $\\frac{4 \\sqrt{3}}{3}$. 故选: C."} {"id": "19911", "image": [], "answer": "D", "solution": "null", "level": "高二", "question": "已知正实数 $a, b$ 满足 $a+2 b=1$, 则 $a^{2}+4 b^{2}+\\frac{1}{a b}$ 最小值为 ( )", "options": "A. $\\frac{7}{2}$\nB. 4\nC. $\\frac{161}{36}$\nD. $\\frac{17}{2}$", "subject": "代数", "analysis": "因为已知正实数 $a, b$ 满足 $a+2 b=1$, 所以 $1=a+2 b \\geq 2 \\sqrt{2 a b}$, 当且仅当 $a=2 b$ 时,取等号, 解得 $a b \\leq \\frac{1}{8}$, 即 $a b \\in\\left(0, \\frac{1}{8}\\right)$, 再由 $a^{2}+4 b^{2}=1-4 a b$, 所以 $a^{2}+4 b^{2}+\\frac{1}{a b}=1-4 a b+\\frac{1}{a b}$,把 $a b$ 当做自变量, 则 $1-4 a b+\\frac{1}{a b}$ 在 $\\left(0, \\frac{1}{8}\\right)$ 上是减函数, 所以当 $a b=\\frac{1}{8}$ 时, $1-4 a b+\\frac{1}{a b}$ 取得最小值为 $1-\\frac{1}{2}+8=\\frac{17}{2}$, 故选 D."} {"id": "19915", "image": [], "answer": "C", "solution": "null", "level": "高二", "question": "若正实数 $a, b$ 满足 $a+b=1$, 则 ( )", "options": "A. $\\frac{1}{a}+\\frac{1}{b}$ 有最大值 4\nB. $a b$ 有最小值 $\\frac{1}{4}$\nC. $\\sqrt{a}+\\sqrt{b}$ 有最大值 $\\sqrt{2}$\nD. $a^{2}+b^{2}$ 有最小值 $\\frac{\\sqrt{2}}{2}$", "subject": "代数", "analysis": "因为正实数 $a, b$ 满足 $a+b=1$, 所以 $\\frac{1}{a}+\\frac{1}{b}=\\frac{a+b}{a}+\\frac{a+b}{b}=2+\\frac{b}{a}+\\frac{a}{b} \\geq 2+2=4$,故 $\\frac{1}{a}+\\frac{1}{b}$ 有最小值 4 , 故 A 不正确; 由基本不等式可得 $a+b=1 \\geq 2 \\sqrt{a b}, \\therefore a b \\leq \\frac{1}{4}$, 故 $a b$ 有最大值 $\\frac{1}{4}$, 故 B 不正确; 由于 $(\\sqrt{a}+\\sqrt{b})^{2}=a+b+2 \\sqrt{a b}=1+2 \\sqrt{a b} \\leq 2, \\therefore \\sqrt{a}+\\sqrt{b} \\leq \\sqrt{2}$, 故 $\\sqrt{a}+\\sqrt{b}$由最大值为 $\\sqrt{2}$, 故 C 正确; $\\because a^{2}+b^{2}=(a+b)^{2}-2 a b=1-2 a b \\geq 1-\\frac{1}{2}=\\frac{1}{2}$, 故 $a^{2}+b^{2}$ 由最小值 $\\frac{1}{2}$,故 D 不正确."} {"id": "19916", "image": [], "answer": "B", "solution": "null", "level": "高二", "question": "已知 $a>1, b>1$, 且 $\\frac{1}{a-1}+\\frac{1}{b-1}=1$, 则 $a+4 b$ 的最小值为", "options": "A. 13\nB. 14\nC. 15\nD. 16", "subject": "代数", "analysis": "$a+4 b=(a-1)+4(b-1)+5=[(a-1)+4(b-1)]\\left(\\frac{1}{a-1}+\\frac{1}{b-1}\\right)+5=10+\\frac{a-1}{b-1}+\\frac{4(b-1)}{a-1}$\n\n$\\geq 10+2 \\sqrt{4}=14$, 当且仅当 $\\frac{a-1}{b-1}=\\frac{4(b-1)}{a-1}$ 时等号成立, 取得最小值 14"} {"id": "19917", "image": [], "answer": "C", "solution": "null", "level": "高二", "question": "已知 $2 x+3 y=3$. 若 $x, y$ 均为正数, 则 $\\frac{3}{x}+\\frac{2}{y}$ 的最小值是 ( )", "options": "A. $\\frac{5}{3}$\nB. $\\frac{8}{3}$\nC. 8\nD. 24", "subject": "代数", "analysis": "$\\because 2 x+3 y=3 . x, y$ 均为正数,\n\n则 $\\frac{3}{x}+\\frac{2}{y}=\\frac{1}{3}\\left(\\frac{3}{x}+\\frac{2}{y}\\right)(2 x+3 y)=\\frac{1}{3}\\left(12+\\frac{9 y}{x}+\\frac{4 x}{y}\\right) \\geq \\frac{12+2 \\sqrt{\\frac{9 y}{x} \\cdot \\frac{4 x}{y}}}{3}=8$\n\n当且仅当 $\\frac{9 y}{x}=\\frac{4 x}{y}$ 且 $2 x+3 y=3$ 即 $x=\\frac{3}{4}, y=\\frac{1}{2}$ 时取等号,\n$\\therefore \\frac{3}{x}+\\frac{2}{y}$ 的最小值是"} {"id": "17812", "image": [], "answer": "C", "solution": "null", "level": "高二", "question": "下列函数中,周期为 $2 \\pi$ 的偶函数是", "options": "A. $y=\\sin \\frac{x}{2}$\nB. $y=\\sin 2 x$\nC. $y=\\left|\\sin \\frac{x}{2}\\right|$\nD. $y=|\\sin 2 x|$", "subject": "计数", "analysis": ""} {"id": "19850", "image": [], "answer": "D", "solution": "null", "level": "高二", "question": "完成一项装修工程, 请木工共需付工资每人 500 元, 请瓦工共需付工资每人 400 元, 现有工人工资预算 20000 元, 设木工 $x$ 人, 瓦工 $y$ 人,则工人满足的关系式是 ( )", "options": "A. $5 x+4 y<200$\nB. $5 x+4 y \\geqslant 200$\nC. $5 x+4 y=200$\nD. $5 x+4 y \\leqslant 200$", "subject": "计数", "analysis": "D 解析据题意知, $500 x+400 y \\leq 20000$, 即 $5 x+4 y \\leq 200$, 故选 D."} {"id": "17837", "image": ["8990.jpg", "8991.jpg", "8992.jpg", "8993.jpg"], "answer": "A", "solution": "null", "level": "高二", "question": "函数 $y=\\tan \\left(\\frac{1}{2} x-\\frac{\\pi}{3}\\right)$ 在一个周期内的大致图象是\n\n", "options": "A.\n\n\n\nB\n\n\n\nC\n\n\n\nD", "subject": "变换几何", "analysis": "解析:由函数周期 $T=\\frac{\\pi}{\\frac{1}{2}}=2 \\pi$ ,排除选项 B、D.\n\n将 $x=\\frac{2}{3} \\pi$ 代入函数解析式中,得\n\n$y=\\tan \\left(\\frac{1}{2} \\times \\frac{2}{3} \\pi-\\frac{\\pi}{3}\\right)=\\tan 0=0$,\n\n故函数图象与 $x$ 轴的一个交点为 $\\left(\\frac{2}{3} \\pi , 0\\right)$ ,排除 C,故选 A.\n\n答案: A"} {"id": "17843", "image": [], "answer": "D", "solution": "null", "level": "高二", "question": "要得到函数 $y=\\sin \\left(x-\\frac{\\pi}{3}\\right)$ 的图象, 只要把函数 $y=\\sin x$ 的图象", "options": "A. 向上平移 $\\frac{\\pi}{3}$ 个单位长度\n\nB. 向下平移 $\\frac{\\pi}{3}$ 个单位长度\n\nC. 向左平移 $\\frac{\\pi}{3}$ 个单位长度\n\nD. 向右平移 $\\frac{\\pi}{3}$ 个单位长度", "subject": "变换几何", "analysis": "解析: 由题意,只要把函数 $y=\\sin x$ 的图象向右平移 $\\frac{\\pi}{3}$ 个单位长度即可.\n\n答案: D"} {"id": "17846", "image": [], "answer": "C", "solution": "null", "level": "高二", "question": "将函数 $y=\\sin 2 x$ 的图象向左平移 $\\varphi(\\varphi>0)$ 个单位长度, 所得图象对应的函数为偶函数, 则 $\\varphi$ 的最小值为", "options": "A. $\\frac{\\pi}{6}$\n B. $\\frac{\\pi}{3}$\n C. $\\frac{\\pi}{4}$\n D. $\\frac{\\pi}{12}$", "subject": "变换几何", "analysis": "解析 : 由题意得,将函数 $y=\\sin 2 x$ 的图象向左平移 $\\varphi(\\varphi>0)$ 个单位长度,得函数 $y=\\sin 2(x$ $+\\varphi)=\\sin (2 x+2 \\varphi)$ 的图象. 因为它是偶函数,所以 $2 \\varphi=\\frac{\\pi}{2}+k \\pi , k \\in \\mathbf{Z}$ ,即 $\\varphi=\\frac{\\pi}{4}+\\frac{k \\pi}{2} , k \\in \\mathbf{Z}$ ,所以 $\\varphi$ 的最小值是 $\\frac{\\pi}{4}$ ,故选 C.\n\n答案:"} {"id": "17851", "image": [], "answer": "A", "solution": "null", "level": "高二", "question": "为了得到 $y=\\cos \\frac{x}{4}$ 的图象, 只需把 $y=\\cos x$ 的图象上的所有点", "options": "A. 横坐标伸长到原来的 4 倍, 纵坐标不变\n\nB. 横坐标缩短到原来的 $\\frac{1}{4}$, 纵坐标不变\n\nC. 纵坐标伸长到原来的 4 倍, 横坐标不变\n\nD. 纵坐标缩短到原来的 $\\frac{1}{4}$, 横坐标不变", "subject": "变换几何", "analysis": "解析: 由图象的周期变换可知, A 正确.\n\n答案: A"} {"id": "17852", "image": [], "answer": "B", "solution": "null", "level": "高二", "question": "要得到函数 $y=\\sin \\left(4 x-\\frac{\\pi}{3}\\right)$ 的图象, 只需将函数 $y=\\sin 4 x$ 的图象", "options": "A. 向左平移 $\\frac{\\pi}{12}$ 个单位长度\n\nB. 向右平移 $\\frac{\\pi}{12}$ 个单位长度\n\nC. 向左平移 $\\frac{\\pi}{3}$ 个单位长度\n\nD. 向右平移 $\\frac{\\pi}{3}$ 个单位长度", "subject": "变换几何", "analysis": "解析: $y=\\sin \\left(4 x-\\frac{\\pi}{3}\\right)=\\sin \\left[4\\left(x-\\frac{\\pi}{12}\\right)\\right]$, 故只需将函数 $y=\\sin 4 x$ 的图象向右平移 $\\frac{\\pi}{12}$ 个单位长\n\n度. 故选 B.\n答案: B"} {"id": "17853", "image": [], "answer": "B", "solution": "null", "level": "高二", "question": "把函数 $y=\\cos x$ 的图象上所有点的横坐标缩短到原来的 $\\frac{1}{2}$, 纵坐标伸长到原来的 2 倍, 最后把图象向左平移 $\\frac{\\pi}{4}$ 个单位长度, 则所得图象表示的函数的解析式为", "options": "A. $y=2 \\sin 2 x$\nB. $y=-2 \\sin 2 x$\nC. $y=2 \\cos \\left(2 x+\\frac{\\pi}{4}\\right)$\nD. $y=2 \\cos \\left(\\frac{x}{2}+\\frac{\\pi}{4}\\right)$", "subject": "变换几何", "analysis": "解析: 把函数 $y=\\cos x$ 的图象上所有点的横坐标缩短到原来的 $\\frac{1}{2}$ ,所得图象的函数解析式为 $y=\\cos 2 x$ ,再把纵坐标伸长到原来的 2 倍,所得图象的函数解析式为 $y=2 \\cos 2 x$ ,最后把图象向左平移 $\\frac{\\pi}{4}$ 个单位长度,所得图象的函数解析式为 $y=2 \\cos \\left[2\\left(x+\\frac{\\pi}{4}\\right)\\right]=-2 \\sin 2 x$.答案: B"} {"id": "17854", "image": [], "answer": "D", "solution": "null", "level": "高二", "question": "把函数 $f(x)=\\sin \\left(2 x-\\frac{\\pi}{3}\\right)$ 的图象向左平移 $\\varphi(0<\\varphi<\\pi)$ 个单位长度可以得到函数 $g(x)$ 的图象. 若 $g(x)$ 的图象关于 $y$ 轴对称, 则 $\\varphi$ 的值为", "options": "A. $\\frac{5 \\pi}{12}$\nB. $\\frac{7 \\pi}{12}$\nC. $\\frac{5 \\pi}{6}$ 或 $\\frac{\\pi}{6}$\nD. $\\frac{5 \\pi}{12}$ 或 $\\frac{11 \\pi}{12}$", "subject": "变换几何", "analysis": "解析: 由题意,得 $g(x)=\\sin \\left[2(x+\\varphi)-\\frac{\\pi}{3}\\right]=\\sin \\left(2 x+2 \\varphi-\\frac{\\pi}{3}\\right)$.\n\n$\\because g(x)$ 的图象关于 $y$ 轴对称, $\\therefore g(x)$ 为偶函数,\n\n$\\therefore 2 \\varphi-\\frac{\\pi}{3}=k \\pi+\\frac{\\pi}{2}(k \\in \\mathbf{Z}), \\quad \\therefore \\varphi=\\frac{k \\pi}{2}+\\frac{5 \\pi}{12}(k \\in \\mathbf{Z})$.\n\n当 $k=0$ 时, $\\varphi=\\frac{5 \\pi}{12}$ ;当 $k=1$ 时, $\\varphi=\\frac{11 \\pi}{12}$ ,故选 D.\n\n答案: D"} {"id": "17860", "image": [], "answer": "D", "solution": "null", "level": "高二", "question": "已知函数 $f(x)=\\sin \\left(2 x-\\frac{\\pi}{6}\\right)$. 若存在 $a \\in(0, \\pi)$, 使得 $f(x+a)=f(x-a)$ 恒成立, 则 $a$ 的值是", "options": "A. $\\frac{\\pi}{6}$\n B. $\\frac{\\pi}{3}$\n C. $\\frac{\\pi}{4}$\n D. $\\frac{\\pi}{2}$", "subject": "图论", "analysis": "解析: 因为 $f(x+a)=f(x-a)$ ,所以函数 $f(x)=\\sin \\left(2 x-\\frac{\\pi}{6}\\right)$ 的周期为 $2 a$. 因为 $a \\in(0 , \\pi)$ ,所以 $2 a=\\frac{2 \\pi}{2}$ ,即 $a=\\frac{\\pi}{2}$ ,故选 D.\n\n答案: D"} {"id": "17874", "image": ["9005.jpg", "9006.jpg", "9007.jpg", "9008.jpg", "9009.jpg", "9010.jpg"], "answer": "C", "solution": "null", "level": "高二", "question": "如图, 设点 $A$ 是单位圆上的一定点, 动点 $P$ 从点 $A$ 出发在圆上按逆时针方向旋转一周, 点 $P$ 所旋转过的弧 $\\widehat{A P}$ 的长为 $l$, 弦 $A P$ 的长为 $d$, 则函数 $d=f(l)$ 的图象大致是 $(\\quad)$\n\n\n\n", "options": "A.\n\n\n\nC\n\n\n\nB\n\n\n\nD", "subject": "组合几何学", "analysis": "解析: 如图,过 $O$ 作 $O D \\perp A P$ 于 $D$.由题意知, $\\angle A O D=\\frac{l}{2} , O A=1 , A D=\\frac{d}{2} , \\therefore \\sin \\frac{l}{2}=\\frac{d}{2}$ ,即 $d=2 \\sin \\frac{1}{2}$结合图象知选 C.\n\n\n\n答案: C"} {"id": "17881", "image": ["9014.jpg"], "answer": "A", "solution": "null", "level": "高二", "question": "如图所示的是一半径为 $3 \\mathrm{~m}$ 的圆形水轮, 水轮的中心 $O$ 距离水面 $2 \\mathrm{~m}$, 已知水轮自点 $B$ 开始旋转, $15 \\mathrm{~s}$ 旋转一圈, 水轮上的点 $P$ 到水面的距离 $y(\\mathrm{~m})$ 与时间 $x(\\mathrm{~s})$ 满足函数关系式 $y=$ $A \\sin (\\omega x+\\varphi)+2$, 则有\n\n", "options": "A. $\\omega=\\frac{2 \\pi}{15}, A=3$\nB. $\\omega=\\frac{15}{2 \\pi}, A=3$\nC. $\\omega=\\frac{2 \\pi}{15}, A=5$\nD. $\\omega=\\frac{15}{2 \\pi}, A=5$", "subject": "组合几何学", "analysis": "解析: $\\because T=15 , \\therefore \\omega=\\frac{2 \\pi}{T}=\\frac{2 \\pi}{15}$.\n\n显然 $y_{\\max }-y_{\\text {min }}=6 ,$\n\n$\\therefore A=\\frac{y_{\\text {max }}-y_{\\text {min }}}{2}=\\frac{6}{2}=3$,\n\n答案: A"} {"id": "18475", "image": [], "answer": "A", "solution": "null", "level": "高二", "question": "已知三角形面积为 $\\frac{1}{4}$, 外接圆面积为 $\\pi$, 则这个三角形的三边之积为 $($ )", "options": "A. 1\nB. 2\nC. $\\frac{1}{2}$\nD. 4", "subject": "立体几何学", "analysis": "解析 设三角形外接圆半径为 $R$, 则由 $\\pi R^{2}=\\pi$,\n\n得 $R=1$, 由 $S_{\\triangle}=\\frac{1}{2} a b \\sin C=\\frac{a b c}{4 R}=\\frac{a b c}{4}=\\frac{1}{4}, \\therefore a b c=1$​."} {"id": "17893", "image": [], "answer": "C", "solution": "null", "level": "高二", "question": "扇形的周长为 $6 \\mathrm{~cm}$, 面积是 $2 \\mathrm{~cm}^{2}$, 则扇形的圆心角的弧度数是", "options": "A. 1\nB. 4\nC. 1 或 4\nD. 2 或 4", "subject": "立体几何学", "analysis": "解析:设扇形的圆心角为 $a \\mathrm{rad}$, 半径为 $R \\mathrm{~cm}$,\n则 $\\left\\{\\begin{array}{l}2 R+\\alpha \\cdot R=6, \\\\ \\frac{1}{2} R^{2} \\cdot \\alpha=2,\\end{array}\\right.$ 解得 $\\alpha=1$ 或 $\\alpha=4$, 选 C.\n\n答案: C"} {"id": "17963", "image": [], "answer": "D", "solution": "null", "level": "高二", "question": "若向量 $\\boldsymbol{a}, \\boldsymbol{b}, \\boldsymbol{c}$ 满足 $\\boldsymbol{a} / / \\boldsymbol{b}$ 且 $\\boldsymbol{a} \\perp \\boldsymbol{c}$, 则 $\\boldsymbol{c} \\cdot(\\boldsymbol{a}+2 \\boldsymbol{b})=$", "options": "A. 4\n B. 3\n C. 2\n D. 0", "subject": "立体几何学", "analysis": "解析: $\\because a / / b, \\therefore b=\\lambda a, \\lambda \\in \\mathbf{R} . \\therefore c \\cdot(a+2 b)=c \\cdot(a+2 \\lambda a)=c \\cdot a(1+2 \\lambda) . \\because a \\perp c, \\therefore a \\cdot c=0 . \\therefore c \\cdot(a$ $+2 b)=0$.\n\n答案: D"} {"id": "17970", "image": [], "answer": "C", "solution": "null", "level": "高二", "question": "已知 $|\\boldsymbol{a}|=|\\boldsymbol{b}|=2, \\boldsymbol{a} \\cdot \\boldsymbol{b}=2$, 则 $|\\boldsymbol{a}-\\boldsymbol{b}|=$", "options": "A. 1 B. $\\sqrt{3}$\nC. 2 D. $\\sqrt{3}$ 或 2", "subject": "立体几何学", "analysis": "解析: $|\\boldsymbol{a}-\\boldsymbol{b}|=\\sqrt{|\\boldsymbol{a}-\\boldsymbol{b}|^{2}}=\\sqrt{(\\boldsymbol{a}-\\boldsymbol{b})^{2}}=\\sqrt{\\boldsymbol{a}^{2}-2 \\boldsymbol{a} \\cdot \\boldsymbol{b}+\\boldsymbol{b}^{2}}=\\sqrt{2^{2}-2 \\times 2+2^{2}}=\\sqrt{4}=2$.\n\n答案: C"} {"id": "18901", "image": [], "answer": "D", "solution": "null", "level": "高二", "question": "在 $\\triangle A B C$ 中, 角 $A, B, C$ 的对边分\n\n别为 $a, b, c$, 若 $a=1, \\sqrt{3} \\sin A \\cos C+(\\sqrt{3} \\sin C+b) \\cos A=0$, 则角 $A=$", "options": "A. $\\frac{2 \\pi}{3}$\nB. $\\frac{\\pi}{3}$\nC. $\\frac{\\pi}{6}$\nD. $\\frac{5 \\pi}{6}$", "subject": "立体几何学", "analysis": " $\\because a=1, \\sqrt{3} \\sin A \\cos C+(\\sqrt{3} \\sin C+b) \\cos A=0$,\n\n$\\therefore \\sqrt{3} \\sin A \\cos C+\\sqrt{3} \\sin C \\cos A=-b \\cos A$,\n$\\therefore \\sqrt{3} \\sin (A+C)=\\sqrt{3} \\sin B=-b \\cos A$,\n\n$\\therefore \\sqrt{3} a \\sin B=-b \\cos A$,\n\n由正弦定理可得: $\\sqrt{3} \\sin A \\sin B=-\\sin B \\cos A$,\n\n$\\because \\sin B>0, \\therefore \\sqrt{3} \\sin A=-\\cos A$, 即 $\\tan A=-\\frac{\\sqrt{3}}{3}$,\n\n$\\because A \\in(0, \\pi), \\therefore A=\\frac{5 \\pi}{6}$. 故选 D."} {"id": "18926", "image": [], "answer": "B", "solution": "null", "level": "高二", "question": "已知 $A, B, C$ 是海面上的三座岛屿, 测得 $\\angle A B C=30^{\\circ}, \\angle B A C=105^{\\circ}$, 从岛屿 $A$ 到岛屿 $C$ 需要 30 分钟, 按照同样的速度, 从岛屿 $A$ 到岛屿 $B$ 需要 (取 $\\sqrt{2} \\approx 1.4, \\sqrt{3} \\approx 1.7$ )", "options": "A. 51 分钟\nB. 42 分钟\nC. 39 分钟\nD. 36 分钟", "subject": "立体几何学", "analysis": "设从从岛屿 $A$ 到岛屿 $C$ 速度为 $v$, 从岛屿 $A$ 到岛屿 $B$ 需要 $t$ 分钟, 由正弦定理, 得\n$\\frac{30 v}{\\sin 30^{\\circ}}=\\frac{v t}{\\sin \\left(180^{\\circ}-30^{\\circ}-105^{\\circ}\\right)}, t=\\frac{30 \\sin 45^{\\circ}}{\\sin 30^{\\circ}}=30 \\sqrt{2} \\approx 42$. 故选 B."} {"id": "18929", "image": [], "answer": "A", "solution": "null", "level": "高二", "question": "在 $\\triangle A B C$ 中, 角 $B$ 为 $\\frac{3 \\pi}{4}, B C$ 边上的高恰为 $B C$ 边长的一半, 则 $\\cos A=$", "options": "A. $\\frac{2 \\sqrt{5}}{5}$\nB. $\\frac{\\sqrt{5}}{5}$\nC. $\\frac{2}{3}$\nD. $\\frac{\\sqrt{5}}{3}$", "subject": "立体几何学", "analysis": "作 $A H \\perp B C$, 垂足点 $H$ 在 $C B$ 的延长线上, $\\triangle A H B$ 为等腰直角三角形, 设 $B C=2 a$, 则 $A B=\\sqrt{2} a, A H=a, C H=3 a$, 由勾股定理得 $A C=\\sqrt{10} a$, 由余弦定理得 $\\cos A=\\frac{2 a^{2}+10 a^{2}-4 a^{2}}{2 \\times \\sqrt{2} a \\times \\sqrt{10} a}=\\frac{2 \\sqrt{5}}{5}$, 故选 A."} {"id": "19709", "image": [], "answer": "B", "solution": "null", "level": "高二", "question": "我国古代数学名著《算法统宗》中有如下问题: “远望巍巍塔七层, 红光点点倍加增,共灯三百八十一, 请问尖头几盛灯? ” 意思是: 一座 7 层塔共挂了 381 盏灯, 且相邻两层中的下一层灯数是上一层灯数的 2 倍, 则塔的顶层共有灯 ( )", "options": "A. 1 盛\n B. 3 盛\n C. 5 盏\n D. 9 盏", "subject": "组合数学", "analysis": "设这个塔顶层有 $\\mathrm{a}$ 盏灯, $\\because$ 宝塔一共有七层, 每层悬挂的红灯数是上一层的 2 倍, $\\therefore$从塔顶层依次向下每层灯数是以 2 为公比、 $\\mathrm{a}$ 为首项的等比数列, 又总共有灯 381 盛, $\\therefore 381=\\frac{a\\left(1-2^{7}\\right)}{1-2}=127 \\mathrm{a}$, 解得 $\\mathrm{a}=3$, 则这个塔顶层有 3 盛灯.故选 $\\mathrm{B}$"} {"id": "19868", "image": [], "answer": "B", "solution": "null", "level": "高二", "question": "已知集合 $A=\\{x \\in N \\mid x \\leq 6\\}, B=\\left\\{x \\in R \\mid x^{2}-4 x>0\\right\\}$, 则 $A \\cap B=( )$", "options": "A. $\\{4,5,6\\}$\nB. $\\{5,6\\}$\nC. $\\{x \\mid 40\\right\\}=\\{x \\in R \\mid x<0$ 或 $x>4\\}, \\therefore A \\cap B=\\{5,6\\}$. 故选 $B$."} {"id": "18885", "image": [], "answer": "D", "solution": "null", "level": "高二", "question": "已知数列 $\\left\\{a_{n}\\right\\}$ 的通项公式为 $a_{n}=2 n+2$, 将这个数列中的项摆放成如图所示的数阵. 记 $b_{n}$ 为数阵从左至右的 $n$ 列, 从上到下的 $n$ 行共 $n^{2}$ 个数的和, 则数列 $\\left\\{\\frac{n}{b_{n}}\\right\\}$ 的前 2020 项和为 ( )", "options": "A. $\\frac{1011}{2020}$\nB. $\\frac{2019}{2020}$\nC. $\\frac{2020}{2021}$\nD. $\\frac{1010}{2021}$", "subject": "组合数学", "analysis": "由题意, 设每一行的和为 $c_{i}$, 故 $c_{i}=a_{i}+a_{i+1}+\\ldots+a_{n+1-i}=\\frac{\\left(a_{i}+a_{n+1-i}\\right) n}{2}=n(n+2 i+1)$\n\n因此: $b_{n}=c_{1}+c_{2}+\\ldots+c_{n}=n[(n+3)+(n+5)+\\ldots+(n+2 n+1)]=2 n^{2}(n+1)$, $\\frac{n}{b_{n}}=\\frac{1}{2 n(n+1)}=\\frac{1}{2}\\left(\\frac{1}{n}-\\frac{1}{n+1}\\right)$, 故 $S_{2020}=\\frac{1}{2}\\left(1-\\frac{1}{2}+\\frac{1}{2}-\\frac{1}{3}+\\ldots+\\frac{1}{2020}-\\frac{1}{2021}\\right)=\\frac{1}{2}\\left(1-\\frac{1}{2021}\\right)=\\frac{1010}{2021}$, 故选 D。"} {"id": "19853", "image": [], "answer": "C", "solution": "null", "level": "高二", "question": "下列命题: (1)若 $a>b$ 且 $a, b$ 同号, 则 $\\frac{1}{-}<\\frac{1}{b}$; (2) 若 $\\frac{1}{-}>1$, 则 $0b, n \\in N_{+} \\Rightarrow a^{2 n-1}>b^{2 n-1}$.\n\n其中真命题个数为 ( )", "options": "A. 1\nB. 2\nC. 3\nD. 4", "subject": "逻辑题", "analysis": "(1)正确.\n\n$$\n\\because a b>0, a>b, \\therefore \\frac{a}{a b}>\\frac{b}{a b}, \\frac{1}{1} \\frac{1}{a} \\text {, 即 } b>a \\text {. }\n$$\n\n$\\xrightarrow{1}>1$\n\n(2)正确, 由 ${ }^{a}$, 可得 $\\mathrm{a}$ 正数, 两边同乘以 $a$ 可得 $0b$, 则得 $a^{2 n-1}>b^{2 n-1}$, 故选 $C$."} {"id": "20152", "image": [], "answer": "A", "solution": "null", "level": "高二", "question": "已知 $a, b \\in \\mathbf{R}$, 则命题 “若 $a>b$, 则 $\\frac{1}{a}<\\frac{1}{b}$ ” 的逆命题、否命题、逆否命题, 这三个命题中真命题的个数有 $($ )", "options": "A. 0\nB. 1\nC. 2\nD. 3", "subject": "逻辑题", "analysis": "解析 :原命题 “ $a>b$, 则 $\\frac{1}{a}<\\frac{1}{b}$ ” 是假命题, 其逆命题 “ $\\frac{1}{a}<\\frac{1}{b}$, 则 $a>b$ ” 也是假命题, 又原命题与逆否命题等价, 逆命题与否命题等价,故三个命题都是假命题。\n\n答案 :A"} {"id": "20153", "image": [], "answer": "C", "solution": "null", "level": "高二", "question": "下列命题:\n\n(1) “全等三角形的面积相等” 的逆命题;\n\n(2) “正三角形的三个角均为 $60^{\\circ}$ ” 的否命题;\n\n(3) “若 $k<0$, 则方程 $x^{2}+(2 k+1) x+k=0$ 必有两相异实数根” 的逆否命题.\n\n其中真命题的个数是( )", "options": "A. 0\nB. 1\nC. 2\nD. 3", "subject": "逻辑题", "analysis": "解析 :(1)的逆命题 “面积相等的三角形必全等” 是假命题.\n\n(2) 的否命题 “不是正三角形的三个内角不全为 $60^{\\circ}$ ” 为真命题.\n\n(3) 当 $k<0$ 时, $\\Delta=(2 k+1)^{2}-4 k=4 k^{2}+1>0$, 方程有两相异实根,原命题与逆否命题均为真命题.\n\n答案 :C"} {"id": "20154", "image": [], "answer": "B", "solution": "null", "level": "高二", "question": "命题 “若方程 $a x^{2}+b x+c=0(a \\neq 0)$ 的 $\\Delta=b^{2}-4 a c<0$, 则方程无实根” 的否命题的逆否命题是( )", "options": "A. 若方程 $a x^{2}+b x+c=0(a \\neq 0)$ 的 $\\Delta=b^{2}-4 a c \\geqslant 0$, 则方程有二实根\n\nB. 若方程 $a x^{2}+b x+c=0(a \\neq 0)$ 无实根, 则其 $\\Delta=b^{2}-4 a c<0$\n\nC. 若方程 $a x^{2}+b x+c=0(a \\neq 0)$ 有二实根, 则其 $\\Delta=b^{2}-4 a c \\geqslant 0$\n\nD. 以上均不对", "subject": "逻辑题", "analysis": ""} {"id": "20171", "image": [], "answer": "D", "solution": "null", "level": "高二", "question": "下列命题为特称命题的是 ( )", "options": "A. 偶函数的图象关于 $y$ 轴对称\n\nB. 正四棱柱都是平行六面体\n\nC. 不相交的两条直线是平行线\n\nD. 存在大于等于 3 的实数", "subject": "逻辑题", "analysis": "解析 :选项 A, B, C 都是全称命题, 选项 D 含有存在量词, 是特称命题.\n\n答案: D"} {"id": "20177", "image": [], "answer": "C", "solution": "null", "level": "高二", "question": "命题 “存在实数 $x$, 使 $x>1$ ” 的否定是 ( )", "options": "A. 对任意实数 $x$, 都有 $x>1$\n\nB. 不存在实数 $x$, 使 $x \\leqslant 1$\n\nC. 对任意实数 $x$ ,都有 $x \\leqslant 1$\n\nD. 存在实数 $x$, 使 $x \\leqslant 1$\n\n", "subject": "逻辑题", "analysis": "C"} {"id": "20201", "image": [], "answer": "C", "solution": "null", "level": "高二", "question": "已知命题 $p_{1}$ :函数 $y=2^{x}-2^{-x}$ 在 $\\mathbf{R}$ 上为增函数; $p_{2}$ :函数 $y$ $=2^{x}+2^{-x}$ 在 $\\mathbf{R}$ 上为减函数. 则在命题 $q_{1}: p_{1} \\vee p_{2}, q_{2}: p_{1} \\wedge p_{2}, q_{3}:$ (綈 $\\left.p_{1}\\right) \\vee p_{2}$ 和 $q_{4}: p_{1} \\wedge$ (綈 $\\left.p_{2}\\right)$ 中,真命题是 $(\\quad)$", "options": "A. $q_{1}, q_{3}$\nB. $q_{2}, q_{3}$\nC. $q_{1}, q_{4}$\nD. $q_{2}, q_{4}$", "subject": "逻辑题", "analysis": "解析 :由题知, $p_{1}$ 为真命题, $p_{2}$ 为假命题,\n\n$\\therefore q_{1}, q_{4}$ 为真命题, 故选 C.\n\n答案 :C"} {"id": "20203", "image": [], "answer": "B", "solution": "null", "level": "高二", "question": "命题 $p$ : 若不等式 $x^{2}+x+m>0$ 恒成立, 则 $m>\\frac{1}{4}$, 命题 $q$ : 在 $\\triangle A B C$ 中, $\\angle A>\\angle B$ 是 $\\sin A>\\sin B$ 的充要条件, 则( )", "options": "A. $p$ 真 $q$ 假\nB. “ $p \\wedge q$ ” 为真\nC. “ $p \\vee q$ ” 为假\nD. “綈 $p \\bigvee$ 綈 $q$ ” 为真", "subject": "逻辑题", "analysis": "解析 :$x^{2}+x+m>0$ 恒成立, 只需 $\\Delta=1-4 m<0$, 即 $m>\\frac{1}{4}, \\therefore$ 命题 $p$ 正确. 在 $\\triangle A B C$ 中, $\\angle A>\\angle B \\Leftrightarrow \\sin A>\\sin B, \\therefore$ 命题 $q$ 正确. 故选 B.\n\n答案 :B"} {"id": "20204", "image": [], "answer": "B", "solution": "null", "level": "高二", "question": "由下列各组命题构成的 “ $p \\vee q$ ” “ $p \\wedge q$ ” “綈 $p$ ” 形式的新命题中, “ $p \\vee q$ ” 为真, “ $p \\wedge q$ ” 为假, “綈 $p$ ” 为真的是 ( )", "options": "A. $p: 3$ 是偶数, $q: 4$ 是奇数\nB. $p: 3+2=6, q: 5>3$\nC. $p: a \\in\\{a, b\\}, q:\\{a\\}\\{a, b\\}$\nD. $p: Q \\quad R, q: N=N^{*}$", "subject": "逻辑题", "analysis": "解析: 选项 B 中, $\\because p: 3+2=6$ 为假命题, $\\therefore$ 綈 $p$ 为真. $q: 5>3$为真命题, $\\therefore “ p \\vee q$ ” 为真, “ $p \\wedge q$ ”为假.\n\n答案 :B"} {"id": "19922", "image": [], "answer": "B", "solution": "null", "level": "高二", "question": "下列语句中,不能称为命题的是 ( )", "options": "A. $5>12$\nB. $x>0$\nC. 若 $a \\perp b$,则 $a \\cdot b=0$\nD.三角形的三条中线交于一点", "subject": "逻辑题", "analysis": "解析:解答:选项 A 能判断为假,选项 C,D 能判断为真,而选项 B 中, 因为在给 $x$ 赋值之前, 不能判断 $x>0$ 的真假,所以 $x>0$ 不是命题., 故选 B.\n\n分析:可以判断真假、用文字或符号表述的语句叫作命题,并不是任何语句都是命题,只有那些能判断真假的语句才是命题.一般来说, 疑问句、祈使句、感叹句都不是命题.在数学或其他领域, 有一类陈述句, 如“每一个不小于 6 的偶数都是两个奇质数的和”, 目前不能判断它的真假,但以后总能确定它的真假,因此通常把它也算作命题."} {"id": "19923", "image": [], "answer": "D", "solution": "null", "level": "高二", "question": "命题: “若 $x^{2}<1$, 则 $-11$, 或 $x<-1$, 则 $x^{2}>1$\n\nD. 若 $x \\geq 1$, 或 $x \\leq-1$, 则 $x^{2} \\geq 1$", "subject": "逻辑题", "analysis": "解析:解答:逆否命题是逆命题的否命题:若 $x^{2}<1$, 则 $-10$\nB. $\\forall x \\in \\mathrm{R}, x^{2}+x-1 \\geq 0$\nC. $\\exists x \\notin \\mathrm{R}, x^{2}+x-1 \\geq 0$\nD. $\\forall x \\notin \\mathrm{R}, x^{2}+x-1>0$", "subject": "逻辑题", "analysis": "解析: 解答: 命题 $p: \\exists x \\in \\mathrm{R}, x^{2}+x-1<0$, 则 $\\neg p: \\forall x \\in \\mathrm{R}, x^{2}+x-1 \\geq 0$. 故选 B\n\n分析: 本题主要考查含有存在量词的命题的否定, 需将存在量词改为全称量词, 并将结论否定;"} {"id": "1292", "image": [], "answer": "A", "solution": "null", "level": "七年级", "question": "2022 的绝对值是 ( )", "options": "2022\nB. $\\frac{1}{2022}$\nC. -2022\nD. $-\\frac{1}{2022}$", "subject": "算术", "analysis": "2022 的绝对值是 2022.\n\n故答案为: A."} {"id": "1293", "image": ["3089.jpg"], "answer": "C", "solution": "null", "level": "七年级", "question": "实数 $\\mathrm{a}$ 在数轴上的位置如图所示, 则 $-\\frac{1}{a}, 1,0$ 的大小顺序是 ( )\n\n", "options": "A. $-\\frac{1}{a}<0<1$\nB. $0<-\\frac{1}{a}<1$\nC. $0<1<-\\frac{1}{a}$\nD. $0<1$ 且 1 和 $-\\frac{1}{a}$ 的大小无法确定", "subject": "算术", "analysis": "because-1<\\mathrm{a}<0$\n\n$-\\frac{1}{a}>1$\n$\\therefore 0<1<-\\frac{1}{a}$.\n\n故答案为: $\\mathrm{C}$"} {"id": "1303", "image": [], "answer": "D", "solution": "null", "level": "七年级", "question": "如果一个数的绝对值是它的相反数, 则这个数是()", "options": "A. 正数\nB. 负数\nC. 正数或零\nD. 负数或零", "subject": "算术", "analysis": "$\\because$ 一个数的绝对值是它的相反数,\n\n设这个数是 $a$, 则\n\n$|a|=-a \\geq 0$\n\n$\\therefore \\mathrm{a} \\leq 0$.\n\n故答案为: D."} {"id": "1309", "image": [], "answer": "A", "solution": "null", "level": "七年级", "question": "如果用・代表间一个自然数 $(\\bullet \\neq 0)$, 那么下面各式中, 得数最大的是( )", "options": "A. $\\bullet \\div \\frac{6}{7}$\nB. $\\frac{6}{7} \\div$\nC. $\\bullet \\times \\frac{6}{7}$\nD. $\\cdot-\\frac{6}{7}$", "subject": "算术", "analysis": "$\\because 0$ 不能作除数,\n\n当 $\\bullet$ 为 1 时,\n\n$1 \\div \\frac{6}{7}=\\frac{7}{6}$\n\n$\\frac{6}{7} \\div 1=\\frac{6}{7}$\n\n$1 \\times \\frac{6}{7}=\\frac{6}{7}$\n\n$1-\\frac{6}{7}=\\frac{1}{7}$\n\n$\\therefore$ 得数最大的数是 $\\frac{6}{7}$.\n\n故答案为: A."} {"id": "1310", "image": [], "answer": "D", "solution": "null", "level": "七年级", "question": "若 $\\mathrm{a}$ 表示一个有理数, 且有 $|-3-a|=3+|a|$, 则 $\\mathrm{a}$ 应该是()", "options": "任意一个有理数\nB. 任意一个正数\nC. 任意一个负数\nD. 任意一个非负数", "subject": "算术", "analysis": "当 $a \\geq 0$ 时, 得 $3+a=3+a, \\therefore a$ 为可以为一切非负数,当 $-3 \\leq a<0$ 时, 得 $3+a=3-a, \\quad \\therefore a$ 为 0 , 不符合题意, 舍去,当 $a<-3$ 时, 得 $3+a=3-a, \\therefore a$ 为 0 , 不符合题意, 舍去,综上 a 为可以为一切非负数,故答案为: D"} {"id": "1312", "image": ["3104.jpg"], "answer": "C", "solution": "null", "level": "七年级", "question": "有理数 $a 、 b$ 满足 $|a-b|=|a|+|b|$, 则 $a 、 b$ 应满足的条件是 ( )", "options": "$a b \\geq 0$\nB. ab $>1$\nC. $a b \\leq 0$\nD. $a b \\leq 1$", "subject": "算术", "analysis": "$\\because$ 有理数 $a 、 b$ 满足 $|a-b|=|a|+|b|$,\n\n当 $a>0, b>0$ 时 $|a-b|\n\nA、 $a b \\geq 0$, 可知 $a 、 b$ 是同号或为 0 , 都为 0 是成立, 同号时条件不成立, 故此选项不正确;\n\nB、 $a b>1$ ,可知 $a 、 b$ 是同号,同号时条件不成立,故此选项不正确;\n\nC、 $a b \\leq 0$, 可知 $a 、 b$ 是异号或为 0 , 满足条件, 故此选项正确;\n\nD、 $a b \\leq 1$, 当 $0-a>-c$;\n(2) $\\frac{a b}{|a b|}-\\frac{a c}{|a c|}=0$\n(3) $|a+b|=|a|+|b|$\n(4) $|a-b|-|c-b|+|a-c|=$\n\n0 . 其中正确的有 $(\\quad)$\n\n", "options": "4 个\nB. 3 个\nC. 2 个\nD. 1 个", "subject": "算术", "analysis": "$\\because|a|<|b|<|c|$,\n\n$\\therefore$ (1)-b>-a>-c, 故(1) 符合题意;\n\n(2) $\\frac{a b}{|a b|}-\\frac{a c}{|a c|}=\\frac{a b}{-a b}-\\frac{a c}{a c}=1+1=2$, 故(2)不符合题意;\n\n(3) $|a+b|=|a|+|b|$, 故(3)符合题意;\n\n(4) $|\\mathrm{a}-\\mathrm{b}|-|\\mathrm{c}-\\mathrm{b}|+|\\mathrm{a}-\\mathrm{c}|=\\mathrm{a}-\\mathrm{b}-(\\mathrm{c}-\\mathrm{b})+(\\mathrm{c}-\\mathrm{a})=\\mathrm{a}-\\mathrm{b}-\\mathrm{c}+\\mathrm{b}+\\mathrm{c}-\\mathrm{a}=0$, 故(4)符合题意:\n\n所以正确的个数有(1)(3)(4), 共 3 个.\n\n故答案为: B."} {"id": "1316", "image": [], "answer": "D", "solution": "null", "level": "七年级", "question": "下列式子中正确的是 $(\\quad)$", "options": "$-|-31|=31$\nB. $(-5)+(-5)+(-5)+(-5)+(-5)=(-5)^{5}$\nC. $-8 \\div(2-4)=-4+2=-2$\nD. $|-3-1|=|-3|+|-1|$", "subject": "算术", "analysis": "A、 $-|-31|=-31$, 此项不符合题意;\n\nB、因为 $(-5)+(-5)+(-5)+(-5)+(-5)=(-5) \\times(-5)$, $(-5)^{5}=(-5) \\times(-5) \\times(-5) \\times(-5) \\times(-5)$,所以 $(-5)+(-5)+(-5)+(-5)+(-5) \\neq(-5)^{5}$, 此项不符合题意;\n\nC、 $-8 \\div(2-4)=-8 \\div(-2)=4$, 此项不符合题意;\n\nD、因为 $|-3-1|=|-4|=4,|-3|+|-1|=3+1=4$,\n\n所以 $|-3-1|=|-3|+|-1|$, 此项符合题意;\n\n故答案为: D."} {"id": "1317", "image": [], "answer": "B", "solution": "null", "level": "七年级", "question": "观察下列式子:\n\n$1 \\times 2+2 \\times 3+3 \\times 4=\\frac{1}{3} \\times 3 \\times 4 \\times 5$\n\n$1 \\times 2+2 \\times 3+3 \\times 4+4 \\times 5=\\frac{1}{3} \\times 4 \\times 5 \\times 6$\n\n$1 \\times 2+2 \\times 3+3 \\times 4+4 \\times 5+5 \\times 6=\\frac{1}{3} \\times 5 \\times 6 \\times 7$\n\n探索以上式子的规律, 与计算 $11 \\times 12+12 \\times 13+\\cdots+18 \\times 19+19 \\times 20$ 的结果相等的算式是", "options": "A. $\\frac{1}{3} \\times(19 \\times 20 \\times 21-9 \\times 10 \\times 11)$\nB. $\\frac{1}{3} \\times(19 \\times 20 \\times 21-10 \\times 11 \\times 12)$\nC. $\\frac{1}{3} \\times(18 \\times 19 \\times 20-9 \\times 10 \\times 11)$\nD. $\\frac{1}{3} \\times(18 \\times 19 \\times 20-10 \\times 11 \\times 12)$", "subject": "算术", "analysis": "$11 \\times 12+12 \\times 13+\\cdots+18 \\times 19+19 \\times 20=(1 \\times 2+2 \\times 3+\\cdots+19 \\times 20)-(1 \\times 2+$ $2 \\times 3+\\cdots+10 \\times 11)=\\frac{1}{3} \\times 19 \\times 20 \\times 21-\\frac{1}{3} \\times 10 \\times 11 \\times 12$\n\n$=\\frac{1}{3} \\times(19 \\times 20 \\times 21-10 \\times 11 \\times 12)$\n\n故答案为: B."} {"id": "1327", "image": [], "answer": "B", "solution": "null", "level": "七年级", "question": "小明做了下列 3 道计算题:\n(1) $\\frac{1}{2}-\\frac{1}{2} \\times 2=0 \\times 2=0$,\n(2) $-2^{3}-(-3)^{2}=-8-9=-17$\n(3) $6 \\div\\left(\\frac{3}{2}-\\frac{2}{3}\\right)=6 \\div \\frac{3}{2}-6 \\div \\frac{2}{3}=$ $9-6=3$. 其中正确的有", "options": "A. 0 道\nB. 1 道\nC. 2 道\nD. 3 道", "subject": "算术", "analysis": "$\\frac{1}{2}-\\frac{1}{2} \\times 2=\\frac{1}{2}-1=-\\frac{1}{2}$, 故(1)计算错误; $-2^{3}-(-3)^{2}=-8-9=-17$, 故(2)计算正确;\n\n$6 \\div\\left(\\frac{3}{2}-\\frac{2}{3}\\right)=6 \\div \\frac{5}{6}=6 \\times \\frac{6}{5}=\\frac{36}{5}$, 故(3)计算错误;\n\n综上, 计算正确的有 1 道.\n\n故答案为: B"} {"id": "1333", "image": [], "answer": "C", "solution": "null", "level": "七年级", "question": "已知两个数的和为负数, 则这两个有理数 ( )", "options": "A. 都为负数\nB. 都为正数\nC. 至少有一个为负数\nD. 必须一正一负", "subject": "算术", "analysis": "两个数的和为负数, 这两个有理数可以都是负数或者有一个是负数且负数的绝对值比另一个数的绝对值大.\n\n故答案为: $\\mathrm{C}$"} {"id": "1334", "image": [], "answer": "C", "solution": "null", "level": "七年级", "question": "预防和控制新冠肺炎最有效的办法就是接种疫苗.截止 2021 年 12 月 1 日, 某市累计接种新冠病毒疫苗超过 350 万剂次,用科学记数法表示 350 万为 $(\\quad)$", "options": "A. $35 \\times 10^{5}$\nB. $3.5 \\times 10^{5}$\nC. $3.5 \\times 10^{6}$\nD. $3.5 \\times 10^{7}$", "subject": "算术", "analysis": "350 万 $=3500000=3.5 \\times 10^{6}$.\n\n故答案为: C."} {"id": "1335", "image": [], "answer": "D", "solution": "null", "level": "七年级", "question": "计算 $2+2 \\stackrel{m \\uparrow}{\\stackrel{m \\uparrow}{n}} 2+2+3 \\times 3 \\widetilde{\\times} \\cdots \\times 3=(\\quad)$", "options": "$2^{m}+3 n$\nB. $m^{2}+3 n$\nC. $2 m+n^{3}$\nD. $2 m+3^{n}$", "subject": "算术", "analysis": "原式 $=2 m+3^{n}$\n\n故答案为: D."} {"id": "1336", "image": [], "answer": "C", "solution": "null", "level": "七年级", "question": "下列说法正确的是", "options": "非零两数的和一定大于任何一个加数\n\nB. 非零两数的差一定小于被减数\n\nC. 大于 1 的两数之积一定大于任何一个因数\n\nD. 小于 1 的两数之商一定小于被除数", "subject": "算术", "analysis": "$\\because$ 两个负数的和小于任何一个加数,\n\n$\\therefore$ 选项 $A$ 不符合题意;\n\n$\\because$ 当减数是小于或等于 0 的数时, 差是大于或等于被减数的,\n\n$\\therefore$ 选项 $B$ 不符合题意;\n\n$\\because$ 大于 1 的两数之积一定大于任何一个因数,\n\n$\\therefore$ 选项 $C$ 符合题意;\n\n$\\because$ 当除数是真分数, 被除数是正数时, 商大于被除数,\n$\\therefore$ 选项 $D$ 不符合题意.\n\n故答案为: C."} {"id": "1337", "image": [], "answer": "B", "solution": "null", "level": "七年级", "question": "现定义两种运算“ $\\oplus$ ”, “*”.对于任意两个整数, $\\quad a \\oplus b=a+b-1, \\quad a * b=a \\times b-1$,则 $(6 \\oplus 8) *(3 \\oplus 5)$ 的结果是 $(\\quad)$", "options": "A. 69\nB. 90\nC. 100\nD. 112", "subject": "算术", "analysis": "由题意知, $(6 \\oplus 8) *(3 \\oplus 5)=(6+8-1) *(3+5-1)=13 * 7=13 \\times 7-1=90$.\n\n故答案为: $\\mathrm{B}$"} {"id": "1338", "image": [], "answer": "A", "solution": "null", "level": "七年级", "question": "观察下列各式: $-\\frac{1}{1 \\times 2}=-1+\\frac{1}{2},-\\frac{1}{2 \\times 3}=-\\frac{1}{2}+\\frac{1}{3} ,-\\frac{1}{3 \\times 4}=-\\frac{1}{3}+\\frac{1}{4},-\\frac{1}{4 \\times 5}=-\\frac{1}{4}+\\frac{1}{5}$, 按照上面的规律, 计算式子 $-\\frac{1}{1 \\times 2}-\\frac{1}{2 \\times 3}-\\frac{1}{3 \\times 4}-\\cdots-\\frac{1}{2020 \\times 2021}$ 的值为 $(\\quad)$", "options": "A. $-\\frac{2020}{2021}$\nB. $\\frac{2020}{2021}$\nC. 2020\nD. 2021", "subject": "算术", "analysis": "原式 $=-\\frac{1}{1 \\times 2}+\\left(-\\frac{1}{2 \\times 3}\\right)+\\left(-\\frac{1}{3 \\times 4}\\right)+\\cdots+\\left(-\\frac{1}{2020 \\times 2021}\\right)$,\n\n$=-1+\\frac{1}{2}+\\left(-\\frac{1}{2}+\\frac{1}{3}\\right)+\\left(-\\frac{1}{3}+\\frac{1}{4}\\right)+\\cdots+\\left(-\\frac{1}{2020}+\\frac{1}{2021}\\right)$,\n\n$=-1+\\frac{1}{2}-\\frac{1}{2}+\\frac{1}{3}-\\frac{1}{3}+\\frac{1}{4}-\\cdots-\\frac{1}{2020}+\\frac{1}{2021}$\n\n$=-1+\\frac{1}{2021}$,\n\n$=-\\frac{2020}{2021}$\n\n故答案为: A."} {"id": "1340", "image": [], "answer": "B", "solution": "null", "level": "七年级", "question": "2022 年 2 月 5 日, 杭州某区最商气湿 $7^{\\circ} \\mathrm{C}$, 最低气温为 $-2^{\\circ} \\mathrm{C}$, 那么这天的最高气温比最低气温高", "options": "A. $5^{\\circ} \\mathrm{C}$\nB. $9^{\\circ} \\mathrm{C}$\nC. $-5^{\\circ} \\mathrm{C}$\nD. $-9^{\\circ} \\mathrm{C}$", "subject": "算术", "analysis": "由题意得\n\n$7-(-2)-9$.\n\n故答案为: B."} {"id": "1341", "image": [], "answer": "C", "solution": "null", "level": "七年级", "question": "如果有 4 个不同的正整数 $a 、 b 、 c 、 d$ 满足 $(2019-a)(2019-b)(2019-c)(2019-$ $d)=9$, 那么 $a+b+c+d$ 的值为()", "options": "0\nB. 9\nC. 8076\nD. 8090", "subject": "算术", "analysis": "because$ 有 4 个不同的正整数 $\\mathrm{a} 、 \\mathrm{~b} 、 \\mathrm{c} 、 \\mathrm{~d}$ 满足 $(2019-a)(2019-b)(2019-c)(2019-d)=$ 9 ,\n\n$\\therefore$ 四个括号内的值分别是: $\\pm 1, \\pm 3$,\n\n不妨设: $2019-a=1,2019-b=-1 , 2019-c=3 , 2019-d=-3 ,$\n\n解得: $a=2018 , b=2020 , c=2016, \\quad d=2022 ,$\n\n$\\therefore a+b+c+d=2018+2020+2016+2022=8076$\n\n故答案为: C."} {"id": "1351", "image": [], "answer": "B", "solution": "null", "level": "七年级", "question": "下列计算结果是负数的是 $(\\quad)$", "options": "A. $(-1) \\times(-2) \\times(-3) \\times 0$\nB. $5 \\times(-0.5) \\div(-1.84)^{2}$\nC. $(-5)^{2}+(-6)^{2}+(-7)^{2}$\nD. $(-1.2) \\times|-3.75| \\times(-0.125)$", "subject": "算术", "analysis": "A、 $(-1) \\times(-2) \\times(-3) \\times 0=0$, 故 A 不符合题意;\n\nB、 $5 \\times(-0.5) \\div(-1.84)^{2}=5 \\times(-0.5) \\div 1.84^{2}<0$, 故 B 符合题意;\n\nC、 $(-5)^{2}+(-6)^{2}+(-7)^{2}=5^{2}+6^{2}+7^{2}>0$, 故 C 不符合题意;\n\nD、 $-1.2 \\times 3.75 \\times(-0.125)=1.2 \\times 3.75 \\times 0.125>0$, 故 D 不符合题意;\n\n故答案为: $\\mathrm{B}$."} {"id": "1356", "image": [], "answer": "C", "solution": "null", "level": "七年级", "question": "两个有理数的和为 0 , 则这两个数", "options": "A. 都是 0\nB. 至少有一个为 0\nC. 互为相反数\nD. 一正一负", "subject": "算术", "analysis": "这两个数的和为零, 必定互为相反数.\n\n故答案为: C."} {"id": "1357", "image": [], "answer": "C", "solution": "null", "level": "七年级", "question": "2021 年是伟大的中国共产党百年华诞, 从南陈北李相约建党历经百年沧桑发展到今天已有近 9200 万党员, 其中 9200 万用科学记数法表示为", "options": "$9.2 \\times 10^{3}$\nB. $92 \\times 10^{6}$\nC. $9.2 \\times 10^{7}$\nD. $0.92 \\times 10^{8}$", "subject": "算术", "analysis": "9200 万 $=92000000=9.2 \\times 10^{7}$.\n\n故答案为: C."} {"id": "1360", "image": [], "answer": "C", "solution": "null", "level": "七年级", "question": "若规定一种运算“※”: $a ※ b=a^{b}+a b$, 则 $(-1) ※ 4=$", "options": "A. 0\nB. -8\nC. -3\nD. 3", "subject": "算术", "analysis": "$\\because a ※ b=a^{b}+a b$ ,\n\n$\\therefore(-1) ※ 4=(-1)^{4}+(-1) \\times 4=-3$.\n\n故答案为: C."} {"id": "1361", "image": [], "answer": "A", "solution": "null", "level": "七年级", "question": "设 $n$ 是自然数, 则 $\\frac{(-1)^{n}+(-1)^{n+1}}{2}$ 的值为 $(\\quad)$", "options": "A. 0\nB. 1\nC. -1\nD. 1 或 -1", "subject": "算术", "analysis": "当 $n$ 为偶数时, $\\frac{(-1)^{n}+(-1)^{n+1}}{2}=\\frac{1+(-1)}{2}=0$ ,\n\n当 $n$ 为偶数时, $\\frac{(-1)^{n}+(-1)^{n+1}}{2}=\\frac{(-1)+1}{2}=0$,\n\n故答案为: A."} {"id": "1362", "image": [], "answer": "B", "solution": "null", "level": "七年级", "question": "2021 减去它的 $\\frac{1}{2}$, 再减去余下的 $\\frac{1}{3}$, 再减去余下的 $\\frac{1}{4}, \\ldots$, 以此类推, 一直减到余下的 $\\frac{1}{2021}$, 则最后剩下的数是 $(\\quad)$", "options": "0\nB. 1\nC. $\\frac{2020}{2021}$\nD. $\\frac{2021}{2020}$", "subject": "算术", "analysis": "根据题意得 $2021 \\times\\left(1-\\frac{1}{2}\\right) \\times\\left(1-\\frac{1}{3}\\right) \\times\\left(1-\\frac{1}{4}\\right) \\times \\cdots \\times\\left(1-\\frac{1}{2021}\\right)$\n\n$=2021 \\times \\frac{1}{2} \\times \\frac{2}{3} \\times \\frac{3}{4} \\times \\cdots \\times \\frac{2020}{2021}=1$.\n\n故答案为: $\\mathrm{B}$."} {"id": "1363", "image": [], "answer": "B", "solution": "null", "level": "七年级", "question": "下列计算结果是负数的是", "options": "A. $3-2$\nB. $3 \\times(-2)$\nC. $3^{-2}$\nD. $\\sqrt{3}$", "subject": "算术", "analysis": "$\\because 3-2=1$, 计算结果是正数,\n\n$\\therefore$ 选项 A 不正确;\n\n$\\because 3 \\times(-2)=-6$, 计算结果是负数,\n\n$\\therefore$ 选项 B 正确;\n\n$\\because 3^{-2}=\\frac{1}{9}$, 计算结果是正数,\n\n$\\therefore$ 选项 $\\mathrm{C}$ 不正确;\n\n$\\because \\sqrt{3}$ 是一个正数,\n\n$\\therefore$ 选项 D 不正确.\n\n故选: B."} {"id": "1374", "image": [], "answer": "D", "solution": "null", "level": "七年级", "question": "定义运算 $\\mathrm{a} \\otimes \\mathrm{b}=\\mathrm{a}(\\mathrm{b}-1)$, 下面给出了关于这种运算的四个结论: (1) $2 \\otimes(-1)=-4$; (2) $\\mathrm{a} \\otimes \\mathrm{b}=\\mathrm{b} \\otimes \\mathrm{a}$; (3)若 $a+b=1$, 则 $a \\otimes a=b \\otimes b$; (4)若 $b \\otimes a=0$, 则 $a=0$ 或 $b=1$. 其中正确结论的序号是", "options": "(2)(4)\nB. (2) (3)\nC. (1) (4)\nD. (1)(3)", "subject": "算术", "analysis": "(1)根据题意得: 原式 $=2 \\times(-1-1)=2 \\times(-2)=-4$, 正确;\n\n(2) 根据题意得: $\\mathrm{a} \\otimes \\mathrm{b}=\\mathrm{a}(\\mathrm{b}-1), \\mathrm{b} \\otimes \\mathrm{a}=\\mathrm{b}(\\mathrm{a}-1)$, 不相等, 错误;\n\n(3) 由 $a+b=1$, 得到 $b=1-a, a=1-b$,\n\n则 $\\mathrm{a}^{\\otimes} \\mathrm{a}=\\mathrm{a}(\\mathrm{a}-1)=-\\mathrm{ab}, \\mathrm{b} \\otimes \\mathrm{b}=\\mathrm{b}(\\mathrm{b}-1)=-\\mathrm{ab}$, 即 $\\mathrm{a} \\otimes \\mathrm{a}=\\mathrm{b} \\otimes \\mathrm{b}$, 正确;\n\n(4) $b \\otimes a=b(a-1)=0$, 得到 $b=0$ 或 $a=1$, 错误,\n\n则正确结论的序号是 (1) (3),\n\n故选 D."} {"id": "1380", "image": [], "answer": "A", "solution": "null", "level": "七年级", "question": "下列不是无理数的是 $(\\quad)$", "options": "A. $\\sqrt{4}$\nB. $3.141141114 \\ldots$\nC. $\\sqrt{3}$\nD. $\\pi$", "subject": "算术", "analysis": "A、 $\\sqrt{4}$ 是有理数, 故 A 错误;\n\nB、3.141141114...是无理数,故 B 正确;\n\nC、 $\\sqrt{3}$ 是无理数,故 C 正确;\n\nD、 $\\pi$ 是无理数, 故 D 正确;\n\n故选: A."} {"id": "1381", "image": [], "answer": "D", "solution": "null", "level": "七年级", "question": "在算式 $(-\\sqrt{0.3})$ 口 $(-\\sqrt{0.3})$ 叶填上运算符号, 使结果最大, 这个运算符号是 ( )", "options": "加号\nB. 减号\nC. 乘号\nD. 除号", "subject": "算术", "analysis": "$(-\\sqrt{0.3})+(-\\sqrt{0.3})=-\\frac{\\sqrt{30}}{5} ;(-\\sqrt{0.3})-(-\\sqrt{0.3})=0 ;(-\\sqrt{0.3}) \\times(-\\sqrt{0.3})=0.3 ;(-$ $\\sqrt{0.3}) \\div(-\\sqrt{0.3})=1$\n\n则使结果最大,这个运算符号为除号.\n\n故选 D."} {"id": "1383", "image": [], "answer": "D", "solution": "null", "level": "七年级", "question": "若 $|a|=4, \\sqrt{b^{2}}=3$, 且 $\\mathrm{a}+\\mathrm{b}<0$, 则 $\\mathrm{a}-\\mathrm{b}$ 的值是()", "options": "1,7\nB. $-1,7$\nC. $1,-7$\nD. $-1,-7$", "subject": "算术", "analysis": "$\\because|\\mathrm{a}|=4, \\sqrt{b^{2}}=3$, 且 $\\mathrm{a}+\\mathrm{b}<0$,\n\n$\\therefore \\mathrm{a}=-4, \\mathrm{a}=-3 ; \\mathrm{a}=-4, \\mathrm{~b}=3$,\n\n则 $\\mathrm{a}-\\mathrm{b}=-1$ 或 -7 .\n\n故选 D"} {"id": "1384", "image": ["3119.jpg"], "answer": "B", "solution": "null", "level": "七年级", "question": "实数 $\\mathrm{a}$ 在数轴上的位置如图所示, 则下列说法不正确的是 ( )\n\n", "options": "A. $\\mathrm{a}$ 的相反数大于 2\nB. $a$ 的相反数是 2\nC. $|a|>2$\nD. $2 \\mathrm{a}<0$", "subject": "算术", "analysis": "由数轴可知, $a<-2$,\n\nA、a 的相反数 $>2$, 故本选项正确, 不符合题意;\n\nB、a 的相反数 $\\neq 2$, 故本选项错误,符合题意;\n\nC、a 的绝对值 $>2$, 故本选项正确, 不符合题意;\n\nD、 $2 \\mathrm{a}<0$, 故本选项正确, 不符合题意.\n\n故选: B"} {"id": "1385", "image": [], "answer": "C", "solution": "null", "level": "七年级", "question": "一个正偶数的算术平方根是 a, 那么与这个正偶数相邻的下一个正偶数的算术平方根是 ( )", "options": "$a+2$\nB. $a^{2}+2$\nC. $\\sqrt{a^{2}+2}$\nD. $\\pm \\sqrt{a+2}$", "subject": "算术", "analysis": "一个正偶数的算术平方根是 $a$, 得到这个正偶数为 $a^{2}$,所以与这个正偶数相邻的下一个正偶数为 $\\mathrm{a}^{2}+2$,从而与这个正偶数相邻的下一个正偶数的算术平方根是 $\\sqrt{a^{2}+2}$故选 C."} {"id": "1386", "image": [], "answer": "A", "solution": "null", "level": "七年级", "question": "已知 $01^{2}$, $\\sqrt{1.01}>1$, 故 D 正确,\n\n故选: D."} {"id": "1407", "image": [], "answer": "B", "solution": "null", "level": "七年级", "question": "当 $a^{2}=b^{2}$ 时, 下列等式中成立的是()", "options": "A. $a=b$\nB. $|a|=\\sqrt{b^{2}}$\nC. $a^{3}=b^{3}$\nD. $\\sqrt{a}=\\sqrt{b}$", "subject": "算术", "analysis": "$\\because \\mathrm{a}^{2}=\\mathrm{b}^{2}$,\n\n$\\therefore|\\mathbf{a}|=|\\mathbf{b}|$,\n\n$\\therefore|a|=\\sqrt{b^{2}}$.\n\n故选: B."} {"id": "1409", "image": [], "answer": "C", "solution": "null", "level": "七年级", "question": "-2 的立方与 -2 的平方的和是 ()", "options": "A. 0\nB. 4\nC. -4\nD. 0 或 -4", "subject": "算术", "analysis": "(分析】 -2 的立方是 $-8,-2$ 的平方是 4 , 求其和即可.\n\n【解答】 $(-2)^{3}+(-2)^{2}=-8+4=-4$.\n\n故选 C."} {"id": "1410", "image": [], "answer": "D", "solution": "null", "level": "七年级", "question": "已知数 $\\mathrm{a}$ 在数轴上的位置如图所示, 则 $\\mathrm{a} 、-\\mathrm{a} 、 \\frac{1}{a} 、-\\frac{1}{a}$ 大小关系正确的是\n\n| -1 | $a$ | 0 | 1 |\n| :--- | :--- | :--- | :--- |", "options": "A. $-\\frac{1}{a}<-a<\\frac{1}{a}1$,\n\n$\\therefore \\frac{1}{a}<\\mathrm{a}<-\\mathrm{a}<-\\frac{1}{a}$.\n\n故答案为: D."} {"id": "1411", "image": [], "answer": "B", "solution": "null", "level": "七年级", "question": "下列运算正确的是 $(\\quad)$", "options": "A. $\\sqrt{4}= \\pm 2$\nB. $-\\sqrt{4}=-2$\nC. $(-2)^{3}=-6$\nD. $-2^{2}=4$", "subject": "算术", "analysis": "A、 $\\sqrt{4}=2$, 故 A 不符合题意;\n\nB、 $-\\sqrt{4}=-2$, 故 B 符合题意;\n\nC、 $(-2)^{3}=-8$, 故 C 不符合题意;\n\nD、 $-2^{2}=-4$ , 故 D 不符合题意;\n\n故答案为: $B$."} {"id": "1412", "image": [], "answer": "C", "solution": "null", "level": "七年级", "question": "已知 $|x|=6, y^{2}=4$, 且 $x y<0$. 则 $x+y$ 的值为 ( )", "options": "4\nB. -4\nC. 4 或 -4\nD. 2 或 -2", "subject": "算术", "analysis": "$|x|=6, y^{2}=4$,\n\n所以 $x= \\pm 6, y= \\pm 2$,\n\n因为 $x y<0$ ,\n\n所以 $x=6, y=-2$ 或 $x=-6, y=2$,\n\n当 $x=6, y=-2$ 时, $x+y=6+(-2)=4$;\n\n当 $x=-6, y=2$ 时, $x+y=-6+2=-4$;\n\n故答案为: $\\mathrm{C}$."} {"id": "1422", "image": [], "answer": "B", "solution": "null", "level": "七年级", "question": "在 $\\frac{22}{7}, 3.14, \\sqrt{16}, 0.1010010001, \\pi-1, \\sqrt[3]{125}$ 中, 无理数的个数有()", "options": "A. 1 个\nB. 2 个\nC. 3 个\nD. 4 个", "subject": "算术", "analysis": "$\\because \\sqrt{16}=4, \\sqrt[3]{125}=5$\n\n无理数有: $\\pi-1$.\n\n故答案为: A."} {"id": "1428", "image": [], "answer": "C", "solution": "null", "level": "七年级", "question": "$\\sqrt{169}$ 的算术平方根为 $(\\quad)$", "options": "13\nB. $\\pm 13$\nC. $\\sqrt{13}$\nD. $\\pm \\sqrt{13}$", "subject": "算术", "analysis": "$\\because \\sqrt{169}=13$,\n\n$\\therefore \\sqrt{169}$ 的算术平方根即为 13 的算术平方根,\n\n结果为 $\\sqrt{13}$,\n\n故答案为: $\\mathrm{C}$."} {"id": "1429", "image": [], "answer": "C", "solution": "null", "level": "七年级", "question": "下列说法正确的是 ( )", "options": "一个数的立方根有两个, 它们互为相反数\n\nB. 负数没有立方根\n\nC. 任何数的立方根都只有一个\n\nD. 如果一个数有立方根, 那么这个数也一定有平方根", "subject": "算术", "analysis": "$\\because$ 一个正数有一个正的立方根, 一个负数有一个负的立方根, 0 的立方根是 0 , 即任何数都\n只有一个立方根,\n\n$\\therefore \\mathrm{A}$ 选项说法不正确;\n\n$\\because$ 一个负数有一个负的立方根,\n\n$\\therefore \\mathrm{B}$ 选项说法不正确;\n\n$\\because$ 一个正数有一个正的立方根, 一个负数有一个负的立方根, 0 的立方根是 0 ,\n\n$\\therefore \\mathrm{C}$ 选项说法正确;\n\n$\\because$ 一个负数有一个负的立方根, 但负数没有平方根,\n\n$\\therefore \\mathrm{D}$ 选项说法不正确.\n\n综上, 说法正确的是 $\\mathrm{C}$ 选项.\n\n故答案为: C."} {"id": "1430", "image": [], "answer": "C", "solution": "null", "level": "七年级", "question": "求 $\\frac{49}{81}$ 的平方根的数学表达式为 $(\\quad)$", "options": "$\\sqrt{\\frac{49}{81}}= \\pm \\frac{7}{9}$\nB. $\\sqrt{\\frac{49}{81}}=-\\frac{7}{9}$\nC. $\\pm \\sqrt{\\frac{49}{81}}= \\pm \\frac{7}{9}$\nD. $\\sqrt{\\frac{49}{81}}=\\frac{7}{9}$", "subject": "算术", "analysis": "求 $\\frac{49}{81}$ 的平方根的数学表达式为 $\\pm \\sqrt{\\frac{49}{81}}= \\pm \\frac{7}{9}$.故答案为: C."} {"id": "1431", "image": [], "answer": "D", "solution": "null", "level": "七年级", "question": "以下各数没有平方根的是 $(\\quad)$", "options": "A. 64\nB. $(-2)^{2}$\nC. $|-3|$\nD. $-2^{-2}$", "subject": "算术", "analysis": "A、 $\\because 64>0$,\n\n$\\therefore 64$ 有平方根, 故 A 不符合题意;\n\nB、 $\\because(-2)^{2}=4>0$,\n\n$\\therefore 4$ 有平方根, 故 B 不符合题意;\n\nC、 $\\because|-3|=3>0$\n\n$\\therefore|-3|$ 有平方根, 故 $\\mathrm{C}$ 不符合题意;\n\nD、 $\\because-2^{-2}=-\\frac{1}{4}<0$\n\n$\\therefore-2^{-2}$ 没有平方根, 故 D 符合题意;故答案为: D."} {"id": "1432", "image": [], "answer": "B", "solution": "null", "level": "七年级", "question": "下列说法正确的是", "options": "倒数等于它本身的数只有 1\nB. 正数的绝对值是它本身\nC. 平方等于它本身的数只有 1\nD. 立方等于它本身的数只有 1", "subject": "算术", "analysis": "A、倒数等于它本身的数有 $\\pm 1$, 此项说法不符合题意;\n\nB、正数的绝对值是它本身, 此项说法符合题意;\n\nC、平方等于它本身的数有 0 和 1 , 此项说法不符合题意;\n\nD、立方等于它本身的数有 0 和 $\\pm 1$, 此项说法不符合题意;\n\n故答案为: $\\mathrm{B}$\nB."} {"id": "1435", "image": [], "answer": "A", "solution": "null", "level": "七年级", "question": "$\\sqrt{81}$ 的算术平方根是 $(\\quad)$", "options": "A. 3\nB. $\\pm 3$\nC. $\\pm 9$\nD. 9", "subject": "算术", "analysis": "$\\because \\sqrt{81}=9$,\n\n$\\therefore \\sqrt{81}$ 的算术平方根是 $\\sqrt{9}=3$,\n\n故答案为: A."} {"id": "1446", "image": [], "answer": "A", "solution": "null", "level": "七年级", "question": "下列各组数中, 不相等的一组是 ( ).", "options": "A. $(-2)^{3}$ 和 $-2^{3}$\nB. $\\sqrt{4}$ 和 $\\sqrt[3]{8}$\nC. $(-2)^{4}$ 和 $-2^{4}$\nD. $\\left|-2^{3}\\right|$ 和 $|-2|^{3}$", "subject": "算术", "analysis": "A、 $\\because(-2)^{3}=-8,-2^{3}=-8$,\n\n$\\therefore(-2)^{3}=-2^{3}$, 故 A 不符合题意;\n\nB、 $\\because \\sqrt{4}=2, \\sqrt[3]{8}=2$,\n\n$\\therefore \\sqrt{4}=\\sqrt[3]{8}$, 故 $\\mathrm{B}$ 不符合题意;\n\nC、 $\\because(-2)^{4}=14,-2^{4}=-16$\n\n$\\therefore(-2)^{4} \\neq-2^{4}$, 故 $\\mathrm{C}$ 符合题意;\n\nD、 $\\because\\left|-2^{3}\\right|=8,|-2|^{3}=8$\n\n$\\therefore\\left|-2^{3}\\right|=|-2|^{3}, \\quad$ 故 D 不符合题意;\n\n故答案为: C."} {"id": "1452", "image": [], "answer": "D", "solution": "null", "level": "七年级", "question": "下列计算正确的是 $(\\quad)$", "options": "A. $\\sqrt{\\frac{4}{25}}= \\pm \\frac{2}{5}$\nB. $\\sqrt{ \\pm \\frac{4}{25}}=\\frac{2}{5}$\nC. $\\pm \\sqrt{\\frac{4}{25}}=\\frac{2}{5}$\nD. $\\pm \\sqrt{\\frac{4}{25}}= \\pm \\frac{2}{5}$", "subject": "算术", "analysis": "A、 $\\sqrt{\\frac{4}{25}}=\\frac{2}{5}$, 错误;\n\nB、 $\\because \\sqrt{-\\frac{4}{25}}$ 无意义,错误;\n\nCD、 $\\pm \\sqrt{\\frac{4}{25}}= \\pm \\frac{2}{5}$, 故 C 错误, D 正确;\n\n故答案为: D."} {"id": "1453", "image": [], "answer": "B", "solution": "null", "level": "七年级", "question": "下列判断: (1)一个数的平方根等于它本身, 这个数是 0 和 1 ; (2)实数包括无理数和有理数;\n\n(3) 2 的算术平方根是 $\\sqrt{2}$; (4)无理数是带根号的数. 正确的有 ( )", "options": "A. 1 个\nB. 2 个\nC. 3 个\nD. 4 个", "subject": "算术", "analysis": "(1)一个数的平方根等于它本身, 只有 0 , 该项不符合题意; (2) 实数包括无理数和有理数, 该项符合题意; (3) 2 的算术平方根是 $\\sqrt{2}$, 该项符合题意; (4)无理数是带根号的数, 例如: $\\sqrt{4}$ 不是无理数, 该项不符合题意.\n\n故答案为:\nB."} {"id": "1455", "image": [], "answer": "A", "solution": "null", "level": "七年级", "question": "一个正数的平方根是 $x-5$ 和 $x+1$, 则 $x$ 的值为 (", "options": "A. 2\nB. -2\nC. 0\nD. 无法确定", "subject": "算术", "analysis": "$\\because$ 一个正数的平方根是 $\\mathrm{x}-5$ 和 $\\mathrm{x}+1$\n\n$\\therefore \\mathrm{x}-5+\\mathrm{x}+1=0$\n\n解之: $x=2$.\n\n故答案为: A."} {"id": "1456", "image": [], "answer": "D", "solution": "null", "level": "七年级", "question": "下列说法正确的是 ( )", "options": "$\\frac{\\sqrt{2}}{2}$ 是分数\nB. 16 的平方根是 $\\pm 4$, 即 $\\sqrt{16}= \\pm 4$\nC. 8.30 万精确到百分位\nD. 若 $\\sqrt{a-2022}+|b+1|=0$, 则 $b^{a}=1$", "subject": "算术", "analysis": "$\\mathrm{A}$ 选项, $\\frac{\\sqrt{2}}{2}$ 是无理数, $\\mathrm{A}$ 选项不正确;\n\nB 选项, 16 的平方根是 $\\pm 4$, 即 $\\pm \\sqrt{16}= \\pm 4$, B 选项不正确;\n\n$\\mathrm{C}$ 选项, 8.30 万精确到百位, $\\mathrm{C}$ 选项不正确;\n\nD 选项, $\\because \\sqrt{a-2022}+|b+1|=0$\n\n$\\therefore \\mathrm{a}-2022=0, \\mathrm{~b}+1=0$\n\n$\\therefore \\mathrm{a}=2022, \\mathrm{~b}=-1$\n\n$\\therefore b^{a}=1$\n\nD 选项正确;\n\n故答案为: D."} {"id": "1458", "image": [], "answer": "B", "solution": "null", "level": "七年级", "question": "一个自然数的算术平方根是 $\\mathrm{a}$, 则它下一个自然数的算术平方根是 ( )", "options": "A. $\\sqrt{a}+1$\nB. $\\sqrt{a^{2}+1}$\nC. $a+1$\nD. $\\sqrt{a+1}$", "subject": "算术", "analysis": "$\\because$ 一个自然数的算术平方根是 a,\n\n$\\therefore$ 这个自然数为 $\\mathrm{a}^{2}$,\n\n$\\therefore$ 它下一个自然数的算术平方根是 $\\sqrt{a^{2}+1}$.\n\n故答案为:\nB."} {"id": "1481", "image": [], "answer": "A", "solution": "null", "level": "七年级", "question": "某商店在甲批发市场以每包 $\\mathrm{a}$ 元的价格进了 50 包茶叶, 又在乙批发市场以每包 $\\mathrm{b}$ 元 $(\\mathrm{a}>\\mathrm{b})$ 的价格进了同样的 70 包茶叶, 如果以每包 $\\frac{a+b}{2}$ 元价格全部卖出这种茶叶, 那么这家商店()", "options": "A. 盈利了\nB. 亏损了\nC. 不盈不亏\nD. 盈亏不能确定", "subject": "算术", "analysis": "$\\because \\mathrm{a}>\\mathrm{b}$,\n\n$\\therefore(50+70) \\times \\frac{a+b}{2}-(50 a+70 b)$\n\n$=60 a+60 b-50 a-70 b$\n\n$=10 \\mathrm{a}-10 \\mathrm{~b}$\n\n$=10(a-b)>0$,\n\n$\\therefore$ 这家商店盈利了,\n故答案为: A."} {"id": "1483", "image": [], "answer": "C", "solution": "null", "level": "七年级", "question": "下列计算正确的是", "options": "A. $2 m^{2} n^{2}+4 m^{2} n^{2}=6 m^{4} n^{4}$\nB. $3 x+2 y=5 x y$\nC. $-\\frac{4}{3} \\times 2 \\pi x^{2}=-\\frac{8}{3} \\pi x^{2}$\nD. $7 x^{3}-4 x^{3}=3$", "subject": "算术", "analysis": "$\\mathrm{A}$ 选项, 原式 $=6 \\mathrm{~m}^{2} \\mathrm{n}^{2}$, 故该选项不符合题意;\n\n$\\mathrm{B}$ 选项, $3 \\mathrm{x}$ 与 $2 \\mathrm{y}$ 不是同类项, 不能合并, 故该选项不符合题意;\n\n$\\mathrm{C}$ 选项, 原式 $=-\\frac{8}{3} \\pi \\mathrm{x}^{2}$, 故该选项符合题意;\n\n$\\mathrm{D}$ 选项, 原式 $=3 \\mathrm{x}^{3}$, 故该选项不符合题意;\n\n故答案为: C."} {"id": "1494", "image": [], "answer": "B", "solution": "null", "level": "七年级", "question": "已知 $2 a^{n} b^{n}$ 与 $3 a^{3} b^{m+2}$ 是同类项, 则 $m+n=$", "options": "A. 3\nB. 4\nC. -4\nD. -3", "subject": "算术", "analysis": "$\\because 2 a^{n} b^{n}$ 与 $3 a^{3} b^{m+2}$ 是同类项,\n\n$\\therefore \\mathrm{m}+2=\\mathrm{n}, \\mathrm{n}=3$,\n\n解得: $m=1$,\n\n则 $m+n=1+3=4$.\n\n故答案为: B."} {"id": "1500", "image": [], "answer": "C", "solution": "null", "level": "七年级", "question": "下列表述正确的是", "options": "A. 单项式 $a b$ 的系数是 0 , 次数是 2\nB. $-2 x^{2} y^{3}$ 的系数是 -2 , 次数是 3\nC. $x-1$ 是一次二项式\nD. $-a b^{2}+3 a-1$ 的项是 $-a b^{2}, 3 \\mathrm{a}, 1$", "subject": "算术", "analysis": "A、单项式 ab 的系数是 1 , 次数是 2 , 故此选项不合题意;\n\nB、 $-2 x^{2} y^{3}$ 的系数是 -2 , 次数是 5, 故此选项不合题意;\n\nC、 $x-1$ 是一次二项式, 故此选项符合题意;\n\nD、 $-a b^{2}+3 a-1$ 的项是 $-a b^{2}, 3 a,-1$, 故此选项不合题意.\n\n故答案为: $\\mathrm{C}$"} {"id": "1501", "image": [], "answer": "B", "solution": "null", "level": "七年级", "question": "如果式子 $-2 m+3 n+6$ 的值为 16 , 那么式子 $-9 n+6 m+2$ 的值等于 ( )", "options": "A. -32\nB. -28\nC. 32\nD. 28", "subject": "算术", "analysis": "$\\because-2 m+3 n+6=16$,\n\n$$\n\\begin{gathered}\n\\therefore 2 m-3 n=-10, \\\\\n\\therefore-9 n+6 m+2=3(2 m-3 n)+2 \\\\\n=3 \\times(-10)+2=-30+2=-28 .\n\\end{gathered}\n$$\n\n故答案为: B"} {"id": "1503", "image": [], "answer": "C", "solution": "null", "level": "七年级", "question": "观察下列三行数:\n\n第一行: $2 、 4 、 6 、 8 、 10 、 12 \\ldots \\ldots$\n\n第二行:3、5、7、9、11、13....\n\n第三行:1、4、9、16、25、36....\n\n设 $\\mathrm{x} 、 \\mathrm{y} 、 \\mathrm{z}$ 分别为第一、第二、第三行的第 100 个数,则 $2 x-y+2 z$ 的值为 ( )", "options": "9999\nB. 10001\nC. 20199\nD. 20001", "subject": "算术", "analysis": "观察第 (1)行:2、4、6、8、10、12、...\n\n$\\therefore$ 第 100 个数为 $100 \\times 2=200$,\n\n观察第(2) 行: 3、5、7、9、11、13、...\n$\\therefore$ 第 100 个数为 $100 \\times 2+1=201$,\n\n观察第(3)行: $1 、 4 、 9 、 16 、 25 、 36 、 \\ldots$\n\n$\\therefore$ 第 100 个数是 $100^{2}=10000$,\n\n即 $x=200 、 y=201 、 z=10000$,\n\n$\\therefore 2 \\mathrm{x}-\\mathrm{y}+2 \\mathrm{z}=20199$,\n\n故答案为: C."} {"id": "1505", "image": [], "answer": "A", "solution": "null", "level": "七年级", "question": "为解决老百姓看病难问题.决定下调药品价格, 某种药品在第一年涨价 $40 \\%$ 后, 第二年在第一年的基础上降价 $70 \\%$ 调至 a 元, 则这种药品在第一年涨价前的价格为()", "options": "$\\frac{100}{42} a$ 元\nB. $\\frac{42}{100} a$ 元\nC. $(1-30 \\%) a$ 元\nD. $\\frac{a}{1-30 \\%}$ 元", "subject": "算术", "analysis": "由题意可得,\n\n这种药品在第一年涨价前的价格为: $a \\div(1-70 \\%) \\div(1+40 \\%)=a \\div 30 \\% \\div 140 \\%=\\frac{100}{42} a($ 元 )\n\n故答案为: A."} {"id": "1524", "image": [], "answer": "B", "solution": "null", "level": "七年级", "question": "为调研大众的低碳环保意识, 小明在某超市出口统计后发现: 一小时内使用自带环保袋的人数比使用超市塑料袋人数的 2 倍少 4 人, 若使用超市塑料袋的为 $\\mathrm{x}$ 人, 则使用自带环保袋的人数为", "options": "A. $2 x+4$\nB. $2 x-4$\nC. $4 x+2$\nD. $4 x-2$", "subject": "算术", "analysis": "由题意, 使用超市塑料袋的为 $\\mathrm{x}$ 人, 则使用自带环保袋的人数为 $2 x-4$,故答案为: $\\mathrm{B}$."} {"id": "1526", "image": [], "answer": "C", "solution": "null", "level": "七年级", "question": "商品的原售价为 $m$ 元,若按该价的 8 折出售,仍获利 $\\mathrm{n} \\%$, 则该商品的进价为 ( ) 元", "options": "$0.8 \\mathrm{~m} \\times \\mathrm{n} \\%$\nB. $0.8 \\mathrm{~m}(1+\\mathrm{n} \\%)$\nC. $\\frac{0.8 m}{1+n \\%}$\nD. $\\frac{0.8 m}{n \\%}$", "subject": "算术", "analysis": "由题意知:进价为 $m \\cdot 80 \\% \\div(1+n \\%)=\\frac{0.8 m}{1+n \\%}$,故答案为: C C."} {"id": "1531", "image": [], "answer": "D", "solution": "null", "level": "七年级", "question": "下列运算中, 正确的是 ( )", "options": "A. $3 a+b=3 a b$\nB. $3 a-a=3$\nC. $-5 a^{2}-3 a^{2}=-2 a^{2}$\nD. $-a^{2} b+2 a^{2} b=a^{2} b$", "subject": "算术", "analysis": "A、 $3 \\mathrm{a}$ 和 $\\mathrm{b}$ 含有不同字母, 不是同类项, 不能合并, 故计算不符合题意;\n\nB、3a-a 合并同类项后应为 $2 \\mathrm{a}$, 不是 3, 故计算不符合题意;\n\nC、 $-5 a^{2}-3 a^{2}$ 合并同类项后应为 $-8 a^{2}$, 不是 $-2 a^{2}$, 故计算不符合题意;\n\nD、 $-a^{2} b+2 a^{2} b=a^{2} b$ ,故计算符合题意.\n\n故答案为: D."} {"id": "1542", "image": [], "answer": "C", "solution": "null", "level": "七年级", "question": "下列说法错误的有\n\n(1) 0 是绝对值最小的数\n\n(2) $3 a-2$ 的相反数是 $-3 a-2$\n\n(3) $5 \\pi R^{2}$ 的系数是 5\n\n(4) 一个有理数不是整数就是分数\n\n(5) $3^{4} x^{3}$ 是 7 次单项式", "options": "A. 1 个\nB. 2 个\nC. 3 个\nD. 4 个", "subject": "算术", "analysis": "(1) 0 是绝对值最小的数, 故(1)符合题意;\n\n(2) $3 a-2$ 的相反数时 2-3a, 故 (2) 不符合题意;\n\n(3) $5 \\pi \\mathrm{R}^{2}$ 的系数是 $5 \\pi$, 故(3)不符合题意;\n\n(4)一个有理数不是整数就是分数, 故(4)符合题意;\n\n(5) $3^{4} x^{3}$ 是 3 次单项式, 故(5)不符合题意;\n\n综上, 错误的有(2)(3)(5)共 3 个,\n\n故答案为: $\\mathrm{C}$"} {"id": "1550", "image": ["3176.jpg"], "answer": "A", "solution": "null", "level": "七年级", "question": "有理数 $\\mathrm{a} 、 \\mathrm{~b} 、 \\mathrm{c}$ 在数轴上位置如图,则 $|c-a|-|a+b|-|b-c|$ 的值为()\n\n", "options": "A. $2 a+2 b-2 c$\nB. 0\nC. $-2 c$\nD. $2 a$", "subject": "算术", "analysis": "解:观察数轴得: $b|a|$,\n\n$\\therefore c-a<0, a+b<0, b-c<0$,\n\n$\\therefore|c-a|-|a+b|-|b-c|$\n\n$$\n\\begin{gathered}\n=-(c-a)-[-(a+b)]-(c-b) \\\\\n=-c+a+a+b-c+b\n\\end{gathered}\n$$\n\n$=2 a+2 b-2 c$.\n\n故答案为: A"} {"id": "1311", "image": [], "answer": "D", "solution": "null", "level": "七年级", "question": "已知 $\\mathrm{a}, \\mathrm{b}, \\mathrm{c}$ 是有理数, 且 $\\mathrm{a}+\\mathrm{b}+\\mathrm{c}=0$, $\\mathrm{abc}$ (乘积) 是负数, 则 $\\frac{b+c}{|a|}+\\frac{a+c}{|b|}+\\frac{a+b}{|c|}$ 的值是", "options": "3\nB. -3\nC. 1\nD. -1", "subject": "代数", "analysis": "由题意知, $\\mathrm{a}, \\mathrm{b}, \\mathrm{c}$ 中只能有一个负数, 另两个为正数, 不妨设 $\\mathrm{a}<0, \\mathrm{~b}>0, \\mathrm{c}>0$.由 $a+b+c=0$ 得出: $a+b=-c, b+c=-a, a+c=-b$,\n\n代入代数式, 原式 $=\\frac{-a}{|a|}+\\frac{-b}{|b|}+\\frac{-c}{|c|}=1-1-1=-1$,\n\n故答案为: D."} {"id": "1313", "image": [], "answer": "C", "solution": "null", "level": "七年级", "question": "已知 $\\mathrm{a}, \\mathrm{b}$ 为实数, 下列说法: (1)若 $\\mathrm{ab}<0$, 且 $\\mathrm{a}, \\mathrm{b}$ 互为相反数, 则 $\\frac{a}{b}=-1$; (2)若 $\\mathrm{a}+\\mathrm{b}<0$, $a b>0$, 则 $|2 a+3 b|=-2 a-3 b$; (3)若 $|a-b|+a-b=0$, 则 $b>a$; (4)若 $|a|>|b|$, 则(a+b) $\\times(a-b)$ 是正数; (5) 若 $a6$, 其中正确的说法有(() 个.", "options": "2\nB. 3\nC. 4\nD. 5", "subject": "代数", "analysis": "(1)若 $\\mathrm{ab}<0$, 且 $\\mathrm{a}, \\mathrm{b}$ 互为相反数, 则 $\\frac{a}{b}=-1$, 正确 ;\n\n(2) $\\because a+b<0, a b>0, \\quad \\therefore a<0, b<0, \\quad \\therefore 2 a+3 b<0, \\quad \\therefore|2 a+3 b|=-2 a-3 b$, 正确;\n\n(3) $\\because|\\mathrm{a}-\\mathrm{b}|+\\mathrm{a}-\\mathrm{b}=0, \\quad \\therefore|\\mathrm{a}-\\mathrm{b}|=\\mathrm{b}-\\mathrm{a} \\geq 0, \\quad \\therefore \\mathrm{b} \\geq \\mathrm{a}$, 错误;\n\n(4) 当 $a>0, \\quad b>0$ 时, 则 $a>b, \\quad \\therefore a-b>0, a+b>0, \\quad \\therefore(a+b)$. (a-b)为正数;\n\n当 $a>0, b<0$ 时, $a-b>0, a+b>0, \\quad \\therefore(a+b) .(a-b)$ 为正数;\n\n当 $a<0, b>0$ 时, $a-b<0, a+b<0, \\quad \\therefore(a+b)$. (a-b) 为正数;\n\n当 $a<0, b<0$ 时, $a-b<0, a+b<0, \\therefore(a+b) .(a-b)$ 为正数;\n\n故 (4) 正确;\n\n(5) $\\because a0, \\quad a<0$,\n\n当 $03$,\n\n$\\because|a-3|<|b-3|$\n\n$\\therefore 3-a6$, 正确.\n\n综上, 正确的有 4 项.\n\n故答案为: C."} {"id": "1339", "image": [], "answer": "C", "solution": "null", "level": "七年级", "question": "求 $1+3+3^{2}+3^{3}+\\cdots+3^{2019}$ 的值, 可令 $S=1+3+3^{2}+3^{3}+\\cdots+3^{2019}$ (1), (1)式两边都乘以 3 , 则 $3 S=3+3^{2}+3^{3}+3^{4}+\\cdots+3^{2020}$ (2), (2)-(1)得 $3 S-S=3^{2020}-1$, 则 $S=\\frac{3^{2020}-1}{2}$仿照以上推理, 计算出 $1+5+5^{2}+5^{3}+5^{4}+\\cdots+5^{2019}$ 的值为 $(\\quad)$", "options": "$5^{2019}-1$\nB. $5^{2020}-1$\nC. $\\frac{5^{2020}-1}{4}$\nD. $\\frac{5^{2010}-1}{4}$", "subject": "代数", "analysis": "令 $S=1+5+5^{2}+5^{3}+5^{4}+\\cdots+5^{2019}(1)$,\n\n(1) 式两边同时乘以 5 , 得 $5 S=5+5^{2}+5^{3}+5^{4}+5^{5}+\\cdots+5^{2020}$ (2),\n\n(2)-(1)得 $4 S=5^{2020}-1$, 即 $S=\\frac{5^{2020}-1}{4}$.\n\n故答案为: C."} {"id": "1358", "image": [], "answer": "B", "solution": "null", "level": "七年级", "question": "已知 $(a-1)^{2}+|b+2|=0$, 则 $(a+b)^{2022}$ 的值是 ( )", "options": "A. -1\nB. 1\nC. 2\nD. 3", "subject": "代数", "analysis": "$\\because(\\mathrm{a}-1)^{2}+|\\mathrm{b}+2|=0$,\n\n$\\therefore \\mathrm{a}-1=0, \\mathrm{~b}+2=0$,\n\n$\\therefore \\mathrm{a}=1, \\mathrm{~b}=-2$,\n\n$\\therefore(1-2)^{2022}=1$,\n\n故答案为: B"} {"id": "1382", "image": [], "answer": "C", "solution": "null", "level": "七年级", "question": "设 $6-\\sqrt{13}$ 的整数部分为 $a$, 小数部分为 $b$, 那么 $2 a-b$ 的值是", "options": "$3-\\sqrt{3}$\nB. $4-\\sqrt{13}$\nC. $\\sqrt{13}$\nD. $4+\\sqrt{13}$", "subject": "代数", "analysis": "$\\because 3<\\sqrt{13}<4$,\n\n$\\therefore-4<-\\sqrt{13}<-3$,\n\n$\\therefore 2<6-\\sqrt{13}<3$,\n\n$\\therefore \\mathrm{a}=2, \\mathrm{~b}=4-\\sqrt{13}$,\n\n$\\therefore 2 \\mathrm{a}-\\mathrm{b}=2 \\times 2-(4-\\sqrt{13})=\\sqrt{13}$,\n\n故选 C."} {"id": "1405", "image": [], "answer": "D", "solution": "null", "level": "七年级", "question": "已知 $\\mathrm{x}$ 没有平方根, 且 $|\\mathrm{x}|=64$, 则 $\\mathrm{x}$ 的立方根为 $(\\quad)$", "options": "A. 8\nB. -8\nC. $\\pm 4$\nD. -4", "subject": "代数", "analysis": "由题意得, $x$ 为负数,\n\n又 $\\because|x|=64$,\n\n$\\therefore \\mathrm{x}=-64$,\n\n故可得: $\\mathrm{x}$ 的立方根为: -4 .\n故选 D."} {"id": "1433", "image": [], "answer": "C", "solution": "null", "level": "七年级", "question": "下列四种说法中:\n\n( 1 )负数没有立方根;(2) 1 的立方根与平方根都是 1;(3) $\\sqrt[3]{8}$ 的平方根是 $\\pm \\sqrt{2}$ ;(4) $\\sqrt[3]{8+\\frac{1}{8}}=2+\\frac{1}{2}=2 \\frac{1}{2}$. 共有 ( ) 个是错误的.", "options": "1\nB. 2\nC. 3\nD. 4", "subject": "代数", "analysis": "(1) 负数的立方根为负数, 故 (1) 错误;\n\n(2) 1 的立方根是 1,1 的平方根是 $\\pm 1$, 故(2)错误;\n\n(3) $\\sqrt[3]{8}$ 的平方根是 $\\pm \\sqrt{2}$, 故 ( 3 ) 正确;\n\n(4) $\\sqrt[3]{8+\\frac{1}{8}}=\\sqrt[3]{\\frac{65}{8}}=\\frac{\\sqrt[3]{65}}{2}$, 故(4)错误;\n\n$\\therefore$ 错误的个数为 3 个.\n\n故答案为: C."} {"id": "1434", "image": [], "answer": "D", "solution": "null", "level": "七年级", "question": "已知 $|a|=5, \\sqrt{b^{2}}=7$, 且 $|a+b|=a+b$, 则 $a-b$ 的值为 ( )", "options": "A. 2 或 12\nB. 2 或 -12\nC. -2 或 12\nD. -2 或 -12", "subject": "代数", "analysis": "根据 $|a|=5, \\sqrt{b^{2}}=7$, 得 $a= \\pm 5, b= \\pm 7$, 因为 $|a+b|=a+b$, 则 $a= \\pm 5, b=$ 7 , 则 $a-b=5-7=-2$ 或 $-5-7=-12$.\n\n故答案为: $\\mathrm{D}$"} {"id": "1454", "image": [], "answer": "B", "solution": "null", "level": "七年级", "question": "已知数 $\\mathrm{a}$ 的平方根与其立方根相同, 数 $\\mathrm{b}$ 和其相反数相等, 则 $\\mathrm{a}+\\mathrm{b}=(\\quad)$", "options": "-1\nB. 0\nC. 1\nD. 2", "subject": "代数", "analysis": "$\\mathrm{a}$ 的平方根与其立方根相同,\n\n$\\therefore a=0$\n\n$\\because$ 数 $\\mathrm{b}$ 和其相反数相等,\n\n$\\therefore \\mathrm{b}=0$,\n\n$\\therefore \\mathrm{a}+\\mathrm{b}=0$.\n\n故答案为: B."} {"id": "1459", "image": [], "answer": "D", "solution": "null", "level": "七年级", "question": "下列计算, 结果正确的是", "options": "A. $4 a^{2} b-5 a b^{2}=-a^{2}-b$\nB. $5 a^{2}+3 a^{2}=8 a^{4}$\nC. $2 x+3 y=5 x y$\nD. $3 x y-5 y x=-2 x y$", "subject": "代数", "analysis": "A、 $4 a^{2} b$ 与 $-5 a b^{2}$, 不是同类项, 不能合并, 故本选项错误, 不符合题意;\n\nB、 $5 a^{2}+3 a^{2}=8 a^{2}$, 故本选项错误, 不符合题意;\n\nC、 $2 x$ 和 $3 y$ ,不是同类项, 不能合并, 故本选项错误, 不符合题意;\n\nD、 $3 x y-5 y x=-2 x y$, 故本选项正确, 符合题意;\n\n故答案为: $\\mathrm{D}$."} {"id": "1470", "image": [], "answer": "C", "solution": "null", "level": "七年级", "question": "若 $-2 a^{n} b^{5}$ 与 $5 a^{3} b^{2 m+n}$ 的差仍是单项式, 则 $m+n$ 的值是 ( )", "options": "A. 2\nB. 3\nC. 4\nD. 5", "subject": "代数", "analysis": "$\\because-2 a^{n} b^{5}$ 与 $5 a^{3} b^{2 m+n}$ 的差仍是单项式,\n\n$\\therefore \\mathrm{n}=3,2 \\mathrm{~m}+\\mathrm{n}=5$,\n\n$\\therefore \\mathrm{n}=3, \\mathrm{~m}=1$,\n\n$\\therefore \\mathrm{m}+\\mathrm{n}=1+3=4$,\n\n故答案为: $\\mathrm{C}$."} {"id": "1476", "image": [], "answer": "C", "solution": "null", "level": "七年级", "question": "下列说法中, 正确的是", "options": "A. 多项式 $a^{2}+2 a^{2} b+3$ 是二次三项式\n\nB. 单项式 $-\\pi x^{2} y$ 的系数是 -1\n\nC. 单项式 $4 m^{2} n$ 和 $-n m^{2}$ 是同类项\nD. $a b+3 b$ 是单项式", "subject": "代数", "analysis": "A、多项式 $a^{2}+2 a^{2} b+3$ 是三次三项式, 故原说法错误;\n\nB、单项式 $-\\pi x^{2} y$ 的系数是 $-\\pi$, 故原说法错误;\n\nC、单项式 $4 m^{2} n$ 和 $-n m^{2}$ 是同类项, 故原说法正确;\n\nD、 $a b+3 b$ 是多项式, 故原说法错误;\n\n故答案为: $\\mathrm{C}$."} {"id": "1477", "image": [], "answer": "A", "solution": "null", "level": "七年级", "question": "如果代数式 $6 y^{2}+3 y+2$ 的值是 5, 那么 $2 y^{2}+y-3$ 的值为 ( )", "options": "A. -2\nB. 2\nC. 1\nD. -3", "subject": "代数", "analysis": "$\\because 6 y^{2}+3 y+2=5$,\n\n$\\therefore 6 y^{2}+3 y=3$,\n\n$\\therefore 2 y^{2}+y=1$,\n\n$\\therefore 2 y^{2}+y-3=1-3=-2$,\n\n故答案为: A."} {"id": "1479", "image": ["3147.jpg"], "answer": "C", "solution": "null", "level": "七年级", "question": "对于代数式 $\\frac{y^{2}-1}{2}$, 第三学习小组讨论后得出如下结论: (1)代数式还可以写成 $\\frac{y^{2}}{2}-\\frac{1}{2}$; (2)如图,较大正方形的边长为 $\\mathrm{y}$, 较小正方形的边长为 1 , 则代数式表示阴影部分的面积; (3)其可以叙述为: $\\mathrm{y}$ 与 1 的平方差的一半; (4) 代数式的值可能是 -1 , 其中正确的个数为\n\n", "options": "1\nB. 2\nC. 3\nD. 4", "subject": "代数", "analysis": "代数式 $\\frac{y^{2}-1}{2}$ 还可以写成 $\\frac{y^{2}}{2}-\\frac{1}{2}$, 则 (1) 符合题意;\n图中阴影部分的面积等于较大正方形的面积与较小正方形的面积之差的一半, 即为 $\\frac{y^{2}-1}{2}$, 则 (2)符合题意;\n\n代数式 $\\frac{y^{2}-1}{2}$ 可以叙述为: $y$ 与 1 的平方差的一半, 则 (3)符合题意;\n\n$\\because y^{2} \\geq 0$\n\n$\\therefore \\frac{y^{2}-1}{2}=\\frac{y^{2}}{2}-\\frac{1}{2} \\geq-\\frac{1}{2}>-1$,\n\n所以代数式 $\\frac{y^{2}-1}{2}$ 的值不可能是 -1 , 即(4)不符合题意;\n\n综上, 正确的个数为 3 个,\n\n故答案为: C."} {"id": "1482", "image": [], "answer": "D", "solution": "null", "level": "七年级", "question": "观察等式: $2+2^{2}=2^{3}-2 ; 2+2^{2}+2^{3}=2^{4}-1 ; 2+2^{2}+2^{3}+2^{4}=2^{5}-2 \\ldots$ 已知按一定规律排列的一组数: $2^{45} 、 2^{46} 、 2^{47} 、 \\cdots 、 2^{89} 、 2^{90}$. 若 $2^{45}=m$, 用含 $m$ 的式子表示这组数的和是 $(\\quad)$", "options": "A. $2 m^{2}-2 m-2$\nB. $2 m^{2}-2 m$\nC. $2 m^{2}+m$\nD. $2 m^{2}-m$", "subject": "代数", "analysis": "$\\because 2+2^{2}=2^{3}-2$\n\n$2+2^{2}+2^{3}=2^{4}-1$\n\n$2+2^{2}+2^{3}+2^{4}=2^{5}-2 \\cdots$\n\n$\\therefore 2+2^{2}+2^{3}+\\ldots+2^{n}=2^{n+1}-2$,\n\n$\\therefore 2^{45}+2^{46}+2^{47}+\\ldots+2^{89}+2^{90}$\n\n$=\\left(2+2^{2}+2^{3}+\\ldots+2^{90}\\right)-\\left(2+2^{2}+2^{3}+\\ldots+2^{44}\\right)$\n\n$=\\left(2^{91}-2\\right)-\\left(2^{45}-2\\right)$\n\n$=2^{91}-2^{45}$,\n\n$\\because 2^{45}=m$\n\n$\\therefore 2^{91}=\\left(2^{45}\\right)^{2 \\cdot} \\cdot 2=2 \\mathrm{~m}^{2}$\n\n$\\therefore$ 原式 $=2 m^{2}-m$.\n\n故答案为: $\\mathrm{D}$"} {"id": "1502", "image": [], "answer": "D", "solution": "null", "level": "七年级", "question": "若代数式 $a x^{2}+4 x-y+3-\\left(2 x^{2}-b x+5 y-1\\right)$ 的值与 $\\mathrm{x}$ 的取值无关, 则 $a+b$ 的值为 ( )", "options": "A. 6\nB. -6\nC. 2\nD. -2", "subject": "代数", "analysis": "$a x^{2}+4 x-y+3-\\left(2 x^{2}-b x+5 y-1\\right)$\n\n$$\n\\begin{gathered}\n=a x^{2}+4 x-y+3-2 x^{2}+b x-5 y+1 \\\\\n=(a-2) x^{2}+(4+b) x-6 y+4\n\\end{gathered}\n$$\n\n由结果与 $\\mathrm{x}$ 的取值无关, 得到 $\\mathrm{a}-2=0, \\mathrm{~b}+4=0$,\n\n解得: $a=2, b=-4$,\n\n$a+b=2-4=-2$,\n\n故答案为: $\\mathrm{D}$"} {"id": "1506", "image": [], "answer": "C", "solution": "null", "level": "七年级", "question": "已知: $(2 x+1)^{3}=a x^{3}+b x^{2}+c x+d$, 那么代数式 $f(x)=\\mathrm{a}+\\mathrm{b}+\\mathrm{c}+\\mathrm{d}$ 的值是 ( )", "options": "A. -1\nB. 1\nC. 27\nD. -27", "subject": "代数", "analysis": "令 $\\mathrm{x}=1$, 原等式变形为: $(2+1)^{3}=a+b+c+d$,即 $\\mathrm{a}+\\mathrm{b}+\\mathrm{c}+\\mathrm{d}=27$,\n$\\therefore$ 代数式 $f(x)=\\mathrm{a}+\\mathrm{b}+\\mathrm{c}+\\mathrm{d}$ 的值是 27 .\n\n故答案为: $\\mathrm{C}$."} {"id": "1507", "image": [], "answer": "C", "solution": "null", "level": "七年级", "question": "下列表述正确的是 $(\\quad)$", "options": "A. 单项式 $\\mathrm{ab}$ 的系数是 0 , 次数是 2\nB. $-2 x^{2} y^{3}$ 的系数是 -2 , 次数是 3\nC. $x-1$ 是一次二项式\nD. $-a b^{2}+3 a-1$ 的项是 $-a b^{2}, 3 a, 1$", "subject": "代数", "analysis": "A、单项式 ab 的系数是 1 , 次数是 2 , 故此选项不合题意;\n\nB、 $-2 x^{2} y^{3}$ 的系数是 -2 , 次数是 5 , 故此选项不合题意;\n\nC、 $x-1$ 是一次二项式, 故此选项符合题意;\n\nD、 $-a b^{2}+3 a-1$ 的项是 $-a b^{2}, 3 a,-1$, 故此选项不合题意.\n\n故答案为: $\\mathrm{C}$.\nC."} {"id": "1518", "image": [], "answer": "D", "solution": "null", "level": "七年级", "question": "已知 $3 x^{6} y^{2}$ 和 $x^{3 m} y^{n}$ 是同类项, 则 $2 m-n$ 的值是 ( )", "options": "6\nB. 5\nC. 4\nD. 2", "subject": "代数", "analysis": "$\\because 3 x^{6} y^{2}$ 和 $x^{3 m} y^{n}$ 是同类项,\n\n$\\therefore 3 m=6, n=2$,\n\n$\\therefore \\mathrm{m}=2$,\n\n$\\therefore 2 m-n=2 \\times 2-2=2$.\n\n故答案为: D."} {"id": "1525", "image": [], "answer": "C", "solution": "null", "level": "七年级", "question": "若代数式 $2 x^{2}+3 x=8$, 则代数式 $4 x^{2}+6 x+15$ 的值是( $\\quad )$", "options": "21\nB. 17\nC. 31\nD. 16", "subject": "代数", "analysis": "$\\because 2 \\mathrm{x}^{2}+3 \\mathrm{x}=8$,\n\n$\\therefore 4 \\mathrm{x}^{2}+6 \\mathrm{x}+15=2\\left(2 x^{2}+3 x\\right)+15=2 \\times 8+15=16+15=31$\n\n故答案为: $\\mathrm{C}$"} {"id": "1527", "image": [], "answer": "B", "solution": "null", "level": "七年级", "question": "已知无论 $x, y$ 取什么值, 多项式 $\\left(3 x^{2}-m y+9\\right)-\\left(n x^{2}+5 y-3\\right)$ 的值都等于定值 12 , 则 $m+n$等于()。", "options": "8\nB. -2\nC. 2\nD. -8", "subject": "代数", "analysis": "$\\left(3 x^{2}-m y+9\\right)-\\left(n x^{2}+5 y-3\\right)$\n\n$=3 x^{2}-m y+9-n x^{2}-5 y+3$\n\n$=(3-n) x^{2}-(m+5) y+12$,\n\n$\\because$ 无论 $x, y$ 取什么值, 多项式的值都等于定值 12,\n\n$\\therefore 3-\\mathrm{n}=0, \\mathrm{~m}+5=0$,\n\n解得: $n=3, m=-5$,\n\n$\\therefore m+n=(-5)+3=-2$,\n\n故答案为: B."} {"id": "1529", "image": [], "answer": "C", "solution": "null", "level": "七年级", "question": "对于任意实数 $\\mathrm{a}$ 和 $\\mathrm{b}$, 如果满足 $\\frac{a}{3}+\\frac{b}{4}=\\frac{a+b}{3+4}+\\frac{2}{3 \\times 4}$ 那么我们称这一对数 $\\mathrm{a}, \\mathrm{b}$ 为“友好数对”, 记为 $(a, b)$. 若 $(x, y)$ 是“友好数对”, 则 $2 x-3[6 x+(3 y-4)]=$", "options": "A. -4\nB. -3\nC. -2\nD. -1", "subject": "代数", "analysis": "$\\because(x, y)$ 是“友好数对”,\n\n$\\therefore \\frac{x}{3}+\\frac{y}{4}=\\frac{x+y}{3+4}+\\frac{2}{3 \\times 4}$\n\n$\\therefore \\frac{x}{3}+\\frac{y}{4}=\\frac{x+y}{7}+\\frac{1}{6}$\n\n整理得: $\\quad 16 x+9 y=14$,\n\n$\\therefore 2 x-3[6 x+(3 y-4)]$\n\n$=-16 x-9 y+12$\n\n$=-(16 x+9 y)+12$\n\n$=\\quad-14+12$\n\n$=-2$\n\n故答案为: C"} {"id": "1548", "image": [], "answer": "B", "solution": "null", "level": "七年级", "question": "一个长方形的周长为 $6 a+8 b$, 其一边长为 $2 a+3 b$, 则另一边长为", "options": "$4 a+5 b$\nB. $a+b$\nC. $a+2 b$\nD. $a+7 b$", "subject": "代数", "analysis": "另一边长为 $(6 a+8 b) \\div 2-(2 a+3 b)=3 a+4 b-2 a-3 b=a+b$;\n\n故答案为: B.\n\n【分析】由于长方形的周长等于两邻边和的 2 倍, 故知道周长及一条边长, 可以用周长除以 2 再减去已知的边长, 据此列出式子, 进而根据整式的加减法法则算出答案."} {"id": "1549", "image": [], "answer": "A", "solution": "null", "level": "七年级", "question": "用字母 $\\mathrm{a}$ 表示任意一个有理数, 下列四个代数式中,值不可能为 0 的是()", "options": "$1+a^{2}$\nB. $|a+1|$\nC. $a^{2}$\nD. $a^{3}+1$", "subject": "代数", "analysis": "当字母 $a$ 表示任意一个有理数时, $a^{2} \\geq 0$,\n\n$\\therefore 1+\\mathrm{a}^{2}>0$, 故 A 选项符合题意;\n\n当 $a=0$ 时, $a^{2}=0$, 故 $C$ 选项不符合题意;\n\n当 $a=-1$ 时, $|a+1|=0, a^{3}+1=0$, 故 B 选项, D 选项均不符合题意;\n\n故答案为: A."} {"id": "1552", "image": [], "answer": "D", "solution": "null", "level": "七年级", "question": "若代数式 $a x^{2}+4 x-y+3-\\left(2 x^{2}-b x+5 y-1\\right)$ 的值与 $\\mathrm{x}$ 的取值无关, 则 $a+b$ 的值为", "options": "A. 6\nB. -6\nC. 2\nD. -2", "subject": "代数", "analysis": "$a x^{2}+4 x-y+3-\\left(2 x^{2}-b x+5 y-1\\right)$\n\n$$\n\\begin{gathered}\n=a x^{2}+4 x-y+3-2 x^{2}+b x-5 y+1 \\\\\n=(a-2) x^{2}+(4+b) x-6 y+4\n\\end{gathered}\n$$\n\n由结果与 $\\mathrm{x}$ 的取值无关, 得到 $\\mathrm{a}-2=0, \\mathrm{~b}+4=0$,\n\n解得: $a=2, b=-4$,\n\n$a+b=2-4=-2$,\n\n故答案为: D"} {"id": "1554", "image": [], "answer": "C", "solution": "null", "level": "七年级", "question": "观察等式: $2^{2}+2^{2}=2^{3}-2 ; 2^{2}+2^{2}+2^{3}=2^{4}-2 ; 2^{2}+2^{2}+2^{3}+2^{4}=2^{5}-2 \\ldots$ 已知按一定规律排列的一组数: $2^{100} 、$ $2^{101} 、 2^{102} \\ldots 、 2^{199} 、 2^{200}$, 设 $2^{100}=a$, 用含 $a$ 的式子表示这组数的和是 ( )", "options": "$2 a^{2}+a$\nB. $2 a^{2}-2 a-2$\nC. $2 a^{2}-a$\nD. $2 a^{2}-2 a$", "subject": "代数", "analysis": "$\\because 2^{100}=\\mathrm{a}$,\n\n$\\therefore 2^{100}+2^{101}+2^{102}+\\ldots+2^{199}+2^{200}$\n\n$=a+2 a+2^{2} a+\\ldots+2^{99} a+2^{100} a$\n\n$=\\mathrm{a}\\left(1+2+2^{2}+\\ldots+2^{99}+2^{100}\\right)$\n$=\\mathrm{a}\\left(1+2^{100}-2+2^{100}\\right)$\n\n$=\\mathrm{a}(2 \\mathrm{a}-1)$\n\n$=2 a^{2}-a$.\n\n故答案为: $\\mathrm{C}$.\nC."} {"id": "1572", "image": [], "answer": "C", "solution": "null", "level": "七年级", "question": "若单项式 $3 a b^{4 n+1}$ 与 $9 a b^{(2 n+2)-1}$ 是同类项, 则 $n$ 的值是 ( )", "options": "A. 7\nB. 2\nC. 0\nD. -1", "subject": "代数", "analysis": "$\\because$ 单项式 $3 a b^{4 n+1}$ 与 $9 a b^{(2 n+2)-1}$ 是同类项,\n\n$\\therefore 4 \\mathrm{n}+1=(2 \\mathrm{n}+2)-1$,\n\n$\\therefore 4 \\mathrm{n}+1=2 \\mathrm{n}+2-1$,\n\n移项得: $2 \\mathrm{n}=0$\n$\\therefore \\mathrm{n}=0$,\n\n故选: C."} {"id": "1575", "image": [], "answer": "D", "solution": "null", "level": "七年级", "question": "下列说法中, 正确的有", "options": "等式两边各加上一个式子, 所得的结果仍是等式\n\nB. 等式两边各乘以一个数, 所得的结果仍是等式\n\nC. 等式两边都除以同一个数, 所得的结果仍是等式\n\nD. 一个等式的左右两边分别与另一个等式的左右两边相加, 所得的结果仍是等式.", "subject": "代数", "analysis": "A、等式两边各加上同一个式子,所得的结果仍是等式,应是加上同一个式子,故此选项不符合题意;\n\nB、等式两边各乘以一个相同的数, 所得的结果仍是等式, 应是乘以相同的数, 故此选项不符合题意;\n\nC、等式两边都除以同一个不为 0 的数, 所得的结果仍是等式, 应是除以不为 0 的数, 故此选项不符合题意;\n\nD、一个等式的左右两边分别与另一个等式的左右两边相加, 所得的结果仍是等式, 故此选项符合题意;\n\n故答案为: D."} {"id": "1578", "image": [], "answer": "B", "solution": "null", "level": "七年级", "question": "《九章算术》是中国传统数学最重要的著作, 奠定了中国传统数学的基本框架. 它的代数成就主要包括开方术、正负术和方程术,其中方程术是其最高的代数成就. 《九章算术》中有这样一个问题: “今有善行者行一百步, 不善行者行六十步,今不善行者先行一百步, 善行者追之, 问几何步及之?\"译文: “相同时间内, 走路快的人走 100 步, 走路慢的人只走 60 步.若走路慢的人先走 100 步, 走路快的人要走多少步才能追上? (注: 步为长度单位) ”设走路快的人要走 $\\mathrm{x}$ 步才能追上,根据题意可列出的方程是", "options": "$x=100-\\frac{60}{100} x$\nB. $x=100+\\frac{60}{100} x$\nC. $\\frac{100}{60} x=100+x$\nD. $\\frac{100}{60} x=100-x$", "subject": "代数", "analysis": "令在相同时间 $\\mathrm{t}$ 内走路快的人走 100 步, 走路慢的人只走 60 步, 从而得到走路快的人的速度 $\\frac{100}{t}$ 步 , 走路慢的人的速度 $\\frac{60}{t}$ 步,\n\n设走路快的人要走 $\\mathrm{x}$ 步才能追上, 根据题意可得 $x=100+\\frac{60}{t} \\times \\frac{x}{\\frac{100}{t}}$,\n\n$\\therefore$ 根据题意可列出的方程是 $x=100+\\frac{60}{100} x$.\n\n故答案为: B."} {"id": "1579", "image": [], "answer": "A", "solution": "null", "level": "七年级", "question": "下列等式的变形中, 正确的是 $(\\quad)$", "options": "A. 如果 $\\frac{a}{2 c}=\\frac{b}{2 c}$, 那么 $\\mathrm{a}=\\mathrm{b}$\nB. 如果 $\\mathrm{a}=\\mathrm{b}$, 那么 $\\frac{a}{2 c+1}=\\frac{b}{2 c+1}$\nC. 如果 $a x=a y$, 那么 $x=y$\nD. 如果 $\\mathrm{m}=\\mathrm{n}$, 那么 $\\frac{m}{c^{2}-4}=\\frac{n}{c^{2}-4}$", "subject": "代数", "analysis": "A. 如果 $\\frac{a}{2 c}=\\frac{b}{2 c}$, 那么 $\\mathrm{a}=\\mathrm{b}$, 符合题意;\n\nB. 当 $c=-\\frac{1}{2}$ 时, 等式不成立, 不符合题意;\n\nC. 当 $\\mathrm{a}=0$ 时, 等式成立, 但 $\\mathrm{x}$ 和 $\\mathrm{y}$ 不一定相等, 不符合题意;\n\nD. 当 $c= \\pm 2$ 时, 等式不成立, 不符合题意.\n\n故答案为: A."} {"id": "1589", "image": [], "answer": "B", "solution": "null", "level": "七年级", "question": "若方程 $2 x+1=-3$ 的解是关于 $x$ 的方程 $7-2(x-a)=3$ 的解, 则 $a$ 的值为 ( )", "options": "A. -2\nB. -4\nC. -5\nD. -6", "subject": "代数", "analysis": "解方程 $2 x+1=-3$ 得: $x=-2$,\n\n把 $x=-2$ 代入方程 $7-2(x-a)=3$ 得:\n\n$7-2(-2-a)=3$,\n\n解得: $a=-4$,\n\n故答案为:\nB."} {"id": "1595", "image": [], "answer": "C", "solution": "null", "level": "七年级", "question": "若等式 $m+a=n-b$ 根据等式的性质变形得到 $m=n$, 则 $a 、 b$ 满足的条件是", "options": "A. 相等\nB. 互为倒数\nC. 互为相反数\nD. 无法确定", "subject": "代数", "analysis": "$m+a=n-b$ 两边都加上 $b$ 得, $m+a+b=n$,\n\n$\\because$ 等式可变形为 $\\mathrm{m}=\\mathrm{n}$,\n\n$\\therefore \\mathrm{a}+\\mathrm{b}=0$,\n\n$\\therefore \\mathrm{a}=-\\mathrm{b}$.\n\n故答案为: C."} {"id": "1596", "image": [], "answer": "C", "solution": "null", "level": "七年级", "question": "若关于 $\\mathrm{x}$ 的一元一次方程 $\\frac{2 x-k}{3}-\\frac{x-3 k}{2}=1$ 的解是 $\\mathrm{x}=-1$, 则 $\\mathrm{k}$ 的值是 ( )", "options": "A. $\\frac{2}{7}$\nB. $-\\frac{3}{11}$\nC. 1\nD. 0", "subject": "代数", "analysis": "将 $\\mathrm{x}=-1$ 代入方程, 得 $\\frac{-2-k}{3}-\\frac{-1-3 k}{2}=1$,\n\n去分母得 $2(-2-k)-3(-1-3 k)=6$\n\n去括号得- $4-2 k+3+9 k=6$\n\n移项、合并同类项得 $7 \\mathrm{k}=7$\n\n系数化为 1 得 $\\mathrm{k}=1$.\n\n故答案为: C."} {"id": "1597", "image": [], "answer": "B", "solution": "null", "level": "七年级", "question": "一元一次方程 $\\frac{2 x-0.3}{0.5}-\\frac{x+0.4}{0.3}=1$ 可以化简成 $\\frac{20 x-3}{5}-\\frac{10 x+4}{3}=1$, 其依据是 ( )", "options": "A. 等式的性质 1\nB. 分数的性质\nC. 分配律\nD. 等式的性质 2", "subject": "代数", "analysis": "由一元一次方程 $\\frac{2 x-0.3}{0.5}-\\frac{x+0.4}{0.3}=1$ 可以化简成 $\\frac{20 x-3}{5}-\\frac{10 x+4}{3}=1$, 其依据是分数的性质;故答案为: B."} {"id": "1600", "image": [], "answer": "A", "solution": "null", "level": "七年级", "question": "阅读: 关于 $\\mathrm{x}$ 方程 $\\mathrm{ax}=\\mathrm{b}$ 在不同的条件下解的情况如下:(1)当 $\\mathrm{a} \\neq 0$ 时,有唯一解 $\\mathrm{x}=\\frac{b}{a}$; (2) 当 $\\mathrm{a}=0, \\mathrm{~b}=0$ 时有无数解; (3) 当 $\\mathrm{a}=0, \\mathrm{~b} \\neq 0$ 时无解. 请你根据以上知识作答: 已知关于 $\\mathrm{x}$ 的方程 $\\frac{x}{3} \\mathrm{a}=$ $\\frac{x}{2}-\\frac{1}{6}(x-6)$ 无解, 则 $\\mathrm{a}$ 的值是 $(\\quad)$", "options": "1\nB. -1\nC. $\\pm 1$\nD. $\\mathrm{a} \\neq 1$", "subject": "代数", "analysis": "要把原方程变形化简,去分母得:2ax=3x - $(x-6)$,去括号得: $2 \\mathrm{ax}=2 \\mathrm{x}+6$,\n\n移项, 合并得, $\\mathrm{x}=\\frac{3}{a-1}$,\n\n因为无解,所以 $\\mathrm{a}-1=0$, 即 $\\mathrm{a}=1$.\n\n故答案为: A."} {"id": "1603", "image": [], "answer": "C", "solution": "null", "level": "七年级", "question": "下列变形正确的是 $(\\quad)$", "options": "A. 由 $\\frac{x-5}{3}-1=\\frac{2 x+1}{5}$ 去分母, 得 $5(x-5)-1=3(2 x+1)$\n\nB. 由 $3(2 x-1)-2(x+5)=4$ 去括号, 得 $6 x-3-2 x+10=4$\n\nC. 由 $-6 x-1=2 x$ 移项,得 $-6 x-2 x=1$\n\nD. 由 $2 x=3$ 系数化为 $1, x=\\frac{2}{3}$", "subject": "代数", "analysis": "A、由 $\\frac{x-5}{3}-1=\\frac{2 x+1}{5}$, 去分母得: $5(x-5)-15=3(2 x+1)$, 不符合题意;\n\nB、由 $3(2 x-1)-2(x+5)=4$, 去括号得: $6 x-3-2 x-10=4$, 不符合题意;\n\nC、由 $-6 x-1=2 x$, 移项得: $-6 x-2 x=1$, 符合题意;\n\nD、由 $2 x=3$, 系数化为 1 , 得: $x=\\frac{3}{2}$, 不符合题意.\n\n故答案为: $\\mathrm{C}$."} {"id": "1614", "image": [], "answer": "D", "solution": "null", "level": "七年级", "question": "已知关于 $x$ 的方程 $2 x+a=1-x$ 与方程 $2 x-3=1$ 的解相同, 则 $a$ 的值为", "options": "2\nB. -2\nC. 5\nD. -5", "subject": "代数", "analysis": "$\\because 2 x-3=1$,\n\n$\\therefore \\mathrm{x}=2$,\n\n$\\because$ 方程 $2 x+a=1-x$ 与方程 $2 x-3=1$ 的解相同,\n\n$\\therefore 2 \\times 2+a=1-2$,\n\n$\\therefore \\mathrm{a}=-5$.\n\n故答案为: D."} {"id": "1623", "image": [], "answer": "B", "solution": "null", "level": "七年级", "question": "某商场把一个双肩包按进价提高 $30 \\%$ 标价, 然后按八折出售, 这样商场每卖出一个书包仍可盈利 10 元.设每个双肩书包的进价是 $\\mathrm{x}$ 元,根据题意列一元一次方程正确的是 ( )", "options": "$30 \\% x \\cdot 80 \\%-x=10$\nB. $(1+30 \\%) x \\cdot 80 \\%-x=10$\nC. $(1+30 \\%) x \\cdot 80 \\%=10$\nD. $(1+30 \\%) x-x=10$", "subject": "代数", "analysis": "根据题意得: $(1+30 \\%) \\mathrm{x} \\cdot 80 \\%-\\mathrm{x}=10$,\n\n故答案为: B."} {"id": "1628", "image": [], "answer": "A", "solution": "null", "level": "七年级", "question": "若不论 $k$ 取什么实数, 关于 $x$ 的方程 $\\frac{2 k x+a}{3}-\\frac{x-b k}{6}=1 ( a 、 b$ 常数)的解总是 $x=$ 1 , 则 $a+b$ 的值是", "options": "-0.5\nB. 0.5\nC. -1.5\nD. 1", "subject": "代数", "analysis": "because$ 关于 $\\mathrm{x}$ 的方程 $\\frac{2 k x+a}{3}-\\frac{x-b k}{6}=1$ 的解总是 $x=1$\n\n$\\therefore \\frac{2 k+a}{3}-\\frac{1-b k}{6}=1$\n\n$\\therefore 4 k+2 a-1+b k=6$\n\n$\\therefore(4+b) k=7-2 a$\n\n$\\therefore\\left\\{\\begin{array}{l}4+b=0 \\\\ 7-2 a=0\\end{array}\\right.$\n解得: $\\left\\{\\begin{array}{c}a=\\frac{7}{2} \\\\ b=-4\\end{array}\\right.$\n\n$\\therefore a+b=\\frac{7}{2}-4=-\\frac{1}{2}$\n\n故答案为: A."} {"id": "1638", "image": [], "answer": "A", "solution": "null", "level": "七年级", "question": "已知代数式 $8 x-7$ 与 $6-2 x$ 的值互为相反数, 那么 $x$ 的值等于 ( )", "options": "A. $\\frac{1}{6}$\nB. $-\\frac{1}{6}$\nC. $\\frac{13}{10}$\nD. $-\\frac{13}{10}$", "subject": "代数", "analysis": "根据题意得: $(8 x-7)+(6-2 x)=0$,\n\n解得: $x=\\frac{1}{6}$.\n\n故答案为: A."} {"id": "1646", "image": [], "answer": "C", "solution": "null", "level": "七年级", "question": "一列火车匀速行驶, 经过一条长 800 米的隧道, 从车头开始进入隧道到车尾离开隧道一共需要 50 秒的时间: 在隧道中央的顶部有一盋灯, 垂直向下发光照在火车上的时间是 18 秒, 设该火车的长度为 $\\mathrm{x}$ 米,根据题意可列一元一次方程为 $(\\quad)$", "options": "A. $18 x-800=50 x$\nB. $18 \\mathrm{x}+800=50$\nC. $\\frac{800+x}{50}=\\frac{x}{18}$\nD. $\\frac{800-x}{50}=\\frac{x}{18}$", "subject": "代数", "analysis": "设该火车的长度为 $\\mathrm{x}$ 米, 根据火车速度 $=$ 火车长度 - 灯照时间 $=($ 隧道长 + 火车长 $) \\div$ 通过隧道的时间即可列出方程.\n\n依题意, 得: $\\frac{800+x}{50}=\\frac{x}{18}$.\n\n故答案为: C."} {"id": "1648", "image": [], "answer": "D", "solution": "null", "level": "七年级", "question": "在光明区举办的“周年艺术季”期间, 小颖一家去欣赏了一台音乐剧, 路上预计用时 25 分钟, 但由于堵车, 所以实际车速比预计的每小时慢了 10 千米, 且路上多用了 5 分钟. 设预计车速为 $\\mathrm{x}$ 千米/时, 根据题意可列方程为", "options": "A. $\\frac{25}{60} x=\\frac{30}{60}(x+10)$\nB. $\\frac{25}{60}(x+10)=\\frac{30}{60} x$\nC. $25 x=30 x-10$\nD. $\\frac{25}{60} x=\\frac{30}{60}(x-10)$", "subject": "代数", "analysis": "$\\because$ 预计车速为 $x$ 千米/时, 实际车速比预计的每小时慢了 10 千米,\n\n$\\therefore$ 实际车速为 $(x-10)$ 千米/时.\n\n依题意得: $\\frac{25}{60} x=\\frac{25+5}{60}(x-10)$,\n\n即 $\\frac{25}{60} x=\\frac{30}{60}(x-10)$.\n\n## 故答案为: $D$."} {"id": "1649", "image": [], "answer": "A", "solution": "null", "level": "七年级", "question": "阅读: 关于 $\\mathrm{x}$ 方程 $\\mathrm{ax}=\\mathrm{b}$ 在不同的条件下解的情况如下: (1) 当 $\\mathrm{a} \\neq 0$ 时, 有唯一解 $\\mathrm{x}=\\frac{b}{a}$; (2) 当 $\\mathrm{a}=0, \\mathrm{~b}=0$ 时有无数解; (3) 当 $\\mathrm{a}=0, \\mathrm{~b} \\neq 0$ 时无解. 请你根据以上知识作答: 已知关于 $\\mathrm{x}$ 的方程 $\\frac{x}{3} \\mathrm{a}=$ $\\frac{x}{2}-\\frac{1}{6}(x-6)$ 无解, 则 $\\mathrm{a}$ 的值是 $(\\quad)$", "options": "1\nB. -1\nC. $\\pm 1$\nD. $\\mathrm{a} \\neq 1$", "subject": "代数", "analysis": "要把原方程变形化简, 去分母得: $2 a x=3 x-(x-6)$,\n\n去括号得: $2 \\mathrm{ax}=2 \\mathrm{x}+6$,\n\n移项, 合并得, $\\mathrm{x}=\\frac{3}{a-1}$,\n\n因为无解, 所以 $\\mathrm{a}-1=0$, 即 $\\mathrm{a}=1$.\n\n故答案为: A."} {"id": "1650", "image": [], "answer": "A", "solution": "null", "level": "七年级", "question": "下列说法正确的是 ( $\\quad )$", "options": "A. 0 的平方根是 0\nB. $(-3)^{2}$ 的平方根是 -3\nC. 1 的立方根是 $\\pm 1$\nD. -4 的平方根是 $\\pm 2$", "subject": "代数", "analysis": "A、 0 的平方根是 0, 正确,\n\nB、 $(-3)^{2}$ 的平方根是 $\\pm 3$, 原命题错误;\n\nC、 1 的立方根是 1 , 原命题错误;\n\nD、 -4 没有平方根,原命题错误,\n\n故答案为: A."} {"id": "1671", "image": [], "answer": "D", "solution": "null", "level": "七年级", "question": "$x 、 y 、 z$ 是有理数且 $x y z<0$, 则 $\\frac{|x|}{x}+\\frac{|y|}{y}+\\frac{|z|}{z}$ 的值是 ( )", "options": "A. -3\nB. 3 或 -1\nC. 1\nD. -3 或 1", "subject": "代数", "analysis": "$\\because x y z<0$,\n\n$\\therefore \\mathrm{x} 、 \\mathrm{y} 、 \\mathrm{z}$ 这三个数中有一个或三个数为负数,\n\n当这三个数中有一个负数时, 假设 $x<0, y>0, z>0$,\n\n则 $\\frac{|x|}{x}+\\frac{|y|}{y}+\\frac{|z|}{z}=\\frac{-x}{x}+\\frac{y}{y}+\\frac{z}{z}=-1+1+1=1$;\n\n当这三个数中有三个负数时,假设 $x<0, y<0, z<0$,\n\n则 $\\frac{|x|}{x}+\\frac{|y|}{y}+\\frac{|z|}{z}=\\frac{-x}{x}+\\frac{-y}{y}+\\frac{-z}{z}=-1-1-1=-3$; 故 D 符合题意.\n\n故答案为:\nD."} {"id": "1674", "image": [], "answer": "D", "solution": "null", "level": "七年级", "question": "为鼓励居民节约用水, 某市对居民用水实行“阶梯收费”, 规定每户每月用水量不超过 10 吨, 水价为每吨 2 元; 超过 10 吨的部分每吨 3.5 元. 已知小莉家某月交水费 34 元, 则小莉家该月用水多少吨?若设小莉家该月用水 $\\mathrm{x}$ 吨,则可列方程为 $(\\quad)$", "options": "A. $2 \\times 10-3.5 \\times(x-10)=34$\nB. $3.5 \\times 10+2 \\times(x-10)=34$\nC. $2 \\times 10+3.5 \\times(10-x)=34$\nD. $2 \\times 10+3.5 \\times(x-10)=34$", "subject": "代数", "analysis": "设小莉家该月用水 $x$ 吨, 根据题意得:\n\n$2 \\times 10+3.5 \\times(x-10)=34$, 故 $\\mathrm{D}$ 符合题意.\n\n故答案为: D."} {"id": "1685", "image": [], "answer": "B", "solution": "null", "level": "七年级", "question": "某车间 28 名工人生产螺栓和螺母, 每人每天平均生产螺栓 12 个或螺母 18 个, 一个螺栓需要两个螺母与之配套, 如何安排生产螺栓才能让螺栓和蝶母正好配套? 设有 $\\mathrm{x}$ 名工人生产螺栓, 其余人生产螺母, 依题意列方程应为 $(\\quad)$", "options": "A. $12 x=18(28-x)$\nB. $2 \\cdot 12 x=18(28-x)$\nC. $12 \\cdot 18 x=18(28-x)$\nD. $12 x=2 \\cdot 18(28-x)$", "subject": "代数", "analysis": "设有 $\\mathrm{x}$ 名工人生产螺栓, 则 $(28-x)$ 人生产螺母, 依题意得,\n\n$2 \\times 12 x=18(28-x)$,\n\n故答案为: B."} {"id": "1694", "image": [], "answer": "D", "solution": "null", "level": "七年级", "question": "我国“DF-41 型”导弹俗称“东风快递”, 速度可达到 26 马赫 (1 马赫 $=340$ 米(秒), 则“DF -41 型”导弹飞行多少分钟能打击到 12000 公里处的目标? 设飞行 $x$ 分钟能打击到目标, 可以得到方程 $(\\quad)$", "options": "$26 \\times 340 \\times 60 x=12000$\nB. $26 \\times 340 x=12000$\nC. $\\frac{26 \\times 340 x}{1000}=12000$\nD. $\\frac{26 \\times 340 \\times 60 x}{1000}=12000$", "subject": "代数", "analysis": "设飞行 $\\mathrm{x}$ 分钟能打击到目标, 根据题意得 $\\frac{26 \\times 340 \\times 60 x}{1000}=12000$.\n\n故答案为: D."} {"id": "1696", "image": [], "answer": "A", "solution": "null", "level": "七年级", "question": "小明与他的爸爸一起做“投篮球”游戏. 两人商定游戏规则为:小明投中 1 个得 2 分, 小明爸爸投中 1 个得 1 分, 两人共投中了 25 个. 经计算, 发现小明比爸爸多得 2 分, 你知道小明投中几个吗?设小明投中 $\\mathrm{x}$ 个, 根据题意, 列方程正确的是 $(\\quad)$", "options": "A. $2 x-(25-x)=2$\nB. $x-2(25-x)=2$\nC. $2 x+(2+x)=25$\nD. $(25-x)-2 x=2$", "subject": "代数", "analysis": "设小明投中数为 $\\mathrm{x}$ 个, 可知小明爸爸投中数为 $25-\\mathrm{x}$ 个, 由题可知小明比爸爸多得 2 分, 根据题意列方程:\n\n$2 x-(25-x)=2$\n\n故答案为: A"} {"id": "1747", "image": [], "answer": "C", "solution": "null", "level": "七年级", "question": "已知 $a, b$ 为实数, 下列说法: (1) 若 $a b<0$, 且 $a, b$ 互为相反数, 则 $\\frac{a}{b}=-1$; (2)若 $a+b<0$, $a b>0$, 则 $|2 a+3 b|=-2 a-3 b$; (3)若 $|a-b|+a-b=0$, 则 $b>a$; (4)若 $|a|>|b|$, 则 $(a+b) \\times(a-b)$ 是正数; (5) 若 $a6$, 其中正确的说法有()个.", "options": "2\nB. 3\nC. 4\nD. 5", "subject": "代数", "analysis": ")若 $\\mathrm{ab}<0$, 且 $\\mathrm{a}, \\mathrm{b}$ 互为相反数, 则 $\\frac{a}{b}=-1$, 正确 ;\n\n(2) $\\because a+b<0, \\quad a b>0, \\quad \\therefore a<0, \\quad b<0, \\quad \\therefore 2 a+3 b<0, \\quad \\therefore|2 a+3 b|=-2 a-3 b$, 正确;\n\n(3) $\\because|\\mathrm{a}-\\mathrm{b}|+\\mathrm{a}-\\mathrm{b}=0, \\quad \\therefore|\\mathrm{a}-\\mathrm{b}|=\\mathrm{b}-\\mathrm{a} \\geq 0, \\quad \\therefore \\mathrm{b} \\geq \\mathrm{a}$, 错误;\n\n(4) 当 $a>0, \\quad b>0$ 时, 则 $a>b, \\quad \\therefore a-b>0, a+b>0, \\quad \\therefore(a+b) .(a-b)$ 为正数;\n\n当 $a>0, b<0$ 时, $a-b>0, a+b>0, \\therefore(a+b) .(a-b)$ 为正数;\n\n当 $a<0, b>0$ 时, $a-b<0, a+b<0, \\therefore(a+b) .(a-b)$ 为正数;\n\n当 $a<0, b<0$ 时, $a-b<0, a+b<0, \\quad \\therefore(a+b) .(a-b)$ 为正数;\n\n故 (4) 正确;\n\n(5) $\\because a0, a<0$,\n\n当 $03$,\n\n$\\because|a-3|<|b-3|$\n\n$\\therefore 3-a6$, 正确.\n\n综上,正确的有 4 项.\n\n故答案为: C."} {"id": "1769", "image": [], "answer": "A", "solution": "null", "level": "七年级", "question": "若 $a b c \\neq 0$, 则 $\\frac{|a|}{a}+\\frac{|b|}{b}+\\frac{c}{|c|}$ 的值为 ( )", "options": "$\\pm 3$ 或 $\\pm 1$\nB. $\\pm 3$ 或 0 或 $\\pm 1$\nC. $\\pm 3$ 或 0\nD. 0 或 $\\pm 1$", "subject": "代数", "analysis": "当 $a 、 b 、 c$ 没有负数时, 原式 $=1+1+1=3$;\n\n当 $a 、 b 、 c$ 有一个负数时, 原式 $=-1+1+1=1$;\n\n当 $a 、 b 、 c$ 有两个负数时, 原式 $=-1-1+1=-1$;\n\n当 $a 、 b 、 c$ 有三个负数时, 原式 $=-1-1-1=-3$.\n\n故答案为: $A$."} {"id": "1789", "image": [], "answer": "A", "solution": "null", "level": "七年级", "question": "若关于 $x 、 y$ 的多项式 $2 x^{2}+m x+5 y-2 n x^{2}-y+5 x+7$ 的值与 $x$ 的取值无关, 则 $m+n=(\\quad)$", "options": "-4\nB. -5\nC. -6\nD. 6", "subject": "代数", "analysis": "$2 x^{2}+m x+5 y-2 n x^{2}-y+5 x+7=(2-2 n) x^{2}+(m+5) x+4 y+7$,\n\n$\\because$ 关于 $x 、 y$ 的多项式 $2 x^{2}+m x+5 y-2 n x^{2}-y+5 x+7$ 的值与 $x$ 的取值无关,\n\n$\\therefore 2-2 n=0$,\n\n解得 $n=1$,\n\n$\\mathrm{m}+5=0$,\n\n解得 $m=-5$,\n\n则 $m+n=-5+1=-4$.\n\n故答案为: A."} {"id": "1790", "image": [], "answer": "B", "solution": "null", "level": "七年级", "question": "已知 $a_{1}+a_{2}=1, a_{2}+a_{3}=2, a_{3}+a_{4}=-3, a_{4}+a_{5}=-4, a_{5}+a_{6}=5, a_{6}+a_{7}=6, a_{7}+a_{8}=-7, a_{8}+a_{9}=-$\n\n$8, \\ldots \\ldots, a_{99}+a_{100}=-99, a_{100}+a_{1}=-100$, 那么 $a_{1}+a_{2}+a_{3}+\\ldots . .+a_{100}$ 的值为 $(\\quad)$", "options": "-48\nB. -50\nC. -98\nD. -100", "subject": "代数", "analysis": "$a_{1}+a_{2}+a_{3}+\\cdots \\ldots+a_{100}$\n\n$$\n\\begin{gathered}\n=\\left(a_{1}+a_{2}\\right)+\\left(a_{3}+a_{4}\\right)+\\left(a_{5}+a_{6}\\right)+\\left(a_{7}+a_{8}\\right)+\\cdots \\ldots+\\left(a_{99}+a_{100}\\right) \\\\\n=1+(-3)+5+(-7)+\\cdots \\ldots+(-99) \\\\\n=-2 \\times 50 \\div 2 \\\\\n=-50\n\\end{gathered}\n$$\n\n故答案为: B"} {"id": "1811", "image": [], "answer": "D", "solution": "null", "level": "七年级", "question": "符合条件 $|\\mathrm{a}+5|+|\\mathrm{a}-3|=8$ 的整数 $\\mathrm{a}$ 的值有().", "options": "A. 4 个\nB. 5 个\nC. 7 个\nD. 9 个", "subject": "代数", "analysis": "$|a+5|$ 表示 $a$ 到 -5 点的距离,\n\n$|a-3|$ 表示 $a$ 到 3 点的距离,\n\n由-5 到 3 点的距离为 8 ,\n\n故 -5 到 3 之间的所有点均满足条件,\n\n即 $-5 \\leq a \\leq 3 ,$\n\n又由 $\\mathrm{a}$ 为整数,\n\n故满足条件的 a 有: $-5,-4,-3,-2,-1,0,1,2,3$ 共 9 个,\n\n故答案为: D."} {"id": "1812", "image": [], "answer": "B", "solution": "null", "level": "七年级", "question": "已知 $|a|=8,|b|=3$, 且 $|a-b|=b-a$, 则 $a+b$ 的值为()", "options": "A. 5 或 11\nB. -5 或 -11\nC. -5\nD. -11", "subject": "代数", "analysis": "$\\because|a|=8,|\\mathbf{b}|=3$,\n\n$$\n\\therefore a= \\pm 8, b= \\pm 3 \\text {, }\n$$\n\n$\\because|\\mathrm{a}-\\mathrm{b}|=\\mathrm{b}-\\mathrm{a}$,\n\n$$\n\\therefore b \\geq a \\text {, }\n$$\n\n$\\therefore a=-8, b=3$ 或 $a=-8, b=-3$,\n\n$\\therefore a+b=-8+3=-5$ 或 $a+b=-8+(-3)=-11$,\n\n故答案为: B"} {"id": "1813", "image": [], "answer": "A", "solution": "null", "level": "七年级", "question": "已知代数式 $8 x-7$ 与 $6-2 x$ 的值互为相反数, 那么 $x$ 的值等于", "options": "A. $\\frac{1}{6}$\nB. $-\\frac{1}{6}$\nC. $\\frac{13}{10}$\nD. $-\\frac{13}{10}$", "subject": "代数", "analysis": "【解答】根据题意得: $(8 x-7)+(6-2 x)=0$,\n\n解得: $x=\\frac{1}{6}$.\n\n故答案为: A."} {"id": "1815", "image": [], "answer": "D", "solution": "null", "level": "七年级", "question": "如图, 数轴上的三个点 $A 、 B 、 C$ 表示的数分别是 $a 、 b 、 C$, 且 $|a|=|b|, A B=B C$, 则下列结论中 (1) $a b<0$; (2) $a=-b$ : (3) $a+c>0$; (4) $3 a+c=0$ 中, 正确的有()个.\n\n| $A$ | 0 | $B$ | $C$ |\n| :--- | :--- | :--- | :--- |", "options": "A. 1 个\nB. 2 个\nC. 3 个\nD. 4 个", "subject": "代数", "analysis": "根据 $a 、 b 、 C$ 在数轴上的位置, 得 $a<00$,\n\n故(1)(3)正确;\n\n$\\because|a|=|b|$,\n\n$\\therefore \\mathrm{a}, \\mathrm{b}$ 互为相反数,\n\n$\\therefore \\mathrm{a}=-\\mathrm{b}$, 故(2)正确:\n\n$\\because \\mathrm{AB}=\\mathrm{BC}, \\mathrm{a}=-\\mathrm{b}$,\n\n$\\therefore c=3 b==-3 a$,\n\n$\\therefore 3 a+c=0$, 故(4)正确,\n\n故答案为: D."} {"id": "1861", "image": [], "answer": "C", "solution": "null", "level": "七年级", "question": "有下列说法: (1) -3 是 $\\sqrt{81}$ 的平方根; (2) 7 是 $(-7)^{2}$ 的算术平方根; (3)27的立方根是 $\\pm 3$;\n(4)1的平方根是 $\\pm 1$;\n(5)0没有算术平方根. 其中正确的有 ( )", "options": "A. 1 个\nB. 2 个\nC. 3 个\nD. 4 个", "subject": "代数", "analysis": "解: (1) $\\sqrt{81}=9$, 故 -3 是 $\\sqrt{81}$ 的平方根, (1)正确;\n\n(2) $(-7)^{2}=49,7$ 是 $(-7)^{2}$ 的算术平方根, (2)正确;\n\n(3) 27 的立方根是 $3 ,(3)$ 错误;\n\n(4)1的平方根是 $\\pm 1$, (4)正确;\n\n(5) 0 的算术平方根是 $0, \\quad(5)$ 错误;\n\n故选: $C$.\n\n根据平方根与立方根的定义即可求出答案.\n\n本题考查平方根与立方根, 解题的关键是正确理解平方根与立方根, 本题属于基础题型."} {"id": "1862", "image": [], "answer": "B", "solution": "null", "level": "七年级", "question": "下列说法中, 正确的有 ( )\n\n(1)只有正数才有平方根; (2) $a$ 一定有立方根; (3) $\\sqrt{-a}$ 没意义; (4) $\\sqrt[3]{-a}=-\\sqrt[3]{a}$;\n\n(5)只有正数才有立方根.", "options": "A. 1 个\nB. 2 个\nC. 3 个\nD. 4 个", "subject": "代数", "analysis": "析】\n\n## 【分析】\n\n本题考查平方根和立方根的性质.利用平方根与立方根的性质, 对各个选项一一判断即可.\n\n## 【解答】\n\n解: 非负数都有平方根, 所以 (1)是错误的;\n\n任何数的立方根都只有一个, 所以(2)是正确的;\n\n$a>0$ 时, $\\sqrt{-a}$ 没意义, 所以所以(3)是错误的;\n\n$\\sqrt[3]{-a}=-\\sqrt[3]{a}$, 所以(4)是正确的.\n\n所以正确的有 2 个.\n\n故选 $B$."} {"id": "2509", "image": [], "answer": "B", "solution": "null", "level": "七年级", "question": "(2022$\\cdot$浙江七年级)若方程组 $\\left\\{\\begin{array}{l}2 x+y=1-3 k \\\\ x+2 y=2\\end{array}\\right.$ 的解满足 $x+y=0$, 则 $k$ 的值为 $(\\quad)$", "options": "-1\nB. 1\nC. 0\nD. 不能确定", "subject": "代数", "analysis": "方程组中两方程相加得到以 $\\mathrm{k}$ 为未知数的方程, 解方程即可得答案.\n\n【详解】解:(1)+(2), 得 $3(x+y)=3-3 k$, 由 $x+y=0$, 得 $3-3 k=0$, 解得 $k=1$, 故选: B.\n\n【点睛】本题考查了二元一次方程组的解, 利用等式的性质是解题关键."} {"id": "2517", "image": [], "answer": "B", "solution": "null", "level": "七年级", "question": "(2022$\\cdot$ 重庆礼嘉中学八年级月考)如果把 $\\frac{2 y}{2 x-3 y}$ 中的 $x$ 和 $y$ 都扩大 6 倍, 那么分式的值()", "options": "A. 扩大 6 倍\nB. 不变\nC. 缩小 6 倍\nD. 扩大 5 倍", "subject": "代数", "analysis": "根据分式的基本性质即可得.\n【详解】把 $X$ 和 $y$ 都扩大 6 倍后的分式为 $\\frac{2 \\times 6 y}{2 \\times 6 x-3 \\times 6 y}=\\frac{2 \\times 6 y}{6(2 x-3 y)}=\\frac{2 y}{2 x-3 y}$, 即分式的值不变,故选: B.\n\n【点睛】本题考查了分式的基本性质, 熟练掌握分式的基本性质是解题关键."} {"id": "2519", "image": [], "answer": "C", "solution": "null", "level": "七年级", "question": "(2022.广东八年级期末) 小明是一位密码翻译爱好者, 在他的密码手册中, 有这样一条信息: $\\boldsymbol{a}-\\boldsymbol{b}$, $x-y, x+y, \\boldsymbol{a}+\\boldsymbol{b}, x^{2}-y^{2}, a^{2}-b^{2}$ 分别对应下列六个字: 头、爱、我、汕、丽、美, 现将 $\\left(x^{2}-y^{2}\\right) a^{2}-\\left(x^{2}-y^{2}\\right) b^{2}$ 因式分解, 结果呈现的密码信息可能是 $(\\quad)$", "options": "我爱美\nB. 汕头美\nC. 我爱汕头\nD. 汕头美丽", "subject": "代数", "analysis": "先提取公因式 $\\left(x^{2}-y^{2}\\right)$, 然后再利用平方法公式因式分解可得.\n\n【详解】 $\\left(x^{2}-y^{2}\\right) a^{2}-\\left(x^{2}-y^{2}\\right) b^{2}=\\left(x^{2}-y^{2}\\right)\\left(a^{2}-b^{2}\\right)=(x+y)(x-y)(a+b)(a-b)$故对应的密码为: 我爱汕头 故选: C\n\n【点睛】本题考查因式分解, 注意, 当式子可提取公因式时, 我们在因式分解中, 往往先提取公因式."} {"id": "2520", "image": [], "answer": "B", "solution": "null", "level": "七年级", "question": "(2022-浙江杭州市$\\cdot$七年级其他模拟)已知被除式是 $x^{3}+3 x^{2}-1$, 商式是 $x$, 余式是 - 1 , 则除式是", "options": "$x^{2}+3 x-1$\n B. $x^{2}+3 x$\n C. $x^{2}-1$\n D. $x^{2}-3 x+1$", "subject": "代数", "analysis": "$x^{2}+3 x-1$\n B. $x^{2}+3 x$\n C. $x^{2}-1$\n D. $x^{2}-3 x+1$\n\n## 【答案】B\n\n分析:按照 “被除式、除式、商式和余式间的关系” 进行分析解答即可.\n\n【解析】由题意可得, 除式为: $\\left[\\left(x^{3}+2 x^{2}-1\\right)-(-1)\\right] \\div x=\\left(x^{3}+2 x^{2}\\right) \\div x=x^{2}+2 x$.故选 B.\n\n点睛: 熟知 “被除式、除式、商式和余式间的关系: 被除式=除式×商式+余式” 是解答本题的关键."} {"id": "2521", "image": [], "answer": "A", "solution": "null", "level": "七年级", "question": "(2021$\\cdot$普定县第二中学九年级二模) 已知关于 $x$ 的分式方程 $\\frac{x}{x-1}-2=\\frac{k}{x-1}$ 的解为正数, 则 $k$的取值范围为", "options": "$k<2$ 且 $k \\neq 1$\nB. $k>-2$ 且 $k \\neq-1$\nC. $k>-2$\nD. $-20$, 且 $2-k \\neq 1$, 解得: $k<2$ 且 $k \\neq 1$, 故选: $A$.\n\n【点睛】此题考查了分式方程的解,始终注意分母不为 0 这个条件."} {"id": "2535", "image": [], "answer": "B", "solution": "null", "level": "七年级", "question": "(2022$\\cdot$浙江金华市$\\cdot$七年级月考)下列等式,错误的是()", "options": "A. $\\left(x^{2} y^{3}\\right)^{2}=x^{4} y^{6}$\nB. $(-x y)^{3}=-x y^{3}$\nC. $\\left(3 m^{2} n^{2}\\right)^{2}=9 m^{4} n^{4}$\nD. $\\left(-a^{2} b^{3}\\right)^{2}=a^{4} b^{6}$\n\n", "subject": "代数", "analysis": "根据幂的乘方和积的乘方法则判断即可.\n\n【详解】解: $A 、\\left(x^{2} y^{3}\\right)^{2}=x^{4} y^{6}$, 故选项正确; $B 、(-x y)^{3}=-x^{3} y^{3}$, 故选项错误;\n\nC、 $\\left(3 m^{2} n^{2}\\right)^{2}=9 m^{4} n^{4}$, 故选项正确; $D 、\\left(-a^{2} b^{3}\\right)^{2}=a^{4} b^{6}$, 故选项正确; 故选 $B$.\n\n【点睛】本题考查了幂的乘方和积的乘方, 解题的关键是掌握运算法则."} {"id": "2543", "image": [], "answer": "B", "solution": "null", "level": "七年级", "question": "(2022$\\cdot$浙江杭州市$\\cdot$七年级期中)在多项式中,(1) $a^{2}-b^{2}+2 a b$ (2) $1-a+a^{2}$ (3) $\\frac{1}{4}-x+x^{2}$\n\n(4) $-4 x^{2}+12 x y-9 y^{2}$ 其中能用完全平方公式分解因式的个数有()", "options": "A. 1 个\nB. 2 个\nC. 3 个\nD. 4 个\n\n", "subject": "代数", "analysis": "直接利用完全平方公式分别分解因式进而判断即可.\n\n【详解】解:(1) $a^{2}-b^{2}+2 a b$ 无法运用完全平方公式分解因式;\n\n(2) $1-a+a^{2}$ 无法运用完全平方公式分解因式;\n\n(3) $\\frac{1}{4}-x+x^{2}=\\left(x-\\frac{1}{2}\\right)^{2}$ ,能运用完全平方公式分解因式;\n\n(4) $-4 x^{2}+12 x y-9 y^{2}=-(2 x-3 y)^{2}$, 能运用完全平方公式分解因式; 故选 B.\n\n【点睛】此题主要考查了公式法分解因式, 正确应用公式是解题关键."} {"id": "1315", "image": ["3106.jpg"], "answer": "B", "solution": "null", "level": "七年级", "question": "数轴上有 $O, A, B, C, D$ 五个点, 各点的位置与所表示的数如图所示, 且 $3<|d|<$ 5 . 若数轴上有一点 $M, M$ 所表示的数为 $m$, 且 $|m-d|=|m-3|$, 则关于点 $M$ 的位置, 下列叙述正确的是\n\n", "options": "A. $M$ 在 $O, B$ 之间\nB. $M$ 在 $O, C$ 之间\nC. $M$ 在 $C, D$ 之间\nD. $M$ 在 $A, D$ 之间", "subject": "度量几何学", "analysis": "由题意可得: 点 $\\mathrm{A}$ 表示的数为 -5 , 点 $\\mathrm{B}$ 表示的数为 3 , 点 $\\mathrm{C}$ 表示的数为 -1 , 点 $\\mathrm{D}$ 表示的数为 $d$, 且 $A C=B C$\n\n$\\because|m-d|=|m-3|$\n\n$\\therefore \\mathrm{MD}=\\mathrm{BD}$,\n\n又 $\\because-5<\\mathrm{d}<-1<3$\n\n$\\therefore \\mathrm{M}$ 点介于 $\\mathrm{O} 、 \\mathrm{C}$ 之间,\n\n故答案为: B."} {"id": "2426", "image": [], "answer": "D", "solution": "null", "level": "七年级", "question": "$\\odot O$ 的半径是 3, 直线 $l$ 与 $\\odot O$ 相交, 圆心 $O$ 到直线 $l$ 的距离是 $d$, 则 $d$ 不可能是 ( )", "options": "A. 0\n B. 1\n C. 2\n D. 3", "subject": "度量几何学", "analysis": "D"} {"id": "2441", "image": [], "answer": "B", "solution": "null", "level": "七年级", "question": "若两圆的半径分别是 $2 \\mathrm{~cm}$ 和 $3 \\mathrm{~cm}$, 圆心距为 $4 \\mathrm{~cm}$, 则这两个圆的位置关系是 ( )", "options": "A. 内切\n B.\n C. 外切\n D. 外离", "subject": "度量几何学", "analysis": "$\\mathrm{B}$"} {"id": "2443", "image": ["3530.jpg"], "answer": "B", "solution": "null", "level": "七年级", "question": "如图, $P A$ 为 $\\odot O$ 的切线, $A$ 为切点, $P O$ 交 $\\odot O$ 于点 $B$, $P A=3, O A=4$, 则 $\\cos \\angle A P O$ 的值为 $(\\quad)$\n\n \n\n 第 5 题", "options": "A. $\\frac{3}{4}$\n B. $\\frac{3}{5}$\n C. $\\frac{4}{5}$\n D. $\\frac{4}{3}$", "subject": "度量几何学", "analysis": "B"} {"id": "2445", "image": ["3531.jpg"], "answer": "A", "solution": "null", "level": "七年级", "question": "如图, $A B$ 是 $\\odot O$ 的直径, $B C$ 交 $\\odot O$ 于点 $D, D E \\perp A C$ 于点 $E$, 要使 $D E$ 是 $\\odot \\mathrm{O}$ 的切线, 还需补充一个条件, 则补充的条件不正确的是 $(\\quad)$\n\n \n\n 第 7 题", "options": "A. $D E=$ DO\n B. $\\mathrm{AB}=\\mathrm{AC}$\n C. $C D=D B$\n D. $\\mathrm{AC} / / \\mathrm{OD}$", "subject": "度量几何学", "analysis": "A"} {"id": "1364", "image": ["3112.jpg"], "answer": "C", "solution": "null", "level": "七年级", "question": "如图, 数轴上 $\\mathrm{C}, \\mathrm{B}$ 两点表示的数分别是 $2, \\sqrt{13}$, 且点 $\\mathrm{C}$ 是 $\\mathrm{AB}$ 的中点, 则点 $\\mathrm{A}$ 表示的数是\n\n", "options": "A. $\\sqrt{13}-4$\nB. $3-\\sqrt{13}$\nC. $4-\\sqrt{13}$\nD. $\\sqrt{13}-3$", "subject": "度量几何学", "analysis": "$\\mathrm{A}$ 表示的数是 $\\mathrm{x}$,\n$\\because$ 点 $\\mathrm{C}$ 是 $\\mathrm{AB}$ 的中点,\n\n$\\therefore A C=B C$,\n\n$\\therefore 2-x=\\sqrt{13}-2$,\n\n解得 $x=4-\\sqrt{13}$,\n\n故答案为: C."} {"id": "1388", "image": ["3120.jpg"], "answer": "A", "solution": "null", "level": "七年级", "question": "如图, 数轴上 $A, B$ 两点表示的数分别为 -1 和 $\\sqrt{3}$, 点 $\\mathrm{B}$ 关于点 $\\mathrm{A}$ 的对称点为 $\\mathrm{C}$, 则点 $\\mathrm{C}$ 所表示的数为 $(\\quad)$\n\n", "options": "A. $-2-\\sqrt{3}$\nB. $-1-\\sqrt{3}$\nC. $-2+\\sqrt{3}$\nD. $1+\\sqrt{3}$", "subject": "度量几何学", "analysis": "$\\mathrm{C}$ 表示的数是 $\\mathrm{x}$,\n\n$\\because \\mathrm{A}, \\mathrm{B}$ 两点表示的数分别为 -1 和 $\\sqrt{3}, \\mathrm{C}, \\mathrm{B}$ 两点关于点 $\\mathrm{A}$ 对称, $\\therefore \\frac{x+\\sqrt{3}}{2}=-1$,\n\n解得 $\\mathrm{x}=-2-\\sqrt{3}$.\n\n故选: A."} {"id": "1408", "image": ["3122.jpg"], "answer": "D", "solution": "null", "level": "七年级", "question": "如图, $A, B, C, D, E$ 分别是数轴上五个连续整数所对应的点, 其中有一点是原点, 数 $a$ 对应的点在 $B$ 与 $C$ 之间, 数 $b$ 对应的点在 $D$ 与 $E$ 之间, 若 $|a|+|b|=3$, 则原点可能是\n\n", "options": "A. $A$ 或 $E$\nB. $A$ 或 $B$\nC. $B$ 或 $C$\nD. $B$ 或 $E$", "subject": "度量几何学", "analysis": "$\\because \\mathrm{AB}=\\mathrm{BC}=\\mathrm{CD}=\\mathrm{DE}=1$,\n\n$\\therefore|\\mathrm{AB}|=|\\mathrm{BC}|=|\\mathrm{CD}|=|\\mathrm{DE}|=1$,\n\n$\\therefore|\\mathrm{AE}|=4$\n\n(1) 当原点在 $C$ 点时, $|a|+|b|<3$, 又因为 $|\\mathbf{a}|+|\\mathbf{b}|=3$, 所以, 原点不可能在 C 点;\n\n(2) 当原点在 $B$ 或 $E$ 时, $|a|+|b|=3$;\n\n(3)当原点在 $\\mathrm{A}$ 点时, $|\\mathrm{a}|+|\\mathrm{b}|>3$, 又因为 $|\\mathbf{a}|+|\\mathrm{b}|=3$, 所以, 原点不可能在 $\\mathrm{A}$ 点;综上所述, 此原点应是在 $\\mathrm{B}$ 或 $\\mathrm{E}$ 点.\n故答案为: D."} {"id": "2447", "image": ["3533.jpg"], "answer": "B", "solution": "null", "level": "七年级", "question": "(2 分)如图所示, 某公园入口处原有三级台阶, 每级台阶高 $20 \\mathrm{~cm}$, 宽 $30 \\mathrm{~cm}$. 为方便残疾人士, 拟将台阶改为斜坡。现台阶的起点为 $\\mathrm{A}$, 斜坡的起始点为 $\\mathrm{C}$, 若将坡角 $\\angle \\mathrm{BCA}$设计为 $30^{\\circ}$, 则 $\\mathrm{AC}$ 的长度应为 $(\\quad)$\n\n", "options": "A. $60 \\sqrt{3} \\mathrm{~cm}$\nB. $60(\\sqrt{3}-1) \\mathrm{cmC} .60 \\mathrm{~cm}$\nD. $60(\\sqrt{3}+1) \\mathrm{cm}$", "subject": "度量几何学", "analysis": "解: 过点 $B$ 作 $B D \\perp A C$ 于 $D$,\n\n根据题意得: $A D=2 \\times 30=60 \\mathrm{~cm}, B D=20 \\times 3=60 \\mathrm{~cm}$,\n\n$\\because$ 坡角 $\\angle B C A=30^{\\circ}$,\n\n$\\therefore \\mathrm{BD}: \\mathrm{CD}=1: \\sqrt{3}$,\n\n$\\therefore \\mathrm{CD}=\\sqrt{3} \\mathrm{BD}=\\sqrt{3} \\times 60=60 \\sqrt{3} \\mathrm{~cm}$,\n\n$\\therefore \\mathrm{AC}=\\mathrm{CD}-\\mathrm{AD}=60 \\sqrt{3}-60=60(\\sqrt{3}-1) \\mathrm{cm}$."} {"id": "2457", "image": ["3544.jpg"], "answer": "B", "solution": "null", "level": "七年级", "question": "(2 分) 如图所示, $\\triangle \\mathrm{ABC}$ 的顶点是正方形网格的格点, 则 $\\sin \\mathrm{A}$ 的值为 ( )\n\n", "options": "A. $\\frac{1}{2}$\nB. $\\frac{\\sqrt{5}}{5}$\nC. $\\frac{\\sqrt{10}}{10}$\nD. $\\frac{2 \\sqrt{5}}{5}$", "subject": "度量几何学", "analysis": "解: 如图: 连接 $\\mathrm{CD}$ 交 $\\mathrm{AB}$ 于 $\\mathrm{O}$,\n根据网格的特点, $\\mathrm{CD} \\perp \\mathrm{AB}$,\n\n在 Rt $\\triangle A O C$ 中,\n\n$\\mathrm{CO}=\\sqrt{1^{2}+1^{2}}=\\sqrt{2}$\n\n$\\mathrm{AC}=\\sqrt{1^{2}+3^{2}}=\\sqrt{10}$\n\n则 $\\sin \\mathrm{A}=\\frac{\\mathrm{OC}}{\\mathrm{AC}}=\\frac{\\sqrt{2}}{\\sqrt{10}}=\\frac{\\sqrt{5}}{5}$."} {"id": "2466", "image": ["3556.jpg"], "answer": "B", "solution": "null", "level": "七年级", "question": "(2 分)如图, 已知: $45^{\\circ}<\\angle \\mathrm{A}<90^{\\circ}$, 则下列各式成立的是()\n\n", "options": "A. $\\sin A=\\cos A$\nB. $\\sin A>\\cos A$\nC. $\\sin A>\\tan A$\nD. $\\sin \\mathrm{A}<\\cos \\mathrm{A}$", "subject": "度量几何学", "analysis": "解: $\\because 45^{\\circ}<\\mathrm{A}<90^{\\circ}$,\n\n$\\therefore$ 根据 $\\sin 45^{\\circ}=\\cos 45^{\\circ}, \\sin \\mathrm{A}$ 随角度的增大而增大, $\\cos \\mathrm{A}$ 随角度的增大而减小,当 $\\angle A>45^{\\circ}$ 时, $\\sin A>\\cos A$,"} {"id": "2470", "image": ["3560.jpg"], "answer": "A", "solution": "null", "level": "七年级", "question": "(2 分) 小明想测量一棵树的高度, 他发现树的影子恰好落在地面和一斜坡上, 如图, 此时测得地面上的影长为 8 米, 坡面上的影长为 4 米. 已知斜坡的坡角为 $30^{\\circ}$, 同一时刻, 一根长为 1 米且垂直于地面放置的标杆在地面上的影长为 2 米, 则树的高度为\n\n", "options": "A. $(6+\\sqrt{3})$ 米\nB. 12 米\nC. $(4-2 \\sqrt{3})$ 米\nD. 10 米", "subject": "度量几何学", "analysis": "解: 延长 $\\mathrm{AC}$ 交 $\\mathrm{BF}$ 延长线于 $\\mathrm{D}$ 点,\n\n则 $\\angle \\mathrm{CFE}=30^{\\circ}$, 作 $\\mathrm{CE} \\perp \\mathrm{BD}$ 于 $\\mathrm{E}$,\n\n在 Rt $\\triangle \\mathrm{CFE}$ 中, $\\angle \\mathrm{CFE}=30^{\\circ}, \\mathrm{CF}=4 \\mathrm{~m}$,\n\n$\\therefore \\mathrm{CE}=2$ (米), $\\mathrm{EF}=4 \\cos 30^{\\circ}=2 \\sqrt{3}$ (米),\n\n在 Rt $\\triangle \\mathrm{CED}$ 中,\n\n$\\because$ 同一时刻, 一根长为 1 米、垂直于地面放置的标杆在地面上的影长为 2 米, $\\mathrm{CE}=2$\n\n(米), $\\mathrm{CE}: \\mathrm{DE}=1: 2$,\n\n$\\therefore \\mathrm{DE}=4($ 米 $)$,\n\n$\\therefore \\mathrm{BD}=\\mathrm{BF}+\\mathrm{EF}+\\mathrm{ED}=12+2 \\sqrt{3}$ (米)\n\n在 Rt $\\triangle \\mathrm{ABD}$ 中, $\\mathrm{AB}=\\frac{1}{2} \\mathrm{BD}=\\frac{1}{2}(12+2 \\sqrt{3})=(\\sqrt{3}+6)$ (米)."} {"id": "2471", "image": ["3561.jpg"], "answer": "B", "solution": "null", "level": "七年级", "question": "(2 分)江郎山位我国典型的丹霞地貌景观,被称为“中国丹霞第一奇峰”。九年级(2)班课题学习小组的同学要测量三块巨石中的最左边的“郎峰”的高度, 他们在山脚的平地上选取一处观测点 $\\mathrm{C}$, 测得 $\\angle B C D=28^{\\circ}, \\angle A C D=48^{\\circ} 25^{\\prime}$, 已知从观测点 $C$ 到“郎峰”脚 $B$ 的垂直高度为 322 米,如图所示,那么“郎峰”AB 的高度约为()\n\n", "options": "A. 152 米\nB. 361 米\nC. 202 米\nD. 683 米", "subject": "度量几何学", "analysis": "解: 在直角 $\\triangle \\mathrm{BCD}$ 中, $\\tan \\angle \\mathrm{BCD}=\\frac{\\mathrm{BD}}{\\mathrm{CD}}$,\n\n则 $\\mathrm{CD}=\\frac{\\mathrm{BD}}{\\tan \\angle \\mathrm{BCD}}=\\frac{322}{\\tan 28^{\\circ}}=\\frac{322}{0.5317} \\approx 605.6$ (米).\n\n$\\therefore \\mathrm{AD}=\\mathrm{CD} \\times \\tan \\angle \\mathrm{ACD}=\\mathrm{CD} \\times \\tan 48^{\\circ} 25^{\\prime}=605.6 \\times 1.127 \\approx 682.5$ (米).\n\n$\\therefore \\mathrm{AB}=\\mathrm{AD}-\\mathrm{BD}=682.5-322 \\approx 361$ (米)."} {"id": "1457", "image": ["3139.jpg"], "answer": "C", "solution": "null", "level": "七年级", "question": "如图是 $5 \\times 5$ 方格子 (每个小正方格的边长为 1 个单位长度)、图中阴影部分是正方形, 则正方形的边长为\n\n", "options": "3\nB. $\\sqrt{7}$\nC. $\\sqrt{13}$\nD. 5", "subject": "度量几何学", "analysis": "由题意可知正方形的边长为: $\\sqrt{2^{2}+3^{2}}=\\sqrt{13}$.故答案为: $\\mathrm{C}$"} {"id": "1478", "image": ["3146.jpg"], "answer": "C", "solution": "null", "level": "七年级", "question": "已知数 $\\mathrm{a}, \\mathrm{b}, \\mathrm{c}$ 在数轴上的位置如图所示, 化简 $|\\mathrm{a}+\\mathrm{b}|-|\\mathrm{a}-\\mathrm{b}|+|\\mathrm{a}+\\mathrm{c}|$ 的结果为\n\n", "options": "$-\\mathrm{a}-\\mathrm{c}$\nB. $-\\mathrm{a}-\\mathrm{b}-\\mathrm{c}$\nC. $-a-2 b-c$\nD. $a-2 b+c$", "subject": "度量几何学", "analysis": "通过数轴得到 $a<0, c>0, b>0,|a|>|c|>|b|$,\n\n$\\therefore \\mathrm{a}+\\mathrm{b}<0, \\mathrm{a}-\\mathrm{b}<0, \\mathrm{a}+\\mathrm{c}<0$\n\n$\\therefore|a+b|-|a-b|+|a+c|=-a-b+a-b-a-c=-a-2 b-c$.\n\n故答案为: C."} {"id": "1480", "image": ["3148.jpg"], "answer": "A", "solution": "null", "level": "七年级", "question": "已知有理数 $a, b, \\quad c$ 在数轴上的位置如图, 且 $|c|>|a|>|b|$, 则 $|a+b|-2|c-b|+$ $|a+c|=(\\quad)$.\n\n", "options": "$c-b$\nB. 0\nC. $3 b-3 c$\nD. $2 a+3 b-c$", "subject": "度量几何学", "analysis": "$\\because|c|>|a|>|b|$, 且 $c<0, b<0, a>0$,\n\n$\\therefore a+b>0, \\quad c-b<0, a+c<0$,\n\n$\\therefore|a+b|-2|c-b|+|a+c|=a+b-2(-c+b)+(-a-c)$\n\n$=a+b+2 c-2 b-a-c=-b+c$.\n\n故答案为: A."} {"id": "1504", "image": ["3155.jpg"], "answer": "A", "solution": "null", "level": "七年级", "question": "已知数轴上的四点 $P, Q, R, S$ 对应的数分别为 $p, q, r, s$. 且 $p, q$, $r, s$ 在数轴上的位置如图所示, 若 $r-p=10, s-p=12, s-q=9$, 则 $r-q$ 等于 $(\\quad)$.\n\n", "options": "7\nB. 9\nC. 11\nD. 13", "subject": "度量几何学", "analysis": "由数轴可知: $\\mathrm{p}<\\mathrm{r}, \\mathrm{p}<\\mathrm{s}, \\mathrm{q}<\\mathrm{s}, \\mathrm{q}<\\mathrm{r}$,\n\n$\\because \\mathrm{r}-\\mathrm{p}=10, \\mathrm{~s}-\\mathrm{p}=12, \\mathrm{~s}-\\mathrm{q}=9$,\n\n$\\therefore \\mathrm{r}-\\mathrm{q}=(\\mathrm{r}-\\mathrm{p})-(\\mathrm{s}-\\mathrm{p})+(\\mathrm{s}-\\mathrm{q})=10-12+9=7$.\n\n故答案为: A."} {"id": "1530", "image": ["3164.jpg"], "answer": "C", "solution": "null", "level": "七年级", "question": "如图, 长为 $\\mathrm{y}(\\mathrm{cm})$, 宽为 $\\mathrm{x}(\\mathrm{cm})$ 的大长方形被分割为 7 小块, 除阴影 $\\mathrm{A}, \\mathrm{B}$ 外, 其余 5 块是形状、大小完全相同的小长方形,其较短的边长为 $4 \\mathrm{~cm}$ ,下列说法中正确的有( )\n\n(1)小长方形的较长边为 $\\mathrm{y}-12$;\n\n(2) 阴影 $\\mathrm{A}$ 的较短边和阴影 $\\mathrm{B}$ 的较短边之和为 $\\mathrm{x}-\\mathrm{y}+4$;\n\n(3)若 $\\mathrm{x}$ 为定值, 则阴影 $\\mathrm{A}$ 和阴影 $\\mathrm{B}$ 的周长和为定值;\n\n(4)当 $\\mathrm{x}=20$ 时, 阴影 $\\mathrm{A}$ 和阴影 $\\mathrm{B}$ 的面积和为定值.\n\n", "options": "1 个\nB. 2 个\nC. 3 个\nD. 4 个", "subject": "度量几何学", "analysis": "(1) $\\because$ 大长方形的长为 $\\mathrm{ycm}$, 小长方形的宽为 $4 \\mathrm{~cm}$,\n\n$\\therefore$ 小长方形的长为 $\\mathrm{y}-3 \\times 4=(\\mathrm{y}-12) \\mathrm{cm}$, 说法(1)正确;\n\n(2) $\\because$ 大长方形的宽为 $\\mathrm{xcm}$, 小长方形的长为 $(\\mathrm{y}-12) \\mathrm{cm}$, 小长方形的宽为 $4 \\mathrm{~cm}$,\n\n$\\therefore$ 阴影 $\\mathrm{A}$ 的较短边为 $\\mathrm{x}-2 \\times 4=(\\mathrm{x}-8) \\mathrm{cm}$, 阴影 $\\mathrm{B}$ 的较短边为 $\\mathrm{x}-(\\mathrm{y}-12)=(\\mathrm{x}-\\mathrm{y}+12) \\mathrm{cm}$,\n\n$\\therefore$ 阴影 $\\mathrm{A}$ 的较短边和阴影 $\\mathrm{B}$ 的较短边之和为 $\\mathrm{x}-8+\\mathrm{x}-\\mathrm{y}+12=(2 \\mathrm{x}+4-\\mathrm{y}) \\mathrm{cm}$, 说法(2)错误;\n\n(3) $\\because$ 阴影 $\\mathrm{A}$ 的较长边为 $(\\mathrm{y}-12) \\mathrm{cm}$, 较短边为 $(\\mathrm{x}-8) \\mathrm{cm}$, 阴影 $\\mathrm{B}$ 的较长边为 $3 \\times 4=12 \\mathrm{~cm}$, 较短边为 $(x-y+12) \\mathrm{cm}$,\n\n$\\therefore$ 阴影 $\\mathrm{A}$ 的周长为 $2(\\mathrm{y}-12+\\mathrm{x}-8)=2(\\mathrm{x}+\\mathrm{y}-20) \\mathrm{cm}$, 阴影 $\\mathrm{B}$ 的周长为 $2(12+\\mathrm{x}-\\mathrm{y}+12)=2(\\mathrm{x}$ $-\\mathrm{y}+24) \\mathrm{cm}$,\n\n$\\therefore$ 阴影 $\\mathrm{A}$ 和阴影 $\\mathrm{B}$ 的周长之和为 $2(\\mathrm{x}+\\mathrm{y}-20)+2(\\mathrm{x}-\\mathrm{y}+24)=2(2 \\mathrm{x}+4)$,\n\n$\\therefore$ 若 $\\mathrm{x}$ 为定值, 则阴影 $\\mathrm{A}$ 和阴影 $\\mathrm{B}$ 的周长之和为定值, 说法(3)正确;\n\n(4) $\\because$ 阴影 $\\mathrm{A}$ 的较长边为 $(\\mathrm{y}-12) \\mathrm{cm}$, 较短边为 $(\\mathrm{x}-8) \\mathrm{cm}$, 阴影 $\\mathrm{B}$ 的较长边为 $3 \\times 4=12 \\mathrm{~cm}$, 较短边为 $(x-y+12) \\mathrm{cm}$,\n\n$\\therefore$ 阴影 $A$ 的面积为 $(y-12)(x-8)=(x y-12 x-8 y+96) \\mathrm{cm}^{2}$, 阴影 $B$ 的面积为 $12(x-y+12)=$ $(12 x-12 y+144) \\mathrm{cm}^{2}$,\n\n$\\therefore$ 阴影 $\\mathrm{A}$ 和阴影 B 的面积之和为 $\\mathrm{xy}-12 \\mathrm{x}-8 \\mathrm{y}+96+12 \\mathrm{x}-12 \\mathrm{y}+144=(\\mathrm{xy}-20 \\mathrm{y}+240) \\mathrm{cm}^{2}$,\n\n当 $x=20$ 时, $x y-20 y+240=240 \\mathrm{~cm}^{2}$, 说法(4)正确,\n\n综上所述, 正确的说法有(1)(3)(4), 共 3 个,\n\n故答案为: C."} {"id": "1532", "image": ["3165.jpg", "3166.jpg"], "answer": "D", "solution": "null", "level": "七年级", "question": "把如图 1 的两张大小相同的长方形卡片放置在图 2 与图 3 中的两个相同大长方形中, 已知这两个大长方形的长比宽长 $20 \\mathrm{~cm}$, 若记图 2 中阴影部分的周长为 $\\mathrm{C}_{1}$, 图 3 中阴影部分的周长为 $\\mathrm{C}_{2}$, 那 $么 \\mathrm{C}_{1}-\\mathrm{C}_{2}=(\\quad)$\n\n\n\n[社|\n\n图2\n\n\n\n图3", "options": "$10 \\mathrm{~cm}$\nB. $20 \\mathrm{~cm}$\nC. $30 \\mathrm{~cm}$\nD. $40 \\mathrm{~cm}$", "subject": "度量几何学", "analysis": "2 与图 3 中的大长方形的宽为 $\\mathrm{acm}$, 则长为 $(a+20) \\mathrm{cm}$,\n\n图 1 中的长方形长为 $\\mathrm{xcm}$, 宽为 $\\mathrm{ycm}$,\n\n由图 2 可知: $C_{1}=(a+a+20) \\times 2=4 a+40$;\n\n由图 3 可知: $x+y=a+20$,\n\n$C_{2}=2(a+20)+2(a-x)+2(a-y)$,\n\n$=2 a+40+4 a-2(x+y)$,\n\n$=6 a+40-2(a+20)$,\n\n$=4 a$,\n\n则 $C_{1}-C_{2}=4 a+40-4 a=40(\\mathrm{~cm})$,\n\n故答案为: D."} {"id": "1580", "image": ["3180.jpg"], "answer": "C", "solution": "null", "level": "七年级", "question": "在长方形 $A B C D$ 中, 放入 6 个形状大小完全相同的小长方形, 所标尺寸如图所示, 则小长方形的宽 $A E$ 的长度为 $(\\quad) \\mathrm{cm}$.\n\n", "options": "1\nB. 1.6\nC. 2\nD. 2.5", "subject": "度量几何学", "analysis": "$\\mathrm{AE}=\\mathrm{xcm}$, 得出大长方形的宽为 $(6+2 \\mathrm{x}) \\mathrm{cm}$, 小长方形的长为 $(14-3 \\mathrm{x}) \\mathrm{cm}$, 从而得出大长方形的宽为 $(\\mathrm{x}+14-3 \\mathrm{x}) \\mathrm{cm}$, 根据用两个不同的式子表示同一个量, 则这两个式子应该相等\n\n依题意, 得: $6+2 x=x+(14-3 x)$,\n\n解得: $x=2$\n\n故答案为: C."} {"id": "1651", "image": ["3186.jpg"], "answer": "C", "solution": "null", "level": "七年级", "question": "如图, 长为 $\\mathrm{y}(\\mathrm{cm})$, 宽为 $\\mathrm{x}(\\mathrm{cm})$ 的大长方形被分割为 7 小块, 除阴影 $\\mathrm{A}, \\mathrm{B}$ 外, 其余 5 块是形状、大小完全相同的小长方形,其较短的边长为 $4 \\mathrm{~cm}$, 下列说法中正确的有()\n\n(1) 小长方形的较长边为 $\\mathrm{y}-12$;\n\n(2) 阴影 $\\mathrm{A}$ 的较短边和阴影 $\\mathrm{B}$ 的较短边之和为 $x-y+4$;\n\n(3)若 $\\mathrm{x}$ 为定值, 则阴影 $\\mathrm{A}$ 和阴影 $\\mathrm{B}$ 的周长和为定值;\n\n(4) 当 $x=20$ 时, 阴影 $A$ 和阴影 $B$ 的面积和为定值.\n\n", "options": "1 个\nB. 2 个\nC. 3 个\nD. 4 个", "subject": "度量几何学", "analysis": ") $\\because$ 大长方形的长为 $\\mathrm{ycm}$, 小长方形的宽为 $4 \\mathrm{~cm}$,\n\n$\\therefore$ 小长方形的长为 $\\mathrm{y}-3 \\times 4=(\\mathrm{y}-12) \\mathrm{cm}$, 说法(1)正确;\n\n(2) $\\because$ 大长方形的宽为 $\\mathrm{xcm}$, 小长方形的长为 $(\\mathrm{y}-12) \\mathrm{cm}$, 小长方形的宽为 $4 \\mathrm{~cm}$,\n$\\therefore$ 阴影 $\\mathrm{A}$ 的较短边为 $\\mathrm{x}-2 \\times 4=(\\mathrm{x}-8) \\mathrm{cm}$, 阴影 $\\mathrm{B}$ 的较短边为 $\\mathrm{x}-(\\mathrm{y}-12)=(\\mathrm{x}-\\mathrm{y}+12) \\mathrm{cm}$,\n\n$\\therefore$ 阴影 $\\mathrm{A}$ 的较短边和阴影 $\\mathrm{B}$ 的较短边之和为 $\\mathrm{x}-8+\\mathrm{x}-\\mathrm{y}+12=(2 \\mathrm{x}+4-\\mathrm{y}) \\mathrm{cm}$, 说法(2)错误;\n\n(3) $\\because$ 阴影 $\\mathrm{A}$ 的较长边为 $(\\mathrm{y}-12) \\mathrm{cm}$, 较短边为 $(\\mathrm{x}-8) \\mathrm{cm}$, 阴影 $\\mathrm{B}$ 的较长边为 $3 \\times 4=12 \\mathrm{~cm}$, 较短边为 $(x-y+12) \\mathrm{cm}$,\n\n$\\therefore$ 阴影 $\\mathrm{A}$ 的周长为 $2(\\mathrm{y}-12+\\mathrm{x}-8)=2(\\mathrm{x}+\\mathrm{y}-20) \\mathrm{cm}$, 阴影 $\\mathrm{B}$ 的周长为 $2(12+\\mathrm{x}-\\mathrm{y}+12)=2(\\mathrm{x}$ $-\\mathrm{y}+24) \\mathrm{cm}$,\n\n$\\therefore$ 阴影 $\\mathrm{A}$ 和阴影 $\\mathrm{B}$ 的周长之和为 $2(\\mathrm{x}+\\mathrm{y}-20)+2(\\mathrm{x}-\\mathrm{y}+24)=2(2 \\mathrm{x}+4)$,\n\n$\\therefore$ 若 $\\mathrm{x}$ 为定值, 则阴影 $\\mathrm{A}$ 和阴影 $\\mathrm{B}$ 的周长之和为定值, 说法(3)正确;\n\n(4) $\\because$ 阴影 $\\mathrm{A}$ 的较长边为 $(\\mathrm{y}-12) \\mathrm{cm}$, 较短边为 $(\\mathrm{x}-8) \\mathrm{cm}$, 阴影 $\\mathrm{B}$ 的较长边为 $3 \\times 4=12 \\mathrm{~cm}$, 较短边为 $(x-y+12) \\mathrm{cm}$,\n\n$\\therefore$ 阴影 A 的面积为 $(\\mathrm{y}-12)(\\mathrm{x}-8)=(\\mathrm{xy}-12 \\mathrm{x}-8 \\mathrm{y}+96) \\mathrm{cm}^{2}$, 阴影 $\\mathrm{B}$ 的面积为 $12(\\mathrm{x}-\\mathrm{y}+12)=$ $(12 x-12 y+144) \\mathrm{cm}^{2}$\n\n$\\therefore$ 阴影 A 和阴影 B 的面积之和为 $\\mathrm{xy}-12 \\mathrm{x}-8 \\mathrm{y}+96+12 \\mathrm{x}-12 \\mathrm{y}+144=(\\mathrm{xy}-20 \\mathrm{y}+240) \\mathrm{cm}^{2}$,\n\n当 $x=20$ 时, $x y-20 y+240=240 \\mathrm{~cm}^{2}$, 说法(4)正确,\n\n综上所述, 正确的说法有①(3)(4), 共 3 个,\n\n故答案为: C."} {"id": "1670", "image": ["3194.jpg"], "answer": "B", "solution": "null", "level": "七年级", "question": "已知有理数 $a 、 b 、 c$ 在数轴上的对应点的位置如图所示, 则下列关系中, 正确的是 ( )\n\n", "options": "$a+b+c>0$\nB. $a b c>0$\nC. $a+b-c>0$\nD. $0<\\frac{a b}{c}<1$", "subject": "度量几何学", "analysis": "由数轴知, $a<-20, \\quad \\mathrm{a}+\\mathrm{b}-\\mathrm{c}<0, \\quad \\frac{a b}{c}>1$,\n\n故答案为: B."} {"id": "1675", "image": [], "answer": "B", "solution": "null", "level": "七年级", "question": "一段跑道长 100 米, 两端分别记为点 A、B. 甲、乙两人分别从 A、B 两端同时出发, 在这段跑道上来回练习跑步, 甲跑步的速度是 $6 \\mathrm{~m} / \\mathrm{s}$, 乙跑步的速度为 $4 \\mathrm{~m} / \\mathrm{s}$, 练习了足够长时间, 他们经过了多次相遇, 相遇点离 A 端不可能是 $(\\quad)$", "options": "60 米\nB. 0 米\nC. 20 米\nD. 100 米", "subject": "度量几何学", "analysis": "步时间为 $\\mathrm{ts} ,$\n\n第一次相遇 $: 100=6 t+4 t$\n$t=10$,\n\n$\\therefore$ 相遇点距 A 为 60 米,故 A 不符合题意;\n\n第二次相遇: $300=6 t+4 t ,$\n\n$t=30$,\n\n$6 \\times 30=180$ (米),\n\n$\\therefore$ 相遇点距 A 为 20 米,故 C 不符合题意;\n\n第三次相遇 $: 500=6 t+4 t$,\n\n$t=50$,\n\n$6 \\times 50=300$ (米),\n\n$\\therefore$ 相遇点距 A 为 100 米, 选项 D 说法符合题意,不符合题意;\n\n第四次相遇 $: 700=6 t+4 t$\n\n$t=70$,\n\n$6 \\times 70=420$ (米),\n\n$\\therefore$ 相遇点距 A 为 20 米;\n\n第五次相遇 $: 900=6 t+4 t$\n\n$t=90$,\n\n$6 \\times 90=540($ 米 $)$,\n\n$\\therefore$ 相遇点距 A 为 60 米;\n\n综上, 相遇点离 $\\mathrm{A}$ 端不可能是 0 米,\n\n故答案为:\nB."} {"id": "1699", "image": ["3197.jpg"], "answer": "B", "solution": "null", "level": "七年级", "question": "如图, 按照上北下南, 左西右东的规定画出方向十字线, $\\angle \\mathrm{AOE}=\\mathrm{m}^{\\circ}, \\angle \\mathrm{EOF}=90^{\\circ}, \\mathrm{OM} 、 \\mathrm{ON}$分别平分 $\\angle \\mathrm{AOE}$ 和 $\\angle \\mathrm{BOF}$, 下面说法:\n\n(1)点 $\\mathrm{E}$ 位于点 $\\mathrm{O}$ 的北偏西 $\\mathrm{m}^{\\circ}$; (2)图中互余的角有 4 对; (3)若 $\\angle \\mathrm{BOF}=4 \\angle \\mathrm{AOE}$, 则 $\\angle \\mathrm{DON}=$ $54^{\\circ}$; (4)若 $\\frac{\\angle M O N}{\\angle A O E+\\angle B O F}=n$, 则 $\\mathrm{n}$ 的倒数是 $\\frac{2}{3}$, 其中正确有 $(\\quad)$\n\n", "options": "A. 3 个\nB. 2 个\nC. 1 个\nD. 0 个", "subject": "度量几何学", "analysis": "$\\because \\angle \\mathrm{AOE}=\\mathrm{m}^{\\circ}$,\n\n$\\therefore \\angle \\mathrm{EOD}=90^{\\circ}-\\mathrm{m}^{\\circ}$,\n\n$\\therefore$ 点 $\\mathrm{E}$ 位于点 $\\mathrm{O}$ 的北偏西 $90^{\\circ}-\\mathrm{m}^{\\circ}$, 故 (1)错误;\n\n$\\because \\angle \\mathrm{EOF}=90^{\\circ}$,\n\n$\\therefore \\angle \\mathrm{EOD}+\\angle \\mathrm{DOF}=90^{\\circ}, \\angle \\mathrm{AOE}+\\angle \\mathrm{BOF}=90^{\\circ}$,\n\n$\\because \\angle \\mathrm{AOD}=\\angle \\mathrm{BOD}=90^{\\circ}$,\n\n$\\therefore \\angle \\mathrm{AOE}+\\angle \\mathrm{EOD}=90^{\\circ}, \\angle \\mathrm{DOF}+\\angle \\mathrm{FOB}=90^{\\circ}$,\n\n$\\angle \\mathrm{AOM}+\\angle \\mathrm{MOD}=90^{\\circ}, \\angle \\mathrm{BON}+\\angle \\mathrm{DON}=90^{\\circ}$,\n\n$\\because \\mathrm{OM} 、 \\mathrm{ON}$ 分别平分 $\\angle \\mathrm{AOE}$ 和 $\\angle \\mathrm{BOF}$,\n\n$\\therefore \\angle \\mathrm{AOM}=\\angle \\mathrm{EOM}, \\angle \\mathrm{BON}=\\angle \\mathrm{FON}$,\n$\\therefore \\angle \\mathrm{EOM}+\\angle \\mathrm{MOD}=90^{\\circ}, \\angle \\mathrm{FON}+\\angle \\mathrm{DON}=90^{\\circ}$,\n\n$\\therefore$ 图中互余的角共有 8 对, 故(2)错误;\n\n$\\because \\angle \\mathrm{BOF}=4 \\angle \\mathrm{AOE}, \\angle \\mathrm{AOE}+\\angle \\mathrm{BOF}=90^{\\circ}$,\n\n$\\therefore \\angle \\mathrm{BOF}=72^{\\circ}$,\n\n$\\therefore \\angle \\mathrm{BON}=36^{\\circ}$,\n\n$\\therefore \\angle \\mathrm{DON}=90^{\\circ}-36^{\\circ}=54^{\\circ}$; 故(3)正确;\n\n$\\because \\angle \\mathrm{AOE}+\\angle \\mathrm{BOF}=90^{\\circ}$\n\n$\\therefore \\angle \\mathrm{MOE}+\\angle \\mathrm{NOF}=\\frac{1}{2}(\\angle A O E+\\angle B O F)=\\frac{1}{2} \\times 90^{\\circ}=45^{\\circ}$,\n\n$\\therefore \\angle M O N=90^{\\circ}+45^{\\circ}=135^{\\circ}$\n\n$\\therefore \\frac{\\angle M O N}{\\angle A O E+\\angle B O F}=\\frac{135^{\\circ}}{90^{\\circ}}=\\frac{3}{2}=n$,\n\n$\\therefore \\mathrm{n}$ 的倒数是 $\\frac{2}{3}$, 故(4)正确;\n\n$\\therefore$ 正确的选项有(3)(4), 共 2 个;\n\n故答案为: $\\mathrm{B}$."} {"id": "1709", "image": ["3208.jpg"], "answer": "B", "solution": "null", "level": "七年级", "question": "如图, 点 $\\mathrm{A}, \\mathrm{O}, \\mathrm{B}$ 在一条直线上, $\\mathrm{OE} \\perp \\mathrm{AB}$ 于点 $\\mathrm{O}$, 如果 $\\angle 1$ 与 $\\angle 2$ 互余, 那么图中相等的角有\n\n", "options": "A. 6 对\nB. 5 对\nC. 4 对\nD. 3 对", "subject": "度量几何学", "analysis": "图中相等的角有 $\\angle 1=\\angle C O A, \\angle 2=\\angle B O D, \\angle A O E=\\angle B O E, \\angle C O D=\\angle B O E, \\angle C O D=$ $\\angle A O E$, 共 5 对\n\n故答案为: B"} {"id": "1715", "image": ["3220.jpg"], "answer": "A", "solution": "null", "level": "七年级", "question": "如图, 点 $\\mathrm{C} 、 \\mathrm{D}$ 分别是线段 $\\mathrm{AB}$ 上两点 ( $\\mathrm{CD}>\\mathrm{AC}, \\mathrm{CD}>\\mathrm{BD})$, 用圆规在线段 $\\mathrm{CD}$ 上截取 $\\mathrm{CE}=\\mathrm{AC}$, $\\mathrm{DF}=\\mathrm{BD}$, 若点 $\\mathrm{E}$ 与点 $\\mathrm{F}$ 恰好重合, $\\mathrm{AB}=8$, 则 $\\mathrm{CD}=(\\quad)$\n\n", "options": "A. 4\nB. 4.5\nC. 5\nD. 5.5", "subject": "度量几何学", "analysis": "$\\because \\mathrm{CE}=\\mathrm{AC}, \\mathrm{DF}=\\mathrm{BD}$,\n\n$\\therefore \\mathrm{CE}=\\frac{1}{2} \\mathrm{AE}, \\quad \\mathrm{DF}=\\frac{1}{2} \\mathrm{FB}$,\n\n$\\therefore \\mathrm{CD}=\\mathrm{CE}+\\mathrm{DF}=\\frac{1}{2} \\mathrm{AE}+\\frac{1}{2} \\mathrm{FB}=\\frac{1}{2} \\mathrm{AB}=4$,\n故答案为: A."} {"id": "1359", "image": [], "answer": "C", "solution": "null", "level": "七年级", "question": "近几年宁波市常住人口总量持续增长, 根据第七次全国人口普查数据显示宁波市常住人口约为 940.43 万人, 940.43 万精确到()", "options": "十分位\nB. 百分位\nC. 百位\nD. 万位", "subject": "计数", "analysis": "940.43 万精确到百位.\n\n故答案为: C."} {"id": "2420", "image": [], "answer": "B", "solution": "null", "level": "七年级", "question": "(3 分) 在李咏主持的“幸运 52 ”栏目中, 曾有一种竟猜游戏, 游戏规则是: 在 20 个商标牌中, 有 5 个商标牌的背面注明了一定的奖金, 其余商标牌的背面是一张“哭脸”, 若翻到“哭\n脸”就不获奖, 参与这个游戏的观众有三次翻牌的机会, 且翻过的牌不能再翻. 有一位观众已翻牌两次,一次获奖,一次不获奖,那么这位观众第三次翻牌获奖的概率是()", "options": "A. $\\frac{1}{5}$\nB. $\\frac{2}{9}$\nC. $\\frac{1}{4}$\nD. $\\frac{5}{18}$", "subject": "计数", "analysis": "解:根据题意, 得\n\n全部还有 18 个商标牌, 其中还有 4 个中奖, 所以第三次翻牌获奖的概率是 $\\frac{4}{18}=\\frac{2}{9}$."} {"id": "2422", "image": ["3518.jpg"], "answer": "B", "solution": "null", "level": "七年级", "question": "(3 分) 一只小鸟自由自在地在空中飞行, 然后随意落在图中所示的某个方格中 (每个方格除颜色外完全一样), 那么小鸟停在黑色方格中的概率是()\n\n", "options": "A. $\\frac{1}{2}$\nB. $\\frac{1}{3}$\nC. $\\frac{1}{4}$\nD. $\\frac{1}{5}$", "subject": "计数", "analysis": "解: 图上共有 15 个方格, 黑色方格为 5 个,\n\n小鸟最终停在黑色方格上的概率是 $\\frac{5}{15}$, 即 $\\frac{1}{3}$."} {"id": "2425", "image": [], "answer": "D", "solution": "null", "level": "七年级", "question": "(3 分) 为了估计湖中有多少条鱼, 先从湖中捕捉 50 条鱼做记号, 然后放回湖里, 经过一段时间, 等带记号的鱼完全混于鱼群中之后, 再捕捞第二次, 鱼共 200 条, 有 10 条做了记号, 则估计湖里有多少条鱼 $(\\quad)$", "options": "A. 400 条\nB. 500 条\nC. 800 条\nD. 1000 条", "subject": "计数", "analysis": "解:设湖中有 $\\mathrm{x}$ 条鱼, 则 200: $10=x: 50$, 解得 $\\mathrm{x}=1000$ (条)."} {"id": "1647", "image": [], "answer": "A", "solution": "null", "level": "七年级", "question": "某次数学竟赛共有 20 道题, 已知做对一道得 4 分, 做错一道或者不做扣 1 分, 某同学最后的得分是 50 分, 则他做对 $(\\quad)$ 道题.", "options": "14\nB. 15\nC. 16\nD. 17", "subject": "计数", "analysis": "设他做对 $\\mathrm{x}$ 道题,\n\n根据题意得: $4 x-(20-x)=50$,\n\n$\\therefore \\mathrm{x}=14$,\n\n$\\therefore$ 他做对 14 道题,\n\n故答案为: A."} {"id": "2518", "image": [], "answer": "D", "solution": "null", "level": "七年级", "question": "(2021$\\cdot$江苏八年级期中) 为了解 2019 年泰兴市八年级学生的视力情况, 从中随机调查了 500 名学生的视力情况. 下列说法正确的是 ( )", "options": "2016 年泰兴市八年级学生是总体\nB. 每一名八年级学生是个体\nC. 500 名八年级学生是总体的一个样本\nD. 样本容量是 500", "subject": "计数", "analysis": "总体是指考查的对象的全体, 个体是总体中的每一个考查的对象, 样本是总体中所抽取的一部分个体, 而样本容量则是指样本中个体的数目. 我们在区分总体、个体、样本、样本容量, 这四个概念时, 首先找出考查的对象. 从而找出总体、个体. 再根据被收集数据的这一部分对象找出样本,最后再根据样本确定出样本容量.\n\n【详解】A. 2019 年泰兴市八年级学生的视力情况是总体, 故 A 错误; B. 每一名八年级学生的视力情况是个体, 故 B 错误; C. 从中随机调查了 500 名学生的视力情况是一个样本, 故 C 错误;\n\nD. 样本容量是 500, 故 D 正确; 故选: D.\n\n【点睛】此题考查总体、个体、样本、样本容量, 解题关键在于掌握它们的定义及区别."} {"id": "2545", "image": ["3627.jpg", "3628.jpg"], "answer": "B", "solution": "null", "level": "七年级", "question": "(2021$\\cdot$河南南阳市$\\cdot$九年级一模)某地区经过三年的乡村振兴建设, 农村的经济收入是振兴前的 2 倍. 为更好地了解该地区农村的经济收入变化情况, 统计了该地区乡村振兴建设前后农村的经济收入构成比例, 绘制了下面的扇形统计图, 则下列说法错误的是 ( )", "options": "A. 乡村振兴建设后, 养殖收入是振兴前的 2 倍\n\nB. 乡村振兴建设后, 种植收入减少\n\nC. 乡村振兴建设后, 其他收入是振兴前的 2 倍以上\n\nD. 乡村振兴建设后, 养殖收入与第三产业收入的总和超过了经济收入的一半\n\n\n\n乡村振兴前\n\n\n\n乡村振兴后\n\n## ", "subject": "计数", "analysis": "根据某地区经过三年的乡村振兴建设, 农村的经济收入是振兴前的 2 倍和扇形统计图, 可以判断各个选项中的说法是否正确, 从而得到正确答案.\n\n【详解】解: 由题意得乡村振兴建设后, 养殖收入是振兴前的 2 倍, 故 A 选项正确;\n\n乡村振兴建设后, 种植收入相当于振兴前的 $37 \\% \\times 2=74 \\%$, 相对于振兴前收入增加了, 故 B 选项错误;乡村振兴建设后, 其他收入是振兴前的 2 倍以上, 故 C 选项正确;\n\n乡村振兴建设后, 养殖收入与第三产业收入的总和占总收入的 $30 \\%+28 \\%=58 \\%$, 故 D 选项正确; 选 B.\n\n【点睛】本题考查了扇形统计图, 解答本题的关键是明确题意, 利用数形结合的思想解答."} {"id": "23499", "image": [], "answer": "A", "solution": "null", "level": "七年级", "question": "小慧去花店购买鲜花, 若买 5 支玫瑰和 3 支百合, 则她所带的钱还剩下 10 元; 若买 3 支玫瑰和 5 支百合, 则她所带的钱还缺 4 元. 若只买 8 支玫瑰, 则她所带的钱还剩下 ( )\n", "options": "A. 31元\nB. 30 元\nC. 25 元\nD. 19 元\n", "subject": "计数", "analysis": "【答案】 $A$\n\n【解析】\n\n【分析】\n\n本题考查了二元一次方程的应用, 找准等量关系, 正确列出二元一次方程是解题的关键.设每支玫瑰 $x$ 元,每支百合 $y$ 元,根据总价=单价 $\\times$ 数量结合小慧带的钱数不变, 可得出关于 $x, y$ 的二元一次方程, 整理后可得出 $y=x+7$, 再将其代入 $5 x+3 y+10-8 x$ 中即可求出结论.\n\n【解答】\n\n解: 设每支玫瑰 $x$ 元,每支百合 $y$ 元,\n\n依题意, 得: $5 x+3 y+10=3 x+5 y-4$,\n\n$\\therefore y=x+7$,\n\n$\\therefore 5 x+3 y+10-8 x=5 x+3(x+7)+10-8 x=31$.\n\n故选 $A$."} {"id": "23757", "image": [], "answer": "D", "solution": "null", "level": "七年级", "question": "为确定本市七、八、九年级学生校服生产计划, 有关部门准备对180名初中学生的身高作调查, 现有四种调查方案, 样本选取正确的是( )\n\n", "options": "A. 测量体校篮球队和排球队中 180 名队员的身高\n\nB. 随机抽取本市一所学校的180名学生的身高\n\nC. 查阅有关外地180名学生身高的统计资料\n\nD. 在本地的市区和郊县各任选一所完全中学、两所初级中学, 在这六所学校的七、八、九年级的一个班中, 用抽签的方法分别选出 10 名学生, 然后测量他们的身高\n\n", "subject": "计数", "analysis": "【答案】 $D$\n\n【解析】解: $A$ 方案所选取的样本太特殊, 不具备代表性;\n\n$B$ 方案只抽取一所学校的学生, 代表性不强;\n\n$C$ 方案所选取的样本与调查对象无关;\n\n$D$ 方案在各层次, 各方面抽取样本, 更具有代表性和科学性;\n\n故选: D。\n\n样本具有代表性是指抽取的样本必须是随机的, 即各个方面, 各个层次的对象都要有所体现。根据抽取的样本是否具有代表性进行分析。\n\n本题考查了样本的选择, 注意抽取的样本一定要具有代表性。"} {"id": "23758", "image": ["11677.jpg", "11678.jpg"], "answer": "A", "solution": "null", "level": "七年级", "question": "某地区经过三年的新农村建设, 年经济收入实现了翻两番(即是原来的 $2^{2}$ 倍). 为了更好地了解该地区的经济收入变化情况, 统计了该地区新农村建设前后的年经济收入构成结构如图, 则下列结论中不正确的是()\n设前年经济收入结构统计图\n\n\n\n设后年经济收入结构统计图\n\n\n\n", "options": "A. 新农村建设后, 种植收入减少了\n\nB. 新农村建设后, 养殖收入实现了翻两番\n\nC. 新农村建设后, 第三产业收入比新农村建设前的年经济收入还多\n\nD. 新农村建设后, 第三产业收入与养殖收入之和超过了年经济收入的一半\n\n", "subject": "计数", "analysis": "【答案】A\n\n【解析】解:设建设前经济收入为 $a$, 建设后经济收入为 $4 a$.\n\nA、建设后,种植收入为 $30 \\% \\times 4 a=120 \\% a$ ,\n\n建设前, 种植收入为 $55 \\% a$,\n\n故新农村建设后, 种植收入增加了, 故 $A$ 项符合题意;\n\nB、建设后,养殖收入为 $30 \\% \\times 4 a=120 \\% a$ ,\n\n建设前, 养殖收入为 $30 \\% a$,\n\n故 $120 \\% a \\div 30 \\% a=4$, 故 B 项不符合题意;\n\nC、建设后,第三产业收入为 $32 \\% \\times 4 a=128 \\% a$, 故第三产业收入比新农村建设前的年经济收入还多, 故 $C$ 项不符合题意;\n\n、、建设后, 养殖收入与第三产业收入总和为 $(30 \\%+32 \\%) \\times 4 a=248 \\% a$,\n\n经济收入的一半为 $2 a$,\n\n故 $248 \\% a>2 a$, 故 $D$ 项不符合题意.\n\n故选: A.\n\n设建设前经济收入为 $a$, 建设后经济收入为 $4 a$. 通过选项逐一分析新农村建设前后, 经济\n收入情况, 利用数据推出结果.\n\n本题主要考查扇形统计图的应用, 命题的真假的判断, 考查发现问题解决问题的能力."} {"id": "23759", "image": [], "answer": "D", "solution": "null", "level": "七年级", "question": "根据我国国民倾向的阅读形式的调查研究发现, 36.7\\%的成年国民倾向于 “拿一本纸质图书阅读” , 有 $43.5 \\%$ 的国民倾向于 “手机阅读” , 有 10.6\\%的国民更倾向于 “网络在线阅读”, 有 $7.8 \\%$ 的人倾向于 “在电子阅读器上阅读” , 有 $1.4 \\%$ 的国民 “习惯从网上下载并打印下来阅读”. 以上数据最适合用下列哪种统计图描述()\n", "options": "A. 条形统计图\nB. 折线图\nC. 频数分布直方图\nD. 扇形统计图\n", "subject": "计数", "analysis": "【答案】D\n\n【解析】解:由题意,用扇形统计图,可以描述国民阅读形式的百分比.\n\n故选: D.\n\n通过扇形统计图可以很清楚地表示出各部分数量与总数之间的关系.\n\n本题考查扇形统计图, 解题的关键是理解题意, 掌握各种统计图的特征, 灵活运用所学知识解决问题."} {"id": "23760", "image": ["11679.jpg", "11680.jpg"], "answer": "C", "solution": "null", "level": "七年级", "question": "根据《居民家庭亲子阅读消费调查报告》中的相关数据制成扇形统计图, 由图可知,下列说法错误的是 ( )\n\n\n\n", "options": "A. 扇形统计图能反映各部分在总体中所占的百分比\n\nB. 每天阅读 30 分钟以上的居民家庭孩子超过 $50 \\%$\n\nC. 每天阅读 1 小时以上的居民家庭孩子占 $20 \\%$\n\nD. 每天阅读 30 分钟至 1 小时的居民家庭孩子对应扇形的圆心角是 $108^{\\circ}$\n\n\n", "subject": "计数", "analysis": "【答案】 $C$\n\n【解析】\n\n【分析】\n\n本题主要考查扇形统计图, 扇形统计图是用整个圆表示总数, 用圆内各个扇形的大小表示各部分数量占总数的百分数. 根据扇形统计图中的百分比的意义逐一判断即可得.\n\n【解答】\n\n解:\n\nA.扇形统计图能反映各部分在总体中所占的百分比, 此选项正确;\n\nB.每天阅读 30 分钟以上的居民家庭孩子的百分比为 $1-40 \\%=60 \\%$, 超过 $50 \\%$, 此选项正确;\n\nC.每天阅读 1 小时以上的居民家庭孩子占 $20 \\%+10 \\%=30 \\%$, 此选项错误; D. 每天阅读 30 分钟至 1 小时的居民家庭孩子对应扇形的圆心角是 $360^{\\circ} \\times(1-40 \\%-$ $10 \\%-20 \\%)=108^{\\circ}$, 此选项正确;\n\n故选 $C$."} {"id": "23768", "image": [], "answer": "C", "solution": "null", "level": "七年级", "question": "今年某市有 4 万名学生参加中考, 为了了解这些考生的数学成绩, 从中抽取 2000 名考生的数学成绩进行统计分析. 在这个问题中, 有下列说法:\n\n(1) 这4万名考生的数学中考成绩的全体是总体;\n\n(2)每名考生是个体;\n\n(3)2000名考生是总体的一个样本;\n\n(4)样本容量是 2000 .\n\n其中正确的有\n", "options": "A. 4 个\nB. 3 个\nC. 2 个\nD. 1 个\n\n", "subject": "计数", "analysis": "【答案】 $C$\n\n【解析】\n\n【分析】\n\n本题考查了总体、个体、样本、样本容量的概念, 解题要分清具体问题中的总体、个体与样本, 关键是明确考查的对象. 总体、个体与样本的考查对象是相同的, 所不同的是范围的大小. 样本容量是样本中包含的个体的数目, 不能带单位.\n\n总体是指考查的对象的全体, 个体是总体中的每一个考查的对象, 样本是总体中所抽取的一部分个体, 而样本容量则是指样本中个体的数目. 我们在区分总体、个体、样本、样本容量, 这四个概念时, 首先找出考查的对象, 从而找出总体、个体. 再根据被收集数据的这一部分对象找出样本, 最后再根据样本确定出样本容量.\n\n【解答】\n\n解: 这4万名考生的数学中考成绩的全体是总体;\n\n每个考生的数学中考成绩是个体;\n\n2000名考生的中考数学成绩是总体的一个样本, 样本容量是 2000 .\n\n故正确的是(1)(4).\n\n故选 $C$."} {"id": "23775", "image": [], "answer": "D", "solution": "null", "level": "七年级", "question": "2018年1 4月我国新能源乘用车的月销量情况如图\n\n2018年1 4月新能源乘用车\n\n月销量统计图\n\n所示,则下列说法错误的是( )\n", "options": "A. 1 月份销量为 2.2 万辆\n\nB. 从2月到 3 月的月销量增长最快\nC. 4 月份销量比 3 月份增加了 1 万辆\nD. $1 \\sim 4$ 月新能源乘用车销量逐月增加\n\n", "subject": "计数", "analysis": "【答案】D\n\n【解析】\n\n【分析】\n\n此题考查数据收集中的折线统计图.\n\n解答此题的关键是仔细阅读分析折线统计图有关数据和折线的形状及倾斜度, 然后根据题意一一辨析选择即可.\n\n【解答】\n\nA.根据折线统计图一月份对应的数值 2.2 , 可知 1 月份销量为 2.2 万辆的说法正确, 故 $A$不符合题意;\n\nB. 2 月份到 3 月份的线段最陡, 所以 2 月到 3 月的月销量增长最快, 说法正确, 故 $B$ 不符合题意;\n\nC. 4 月份销量为 4.3 万辆, 3 月份销量为 3.3 万量, $4.3-3.3=1$ (万辆), 说法正确, 故不符合题意;\n\nD.统计图中1月到2月线是下降的表示这期间销量是减少的, 所以新能源乘用车销量逐月增加的说法错误, 故 $D$ 符合题意.\n\n故选 $D$."} {"id": "2401", "image": [], "answer": "C", "solution": "null", "level": "七年级", "question": "(3 分) 在一副 52 张扑克牌中(没有大小王)任抽一张牌是方块的机会是()", "options": "A. $\\frac{1}{2}$\nB. $\\frac{1}{3}$\nC. $\\frac{1}{4}$\nD. 0", "subject": "组合数学", "analysis": "解: $\\mathrm{P}$ (方块) $\\frac{13}{52}=\\frac{1}{4}$."} {"id": "2402", "image": ["3507.jpg"], "answer": "C", "solution": "null", "level": "七年级", "question": "(3 分) 如图所示, 小明走进迷宫, 站在 A 处, 迷宫的 8 扇门每一扇门都相同, 其中 6 号门为迷宫出口,则小明一次就能走出迷宫的概率是()\n\n", "options": "A. $\\frac{1}{2}$\nB. $\\frac{1}{3}$\nC. $\\frac{1}{6}$\nD. $\\frac{1}{8}$", "subject": "组合数学", "analysis": "解: 由图易得共有 $2 \\times 3=6$ 种可能, 小明一次就能走出迷宫的有 1 种, 所以概率是 $\\frac{1}{6}$."} {"id": "2403", "image": [], "answer": "B", "solution": "null", "level": "七年级", "question": "(3 分) 有五条线段长分别为 $1,3,5,7,9$, 从中任取三条, 能组成三角形的概率是 ( )", "options": "A. $\\frac{1}{5}$\nB. $\\frac{3}{10}$\nC. $\\frac{1}{2}$\nD. $\\frac{3}{5}$", "subject": "组合数学", "analysis": "解: 从 5 个数中取 3 个数, 共有 10 种可能的结果,\n\n能构成三角形,满足两边之和大于第三边的有: 3、5、7; 3、7、9;5、7、9 三种,\n\n$\\therefore \\mathrm{P}($ 从中任取三条, 能组成三角形 $)=\\frac{3}{10}$."} {"id": "2412", "image": [], "answer": "A", "solution": "null", "level": "七年级", "question": "(3 分) 某商场利用转盘进行有奖促销, 转盘扇形区域的圆心角及奖品设置如下表:\n\n| | 特等奖 | 一等奖 | 二等奖 | 三等奖 | 鼓励奖 |\n| :--- | :--- | :--- | :--- | :--- | :--- |\n| 圆心角 | $1^{\\circ}$ | $10^{\\circ}$ | $60^{\\circ}$ | $90^{\\circ}$ | $199^{\\circ}$ |\n| 奖品 | 冰箱 | 彩电 | 学习机 | 自行车 | 笔记本 |\n\n小英有一次转转盘的机会, 她能获得奖品为学习机的概率是 ( )", "options": "A. $\\frac{1}{6}$\nB. $\\frac{1}{5}$\nC. $\\frac{1}{4}$\nD. $\\frac{1}{3}$", "subject": "组合数学", "analysis": "解: 由题意可得出:她能获得奖品为学习机所占圆心角为 $60^{\\circ}$,\n\n$\\therefore$ 她能获得奖品为学习机的概率是: $\\frac{60}{360}=\\frac{1}{6}$."} {"id": "2421", "image": ["3517.jpg"], "answer": "C", "solution": "null", "level": "七年级", "question": "(3 分) 如图, 两个标有数字的轮子可以分别绕轮子中心旋转, 旋转停止时, 每个轮子上方的箭头各指着轮子上的一个数字, 这两个数字和为偶数的概率是 ( )\n", "options": "A. $\\frac{1}{2}$\nB. $\\frac{1}{6}$\nC. $\\frac{5}{12}$\nD. $\\frac{3}{4}$", "subject": "组合数学", "analysis": "解:根据题意, 旋转停止时, 按箭头指着轮子上的数字, 共 $3 \\times 4=12$ 种情况,\n\n其中两数字和为偶数的有 $2 、 6 ; 4 、 6 ; 8 、 6 ; 5 、 3 ; 5 、 7$, 共 5 种情况;\n\n根据概率的计算公式,可得其概率为 $\\frac{5}{12} ;$"} {"id": "2436", "image": [], "answer": "C", "solution": "null", "level": "七年级", "question": "在平面直角坐标系中, 以点 $(2,1)$ 为圆心、 1 为半径的圆必与 ( )", "options": "A. $x$ 轴相交\n B. $y$ 轴相交\n C. $x$ 轴相切\n D. $y$ 轴相切", "subject": "解析几何", "analysis": "$\\mathrm{C}$"} {"id": "2442", "image": [], "answer": "B", "solution": "null", "level": "七年级", "question": "已知 $\\odot O_{1}$ 与 $\\odot O_{2}$ 内切, 它们的半径分别为 2 和 3 , 则这两圆的圆心距 $d$ 满足 ( )", "options": "A. $d=5$\n B. $d=1$\n C. $15$", "subject": "解析几何", "analysis": "B"} {"id": "2444", "image": [], "answer": "A", "solution": "null", "level": "七年级", "question": "已知正三角形的内切圆半径为 $\\frac{\\sqrt{3}}{3} \\mathrm{~cm}$, 则它的边长是 ( )", "options": "A. $2 \\mathrm{~cm}$\nB. $\\frac{4}{3} \\mathrm{~cm}$\nC. $2 \\sqrt{3} \\mathrm{~cm}$\nD. $\\sqrt{3} \\mathrm{~cm}$", "subject": "解析几何", "analysis": "A"} {"id": "2446", "image": ["3532.jpg"], "answer": "B", "solution": "null", "level": "七年级", "question": "(2 分) 如图, $\\mathrm{P}$ 是 $\\angle \\alpha$ 的边 $\\mathrm{OA}$ 上一点, 且点 $\\mathrm{P}$ 的坐标为 (3, 4), 则 $\\sin \\alpha=(\\quad)$\n\n", "options": "A. $\\frac{3}{5}$\nB. $\\frac{4}{5}$\nC. $\\frac{3}{4}$\nD. $\\frac{4}{3}$", "subject": "解析几何", "analysis": "解: $\\because$ 点 $\\mathrm{P}$ 的坐标为 $(3,4), \\therefore \\mathrm{OP}=5$.\n\n$\\therefore \\sin \\alpha=\\frac{4}{5}$."} {"id": "2465", "image": [], "answer": "D", "solution": "null", "level": "七年级", "question": "(2 分) 在 Rt $\\triangle A B C$ 中, $\\angle C=90^{\\circ}$, 如果 $\\cos A=\\frac{4}{5}$, 那么 $\\tan B$ 的值为", "options": "A. $\\frac{3}{5}$\nB. $\\frac{5}{4}$\nC. $\\frac{3}{4}$\nD. $\\frac{4}{3}$", "subject": "解析几何", "analysis": "解: $\\because$ 在 Rt $\\triangle \\mathrm{ABC}$ 中, $\\angle \\mathrm{C}=90^{\\circ}$,\n\n$\\therefore \\cos A=\\frac{b}{c}, \\tan B=\\frac{b}{a}, a^{2}+b^{2}=c^{2}$.\n\n$\\because \\cos A=\\frac{4}{5}$, 设 $b=4 x$, 则 $c=5 x, a=3 x$.\n\n$\\therefore \\tan B=\\frac{b}{a}=\\frac{4 x}{3 x}=\\frac{4}{3}$."} {"id": "2467", "image": ["3557.jpg"], "answer": "B", "solution": "null", "level": "七年级", "question": "(2 分) 小明在学习“锐角三角函数”中发现, 将如图所示的矩形纸片 ABCD 沿过点 B 的直线折叠, 使点 $\\mathrm{A}$ 落在 $\\mathrm{BC}$ 上的点 $\\mathrm{E}$ 处, 还原后, 再沿过点 $\\mathrm{E}$ 的直线折叠, 使点 $\\mathrm{A}$ 落在 $\\mathrm{BC}$上的点 $\\mathrm{F}$ 处,这样就可以求出 $67.5^{\\circ}$ 角的正切值是 $(\\quad)$\n\n", "options": "A. $\\sqrt{3}+1$\nB. $\\sqrt{2}+1$\nC. 2.5\nD. $\\sqrt{5}$", "subject": "解析几何", "analysis": "解: $\\because$ 将如图所示的矩形纸片 $\\mathrm{ABCD}$ 沿过点 $\\mathrm{B}$ 的直线折叠, 使点 $\\mathrm{A}$ 落在 $\\mathrm{BC}$ 上的点 $\\mathrm{E}$处,\n\n$\\therefore \\mathrm{AB}=\\mathrm{BE}, \\angle \\mathrm{AEB}=\\angle \\mathrm{EAB}=45^{\\circ}$,\n\n$\\because$ 还原后, 再沿过点 $\\mathrm{E}$ 的直线折叠, 使点 $\\mathrm{A}$ 落在 $\\mathrm{BC}$ 上的点 $\\mathrm{F}$ 处,\n$\\therefore \\mathrm{AE}=\\mathrm{EF}, \\quad \\angle \\mathrm{EAF}=\\angle \\mathrm{EFA}=\\frac{45^{\\circ}}{2}=22.5^{\\circ}$,\n\n$\\therefore \\angle \\mathrm{FAB}=67.5^{\\circ}$,\n\n设 $A B=x$\n\n则 $\\mathrm{AE}=\\mathrm{EF}=\\sqrt{2} \\mathrm{x}$,\n\n$\\therefore \\tan \\angle \\mathrm{FAB}=\\tan 67.5^{\\circ}=\\frac{\\mathrm{FB}}{\\mathrm{AB}}=\\frac{\\sqrt{2} \\mathrm{x}+\\mathrm{x}}{\\mathrm{x}}=\\sqrt{2}+1$."} {"id": "2469", "image": ["3559.jpg"], "answer": "D", "solution": "null", "level": "七年级", "question": "(2 分) 如图, 客轮在海上以 $30 \\mathrm{~km} / \\mathrm{h}$ 的速度由 $B$ 向 $C$ 航行, 在 $B$ 处测得灯塔 $A$ 的方位角为北偏东 $80^{\\circ}$, 测得 $\\mathrm{C}$ 处的方位角为南偏东 $25^{\\circ}$, 航行 1 小时后到达 $\\mathrm{C}$ 处, 在 $\\mathrm{C}$ 处测得 $\\mathrm{A}$ 的方位角为北偏东 $20^{\\circ}$, 则 C 到 A 的距离是 $(\\quad)$\n\n", "options": "A. $15 \\sqrt{6} \\mathrm{~km}$\nB. $15 \\sqrt{2} \\mathrm{~km}$\nC. $15(\\sqrt{6}+\\sqrt{2}) \\mathrm{km}$ D. $5(\\sqrt{6}+3 \\sqrt{2}) \\mathrm{km}$", "subject": "解析几何", "analysis": "解: 过点 $\\mathrm{B}$ 作 $\\mathrm{BD} \\perp \\mathrm{AC}$ 于点 $\\mathrm{D}$.\n\n过 $C$ 作方位线, 由平行得到 $\\angle 1=\\angle 2=25^{\\circ}$, 又 $\\angle 3=20^{\\circ}$,\n\n$\\therefore \\angle \\mathrm{BCD}=45^{\\circ}$,\n\n$\\therefore \\triangle \\mathrm{BCD}$ 为等腰直角三角形,\n\n$\\therefore B D=C D=30 \\times \\frac{\\sqrt{2}}{2}=15 \\sqrt{2}$.\n\n$\\because \\mathrm{AD}=\\mathrm{BD} \\cdot \\tan 30^{\\circ}=5 \\sqrt{6}$,\n\n$\\therefore C A=15 \\sqrt{2}+5 \\sqrt{6}=5(\\sqrt{6}+3 \\sqrt{2})$."} {"id": "1698", "image": [], "answer": "C", "solution": "null", "level": "七年级", "question": "木匠在木料上画线, 先确定两个点的位置, 就能把线画得很准确, 其依据是 ( )", "options": "A. 两点之间线段最短\n\nB. 过一点有无数条直线\n\nC. 两点确定一条直线\n\nD. 两点之间线段的长度叫做这两点之间的距离", "subject": "解析几何", "analysis": "木匠在木料上画线, 先确定两个点的位置, 就能把线画得很准确, 其依据是:两点确定一条直线.\n\n故答案为: C."} {"id": "1719", "image": [], "answer": "D", "solution": "null", "level": "七年级", "question": "已知, 点 $C$ 在直线 $A B$ 上, $A C=a, B C=b$, 且 $a \\neq b$, 点 $M$ 是线段 $A B$ 的中点, 则线段 $\\mathrm{MC}$ 的长为", "options": "A. $\\frac{a+b}{2}$\nB. $\\frac{a-b}{2}$\nC. $\\frac{a+b}{2}$ 或 $\\frac{a-b}{2}$\nD. $\\frac{a+b}{2}$ 或 $\\frac{|a-b|}{2}$", "subject": "解析几何", "analysis": "由于点 B 的位置不能确定, 故应分四种情况讨论:\n\n(1)当 $a>b$ 且点 $C$ 在线段 $A B$ 上时, 如图 1.\n\n## $\\overrightarrow{\\mathrm{A}} \\quad \\dot{\\mathrm{M}} \\quad \\dot{\\mathrm{C}} \\quad \\dot{\\mathrm{B}}$\n\n图1\n\n$\\because \\mathrm{AC}=\\mathrm{a}, \\mathrm{BC}=\\mathrm{b}$,\n\n$\\therefore \\mathrm{AB}=\\mathrm{AC}+\\mathrm{BC}=\\mathrm{a}+\\mathrm{b}$.\n\n$\\because$ 点 $\\mathrm{M}$ 是 $\\mathrm{AB}$ 的中点,\n\n$\\therefore \\mathrm{AM}=\\frac{1}{2} \\mathrm{AB}=\\frac{1}{2}(a+b)$,\n\n$\\therefore \\mathrm{MC}=\\mathrm{AC}-\\mathrm{AM}=a-\\frac{1}{2}(a+b)=\\frac{a-b}{2}$.\n\n(2) 当 $a>b$ 且点 $C$ 在线段 $A B$ 的延长线上时, 如图 2 .\n\n$\\overrightarrow{\\mathrm{A}} \\quad \\dot{\\mathrm{M}} \\quad \\overrightarrow{\\mathrm{B}} \\quad \\dot{\\mathrm{C}}$\n\n图2\n\n$\\because \\mathrm{AC}=\\mathrm{a}, \\quad \\mathrm{BC}=\\mathrm{b}$,\n\n$\\therefore A B=A C-B C=a-b$.\n\n$\\because$ 点 $\\mathrm{M}$ 是 $\\mathrm{AB}$ 的中点,\n\n$\\therefore \\mathrm{AM}=\\frac{1}{2} \\mathrm{AB}=\\frac{1}{2}(a-b)$,\n\n$\\therefore \\mathrm{MC}=\\mathrm{AC}-\\mathrm{AM}=a-\\frac{1}{2}(a-b)=\\frac{a+b}{2}$.\n\n(3)当 $\\mathrm{a}<\\mathrm{b}$ 且点 $\\mathrm{C}$ 在线段 $A B$ 上时, 如图 3.\n\n$\\vec{A} \\quad \\dot{C} \\quad \\dot{\\mathrm{M}}$\n\n图3\n\n$\\because \\mathrm{AC}=\\mathrm{a}, \\quad \\mathrm{BC}=\\mathrm{b}$\n\n$\\therefore \\mathrm{AB}=\\mathrm{AC}+\\mathrm{BC}=\\mathrm{a}+\\mathrm{b}$.\n\n$\\because$ 点 $\\mathrm{M}$ 是 $A B$ 的中点,\n\n$\\therefore \\mathrm{AM}=\\frac{1}{2} \\mathrm{AB}=\\frac{1}{2}(a+b)$\n\n$\\therefore \\mathrm{MC}=\\mathrm{AM}-\\mathrm{AC}=\\frac{1}{2}(a+b)-a=\\frac{b-a}{2}$.\n\n(4) 当 $\\mathrm{a}<\\mathrm{b}$ 且点 $\\mathrm{C}$ 在线段 $\\mathrm{AB}$ 的方向延长线上时, 如图 4.\n\n| $\\dot{\\mathrm{C}}$ | $\\vec{A}$ | $\\dot{\\mathrm{M}}$ | $\\overrightarrow{\\mathrm{B}}$ |\n| :--- | :--- | :--- | :--- |\n\n图4\n\n$\\because \\mathrm{AC}=\\mathrm{a}, \\quad \\mathrm{BC}=\\mathrm{b}$,\n\n$\\therefore A B=B C-A C=b-a$.\n\n$\\because$ 点 $M$ 是 $A B$ 的中点,\n$\\therefore \\mathrm{AM}=\\frac{1}{2} \\mathrm{AB}=\\frac{1}{2}(b-a)$,\n\n$\\therefore \\mathrm{MC}=\\mathrm{AC}+\\mathrm{AM}=a+\\frac{1}{2}(b-a)=\\frac{a+b}{2}$.\n\n综上所述: $\\mathrm{MC}$ 的长为 $\\frac{a+b}{2}$ 或 $\\frac{a-b}{2}(\\mathrm{a}>\\mathrm{b})$ 或 $\\frac{b-a}{2}(\\mathrm{a}<\\mathrm{b})$, 即 $\\mathrm{MC}$ 的长为 $\\frac{a+b}{2}$ 或 $\\frac{|a-b|}{2}$.故答案为: D."} {"id": "1723", "image": ["3228.jpg"], "answer": "B", "solution": "null", "level": "七年级", "question": "如图, 点 $O$ 为线段 $A D$ 外一点, 点 $M, C, B, N$ 为 $A D$ 上任意四点, 连接 $O M$, $O C, O B, O N$, 下列结论错误的是 $(\\quad)$\n\n", "options": "A. 以 $O$ 为顶点的角共有 15 个\n\nB. 若 $M C=C B, M N=N D$, 则 $C D=2 C N$\n\nC. 若 $M$ 为 $A B$ 中点, $N$ 为 $C D$ 中点, 则 $M N=\\frac{1}{2}(A D-C B)$\n\nD. 若 $O M$ 平分 $\\angle A O C, O N$ 平分 $\\angle B O D, \\angle A O D=5 \\angle C O B$, 则 $\\angle M O N=\\frac{3}{2}(\\angle M O C+$ $\\angle B O N)$", "subject": "解析几何", "analysis": "$\\mathrm{O}$ 为顶点的角有 $\\frac{6 \\times 5}{2}=15$ 个,\n\n所以 $\\mathrm{A}$ 选项不符合题意;\n\n$\\because M N=N D$\n\n$\\therefore N D>C N$,\n\n$\\therefore C D=C N+N D>C N+C N ,$ 即 $C D>2 C N$ ,\n\n所以 $\\mathrm{B}$ 选项符合题意;\n\n由中点定义可得: $M B=\\frac{1}{2} A B, \\quad N C=\\frac{1}{2} C D$,\n\n$\\therefore M N=M B+C N-C B=\\frac{1}{2} A B+\\frac{1}{2} C D-C B=\\frac{1}{2}(A B+C D)-C B$,\n\n$\\because A B+C D=A D+C B$\n\n$\\therefore M N=\\frac{1}{2}(A D+C B)-C B=\\frac{1}{2}(A D-C B)$,\n\n所以 $\\mathrm{C}$ 选项不符合题意;\n\n由角平分线的定义可得: $\\angle A O C=2 \\angle M O C, \\angle B O D=2 \\angle B O N$,\n\n$\\because \\angle A O D=\\angle A O C+\\angle C O B+\\angle D O B=5 \\angle C O B$,\n\n$\\therefore 2 \\angle M O C+2 \\angle B O N+\\angle B O C=5 \\angle B O C$,\n\n$\\therefore \\angle M O C+\\angle B O N=2 \\angle B O C$,\n\n$$\n\\therefore \\angle M O N=\\angle M O C+\\angle C O B+\\angle B O N=2 \\angle C O B+\\angle C O B=3 \\angle C O B\n$$\n\n$\\frac{3}{2}(\\angle M O C+\\angle B O N)=\\frac{3}{2} \\times 2 \\angle C O B=3 \\angle C O B$,\n\n$\\therefore \\angle M O N=\\frac{3}{2}(\\angle M O C+\\angle B O N)$\n\n所以 $D$ 选项不符合题意,\n\n所以错误的只有 $B$,\n\n故答案为: B."} {"id": "2468", "image": ["3558.jpg"], "answer": "B", "solution": "null", "level": "七年级", "question": "(2 分) 如图, $A B$ 是 $\\odot O$ 的直径, 弦 $A C, B D$ 相交于点 $P$, 则 $\\frac{C D}{A B}$ 等于 ( )\n\n", "options": "A. $\\sin \\angle \\mathrm{BPC}$\nB. $\\cos \\angle \\mathrm{BPC}$\nC. $\\tan \\angle \\mathrm{BPC}$\nD. 以上都不对", "subject": "组合几何学", "analysis": "解: 连接 $\\mathrm{BC}$.\n\n$\\because \\mathrm{AB}$ 是 $\\odot \\mathrm{O}$ 的直径,\n\n$\\therefore \\angle \\mathrm{BCP}=90^{\\circ}$.\n\n根据同弧所对的圆周角相等得:\n\n$\\angle \\mathrm{A}=\\angle \\mathrm{D}, \\angle \\mathrm{DCA}=\\angle \\mathrm{PBA}$\n\n$\\therefore \\triangle \\mathrm{PCD} \\sim \\triangle \\mathrm{PBA}$.\n\n$\\therefore \\frac{C D}{A B}=\\frac{P C}{P B}=\\cos \\angle \\mathrm{BPC}$."} {"id": "1528", "image": ["3161.jpg", "3162.jpg", "3163.jpg"], "answer": "D", "solution": "null", "level": "七年级", "question": "如图 1 所示, 在一个边长为 $\\mathrm{a}$ 的正方形纸片上剪去两个小长方形, 得到一个如图 2 的图案所示,再将剪下的两个小长方形拼成一个新的长方形, 如图 3 所示, 则新长方形的周长可表示为 ( )\n\n\n\n图1\n\n\n\n图2\n\n\n\n图3", "options": "$2 a-3 b$\nB. $4 a-10 b$\nC. $2 a-4 b$\nD. $4 a-8 b$", "subject": "组合几何学", "analysis": "根据题意得小长方形的长为: $\\mathrm{a}-\\mathrm{b}$, 宽为: $\\frac{a-3 b}{2}$,\n\n$\\therefore$ 新长方形的周长为: $\\left(\\frac{a-3 b}{2} \\times 2+a-b\\right) \\times 2=4 a-8 b$.\n\n故答案为: D."} {"id": "1626", "image": ["3184.jpg"], "answer": "C", "solution": "null", "level": "七年级", "question": "在长方形 $A B C D$ 中, 放入 6 个形状大小完全相同的小长方形, 所标尺寸如图所示, 则小长方形的宽 $A E$ 的长度为 $(\\quad) \\mathrm{cm}$.\n\n", "options": "A. 1\nB. 1.6\nC. 2\nD. 2.5", "subject": "组合几何学", "analysis": "设 $\\mathrm{AE}=\\mathrm{xcm}$,\n\n依题意, 得: $6+2 x=x+(14-3 x)$,\n\n解得: $x=2$\n\n故答案为: C."} {"id": "1436", "image": ["3130.jpg"], "answer": "C", "solution": "null", "level": "七年级", "question": "将尺寸如图的 4 块完全相同的长方形薄木块(厚度忽略不计)进行拼摆, 恰好可以不重叠地摆放在如图的甲、乙两个方框内. 已知小木块的宽为 2 , 图甲中阴影部分面积为 19 , 则图乙中 $\\mathrm{AD}$ 的长为 $(\\quad)$\n\n", "options": "A. $2 \\sqrt{19}+2$\nB. $\\sqrt{19}+4$\nC. $2 \\sqrt{19}+4$\nD. $\\sqrt{19}+2$", "subject": "立体几何学", "analysis": "块的长为 $\\mathrm{x}$,\n\n根据题意, 知: $(x-2)^{2}=19$,\n\n则 $x-2= \\pm \\sqrt{19}$,\n\n$\\therefore x=2+\\sqrt{19}$ 或 $x=2-\\sqrt{19}<2$ (舍去)\n\n则 $B C=2 x=2 \\sqrt{19}+4$ ,\n\n故选: C."} {"id": "1667", "image": ["3193.jpg"], "answer": "D", "solution": "null", "level": "七年级", "question": "如图, 该平面展开图按虚线折叠成正方体后, 相对面上两个数之和为 8 , 则 $\\mathrm{x}+\\mathrm{y}$ 的值是 ( )\n\n", "options": "A. 7\nB. 8\nC. 9\nD. 10", "subject": "立体几何学", "analysis": "$\\because$ 与 $\\mathrm{x}$ 相对的面是 2 , 与 $\\mathrm{y}$ 相对的面是 4 ,\n\n$\\therefore \\mathrm{x}+2=8, \\mathrm{y}+4=8$,\n\n$\\therefore \\mathrm{x}=6, \\mathrm{y}=4$,\n\n$\\therefore \\mathrm{x}+\\mathrm{y}=10$.\n\n故答案为: D."} {"id": "1717", "image": [], "answer": "B", "solution": "null", "level": "七年级", "question": "如图, 点 $\\mathrm{C}$ 在线段 $\\mathrm{AB}$ 上, $\\mathrm{AB}=10, \\mathrm{AC}=4$, 点 $\\mathrm{D}$ 是 $\\mathrm{BC}$ 的中点, 则 $\\mathrm{BD}$ 的长为 $(\\quad)$", "options": "2\nB. 3\nC. 5\nD. 6", "subject": "立体几何学", "analysis": "$\\because \\mathrm{AB}=10, \\mathrm{AC}=4$,\n\n$\\therefore \\mathrm{BC}=\\mathrm{AB}-\\mathrm{AC}=6$,\n\n$\\because$ 点 $\\mathrm{D}$ 是 $\\mathrm{BC}$ 的中点,\n$\\therefore \\mathrm{BD}=\\frac{1}{2} \\mathrm{BC}=3$.\n\n故答案为: B"} {"id": "1460", "image": ["3140.jpg", "3141.jpg", "3142.jpg"], "answer": "D", "solution": "null", "level": "七年级", "question": "把如图 1 的两张大小相同的长方形卡片放置在图 2 与图 3 中的两个相同大长方形中, 已知这两个大长方形的长比宽长 $20 \\mathrm{~cm}$, 若记图 2 中阴影部分的周长为 $\\mathrm{C}_{1}$, 图 3 中阴影部分的周长为 $\\mathrm{C}_{2}$, 那 $么 \\mathrm{C}_{1}-\\mathrm{C}_{2}=(\\quad)$\n\n\n\n图।\n\n\n\n图2\n\n\n\n图3", "options": "$10 \\mathrm{~cm}$\nB. $20 \\mathrm{~cm}$\nC. $30 \\mathrm{~cm}$\nD. $40 \\mathrm{~cm}$", "subject": "画法几何学", "analysis": "设图 2 与图 3 中的大长方形的宽为 $\\mathrm{acm}$, 则长为 $(a+20) \\mathrm{cm}$,\n\n图 1 中的长方形长为 $\\mathrm{xcm}$, 宽为 $\\mathrm{ycm}$,\n\n由图 2 可知: $C_{1}=(a+a+20) \\times 2=4 a+40$;\n\n由图 3 可知: $x+y=a+20$,\n\n$C_{2}=2(a+20)+2(a-x)+2(a-y)$,\n$=2 a+40+4 a-2(x+y)$,\n\n$=6 a+40-2(a+20)$,\n\n$=4 a$,\n\n则 $C_{1}-C_{2}=4 a+40-4 a=40(\\mathrm{~cm})$,\n\n故答案为: D."} {"id": "1484", "image": ["3149.jpg", "3150.jpg", "3151.jpg"], "answer": "B", "solution": "null", "level": "七年级", "question": "如图, 将图 1 中的长方形纸片前成(1)号、(2) 号、(3) 号、(4) 号正方形和(5)号长方形, 并将它们按图 2 的方式无重叠地放入另一个大长方形中, 若需求出没有覆盖的阴影部分的周长, 则下列说法中错误的是 $(\\quad)$\n\n\n\n图 1\n\n\n\n图 2", "options": "只需知道图 1 中大长方形的周长即可\n\nB. 只需知道图 2 中大长方形的周长即可\n\nC. 只需知道 $(3)$ 号正方形的周长即可\n\nD. 只需知道 $(5)$ 号长方形的周长即可", "subject": "画法几何学", "analysis": "(1)号正方形的边长为 $\\mathrm{a}$, (2)号正方形的边长为 $\\mathrm{b}$, 则 (3)号正方形的边长为 $\\mathrm{a}+\\mathrm{b}$, (4)号正方形的边长为 $2 a+b$, (5) 号长方形的长为 $3 a+b$, 宽为 $b-a$,\n\n$$\n\\text { 如图, } A D=b-a+b+a=2 b, \\quad A B=a+b+2 a+b-b=3 a+b\n$$\n\n\n\n$\\therefore$ 矩形 $\\mathrm{ABCD}$ 的周长为 $2(A B+A D)=2(3 a+b+b-a)=6(a+b)$\n\n图 1 中大长方形的周长为: $2(a+b+b+a+b+2 a+b)=8(a+b)$\n\n图 2 中大长方形的周长为 $2(b-a+b+2 a+b+3 a+2 b)=2(5 b+4 a)$\n\n(3) 号正方形周长为 $4(a+b)$\n\n(5)号正方形周长为 $2(2 a+b+b-a)=4(a+b)$\n\n所以, 只有 $2(5 b+4 a)$ 不能得出 $6(a+b)$ 的值,\n\n故答案为: B."} {"id": "1508", "image": ["3156.jpg", "3157.jpg"], "answer": "B", "solution": "null", "level": "七年级", "question": "如图, 将长方形 $\\mathrm{ABCD}$ 分成 2 个长方形与 2 个正方形, 其中 (3)、(4)为正方形, 记长方形(1)的周长为 $C_{1}$, 长方形 (2)的周长为 $C_{2}$, 则 $C_{1}$ 与 $C_{2}$ 的大小为 ( $)$\n\n", "options": "A. $C_{1}>C_{2}$\nB. $C_{1}=C_{2}$\nC. $C_{1}\n\n$\\because$ 将长方形 $\\mathrm{ABCD}$ 分成 2 个长方形与 2 个正方形, 其中(3)、(4)为正方形\n\n$\\therefore C G=B E, \\quad A E=D G, \\quad B C=A D, \\quad A B=C D$\n\n设正方形(3)的边长为 $\\mathrm{a}$, 正方形(4)的边长为 $\\mathrm{b}$\n\n$\\therefore C G=B E=a, \\quad C F=B C-B E=A D-a, \\quad A E=D G=b, \\quad A H=A D-D H=A D-b$\n\n$\\therefore$ 长方形(1)的周长为 $C_{1}=2 A H+2 A E=2 A D-2 b+2 b=2 A D$,\n\n长方形(2)的周长为 $C_{2}=2 C F+2 C G=2 A D-2 a+2 a=2 A D$\n\n$\\therefore C_{1}=C_{2}$\n\n故答案为:\nB."} {"id": "2523", "image": ["3601.jpg", "3602.jpg", "3603.jpg"], "answer": "D", "solution": "null", "level": "七年级", "question": "(2021$\\cdot$浙江温州市$\\cdot$七年级期末)将图 1 中周长为 32 的长方形纸片剪成 1 号、 2 号、 3 号、 4 号正方形和 5 号长方形, 并将它们按图 2 的方式放入周长为 48 的长方形中, 则没有覆盖的阴影部分的周\n长为 $(\\quad)$\n\n\n\n(图 1)\n\n\n\n(图 2 )", "options": "A. 16\nB. 24\nC. 30\nD. 40", "subject": "画法几何学", "analysis": "设 1 号正方形的边长为 $\\mathrm{x}, 2$ 号正方形的边长为 $\\mathrm{y}$, 则 3 号正方形的边长为 $\\mathrm{x}+\\mathrm{y}, 4$ 号正方形的边长为 $2 \\mathrm{x}+\\mathrm{y}, 5$ 号长方形的长为 $3 \\mathrm{x}+\\mathrm{y}$, 宽为 $\\mathrm{y}-\\mathrm{x}$, 根据图 1 中长方形的周长为 32 , 求得 $\\mathrm{x}+\\mathrm{y}=4$, 根据图 2 中长方形的周长为 48 , 求得 $A B=24-3 x-4 y$, 根据平移得: 没有覆盖的阴影部分的周长为四边形 $A B C D$ 的周长 $=2(A B+A D)$, 计算即可得到答案.\n\n【详解】设 1 号正方形的边长为 $\\mathrm{x}, 2$ 号正方形的边长为 $\\mathrm{y}$, 则 3 号正方形的边长为 $\\mathrm{x}+\\mathrm{y}, 4$ 号正方形的边长为 $2 x+y, 5$ 号长方形的长为 $3 x+y$, 宽为 $y-x$,\n\n由图 1 中长方形的周长为 32 , 可得, $y+2(x+y)+(2 x+y)=16$, 解得: $x+y=4$,\n\n如图, $\\because$ 图 2 中长方形的周长为 $48, \\therefore A B+2(x+y)+2 x+y+y-x=24, \\quad \\therefore A B=24-3 x-4 y$,\n\n根据平移得:没有覆盖的阴影部分的周长为四边形 $\\mathrm{ABCD}$ 的周长,\n\n$\\therefore 2(A B+A D)=2(24-3 x-4 y+x+y+2 x+y+y-x)=2(24-x-y)=48-2(x+y)=48-8=40$, 故选: D.\n\n\n\n【点睛】此题考查整式加减的应用, 平移的性质, 利用平移的性质将不规则图形变化为规则图形进而求解, 解题的关键是设出未知数, 列代数式表示各线段进而解决问题."} {"id": "2525", "image": ["3605.jpg", "3606.jpg"], "answer": "B", "solution": "null", "level": "七年级", "question": "(2021 浙江七年级期中) 将一张边沿互相平行的纸条如图折叠后, 若边 $A D / / B C$, 则翻折角 $\\angle 1$与 $\\angle 2$ 一定满足的关系是 $(\\quad)$\n\n", "options": "A. $\\angle 1=2 \\angle 2$\nB. $\\angle 1+\\angle 2=90^{\\circ}$\nC. $\\angle 1-\\angle 2=30^{\\circ}$\nD. $2 \\angle 1-3 \\angle 2=30^{\\circ}$\n\n## ", "subject": "画法几何学", "analysis": "根据平行可得出 $\\angle D A B+\\angle C B A=180^{\\circ}$, 再根据折叠和平角定义可求出 $\\angle 1+\\angle 2=90^{\\circ}$.\n\n【详解】解:由翻折可知, $\\angle D A E=2 \\angle 1, \\angle C B F=2 \\angle 2$,\n\n$\\because A D / / B C, \\therefore \\angle D A B+\\angle C B A=180^{\\circ}, \\therefore \\angle D A E+\\angle C B F=180^{\\circ}$,\n\n即 $2 \\angle 1+2 \\angle 2=180^{\\circ}, \\therefore \\angle 1+\\angle 2=90^{\\circ}$ ,故选: $B$.\n\n\n\n【点睛】本题考查了平行线的性质和角平分线的性质, 解题关键是熟练运用平行线的性质进行推理计\n\n算.\n\n##"} {"id": "23418", "image": [], "answer": "A", "solution": "null", "level": "七年级", "question": "有下列语句: (1)不相交的两条直线叫平行线;\n\n(2)两条直线的位置关系只有两种: 相交和平行;\n\n(3)如果线段 $A B$ 和线段 $C D$ 不相交, 那么直线 $A B$ 和直线 $C D$ 平行;\n\n(4)如果两条直线都和第三条直线平行, 那么这两条直线平行;\n\n(5)过一点有且只有一条直线与已知直线平行. 正确的个数是\n", "options": "A. 1\nB. 2\nC. 3\nD. 4\n\n", "subject": "逻辑题", "analysis": "【答案】A\n\n【解析】解:(1)不相交的两条直线叫平行线, 必须是在同一平面内, 故错误; (2)在同一平面内, 不重合的两条直线的位置关系只有两种: 相交和平行, 故错误;\n\n(3)线段 $A B$ 和线段 $C D$ 不相交, 直线 $A B$ 和直线 $C D$ 不一定平行, 故错误;\n\n(4)如果两条直线都和第三条直线平行, 那么这两条直线平行, 故正确;\n\n(5)过直线外一点有且只有一条直线与已知直线平行, 故错误, 故选 A."} {"id": "23427", "image": ["11556.jpg"], "answer": "C", "solution": "null", "level": "七年级", "question": "将一张长方形纸片沿 $E F$ 折叠, 折叠后的位置如图所示, 若 $\\angle E F B=65^{\\circ}$, 则 $\\angle A E D^{\\prime}$ 等于( )\n", "options": "A. $70^{\\circ}$\nB. $65^{\\circ}$\nC. $50^{\\circ}$\n\n\nD. $25^{\\circ}$\n\n", "subject": "变换几何", "analysis": "【答案】C\n\n【解析】解: $\\because$ 四边形 $A B C D$ 是矩形,\n\n$\\therefore A D / / B C$,\n\n$\\therefore \\angle D E F=\\angle E F B=65^{\\circ}$,\n\n由翻折不变性可知: $\\angle D E F=\\angle F E D^{\\prime}=65^{\\circ}$,\n\n$\\therefore \\angle D E D^{\\prime}=130^{\\circ}$,\n\n$\\therefore \\angle A E D^{\\prime}=180^{\\circ}-130^{\\circ}=50^{\\circ}$,\n\n故选: $C$.\n\n利用平行线的性质以及翻折不变性解决问题即可.\n\n本题考查平行线的性质, 矩形的性质, 翻折变换等知识, 解题的关键是熟练掌握基本知识, 属于中考常考题型."} {"id": "23429", "image": ["11558.jpg"], "answer": "C", "solution": "null", "level": "七年级", "question": "如图, 将三角形 $A B C$ 沿射线 $A B$ 平移到三角形 $D E F$ 的位置, 则以下说法不正确的是 ( )\n", "options": "A. $A C / / D F$.\nB. $\\angle C=\\angle F$.\n\n\nC. $A C=D B$.\nD. $A D=B E$\n\n", "subject": "变换几何", "analysis": "【答案】 $C$\n\n解: $A$. 因为 $\\angle A=\\angle F D E$, 所以 $A C / / D F$, 故 $A$ 正确;\n\nB. B 正确;\n\n$C . A C \\neq D B$, 故 $C$ 错误;\n\n$D$. 因为 $A B=D E$, 所以 $A D=B E$. 所以 $D$ 正确.\n\n故选 $C$."} {"id": "23431", "image": ["11559.jpg"], "answer": "C", "solution": "null", "level": "七年级", "question": "如图, 将三角形 $A B C$ 沿水平方向向右平移到三角形 $D E F$ 的位置. 已知点 $A, D$ 之间的距离为 $1, C E=2$, 则 $B F$ 的长为 $(\\quad)$\n\n\n", "options": "A. 2 .\nB. 3 .\nC. 4 .\nD. 5 .\n\n", "subject": "变换几何", "analysis": "解【答案】 $C$\n\n【解析】解: $\\because$ 三角形 $A B C$ 沿水平方向向右平移到三角形 $D E F$,\n\n$\\therefore B E=C F=A D=1$,\n\n$\\therefore B F=B E+C E+C F=1+2+1=4$.\n\n根据平移的性质得到 $B E=C F=A D=1$, 然后计算 $B E+C E+C F$ 即可.\n\n本题考查了平移的性质."} {"id": "23432", "image": [], "answer": "B", "solution": "null", "level": "七年级", "question": "以下现象中, 属于平移的是( )\n(1)在荡秋千的小朋友;\n(2)电梯上升过程;\n\n(3)宇宙中行星的运动; (4)生产过程中传送带上的电视机的移动过程.\n", "options": "A. (1)(2)\nB. (2)(4)\nC. (2)(3)\nD. (3)(4)\n\n", "subject": "变换几何", "analysis": "【答案】 $B$\n\n解: (1)在荡秋千的小朋友是旋转运动, 不是平移;\n\n(2)电梯上升过程是平移;\n\n(3)宇宙中行星的运动不是平移;\n\n(4)生产过程中传送带上的电视机的移动过程是平移;\n\n故选 $B$."} {"id": "2387", "image": ["3504.jpg"], "answer": "C", "solution": "null", "level": "七年级", "question": "如图, 圆的周长为 4 个单位长度, 在该圆的四等分点处分别标上数字 $0 、 1 、 2 、 3$, 先让圆周上表示数字 0 的点与数轴上表示数 -1 的点重合, 再将数轴按逆时针方向环绕在该圆上, 则数轴上表示数 -2019 的点与圆周上重合的点表示的数字为 $(\\quad)$\n\n", "options": "A. 0\nB. 1\nC. 2\nD. 3", "subject": "图论", "analysis": "解: 因为 -1 与 -2019 之间的距离是 2018 个单位长度, 而 $2018 \\div 4=504 \\cdots \\cdots 2$, 所以数轴上表示数-2019的点与圆周上表示数字 2 的点重合, 故选 $C$."} {"id": "2971", "image": [], "answer": "B", "solution": "null", "level": "七年级", "question": "在扇形统计图中, 其中一个扇形的圆心角为 $72^{\\circ}$, 则这个扇形所表示的区域占总体区域的( )\n", "options": "A. $10 \\%$\nB. $20 \\%$\nC. $30 \\%$\nD. $50 \\%$\n", "subject": "统计数学", "analysis": " 根据题意, 这个扇形所表示的区域占总体区域的 $\\frac{72}{360} \\times 100 \\%=20 \\%$,\n故选: $B$.\n\n"} {"id": "3572", "image": ["4865.jpg", "4866.jpg"], "answer": "A", "solution": "null", "level": "九年级", "question": "已知二次函数 $y=x^{2}$, 当 $a \\leq x \\leq b$ 时 $m \\leq y \\leq n$, 则下列说法正确的是", "options": "A. 当 $n-m=1$ 时, $b-a$ 有最大值\nB. 当 $n-m$ 时,$b-a$ 有最小值\nC. 当 $b-a=1$ 时, $n-m$ 无最小值\nD. 当 $b-a=1$ 时, $n-m$ 有最大值", "subject": "解析几何", "analysis": "(1)当 $b-a=1$ 时, 如图 1 , 过点 $B$ 作 $B C \\perp A D$ 于 $C$,\n\n\n\n图 1\n\n$\\therefore \\angle B C D=90^{\\circ}$,\n\n$\\because \\angle A D E=\\angle B E D=90^{\\circ}$,\n\n$\\therefore \\angle A D O=\\angle B C D=\\angle B E D=90^{\\circ}$,\n\n$\\therefore$ 四边形 $B C D E$ 是矩形,\n\n$\\therefore B C=D E=b-a=1, C D=B E=m$,\n\n$\\therefore A C=A D-C D=n-m$,\n\n在Rt $\\triangle A C B$ 中,\n\n$\\tan \\angle A B C=\\frac{A C}{B C}=n-m$,\n\n$\\because$ 点 $A, B$ 在抛物线 $y=x^{2}$ 上,\n\n$\\therefore 0^{\\circ} \\leq \\angle A B C<90^{\\circ}$,\n\n$\\therefore \\tan \\angle A B C \\geq 0$,\n\n$\\therefore n-m \\geq 0$,\n\n即 $n-m$ 无最大值, 有最小值, 最小值为 0 , 故选项 $C, D$ 都错误;\n\n(2)当 $n-m=1$ 时, 如图 2 , 过点 $N$ 作 $N H \\perp M Q$ 于 $H$,\n\n图 2\n\n\n同 (1)的方法得, $N H=P Q=b-a, H Q=P N=m$,\n\n$\\therefore M H=M Q-H Q=n-m=1$,\n\n在 $R t \\triangle M H Q$ 中,\n\n$\\tan \\angle M N H=\\frac{M H}{N H}=\\frac{1}{b-a}$,\n\n$\\because$ 点 $M, N$ 在抛物线 $y=x^{2}$ 上,\n\n$\\therefore m \\geq 0$,\n\n当 $m=0$ 时, $n=1$,\n\n$\\therefore$ 点 $N(0,0), M(1,1)$,\n\n$\\therefore N H=1$,\n\n此时, $\\angle M N H=45^{\\circ}$,\n\n$\\therefore 45^{\\circ} \\leq \\angle M N H<90^{\\circ}$,\n\n$\\therefore \\tan \\angle M N H \\geq 1$,\n\n$\\therefore \\frac{1}{b-a} \\geq 1$,\n\n$\\therefore b-a$ 无最小值, 有最大值, 最大值为 1 , 故选项 $B$ 错误.\n\n故选: $A$."} {"id": "3573", "image": ["4867.jpg", "4868.jpg"], "answer": "C", "solution": "null", "level": "九年级", "question": "如图, $\\odot O$ 的半径为 $2, A B, C D$ 是互相垂直的两条直径, 点 $P$是 $\\odot O$ 上任意一点, 过点 $P$ 作 $P M \\perp A B$ 于 $M, P N \\perp C D$ 于 $N$,点 $Q$ 是 $M N$ 的中点, 当点 $P$ 沿着圆周从点 $D$ 逆时针方向运动到点 $C$ 的过程中, 当 $\\angle Q C N$ 度数取最大值时, 线段 $C Q$ 的长为 $(\\quad)$", "options": "A. 2\nB. 1\n\n\nC. $\\sqrt{3}$\nD. $\\sqrt{3}-1$", "subject": "解析几何", "analysis": "连接 $O Q$,\n\n\n\n$\\because M N=O P$ (矩形对角线相等), $\\odot O$ 的半径为 2 ,\n\n$\\therefore O Q=\\frac{1}{2} M N=\\frac{1}{2} O P=1$,\n\n可得点 $Q$ 的运动轨迹是以 $O$ 为圆心, 1 为半径的圆, 当 $C Q$ 与此圆相切时, $\\angle Q C N$ 最大,\n\n此时, 在直角三角形 $C Q^{\\prime} O$ 中,\n\n$\\angle C Q^{\\prime} O=90^{\\circ}, O Q^{\\prime}=1, C O=2$,\n\n$\\therefore C Q^{\\prime}=\\sqrt{C O^{2}-O Q^{\\prime 2}}=\\sqrt{2^{2}-1^{2}}=\\sqrt{3}$,\n\n即线段 $C Q$ 的长为 $\\sqrt{3}$.\n\n故选 $C$."} {"id": "3591", "image": ["4905.jpg"], "answer": "D", "solution": "null", "level": "九年级", "question": "$P A 、 P B$ 分别切 $\\odot O$ 于点 $A 、 B$, 点 $C$ 为 $\\odot O$ 上一个动点 (不与 $A 、 B$ 重合), 若 $\\angle A P B=40^{\\circ}$. 则 $\\angle A C B$的度数为 ( $)$", "options": "A. $70^{\\circ}$\nB. $140^{\\circ}$ 或 $40^{\\circ}$\nC. $140^{\\circ}$\nD. $70^{\\circ}$ 或 $110^{\\circ}$", "subject": "解析几何", "analysis": "连接 $O A 、 O B$,\n\n\n$\\because P A 、 P B$ 分别切 $\\odot O$ 于 $A 、 B$ 两点,\n\n$\\therefore O A \\perp A P, O B \\perp P B$,\n\n(1) 当点 $C$ 在优弧 $A B$ 上时,\n\n$\\angle A O B=180^{\\circ}-\\angle A P B=140^{\\circ}$,\n\n$\\therefore \\angle A C^{\\prime} B=70^{\\circ}$;\n\n(2)当点 $C$ 在劣弧 $A B$ 上时, $\\angle A C B=180^{\\circ}-\\angle A C^{\\prime} B=110^{\\circ}$.\n\n综上可得: $\\angle A C B=70^{\\circ}$ 或 $110^{\\circ}$.\n\n故选 $D$."} {"id": "3594", "image": [], "answer": "B", "solution": "null", "level": "九年级", "question": "$\\triangle A B C$ 中 $\\angle A, \\angle B$ 为锐角, 且 $\\left|\\tan ^{2} B-3\\right|+(2 \\sin A-\\sqrt{3})^{2}=0$, 则 $\\triangle A B C(\\quad)$", "options": "A. 直角三角形\nB. 等边三角形\nC. 等腰三角形\nD. 等腰直角三角形", "subject": "解析几何", "analysis": "由 $\\left|\\tan ^{2} B-3\\right|+(2 \\sin A-\\sqrt{3})^{2}=0$, 得\n\n$\\tan ^{2} B-3=0,2 \\sin A-\\sqrt{3}=0$,\n\n由 $\\angle A, \\angle B$ 均为锐角, 得\n\n$\\tan B=\\sqrt{3}, \\sin A=\\frac{\\sqrt{3}}{2}$,\n\n$A=60^{\\circ}, B=60^{\\circ}$,\n\n$\\angle C=180^{\\circ}-\\angle A-\\angle B=60^{\\circ}$,\n\n$\\therefore \\angle C=\\angle A=\\angle B=60^{\\circ}$,\n\n$\\therefore \\triangle A B C$ 是等边三角形,\n\n故选 $B$."} {"id": "3595", "image": [], "answer": "A", "solution": "null", "level": "九年级", "question": "在 $\\triangle A B C$ 中, $\\angle C=90^{\\text {” }}, \\tan A$ 和 $\\tan B$ 的值是关于 $x$ 的方程 $x^{2}-\\frac{4 \\sqrt{3}}{3} x+m=0$ 的根, 则 $m$ 等于 $(\\quad)$", "options": "A. 1\nB. -1\nC. $\\frac{4 \\sqrt{3}}{3}$\nD. $-\\frac{4 \\sqrt{3}}{3}$", "subject": "解析几何", "analysis": ": $\\angle C=90^{\\circ}$,\n\n$\\therefore \\tan A \\cdot \\tan B=\\frac{a}{b} \\cdot \\frac{b}{a}=1$,\n\n$\\because \\tan A$ 和 $\\tan B$ 的值是关于 $x$ 的方程 $x^{2}-\\frac{4 \\sqrt{3}}{3} x+m=0$ 的根,\n\n$\\therefore m=\\tan A \\cdot \\tan B=1$.\n\n故选 $A$."} {"id": "3638", "image": [], "answer": "A", "solution": "null", "level": "九年级", "question": "下列说法: (1)平分弦的直径垂直于弦; (2)三点确定一个圆; (3)相等的圆心角所对的弧相等; (4)\n垂直于半径的直线是圆的切线; (5)三角形的内心到该三角形三条边的距离相等。其中正确的个数为 $(\\quad)$", "options": "A. 1 个\nB. 2 个\nC. 3 个\nD. 4 个", "subject": "解析几何", "analysis": "1)当两弦都是直径时, 它们相互平分,但可以不垂直, 1错误;\n\n(2)当三点共线时不能确定一个圆; (2)错误;\n\n(3)当圆半径相等时才有相等的圆心角所对的弧相等, (3)错误;\n\n(4)过半径的外端点且垂直于半径的直线才是圆的切线, (4)错误;\n\n(5)根据角平分线的性质即可得出三角形的内心到三角形的三边距离相等, $\\therefore$ (5)正确;\n\n故选 $A$."} {"id": "3639", "image": [], "answer": "D", "solution": "null", "level": "九年级", "question": "在直角坐标系中, 我们把横、纵坐标都为整数的点叫做整点. 对于一条直线, 当它与一个圆的公共点都是整点时, 我们把这条直线称为这个圆的 “整点直线”. 已知 $\\odot O$ 是以原点为圆心, 半径为 $2 \\sqrt{2}$ 的圆, 则 $\\odot O$ 的 “整点直线” 共有 $(\\quad)$ 条", "options": "A. 7\nB. 8\nC. 9\nD. 10", "subject": "解析几何", "analysis": "$\\because$ 圆的半径为 $2 \\sqrt{2}$,\n\n$\\therefore$ 圆上的整数点有四个, $(2,2),(2,-2),(-2,2),(-2,-2)$,\n若直线与圆有两个交点, 则两点确定一直线, 可以画 6 条,\n\n若直线与圆只有一个交点, 则分别过这四个点画圆的切线, 可以有 4 条,\n\n$\\therefore$ 一共有 10 条,\n\n故选 $D$."} {"id": "3893", "image": [], "answer": "C", "solution": "null", "level": "九年级", "question": "正六边形的外接圆的半径与内切圆的半径之比为 $(\\quad)$", "options": "A. $1: \\sqrt{3}$\nB. $\\sqrt{3}: 2$\nC. $2: \\sqrt{3}$\nD. $\\sqrt{3}: 1$", "subject": "解析几何", "analysis": "设正六边形的半径是 $r$,则外接圆的半径是 $r$, 内切圆的半径是正六边形的边心距, 因而是 $\\frac{\\sqrt{3}}{2} r$,\n因而正六边形的外接圆的半径与内切圆的半径之比为 $2: \\sqrt{3}$. 故选 $C$."} {"id": "3894", "image": ["5479.jpg", "5480.jpg"], "answer": "C", "solution": "null", "level": "九年级", "question": "如图, $\\odot \\mathrm{O}$ 中, 弦 $\\mathrm{AB}$ 的长为 $8 \\mathrm{~cm}$, 圆心 $\\mathrm{O}$ 到 $\\mathrm{AB}$ 的距离为 $3 \\mathrm{~cm}$, 则 $\\odot \\mathrm{O}$ 的半径长为 ( )\n\n", "options": "A. $3 \\mathrm{~cm}$\nB. $4 \\mathrm{~cm}$\nC. $5 \\mathrm{~cm}$\nD. $6 \\mathrm{~cm}$", "subject": "解析几何", "analysis": "\n\n过点 $\\mathrm{O}$ 作 $\\mathrm{OC} \\perp \\mathrm{AB}$ 于 $\\mathrm{C}$, 连接 $\\mathrm{OA}, \\therefore \\mathrm{OC}=3 \\mathrm{~cm}, \\mathrm{AC}=\\frac{1}{2} \\mathrm{AB}=\\frac{1}{2} \\times 8=4(\\mathrm{~cm})$,\n\n$\\therefore$ 在 Rt $\\triangle \\mathrm{AOC}$ 中, $\\mathrm{OA}=\\sqrt{A C^{2}+O C^{2}}=5 \\mathrm{~cm}$. 故选: C."} {"id": "3919", "image": ["5530.jpg"], "answer": "D", "solution": "null", "level": "九年级", "question": "如图, 直线 1 是 $\\odot O$ 的切线, 点 $A$ 为切点, $B$ 为直线 1 上一点, 连接 $O B$ 交 $\\odot O$ 于点 $C, D$ 是优弧 $\\mathrm{AC}$ 上一点, 连接 $\\mathrm{AD}, \\mathrm{CD}$. 若 $\\angle \\mathrm{ABO}=40^{\\circ}$. 则 $\\angle \\mathrm{D}$ 的大小是 $(\\quad)$\n\n", "options": "A. $50^{\\circ}$\nB. $40^{\\circ}$\nC. $35^{\\circ}$\nD. $25^{\\circ}$", "subject": "解析几何", "analysis": "$\\because$ 直线 $l$ 是 $\\odot O$ 的切线, 点 $A$ 为切点, $\\therefore \\angle O A B=90^{\\circ}, \\therefore \\angle A O B=90^{\\circ}-40^{\\circ}=50^{\\circ}$, $\\therefore \\angle D=\\frac{1}{2} \\angle A O B=25^{\\circ}$. 故选 D."} {"id": "3921", "image": ["5531.jpg"], "answer": "D", "solution": "null", "level": "九年级", "question": "如图, 边长为 1 的小正方形构成的网格中, 半径为 1 的 $\\odot \\mathrm{O}$ 的圆心 $\\mathrm{O}$ 在格点上, 则 $\\angle \\mathrm{BED}$ 的正切值等于 $(\\quad)$\n\n", "options": "A. $\\frac{2 \\sqrt{5}}{5}$\nB. $\\frac{\\sqrt{5}}{5}$\nC. 2\nD. $\\frac{1}{2}$", "subject": "解析几何", "analysis": "$\\because \\angle \\mathrm{DAB}=\\angle \\mathrm{DEB}$,\n\n$\\therefore \\tan \\angle \\mathrm{DEB}=\\tan \\angle \\mathrm{DAB}=\\frac{1}{2}$, 故选 D."} {"id": "3937", "image": [], "answer": "A", "solution": "null", "level": "九年级", "question": "Rt $\\triangle A B C$ 中, $\\angle C=90^{\\circ}, \\cos A=\\frac{3}{5}, A C=6 \\mathrm{~cm}$, 那么 $B C$ 等于 ( )", "options": "A. $8 \\mathrm{~cm}$\nB. $\\frac{24}{5} \\mathrm{~cm}$\nC. $\\frac{18}{5} \\mathrm{~cm}$\nD. $\\frac{6}{5} \\mathrm{~cm}$", "subject": "解析几何", "analysis": "$\\because$ 在 $\\mathrm{Rt} \\triangle \\mathrm{ABC}$ 中, $\\angle \\mathrm{C}=90^{\\circ}, \\cos \\mathrm{A}=\\frac{A C}{A B}=\\frac{3}{5}, \\mathrm{AC}=6 \\mathrm{~cm}$,\n\n$\\therefore \\mathrm{AB}=10 \\mathrm{~cm}, \\quad \\therefore \\mathrm{BC}=\\sqrt{A B^{2}-A C^{2}}=8 \\mathrm{~cm}$. 故选: A."} {"id": "3938", "image": ["5549.jpg", "5550.jpg", "5551.jpg"], "answer": "D", "solution": "null", "level": "九年级", "question": "已知等腰 $\\triangle A B C$ 内接于 $\\odot O, \\odot O$ 的半径为 5 , 如果底边 $B C$ 的长为 6 , 则底角的正切值为 ( )\n\n", "options": "A. 3\nB. $\\frac{1}{3}$\nC. $\\frac{8}{3}$\nD. 3 或 $\\frac{1}{3}$", "subject": "解析几何", "analysis": "\n\n(1)\n\n\n\n(2)\n\n如图 (1), 可求得 $A D=O A+O D=9, \\quad \\tan \\angle A B D=\\frac{A D}{B D}=3$,\n\n如图 (2) , 可求得 $A D=O A-O D=1, \\tan \\angle A B D=\\frac{A D}{B D}=\\frac{1}{3}$, 综上, $\\tan \\angle A B D=3$ 或 $\\frac{1}{3}$. 故选: D."} {"id": "3940", "image": [], "answer": "C", "solution": "null", "level": "九年级", "question": "已知 $\\alpha$ 为锐角, 且 $\\tan \\alpha=3$, 则 $\\frac{\\sin \\alpha-2 \\cos \\alpha}{2 \\cos \\alpha+\\sin \\alpha}$ 的值为 ()", "options": "A. $\\frac{1}{3}$\nB. $\\frac{1}{4}$\nC. $\\frac{1}{5}$\nD. $\\frac{1}{6}$", "subject": "解析几何", "analysis": "$\\because \\alpha$ 为锐角, 且 $\\tan \\alpha=3$,\n\n$\\therefore \\frac{\\sin \\alpha-2 \\cos \\alpha}{2 \\cos \\alpha+\\sin \\alpha}=\\frac{\\tan \\alpha-2}{2+\\tan \\alpha}=\\frac{3-2}{2+3}=\\frac{1}{5}$. 故选 C."} {"id": "3942", "image": ["5554.jpg"], "answer": "B", "solution": "null", "level": "九年级", "question": "如图一艘轮船由海平面上 A 地出发向南偏西 $40^{\\circ}$ 的方向行驶 40 海里到达 B 地, 再由 B 地向北偏西 $20^{\\circ}$ 的方向行驶 40 海里到达 C 地, 则 A、C 两地相距()\n\n", "options": "A. 30 海里\nB. 40 海里\nC. 50 海里\nD. 60 海里", "subject": "解析几何", "analysis": "由题意得 $\\angle A B C=60^{\\circ}, A B=B C=40$\n\n$\\therefore \\triangle \\mathrm{ABC}$ 是等边三角形\n\n$\\therefore \\mathrm{AC}=\\mathrm{AB}=40$ 海里. 故选 $\\mathrm{B}$."} {"id": "3298", "image": [], "answer": "B", "solution": "null", "level": "九年级", "question": "小林同学在教学活动课中, 用一块长方形硬纸板在阳光下做投影实验, 通过观察, 发现这块长方形硬纸板在平整的地面上不可能出现的投影是 $(\\quad)$", "options": "A. 线段\nB. 三角形\nC. 平行四边形\nD. 正方形", "subject": "解析几何", "analysis": "解:将长方形硬纸的板面与投影线平行时, 形成的影子为线段;\n\n将长方形硬纸板与地面平行放置时, 形成的影子为矩形;\n\n将长方形硬纸板倾斜放置形成的影子为平行四边形;\n\n由物体同一时刻物高与影长成比例, 且长方形对边相等, 故得到的投影不可能是三角形.\n\n故选: $B$.\n\n根据平行投影的性质进行分析即可得出答案.\n\n本题考查了投影与视图的有关知识, 是一道与实际生活密切相关的热点试题, 灵活运用平行投影的性质是解题的关键."} {"id": "2976", "image": [], "answer": "A", "solution": "null", "level": "九年级", "question": "(本题 3 分) (2022-浙江$\\cdot$九年级专题练习) 已知抛物线 $y=a x^{2}+b x+c(a \\neq 0)$ 与 $x$ 轴的交点为 $A 1,0$ 和 $B(3,0)$, 点 $P_{1}\\left(x_{1}, y_{1}\\right), P_{2}\\left(x_{2}, y_{2}\\right)$ 是抛物线上不同于 $A, B$ 的两个点,记 $\\triangle P_{1} A B$ 的面积为 $S_{1}, \\triangle P_{2} A B$ 的面积为 $S_{2}$. 有下列结论: (1)当 $x_{1}>x_{2}+2$ 时, $S_{1}>S_{2}$;当 $x_{1}<2-x_{2}$ 时, $S_{1}\\left|x_{2}-2\\right|>1$ 时, $S_{1}>S_{2}$; (4)当 $\\left|x_{1}-2\\right|>\\left|x_{2}+2\\right|>1$时, $S_{1}x_2+2时与当x_1<2-x_2时无法确定P_1left(x_1,y_1right),P_2left(x_2,y_2right)在抛物线上的相对位置,故(1)和(2)都不正确;当left|x_1-2right|>left|x_2-2right|>1时,P_1left(x_1,y_1right)比P_2left(x_2,y_2right)离对称轴更远,且同在x轴上方或者下方,thereforeleft|y_1right|>left|y_2right|,thereforeS_1>S_2,故(3)正确;当left|x_1-2right|>left|x_2+2right|>1时,即在x轴上x_1到2的距离比x_2到-2的距离大,且都大于1,可知在x轴上x_1到2的距离大于1,x_2到2的距离不能确定,所以无法比较P_1left(x_1,y_1right)与P_2left(x_2,y_2right)谁离对称轴更远,故无法比较面积,故(4)错误;故选:A.【点睛】本题考查二次函数的图象与性质,掌握二次函数的对称性是解题的关键."} {"id": "2986", "image": [], "answer": "C", "solution": "null", "level": "九年级", "question": "(本题 3 分) (2020 浙江$\\cdot$九年级期末) 抛物线 $y=3(x-2)^{2}+5$ 的顶点坐标是", "options": "A. $(-2,5)$B. $(-2,-5)$C. $(2,5)$D. $(2,-5)$", "subject": "解析几何", "analysis": "C【解析】【分析】根据二次函数的性质y=a(x-h)^2+k的顶点坐标是(h,k)进行求解即可.【详解】because抛物线解析式为y=3(x-2)^2+5,therefore二次函数图象的顶点坐标是(2,5).故选C.【点睛】本题考查了二次函数的性质,根据抛物线的顶点式,可确定抛物线的开口方向,顶点坐标(对称轴),最大(最小)值,增减性等."} {"id": "2991", "image": [], "answer": "D", "solution": "null", "level": "九年级", "question": "(本题 3 分) (2020 浙江省温岭市第四中学九年级期中) 对于二次函数 $\\mathrm{y}=2(\\mathrm{x}-2)$${ }^{2}+1$, 下列说法中正确的是 $(\\quad)$", "options": "A. 图象的开口向下B. 函数的最大值为 1C. 图象的对称轴为直线 $\\mathrm{x}=-2$D. 当 $\\mathrm{x}<2$ 时 $\\mathrm{y}$ 随 $\\mathrm{x}$ 的增大而减小", "subject": "解析几何", "analysis": "D【解析】【分析】根据二次函数的图象和性质,可以判断各个选项中的说法是否正确.【详解】二次函数y=2(x-2)2+1,a=2>0,therefore该函数的图象开口向上,故选项A错误,函数的最小值是y=1,故选项B错误,图象的对称轴是直线x=2,故选项C错误,当x<2时y随x的增大而减小,故选项D正确,故选D.【点睛】考查二次函数的性质、二次函数的最值,解答本题的关键是明确题意,利用二次函数的性质解答."} {"id": "2992", "image": [], "answer": "B", "solution": "null", "level": "九年级", "question": "(本题 3 分)(2020-浙江$\\cdot$浣江教育九年级期中)将抛物线 $y=2 x^{2}$ 向上平移 3 个单位长度, 再向右平移 2 个单位长度, 所得到的抛物线为 $(\\quad)$.", "options": "A. $y=2(x+2)^{2}+3$B. $y=2(x-2)^{2}+3$;C. $y=2(x-2)^{2}-3$D. $y=2(x+2)^{2}-3$.", "subject": "解析几何", "analysis": "B【解析】【分析】根据抛物线图像的平移规律“左加右减,上加下减”即可确定平移后的抛物线解析式.【详解】解:将抛物线y=2x^2向上平移3个单位长度,再向右平移2个单位长度,得到的抛物线的解析式为y=2(x-2)^2+3,故选B.【点睛】本题考查了二次函数的平移规律,熟练掌握其平移规律是解题的关键."} {"id": "2993", "image": [], "answer": "C", "solution": "null", "level": "九年级", "question": "(本题 3 分) (2020-浙江温州$\\cdot$九年级阶段练习) 烟花厂某种礼炮的升空高度 $h(m)$与飞行时间 $t(s)$ 的关系式是 $h=-2 t^{2}+20 t+1$, 若这种礼炮在点火升空到最高点处引爆, 则从点火升空到引爆需要的时间为", "options": "A. $3 s$B. $4 s$C. $5 s$D. $10 s$", "subject": "解析几何", "analysis": "C【解析】【分析】将h关于t的函数关系式变形为顶点式,即可得出升到最高点的时间,从而得出结论.【详解】解:becauseh=-2t^2+20t+1=-2(t-5)^2+51,therefore当t=5时,礼炮升到最高点.故选:C.【点睛】本题考查了二次函数的应用,解题的关键是将二次函数的关系式变形为顶点式.本题属于基础题,难度不大,解决该题型题目时,将函数的关系式进行变换找出顶点坐标即可."} {"id": "2994", "image": ["3895.jpg", "3896.jpg", "3897.jpg", "3898.jpg"], "answer": "D", "solution": "null", "level": "九年级", "question": "(本题 3 分) (2017$\\cdot$全国$\\cdot$九年级课时练习) 在平面直角坐标系中, 二次函数 $y=a(x-h)^{2} \\quad(a \\neq 0)$ 的图象可能是 ( $)$", "options": "A.B.C.D.", "subject": "解析几何", "analysis": "D【解析】【分析】根据二次函数y=a(x-h)^2(aneq0)的顶点坐标为(h,0),它的顶点坐标在x轴上,即可解答.【详解】二次函数y=a(x-h)^2(aneq0)的顶点坐标为(left.h,0right),它的顶点坐标在x轴上,故选D."} {"id": "2997", "image": [], "answer": "A", "solution": "null", "level": "九年级", "question": "(本题 3 分) (2020$\\cdot$浙江省临海市大成中学九年级期中) 已知点 $A\\left(1, y_{1}\\right), B\\left(2, y_{2}\\right)$ 在抛物线 $y=-(x+1)^{2}+2$ 上, 则下列结论正确的是()", "options": "A. $2>y_{1}>y_{2}$B. $2>y_{2}>y_{1}$C. $y_{1}>y_{2}>2$D. $y_{2}>y_{1}>2$", "subject": "解析几何", "analysis": "A【解析】【分析】分别计算自变量为1和2对应的函数值,然后对各选项进行判断.【详解】当x=1时,y1=-(x+1)^2+2=-(1+1)^2+2=-2;当x=2时,y_1=-(x+1)^2+2=-(2+1)^2+2=-7;所以2>y_1>y_2.故选A【点睛】此题考查二次函数顶点式以及二次函数的性质,解题关键在于分析函数图象的情况"} {"id": "3009", "image": [], "answer": "C", "solution": "null", "level": "九年级", "question": "(本题 3 分) (2020 浙江绍兴$\\cdot$九年级期中) 对于二次函数 $y=(x-1)^{2}+2$ 的图象, 下列说法正确的是 $(\\quad)$", "options": "A. 开口向下B. 对称轴是 $x=-1$C. 顶点坐标是 $(1$,2) D. 与 $x$ 轴有两个交点", "subject": "解析几何", "analysis": "C【解析】【分析】根据抛物线的性质由a=1得到图象开口向上,根据顶点式得到顶点坐标为(1,2),对称轴为直线x=1,从而可判断抛物线与x轴没有公共点.【详解】解:二次函数y=(x-1)^2+2的图象开口向上,顶点坐标为(1,2),对称轴为直线x=1,抛物线与x轴没有公共点.故选:C.【点睛】本题考查了二次函数的性质,掌握二次函数的顶点式是解题的关键,即在y=a(x-h)^2+k中,其顶点坐标为(h,k),对称轴为x=h.当a>0时,抛物线开口向上,当a<0时,抛物线开口向下."} {"id": "3015", "image": [], "answer": "A", "solution": "null", "level": "九年级", "question": "(本题 3 分)(2021 $\\cdot$九年级期中)将 $y=x^{2}$ 向上平移 2 个单位后所得的抛物线的解析式为 $(\\quad)$", "options": "A. $y=x^{2}+2$B. $y=x^{2}-2$C. $y=(x+2)^{2}$D. $y=(x-2)^{2}$", "subject": "解析几何", "analysis": "A【解析】【详解】试题分析:抛物线y=x^2的顶点坐标为(0,0),把点(0,0)向上平移2个单位得到的点的坐标为(0,2),所以平移后的抛物线的解析式为y=x^2+2.故选A.考点:二次函数图象与几何变换."} {"id": "3016", "image": ["3912.jpg", "3913.jpg", "3914.jpg", "3915.jpg"], "answer": "A", "solution": "null", "level": "九年级", "question": "(本题 3 分)(2019$\\cdot$浙江$\\cdot$九年级期末)在同一直角坐标系中 $y=a x^{2}+b$ 与 $\\mathrm{y}=\\mathrm{ax}+\\mathrm{b}(\\mathrm{a} \\neq 0, \\mathrm{~b} \\neq 0)$ 图象大致为 $(\\quad)$", "options": "A.B.C.D.", "subject": "解析几何", "analysis": "A【解析】【分析】本题由一次函数y=ax+b图象得到字母系数的正负,再与二次函数y=ax^2+b的图象相比较看是否一致.【详解】解:A、由抛物线可知,a<0,~b<0,由直线可知,a<0,~b<0,故本选项正确;B、由抛物线可知,a<0,~b>0,由直线可知,a>0,~b>0,故本选项错误;C、由抛物线可知,a>0,~b<0,由直线可知,a>0,~b>0,故本选项错误;D、由抛物线可知,a>0,b>0,由直线可知,a<0,b>0,故本选项错误.故选A.【点睛】本题考查了一次函数和二次函数的图象cdot解答该题时,一定要熟记一次函数、二次函数的图象的性质."} {"id": "3017", "image": [], "answer": "D", "solution": "null", "level": "九年级", "question": "(本题 3 分) (2019- 浙江台州$\\cdot$九年级期末) 抛物线 $y=x^{2}+2 x-3$ 的最小值是", "options": "A. 3B. -3C. 4D. -4", "subject": "解析几何", "analysis": "D【解析】【分析】把y=x^2+2x-3配方变成顶点式,求出顶点坐标即可得抛物线的最小值.【详解】becausey=x2+2x-3=(x+1)2-4therefore顶点坐标为(-1,-4),becausea=1>0,therefore开口向上,有最低点,有最小值为-4.故选D.【点睛】本题考查二次函数最值的求法:求二次函数的最大(小)值有三种方法,第一种可由图象直接得出,第二种是配方法,第三种是公式法,熟练掌握并灵活运用适当方法是解题关键,"} {"id": "3018", "image": ["3916.jpg"], "answer": "D", "solution": "null", "level": "九年级", "question": "(本题 3 分) (2019$\\cdot$浙江台州$\\cdot$九年级期末) 已知二次函数 $y=a x^{2}+b x+c$ 的部分图象如图所示, 下列关于此函数图象的描述中,错误的是", "options": "A. 对称轴是直线 $x=1$B. 当 $\\mathrm{x}<0$ 时, 函数 $\\mathrm{y}$ 随 $\\mathrm{x}$ 增大而增大C. 图象的顶点坐标是 $(1,4)$D. 图象与 $x$ 轴的另一个交点是 $(4,0)$", "subject": "解析几何", "analysis": "D【解析】【分析】利用二次函数的图像与性质,判断选项的正误即可.【详解】由函数图像可知,对称轴是直线x=1故选项A正确;当x<0时,函数y随x增大而增大,故选项B正确;图象的顶点坐标是(1,4),故选项C正确;图象与x轴的另一个交点是(3,0),故选项D错误.故选D【点睛】本题考查了二次函数的图像与性质,熟练掌握性质是解题的关键."} {"id": "3019", "image": ["3917.jpg", "3918.jpg"], "answer": "D", "solution": "null", "level": "九年级", "question": "(本题 3 分)(2018$\\cdot$浙江$\\cdot$九年级期中)对于题目“一段抛物线 $L: y=-x(x-3)+c$ $(0 \\leq x \\leq 3)$ 与直线 $1: y=x+2$ 有唯一公共点, 若 $\\mathrm{c}$ 为整数, 确定所有 $\\mathrm{c}$ 的值, \"甲的结果是 $\\mathrm{c}=1$, 乙的结果是 $\\mathrm{c}=3$ 或 4 , 则", "options": "A. 甲的结果正确B. 乙的结果正确C. 甲、乙的结果合在一起才正确D. 甲、乙的结果合在一起也不正确", "subject": "解析几何", "analysis": "D【解析】【分析】分两种情况进行讨论,(1)当抛物线与直线相切,triangle=0求得c=1,(2)当抛物线与直线不相切,但在0leqxleq3上只有一个交点时,找到两个临界值点,可得c=3,4,5,故c=3,4,5【详解】解:because抛物线L:y=-x(x-3)+c(0leqxleq3)与直线1:y=x+2有唯一公共点therefore(1)如图1,抛物线与直线相切,图1联立解析式leftbeginarrayly=-x(x-3)+cy=x+2endarrayright.得x^2-2x+2-c=0Delta=(-2)^2-4(2-c)=0解得:c=1,当c=1时,相切时只有一个交点,和题目相符所以不用舍去;(2)如图2,抛物线与直线不相切,但在0leqxleq3上只有一个交点图2此时两个临界值分别为(0,2)和(3,5)在抛物线上thereforec的最小值=2,但取不到,c的最大值=5,能取到therefore20$, (2) $4 a c2$ ,其中正确的结论的个数是( $\\quad$ )", "options": "A. 1B. 2C. 3D. 4", "subject": "解析几何", "analysis": "C【解析】【详解】(1)because抛物线开口向下,quadthereforea<0,because抛物线的对称轴为直线x=-fracb2a=-1,quadthereforeb=2a<0,because抛物线与y轴的交点在x轴上方,thereforec>0,thereforeabc>0,所以(1)正确,符合题意;(2)because抛物线与x轴有2个交点,thereforetriangle=b^2-4ac>0,therefore4ac0,thereforea-b+c>2,所以(4)正确,符合题意.故选C."} {"id": "3043", "image": [], "answer": "A", "solution": "null", "level": "九年级", "question": "(本题 3 分) (2022$\\cdot$浙江衢州 $\\cdot$ 九年级期末) 已知 $\\odot O$ 的半径是 3 , 若 $O A=3$, 则点 $A$", "options": "A. 在 $\\odot O$ 上B. 在 $\\odot O$ 内C. 在 $\\odot O$ 外D. 无法判定", "subject": "解析几何", "analysis": "A【解析】【分析】根据点与圆的位置关系判断即可.【详解】becauseodotO的半径是3,OA=3,3=3,therefore点A在odotO上,故选:A.【点睛】本题考查了点与圆的位置关系,能熟记点与圆的位置关系的内容是解此题的关键,已知odotO的半径为r,点P到圆心O的距离为d,(1)当dr时,点P在odotO外,反之亦然."} {"id": "3044", "image": ["3934.jpg", "3935.jpg"], "answer": "D", "solution": "null", "level": "九年级", "question": "(本题 3 分)(2022$\\cdot$浙江$\\cdot$九年级专题练习)如图,在平面直角坐标系中, $O$ 为坐标原点, 半径为 2 的 $\\odot O$ 与 $x$ 轴的负半轴交于点 $\\mathrm{A}$, 点 $B$ 是 $\\odot O$ 上一动点, 点 $P$ 为弦 $A B$的中点, 直线 $y=-\\frac{4}{3} x+4$ 与 $x$ 轴、 $y$ 轴分别交于点 $C, E$, 则 $\\triangle P C E$ 面积的最小值为 $(\\quad)$", "options": "A. 5B. 6C. $\\frac{25}{4}$D. $\\frac{11}{2}$", "subject": "解析几何", "analysis": "D【解析】【分析】连接OP,根据点P为弦AB的中点,可得点P在以OA为直径的圆上,以OA为直径作odotN,过N点作直线NFperpCE于F,交odotN于M,则odotN上到直线CE上最短的距离是MF,则可得triangleMCE即trianglePCE的面积最小,根据一次函数的性质,求得OE=4,quadOC=3根据勾股定理可得CE=5,再根据odotO的半径为2,可知OA=2,NM=NO=1,NC=4,由等积法可求得NF=frac165,MF=NF-NM=frac115,根据S_triangleMCE=frac12CEcdotMF可求得面积最小是frac112.【详解】解:连接OP,如图,because点P为弦AB的中点,thereforeOPperpABthereforeangleAPO=90^circ,therefore点P在以OA为直径的圆上,以OA为直径作odotN,过N点作直线NFperpCE于F,交odotN于M,则odotN上到直线CE上最短的距离是MF,此时,triangleMCE即trianglePCE的面积最小,当x=0时,y=-frac43x+4=4,则E(0,4),当y=0时,-frac43x+4=0,解得x=3,则C(3,0),thereforeOE=4,quadOC=3thereforeCE=sqrtOC^2+OE^2=sqrt3^2+4^2=5,becauseodotO的半径为2,thereforeOA=2,thereforeNM=NO=1,quadNC=ON+OC=3+1=4由等积法可知:frac12NCcdotOE=frac12ECcdotNFthereforeNF=fracNCcdotOEEC=frac4times45=frac165thereforeMF=NF-NM=frac165-1=frac115,thereforeS_wedgeMCE=frac12CEcdotMF=frac12times5timesfrac115=frac112,即trianglePCE的面积最小是frac112,故选:D【点睛】本题考查了垂径定理,圆周角定理,一次函数的性质和等积法等知识点,属性相关性质是解题的关键."} {"id": "3087", "image": ["4001.jpg", "4002.jpg"], "answer": "C", "solution": "null", "level": "九年级", "question": "(本题 3 分)(2022$\\cdot$浙江宁波$\\cdot$三模)已知 $\\odot O$ 的直径 $C D=10 \\mathrm{~cm}, A B$ 是 $\\odot O$ 的弦,$A B \\perp C D$, 垂足为 $M$, 且 $A B=8 \\mathrm{~cm}$, 则 $A C$ 的长为 ( )", "options": "A. $2 \\sqrt{5} \\mathrm{~cm}$B. $4 \\sqrt{3} \\mathrm{~cm}$C. $2 \\sqrt{5} \\mathrm{~cm}$ 或 $4 \\sqrt{5} \\mathrm{~cm}$D. $2 \\sqrt{3} \\mathrm{~cm}$ 或$4 \\sqrt{3} \\mathrm{~cm}$", "subject": "解析几何", "analysis": "C【分析】先画好一个圆,标上直径CD,已知AB的长为8~cm,可知分为两种情况,第一种情况AB与OD相交,第二种情况AB与OC相交,利用勾股定理即可求出两种情况下的AC的长;【详解】连接AC,AO,图1图2because圆O的直径CD=10~cm,ABperpCD,AB=8~cm,thereforeAM=frac12AB=frac12times8=4~cm,OD=OC=5~cm,当C点位置如图1所示时,becauseOA=5~cm,AM=4~cm,CDperpABthereforeOM=sqrtOA^2-AM^2=sqrt5^2-4^2=3~cm,thereforeCM=OC+OM=5+3=8~cm,thereforeAC=sqrtAM^2+CM^2=sqrt4^2+8^2=4sqrt5~cm;当C点位置如图2所示时,同理可得OM=3~cm,becauseOC=5~cmthereforeMC=5-3=2~cm,在RttriangleAMC中,AC=sqrtAM^2+CM^2=sqrt4^2+2^2=2sqrt5~cm.故选C.【点睛】本题考查垂径定理和勾股定理,根据题意正确画出图形进行分类讨论,熟练运用垂径定理是解决本题的关键."} {"id": "3114", "image": ["4043.jpg", "4044.jpg"], "answer": "D", "solution": "null", "level": "九年级", "question": "(本题 3 分)(2022$\\cdot$浙江$\\cdot$金华市婺城区教育局教研室模拟预测)如图是中国古代数学家赵爽用来证明勾股定理的弦图的示意图, 它是由四个全等的直角三角形和一个小正方形 $E F G H$ 组成, 恰好拼成一个大正方形 $A B C D$ 。 连结 $E G$ 并延长交 $B C$ 于点$M$. 若 $A B=\\sqrt{13}, E F=1$, 则 $G M$ 有长为 $(\\quad)$", "options": "A. $\\frac{2 \\sqrt{2}}{5}$B. $\\frac{2 \\sqrt{2}}{3}$C. $\\frac{3 \\sqrt{2}}{4}$D. $\\frac{4 \\sqrt{2}}{5}$", "subject": "解析几何", "analysis": "D【分析】添加辅助线,过F点作FI//HM,通过证明两组三角形相似,得到FI和GM的两个关系式,从而求解GM.【详解】如图所示,过F点作FI//HM,交BC于点I,because证明勾股定理的弦图的示意图是由四个全等的直角三角形和一个小正方形EFGH组成thereforeangleAEB=90^circ,BF=AE=CG,CF=BE,FG=EF=1,EG=sqrt2又becauseAB=sqrt13,EF=1thereforeAE^2+BE^2=AB^2,即BF^2+(BF+1)^2=(sqrt13)^2解得BF=2或BF=-3(舍去)thereforeBF=AE=CG=2,CF=BE=3becauseFI//HMthereforetriangleCGMsimtriangleCFI,triangleBFIsimtriangleBEMthereforefracFIGM=fracCFCG=frac32,quadfracEMFI=fracBEBF=frac32thereforeFI=frac32GM,quadfracEG+GMFI=fracsqrt2+GMFI=frac32thereforefracsqrt2+GMfrac32GM=frac32解得:GM=frac45sqrt2经检验:GM=frac45sqrt2符合题意,故选:D.【点睛】本题考查了相似三角形和勾股定理.本题的关键在于添加辅助线,建立所求线段与已知条件之间的联系."} {"id": "3129", "image": ["4073.jpg", "4074.jpg", "4075.jpg", "4076.jpg", "4077.jpg"], "answer": "A", "solution": "null", "level": "九年级", "question": "(本题 3 分) (2018 浙江杭州 $\\cdot$九年级期末) 如图, 在边长为 1 的格点图形中, 与 $\\triangle \\mathrm{ABC}$ 相似的是 $(\\quad)$", "options": "A.C.B.D.", "subject": "解析几何", "analysis": "A【分析】根据勾股定理求出triangleABC的三边,并求出三边之比,然后根据网格结构利用勾股定理求出三角形的三边之比,再根据三边对应成比例,两三角形相似选择答案.【详解】解:已知给出的三角形的各边分别为sqrt2、2、sqrt10,所以triangleABC的三边之比为sqrt2:2:sqrt10=1:sqrt2:sqrt5,A、三角形的三边分别为1,sqrt2,sqrt5,三边之比为1:sqrt2:sqrt5,故A选项正确;B、三角形的三边分别为sqrt2,sqrt5,3,三边之比为sqrt2:sqrt5:3,故B选项错误;C、三角形的三边分别为1,sqrt5,2sqrt2,三边之比为1:sqrt5:2sqrt2,故C选项错误;D、三角形的三边分别为:2,sqrt5,sqrt13,三边之比为2:sqrt5:sqrt13,故D选项错误.故选A.【点睛】题主要考查了相似三角形的判定与网格结构的知识,根据网格结构分别求出各三角形的三条边的长,并求出三边之比是解题的关键."} {"id": "3133", "image": ["4086.jpg", "4087.jpg"], "answer": "D", "solution": "null", "level": "九年级", "question": "(本题 3 分) (2022- 浙江唯州$\\cdot$二模) 如图, 正方形 $A B C D$ 的顶点 $B$ 在 $x$ 轴上, 点 $A 、$ 点 $C$ 在反比例函数 $y=\\frac{k}{x}(k>0, x>0)$ 图象上. 若直线 $B C$ 的函数表达式为 $y=\\frac{1}{2}$ $x-4$, 则反比例函数表达式为 $(\\quad)$", "options": "A. $y=\\frac{6}{x}$B. $y=\\frac{12}{x}$C. $y=\\frac{16}{x}$D. $y=\\frac{24}{x}$", "subject": "解析几何", "analysis": "D【分析】解方程求得B(8,0),G(0,-4),得到OB=8,OG=4,过A作AEperpx轴于E,过C作CFperpx轴于F,根据正方形的性质得到AB=BC,angleABC=90^circ,根据全等三角形的性质得到AE=BF,BE=CF,根据相似三角形的性质以及反比例函数图象上点的坐标特征即可得到结论.【详解】解:在y=frac12x-4中,令y=0,则x=8,令x=0,则y=-4,thereforeB(8,0),G(0,-4),thereforeOB=8,OG=4,过A作AEperpx轴于E,过C作CFperpx轴于F,because四边形ABCD是正方形,thereforeAB=BC,angleABC=90^circ,thereforeangleEAB+angleABE=angleABE+angleCBF=90^circ,thereforeangleEAB=angleCBF在triangleAEB与triangleBFC中,leftbeginarraycangleAEB=angleBFC=90^circangleBAE=angleFBCAB=BCendarrayright.,thereforetriangleAEBcongtriangleBFC(AAS),thereforeAE=BF,BE=CF,becauseangleBOG=angleBFC=90^circ,angleOBG=angleCBFthereforetriangleOBGsimtriangleFBCthereforefracCFBF=fracOGOB=frac12,therefore设CF=x,BF=2x,thereforeAE=2x,BE=x,thereforeA(8-x,2x),C(8+2x,x)because点A,点C在反比例函数y=frackx(k>0,x>0)图象上,therefore2x(8-x)=x(8+2x),thereforex=2,x=0(不合题意舍去),thereforeA(6,4),thereforek=4times6=24,therefore反比例函数表达式为y=frac24x,故选:D.【点睛】本题考查了反比例函数的综合题,待定系数法求反比例函数的解析式,一次函数的性质,全等三角形的判定和性质,正方形的性质,相似三角形的判定和性质,正确地作出辅助线是解题的关键."} {"id": "3157", "image": ["4123.jpg", "4124.jpg"], "answer": "D", "solution": "null", "level": "九年级", "question": "(本题 3 分)(2022$\\cdot$浙江绍兴-一模)如图, 在平面直角坐标系中, 矩形 $A B C O$ 的边 $O A$ 在 $x$ 轴上, 边 $O C$ 在 $y$ 轴上, 点 $B$ 的坐标为 $(1,3)$, 将矩形沿对角线 $A C$ 折叠, 使点 $B$ 落在 $D$ 点的位置, 且交 $y$ 轴交于点 $E$, 则点 $D$ 的坐标是 $(\\quad)$", "options": "A. $\\left(-\\frac{3}{5}, \\frac{8}{3}\\right)$B. $\\left(-\\frac{3}{5}, 2\\right)$C. $\\left(-\\frac{4}{5}, \\frac{14}{5}\\right)$D. $\\left(-\\frac{4}{5}, \\frac{12}{5}\\right)$", "subject": "解析几何", "analysis": "D【分析】过D作DFperpAO于F,根据折叠可以证明triangleCDEcongtriangleAOE,然后利用全等三角形的性质得到OE=DE,OA=CD=1,设OE=m,那么CE=3-m,DE=m,利用勾股定理即可求出m,然后利用已知条件可以证明triangleAEOsimtriangleADF,而AD=AB=3,接着利用相似三角形的性质即可求出DF、AF的长度,也就求出了点D的坐标.【详解】解:如图,过D作DFperpAO于F,because点B的坐标为(1,3),thereforeAO=1,AB=3,根据折叠可知:CD=OA,而angleD=angleAOE=90^circ,angleDEC=angleAEO,thereforetriangleCDEcongtriangleAOE,thereforeOE=DE,OA=CD=1,设OE=m,那么CE=3-m,DE=m,therefore在RttriangleDCE中,CE^2=DE^2+CD^2,therefore(3-m)^2=m^2+1^2,解得m=frac43,becauseDFperpAF,thereforeDF//EO,thereforetriangleAEOsimtriangleADF,而AD=AB=3,thereforeAE=CE=3-frac43=frac53,thereforefracAEAD=fracEODF=fracAOAF即frac53=frac43=fracAODF,thereforeDF=frac125,AF=frac95thereforeOF=frac95-1=frac45,thereforeD的坐标为left(-frac45,frac125right).故选:D.【点睛】此题主要考查了图形的折叠问题,也考查了坐标与图形的性质,解题的关键是把握折叠的隐含条件,利用隐含条件得到全等三角形和相似三角形,然后利用它们的性质即可解决问题."} {"id": "3158", "image": ["4125.jpg"], "answer": "D", "solution": "null", "level": "九年级", "question": "(本题 3 分)(2022$\\cdot$浙江$\\cdot$九年级专题练习)如图, 在平行四边形 $A B C D$ 中, 对角线 $A C 、 B D$ 交于点 $O, M$ 为 $A D$ 中点, 连接 $C M$ 交 $B D$ 于点 $N$, 则 $S_{\\triangle M D N}: S_{\\triangle} B C D=$", "options": "A. $1: 3$B. $1: 5$C. $2: 3$D. $1: 6$", "subject": "解析几何", "analysis": "D【分析】先证triangleBCNsimtriangleDMN,得出BN:DN=BC:DM=2:1,从而得出fracS_triangleMNDS_triangleBCN=left(fracDNBNright)^2=left(frac12right)^2=frac14,fracS_triangleCNDS_triangleBCN=fracDNBN=frac12,所以StriangleMND=frac14StriangleBCN,StriangleCND=frac12StriangleBCD,然后代入StriangleMDN:StriangleBCD即可求解.【详解】解:because四边形ABCD是平行四边形,thereforeAD=BC,AD//BC,thereforetriangleBCNsimtriangleDMNthereforeBN:DN=BC:DMbecauseM为AD中点,AD=BC,thereforeBC=AD=2DM,thereforeBN:DN=2:1,thereforefracS_triangleMNDS_triangleBCN=left(fracDNBNright)^2=left(frac12right)^2=frac14,quadfracS_triangleCNDS_triangleBCN=fracDNBN=frac12thereforeStriangleMND=frac14StriangleBCN,StriangleCND=frac12StriangleBCDbecauseStriangleBCD=StriangleBCN+StriangleCND=StriangleBCN+frac12StriangleBCN=frac32quadStriangleBCN,thereforeStriangleMDN:StriangleBCD=frac14StriangleBCN:frac32StriangleBCN=1:6,故选:D.【点睛】本题考查平行四边形的性质,相似三角形判定与性质,熟练掌握平行四边形的性质、相似三角形判定与性质是解题的关键."} {"id": "3179", "image": ["4151.jpg", "4152.jpg"], "answer": "D", "solution": "null", "level": "九年级", "question": "(本题 3 分) (2021-信达外国语学校九年级期中) 如图, 在 $\\triangle A B C$ 中, $D$ 为边 $B C$ 上一点, 已知 $\\frac{B D}{D C}=\\frac{5}{3}, E$ 为 $A D$ 的中点, 延长 $B E$ 交 $A C$ 于 $F$, 则 $\\frac{A F}{A C}=(\\quad)$", "options": "A. $\\frac{3}{5}$B. $\\frac{5}{8}$C. $\\frac{3}{13}$D. $\\frac{5}{13}$", "subject": "解析几何", "analysis": "D【分析】过点 $D$ 作 $D G / / A C$ 交 $B F$ 于 $G$, 先证 $\\triangle A E F \\cong \\triangle D E G$, 得 $D G=A F$, 再由平行线的性质可得对应线段成比例, 从而得答案.【详解】解:过点 $D$ 作 $D G / / A C$ 交 $B F$ 于 $G$, 如图所示,$\\therefore \\angle E A F=\\angle E D G, \\angle E F A=\\angle E G D$,$\\because E$ 是 $A D$ 的中点,$\\therefore A E=D E$$\\therefore \\triangle A E F \\cong \\triangle D E G(A A S)$,$\\therefore D G=A F$,$\\because D G / / A C, \\frac{B D}{D C}=\\frac{5}{3}$,$\\therefore \\frac{D G}{C F}=\\frac{B D}{B C}=\\frac{5}{8}$,$\\therefore \\frac{A F}{C F}=\\frac{5}{8}$,$\\therefore \\frac{A F}{A C}=\\frac{5}{13}$故选 D.【点睛】此题考查了全等三角形的判定与性质、平行线分线段成比例的性质定理, 熟练掌握这些判定与性质是解答此题的关键."} {"id": "3193", "image": ["4167.jpg"], "answer": "D", "solution": "null", "level": "九年级", "question": "(本题 3 分) (2022$\\cdot$浙江舟山$\\cdot$九年级期末) 如图, 在直角坐标系中, 点 $\\mathrm{A}$ 在第一象限内, 点 $B$ 在 $x$ 轴正半轴上, 以点 $O$ 为位似中心, 在第三象限内与 $\\triangle O A B$ 的位似比为 $\\frac{1}{3}$的位似图形 $\\triangle O C D$. 若点 $C$ 的坐标为 $\\left(-1,-\\frac{2}{3}\\right)$, 则点 $\\mathrm{A}$ 的坐标为 ( )", "options": "A. $\\left(\\frac{2}{3}, 2\\right)$B. $\\left(\\frac{2}{3}, 2\\right)$C. $\\left(3, \\frac{2}{3}\\right)$D. $(3,2)$", "subject": "解析几何", "analysis": "D【分析】根据关于以原点为位似中心的对应点的坐标的关系,把C点的横纵坐标都乘以-3即可.【详解】在第三象限内与triangleOAB的位似比为frac13的位似图形triangleOCD,相当于在第一象限内与triangleOCD的位似比为3:1的位似图形triangleOAB,则以点O为位似中心,位似比为3:1,而点C的坐标为left(-1,-frac23right),thereforeC点的对应点A的坐标为(3,2),故D正确.故选:D.【点睛】本题考查了位似变换,在平面直角坐标系中,如果位似变换是以原点为位似中心,位似比为k,那么位似图形对应点的坐标的比等于k或-k."} {"id": "3198", "image": ["4182.jpg"], "answer": "C", "solution": "null", "level": "九年级", "question": "(本题 3 分)(2022$\\cdot$浙江温州$\\cdot$九九年级期末)如图, $l_{1}, l_{2}, l_{3}$ 是一组平行线, 直线 $A C, D F$ 分别与这组平行线依次相交于点 $A, B, E, F$. 若 $\\frac{A B}{B C}=\\frac{2}{3}$, 则 $\\frac{E F}{D F}$ 的值为", "options": "A. $\\frac{2}{5}$B. $\\frac{1}{2}$C. $\\frac{3}{5}$D. $\\frac{2}{3}$", "subject": "解析几何", "analysis": "C【分析】根据平行线分线段成比例定理(两条直线被一组平行线(不少于3条)所截,截得对应线段的长度成比例)及比例的性质即可得.【详解】解:becausel_1//l_2//l_3且直线AC、DF均被平行线所截,thereforefracABBC=fracDEEF=frac23,therefore设DE=2k,则EF=3k,thereforefracEFDF=fracEFDE+EF=frac3k2k+3k=frac35,故选:C.【点睛】题目主要考查平行线分线段成比例定理及比例的性质,深刻理解平行线分线段成比例定理是解题关键."} {"id": "3359", "image": ["4408.jpg", "4409.jpg"], "answer": "C", "solution": "null", "level": "九年级", "question": "如图, 四边形 $A B C D$ 内接于 $\\odot O, A B$ 为直径, $A D=C D$, 过点 $D$ 作 $D E \\perp A B$ 于点 $E$, 连接 $A C$交 $D E$ 于点 $F$. 若 $\\sin \\angle C A B=\\frac{3}{5}, D F=5$, 则 $B C$ 的长为 ()", "options": "A. 8\n B. 10\n C. 12\n D. 16", "subject": "解析几何", "analysis": "【分析】\n\n本题考查了圆周角定理: 在同圆或等圆中, 同弧或等弧所对的圆周角相等, 都等于这条弧所对的圆心角的一半. 推论: 半圆 (或直径)所对的圆周角是直角, $90^{\\circ}$ 的圆周角所对的弦是直径. 也考查了解直角三角形.\n\n连接 $B D$, 如图, 先利用圆周角定理证明 $\\angle A D E=\\angle D A C$ 得到 $F D=F A=5$, 再根据正弦的定义计算出 $E F=3$, 则 $A E=4, D E=8$, 接着证明 $\\triangle A D E \\sim \\triangle D B E$, 利用相似比得到 $B E=16$, 所以 $A B=$ 20 , 然后在 $R t \\triangle A B C$ 中利用正弦定义计算出 $B C$ 的长.\n\n【解答】\n\n解: 连接 $B D$, 如图,\n\n$\\because A B$ 为直径,\n\n\n$\\therefore \\angle A D B=\\angle A C B=90^{\\circ}$,\n\n$\\because A D=C D$,\n\n$\\therefore \\angle D A C=\\angle D C A$,\n\n而 $\\angle D C A=\\angle A B D$,\n\n$\\therefore \\angle D A C=\\angle A B D$,\n\n$\\because D E \\perp A B$,\n\n$\\therefore \\angle A B D+\\angle B D E=90^{\\circ}$,\n\n而 $\\angle A D E+\\angle B D E=90^{\\circ}$,\n\n$\\therefore \\angle A B D=\\angle A D E$,\n\n$\\therefore \\angle A D E=\\angle D A C$,\n\n$\\therefore F D=F A=5$,\n\n在Rt $\\triangle A E F$ 中, $\\because \\sin \\angle C A B=\\frac{E F}{A F}=\\frac{3}{5}$,\n\n$\\therefore E F=3$,\n\n$\\therefore A E=\\sqrt{5^{2}-3^{2}}=4, D E=5+3=8$,\n\n$\\because \\angle A D E=\\angle D B E, \\angle A E D=\\angle B E D$,\n\n$\\therefore \\triangle A D E \\sim \\triangle D B E$,\n\n$\\therefore D E: B E=A E: D E$, 即 $8: B E=4: 8$,\n\n$\\therefore B E=16$,\n\n$\\therefore A B=4+16=20$,\n\n在Rt $\\triangle A B C$ 中, $\\because \\sin \\angle C A B=\\frac{B C}{A B}=\\frac{3}{5}$,\n\n$\\therefore B C=20 \\times \\frac{3}{5}=12$.\n\n故选 $C$."} {"id": "3366", "image": ["4421.jpg", "4422.jpg", "4423.jpg"], "answer": "D", "solution": "null", "level": "九年级", "question": "如图, 在 $R t \\triangle B A D$ 中, 延长斜边 $B D$ 到点 $C$, 使 $D C=\\frac{1}{2} B D$, 连接 $A C$, 若 $\\tan B=\\frac{5}{3}$, 则 $\\tan \\angle C A D$ 的值为 $(\\quad)$", "options": "A. $\\frac{\\sqrt{3}}{3}$\n B. $\\frac{\\sqrt{3}}{5}$\n C. $\\frac{1}{3}$\n D. $\\frac{1}{5}$", "subject": "解析几何", "analysis": "解法 1 : 由 $\\tan B=\\frac{5}{3}$, 设 $A D=5 k, A B=3 k$, 如图, 过点 $D$ 作 $D E / / A B$ 交 $A C$ 于点 $E$, 则 $\\angle A D E=90^{\\circ}, \\frac{D E}{A B}=\\frac{C D}{B C}$.\n\n$\\because D C=\\frac{1}{2} B D, \\quad \\therefore \\frac{C D}{B C}=\\frac{1}{3}$\n\n$\\therefore D E=\\frac{1}{3} A B$\n$\\therefore \\tan \\angle C A D=\\frac{D E}{A D}=\\frac{1}{3} \\times \\frac{A B}{A D}=\\frac{1}{5}$.\n\n\n\n解法2: 如图, 延长 $A D$, 过点 $C$ 作 $C E \\perp A D$, 垂足为 $E$.\n\n$\\because \\tan B=\\frac{5}{3}$, 即 $\\frac{A D}{A B}=\\frac{5}{3}$ ,\n\n$\\therefore$ 设 $A D=5 x$, 则 $A B=3 x$.\n\n$\\because \\angle C D E=\\angle B D A, \\quad \\angle C E D=\\angle B A D, \\quad \\therefore \\triangle C D E \\sim \\triangle B D A$,\n\n$\\therefore \\frac{C E}{A B}=\\frac{D E}{A D}=\\frac{C D}{B D}=\\frac{1}{2}$\n\n$\\therefore C E=\\frac{3}{2} x, D E=\\frac{5}{2} x$,\n\n$\\therefore A E=\\frac{15}{2} x$\n\n$\\therefore \\tan \\angle C A D=\\frac{E C}{A E}=\\frac{1}{5}$.\n\n"} {"id": "3370", "image": [], "answer": "C", "solution": "null", "level": "九年级", "question": "在Rt $\\triangle A B C$ 中, $\\angle C=90^{\\circ}, \\cos A=\\frac{\\sqrt{3}}{2}, \\angle B$ 的平分线 $B D$ 交 $A C$ 于点 $D$, 若 $A D=16$, 则 $B C$ 的长为 $(\\quad)$", "options": "A. 6\nB. 8\nC. $8 \\sqrt{3}$\nD. 12", "subject": "解析几何", "analysis": "略"} {"id": "3373", "image": ["4431.jpg", "4432.jpg"], "answer": "A", "solution": "null", "level": "九年级", "question": "如图, 已知 $\\triangle A B C$ 中, $\\angle B=90^{\\circ}, D, E$ 分别为 $B C, A C$ 的中点, 连结 $D E$, 过 $D$ 作 $A C$ 的平行线与 $\\angle C A B$ 的角平分线交于点 $F$, 连结 $E F$, 若 $E F \\perp D F, A C=2$, 则 $\\angle D E F$ 的正弦值为 ( )\n\n", "options": "A. $\\frac{\\sqrt{5}-1}{2}$\nB. $\\frac{\\sqrt{5}+1}{4}$\nC. $\\frac{\\sqrt{5}-1}{4}$\nD. $\\frac{3+\\sqrt{5}}{4}$", "subject": "解析几何", "analysis": "【分析】\n\n本题考查三角形中位线定理, 勾股定理, 全等三角形的性质及判定, 锐角三角函数等知识, 解题的关键是学会利用参数构建方程解决问题, 属于中考填空题中的压轴题. 延长 $D F$ 交 $A B$ 于 $H$, 作 $F T \\perp$ $A B$ 于 $T$. 设 $D F=x$. 构建方程求出 $x$ 即可解决问题.\n\n【解答】\n\n解: 延长 $D F$ 交 $A B$ 于 $H$, 作 $F T \\perp A B$ 于 $T$. 设 $D F=x$.\n\n\n\n$\\because D H / / A C, B D=D C$,\n\n$\\therefore B H=A H$,\n\n$\\therefore D H=\\frac{1}{2} A C=1$,\n\n$\\therefore F H=1-x$,\n\n$\\because F A$ 平分 $C A B, F E \\perp A C, F T \\perp A B$,\n\n$\\therefore F E=F T$,\n\n$\\therefore R t \\triangle C F E \\cong R t \\triangle A F T(H L)$,\n\n$\\therefore A E=A T=1$,\n\n$\\because \\angle F A E=\\angle A F H=\\angle F A H$,\n\n$$\n\\therefore F H=A H=B H=1-x\n$$\n\n$\\therefore T H=1-(1-x)=x$,\n\n$\\because \\angle C=\\angle B D H=\\angle T F H$,\n\n$\\therefore \\tan \\angle C=\\tan \\angle T F H$,\n\n$\\therefore \\frac{2-2 x}{2}=\\frac{x}{1-x}$,\n解得 $x=\\frac{3-\\sqrt{5}}{2}$ 或 $\\frac{3+\\sqrt{5}}{2}$ (舍弃),\n\n$\\therefore A B=2-2 x=\\sqrt{5}-1$,\n\n$\\because D E=\\frac{1}{2} A B=\\frac{\\sqrt{5}-1}{2}$,\n\n$\\therefore \\sin \\angle D E F=\\frac{D F}{D E}=\\frac{\\sqrt{5}-1}{2}$.\n\n故选 $A$."} {"id": "3374", "image": ["4433.jpg", "4434.jpg"], "answer": "C", "solution": "null", "level": "九年级", "question": "如图, $O A=4$, 线段 $O A$ 的中点为 $B$, 点 $P$ 在以 $O$ 为圆心, $O B$ 为半径的圆上运动, $P A$ 的中点为 $Q$. 当点 $Q$ 也落在 $\\odot O$ 上时, $\\cos \\angle O Q B$ 的值等于 $(\\quad)$\n\n", "options": "A. $\\frac{1}{2}$\nB. $\\frac{1}{3}$\nC. $\\frac{1}{4}$\nD. $\\frac{2}{3}$", "subject": "解析几何", "analysis": "【分析】\n\n本题目考查了等腰三角形的性质、三角形中位线的性质、锐角三角函数, 关键是要根据点 $Q$ 是 $A P$ 的\n中点来分析作辅助线解答.\n\n【解答】\n\n解:当点 $Q$ 也落在 $\\odot O$ 上时, 如图, 连结 $O P 、 O Q 、 B Q$.\n\n\n\n$\\therefore Q$ 为 $A P$ 的中点, $B$ 为 $O A$ 的中点,\n\n$\\therefore B Q=\\frac{1}{2} P O=1$,\n\n作 $O H \\perp B Q$ 于 $H$,\n\n$\\because O B=O Q$,\n\n$\\therefore Q H=\\frac{1}{2} B Q=\\frac{1}{2}$,\n\n$\\therefore \\cos \\angle O Q B=\\frac{Q H}{O Q}=\\frac{1}{4}$,\n\n故选 $C$."} {"id": "3392", "image": [], "answer": "A", "solution": "null", "level": "九年级", "question": "在 $R t \\triangle A B C$ 中, $\\angle C=90^{\\circ}, \\cos B=\\frac{1}{2}$, 则 $\\sin A$ 的值为 ( )", "options": "A. $\\frac{1}{2}$\nB. $\\frac{\\sqrt{2}}{2}$\nC. $\\frac{\\sqrt{3}}{2}$\nD. $\\sqrt{3}$", "subject": "解析几何", "analysis": "解: 在Rt $\\triangle A B C$ 中, $\\angle C=90^{\\circ}, \\cos B=\\frac{1}{2}$,\n\n则 $\\sin A=\\cos B=\\frac{1}{2}$,\n\n故选: $A$.\n\n利用互余两角三角函数的关系判断即可.\n\n此题考查了互余两角三角函数的关系, 以及特殊角的三角函数值, 熟练掌握各自的性质是解本题的关键."} {"id": "3399", "image": [], "answer": "B", "solution": "null", "level": "九年级", "question": "下列结论中: (1) $\\triangle A B C$ 的内切圆半径为 $r, \\triangle A B C$ 的周长为 $L$, 则 $\\triangle A B C$ 的面积是 $\\frac{1}{2} L r$; (2)同时抛郑两枚质地均匀的硬币, 两枚硬币全部正面向上的概率为 $\\frac{1}{2}$; (3)圆内接平行四边形是矩形; (4)无论 $p$ 取何值, 方程 $(x-3)(x-2)-p^{2}=0$ 总有两个不等的实数根.其中正确的结论有 $(\\quad)$", "options": "A. 4 个\nB. 3 个\nC. 2 个\nD. 1 个", "subject": "解析几何", "analysis": "解: (1) $\\triangle A B C$ 的内切圆半径为 $r, \\triangle A B C$ 的周长为 $L$, 则 $\\triangle A B C$ 的面积是 $\\frac{1}{2} L r$, 故(1)正确;\n\n(2)同时抛掷两枚质地均匀的硬币,两枚硬币全部正面向上的概率为 $\\frac{1}{4}$, 故(2)错误;\n\n(3)圆内接平行四边形是矩形; 故③正确;\n\n(4) $\\because$ 方程 $(x-3)(x-2)-p^{2}=0$,\n\n$\\therefore \\Delta=25-4\\left(6-p^{2}\\right)=1+p^{2}>0$,\n\n$\\therefore$ 无论 $p$ 取何值, 总有两个不等的实数根. 故(4)正确;\n\n故选: $B$.\n\n利用圆的内切圆的知识, 平行四边形的性质, 概率, 根的判别式的知识, 依次判断可求解.\n\n本题考查了平行四边形的性质, 矩形的判定和性质, 根的判定式等知识, 灵活运用这些性质解决问题是解题的关键."} {"id": "3416", "image": [], "answer": "C", "solution": "null", "level": "九年级", "question": "在平面直角坐标系中, 以点 $(2,1)$ 为圆心, 1 为半径的圆与坐标轴的位置关系为 ( )", "options": "A. 与 $x$ 轴相离、与 $y$ 轴相切\nB. 与 $x$ 轴、 $y$ 轴都相离\nC. 与 $x$ 轴相切、 $y$ 轴相离\nD. 与 $x$ 轴、 $y$ 轴都相切", "subject": "解析几何", "analysis": "解: $\\because$ 点 $(2,1)$ 为圆心, 1 为半径的圆,\n\n则有 $1=1,2>1$ ,\n\n$\\therefore$ 这个圆与 $x$ 轴相切, 与 $y$ 轴相离.\n\n故选:C.\n\n本题应将该点的横纵坐标分别与半径对比, 大于半径时, 则该圆与坐标轴相离; 若等于半径时,则该圆与坐标轴相切.\n\n本题考查了直线与圆的位置关系、坐标与图形性质. 直线与圆相切, 圆心到直线的距离等于半径;与圆相离, 圆心到直线的距离大于半径."} {"id": "3417", "image": ["4514.jpg", "4515.jpg"], "answer": "D", "solution": "null", "level": "九年级", "question": "如图, 菱形 $O A B C$ 的顶点 $A, B, C$ 在 $\\odot O$ 上, 过点 $B$ 作 $\\odot O$ 的切线交 $O A$ 的延长线于点 $D$.若 $\\odot O$ 的半径为 1 , 则 $B D$ 的长为 $(\\quad)$", "options": "A. 1\nB. 2\nC. $\\sqrt{2}$\nD. $\\sqrt{3}$\n\n", "subject": "解析几何", "analysis": "【分析】\n\n本题考查了切线的性质, 菱形的性质, 等边三角形的判定和性质, 熟练切线的性质定理是解题的关键. 连接 $O B$, 根据菱形的性质得到 $O A=A B$, 求得 $\\angle A O B=60^{\\circ}$, 根据切线的性质得到 $\\angle D B O=$ $90^{\\circ}$ ,即可得到结论.\n\n【解答】\n\n解:连接 $O B$,\n\n$\\because$ 四边形 $O A B C$ 是菱形,\n\n\n$\\therefore O A=A B$,\n\n$\\because O A=O B$,\n\n$\\therefore O A=A B=O B$,\n\n$\\therefore \\angle A O B=60^{\\circ}$,\n\n$\\because B D$ 是 $\\odot O$ 的切线,\n\n$\\therefore \\angle D B O=90^{\\circ}$,\n\n$\\because O B=1$,\n\n$\\therefore B D=\\sqrt{3} O B=\\sqrt{3}$,\n\n故选 $D$."} {"id": "3420", "image": ["4520.jpg"], "answer": "C", "solution": "null", "level": "九年级", "question": "如图, 从圆 $O$ 外一点 $P$ 引圆 $O$ 的两条切线 $P A, P B$, 切点分别为 $A, B$, 如果 $\\angle A P B=60^{\\circ}$, $P A=8$, 那么弦 $A B$ 的长是 $(\\quad)$\n\n", "options": "A. 4\nB. $4 \\sqrt{3}$\nC. 8\nD. $8 \\sqrt{3}$", "subject": "解析几何", "analysis": "解: $\\because P A, P B$ 为 $\\odot O$ 的切线,\n\n$\\therefore P A=P B$,\n\n$\\because \\angle A P B=60^{\\circ}$,\n\n$\\therefore \\triangle A P B$ 为等边三角形,\n\n$\\therefore A B=P A=8$.\n\n故选 $C$.\n\n先利用切线长定理得到 $P A=P B$, 再利用 $\\angle A P B=60^{\\circ}$ 可判断 $\\triangle A P B$ 为等边三角形, 然后根据等边三角形的性质求解.\n\n本题考查切线长定理, 以及等边三角形的判定和性质."} {"id": "3434", "image": ["4554.jpg", "4555.jpg"], "answer": "B", "solution": "null", "level": "九年级", "question": "如图, 在 $R t \\triangle A B C$ 中, $\\angle C=90^{\\circ}, A C=4, B C=3$, 点 $O$ 是 $A B$ 的三等分点, 半圆 $O$ 与 $A C$ 相切, $M, N$ 分别是 $B C$ 与半圆弧上的动点, 则 $M N$ 的最小值和最大值之和是 ( )\n\n", "options": "A. 5\nB. 6\nC. 7\nD. 8", "subject": "解析几何", "analysis": "【分析】\n\n本题考查切线的性质、三角形中位线定理等知识, 解题的关键是正确找到点 $M N$ 取得最大值、最小值时的位置, 属于中考常考题型.\n\n设 $\\odot O$ 与 $A C$ 相切于点 $D$, 连接 $O D$, 作 $O P \\perp B C$ 垂足为 $P$ 交 $\\odot O$ 于 $F$, 此时垂线段 $O P$ 最短, $M N$ 最小值为 $O P-O F=\\frac{5}{3}$, 当 $N$ 在 $A B$ 边上时, $M$ 与 $B$ 重合时, $M N$ 最大值 $=\\frac{10}{3}+1=\\frac{13}{3}$, 由此不难解决问题.\n\n【解答】\n\n解:如图, 设 $\\odot O$ 与 $A C$ 相切于点 $D$, 连接 $O D$, 作 $O P \\perp B C$ 垂足为 $P$ 交 $\\odot O$ 于 $F$,\n\n\n\n此时垂线段 $O P$ 最短, $P F$ 最小值为 $O P-O F$,\n\n$\\because A C=4, B C=3$,\n\n$$\n\\therefore A B=5\n$$\n\n$\\because \\angle O P B=90^{\\circ}$,\n\n$$\n\\therefore O P / / A C\n$$\n\n$\\because$ 点 $O$ 是 $A B$ 的三等分点,\n\n$\\therefore O B=\\frac{2}{3} \\times 5=\\frac{10}{3}, \\frac{O P}{A C}=\\frac{O B}{A B}=\\frac{2}{3}$,\n$\\therefore O P=\\frac{8}{3}$,\n\n$\\because \\odot O$ 与 $A C$ 相切于点 $D$,\n\n$\\therefore O D \\perp A C$,\n\n$\\therefore O D / / B C$\n\n$\\therefore \\frac{O D}{B C}=\\frac{O Q}{A B}=\\frac{1}{3}$,\n\n$\\therefore O D=1$,\n\n$\\therefore M N$ 最小值为 $O P-O F=\\frac{8}{3}-1=\\frac{5}{3}$,\n\n如图, 当 $N$ 在 $A B$ 边上时, $M$ 与 $B$ 重合时, $M N$ 经过圆心, 经过圆心的弦最长,\n\n$M N$ 最大值 $=\\frac{10}{3}+1=\\frac{13}{3} ,$\n\n$\\therefore M N$ 长的最大值与最小值的和是 6.\n\n故选 $B$."} {"id": "3470", "image": ["4633.jpg", "4634.jpg"], "answer": "A", "solution": "null", "level": "九年级", "question": "如图, 在矩形 $A B C D$ 中, $A B=4, A D=5, A D, A B, B C$ 分别与 $\\odot O$ 相切于 $E, F, G$ 三点, 过点 $D$ 作 $\\odot O$ 的切线交 $B C$ 于点 $M$, 切点为 $N$, 则 $D M$ 的长为 $(\\quad)$\n\n", "options": "A. $\\frac{13}{3}$\nB. $\\frac{9}{2}$\nC. $\\frac{4}{3} \\sqrt{13}$\nD. $2 \\sqrt{5}$", "subject": "解析几何", "analysis": "解:如图, 连接 $O E, O F, O N, O G$,\n\n\n\n在矩形 $A B C D$ 中,\n\n$\\because \\angle A=\\angle B=90^{\\circ}, C D=A B=4, A D, A B, B C$ 分别与 $\\odot O$ 相切于 $E, F, G$ 三点,\n\n$\\therefore \\angle A E O=\\angle A F O=\\angle O F B=\\angle B G O=90^{\\circ}, O E=O F=O N=O G$,\n\n$\\therefore$ 四边形 $A F O E$ 和四边形 $F B G O$ 是正方形,\n\n$\\therefore A F=B F=A E=B G=2$,\n\n$\\therefore D E=3$,\n\n$\\because D M$ 是 $\\odot O$ 的切线, $B C$ 是 $\\odot O$ 的切线, $A D$ 是 $\\odot O$ 的切线,\n\n$\\therefore D N=D E=3, M N=M G$,\n\n$\\therefore C M=5-2-M N=3-M N, D M=3+M N$,\n\n在Rt $\\triangle D M C$ 中, $D M^{2}=C D^{2}+C M^{2}$,\n\n$\\therefore(3+N M)^{2}=(3-N M)^{2}+4^{2}$,\n\n$\\therefore N M=\\frac{4}{3}$,\n\n$\\therefore D M=3+\\frac{4}{3}=\\frac{13}{3}$.\n\n故选: $A$."} {"id": "3498", "image": [], "answer": "B", "solution": "null", "level": "九年级", "question": "在 $R t \\triangle A B C$ 中, $\\angle C=90^{\\circ}, a: b=3: 4$, 运用计算器计算, $\\angle A$ 的度数为 $\\left(\\right.$ 精确到 $\\left.1^{\\circ}\\right)(\\quad)$", "options": "A. $30^{\\circ}$\nB. $37^{\\circ}$\nC. $38^{\\circ}$\nD. $39^{\\circ}$", "subject": "解析几何", "analysis": "【分析】\n\n本题考查在直角三角形中解题, 根据角的正弦值求出三角形的角度,根据题中所给的条件, 在直角三角形中解题, 根据角的正弦值与三角形边的关系, 可求出各边的长, 然后求出 $\\angle A$.\n\n【解答】\n\n解: $\\because a: b=3: 4$,\n\n$\\therefore$ 可设 $a=3 x, b=4 x, x>0$,\n\n由勾股定理得 $c=5 x$,\n\n$\\therefore \\sin A=\\frac{a}{c}=\\frac{3 x}{5 x}=0.6$,\n\n利用计算器可求得 $\\angle A \\approx 37^{\\circ}$.\n\n故选 $B$"} {"id": "3500", "image": ["4701.jpg", "4702.jpg"], "answer": "B", "solution": "null", "level": "九年级", "question": "如图, 线段 $A B$ 经过 $\\odot O$ 的圆心, $A C, B D$ 分别与 $\\odot O$ 相切于点 $C, D$. 若 $A C=B D=4, \\angle A=$ $45^{\\circ}$, 则 $\\overparen{C D}$ 的长为 $(\\quad)$\n\n", "options": "A. $\\pi$\nB. $2 \\pi$\nC. $2 \\sqrt{2} \\pi$\nD. $4 \\pi$", "subject": "解析几何", "analysis": "如答图, 连结 $C O, D O$.\n\n\n\n$\\because A C, B D$ 分别与 $\\odot O$ 相切于点 $C, D$,\n\n$\\therefore \\angle A C O=\\angle B D O=90^{\\circ}$,\n\n$\\therefore \\angle A O C=\\angle A=45^{\\circ}$,\n\n$\\therefore C O=A C=4$.\n\n$\\because A C=B D, \\quad C O=D O$,\n\n$\\therefore \\triangle A C O \\cong \\triangle B D O(S A S)$,\n\n$\\therefore \\angle D O B=\\angle A O C=45^{\\circ}$,\n\n$\\therefore \\angle D O C=180^{\\circ}-\\angle D O B-\\angle A O C=180^{\\circ}-45^{\\circ}-45^{\\circ}=90^{\\circ}$,\n$\\therefore l_{\\overparen{C D}}=\\frac{90 \\pi \\times 4}{180}=2 \\pi$."} {"id": "3516", "image": ["4734.jpg", "4735.jpg"], "answer": "D", "solution": "null", "level": "九年级", "question": "如图, $\\triangle A B C$ 的顶点都是正方形网格中的格点, 则 $\\cos ^{\\angle A C B}$ 等于 ( )\n\n", "options": "A. $\\frac{4}{5}$\nB. $\\frac{3}{5}$\nC. $\\frac{3}{4}$\nD. $\\frac{\\sqrt{10}}{10}$", "subject": "解析几何", "analysis": "【分析】\n\n此题考查了勾股定理, 以及锐角三角函数定义, 熟练掌握勾股定理是解本题的关键.\n\n如图所示, 利用勾股定理求出 $B C$ 和 $A C$ 的长, 再利用锐角三角函数定义\n\n\n即可求出 $\\cos \\angle A C B$ 的值.\n\n【解答】\n\n解: $\\because A B=5, A C=\\sqrt{3^{2}+1^{2}}=10, B C=\\sqrt{3^{2}+4^{2}}=5$,\n\n$\\therefore B A=B C$,\n\n$\\therefore \\angle A C B=\\angle C A B$,\n\n$\\therefore \\cos \\angle A C B=\\cos \\angle C A B=\\frac{1}{\\sqrt{10}}=\\frac{\\sqrt{10}}{10}$,\n\n故选 $D$."} {"id": "3534", "image": ["4765.jpg"], "answer": "C", "solution": "null", "level": "九年级", "question": "如图, $\\odot O$ 的直径 $A B$ 垂直于弦 $C D$, 垂足为 $E, C D=4, \\tan C=\\frac{1}{2}$, 则 $A B$ 的长为 $(\\quad)$", "options": "A. 2.5\nB. 4\nC. 5\nD. 10\n\n", "subject": "解析几何", "analysis": "解: $\\because A B \\perp C D, C D=4$,\n\n$\\therefore C E=D E=2$,\n\n$\\because \\angle B=\\angle C, \\tan C=\\frac{1}{2}$,\n\n$\\therefore \\tan B=\\frac{1}{2}$,\n\n$\\therefore A E=1, B E=4$,\n\n$\\therefore A B=A E+B E=1+4=5$,\n\n故选: $C$.\n\n首先根据垂径定理和 $C D$ 的长求得 $C E$ 和 $D E$ 的长, 然后根据同弧所对的圆周角相等确定 $\\angle B=\\angle C$,\n\n根据正切的定义求得 $A E$ 和 $B E$ 的长即可求得答案.\n\n本题考查了圆周角定理及垂径定理, 锐角三角函数的定义知识, 解题的关键是根据垂径定理求得 $C E$ 和 $D E$ 的长, 难度不大."} {"id": "3548", "image": ["4803.jpg", "4804.jpg"], "answer": "C", "solution": "null", "level": "九年级", "question": "如图, 在矩形 $A B C D$ 中, $A B=3, A D=4, C E$ 平分 $\\angle A C B$,与对角线 $B D$ 相交于点 $N, F$ 是线段 $C E$ 的中点, 则下列结论中正确的有个. ( )\n\n(1) $O F=\\frac{5}{6}$; (2) $O N=\\frac{25}{26} ;$ (3) $S_{\\triangle C O N}=\\frac{15}{13} ;$ (4) $\\sin \\angle A C E=\\frac{5}{13}$.\n\n", "options": "A. 1\nB. 2\nC. 3\nD. 4", "subject": "解析几何", "analysis": "解: (1)如图, 过点 $E$ 作 $E H \\perp A C$ 于 $H$,\n\n\n\n$\\because A B=C D=3, A D=B C=4$,\n\n$\\therefore A C=\\sqrt{A B^{2}+B C^{2}}=\\sqrt{9+16}=5$,\n\n$\\because$ 四边形 $A B C D$ 是矩形,\n\n$\\therefore A O=C O=D O=B O=\\frac{5}{2}$,\n\n$\\because C E$ 平分 $\\angle A C B, E H \\perp A C, \\angle A B C=90^{\\circ}$,\n\n$\\therefore B E=E H$,\n\n$\\because S_{\\triangle A B C}=S_{\\triangle A E C}+S_{\\triangle B C E}$,\n\n$\\therefore \\frac{1}{2} \\times A B \\times B C=\\frac{1}{2} \\times A C \\times E H+\\frac{1}{2} \\times B C \\times B E$,\n\n$\\therefore 3 \\times 4=5 \\times E H+4 \\times E H$,\n\n$\\therefore E H=\\frac{4}{3}=B E$,\n\n$\\therefore A E=A B-B E=\\frac{5}{3}$,\n\n$\\because F$ 是线段 $C E$ 的中点, $A O=C O$,\n\n$\\therefore O F=\\frac{1}{2} A E=\\frac{5}{6}$,\n\n故(1)正确;\n\n(2) $\\because O F / / A B$,\n\n$\\therefore \\triangle O N F \\sim \\triangle B N E$,\n\n$\\therefore \\frac{O F}{B E}=\\frac{O N}{B N}=\\frac{\\frac{5}{6}}{\\frac{4}{3}}=\\frac{5}{8}$,\n\n$\\therefore O N=\\frac{5}{8} B N$,\n\n$\\because O N+B N=B O=\\frac{5}{2}$,\n$\\therefore B N=\\frac{20}{13}, N O=\\frac{25}{26}$,\n\n故(2)正确;\n\n(3) $\\because S_{\\triangle B O C}=\\frac{1}{4} S_{\\text {矩形 } A B C D}$,\n\n$\\therefore S_{\\triangle B O C}=\\frac{1}{4} \\times 3 \\times 4=3$,\n\n$\\because O N=\\frac{5}{8} B N$,\n\n$\\therefore S_{\\triangle C O N}=\\frac{5}{13} \\times 3=\\frac{15}{13}$,\n\n故(3)正确;\n\n(4) $\\because B E=\\frac{4}{3}, B C=4$,\n\n$\\therefore E C=\\sqrt{B E^{2}+B C^{2}}=\\sqrt{\\frac{16}{9}+16}=\\frac{4 \\sqrt{10}}{3}$,\n\n$\\therefore \\sin \\angle A C E=\\frac{E H}{E C}=\\frac{\\frac{4}{3}}{\\frac{4 \\sqrt{10}}{3}}=\\frac{\\sqrt{10}}{10}$,\n\n故(4)错误,\n\n故选: $C$.\n\n利用面积法可求 $B E$ 的长, 由三角形的中位线定理可求 $O F$ 的长, 可判断 1 ; ; 由相似三角形的判定和性质可求 $O N$ 的长, 可判断(2); 由面积关系可求 $\\triangle O N C$, 可判断(3); 由勾股定理可求 $E C$ 的长,由锐角三角函数可求 $\\sin \\angle A C E$ 的值, 可判断(4), 即可求解.\n\n本题是四边形综合题, 考查了矩形的性质, 三角形中位线定理, 相似三角形的判定和性质, 勾股定理, 锐角三角函数等知识, 灵活运用这些性质解决问题是解题的关键."} {"id": "3559", "image": ["4829.jpg"], "answer": "B", "solution": "null", "level": "九年级", "question": "如图, 抛物线 $y=\\frac{1}{4}(x+2)(x-8)$ 与 $x$ 轴交于 $A, B$ 两点, 与 $y$ 轴交于点 $C$, 顶点为 $M$, 以 $A B$为直径作 $\\odot D$.下列结论:(1)抛物线的对称轴是直线 $x=3$; (2) $\\odot D$ 的面积为 $16 \\pi$; (3)抛物线上存在点 $E$, 使四边形 $A C E D$ 为平行四边形; (4)直线 $C M$ 与 $\\odot D$ 相切. 其中正确结论的个数是 ( )\n\n", "options": "A. 1\nB. 2\nC. 3\nD. 4", "subject": "解析几何", "analysis": "【分析】本题主要考查了二次函数的综合题, 关键是熟练掌握二次函数的性质及圆的性质. 先根据二次函数确定 $A$ 和 $B$ 的坐标, 从而可得对称轴, 即可判断(1); 根据坐标先求圆的直径, 可得圆的面积, 从而判断(2); 然后利用二次函数解析式求 $C$ 和 $E$ 的坐标, 从而可得 $A D$, 进而判断(3); 确定顶点坐标, 根据勾股定理和勾股定理的逆定理得出直角, 利用切线的判定定理判断(4)即可.\n\n【解答】\n\n解析: 因为在 $y=\\frac{1}{4}(x+2)(x-8)$ 中, 当 $y=0$ 时, $x=-2$ 或 $x=8$, 所以点 $A$ 的坐标为 $(-2,0)$, 点 $B$ 的坐标为 $(8,0)$.\n所以抛物线的对称轴为直线 $x=\\frac{-2+8}{2}=3$, 故 (1)正确;\n\n因为 $\\odot D$ 的直径为 $8-(-2)=10$, 即半径为 5 , 所以 $\\odot D$ 的面积为 $25 \\pi$, 故 (2)错误;\n\n在 $y=\\frac{1}{4}(x+2)(x-8)=\\frac{1}{4} x^{2}-\\frac{3}{2} x-4$ 中, 当 $x=0$ 时, $y=-4$, 所以点 $C$ 的坐标为 $(0,-4)$.\n\n当 $y=-4$ 时, 由 $\\frac{1}{4} x^{2}-\\frac{3}{2} x-4=-4$, 解得 $x_{1}=0, x_{2}=6$, 所以点 $E$ 的坐标为 $(6,-4)$.\n\n则 $C E=6$.\n\n因为 $A D=3-(-2)=5$, 所以 $A D \\neq C E$, 所以四边形 $A C E D$ 不是平行四边形, 故 (3)错误;\n\n因为 $y=\\frac{1}{4} x^{2}-\\frac{3}{2} x-4=\\frac{1}{4}(x-3)^{2}-\\frac{25}{4}$, 所以点 $M$ 的坐标为 $\\left(3,-\\frac{25}{4}\\right)$.\n\n作 $C H \\perp D M$ 于点 $H$, 连接 $C D$, 则点 $H$ 的坐标为 $(3,-4)$.\n\n因为点 $C$ 的坐标为 $(0,-4)$, 所以 $C H=3, H M=\\frac{25}{4}-4=\\frac{9}{4}$.\n\n在Rt $\\triangle C H M$ 中, $C M^{2}=C H^{2}+H M^{2}=\\frac{225}{16}$.\n\n因为 $C D^{2}=O D^{2}+O C^{2}=25, D M^{2}=\\left(\\frac{25}{4}\\right)^{2}=\\frac{625}{16}$,\n\n$C M^{2}+C D^{2}=\\frac{225}{16}+25=\\frac{625}{16}=D M^{2}$,\n\n所以 $\\triangle C D M$ 是直角三角形, $\\angle D C M=90^{\\circ}$,\n\n所以 $D C \\perp C M$. 又 $C D$ 是 $\\odot D$ 的半径, 所以 $M C$ 与 $\\odot D$ 相切, 故 (4)正确.\n\n故选 $B$."} {"id": "1", "image": ["10109.jpg", "10110.jpg"], "answer": "B", "solution": "null", "level": "九年级", "question": "如图, 拋物线 $y=a x^{2}+b x+c$ 与 $x$ 轴正半轴交于 $A, B$ 两点\n\n\n点 $B(4,0)$. (1) $a b c>0$; (2) $4 a+b>0$; (3) $b^{2}-4 a c \\geq 0$; (4)若 $A B \\geq 3$, 则 $4 b+3 c>0$;\n\n若抛物线的对称轴是直线 $x=3, k$ 为任意实数, 则 $a(k-3)(k+3) \\leq b(3-k)$; 则上述结论中, 正确的个数是()\n\n\n\n", "options": "A. 5 个\nB. 4 个\nC. 3 个\nD. 2 个\n\n##", "subject": "解析几何", "analysis": "根据函数的开口,判断 $a$ 的符号,根据对称轴,判断 $b$ 的符号,根据于 $y$ 轴交点,判断 $c$ 的符号,即可判断(1); 把点 $B(4,0)$ 代入 $y=a x^{2}+b x+c$ 得 $0=16 a+4 b+c$, 整理得到\n\n$4 a+b=-\\frac{c}{4}$, 即可判断(2); 根据该函数图象与 $x$ 轴的交点个数, 即可判断(3); 根据 $A B \\geq 3$ 可得 $O A \\leq 1$, 则当 $x=1$ 时, $y \\geq 0$, 把 $x=1$ 和 $x=4$ 分别代入, 消去 $a$, 即可判断(4); 根据函数开口向下, 对称轴为直线 $x=3$, 可知函数的最大值为 $x=3$ 对应的函数值, 则当 $x=k$ 时, 函数值不大于 $x=3$ 对应的函数值, 即可判断(5).\n\n【详解】解: $\\because$ 函数开口向下, $\\therefore a<0$,\n$\\because$ 函数对称轴在 $y$ 轴左侧,\n\n$\\therefore-\\frac{b}{2 a}>0$, 则 $b>0$,\n\n$\\because$ 函数图象与 $y$ 轴相交于负半轴,\n\n$\\therefore c<0$,\n\n$\\therefore a b c>0$, 故(1)正确;\n\n把点 $B(4,0)$ 代入 $y=a x^{2}+b x+c$ 得:\n\n$0=16 a+4 b+c$,\n\n$\\therefore 16 a+4 b=-c$, 则 $4 a+b=-\\frac{c}{4}$,\n\n$\\because c<0$\n\n$\\therefore 4 a+b=-\\frac{c}{4}>0$, 故(2)正确;\n\n$\\because$ 该函数图象与 $x$ 轴有 2 个交点,\n\n$\\therefore b^{2}-4 a c>0$, 故(3)不正确;\n\n$\\because B(4,0)$,\n\n$\\therefore O B=4$,\n\n$\\because A B \\geq 3$,\n\n$\\therefore O A \\leq 1$,\n\n$\\therefore$ 当 $x=1$ 时, $y \\geq 0$,\n\n把 $x=1$ 代入 $y=a x^{2}+b x+c$ 得: $a+b+c \\geq 0$, 即 $16 a+16 b+16 c \\geq 0$\n\n$\\because 16 a+4 b+c=0$,\n\n$\\therefore 12 b+15 c \\geq 0$, 整理得: $4 b+3 c \\geq-2 c$,\n\n$\\because c<0$,\n\n$\\therefore 4 b+3 c \\geq-2 c>0$, 故(4)正确;\n\n把 $x=3$ 代入 $y=a x^{2}+b x+c$ 得: $y=9 a+3 b+c$,\n\n$\\because$ 拋物线的对称轴是直线 $x=3$, 函数图象开口向下,\n\n$\\therefore$ 该函数的顶点坐标为: $(3,9 a+3 b+c)$, 即该函数最大值为 $9 a+3 b+c$,\n\n当 $x=k$ 时, $y=a k^{2}+b k+c$,\n\n$\\therefore a k^{2}+b k+c \\leq 9 a+3 b+c$,\n\n整理得: $a k^{2}-9 a \\leq 3 b-b k$, 即 $a(k+3)(k-3) \\leq b(3-k)$, 故(5)正确;\n\n综上:正确的有(1)(2)(4)(5), 共 4 个;\n\n故选: B.\n\n【点睛】本题主要考查了二次函数的图象和性质, 解题的关键是熟练掌握二次函数的图象和系数的关系.\n\n"} {"id": "11", "image": [], "answer": "D", "solution": "null", "level": "九年级", "question": "下列关于二次函数 $y=-x^{2}+3$ 的图像说法中错误的是 ( )\n", "options": "A. 它的对称轴是直线 $x=0$\nB. 它的图像有最高点\nC. 它的顶点坐标是 $(0,3)$\nD. 在对称轴的左侧, $y$ 随着 $x$ 的增大而减小\n", "subject": "解析几何", "analysis": "已知二次函数的解析式可以找到相应的系数, 根据系数与图象的关系可以找到二次函数的图象的特征: $a>0$ 开口向上, $a<0$ 开口向下, 根据对称轴公式 $x=-\\frac{b}{2 a}$ 可以找到对称轴, 进而找到顶点, 再根据图象了解该拋物线的其他特征.\n\n【详解】 $\\because$ 二次函数的表达式为 $y=-x^{2}+3$\n\n$\\therefore a<0$, 开口向下, 抛物线有最高点, 在对称轴的左侧, $y$ 随 $x$ 的增大而增大, 在对称轴的右侧, $y$ 随 $x$ 的增大而减小\n\n$\\because b=0, \\therefore$ 对称轴 $x=0$\n\n将 $x=0$ 代入解析式得 $y=3$\n\n$\\therefore$ 顶点坐标为 $(0,3)$\n\n故选: D\n\n【点睛】本题考查了二次函数系数与图象的关系, 掌握通过二次函数的系数判断二次函数图象的开口方向、对称轴、顶点坐标是解决本题的关键.\n\n"} {"id": "17", "image": ["10118.jpg", "10119.jpg", "10120.jpg", "10121.jpg"], "answer": "B", "solution": "null", "level": "九年级", "question": "二次函数 $y=(x-1)^{2}+1$ 的大致图象是 ( )\n", "options": "A.\n\n\nB.\n\n\nC.\n\n\nD.\n\n\n\n", "subject": "解析几何", "analysis": "根据二次函数解析式写出顶点坐标和对称轴, 即可判断.\n\n【详解】解: $\\because$ 二次函数 $y=(x-1)^{2}+1$,\n\n则可得二次函数的顶点是: $(1,1)$ ,对称轴是 $x=1$ ,\n\n又 $\\because a=1>0$\n\n$\\therefore$ 图像开口向上,\n\n所以选项 B 图像符合.\n\n故选 B.\n\n【点睛】本题考查了二次函数的图像与性质, 二次函数顶点式, 通过顶点式写出顶点坐标和对称轴是解题关键.\n\n"} {"id": "18", "image": [], "answer": "A", "solution": "null", "level": "九年级", "question": "若 $A\\left(-2, y_{1}\\right), B\\left(1, y_{2}\\right), C\\left(2, y_{3}\\right)$ 是抛物线 $y=x^{2}+2 x+3$ 上的三点, 则 $y_{1}, y_{2}, y_{3}$ 的大小关系是()\n ", "options": "A. $y_{1}-1$ 时, $y$ 随 $x$ 的增大而增大,\n\n$\\therefore x=0$ 时的函数值与 $x=-2$ 时的函数值相等, 即为 $y_{1}$,\n$\\because\\left(0, y_{1}\\right), B\\left(1, y_{2}\\right), C\\left(2, y_{3}\\right)$ 是拋物线 $y=x^{2}+2 x+3$ 上的三点,且 $2>1>0>-1$,\n\n$\\therefore y_{1}\n\n (第 7 题)\n\n ", "options": "A. $-1,3$\n B. $-2,3$\n C. 1,3\n D. 3,4\n\n", "subject": "解析几何", "analysis": "根据二次函数的性质,从函数的图象可知函数的对称轴及与 $\\mathrm{x}$ 轴一个交点坐标,即可求解.\n\n【详解】解:由图象可知:二次函数 $y=a x^{2}+b x+c(a \\neq 0)$ 的对称轴是直线 $\\mathrm{x}=1$,\n\n函数与 $\\mathrm{x}$ 轴的一个交点为 $(3,0)$,\n\n则该函数与 $\\mathrm{x}$ 轴的另一个交点横坐标为: 1-(3-1)=-1,\n\n$\\therefore$ 交点为 $(3,0)$ 和 $(-1,0)$,\n\n$\\therefore$ 方程 $a x^{2}+b x+c=0$ 的解应为: $\\mathrm{x}=-1$ 或 $\\mathrm{x}=3$,\n\n故选: A.\n\n【点睛】本题考查抛物线与 $\\mathrm{x}$ 轴的交点坐标,解答本题的关键是明确题意,利用二次函数的性质和数形结合的思想解答.\n\n"} {"id": "22", "image": ["10123.jpg"], "answer": "B", "solution": "null", "level": "九年级", "question": "二次函数 $y=a x^{2}+b x+c \\quad(a \\neq 0)$ 的图象如图所示,下列说法错误的是()\n\n \n\n (第 8 题)\n\n ", "options": "A. $a b c>0$\n B. $4 a c-b^{2}>0$\n C. $3 a+c=0$\n D. $a x^{2}+b x+c=$\n\n", "subject": "解析几何", "analysis": "根据函数图象确定 $\\mathrm{a} 、 \\mathrm{~b} 、 \\mathrm{c}$ 的符号判断 $\\mathrm{A}$; 根据抛物线与 $\\mathrm{x}$ 轴的交点判断 $\\mathrm{B}$; 利用抛物线的对称轴得到 $\\mathrm{b}=2 \\mathrm{a}$, 再根据拋物线的对称性求得 $\\mathrm{c}=-3 \\mathrm{a}$ 即可判断 $\\mathrm{C}$; 利用拋物线的顶点坐标判断抛物线与直线 $\\mathrm{y}=\\mathrm{n}+1$ 即可判断 $\\mathrm{D}$.\n\n【详解】由函数图象知 $a<0, \\quad c>0$, 由对称轴在 $\\mathrm{y}$ 轴左侧, $\\mathrm{a}$ 与 $\\mathrm{b}$ 同号, 得 $b<0$, 故 $\\mathrm{abc}>0$,选项 A 正确;\n\n二次函数与 $x$ 轴有两个交点, 故 $\\Delta=b^{2}-4 a c>0$, 则选项 $\\mathrm{B}$ 错误,\n\n由图可知二次函数对称轴为 $x=-1$, 得 $b=2 a$,\n\n根据对称性可得函数与 $x$ 轴的另一交点坐标为 $(1,0)$,\n\n代入解析式 $y=a x^{2}+b x+c$ 可得 $c=-3 a$,\n\n$\\therefore 3 a+c=0$, 选项 C 正确;\n\n$\\because$ 二次函数 $\\mathrm{y}=\\mathrm{ax}{ }^{2}+\\mathrm{bx}+\\mathrm{c}$ 的顶点坐标为 $(-1, \\mathrm{n})$,\n\n$\\therefore$ 抛物线与直线 $\\mathrm{y}=\\mathrm{n}+1$ 没有交点, 故 D 正确;\n\n故选: B.\n\n【点睛】此题考查抛物线的性质, 抛物线的图象与点坐标, 抛物线的对称性, 正确理解和掌握 $\\mathrm{y}=\\mathrm{ax}{ }^{2}+\\mathrm{bx}+\\mathrm{c}$ 型抛物线的性质及特征是解题的关键.\n\n"} {"id": "23", "image": [], "answer": "C", "solution": "null", "level": "九年级", "question": "已知点 $\\left.A\\left(m, y_{1}\\right) 、 B\\left(m+2, y_{2}\\right)\\right) 、 C\\left(x_{0}, y_{0}\\right)$ 在二次函图像上. 且 $C$ 为抛物线的顶点. 若 $y_{0} \\geq y_{2}>y_{1}$, 则 $m$ 的取们\n", "options": "A. $m<-3$\nB. $m>-3$\nC. $m<-2$\n\n", "subject": "解析几何", "analysis": "先根据 $y_{0} \\geq y_{2}>y_{1}$ 得到拋物线开口向下,离对称轴越远函数值越小,再求出拋物线对称轴为直线 $x=-1$, 进而根据二次函数的性质列出对应的不等式进行求解即可.\n\n【详解】解: $\\because$ 点 $C$ 为抛物线的顶点, 点 $\\left.A\\left(m, y_{1}\\right) 、 B\\left(m+2, y_{2}\\right)\\right) 、 C\\left(x_{0}, y_{0}\\right)$ 在二次函数 $y=a x^{2}+2 a x+c(a \\neq 0)$ 的图像上且 $y_{0} \\geq y_{2}>y_{1}$,\n\n$\\therefore$ 抛物线开口向下,离对称轴越远函数值越小,\n\n$\\because$ 拋物线对称轴为直线 $x=\\frac{2 a}{-2 a}=-1$,\n\n$\\therefore m+2 \\leq-1$ 或 $m<-1 \\leq m+2$ 且 $m+2-(-1)<-1-m$,\n\n$\\therefore m \\leq-3$ 或 $-3 \\leq m<-2$,\n\n$\\therefore m<-2$,\n\n故选 C.\n\n【点睛】本题主要考查了二次函数图象的性质, 正确理解题意得到抛物线开口向上, 离对称轴越远函数值越小是解题的关键.\n\n"} {"id": "35", "image": [], "answer": "B", "solution": "null", "level": "九年级", "question": "抛物线 $y=-x^{2}$ 开口方向是 ( )\n", "options": "A. 向上\nB. 向下\nC. 向左\nD. 向右\n", "subject": "解析几何", "analysis": "根据拋物线的解析式和二次函数的性质, 可以解答本题.\n\n【详解】 $\\because a=-1<0$\n\n$\\therefore$ 抛物线的开口向下.\n\n故选: B.\n\n【点睛】本题考查了二次函数的性质, 解答本题的关键是明确题意, 利用二次函数的性质解答.\n\n"} {"id": "3583", "image": [], "answer": "C", "solution": "null", "level": "九年级", "question": "下列说法正确的是:", "options": "A. 与圆有公共点的直线是圆的切线\nB. 过三点一定可以作一个圆\nC. 垂直于弦的直径一定平分这条弦\nD. 三角形的外心到三边的距离相等", "subject": "立体几何学", "analysis": "$A$. 应为与圆只有一个交点的直线是圆的切线, 故 $A$ 选项错误;\n\n$B$ B. 过不在同一直线上的三点才能作一个圆, 故 $B$ 选项错误;\n\nC.垂直于弦的直径一定平分这条弦, 故 $C$ 选项正确;\n\n$D$. 到三角形三边距离相等的是三角形的内心, 故 $D$ 选项错误,\n\n故选 $C$."} {"id": "3588", "image": ["4894.jpg", "4895.jpg", "4896.jpg", "4897.jpg"], "answer": "B", "solution": "null", "level": "九年级", "question": "是三棱雉的表面展开图的是 ( )", "options": "A.\n\n\nB.\n\n\nC.\n\n\nD.\n\n", "subject": "立体几何学", "analysis": "$A$.不组成三棱雉, 故 $A$ 不符合题意.\n\nB. 能组成三棱雉, 故 $B$ 符合题意.\n\n$C$.组成的是四棱雉, 故 $C$ 不符合题意.\n\n$D$.组成的是三棱柱, 故 $D$ 不符合题意.\n\n故选 $B$."} {"id": "3593", "image": ["4908.jpg", "4909.jpg", "4910.jpg", "4911.jpg", "4912.jpg"], "answer": "D", "solution": "null", "level": "九年级", "question": "已知 $O$ 为圆锥的顶点, $M$ 为圆锥底面圆上一点, 点 $P$ 在 $O M$ 上. 一只蜗牛从 $P$ 点出发, 绕圆锥侧面爬行, 回到 $P$ 点时, 所爬过的最短路线的痕迹如图所示. 若沿 $O M$ 将圆锥侧面剪开并展平, 所得侧面展开图是 ( )", "options": "A.\n\n\nB.\n\n\n\n\n\nC.\n\n\n\nD.\n\n", "subject": "立体几何学", "analysis": "蜗牛绕圆雉侧面爬行的最短路线应该是一条线段, 因此选项 $A$ 和 $B$ 错误, 又因为蜗牛从 $P$ 点出\n发, 绕圆锥侧面爬行后, 又回到起始点 $P$ 处, 那么如果将选项 $C 、 D$ 的圆雉侧面展开图还原成圆锥后,位于母线 $O M$ 上的点 $P$ 应该能够与母线 $O M^{\\prime}$ 上的点 $\\left(P^{\\prime}\\right)$ 重合, 而选项 $C$ 还原后两个点不能够重合.\n\n故选 $D$."} {"id": "3606", "image": [], "answer": "D", "solution": "null", "level": "九年级", "question": "已知 $\\frac{1}{2}<\\cos \\alpha<\\sin 80^{\\circ}$, 则锐角 $\\alpha$ 的取值范围是 $(\\quad)$", "options": "A. $30^{\\circ}<\\alpha<80^{\\circ}$\nB. $10^{\\circ}<\\alpha<80^{\\circ}$\nC. $60^{\\circ}<\\alpha<80^{\\circ}$\nD. $10^{\\circ}<\\alpha<60^{\\circ}$", "subject": "立体几何学", "analysis": "$\\because \\cos 60^{\\circ}=\\frac{1}{2}, \\sin 80^{\\circ}=\\cos 10^{\\circ}$,\n\n余弦函数随角增大而减小,\n\n$\\therefore 10^{\\circ}<\\alpha<60^{\\circ}$.\n\n故选 $D$."} {"id": "3630", "image": [], "answer": "B", "solution": "null", "level": "九年级", "question": "如图, 一个菱形的边长与它的一边相外切的圆的周长相等, 当这个圆按箭头方向从某一位置沿此菱形的四边做无滑动旋转, 直至回到原出发位置时, 这个圆共转了( $)$", "options": "A. 6 圈\nB. 5 圈\nC. 4.5 圈\nD. 4 圈", "subject": "立体几何学", "analysis": ": 菱形的边长与它的一边相外切的圆的周长相等\n\n$\\therefore$ 圆在菱形的边上转了 4 圈\n\n$\\because$ 圆在菱形的四个顶点处共转了 $360^{\\circ}$,\n\n$\\therefore$ 圆在菱形的四个顶点处共转 1 圈\n\n$\\therefore$ 回到原出发位置时, 这个圆共转了 5 圈.\n\n故选: $B$."} {"id": "3659", "image": ["5021.jpg"], "answer": "C", "solution": "null", "level": "九年级", "question": "有若干个完全相同的小正方体堆成一个如图所示几何体, 若现在你手头还有一些相同的小正方体, 如果保持俯视图和左视图不变, 最多可以再添加小正方体的个数为 $(\\quad)$\n\n", "options": "A. 2\nB. 3\nC. 4\nD. 5", "subject": "立体几何学", "analysis": "若要保持俯视图和左视图不变, 可以往第 2 排右侧正方体上添加 1 个, 往第 3 排中间正方体上添加 2 个、右侧两个正方体上再添加 1 个,\n\n即一共添加 4 个小正方体,\n故选 $C$."} {"id": "3660", "image": ["5022.jpg", "5023.jpg", "5024.jpg"], "answer": "B", "solution": "null", "level": "九年级", "question": "某几何体由若干个大小相同的小正方体搭成, 其主视图与左视图如图所示, 则搭成这个几何体的小正方体最少有 ( $)$\n\n\n\n主视图\n\n", "options": "A. 4 个\nB. 5 个\nC. 6 个\nD. 7 个", "subject": "立体几何学", "analysis": "由主视图和左视图可确定所需正方体个数最少时的俯视图(图中的数字表示在该位置上小正方体的个数)为:\n\n\n\n故组成这个几何体的小正方体最少有 5 个.\n\n故选 $B$."} {"id": "3662", "image": ["5026.jpg", "5027.jpg", "5028.jpg", "5029.jpg"], "answer": "C", "solution": "null", "level": "九年级", "question": "如图是一个正方体的表面展开图, 则这个正方体是( )", "options": "A.\n\n\nB.\n\n\nD.\n\n\n\n", "subject": "立体几何学", "analysis": "通过具体折叠结合图形的特征, 判断图中小正方形内部的线段折叠后只能互相垂直, 且无公共点,所以折叠成正方体后的立体图形是 $C$.\n\n故选: $C$."} {"id": "3663", "image": [], "answer": "C", "solution": "null", "level": "九年级", "question": "将一个棱长为 $m\\left(m>2\\right.$ 且 $m$ 为正整数)的正方体木块的表面染上红色, 然后切成 $m^{3}$ 个棱长为 1 的小正方体, 发现只有一个表面染有红色的小正方体的数量是恰有两个表面染有红色的小正方体的数量的 12 倍, 则 $m$ 等于 $(\\quad)$", "options": "A. 16\nB. 18\nC. 26\nD. 32", "subject": "立体几何学", "analysis": "将一个棱长为 $m(m>2$ 且 $m$ 为正整数 $)$ 的正方体木块的表面染上红色, 然后切成 $m^{3}$ 个棱长为 1 的小正方体, 则\n\n只有一个表面染有红色的小正方体的数量为 $6(m-2)^{2}$,\n\n恰有两个表面染有红色的小正方体的数量 $12(m-2)$,\n\n$\\because$ 只有一个表面染有红色的小正方体的数量是恰有两个表面染有红色的小正方体的数量的 12 倍,\n\n$\\therefore 6(m-2)^{2}=12 \\times 12(m-2)$,\n\n解得 $m_{1}=26, m_{2}=2$ (舍去),\n故选: $C$."} {"id": "3667", "image": ["5037.jpg", "5038.jpg", "5039.jpg", "5040.jpg", "5041.jpg"], "answer": "C", "solution": "null", "level": "九年级", "question": "已知 $O$ 为圆锥的顶点, $M$ 为圆雉底面上一点, 点 $P$ 在 $O M$ 上.一只蜗牛从 $P$ 点出发, 绕圆锥侧面爬行, 回到 $P$ 点时所爬过的最短路线的痕迹如图所示. 若沿 $O M$ 将圆锥侧面剪开并展开, 所得侧面展开图是 $(\\quad)$.\n\n", "options": "A.\n\n\nB.\n\n\nC.\n\n\nD.\n\n", "subject": "立体几何学", "analysis": "蜗牛绕圆雉侧面爬行的最短路线应该是一条线段, 因此选项 $A$ 和 $B$ 错误, 又因为蜗牛从 $P$ 点出发, 绕圆锥侧面爬行后, 又回到起始点 $P$ 处, 那么如果将选项 $C 、 D$ 的圆雉侧面展开图还原成圆锥后,位于母线 $O M$ 上的点 $P$ 应该能够与母线 $O M^{\\prime}$ 上的点 $\\left(P^{\\prime}\\right)$ 重合, 而选项 $D$ 还原后两个点不能够重合.故选 $C$."} {"id": "3682", "image": [], "answer": "B", "solution": "null", "level": "九年级", "question": "坚直放置的正四棱柱(即底面是水平放置的), 用水平面去截得的截面的形状是( )", "options": "A. 长方形\nB. 正方形\nC. 梯形\nD. 截面形状不定", "subject": "立体几何学", "analysis": "坚直放置的正四棱柱的上下底面是正方形,\n\n当水平面去截坚直放置的正四棱柱时, 截得的截面是正方形.\n\n故选 $B$."} {"id": "3683", "image": ["5063.jpg", "5064.jpg", "5065.jpg", "5066.jpg", "5067.jpg"], "answer": "C", "solution": "null", "level": "九年级", "question": "用两块完全相同的长方体搭成如图所示几何体, 这个几何体的主视图是 ( )", "options": "A.\n\n\nB.\n\n\nC.\n\n\nD.\n\n\n\n\n正而", "subject": "立体几何学", "analysis": "从物体正面看, 左边 1 列, 右边 1 列上下各一个正方形, 且左右正方形中间是虚线.\n\n故选 $C$."} {"id": "3684", "image": ["5068.jpg", "5069.jpg", "5070.jpg", "5071.jpg", "5072.jpg", "5073.jpg", "5074.jpg", "5075.jpg", "5076.jpg", "5077.jpg", "5078.jpg"], "answer": "B", "solution": "null", "level": "九年级", "question": "如图, 图 1 是由 5 个完全相同的正方体堆成的几何体, 现将标有 $E$ 的正方体平移至如图 2 所示的位置, 下列说法中正确的是 $(\\quad)$", "options": "A. 左、右两个几何体的主视图相同\n\nB. 左、右两个几何体的左视图相同\n\nC. 左、右两个几何体的俯视图不相同\n\n\n\n正面\n\n图 1\n\n\n\n正面\n\n图 2\n\nD. 左、右两个几何体的三视图不相同\n\n\n\n主视图\n\n\n\n左视图\n\n\n\n俯枧图", "subject": "立体几何学", "analysis": "$A$ 、左、右两个几何体的主视图为:\n\n\n\n主视图1\n\n\n\n主视图2\n\n不相同,故此选项错误;\n\n$B$ 、左、右两个几何体的左视图为:\n\n\n\n左视图1\n\n\n\n左视图2\n\n相同, 故此选项正确;\n\nC、左、右两个几何体的俯视图为:\n\n\n\n俯视图1\n\n\n\n俯视图2\n\n相同,故此选项错误;\n\n$D$ 、由以上可得,此选项错误;\n\n故选: $B$."} {"id": "3685", "image": [], "answer": "A", "solution": "null", "level": "九年级", "question": "北附小卖部货摆放着某品牌方便面, 它们的三视图如图, 则货架上的方便面至少有 ( )", "options": "A. 7 盒\nB. 8 盒\nC. 9 盒\nD. 10 盒", "subject": "立体几何学", "analysis": "易得第一层有 4 碗,第二层最少有 2 碗,第三层最少有 1 碗,所以至少共有 7 盒.\n\n故选: $A$."} {"id": "3687", "image": ["5080.jpg"], "answer": "A", "solution": "null", "level": "九年级", "question": "如图, 一个空间立体几何体的主视图和左视图都是边长为 2 的正三角形, 俯视图是一个圆, 那么这个几何体的表面积是 $(\\quad)$\n", "options": "A. $3 \\pi$\nB. $\\pi+2 \\sqrt{3}$\nC. $2 \\pi$\nD.\n\n$5 \\pi$", "subject": "立体几何学", "analysis": "综合主视图, 俯视图, 左视图可以看出这个几何体应该是圆锥, 且底面圆的半径为 $2 \\div 2=1$,母线长为 2,\n\n因此侧面面积为 $1 \\times \\pi \\times 2=2 \\pi$, 底面积为 $\\pi \\times(1)^{2}=\\pi$.\n\n表面积为 $2 \\pi+\\pi=3 \\pi$.\n\n故选 $A$."} {"id": "3830", "image": ["5320.jpg", "5321.jpg", "5322.jpg"], "answer": "D", "solution": "null", "level": "九年级", "question": "一物体及其正视图如下图所示, 则它的左视图与俯视图分别是右侧图形中的().\n\n", "options": "A. (1)(2)\n\n\n\n正视目\n\n\nC. (1) (4)\n\nD. (3)(2)", "subject": "立体几何学", "analysis": "先细心观察原立体图形和俯视图中两个长方体的位置关系, 从几何体的左边看去是 2 个长方体叠在一起, 所以左视图是(3), 从上面看下来是 3 个长方体并排, 所以俯视图是(2).\n\n故选 D."} {"id": "3831", "image": ["5323.jpg", "5324.jpg", "5325.jpg", "5326.jpg", "5327.jpg"], "answer": "B", "solution": "null", "level": "九年级", "question": "如图的几何体, 左视图是 $\\quad(\\quad)$\n\n", "options": "A.\n\n\nB.\n\n\nC.\n\n\nD.\n\n", "subject": "立体几何学", "analysis": "从左边看去, 左边是两个正方形, 右边是一个正方形. 故选 B."} {"id": "3841", "image": ["5345.jpg", "5346.jpg", "5347.jpg", "5348.jpg", "5349.jpg"], "answer": "A", "solution": "null", "level": "九年级", "question": "如图, 从左面观察这个立体图形, 能得到的平面图形是 ( )\n\n", "options": "A.\n\n\n\n\nC.\n\n\nD.\n\n", "subject": "立体几何学", "analysis": "从左面看下面一个正方形, 上面一个正方形, 故选 A."} {"id": "3847", "image": ["5366.jpg", "5367.jpg", "5368.jpg", "5369.jpg", "5370.jpg"], "answer": "B", "solution": "null", "level": "九年级", "question": "明明用纸 (如图) 折成了一个正方体的盒子, 里面装了一瓶墨水, 与其它空盒子混放在一起, 只凭观察, 选出墨水在哪个盒子中 ( $\\quad$ )\n\n", "options": "A.\n\n\nB.\n\n\nC.\n\n\nD.\n\n", "subject": "立体几何学", "analysis": "根据展开图中各种符号的特征和位置, 可得墨水在 B 盒子里面.故选 B."} {"id": "3589", "image": ["4898.jpg", "4899.jpg", "4900.jpg", "4901.jpg", "4902.jpg", "4903.jpg"], "answer": "B", "solution": "null", "level": "九年级", "question": "如图, 在菱形 $A B C D$ 中, $\\angle B=60^{\\circ}, A B=2$, 动点 $P$ 从点 $B$ 出发, 以每秒 1 个单位长度的速度沿折线 $B A \\rightarrow A C$ 运动到点 $C$, 同时动点 $Q$ 从点 $A$ 出发,以相同速度沿折线 $A C \\rightarrow C D$ 运动到点 $D$, 当一个点停止运动时, 另一个点\n\n\n也随之停止. 设 $\\triangle A P Q$ 的面积为 $y$, 运动时间为 $x$ 秒, 则下列图象能大致反映 $y$ 与 $x$ 之间函数关系的是( $)$", "options": "A.\n\n\nB.\n\n\nC.\n\n\nD.\n\n", "subject": "组合几何学", "analysis": "(1) 当 $P 、 Q$ 分别在 $A B 、 A C$ 上运动时,\n\n$\\because A B C D$ 是菱形, $\\angle B=60^{\\circ}$, 则 $\\triangle A B C 、 \\triangle A C D$ 为边长为 2 的等边三角形,\n\n过点 $Q$ 作 $Q H \\perp A B$ 于点 $H$,\n\n\n\n$y=\\frac{1}{2} A P \\times Q H=\\frac{1}{2}(2-t) \\times t \\sin 60^{\\circ}=-\\frac{\\sqrt{3}}{4} t^{2}+\\frac{\\sqrt{3}}{2} t$,\n\n函数最大值为 $\\frac{\\sqrt{3}}{4}$, 符合条件的有 $A 、 B 、 D$;\n\n(2)当 $P 、 Q$ 分别在 $A C 、 D C$ 上运动时,\n\n同理可得: $y=\\frac{\\sqrt{3}}{4}(t-2)^{2}$,\n\n符合条件的有 $B$;\n\n故选: $B$."} {"id": "3592", "image": ["4906.jpg", "4907.jpg"], "answer": "B", "solution": "null", "level": "九年级", "question": "如图, 直角梯形 $A B C D$ 中, $\\angle A D C=9$ ॠ० $^{\\circ}, A B=6, A D=2, B C=4$, 你可以在 $C D$ 边上找到多少个点, 使其与点 $A 、 B$ 构成一个直角三角形 $(\\quad)$\n\n", "options": "A. 1 个\nB. 2 个\nC. 3 个\nD. 无数多个", "subject": "组合几何学", "analysis": "如图:\n\n\n\n设 $A B$ 的中点为 $O$, 过 $O$ 作 $O M / / A D$ 交 $C D$ 于 $M$,\n\n则 $O M$ 是直角梯形 $A B C D$ 的中位线,\n\n$\\therefore O M=\\frac{1}{2}(A D+B C)=3$;\n\n$\\because O M / / A D, A D \\perp C D$,\n\n$\\therefore O M \\perp A C$\n\n以 $O$ 为圆心, $A B$ 为直径作圆,\n\n由于 $O M=3=\\frac{1}{2} A B$, 且 $O M \\perp C D$,\n\n所以 $C D$ 与 $\\odot O$ 相切, 因此在 $C D$ 上, 点 $M$ 符合要求,\n\n过点 $A$ 作 $A E \\perp A B$ 于点 $A$,\n\n$\\because \\angle D A B$ 是针角,\n\n$\\therefore E$ 点在 $C D$ 上,\n\n故符合条件的点有两个,即点 $E$ 、点 $M$.\n\n故选 $B$."} {"id": "3613", "image": ["4931.jpg", "4932.jpg"], "answer": "B", "solution": "null", "level": "九年级", "question": "如图, 有一个边长不定的正方形 $A B C D$, 它的两个相对的顶点 $A, C$ 分别在边长为 1 的正六边形一组平行的对边上, 另外两个顶点 $B, D$ 在正六边形内部(包括边界), 则正方形边长 $a$ 的取值范围 ( )", "options": "A. $\\frac{2 \\sqrt{3}}{3} \\leq a \\leq 3-\\sqrt{3}$\nB. $\\frac{\\sqrt{6}}{2} \\leq a \\leq 3-\\sqrt{3}$\nC. $\\frac{2 \\sqrt{3}}{3} \\leq a \\leq \\sqrt{2}$\nD. $\\frac{\\sqrt{6}}{2} \\leq a \\leq \\sqrt{2}$", "subject": "组合几何学", "analysis": "(1)当正方形 $A B C D$ 的对角线 $A C$ 在正六边形一组平行的对边的中点上时,正方形边长 $a$ 的值最小, $A C$ 是正方形的对角线,\n\n$\\therefore A C=A^{\\prime} D=\\sqrt{3}$,\n\n$\\therefore a=\\frac{\\sqrt{6}}{2}$,\n\n\n\n图1\n\n(2) 当正方形 $A B C D$ 的四个顶点都在正六边形的边上时, 正方形边长 $a$ 的值最大,\n\n$A C$ 是正方形的对角线 $A C$,\n设 $A^{\\prime}\\left(t, \\frac{\\sqrt{3}}{2}\\right)$ 时, 正方形的边长最大,\n\n$\\because O B^{\\prime} \\perp O A^{\\prime}$,\n\n$\\therefore B^{\\prime}\\left(-\\frac{\\sqrt{3}}{2}, t\\right)$,\n\n设直线 $M N$ 的解析式为 $y=k x+b, M(-1,0), N\\left(-\\frac{1}{2},-\\frac{\\sqrt{3}}{2}\\right)$,\n\n$\\therefore\\left\\{\\begin{array}{l}-k+b=0 \\\\ -\\frac{1}{2} k+b=-\\frac{\\sqrt{3}}{2}\\end{array}\\right.$\n\n$\\therefore\\left\\{\\begin{array}{l}k=-\\sqrt{3} \\\\ b=-\\sqrt{3}\\end{array}\\right.$.\n\n$\\therefore$ 直线 $M N$ 的解析式为 $y=-\\sqrt{3} x-\\sqrt{3}$,\n\n将 $B^{\\prime}\\left(-\\frac{\\sqrt{3}}{2}, t\\right)$ 代入得 $t=\\frac{3}{2}-\\sqrt{3}$,\n\n此时, $A^{\\prime} B^{\\prime}$ 取最大值,\n\n$\\therefore a=\\sqrt{\\left(\\frac{3}{2}-\\sqrt{3}+\\frac{\\sqrt{3}}{2}\\right)^{2}+\\left(\\frac{\\sqrt{3}}{2}-\\frac{3}{2}+\\sqrt{3}\\right)^{2}}=3-\\sqrt{3}$,\n\n\n\n$\\therefore$ 正方形边长 $a$ 的取值范围是: $\\frac{\\sqrt{6}}{2} \\leq a \\leq 3-\\sqrt{3}$,\n\n故选 $B$."} {"id": "3620", "image": ["4943.jpg", "4944.jpg"], "answer": "A", "solution": "null", "level": "九年级", "question": "如图, 将 $R t \\triangle A B C$ 平移到 $\\triangle A^{\\prime} B^{\\prime} C^{\\prime}$ 的位置, 其中 $\\angle C=90^{\\circ}$, 使得点 $C^{\\prime}$与 $\\triangle A B C$ 的内心重合, 已知 $A C=4, B C=3$, 则阴影部分的周长为 $(\\quad)$", "options": "A. 5\nB. 6\nC. 7\nD. 8\n\n", "subject": "组合几何学", "analysis": "连接 $A C^{\\prime} 、 B C^{\\prime}, A^{\\prime} C^{\\prime} 、 B^{\\prime} C^{\\prime}$ 交 $A B$ 于 $D 、 E$, 如图,\n\n\n\n在Rt $\\triangle A B C$ 中, $\\angle C=90^{\\circ}, A C=4, B C=3$,\n\n$\\therefore A B=\\sqrt{3^{2}+4^{2}}=5$,\n\n$\\because$ 将 $R t \\triangle A B C$ 平移到 $\\triangle A^{\\prime} B^{\\prime} C^{\\prime}$ 的位置,\n\n$\\therefore A C / / A^{\\prime} C^{\\prime}, B C / / B^{\\prime} C^{\\prime}$,\n\n$\\therefore \\angle C A C^{\\prime}=\\angle D C^{\\prime} A, \\angle C B C^{\\prime}=\\angle B C^{\\prime} E$,\n\n$\\because$ 点 $C^{\\prime}$ 为 $\\triangle A B C$ 的内心,\n\n$\\therefore \\angle C A C^{\\prime}=\\angle D A C^{\\prime}, \\angle C B C^{\\prime}=\\angle E B C^{\\prime}$,\n\n$\\therefore \\angle D C^{\\prime} A=\\angle D A C^{\\prime}, \\angle B C^{\\prime} E=\\angle E B C^{\\prime}$,\n\n$\\therefore D C^{\\prime}=D A, E B=E C^{\\prime}$,\n\n$\\therefore$ 阴影部分的周长 $=D C^{\\prime}+D E+E C^{\\prime}=D A+D E+E B=A B=5$.\n\n故选:A."} {"id": "3641", "image": ["4984.jpg", "4985.jpg"], "answer": "D", "solution": "null", "level": "九年级", "question": "如图, $A B$ 为半圆 $O$ 的直径, $A D 、 B C$ 分别切 $\\odot O$ 于 $A, B$ 两点, $C D$ 切 $\\odot O$于点 $E, A D$ 与 $C D$ 相交于 $D, B C$ 与 $C D$ 相交于 $C$, 连接 $O D 、 O C, A D=3$, $B C=\\frac{16}{3}$, 则四边形 $A B C D$ 的周长为 $(\\quad)$\n\n", "options": "A. $\\frac{25}{3}$\nB. $\\frac{50}{3}$\nC. $\\frac{62}{3}$\nD. $\\frac{74}{3}$", "subject": "组合几何学", "analysis": "过 $D$ 作 $D F \\perp B C$ 于 $F$ ,则 $\\angle D F B=90^{\\circ}$,\n\n\n\n$\\because A D 、 B C$ 分别切 $\\odot O$ 于 $A 、 B$ 两点, $C D$ 切 $\\odot O$ 于点 $E$,\n\n$\\therefore A D=D E, B C=C E, \\angle D A B=\\angle C B A=90^{\\circ}$,\n\n$\\therefore$ 四边形 $A D F B$ 是矩形,\n\n$\\therefore A D=B F, A B=D F$,\n\n$\\because A D=3, B C=\\frac{16}{3} \\frac{16}{3}, A D=D E, B C=C E$,\n\n$\\therefore D E=3, C E=\\frac{16}{3} \\frac{16}{3}$,\n\n$\\therefore D C=3+\\frac{16}{3} \\frac{16}{3}=\\frac{25}{3} \\frac{25}{3}, C F=B C-A D=\\frac{16}{3} \\frac{16}{3}-3=\\frac{7}{3} \\frac{7}{3}$,\n\n在 $R t \\triangle C F D$ 中, 由勾股定理得: $D F=\\sqrt{D C^{2}-C F^{2}} \\sqrt{D C^{2}-C F^{2}}=\\sqrt{\\left(\\frac{25}{3}\\right)^{2}-\\left(\\frac{7}{3}\\right)^{2}}=8$,\n\n即 $A B=D F=8 ,$\n\n即四边形 $A B C D$ 的周长是 $A D+D C+B C+A B=3+\\frac{25}{3}+\\frac{16}{3}+8=\\frac{74}{3}$,\n\n故选 $D$."} {"id": "3874", "image": ["5446.jpg", "5447.jpg"], "answer": "C", "solution": "null", "level": "九年级", "question": "如图, $O$ 是 $\\triangle A B C$ 的内心, 过点 $O$ 作 $E F / / A B$, 与 $A C 、 B C$ 分别交 $E 、 F$, 则 ( )\n\n", "options": "A. $\\mathrm{EF}>\\mathrm{AE}+\\mathrm{BF}$\nB. $\\mathrm{EF}<\\mathrm{AE}+\\mathrm{BF}$\nC. $\\mathrm{EF}=\\mathrm{AE}+\\mathrm{BF}$\nD. $\\mathrm{EF} \\leq \\mathrm{AE}+\\mathrm{BF}$", "subject": "组合几何学", "analysis": "连接 $\\mathrm{OA} 、 \\mathrm{OB}$,\n\n\n\n$\\because \\mathrm{O}$ 是 $\\triangle \\mathrm{ABC}$ 的内心, $\\therefore \\mathrm{OA} 、 \\mathrm{OB}$ 分别是 $\\angle \\mathrm{CAB}$ 及 $\\angle \\mathrm{ABC}$ 的平分线,\n\n$\\therefore \\angle \\mathrm{EAO}=\\angle \\mathrm{OAB}, \\angle \\mathrm{ABO}=\\angle \\mathrm{FBO}, \\because \\mathrm{EF} / / \\mathrm{AB}, \\therefore \\angle \\mathrm{AOE}=\\angle \\mathrm{OAB}$,\n\n$\\angle \\mathrm{BOF}=\\angle \\mathrm{ABO}, \\therefore \\angle \\mathrm{EAO}=\\angle \\mathrm{AOE}$,\n\n$\\angle \\mathrm{FBO}=\\angle \\mathrm{BOF}, \\therefore \\mathrm{AE}=\\mathrm{OE}, \\mathrm{OF}=\\mathrm{BF}, \\therefore \\mathrm{EF}=\\mathrm{AE}+\\mathrm{BF}$, 故选 $\\mathrm{C}$."} {"id": "3897", "image": ["5485.jpg", "5486.jpg"], "answer": "C", "solution": "null", "level": "九年级", "question": "如图, $\\odot O$ 中, $\\mathrm{PC}$ 切 $\\odot \\mathrm{O}$ 于点 $\\mathrm{C}$, 连 $\\mathrm{PO}$ 交于 $\\odot \\mathrm{O}$ 点 $\\mathrm{A} 、 \\mathrm{~B}$, 点 $\\mathrm{F}$ 是 $\\odot \\mathrm{O}$ 上一点, 连 $\\mathrm{PF}, \\mathrm{CD} \\perp \\mathrm{AB}$于点 $\\mathrm{D}, \\mathrm{AD}=2, \\mathrm{CD}=4$, 则 $\\mathrm{PF}: \\mathrm{DF}$ 的值是 ( )\n\n", "options": "A. 2\nB. $\\sqrt{5}$\nC. $5: 3$\nD. $4: 3$", "subject": "组合几何学", "analysis": "连接 $A C 、 O C 、 O F 、 B C$. 如图所示:\n\n\n\n$\\because \\mathrm{AB}$ 是直径, $\\therefore \\angle \\mathrm{ACB}=90^{\\circ}$,\n\n$\\because \\mathrm{CD} \\perp \\mathrm{AB}, \\quad \\therefore \\angle \\mathrm{ADC}=\\angle \\mathrm{BDC}=90^{\\circ}$,\n\n$\\therefore \\angle \\mathrm{ACD}+\\angle \\mathrm{CAD}=90^{\\circ}, \\angle \\mathrm{ACD}+\\angle \\mathrm{BCD}=90^{\\circ}, \\therefore \\angle \\mathrm{CAD}=\\angle \\mathrm{BCD}$,\n\n$\\therefore \\triangle \\mathrm{ADC} \\sim \\triangle \\mathrm{CDB}, \\quad \\therefore \\frac{A D}{C D}=\\frac{C D}{D B}$,\n\n$\\therefore \\frac{2}{4}=\\frac{4}{D B}, \\quad \\therefore \\mathrm{DB}=8, \\mathrm{OA}=\\mathrm{OB}=5, \\mathrm{OD}=3$\n\n$\\because \\mathrm{PC}$ 是切线, $\\therefore \\mathrm{OC} \\perp \\mathrm{PC}$,\n\n$\\because \\angle \\mathrm{DOC}=\\angle \\mathrm{POC}, \\angle \\mathrm{ODC}=\\angle \\mathrm{OCP}$,\n\n$\\therefore \\triangle \\mathrm{ODC} \\sim \\triangle \\mathrm{OCP}, \\quad \\therefore \\frac{O D}{O C}=\\frac{O C}{O P}$,\n\n$\\therefore \\mathrm{OC}^{2}=\\mathrm{OD} \\cdot \\mathrm{OP}$,\n$\\therefore \\mathrm{OF}^{2}=\\mathrm{OD} \\cdot \\mathrm{OP}, \\quad \\therefore \\frac{O F}{O D}=\\frac{O P}{O F}$,\n\n$\\because \\angle \\mathrm{DOF}=\\angle \\mathrm{POF}, \\therefore \\triangle \\mathrm{DOF} \\sim \\triangle \\mathrm{FOP}, \\therefore \\frac{P F}{D F}=\\frac{O F}{O D}=\\frac{5}{3}$, 故选 C."} {"id": "3907", "image": ["5504.jpg", "5505.jpg"], "answer": "B", "solution": "null", "level": "九年级", "question": "如图, 已知 $A B$ 是 $\\odot O$ 直径, $B C$ 是弦, $\\angle A B C=40^{\\circ}$, 过圆心 $O$ 作 $O D \\perp B C$ 交弧 $B C$ 于点 $D$, 连接 $D C$, 则 $\\angle D C B$ 为 $(\\quad)$\n\n", "options": "A. $20^{\\circ}$\nB. $25^{\\circ}$\nC. $30^{\\circ}$\nD. $35^{\\circ}$", "subject": "组合几何学", "analysis": "$\\because O D \\perp B C, \\angle A B C=40^{\\circ}, \\therefore$ 在 Rt $\\triangle O B E$ 中, $\\angle B O E=50^{\\circ}$ (直角三角形的两个锐角互余). 又 $\\because \\angle D C B=\\frac{1}{2} \\angle D O B$ (同弧所对的圆周角是所对的圆心角的一半), $\\therefore \\angle D C B=25^{\\circ}$. 故选 B.\n\n"} {"id": "3913", "image": ["5520.jpg", "5521.jpg", "5522.jpg"], "answer": "C", "solution": "null", "level": "九年级", "question": "已知, 如图, 在 $\\triangle A B C$ 中, $A B=A C$, 以 $A B$ 为直径作 $\\odot O$ 分别交 $A C, B C$ 于 $D, E$ 两点,过 $B$ 点的切线交 $O E$ 的延长线于点 $F$. 下列结论:\n\n(1) $O E / / A C$; (2)两段劣弧 $D E=B E$; (3) $F D$ 与 $\\odot O$ 相切; (4) $S_{\\triangle B D E}: S_{\\triangle B A C}=1: 4$.\n\n其中一定正确的有( $\\quad$ )个.\n\n", "options": "A. 1\nB. 2\nC. 3\nD. 4", "subject": "组合几何学", "analysis": "$(1) \\because A B=A C, O B=O E, \\therefore \\angle A B C=\\angle A C B, \\angle O B E=\\angle O E B, \\therefore \\angle O E B=\\angle A C B, \\therefore O E / / A C$,故(1)正确; (2)连接 $O D$, 如图所示:\n\n$\\because O E / / A C, \\quad \\therefore \\angle B O E=\\angle O A D, \\angle E O D=\\angle A D O$.\n\n$\\because O A=O D, \\therefore \\angle O A D=\\angle O D A, \\therefore \\angle B O E=\\angle E O D, \\therefore D E=B E$, 故(2)正确;\n\n\n\n$\\because B F$ 与 $\\odot O$ 相切于点 $B, \\therefore \\angle O B F=90^{\\circ}, \\therefore \\angle O D F=90^{\\circ}, \\therefore D F$ 与 $\\odot O$ 相切, 故(3)正确;\n\n(4) $\\because O E / / A C, \\therefore \\triangle B O E \\sim \\triangle B A C, \\therefore \\frac{S_{\\triangle B O E}}{S_{\\triangle B A C}}=\\left(\\frac{B O}{B A}\\right)^{2}=\\left(\\frac{1}{2}\\right)^{2}=\\frac{1}{4}$, 而 $\\triangle B D E$ 的面积 $\\neq \\triangle B O E$ 的面积, 故(4)不正确; 正确的有 3 个. 故选 C.\n\n"} {"id": "3323", "image": ["4326.jpg", "4327.jpg", "4328.jpg"], "answer": "A", "solution": "null", "level": "九年级", "question": "若干个桶装方便面摆放在桌子上, 小明从三个不同方向看到的图形如图所示, 则这一堆方便面共有 $(\\quad)$", "options": "A. 5 桶\nB. 6 桶\nC. 9 桶\n\n\nD. 12 桶", "subject": "组合几何学", "analysis": "解: 根据从三个不同方向看到的图形, 可得到, 图形相应位置上放置的个数,进而得出总数量;\n\n\n图中的数, 表示该位置放的数量, 因此 $2+2+1=5$.\n\n故选: $A$.\n\n根据从三个不同方向看到的图形, 可得到, 图形相应位置上放置\n\n\n\n$(1)$\n\n的桶装方便面的个数, 进而得出答案.\n\n考查图形的实际应用, 根据从三个不同方向看到的图形, 分析得到相应位置上放置的个数是解题的关键."} {"id": "3062", "image": [], "answer": "C", "solution": "null", "level": "九年级", "question": "(本题 3 分) (2021$\\cdot$浙江$\\cdot$杭州市采荷中学九年级期中)下列关于正多边形的叙述,正确的是 $(\\quad)$", "options": "A. 正九边形既是轴对称图形又是中心对称图形B. 存在一个正多边形, 它的外角和为 $720^{\\circ}$C. 任何正多边形都有一个外接圆D. 不存在每个外角都是对应每个内角两倍的正多边形", "subject": "组合几何学", "analysis": "C【解析】【分析】根据正多边形、轴对称、中心对称的性质分析,即可判断选项A;根据多边形外角和的性质,即可判断选项B;根据正多边形与圆的性质分析,即可判断选项C;根据正多边形和外角的性质分析,即可判断选项D,从而得到答案.【详解】正九边形是轴对称图形,不是中心对称图形,故选项A不正确;任何多边形的外角和都为360^circ,故选项B不正确;任何正多边形都有一个外接圆,故选项C正确;等边三角形的每个外角都是对应每个内角两倍,故选项D不正确;故选:C.【点睛】本题考查了正多边形、轴对称、中心对称、正多边形与圆、外角的知识;解题的关键是熟练掌握正多边形、轴对称、中心对称、正多边形与圆、外角的性质,从而完成求解."} {"id": "3063", "image": ["3963.jpg"], "answer": "A", "solution": "null", "level": "九年级", "question": "(本题 3 分)(2022$\\cdot$浙江眸州 $\\cdot$二模)如图, $A B$ 是 $\\odot O$ 的直径, $C, D$ 为 $\\odot O$ 上的点, 且点 $D$ 在弧 $A C$ 上. 若 $\\angle D=120^{\\circ}$, 则 $\\angle C A B$ 的度数为", "options": "A. $30^{\\circ}$B. $40^{\\circ}$C. $50^{\\circ}$D. $60^{\\circ}$", "subject": "组合几何学", "analysis": "A【解析】【分析】利用圆内接四边形的性质求出angleB=60^circ,由圆周角定理推论得出angleACB=90^circ,再由直角三角形两锐角互余求解即可.【详解】解:becauseangleD+angleB=180^circ,angleD=120^circ,thereforeangleB=60^circ,becauseAB是直径,thereforeangleACB=90^circ,thereforeangleCAB=90^circ-angleB=30^circ,故选:A.【点睛】本题考查圆周角定理的推论,圆内接四边形的性质等知识,解题的关键是熟练掌握直径所对圆周角是直角、圆内接四边形的性质,属于中考常考题型."} {"id": "3084", "image": ["3998.jpg"], "answer": "A", "solution": "null", "level": "九年级", "question": "(本题 3 分) (2022 浙江湖州 $\\cdot$年级期末) 如图, 正五边形 $A B C D E$ 内接于 $\\odot O$, 连接 $A C$, 则 $\\angle A C D$ 的度数是 ()", "options": "A. $72^{\\circ}$B. $70^{\\circ}$C. $60^{\\circ}$D. $45^{\\circ}$", "subject": "组合几何学", "analysis": "A【分析】由正五边形的性质可知triangleABC是等腰三角形,求出angleB,angleACB的度数即可解决问题.【详解】解:在正五边形ABCDE中,angleB=angleBCD=frac15times(5-2)times180=108^circ,AB=BC,thereforeangleBCA=angleBAC=frac12left(180^circ-108^circright)=36^circ,thereforeangleACD=angleBCD-angleACB=108^circ-36^circ=72^circ.故选:A.【点睛】本题主要考查了正多边形与圆,多边形内角与外角的知识点,解答本题的关键是求出正五边形的内角,此题基础题,比较简单."} {"id": "3086", "image": ["4000.jpg"], "answer": "D", "solution": "null", "level": "九年级", "question": "(本题 3 分) (2022$\\cdot$浙江金华$\\cdot$九年级期末) 如图, 点 $A, B, C, D$ 是 $\\odot O$ 上的四个点,且 $A B=C D, O E \\perp A B, O F \\perp C D$, 则下列结论错误的是( )", "options": "A. $A B=C D$B. $O E=O F$C. $\\angle A O B=\\angle C O D$D. $A C=B C$", "subject": "组合几何学", "analysis": "D【分析】在同圆中,根据圆心角、弧和弦之间的关系即可判断.【详解】解:在odotO中,becauseAB=CDthereforeAB=CD,angleAOB=angleCOD故A、C选项正确,不符合题意;becauseAB=CD,OA=OD,OB=OCthereforetriangleOABcongtriangleODCthereforeS_triangleOAB=S_triangleODCbecauseOEperpAB,quadOFperpCD,thereforefrac12ABcdotOE=frac12CDcdotOFthereforeOE=OF故B选项正确,不符合题意.故选D【点睛】本题考查圆的对称性,理解同圆中圆心角、弧和弦之间的关系是解题的关键."} {"id": "3155", "image": ["4120.jpg"], "answer": "C", "solution": "null", "level": "九年级", "question": "(本题 3 分) (2019$\\cdot$浙江$\\cdot$九年级期末) 点 $G$ 是 $\\triangle A B C$ 的重心, 过点 $G$ 画 $M N / / B C$ 分别交 $A B, A C$ 于点 $M, N$, 则 $\\triangle A M N$ 与 $\\triangle A B C$ 的面积之比是( )", "options": "$\\frac{1}{2}$B. $\\frac{2}{3}$C. $\\frac{4}{9}$D. $\\frac{4}{25}$", "subject": "组合几何学", "analysis": "C【分析】延长AG交BC于H.由G是triangleABC的重心,推出AG:GH=2:1,推出AG:AH=2:3,由MN//BC,推出triangleAMNsimtriangleABC,fracAGAH=fracAMAB=frac23,可得fracS_triangleAMNS_triangleABC=left(fracAMABright)^2,即可解决问题.【详解】解:延长AG交BC于H.becauseG是triangleABC的重心,thereforeAG:GH=2:1,thereforeAG:AH=2:3,becauseMN//BCthereforetriangleAMNsimtriangleABC,fracAGAH=fracAMAB=frac23,thereforefracS_triangleAMN~S_triangleABC=left(fracAMABright)^2=frac49,故选C.【点睛】本题考查三角形的重心,平行线的性质,相似三角形的判定和性质等知识,解题的关键是熟练掌握基本知识,属于中考常考题型."} {"id": "3178", "image": ["4150.jpg"], "answer": "D", "solution": "null", "level": "九年级", "question": "(本题 3 分) (2022$\\cdot$浙江$\\cdot$宁波市鄞州蓝青学校九年级期末) 如图, 点 $P$ 在 $\\triangle A B C$ 的边 $A C$ 上, 要判断 $\\triangle A B P \\sim \\triangle A C B$, 添加一个条件, 不正确的是()", "options": "A. $\\angle A B P=\\angle C$B. $\\angle A P B=\\angle A B C$C. $\\frac{A P}{A B}=\\frac{A B}{A C}$D. $\\frac{A B}{A P}=\\frac{A C}{C B}$", "subject": "组合几何学", "analysis": "D【分析】根据相似三角形的判定方法, 逐项判断即可.【详解】解:在 $\\triangle A B P$ 和 $\\triangle A C B$ 中, $\\angle B A P=\\angle C A B$,$\\therefore$ 当 $\\angle A B P=\\angle C$ 时, 满足两组角对应相等, 可判断 $\\triangle A B P \\sim \\triangle A C B$, 故 $A$ 正确;当 $\\angle A P B=\\angle A B C$ 时, 满足两组角对应相等, 可判断 $\\triangle A B P \\sim \\triangle A C B$, 故 $B$ 正确;当 $\\frac{A P}{A B}=\\frac{A B}{A C}$ 时, 满足两边对应成比例且夹角相等, 可判断 $\\triangle A B P \\sim \\triangle A C B$, 故 $C$ 正确;当 $\\frac{A B}{A P}=\\frac{A C}{C B}$ 时, 其夹角不相等, 则不能判断 $\\triangle A B P \\sim \\triangle A C B$, 故 $D$ 不正确;故选: $D$.【点睛】本题主要考查相似三角形的判定, 掌握相似三角形的判定方法是解题的关键,即在两个三角形中, 满足三边对应成比例、两边对应成比例且夹角相等或两组角对应相等, 则这两个三角形相似."} {"id": "3590", "image": ["4904.jpg"], "answer": "B", "solution": "null", "level": "九年级", "question": "如图, 为了测量一条河流的宽度, 一测量员在河岸边相距 200 米的 $P 、 Q$ 两点分别测定对岸一棵\n\n\n\n树 $T$ 的位置, $T$ 在 $P$ 的正北方向, 且 $T$ 在 $Q$ 的北偏西 $70^{\\circ}$ 方向, 则河宽 $(P T$ 的长)可以表示为", "options": "A. $200 \\tan 70^{\\circ}$ 米\nB. $\\frac{200}{\\tan 70!}$ 米\nC. $200 \\sin 70^{\\circ}$ 米\nD. $\\frac{200}{\\sin 70!}$ 米", "subject": "度量几何学", "analysis": "在Rt $\\triangle P Q T$ 中,\n\n$\\because \\angle Q P T=90^{\\circ}, \\angle P Q T=90^{\\circ}-70^{\\circ}=20^{\\circ}$,\n\n$\\therefore \\angle P T Q=70^{\\circ}$,\n\n$\\therefore \\tan 70^{\\circ}=\\frac{P Q}{P T}$,\n\n$\\therefore P T=\\frac{P Q}{\\tan 70^{\\circ}}=\\frac{200}{\\tan 70^{\\circ}}$,\n\n即河宽 $\\frac{200}{\\tan 70^{\\circ}}$ 米,\n\n故选: $B$."} {"id": "3596", "image": [], "answer": "D", "solution": "null", "level": "九年级", "question": "关于三角函数有如下公式:\n\n$\\sin (\\alpha+\\beta)=\\sin \\alpha \\cos \\beta+\\cos \\alpha \\sin \\beta, \\sin (\\alpha-\\beta)=\\sin \\alpha \\cos \\beta-\\cos \\alpha \\sin \\beta$\n\n$\\cos (\\alpha+\\beta)=\\cos \\alpha \\cos \\beta-\\sin \\alpha \\sin \\beta, \\cos (\\alpha-\\beta)=\\cos \\alpha \\cos \\beta+\\sin \\alpha \\sin \\beta$\n\n$\\tan (\\alpha+\\beta)=\\frac{\\tan \\alpha+\\tan \\beta}{1-\\tan \\alpha \\tan \\beta}(1-\\tan \\alpha \\tan \\beta \\neq 0)$, 合理利用这些公式可以将一些角的三角函数值转化\n为特殊角的三角函数来求值, 如 $\\sin 90^{\\circ}=\\sin \\left(30^{\\circ}+60^{\\circ}\\right)=\\sin 30^{\\circ} \\cos 60^{\\circ}+\\cos 30^{\\circ} \\sin 60^{\\circ}=$ $\\frac{1}{2} \\times \\frac{1}{2}+\\frac{\\sqrt{3}}{2} \\times \\frac{\\sqrt{3}}{2}=1$\n\n利用上述公式计算下列三角函数 $(1) \\sin 105^{\\circ}=\\frac{\\sqrt{6}+\\sqrt{2}}{4},(2) \\tan 105^{\\circ}=-2-\\sqrt{3},(3) \\sin 15^{\\circ}=\\frac{\\sqrt{6}-\\sqrt{2}}{4}$,\n\n(4) $\\cos 90^{\\circ}=0$\n\n其中正确的个数有 $(\\quad)$", "options": "A. 1 个\nB. 2 个\nC. 3 个\nD. 4 个", "subject": "度量几何学", "analysis": "(1) $\\sin 105^{\\circ}=\\sin \\left(45^{\\circ}+60^{\\circ}\\right)$\n\n$$\n\\begin{gathered}\n=\\sin 60^{\\circ} \\cos 45^{\\circ}+\\cos 60^{\\circ} \\sin 45^{\\circ} \\\\\n=\\frac{\\sqrt{3}}{2} \\times \\frac{\\sqrt{2}}{2}+\\frac{1}{2} \\times \\frac{\\sqrt{2}}{2}\n\\end{gathered}\n$$\n\n$=\\frac{\\sqrt{6}+\\sqrt{2}}{4}$, 故此选项正确;\n\n$$\n\\begin{gathered}\n\\text { (2) } \\tan 105^{\\circ}=\\tan \\left(60^{\\circ}+45^{\\circ}\\right) \\\\\n=\\frac{\\tan 45^{\\circ}+\\tan 60^{\\circ}}{1-\\tan 45^{\\circ} \\tan 60^{\\circ}} \\\\\n=\\frac{1+\\sqrt{3}}{1-\\sqrt{3}} \\\\\n=\\frac{(1+\\sqrt{3})^{2}}{-2}\n\\end{gathered}\n$$\n\n$=-2-\\sqrt{3}$, 故此选项正确;\n\n$$\n\\begin{gathered}\n\\text { (3) } \\sin 15^{\\circ}=\\sin \\left(60^{\\circ}-45^{\\circ}\\right) \\\\\n=\\sin 60^{\\circ} \\cos 45^{\\circ}-\\cos 60^{\\circ} \\sin 45^{\\circ} \\\\\n=\\frac{\\sqrt{3}}{2} \\times \\frac{\\sqrt{2}}{2}-\\frac{1}{2} \\times \\frac{\\sqrt{2}}{2}\n\\end{gathered}\n$$\n\n$=\\frac{\\sqrt{6}-\\sqrt{2}}{4}$, 故此选项正确;\n\n$$\n\\begin{gathered}\n\\text { (4) } \\cos 90^{\\circ}=\\cos \\left(45^{\\circ}+45^{\\circ}\\right) \\\\\n=\\cos 45^{\\circ} \\cos 45^{\\circ}-\\sin 45^{\\circ} \\sin 45^{\\circ} \\\\\n=\\frac{\\sqrt{2}}{2} \\times \\frac{\\sqrt{2}}{2}-\\frac{\\sqrt{2}}{2} \\times \\frac{\\sqrt{2}}{2}\n\\end{gathered}\n$$\n\n$=0$, 故此选项正确;\n\n故正确的有 4 个.\n\n故选: $D$."} {"id": "3614", "image": ["4933.jpg", "4934.jpg"], "answer": "D", "solution": "null", "level": "九年级", "question": "如图, 有两张矩形纸片 $A B C D$ 和 $E F G H, A B=E F=2 \\mathrm{~cm}, B C=F G=$ $8 \\mathrm{~cm}$.把纸片 $A B C D$ 交叉叠放在纸片 $E F G H$ 上, 使重叠部分为平行四边形,且点 $D$ 与点 $G$ 重合.当两张纸片交叉所成的角 $\\alpha$ 最小时, $\\tan \\alpha$ 等于 $(\\quad)$", "options": "A. $\\frac{1}{4}$\nB. $\\frac{1}{2}$\nC. $\\frac{8}{17}$\nD. $\\frac{8}{15}$\n", "subject": "度量几何学", "analysis": "如图,\n\n\n\n$$\n\\because \\angle A D C=\\angle H D F=90^{\\circ}\n$$\n\n$\\therefore \\angle C D M=\\angle N D H$, 且 $C D=D H, \\angle H=\\angle C=90^{\\circ}$\n\n$\\therefore \\triangle C D M \\cong \\triangle H D N(A S A)$\n\n$\\therefore M D=N D$, 且四边形 $D N K M$ 是平行四边形\n\n$\\therefore$ 四边形 $D N K M$ 是菱形\n\n$$\n\\begin{gathered}\n\\therefore K M=D M \\\\\n\\because \\sin \\alpha=\\sin \\angle D M C=\\frac{C D}{M D}\n\\end{gathered}\n$$\n\n$\\therefore$ 当点 $B$ 与点 $E$ 重合时, 两张纸片交叉所成的角 $a$ 最小,\n\n设 $M D=a=B M ,$ 则 $C M=8-a ,$\n\n$\\because M D^{2}=C D^{2}+M C^{2}$,\n\n$\\therefore a^{2}=4+(8-a)^{2}$,\n\n$$\n\\begin{gathered}\n\\therefore a=\\frac{17}{4} \\\\\n\\therefore C M=\\frac{15}{4} \\\\\n\\therefore \\tan \\alpha=\\tan \\angle D M C=\\frac{C D}{M C}=\\frac{8}{15}\n\\end{gathered}\n$$\n\n故选: $D$."} {"id": "3615", "image": ["4935.jpg", "4936.jpg"], "answer": "B", "solution": "null", "level": "九年级", "question": "如图, 已知直线 $l_{1} / / l_{2} / / l_{3} / / l_{4}$, 相邻两条平行直线间的距离都是 1 , 如果正方形 $A B C D$ 的四个顶点分别在四条直线上, 则 $\\sin \\alpha=(\\quad)$", "options": "A. $\\frac{1}{2}$\nB. $\\frac{\\sqrt{5}}{5}$\nC. $\\frac{2 \\sqrt{5}}{5}$\nD. $\\frac{\\sqrt{3}}{2}$\n\n", "subject": "度量几何学", "analysis": "过 $D$ 作 $E F \\perp l_{1}$, 交 $l_{1}$ 于 $E$, 交 $l_{4}$ 于 $F$,\n\n$\\because E F \\perp l_{1}, l_{1} / / l_{2} / / l_{3} / / l_{4}$,\n\n$\\therefore E F$ 和 $l_{2}, l_{3}, l_{4}$ 的夹角都是 $90^{\\circ}$,\n\n即 $E F$ 与 $l_{2}, l_{3}, l_{4}$ 都垂直,\n\n\n\n$\\therefore D E=1, D F=2$.\n\n$\\because$ 四边形 $A B C D$ 是正方形,\n\n$\\therefore \\angle A D C=90^{\\circ}, A D=C D$,\n\n$\\therefore \\angle A D E+\\angle C D F=90^{\\circ}$,\n\n又 $\\because \\angle \\alpha+\\angle A D E=90^{\\circ}$,\n\n$\\therefore \\angle \\alpha=\\angle C D F$,\n\n$\\because A D=C D, \\angle A E D=\\angle D F C=90^{\\circ}$,\n\n$\\therefore \\triangle A D E \\cong \\triangle D C F$,\n\n$\\therefore D E=C F=1$,\n\n$\\therefore$ 在Rt $\\triangle C D F$ 中, $C D=\\sqrt{C F^{2}+D F^{2}}=\\sqrt{5}$,\n\n$\\therefore \\sin \\alpha=\\sin \\angle C D F=\\frac{C F}{C D}=\\frac{1}{\\sqrt{5}}=\\frac{\\sqrt{5}}{5}$.\n\n故选: $B$."} {"id": "3617", "image": ["4939.jpg"], "answer": "B", "solution": "null", "level": "九年级", "question": "如图, 在 $2 \\times 2$ 的正方形网格中, 以格点为顶点的 $\\triangle A B C$ 的面积等于 $\\frac{3}{2}$, 则 $\\sin \\angle C A B$ 的值是 $(\\quad)$", "options": "A. $\\frac{3}{2} \\sqrt{3}$\nB. $\\frac{3}{5}$\nC. $\\frac{\\sqrt{10}}{5}$\nD. $\\frac{3}{10}$", "subject": "度量几何学", "analysis": "如图:作 $C D \\perp A B$ 于 $D, A E \\perp B C$ 于 $E$,\n\n\n\n由勾股定理, 得\n\n$A B=A C=\\sqrt{5}, B C=\\sqrt{2}$.\n\n由等腰三角形的性质, 得\n\n$B E=\\frac{1}{2} B C=\\frac{\\sqrt{2}}{2}$.\n\n由勾股定理, 得 $A E=\\sqrt{A B^{2}-B E^{2}}=\\frac{3 \\sqrt{2}}{2}$,\n\n由三角形的面积, 得\n\n$\\frac{1}{2} A B \\cdot C D=\\frac{1}{2} B C \\cdot A E$.\n\n即 $C D=\\frac{\\sqrt{2} \\times \\frac{3 \\sqrt{2}}{2}}{\\sqrt{5}}=\\frac{3 \\sqrt{5}}{5}$,\n\n$\\sin \\angle C A B=\\frac{C D}{A C}=\\frac{\\frac{3 \\sqrt{5}}{5}}{\\sqrt{5}}=\\frac{3}{5}$.\n\n故选 $B$."} {"id": "3618", "image": ["4940.jpg", "4941.jpg"], "answer": "A", "solution": "null", "level": "九年级", "question": "一个正方体沿斜坡向下滑动, 其截面如图所示: $\\angle B=90^{\\circ}$, 正方形 $D E F H$的边长为 $1 m, B C=4 m, A C=8 m$. 当正方形 $D E F H$ 运动到某位置, 使得 $D C^{2}=A E^{2}+B C^{2}$, 此时 $A E$ 的长为 $(\\quad) m$.", "options": "A. $\\frac{49}{16}$\nB. 4\nC. $\\sqrt{17}$\nD. $2 \\sqrt{3}$\n\n", "subject": "度量几何学", "analysis": "如图, 连接 $C D$,\n\n\n\n设 $A E=x$,\n\n可得 $E C=8-x$.\n$\\because$ 正方形 $D E F H$ 的边长为 1 米, 即 $D E=1$ 米,\n\n$\\therefore D C^{2}=D E^{2}+E C^{2}=1+(8-x)^{2}, A E^{2}+B C^{2}=x^{2}+16$,\n\n$\\because D C^{2}=A E^{2}+B C^{2}$,\n\n$\\therefore 1+(8-x)^{2}=x^{2}+16$,\n\n解得: $x=\\frac{49}{16}$,\n\n所以, 当 $A E=\\frac{49}{16}$ 米时, 有 $D C^{2}=A E^{2}+B C^{2}$.\n\n故选 $A$."} {"id": "3619", "image": ["4942.jpg"], "answer": "A", "solution": "null", "level": "九年级", "question": "如果一个三角形的面积和周长都被同一直线所平分, 那么该直线必通过这个三角形的 ( )", "options": "A. 内心\nB. 外内\nC. 重心\nD. 垂心", "subject": "度量几何学", "analysis": "设直线 $D E$ 平分 $\\triangle A B C$ 的周长和面积,点 $D 、 E$ 分别在边 $A B$ 和 $A C$ 上, 作 $\\angle A$ 的平分线交 $D E$ 于点 $P$, 记点 $P$ 到 $A B 、 A C$ 的距离为 $r, P$ 到 $B C$ 的距离为 $r_{1}$, 于是依题意, 有 $\\left\\{\\begin{array}{l}A D+A E=D B+B C+C E, \\\\ \\frac{r}{2}(A D+A E)=\\frac{r}{2}(D B+C E)+\\frac{r_{1}}{2} B C, \\text { 解得 } r=r_{1} \\text {, 即点 } P \\text { 为 } \\triangle A B C \\text { 的内心. 故选 } A .\\end{array}\\right.$\n\n"} {"id": "3636", "image": ["4978.jpg", "4979.jpg"], "answer": "C", "solution": "null", "level": "九年级", "question": "如图, $\\triangle A B C$ 是等腰直角三角形, $A C=B C=2$, 以斜边 $A B$ 上的点 $O$ 为圆心的圆分别与 $A C 、 B C$ 相切于点 $E 、 F$, 与 $A B$ 分别相交于点 $G 、 H$, 且 $E H$的延长线与 $C B$ 的延长线交于点 $D$, 则 $C D$ 的长为 $(\\quad)$", "options": "A. $2 \\sqrt{2}-1$\nB. $2 \\sqrt{2}$\nC. $\\sqrt{2}+1$\nD. $2 \\sqrt{2}-\\frac{1}{2}$\n\n", "subject": "度量几何学", "analysis": "如右图所示, 连接 $O E 、 O F$,\n\n$\\because \\odot O$ 与 $A C 、 B C$ 切于点 $E 、 F$,\n\n$\\therefore \\angle O E C=\\angle O F C=90^{\\circ}, O E=O F$,\n\n又 $\\because \\triangle A B C$ 是等腰直角三角形,\n\n\n\n$\\therefore \\angle C=90^{\\circ}$,\n\n$\\therefore$ 四边形 $C E O F$ 是正方形,\n\n$\\therefore O E / / B C$,\n\n又 $\\because$ 以斜边 $A B$ 上的点 $O$ 为圆心的圆分别与 $A C 、 B C$ 相切于点 $E 、 F, O E=O F$,\n\n$\\therefore O$ 在 $\\angle A C B$ 的角平分线上,\n\n$\\because A C=B C$\n\n$\\therefore O$ 是 $A B$ 中点,\n\n$\\therefore A E=C E$,\n\n又 $\\because A C=2$,\n\n$\\therefore A E=C E=1$,\n\n$\\therefore O E=O F=C E=1$,\n\n$\\therefore O H=1$,\n\n$\\because O E / / C D$,\n\n$\\therefore \\triangle O E H \\sim \\triangle B D H$\n\n$\\therefore \\frac{O E}{O H}=\\frac{D B}{B H}$\n\n又 $\\because A B=\\sqrt{A C^{2}+B C^{2}}=2 \\sqrt{2}$,\n\n$\\therefore O B=\\sqrt{2}$,\n$\\therefore \\frac{1}{1}=\\frac{D B}{\\sqrt{2}-1}$,\n\n$\\therefore B D=\\sqrt{2}-1$,\n\n$\\therefore C D=2+B D=\\sqrt{2}+1$,\n\n故选: $C$."} {"id": "3637", "image": ["4980.jpg", "4981.jpg"], "answer": "B", "solution": "null", "level": "九年级", "question": "如图, $\\odot O$ 的外切正八边形 $A B C D E F G H$ 的边长 2 , 则 $\\odot O$ 的半径为 ( )", "options": "A. 2\nB. $1+\\sqrt{2}$\nC. 3\n\n\nD. $2+\\sqrt{2}$", "subject": "度量几何学", "analysis": "设 $D E$ 与 $\\odot O$ 相切于点 $N$, 连接 $O D 、 O E 、 O N$, 作 $D M \\perp O E$ 于 $M$, 如图所示:\n\n则 $O N \\perp D E, D E=2, O D=O E, \\angle D O E=\\frac{360^{\\circ}}{8}=45^{\\circ}$,\n\n$\\because D M \\perp O E$,\n\n$\\therefore \\triangle O D M$ 是等腰直角三角形,\n\n\n\n$\\therefore D M=O M, O E=O D=\\sqrt{2} D M$,\n\n设 $O M=D M=x$, 则 $O D=O E=\\sqrt{2} x, E M=O E-O M=(\\sqrt{2}-1) x$,\n\n在Rt $\\triangle D E M$ 中, 由勾股定理得: $x^{2}+(\\sqrt{2}-1)^{2} x^{2}=2^{2}$,\n\n解得: $x^{2}=2+\\sqrt{2}$,\n\n$\\because \\triangle O D E$ 的面积 $=\\frac{1}{2} D E \\times O N=\\frac{1}{2} O E \\times D M$,\n\n$\\therefore O N=\\frac{O E \\times D M}{D E}=\\frac{\\sqrt{2} x^{2}}{2}=\\frac{\\sqrt{2}(2+\\sqrt{2})}{2}=\\sqrt{2}+1$,\n\n即 $\\odot O$ 的半径为: $1+\\sqrt{2}$;\n\n故选:B."} {"id": "3661", "image": ["5025.jpg"], "answer": "D", "solution": "null", "level": "九年级", "question": "已知某几何体的三视图如图所示, 其中俯视图为等边三角形, 则该几何体的表面积为( $)$", "options": "A. $6 \\mathrm{~cm}^{2}$\nB. $18 \\mathrm{~cm}^{2}$\nC. $2 \\sqrt{3} \\mathrm{~cm}^{2}$\nD. $(18+2 \\sqrt{3}) \\mathrm{cm}^{2}$\n\n\n\n主视图 左视图\n\n$\\checkmark$\n\n俯视图", "subject": "度量几何学", "analysis": "根据三视图可得几何体是三棱柱, 底面三角形是正三角形, 边长为 $2 \\mathrm{~cm}$, 棱柱的高为 $3 \\mathrm{~cm}$,\n\n底面面积为: $2 \\times \\sqrt{3} \\div 2=\\sqrt{3}\\left(\\mathrm{~cm}^{2}\\right)$,\n\n侧面积是 $6 \\times 3=18\\left(\\mathrm{~cm}^{2}\\right)$,\n\n则这个几何体的表面积是 $(18+2 \\sqrt{3}) \\mathrm{cm}^{2}$.\n\n故选 $D$."} {"id": "3666", "image": ["5036.jpg"], "answer": "B", "solution": "null", "level": "九年级", "question": "在阳光下, 一名同学测得一根长为 1 米的垂直地面的竹竿的影长为 0.6 米,同时另一名同学测量树的高度时, 发现树的影子不全落在地面上, 有一部分落在教学楼的第一级台阶上, 测得此影子长为 0.2 米, 一级台阶高为 0.3 米,如图所示, 若此时落在地面上的影长为 4.42 米, 则树高为 ( )", "options": "A. 6.93 米\nB. 8 米\nC. 11.8 米\nD. 12 米", "subject": "度量几何学", "analysis": "如图,\n\n\n\n$\\because \\frac{D E}{E H}=\\frac{1}{0.6}$,\n\n$\\therefore E H=0.3 \\times 0.6=0.18$\n\n$\\therefore A F=A E+E H+H F=4.42+0.18+0.2=4.8$,\n\n$\\because \\frac{A B}{A F}=\\frac{1}{0.6}$,\n\n$\\therefore A B=\\frac{4.8}{0.6}=8$ (米),\n\n故选 $B$."} {"id": "3677", "image": [], "answer": "C", "solution": "null", "level": "九年级", "question": "在同一时刻物体的高度与它的影长成正比例. 在某一时刻, 有人测得一个高为 1 米的竹笔的影长为 3 米; 某高楼的影长为 60 米, 那么高楼的高度是 $(\\quad)$", "options": "A. 1 米\nB. 10 米\nC. 20 米\nD. 30 米", "subject": "度量几何学", "analysis": "设此高楼的高度为 $h$ 米,\n\n$\\because$ 在同一时刻, 有人测得一高为 1.8 米得竹等的影长为 3 米, 某高楼的影长为 60 米,\n\n$\\therefore \\frac{1}{3}=\\frac{h}{60}$, 解得 $h=20$.\n\n$\\therefore$ 高楼的高度是 20 米.\n\n故选: $C$."} {"id": "3686", "image": ["5079.jpg"], "answer": "A", "solution": "null", "level": "九年级", "question": "如图, 已知 $B C$ 是圆柱的底面直径, $A B$ 是圆柱的高, 在圆柱的侧面上,过点 $A 、 C$ 嵌有一圈路径最短的金属丝, 现将圆柱侧面沿 $A B$ 剪开, 若展开图中, 金属丝与底面周长围成的图形的面积是 $5 \\mathrm{~cm}^{2}$, 该圆柱的侧面积是 $(\\quad) \\mathrm{cm}^{2}$", "options": "A. 10\nB. 20\nC. $10 \\pi$\nD. $20 \\pi$", "subject": "度量几何学", "analysis": "如图, 圆柱的侧面展开图为长方形, $A C=A^{\\prime} C$, 且点 $C$ 为 $B B^{\\prime}$ 的中点,\n\n\n\n$\\because A A^{\\prime} / / B B^{\\prime}$, 四边形 $A B B^{\\prime} A^{\\prime}$ 是矩形,\n\n$\\therefore S_{\\triangle A A C C}=\\frac{1}{2} S_{\\text {长方形 } A B B, A,}$,\n\n又 :展开图中, $S_{\\triangle A A \\prime C}=5 \\mathrm{~cm}^{2}$,\n\n$\\therefore$ 圆柱的侧面积是 $10 \\mathrm{~cm}^{2}$.\n故选 $A$."} {"id": "3884", "image": ["5464.jpg", "5465.jpg"], "answer": "C", "solution": "null", "level": "九年级", "question": "如图, $\\mathrm{PO}$ 是 $\\odot \\mathrm{O}$ 外一点, $\\mathrm{PA}$ 是 $\\odot \\mathrm{O}$ 的切线, $\\mathrm{PO}=26 \\mathrm{~cm}, \\mathrm{PA}=\" 24 \" \\mathrm{~cm}$, 则 $\\odot \\mathrm{O}$ 的周长为 $(\\mathrm{)}$\n\n", "options": "A. $18 \\pi \\mathrm{cm}$\nB. $16 \\pi \\mathrm{cm}$\nC. $20 \\pi \\mathrm{cm}$\nD. $24 \\pi \\mathrm{cm}$", "subject": "度量几何学", "analysis": "如图, 设 $\\mathrm{PA}$ 与 $\\odot \\mathrm{O}$ 相交于点 $\\mathrm{D}$, 延长 $\\mathrm{PO}$ 交 $\\odot \\mathrm{O}$ 于另一点 $\\mathrm{C}$,\n\n\n\n若 $\\odot \\mathrm{O}$ 的半径为 $\\mathrm{r}$, 则 $\\because \\mathrm{PO}=26, \\therefore \\mathrm{PD}=26-\\mathrm{r}, \\mathrm{PC}=26+\\mathrm{r}$.\n\n$\\because \\mathrm{PA}$ 是 $\\odot \\mathrm{O}$ 的切线, $\\therefore$ 根据切割线定理, $\\mathrm{PA}^{2}=\\mathrm{PC} \\cdot \\mathrm{PD} . \\because \\mathrm{PA}=\" 24 \" \\mathrm{~cm}$,\n\n$\\therefore 24^{2}=(26+r) \\cdot(26-r) \\Rightarrow 24^{2}=26^{2}-r^{2} \\Rightarrow r^{2}=26^{2}-24^{2}=(26+24)(6-24)=50 \\times 2=100 \\Rightarrow \\mathrm{r}=10$.\n\n$\\therefore \\odot \\mathrm{O}$ 的周长为 $2 \\pi \\mathrm{r}=20 \\pi(\\mathrm{cm})$. 故选 C."} {"id": "3890", "image": ["5476.jpg"], "answer": "B", "solution": "null", "level": "九年级", "question": "如图, 一圆内切四边形 $A B C D$, 且 $A B=16, C D=10$, 则四边形的周长为 ( )\n\n", "options": "A. 50\nB. 52\nC. 54\nD. 56", "subject": "度量几何学", "analysis": "根据切线长定理, 可以证明圆外切四边形的性质: 圆外切四边形的两组对边的和相等,所以四边形的周长为: $2 \\times(16+10)=52$. 故选 B."} {"id": "3891", "image": ["5477.jpg"], "answer": "C", "solution": "null", "level": "九年级", "question": "如图, $A B$ 为半圆 $O$ 的直径, $A D 、 B C$ 分别切 $\\odot O$ 于 $A, B$ 两点, $C D$ 切 $\\odot O$ 于点 $E, A D$ 与 $C D$相交于 $\\mathrm{D}, \\mathrm{BC}$ 与 $\\mathrm{CD}$ 相交于 $\\mathrm{C}$, 连结 $\\mathrm{OD} 、 \\mathrm{OE} 、 \\mathrm{OC}$, 对于下列结论: (1) $\\mathrm{AD}+\\mathrm{BC}=\\mathrm{CD}$; (2) $\\angle \\mathrm{DOC}=90^{\\circ}$; (3) $\\mathrm{S}_{\\text {粠形 } \\mathrm{ABCD}}=\\frac{1}{2} \\mathrm{CD} \\cdot \\mathrm{OA}$; (4) $\\frac{O D}{D E}=\\frac{C D}{O D}$. 其中结论正确的个数是( )\n\n", "options": "A. 1\nB. 2\nC. 3\nD. 4", "subject": "度量几何学", "analysis": "根据切线的性质可得: $\\mathrm{AD}=\\mathrm{DE}, \\mathrm{BC}=\\mathrm{CE}$, 则 $\\mathrm{AD}+\\mathrm{BC}=\\mathrm{DE}+\\mathrm{CE}=\\mathrm{CD}$, 则(1)正确; 根据题意可知: $\\triangle \\mathrm{AOD}$ 和 $\\triangle \\mathrm{EOD}$ 全等, $\\triangle \\mathrm{BOC}$ 和 $\\triangle \\mathrm{EOC}$ 全等, 则 $\\angle \\mathrm{DOC}=90^{\\circ}$, 故(2)正确; 梯形的面积 $=(\\mathrm{AD}+\\mathrm{BC}) \\cdot \\mathrm{AB} \\div 2=\\mathrm{CD} \\cdot \\mathrm{AB} \\div 2=\\mathrm{CD} \\cdot \\mathrm{OA}$, 则(3)错误; 根据题意可知: $\\triangle \\mathrm{DOE}$ 和 $\\triangle \\mathrm{DCO}$ 相似, 则(4)正确,故选 C."} {"id": "3892", "image": ["5478.jpg"], "answer": "B", "solution": "null", "level": "九年级", "question": "如图, $A B$ 是 $\\odot O$ 的切线, $B$ 为切点, $A O$ 与 $\\odot O$ 交于点 $C$, 若 $\\angle B A O=30^{\\circ}$, 则 $\\angle O C B$ 的度数为 ( )\n\n", "options": "A. $30^{\\circ}$\nB. $60^{\\circ}$\nC. $50^{\\circ}$\nD. $40^{\\circ}$", "subject": "度量几何学", "analysis": "$\\because A B$ 是 $\\odot O$ 的切线, $B$ 为切点, $\\therefore \\angle O B A=90^{\\circ} . \\because \\angle B A O=30^{\\circ}, \\therefore \\angle O=60^{\\circ} . \\because O B=O C$, $\\therefore \\triangle O B C$ 是等边三角形, $\\therefore \\angle O C B=60^{\\circ}$. 故选 B."} {"id": "3896", "image": ["5483.jpg", "5484.jpg"], "answer": "A", "solution": "null", "level": "九年级", "question": "如图, $P A 、 P B$ 分别切 $\\odot O$ 于 $A 、 B$ 两点, 点 $C$ 在优弧 $A C B$ 上, $\\angle P=80^{\\circ}$, 则 $\\angle C$ 的度数为\n\n", "options": "A. $50^{\\circ}$\nB. $60^{\\circ}$\nC. $70^{\\circ}$\nD. $80^{\\circ}$", "subject": "度量几何学", "analysis": "$\\because \\mathrm{PA}$ 是圆的切线.\n\n\n\n$\\therefore \\angle \\mathrm{OAP}=90^{\\circ}$, 同理 $\\angle \\mathrm{OBP}=90^{\\circ}$,\n\n根据四边形内角和定理可得:\n\n$\\angle \\mathrm{AOB}=360^{\\circ}-\\angle \\mathrm{OAP}-\\angle \\mathrm{OBP}-\\angle \\mathrm{P}=360^{\\circ}-90^{\\circ}-90^{\\circ}-80^{\\circ}=100^{\\circ}$,\n\n$\\therefore \\angle \\mathrm{C}=\\frac{1}{2} \\angle \\mathrm{AOB}=50^{\\circ}$. 故选 A."} {"id": "3915", "image": ["5525.jpg"], "answer": "B", "solution": "null", "level": "九年级", "question": "在 $\\triangle \\mathrm{ABC}$ 中, $\\angle \\mathrm{A}=90^{\\circ}, \\mathrm{AB}=3 \\mathrm{~cm}, \\mathrm{AC}=4 \\mathrm{~cm}$, 若以顶点 $\\mathrm{A}$ 为圆心, $3 \\mathrm{~cm}$ 长为半径作 $\\odot \\mathrm{A}$, 则\n$\\mathrm{BC}$ 与 $\\odot \\mathrm{A}$ 的位置关系是 $(\\quad)$", "options": "A. 相切\nB. 相交\nC. 相离\nD. 无法确定", "subject": "度量几何学", "analysis": "做 $\\mathrm{AD} \\perp \\mathrm{BC}$,\n\n\n\n$\\because \\angle A=90^{\\circ}, A B=3 \\mathrm{~cm}, A C=4 \\mathrm{~cm}$, 若以 $A$ 为圆心 $3 \\mathrm{~cm}$ 为半径作 $\\odot A$,\n\n$\\therefore \\mathrm{BC}=5, \\therefore \\mathrm{AD} \\times \\mathrm{BC}=\\mathrm{AC} \\times \\mathrm{AB}$, 即 $\\mathrm{AD} \\times 5=4 \\times 3$ 解得: $\\mathrm{AD}=2.4,2.4<3$,\n\n$\\therefore$ 直线 $\\mathrm{BC}$ 与 $\\odot \\mathrm{A}$ 的位置关系是: 相交. 故选: $\\mathrm{B}$."} {"id": "3916", "image": ["5526.jpg"], "answer": "A", "solution": "null", "level": "九年级", "question": "如图, 在 Rt $\\triangle A B C$ 中, $A C=5, B C=12, \\odot O$ 分别与边 $A B, A C$ 相切, 切点分别为 $E, C$,则 $\\odot O$ 的半径是 ()\n\n", "options": "A. $\\frac{10}{3}$\nB. $\\frac{16}{3}$\nC. $\\frac{20}{3}$\nD. $\\frac{23}{3}$", "subject": "度量几何学", "analysis": "$\\because \\mathrm{AE}=\\mathrm{AC}=5, \\mathrm{AC}=5, \\mathrm{BC}=12$,\n\n$\\therefore \\mathrm{AB}=13, \\quad \\therefore \\mathrm{BE}=8$\n\n$\\because \\mathrm{BE}^{2}=\\mathrm{BD} \\cdot \\mathrm{BC}, \\quad \\therefore \\mathrm{BD}=\\frac{16}{3}$,\n\n$\\therefore \\mathrm{CD}=\\frac{20}{3}, \\therefore$ 圆的半径是 $\\frac{10}{3}$, 故选 A."} {"id": "3918", "image": ["5528.jpg", "5529.jpg"], "answer": "A", "solution": "null", "level": "九年级", "question": "如图, 点 $\\mathrm{O}$ 是 $\\angle \\mathrm{BAC}$ 的边 $\\mathrm{AC}$ 上的一点, $\\odot \\mathrm{O}$ 与边 $\\mathrm{AB}$ 相切于点 $\\mathrm{D}$, 与线段 $\\mathrm{AO}$ 相交于点 $\\mathrm{E}$, 若点 $\\mathrm{P}$ 是 $\\odot \\mathrm{O}$ 上一点, 且 $\\angle \\mathrm{EPD}=35^{\\circ}$, 则 $\\angle \\mathrm{BAC}$ 的度数为 $(\\quad)$\n\n", "options": "A. $20^{\\circ}$\nB. $35^{\\circ}$\nC. $55^{\\circ}$\nD. $70^{\\circ}$", "subject": "度量几何学", "analysis": "连接 $\\mathrm{OD}$,\n\n\n\n$\\because \\odot \\mathrm{O}$ 与边 $\\mathrm{AB}$ 相切于点 $\\mathrm{D}, \\quad \\therefore \\mathrm{OD} \\perp \\mathrm{AD}$,\n\n$\\therefore \\angle \\mathrm{ADO}=90^{\\circ}, \\because \\angle \\mathrm{EPD}=35^{\\circ}$,\n\n$\\therefore \\angle \\mathrm{EOD}=2 \\angle \\mathrm{EPD}=70^{\\circ}, \\therefore \\angle \\mathrm{BAC}=90^{\\circ}-\\angle \\mathrm{EOD}=20^{\\circ}$. 故选 A."} {"id": "3920", "image": [], "answer": "C", "solution": "null", "level": "九年级", "question": "$\\sin 60^{\\circ}$ 的值等于 $(\\quad)$", "options": "A. $\\frac{1}{2}$\nB. $\\frac{\\sqrt{2}}{2}$\nC. $\\frac{\\sqrt{3}}{2}$\nD. 1", "subject": "度量几何学", "analysis": "根据特殊角的三角函数值,可知:\n\n$\\sin 60^{\\circ}=\\frac{\\sqrt{3}}{2}$. 故选 C."} {"id": "3931", "image": [], "answer": "C", "solution": "null", "level": "九年级", "question": "下列命题: (1)所有锐角三角函数值都为正数; (2)解直角三角形时只需已知除直角外的两个元素; (3)Rt $\\triangle A B C$ 中, $\\angle B=90^{\\circ}$, 则 $\\sin ^{2} A+\\cos ^{2} A=1$; (4) $R t \\triangle A B C$ 中, $\\angle A=90^{\\circ}$, 则 $\\tan C \\cdot \\sin C=\\cos C$. 其中正确的命题有 $(\\quad)$", "options": "A. 0 个\nB. 1 个\nC. 2 个\nD. 3 个", "subject": "度量几何学", "analysis": "(1)根据锐角三角函数的定义知所有的锐角三角函数值都是正数, 故正确;\n\n(2)两个元素中, 至少得有一条边, 故错误;\n\n(3)根据锐角三角函数的概念, 以及勾股定理, 得则 $\\sin ^{2} A+\\cos ^{2} A=\\frac{a 2+c 2}{b 2}=1$, 故正确;\n\n(4)根据锐角三角函数的概念, 得 $\\tan C=\\frac{c}{b}, \\sin C=\\frac{c}{a}, \\cos C=\\frac{b}{a}$, 则 $\\tan C \\cdot \\cos C=\\sin C$, 故错误. 故选 C."} {"id": "3939", "image": [], "answer": "A", "solution": "null", "level": "九年级", "question": "将 $R t \\triangle A B C$ 的三边分别扩大 2 倍, 得到 $R t \\triangle A^{\\prime} B^{\\prime} C^{\\prime}$, 则 ( )", "options": "A. $\\sin A=\\sin A^{\\prime}$\nB. $\\sin A>\\sin A^{\\prime}$\nC. $\\sin A<\\sin A^{\\prime}$\nD. 不能确定", "subject": "度量几何学", "analysis": "Rt $\\triangle A B C$ 的三边分别扩大 2 倍, 得到 Rt $\\triangle A^{\\prime} B^{\\prime} C^{\\prime}$, 对边与斜边的比值不变,故 A 正确; 故选: A."} {"id": "3346", "image": ["4392.jpg", "4393.jpg"], "answer": "A", "solution": "null", "level": "九年级", "question": "如图, 按照三视图确定该几何体的侧面积是(单位: $\\mathrm{cm})($ )\n\n\n", "options": "A. $24 \\pi \\mathrm{cm}^{2}$\nB. $48 \\pi \\mathrm{cm}^{2}$\nC. $60 \\pi c m^{2}$\nD. $80 \\pi c m^{2}$", "subject": "度量几何学", "analysis": "解:由主视图和左视图为三角形判断出是锥体, 由俯视图是圆形可判断出这个几何体应该是圆锥;\n\n根据三视图知: 该圆锥的母线长为 $6 \\mathrm{~cm}$, 底面半径为 $8 \\div 2=4 \\mathrm{~cm}$,\n\n故侧面积 $=\\pi r l=\\pi \\times 6 \\times 4=24 \\pi \\mathrm{cm}^{2}$.\n\n故选: $A$.\n\n由主视图和左视图确定是柱体, 锥体还是球体, 再由俯视图确定具体形状, 确定圆锥的母线长和底面半径, 从而确定其侧面积.\n\n此题考查学生对三视图掌握程度和灵活运用能力, 同时也体现了对空间想象能力方面的考查."} {"id": "3347", "image": ["4394.jpg", "4395.jpg", "4396.jpg"], "answer": "D", "solution": "null", "level": "九年级", "question": "如图, 一个几何体的三视图分别是两个矩形、一个扇形, 则这个几何体表面积的大小为( )\n\n\n\n(主视图)\n\n\n\n(左视图)\n\n\n\n(俯视图)", "options": "A. $12 \\pi$\nB. $15 \\pi$\nC. $12 \\pi+6$\nD. $15 \\pi+12$", "subject": "度量几何学", "analysis": "【分析】\n\n本题主要考查由三视图判断几何体, 由几何体的三视图得出该几何体是几何体是长方体与三棱柱的组合体,结合图中数据求出组合体的表面积即可.\n\n【解答】\n\n解: 由几何体的三视图可得: 该几何体是长方体、两个扇形和一个矩形的组合体, 该组合体的表\n\n面积为: $S=2 \\times 2 \\times 3+\\frac{270 \\pi \\times 2^{2}}{360} \\times 2+\\frac{270 \\pi \\times 2}{180} \\times 3=12+15 \\pi$,\n\n故选 $D$."} {"id": "3020", "image": ["3919.jpg", "3920.jpg"], "answer": "B", "solution": "null", "level": "九年级", "question": "(本题 3 分)(2020$\\cdot$浙江宁波$\\cdot$九年级期中)三孔桥横截面的三个孔都呈抛物线形,两小孔形状、大小完全相同. 当水面刚好淹没小孔时, 大孔水面宽度为 10 米, 孔顶离水面 1.5 米; 当水位下降, 大孔水面宽度为 14 米时, 单个小孔的水面宽度为 4 米, 若大孔水面宽度为 20 米,则单个小孔的水面宽度为()", "options": "A. $4 \\sqrt{3}$ 米B. $5 \\sqrt{2}$ 米C. $2 \\sqrt{13}$ 米D. 7 米", "subject": "度量几何学", "analysis": "B【解析】【分析】根据题意,可以画出相应的拋物线,然后即可得到大孔所在抛物线解析式,再求出顶点为A的小孔所在抛物线的解析式,将x=-10代入可求解.【详解】解:如图,建立如图所示的平面直角坐标系,由题意可得MN=4,EF=14,BC=10,DO=frac32,设大孔所在抛物线解析式为y=ax^2+frac32,becauseBC=10,therefore点B(-5,0),therefore0=atimes(-5)^2+frac32,thereforea=-frac350,therefore大孔所在抛物线解析式为y=-frac350x^2+frac32,设点A(b,0),则设顶点为A的小孔所在拖物线的解析式为y=m(x-b)^2,becauseEF=14,therefore点E的横坐标为-7,therefore点E坐标为left(-7,-frac3625right),therefore-frac3625=m(x-b)^2,thereforex_1=frac65sqrt-frac1m+b,x_2=-frac65sqrt-frac1m+b,thereforeMN=4,thereforeleft|frac65sqrt-frac1m+b-left(-frac65sqrt-frac1m+bright)right|=4thereforem=-frac925,therefore顶点为A的小孔所在抛物线的解析式为y=-frac925(x-b)^2,because大孔水面宽度为20米,therefore当x=-10时,y=-frac92,therefore-frac92=-frac925(x-b)^2,thereforex_1=frac52sqrt2+b,x_2=-frac5sqrt22+btherefore单个小孔的水面宽度=left|left(frac52sqrt2+bright)-left(-frac52sqrt2+bright)right|=5sqrt2(米),故选:B.【点睛】本题考查二次函数的应用,解答本题的关键是明确题意,利用二次函数的性质和数形结合的思想解答."} {"id": "3061", "image": [], "answer": "B", "solution": "null", "level": "九年级", "question": "(本题 3 分) (2022 浙江绍兴$\\cdot$九年级期末) 已知扇形的圆心角为 $120^{\\circ}$, 半径为 $3 \\mathrm{~cm}$, 则弧长为", "options": "A. $\\frac{2 \\pi}{3} \\mathrm{~cm}$B. $2 \\pi \\mathrm{cm}$C. $4 \\mathrm{~cm}$D. $\\frac{\\pi}{3} \\mathrm{~cm}$", "subject": "度量几何学", "analysis": "B【解析】【分析】扇形的弧长=圆形周长timesfractext扇形圆心角360^circ,根据公式列出算式计算即可.【详解】解:扇形的弧长:2times3timespitimesfrac120^circ360^circ=2pi,故选:B.【点睛】本题考查扇形的弧长,掌握扇形的弧长公式是解决本题的关键."} {"id": "3064", "image": ["3964.jpg", "3965.jpg"], "answer": "C", "solution": "null", "level": "九年级", "question": "(本题 3 分) (2022$\\cdot$浙江湖州$\\cdot$模拟预测)如图, $\\triangle A B C$ 内接于 $\\odot O, \\angle A=$ $50^{\\circ} . O D \\perp B C$, 垂足为 $E$, 连接 $B D$, 则 $\\angle C B D$ 的大小为 ( ).", "options": "A. $50^{\\circ}$B. $60^{\\circ}$C. $25^{\\circ}$D. $30^{\\circ}$", "subject": "度量几何学", "analysis": "C【解析】【分析】连接CD,根据圆内接四边形的性质得到angleCDB=180^circ-angleA=130^circ,根据垂径定理得到E是边BC的中点,由垂直平分线的性质得到BD=CD,根据等腰三角形的性质与三角形内角和定理即可得到结论.【详解】解:连接CDbecause四边形ABDC是圆内接四边形,angleA=50^circthereforeangleCDB+angleA=180^circthereforeangleCDB=180^circ-angleA=130^circbecauseODperpBCthereforeE是边BC的中点thereforeBD=CDthereforeangleCBD=angleBCD=frac12left(180^circ-angleCDBright)=frac12left(180^circ-130^circright)=25^circ故选:C.【点睛】本题考查三角形的外接圆和外心,圆内接四边形的性质,垂径定理,等腰三角形的性质.正确作出辅助线(即连接CD)构造等腰三角形是解题的关键."} {"id": "3065", "image": ["3966.jpg", "3967.jpg"], "answer": "C", "solution": "null", "level": "九年级", "question": "(本题 3 分)(2022$\\cdot$浙江$\\cdot$温州绣山中学二模)如图, $A B$ 是半圆 $O$ 的直径,$A B=20$, 弦 $D C / / A B, E$ 是 $D C$ 上的点, 连结 $O E, E B$. 若 $O E=E B$,$E C: E D=3: 13$, 则 $O E$ 的长为 $(\\quad)$", "options": "A. $\\sqrt{69}$B. 8C. $\\sqrt{61}$D. $5 \\sqrt{2}$", "subject": "度量几何学", "analysis": "C【解析】【分析】过点O作OMperpCD,过点E作ENperpAB,连接OC,设EC=3x,则ED=13x,先证明四边形MONE是矩形,求出x的值,再根据勾股定理求出OM及OE的值.【详解】解:过点O作OMperpCD,过点E作ENperpAB,连接OC,垂足分别为点M、N,becauseEC:ED=3:13therefore设EC=3x,则ED=13x,thereforeCD=16x,becauseOMperpCDthereforeCM=DM=8x,thereforeME=CM-CE=8x-3x=5x,becauseOMperpCD,quadDC//AB,ENperpAB,thereforeangleMON=angleOME=angleONE=90^circ,therefore四边形MONE是矩形,thereforeON=ME=5x,becauseAB=20,thereforeOB=10,becauseOE=EB,ENperpAB,thereforeON=BN=5,therefore5x=5,即x=1,thereforeCM=8,thereforeOM=sqrtOC^2-CM^2=sqrt10^2-8^2=6,thereforeOE=sqrtOM^2+EM^2=sqrt6^2+5^2=sqrt61故选:C【点睛】本题考查垂径定理,勾股定理等知识,解题的关键是掌握垂径定理的性质,灵活运用所学知识解决问题."} {"id": "3083", "image": [], "answer": "B", "solution": "null", "level": "九年级", "question": "(本题 3 分) (2021$\\cdot$浙江温州$\\cdot$九年级期末) 已知一个扇形的半径长是 6 , 圆心角为 $90^{\\circ}$,则这个扇形的面积为 $(\\quad)$", "options": "A. $12 \\pi$B. $9 \\pi$C. $6 \\pi$D. $3 \\pi$", "subject": "度量几何学", "analysis": "B【分析】根据扇形的面积公式直接求解即可.【详解】解:由扇形的面积公式可得,这个扇形的面积为frac90^circpitimes6^2360^circ=9pi故选B【点睛】此题考查了扇形面积的计算,掌握扇形面积的计算公式是解题的关键."} {"id": "3085", "image": ["3999.jpg"], "answer": "D", "solution": "null", "level": "九年级", "question": "(本题 3 分) (2022- 浙江$\\cdot$九年级专题练习) 如图, 四边形 $A B C D$ 内接于 $\\odot O, \\angle D-$ $\\angle B=40^{\\circ}$, 连接 $A O, C O$, 则 $\\angle A O C$ 的度数为 $(\\quad)$", "options": "A. $110^{\\circ}$B. $120^{\\circ}$C. $130^{\\circ}$D. $140^{\\circ}$", "subject": "度量几何学", "analysis": "D【分析】根据圆内接四边形的性质得出angleB+angleD=180^circ,根据angleD-angleB=40^circ求出angleD=110^circ,angleB=70^circ,根据圆周角定理得出angleAOC=2angleB,再代入求出答案即可.【详解】because四边形ABCD内接于odotO,thereforeangleB+angleD=180^circ,becauseangleD-angleB=40^circthereforeangleD=110^circ,angleB=70^circ,thereforeangleAOC=2angleB=140^circ,故选:D.【点睛】本题考查了圆周角定理及圆内接四边形的性质,解题的关键是熟记圆内接四边形的对角互补."} {"id": "3090", "image": [], "answer": "A", "solution": "null", "level": "九年级", "question": "(本题 3 分) (2021 浙江權州$\\cdot$九年级阶段练习) 已知点 $P$ 在半径为 8 的 $\\odot O$ 外, 则", "options": "A. $O P>8$B. $O P=8$C. $O P<8$D. $O P \\geq 8$", "subject": "度量几何学", "analysis": "A【解析】【分析】根据点P与odotO的位置关系即可确定OP的范围.【详解】解:because点P在圆O的外部,therefore点P到圆心O的距离大于8,故选:A.【点睛】本题主要考查点与圆的位置关系,关键是要牢记判断点与圆的位置关系的方法."} {"id": "3091", "image": ["4008.jpg"], "answer": "B", "solution": "null", "level": "九年级", "question": "(本题 3 分) (2020$\\cdot$浙江杭州$\\cdot$九年级期末) 如图, $A B$ 是 $\\odot O$ 的直径, 点 $C$, 点 $D$是半圆上两点, 连结 $A C, B D$ 相交于点 $P$, 连结 $A D, O D$. 已知 $O D \\perp A C$ 于点 $E$, $A B=2$. 下列结论: (1) $A D^{2}+A C^{2}=4$; (2) $\\angle D B C+\\angle A D O=90^{\\circ}$; (3)若 $A C=B D$, 则 $D E=O E$ ; (4)若点 $P$ 为 $B D$ 的中点, 则 $D E=2 O E$. 其中正确的是( )", "options": "A. (1)(2)(3)B. (2)(3)(4)C. (3)(4)D. (2)(4)", "subject": "度量几何学", "analysis": "B【解析】【分析】证明AC^2+BC^2=AB^2=4即可判断(1);根据ODperpAC,得到angleDAE+angleADO=90^circ,根据angleDAE=angleDBC,即可判断(2);推出triangleAOD是等边三角形,即可判断(3);利用全等三角形的性质证明DE=BC,再利用三角形的中位线定理证明BC=2OE即可判断(4).【详解】解:becauseAB是直径,thereforeangleADB=angleACB=90^circ,在RttriangleACB中,thereforeAC^2+BC^2=AB^2=4,由已知条件无法得到AD与BC之间的大小关系,故无法得到AD^2+AC^2与4的大小关系,故(1)错误;becauseODperpACthereforeangleAED=90^circ,thereforeangleDAE+angleADO=90^circ,becauseangleDAE=angleDBC,thereforeangleDBC+angleADO=90^circ,故(2)正确;becauseAEperpOE,thereforeAD=CD,becauseAC=BDthereforeAC=BD,thereforeAD=CD=BC,thereforeangleAOD=60^circ,becauseOA=ODthereforetriangleOAD是等边三角形,becauseAEperpODthereforeDE=OE,故(3)正确,becauseangleDEP=angleBCP=90^circ,DP=PB,angleDPE=angleBPC,thereforetrianglePDEcongtrianglePBC(AAS),thereforeDE=BCbecauseOE//BC,AO=OBthereforeAE=EC,thereforeBC=2OE,thereforeDE=2OE,故(4)正确.故选:B.【点睛】本题考查圆周角定理,垂径定理,全等三角形的判定和性质等知识,解题的关键是熟练掌握基本知识,属于中考常考题型"} {"id": "3107", "image": [], "answer": "C", "solution": "null", "level": "九年级", "question": "(本题 3 分) (2022$\\cdot$浙江湖州$\\cdot$九年级期末) 已知圆心角度数为 $60^{\\circ}$, 半径为 30 , 则这个圆心角所对的弧长为", "options": "A. $20 \\pi$B. $15 \\pi$C. $10 \\pi$D. $5 \\pi$", "subject": "度量几何学", "analysis": "C【解析】【分析】根据弧长公式求出答案即可.【详解】解:圆心角是60^circ,半径为30的扇形的弧长是frac60pitimes30180=10pi.故选:C【点睛】本题主要考查了求弧长,熟练掌握弧长公式l=fracnpir180(其中n,r分别为圆心角,半径)是解题的关键."} {"id": "3109", "image": ["4036.jpg"], "answer": "B", "solution": "null", "level": "九年级", "question": "(本题 3 分)(2022$\\cdot$浙江湖州$\\cdot$年级期中)如图, 四边形 $A B C D$ 内接于 $\\odot O, C D$ 是 $\\odot O$ 的直径, 若 $\\angle B D C=20^{\\circ}$, 则 $\\angle A$ 的度数是( )", "options": "A. $100^{\\circ}$B. $110^{\\circ}$C. $120^{\\circ}$D. $130^{\\circ}$", "subject": "度量几何学", "analysis": "B【解析】【分析】由直角三角形的性质可得angleC的度数,再由圆内接四边形的性质即可求得结果.【详解】becauseCD是odotO的直径thereforeangleDBC=90^circbecauseangleBDC=20thereforeangleC=90^circ-angleBDC=70^circbecause四边形ABCD内接于odotOthereforeangleA=180^circ-angleC=110^circ故选:B【点睛】本题考查了直径所对的圆周角是直角,圆内接四边形的性质,掌握这两个知识点是关键."} {"id": "3110", "image": ["4037.jpg", "4038.jpg"], "answer": "B", "solution": "null", "level": "九年级", "question": "(本题 3 分) (2022$\\cdot$浙江杭州$\\cdot$九年级开学考试) 如图, $\\odot O$ 为 $\\triangle A B C$ 的外接圆, $\\angle A$ $=45^{\\circ}, \\odot O$ 的半径为 2 , 则 $B C$ 的长为 $(\\quad)$", "options": "A. 2B. $2 \\sqrt{2}$C. 4D. $2 \\sqrt{3}$", "subject": "度量几何学", "analysis": "B【解析】【分析】由odotO是triangleABC的外接圆,angleA=45^circ,易得triangleOBC是等腰直角三角形,继而求得答案.【详解】解:如图,连接OB,OC,becauseodotO是triangleABC的外接圆,angleA=45^circ,thereforeangleBOC=2angleA=90^circ,becauseOB=OC=2,thereforetriangleOBC是等腰直角三角形,thereforeBC=sqrtOB^2+OC^2=sqrt4+4=sqrt8=2sqrt2.故选:B.【点睛】本题主要考查圆周角定理,勾股定理,掌握圆周角定理以及勾股定理是解决问题的关键."} {"id": "3111", "image": ["4039.jpg", "4040.jpg"], "answer": "C", "solution": "null", "level": "九年级", "question": "(本题 3 分) (2022$\\cdot$浙江杭州 $\\cdot$九年级期末) 如图, $B D$ 是 $\\odot O$ 的直径, $A, C$ 是圆上不与点 $B, D$ 重合的两个点, 若 $\\angle A B D=30^{\\circ}$, 则 $\\angle A C B$ 的度数为 ( )", "options": "A. $30^{\\circ}$B. $45^{\\circ}$C. $60^{\\circ}$D. $75^{\\circ}$", "subject": "度量几何学", "analysis": "C【解析】【分析】根据直径所对的圆周角等于直角,三角形内角和定理即可求得angleADB的度数,再利用在同圆或等圆中,同弧所对的圆周角相等。即可求出angleACB.【详解】解:连接AD,becauseBD是odotO的直径,thereforeangleBAD=90^circ,becauseangleABD=30^circthereforeangleADB=60^circ.thereforeangleACB=angleADB=60^circ.故选:D.【点睛】本题考查直径所对的圆周角等于直角,三角形内角和定理,在同圆或等圆中,同弧所对的圆周角相等。解题的关键求出angleADB=60^circ."} {"id": "3112", "image": ["4041.jpg", "4042.jpg"], "answer": "D", "solution": "null", "level": "九年级", "question": "(本题 3 分) (2022$\\cdot$浙江杭州$\\cdot$九年级期末) 如图, 在 $\\triangle A B C$ 中, $\\angle A C B=90^{\\circ}$, 点 $D$为边 $A B$ 的中点, 以点 $A$ 为圆心,线段 $A D$ 的长为半径画弧,与 $A C$ 边交于点 $E$; 以点 $B$ 为圆心, 线段 $B D$ 的长为半径画弧, 与 $B C$ 边交于点 $F$. 若 $B C=6, A C=8$, 则图中阴影部分的面积为 $(\\quad)$", "options": "A. $48-\\frac{25 \\pi}{2}$B. $48-\\frac{25 \\pi}{4}$C. $24-\\frac{25 \\pi}{2}$D. $24-\\frac{25 \\pi}{4}$", "subject": "度量几何学", "analysis": "D【解析】【分析】根据勾股定理得到AB=sqrtAC^2+BC^2=10,根据线段中点的定义得到AD=BD=5,根据扇形和三角形的面积公式即可得到结论.【详解】解:becauseangleACB=90^circ,BC=6,AC=8,thereforeAB=sqrtAC^2+BC^2=10,angleA+angleB=90^circ,because点D为边AB的中点,thereforeAD=BD=5,如图所示,扇形ADE面积与扇形BGD面积相等,therefore图中阴影部分的面积==frac12times6times8-frac90cdotpitimes5^2360=24-frac25pi4,故选:D.【点睛】本题考查了扇形面积的计算,三角形的面积公式,勾股定理,熟练掌握扇形的面积公式是解题的关键."} {"id": "3113", "image": [], "answer": "A", "solution": "null", "level": "九年级", "question": "(本题 3 分)(2021$\\cdot$浙江$\\cdot$宁波市镇海蛟川书院九年级期中)下面四组线段中, 成比例的是 $(\\quad)$", "options": "A. $a=1, b=2, c=2, d=4$B. $a=2, b=3, c=4, d=5$C. $a=4, b=6, c=8, d=10$D. $a=\\sqrt{2}, c=3, b=\\sqrt{3}, c=\\sqrt{3}$", "subject": "度量几何学", "analysis": "A【分析】如果其中两条线段的乘积等于另外两条线段的乘积,则四条线段叫成比例线段.对选项一一分析,排除错误答案.【详解】解:A、1times4=2times2,故选项符合题意;B、2times5neq3times4,故选项不符合题意;C、4times10neq6times8,故选项不符合题意;C、sqrt2times3neqsqrt3timessqrt3,故选项不符合题意;故选:A.【点睛】此题考查了比例线段,根据成比例线段的概念,注意在相乘的时候,最小的和最大的相乘,另外两个相乘,看它们的积是否相等,同时注意单位要统一."} {"id": "3131", "image": ["4080.jpg", "4081.jpg", "4082.jpg", "4083.jpg", "4084.jpg"], "answer": "D", "solution": "null", "level": "九年级", "question": "(本题 3 分) (2021$\\cdot$浙江得州$\\cdot$九年级阶段练习) 如图, 在 Rt $\\triangle A B C$ 中, $\\angle C=90^{\\circ}$, $\\angle A=30^{\\circ}, B C=3$, 按图中虚线剪下的三角形与 $\\triangle A B C$ 不相似的是 ( )", "options": "A.B.C.D.", "subject": "度量几何学", "analysis": "D【分析】由相似三角形的判定依次判断可求解.【详解】解:A、由两角对应相等,两三角形相似,可证图中虚线剪下的三角形与triangleABC相似,故选项A不符合题意;B、由两组对边对应成比例且夹角相等,两三角形相似,可证图中虚线剪下的三角形与triangleABC相似,故选项B不符合题意;C、由两角对应相等,两三角形相似,可证图中虚线剪下的三角形与triangleABC相似,故选项C不符合题意;D、无法证明图中虚线剪下的三角形与triangleABC相似,故选项D符合题意;故选:D.【点睛】本题考查了相似三角形的判定,直角三角形的性质,掌握相似三角形的判定方法是本题的关键."} {"id": "3132", "image": ["4085.jpg"], "answer": "D", "solution": "null", "level": "九年级", "question": "(本题 3 分) (2020:浙江杭州 九九年级期末) $\\triangle A B C$ 中, $P$ 为 $A B$ 上的一点. 下列四个条件: (1) $\\angle A C P=\\angle B$ : (2) $\\angle A P C=\\angle A C B$ : (3) $\\frac{A C}{A B}=\\frac{A P}{A C}$; (4) $\\frac{A B}{A C}=\\frac{B C}{C P}$ 等, 其中能判断 $\\triangle A P C \\sim \\triangle A C B$ 的有 $(\\quad)$", "options": "A. (1)(2)(4)B. (1)(3)(4)C. (2)(3)(4)D. (1)(2)(3)", "subject": "度量几何学", "analysis": "D【分析】由相似三角形的判定依次判断可求解.【详解】解:(1)angleACP=angleB,angleA=angleA,可证triangleAPCsimtriangleACB,故(1)符合题意;(2)angleAPC=angleACB,angleA=angleA,可证triangleAPCsimtriangleACB,故(2)符合题意;(3)fracACAB=fracAPAC,angleA=angleA,可证triangleAPCbacksimtriangleACB,故(3)符合题意;(4)fracABAC=fracBCCP,angleA=angleA,不能证明triangleAPCbacksimtriangleACB,故(4)不符合题意;故选:D.【点睛】本题考查了相似三角形的判定,灵活运用相似三角形的判定是本题的关键."} {"id": "3134", "image": ["4088.jpg"], "answer": "D", "solution": "null", "level": "九年级", "question": "(本题 3 分) (2022$\\cdot$浙江彊州 $\\cdot$九年级期末) 如图, 在 Rt $\\triangle A B C$ 中, $\\angle B A C=90^{\\circ}$, 中线 $A D, B E$ 相交于点 $F . E G / / B C$, 交 $A D$ 于点 $G . G F=1$, 则 $B C$ 的长为 $(\\quad)$", "options": "A. 5B. 6C. 10D. 12", "subject": "度量几何学", "analysis": "D【分析】首先根据GE//CD得到triangleAGFsimtriangleADC、triangleFEGsimtriangleFBD,求出AD=6,然后利用直角三角形斜边的中线性质得出结果.【详解】解:becauseGE//CD,thereforetriangleAGEsimtriangleADC,triangleFEGsimtriangleFBDthereforefracAGAD=fracGECD=fracAEAC=frac12thereforefracGEBD=fracGFDF又becauseBD=CD,thereforefracGFDF=frac12,thereforeDF=2GF=2,thereforeDG=DF+GF=3thereforeAD=2DG=6,在直角triangleABC中,angleBAC=90^circ,thereforeBC=2AD=12,故选D.【点睛】本题考查相似三角形的性质与判定以及直角三角形的性质,根据平行得到相似三角形是解决问题的关键."} {"id": "3135", "image": ["4089.jpg", "4090.jpg"], "answer": "C", "solution": "null", "level": "九年级", "question": "(本题 3 分) (2022$\\cdot$浙江温州$\\cdot$模拟预测)如图, 在正方形 $A B C D$ 中, 延长 $D C$ 至点 $G$,以 $C G$ 为边向下画正方形 $C E F G$. 延长 $A B$ 交边 $F G$ 于点 $H$, 连结 $C F, A F$ 分别交 $A H, C E$ 于点 $M, N$. 收录在清朝四库全书的《几何通解》利用此图得: $2 A B^{2}+2 B H^{2}=A H^{2}+M H^{2}$. 若正方形 $A B C D$ 与 $C E F G$ 的面积之和为 $68, C N=3 N E$, 则 $A H$ 的长为 $(\\quad)$", "options": "A. $4 \\sqrt{2}$B. 8C. $8 \\sqrt{2}$D. 16", "subject": "度量几何学", "analysis": "C【分析】根据勾股定理验证清朝四库全书的《几何通解》中的结论,设正方形ABCD与CEFG的边长分别为a,b,根据图形可得FH=MH=BH-BM=b-a(1),证明triangleEFNsimtriangleBAN,可得BN=frac14a,证明triangleABNsimtriangleAFH,可得FH=fracBNcdotAHAB=fracfrac14acdot(a+b)a(2),从而求得3b=5a,代入a^2+b^2=68,解一元二次方程,继而求得a,b的值,根据AH=a+b求解即可【详解】解:如图,连接AC,because四边形ABCD,CEFG是正方形,AC,CF是对角线thereforeangleACB=angleECF=angleCFG=45^circ,AC^2=2AB^2,CF^2=2CG^2thereforetriangleAFC是Rt_trianglethereforeAF^2=AC^2+CF^2=2AB^2+2CG^2becauseAH//DGthereforeBHperpFGbecauseCEFG是正方形,thereforeangleG=angleBCE=90^circtherefore四边形BHGC是矩形thereforeBH=CGAF^2=2AB^2+2BH^2becauseangleCFG=45^circ,AHperpFGthereforeMH=FH在RttriangleAFH中,AF^2=FH^2+AH^2thereforeAF^2=MH^2+AH^2therefore2AB^2+2BH^2=AH^2+MH^2设正方形ABCD与CEFG的边长分别为a,bbecause正方形ABCD与CEFG的面积之和为68,CN=3NE,thereforea^2+b^2=68,quadCN=frac34b,EN=frac14btherefore2a^2+2b^2=AH^2+MH^2=136becauseAHperpBC,angleBCF=45^circthereforeFH=MH=BH-BM=b-abecauseEF//ABthereforetriangleEFNsimtriangleBANthereforefracENBN=fracEFAB即frac14bthereforeBN=frac14abecauseBN//FHthereforetriangleABNbacksimtriangleAFHthereforefracBNFH=fracABAHthereforeFH=fracBNcdotAHAB=fracfrac14acdot(a+b)a(2)由(1)(2)可得b-a=fracfrac14b(a+b)a整理得5a^2=3abbecauseaneq0therefore3b=5abecausea^2+b^2=68thereforeleft(frac35bright)^2+b^2=68解得b=5sqrt2(负值舍去)thereforea=3sqrt3thereforeAH=a+b=8sqrt2故选C【点睛】本题考查了正方形的性质,相似三角形的性质与判定,勾股定理,解一元二次方程,通过设参数的方法建立方程求解是解题的关键."} {"id": "3136", "image": [], "answer": "C", "solution": "null", "level": "九年级", "question": "(本题 3 分)(2022 : 浙江$\\cdot$九年级专题练习) 已知线段 $a$ 是线段 $b, c$ 的比例中项, $b=4 \\mathrm{~cm}$, $c=9 \\mathrm{~cm}$, 则 $a$ 为 () $\\mathrm{cm}$.", "options": "A. 36B. -36C. 6D. -6", "subject": "度量几何学", "analysis": "C【详解】根据题意可得a^2=bc,代入数值,解答出即可,注意线段为正值.【解答】解:由题意得,a^2=bcbecauseb=4~cm,quadc=9~cmthereforea^2=bcthereforea_1=6,a_2=-6(舍)thereforea=6.故选:C.【点睛】本题主要考查了比例线段,解题的关键是理解比例中项的定义."} {"id": "3137", "image": ["4091.jpg", "4092.jpg"], "answer": "D", "solution": "null", "level": "九年级", "question": "(本题 3 分)(2022$\\cdot$浙江$\\cdot$慈溪育才中学九年级阶段练习)如图, 在 Rt $\\triangle A B C$ 中, $\\angle A C B=90^{\\circ}, A C=9, B C=12$, 点 $E$ 是 $A B$ 中点, $P$ 是直线 $C E$ 上一动点, 连接 $A P$, 以 $A P$ 为斜边在其左侧作 Rt $\\triangle A P M$, 使 $\\angle A P M=\\angle B$, 连接 $C M$, 则 $C M$ 的最小值为 ( )", "options": "A. $\\frac{9}{2}$B. $\\frac{36}{5}$C. $\\frac{27}{5}$D. $\\frac{108}{25}$", "subject": "度量几何学", "analysis": "D【分析】连接BP.由题意易证triangleAPMsimtriangleABC,即得出angleMAP=angleCAB,fracAMAC=fracAPAB,从而得出angleMAC=anglePAB,即又易证triangleMACsimtrianglePAB,得出fracCMBP=fracACAB.再根据勾股定理可求出AB=15,从而得出fracCMBP=frac915,即说明当BP最小时,CM最小.又根据当BPperpCE时,BP最小,结合三角形相似的判定和性质求出此时BP的值,即如图BP^prime的值,进而即可求出CM的最小值.【详解】如图,连接BP.becauseangleAPM=angleABC,quadangleAMP=angleACB=90^circ,thereforetriangleAPMsimtriangleABCthereforeangleMAP=angleCAB,fracAMAC=fracAPAB,thereforeangleMAP-angleCAP=angleCAB-angleCAP,即angleMAC=anglePAB,thereforetriangleMACsimtrianglePABthereforefracCMBP=fracACAB.becauseangleACB=90^circ,AC=9,BC=12thereforeAB=sqrtAC^2+BC^2=15,thereforefracCMBP=frac915,therefore当BP最小时,CM最小.because点P是直线CE上一动点,therefore当BPperpCE时,BP最小,如图BP^prime即为最小时,此时所作的三角形为Rt_triangleAP^primeM^prime.because点E是AB中点,angleACB=90^circ,thereforeCE=BE,thereforeangleP^primeCB=angleCBA又becauseangleBP^primeC=angleACB=90^circ,thereforetriangleP^primeCB^simsimtriangleCBAthereforefracB^primePAC=fracBCAB,即fracB^primeP9=frac1215,解得:B^primeP=frac365,即BP的最小值为frac365,thereforefracCMBP=fracCMfrac365=frac915,解得:CM=frac10825.故选D.【点睛】本题考查三角形相似的判定和性质,直角三角形斜边中线的性质,勾股定理等知识,较难.证明出当BPperpCE时,BP最小,此时CM最小是解题关键."} {"id": "3147", "image": ["4101.jpg"], "answer": "C", "solution": "null", "level": "九年级", "question": "(本题 3 分) (2021 浙江: 温州市实验中学九年级期中) 如图, 直线 $a / / b / / c$, 直线 $A C$分别交 $a, b, c$ 于点 $A, B, C$, 直线 $D F$ 分别交 $a, b, c$ 于点 $D, E, F$. 若 $D E=2 E F$, $A C=6$, 则 $A B$ 的长为", "options": "2B. 3C. 4D. 5", "subject": "度量几何学", "analysis": "C【分析】根据平行线分线段成比例定理列出比例式,把已知数据代入计算即可.【详解】解:becausea//b//c,thereforefracDEEF=fracABBC,becauseDE=2EF,AC=6,thereforefracAB6-AB=2,解得:AB=4,故选:C.【点睛】本题考查的是平行线分线段成比例定理,灵活运用定理、找准对应关系是解题的关键."} {"id": "3152", "image": ["4118.jpg"], "answer": "D", "solution": "null", "level": "九年级", "question": "(本题 3 分) (2021 浙江$\\cdot$温州市第十二中学九年级期中) 如图, $\\triangle A B C$ 中, $D$ 是 $A B$边上一点, 添加下列条件, 不能判定 $\\triangle A B C \\sim \\triangle A C D$ 的是 ( )", "options": "A. $\\angle A C D=\\angle B$B. $\\angle A D C=\\angle A C B$C. $\\frac{A D}{A C}=\\frac{A C}{A B}$D. $\\frac{A D}{A C}=\\frac{C D}{B C}$", "subject": "度量几何学", "analysis": "D【分析】根据三角形相似的判定定理逐一分析判断即可.【详解】解:A、becauseangleACD=angleB,angleDAC=angleCABthereforetriangleABCsimtriangleACD所以选项A不符合题意;B、becauseangleADC=angleACB,angleDAC=angleCABthereforetriangleABCsimtriangleACD所以选项B不符合题意;C、becausefracADAC=fracACAB,quadangleDAC=angleCABthereforetriangleABCsimtriangleACD所以选项C不符合题意;D、fracADAC=fracCDBC,对应边成比例,但是angleADC不确定是否与angleACB相等,所以不能判定triangleABCsimtriangleACD,所以选项D符合题意.故选:D【点睛】本题考查三角形相似的判定定理,牢记定理的内容是解题的重点."} {"id": "3156", "image": ["4121.jpg", "4122.jpg"], "answer": "A", "solution": "null", "level": "九年级", "question": "(本题 3 分) (2022: 浙江省义乌市稠江中学九年级阶段练习) 如图, 在 $\\triangle A B C$ 中, $A H \\perp B C$于 $H, B C=12, A H=8, D 、 E$ 分别为 $A B 、 A C$ 上的点, $G 、 F$ 是 $B C$ 上的两点, 四边形 $D E F G$ 是正方形, 正方形的边长 $D E$ 为 $(\\quad)$", "options": "A. 4.8B. 4C. 6.4D. 6", "subject": "度量几何学", "analysis": "A【分析】利用相似三角形对应高的比也等于相似比,可以求出x,注意所画图形是正方形,用同一未知数表示未知边,即可求出.【详解】解:设triangleABC的高AH交DE于点M,正方形的边长为x.由正方形DEFG得,DE//FG,即DE//BC,becauseAHperpBCthereforeAMperpDE.由DE//BC得triangleADEsimtriangleABC,thereforefracDEBC=fracAMAH,把BC=12,AH=8,DE=x,AM=8-x代入上式得:fracx12=frac8-x8,解得:x=4.8.答:正方形的边长是4.8.故选:A.【点睛】本题考查了相似三角形的判定与性质.关键是由平行线得到相似三角形,利用相似三角形的性质列方程."} {"id": "3159", "image": [], "answer": "A", "solution": "null", "level": "九年级", "question": "(本题 3 分)(2022$\\cdot$浙江省义乌市廿三里初级中学九年级期中)下列各组线段中,成比例的是 $(\\quad)$", "options": "$1,2,2,4$B. $1,2,3,4$C. $3,5,9,13$D. $1,2,2,3$", "subject": "度量几何学", "analysis": "A【分析】如果其中两条线段的乘积等于另外两条线段的乘积,则四条线段叫成比例线段,对选项一一分析,排除错误答案.【详解】 解: A、 $1 \\times 4=2 \\times 2$, 故选项符合题意;B、 $1 \\times 4 \\neq 2 \\times 3$, 故选项不符合题意;C、 $3 \\times 13 \\neq 5 \\times 9$, 故选项不符合题意;D、 $1 \\times 3 \\neq 2 \\times 2$, 故选项不符合题意.故选: A.【点睛】此题考查了比例线段,根据成比例线段的概念,注意在相乘的时候,最小的和最大的相乘,另外两个相乘,看它们的积是否相等."} {"id": "3170", "image": [], "answer": "D", "solution": "null", "level": "九年级", "question": "(本题 3 分) (2022$\\cdot$浙江宁波$\\cdot$九年级期末)下列与相似有关的命题中,正确的是 ( )(1)所有的等腰三角形都相似;(2)所有的矩形都相似;(3)所有的正六边形都相似.", "options": "(1)(2)(3)B. (1)C. (2)D. (3)", "subject": "度量几何学", "analysis": "D【分析】利用相似图形的定义分别判断后即可确定正确的选项.【详解】解:(1)所有的等腰三角形都不一定相似,故原说法错误,不符合题意;(2)所有的矩形的对应角相等, 但对应边的比不一定相等, 不都相似, 故原命题错误, 不符合题意;(3)所有的正六边形都相似,正确,符合题意,故选: D.【点睛】考查了命题与定理的知识, 解题的关键是了解相似图形的定义."} {"id": "3180", "image": ["4153.jpg"], "answer": "A", "solution": "null", "level": "九年级", "question": "(本题 3 分) (2021$\\cdot$浙江$\\cdot$温州市实验中学九年级期中) 如图, 点 $C$ 为线段 $A B$ 的中点,在 $A C$ 上取点 $D$, 分别以 $A D, C D, B C, B D$ 为边向上作正方形 $A D G H, C D K L, B C I J$, $D B E F$, 将其面积依次记为 $S_{1}, S_{2}, S_{3}, S_{4}$, 在《几何原本》有这样一个结论; $S_{1}+S_{4}=2\\left(S_{2}+S_{3}\\right)$. 当 $A B=2$ 时, 若 $A, K, J$ 共线, 则图中阴影部分的面积为 ( )", "options": "A. $\\frac{10}{9}$B. $\\frac{11}{10}$C. $\\frac{2 \\sqrt{3}}{3}$D. $\\frac{3 \\sqrt{2}}{4}$", "subject": "度量几何学", "analysis": "A【分析】根据正方形的性质证明 $\\triangle A D K \\sim \\triangle A B J$, 设 $C D=x$, 则 $A D=1-x, K D=C D=x$, 所以 $\\frac{x}{1}=\\frac{1-x}{2}$, 解得 $x=\\frac{1}{3}$, 再根据图形可得 $S_{\\text {阴影 }}+S_{2}+S_{3}=S_{1}+S_{4}=2\\left(S_{2}+S_{3}\\right)$, 所以 $S_{\\text {阴影 }}=S_{2}+S_{3}=C D^{2}+B C^{2}$, 进而可得结果.【详解】解:根据题意可知:DK// $B J$,$\\therefore \\triangle A D K \\sim \\triangle A B J$,$\\therefore \\frac{D K}{B J}=\\frac{A D}{A B}$$\\because$ 点 $C$ 为线段 $A B$ 的中点, $A B=2$,$\\therefore A C=B C=B J=1$,设 $C D=x$,则 $A D=1-x, K D=C D=x$,$\\therefore \\frac{x}{1}=\\frac{1-x}{2}$,解得 $x=\\frac{1}{3}$,$\\therefore A D=1-x=\\frac{2}{3}, C D=D K=\\frac{1}{3}$,$\\because S_{\\text {阴影 }}+S_{2}+S_{3}=S_{1}+S_{4}=2\\left(S_{2}+S_{3}\\right)$,$\\therefore S_{\\text {阴影 }}=S_{2}+S_{3}=C D^{2}+B C^{2}$$=\\left(\\frac{1}{3}\\right)^{2}+1^{2}$$=\\frac{1}{9}+1$$=\\frac{10}{9}$.故选: A.【点睛】本题考查了相似三角形的判定与性质, 正方形的性质, 解决本题的关键是掌握相似三角形面积比等于相似比的平方."} {"id": "3181", "image": ["4154.jpg"], "answer": "B", "solution": "null", "level": "九年级", "question": "(本题 3 分)(2022$\\cdot$浙江金华$\\cdot$九年级期末)如图,在 $\\triangle A B C$ 中, $C H \\perp A B, C H=5$, $A B=10$, 若内接矩形 $D E F G$ 邻边 $D G: G F=1: 2$, 则 $\\triangle G F C$ 与四边形边形 $A B F G$ 的面积比为 $(\\quad)$", "options": "A. $\\frac{1}{3}$B. $\\frac{1}{4}$C. $\\frac{1}{2}$D. $\\frac{\\sqrt{2}}{2}$", "subject": "度量几何学", "analysis": "B【分析】设 $G D=x$, 则 $G F=2 x$. 根据题意还可求出 $I H=x, C I=5-x$. 根据 $G F / / A B$,可推出 $\\triangle C G F \\sim \\triangle C A B$, 即得出 $\\frac{C I}{C H}=\\frac{G F}{A B}$, 代入数值, 即可求出 $x$ 的值, 即可得出 $G F$和 $C I$ 的值, 最后根据三角形面积公式求出 $S_{\\triangle C G F}$ 和 $S_{\\triangle A B C}$, 作比即可.【详解】解:设 $G D=x$, 则 $G F=2 x$.$\\because$ 四边形 $D E F G$ 为 $\\triangle A B C$ 的内接矩形, $C H \\perp A B$,$\\therefore I H=x$,$\\therefore C I=C H-I H=5-x$.$\\because G F / / D E$ ,即 $G F / / A B$ ,$\\therefore \\triangle C G F \\sim \\triangle C A B$$\\therefore \\frac{C I}{C H}=\\frac{G F}{A B}$, 即 $\\frac{5-x}{5}=\\frac{2 x}{10}$,解得 $x=\\frac{5}{2}$.$\\therefore G F=2 \\times \\frac{5}{2}=5, C I=5-\\frac{5}{2}=\\frac{5}{2}$$\\therefore S_{\\triangle C G F}=\\frac{1}{2} G F \\cdot C I=\\frac{1}{2} \\times 5 \\times \\frac{5}{2}=\\frac{25}{4}$.$\\because S_{\\triangle A B C}=\\frac{1}{2} A B \\cdot C H=\\frac{1}{2} \\times 10 \\times 5=25$,$\\therefore \\frac{S_{\\triangle C G F}}{S_{\\triangle A B C}}=\\frac{\\frac{25}{4}}{25}=\\frac{1}{4}$.故选 B【点睛】本题考查矩形的性质, 三角形相似的判定和性质. 利用数形结合的思想是解答本题的关键."} {"id": "3200", "image": ["4183.jpg"], "answer": "C", "solution": "null", "level": "九年级", "question": "(本题 3 分) (2022$\\cdot$浙江绍兴$\\cdot$二模)如图, 如果 $\\angle B A D=\\angle C A E$, 那么添加下列一个条件后,仍不能确定 $\\triangle A D E$ 与 $\\triangle A B C$ 相似的是( )", "options": "A. $B=\\angle D$B. $\\angle C=\\angle A E D$C. $\\frac{A B}{A D}=\\frac{D E}{B C}$D. $\\frac{A B}{A D}=\\frac{A C}{A E}$", "subject": "度量几何学", "analysis": "C【分析】triangleADEcongtriangleABC根据题意可得angleEAD=angleCAB,然后根据相似三角形的判定定理逐项判断,即可求解.【详解】解:becauseangleBAD=angleCAE,thereforeangleEAD=angleCABA.若添加angleB=angleD,可用两角对应相等的两个三角形相似,证明triangleADEcongtriangleABC,故本选项不符合题意;B.若添加angleC=angleAED,可用两角对应相等的两个三角形相似,证明triangleADEcongtriangleABC,故本选项不符合题意;C.若添加fracABAD=fracDEBC,不能证明triangleADEcongtriangleABC,故本选项符合题意;D.若添加fracABAD=fracACAE,可用两边对应成比例,且夹角相等的两个三角形相似,证明triangleADEcongtriangleABC,故本选项不符合题意;故选:C.【点睛】本题主要考查了相似三角形的判定,熟练掌握相似三角形的判定定理是解题的关键."} {"id": "3202", "image": ["4185.jpg", "4186.jpg"], "answer": "D", "solution": "null", "level": "九年级", "question": "(本题 3 分)(2022$\\cdot$浙江宁波$\\cdot$九年级期末)如图,将 5 个全等的等腰三角形拼成内外两个大小不同的正五边形图案, 设小正五边形边长为 1 , 则大正五边形边长为", "options": "A. $\\frac{\\sqrt{5}-1}{2}$B. $\\frac{\\sqrt{5}+1}{2}$C. $\\frac{3-\\sqrt{5}}{2}$D. $\\frac{3+\\sqrt{5}}{2}$", "subject": "度量几何学", "analysis": "D【分析】根据多边形的内角和定理得到angleABE=frac(5-2)times180^circ5=108^circ,等量代换得到angleCBE+angleABC=angleBAC+angleABC=108^circ,如图,作angleACB的平分线CD交AB于D,根据相似三角形的性质即可得到结论.【详解】在正五边形ABEFG中,angleABE=frac(5-2)times180^circ5=108^circ,because将5个全等的等腰三角形拼成内外两个大小不同的正五边形图案,thereforeangleCBE+angleABC=angleBAC+angleABC=108^circ,如图,作angleACB的平分线CD交AB于D,becauseAB=AC,quadthereforeangleABC=angleACBthereforeangleACB+angleBAC+angleABC=108^circ+angleACB=180^circthereforeangleABC=angleACB=72^circ,quadthereforeangleBAC=36^circ,angleACD=angleBCD=angleBAC=36^circ,thereforeangleBCD=angleBAC,quadAD=CD=BCthereforetriangleABCsimtriangleCBD,quadthereforefracBCAB=fracBDBC,becauseAB=BC+1,quadthereforeBD=AB-AD=AB-BC=1,thereforefracBCBC+1=frac1BC,quadthereforeBC=frac1+sqrt52,thereforeAB=BC+1=frac3+sqrt52,故选:D.【点睛】本题考查了正多边形的性质,等腰三角形的性质,相似三角形的性质,熟练掌握相似三角形的性质是解题的关键."} {"id": "3348", "image": ["4397.jpg"], "answer": "D", "solution": "null", "level": "九年级", "question": "如图, 在 $R t \\triangle A B C$ 中, $\\angle A C B=90^{\\circ}, B C=1, A B=2$, 则下列结论正确的是 $($ )", "options": "A. $\\sin A=\\frac{\\sqrt{3}}{2}$\n B. $\\tan A=\\frac{1}{2}$\n C. $\\cos B=\\frac{\\sqrt{3}}{2}$\n D. $\\tan B=\\sqrt{3}$", "subject": "度量几何学", "analysis": "【分析】\n\n本题主要考查了锐角三角函数的定义,正确记忆相关比例关系是解题关键.\n\n先根据勾股定理求出 $A C=\\sqrt{3}$, 再根据三角函数的定义分别求解可得.\n\n【解答】\n\n解: $\\because$ 在Rt $\\triangle A B C$ 中, $\\angle A C B=90^{\\circ}, B C=1, A B=2$,\n\n$\\therefore A C=\\sqrt{3}$,\n\nA、 $\\sin A=\\frac{B C}{A B}=\\frac{1}{2}$, 故该选项不符合题意;\n\nB、 $\\tan A=\\frac{B C}{A C}=\\frac{1}{\\sqrt{3}}=\\frac{\\sqrt{3}}{3}$, 故该选项不符合题意;\n\nC、 $\\cos B=\\frac{B C}{A B}=\\frac{1}{2}$, 故该选项不符合题意;\n\n$D 、 \\tan B=\\frac{A C}{B C}=\\sqrt{3}$, 故该选项符合题意;\n\n故选: $D$."} {"id": "3349", "image": ["4398.jpg"], "answer": "C", "solution": "null", "level": "九年级", "question": "如图, 在 $\\triangle A B C$ 中, $\\angle B=45^{\\circ}, \\angle C=60^{\\circ}, A D \\perp B C$ 于点 $D, B D=\\sqrt{3}$. 若 $E, F$ 分别为 $A B$, $B C$ 的中点, 则 $E F$ 的长为 $(\\quad)$", "options": "A. $\\frac{\\sqrt{3}}{3}$\n B. $\\frac{\\sqrt{3}}{2}$\n C. 1\n D. $\\frac{\\sqrt{6}}{2}$", "subject": "度量几何学", "analysis": "解: $\\because A D \\perp B C$,\n\n$\\therefore \\angle A D B=\\angle A D C=90^{\\circ}$,\n\n$\\because \\angle B=45^{\\circ}, B D=\\sqrt{3}$,\n\n$\\therefore A D=B D=\\sqrt{3}$,\n\n$\\because \\angle C=60^{\\circ}$,\n\n$\\therefore D C=\\frac{A D}{\\tan 60^{\\circ}}=\\frac{\\sqrt{3}}{\\sqrt{3}}=1$,\n\n$\\therefore A C=2$,\n\n$\\because E, F$ 分别为 $A B, B C$ 的中点,\n\n$\\therefore E F=\\frac{1}{2} A C=1$.\n\n故选: $C$.\n\n由等腰直角三角形的性质求出 $A D=B D=\\sqrt{3}$, 由锐角三角函数的定义求出 $D C=1$, 由三角形的中位线定理可求出答案.\n\n本题考查了等腰直角三角形的性质, 三角形中位线定理, 锐角三角函数, 熟练掌握三角形的中位线定理是解题的关键."} {"id": "3350", "image": ["4399.jpg", "4400.jpg"], "answer": "D", "solution": "null", "level": "九年级", "question": "如图, 一块矩形木板 $A B C D$ 斜靠在墙边 $(O C \\perp O B$, 点 $A, B, C, D, O$ 在同一平面内),已知 $A B=a, A D=b, \\angle B C O=\\alpha$, 则点 $A$ 到 $O C$ 的距离等于 $(\\quad)$", "options": "A. $a \\cdot \\sin \\alpha+b \\cdot \\sin \\alpha$\n B. $a \\cdot \\cos \\alpha+b \\cdot \\cos \\alpha$\n C. $a \\cdot \\sin \\alpha+b \\cdot \\cos \\alpha$\n D. $a \\cdot \\cos \\alpha+b \\cdot \\sin \\alpha$", "subject": "度量几何学", "analysis": "【分析】\n\n作 $A E \\perp O B$ 交 $O B$ 的延长线于点 $E$, 在直角三角形 $A B E$ 和直角三角形 $B O C$ 中解直角三角形可求出点 $A$到 $O C$ 的距离.\n\n此题主要考查解直角三角形的应用, 解题的关键是作辅助线将点 $A$ 到 $O C$ 的距离转化为线段 $O E$ 的长.\n\n【解答】\n\n解: 如图, 作 $A E \\perp O B$ 交 $O B$ 的延长线于点 $E$,\n\n$\\because O C \\perp O B$,\n\n$\\therefore \\angle A E B=\\angle B O C=90^{\\circ}$,\n\n$\\because$ 四边形 $A B C D$ 是矩形,\n\n$\\therefore B C=A D=b, \\angle A B C=90^{\\circ}$,\n\n$\\therefore \\angle A B E=90^{\\circ}-\\angle O B C=\\angle B C O=\\alpha$,\n\n\n$\\because \\frac{B E}{A B}=\\cos \\angle A B E=\\cos \\alpha$,\n\n$\\therefore B E=A B \\cdot \\cos \\alpha=a \\cdot \\cos \\alpha$,\n\n$\\because \\frac{O B}{B C}=\\sin \\angle B C O=\\sin \\alpha$,\n\n$\\therefore O B=B C \\cdot \\sin \\alpha=b \\cdot \\sin \\alpha$,\n\n$\\therefore O E=B E+O B=a \\cdot \\cos \\alpha+b \\cdot \\sin \\alpha$,\n\n$\\because A E / / O C$,\n\n$\\therefore$ 点 $A 、$ 点 $E$ 到 $O C$ 的距离相等,\n\n$\\therefore$ 点 $A$ 到 $O C$ 的距离等于 $a \\cdot \\cos \\alpha+b \\cdot \\sin \\alpha$,\n\n故选: $D$."} {"id": "3351", "image": ["4401.jpg", "4402.jpg"], "answer": "B", "solution": "null", "level": "九年级", "question": "如图, 某社会实践活动小组实地测量两岸互相平行的一段河的宽度, 在河的南岸点 $A$ 处,测得河的北岸边点 $B$ 在其北偏东 $45 \\div \\circ$ 方向然后向西走 80 米到达 $C$ 点, 测得点 $B$ 在点 $C$ 的北偏东 $60 \\div{ }^{\\circ}$ 方向, 则这段河的宽度为 $(\\quad)$", "options": "A. $80(\\sqrt{3}+1)$ 米\n B. $40(\\sqrt{3}+1)$ 米\n C. $(120-40 \\sqrt{3})$ 米\n D. $40(\\sqrt{3}-1)$ 米", "subject": "度量几何学", "analysis": "【分析】\n\n本题考查的是解直角三角形的应用一方向角问题.\n\n过点 $B$ 作 $B D \\perp C A$ 交 $C A$ 的延长线于 $D$, 设 $B D=x$ 米, 根据正切的定义用 $x$ 表示出 $C D 、 A D$, 根据题意列出方程, 解方程即可.\n\n【解答】\n\n解:过点 $B$ 作 $B D \\perp C A$ 交 $C A$ 的延长线于 $D$,\n\n\n\n设 $B D=x$ 米,\n\n$\\because \\angle B C A=30^{\\circ}$,\n\n$\\therefore C D=\\frac{B D}{\\tan 30^{\\circ}}=\\sqrt{3} x$ 米,\n\n$\\because \\angle B A D=45^{\\circ}$,\n\n$\\therefore A D=B D=x$ 米,\n\n$\\because A C=C D-A D=80$ 米,\n\n$\\therefore \\sqrt{3} x-x=80$,\n解得 $x=\\frac{80}{\\sqrt{3}-1}=40(\\sqrt{3}+1)$ 米,\n\n答: 这段河的宽度为 $40(\\sqrt{3}+1)$ 米."} {"id": "3368", "image": ["4424.jpg", "4425.jpg"], "answer": "D", "solution": "null", "level": "九年级", "question": "如图, $\\triangle A B C$ 的三个顶点分别在正方形网格的格点上, 下列三角函数值错误的是 ( )", "options": "A. $\\sin B=\\frac{3}{5}$\n B. $\\cos B=\\frac{4}{5}$\n C. $\\tan B=\\frac{3}{4}$\n D. $\\tan A=\\frac{4}{3}$", "subject": "度量几何学", "analysis": "【分析】\n\n本题考查了勾股定理, 锐角三角函数的定义及其增减性, 计算时要将锐角置于直角三角形中并要充分利用格点. 根据锐角三角函数的定义, 将 $\\angle B$ 放在直角三角形 $A B D$ 中, 分别计算 $\\sin B, \\cos B$, $\\tan B$, 可判断 $A 、 B 、 C$, 再根据锐角正切函数的增减性可判断 $D$.\n\n【解答】\n\n解: 如图, 过点 $A$ 作 $A D \\perp B C$ 于 $D$.\n\n\n\n在 $\\triangle A B D$ 中, $\\because \\angle A D B=90^{\\circ}, A D=3, B D=4$,\n\n$\\therefore A B=5$,\n\n$\\therefore \\sin B=\\frac{A D}{A B}=\\frac{3}{5}$, 故 $A$ 正确, 不符合题意;\n\n$\\cos B=\\frac{B D}{A B}=\\frac{4}{5}$, 故 $B$ 正确, 不符合题意;\n\n$\\tan B=\\frac{A D}{B D}=\\frac{3}{4}$, 故 $C$ 正确, 不符合题意;\n\n$\\because \\tan \\angle B A D=\\frac{B D}{A D}=\\frac{4}{3}, \\angle A<\\angle B A D$,\n\n$\\therefore \\tan A<\\frac{4}{3}$, 故 $D$ 错误, 符合题意.\n\n故选 $D$."} {"id": "3371", "image": ["4427.jpg", "4428.jpg"], "answer": "D", "solution": "null", "level": "九年级", "question": "如图, 若锐角 $\\triangle A B C$ 内接于 $\\odot O$, 点 $D$ 在 $\\odot O$ 外(与点 $C$ 在 $A B$ 同侧), 则下列三个结论: (1) $\\sin \\angle C>\\sin \\angle D$; (2) $\\cos \\angle C>\\cos \\angle D$; (3) $\\tan \\angle C>\\tan \\angle D$ 中, 正确的结论为( )", "options": "A. (1) (2);\n B. (2)(3);\n C. (1)(2) (3);\n D. (1) (3);", "subject": "度量几何学", "analysis": "【分析】本题考查了圆周角定理、锐角三角函数的定义、锐角三角形函数的增减性, 解决本题的关键是比较出 $\\angle C>\\angle D$.\n\n连接 $B E$, 根据圆周角定理, 可得 $\\angle C=\\angle A E B$, 因为 $\\angle A E B=\\angle D+\\angle D B E$, 所以 $\\angle A E B>\\angle D$, 所以 $\\angle C>\\angle D$, 根据锐角三角函数的增减性, 即可判断.\n\n【解答】 解: 如图, 设 $A D$ 与圆的交点为 $E$, 连接 $B E$,\n\n\n\n根据圆周角定理, 可得 $\\angle C=\\angle A E B$,\n\n$\\because \\angle A E B=\\angle D+\\angle D B E$,\n\n$\\therefore \\angle A E B>\\angle D$,\n\n$\\therefore \\angle C>\\angle D$,\n\n锐角三角函数的增减性, 可得 $\\sin C>\\sin D$, 故 1 )正确;\n\n$\\cos C<\\cos D$, 故(2)错误;\n\n$\\tan C>\\tan D$, 故(3)正确.\n\n故选 $D$."} {"id": "3372", "image": ["4429.jpg", "4430.jpg"], "answer": "B", "solution": "null", "level": "九年级", "question": "某简易房示意图如图所示, 它是一个轴对称图形, 则坡屋顶上弦杆 $A B$ 的长为( )", "options": "A. $\\frac{9}{5 \\sin \\alpha}$ 米\n\nB. $\\frac{9}{5 \\cos \\alpha}$ 米\n\nC. $\\frac{5}{9 \\sin \\alpha}$ 米\n\nD. $\\frac{5}{9 \\cos \\alpha}$ 米", "subject": "度量几何学", "analysis": "解: 作 $A D \\perp B C$ 于点 $D$, 则 $B D=\\frac{3}{2}+0.3=\\frac{9}{5}$,\n\n\n\n$\\because \\cos \\alpha=\\frac{B D}{A B}$,\n\n$\\therefore \\cos \\alpha=\\frac{\\frac{9}{5}}{A B}$,\n\n解得, $A B=\\frac{9}{5 \\cos \\alpha}$ 米,\n\n故选: $B$.\n\n根据题意作出合适的辅助线, 然后利用锐角三角函数即可表示出 $A B$ 的长.\n\n本题考查解直角三角形的应用、轴对称图形, 解答本题的关键是明确题意, 利用数形结合的思想解答."} {"id": "3375", "image": ["4435.jpg", "4436.jpg", "4437.jpg"], "answer": "C", "solution": "null", "level": "九年级", "question": "如图, 正方形 $A B C D$ 的对角线 $A C, B D$ 相交于点 $O$, 点 $F$ 是 $C D$ 上一点, $O E \\perp O F$ 交 $B C$ 于点 $E$, 连接 $A E, B F$ 交于点 $P$, 连接 $O P$. 则下列结论: (1) $A E \\perp B F$ (2) $\\triangle O A P \\sim \\triangle E A C$; (3)四边形 $O E C F$ 的面积是正方形 $A B C D$ 面积的 $\\frac{1}{4}$; (4) $A P-B P=\\sqrt{2} O P$\n\n(5) 若 $B E: C E=2: 3$, 则 $\\tan \\angle C A E=\\frac{4}{7}$. 其中正确的结论有 $(\\quad)$ 个\n\n", "options": "A. 2 个\nB. 3 个\nC. 4 个\nD. 5 个", "subject": "度量几何学", "analysis": "【分析】\n\n本题考查了正方形的性质, 全等三角形的判定和性质, 相似三角形的判定和性质, 解直角三角形等知识, 解决问题的关键是熟练掌握有关基础知识.\n\n(1) 可证明 $\\triangle C O F \\cong \\triangle B O E$, 进而证明 $\\triangle A B E \\cong \\triangle B C F$, 进一步得出结论;\n\n(2)可证明 $\\triangle A B P \\sim \\triangle A E B$, 从而 $A B^{2}=A P \\cdot A E$, 可证明 $\\triangle A O B \\sim \\triangle A B C$, 从而 $A B^{2}=O A \\cdot A C$, 进而得出 $A P \\cdot A E=O A \\cdot A C$, 从而得出结论;\n\n(3) 由四边形 $O E C F$ 的面积等于 $\\triangle C O E$ 的面积加 $\\triangle C O F$ 的面积可得四边形 $O E C F$ 的面积等于 $\\triangle C O E$的面积加 $\\triangle B O E$ 的面积, 从而四边形 $O E C F$ 的面积等于 $\\triangle B O C$ 的面积, 进而得出结论;\n\n(4) 作 $\\angle P O Q=90^{\\circ}$, 交 $A P$ 于 $Q$, 可证得 $P Q=\\sqrt{2} O P$ 及 $\\triangle A O Q \\cong \\triangle B O P$, 进一步得出结论;\n\n(5)作 $F G \\perp B D$ 于 $G$, 设 $C F=2 a$, 则 $C D=B C=5 a, B D=\\sqrt{2} B C=5 \\sqrt{2} a$, 可得出 $\\tan \\angle D B F=\\frac{3}{7}$,\n可证得 $\\angle D B F=\\angle C A E$, 从而得出 $\\tan \\angle C A E=\\frac{3}{7}$, 从而得出结论.\n\n【解答】\n\n解: :四边形 $A B C D$ 是正方形,\n\n$\\therefore O B=O C, \\angle O C F=\\angle O B E=45^{\\circ}, \\angle B O C=90^{\\circ}, A B=B C, \\angle A B C=\\angle B C D=90^{\\circ}$,\n\n$\\therefore \\angle B O C=\\angle E O F=90^{\\circ}$,\n\n$\\therefore \\angle B O C-\\angle C O E=\\angle E O F-\\angle C O E$,\n\n$\\therefore \\angle C O F=\\angle B O E$,\n\n$\\therefore \\triangle C O F \\cong \\triangle B O E(A A S)$,\n\n$\\therefore C F=B E$,\n\n$\\therefore \\triangle A B E \\cong \\triangle B C F(S A S)$,\n\n$\\therefore \\angle C B F=\\angle B A E$,\n\n$\\because \\angle A B E+\\angle C B F=\\angle A B C=90^{\\circ}$,\n\n$\\therefore \\angle A B E+\\angle B A E=90^{\\circ}$,\n\n$\\therefore \\angle A P B=90^{\\circ}$,\n\n$\\therefore A E \\perp B F$,\n\n故(1)正确;\n\n由 $\\triangle A B P \\sim \\triangle A E B$ 得, $A B^{2}=A P \\cdot A E$,\n\n由 $\\triangle A O B \\backsim \\triangle A B C$ 得, $A B^{2}=O A \\cdot A C$,\n\n$\\therefore A P \\cdot A E=O A \\cdot A C$,\n\n$\\because \\angle P O A=\\angle C A E$,\n\n$\\therefore \\triangle A O P \\sim \\triangle A E C$,\n\n故(2)正确;\n\n由(1)知: $\\triangle C O F \\cong \\triangle B O E$,\n\n$\\because$ 四边形 $O E C F$ 的面积等于 $\\triangle C O E$ 的面积加 $\\triangle C O F$ 的面积,\n\n$\\therefore$ 四边形 $O E C F$ 的面积等于 $\\triangle C O E$ 的面积加 $\\triangle B O E$ 的面积,\n\n$\\therefore$ 四边形 $O E C F$ 的面积等于 $\\triangle B O C$ 的面积,\n\n而 $\\triangle B O C$ 的面积等于正方形 $A B C D$ 的面积的 $\\frac{1}{4}$,\n\n$\\therefore$ 四边形 $O E C F$ 的面积是正方形 $A B C D$ 面积的 $\\frac{1}{4}$;\n\n故(3)正确;\n如图,\n\n\n\n作 $\\angle P O Q=90^{\\circ}$, 交 $A P$ 于 $Q$,\n\n$\\because \\angle A P O=45^{\\circ}$,\n\n$\\therefore \\angle O Q P=90^{\\circ}-\\angle A P O=45^{\\circ}$,\n\n$\\therefore O Q=O P, P Q=\\sqrt{2} O P$,\n\n$\\because \\angle A O B=\\angle P O Q=90^{\\circ}$,\n\n$\\therefore \\angle A O Q=\\angle B O P$,\n\n$\\because O A=O B$,\n\n$\\therefore \\triangle A O Q \\cong \\triangle B O P(S A S)$,\n\n$\\therefore A Q=B P$,\n\n$\\because A P-A Q=P Q$,\n\n$\\therefore A P-P Q=\\sqrt{2} O P$,\n\n故(4)正确;\n\n如图2,\n\n\n\n图2\n\n作 $F G \\perp B D$ 于 $G$,\n\n$\\because B E: C E=2: 3$,\n\n$\\therefore B E: B C=2: 5$,\n\n$\\because C F=B E$,\n\n$\\therefore C F: B C=2: 5$,\n\n设 $C F=2 a$, 则 $C D=B C=5 a, B D=\\sqrt{2} B C=5 \\sqrt{2} a$,\n$\\therefore D F=3 a$,\n\n$\\therefore F G=D G=\\frac{\\sqrt{2}}{2} D F=\\frac{3 \\sqrt{2}}{2} a$,\n\n$\\therefore B G=B D-D G=5 \\sqrt{2} a-\\frac{3 \\sqrt{2}}{2}=\\frac{7 \\sqrt{2}}{2} a$,\n\n$\\therefore \\tan \\angle D B F=\\frac{F G}{B G}=\\frac{\\frac{3 \\sqrt{2}}{2} a}{\\frac{7 \\sqrt{2}}{2} a}=\\frac{3}{7}$,\n\n$\\because \\angle A B D=\\angle A P O=45^{\\circ}$,\n\n$\\therefore$ 点 $A 、 B 、 P 、 O$ 共圆,\n\n$\\therefore \\angle D B F=\\angle C A E$,\n\n$\\therefore \\tan \\angle C A E=\\frac{3}{7}$,\n\n故(5)不正确,\n\n$\\therefore$ (1)(2)(3)正确,\n\n故选: $C$."} {"id": "3384", "image": ["4453.jpg"], "answer": "D", "solution": "null", "level": "九年级", "question": "在 $\\triangle A B C$ 中, 已知 $\\tan A=\\tan B$, 则下列说法不正确的是 ( )", "options": "A. 边 $A B$ 上任意一点 $P$ 到边 $A C 、 B C$ 的距离之和等于点 $B$ 到 $A C$ 的距离\n\nB. 边 $A B$ 的垂直平分线是 $\\triangle A B C$ 的对称轴\nC. $\\triangle A B C$ 的外心可能在 $\\triangle A B C$ 内部、边上或外部\n\nD. 如果 $\\triangle A B C$ 的周长是 $l$, 那么 $B C=l-2 A B$", "subject": "度量几何学", "analysis": "解:\n\n在 $\\triangle A B C$ 中, $\\tan A=\\tan B$,\n\n$\\therefore \\angle A=\\angle B$,\n\n$P$ 为 $A B$ 边上任意一点, 作 $P E \\perp B C$ 交 $B C$ 于 $E$ 点, 作 $P F \\perp A C$ 于 $F$ 点, 作 $B D \\perp A C$ 于 $D$ 点, 如图:\n\n\n\n$\\therefore \\sin A=\\frac{P F}{P A}, \\sin \\angle P B E=\\frac{P E}{P B}, \\sin A=\\frac{B D}{A B}$,\n\n$\\therefore P F=P A \\sin A, P E=P B \\sin B=P B \\sin A, B D=A B \\sin A$,\n\n$\\therefore P F+P E=P A \\sin A+P B \\sin A=(P A+P B) \\sin A=A B \\sin A=B D$,\n\n$\\therefore$ :边 $A B$ 上任意一点 $P$ 到边 $A C 、 B C$ 的距离之和等于点 $B$ 到 $A C$ 的距离, 故 $A$ 中说法正确;\n\n$\\because \\angle A=\\angle A B C$,\n\n$\\therefore C A=C B$,\n\n$\\therefore$ 边 $A B$ 的垂直平分线是 $\\triangle A B C$ 的对称轴, 故 $B$ 中说法正确;\n\n$\\because \\angle A=\\angle B$,\n\n$\\therefore \\angle A, \\angle B$ 均为锐角,\n\n$\\therefore \\triangle A B C$ 可能为锐角三角形, 也可能为直角三角形, 也可能为针角三角形,\n$\\therefore \\triangle A B C$ 的外心可能在 $\\triangle A B C$ 内部、边上或外部, 故 $C$ 中说法正确;\n\n$\\because \\triangle A B C$ 的周长是 $l$,\n\n$\\therefore A B+A C+B C=l$\n\n$\\because \\angle A=\\angle B$,\n\n$\\therefore A C=B C$,\n\n$\\therefore 2 B C+A B=l$,\n\n$\\therefore B C=\\frac{l-A B}{2} \\neq l-2 A B$, 故 $D$ 中说法错误.\n\n故选 $D$."} {"id": "3393", "image": ["4478.jpg"], "answer": "D", "solution": "null", "level": "九年级", "question": "如图, $\\overparen{A B}$ 是半径为 1 的半圆弧, $\\triangle A O C$ 为等边三角形, 点 $D$ 是 $\\overparen{B C}$ 上的一动点、则 $\\triangle C O D$ 的面积 $S$ 的最大值是 $\\quad(\\quad)$\n\n", "options": "A. $\\frac{\\sqrt{3}}{4}$\nB. $\\frac{\\sqrt{3}}{3}$\nC. $\\frac{\\sqrt{3}}{2}$\nD. $\\frac{1}{2}$", "subject": "度量几何学", "analysis": "【分析】\n\n本题考查了三角形的面积的求法与锐角三角函数的增减性有关知识, 根据三角形的面积公式\n\n$S_{\\triangle C O D}=\\frac{1}{2} C O \\cdot O D \\sin \\angle C O D$, 因为 $C O, O D$ 都是圆的半径 1 , 所以 $S_{\\triangle C O D}=\\frac{1}{2} \\sin \\angle C O D$, 根据 $\\angle C O D$的范围, 分锐角, 钝角两种情况分析, 进行解答即可.\n\n【解答】\n\n解: $S_{\\triangle C O D}=\\frac{1}{2} C O \\cdot O D \\sin \\angle C O D$,\n\n$\\because C O=O D=1$,\n\n$\\therefore S_{\\triangle C O D}=\\frac{1}{2} \\sin \\angle C O D$,\n\n$\\because \\triangle A O C$ 为等边三角形,\n\n$\\therefore \\angle C O B=120^{\\circ}$,\n\n$\\therefore 0^{\\circ}<\\angle C O D \\leq 120^{\\circ}$,\n\n当 $0^{\\circ}<\\angle C O D \\leq 90^{\\circ}$ 时, 随着 $\\angle C O D$ 角度的增大, $\\sin \\angle C O D$ 增大,\n\n当 $90^{\\circ}<\\angle C O D \\leq 120^{\\circ}$ 时, 随着 $\\angle C O D$ 角度的增大, 三角形的底不变, 高在减小, 面积 $S$ 在减小, $\\therefore$ 当 $\\angle C O D=90^{\\circ}$ 时, $\\sin \\angle C O D$ 最大, 最大值是 1 ,\n\n$\\therefore \\triangle C O D$ 的面积 $S$ 的最大值是 $\\frac{1}{2}$.\n\n故选 $D$."} {"id": "3395", "image": ["4481.jpg"], "answer": "B", "solution": "null", "level": "九年级", "question": "已知圆内接正三角形的面积为 $\\sqrt{3}$, 则该圆的内接正六边形的边心距是 ( )", "options": "A. 2\nB. 1\nC. $\\sqrt{3}$\nD. $\\frac{\\sqrt{3}}{2}$", "subject": "度量几何学", "analysis": "【分析】\n\n本题考查正多边形和圆, 解答本题的关键是明确题意, 求出相应的图形的边心距.\n\n根据题意可以求得半径, 进而解答即可.\n\n【解答】\n\n解: 如图(1), $O$ 为 $\\triangle A B C$ 的中心,\n\n$A D$ 为 $\\triangle A B C$ 的边 $B C$ 上的高,\n\n则 $O D$ 为边心距,\n\n$\\therefore \\angle B A D=30^{\\circ}$,\n\n又 $\\because A O=B O$,\n\n$\\therefore \\angle A B O=\\angle B A D=30^{\\circ}$,\n\n$\\therefore \\angle O B D=60^{\\circ}-30^{\\circ}=30^{\\circ}$,\n\n\n\n图1\n\n在 $R t \\triangle O B D$ 中, $B O=2 D O$,\n\n即 $A O=2 D O$,\n\n$\\therefore O D: O A: A D=1: 2: 3$.\n\n在正 $\\triangle A B C$ 中, $A D$ 是高, 设 $B D=x$,\n\n则 $A D=B D \\cdot \\tan 60^{\\circ}=\\sqrt{3} B D=\\sqrt{3} x$.\n\n$\\because$ 正三角形 $A B C$ 面积为 $\\sqrt{3}$,\n\n$\\therefore \\frac{1}{2} B C \\cdot A D=\\sqrt{3}$,\n\n$\\therefore \\frac{1}{2} \\times 2 x \\cdot \\sqrt{3} x=\\sqrt{3}$,\n\n$\\therefore x=1$.\n\n即 $B D=1$, 则 $A D=\\sqrt{3}$,\n\n$\\because O D: O A: A D=1: 2: 3$,\n\n$\\therefore A O=\\sqrt{3} \\times \\frac{2}{3}=\\frac{2 \\sqrt{3}}{3}$.\n\n即这个圆的半径为 $\\frac{2 \\sqrt{3}}{3}$.\n所以该圆的内接正六边形的边心距 $\\frac{2 \\sqrt{3}}{3} \\times \\sin 60^{\\circ}=\\frac{2 \\sqrt{3}}{3} \\times \\frac{\\sqrt{3}}{2}=1$,\n\n故选: $B$."} {"id": "3397", "image": ["4483.jpg", "4484.jpg", "4485.jpg", "4486.jpg"], "answer": "C", "solution": "null", "level": "九年级", "question": "四巧板是一种类似七巧板的传统智力玩具, 它是由一个长方形按如图1分割而成, 这几个多边形的内角除了有直角外, 还有 $45^{\\circ} 、 135^{\\circ} 、 270^{\\circ}$ 角. 小明发现可以将四巧板拼搭成如图 2 的\n$T$ 字形和 $V$ 字形, 那么 $T$ 字形图中高与宽的比值 $\\frac{h}{l}$ 为 $($ )\n\n\n\n图1\n\n\n\n图2", "options": "A. $\\sqrt{2}$\nB. $\\frac{\\sqrt{2}+1}{2}$\nC. $\\frac{4+\\sqrt{2}}{4}$\nD. $\\frac{3 \\sqrt{2}}{2}$", "subject": "度量几何学", "analysis": "解: 如图1中, 设 $A B=a$,\n\n则 $A C=D E=\\sqrt{2} a, C E=2 \\sqrt{2} a$,\n\n$\\therefore h=a+2 \\sqrt{2} a, \\quad l=2 \\sqrt{2} a$,\n\n$\\therefore \\frac{h}{l}=\\frac{a+2 \\sqrt{2} a}{2 \\sqrt{2} a}=\\frac{4+\\sqrt{2}}{4}$\n\n故选: $C$.\n\n\n\n图1\n\n\n\n图2\n\n如图1中, 设 $A B=a$, 则 $A C=D E=\\sqrt{2} a, C E=2 \\sqrt{2} a$, 求出 $h, l$, 可得结论.\n\n本题考查解直角三角形, 四巧板, 等腰直角三角形的性质等知识, 解题的关键是读懂图象信息,学会利用参数解决问题."} {"id": "3401", "image": ["4492.jpg"], "answer": "C", "solution": "null", "level": "九年级", "question": "如图, 点 $O$ 是 $\\triangle A B C$ 的内切圆的圆心, 若 $\\angle O A C=40^{\\circ}$, 则 $\\angle B O C$ 的度数为 ( )\n\n", "options": "A. $80^{\\circ}$\nB. $100^{\\circ}$\nC. $130^{\\circ}$\nD. $140^{\\circ}$\n第 II 卷(非选择题)", "subject": "度量几何学", "analysis": "略"} {"id": "3409", "image": ["4503.jpg", "4504.jpg"], "answer": "C", "solution": "null", "level": "九年级", "question": "如图, $A B$ 是 $\\odot O$ 的弦, $A C$ 是 $\\odot O$ 的切线, $A$ 为切点, $B C$ 经过圆心, 若 $\\angle B=25^{\\circ}$, 则 $\\angle C$ 的大小等于 $(\\quad)$\n\n", "options": "A. $20^{\\circ}$\nB. $25^{\\circ}$\nC. $40^{\\circ}$\nD. $50^{\\circ}$", "subject": "度量几何学", "analysis": "【分析】本题考查切线的性质, 等腰三角形的性质, 三角形外角的性质, 关键是连接 $O A$,先根据等腰三角形的性质得所以 $\\angle B=\\angle B A O$, 根据切线的性质和直角三角形的性质即可求得 $\\angle C$ 的\n度数.\n\n【解答】\n\n解: 连接 $O A, \\odot O$ 中, $O A=O B$, 所以 $\\angle B=\\angle B A O=25^{\\circ}$,\n\n因为 $\\angle A O C$ 是 $\\triangle O A B$ 的外角, 所以 $\\angle A O C=\\angle B+\\angle B A O=50^{\\circ}$, 又因为 $A C$ 是 $\\odot O$ 的切线, 所以 $O A \\perp$ $A C$, 在Rt $\\triangle O A C$ 中, $\\angle C=90^{\\circ}-\\angle A O C=40^{\\circ}$, 故选 $C$.\n\n"} {"id": "3418", "image": ["4516.jpg", "4517.jpg"], "answer": "C", "solution": "null", "level": "九年级", "question": "如图, $\\odot C$ 与 $\\angle A O B$ 的两边分别相切, 其中 $O A$ 边与 $\\odot C$ 相切于点 $P$, 若 $\\angle A O B=90^{\\circ}, O P=4$,则 $O C$ 的长为 $(\\quad)$\n\n", "options": "A. 8\nB. $16 \\sqrt{2}$\nC. $4 \\sqrt{2}$\nD. $2 \\sqrt{2}$", "subject": "度量几何学", "analysis": "解:如图, 连接 $C P$,\n\n\n\n$\\because O A$ 边与 $\\odot C$ 相切于点 $P$,\n\n$\\therefore C P \\perp O A$,\n\n$\\therefore \\angle O P C=90^{\\circ}$,\n\n$\\because \\odot C$ 与 $\\angle A O B$ 的两边分别相切, $\\angle A O B=90^{\\circ}$,\n\n$\\therefore O C$ 平分 $\\angle A O B$,\n\n$\\therefore \\angle C O P=\\frac{1}{2} \\angle A O B=\\frac{1}{2} \\times 90^{\\circ}=45^{\\circ}$,\n\n$\\therefore \\triangle O C P$ 为等腰直角三角形,\n\n$\\because O P=4$,\n\n$\\therefore O C=\\sqrt{2} O P=4 \\sqrt{2}$.\n\n故选: $C$.\n\n如图, 连接 $C P$, 先利用切线的性质得到 $\\angle O P C=90^{\\circ}$, 再利用切线长定理得到 $O C$ 平分 $\\angle A O B$, 则 $\\triangle O C P$ 为等腰直角三角形, 从而得到 $O C=\\sqrt{2} O P$.\n\n本题考查了切线的性质: 圆的切线垂直于经过切点的半径. 也考查了切线长定理."} {"id": "3612", "image": [], "answer": "B", "solution": "null", "level": "九年级", "question": "在Rt $\\triangle A B C$ 中, $\\angle C=90^{\\circ}, \\angle A 、 \\angle B$ 的对边分别是 $a 、 b$, 且满足 $a^{2}-a b-b^{2}=0$, 则 $\\tan A$ 等于 $(\\quad)$", "options": "A. 1\nB. $\\frac{1+\\sqrt{5}}{2}$\nC. $\\frac{1-\\sqrt{5}}{2}$\nD. $\\frac{1 \\pm \\sqrt{5}}{2}$", "subject": "代数", "analysis": "$\\because a 、 b$ 满足 $a^{2}-a b-b^{2}=0$,\n\n等式两边同时除以 $b^{2}$ 得: $\\left(\\frac{a}{b}\\right)^{2}-\\frac{a}{b}-1=0$,\n\n解得 $\\frac{a}{b}=\\frac{1 \\pm \\sqrt{5}}{2} ,$\n\n$\\because \\tan A=\\frac{a}{b}>0$,\n\n故 $\\tan A=\\frac{1+\\sqrt{5}}{2}$.\n\n故选 $B$."} {"id": "2975", "image": [], "answer": "B", "solution": "null", "level": "九年级", "question": "(本题 3 分) (2019$\\cdot$浙江$\\cdot$长兴县实验中学九年级期中) 若关于 $x$ 的函数 $y=(2-a)$ $x^{2}-x$ 是二次函数,则 $a$ 的取值范围是 ( )", "options": "A. $a \\neq 0$B. $a \\neq 2$C. $a<2$D. $a>2$", "subject": "代数", "analysis": "B【解析】【详解】解:because函数y=(2-a)x^2-x是二次函数,therefore2-aneq0,即aneq2,故选B."} {"id": "2995", "image": ["3899.jpg"], "answer": "B", "solution": "null", "level": "九年级", "question": "(本题 3 分) (2019$\\cdot$浙江$\\cdot$宁波市第七中学九年级阶段练习)函数 $y=x^{2}+b x+c$ 与 $y=x$ 的图象如图所示, 有以下结论: (1) $b^{2}-4 c>0$; (2) $b+c+1=0$; (3) $3 b+c+6=0$; (4)当 $1", "options": "A. 1B. 2C. 3D. 4", "subject": "代数", "analysis": "B【解析】【详解】解:because函数y=x^2+bx+c与x轴无交点,thereforeb^2-4c<0;故①错误.当x=1时,y=1+b+c=1,故(2)错误.because当x=3时,y=9+3b+c=3,therefore3b+c+6=0.故(3)正确.because当10,所以存在实数m,故符合题意;对于B选项则有m^2+m+1=0,由一元二次方程根的判别式可得:b^2-4ac=1-4=-3<0,所以不存在实数m,故不符合题意;对于C选项则有-frac1m-m-1=0,化简得:m^2+m+1=0,由一元二次方程根的判别式可得:b^2-4ac=1-4=-3<0,所以不存在实数m,故不符合题意;对于D选项则有-frac1m-m+1=0,化简得:m^2-m+1=0,由一元二次方程根的判别式可得:b^2-4ac=1-4=-3<0,所以不存在实数m,故不符合题意;故选A.【点睛】本题主要考查一元二次方程根的判别式、二次函数与反比例函数的性质,熟练掌握一元二次方程根的判别式、二次函数与反比例函数的性质是解题的关键."} {"id": "2998", "image": [], "answer": "C", "solution": "null", "level": "九年级", "question": "(本题 3 分) (2019$\\cdot$浙江金华・九年级期中) 下列函数是二次函数的是 ( )", "options": "A. $y=2 x+1$B. $y=-2 x+1$C. $y=x^{2}+2$D. $y=\\frac{1}{2} x-2$", "subject": "代数", "analysis": "C【解析】【详解】根据二次函数的定义,形如y=axx^2+bx+c(其中a,b,c是常数,aneq0)的函数叫做二次函数,所给函数中是二次函数的是y=x^2+2.故选C."} {"id": "3177", "image": [], "answer": "A", "solution": "null", "level": "九年级", "question": "(本题 3 分) (2021$\\cdot$浙江$\\cdot$杭州市建兰中学九年级期中)一本书的宽与长之比为黄金比,书的宽为 $14 \\mathrm{~cm}$, 则它的长为 $(\\quad) \\mathrm{cm}$", "options": "A. $7 \\sqrt{5}+7$B. $21-7 \\sqrt{5}$C. $7 \\sqrt{5}-7$D. $7 \\sqrt{5}-21$", "subject": "代数", "analysis": "A【分析】根据黄金比值是 $\\frac{\\sqrt{5}-1}{2}$ 进行计算即可.【详解】解: $\\because$ 一本书的宽与长之比为黄金比,$\\therefore$ 这本书的长 $=\\frac{14}{\\frac{\\sqrt{5}-1}{2}} \\mathrm{~cm}=(7 \\sqrt{5}+7) \\mathrm{cm}$,故选: A 。【点睛】本题考查的是黄金分割, 掌握黄金比值是 $\\frac{\\sqrt{5}-1}{2}$ 是解题的关键."} {"id": "3182", "image": [], "answer": "D", "solution": "null", "level": "九年级", "question": "(本题 3 分)(2022$\\cdot$浙江舟山$\\cdot$九年级期末)已知 $\\frac{b}{a}=\\frac{3}{2}$, 下列变形正确的是", "options": "$a b=6$B. $2 a=3 b$C. $a=\\frac{3}{2} b$D. $3 a=2 b$", "subject": "代数", "analysis": "D【分析】根据比例的性质得出3a=2b,再判断即可.【详解】解:becausefracba=frac32,therefore等式两边都乘以2a,得2b=3a,故选:D.【点睛】本题考查了比例的性质,能正确运用比例的性质进行变形是解此题的关键,注意:如果ab=cd,那么fracac=fracdb,反之反亦然."} {"id": "19", "image": [], "answer": "B", "solution": "null", "level": "九年级", "question": "在下列四个函数中,当 $x>0$ 时, $y$ 随 $x$ 的增大而减小的函数是()\n ", "options": "A. $y=2 x$\n B. $y=\\frac{3}{x}$\n C. $y=3 x-2$\n D. $y=x^{2}$\n\n", "subject": "代数", "analysis": "分别根据正比例函数、反比例函数、一次函数和二次函数的性质逐项判断即得答案.\n\n【详解】解: A、 $\\because 2>0, \\therefore$ 当 $x>0$ 时, 函数 $y=2 x$ 是 $y$ 随着 $x$ 增大而增大, 故本选项错误;\n\nB、 $\\because 3>0, \\therefore$ 当 $x>0$ 时, 函数 $y=\\frac{3}{x}$ 是 $y$ 随着 $x$ 增大而减小, 故本选项正确;\n\nC、 $\\because 3>0, \\therefore$ 当 $x>0$ 时,函数 $y=3 x-2$ 是 $y$ 随着 $x$ 增大而增大,故本选项错误;\n\nD、函数 $y=x^{2}$, 当 $x<0$ 时, $y$ 随着 $x$ 增大而减小,当 $x>0$ 时, $y$ 随着 $x$ 增大而增大,故本选项错误.\n\n故选: B.\n\n【点睛】本题考查了初中阶段三类常见函数的性质,属于基础题型,熟练掌握一次函数、反比例函数和二次函数的性质是解题的关键.\n\n"} {"id": "20", "image": [], "answer": "A", "solution": "null", "level": "九年级", "question": "若二次函数 $y=x^{2}-4 x+c$ 的图像经过点 $(0,3)$, 则函数的最小值是 ( )\n ", "options": "A. -1\n B. 3\n C. 5\n D. 7\n\n", "subject": "代数", "analysis": "把点 $(0,3)$ 代入二次函数 $y=x^{2}-4 x+c$ 求出 $c$ 的值, 根据二次函数解析式变为顶点式, 由此即可求出最小值.\n\n【详解】解:二次函数 $y=x^{2}-4 x+c$ 的图像经过点 $(0,3)$,\n\n$\\therefore c=3$, 则二次函数解析式为 $y=x^{2}-4 x+3$,\n\n$\\therefore$ 二次函数的顶点式为 $y=(x-2)^{2}-1$,\n\n$\\therefore$ 顶点为 $(2,-1)$, 最小值为 -1 ,\n\n故选: A.\n\n【点睛】本题主要考查二次函数的最值问题,掌握二次函数顶点式是解题的关键.\n\n"} {"id": "24", "image": [], "answer": "B", "solution": "null", "level": "九年级", "question": "下列函数中是二次函数的是 ( )\n", "options": "A. $y=x+\\frac{1}{2}$\nB. $y=3(x-1)^{2}$\nC. $y=(x+1)^{2}-x^{2}$\nD. $y=\\frac{1}{x^{2}}-x$\n", "subject": "代数", "analysis": "整理成一般形式后,根据二次函数的定义判定即可.\n\n【详解】解: A 选项: $y=x+\\frac{1}{2}$ 是一次函数, 故此选项错误;\n\nB 选项: $y=3(x-1)^{2}=3 x^{2}-6 x+3$, 是二次函数, 故此选项正确;\n\nC 选项: $y=(x+1)^{2}-x^{2}=2 x+1$, 为一次函数, 故此选项错误;\n\n$\\mathrm{D}$ 选项: $y=\\frac{1}{x^{2}}-x$ 是组合函数, 不是二次函数, 故此选项错误.\n\n故选: B.\n\n【点睛】本题考查了二次函数的定义: 函数 $y=a x^{2}+b x+c \\quad(a \\neq 0, a 、 b 、 c$ 为常数 $)$ 叫二次函数.\n\n"} {"id": "25", "image": [], "answer": "B", "solution": "null", "level": "九年级", "question": "已知二次函数 $y=a(x-2)^{2}-a(a \\neq 0)$, 当 $-1 \\leq x \\leq 4$ 时, $y$ 的最小值为 -4 , 则 $a$ 的值为 ( )\n", "options": "A. $\\frac{1}{2}$ 或 4\nB. 4 或 $-\\frac{1}{2}$\nC. $-\\frac{4}{3}$ 或 4\nD. $-\\frac{1}{2}$ 或 $\\frac{4}{3}$\n\n##", "subject": "代数", "analysis": "根据表达式求出对称轴, 对 $a$ 的正负进行分类讨论, 求出每种情况的最小值即可.\n\n【详解】解: $y=a(x-2)^{2}-a$ 的对称轴为直线 $x=2$,\n\n顶点坐标为 $(2,-a)$,\n\n当 $a>0$ 时, 在 $-1 \\leq x \\leq 4$,\n\n$\\because y$ 的最小值为 -4 ,\n\n$\\therefore-a=-4$,\n\n$\\therefore a=4$;\n\n$a<0$ 时, 在 $-1 \\leq x \\leq 4$,\n\n当 $x=-1$ 时函数有最小值,\n\n$\\therefore a(-1-2)^{2}-a=-4$,\n\n解得 $a=-\\frac{1}{2}$;\n\n综上所述: $a$ 的值为 4 或 $-\\frac{1}{2}$,\n\n故选: B.\n\n【点睛】本题考查了二次函数的性质, 对 $a$ 的分类讨论是本题的解题关键.\n\n"} {"id": "3616", "image": ["4937.jpg", "4938.jpg"], "answer": "A", "solution": "null", "level": "九年级", "question": "如图, 在高度是 90 米的小山 $A$ 处测得建筑物 $C D$ 顶部 $C$ 处的仰角为 $30^{\\circ}$, 底部 $D$ 处的俯角为 $45^{\\circ}$,\n\n\n\n则这个建筑物的高度 $C D$ 是 $(\\quad)$ (结果可以保留根号)", "options": "A. $30(3+\\sqrt{3})$ 米\nB. $45(2+\\sqrt{3})$ 米\nC. $30(1+3 \\sqrt{3})$ 米\nD. $45(1+\\sqrt{2})$ 米", "subject": "画法几何学", "analysis": "作 $A E \\perp C D$ 于点 $E$.\n\n在直角 $\\triangle A B D$ 中, $\\angle A D B=45^{\\circ}$,\n\n\n$\\therefore D E=A E=B D=A B=90$ (米),\n\n在直角 $\\triangle A E C$ 中, $C E=A E \\cdot \\tan \\angle C A E=90 \\times \\frac{\\sqrt{3}}{3}=30 \\sqrt{3}$ (米).\n\n则 $C D=(90+30 \\sqrt{3})$ 米.\n\n故选: $A$."} {"id": "3635", "image": ["4976.jpg", "4977.jpg"], "answer": "D", "solution": "null", "level": "九年级", "question": "如图, $\\odot O$ 的圆心在矩形 $A B C D$ 的对角线 $A C$ 上, 且 $\\odot O$ 与 $A B, B C$ 相切, $A B=3, B C=4$, 则 $\\odot O$ 截 $A D$ 的所得的弦 $E F$ 的长是 $(\\quad)$", "options": "A. 3\nB. $\\sqrt{5}$\nC. $\\frac{6 \\sqrt{5}}{7}$\nD. $\\frac{6 \\sqrt{7}}{7}$\n\n", "subject": "画法几何学", "analysis": "如图, $\\because \\odot O$ 与 $A B, B C$ 相切,\n\n$\\therefore$ :设切点为 $G, H$, 连接 $O G, H O$ 并延长交 $A D$ 于 $K$, 连接 $O F$, 则四边形 $O G B H$ 为正方形,\n\n\n\n设正方形边长为 $x$,\n\n$\\because$ 四边形 $A B C D$ 是矩形,\n\n$\\therefore A B \\perp B C$\n\n$\\because O G \\perp A B$,\n\n$\\therefore O G / / B C$,\n\n$\\therefore \\triangle A B C \\sim \\triangle A G O$,\n\n$\\therefore \\frac{A G}{A B}=\\frac{G O}{B C}$,\n\n$\\therefore \\frac{3-x}{3}=\\frac{x}{4}$,\n解得: $x=\\frac{12}{7} ,$\n\n$\\therefore O K=A G=3-\\frac{12}{7}=\\frac{9}{7}$, 由\n\n垂径定理, $O K \\perp E F, E K \\perp K F$,\n\n$\\therefore$ 在Rt $\\triangle O K F$ 中, $O F=x=\\frac{12}{7}, K F=\\sqrt{O F^{2}-O K^{2}}=\\sqrt{\\left(\\frac{12}{7}\\right)^{2}-\\left(\\frac{9}{7}\\right)^{2}}=\\frac{3 \\sqrt{7}}{7}$,\n\n$\\therefore E F=2 K F=\\frac{6 \\sqrt{7}}{7}$,\n\n故选: $D$."} {"id": "3640", "image": ["4982.jpg", "4983.jpg"], "answer": "A", "solution": "null", "level": "九年级", "question": "如图, $\\triangle A B C$ 内切圆是 $\\odot O$, 折叠矩形 $A B C D$, 使点 $D 、 O$ 重合, $F G$是折痕, 点 $F$ 在 $A D$ 上, $G$ 在 $A B C$ 上, 连结 $O G, D G$, 若 $O G$ 垂直 $D G$, 且 $\\odot O$ 的半径为 1 , 则下列结论不成立的是 $(\\quad)$", "options": "A. $C D+D F=4$\nB. $C D-D F=2 \\sqrt{3}-3$\nC. $B C+A B=2 \\sqrt{3}+4$\nD. $B C-A B=2$\n\n", "subject": "画法几何学", "analysis": "如图,\n\n\n\n设 $\\odot O$ 与 $B C$ 的切点为 $M$, 连接 $M O$ 并延长 $M O$ 交 $A D$ 于点 $N$,\n\n$\\because$ 将矩形 $A B C D$ 按如图所示的方式折叠, 使点 $D$ 与点 $O$ 重合, 折痕为 $F G$,\n\n$\\therefore O G=D G$,\n\n$\\because O G \\perp D G$,\n\n$\\therefore \\angle M G O+\\angle D G C=90^{\\circ}$,\n\n$\\because \\angle M O G+\\angle M G O=90^{\\circ}$,\n\n$\\therefore \\angle M O G=\\angle D G C$\n\n在 $\\triangle O M G$ 和 $\\triangle G C D$ 中, $\\left\\{\\begin{array}{l}\\angle O M G=\\angle D C G=90^{\\circ} \\\\ \\angle M O G=\\angle D G C \\\\ O G=D G\\end{array}\\right.$,\n\n$\\therefore \\triangle O M G \\cong \\triangle G C D$,\n\n$\\therefore O M=G C=1, C D=G M=B C-B M-G C=B C-2$.\n\n$\\because A B=C D$,\n\n$\\therefore B C-A B=2$.\n\n设 $A B=a, B C=b, A C=c, \\odot O$ 的半径为 $r$,\n\n$\\odot O$ 是 $R t \\triangle A B C$ 的内切圆可得 $r=\\frac{1}{2}(a+b-c)$,\n\n$\\therefore c=a+b-2$.\n\n在Rt $\\triangle A B C$ 中, 由勾股定理可得 $a^{2}+b^{2}=(a+b-2)^{2}$,\n\n整理得 $2 a b-4 a-4 b+4=0$,\n又 $\\because B C-A B=2$ 即 $b=2+a$, 代入可得 $2 a(2+a)-4 a-4(2+a)+4=0$,\n\n解得 $a=1+\\sqrt{3}$ 或 $a=1-\\sqrt{3}$ (不合题意舍去),\n\n$\\therefore B C+A B=2 \\sqrt{3}+4$.\n\n再设 $D F=x$, 在 $R t \\triangle O N F$ 中, $F N=3+\\sqrt{3}, O F=x, O N=1+\\sqrt{3}$,\n\n由勾股定理可得 $(2+\\sqrt{3}-x)^{2}+(\\sqrt{3})^{2}=x^{2}$,\n\n解得 $x=4-\\sqrt{3}$,\n\n$\\therefore C D-D F=\\sqrt{3}, C D+D F=\\sqrt{3}$.\n\n综上只有选项 $A$ 错误,\n\n故选: $A$."} {"id": "3642", "image": ["4986.jpg", "4987.jpg"], "answer": "B", "solution": "null", "level": "九年级", "question": "如图, 太阳光线与地面成 $60^{\\circ}$ 角, 一棵倾斜的大树 $A B$ 与地面成 $30^{\\circ}$角, 这时测得大树在地面的影长 $B C$ 为 $10 \\mathrm{~m}$, 则大树的长为 $(\\quad)$", "options": "A. $5 \\sqrt{3} m$\nB. $10 \\sqrt{3} m$\nC. $15 \\sqrt{3} m$\nD. $20 \\sqrt{3} m$\n\n", "subject": "画法几何学", "analysis": "如图, 作 $A D \\perp B C$, 角 $B C$ 的延长线于 $D$ 点,\n\n\n\n因为 $\\angle B=30^{\\circ}, \\angle A C D=60^{\\circ}$,\n\n且 $\\angle A C D=\\angle B+\\angle C A B$,\n\n$\\therefore \\angle C A B=30^{\\circ}$.\n\n$\\therefore B C=A C=10 \\mathrm{~m}$,\n\n在 $R t \\triangle A C D$ 中, $C D=A C \\cdot \\cos 60^{\\circ}=10 \\times 0.5=5 \\mathrm{~m}$,\n\n$\\therefore B D=15$.\n\n$\\therefore$ 在Rt $\\triangle A B D$ 中,\n\n$$\nA B=B D \\div \\cos 30^{\\circ}=15 \\div \\frac{\\sqrt{3}}{2}=10 \\sqrt{3} \\mathrm{~m}\n$$\n\n故选 $B$."} {"id": "3653", "image": ["5001.jpg"], "answer": "D", "solution": "null", "level": "九年级", "question": "如图, 小树 $A B$ 在路灯 $O$ 的照射下形成投影 $B C$. 若树高 $A B=2 m$,树影 $B C=3 m$, 树与路灯的水平距离 $B P=4.5 m$. 则路灯的高度 $O P$ 为 $(\\quad)$", "options": "A. $3 m$\nB. $4 m$\nC. $4.5 m$\nD. $5 m$\n\n", "subject": "画法几何学", "analysis": "$\\because A B / / O P$,\n\n$\\therefore \\triangle C A B \\sim \\triangle C O P$,\n\n$\\therefore \\frac{C B}{C P}=\\frac{A B}{O P}$,\n\n$\\therefore \\frac{3}{7.5}=\\frac{2}{O P}$,\n\n$\\therefore O P=5(m)$,\n\n故选: $D$."} {"id": "3664", "image": ["5030.jpg"], "answer": "A", "solution": "null", "level": "九年级", "question": "如图, 一个正六棱柱的表面展开后恰好放入一个矩形内, 把其中一部分图形挪动了位置, 发现矩\n\n\n形的长留出 $5 \\mathrm{~cm}$, 宽留出 $1 \\mathrm{~cm}$, 则该正六棱柱的侧面积是 $(\\quad)$", "options": "A. $108-24 \\sqrt{3}$\nB. $108-12 \\sqrt{3}$\nC. $54-24 \\sqrt{3}$\nD. $54-12 \\sqrt{3}$", "subject": "画法几何学", "analysis": "设正六棱柱的底面边长为 $a c m$, 高为 $h \\mathrm{~cm}$,\n\n挪动前长为 $(2 h+2 \\sqrt{3} a) \\mathrm{cm}$, 宽为 $\\left(4 a+\\frac{1}{2} a\\right) \\mathrm{cm}$,\n\n挪动后长为 $(h+2 a+\\sqrt{3} a) c m$, 宽为 $4 a c m$,\n\n由题意得: $(2 h+2 \\sqrt{3} a)-(h+2 a+\\sqrt{3} a)=5,\\left(4 a+\\frac{1}{2} a\\right)-4 a=1$,\n\n$\\therefore a=2, h=9-2 \\sqrt{3}$,\n\n$\\therefore$ 六棱柱的侧面积是 $6 a h=6 \\times 2 \\times(9-2 \\sqrt{3})=108-24 \\sqrt{3}$;\n\n故选: $A$."} {"id": "3665", "image": ["5031.jpg", "5032.jpg", "5033.jpg", "5034.jpg", "5035.jpg"], "answer": "B", "solution": "null", "level": "九年级", "question": "如图所示的无盖水杯, 俯视图是 $(\\quad)$", "options": "A.\n\n\nB.\n\n\nC.\n\n\nD.\n\n\n\n", "subject": "画法几何学", "analysis": "该立体图形的俯视图为 $B$ 所示.\n\n故选 $B$."} {"id": "3688", "image": ["5081.jpg", "5082.jpg"], "answer": "A", "solution": "null", "level": "九年级", "question": "如图, 相邻两根电杆都用钢索在地面上固定, 一根电杆的钢索系在离地面 $4 m$ 处,另一根电杆的钢索系在离地面 $6 m$ 处,则中间两根钢\n\n\n索相交处点 $P$ 离地面 $(\\quad)$", "options": "A. $2.4 m$\nB. $2.6 m$\nC. $2.8 m$\nD. $3 m$", "subject": "画法几何学", "analysis": "如图, 过点 $P$ 作 $P E \\perp C B, \\therefore P E / / C D, P E / / A B . \\because C D / / A B, \\therefore \\triangle A P B \\sim \\triangle C P D . \\therefore \\frac{D P}{B P}=\\frac{C D}{A B} \\because C D / / P E$, $\\therefore \\frac{D P}{B P}=\\frac{C E}{B E} . \\therefore \\frac{C E}{B E}=\\frac{C D}{A B} . \\therefore \\frac{C E}{B E}=\\frac{2}{3} . \\therefore \\frac{B C}{B E}=\\frac{5}{3} . \\because C D / / P E, \\therefore \\triangle B P E \\sim \\triangle B D C . \\therefore \\frac{P E}{D C}=\\frac{B E}{B C}$, 即 $\\frac{P E}{4}=\\frac{3}{5}$, 解得 $P E=$ $2.4 m$. 故选 $A$.\n\n"} {"id": "3849", "image": ["5372.jpg"], "answer": "D", "solution": "null", "level": "九年级", "question": "长方体的主视图、俯视图如图所示, 则长方体的表面积为 ( )\n\n", "options": "A. 12\nB. 19\nC. 24\nD. 38", "subject": "画法几何学", "analysis": "由主视图可得长方体的长为 4 , 高为 1 ,由俯视图可得长方体的宽为 3 , 则这个长方体的表面积是 $(4 \\times 1+4 \\times 3+3 \\times 1) \\times 2$ $=19 \\times 2=38$. 故这个长方体的表面积是 38. 故选 D."} {"id": "3871", "image": ["5444.jpg"], "answer": "C", "solution": "null", "level": "九年级", "question": "如图, 是一个用若干个相同的小立方块搭成的几何体的三视图, 则组成这个几何体的小立方块的个数是 $(\\quad)$\n\n", "options": "A. 2\nB. 3\nC. 4\nD. 5", "subject": "画法几何学", "analysis": "综合三视图可知, 这个几何体的底层应该有 3 个小正方体,第二层有 1 个小正方体,因此搭成这个几何体所用小正方体的个数是 $3+1=4$ 个.故选 C."} {"id": "3873", "image": ["5445.jpg"], "answer": "A", "solution": "null", "level": "九年级", "question": "若一个几何体的三视图如图所示, 则这个几何体是 ( )\n\n\n\n主视图 俯视图 左视图\n\n(第 4 题)", "options": "A. 三棱柱\nB. 四棱柱\nC. 五棱柱\nD. 长方体", "subject": "画法几何学", "analysis": "根据图中三视图的形状, 符合条件的只有直三棱柱, 因此这个几何体的名称是直三棱柱.故选 A."} {"id": "3895", "image": ["5481.jpg", "5482.jpg"], "answer": "A", "solution": "null", "level": "九年级", "question": "如图, $\\mathrm{PA} 、 \\mathrm{~PB}$ 分别切 $\\odot \\mathrm{O}$ 于 $\\mathrm{A} 、 \\mathrm{~B}$ 两点, 点 $\\mathrm{C}$ 在优孤 $A C B$ 上, $\\angle \\mathrm{P}=80^{\\circ}$, 则 $\\angle \\mathrm{C}$ 的度数为 ( )\n\n", "options": "A. $50^{\\circ}$\nB. $60^{\\circ}$\nC. $70^{\\circ}$\nD. $80^{\\circ}$", "subject": "画法几何学", "analysis": "连接 $\\mathrm{OA}, \\mathrm{OB}, \\because \\mathrm{PA}$ 是圆的切线.\n\n$\\therefore \\angle \\mathrm{OAP}=90^{\\circ}$, 同理 $\\angle \\mathrm{OBP}=90^{\\circ}$,\n\n\n\n根据四边形内角和定理可得:\n\n$\\angle \\mathrm{AOB}=360^{\\circ}-\\angle \\mathrm{OAP}-\\angle \\mathrm{OBP}-\\angle \\mathrm{P}=360^{\\circ}-90^{\\circ}-90^{\\circ}-80^{\\circ}=100^{\\circ}$,\n\n$\\therefore \\angle C=\\frac{1}{2} \\angle A O B=50^{\\circ}$. 故选 A."} {"id": "3914", "image": ["5523.jpg", "5524.jpg"], "answer": "C", "solution": "null", "level": "九年级", "question": "如图, $A B$ 为 $\\odot O$ 的直径, $P$ 点在 $A B$ 的延长线上, $P M$ 切 $\\odot O$ 于 $M$ 点, 若 $O A=a, P M=\\sqrt{3} a$,那么 $\\triangle P M B$ 的周长是 $(\\quad)$\n\n", "options": "A. $2 \\sqrt{3} a$\nB. $(1+\\sqrt{3}) a$\nC. $(2+\\sqrt{3}) a$\nD. $3 \\sqrt{3} a$", "subject": "画法几何学", "analysis": "\n\n连接 $O M, \\because P M$ 是圆 $O$ 的切线, $\\therefore O M \\perp P M$,\n\n$\\because O A=a, \\quad \\therefore O M=a, \\therefore \\tan \\angle M O P=\\frac{M P}{M O}=\\sqrt{3}$,\n\n$\\therefore \\angle M O P=60^{\\circ}, \\therefore \\triangle M O B$ 是等边三角形, $\\angle P=30^{\\circ}$,\n\n$\\therefore M B=O B, O P=2 a, \\therefore C_{\\triangle P M B}=P M+P B+M B=P M+P B+O B=P M+O P=\\sqrt{3} a+2 a=(2+\\sqrt{3}) a$.故选 C."} {"id": "3917", "image": ["5527.jpg"], "answer": "D", "solution": "null", "level": "九年级", "question": "如图, $\\mathrm{PA} 、 \\mathrm{~PB} 、 \\mathrm{CD}$ 分别切 $\\odot \\mathrm{O}$ 于 $\\mathrm{A} 、 \\mathrm{~B} 、 \\mathrm{E}, \\mathrm{CD}$ 交 $\\mathrm{PA} 、 \\mathrm{~PB}$ 于 $\\mathrm{C} 、 \\mathrm{D}$ 两点, 若 $\\angle \\mathrm{P}=40^{\\circ}$, 则 $\\angle \\mathrm{PAE}+\\angle \\mathrm{PBE}$的度数为 $(\\quad)$\n\n", "options": "A. $50^{\\circ}$\nB. $62^{\\circ}$\nC. $66^{\\circ}$\nD. $70^{\\circ}$", "subject": "画法几何学", "analysis": "$\\because$ PA、PB、CD 分别切 $\\odot O$ 于 A. B. E, CD 交 PA、PB 于 C. D 两点,\n\n$\\therefore \\mathrm{CE}=\\mathrm{CA}, \\mathrm{DE}=\\mathrm{DB}, \\therefore \\angle \\mathrm{CAE}=\\angle \\mathrm{CEA}, \\angle \\mathrm{DEB}=\\angle \\mathrm{DBE}$,\n$\\therefore \\angle \\mathrm{PCD}=\\angle \\mathrm{CAE}+\\angle \\mathrm{CEA}=2 \\angle \\mathrm{CAE}, \\angle \\mathrm{PDC}=\\angle \\mathrm{DEB}+\\angle \\mathrm{DBE}=2 \\angle \\mathrm{DBE}$,\n\n$\\therefore \\angle \\mathrm{CAE}=\\frac{1}{2} \\angle \\mathrm{PCD}, \\angle \\mathrm{DBE}=\\frac{1}{2} \\angle \\mathrm{PDC}$, 即 $\\angle \\mathrm{PAE}=\\frac{1}{2} \\angle \\mathrm{PCD}, \\angle \\mathrm{PBE}=\\frac{1}{2} \\angle \\mathrm{PDC}$,\n\n$\\because \\angle \\mathrm{P}=40^{\\circ}, \\therefore \\angle \\mathrm{PAE}+\\angle \\mathrm{PBE}=\\frac{1}{2} \\angle \\mathrm{PCD}+\\frac{1}{2} \\angle \\mathrm{PDC}=\\frac{1}{2}(\\angle \\mathrm{PCD}+\\angle \\mathrm{PDC})=\\frac{1}{2}\\left(180^{\\circ}-\\angle \\mathrm{P}\\right)=70^{\\circ}$.\n\n故答案选: D."} {"id": "3941", "image": ["5552.jpg", "5553.jpg"], "answer": "A", "solution": "null", "level": "九年级", "question": "如图是某水库大坝横断面示意图. 其中 $\\mathrm{AB} 、 \\mathrm{CD}$ 分别表示水库上下底面的水平线, $\\angle \\mathrm{ABC}=120^{\\circ}$, $\\mathrm{BC}$ 的长是 $50 \\mathrm{~m}$, 则水库大坝的高度 $\\mathrm{h}$ 是\n\n", "options": "A. $25 \\sqrt{3} \\mathrm{~m}$\nB. $25 \\mathrm{~m}$\nC. $25 \\sqrt{2} \\mathrm{~m}$\nD. $\\frac{50 \\sqrt{3}}{3} \\mathrm{~m}$", "subject": "画法几何学", "analysis": "过点 $C$ 作 $C E \\perp A B$ 于点 $\\mathrm{E}$,\n\n\n\n$\\because \\angle \\mathrm{ABC}=120^{\\circ}$,\n\n$\\therefore \\angle \\mathrm{CBE}=60^{\\circ}$ 。\n\n在 Rt $\\triangle \\mathrm{CBE}$ 中, $\\mathrm{BC}=50 \\mathrm{~m}$,\n\n$\\therefore \\mathrm{CE}=\\mathrm{BC} \\cdot \\sin 60^{\\circ}=25 \\sqrt{3}(\\mathrm{~m})$. 故选 A."} {"id": "3309", "image": ["4292.jpg", "4293.jpg", "4294.jpg", "4295.jpg", "4296.jpg"], "answer": "D", "solution": "null", "level": "九年级", "question": "如图, 正方形纸板的一条对角线垂直于地面, 纸板上方的灯(看作一个点)与这条对角线所确定的平面垂直于纸板.在灯光照射下,正方形纸板在地面上形成的影子的形状可以是( )\n\n", "options": "A.\n\n\nB.\n\n\nC.\n\n\nD.\n\n", "subject": "画法几何学", "analysis": "略"} {"id": "3643", "image": ["4988.jpg"], "answer": "B", "solution": "null", "level": "九年级", "question": "将正方体股子(相对面上的点数分别为 1 和 $6 、 2$ 和 $5 、 3$ 和 4 )放置于水平桌面上, 如图 1 在图 2 中, 将股子向右翻滚 $90^{\\circ}$, 然后在桌面上按逆时针方向旋转 $90^{\\circ}$, 则完成一次变换. 若股子的初始位置为图 1所示的状态, 那么按上述规则连续完成 2016 次变换后, 骰子朝上一面的点数是 ( )\n\n", "options": "A. 1\nB. 3\nC. 5\nD. 6", "subject": "变换几何", "analysis": "根据题意可知连续 3 次变换是一循环. 所以 $2016 \\div 3=672$.所以是初始位置的图形, 䛇子朝上一面的点数是 3\n\n故选 $B$."} {"id": "2999", "image": ["3900.jpg", "3901.jpg"], "answer": "D", "solution": "null", "level": "九年级", "question": "(本题 3 分)(2022$\\cdot$浙江金华$\\cdot$九年级期末)如图, 将函数 $y=\\frac{1}{2}(x-2)^{2}+1$ 的图象沿 $y$ 轴向上平移得到一条新函数的图象, 其中点 $A(1, m), B(4, n)$ 平移后的对应点分别为点 $A^{\\prime} 、 B^{\\prime}$. 若曲线段 $A B$ 扫过的面积为 9 (图中的阴影部分), 则新图象的函数表达式是()", "options": "A. $y=\\frac{1}{2}(x-2)^{2}-2$B. $y=\\frac{1}{2}(x-2)^{2}+7$C. $y=\\frac{1}{2}(x-2)^{2}-5$D. $y=\\frac{1}{2}(x-2)^{2}+4$", "subject": "变换几何", "analysis": "D【解析】【分析】连接AB、A^primeB^prime,过A作AC//x轴,交B^primeB的延长线于点C,由平移的性质得四边形ABB^primeA^prime的面积等于阴影部分的面积,由此关系可确定平移的距离,则可求得平移后抛物线的解析式.【详解】because函数y=frac12(x-2)^2+1的图象过点A(1,m),B(4,n),thereforem=frac12(1-2)^2+1=frac32,n=frac12(4-2)^2+1=3,thereforeAleft(1,frac32right),B(4,3),如图,连接AB、A^primeB^prime,过A作AC//x轴,交B^primeB的延长线于点C,则Cleft(4,frac32right),且四边形ABB^primeA^prime是平行四边形,thereforeAC=4-1=3,because曲线段AB扫过的面积为9(图中的阴影部分),therefore阴影部分的面积等于平行四边形ABB^primeA^prime的面积,thereforeACcdotAA^prime=3AA^prime=9,thereforeAA^prime=3,即将函数y=frac12(x-2)^2+1的图象沿y轴向上平移3个单位长度得到一条新函数的图象,therefore新图象的函数表达式是y=frac12(x-2)^2+4.故选:D.【点睛】本题考查了二次函数图象的平移,关键是确定平移的距离,难点是通过割补把不规则图形面积转化为规则图形面积."} {"id": "3054", "image": ["3950.jpg"], "answer": "D", "solution": "null", "level": "九年级", "question": "(本题 3 分) (2021 浙江台州$\\cdot$九年级期末) 如图, $\\triangle D E C$ 是由 $\\triangle A B C$ 绕点 $C$ 顺时针旋转 $30^{\\circ}$ 所得, 边 $D E, A C$ 相交于点 $F$. 若 $\\angle A=35^{\\circ}$, 则 $\\angle E F C$ 的度数为", "options": "A. $50^{\\circ}$B. $55^{\\circ}$C. $60^{\\circ}$D. $65^{\\circ}$", "subject": "变换几何", "analysis": "D【解析】【分析】由旋转的性质可得angleA=angleD=35^circ,angleACD=30^circ,由三角形外角的性质可求解.【详解】解:becausetriangleDEC是由triangleABC绕点C顺时针旋转30^circ所得,thereforeangleA=angleD=35^circ,angleACD=30^circ,thereforeangleEFC=angleD+angleACD=65^circ故选:D.【点睛】本题考查了旋转的性质,三角形的外角性质,掌握旋转的性质是解题的关键."} {"id": "3108", "image": ["4035.jpg"], "answer": "B", "solution": "null", "level": "九年级", "question": "(本题 3 分)(2021$\\cdot$浙江金华一一模)将正方形纸片按图(1)方式依次对折得图(2)的 $\\triangle A B C$, 点 $D$ 是 $A C$ 边上一点, 沿线段 $B D$ 剪开, 展开后得到一个正八边形, 则点 $D$ 应满足 $(\\quad)$", "options": "A. $B D \\perp A C$B. $A D=A B$C. $\\angle A D B=60^{\\circ}$D. $A D=D B$", "subject": "变换几何", "analysis": "B【解析】【分析】根据折叠的性质易得angleBAC=45^circ,然后由正多边形的性质可进行排除选项.【详解】解:由题意得:angleBAC=45^circ,therefore沿线段BD剪开,展开图即为八边形,若使展开后得到的是一个正八边形,则需满足以点A为圆心,AD、AB为半径即可,thereforeAD=AB故选B.【点睛】本题主要考查正多边形和圆、正方形的性质及折叠的性质,熟练掌握正多边形和圆、正方形的性质及折叠的性质是解题的关键."} {"id": "3176", "image": ["4149.jpg"], "answer": "A", "solution": "null", "level": "九年级", "question": "(本题 3 分) (2022-浙江温州$\\cdot$九年级期末) 如图, $\\triangle A B C$ 与 $\\triangle D E F$ 是位似图形, $O$ 为位似中心,位似比为 $2: 3$ 。若 $A B=4$, 则 $D E$ 的长为 $(\\quad)$", "options": "A. 6B. 8C. 9D. 10", "subject": "变换几何", "analysis": "A【分析】位似图形就是特殊的相似图形位似比等于相似比. 利用相似三角形的性质即可求解.【详解】解: $\\because \\triangle A B C$ 与 $\\triangle D E F$ 是位似图形, 位似比为 $2: 3$,$\\therefore A B: D E=2: 3$.$\\because A B=4$,$\\therefore D E=6$.故选: A.【点睛】本题主要考查位似变换. 解题的关键是掌握位似图形是相似图形的特殊形式,位似比等于相似比的特点."} {"id": "0", "image": [], "answer": "C", "solution": "null", "level": "九年级", "question": "若将函数 $y=2 x^{2}$ 的图象向上平移 5 个单位, 再向右平行移动 1 个单位, 得到的拋物线是 ( )\n", "options": "A. $y=2(x+5)^{2}+1$\nB. $y=2(x+5)^{2}-1$\nC. $y=2(x-1)^{2}+5$\nD. $y=2(x-1)^{2}-5$\n", "subject": "变换几何", "analysis": "根据图象右移减、左移加, 上移加、下移减,可得答案.\n\n【详解】原拋物线的顶点为 $(0,0)$, 向右平行移动 1 个单位, 再向上平移 5 个单位, 那么新抛物线的顶点为 $(1,5)$. 可设新抛物线的解析式为 $\\mathrm{y}=2(x-h)^{2}+\\mathrm{k}$, 代入可得: $\\mathrm{y}=2(x-1)$ ${ }^{2}+5$.\n\n故选 C.\n\n【点睛】本题考查了二次函数图象与几何变换, 图象右移减、左移加, 上移加、下移减是解题关键.\n\n"} {"id": "1070", "image": ["11273.jpg", "11274.jpg"], "answer": "A", "solution": "null", "level": "九年级", "question": "如图, 在 $\\triangle A B C$ 中, $\\angle B A C=30^{\\circ}, A B=8, A C=5$, 将 $\\triangle A B C$ 绕点 $\\mathrm{A}$ 顺时针旋转 $30^{\\circ}$ 得到 $\\triangle A D E$ 连接 $C D$, 则 $C D$ 的长是 ( )\n\n\n", "options": "A. 7\nB. 8\nC. 12\nD. 13\n\n", "subject": "变换几何", "analysis": "过点 $\\mathrm{D}$ 作 $D F \\perp A C$ 与 $\\mathrm{F}$, 由旋转的性质可得 $\\mathrm{AD}=\\mathrm{AB}=8, \\angle B A C=\\angle \\mathrm{DAB}=30^{\\circ}$,由直角三角形的性质可得 $\\mathrm{AF}=4, \\quad \\mathrm{DF}=\\sqrt{3} \\mathrm{AF}=4 \\sqrt{3}$, 由勾股定理可求解.\n\n【详解】解:过点 $\\mathrm{D}$ 作 $D F \\perp A C$ 与 $\\mathrm{F}$,\n\n$\\because$ 将 $\\triangle A B C$ 绕点 $\\mathrm{A}$ 顺时针旋转 $30^{\\circ}$ 得到 $\\triangle A D E$,\n\n$\\therefore A D=A B=8, \\angle B A C=\\angle D A B=30^{\\circ}$,\n\n$\\therefore \\angle C A D=60^{\\circ}$, 且 $D F \\perp A C, A D=8$\n\n$\\therefore A F=4, D F=\\sqrt{3} A F=4 \\sqrt{3}$,\n\n$\\therefore C F=1$,\n\n$\\therefore C D=\\sqrt{D F^{2}+C F^{2}}=\\sqrt{48+1}=7$\n\n故选 A.\n\n\n\n【点睛】本题考查了旋转的性质、勾股定理, 添加合适的辅助线构造直角三角形是解题的关键.\n\n"} {"id": "1071", "image": ["11275.jpg"], "answer": "C", "solution": "null", "level": "九年级", "question": "如图, 在 Rt $\\triangle A B C$ 中, $\\angle A C B=90^{\\circ}$, 将 Rt $\\triangle A B C$ 绕点 $C$ 按顺时针方向旋转一定角度得到 Rt $\\triangle D E C$, 点 $D$ 恰好落在边 $A B$ 上若 $\\angle B=25^{\\circ}$, 则 $\\angle B C E$ 的度数为()\n\n\n", "options": "A. $20^{\\circ}$\nB. $40^{\\circ}$\nC. $50^{\\circ}$\nD. $70^{\\circ}$\n\n", "subject": "变换几何", "analysis": "由旋转的性质可得 $A C=C D, \\angle A C D=\\angle B C E$, 可得 $\\angle A=\\angle A D C=65^{\\circ}$, 由三角形内角和定理可得 $\\angle A C D=50^{\\circ}$, 即可求解.\n\n【详解】 $\\because \\angle A C B=90^{\\circ}, \\angle B=25^{\\circ}$,\n\n$\\therefore \\angle A=65^{\\circ}$,\n\n$\\because$ 将 Rt $\\triangle A B C$ 绕点 $C$ 顺时针方向旋转一定角度得到 Rt $\\triangle D E C$,\n\n$\\therefore A C=C D, \\angle A C D=\\angle B C E$,\n\n$\\therefore \\angle A=\\angle A D C=65^{\\circ}$,\n\n$\\therefore \\angle A C D=50^{\\circ}$,\n\n$\\therefore \\angle B C E=50^{\\circ}$,\n\n故选: $C$.\n\n【点睛】本题考查了旋转的性质, 等腰三角形的性质, 掌握旋转的性质是解题的关键.\n\n"} {"id": "1075", "image": ["11280.jpg"], "answer": "A", "solution": "null", "level": "九年级", "question": "如图, 将 $\\triangle A B C$ 绕点 $A$ 逆时针旋转得到 $\\triangle A B^{\\prime} C^{\\prime}$, 若 $B^{\\prime}$ 落在 $B C$ 边上, $\\angle B=50^{\\circ}$, 则 $\\angle C B^{\\prime} C^{\\prime}$ 为 $(\\quad)$\n\n\n", "options": "A. $80^{\\circ}$\nB. $70^{\\circ}$\nC. $60^{\\circ}$\nD. $50^{\\circ}$\n\n", "subject": "变换几何", "analysis": "依据旋转的性质可求得 $A B=A B^{\\prime}, \\angle A B^{\\prime} C^{\\prime}$ 的度数, 依据等边对等角的性质可得到 $\\angle B=\\angle B B^{\\prime} A$, 于是可得到 $\\angle C B^{\\prime} C^{\\prime}$ 的度数.\n\n【详解】解: 由旋转的性质可知: $A B=A B^{\\prime}, \\angle B=\\angle A B^{\\prime} C^{\\prime}=50^{\\circ}$,\n\n$\\because A B=A B^{\\prime}$,\n$\\therefore \\angle B=\\angle B B^{\\prime} A=50^{\\circ}$.\n\n$\\therefore \\angle B B^{\\prime} C^{\\prime}=50^{\\circ}+50^{\\circ}=100^{\\circ}$,\n\n$\\therefore \\angle C B^{\\prime} C^{\\prime}=180^{\\circ}-100^{\\circ}=80^{\\circ}$,\n\n故选: A.\n\n【点睛】本题主要考查的是旋转的性质, 等腰三角形的性质, 求得 $\\angle A B^{\\prime} C^{\\prime}$ 和 $\\angle B B^{\\prime} A$ 的度数是解题的关键.\n\n"} {"id": "1122", "image": [], "answer": "D", "solution": "null", "level": "九年级", "question": "下列说法正确的是( )\n\n", "options": "A. 小红小学毕业时的照片和初中毕业时的照片相似\n\nB. 商店新头来的一副三角板是相似的\n\nC. 所有的课本都是相似的\n\nD. 国旗的五角星都是相似的\n\n", "subject": "变换几何", "analysis": "观察图形,看它们的形状是否相同,形状相同的两个图形是相似图形.\n\n【详解】A. 小明上幼儿园时的照片和初中毕业时的照片, 形状不相同, 不相似;\n\nB. 商店新买来的一副三角板, 形状不相同, 不相似;\n\nC. 所有的课本都是相似的, 形状不相同, 不相似;\n\nD. 国旗的五角星都是相似的,形状相同,相似.\n\n故选 D.\n\n【点睛】本题考查了相似图形, 相似图形是指形状相同的图形, 仔细观察看每组图形是否相同,如果相同就相似,否则就不相似.\n\n"} {"id": "1123", "image": ["11328.jpg"], "answer": "D", "solution": "null", "level": "九年级", "question": "如图, $\\triangle A B C$ 和 $\\triangle A^{\\prime} B^{\\prime} C^{\\prime}$ 位似, 位似中心为点 $O$, 点 $A(-1,2)$ 、点 $A^{\\prime}(2,-4)$, 若 $\\triangle A B C$ 的面积为 4 , 则 $\\triangle A^{\\prime} B^{\\prime} C^{\\prime}$ 的面积是 ( )\n\n\n", "options": "A. 2\nB. 4\nC. 8\nD. 16\n\n", "subject": "变换几何", "analysis": "直接利用位似图形对应点坐标得出相似比, 进而利用相似三角形的性质得出答案.\n\n【详解】解: $\\because \\triangle A B C$ 和 $\\triangle A^{\\prime} B^{\\prime} C^{\\prime}$ 位似, 位似中心为原点 $O$, 点 $A(-1,2)$ 、点 $A^{\\prime}(2,-4)$, $\\therefore \\triangle A B C$ 和 $\\triangle A^{\\prime} B^{\\prime} C^{\\prime}$ 的相似比为: $1: 2$,\n\n$\\therefore \\triangle A B C$ 和 $\\triangle A^{\\prime} B^{\\prime} C^{\\prime}$ 的面积比为: $1: 4$,\n\n$\\because \\triangle A B C$ 的面积为 4 ,\n\n$\\therefore \\triangle A^{\\prime} B^{\\prime} C^{\\prime}$ 的面积是: 16 .\n\n故选: D.\n\n【点睛】此题主要考查了位似变换, 正确得出相似比是解题关键.\n\n"} {"id": "3022", "image": [], "answer": "B", "solution": "null", "level": "九年级", "question": "(本题 3 分)(2022:浙江宁波$\\cdot$九年级期末)下列事件中是不可能事件的是()", "options": "A. 从一副扑克牌中任抽一张牌恰好是“红桃”B. 在装有白球和黑球的袋中摸球,摸出了红球C. 2022 年大年初一早晨艳阳高照D. 从两个班级中任选三名学生, 至少有两名学生来自同一个班级", "subject": "计数", "analysis": "B【解析】【分析】根据随机事件,必然事件,不可能事件的特点判断即可.【详解】解:A.从一副扑克牌中任抽一张牌恰好是“红桃”,这是随机事件,故A不符合题意;B.在装有白球和黑球的袋中摸球,摸出了红球,这是不可能事件,故B符合题意;C.2022年大年初一早晨绝阳高照,这是随机事件,故C不符合题意;D.从两个班级中任选三名学生,至少有两名学生来自同一个班级,这是必然事件,故D不符合题意;故选:B.【点睛】本题考查了随机事件,熟练掌握随机事件,必然事件,不可能事件的特点是解题的关键."} {"id": "3023", "image": [], "answer": "C", "solution": "null", "level": "九年级", "question": "(本题 3 分)(2021$\\cdot$浙江$\\cdot$宁波市镇海蛟川书院九年级期中)育种小组对某品种小麦发芽情况进行测试,在测试基本情况相同的条件下,得到如下数据:| 抽查小
麦粒数 | 100 | 500 | 1000 | 2000 | 3000 | 4000 || :--- | :--- | :--- | :--- | :--- | :--- | :--- || 发芽粒
数 | 95 | 486 | 968 | 1940 | 2907 | $a$ |则 $a$ 的值最有可能是)", "options": "A. 3680B. 3720C. 3880D. 3960", "subject": "计数", "analysis": "C【解析】【分析】分别计算出每一次抽取样本的发芽率,从而判断出小麦的发芽的频率稳定在0.97左右,从而得出答案.【详解】解:95div100=0.95,486div500=0.972,968div1000=0.9681940div2000=0.972907div3000=0.969由抽取的样本数据,我们发现小麦发芽的频率稳定在0.97左右,即用频率估计概率,我们可估计小麦发芽的概率为0.97,所以,a=4000times0.97=3880,所以,a最有可能为3880,故选:C.【点睛】本题考查了统计与概率,解题的关键是用频率估计概率以及对频率计算公式的理解."} {"id": "3032", "image": [], "answer": "B", "solution": "null", "level": "九年级", "question": "(本题 3 分) (2022- 浙江绍兴$\\cdot$九年级期末) 在一个不透明的箱子里放有 5 个球, 其中 2 个红球, 3 个白球, 它们除颜色外其余都相同. 从箱子里任意摸出 1 个球, 摸到红球的概率是", "options": "A. $\\frac{1}{5}$B. $\\frac{2}{5}$C. $\\frac{3}{5}$D. $\\frac{2}{3}$", "subject": "计数", "analysis": "B【解析】【分析】直接利用概率公式:由红球的数量除以球的总数即可求解.【详解】解:because一个不透明的箱子里有5个球,其中2个红球,3个白球,therefore从箱子中随机摸出一个球是红球的概率是:frac25.故选:B.【点睛】本题考查了概率公式的应用.用到的知识点为:概率=所求情况数与总情况数之比."} {"id": "3036", "image": ["3930.jpg"], "answer": "C", "solution": "null", "level": "九年级", "question": "(本题 3 分)(2022$\\cdot$浙江温州$\\cdot$九年级期末)如图是一个游戏转盘. 自由转动转盘,当转盘停止转动后, 指针落在数字 $1,2,3,4$ 所示区域内可能性最大的是 ( )", "options": "A. 1 号B. 2 号C. 3 号D. 4 号", "subject": "计数", "analysis": "C【解析】【分析】根据圆周角可得1区域的圆心角度数,然后计算各个区域的可能性,比较大小即可得.【详解】解:1区域的圆心角为:360^circ-50^circ-125^circ-65^circ=120^circ,therefore落在1区域的可能性为:frac120^circ360^circ=frac13,落在2区域的可能性为:frac50^circ360^circ=frac536,落在3区域的可能性为:frac125^circ360^circ=frac2572,落在4区域的可能性为:frac65^circ360^circ=frac1372,becausefrac536", "options": "A. 朝上的点数是 5 的概率B. 朝上的点数是奇数的概率C. 朝上的点数大于 2 的概率D. 朝上的点数是 3 的倍数的概率", "subject": "计数", "analysis": "D【解析】【分析】计算出各个选项中事件的概率,根据概率即可作出判断.【详解】A、朝上的点数是5的概率为frac16approx16.67%,不符合试验的结果;B、朝上的点数是奇数的概率为frac36=frac12=50%,不符合试验的结果;C、朝上的点数大于2的概率frac46approx66.67%,不符合试验的结果;D、朝上的点数是3的倍数的概率是frac26approx33.33%,基本符合试验的结果.故选:D.【点睛】本题考查了频率估计概率,当试验的次数较多时,频率稳定在某一固定值附近,这个固定值即为概率."} {"id": "3042", "image": ["3933.jpg"], "answer": "B", "solution": "null", "level": "九年级", "question": "(本题 3 分) (2018$\\cdot$浙江杭州$\\cdot$九年级期末) 甲、乙两位同学在一次用频率去估计概率的实验中统计了某一结果出现的频率, 绘出的统计图如图所示, 则符合这一结果的实验可能是 $(\\quad)$", "options": "A. 掷一枚正六面体的股子, 出现 1 点的概率B. 一个袋子中有 2 个白球和 1 个红球, 从中任取一个球, 则取到红球的概率C. 抛一枚硬币, 出现正面的概率D. 任意写一个整数, 它能被 2 整除的概率", "subject": "计数", "analysis": "B【解析】【分析】根据统计图可知,试验结果在0.33附近波动,即其概率Papprox0.33,计算四个选项的概率,约为0.33者即为正确答案.【详解】解:A、郑一枚正六面体的股子,出现1点的概率为frac16,故此选项不符合题意;B、一个袋子中有2个白球和1个红球,从中任取一个球,则取到红球的概率frac13approx0.33,故此选项符合题意;C、掷一枚硬币,出现正面朝上的概率为frac12,故此选项不符合题意;D、任意写出一个整数,能被2整除的概率为frac12,故此选项不符合题意.故选:B.【点睛】此题考查了利用频率估计概率,大量反复试验下频率稳定值即概率.用到的知识点为:频率=所求情况数与总情况数之比.同时此题在解答中要用到概率公式."} {"id": "1232", "image": ["11409.jpg"], "answer": "D", "solution": "null", "level": "九年级", "question": "不透明袋子中装有无差别的两个小球,分别写有“问天”和“梦天”. 随机取出一个小球后,放回并摇匀,再随机取出一个小球,则两次都取到写有“问天”的小球的概率为 ( )\n", "options": "A. $\\frac{3}{4}$\nB. $\\frac{1}{2}$\nC. $\\frac{1}{3}$\nD. $\\frac{1}{4}$\n", "subject": "计数", "analysis": "画树状图, 共有 4 种等可能的结果, 两次都取到写有“问天”的小球的结果有 1 种,再由概率公式求解即可.\n\n【详解】解: 设“问天”为 1 , “梦天”为 2 , 画树状图如图:\n\n\n\n共有 4 种等可能的结果,两次都取到写有“问天”的小球的结果有 1 种, $\\therefore$ 两次都取到写有“问天”的小球的概率为 $\\frac{1}{4}$,\n\n故选: D.\n\n【点睛】此题考查的是用列表法或树状图法求概率. 列表法可以不重复不遗漏的列出所有可能的结果, 适合于两步完成的事件; 树状图法适合两步或两步以上完成的事件; 解题时要注意此题是放回试验还是不放回试验. 用到的知识点为: 概率 $=$ 所求情况数与总情况数之比.\n\n"} {"id": "1233", "image": [], "answer": "C", "solution": "null", "level": "九年级", "question": "下列说法中正确的是 ( )\n\n", "options": "A. 要了解一批灯泡的使用寿命应采用普方式\n\nB. “任意画出一个等边三角形, 它是轴对称图形”是随机事件\n\nC. 有一组数据: $3,5,7,6,4$, 这组数据的中位数是 5 .\n\nD. 任意掷一枚质地均匀的硬币 10 次, 正面向上的一定是 5 次\n\n", "subject": "计数", "analysis": "分析:利用随机事件和必然事件的定义对个选项进行判断得出答案.\n\n详解: A.要了解一批灯泡的使用寿命应抽样调查, 故此选项错误;\n\nB. “任意画出一个等边三角形, 它是轴对称图形”是必然事件, 故此选项错误;\n\nC. 有一组数据: $3,5,7,6,4$, 这组数据的中位数是 5, 故此选项正确;\n\nD.任意掷一枚质地均匀的硬币 10 次,正面向上的不一定是 5 次, 故此选项错误.\n\n点睛:此题主要考查了随机事件,正确把握相关事件的定义是解题的关键.\n\n"} {"id": "1234", "image": [], "answer": "D", "solution": "null", "level": "九年级", "question": "试验中, 随着次数的逐渐增多, 事件发生频率的变化规律是 ( )\n\n", "options": "A. 波澜起伏\n\nB. 风平浪静\n\nC. 先风平浪静后波澜起伏\n\nD. 先波澜起伏后风平浪静\n\n", "subject": "计数", "analysis": "根据大量重复试验事件发生的频率逐渐稳定到某个常数附近,可以用这个常数估计这个事件发生的概率解答.\n\n【详解】解: $\\because$ 大量重复试验事件发生的频率逐渐稳定到某个常数附近, 可以用这个常数估计这个事件发生的概率,\n\n$\\therefore$ 事件发生频率的变化规律是先波澜起伏后风平浪静;\n\n故选择: D.\n\n【点睛】本题考查了利用频率估计概率的知识,大量重复试验事件发生的频率逐渐稳定到某个常数附近, 可以用这个常数估计这个事件发生的概率.\n"} {"id": "1235", "image": [], "answer": "C", "solution": "null", "level": "九年级", "question": "下列说法正确的是 ( )\n\n", "options": "A. “翻开九年上册数学课本, 恰好是第 88 页”是不可能事件\n\nB. “太阳从西方升起”是必然事件\n\nC. “明天会下雨”描述的事件是随机事件\n\nD. 射击运动员射击一次, 命中十环是必然事件\n\n", "subject": "计数", "analysis": "根据确定事件和随机事件的定义来区分判断即可, 必然事件和不可能事件统称确定\n性事件; 必然事件: 在一定条件下, 一定会发生的事件称为必然事件; 不可能事件: 在一定条件下, 一定不会发生的事件称为不可能事件; 随机事件: 在一定条件下, 可能发生也可能不发生的事件称为随机事件.\n\n【详解】A. “翻开九年上册数学课本, 恰好是第 88 页”是随机事件, 故该选项不正确, 不符合题意;\n\nB. “太阳从西方升起”是不可能事件, 故该选项不正确, 符合题意;\n\nC. “明天会下雨”描述的事件是随机事件, 故该选项正确, 不符合题意;\n\nD. 射击运动员射击一次, 命中十环是随机事件, 故该选项不正确, 不符合题意;\n\n故选: C.\n\n【点睛】本题考查了确定事件和随机事件的定义,熟悉定义是解题的关键.\n\n"} {"id": "1236", "image": [], "answer": "C", "solution": "null", "level": "九年级", "question": "在一个不透明的盒子中装有 2 个白球, 若干个黄球, 它们除了颜色不同外, 其余均相同.若从中随机摸出一个白球的概率是 $\\frac{1}{3}$, 则黄球的个数为\n", "options": "A. 2\nB. 3\nC. 4\nD. 6\n", "subject": "计数", "analysis": "试题分析: 设黄球的个数为 $\\mathrm{x}$ 个, 根据题意得: $\\frac{12}{12+x}=\\frac{1}{3}$, 解得: $\\mathrm{x}=24$, 经检验: $\\mathrm{x}=24$ 是原分式方程的解; $\\therefore$ 黄球的个数为 24 . 故选 C.\n\n考点: 概率公式.\n\n"} {"id": "1238", "image": [], "answer": "B", "solution": "null", "level": "九年级", "question": "下列事件中, 属于随机事件的是 ( )\n\n", "options": "A. 掷一枚普通正六面体骰子所得点数不超过 6\n\nB. 买一张体育彩票中奖;\n\nC. 太阳从西边落下;\n\nD. 口袋中装有 10 个红球, 从中摸出一个白球\n\n", "subject": "计数", "analysis": "解:A. 是必然事件, 故本选项错误;\n\nB. 是随机事件, 故本选项正确;\n\nC. 是必然事件, 故本选项错误;\n\nD. 是随机事件, 故本选项正确.\n故选 B.\n\n点睛: 本题考查的是随机事件, 事件分为确定事件和不确定事件 (随机事件), 确定事件又分为必然事件和不可能事件.\n\n"} {"id": "1241", "image": [], "answer": "D", "solution": "null", "level": "九年级", "question": "下列说法错误的是()\n\n", "options": "A. 必然事件的概率为 1\n\nB. 数据 $1 、 2 、 2 、 3$ 的平均数是 2\n\nC. 连续掷一枚硬币, 若 5 次都是正面朝上, 则第六次仍然可能正面朝上\n\nD. 如果某种活动的中奖率为 $40 \\%$, 那么参加这种活动 10 次必有 4 次中奖\n\n", "subject": "计数", "analysis": "必然事件: 在一定的条件下重复进行试验时, 有的事件在每次试验中必然会发生,这样的事件叫必然事件, 概率为 1 .\n\n随机事件:在一定条件下,可能发生也可能不发生的事件为随机事件.\n\n概率: 某事件在一次观测或实验中出现的可能性的大小或机会.\n\n平均数:所有数据之和再除以数据的个数.\n\n【详解】A. 概率值反映了事件发生的机会的大小, 必然事件是一定发生的事件, 所以概率为 1 , 本项正确;\n\nB. 数据 $1 、 2 、 2 、 3$ 的平均数是 $(1+2+2+3) \\div 4=2$, 本项正确;\n\nC. 掷硬币属于随机事件, 可能正面向上,也可能反面向上.所以无论第几次掷, 都有可能正面向上, 故本项正确;\n\nD. 某种游戏活动的中奖率为 $40 \\%$, 属于随机事件, 可能中奖, 也可能不中奖, 故本说法错误.\n\n【点睛】此题主要考查查了概率的意义, 求算术平均数的方法, 比较简单.\n\n"} {"id": "1242", "image": ["11411.jpg"], "answer": "A", "solution": "null", "level": "九年级", "question": "从 $-1,2,-3$ 三个数中, 随机抽取两个数相乘, 积是正数的概率是()\n", "options": "A. $\\frac{1}{3}$\nB. $\\frac{2}{3}$\nC. $\\frac{1}{6}$\nD. 1\n", "subject": "计数", "analysis": "首先根据题意画出树状图, 然后由树状图求得所有等可能的结果与积是正数的情况,再利用概率公式即可求得答案.\n\n【详解】解:画树状图如下:\n\n\n\n共有 6 种情况,积是正数的有 2 种情况,\n\n所以, $\\mathrm{P}$ (积是正数) $=\\frac{2}{6}=\\frac{1}{3}$,\n\n故选: A.\n\n【点睛】考查了列表法与树状图法, 本题用到的知识点为: 概率=所求情况数与总情况数之比.\n\n"} {"id": "1244", "image": [], "answer": "B", "solution": "null", "level": "九年级", "question": "口袋中有 14 个红球和若干个白球,这些球除颜色外都相同,从口袋中随机摸出一个球,记下颜色后放回,多次实验后发现摸到白球的频率稳定在 0.3 , 则白球的个数是 ( )\n", "options": "A. 5\nB. 6\nC. 7\nD. 8\n", "subject": "计数", "analysis": "设白球的个数为 $x$, 利用概率公式即可求得.\n\n【详解】设白球的个数为 $x$,\n\n由题意得, 从 14 个红球和 $x$ 个白球中, 随机摸出一个球是白球的概率为 0.3 ,\n\n则利用概率公式得: $\\frac{x}{14+x}=0.3$,\n\n解得: $x=6$,\n\n经检验, $x=6$ 是原方程的根,\n\n故选: B.\n\n【点睛】本题考查了等可能下概率的计算,理解题意利用概率公式列出等式是解题关键.\n\n"} {"id": "3039", "image": ["3932.jpg"], "answer": "B", "solution": "null", "level": "九年级", "question": "(本题 3 分) (2021 浙江绍兴$\\cdot$九年级期中) 如图, 电路图上有 4 个开关 $A, B, C$, $D$ 和 1 个小灯泡, 同时闭合开关 $A, B$ 或同时闭合开关 $C, D$ 都可以使小灯泡发光. 下列操作中,“小灯泡发光”这个事件是随机事件的是()", "options": "A. 只闭合 1 个开关B. 只闭合 2 个开关C. 只闭合 3 个开关D. 闭合 4 个开关", "subject": "逻辑题", "analysis": "B【解析】【分析】根据随机事件的定义,即可能发生也可能不发生的事件称为随机事件,进行判断即可.【详解】A、“只闭合1个开关”小灯泡不可能发光,则“小灯泡发光”这个事件是不可能事件,此项不符合题意;B、“只闭合2个开关”小灯泡有可能发光(如闭合A,B),也有可能不发光(如闭合A,C),则“小灯泡发光”这个事件是随机事件,此项符合题意;C、“只闭合3个开关”小灯泡一定发光,则“小灯泡发光”这个事件是必然事件,此项不符合题意;D、“闭合4个开关”小灯泡一定发光,则“小灯泡发光”这个事件是必然事件,此项不符合题意.故选:B.【点睛】本题考查了随机事件、必然事件、不可能事件,掌握各事件的定义是解题的关键."} {"id": "1237", "image": [], "answer": "C", "solution": "null", "level": "九年级", "question": "下列说法正确的是 ( )\n", "options": "A. 可能性很大的事情是必然发生的\nB. 可能性很小的事情是不可能发生的\nC. 如果圆的半径为 $r$, 则该圆的面积为 $\\pi r^{2}$ 是必然的\nD. 冬季里下雪是一定发生的\n", "subject": "逻辑题", "analysis": "根据必然事件, 不可能事件, 随机事件的定义进行解答即可.\n\n【详解】解:依次分析选项可得:\n\nA 、可能性很大的事情是随机事件, 不一定发生, 错误;\n\n$B 、$ 可能性很小的事情是随机事件, 也有可能发生, 错误;\n\n$C 、$ 根据圆的面积公式, 易得其正确;\n\n$D$ 、冬季里下雪是可能性很大的事情, 不一定发生, 错误;\n\n故选: C.\n\n【点睛】本题主要考查了必然事件, 不可能事件, 随机事件的定义, 能够正确判断每个事件是解题关键.\n\n"} {"id": "3040", "image": [], "answer": "B", "solution": "null", "level": "九年级", "question": "(本题 3 分)(2022$\\cdot$浙江温州$\\cdot$九年级期末)某班从 4 名男生和 2 名女生中任选 1 人参加“我的数学故事”演讲比赛, 则选中女生的概率是", "options": "A. $\\frac{1}{2}$B. $\\frac{1}{3}$C. $\\frac{1}{4}$D. $\\frac{2}{3}$", "subject": "组合数学", "analysis": "B【解析】【分析】让女生的人数除以学生总数即为所求的概率.【详解】某班从4名男生和2名女生中任选1人参加“我的数学故事”演讲比赛,则选中女生的概率是frac24+2=frac13,故B正确.故选:B.【点睛】本题主要考查了概率公式的应用,明确概率的意义是解答的关键,用到的知识点为:概率等于所求情况数与总情况数之比."} {"id": "3041", "image": [], "answer": "B", "solution": "null", "level": "九年级", "question": "(本题 3 分)(2022$\\cdot$浙江睤州 $\\cdot$ 九年级期末)某班甲、乙、丙三个综合实践活动小组准备向全班同学展示成果, 现通过抽签确定三个小组展示的先后顺序. 三个小组排列的顺序有( $\\quad )$ 种不同可能.", "options": "A. 3B. 6C. 9D. 12", "subject": "组合数学", "analysis": "B【解析】【分析】根据题意列举事件所有的情况即可.【详解】解:由题意知:有甲乙丙,甲丙乙,乙丙甲,乙甲丙,丙甲乙,丙乙甲,共6种可能;故选B.【点睛】本题考查了列举法.解题的关键在于列举所有的情况."} {"id": "3569", "image": ["4861.jpg"], "answer": "A", "solution": "null", "level": "九年级", "question": "如图, 已知一个正方体的六个面上分别写着 6 个连续整数, 且相对面上两个数的和相等,图中所能看到的数是 1,3 和 4 , 则这 6 个整数的和是 $(\\quad)$\n\n", "options": "A. 9\nB. 9 或 15\nC. 15 或 21\nD. 9,15 或 21", "subject": "组合数学", "analysis": "【分析】\n\n此题考查了空间图形, 主要培养学生的观察能力和空间想象能力, 由平面图形的折叠及立体图形的表面展开图的特点解题\n\n【解答】\n\n根据题意分析可得: 六个面上分别写着六个连续的整数,\n\n故六个整数可能为 $1 、 2 、 3 、 4 、 5 、 6$ 或 $0 、 1 、 2 、 3 、 4 、 5$ 或 $-1 、 0 、 1 、 2 、 3 、 4$;\n\n当这几个数为 $1 、 2 、 3 、 4 、 5 、 6$ 时, $3+4=7$ 在相邻的两个面上, 所以此时相对的面两个数的和不相等;\n\n当这几个数为 $0 、 1 、 2 、 3 、 4 、 5$ 时, $1+4=5$ 在相邻的两个面上, 所以此时相对的面两个数的和不相等;\n\n故只可能为 $-1 、 0 、 1 、 2 、 3 、 4$ 其和为 9.\n\n故选 $A$."} {"id": "1239", "image": ["11410.jpg"], "answer": "A", "solution": "null", "level": "九年级", "question": "如图是小鹏自己制作的正方形飞镖盘, 并在盘内画了两个小正方形, 则小鹏在投掷飞镖时, 飞镖扎在阴影部分的概率为( )\n\n\n", "options": "A. $\\frac{1}{4}$\nB. $\\frac{1}{5}$\nC. $\\frac{3}{8}$\nD. $\\frac{1}{3}$\n\n", "subject": "组合数学", "analysis": "根据概率公式计算可得: 飞镖扎在阴影部分的概率是 $\\frac{1}{4}$, 故选 A.\n\n"} {"id": "1240", "image": [], "answer": "A", "solution": "null", "level": "九年级", "question": "5 月 9 号重庆实验外国语学校就行了“五四表彰大会”, 初三某班老师准备从包括小明在内的四名优秀团员中, 随机抽取了 2 名学生参加表彰大会, 则抽到小明的概率是 ( )\n", "options": "A. $\\frac{1}{2}$\nB. $\\frac{1}{3}$\nC. $\\frac{1}{4}$\nD. $\\frac{1}{6}$\n", "subject": "组合数学", "analysis": "试题分析: $\\because$ 初三某班老师准备从包括小明在内的四名优秀团员中, 随机抽取了 2 名学生参加表彰大会,\n\n$\\therefore$ 抽到小明的概率是: $\\frac{2}{4}=\\frac{1}{2}$. 故选 A.\n\n【考点】概率公式.\n\n"} {"id": "1250", "image": [], "answer": "A", "solution": "null", "level": "九年级", "question": "布袋中有除颜色外完全相同的 5 个红球, 2 个黄球,3 个白球,从布袋中同时随机摸出两个球都是红球的概率为 ( $)$\n", "options": "A. $\\frac{2}{9}$\nB. $\\frac{1}{4}$\nC. $\\frac{17}{18}$\nD. $\\frac{1}{2}$\n", "subject": "组合数学", "analysis": "分两步进行,第一步来摸球一共 10 球摸出一个共 10 种,再从剩下 9 个球再摸一个,所有情况有 $10 \\times 9$ 种情况, 按同样的方法摸红球, 用概率公式计算, 两次摸出红球情况除以所有情况即可.\n\n【详解】由题可得:第一次描出 10 球之一共有十种,再从剩下的 9 个球中摸出一个球有 9 种情况, 一共有 $10 \\times 9=90$ 种情况, 其中第一次描出 5 个红球之一共有 5 种, 再从剩下的 4 个红球中摸出一个球,两个球都是红球的有 $5 \\times 4=20$ 种情况,\n\n因此摸出的两球都是红球的概率是 $=\\frac{20}{90}=\\frac{2}{9}$.\n\n故选择: A.\n\n【点睛】本题考查概率问题, 掌握概率的概念与公式, 会用树状图的思想分步解决所有情况,第一层有 10 个球即 10 种情况, 画 10 个分支, 第二层还有 9 个球即 9 种情况, 在每个分支的基础上再画 9 个小分支故可知所有情况是解题关键.\n\n"} {"id": "1251", "image": [], "answer": "B", "solution": "null", "level": "九年级", "question": "已知事件:(1)郑一次股子,向上一面的点数是偶数; (2)在 13 位同学中至少有 2 人生肖相同;(3)若彩票中奖率 10\\%,那么买 10 张彩票一定中奖;(4)任意画一个三角形,其内角和为 $360^{\\circ}$ ,其中随机事件是()\n", "options": "A. (1)(2)\nB. (1)(3)\nC. (2) (4)\nD. (3)4)\n", "subject": "组合数学", "analysis": "根据随机事件、必然事件以及不可能事件的定义即可作出判断.\n\n【详解】解:随机事件:(1)(3);\n\n必然事件: (2);\n不可能事件: (4).\n\n故选: B.\n\n【点睛】此题主要考查事件的分类, 解题的关键是熟知随机事件、必然事件以及不可能事件的定义.\n\n"} {"id": "3154", "image": [], "answer": "B", "solution": "null", "level": "九年级", "question": "(本题 3 分) (2021 浙江$\\cdot$翠苑中学九年级期中) 下列每个选项中的两个图形一定相似的是", "options": "A. 任意两个矩形B. 任意两个正五边形C. 任意两个平行四边形D.任意两个等腰三角形", "subject": "图论", "analysis": "B【分析】根据题意,任意正多边形是相似图形,其余不是,据此分析即可.【详解】A.任意两个矩形,不一定相似,故该选项不正确,不符合题意;B.任意两个正五边形,一定相似,故该选项正确,符合题意;C.任意两个平行四边形,不一定相似,故该选项不正确,不符合题意;D.任意两个等腰三角形,不一定相似,故该选项不正确,不符合题意;故选B【点睛】本题考查了相似图形的判定,理解任意正多边形是相似图形是解题的关键."} {"id": "3367", "image": [], "answer": "B", "solution": "null", "level": "九年级", "question": "在实数 $\\pi, \\frac{1}{3}, \\sqrt{2}, \\sin 30^{\\circ}$ 中, 无理数的个数为 $(\\quad)$", "options": "A. 1\nB. 2\nC. 3\nD. 4", "subject": "算术", "analysis": "先把 $\\sin 30^{\\circ}$ 化为 $\\frac{1}{2}$ 的形式, 再根据无理数的定义进行解答."} {"id": "1114", "image": [], "answer": "C", "solution": "null", "level": "九年级", "question": "已知 $2 x=3 y$, 则下列比例式成立的是()\n", "options": "A. $\\frac{x}{2}=\\frac{y}{3}$\nB. $\\frac{x}{y}=\\frac{2}{3}$\nC. $\\frac{x}{3}=\\frac{y}{2}$\nD. $\\frac{x+y}{x}=\\frac{5}{2}$\n", "subject": "算术", "analysis": "依据比例的性质, 将各选项变形即可得到正确结论.\n\n【详解】解: A. 由 $\\frac{x}{2}=\\frac{y}{3}$ 可得, $2 \\mathrm{y}=3 \\mathrm{x}$, 不合题意;\n\nB. 由 $\\frac{x}{y}=\\frac{2}{3}$ 可得, $2 y=3 x$, 不合题意;\n\nC. 由 $\\frac{x}{3}=\\frac{y}{2}$ 可得, $3 \\mathrm{y}=2 \\mathrm{x}$, 符合题意;\n\nD. 由 $\\frac{x+y}{x}=\\frac{5}{2}$ 可得, $3 \\mathrm{x}=2 \\mathrm{y}$, 不合题意;\n\n故选: C.\n\n【点睛】本题主要考查了比例的性质, 解决问题的关键是掌握: 内项之积等于外项之积.\n\n"} {"id": "1127", "image": [], "answer": "C", "solution": "null", "level": "九年级", "question": "由 $\\frac{x}{y}=\\frac{2}{3}$ 不能推出的比例式是 ( )\n", "options": "A. $\\frac{x}{2}=\\frac{y}{3}$\nB. $\\frac{x+y}{y}=\\frac{5}{3}$\nC. $\\frac{x-y}{y}=\\frac{1}{3}$\nD. $\\frac{x+2}{y+3}=\\frac{2}{3}(y \\neq-3)$\n", "subject": "算术", "analysis": "根据比例的性质依次判断即可.\n\n【详解】设 $\\mathrm{x}=2 \\mathrm{a}, \\mathrm{y}=3 \\mathrm{a}$,\n\nA. $\\frac{x}{2}=\\frac{y}{3}$ 正确, 不符合题意;\n\nB. $\\frac{x+y}{y}=\\frac{2 a+3 a}{3 a}=\\frac{5}{3}$, 故该项正确, 不符合题意;\n\nC. $\\frac{x-y}{y}=\\frac{2 a-3 a}{3 a}=-\\frac{1}{3}$, 故该项不正确, 符合题意;\n\nD. $\\frac{x+2}{y+3}=\\frac{2 a+2}{3 a+3}=\\frac{2(a+1)}{3(a+1)}=\\frac{2}{3}(y \\neq-3)$ 正确, 不符合题意;\n\n【点睛】此题考查比例的基本性质, 熟记性质并运用解题是解此题的关键.\n\n"} {"id": "1243", "image": [], "answer": "D", "solution": "null", "level": "九年级", "question": "某气象局预报称:“明天本市的降水概率为 $70 \\%$ ”.这句话指的是()\n\n", "options": "A. 明天本市 $70 \\%$ 的时间下雨, $30 \\%$ 的时间不下雨\n\nB. 明天本市 $70 \\%$ 的地方下雨, $30 \\%$ 的地方不下雨\n\nC. 明天本市一定下雨\n\nD. 明天本市下雨的可能性是 $70 \\%$\n\n", "subject": "统计数学", "analysis": "降水概率是指降水这件事的可能性, 根据概率的意义作答即可.\n\n【详解】明天本市的降水概率为 $70 \\%$, 这句话指的是明天本市下雨的可能性是 $70 \\%$.\n\n故选: D.\n\n【点睛】本题主要考查了概率的意义, 概率是反映事件发生的可能性大小.\n\n"} {"id": "1246", "image": [], "answer": "D", "solution": "null", "level": "九年级", "question": "下列事件中,属于不可能事件的是()\n\n", "options": "A. 明天某地区早晨有雾\n\nB. 抛掷一枚质地均匀的散子, 向上一面的点数是 6\n\nC. 明天见到的第一辆公交车的牌照的末位数字是偶数\n\nD. 一只不透明的袋子中有 2 个红球和 1 个白球, 从中摸出 1 个球, 该球是黄球\n\n", "subject": "统计数学", "analysis": "不可能事件就是一定不会发生的事件, 依据这一定义即可做出判断.\n\n【详解】解: $A$ 、明天某地区早晨有雾, 是随机事件, 本选项不符合题意;\n\n$B$ 、抛掷一枚质地均匀的散子, 向上一面的点数是 6 , 是随机事件, 本选项不符合题意;\n\n$C$ 、明天见到的第一辆公交车的牌照的末位数字是偶数, 是随机事件, 本选项不符合题意;\n\n$D$ 、一只不透明的袋子中有 2 个红球和 2 个白球, 从中摸出 1 个球, 该球是黄球,是不可能事件, 故本选项符合题意.\n\n故选: D.\n\n【点睛】本题考查不可能事件的定义, 解题的关键是正确理解必然事件、不可能事件、随机事件的概念,不可能事件是指在一定条件下,一定不会发生的事件.\n\n"} {"id": "13154", "image": [], "answer": "B", "solution": "null", "level": "六年级", "question": "把一个圆柱的侧面展开, 不可能得到的是 ( )。", "options": "A. 长方形\nB. 三角形\nC. 平行四边形\nD. 正方形", "subject": "立体几何学", "analysis": "答案: $\\mathrm{B}$"} {"id": "13041", "image": [], "answer": "A", "solution": "null", "level": "六年级", "question": "将圆柱的侧面展开, 将得不到 $(\\quad)$", "options": "A. 平行四边形\nB. 长方形\nC. 梯形\nD. 正方形", "subject": "立体几何学", "analysis": "答案: $A$"} {"id": "13047", "image": ["2742.jpg"], "answer": "B", "solution": "null", "level": "六年级", "question": "一个底面积是 $20 \\mathrm{~cm}^{2}$ 的圆柱, 斜着截去了一段后, 剩下的图形如下图。截后剩下的图形的体积是 ( ) $\\mathrm{cm}^{3}$ 。\n\n", "options": "A. 140\nB. 180\nC. 220\nD. 360", "subject": "立体几何学", "analysis": "答案: $\\mathrm{B}$"} {"id": "13053", "image": ["2743.jpg", "2744.jpg", "2745.jpg"], "answer": "D", "solution": "null", "level": "六年级", "question": "下列立体图形中,截面形状不可能是长方形的是()。", "options": "A.\n\n![](本地图片-4695\\.jpg)\nB.\n\n\nC.\n\n\nD.\n\n", "subject": "立体几何学", "analysis": "D"} {"id": "13055", "image": ["2746.jpg", "2747.jpg", "2748.jpg", "2749.jpg"], "answer": "C", "solution": "null", "level": "六年级", "question": "在下图中, 以直线为轴旋转, 可以得到圆柱体的是( )", "options": "A.\n\n\nB.\n\n\nC.\n\n\n\n", "subject": "立体几何学", "analysis": "答案: $\\mathrm{C}$"} {"id": "13056", "image": [], "answer": "C", "solution": "null", "level": "六年级", "question": "圆柱的底面半径扩大到原来的 2 倍,高也扩大到原来的 2 倍,体积()", "options": "A. 扩大到原来的 4 倍\nB. 不变\nC. 扩大到原来的 8 倍\nD. 不能确定", "subject": "立体几何学", "analysis": "C"} {"id": "27007", "image": [], "answer": "B", "solution": "null", "level": "六年级", "question": "一个圆柱, 底面周长是 25.12 厘米, 高是 8 厘米, 如果沿底面直径垂直切开, 它的截面是()。\n", "options": "A. 长方形\nB. 正方形\nC. 三角形\nD. 圆\n", "subject": "立体几何学", "analysis": "B"} {"id": "27018", "image": [], "answer": "D", "solution": "null", "level": "六年级", "question": " 一个圆柱与一个圆雉的底面积一样大, 要使它们的体积相等, 圆柱的高是 6 厘米, 圆雉的高应是 ( ) 厘米。\n", "options": "A. 2\nB. 6\nC. 12\nD. 18", "subject": "立体几何学", "analysis": "D"} {"id": "27035", "image": [], "answer": "D", "solution": "null", "level": "六年级", "question": " 如果正方体、圆柱、圆雉的底面积相等, 高也相等。下面说法正确的是\n", "options": "A. 圆柱的体积比正方体的体积小\nB. 圆柱和正方体的表面积相同\nC. 圆柱的体积是圆雉的 $\\frac{1}{3}$\nD. 圆雉的体积是正方体的 $\\frac{1}{3}$", "subject": "立体几何学", "analysis": "D"} {"id": "27164", "image": ["13269.jpg", "13270.jpg", "13271.jpg", "13272.jpg"], "answer": "D", "solution": "null", "level": "六年级", "question": " 在下图中, 以虚线为轴旋转, 可以得到圆柱形体的是 ()。\n", "options": "A.\n\n\nB.\n\n\nC.\n\n\nD.\n\n\n", "subject": "立体几何学", "analysis": "D"} {"id": "27175", "image": [], "answer": "D", "solution": "null", "level": "六年级", "question": " 18 个铁圆雉体, 可以熔铸成()个和它等底等高的圆柱体。\n", "options": "A. 72\nB. 18\nC. 9\nD. 6", "subject": "立体几何学", "analysis": "D"} {"id": "27186", "image": [], "answer": "C", "solution": "null", "level": "六年级", "question": " 把一个圆柱制成一个最大的圆雉,削去部分的体积是圆柱体积的()\n", "options": "A. 3 倍\nB. 2 倍\nC. 2\nD", "subject": "立体几何学", "analysis": "C"} {"id": "27189", "image": ["13291.jpg", "13292.jpg", "13293.jpg"], "answer": "A", "solution": "null", "level": "六年级", "question": " 直角三角形绕其一条直角边旋转一周可能得到的图形是()\n", "options": "A.\n\n\nB.\n\n\nC.\n\n\n", "subject": "立体几何学", "analysis": "A"} {"id": "13160", "image": [], "answer": "B", "solution": "null", "level": "六年级", "question": "20 名少先队员参加义务劳动, 平均分成若干小组 (组数和每组的人数都不少于 2 ) , 最多有 ( $\\quad$ ) 种分法。", "options": "A. 2\nB. 4\nC. 6\nD. 8", "subject": "组合数学", "analysis": "B"} {"id": "13168", "image": [], "answer": "C", "solution": "null", "level": "六年级", "question": "把 26 枝花插到 4 个花瓶中, 总有一个花瓶至少插 ( ) 枝花。", "options": "A. 9\nB. 8\nC. 7\nD. 6", "subject": "组合数学", "analysis": "答案:$\\mathrm{C}$"} {"id": "13122", "image": [], "answer": "C", "solution": "null", "level": "六年级", "question": "从一幅扑克牌中抽出 2 张王牌, 在剩下的 52 张中任意抽()张, 才能保证有两张是相同花色的。", "options": "A. 4\nB. 6\nC. 5\nD. 9", "subject": "组合数学", "analysis": "答案:$\\mathrm{C}$"} {"id": "13129", "image": [], "answer": "C", "solution": "null", "level": "六年级", "question": "小明参加飞镖比赛, 投了 10 镖, 成绩是 91 环, 小明至少有一镖不低于 $(\\quad)$ 环。", "options": "A. 8\nB. 9\nC. $\\quad 10$", "subject": "组合数学", "analysis": "答案: $\\mathrm{C}$"} {"id": "13131", "image": [], "answer": "B", "solution": "null", "level": "六年级", "question": "一个袋子里装着红、黄、二种颜色球各 3 个, 这些球的大小都相同, 问一次摸出 3 个球,其中至少有( )个球的颜色相同。", "options": "A. 1\nB. 2\nC. 3", "subject": "组合数学", "analysis": "答案:$\\mathrm{B}$"} {"id": "13132", "image": [], "answer": "B", "solution": "null", "level": "六年级", "question": "口袋里放有红、黄、白三种颜色的同样的钮扣各 10 枚,至少取出()枚钮扣, 才能保证三种颜色的钮扣都取到。", "options": "A. 13\nB. 21\nC. 30", "subject": "组合数学", "analysis": "答案:$\\mathrm{B}$"} {"id": "13133", "image": [], "answer": "C", "solution": "null", "level": "六年级", "question": "把()种颜色的球各 8 个放在一个盒子里,至少取出 4 个球,可以保证取到两个颜色相同的球。", "options": "A. 1\nB. 2\nC. 3\nD. 4", "subject": "组合数学", "analysis": "答案:C"} {"id": "13167", "image": [], "answer": "B", "solution": "null", "level": "六年级", "question": "把一根绳子连续对折两次, 每一小段是全长的 ( )。", "options": "A. $\\frac{1}{2}$\nB. $\\frac{1}{4}$\nC. $\\frac{1}{8}$\nD. $\\frac{1}{16}$", "subject": "算术", "analysis": "B"} {"id": "13063", "image": [], "answer": "A", "solution": "null", "level": "六年级", "question": "王叔叔把 10000 元钱存入银行, 这里的 “10000 元” 是 ( )。", "options": "A. 本金\nB. 利息\nC. 利率", "subject": "算术", "analysis": ""} {"id": "13064", "image": [], "answer": "A", "solution": "null", "level": "六年级", "question": "数轴上, -4 在 -3 的 ( $)$ 边。\nA、左\nB、右\nC、无法确定\n3. 一个圆锥的体积是 $135 \\mathrm{~cm}^{3}$, ( ) 是它等底等高的圆柱体体积。\nA、 $45 \\mathrm{~cm}^{3}$\nB、 $405 \\mathrm{~cm}^{3}$\nC、 $270 \\mathrm{~cm}^{3}$\n4. 表示 $x$ 和 $y$ 成反比例关系的式子是 $(\\quad)$ 。", "options": "A. $y-x=8$\nB. $x \\frac{1}{8} \\div y$\nC. $x \\div y=8$", "subject": "算术", "analysis": ""} {"id": "13065", "image": [], "answer": "C", "solution": "null", "level": "六年级", "question": "把 5 克糖溶解在 100 克水中,糖和糖水的比是()。", "options": "A. $1: 20$\nB. $20: 1$\nC. $1: 21$", "subject": "算术", "analysis": ""} {"id": "13066", "image": [], "answer": "D", "solution": "null", "level": "六年级", "question": "一件商品按五五折出售就是说现价()。", "options": "A. 比原价降低 $55 \\%$\nB. 是原价的 $", "subject": "算术", "analysis": "D"} {"id": "13072", "image": [], "answer": "B", "solution": "null", "level": "六年级", "question": "下面的百分率可能大于 $100 \\%$ 的是()。", "options": "A. 成活率\nB. 增长率\nC. 出勤率\nD. 发芽率", "subject": "算术", "analysis": "答案: $\\mathrm{B}$"} {"id": "13078", "image": [], "answer": "C", "solution": "null", "level": "六年级", "question": "六 (2) 班有四成的学生是女生, 那么男生占全班人数的()。", "options": "A. $\\frac{2}{3}$\nB. $40 \\%$\nC. $\\frac{3}{5}$\nD. 五成", "subject": "算术", "analysis": "答案: $\\mathrm{C}$"} {"id": "13079", "image": [], "answer": "C", "solution": "null", "level": "六年级", "question": "一件衣服 300 元,打七折后,便宜了()元.", "options": "A. 30\nB. 210\nC. 90\nD. 70", "subject": "算术", "analysis": "C"} {"id": "13080", "image": [], "answer": "B", "solution": "null", "level": "六年级", "question": "下面三句话中,() 是正确的。", "options": "A. 一种商品打六折出售, 就是按原价的 $6 \\%$ 出售\n\nB. 今年的水稻产量是去年的 $110 \\%$, 表示今年的水稻比去年增产 $\\frac{1}{10}$\n\nC. 在 100 克水中加入 2 克盐, 这种盐水的含盐率是 $2 \\%$", "subject": "算术", "analysis": "答案: $\\mathrm{B}$"} {"id": "13084", "image": [], "answer": "B", "solution": "null", "level": "六年级", "question": "$-5^{\\circ} \\mathrm{C}$ 比 $0^{\\circ} \\mathrm{C}$ 要 $(\\quad)$", "options": "A. 高 $3^{\\circ} \\mathrm{C}$\nB. 低 $5^{\\circ} \\mathrm{C}$\nC. 低 $10^{\\circ} \\mathrm{C}$", "subject": "算术", "analysis": "B"} {"id": "13090", "image": [], "answer": "D", "solution": "null", "level": "六年级", "question": "在 $-4,-9,-\\frac{4}{5},-0.1$ 这些数中, 最大的数是 ( )。", "options": "A. -4\nB. -9\nC. $-\\frac{4}{5}$\nD. -", "subject": "算术", "analysis": "答案: $\\mathrm{D}$"} {"id": "13097", "image": [], "answer": "C", "solution": "null", "level": "六年级", "question": "规定收入为正,那么支出 200 元则为( )", "options": "A. 200\nB. +200\nC. -200\nD. 不知道", "subject": "算术", "analysis": "答案:C"} {"id": "13098", "image": [], "answer": "D", "solution": "null", "level": "六年级", "question": "如果一个人先向东走 $6 \\mathrm{~m}$ 记作 $+6 \\mathrm{~m}$, 后来这个人又走 $-7 \\mathrm{~m}$, 结果是 ( )", "options": "A. 相当于从起点向东走了 $13 \\mathrm{~m}$\nB. 相当于从起点向东走了 $1 \\mathrm{~m}$\nC. 相当于从起点向西走了 $13 \\mathrm{~m}$\nD. 相当于从起点向西走了 $1 \\mathrm{~m}$", "subject": "算术", "analysis": "答案: D"} {"id": "13108", "image": [], "answer": "D", "solution": "null", "level": "六年级", "question": "能与 $3: 8$ 组成比例的是 $(\\quad)$ 。", "options": "A. $8: 3$\nB. $\\frac{1}{6}: \\frac{1}{8}$\nC. $", "subject": "算术", "analysis": "答案: $\\mathrm{D}$"} {"id": "13114", "image": [], "answer": "A", "solution": "null", "level": "六年级", "question": "$X=3 Y ( X 、 Y$ 不等于 $0 ), X$ 与 $Y ( ) 。$", "options": "A. 成正比\nB. 成反比\nC. 不成比例", "subject": "算术", "analysis": "A"} {"id": "13147", "image": [], "answer": "A", "solution": "null", "level": "六年级", "question": "甲乙两数都被遮住了部分, 甲: $5 \\square \\square \\square$ 乙: $9 \\square \\square . \\square$, 那么甲()乙。", "options": "A.大于\nB .小于\nC.等 于\nD.无法确定\n\n4 甲乙两地相距 170 千米, 在地图上量的距离是", "subject": "算术", "analysis": "A"} {"id": "13151", "image": [], "answer": "D", "solution": "null", "level": "六年级", "question": "两根同样长的绳子, 第一根截去它的 $\\frac{1}{3}$, 第二根截去 $\\frac{1}{3}$ 米, 余下的部分 $(\\quad)$ 。", "options": "A. $2 \\pi$\nB. $12 \\pi$\nC. $24 \\pi$\nD. $48 \\pi$", "subject": "算术", "analysis": "答案:D"} {"id": "26952", "image": [], "answer": "C", "solution": "null", "level": "六年级", "question": " 在 $-8,+11,-4,0,15,-16,+37,8848.86$ 这 8 个数中, 正数有 ( )个。\n", "options": "A. 2\nB. 3\nC. 4\nD. 5", "subject": "算术", "analysis": "C"} {"id": "26963", "image": [], "answer": "B", "solution": "null", "level": "六年级", "question": " 直线上, -3 和 +3 之间有 ( ) 个自然数。\n", "options": "A. 5\nB. 3\nC. 2\nD. 1", "subject": "算术", "analysis": "B"} {"id": "26971", "image": [], "answer": "C", "solution": "null", "level": "六年级", "question": " 下面最接近 0 的数是 ( )。\n", "options": "A. -3\nB. 2\nC. -1\nD. +2", "subject": "算术", "analysis": "C"} {"id": "26974", "image": [], "answer": "A", "solution": "null", "level": "六年级", "question": " 下列各数中是负数的是 ( )。\n", "options": "A. -2\nB. 0\nC. 1\nD. $\\frac{1}{3}$\n", "subject": "算术", "analysis": "A"} {"id": "26979", "image": [], "answer": "C", "solution": "null", "level": "六年级", "question": " 一双鞋打八折后是 64 元, 这双鞋原来()元。\n", "options": "A. 65\nB. 75\nC. 80", "subject": "算术", "analysis": "C"} {"id": "26989", "image": [], "answer": "C", "solution": "null", "level": "六年级", "question": " 一支钢笔的原价 10 元,先提价 $20 \\%$, 再打八折出售, 现价是()。\n", "options": "A. 12\nB. 10\nC. 9.6", "subject": "算术", "analysis": "C"} {"id": "27000", "image": [], "answer": "C", "solution": "null", "level": "六年级", "question": " 一种懒人拖把原来 80 元, 现在打七折出售, 现在买可以节约()元。\n", "options": "A. 10\nB. 56\nC. 24", "subject": "算术", "analysis": "C"} {"id": "27001", "image": [], "answer": "A", "solution": "null", "level": "六年级", "question": " 奶奶在银行存了两万元, 存期为两年, 年利率是 $3.15 \\%$, 到期后她能拿到()元利息。\n", "options": "A. $20000 \\times 3.15 \\% \\times 2$\nB. $20000 \\times 3.15 \\%+20000$\nC. $20000 \\times 3.15 \\% \\times 2+20000$\nD. $(20000 \\times 3.15 \\%+20000) \\times 2$", "subject": "算术", "analysis": "A"} {"id": "27002", "image": [], "answer": "D", "solution": "null", "level": "六年级", "question": " 一台电脑打九折后的售价 2700 元,这台电脑原来售价()元。\n", "options": "A. 270\nB. 300\nC. 2430\nD. 3000\n", "subject": "算术", "analysis": "D"} {"id": "27041", "image": [], "answer": "D", "solution": "null", "level": "六年级", "question": " 下面每组的四个数中, 能组成比例的是()。\n ", "options": "A. $5 、 3 、 15 、 12$\n B. $5 、 4 、 2 、 3$\n C. $\\frac{1}{2} 、 2 、 1.6 、 6$\n D. $\\frac{1}{6} ` \\frac{1}{4} ` \\frac{1}{3} ` \\frac{1}{2}$", "subject": "算术", "analysis": "D"} {"id": "27052", "image": [], "answer": "C", "solution": "null", "level": "六年级", "question": " 小林做 10 道数学题, 已做的题数和没有做的题数 ( )。\n ", "options": "A. 成正比例\n B. 成反比例\n C. 不成比例\n D. 无法判断", "subject": "算术", "analysis": "C"} {"id": "27063", "image": [], "answer": "B", "solution": "null", "level": "六年级", "question": " 生产同样的零件, 小张用了 4 小时, 小李用了 6 小时, 小李和小张的工作效率比是()。\n ", "options": "A. 6: 4\n B. $2: 3$\n C. 3: 2", "subject": "算术", "analysis": "B"} {"id": "27067", "image": [], "answer": "B", "solution": "null", "level": "六年级", "question": " 根据 $\\mathrm{a} \\times \\mathrm{b}=\\mathrm{c} \\times \\mathrm{d}(\\mathrm{a} 、 \\mathrm{~b} 、 \\mathrm{c} 、 \\mathrm{~d}$ 均不为 0 ), 改写成比例是()。\n ", "options": "A. c: $a=d: b$\n B. c: $a=b: d$\n C. a: b=c: d\n D. a: c=b: d\n", "subject": "算术", "analysis": "B"} {"id": "27102", "image": ["13251.jpg"], "answer": "B", "solution": "null", "level": "六年级", "question": " 如图, 点 $\\mathrm{A}$ 表示的数是 ( )。\n\n\n", "options": "A. 2\nB. -1\nC. -3\n", "subject": "算术", "analysis": "B"} {"id": "27113", "image": [], "answer": "B", "solution": "null", "level": "六年级", "question": " 体育老师记录成绩时用这样一种记数方法: 以 90 分为标准, 96 分记作+6 分, 某个同学的得分为 87 分, 应记作 ( $)$ 分。\n", "options": "A. +3\nB. -3\nC. -87", "subject": "算术", "analysis": "B"} {"id": "27123", "image": [], "answer": "A", "solution": "null", "level": "六年级", "question": " 如果把向南走 10 米记作 +10 , 那么把向()走 20 米记作-20 米。\n", "options": "A. 北\nB. 西\nC. 南\nD. 东", "subject": "算术", "analysis": "A"} {"id": "27124", "image": [], "answer": "D", "solution": "null", "level": "六年级", "question": " 菏泽某天的气温: 最低温度是 $-4^{\\circ} \\mathrm{C}$, 最高气温是 $6^{\\circ} \\mathrm{C}$, 这天的温差是 ( ) ${ }^{\\circ} \\mathrm{C}$ 。\n", "options": "A. 2\nB. 6\nC. 4\nD. 10", "subject": "算术", "analysis": "D"} {"id": "27130", "image": [], "answer": "B", "solution": "null", "level": "六年级", "question": " 六一前夕各商场搞优惠活动, 甲商店宣传册上说“每满 80 元减 20 元”; 乙商店门口海报写着: “全部商品七五折”出售,小玲看中了一双 500 元的运动鞋, 购买的实际价格()。\n", "options": "A. 甲商店便宜\nB. 乙商店便宜\nC. 甲乙商店价格一样\nD. 无法确定", "subject": "算术", "analysis": "B"} {"id": "27141", "image": [], "answer": "D", "solution": "null", "level": "六年级", "question": " 爸爸在银行存入 200000 元, 定期 3 年, 年利率 $2.75 \\%$, 到期时他获得利息是 ( ) 元。\n", "options": "A. 5500\nB. 5550\nC. 1650\nD. 16500", "subject": "算术", "analysis": "D"} {"id": "27152", "image": [], "answer": "B", "solution": "null", "level": "六年级", "question": " 甲、乙、丙、丁四个商店同时促销一种原价为 100 元的花生油。甲商店按原价的 $85 \\%$ 出售;乙商店满 200 元一律降价 $25 \\%$ 出售; 丙商店买四送一; 丁商店第二桶打六折。妈妈要买 2 桶这样的花生油,想花钱最少,应该到()商店去买。\n", "options": "A. 甲\nB. 乙\nC. 丙\nD. 丁", "subject": "算术", "analysis": "B"} {"id": "27158", "image": [], "answer": "A", "solution": "null", "level": "六年级", "question": " 王叔叔的月工资是 4800 元,扣除 3500 元个税免征额后的部分按 $3 \\%$ 的税率缴纳个人所得税,王叔叔应缴纳个人所得税()元。\n", "options": "A. 39\nB. 105\nC. 144", "subject": "算术", "analysis": "A"} {"id": "27205", "image": [], "answer": "A", "solution": "null", "level": "六年级", "question": " 式子 $\\frac{m-10}{x}=\\mathrm{y}$, 且 $\\mathrm{x}$ 和 $\\mathrm{y}$ 都不为 0 , 当 $\\mathrm{m}$ 一定时, $\\mathrm{x}$ 和 $\\mathrm{y}(\\quad)$\n", "options": "A. 成反比例关系\nB. 成正比例关系\nC. 不成比例关系\nD. 以上都不对", "subject": "算术", "analysis": "A"} {"id": "27219", "image": [], "answer": "B", "solution": "null", "level": "六年级", "question": " 下列各比, 能与\n\n$$\n\\frac{8}{15}: \\frac{4}{9} \\text { 组成比例的是 ( )。 }\n$$\n\n", "options": "A. $\\frac{4}{9}: \\frac{8}{15}$\nB. 6: 5\nC. $\\frac{15}{8}: 8$\nD. 5: 6\n", "subject": "算术", "analysis": "B"} {"id": "27220", "image": [], "answer": "B", "solution": "null", "level": "六年级", "question": " 在一个没有余数的除法算式中,若除数(不为 0 )一定,则被除数和商()\n", "options": "A. 成反比例关系\nB. 成正比例关系\nC. 不成比例关系\n", "subject": "算术", "analysis": "B"} {"id": "27251", "image": [], "answer": "A", "solution": "null", "level": "六年级", "question": " 任意 5 个自然数的和是偶数, 则其中至少有()个偶数。\n", "options": "A. 1\nB. 2\nC. 3\n", "subject": "算术", "analysis": "A"} {"id": "26081", "image": [], "answer": "B", "solution": "null", "level": "六年级", "question": " “大羊只数的 $\\frac{3}{8}$, 正好相当于小羊的只数” ,()是单位 “1”。\n ", "options": "A. 小羊的只数\n B. 大羊的只数\n C. 无法确定", "subject": "算术", "analysis": "B"} {"id": "26082", "image": [], "answer": "C", "solution": "null", "level": "六年级", "question": " 当 $\\mathrm{a}(\\quad)$ 时, $\\frac{2}{13} \\times \\mathrm{a}>\\frac{2}{13}$ 。\n ", "options": "A. 小于 1\n B. 等于 1\n C. 大于 1", "subject": "算术", "analysis": "C"} {"id": "26083", "image": [], "answer": "C", "solution": "null", "level": "六年级", "question": " 今年的产量比去年多 $\\frac{1}{10}$ ,今年的产量就相当于去年的()。\n ", "options": "A. $\\frac{1}{10}$\n B. $\\frac{9}{10}$\n C. $\\frac{11}{10}$", "subject": "算术", "analysis": "C"} {"id": "26085", "image": [], "answer": "C", "solution": "null", "level": "六年级", "question": " 一块长方形菜地, 长 $20 \\mathrm{~m}$, 宽是长的 $\\frac{3}{4}$, 求菜地面积的算式是 ( )。\n ", "options": "A. $20 \\times \\frac{3}{4}$\n B. $20 \\times \\frac{3}{4}+20$\n C. $20 \\times\\left(20 \\times \\frac{3}{4}\\right)$", "subject": "算术", "analysis": "C"} {"id": "26086", "image": [], "answer": "B", "solution": "null", "level": "六年级", "question": " 两根都是 $10 \\mathrm{~m}$ 长的电线, 甲用去全长的 $\\frac{2}{5}$, 乙用去 $\\frac{2}{5} \\mathrm{~m}$, 剩下的部分 ( )。\n ", "options": "A. 甲长\n B. 乙长\n C. 同样长\n", "subject": "算术", "analysis": "B"} {"id": "26106", "image": [], "answer": "B", "solution": "null", "level": "六年级", "question": " 白兔有 51 只, 占兔子总数的 $\\frac{3}{5}$, 要求( )可以列式为 “ $51 \\div \\frac{3}{5}$\n ", "options": "A. 黑兔只数\n B. 兔子总数\n C. 白兔只数\n", "subject": "算术", "analysis": "B"} {"id": "26107", "image": [], "answer": "C", "solution": "null", "level": "六年级", "question": " 下列除法算式中,商大于被除数的是( )\n ", "options": "A. $12 \\div 1 \\frac{1}{6}$\n B. $1 \\frac{3}{5} \\div 1 \\frac{1}{3}$\n C. $\\frac{9}{10} \\div \\frac{3}{4}$\n", "subject": "算术", "analysis": "C"} {"id": "26108", "image": [], "answer": "B", "solution": "null", "level": "六年级", "question": " 一根绳子长 $3 \\mathrm{~m}$, 比另一根绳子短 $\\frac{1}{3} \\mathrm{~m}$, 另一根绳子长()。\n ", "options": "A. $\\frac{8}{3} m$\n B. $\\frac{10}{3} \\mathrm{~m}$\n C. $2 \\mathrm{~m}$", "subject": "算术", "analysis": "B"} {"id": "26109", "image": [], "answer": "A", "solution": "null", "level": "六年级", "question": " 甲数的 $\\frac{5}{9}$ 和乙数的 $\\frac{3}{5}$ 相等(甲、乙都不为 0 ), 那么()。\n ", "options": "A. 甲数大\n B. 乙数大\n C.甲、乙大小无法比较", "subject": "算术", "analysis": "A"} {"id": "26110", "image": [], "answer": "D", "solution": "null", "level": "六年级", "question": " 下面的算式中,计算结果最大的是( )。\n ", "options": "A. $\\frac{12}{13} \\times\\left(\\frac{5}{6}+\\frac{3}{8}\\right)$\n B. $\\frac{12}{13} \\div\\left(\\frac{5}{6}+\\frac{3}{8}\\right.$\n C. A. $\\frac{12}{1 \\overline{3}}\\left(\\frac{5}{6}\\right) \\frac{3}{8}$\n D. $\\frac{12}{13} \\quad \\frac{5}{6}, \\frac{3}{8}$\n", "subject": "算术", "analysis": "D"} {"id": "13095", "image": [], "answer": "A", "solution": "null", "level": "六年级", "question": "点 $\\mathrm{A}$ 为数轴上 -1 的点, 将点 $\\mathrm{A}$ 沿数轴向左移动 2 个单位长度到达点 $\\mathrm{B}$, 则点 $\\mathrm{B}$ 表示的数为 ( )", "options": "A. -3\nB. 3\nC. 1\nD. 1 或 -3", "subject": "度量几何学", "analysis": "A"} {"id": "13099", "image": [], "answer": "A", "solution": "null", "level": "六年级", "question": "如果顺时针旋转 $60^{\\circ}$ 记作 $-60^{\\circ}$, 那么逆时针旋转 $45^{\\circ}$ 记作 ( )。", "options": "A. $45^{\\circ}$\nB. $-45^{\\circ}$\nC. 无法表示", "subject": "度量几何学", "analysis": "答案: A"} {"id": "13102", "image": [], "answer": "A", "solution": "null", "level": "六年级", "question": "某种型号的铁丝, 它的长度与质量()。", "options": "A. 成正比例\nB. 成反比例\nC. 不成比例", "subject": "度量几何学", "analysis": "答案: A"} {"id": "13118", "image": [], "answer": "A", "solution": "null", "level": "六年级", "question": "比例尺一定,图上距离和实际距离( )。", "options": "A. 正比例\nB. 反比例\nC. 不成比例", "subject": "度量几何学", "analysis": "答案:A"} {"id": "13142", "image": [], "answer": "B", "solution": "null", "level": "六年级", "question": "你平时上课的宽敞的教室地面面积大约是()。", "options": "A. 50 平方分米\nB. 50 平方米\nC. 50 平方厘米\nD. 5000 平方分米", "subject": "度量几何学", "analysis": "答案:B"} {"id": "13150", "image": [], "answer": "A", "solution": "null", "level": "六年级", "question": "钟面上时针的长度 1 分米, 一昼夜时针扫过的面积 ( ) 平方分米。", "options": "A. $1: 500$\nB. $1: 5000000$\nC. $1: 50000$\nD. $1: 500000$", "subject": "度量几何学", "analysis": "答案:A"} {"id": "12492", "image": [], "answer": "C", "solution": "null", "level": "六年级", "question": "将周长 25.12 厘米的圆形纸片剪成两个半圆,每个半圆的周长是()", "options": "A.", "subject": "度量几何学", "analysis": "答案: $C$"} {"id": "12496", "image": [], "answer": "A", "solution": "null", "level": "六年级", "question": "经过 1 小时,钟面上分针转过的角度与时针转过的角度相差 ( )。", "options": "A. $330^{\\circ}$\nB. $300^{\\circ}$\nC. $150^{\\circ}$\nD. $120^{\\circ}$", "subject": "度量几何学", "analysis": "答案: A"} {"id": "12498", "image": ["2665.jpg"], "answer": "C", "solution": "null", "level": "六年级", "question": "如图:一个三角形的三个顶点分别为三个半径为 3 厘米的圆的圆心,则图中阴影部分的面积是\n\n", "options": "A. $\\pi$ 平方厘米\nB. $9 \\pi$ 平方厘米\nC. $", "subject": "度量几何学", "analysis": "答案: $\\mathrm{C}$"} {"id": "26973", "image": ["13225.jpg"], "answer": "C", "solution": "null", "level": "六年级", "question": " 下图中, $\\mathrm{N}$ 是数轴的一个数, $\\mathrm{N}$ 适合表示 ( )。\n\n\n", "options": "A. 1.5\nB. 0.15\nC. -1.5\n", "subject": "度量几何学", "analysis": "C"} {"id": "27066", "image": [], "answer": "C", "solution": "null", "level": "六年级", "question": " 一幅地图上的线段比例尺是 $0 \\quad 20 \\quad 40$ 千米, 把这个线段比例尺改写为数值比例尺是()。\n ", "options": "A. $1: 20$\n B. $20: 1$\n C. 1: 2000000\n D. 2000000: 1", "subject": "度量几何学", "analysis": "C"} {"id": "27195", "image": [], "answer": "A", "solution": "null", "level": "六年级", "question": " 下列说法错误的是 ( )。\n\n", "options": "A. 比例尺中, 图上距离一定小于实际距离。\n\nB. 长方形的面积一定, 长与宽成反比例。\n\nC. 如果学校在书店的正东方向上, 那么书店就在学校的正西方向上。\n", "subject": "度量几何学", "analysis": "A"} {"id": "26128", "image": [], "answer": "A", "solution": "null", "level": "六年级", "question": " 正方形的周长与边长的比是 ( )\n ", "options": "A.4:1\n B.3:1\n C.2:1\n", "subject": "度量几何学", "analysis": "A"} {"id": "26133", "image": [], "answer": "A", "solution": "null", "level": "六年级", "question": " 一个三角形三个内角度数的比是 $5: 4: 3$ 这个三角形是()三角形。\n ", "options": "A.锐角\n B. 钝角\n C.直角\n", "subject": "度量几何学", "analysis": "A"} {"id": "26151", "image": [], "answer": "C", "solution": "null", "level": "六年级", "question": " 两个圆的周长不相等, 是因为两个圆的()不一样。\n ", "options": "A.圆周率\n B. 圆心的位置\n C. 半径的长度\n", "subject": "度量几何学", "analysis": "C"} {"id": "26153", "image": [], "answer": "C", "solution": "null", "level": "六年级", "question": " 周长相等的长方形、正方形和圆,()的面积最大。\n ", "options": "A.长方形\n B.正方形\n C.圆\n", "subject": "度量几何学", "analysis": "C"} {"id": "26154", "image": [], "answer": "C", "solution": "null", "level": "六年级", "question": " 一个圆的周长是 $18.84 \\mathrm{~cm}$, 它的面积是 ( ) cm。\n ", "options": "A. 9.42\n B. 18.84\n C. 28.26\n\n \n", "subject": "度量几何学", "analysis": "C"} {"id": "26170", "image": [], "answer": "C", "solution": "null", "level": "六年级", "question": " 圆的半径扩大到原来的 3 倍, 它的面积扩大到原来的()倍\n ", "options": "A. 3\n B. 6\n C. 9\n", "subject": "度量几何学", "analysis": "C"} {"id": "13117", "image": [], "answer": "B", "solution": "null", "level": "六年级", "question": "在比例尺是 1: 4000000 的地图上, 甲、乙两地相距 5 厘米, 如果画在比\n\n例尺是 $\\begin{array}{llll}0 & 50 \\quad 100 \\quad 150\\end{array}$ 千米\n\n的地图上, 那么应画 $(\\quad)$ 厘米。", "options": "A. 3\nB. 4\nC. 5", "subject": "画法几何学", "analysis": "B"} {"id": "27029", "image": ["13235.jpg", "13236.jpg", "13237.jpg"], "answer": "D", "solution": "null", "level": "六年级", "question": " 下面四幅图中,不是圆柱侧面展开图的是()。\n", "options": "A. $\\square$\nB.\n\n\nC.\n\n\nD.\n\n\n", "subject": "画法几何学", "analysis": "D"} {"id": "12487", "image": [], "answer": "D", "solution": "null", "level": "六年级", "question": "在一个边长 6 分米的正方形中画一个最大的圆,圆的半径是()分米.", "options": "A. 8\nB. 6\nC. 4\nD. 3", "subject": "解析几何", "analysis": "D"} {"id": "12497", "image": [], "answer": "A", "solution": "null", "level": "六年级", "question": "周长都相等的圆,正方形和长方形,它们的面积( )。", "options": "A. 圆最大\nB. 一样大\nC. 正方形最大", "subject": "解析几何", "analysis": "答案:A"} {"id": "27072", "image": [], "answer": "A", "solution": "null", "level": "六年级", "question": "盒子里有 5 个黑球、 3 个黄球、 2 个绿球,任意拿出 6 个, 最少有一个( )。\n", "options": "A. 黑球\nB. 黄球\nC. 绿球\nD. 白球\n", "subject": "计数", "analysis": "A"} {"id": "27159", "image": [], "answer": "D", "solution": "null", "level": "六年级", "question": " 某商品的标价是 1000 元,打八折出售后仍盈利 100 元,则该商品进价是()元\n", "options": "A. 100\nB. 850\nC. 750\nD. 700", "subject": "计数", "analysis": "D"} {"id": "27160", "image": [], "answer": "A", "solution": "null", "level": "六年级", "question": " 小军把 800 元存入银行, 这里的“ 800 元”是()。\n", "options": "A. 本金\nB. 利息\nC. 利率\n", "subject": "计数", "analysis": "A"} {"id": "26194", "image": [], "answer": "C", "solution": "null", "level": "六年级", "question": " 一种纺织品的合格率是 $98 \\% , 300$ 件产品中有()件不合格。\n ", "options": "A. 2\n B. 4\n C. 6\n D. 294", "subject": "计数", "analysis": "C"} {"id": "26212", "image": ["13828.jpg"], "answer": "C", "solution": "null", "level": "六年级", "question": " 右面是两个扇形统计图, 其中说法不正确的是( )。\n ", "options": "A. 甲班的女生占全班人数的 $\\frac{2}{5}$ 。\n B. 乙班的男生占全班人数的 $\\frac{7}{10}$ 。\n C. 乙班的男生一定比甲班的男生多\n\n\n", "subject": "计数", "analysis": "C"} {"id": "26225", "image": ["13100.jpg"], "answer": "B", "solution": "null", "level": "六年级", "question": " 如下图, 某校对 300 名学生最喜欢的球类进行了调查, 最喜欢篮球的有 ( )人。\n ", "options": "A. 50\n B. 60\n C. 90\n D. 75\n\n\n", "subject": "计数", "analysis": "B"} {"id": "26226", "image": [], "answer": "B", "solution": "null", "level": "六年级", "question": " 为了表示一个病人的体温变化情况,应选择制作( )。\n ", "options": "A.条形统计图\n B.折线统计图\n C.扇形统计图", "subject": "计数", "analysis": "B"} {"id": "26249", "image": [], "answer": "D", "solution": "null", "level": "六年级", "question": " 六年级每周的体育兴趣小组活动时间, 全年级有一半的同学报名参加篮球、足球和排球的训练, 参与的人数比是 $5: 2: 2$, 画成扇形统计图, 参加篮球训练的人数所在的\n 扇形的圆心角的度数是 ( )。\n ", "options": "A. $50^{\\circ}$\n B. $100^{\\circ}$\n C. 150\n D. $200^{\\circ}$", "subject": "计数", "analysis": "D"} {"id": "26250", "image": ["13101.jpg", "13102.jpg", "13103.jpg", "13104.jpg"], "answer": "C", "solution": "null", "level": "六年级", "question": " 六(1)班统计学生最喜欢阅读的书籍: 文学类 30 人,科技类 15 人,漫画类 15 人。下面的图( ), 能准确表示这一结果。\n\n\n\n", "options": "A.\n\n\n\n$\\mathrm{B}$.\n\n\n\nC.\n\n\n\n$\\mathrm{D}$.\n", "subject": "计数", "analysis": "C"} {"id": "26259", "image": [], "answer": "C", "solution": "null", "level": "六年级", "question": " 扇形统计图最突出的特点是()。\n ", "options": "A.表示数据的多少\n B.表示数据的增减变化\n C.表示各部分与整体的关系", "subject": "计数", "analysis": "C"} {"id": "26084", "image": [], "answer": "B", "solution": "null", "level": "六年级", "question": " 如果 $\\mathrm{ax} \\frac{1}{6}=\\mathrm{bx} \\frac{2}{3}$ ( $\\mathrm{ab}$ 都不为 0$)$, 那么 ()。\n ", "options": "A. $ab$\n C. $a=b$", "subject": "代数", "analysis": "B"} {"id": "26229", "image": [], "answer": "A", "solution": "null", "level": "六年级", "question": " 男生人数比女生多,则女生人数与男生人数的比是()\n ", "options": "A.4:5\n B.5:4\n C. $1: 4$\n D.1: $\\frac{1}{4}$", "subject": "代数", "analysis": "A"} {"id": "26171", "image": [], "answer": "B", "solution": "null", "level": "六年级", "question": " 下列各图形中,()不是轴对称图形。\n ", "options": "A.长方形\n B. 平行四边形\n C.圆\n", "subject": "变换几何", "analysis": "B"} {"id": "26213", "image": ["13089.jpg", "13090.jpg", "13091.jpg"], "answer": "B", "solution": "null", "level": "六年级", "question": " 按下面的图形摆放桌椅, 想一想这样的 10 张桌子连在一起可以坐( )人, 如果有 $\\mathrm{n}$ 张这样的桌子连在一起, 一共可以坐( )人。\n\n\n\n", "options": "A. 20\n\n\nB. 22\n\n\nC. $2 n+2$\nD. 4 n-2\n\n\n", "subject": "组合几何学", "analysis": "B"} {"id": "26224", "image": [], "answer": "C", "solution": "null", "level": "六年级", "question": " 在扇形统计图中, 有一个扇形的面积占整个圆面积的 $\\frac{1}{5}$, 这个扇形的圆心角是\n ", "options": "A. $36^{\\circ}$\n B. $50^{\\circ}$\n C. $72^{\\circ}$", "subject": "统计数学", "analysis": "C"} {"id": "12560", "image": ["2687.jpg"], "answer": "B", "solution": "null", "level": "六年级", "question": "1 路汽车从火车站开往幸福村,前进的方向是 ( ) .\n\n", "options": "A. 先向西-再向西南-最后向西北\n\nB. 先向东 - 再向东南 - 最后向东北\n\nC. 先向东-再向西南 - 最后向东北\n\nD. 无选项", "subject": "图论", "analysis": "答案:B"} {"id": "12736", "image": [], "answer": "A", "solution": "null", "level": "六年级", "question": "$\\frac{1}{2021} \\times 2020+\\frac{1}{2021}=\\frac{1}{2021} \\times(2020+1)=1$, 这是根据 )计算的。", "options": "A. 乘法分配律\nB. 乘法结合律\nC. 乘法交换律", "subject": "逻辑题", "analysis": "A"} {"id": "5686", "image": [], "answer": "A", "solution": "null", "level": "八年级", "question": "在体育课上, 九年级 2 名学生各练习 10 次立定跳远, 要判断哪一名学生的成绩比较稳定, 通常需要比较这 2 名学生立定跳远成绩的()", "options": "A. 方差\nB. 平均数\nC. 频率分布\nD. 众数", "subject": "计数", "analysis": "由于方差能反映数据的稳定性, 需要比较这 2 名学生立定跳远成绩的方差.\n\n故答案为: $A$"} {"id": "5696", "image": [], "answer": "D", "solution": "null", "level": "八年级", "question": "有一组数据: $1,3,3,6,7,8$, 这组数据的中位数是( )", "options": "A. 3\nB. 3.5\nC. 4\nD. 4.5", "subject": "计数", "analysis": "排序为: $1,3,3,6,7,8$\n\n处于最中间的两个数是 3 和 6\n\n$\\therefore$ 这组数据的中位数为 ${ }^{\\frac{3+6}{2}}=4.5$\n\n故答案为: D"} {"id": "5705", "image": [], "answer": "A", "solution": "null", "level": "八年级", "question": "在抗击“新冠肺炎”时期, 开展停课不停学活动, 戴老师从 3 月 1 号到 7 号在网上答题个数记录如下\n\n| 日期 | 1 号 | 2 号 | 3 号 | 4 号 | 5 号 | 6 号 | 7 号 |\n| :--- | :--- | :--- | :--- | :--- | :--- | :--- | :--- |\n| 答题个数 | 68 | 55 | 50 | 56 | 54 | 48 | 68 |\n\n在戴老师每天的答题个数所组成的这组数据中,众数和中位数依次是()", "options": "A. 68,55\nB. 55,68\nC. 68,57\nD. 55,57", "subject": "计数", "analysis": "这组数据: $68,55,50,56,54,48,68$ 中出现次数最多的为 68 , 故众数是 68 ;把这组数据从小到大排列为: $48,50,54,55,56,68,68$ 中间的数据为 55 , 故中位数是 55 .\n\n故答案为: A."} {"id": "5707", "image": [], "answer": "D", "solution": "null", "level": "八年级", "question": "数据 $2,6,5,0,1,6,8$ 的中位数和众数分别是 $(\\quad)$", "options": "A. 0 和 6\nB. 0 和 8\nC. 5 和 8\nD. 5 和 6", "subject": "计数", "analysis": "从小到大排列为 $0,1,2,5,6,6,8$\n\n最中间的数是 5,\n\n$\\therefore$ 中位数是 5 ;\n\n$\\because 6$ 出现了 2 次, 是出现次数最多的数,\n\n$\\therefore$ 这组数据的众数是 6 .\n\n故答案为: D"} {"id": "5708", "image": ["7157.jpg"], "answer": "A", "solution": "null", "level": "八年级", "question": "如表是某皮鞋专卖店一周的同一款男士皮鞋四种尺码的销售分布情况:\n\n| 尺码/码 | 38 | 39 | 40 | 41 |\n| :---: | :---: | :---: | :---: | :---: |\n| 频数 | 5 | 15 | a | $10-\\mathrm{a}$ |\n\n对于不同的 a, 下列关于皮鞋尺码的四个统计量(1)众数, (2)中位数, (3)平均数, (4)方差中, 不会发生改变的是 $(\\quad)$", "options": "A. (1)(2)\nB. (2)(3)\nC. (3)(4)\nD. (1) (4)", "subject": "计数", "analysis": "由表可知,尺码为 $40 、 41$ 的频数和为 $a+10-a=10$,则总人数为: $5+15+10=30$,\n\n\n即对于不同的 $\\mathrm{x}$, 关于皮鞋尺码的四个统计量不会发生改变的是众数和中位数,故答案为: A."} {"id": "5710", "image": [], "answer": "B", "solution": "null", "level": "八年级", "question": "某班要从 9 名百米跑成绩各不相同的同学中选 4 名参加 $4 \\times 100$ 米接力赛, 而这 9 名同学只知道自己的成绩,要想让他们知道自己是否入选,老师只需公布他们成绩的( )", "options": "A. 平均数\nB. 中位数\nC. 众数\nD. 方差", "subject": "计数", "analysis": "总共有 9 名同学, 只要确定每个人与成绩的第五名的成绩的多少即可判断, 然后根据中位数定义即可判断. 知道自己是否入选, 老师只需公布第五名的成绩, 即中位数.\n\n故答案为: B"} {"id": "5711", "image": [], "answer": "C", "solution": "null", "level": "八年级", "question": "某气象台报告一周中白天的气温(单位: ${ }^{\\circ} \\mathrm{C}$ )为: $3,4,0,3,1,-1,-3$, 这一周内白天温度的标准差(精确到 0.1$)$ 是 $(\\quad)$", "options": "A. 2.1\nB. 2.2\nC. 2.3\nD. 2.4", "subject": "计数", "analysis": "总共有 9 名同学, 只要确定每个人与成绩的第五名的成绩的多少即可判断, 然后根据中位数定义即可判断. 知道自己是否入选, 老师只需公布第五名的成绩, 即中位数.\n\n故答案为: B"} {"id": "5712", "image": [], "answer": "A", "solution": "null", "level": "八年级", "question": "为了解当地气温变化情况, 某研究小组记录了寒假期间连续 6 天的最高气温, 结果如下 (单位: ${ }^{\\circ} \\mathrm{C}$ ): $-6,-3, x, 2,-1,3$. 若这组数据的中位数是 -1 , 则下列结论错误的是 ( )", "options": "A. 方差是 8\nB. 极差是 9\nC. 众数是 -1\nD. 平均数是 -1\n\n10 .一组数据 $3 、 5 、 8 、 3 、 4$ 的众数与中位数分别是 $(\\quad)$\nA. 3,8\nB. 3,3\nC. 3,4\nD. 4,3", "subject": "计数", "analysis": "根据题意可知 $x=-1$,\n\n平均数 $=(-6-3-1-1+2+3) \\div 6=-1$,\n\n$\\because$ 数据 -1 出现两次最多,\n\n$\\therefore$ 众数为 -1 ,\n\n极差 $=3-(-6)=9$,\n\n方差 $=\\frac{1}{6}\\left[(-6+1)^{2}+(-3+1)^{2}+(-1+1)^{2}+(2+1)^{2}+(-1+1)^{2}+(3+1)^{2}\\right]=9$.\n\n故选 A."} {"id": "5802", "image": [], "answer": "C", "solution": "null", "level": "八年级", "question": "已知一组数据 $1,3,2,5, \\mathrm{x}$ 的众数是 3 , 则这组数据的中位数是 ( )", "options": "A. 2.8\nB. 2\nC. 3\nD. 5", "subject": "计数", "analysis": "根据题意, $x=3$.\n\n把这组数据从小到大排列为: $1,2,3,3,5$. 所以中位数为 3 .\n\n故选 C."} {"id": "5824", "image": [], "answer": "B", "solution": "null", "level": "八年级", "question": "近年来, 我国持续大面积的雾䨵天气让环保和健康问题成为焦点. 为进一步普及环保和健康知识,我市某校举行了“建设宜居成都, 关注环境保护”的知识竟赛, 某班的学生成绩统计如下:\n\n| 成绩(分) | 60 | 70 | 80 | 90 | 100 |\n| :--- | :--- | :--- | :--- | :--- | :--- | :--- |\n| 人 数 | 4 | 8 | 12 | 11 | 5 |\n\n则该办学生成绩的众数和中位数分别是 $(\\quad)$", "options": "A. 70 分, 80 分\nB. 80 分, 80 分\nC. 90 分, 80 分\nD. 80 分, 90 分", "subject": "计数", "analysis": "众数是在一组数据中, 出现次数最多的数据, 这组数据中 80 出现 12 次, 出现的次数最多, 故这组数据的众数为 80 分;\n\n中位数是一组数据从小到大 (或从大到小) 排列后, 最中间的那个数(最中间两个数的平均数). 因此这组 40 个按大小排序的数据中, 中位数是按从小到大排列后第 20,21 个数的平均数, 而第 20 , 21 个数都在 80 分组, 故这组数据的中位数为 80 分.\n\n故答案为: B."} {"id": "5079", "image": [], "answer": "B", "solution": "null", "level": "八年级", "question": "已知一组数据: $1 、 2 、 3 、 1 、 5$, 这组数据的中位数是", "options": "A. 1B. 2C. 3D. 5", "subject": "计数", "analysis": "【点拨】利用中位数的定义求解即可.【解析】解:把这组数据从小到大排列为1,1,2,3,5,故中位数为2;故选B.【点睛】本题考查了确定一组数据的中位数的能力.注意找中位数的时候一定要先排好顺序,然后再根据奇数和偶数个来确定中位数,如果数据有奇数个,则正中间的数字即为所求,如果是偶数个,则找中间两位数的平均数."} {"id": "5709", "image": [], "answer": "B", "solution": "null", "level": "八年级", "question": "某演讲比赛中, 评委将从演讲内容、演讲能力、演讲效果三个方面为选手打分, 然后再按演讲内容占 $50 \\%$ 、演讲能力占 $40 \\%$ 、演讲效果占 $10 \\%$ 的比例计算选手的综合成绩.某选手的演讲内容、演讲能力、演讲效果成绩依次为 $85 、 90 、 95$ ,则该选手的综合成绩为 $(\\quad)$", "options": "A. 92\nB. 88\nC. 90\nD. 95", "subject": "算术", "analysis": "该选手的综合成绩为 $85 \\times 50 \\%+90 \\times 40 \\%+95 \\times 10 \\%=88$ 分.\n\n故答案为: B."} {"id": "5739", "image": [], "answer": "D", "solution": "null", "level": "八年级", "question": "某商场进来一批电视机, 进价为 2300 元, 为答谢新老顾客, 商店按标价的九折销售, 利润仍为 $20 \\%$,则该电视的标价是()。", "options": "A. 2760 元\nB. 3286 元\nC. 2875 元\nD. 3067 元", "subject": "算术", "analysis": "设标价为 $x$ 元,根据题意列方程得:\n\n$90 \\% x=2300 \\times(1+20 \\%)$,\n\n解得: $x=3067$,\n\n故选: D."} {"id": "5743", "image": [], "answer": "D", "solution": "null", "level": "八年级", "question": "下列计算错误的是 $(\\quad)$", "options": "A. $3 \\sqrt{5}-\\sqrt{5}=2 \\sqrt{5}$\nB. $\\sqrt{2} \\times \\sqrt{3}=\\sqrt{6}$\nC. $\\sqrt{\\frac{7}{4}}=\\frac{\\sqrt{7}}{2}$\nD. $\\sqrt{3}+\\sqrt{6}=\\sqrt{3+6}=\\sqrt{9}=3$", "subject": "算术", "analysis": "根据二次根式的加减法, 合并同类二次根式, 可知 $3 \\sqrt{5}-\\sqrt{5}=2 \\sqrt{5}$, 故不符合题意;\n\n根据二次根式的乘法, 可知 $\\sqrt{2} \\times \\sqrt{3}=\\sqrt{6}$, 故不符合题意;\n\n根据二次根式的性质和化简, 由分母有理化可得 $\\sqrt{\\frac{7}{4}}=\\frac{\\sqrt{7}}{2}$, 故不符合题意;\n\n根据二次根式的加减, 可知 $\\sqrt{3}$ 与 $\\sqrt{6}$ 不是同类二次根式, 故符合题意.\n\n故答案为: D."} {"id": "5754", "image": [], "answer": "B", "solution": "null", "level": "八年级", "question": "下列各式中不是二次根式的是 ( )", "options": "A. $\\sqrt{x^{2}+1}$\nB. $\\sqrt{-4}$\nc. $\\sqrt{0}$\n$\\sqrt{(a-b)^{2}}$\nD.", "subject": "算术", "analysis": "A、 $\\sqrt{x^{2}+1}, \\because \\mathrm{x}^{2}+1 \\geq 1>0, \\therefore \\sqrt{x^{2}+1}$ 符合二次根式的定义; 故本选项正确;\n\nB、 $\\because-4<0, \\therefore \\sqrt{-4}$ 不是二次根式; 故本选项错误;\n\nC、 $\\because 0 \\geq 0, \\therefore \\sqrt{0}$ 符合二次根式的定义; 故本选项正确;\n\nD、 $\\sqrt{(a-b)^{2}}$ 符合二次根式的定义; 故本选项正确.\n\n故选 B."} {"id": "5770", "image": [], "answer": "D", "solution": "null", "level": "八年级", "question": "下列计算正确的是 ( )", "options": "A. $\\sqrt{(-3)^{2}}=-3$\nB. $\\left(\\frac{1}{3}\\right)^{-2}=\\frac{1}{9}$\nC. $\\left(-a^{2}\\right)^{3}=a^{6}$\nD. $a^{6} \\div\\left(\\frac{1}{2} a^{2}\\right)=2 a^{4}$", "subject": "算术", "analysis": "$A 、 \\sqrt{(-3)^{2}}=|-3|=3$, 错误, 不符合题意;\n\nB、 $\\left(\\frac{1}{3}\\right)^{-2}=\\frac{1}{\\left(\\frac{1}{3}\\right)^{2}}=9$ ,错误,不符合题意;\n\nC、 $\\left(-a^{2}\\right)^{3}=-a^{6}$ ,错误,不符合题意;\n\nD、\n\n$$\na^{6} \\div\\left(\\frac{1}{2} a^{2}\\right)=a^{6} \\times \\frac{2}{a^{2}}=2 a^{4} \\text {, 正确, 符合题意。 }\n$$\n\n故答案为: D。"} {"id": "4998", "image": [], "answer": "C", "solution": "null", "level": "八年级", "question": "下列二次根式是最简二次根式的是 $(\\quad)$", "options": "A. $\\sqrt{\\frac{1}{3}}$B. $\\sqrt{8}$C. $\\sqrt{14}$D. $\\sqrt{12}$", "subject": "算术", "analysis": "【点拨】判定一个二次根式是不是最简二次根式的方法,就是逐个检查最简二次根式的两个条件是否同时满足,同时满足的就是最简二次根式,否则就不是.【解析】解:sqrtfrac13=fracsqrt33不是最简二次根式;sqrt8=2sqrt2不是最简二次根式;sqrt14是最简二次根式;sqrt12=2sqrt3不是最简二次根式.故选C.【点睛】本题考查最简二次根式的定义.解决此题的关键,是掌握最简二次根式的定义,最简二次根式必须满足两个条件:(1)被开方数不含分母;(2)被开方数不含能开得尽方的因数或因式."} {"id": "5003", "image": [], "answer": "A", "solution": "null", "level": "八年级", "question": "化简 $\\sqrt{(3-\\pi)^{2}}$ 得 $(\\quad)$", "options": "A. $\\pi-3$B. $3-\\pi$C. $-\\pi-3$D. $\\pi+3$", "subject": "算术", "analysis": "【点拨】根据二次根式的性质进行化简即可求解.【解析】解:sqrt(3-pi)^2=|3-pi|=pi-3,故选A.【点睛】本题主要考查了二次根式的化简,掌握二次根式的性质是解题的关键."} {"id": "5005", "image": [], "answer": "B", "solution": "null", "level": "八年级", "question": "下列二次根式中, 能与 $\\sqrt{2}$ 合并的是 $(\\quad)$", "options": "A. $\\sqrt{12}$B. $\\sqrt{\\frac{1}{2}}$C. $\\sqrt{20}$D. $\\sqrt{9}$", "subject": "算术", "analysis": "【点拨】先化简二次根式,根据同类二次根式的定义即可得出答案.【解析】解:A.sqrt12=2sqrt3,不能与sqrt2合并,故该选项不符合题意;B.sqrtfrac12=fracsqrt22,能与sqrt2合并,故该选项符合题意;C.sqrt20=2sqrt5,不能与sqrt2合并,故该选项不符合题意;D.sqrt9=3,不能与sqrt2合并,故该选项不符合题意.故选B.【点睛】本题考查了同类二次根式,二次根式的性质与化简,掌握一般地,把几个二次根式化为最简二次根式后,如果它们的被开方数相同,就把这几个二次根式叫做同类二次根式是解题的关键."} {"id": "5006", "image": [], "answer": "D", "solution": "null", "level": "八年级", "question": "计算 $\\sqrt{12}-\\sqrt{27}$ 的结果是 $(\\quad)$", "options": "A. $\\sqrt{6}$B. -1C. $\\sqrt{3}$D. $-\\sqrt{3}$", "subject": "算术", "analysis": "【点拨】直接化简二次根式,进而合并二次根式得出答案.【解析】解:原式=2sqrt3-3sqrt3=-sqrt3.故选D.【点睛】此题主要考查了二次根式的加减,正确化简二次根式是解题关键."} {"id": "5010", "image": [], "answer": "D", "solution": "null", "level": "八年级", "question": "下列各式是最简二次根式的是 $(\\quad)$", "options": "A. $\\sqrt{9}$B. $\\sqrt{\\frac{25}{2}}$C. $\\sqrt{50}$D. $\\sqrt{\\mathrm{x}^{4}+\\mathrm{y}^{4}}$", "subject": "算术", "analysis": "【点拨】根据最简二次根式的特点:(1)被开方的因数是整数,因式是整式,(2)被开方数中不能含有开得尽方的因数或因式,进行判断即可.【解析】解:A、sqrt9=3,不是最简二次根式,故A不符合题意;B、sqrtfrac252=frac5sqrt22,不是最简二次根式,故B不符合题意;C、sqrt50=5sqrt2,不是最简二次根式,故C不符合题意;D、sqrtx^4+y^4是最简二次根式,故D符合题意.故选D.【点睛】本题考查了最简二次根式的定义,熟练掌握最简二次根式的特点(1)被开方的因数是整数,因式是整式,(2)被开方数中不能含有开得尽方的因数或因式是解题的关键."} {"id": "5021", "image": [], "answer": "D", "solution": "null", "level": "八年级", "question": "下列计算:(1) $(\\sqrt{2})^{2}=2$, (2) $\\sqrt{(-2)^{2}}=2$, (3) $(-2 \\sqrt{3})^{2}=12$, (4) $(\\sqrt{2}+1)(\\sqrt{2}$ $-1)=1$. 其中结果正确的个数为 $(\\quad)$", "options": "A. 1B. 2C. 3D. 4", "subject": "算术", "analysis": "【点拨】利用二次根式的化简计算和平方差公式进行解答.【解析】解:(1)(sqrt2)^2=2,结果正确;(2)sqrt(-2)^2=2,结果正确;(3)(-2sqrt3)^2=12,结果正确;(4)(sqrt2+1)(sqrt2-1)=(sqrt2)^2-1^2=2-1=1,结果正确;故选D.【点睛】考查了二次根式的混合运算,平方差公式,属于基础计算题,熟记计算法则或公式即可解题."} {"id": "5032", "image": [], "answer": "D", "solution": "null", "level": "八年级", "question": "若 $\\sqrt{20 \\mathrm{n}}$ 是整数, 则正整数 $n$ 的最小值是 ( )", "options": "A. 2B. 3C. 4D. 5", "subject": "算术", "analysis": "【点拨】先化简sqrt20,然后根据二次根式的定义判断即可.【解析】解:becausesqrt20=2sqrt5,therefore正整数n的最小值是:5,故选D.【点睛】本题考查了二次根式的定义,熟练掌握二次根式的定义是解题的关键."} {"id": "5078", "image": [], "answer": "B", "solution": "null", "level": "八年级", "question": "王阿姨的水果店以 4 元/千克的价格购入了一批苹果, 再以 6 元/千克的价格出售, 每天可售出 200 千克, 为了促销, 王阿姨决定降价销售, 销售过程中发现, 这种苹果每降价 0.2 元/千克, 每天可多售出 20 千克, 另外, 每天的房租等固定成本为 50 元, 若王阿姨每天要想盈利 250 元, 设应将每千克苹果的售价降低 $x$ 元, 则以下方程正确的为", "options": "A. $(6-x)\\left(200+20 \\times \\frac{x}{0.2}\\right)-50=250$B. $(6-x-4)\\left(200+20 \\times \\frac{x}{0.2}\\right)-50=250$C. $(6-x-4)\\left(200+20 \\times \\frac{x}{0.2}\\right)=250$D. $(6-x-4)(200+20 x)-50=250$", "subject": "算术", "analysis": "【点拨】设每千克苹果的售价降低x元,根据总利润=单个的利润times销售量,列出方程即可.【解析】解:设应将每千克苹果的售价降低x元,根据题意得:(6-x-4)left(200+20timesfracx0.2right)-50=250,故选B.【点睛】本题主要考查了一元二次方程的应用,解题的关键是熟练掌握总利润=单个的利润times销售量."} {"id": "5713", "image": [], "answer": "D", "solution": "null", "level": "八年级", "question": "下列方程是一元二次方程的是 $(\\quad)$", "options": "A. $3 x+1=0$\nB. $5 x^{2}-6 y-3=0$\nC. $a x^{2}-x+2=0$\nD. $3 x^{2}-2 x-1=0$", "subject": "代数", "analysis": "A.是一元一次方程, 故本选项不符合题意;\n\nB. 是二元二次方程, 故本选项不符合题意;\n\nC.当 $a \\neq 0$ 时, 是一元二次方程, 当 $a=0$ 时, 是一元一次方程, 故本选项不符合题意;\n\nD.是一元二次方程, 故本选项符合题意.\n\n故答案为: D."} {"id": "5714", "image": [], "answer": "B", "solution": "null", "level": "八年级", "question": "若方程 $(a-b) x^{2}+(b-c) x+( c-a )=0$ 是关于 $x$ 的一元二次方程, 则必有 ( )", "options": "A. $a=b=c$\nB. 一根为 1\nC. 一根为 -1\nD. 以上都不对", "subject": "代数", "analysis": "A. 当 $a=b=c$ 时, $a-b=0, b-c=0$, 则式子不是方程, 故错误;\n\nB. 把 $x=1$ 代入方程的左边: $a-b+b-c+c-a=0$. 方程成立,所以 $x=1$ 是方程 ( $a-b$ ) $x^{2}+(b-c) x+(c-a)$ $=0$ 的解;\n\nC. 把 $x=-1$ 代入方程的左边: $a-b+c-b+c-a=2$ ( $c-b)=0$ 不一定成立, 故选项错误所以选 B."} {"id": "5724", "image": [], "answer": "A", "solution": "null", "level": "八年级", "question": "方程 $(x-1)^{2}=16$ 的解是 $(\\quad)$", "options": "A. $x_{1}=5, x_{2}=-3$\nB. $x_{1}=-5, x_{2}=4$\nC. $\\mathrm{x}_{1}=17, \\mathrm{x}_{2}=-15$\nD. $x_{1}=5, x_{2}=-5$", "subject": "代数", "analysis": "$(\\mathrm{x}-1)^{2}=16, \\therefore \\mathrm{x}-1= \\pm 4, \\therefore \\mathrm{x}_{1}=5, \\mathrm{x}_{2}=-3$. 故答案为: $\\mathrm{A}$."} {"id": "5735", "image": [], "answer": "C", "solution": "null", "level": "八年级", "question": "下列四个备选项所列的方程中, 其中有两个不相等实数根的方程是 ( )", "options": "A. $2 x^{2}+8=0$\nB. $x^{2}-6 x+9=0$\nC. $x^{2}-4 x-1=0$\nD. $2 x^{2}=-8 x-9$", "subject": "代数", "analysis": "A. $2 \\mathrm{x}^{2}+8=0, \\triangle=0-4 \\times 2 \\times 8=-64<0$, 方程无解; 故 A 不符合题意;\nB. $x^{2}-6 x+9=0, \\triangle=(-6)^{2}-4 \\times 9=0$, 方程有两个相等的实根; 故 B 不符合题意;\nC. $x^{2}-4 x-1=0, \\triangle=(-4)^{2}-4 \\times(-1)=20>0$, 方程有两个不相等的实根; 故 C 不符合题意;\nD. $2 x^{2}=-8 \\mathrm{x}-9,2 \\mathrm{x}^{2}+8 \\mathrm{x}+9=0, \\triangle=8^{2}-4 \\times 2 \\times 9=-8<0$, 方程无解; 故 $\\mathrm{D}$ 不符合题意;\n\n故答案为: C."} {"id": "5737", "image": [], "answer": "D", "solution": "null", "level": "八年级", "question": "下列方程中, 没有实数根的是 $(\\quad)$", "options": "A. $x^{2}+x=0$\nB. $x^{2}-2=0$\nC. $x^{2}+x-1=0$\nD. $x^{2}-x+1=0$", "subject": "代数", "analysis": "$A 、 x^{2}+x=0$ 中, $\\Delta=b^{2}-4 a c=1>0$, 有实数根;\n\nB、 $x^{2}-2=0$ 中, $\\triangle=b^{2}-4 a c=8>0$, 有实数根;\n\nC、 $x^{2}+x-1=0$ 中, $\\triangle=b^{2}-4 a c=5>0$, 有实数根;\n\nD、 $x^{2}-x+1=0$ 中, $\\triangle=b^{2}-4 a c=-3$, 没有实数根.\n\n故答案为: $D$.\nD."} {"id": "5738", "image": [], "answer": "D", "solution": "null", "level": "八年级", "question": "已知 $x_{1} 、 x_{2}$ 是一元二次方程 $x^{2}-2 x=0$ 的两个实数根, 下列结论错误的是 ( )", "options": "A. $x_{1} \\neq x_{2}$\nB. $x_{1}^{2}-2 x_{1}=0$\nC. $x_{1}+x_{2}=2$\nD. $x_{1} \\cdot x_{2}=2$", "subject": "代数", "analysis": "$\\mathrm{x}_{1} 、 \\mathrm{x}_{2}$ 是一元二次方程 $\\mathrm{x}^{2}-2 \\mathrm{x}=0$ 的两个实数根,\n\n这里 $a=1, b=-2, c=0$,\n\n$\\mathrm{b}^{2}-4 \\mathrm{ac}=(-2)^{2}-4 \\times 1 \\times 0=4>0$,\n\n所以方程有两个不相等的实数根, 即 $x_{1} \\neq x_{2}$, 故 $\\mathrm{A}$ 选项符合题意, 不符合题意; $x_{1}^{2}-2 x_{1}=0$, 故 B 选项符合题意, 不符合题意;\n$x_{1}+x_{2}=-\\frac{b}{a}=-\\frac{-2}{1}=2$, 故 $\\mathrm{C}$ 选项符合题意, 不符合题意;\n\n$x_{1} \\cdot x_{2}=\\frac{c}{a}=0$\n\n, 故 D 选项不符合题意, 符合题意,\n\n故答案为: D."} {"id": "5740", "image": [], "answer": "D", "solution": "null", "level": "八年级", "question": "已知关于 $\\mathrm{x}$ 的一元二次方程 $\\mathrm{x}^{2}-2 \\mathrm{x}-\\mathrm{k}=0$ 有两个不相等的实数根, 则实数 $\\mathrm{k}$ 的取值范围是 ( )", "options": "A. $\\mathrm{k} \\geq 1$\nB. $\\mathrm{k}>1$\nC. $\\mathrm{k} \\geq-1$\nD. $\\mathrm{k}>-1$", "subject": "代数", "analysis": "$\\because$ 关于 $\\mathrm{x}$ 的一元二次方程 $\\mathrm{x}^{2}-2 \\mathrm{x}-\\mathrm{k}=0$ 有两个不相等的实数根,\n\n$\\therefore \\Delta=(-2)^{2}+4 \\mathrm{k}>0$,\n\n解得 $\\mathrm{k}>-1$.\n\n故选: D."} {"id": "5741", "image": [], "answer": "A", "solution": "null", "level": "八年级", "question": "已知 $\\left(1-m^{2}-n^{2}\\right)\\left(m^{2}+n^{2}\\right)=-6$, 则 $\\mathrm{m}^{2}+\\mathrm{n}^{2}$ 的值是 $(\\quad)$", "options": "A. 3\nB. 3 或 -2\nC. 2 或 -3\nD. 2", "subject": "代数", "analysis": "设 $m^{2}+n^{2}=x$,\n\n原方程变形为 $(1-\\mathrm{x}) \\mathrm{x}=-6$,\n\n解得 $x=-2$ 或 3 ,\n\n$\\because m^{2}+n^{2} \\geq 0$,\n\n$\\therefore \\mathrm{x}=3$,\n\n$\\therefore \\mathrm{m}^{2}+\\mathrm{n}^{2}=3$.\n\n故选 A."} {"id": "5742", "image": [], "answer": "A", "solution": "null", "level": "八年级", "question": "方程 $-x^{2}+3 x=1$ 用公式法求解, 先确定 a , b , c 的值, 正确的是 ( )", "options": "A. $a=-1, b=3, c=-1$\nB. $a=-1, b=3, c=1$\nC. $\\mathrm{a}=-1, \\mathrm{~b}=-3, \\mathrm{c}=-1$\nD. $\\mathrm{a}=1, \\mathrm{~b}=-3, \\mathrm{c}=-1$", "subject": "代数", "analysis": "将 $-x^{2}+3 x=1$ 整理为一般形式得: $-x^{2}+3 x-1=0$,\n\n可得出 $a=-1, b=3, c=-1$.\n\n故选 A"} {"id": "5744", "image": [], "answer": "C", "solution": "null", "level": "八年级", "question": "已知 $\\mathrm{x}$ 为实数, 化简 $\\sqrt{-x^{3}}-x \\sqrt{-\\frac{1}{x}}$ 的结果为 ()", "options": "A. $(x-1) \\sqrt{-x}$\nB. $(-1-x) \\sqrt{-x}$\nC. $(1-x) \\sqrt{-x}$\nD. $(1+x) \\sqrt{-x}$", "subject": "代数", "analysis": "由原式成立, 所以 $\\mathrm{x}<0$, 所以原式 $=-x \\sqrt{-x}+\\sqrt{-x}=(1-x) \\sqrt{-x}$ ,故选 C."} {"id": "5765", "image": [], "answer": "B", "solution": "null", "level": "八年级", "question": "若单项式 $-3 x^{2} y^{2 m+n}$ 与 $2 x^{m+n} y^{4}$ 是同类项, 则 $m^{2}+2 m n$ 的算术平方根为", "options": "A. 0\nB. 2\nC. -2\nD. $\\pm 2$", "subject": "代数", "analysis": "$\\because$ 单项式 $-3 x^{2} y^{2 m+n}$ 与 $2 x^{m+n} y^{4}$ 是同类项,\n\n$\\therefore\\left\\{\\begin{array}{c}m+n=2 \\\\ 2 m+n=4\\end{array}\\right.$,\n\n$\\therefore\\left\\{\\begin{array}{l}m=2 \\\\ n=0\\end{array}\\right.$\n\n$\\therefore \\mathrm{m}^{2}+2 \\mathrm{mn}=4$,\n\n$\\therefore \\mathrm{m}^{2}+2 \\mathrm{mn}$ 的算术平方根为 2 .\n\n故答案为: B."} {"id": "5767", "image": [], "answer": "C", "solution": "null", "level": "八年级", "question": "下列运算正确的是 ( )", "options": "A. $a \\cdot a^{2}=a^{2}$\nB. $(a b)^{2}=a b$\nC. $3^{-1}=\\frac{1}{3}$\nD. $\\sqrt{5}+\\sqrt{5}=\\sqrt{10}$", "subject": "代数", "analysis": "$A 、$ 原式 $=a^{3}$, 所以 $A$ 选项不符合题意;\n\nB、原式 $=a^{2} b^{2} , \\quad$ 所以 $B$ 选项不符合题意;\n\nC、原式 $={ }^{\\frac{1}{3}}$, 所以 $\\mathrm{C}$ 选项符合题意;\n\nD、原式 $=2 \\sqrt{5}$, 所以 $\\mathrm{D}$ 选项不符合题意.\n\n故答案为: C."} {"id": "5768", "image": [], "answer": "A", "solution": "null", "level": "八年级", "question": "下列函数中白变量 $x$ 的取值范围是 $x>1$ 的是 ( )", "options": "A. $y=\\frac{1}{\\sqrt{x-1}}$\nB. $y=\\sqrt{x-1}$\nC. $y=\\frac{1}{\\sqrt{x}-1}$\nD. $y=\\frac{1}{\\sqrt{1-x}}$", "subject": "代数", "analysis": "A、二次根式和分式有意义, $x-1>0$, 解得 $x>1$, 符合题意;\n\nB、二次根式有意义, $x-1 \\geq 0$, 解得 $x \\geq 1$, 不符合题意;\n\nC、二次根式和分式有意义, $x \\geq 0$ 且 $\\sqrt{x}-1 \\neq 0$, 解得 $x \\geq 0$ 且 $x \\neq 1$, 不符合题意;\n\nD、二次根式和分式有意义 $1-x>0$, 解得 $x<1$, 不符合题意.\n\n故选 A."} {"id": "5769", "image": [], "answer": "A", "solution": "null", "level": "八年级", "question": "已知 $\\mathrm{y}=3 \\sqrt{x-2}+\\sqrt{2-x}+6$, 则 $\\mathrm{x}+\\mathrm{y}$ 的立方根是 ( )", "options": "A. 2\nB. -2\nC. $\\pm 2$\nD. 8", "subject": "代数", "analysis": "$\\because \\mathrm{y}=3 \\sqrt{x-2}+\\sqrt{2-x}+6, \\quad \\therefore \\mathrm{x}-2=0$, 则 $\\mathrm{x}=2$,\n\n故 $\\mathrm{y}=6$,\n\n则 $x+y=8$ 的立方根是: 2 .\n\n故选: A."} {"id": "5771", "image": [], "answer": "D", "solution": "null", "level": "八年级", "question": "下列根式中, 最简二次根式是 ( )", "options": "A. $\\sqrt{\\frac{x}{5}}$\nB. $\\sqrt{12 x}$\nC. $\\sqrt{7 x^{3}}$\nD. $\\sqrt{x^{2}+1}$", "subject": "代数", "analysis": "最简二次根式应满足: (1)被开方数不含分母; (2)被开方数中不含能开得尽方的因数或因式.A 选项中被开方数含有分母; $\\mathrm{B}$ 选项被开方数含有能开得尽方的因数 $4 ; \\mathrm{C}$ 选项被开方数含有能开得尽方的因式, 只有 $\\mathrm{D}$ 选项符合最简二次根式的两个条件, 故答案应选择 D."} {"id": "5772", "image": [], "answer": "C", "solution": "null", "level": "八年级", "question": "下列运算正确的有 ( )", "options": "A. $5 \\mathrm{ab}-\\mathrm{ab}=4$\nB. $3 \\sqrt{2}-\\sqrt{2}=3$\nC. $a^{6} \\div a^{3}=a^{3}$\nD. ${ }^{\\frac{1}{a}}+{ }^{\\frac{1}{b}}=\\frac{2}{a+b}$", "subject": "代数", "analysis": "A、5ab-ab=4ab, 故此选项错误, 不合题意;\n\nB、 $3 \\sqrt{2}-\\sqrt{2}=2 \\sqrt{2}$, 故此选项错误, 不合题意;\n\nC、 $a^{6} \\div \\mathrm{a}^{3}=\\mathrm{a}^{3}, \\quad$ 正确, 符合题意;\n\nD、 ${ }^{\\frac{1}{a}}+{ }^{\\frac{1}{b}}={ }^{\\frac{b}{a k}}+{ }^{\\frac{a}{a k}}=\\frac{a+b}{a b}$, 故此选项错误, 不合题意;\n\n故选: C."} {"id": "5813", "image": [], "answer": "D", "solution": "null", "level": "八年级", "question": "若三角形的三边分别是 $\\mathrm{a}, \\mathrm{b}, \\mathrm{c}$, 且 $(a-2 \\sqrt{5})^{2}+\\sqrt{a-b-1}+|c-4|=0=0$, 则这个三角形的周长是 $(\\quad)$", "options": "A. $2 \\sqrt{5}+5$\nB. $4 \\sqrt{5}-3$\nC. $4 \\sqrt{5}+5$\nD. $4 \\sqrt{5}+3$", "subject": "代数", "analysis": "由原式得 $\\mathrm{a}=2 \\sqrt{5}, \\mathrm{~b}=2 \\sqrt{5}-1, \\mathrm{c}=4$, 故此三角形的周长为 $2 \\sqrt{5}+2 \\sqrt{5}-1+4=4 \\sqrt{5}+3$, 故答案为: D"} {"id": "5826", "image": [], "answer": "C", "solution": "null", "level": "八年级", "question": "下列说法中正确的是 ( )", "options": "A. 使式子 $\\sqrt{x+3}$ 有意义的 $\\mathrm{x}$ 的取值范围是 $\\mathrm{x}>-3$\n\nB. 若正方形的边长为 $3 \\sqrt{10} \\mathrm{~cm}$,则面积为 $30 \\mathrm{~cm}^{2}$\n\nC. 使 $\\sqrt{12 n}$ 是正整数的最小整数 $\\mathrm{n}$ 是 3\n\nD. 计算 $3 \\div \\sqrt{3} \\times \\frac{1}{\\sqrt{3}}$ 的结果是 3", "subject": "代数", "analysis": "A. $\\because$ 使式子 $\\sqrt{x+3}$ 有意义的 $\\mathrm{x}$ 的取值范围是 $x \\geq-3$, 故不正确;\n\nB. $\\because$ 若正方形的边长为 $3 \\sqrt{10} \\mathrm{~cm}$,则面积为 $90 \\mathrm{c} \\mathrm{m}^{2}$, 故不正确;\n\nC. $\\because$ 使 $\\sqrt{12 n}$ 是正整数的最小整数 $\\mathrm{n}$ 是 3 ,故正确;\n\nD. $\\because$ 计算 $3 \\div \\sqrt{3} \\times \\frac{1}{\\sqrt{3}}$ 的结果是 1 , 故不正确;\n\n故选 C."} {"id": "5831", "image": ["7228.jpg"], "answer": "C", "solution": "null", "level": "八年级", "question": "已知 $b^{2}-4 a c$ 是一元二次方程 $a x^{2}+b x+c=0(a \\neq 0)$ 的一个实数根, 则 $a b$ 的取值范围为 ( )", "options": "A. $a b \\geq \\frac{1}{8}$\nB. $a b \\geq \\frac{1}{4}$\nC. $a b \\leq \\frac{1}{8}$\nD. $a b \\leq \\frac{1}{4}$", "subject": "代数", "analysis": "由题意知, 这 3 种多边形的 3 个内角之和为 360 度, 已知正多边形的边数为 $\\mathrm{x} 、 \\mathrm{y} 、 \\mathrm{z}$, 那么这三个多边形的内角和可表示为: $\\frac{(x-2) x_{180}}{x}+\\frac{(y-2) \\times 180}{y}+\\frac{(z-2) x_{180}}{z}=360$, 两边都除以 180 得: 1\n\n\n\n故答案为: C."} {"id": "4987", "image": [], "answer": "D", "solution": "null", "level": "八年级", "question": "下列式子中, 一定属于二次根式的是", "options": "A. $\\sqrt{-6}$B. $\\sqrt{x-2}$C. $\\sqrt[3]{9}$D. $\\sqrt{3}$", "subject": "代数", "analysis": "【点拨】根据二次根式的定义判断即可.【解析】解:A、被开方数不是非负数,没有意义,故该选项不符合题意;B、被开方数不能保证x-2是非负数,故该选项不符合题意;C、sqrt[3]9是三次根式,故该选项不符合题意;D、sqrt3是二次根式,故该选项符合题意.故选D.【点睛】本题考查了二次根式的定义,掌握二次根式的定义是解题的关键,一般地,我们把形如sqrta(ageqslant0)的式子叫做二次根式."} {"id": "4988", "image": [], "answer": "A", "solution": "null", "level": "八年级", "question": "若 $|a-2|+b^{2}+4 b+4+\\sqrt{c^{2}-c+\\frac{1}{4}}=0$, 则 $\\sqrt{\\mathrm{b}^{2}}-\\sqrt{\\mathrm{a}}-\\sqrt{\\mathrm{c}}$ 的值是()", "options": "A. $2-\\frac{3}{2} \\sqrt{2}$B. 4C. 1D. 8", "subject": "代数", "analysis": "【点拨】通过因式分解把|a-2|+b^2+4b+4+sqrtc^2-c+frac14=0化为|a-2|+(b+2)^2+sqrtleft(c-frac12right)^2=0,再根据非负数的性质求得a、b、c,进而代值计算原式便可.【解析】解:because|a-2|+b^2+4b+4+sqrtc^2-c+frac14=0,therefore|a-2|+(b+2)^2+sqrtleft(c-frac12right)^2=0,thereforea-2=0,b+2=0,c-frac12=0,thereforea=2,b=-2,c=frac12,thereforesqrtb^2-sqrta-sqrtc=2-sqrt2-fracsqrt22=2-frac32sqrt2.故选A.【点睛】本题考查了二次根式的化简求值,非负数的性质,关键是根据非负数性质求得a、b、c."} {"id": "5004", "image": [], "answer": "C", "solution": "null", "level": "八年级", "question": "代数式 $\\sqrt{6-2 x}$ 有意义,那么 $x$ 应满足的条件是 ( )", "options": "A. $x \\geqslant 3$B. $x<3$C. $x \\leqslant 3$D. $x \\neq 3$", "subject": "代数", "analysis": "【点拨】直接利用二次根式有意义的条件,被开方数是非负数,进而得出答案.【解析】解:because代数式sqrt6-2x有意义,therefore6-2xgeqslant0,解得:xleqslant3.故选C.【点睛】此题主要考查了二次根式有意义的条件,正确掌握相关定义是解题关键."} {"id": "5007", "image": [], "answer": "D", "solution": "null", "level": "八年级", "question": "下列各式计算正确的是", "options": "A. $\\sqrt{(-25) \\times(-36)}=\\sqrt{-25} \\times \\sqrt{-36}=-5 \\times(-6)=30$B. $\\sqrt{8 a^{2} b^{2}}=4 a^{2} b$C. $\\sqrt{5^{2}+4^{2}}=5+4=9$D. $\\sqrt{15^{2}-12^{2}}=\\sqrt{15+12} \\times \\sqrt{15-12}=9$", "subject": "代数", "analysis": "【点拨】根据二次根式的运算法则,进行计算逐一判断即可解答.【解析】解:A、sqrt(-25)times(-36)=sqrt25timessqrt36=5times6=30,故A不符合题意;B、sqrt8a^2~b^2=2|ab|sqrt2,故B不符合题意;C、sqrt5^2+4^2=sqrt41,故C不符合题意;D、sqrt15^2-12^2=sqrt15+12timessqrt15-12=9,故D符合题意;故选D.【点睛】本题考查了二次根式的混合运算,准确熟练地进行计算是解题的关键."} {"id": "5008", "image": [], "answer": "A", "solution": "null", "level": "八年级", "question": "若 $\\sqrt{x(x-6)}=\\sqrt{x} \\cdot \\sqrt{x-6}$ 则()", "options": "A. $x \\geqslant 6$B. $x \\geqslant 0$C. $0 \\leqslant x \\leqslant 6$D. $x$ 为一切实数", "subject": "代数", "analysis": "【点拨】利用二次根式的乘法法则和二次根式有意义的条件得到xgeqslant0且x-6geqslant0,然后求出两不等式的公共部分即可.【解析】解:根据题意得xgeqslant0且x-6geqslant0,所以xgeqslant6.故选A.【点睛】本题考查了二次根式的乘法:sqrtacdotsqrtb=sqrtab(ageqslant0,bgeqslant0);"} {"id": "5026", "image": [], "answer": "C", "solution": "null", "level": "八年级", "question": "计算 $\\sqrt{15} \\div 4 \\sqrt{3} \\times \\sqrt{\\frac{6}{5}}$ 的结果是 $(\\quad)$", "options": "A. 1B. $\\frac{3}{4}$C. $\\frac{1}{4} \\sqrt{6}$D. $\\frac{3}{2}$", "subject": "代数", "analysis": "【点拨】直接利用二次根式的乘除法运算法则化简,进而得出答案.【解析】解:sqrt15div4sqrt3timessqrtfrac65=fracsqrt54timessqrtfrac65=frac14timessqrtfrac65times5=frac14sqrt6.故选C.【点睛】此题主要考查了二次根式的乘除运算,正确化简二次根式是解题关键."} {"id": "5027", "image": [], "answer": "B", "solution": "null", "level": "八年级", "question": "若 $\\sqrt{(x-3)^{2}}=x-3$, 则 $x$ 的取值范围是 ( )", "options": "A. $x>3$B. $x \\geqslant 3$C. $x<3$D. $x \\leqslant 3$", "subject": "代数", "analysis": "【点拨】根据题意可知x-3geqslant0,直接解答即可.【解析】解:becausesqrt(x-3)^2=x-3,即x-3geqslant0,解得xgeqslant3,故选B.【点睛】考查二次根式的性质与化简,掌握二次根式的化简方法是解题的关键."} {"id": "5773", "image": ["7175.jpg", "7176.jpg", "7177.jpg", "7178.jpg"], "answer": "D", "solution": "null", "level": "八年级", "question": "下列图案中, 是中心对称图形的是 ( )", "options": "A.\n\n\nB.\n\n\nC.\n\n\nD.\n\n", "subject": "画法几何学", "analysis": "A.此图案不是中心对称图形, 是轴对称图形,不合题意;\n\nB.此图案不是中心对称图形, 是轴对称图形,不合题意;\n\nC.此图案不是中心对称图形, 是轴对称图形,不合题意;\n\nD.此图案是中心对称图形, 符合题意;\n\n故答案为: D."} {"id": "5803", "image": ["7206.jpg", "7207.jpg"], "answer": "B", "solution": "null", "level": "八年级", "question": "将正方形 $\\mathrm{ABCD}$ 绕点 $\\mathrm{A}$ 按逆时针方向旋转 $30^{\\circ}$, 得正方形 $\\mathrm{AB}_{1} \\mathrm{C}_{1} \\mathrm{D}_{1}, \\mathrm{~B}_{1} \\mathrm{C}_{1}$ 交 $\\mathrm{CD}$ 于点 $\\mathrm{E}$, $\\mathrm{AB}=\\sqrt{3}$, 则四边形 $\\mathrm{AB}_{1} \\mathrm{ED}$ 的内切圆半径为 $(\\quad)$", "options": "A. $\\frac{\\sqrt{3}+1}{2}$\nB. $\\frac{3-\\sqrt{3}}{2}$\nC. $\\frac{\\sqrt{3}+1}{3}$\nD. $\\frac{3-\\sqrt{3}}{3}$", "subject": "画法几何学", "analysis": "作 $\\angle \\mathrm{DAF}$ 与 $\\angle \\mathrm{AB} \\mathrm{B}_{1} \\mathrm{G}$ 的角平分线交于点 $\\mathrm{O}$, 过 $\\mathrm{O}$ 作 $\\mathrm{OF} \\perp \\mathrm{AB}_{1}$,\n\n则 $\\angle \\mathrm{OAF}=30^{\\circ}, \\angle \\mathrm{AB}_{1} \\mathrm{O}=45^{\\circ}$,\n\n故 $\\mathrm{B}_{1} \\mathrm{~F}=\\mathrm{OF}={ }^{\\frac{1}{2}} \\mathrm{OA}$,\n\n设 $B_{1} F=x$, 则 $A F=\\sqrt{3}-x$,\n\n故 $(\\sqrt{3}-x)^{2}+x^{2}=(2 x)^{2}$,\n\n\n\n$\\therefore$ 四边形 $\\mathrm{AB}_{1} \\mathrm{ED}$ 的内切圆半径为: $\\frac{-\\sqrt{3}+3}{2}$.\n\n故选: B.\n\n"} {"id": "5074", "image": ["6620.jpg"], "answer": "D", "solution": "null", "level": "八年级", "question": "如图, 有一张长 $12 \\mathrm{~cm}$, 宽 $9 \\mathrm{~cm}$ 的矩形纸片, 在它的四个角各剪去一个同样大小的小正方形, 然后折叠成一个无盖的长方体纸盒. 若纸盒的底面 (图中阴影部分) 面积是 $70 \\mathrm{~cm}^{2}$, 求剪去的小正方形的边长. 设剪去的小正方形的边长是 $x \\mathrm{~cm}$, 根据题意, 可列方程为 ( )", "options": "A. $12 \\times 9-4 \\times 9 x=70$B. $12 \\times 9-4 x^{2}=70$C. $(12-x)(9-x)=70$D. $(12-2 x)(9-2 x)=70$", "subject": "画法几何学", "analysis": "【点拨】设剪去的小正方形的边长是x~cm,则纸盒底面的长为(12-2x)cm,宽为(9-2x)cm,根据纸盒的底面(图中阴影部分)面积是70~cm^2,得出关于x的一元二次方程,从而得到答案.【解析】解:设剪去的小正方形的边长是x~cm,则纸盒底面的长为(12-2x)cm,宽为(9-2x)cm,because纸盒的底面(图中阴影部分)面积是70~cm^2,therefore(12-2x)(9-2x)=70,故选D.【点睛】本题考查一元二次方程解实际问题,读懂题意,找准等量关系,正确列出一元二次方程是解题的关键."} {"id": "5124", "image": ["6638.jpg", "6639.jpg", "6640.jpg", "6641.jpg"], "answer": "A", "solution": "null", "level": "八年级", "question": "下列图形中既是中心对称图形又是轴对称图形的是()", "options": "A.B.C.D.", "subject": "画法几何学", "analysis": "【点拨】根据轴对称图形和中心对称图形的定义判断即可.【解析】解:A.该图形既是中心对称图形,也是轴对称图形,符合题意;B.该图形是轴对称图形,不是中心对称图形,故此选项不合题意;C.该图形是轴对称图形,不是中心对称图形,故此选项不合题意;D.该图形是轴对称图形,不是中心对称图形,故此选项不合题意.故选A.【点睛】本题考查中心对称图形和轴对称图形的知识,掌握好中心对称图形与轴对称图形的概念.轴对称图形的关键是寻找对称轴,图形两部分折叠后可重合,中心对称图形是要寻找对称中心,图形旋转180^circ后与原图重合是解题的关键."} {"id": "5147", "image": ["6671.jpg", "6672.jpg", "6673.jpg", "6674.jpg"], "answer": "B", "solution": "null", "level": "八年级", "question": "如图图形中既是轴对称图形又是中心对称图形的是 ( )", "options": "A.B.C.D.", "subject": "画法几何学", "analysis": "【点拨】根据轴对称图形和中心对称图形的定义,逐项判断即可求解.【解析】解:A、是轴对称图形,但不是中心对称图形,不符合题意;B、既是轴对称图形又是中心对称图形,符合题意;C、是中心对称图形,但不是轴对称图形,不符合题意;D、是轴对称图形,但不是中心对称图形,不符合题意.故选B.【点睛】本题主要考查了轴对称图形和中心对称图形,熟练掌握如果一个图形沿着一条直线对折后两部分完全重合,这样的图形叫做轴对称图形;在平面内,把一个图形绕着某个点旋转180^circ,如果旋转后的图形能与原来的图形重合,那么这个图形叫做中心对称图形是解题的关键."} {"id": "5169", "image": ["6716.jpg", "6717.jpg"], "answer": "B", "solution": "null", "level": "八年级", "question": "如图, 在 $\\square A B C D$ 中, $\\angle B$ 是锐角, 点 $F$ 是 $A B$ 边的中点, $A E \\perp B C$ 于点 $E$, 连接 $D F, E F$, 若 $\\angle$ $E F D=90^{\\circ}, A D=2, A B=\\sqrt{6}$, 则 $A E$ 长为 $(\\quad)$", "options": "A. 2B. $\\sqrt{5}$C. $\\frac{3}{2} \\sqrt{2}$D. $\\frac{3}{2} \\sqrt{3}$", "subject": "画法几何学", "analysis": "【点拨】延长EF交DA的延长线于Q,连接DE,设BE=x.首先证明DQ=DE=x+2,利用勾股定理构建方程即可解决问题.【解析】解:如图,延长EF交DA的延长线于Q,连接DE,设BE=x,because四边形ABCD是平行四边形,thereforeDQ//BC,thereforeangleAQF=angleBEF,becauseAF=FB,angleAFQ=angleBFE,thereforetriangleQFAcongtriangleEFB(AAS),thereforeAQ=BE=x,QF=EF,becauseangleEFD=90^circ,thereforeDFperpQE,thereforeDQ=DE=x+2,becauseAEperpBC,BC//AD,thereforeAEperpAD,thereforeangleAEB=angleEAD=90^circ,becauseAE^2=DE^2-AD^2=AB^2-BE^2,therefore(x+2)^2-2^2=(sqrt6)^2-x^2,整理得:x^2+2x-3=0,解得x=1或x=-3(舍去),thereforeBE=1,thereforeAE=sqrtAB^2-BE^2=sqrt(sqrt6)^2-1^2=sqrt5,故选B.【点睛】本题考查平行四边形的性质,线段的垂直平分线的性质,勾股定理,全等三角形的判定和性质等知识,解题的关键是学会添加常用辅助线,构造全等三角形解决问题,属于中考选择题中的压轴题."} {"id": "5208", "image": ["6780.jpg"], "answer": "C", "solution": "null", "level": "八年级", "question": "如图, 将矩形纸片 $A B C D$ 沿 $B E$ 折叠, 使点 $A$ 落在对角线 $B D$ 上的 $A^{\\prime}$ 处. 若 $\\angle D B C=24^{\\circ}$, 则 $\\angle$ $A^{\\prime} E B$ 等于 $(\\quad)$", "options": "A. $66^{\\circ}$B. $60^{\\circ}$C. $57^{\\circ}$D. $48^{\\circ}$", "subject": "画法几何学", "analysis": "【点拨】由矩形的性质得angleA=angleABC=90^circ,由折叠的性质得angleBA^primeE=angleA=90^circ,angleA^primeBE=angleABE=frac12left(90^circ-angleDBCright)=33^circ,即可得出答案.【解析】解:because四边形ABCD是矩形,thereforeangleA=angleABC=90^circ,由折叠的性质得:angleBA^primeE=angleA=90^circ,angleA^primeBE=angleABE,thereforeangleA^primeBE=angleABE=frac12left(90^circ-angleDBCright)=frac12left(90^circ-24^circright)=33^circ,thereforeangleA^primeEB=90^circ-angleA^primeBE=90^circ-33^circ=57^circ.故选C.【点睛】本题考查了矩形的性质、折叠的性质以及直角三角形的性质;熟练掌握矩形的性质和折叠的性质是解题的关键."} {"id": "5209", "image": ["6781.jpg"], "answer": "C", "solution": "null", "level": "八年级", "question": "一个四边形顺次添加下列条件中的三个条件便得到正方形:a. 两组对边分别相等b. 一组对边平行且相等c. 一组邻边相等d. 一个角是直角顺次添加的条件: (1) $a \\rightarrow c \\rightarrow d(2) b \\rightarrow d \\rightarrow c(3) a \\rightarrow b \\rightarrow c$则正确的是 $(\\quad)$", "options": "A. 仅 $(1)$B. 仅(3)C. (1) (2)D. (2)(3)", "subject": "画法几何学", "analysis": "【点拨】(1)由条件a可得到四边形是平行四边形,添加c得到平行四边形是菱形,再添加d得到菱形是正方形,(1)正确;(2)由条件b得到四边形是平行四边形,添加d平行四边形是矩形,再添加c矩形是正方形,(2)正确;(3)由a和b都可得到四边形是平行四边形,再添加c得到平行四边形是菱形,不能得到四边形是正方形,(3)不正确.【解析】解:(1)由a得到两组对边分别相等的四边形是平行四边形,添加c即一组邻边相等的平行四边形是菱形,再添加d即一个角是直角的菱形是正方形,故①正确;(2)由b得到一组对边平行且相等的四边形是平行四边形,添加d即有一个角是直角的平行四边形是矩形,再添加c即一组邻边相等的矩形是正方形,故(2)正确;(3)由a得到两组对边分别相等的四边形是平行四边形,添加b得到一组对边平行且相等的平行四边形仍是平行四边形,再添加c即一组邻边相等的平行四边形是菱形,不能得到四边形是正方形,故(3)不正确;故选C.【点睛】本题主要考查了正方形的判定,熟练掌握正方形的判定方法是解决问题的关键."} {"id": "5774", "image": ["7179.jpg", "7180.jpg"], "answer": "C", "solution": "null", "level": "八年级", "question": "如图, $\\triangle A O B$ 的外角 $\\angle C A B, \\angle D B A$ 的平分线 $A P, B P$ 相交于点 $\\mathrm{P}, P E \\perp O C$ 于 $\\mathrm{E}, P F \\perp O D$ 于 $\\mathrm{F}$, 下列结论: (1) $P E=P F$; (2) 点 $\\mathrm{P}$ 在 $\\angle C O D$ 的平分线上; (3) $\\angle A P B=90^{\\circ}-\\angle O$,其中正确的有 $(\\quad)$\n\n", "options": "A. 0 个\nB. 1 个\nC. 2 个\nD. 3 个", "subject": "度量几何学", "analysis": "过点 $\\mathrm{P}$ 作 $\\mathrm{PG} \\perp \\mathrm{AB}$, 如图:\n\n\n\n$\\because \\mathrm{AP}$ 平分 $\\angle \\mathrm{CAB}, \\mathrm{BP}$ 平分 $\\angle \\mathrm{DBA}, P E \\perp O C, P F \\perp O D, \\mathrm{PG} \\perp \\mathrm{AB}$,\n\n$\\therefore P E=P G=P F$; 故 (1) 正确;\n$\\therefore$ 点 $\\mathrm{P}$ 在 $\\angle C O D$ 的平分线上; 故(2)正确;\n\n$\\because \\angle A P B=\\angle A P G+\\angle B P G=\\frac{1}{2} \\angle E P F$,\n\n又 $\\angle E P F+\\angle O=180^{\\circ}$,\n\n$\\therefore \\angle A P B=\\frac{1}{2} \\times\\left(180^{\\circ}-\\angle O\\right)=90^{\\circ}-\\frac{1}{2} \\angle O$; 故 (3) 错误;\n\n$\\therefore$ 正确的选项有 2 个;\n\n故答案为: C."} {"id": "5800", "image": ["7203.jpg"], "answer": "C", "solution": "null", "level": "八年级", "question": "在 $\\triangle A B C D$ 中, 若 $\\angle A+\\angle C=80^{\\circ}$, 则 $\\angle B$ 的度数为 ( )", "options": "A. $100^{\\circ}$\nB. $130^{\\circ}$\nC. $140^{\\circ}$\nD.\n\n$150^{\\circ}", "subject": "度量几何学", "analysis": "如图,\n\n\n由 $=A B C D$ 可知, $\\mathrm{AD} / / \\mathrm{BC}, \\angle \\mathrm{A}=\\angle \\mathrm{C}$,\n\n$\\therefore \\angle \\mathrm{A}+\\angle \\mathrm{B}=180^{\\circ}$,\n\n$\\because \\angle A+\\angle C=80^{\\circ}$\n\n$\\therefore \\angle A=\\angle C=40^{\\circ}$,\n\n$\\therefore \\angle B=180^{\\circ}-\\angle A=180^{\\circ}-40^{\\circ}=140^{\\circ}$,\n\n故答案为: C.\n\n【分析】通过平行四边形的性质可得到 $\\angle \\mathrm{A}=\\angle \\mathrm{C}, \\angle \\mathrm{A}+\\angle \\mathrm{B}=180^{\\circ}$, 从而可求出 $\\angle B$ 的度数."} {"id": "5827", "image": [], "answer": "C", "solution": "null", "level": "八年级", "question": "已知四边形 $\\mathrm{ABCD}$ 中, $\\angle \\mathrm{A}$ 与 $\\angle \\mathrm{B}$ 互补, $\\angle \\mathrm{D}=70^{\\circ}$, 则 $\\angle \\mathrm{C}$ 的度数为 ( )", "options": "A. $70^{\\circ}$\nB. $90^{\\circ}$\nC. $110^{\\circ}$\nD. $140^{\\circ}$", "subject": "度量几何学", "analysis": "$\\because \\angle \\mathrm{A}$ 与 $\\angle \\mathrm{B}$ 互补, $\\quad \\therefore \\angle \\mathrm{A}+\\angle \\mathrm{B}=180^{\\circ}$,\n\n$\\because \\angle \\mathrm{A}+\\angle \\mathrm{B}+\\angle \\mathrm{C}+\\angle \\mathrm{D}=180^{\\circ}$,\n\n$\\because \\angle \\mathrm{D}=70^{\\circ}$,\n\n$\\therefore \\angle \\mathrm{C}=110^{\\circ}$,\n\n故选 C."} {"id": "5828", "image": ["7226.jpg"], "answer": "A", "solution": "null", "level": "八年级", "question": "如图, 某小区计划在一块长为 $32 \\mathrm{~m}$, 宽为 $20 \\mathrm{~m}$ 的矩形空地上修建三条同样宽的道路, 剩余的空地上种植草坪, 使草坪的面积为 $570 \\mathrm{~m}^{2}$, 若设道路的宽为 $x \\mathrm{~m}$, 则下面所列方程正确的是 $(\\quad)$", "options": "A. $(32-2 x)(20-x)=570$\nB. $32 x+2 \\times 20 x=32 \\times 20-570$\nC. $(32-x)(20-x)=32 \\times 20-570$\nD. $32 x+2 \\times 20 x-2 x^{2}=570$\n\n", "subject": "度量几何学", "analysis": "设道路的宽为 $\\mathrm{xm}$, 根据题意得: $(32-2 x)(20-x)=570$,\n\n故答案为: A."} {"id": "5829", "image": ["7227.jpg"], "answer": "A", "solution": "null", "level": "八年级", "question": "实数 $\\mathrm{a}, \\mathrm{b}$ 在数轴上对应点的位置如图所示, 化简 $|\\mathrm{a}|+\\sqrt{(a-b)^{2}}$ 的结果是()\n\n", "options": "A. $-2 a+b$\nB. $2 \\mathrm{a}-\\mathrm{b}$\nC. $-b$\nD. b", "subject": "度量几何学", "analysis": "如图所示: $\\mathrm{a}<0, \\mathrm{a}-\\mathrm{b}<0$, 则 $|\\mathrm{a}|+\\sqrt{(a-b)^{2}}$\n\n$$\n\\begin{aligned}\n& =-a-(a-b) \\\\\n& =-2 a+b\n\\end{aligned}\n$$\n\n故选: A."} {"id": "5009", "image": ["6613.jpg"], "answer": "C", "solution": "null", "level": "八年级", "question": "如图, 在一个长方形中无重叠的放入面积分别为 $32 \\mathrm{~cm}^{2}$ 和 $2 \\mathrm{~cm}^{2}$ 的两张正方形纸片, 则图中阴影部分的面积为 $(\\quad)$", "options": "A. $3.2 \\mathrm{~cm}^{2}$B. $6 \\sqrt{2} \\mathrm{~cm}^{2}$C. $6 \\mathrm{~cm}^{2}$D. $12 \\mathrm{~cm}^{2}$", "subject": "度量几何学", "analysis": "【点拨】根据题目中的数据和图形,可以写出阴影部分的长和宽,然后即可计算出阴影部分的面积.【解析】解:由图可知,阴影部分的长为sqrt32-sqrt2=4sqrt2-sqrt2=3sqrt2(~cm),宽为:sqrt2~cm,therefore阴影部分的面积为:3sqrt2timessqrt2=6left(~cm^2right),故选C.【点睛】本题考查二次函数的应用,解答本题的关键是明确题意,求出阴影部分的长和宽."} {"id": "5011", "image": [], "answer": "C", "solution": "null", "level": "八年级", "question": "等腰三角形的两条边分别为 $2 \\sqrt{3}$ 和 $3 \\sqrt{2}$, 则这个三角形的周长为 ( )", "options": "A. $4 \\sqrt{3}+3 \\sqrt{2}$B. $2 \\sqrt{3}+6 \\sqrt{2}$C. $4 \\sqrt{3}+3 \\sqrt{2}$ 或 $2 \\sqrt{3}+6 \\sqrt{2}$D. $4 \\sqrt{3}+6 \\sqrt{2}$ 或 $2 \\sqrt{3}+6 \\sqrt{2}$", "subject": "度量几何学", "analysis": "【点拨】分2sqrt3是腰长和底边两种情况讨论求解。【解析】解:2sqrt3是腰长时,三角形的三边分别为2sqrt3、2sqrt3、3sqrt2,能组成三角形,周长=2sqrt3+2sqrt3+3sqrt2=4sqrt3+3sqrt2;2sqrt3是底边时,三角形的三边分别为2sqrt3、3sqrt2、3sqrt2,能组成三角形,周长=2sqrt3+3sqrt2+3sqrt2=2sqrt3+6sqrt2,综上所述,这个三角形的周长为4sqrt3+3sqrt2或2sqrt3+6sqrt2.故选C.【点睛】本题考查了二次根式的应用,主要利用了同类二次根式的加减运算和等腰三角形的性质,难点在于分情况讨论."} {"id": "5055", "image": ["6618.jpg"], "answer": "A", "solution": "null", "level": "八年级", "question": "如图, 学校课外小组的试验园地是长 20 米、宽 15 米的矩形, 为便于管理, 现要在中间开辟一横两纵共三条等宽的小道, 使种植面积为 252 平方米, 设小道宽为 $x$ 米, 根据题意, 下面所列方程正确的是 $(\\quad)$", "options": "A. $(20-2 x)(15-x)=252$B. $(20-x)(15-x)=252$C. $(20-x)(15-2 x)=252$D. $(20-2 x)(15-2 x)=252$", "subject": "度量几何学", "analysis": "【点拨】根据题意知:小道的宽为x米,图形可以变换成如图的形状,种植面积和图中蓝色矩形的面积相等,而蓝色矩形的长、宽分别为(20-2x)、(15-x),根据矩形的面积公式就可以列出方程即可.【解析】解:根据题意知:小道的宽为x米,则(20-2x)(15-x)=252,故选A.【点睛】本题考查了一元二次方程的应用,找准等量关系,正确列出一元二次方程是解题的关键."} {"id": "5140", "image": ["6662.jpg"], "answer": "C", "solution": "null", "level": "八年级", "question": "如图, 平行四边形 $A B C D$ 的对角线 $A C, B D$ 相交于点 $O$, 则下列说法一定正确的是 ( )", "options": "A. $A O=O B$B. $A O \\perp O D$C. $A O=O C$D. $A O \\perp A B$", "subject": "度量几何学", "analysis": "【点拨】由平行四边形的性质容易得出结论.【解析】解:because四边形ABCD是平行四边形,thereforeOA=OC;故选C.【点睛】本题考查了平行四边形的性质;熟记平行四边形的对角线互相平分是解决问题的关键."} {"id": "5141", "image": ["6663.jpg"], "answer": "D", "solution": "null", "level": "八年级", "question": "如图, 平行四边形 $A B C D$ 的周长为 $40, \\triangle B O C$ 的周长比 $\\triangle A O B$ 的周长多 10 , 则 $A B$ 长为 ( )", "options": "A. 20B. 15C. 10D. 5", "subject": "度量几何学", "analysis": "【点拨】由于平行四边形的对角线互相平分,那么triangleAOB、triangleBOC的周长差,实际是AB、BC的差,联立平行四边形的周长,即可得解.【解析】解:because,triangleBOC的周长比triangleAOB的周长多10,即BC-AB=10,because平行四边形ABCD的周长是40,即BC+AB=20,thereforeAB=5.故选D.【点睛】本题考查平行四边形的性质,比较简单,关键是利用平行四边形的性质解题:平行四边形的对角线互相平分."} {"id": "5143", "image": ["6664.jpg", "6665.jpg"], "answer": "D", "solution": "null", "level": "八年级", "question": "已知, 在平行四边形 $A B C D$ 中, $\\angle A$ 的平分线分 $B C$ 成 $4 \\mathrm{~cm}$ 和 $3 \\mathrm{~cm}$ 两条线段, 则平行四边形 $A B C D$的周长为 $(\\quad) \\mathrm{cm}$.", "options": "A. 11B. 22C. 20D. 20 或 22", "subject": "度量几何学", "analysis": "【点拨】设angleA的平分线交BC于点E,可证明AB=EB,再分两种情况讨论,一是EB=4~cm,EC=3~cm,则AB=EB=4~cm,BC=EB+EC=7~cm;二是EB=3~cm,EC=4~cm时,则AB=EB=3~cm,BC=EB+EC=7~cm,分别求出平行四边形ABCD的周长即可.【解析】解:设angleA的平分线交BC于点E,because四边形ABCD是平行四边形,thereforeBC//AD,thereforeangleBEA=angleDAE,becauseangleBAE=angleDAE,thereforeangleBEA=angleBAE,thereforeAB=EB,当EB=4~cm,EC=3~cm时,如图1,则AB=EB=4~cm,BC=EB+EC=7~cm,therefore2AB+2BC=2times4+2times7=22(~cm);当EB=3~cm,EC=4~cm时,如图2,则AB=EB=3~cm,BC=EB+EC=7~cm,therefore2AB+2BC=2times3+2times7=20(~cm),therefore平行四边形ABCD的周长为22~cm或20~cm,故选D.图1图2【点睛】此题重点考查平行四边形的性质、平行线的性质、等腰三角形的判定等知识,熟练掌握平行四边形的性质和等腰三角形的判定是解题的关键."} {"id": "5146", "image": ["6670.jpg"], "answer": "A", "solution": "null", "level": "八年级", "question": "如图, 四边形 $A B C D$ 是平行四边形, $O$ 是对角线 $A C$ 与 $B D$ 的交点, $A B \\perp A C$, 若 $A B=8, A C=12$,则 $B D$ 的长是", "options": "A. 20B. 21C. 22D. 23", "subject": "度量几何学", "analysis": "【点拨】由四边形ABCD是平行四边形,根据平行四边形的对角线互相平分,可得OA的长,然后由ABperpAC,AB=8,AC=12,根据勾股定理可求得OB的长,继而求得答案.【解析】解:because四边形ABCD是平行四边形,AC=12,thereforeOA=frac12AC=6,BD=2OB,becauseABperpAC,AB=8,thereforeOB=sqrtO~A^2+AB^2=10,thereforeBD=2OB=20.故选A.【点睛】此题考查了平行四边形的性质以及勾股定理.注意掌握平行四边形的对角线互相平分."} {"id": "5148", "image": ["6675.jpg", "6676.jpg"], "answer": "C", "solution": "null", "level": "八年级", "question": "如图, $F$ 是 $\\square A B C D$ 的边 $C D$ 上的点, $Q$ 是 $B F$ 中点, 连接 $C Q$ 并延长交 $A B$ 于点 $E$, 连接 $A F$ 与 $D E$ 相交于点 $P$, 若 $S_{\\triangle A P D}=2 \\mathrm{~cm}^{2}, S_{\\triangle B Q C}=8 \\mathrm{~cm}^{2}$, 则阴影部分的面积为 $(\\quad) \\mathrm{cm}^{2}$", "options": "A. 24B. 17C. 18D. 10", "subject": "度量几何学", "analysis": "【点拨】连接EF,证明四边形EBCF是平行四边形,求出S_triangleBEF=16cm^2,再得出S_triangleAPD=S_triangleEPF=2~cm^2即可求出阴影部分的面积.【解析】解:连接EF,becauseF是squareABCD的边CD上的点,thereforeBE//CF,thereforeangleEBF=angleCFB,quadangleBEC=angleFCE,becauseBQ=FQ,thereforetriangleEBQcongtriangleCFQ,thereforeEQ=CQ,therefore四边形EBCF是平行四边形,thereforeS_triangleBEF=2S_triangleBQC=16~cm^2,becauseS_triangleAED=S_triangleAEF,thereforeS_triangleAPD=S_triangleEPF=2~cm^2,thereforeS_text阴影=S_triangleEPF+S_triangleEEF=18~cm^2,故选C.【点睛】本题考查了平行四边形的性质与判定,熟练运用平行四边形的性质与判定进行证明与计算是解题的关键."} {"id": "5164", "image": ["6711.jpg", "6712.jpg"], "answer": "B", "solution": "null", "level": "八年级", "question": "如图, 五边形 $A B C D E$ 中, $A B / / C D, \\angle 1 、 \\angle 2 、 \\angle 3$ 是外角, 则 $\\angle 1+\\angle 2+\\angle 3$ 等于 ( )", "options": "A. $100^{\\circ}$B. $180^{\\circ}$C. $210^{\\circ}$D. $270^{\\circ}$", "subject": "度量几何学", "analysis": "【点拨】先根据平行线的性质得出angle4+angle5=180^circ,再由多边形的外角和为360^circ即可得出结论.【解析】解:延长AB,DC,becauseAB//CD,thereforeangle4+angle5=180^circbecause多边形的外角和为360^circ,thereforeangle1+angle2+angle3+angle4+angle5=360^circ,thereforeangle1+angle2+angle3=360^circ-(angle4+angle5)=360^circ-180^circ=180^circ.故选B.【点睛】本题考查的是多边形的外角与内角,熟知多边形的外角和等于360^circ是解题的关键."} {"id": "5165", "image": [], "answer": "C", "solution": "null", "level": "八年级", "question": "四边形 $A B C D$ 中, 对角线 $A C 、 B D$ 交于点 $O$, 给出下列四组条件, 一定能判定四边形 $A B C D$ 是平行四边形的条件有(1) $A B / / C D, A D / / B C$ (2) $A B / / C D, \\angle A=\\angle C$ (3) $A O=C O, B O=D O$ (4) $A B / / C D, A D=B C$.", "options": "A. 1 组B. 2 组C. 3 组D. 4 组", "subject": "度量几何学", "analysis": "【点拨】根据平行四边形的判断定理可作出判断.【解析】解:(1)根据平行四边形的判定定理:二组对边分别平行的四边形是平行四边形,可知(1)能判断这个四边形是平行四边形;(2)根据条件可以证明二组对边分别平行,可知(2)能判断这个四边形是平行四边形;(3)根据平行四边形的判定定理:两条对角线互相平分的四边形是平行四边形,可知(3)能判断这个四边形是平行四边形;(4)可能是等腰梯形,知(4)不能判断这个四边形是平行四边形;故给出下列四组条件中,(1)(2)(3)能判断这个四边形是平行四边形,故选C.【点睛】此题主要考查了平行四边形的判定定理,准确无误的掌握定理是做题的关键."} {"id": "5166", "image": ["6713.jpg"], "answer": "A", "solution": "null", "level": "八年级", "question": "如图, 在 Rt $\\triangle A B C$ 中, $\\angle C=90^{\\circ}, D 、 E$ 分别为 $C A 、 C B$ 的中点, $A F$ 平分 $\\angle B A C$, 交 $D E$ 于点 $F$,若 $A C=3, B C=4$, 则 $E F$ 的长为 $(\\quad)$", "options": "A. 1B. $\\frac{1}{2}$C. 2D. $\\frac{3}{2}$", "subject": "度量几何学", "analysis": "【点拨】根据勾股定理得到AB=sqrtAC^2+BC^2=5,根据三角形中位线定理得到DE//AB,DE=frac12AB=frac52,根据平行线的性质得到angleDFA=angleFAB,根据角平分线的定义得到angleDAF=angleBAF,求得angleDAF=angleDFA,于是得到结论.【解析】解:在RttriangleABC中,angleC=90^circ,AC=3,BC=4,thereforeAB=sqrtAC^2+BC^2=5,becauseD、E分别为CA、CB的中点,thereforeDE是triangleABC的中位线,thereforeDE//AB,DE=frac12AB=frac52,thereforeangleDFA=angleFAB,becauseAF平分angleBAC,thereforeangleDAF=angleBAFthereforeangleDAF=angleDFAthereforeDF=AD=frac12AC=frac12times3=frac32,thereforeEF=DE-DF=1,故选A.【点睛】本题考查的是三角形中位线定理、勾股定理、平行线的性质,掌握三角形的中位线平行于第三边,且等于第三边的一半是解题的关键."} {"id": "5184", "image": ["6736.jpg"], "answer": "B", "solution": "null", "level": "八年级", "question": "如图, 在矩形 $A B C D$ 中, 对角线 $A C, B D$ 相交于点 $O, \\angle A C B=30^{\\circ}$, 则 $\\angle A O B$ 的大小为 ( )", "options": "A. $30^{\\circ}$B. $60^{\\circ}$C. $90^{\\circ}$D. $120^{\\circ}$", "subject": "度量几何学", "analysis": "【点拨】根据矩形的对角线互相平分且相等可得OB=OC,再根据等边对等角可得angleOBC=angleACB,然后根据三角形的一个外角等于与它不相邻的两个内角的和列式计算即可得解.【解析】解:because矩形ABCD的对角线AC,BD相交于点O,thereforeOB=OC,thereforeangleOBC=angleACB=30^circ,thereforeangleAOB=angleOBC+angleACB=30^circ+30^circ=60^circ.故选B.【点睛】本题考查了矩形的性质,等边对等角的性质以及三角形的一个外角等于与它不相邻的两个内角的和的性质,熟记各性质是解题的关键."} {"id": "5187", "image": ["6738.jpg"], "answer": "B", "solution": "null", "level": "八年级", "question": "如图, 在矩形 $A B C D$ 中, 对角线 $A C, B D$ 相交于点 $O, D E \\perp A C$ 于点 $E, \\angle E D C: \\angle E D A=1: 2$,且 $A C=8$, 则 $E C$ 的长度为 $(\\quad)$", "options": "A. $2 \\sqrt{3}$B. 2C. 4D. $\\sqrt{3}$", "subject": "度量几何学", "analysis": "【点拨】根据angleEDC:angleEDA=1:2,可得angleEDC=30^circ,angleEDA=60^circ,进而得出DC=frac12AC,进而求得CE的长.【解析】解:because四边形ABCD是矩形,thereforeangleADC=90^circ,AC=BD=8,OA=OC=frac12AC=4,OB=OD=frac12BD=4,thereforeOC=OD,thereforeangleODC=angleOCD,becauseangleEDC:angleEDA=1:2,angleEDC+angleEDA=90^circ,thereforeangleEDC=30^circ,angleEDA=60^circ,becauseDEperpAC,thereforeangleDEC=90^circ,thereforeangleDAC=30^circ,thereforeDC=frac12AC=4,thereforeEC=frac12DC=2,故选B.【点睛】本题考查了直角三角形的性质和矩形的性质,根据已知得出angleDAC=30^circ是解题关键."} {"id": "5189", "image": ["6739.jpg", "6740.jpg"], "answer": "A", "solution": "null", "level": "八年级", "question": "如图, 在边长为 1 的正方形 $A B C D$ 中, $\\angle C A D$ 的平分线交 $C D$ 于点 $E$, 交 $B C$ 的延长线于点 $F$,则 $D E$ 的长为 $(\\quad)$", "options": "A. $\\sqrt{2}-1$B. $\\frac{\\sqrt{2}}{4}$C. $\\frac{\\sqrt{2}}{3}$D. $2-\\sqrt{2}$", "subject": "度量几何学", "analysis": "【点拨】设DE的长为x,过点E作EGperpAC于点G,根据角平分线上的点到角两边的距离相等可得EG=ED=x,再根据正方形的性质可得triangleEGC是等腰直角三角形,可得EC=sqrt2x,根据DC=DE+EC=x+sqrt2x=1,从而求出x的值,即DE的长.【解析】解:过点E作EGperpAC于点G,如图所示,设DE的长为x,because四边形ABCD是正方形,thereforeangleD=90^circ,angleACD=45^circ,CD=1.becauseEGperpAC,且AE平分angleCAD,thereforeEG=DE=x.在triangleEGC中,angleEGC=90^circ,angleECG=45^circ.thereforeangleCEG=angleECG=45^circ,thereforeCG=EG=x,thereforeEC=sqrtEG^2+CG^2=sqrtx^2+x^2=sqrt2x.thereforeDC=DE+EC=x+sqrt2x=1.解得x=sqrt2-1.thereforeDE的长为sqrt2-1.故选A.【点睛】本题主要考查了正方形的性质、角平分线的性质等,利用角平分线的性质添加辅助线是解题的关键."} {"id": "5190", "image": ["6741.jpg", "6742.jpg"], "answer": "A", "solution": "null", "level": "八年级", "question": "如图, 已知点 $P$ 是矩形 $A B C D$ 内一点 (不含边界), 设 $\\angle P A D=\\theta_{1}, \\angle P B A=\\theta_{2}, \\angle P C B=\\theta_{3}, \\angle$ $P D C=\\theta_{4}$, 若 $\\angle A P B=80^{\\circ}, \\angle C P D=50^{\\circ}$, 则 $(\\quad)$", "options": "A. $\\left(\\theta_{1}+\\theta_{4}\\right)-\\left(\\theta_{2}+\\theta_{3}\\right)=30^{\\circ}$B. $\\left(\\theta_{2}+\\theta_{4}\\right)-\\left(\\theta_{1}+\\theta_{3}\\right)=40^{\\circ}$C. $\\left(\\theta_{1}+\\theta_{2}\\right)-\\left(\\theta_{3}+\\theta_{4}\\right)=70^{\\circ}$D. $\\left(\\theta_{1}+\\theta_{2}\\right)+\\left(\\theta_{3}+\\theta_{4}\\right)=180^{\\circ}$", "subject": "度量几何学", "analysis": "【点拨】依据矩形的性质以及三角形内角和定理,可得theta_2-theta_1=10^circ,theta_4-theta_3=40^circ,两式相减即可得到left(theta_1+theta_4right)-left(theta_2+theta_3right)=30^circ.【解析】解:because矩形ABCD,thereforeangleBAD=angleBCD=90^circthereforeangleBAP=90^circ-theta_1,angleDCP=90^circquad-theta_3,thereforetriangleABP中,90^circ-theta_1+theta_2+80^circ=180^circ,即theta_2-theta_1=10^circ,(1)triangleDCP中,90^circ-theta_3+theta_4+50^circ=180^circ,即theta_4-theta_3=40^circ,(2)由(2)-(1),可得left(theta_4-theta_3right)-left(theta_2-theta_1right)=30^circ,即left(theta_1+theta_4right)-left(theta_2+theta_3right)=30^circ,故选A.【点睛】本题主要考查了矩形的性质以及三角形内角和定理的运用,解决问题的关键是掌握:矩形的四个角都是直角."} {"id": "5202", "image": ["6766.jpg"], "answer": "A", "solution": "null", "level": "八年级", "question": "如图, 点 $E 、 F$ 在矩形 $A B C D$ 的对角线 $B D$ 所在的直线上, $B E=D F$, 则四边形 $A E C F$ 是 ( )", "options": "A. 平行四边形B. 矩形C. 菱形D. 正方形", "subject": "度量几何学", "analysis": "【点拨】根据对角线互相平分可判断A;根据对角线不相等的平行四边形不是矩形可判断B,D;根据无法证明对角线互相垂直可判断C.【解析】解:A.because四边形ABCD是矩形,thereforeAO=CO,BO=DO,becauseBE=DF,thereforeEO=FO,therefore四边形AECF是平行四边形,故本选项符合题意;B.because四边形ABCD是矩形,thereforeAC=BD,thereforeACneqEF,therefore四边形AECF不是矩形,故本选项不符合题意;C.because四边形ABCD是矩形,therefore不能证明ACperpBD,therefore不能证明ACperpEF,故本选项不符合题意;D.because四边形ABCD是矩形,thereforeAC=BD,thereforeACneqEF,therefore四边形AECF不是正方形,故本选项不符合题意;故选A.【点睛】本题主要考查了平行四边形的判定,矩形的性质和判定,菱形的判定,正方形的判定,熟练掌握平行四边形和特殊平行四边形的判定方法是解决问题的关键."} {"id": "5207", "image": ["6779.jpg"], "answer": "C", "solution": "null", "level": "八年级", "question": "如图, 菱形 $A B C D$ 的对角线 $A C 、 B D$ 相交于点 $O$, 过点 $D$ 作 $D H \\perp A B$ 于点 $H$, 连接 $O H, O H=4$,若菱形 $A B C D$ 的面积为 $32 \\sqrt{3}$, 则 $C D$ 的长为 $(\\quad)$", "options": "A. 4B. $4 \\sqrt{3}$C. 8D. $8 \\sqrt{3}$", "subject": "度量几何学", "analysis": "【点拨】在RttriangleBDH中先求得BD的长,根据菱形面积公式求得AC长,再根据勾股定理求得CD长.【解析】解:becauseDHperpAB,thereforeangleBHD=90^circ,because四边形ABCD是菱形,thereforeOB=OD,OC=OA=frac12AC,ACperpBD,thereforeOH=OB=OD=frac12BD(直角三角形斜边上中线等于斜边的一半),thereforeOD=4,BD=8,由frac12ACcdotBD=32sqrt3得,frac12times8cdotAC=32sqrt3,thereforeAC=8sqrt3,thereforeOC=frac12AC=4sqrt3,thereforeCD=sqrtOC^2+OD^2=8,故选C.【点睛】本题考查了菱形性质,直角三角形性质,勾股定理等知识,解决问题的关键是先求得BD的长."} {"id": "5210", "image": ["6782.jpg", "6783.jpg"], "answer": "C", "solution": "null", "level": "八年级", "question": "如图, 在周长为 12 的菱形 $A B C D$ 中, $A E=1, A F=2$, 若 $P$ 为对角线 $B D$ 上一动点, 则 $E P+F P$ 的最小值为 $(\\quad)$", "options": "A. 1B. 2C. 3D. 4", "subject": "度量几何学", "analysis": "【点拨】作F点关于BD的对称点F^prime,连接EF^prime交BD于点P,则PF=PF^prime,由两点之间线段最短可知当E、P、F^prime在一条直线上时,EP+FP有最小值,然后求得EF^prime的长度即可.【解析】解:作F点关于BD的对称点F^prime,连接EF^prime交BD于点P,则PF=PF^prime.thereforeEP+FP=EP+F^primeP.由两点之间线段最短可知:当E、P、F^prime在一条直线上时,EP+FP的值最小,此时EP+FP=EP+F^primeP=EF^prime.because四边形ABCD为菱形,周长为12,thereforeAB=BC=CD=DA=3,AB//CD,becauseAF=2,AE=1,thereforeDF=DF^prime=AE=1,therefore四边形AEF^primeD是平行四边形,thereforeEF^prime=AD=3.thereforeEP+FP的最小值为3.故选C.【点睛】本题主要考查的是菱形的性质、轴对称--路径最短问题,明确当E、P、F^prime在一条直线上时EP+FP有最小值是解题的关键."} {"id": "5211", "image": ["6784.jpg", "6785.jpg"], "answer": "C", "solution": "null", "level": "八年级", "question": "如图, 在正方形 $A B C D$ 中, $A B=4, E$ 为对角线 $A C$ 上与 $A, C$ 不重合的一个动点, 过点 $E$ 作 $E F$ $\\perp A B$ 于点 $F, E G \\perp B C$ 于点 $G$, 连接 $D E, F G$, 下列结论: (1) $D E=F G$; (2) $D E \\perp F G$; (3) $\\angle B F G=$ $\\angle A D E$; (4) $F G$ 的最小值为 3 . 其中正确结论的个数有( )", "options": "A. 1 个B. 2 个C. 3 个D. 4 个", "subject": "度量几何学", "analysis": "【点拨】(1)连接BE,易知四边形EFBG为矩形,可得BE=FG;由triangleAEBcongtriangleAED可得DE=BE,所以DE=FG;(2)延长DE,交FG于M,交FB于点H,由矩形EFBG可得OF=OB,则angleOBF=angleOFB;由angleOBF=angleADE,则angleOFB=angleADE;由四边形ABCD为正方形可得angleBAD=90^circ,即angleAHD+angleADH=90^circ,所以angleAHD+angleOFH=90^circ,即angleFMH=90^circ,可得DEperpFG;(3)由(2)中的结论可得angleBFG=angleADE;(4)由于点E为AC上一动点,当DEperpAC时,根据垂线段最短可得此时DE最小,最小值为2sqrt2,由(1)知FG=DE,所以FG的最小值为2sqrt2;【解析】解:(1)连接BE,交FG于点O,如图,becauseEFperpAB,EGperpBC,thereforeangleEFB=angleEGB=90^circ.becauseangleABC=90^circ,therefore四边形EFBG为矩形.thereforeFG=BE,OB=OF=OE=OG.because四边形ABCD为正方形,thereforeAB=AD,angleBAC=angleDAC=45^circ在triangleABE和triangleADE中,leftbeginarraylAE=AEangleBAC=angleDAC,AB=ADendarrayright.thereforetriangleABEcongtriangleADE(SAS).thereforeBE=DE.thereforeDE=FG.therefore(1)正确;(2)延长DE,交FG于M,交FB于点H,becausetriangleABEcongtriangleADE,thereforeangleABE=angleADE.由(1)知:OB=OF,thereforeangleOFB=angleABE.thereforeangleOFB=angleADE.becauseangleBAD=90^circ,thereforeangleADE+angleAHD=90^circ.thereforeangleOFB+angleAHD=90^circ.即:angleFMH=90^circ,thereforeDEperpFG.therefore(2)正确;(3)由(2)知:angleOFB=angleADE.即:angleBFG=angleADE.therefore(3)正确;(4)because点E为AC上一动点,therefore根据垂线段最短,当DEperpAC时,DE最小.becauseAD=CD=4,angleADC=90^circ,thereforeAC=sqrtAD^2+CD^2=4sqrt2.thereforeDE=frac12AC=2sqrt2.由(1)知:FG=DE,thereforeFG的最小值为2sqrt2,therefore(4)错误.综上,正确的结论为:(1)(2)(3).故选C.【点睛】本题主要考查了正方形的性质,垂线段最短,三角形全等的判定与性质,矩形的判定与性质,垂直的定义.根据图形位置的特点通过添加辅助线构造全等是解题的关键,也是解决此类问题常用的方法."} {"id": "5212", "image": ["6786.jpg", "6787.jpg"], "answer": "D", "solution": "null", "level": "八年级", "question": "如图, 在正方形 $A B C D$ 中, 点 $E, F$ 将对角线 $A C$ 三等分, 且 $A C=12$, 点 $P$ 在正方形的边上, 则满足 $P E+P F=9$ 的点 $P$ 的个数是 $(\\quad)$", "options": "A. 0B. 4C. 6D. 8", "subject": "度量几何学", "analysis": "【点拨】作点F关于BC的对称点M,连接FM交BC于点N,连接EM,交BC于点H,可得点H到点E和点F的距离之和最小,可求最小值,即可求解.【解析】解:如图,作点F关于BC的对称点M,连接FM交BC于点N,连接EM,交BC于点Hbecause点E,F将对角线AC三等分,且AC=12,thereforeEC=8,FC=4=AE,because点M与点F关于BC对称thereforeCF=CM=4,angleACB=angleBCM=45^circthereforeangleACM=90^circthereforeEM=sqrtEC^2+CM^2=4sqrt5则在线段BC存在点H到点E和点F的距离之和最小为4sqrt5<9在点H右侧,当点P与点C重合时,则PE+PF=12therefore点P在CH上时,4sqrt5", "options": "1\nB. 2\nC. 3\nD. 4", "subject": "度量几何学", "analysis": ""} {"id": "4097", "image": ["5779.jpg"], "answer": "A", "solution": "null", "level": "八年级", "question": "如图, 点 $F, B, E, C$ 在同一条直线上, $A C / / D F, C E=F B$,\n\n添加下列哪个条件后,仍不能判定出 $\\triangle A B C \\cong \\triangle D E F ( )$\n\n", "options": "A. $A B=D E$\nB. $A B / / D E$\nC. $\\angle A=\\angle D$\nD. $A C=D F$", "subject": "度量几何学", "analysis": ""} {"id": "4098", "image": ["5780.jpg"], "answer": "C", "solution": "null", "level": "八年级", "question": "如图, $\\triangle A B C$ 中, $A D \\perp B C, A E$ 平分 $\\angle B A C$, 若 $\\angle B=72^{\\circ}, \\angle C=38^{\\circ}$, 则 $\\angle D A E=$ ( )\n\n", "options": "$7^{\\circ}$\nB. $12^{\\circ}$\nC. $17^{\\circ}$\nD. $22^{\\circ}$", "subject": "度量几何学", "analysis": ""} {"id": "4099", "image": ["5781.jpg"], "answer": "A", "solution": "null", "level": "八年级", "question": "如图, $\\triangle \\mathrm{ABC} \\cong \\triangle \\mathrm{AED}$, 点 $\\mathrm{E}$ 在线段 $\\mathrm{BC}$ 上, $\\angle 1=48^{\\circ}$, 则 $\\angle \\mathrm{AED}$ 的度数是()\n\n", "options": "A. $66^{\\circ}$\nB. $65^{\\circ}$\nC. $62^{\\circ}$\nD. $60^{\\circ}$", "subject": "度量几何学", "analysis": ""} {"id": "4101", "image": ["5783.jpg"], "answer": "B", "solution": "null", "level": "八年级", "question": "如图, 在 Rt $\\triangle A B C$ 中, $\\angle C=90^{\\circ}$ , 直线 $D E$ 是斜边 $A B$ 的垂直平分线交 $A C$ 于 $D$.\n\n若 $A C=8, B C=6$, 则 $\\triangle D B C$ 的周长为 $(\\quad)$\n\n", "options": "A. 12\nB. 14\nC. 16\nD. 无法计算", "subject": "度量几何学", "analysis": ""} {"id": "4113", "image": ["5794.jpg"], "answer": "C", "solution": "null", "level": "八年级", "question": "如图, 一棵大树在一次强台风中于离地面 $10 \\mathrm{~m}$ 处折断倒下,倒下部分的树梢到树的距离为 $24 \\mathrm{~m}$, 则这棵大树折断处到树顶的长度是()\n\n", "options": "A. $10 \\mathrm{~m}$\nB. $15 \\mathrm{~m}$\nC. $26 \\mathrm{~m}$\nD. $30 \\mathrm{~m}$", "subject": "度量几何学", "analysis": ""} {"id": "4120", "image": ["5803.jpg"], "answer": "C", "solution": "null", "level": "八年级", "question": "如图, $\\triangle A C B \\cong \\triangle A^{\\prime} C B^{\\prime}, A^{\\prime} B^{\\prime}$ 经过点 $A, \\angle B A C=70^{\\circ}$, 则 $\\angle A C A^{\\prime}$ 的度数为 ( )\n\n", "options": "A. $20^{\\circ}$\nB. $30^{\\circ}$\nC. $40^{\\circ}$\nD. $50^{\\circ}$", "subject": "度量几何学", "analysis": ""} {"id": "4122", "image": ["5804.jpg"], "answer": "B", "solution": "null", "level": "八年级", "question": "工人师傅常用角尺平分一个任意角. 作法如下:\n\n如图所示, $\\angle A O B$ 是一个任意角, 在边 $O A, O B$ 上分别取 $O M=O N$,\n\n移动角尺,使角尺两边相同的刻度分别与 $M, N$ 重合,\n\n过角尺顶点 $C$ 的射线 $O C$ 即是 $\\angle A O B$ 的平分线. 这种作法的道理是 ( )\n\n", "options": "$\\mathrm{HL}$\nB. $\\mathrm{SSS}$\nC. SAS\nD. ASA", "subject": "度量几何学", "analysis": ""} {"id": "4123", "image": ["5805.jpg"], "answer": "D", "solution": "null", "level": "八年级", "question": "如图, $A D$ 是 $\\triangle A B C$ 的角平分线, $D E \\perp A B$, 垂足为 $E$,若 $S_{\\triangle A B C}=7, D E=2, A B=4$, 则 $A C$ 的长为 $(\\quad)$\n\n", "options": "6\nB. 5\nC. 4\nD. 3", "subject": "度量几何学", "analysis": ""} {"id": "4124", "image": ["5806.jpg"], "answer": "D", "solution": "null", "level": "八年级", "question": "如图, 等腰 $\\triangle A B C$ 的底边 $B C$ 长为 $4 \\mathrm{~cm}$, 面积为 $16 \\mathrm{~cm}^{2}$,\n\n腰 $A C$ 的垂直平分线 $E F$ 交 $A C$ 于点 $E$, 交 $A B$ 于点 $F, D$ 为 $B C$ 的中点, $M$ 为直线 $E F$ 上的动点.\n则 $\\triangle C D M$ 周长的最小值为 $(\\quad)$\n\n", "options": "$6 \\mathrm{~cm}$\nB. $8 \\mathrm{~cm}$\nC. $9 \\mathrm{~cm}$\nD. $10 \\mathrm{~cm}$", "subject": "度量几何学", "analysis": ""} {"id": "4126", "image": ["5808.jpg"], "answer": "C", "solution": "null", "level": "八年级", "question": "如图, 某同学把一块三角形的玻璃打碎成 3 块, 现要到玻璃店去配一块完全一样的玻璃,那么最省事的方法是 \\$ \\qquad \\$ , 这么做的依据是 \\$ \\qquad \\$ .\n\n", "options": "带(1)去, $S A S$\nB. 带(2)去,SAS\nC. 带 $(3)$ 去, $A S A$\nD. (1)(2)(3)都带去, $S S S$", "subject": "度量几何学", "analysis": ""} {"id": "4143", "image": [], "answer": "C", "solution": "null", "level": "八年级", "question": "已知等腰三角形的一个内角为 $40^{\\circ}$, 则这个等腰三角形的顶角为 ( )", "options": "$100^{\\circ}$\nB. $40^{\\circ}$\nC. $40^{\\circ}$ 或 $100^{\\circ}$\nD. $40^{\\circ}$ 或 $70^{\\circ}$", "subject": "度量几何学", "analysis": ""} {"id": "4146", "image": ["5829.jpg"], "answer": "C", "solution": "null", "level": "八年级", "question": "如图是一副三角尺拼成的图案, 则 $\\angle A E B$ 的度数为 $(\\quad)$\n\n", "options": "$105^{\\circ}$\nB. $90^{\\circ}$\nC. $75^{\\circ}$\nD. $60^{\\circ}$", "subject": "度量几何学", "analysis": ""} {"id": "4150", "image": ["5836.jpg"], "answer": "B", "solution": "null", "level": "八年级", "question": "如图, $A D, A E$ 分别为 $\\triangle A B C$ 的高线和角平分线, $D F \\perp A E$ 于点 $F$,当 $\\angle A D F=69^{\\circ}, \\angle C=65^{\\circ}$ 时, $\\angle B$ 的度数为 $(\\quad)$\n\n", "options": "A. $21^{\\circ}$\nB. $23^{\\circ}$\nC. $25^{\\circ}$\nD. $30^{\\circ}$", "subject": "度量几何学", "analysis": ""} {"id": "5784", "image": [], "answer": "A", "solution": "null", "level": "八年级", "question": "下面关于平行四边形的说法中错误的是( )", "options": "A. 平行四边形的两条对角线相等\nB. 平行四边形的两条对角线互相平分\nC. 平行四边形的对角相等\nD. 平行四边形的对边相等", "subject": "解析几何", "analysis": "$\\because$ 平行四边形的对边相等、对角相等、对角线互相平分,\n\n$\\therefore \\mathrm{B} 、 \\mathrm{C} 、 \\mathrm{D}$ 说法符合题意;\n\n只有矩形的对角线才相等,故 A 说法不符合题意,\n\n故答案为: A."} {"id": "5801", "image": ["7204.jpg", "7205.jpg"], "answer": "D", "solution": "null", "level": "八年级", "question": "如图, 平行四边形 $\\mathrm{ABCD}$ 中, $\\mathrm{AB}: \\mathrm{BC}=3: 2, \\angle \\mathrm{DAB}=60^{\\circ}, \\mathrm{E}$ 在 $\\mathrm{AB}$ 上, 且 $\\mathrm{AE}: \\mathrm{EB}=1: 2, \\mathrm{~F}$ 是 $\\mathrm{BC}$ 的中点, 过 $\\mathrm{D}$ 分别作 $\\mathrm{DP} \\perp \\mathrm{AF}$ 于 $\\mathrm{P}, \\mathrm{DQ} \\perp \\mathrm{CE}$ 于 $\\mathrm{Q}$, 则 $\\mathrm{DP}: \\mathrm{DQ}$ 等于 ( )\n\n", "options": "A. $3: 4$\nB. $\\sqrt{13}: 2 \\sqrt{5}$\nC. $\\sqrt{13}: 2 \\sqrt{6}$\nD. $2 \\sqrt{3}: \\sqrt{13}$", "subject": "解析几何", "analysis": "连接 $\\mathrm{DE} 、 \\mathrm{DF}$, 过 $\\mathrm{F}$ 作 $\\mathrm{FN} \\perp \\mathrm{AB}$ 于 $\\mathrm{N}$, 过 $\\mathrm{C}$ 作 $\\mathrm{CM} \\perp \\mathrm{AB}$ 于 $\\mathrm{M}$,\n\n\n\n根据三角形的面积和平行四边形的面积得出\n\n$$\n\\mathrm{S}_{\\triangle \\mathrm{DEC}}=\\mathrm{S}_{\\triangle \\mathrm{DFA}}=\\frac{1}{2} \\mathrm{~S}_{\\text {\\#行W边形 } A B C D}\n$$\n\n即 ${ }^{\\frac{1}{2}} \\mathrm{AF} \\times \\mathrm{DP}={ }^{\\frac{1}{2}} \\mathrm{CE} \\times \\mathrm{DQ}$, 求出 $\\mathrm{AF} \\times \\mathrm{DP}=\\mathrm{CE} \\times \\mathrm{DQ}$, 设 $\\mathrm{AB}=3 \\mathrm{a}, \\mathrm{BC}=2 \\mathrm{a}$,\n\n则 $B F=a, \\quad B E=2 a, B N={ }^{\\frac{1}{2}} a, B M=a, \\quad F N={ }^{\\frac{\\sqrt{3}}{2}} a, C M=\\sqrt{3} a$,\n\n求出 $\\mathrm{AF}=\\sqrt{13} \\mathrm{a}, \\mathrm{CE}=2 \\sqrt{3} \\mathrm{a}$, 代入可得 $\\sqrt{13} \\mathrm{a} \\cdot \\mathrm{DP}=2 \\sqrt{3} \\mathrm{a} \\cdot \\mathrm{DQ}$,\n\n即 $\\mathrm{DP}: \\mathrm{DQ}=2 \\sqrt{3}: \\sqrt{13}$\n\n故答案为: D."} {"id": "5029", "image": ["6616.jpg"], "answer": "B", "solution": "null", "level": "八年级", "question": "实数 $a, b$ 在数轴上的位置如图所示, 化简 $\\sqrt{(a+1)^{2}}+\\sqrt{(b-1)^{2}}-\\sqrt{(a-b)^{2}}$ 的结果是 ( )", "options": "A. 0B. -2C. $-2 a$D. $2 b$", "subject": "解析几何", "analysis": "【点拨】利用已知条件确定出a+1,b-1,a-b的符号,再利用二次根式的性质和绝对值的意义化简运算即可.【解析】解:由题意得:a<-1,b>1,thereforea+1<0,b-1>0,a-b<0,therefore原式=|a+1|+|b-1|-|a-b|=-(a+1)+b-1-(b-a)=-a-1+b-1-b+a=-2.故选B.【点睛】本题主要考查了二次根式的性质,绝对值的意义,正确利用上述法则与性质解答是解题的关键."} {"id": "5030", "image": [], "answer": "B", "solution": "null", "level": "八年级", "question": "若式子 $\\frac{1}{\\sqrt{-\\mathrm{a}}}+\\frac{1}{\\sqrt{\\mathrm{b}}}$ 有意义, 则点 $P(a, b)$ 在 ( )", "options": "A. 第一象限B. 第二象限C. 第三象限D. 第四象限", "subject": "解析几何", "analysis": "【点拨】根据二次根式有意义的条件求出a,b的取值范围,进而可得出结论.【解析】解:由题意得,-a>0,b>0,thereforea<0,therefore点P(a,b)在第二象限.故选B.【点睛】本题考查的是二次根式有意义的条件,熟知二次根式具有非负性是解题的关键."} {"id": "5145", "image": ["6668.jpg", "6669.jpg"], "answer": "B", "solution": "null", "level": "八年级", "question": "已知: 如图, $A D 、 B E$ 分别是 $\\triangle A B C$ 的中线和角平分线, $A D \\perp B E, A D=B E=6$, 求 $A C$ 的长为 $(\\quad)$", "options": "A. 10B. $\\frac{9}{2} \\sqrt{5}$C. $6 \\sqrt{3}$D. $8 \\sqrt{2}$", "subject": "解析几何", "analysis": "【点拨】过D点作DF//BE,则DF=frac12BE,F为EC中点,在RttriangleADF中求出AF的长度,根据已知条件易知G为AD中点,因此E为AF中点,则AC=frac32AF.【解析】解:过D点作DF//BE,becauseAD是triangleABC的中线,ADperpBE,thereforeF为EC中点,ADperpDF,becauseAD=BE=6,则DF=3,AF=sqrtAD^2+DF^2=3sqrt5,becauseBE是triangleABC的角平分线,ADperpBE,thereforeangleAGB=angleDGB=90^circ,angleABG=angleDBG,becauseBG=BG,thereforetriangleABGcongtriangleDBG(ASA),thereforeG为AD中点,thereforeE为AF中点,thereforeAC=frac32AF=frac92sqrt5.故选B.【点睛】本题考查了三角形中线和角平分线的性质以及勾股定理的应用,作出辅助线构建直角三角形是解题的关键."} {"id": "5158", "image": [], "answer": "C", "solution": "null", "level": "八年级", "question": "若一个多边形的内角和是它的外角和的 3 倍, 则该多边形的边数为 ( )", "options": "A. 6B. 7C. 8D. 9", "subject": "解析几何", "analysis": "【点拨】设这个多边形的边数为x,根据多边形的内角和公式、任意多边形的外角和等于360^circ列出方程,从而解决此题.【解析】解:设这个多边形的边数为x.由题意得,180^circ(x-2)=360^circtimes3.thereforex=8.故选C.【点睛】本题主要考查多边形的内角与外角,熟练掌握多边形的内角和公式、任意多边形的外角和等于360度是解决本题的关键."} {"id": "5163", "image": ["6710.jpg"], "answer": "A", "solution": "null", "level": "八年级", "question": "小玲的爸爸在钉制平行四边形框架时, 采用了一种方法: 如图所示, 将两根木条 $A C 、 B D$ 的中点重叠,并用钉子固定,则四边形 $A B C D$ 就是平行四边形,这种方法的依据是()", "options": "A. 对角线互相平分的四边形是平行四边形B. 两组对角分别相等的四边形是平行四边形C. 两组对边分别相等的四边形是平行四边形D. 两组对边分别平行的四边形是平行四边形", "subject": "解析几何", "analysis": "【点拨】已知AC和BD是对角线,取各自中点,则对角线互相平分(即AO=OC,BO=DO)的四边形是平行四边形.【解析】解:由已知可得AO=CO,BO=DO,所以四边形ABCD是平行四边形,依据是对角线互相平分的四边形是平行四边形.故选A.【点睛】本题主要考查了平行四边形的判定,熟记平行四边形的判定方法是解题的关键."} {"id": "5191", "image": ["6743.jpg"], "answer": "D", "solution": "null", "level": "八年级", "question": "如图, $\\square A B C D$ 的对角线 $A C$ 和 $B D$ 相交于点 $O$, 下列说法正确的是 ( )", "options": "A. 若 $O B=O D$, 则 $\\square A B C D$ 是菱形B. 若 $A C=B D$, 则 $\\square A B C D$ 是菱形C. 若 $O A=O D$, 则 $\\square A B C D$ 是菱形D. 若 $A C \\perp B D$, 则 $\\square A B C D$ 是菱形", "subject": "解析几何", "analysis": "【点拨】由矩形的判定和菱形的判定分别对各个选项进行判断即可.【解析】解:A、because四边形ABCD是平行四边形,thereforeOB=OD,故选项A不符合题意;B、because四边形ABCD是平行四边形,AC=BD,thereforesquareABCD是矩形,故选项B不符合题意;C、because四边形ABCD是平行四边形,thereforeOA=OC=frac12AC,OB=OD=frac12BD,becauseOA=OD,thereforeAC=BD,thereforesquareABCD是矩形,故选项C不符合题意;D、because四边形ABCD是平行四边形,ACperpBD,thereforesquareABCD是菱形,故选项D符合题意;故选D.【点睛】本题考查了菱形的判定、矩形的判定以及平行四边形的性质,熟练掌握菱形的判定和矩形的判定是解题的关键."} {"id": "4100", "image": ["5782.jpg"], "answer": "B", "solution": "null", "level": "八年级", "question": "如图, 在三角形纸片 $A B C$ 中, $A B=8 \\mathrm{~cm}, B C=7 \\mathrm{~cm}, A C=5 \\mathrm{~cm}$,\n\n将 $\\triangle C D B$ 沿过点 $B$ 的直线折叠, 使顶点 $C$ 落在 $A B$ 边上的点 $E$ 处, 折痕为 $B D$,则 $\\triangle A E D$ 的周长为 $(\\quad)$\n\n", "options": "A. $5 \\mathrm{~cm}$\nB. $6 \\mathrm{~cm}$\nC. $7 \\mathrm{~cm}$\nD. $8 \\mathrm{~cm}$", "subject": "解析几何", "analysis": ""} {"id": "4103", "image": ["5788.jpg"], "answer": "A", "solution": "null", "level": "八年级", "question": "如图, 在边长为 6 的正方形 $A B C D$ 内作 $\\angle E A F=45^{\\circ}, A E$ 交 $B C$ 于点 $E, A F$ 交 $C D$ 于点 $F$,连接 $E F$. 若 $D F=3$, 则 $B E$ 的长为 $(\\quad)$\n\n", "options": "2\nB. 3\nC. 4\nD. 5", "subject": "解析几何", "analysis": ""} {"id": "4121", "image": [], "answer": "B", "solution": "null", "level": "八年级", "question": "在 $\\triangle A B C$ 中, $\\angle A 、 \\angle B 、 \\angle C$ 的对边分别为 $a 、 b 、 c$, 下列条件中,\n不能判断 $\\triangle A B C$ 是直角三角形的是 $(\\quad)$", "options": "$a=3, b=4, c=5$\nB. $a=b, \\angle C=45^{\\circ}$\nC. $\\angle A: \\angle B: \\angle C=1: 2: 3$\nD. $a=9, b=40, c=41$", "subject": "解析几何", "analysis": ""} {"id": "5213", "image": [], "answer": "D", "solution": "null", "level": "八年级", "question": "若反比例函数的图象经过点 $(1,2)$, 则该反比例函数的表达式是()", "options": "A. $y=2 x$B. $y=x+1$C. $\\mathrm{y}=\\frac{1}{\\mathrm{x}}$D. $\\mathrm{y}=\\frac{2}{\\mathrm{x}}$", "subject": "解析几何", "analysis": "【点拨】先设y=frackx,再把已知点的坐标代入可求出k值,即得到反比例函数的解析式.【解析】解:设反比例函数的表达式为y=frackx,把点(1,2)代入y=frackx,得2=frack1,thereforek=2,则反比例函数的解析式为y=frac2x,故选D.【点睛】本题主要考查了用待定系数法求反比例函数的解析式,熟练掌握待定系数法是解题的关键."} {"id": "5214", "image": ["6788.jpg", "6789.jpg"], "answer": "B", "solution": "null", "level": "八年级", "question": "如图, 四边形 $O A B C$ 是矩形, $A D E F$ 是正方形, 点 $A 、 D$ 在 $x$ 轴的正半轴上, 点 $C$ 在 $y$ 轴的正半轴上, 点 $F$ 在 $A B$ 上, 点 $B 、 E$ 在反比例函数 $\\mathrm{y}=\\frac{\\mathrm{k}}{\\mathrm{x}}$ 的图象上, $O A=1, O C=6$, 则正方形 $A D E F$ 的边长为 $(\\quad)$", "options": "A. 1B. 2C. $\\sqrt{3}$D. 3", "subject": "解析几何", "analysis": "【点拨】先确定B点坐标(1,6),根据反比例函数图象上点的坐标特征得到k=6,则反比例函数解析式为y=frac6x,设AD=t,则OD=1+t,所以E点坐标为(1+t,t),再根据反比例函数图象上点的坐标特征得(1+t)cdott=6,利用因式分解法可求出t的值.【解析】解:becauseOA=1,OC=6,thereforeB点坐标为(1,6),thereforek=6times1=6,therefore反比例函数解析式为y=frac6x,设AD=t,则OD=1+t,thereforeE点坐标为(1+t,t),therefore(1+t)cdott=6,整理为t^2+t-6=0,解得t_1=-3(舍去),t_2=2,therefore正方形ADEF的边长为2.故选B.【点睛】本题考查了反比例函数图象上点的坐标特征:反比例函数y=frackx(k为常数,kneq0)的图象是双曲线,图象上的点(x,y)的横纵坐标的积是定值k,即xy=k."} {"id": "5224", "image": [], "answer": "D", "solution": "null", "level": "八年级", "question": "若点 $A\\left(-1, y_{1}\\right), B\\left(\\sqrt{2}, y_{2}\\right), C\\left(\\sqrt{3}, \\mathrm{y}_{3}\\right)$ 在反比例函数的图象 $\\mathrm{y}=\\frac{4}{\\mathrm{x}}$ 上, 则 $y_{1} 、 y_{2} 、 y_{3}$的大小关系是 $(\\quad)$", "options": "A. $y_{1}>y_{2}>y_{3}$B. $y_{3}>y_{2}>y_{1}$C. $y_{1}>y_{3}>y_{2}$D. $y_{2}>y_{3}>y_{1}$", "subject": "解析几何", "analysis": "【点拨】先由k=4>0得到函数在第一、三象限,在每个象限内的函数值随x的增大而减小,然后根据点的坐标特征以及函数的增减性得到y_1,y_2,y_3的大小关系.【解析】解:because反比例系数k=4>0,therefore函数在第一、三象限,在每个象限内的函数值随x的增大而减小,because-1<0y_3>y_1,故选D.【点睛】本题考查了反比例函数图象上点的坐标特征,解题的关键是熟知反比例函数的增减性."} {"id": "5229", "image": ["6797.jpg"], "answer": "C", "solution": "null", "level": "八年级", "question": "如图, 已知直线 $y=m x$ 与双曲线 $\\mathrm{y}=\\frac{\\mathrm{k}}{\\mathrm{x}}$ 的一个交点坐标为 $(-1,3)$, 则它们的另一个交点坐标是", "options": "A. $(1,3)$B. $(3,1)$C. $(1,-3)$D. $(-1,3)$", "subject": "解析几何", "analysis": "【点拨】反比例函数的图象是中心对称图形,则与经过原点的直线的两个交点一定关于原点对称.【解析】解:因为直线y=mx过原点,双曲线y=frackx的两个分支关于原点对称,所以其交点坐标关于原点对称,一个交点坐标为(-1,3),另一个交点的坐标为(1,-3).故选C.【点睛】此题考查了函数交点的对称性,通过数形结合和中心对称的定义很容易解决."} {"id": "5230", "image": [], "answer": "D", "solution": "null", "level": "八年级", "question": "已知反比例函数 $\\mathrm{y}=\\frac{6}{\\mathrm{x}}$, 下列说法不正确的是 ( )", "options": "A. 图象经过点 $(-3,2)$B. 图象分别位于第二、四象限内C. 在每个象限内 $y$ 的值随 $x$ 的值增大而增大D. $x \\geqslant-1$ 时, $y \\geqslant 6$", "subject": "解析几何", "analysis": "【点拨】根据反比例函数的性质逐一判断即可.【解析】解:因为(-3)times2=-6,所以A正确,不符合题意;因为反比例函数y=frac6x,所以图象分别位于第二、四象限内;在每个象限内y的值随x的值增大而增大;所以B、C正确,不符合题意;当xgeqslant-1时,ygeqslant6或y<0,所以D错误,符合题意,故选D.【点睛】本题考查了反比例函数的性质,熟练掌握反比例函数的基本性质是解题的关键."} {"id": "5231", "image": ["6798.jpg"], "answer": "D", "solution": "null", "level": "八年级", "question": "如图, 一次函数 $y_{1}=x-1$ 的图象与反比例函数 $\\mathrm{y}_{2}=\\frac{\\mathrm{k}}{\\mathrm{x}}$ 的图象交于点 $A(2, m), B(n,-2)$,当 $y_{1}>y_{2}$ 时, $x$ 的取值范围是 $(\\quad)$", "options": "A. $x<-1$ 或 $x>2$B. $x<-1$ 或 $02$", "subject": "解析几何", "analysis": "【点拨】先把B(n,-2)代入y_1=x-1,求出n值,再根据图象直接求解即可.【解析】解:把B(n,-2)代入y_1=x-1,得-2=n-1,解得:n=-1,thereforeB(-1,-2),because图象交于A(2,m)、B(-1,-2)两点,therefore当y_1>y_2时,-12.故选D.【点睛】本题考查了一次函数与反比例函数交点问题,掌握利用图象法求自变量的取值范围是解题的关键."} {"id": "5232", "image": ["6799.jpg", "6800.jpg", "6801.jpg"], "answer": "B", "solution": "null", "level": "八年级", "question": "函数 $y=k x-3$ 与 $y=\\frac{\\mathrm{k}}{\\mathrm{x}}$ 在同一坐标系内的图象可能是 ( )", "options": "A.C.D.", "subject": "解析几何", "analysis": "【点拨】根据当k>0、当k<0时,y=kx-3和y=frackx(kneq0)经过的象限,二者一致的即为正确答案.【解析】解:because当k>0时,y=kx-3过一、三、四象限,反比例函数y=frackx过一、三象限,当k<0时,y=kx-3过二、三、四象限,反比例函数y=frackx过二、四象限,thereforeB正确;故选B.【点睛】本题主要考查了反比例函数的图象性质和一次函数的图象性质,关键是由k的取值确定函数所在的象限."} {"id": "5233", "image": [], "answer": "A", "solution": "null", "level": "八年级", "question": "在研究反比例函数的图象时, 小明想通过列表、描点的方法画出反比例函数的图象, 但是在作图时,小明发现计算有错误,四个点中有一个不在该函数图象上,那么这个点是()| $x$ | $\\cdots$ | -2 | $-\\frac{1}{2}$ | 1 | 2 | $\\cdots$ || :---: | :---: | :---: | :---: | :---: | :---: | :---: || $y$ | $\\cdots$ | -1 | 4 | -2 | -1 | $\\cdots$ |", "options": "A. $(-2,-1)$B. $\\left(-\\frac{1}{2}, 4\\right)$C. $(1,-2)$D. $(2,-1)$", "subject": "解析几何", "analysis": "【点拨】根据反比例函数中k=xy的特点进行解答即可.【解析】解:because(-2)times(-1)neq-frac12times4=1times(-2)=2times(-1),therefore这个点是(-2,-1).故选A.【点睛】本题考查的是反比例函数图象上点的坐标特点,熟知反比例函数图象上各点的坐标一定适合此函数的解析式是解答此题的关键."} {"id": "5234", "image": [], "answer": "D", "solution": "null", "level": "八年级", "question": "已知反比例函数 $\\mathrm{y}=\\frac{\\mathrm{k}}{\\mathrm{x}}$ 图象过点 $(2,-4)$, 若 $-12$D. $y<-2$ 或 $y>8$", "subject": "解析几何", "analysis": "【点拨】先求得反比例函数的解析式,然后分别求出x=-1和4时,y的值,再根据反比例函数的性质求解.【解析】解:反比例函数y=frackx图象过点(2,-4),thereforek=2times(-4)=-8,therefore反比例函数y=-frac8x图象在二、四象限,令x=-1,得:y=8;令x=4,得:y=-2,若-18或y<-2,故选D.【点睛】本题考查了反比例函数图象上点的坐标特征,点在图象上,点的横纵坐标满足图象的解析式;也考查了反比例函数的性质,三角形的面积公式以及代数式的变形能力."} {"id": "5235", "image": ["6802.jpg"], "answer": "C", "solution": "null", "level": "八年级", "question": "在一个可以改变体积的密闭容器内装有一定质量的气体, 当改变容器的体积时, 气体的密度也会随之改变, 密度 $\\rho\\left(\\mathrm{kg} / \\mathrm{m}^{3}\\right)$ 是体积 $V\\left(\\mathrm{~m}^{3}\\right)$ 的反比例函数, 它的图象如图所示, 根据图象可知,下列说法正确的是", "options": "A. 密度 $\\rho\\left(\\mathrm{kg} / \\mathrm{m}^{3}\\right)$ 随体积 $V\\left(\\mathrm{~m}^{3}\\right)$ 的增大而增大B. 密度 $\\rho\\left(\\mathrm{kg} / \\mathrm{m}^{3}\\right)$ 和体积 $V\\left(\\mathrm{~m}^{3}\\right)$ 的关系式为 $\\rho=\\frac{4}{V}$C. 密度 $\\rho \\geqslant 2 \\mathrm{~kg} / \\mathrm{m}^{3}$ 时, 体积 $V$ 的范围为 $00$ 时, $y$ 随 $x$ 的增大而增大D. 当 $x<0$ 时, $y$ 随 $x$ 的增大而减小", "subject": "解析几何", "analysis": "【点拨】对于反比例函数y=frac2x,可得k=-2<0,x=-2时,y=1,图象在第二、四象限,在每一个象限内,y随x的增大而增大,逐项分析判断即可求解.【解析】解:对于反比例函数y=frac2x,A.当x=-2时,y=1,则点(-2,1)在它的图象上,正确,不符合题意;B.k=-2<0,则它的图象在第二、四象限,正确,不符合题意;C.当x>0时,y随x的增大而增大,正确,不符合题意;D.当x<0时,y随x的增大而增大,原说法不正确,符合题意.故选D.【点睛】本题考查了反比例函数的性质,熟练掌握反比例函数的图象与性质是解题的关键."} {"id": "5237", "image": ["6803.jpg"], "answer": "C", "solution": "null", "level": "八年级", "question": "两个反比例函数 $\\mathrm{y}=\\frac{\\mathrm{k}}{\\mathrm{x}}$ 和 $\\mathrm{y}=\\frac{1}{\\mathrm{x}}$ 在第一象限内的图象如图所示, 点 $P$ 在 $\\mathrm{y}=\\frac{\\mathrm{k}}{\\mathrm{x}}$ 的图象上, $P C \\perp x$ 轴于点 $C$, 交 $\\mathrm{y}=\\frac{1}{\\mathrm{x}}$ 的图象于点 $A, P D \\perp y$ 轴于点 $D$, 交 $\\mathrm{y}=\\frac{1}{\\mathrm{x}}$ 的图象于点 $B$, 当点 $P$ 在 $\\mathrm{y}=\\frac{\\mathrm{k}}{\\mathrm{x}}$ 的图象上运动时, 以下结论:(1) $\\triangle O D B$ 与 $\\triangle O C A$ 的面积相等; (2)四边形 $P A O B$ 的面积不会发生变化;(3) $P A$ 与 $P B$ 始终相等; (4)当点 $A$ 是 $P C$ 的中点时, 点 $B$ 一定是 $P D$ 的中点.其中, 正确的结论有", "options": "A. 1B. 2C. 3D. 4", "subject": "解析几何", "analysis": "【点拨】根据反比例函数的图象那个和性质,特别是根据反比例函数k的几何意义,对四个选项逐一进行分析,即可得出正确答案.【解析】解:(1)由于点A和点D均在同一个反比例函数y=frac1x的图象上,所以S_triangleODB=frac12,S_triangleOCA=frac12;故triangleODB与triangleOCA的面积相等,故本选项正确;(2)根据反比例函数的几何意义,四边形PAOB的面积始终等于|k|-1,故本选项正确;(3)由图可知,当OCPB,故本选项错误;(4)由于反比例函数是轴对称图形,当A为PC的中点时,B为PD的中点,故本选项正确.故选C.【点睛】此题考查了反比例函数k的几何意义,还要熟悉反比例函数的对称性,体现了数形结合的魅力."} {"id": "5795", "image": [], "answer": "D", "solution": "null", "level": "八年级", "question": "下列图形中, 既是轴对称图形又是中心对称图形的是()", "options": "A. 等边三角形\nB. 直角梯形\nC. 平行四边形\nD. 菱形", "subject": "变换几何", "analysis": "A、是轴对称图形, 不是中心对称图形. A 不符合题意;\n\nB、不是轴对称图形,也不是中心对称图形. B 不符合题意;\n\nC、不是轴对称图形, 是中心对称图形. C 不符合题意;\n\nD、是轴对称图形, 也是中心对称图形. D 符合题意.\n\n故答案为: D."} {"id": "5798", "image": ["7200.jpg", "7201.jpg"], "answer": "C", "solution": "null", "level": "八年级", "question": "观察下列图案,既是轴对称图形又是中心对称图形的是()", "options": "A.\n\n\nB.\n\n\n$(8$", "subject": "变换几何", "analysis": "$A$ 、不是轴对称图形, 是中心对称图形. 故不符合题意;\n\n$B 、$ 是轴对称图形, 不是中心对称图形. 故不符合题意;\n\nC、是轴对称图形, 也是中心对称图形. 故符合题意;\n\n$D$ 、不是轴对称图形, 也不是中心对称图形. 故不符合题意.\n\n故答案为: $C$."} {"id": "4136", "image": ["5815.jpg", "5816.jpg", "5817.jpg", "5818.jpg"], "answer": "A", "solution": "null", "level": "八年级", "question": "在以下 “绿色食品”、“节能减排”、“循环回收、质量安全”四个标志中,是轴对称图形的是 $(\\quad)$", "options": "\nB.\n\n\nC.\n\n\nD.\n\n", "subject": "变换几何", "analysis": ""} {"id": "4170", "image": ["5855.jpg"], "answer": "A", "solution": "null", "level": "八年级", "question": "如图, 有一张直角三角形的纸片, 两直角边 $A C=6 \\mathrm{~cm}, B C=8 \\mathrm{~cm}$,\n\n现将直角边 $A C$ 沿直线 $A D$ 折叠, 使它落在斜边 $A B$ 上且与 $A E$ 重合, 则 $B D$ 的长为 (\n\n", "options": "$5 \\mathrm{~cm}$\nB. $4 \\mathrm{~cm}$\nC. $3 \\mathrm{~cm}$\nD. $2 \\mathrm{~cm}$", "subject": "变换几何", "analysis": ""} {"id": "4172", "image": ["5857.jpg", "5858.jpg", "5859.jpg", "5860.jpg"], "answer": "B", "solution": "null", "level": "八年级", "question": "我国的生活垃圾一般可分为四大类: 厨余垃圾、可回收垃圾、有害垃圾和其他垃圾,图标如下,其中不是轴对称图形的是( )", "options": "\nB.\n\n\nC.\n\n\nD.\n\n", "subject": "变换几何", "analysis": ""} {"id": "4197", "image": ["5883.jpg", "5884.jpg", "5885.jpg", "5886.jpg"], "answer": "B", "solution": "null", "level": "八年级", "question": "我国的生活垃圾一般可分为四大类: 厨余垃圾、可回收垃圾、有害垃圾和其他垃圾,图标如下,其中不是轴对称图形的是( )", "options": "\nB.\n\n\nC.\n\n\nD.\n\n", "subject": "变换几何", "analysis": ""} {"id": "4233", "image": ["5923.jpg"], "answer": "B", "solution": "null", "level": "八年级", "question": "如图, $\\triangle A B C$ 与 $\\triangle D E F$ 关于直线 1 对称, 且 $\\angle A=108^{\\circ}, \\angle F=32^{\\circ}$, 则 $\\angle B$ 的度数为 ( )\n\n", "options": "A. $32^{\\circ}$\nB. $40^{\\circ}$\nC. $50^{\\circ}$\nD. $108^{\\circ}$", "subject": "变换几何", "analysis": ""} {"id": "4259", "image": ["5955.jpg"], "answer": "B", "solution": "null", "level": "八年级", "question": "如图, $\\triangle A B C$ 与 $\\triangle D E F$ 关于直线 1 对称, 且 $\\angle A=108^{\\circ}, \\angle F=32^{\\circ}$, 则 $\\angle B$ 的度数为 ( )\n\n", "options": "A. $32^{\\circ}$\nB. $40^{\\circ}$\nC. $50^{\\circ}$\nD. $108^{\\circ}$", "subject": "变换几何", "analysis": ""} {"id": "5797", "image": [], "answer": "C", "solution": "null", "level": "八年级", "question": "下列描述不正确的是 ( )", "options": "A. 单项式 $-\\frac{a b^{2}}{3}$ 的系数是 $-\\frac{1}{3}$, 次数是 3 次\n\nB. 用一个平面去截一个圆柱, 截面的形状可能是一个长方形\n\nC. 过七边形的一个顶点有 5 条对角线\n\nD. 五棱柱有 7 个面, 15 条棱", "subject": "立体几何学", "analysis": "A、单项式- ${ }^{\\frac{a b^{2}}{3}}$ 的系数是- ${ }^{\\frac{1}{3}}$, 次数是 3 次, 故 A 正确;\n\nB、用一个平面去截一个圆柱, 截面的形状可能是一个长方形, 故 B 正确;\n\nC、过七边形的一个顶点有 4 条对角线, 故 C 错误;\n\nD、五棱柱有 7 个面, 15 条棱, 故 D 正确;\n\n故答案为: C."} {"id": "5135", "image": [], "answer": "D", "solution": "null", "level": "八年级", "question": "一个八边形的内角和度数为", "options": "A. $360^{\\circ}$B. $720^{\\circ}$C. $900^{\\circ}$D. $1080^{\\circ}$", "subject": "立体几何学", "analysis": "【点拨】应用多边形的内角和公式计算即可.【解析】解:(n-2)cdot180=(8-2)times180^circ=1080^circ.故选D.【点睛】此题主要考查了多边形内角和定理,关键是熟练掌握计算公式:(n-2)cdot180quad(ngeqslant3)且n为整数)."} {"id": "5170", "image": [], "answer": "B", "solution": "null", "level": "八年级", "question": "矩形具有而菱形不一定具有的性质是", "options": "A. 四条边都相等B. 对角线相等C. 对角线互相垂直且平分D. 对角线平分一组对角", "subject": "立体几何学", "analysis": "【点拨】根据正方形与菱形的性质即可求得答案,注意排除法在解选择题中的应用.【解析】解:正方形的性质有:四条边都相等,四个角都是直角,对角线互相平分垂直且相等,而且平分一组对角;菱形的性质有:四条边都相等,对角线互相垂直平分.故正方形具有而菱形不一定具有的性质是:对角线相等.故选B.【点睛】此题考查了正方形与菱形的性质。此题比较简单,解题的关键是熟记正方形与菱形的性质定理."} {"id": "5179", "image": [], "answer": "C", "solution": "null", "level": "八年级", "question": "已知菱形的面积为 $120 \\mathrm{~cm}^{2}$, 一条对角线长为 $10 \\mathrm{~cm}$, 则这个菱形的周长为() $\\mathrm{cm}$.", "options": "A. 13B. 24C. 52D. 60", "subject": "立体几何学", "analysis": "【点拨】根据菱形的面积可求得另一条对角线的长,再根据勾股定理求得其边长,从而就不难求得其周长.【解析】解:because菱形的一条对角线长为10~cm,面积为120~cm^2,therefore另一对角线长为frac120times210=24(~cm),根据勾股定理,菱形的边长为sqrt12^2+5^2=13(~cm),则菱形的周长=13times4=52(~cm).故选C.【点睛】本题主要考查了菱形的性质,菱形的面积公式,勾股定理,熟练掌握菱形的性质是解题的关键."} {"id": "5186", "image": [], "answer": "D", "solution": "null", "level": "八年级", "question": "下列说法正确的是 ( $\\quad )$", "options": "A. 四边相等的四边形是正方形B. 对角线互相垂直且相等的四边形是正方形C. 对角线相等的四边形是矩形D. 对角线互相垂直平分的四边形是菱形", "subject": "立体几何学", "analysis": "【点拨】由矩形,菱形,正方形的判定方法,即可判断.【解析】解:A、四边相等的四边形是菱形,故A不符合题意;B、对角线互相垂直且相等的四边形不一定是正方形,故B不符合题意;C、、对角线相等的四边形不一定是矩形,故C不符合题意;D、对角线互相垂直平分的四边形是菱形,正确,故D符合题意.故选D.【点睛】本题考查菱形,矩形,正方形,关键是掌握菱形,矩形,正方形的判定方法."} {"id": "5188", "image": [], "answer": "A", "solution": "null", "level": "八年级", "question": "下列说法正确的有 $(\\quad)$ 个(1)菱形的对角线相等; (2)对角线互相垂直的四边形是菱形; (3)正方形既是菱形又是矩形;(4)有两个角是直角的四边形是矩形; (5)矩形的对角线相等且互相垂直平分.", "options": "A. 1B. 2C. 3D. 4", "subject": "立体几何学", "analysis": "【点拨】菱形的对角线互相垂直平分,但菱形的对角线不一定相等,可判断(1)错误;对角线互相垂直的四边形不一定是菱形,可判断(2)错误;根据正方形的定义,有一组邻边相等且有一个角是直角的平行四边形是正方形,可知正方形既是菱形又是矩形,可判断(3)正确;有两个角是直角的四边形,如直角梯形不一定是矩形,可判断(4)错误;矩形的对角线相等且互相平分,但不一定互相垂直,可判断(5)错误,于是得到问题的答案.【解析】解:菱形的对角线互相垂直平分,但不一定相等,故(1)错误;对角线互相垂直平分的四边形是菱形,但对角线互相垂直的四边形不一定是菱形,故(2)错误;有一组邻边相等且有一个角是直角的平行四边形是正方形,所以正方形既是菱形又是矩形,故(3)正确;有两个角是直角的四边形不一定是矩形,如直角梯形有两个角是直角,故(4)错误;矩形的对角线相等且互相平分,但不一定互相垂直,故(5)错误,故选A.【点睛】此题重点考查矩形、菱形与正方形的判定,熟练掌握矩形、菱形与正方形的判定定理是解题的关键."} {"id": "5799", "image": ["7202.jpg"], "answer": "D", "solution": "null", "level": "八年级", "question": "如图, $=A B C D$ 的对角线 $A C, B D$ 交于点 $O, A C \\perp A B, A B=\\sqrt{5}, B O=3$, 那么 $A C$ 的长为 $(\\quad)$\n\n", "options": "A. $2 \\sqrt{5}$\nB. $\\sqrt{5}$\nC. 3\nD. 4", "subject": "组合几何学", "analysis": "$\\because$ 四边形 $\\mathrm{ABCD}$ 是平行四边形,\n\n$\\therefore \\mathrm{OA}=\\mathrm{OC}, \\mathrm{OB}=\\mathrm{OD}$\n\n$\\because \\mathrm{AC} \\perp \\mathrm{AB}, \\quad A B=\\sqrt{5}, \\quad \\mathrm{BO}=3$,\n\n$\\therefore \\mathrm{OB}^{2}=\\mathrm{AB}^{2}+\\mathrm{OA}^{2} \\quad ,$ 即 $3^{2}=(\\sqrt{5})^{2}+\\mathrm{OA}^{2}$,\n\n$\\therefore \\mathrm{OA}^{2}=4$,\n\n$\\because \\mathrm{OA}>0$\n\n$\\therefore \\mathrm{OA}=2$,\n\n$\\therefore \\mathrm{AC}=2 \\mathrm{OA}=4$.\n\n故答案为: D."} {"id": "5125", "image": ["6642.jpg"], "answer": "A", "solution": "null", "level": "八年级", "question": "如图, 平行四边形 $A B C D$ 的对角线 $A C, B D$ 相交于点 $O$. 点 $E$ 为 $B C$ 的中点, 连接 $E O$ 并延长交 $A D$ 于点 $F, \\angle A B C=60^{\\circ}, B C=2 A B$. 下列结论:(1) $S \\square_{A B C D}=A B \\cdot A C$; (2) $A D=4 O E ;$ (3) $E F \\perp A C$; (4) $S_{\\triangle B O E}=\\frac{1}{4} S_{\\triangle A B C}$. 其中正确结论的个数是 $(\\quad)$", "options": "A. 4B. 3C. 2D. 1", "subject": "组合几何学", "analysis": "【点拨】通过判定triangleABE为等边三角形求得angleBAE=60^circ,利用等腰三角形的性质求得angleEAC=30^circ,从而判断(1);利用有一组邻边相等的平行四边形是菱形判断(3),然后结合菱形的性质和含30^circ直角三角形的性质判断(2);根据三角形中线的性质判断(4).【解析】解:because点E为BC的中点,thereforeBC=2BE=2CE,又becauseBC=2AB,thereforeAB=BE,becauseangleABC=60^circ,thereforetriangleABE是等边三角形,thereforeangleBAE=angleBEA=60^circ,thereforeangleEAC=angleECA=30^circ,thereforeangleBAC=angleBAE+angleEAC=90^circ,即ABperpAC,故(1)正确;在平行四边形ABCD中,AD//BC,AD=BC,AO=CO,thereforeangleCAD=angleACB,在triangleAOF和triangleCOE中,leftbeginarraylangleCAD=angleACBOA=OCangleAOF=angleCOEendarrayright.,thereforetriangleAOFcongtriangleCOE(ASA),thereforeAF=CE,therefore四边形AECF是平行四边形,又becauseABperpAC,点E为BC的中点,thereforeAE=CE,therefore平行四边形AECF是菱形;thereforeACperpEF,故(3)正确,在RttriangleCOE中,angleACE=30^circ,thereforeOO=frac12CE=frac14BC=frac14AD,故(2)正确;在平行四边形ABCD中,OA=OC,又because点E为BC的中点,thereforeS_triangleBOE=frac12S_triangleBOC=frac14S_triangleABC,故(4)正确;综上所述:正确的结论有4个,故选A.【点睛】本题考查平行四边形的性质,等边三角形的判定和性质,菱形的判定和性质,含30^circ的直角三角形的性质,掌握菱形的判定是解题关键."} {"id": "5144", "image": ["6666.jpg", "6667.jpg"], "answer": "C", "solution": "null", "level": "八年级", "question": "如图, 四边形 $A B C D$ 的对角线 $A C, B D$ 交于点 $O$, 则不能判断四边形 $A B C D$ 是平行四边形的是 (", "options": "A. $\\angle A B D=\\angle B D C, O A=O C$B. $\\angle A B C=\\angle A D C, A D / / B C$C. $\\angle A B C=\\angle A D C, A B=C D$D. $\\angle A B D=\\angle B D C, \\angle B A D=\\angle D C B$", "subject": "组合几何学", "analysis": "【点拨】利用所给条件结合平行四边形的判定方法进行分析即可.【解析】解:A、becauseangleABD=angleBDC,OA=OC,又angleAOB=angleCOD,thereforetriangleAOBcongtriangleCOD(AAS),thereforeDO=BO,therefore四边形ABCD是平行四边形,故此选项不合题意;B、becauseAD//BC,thereforeangleABC+angleBAD=180^circ,becauseangleABC=angleADCthereforeangleADC+angleBAD=180^circ,thereforeAB//CD,therefore四边形ABCD是平行四边形,故此选项不合题意;C、angleABC=angleADC,AB=CD不能判断四边形ABCD是平行四边形,故此选项符合题意;D、becauseangleABD=angleBDC,angleBAD=angleDCB,thereforeangleADB=angleCBDthereforeAD//CB,becauseangleABD=angleBDCthereforeAB//CD,therefore四边形ABCD是平行四边形,故此选项不合题意;故选C.【点睛】此题主要考查了平行四边形的判定,关键是掌握(1)两组对边分别平行的四边形是平行四边形.(2)两组对边分别相等的四边形是平行四边形.(3)一组对边平行且相等的四边形是平行四边形.(4)两组对角分别相等的四边形是平行四边形.(5)对角线互相平分的四边形是平行四边形."} {"id": "5168", "image": ["6714.jpg", "6715.jpg"], "answer": "D", "solution": "null", "level": "八年级", "question": "一个多边形截去一个角后, 形成另一个多边形的内角和为 $720^{\\circ}$, 那么原多边形的边数为", "options": "A. 5B. 5 或 6C. 6 或 7D. 5 或 6 或 7", "subject": "组合几何学", "analysis": "【点拨】首先求得内角和为720^circ的多边形的边数,即可确定原多边形的边数.【解析】解:如图,剪切的三种情况:(1)不经过顶点剪,则比原来边数多1,(2)只过一个顶点剪,则和原来边数相等,(3)按照顶点连线剪,则比原来的边数少1,设内角和为720^circ的多边形的边数是n,therefore(n-2)cdot180=720,解得:n=6.则原多边形的边数为5或6或7.故选D.【点睛】本题考查了多边形的内角和定理,理解分三种情况是关键."} {"id": "5171", "image": ["6718.jpg"], "answer": "C", "solution": "null", "level": "八年级", "question": "如图, 在 $\\triangle A B C$ 中, 点 $D, E, F$ 分别在边 $B C, A B, C A$ 上, 且 $D E / / C A, D F / / B A$. 下列四种说法:(1)四边形 $A E D F$ 是平行四边形;(2)如果 $\\angle B A C=90^{\\circ}$, 那么四边形 $A E D F$ 是矩形;(3)如果 $A D$ 平分 $\\angle B A C$, 那么四边形 $A E D F$ 是菱形;(4) 如果 $A D \\perp B C$, 且 $A B=A C$, , 那么四边形 $A E D F$ 是正方形.其中, 正确的有", "options": "A. 1 个B. 2 个C. 3 个D. 4 个", "subject": "组合几何学", "analysis": "【点拨】先由两组对边分别平行的四边形为平行四边形,根据DE//CA,DF//BA,得出AEDF为平行四边形,得出(1)正确;当angleBAC=90^circ,根据推出的平行四边形AEDF,利用有一个角为直角的平行四边形为矩形可得出(2)正确;若AD平分angleBAC,得到一对角相等,再根据两直线平行内错角相等又得到一对角相等,等量代换可得angleEAD=angleEDA,利用等角对等边可得一组邻边相等,根据邻边相等的平行四边形为菱形可得出(3)正确;由AB=AC,ADperpBC,根据等腰三角形的三线合一可得AD平分angleBAC,同理可得四边形AEDF是菱形,(4)错误,进而得到正确说法的个数.【解析】解:becauseDE//CA,DF//BA,therefore四边形AEDF是平行四边形,选项(1)正确;若angleBAC=90^circ,therefore平行四边形AEDF为矩形,选项(2)正确:若AD平分angleBAC,thereforeangleEAD=angleFAD,又DE//CA,thereforeangleEDA=angleFAD,thereforeangleEAD=angleEDA,thereforeAE=DE,therefore平行四边形AEDF为菱形,选项(2)正确;若AB=AC,ADperpBC,thereforeAD平分angleBAC,同理可得平行四边形AEDF为菱形,选项(4)错误,则其中正确的个数有3个.故选C.【点睛】此题考查了平行四边形的定义,菱形、矩形的判定,涉及的知识有:平行线的性质,角平分线的定义,以及等腰三角形的判定与性质."} {"id": "5185", "image": ["6737.jpg"], "answer": "C", "solution": "null", "level": "八年级", "question": "如图, 菱形 $A B C D$ 的对角线 $A C, B D$ 相交于点 $O$, 若 $A C=6, B D=8, A E \\perp B C$, 垂足为 $E$, 则 $A E$ 的长为 $(\\quad)$", "options": "A. 12B. 14C. $\\frac{24}{5}$D. $\\frac{48}{5}$", "subject": "组合几何学", "analysis": "【点拨】利用菱形的面积公式:frac12ACcdotBD=BCcdotAE,即可解决问题.【解析】解:because四边形ABCD是菱形,thereforeACperpBD,OA=OC=frac12AC=3,OB=OD=frac12BD=4,thereforeAB=BC=sqrt3^2+4^2=5,becausefrac12ACcdotBD=BCcdotAE,thereforefrac12times6times8=5AE,thereforeAE=frac245故选C.【点睛】本题考查菱形的性质、勾股定理等知识,解题的关键是学会利用面积法求线段的长,属于中考常考题型."} {"id": "5192", "image": ["6744.jpg", "6745.jpg", "6746.jpg"], "answer": "C", "solution": "null", "level": "八年级", "question": "如图, 在菱形 $A B C D$ 中, $A B=B D$, 点 $E 、 F$ 分别是 $A B 、 A D$ 上任意的点 (不与端点重合), 且 $A E=D F$, 连接 $B F$ 与 $D E$ 相交于点 $G$, 连接 $C G$ 与 $B D$ 相交于点 $H$. 给出如下几个结论:(1) $\\angle A D E=\\angle D B F$;(2) $\\triangle D A E \\cong \\triangle B D G ;$(3)若 $A F=2 D F$, 则 $B G=6 G F ;$ (4) $C G$ 与 $B D$ 一定不垂直;(5) $\\angle B G E=60^{\\circ}$. 其中正确的结论个数为 ( )", "options": "A. 5B. 4C. 3D. 2", "subject": "组合几何学", "analysis": "【点拨】(1)先证明triangleABD为等边三角形,根据“SAS”证明triangleAEDcongtriangleDFB,利用全等三角形的性质解答即可;(2)先证明triangleABD为等边三角形,根据“SAS”证明triangleAEDcongtriangleDFB;(3)过点F作FP//AE于P点,根据题意有FP:AE=DF:DA=1:3,则FP:BE=1:6=FG:BG,即BG=6GF;(4)因为点E、F分别是AB、AD上任意的点(不与端点重合),且AE=DF,当点E,F分别是AB,AD中点时,CGperpBD;(5)angleBGE=angleBDG+angleDBF=angleBDG+angleGDF=60^circ【解析】解:(1)becauseABCD为菱形,thereforeAB=AD,becauseAB=BD,thereforetriangleABD为等边三角形,thereforeangleA=angleBDF=60^circ又becauseAE=DF,AD=BD,thereforetriangleAEDcongtriangleDFB,thereforeangleADE=angleDBF,故本选项正确;(2)becauseABCD为菱形,thereforeAB=AD,becauseAB=BD,thereforetriangleABD为等边三角形,thereforeangleA=angleBDF=60^circ,又becauseAE=DF,AD=BD,thereforetriangleAEDcongtriangleDFB,故本选项错误;(3)过点F作FP//AE交DE于P点(如图2),becauseAF=2FD,thereforeFP:AE=DF:DA=1:3,becauseAE=DF,AB=AD,thereforeBE=2AE,thereforeFP:BE=FP:2AE=1:6,becauseFP//AE,thereforePF//BE,thereforeFG:quadBG=FP:quadBE=1:quad6,即BG=6GF,故本选项正确;(4)当点E,F分别是AB,AD中点时(如图3),由(1)知,triangleABD,triangleBDC为等边三角形,because点E,F分别是AB,AD中点,thereforeangleBDE=angleDBG=30^circ,thereforeDG=BG,在triangleGDC与triangleBGC中,leftbeginarraylDG=BGCG=CGCD=CBendarrayright.thereforetriangleGDCcongtriangleBGC,thereforeangleDCG=angleBCG,thereforeCHperpBD,即CGperpBD,故本选项错误;(5)becauseangleBGE=angleBDG+angleDBF=angleBDG+angleGDF=60^circ,为定值,故本选项正确;综上所述,正确的结论有(1)(3),共3个,故选C.图3图2【点睛】此题综合考查了菱形的性质,等边三角形的判定与性质,全等三角形的判定和性质,作出辅助线构造出全等三角形,把不规则图形的面转化为两个全等三角形的面积是解题的关键."} {"id": "4095", "image": ["5778.jpg"], "answer": "A", "solution": "null", "level": "八年级", "question": "如图是一个平分角的仪器, 其中 $A B=A D, B C=D C$.\n\n将点 $A$ 放在角的顶点, $A B$ 和 $A D$ 沿着角的两边固定,\n\n沿 $A C$ 画一条射线 $A E, A E$ 就是这个角的平分线. 此仪器的原理是 ( )\n\n", "options": "A. SSS\nB. SAS\nC. ASA\nD. AAS", "subject": "组合几何学", "analysis": ""} {"id": "4125", "image": ["5807.jpg"], "answer": "C", "solution": "null", "level": "八年级", "question": "如图, 在 $\\triangle A B C$ 中, 已知点 $D, E, F$ 分别为边 $A C, B D, C E$ 的中点,且阴影部分图形面积等于 4 平方厘米,则 $\\triangle A B C$ 的面积为( )\n\n", "options": "8 平方厘米\nB. 12 平方厘米\nC. 16 平方厘米\nD. 18 平方厘米", "subject": "组合几何学", "analysis": ""} {"id": "4127", "image": ["5809.jpg"], "answer": "A", "solution": "null", "level": "八年级", "question": "如图, 点 $E$ 在 $\\triangle D B C$ 的边 $D B$ 上, 点 $A$ 在 $\\triangle D B C$ 内部, $\\angle D A E=\\angle B A C=90^{\\circ}, A D=A E, A B=A C$.\n\n给出下列结论:\n(1) $B D=C E$;\n(2) $\\angle A B D+\\angle E C B=45^{\\circ}$\n(3) $B D \\perp C E ;$\n(4) $B E^{3}=2\\left(A D^{2}+A B^{2}\\right)-C D^{2}$. 其中正确的是( )\n\n", "options": "(1)(2)(3)(4)\nB. (2)(4)\nC. (1)(2)(3)\nD. (1)(3)(4)", "subject": "组合几何学", "analysis": ""} {"id": "4148", "image": ["5831.jpg"], "answer": "D", "solution": "null", "level": "八年级", "question": "如图, $\\angle A=15^{\\circ}, A B=B C=C D=D E=E F$, 则 $\\angle D E F$ 等于()\n\n", "options": "$90^{\\circ}$\nB. $75^{\\circ}$\nC. $70^{\\circ}$\nD. $60^{\\circ}$", "subject": "组合几何学", "analysis": ""} {"id": "5830", "image": ["7228.jpg"], "answer": "C", "solution": "null", "level": "八年级", "question": "用三种边长相等的正多边形地砖铺地, 其顶点拼在一起, 刚好能完全铺满地面.已知正多边形的边\n\n数为 $\\mathrm{x}, \\mathrm{y}, \\mathrm{z}$, 则 $\\frac{1}{x}+\\frac{1}{y}+\\frac{1}{2}$ 的值为 $(\\quad)$", "options": "A. 1\n$\\frac{2}{3}$\n$\\frac{1}{2}$\n$\\frac{1}{3}$", "subject": "组合数学", "analysis": "由题意知, 这 3 种多边形的 3 个内角之和为 360 度, 已知正多边形的边数为 $\\mathrm{x} 、 \\mathrm{y} 、 \\mathrm{z}$, 那么这三个多边形的内角和可表示为: $\\frac{(x-2) x_{180}}{x}+\\frac{(y-2) \\times 180}{y}+\\frac{(z-2) x_{180}}{z}=360$, 两边都除以 180 得: 1\n\n\n\n故答案为: C."} {"id": "5142", "image": [], "answer": "A", "solution": "null", "level": "八年级", "question": "用反证法证明命题 “在三角形中, 至少有一个内角大于或等于 $60^{\\circ}$ ” 时, 先假设 ( )", "options": "A. 每个内角都小于 $60^{\\circ}$B. 每个内角都大于 $60^{\\circ}$C. 没有一个内角小于等于 $60^{\\circ}$D. 每个内角都等于 $60^{\\circ}$", "subject": "逻辑题", "analysis": "【点拨】假设命题的结论不成立,假定命题的结论反面成立即可.【解析】解:用反证法证明“在三角形中,至少有一个内角大于或等于60^circ”时,应先假设在三角形中,没有一个内角大于或等于60^circ,即每个内角都小于60^circ.故选A.【点睛】本题考查了反证法:掌握反证法的一般步骤(假设命题的结论不成立;从这个假设出发,经过推理论证,得出矛盾;由矛盾判定假设不正确,从而肯定原命题的结论正确)."} {"id": "5167", "image": [], "answer": "A", "solution": "null", "level": "八年级", "question": "用反证法证明命题 “三角形中最多有一个角是钝角时, 下列假设正确的是", "options": "A. 三角形中至少有两个角是钝角B. 三角形中没有一个角是钝角C. 三角形中三个角都是针角D. 三角形中至少有一个角是钝角", "subject": "逻辑题", "analysis": "【点拨】在反证法的步骤中,第一步是假设结论不成立,可据此进行解答.【解析】解:根据反证法的步骤,则可假设三角形中至少有两个角是钝角.故选A.【点睛】本题考查了反证法,解此题关键要懂得反证法的意义及步骤."} {"id": "4195", "image": [], "answer": "B", "solution": "null", "level": "八年级", "question": "下列说法中错误的个数是 $(\\quad)$\n\n(1) 任何一个命题都有逆命题\n\n(2)若原命题是假命题, 则它的逆命题也是假命题\n\n(3)任何一个定理都有逆定理\n\n(4)若原命题是真命题, 则它的逆命题也是真命题", "options": "4\nB. 3\nC. 2\nD. 1", "subject": "逻辑题", "analysis": ""} {"id": "14087", "image": ["2908.jpg", "2909.jpg"], "answer": "C", "solution": "null", "level": "四年级", "question": "下列几款汽车标志, 不是轴对称图形的是 ( )。", "options": "A.\n\n\nB.\n\n\nc.", "subject": "度量几何学", "analysis": "答案: C"} {"id": "13982", "image": [], "answer": "B", "solution": "null", "level": "四年级", "question": "下列说法中不正确的是 ( $\\quad)$ 。", "options": "A. 等边三角形是锐角三角形\n\nB. 直角三角形不可能是等腰三角形\n\nC.最大角是直角的三角形一定是直角三角形", "subject": "度量几何学", "analysis": "答案:B \\quad"} {"id": "13983", "image": [], "answer": "C", "solution": "null", "level": "四年级", "question": "周长是 $12 \\mathrm{~cm}$ 的三角形, 其中两条边的长度可以是( )。", "options": "A. $2 \\mathrm{~cm}, 6 \\mathrm{~cm}$\nB. $4 \\mathrm{~cm}, 6 \\mathrm{~cm}$\nC. $3 \\mathrm{~cm}, 4 \\mathrm{~cm}$", "subject": "度量几何学", "analysis": "答案: C \\quad"} {"id": "13984", "image": [], "answer": "C", "solution": "null", "level": "四年级", "question": "等腰三角形中, 有一个内角是 $50^{\\circ}$, 另外两个内角分别是 ( )。", "options": "A. $50^{\\circ} 、 80^{\\circ}$\nB. $65^{\\circ} 、 65^{\\circ}$\nC. $50^{\\circ} 、 80^{\\circ}$ 或 $65^{\\circ} 、 65^{\\circ}$", "subject": "度量几何学", "analysis": "C"} {"id": "13985", "image": ["2867.jpg"], "answer": "D", "solution": "null", "level": "四年级", "question": "将一个正方形剪去一个角后, 剩下图形的内角和不可能是 ( )。", "options": "A. $180^{\\circ}$\nB. $360^{\\circ}$\nC. $540^{\\circ}$\nD. $720^{\\circ}$", "subject": "度量几何学", "analysis": ""} {"id": "14036", "image": [], "answer": "A", "solution": "null", "level": "四年级", "question": "用米作单位计算, 3 米 50 厘米 +6 米 5 分米的正确算式是 ( )。", "options": "A.", "subject": "度量几何学", "analysis": "A"} {"id": "14060", "image": ["2895.jpg"], "answer": "B", "solution": "null", "level": "四年级", "question": "下图是由两个边长是 $2 \\mathrm{dm}$ 的正方形拼成的图形, 图中阴影部分的面积是 $(\\quad) \\mathrm{dm}^{2}$ 。", "options": "A. 2\nB. 4\nC. 8\n\n", "subject": "度量几何学", "analysis": "答案:\\mathrm{B} \\quad"} {"id": "14062", "image": ["2900.jpg"], "answer": "A", "solution": "null", "level": "四年级", "question": "下面四个图形中,周长与其他三个图形不同的是()。\n\n\n\nA\n\nB\n\nC\n\nD", "options": "A. 6\nB. 4\nC. 3\nD. 2", "subject": "度量几何学", "analysis": "答案:\\mathrm{A}$"} {"id": "28071", "image": ["13587.jpg"], "answer": "B", "solution": "null", "level": "四年级", "question": "如下图所示, $\\mathrm{M}$ 点可能表示的数是()。\n\n\n", "options": "A. 1.9\nB. 1.7\nC. 1.5\nD. 0.8\n\n", "subject": "度量几何学", "analysis": "解B\n\n【分析】 M 点在 1 2 之间, 超过了一半的位置, 所以表示的是大于 1.5 而小于 2 的数, 据此解答。\n\n【详解】如下图所示, M 点可能表示的数是(1.7)。\n\n故答案为: B\n\n【点睛】本题考查小数的意义, 应熟练掌握并灵活运用。"} {"id": "28251", "image": ["13607.jpg"], "answer": "B", "solution": "null", "level": "四年级", "question": "用一个大正方形表示 1 平方分米,阴影部分表示 1 平方分米 7 平方厘米的是()。\n\n", "options": "A.\n\n\n", "subject": "度量几何学", "analysis": "解B\n\n【分析】1 平方分米是一个大正方形, 1 平方分米 $=100$ 平方厘米; 把 1 平方分米大正方形平均分成 100 份, 每份是 1 平方厘米, 1 平方厘米是 $\\frac{1}{100}$ 平方分米, 7 平方厘米取其中的 7 份。\n\n【详解】 1 平方分米 7 平方厘米, 涂色一个大正方形表示 1 平方分米, 再把一个大正方形平均分成 100 份, 涂其中的 7 份。\n\n故答案为: B\n\n【点睛】根据小数的意义进行判断: 把一个整体平均分成 10 份, 每一份是 $\\frac{1}{10}$, 也就是 0.1 , 再看取几份就可以;\n\n把一个整体平均分成 100 份, 每一份是 $\\frac{1}{100}$, 也就是 0.01 , 再看取几份就可以......"} {"id": "28678", "image": [], "answer": "A", "solution": "null", "level": "四年级", "question": "下面几组纸条中(单位:厘米),不能摆成三角形的是()。\n", "options": "A. $1,2 、 3$\nB. $3 、 4 、 5$\nC. $4 、 5 、 6$\n", "subject": "度量几何学", "analysis": "解A\n\n【分析】根据三角形三边关系: 三角形的两边之和大于第三边, 三角形的两边的差一定小于第三边;进行解答即可。\n\n【详解】A. 1 厘米 +2 厘米 $=3$ 厘米, 3 厘米 $=3$ 厘米, 不符合三角形三边关系, 所以不能摆成三角形。\n\nB. 3 厘米 +4 厘米 $>5$ 厘米, 4 厘米 -3 厘米 $<5$ 厘米, 符合三角形三边关系, 所以能摆成三角形。\n\nC. 4 厘米 +5 厘米 $>6$ 厘米, 6 厘米 -4 厘米 $<5$ 厘米, 符合三角形三边关系, 所以能摆成三角形;故答案为: A\n\n【点睛】熟练掌握三角形三边关系, 是解答本题的关键。"} {"id": "28689", "image": [], "answer": "C", "solution": "null", "level": "四年级", "question": "在一个三角形中, $\\angle 1=20^{\\circ}, \\angle 2=40^{\\circ}$, 这是一个()。\n", "options": "A. 等边三角形\nB. 直角三角形\nC. 钝角三角形\n", "subject": "度量几何学", "analysis": "解C\n\n【分析】三角形的内角和是 $180^{\\circ}$, 用减法计算出 $\\angle 3$ 的度数, 再根据三角形的分类标准进行选择即可。三条边都相等, 三个角都相等的三角形是等边三角形; 有一个角是直角的三角形是直角三角形; 有一个角是钝角的三角形是钝角三角形, 依此解答即可。\n\n【详解】 $180^{\\circ}-20^{\\circ}-40^{\\circ}$\n\n$=160^{\\circ}-40^{\\circ}$\n\n$=120^{\\circ}$\n\n$90^{\\circ}<120^{\\circ}<180^{\\circ}$, 即这是一个钝角三角形。\n\n故答案为: C\n\n【点睛】解答此题的关键是要熟练掌握三角形的分类标准, 以及熟记三角形的内角和度数。"} {"id": "28696", "image": ["13720.jpg", "13721.jpg"], "answer": "C", "solution": "null", "level": "四年级", "question": "下图是一个四边形, $\\angle 1+\\angle 2+\\angle 3+\\angle 4$ 等于()。\n\n\n", "options": "A. $180^{\\circ}$\nB. $540^{\\circ}$\nC. $360^{\\circ}$\n\n", "subject": "度量几何学", "analysis": "解C\n\n【分析】把该四边形作出对角线, 可将四边形分成 2 个三角形,所以该四边形的内角和等于三角形内角和的 2 倍, 而三角形内角和为 $180^{\\circ}$, 据此解答。\n\n【详解】如图所示:\n\n\n\n$\\angle 1+\\angle 2+\\angle 3+\\angle 4=180^{\\circ} \\times 2=360^{\\circ}$ 。\n\n故答案为: C\n\n【点睛】本题考查多边形的内角和, 可把多边形分为若干个三角形, 结合三角形内角和为 $180^{\\circ}$ 进行计算。"} {"id": "14130", "image": [], "answer": "A", "solution": "null", "level": "四年级", "question": "在一个三角形中, 有两个内角都是 $60^{\\circ}$, 这个三角形是 ( ) 三角形。", "options": "A. 锐角\nB. 直角\nC. 钝角", "subject": "度量几何学", "analysis": ""} {"id": "14133", "image": [], "answer": "A", "solution": "null", "level": "四年级", "question": "用放大 20 倍的放大镜看一个 $50^{\\circ}$ 的角, 这个角是 ( )。", "options": "A. $50^{\\circ}$\nB. $100^{\\circ}$\nC. $1000^{\\circ}$", "subject": "度量几何学", "analysis": ""} {"id": "14149", "image": ["2928.jpg"], "answer": "B", "solution": "null", "level": "四年级", "question": "下列事物中运用了三角形稳定性的是()。\n", "options": "A. 升降机\nB. 折叠椅\nC. 七巧板中的三角形", "subject": "度量几何学", "analysis": "答案: $\\mathrm{B}$"} {"id": "14151", "image": [], "answer": "C", "solution": "null", "level": "四年级", "question": "下面 3 组角中, ( ) 组中的三个角可以是一个三角形的三个内角。", "options": "A. $50^{\\circ} 、 50^{\\circ} 、 50^{\\circ}$\nB. $30^{\\circ} 、 75^{\\circ} 、 65$\nC. $80^{\\circ} 、 88^{\\circ} 、 12^{\\circ}$\n\n20.针角三角形中两个锐角的和一定() $90^{\\circ}$\nA. 小于\nB. 大于\nC. 等于", "subject": "度量几何学", "analysis": "答案:$\\mathrm{C}$"} {"id": "14249", "image": [], "answer": "B", "solution": "null", "level": "四年级", "question": "下列各种长度的小棒, 不能拼成三角形的一组是()。", "options": "A. $3 \\mathrm{~m} 、 6 \\mathrm{~m} 、 8 \\mathrm{~m}$\nB. $1 \\mathrm{dm} 、 2 \\mathrm{dm} 、 3 \\mathrm{dm}$\nC. $3 \\mathrm{~cm} 、 4 \\mathrm{~cm} 、 5 \\mathrm{dm}$", "subject": "度量几何学", "analysis": "答案: $\\mathrm{B}$"} {"id": "14250", "image": [], "answer": "A", "solution": "null", "level": "四年级", "question": "三角形按角分, 可以分为 $(\\quad)$ 。", "options": "A. 直角三角形、锐角三角形、钝角三角形\n\nB. 等腰三角形、等边三角形、针角三角形\n\nC. 等要三角形、直角三角形、锐角三角形", "subject": "度量几何学", "analysis": "答案: A"} {"id": "14251", "image": [], "answer": "B", "solution": "null", "level": "四年级", "question": "七边形的内角和是 ( ) 。", "options": "A. $630^{\\circ}$\nB. $900^{\\circ}$\nC. $1260^{\\circ}$", "subject": "度量几何学", "analysis": "答案: $\\mathrm{B}$"} {"id": "14252", "image": [], "answer": "C", "solution": "null", "level": "四年级", "question": "如果一个等腰三角形最小的一个内角大于 $45^{\\circ}$, 这个三角形一定是 ( ) 三角形。", "options": "A. 钝角\nB. 直角\nC. 锐角", "subject": "度量几何学", "analysis": "答案: C"} {"id": "14253", "image": [], "answer": "A", "solution": "null", "level": "四年级", "question": "当一个三角形其中的两条边分别是 3 厘米和 8 厘米时, 第三条边的长度可能是 ()。", "options": "A. 6 厘米\nB. 5 厘米\nC. 4 厘米", "subject": "度量几何学", "analysis": "答案:A"} {"id": "28753", "image": ["13731.jpg"], "answer": "C", "solution": "null", "level": "四年级", "question": "钝角三角形有 ( )条高。\n", "options": "A. 1\nB. 2\nC. 3\nD. 无数条\n", "subject": "度量几何学", "analysis": "解$\\mathrm{C}$\n\n【分析】有一个角是钝角的三角形是钝角三角形, 由三条边组成的封闭图形是三角形, 从三角形的一个顶点到它的对边作一条垂线, 顶点和垂足之间的线段叫做三角形的高, 这条对边叫做三角形的底,依此选择。\n\n\n\n由此可知, 钝角三角形有 3 条高。\n\n故答案为: C\n\n【点睛】解答此题的关键是应熟练掌握钝角三角形的特点, 以及三角形的高及画法。"} {"id": "28771", "image": ["13748.jpg"], "answer": "A", "solution": "null", "level": "四年级", "question": "一块三角形玻璃破碎后的形状如下图, 这个三角形是()。\n\n\n", "options": "A. 钝角三角形\nB. 直角三角形\nC. 锐角三角形\nD. 以上都有可能\n\n", "subject": "度量几何学", "analysis": "解A\n\n【分析】因为三角形的内角度数和是 $180^{\\circ}$, 根据图中的两个内角度数可求打碎的一个角的度数, 再根据三角形的分类即可作出判断。\n\n【详解】 $180^{\\circ}-30^{\\circ}-40^{\\circ}$\n\n$=150^{\\circ}-40^{\\circ}$\n\n$=110^{\\circ}$\n\n故是一个钝角三角形。\n\n故答案为: A\n\n【点睛】考查了三角形的内角和定理:三角形的内角和等于 $180^{\\circ}$; 同时考查了三角形的按角分类,关键明确: (1)有一个角是直角的三角形是直角三角形, (2)三个角都为锐角的三角形是锐角三角形, (3)\n有一个角是钝角的三角形是钝角三角形。"} {"id": "28772", "image": [], "answer": "B", "solution": "null", "level": "四年级", "question": "用三根小棒拼三角形, 其中两根小棒分别是 $12 \\mathrm{~cm} 、 17 \\mathrm{~cm}$, 第三根小棒的长度是()。\n", "options": "A. $30 \\mathrm{~cm}$\nB. $7 \\mathrm{~cm}$\nC. $5 \\mathrm{~cm}$\nD. $4 \\mathrm{~cm}$\n", "subject": "度量几何学", "analysis": "解$\\mathrm{B}$\n\n【分析】任意三角形的两边之和必须大于第三边, 任意两边的差必须小于第三边, 据此解答。\n\n【详解】 $12+17=29(\\mathrm{~cm})$\n\n$17-12=5(\\mathrm{~cm})$\n\n根据三角形的三边关系可知, 第三边要大于 $5 \\mathrm{~cm}$ 小于 $29 \\mathrm{~cm}, 7 \\mathrm{~cm}$ 符合题意。\n\n故答案为: B\n\n【点睛】本题考查了三角形的三边关系的应用。"} {"id": "28773", "image": [], "answer": "B", "solution": "null", "level": "四年级", "question": "下面说法正确的是 ( )。\n", "options": "A. 三角形和平行四边形都具有稳定性\nB. 同一平面内, 两条直线不相交就互相平行\nC. 个 (一) 、十、百、千 .都是数位\nD. 把线段的一端无限延长就得到一条直线\n", "subject": "度量几何学", "analysis": "解B\n\n【分析】三角形具有稳定性, 平行四边形容易变形; 两条直线相交所成的四个角中, 有一个角是直角时, 就说这两条直线互相垂直; 在同一平面内, 不相交的两条直线叫做平行线; 计数单位就是数字计数单位, 计数单位应包含整数部分和小数部分, 整数部分从小到大依次是: 个、十、百、千、万、十万、百万、千万..... ; 小数部分是: 十分之一、百分之一、千分之一..... ; 整数部分没有最大的计数单位, 小数部分没有最小的计数单位; 计数单位按照一定顺序排列起来, 它们所占的位置叫数位; 线段有两个端点, 有具体的长度; 直线没有端点, 可以无限延长, 据此解答。\n\n【详解】A. 三角形具有稳定性, 平行四边形不具有稳定性, 故该选项错误;\n\nB. 同一平面内, 两条直线不相交就互相平行, 故该选项正确;\n\nC. 个 (一)、十、百、千...... 都是计数单位而不是数位, 故该选项错误;\n\nD. 线段有两个端点, 是有具体的长度, 直线没有端点, 可以无限延长, 故该选项错误。由分析可知:说法正确的是(同一平面内,两条直线不相交就互相平行)。\n\n故答案为: B\n\n【点睛】本题考查三角形具有稳定性, 平行和垂直的性质, 线段和直线的特点, 熟练掌握并灵活运用。"} {"id": "28868", "image": [], "answer": "C", "solution": "null", "level": "四年级", "question": "(2021 四下.宁津期末) 用“米”作单位计算, “ 8 米 6 厘米 +5 米 60 厘米”的正确算式是 ( )\n", "options": "A. $8.6+5.6$\nB. $8.06+5.06$\nC. $8.06+5.6$\n", "subject": "度量几何学", "analysis": "解C"} {"id": "28909", "image": [], "answer": "B", "solution": "null", "level": "四年级", "question": "下面三组小棒中, 不能围成三角形的是()。\n", "options": "A. $7 \\mathrm{~cm} 、 7 \\mathrm{~cm} 、 5 \\mathrm{~cm}$\nB. $4 \\mathrm{~cm} 、 4 \\mathrm{~cm} 、 8 \\mathrm{~cm}$\nC. $4 \\mathrm{~cm} 、 4 \\mathrm{~cm} 、 4 \\mathrm{~cm}$\n", "subject": "度量几何学", "analysis": "解$\\mathrm{B}$\n\n【分析】三角形的任意两边之和大于第三边, 任意两边之差小于第三边。将三个选项运用三角形三边关系判定, 即可解答此题。\n\n【详解】A. $7 \\mathrm{~cm}+7 \\mathrm{~cm}>5 \\mathrm{~cm}, 7 \\mathrm{~cm}-7 \\mathrm{~cm}<5 \\mathrm{~cm}$, 两边之和大于第三边, 两边之差小于第三边, A 选项能围成三角形;\n\nB. $4 \\mathrm{~cm}+4 \\mathrm{~cm}=8 \\mathrm{~cm}$, 两边之和等于第三边, B 选项不能围成三角形;\n\nC. $4 \\mathrm{~cm}+4 \\mathrm{~cm}>4 \\mathrm{~cm}, 4 \\mathrm{~cm}-4 \\mathrm{~cm}<4 \\mathrm{~cm}$, 两边之和大于第三边, 两边之差小于第三边, C 选项能围成三角形。\n\n故答案为: B\n\n【点睛】本题考查三角形三边关系的应用。熟练掌握三角形三边关系是解决此题的关键。"} {"id": "28920", "image": ["13768.jpg"], "answer": "C", "solution": "null", "level": "四年级", "question": "一个三角形的三个内角分别是 $\\angle 1 、 \\angle 2$ 和 $\\angle 3$, 那么 $\\angle 1+\\angle 2+\\angle 3=$ ()。\n", "options": "A. $90^{\\circ}$\nB. $120^{\\circ}$\nC. $180^{\\circ}$\n", "subject": "度量几何学", "analysis": "解C\n\n【分析】将三角形中 $\\angle 1 、 \\angle 2$ 和 $\\angle 3$ 剪下来, 刚好可以拼成一个平角 (如下图所示), 1 平角 $=180^{\\circ}$,依此选择即可。\n\n\n【详解】三角形的内角和为 $180^{\\circ}$, 即一个三角形的三个内角分别是 $\\angle 1 、 \\angle 2$ 和 $\\angle 3$, 那么 $\\angle 1+\\angle 2$ $+\\angle 3=180^{\\circ}$ 。\n\n故答案为: $\\mathrm{C}$\n\n【点睛】熟记三角形的内角和度数是解答此题的关键。"} {"id": "28927", "image": [], "answer": "B", "solution": "null", "level": "四年级", "question": "一个等腰三角形的两条边长分别是 6 厘米和 15 厘米, 那么这个三角形的周长是()。\n", "options": "A. 27 厘米\nB. 36 厘米\nC. 30 厘米\n", "subject": "度量几何学", "analysis": "解$\\mathrm{B}$\n\n【分析】根据三角形的三边关系:任意两边之和大于第三边, 任意两边之差小于第三边, 可知这个等腰三角形的底是 6 厘米, 腰长是 15 厘米, 再根据三角形的周长公式解答即可。\n\n【详解】 $6+15+15$\n\n$=21+15$\n\n$=36$ (厘米)\n\n一个等腰三角形的两条边长分别是 6 厘米和 15 厘米, 那么这个三角形的周长是(36 厘米)。故答案为: B\n\n【点睛】此题考查三角形周长公式的灵活运用以及等腰三角形的特征和应用, 解答的关键是根据三边\n关系确定三角形的三条边。"} {"id": "28928", "image": [], "answer": "C", "solution": "null", "level": "四年级", "question": "一个三角形的两条边分别长 $6 \\mathrm{~cm} 、 5 \\mathrm{~cm}$, 第三条边可以是()。\n", "options": "A. $12 \\mathrm{~cm}$\nB. $11 \\mathrm{~cm}$\nC. $8 \\mathrm{~cm}$\n", "subject": "度量几何学", "analysis": "解C\n\n【分析】根据三角形三边关系:两边之和大于第三边, 两边的差一定小于第三边, 进行解答即可。\n\n【详解】由三角形三边关系可知: $6-5<$ 第三条边 $<6+5$;\n\n所以: $1<$ 第三条边 $<11$, 即第三条边的长在 $1 \\mathrm{~cm} \\sim 11 \\mathrm{~cm}$ 之间(不包括 $1 \\mathrm{~cm} 、 11 \\mathrm{~cm}$ );\n\n结合选项中的数据可知, 第三条边的长可能是 $8 \\mathrm{~cm}$ 。\n\n故答案为: C\n\n【点睛】熟练掌握三角形三边关系, 是解答此题的关键。"} {"id": "28929", "image": [], "answer": "C", "solution": "null", "level": "四年级", "question": "一个三角形的最大角是最小角的 5 倍, 另一个角是最小角的 3 倍, 这是一个()。\n", "options": "A. 锐角三角形\nB. 直角三角形\nC. 钝角三角形\n", "subject": "度量几何学", "analysis": "解C\n\n【分析】根据题意可假设最小角的度数为 1 份, 则最大的角的度数为 5 份, 另一个角的度数为 3 份,则一共有 $(1+5+3)$ 份, 三角形的内角和是 $180^{\\circ}$, 因此用 $180^{\\circ}$ 除以一共的份数, 从而计算出每份的度数, 再根据最大角所对应的份数计算出最大角的度数, 最后根据三角形的分类标准进行选择即可。\n\n【详解】 $1+5+3=9$ (份)\n\n$180 \\div 9=20^{\\circ}$\n\n$20^{\\circ} \\times 5=100^{\\circ}$\n\n$100^{\\circ}>90^{\\circ}$, 即最大的角是一个钝角, 因此这是一个钝角三角形。\n\n故答案为: C\n\n【点睛】此题考查的是三角形的内角和, 以及三角形的分类, 应先计算出三角形中最大角的度数再进行判断。"} {"id": "28961", "image": [], "answer": "D", "solution": "null", "level": "四年级", "question": "四根长度分别为 $3 \\mathrm{~cm} 、 4 \\mathrm{~cm} 、 7 \\mathrm{~cm} 、 10 \\mathrm{~cm}$ 的木条, 以其中三根的长为边长钉成一个三角形框架,那么这个框架的周长可能是()。\n", "options": "A. $14 \\mathrm{~cm}$\nB. $17 \\mathrm{~cm}$\nC. $20 \\mathrm{~cm}$\nD. $21 \\mathrm{~cm}$\n", "subject": "度量几何学", "analysis": "解$\\mathrm{D}$\n\n【分析】先写出任意三根木条的所有组合情况, 再根据三角形的三边关系“任意两边之和大于第三边”进行分析解答。\n\n【详解】任意三根木条的组合有四种情况:\n\n(1) $3 \\mathrm{~cm} 、 4 \\mathrm{~cm} 、 7 \\mathrm{~cm} ; 3+4=7$, 不能组成三角形;\n\n(2) $3 \\mathrm{~cm} 、 4 \\mathrm{~cm} 、 10 \\mathrm{~cm} ; 3+4=7,7<10$, 不能组成三角形;\n\n(3) $3 \\mathrm{~cm} 、 7 \\mathrm{~cm} 、 10 \\mathrm{~cm} ; 3+7=10$, 不能组成三角形;\n\n(4) $4 \\mathrm{~cm} 、 7 \\mathrm{~cm} 、 10 \\mathrm{~cm} ; 4+7=11,11>10$, 能组成三角形;\n\n周长: $4+7+10=21(\\mathrm{~cm})$\n\n故答案为: D\n\n【点睛】本题考查三角形三边关系的应用, 掌握判断能否组成三角形的方法是看较小的两条边的和是否大于第三条边。"} {"id": "28980", "image": [], "answer": "C", "solution": "null", "level": "四年级", "question": "三角形中, 其中一个角是另两个角的和, 则此三角形是()。\n", "options": "A. 锐角三角形\nB. 钝角三角形\nC. 直角三角形\nD. 无法判断\n", "subject": "度量几何学", "analysis": "解C\n\n【分析】三角形的内角和是 $180^{\\circ}$, 若其中一个角是另两个角的和, 则这个角的度数是 $90^{\\circ}$, 据此解答即可。\n\n【详解】由分析可知:\n\n三角形中, 其中一个角是另两个角的和, 则此三角形是直角三角形。\n\n故答案为: C\n\n【点睛】本题考查直角三角形, 明确直角三角形的两个锐角的和等于 $90^{\\circ}$ 是解题的关键。"} {"id": "28983", "image": ["13796.jpg"], "answer": "B", "solution": "null", "level": "四年级", "question": "如图, $\\angle \\mathrm{C}-\\angle \\mathrm{A}=$ ( )。\n\n\n", "options": "A. $19^{\\circ}$\nB. $29^{\\circ}$\nC. $32^{\\circ}$\nD. $40^{\\circ}$\n", "subject": "度量几何学", "analysis": "解$\\mathrm{B}$\n\n【分析】在三角形 $\\mathrm{ABD}$ 中, 根据三角形的内角和为 $180^{\\circ}$ 求出 $\\angle \\mathrm{A}$ 的度数, 在三角形 $\\mathrm{ABC}$ 中, 根据三角形的内角和求出 $\\angle \\mathrm{C}$ 的度数, 最后求出 $\\angle \\mathrm{C}$ 与 $\\angle \\mathrm{A}$ 的度数差。\n\n【详解】三角形的内角和是 $180^{\\circ}$ 。\n\n$\\angle \\mathrm{A}: 180^{\\circ}-\\left(25^{\\circ}+108^{\\circ}\\right)$\n\n$=180^{\\circ}-133^{\\circ}$\n\n$=47^{\\circ}$\n\n$\\angle \\mathrm{C}: 180^{\\circ}-\\left(32^{\\circ}+25^{\\circ}+47^{\\circ}\\right)$\n\n$=180^{\\circ}-104^{\\circ}$\n\n$=76^{\\circ}$\n\n$\\angle \\mathrm{C}-\\angle \\mathrm{A}=76^{\\circ}-47^{\\circ}=29^{\\circ}$\n\n故答案为: B\n\n【点睛】本题主要考查三角形内角和的应用, 熟记三角形的内角和为 $180^{\\circ}$ 是解答题目的关键。"} {"id": "28998", "image": [], "answer": "B", "solution": "null", "level": "四年级", "question": "一个三角形中至少有几个角是锐角()。\n", "options": "A. 1 个\nB. 2 个\nC. 3 个\n", "subject": "度量几何学", "analysis": "解$\\mathrm{B}$\n\n【分析】由三角形内角和为 180 度可知, 若一个三角形中只有一个锐角, 另外两个角的和一定大于或等于 180 度, 就不符合三角形内角和定理, 从而可以判断出一个三角形中至少有 2 个锐角; 据此解答。\n\n【详解】假设一个三角形中至少有 1 个锐角,\n\n则另外两个内角的度数和就会等于或大于 180 度,\n\n那么三角形的内角和就大于 180 度,\n\n这与三角形的内角和是 180 度是相违背的, 故假设不成立,\n\n所以一个三角形中至少有 2 个角是锐角。\n\n故答案为: B\n\n【点睛】本题考查了三角形内角和定理, 属于基础题, 关键是掌握三角形内角和为 180 度。"} {"id": "29003", "image": [], "answer": "B", "solution": "null", "level": "四年级", "question": "一个等腰三角形的两条边的长度分别是 3 厘米和 7 厘米, 这个三角形的周长是 ( ) 厘米。\n", "options": "A. 13\nB. 17\nC. 13 或 17\n", "subject": "度量几何学", "analysis": "解$\\mathrm{B}$\n\n【分析】等腰三角形的两条腰相等, 根据三角形三边关系: 三角形两边之和大于第三边, 三角形的两边的差一定小于第三边, 可得 $3+3<7$, 所以判断出该等腰三角形的腰长为 7 厘米, 底边长为 3 厘米,进而根据三角形的周长计算方法解答即可。\n\n【详解】因为 $3+3<7$,\n\n所以等腰三角形的腰长为 7 厘米, 底边长为 3 厘米,\n\n$7+7+3$\n\n$=14+3$\n\n$=17$ (厘米)\n\n这个三角形的周长是 17 厘米。\n\n故答案为: B\n【点睛】熟练掌握三角形三边关系和等腰三角形的特征, 是解答此题的关键。"} {"id": "29005", "image": [], "answer": "A", "solution": "null", "level": "四年级", "question": "等腰直角三角形的底角是()。\n", "options": "A. $45^{\\circ}$\nB. $60^{\\circ}$\nC. $90^{\\circ}$\n", "subject": "度量几何学", "analysis": "解A\n\n【分析】等腰直角三角形的两个底角相等, 并且顶角是直角, 三角形的内角和是 $180^{\\circ}$, 因此用 $180^{\\circ}$减去 $90^{\\circ}$ 后, 再除以 2 即可, 依此计算并选择。\n\n【详解】 $180^{\\circ}-90^{\\circ}=90^{\\circ}$\n\n$90^{\\circ} \\div 2=45^{\\circ}$\n\n即等腰直角三角形的底角是 $45^{\\circ}$ 。\n\n故答案为: A\n\n【点睛】此题考查的是等腰直角三角形的特点, 以及三角形的内角和, 应熟练掌握。"} {"id": "14094", "image": ["2911.jpg"], "answer": "A", "solution": "null", "level": "四年级", "question": "是从下面哪张纸上剪下来的?()。", "options": "A.\n\n\nB.\nc. $\\sqrt{3}$", "subject": "组合几何学", "analysis": "答案: A"} {"id": "14098", "image": [], "answer": "C", "solution": "null", "level": "四年级", "question": "下列图形中, 对称轴最多的是 ( )。", "options": "A. 等边三角形\nB. 长方形\nC. 圆形", "subject": "组合几何学", "analysis": "答案:C"} {"id": "14099", "image": [], "answer": "B", "solution": "null", "level": "四年级", "question": "( ) 一定是轴对称图形。", "options": "A. 三角形\nB. 正方形\nC. 梯形", "subject": "变换几何", "analysis": "答案: B"} {"id": "14107", "image": [], "answer": "B", "solution": "null", "level": "四年级", "question": "在 32+ (21-18) $\\div 3$ 的算式里, 最后一步求 ( )。", "options": "A. 商\nB. 和\nC. 差", "subject": "算术", "analysis": "B"} {"id": "14110", "image": [], "answer": "B", "solution": "null", "level": "四年级", "question": "下面各数去掉 0 后,大小不变的是 ( )。", "options": "A. 650\nB.", "subject": "算术", "analysis": "答案: B"} {"id": "27256", "image": [], "answer": "A", "solution": "null", "level": "四年级", "question": "下面的算式,得数不是 0 的是()。\n", "options": "A. $0+504$\nB. $0 \\div 36$\nC. $8 \\times 0 \\times 9$\nD. $127-127$\n", "subject": "算术", "analysis": "解A\n\n【分析】任何一个数与 0 相加, 还得这个数; 0 除以任何一个不为 0 的数, 还得 0 ; 连续几个数相乘,只要有一个因数是 0 , 这几个数的乘积就是 0 ; 两个相同的数相减, 结果得 0 , 依此计算并选择。\n\n【详解】A. $0+504=504$ 。\n\nB. $0 \\div 36=0$ 。\n\nC. $8 \\times 0 \\times 9=0$ 。\n\nD. $127-127=0$ 。\n\n故答案为: A\n\n【点晴】熟练掌握有关 0 的运算, 是解答此题的关键。"} {"id": "27266", "image": [], "answer": "C", "solution": "null", "level": "四年级", "question": "关于减法, 下面说法错误的是()\n\n", "options": "A. 如果被减数等于减数,那么它们的差一定是 0 。\n\nB. 一个数减 50 , 差也是 50 , 那么这个数是 100 。\n\nC. 如果被减数和减数都增加 15, 那么它们的差也增加 15 。\n\nD. 减数等于被减数减差。\n\n", "subject": "算术", "analysis": "解C\n\n【分析】被减数一减数=差、减数十差=被减数, 减数=被减数一差, 据此解题即可。\n\n【洋解】A. 被减数一减数=差, 所以, 如果被减数等于减数, 那么它们的差一定是 0 。\n\nB. 减数十差 $=$ 被减数, 据此可得: $50+50=100$, 所以, 一个数减 50 , 差也是 50 , 那么这个数是 100 。\n\nC. 假设被减数是 10 , 减数是 6 , 差是 $4,(10+15)-(6+15)=25-21=4$, 所以, 如果被减数和减数都增加 15 , 那么它们的差也增加 15 , 是错误的。\n\nD. 减数=被减数一差, 所以, 减数等于被减数减差是正确的。\n\n故答案为: C\n\n【点睛】正确理解减法各部分之间的意义与关系, 是解答此题的关键。"} {"id": "27273", "image": [], "answer": "C", "solution": "null", "level": "四年级", "question": "有 5 名学生去古桑洲游玩, 共付船票费 60 元,每人乘车用 2 元。平均每人花了多少钱?下面的列式正确的有()。\n", "options": "A. $(60+2) \\div 5$\nB. $60 \\div 5+2 \\times 5$\nC. $60 \\div 5+2$\nD. $60 \\div(5+2)$\n", "subject": "算术", "analysis": "解C\n\n【分析】根据题意, 用 60 元除以学生人数, 求出每人付船费需要的钱数, 再加上每人乘车所用的 2 元, 即可求出平均每人花的钱数; 据此列式为: $60 \\div 5+2$ 。\n\n【详解】 $60 \\div 5+2$\n\n$=12+2$\n\n$=14$ (元)\n\n所以, 平均每人花了 14 元钱, 正确列式为: $60 \\div 5+2$ 。\n\n故答案为: C\n\n【点睛】理清题中数量关系, 正确列式是解答此题的关键。"} {"id": "27274", "image": [], "answer": "D", "solution": "null", "level": "四年级", "question": "下面算式中,去掉括号后结果不变的是()。\n", "options": "A. $(72+18) \\times 54$\nB. $512-(38+12)$\nC. $(702-12) \\times 5$\nD. $63+(45 \\times 6)$\n", "subject": "算术", "analysis": "解D\n\n【分析】四则混合运算的顺序: 一个算式里, 如果只有加减法或者只有乘除法, 按照从左到右的顺序依次计算; 如果既有加减法、又有乘除法, 先算乘除法、再算加减法; 如果有括号, 先算括号里面的,\n然后逐项分析各个选项, 判断并得出答案即可, 据此解答。\n\n【详解】A. $(72+18) \\times 54$, 去掉括号后先计算乘法, 再计算加法, 计算结果会变, 故该选项错误;\n\nB. $512-(38+12)$, 去掉括号后先计算减法, 再计算加法, 计算结果会变, 故该选项错误;\n\nC. (702-12) ×5,去掉括号后先计算乘法,再计算减法,计算结果会变,故该选项错误;\n\nD. $63+(45 \\times 6)$, 去掉括号后,依旧先计算乘法,再计算加法,计算结果不会变,故该选项正确。故答案为: $\\mathrm{D}$\n\n【点睛】本题考查带有小括号的混合运算的运算顺序,应熟练掌握并正确计算。"} {"id": "27275", "image": [], "answer": "D", "solution": "null", "level": "四年级", "question": "下面的算式中, “口”一定不是 0 的是()。\n", "options": "A. $\\square+\\boldsymbol{\\Delta}=\\boldsymbol{\\Delta}$\nB. $\\square \\times \\Delta=0$\nC.\nD. $\\boldsymbol{\\Delta} \\div \\square=0$\n", "subject": "算术", "analysis": "解$\\mathrm{D}$\n\n【分析】有关 0 的运算: 0 乘任何数都得 0,0 除以任何非 0 数都得 0,0 加任何数都得任何数,任何数减 0 都得任何数, 据此解答即可。\n\n【详解】A. $\\square+\\boldsymbol{\\Delta}=\\boldsymbol{\\Delta}, 0$ 加任何数都得任何数, 所以“口”一定是 0 ;\n\nB. $\\square \\times \\boldsymbol{\\Delta}=0,0$ 乘任何数都得 0 , 所以“व”可能是 0 ;\n\nC. $\\boldsymbol{\\Delta}-\\square=\\boldsymbol{\\Delta}$ ,任何数减 0 都得任何数,所以“口”一定是 0 ;\n\nD. $\\Delta \\div \\square=0$, 因为“ $\\square$ ”是除数, 所以“口”一定不是 0 ;\n\n故答案为: D\n\n【点睛】此题重点考查有关 0 参与的运算。"} {"id": "27276", "image": [], "answer": "D", "solution": "null", "level": "四年级", "question": "小明在计算 $50 \\div(2+3)$ 时, 把算式写成 $50 \\div 2+3$, 这样两题的计算结果相差()。\n", "options": "A. 15\nB. 14\nC. 16\nD. 18\n", "subject": "算术", "analysis": "解$\\mathrm{D}$\n\n【分析】分别计算出两个算式的结果, 比较结果即可。据此解答。\n\n【详解】 $50 \\div(2+3)$\n\n$=50 \\div 5$\n\n$=10$\n\n$50 \\div 2+3$\n\n$=25+3$\n\n$=28$\n\n$28-10=18$, 选项 $\\mathrm{D}$ 符合。\n\n故答案为: $\\mathrm{D}$\n\n【点睛】本题考查学生对四则运算的掌握。在有小括号的算式中, 要先算括号里面的, 再算括号外面的。"} {"id": "13967", "image": [], "answer": "A", "solution": "null", "level": "四年级", "question": "下面算式中不能运用除法的性质进行简便计算的是( )。", "options": "A. $475 \\div 25 \\times 4$\nB. $6000 \\div 4 \\div 15$\nC. $10000 \\div 40 \\div 25$", "subject": "算术", "analysis": "答案:A $\\quad"} {"id": "13972", "image": [], "answer": "A", "solution": "null", "level": "四年级", "question": "同同在计算 $36 \\times 54$ 时, 不小心看成了 $63 \\times 54$, 她可以用得到的积再 ( )就能得到正确的得数。", "options": "A.减去 $54 \\times 27$\nB. 加上 $27 \\times 54$\nC.减去 $29 \\times 54$", "subject": "算术", "analysis": "答案: \\mathrm{A} \\quad"} {"id": "13973", "image": [], "answer": "C", "solution": "null", "level": "四年级", "question": "下面各组算式得数不相等的是 ( )。", "options": "A. $\\left\\{\\begin{array}{l}205 \\times 102-205 \\times \\\\ 205 \\times(102-2)\\end{array}\\right.$\nB. $\\left\\{\\begin{array}{l}4 \\times 32 \\times 25 \\\\ 32 \\times(4 \\times 25)\\end{array}\\right.$\nC. $\\left\\{\\begin{array}{l}78 \\times 100+1 \\\\ 78 \\times 101\\end{array}\\right.$", "subject": "算术", "analysis": "答案: \\mathrm{C} \\quad$"} {"id": "13993", "image": [], "answer": "D", "solution": "null", "level": "四年级", "question": "下面的小数中, 最接近 3 的是 ( )。", "options": "A.3.03B. 2.997C.3.1D.3.001", "subject": "算术", "analysis": ""} {"id": "13994", "image": [], "answer": "B", "solution": "null", "level": "四年级", "question": "出 $+o=10$, 则 $4.56 \\times$ 玄 $+4.56 \\times 0$ 的结果是 $(\\quad)$ 。", "options": "A.4.56B.45.6C. 456", "subject": "算术", "analysis": ""} {"id": "13995", "image": [], "answer": "C", "solution": "null", "level": "四年级", "question": "涛涛买了一盒 10 支装的铅笔, 又买了一本 18.5 元的笔记本, 付了 50 元, 找回 20.5 元。一支铅笔多少元? 解决这个问题要用到的信息是( $\\quad$ 。", "options": "A. 50 元, 20.5 元B. 18.5 元, 50 元, 20.5 元C. 10 支, 18.5 元, 50 元, 20.5 元", "subject": "算术", "analysis": ""} {"id": "13996", "image": [], "answer": "A", "solution": "null", "level": "四年级", "question": "已知 $1 \\square .5+8 . \\square 7$ 是一个小数加法算式, 算式的结果不可能是", "options": "A.35.57B. 18.57C. 27.67", "subject": "算术", "analysis": ""} {"id": "13997", "image": [], "answer": "B", "solution": "null", "level": "四年级", "question": "在一道减法算式中, 减数增加 5.6, 被减数增加 2.7 , 差( )。", "options": "A.增加 8.3B.减少 2.9C.增加 2.9", "subject": "算术", "analysis": ""} {"id": "13998", "image": ["2868.jpg", "2869.jpg", "2870.jpg"], "answer": "B", "solution": "null", "level": "四年级", "question": "一套书有 4 本 (如下图), 蓝蓝买了其中的 2 本, 递给收银员 30 元,蓝蓝可能买了哪两本 $?(\\quad)$ 。\n\n\n\n(1) 16.85 元\n\n![](本地图片-5241/.jpg)\n\n(2) 17.1 元\n\n\n\n(3) 12.99 元\n\n\n\n(4) 18.55 元", "options": "A.(1)(2)\nB. (1)(3)\nC. (1) (4)", "subject": "算术", "analysis": ""} {"id": "14010", "image": [], "answer": "B", "solution": "null", "level": "四年级", "question": "龙龙三次拍球练习的成绩从低到高排列是 92 个, $a$ 个, 96 个, 他的平均成绩可能是 $(\\quad)$ 个。", "options": "A. 91\nB. 94\nC. 97", "subject": "算术", "analysis": "答案: \\mathrm{B} \\quad"} {"id": "14012", "image": [], "answer": "B", "solution": "null", "level": "四年级", "question": "停车场停着共享汽车 (四轮) 和共享单车 (两轮)一共 10 辆, 总共有 32 个轮子, 共享单车有 $(\\quad)$ 辆。", "options": "A. 6\nB. 4\nC. 7\nD. 3", "subject": "算术", "analysis": "答案:B$"} {"id": "14015", "image": [], "answer": "C", "solution": "null", "level": "四年级", "question": "下面的问题能用 $22 \\times 15$ 解决的是 ( )。\n\n(1) 一碗热干面 15 元,卖 22 碗一共卖多少元\n\n(2) 围一个长 22 米, 宽 15 米的长方形框架要用多长的铁丝\n\n(3) 李叔叔每小时检测 22 个零件, 15 小时检测多少个零件", "options": "A.(1)(2)\nB. (2)(3)\nC.(1)(3)", "subject": "算术", "analysis": "C"} {"id": "14022", "image": [], "answer": "A", "solution": "null", "level": "四年级", "question": "服装店出售一款套装,每件上衣 275 元, \\$ \\qquad \\$ , 买 14 套这样的套装需要多少钱?如果列式解答为 $(275+185) \\times 14$, 那么横线上的信息应是( $\\quad$ )。", "options": "A. 每条裤子 185 元\nB.裤子的价格比上衣贵 185 元\nC.帽子的价格比上衣贵 185 元\nD.裤子的价格比上衣便宜 185 元", "subject": "算术", "analysis": "答案:\\mathrm{A} \\quad"} {"id": "14023", "image": [], "answer": "B", "solution": "null", "level": "四年级", "question": "荣老师和德老师买价格相同的中性笔, 荣老师买了 60 支, 德老师买了 78 支,选择下面条件( ) 就能求出一支中性笔多少钱。", "options": "A.荣老师还买了 36 元的橡皮\n\nB. 荣老师和德老师一共花了 276 元\n\nC.荣老师和德老师一共买了 138 支中性笔", "subject": "算术", "analysis": "答案: \\mathrm{B} \\quad"} {"id": "14028", "image": [], "answer": "B", "solution": "null", "level": "四年级", "question": "关于 $32 \\times 25$ 这道题目, 三名同学有不同的算法, ( )的算法是错误的。", "options": "A. 典典: $32 \\times 25=8 \\times(4 \\times 25)$\n\nB. 聪聪: $32 \\times 25=28+4 \\times 25$\n\nC. 梦梦: $32 \\times 25=32 \\times 100 \\div 4$", "subject": "算术", "analysis": "答案: \\mathrm{B} & \\text"} {"id": "14035", "image": [], "answer": "C", "solution": "null", "level": "四年级", "question": "同同在计算 $36 \\times 92+36 \\times \\mathbf{\\Lambda}-36 \\times 6$ 时, 利用乘法分配律计算的准确结果是 3600, 则 $\\mathbf{\\Delta}$ 代表的数是 $(\\quad)$ 。", "options": "A. 12\nB. 18\nC. 14\nD. 52", "subject": "算术", "analysis": "C"} {"id": "14037", "image": ["2889.jpg", "2890.jpg"], "answer": "D", "solution": "null", "level": "四年级", "question": "下面计算“ $1.42+2.15$ ”的方法中, ( )是合理的。\n(1)\n\n\n(2)\n\n\n(3) $1.4 \\quad 2$\n$\\begin{array}{r}1.42 \\\\ +2.15 \\\\ \\hline 3.57\\end{array}$", "options": "A. 只有 (3)\nB. 只有 (1)(3)\nC. 只有 (2) (3)\nD. (1)(2)(3)", "subject": "算术", "analysis": "答案:D & \\text"} {"id": "14038", "image": [], "answer": "B", "solution": "null", "level": "四年级", "question": "下列算式中, 去掉括号后不改变结果的是 ( )。", "options": "A. $(68+14) \\times(43-18)$\nB. $450+(180-4 \\times 35)$\nC. $(32 \\times 5-120) \\div 20$\nD. $720 \\div 20 \\times(15+73)$", "subject": "算术", "analysis": "B"} {"id": "14039", "image": [], "answer": "C", "solution": "null", "level": "四年级", "question": "下面说法正确的是 ( $\\quad )$ 。", "options": "A.小数点向右移动两位后, 这个数缩小到原来的 $\\frac{1}{100}$\n\nB. 7 吨 30 千克 -2 吨 8 千克 $=", "subject": "算术", "analysis": "答案:C \\end{array}$"} {"id": "14054", "image": [], "answer": "C", "solution": "null", "level": "四年级", "question": "华华读一本连环画, 第一天上午读了 21 页, 下午读了 16 页, 第二天读了 17 页, 正好读完, 华华平均每天读( )页。", "options": "A. 17\nB. 21\nC. 27", "subject": "算术", "analysis": "答案: \\mathrm{C} \\quad"} {"id": "14058", "image": [], "answer": "B", "solution": "null", "level": "四年级", "question": "在 0.7 和 0.8 之间的最大的两位小数是 $(\\quad)$ 。", "options": "A.", "subject": "算术", "analysis": "答案:\\mathrm{B} \\quad"} {"id": "14076", "image": [], "answer": "C", "solution": "null", "level": "四年级", "question": "下面各数中, 与“ 10 ”最接近的数是 ( )。", "options": "A.", "subject": "算术", "analysis": "答案:C $\\quad"} {"id": "14082", "image": [], "answer": "C", "solution": "null", "level": "四年级", "question": "与 $420 \\div 35$ 结果相等的式子是 $(\\quad)$ 。", "options": "A. $420 \\div 7 \\times 5$\nB. $420 \\div 30 \\div 5$\nC. $420 \\div 7 \\div 5$", "subject": "算术", "analysis": "答案: \\mathrm{C} \\quad"} {"id": "14083", "image": [], "answer": "B", "solution": "null", "level": "四年级", "question": "在 $72 \\times \\square+28 \\times 88$ 的方框里填上 ( ), 可以使算式简便计算。", "options": "A. 12\nB. 88\nC. 17", "subject": "算术", "analysis": "B"} {"id": "14084", "image": [], "answer": "B", "solution": "null", "level": "四年级", "question": "下面算式中正确的有 $(\\quad)$ 个。\n\n$106-29+71=106-(29+71) \\quad 45 \\times 18=45 \\times 2 \\times 9$\n\n$540 \\div 45=540 \\div 9 \\div 5$ $27 \\times 102-2=27 \\times 100$", "options": "A. 1\nB. 2\nC. 3\nD. 4", "subject": "算术", "analysis": "B"} {"id": "27280", "image": [], "answer": "B", "solution": "null", "level": "四年级", "question": "(2022 秋 $\\cdot$ 通州区期末) $O+10=\\square \\div 2=\\triangle \\times 5 ( O 、 \\square 、 \\triangle$ 都大于 0 ), 那么()表示的数最大。\n", "options": "A. $\\mathrm{O}$\nB.\nC. $\\triangle$\n", "subject": "算术", "analysis": " $O+10=\\square \\div 2=\\triangle \\times 5(O 、 \\square 、 \\triangle$ 都大于 0$)$, 那么 $\\square$ 表示的数最大。故选: $B$ 。"} {"id": "27291", "image": [], "answer": "A", "solution": "null", "level": "四年级", "question": "(2022 秋・东昌府区期末)用()可以求出除数。\n", "options": "A. 被除数 $\\div$ 商\nB. 被除数 - 商\nC. 被除数 $\\times$ 商\n", "subject": "算术", "analysis": "解【解答】解:用除数%商可以求出除数。\n\n故选: $A$ 。"} {"id": "27302", "image": [], "answer": "B", "solution": "null", "level": "四年级", "question": "(2022 秋 $\\cdot$ 唐河县期末)小强在计算除法时, 把除数 15 写成了 45 , 结果得到的商是 15 , 那么正确的商是 ( $\\quad$ )\n", "options": "A. 54\nB. 45\nC. 99\n", "subject": "算术", "analysis": "解【解答】解: $15 \\times 45 \\div 15$\n\n$=675 \\div 15$\n\n$=45$\n\n故选: $B$ 。"} {"id": "27304", "image": [], "answer": "C", "solution": "null", "level": "四年级", "question": "(2022 秋・灌云县期末)计算 $675-75 \\div 5 \\times 4$ 时, 先计算()\n", "options": "A. $675-75$\nB. $5 \\times 4$\nC. $75 \\div 5$\nD. $675 \\div 5$\n", "subject": "算术", "analysis": "解【解答】解: $675-75 \\div 5 \\times 4$\n\n$=675-15 \\times 4$\n\n$=675-60$\n\n$=615$\n\n是先算 $75 \\div 5$ 。\n\n故选: $C$ 。"} {"id": "27306", "image": [], "answer": "C", "solution": "null", "level": "四年级", "question": "(2022 春・坪山区期末) $24 \\times 3 \\div 24 \\times 3$ 的结果是 ( )\n", "options": "A. 1\nB. 6\nC. 9\nD. 576\n", "subject": "算术", "analysis": "解【解答】解: $24 \\times 3 \\div 24 \\times 3$\n\n$=72 \\div 24 \\times 3$\n\n$=3 \\times 3$\n\n$=9$\n\n故选: $C$ 。"} {"id": "27307", "image": [], "answer": "A", "solution": "null", "level": "四年级", "question": "(2022 秋 - 上蔡县期中)拉拉在计算一道减法题时, 不小心把减数 250 写成了 230 , 算出来的结果是 180 , 正确的结果是 ( )\n", "options": "A. 160\nB. 200\nC. 170\n\n", "subject": "算术", "analysis": "【解答】解: $180+230-250$\n\n$=410-250$\n\n$=160$\n\n答: 正确的结果是 160 。\n\n故选: $A$ 。"} {"id": "27308", "image": ["13305.jpg"], "answer": "B", "solution": "null", "level": "四年级", "question": "\n ", "options": "A. 相等\n B. 不相等\n C. 无法确定\n", "subject": "算术", "analysis": "解【解答】解: $9+3 \\times 7$\n\n$=9+21$\n\n$=30$\n\n$(9+3) \\times 7$\n\n$=12 \\times 7$\n\n$=84$\n\n结果不相等。\n\n故选: $B$ 。"} {"id": "27310", "image": [], "answer": "A", "solution": "null", "level": "四年级", "question": "4 人 5 天植树 360 棵,平均每人每天植树多少棵?列式是()。\n", "options": "A. $360 \\div 5 \\div 4$\nB. $360 \\div 4 \\times 5$\nC. $360 \\div(5 \\div 4)$\n", "subject": "算术", "analysis": "解A\n\n【分析】 4 人平均每天植树的棵数 $=$ 植树的总棵数 $\\div$ 天数, 平均每人每天植树的棵数 $=4$ 人平均每天植树的棵数 $\\div$ 数, 或者可以先求出 1 人 5 天植树的棵数, 再求出平均每人每天植树多少棵再或者可以带上括号, 先求出 $4 \\times 5=20$, 再用总棵数 $\\div 20$ 来解答。\n\n【详解】根据分析可知, 平均每人每天植树的棵数是: $360 \\div 5 \\div 4$ 或 $360 \\div 4 \\div 5$ 或 $360 \\div(5 \\times 4)$ 结合选项可得 $\\mathrm{A}$ 选项是正确的。\n\n故答案为: A\n\n【点睛】分析题目数量之间的关系, 根据数量之间的关系列出数量关系式, 根据数量关系式解决问题。"} {"id": "27321", "image": [], "answer": "C", "solution": "null", "level": "四年级", "question": "被减数、减数、差的和, 与被减数的关系是()。\n", "options": "A. 相等\nB. 无法确定\nC. 是被减数的 2 倍\n", "subject": "算术", "analysis": "解C\n\n【分析】被减数一减数=差, 减数十差=被减数, 依此进行选择即可。\n\n【详解】减数十差二被减数\n\n被减数十减数十差=被减数 $\\times 2$\n\n因此被减数、减数、差的和是被减数的 2 倍。\n\n故答案为: C\n\n【点睛】熟练掌握加、减法的意义和各部分之间的关系是解答此题的关键。"} {"id": "27332", "image": [], "answer": "C", "solution": "null", "level": "四年级", "question": "下列算式中,应先算加法的是()。\n", "options": "A. $52+48 \\times 4$\nB. $187-(13+25 \\div 5)$\nC. $11 \\times(14+9)-63$\n", "subject": "算术", "analysis": "解C\n\n【分析】根据整数的四则混合运算法则, 先算乘除, 再算加减, 有括号的先算括号里面的, 逐项判断即可。\n\n【详解】A. 应先算乘法, 再算加法;\n\nB. 应先算括号里面的除法, 再算括号里面的加法, 最后算减法;\n\nC. 应先算括号里面的加法, 再算乘法, 最后算加法。\n\n故答案为: $\\mathrm{C}$\n\n【点睛】本题解题的关键是掌握整数的四则混合运算法则。"} {"id": "27333", "image": [], "answer": "B", "solution": "null", "level": "四年级", "question": "如果 $\\square \\times \\circ=\\triangle$, 那么下面的算式中, 错误的是()。\n", "options": "A. $\\triangle \\div \\circ=\\square$\nB. $\\circ \\div \\square=\\triangle$\nC. $\\triangle \\div \\square=0$\n", "subject": "算术", "analysis": "解$\\mathrm{B}$\n\n【分析】因数 $\\times$ 因数 $=$ 积, 积 $\\div$ 因数 $=$ 另一个因数, 据此解答。\n\n【详解】A. $\\triangle \\div \\circ=\\square$, 算式正确;\n\nB. $0 \\times \\square=\\triangle$, 而不是 $0 \\div \\square=\\triangle$, 算式错误;\n\nC. $\\triangle \\div \\square=\\circ$, 算式正确;\n\n故答案为: B\n\n【点睛】本题考查乘法各部分之间的关系, 需熟练掌握。"} {"id": "27334", "image": [], "answer": "C", "solution": "null", "level": "四年级", "question": "一个数除以 65 , 商是最大的两位数, 余数是 7, 这个数是()。\n", "options": "A. 82\nB. 657\nC. 6442\n", "subject": "算术", "analysis": "解$\\mathrm{C}$\n\n【分析】最大的两位数是 99 , 由题意可知: 商是 99 , 求被除数, 根据被除数 $=$ 商 ×除数 + 余数, 解答即可。\n\n【详解】 $99 \\times 65+7$\n\n$=6435+7$\n\n$=6442$\n\n这个数是 6442 。\n\n故答案为: $\\mathrm{C}$\n\n【点睛】明确被除数与除数、商和余数的关系是解决本题的关键。"} {"id": "27335", "image": [], "answer": "B", "solution": "null", "level": "四年级", "question": "把 $91 \\div 7=13,85-13=72,72 \\times 4=288$ 列成一个综合算式()。\n", "options": "A. $85-91 \\div 7 \\times 4=288$\nB. $(85-91 \\div 7) \\times 4=288$\nC. $85-(91 \\div 7) \\times 4=288$\n", "subject": "算术", "analysis": "解$\\mathrm{B}$\n\n【分析】根据题意可知, 此题是先算除法, 再算减法, 最后算乘法, 根据混合运算的计算顺序列出综合算式并选择即可。\n\n【详解】 $(85-91 \\div 7) \\times 4$\n\n$=(85-13) \\times 4$\n\n$=72 \\times 4$\n\n$=288$\n\n即把 $91 \\div 7=13,85-13=72,72 \\times 4=288$ 列成一个综合算式是: $\\quad(85-91 \\div 7 ) \\times 4=288$ 。\n\n故答案为: B\n\n【点睛】熟练掌握混合运算的计算顺序是解答此题的关键。"} {"id": "27336", "image": [], "answer": "C", "solution": "null", "level": "四年级", "question": "给算式 $15+900 \\div 15-12$ 加上括号, 使运算顺序变为减 $\\rightarrow$ 除 $\\rightarrow$ 加, 正确的算式是()。\n", "options": "A. $15+(900 \\div 15-12)$\nB. $(15+900) \\div 15-12$\nC. $15+900 \\div(15-12)$\n", "subject": "算术", "analysis": "解$\\mathrm{C}$\n\n【分析】 $15+900 \\div 15-12$ 是先算除法, 再算加法, 最后算减法, 变成先算减法, 再算除法, 最后算加法, 是把减法提前到除法的前面,所以要给减法加上小括号,由此求解。\n\n【详解】 $15+900 \\div(15-12)$\n\n$=15+900 \\div 3$\n\n$=15+300$\n\n$=315$\n\n给算式 $15+900 \\div 15-12$ 加上括号, 使运算顺序变为减 $\\rightarrow$ 除 $\\rightarrow$ 加, 正确的算式是 $15+900 \\div(15-12)$ 。故答案为:\n\n【点睛】解决本题关键是找清楚计算顺序的变化,合理利用小括号进行求解。"} {"id": "27337", "image": [], "answer": "B", "solution": "null", "level": "四年级", "question": "星星小学 2 名老师和 70 名学生租车去科技馆参观, 现有大、小两种车型可以选择:\n\n大车: 限乘 30 人, 租金 600 元。\n\n小车: 限乘 15 人, 租金 400 元。\n\n租 $\\qquad$辆大车和 $\\qquad$辆小车最省钱。\n", "options": "A. 1,3\nB. 2,1\nC. 3, 0\n", "subject": "算术", "analysis": "解B\n\n【分析】先根据“单价 $=$ 总价 $\\div$ 数量”分别计算出租大车和小车时每人需要的钱, 要使租车最省钱, 则\n应尽量租最便宜的一种车型, 并且尽量使每辆车都坐满; 因此用总人数除以最便宜的一种车型可坐的人数, 再根据计算出的结果进行解答即可。\n\n【详解】大车: $600 \\div 30=20$ (元/人)\n\n小车: $400 \\div 15=26$ (元/人).....10(元)\n\n$400 \\div 15>600 \\div 30$, 所以尽量租大车。\n\n$2+70=72$ (人)\n\n$72 \\div 30=2$ (辆) $\\ldots . .12$ (人)\n\n1 辆小车限乘 15 人, 而剩下的 12 人可以租 1 辆小车\n\n所以租 2 辆大车和 1 辆小车最省钱。\n\n故答案为: B\n\n【点睛】租车优化问题首先要使便宜的车满座, 如果剩余的人数比较较多又接近满座, 可以考虑剩下的人再租用同一种车, 如果剩余的人数比较少可以通过调整租用其它载人少的车。"} {"id": "27339", "image": [], "answer": "B", "solution": "null", "level": "四年级", "question": "已知 $\\triangle-\\triangle=\\bullet$, 下面的算式 ( ) 是正确的。\n", "options": "A. $\\triangle-\\Delta=\\bullet$\nB. $\\square+\\bullet=\\boldsymbol{\\Lambda}$\nC. $\\bullet+\\Delta=\\square$\nD. $\\bullet-\\boldsymbol{\\Delta}=\\square$\n", "subject": "算术", "analysis": "解$\\mathrm{B}$\n\n【分析】被减数一减数=差, 被减数=差+减数; 减数=被减数一差, 依此选择。\n\n【详解】已知 $\\triangle-\\triangle=\\bullet$, 则:\n\n故答案为: $\\mathrm{B}$\n\n【点睛】熟练掌握加、减法的意义和各部分之间的关系是解答此题的关键。"} {"id": "27350", "image": [], "answer": "C", "solution": "null", "level": "四年级", "question": "50 减去 25 的差, 乘 20 加 13 的和, 正确的算式是 ( )。\n", "options": "A. $50-25 \\times 20+13$\nB. $(50-25) \\times 20+13$\nC. $(50-25) \\times(20+13)$\nD. $50-25 \\times(20+13)$\n", "subject": "算术", "analysis": "解C\n\n【分析】先分别求出 50 减去 25 的差与 20 加 13 的和, 再把求出的差乘求出的和, 即可解答。\n\n【详解】 $(50-25) \\times(20+13)$\n\n$=25 \\times 33$\n\n$=825$\n\n所以 50 减去 25 的差, 乘 20 加 13 的和, 正确的算式是(50-25)×(20+13)。\n\n故答案为: C\n\n【点睛】列式计算注意语言叙述的运算顺序,正确选择运算符号和括号列式计算即可。"} {"id": "27360", "image": [], "answer": "D", "solution": "null", "level": "四年级", "question": "计算 $700-150 \\div 50 \\times 3$ 时, 计算顺序是 ( )。\n", "options": "A. 先算乘法, 再算减法, 最后算除法\nB. 先算乘法, 再算除法, 最后算减法\nC. 先算除法, 再算减法, 最后算乘法\nD. 先算除法, 再算乘法, 最后算减法\n", "subject": "算术", "analysis": "解D\n\n【分析】没有括号的混合运算的计算顺序是:先算乘、除法,再算加、减法,依此选择即可。\n\n【详解】计算 $700-150 \\div 50 \\times 3$ 时, 应先算除法, 再算乘法, 最后算减法。\n\n故答案为: D\n\n【点睛】熟练掌握混合运算的计算顺序是解答此题的关键。"} {"id": "27361", "image": [], "answer": "A", "solution": "null", "level": "四年级", "question": "如果 $\\mathrm{A} \\times \\mathrm{B}=\\mathrm{C}$ (其中 $\\mathrm{A} \\neq \\mathrm{B} \\neq \\mathrm{C}$ ), 那么()。\n", "options": "A. $\\mathrm{C} \\div \\mathrm{A}=\\mathrm{B}$\nB. $\\mathrm{A} \\div \\mathrm{C}=\\mathrm{B}$\nC. $\\mathrm{B} \\div \\mathrm{C}=\\mathrm{A}$\nD. $\\mathrm{C} \\times \\mathrm{B}=\\mathrm{A}$\n", "subject": "算术", "analysis": "解A\n\n【分析】根据积除以一个因数等于另一个因数, 解答即可。\n\n【详解】如果 $\\mathrm{A} \\times \\mathrm{B}=\\mathrm{C}$ (其中 $\\mathrm{A} \\neq \\mathrm{B} \\neq \\mathrm{C}$ ), 那么 $\\mathrm{C} \\div \\mathrm{A}=\\mathrm{B}, \\mathrm{C} \\div \\mathrm{B}=\\mathrm{A}$ 。\n\n故答案为: A\n\n【点睛】本题考查了乘法的意义和各部分之间的关系。"} {"id": "27362", "image": [], "answer": "B", "solution": "null", "level": "四年级", "question": "张老师用 400 元钱购买了 18 本《少儿科幻》, 还剩 40 元。小明列式为“ $40 \\div[(400-40) \\div 18]$ ”,他要解决的问题是 ( )。\n\n", "options": "A. 《少儿科幻》每本多少钱\n\nB. 剩下的钱还能买几本《少儿科幻》\n\nC. 买 18 本《少儿科幻》用了多少钱\n\nD. 400 元钱一共能买几本《少儿科幻》\n\n", "subject": "算术", "analysis": "解$\\mathrm{B}$\n\n【分析】算式“40[ (400-40) $\\div 18]$ ”, 是先算小括号里面的的减法, 再算中括号里面的的除法, 最后算括号外的除法, 第一步小括号里面 400-40 是用总钱数减去剩下的钱数, 表示 18 本《少儿科幻》花了多少钱, 第二步 18 本的总价除以 18 , 表示每一本的价格是多少元, 最后一步, 用剩下的 40 元除以每一本的价格, 就表示剩下的钱数还能买几本《少儿科幻》, 由此求解。\n\n【详解】根据分析可知:张老师用 400 元钱购买了 18 本《少儿科幻》, 还剩 40 元。小明列式为“ $40 \\div$ [(400 - 40)18]”, 他要解决的问题是剩下的钱还能买几本《少儿科幻》。\n故答案为: B\n\n【点睛】解决本题关键是理解题目中的数据表示的含义, 根据计算顺序分析每一步算式表示的含义,从而解决问题。"} {"id": "27363", "image": [], "answer": "C", "solution": "null", "level": "四年级", "question": "要把 $725-475 \\div 25 \\times 4$ 的运算顺序改变为: 先算除法, 再算减法, 最后算乘法。则原式变为 ( )。\n", "options": "A. $725-(475 \\div 25 \\times 4)$\nB. $(725-475) \\div 25 \\times 4$\nC. $(725-475 \\div 25) \\times 4$\nD. $725-(475 \\div 25) \\times 4$\n", "subject": "算术", "analysis": "解C\n\n【分析】把 $725-475 \\div 25 \\times 4$ 的运算顺序是先算除法, 再算乘法, 最后算减法, 要想改变为: 先算除法,再算减法, 把 $725-475 \\div 25$ 用括号括起来, 据此解答。\n\n【详解】把 $725-475 \\div 25 \\times 4$ 的运算顺序改变为: 先算除法, 再算减法, 最后算乘法, 改变后的算式是: $(725-475 \\div 25) \\times 4$ 。\n\n故答案为: $\\mathrm{C}$\n\n【点睛】考查了小括号的作用, 能改变算式的运算顺序。"} {"id": "27364", "image": [], "answer": "A", "solution": "null", "level": "四年级", "question": "用一个杯子向空水企里倒水, 如果倒进 2 杯水, 连企重 520 克; 如果倒进 5 杯水, 连壸重 700 克;每杯水重()克。\n", "options": "A. 60\nB. 140\nC. 180\nD. 260\n", "subject": "算术", "analysis": "解$\\mathrm{A}$\n\n【分析】用倒进 5 杯水后的重量减去倒进 2 杯水后的重量, 就是 3 杯水的重量, 然后再除以 3 , 即可求出每杯水的重量。\n\n【详解】 $(700-520) \\div 3$\n\n$=180 \\div 3$\n\n$=60$ (克)\n\n故答案为: $\\mathrm{A}$\n\n【点睛】解答此题的关键是明确倒进 5 杯水后的重量减去倒进 2 杯水后的重量, 是 3 杯水的重量。"} {"id": "27365", "image": [], "answer": "D", "solution": "null", "level": "四年级", "question": "工厂加工一批零件, 前 3 天每天加工 120 个, 后 5 天一共加工 760 个。根据以上信息, 不能解决的问题是 ( ) 。\n", "options": "A. 8 天一共加工多少个零件?\nB. 后 5 天平均每天加工多少个零件?\nC. 这 8 天平均每天加工多少个零件?\nD. 这批零件还剩多少个没有加工?\n", "subject": "算术", "analysis": "解$\\mathrm{D}$\n\n【分析】前 3 天每天加工的个数十后 5 天一共加工的个数 $=8$ 天一共加工零件的个数; 后 5 天一共加工零件的个数 $\\div 5=$ 后 5 天平均每天加工零件的个数; 8 天一共加工零件的个数 $\\div 8=$ 这 8 天平均每天加工零件的个数, 依此选择。\n\n【详解】A. 这 8 天一共加工零件的个数可列式为: $120+760=880$ (个);\n\nB. 后 5 天平均每天加工零件的个数可列式为: $760 \\div 5=152$ (个);\n\nC. 这 8 天平均每天加工零件的个数可列式为: $(120+760) \\div(5+3)=880 \\div 8=110$ (个);\n\nD. 这批零件还剩多少个没有加工?由于不知道需要加工零件的总个数, 因此剩下没有加工的零件个数无法计算。\n\n故答案为: D\n\n【点睛】此题考查的是工程问题的计算,先根据题意找到对应的关系是解答此题的关键。"} {"id": "27367", "image": [], "answer": "A", "solution": "null", "level": "四年级", "question": "如果 $\\mathrm{A} \\times \\mathrm{B}=\\mathrm{C}$ (其中 $\\mathrm{A}, \\mathrm{B}, \\mathrm{C}$ 都不相等,且都不等于 0 ), 那么()。\n", "options": "A. $\\mathrm{C} \\div \\mathrm{A}=\\mathrm{B}$\nB. $\\mathrm{C} \\times \\mathrm{B}=\\mathrm{A}$\nC. $\\mathrm{A} \\div \\mathrm{C}=\\mathrm{B}$\nD. $\\mathrm{B} \\div \\mathrm{C}=\\mathrm{A}$\n", "subject": "算术", "analysis": "A"} {"id": "27386", "image": [], "answer": "D", "solution": "null", "level": "四年级", "question": "小明在计算 $50 \\div(2+3)$ 时, 把算式写成 $50 \\div 2+3$, 这样两题的计算结果相差()。\n", "options": "A. 15\nB. 14\nC. 16\nD. 18\n", "subject": "算术", "analysis": "解$\\mathrm{D}$\n\n【分析】分别计算出两个算式的结果, 比较结果即可。据此解答。\n\n【详解】 $50 \\div(2+3)$\n\n$=50 \\div 5$\n\n$=10$\n\n$50 \\div 2+3$\n\n$=25+3$\n\n$=28$\n\n$28-10=18$, 选项 $\\mathrm{D}$ 符合。\n\n故答案为: $\\mathrm{D}$\n\n【点睛】本题考查学生对四则运算的掌握。在有小括号的算式中, 要先算括号里面的, 再算括号外面的。"} {"id": "27387", "image": [], "answer": "B", "solution": "null", "level": "四年级", "question": "把 $91 \\div 13=7,65-7=58,58 \\times 5=290$ 列成一个综合算式是()。\n", "options": "A. $65 \\times 5-91 \\div 13$\nB. $(65-91 \\div 13) \\times 5$\nC. $65-91 \\div 13 \\times 5$\nD. $(91-65) \\div 13 \\times 5$\n", "subject": "算术", "analysis": "解 $\\mathrm{B}$"} {"id": "27388", "image": [], "answer": "B", "solution": "null", "level": "四年级", "question": "张老师用 400 元钱购买了 18 本《少儿科幻》,还剩 40 元。小明列式为“40 $\\div[(400-40) \\div 18]$,他要解决的问题是()。\n\n", "options": "A. 《少儿科幻》每本多少钱\n\nB. 剩下的钱还能买几本《少儿科幻》\n\nC. 买 18 本《少儿科幻》用了多少钱\nD. 400 元钱一共能头几本《少儿科幻》\n\n", "subject": "算术", "analysis": "解B"} {"id": "27389", "image": [], "answer": "D", "solution": "null", "level": "四年级", "question": "下面算式中, 去掉括号后结果不变的是( )。\n", "options": "A. $(72+18) \\times 54$\nB. $512-(38+12)$\nC. $(702-12) \\times 5$\nD. $63+(45 \\times 6)$\n", "subject": "算术", "analysis": "解D"} {"id": "27390", "image": [], "answer": "D", "solution": "null", "level": "四年级", "question": "下列说法不正确的是()。\n", "options": "A. 一个数和 0 相乘仍是 0 。\nB. 被减数等于减数, 差是 0 。\nC. 除法是乘法的逆运算。\nD. 因为 $0 \\div 7=0$, 所以 $7 \\div 0=0$\n", "subject": "算术", "analysis": "解D"} {"id": "27391", "image": [], "answer": "D", "solution": "null", "level": "四年级", "question": "48 与 120 的和除以 12 与 4 的差,商是多少,下列算式正确的是()\n", "options": "A. $48+120 \\div 12-4$\nB. $(48+120) \\div 12-4$\nC. $48+120 \\div(12-4)$\nD. $(48+120) \\div(12-4)$\n", "subject": "算术", "analysis": "解D"} {"id": "27393", "image": [], "answer": "C", "solution": "null", "level": "四年级", "question": "张叔叔带了 80 元, 买了单价 18 元的葡萄 4 千克。下面四个问题中适合用估算是()。\n\n", "options": "A. 老板确认应该收多少钱\n\nB. 老板计算要找回多少钱\n\nC. 张叔叔思考 80 元到底够不够\n\n", "subject": "算术", "analysis": "解C\n\n【详解】在实际生活中, 老板应该收的钱数以及老板要找回的钱数都需要计算出准确值, 而张叔叔想要知道自己带的钱数够不够可以采用估算的方法, 据此解答。\n\n【解答】A. $18 \\times 4=72$ (元)\n\n所以,老板应该收 72 元,不可以估算;\n\nB. $80-18 \\times 4$\n\n$=80-72$\n\n$=8$ (元)\n\n所以, 老板要找回 8 元, 不可以估算;\n\nC. 18 元 $\\approx 20$ 元\n\n$20 \\times 4=80 ($ 元)\n\n80 元 $=80$ 元\n\n所以, 张叔叔 80 元钱够买 4 千克葡萄, 可以估算。\n\n故答案为: C\n\n【点睛】本题主要考查估算在实际生活中的应用, 生活中有些情况不需要计算准确值, 只要计算出比较接近的数即可。"} {"id": "27404", "image": [], "answer": "C", "solution": "null", "level": "四年级", "question": "如果 $\\mathrm{A}=5$, 下面算式不成立的是 ( )。\n", "options": "A. $0+\\mathrm{A}=\\mathrm{A}$\nB. $0 \\times \\mathrm{A}=0$\nC. $\\mathrm{A} \\div 0=0$\n", "subject": "算术", "analysis": "解C\n\n【分析】 0 和任何数相加还得这个数; 0 乘任何数还得 0 ; 除数不能为 0 , 任何数减 0 还得这个数, 依此选择。\n\n【详解】A. $0+5=5$, 即 $0+\\mathrm{A}=\\mathrm{A}$ 成立, 该选项正确;\n\nB. $0 \\times 5=0$, 即 $0 \\times \\mathrm{A}=0$ 成立, 该选项正确;\n\nC. 除数不能为 0 , 即 $\\mathrm{A} \\div 0=0$ 不成立, 该选项错误。\n\n如果 $\\mathrm{A}=5$, 下面算式不成立的是 $(\\mathrm{A} \\div 0=0)$ 。\n\n故答案为: $\\mathrm{C}$\n\n【点睛】熟练掌握有关 0 的计算是解答此题的关键。"} {"id": "27413", "image": [], "answer": "B", "solution": "null", "level": "四年级", "question": "把 $46+36=82,82 \\div 2=41$ 改写成一个综合算式 ( )。\n", "options": "A. $46+36 \\div 2=41$\nB. $(46+32) \\div 2=41$\nC. $46+(36 \\div 2)=41$\n", "subject": "算术", "analysis": "解$\\mathrm{B}$\n\n【分析】根据题意分析, 先计算括号里面的 $46+32$ 的和, 再除以 2 , 即可求解。\n\n【详解】 $46+36=82,82 \\div 2=41$ 改写成一个综合算式为 $(46+32) \\div 2=41$ 。\n\n故答案为: B\n\n【点睛】本题考查含有括号的混合运算。"} {"id": "27414", "image": ["13311.jpg"], "answer": "B", "solution": "null", "level": "四年级", "question": "根据紟 $\\times \\triangle=$ (三个数不为零), 下面三个算式中正确的是()。\n", "options": "A. $\\triangle \\div \\hbar=0$\nB. $0 \\div i \\xi=\\triangle$\nC. $0 \\times \\triangle=$ 邻\n", "subject": "算术", "analysis": "解$\\mathrm{B}$\n\n【分析】根据题目所给算式,因数 $\\times$ 因数 $=$ 积,积 $\\div$ 因数 $=$ 另一个因数,即可求解。\n\n\n\n故答案为: B\n\n【点睛】本题考查的知识点是乘法各部分间的关系。"} {"id": "27415", "image": ["13312.jpg", "13313.jpg"], "answer": "C", "solution": "null", "level": "四年级", "question": "根据如图列综合算式, 正确的是 ( )。\n\n\n", "options": "A. $920+438 \\div 73 \\times 34$\nB. $(920+438) \\div 73 \\times 34$\nC. $(920+438 \\div 73) \\times 34$\n\n", "subject": "算术", "analysis": "解C\n\n【分析】先用 438 除以 73 求出商, 再用 920 加上求出的商得到和, 然后用得到的和乘上 34 求出积,即可解答。\n\n【详解】根据分析可知,\n\n\n\n综合算式是: $\\quad(920+438 \\div 73 ) \\times 34$ 。\n\n故答案为: $\\mathrm{C}$\n\n【点睛】列综合算式, 关键是弄清运算顺序。"} {"id": "27416", "image": [], "answer": "B", "solution": "null", "level": "四年级", "question": "艺术宫举办“建党 100 周年”儿童作品展, 上午 $9 : 00 \\sim 10: 30$ 有 105 人参观, 10: $30 \\sim 12: 00$ 之间参观的人数是上一时段的 2 倍, 下午参观的总人数有 279 人。算式: $105+105 \\times 2+279$ 解决的问题是 ( ) 。\n", "options": "A. 上午参观的有多少人?\nB. 全天参观的有多少人?\nC. 9: $00 \\sim 10: 30$ 参观的有多少人?\n", "subject": "算术", "analysis": "解$\\mathrm{B}$\n\n【分析】算式: $105+105 \\times 2+279$ 中,105 是上午 9: $00 \\sim 10: 30$ 的参观人数, $105 \\times 2$ 是 10: $30 \\sim 12$ : 00 的参观人数, $105+105 \\times 2$ 就等于上午参观的总人数, 279 是下午参观的总人数, 所以 $105+105 \\times 2$ +279 就等于全天参观的总人数, 据此即可解答。\n\n【详解】根据分析可知, $105+105 \\times 2+279$ 等于全天参观的总人数。\n\n故答案为: B\n\n【点睛】本题主要考查学生的综合分析能力, 算式计算出来的结果是什么, 解决的问题就是什么。"} {"id": "27417", "image": [], "answer": "A", "solution": "null", "level": "四年级", "question": "$[470+(100-74)] \\times 28$ 的运算顺序是 ( )。\n", "options": "A. 减法 $\\rightarrow$ 加法 $\\rightarrow$ 乘法\nB. 加法 $\\rightarrow$ 减法 $\\rightarrow$ 乘法\nC. 乘法 $\\rightarrow$ 减法 $\\rightarrow$ 加法\n", "subject": "算术", "analysis": "解$\\mathrm{A}$\n\n【分析】四则混合运算的顺序:在一个没有括号的算式里,如果只含同一级运算,按照从左往右的顺序依次计算; 如果含有两级运算, 要先算乘除法, 再算加减法; 在一个有括号的算式里, 要先算小括号里的, 再算中括号里的, 最后算括号外的。据此可知, $[470+(100-74)] \\times 28$ 的运算顺序是先算小括号里面的减法,再算中括号里面的加法,最后计算中括号外面的乘法。\n\n【详解】根据分析可知,\n\n$[470+( 100-74 )] \\times 28$ 的运算顺序是先算小括号里面的减法, 再算中括号里面的加法, 最后计算中括号外面的乘法。\n故答案为: $\\mathrm{A}$\n\n【点睛】熟练掌握整数四则混合运算的计算方法, 是解答此题的关键。"} {"id": "27418", "image": [], "answer": "C", "solution": "null", "level": "四年级", "question": "小明在计算 $(30+25) \\times 2$ 时, 错算成了 $30+25 \\times 2$, 这样与正确结果相差 ( )。\n", "options": "A. 110\nB. 80\nC. 30\n", "subject": "算术", "analysis": "解$\\mathrm{C}$\n\n【分析】分别计算出两个算式的结果, 然后相减即可解答。\n\n【详解】 $(30+25) \\times 2$\n\n$=55 \\times 2$\n\n$=110$\n\n$30+25 \\times 2$\n\n$=30+50$\n\n$=80$\n\n$110-80=30$\n\n故答案为: $\\mathrm{C}$\n\n【点睛】熟练掌握整数混合运算的运算法则是解答本题的关键。"} {"id": "27492", "image": [], "answer": "C", "solution": "null", "level": "四年级", "question": " 下面和 $(18+\\square ) \\times 5$ 的计算结果相等的算式是()。\n", "options": "A. $(18+5) \\times_{\\square}$\nB. $18+\\square \\times 5$\nC. $18 \\times 5+\\square \\times 5$\n", "subject": "算术", "analysis": "解$\\mathrm{C}$\n\n【分析】乘法分配律是指两个数的和同一个数相乘, 等于把两个加数分别同这个数相乘, 再把两个积加起来, 结果不变, 由此进行解答即可。\n\n【详解】根据乘法分配律可得: $(18+\\square ) \\times 5=18 \\times 5+\\square \\times 5$;\n\n所以和 $(18+\\square ) \\times 5$ 的计算结果相等的算式是: $18 \\times 5+\\square \\times 5$ 。\n\n故答案为: $\\mathrm{C}$\n\n【点睛】熟练掌握乘法分配律的特点是解答此题的关键。"} {"id": "27503", "image": [], "answer": "B", "solution": "null", "level": "四年级", "question": "下列算式中,与“ $98 \\times 99 ”$ 的计算结果不相同的是()。\n", "options": "A. $98 \\times 100-98$\nB. $99 \\times 100-99$\nC. $99 \\times 90+99 \\times 8$\n", "subject": "算术", "analysis": "解$\\mathrm{B}$\n\n【分析】乘法分配律是指两个数的和与一个数相乘, 可以先把它们分别与这个数相乘, 再相加; 或两个数的差与一个数相乘, 可以先把它们分别与这个数相乘, 再相减; $98 \\times 99=98 \\times 100-98,98 \\times 99=$ $99 \\times 90+99 \\times 8$; 据此解答。\n\n【详解】根据分析:\n\nA. $98 \\times 100-98$\n\n$=98 \\times(100-1)$\n\n$=98 \\times 99$\n\nB. $99 \\times 100-99$\n\n$=99 \\times(100-1)$\n\n$=99 \\times 99$\n\nC. $99 \\times 90+99 \\times 8$\n\n$=99 \\times(90+8)$\n\n$=99 \\times 98$\n\n故答案为: $\\mathrm{B}$\n\n【点睛】正确理解乘法分配律的意义, 是解答此题的关键。"} {"id": "27511", "image": [], "answer": "B", "solution": "null", "level": "四年级", "question": "一正药房采购 48 箱 N95 型口罩, 每箱价格 205 元, 一共需要付多少元?王明的算式是“ $48 \\times 205$ ”,他想采用乘法分配律计算,下面正确的算式是()。\n", "options": "A. $200 \\times 40+5 \\times 8$\nB. $200 \\times 48+5 \\times 48$\nC. $205 \\times 6 \\times 8$\n", "subject": "算术", "analysis": "解$\\mathrm{B}$\n\n【分析】用乘法分配律计算 $48 \\times 205$ 时, 可将 205 写成 $200+5$, 然后再根据乘法分配律的特点进行计算, 依此选择。\n\n【详解】 $48 \\times 205$\n\n$=48 \\times(200+5)$\n\n$=48 \\times 200+48 \\times 5$\n\n$=9600+240$\n$=9840 ($ 元)\n\n算式正确的是 $200 \\times 48+5 \\times 48$ 。\n\n故答案为: B\n\n【点睛】熟练掌握乘法分配律的特点是解答此题的关键。"} {"id": "27512", "image": [], "answer": "A", "solution": "null", "level": "四年级", "question": "下面可以用乘法分配律进行简便计算的算式是()。\n", "options": "A. $122 \\times 8+78 \\times 8$\nB. $52 \\times 25 \\times 4$\nC. $76 \\times 8+16 \\times 25$\n", "subject": "算术", "analysis": "解A\n\n【分析】能利用乘法分配律简算的算式应满足 3 个条件: (1)算式包含两级运算,即乘加或乘减。\n\n(2)有一个因数是相同的。(3)另外两个因数可凑成整十、整百、整千。\n\n【详解】A. $122 \\times 8+78 \\times 8$\n\n$=(122+78) \\times 8$\n\n$=200 \\times 8$\n\n$=1600$\n\n利用乘法分配律简算。\n\nB. $52 \\times 25 \\times 4$\n\n$=52 \\times(25 \\times 4)$\n\n$=52 \\times 100$\n\n$=5200$\n\n利用乘法结合律简算。\n\nC. $76 \\times 8+16 \\times 25$\n\n$=608+400$\n\n$=1008$\n\n先算乘法,再算加法。\n\n故答案为: $\\mathrm{A}$\n\n【点睛】本题考查学生对乘法分配律的认识和应用。"} {"id": "27513", "image": [], "answer": "B", "solution": "null", "level": "四年级", "question": "计算 $124 \\times 5 \\times 20 \\times 37=(124 \\times 5) \\times 20 \\times 37$, 运用了 ( )。\n", "options": "A. 乘法交换律\nB. 乘法结合律\nC. 乘法分配律\n", "subject": "算术", "analysis": "解B\n\n【分析】乘法交换律的特点是两个数相乘,交换两个因数的位置,积不变。\n\n乘法结合律的特点是三个数相乘, 先把前两个数相乘, 或先把后两个数相乘, 积不变。乘法分配律的特点是两个数的和与一个数相乘, 可以先把它们与这个数分别相乘, 再相加。依此选择。\n\n【详解】根据分析可知, 计算 $124 \\times 5 \\times 20 \\times 37=( 124 \\times 5 ) \\times 20 \\times 37$, 运用了乘法结合律。故答案为: B\n\n【点睛】熟练掌握乘法交换律、乘法结合律、乘法分配律的特点, 是解答此题的关键。"} {"id": "27514", "image": [], "answer": "B", "solution": "null", "level": "四年级", "question": "下面的算式, 计算结果与“ $672-36+64$ ”相同的是 ( )。\n", "options": "A. $672-(36+64)$\nB. $672+64-36$\nC. $672-(64-36)$\n", "subject": "算术", "analysis": "解$\\mathrm{B}$\n【分析】加减同级运算从左往右算起, 有小括号先算小括号里的数。\n\n【详解】 $672-36+64$\n\n$=636+64$\n\n$=700$\n\nA. $672-(36+64)$\n\n$=672-100$\n\n$=572$\n\n$572<700$\n\nB. $672+64-36$\n\n$=736-36$\n\n$=700$\n\n$700=700$\n\nC. $672-(64-36)$\n\n$=672-28$\n\n$=644$\n\n$644<700$\n\n故答案为: B\n\n【点睛】要注意加减同级运算交换位置, 要带数前面的符号一起交换。"} {"id": "27515", "image": [], "answer": "C", "solution": "null", "level": "四年级", "question": "计算器上数字键“ 3 ”坏了, 小明要计算 201636, 可以用下面算式 ( )来代替。\n", "options": "A. $2016 \\div 20 \\div 16$\nB. $2016 \\div 18 \\div 18$\nC. $2016 \\div 2 \\div 18$\n", "subject": "算术", "analysis": "解C\n\n【分析】根据题意可知, 36 可以写成 $6 \\times 6$, 或将 36 写成 $2 \\times 18$, 这样就不会使用数字键 “ 3 ”, 因此可根据整数除法的性质进行简算, 依此选择。\n\n【详解】 $2016 \\div 36=2016 \\div(6 \\times 6)=2016 \\div 6 \\div 6$ 。\n\n$2016 \\div 36=2016 \\div(2 \\times 18)=2016 \\div 2 \\div 18$ 。\n\n故答案为: $\\mathrm{C}$\n\n【点睛】熟练掌握整数除法的性质是解答此题的关键。"} {"id": "27516", "image": [], "answer": "C", "solution": "null", "level": "四年级", "question": "在列坚式计算 $403 \\times 21$ 时, 我们是这样计算的(如下), 这个过程利用的是()。\n\n| 403 |\n| ---: |\n| $\\times \\quad 21$ |\n| 403 |\n| 806 |\n| 8463 |\n\n", "options": "A. 乘法交换律\nB. 乘法结合律\nC. 乘法分配律\n", "subject": "算术", "analysis": "解C\n\n【分析】三位数乘两位数的计算方法: 先是用两位数的个位上的数与三位数相乘; 接着用两位数的十位上的数与三位数相乘, 最后把两次乘得的积相加;\n\n乘法交换律的特点是两个数相乘, 交换两个因数的位置, 积不变。\n\n乘法结合律的特点是三个数相乘, 先把前两个数相乘, 或先把后两个数相乘, 积不变。\n\n乘法分配律的特点是两个数的和与一个数相乘, 可以先把它们与这个数分别相乘, 再相加。依此选择\n即可。\n\n【详解】根据分析可知, 在列坚式计算 $403 \\times 21$ 时, 我们是这样计算的(如下), 这个过程利用的是乘法分配律。\n\n$$\n\\begin{array}{r}\n403 \\\\\n\\times \\quad 21 \\\\\n\\hline 403 \\\\\n+806 \\\\\n\\hline 8463\n\\end{array}\n$$\n\n故答案为:\n\n【点睛】此题考查的是三位数与两位数的乘法计算, 以及乘法交换律、乘法结合律、乘法分配律的特点是解答此题的关键。"} {"id": "27518", "image": [], "answer": "C", "solution": "null", "level": "四年级", "question": "已知 $O+\\triangle=$ 。。下列算式正确的是()。\n", "options": "A. $O+\\square=\\triangle$\nB. $\\triangle+\\square=O$\nC. $\\square-\\triangle=O$\n", "subject": "算术", "analysis": "解$\\mathrm{C}$\n\n【分析】根据加数十加数=和, 和一另一个加数=加数求解即可。\n\n【详解】已知 $\\mathrm{O}+\\triangle=\\square$, 可得以下算式:\n\n$\\square-\\mathrm{O}=\\triangle$\n\n$\\square-\\triangle=O$\n\n所以上列算式正确的是 $\\square-\\triangle=O$ 。\n\n故答案为: $\\mathrm{C}$\n\n【点睛】熟练掌握加法各部分间的关系, 是解答本题的关键。"} {"id": "27529", "image": [], "answer": "A", "solution": "null", "level": "四年级", "question": "下面按照(1)加法(2)除法(3)乘法的顺序运算的式是()。\n", "options": "A. $158 \\times[(27+54) \\div 9]$\nB. $[158 \\times(27+54)] \\div 9$\nC. $158 \\times(27+54 \\div 9)$\n", "subject": "算术", "analysis": "解A\n\n【分析】四则混合运算的运算顺序:同级运算,从左往右依次进行计算; 既有加减, 又有乘除的, 先算乘除, 再算加减; 有括号, 先算括号里面的, 如果既有小括号又中括号的, 先算小括号里面的, 再算中括号里面的。\n\n【详解】A. $158 \\times[(27+54) \\div 9]$, 先算加法, 再算除法, 最后算乘法;\n\nB. $[158 \\times(27+54)] \\div 9$, 先算加法, 再算乘法, 最后算除法;\n\nC. $158 \\times ( 27+54 \\div 9 )$, 先算除法,再算加法,最后算乘法;\n\n故答案为: A\n\n【点睛】本题主要考查学生对整数混合运算顺序的掌握和灵活运用。"} {"id": "27539", "image": [], "answer": "C", "solution": "null", "level": "四年级", "question": "已知 $\\diamond+0=\\triangle$, $\\square \\times 0=\\leadsto$ (所有图形均不为 0 ), 下面算式中, 正确的是()。\n", "options": "A. $\\triangle+o=\\diamond$\nB. $\\diamond-\\circ=\\triangle$\nC. $\\dot{\\Re} \\div \\square=0$\n", "subject": "算术", "analysis": "解C\n\n【分析】在加法中, 加数加加数等于和, 一个加数等于和减去另一个加数; 在乘法中, 乘数乘乘数等于积, 一个乘数等于积除以另一个乘数, 据此解答。\n\n【详解】已知 $\\diamond+o=\\triangle, \\square \\times 0=$ 沂(所有图形均不为 0 ), 下面算式中, 正确的是(㾎 $\\div=0$ )。故答案为: C\n\n【点睛】熟练掌握加法、乘法各部分之间的关系并灵活运用是解答本题的关键。"} {"id": "27540", "image": [], "answer": "B", "solution": "null", "level": "四年级", "question": "学校头了 5 个同样的足球和 8 个同样的蓝球。一共花了 976 元。一个足球 80 元, 一个篮球多少钱?正确的列式是()。\n", "options": "A. $976-80 \\times 5 \\div 8$\nB. $(976-80 \\times 5) \\div 8$\nC. $(976-80) \\div 8$\n", "subject": "算术", "analysis": "解$\\mathrm{B}$\n\n【分析】买了 5 个足球, 一个足球 80 元, 先用 80 乘 5 求出买足球花的钱数, 然后再用花去的总钱数减去买足球花的钱数求出买篮球花的钱数, 最后再用买篮球的钱数除以买篮球的个数即可。\n\n【详解】 $(976-80 \\times 5) \\div 8$\n\n$=(976-400) \\div 8$\n\n$=576 \\div 8$\n\n$=72$ (元)\n故答案为: B\n\n【点睛】此题主要考查了减法、除法的意义的应用, 解答此题的关键是求出买篮球花的总钱数。"} {"id": "27541", "image": [], "answer": "A", "solution": "null", "level": "四年级", "question": "工人王师傅和徒弟要把加工好的零件装箱, 每 28 个装一箱, 装了 7 箱, 还有 48 个零件没有装入纸箱。根据所给信息,不能解决的问题是()。\n", "options": "A. 两人各装了多少个零件\nB. 一共需要几个纸箱\nC. 一共加工了多少个零件\n", "subject": "算术", "analysis": "解A\n\n【分析】用没有装箱零件个数除以一箱装零件个数, 可以求出没有装箱零件需要纸箱数量, 再加上已经装箱数量, 可以求出一共需要纸箱数量。用一箱装零件个数乘已经装箱数量, 求出已经装箱零件个数, 再加上没有装箱零件个数, 求出一共加工零件个数。这些零件是两人一共加工的, 在现有条件下,不能求出两人各装零件个数。据此得解。\n\n【详解】由分析得:\n\n不能求出两人各装了多少个零件, 可以求出一共需要几个纸箱以及一共加工了多少个零件。\n\n故答案为: $\\mathrm{A}$\n\n【点睛】解决本题时应从条件进行分析, 理清量与量之间的关系, 再进行解答。"} {"id": "27542", "image": [], "answer": "C", "solution": "null", "level": "四年级", "question": "求“ 5 除 975 与 150 的差, 商是多少”的正确算式是()。\n", "options": "A. $975-150 \\div 5$\nB. $975 \\div 5-150$\nC. $(975-150) \\div 5$\n", "subject": "算术", "analysis": "解C\n\n【分析】分析题干可知, 要求商是多少, 所以在这个算式中先算减法, 再算除法。\n\n【详解】经过分析, 列式为: $(975-150) \\div 5$ 。\n\n故答案为: $\\mathrm{C}$\n\n【点睛】解答此题需要注意的是“除”和“除以”不同, 一个数除另一个数, 是用另一个数除以一个数。"} {"id": "27543", "image": [], "answer": "B", "solution": "null", "level": "四年级", "question": "营养专家说:10岁左右的儿童从每顿午餐中获取的脂肪应不超过 $50 \\mathrm{~g}$, 热量应不低于 2926 千焦。小米选择的菜肴是(1)(2)(3), 小乐选择的菜肴是(2)(3)(4)。()选择的符合营养标准。\n\n| 菜名 | 热量/千焦 | 脂肪/g |\n| :---: | :---: | :---: |\n| (1)土豆炖牛肉 | 1095 | 23 |\n| (2)炸鸡排 | 1254 | 19 |\n| (3)西红柿鸡蛋 | 899 | 15 |\n| (4)香菇油菜 | 911 | 11 |\n\n", "options": "A. 小米\nB. 小乐\nC. 小米和小乐\n\n", "subject": "算术", "analysis": "解$\\mathrm{B}$\n\n【分析】将菜肴(1)(2)(3)的热量相加, 看总热量是否不低于 2926 千焦。将菜肴(1)(2)(3)的脂肪相加, 求出总脂肪是否不超过 $50 \\mathrm{~g}$ 。同理判断菜肴(2)(3)(4)的热量和脂肪是否符号要求。\n\n【详解】 $1095+1254+899=3248$ (千焦),3248>2926\n\n$23+19+15=57(\\mathrm{~g}), 57>50$\n\n则菜肴(1)(2)(3)不符合营养标准。\n\n$1254+899+911=3064$ (千焦),3064>2926\n\n$19+15+11=45(\\mathrm{~g}), 45<50$\n\n则菜肴(2)(3)(4)符合营养标准。\n\n故答案为: B\n\n【点睛】本题关键是正确理解题意, 根据整数加法计算方法求出各种菜肴的总热量和总脂肪量。"} {"id": "27544", "image": [], "answer": "B", "solution": "null", "level": "四年级", "question": "把“ $26+14=40,280 \\div 40=7,52-7=45$ ”改写成综合算式, 正确的是()。\n", "options": "A. $52-(280 \\div 26+14)$\nB. $52-280 \\div(26+14)$\nC. $280 \\div(26+14)-52$\n", "subject": "算术", "analysis": "解$\\mathrm{B}$\n\n【分析】先计算 26 和 14 的和, 再用 280 除以它们的和, 最后再用 52 减去商即可, 据此解答。\n\n【详解】把“ $26+14=40,280 \\div 40=7,52-7=45$ ”改写成综合算式为 $52-280 \\div(26+14)$ 。\n\n故答案为: $\\mathrm{B}$\n【点睛】熟练掌握把所给的式子列乘一个综合算式, 注意运算顺序是解答本题的关键。"} {"id": "27546", "image": [], "answer": "A", "solution": "null", "level": "四年级", "question": "(2022 春・六枝特区期末)已知 $\\square \\times \\triangle=$ 认( 、 $\\triangle$ 、认均不为 0 ), 下面式子不成立的是 ( )\n", "options": "A. $\\triangle \\div i \\gtrless=\\square$\nB. $i \\div \\triangle=\\square$\nC. $\\triangle \\times \\square=\\AA$\nD. $i \\div \\square=\\triangle$\n", "subject": "算术", "analysis": "解: $A . \\triangle \\div \\Re \\neq \\square$, 积 $\\div$ 一个因数 $=$ 另一个因数, 该式子错误;\nB. $\\sim \\triangle=\\square$, 积 $\\div$ 一个因数 $=$ 另一个因数, 该式子正确;\nC. $\\triangle \\times \\square=$, 因数 $\\times$ 因数 $=$ 积, 该式子正确;\nD. $\\div \\square=\\triangle$, 积 $\\div$ 一个因数 $=$ 另一个因数, 该式子正确。\n\n已知 $\\square \\times \\triangle=$ 出( $\\square 、 \\triangle$ 、令均不为 0 ), 式子不成立的是( $\\triangle \\div \\hat{\\sim}=\\square ) 。$\n\n故选: $A$ 。"} {"id": "27557", "image": [], "answer": "B", "solution": "null", "level": "四年级", "question": "(2022 春・坪山区期末) 一台电梯限重 1000 千克, 一个成人约重 80 千克, 一个儿童约重 30 千克,下列答案中会超重的是()\n", "options": "A. 11 个成人, 3 个儿童\nB. 12 个成人, 2 个儿童\nC. 10 个成人, 4 个儿童\nD. 9 个成人, 5 个儿童\n", "subject": "算术", "analysis": "解【解答】解: $A$. $11 \\times 80+3 \\times 30$\n\n$=880+90$\n\n$=970$ (千克)\nB. $12 \\times 80+2 \\times 30$\n\n$=960+60$\n\n$=1020$ (千克)\nC. $10 \\times 80+4 \\times 30$\n\n$=800+120$\n\n$=920$ (千克)\nD. $9 \\times 80+5 \\times 30$\n\n$=720+150$\n\n$=870$ (千克)\n\n1020 千克 $>1000$ 千克 $>970$ 千克 $>920$ 千克\n\n所以答案中会超重的是选项 $B$ 。\n\n故选: $B$ 。"} {"id": "27567", "image": [], "answer": "C", "solution": "null", "level": "四年级", "question": "(2022 春・安龙县期中)下面说法错误的是()\n", "options": "A. 减数 $=$ 被减数 - 差\nB. 除数 $=$ 被除数 $\\div$ 商\nC. 被除数 $=$ 商 $\\div$ 除数\nD. 被减数=差+减数\n", "subject": "算术", "analysis": "【解答】解: $A$. 减数=被减数 - 差, 原题说法正确。\n\nB. 除数=被除数 $\\div$ 商, 原题说法正确。\n\n$C$. 被除数 $=$ 商 $\\times$ 除数, 原题说法错误。\n\nD. 被减数=差+减数, 原题说法正确。\n\n故选: $C$ 。"} {"id": "27568", "image": [], "answer": "D", "solution": "null", "level": "四年级", "question": "(2022 秋・灵宝市期中)进入知识宫的密码是 $\\star 0$ O $6 \\div \\star=2,62-\\bullet=56$, 知识宫的密码是\n", "options": "A. 466\nB. 344\nC. 336\nD. 366\n", "subject": "算术", "analysis": "解【解答】解:因为 $6 \\div \\star=2$\n\n所以 $\\star=3$\n\n因为 $62-0=56$\n\n所以 $=6$\n\n答: 知识宫的密码是 366 。\n\n故选: $D$ 。"} {"id": "27569", "image": [], "answer": "A", "solution": "null", "level": "四年级", "question": "(2021 秋・镇巴县期末)学校买了 129 盒彩色粉笔, 735 盒白色粉笔, 再买()盒白色粉笔,白色粉笔的盒数是彩色粉笔的 7 倍。\n", "options": "A. 168\nB. 178\nC. 78\nD. 68\n", "subject": "算术", "analysis": "【解答】解: $129 \\times 7-735$\n\n$=903-735$\n\n$=168$ (盒)\n\n答: 再买 168 盒白色粉笔, 白色粉笔的盒数是彩色粉笔的 7 倍。\n\n故选: $A$ 。"} {"id": "27570", "image": [], "answer": "B", "solution": "null", "level": "四年级", "question": "(2021 秋 $\\cdot$ 海曙区期末)明明在计算一个数除以 21 时, 不小心把 21 看成了 12 , 得到的商是 63 ,那么正确的商应该是()\n", "options": "A. 72\nB. 36\nC. 63\nD. 不太确定\n", "subject": "算术", "analysis": "解【解答】解: $63 \\times 12 \\div 21$\n\n$=63 \\div 21 \\times 12$\n\n$=3 \\times 12$\n\n$=36$\n\n答: 正确的商应该是 36。\n\n故选: $B$ 。"} {"id": "27571", "image": [], "answer": "A", "solution": "null", "level": "四年级", "question": "( 2021 秋 ・秦都区期末) 小辉把 $2+4 \\times 3$ 错算成 $(2+4) \\times 3$, 这样得到的结果与正确结果相差 $($ )\n", "options": "A. 4\nB. 12\nC. 14\nD. 18\n", "subject": "算术", "analysis": "解【解答】解: $2+4 \\times 3$\n\n$=2+12$\n\n$=14$\n\n$(2+4) \\times 3$\n\n$=6 \\times 3$\n\n$=18$\n\n$18-14=4$\n\n答: 小辉把 $2+4 \\times 3$ 错算成 $(2+4) \\times 3$, 这样得到的结果与正确结果相差 4 。\n\n故选: $A$ 。"} {"id": "27572", "image": [], "answer": "B", "solution": "null", "level": "四年级", "question": "(2021 秋 ・郯城县期末)已知被减数、减数、差的和等于 360 , 则被减数等于()\n", "options": "A. 120\nB. 180\nC. 90\nD. 100\n", "subject": "算术", "analysis": "解【解答】解: 被减数+减数 + 差 $=360$, 已知差 + 减数 $=$ 被减数, 则被减数 + 被减数 $=360$\n\n被减数 $=360 \\div 2=180$\n\n答: 被减数等于 180 。\n\n故选: $B$ 。"} {"id": "27574", "image": [], "answer": "B", "solution": "null", "level": "四年级", "question": "(2022 秋・南京期末)下面算式的得数与 “ $720 \\div(45 \\times 2)$ ”相等的是()\n", "options": "A. $720 \\div 45 \\times 2$\nB. $720 \\div 45 \\div 2$\nC. $720 \\times 45 \\div 2$\nD. $720 \\times 45 \\times 2$\n", "subject": "算术", "analysis": "【解答】解: $720 \\div(45 \\times 2)$\n\n$=720 \\div 90$\n\n$=8$\nA. $720 \\div 45 \\times 2$\n\n$=16 \\times 2$\n\n$=32$\nB. $720 \\div 45 \\div 2$\n\n$=16 \\div 2$\n\n$=8$\nC. $720 \\times 45 \\div 2$\n\n$=32400 \\div 2$\n\n$=16200$\nD. $720 \\times 45 \\times 2$\n\n$=32400 \\times 2$\n\n$=64800$\n\n故答案为: $B$ 。"} {"id": "27585", "image": [], "answer": "B", "solution": "null", "level": "四年级", "question": "(2022 秋・芝罧区期末)下面算式中,属于正确运用乘法分配律的是()\n", "options": "A. $6 \\times(4+28)=6 \\times 4+28$\nB. $(4+8) \\times 25=4 \\times 25+8 \\times 25$\nC. $24 \\times(5+12)=24 \\times 5+5 \\times 12$\nD. $8 \\times(5 \\times 7)=8 \\times 5+8 \\times 7$\n", "subject": "算术", "analysis": "解【解答】解: $6 \\times(4+28)=6 \\times 4+6 \\times 28$\n\n$(4+8) \\times 25=4 \\times 25+8 \\times 25$\n\n$24 \\times(5+12)=24 \\times 5+24 \\times 12$\n\n$8 \\times(5 \\times 7)=(8 \\times 5) \\times 7$, 运用了乘法结合律。\n\n故选: B。"} {"id": "27592", "image": [], "answer": "B", "solution": "null", "level": "四年级", "question": "(2022 秋・北碚区期末) $87+136+64=87+( 136+64 )$, 这里运用了()\n\n", "options": "A. 加法交换律\n\nB. 加法结合律\n\nC. 加法交换律和加法结合律\n\nD. 无法确定\n\n", "subject": "算术", "analysis": "解【解答】解: $87+136+64=87+(136+64)$, 这里运用了加法结合律。\n\n故选: $B$ 。"} {"id": "27593", "image": [], "answer": "C", "solution": "null", "level": "四年级", "question": "(2022 秋 $\\cdot$ 大治市期末)与算式 $125 \\times 16$ 的计算结果不相等是()\n", "options": "A. $125 \\times(8+8)$\nB. $125 \\times 8 \\times 2$\nC. $125 \\times 8+125 \\times 2$\nD. $125 \\times 10+125 \\times 6$\n", "subject": "算术", "analysis": "【解答】解: $A$.因为 $8+8=16$, 所以 $125 \\times 16=125 \\times(8+8)$;\n\nB. 因为 $8 \\times 2=16$, 所以 $125 \\times 16=125 \\times 8 \\times 2$;\n\nC. 因为 $125 \\times 8+125 \\times 2=125 \\times(8+2)=125 \\times 10,10 \\neq 16$, 所以 $125 \\times 16 \\neq 125 \\times 8+125 \\times 2$;\n\nD. 因为 $125 \\times 10+125 \\times 6=125 \\times(10+6)=125 \\times 16$ 。\n\n故选: $C$ 。"} {"id": "27594", "image": [], "answer": "C", "solution": "null", "level": "四年级", "question": "(2022 秋 $\\cdot$ 浑南区期末) 与 $37 \\times 101$ 计算结果相同的算式是 ( )\n", "options": "A. $37 \\times 100+1$\nB. $37 \\times 100-1$\nC. $37 \\times 100+37$\nD. $37 \\times 100 \\times 1$\n", "subject": "算术", "analysis": "【解答】解: $37 \\times 101$\n\n$=37 \\times(100+1)$\n\n$=37 \\times 100+37 \\times 1$\n\n$=37 \\times 100+37$\n\n选项 $C$ 是正确的。\n\n故选: $C$ 。"} {"id": "27595", "image": [], "answer": "B", "solution": "null", "level": "四年级", "question": "(2022 秋 ・莱阳市期末)与 $36.5 \\div 8 \\div 2$ 不相等的是 ( )\n", "options": "A. $36.5 \\div 2 \\div 8$\nB. $36.5 \\div(8 \\div 2)$\nC. $36.5 \\div(8 \\times 2)$\nD. $36.5 \\div 16$\n", "subject": "算术", "analysis": "解【解答】解: $36.5 \\div 8 \\div 2=36.5 \\div 2 \\div 8$\n\n或: $36.5 \\div 8 \\div 2$\n\n$=36.5 \\div(8 \\times 2)$\n\n$=36.5 \\div 16$\n\n所以选项 $A C D$ 是正确的, 不正确是 $B$ 。\n\n故选: $B$ 。"} {"id": "27596", "image": [], "answer": "D", "solution": "null", "level": "四年级", "question": "(2022 秋・中山区期末)下列算式中, 积在 4000 到 5000 之间的是()\n", "options": "A. $11 \\times 502$\nB. $101 \\times 53$\nC. $29 \\times 128$\nD. $22 \\times 211$\n", "subject": "算术", "analysis": "解【解答】解: $A 、 11 \\times 502 \\approx 5500$\n\nB、 $101 \\times 53 \\approx 5300$\n\nC、 $29 \\times 128 \\approx 3900$\n\n$D 、 22 \\times 211 \\approx 4400$\n\n得数在 4000 至 5000 之间的是 $22 \\times 211$ 。\n\n故选: $D$ 。"} {"id": "27610", "image": [], "answer": "C", "solution": "null", "level": "四年级", "question": "给 $720-10 \\times 35 \\div 5$ 添上括号, 使运算顺序改成先算乘法, 再算减法, 最后算除法, 下面说法正确的是()。\n", "options": "A. $(720-10) \\times 35 \\div 5$\nB. $720-10 \\times(35 \\div 5)$\nC. $(720-10 \\times 35) \\div 5$\n", "subject": "算术", "analysis": "C"} {"id": "27619", "image": [], "answer": "A", "solution": "null", "level": "四年级", "question": "根据下面的算式 ( ) 可以直接写出 $4128 \\div 16$ 的结果\n", "options": "A. $258 \\times 16=4128$\nB. $4128 \\times 16=66048$\nC. $24 \\times 172=4128$\n", "subject": "算术", "analysis": "A"} {"id": "27620", "image": [], "answer": "B", "solution": "null", "level": "四年级", "question": "下面算式中去掉括号后,不改变结果的是()\n", "options": "A. $(70+12) \\times(25-13)$\nB. $620+(256-51 \\times 2)$\nC. $(205 \\div 5-8) \\times 7$\n", "subject": "算术", "analysis": "B"} {"id": "14108", "image": ["3065.jpg"], "answer": "A", "solution": "null", "level": "四年级", "question": "经\n\n从 (\n\n面看到的形状不变。", "options": "A. 左\nB. 前\nC. 上", "subject": "立体几何学", "analysis": "答案:\\mathrm{A}$"} {"id": "27458", "image": ["13342.jpg", "13343.jpg", "13344.jpg", "13345.jpg"], "answer": "C", "solution": "null", "level": "四年级", "question": "下面的几何体从左面看, 看到的图形是 $\\square$ 的有()。\n\n\n(1)\n\n\n(2)\n\n\n(3)\n", "options": "A. (1)(2)\nB. (2)(3)\nC. (1)(3)\n\n", "subject": "立体几何学", "analysis": "解$\\mathrm{C}$\n\n【分析】分别从左面看各个图形,看哪个图形从左面看是\n\n\n\n【详解】从左面观察图形(1), 看到\n\n| $\\square$ |\n| :--- |\n| $\\square$ |\n\n从左面观察图形(2), 看到\n\n| $\\square$ | 。 |\n| :--- | :--- |\n\n从左面观察图形(3),看 $\\qquad$则看到的图形是 $\\square$ 的的有(1)(3)。\n\n故答案为: $\\qquad$\n【点睛】本题考查了从不同的方向观察物体, 需要学生有较强的空间想象和推理能力。"} {"id": "27464", "image": ["13361.jpg", "13362.jpg", "13363.jpg", "13364.jpg", "13365.jpg", "13366.jpg", "13367.jpg", "13368.jpg"], "answer": "C", "solution": "null", "level": "四年级", "question": "下面两个几何体, 从 ( ) 面看到的形状相同。\n\n", "options": "A. 前\nB. 上\nC. 左\n", "subject": "立体几何学", "analysis": "解C\n\n【分析】画出两个几何体从前、上、左观察看到的图形即可解答。\n\n\n\n从前面看到的形状是\n\n\n\n从上面看到的形状是\n\n\n\n面看到的形状是\n\n\n, 从前面看到的形状是\n\n\n\n,从上面看到的形状是\n\n\n\n从左面看到的形状是\n\n\n\n故答案为: $\\mathrm{C}$\n\n【点睛】本题主要考查学生对三视图知识的掌握和灵活运用。"} {"id": "13975", "image": ["3055.jpg", "3056.jpg", "3057.jpg"], "answer": "B", "solution": "null", "level": "四年级", "question": "用木条围成下列图形并用钉子固定, 最为牢固的是( )。", "options": "A.\n\n\nB.\n\n\nc. 4\nD.\n\n", "subject": "立体几何学", "analysis": "答案:B \\quad"} {"id": "14065", "image": [], "answer": "C", "solution": "null", "level": "四年级", "question": "把 13 厘米长的线段分成三段, 围成一个等腰三角形, ( )是正确的。(单位: 厘米)", "options": "A. $3,4,6$\nB. $7,3,3$\nC. $4,5,4$", "subject": "立体几何学", "analysis": "答案:C $\\quad"} {"id": "14073", "image": [], "answer": "C", "solution": "null", "level": "四年级", "question": "下面说法正确的是 ( $\\quad)$ 。", "options": "A. 小数点左边是十位, 右边是十分位\n\nB. 有一个角是 $40^{\\circ}$ 的等腰三角形一定是针角三角形\n\nC.", "subject": "立体几何学", "analysis": "答案:C"} {"id": "14081", "image": ["2906.jpg", "2907.jpg"], "answer": "A", "solution": "null", "level": "四年级", "question": "分别从左面和前面看下面的物体, 看到图形相同的是 ( )。", "options": "A.\n\n\n\n\nB.\nс. $\\square$", "subject": "立体几何学", "analysis": "A"} {"id": "27470", "image": ["13384.jpg", "13385.jpg", "13386.jpg", "13387.jpg"], "answer": "C", "solution": "null", "level": "四年级", "question": "观察下面的 4 个物体, 从左面看, 看到的图形与其他三个不同的是()。\n", "options": "A.\n\n\nB.\n\n\nC.\n\n\nD.\n\n\n\n", "subject": "立体几何学", "analysis": "解C\n\n【分析】分别对每个物体从左面看到的图形进行分析, 然后再选择即可;\n\nA. 此图从左面看, 可看到 2 层, 第 1 层可看到 2 个小正方形, 第 2 层可看到 1 个小正方形, 左齐;\n\nB. 此图从左面看, 可看到 2 层, 第 1 层可看到 2 个小正方形,第 2 层可看到 1 个小正方形,左齐;\n\nC. 此图从左面看, 可看到 2 层, 第 1 层可看到 3 个小正方形,第 2 层可看到 1 个小正方形, 左齐;\n\nD. 此图从左面看, 可看到 2 层, 第 1 层可看到 2 个小正方形, 第 2 层可看到 1 个小正方形, 左齐。\n\n【详解】根据分析可知, A、B、D 图从左面看, 都可看到 2 层, 第 1 层都可看到 2 个小正方形, 第 2 层都可看到 1 个小正方形, 都是左齐; C 图从左面看, 可看到 2 层, 第 1 层可看到 3 个小正方形, 第 2 层可看到 1 个小正方形, 左齐; 因此 C 图从左面看, 看到的图形与其他三个不同。\n\n故答案为: C\n\n【点晴】熟练掌握对物体三视图的认识是解答此题的关键。"} {"id": "27448", "image": ["13316.jpg", "13317.jpg", "13318.jpg", "13319.jpg", "13320.jpg", "13321.jpg", "13322.jpg"], "answer": "B", "solution": "null", "level": "四年级", "question": "下面物体()从左面看, 所看见的图形是\n\n\n", "options": "A.\n\n\nB.\n\n\nC.\n\n\n\n", "subject": "画法几何学", "analysis": "解$\\mathrm{B}$\n\n【分析】先分析出每个选项中的物体从左面看到的图形,然后再进行选择即可;\n\n| |\n| :--- |\n| |\n\n此图为 2 层,每层都是 1 个小正方形, 依此选择。\n\n【详解】A.\n\n\n此物体从左面看, 可看到 2 层, 第 1 层可看到 2 个小正方形, 第 2 层可看到 1 个小正方形, 左齐;\n\nB.\n\n\n此物体从左面看, 可看到 2 层, 每层都可看到 1 个小正方形;\n\nC.\n\n\n\n此物体从左面看, 可看到 2 层, 第 1 层可看到 2 个小正方形, 第 2 层可看到 1 个小正方形,左齐。\n\n故答案为: B\n\n【点睛】熟练掌握对物体三视图的认识是解答此题的关键。"} {"id": "27463", "image": ["13356.jpg", "13357.jpg", "13358.jpg", "13359.jpg", "13360.jpg"], "answer": "C", "solution": "null", "level": "四年级", "question": "下面的物体从左面看, 看到的图形是 ( )。\n\n\n", "options": "A.\n\n\nB.\n\n\nC.\n\n\n\n", "subject": "画法几何学", "analysis": "解C\n\n【分析】\n\n\n\n此图从左面看, 可看到 3 层, 第 1 层可看到 2 个小正方形, 第 2 层、第 3 层均可看到 1 个小正方形, 都是右齐, 依此选择。\n【详解】A. 此图为 3 层, 第 1 层为 2 个小正方形, 第 2 层、第 3 层均为 1 个小正方形, 都是左齐,因此不满足;\n\nB. 此图为 2 层, 第 1 层为 2 个小正方形, 第 2 层为 1 个小正方形, 右齐, 因此不满足;\n\nC. 此图为 3 层, 第 1 层为 2 个小正方形, 第 2 层、第 3 层均为 1 个小正方形, 都是右齐, 因此满足;故答案为: $\\mathrm{C}$\n\n【点睛】熟练掌握对三视图的认识是解答此题的关键。"} {"id": "27465", "image": ["13369.jpg", "13370.jpg", "13371.jpg"], "answer": "C", "solution": "null", "level": "四年级", "question": "观察左图, 从正面、侧面和上面看到的图形依次应该选择()。\n\n(1)\n\n\n(2)\n\n\n(3)\n", "options": "A. (1)、(3)、(2)\nB. (3)、(2)、(1)\nC. (2)、(3)、(1)\n\n", "subject": "画法几何学", "analysis": "解$\\mathrm{C}$\n\n【分析】观察图形可知:\n\n从正面看, 是 3 个正方形, 分两层 , 上边一层 1 个, 下边一层 2 个, 左边对齐;从侧面看, 是 3 个正方形, 分两层 , 上边一层 1 个, 下边一层 2 个, 右边对齐;从上面看, 是 3 个正方形, 分两层,上边一层 2 个, 下边一层 1 个, 左边对齐。\n\n【详解】根据分析可知:从正面看到的图形是(2); 从侧面看到的图形是(3); 从上面看到的图形是(1)。故答案为: C\n\n【点睛】此题考查了从不同方向观察几何体,可以锻炼学生的空间想象力和抽象思维力。"} {"id": "13970", "image": ["3078.jpg"], "answer": "B", "solution": "null", "level": "四年级", "question": "有一堆钢管如图摆放, 计算钢管根数的正确算式是( )。", "options": "A. $(5+10) \\times 6$\nB. $(5+10) \\times 6 \\div 2$\nC. $(5+10) \\times 2 \\times 6$\nD. $(5+10) \\times 2 \\div 6$\n\n", "subject": "组合数学", "analysis": "B"} {"id": "14024", "image": [], "answer": "B", "solution": "null", "level": "四年级", "question": "某商场举行购物抽奖活动,设置两个奖项,一等奖 100 元,二等奖 80 元,共有 35 个中奖名额,奖金总额是 3000 元。一等奖有 ( )个。", "options": "A. 25\nB. 10\nC. 20", "subject": "组合数学", "analysis": "B"} {"id": "28661", "image": ["13701.jpg"], "answer": "C", "solution": "null", "level": "四年级", "question": "把整个图形看作“1”, 涂色部分不能用 0.4 表示的是()。\n", "options": "A.\n\n| | |\n| :--- | :--- |\n\nB.\n\n\nC.\n\n| |\n| :--- |\n\nD. 无法确定\n\n", "subject": "组合数学", "analysis": "解$\\mathrm{C}$\n\n【分析】把一个图形平均分成 10 份, 阴影部分占其中的 4 份, 分数表示为 $\\frac{4}{10}$, 小数表示为 0.4 , 写出各个选项表示的数即可解答。\n\n【详解】A. 把长方形平均分成 10 份, 阴影部分占其中的 4 份, 分数表示为 $\\frac{4}{10}$, 小数表示为 0.4 。\n\nB. 把长方形平均分成 10 份, 阴影部分占其中的 4 份, 分数表示为 $\\frac{4}{10}$, 小数表示为 0.4 。\n\nC. 把正方形平均分成 9 份, 阴影部分占其中的 4 份, 分数表示为 $\\frac{4}{9}$ 。\n\nD. 把图形平均分成 10 份, 阴影部分占其中的 4 份, 分数表示为 $\\frac{4}{10}$, 小数表示为 0.4 。故答案为: $\\mathrm{C}$\n\n【点睛】熟练掌握小数的意义是解答本题的关键。"} {"id": "14004", "image": [], "answer": "C", "solution": "null", "level": "四年级", "question": "龙龙班上同学的平均身高是 $132 \\mathrm{~cm}$, 华华班上同学的平均身高是 $135 \\mathrm{~cm}$, 龙龙与华华的身高相比较, ( )。", "options": "A. 华华高\nB. 龙龙高\nC. 无法确定谁高", "subject": "计数", "analysis": "答案: \\mathrm{C} \\quad"} {"id": "14011", "image": ["2877.jpg", "2878.jpg", "2879.jpg", "2880.jpg"], "answer": "B", "solution": "null", "level": "四年级", "question": "下面的统计图中, 虚线所在位置能反映这三个数的平均数的是图\n![](本地图片-5287、.jpg)\n\n4. 下面四个小木块, 你能确定( )木块的高度比较接近它们的平均高度。\n\n\n\nA\n\n\n\nB\n\n\n\nC\n\n\n\nD\n\n5.学校共有 26 副象棋和跳棋, 2 人下一副象棋, 6 人下一副跳棋, 恰\n巧可供 108 人同时进行活动。象棋有 ( )副。", "options": "A. 12\nB. 14\nC. 16\nD. 18", "subject": "计数", "analysis": "答案: \\mathrm{B} \\quad"} {"id": "14021", "image": [], "answer": "C", "solution": "null", "level": "四年级", "question": "在全民阅读调查中, 星光城小区通过网络阅读的有 1500 人, \\$ \\qquad \\$ ,通过书籍阅读的有几人?下列补充条件能用 $1500 \\times 3$ -500 解决的是 $(\\quad)$ 。", "options": "A. 比通过书籍阅读的人数的 3 倍少 500 人\n\nB. 比通过书籍阅读的人数的 3 倍多 500 人\n\nC.通过书籍阅读的人数比通过网络阅读的人数的 3 倍少 500 人", "subject": "计数", "analysis": "答案: \\mathrm{C} \\quad"} {"id": "14072", "image": [], "answer": "D", "solution": "null", "level": "四年级", "question": "六名同学的身高如下, 估计他们的平均身高, 说法正确的是( )。\n\n| 身高/cm | 142 | 137 | 153 | 148 | 135 | 140 |\n| :--- | :--- | :--- | :--- | :--- | :--- | :--- |", "options": "A. 小于 $135 \\mathrm{~cm}$\nB. 小于 $137 \\mathrm{~cm}$\nC. 大于 $153 \\mathrm{~cm}$\nD. 在 $135 \\sim 153 \\mathrm{~cm}$ 之间", "subject": "计数", "analysis": "答案: D$"} {"id": "27597", "image": [], "answer": "A", "solution": "null", "level": "四年级", "question": "(2022 秋・石景山区期末)下面是小博士文具店 2022 年 9 月 1 日至 7 日每天的营业额。(单位:元)\n\n| 日期 | 1 | 2 | 3 | 4 | 5 | 6 | 7 |\n| :---: | :---: | :---: | :---: | :---: | :---: | :---: | :---: |\n| 营业额 | 793 | 836 | 772 | 797 | 802 | 785 | 812 |\n\n估一估, 这 7 天的总营业额大约是()元。\n", "options": "A. 5600\nB. 800\nC. 8000\nD. 56000\n", "subject": "计数", "analysis": "解【解答】解:把每天的营业额看作 800 元。\n\n$800 \\times 7=5600 ($ 元)\n\n答:这 7 天的总营业额大约是 5600 元。\n\n故选: $A$ 。"} {"id": "28030", "image": [], "answer": "C", "solution": "null", "level": "四年级", "question": "用写有数字 $1 、 2 、 3$ 和“.”四张卡片组成不同的小数(每张都用)有()。\n", "options": "A. 6 个\nB. 8 个\nC. 12 个\n", "subject": "计数", "analysis": "解C\n\n【分析】利用枚举法, 不重不漏地列举出所有的情况, 再数出这样的数有几个, 据此解答。\n\n【详解】根据题意, 符合条件的有: $1.23 、 1.32 、 2.31 、 2.13 、 3.12 、 3.21 、 12.3 、 13.2 、 23.1 、 21.3$ 、\n\n$31.2 、 32.1$, 共 12 个。\n\n故答案为: C\n\n【点睛】此题主要使用了枚举法来解决简单的排列组合问题, 要熟练掌握。"} {"id": "28080", "image": [], "answer": "C", "solution": "null", "level": "四年级", "question": "学校举行夏季运动会, 参加 200 米短跑比赛运动员的成绩如下, 那么冠军是 ( )。\n\n| 姓名 | 王强 | 李刚 | 田齐 | 张华 |\n| :---: | :---: | :---: | :---: | :---: |\n| 成绩/秒 | 37.1 | 36.8 | 35.08 | 35.8 |\n\n", "options": "A. 王强\nB. 李刚\nC. 田齐\nD. 张华\n\n", "subject": "计数", "analysis": "解C\n\n【分析】小数比较大小先看整数部分, 整数部分相同则从十分位开始比较, 依次类推, 本题比的是准快, 则就看谁的用时最少谁就是最快, 所以选出最小的小数即可, 据此解答。\n\n【详解】 37.1 秒 $>36.8$ 秒 $>35.8$ 秒 $>35.08$ 秒\n\n学校举行夏季运动会, 参加 200 米短跑比赛运动员的成绩如下, 那么冠军是(田齐)。\n\n故答案为: C\n\n【点睛】本题考查多位小数的大小比较, 应熟练掌握并灵活运用。"} {"id": "28108", "image": [], "answer": "C", "solution": "null", "level": "四年级", "question": "2018 年第 23 届冬季奥林匹克运动会短道速滑男子 500 米决赛中, 我国运动健儿武大靖夺得金牌,并刷新世界纪录和奥运会纪录,下列选项中的时间分别是他和其他两名选手的成绩,武大靖所花时间是()。\n", "options": "A. 39.919 秒\nB. 39.854 秒\nC. 39.584 秒\n", "subject": "计数", "analysis": "解C\n\n【分析】要考虑 5.0 是一个两位小数的近似数, 有两种情况:\n\n“四舍”得到的 5.0 , 有 $5.00 、 5.01 、 5.02 、 5.03 、 5.04$, 其中最大是 5.04; “五入”得到的 5.0, 有 4.95、4.96、4.97、4.98、4.99, 其中最小是 4.95。\n\n【详解】一个两位小数精确到十分位约是 5.0 , 这个数最小是 4.95 。\n\n故答案为: $\\mathrm{C}$\n\n【点睛】已知小数的近似数, 利用“四舍”法得到最大的原数, “五入”法得到最小的原数。"} {"id": "28234", "image": [], "answer": "D", "solution": "null", "level": "四年级", "question": "在小数 1.356 中,数字“ 6 ”的计数单位是()。\n", "options": "A. 1\nB. 0.1\nC. 0.01\nD. 0.001\n", "subject": "计数", "analysis": "解D\n\n【分析】根据小数的组成, 首先搞清这个数字在什么数位上和这个数位的计数单位, 这个数位上是几它就表示有几个这样的计数单位。据此解答。\n\n【详解】在小数 1.356 中, 数字“ 6 ”在千分位上, 千分位的计数单位是千分之一, 写成小数是 0.001 。选项 D 符合题意。\n\n故答案为: D\n【点睛】本题考查了小数的组成及数位和计数单位知识, 结合题意分析解答即可。"} {"id": "14061", "image": ["2896.jpg", "2897.jpg", "2898.jpg", "2899.jpg"], "answer": "A", "solution": "null", "level": "四年级", "question": "要使大、小两个圆组成的图形有无数条对称轴, 应采取图( )的画法。", "options": "A.\n\n\nB.\n\n\nC.\n\n\nD.\n\n", "subject": "图论", "analysis": "答案: \\mathrm{A} \\quad"} {"id": "27305", "image": [], "answer": "C", "solution": "null", "level": "四年级", "question": "(2022 春・安新县期末)已知 $O+\\triangle=\\square$, 下面算式正确的是 ( )\n", "options": "A. $\\triangle+\\square=O$\nB. $O-\\square=\\triangle$\nC. $\\square-\\triangle=O$\n", "subject": "代数", "analysis": "解【解答】解: 已知 $O+\\triangle=\\square$, 下面算式正确的是 $\\square-\\triangle=O$ 。\n\n故选: $C$ 。"} {"id": "27599", "image": ["13514.jpg", "13515.jpg"], "answer": "C", "solution": "null", "level": "四年级", "question": "\n ", "options": "A. $\\tilde{\\Psi}+\\square=0$\n B. $0-\\square=\\underset{\\aleph}{ }$\n\n\n\n", "subject": "代数", "analysis": "解C"} {"id": "27378", "image": [], "answer": "A", "solution": "null", "level": "四年级", "question": "已知 $\\bigcirc+\\square=※, \\triangle \\times \\odot=\\otimes$, 下面算式中错误的是()。\n", "options": "A. $\\square+※=0$\nB. $※-\\bigcirc=\\square$\nC. $\\otimes \\div \\odot=\\triangle$\nD. $\\odot \\times \\triangle=\\otimes$\n", "subject": "逻辑题", "analysis": "解A"} {"id": "14131", "image": [], "answer": "C", "solution": "null", "level": "四年级", "question": "平行四边形的内角和是 ( $\\quad)^{\\circ}$ 。", "options": "A. 180\nB. 90\nC. 360", "subject": "解析几何", "analysis": ""} {"id": "28982", "image": [], "answer": "A", "solution": "null", "level": "四年级", "question": "一个三角形中最大的角是 $88^{\\circ}$, 这是一个 ( ) 三角形。\n", "options": "A. 锐角\nB. 直角\nC. 钝角\nD. 等边\n", "subject": "解析几何", "analysis": "解A\n\n【分析】因为最大的角小于 90 度, 即最大的角是锐角, 则三角形的三个内角都是锐角, 根据锐角三角形的含义: 三个角都是锐角的三角形, 叫做锐角三角形。\n\n【详解】最大的角小于 $90^{\\circ}$ 的三角形, 一定是锐角三角形。\n\n故答案为: A\n\n【点睛】此题主要是根据三角形内角和定理来判断另两个角分别是什么角, 从而根据三角形的分类方法来判定。"} {"id": "11029", "image": [], "answer": "C", "solution": "null", "level": "二年级", "question": "5 个苹果约重 ( $\\quad$ )。", "options": "A.20 克\n B. 200 克\n C.$ 2$ 千克", "subject": "算术", "analysis": "C"} {"id": "11030", "image": [], "answer": "A", "solution": "null", "level": "二年级", "question": "每只小猫重 2 千克, ()只小猫重 8000 克。", "options": "A.4\n B. 16\n C. 4000", "subject": "算术", "analysis": "A"} {"id": "11032", "image": [], "answer": "A", "solution": "null", "level": "二年级", "question": "1 只鹅的质量等于 2 只鸭的质量, 已知 1 只鹅重 8 千克, 1 只鸭重()千克。", "options": "A.4\n B. 16\n C. 10", "subject": "算术", "analysis": "A"} {"id": "11034", "image": [], "answer": "B", "solution": "null", "level": "二年级", "question": "买 1 千克苹果需要 5 元钱, 现有 32 元钱, 最多可买 (\n\n) 千克的苹果。", "options": "A.5\nB. 6\nC. 7", "subject": "算术", "analysis": "B"} {"id": "10267", "image": [], "answer": "C", "solution": "null", "level": "二年级", "question": "亮亮今年上二年级,他的身高大约是()。", "options": "A. 50 厘米\nB. 5 米\nC. 1 米 30 厘米", "subject": "算术", "analysis": "$\\mathrm{C}$"} {"id": "10273", "image": [], "answer": "C", "solution": "null", "level": "二年级", "question": "计算算式 96-32+28, 得数是 ( $\\quad$ )。", "options": "A. 29\nB. 60\nC. 92", "subject": "算术", "analysis": "$\\mathrm{C}$"} {"id": "10284", "image": [], "answer": "B", "solution": "null", "level": "二年级", "question": "一本 50 页纸的本子,上星期用去 21 页,这个星期用去()页,还剩 6 页。", "options": "A. 18\nB. 23\nC. 29", "subject": "算术", "analysis": "$\\mathrm{B}$"} {"id": "10294", "image": [], "answer": "A", "solution": "null", "level": "二年级", "question": "车上原来有 43 人, 下来了 24 人, 又上去了 19 人, 现在车上有 ( ) 人。", "options": "A. 38\nB. 39\nC. 37", "subject": "算术", "analysis": "A"} {"id": "10295", "image": [], "answer": "C", "solution": "null", "level": "二年级", "question": "一根长 80 米的绳子, 第一次剪去 36 米, 第二次剪去 25 米, 这根绳子比原来短了多少米?正确的列式是(", "options": "A. $80-36-25$\nB. $80-25+36$\nC. $25+36$", "subject": "算术", "analysis": "$\\mathrm{C}$"} {"id": "10296", "image": [], "answer": "A", "solution": "null", "level": "二年级", "question": "妈妈买鱼花了 15 元,付了一张 50 元,应找回()元。", "options": "A. 35\nB. 40\nC. 45", "subject": "算术", "analysis": "A"} {"id": "10329", "image": [], "answer": "B", "solution": "null", "level": "二年级", "question": "下面不能用 “三五十五” 来计算的算式是()。", "options": "A. $5 \\times 3$\nB. $5+5+5+5+5 \\quad C$\nC. $3+3+3+3+3$", "subject": "算术", "analysis": "$\\mathrm{B}$"} {"id": "10340", "image": [], "answer": "B", "solution": "null", "level": "二年级", "question": "3 个 4 加上 5 , 和是多少?正确列式是()。", "options": "A. $3+4+5$\n B. $3 \\times 4+5$\n C. $3 \\times(4+5)$", "subject": "算术", "analysis": "$\\mathrm{B}$"} {"id": "10350", "image": [], "answer": "A", "solution": "null", "level": "二年级", "question": "平均每只信鹋送 6 封信 4 只信鹋可以送多少封信? 列式为", "options": "A. $4 \\times 6=24$\nB. $24 \\div 6=4$\nC. $24 \\div 4=6$", "subject": "算术", "analysis": "A"} {"id": "10355", "image": [], "answer": "A", "solution": "null", "level": "二年级", "question": "6+6+6+5, 不可以改写成算式 $(\\quad) 。$", "options": "A. $6 \\times 4$\nB. $6 \\times 3+5$\nC. $6 \\times 4-1$", "subject": "算术", "analysis": "A"} {"id": "10356", "image": [], "answer": "B", "solution": "null", "level": "二年级", "question": "芳芳看一本书, 从第一页看起, 每天看 9 页, 到了第 6 天她应该从第几页看起? ()", "options": "A. 45 页\nB. 46 页\nC. 55 页", "subject": "算术", "analysis": "B"} {"id": "10372", "image": [], "answer": "C", "solution": "null", "level": "二年级", "question": "计算算式 96-32+28, 得数是 ( )。", "options": "A. 29\nB. 60\nC. 92", "subject": "算术", "analysis": "$\\mathrm{C}$"} {"id": "10390", "image": [], "answer": "A", "solution": "null", "level": "二年级", "question": "$6+6+6+5$, 不可以改写成算式 ( ) 。", "options": "A. $6 \\times 4$\nB. $6 \\times 3+5$\nC. $6 \\times 4-1$", "subject": "算术", "analysis": "A"} {"id": "10395", "image": [], "answer": "A", "solution": "null", "level": "二年级", "question": "$7 \\times 8=56$, 算式中的 “ 56 ” 是( )。", "options": "A. 积\nB. 乘数\nC. 商", "subject": "算术", "analysis": "A"} {"id": "10406", "image": [], "answer": "A", "solution": "null", "level": "二年级", "question": "比较大小: $36+28 \\mathrm{O} 6 \\times 9$, $\\mathrm{O}$ 里应填 ( )。", "options": "A. $>$\nB. $<$\nC. $=$", "subject": "算术", "analysis": "A"} {"id": "10417", "image": [], "answer": "B", "solution": "null", "level": "二年级", "question": "下列口诀中只能用来计算一个乘法算式的是()。", "options": "A. 二三得六\nB. 七七四十九\nC. 三五十五", "subject": "算术", "analysis": "$\\mathrm{B}$"} {"id": "10420", "image": [], "answer": "B", "solution": "null", "level": "二年级", "question": "6 个 8 连加得多少?算式是 ( $\\quad$ )。", "options": "A. $8+6$\nB. $8 \\times 6$\nC. $6+8$", "subject": "算术", "analysis": "$\\mathrm{B}$"} {"id": "10421", "image": [], "answer": "C", "solution": "null", "level": "二年级", "question": "买一个西瓜 9 元钱,买 3 个西瓜需要多少元钱?列式计算为()。", "options": "A. $9+3=12$\nB. $9 \\div 3=3$\nC. $9 \\times 3=27$", "subject": "算术", "analysis": "$\\mathrm{C}$"} {"id": "10426", "image": [], "answer": "B", "solution": "null", "level": "二年级", "question": "一场电影的播放时间是 80 分钟, 再加上()分钟是 2 小时。", "options": "A. 20\nB. 40\nC. 80", "subject": "算术", "analysis": "$\\mathrm{B}$"} {"id": "10449", "image": [], "answer": "B", "solution": "null", "level": "二年级", "question": "时针刚走过数字 4 , 分针从 12 开始走()个大格, 就是 4 时 25 分。", "options": "A. 4\nB. 5\nC. 8", "subject": "算术", "analysis": "B"} {"id": "11438", "image": [], "answer": "A", "solution": "null", "level": "二年级", "question": "$30 \\div 6=5$, 读作 ( )。", "options": "A. 30 除以 6 等于 5\nB. 30 除以 5 等于 6", "subject": "算术", "analysis": "A"} {"id": "11449", "image": [], "answer": "B", "solution": "null", "level": "二年级", "question": "下列算式中, 商最大的算式是 ()。", "options": "A. $12 \\div 3$\nB. $36 \\div 6$\nC. $20 \\div 4$\nD. $24 \\div 6$", "subject": "算术", "analysis": "B"} {"id": "11450", "image": [], "answer": "B", "solution": "null", "level": "二年级", "question": "二年(2)班参加舞蹈队的同学站了 5 排, 每排站 6 人, 其中男生有 9 人, 求女生有多少人。用算式表示是( $\\quad$ 。", "options": "A. $5+6-9$\nB. $5+6-9$\nC. $5 \\times 6-9$\nD. $5+6-9$", "subject": "算术", "analysis": "B"} {"id": "10465", "image": ["2140.jpg", "2141.jpg", "2142.jpg", "2143.jpg"], "answer": "B", "solution": "null", "level": "二年级", "question": "下面四种袜子的质量是一样的, 买哪一种最便宜?()", "options": "A.\n\n\nB.\n\n\nC.\n\n\nD.\n\n", "subject": "算术", "analysis": "$\\mathrm{B}$\n\n【分析】分别求出每种袜子的单价再作比较, 单价最低的就是最便宜的。\n\n【详解】A. $12 \\div 2=6 ($ 元 $)$\n\nB. $15 \\div 3=5$ (元)\n\nC. $30 \\div 5=6 ($ 元 $)$\n\nD. $7 \\div 1=7 ($ 元 $)$\n\n$5<6=6<7$\n\n所以 15 元 3 双的最便宜。\n\n故答案为: B\n\n【点睛】算出每种袜子的单价是解题关键。"} {"id": "10476", "image": [], "answer": "C", "solution": "null", "level": "二年级", "question": "下面的算式中, 得数最大的是( )。", "options": "A. $48 \\div 6$\nB. $54 \\div 6$\nC. $2 \\times 5$\nD. $30 \\div 6$", "subject": "算术", "analysis": "$\\mathrm{C}$\n\n【分析】根据表内乘法依次计算出各个算式的得数, 然后进行比较即可。\n\n【详解】A. $48 \\div 6=8$\n\nB. $54 \\div 6=9$\n\nC. $2 \\times 5=10$\n\nD. $30 \\div 6=5$\n\n故答案为: C\n\n【点睛】熟练掌握内乘法口诀, 利用口诀正确计算出结果是解答此题的关键, 要注意平时计算能力的培养。"} {"id": "10484", "image": [], "answer": "C", "solution": "null", "level": "二年级", "question": "下面计算中,不能利用乘法口诀“六八四十八”来计算的是( )。", "options": "A. $6 \\times 8$\nB. $48 \\div 6$\nC. $32 \\div 8$\nD. $48 \\div 8$", "subject": "算术", "analysis": "$\\mathrm{C}$\n\n【分析】思考每个选项中的算式所应用的乘法口诀, 逐项分析后选择正确的答案。\n\n【详解】A. 计算 $6 \\times 8$ 所应用的乘法口诀是:六八四十八;\n\nB. 计算 $48 \\div 6$ 所应用的乘法口诀是:六八四十八;\n\nC. 计算 $32 \\div 8$ 所应用的乘法口诀是:四八三十二;\n\nD. 计算 $48 \\div 8$ 所应用的乘法口诀是:六八四十八;\n\n所以,不能利用乘法口诀“六八四十八”来计算的是 $32 \\div 8$ 。\n\n故答案为: C\n\n【点睛】本题考查乘法口诀的应用, 要熟记每句口诀和对应算式的关系。"} {"id": "10485", "image": [], "answer": "B", "solution": "null", "level": "二年级", "question": "21 根平均分给 3 只小白兔, 每只小白兔分到几根? 列式为:", "options": "A. $21 \\div 7=3$\nB. $21 \\div 3=7$\nC. $3 \\times 7=21$\nD. $7 \\times 3=21$", "subject": "算术", "analysis": "$\\mathrm{B}$\n\n【分析】分析题意是平均分给 3 只小白兔, 每只小白兔分到的根数相同, 用除法计算, 运用 7 的乘法\n口诀求出商。\n\n【详解】根据分析列式为: $21 \\div 3=7$ (根) , 所以每只小白兔分到 7 根。\n\n故答案为: B\n\n【点睛】 掌挃除法的意义, 能运用 7 的乘法口诀求商。"} {"id": "10486", "image": [], "answer": "B", "solution": "null", "level": "二年级", "question": "$8 \\times$ 口 $<64$, 最大可以填()。", "options": "A. 6\nB. 7\nC. 8\nD. 9", "subject": "算术", "analysis": "$\\mathrm{B}$\n\n【分析】利用 8 的乘法口诀求商解答。\n\n【详解】 $64 \\div 8=8$\n\n$8-1=7$\n\n因此最大填 7 。\n\n故答案为: B\n\n【点睛】本题考查了用 8 的乘法口诀求商的应用。"} {"id": "10487", "image": [], "answer": "C", "solution": "null", "level": "二年级", "question": "求 42 里面有几个 7 ? 正确的解答是()。", "options": "A. $42 \\div 6=7$\nB. $42+7=49$\nC. $42 \\div 7=6$\nD. $42-7=35$", "subject": "算术", "analysis": "$\\mathrm{C}$"} {"id": "10491", "image": [], "answer": "B", "solution": "null", "level": "二年级", "question": "根据“七八五十六”可以写出()道除法算式。", "options": "A. 1\n B. 2\n C. 3", "subject": "算术", "analysis": "$\\mathrm{B}$\n\n【分析】根据“七八五十六”可以写出的算式有: $7 \\times 8=56 、 8 \\times 7=56 、 56 \\div 7=8 、 56 \\div 8=7$ 。\n\n【详解】则经分析得知根据“七八五十六”可以写出 2 道除法算式。\n\n故答案为: B。\n\n【点睛】要牢记并灵活运用乘法口诀。"} {"id": "10502", "image": [], "answer": "B", "solution": "null", "level": "二年级", "question": "一袋饼干有 40 块, 每次从袋中拿出 5 块, ( ) 次可以拿完。", "options": "A. 7\nB. 8\nC. 9", "subject": "算术", "analysis": "$\\mathrm{B}$"} {"id": "10511", "image": [], "answer": "A", "solution": "null", "level": "二年级", "question": "27 里面有几个 9?列式是()。", "options": "A. $27 \\div 9$\nB. $27+9$\nC. $27-9$", "subject": "算术", "analysis": "A"} {"id": "10512", "image": [], "answer": "C", "solution": "null", "level": "二年级", "question": "下面哪个问题不可以用 $42 \\div 7$ 来解决?", "options": "A. 42 个苹果, 平均分给小明和他的 6 名同学, 每人分几个?\n\nB. 42 个苹果, 每 7 个装一袋, 可以装几袋?\n\nC. 42 个苹果, 已经吃了 7 个, 还剩几个?", "subject": "算术", "analysis": "$\\mathrm{C}$"} {"id": "10513", "image": [], "answer": "C", "solution": "null", "level": "二年级", "question": "一道除法题, 除数是 9 , 乐乐把被除数的十位数字和个位数字看颠倒了, 结果除得的商是 4 , 这道题正确的商应该是 ( $\\quad$ )。", "options": "A. 36\nB. 13\nC. 7\n\n6 . _从 56 里面连续减 7 , 减 $(\\quad)$ 次, 结果是 0 。\nA. 8\nB. 6\nC. 9", "subject": "算术", "analysis": "C"} {"id": "10517", "image": [], "answer": "C", "solution": "null", "level": "二年级", "question": "计算下列算式时, 不用“七九六十三”这句口块的是()。", "options": "A. $7 \\times 9$\nB. $63 \\div 7$\nC. $7+9$", "subject": "算术", "analysis": "$\\mathrm{C}$"} {"id": "10527", "image": [], "answer": "C", "solution": "null", "level": "二年级", "question": "一根 35 米长的木头, 如果锯成若干段, 每段长 7 米, 要锯 ( ) 次。", "options": "A. 6\nB. 5\nC. 4", "subject": "算术", "analysis": "$\\mathrm{C}$"} {"id": "10536", "image": [], "answer": "B", "solution": "null", "level": "二年级", "question": "王老师每周工作 40 小时, 一周有 5 个工作日, 平均每天工作( )小时。", "options": "A. 7\nB. 8\nC. 9", "subject": "算术", "analysis": "$\\mathrm{B}$"} {"id": "10537", "image": [], "answer": "C", "solution": "null", "level": "二年级", "question": "一支 5 元钱,小明有 40 元钱,能买 $(\\quad)$ 支支", "options": "A. 6\nB. 5\nC. 8", "subject": "算术", "analysis": "$\\mathrm{C}$"} {"id": "10543", "image": [], "answer": "C", "solution": "null", "level": "二年级", "question": "$48 \\div 8=(\\quad)$ 。", "options": "A. 2\nB. 3\nC. 6", "subject": "算术", "analysis": "$\\mathrm{C}$\n\n【分析】此题可根据乘法口诀和表内除法来解答。\n\n【详解】除数是 8 , 根据 8 的乘法口诀“六八四十八”, 可得出结果是 6 。\n\n故答案为: C\n\n【点睛】本题考查的要点是熟练掌握乘法口诀和表内除法。"} {"id": "10554", "image": [], "answer": "A", "solution": "null", "level": "二年级", "question": "下面不是用口诀三七二十一计算的式子是()。", "options": "A. $7 \\times 7$\nB. $3 \\times 7$\nC. $21 \\div 3$", "subject": "算术", "analysis": "A\n\n【分析】口诀三七二十一可以写成 3 与 7 相乘, 也可以用来计算 21 除以 3 或 7。\n\n【详解】A. $7 \\times 7$ 用的口诀是七七四十九;\n\nB. $3 \\times 7$ 用的口诀是三七二十一;\n\nC. $21 \\div 3$ 用的口诀是三七二十一。\n\n故答案为: A\n\n【点睛】灵活运用表内乘法口诀是解题关键。"} {"id": "10564", "image": [], "answer": "C", "solution": "null", "level": "二年级", "question": "有 30 棵柳树, 平均栽成 5 行。当提出的问题是( )时, 算式才能列为 $30 \\div 5=6$ 。", "options": "A. 平均裁成几行?\nB. 有多少棵柳树?\nC. 每行有几棵?", "subject": "算术", "analysis": "$\\mathrm{C}$\n\n【分析】由题干可知, 共有 30 棵柳树, 平均栽成 5 行, 根据平均分的意义, 则(30:5)表示每行裁了多少棵, 即可进行选择。\n\n【详解】有 30 棵柳树, 平均栽成 5 行。当提出的问题是:每行有几棵时, 算式才能列为 $30 \\div 5=6$ 。故答案为: C\n\n【点睛】本题考查的是有关除法的相关应用题, 关键是能够理清关系, 理解平法平均分的意义。"} {"id": "10565", "image": [], "answer": "B", "solution": "null", "level": "二年级", "question": "计算 72 里有几个 9 , 正确的算式是()。", "options": "A. $72 \\div 8$\nB. $72 \\div 9$\nC. $8 \\times 9$", "subject": "算术", "analysis": "$\\mathrm{B}$\n\n【分析】计算 72 里有几个 9, 根据除法的意义, 即用 72 除以 9 即可。\n\n【详解】计算 72 里有几个 9 , 正确的算式应该是 $72 \\div 9$ 。\n\n故答案为: $\\mathrm{B}$\n\n【点睛】除法算式中, 总数作被除数, 平均每组的个数作除数。"} {"id": "10566", "image": [], "answer": "B", "solution": "null", "level": "二年级", "question": "一道除法题, 除数是 3 , 小明把被除数的十位数字和个位数字看颠倒了, 结果除得的商是 7 。这道题被除数应该是 $(\\quad)$ 。", "options": "A. 21\nB. 12\nC. 24", "subject": "算术", "analysis": "$\\mathrm{B}$\n\n【分析】根据被除数=除数 $\\times$ 商求出颠倒后的被除数, 再将个位和十位对调即可。\n\n【详解】 $3 \\times 7=21$ 所以正确的被除数是 12 。\n\n故答案为: B\n\n【点睛】本题考查了被除数、除数和商之间的关系, 需熟练掌握, 灵活使用。"} {"id": "10567", "image": [], "answer": "B", "solution": "null", "level": "二年级", "question": "$\\circ+\\circ+\\circ+\\circ=36$, $\\circ$ 代表的数是()。", "options": "A. 7\nB. 9\nC. 8", "subject": "算术", "analysis": "$\\mathrm{B}$\n【分析】一共有 4 个○相加得 36 , 也就是把 36 平均分成 4 份, 每份是多少, 用除法计算即 36 除以 4 ;据此解答。\n\n【详解】 $36 \\div 4=9$, 所以 0 代表的数是 9 。\n\n故答案为: B\n\n【点睛】本题考查表内除法的计算。"} {"id": "10571", "image": [], "answer": "B", "solution": "null", "level": "二年级", "question": "1 件上衣需要钉 6 个纽扣, 42 个纽扣能钉 ( ) 件这样的上衣。", "options": "A. 6\nB. 7\nC. 8", "subject": "算术", "analysis": "$\\mathrm{B}$\n\n【分析】要计算 42 个纽扣能钉几件这样的上衣, 就是求 42 里面有几个 6 , 列式为: $42 \\div 6$ 。\n\n【详解】 $42 \\div 6=7$ (件)\n\n故答案为: $\\mathrm{B}$"} {"id": "10582", "image": [], "answer": "A", "solution": "null", "level": "二年级", "question": "与 $6 \\times 8$ 使用同一句口诀的算式是 ( )。", "options": "A. $48 \\div 8$\nB. $6 \\times 7$\nC. $24 \\div 6$", "subject": "算术", "analysis": "A\n\n【分析】 $6 \\times 8$ 使用的乘法口诀是:六八四十八,由此对各个选项进行判断即可求解。\n\n【详解】A. $48 \\div 8=6$, 使用的乘法口诀是“六八四十八”;\n\nB. $6 \\times 7=42$, 使用的乘法口诀是“六七四十二”;\n\nC. $24 \\div 6=4$, 使用的乘法口诀是“四六二十四”;\n\n所以与 $6 \\times 8$ 使用同一句口诀的算式是: $48 \\div 8$ 。\n\n故答案为: A\n\n【点睛】解答本题的关键是熟练掌握乘法口诀。"} {"id": "10590", "image": [], "answer": "B", "solution": "null", "level": "二年级", "question": "45 里面有几个 9? 列式正确的是 ( )。", "options": "A. $45 \\div 5=9$\nB. $45 \\div 9=5$\nC. $45+9=45$", "subject": "算术", "analysis": "$\\mathrm{B}$\n\n【分析】要求 45 里面有几个 9 , 用 $45 \\div 9$ 即可得出答案。\n\n【详解】要求 45 里面有几个 9 , 列式为: $45 \\div 9=5$ 。\n\n故答案为: B\n\n【点睛】本题考查学生对表内乘法的掌握和运用。"} {"id": "10591", "image": [], "answer": "C", "solution": "null", "level": "二年级", "question": "下列算式中, 得数不是 9 的是 ( )。", "options": "A. $36 \\div 4$\nB. $9 \\times 1$\nC. $9-1$", "subject": "算术", "analysis": "C"} {"id": "10592", "image": [], "answer": "B", "solution": "null", "level": "二年级", "question": "$48 \\div 6=8$, 当被除数和除数都增加 1 时, 商是 ( )。", "options": "A. 6\nB. 7\nC. 9", "subject": "算术", "analysis": "$\\mathrm{B}$"} {"id": "10593", "image": [], "answer": "A", "solution": "null", "level": "二年级", "question": "从 27 里面连续减去 $9,()$ 次减完。", "options": "A. 3\nB. 7\nC. 9", "subject": "算术", "analysis": "A\n\n【分析】问几次能减完, 就是求 27 里面有几个 9, 用除法计算, 根据 9 的表内乘法口诀求商。\n\n【详解】 $27 \\div 9=3$\n\n故答案为: A\n\n【点睛】理解包含除法的意义及应用。"} {"id": "10597", "image": [], "answer": "A", "solution": "null", "level": "二年级", "question": "把 72 平均分成 8 份, 求每份是多少? 列算式是 ( )。", "options": "A. $72 \\div 8$\nB. $72 \\div 9$\nC. $72-9$", "subject": "算术", "analysis": "A\n\n【分析】每份分得同样多, 叫平均分, 除法就是用来解决平均分问题的。结合题意可列算式: 72 $\\div$ 。\n\n【详解】 $72 \\div 8=9$\n\n故答案为: A"} {"id": "10616", "image": [], "answer": "B", "solution": "null", "level": "二年级", "question": "把 27 根棒棒糖平均分给 9 个小朋友, 每人得 ( ) 根。", "options": "A. 18\nB. 3\nC. 9", "subject": "算术", "analysis": "$\\mathrm{B}$\n\n【详解】要计算每人分得几根棒棒糖, 就是看 27 里面有几个 9 , 列式为: $27 \\div 9=3$ (根)。故答案为: B"} {"id": "10617", "image": [], "answer": "C", "solution": "null", "level": "二年级", "question": "下面的口诀中, 只能写一道除法算式的是( )。", "options": "A. 一六得六\nB. 二四得八\nC. 七七四十九", "subject": "算术", "analysis": "$\\mathrm{C}$\n\n【分析】可逐项分析每个口诀并写出相应的算式, 根据算式的具体情况做选择。\n\n【详解】A. $6 \\div 6=1,6 \\div 1=6$;\n\nB. $8 \\div 4=2,8 \\div 2=4$;\n\nC. $49 \\div 7=7$ 。\n\n故答案为: C"} {"id": "10618", "image": [], "answer": "A", "solution": "null", "level": "二年级", "question": "除数和商都是 8 , 被除数是 $(\\quad)$ 。", "options": "A. 64\nB. 1\nC. 0", "subject": "算术", "analysis": "A"} {"id": "10619", "image": [], "answer": "C", "solution": "null", "level": "二年级", "question": "妈妈用 54 元买了 9 只盘子, 一只盘子 ( ) 元。", "options": "A. 7\nB. 8\nC. 6", "subject": "算术", "analysis": "$\\mathrm{C}$"} {"id": "10622", "image": [], "answer": "C", "solution": "null", "level": "二年级", "question": "只能写一道乘法算式和一道除法算式的口诀是 ( )。", "options": "A. 四八三十二\nB. 二四得八\nC. 七七四十九", "subject": "算术", "analysis": "$\\mathrm{C}$"} {"id": "10633", "image": [], "answer": "B", "solution": "null", "level": "二年级", "question": "有 28 个苹果, $(\\quad)$, 平均每箱装多少个?", "options": "A. 每箱装 7 个\nB. 装在 7 个箱子里\nC. 又买来了 7 个", "subject": "算术", "analysis": "$\\mathrm{B}$"} {"id": "10641", "image": [], "answer": "B", "solution": "null", "level": "二年级", "question": "54 里有几个 9 ? 可以列式为( )。", "options": "A. $54 \\div 6=9$\nB. $54 \\div 9=6$\nC. $6 \\times 9=54$", "subject": "算术", "analysis": "$\\mathrm{B}$"} {"id": "10642", "image": [], "answer": "A", "solution": "null", "level": "二年级", "question": "下列算式中,商最小的算式是 $(\\quad )$ 。", "options": "A. $12 \\div 6$\nB. $24 \\div 8$\nC. $36 \\div 6$", "subject": "算术", "analysis": "A"} {"id": "10643", "image": [], "answer": "B", "solution": "null", "level": "二年级", "question": "40 个苹果每盘放 8 个, 可以放多少盘? 列式为 ( $\\quad$ 。", "options": "A. $40 \\div 5=8$\nB. $40 \\div 8=5$\nC. $5 \\times 8=40$", "subject": "算术", "analysis": "B\n\n【分析】用苹果的总数除以每盘放的苹果数量即可解答。\n\n【详解】由分析可得:正确列式为: $40 \\div 8=5$ (盘)\n\n故答案为: B\n\n【点睛】熟练掌握表内乘法是解题的关键。"} {"id": "10644", "image": [], "answer": "A", "solution": "null", "level": "二年级", "question": "$63 \\div 7 \\bigcirc 63 \\div 9$,里应填 ( )。", "options": "A. $>$\nB. $<$\nC. $=$", "subject": "算术", "analysis": "A"} {"id": "11031", "image": [], "answer": "B", "solution": "null", "level": "二年级", "question": "估计一下,一个成年人每天吃进的食物大约有()。", "options": "A.100 克\n B. 1 千克\n C. 10 千克", "subject": "计数", "analysis": "B"} {"id": "11448", "image": [], "answer": "C", "solution": "null", "level": "二年级", "question": "20 里面有 4 个 ( )。", "options": "A. 10\nB. 4\nC. 5\nD. 16", "subject": "计数", "analysis": "C"} {"id": "11528", "image": [], "answer": "C", "solution": "null", "level": "二年级", "question": "淘气调查了学校五个同学的年龄, 数据如下表。\n\n| 姓名 | 李刚 | 赵四 | 张三 | 陈梅 | 王芳 |\n| :--: | :--: | :--: | :--: | :--: | :--: |\n| 年龄 | 7 | 9 | 8 | 10 | 8 |\n\n下列说法不正确的是 $(\\quad)$ 。", "options": "A.李刚年龄最小\nB.陈梅年龄最大\nC.张三和王芳一定同班", "subject": "计数", "analysis": "C"} {"id": "10246", "image": ["2044.jpg"], "answer": "B", "solution": "null", "level": "二年级", "question": "橡皮擦长 ( ) 厘米。\n\n", "options": "A. 1\nB. 2\nC. 3", "subject": "度量几何学", "analysis": "$\\mathrm{B}$"} {"id": "10257", "image": [], "answer": "B", "solution": "null", "level": "二年级", "question": "下面的物体中, ( ) 的高度最接近 1 米。", "options": "A. 台灯\nB. 讲台\nC. 教室的门", "subject": "度量几何学", "analysis": "$\\mathrm{B}$"} {"id": "10266", "image": [], "answer": "C", "solution": "null", "level": "二年级", "question": "二(1)班小亮的一拃大约有()厘米。", "options": "A. 13\nB. 25\nC. 8", "subject": "度量几何学", "analysis": "$\\mathrm{C}$"} {"id": "10268", "image": [], "answer": "C", "solution": "null", "level": "二年级", "question": "下面可以用来计量物体长度的单位是()。", "options": "A. 时\nB. 分\nC. 米", "subject": "度量几何学", "analysis": "$\\mathrm{C}$"} {"id": "10301", "image": ["2053.jpg", "2054.jpg"], "answer": "C", "solution": "null", "level": "二年级", "question": "如图, (1)号角和(2)号角相比,()。\n\n\n\n(1)\n\n\n\n(2)", "options": "A. (1)号角大\nB. (2)号角大\nC. 一样大", "subject": "度量几何学", "analysis": "$\\mathrm{C}$"} {"id": "10312", "image": [], "answer": "B", "solution": "null", "level": "二年级", "question": "三角尺上的直角与黑板上的直角大小()。", "options": "A. 不相等\nB. 相等\nC. 不能比较", "subject": "度量几何学", "analysis": "B"} {"id": "10323", "image": [], "answer": "B", "solution": "null", "level": "二年级", "question": "一个三角板上, 有 ( ) 个锐角。", "options": "A. 1\nB. 2\nC. 3", "subject": "度量几何学", "analysis": "B"} {"id": "10322", "image": ["2074.jpg", "2075.jpg", "2076.jpg", "2077.jpg"], "answer": "D", "solution": "null", "level": "二年级", "question": "下列图形中, 有一个直角的是 (", "options": "A.\n\n\nB.\n\n\nC.\n\n\nD.\n\n", "subject": "画法几何学", "analysis": "$\\mathrm{D}$"} {"id": "10665", "image": ["2210.jpg", "2211.jpg", "2212.jpg", "2213.jpg", "2214.jpg", "2215.jpg", "2216.jpg"], "answer": "B", "solution": "null", "level": "二年级", "question": "下面图形\n\n", "options": "A.\n\n\nB.\n\n\nC.\n\n", "subject": "画法几何学", "analysis": "$\\mathrm{B}$\n\n【分析】把一个图形整体沿某一方向移动一定的距离, 图形的这种移动, 叫做平移; 在平面内, 将一个图形绕一点按某个方向转动一个角度, 这样的运动叫做图形的旋转; 据此解答即可。\n\n【详解】A.\n\n\n\nB.\n\n\n\nC.\n\n\n\n故答案为: B\n\n【点睛】本题主要考查平移和旋转的意义, 在实际当中的运用。"} {"id": "10383", "image": ["2108.jpg", "2109.jpg", "2110.jpg", "2111.jpg"], "answer": "D", "solution": "null", "level": "二年级", "question": "下列图形中, 有一个直角的是()。", "options": "A.\n\n\nB.\n\n\nC.\n\n\n\n", "subject": "解析几何", "analysis": "D"} {"id": "10437", "image": [], "answer": "B", "solution": "null", "level": "二年级", "question": "用 $3 、 5 、 0$ 三个数字组成的两位数有 $(\\quad)$ 个。", "options": "A. 2\nB. 4\nC. 6", "subject": "组合数学", "analysis": "$\\mathrm{B}$"} {"id": "10448", "image": [], "answer": "B", "solution": "null", "level": "二年级", "question": "有一件上衣, 四条不同的裤子, 共有 ( ) 种不同的穿法。", "options": "A. 3\nB. 4\nC. 5", "subject": "组合数学", "analysis": "$\\mathrm{B}$"} {"id": "10538", "image": ["2164.jpg"], "answer": "B", "solution": "null", "level": "二年级", "question": "64 根火柴棒可以拼成 ( )\n\n", "options": "A. 7\nB. 8\nC. 9", "subject": "组合数学", "analysis": "B"} {"id": "10450", "image": ["2137.jpg"], "answer": "C", "solution": "null", "level": "二年级", "question": "图中一共有()个长方形。\n\n", "options": "A. 3\nB. 4\nC. 6", "subject": "组合几何学", "analysis": "$\\mathrm{C}$"} {"id": "11463", "image": [], "answer": "A", "solution": "null", "level": "二年级", "question": "小朋友玩滑梯属于( $\\quad$ 现象。", "options": "A. 平移 B. 旋转", "subject": "变换几何", "analysis": "A"} {"id": "11464", "image": ["2415.jpg", "2416.jpg", "2417.jpg"], "answer": "D", "solution": "null", "level": "二年级", "question": "下列物品中, ( )不是轴对称的。", "options": "A.\n \n C.\n\n\n\n", "subject": "变换几何", "analysis": "D"} {"id": "11465", "image": [], "answer": "A", "solution": "null", "level": "二年级", "question": "红领巾有 ( ) 条对称轴。", "options": "A. 1\n B. 2\n C. 无数", "subject": "立体几何学", "analysis": "A"} {"id": "11182", "image": [], "answer": "A", "solution": "null", "level": "二年级", "question": "小龙、小东和小强三人分别在一、二、三班。小龙是二班的, 小强下课后去三班找小东玩。小强是 $(\\quad)$ 班的。", "options": "A.一 B.二 C.三", "subject": "逻辑题", "analysis": "A"} {"id": "11183", "image": [], "answer": "B", "solution": "null", "level": "二年级", "question": "盘子里有桃子、苹果、雪梨三种水果各一个, 小林、小丽、小南每人只能选择一个。小南说:\n\n“我不吃苹果。” ; 小林说: “我既不吃苹果, 也不吃雪梨。”; 那么小丽吃的是 ( )。", "options": "A.桃子\nB.苹果\nC.雪梨", "subject": "逻辑题", "analysis": "B"} {"id": "21052", "image": ["9518.jpg"], "answer": "C", "solution": "null", "level": "高三", "question": "已知命题 $p:$ 若 $x>y$, 则 $-x<-y$; 命题 $q$ : 若 $x>y$, 则 $x^{2}>y^{2}$, 在命题 (1)p^q;\n\n", "options": "A. (1)(3)\nB. (1)(4)\nC. (2) (3)\nD. (2)(4)", "subject": "逻辑题", "analysis": "答案:C 解析:根据不等式的性质可知, 若 $x>y$, 则 $-x<-y$ 成立, 即 $p$ 为真命题,当 $\\mathrm{x}=1, \\mathrm{y}=-1$ 时, 满足 $\\mathrm{x}>\\mathrm{y}$, 但 $\\mathrm{x}^{2}>\\mathrm{y}^{2}$ 不成立, 即命题 $\\mathrm{q}$ 为假命题,则 (1) $p \\wedge q$ 为假命题; (2) $p v q$ 为真命题; (3) $p \\wedge(\\neg q)$ 为真命题; (4) ( $\\neg \\mathrm{p}) \\mathrm{vq}$为假命题, 故选 C.\n\n2.答案:C 解析: 若 $\\neg(p \\wedge q)$ 为假命题, 则 $p \\wedge q$ 为真命题, 则 $p$ 为真命题, $q$ 为真命题"} {"id": "21056", "image": [], "answer": "C", "solution": "null", "level": "高三", "question": "若ᄀ $(\\mathrm{p} \\wedge \\mathrm{q})$ 为假命题, 则 ( )", "options": "A. $p$ 为真命题, $q$ 为假命题\nB. $p$ 为假命题, $q$ 为假命题\nC. $p$ 为真命题, $q$ 为真命题\nD. $p$ 为假命题, $q$ 为真命题", "subject": "逻辑题", "analysis": "解析: 若 $\neg(p \\wedge q)$ 为假命题, 则 $p \\wedge q$ 为真命题, 则 $p$ 为真命题, $q$ 为真命题"} {"id": "21057", "image": [], "answer": "B", "solution": "null", "level": "高三", "question": "已知命题 “ (pvq) ” 为真, “ $\\neg \\mathrm{p}$ ” 为真, 则 (", "options": "A. $\\mathrm{p}$ 假 $\\mathrm{q}$ 假\nB. $\\mathrm{p}$ 假 $\\mathrm{q}$ 真\nC. $\\mathrm{p}$ 真 $\\mathrm{q}$ 假\nD. $\\mathrm{p}$ 真 $\\mathrm{q}$ 真", "subject": "逻辑题", "analysis": "答案:B.\n\n4.答案:A 解析: : “pvq 为真命题, 则 $p 、 q$ 中只要有一个命题为真命题即可, $p \\wedge q$为真命题, 则需两个命题都为真命题, $\\therefore \\mathrm{p} \\vee \\mathrm{q}$ 为真命题不能推出 $\\mathrm{p} \\wedge \\mathrm{q}$ 为真命题, 而 $p \\wedge q$ 为真命题能推出 $p \\vee q$ 为真命题: “ $p \\wedge q$ 是真命题” 是 “ $p \\vee q$ 是真命题” 的充分不必要条件,\n\n5.答案:C 解析: 命题 p: 若 $x>y$, 则 $-x<-y$, 为真命题; 命题 $q$ : 若 $x$ $\\mathrm{y}^{2}$, 为假命题, $\\therefore(1) \\mathrm{p} \\wedge \\mathrm{q}$ 为假命题; (2) pvq 为真命题; (3) $\\wedge \\wedge(\\neg \\mathrm{q})$ 为真命题;\n\n(4) ( $\\neg \\mathrm{p}) \\mathrm{vq}$ 为假命题故选: C.\n\n6.答案:C 解析: 对于命题 $p: 4+2=5$, 是假命题; 对于 $q: 3 \\geq 2$, 是真命题. $\\therefore \\mathrm{p} q \\mathrm{q}$ 为真命题, $\\mathrm{p} \\wedge \\mathrm{q}$ 是假命题, $\\rightarrow \\mathrm{p}$ 为真命题, $\\neg \\mathrm{q}$ 为假命题. $\\therefore \\mathrm{C}$ 是假命题 7.B 解析: 命题 p: $\\because \\sqrt{3}<2<\\mathrm{e}, \\therefore \\sqrt{3}>\\mathrm{e}$, 是假命题; 命题 q: $\\because \\Delta=1-4=-3<0$, $\\therefore$ 方程 $\\mathrm{x}^{2}-\\mathrm{x}+1=0$ 无实根,是真命题。 $\\therefore \\mathrm{p}$ 假 $\\mathrm{q}$ 真,故选:B.\n\n8.答案:B 解析: 命题 $p: 1 \\in Q$ 为真命题; 函数 $f(x)=\\frac{1}{\\sqrt{x-1}}$ 的定义域是 $x-1>0$, 即 $x>1$, $\\therefore$ 命题 $\\mathrm{q}$ 为假命题, $\\therefore$ 仅有 $\\mathrm{B}$ 选项 $\\mathrm{pvq}$ 为真命题, 故选 $\\mathrm{B}$.\n\n9.答案:C 解析: 命题 “ $(\\neg \\mathrm{p}) \\vee(\\neg \\mathrm{q})$ ” 是假命题, $\\therefore$ 命题 $(\\neg \\mathrm{p})$ 与 $(\\neg \\mathrm{q})$ 都是假命题, $\\therefore$ 命题 $\\mathrm{p}, \\mathrm{q}$ 都为真命题. 给出下列四个结论:可得命题 “ $\\mathrm{p} \\wedge \\mathrm{q}$ ”\n是真命题; 命题 “ $\\mathrm{pvq}$ ” 是真命题. 其中正确的结论为(1)(4). 故选: C.\n\n10.答案: \\quad 1,2$.\n\n解析: 由命题 $p:|x-1| \\geqslant 2$, 得到命题 $P: x-1 \\geqslant 2$ 或 $x-1 \\leqslant-2$, 即命题 $P: x \\geqslant 3$ 或 $x \\leqslant-1$; $\\because \\neg q$ 为假命题, $\\therefore$ 命题 $q: x \\in Z$ 为真命题, 再由 “ $p$ 且 $q$ ” 为假命题, 知命题 $P: x \\geqslant 3$或 $x \\leqslant-1$ 是假命题, 故 $-10 \\\\ 1-4 a^{2}<0\\end{array}\\right.$, 解析得 $a>\\frac{1}{2}$. 要 “ $p$ 且 $q$ ” 为真命题, 则 $p$ 与 $q$ 都是真命题, 所以 $a$ 的取值范围是 $\\frac{1}{2}<$ $a<2$. 故答案为 $\\left(\\frac{1}{2}, 2\\right)$.\n\n12. 答案: 解析: (1) 若 $\\mathrm{p}$ 为真命题, 则应有 $\\Delta=8-4 \\mathrm{~m}>0$, 解析得 $\\mathrm{m}<2$.\n\n(2) 若 $q$ 为真命题, 则有 $m+1<2$, 即 $m<1$, 因为 $p \\vee q$ 为真命题, $p \\wedge q$ 为假命题, 则 $\\mathrm{p}, \\mathrm{q}$ 应一真一假.\n\n(1)当 $\\mathrm{p}$ 真 $\\mathrm{q}$ 假时, 有 $\\left\\{\\begin{array}{l}\\mathrm{m}<2 \\\\ \\mathrm{~m} \\geq 1\\end{array}\\right.$, 得 $1 \\leq \\mathrm{m}<2$;\n\n(2) 当 $\\mathrm{p}$ 假 $\\mathrm{q}$ 真时, 有 $\\left\\{\\begin{array}{l}\\mathrm{m} \\geq 2 \\\\ \\mathrm{~m}<1\\end{array}\\right.$, 无解析. 综上, $\\mathrm{m}$ 的取值范围是 $[1,2)$."} {"id": "21058", "image": [], "answer": "A", "solution": "null", "level": "高三", "question": "“ $\\mathrm{p} \\Wedge \\mathrm{q}$ 是真命题” 是 “ $\\mathrm{p} \\vee \\mathrm{q}$ 是真命题” 的 ( )", "options": "A. 充分不必要条件\nB. 必要不充分条件\nC. 充要条件\nD. 既不充分也不必要条件", "subject": "逻辑题", "analysis": "“pvq 为真命题, 则 $p 、 q$ 中只要有一个命题为真命题即可, $p \\wedge q$为真命题, 则需两个命题都为真命题, $\therefore \\mathrm{p} \\vee \\mathrm{q}$ 为真命题不能推出 $\\mathrm{p} \\Wedge \\mathrm{q}$ 为真命题, 而 $p \\Wedge q$ 为真命题能推出 $p \\vee q$ 为真命题: “ $p \\Wedge q$ 是真命题” 是 “ $p \\vee q$ 是真命题” 的充分不必要条件,"} {"id": "21059", "image": [], "answer": "C", "solution": "null", "level": "高三", "question": "已知命题 $p$ : 若 $x>y$, 则 $-x<-y$; 命题 $q$ : 若 $xy^{2}$, 在命题 (1) $p \\wedge q$; (2) pvq; (3) p $\\wedge(\\neg \\mathrm{q})$; (4) ( $\\neg \\mathrm{p}) \\vee \\mathrm{qq}$ 中, 真命题是 ( )", "options": "A. (1)(3)\nB. (1) (4)\nC. (2)(3)\nD. (2) (4)", "subject": "逻辑题", "analysis": "命题 p: 若 $x>y$, 则 $-x<-y$, 为真命题; 命题 $q$ : 若 $x$ $\\mathrm{y}^{2}$, 为假命题, $\therefore(1) \\mathrm{p} \\wedge \\mathrm{q}$ 为假命题; (2) pvq 为真命题; (3) $\\wedge \\wedge(\neg \\mathrm{q})$ 为真命题;"} {"id": "21060", "image": [], "answer": "C", "solution": "null", "level": "高三", "question": "已知 $\\mathrm{p}: 4+2=5, \\mathrm{q}: 3 \\geq 2$, 则下列判断中, 错误的是 ( )", "options": "A. $\\mathrm{p}$ 或 $\\mathrm{q}$ 为真,非 $\\mathrm{q}$ 为假\nB. $p$ 或 $q$ 为真,非 $p$ 为真\nC. $\\mathrm{p}$ 且 $\\mathrm{q}$ 为假,非 $\\mathrm{p}$ 为假\nD. $p$ 且 $q$ 为假, $p$ 或 $q$ 为真", "subject": "逻辑题", "analysis": "对于命题 $p: 4+2=5$, 是假命题; 对于 $q: 3 \\geq 2$, 是真命题. $\therefore \\mathrm{p} q \\mathrm{q}$ 为真命题, $\\mathrm{p} \\wedge \\mathrm{q}$ 是假命题, $\rightarrow \\mathrm{p}$ 为真命题, $\neg \\mathrm{q}$ 为假命题. $\therefore \\mathrm{C}$ 是假命题 7.B 解析: 命题 p: $\because \\sqrt{3}<2<\\mathrm{e}, \therefore \\sqrt{3}>\\mathrm{e}$, 是假命题; 命题 q: $\because \\Delta=1-4=-3<0$, $\therefore$ 方程 $\\mathrm{x}^{2}-\\mathrm{x}+1=0$ 无实根,是真命题。 $\therefore \\mathrm{p}$ 假 $\\mathrm{q}$ 真,故选:B."} {"id": "21061", "image": [], "answer": "B", "solution": "null", "level": "高三", "question": "已知命题 $\\mathrm{p}: 1 \\in \\mathrm{Q}$, 命题 $\\mathrm{q}$ : 函数 $\\mathrm{f}(\\mathrm{x})=\\frac{1}{\\sqrt{\\mathrm{x}-1}}$ 的定义域是 $[1,+\\infty)$, 则以下为真命题的是 ( )", "options": "A. $\\mathrm{p} \\wedge \\mathrm{q}$\nB. $\\mathrm{p} \\vee \\mathrm{q}$\nC. $\\neg \\mathrm{p} \\wedge \\mathrm{q}$\nD. $\\neg \\mathrm{p} \\vee \\mathrm{q}$", "subject": "逻辑题", "analysis": "命题 $p: 1 \\in Q$ 为真命题; 函数 $f(x)=\frac{1}{\\sqrt{x-1}}$ 的定义域是 $x-1>0$, 即 $x>1$, $\therefore$ 命题 $\\mathrm{q}$ 为假命题, $\therefore$ 仅有 $\\mathrm{B}$ 选项 $\\mathrm{pvq}$ 为真命题, 故选 $\\mathrm{B}$."} {"id": "21062", "image": [], "answer": "C", "solution": "null", "level": "高三", "question": "已知命题 “ ( $\\neg \\mathrm{p}) \\vee(\\neg \\mathrm{q})$ ” 是假命题, 给出下列四个结论:\n(1)命题 “ $\\mathrm{p} \\wedge \\mathrm{q}$ ” 是真命题;\n(2)命题 “ $\\mathrm{p} \\wedge \\mathrm{q}$ ” 是假命题;\n\n(3)命题 “ pvq” 是假命题;\n\n(4)命题 “ $\\mathrm{pVq}$ ” 是真命题.其中正确的结论为()", "options": "A. (1)(3)\nB. (2) (3)\nC. (1)(4)\nD. (2)(4)", "subject": "逻辑题", "analysis": "命题 “ $(\neg \\mathrm{p}) \\vee(\neg \\mathrm{q})$ ” 是假命题, $\therefore$ 命题 $(\neg \\mathrm{p})$ 与 $(\neg \\mathrm{q})$ 都是假命题, $\therefore$ 命题 $\\mathrm{p}, \\mathrm{q}$ 都为真命题. 给出下列四个结论:可得命题 “ $\\mathrm{p} \\wedge \\mathrm{q}$ ”是真命题; 命题 “ $\\mathrm{pvq}$ ” 是真命题. 其中正确的结论为(1)(4). 故选: C."} {"id": "21063", "image": [], "answer": "A", "solution": "null", "level": "高三", "question": "命题 p: $\\forall \\mathrm{x}>0, \\mathrm{x}^{2}-2 \\mathrm{x}+1>0$; 命题 $\\mathrm{q}: \\exists \\mathrm{x}_{0}>0, \\mathrm{x}_{0}{ }^{2}-2 \\mathrm{x}_{0}+1 \\leq 0$, 下列选项真命题的是 ( )", "options": "A. $\\neg \\mathrm{p} \\wedge \\mathrm{q}$\nB. $p \\wedge q$\nC. $p \\vee \\neg q$\nD. $\\neg p \\wedge \\neg q$", "subject": "逻辑题", "analysis": "因为 $\\mathrm{x}=1$ 时不成立, 故命题 $\\mathrm{p}: \forall \\mathrm{x}>0, \\mathrm{x}^{2}-2 \\mathrm{x}+1>0$ 是假命题;命题 q: $\\exists x_{0}>0, x_{0}{ }^{2}-2 x_{0}+1 \\leq 0$, 当 $x_{0}=1$ 时, 命题成立, 所以是真命题.所以 $\neg \\mathrm{p} \\wedge \\mathrm{q}$ 是真命题; $\\mathrm{p} \\wedge \\mathrm{q}$ 是假命题; $\\mathrm{p} \\vee \neg \\mathrm{q}$ 是假命题; $\neg \\mathrm{p} \\wedge \neg \\mathrm{q}$ 是假命题;故选 A."} {"id": "21064", "image": [], "answer": "A", "solution": "null", "level": "高三", "question": "已知命题 p: $\\exists m \\in R$, 使 $f(x)=x^{2}+m x$ 是偶函数; $q:$ 若 $a\\frac{1}{b}$,下列为真命题 的是 ( )", "options": "A. $\\mathrm{p} \\wedge \\neg \\mathrm{q}$\nB. $\\neg \\mathrm{p} \\wedge \\mathrm{q}$\nC. $\\mathrm{p} \\wedge \\mathrm{q}$\nD. $\\neg \\mathrm{p} \\wedge \\neg \\mathrm{q}$", "subject": "逻辑题", "analysis": "命题 p: $\\exists m=0 \\in R$, 使 $\\mathrm{f}(\\mathrm{x})=\\mathrm{x}^{2}+\\mathrm{mx}$ 是偶函数, 因此为真命题;$\\mathrm{q}$ : 若 $\\mathrm{a}<\\mathrm{b}$, 则 $\frac{1}{\\mathrm{a}}>\frac{1}{\\mathrm{~b}}$, 为假命题, 例如取 $\\mathrm{a}=-1, \\mathrm{~b}=2$. 下列为真命题 的是 $\\mathrm{p} \\wedge$ ( $\neg \\mathrm{q})$ ,其余为假命题. 故选:A."} {"id": "21068", "image": [], "answer": "C", "solution": "null", "level": "高三", "question": "下列命题中错误的是 ( )", "options": "A. 若命题 $\\mathrm{p}$ 为真命题,命题 $\\mathrm{q}$ 为假命题, 则命题 “ $\\mathrm{p} v(\\neg \\mathrm{q})$ ” 为真命题\n\nB. 命题 “若 $a+b \\neq 7$, 则 $a \\neq 2$ 或 $b \\neq 5$ ” 为真命题\n\nC. 命题 “若 $x^{2}-x=0$, 则 $x=0$ 或 $x=1$ ” 的否命题为 “若 $x^{2}-x=0$, 则 $x \\neq 0$ 且 $x \\neq 1$ \"\n\nD. 命题 $p: ~ \\exists x>0, \\sin x>2^{x}-1$, 则 $\\neg p$ 为 $\\forall x>0, \\sin x \\leq 2^{x}-1$", "subject": "逻辑题", "analysis": "A、若 $\\mathrm{q}$ 为假, 则 $\neg \\mathrm{q}$ 为真, 故 $\\mathrm{pV}(\neg \\mathrm{q})$ 为真, 故 A 正确;$B$ 、命题的逆否命题为: 若 $a=2$ 且 $b=5$, 则 $a+b=7$, 显然正确, 故原命题正确, 故 B 正确;C、命题 “若 $x^{2}-x=0$, 则 $x=0$ 或 $x=1$ ” 的否命题应为 “若 $x^{2}-x \neq 0$ 则 $x \neq 0$ 且 $x \neq 1$ ”,故 C 错误;D、根据含有一个量词的命题的否定易得 D 正确.综上可得:错误的为 C. 故选: C."} {"id": "21069", "image": [], "answer": "B", "solution": "null", "level": "高三", "question": "已知命题 $p: \\forall x>0$, 总有 $(x+1) e^{x}>1$, 则 $7 p$ 为", "options": "A. $\\exists x_{0} \\leq 0$, 使得 $\\left(x_{0}+1\\right) \\mathrm{e}^{x_{0}} \\leq 1$\nB. $\\exists x_{0}>0$, 使得 $\\left(x_{0}+1\\right) \\mathrm{e}^{x_{0}} \\leq 1$\nC. $\\forall x>0$,总有 $(x+1) \\mathrm{e}^{\\mathrm{x}} \\leq 1$\nD. $\\forall \\mathrm{x} \\leq 0$,总有 $(\\mathrm{x}+1) \\mathrm{e}^{\\mathrm{x}} \\leq 1$", "subject": "逻辑题", "analysis": "根据全称命题的否定为特称命题可知, $\rightarrow \\mathrm{p}$ 为 $\\exists \\mathrm{x}_{0}>0$, 使得 $\\left(\\mathrm{x}_{0}+1\right)$ $\\mathrm{e}^{\\mathrm{x}_{0}} \\leq 1$, 故选: $\\mathrm{B}$."} {"id": "21070", "image": [], "answer": "C", "solution": "null", "level": "高三", "question": "已知下列四个命题: (1) “若 $x^{2}-2=0$, 则 $x=0$ 或 $x=1$ ” 的逆否命题为 “若 $x \\neq 0$且 $x \\neq 1$, 则 $x^{2}-x \\neq 0$ ” ; (2) “ $x<1$ ” 是 “ $x^{2}-3 x+2>0$ ” 的充分不必要条件; (3)命题 $\\mathrm{p}: \\exists \\mathrm{x}_{0} \\in \\mathrm{R}$, 使得 $\\mathrm{x}_{0}{ }^{2}+\\mathrm{x}_{0}+1<0$, 则 $\\neg \\mathrm{p}: \\forall \\mathrm{x} \\in \\mathrm{R}$, 都有 $\\mathrm{x}^{2}+\\mathrm{x}+1 \\geq 0$; (4)若 $\\mathrm{p} \\wedge \\mathrm{q}$ 为假命题, 则 $\\mathrm{p}, \\mathrm{q}$ 均为假命题. 其中真命题个数为 $(\\quad)$", "options": "A. 1\nB. 2\nC. 3\nD. 4", "subject": "逻辑题", "analysis": "(1)若 $x 2-2=0$, 则 $x=0$ 或 $x=1$ ” 的逆否命题为 “若 $x \neq 0$ 且 $x \neq 1$, 则 $x 2$ $-2 \neq 0$ ” 正确;(2) $x^{2}-3 x+2>0$ 可得 $x<1$ 或 $x>2$, 故 “ $x<1$ ” 是 “ $x 2-3 x+2>0$ ” 的充分不必要条件, 正确;(3)命题 $p$ : 存在 $x 0 \\in R$, 使得 $x_{0}{ }^{2}+x_{0}+1<0$, 则 $\neg p$ : 任意 $x \\in R$, 都有 $x 2+$ $x+1 \\geq 0$, 正确;(4)若 $\\mathrm{p} \\wedge \\mathrm{q}$ 为假命题, 则 $\\mathrm{p}, \\mathrm{q}$ 至少一个为假命题, 故(4)错误.故选 C."} {"id": "21072", "image": [], "answer": "C", "solution": "null", "level": "高三", "question": "已知命题 $\\mathrm{p}: \\exists x \\in R$, 使得 $\\mathrm{x}^{2}-x+2<0$; 命题 $\\mathrm{q}: \\forall \\mathrm{x} \\in[1,2]$, 使得 $\\mathrm{x}^{2} \\geq 1$. 以下命题为真命题的是 ( )", "options": "A. $\\neg \\mathrm{p} \\wedge \\neg \\mathrm{q}$\nB. $p \\vee \\neg q$\nC. $\\neg \\mathrm{p} \\wedge \\mathrm{q}$\nD. $\\mathrm{p} \\wedge \\mathrm{q}$", "subject": "逻辑题", "analysis": "$\because$ 判别式 $\triangle=1-4 \times 2=1-7=-6<0, \therefore \forall x \\in R$, 使得 $x^{2}-x+2>0$; 即命题 $p$ : $\\exists x \\in R$, 使得 $x^{2}-x+2<0$ 为假命题, 当 $x \\in[1,2]$ 时, $x^{2} \\geq 1$ 恒成立, 即命题 $q$是真命题, 则 $\neg \\mathrm{p} \\wedge \\mathrm{q}$ 是真命题, 其余为假命题, 故选 C."} {"id": "21073", "image": [], "answer": "C", "solution": "null", "level": "高三", "question": "下列有三种说法:\n\n(1)命题 “ $\\exists x \\in R, x^{2}+1>3 x$ ” 的否定是 “ $\\forall x \\in R, x^{2}+1<3 x$ ” ;\n\n(2) 已知 $\\mathrm{p} 、 \\mathrm{q}$ 为两个命题, 若 $\\mathrm{p} v \\mathrm{q}$ 为假命题, 则 $(\\neg \\mathrm{p}) \\wedge(\\neg \\mathrm{q})$ 为真命题;\n\n(3)命题 “若 $x y=0$, 则 $x=0$ 且 $y=0$ ” 为真命题. 其中正确的个数为 ( )", "options": "A. 3 个\nB. 2 个\nC. 1 个\nD. 0 个", "subject": "逻辑题", "analysis": ""} {"id": "21074", "image": [], "answer": "C", "solution": "null", "level": "高三", "question": "下列结论错误的是 ( )", "options": "A. 命题 “若 $p$, 则 $q$ ” 与命题 “若 $\\neg q$, 则 $\\neg p$ ” 互为逆否命题\n\nB. 命题 p: $\\forall x \\in[0,1], e^{x} \\geq 1$, 命题 $q: \\exists x \\in \\mathbf{R}, x^{2}+x+1<0$, 则 “ $p \\vee q$ ”为真\n\nC. “若 $a m^{2}<\\mathrm{bm}^{2}$, 则 $\\mathrm{a}<\\mathrm{b}$ ” 的逆命题为真命题\n\nD. 命题 $P$ : “ $\\exists x_{0} \\in \\mathbf{R}$, 使得 $x_{0}{ }^{2}-2 \\geq 0$ ” 的否定为 $\\neg P$ : “ $\\forall x \\in \\mathbf{R}, x^{2}-$ $2<0$", "subject": "逻辑题", "analysis": "命题 “若 $\\mathrm{p}$ 则 $\\mathrm{q}$ ” 与命题 “若 $\neg \\mathrm{q}$ 则 $\\mathrm{p}$ ” 互为逆否命题, 故 A 正确;命题 $p: \forall x \\in[0,1], \\mathrm{e}^{\\mathrm{x}} \\geqslant 1$, 由 $\\mathrm{e}^{\\mathrm{x}} \\in[1, \\mathrm{e}]$, 可得 $\\mathrm{p}$ 真; 命题 $q: \\exists x \\in R, \\mathrm{x}^{2}+\\mathrm{x}+1<$ 0 , 由于 $x^{2}+x+1=\\left(x+\frac{1}{2}\right)^{2}+\frac{3}{4} \\geqslant \frac{3}{4}$, 则 $q$ 假, 则 “ $p \\vee q$ ” 为真, 故 B 正确; “若 $\\mathrm{am}^{2}<\\mathrm{bm}^{2}$, 则 $\\mathrm{a}<\\mathrm{b}$ ” 的逆命题为 “若 $\\mathrm{a}<\\mathrm{b}$, 则 $\\mathrm{am}^{2}<\\mathrm{bm}^{2}$ ” 错误,如果 $\\mathrm{m}=0$, 不成立, 故 C 不正确;命题 P: “ $\\exists x_{0} \\in R$, 使得 $x_{0}{ }^{2}-2 \\geq 0$ ” 的否定为 $P \\mathrm{P}:$ “ $\forall x \\in R, x^{2}-2<0$ ”,故 D 正确. 故选 C."} {"id": "21075", "image": [], "answer": "C", "solution": "null", "level": "高三", "question": "已知命题 p: “ $\\forall \\mathrm{a}>\\mathrm{b},|\\mathrm{a}|>|\\mathrm{b}|$ ” , 命题 $\\mathrm{q}$ : “ $\\exists \\mathrm{x}_{0}<0,2^{\\mathrm{x}_{0}}>0$ ”, 则下列为真命题的是", "options": "A. $\\mathrm{p} \\wedge \\mathrm{q}$\nB. $\\neg \\mathrm{p} \\wedge \\neg \\mathrm{q}$\nC. $\\mathrm{p} \\vee \\mathrm{q}$\nD. $\\mathrm{p} \\vee \\neg \\mathrm{q}$", "subject": "逻辑题", "analysis": "命题 $\\mathrm{p}$ : “ $\forall \\mathrm{a}>\\mathrm{b},|\\mathrm{a}|>|\\mathrm{b}|$ ” 是假命题,"} {"id": "21076", "image": [], "answer": "D", "solution": "null", "level": "高三", "question": "当 $m \\in N^{*}$, 命题 “若 $m>0$, 则方程 $x^{2}+x-m=0$ 有实根” 的逆否命题是 ( )", "options": "A. 若方程 $x^{2}+x-m=0$ 有实根, 则 $m>0$\n\nB. 若方程 $x^{2}+x-m=0$ 有实根, 则 $m \\leq 0$\n\nC. 若方程 $x^{2}+x-m=0$ 没有实根, 则 $m>0$\n\nD. 若方程 $x^{2}+x-m=0$ 没有实根, 则 $m \\leq 0$", "subject": "逻辑题", "analysis": "由逆否命题的定义可知:当 $m \\in N^{*}$, 命题 “若 $m>0$,则方程 $x^{2}+x-m=0$ 有实根” 的逆否命题是: 若方程 $x^{2}+x-m=0$ 没有实根, 则 $m \\leq 0$,故选 D."} {"id": "21077", "image": [], "answer": "B", "solution": "null", "level": "高三", "question": "一个命题与它的逆命题、否命题、逆否命题这四个命题中, 真命题的个数 ( )", "options": "A. 一定是奇数\nB. 一定是偶数\nC. 可能是奇数也可能是偶数\nD. 上述判断都不正确", "subject": "逻辑题", "analysis": "根据四种命题及其关系理论:原命题 $\\Leftrightarrow$ 逆否命题, 逆命题 $\\Leftrightarrow$ 否命题如果原命题是真命题, 逆命题是假命题, 则真命题共有两个;如果原命题是真命题, 逆命题也是真命题, 则真命题共有四个;如果原命题是假命题, 逆命题也是假命题, 则真命题共有 0 个;故一个命题与它的逆命题、否命题、逆否命题这四个命题中, 真命题的个数一定是偶数, 故选: B"} {"id": "21081", "image": [], "answer": "D", "solution": "null", "level": "高三", "question": "命题 “若 $x^{2} \\neq 4$, 则 $x \\neq 2$ 且 $x \\neq-2$ ” 的否命题为 ( )", "options": "A. 若 $x^{2}=4$,则 $x \\neq 2$ 且 $x \\neq-2$\nB. 若 $x^{2} \\neq 4$,则 $x=2$ 且 $x=-2$\nC. 若 $x^{2} \\neq 4$,则 $x=2$ 或 $x=-2$\nD. 若 $x^{2}=4$,则 $x=2$ 或 $x=-2$", "subject": "逻辑题", "analysis": "否命题, 条件和结论不等号变等号, 且变成或, “若 $x^{2} \neq 4$, 则 $x \neq 2$且 $x \neq-2$ ” 的否命题是: “若 $x^{2}=4$, 则 $x=2$ 或 $x=-2$ ” , 故选 D."} {"id": "21082", "image": [], "answer": "B", "solution": "null", "level": "高三", "question": "下列命题:\n\n(1) “若 $\\mathrm{a} \\leq \\mathrm{b}$, 则 $\\mathrm{a}<\\mathrm{b}$ ” 的否命题;\n\n(2) “若 $a=1$, 则 $a x^{2}-x+3 \\geq 0$ 的解集为 $R$ ” 的逆否命题;\n\n(3) “周长相同的圆面积相等” 的逆命题;\n\n(4) “若 $\\sqrt{2} x$ 为有理数, 则 $x$ 为无理数” 的逆否命题.\n\n其中真命题序号为 ( )", "options": "A. (2) (4)\nB. (1)(2)(3)\nC. (2) (3) (4)\nD. (1)(2)(3)(4)", "subject": "逻辑题", "analysis": "对于(1), 逆命题为真, 故否命题为真; 对于(2), “若 $\\mathrm{a}=1$, 则 $a x^{2}-\\mathrm{x}+3 \\geq 0$的解集为 $\\mathrm{R}$ ” 原命题为真, 故逆否命题为真; 对于(3), “周长相同的圆面积相等” 的逆命题 “面积相等的圆周长相同” 为真; 对于(4), “若 $\\sqrt{2} \\mathrm{x}$ 为有理数, 则 $\\mathrm{x}$ 为 0 或无理数”, 故原命题为假, 逆否命题为假. 故选 B."} {"id": "21083", "image": [], "answer": "C", "solution": "null", "level": "高三", "question": "原命题: “设 $a, b, c \\in R$, 若 $a>b$, 则 $a c^{2}>b c^{2}$ ”, 在原命题以及它的逆命题、否命题、逆否命题中, 真命题的个数为 ( )", "options": "A. 0\nB. 1\nC. 2\nD. 4", "subject": "逻辑题", "analysis": "逆命题: 设 $\\mathrm{a}, \\mathrm{b}, \\mathrm{c} \\in \\mathrm{R}$, 若 $\\mathrm{ac}^{2}>\\mathrm{bc}^{2}$, 则 $\\mathrm{a}>\\mathrm{b}$;$\because$ 由 $\\mathrm{ac}^{2}>\\mathrm{bc}^{2}$ 可得 $\\mathrm{c}^{2}>0, \therefore$ 能得到 $\\mathrm{a}>\\mathrm{b}$, 所以该命题为真命题;否命题: 设 $a, b, c \\in R$, 若 $a \\leq b$, 则 $a c^{2} \\leq b c^{2} ; \because c^{2} \\geq 0, \therefore$ 由 $a \\leq b$ 可以得到 $a c^{2} \\leq b c^{2}$,所以该命题为真命题; 因为原命题和它的逆否命题具有相同的真假性, 所以只需判断原命题的真假即可; $\because c^{2}=0$ 时, $a c^{2}=b c^{2}$, 所以由 $\\mathrm{a}>b$ 得到 $a c^{2} \\geq b c^{2}$,所以原命题为假命题, 即它的逆否命题为假命题; $\therefore$ 为真命题的有 2 个. 故选 C."} {"id": "21084", "image": [], "answer": "A", "solution": "null", "level": "高三", "question": "给出下列命题:\n\n(1)命题 “若 $b^{2}-4 a c<0$, 则方程 $a x^{2}+b x+c=0 \\quad(a \\neq 0)$ 无实根” 的否命题;\n\n(2)命题 “在 $\\triangle A B C$ 中, $A B=B C=C A$, 那么 $\\triangle A B C$ 为等边三角形” 的逆命题;\n\n(3)命题 “若 $a>b>0$, 则 $\\sqrt[3]{a}>\\sqrt[3]{b}>0$ ” 的逆否命题;\n\n(4) “若 $m \\geq 1$, 则 $m x^{2}-2(m+1) x+(m+3)>0$ 的解集为 $R$ ” 的逆命题.其中真命题的序号为 ( )", "options": "A. (1)(2)(3)\nB. (1)(2)(4)\nC. (2) (4)\nD. (1)(2)(3)(4)", "subject": "逻辑题", "analysis": "1)命题 “若 $b^{2}-4 a c<0$, 则方程 $a x^{2}+b x+c=0 \\quad(a \neq 0)$ 无实根” 的否命题是 “若 $b^{2}-4 a c \\geq 0$ ,则方程 $a x^{2}+b x+c=0 \\quad(a \neq 0)$ 有实根” ,是正确的,(2)命题 “ $\triangle A B C$ 中, $A B=B C=C A$, 那么 $\triangle A B C$ 为等边三角形” 的逆命题是 “ $\triangle A B C$ 是等边三角形, 则 $A B=B C=C A$ ” , 是正确的,(3)命题 “若 $a>b>0$, 则 $\\sqrt[3]{a}>\\sqrt[3]{b}>0$ ” 是正确的, 所以原命题的逆否命题也是正确的,(4) 命题 “若 $m \\geq 1$, 则 $m x^{2}-2(m+1) x+(m+3)>0$ 的解集为 $R$ ” 的逆命题是 “若 $m x^{2}-2(m+1) x+(m+3)>0$ 的解集为 $R$, 则 $m \\geq 1$ ”,若不等式的解集为 $R$, 所以 $\\left\\{\begin{array}{l}m>0 \\ 4(m+1)^{2}-4 m(m+3)<0\\end{array}\right.$, 解得 $m>1$, 所以逆命题是错误的,所以真命题的序号为(1)(2)(3). 故选 A."} {"id": "21085", "image": [], "answer": "B", "solution": "null", "level": "高三", "question": "命题 “若 $a^{2}+b^{2}=0$, 则 $a, b$ 都为零” 的否命题是 ( )", "options": "A. 若 $a^{2}+b^{2} \\neq 0$, 则 $a, b$ 都不为零\nB. 若 $a^{2}+b^{2} \\neq 0$, 则 $a, b$ 不都为零\nC. 若 $a, b$ 都不为零, 则 $a^{2}+b^{2} \\neq 0$\nD. 若 $a, b$ 不都为零, 则 $a^{2}+b^{2} \\neq 0$", "subject": "逻辑题", "analysis": "命题 “若 $a^{2}+b^{2}=0$, 则 $a$, $b$ 都为零” 的否命题是 “若 $a^{2}+b^{2} \neq 0$, 则 $a, b$不都为零”. 故选 B."} {"id": "21086", "image": [], "answer": "A", "solution": "null", "level": "高三", "question": "已知命题: “若 $x^{2}>y^{2}$, 则 $x>y$ ” 则原命题、逆命题、否命题、逆否命题这四个命题中, 真命题的个数是 ( )", "options": "A. 0\nB. 1\nC. 2\nD. 4", "subject": "逻辑题", "analysis": "由题意, 原命题为: 若 $x^{2}>y^{2}$, 则 $x>y$ ” , 当 $x=-2, y=1$, 不满足, 故为假命题; 逆命题为: 若 $x>y$, 则 $x^{2}>y^{2}$, 当 $x=1, y=-2$ 是不满足, 故为假命题; 因为原命题与逆否命题等价, 故逆否命题为假; 逆命题与否命题等价,故否命题为假. 综上, 真命题的个数为 0 . 故选 A."} {"id": "21087", "image": [], "answer": "A", "solution": "null", "level": "高三", "question": "下列说法正确的是 ( )", "options": "A. 若一个命题的逆命题是真命题, 则它的否命题一定是真命题\n\nB. 若一个命题的逆命题是真命题, 则它的逆否命题一定是真命题\n\nC. 若一个命题的逆命题是真命题,则它的否命题一定是假命题\n\nD. 若一个命题的逆命题是真命题, 则它的逆否命题一定是真命题", "subject": "逻辑题", "analysis": "一个命题的逆命题和它的否命题是互为逆否命题, 它们的真假性相同,所以若一个命题的逆命题是真命题, 则它的否命题一定是真命题. 故选: A."} {"id": "21088", "image": [], "answer": "B", "solution": "null", "level": "高三", "question": "命题 “若 $a>-3$, 则 $a>-6$ ” 以及它的逆命题、否命题、逆否命题中, 假命题的个数为 ( )", "options": "A. 1\nB. 2\nC. 3\nD. 4", "subject": "逻辑题", "analysis": "在命题的四种形式中原命题和逆否命题互为逆否命题, 同真同假, 否命题和逆命题互为逆否命题同真同假.$\because$ 命题 “若 $\\mathrm{a}>-3$, 则 $\\mathrm{a}>-6$ ” 为真命题; 逆命题是假命题, $\therefore$ 命题的逆否命题为真命题,否命题是假命题,故选:B"} {"id": "21094", "image": [], "answer": "C", "solution": "null", "level": "高三", "question": "命题 “若 $x=3$, 则 $x^{2}-9 x+18=0$ ” 的逆命题、否命题和逆否命题中, 假命题的个数为 ( )", "options": "A. 0\nB. 1\nC. 2\nD. 3", "subject": "逻辑题", "analysis": "由题意, 原命题为: 若 $x=3$, 则 $x^{2}-9 x+18=0$, 显然 3 是方程的解, 为真命题;逆命题为: 若 $x^{2}-9 x+18=0$, 则 $x=3$, 因为方程还有另一根为 6 , 故为假命题;因为原命题与逆否命题等价, 故逆否命题为真; 逆命题与否命题等价, 故否命题为假. 故选 C."} {"id": "21095", "image": [], "answer": "D", "solution": "null", "level": "高三", "question": "命题 “若 $x^{2} \\leq 1$, 则 $-1 \\leq x \\leq 1$ ” 的逆否命题是 ( )", "options": "A. 若 $x^{2} \\geq 1$, 则 $x \\geq 1$, 或 $x \\leq-1$\nB. 若 $-11$ 或 $x<-1$, 则 $x^{2}>1$", "subject": "逻辑题", "analysis": "命题 “若 $x^{2} \\leq 1$, 则 $-1 \\leq x \\leq 1$ ” 的逆否命题是 “若 $x<-1$ 或 $x>1$, 则 $x^{2}$ $>1$ ”. 故选: D."} {"id": "21096", "image": [], "answer": "B", "solution": "null", "level": "高三", "question": "下列命题:\n\n(1) “若 $\\mathrm{a} \\leq \\mathrm{b}$, 则 $\\mathrm{a}<\\mathrm{b}$ ” 的否命题;\n\n(2) “若 $a=1$, 则 $a x^{2}-x+3 \\geq 0$ 的解集为 $R$ ” 的逆否命题;\n\n(3) “周长相同的圆面积相等” 的逆命题;\n\n(4) “若 $\\sqrt{2} x$ 为有理数, 则 $x$ 为无理数” 的逆否命题.\n\n其中真命题序号为 ( )", "options": "A. (2) (4)\nB. (1) (2) (3)\nC. (2)(3)(4)\nD. (1)(2)(3)(4)", "subject": "逻辑题", "analysis": "对于(1), 逆命题为真, 故否命题为真;对于 (2) “若 $a=1$, 则 $a x^{2}-x+3 \\geq 0$ 的解集为 $R$ ” 原命题为真, 故逆否命题为真;对于 (3) “面积相等的圆周长相同” 为真;对于 (4) “若 $\\sqrt{2} x$ 为有理数, 则 $x$ 为 0 或无理数” , 故原命题为假, 逆否命题为假."} {"id": "21071", "image": [], "answer": "B", "solution": "null", "level": "高三", "question": "已知条件 $\\mathrm{p}: \\mathrm{k}=\\sqrt{3}$; 条件 $\\mathrm{q}:$ 直线 $y=k x_{+} 2$ 与圆 $\\mathrm{x}^{2}+\\mathrm{y}^{2}=1$ 相切, 则 $\\neg \\mathrm{p}$ 是 $\\neg \\mathrm{q}$的( )", "options": "A. 充分必要条件\nB. 必要不充分条件\nC. 必要不充分条件\nD. 既不充分也不必要条件", "subject": "解析几何", "analysis": "条件 $\\mathrm{q}$ : 直线 $\\mathrm{y}=\\mathrm{kx}+2$ 与圆 $\\mathrm{x}^{2}+\\mathrm{y}^{2}=1$ 相切, 可得: $\frac{2}{\\sqrt{1+\\mathrm{k}^{2}}}=1$, 解得 $\\mathrm{k}= \\pm \\sqrt{3}$. $\therefore \\mathrm{p}$ 是 $\\mathrm{q}$ 的充分不必要条件. 则 $7 \\mathrm{p}$ 是 $7 \\mathrm{q}$ 的必要不充分条件. 故选 B."} {"id": "22976", "image": [], "answer": "A", "solution": "null", "level": "高三", "question": "故选 C.\n\n分析: 本题主要考查了基本不等式在最值问题中的应用, 解决问题的关键是根据基本不等式分析计算即可\n\n10. 设 $a>0, b>0, A(1,-2), B(a,-1), C(-b, 0)$, 若", "options": "A.B.C 三点共线, 则 $\\frac{1}{a}+\\frac{1}{b}$的最小值是 ( )\nA. $3+2 \\sqrt{2}$\nB. $4 \\sqrt{2}$\nC. 6\nD. $\\frac{9}{2}$", "subject": "解析几何", "analysis": "解析: 解答: 由题设知: $\\overrightarrow{A B}=(a-1,1), \\overrightarrow{A C}=(-b-1,2)$\n\n因为 $A . B . C$ 三点共线,所以向量与向量共线,\n\n所以 $2(a-1)+b+1=0 \\Rightarrow 2 a+b=1$\n\n又因为 $a>0, b>0$,\n所以 $\\frac{1}{a}+\\frac{1}{b}=(2 a+b) \\cdot\\left(\\frac{1}{a}+\\frac{1}{b}\\right)=2+\\frac{2 a}{b}+\\frac{b}{a}+1 \\geq 3+2 \\sqrt{\\frac{2 a}{b} \\times \\frac{b}{a}}=3+2 \\sqrt{2}$\n\n当且仅当 $a=\\frac{1}{4}, b=\\frac{1}{2}$ 时等号成立; 所以 $\\frac{1}{a}+\\frac{1}{b}$ 的最小值是 $3+2 \\sqrt{2}$, 故选 $\\mathrm{A}$\n\n分析: 本题主要考查了基本不等式在最值问题中的应用, 解决问题的关键是根据不等式性质分析计算即可"} {"id": "21764", "image": [], "answer": "B", "solution": "null", "level": "高三", "question": "设 $f^{\\prime}\\left(x_{0}\\right)=0$, 则曲线 $y=f(x)$ 在点 $\\left(x_{0}, f\\left(x_{0}\\right)\\right)$ 处的切线", "options": "A. 不存在\nB. 与 $X$ 轴平行或重合\nC. 与 $x$ 轴垂直\nD. 与 $x$ 轴相交但不垂直", "subject": "解析几何", "analysis": "解析:曲线 $y=f(x)$ 在点 $\\left(x_{0}, f\\left(x_{0}\\right)\\right)$ 处的切线斜率为 0 , 切线与 $x$ 轴平行或重合. 故选 B."} {"id": "21766", "image": [], "answer": "B", "solution": "null", "level": "高三", "question": "若曲线 $y=h(x)$ 在点 $P(a, h(a))$ 处的切线方程为 $2 x+y+1=0$, 则", "options": "A. $h^{\\prime}(a)=0$\nB. $h^{\\prime}(a)<0$\nC. $h^{\\prime}(a)>0$\nD. $h^{\\prime}(a)$ 不确定", "subject": "解析几何", "analysis": "解析:由 $2 x+y+1=0$, 得 $y=-2 x-1$, 由导数的几何意义知, $h^{\\prime}(a)=-2<0$. 故选 B."} {"id": "21768", "image": ["9661.jpg"], "answer": "A", "solution": "null", "level": "高三", "question": "已知 $y=f(x)$ 的图象如图所示, 则 $f^{\\prime}\\left(x_{A}\\right)$ 与 $f^{\\prime}\\left(x_{B}\\right)$ 的大小关系是\n\n", "options": "A. $f^{\\prime}\\left(x_{A}\\right)>f^{\\prime}\\left(x_{B}\\right)$\nB. $f^{\\prime}\\left(x_{A}\\right)=f^{\\prime}\\left(x_{B}\\right)$\nC. $f^{\\prime}\\left(x_{A}\\right)k_{B}$, 根据导数的几何意义有 $f^{\\prime}\\left(x_{A}\\right)>f^{\\prime}\\left(x_{B}\\right)$. 故选 A."} {"id": "21769", "image": [], "answer": "C", "solution": "null", "level": "高三", "question": "已知函数 $f(x)=2 x^{2}-4$ 的图象上一点 $(1,-2)$ 及附近一点 $(1+\\Delta x,-2+\\Delta y)$, 则 $\\frac{\\Delta y}{\\Delta x}=(\\quad)$", "options": "A. 4\nB. $4 x$\nC. $4+2 \\Delta x$\nD. $4+2(\\Delta x)^{2}$", "subject": "解析几何", "analysis": ""} {"id": "21770", "image": [], "answer": "B", "solution": "null", "level": "高三", "question": "曲线 $y=-\\frac{1}{x}$ 在点 $\\left(\\frac{1}{2},-2\\right)$ 处的切线方程为", "options": "A. $y=4 x$\nB. $y=4 x-4$\nC. $y=4 x+4$\nD. $y=2 x+4$", "subject": "解析几何", "analysis": "解析:由题可得 $\\Delta y=-\\frac{1}{x+\\Delta x}+\\frac{1}{x}=\\frac{\\Delta x}{x(x+\\Delta x)}$, 所以 $\\frac{\\Delta y}{\\Delta x}=\\frac{1}{x(x+\\Delta x)}$, 所以 $y^{\\prime}=\\lim _{\\Delta x \\rightarrow 0} \\frac{\\Delta y}{\\Delta x}=\\frac{1}{x^{2}}$, 所以切线的斜率 $k=\\left.y^{\\prime}\\right|_{x=\\frac{1}{2}}=4$, 故所求切线方程为 $y+2=4\\left(x-\\frac{1}{2}\\right)$, 即 $y=4 x-4$. 故选 B."} {"id": "21771", "image": [], "answer": "C", "solution": "null", "level": "高三", "question": "已知曲线 $y=\\frac{4}{x}$ 在点 $P(1,4)$ 处的切线与直线 $l$ 平行且距离为 $\\sqrt{17}$, 则直线 $l$ 的方程为", "options": "A. $4 x-y+9=0$\nB. $4 x-y+9=0$ 或 $4 x-y+25=0$\nC. $4 x+y+9=0$ 或 $4 x+y-25=0$\nD. 以上均不对", "subject": "解析几何", "analysis": "解析:由题可得 $y^{\\prime}=\\lim _{\\Delta x \\rightarrow 0} \\frac{\\Delta y}{\\Delta x}=-\\frac{4}{x^{2}}$, 点 $P(1,4)$ 在曲线 $y=\\frac{4}{x}$ 上, 所以切线的斜率 $k=\\left.y^{\\prime}\\right|_{x=1}=-4$, 故切线方程为 $y-4=-4(x-1)$, 即 $4 x+y-8=0$, 设 $l: 4 x+y+c=0$, 由题意可得 $\\sqrt{17}=\\frac{|c+8|}{\\sqrt{4^{2}+1^{2}}}$, 解得 $c=9$ 或 -25 , 故选 C."} {"id": "22506", "image": [], "answer": "A", "solution": "null", "level": "高三", "question": "梯形的中位线长为 $15 \\mathrm{~cm}$,一条对角线把中位线分成 $3: 2$ 两段,那么梯形的两底长分别为 ( )", "options": "A. $12 \\mathrm{~cm}$ 和 $18 \\mathrm{~cm}$\nB. $20 \\mathrm{~cm}$ 和 $10 \\mathrm{~cm}$\nC. $14 \\mathrm{~cm}$ 和 $16 \\mathrm{~cm}$\nD. $6 \\mathrm{~cm}$ 和 $9 \\mathrm{~cm}$", "subject": "解析几何", "analysis": "解析: 解答: 如图, 不妨设 $M P: P N=2: 3$, 则 $M P=6 \\mathrm{~cm}, P N=9 \\mathrm{~cm} . \\because M$ 为梯形 $A B C D$ 的中位线, $\\therefore \\mathrm{MN} / / \\mathrm{AD} . \\therefore$ 在 $\\triangle \\mathrm{BAD}$ 中, $\\mathrm{MP}$ 为其中位线, $\\therefore \\mathrm{AD}=2 \\mathrm{MP}=12 \\mathrm{~cm}$. 同理可得 $\\mathrm{BC}=2 \\mathrm{PN}=18 \\mathrm{~cm}$.\n\n![](https://cdn.mathpix.com/cropped/2024_04_19_81303e2fc6b69da0f33fg-"} {"id": "22507", "image": [], "answer": "C", "solution": "null", "level": "高三", "question": "在 $\\triangle A B C$ 中, $D, E$ 分别是 $A B, A C$ 边的中点, 且 $B C=8$, 则 $D E$ 等于( )", "options": "A. 1\nB. 2\nC. 4\nD. 8", "subject": "解析几何", "analysis": "解析: 解答: $\\because \\mathrm{DE}$ 是 $\\triangle \\mathrm{ABC}$ 的中位线, $\\therefore \\mathrm{DE}=\\frac{1}{2} \\mathrm{BC}=4$.\n\n分析: 本题主要考查了平行线等分线段定理, 解决问题的关键是根据平行线等分线段定理计算即可."} {"id": "22779", "image": [], "answer": "D", "solution": "null", "level": "高三", "question": "已知等腰 $\\triangle A B C$ 中, $\\angle C=90^{\\circ}, A(-1,0), B(3,2)$, 则点 $C$ 的坐标为 $(\\quad)$", "options": "A. $(3,-3)$\nB. $(0,3)$ 或 $(3,-3)$\nC. $(2,-1)$\nD. $(0,3)$ 或 $(2,-1)$", "subject": "解析几何", "analysis": "解析: 解答: 方法一: 若点 $c(3,-3)$, 则 $\\overrightarrow{A C} \\cdot \\overrightarrow{B C}=(4,-3) \\cdot(0,-5)=15 \\neq 0$, 所以不满足 $A C \\perp B C ,$ 排除选项 $\\mathrm{A}, \\mathrm{B}$;\n\n若点 $C(2,-1)$, 则 $\\overrightarrow{A C} \\cdot \\overrightarrow{B C}=(3,-1) \\cdot(-1,-3)=0$, 所以 $A C \\perp B C$, 又 $|C A|^{2}=|C B|^{2}=10$, 故 $C(2,-1)$ 满足题意, 由于 $A B$ 的中点坐标为 $(1,1)$, 由对称性,得另一点 $C$ 的坐标为 $(0,3)$, 故选 D.\n\n方法二: 设 $C(x, y)$, 由 $A C \\perp B C$, 得 $\\overrightarrow{A C} \\cdot \\overrightarrow{B C}=0$, 即 $(x+1, y) \\cdot(x-3, y-2)=0$, 所以 $(x+1)(x-3)+y(y-2)=0$, 即 $(x-1)^{2}+(y-1)^{2}=5$ (1).\n\n由 $|C A|^{2}=|C B|^{2}$, 得 $(x+1)^{2}+y^{2}=(x-3)^{2}+(y-2)^{2}$,化简,得 $y=-2 x+3$.\n\n代入(1)整理, 得 $x^{2}-2 x=0$, 解得 $x_{1}=0, x_{2}=2$, 所以 $y_{1}=3, y_{2}=-1$,所以点 $C$ 的坐标为 $(0,3)$ 或 $(2,-1)$.\n\n分析: 本题主要考查了平面直角坐标轴中的伸缩变换, 解决问题的关键是根据伸缩变换的原理结合所给条件计算即可"} {"id": "22782", "image": [], "answer": "A", "solution": "null", "level": "高三", "question": "圆 $\\rho=\\sqrt{2}(\\cos \\theta+\\sin \\theta)$ 的圆心坐标是 $(\\quad)$", "options": "A. $\\left(1, \\frac{\\pi}{4}\\right)$ B. $\\left(\\frac{1}{2}, \\frac{\\pi}{4}\\right)$ C. $\\left(\\sqrt{2}, \\frac{\\pi}{4}\\right)$ D. $\\left(2, \\frac{\\pi}{4}\\right)$", "subject": "解析几何", "analysis": "解析: 解答: 法一: $\\because$ 圆 $\\rho=\\sqrt{2}(\\cos \\theta+\\sin \\theta)=2 \\sin \\left(\\theta+\\frac{\\pi}{4}\\right)$, 可以看作由圆 $\\rho=2 \\sin \\theta$顺时针旋转 $\\frac{\\pi}{4}$ 得到.而 $\\rho=2 \\sin \\theta$ 的圆心为 $\\left(1, \\frac{\\pi}{2}\\right)$, 顺时针旋转 $\\frac{\\pi}{4}$ 得到 $\\left(1, \\frac{\\pi}{4}\\right), \\therefore$ $\\rho=\\sqrt{2}(\\cos \\theta+\\sin \\theta)$ 的圆心坐标为 $\\left(1, \\frac{\\pi}{4}\\right)$.\n\n法二: 圆 $\\rho=\\sqrt{2}(\\cos \\theta+\\sin \\theta)$ 直角坐标方程为 $x^{2}+y^{2}-\\sqrt{2} x-\\sqrt{2} y=0 , \\therefore$ $\\left(x-\\frac{\\sqrt{2}}{2}\\right)^{2}+\\left(y-\\frac{\\sqrt{2}}{2}\\right)^{2}=1$, 圆心的直角坐标为 $\\left(\\frac{\\sqrt{2}}{2}, \\frac{\\sqrt{2}}{2}\\right)$, 化为极坐标为 $\\left(1, \\frac{\\pi}{4}\\right)$.分析: 本题主要考查了极坐标系, 解决问题的关键是根据极坐标与直角坐标系关系互化化简计算即可"} {"id": "22792", "image": [], "answer": "C", "solution": "null", "level": "高三", "question": "极坐标系中, 集合 $\\{(\\rho, \\theta) \\mid \\rho=2014,0 \\leq \\theta<2 \\pi\\}$ 表示的图形是( )", "options": "A. 射线 B. 直线 C. 圆 D. 半圆", "subject": "解析几何", "analysis": "解析: 解答: 由于 $\\rho=2014$ 表示到极点距离等于 2014 的点的集合, 又 $0 \\leq \\theta<2 \\pi$, 即以极点为圆心, 半径为 2014 的圆.\n\n分析: 本题主要考查了极坐标系, 解决问题的关键是根据极坐标的意义分析即可"} {"id": "22799", "image": [], "answer": "B", "solution": "null", "level": "高三", "question": "极坐标系中, 与点 $\\left(5, \\frac{5 \\pi}{3}\\right)$ 的距离为 1 且与极点距离最近的点的直角坐标为 ( )", "options": "A. $(2,2 \\sqrt{3})$ B. $(2,-2 \\sqrt{3})$ C. $(-2,-2 \\sqrt{3})$ D. $(-2,2 \\sqrt{3})$", "subject": "解析几何", "analysis": "解析: 解答: 依题意, 所求的点与极点.点 $\\left(5, \\frac{5 \\pi}{3}\\right)$ 三点共线, 且该点的极径为 4 , 故该点的极坐标可以表示为 $\\left(4, \\frac{5 \\pi}{3}\\right)$, 化为直角坐标为 $(2,-2 \\sqrt{3})$.\n\n分析: 本题主要考查了点的极坐标和直角坐标的互化, 解决问题的关键是根据极坐标系与直角坐标系的关系分析即可"} {"id": "22801", "image": [], "answer": "C", "solution": "null", "level": "高三", "question": "已知点 $P$ 的坐标为 $(1, \\pi)$, 则过点 $P$ 且垂直极轴的直线方程是 $(\\quad)$", "options": "A. $\\rho=1$ B. $\\rho=\\cos \\theta$ C. $\\rho=-\\frac{1}{\\cos \\theta}$ D. $\\rho=\\frac{1}{\\cos \\theta}$", "subject": "解析几何", "analysis": "解析: 解答: 由点 $\\mathrm{P}$ 的坐标可知, 过点 $\\mathrm{P}$ 且垂直极轴的直线方程在直角坐标中为 $\\mathrm{x}=-1$, 即 $\\rho \\cos \\theta=-1$.\n\n分析: 本题主要考查了点的极坐标和直角坐标的互化, 解决问题的关键是根据极坐标的意义互化即可"} {"id": "22983", "image": [], "answer": "B", "solution": "null", "level": "高三", "question": "关于 $\\mathrm{x}$ 的不等式 $|\\mathrm{x}-3|+|\\mathrm{x}-4|1$\nC. $0
1$. 故选 B 分析: 本题主要考查了绝对值不等式的解法, 解决问题的关键是根据绝对值不等式的性质及转化思想,分析解决问题即可."} {"id": "22984", "image": [], "answer": "A", "solution": "null", "level": "高三", "question": "如果 $x, y \\in R$, 那么\" $x y>0 \"$ 是\" $|x+y|=|x|+|y| \"$ 成立的", "options": "A. 充分不必要条件\nB. 必要不充分条件\nC. 充要条件\nD. 既不充分也不必要条件", "subject": "解析几何", "analysis": "解析: 解答: 由已知中 $x, y \\in R$, 根据绝对值的性质, 分别讨论“ $x y>0$ ” $\\Rightarrow|x+y||| x|+| y \\mid ”$, 与 $“|x+y|=|x|+|y| \" \\Rightarrow$ “ $x y>0$ ”, 的真假, 然后根据充要条件的定义, 即可得到答案.\n\n解答: 解: 若“ $x y>0$ ”, 则 $x, y$ 同号, 则“ $|x+y|=|x|+|y|$ ”成立\n\n即“ $x y>0$ ”是“ $|x+y|=|x|+|y|$ ”成立的充分条件\n\n但“|x+y|=|x|+|y|”成立时, $x, y$ 不异号, “ $x y \\geq 0$ ”, “ $x y>0$ ”不一定成立,\n\n即“ $x y>0$ ”是“ $|x+y|=|x|+|y| ”$ ”成立的不必要条件\n\n即“ $x y>0$ ”是“ $|x+y|=|x|+|y|$ ”成立的充分不必要条件\n\n故选 A\n\n分析: 本题主要考查了不等式的基本性质, 解决问题的关键是根据绝对值的性质, 判断“ $x y$ $>0$ ” $\\Rightarrow$ “| $x+y|=| x|+| y|”, ~ 与 “| x+y|=| x|+| y \\mid ” \\Rightarrow$ “ $x y>0$ ”的真假, 是解答本题的关键."} {"id": "22985", "image": [], "answer": "C", "solution": "null", "level": "高三", "question": "若 $a>0$, 使不等式 $|x-4|+|x-3|0)$ 所表示的图形的交点的极坐标是", "options": "A. $(1,1)$\nB. $\\left(1, \\frac{\\pi}{4}\\right)$\nC. $\\left(\\sqrt{2}, \\frac{\\pi}{4}\\right)$\nD. $\\left(\\sqrt{2}, \\frac{\\pi}{2}\\right)$", "subject": "解析几何", "analysis": "解析:解答:圆 $\\rho=2 \\cos \\theta$ 的直角坐标方程为: $x^{2}+y^{2}-2 x=0$,\n\n方程 $\\theta=\\frac{\\pi}{4}$ 的直角坐标方程为: $\\mathrm{y}=\\mathrm{x}$\n\n解方程组:\n\n$\\mathrm{x}^{2}+\\mathrm{y}^{2}-2 \\mathrm{x}=0$\n\n$y=x(x>0)$, 得交点的坐标是 (1, 1),\n\n$\\therefore$ 交点的极坐标是 $\\left(\\sqrt{2}, \\frac{\\pi}{4}\\right)$, 故选 C\n\n分析: 本题主要考查了圆的极坐标方程, 解决问题的关键是根据所给方程化为普通方程分析即可"} {"id": "22828", "image": [], "answer": "A", "solution": "null", "level": "高三", "question": "在极坐标系下, 已知圆 $\\mathrm{C}$ 的方程为 $\\rho=2 \\cos \\theta$, 则下列各点中, 在圆 $\\mathrm{C}$ 上的是 ( )", "options": "A. $\\left(1,-\\frac{\\pi}{3}\\right)$\nB. $\\left(1, \\frac{\\pi}{6}\\right)$\nC. $\\left(\\sqrt{2}, \\frac{3 \\pi}{4}\\right)$\nD. $\\left(\\sqrt{2}, \\frac{5 \\pi}{4}\\right)$", "subject": "解析几何", "analysis": "解析: 解答: 将四个选项极坐标分别代入及坐标方程验证可知只有 A 项成立, 所以点 $\\left(1,-\\frac{\\pi}{3}\\right)$\n\n在圆 C 上\n\n分析: 本题主要考查了圆的极坐标方程, 解决问题的关键是点是否在曲线上只需将点的坐标带入验证"} {"id": "22829", "image": [], "answer": "A", "solution": "null", "level": "高三", "question": "极坐标系中, 以 $\\left(9, \\frac{\\pi}{3}\\right)$ 为圆心, 9 为半径的圆的极坐标方程为( )", "options": "A. $\\rho=18 \\cos \\left(\\frac{\\pi}{3}-\\theta\\right)$\nB. $\\rho=-18 \\cos \\left(\\frac{\\pi}{3}-\\theta\\right)$\nC. $\\rho=18 \\sin \\left(\\frac{\\pi}{3}-\\theta\\right)$\nD. $\\rho=9 \\cos \\left(\\frac{\\pi}{3}-\\theta\\right)$", "subject": "解析几何", "analysis": "解析: 解答: 结合图形分析, 借助于直角三角形中的边角关系, 极坐标系中, 以 ( $9, \\frac{\\pi}{3}$ )为圆心, 9 为半径的圆的极坐标方程为 $\\rho=18 \\cos \\left(\\frac{\\pi}{3}-\\theta\\right)$, 选 A。\n\n![](https://cdn.mathpix.com/cropped/2024_04_19_b4d16be27c2fb355fe7ag-"} {"id": "22834", "image": ["10088.jpg"], "answer": "B", "solution": "null", "level": "高三", "question": "已知圆的直角坐标方程为 $x^{2}+y^{2}-2 y=0$. 在以原点为极点, $x$ 轴正半轴为极轴的极坐标系中, 该圆的方程为 ( )", "options": "A. $\\rho=2 \\cos \\theta$\nB. $\\rho=2 \\sin \\theta$\nC. $\\rho=-2 \\cos \\theta$\nD. $\\rho=-2 \\sin \\theta$", "subject": "解析几何", "analysis": "解析: 解答: 法一: 利用直角坐标与极坐标间的关系, 即利用 $\\rho \\cos \\theta=x, \\rho \\sin \\theta=y, \\rho^{2}=x^{2}+y^{2}$, 进行代换,\n\n圆的直角坐标方程为 $x^{2}+y^{2}-2 y=0$, 所以 $\\rho^{2}-2 \\rho \\sin \\theta=0$, 即 $\\rho=2 \\sin \\theta$.\n\n法二: 圆的直角坐标方程为 $x^{2}+y^{2}-2 y=0$, 即 $x^{2}+(y-1)^{2}=1$,\n\n设 $M(\rho, \theta)$ 是圆 $C$ 上任一点, $\\angle M O x=\theta$, 若 $\theta$ 为针角, 则 $\\sin (\\pi-\theta)=\\sin \theta 10$所以 $2 \\sin \theta=\rho$.故选 B. 分析: 本题主要考查了圆的极坐标方程, 解决问题的关键是根据所给极坐标方程转化为普通方程计算即可"} {"id": "22841", "image": [], "answer": "A", "solution": "null", "level": "高三", "question": "极坐标方程 $\\rho=\\cos \\theta$ 和参数方程 $\\left\\{\\begin{array}{l}x=-1-t \\\\ y=2+3 t\\end{array}\\right.$ ( $t$ 为参数) 所表示的图形分别是 ( )", "options": "A. 圆.直线\nB.直线.圆\nC.圆.圆\nD.直线.直线", "subject": "解析几何", "analysis": "解析:解答: $\\because$ 极坐标 $\\mathrm{p}=\\cos \\theta, \\mathrm{x}=\\mathrm{p} \\cos \\theta, \\mathrm{y}=\\mathrm{p} \\sin \\theta$, 消去 $\\theta$ 和 $\\mathrm{p}, \\therefore \\mathrm{x}^{2}+\\mathrm{y}^{2}=\\mathrm{x}, \\mathrm{x}^{2}+\\mathrm{y}^{2}=\\mathrm{x}$ 为圆的方程; 参数方程 $\\left\\{\\begin{array}{l}x=-1-t \\\\ y=2+3 t\\end{array}\\right.$ ( $\\mathrm{t}$ 为参数) 消去 $\\mathrm{t}$ 得, $3 \\mathrm{x}+\\mathrm{y}+1=0$, 为直线的方程, 故选 $\\mathrm{A}$.分析: 本题主要考查了圆的极坐标方程, 解决问题的关键是参数方程.极坐标方程与普通方程的区别和联系, 两者要会互相转化, 根据实际情况选择不同的方程进行求解, 这也是每年高考必考的热点问题"} {"id": "22842", "image": [], "answer": "C", "solution": "null", "level": "高三", "question": "极坐标方程 $\\rho \\cos \\theta=2 \\sin 2 \\theta$ 表示的曲线为()", "options": "A. 一条射线和一个圆\nB. 两条直线\nC. 一条直线和一个圆\nD. 一个圆", "subject": "解析几何", "analysis": "解析:解答:因为极坐标方程\n\n$\\rho \\cos \\theta=2 \\sin 2 \\theta \\Leftrightarrow \\rho \\cos \\theta=4 \\sin \\theta \\cos \\theta \\Leftrightarrow \\rho=4 \\sin \\theta, \\cos \\theta=0$,\n\n因此表示一个圆和一条直线。选 C\n\n分析: 本题主要考查了圆的极坐标方程, 解决问题的关键是根据所给极坐标方程化简分析即可"} {"id": "22843", "image": [], "answer": "C", "solution": "null", "level": "高三", "question": "极坐标方程 $(\\rho-1)(\\theta-\\pi)=0(\\rho \\geq 0)$ 表示的图形是 ( )", "options": "A. 两个圆\nB. 两条直线\nC. 一个圆和一条射线\nD. 一条直线和一条射线", "subject": "解析几何", "analysis": "解析:解答:方程 $(\\rho-1)(\\theta-\\pi)=0 \\Rightarrow \\rho=1$ 或 $\\theta=\\pi$,\n$\\rho=1$ 是半径为 1 的圆,\n\n$\\theta=\\pi$ 是一条射线.\n\n故选 C.\n\n分析: 本题主要考查了圆的极坐标方程, 解决问题的关键是化简分析即可"} {"id": "22844", "image": [], "answer": "B", "solution": "null", "level": "高三", "question": "极坐标方程 $\\rho \\cos ^{2} \\theta+\\rho \\sin ^{2} \\theta=4 \\sin \\theta$ 所表示的曲线是 ( )", "options": "A. 一条直线\nB. 一个圆\nC. 一条抛物线\nD. 一条双曲线", "subject": "解析几何", "analysis": "解析: 解答: 由已知得 $\\rho^{2} \\cos ^{2} \\theta+\\rho^{2} \\sin ^{2} \\theta=4 \\rho \\sin \\theta$, 故 $x^{2}+y^{2}=4 y$, 故表示一个圆分析: 本题主要考查了圆的极坐标方程, 解决问题的关键是化为普通方程分析即可"} {"id": "22845", "image": [], "answer": "B", "solution": "null", "level": "高三", "question": "在极坐标系中, 圆 C 过极点, 且圆心的极坐标是 $\\left(a, \\frac{\\pi}{2}\\right)(a>0)$, 则圆 C 的极坐标方程是", "options": "A. $\\rho=-2 a \\sin \\theta$.\nB. $\\rho=2 a \\sin \\theta$.\nC. $\\rho=-2 a \\cos \\theta$.\nD. $\\rho=2 a \\cos \\theta$.", "subject": "解析几何", "analysis": "解析:解答:由题意得圆心的直角坐标是 $(0, a)$, 半径为 $a$, 直角坐标方程为 $x^{2}+(y-a)^{2}=a^{2}, x^{2}+y^{2}-2 a y=0$, 极坐标方程是 $\\rho^{2}-2 a \\rho \\sin \\theta=0, \\rho=2 a \\sin \\theta$.分析: 本题主要考查了圆的极坐标方程, 解决问题的关键是根据普通方程转化为极坐标方程即可"} {"id": "22846", "image": [], "answer": "D", "solution": "null", "level": "高三", "question": "已知在极坐标系下两圆的极坐标方程分别为 $\\rho=\\cos \\theta, \\rho=\\sqrt{3} \\sin \\theta$, 则此两圆的圆心距为 $(\\quad)$", "options": "A. $\\frac{\\sqrt{3}}{4}$\nB. $\\frac{\\sqrt{3}}{2}$\nC. $\\frac{1}{2}$\nD. 1", "subject": "解析几何", "analysis": "解析: 解答: $\\rho=\\cos \\theta \\Longrightarrow \\rho^{2}=\\rho \\cos \\theta$, 化为直角坐标方程为 $x^{2}+y^{2}=x$, 即 $\\left(x-\\frac{1}{2}\\right)^{2}+y=\\frac{1}{4}$, 圆心为 $\\left(\\frac{1}{2}, 0\\right) ; \\rho=\\sqrt{3} \\sin \\theta \\Rightarrow \\rho^{2}=\\sqrt{3} \\rho \\sin \\theta$ 化为直角坐标方程为 $x^{2}+y^{2}=\\sqrt{3} y$, 即 $x^{2}+\\left(y-\\frac{\\sqrt{3}}{2}\\right)=\\frac{3}{4}$, 圆心为 $\\left(0, \\frac{\\sqrt{3}}{2}\\right)$; 则此两圆的圆心距为 $\\sqrt{\\left(\\frac{1}{2}-0\\right)^{2}+\\left(0-\\frac{\\sqrt{3}}{2}\\right)^{2}}=1$. 故选 D\n\n分析: 本题主要考查了圆的极坐标方程, 解决问题的关键是化为普通方程分析计算即可"} {"id": "22534", "image": [], "answer": "D", "solution": "null", "level": "高三", "question": "线段 $M N$ 在直线 $l$ 上的射影不可能是( )", "options": "A. 点\nB.线段\nC.与 $\\mathrm{MN}$ 等长的线段\nD. 直线", "subject": "解析几何", "analysis": "解析:解答:当 $M N \\perp 1$ 时,射影是一个点;当 $M N$ 与 1 不垂直时,射影是一条线段;特别地,当 $\\mathrm{MN} / / 1$ 或 $\\mathrm{MN}$ 在 1 上时,射影与 $\\mathrm{MN}$ 等长,线段 $\\mathrm{MN}$ 的射影不可能是直线.\n\n分析: 本题主要考查了直角三角形的射影定理, 解决问题的关键是根据直角三角形的射影定理的原理分析即可"} {"id": "22858", "image": [], "answer": "C", "solution": "null", "level": "高三", "question": "柱坐标 $\\left(2, \\frac{\\pi}{3}, 1\\right)$ 对应的点的直角坐标是 ( )", "options": "A. $(\\sqrt{3},-1,1)$\nB. $(\\sqrt{3}, 1,1)$ C. $(1, \\sqrt{3}, 1)$ D. $(-1, \\sqrt{3}, 1)$", "subject": "解析几何", "analysis": "解析: 解答: 由直角坐标与柱坐标之间的变换公式 $\\left\\{\\begin{array}{l}x=\\rho \\cos \\theta, \\\\ y=\\rho \\sin \\theta \\text {, 可得 } \\\\ z=z, \\\\ y=\\sqrt{3}, \\\\ z=1,\\end{array}\\right.$\n\n分析: 本题主要考查了柱坐标刻画点的位置, 解决问题的关键是根据柱坐标刻画点的位置转化分析即可"} {"id": "21102", "image": [], "answer": "B", "solution": "null", "level": "高三", "question": "椭圆 $\\frac{x^{2}}{25}+\\frac{y^{2}}{9}=1$ 上一点 $P$ 到一个焦点的距离为 5 , 则 $P$ 到另一个的焦点距离为 ( )", "options": "A. 4\nB. 5\nC. 6\nD. 10", "subject": "解析几何", "analysis": "$: \frac{\\mathrm{x}^{2}}{25}+\frac{\\mathrm{y}^{2}}{9}=1, \therefore \\mathrm{a}=5$, 由于点 $\\mathrm{P}$ 到一个焦点的距离为 5 , 由椭圆的定义知, $\\mathrm{P}$ 到另一个焦点的距离为 $2 \\mathrm{a}-5=5$. 故选 B."} {"id": "21107", "image": [], "answer": "D", "solution": "null", "level": "高三", "question": "已知椭圆的标准方程为 $\\frac{x^{2}}{10}+y^{2}=1$, 则粗圆的焦点坐标为 ( )", "options": "A. $(\\sqrt{10}, 0),(-\\sqrt{10}, 0)$\nB. $(0, \\sqrt{10}),(0,-\\sqrt{10})$\nC. $(0,3),(0,-3)$\nD. $(3,0),(-3,0)$", "subject": "解析几何", "analysis": "由粗圆的标准方程 $\frac{x^{2}}{10}+y^{2}=1$, 得 $a^{2}=10, b^{2}=1, \therefore c^{2}=a^{2}-b^{2}=10-1=9$, 则 $c=3, \therefore$ 椭圆的焦点坐标为 $(3,0),(-3,0)$. 故选 D."} {"id": "21108", "image": [], "answer": "B", "solution": "null", "level": "高三", "question": "椭圆的焦距为 8 , 且椭圆上的点到两个焦点距离之和为 10 , 则该粗圆的标准方程是", "options": "A. $\\frac{x^{2}}{25}+\\frac{y^{2}}{9}=1$\nB. $\\frac{y^{2}}{25}+\\frac{x^{2}}{9}=1$ 或 $\\frac{x^{2}}{25}+\\frac{y^{2}}{9}=1$\nC. $\\frac{y^{2}}{25}+\\frac{x^{2}}{9}=1$\nD. $\\frac{y^{2}}{25}+\\frac{x^{2}}{16}=1$ 或 $\\frac{x^{2}}{16}+\\frac{y^{2}}{9}=1$", "subject": "解析几何", "analysis": "由题意可知: 焦距为 $2 c=8$, 则 $c=4,2 a=10, a=5, \\quad b^{2}=a^{2}-c^{2}=9, \therefore$ 当椭圆的焦点在 $x$轴上时, 椭圆的标准方程: $\frac{x^{2}}{25}+\frac{y^{2}}{9}=1$, 当椭圆的焦点在 $y$ 轴上时, 椭圆的标准方程: $\frac{y^{2}}{25}$ $+\frac{x^{2}}{9}=1$, 故椭圆的标准方程为: $\frac{x^{2}}{25}+\frac{y^{2}}{9}=1$ 或 $\frac{y^{2}}{25}+\frac{x^{2}}{9}=1$, 故选 B."} {"id": "21109", "image": [], "answer": "A", "solution": "null", "level": "高三", "question": "已知椭圆: $\\frac{x^{2}}{\\mathrm{k}}+\\frac{\\mathrm{y}^{2}}{2}=1$, 若粗圆的焦距为 2 , 则 $\\mathrm{k}$ 为 ( )", "options": "A. 1 或 3\nB. 1\nC. 3\nD. 6", "subject": "解析几何", "analysis": "(1) 若焦点在 $y$ 轴上, 椭圆 $\frac{x^{2}}{k}+\frac{y^{2}}{2}=1$ 中, $a^{2}=2, b^{2}=k$, 则 $c=\\sqrt{2-k}: 2 c=2 \\sqrt{2-k}=2$,解得 $\\mathrm{k}=1$. (2)若焦点在 $\\mathrm{x}$ 轴上, 籿圆 $\frac{\\mathrm{x}^{2}}{\\mathrm{k}}+\frac{\\mathrm{y}^{2}}{2}=1$ 中, $\\mathrm{a}^{2}=\\mathrm{k}, \\mathrm{b}^{2}=2$, 则 $c=\\sqrt{\\mathrm{k}-2}, \therefore 2 \\mathrm{c}=$ $2 \\sqrt{\\mathrm{k}-2}=2$, 解得 $\\mathrm{k}=3$. 综上所述, $\\mathrm{k}$ 的值是 1 或 3 . 故选 $\\mathrm{A}$."} {"id": "21110", "image": [], "answer": "B", "solution": "null", "level": "高三", "question": "已知椭圆 $\\frac{x^{2}}{25}+\\frac{y^{2}}{m^{2}}=1(m>0)$ 的左焦点为 $F_{1}(-4,0)$, 则 $m=(\\quad)$", "options": "A. 2\nB. 3\nC. 4\nD. 9", "subject": "解析几何", "analysis": "$\because$ 椭圆 $\frac{\\mathrm{x}^{2}}{25}+\frac{\\mathrm{y}^{2}}{\\mathrm{~m}^{2}}=1(\\mathrm{~m}>0)$ 的左焦点为 $\\mathrm{F}_{1}(-4,0), \therefore 25-\\mathrm{m}^{2}=16, \because \\mathrm{m}>0, \therefore \\mathrm{m}=3$, 故选 B."} {"id": "21111", "image": [], "answer": "C", "solution": "null", "level": "高三", "question": "过点 $(3,2)$ 且与椭圆 $3 x^{2}+8 y^{2}=24$ 有相同焦点的粗圆方程为 $(\\quad)$", "options": "A. $\\frac{x^{2}}{5}+\\frac{y^{2}}{10}=1$\nB. $\\frac{x^{2}}{10}+\\frac{y^{2}}{15}=1$\nC. $\\frac{x^{2}}{15}+\\frac{y^{2}}{10}=1$\nD. $\\frac{x^{2}}{25}+\\frac{y^{2}}{10}=1$", "subject": "解析几何", "analysis": "椭圆 $3 x^{2}+8 y^{2}=24$ 的焦点 $( \\pm \\sqrt{5}, 0)$, 可得 $c=\\sqrt{5}$, 设陏圆的方程为: $\frac{x^{2}}{a^{2}}+\frac{y^{2}}{b^{2}}=1(a>b>0)$,可得: $\frac{9}{a^{2}}+\frac{4}{b^{2}}=1, a^{2}-b^{2}=5$, 解得 $a=\\sqrt{15}, b=\\sqrt{10}$, 所求的粗圆方程为: $\frac{x^{2}}{15}+\frac{y^{2}}{10}=1$. 故选 C."} {"id": "21112", "image": [], "answer": "B", "solution": "null", "level": "高三", "question": "已知椭圆的焦点为 $(-1,0)$ 和 $(1,0)$, 点 $P(2,0)$ 在粗圆上, 则陏圆的标准方程为 ( )", "options": "A. $\\frac{x^{2}}{4}+y^{2}=1$\nB. $\\frac{\\mathrm{x}^{2}}{4}+\\frac{\\mathrm{y}^{2}}{3}=1$\nC. $\\frac{y^{2}}{4}+x^{2}=1$\nD. $\\frac{y^{2}}{4}+\\frac{x^{2}}{3}=1$", "subject": "解析几何", "analysis": "设粗圆方程为 $\frac{x^{2}}{a^{2}}+\frac{y^{2}}{b^{2}}=1(a>b>0)$, 由题意可得 $c=1, a=2, b=\\sqrt{3}$, 即有椭圆方程为 $\frac{\\mathrm{x}^{2}}{4}+\frac{\\mathrm{y}^{2}}{3}=1$. 故选: $\\mathrm{B}$"} {"id": "21113", "image": ["9520.jpg"], "answer": "B", "solution": "null", "level": "高三", "question": "已知椭圆过点 $P\\left(1, \\frac{\\sqrt{3}}{2}\\right)$ 和 $Q(2,0)$, 则粗圆的方程为 ( )", "options": "A. $\\frac{x^{2}}{4}+\\frac{y^{2}}{2}=1$\nB. $\\frac{x^{2}}{4}+y^{2}=1$\nC. $\\frac{x^{2}}{4}+\\frac{y^{2}}{3}=1$\nD. $\\frac{y^{2}}{4}+x^{2}=1$", "subject": "解析几何", "analysis": "设椭圆方程为: $m x^{2}+n y^{2}=1(m>0, n>0 . m \neq n)$, 粗圆过点 $P\\left(1, \frac{\\sqrt{3}}{2}\right)$ 和 $Q(2,0)$, 可得: $m+\frac{3}{4} n=1,4 m=1$, 解得 $m=\frac{1}{4}$,$\\mathrm{n}=1$. 所以所求椭圆方程为: $\frac{\\mathrm{x}^{2}}{4}+\\mathrm{y}^{2}=1$. 故选 $\\mathrm{B}$."} {"id": "21114", "image": [], "answer": "C", "solution": "null", "level": "高三", "question": "“a>0, b>0” 是 “方程 $a x^{2}+b y^{2}=1$ 表示粗圆” 的()", "options": "A. 充要条件\nB. 充分非必要条件\nC. 必要非充分条件\nD. 既不充分也不必要条件", "subject": "解析几何", "analysis": "$a>0, b>0$, 方程 $a x^{2}+b y^{2}=1$ 不一定表示椭圆, 如 $a=b=1$; 反之, 若方程 $a x^{2}+b y^{2}=1$表示椭圆, 则 $\\mathrm{a}>0, \\mathrm{~b}>0, \therefore$, $\\mathrm{a}>0, \\mathrm{~b}>0$ ”是“方程 $\\mathrm{ax}^{2}+b \\mathrm{y}^{2}=1$ 表示椭圆”的必要非充分条件. 故选: C."} {"id": "21115", "image": [], "answer": "B", "solution": "null", "level": "高三", "question": "椭圆 $\\frac{\\mathrm{x}^{2}}{9}+\\frac{\\mathrm{y}^{2}}{4}=1$ 的离心率是 ( ).", "options": "A. $\\frac{\\sqrt{13}}{3}$\nB. $\\frac{\\sqrt{5}}{3}$\nC. $\\frac{2}{3}$\nD. $\\frac{5}{9}$", "subject": "解析几何", "analysis": "椭圆 $\frac{x^{2}}{9}+\frac{y^{2}}{4}=1$, 可得 $a=3, b=2$, 则 $c=\\sqrt{9-4}=\\sqrt{5}$, 所以椭圆的离心率为: $\frac{c}{a}=\frac{\\sqrt{5}}{3}"} {"id": "21116", "image": ["9521.jpg", "9522.jpg", "9523.jpg"], "answer": "B", "solution": "null", "level": "高三", "question": "下列三个图中的多边形均为正多边形, A (B) 是正多边形的顶点, 椭圆过 A (B) 且均以图中的 $\\mathrm{F}_{1}, \\mathrm{~F}_{2}$ 为焦点, 设图(1), (2), (3)中的椭圆的离心率分别为 $\\mathrm{e}_{1}, \\mathrm{e}_{2}, \\mathrm{e}_{3}$, 则\n\n\n\n(1)\n\n\n\n(2)\n\n\n\n(3)", "options": "A. $\\mathrm{e}_{1}>\\mathrm{e}_{2}>\\mathrm{e}_{3}$\nB. $e_{3}>e_{1}>e_{2}$\nC. $\\mathrm{e}_{1}<\\mathrm{e}_{3}<\\mathrm{e}_{2}$\nD. $\\mathrm{e}_{1}<\\mathrm{e}_{2}<\\mathrm{e}_{3}$", "subject": "解析几何", "analysis": "由图(1)知, $a=2 c, \therefore e_{1}=\frac{c}{a}=\frac{1}{2}$; 由图(2)知, 点 B (c, 2c) 在籿圆 $\frac{x^{2}}{a^{2}}+\frac{y^{2}}{b^{2}}=1$ 上, $\therefore \frac{c^{2}}{a^{2}}+\frac{4 c^{2}}{b^{2}}=1$, 则 $\frac{c^{2}}{a^{2}}+\frac{4 c^{2}}{a^{2}-c^{2}}=1$, 整理得: $c^{4}-6 a^{2} c^{2}+a^{4}=0$, 得 $e_{2}=\\sqrt{2}-1$; 由图(3)知, $B\\left(\frac{c}{2}, \frac{\\sqrt{3} c}{2}\right)$ 在椭圆 $\frac{x^{2}}{a^{2}}+\frac{y^{2}}{b^{2}}=1$ 上, $\therefore \frac{c^{2}}{4 a^{2}}+\frac{3 c^{2}}{4 b^{2}}=1$, 则 $\frac{c^{2}}{4 a^{2}}+\frac{3 c^{2}}{4\\left(a^{2}-c^{2}\right)}=1$, 整理得: $e_{3}=\\sqrt{3}-$ 1. $\therefore \\mathrm{e}_{3}>\\mathrm{e}_{1}>\\mathrm{e}_{2}$. 故选 B."} {"id": "21119", "image": [], "answer": "C", "solution": "null", "level": "高三", "question": "若焦点在 $\\mathrm{x}$ 轴上的椭圆 $\\mathrm{C}: \\frac{\\mathrm{x}^{2}}{\\mathrm{a}^{2}}+\\frac{\\mathrm{y}^{2}}{5}=1(\\mathrm{a}>0)$ 的离心率为 $\\frac{2}{3}$, 则 $\\mathrm{a}$ 的值为 $(\\quad)$", "options": "A. 9\nB. 6\nC. 3\nD. 2", "subject": "解析几何", "analysis": "焦点在 $x$ 轴上的椭圆C: $\frac{x^{2}}{a^{2}}+\frac{y^{2}}{5}=1(a>0)$, 可得 $c=\\sqrt{a^{2}-5}$, 离心率为 $\frac{2}{3}$, 可得: $\frac{\\sqrt{a^{2}-5}}{a}=$ $\frac{2}{3}$, 得 $\\mathrm{a}=3$. 故选 C."} {"id": "21120", "image": [], "answer": "C", "solution": "null", "level": "高三", "question": "焦点在 $x$ 轴上, 长短半轴长之和为 10 , 焦距为 $4 \\sqrt{5}$, 则粗圆的标准方程为", "options": "A. $\\frac{\\mathrm{x}^{2}}{6}+\\frac{\\mathrm{y}^{2}}{4}=1$\nB. $\\frac{\\mathrm{x}^{2}}{16}+\\frac{\\mathrm{y}^{2}}{36}=1$\nC. $\\frac{x^{2}}{36}+\\frac{y^{2}}{16}=1$\nD. $\\frac{x^{2}}{49}+\\frac{y^{2}}{9}=1$", "subject": "解析几何", "analysis": "焦点在 $\\mathrm{x}$ 轴上, 长、短半轴长之和为 10 , 焦距为 $4 \\sqrt{5}$, 可得 $\\mathrm{a}+\\mathrm{b}=10,2 \\mathrm{c}=4 \\sqrt{5}, \\mathrm{c}=2 \\sqrt{5}$,即 $a^{2}-b^{2}=20$, 得 $a^{2}=36, b^{2}=16$, 所求椭圆方程为: $\frac{x^{2}}{36}+\frac{y^{2}}{16}=1$. 故选 C."} {"id": "21121", "image": [], "answer": "C", "solution": "null", "level": "高三", "question": "若椭圆 C: $\\frac{x^{2}}{a^{2}}+\\frac{y^{2}}{b^{2}}=1(a>b>0)$ 的短轴长等于焦距, 则粗圆的离心率为 ( )", "options": "A. $\\frac{1}{2}$\nB. $\\frac{\\sqrt{3}}{3}$\nC. $\\frac{\\sqrt{2}}{2}$\nD. $\\frac{\\sqrt{2}}{4}$", "subject": "解析几何", "analysis": "依题意可知 $2 \\mathrm{c}=2 \\mathrm{~b}$, 即 $\\mathrm{b}=\\mathrm{c}$, 所以 $\\mathrm{a}=\\sqrt{\\mathrm{b}^{2}+\\mathrm{c}^{2}}=\\sqrt{2} \\mathrm{c}, \therefore$ 粗圆的离心率 $\\mathrm{e}=\frac{\\mathrm{c}}{\\mathrm{a}}=\frac{\\sqrt{2}}{2}$."} {"id": "21122", "image": [], "answer": "B", "solution": "null", "level": "高三", "question": "焦点在 $\\mathrm{x}$ 轴上的椭圆 $\\frac{\\mathrm{x}^{2}}{3 \\mathrm{n}}+\\mathrm{y}^{2}=1(\\mathrm{n}>0)$ 的焦距为 $4 \\sqrt{2}$, 则长轴长是 $(\\quad)$", "options": "A. 3\nB. 6\nC. $6 \\sqrt{2}$\nD. 2", "subject": "解析几何", "analysis": "椭圆 $\frac{x^{2}}{3 n}+y^{2}=1(n>0)$ 的 $a=\\sqrt{3 n}, b=1, c=\\sqrt{3 n-1}$,由题意可知 $\\left\\{\begin{array}{l}3 n>1 \\ 2 \\sqrt{3 n-1}=4 \\sqrt{2}\\end{array} \\Rightarrow n=3\right.$, 所以长轴长为 $2 a=6$, 故选 $B$."} {"id": "21124", "image": [], "answer": "D", "solution": "null", "level": "高三", "question": "粗圆 $\\frac{x^{2}}{16}+\\frac{y^{2}}{12}=1$ 上一点 $P$ 到焦点距离的最大值为 ( )", "options": "A. 4\nB. 2\nC. $2 \\sqrt{3}$\nD. 6", "subject": "解析几何", "analysis": "由椭圆 $\frac{\\mathrm{x}^{2}}{16}+\frac{\\mathrm{y}^{2}}{12}=1$ 可知: 焦点在 $\\mathrm{x}$ 轴上, $\\mathrm{a}=4, \\mathrm{~b}=2 \\sqrt{3}, \\mathrm{c}=\\sqrt{\\mathrm{a}^{2}-\\mathrm{b}^{2}}=2$, 由粗圆的性质可知: $P$ 到焦点距离的最大值 $a+c=4+2=6$, 则 $P$ 到焦点距离的最大值为 6 ,"} {"id": "21125", "image": [], "answer": "A", "solution": "null", "level": "高三", "question": "已知 $F$ 是椭圆 $\\frac{x^{2}}{a^{2}}+\\frac{y^{2}}{b^{2}}=1(a>b>0)$ 的左焦点, $A$ 为右顶点, $P$ 是椭圆上的一点, $P F \\perp x$轴, 若 $|\\mathrm{PF}|=\\frac{3}{4}|\\mathrm{AF}|$, 则该陏圆的离心率是 ( )", "options": "A. $\\frac{1}{4}$\nB. $\\frac{1}{3}$\nC. $\\frac{1}{2}$\nD. $\\frac{\\sqrt{2}}{2}$", "subject": "解析几何", "analysis": "根据椭圆几何性质可知 $|P F|=\frac{b^{2}}{a},|A F|=a+c$, 所以 $\frac{b^{2}}{a}=\frac{3}{4}(a+c)$, 即 $4 b^{2}=3 a^{2}+3 a c$,因为 $b^{2}=a^{2}-c^{2}$, 所以有 $4 a^{2}-4 c^{2}=3 a^{2}+3 a c$, 整理可得 $4 c^{2}+3 a c-a^{2}=0$, 两边同除以 $a^{2}$ 得: $4 \\mathrm{e}^{2}+3 \\mathrm{e}-1=0$, 所以 $(4 \\mathrm{e}-1) \\quad(\\mathrm{e}+1)=0$, 由于 $0<\\mathrm{e}<1$, 所以 $\\mathrm{e}=\frac{1}{4}$. 故选 A."} {"id": "21126", "image": [], "answer": "A", "solution": "null", "level": "高三", "question": "已知椭圆过点 $\\left(-4, \\frac{3}{5}\\right)$ 和 $\\left(3,-\\frac{4}{5}\\right)$, 则椭圆离心率 $e=(\\quad)$", "options": "A. $\\frac{2 \\sqrt{6}}{5}$\nB. $\\frac{\\sqrt{6}}{5}$\nC. $\\frac{1}{5}$\nD. $\\frac{2}{5}$", "subject": "解析几何", "analysis": "椭圆 $\frac{x^{2}}{a^{2}}+\frac{y^{2}}{b^{2}}=1$ 过点 $\\left(-4, \frac{3}{5}\right)$ 和 (3, $\\left.-\frac{4}{5}\right)$, 则 $\\left\\{\begin{array}{l}\frac{16}{a^{2}}+\frac{9}{25 b^{2}}=1 \\ \frac{9}{a^{2}}+\frac{16}{25 b^{2}}=1\\end{array}\right.$, 得 $a=5, b=1$, $\therefore \\mathrm{c}^{2}=\\mathrm{a}^{2}-\\mathrm{b}^{2}=24, \therefore \\mathrm{c}=2 \\sqrt{6}, \therefore \\mathrm{e}=\frac{\\mathrm{c}}{\\mathrm{c}}=\frac{2 \\sqrt{6}}{5}$, 故选: A."} {"id": "21127", "image": [], "answer": "B", "solution": "null", "level": "高三", "question": "已知椭圆的焦点为 $(-1,0)$ 和 $(1,0)$, 点 $P(2,0)$ 在粗圆上, 则陏圆的标准方程为", "options": "A. $\\frac{x^{2}}{4}+y^{2}=1$\nB. $\\frac{x^{2}}{4}+\\frac{y^{2}}{3}=1$\nC. $\\frac{y^{2}}{4}+x^{2}=1$\nD. $\\frac{y^{2}}{4}+\\frac{x^{2}}{3}=1$", "subject": "解析几何", "analysis": "设粗圆方程为 $\frac{x^{2}}{a^{2}}+\frac{y^{2}}{b^{2}}=1(a>b>0)$, 由题意可得 $c=1, a=2, b=\\sqrt{3}$, 即有椭圆方程为 $\frac{x^{2}}{4}+\frac{y^{2}}{3}=1$. 故选: $B$."} {"id": "21128", "image": ["9524.jpg"], "answer": "A", "solution": "null", "level": "高三", "question": "如图, 已知 $F_{1}, F_{2}$ 是椭圆的左、右焦点, 点 $P$ 在粗圆上, 线段 $P F_{2}$ 与圆相切于点 $Q$, 且点 $Q$ 为线段 $P F_{2}$ 的中点, 则椭圆的离心率为 ( )", "options": "A. $\\frac{\\sqrt{5}}{3}$\nB. $\\frac{\\sqrt{3}}{5}$\nC. $\\frac{\\sqrt{5}}{4}$\nD. $\\frac{\\sqrt{2}}{5}$\n\n![](https://cdn.mathpix.com/cropped/2024_04_19_c70ed9230755576ee53ag-", "subject": "解析几何", "analysis": "如图: 连接 $O Q, P F_{1}, \because$ 点 $Q$ 为线段 $P F_{2}$ 的中点, $\therefore O Q \\| P F_{1}$, $|O Q|=\frac{1}{2}\\left|P F_{1}\right|, \therefore\\left|P F_{1}\right|=2|O Q|=2 b$, 由椭圆定义, $\\left|P F_{1}\right|+\\left|P F_{2}\right|=2 a$, $\therefore\\left|P F_{2}\right|=2 a-2 b, \because$ 线段 $P F_{2}$ 与圆 $x^{2}+y^{2}=b^{2}$ 相切于点 $Q, \therefore O Q \\perp P F_{2}$,$\therefore P F_{1} \\perp P F_{2}$, 且 $\\left|F_{1} F_{2}\right|=2 c, \therefore(2 b)^{2}+(2 a-2 b)^{2}=(2 c)^{2}$, 即 $3 b=2 a, 5 a^{2}=9 c^{2}, \therefore e=\frac{c}{a}=\frac{\\sqrt{5}}{3}$.故选 $A$."} {"id": "21131", "image": [], "answer": "C", "solution": "null", "level": "高三", "question": "若粗圆 $C: \\frac{x^{2}}{a^{2}}+\\frac{y^{2}}{b^{2}}=1(a>b>0)$ 的短轴长等于焦距, 则椭圆的离心率为 ( )", "options": "A. $\\frac{1}{2}$\nB. $\\frac{\\sqrt{3}}{3}$\nC. $\\frac{\\sqrt{2}}{2}$\nD. $\\frac{\\sqrt{2}}{4}$", "subject": "解析几何", "analysis": "依题意可知 $2 c=2 b$, 即 $b=c$, 所以 $a=\\sqrt{b^{2}+c^{2}}=\\sqrt{2} c, \therefore$ 粗圆的离心率 $e=\frac{c}{a}=\frac{\\sqrt{2}}{2}$."} {"id": "21132", "image": [], "answer": "D", "solution": "null", "level": "高三", "question": "已知三个数 $1, a, 9$ 成等比数列, 则圆雉曲线 $\\frac{x^{2}}{a}+\\frac{y^{2}}{2}=1$ 的离心率为", "options": "A. $\\frac{\\sqrt{3}}{3}$\nB. $\\sqrt{5}$\nC. $\\sqrt{5}$ 或 $\\frac{\\sqrt{10}}{2}$\nD. $\\frac{\\sqrt{3}}{3}$ 或 $\\frac{\\sqrt{10}}{2}$", "subject": "解析几何", "analysis": "$:$ 三个数 $1, a, 9$ 成等比数列, $\therefore a^{2}=9$, 则 $a= \\pm 3$. 当 $a=3$ 时, 曲线方程为 $\frac{x^{2}}{3}+\frac{y^{2}}{2}=1$,表示椭圆, 则长半轴长为 $\\sqrt{3}$, 半焦距为 1 , 离心率为 $\frac{\\sqrt{3}}{3}$; 当 $a=-3$ 时, 曲线方程为 $\frac{y^{2}}{2}-\frac{x^{2}}{3}=1$,表示双曲线, 则实半轴长为 $\\sqrt{2}$, 半焦距为 $\\sqrt{5}$, 离心率为 $\frac{\\sqrt{5}}{\\sqrt{2}}=\frac{\\sqrt{10}}{2}$."} {"id": "21133", "image": ["9525.jpg"], "answer": "A", "solution": "null", "level": "高三", "question": "设 $F_{1} 、 F_{2}$ 为粗圆的两个焦点, $M$ 为椭圆上一点, $M F_{1} \\perp M F_{2}$, 且 $\\left|M F_{2}\\right|=|M O|$ (其中点 $O$ 为椭圆的中心),则该椭圆的离心率为", "options": "A. $\\sqrt{3}-1$\nB. $2-\\sqrt{3}$\nC. $\\frac{\\sqrt{2}}{2}$\nD. $\\frac{\\sqrt{3}}{2}$", "subject": "解析几何", "analysis": "由题意可知: $M F_{1} \\perp M F_{2}$, 则 $\triangle F_{1} M F_{2}$ 为直角三角形, 由 $\\left|M F_{2}\right|=|M O|$, $O$ 为 $F_{1} F_{2}$ 中点, 则 $|O M|=\\left|O F_{2}\right|, \therefore \triangle O M F_{2}$ 为等边三角形, $\\angle O F_{2} M=60^{\\circ}:\\left|M F_{2}\right|=c, \\quad \therefore\\left|M F_{1}\right|=\\sqrt{3} c$, 由粗圆的定义可知: $\\mid M F_{1}$$|+| M F_{2} \\mid=2 a=\\sqrt{3} c+c=(\\sqrt{3}+1) c, a=\frac{(\\sqrt{3}+1)}{2}$, 则该椭圆的离心率 $e=\frac{c}{a}=\frac{c}{\frac{\\sqrt{3}+1}{2} c}=\\sqrt{3}-1$,"} {"id": "21134", "image": [], "answer": "B", "solution": "null", "level": "高三", "question": "直线 $l$ 经过粗圆的一个顶点和一个焦点, 若椭圆中心到 $l$ 的距离为其短轴长的 $\\frac{1}{4}$, 则该椭圆的离心率为 ( )", "options": "A. $\\frac{1}{3}$\nB. $\\frac{1}{2}$\nC. $\\frac{2}{3}$\nD. $\\frac{3}{4}$", "subject": "解析几何", "analysis": "设椭圆的方程为: $\frac{x^{2}}{a^{2}}+\frac{y^{2}}{b^{2}}=1$, 直线 $l$ 经过椭圆的一个顶点和一个焦点, 设直线方程为: $\frac{x}{c}+\frac{y}{b}=1$, 粗圆中心到 $l$ 的距离为其短轴长的 $\frac{1}{4}$, 可得: $\frac{1}{\\sqrt{\frac{1}{c^{2}}+\frac{1}{b^{2}}}}=\frac{b}{2}$, $\therefore 4=b^{2}\\left(\frac{1}{c^{2}}+\frac{1}{b^{2}}\right), \therefore \frac{b^{2}}{c^{2}}=3, \therefore \frac{a^{2}-c^{2}}{c^{2}}=3, \therefore \frac{a^{2}}{c^{2}}=4, \therefore e=\frac{c}{a}=\frac{1}{2}$. 故选: $B$."} {"id": "21135", "image": [], "answer": "A", "solution": "null", "level": "高三", "question": "已知陏圆 $C: \\frac{x^{2}}{a^{2}}+\\frac{y^{2}}{b^{2}}=1(a>b>0)$ 的左、右顶点分别为 $A_{1}, A_{2}$, 且以线段 $A_{1} A_{2}$ 为直径的圆与直线 $\\mathrm{bx}-\\mathrm{ay}+2 \\mathrm{ab}=0$ 相切, 则 $C$ 的离心率为 $(\\quad)$", "options": "A. $\\frac{\\sqrt{6}}{3}$\nB. $\\frac{\\sqrt{3}}{3}$\nC. $\\frac{\\sqrt{2}}{3}$\nD. $\\frac{1}{3}$", "subject": "解析几何", "analysis": "以线段 $A_{1} A_{2}$ 为直径的圆与直线 $b x-a y+2 a b=0$ 相切, $\therefore$ 原点到直线的距离 $\frac{2 a b}{\\sqrt{a^{2}+b^{2}}}=a$, 化为: $a^{2}=3 b^{2} . \therefore$ 椭圆 $C$ 的离心率 $e=\frac{c}{a}=\\sqrt{1-\frac{b^{2}}{a^{2}}}=\frac{\\sqrt{6}}{3}$. 故选 $A$."} {"id": "21136", "image": [], "answer": "B", "solution": "null", "level": "高三", "question": "已知 $F_{1}, F_{2}$ 是粗圆 $C: \\frac{x^{2}}{8}+\\frac{y^{2}}{4}=1$ 的两个焦点, 在 $C$ 上满足 $\\overrightarrow{P F_{1}} \\cdot \\overrightarrow{P F_{2}}=0$ 的点 $P$ 的个数为", "options": "A. 0\nB. 2\nC. 4\nD. 无数个", "subject": "解析几何", "analysis": "由 $C: \frac{x^{2}}{8}+\frac{y^{2}}{4}=1$, 得 $a=2 \\sqrt{2}, b=2, c=2, \because b=c=2, \therefore$ 以原点为圆心, $c$ 为半径的圆与椭圆有 2 个交点, $\therefore P F_{1} \\perp P F_{2}$ 的点 $P$ 的个数为 2 , 即满足 $\\overrightarrow{P F_{1}} \\cdot \\overrightarrow{P F_{2}}=0$ 的点 $P$ 的个数为 2 ,故选: $B$."} {"id": "21138", "image": [], "answer": "D", "solution": "null", "level": "高三", "question": "已知 $F_{1}, F_{2}$ 是椭圆 $C$ 的两个焦点, $P$ 是 $C$ 上的一点, 若 $P F_{1} \\perp P F_{2}$, 且 $\\angle P F_{2} F_{1}=60^{\\circ}$,则 $C$ 的离心率为 $(\\quad)$", "options": "A. $1-\\frac{\\sqrt{3}}{2}$\nB. $2-\\sqrt{3}$\nC. $\\frac{\\sqrt{3}-1}{2}$\nD. $\\sqrt{3}-1$", "subject": "解析几何", "analysis": "不妨设 $F_{1}, F_{2}$ 分别是椭圆的左、右焦点, $\because P F_{1} \\perp P F_{2}, \therefore \\Delta P F_{1} F_{2}$ 是直角三角形, 在 $R t \\Delta$ $P F_{1} F_{2}$ 中, $\\angle P F_{2} F_{1}=60^{\\circ},\\left|F_{1} F_{2}\right|=2 c, \therefore\\left|P F_{1}\right|=\\sqrt{3} c,\\left|P F_{2}\right|=c$. 由椭圆的定义可知 $\\left|P F_{1}\right|+\\left|P F_{2}\right|=2 a$, 即 $\\sqrt{3} c+c=2 a$, 则 $\frac{c}{a}=\frac{2}{\\sqrt{3}+1}=\\sqrt{3}-1$, 则 $C$ 的离心率为 $\\sqrt{3}-1$. 故选 $D$."} {"id": "22959", "image": [], "answer": "D", "solution": "null", "level": "高三", "question": "若 $\\mathrm{a}, \\mathrm{b}, \\mathrm{c}$ 为实数, 且 $a\\frac{a}{b}$\nD. $a^{2}>a b>b^{2}$", "subject": "代数", "analysis": "解析: 解答: 因为 $a\\frac{1}{b}, \\frac{b}{a}<1, \\frac{a}{b}>1$, 即 $\\frac{1}{a}<\\frac{1}{b}, \\frac{b}{a}>\\frac{a}{b}$ 均不成立; 当 $c^{2}=0$时, $a c^{2}0, b>0$, 若 $\\sqrt{3}$ 是 $3^{a}$ 与 $3^{b}$ 的等比中项, 则 $\\frac{1}{a}+\\frac{4}{b}$ 的最小值为", "options": "A. 8\nB. 9\nC. 4\nD. $\\frac{1}{4}$", "subject": "代数", "analysis": "解析: 解答: 由 $\\sqrt{3}$ 是 $3^{a}$ 与 $3^{b}$ 的等比中项, 所以 $3^{a} \\cdot 3^{b}=3$, 即 $3^{a+b}=3$, 所以 $\\mathrm{a}+\\mathrm{b}=1$.\n\n又 $\\mathrm{a}>0, \\mathrm{~b}>0$, 则 $\\frac{1}{a}+\\frac{4}{b}=(a+b)\\left(\\frac{1}{a}+\\frac{4}{b}\\right)=1+4+\\frac{b}{a}+\\frac{4 a}{b} \\geq 5+2 \\sqrt{\\frac{b}{a} \\cdot \\frac{4 a}{b}}=9$. 故选 D.\n\n分析: 本题主要考查了基本不等式在最值问题中的应用, 解决问题的关键是根据基本不等式分析计算即可"} {"id": "22961", "image": [], "answer": "D", "solution": "null", "level": "高三", "question": "设 $0b^{3}$\nB. $\\frac{1}{a}<\\frac{1}{b}$\nC. $a^{b}>1$\nD. $\\lg (b-a)<0$", "subject": "代数", "analysis": "解析: 解答: $\\because 0\\frac{a}{b}$\nD. $a^{2}>a b>b^{2}$", "subject": "代数", "analysis": "解析: 解答: 因为 $a\\frac{1}{b}, \\frac{b}{a}<1, \\frac{a}{b}>1$, 即 $\\frac{1}{a}<\\frac{1}{b}, \\frac{b}{a}>\\frac{a}{b}$ 均不成立; 当 $c^{2}=0$时, $a c^{2}|b|$\nB. $\\frac{1}{a}>\\frac{1}{b}$\nC. $a^{2}+b^{2}>2 a b$\nD. $a+b>-2 \\sqrt{a b}$", "subject": "代数", "analysis": "解析: 解答: 由不等式的基本性质可知 $\\mathrm{A} . \\mathrm{~B}$ 是正确的; 选项 C 是重要不等式 $a^{2}+b^{2} \\geq 2 a b$,由于 $a0)$, 则 $\\frac{1}{m}+\\frac{1}{n}$ 的最小值为 ( )", "options": "A. 1\nB. 2\nC. 3\nD. 4", "subject": "代数", "analysis": "解析: 解答: 由 $m+n=1(m n>0)$, 则 $\\frac{1}{m}+\\frac{1}{n}=\\frac{m+n}{m n}=\\frac{1}{m n} \\geq \\frac{1}{\\left(\\frac{m+n}{2}\\right)^{2}}=4$.故选 D.\n\n分析: 本题主要考查了基本不等式在最值问题中的应用, 解决问题的关键是根据基本不等式分析计算即可"} {"id": "21765", "image": [], "answer": "B", "solution": "null", "level": "高三", "question": "若 $f(x)$ 在 $x=x_{0}$ 处存在导数, 则 $\\lim _{h \\rightarrow 0} \\frac{f\\left(x_{0}+h\\right)-f\\left(\\mathrm{x}_{0}\\right)}{h}$ ())", "options": "A. 与 $x_{0}, h$ 都有关\n\nB. 仅与 $x_{0}$ 有关, 而与 $h$ 无关\n\nC. 仅与 $h$ 有关, 而与 $x_{0}$ 无关\n\nD. 以上答案都不对", "subject": "代数", "analysis": ""} {"id": "21767", "image": [], "answer": "C", "solution": "null", "level": "高三", "question": "设函数 $f(x)$ 在点 $x_{0}$ 附近有定义, 且有 $f\\left(x_{0}+\\Delta x\\right)-f\\left(x_{0}\\right)=a \\Delta x+b(\\Delta x)^{2}(a, b$ 为常数),则( )", "options": "A. $f^{\\prime}(x)=a$\nB. $f^{\\prime}(x)=b$\nC. $f^{\\prime}\\left(x_{0}\\right)=a$\nD. $f^{\\prime}\\left(x_{0}\\right)=b$", "subject": "代数", "analysis": ""} {"id": "21101", "image": [], "answer": "A", "solution": "null", "level": "高三", "question": "下列命题是真命题的为 ( )", "options": "A. 若 $\\frac{1}{x}=\\frac{1}{y}$ 则 $x=y$\nB. 若 $x^{2}=1$, 则 $x=1$\nC. 若 $x=y$, 则 $\\sqrt{x}=\\sqrt{y}$\nD. 则 $x2 x$\nC. $\\sqrt{x}+\\sqrt{1-x} \\leq 1$\nD. $|x-1|-|x+2| \\leq 3$", "subject": "代数", "analysis": "解析: 解答: 当 $x=-1$ 时, $x+\\frac{1}{x}=-2<2$, 故 $\\mathrm{A}$ 错;\n\n当 $x=1$ 时, $x^{2}+1=2$, 故 B 错;\n\n当 $x=\\frac{1}{2}$ 时, $\\sqrt{x}+\\sqrt{1-x}=\\sqrt{\\frac{1}{2}}+\\sqrt{\\frac{1}{2}}=\\sqrt{2}>1$, 故 C 错;\n\n由绝对值的几何意义知, $|x-1|-|x+2|$ 表示数轴上的点到 $A(1)$ 和 $B(-2)$ 的距离之差, 其最小值为 3 , 故 D 正确\n\n分析: 本题主要考查了绝对值不等, 解决问题的关键是根据绝对值的几何意义分析判断即可"} {"id": "22999", "image": [], "answer": "A", "solution": "null", "level": "高三", "question": "不等式 $\\left|\\frac{x-2}{x}\\right|>\\frac{x-2}{x}$ 的解集是 ( )", "options": "A. $(0,2)$\nB. $(-\\infty, 0)$\nC. $(2,+\\infty)$\nD. $(-\\infty, 0) \\cup(0,+\\infty)$", "subject": "代数", "analysis": "解析: 解答: 本题考查绝对值的含义,不等式的解法, 等价转化思想.\n\n因为 $a \\geq 0$ 时, $|a|=a ; a<0$ 时, $|a|=-a$, 则 $|a|>a \\Leftrightarrow a<0$; 所以不等式 $\\left|\\frac{x-2}{x}\\right|>\\frac{x-2}{x}$ 可化为 $\\frac{x-2}{x}<0$, 即 $x(x-2)<0$, 解得 $0|a-b|$\nB. $|a+b|<|a-b|$\nC. $|a-b|<|| a|-| b||$\nD. $|a-b|<|a|+|b|$", "subject": "代数", "analysis": "解析:解答:本题考查绝对值不等式的性质及推理能力. $\\because a b<0, \\therefore|a+b|^{2}-|a-b|^{2}=2 a b<0$\n\n所以 $|a+b|^{2}<|a-b|^{2}$, 所以 $|a+b|<|a-b|$.\n\n$|a-b|^{2}-\\|a|-| b\\|^{2}=-2 a b+2|a b|=-2 a b-2 a b>0$, 所以 $|a-b|>|| a|-| b \\| ;$\n\n$|a-b|^{2}-(|a|+|b|)^{2}=-2 a b-2|a b|=-2 a b+2 a b=0$, 所以 $|a-b|=|a|+|b|$;\n\n故选 B\n\n分析: 本题主要考查了绝对值不等, 解决问题的关键是根据绝对值的性质分析判断即可"} {"id": "23002", "image": [], "answer": "B", "solution": "null", "level": "高三", "question": "若实数 $a, b, c$ 满足 $|a-c|<|b|$, 则下列不等式中成立的是( )", "options": "A. $|a|>|b|-|c|$\nB. $|a|<|b|+|c|$\nC. $a>c-b$\nD. $a2^{n}$\nB. $n \\geq 3$ 时, $n^{2}>2^{n}$\nC. $n \\geq 4$ 时, $2^{n}>n^{2}$\nD. $n \\geq 5$ 时, $2^{n}>n^{2}$", "subject": "代数", "analysis": "解析: 解答: 当 $n=1$ 时, $2^{1}>1^{2}$, 即 $2^{n}>n^{2}$; 当 $n=2$ 时, $2^{2}=2^{2}$, 即 $2^{n}=n^{2}$; 当 $n=3$ 时,\n$2^{3}<3^{2}$, 即 $2^{n}5^{2}$, 即 $2^{n}>n^{2}$;当 $\\mathrm{n}=6$ 时, $2^{6}>6^{2}$; 猜测当 $\\mathrm{n} \\geq 5$ 时, $2^{n}>n^{2}$; 下面我们用数学归纳法证明猜测成立, (1)当 $\\mathrm{n}=5$ 时, 由以上可知猜测成立, (2) 设 $n=k(k \\geq 5)$ 时, 命题成立, 即 $2^{k}>k^{2}$, 当 $\\mathrm{n}=\\mathrm{k}+1$时, $2^{k+1}=2 \\cdot 2^{k}>2 k^{2}=k^{2}+k^{2}>k^{2}+(2 k+1)=(k+1)^{2}$, 即 $\\mathrm{n}=\\mathrm{k}+1$ 时, 命题成立, 由 (1)和 (2) 可得 $n \\geq 5$ 时, $2^{n}$ 与 $n^{2}$ 的大小关系为: $2^{n}>n^{2}$; 故选 D.\n\n分析: 本题主要考查了归纳推理, 解决问题的关键是根据归纳方法进行发现计算即可."} {"id": "23005", "image": [], "answer": "C", "solution": "null", "level": "高三", "question": "设 $a, b>0, A=\\sqrt{a}+\\sqrt{b}, B=\\sqrt{a+b}$, 则 $A, B$ 的大小关系是( )", "options": "A. $A=B$\nB. $AB$\nD. 大小不确定", "subject": "代数", "analysis": "解析:【解答】用综合法:\n\n$$\n(\\sqrt{a}+\\sqrt{b})^{2}=a+2 \\sqrt{a b}+b\n$$\n\n所以 $A^{2}-B^{2}>0$. 所以 $A^{2}>B^{2}$.\n\n又 $A>0, B>0$,\n\n所以 $A>B$.\n\n【分析】本题主要考查了分析法与综合法, 解决问题的关键是对所给关系式 $A, B$ 两边平方作差比较即可."} {"id": "23006", "image": [], "answer": "C", "solution": "null", "level": "高三", "question": "要证明 $\\sqrt{a}+\\sqrt{a+7}<\\sqrt{a+3}+\\sqrt{a+4}(a \\geq 0)$ 可选择的方法有多种, 其中最合理的是", "options": "A. 综合法\nB. 类比法\nC.分析法\nD.归纳法", "subject": "代数", "analysis": "解析:【解答】要证 $\\sqrt{a}+\\sqrt{a+7}<\\sqrt{a+3}+\\sqrt{a+4}$,\n\n只需证\n\n$2 a+7+2 \\sqrt{a(a+7)}<2 a+7+2 \\sqrt{(a+3)(a+4)}$\n\n$2 a+7+2 \\sqrt{a(a+7)}<2 a+7+2 \\sqrt{(a+3)(a+4)}$, 只需证 $\\sqrt{a(a+7)}<\\sqrt{(a+3)(a+4)}$,\n\n只需证 $a(a+7)<(a+3)(a+4)$,\n\n只需证 $0<12$,\n\n故选用分析法最合理.\n\n【分析】本题主要考查了分析法与综合法, 解决问题的关键是根据分析法证明不等式的步骤分析计算即可."} {"id": "23008", "image": [], "answer": "C", "solution": "null", "level": "高三", "question": "分析法又叫执果索因法, 若使用分析法证明: 设 $a>b>c$, 且 $a+b+c=0$, 求证:\n\n$\\sqrt{b^{2}-a c}<\\sqrt{3} a$ 索的因应是 $(\\quad)$", "options": "A. $a-b>0$ B. $a-c>0$ C. $(a-b)(a-c)>0$ D. $(a-b)(a-c)<0$", "subject": "代数", "analysis": "解析:【解答】要证 $\\sqrt{b^{2}-a c}<\\sqrt{3} a$,\n\n只需证 $b^{2}-a c<3 a^{2} b^{2}-a c<3 a^{2}$, 只需证 $b^{2}-a(-b-a)<3 a^{2}$,\n\n只需证 $2 a^{2}-a b-b^{2}>0$, 只需证 $(2 a+b)(a-b)>0$, 只需证\n\n$(a-c)(a-b)>0(a-c)(a-b)>0$, 故索的因应为 C.\n\n【分析】本题主要考查了分析法与综合法, 解决问题的关键是根据分析法由结论解析逆推即可"} {"id": "23022", "image": [], "answer": "A", "solution": "null", "level": "高三", "question": "若 $a, b, c$ 是常数, 则 “ $a>0$, 且 $b^{2}-4 a c<0$ ”是“对任意 $x \\in \\mathbf{R}$, 有 $a x^{2}+b x+c>0$ ”的( )", "options": "A. 充分不必要条件 B. 必要不充分条件 C. 充要条件 D. 既不充分也不必要条件", "subject": "代数", "analysis": "解析: 【解答】因为 $a>0$, 且 $b^{2}-4 a<0 \\Rightarrow a x^{2}+b x+c>0$ 对任意 $x \\in \\mathbf{R}$ 恒成立.反之, $a x^{2}$ $+b x+c>0$ 对任意 $x \\in \\mathbf{R}$ 恒成立不能推出 $a>0$, 且 $b^{2}-4 a c<0$, 反例为: 当 $a=b=0$, 且 $c$ $>0$ 时也有 $a x^{2}+b x+c>0$ 对任意 $x \\in \\mathbf{R}$ 恒成立, 所以“ $a>0$, 且 $b^{2}-4 a c<0$ ”是“对任意 $x \\in \\mathbf{R}$,有 $a x^{2}+b x+c>0$ ”的充分不必要条件\n\n【分析】本题主要考查了分析法与综合法, 解决问题的关键是根据所给不等式结合充分条件与必要条件成立的关系解析分析验证即可."} {"id": "23025", "image": [], "answer": "D", "solution": "null", "level": "高三", "question": "函数 $f(x)=(x-3) e^{x}$ 的单调递增区间是 $(\\quad)$", "options": "A. $(-\\infty, 2)$ B. $(0,3)$ C. $(1,4)$ D. $(2,+\\infty)$", "subject": "代数", "analysis": "解析: 【解答 $f^{\\prime}(x)=(x-3)^{\\prime} \\mathrm{e}^{x}+(x-3) \\cdot\\left(\\mathrm{e}^{x}\\right)^{\\prime}=(x-2) \\mathrm{e}^{x}$, 令 $f^{\\prime}(x)>0$, 解得 $x>2$,故选 D.\n\n【分析】本题主要考查了分析法与综合法, 解决问题的关键是根据所给函数求导, 利用导数\n的性质分析求解即可得到函数的得到区间."} {"id": "20495", "image": [], "answer": "D", "solution": "null", "level": "高三", "question": "(1)已知 $p^{3}+q^{3}=2$, 求证: $p+q \\leq 2$. 用反证法证明时, 可假设 $p+q \\geq 2$;\n\n(2) 若 $a 、 b \\in R,|a|+|b|<1$, 求证: 方程 $x^{2}+a x+b=0$ 的两根的绝对值都小于 1 . 用反证法证明时可假设方程有一根 $x_{1}$ 的绝对值大于或等于 1 , 即假设 $\\left|x_{1}\\right| \\geq 1$;\n\n以下结论正确的是 ( )", "options": "A. (1)与(2)的假设都错误\nB. (1)的假设正确; (2)的假设错误\nC. (1)与(2)的假设都正确\nD. (1)的假设错误; (2)的假设正确", "subject": "代数", "analysis": "解析:解答:(1)的结论的否定应该是 $p+q>2$, 故错; (2)的否定是方程的两根至少有一个大于等于 1 , 故(2)正确.\n\n分析: 本题主要考查了反证法, 解决问题的关键是根据反证法的步骤特征进行具体分析即可."} {"id": "20496", "image": [], "answer": "B", "solution": "null", "level": "高三", "question": "不相等的三个正数 $\\mathrm{a} 、 \\mathrm{~b} 、 \\mathrm{c}$ 成等差数列, 并且 $x$ 是 $\\mathrm{a} 、 \\mathrm{~b}$ 的等比中项, $\\mathrm{y}$ 是 $\\mathrm{b} 、 \\mathrm{c}$ 的等比中项, 则 $x^{2} 、 \\mathrm{~b}^{2} 、 \\mathrm{y}^{2}$ 三数( )", "options": "A. 成等比数列而非等差数列\n\nB. 成等差数列而非等比数列\n\nC. 既成等差数列又成等比数列\n\nD. 既非等差数列又非等比数列", "subject": "代数", "analysis": "解析:解答:由已知条件, 可得\n\n$\\left\\{\\begin{array}{l}a+c=2 b(1) \\\\ x^{2}=a b(2) \\\\ y^{2}=b c(3)\\end{array}\\right.$\n\n由(2)(3)得 $\\left\\{\\begin{array}{l}a=\\frac{x^{2}}{b} \\\\ c=\\frac{y^{2}}{b}\\end{array}\\right.$\n\n代入(1), 得 $\\frac{x^{2}}{b}+\\frac{y^{2}}{b}=2 \\mathrm{~b}$,\n\n即 $x^{2}+\\mathrm{y}^{2}=2 \\mathrm{~b}^{2}$.\n\n故 $x^{2} 、 \\mathrm{~b}^{2} 、 \\mathrm{y}^{2}$ 成等差数列,\n\n故选 B.\n\n分析: 本题主要考查了综合法的思考过程、特点及应用, 解决问题的关键是根据所给条件进\n行综合分析得到对应的结论即可得到正确选项."} {"id": "20497", "image": [], "answer": "C", "solution": "null", "level": "高三", "question": "设 $\\mathrm{a}, \\mathrm{b}$ 是两个实数, 给出下列条件:\n\n(1) $a+b>1$; (2) $a+b=2$; (3) $a+b>2$; (4) $a^{2}+b^{2}>2$; (5) $a b>1$.\n\n其中能推出: “a, b 中至少有一个大于 1 ”的条件是( )", "options": "A. (2)(3)\nB. (1)(2)(3)\nC. (3)\nD. (3)(4)(5)", "subject": "代数", "analysis": "解析: 解答: (1)中若 $\\mathrm{a}=\\frac{3}{4}, \\mathrm{~b}=\\frac{1}{2}$, 则 $\\mathrm{a}+\\mathrm{b}>1$, 故(1)不能; (2)中若 $\\mathrm{a}=\\mathrm{b}=1$, 则 $\\mathrm{a}+\\mathrm{b}=2$,故(2)不能; (3)能, (4)中若 $a=b=-2$, 则 $a^{2}+b^{2}>2$, 故(4)不能; (5)中若 $a=b=-2$, 则 $a b>1$,故(5)不能. $\\therefore$ 只有(3)能, 选 C.\n\n分析: 本题主要考查了分析法, 解决问题的关键是根据所给条件与结论运用分析法进行具体分析即可."} {"id": "20498", "image": [], "answer": "C", "solution": "null", "level": "高三", "question": "分析法又称执果索因法, 若用分析法证明: “设 $a>b>c$, 且 $a+b+c=0$, 求证 $\\sqrt{b^{2}-a c}<\\sqrt{3}$ a” 索的因应是( )", "options": "A. $\\mathrm{a}-\\mathrm{b}>0$\nB. $a-c>0$\nC. $(a-b)(a-c)>0$\nD. $(a-b)(a-c)<0$", "subject": "代数", "analysis": "解析: 解答: $\\sqrt{b^{2}-a c}<\\sqrt{3}$ a\n\n$\\Leftrightarrow \\mathrm{b}^{2}-\\mathrm{ac}<3 \\mathrm{a}^{2}$\n\n$\\Leftrightarrow(\\mathrm{a}+\\mathrm{c})^{2}-\\mathrm{ac}<3 \\mathrm{a}^{2}$\n\n$\\Leftrightarrow \\mathrm{a}^{2}+2 \\mathrm{ac}+\\mathrm{c}^{2}-\\mathrm{ac}-3 \\mathrm{a}^{2}<0$\n\n$\\Leftrightarrow-2 \\mathrm{a}^{2}+\\mathrm{ac}+\\mathrm{c}^{2}<0$\n\n$\\Leftrightarrow 2 \\mathrm{a}^{2}-\\mathrm{ac}-\\mathrm{c}^{2}>0$\n\n$\\Leftrightarrow(a-c)(2 a+c)>0$\n\n$\\Leftrightarrow(\\mathrm{a}-\\mathrm{c})(\\mathrm{a}-\\mathrm{b})>0$.\n\n分析: 本题主要考查了分析法的思考过程、特点及应用, 解决问题的关键是根据所给条件与所求结论运用分析法进行分析推理即可."} {"id": "20511", "image": [], "answer": "B", "solution": "null", "level": "高三", "question": "要证明 $\\sqrt{3}+\\sqrt{7}<2 \\sqrt{5}$ 可选择的方法有以下几种, 其中最合理的是 ( )", "options": "A. 综合法\nB. 分析法\nC. 反证法\nD. 归纳法", "subject": "代数", "analysis": "解析: 解答: 综合法由已知条件入手开始证明, 分析法从所求的结论入手寻找使其成立的条件, 反证法适合证明含有存在, 唯一等字眼的题目, 归纳法证明与正整数有关的题目; 结合以上特点, 本题的证明适合采用分析法\n\n分析: 本题主要考查了分析法的思考过程、特点及应用; 综合法的思考过程、特点及应用,\n解决问题的关键是根据所给选项结合对应的方法进行具体分析即可."} {"id": "20514", "image": [], "answer": "A", "solution": "null", "level": "高三", "question": "若 $a>0, b>0$, 那么必有 ( )", "options": "A. $a^{3}+b^{3} \\geqslant a^{2} b+a b^{2}$\nB. $a^{3}+b^{3}>a^{2} b+a b^{2}$\nC. $a^{3}+b^{3} \\leqslant a^{2} b+a b^{2}$\nD. $a^{3}+b^{3}a^{2}, \\mathrm{~b}>b^{2}, \\mathrm{a}+\\mathrm{b}>a^{2}+b^{2}>2 a b$由均值定理 $\\mathrm{a}+\\mathrm{b}>2 \\sqrt{a b}$, 即 $\\mathrm{a}+\\mathrm{b}$ 最大, 故选 A 分析: 本题主要考查了分析法和综合法, 解决问题的关键是根据分析法和综合法进行分析比较即可."} {"id": "23046", "image": [], "answer": "D", "solution": "null", "level": "高三", "question": "(1)已知 $p^{3}+q^{3}=2$, 求证 $p+q \\leq 2$. 用反证法证明时, 可假设 $p+q \\geq 2$. (2)已知 $a, b \\in \\mathbf{R},|a|$ $+|b|<1$, 求证方程 $x^{2}+a x+b=0$ 的两根的绝对值都小于 1 . 用反证法证明时可假设方程有一根 $x_{1}$ 的绝对值大于或等于 1 , 即假设 $\\left|x_{1}\\right| \\geq 1$. 以下结论正确的是 ( )", "options": "A.(1)与(2)的假设都错误\nB.(1)与(2)的假设都正确\nC.(1)的假设正确; (2)的假设错误\nD.(1)的假设错误; (2)的假设正确", "subject": "代数", "analysis": "解析:【解答】(1)的假设应为 $p+q>2$; (2)的假设正确\n\n【分析】本题主要考查了反证法与放缩法, 解决问题的关键是根据反证法证明的步骤分析即可."} {"id": "22666", "image": [], "answer": "C", "solution": "null", "level": "高三", "question": "已知 $\\odot \\mathrm{O}$ 的弦 $\\mathrm{AB}$ 过弦 $\\mathrm{CD}$ 的三等分点 $\\mathrm{M}, \\mathrm{AM}$ 和 $\\mathrm{BM}$ 是方程 $3 \\mathrm{x}^{2}+2 \\mathrm{mx}+18=0$ 的两个根,则 $\\mathrm{CD}$ 的长为 ( )", "options": "A. $\\sqrt{3}$\nB. $2 \\sqrt{3}$\nC. $3 \\sqrt{3}$\nD. $4 \\sqrt{3}$", "subject": "代数", "analysis": "解析: 解答: $\\because \\mathrm{AM}$ 和 $\\mathrm{BM}$ 是 $3 \\mathrm{x}^{2}+2 \\mathrm{mx}+18=0$ 的两根, $\\therefore \\mathrm{AM} \\cdot \\mathrm{BM}=\\frac{18}{3}=6$.\n\n又 $\\mathrm{AB}$ 和 $\\mathrm{CD}$ 相交于点 $\\mathrm{M}, \\therefore \\mathrm{CM} \\cdot \\mathrm{MD}=\\mathrm{AM} \\cdot \\mathrm{BM}=6$.\n\n$\\therefore \\frac{1}{3} \\mathrm{CD} \\cdot \\frac{2}{3} \\mathrm{CD}=6, \\therefore \\mathrm{CD}=3 \\sqrt{3}$.\n\n分析: 本题主要考查了与圆有关的比例线段, 解决问题的关键是根据与圆有关的比例线段分析满足的性质计算即可"} {"id": "20805", "image": [], "answer": "B", "solution": "null", "level": "高三", "question": "已知 $z_{1}=m^{2}-3 m+m^{2} \\mathrm{i}, z_{2}=4+(5 m+6) \\mathrm{i}$, 其中 $\\mathrm{m}$ 为实数, $\\mathrm{i}$ 为虚数单位, 若 $z_{1}-z_{2}=0$, 则 $\\mathrm{m}$ 的值为 $(\\quad)$", "options": "A. 4\nB. -1\nC. 6\nD. 0", "subject": "代数", "analysis": "答案: B\n\n解析:由题意可得 $\\left\\{\\begin{array}{l}m^{2}-3 m=4, \\\\ m^{2}=5 m+6, \\therefore m=-1 .\\end{array}\\right.$"} {"id": "20806", "image": [], "answer": "D", "solution": "null", "level": "高三", "question": "已知对于 $x$ 的方程 $x^{2}+x+3 m-(2 x+1) i=0$ 有实根, 则实数 $m$ 满足 ( )", "options": "A. $m \\leq-\\frac{1}{4}$\nB. $\\geq-\\frac{1}{4}$\nC. $m=-\\frac{1}{12}$\nD. $m=\\frac{1}{12}$", "subject": "代数", "analysis": "答案: D\n\n解析: 由已知 $\\left\\{\\begin{array}{c}x^{2}+x+3 m=0(1) \\\\ -2 x-1=0(2)\\end{array} \\quad x=-\\frac{1}{2}\\right.$ 由(2)解得 $\\quad$ 代入(1)中, 解得 $m=\\frac{1}{12}$. 故选 D."} {"id": "20810", "image": [], "answer": "D", "solution": "null", "level": "高三", "question": "设复数 $z=\\frac{2 \\mathrm{i}}{1+\\mathrm{i}}$, 则复数 $z$ 的共轭复数 $z$ 在复平面内对应的点在( )", "options": "A.第一象限\nB.第二象限\nC.第三象限\nD.第四象限", "subject": "代数", "analysis": "答案: D\n\n解析: 因为 ${ }^{z=\\frac{2 \\mathrm{i}}{1+\\mathrm{i}}}=1+\\mathrm{i}$, 所以 $\\bar{z}=1-\\mathrm{i}, \\bar{z}$ 在复平面内对应的点为 ${ }^{(1,-1)}$, 在第四象限.故选 D."} {"id": "20812", "image": [], "answer": "A", "solution": "null", "level": "高三", "question": "已知复数 $z$ 满足 ${ }^{(1-i) z=-2-2 i}$, 则 $|z|=(\\quad)$", "options": "A. 2\nB. $\\sqrt{2}$\nC. 1\nD. $\\frac{\\sqrt{2}}{2}$", "subject": "代数", "analysis": "答案: A\n\n解析:由题意得 $z=-2 \\times \\frac{1+\\mathrm{i}}{1-\\mathrm{i}}=-(1+\\mathrm{i})^{2}=-2 \\mathrm{i}$, 所以 $|z|=2$, 故选 A."} {"id": "20813", "image": [], "answer": "B", "solution": "null", "level": "高三", "question": "已知复数 $z=\\frac{2}{1-i}$, 则 $|z|$ 等于()", "options": "A. 1\nB. $\\sqrt{2}$\nC. 2\nD. $2 \\sqrt{2}$", "subject": "代数", "analysis": "答案: B\n\n解析: $\\because$ 复数 $z=\\frac{2}{1-i}=\\frac{2(1+i)}{(1-i)(1+i)}=\\frac{2(1+i)}{1-i^{2}}=1+i$\n\n$\\therefore|z|=\\sqrt{1+1}=\\sqrt{2}$\n\n故选 B"} {"id": "20814", "image": [], "answer": "A", "solution": "null", "level": "高三", "question": "若复数 $z$ 满足 $z(1-2 i)=3-i$ ( $i$ 为虚数单位), 则复数 $z$ 的共轭复数为 $(\\quad)$", "options": "A. $1-\\mathrm{i}$\nB. $1+\\mathrm{i}$\nC. $-1-\\mathrm{i}$\nD. $-1+\\mathrm{i}$", "subject": "代数", "analysis": "答案: A\n\n解析: 因为 $z(1-2 \\mathrm{i})=3-\\mathrm{i}$, 所以 $z=\\frac{3-\\mathrm{i}}{1-2 \\mathrm{i}}=\\frac{(3-\\mathrm{i})(1+2 \\mathrm{i})}{(1-2 \\mathrm{i})(1+2 \\mathrm{i})}=1+\\mathrm{i}$, 所以复数 $z$ 的共轭复数为 $1-\\mathrm{i}$. 故选\n\nA."} {"id": "20818", "image": ["9483.jpg"], "answer": "B", "solution": "null", "level": "高三", "question": "执行下面的程序框图,如果输入的 $a=4, b=6$, 那么输出的 $n=(\\quad)$\n\n", "options": "A. 3\nB. 4\nC. 5\nD. 6", "subject": "代数", "analysis": "答案: B\n\n解析:程序运行如下:\n\n开始 $a=4, b=6, n=0, s=0$.\n\n第1次循环: $a=2, b=4, a=6, s=6, n=1$ ;\n\n第2 次循环: $a=-2, b=6, a=4, s=10, n=2$;\n\n第3 次循环: $a=2, b=4, a=6, s=16, n=3$;\n\n第4 次循环: $a=-2, b=6, a=4, s=20, n=4$.\n\n此时, 满足条件 $s>16$, 退出循环, 输出 $n=4$."} {"id": "20826", "image": ["9488.jpg"], "answer": "C", "solution": "null", "level": "高三", "question": "如图所示是数列一章的知识结构图,下列说法正确的是( )\n\n", "options": "A.“概念”与“分类”是从属关系\n\nB. “等差数列”与“等比数列”是从属关系\n\nC.“数列”与 “等差数列”是从属关系\n\nD.“数列”与与“等差数列”是从属关系,但“数列”与“分类”不是从属关系", "subject": "代数", "analysis": "画某一章节的知识结构图时, 首先应对本章节的知识有全面的把握, 然后明确各知识点之间在逻辑上的先后顺序、概念上的从属关系.按从上到下、从左到右的顺序画图, 在 $\\mathrm{A} 、 \\mathrm{~B} 、 \\mathrm{C} 、 \\mathrm{D}$ 四个选项中只有 C 正确."} {"id": "20831", "image": ["9493.jpg"], "answer": "C", "solution": "null", "level": "高三", "question": "在下面的知识结构图中:\n\n\n\n“求简单函数的导数”的“上位”要素有( )", "options": "A. 1 个\nB. 2 个\nC. 3 个\nD. 4 个", "subject": "代数", "analysis": "由结构图可知, “求简单函数的导数”的 “上位”要素有“基本导数公式”“函数四则运算求导法则”“复合函数求导法则”这 3 个."} {"id": "20839", "image": [], "answer": "D", "solution": "null", "level": "高三", "question": "设复数 $z=\\frac{2 i}{1+i}$, 则复数 $z$ 的共轭复数 $z$ 在复平面内对应的点在 $(\\quad)$", "options": "A. 第一象限\nB. 第二象限\nC.第三象限\nD.第四象限", "subject": "代数", "analysis": "D"} {"id": "20840", "image": [], "answer": "D", "solution": "null", "level": "高三", "question": "已知复数 ${ }^{z}$ 满足 ${ }^{z=\\frac{2+\\mathrm{i}}{\\mathrm{i}}}$, 其中 ${ }^{\\mathrm{i}}$ 是虚数单位, 则 ${ }^{z}$ 在复平面内对应的点位于 ( )", "options": "A. 第一象限\nB. 第二象限\nC. 第三象限\nD. 第四象限", "subject": "代数", "analysis": "答案: D\n\n解析: $z=\\frac{2+\\mathrm{i}}{\\mathrm{i}}=\\frac{(2+\\mathrm{i}) \\mathrm{i}}{\\mathrm{i}^{2}}=1-2 \\mathrm{i}$, 所以 ${ }^{z}$ 在复平面对应的点 $(1,-2)$ 位于第四象限, 故选 D"} {"id": "20841", "image": [], "answer": "A", "solution": "null", "level": "高三", "question": "已知复数 $z$ 满足 $(1+i) z=2 i$, 则 $z$ 在复平面内对应的点位于 $(\\quad)$", "options": "A.第一象限\nB.第二象限\nC.第三象限\nD.第四象限", "subject": "代数", "analysis": "答案: A\n\n解析: 由题, $z=\\frac{2 \\mathrm{i}}{1+\\mathrm{i}}=\\frac{2 \\mathrm{i}(1-\\mathrm{i})}{(1+\\mathrm{i})(1-\\mathrm{i})}=\\frac{2+2 \\mathrm{i}}{2}=1+\\mathrm{i}$, 所以 $z$ 在复平面内对应的点为 $(1,1)$,\n\n故选:A"} {"id": "20843", "image": [], "answer": "D", "solution": "null", "level": "高三", "question": "设复数 $z$ 满足 $z(1+i)=2$, 则 $z$ 在复平面内所对应的点位于 $(\\quad)$", "options": "A. 第一象限\nB. 第二象限\nC. 第三象限\nD. 第四象限", "subject": "代数", "analysis": "答案: D\n\n解析: 由 ${ }^{z(1+i)=2}$, 得 ${ }^{z=\\frac{2}{1+\\mathrm{i}}=1-\\mathrm{i}}, \\therefore^{z}$ 在复平面内所对应的点坐标为 $(1,-1)$, 位于第四象限"} {"id": "21323", "image": [], "answer": "D", "solution": "null", "level": "高三", "question": "下列各式中正确的是 ( )", "options": "A. $\\left(\\log _{a} x\\right)^{\\prime}=\\frac{1}{x}$\nB. $\\left(\\log _{2} x\\right)^{\\prime}=\\frac{\\ln 10}{x}$\n\nC. D.", "subject": "代数", "analysis": "由 $\\left(\\log _{a} x\right)^{\\prime}=\frac{1}{x \\ln a}$, 可知 $\\mathrm{A} 、 \\mathrm{~B}$ 均错由, 可知 $\\mathrm{D}$ 正确,故选 D."} {"id": "21324", "image": [], "answer": "D", "solution": "null", "level": "高三", "question": "已知函数 $f(x)=\\ln x-\\frac{1}{2} a x^{2}+(a-1) x+a(a>0)$ 的值域与函数\n\n$y=f(f(x))$ 的值域相同, 则实数 $a$ 的取值范围为()", "options": "A. $(0,1]$\nB. $(1,+\\infty)$\nC. $\\left(0, \\frac{4}{3}\\right]$\nD. $\\left[\\frac{4}{3},+\\infty\\right)$", "subject": "代数", "analysis": "由题可知,函数 $f(x)$ 的定义域为 $(0,+\\infty)$.$\because f(x)=\\ln x-\frac{1}{2} a x^{2}+(a-1) x+a(a>0)$,$\therefore f^{\\prime}(x)=\frac{1}{x}-a x+a-1=\frac{(a x+1)(1-x)}{x}$,$\therefore$ 当 $x>1$ 时, $f^{\\prime}(x)<0$; 当 $00$, $\therefore f(x)$ 在 $(0,1)$ 上单调递增, 在 $(1,+\\infty)$ 上单调递减, $\therefore f(x)_{\text {max }}=f(1)=\frac{3}{2} a-1$, 即 $f(x)$ 的值域为 $\\left(-\\infty, \frac{3}{2} a-1\right]$.要使 $y=f(f(x))$ 的值域也为 $\\left(-\\infty, \frac{3}{2} a-1\right]$, 则只要 $f(x)_{\text {max }} \\geq 1$,则 $\frac{3}{2} a-1 \\geq 1$, 即 $a \\geq \frac{4}{3}$, 故选 D."} {"id": "21325", "image": [], "answer": "C", "solution": "null", "level": "高三", "question": "已知函数 $f(x)=x+\\ln x, g(x)=x \\ln x$, 若 $f\\left(x_{1}\\right)=\\ln t, g\\left(x_{2}\\right)=t$,则 $x_{1} x_{2} \\ln t$ 的最小值为 ( )\n\n$\\begin{array}{llll}\\text {", "options": "A. } \\frac{1}{e^{2}} & \\text { B. } \\frac{2}{e} & \\text { C. }-\\frac{1}{e} & \\text { D. }-\\frac{1}{e^{2}}\\end{array}$", "subject": "代数", "analysis": "$\because f\\left(x_{1}\right)=x_{1}+\\ln x_{1}=\\ln t, \therefore t=e^{x_{1}} \\cdot x_{1}$ (1), $g\\left(x_{2}\right)=x_{2} \\ln x_{2}=t, \\quad \therefore t=e^{\\ln x_{2}} \\cdot \\ln x_{2}(2)$,由(1)(2)得 $e^{x_{1}} \\cdot x_{1}=e^{\\mathrm{h} x_{2}} \\cdot \\ln x_{2}$,$\because y=x e^{x}$ 在 $(0,+\\infty)$ 单调递增, $\therefore x_{1}=\\ln x_{2}$, 则 $x_{1} x_{2}=t$,$x_{1} x_{2} \\ln t=t \\ln t$,令 $h(x)=t \\ln t(t>0)$, 则 $h^{t}(t)=\\ln t+1$,令 $h^{t}(t)>0$, 解得 $t>\frac{1}{e}$; 令 $h^{t}(t)<0$, 解得 $0 要使方程 $F(x)=f(x)-a x$ 有 4 个零点, 则 $f(x)$ 的图象与直线 $y=a x$ 有 4 个不同的交点,所以只需 $a$ 小于 $y=\\ln x$ 在区间 $[1, e]$ 上的过坐标原点的切线的斜率即可.由 $y=\\ln x$, 得 $y^{t}=\frac{1}{x}$, 设切点坐标为 $\\left(x_{0}, y_{0}\right)$, 则切线方程为 $y-\\ln x_{0}=\frac{1}{x_{0}}\\left(x-x_{0}\right)$,又切线过 $(0,0)$, 所以 $-\\ln x_{0}=\frac{1}{x_{0}}\\left(-x_{0}\right)$, 解得 $x_{0}=e$, 故此时切线的斜率为 $\frac{1}{x_{0}}=\frac{1}{e}$, 故 $a \\in\\left(0, \frac{1}{e}\right)$, 结合选项知, 选 A."} {"id": "21337", "image": ["9548.jpg"], "answer": "D", "solution": "null", "level": "高三", "question": "已知函数 $f(x)$ 的导函数为 $f^{\\prime}(x)$, 函数 $g(x)=(x-1) f^{\\prime}(x)$ 的图象如图所示,则下列结论正确的是 (\n\n", "options": "A. $f(x)$ 在 $(-\\infty,-2),(1,2)$ 上为减函数\n\nB. $f(x)$ 在 $(-2,1),(2,+\\infty)$ 上为增函数\n\nC. $f(x)$ 的极小值为 $f(-2)$, 极大值为 $f(2)$\n\nD. $f(x)$ 的极大值为 $f(-2)$, 极小值为 $f(2)$", "subject": "代数", "analysis": "根据函数 $g(x)=(x-1) f^{\\prime}(x)$ 的图象可知:当 $x>2$ 时, $g(x)>0$, 即 $(x-1) f^{\\prime}(x)>0 \\Rightarrow f^{\\prime}(x)>0$, 因此当 $x>2$ 时, 函数 $f(x)$ 单调递增;当 $10$, 即 $(x-1) f^{\\prime}(x)>0 \\Rightarrow f^{\\prime}(x)<0$, 因此当 $-20$, 因此当 $x<-2$ 时, 函数 $f(x)$ 单调递增,显然当 $x=-2$, 函数有极大值, 极大值为 $f(-2)$,由上可以判断 D 是正确的, 故选 D."} {"id": "21339", "image": [], "answer": "D", "solution": "null", "level": "高三", "question": "若函数 $f(x)=a x^{3}+3 x^{2}+x+b(a>0, b \\in \\mathbf{R})$ 恰好有三个不同的单调区间,\n\n则实数 $a$ 的取值范围是 ( )", "options": "A. $(0,3) \\cup(3,+\\infty)$\nB. $[3,+\\infty)$\nC. $(0,3]$\nD. $(0,3)$", "subject": "代数", "analysis": "由题意得 $f^{\\prime}(x)=3 a x^{2}+6 x+1(a>0)$,$\because$ 函数 $f(x)$ 恰好有三个不同的单调区间, $\therefore f^{\\prime}(x)$ 有两个不同的零点,所以, $\\left\\{\begin{array}{l}\\Delta=36-12 a>0 \\ a>0\\end{array}\right.$, 解得 $00$, 若函数 $g(x)$ 满足 $e^{x} g(x)=f(x)$, 下列结论错误的是 ( )", "options": "A. 函数 $g(x)$ 在 $(2,+\\infty)$ 上为增函数\nB. $x=2$ 是函数 $\\mathrm{g}(x)$ 的极小值点\n\nC. $x \\leq 0$ 时, 不等式 $f(x) \\leq 2 e^{x}$ 恒成立 D. 函数 $g(x)$ 至多有两个零点", "subject": "代数", "analysis": "$\because e^{x} g(x)=f(x), \therefore g(x)=\frac{f(x)}{e^{x}}$, 则 $g^{\\prime}(x)=\frac{f^{\\prime}(x)-f(x)}{e^{x}}$,由题意得当 $x>2$ 时, $f^{\\prime}(x)-f(x)>0$, 故 $y=g(x)$ 在 $(2,+\\infty)$ 递增,选项 A 正确当 $x<2$ 时, $f^{\\prime}(x)-f(x)<0$, 故 $y=g(x)$ 在 $(-\\infty, 2)$ 递减,故 $x=2$ 是函数 $y=g(x)$ 的极小值点, 故选项 B 正确;由 $y=g(x)$ 在 $(-\\infty, 2)$ 递减, 则 $y=g(x)$ 在 $(-\\infty, 0)$ 递减,由 $g(0)=\frac{f(0)}{e^{0}}=2$, 得 $x \\leq 0$ 时, $g(x) \\geq g(0)$,$\therefore \frac{f(x)}{e^{x}} \\geq 2$, 故 $f(x) \\geq 2 e^{x}$, 故选项 C 错误;若 $g(2)<0$, 则 $y=g(x)$ 有 2 个零点:若 $\boldsymbol{g}(2)=0$ ,则函数 $y=g(x)$ 有 1 个零点;若 $g(2)>0$, 则函数 $y=g(x)$ 没有零点, 故选项 D 正确,故选 C."} {"id": "21366", "image": [], "answer": "A", "solution": "null", "level": "高三", "question": "设 $a>0, b>0, e$ 是自然对数的底数 ( )", "options": "A. 若 $e^{a}+2 a=e^{b}+3 b$, 则 $a>b$\nB. 若 $e^{a}+2 a=e^{b}+3 b$, 则 $ab$\n\nD. 若 $e^{a}-2 a=e^{b}-3 b$, 则 $ae^{b}+2 b$构造函数: $f(x)=e^{x}+2 x$, 则 $f(a)>f(b)$, 则 $f^{\\prime}(x)=e^{x}+2>0$ 恒成立,故有函数 $f(x)=e^{x}+2 x$ 在 $x>0$ 上单调递增, 所以 $a>b$ 成立,故选 A."} {"id": "22962", "image": [], "answer": "B", "solution": "null", "level": "高三", "question": "若 $a, b, c$ 为实数, 则下列命题正确的是 ( )", "options": "A. 若 $a>b$, 则 $a c^{2}>b c^{2}$\nB. 若 $aa b>b^{2}$\nC. 若 $a\\frac{a}{b}$", "subject": "算术", "analysis": "解析: 解答: 对于 $\\mathrm{A}$, 当 $c=0$ 时, 不等式不成立, 故 $\\mathrm{A}$ 错; 对于 $\\mathrm{C}$, 因为 $a0$, 所以 $\\frac{1}{a}>\\frac{1}{b}$, 故 C 错; 对于 D, 因为 $-a>-b>0,-\\frac{1}{b}>-\\frac{1}{a}>0$, 所以 $\\frac{a}{b}>\\frac{b}{a}$, 故 $\\mathrm{D}$ 错, 所以选 $\\mathrm{B}$.\n\n分析: 本题主要考查了不等关系与不等式, 解决问题的关键是根据不等式性质分析计算即可"} {"id": "22964", "image": [], "answer": "B", "solution": "null", "level": "高三", "question": "若 $aa b>b^{2}$\nC. $a^{2}b^{2}>a b$", "subject": "算术", "analysis": "解析: 解答: 因为 $aa b>b^{2}$, 故选 B.\n\n分析: 本题主要考查了不等关系与不等式, 解决问题的关键是根据不等关系与不等式分析即可"} {"id": "22969", "image": [], "answer": "C", "solution": "null", "level": "高三", "question": "若 $a>b>0, c\\frac{b}{d}$\nB. $\\frac{a}{d}>\\frac{b}{c}$\nC. $\\frac{a}{d}<\\frac{b}{c}$\nD. $\\frac{a}{c}<\\frac{b}{d}$", "subject": "算术", "analysis": "解析: 解答: 因为 $\\mathrm{c}<\\mathrm{d}<0$, 所以 $\\frac{1}{d}<\\frac{1}{c}<0$, 即 $-\\frac{1}{d}>-\\frac{1}{c}>0$,\n\n与 $\\mathrm{a}>\\mathrm{b}>0$ 对应相乘得, $-\\frac{a}{d}>-\\frac{b}{c}>0$, 所以 $\\frac{a}{d}<\\frac{b}{c}$. 故选 C.\n\n分析: 本题主要考查了不等式与不等关系, 解决问题的关键是根据不等式的性质分析即可"} {"id": "22975", "image": [], "answer": "C", "solution": "null", "level": "高三", "question": "若 $a, b, c \\in \\mathbf{R}, a>b$, 则下列不等式成立的是 ( )", "options": "A. $\\frac{1}{a}<\\frac{1}{b}$\nB. $a^{2}>b^{2}$\nC. $\\frac{a}{c^{2}+1}>\\frac{b}{c^{2}+1}$\nD. $a|c|>b|c|$", "subject": "算术", "analysis": "解析: 解答: 根所题意及选项特征宜用特殊值法进行求解.不妨设 $a=1, b=-1$, 则可排除 $\\mathrm{A} . \\mathrm{~B}$ 选项,当 $c=0$ 时 $\\mathrm{D}$ 选项显然不成立,故正确答案为选项 C.\n\n分析: 本题主要考查了不等式与不等关系, 解决问题的关键是根据题意及选项特征宜用特殊值法进行求解"} {"id": "22977", "image": [], "answer": "B", "solution": "null", "level": "高三", "question": "若 $a, b, c$ 为实数, 则下列命题正确的是 ( )", "options": "A. 若 $a>b$, 则 $a c^{2}>b c^{2}$\n\nB. 若 $aa b>b^{2}$\n\nC. 若 $a\\frac{a}{b}$", "subject": "算术", "analysis": "解析:解答:选项 A: 当 $c=0$ 时, $a c^{2}=b c^{2}=0$ (舍); 选项 B: $\\because aa b, a b>b^{2}$, 即 B 正确; 选项 C: $\\because y=\\frac{1}{x}$ 在 $(-\\infty, 0)$ 上为减函数, 且 $a\\frac{1}{b}$ (舍);选项 D: $\\because ab^{2}$, 所以 $\\frac{b}{a}-\\frac{a}{b}=\\frac{b^{2}-a^{2}}{a b}<0$, 即 $\\frac{b}{a}<\\frac{a}{b}$ (舍); 故选 B 分析: 本题主要考查了不等式与不等关系, 解决问题的关键是根据不等式的性质分析即可"} {"id": "22978", "image": [], "answer": "D", "solution": "null", "level": "高三", "question": "下列选项一定正确的是 ( )", "options": "A.若 $a>b$, 则 $a c>b c$\n\nB.若 $\\sqrt{a}>\\sqrt{b}$, 则 $a>b$\n\nC.若 $a^{2}>b^{2}$, 则 $a>b$\n\nD.若 $\\frac{1}{a}<\\frac{1}{b}$, 则 $a>b$", "subject": "算术", "analysis": "解析: 解答: 若 $c<0$, 选项 A 错误; 若 $\\sqrt{a}>\\sqrt{b}$, 两边平方, 则 $a>b$, 故 B 正确; 若 $a^{2}>b^{2}$,则 $|a|>|b|$, 故 C 错误; 若 $\\mathrm{a}<0, \\mathrm{~b}>0$, 则选项 D 错误; 故选 D.\n\n分析: 本题主要考查了不等式与不等关系, 解决问题的关键是根据不等式的性质分析判断即可"} {"id": "21759", "image": [], "answer": "B", "solution": "null", "level": "高三", "question": "函数 $f(x)=x^{2}-1$ 在区间 $[1, m]$ 上的平均变化率为 3 , 则实数 $m$ 的值为", "options": "A. 3\nB. 2\nC. 1\nD. 4", "subject": "算术", "analysis": ""} {"id": "21761", "image": [], "answer": "C", "solution": "null", "level": "高三", "question": "设函数 $f(x)$ 可导, 则 $\\lim _{\\Delta x \\rightarrow 0} \\frac{f(1+\\Delta x)-f(1)}{3 \\Delta x}$ 等于 $(\\quad)$", "options": "A. $f^{\\prime}(1)$\nB. $3 f^{\\prime}(1)$\nC. $\\frac{1}{3} f^{\\prime}(1)$\nD. $f^{\\prime}(3)$", "subject": "算术", "analysis": "]"} {"id": "21090", "image": [], "answer": "C", "solution": "null", "level": "高三", "question": "下列命题中, 正确的是 ( )", "options": "A. 若 $a>b, c>d$, 则 $a c>b d$\nB. 若 $a c>b c$, 则 $a>b$\nC. 若 $\\frac{a}{\\mathrm{c}^{2}}<\\frac{\\mathrm{b}}{\\mathrm{c}^{2}}$, 则 $\\mathrm{a}<\\mathrm{b}$\nD. 若 $a>b, c>d$, 则 $a-c>b-d$", "subject": "算术", "analysis": "A. 举出反例: 虽然 $5>2,-1>-2$, 但是 $5 \times(-1)<2 \times(-2)$, 故 A 不正确; B. 举出反例: 虽然 $2 \times(-1)>3 \times(-1)$, 但是 $\\mathrm{a}<\\mathrm{b}$, 故 B 不正确;C. $\because \frac{\\mathrm{a}}{\\mathrm{c}^{2}}<\frac{\\mathrm{b}}{\\mathrm{c}^{2}}, \therefore \frac{\\mathrm{a}}{\\mathrm{c}^{2}} \times \\mathrm{c}^{2}<\frac{\\mathrm{b}}{\\mathrm{c}^{2}} \times \\mathrm{c}^{2}, \therefore \\mathrm{a}<\\mathrm{b}$, 故 C 正确; D. 举出反例: 虽然 5 $>4,3>1$, 但是 $5-3<4-1$, 故 D 不正确. 综上可知: C 正确."} {"id": "23001", "image": [], "answer": "D", "solution": "null", "level": "高三", "question": ". 不等式 $|4-3 x|-5 \\leq 0$ 的解集是()", "options": "A. $\\left\\{x \\left\\lvert\\,-\\frac{1}{3}0$ ”, 你认为这个推理 ( )", "options": "A. 大前提错误\nB. 小前提错误\nC. 推理形式错误\nD. 是正确的", "subject": "算术", "analysis": "解析: 解答: 要分析一个演绎推理是否正确, 主要观察所给的大前提, 小前提和结论是否正确, 根据三个方面都正确, 才能得到结论. 在本题中, 因为任何实数的平方大于 0 , 因为 $a$是实数, 所以 $a^{2}>0$, 大前提为: 任何实数的平方大于 0 是不正确的, 0 的平方就不大于 0 . 故选 A.\n\n分析: 本题主要考查了演绎推理的基本方法, 解决问题的关键是根据演绎推理的基本方法结合三段论进行分析即可."} {"id": "20489", "image": [], "answer": "B", "solution": "null", "level": "高三", "question": "根据给出的数塔猜测 $123456 \\times 9+7=( \\quad)$\n\n$1 \\times 9+2=11$\n\n$12 \\times 9+3=111$\n\n$123 \\times 9+4=1111$\n\n$1234 \\times 9+5=11111$\n\n$12345 \\times 9+6=111111$\n\n......", "options": "A. 1111110\nB. 1111111\n\nC. 1111112\nD. 111113", "subject": "算术", "analysis": "解析:解答:由数塔等号右侧数字规律易得 $123456 \\times 9+7=1111111$.\n\n分析: 本题主要考查了归纳推理, 解决问题的关键是根据所给数列的规律进行归纳即可."} {"id": "22369", "image": [], "answer": "C", "solution": "null", "level": "高三", "question": "高二年级的三个班去甲, 乙, 丙, 丁四个工厂参加社会实践, 但去何工厂可自由选择, 甲工厂必须有班级要去, 则不同的分配方案有( )", "options": "A. 16 种\nB. 18 种\nC. 37 种\nD. 48 种", "subject": "组合数学", "analysis": "答案: C\n\n【解析】 方法一: 满足题意的不同的分配方案有以下三类:\n\n(1)三个班中只有一个班去甲工厂有 $\\mathrm{C}_{3}^{1} \\times 3^{2}=27$ 种方案; (2)三个班中只有两个班去甲工厂有 $\\mathrm{C}_{3}^{2} \\times 3=9$ 种方案; (3)三个班都去甲工厂, 有 1 种方案. 综上可知, 共有 $27+9+1=37$ 种不同方案, 故选 C.\n\n方法二:高二年级的三个班到甲、乙、丙、丁四个工厂进行社会实践,有 $4^{3}$ 种不同的分配方案, 若三个班都不去甲工厂, 则有 $3^{3}$ 种不同的分配方案, 则满足条件的不同的分配方案有 $4^{3}-3^{3}=37$ 种, 故选 C."} {"id": "22370", "image": [], "answer": "D", "solution": "null", "level": "高三", "question": "4 位同学各自在周六、周日两天中任选一天参加公益活动, 则周六、周日都有同学参加公益活动的概率为 (", "options": "A. $\\frac{1}{8}$\nB. $\\frac{3}{8}$\nC. $\\frac{5}{8}$\nD. $\\frac{7}{8}$", "subject": "组合数学", "analysis": "答案: D\n\n【解析】 方法一: 4 为同学各自在周六、日任选一天参加公益活动共有 $2^{4}=16$ (种)结果,而周六、日都有同学参加公益活动有两种情况:(1)一天一人,另一天三人, $\\mathrm{C}_{4}^{1} \\mathrm{C}_{2}^{2}=8$ (种);(2)每天二人, 有 $\\mathrm{C}_{4}^{2}=6$ (种), 所以 $P=\\frac{8+6}{16}=\\frac{7}{8}$.\n\n方法二(间接法): 4 位同学各自在周六、日任选一天参加公益活动,共有 $2^{4}=16$ (种)结果,而 4 人都选周六或周日有 2 种结果, 所以 $P=1-\\frac{2}{16}=\\frac{7}{8}$."} {"id": "22371", "image": [], "answer": "D", "solution": "null", "level": "高三", "question": "体育场南侧有 4 个大门, 北侧有 3 个大门,某学生到该体育场练跑步, 则他进出门的方案有", "options": "A. 12 种\nB. 7 种\nC. 24 种\nD. 49 种", "subject": "组合数学", "analysis": "答案: D\n\n【解析】 第一步, 他进门, 有 7 种选择; 第二步, 他出门, 有 7 种选择. 根据分步乘法计数原理可得他进出门的方案有 $7 \\times 7=49$ (种)."} {"id": "22372", "image": [], "answer": "C", "solution": "null", "level": "高三", "question": "王刚同学衣服上左、右各有一个口袋,左边口袋里装有 30 个英语单词卡片,右边口袋里装有 20 个英语单词卡片, 这些英语单词卡片都互不相同,则从两个口袋里任取一张英语单词卡片,不同取法的种数为()", "options": "A. 20\nB. 30\nC. 50\nD. 600", "subject": "组合数学", "analysis": "答案: C\n【解析】从口袋里任取一张英语单词卡片的方案分两类:\n\n第一类:从左边口袋里取一张英语单词卡片, 有 30 种不同的取法;\n\n第二类: 从右边口袋里取一张英语单词卡片, 有 ${ }^{20}$ 种不同的取法. 由分类加法计数原理知, 从两个口袋里任取一张英语单词卡片的取法种数为 $30+20=50$."} {"id": "22373", "image": [], "answer": "C", "solution": "null", "level": "高三", "question": "从 6 人中选出 4 人分别参加 2018 年北京大学举办的数学、物理、化学、生物暑期夏令营,每人只能参加其中一项, 其中甲、乙两人都不能参加化学比赛, 则不同的参赛方案的种数为", "options": "A. 94\nB. 180\nC. 240\nD. 286", "subject": "组合数学", "analysis": "答案: C\n\n【解析】 第一步, 因为甲、乙两人都不能参加化学比赛. 所以从剩下的 4 人中选 1 人参加化学比赛.共有 4 种选法;\n\n第二步,在剩下的 5 人中任选 3 人参加数学、物理、生物比赛,共有 $5 \\times 4 \\times 3=60$ 种选法.\n\n由分步乘法计数原理,得不同的参赛方案的种数为 $4 \\times 60=240$,故选 C."} {"id": "22374", "image": [], "answer": "D", "solution": "null", "level": "高三", "question": "甲与其四位同事各有一辆私家车, 车牌尾数分别是 $0 、 0 、 2 、 1 、 5$, 为遵守当地某月 5 日至 9 日 5 天的限行规定(奇数日车牌尾数为奇数的车通行, 偶数日车牌尾数为偶数的车通行), 五人商议拼车出行, 每天任选一辆符合规定的车, 但甲的车最多只能用一天, 则不同的用车方案种数为( )", "options": "A. 5\nB. 24\nC. 32\nD. 64", "subject": "组合数学", "analysis": "答案: D\n\n【解析】 5 日至 9 日,分别为 $5,6,7,8,9$, 有 3 天奇数日, 2 天偶数日,第一步安排奇数日出行,每天都有 2 种选择, 共有 $2^{3}=8$ 种, 第一步安排偶数日出行分两类, 第一类, 先选 1 天安排甲的车, 另外一天安排其他车, 有 $2 \\times 2=4$ 种, 第二类, 不安排甲的车, 每天都有 2 种选择, 共有 $2^{2}=4$ 种, 共计 4 ? 4 ? $=8$, 根据分布计数原理, 不同的用车方案种数共有 $8 \\times 8=64$. 故选 D."} {"id": "22377", "image": [], "answer": "B", "solution": "null", "level": "高三", "question": "六个人从左至右排成一行,最左端只能排甲或乙,最右端不能排甲, 则不同的排法共有 ( )", "options": "A. 192 种\nB. 216 种\nC. 240 种\nD. 288 种", "subject": "组合数学", "analysis": "答案: B\n\n【解析】 若最左端排甲, 其他位置共有 $A_{5}^{5}=120$ (种) 排法; 若最左端排乙, 最右端共有 4 种排法,其余 4 个位置有 $A_{4}^{4}=24$ (种)排法,所以共有 $120+4 \\times 24=216$ (种)排法。"} {"id": "22378", "image": ["9788.jpg"], "answer": "C", "solution": "null", "level": "高三", "question": "某单位安排 7 位员工在 10 月 1 日至 7 日值班,每天安排 1 人,每人值班 1 天。若 7 位员工中的甲、乙排在相邻两天,丙不排在 10 月 1 日,丁不排在 10 月 7 日,则不同的安排方案共有( )", "options": "A. 504 种\nB. 960 种\nC. 1008 种\nD. 1108 种", "subject": "组合数学", "analysis": "答案: C\n\n\n甲乙排中间,丙排 7 号或不排 7 号,共有 $4 A_{2}^{2}\\left(A_{+}^{+}+A_{3}^{2} A_{3}^{1} A_{3}^{3}\\right)$ 种方法,故共有 1008 种不同的排法.故选 C.\n\n3.答案: A\n\n【解析】 安排人员去甲地可分为两步: 第一步安排教师, 有 ${ }^{1}$ 种方案; 第二步安排学生, 有 ${ }^{2}$ 种方案.其余的教师和学生去乙地,所以不同的安排方案共有 $C_{2}^{1} C_{4}^{R}=12$ 种,选 A."} {"id": "22379", "image": [], "answer": "A", "solution": "null", "level": "高三", "question": "将 2 名教师, 4 名学生分成 2 个小组,分别安排到甲、乙两地参加社会实践活动, 每个小组由 1 名教师和 2 名学生组成,不同的安排方案共有( )", "options": "A. 12 种\nB. 10 种\nC. 9 种\nD. 8 种", "subject": "组合数学", "analysis": "安排人员去甲地可分为两步: 第一步安排教师, 有 ${ }^{1}$ 种方案; 第二步安排学生, 有 ${ }^{2}$ 种方案.其余的教师和学生去乙地,所以不同的安排方案共有 $C_{2}^{1} C_{4}^{R}=12$ 种,选 A."} {"id": "22380", "image": [], "answer": "D", "solution": "null", "level": "高三", "question": "从 $1,2,3, \\ldots, 9$ 这 9 个整数中同时取 4 个不同的数, 其和为偶数, 则不同的取法共有 ( )", "options": "A. 60 种\nB. 63 种\nC. 65 种\nD. 66 种", "subject": "组合数学", "analysis": "答案: D\n\n【解析】 共有 4 个不同的偶数和 5 个不同的基数,要使和为偶数, 则 4 个数全为奇数,或全为偶数,或 2 个奇数、 2 个偶数,故不同的取法有 $C_{5}^{4}+C_{4}^{4}+C_{5}^{2} C_{4}^{2}=66$ (种)。"} {"id": "22381", "image": [], "answer": "C", "solution": "null", "level": "高三", "question": "从 1,3,5,7,9 这五个数中, 每次取出两个不同的数作为 $a, b$, 共可得到 $\\lg a-1 \\mathrm{l} b$ 的不同值的个数是( )", "options": "A. 9\nB. 10\nC. 18\nD. 20", "subject": "组合数学", "analysis": "答案: C\n\n【解析】 从 1,3,5,7,9 中,每次取出两个不同的数作为 $a, b$, 可以得到不同的差式 $\\lg a-\\lg b$ 共计 $A_{5}^{2}=20$ 个, 但其中 $\\lg 9-\\lg 3=\\lg 3-\\lg 1, \\lg 3-\\lg 9=\\lg 1-\\lg 3$, 故不同的值只有 18 个."} {"id": "22382", "image": ["9789.jpg"], "answer": "C", "solution": "null", "level": "高三", "question": "如图是某汽车维修公司的维修点环形分布图.公司在年初分配给 $A, B, C, D$ 四个维修点某种配件各 50 件.在使用前发现需将 $A, B, C, D$ 四个维修点的这批配件分别调整为 $40,45,54,61$ 件,但调整只能在相邻维修点之间进行,那么要完成上述调整,最少的调动件次 $(n$ 件配件从一个维修点调整到相邻维修点的调动件次为 $n$ )为 ( )\n\n", "options": "A. 18\nB. 17\nC. 16\nD. 15", "subject": "组合数学", "analysis": "答案: C\n【解析】只需 $A$ 处给 $D$ 处 10 件, $B$ 处给 $C$ 处 5 件, $C$ 处给 $D$ 处 1 件, 共 16 件次,故选 C."} {"id": "22386", "image": [], "answer": "C", "solution": "null", "level": "高三", "question": "某班级需要把 6 名同学安排到周一、周二、周三这三天值日, 每天安排 2 名同学,已知甲不能安排到周一,乙和丙不能安排到同一天,则安排方案的种数为( )", "options": "A. 24\nB. 36\nC. 48\nD. 72", "subject": "组合数学", "analysis": "答案: C\n\n【解析】 根据题意, 分 2 种情况讨论:\n\n(1)甲、乙、丙三人分在不同的三天值班,\n\n甲可以分在周二、周三, 有 2 种安排方法, 将乙、丙全排列,分在其他 2 天, 有 $A_{2}^{2}=3$ 种安排方法,\n\n剩余的 3 人,全排列,安排在周一、周二、周三这三天,有 $A_{3}^{3}=6$ 种安排方法,\n\n则此时有 $2 \\times 2 \\times 6=24$ 种安排方法;\n\n(2), 甲和乙、丙中的 1 人, 安排在同一天值班,\n\n在乙、丙中选出 1 人,和甲一起分在周二、周三值班,有 $2 \\times 2=4$ 种情况,\n\n剩余 4 人,平均分成 2 组,有 $\\frac{1}{2} C_{4}^{2}=3$ 种分组方法,\n\n再将 2 组全排列,对应剩下的 2 天值班, 有 $A_{2}^{2}=2$ 种安排方法,\n\n则此时有 $4 \\times 3 \\times 2=24$ 种安排方法;\n\n则有 $24+24=48$ 种不同的安排方案,\n\n故选: C."} {"id": "22387", "image": [], "answer": "A", "solution": "null", "level": "高三", "question": "将 2 名教师, 4 名学生分成 2 个小组,分别安排到甲、乙两地参加社会实践活动, 每个小组由 1 名教师和 2 名学生组成,不同的安排方案共有( )", "options": "A. 12 种\nB. 10 种\nC. 9 种\nD. 8 种", "subject": "组合数学", "analysis": "安排人员去甲地可分为两步: 第一步安排教师, 有 ${ }^{1}$ 种方案; 第二步安排学生, 有 ${ }^{2}$ 种方案.其余的教师和学生去乙地,所以不同的安排方案共有 $C_{2}^{1} C_{4}^{R}=12$ 种,选 A."} {"id": "22388", "image": [], "answer": "D", "solution": "null", "level": "高三", "question": "从 $1,2,3, \\ldots, 9$ 这 9 个整数中同时取 4 个不同的数, 其和为偶数, 则不同的取法共有 ( )", "options": "A. 60 种\nB. 63 种\nC. 65 种\nD. 66 种", "subject": "组合数学", "analysis": "答案: D\n\n【解析】 共有 4 个不同的偶数和 5 个不同的基数,要使和为偶数, 则 4 个数全为奇数,或全为偶数,或 2 个奇数、 2 个偶数,故不同的取法有 $C_{5}^{4}+C_{4}^{4}+C_{5}^{2} C_{4}^{2}=66$ (种)。"} {"id": "22389", "image": [], "answer": "C", "solution": "null", "level": "高三", "question": "某班班会准备从甲、乙等 7 名学生中选派 4 名学生发言, 要求甲、乙两名同学至少有一人参加,且若甲、乙同时参加,则他们发言时不能相邻,那么不同的发言顺序的种数为( )", "options": "A. 360\nB. 520\nC. 600\nD. 720", "subject": "组合数学", "analysis": "答案: C\n\n【解析】 分两类:第一类,甲、乙中只有一人参加,则有 $\\mathrm{C}_{2}^{1} \\mathrm{C}_{5}^{3} \\mathrm{~A}_{4}^{4}=2 \\times 10 \\times 24=480$ 种选法.\n第二类,甲、乙都参加时,则有 $C_{5}^{2}\\left(\\hat{A}_{4}^{4}-\\hat{A}_{2}^{2} \\hat{A}_{3}^{3}\\right)=10 \\times(24-12)=120$ 种选法.\n\n$\\therefore$ 共有 $480+120=600$ 种选法."} {"id": "22390", "image": [], "answer": "B", "solution": "null", "level": "高三", "question": "将 3 本相同的语文书和 2 本相同的数学书分给四名同学, 每人至少 1 本, 不同的分配方法数为( )", "options": "A. 24\nB. 28\nC. 32\nD. 36", "subject": "组合数学", "analysis": "答案: B\n\n【解析】 第一类, 先选 1 人得到两本语文书, 剩下的 3 人各得本, 有 $\\mathrm{C}_{4}^{1} \\mathrm{C}_{3}^{1}=12$ 种; 第二类,\n\n先选 1 人得到一本语文书和一本数学书, 剩下的 3 人各得一本, 有 ${ }^{C_{4}^{1} C_{3}^{1}}=12$ 种; 第三类,先选 1 人得到两本数学书, 剩下的 3 人各得一本, 有 ${ }^{1}=4$ 种, 根据分类加法计数原理可得共有 $12+12+4=28$ 种方法, 故选 B."} {"id": "22391", "image": [], "answer": "C", "solution": "null", "level": "高三", "question": "在航天员进行的一项太空实验中, 要先后实施 6 个程序, 其中程序 $A$ 只能出现在第一步或最后一步, 程序 $B$ 和 $C$ 实施时必须相邻, 请问实验顺序的编排方法共有 ( )", "options": "A. 24 种\nB. 48 种\nC. 96 种\nD. 144 种", "subject": "组合数学", "analysis": "答案: C\n\n【解析】 由题意知程序 $A$ 只能出现在第一步或最后一步, $\\therefore$ 从第一个位置和最后一个位置选一个位置把 $A$ 排列, 有 $A_{2}^{1}=2$ 种方法, $\\mathrm{Q}$ 程序 $B$ 和 $C$ 实施时必须相邻, $\\therefore$ 把 $B$ 和 $C$ 看作一个元素, 同除 $A$ 外的 3 个元素排列, 注意 $B$ 和 $C$ 之间还有一个排列, 共有 $A_{4}^{4} A_{2}^{2}=48$ 种方法,根据分步乘法计数原理知共有 $2 \\times 48=96$ 种编排方法, 故选 C."} {"id": "22397", "image": [], "answer": "C", "solution": "null", "level": "高三", "question": "$\\left(x-\\frac{1}{x}\\right)^{16}$ 的二项展开式中, 第 4 项是 ( )", "options": "A. 1\nB. -1\nC. 0\nD. 2", "subject": "组合数学", "analysis": "答案: C\n\n【解析】 展开式的通项为 $T_{\\mathrm{r+1}}=\\mathrm{C}_{1 \\mathrm{r}}^{\\mathrm{r}} \\cdot x^{1 \\mathrm{kr}} \\cdot\\left(-\\frac{1}{x}\\right)^{r}=(-1)^{r} \\cdot \\mathrm{C}_{1 \\mathrm{k}}^{\\mathrm{r}} \\cdot x^{16-1 \\mathrm{x}}$, 所以第 4 项为 $T_{4}=(-1)^{3} \\times C_{16}^{3} x^{10}=-C_{16}^{3} x^{10}$. 故选 C."} {"id": "22398", "image": [], "answer": "C", "solution": "null", "level": "高三", "question": "若二项式 $\\left(2 x-\\frac{1}{\\sqrt{x}}\\right)^{n}\\left(n \\in \\mathrm{N}^{*}\\right)$ 的展开式中第 2 项与第 3 项的二项式系数比是 $2: 5$, 则 $x^{3}$ 的系数为( )", "options": "A. ${ }^{2}{ }_{16}^{12}$\nB. $\\mathrm{C}^{3}{ }^{3} x^{10}$\nC. $\\mathrm{C}^{3} x^{10}$\nD. ${ }^{4} x^{4} x^{8}$", "subject": "组合数学", "analysis": "答案: C\n\n【解析】 二项展开式的第 $r+1$ 项的通项公式为 $T_{r+1}=C_{n}^{n}(2 x)^{n-r}\\left(-\\frac{1}{\\sqrt{x}}\\right)^{r}$\n由展开式中第 2 项与第 3 项的二项式系数之比是 $2: 5$, 可得: $C_{n}^{1}: C_{n}^{2}=2: 5$.\n\n解得: $n=6$.\n\n所以 $T_{r+1}=C_{n}^{r}(2 x)^{n-r}\\left(-\\frac{1}{\\sqrt{x}}\\right)^{r}=C_{r}^{r} 2^{r-r}(-1)^{r} x^{r-\\frac{3}{2}}$\n\n令 $6-\\frac{3}{2} r=3$, 解得: $r=2$,\n\n所以 $x^{3}$ 的系数为 $C_{6}^{2} 2^{-12}(-1)^{2}=240$\n\n故选 C."} {"id": "22401", "image": ["9791.jpg"], "answer": "A", "solution": "null", "level": "高三", "question": "我国南宋数学家杨辉 1261 年所著的《详解九章算法》一书里出现了如图所示的表, 即杨辉三角, 这是数学史上的一个伟大成就.在“杨辉三角” 中, 第 $n$ 行的所有数字之和为 $2^{2^{n-1}}$, 若去除所有为 1 的项, 依次构成数列 $2,3,3,4,6,5,10,10,5, \\cdots$, 则此数列的前 55 项和为 ( )\n\n", "options": "A. 4072\nB. ${ }^{2026}$\nC. ${ }^{4096}$\nD. ${ }^{2048}$", "subject": "组合数学", "analysis": "答案:A\n\n【解析】记数列 $2,3,3,4,6,5,10,10,5, \\cdots$ 为 $\\left\\{a_{n}\\right\\}, n \\in N^{*}$, 则在杨辉三角中, 第3行出现 $\\left\\{a_{n}\\right\\}$ 中的1 项, 第4行出现 $\\left\\{a_{n}\\right\\}$ 中的2项, 第5行出现 $\\left\\{a_{n}\\right\\}$ 中的3项, $\\cdots$,第n行出现 $\\left\\{a_{n}\\right\\}$ 中的 ${ }^{(n-2)}$ 项, 设第55项出现在第 $n+1$ 行 $(n>2)$, 则 $1+2+3+\\cdots+(n-1) .55$, 解得 $n \\ldots 11, \\therefore$ 第55项出现在第12 行从右向左数第 2 个. 又在“杨辉三角” 中, 第 $n$ 行的数字之和为 $2^{n-1}$, 从第二行起每一行再减去 2 个 $1, \\therefore$ 前 55 项的和为 $2^{0}+2^{1}+2^{2}+\\cdots+2^{11}-1-2 \\times 11=4072$, 故选A."} {"id": "22402", "image": ["9792.jpg"], "answer": "C", "solution": "null", "level": "高三", "question": "我国南宋数学家杨辉所著的《详解九章算法》一书中记录了一个由数字构成的三角形数表,我们通常称之为杨辉三角. 以下数表的构造思路就来源于杨辉三角.\n\n\n\n从第二行起, 每一行中的数字均等于其“肩上”两数之和, 表中最后一行仅有一个数 $\\mathrm{a}$, 则 $\\mathrm{a}$的值为( )", "options": "A. $2018 \\times 2^{1008}$\nB. $2018 \\times 2^{1009}$\nC. $2020 \\times 2^{1008}$\nD. $2020 \\times 2^{1009}$", "subject": "组合数学", "analysis": "答案:C\n\n【解析】观察数表中每一行的数, 第 1 行共 1010 个数, 第 1 个数是 $1 \\times 2^{0}$ : 第 2 行共 1009 个数, 第 1 个数是 $2 \\times 2^{1}$; 第 3 行共 1008 个数, 第 1 个数是 $3 \\times 2^{2}$; 第 4 行共 1007 个数, 第 1 个数是 $4 \\times 2^{3} ; \\ldots$ 第 1010 行共 1 个数, 这个数是 $1010 \\times 2^{1009}=2020 \\times 2^{1008}$ 。"} {"id": "22403", "image": [], "answer": "B", "solution": "null", "level": "高三", "question": "以下数表的构造思路源于我国南宋数学家杨辉所著的《详解九章算法》一书中的“杨辉三角”.\n\n$\\begin{array}{llllllllllll}1 & 2 & 3 & 4 & 5 & \\cdots \\cdots & 2013 & 2014 & 2015 & 2016\\end{array}$\n\n$\\begin{array}{llllllllll}3 & 5 & 7 & 9 & \\cdots \\cdots & 4027 & 4029 & 4031\\end{array}$\n\n$\\begin{array}{llllll}8 & 12 & 16 & \\cdots \\cdots & 8056 \\quad 8060\\end{array}$\n\n$\\begin{array}{llll}20 & 28 & \\cdots \\cdots & 16116\\end{array}$\n\n该表由若干行数字组成, 从第二行起, 每一行中的数字均等于其“肩上”两数之和, 表中最后一行仅有一个数, 则这个数为 ( )", "options": "A. $2017 \\times 2^{2015}$\nB. $2017 \\times 2^{2014}$\nC. $2016 \\times 2^{2015}$\nD. $2016 \\times 2^{2014}$", "subject": "组合数学", "analysis": "答案:B\n\n【解析】由题意得, 数表的第一行有 2016 个数, 第二行有 2015 个数, ..., 所以最后一行有 1 个数, 最后一行为第 2016 行. 数表的每一行都是等差数列, 且第一行公差为 1 , 第二行公差为\n\n2 , 第三行公差为 $4, \\cdots$, 第 2015 行公差为 $2^{2014}$. 第一行的第一个数为 $2 \\times 2^{-1}$, 第二行的第一个数为 $3 \\times 2^{0}$, 第三行的第一个数为 $4 \\times 2^{1}, \\ldots$, 第 $\\mathrm{n}$ 行的第一个数为 $(n+1) \\times 2^{n-2}$, 第 2016 行只有数 $M=(1+2016) \\times 2^{2014}=2017 \\times 2^{2014}$, 故选 B."} {"id": "21089", "image": [], "answer": "D", "solution": "null", "level": "高三", "question": "设 $\\mathrm{m}, \\mathrm{n}$ 是两条不同的直线, $\\alpha, \\beta$ 是两个不同的平面, 则下列命题正确的是 ( )", "options": "A. 若 $\\mathrm{m}\\|\\alpha, \\mathrm{n}\\| \\alpha$, 则 $\\mathrm{m} \\| \\mathrm{n}$\nB. 若 $\\alpha \\| \\beta, \\mathrm{m} \\subset \\alpha, \\mathrm{n} \\subset \\beta$,则 $\\mathrm{m} \\| \\mathrm{n}$\nC. 若 $\\alpha \\cap \\beta=\\mathrm{m}, \\mathrm{n} \\subset \\alpha, n \\perp \\mathrm{m}$, 则 $\\mathrm{n} \\perp \\beta$\nD. 若 $m \\perp \\alpha, m \\| n, n \\subset \\beta$ 则 $\\alpha \\perp \\beta$", "subject": "立体几何学", "analysis": "答案:D\n\n解析: A. 错误, 同时和一个平面平行的两直线不一定平行, 可能相交, 可能异面;\n\nB. 错误, 两平面平行, 两平面内的直线不一定平行, 可能异面;\n\nC. 错误, 一个平面内垂直于两平面交线的直线, 不一定和另一平面垂直,可能斜交;\n\nD. 正确, 由 $\\mathrm{m} \\perp \\alpha, \\mathrm{m} \\| \\mathrm{n}$ 便得 $n \\perp \\alpha$, 又 $\\mathrm{n} \\subset \\beta , \\therefore \\beta \\perp \\alpha$ ,即 $\\alpha \\perp \\beta$.\n\n2.答案;C\n\n解析: 由题意, 原命题为: 若 $x=3$, 则 $x^{2}-9 x+18=0$, 显然 3 是方程的解, 为真命题;\n\n逆命题为: 若 $x^{2}-9 x+18=0$, 则 $x=3$, 因为方程还有另一根为 6 , 故为假命题;因为原命题与逆否命题等价, 故逆否命题为真; 逆命题与否命题等价, 故否命题为假. 故选 C.\n\n3.答案:D 解析: 命题 “若 $x^{2} \\leq 1$, 则 $-1 \\leq x \\leq 1$ ” 的逆否命题是 “若 $x<-1$ 或 $x>1$, 则 $x^{2}$ $>1$ ”. 故选: D.\n\n4.答案:B\n\n解析:对于(1), 逆命题为真, 故否命题为真;\n\n对于 (2) “若 $a=1$, 则 $a x^{2}-x+3 \\geq 0$ 的解集为 $R$ ” 原命题为真, 故逆否命题为真;\n\n对于 (3) “面积相等的圆周长相同” 为真;\n\n对于 (4) “若 $\\sqrt{2} x$ 为有理数, 则 $x$ 为 0 或无理数” , 故原命题为假, 逆否命题为假.\n\n5.答案:C\n\n解析:对于原命题 “若 $x=3$, 则 $x^{2}=9$ ” 当 $x=3$ 时, 显然必有 $x^{2}=9$, 所以原命题成立是真命题.\n\n又因为逆命题为 “若 $x^{2}=9$, 则 $x=3$. ” 可知 $x^{2}=9$ 即 $x=3$ 或 $x=-3$, 从而推不出 $\\mathrm{x}$ 一定等于 3 , 故逆命题错误是假命题;\n\n6.答案:D 解析: 能够判断真假的陈述句叫命题, 则只有 $\\sin 30^{\\circ}=\\frac{1}{2}$, 能够判断真假,故只有 $\\mathrm{D}$ 是命题,\n\n7.答案:B\n\n【解答】\n\n解析: (1)若 $a \\perp b, b \\perp c$, 则 $a \\perp c$, 垂直于同一直线的两条直线相交、平行、异面皆有可能, 故命题不正确;\n\n(2)若 $a 、 b$ 是异面直线, $b 、 c$ 是异面直线,则 $a 、 c$ 也是异面直线,与同一直线异面的两直线可能是平行的, 即异面关系不具有传递性, 故命题不正确;\n\n(3)若 $\\mathrm{a}$ 和 $\\mathrm{b}$ 相交, $\\mathrm{b}$ 和 $\\mathrm{c}$ 相交, 则 $\\mathrm{a}$ 和 $\\mathrm{c}$ 也相交, 相交关系不具有传递性,故命题不正确;\n\n(4)若 $\\mathrm{a}$ 和 $\\mathrm{b}$ 共面, $\\mathrm{b}$ 和 $\\mathrm{c}$ 共面, 则 $\\mathrm{a}$ 和 $\\mathrm{c}$ 也共面, 线线间共面关系不具有传递性, $a \\| b, b$ 与 $c$ 相交, 则 $a, c$ 可以是异面关系, 故命题不正确;\n\n(5)若 $a\\|b , b\\| c$ ,则 $a \\| c ,$ 此是空间两直线平行公理,是正确命题;\n综上,仅有(5)正确.故选 B.\n\n8.答案:D\n\n解析: 实数 $\\mathrm{a}, \\mathrm{b}, \\mathrm{c}$ 不全为 0 等价于为 $\\mathrm{a}, \\mathrm{b}, \\mathrm{c}$ 中至少有一个不为 0 , 故选: D \n9.答案:A\n\n解析: A, 显然成立, 故是真命题; B, $x=-1$ 或 $x=1$, 故是假命题; C, 当 $x<0, y<0$时, 不成立, 故是假命题; $\\mathrm{D}$, 如果 $\\mathrm{x}=-3, \\mathrm{y}=1$ 时, 不成立, 故是假命题.故选 A.\n\n10.答案:C\n\n解析: A. 举出反例: 虽然 $5>2,-1>-2$, 但是 $5 \\times(-1)<2 \\times(-2)$, 故 A 不正确; B. 举出反例: 虽然 $2 \\times(-1)>3 \\times(-1)$, 但是 $\\mathrm{a}<\\mathrm{b}$, 故 B 不正确;\n\nC. $\\because \\frac{\\mathrm{a}}{\\mathrm{c}^{2}}<\\frac{\\mathrm{b}}{\\mathrm{c}^{2}}, \\therefore \\frac{\\mathrm{a}}{\\mathrm{c}^{2}} \\times \\mathrm{c}^{2}<\\frac{\\mathrm{b}}{\\mathrm{c}^{2}} \\times \\mathrm{c}^{2}, \\therefore \\mathrm{a}<\\mathrm{b}$, 故 C 正确; D. 举出反例: 虽然 5 $>4,3>1$, 但是 $5-3<4-1$, 故 D 不正确. 综上可知: C 正确.\n\n11.答案:(1)(4)\n\n解析: (1)由面面垂直的判定定理可知, 如果 $\\mathrm{m} \\perp \\alpha, \\mathrm{m} \\subset \\beta$, 那么 $\\alpha \\perp \\beta$; 故(1)正确.\n\n(2)如果 $\\mathrm{m} \\perp \\mathrm{n}, \\mathrm{m} \\perp \\alpha$ ,那么 $\\mathrm{n} / / \\alpha$ 或 $\\mathrm{n} \\subset \\alpha$; 故(2)错误;\n\n(3)如果 $\\alpha \\perp \\beta, \\mathrm{m} / / \\alpha$ ,那么 $\\mathrm{m}, \\beta$ 位置关系不确定; 故(3)错误;\n\n(4)如果 $\\alpha / / \\beta, \\alpha \\cap \\gamma=m, \\beta \\cap \\gamma=n$, 那么 $m / / n$. 正确.\n\n故答案为(1)(4).\n\n12.答案:1\n\n解析: 因为在命题的四种形式中原命题和逆否命题互为逆否命题, 同真同假,否命题和逆命题互为逆否命题同真同假.\n\n$\\because$ 原命题 “若 $a>-3$, 则 $a>-6$ ” 为真命题; 逆命题 “若 $a>-6$, 则 $a>-$ 3 ” 是假命题, $\\therefore$ 命题的逆否命题为真命题, 否命题为假命题. 故答案为 1 .\n\n13.答案: 解析: 原命题: 若 $m>0$, 则方程 $x^{2}+x-m=0$ 有实数根, 它是真命题;\n\n逆命题: 若方程 $x^{2}+x-m=0$ 有实数根, 则 $m>0$, 它是假命题;\n\n否命题: 若 $\\mathrm{m} \\leq 0$, 则方程 $\\mathrm{x}^{2}+\\mathrm{x}-\\mathrm{m}=0$ 没有实数根, 它是假命题;\n\n逆否命题: 若方程 $x^{2}+x-m=0$ 没有实数根, 则 $m \\leq 0$, 它是真命题."} {"id": "21099", "image": [], "answer": "B", "solution": "null", "level": "高三", "question": "设是空间的三条直线, 给出以下五个命题:\n\n(1)若 $\\mathrm{A} \\perp \\mathrm{b}, \\mathrm{b} \\perp \\mathrm{c}$, 则 $\\mathrm{a} \\perp \\mathrm{c}$;\n\n(2)若 $a 、 b$ 是异面直线, $b 、 c$ 是异面直线, 则 $a 、 c$ 也是异面直线;\n\n(3)若 $\\mathrm{a}$ 和 b 相交, b 和 $\\mathrm{c}$ 相交, 则 $\\mathrm{a}$ 和 $\\mathrm{c}$ 也相交;\n\n(4)若 $\\mathrm{a}$ 和 b 共面, b 和 $c$ 共面, 则 $\\mathrm{a}$ 和 $\\mathrm{c}$ 也共面;\n\n(5)若 $\\mathrm{a} / / \\mathrm{b}, \\mathrm{b} / / \\mathrm{c}$, 则 $\\mathrm{a} / / \\mathrm{c}$;\n\n其中正确的命题的个数是 ( )", "options": "A. 0\nB. 1\nC. 2\nD. 3", "subject": "立体几何学", "analysis": "(1)若 $a \\perp b, b \\perp c$, 则 $a \\perp c$, 垂直于同一直线的两条直线相交、平行、异面皆有可能, 故命题不正确;(2)若 $a 、 b$ 是异面直线, $b 、 c$ 是异面直线,则 $a 、 c$ 也是异面直线,与同一直线异面的两直线可能是平行的, 即异面关系不具有传递性, 故命题不正确;(3)若 $\\mathrm{a}$ 和 $\\mathrm{b}$ 相交, $\\mathrm{b}$ 和 $\\mathrm{c}$ 相交, 则 $\\mathrm{a}$ 和 $\\mathrm{c}$ 也相交, 相交关系不具有传递性,故命题不正确;(4)若 $\\mathrm{a}$ 和 $\\mathrm{b}$ 共面, $\\mathrm{b}$ 和 $\\mathrm{c}$ 共面, 则 $\\mathrm{a}$ 和 $\\mathrm{c}$ 也共面, 线线间共面关系不具有传递性, $a \\| b, b$ 与 $c$ 相交, 则 $a, c$ 可以是异面关系, 故命题不正确;(5)若 $a\\|b , b\\| c$ ,则 $a \\| c ,$ 此是空间两直线平行公理,是正确命题;综上,仅有(5)正确.故选 B."} {"id": "22847", "image": [], "answer": "B", "solution": "null", "level": "高三", "question": "设点 $M$ 的柱坐标为 $\\left(\\sqrt{2}, \\frac{5 \\pi}{4}, \\sqrt{2}\\right)$, 则它的球坐标为 ( )\n\n$$\n\\text {", "options": "A. }\\left(2, \\frac{\\pi}{4}, \\frac{\\pi}{4}\\right) \\text { B. }\\left(2, \\frac{\\pi}{4}, \\frac{5 \\pi}{4}\\right) \\text { C. }\\left(2, \\frac{5 \\pi}{4}, \\frac{\\pi}{4}\\right) \\text { D. }\\left(2, \\frac{3 \\pi}{4}, \\frac{\\pi}{4}\\right)\n$$", "subject": "立体几何学", "analysis": "解析: 解答: 设点 $M$ 的直角坐标为 $(x, y, z)$, 则 $\\left\\{\\begin{array}{c}x=\\sqrt{2} \\cos \\frac{5 \\pi}{4}, \\\\ y=\\sqrt{2} \\sin \\frac{5 \\pi}{4} \\\\ z=\\sqrt{2},\\end{array}\\right.$ 故 $\\left\\{\\begin{array}{c}x=-1, \\\\ y=-1, \\\\ z=\\sqrt{2}\\end{array}\\right.$.点 $M$ 的球坐标为 $(\\rho, \\varphi, \\theta)$,\n\n则 $\\rho=\\sqrt{(-1)^{2}+(-1)^{2}+(\\sqrt{2})^{2}}=2$, 由 $\\sqrt{2}=2 \\cos \\varphi$ 知 $\\varphi=\\frac{\\pi}{4}$,\n\n又 $\\tan \\theta=\\frac{-1}{-1}=1$, 故 $\\theta=\\frac{5 \\pi}{4}$,\n\n故点 $M$ 的球坐标为 $\\left(2, \\frac{\\pi}{4}, \\frac{5 \\pi}{4}\\right)$.\n\n分析: 本题主要考查了柱坐标系与球坐标系, 解决问题的关键是根据柱坐标系与球坐标系的关系转化计算即可"} {"id": "22859", "image": [], "answer": "D", "solution": "null", "level": "高三", "question": "若点 $P$ 的柱坐标为 $\\left(2, \\frac{\\pi}{6}, \\sqrt{3}\\right)$, 则 $P$ 到直线 $O y$ 的距离为 $(\\quad)$", "options": "A. 1\nВ . 2\nC. $\\sqrt{3}$\nD. $\\sqrt{6}$", "subject": "立体几何学", "analysis": "解析: 解答: 由于点 $P$ 的柱坐标为 $(\\rho, \\theta, z)=\\left(2, \\frac{\\pi}{6}, \\sqrt{3}\\right)$, 故点 $P$ 在平面 $x O y$ 内的射影 $Q$ 到直线 $O y$ 的距离为 $\\rho \\cos \\frac{\\pi}{6}=\\sqrt{3}$, 结合图形, 得 $P$ 到直线 $O y$ 的距离为 $\\sqrt{6}$.\n\n分析: 本题主要考查了柱坐标刻画点的位置, 解决问题的关键是根据柱坐标刻画点的位置转化为极坐标方程分析即可"} {"id": "22860", "image": [], "answer": "A", "solution": "null", "level": "高三", "question": "已知点 $P_{1}$ 的球坐标为 $\\left(4, \\frac{\\pi}{2}, \\frac{5 \\pi}{3}\\right), P_{2}$ 的柱坐标为 $\\left(2, \\frac{\\pi}{6}, 1\\right)$, 则 $\\left|P_{1} P_{2}\\right|=(\\quad)$", "options": "A. $\\sqrt{21}$\nB. $\\sqrt{29}$\nC. $\\sqrt{30}$\nD. $4 \\sqrt{2}$", "subject": "立体几何学", "analysis": "解析: 解答: 设点 $P_{1}$ 的直角坐标为 $\\left(x_{1}, y_{1}, z_{1}\\right)$,\n\n则 $\\left\\{\\begin{array}{c}x_{1}=4 \\sin \\frac{\\pi}{2} \\cos \\frac{5 \\pi}{3}, \\\\ y_{1}=4 \\sin \\frac{\\pi}{2} \\sin \\frac{5 \\pi}{3}, \\\\ z_{1}=4 \\cos \\frac{\\pi}{2}\\end{array}\\right.$ 得 $\\left\\{\\begin{array}{c}x_{1}=2, \\\\ y_{1}=-2 \\sqrt{3}, \\\\ z_{1}=0 .\\end{array}\\right.$\n\n故 $P_{1}(2,-2 \\sqrt{3}, 0)$,\n\n设点 $P_{2}$ 的直角坐标为 $\\left(x_{2}, y_{2}, z_{2}\\right)$,\n\n故 $\\left\\{\\begin{array}{c}x_{2}=2 \\cos \\frac{\\pi}{6}, \\\\ y_{2}=2 \\sin \\frac{\\pi}{6}, \\\\ z_{2}=1\\end{array}\\right.$ 得 $\\left\\{\\begin{array}{c}x_{2}=\\sqrt{3}, \\\\ y_{2}=1, \\\\ z_{2}=1 .\\end{array}\\right.$\n\n故 $P_{2}(\\sqrt{3}, 1,1)$.\n\n则 $\\left|P_{1} P_{2}\\right|=\\sqrt{(2-\\sqrt{3})^{2}+(-2 \\sqrt{3}-1)^{2}+(0-1)^{2}}=\\sqrt{21}$.\n\n分析: 本题主要考查了柱.球坐标系与空间直角坐标系的区别, 解决问题的关键是根据柱.\n球坐标系与空间直角坐标系的区别分析计算即可"} {"id": "21137", "image": ["9526.jpg"], "answer": "C", "solution": "null", "level": "高三", "question": "如图, 一个底面半径为 2 的圆柱被与其底面所成角是 $30^{\\circ}$ 的平面所截,截面是一个粗圆,则该粗圆的离心率等于()\n\n", "options": "A. $\\frac{\\sqrt{3}}{2}$\nB. $\\frac{\\sqrt{2}}{2}$\nC. $\\frac{1}{2}$\nD. $\\frac{1}{4}$", "subject": "立体几何学", "analysis": "$\because$ 底面半径为 2 的圆柱被与底面成 $30^{\\circ}$ 的平面所截, 其截口是一个椭圆, $\therefore$ 这个粗圆的短半轴长为 2 , 长半轴长为 $\frac{2}{\\cos 30^{\\circ}}=\frac{4 \\sqrt{3}}{3}, \because a^{2}=b^{2}+c^{2}, \\quad \therefore c=\\sqrt{\\left(\frac{4}{3} \\sqrt{3}\right)^{2}-2^{2}}=\frac{2 \\sqrt{3}}{3}$, $\therefore$ 椭圆的离心率为: $e=\frac{c}{a}=\frac{1}{2}$. 故选 $C$."} {"id": "20473", "image": ["9364.jpg"], "answer": "C", "solution": "null", "level": "高三", "question": "六个面都是平行四边形的四棱柱称为平行六面体。如, 在平行四边形 $A B C D$ 中, 有 $A C^{2}+B D^{2}=2\\left(A B^{2}+A D^{2}\\right)$, 那么在图 (2) 的平行六面体 $A B C D-A_{1} B_{1} C_{1} D_{1}$ 中有 $A C_{1}^{2}+B D_{1}^{2}+C A_{1}{ }^{2}+D B_{1}{ }^{2}$ 等于 $(\\quad)$\n", "options": "A. $2\\left(A B^{2}+A D^{2}+A A_{1}^{2}\\right)$\nB. $3\\left(A B^{2}+A D^{2}+A A_{1}^{2}\\right)$\nC. $4\\left(A B^{2}+A D^{2}+A A_{1}^{2}\\right)$\nD. $3\\left(A B^{2}+A D^{2}\\right)$", "subject": "立体几何学", "analysis": "解析: 解答: 平行四边形 $A B C D$ 中, 有 $A C^{2}+B D^{2}=2\\left(A B^{2}+A D^{2}\\right)$, 两对角线的平方和等于两邻边平方和的两倍, 其中的 2 倍代表的是两条对角线, 类比平面图形, 在空间图形中, 对角线的平方和等于三条邻边的平方和的 4 倍。 4 倍代表是的 4 条对角线。分析: 本题主要考查了类比推理, 解决问题的关键是根据所给平面图形的性质通过类比推理得到空间几何体的有关性质."} {"id": "20492", "image": [], "answer": "B", "solution": "null", "level": "高三", "question": "由“在平面内三角形的内切圆的圆心到三边的距离相等”联想到“在空间中内切于三棱锥的球的球心到三棱雉四个面的距离相等”这一推理过程是 ( )", "options": "A.归纳推理\nB.类比推理\nC. 演绎推理\nD. 联想推理", "subject": "立体几何学", "analysis": "解析:解答:圆的圆心 $\\Rightarrow$ 三棱雉的球的球心,相同类型,用类比方法\n\n分析: 本题主要考查了类比推理, 解决问题的关键是根据所给定义运用类比推理方法解决即可."} {"id": "20513", "image": [], "answer": "C", "solution": "null", "level": "高三", "question": "已知 $\\alpha, \\beta$ 是两个平面, 直线 $l$ 不在平面 $\\alpha$ 内, $l$ 也不在平面 $\\beta$ 内, 设(1) $l \\perp \\alpha$; (2) $l / / \\beta$;\n\n(3) $\\alpha \\perp \\beta$. 若以其中两个作为条件, 另一个作为结论, 则正确命题的个数为()", "options": "A. 0\nB. 1\nC. 2\nD. 3", "subject": "立体几何学", "analysis": "解析: 解答: 以(1) $l \\perp \\alpha$; (2) $l / / \\beta$; (3) $\\alpha \\perp \\beta$ 中两个作为条件, 另一个作为结论, 可得命题有(1)若 $l \\perp \\alpha, \\quad l / / \\beta$, 则 $\\alpha \\perp \\beta$, 真命题;\n\n若 $l \\perp \\alpha, \\alpha \\perp \\beta$, 则 $l / / \\beta$ 真命题; (3) $l / / \\beta, \\alpha \\perp \\beta$, 则 $l \\perp \\alpha$, 假命题, 故选 C.\n\n分析: 本题主要考查了综合法的思考过程、特点及应用, 解决问题的关键是根据所给命题之间的关系运用综合法进行分析即可."} {"id": "23047", "image": [], "answer": "C", "solution": "null", "level": "高三", "question": "用反证法证明“三角形中最多只有一个内角为针角”, 下列假设中正确的是( )", "options": "A. 有两个内角是针角\nB. 有三个内角是针角\nC. 至少有两个内角是钝角\nD. 没有一个内角是针角", "subject": "立体几何学", "analysis": "解析:【解答】“最多有一个”的反设是“至少有两个”.\n\n【分析】本题主要考查了反证法与放缩法, 解决问题的关键是根据三角形内角性质解析反设即可."} {"id": "20827", "image": ["9489.jpg"], "answer": "C", "solution": "null", "level": "高三", "question": "在空间中直线与平面的位置关系依次填入下图的 $M, N, E, F$ 中,顺序较为恰当的是( )\n\n\n\n(1)直线在平面内;(2)直线与平面平行;(3)直线在平面外;(4)直线与平面相交.", "options": "A.(1)(2)(3)(4)\nB.(1)(4)(2)(3)\nC.(1)(3)(2)(4)\nD.(2)(1)(4)(3)", "subject": "立体几何学", "analysis": "直线与平面的位置关系可分为直线在平面内和直线在平面外, 而直线在平面外又可分为直线与平面平行和直线与平面相交,所以 $M$ 为(1), $N$ 为(3), $E, F$ 分别为(2)、(4)或(4)、(2). 故选 C."} {"id": "21345", "image": [], "answer": "D", "solution": "null", "level": "高三", "question": "以 $B C$ 为斜边的 Rt $\\triangle A B C$ 中, $B C^{2}=A B^{2}+A C^{2}$, 由类比推理, 在三棱雉 $P-A B C$ 中, 若 $P A 、 P B 、 P C$ 两两垂直, $P A=a, P B=b, P C=c$, $S_{\\triangle B P^{P} C}=s_{1}, S_{\\triangle C P A}=s_{2}, S_{\\triangle A P B}=s_{3}$, 则 $S_{\\triangle A B C}=(\\quad)$", "options": "A. $\\sqrt{a^{2} b^{2}+b^{2} c^{2}+a^{2} c^{2}}$\nB. $\\sqrt{s_{1}^{2} s_{2}^{2}+s_{2}^{2} s_{3}^{2}+s_{3}^{2} s_{1}^{2}}$\nC. $\\sqrt{a^{2}+b^{2}+c^{2}}$\nD. $\\sqrt{s_{1}^{2}+s_{2}^{2}+s_{3}^{2}}$", "subject": "立体几何学", "analysis": "根据几何体和平面图形的类比关系,三角形的边应与四面体的各个面进行类比, 将三角形各边边长与四面体各面面积进行类比,在以 $B C$ 为斜边的 Rt $\triangle A B C$ 中, $B C^{2}=A B^{2}+A C^{2}$,对应地, 在三棱雉 $P-A B C$ 中, 若 $P A 、 P B 、 P C$ 两两垂直, $P A=a, P B=b, P C=c, S_{\triangle B^{P} C}=s_{1}$, $S_{\triangle C P A}=s_{2}, S_{\triangle A P B}=s_{3}$,所以, $S_{\triangle A B C}^{2}=s_{1}^{2}+s_{2}^{2}+s_{3}^{2}$, 即 $S_{\triangle A B C}=\\sqrt{s_{1}^{2}+s_{2}^{2}+s_{3}^{2}}$, 故选 D."} {"id": "20647", "image": [], "answer": "C", "solution": "null", "level": "高三", "question": "已知 $\\triangle A B C$ 的边长分别为 $a 、 b 、 c, \\triangle A B C$ 的面积为 $S$, 内切圆半径为 $r$,则 $r=\\frac{2 S}{a+b+c}$, 类比这一结论可知: 若三棱雉 $A-B C D$ 的四个面的面积分别为 $S_{1} 、 S_{2} 、 S_{3} 、 S_{4}$, 内切球半径为 $R$, 三棱雉 $A-B C D$ 的体积为 $V$, 则 $R=(\\quad)$", "options": "A. $\\frac{V}{S_{1}+S_{2}+S_{3}+S_{4}}$\nB. $\\frac{2 V}{S_{1}+S_{2}+S_{3}+S_{4}}$\n\nC. $\\frac{3 V}{S_{1}+S_{2}+S_{3}+S_{4}}$\nD. $\\frac{4 V}{S_{1}+S_{2}+S_{3}+S_{4}}$", "subject": "立体几何学", "analysis": "$\triangle A B C$ 的边长分别为 $a 、 b 、 c, \triangle A B C$ 的面积为 $S$, 内切圆半径为 $r$由等面积法可得 $S=\frac{1}{2} r(a+b+c), \therefore r=\frac{2 S}{a+b+c}$.类比这个结论:三棱雉 $A-B C D$ 的四个面的面积分别为 $S_{1} 、 S_{2} 、 S_{3} 、 S_{4}$, 内切球半径为 $R$, 三棱雉 $A-B C D$ 的体积为 $V$,由等体积法可得 $V=\frac{1}{3} R\\left(S_{1}+S_{2}+S_{3}+S_{4}\right), \therefore R=\frac{3 V}{S_{1}+S_{2}+S_{3}+S_{4}}$, 故选 C."} {"id": "21402", "image": [], "answer": "D", "solution": "null", "level": "高三", "question": "三角形的面积为 $S=\\frac{1}{2}(a+b+c) \\cdot r$, 其中 $a, b, c$ 为三角形的边长, $r$ 为三角形内切圆的半径, 则利用类比推理, 可得出四面体的体积为 ( )", "options": "A. $V=\\frac{1}{3} a b c$\n\nB. $V=\\frac{1}{3} S h$\n\nC. $V=\\frac{1}{3}(a b+b c+c a) h,(h$ 为四面体的高)\n\nD. $V=\\frac{1}{3}\\left(S_{1}+S_{2}+S_{3}+S_{4}\\right) r,\\left(S_{1}, S_{2}, S_{3}, S_{4}\\right.$ 分别为四面体的四个面的面积, $r$为四面体内切球的半径)", "subject": "立体几何学", "analysis": "设四面体的内切球的球心为 $O$, 则球心 $O$ 到四个面的距离都是 $r$, 将 $O$ 与四顶点连起来,可得四面体的体积等于以 $O$ 为顶点, 分别以四个面为底面的 4 个三棱雉体积的和,$\therefore V=\frac{1}{3}\\left(S_{1}+S_{2}+S_{3}+S_{4}\right) r$,故选 D."} {"id": "22669", "image": [], "answer": "D", "solution": "null", "level": "高三", "question": "$\\triangle \\mathrm{ABC}$ 在平面 $\\alpha$ 上的正射影是 ( )", "options": "A.三角形\nB. 直线\nC.线段\nD.三角形或线段", "subject": "立体几何学", "analysis": "解析:解答:当 $\\triangle \\mathrm{ABC}$ 所在平面垂直于 $\\alpha$ 时, $\\triangle \\mathrm{ABC}$ 在 $\\alpha$ 上的正射影是一条线段,否则是三角形.\n\n分析: 本题主要考查了平行射影, 解决问题的关键是根据平行射影的性质分析即可"} {"id": "22670", "image": [], "answer": "B", "solution": "null", "level": "高三", "question": "在长方体 $A B C D-A_{1} B_{1} C_{1} D_{1}$ 中, 四边形 $A_{1} A B B_{1}$ 在平面 $A B C D$ 上的正射影是 ( )", "options": "A. 四边形 $\\mathrm{ABCD}$\nB. 线段 $A B$\nC. $\\triangle \\mathrm{ABC}$\nD. 线段 $\\mathrm{A}_{1} \\mathrm{~B}_{1}$", "subject": "立体几何学", "analysis": "解析:解答:由于平面 $A_{1} A B B_{1} \\perp$ 平面 $A B C D$, 则四边形 $A_{1} A B B_{1}$ 在平面 $A B C D$ 上的正射影是线段 $A B$\n\n分析: 本题主要考查了平行射影, 解决问题的关键是根据平行射影的性质结合所给长方体的\n性质分析即可"} {"id": "22677", "image": [], "answer": "D", "solution": "null", "level": "高三", "question": "两条异面直线 $\\mathrm{m}$ 和 $\\mathrm{n}$ 在平面 $\\alpha$ 上的平行射影是 ( )", "options": "A. 一条直线和直线外一个点\nB. 两条相交直线\nC. 两条平行直线\nD.以上都有可能", "subject": "立体几何学", "analysis": "解析: 解答: 当 $\\mathrm{m}$ 和 $\\mathrm{n}$ 中有一条直线与投影方向平行时, 它们的平行射影是一个点和一条直线;否则是两条平行直线或相交直线.\n\n分析: 本题主要考查了平行射影, 解决问题的关键是根据平行射影的性质分析即可"} {"id": "21254", "image": [], "answer": "C", "solution": "null", "level": "高三", "question": "在空间四边形 $O A B C$ 中, $\\overrightarrow{\\mathrm{OA}}+\\overrightarrow{\\mathrm{AB}}-\\overrightarrow{\\mathrm{CB}}$ 等于()", "options": "A. $\\overrightarrow{\\mathrm{OA}}$\nB. $\\overrightarrow{\\mathrm{AB}}$\nC. $\\overrightarrow{\\mathrm{OC}}$\nD. $\\overrightarrow{\\mathrm{AC}}$", "subject": "立体几何学", "analysis": "根据向量的加法、减法法则, 得 $\\overrightarrow{\\mathrm{OA}}+\\overrightarrow{\\mathrm{AB}}-\\overrightarrow{\\mathrm{CB}}=\\overrightarrow{\\mathrm{OB}}-\\overrightarrow{\\mathrm{CB}}=\\overrightarrow{\\mathrm{OB}}+\\overrightarrow{\\mathrm{BC}}=\\overrightarrow{\\mathrm{OC}}$. 故选: $C$."} {"id": "22490", "image": ["9799.jpg"], "answer": "A", "solution": "null", "level": "高三", "question": "如图,,$l_{1} / / l_{2} / / l_{3}$, 直线 $a$ 分别与 $l_{1}, l_{2}, l_{3}$ 相交于点 $A, B, C$, 且 $A B=B C$, 直线 $b$ 分别与 $l_{1}, l_{2}, l_{3}$ 相交于点 $\\mathrm{A}_{1}, \\mathrm{~B}_{1}, \\mathrm{C}_{1}$, 则有 $(\\quad)$\n\n", "options": "A. $\\mathrm{A}_{1} \\mathrm{~B}_{1}=\\mathrm{B}_{1} \\mathrm{C}_{1}$\n\nB. $\\mathrm{A}_{1} \\mathrm{~B}_{1}>\\mathrm{B}_{1} \\mathrm{C}_{1}$\n\nC. $\\mathrm{A}_{1} \\mathrm{~B}_{1}<\\mathrm{B}_{1} \\mathrm{C}_{1}$\n\nD. $A_{1} B_{1}$ 与 $B_{1} C_{1}$ 的大小不确定", "subject": "变换几何", "analysis": "解析: 解答: $\\because \\mathrm{l}_{1} / / \\mathrm{l}_{2} / / \\mathrm{l}_{3}, \\mathrm{AB}=\\mathrm{BC}$,根据平行线等分线段定理, $\\therefore \\mathrm{A}_{1} \\mathrm{~B}_{1}=\\mathrm{B}_{1} \\mathrm{C}_{1}$. 故选 $\\mathrm{A}$.\n\n分析: 本题主要考查了平行线等分线段定理, 解决问题的关键是根据平行线等分线段定理方向即可."} {"id": "22757", "image": ["10064.jpg"], "answer": "B", "solution": "null", "level": "高三", "question": "直线 $2 x+3 y=0$ 经伸缩变换后变为 $x^{f}+y^{\\prime}=0$, 则该伸缩变换为 $(\\quad)$", "options": "A. $\\left\\{\\begin{array}{l}x^{\\prime}=\\frac{1}{2} x \\\\ y^{\\prime}=3 y\\end{array}\\right.$\nB. $\\left\\{\\begin{array}{l}x^{\\prime}=2 x \\\\ y^{\\prime}=3 y\\end{array}\\right.$\n$\\left\\{\\begin{array}{l}x^{\\prime}=2 x \\\\ y^{\\prime}=\\frac{1}{3} y\\end{array}\\right.$\nD. $\\left\\{\\begin{array}{l}x^{\\prime}=\\frac{1}{2} x \\\\ y^{\\prime}=\\frac{1}{3} y\\end{array}\\right.$", "subject": "变换几何", "analysis": "$2 x+3 y=0, \therefore \\lambda=2, \\mu=3$. 即 $\\left\\{\begin{array}{l}x^{\\prime}=2 x, \\ y^{\\prime}=3 y,\\end{array}\right.$"} {"id": "22774", "image": [], "answer": "A", "solution": "null", "level": "高三", "question": "将曲线 $y^{2}=4 x$ 按 $\\varphi:\\left\\{\\begin{array}{c}x^{s}=2 x, \\\\ 2 y^{\\prime}=y\\end{array}\\right.$ 变换后得到曲线的焦点坐标为( )", "options": "A. $\\left(\\frac{1}{6}, 0\\right)$ B. $\\left(\\frac{1}{4}, 0\\right)$\nC. $\\left(\\frac{1}{2}, 0\\right)$\nD. $(1,0)$", "subject": "变换几何", "analysis": "解析: 解答: 将曲线 $y^{2}=4 x$ 按 $\\varphi:\\left\\{\\begin{array}{l}x^{x}=2 x, \\\\ 2 y^{\\prime}=y,\\end{array}\\right.$ 变换后得到曲线方程为 $y^{\\prime 2}=\\frac{1}{2} x^{x}$, 所以焦点坐标为 $\\left(\\frac{1}{a}, 0\\right)$.\n\n分析: 本题主要考查了平面直角坐标轴中的伸缩变换, 解决问题的关键是根据伸缩变换计算即可"} {"id": "22491", "image": ["9800.jpg"], "answer": "C", "solution": "null", "level": "高三", "question": "如图, 在 $\\triangle A B C$ 中, $D, E$ 三等分 $A B, D F / / B C, E G / / B C$, 分别交 $A C$ 于 $F, G$, 若 $A C=15 \\mathrm{~cm}$, 则 $\\mathrm{FC}=(\\mathrm{r})$\n\n", "options": "A. 8 B. 9 C. 10 D. 11", "subject": "组合几何学", "analysis": "解析: 解答: $\\because \\mathrm{DF} / / \\mathrm{BC}, \\mathrm{EG} / / \\mathrm{BC}, \\therefore \\mathrm{DF} / / \\mathrm{EG} / / \\mathrm{BC}$. 由已知,得 $\\mathrm{AD}=\\mathrm{DE}=\\mathrm{EB}, \\therefore \\mathrm{AF}=\\mathrm{FG}=\\mathrm{GC}$.\n\n又 $\\because \\mathrm{AC}=15 \\mathrm{~cm}, \\therefore \\mathrm{FG}=\\mathrm{GC}=\\frac{1}{3} \\mathrm{AC}=5 \\mathrm{~cm} . \\therefore \\mathrm{FC}=\\mathrm{FG}+\\mathrm{GC}=10 \\mathrm{~cm}$.\n\n分析: 本题主要考查了平行线等分线段定理, 解决问题的关键是根据平行线等分线段定理计算即可."} {"id": "22492", "image": ["9801.jpg"], "answer": "A", "solution": "null", "level": "高三", "question": "如图, 在梯形 $\\mathrm{ABCD}$ 中, $\\mathrm{AD} / / \\mathrm{BC}, \\mathrm{AD}=2, \\mathrm{BC}=6, \\mathrm{E}, \\mathrm{F}$ 分别为对角线 $\\mathrm{BD}, \\mathrm{AC}$ 的中点, 则 $\\mathrm{EF}=(\\quad)$\n\n\n\n$\\begin{array}{llll}\\text {", "options": "A. } 2 & \\text { B. } 4 & \\text { C. } 6 & \\text { D. } 8\\end{array}$", "subject": "组合几何学", "analysis": "解析: 解答: 如图, 过点 $E$ 作 $\\mathrm{GE} / / \\mathrm{BC}$ 交 BA 于点 $\\mathrm{G} . \\because \\mathrm{E}$ 是 $\\mathrm{DB}$ 的中点, $\\therefore \\mathrm{G}$ 是 $\\mathrm{AB}$ 的中点.\n\n又 $\\because \\mathrm{F}$ 是 $\\mathrm{AC}$ 的中点, $\\therefore \\mathrm{GF} / / \\mathrm{BC}, \\therefore \\mathrm{G}, \\mathrm{E}, \\mathrm{F}$ 三点共线, $\\therefore \\mathrm{GE}=\\frac{1}{2} \\mathrm{AD}=1, \\mathrm{GF}=\\frac{1}{2} \\mathrm{BC}=3 . \\therefore$\n\n$\\mathrm{EF}=\\mathrm{GF}-\\mathrm{GE}=3-1=2$.\n\n![](https://cdn.mathpix.com/cropped/2024_04_19_81303e2fc6b69da0f33fg-"} {"id": "22509", "image": ["9824.jpg"], "answer": "D", "solution": "null", "level": "高三", "question": "如图, $, \\mathrm{A}, \\mathrm{B}, \\mathrm{C}, \\mathrm{D}$ 把 $\\mathrm{OE}$ 五等分, 且 $\\mathrm{AA}^{\\prime} / / / \\mathrm{BB}^{\\prime} / / \\mathrm{CC}^{\\prime} / / \\mathrm{DD}^{\\prime} / / \\mathrm{EE}^{\\prime}$,如果 $\\mathrm{OE}^{\\prime}=20 \\mathrm{~cm}$,那么 $\\mathrm{B}^{\\prime} \\mathrm{D}^{\\prime}$ 等于\n\n", "options": "A. $12 \\mathrm{~cm}$\nB. $10 \\mathrm{~cm}$\nC. $6 \\mathrm{~cm}$\nD. $8 \\mathrm{~cm}$", "subject": "度量几何学", "analysis": "解析: 解答: $\\because \\mathrm{A}, \\mathrm{B}, \\mathrm{C}, \\mathrm{D}$ 把 OE 五等分, $\\mathrm{AA}^{\\prime} / / \\mathrm{BB}^{\\prime} / / / \\mathrm{CC}^{\\prime} / / \\mathrm{DD}^{\\prime}, \\therefore \\mathrm{OA}^{\\prime}=\\mathrm{A}^{\\prime} \\mathrm{B}^{\\prime}=\\mathrm{B}^{\\prime} \\mathrm{C}^{\\prime}=\\mathrm{C}^{\\prime} \\mathrm{D}^{\\prime}=\\mathrm{D}^{\\prime} \\mathrm{E}^{\\prime}$.\n又 $\\because \\mathrm{OE}^{\\prime}=20 \\mathrm{~cm}, \\therefore \\mathrm{OA}^{\\prime}=\\mathrm{A}^{\\prime} \\mathrm{B}^{\\prime}=\\mathrm{B}^{\\prime} \\mathrm{C}^{\\prime}=\\mathrm{C}^{\\prime} \\mathrm{D}^{\\prime}=\\mathrm{D}^{\\prime} \\mathrm{E}^{\\prime}=4 \\mathrm{~cm} . \\therefore \\mathrm{B}^{\\prime} \\mathrm{D}^{\\prime}=\\mathrm{B}^{\\prime} \\mathrm{C}^{\\prime}+\\mathrm{C}^{\\prime} \\mathrm{D}^{\\prime}=8 \\mathrm{~cm}$.\n\n分析: 本题主要考查了平行线等分线段定理, 解决问题的关键是根据平行线等分线段定理计算即可."} {"id": "22780", "image": [], "answer": "A", "solution": "null", "level": "高三", "question": "有相距 $1400 \\mathrm{~m}$ 的 A, B 两个观察站, 在 A 站听到爆炸声的时间比在 B 站听到爆炸声的时间早 $4 \\mathrm{~s}$. 已知当时声音速度为 $340 \\mathrm{~m} / \\mathrm{s}$, 则爆炸点所在的曲线为 ( )", "options": "A.又曲线\nB.直线\nC.制圆\nD.抛物线", "subject": "度量几何学", "analysis": "解析:解答:由题意知, 爆炸点与点的距离差为定值 $340 \\times 4=1360<1400$,\n\n故爆炸点位于以为焦点的且离点 B 较远的双曲线的一支上.\n\n分析: 本题主要考查了平面直角坐标系与曲线方程, 解决问题的关键是根据所给曲线满足的条件分析计算即可"} {"id": "22528", "image": ["9849.jpg"], "answer": "B", "solution": "null", "level": "高三", "question": "如图, $\\mathrm{AB} / / \\mathrm{CD}, \\mathrm{AC}, \\mathrm{BD}$ 相交于 $\\mathrm{O}$ 点, $\\mathrm{BO}=7, \\mathrm{DO}=3, \\mathrm{AC}=25$, 则 $\\mathrm{AO}$ 的长为 $(\\quad)$\n\n", "options": "A. 10\nB", "subject": "度量几何学", "analysis": "."} {"id": "22529", "image": ["9850.jpg"], "answer": "D", "solution": "null", "level": "高三", "question": "如图,在 $\\triangle \\mathrm{ABC}$ 中, $\\mathrm{DE} / / \\mathrm{AB}, \\frac{\\mathrm{CD}}{\\mathrm{DA}}=\\frac{3}{2}$, 则 $\\frac{\\mathrm{DE}}{\\mathrm{AB}}$ 等于( )\n\n", "options": "A. $\\frac{1}{2}$\nB. $\\frac{2}{3}$\nC. $\\frac{3}{2}$\nD. $\\frac{3}{5}$", "subject": "度量几何学", "analysis": "解析: 解答: $\\because \\frac{\\mathrm{CD}}{\\mathrm{DA}}=\\frac{3}{2}, \\therefore \\frac{\\mathrm{CD}}{\\mathrm{CA}}=\\frac{3}{5}$. 又 $\\because \\mathrm{DE} / / \\mathrm{AB}, \\therefore \\frac{\\mathrm{DE}}{\\mathrm{AB}}=\\frac{\\mathrm{CD}}{\\mathrm{CA}}=\\frac{3}{5}$.\n\n分析: 本题主要考查了平行线分线段成比例定理, 解决问题的关键是根据平行线分线段成比例定理分析计算即可"} {"id": "22530", "image": ["9851.jpg"], "answer": "C", "solution": "null", "level": "高三", "question": "如图,已知 $\\mathrm{AD} / / \\mathrm{BE} / / \\mathrm{CF}, \\mathrm{EG} / / \\mathrm{FH}$,则下列等式成立的是( )\n\n", "options": "A. $\\frac{\\mathrm{AD}}{\\mathrm{BE}}=\\frac{\\mathrm{EG}}{\\mathrm{HF}}$\nB. $\\frac{\\mathrm{BE}}{\\mathrm{CF}}=\\frac{\\mathrm{EG}}{\\mathrm{HF}}$\nC. $\\frac{\\mathrm{AB}}{\\mathrm{AC}}=\\frac{\\mathrm{EG}}{\\mathrm{HF}}$\nD. $\\frac{\\mathrm{EF}}{\\mathrm{DE}}=\\frac{\\mathrm{AB}}{\\mathrm{AC}}$", "subject": "度量几何学", "analysis": "解析: 解答: $\\because \\mathrm{AD} / / \\mathrm{BE} / / \\mathrm{CF}, \\therefore \\frac{\\mathrm{AB}}{\\mathrm{AC}}=\\frac{\\mathrm{DE}}{\\mathrm{DF}}$. 又 $\\because \\mathrm{EG} / / \\mathrm{FH}, \\therefore \\frac{\\mathrm{BE}}{\\mathrm{CF}}=\\frac{\\mathrm{EG}}{\\mathrm{HF}}$.\n\n$\\therefore \\frac{\\mathrm{AB}}{\\mathrm{AC}}=\\frac{\\mathrm{EG}}{\\mathrm{HF}}, \\therefore$ 选项 C 成立; $\\because \\frac{\\mathrm{AD}}{\\mathrm{BE}} \\neq \\frac{\\mathrm{DE}}{\\mathrm{DF}}, \\therefore \\frac{\\mathrm{AD}}{\\mathrm{BE}} \\neq \\frac{\\mathrm{EG}}{\\mathrm{HF}}$.\n\n$\\therefore$ 选项 A 不成立;同理选项 B 不成立;\n\n很明显 $\\frac{\\mathrm{EF}}{\\mathrm{DE}}=\\frac{\\mathrm{BC}}{\\mathrm{BA}} \\neq \\frac{\\mathrm{AB}}{\\mathrm{AC}}, \\therefore$ 选项 $\\mathrm{D}$ 不成立,故选 C.\n\n分析: 本题主要考查了平行线分线段成比例定理, 解决问题的关键是根据平行线分线段成比例定理分析计算即可"} {"id": "22531", "image": ["9852.jpg"], "answer": "C", "solution": "null", "level": "高三", "question": "如图, $\\mathrm{AB} / / \\mathrm{EF} / / \\mathrm{CD}$,已知 $\\mathrm{AB}=20, \\mathrm{DC}=80$,那么 $\\mathrm{EF}$ 的长是 ( )\n\n", "options": "A. 10\nB. 12\nC. 16\nD. 18", "subject": "度量几何学", "analysis": "解析: 解答: $\\because \\mathrm{AB} / / \\mathrm{EF} / / \\mathrm{CD}, \\therefore \\frac{\\mathrm{AE}}{\\mathrm{EC}}=\\frac{\\mathrm{AB}}{\\mathrm{DC}}=\\frac{20}{80}=\\frac{1}{4} . \\therefore \\frac{\\mathrm{EF}}{\\mathrm{AB}}=\\frac{\\mathrm{EC}}{\\mathrm{AC}}=\\frac{4}{5}$.\n\n$\\therefore \\mathrm{EF}=\\frac{4}{5} \\mathrm{AB}=\\frac{4}{5} \\times 20=16$.\n\n分析: 本题主要考查了平行线分线段成比例定理, 解决问题的关键是根据平行线分线段成比例定理分析计算即可"} {"id": "22532", "image": ["9853.jpg"], "answer": "C", "solution": "null", "level": "高三", "question": "如图, 在平行四边形 $\\mathrm{ABCD}$ 中, $\\mathrm{N}$ 是 $\\mathrm{AB}$ 延长线上一点, 则 $\\frac{\\mathrm{BC}}{\\mathrm{BM}}-\\frac{\\mathrm{AB}}{\\mathrm{BN}}$ 的值为( )\n\n", "options": "A. $\\frac{1}{2}$\nB. $\\frac{2}{3}$\nC. 1\nD. $\\frac{3}{2}$", "subject": "度量几何学", "analysis": "解析: 解答: $\\because \\mathrm{DC} / / \\mathrm{BN}, \\therefore \\frac{\\mathrm{BC}}{\\mathrm{BM}}=\\frac{\\mathrm{ND}}{\\mathrm{MN}}$. 又 $\\because \\mathrm{BM} / / \\mathrm{AD}, \\therefore \\frac{\\mathrm{AB}}{\\mathrm{BN}}=\\frac{\\mathrm{DM}}{\\mathrm{MN}}$.\n$\\therefore \\frac{\\mathrm{BC}}{\\mathrm{BM}}-\\frac{\\mathrm{AB}}{\\mathrm{BN}}=\\frac{\\mathrm{ND}}{\\mathrm{MN}}-\\frac{\\mathrm{DM}}{\\mathrm{MN}}=\\frac{\\mathrm{ND}-\\mathrm{DM}}{\\mathrm{MN}}=\\frac{\\mathrm{MN}}{\\mathrm{MN}}=1$.\n\n分析: 本题主要考查了平行线分线段成比例定理, 解决问题的关键是根据平行线分线段成比例定理分析计算即可"} {"id": "22781", "image": [], "answer": "D", "solution": "null", "level": "高三", "question": "$\\mathrm{O}$ 为极点, $A\\left(2, \\frac{\\pi}{3}\\right), B\\left(5,-\\frac{7 \\pi}{6}\\right)$, 则 $S_{\\triangle A O B}=(\\quad)$", "options": "A. 2\nB. 3\nC. 4\nD. 5", "subject": "度量几何学", "analysis": "解析: 解答: 因为 $A\\left(2, \\frac{\\pi}{3}\\right), B\\left(5,-\\frac{7 \\pi}{6}\\right)$, 所以 $\\angle A O B=\\frac{\\pi}{2}$, 则三角形为直角三角形,则面积为 $\\frac{1}{2} \\times 2 \\times 5=5$.\n\n分析: 本题主要考查了点的极坐标和直角坐标的互化, 解决问题的关键是根据极坐标的点对应的直角坐标系中的点解析分析其几何关系计算即可."} {"id": "22803", "image": [], "answer": "A", "solution": "null", "level": "高三", "question": "在极坐标系中, 由三条直线 $\\theta=0, \\theta=\\frac{\\pi}{3}, \\rho \\cos \\theta+\\rho \\sin \\theta=1$ 围成图形的面积是", "options": "A. $\\frac{3-\\sqrt{3}}{4}$\nB. $\\frac{3+\\sqrt{3}}{4}$\nC. $\\frac{3+\\sqrt{3}}{2}$\nD. $\\frac{3-\\sqrt{3}}{2}$", "subject": "度量几何学", "analysis": "解析: 解答: 因为三条直线 $\\theta=0, \\theta=\\frac{\\pi}{3}, \\rho \\cos \\theta+\\rho \\sin \\theta=1$ 的直角坐标方程分别为 $y=0, y=\\sqrt{3} x, x+y=1$, 将 $x=\\frac{1}{\\sqrt{3}} y$ 代入 $x+y=1$, 解得 $y=\\frac{3-\\sqrt{3}}{2}$. 那么结合图形可知, 这三条直线 $\\theta=0, \\theta=\\frac{\\pi}{3}, \\rho \\cos \\theta+\\rho \\sin \\theta=1$ 围成的三角形的面积是 $S=\\frac{1}{2} \\times 1 \\times \\frac{3-\\sqrt{3}}{2}=\\frac{3-\\sqrt{3}}{4}$.\n\n分析: 本题主要考查了简单曲线的极坐标方程, 解决问题的关键是根据简单曲线的极坐标方程化简计算即可"} {"id": "22545", "image": ["9865.jpg"], "answer": "A", "solution": "null", "level": "高三", "question": "如图, 在 Rt $\\triangle A B C$ 中, $A C \\perp C B, C D \\perp A B$ 于点 $D$, 且 $C D=4$,则 $A D \\cdot D B$ 等于()\n\n", "options": "A. 16\nB. 4\nC. 2\nD.不确定", "subject": "度量几何学", "analysis": "解析: 解答: $\\because \\mathrm{AC} \\perp \\mathrm{CB}, \\mathrm{CD} \\perp \\mathrm{AB}, \\therefore \\mathrm{AD} \\cdot \\mathrm{DB}=\\mathrm{CD}^{2}$. 又 $\\because \\mathrm{CD}=4, \\therefore \\mathrm{AD} \\cdot \\mathrm{DB}=4^{2}=16$.\n\n分析: 本题主要考查了直角三角形的射影定理, 解决问题的关键是根据直角三角形的射影定理分析计算即可"} {"id": "22552", "image": ["9873.jpg"], "answer": "C", "solution": "null", "level": "高三", "question": "在 Rt $\\triangle \\mathrm{MNP}$ 中, $\\mathrm{MN} \\perp \\mathrm{MP}, \\mathrm{MQ} \\perp \\mathrm{PN}$ 于点 $\\mathrm{Q}$, 如图, 若 $\\mathrm{NQ}=3$, 则 $\\mathrm{MN}$ 等于()\n\n", "options": "A.3PN\nB. $\\frac{1}{3} \\mathrm{PN}$\nC. $\\sqrt{3 \\mathrm{PN}}$\nD.9PN", "subject": "度量几何学", "analysis": "解析: 解答: $\\because \\mathrm{MN} \\perp \\mathrm{MP}, \\mathrm{MQ} \\perp \\mathrm{PN}, \\therefore \\mathrm{MN}^{2}=\\mathrm{NQ} \\cdot \\mathrm{PN}$. 又 $\\because \\mathrm{NQ}=3, \\therefore \\mathrm{MN}=\\sqrt{\\mathrm{NQ} \\cdot \\mathrm{PN}}=\\sqrt{3 \\mathrm{PN}}$分析: 本题主要考查了直角三角形的射影定理, 解决问题的关键是根据直角三角形的射影定理分析计算即可"} {"id": "22553", "image": ["9874.jpg"], "answer": "C", "solution": "null", "level": "高三", "question": "在 Rt $\\triangle \\mathrm{ABC}$ 中, $\\angle \\mathrm{BAC}=90^{\\circ}, \\mathrm{AD} \\perp \\mathrm{BC}$ 于点 $\\mathrm{D}$, 若 $\\frac{\\mathrm{AC}}{\\mathrm{AB}}=\\frac{3}{4}$, 则 $\\frac{\\mathrm{BD}}{\\mathrm{CD}}$ 等于()\n\n", "options": "A. $\\frac{3}{4}$\nB. $\\frac{4}{3}$\nC. $\\frac{16}{9}$\nD. $\\frac{9}{16}$", "subject": "度量几何学", "analysis": "解析: 解答: 如图, 由射影定理,得 $\\mathrm{AC}^{2}=\\mathrm{CD} \\cdot \\mathrm{BC}, \\mathrm{AB}^{2}=\\mathrm{BD} \\cdot \\mathrm{BC}, \\therefore \\frac{\\mathrm{AC}^{2}}{\\mathrm{AB}^{2}}=\\frac{\\mathrm{CD}}{\\mathrm{BD}}=\\left(\\frac{3}{4}\\right)^{2}$\n\n即 $\\frac{\\mathrm{CD}}{\\mathrm{BD}}=\\frac{9}{16}, \\therefore \\frac{\\mathrm{BD}}{\\mathrm{CD}}=\\frac{16}{9}$\n\n分析: 本题主要考查了直角三角形的射影定理, 解决问题的关键是根据直角三角形的射影定理分析计算即可"} {"id": "22554", "image": [], "answer": "A", "solution": "null", "level": "高三", "question": "在矩形 $\\mathrm{ABCD}$ 中, $\\mathrm{AB}=4, \\mathrm{BC}=6$, 则线段 $\\mathrm{AC}$ 在 $\\mathrm{AB}$ 上的射影长等于()", "options": "A. 4\nB. 6\nC. 2\nD. $2 \\sqrt{13}$", "subject": "度量几何学", "analysis": "解析: 解答: $\\because \\mathrm{BC} \\perp \\mathrm{AB}, \\therefore \\mathrm{AC}$ 在 $\\mathrm{AB}$ 上的射影是 $\\mathrm{AB}$.\n\n分析: 本题主要考查了直角三角形的射影定理, 解决问题的关键是根据直角三角形的射影定理分析计算即可, 难度不大"} {"id": "22555", "image": ["9875.jpg"], "answer": "B", "solution": "null", "level": "高三", "question": "如图, 在 $\\triangle A B C$ 中, $A B \\perp A C, A D \\perp B C$ 于点 $D$, 若 $B D \\cdot D C=16$, 则 $A D$ 等于 ( )\n\n", "options": "A. 2\nB. 4\nC. 16\n\nD.不确定", "subject": "度量几何学", "analysis": "解析:解答:由题意知, $\\mathrm{AD}^{2}=\\mathrm{BD} \\cdot \\mathrm{DC}=16$,故 $\\mathrm{AD}=4$.\n\n分析: 本题主要考查了直角三角形的射影定理, 解决问题的关键是根据直角三角形的射影定理分析计算即可解决问题"} {"id": "22556", "image": ["9876.jpg"], "answer": "A", "solution": "null", "level": "高三", "question": "如图, 在 Rt $\\triangle \\mathrm{MNP}$ 中, $\\mathrm{MN} \\perp \\mathrm{MP}, \\mathrm{MQ} \\perp \\mathrm{PN}$ 于点 $\\mathrm{Q}, \\mathrm{MN}=3, \\mathrm{PN}=9$, 则 $N Q$ 等于()\n\n", "options": "A. 1\nB. 3\nC. 9\nD. 27", "subject": "度量几何学", "analysis": "解析: 解答: $\\because \\mathrm{MN}^{2}=\\mathrm{NQ} \\cdot \\mathrm{NP}, \\therefore 3^{2}=9 \\mathrm{NQ} . \\therefore \\mathrm{NQ}=1$.\n\n分析: 本题主要考查了直角三角形的射影定理, 解决问题的关键是根据直角三角形的射影定理计算即可, 属于普通题目"} {"id": "22557", "image": ["9877.jpg"], "answer": "C", "solution": "null", "level": "高三", "question": "如图, 在 $\\triangle A B C$ 中, $\\angle A C B=90^{\\circ}, C D \\perp A B$ 于点 $D, A D=3, B D=2$, 则 $A C: B C$ 的值是()\n\n", "options": "A. $3: 2$\nB. $9: 4$\nC. $\\sqrt{3}: \\sqrt{2}$\nD. $\\sqrt{2}: \\sqrt{3}$", "subject": "度量几何学", "analysis": "解析: 解答: 在 Rt $\\triangle \\mathrm{ABC}$ 中, $\\angle \\mathrm{ACB}=90^{\\circ}, \\mathrm{CD} \\perp \\mathrm{AB}$, 由射影定理知, $\\mathrm{AC}=\\mathrm{AD} \\cdot \\mathrm{AB}, \\mathrm{BC}^{2}=\\mathrm{BD} \\cdot \\mathrm{AB}$.又 $\\because A D=3, B D=2, \\quad \\therefore A B=A D+B D=5$,\n\n$\\therefore \\mathrm{AC}^{2}=3 \\times 5=15, \\mathrm{BC}^{2}=2 \\times 5=10 . \\therefore \\frac{\\mathrm{AC}}{\\mathrm{BC}}=\\frac{\\sqrt{15}}{\\sqrt{10}}=\\frac{\\sqrt{3}}{\\sqrt{2}}$, 即 $\\mathrm{AC}: \\mathrm{BC}=\\sqrt{3}: \\sqrt{2}$\n\n分析: 本题主要考查了直角三角形的射影定理, 解决问题的关键是根据直角三角形的射影定理结合所给条件复数计算即可解决问题, 有一定难度"} {"id": "21103", "image": [], "answer": "B", "solution": "null", "level": "高三", "question": "如图, 已知粗圆 $C$ 的中心为原点 $0, F(-2 \\sqrt{5}, 0)$ 为 $C$ 的左焦点, $P$为 $\\mathrm{C}$ 上一点, 满足 $|\\mathrm{OP}|=|\\mathrm{OF}|$, 且 $|\\mathrm{PF}|=4$, 则粗圆 $\\mathrm{C}$ 的方程为 ( )", "options": "A. $\\frac{\\mathrm{x}^{2}}{25}+\\frac{\\mathrm{y}^{2}}{5}=1$\nB. $\\frac{x^{2}}{36}+\\frac{y^{2}}{16}=1$\nC. $\\frac{x^{2}}{30}+\\frac{y^{2}}{10}=1$\nD. $\\frac{\\mathrm{x}^{2}}{45}+\\frac{\\mathrm{y}^{2}}{25}=1$\n\n![](https://cdn.mathpix.com/cropped/2024_04_19_c70ed9230755576ee53ag-", "subject": "度量几何学", "analysis": "设椭圆标准方程为 $\frac{\\mathrm{x}^{2}}{\\mathrm{a}^{2}}+\frac{\\mathrm{y}^{2}}{\\mathrm{~b}^{2}}=1(\\mathrm{a}>\\mathrm{b}>0)$, 焦距为 $2 \\mathrm{c}$, 右焦点为 $\\mathrm{F}^{\\prime}$, 连接 $\\mathrm{PF}^{\\prime}$, 如图所示. 因为 $\\mathrm{F}(-2 \\sqrt{5}, 0)$ 为 $\\mathrm{C}$ 的左焦点, 所以 $\\mathrm{c}=2 \\sqrt{5}$. 由 $|\\mathrm{OP}|=|\\mathrm{OF}|=\\left|\\mathrm{OF}^{\\prime}\right|$ 知, $\\angle \\mathrm{PFF}^{\\prime}=\\angle \\mathrm{FPO}$, $\\angle \\mathrm{OF}^{\\prime} \\mathrm{P}=\\angle \\mathrm{OPF}^{\\prime}$, 所 $\\angle \\mathrm{PFF}^{\\prime}+\\angle \\mathrm{OF}^{\\prime} \\mathrm{P}=\\angle \\mathrm{FPO}+\\angle \\mathrm{OPF}^{\\prime}$, 由 $\\angle \\mathrm{PFF}^{\\prime}+\\angle \\mathrm{OF}^{\\prime} \\mathrm{P}+\\angle \\mathrm{FPO}+\\angle \\mathrm{OPF}^{\\prime}=180^{\\circ}$ 知, $\\angle \\mathrm{FPO}+\\angle \\mathrm{OPF}=90^{\\circ}$, 即 $\\mathrm{PF} \\perp \\mathrm{PF}^{\\prime}$. 在 Rt $\triangle \\mathrm{PFF}^{\\prime}$ 中, 由勾股定理, 得 $\\left|\\mathrm{PF}^{\\prime}\right|=\\sqrt{\\left|\\mathrm{FF}^{\\prime}\right|^{2}-|\\mathrm{PF}|^{2}}=$ $\\sqrt{(4 \\sqrt{5})^{2}-4^{2}}=8$, 由椭圆定义, 得 $|P F|+\\left|P F^{\\prime}\right|=2 a=4+8=12$, 从而 $a=6$, 得 $a^{2}=36$, 于是 $b^{2}=$ $a^{2}-c^{2}=36-(2 \\sqrt{5})^{2}=16$, 所以粗圆的方程为 $\frac{x^{2}}{36}+\frac{y^{2}}{16}=1$. 故选 B."} {"id": "21123", "image": [], "answer": "D", "solution": "null", "level": "高三", "question": "以椭圆短轴为直径的圆经过此椭圆长轴的两个三等分点, 则粗圆离心率是 ( )", "options": "A. $\\frac{1}{3}$\nB. $\\frac{\\sqrt{3}}{3}$\nC. $\\frac{\\sqrt{3}}{4}$\nD. $\\frac{2 \\sqrt{2}}{3}$", "subject": "度量几何学", "analysis": "根据题意, 以粗圆短轴为直径的圆经过此椭圆的长轴的两个三等分点, 则有 $2 \\mathrm{~b}=\frac{2 \\mathrm{a}}{3}$,即 $a=3 b$, 则 $c=\\sqrt{a^{2}-b^{2}}=2 \\sqrt{2} b$, 则椭圆的离心率 $e=\frac{c}{a}=\frac{2 \\sqrt{2}}{3}$; 故选: D."} {"id": "22559", "image": ["9879.jpg"], "answer": "B", "solution": "null", "level": "高三", "question": "如图,在 $\\odot \\mathrm{O}$ 中, $\\angle \\mathrm{BAC}=25^{\\circ}$, 则 $\\angle \\mathrm{BOC}$ 等于 ( )\n\n", "options": "A. $25^{\\circ} \\quad$ B. $50^{\\circ}$\n\nC. $30^{\\circ}$\nD.", "subject": "度量几何学", "analysis": "$"} {"id": "22576", "image": ["9901.jpg"], "answer": "A", "solution": "null", "level": "高三", "question": "如图, $A B$ 是 $\\odot O$ 的直径, $C$ 是 $A B$ 上的一点, 且 $A C=4, B C=3$, 则 $\\odot O$ 的半径 $r$ 等于 ( )\n\n", "options": "A. $\\frac{5}{2}$\nB. 5\nC. 10\nD. 不确定", "subject": "度量几何学", "analysis": "解析: 解答: $\\because \\mathrm{AB}$ 是 $\\odot \\mathrm{O}$ 的直径,\n\n$\\therefore \\angle \\mathrm{ACB}=90^{\\circ}$.\n\n$\\therefore \\mathrm{AB}=\\sqrt{\\mathrm{AC}^{2}+\\mathrm{BC}^{2}}=\\sqrt{4^{2}+3^{2}}=5$.\n\n$\\therefore 2 \\mathrm{r}=\\mathrm{AB}=5 . \\therefore \\mathrm{r}=\\frac{5}{2}$.\n分析: 本题主要考查了圆周角定理, 解决问题的关键是根据圆周角定理结合所给条件计算即可"} {"id": "22411", "image": [], "answer": "C", "solution": "null", "level": "高三", "question": "10 件产品中有 3 件次品, 从中任取 2 件, 可作为随机变量的是( )", "options": "A. 取到产品的件数\nB. 取到正品的概率\nC.取到次品的件数\nD. 取到次品的概率", "subject": "计数", "analysis": "答案:C\n\n【解析】逐一考查所给的选项, A 中取到产品的件数是一个常量而不是变量, B,D 中的量也是一个定值, 而 C 中取到次品的件数可能是 $0,1,2$, 是随机变量."} {"id": "22412", "image": [], "answer": "B", "solution": "null", "level": "高三", "question": "已知下列随机变量:\n\n(1) 10 件产品中有 2 件次品, 从中任选 3 件, 取到次品的件数 $X$;\n\n(2) 一位射击手对目标进行射击,击中目标得 1 分,未击中目标得 0 分, 该射击手在一次射击中的得分 $X$;\n\n(3) 一天内的温度 $X$;\n\n(4) 在体育彩票的抽奖中,一次摇号产生的号码数 $X$.\n\n其中 $X$ 是离散型随机变量的是( )", "options": "A.(1)(2)(3)\nB.(1)(2)(4)\nC.(2)(3)(4)\nD. (3)(4)", "subject": "计数", "analysis": "答案:B\n\n【解析】易知(1)(2)(4)中的 $X$ 是离散型随机变量.对于(3),一天内的温度不能一一列出,它可以取一天内的最低与最高温度之间的所有值,故不是离散型随机变量."} {"id": "22413", "image": [], "answer": "C", "solution": "null", "level": "高三", "question": "在下列叙述中,是离散型随机变量的为( )", "options": "A. 某人早晨在车站等出租车的时间\n\nB. 将一颗均匀硬币郑十次, 出现正面或反面的次数\n\nC.连续不断的射击,首次命中目标所需要的次数\n\nD. 袋中有 2 个黑球、6 个红球,任取 2 个,取得一个红球的可能性", "subject": "计数", "analysis": "答案:C\n\n【解析】A 项是随机变量,但不能一一列出,不是离散型. B 项掷硬币不是正面就是反面,次数之和为十,是常量. $D$ 项事件发生的可能性不是随机变量,所以选 C."} {"id": "22414", "image": [], "answer": "C", "solution": "null", "level": "高三", "question": "某人进行射击, 共有 10 发子弹, 若击中目标或子弹打完就停止射击, 射击次数为 $\\xi$ 则 $\\xi=10$,表示的试验结果是( )", "options": "A.第 10 次击中目标\nB.第 10 次未击中目标\nC. 前 9 次未击中目标\nD. 第 9 次击中目标", "subject": "计数", "analysis": "答案:C\n\n【解析】击中目标或子弹打完就停止射击,射击次数 $\\xi=10$, 则说明前 9 次均未击中目标, 第 10 次击中目标或未击中目标,故选 C."} {"id": "22418", "image": [], "answer": "B", "solution": "null", "level": "高三", "question": "随机变量 $\\xi^{\\xi}$ 的分布列如表所示, 则 $E(\\xi)$ 的最大值是( )\n\n| $\\xi$ | -1 | 0 | $a$ |\n| :---: | :---: | :---: | :---: |\n| $P$ | $\\frac{1}{4}$ | $\\frac{1}{2}+a$ | $\\frac{1}{4}-b$ |", "options": "A. $-\\frac{5}{8}$\nB. $-\\frac{15}{64}$\nC. 4\nD. $-\\frac{19}{64}$", "subject": "计数", "analysis": "答案:B\n\n【解析】根据随机变量分布列的性质, 所有的概率之和为 1 , 且每个概率都介于 0 和 1 之间,\n\n得到 $b-a=0,-\\frac{1}{2}0)}$, 且 $E(Y)=10, D(Y)=4$, 则 $a$ 与 $b$ 的值为 ( )", "options": "A. ${ }^{a=10, b=3}$\nB. ${ }^{a=3, b=10}$\nC. ${ }^{a=5, b=6}$\nD. $a=6, b=5$", "subject": "计数", "analysis": "答案:C\n\n【解析】由随机变量 $X$ 的分布列可知, $m=1-"} {"id": "22420", "image": [], "answer": "B", "solution": "null", "level": "高三", "question": "已知 $X$ 为随机变量, 则下列说法错误的是( )", "options": "A. $P\\left(|X| \\leq \\frac{1}{2}\\right) \\leq P\\left(X^{2} \\leq \\frac{1}{2}\\right)$\nB. $D\\left(X^{2}\\right)=D\\left((1-X)^{2}\\right)$\nC. $D(X)=D(1-X)$\nD. $[E(X)]^{2} \\leq E\\left(X^{2}\\right)$", "subject": "计数", "analysis": "】对于 A $P\\left(|X| \frac{1}{2}\right)=P\\left(-\frac{1}{2} \\leq X \\leq \frac{1}{2}\right), P\\left(X^{2} \\leq \frac{1}{2}\right)=P\\left(-\frac{\\sqrt{2}}{2} \\leq X \\leq \frac{\\sqrt{2}}{2}\right)$ 故 A 正确.对于 $\\mathrm{B}$, 取特殊值 $P(X=1)=P(X=-1)=\frac{1}{2}$, 则 $E\\left(X^{2}\right)=\frac{1}{2} \times 1+\frac{1}{2} \times 1=1$ $D\\left(X^{2}\right)=(1-1)^{2} \times \frac{1}{2}+(1-1)^{2} \times \frac{1}{2}=0 E\\left((1-X)^{2}\right)=\frac{1}{2} \times 0+\frac{1}{2} \times 4=2$ $D\\left((1-X)^{2}\right)=\frac{1}{2} \times(0-2)^{2}+\frac{1}{2} \times(4-2)^{2}=4$, 故 $D\\left((1-X)^{2}\right) \neq D\\left(X^{2}\right) \\mathrm{B}$ 错对于 C, $D(1-X)=(-1)^{2} D(X)=D(X)$, C 正确对于 $\\mathrm{D}, D(X)=E\\left(X^{2}\right)-[E(X)]^{2} \\geq 0$, D 正确"} {"id": "22421", "image": [], "answer": "B", "solution": "null", "level": "高三", "question": "已知随机变量 $X, Y$ 满足 $Y=2 X+1$, 且随机变量 ${ }^{X}$ 的分布列如下:\n\n| $X$ | 0 | 1 | 2 |\n| :---: | :---: | :---: | :---: |\n| $P$ | $\\frac{1}{6}$ | $\\frac{1}{3}$ | $a$ |\n\n则随机变量 ${ }^{Y}$ 的方差 $D(Y)=(\\quad)$", "options": "A. $\\frac{5}{9}$\nB. $\\frac{20}{9}$\nC. $\\frac{4}{3}$\nD. $\\frac{29}{9}$", "subject": "计数", "analysis": "答案:B\n\n【解析】由分布列的性质, 得 $a=1-\\frac{1}{6}-\\frac{1}{3}=\\frac{1}{2}$ 所以 $E(X)=0 \\times \\frac{1}{6}+1 \\times \\frac{1}{3}+2 \\times \\frac{1}{2}=\\frac{4}{3}$, 所以 $D(X)=\\left(0-\\frac{4}{3}\\right)^{2} \\times \\frac{1}{6}+\\left(1-\\frac{4}{3}\\right)^{2} \\times \\frac{1}{3}+\\left(2-\\frac{4}{3}\\right)^{2} \\times \\frac{1}{2}=\\frac{5}{9}$, 又 $Y=2 X+1$, 所以 $D(Y)=4 D(X)=\\frac{20}{9}$."} {"id": "22422", "image": [], "answer": "C", "solution": "null", "level": "高三", "question": "已知随机变量 $\\xi-B(n, p), E \\xi=15, D \\xi=\\frac{45}{4}$, 则 $n$ 及 $p$ 的值分别为 ( )", "options": "A. $50 ; \\frac{1}{4}$\nB. $50 ; \\frac{3}{4}$\nC. $60 ; \\frac{1}{4}$\nD. $60 ; \\frac{3}{4}$", "subject": "计数", "analysis": "答案:C\n\n【解析】$\\because \\xi-B(n, p)$ 且 $\\xi=15, D \\xi=\\frac{45}{4} \\therefore\\left\\{\\begin{array}{c}n p=15 \\\\ n p(1-p)=\\frac{45}{4}\\end{array}\\right.$, 解得 $\\left\\{\\begin{array}{c}n=60 \\\\ p=\\frac{1}{4}\\end{array}\\right.$"} {"id": "22423", "image": [], "answer": "A", "solution": "null", "level": "高三", "question": "随机变量 $X$ 的分布列为:\n\n| $X$ | 1 | 2 | 4 |\n| :--- | :--- | :--- | :--- |\n\n\n| $P$ | 0.4 | 0.3 | 0.3 |\n| :--- | :--- | :--- | :--- |\n\n那么 $E(5 X+4)$ 等于( )", "options": "A. 15\nB. 11\nC.", "subject": "计数", "analysis": "答案:A\n\n【解析】由分布列可得出 $E(X)=1 \\times"} {"id": "22426", "image": [], "answer": "A", "solution": "null", "level": "高三", "question": "甲、乙两人独立地对同一目标各射击一次, 命中率分别为 0.6 和 0.7 , 在目标被击中的情况下,甲、乙同时击中目标的概率为( )", "options": "A. $\\frac{21}{44}$\nB. $\\frac{15}{22}$\nC. $\\frac{21}{50}$\nD. $\\frac{9}{25}$", "subject": "计数", "analysis": "答案: A\n\n【解析】根据题意, 记“甲击中目标”为事件 $A$, “乙击中目标”为事件 B, “目标被击中”为事件 $C$, 则 $P(C)=1-F(\\bar{A}) P(\\bar{B})=1-(1-"} {"id": "22427", "image": [], "answer": "D", "solution": "null", "level": "高三", "question": "小明早上步行从家到学校要经过有红绿灯的两个路口, 根据经验, 在第一个路口遇到红灯的概率为 0.4 , 在第二个路口遇到红灯的概率为 0.5 , 在两个路口连续遇到红灯的概率是 0.2 .某天早上小明在第一个路口遇到了红灯, 则他在第二个路口也遇到红灯的概率是( )", "options": "A.", "subject": "计数", "analysis": "记“小明在第一个路口遇到红灯”为事件 $A$, “小明在第二个路口遇到红灯”为事件 $B$,则 $P(A)=0.4, P(B)=0.5, P(A \\cap B)=0.2$, 所以 $P(B \\mid A)=\frac{P(A \\cap B)}{P(A)}=\frac{0.2}{0.4}=0.5$, 故选 D."} {"id": "22428", "image": [], "answer": "C", "solution": "null", "level": "高三", "question": "已知 $P(B \\mid A)=\\frac{1}{3}, P(A)=\\frac{2}{5}$, 则 $P(A \\cap B)$ 等于( )", "options": "A. $\\frac{5}{6}$\nB. $\\frac{9}{10}$\nC. $\\frac{2}{15}$\nD. $\\frac{1}{15}$", "subject": "计数", "analysis": "由乘法公式得 $P(A \\cap B)=P(B \\mid A) P(A)=\frac{1}{3} \times \frac{2}{5}=\frac{2}{15}$, 故选 C."} {"id": "22429", "image": [], "answer": "A", "solution": "null", "level": "高三", "question": "某种疾病的患病率为 $0.5 \\%$, 已知在患该种疾病的条件下血检呈阳性的概率为 $99 \\%$, 则患该种疾病且血检呈阳性的概率为( )", "options": "A. $", "subject": "计数", "analysis": "答案: A\n\n【解析】 设事件 $A=$ 血检呈阳性”, $B=$ “患该种疾病”, 依题意知 $P(B)="} {"id": "22430", "image": [], "answer": "A", "solution": "null", "level": "高三", "question": "高一新生健康检查的统计结果: 体重超重者占 $40 \\%$, 血压异常者占 $15 \\%$, 两者都有的占 $8 \\%$今任选一人进行健康检查, 已知此人体重超重, 他血压异常的概率为( )", "options": "A. $\\frac{1}{5}$\nB. $\\frac{2}{5}$\nC. $\\frac{3}{5}$\nD. $\\frac{4}{5}$", "subject": "计数", "analysis": "记事件 $A$ 表示体重超重, 事件 $B$ 表示血压异常.则 $P(B \\mid A)=\frac{P(A \\cap B)}{P(A)}=\frac{0.08}{0.4}=\frac{1}{5}$ 故选A."} {"id": "22431", "image": [], "answer": "D", "solution": "null", "level": "高三", "question": "一个家庭有两个小孩,假设生男生女是等可能的,已知这个家庭有一个是女孩的条件下, 这时另一个也是女孩的概率是( )", "options": "A. $\\frac{1}{4}$\nB. $\\frac{2}{3}$\nC. $\\frac{1}{2}$\nD. $\\frac{1}{3}$", "subject": "计数", "analysis": "一个家庭中有两个小孩只有 4 种可能:(男,男),(男,女),(女,男),(女,女).记事件 A 为“其中一个是女孩”, 事件 B 为“另一个是女孩”, 则 $\\mathrm{A}=\\{$ (男,女),(女,男),(女,女) $\\}, \\mathrm{B}=\\{$ (男,女),(女,男),(女,女) $\\}, \\mathrm{AB}=\\{($ 女,女) $\\}$.于是可知 $P(A)=\frac{3}{4}, P(A B)=\frac{1}{4}$ 问题是求在事件 A 发生的情况下, 事件 B 发生的概率, 即求$\\mathrm{P}(\\mathrm{B} \\mid \\mathrm{A})$,由条件概率公式, 得 $$ P(B \\mid A)=\frac{\frac{1}{4}}{\frac{3}{4}}=\frac{1}{3}$$"} {"id": "22437", "image": [], "answer": "D", "solution": "null", "level": "高三", "question": "甲、乙两人参加“社会主义价值观”知识竞赛, 甲、乙两人能荣获一等奖的概率分別为 $\\frac{2}{3}$ 和 $\\frac{3}{4}$ ,甲、乙两人是否获得一等奖相互独立, 则这两个人中恰有一人获得一等奖的概率为( )", "options": "A. $\\frac{3}{4}$\nB. ${ }^{\\frac{2}{3}}$\nC. $\\frac{5}{7}$\nD. $\\frac{5}{12}$", "subject": "计数", "analysis": "答案: D\n\n解析: 根据题意, 恰有一人获得一等奖就是甲获得乙没获得或甲没获得乙获得, 则所求概率\n\n是 $\\frac{2}{3} \\times\\left(1-\\frac{3}{4}\\right)+\\frac{3}{4} \\times\\left(1-\\frac{2}{3}\\right)=\\frac{5}{12}$, 故选 D"} {"id": "22438", "image": [], "answer": "A", "solution": "null", "level": "高三", "question": "某高三学生进行考试心理素质测试, 场景相同的条件下每次通过测试的概率为 $\\frac{4}{5}$, 则连续测试 4 次, 至少有 3 次通过的概率为 ( )", "options": "A. $\\frac{512}{625}$\nB. $\\frac{256}{625}$\nC. $\\frac{64}{625}$\nD. $\\frac{64}{125}$", "subject": "计数", "analysis": "答案: A\n\n解析: 某高三学生进行考试心理素质测试,场景相同的条件下每次通过测试的概率为 $\\frac{4}{5}$,则连续测试 4 次, 至少有 3 次通过的概率为:\n\n$P=C_{4}^{3}\\left(\\frac{4}{5}\\right)^{3}\\left(\\frac{1}{5}\\right)+C_{4}^{4}\\left(\\frac{4}{5}\\right)^{4}=\\frac{512}{625}$.\n\n故选: A."} {"id": "22442", "image": [], "answer": "B", "solution": "null", "level": "高三", "question": "投篮测试中,每人投 3 次, 至少投中 2 次才能通过测试.已知某同学每次投篮投中的概率为 0.6 ,且各次投篮是否投中相互独立, 则该同学通过测试的概率为( )", "options": "A.", "subject": "计数", "analysis": "\\Rightarrow p=\\frac{2}{5}$ 或 $p=\\frac{3}{5}$\n\n又 $p(x=4)=C_{10}^{4} \\cdot p^{4} \\cdot(1-p)^{6}\\frac{1}{2}$\n\n所以 $p=\\frac{3}{5}=0.6$\n\n故答案为: B\n\n4.答案: B\n\n【解析】$\\because$ 随机变量 ${ }^{\\xi \\sim B(2, p)}$,\n\n$\\therefore P(\\xi=0)=(1-p)^{2}$,\n$\\therefore P(\\xi \\ldots 1)=1-P(\\xi=0)=1-(1-p)^{2}=\\frac{5}{9}$,\n\n解得 $p=\\frac{1}{3}$.\n\n故选: B."} {"id": "22443", "image": [], "answer": "B", "solution": "null", "level": "高三", "question": "某群体中的每位成员使用移动支付的概率都为 $p$, 各成员的支付方式相互独立, 设 $X$ 为该群体的 10 位成员中使用移动支付的人数, $D X=2.4, P(X=4)\frac{1}{2}$所以 $p=\frac{3}{5}=0.6$故答案为: B"} {"id": "22444", "image": [], "answer": "B", "solution": "null", "level": "高三", "question": "设随机变量 $\\zeta^{\\sim} \\sim B(2, p)$, 若 $P(\\xi \\geq 1)=\\frac{5}{9}$, 则 ${ }^{p}$ 的值为( )", "options": "A. $\\frac{1}{4}$\nB. $\\frac{1}{3}$\nC. $\\frac{2}{3}$\nD. $\\frac{16}{27}$", "subject": "计数", "analysis": "$\because$ 随机变量 ${ }^{\\xi \\sim B(2, p)}$,$\therefore P(\\xi=0)=(1-p)^{2}$,$\therefore P(\\xi \\ldots 1)=1-P(\\xi=0)=1-(1-p)^{2}=\frac{5}{9}$,解得 $p=\frac{1}{3}$."} {"id": "22445", "image": [], "answer": "A", "solution": "null", "level": "高三", "question": "设随机变量 $\\mathrm{X}$ 服从二项分布 $B\\left(6, \\frac{1}{2}\\right)$, 则 $P(X=3)$ 等于( )", "options": "A. $\\frac{5}{16}$\nB. $\\frac{3}{16}$\nC. $\\frac{5}{8}$\nD. $\\frac{3}{8}$", "subject": "计数", "analysis": "答案: A\n\n【解析】根据条件中所给的变量符合二项分布, 写出变量取值不同时对应的概率公式, 本题 $X=3$, 代入公式得到要求的概率: $P(X=3)=\\mathrm{C}_{6}^{3} \\times\\left(\\frac{1}{2}\\right)^{3} \\times\\left(\\frac{1}{2}\\right)^{3}=\\frac{5}{16}$."} {"id": "22446", "image": [], "answer": "B", "solution": "null", "level": "高三", "question": "设随机变量 $\\xi-B(2, P), \\eta-B(4, p)$, 若 ${ }^{P(\\xi \\geq 1)=\\frac{5}{9}}$, 则 $P(\\eta \\geq 2)$ 的值为 ( )", "options": "A. $\\frac{32}{81}$\nB. $\\frac{11}{27}$\nC. $\\frac{65}{81}$\nD. $\\frac{16}{81}$", "subject": "计数", "analysis": "答案: B\n\n【解析】$P(\\xi \\geq 1)=\\frac{5}{9} \\Rightarrow C_{2}^{1} p(1-p)+C_{2}^{2} p^{2}=2 p-p^{2}=\\frac{5}{9} \\Rightarrow p=\\frac{1}{3}$,\n\n$P(\\eta \\geq 2)=C_{4}^{2} p^{2}(1-p)^{2}+C_{4}^{3} p^{3}(1-p)+C_{4}^{4} p^{4}=6 \\times \\frac{1}{9} \\times \\frac{4}{9}+4 \\times \\frac{1}{27} \\times \\frac{2}{3}+\\frac{1}{81}=\\frac{11}{27}$"} {"id": "22449", "image": [], "answer": "A", "solution": "null", "level": "高三", "question": "某班有 14 名学生数学成绩优秀, 如果从该班随机找出 5 名学生, 那么其中数学成绩优秀的学生数 $X \\sim B\\left(5, \\frac{1}{4}\\right)$, 则 $E(2 X+1)=(\\quad)$", "options": "A. $\\frac{5}{4}$\nB. $\\frac{5}{2}$\nC. 3\nD. $\\frac{7}{2}$", "subject": "计数", "analysis": "$,\n\n解得 ${ }^{p>\\frac{5}{2}}{ }_{\\text {或 }} p<\\frac{1}{2}$,\n\n又由 ${ }^{p \\in(0,1)}$, 可得 $p \\in\\left(0, \\frac{1}{2}\\right)$.\n\n7.答案: 6\n\n【解析】$\\because{ }^{\\xi \\sim B\\left(n, \\frac{1}{3}\\right)}, \\therefore E(\\xi)=\\frac{1}{3} n$,\n$\\therefore E(3 \\xi+2)=3 E(\\xi)+2=3 \\times \\frac{1}{3} n+2=8$\n\n$\\therefore n=6$, 故答案为 6"} {"id": "22451", "image": [], "answer": "B", "solution": "null", "level": "高三", "question": "已知某离散型随机变量 $X$ 的分布列为\n\n| $X$ | 0 | 1 | 2 | 3 |\n| :--- | :--- | :--- | :--- | :--- |\n| $P$ | $\\frac{8}{27}$ | $\\frac{4}{9}$ | $m$ | $\\frac{1}{27}$ |\n\n则 $X$ 的数学期望 $E(X)=(\\quad)$", "options": "A. $\\frac{2}{3}$\nB. 1\nC. $\\frac{3}{2}$\nD. 2", "subject": "计数", "analysis": "答案: B\n\n【解析】由题意可得: $\\frac{8}{27}+\\frac{4}{9}+m+\\frac{1}{27}=1$.\n\n可得 $m=\\frac{2}{9}$. $E(X)=0 \\times \\frac{8}{27}+1 \\times \\frac{4}{9}+2 \\times \\frac{2}{9}+3 \\times \\frac{1}{27}=1$.\n\n故选: B."} {"id": "22452", "image": [], "answer": "D", "solution": "null", "level": "高三", "question": "交强险是车主必须为机动车购买的险种, 若普通 6 座以下私家车投保交强险的基准保费为 $a$ 元, 在下一年续保时, 实行费率浮动机制, 保费与车辆发生道路交通事故出险的情况相联系, 最终保费 $==$ 基准保费 $\\times(1+$ 与道路交通事故相联系的浮动比率), 具体情况如表:\n\n| 交强险浮动因素和浮动费率比率表 | | 浮动比率 |\n| :--- | :--- | :--- |\n| 类别 | 浮动因素 | 下浮 $10 \\%$ |\n| $A_{1}$ | 上一个年度未发生有责任道路交通事故 | 下浮 $20 \\%$ |\n| $A_{2}$ | 上两个年度未发生有责任道路交通事故 | |\n| $A_{3}$ | 上三个及以上年度未发生有责任道路交通事故 | 下浮 $30 \\%$ |\n| $A_{4}$ | 上一个年度发生一次有责任不涉及死亡的道路交通事故 | $0 \\%$ |\n\n\n| $A_{3}$ | 上一个年度发生两次及两次以上有责任道路交通事故 | 上浮 $10 \\%$ |\n| :--- | :--- | :--- |\n| $A_{8}$ | 上一个年度发生有责任道路交通死亡事故 | 上浮 $30 \\%$ |\n\n为了解某一品牌普通 6 座以下私家车的投保情况, 随机抽取了 100 辆车龄已满三年的该品牌同型号私家车的下一年续保时的情况,统计如表:\n\n| 类型 | $A_{1}$ | $A_{2}$ | $A_{3}$ | $A_{4}$ | $A_{3}$ | $A_{4}$ |\n| :--- | :--- | :--- | :--- | :--- | :--- | :--- |\n| 数量 | 20 | 10 | 10 | 38 | 20 | 2 |\n\n若以这 100 辆该品牌的投保类型的频率代替一辆车投保类型的概率, 则随机抽取一辆该品牌车在第四年续保时的费用的期望为 ( )", "options": "A. $a$ 元\nB. $", "subject": "计数", "analysis": "答案: D\n\n【解析】设一辆该品牌车在第四年续保时的费用为 $X$,\n\n由题意可知: $X$ 的可能取值为 $"} {"id": "22453", "image": [], "answer": "B", "solution": "null", "level": "高三", "question": "已知随机变量 $X, Y$ 的分布列如下:\n\n| $X$ | 1 | 2 | 3 |\n| :---: | :---: | :---: | :---: |\n| $P$ | $m^{2}$ | $1-2 m^{2}$ | $m^{2}$ |\n| $Y$ | 1 | 2 | 3 |\n| $P$ | $m$ | $1-3 m^{3}$ | $3 m^{3}-m$ |\n\n则 $3 D(X), 2 E(Y)$ 的大小关系为( )", "options": "A. $3 D(X)>2 E(Y)$\nB. ${ }^{3 D(X)<2 E(Y)}$\nC. ${ }^{3 D(X)}=2 E(Y)$\nD. 无法确定", "subject": "计数", "analysis": "易知 $E(X)=m^{2}+2\\left(1-2 m^{2}\right)+3 m^{2}=2$ , 所以 $D(X)=m^{2}+m^{2}=2 m^{2} \\cdot E(Y)=m+2\\left(1-3 m^{3}\right)+3\\left(3 m^{3}-m\right)=3 m^{3}-2 m+2$,所以 $3 D(X)-2 E(Y)=6 m^{2}-6 m^{3}+4 m-4=-2\\left(3 m^{3}-3 m^{2}-2 m+2\right)=-2\\left(3 m^{2}-2\right)(m-1)$.易知 $\frac{1}{\\sqrt{3}} \\leq m \\leq \frac{1}{\\sqrt[3]{3}}$ ,所以 $m^{2} \\leq \frac{1}{3^{\frac{2}{3}}}<\frac{1}{3^{\frac{1}{2}}}<\frac{1}{2}=\frac{2}{2}$, 所以 $3 m^{2}-2<0$, 又 ${ }^{m-1<0}$, 所以 $3 D(X)-2 E(Y)<0$, 即 $3 D(X)<2 E(Y)$ ,故选 B."} {"id": "22457", "image": [], "answer": "B", "solution": "null", "level": "高三", "question": "有甲、乙两种水稻, 测得每种水稻各 10 株的分穕数据, 计算出样本均值相等, 方差分别为 $D\\left(X_{\\text {甲 }}\\right)=11, D\\left(X_{Z}\\right)=3.4$. 由此可以估计()", "options": "A.甲种水稻比乙种水稻分菜整齐\nB.乙种水稻比甲种水稻分菜整齐\nC.甲、乙两种水稻分省整齐程度相同\nD.甲、乙两种水稻分穕整齐程度不能比较", "subject": "计数", "analysis": "答案: $\\mathrm{B}$\n\n【解析】 $\\mathrm{Q} D\\left(X_{\\text {m }}\\right)>D\\left(X_{Z}\\right), \\therefore$ 乙种水稻比甲种水稻分葉整齐."} {"id": "22458", "image": [], "answer": "C", "solution": "null", "level": "高三", "question": "若随机变量 $X$ 的分布列如表所示, 且 ${ }^{E(X)=6.3}$, 则 $D(X)=(\\quad)$\n\n| $X$ | 4 | $a$ | 9 |\n| :---: | :---: | :---: | :---: |\n| $P$ | 0.5 | 0.1 | $b$ |", "options": "A. -", "subject": "计数", "analysis": "答案: C\n\n【解析】由题可得 $"} {"id": "22459", "image": [], "answer": "B", "solution": "null", "level": "高三", "question": "一个箱子中装有形状完全相同的 5 个白球和 $n\\left(n \\in \\mathbb{N}^{*}\\right)$ 个黑球. 现从中有放回地摸取 4 次,每次都是随机摸取 1 球, 设摸得白球的个数为 $X$, 若 $D(X)=1$, 则 $E(X)=(\\quad)$", "options": "A. 1\nB. 2\nC. 3\nD. 4", "subject": "计数", "analysis": "设每次随机摸取 1 球, 摸得白球的概率为 $P$, 由题意, 知 $X \\sim B(4, P) . \because$$D(X)=4 P(1-P)=1, \therefore P=\frac{1}{2}, \therefore E(X)=4 P=4 \times \frac{1}{2}=2$. 故选 $\\mathrm{B}$"} {"id": "22460", "image": [], "answer": "D", "solution": "null", "level": "高三", "question": "某公司 10 位员工的月工资(单位:元)为 $x_{1}, x_{2}, \\cdots, x_{10}$, 其均值和方差分别为 $\\bar{x}$ 和 $s^{2}$, 若从下月起每位员工的月工资增加 100 元,则这 10 位员工下月工资的均值和方差分别为( )", "options": "A. $\\bar{x}, 5^{2}+100^{2}$\nB. $\\bar{x}+100, s^{2}+100^{2}$\nC. $\\bar{x}, s^{2}$\nD. $\\bar{x}+100, s^{2}$", "subject": "计数", "analysis": "答案: D\n\n【解析】设增加工资后 10 位员工下月工资均值为 ${ }^{X^{\\prime}}$, 方差为 $5^{\\circledR}$,\n\n则平均数 $\\pi^{\\prime}=\\frac{1}{10}\\left[\\left(x_{1}+100\\right)+\\left(x_{2}+100\\right)+\\cdots+\\left(x_{10}+100\\right)\\right]$\n\n$=\\frac{1}{10}\\left(x_{1}+x_{1}+\\cdots+x_{10}\\right)+100=x+100 ;$\n\n$s^{a}=\\frac{1}{10}\\left[\\left(x_{1}+100-x^{\\prime}\\right)^{2}+\\left(x_{2}+100-x^{\\prime}\\right)^{2}+\\cdots+\\left(x_{10}+100-x^{\\prime}\\right)^{2}\\right]$\n\n$=\\frac{1}{10}\\left[\\left(x_{1}-x\\right)^{2}+\\left(x_{1}-x\\right)^{2}+\\cdots+\\left(x_{10}-x\\right)^{2}\\right]=s^{2}$. 故选 $D$."} {"id": "22461", "image": [], "answer": "B", "solution": "null", "level": "高三", "question": "已知 $X$ 为随机变量, 则下列说法错误的是 ( )", "options": "A. $P\\left(|X| \\leq \\frac{1}{2}\\right) \\leq P\\left(X^{2} \\leq \\frac{1}{2}\\right)$\nB. $D\\left(X^{2}\\right)=D\\left((1-X)^{2}\\right)$\nC. $D(X)=D(1-X)$\nD. $[E(X)]^{2} \\leq E\\left(X^{2}\\right)$", "subject": "计数", "analysis": "答案: B\n【解析】 对于 A $P\\left(|X| \\frac{1}{2}\\right)=P\\left(-\\frac{1}{2} \\leq X \\leq \\frac{1}{2}\\right), P\\left(X^{2} \\leq \\frac{1}{2}\\right)=P\\left(-\\frac{\\sqrt{2}}{2} \\leq X \\leq \\frac{\\sqrt{2}}{2}\\right)$ 故 A 正确.\n\n对于 B, 取特殊值 $P(X=1)=P(X=-1)=\\frac{1}{2}$, 则 $E\\left(X^{2}\\right)=\\frac{1}{2} \\times 1+\\frac{1}{2} \\times 1=1$ $D\\left(X^{2}\\right)=(1-1)^{2} \\times \\frac{1}{2}+(1-1)^{2} \\times \\frac{1}{2}=0 E\\left((1-X)^{2}\\right)=\\frac{1}{2} \\times 0+\\frac{1}{2} \\times 4=2$ $D\\left((1-X)^{2}\\right)=\\frac{1}{2} \\times(0-2)^{2}+\\frac{1}{2} \\times(4-2)^{2}=4$, 故 $D\\left((1-X)^{2}\\right) \\neq D\\left(X^{2}\\right)$ B 错误\n\n对于 C, $D(1-X)=(-1)^{2} D(X)=D(X)$, C 正确\n\n对于 $\\mathrm{D}, D(X)=E\\left(X^{2}\\right)-[E(X)]^{2} \\geq 0, \\mathrm{D}$ 正确"} {"id": "22462", "image": [], "answer": "B", "solution": "null", "level": "高三", "question": "已知随机变量 $X, Y$ 满足 $Y=2 X+1$, 且随机变量 ${ }^{X}$ 的分布列如下:\n\n| $X$ | 0 | 1 | 2 |\n| :---: | :---: | :---: | :---: |\n| $P$ | $\\frac{1}{6}$ | $\\frac{1}{3}$ | $a$ |\n\n则随机变量 ${ }^{Y}$ 的方差 ${ }^{D(Y)=}(\\quad)$", "options": "A. $\\frac{5}{9}$\nB. $\\frac{20}{9}$\nC. ${ }^{\\frac{4}{3}}$\nD. $\\frac{29}{9}$", "subject": "计数", "analysis": "答案: B\n\n【解析】 由分布列的性质, 得 $a=1-\\frac{1}{6}-\\frac{1}{3}=\\frac{1}{2}$ 所以 $E(X)=0 \\times \\frac{1}{6}+1 \\times \\frac{1}{3}+2 \\times \\frac{1}{2}=\\frac{4}{3}$, 所以\n\n$$\nD(X)=\\left(0-\\frac{4}{3}\\right)^{2} \\times \\frac{1}{6}+\\left(1-\\frac{4}{3}\\right)^{2} \\times \\frac{1}{3}+\\left(2-\\frac{4}{3}\\right)^{2} \\times \\frac{1}{2}=\\frac{5}{9} \\text {, 又 } Y=2 X+1 \\text {, 所以 } D(Y)=4 D(X)=\\frac{20}{9} \\text {. }\n$$"} {"id": "22465", "image": [], "answer": "B", "solution": "null", "level": "高三", "question": "已知某射击运动员每次击中目标的概率是 0.8 , 则该射击运动员射击 4 次至少击中 3 次的概率大约为( )", "options": "A.", "subject": "计数", "analysis": "答案: B\n\n【解析】 设运动员射击 4 次, 击中目标的次数为 $X$, 则 $P(X \\geq 3)=\\mathrm{C}_{4}^{4}"} {"id": "22466", "image": ["9795.jpg", "9796.jpg"], "answer": "A", "solution": "null", "level": "高三", "question": "已知随机变量 $X \\sim N(2,1)$, 其正态分布密度曲线如图所示, 则图中阴影部分的面积为( ) ( 附: 若随机变量 $\\xi \\sim N\\left(\\mu, \\sigma^{2}\\right)$, 则 $P(\\mu-\\sigma<\\xi \\leq \\mu+\\sigma)=0.6827, P(2 \\mu-2 \\sigma<\\xi \\leq 2 \\mu+2 \\sigma)=0.9545)$\n\n", "options": "A.", "subject": "计数", "analysis": "根据题意,随机变量 $X$ 满足正态分布 $N(2,1)$,得 $\\mu=2, \\sigma^{2}=1$, 则对称轴为 $x=2$, 且 $\\sigma=1$,根据正态分布密度曲线的性质,可得阴影部分的面积 $S=P 0<$$\\left.\\mu+2 \\sigma)-P\\left(\\mu-\\sigma0)$,试卷满分 150 分, 统计结果显示数学成绩优秀 (高于 120 分) 的人数占总人数的 $\\frac{1}{5}$, 则此次数学考试成绩在 90 分到 105 分之间的人数约为( )", "options": "A. 150\nB. 200\nC. 300\nD. 400", "subject": "计数", "analysis": "$$\\mathrm{Q} P(X<90)=P(X>120)=\frac{1}{5}, F(90 \text { 剟|X } 120)=1-\frac{2}{5}=\frac{3}{5}, \therefore P(90 \text { 剟|X } 105)=\frac{3}{10}$$"} {"id": "22469", "image": [], "answer": "B", "solution": "null", "level": "高三", "question": "已知随机变量 $X \\sim N(7,4)$, 且 $P(5", "options": "A. 7\nB. 12\nC. 17\nD. 34", "subject": "计数", "analysis": "答案: C\n\n解析: 第一次循环, $a=2, s=0 \\times 2+2=2, k=1$; 第二次循环, $a=2, s=2 \\times 2+2=6, k=2$; 第三次循\n\n环, $a=5, s=6 \\times 2+5=17, k=3>2$, 跳出循环. 故输出的 $5=17$. 故选 C."} {"id": "20820", "image": ["9485.jpg"], "answer": "B", "solution": "null", "level": "高三", "question": "相传,黄帝在制定乐律时,用 “三分损益”的方法得到不同的竹管,吹出不同的音调“三分损益”包含“三分损一”和“三分益一”,用现代数学的方法解释如下,“三分损一”是在原来的长度上减去三分之一,即变为原来的三分之二: “三分益一”是在原来的长度上增加分之一,即变为原来的三分之四,如图的程序是与“三分损益”结合的计算过程,若输入的 $x$ 的值为 1 ,则输出的 $x$ 的值为 ( )\n\n", "options": "A. $\\frac{16}{27}$\nB. $\\frac{32}{27}$\nC. $\\frac{8}{9}$\nD. $\\frac{3}{2}$", "subject": "计数", "analysis": "$x=1 \\Rightarrow x=\frac{2}{3}, i=2 \\Rightarrow x=\frac{8}{9}, i=3 \\Rightarrow x=\frac{32}{27}, i=4$, 结束循环,输出的结果为 $x=\frac{32}{27}$, 故选 B."} {"id": "20832", "image": [], "answer": "D", "solution": "null", "level": "高三", "question": "4 x+2.3$\nB. $\\hat{y}=2 x-2.4$\nC. $\\hat{y}=-2 x+9.5$\nD. $\\hat{y}=-0.3 x+4.4$\n3. 某单位为了践行“绿水青山就是金山银山”的理念,打算制定节能减排的目标. 他们调查了用电量 $y$ (单位:千瓦-时)与气温 $x$ (单位 $\\left.{ }^{\\circ} \\mathrm{C}\\right)$ 之间的关系,随机选取了 4 天的用电量与当天气温, 并制作了以下对照表:\n\n| $x$ (单位: ${ }^{\\circ} \\mathrm{C}$ ) | 17 | 14 | 10 | -1 |\n| :---: | :---: | :---: | :---: | :---: |\n| $y$ (单位:千瓦$\\cdot$时) | 24 | 34 | 38 | 64 |\n\n由表中数据得线性回归方程, $\\hat{y}=-2 x+\\hat{a}$ 则由此估计,当某天气温为 $2^{\\circ} \\mathrm{C}$ 时, 当天用电量约为()", "options": "A. 56 千瓦$\\cdot$时\nB. 62 千瓦$\\cdot$时\nC. 64 千瓦$\\cdot$时\nD. 68 千瓦$\\cdot$时", "subject": "计数", "analysis": "x-85.71$ 知, $k=0.85>0$, 所以 $y$ 与 $x$ 具有正的线性相关关系的, 故选项 A 正确;\n\n由回归直线方程恒过样本点的中心 $(\\bar{x}, \\bar{y})$ 知, 选项 B 正确;\n\n若该大学某女生身高增加 $1 \\mathrm{~cm}$, 则由 $\\$=0.85 x-85.71$ 知其体重约增加 $0.85 \\mathrm{~kg}$, 因此 C 选项正确;\n\n若该大学某女生身高为 $170 \\mathrm{~cm}$, 则可预测或估计其体重为 $58.79 \\mathrm{~kg}$, 并不一定为 $58.79 \\mathrm{~kg}$, 因此选项 D 不正确.故答案为 D.\n\n2.答案: A\n\n解析: 变量 ${ }^{x}$ 与 ${ }^{y}$ 正相关,可以排除 C,D;样本平均数代入可求这组样本数据的回归直线方程.\n\n$\\because$ 变量 $^{x}$ 与 $^{y}$ 正相关,\n\n$\\therefore$ 可以排除 C,D;\n\n样本平均数 $\\bar{x}=3, \\bar{y}=3.5$, 代入 $\\mathrm{A}$ 符合, $\\mathrm{B}$ 不符合."} {"id": "20834", "image": [], "answer": "C", "solution": "null", "level": "高三", "question": "为研究变量 $x$ 和 $y$ 的线性相关性,甲、乙二人分别作了研究,利用线性回归方法得到回归直线方程 $l_{1}$ 和 $l_{2}$,两人计算知 $\\bar{x}$ 相同, $\\bar{y}$ 也相同,下列正确的是 ( )", "options": "A. $l_{1}$ 与 $l_{2}$ 重合\nB. $l_{1}$ 与 $l_{2}$ 一定平行\nC. $l_{1}$ 与 $l_{2}$ 相交于点 $(\\bar{x}, \\bar{y})$\nD. 无法判断 $l_{1}$ 和 $l_{2}$ 是否相交", "subject": "计数", "analysis": "命题人考査线性回归方程恒过样本中心点 $(\bar{x}, \bar{y})$. 因为两人计算得 $\bar{x}$ 与 $\bar{y}_{\text {均相同, }}$故知选 $C$."} {"id": "20835", "image": [], "answer": "C", "solution": "null", "level": "高三", "question": "登山族为了了解某山高 $y(\\mathrm{~km})$ 与气温 $x^{\\left({ }^{\\circ} \\mathrm{C}\\right)}$ 之间的关系, 随机统计了 4 次山高与相应的气温, 并制作了对照表如下:\n\n| 气温 $\\left({ }^{\\circ} \\mathrm{C}\\right)$ | 18 | 13 | 10 | -1 |\n| :--- | :--- | :--- | :--- | :--- |\n| 山高 $(\\mathrm{km})$ | 24 | 34 | 38 | 64 |\n\n由表中数据得到线性回归方程 $\\hat{y}=-2 x+\\hat{a}(\\hat{a} \\in \\mathrm{R})$, 由此估计山高为 $72 \\mathrm{~km}$ 处气温的度数是( )", "options": "A. -10\nB.-8\nC.-6\nD. -4", "subject": "计数", "analysis": "由题意得 $\bar{x}=\frac{18+13+10-1}{4}=10 \bar{y}=\frac{24+34+38+64}{4}=40$, 代入线性回归方程 $\\hat{y}=-2 x+\\hat{a}$, 可得 $\\hat{a}=60, \therefore \\hat{y}=-2 x+60$.由 $\\hat{y}=-2 x+60=72$, 可得 $x=-6$."} {"id": "22441", "image": [], "answer": "B", "solution": "null", "level": "高三", "question": "某群体中的每位成员使用移动支付的概率都为 $p$, 各成员的支付方式相互 ${ }^{p=}$ 独立. 设 $X$ 为该群体的 10 位成员中使用移动支付的人数, $D(X)=2.4, P(X=4)", "options": "A. 流程图\nB. 结构图\nC. 程序框图\nD. 工序流程图", "subject": "图论", "analysis": "解析:解答: 本框图显然属于顺序结构的流程图.\n\n分析: 本题主要考查了流程图的概念, 解决问题的关键是根据流程图的定义进行分析即可.\n\n2 要描述一个工厂某种产品的生产步骤, 应用( )\nA.程序框图\nB.工序流程图\nC.知识结构图\nD.组织结构图\n\n答案: B\n\n解析: 解答: $\\because$ 工序流程图又称统筹图, 常见的一种画法是将一个工作或工程从头到尾依先后顺序分为若干道工序, 每一道工序用矩形框表示, 并在该矩形框内注明此工序的名称与代号. 两个相邻工序之间用流程线相连. 两相邻工序之间用流程线相连. 有时为合理安排工作进度,\n\n还在每道工序框上注明完成该工序所需时间. $\\therefore$ 要描述一个工厂某种产品的生产步骤, 应用工序流程图, 故选 B.\n\n分析: 本题主要考查了工序流程图 (即统筹图), 解决问题的关键是工序流程图是将一个工作或工程从头到尾依先后顺序分为若干道工序, 每一道工序用矩形框表示, 并在该矩形框内注明此工序的名称与代号. 两个相邻工序之间用流程线相连. 两相邻工序之间用流程线相连."} {"id": "20569", "image": ["9368.jpg"], "answer": "A", "solution": "null", "level": "高三", "question": "某医院医疗就诊流程如图所示, 则病人到医院就诊至少需要的步骤是 ( )\n\n", "options": "A. 6 个\nB. 7 个\nC. 8 个\nD. 9 个", "subject": "图论", "analysis": "解析: 解答: 根据该医院的就诊流程图知, 病人到医院就诊至少需要的步骤是: 凭医疗卡挂号 $\\rightarrow$ 相应珍区候诊 $\\rightarrow$ 门诊医师诊治 $\\rightarrow$ 划价、交费 $\\rightarrow$ 功能科室 (化验室、放射科、药房等) $\\rightarrow$离院;\n\n所以,就诊至少有 6 个步骤. 故选: A.\n\n分析: 本题主要考查了绘制简单实际问题的流程图, 解决问题的关键是根据该医院的就诊流程图, 得出病人到医院就诊至少需要的步骤是什么."} {"id": "20584", "image": ["9386.jpg"], "answer": "A", "solution": "null", "level": "高三", "question": "某成品的组装工序流程图如图所示, 箭头上的数字表示组装过程中所需要的时间 (小时),不同车间可同时工作, 同一车间不能同时做两种或两种以上的工作, 则组装该产品所需要的最短时间是 ( )\n\n", "options": "A. 11 小时\nB. 13 小时\nC. 15 小时\nD. 10 小时", "subject": "图论", "analysis": "解析: 解答: A 到 E 的时间, 为 $2+4=6$ 小时, 或 5 小时, A 经 C 到 D 的时间为 $3+4=7$ 小时,故 $A$ 到 $F$ 的最短时间就为 9 小时, 则 $A$ 经 $F$ 到 $G$ 的时间为 $9+2=11$ 小时, 即组装该产品所需要的最短时间是 11 小时, 故选: A.\n\n分析: 本题主要考查了设计程序框图解决实际问题, 解决问题的关键是由已知中的工序流程图, 我们可以计算出每条组装劳工序从开始到结束的时间, 进而根据从工程设计到结束试生产需要的最短时间为并联事件中的最大值, 串联事件的和, 进而得到答案."} {"id": "14950", "image": [], "answer": "B", "solution": "null", "level": "高一", "question": "下列各组对象不能构成集合的是()", "options": "A. 拥有手机的人\nB. 2019 年高考数学难题\nC. 所有有理数\nD. 小于 $\\pi$ 的正整数", "subject": "逻辑题", "analysis": "对 A,拥有手机的人属于确定的概念,故能构成集合.\n\n对 B, 2019 年高考数学难题界定不明确,不能构成集合\n\n对 C,任意给一个数都能判断是否为有理数,故能构成集合\n\n对 D,小于 $\\pi$ 的正整数分别为 $1,2,3$, 能够组成集合\n\n故答案为: B\n\n【分析】根据集合的确定性逐个判断即可."} {"id": "14974", "image": [], "answer": "D", "solution": "null", "level": "高一", "question": "已知集合 $M, N, P$ 为全集 $U$ 的子集, 且满足 $M \\subseteq P \\subseteq N$, 则下列结论不正确的是( )", "options": "A. $\\complement_{\\mathrm{UN} \\subseteq \\mathrm{C}_{\\mathrm{U}} \\mathrm{P}}$\nB. $\\complement_{N} \\mathrm{P} \\subseteq \\mathrm{C}_{\\mathrm{N}} \\mathrm{M}$\nC. ( $\\left.\\complement_{U} P\\right) \\cap M=\\varnothing$\nD. $\\left(\\complement_{U M}\\right) \\cap N=\\varnothing$", "subject": "逻辑题", "analysis": "因为 $\\mathrm{P} \\subseteq \\mathrm{N}$ ,所以 $\\mathrm{C}_{\\mathrm{U}} \\mathrm{N} \\subseteq \\mathrm{C}_{\\mathrm{U}} \\mathrm{P}, \\mathrm{A}$ 符合题意;\n\n因为 $\\mathrm{M} \\subseteq \\mathrm{P}$, 所以 $\\complement_{N} \\mathrm{P} \\subseteq \\mathrm{C}_{N} \\mathrm{M}, \\mathrm{B}$ 符合题意;\n\n因为 $M \\subseteq P$, 所以 $\\left(C_{U} P\\right) \\cap M=\\varnothing, C$ 符合题意;\n\n因为 $M \\subseteq N$, 所以 $\\left(\\complement_{U} M\\right) \\cap N \\neq \\varnothing$. $\\mathrm{D}$ 不正确.\n\n故答案为: D.\n【分析】利用集合的包含关系进行判断, 即可得结果."} {"id": "14983", "image": [], "answer": "D", "solution": "null", "level": "高一", "question": "若 $P=\\{x \\mid x<1\\}, Q=\\{x \\mid x>1\\}$, 则( )", "options": "A. $P \\subseteq Q$\nB. $Q \\subseteq P$\nC. $C_{R} P \\subseteq Q$\nD. $Q \\subseteq C_{R} P$", "subject": "逻辑题", "analysis": "$\\because P=\\{x \\mid x<1\\}, \\therefore \\mathrm{C}_{R} P=\\{x \\mid x \\geqslant 1\\}$,\n\n$\\because Q=\\{x \\mid x>1\\}, \\quad \\therefore Q \\subseteq \\mathrm{C}_{R} P$,\n\n故选: $D$.\n\n【分析】利用集合的补集的定义求出 $P$ 的补集; 利用子集的定义判断出 $Q \\subseteq \\mathrm{C}_{R} P$."} {"id": "15005", "image": ["7473.jpg"], "answer": "B", "solution": "null", "level": "高一", "question": "如图所示, $I$ 是全集, $A, B, C$ 是它的子集,则阴影部分所表示的集合是()\n\n", "options": "A. $(A \\cap B) \\cap C$\nB. $\\left(A \\cap C_{I} B\\right) \\cap C$\nC. $(A \\cap B) \\cap C_{I} C$\nD. $C_{I}(A \\cap B) \\cap C$", "subject": "逻辑题", "analysis": "解: 由图中阴影部分可知: 该部分表示的是集合 $A$ 与集合 $B$ 在全集 $I$ 中的补集的交集再与集合 $C$ 的交集运算,即用数学式子表示为: $\\left(A \\cap C_{I} B\\right) \\cap C$.\n\n故答案为: B\n\n【分析】根据图中阴影部分表示的是集合 $A$ 与集合 $B$ 在全集 $I$ 中的补集的交集再与集合 $C$ 的交集运算,用数学符号表示即可."} {"id": "16916", "image": ["8734.jpg"], "answer": "D", "solution": "null", "level": "高一", "question": "(2020 春・宣城期末)如图给出的是计算 $\\frac{1}{2}+\\frac{1}{4}+\\frac{1}{6}+\\cdots+\\frac{1}{20}$ 的值的一个流程图, 其中判断框内应填入的条件是 ( )\n\n", "options": "A. $i \\leqslant 21$\nB. $i \\leqslant 11$\nC. $i \\geqslant 21$\nD. $i \\geqslant 11$", "subject": "逻辑题", "analysis": "解: $\\because S=\\frac{1}{2}+\\frac{1}{4}+\\frac{1}{6}+\\cdots+\\frac{1}{20}$\n\n并由流程图中 $S=S+\\frac{1}{2 i}$\n\n故循环的初值为 1\n\n终值为 10、步长为 1\n\n故经过 10 次循环才能算出 $S=\\frac{1}{2}+\\frac{1}{4}+\\frac{1}{6}+\\cdots+\\frac{1}{20}$ 的值,\n\n故 $i \\leqslant 10$, 应不满足条件, 继续循环\n\n$\\therefore$ 当 $i \\geqslant 11$, 应满足条件, 退出循环\n\n填入 “ $i \\geqslant 11 ”$.\n\n故选: D."} {"id": "16922", "image": ["8739.jpg"], "answer": "C", "solution": "null", "level": "高一", "question": "(2018 秋-芜湖期末) 阅读如图所示的程序框图, 运行相应的程序, 若输出的 $S$ 为 $\\frac{11}{12}$, 则判断框中填写的内容可以是()\n\n", "options": "A. $n<5$\nB. $n<6$\nC. $n \\leqslant 6$\nD. $n<9$", "subject": "逻辑题", "analysis": "解:模拟执行程序框图, 可得\n\n$S=0, n=2$;\n\n满足条件, $S=\\frac{1}{2}, n=4$;\n\n满足条件, $S=\\frac{1}{2}+\\frac{1}{4}=\\frac{3}{4}, n=6$;\n\n满足条件, $S=\\frac{1}{2}+\\frac{1}{4}+\\frac{1}{6}=\\frac{11}{12}, n=8$;\n\n由题意, 此时应该不满足条件, 退出循环, 输出 $S$ 的值为 $\\frac{11}{12}$;\n\n故判断框中填写的内容可以是 $n \\leqslant 6$.\n\n故选: C."} {"id": "14951", "image": [], "answer": "D", "solution": "null", "level": "高一", "question": "若集合 $A=\\left\\{x \\mid m x^{2}+2 x+m=0, m \\in R\\right\\}$ 中有且只有一个元素, 则 $m$ 的取值集合是 ( )", "options": "A. $\\{1\\}$\nB. $\\{-1\\}$\nC. $\\{0,1\\}$\nD. $\\{-1,0,1\\}$", "subject": "代数", "analysis": "集合 A 中只含有一个元素,\n\n所以方程 $m x^{2}+2 x+m=0$ 为一次方程或二次方程有两个相等的实数根,\n\n因此 $m=0$ 或 $\\left\\{\\begin{array}{l}m \\neq 0 \\\\ \\Delta=0\\end{array}, \\therefore m=0,1,-1\\right.$,\n\n故 $\\mathrm{m}$ 的取值集合是 $\\{-1,0,1\\}$.\n\n故答案为:D.\n\n【分析】集合 $\\mathrm{A}$ 中只含有一个元素, 所以方程 $m x^{2}+2 x+m=0$ 为一次方程或二次方程有两个相等的实数根, 得到 $\\mathrm{m}$ 的值."} {"id": "14952", "image": ["7467.jpg"], "answer": "C", "solution": "null", "level": "高一", "question": "有下列说法:\n\n(1) 0 与 $\\{0\\}_{\\text {表示同一个集合; }}$\n\n(2)由 1, 2,3 组成的集合可表示为 $\\{1,2,3\\}$ 或 $\\{3,2,1\\}$;\n\n(3) 方程 $(x+1)(x-2)^{2}=0$ 的所有解的集合可表示为 $\\{-1,2,2\\}$;\n\n(4)集合 $\\{x \\mid-3\n\n(2)中, 根据集合的表示方法, 可得由 $1,2,3$ 组成的集合可表示为 $\\{1,2,3\\}$ 或 $\\{3,2,1\\}$ ,所以(2)是正确的; (3) 中, 根据集合的表示方法, 得方程 $(x+1)(x-2)^{2}=0$ 的所有解的集合可表示为 $\\{-1,2\\}$, 所以(3)不正确;(4)中,集合 $\\{x \\mid-30, \\mathrm{y}>0, \\mathrm{~m}=3$,\n\n$\\mathrm{x}>0, \\mathrm{y}<0, \\mathrm{~m}=-1$,\n\n$\\mathrm{x}<0, \\mathrm{y}>0, \\mathrm{~m}=-1$,\n$\\mathrm{x}<0, \\mathrm{y}<0, \\mathrm{~m}=-1$,\n\n$\\therefore \\mathrm{M}=\\{-1,3\\}$.\n\n故答案为: C.\n\n【分析】先分四种情况讨论 $\\mathrm{x}$ 与 $\\mathrm{y}$, 得到 $\\mathrm{m}$ 的值, 再利用集合的互异性, 即可求出集合 $\\mathrm{M}$."} {"id": "14971", "image": [], "answer": "C", "solution": "null", "level": "高一", "question": "已知集合 $M=\\left\\{2, a^{2}-3 a+5,5\\right\\}$, 集合 $N=\\left\\{1, a^{2}-6 a+10,3\\right\\}$, 且 $M \\cap N=\\{2,3\\}$, 则 $a$ 的值是( )", "options": "A. 1 或 2\nB. 2 或 4\nC. 2\nD. 1", "subject": "代数", "analysis": "因为 $M \\cap N=\\{2,3\\}$, 所以有 $2 \\in N, 3 \\in M$, 所以 $\\left\\{\\begin{array}{l}a^{2}-3 a+5=3 \\\\ a^{2}-6 a+10=2,\\end{array}\\right.$ 解得 $a=2$, 故答案为: $\\mathrm{C}$\n\n【分析】利用交集的运算法则结合已知条件,再利用解一元二次方程解集的公式和元素的确定性、互异性和无序性, 从而求出 $\\mathrm{a}$ 的值。"} {"id": "14972", "image": [], "answer": "D", "solution": "null", "level": "高一", "question": "已知集合 $A=\\left\\{x \\mid x^{2}-3 x-10 \\leq 0\\right\\}, B=\\{x \\mid m+1 \\leq x \\leq 2 m-1\\}$, 若 $B \\subseteq A$, 则实数 $m$ 的取值范围是 ( )", "options": "A. $-2 \\leq m \\leq 3$\nB. $-3 \\leq m \\leq 2$\nC. $m \\geq 2$\nD. $m \\leq 3$", "subject": "代数", "analysis": "$A=\\left\\{x \\mid x^{2}-3 x-10 \\leq 0\\right\\}=\\{x \\mid-2 \\leq x \\leq 5\\}$\n\n$B=\\{x \\mid m+1 \\leq x \\leq 2 m-1\\}$ 且 $B \\subseteq A ,$\n\n当 $B=\\varnothing$ 时: $m+1>2 m-1, m<2$\n\n当 $B \\neq \\varnothing$ 时: $m+1 \\leq 2 m-1, m \\geq 2$ 且 $\\left\\{\\begin{array}{l}2 m-1 \\leq 5 \\\\ m+1 \\geq-2\\end{array}-3 \\leq m \\leq 3\\right.$ 即 $2 \\leq m \\leq 3$\n\n综上所述: $m \\leq 3$\n\n故答案为: D\n\n【分析】通过解不等式求出集合 A, 对 B 的情况分类讨论, 结合集合间的关系, 解不等式组, 即可求出实数 $\\mathrm{m}$ 的取值范围."} {"id": "14980", "image": [], "answer": "B", "solution": "null", "level": "高一", "question": "设集合 $A=\\{x|x-a|=1\\}, B=\\{1,0, b\\}(b>0)$, 若 $A \\subseteq B$, 则对应的实数 $(a, b)$ 有 $\\quad$", "options": "A. 1 对\nB. 2 对\nC. 3 对\nD. 4 对", "subject": "代数", "analysis": "因为 $A=\\{x|x-a|=1\\}=\\{a-1 a+1\\}$, 若 $A \\subseteq B$, 而 $B=\\{1,0, b\\}(b>0)$,\n\n$(a+1)-(a-1)=2$, 所以, 只能 $\\left\\{\\begin{array}{l}a-1=0 \\\\ a+1=b\\end{array}\\right.$ 或 $\\left\\{\\begin{array}{l}a-1=1 \\\\ a+1=b\\end{array}\\right.$, 解得 $\\left\\{\\begin{array}{l}a=1 \\\\ b=2 \\text { 或 }\\end{array}\\left\\{\\begin{array}{l}a=2 \\\\ b=3\\end{array}\\right.\\right.$.\n\n故答案为: B.\n\n【分析】先解出集合 $A$, 再根据集合的包含关系, 即可确定."} {"id": "14985", "image": [], "answer": "D", "solution": "null", "level": "高一", "question": "已知集合 $A=\\{-12\\}, B=\\{x \\mid a x=1\\}$, 若 $B \\subseteq A$, 则由实数 $a$ 的所有可能的取值组成的集合为", "options": "A. $\\left\\{1, \\frac{1}{2}\\right\\}$\nB. $\\left\\{-1, \\frac{1}{2}\\right\\}$\nC. $\\left\\{0,1, \\frac{1}{2}\\right\\}$\nD. $\\left\\{-1,0, \\frac{1}{2}\\right\\}$", "subject": "代数", "analysis": "因为集合 $A=\\{-12\\}, B=\\{x \\mid a x=1\\}, B \\subseteq A$,\n若 $B$ 为空集,则方程 $a x=1$ 无解,解得 $a=0$;\n\n若 $B$ 不为空集,则 $a \\neq 0$; 由 $a x=1$ 解得 $x=\\frac{1}{a}$,\n\n所以 $\\frac{1}{a}=-1$ 或 $\\frac{1}{a}=2$, 解得 $a=-1$ 或 $a=\\frac{1}{2}$,\n\n综上, 由实数 $a$ 的所有可能的取值组成的集合为 $\\left\\{-1,0, \\frac{1}{2}\\right\\}$.\n\n故答案为: D\n\n【分析】分 $B$ 为空集和 $B$ 不为空集两种情况讨论, 分别求出 $a$ 的范围, 即可得出结果."} {"id": "14986", "image": [], "answer": "A", "solution": "null", "level": "高一", "question": "设 $a, b \\in R$, 集合 $A=\\{1 a+b, a\\}, B=\\left\\{0, \\frac{b}{a}, b\\right\\}$, 若 $A=B$, 则 $b-a=$ ( )", "options": "A. 2\nB. -1\nC. 1\nD. -2", "subject": "代数", "analysis": "由已知, $a \\neq 0$, 故 $a+b=0$, 则 $\\frac{b}{a}=-1$, 所以 $a=-1, b=1$.\n\n故答案为: A\n\n【分析】由已知集合相等 $A=B$ 列式, 得到 $a=-1, b=1$, 即可求出 $\\mathrm{b}-\\mathrm{a}$ 的值."} {"id": "14988", "image": [], "answer": "A", "solution": "null", "level": "高一", "question": "若集合 $A=\\left\\{x \\mid(k+2) x^{2}+2 k x+1=0\\right\\}$ 有且仅有 1 个元素, 则实数 $k$ 的值是()", "options": "A. $\\pm 2$ 或- 1\nB. -2 或 -1\nC. 2 或- 1\nD. -2", "subject": "代数", "analysis": "当 $k+2=0$, 解得 $k=-2, \\therefore-4 k+1=0$, 得 $x=\\frac{1}{4}$, 符合题意,\n\n当 $k+2 \\neq 0$ 时, $(2 k)^{2}-4(k+2) \\times 1=0$, 解得 $k=2$ 或 $k=-1$,\n\n故答案为: A.\n\n【分析】由已知集合 $\\mathrm{A}$ 有且仅有 1 个元素, 分两种情况讨论, 当 $k+2=0$, 可得 $x=\\frac{1}{4}$ 符合题意;\n\n当 $k+2 \\neq 0$ 时, 解得 $k=2$ 或 $k=-1$ 符合题意."} {"id": "14991", "image": [], "answer": "D", "solution": "null", "level": "高一", "question": "中国古代重要的数学著作《孙子算经》下卷有题:\n\n今有物, 不知其数, 三三数之, 剩二; 五五数之, 剩三; 七七数之, 剩二.问: 物几何? 现有如下表\n\n示: 已知 $A=\\left\\{x \\mid x=3 n+2, n \\in N^{*}\\right\\}, B=\\left\\{x \\mid x=5 n+3, n \\in N^{*}\\right\\}, C=\\left\\{x \\mid x=7 n+2, n \\in N^{*}\\right\\}$, 若 $x \\in A \\cap B \\cap C$, 则整数 $x$ 的最小值为 ( )", "options": "A. 128\nB. 127\nC. 37\nD. 23", "subject": "代数", "analysis": "解: 因为求整数 $x$ 的最小值, 所以从最小的数开始带入检验即可:\n\n当 $x=23$ 时, $23=3 \\times 7+2$, 故 $x \\in A ; 23=5 \\times 4+3$, 故 $x \\in B ; 23=7 \\times 3+2$, 故 $x \\in C$,\n\n$\\therefore 23 \\in A \\cap B \\cap C$,\n\n故答案为: D.\n\n【分析】将选项中的数字带入集合 $A, B, C$ 检验是否为 $A, B, C$ 的元素, 找出最小的一个即可."} {"id": "15000", "image": [], "answer": "C", "solution": "null", "level": "高一", "question": "设集合 $A=\\{x \\mid 1 \\leq x \\leq 3\\} , B=\\{x \\mid 2\n\n所以 $f(x+1)=-(x+1)((x+1)-1]=-x(x+1)$,\n\n又 $2 f(x+1)=f(x)$,\n\n所以当 $x \\in(-1,0]_{\\text {时, }} f(x)=2 f(x+1)=-2 x(x+1)$,\n\n\n\n做出示意图如下图所示:\n\n\n\n要使 $f(x) \\leq \\frac{8}{9}$, 则需 $x \\geq x_{1}$,\n\n而由 $-4(x+1)(x+2)=\\frac{8}{9}$ ,\n\n解得 $x_{1}=-\\frac{4}{3}$, 所以 $m \\geq-\\frac{4}{3}$,\n\n故选: D.\n\n\n$x \\geq x_{1}$, 而由 $-(x+1)(x+2)=\\frac{8}{9}$ 可解得 $x_{1}=-\\frac{4}{3}$, 从而得出 $m$ 的范围."} {"id": "15012", "image": [], "answer": "A", "solution": "null", "level": "高一", "question": "已知函数 $f(x)$ 对任意 $x, y \\in R$ 都有 $f(x+y)=f(x)+f(y)$ 成立, 且 $f(2)=4$, 则 $f(-1)=$", "options": "A. -2\nB. 1\nC. $\\frac{1}{2}$\nD. 2", "subject": "代数", "analysis": "解: 令 $x=y=0$, 则有 $f(0+0)=f(0)+f(0)$, 即 $f(0)=2 f(0)$, 得 $f(0)=0$;\n\n令 $x=y=1$, 则有 $f(1)+f(1)=f(1+1)=f(2)=4$, 即 $f(1)=2$;\n\n令 $x=-1 y=1$, 则有 $f(-1)+f(1)=f\\lfloor(-1)+1\\rfloor=f(0)=0$;\n\n$\\therefore f(-1)=-2$.\n\n故答案为: A.\n【分析】 分别令 $x=y=0, x=y=1, x=-1, y=1$, 即可得解."} {"id": "15018", "image": ["7479.jpg"], "answer": "C", "solution": "null", "level": "高一", "question": "下列四组函数中表示同一函数的是()", "options": "A. $f(x)=x, \\quad g(x)=(\\sqrt{x})^{2}$\nB. $f(x)=x^{2}, g(x)=(x+1)^{2}$\nC. $f(x)=\\sqrt{x^{2}}, \\quad g(x)=|x|$\nD. $f(x)=0, \\quad g(x)=\\sqrt{x-1}+\\sqrt{1-x}$\n\n\nA. 1\nB. 2\nC. -3\nD. $\\frac{1}{2}$", "subject": "代数", "analysis": "由于函数 $f(x)=x$ 的定义域为 $R$, 而函数 $g(x)=(\\sqrt{x})^{2}$ 的定义域为 $\\{x \\mid x \\geq 0\\}$ ,这 2 个函数的定义域不同, 故不是同一个函数, 故排除 $\\mathrm{A}$.\n\n由于函数 $f(x)=x^{2}, g(x)=(x+1)^{2}$ 的定义域均为 $R$, 但这 2 个函数的对应关系不同,故不是同一个函数, 故排除 B.\n\n由于函数 $f(x)=\\sqrt{x^{2}}$ 的定义域与函数 $g(x)=|x|$ 的定义域, 对应关系, 值域完全相同,\n\n故这 2 个函数是同一个函数.\n\n由于函数 $f(x)=0$ 的定义域为 $R$, 函数 $g(x)=\\sqrt{x-1}+\\sqrt{1-x}$ 的定义域为 $\\{x \\mid x=1\\}$ ,\n\n定义域不同, 故不是同一个函数. 故排除 D\n\n故答案为: C.\n\n【分析】根据函数的定义, 确定两函数的定义域和对应关系是否相同即可确定两函数是否为同一个."} {"id": "15023", "image": [], "answer": "C", "solution": "null", "level": "高一", "question": "已知函数 $f(x)=\\left\\{\\begin{array}{l}3^{-x}+1(x \\leq 0) \\\\ x^{a}+2(x>0),\\end{array}\\right.$ 若 $f(f(-1))=18$, 那么实数 $a$ 的值是 ( )", "options": "A. 4\n.B. 1\nC. 2\nD. 3", "subject": "代数", "analysis": "$f(-1)=4, f(f(-1))=18$ 变成 $f(4)=18$, 即 $4^{a}+2=18$, 解之得: $a=2$.\n\n故答案为: C.\n\n【分析】先求出 $f(-1)=4, f(f(-1))=18$ 变成 $f(4)=18$, 可得到 $4^{a}+2=18$, 解方程即可得解."} {"id": "15024", "image": [], "answer": "B", "solution": "null", "level": "高一", "question": "若 $f(x)_{\\text {满足关系式 }} f(x)+2 f\\left(\\frac{1}{x}\\right)=3 x$, 则 $f(2)$ 的值为 ( )", "options": "A. 1\nB. -1\nC. $-\\frac{3}{2}$\nD. $\\frac{3}{2}$", "subject": "代数", "analysis": "$\\because \\mathrm{f}(\\mathrm{x})$ 满足关系式 $\\mathrm{f}(\\mathrm{x})+2 \\mathrm{f}\\left(\\frac{1}{x}\\right)=3 \\mathrm{x}$,\n\n$\\therefore\\left\\{\\begin{array}{l}f(2)+2 f\\left(\\frac{1}{2}\\right)=6,(1) \\\\ f\\left(\\frac{1}{2}\\right)+2 f(2)=\\frac{3}{2},(2),\\end{array}\\right.$\n\n(1) - (2) $\\times 2$ 得 $-3 f(2)=3$,\n\n$\\therefore \\mathrm{f}(2)=-1$,\n\n故答案为: B.\n\n【分析】由已知条件得 $\\left\\{\\begin{array}{l}f(2)+2 f\\left(\\frac{1}{2}\\right)=6 \\text {, (1) } \\\\ f\\left(\\frac{1}{2}\\right)+2 f(2)=\\frac{3}{2} \\text {, (2), 由此能求出 } \\mathrm{f} \\text { (2) 的值. }\\end{array}\\right.$"} {"id": "15027", "image": [], "answer": "C", "solution": "null", "level": "高一", "question": "函数 $f(x)=\\left\\{\\begin{array}{l}2 x-x^{2}(0 \\leq x \\leq 3) \\\\ x^{2}+6 x(-2 \\leq x<0) \\text { 的值域是 ( ) }\\end{array}\\right.$", "options": "A. $R$\nB. $[-9,+\\infty)$\nC. $[-8,1]$\nD. $[-9,1]$", "subject": "代数", "analysis": "$\\because$ 函数 $y=2 x-x^{2}, 0 \\leq x \\leq 3$ 的对称轴为 $x=1, f(x)$ 最大值为 $f(1)=1$, 最小值为 $f(3)=-3$,值 域 $B=\\lfloor-3,1\\rfloor$, 函数 $y=x^{2}+6 x,-2 \\leq x \\leq 0$ 的值域 $C=\\lfloor-8,0\\rfloor$, 故函数 $f(x)=\\left\\{\\begin{array}{l}2 x-x^{2}(0 \\leq x \\leq 3) \\\\ x^{2}+6 x(-2 \\leq \\leq 0) \\text { 的值域是 } B \\cup C=\\lfloor-8,1\\rfloor 。\\end{array}\\right.$\n\n故答案为: C.\n\n【分析】利用分段函数的解析式画出分段函数图象,再利用分段函数图象的对称性结合二次函数求最值的方法, 求出分段函数的值域。"} {"id": "15029", "image": ["7485.jpg", "7486.jpg", "7487.jpg", "7488.jpg"], "answer": "B", "solution": "null", "level": "高一", "question": "下列四个图象中, 是函数图象的是()\n\n\n\n(1)\n\n\n\n(2)\n\n\n\n(3)\n\n\n\n(4)", "options": "A. (1)\nB. (1)(3)(4)\nC. (1)(2)(3)\nD. (3)(4)", "subject": "代数", "analysis": "由函数的定义知, 对于定义域中的每一个自变量 $x$, 只能有唯一的 $\\mathrm{y}$ 与之对应, 故(2)不是函数,\n\n(1)(3)(4)是函数.\n\n故答案为:\nB.\n\n【分析】函数定义中要求: 1.两个函数都是非空集合; $2 . \\mathrm{A}$ 中的每个元素在 B 中都有与之对应的元素; 3.对应形式为“一对一”或“多对一”,但不能是“一对多”(一个 $x$ 对应多个 $y$; 只有满足了这几个特点的对应关系才是函数关系. 本题解题的关键是观察: 图象对应的是否是函数; 定义域与值域是否正确."} {"id": "15039", "image": [], "answer": "D", "solution": "null", "level": "高一", "question": "已知函数 $f(x)$ 满足 $2 f(x)+f(-x)=3 x+2$, 则 $f(2)=(\\quad)$", "options": "A. $-\\frac{16}{3}$\nB. $-\\frac{20}{3}$\nC. $\\frac{16}{3}$\nD. $\\frac{20}{3}$", "subject": "代数", "analysis": "根据题意得: $2 f(2)+f(-2)=3 \\times 2+2=8_{(1)}$,\n\n令 $x=-2$ 可得: $2 f(-2)+f(2)=3 \\times(-2)+2=-4$ (2),\n\n(1) $\\times 2-$ (2)联立可得 $f(2)=\\frac{20}{3}$,\n\n故答案为:\n\n【分析】由已知利用赋值法, 分别令 $\\mathrm{x}=2$ 和 $\\mathrm{x}=-2$ 列式, 联立即可求出 $f(2)$ 的值."} {"id": "15040", "image": [], "answer": "B", "solution": "null", "level": "高一", "question": "已知 $f(x)$ 是一次函数, 且 $f(x-1)=3 x-5$, 则 $f(x)$ 的解析式为 $(\\quad)$", "options": "A. $f(x)=3 x+2$\nB. $f(x)=3 x-2$\nC. $f(x)=2 x+3$\nD. $f(x)=2 x-3$", "subject": "代数", "analysis": "设 $f(x)=k x+b, \\quad(k \\neq 0)$\n\n$$\n\\therefore f(x-1)=k(x-1)+b=3 x-5 \\text {, }\n$$\n\n即 $k x-k+b=3 x-5$,\n\n所以 $\\left\\{\\begin{array}{l}k=3 \\\\ b-k=-5,\\end{array}\\right.$ 解得 $k=3, b=-2$,\n\n$\\therefore f(x)=3 x-2$,\n\n故答案为: B.\n\n【分析】设 $f(x)=k x+b, \\quad(k \\neq 0)$, 利用 $f(x-1)=3 x-5$ 两边恒等求出 $k$ 即可得结果."} {"id": "15041", "image": [], "answer": "B", "solution": "null", "level": "高一", "question": "已知函数 $\\mathrm{f}(\\mathrm{x}+2)=\\mathrm{x}^{2}$, 则 $\\mathrm{f}(\\mathrm{x})$ 等于 ( )", "options": "A. $x^{2}+2$\nB. $x^{2}-4 x+4$\nC. $x^{2}-2$\nD. $x^{2}+4 x+4$", "subject": "代数", "analysis": "令 $x+2=t \\therefore x=t-2 \\therefore f(t)=(t-2)^{2} \\therefore f(x)=(x-2)^{2}=x^{2}-4 x+4$,\n\n故答案为: $\\mathrm{B}$\nB.\n【分析】利用换元法求函数解析式."} {"id": "15042", "image": [], "answer": "D", "solution": "null", "level": "高一", "question": "给出函数 $\\mathrm{f}(\\mathrm{x}), \\mathrm{g}(\\mathrm{x})$ 如表, 则 $f[g(2)]=(\\quad)$\n\n| $x$ | 1 | 2 | 3 | 4 |\n| :--- | :--- | :--- | :--- | :--- |\n| $f(x)$ | 4 | 3 | 2 | 1 |\n\n\n| $\\mathrm{X}$ | 1 | 2 | 3 | 4 |\n| :--- | :--- | :--- | :--- | :--- |\n| $\\mathrm{g}(\\mathrm{x})$ | 1 | 1 | 3 | 3 |", "options": "A. 1\nB. 2\nC. 3\nD. 4", "subject": "代数", "analysis": "查表可知, $g(2)=1$\n\n$\\therefore f[g(2)]=f(1)=4$\n\n故答案为: D。\n\n【分析】根据题意, 查表格得出 $g(2)$ 的值, 再将结果代入 $f(x)$ 中, 再次查表格即可得出 $f[g(2)]$ 的值。"} {"id": "15043", "image": [], "answer": "C", "solution": "null", "level": "高一", "question": "已知函数 $f\\left(x+\\frac{1}{x}\\right)=x^{2}+\\frac{1}{x^{2}}+3$, 则 $f(3)=( )$", "options": "A. 8\nB. 9\nC. 10\nD. 11", "subject": "代数", "analysis": "由题意得 $f\\left(x+\\frac{1}{x}\\right)=x^{2}+\\frac{1}{x^{2}}+3=\\left(x+\\frac{1}{x}\\right)^{2}+1$,\n\n$\\therefore f(x)=x^{2}+1(x \\leq-2$ 或 $x \\geq 2)$,\n\n$\\therefore f(3)=3^{2}+1=10$.\n\n故答案为: C.\n\n【分析】先用换元法求得 $\\mathrm{f}(\\mathrm{x})$ 的解析式, 在代入求得 $\\mathrm{f}(3)$ 的值。"} {"id": "15047", "image": ["7498.jpg"], "answer": "A", "solution": "null", "level": "高一", "question": "下列四个函数中, 在 $(0,+\\infty)$ 上是增函数的是 $(\\quad)$", "options": "A. $f(x)=\\frac{x}{x+1}$\nB. $f(x)=x^{2}-3 x$\nC. $f(x)=3-x$\nD. $f(x)=-|x|$\n\n\nA. -3\nB. 3\nC. -6\nD. 6", "subject": "代数", "analysis": "对于 $\\mathrm{A}$ 选项, 函数 $f(x)=\\frac{x}{x+1}=\\frac{x+1-1}{x+1}=1-\\frac{1}{x+1}$, 其在 $(0,+\\infty)$ 上递增, 符合题意.\n\n对于 $\\mathrm{B}$ 选项, 函数 $f(x)=x^{2}-3 x$ 的对称轴为 $x=\\frac{3}{2}$, 其在 $\\left(0, \\frac{3}{2}\\right)$ 上递减, 不符合题意.\n\n对于 C 选项,函数 $f(x)=3-x_{\\text {在 }}(0,+\\infty)$ 上递减, 不符合题意.\n\n对于 $\\mathrm{D}$ 选项, 函数 $f(x)=-|x|$ 在 $(0,+\\infty)$ 上递减, 不符合题意.\n\n故答案为: A\n\n【分析】对选项逐一分析函数的单调性, 由此判断出正确选项."} {"id": "15048", "image": [], "answer": "A", "solution": "null", "level": "高一", "question": "已知函数 $f(x)=\\frac{2 x+1}{x-1}$, 其定义域是 $\\lfloor-8,-4)$, 则下列说法正确的是()", "options": "A. $f(x)_{\\text {有最大值 }} \\frac{5}{3}$, 无最小值\nB. $f(x)_{\\text {有最大值 }} \\frac{5}{3}$, 最小值 $\\frac{7}{5}$\nC. $f(x)_{\\text {有最大值 }} \\frac{7}{5}$, 无最小值\nD. $f(x)_{\\text {无最大值, 最小值 } \\frac{7}{5}}$", "subject": "代数", "analysis": "因为函数 $f(x)=\\frac{2 x+1}{x-1}=\\frac{2(x-1)+3}{x-1}=2+\\frac{3}{x-1}$,\n\n所以 $f(x)$ 在 $\\lfloor-8,-4)$ 上单调递减,\n\n则 $f(x)$ 在 $x=-8$ 处取得最大值, 最大值为 $\\frac{5}{3}$,\n\n在 $x=-4$ 取不到函数值, 即最小值取不到.\n\n故答案为: A.\n\n【分析】先化简函数 $f(x)$, 再根据反比例函数单调性确定函数最值取法"} {"id": "15049", "image": [], "answer": "D", "solution": "null", "level": "高一", "question": "已知 $f(4-x)=f(2+x), f(x)$ 在 $(-\\infty, 3]$ 上单调递减, $f(0)=0$, 则 $f(2-3 x)>0$ 的解集是 ( )", "options": "A. $\\left(-\\infty, \\frac{2}{3}\\right) \\cup(2,+\\infty)$\nB. $\\left(\\frac{2}{3}, 2\\right)$\nC. $\\left(-\\frac{2}{3}, \\frac{2}{3}\\right)$\nD. $\\left(-\\infty,-\\frac{4}{3}\\right) \\cup\\left(\\frac{2}{3},+\\infty\\right)$\n\n$f(x)=\\left\\{\\begin{array}{l}(3 a-1) x+4 a,(x<1) \\\\ -a x,(x \\geq 1) \\quad \\text { 是定义在 }(-\\infty,+\\infty) \\text { 上的减函数, 则 } a \\text { 的范围是 }()\\end{array}\\right.$\nA. $\\left[0, \\frac{1}{3}\\right]$\nB. $\\left[\\frac{1}{8}, \\frac{1}{3}\\right)$\nC. $\\left(0, \\frac{1}{3}\\right)$\n.D. $\\left(-\\infty, \\frac{1}{3}\\right]$", "subject": "代数", "analysis": "$\\because f(4-x)=f(2+x)$,\n\n$\\therefore f(x)$ 的图象关于直线 $x=3$ 对称,\n\n$\\because f(0)=0, \\therefore f(6)=0$,\n\n$\\because f(x)$ 在 $(-\\infty, 3]$ 上单调递减,\n\n$\\therefore f(x)$ 在 $\\lfloor 3,+\\infty)$ 上单调递增,\n\n$\\therefore f(2-3 x)>0$, 可得 $2-3 x<0$ 或 $2-3 x>6$,\n\n解得: $x>\\frac{2}{3}$ 或 $x<-\\frac{4}{3}$.\n\n故选:D.\n\n【分析】由 $f(4-x)=f(2+x)$ 可知 $f(x)$ 的图象关于直线 $x=3$ 对称, 由 $f(0)=0$ 可知 $f(6)=0$,则 $f(2-3 x)>0$ 可转化为 $2-3 x<0$ 或 $2-3 x>6$, 即可求得结果."} {"id": "15059", "image": ["7501.jpg"], "answer": "D", "solution": "null", "level": "高一", "question": "函数 $f(x)=1+\\frac{1}{x-1}(\\quad)$", "options": "A. 在 $(-1,+\\infty)$ 上单调递增\nB. 在 $(1,+\\infty)$ 上单调递增\nC. 在 $(-1,+\\infty)$ 上单调递减\nD. 在 $(1,+\\infty)$ 上单调递减\n\n\nA. 1\nB. $\\frac{7}{2}$\nC. $-\\frac{7}{2}$\nD. -1", "subject": "代数", "analysis": "函数 $f(x)=1+\\frac{1}{x-1}$ 的图象是由 $y=\\frac{1}{x}$ 的图象向右平移一个单位, 再向上一个单位而得到,\n\n所以函数 $f(x)=1+\\frac{1}{x-1}$ 在 $(-\\infty, 1),(1,+\\infty)$ 上单调递减,\n\n故答案为: D\n\n【分析】根据分式函数的性质即可得到结论."} {"id": "15060", "image": [], "answer": "B", "solution": "null", "level": "高一", "question": "已知 $f(x)$ 是定义在 $\\lfloor-1,1\\rfloor$ 上的增函数, 且 $f(x-1)0 \\\\ \\frac{1}{4 a}>1\\end{array}\\right.$ 或 $\\left\\{\\begin{array}{l}a<0 \\\\ \\frac{1}{4 a}>1\\end{array}\\right.$, 解得: $00$,\n\n即 $f\\left(x_{1}\\right)>f\\left(x_{2}\\right)$, 函数 $f(x)$ 单调递减;\n\n所以 $f(x)_{\\text {max }}=f(-\\sqrt{2})=2-2 \\sqrt{2}$;\n\n(2)当 $x \\geq 0$ 时, $f(x)=-x^{2}-1$ 单调递减,\n\n所以 $f(x)_{\\text {max }}=f(0)=-1$;\n\n而 $2-2 \\sqrt{2}>-1$, 所以 $f(x)_{\\max }=2-2 \\sqrt{2}$,\n\n故答案为: B\n\n【分析】根据函数单调性, 分别求出 $x<0$ 和 $x \\geq 0$ 时的最大值, 比较大小, 即可得出结果."} {"id": "15064", "image": [], "answer": "D", "solution": "null", "level": "高一", "question": "函数 $f(x)=\\sqrt{x^{2}+4 x-5}$ 的单调递增区间是()", "options": "A. $(-\\infty,-5]$\nB. $(-\\infty,-2]$\nC. $\\lfloor-2,+\\infty)$\nD. $\\lfloor 1+\\infty)$", "subject": "代数", "analysis": "由题可知, 函数可看作 $f(t)=\\sqrt{t}, t=x^{2}+4 x-5$, 外层函数为增函数,\n\n根据同增异减的性质, 则内层函数需先满足 $x^{2}+4 x-5 \\geq 0$, 即 $x \\geq 1$ 或 $x \\leq-5$,\n\n当 $x \\geq 1$ 时, 内层函数为增函数, 则复合函数在 $x \\geq 1$ 时为增函数;\n\n故答案为: D\n\n【分析】根据复合函数同增异减的性质即可求解."} {"id": "15065", "image": [], "answer": "B", "solution": "null", "level": "高一", "question": "下列函数为偶函数的是 ( )", "options": "A. $f(x)=x+3$\nB. $f(x)=x^{2}-2$\nC. $f(x)=x^{3}$\nD. $f(x)=\\frac{1}{x}$", "subject": "代数", "analysis": "当 $f(x)=x^{2}-2$ 时, $f(-x)=(-x)^{2}-2=x^{2}-2=f(x)$, 所以 $f(x)=x^{2}-2$ 为偶函数,\n\n又 $f(x)=x+3$ 为非奇非偶函数函数, $f(x)=x^{3}$ 与 $f(x)=\\frac{1}{x}$ 为奇函数.\n\n故答案为: B\n\n【分析】根据偶函数的定义逐个判断可得答案."} {"id": "15066", "image": ["7502.jpg"], "answer": "A", "solution": "null", "level": "高一", "question": "奇函数 $f(x)$ 在 $(-\\infty, 0)$ 上单调递减, 且 $f(2)=0$, 则不等式 $f(x)>0$ 的解集是 $(\\quad)$", "options": "A. $(-\\infty,-2) \\cup(0,2)$\nB. $(-\\infty, 0) \\cup(2,+\\infty)$\nC. $(-2,0) \\cup(0,2)$\nD. $(-2,0) \\cup(2,+\\infty)$", "subject": "代数", "analysis": "因为函数式奇函数, 在 $(-\\infty, 0)$ 上单调递减,\n\n根据奇函数的性质得到在 $(0,+\\infty)$ 上函数仍是减函数,\n\n再根据 $f(2)=0$ 可画出函数在 $(0,+\\infty)$ 上的图像,\n\n根据对称性画出在 $(-\\infty, 0)$ 上的图像.\n\n\n\n根据图像得到 $f(x)>0$ 的解集是: $(-\\infty,-2) \\cup(0,2)$.\n\n故选 A.\n\n【分析】由已知利用奇函数的性质, 得到函数 $f(x)$ 在 $(0,+\\infty)$ 上函数是减函数, 画出函数图象, 利用图象即可求出 $f(x)>0$ 的解集."} {"id": "15067", "image": ["7503.jpg"], "answer": "C", "solution": "null", "level": "高一", "question": "已知定义在 $R$ 上的函数 $f(x)$ 在 $(-\\infty,-2)$ 上是减函数, 若 $g(x)=f(x-2)$ 是奇函数, 且 $g(2)=0$ ,则不等式 $x f(x) \\leq 0$ 的解集是 $(\\quad)$", "options": "A. $(-\\infty,-2] \\cup[2,+\\infty)$\nB. $\\lfloor-4,-2\\rfloor \\cup[0,+\\infty)$\nC. $(-\\infty,-4] \\cup[-2,+\\infty)$\nD. $(-\\infty,-4] \\cup[0,+\\infty)$", "subject": "代数", "analysis": "由 $g(x)=f(x-2)$ 是把函数 $f(x)$ 向右平移 2 个单位得到的,\n\n且 $g(2)=g(0)=0, f(-4)=g(-2)=-g(2)=0, f(-2)=g(0)=0$,\n\n画出 $f(x)$ 的大致形状\n\n\n\n结合函数的图像可知, 当 $x \\leq-4$ 或 $x \\geq-2$ 时, $x f(x) \\leq 0$, 故选 C.\n\n【分析】由 $g(x)=f(x-2)$ 是奇函数, 可得 $f(x)$ 的图像关于 $(-2,0)$ 中心对称, 再由已知可得函数 $f(x)$ 的三个零点为 $-4,-2,0$, 画出 $f(x)$ 的大致形状, 数形结合得出答案."} {"id": "15068", "image": ["7504.jpg", "7505.jpg"], "answer": "A", "solution": "null", "level": "高一", "question": "已知定义在 $R$ 上的函数 $f(x)$ 满足 $f(x)=f(-x)$, 且在 $(0,+\\infty)$ 上是增函数, 不等式\n\n", "options": "A. $\\left[-\\frac{3}{2},-1\\right]$\nB. $\\left[-1,-\\frac{1}{2}\\right]$\nC. $\\left[-\\frac{1}{2}, 0\\right]$\nD. $\\lfloor 0,1\\rfloor$", "subject": "代数", "analysis": "$\\because f(x)=f(-x) \\quad \\therefore f(x)$ 为定义在 $R$ 上的偶函数, 图象关于 $y_{\\text {轴对称 }}$\n\n又 $f(x)$ 在 $(0,+\\infty)$ 上是增函数 $\\therefore f(x)$ 在 $(-\\infty, 0)$ 上是减函数\n\n$\\because f(a x+2) \\leq f(-1) \\quad \\therefore|a x+2| \\leq 1$, 即 $-1 \\leq a x+2 \\leq 1$\n\n\n\n$\\therefore-\\frac{3}{2} \\leq a \\leq-1$, 即 $a$ 的取值范围为: $\\left[-\\frac{3}{2},-1\\right]$\n\n故答案为: $A$\n\n【分析】根据奇偶性定义和性质可判断出函数为偶函数且在 $(-\\infty, 0)$ 上是减函数, 由此可将不等式化为 $-1 \\leq a x+2 \\leq 1$; 利用分离变量法可得 $-\\frac{3}{x} \\leq a \\leq-\\frac{1}{x}$, 求得 $-\\frac{3}{x}$ 的最大值和 $-\\frac{1}{x}$ 的最小值即可得到结果.\n\n二、填空题"} {"id": "15076", "image": [], "answer": "D", "solution": "null", "level": "高一", "question": "已知 $f(x)$ 是 $R$ 上的偶函数, 当 $x \\leq 0$ 时, $f(x)=x+1$, 则 $f\\left(x^{2}\\right)$ 的表达式为 $(\\quad)$.", "options": "A. $-(\\mathrm{x}+1)^{2}+1$\nB. $(\\mathrm{x}+1)^{2}$\nC. $x^{2}-1$\nD. $-x^{2}+1$", "subject": "代数", "analysis": "设 $\\mathrm{x}>0$, 所以 $-x<0$, 所以 $f(-x)=-x+1$,\n\n所以 $f(x)=-x+1$, 所以 $f\\left(x^{2}\\right)=-x^{2}+1$.\n\n故答案为:\n\n【分析】先求出 $\\mathrm{x}>0$ 的解析式, 再求 $\\mathrm{f}\\left(\\mathrm{x}^{2}\\right)$ 的表达式."} {"id": "15080", "image": [], "answer": "A", "solution": "null", "level": "高一", "question": "下列函数是奇函数的是 ( )", "options": "A. $y=x$\nB. $y=2 x^{2}-3$\nC. $y=\\sqrt{x}$\nD. $y=x^{2}, x \\in[0,1]$", "subject": "代数", "analysis": "$\\mathrm{A}$ 中, $\\mathrm{y}=\\mathrm{x}$ 是奇函数, $\\mathrm{B}$ 中, $\\mathrm{y}=2 \\mathrm{x}^{2}-3$ 是偶函数,\n\n$\\mathrm{C}$ 中, $\\mathrm{y}=\\sqrt{x}$ 是非奇非偶函数, $\\mathrm{D}$ 中, $\\mathrm{y}=\\mathrm{x}^{2}, \\mathrm{x} \\in[0,1]$ 是非奇非偶函数.\n\n故答案为: A\n\n【分析】利用函数奇偶性的定义分别判断各选项, 即可得结果."} {"id": "15081", "image": [], "answer": "A", "solution": "null", "level": "高一", "question": "已知函数 $y=f(x)+x^{2}$ 是奇函数, 且 $f(1)=1$, 则 $f(-1)=$", "options": "A. -3\nB. -1\nC. 0\nD. 2", "subject": "代数", "analysis": "$\\because y=f(x)+x^{2}$ 为奇函数 $\\therefore f(-x)+x^{2}=-f(x)-x^{2}$\n\n$\\therefore f(-1)+1=-f(1)-1=-2 \\quad \\therefore f(-1)=-3$\n\n故答案为: $A$\n\n【分析】由奇函数定义可得 $f(-x)+x^{2}=-f(x)-x^{2}$, 代入 $x=1$ 可求得结果."} {"id": "15083", "image": [], "answer": "D", "solution": "null", "level": "高一", "question": "已知 $f(x)$ 是定义在 $\\mathrm{R}$ 上的偶函数, 并满足 $f(x+2)=-\\frac{1}{f(x)}$, 当 $1 \\leq x \\leq 2$ 时, $f(x)=x-2$,则 $f(6.5)=(\\quad)$", "options": "A. 4.5\nB. -4.5\nC. 0.5\nD. -0.5", "subject": "代数", "analysis": "$\\because f(x+4)=-\\frac{1}{f(x+2)}=f(x)$\n\n$\\therefore f(6.5)=f(4+2.5)=f(2.5)=f(-1.5+4)=f(-1.5)=f(1.5)=1.5-2=-0.5$\n\n故答案为: D\n\n【分析】由题意得出 $f(x+4)=f(x)$, 结合偶函数的性质, 即可得出 $f(6.5)$ 的值."} {"id": "15084", "image": [], "answer": "B", "solution": "null", "level": "高一", "question": "已知函数 $f(x)=x \\sqrt{9-x^{2}}$, 则 $(\\quad)$", "options": "A. $f(1)>f(2)$\nB. $f(x)$ 的定义域为 $\\lfloor-3,3\\rfloor$\nC. $f(x)_{\\text {为偶函数 }}$\nD. $f(x)_{\\text {在 }}\\lfloor 0,3\\rfloor$ 上为增函数", "subject": "代数", "analysis": "因为 $f(1)=2 \\sqrt{2}<2 \\sqrt{5}=f(2)$, 所以 $\\mathrm{A}$ 不符合题意;\n\n由 $9-x^{2} \\geq 0$, 得 $-3 \\leq x \\leq 3$,\n\n所以 $f(x)$ 的定义域为 $\\lfloor-3,3\\rfloor$, 所以 $\\mathrm{B}$ 符合题意;\n\n$f(x)$ 为奇函数, 所以 $\\mathrm{C}$ 不符合题意;\n\n因为 $f(0)=f(3)=0$, 所以 $\\mathrm{D}$ 不符合题意.\n\n故答案为: B\n\n【分析】逐项判断各选项中的结论正确与否后可得正确的选项."} {"id": "15085", "image": [], "answer": "D", "solution": "null", "level": "高一", "question": "函数 $f(x)$ 在 $(-\\infty,+\\infty)$ 单调递减, 且为奇函数. 若 $f(1)=-1$, 则满足 $-1 \\leq f(x-2) \\leq 1$ 的 $x$取值范围是( )", "options": "A. $[-2,2]$\nB. $[-1,1]$\nC. $[0,4]$\nD. $[1,3]$", "subject": "代数", "analysis": "$\\because f(x)$ 为奇函数, $\\therefore f(-x)=-f(x)$.\n\n$\\because f(1)=-1, \\quad \\therefore f(-1)=-f(1)=1$.\n\n故由 $-1 \\leq f(x-2) \\leq 1$, 得 $f(1) \\leq f(x-2) \\leq f(-1)$.\n\n又 $f(x)$ 在 $(-\\infty,+\\infty)$ 单调递减, $\\therefore-1 \\leq x-2 \\leq 1$,\n\n$\\therefore 1 \\leq x \\leq 3$.\n\n故选: D\n\n【分析】根据奇函数的性质由 $f(1)=-1$, 可以求出 $f(-1)$ 的值, 再利用函数的单调性结合已知 $-1 \\leq f(x-2) \\leq 1$, 可以求出 $x$ 取值范围."} {"id": "15086", "image": ["7515.jpg"], "answer": "D", "solution": "null", "level": "高一", "question": "若定义在 $\\mathrm{R}$ 的奇函数 $\\mathrm{f}(\\mathrm{x})$ 在 $(-\\infty, 0)$ 单调递减, 且 $\\mathrm{f}(2)=0$, 则满足 $x f(x-1) \\geq 0$ 的 $\\mathrm{x}$ 的取值范围是 ( )", "options": "A. $[-1,1] \\cup[3,+\\infty)$\nB. $[-3,-1] \\cup[0,1]$\nC. $[-1,0] \\cup[1,+\\infty)$\nD. $[-1,0] \\cup[1,3]$", "subject": "代数", "analysis": "因为定义在 $\\mathrm{R}$ 上的奇函数 $f(x)$ 在 $(-\\infty, 0)$ 上单调递减, 且 $f(2)=0$,\n\n所以 $f(x)$ 在 $(0,+\\infty)$ 上也是单调递减, 且 $f(-2)=0, f(0)=0$,\n\n所以当 $x \\in(-\\infty,-2) \\cup(0,2)$ 时, $f(x)>0$, 当 $x \\in(-2,0) \\cup(2,+\\infty)$ 时, $f(x)<0$,\n所以由 $x f(x-1) \\geq 0$ 可得:\n\n\n\n解得 $-1 \\leq x \\leq 0$ 或 $1 \\leq x \\leq 3$,\n\n所以满足 $x f(x-1) \\geq 0$ 的 $x$ 的取值范围是 $[-1,0] \\cup[1,3]$,\n\n故答案为: D.\n\n【分析】首先根据函数奇偶性与单调性, 得到函数 $f(x)$ 在相应区间上的符号, 再根据两个数的乘积大于等于零, 分类转化为对应自变量不等式, 最后求并集得结果."} {"id": "15087", "image": [], "answer": "B", "solution": "null", "level": "高一", "question": "化简 $\\left(\\sqrt[3]{(-5)^{2}}\\right)^{\\frac{3}{4}}$ 的结果为 ( )", "options": "A. 5\nB. $\\sqrt{5}$\nC. $-\\sqrt{5}$\nD. -5", "subject": "代数", "analysis": "$\\left(\\sqrt[3]{(-5)^{2}}\\right)^{\\frac{3}{4}}=\\left(5^{2}\\right)^{\\frac{1}{3} \\times \\frac{3}{4}}=5^{2 \\times \\frac{1}{4}}=5^{\\frac{1}{2}}=\\sqrt{5}$\n\n故答案为: $B$\n\n【分析】先将根式外面写成幕的形式,再依次化简即可."} {"id": "15089", "image": [], "answer": "A", "solution": "null", "level": "高一", "question": "当 $\\sqrt{2-x}$ 有意义时, 化简 $\\sqrt{x^{2}-4 x+4}-\\sqrt{x^{2}-6 x+9}$ 的结果是()", "options": "A. -1\nB. $-2 x-1$\nC. $2 x-5$\nD. $5-2 x$", "subject": "代数", "analysis": "由题意知 $2-x \\geq 0$, 即 $x \\leq 2$, 原式 $=\\sqrt{(x-2)^{2}}-\\sqrt{(x-3)^{2}}$\n\n$=|x-2|-|x-3|=(2-x)-(3-x)=-1$.\n\n故答案为:A.\n\n【分析】先求出 $\\mathrm{x}$ 的范围,再对目标式进行化简.\n\n二、填空题"} {"id": "15099", "image": [], "answer": "C", "solution": "null", "level": "高一", "question": "化简: $\\sqrt{(3-\\pi)^{2}}+\\pi=(\\quad)$", "options": "A. 3\nB. $3-2 \\pi$\nC. $2 \\pi-3$\nD. $2 \\pi-3$ 或 3", "subject": "代数", "analysis": "$\\sqrt{(3-\\pi)^{2}}+\\pi=|3-\\pi|+\\pi=\\pi-3+\\pi=2 \\pi-3$\n\n故答案为:\n\nC\n\n【分析】根据根式的性质 $\\sqrt{a^{2}}=\\mid a_{\\mid}$化简即可."} {"id": "15101", "image": [], "answer": "A", "solution": "null", "level": "高一", "question": "已知 $x^{\\frac{1}{2}}-x^{-\\frac{1}{2}}=\\sqrt{5}$, 则 $x+\\frac{1}{x}$ 的值为 ( )", "options": "A. 7\nB. $3 \\sqrt{5}$\nC. $\\pm 3 \\sqrt{5}$\nD. 27", "subject": "代数", "analysis": "由 $x^{\\frac{1}{2}}-x^{-\\frac{1}{2}}=\\sqrt{5}$, 两边平方得: $x-2+\\frac{1}{x}=5$,\n\n则 $x+\\frac{1}{x}=7$,\n\n故答案为: A.\n\n【分析】直接把已知等式两边平方求解即可."} {"id": "15104", "image": [], "answer": "B", "solution": "null", "level": "高一", "question": "已知 $a>0$, 则 $\\sqrt{a^{\\frac{1}{3}} \\sqrt{a^{\\frac{1}{2}} \\sqrt{a}}}$ 化为()", "options": "A. $a^{\\frac{7}{12}}$\nB. $a_{12}^{\\frac{5}{12}}$\nC. $a^{\\frac{3}{6}}$\nD. $a^{\\frac{1}{3}}$", "subject": "代数", "analysis": "$a>0, \\sqrt{a^{\\frac{1}{3}} \\sqrt{a^{\\frac{1}{2}} \\sqrt{a}}}=\\sqrt{a^{\\frac{1}{3}} \\sqrt{a^{\\frac{1}{2}} \\cdot a^{\\frac{1}{2}}}}=\\sqrt{a^{\\frac{1}{3}} \\cdot \\sqrt{a}}=\\sqrt{a^{\\frac{1}{3}} \\cdot a^{\\frac{1}{2}}}=\\sqrt{a^{\\frac{5}{6}}}=a^{\\frac{5}{12}}$.\n\n故答案为: B\n\n【分析】利用 $\\mathrm{a}$ 的取值范围结合根式和分数指数幂的对应关系, 从而化简得出结果。"} {"id": "15105", "image": ["7516.jpg"], "answer": "D", "solution": "null", "level": "高一", "question": "若 $2^{x}=7,2^{y}=6$, 则 $4^{x-y}$ 等于 ( )", "options": "A. $\\frac{36}{49}$\nB. $\\frac{7}{6}$\nC. $\\frac{6}{7}$\nD. $\\frac{49}{36}$\n\n\nA. $\\frac{16}{15}$\nB. $3 \\frac{17}{30}$\nC. $-8 \\frac{5}{6}$\nD. 0", "subject": "代数", "analysis": "$2^{x}=7,2^{y}=6$ 则 $4^{x-y}=2^{2 x-2 y}=\\frac{2^{2 x}}{2^{2 y}}=\\frac{49}{36}$;\n\n故答案为: D.\n\n【分析】根据已知条件得出 $4^{x-y}=2^{2 x-2 y}=2^{2 y}$, 进而得出结果。"} {"id": "15106", "image": [], "answer": "D", "solution": "null", "level": "高一", "question": "若函数 $f(x)=\\left(\\frac{1}{2} a-3\\right) \\cdot a^{x}$ 是指数函数, 则 $f\\left(\\frac{1}{2}\\right)$ 的值为 $(\\quad)$", "options": "A. 2\nB. -2\nC. $-2 \\sqrt{2}$\nD. $2 \\sqrt{2}$", "subject": "代数", "analysis": "$\\because$ 函数 $\\mathrm{f}(\\mathrm{x})=\\left(\\frac{1}{2} \\mathrm{a}-3\\right) \\cdot \\mathrm{a}^{\\mathrm{x}}$ 是指数函数,\n\n$\\therefore \\frac{1}{2} a-3=1, a>0, a \\neq 1$, 解得 $a=8$,\n\n$\\therefore \\mathrm{f}(\\mathrm{x})=8^{\\mathrm{x}}$,\n\n$\\therefore f\\left(\\frac{1}{2}\\right)=\\sqrt{8}={ }_{2} \\sqrt{2}$,\n\n故答案为: D.\n\n【分析】根据指数函数的定义可得 $\\frac{1}{2} \\mathrm{a}-3=1, \\mathrm{a}>0, \\mathrm{a} \\neq 1$, 先求出函数解析式, 将 $\\mathrm{x}=\\frac{1}{2}$ 代入可得答案."} {"id": "15124", "image": [], "answer": "C", "solution": "null", "level": "高一", "question": "函数 $y=\\sqrt{4-2^{x}}$ 的值域是 ( )", "options": "A. $[0,+\\infty)$\nB. $[0,2]$\nC. $[0,2)$\nD. $(0,2)$", "subject": "代数", "analysis": "$\\because 2^{x}>0$, 故 $0 \\leq 4-2^{x}<4$,\n\n$\\therefore$ 函数值域为 $[0,2)$.\n\n故答案为: C\n\n【分析】利用指数函数的值域结合构造法求出函数 $y=\\sqrt{4-2^{x}}$ 的值域。"} {"id": "15126", "image": [], "answer": "C", "solution": "null", "level": "高一", "question": "若指数函数 $y=(2 a-3)^{x}$ 在 $R$ 上是增函数, 则实数 $a$ 的取值范围是 ( )", "options": "A. $(-\\infty, 2)$\nB. $(-\\infty, 2]$\nC. $(2,+\\infty)$\nD. $[2,+\\infty)$", "subject": "代数", "analysis": "由指数函数单调性可知 $2 a-3>1 \\therefore a>2$, 实数 $a$ 的取值范围是 $(2,+\\infty)$\n\n故答案为: C\n\n【分析】利用指数函数的单调性求出实数 $a$ 的取值范围。"} {"id": "15128", "image": [], "answer": "C", "solution": "null", "level": "高一", "question": "函数 $y=\\left(a^{2}-4 a+4\\right) a^{x}$ 是指数函数, 则 $a$ 的值是 ( )", "options": "A. 4\nB. 1 或 3\nC. 3\nD. 1", "subject": "代数", "analysis": "由题意 $\\left\\{\\begin{array}{l}a^{2}-4 a+4=1 \\\\ a>0 \\text { 且 } a \\neq 1\\end{array}\\right.$, 解得 $a=3$.\n\n故答案为: C.\nC.\n\n【分析】由已知利用指数函数的概念列式, 即可求出 $\\mathrm{a}$ 的值."} {"id": "15129", "image": [], "answer": "B", "solution": "null", "level": "高一", "question": "设 $a \\log _{3} 4=2$, 则 $4^{-a}=(\\quad)$", "options": "A. $\\frac{1}{16}$\nB. $\\frac{1}{9}$\nC. $\\frac{1}{8}$\nD. $\\frac{1}{6}$", "subject": "代数", "analysis": "由 $a \\log _{3} 4=2$ 可得 $\\log _{3} 4^{a}=2$, 所以 $4^{a}=9$,\n\n所以有 $4^{-a}=\\frac{1}{9}$,\n\n故答案为: B.\n\n【分析】首先根据题中所给的式子, 结合对数的运算法则, 得到 $\\log _{3} 4^{a}=2$, 即 $4^{a}=9$, 进而求得 $4^{-a}=\\frac{1}{9}$, 得到结果."} {"id": "15130", "image": [], "answer": "D", "solution": "null", "level": "高一", "question": "已知 $\\log _{4} 3=\\mathrm{p}, \\log _{3} 25=\\mathrm{q}$, 则 $\\lg 5=$", "options": "A. $\\frac{P q}{p+q}$\nB. $\\frac{p+q}{p q}$\nC. $\\frac{1+p q}{p+q}$\nD. $\\frac{p q}{1+p q}$", "subject": "代数", "analysis": "答案: D\n解: 解: (换底公式) $p q=\\log _{4} 3 \\cdot \\log _{3} 25=\\frac{\\lg 3}{\\lg 4} \\cdot \\frac{\\lg 25}{\\lg 3}=\\frac{2 \\lg 5}{2 \\lg 2}=\\frac{\\lg 5}{1-\\lg 5}$,\n\n$\\therefore \\lg 5=\\frac{p q}{1+p q}$,\n\n故答案为: D.\n\n【分析】计算 $p q$, 利用对数换底公式、对数运算性质变形, 化为 $\\lg 5$ 的式子后可得 $\\lg 5$."} {"id": "15131", "image": ["7523.jpg"], "answer": "B", "solution": "null", "level": "高一", "question": "设 $P=\\frac{1}{\\log _{2} 11}+\\frac{1}{\\log _{3} 11}+\\frac{1}{\\log _{4} 11}+\\frac{1}{\\log _{5} 11}$, 则 $(\\quad)$", "options": "A. $0<\\mathrm{P}<1$\nB. $1<\\mathrm{P}<2$\nC. $2<\\mathrm{P}<3$\nD. $3<\\mathrm{P}<4$\n\n\n$\\lfloor 2 \\cdot 1]=2$; 则 $\\left[\\log _{3} 1\\right]+\\left[\\log _{3} 2\\right]+\\left[\\log _{3} 3\\right]+\\cdots \\cdot+\\left[\\log _{3} 27\\right]$ 的值为 $(\\quad)$\nA. 42\nB. 43\nC. 44\nD. 45", "subject": "代数", "analysis": "$$\nP=\\frac{1}{\\log _{2} 11}+\\frac{1}{\\log _{3} 11}+\\frac{1}{\\log _{4} 11}+\\frac{1}{\\log _{5} 11}\n$$\n\n$=\\log _{11} 2+\\log _{11} 3+\\log _{11} 4+\\log _{11} 5=\\log _{11}(2 \\times 3 \\times 4 \\times 5)=\\log _{11} 120$.\n\n$\\therefore \\log _{11} 11=1<\\log _{11} 120<\\log _{11} 121=2$.\n\n故答案为: B.\n\n【分析】根据对数性质化简为同底数的对数的和, 再根据对数运算性质化简求结果, 最后确定取值范围."} {"id": "15137", "image": [], "answer": "C", "solution": "null", "level": "高一", "question": "若 $7^{x}=8$, 则 $x=(\\quad)$", "options": "A. $\\frac{8}{7}$\nB. $\\log _{8} 7$\nC. $\\log _{7} 8$\nD. $\\log _{7} x$", "subject": "代数", "analysis": "$7^{x}=8$ 则 $x=\\log _{7} 8$\n\n故答案为: C\n\n【分析】利用指对互化求解即可."} {"id": "15143", "image": [], "answer": "C", "solution": "null", "level": "高一", "question": "已知 $\\ln \\left(\\log _{4}\\left(\\log _{2} x\\right)\\right)=0$, 那么 $x^{-\\frac{1}{2}}=(\\quad)$", "options": "A. 4\nB. -4\nC. $\\frac{1}{4}$\nD. $-\\frac{1}{4}$", "subject": "代数", "analysis": "因为 $\\ln \\left(\\log _{4}\\left(\\log _{2} x\\right)\\right)=0$,\n\n所以 $\\log _{4}\\left(\\log _{2} x\\right)=1$, 即 $\\log _{2} x=4$,\n\n所以 $x=2^{4}=16, x^{-\\frac{1}{2}}=16^{-\\frac{1}{2}}=\\frac{1}{4}$,\n\n故答案为: C\n\n【分析】根据对数的性质及指数幂的运算法则求解即可."} {"id": "15145", "image": [], "answer": "C", "solution": "null", "level": "高一", "question": "下列等式成立的是( )", "options": "A. $\\log _{2}(8-4)=\\log _{2} 8-\\log _{2} 4$\nB. $\\frac{\\log _{2} 8}{\\log _{2} 4}=\\log _{2} \\frac{8}{4}$\nC. $\\log _{2} 2^{3}=3 \\log _{2} 2$\nD. $\\log _{2}(8+4)=\\log _{2} 8+\\log _{2} 4$", "subject": "代数", "analysis": "根据对数的运算性质逐个进行判断可得, A,B,D 都不符合对数的运算性质, C 符合. 所以 C 符合题意.\n\n故答案为: C.\n【分析】根据对数的运算性质进行分析、判断即可得到答案."} {"id": "15146", "image": [], "answer": "D", "solution": "null", "level": "高一", "question": "若实数 $\\mathrm{a}, \\mathrm{b}$ 满足 $3^{a}=4^{b}=12$, 则 $\\frac{1}{a}+\\frac{1}{b}=$ ()", "options": "A. $\\frac{1}{2}$\nB. $\\frac{1}{5}$\nC. $\\frac{1}{6}$\nD. 1", "subject": "代数", "analysis": "因为 $3^{a}=4^{b}=12$, 所以 $a=\\log _{3} 12, b=\\log _{4} 12$,\n\n$\\frac{1}{a}+\\frac{1}{b}=\\frac{1}{\\log _{3} 12}+\\frac{1}{\\log _{4} 12}=\\log _{12} 3+\\log _{12} 4=\\log _{12} 12=1$.\n\n故答案为: D.\n\n【分析】先将指数式化成对数式, 求出 $a, b$, 再利用换底公式的推论 $\\log _{a} b \\cdot \\log _{b} a=1$ 以及对数的运算法则即可求出."} {"id": "15148", "image": [], "answer": "B", "solution": "null", "level": "高一", "question": "已知 $x, y, z$ 都是大于 1 的正数, $m>0$, 且 $\\log _{x} m=24, \\log _{y} m=40, \\log _{x y z} m=12$, 则 $\\log _{z} m$ 的值为 $(\\quad)$", "options": "A. $\\frac{1}{60}$\nB. 60\nC. $\\frac{200}{3}$\nD. $\\frac{3}{200}$", "subject": "代数", "analysis": "由已知得 $\\log _{\\mathrm{m}}(\\mathrm{xyz})=\\log _{\\mathrm{m}} \\mathrm{x}+\\log _{\\mathrm{m}} \\mathrm{y}+\\log _{\\mathrm{m}} \\mathrm{z}=\\frac{1}{12}$,\n\n而 $\\log _{\\mathrm{m}} \\mathrm{x}=\\frac{1}{24}, \\log _{\\mathrm{m}} \\mathrm{y}=\\frac{1}{40}$,\n\n故 $\\log _{\\mathrm{m}} \\mathrm{z}=\\frac{1}{12}-\\log _{\\mathrm{m}} \\mathrm{x}-\\log _{\\mathrm{m}} \\mathrm{y}=\\frac{1}{12}-\\frac{1}{24}-\\frac{1}{40}=\\frac{1}{60}$, 即 $\\log _{\\mathrm{z}} \\mathrm{m}=60$.\n\n故答案为: B\n\n【分析】先求出 $\\log _{m}(x y z)=\\log _{m} x+\\log _{m} y+\\log _{m} z=\\frac{1}{12}$, 再计算出 $\\log _{m} z$, 即得 $\\log _{z} m$ 的值."} {"id": "15151", "image": ["7524.jpg", "7525.jpg", "7526.jpg", "7527.jpg", "7528.jpg"], "answer": "D", "solution": "null", "level": "高一", "question": "函数 $f(x)=x \\cdot \\ln |x|$ 的图象可能是( )", "options": "A.\n\n\n\nC.\n\n\n\nB.\n\n\n\nD.\n\n", "subject": "代数", "analysis": "\n当 $x=\\frac{1}{e}$ 时, $y=-\\frac{1}{e}$, 对应点在 $\\mathrm{x}$ 轴下方, 排除 $\\mathrm{B}$;\n\n故答案为: D.\n\n【分析】根据奇偶性的定义, 判断函数为奇函数, 结合图象的对称性及函数的取值逐一排除即可."} {"id": "15152", "image": ["7529.jpg"], "answer": "B", "solution": "null", "level": "高一", "question": "若 $\\log _{a}\\left(a^{2}+1\\right)<0<\\log _{a}(2 a)$, 那么实数 $a$ 的取值范围是 $(\\quad)$", "options": "A. $(0,1)$\nB. $\\left(0, \\frac{1}{2}\\right)$\nC. $\\left(\\frac{1}{2}, 1\\right)$\nD. $(1,+\\infty)$", "subject": "代数", "analysis": "当 $a>1$ 时, $\\log _{a}\\left(a^{2}+1\\right)>0$, 显然不适合题意;\n\n\n\n即 $01,0f(4-a)$, 则实数 $a$ 的取值范围是 ( )", "options": "A. $(1,2)$\nB. $(2,3)$\nC. $(1,3)$\nD. $(2,4)$", "subject": "代数", "analysis": "函数 $f(x)=\\mid \\ln (x-1)$ 的定义域为 $(1+\\infty)$,\n\n由 $f(a)>f(4-a)$ 可得: $|\\ln (a-1)|>|\\ln (4-a-1)|=|\\ln (3-a)|$,\n\n两边平方: $[\\ln (a-1)]^{2}>[\\ln (3-a)]^{2} \\Leftrightarrow[\\ln (a-1)-\\ln (3-a)[\\ln (a-1)+\\ln (3-a)]>0$,\n\n\n\n解 (1) 得: $a$ 无解 , 解 (2) 得: $1f(4-a)$ 代入函数中化简, 解出不等式的解, 即可得到答案。\n\n二、填空题"} {"id": "15162", "image": ["7537.jpg"], "answer": "C", "solution": "null", "level": "高一", "question": "已知 $a=\\frac{3}{5}, b=\\log _{0.2} 0.1, c=\\log _{3} 2$, 则 $a 、 b 、 c$ 的大小关系是 ( )", "options": "A. $c\\log _{0.2} 0.2=1$.\n\n函数 $y=\\log _{3} x$ 为增函数, 则 $c=\\log _{3} 2<\\log _{3} 3=1$.\n\n下面来比较 $a$ 与 $c$ 的大小关系,\n\n\n\n$\\because 5 \\log _{3} 2=\\log _{3} 2^{3}=\\log _{3} 32>\\log _{3} 3^{3}=3, \\quad \\therefore c>a$,\n\n因此, $a1, c<1$, 再比较 $a$ 与 $c$ 的大小关系, 可得出 $a 、 b$ 、 $c$ 三个数的大小关系."} {"id": "15164", "image": [], "answer": "C", "solution": "null", "level": "高一", "question": "若 $a=\\log _{6} 7, \\quad b=\\log _{5} 4, \\quad c=\\log _{\\frac{1}{3}} 4$, 则 ( )", "options": "A. $a\\log _{6} 6=1, \\quad 0=\\log _{5} 11, \\quad 01)$ 的图象必不过 $(\\quad)$.", "options": "A. 第一象限\nB. 第二象限\nC. 第三象限\nD. 第四象限", "subject": "代数", "analysis": "因为 $a>1$, 所以对数函数 $y=\\log _{a} x$ 经过点 $(1,0)$, 经过第一、四象限,\n\n函数 $\\mathrm{f}(\\mathrm{x})=\\log _{\\mathrm{a}}(\\mathrm{x}+2)(\\mathrm{a}>1)$ 的图象就是把函数 $y=\\log _{a} x$ 的图象向左平移 2 个单位,\n\n所以函数 $\\mathrm{f}(\\mathrm{x})=\\log _{\\mathrm{a}}(\\mathrm{x}+2)(\\mathrm{a}>1)$ 的图象必不经过第四象限.\n\n故答案为:\n\n"} {"id": "15166", "image": [], "answer": "C", "solution": "null", "level": "高一", "question": "已知 $0\\log _{b} a$\nC. $a^{b}\\log _{a} b, \\log _{b} a>\\log _{b} b$,\n\n从而得到 $\\log _{a} b<1<\\log _{b} a$, 所以错误;\n\n对于 C: $\\because 00, \\therefore y=x^{a}$ 在 $(0,+\\infty)$ 上单调递增,\n\n由 $a0, f(x)$ 单调递增;\n\n因为 $0\n\n所以可知 $a=\\log _{8} 7>\\log _{9} 7$,\n\n而 $b=\\log _{3} 2=\\log _{9} 4$,\n\n\n\n且 $1=\\log _{9} 9>\\log _{9} 7>\\log _{9} 4>\\log _{9} 1=0$,\n\n所以 $0} \\pi^{0}=1$,\n\n所以 $c>1$, 故 $c>a>b$,\n\n故答案为: C\n\n【分析】根据 $\\log _{8} 7>\\log _{9} 7$, 将 $b=\\log _{3} 2=\\log _{9} 4$, 利用对数函数的单调性, 可得 $a, b$ 大小关系,然后借助中间值 1 , 以及指数函数的单调性, 可得结果."} {"id": "15169", "image": [], "answer": "A", "solution": "null", "level": "高一", "question": "下列函数为幂函数的是 ( )", "options": "A. $y=x^{2}$\nB. $y=-x^{2}$\nC. $y=2^{x}$\nD. $y=2 x^{2}$", "subject": "代数", "analysis": "由幂函数的定义 $f(x)=x^{a}$ 可知,\n\n故答案为: A\n\n【分析】由已知利用幂函数的定义分别判断各选项, 即可得结果."} {"id": "15173", "image": ["7541.jpg", "7542.jpg", "7543.jpg", "7544.jpg"], "answer": "A", "solution": "null", "level": "高一", "question": "函数 $f(x)=x^{-\\frac{1}{2}}$ 的大致图象是 ( )", "options": "A.\n\n\n\nC.\n\n\n\nB.\n\n\n\nD.\n\n", "subject": "代数", "analysis": "由题意得, $f(x)=x^{-\\frac{1}{2}}=\\frac{1}{\\sqrt{x}}$, 所以函数的定义域为 $\\{x \\mid x>0\\}$,\n\n因为 $-\\frac{1}{2}<0$, 根据幂函数的性质, 可知函数 $f(x)=x-\\frac{1}{2}$ 在第一象限为单调递减函数,\n\n故答案为: A.\n\n【分析】由已知幂函数, 得到函数的定义域为 $\\{x \\mid x>0\\}$, 利用幂函数的图象在第一象限单调递减,即可判断函数的大致图象."} {"id": "15175", "image": [], "answer": "A", "solution": "null", "level": "高一", "question": "函数 $f(x)=\\left(m^{2}-m-1\\right) x^{4} m^{2}-m^{s}-1$ 是幂函数, 对任意的 $x_{1}, x_{2} \\in(0,+\\infty)$, 且 $x_{1} \\neq x_{2}$, 满足 $\\frac{f\\left(x_{1}\\right)-f\\left(x_{2}\\right)}{x_{1}-x_{2}}>0$, 若 $a, b \\in R$, 且 $a+b>0$, 则 $f(a)+f(b)$ 的值 ( )", "options": "A. 恒大于 0\nB. 恒小于 0\nC. 等于 0\nD. 无法判断", "subject": "代数", "analysis": "因为对任意的 $x_{1}, x_{2} \\in(0,+\\infty)$, 且 $x_{1} \\neq x_{2}$, 满足 $\\frac{f\\left(x_{1}\\right)-f\\left(x_{2}\\right)}{x_{1}-x_{2}}>0$,\n\n所以幂函数 $f(x)$ 在 $(0,+\\infty)$ 上是增函数,\n\n$\\therefore\\left\\{\\begin{array}{l}m^{2}-m-1=1 \\\\ 4 m^{9}-m^{5}-1>0,\\end{array}\\right.$ 解得 $m=2$, 则 $f(x)=x^{2015}$,\n\n$\\therefore$ 函数 $f(x)=x^{2015}$ 在 $R$ 上是奇函数, 且为增函数.\n\n由 $a+b>0$, 得 $a>-b$,\n\n$\\therefore f(a)>f(-b)=-f(b)$\n\n$$\n\\therefore f(a)+f(b)>0,\n$$\n\n故答案为: A.\n\n【分析】根据题意及幂函数的基本性质求得 $\\mathrm{m}$, 得到幂函数 $f(x)=x^{2015}$, 该函数为奇函数且单调递增,代入数据计算, 即可得出答案。\n\n二、填空题"} {"id": "15180", "image": [], "answer": "C", "solution": "null", "level": "高一", "question": "已知函数 $f(x)=(m-1)^{2} x^{m^{2}-4 m+2}$ 是在 $(0,+\\infty)$ 上单调递增的幂函数, 则 $m=(\\quad)$", "options": "A. 0 或 4\nB. 0 或 2\nC. 0\nD. 2", "subject": "代数", "analysis": "$\\because f(x)$ 是幂函数,\n\n$\\therefore(m-1)^{2}=1$, 得 $m=0$, 或 $m=2$,\n\n$\\because f(x)$ 在 $(0,+\\infty)$ 上单调递增,\n\n$\\therefore m^{2}-4 m+2>0$,\n\n则当 $m=0$ 时, $2>0$ 成立,\n\n当 $m=2$ 时, $4-8+2=-2$, 不成立,\n\n故选 C.\n\n【分析】根据幂函数的定义求出 $m$ 的值, 结合幂函数的单调性进行求解即可."} {"id": "15187", "image": [], "answer": "D", "solution": "null", "level": "高一", "question": "若函数 $\\mathrm{f}(\\mathrm{x})$ 是幂函数, 且满足 $\\frac{f(4)}{f(2)}=3$, 则 $f\\left(\\frac{1}{2}\\right)$ 的值为 ( )", "options": "A. -3\nB. $-\\frac{1}{3}$\nC. 3\nD. $\\frac{1}{3}$", "subject": "代数", "analysis": "设 $f(x)=x^{a}$, 则由 $\\frac{f(4)}{f(2)}=3$, 得 $\\frac{4^{a}}{2^{a}}=3$.\n\n所以 $2^{a}=3$, 故 $f\\left(\\frac{1}{2}\\right)=\\left(\\frac{1}{2}\\right)^{a}=\\frac{1}{3}$.\n\n故答案为: D.\n【分析】设出幂函数的一般形式, 从而把 $\\frac{f(4)}{f(2)}=3$ 转化为关于幂指数的方程, 解出幂指数后可求 $f\\left(\\frac{1}{2}\\right)$."} {"id": "15189", "image": [], "answer": "A", "solution": "null", "level": "高一", "question": "已知幂函数 $y=f(x)$ 的图象过 $\\left(2 \\frac{\\sqrt{2}}{2}\\right)$, 则下列求解正确的是 ( )", "options": "A. $f(x)=x^{\\frac{1}{2}}$\nB. $f(x)=x^{2}$\nC. $f(x)=x^{\\frac{3}{2}}$\nD. $f(x)=x^{-\\frac{1}{2}}$", "subject": "代数", "analysis": "$\\because$ 幂函数 $\\mathrm{y}=\\mathrm{x}^{\\alpha}$ 的图象过点 $(2, \\sqrt{2})$,\n\n$\\therefore \\sqrt{2}={ }_{2}^{\\alpha}$, 解得 $\\alpha=\\frac{1}{2}$,\n\n故 $\\mathrm{f}(\\mathrm{x})=\\sqrt{x}$, 即 $f(x)=x^{\\frac{1}{2}}$,\n\n故答案为: A\n\n【分析】利用幂函数过的点求出幂函数的解析式即可逐项判断正误."} {"id": "15190", "image": ["7547.jpg", "7548.jpg"], "answer": "A", "solution": "null", "level": "高一", "question": "如图所示的曲线是幂函数 $y=x^{n}$ 在第一象限内的图象. 已知 $n$ 分别取 $-1,1, \\frac{1}{2}, 2$ 四个值, 则与\n\n\n\n", "options": "A. $2,1, \\frac{1}{2},-1$\nB. $2,-1,1, \\frac{1}{2}$\nC. $\\frac{1}{2}, 1,2,-1$\nD. $-1,1,2, \\frac{1}{2}$", "subject": "代数", "analysis": "幕函数在区间 $(0,1)$ 上, 图象“指大图低”, 所以从上至下依次为 $-1 \\frac{1}{2}, 1,2$,\n\n对应曲线有\n\n$$\nC_{1}: n=2, C_{2}: n=1 C_{3}: n=\\frac{1}{2}, C_{4}: n=-1 .\n$$\n\n故选: A\n\n【分析】由幂函数的图象性质,在区间 $(0,1)$ 上,图象“指大图低”, 观察得答案."} {"id": "15191", "image": [], "answer": "B", "solution": "null", "level": "高一", "question": "若函数 $f(x)=(m+2) x^{a}$ 是幂函数, 且其图象过点 $(2,4)$, 则函数 $g(x)=\\log _{a}(x+m)$ 的单调增区间为 ( )", "options": "A. $(-2,+\\infty)$\nB. $(1+\\infty)$\nC. $(-1,+\\infty)$\nD. $(2,+\\infty)$", "subject": "代数", "analysis": "由题意得: $m+2=1$, 解得: $m=-1$, 故 $f(x)=x^{a}$,\n\n将 $(2,4)$ 代入函数的解析式得: $2^{a}=4$, 解得: $a=2$,\n\n故 $g(x)=\\log _{a}(x+m)=\\log _{2}(x-1)$ ,\n\n令 $x-1>0$, 解得: $x>1$,\n\n故 $g(x)_{\\text {在 }}(1,+\\infty)_{\\text {递增, }}$\n故答案为: B.\n\n【分析】分别求出 $\\mathrm{m}, \\mathrm{a}$ 的值, 求出函数 $g(x)$ 的单调区间即可."} {"id": "15192", "image": [], "answer": "A", "solution": "null", "level": "高一", "question": "已知函数 $f(x)=(2 n-1) x^{-m}{ }^{2}+2 m+3$, 其中 $m \\in N$, 若函数 $f(x)$ 为幂函数且其在 $(0,+\\infty)$ 上是单调递增的, 并且在其定义域上是偶函数, 则 $m+n=( )$", "options": "A. 2\nB. 3\nC. 4\nD. 5", "subject": "代数", "analysis": "因为函数 $f(x)$ 为幂函数, 所以 $2 n-1=1$, 所以 $n=1$,\n\n又因为函数 $f(x)$ 在 $(0,+\\infty)$ 上是单调递增函数, 所以 $-m^{2}+2 m+3>0$,\n\n所以 $-1", "options": "A. $\\{3,4,7,8\\}$\nB. $\\{3,4,5,6,7,8\\}$\nC. $\\{1,2,9\\}$\nD. $\\{5,6\\}$", "subject": "组合数学", "analysis": "由题意, 全集 $I=\\{1,2,3,4,5,6,7,8,9\\}$, 集合 $A=\\{3,4,5,6\\}$, 集合 $B=\\{5,6,7,8\\}$,\n\n可得 $A \\cup B=\\{3,4,5,6,7,8\\}, A \\cap B=\\{5,6\\}$,\n\n所以 $C_{I}(A \\cap B)=\\{1,2,3,4,7,8,9\\}$,\n\n由图象可得阴影部分表示的集合为 $(A \\cup B) \\cap C_{I}(A \\cap B)=\\{3,4,7,8\\}$.\n\n故答案为: A.\n\n【分析】由图象可知阴影部分对应的集合为 $(A \\cup B) \\cap C_{I}(A \\cap B)$, 根据集合的运算, 即可求解."} {"id": "14989", "image": [], "answer": "B", "solution": "null", "level": "高一", "question": "若全集 $U=\\{x \\in N \\mid-210\\}$ 的是()", "options": "A. $(0,5) \\cup(10,+\\infty)$\nB. $\\lfloor 0,5) \\cup(10,+\\infty)$\nC. $(0,5] \\cup[10,+\\infty)$\nD. $\\lfloor 0,5\\rfloor \\cup(10,+\\infty)$", "subject": "算术", "analysis": "根据区间的定义可知数集 $A=\\{x \\mid 0 \\leq x<5$ 或 $x>10\\}$ 可以用区间 $[0,5) \\cup(10,+\\infty)$ 表示.\n\n故答案为: $\\mathrm{B}$.\nB.\n\n【分析】根据区间的定义,将集合 $A$ 表示为区间的形式,由此确定正确选项.\n\n4.答案: A\n\n解: $\\because C=\\{x \\mid x=m-n, m \\in A, n \\in B\\}$,\n\n$\\mathrm{A}=\\{4,5,6\\}, \\mathrm{B}=\\{1,2,3\\}$,\n\n$\\therefore C=\\{1,2,3,4,5\\}$,\n\n$\\therefore$ 集合 C 中的所有元素之和 $=1+2+3+4+5=15$.\n\n故答案为: A.\n\n【分析】由 $\\mathrm{C}=\\{\\mathrm{x} \\mid \\mathrm{x}=\\mathrm{m}-\\mathrm{n}, \\mathrm{m} \\in \\mathrm{A}, \\mathrm{n} \\in \\mathrm{B}\\}, \\mathrm{A}=\\{4,5,6\\}, \\mathrm{B}=\\{1,2,3\\}$, 先求出 $\\mathrm{C}$, 然后再求集合 C 中的所有元素之和."} {"id": "14990", "image": [], "answer": "D", "solution": "null", "level": "高一", "question": "1.已知集合 $A=\\left\\{x \\mid x^{2}-3 x-4<0\\right\\}, B=\\{-4,1,3,5\\}$ ,则 $A \\cap B=( )$", "options": "A. $\\{-4,1\\}$\\nB. $\\{1,5\\}$\\nc. $\\{3,5\\}$\\nD. $\\{1,3\\}$", "subject": "算术", "analysis": "解: 由 $x^{2}-3 x-4<0$ 解得 $-1c>b$\nC. $c>b>a$\nD. $b>a>c$", "subject": "算术", "analysis": "$\\because b=\\left(\\frac{1}{2}\\right)^{0.3}1$,\n\n$\\therefore \\mathrm{a}>\\mathrm{c}>\\mathrm{b}$,\n\n故答案为: B.\n\n【分析】首先根据指数函数判断出 $\\mathrm{b}, \\mathrm{c}$ 的大小,再和 1 进行比较得出。"} {"id": "15097", "image": [], "answer": "C", "solution": "null", "level": "高一", "question": "计算 $\\frac{a^{2}}{\\sqrt{a} \\cdot \\sqrt[3]{a^{2}}}$ 的结果为 ( )", "options": "A. $a^{\\frac{3}{2}}$\nB. $a^{\\frac{1}{2}}$\nC. $a^{\\frac{3}{6}}$\nD. $a^{\\frac{6}{5}}$", "subject": "算术", "analysis": "$\\frac{a^{2}}{\\sqrt{a} \\cdot \\sqrt[3]{a^{2}}}=a^{2} \\cdot a^{-\\frac{1}{2}} \\cdot a^{-\\frac{2}{3}}=a^{\\frac{5}{6}}$\n\n故答案为:\n\nC\n\n【分析】直接利用指数幂运算法则得到答案."} {"id": "15100", "image": [], "answer": "D", "solution": "null", "level": "高一", "question": "对任意的正实数 $\\mathrm{a}$ 及 $m, n \\in Q$, 下列运算正确的是 ( )", "options": "A. $\\left(a^{m}\\right)^{n}=a^{m+n}$\nB. $\\left(a^{m}\\right)^{n}=a^{m n}$\nC. $\\left(a^{m}\\right)^{n}=a^{m-n}$\nD. $\\left(a^{m}\\right)^{n}=a^{m n}$", "subject": "算术", "analysis": "根据指数的运算性质 $\\left(a^{m}\\right)^{n}=a^{m n}$ 排除 $\\mathrm{ABC}$.\n\n故答案为: D\n\n【分析】直接根据指数的运算性质即可得出答案."} {"id": "15102", "image": [], "answer": "B", "solution": "null", "level": "高一", "question": "$\\sqrt[3]{8}+\\sqrt[3]{-27}+\\sqrt{2 \\frac{1}{4}}=$", "options": "A. $\\frac{5}{2}$\nB. $\\frac{1}{2}$\nC. $\\frac{13}{2}$\nD. $-\\frac{7}{2}$", "subject": "算术", "analysis": "依题意, 原式 $=\\left(2^{3}\\right)^{\\frac{1}{3}}+\\left[(-3)^{3}\\right]^{\\frac{1}{3}}+\\left[\\left(\\frac{3}{2}\\right)^{2}\\right]^{\\frac{1}{2}}=2-3+\\frac{3}{2}=\\frac{1}{2}$.\n故答案为: B.\n\n【分析】根据根式运算公式以及指数运算公式, 化简所求表达式."} {"id": "15103", "image": [], "answer": "B", "solution": "null", "level": "高一", "question": "已知 $\\left(\\frac{1}{3}\\right)^{x}=27 \\times \\sqrt{3^{4 x}}$, 则 $\\mathrm{x}=(\\quad)$", "options": "A. 1\nB. -1\nC. 3\nD. -3", "subject": "算术", "analysis": "方程 $\\left(\\frac{1}{3}\\right)^{x}=27 \\times \\sqrt{3^{4 x}} \\Leftrightarrow 3^{-x}=3^{3} \\cdot 3^{2 x}=3^{2 x+3} \\Rightarrow-x=2 x+3 \\Rightarrow x=-1$\n\n故答案选: B\n\n【分析】将等式两侧同时化简成关于底数为 3 的指数型表达式, 求解对应的 $\\mathrm{x}$ 即可."} {"id": "15107", "image": [], "answer": "B", "solution": "null", "level": "高一", "question": "设 $a=e^{-\\frac{1}{2}}, b=4 e^{-2}, c=2 e^{-1}, d=3 e^{-\\frac{3}{2}}$, 则 $a, b, c, d$ 的大小关系为 $(\\quad)$", "options": "A. $c>b>d>a$\nB. $c>d>a>b$\nC. $c>b>a>d$\nD. $c>d>b>a$.", "subject": "算术", "analysis": "$a^{2}=\\frac{1}{e}=\\frac{e^{3}}{e^{4}}, \\quad b^{2}=\\frac{16}{e^{4}}, \\quad c^{2}=\\frac{4}{e^{2}}=\\frac{4 e^{2}}{e^{4}}, \\quad d^{2}=\\frac{9 e}{e^{4}}$,\n\n由于 $e \\approx 2.7, e^{2} \\approx 7.39, e^{3} \\approx 20.09$, 所以 $c>d>a>b$,\n\n故答案为: B.\n\n【分析】利用指数幂的运算性质化成同分母,再求出分子的近似值即可判断大小."} {"id": "15109", "image": [], "answer": "B", "solution": "null", "level": "高一", "question": "复利是一种计算利息的方法.即把前一期的利息和本金加在一起算作本金,再计算下一期的利息.某同学有压岁钱 1000 元, 存入银行, 年利率为 $2.25 \\%$; 若放入微信零钱通或者支付宝的余额宝, 年利率可达 $4.01 \\%$. 如果将这 1000 元选择合适方式存满 5 年, 可以多获利息()元. (参考数据 $1.0225^{4}=1.093,1.0225^{3}=1.117,1.0401^{4}=1.170,1.0401^{3}=1.217$ )", "options": "A. 176\nB. 100\nC. 77\nD. 88", "subject": "算术", "analysis": "由题意, 某同学有压岁钱 1000 元, 存入银行, 年利率为 $2.25 \\%$,\n\n若在银行存放 5 年, 可得金额为 $S_{1}=1000(1+2.25 \\%)^{3}=1117$ 元,即利息为 117 元,\n\n若放入微信零钱通或者支付宝的余额宝时, 利率可达 $4.01 \\%$,\n\n若存放 5 年, 可得金额为 $S_{1}=1000(1+4.01 \\%)^{3}=1217$ 元,即利息为 217 元,\n\n所以将这 1000 元选择合适方式存满 5 年, 可以多获利息 $217-117=100$ 元,\n\n故答案为: B。\n\n【分析】由题意, 某同学有压岁钱 1000 元, 分别计算存入银行和放入微信零钱通或者支付宝的余额宝所得利息, 即可得到答案。\n\n二、多选题"} {"id": "15116", "image": [], "answer": "D", "solution": "null", "level": "高一", "question": "不等式 $2^{2 x-7}<2^{4 x-1}$ 的解集是 ( )", "options": "A. $(-\\infty,-3)$\nB. $(-\\infty, 3)$\nC. $(3,+\\infty)$\nD. $(-3,+\\infty)$", "subject": "算术", "analysis": "因为 $y=2^{x}$ 在 $\\mathrm{R}$ 上是增函数, $2^{2 x-1}<2^{4 x-1}$,\n\n所以 $2 x-7<4 x-1$ ,即 $x>-3$\n\n所以不等式的解集是 $\\{x \\mid x>-3\\}$,\n\n故选 D.\n\n【分析】利用指数函数 $y=2^{x}$ 在 $\\mathrm{R}$ 上的单调性, 得出关于 $x$ 的不等式 $2 x-7<4 x-1$, 解此不等式, 从而得出不等式的解集;"} {"id": "15127", "image": [], "answer": "D", "solution": "null", "level": "高一", "question": "设 $y_{1}=4^{0.9}, y_{2}=8^{0.44}, y_{3}=\\left(\\frac{1}{2}\\right)^{-1.2}$,", "options": "A. $\\mathrm{y}_{3}>\\mathrm{y}_{1}>\\mathrm{y}_{2}$\nB. $\\mathrm{y}_{2}>\\mathrm{y}_{1}>\\mathrm{y}_{3}$\nC. $\\mathrm{y}_{1}>\\mathrm{y}_{2}>\\mathrm{y}_{3}$\nD. $\\mathrm{y}_{1}>\\mathrm{y}_{3}>\\mathrm{y}_{2}$", "subject": "算术", "analysis": "因为 $y_{1}=4^{0.9}=2^{1.8}, y_{2}=8^{0.44}=2^{1.32}, y_{3}=\\left(\\frac{1}{2}\\right)^{-1.5}=2^{1.5}, \\mathrm{y}=2^{x}$ 为单调递增函数,\n\n所以 $2^{1.8}>2^{1.5}>2^{1.32}$, 即 $\\mathrm{y}_{1}>\\mathrm{y}_{3}>\\mathrm{y}_{2}$,\n\n故答案为: D.\n\n【分析】根据条件化为底为 2 的指数, 再根据指数函数单调性确定大小"} {"id": "15144", "image": [], "answer": "A", "solution": "null", "level": "高一", "question": "${ }^{10^{\\lg 2}+\\lg 5+\\lg 2=(\\quad)}$", "options": "A. 3\nB. 2\nC. 1\nD. 0", "subject": "算术", "analysis": "$10^{\\lg 2}+\\lg 5+\\lg 2=2+\\lg 10=2+1=3$\n\n故答案为: A\n\n【分析】根据对数运算性质化简求值即可."} {"id": "15147", "image": [], "answer": "A", "solution": "null", "level": "高一", "question": "表达式 $\\lg 25+\\lg 2 \\lg 50+(\\lg 2)^{2}$ 的运算结果为 ( )", "options": "A. 2\n.B. 0\nC. 1\nD. $\\sqrt{2}$", "subject": "算术", "analysis": "$\\lg 25+\\lg 2 \\lg 50+(\\lg 2)^{2}=\\lg 25+\\lg 2(\\lg 50+\\lg 2)=\\lg 25+2 \\lg 2=\\lg 25+\\lg 4$\n\n$=\\lg 100=2$\n\n故答案为: A\n\n【分析】直接利用对数的运算法则得到答案."} {"id": "15163", "image": [], "answer": "B", "solution": "null", "level": "高一", "question": "若 $\\lg (2 x-4) \\leq 1$, 则 $x$ 的取值范围是( )", "options": "A. $(-\\infty, 7]$\nB. $(2,7]$\n.C. $(7,+\\infty)$\nD. $(2,+\\infty)$", "subject": "算术", "analysis": "$\\lg (2 x-4) \\leq 1$, 则满足: $\\left\\{\\begin{array}{l}2 x-4 \\leq 10 \\\\ 2 x-4>0 \\text { 解得 } 20\\end{array}\\right.$ 计算得到答案."} {"id": "15168", "image": [], "answer": "D", "solution": "null", "level": "高一", "question": "已知 $t>1, x=\\log _{2} t, y=\\log _{3} t, z=\\log _{5} t$, 则", "options": "A. $2 x<3 y<5 z$\nB. $5 z<2 x<3 y$\n.C. $3 y<5 z<2 x$\nD. $3 y<2 x<5 z$", "subject": "算术", "analysis": "由题意 $2 x=2 \\log _{2} t=\\log _{\\sqrt{2}} t, \\quad 3 y=3 \\log _{3} t=\\log _{\\sqrt[3]{3}} t \\quad 5 z=5 \\log _{5} t=\\log _{\\sqrt[5]{5}} t$,\n\n又 $\\sqrt{2}=2^{\\frac{1}{2}}=8^{\\frac{1}{6}}, \\sqrt[3]{3}=3^{\\frac{1}{3}}=9^{\\frac{1}{6}}$,\n\n易知 $2^{\\frac{1}{2}}<3^{\\frac{1}{3}}, 5^{\\frac{1}{5}}=25^{\\frac{1}{10}}, 2^{\\frac{1}{2}}=32^{\\frac{1}{10}}$, 即 $5^{\\frac{1}{5}}<2^{\\frac{1}{2}}$,\n\n$\\therefore 1<5^{\\frac{1}{5}}<2^{\\frac{1}{2}}<3^{\\frac{1}{3}}$, 又 $t>1, \\therefore 3 y<2 x<5 z$,\n\n故答案为:\nD.\n\n【分析】利用 $\\mathrm{t}$ 的取值范围结合对数函数的单调性和特殊值 1 对应的对数, 再利用 $2 \\mathrm{x}, 3 \\mathrm{y}, 5 \\mathrm{z}$ 与 1 的大小关系, 从而推出 $2 \\mathrm{x}, 3 \\mathrm{y}, 5 \\mathrm{z}$ 的大小关系。"} {"id": "15174", "image": [], "answer": "D", "solution": "null", "level": "高一", "question": "已知 $a=\\left(\\frac{2}{3}\\right)^{\\frac{1}{3}}, b=\\left(\\frac{3}{2}\\right)^{\\frac{1}{3}}, c=\\log _{3} \\frac{1}{2}$, 则()", "options": "A. $ca>0$\n\n而 $c=\\log _{3} \\frac{1}{2}<\\log _{3} 1=0$, 所以 $ca>0$, 再判断 $c<0$ 得到答案."} {"id": "16925", "image": [], "answer": "C", "solution": "null", "level": "高一", "question": "(2017・東阳市校级一模)如果输入 $n=2$, 那么执行如图中算法的结果是\n\n## 第一歩: 输入 $n$\n\n第二步: $n=n+1$\n\n第三步: $n=n+2$\n\n第四步: 输出 $n$", "options": "A. 输出 3\n\nB. 输出 4\n\nC. 输出 5\n\nD. 程序出错, 输不出任何结果", "subject": "算术", "analysis": "解: 第一步: 输入 $n=2$\n\n第二步: $n=2+1=3$\n\n第三步: $n=3+2=5$\n\n第四步: 输出 5\n\n故选: $C$."} {"id": "16938", "image": [], "answer": "D", "solution": "null", "level": "高一", "question": "(2020 秋・哈尔滨期末) 把 77 化成四进制数的末位数字为 ( )", "options": "A. 4\n B. 3\n C. 2\n D. 1", "subject": "算术", "analysis": "解: $\\because 77 \\div 4=19 \\cdots 1$\n\n$19 \\div 4=4 \\cdots 3$\n\n$4 \\div 4=1 \\cdots 0$\n\n$1 \\div 4=0 \\cdots 1$\n\n故 $77_{(10)}=1031_{(4)}$\n\n末位数字为 1 .\n\n故选: D."} {"id": "16942", "image": [], "answer": "B", "solution": "null", "level": "高一", "question": "(2020 秋・尖山区校级月考)秦九韶算法的先进性主要体现在减少运算次数, 下列说法正确的是", "options": "A. 可以减少加法运算次数\n\nB. 可以减少乘法运算次数\n\nC. 同时减少加法和乘法的运算次数\n\nD. 加法次数和乘法次数都有可能减少", "subject": "算术", "analysis": "解:由秦九韶算法中, 把一个 $n$ 次多项式的值转化为求 $n$ 个一次多项的值,\n\n即可以把至多 $\\frac{n(n+1)}{2}$ 次的乘法运算次数减少至多 $n$ 次计算, 而加法运算数不变.\n\n故选: $B$."} {"id": "16944", "image": [], "answer": "B", "solution": "null", "level": "高一", "question": "(2020 秋・让胡路区校级月考)下列各数转化成十进制后最大的数是()", "options": "A. $111111_{(2)}$\n B. 210 (6)\n C. 1000 (4)\n D. $81_{(9)}$", "subject": "算术", "analysis": "解: $111111_{(2)}=2^{5}+2^{4}+2^{3}+2^{2}+2^{1}+2^{0}=63$.\n\n$210{ }_{(6)}=2 \\times 6^{2}+1 \\times 6=78 ;$\n\n$1000(4)=1 \\times 4^{3}=64 ;$\n\n$81_{(9)}=8 \\times 9+1=73 ;$\n\n故 210 (6) 最大,\n\n故选: B."} {"id": "16946", "image": [], "answer": "A", "solution": "null", "level": "高一", "question": "(2018 秋・潞州区校级期末)用秦九韶算法计算多项式 $f(x)=4 x^{5}+5 x^{4}+6 x^{3}+7 x^{2}+8 x+1$ 当 $x=0.4$的值时, 需要做乘法和加法的次数分别是 ( )", "options": "5,5\n B. 4,5\n C. 4,4\n D. 5,4", "subject": "算术", "analysis": "根据题意最高次数为 5 次, 因此需做 5 次加法, 5 次乘法;\n\n故选: A."} {"id": "14966", "image": [], "answer": "C", "solution": "null", "level": "高一", "question": "方程组 $\\left\\{\\begin{array}{l}2 x+y=5 \\\\ x-y=1 \\text { 的解集不可以表示为 }(\\text { ) }\\end{array}\\right.$\n$\\left\\{(x, y) \\left\\lvert\\, \\begin{array}{c}2 x+y=5 \\\\ x-y=1\\end{array}\\right.\\right\\}$\nB. $\\left\\{(x, y) \\left\\lvert\\, \\begin{array}{c}x=2 \\\\ y=1\\end{array}\\right.\\right\\}$\nC. $\\{2,1\\}$\nD. $\\{(2,1)\\}$\n\n6.已知 $A=\\{x \\mid x \\leq 2 \\sqrt{3}, x \\in R\\}, a=\\sqrt{14}, b=2 \\sqrt{2}$, 则 ( )", "options": "A. $a \\in A$, 且 $b \\notin A$\nB. $a \\notin A$, 且 $b \\in A$\nC. $a \\in A$, 且 $b \\in A$\nD. $a \\notin A$, 且 $b \\notin A$", "subject": "解析几何", "analysis": "解: 由题意, 方程组 $\\left\\{\\begin{array}{l}2 x+y=5 \\\\ x-y=1\\end{array}\\right.$ 的解集所表示的集合应为点集, 根据集合的表示方法, 可得方程组的解集可表示为 $\\mathrm{A} 、 \\mathrm{~B} 、 \\mathrm{D}$ 的形式,\n\n而集合 $\\{2,1\\}$ 为两个元素的数集, 所以不正确,\n\n故答案为: C.\n\n【分析】由方程组 $\\left\\{\\begin{array}{l}2 x+y=5 \\\\ x-y=1\\end{array}\\right.$ 的解集所表示的集合应为点集, 根据集合的表示方法, 即作出判定,得到答案.\n\n6.答案: B\n\n解: $\\because A=\\{x \\mid x \\leq 2 \\sqrt{3}, x \\in R\\}, a=\\sqrt{14}, b=2 \\sqrt{2}$,\n\n由 $\\sqrt{14}>2 \\sqrt{3}$, 可得 $a \\notin A, 2 \\sqrt{2}<2 \\sqrt{3}$, 可得 $b \\in A$,\n\n故答案为: B.\n\n【分析】比较 $\\mathrm{a}, \\mathrm{b}$ 与 $^{2} \\sqrt{3}$ 的大小, 即可判断 $\\mathrm{a}, \\mathrm{b}$ 与集合 $\\mathrm{A}$ 的关系."} {"id": "15010", "image": [], "answer": "C", "solution": "null", "level": "高一", "question": "函数 $f(x)=2 x+\\sqrt{x-1}(1 \\leq x \\leq 5)$ 的值域是 $(\\quad)$", "options": "A. $(-\\infty, 12]$\nB. $(-\\infty, 2)$\nC. $[2,12]$\nD. $22,+\\infty)$", "subject": "解析几何", "analysis": "$f(x)=2 x+\\sqrt{x-1}(1 \\leq x \\leq 5)$, 设 $\\sqrt{x-1}=t \\therefore x=t^{2}+1, t \\in[0,2]$\n\n变换得到函数 $y=2 t^{2}+t+2=2\\left(t+\\frac{1}{4}\\right)^{2}+\\frac{15}{8}$ 在 $[0,2\\rfloor$ 单调递增.\n\n故 $y_{\\text {max }}=12, y_{\\text {min }}=2$, 即 $f(x)_{\\text {min }}=f(1)=2, f(x)_{\\text {max }}=f(5)=12$\n\n故答案为: $C$\n\n【分析】换元 $\\sqrt{x-1}=t$, 变换得到 $y=2 t^{2}+t+2$, 根据函数的单调性得到函数值域."} {"id": "15025", "image": [], "answer": "C", "solution": "null", "level": "高一", "question": "当 $x \\in[1 , 4]$ 时, 函数 $f(x)=x^{2}-3 x+2$ 的值域为 $()$", "options": "A. $\\lfloor 1,6\\rfloor$\nB. $\\lfloor 0,6\\rfloor$\nC. $\\left[-\\frac{1}{4}, 6\\right]$\nD. $\\left[-\\frac{5}{4}, 6\\right]$", "subject": "解析几何", "analysis": "由题, $f(x)=x^{2}-3 x+2=\\left(x-\\frac{3}{2}\\right)^{2}-\\frac{1}{4}$,\n\n因为 $x \\in[1,4]$, 则当 $x=\\frac{3}{2}$ 时, $f\\left(\\frac{3}{2}\\right)=f(x)_{\\text {min }}=-\\frac{1}{4}$;\n\n当 $x=4$ 时, $f(4)=f(x)_{\\text {max }}=\\left(4-\\frac{3}{2}\\right)^{2}-\\frac{1}{4}=6$;\n\n故答案为: C\n\n【分析】先利用配方法可得 $f(x)=\\left(x-\\frac{3}{2}\\right)^{2}-\\frac{1}{4}$, 则在 $x=\\frac{3}{2}$ 与 $x=4$ 时分别能取得最小值与最大值,即可得到值域."} {"id": "15044", "image": ["7494.jpg", "7495.jpg", "7496.jpg", "7497.jpg"], "answer": "C", "solution": "null", "level": "高一", "question": "函数 $y=-\\frac{x^{2}}{|x|}$ 的图象的大致形状是()", "options": "A.\n\n\n\nB.\n\n\n\nC.\n\n\n\nD.\n\n", "subject": "解析几何", "analysis": "方法一: $y=-\\frac{x^{2}}{|x|}=\\left\\{\\begin{array}{l}-x, x>0 \\\\ x, x<0\\end{array}\\right.$,\n\n观察选项, C 符合;\n\n方法二: $y=-\\frac{x^{2}}{|x|}<0$, 故图像全部在 $x$ 轴下方, 只有 C 符合,\n\n故答案为: C.\n\n【分析】通过分类讨论去绝对值, 得到函数的解析式的分段形式, 再观察图像即可得结果."} {"id": "15045", "image": [], "answer": "A", "solution": "null", "level": "高一", "question": "将函数 $y=2(x+1)^{2}-3$ 的图像向右平移 1 个单位长度, 再向上平移 3 个单位长度, 所得的图像所对应的函数解析式为 ( )", "options": "A. $y=2 x^{2}$\nB. $y=2 x^{2}-6$\nC. $y=2(x+2)^{2}-6$\nD. $y=2(x+2)^{2}$", "subject": "解析几何", "analysis": "设 $\\mathrm{f}(\\mathrm{x})=2(\\mathrm{x}+1)^{2}-3$, 得函数 $\\mathrm{y}=2(\\mathrm{x}+1)^{2}-3$ 的图象向右平移 1 个单位长度,\n\n得到的图象对应函数解析式为: $y=f(x-1)=2[(x+1)-1]^{2}-3=2 x^{2}-3$,\n\n再将所得图象向上平移 3 个单位长度,\n\n得到的图象对应函数表达式为: $y=f(x-1)+3=2 x^{2}-3+3=2 x^{2}$,\n\n即最终得到的图象对应函数解析式为: $y=2 x^{2}$\n\n故答案为: A\n\n【分析】函数 $\\mathrm{y}=\\mathrm{f}(\\mathrm{x})$ 图象向右平移 1 个单位长度, 得到图象对应的解析式为: $\\mathrm{y}=\\mathrm{f}(\\mathrm{x}-1)$, 然后再将所得图象向上平移 3 个单位长度, 得到的图象对应函数表达式为: $y=f(x-1)+3$. 依此规律代入题\n中函数解析式。"} {"id": "15046", "image": [], "answer": "A", "solution": "null", "level": "高一", "question": "具有性质 $f\\left(\\frac{1}{x}\\right)=-f(x)$ 的函数, 我们称为满足 “倒负” 变换的函数, 给出下列函数:\n\n$f(x)=x-\\frac{1}{x}$\n\n(2) $f(x)=x+\\frac{1}{x}$\n\n$f(x)=\\left\\{\\begin{array}{c}x, 01\\end{array}\\right.$", "options": "A. (1)(3)\nB. (2)(3)\nC. (1)(2)(3)\nD. (1)(2)", "subject": "解析几何", "analysis": "对于(1): $f\\left(\\frac{1}{x}\\right)+f(x)=\\frac{1}{x}-\\frac{1}{\\frac{1}{x}}+x-\\frac{1}{x}=\\frac{1}{x}-x+x-\\frac{1}{x}=0$, 满足题意;\n\n对于(2): $f\\left(\\frac{1}{x}\\right)+f(x)=\\frac{1}{x}+\\frac{1}{\\frac{1}{x}}+x+\\frac{1}{x}=\\frac{1}{x}+x+x+\\frac{1}{x}=2 x+\\frac{2}{x}$, 不满足题意;\n\n$f\\left(\\frac{1}{x}\\right)=\\left\\{\\begin{array}{c}\\frac{1}{x}, 0<\\frac{1}{x}<1 \\\\ 0, \\frac{1}{x}=1 \\\\ -x, \\frac{1}{x}>1\\end{array}=\\left\\{\\begin{array}{cc}\\frac{1}{x}, & x>1 \\\\ 0, & x=1 \\\\ -x, & 0\n\nC.\n\n\n\nB.\n\n\n\nD.\n\n", "subject": "解析几何", "analysis": "由函数的解析式可得: $f(-x)=\\frac{-4 x}{x^{2}+1}=-f(x)$,\n\n则函数 $f(x)$ 为奇函数, 其图象关于坐标原点对称, $\\mathrm{CD}$ 不符合题意;\n\n当 $x=1$ 时, $y=\\frac{4}{1+1}=2>0, \\mathrm{~B}$ 不符合题意.\n故答案为: A.\n\n【分析】由题意首先确定函数的奇偶性, 然后考查函数在特殊点的函数值排除错误选项即可确定函数的图象."} {"id": "15423", "image": ["7658.jpg"], "answer": "C", "solution": "null", "level": "高一", "question": "三条两两相交的直线最多可确定的平面的个数为( )", "options": "A. 1\nB. 2\nC. 3\nD. 无数", "subject": "解析几何", "analysis": "在空间中,两两相交的三条直线最多可以确定 3 个平面,如图所示:\n\n\n\n$P A , P B , P C$ 相交于一点 $P$ ,则 $P A , P B , P C$ 不共面,则 $P A , P B$ 确定一个平面 $P A B , P B$ , $P C$ 确定一个平面 $P B C , P A , P C$ 确定一个平面 $P A C$.故选 C."} {"id": "15108", "image": [], "answer": "A", "solution": "null", "level": "高一", "question": "若 $0<\\mathrm{a}<1, \\mathrm{~b}>0$ 则函数 $\\mathrm{f}(\\mathrm{x})=\\mathrm{a}^{\\mathrm{x}}+\\mathrm{b}$ 的图象一定经过 ( )", "options": "A. 第一、二象限\n.B. 第二、四象限\nc. 第一、二、四象限\n。D. 第二、三、四象限", "subject": "解析几何", "analysis": "因为 $0<\\mathrm{a}<1$, 则函数 $\\mathrm{f}(\\mathrm{x})=\\mathrm{a}^{\\mathrm{x}}$ 单调递减, 过一、二象限,\n\n又因为 $\\mathrm{b}>0$, 把图像向上平移, 即函数图象一定过第二象限,\n\n故选 A.\n\n【分析】利用指数函数的图象与性质, 得到当 $0<\\mathrm{a}<1$, 函数 $\\mathrm{f}(\\mathrm{x})=\\mathrm{a}^{\\mathrm{x}}$ 单调递减且过一、二象限, 再把图象向上平移即可得结果."} {"id": "15122", "image": [], "answer": "D", "solution": "null", "level": "高一", "question": "函数 $y=a^{x-2}+4(a>0$ 且 $a \\neq 1)$ 的图象必经过点()", "options": "A. $(0,1)$\nB. $(1,1)$\nC. $(2,4)$\nD. $(2,5)$", "subject": "解析几何", "analysis": "当 $x=2$ 时, $y=5$, 故函数图像必经过点 $(2,5)$.\n\n故答案为: $\\mathrm{D}$.\n\n【分析】根据指数 $a^{0}=1$ 直接计算得到定点."} {"id": "15123", "image": ["7519.jpg", "7520.jpg", "7521.jpg", "7522.jpg"], "answer": "A", "solution": "null", "level": "高一", "question": "函数 $f(x)=a^{x}$ 与 $g(x)=-x+2$ 在同一坐标系中的图象可能是 $(\\quad)$", "options": "A.\n\n\n\nC.\n\n\nB.\n\n\n\nD.\n\n", "subject": "解析几何", "analysis": "由函数 $g(x)=-x+2$ 易知 C、D 选项不正确,\n\n对于 A、B 选项可知函数 $f(x)=a^{x}$ 为 $a>1$ 时的图象,\n\n但函数 $f(x)=a^{x}$ 的图象过 $(0,1)$ 点,\n\n而函数 $g(x)=-x+2$ 的图象过 $(0,2)$ 点,\n所以 $\\mathrm{B}$ 选项不正确, $\\mathrm{A}$ 选项正确.\n\n故答案为: A.\n\n【分析】先根据 $g(x)=-x+2$ 的图象排除选项, 再利用函数 $f(x)=a^{x}$ 的图象排除剩余选项即可."} {"id": "15150", "image": [], "answer": "A", "solution": "null", "level": "高一", "question": "函数 $f(x)=\\frac{\\sqrt{-x^{2}-3 x+4}}{\\lg (x+1)}$ 的定义域为 $(\\quad)$", "options": "A. $(-1,0) \\cup(0,1]$\nB. $(-1,1]$\nC. $(-4,1]$\nD. $(-4,0) \\cup(0,1]$", "subject": "解析几何", "analysis": "要使函数有意义, 则 $\\left\\{\\begin{array}{l}-x^{2}-3 x+4 \\geq 0 \\\\ x+1>0 \\\\ x+1 \\neq 1\\end{array}\\right.$,\n\n解得 $-10, a \\neq 1)$ 必过点 $(1,0)$,\n\n所以当 $x+2=1$, 即 $x=-1$ 时, 有 $y=0+1=1$,\n\n所以函数 $y=\\log _{a}(x+2)+1_{\\text {必过点 }}(-1,1)$.\n\n故答案为: D\n\n【分析】根据对数函数 $y=\\log _{a} x(a>0, a \\neq 1)$ 必过点 $(1,0)$, 即可确定相应函数图象所过定点坐标."} {"id": "15170", "image": [], "answer": "D", "solution": "null", "level": "高一", "question": "若幂函数 $f(x)$ 的图象过点 $(16,8)$, 则 $f(x)0$,\n\n又函数 $f(x)$ 在定义域是 $[0,+\\infty)$,\n\n且 $f(x)$ 在 $[0,+\\infty)$ 递增, 故 $0 \\leq x1$.\n\n故答案为: $\\mathrm{D}$.\n\n【分析】先根据幂函数 $f(x)$ 的图象过点 $(16,8)$ 求出 $\\alpha=\\frac{3}{4}>0$, 再根据幂函数的单调性得到 $00$ 时, 幂函数 $\\mathrm{y}=\\mathrm{x}^{\\mathrm{n}}$ 是增函数; (5)当 $\\mathrm{n}<0$ 时, 幂函数在第一象限内的函数值随 $\\mathrm{x}$ 的值增大而减小。其中正确的是()", "options": "A. (1)和(4)\nB. (4)和(5)\nC. (2)和(3)\nD. (2)和(5)", "subject": "解析几何", "analysis": "当 $y=x^{-1}$ 时, 不过 $(0,0)$ 点, (1)错误;\n\n当 $x>0$ 时, $y>0$, 故幂函数的图象不可能在第四象限内, 故(2)对;\n\n当 $n=0$ 时, $y=x^{n}$ 中 $x \\neq 0$, 故其图象是去掉 $(0,0)$ 点的一条直线, (3)错;\n\n$y=x^{2}$ 在 $(-\\infty, 0)$ 上是减函数, $(0,+\\infty)$ 上是增函数, (4)错;\n\n幂函数 $y=x^{n}$, 当 $\\mathrm{n}<0$ 时, 在第一象限内函数值随 $x$ 值的增大而减小, (5)对.\n\n故答案为: D\n\n【分析】利用幕函数的图象特征结合幕函数的单调性, 从而找出正确命题的序号。"} {"id": "15172", "image": [], "answer": "A", "solution": "null", "level": "高一", "question": "已知幂函数 $f(x)=\\left(m^{2}-2 m+1\\right) x^{m 2}+m-2$ 的图象不过原点, 则 $m$ 的值为 ( )", "options": "A. 0\nB. -1\nC. 2\nD. 0 或 2", "subject": "解析几何", "analysis": "$\\because$ 函数是幂函数,\n$\\therefore m^{2}-2 m+1=1$, 解得: $m=2$ 或 $m=0$,\n\n当 $m=2$ 时, $f(x)=x^{4}$, 过原点, 不满足条件;\n\n当 $m=0$ 时, $f(x)=x^{-2}$, 不过原点, 满足条件,\n\n$\\therefore m=0$.\n\n故答案为: A.\n\n【分析】根据函数是幂函数可知 $m^{2}-2 m+1=1$, 得出: $m=2$ 或 $m=0$, 然后验证, 得到 $m$ 的值."} {"id": "15186", "image": [], "answer": "B", "solution": "null", "level": "高一", "question": "已知幂函数 $f(x)=x^{n}$ 的图象过点 $\\left(8, \\frac{1}{4}\\right)$, 且 $f(a+1)3$, 解得 $a<-4$ 或 $a>2$.\n\n故答案为: B.\n\n【分析】求出幂函数解析式, 根据函数单调性求解不等式."} {"id": "15188", "image": [], "answer": "B", "solution": "null", "level": "高一", "question": "已知幂函数 $y=f(x)$ 的图象过点 $\\left(\\frac{1}{2}, \\frac{\\sqrt{2}}{2}\\right)$, 则 $\\log _{4} f(2)$ 的值为 ( )", "options": "A. $-\\frac{1}{4}$\nB. $\\frac{1}{4}$\nC. -2\nD. 2", "subject": "解析几何", "analysis": "设幕函数的表达式为 $f(x)=x^{n}$, 则 $\\left(\\frac{1}{2}\\right)^{n}=\\frac{\\sqrt{2}}{2}$, 解得 $n=\\frac{1}{2}$,\n\n所以 $f(x)=x^{\\frac{1}{2}}$, 则 $\\log _{4} f(2)=\\log _{4} 2^{\\frac{1}{2}}=\\frac{1}{2} \\log _{2} 2^{\\frac{1}{2}}=\\frac{1}{2} \\times \\frac{1}{2}=\\frac{1}{4}$.\n\n故答案为: B,\n\n【分析】利用幕函数图象过点 $\\left(\\frac{1}{2}, \\frac{\\sqrt{2}}{2}\\right)$ 可以求出函数解析式, 然后求出 $\\log _{4} f(2)$ 即可。"} {"id": "17014", "image": [], "answer": "B", "solution": "null", "level": "高一", "question": "(2020 秋・齐齐哈尔期末)在区间 $[-1,2]$ 上任取一个实数 $k$, 则直线 $y=k(x-\\sqrt{5})$ 与圆 $x^{2}+y^{2}=1$相交的概率为()", "options": "A. $\\frac{3}{4}$\n B. $\\frac{1}{3}$\n C. $\\frac{2}{3}$\n D. $\\frac{1}{2}$", "subject": "解析几何", "analysis": "解:圆 $x^{2}+y^{2}=1$ 的圆心为 $(0,0)$,\n\n圆心到直线 $y=k(x-\\sqrt{5})$ 的距离为 $\\frac{|\\sqrt{5} k|}{\\sqrt{k^{2}+1^{2}}}$,\n\n要使直线 $y=k(x-\\sqrt{5})$ 与圆 $x^{2}+y^{2}=1$ 相交, 则 $\\frac{|\\sqrt{5} k|}{\\sqrt{k^{2}+1^{2}}}<1$, 解得 $-\\frac{1}{2}", "options": "A. 没有公切线\nB. 只有一条\nC. 恰好有两条\nD. 有超过两条", "subject": "解析几何", "analysis": "解: 由程序语言知, 圆心坐标 $(m n, 2 m n)$ 在直线 $y=2 x$ 上, 圆的半径为 $|m| n$,\n$\\therefore$ 相邻两圆半径之差为 $|m|$, 相邻两圆圆心距均为\n\n$d=\\sqrt{[m(n+1)-m n]^{2}+[2 m(n+1)-2 m n]^{2}}=\\sqrt{5}|m|$,\n\n$\\therefore$ 这 2018 个圆的公切线恰好有两条,是外公切线;\n\n故选: $C$."} {"id": "14361", "image": ["7308.jpg", "7309.jpg", "7310.jpg", "7311.jpg"], "answer": "A", "solution": "null", "level": "高一", "question": "函数 $y=2^{x}-x^{2}$ 的图象大致是 ()", "options": "A.\n\n\nB.\n\n\nC.\n\n\nD.\n\n", "subject": "解析几何", "analysis": "\n\n因为 2、 4 是函数的零点, 所以排除 B、C;\n\n因为 $x=-1$ 时 $y<0$, 所以排除 $\\mathrm{D}$, 故选 $\\mathrm{A}$"} {"id": "14373", "image": [], "answer": "A", "solution": "null", "level": "高一", "question": "函数 $y=x^{-2}$ 在区间 $\\left[\\frac{1}{2}, 2\\right]$ 上的最小值是 ( )", "options": "A. $\\frac{1}{4}$\nB. $-\\frac{1}{4}$\nC. 4\nD. -4", "subject": "解析几何", "analysis": "【详解】\n\n$\\because$ 函数 $y=x^{-2}$ 在区间 $\\left[\\frac{1}{2}, 2\\right]$ 上是减函数,\n\n$\\therefore y_{\\text {min }}=2^{-2}=\\frac{1}{4}$,\n\n故选: A.\n\n【点睛】\n\n此题考查由函数的单调性求最值, 属于基础题"} {"id": "14375", "image": [], "answer": "D", "solution": "null", "level": "高一", "question": "已知点 $\\left(2, \\frac{1}{4}\\right)$ 在幂函数 $y=f(x)$ 的图象上, 设 $a=f\\left(\\log _{0.5} 0.3\\right), b=f\\left(0.5^{0.3}\\right), c=f\\left(0.3^{0.5}\\right)$, 则 $a, b, c$ 的大小关系是 ( )", "options": "A. $b1,0<0.5^{0.3}<1,0<0.3^{0.5}<1$, 且 $0.0 .3^{0.5}<0.3^{0.3}<0.5^{0.3}$,\n\n所以 $0.3^{0.5}<0.5^{0.3}<\\log _{0.5} 0.3$, 所以 $a0, f(1.25)<0$, 则方程的根落在区间 ( )", "options": "A. $(1,1.25)$\nB. $(1.25,1.5)$\nC. $(1.5,2)$\nD. 不能确定", "subject": "解析几何", "analysis": "$\\because f(x)=3^{x}+3 x-8$\n\n又 $\\because f(1.5)>0, f(1.25)<0$\n\n$\\therefore \\quad f(1.5) \\cdot f(1.25)<0$\n\n由零点存在定理可得 $f(x)$ 在区间 $(1.25,1.5)$ 存在零点.\n\n$\\therefore 3^{x}+3 x-8=0$ 方程的根落在区间 $(1.25,1.5)$\n\n故选: $B$."} {"id": "15208", "image": [], "answer": "C", "solution": "null", "level": "高一", "question": "若函数 $f(x)=\\ln x-\\frac{1}{x}+a$ 在区间 $(1, e)$ 上存在零点, 则常数 $a$ 的取值范围为 $(\\quad)$", "options": "A. $00$,\n\n可得 $\\frac{1}{e}-10$, 解不等式即可求得 $\\mathrm{a}$ 的取值范围."} {"id": "15211", "image": [], "answer": "C", "solution": "null", "level": "高一", "question": "已知定义在 $R$ 上的函数 $f(x)$ 的图像是连续的, 且有如下对应值表, 那么 $f(x)$ 一定存在零点的区间是 ( )\n\n| $x$ | 1 | 2 | 3 |\n| :--- | :--- | :--- | :--- |\n| $f(x)$ | 5.1 | 4.2 | -3.3 |", "options": "A. $(-\\infty, 1)$\nB. $(1,2)$\nC. $(2,3)$\nD. $(3,+\\infty)$", "subject": "解析几何", "analysis": "因为 $f(1)=5.1>0, f(2)=4.2>0, f(3)=-3.3<0$,\n\n所以 $f(2) \\cdot f(3)<0$,\n\n又函数 $f(x)$ 在 $R$ 上连续, 由函数零点存在定理, 可得: $f(x)$ 在区间 $(2,3)$ 上必有零点.\n\n故答案为: C\n\n【分析】根据函数零点存在定理, 结合题中数据, 即可得出结果."} {"id": "14297", "image": [], "answer": "C", "solution": "null", "level": "高一", "question": "已知集合 $M=\\{(x, y) \\mid x+y \\leq 1, x \\in \\boldsymbol{Z}, y \\in \\mathbf{Z}\\}, N=\\left\\{(x, y) \\mid x^{2}+y^{2} \\leq 2\\right\\}$, 则集合 $M \\cap N$ 中元素的个数是 ( )", "options": "A. 6\nB. 7\nC. 8\nD. 9", "subject": "解析几何", "analysis": "【分析】\n\n先由 $N$ 中的不等式求得 $x, y$ 的取值范围, 再列举出其中的整点, 然后检验是否满足 $M$ 中的不等式, 即得到交集中的元素个数.\n\n## 【详解】\n\n由 $x^{2}+y^{2} \\leq 2$ 可得, $x^{2} \\leq 2, y^{2} \\leq 2$, 即 $|x| \\leq \\sqrt{2},|y| \\leq \\sqrt{2}$,\n\n$N$ 中的满足 $x \\in Z, y \\in Z$ 的整点有:\n\n$(0,0),(0,1),(0,-1),(-1,0),(1,0),(1,1),(1,-1),(-1,1),(-1,-1)$, 共 9 个点,\n\n其中只有 $(1,1)$ 这一个点不满足 $x+y \\leq 1$,\n\n故 $M \\cap N$ 中的元素个数为 8 个,\n\n故选: C.\n\n## 【点睛】\n\n本题考查集合的交集,关键是寻找 $M$ 中同时符合 $N$ 中的条件的元素."} {"id": "14317", "image": [], "answer": "C", "solution": "null", "level": "高一", "question": "若函数 $f(x)=\\ln x-\\frac{1}{x}+a$ 在区间 $(1, e)$ 上存在零点, 则常数 $a$ 的取值范围为 $(\\quad)$", "options": "A. $00$,\n\n可得 $\\frac{1}{e}-10$, 解不等式即可求得 $\\mathrm{a}$ 的取值范围."} {"id": "14318", "image": [], "answer": "B", "solution": "null", "level": "高一", "question": "已知函数 $f(x)=\\left\\{\\begin{array}{ll}\\log _{2}(1-x), & x \\leq 0 \\\\ -x^{2}+4 x, & x>0,\\end{array}\\right.$ 则函数 $g(x)=f\\lfloor f(x)\\rfloor-1$ 的零点个数为 $(\\quad)$", "options": "A. 4\nB. 7\nC. 8\nD. 9", "subject": "解析几何", "analysis": "令 $f(x)=t$, 则可得 $f(t)=1$,\n\n当 $t \\leq 0$ 时, 即可得 $\\log _{2}(1-t)=1$, 解得 $t=-1$;\n\n当 $t>0$ 时, 即可得 $-t^{2}+4 t=1$, 解得 $t=2^{ \\pm} \\sqrt{3}$.\n\n则 $f(x)=-1$, 或 $f(x)=2+\\sqrt{3}$, 或 $f(x)=2-\\sqrt{3}$\n\n当 $x \\leq 0$ 时,\n\n令 $\\log _{2}(1-x)=-1$, 解得 $x=\\frac{1}{2}$, 不满足题意;\n令 $\\log _{2}(1-x)=2+\\sqrt{3}$, 解得 $x=1-2^{2+\\sqrt{3}} \\leq 0$, 满足题意;\n\n令 $\\log _{2}(1-x)=2-\\sqrt{3}$, 解得 $x=1-2^{2-\\sqrt{3}} \\leq 0$, 满足题意.\n\n当 $x>0$ 时,\n\n令 $-x^{2}+4 x=-1$, 解得 $x=2+\\sqrt{5}$ 或 $x=2-\\sqrt{5}$ (舍);\n\n令 $-x^{2}+4 x=2+\\sqrt{3}$, 整理得 $x^{2}-4 x+2+\\sqrt{3}=0$,\n\n解得 $x=2+\\frac{\\sqrt{6}-\\sqrt{2}}{2}$ 或 $x=2-\\frac{\\sqrt{6}-\\sqrt{2}}{2}$ 满足题意;\n\n令 $-x^{2}+4 x=2-\\sqrt{3}$, 整理得 $x=2+\\frac{\\sqrt{2}+\\sqrt{6}}{2}$ 或 $x=2-\\frac{\\sqrt{2}+\\sqrt{6}}{2}$ 满足题意.\n\n综上所述, 函数零点有 $1-2^{2+\\sqrt{3}}, 1-2^{2-\\sqrt{3}}, 2+\\sqrt{5}, 2 \\pm \\frac{\\sqrt{6}-\\sqrt{2}}{2}, 2 \\pm \\frac{\\sqrt{2}+\\sqrt{6}}{2}$\n\n共计7个.\n\n故答案为: B\n\n【分析】令 $f(x)=t$, 求得 $f(t)=1$ 的根, 再求 $f(x)=t$ 的根, 则问题得解."} {"id": "14327", "image": [], "answer": "D", "solution": "null", "level": "高一", "question": "已知函数 $f(x)$ 的定义域为 $R$, 图象恒过 $(0,1)$ 点, 对任意 $x_{1}, x_{2} \\in R$ 当 $x_{1} \\neq x_{2}$ 时, 都有 $\\frac{f\\left(x_{1}\\right)-f\\left(x_{2}\\right)}{x_{1}-x_{2}}>1$,则不等式 $f\\left[\\ln \\left(e^{x}-1\\right)\\right]<1+\\ln \\left(e^{x}-1\\right)$ 的解集为 ( )", "options": "A. $(\\ln 2,+\\infty)$\nB. $(-\\infty, \\ln 2)$\nC. $(\\ln 2,1)$\nD. $(0, \\ln 2)$", "subject": "解析几何", "analysis": "因为 $\\frac{f\\left(x_{1}\\right)-f\\left(x_{2}\\right)}{x_{1}-x_{2}}>1$, 不妨设 $x_{1}>x_{2}$, 则 $f\\left(x_{1}\\right)-x_{1}>f\\left(x_{2}\\right)-x_{2}$,\n令 $g(x)=f(x)-x$, 在 $\\mathrm{R}$ 上递增,\n\n又 $f(0)=1$, 所以不等式 $f\\left[\\ln \\left(e^{x}-1\\right)\\right]<1+\\ln \\left(e^{x}-1\\right)$,\n\n即为 $f\\left[\\ln \\left(e^{x}-1\\right)\\right]-\\ln \\left(e^{x}-1\\right)<1=f(0)-0$, 即 $g\\left[\\ln \\left(e^{x}-1\\right)\\right]", "options": "A. $(P \\cup Q) \\cap C_{U} S$\nB. $(P \\cap Q) \\cup C_{U} S$\nC. $(P \\cap Q) \\cap C_{U} S$\nD. $(P \\cup Q) \\cup C_{U} S$", "subject": "组合几何学", "analysis": "图中阴影是集合 $P, Q$ 的公共部分,但不包含集合 $S$ 中的元素,\n\n$\\therefore$ 阴影部分表示为 $(P \\cap Q) \\cap C_{U} S$.\n\n故答案为:\n\n【分析】根据图中阴影部分直接求集合的运算."} {"id": "14973", "image": [], "answer": "A", "solution": "null", "level": "高一", "question": "用 $\\mathrm{d}(\\mathrm{A})$ 表示集合 $\\mathrm{A}$ 中的元素个数, 若集合 $\\mathrm{A}=\\{0,1\\}, B=\\left\\{x \\mid\\left(x^{2}-a x\\right) \\quad\\left(x^{2}-a x+1\\right)=0\\right\\}$, 且 $\\mid d(A)$ $-d ( B ) \\mid=1$. 设实数 $\\mathrm{a}$ 的所有可能取值构成集合 $\\mathrm{M}$ ,则 $\\mathrm{d}(\\mathrm{M})=(\\mathrm{)}$", "options": "A. 3\nB. 2\nC. 1\nD. 4", "subject": "度量几何学", "analysis": "由题意, $|\\mathrm{d}(\\mathrm{A})-\\mathrm{d}(\\mathrm{B})|=1, \\mathrm{~d}(\\mathrm{~A})=2$, 可得 $\\mathrm{d}(\\mathrm{B})$ 的值为 1 或 3\n\n若 $d(B)=1$, 则 $x^{2}-a x=0$ 仅有一根, 必为 0 , 此时 $a=0$, 则 $x^{2}-a x+1=x^{2}+1=0$ 无根, 符合题意\n\n若 $d(B)=3$, 则 $x^{2}-a x=0$ 有一根, 必为 0 , 此时 $a=0$, 则 $x^{2}-a x+1=x^{2}+1=0$ 无根, 不合题意\n\n故 $\\mathrm{x}^{2}-\\mathrm{ax}=0$ 有二根, 一根是 0 , 另一根是 $\\mathrm{a}$, 所以 $\\mathrm{x}^{2}-\\mathrm{ax}+1=0$ 必仅有一根,\n\n所以 $\\triangle=a^{2}-4=0$, 解得 $\\mathrm{a}= \\pm 2$, 此时 $\\mathrm{x}^{2}-\\mathrm{ax}+1=0$ 为 1 或 -1 , 符合题意,\n\n综上实数 $\\mathrm{a}$ 的所有可能取值构成集合 $\\mathrm{M}=\\{0,-2,2\\}, \\mathrm{D}(\\mathrm{M})=3$.\n\n故答案为: A.\n\n【分析】先由已知可得元素个数 $\\mathrm{d}$ (B) 的值为 1 或 3 , 再分别判断是否符合题意, 即可求出集合 M 得结果."} {"id": "15028", "image": ["7481.jpg", "7482.jpg", "7483.jpg", "7484.jpg"], "answer": "C", "solution": "null", "level": "高一", "question": "设 $x \\in R$, 定义符号函数 $\\operatorname{sgn} x=\\left\\{\\begin{array}{l}1, x>0 \\\\ 0, x=0 \\\\ -1, x<0\\end{array}\\right.$, 则函数 $f(x)=|x| \\operatorname{sgn} x$ 的图象大致是 ( )", "options": "A.\n\n\n\nC.\n\n\n\n.B.\n\n\n\nD.\n\n", "subject": "度量几何学", "analysis": "函数 $\\mathrm{f}(\\mathrm{x})=|\\mathrm{x}| \\operatorname{sgn} \\mathrm{x}=\\left\\{\\begin{array}{l}x, x>0 \\\\ 0, x=0 \\\\ x, x<0=\\mathrm{x},\\end{array}\\right.$\n\n故函数 $\\mathrm{f}(\\mathrm{x})=|\\mathrm{x}| \\operatorname{sgn} \\mathrm{x}$ 的图象为 $\\mathrm{y}=\\mathrm{x}$ 所在的直线。\n\n故答案为: C。\n\n【分析】利用符号函数的定义结合分段函数图象的画法找出函数 $f(x)=|x| \\operatorname{sgn} x$ 的大致图象。"} {"id": "15438", "image": ["7669.jpg"], "answer": "A", "solution": "null", "level": "高一", "question": "已知在空间四边形 $A B C D$ 中, $M, N$ 分别是 $A B, C D$ 的中点, 且 $A C=4, B D=6$, 则 ( )", "options": "A. $1"} {"id": "15466", "image": [], "answer": "C", "solution": "null", "level": "高一", "question": "下列四个说法中正确的是( )", "options": "平面 $\\alpha$ 内有无数个点到平面 $\\beta$ 的距离相等, 则 $\\alpha / / \\beta$\n\nB. $\\alpha \\cap \\gamma=a, \\alpha \\cap \\beta=b$, 且 $a / / b(\\alpha, \\beta, \\gamma$ 分别表示平面, $a, b$ 表示直线), 则 $\\gamma / / \\beta$\n\nC. 平面 $\\alpha$ 内一个三角形三边分别平行于平面 $\\beta$ 内的一个三角形的三条边, 则 $\\alpha / / \\beta$\n\nD. 平面 $\\alpha$ 内的一个平行四边形的两边与平面 $\\beta$ 内的一个平行四边形的两边对应平行, 则 $\\alpha / / \\beta$", "subject": "度量几何学", "analysis": "由面面平行的判定定理知 C 正确."} {"id": "15517", "image": ["7729.jpg"], "answer": "D", "solution": "null", "level": "高一", "question": "下列不能确定两个平面垂直的是( )", "options": "A. 两个平面相交, 所成二面角是直二面角\n\nB. 一个平面垂直于另一个平面内的一条直线\n\nC. 一个平面经过另一个平面的一条垂线\n\nD. 平面 $\\alpha$ 内的直线 $a$ 垂直于平面 $\\beta$ 内的直线 $b$", "subject": "度量几何学", "analysis": "如图所示,在正方体 $A B C D-A_{1} B_{1} C_{1} D_{1}$ 中,平面 $A_{1} B_{1} C D$ 内的直线 $A_{1} B_{1}$ 垂直于平面 $A B C D$ 内的一条直线 $B C$ ,但平面 $A_{1} B_{1} C D$ 与平面 $A B C D$ 显然不垂直.\n\n"} {"id": "17013", "image": ["8770.jpg"], "answer": "C", "solution": "null", "level": "高一", "question": "(2020 春・荔湾区期末)七巧板是中国古代劳动人民的发明,在 18 世纪,七巧板流传到了国外,被誉为 “东方魔板” , 至今英国剑桥大学的图书馆里还珍藏着一部《七巧新谱》. 完整图案为一正方形(如图)五块等腰直角三角形、一块正方形和一块平行四边形, 如果向此正方形内部随机撒 800 个点, 则估计落在阴影部分的点的个数约为()\n\n", "options": "A. 250\nB. 300\nC. 350\nD. 400", "subject": "度量几何学", "analysis": "解: 根据题意, 设大正方形的边长为 4 , 则面积 $S=4 \\times 4=16$,\n\n阴影部分的面积 $S^{\\prime}=\\frac{1}{2} S-\\left(\\frac{1}{2} \\times \\sqrt{2} \\times \\sqrt{2}\\right)=7$,则在此正方形中随机取一点, 那么此点取自阴影部分的概率 $P=\\frac{7}{16}$,如果向此正方形内部随机撒 800 个点, 则估计落在阴影部分的点的个数约 $800 \\times \\frac{7}{16}=350$,故选: C."} {"id": "17015", "image": [], "answer": "C", "solution": "null", "level": "高一", "question": "(2021$\\cdot$二模拟) 某人向直角边长分别为 6 和 8 的一个直角三角形中投郑一个点, 求此点落在此直角三角形内切圆的内部的概率是( )", "options": "A. $\\frac{\\pi}{2}$\n B. $\\frac{\\pi}{4}$\n C. $\\frac{\\pi}{6}$\n D. $\\frac{\\pi}{8}$", "subject": "度量几何学", "analysis": "解:由勾股定理可得斜边长为 $\\sqrt{6^{2}+8^{2}}=10$,\n\n设其内切圆的半径为 $r$,\n\n则由等面积法, 可得 $\\frac{1}{2}(8+6+10) r=\\frac{1}{2} \\times 8 \\times 6$, 则 $r=2$.\n\n$\\because S_{\\triangle A B C}=\\frac{1}{2} \\times 8 \\times 6=24, S_{\\text {圆 }}=\\pi \\times 2^{2}=4 \\pi$.\n\n$\\therefore$ 往该直角三角形中随机投郑一个点, 则该点落在此三角形内切圆内的概率为 $\\frac{4 \\pi}{24}=\\frac{\\pi}{6}$.故选: $C$."} {"id": "17016", "image": ["8771.jpg"], "answer": "D", "solution": "null", "level": "高一", "question": "(2021$\\cdot$十一模拟) 某游乐场制作了如图所示的游戏盘, 其中 $\\triangle A B C$ 为等腰三角形, $A=\\frac{2 \\pi}{3}, O$ 为 $B C$ 的中点, 分别以 $A, O$ 为圆心, $A B, B O$ 为半径画弧, 交于另一点 $C$. 向游戏盘内投飞镖(不考虑投不中的情况),则飞镖落入阴影部分的概率为\n\n", "options": "$\\frac{5 \\pi+6 \\sqrt{3}}{6(\\pi+\\sqrt{3})}$\nB. $\\frac{8 \\pi+6 \\sqrt{3}}{9 \\pi+6 \\sqrt{3}}$\nC. $\\frac{3 \\sqrt{3}+\\pi}{6(\\pi+\\sqrt{3})}$\nD. $\\frac{\\pi+6 \\sqrt{3}}{9 \\pi+6 \\sqrt{3}}$", "subject": "度量几何学", "analysis": "设 $A B=2$, 则 $B O=\\sqrt{3}, S_{\\text {雇形 } A C B}=\\frac{1}{3} \\times \\pi \\times 2^{2}=\\frac{4 \\pi}{3}$,以 $B C$ 为直径的半圆的面积 $S=\\frac{1}{2} \\pi(\\sqrt{3})^{2}=\\frac{3 \\pi}{2}$, $S_{\\triangle A B C}=\\frac{1}{2} \\times 2 \\sqrt{3} \\times 1=\\sqrt{3}$,故阴影部分的面积为 $\\frac{3 \\pi}{2}+\\sqrt{3}-\\frac{4 \\pi}{3}=\\frac{\\pi}{6}+\\sqrt{3}$,故所求概率 $P=\\frac{\\frac{\\pi}{6}+\\sqrt{3}}{\\frac{3 \\pi}{2}+\\sqrt{3}}=\\frac{\\pi+6 \\sqrt{3}}{9 \\pi+6 \\sqrt{3}}$,\n\n故选: D."} {"id": "17164", "image": [], "answer": "D", "solution": "null", "level": "高一", "question": "(2020 秋・二道区校级期末) 已知边长为 2 的正方形 $A B C D$, 在正方形 $A B C D$ 内随机取一点, 则取到的点到正方形四个顶点 $A, B, C, D$ 的距离都大于 1 的概率为 ( )", "options": "A. $\\frac{\\pi}{16}$\n B. $\\frac{\\pi}{4}$\n C. $\\frac{3-2 \\sqrt{2}}{4} \\pi$\n D. $1-\\frac{\\pi}{4}$", "subject": "度量几何学", "analysis": "解: 在正方形 $A B C D$ 内随机取一点 $P$, 点 $P$ 到点 $A$ 的距离大于 1 的部分是以 $A$ 为圆心, 1 为半径的 $\\frac{1}{4}$ 圆的外部,\n\n同理: 到其余 3 个顶点的距离大于 1 的部分为以 1 为半径的 $\\frac{1}{4}$ 圆的外部,\n\n符合条件的面积之和为 $2^{2}-\\pi \\times 1^{2}=4-\\pi$,\n\n$\\because$ 正方形的面积为 $2 \\times 2=4$,\n\n$\\therefore$ 点 $P$ 到正方形各顶点的距离大于 1 的概率为 $\\frac{4-\\pi}{4}=1-\\frac{\\pi}{4}$.\n\n故选: D."} {"id": "15030", "image": ["7489.jpg"], "answer": "D", "solution": "null", "level": "高一", "question": "汽车的“燃油效率”是指汽车每消耗 1 升汽油行驶的里程, 下图描述了甲、乙、丙三辆汽车在不同速度下的燃油效率情况. 下列叙述中正确的是()\n\n", "options": "A. 消耗 1 升汽油, 乙车最多可行驶 5 千米\n\nB. 以相同速度行驶相同路程, 三辆车中, 甲车消耗汽油最多\n\nC. 甲车以 80 千米/小时的速度行驶 1 小时, 消耗 10 升汽油\n\nD. 某城市机动车最高限速 80 千米/小时. 相同条件下, 在该市用丙车比用乙车更省油", "subject": "计数", "analysis": "对于 A, 由图象可知当速度大于 $40 \\mathrm{~km} / \\mathrm{h}$ 时, 乙车的燃油效率大于 $5 \\mathrm{~km} / \\mathrm{L}$,\n\n$\\therefore$ 当速度大于 $40 \\mathrm{~km} / \\mathrm{h}$ 时, 消耗 1 升汽油, 乙车的行驶距离大于 $5 \\mathrm{~km}, \\mathrm{~A}$ 不符合题意;\n\n对于 B,由图象可知当速度相同时,甲车的燃油效率最高,\n\n即当速度相同时, 消耗 1 升汽油, 甲车的行驶路程最远,\n\n$\\therefore$ 以相同速度行驶相同路程, 三辆车中, 甲车消耗汽油最少, $\\mathrm{B}$ 不符合题意;\n\n对于 C, 由图象可知当速度为 $80 \\mathrm{~km} / \\mathrm{h}$ 时, 甲车的燃油效率为 $10 \\mathrm{~km} / \\mathrm{L}$,\n\n即甲车行驶 $10 \\mathrm{~km}$ 时, 耗油 1 升, 故行驶 1 小时, 路程为 $80 \\mathrm{~km}$, 燃油为 8 升, C 不符合题意;\n\n对于 D, 由图象可知当速度小于 $80 \\mathrm{~km} / \\mathrm{h}$ 时, 丙车的燃油效率大于乙车的燃油效率,\n\n$\\therefore$ 用丙车比用乙车更省油, $\\mathrm{D}$ 符合题意\n\n故答案为: D.\n\n【分析】根据函数图像的几何意义对每个选项逐一分析即可得出正确答案即可.\n\n二、填空题"} {"id": "16488", "image": [], "answer": "C", "solution": "null", "level": "高一", "question": "从 50 份高三学生期中考试试卷中随机抽出 15 份进行教研分析, 则下列说法正确的是", "options": "A. 15 名样本\nB. 50 名学生是总体\nC. 样本容量是 15\nD. 样本容量是 50", "subject": "计数", "analysis": "样本是抽取的 15 份试卷, 总体是 50 份试卷, 总体容量是 50 , 样本容量是 15 .答案: C"} {"id": "16495", "image": [], "answer": "B", "solution": "null", "level": "高一", "question": "抽签法中确保样本代表性的关键是()", "options": "A. 制签\n\nB. 拌均匀\n\nC. 逐一抽取\n\nD. 抽取不放回", "subject": "计数", "analysis": "逐一抽取、抽取不放回是简单随机抽样的特点, 但不是确保代表性的关键,要使样本具有代表性, 则应保证总体中每个个体被抽到的机会均等,而 “搅拌均匀” 是每个个体被抽到的机会均等的关键.\n\n答案: B"} {"id": "16496", "image": [], "answer": "D", "solution": "null", "level": "高一", "question": "下列抽样方法是简单随机抽样的是 ()", "options": "A. 从 100 个学生家长中一次性随机抽取 10 人做家访\n\nB. 从 38 本教辅参考资料中有放回地随机抽取 3 本作为教学参考\n\nC. 从偶数集中一次性抽取 20 个进行奇偶性分析\n\nD. 某参会人员从最后一排 20 个座位中随机选择一个坐下", "subject": "计数", "analysis": "A 选项错在 “一次性” 抽取; B 选项错在 “有放回” 抽取; C 选项错在 “一次性” “总体容量无限”. 故正确选项为 D.\n\n答案: D"} {"id": "16497", "image": [], "answer": "B", "solution": "null", "level": "高一", "question": "下列抽样中,适合用抽签法的是()", "options": "A. 从某厂生产的 3000 件产品中抽取 600 件进行质量检验\n\nB. 从某厂生产的两箱 (每箱 15 件) 产品中抽取 6 件进行质量检验\n\nC. 从甲、乙两厂生产的两箱(每箱 15 件)产品中抽取 6 件进行质量检验\n\nD. 从某厂生产的 3000 件产品中抽取 10 件进行质量检验", "subject": "计数", "analysis": "A、D 中个体的总数较大, 不适合用抽签法; C 中甲、乙两厂生产的两箱产品质量可能差别较大, 不适合用抽签法; B 中个体的总数和样本容量均较小, 且是同厂生产的两箱产品, 质量差别不大,适合用抽签法.答案:B"} {"id": "16498", "image": [], "answer": "C", "solution": "null", "level": "高一", "question": "下列调查方式中, 不合适的是 ( )", "options": "A. 浙江卫视 “奔跑吧兄弟” 综艺节目的收视率,采用抽查的方式\n\nB. 了解某渔场中青鱼的平均质量, 采用抽查的方式\n\nC. 了解 iPhone6s 手机的使用寿命, 采用普查的方式\n\nD. 了解一批汽车的刹车性能, 采用普查的方式", "subject": "计数", "analysis": "对于 A, 收视率采用抽查方式. 例如央视-索福瑞媒介研究有限公司 (CSM)目前采用的收视率数据采集方法有两种, 即日记法和人员测量仪法. 日记法是指通过由样本户中所有 4 岁及以上家庭成员填空日记卡来收集收视信息的方法. 人员测量仪法是指利用 “人员测量仪” 来收集电视收视信息的方法. 以北京为例,2000 多万人口中抽取 500 个样本家庭用户, 调查约 1600 个观众的收视行为. 按照国际惯例, 央视-索福瑞的测量仪网样本户每年更换比例不低于 $25 \\%$, 最多四年即更换一轮; 日记卡网则一年更换一轮.对于 B, 没有必要采用普查的方式, 因此采用抽查合适对于 C, 了解手机的寿命的过程会有破坏性,因此没有必要对所有 iphone6s 手机进行调查,因此采用普查的方式不合适.对于 D, 了解汽车刹车性能, 因为涉及人身安全, 且对汽车没有破坏性, 因此, 应采用普查的方式.\n\n答案: C"} {"id": "16499", "image": ["8565.jpg", "8566.jpg", "8567.jpg"], "answer": "C", "solution": "null", "level": "高一", "question": "利用随机数表法对一个容量为 500 编号为 0001,002, , 999 品进行抽样检验, 抽取一个容量为 10 的样本, 若选定从第 12 行第 4 列的数开始向右读数(下面摘取了随机数表中的第 11 行至第 15 行 $)$, 则根据下面的随机数表,读出的第 3 个数是()\n$\\begin{array}{lllllllllllllllllllllll}18 & 18 & 07 & 92 & 45 & 44 & 17 & 16 & 58 & 09 & 79 & 83 & 86 & 19 & 62 & 06 & 76 & 5003 & 10 & 55 & 23 & 64 & 05\\end{array}$\n\n\n\n\n$\\begin{array}{llllllllllllllllllllll}52 & 36 & 28 & 19 & 95 & 50 & 92 & 26 & 11 & 97 & 00 & 56 & 76 & 31 & 38 & 80 & 22 & 0253 & 53 & 86 & 60 & 42 \\\\ 04 & 53\\end{array}$\n\n", "options": "A. 584\nB. 114\nC. 311\nD. 146", "subject": "计数", "analysis": "从第 12 行第 4 列的数开始向右读数可得: $238,160,311,463,224$, . 所以读出的第 3 个数是 311 . 选 C.\n\n答案: C"} {"id": "16503", "image": [], "answer": "C", "solution": "null", "level": "高一", "question": "某报告厅有 50 排座位, 每排有 60 个座号, 一次报告会坐满了听众, 会后留下座位号为 $18,78,138,198 \\cdots$ 的 50 位听众进行座谈, 这种抽取样本的方法是 ( )", "options": "A. 抽签法\nB. 随机数表法\nC. 系统抽样\nD. 有放回抽样", "subject": "计数", "analysis": "总体容量 (3000) 较大, 抽取间隔相等, 符合系统抽样的特点, 是系统抽样。答案: C"} {"id": "16510", "image": [], "answer": "A", "solution": "null", "level": "高一", "question": "采用系统抽样的方法从 2005 个个体中抽取一个容量为 50 的样本, 则抽样间隔和随机剔除的个体数分别为 ( )", "options": "A. 40,5\nB. 50,5\nC. 5,40\nD. 5,50", "subject": "计数", "analysis": "因为 $2005 \\div 50=40$ 余 5 , 所以用系统抽样的方法从 2005 个个体中抽取一个容量为 50 的样本, 抽样间隔是 40 , 且应随机剔除的个体数为 5 .\n\n答案: A"} {"id": "16511", "image": [], "answer": "C", "solution": "null", "level": "高一", "question": "某校高二年级共有 600 名学生, 编号为 $001^{\\sim} 600$. 为了分析该年级学生上学期期末的数学考试情况, 用系统抽样的方法抽取了一个样本容量为 60 的样本. 如果编号 006, 016, 026 在样本中,那么下列编号在样本中的是 (", "options": "A. 010\nB. 020\nC. 036\nD. 042", "subject": "计数", "analysis": "样本容量为 60 , 抽样间隔为 $10,16-6=26-16=36-26=10$, 故选 C.\n\n答案: C"} {"id": "16512", "image": [], "answer": "D", "solution": "null", "level": "高一", "question": "从 72 名志愿者中抽取 10 名志愿者进行专业兴趣调查, 若采用系统抽样, 则每名志愿者被抽到的可能性是 ( )", "options": "A. $\\frac{1}{72}$\nB. $\\frac{1}{7}$\nC. $\\frac{1}{3}$\nD. $\\frac{5}{36}$", "subject": "计数", "analysis": "由于 72 不能被 10 整除, 则需先剔除 2 名志愿者, 确定分段间隔 $k=10$, 但是每名志愿者被剔除的可能性一样, 所以虽然剔除了 2 名志愿者, 但这 72 名志愿者中每名志愿者被抽到的可能性是一样的, 都是 $\\frac{10}{72}=\\frac{5}{36}$, 故选 D.\n\n答案: D"} {"id": "16513", "image": [], "answer": "A", "solution": "null", "level": "高一", "question": "下列有关系统抽样的说法正确的是 ( )\n\n(1)从某厂生产的 2000 个电子元件中随机抽取 50 个入样,适宜用系统抽样法;\n\n(2)人们打桥牌时, 从洗好的扑克牌(52 张)中随机确定一张作为起始牌, 这时, 开始按次序发牌, 对任何一家来说, 都是从 52 张总体中抽取一个容量为 13 的样本, 这里运用的抽样方法是系统抽样;\n\n(3)有 1252 名学生的成绩, 采用系统抽样的方法抽取一个容量为 50 的样本, 则总体中随机剔除的个体数目是 2 , 但对于被剔除的 2 名学生来说, 这样做是不公平的.", "options": "A. (1)(2)\nB. (1)(3)\nC. (2)(3)\nD. (1)(2)(3)", "subject": "计数", "analysis": "(1)正确, 因为总体容量较大, 适宜用系统抽样法; (2)正确, 简单随机抽样的实质是逐个地从总体中随机抽取样本, 而这里只是随机确定了起始张, 这时其他各张显然是逐张地被抽取, 其实各张在谁手里已确定, 所以, 根据其等距起牌的特点, 应将其定位为系统抽样; (3)错误, 整个抽样过程中每个个体被抽到的可能性仍然相等, 因为每个个体被抽到的机会相等, 所以每个个体被剔除的机会也相等. 故选 A.\n\n答案: A"} {"id": "16514", "image": [], "answer": "B", "solution": "null", "level": "高一", "question": "将参加夏令营的 600 名学生编号为: $001,002, \\cdots, 600$. 采用系统抽样方法抽取一个容量为 50 的样本, 且随机抽得的号码为 003. 这 600 名学生分住在三个营区, 从 001 到 300 在第 I 营区, 从 301 到 495 在第 II 营区, 从 496 到 600 在第III营区, 三个营区被抽中的人数依次为", "options": "A. $26,16,8$\nB. $25,17,8$\nC. $25,16,9$\nD. $24,17,9$", "subject": "计数", "analysis": "依题意及系统抽样的意义可知, 将这 600 名学生按编号依次分成 50 组, 每一组有 12 名学生, 第 $k\\left(k \\in \\mathbf{N}_{+}\\right)$组抽中的号码是 $3+12(k-1)$. 令 $3+12(k-1) \\leqslant 300$, 得 $k \\leqslant \\frac{103}{4}$,因此第 I 营区被抽中的人数是 25 ; 令 $300<3+12(k-1) \\leqslant 495$, 得 $\\frac{103}{4}\n则 7 个剩余分数的方差为", "options": "A. $\\frac{116}{9}$\nB. $\\frac{36}{7}$\nC. 36\nD. $\\frac{6 \\sqrt{7}}{7}$", "subject": "计数", "analysis": "由题中茎叶图可知, 去掉的两个数是 87,99 , 所以 $87+90 \\times 2+91 \\times 2+94+90+x=91$\n\n$\\times 7$, 解得 $x=4$. 故\n\n$s^{2}=\\frac{1}{7} \\times\\left[(87-91)^{2}+(90-91)^{2} \\times 2+(91-91)^{2} \\times 2+(94-91)^{2} \\times 2\\right]=\\frac{36}{7}$\n\n答案: B"} {"id": "16540", "image": [], "answer": "C", "solution": "null", "level": "高一", "question": "某班级有 50 名学生, 其中有 30 名男生和 20 名女生. 随机询问了该班 5 名男生和 5 名女生在某次数学测验中的成绩, 5 名男生的成绩分别为 $86,94,88,92,90,5$ 名女生的成绩分别为 $88,93,93,88,93$. 下列说法一定正确的是 ( )", "options": "A. 这种抽样方法是分层抽样\n\nB. 这种抽样方法是系统抽样\n\nC. 这 5 名男生成绩的方差大于这 5 名女生成绩的方差\n\nD. 该班男生成绩的平均数小于该班女生成绩的平均数", "subject": "计数", "analysis": "若抽样方法是分层抽样, 男生、女生应分别捆取 6 人、 4 人, 所以 $\\mathrm{A}$ 错; 由题目看不出是系统抽样, 所以 $\\mathrm{B}$ 错; 这 5 名男生成绩的平均数\n\n$\\bar{x}_{1}=\\frac{86+94+88+92+90}{5}=90$, 这 5 名女生成绩的平均数 $\\bar{x}_{2}=\\frac{88+93+93+88+93}{5}=91$,\n\n故这 5 名男生成绩的方差为\n\n$\\frac{1}{5} \\times\\left[(86-90)^{2}+(94-90)^{2}+(88-90)^{2}+(92-90)^{2}+(90-90)^{2}\\right]=8 \\div x$, 这 5 名女生成绩的方差为号 $\\frac{1}{5} \\times\\left[(88-91)^{2} \\times 2+(93-91)^{2} \\times 3\\right]=6$, 所以这 5 名男生成绩的方差大于这 5 名女生成绩的方差, 但该班男生成绩的平均数不一定小于女生成绩的平均数, 所以 C 正确, D 错.\n\n答案: C"} {"id": "16541", "image": ["8572.jpg"], "answer": "B", "solution": "null", "level": "高一", "question": "图是我市甲、乙两地五月上旬日平均气温的统计图, 则甲、乙两地这十天的日平均气温 $\\bar{x}_{\\text {甲 }}, \\bar{x}_{\\text {乙 }}$ 和日平均气温的标准差 $s_{\\text {甲 }}, s_{\\text {乙 }}$ 的大小关系应为 ( )\n\n", "options": "A. $\\bar{x}_{\\text {甲 }}=\\bar{x}_{\\text {乙 }}, s_{\\text {甲 }}s_{\\text {乙 }}$\nC. $\\bar{x}_{\\text {甲 }}>\\bar{x}_{\\text {乙 }}, s_{\\text {甲 }}\\bar{x}_{\\text {乙 }}, s_{\\text {甲 }}>s_{\\text {乙 }}$", "subject": "计数", "analysis": "由题中折线统计图可得甲、乙两地五月上旬日平均气温的值, 由标准差的统计意义可得乙的标准差比较小, 则只需计算十天的日平均气温 $\\bar{x}_{\\text {甲 }}$ 和 $\\bar{x}_{\\text {乙 }}$ 即可.\n\n$\\bar{x}_{\\text {甲 }}=\\frac{24+30+28+24+22+26+27+26+29+24}{10}=26$\n\n$\\bar{x}_{\\text {乙 }}=\\frac{24+26+25+26+24+27+28+26+28+26}{10}=26$, 故选 B.\n\n答案: B"} {"id": "16542", "image": ["8573.jpg"], "answer": "B", "solution": "null", "level": "高一", "question": "为比较甲、乙两地某月 14 时的气温状况,随机选取该月中的 5 天,将这 5 天中 14 时的气温数据(单位: ${ }^{\\circ} \\mathrm{C}$ ) 制成如图所示的茎叶图. 考虑以下结论:\n\n(1)甲地该月 14 时的平均气温低于乙地该月 14 时的平均气温;\n\n(2)甲地该月 14 时的平均气温高于乙地该月 14 时的平均气温;\n\n(3)甲地该月 14 时的气温的标准差小于乙地该月 14 时的气温的标准差;\n\n(4)甲地该月 14 时的气温的标准差大于乙地该月 14 时的气温的标准差。其中根据茎叶图能得到的统计结论的编号为\n\n", "options": "A. (1)(3)\nB. (1)(4)\nC. (2)(3)\nD. (2)(4)", "subject": "计数", "analysis": "由题中茎叶图, 知 $\\bar{x}_{\\mathrm{F}}=\\frac{26+28+29+31+31}{5}=29$\n\n$S_{\\varphi}=\\sqrt{\\frac{1}{5} \\times\\left[(26-29)^{2}+(28-29)^{2}+(29-29)^{2}+(31-29)^{2}+(31-29)^{2}\\right]}=\\frac{3 \\sqrt{10}}{5}$\n\n$\\bar{x}_{c}=\\frac{28+29+30+31+32}{5}=30$\n$s_{z}=\\sqrt{\\frac{1}{5} \\times\\left[(8-30)^{2}+(29-30)^{2}+(30-30)^{2}+(31-30)^{2}+(32-30)^{2}\\right]}=\\sqrt{2}$\n\n所以 $\\bar{x}_{\\text {甲 }}<\\bar{x}_{\\text {乙 }}, s_{\\text {甲 }}>s_{\\text {乙 }}$\n\n答案: $B$"} {"id": "16927", "image": [], "answer": "C", "solution": "null", "level": "高一", "question": "(2020 秋 -秦安县校级期末)若用秦九韶算法求多项式 $f(x)=4 x^{5}-x^{2}+2$ 当 $x=3$ 时的值, 则需要做乘法运算和加减法运算的次数分别为 ( )", "options": "A. 4,2\n B. 5,3\n C. 5,2\n D. 6,2", "subject": "计数", "analysis": "解: $\\because f(x)=(((4 x) x) x-1) x) x+2$,\n\n$\\therefore$ 乘法要运算 5 次, 加减法要运算 2 次.\n\n故选: $C$."} {"id": "16992", "image": [], "answer": "A", "solution": "null", "level": "高一", "question": "(2021$\\cdot$十八模拟)连续抛郑一枚硬币 4 次, 落地后第 2 次和第 4 次恰好都是正面向上的概率是 ( )", "options": "$\\frac{1}{4}$\n B. $\\frac{3}{4}$\n C. $\\frac{3}{5}$\n D. $\\frac{2}{5}$", "subject": "计数", "analysis": "抛郑一枚硬币,落地后可能出现正面和反面两种情况,\n\n连续抛郑一枚硬币 4 次的所有结果为:\n\n(正, 正, 正, 正)(正, 正, 正, 反)\n\n(正, 正, 反, 正) (正, 正, 反, 反)\n\n(正, 反, 正, 正) (正, 反, 正, 反)\n\n(正, 反, 反, 正) (正, 反, 反, 反)\n\n(反, 正, 正, 正) (反, 正, 正, 反)\n\n(反, 正, 反, 正) (反, 正, 反, 反)\n\n(反, 反, 正, 正) (反, 反, 正, 反)\n\n(反, 反, 反, 正) (反, 反, 反, 反)\n\n共 16 种情况,\n\n落地后第 2 次和第 4 次恰好都是正面向上的结果为:\n\n(正, 正, 正, 正) (正, 正, 反, 正)\n\n(反, 正, 正, 正) (反, 正, 反, 正)\n\n共 4 种情况,\n\n故所求事件的概率 $P=\\frac{4}{16}=\\frac{1}{4}$,\n\n故选: A."} {"id": "16995", "image": [], "answer": "C", "solution": "null", "level": "高一", "question": "(2020 秋 - 永昌县校级期末)在区间 $\\left[-\\frac{\\pi}{4}, \\frac{\\pi}{4}\\right]$ 上随机取一个数 $x$, 则 $\\cos x$ 的值介于 $\\frac{\\sqrt{3}}{2}$ 到 1 的概率为 ( )", "options": "A. $\\frac{1}{3}$\n B. $\\frac{1}{2}$\n C. $\\frac{2}{3}$\n D. $\\frac{3}{4}$", "subject": "计数", "analysis": "解:根据题意, 区间 $\\left[-\\frac{\\pi}{4}, \\frac{\\pi}{4}\\right]$ 上, 若 $\\frac{\\sqrt{3}}{2}<\\cos x<1$, 则有 $-\\frac{\\pi}{6}\n\nB.\n\n\n\nC.\n\n\n\nD.\n\n", "subject": "画法几何学", "analysis": "画两个相交平面时,被遮住的部分用虚线表示."} {"id": "15622", "image": ["7910.jpg", "7911.jpg", "7912.jpg", "7913.jpg"], "answer": "A", "solution": "null", "level": "高一", "question": "用斜二测画法画一个水平放置的平面图形的直观图是如图所示的一个正方形, 则原来的图形是\n\n", "options": "\nB.\n\n\nC.\n\n\nD. $\\xrightarrow[O]{\\underbrace{y_{\\uparrow}}_{\\uparrow}}$", "subject": "画法几何学", "analysis": "试题分析:由斜二测画法的规则知与 $x^{\\prime}$ 轴平行或重合的线段与与 $x^{\\prime}$ 轴平行或重合, 其长度不变, 与 $y$轴平行或重合的线段与 $x^{\\prime}$ 轴平行或重合, 其长度变成原来的一半, 正方形的对角线在 $y^{\\prime}$ 轴上, 可求得其长度为 $\\sqrt{2}$, 故在平面图中其在 $\\mathrm{y}$ 轴上, 且其长度变为原来的 2 倍, 长度为 $2 \\sqrt{2}$, 观察四个选项, A 选项符合题意. 故应选 A.\n\n考点: 斜二测画法.\n\n点评: 注意斜二测画法中线段长度的变化."} {"id": "15638", "image": ["7941.jpg", "7942.jpg"], "answer": "A", "solution": "null", "level": "高一", "question": "如图, 正方形 $O^{\\prime} A^{\\prime} B^{\\prime} C^{\\prime}$ 的边长为 $1 \\mathrm{~cm}$, 它是水平放置的一个平面图形用斜二测画法得到的直观图, 则原图形的周长是 ( )\n\n", "options": "$8 \\mathrm{~cm}$\nB. $6 \\mathrm{~cm}$\nC. $2(1+\\sqrt{3}) \\mathrm{cm}$\nD. $2(1+\\sqrt{2}) \\mathrm{cm}$", "subject": "画法几何学", "analysis": "【分析】\n\n将直观图还原为平面图形是平行四边形,然后计算.\n\n## 【详解】\n\n解: 将直观图还原为平面图形,如图所示.\n\n$$\nO B=2 O^{\\prime} B^{\\prime}=2 \\sqrt{2}, O A=O^{\\prime} A^{\\prime}=1 \\text {, 所以 } A B=\\sqrt{1^{2}+(2 \\sqrt{2})^{2}}=3 \\text {, }\n$$\n\n\n\n所以原图形的周长为 $8 \\mathrm{~cm}$,\n\n故选: A.\n\n【点睛】\n\n本题考查斜二测画法, 掌握斜二测画法的定义是解题关键. 根据斜二测画法的定义才能根据直观图中直线的位置关系确定原图形中直线的位置关系, 从而解决原图形中的问题."} {"id": "15420", "image": [], "answer": "C", "solution": "null", "level": "高一", "question": "下列说法中正确的是( )", "options": "A. 三点确定一个平面\n\nB. 四边形一定是平面图形\n\nC. 梯形一定是平面图形\n\nD. 两个不同平面 $\\alpha$ 和 $\\beta$ 有不在同一条直线上的三个公共点", "subject": "立体几何学", "analysis": "不共线的三点确定一个平面,故 A 不正确; 四边形有时指空间四边形,故 B 不正确;梯形的上底和下底平行,可以确定一个平面,故 C 正确;两个平面如果相交,一定有一条交线,所有这两个平面的公共点都在这条交线上,故 D 不正确. 故选 C."} {"id": "15421", "image": [], "answer": "B", "solution": "null", "level": "高一", "question": "如果空间四点 $A, B, C, D$ 不共面, 那么下列判断中正确的是( )", "options": "A. $A, B, C, D$ 四点中必有三点共线\n\nB. $A, B, C, D$ 四点中不存在三点共线\n\nC. 直线 $A B$ 与 $C D$ 相交\n\nD. 直线 $A B$ 与 $C D$ 平行", "subject": "立体几何学", "analysis": "两条平行直线、两条相交直线、直线及直线外一点都分别确定一个平面."} {"id": "15422", "image": [], "answer": "A", "solution": "null", "level": "高一", "question": "如果直线 $a \\subset$ 平面 $\\alpha$, 直线 $b \\subset$ 平面 $\\alpha, M \\in a, N \\in b, M \\in l, N \\in l$, 则 ()", "options": "A. $l \\subset \\alpha$\nB. $14 \\alpha$\nC. $l \\cap \\alpha=M$\nD. $l \\cap \\alpha=N$", "subject": "立体几何学", "analysis": "$\\because M \\in a , a \\subset \\alpha$ ,\n$\\therefore M \\in \\alpha$ ,同理, $N \\in \\alpha ,$\n\n又 $M \\in l, N \\in l$ ,故 $l \\subset \\alpha$."} {"id": "15424", "image": [], "answer": "C", "solution": "null", "level": "高一", "question": "已知 $\\alpha, \\beta$ 为平面, $A, B, M, N$ 为点, $a$ 为直线, 下列推理错误的是 ( )", "options": "A. $A \\in a, A \\in \\beta, B \\in a, B \\in \\beta \\Rightarrow a \\subset \\beta$\n\nB. $M \\in \\alpha, M \\in \\beta, N \\in \\alpha, N \\in \\beta \\Rightarrow \\alpha \\cap \\beta=M N$\n\nC. $A \\in \\alpha, A \\in \\beta \\Rightarrow \\alpha \\cap \\beta=A$\n\nD. $A, B, M \\in \\alpha, A, B, M \\in \\beta$, 且 $A, B, M$ 不共线 $\\Rightarrow \\alpha, \\beta$ 重合", "subject": "立体几何学", "analysis": "$\\because A \\in \\alpha , A \\in \\beta , \\therefore A \\in(\\alpha \\cap \\beta)$.\n\n由公理可知 $\\alpha \\cap \\beta$ 为经过 $A$ 的一条直线而不是点 $A$.\n\n故 $\\alpha \\cap \\beta=A$ 的写法错误."} {"id": "15425", "image": [], "answer": "C", "solution": "null", "level": "高一", "question": "一条直线和直线外的三点所确定的平面有( )", "options": "A. 1 个或 3 个\n\nB. 1 个或 4 个\n\nC. 1 个, 3 个或 4 个\n\nD. 1 个, 2 个或 4 个", "subject": "立体几何学", "analysis": "若三点在同一直线上,且与已知直线平行或相交,或该直线在由该三点确定的平面内,则均确定 1 个平面;若三点有两点连线和已知直线平行时可确定 3 个平面;若三点不共线,且该直线在由该三点确定的平面外,则可确定 4 个平面."} {"id": "15426", "image": [], "answer": "D", "solution": "null", "level": "高一", "question": "空间中有五个点, 其中有四个点在同一平面内, 但没有任何三点共线, 这样的五个点确定平面的个数最多可以是( )", "options": "A. 4\nB. 5\nC. 6\nD. 7", "subject": "立体几何学", "analysis": "想象四棱锥的 5 个顶点,它们总共确定 7 个平面."} {"id": "15428", "image": [], "answer": "B", "solution": "null", "level": "高一", "question": "若空间三条直线 $a, b, c$ 满足 $a \\perp b, b / / c$, 则直线 $a$ 与 $c(\\quad)$", "options": "A. 一定平行\nB. 一定垂直\nC. 一定是异面直线\nD. 一定相交", "subject": "立体几何学", "analysis": "$\\because a \\perp b, b / / c , \\therefore a \\perp c$."} {"id": "15433", "image": [], "answer": "D", "solution": "null", "level": "高一", "question": "分别和两条异面直线平行的两条直线的位置关系是( )", "options": "A. 一定平行\nB. 一定相交\nC. 一定异面\nD. 相交或异面", "subject": "立体几何学", "analysis": "可能相交也可能异面,但一定不平行(否则与条件矛盾)."} {"id": "15434", "image": [], "answer": "D", "solution": "null", "level": "高一", "question": "两等角的一组对应边平行, 则( )", "options": "另一组对应边平行\n\nB. 另一组对应边不平行\n\nC. 另一组对应边不可能垂直\n\nD. 以上都不对", "subject": "立体几何学", "analysis": "另一组对应边可能平行,也可能不平行,也可能垂直. 注意和等角定理(若两个角的对应边平行,则这两个角相等或互补)的区别."} {"id": "15435", "image": ["7666.jpg"], "answer": "C", "solution": "null", "level": "高一", "question": "在正方体 $A B C D-A_{1} B_{1} C_{1} D_{1}$ 中, $E, F$ 分别是侧面 $A A_{1} D_{1} D$, 侧面 $C C_{1} D_{1} D$ 的中心, $G, H$分别是线段 $A B, B C$ 的中点, 则直线 $E F$ 与直线 $G H$ 的位置关系是 ( )", "options": "A. 相交\nB. 异面\nC. 平行\nD. 垂直", "subject": "立体几何学", "analysis": "如图,连接 $A D_{1} , C D_{1} , A C$ ,则 $E , F$ 分别为 $A D_{1} , C D_{1}$ 的中点. 由三角形的中位线定理,知 $E F / / A C , G H / / A C$ ,所以 $E F / / G H$ ,故选 C.\n\n"} {"id": "15436", "image": ["7667.jpg"], "answer": "C", "solution": "null", "level": "高一", "question": "长方体的一条体对角线与长方体的棱所组成的异面直线有 ( )", "options": "A. 2 对\nB. 3 对\nC. 6 对\nD. 12 对", "subject": "立体几何学", "analysis": "如图所示,在长方体中没有与体对角线平行的棱,要求与长方体体对角线 $A C_{1}$ 异面的棱所在的直线,只要去掉与 $A C_{1}$ 相交的六条棱,其余的都与体对角线异面, $\\therefore$ 与 $A C_{1}$ 异面的棱有 $B B_{1} , A_{1} D_{1} , A_{1} B_{1} , B C , C D , D D_{1} , \\therefore$ 长方体的一条对角线与长方体的棱所组成的异面直线有 6 对,故选 C.\n\n"} {"id": "15437", "image": ["7668.jpg"], "answer": "A", "solution": "null", "level": "高一", "question": "已知直线 $a, b, c$, 下列三个命题:\n\n(1)若 $a$ 与 $b$ 异面, $b$ 与 $c$ 异面, 则 $a$ 与 $c$ 异面;\n\n(2)若 $a / / b, a$ 和 $c$ 相交, 则 $b$ 和 $c$ 也相交;\n\n(3)若 $a \\perp b, a \\perp c$, 则 $b / / c$.\n\n其中, 正确命题的个数是 ( )", "options": "A. 0\nB. 1\nC. 2\nD. 3", "subject": "立体几何学", "analysis": "(1)不正确,如图; (2)不正确,有可能相交,也有可能异面;(3)不正确,可能平行,可能相交,也可能异面.\n\n"} {"id": "15439", "image": ["7670.jpg"], "answer": "C", "solution": "null", "level": "高一", "question": "如图, 点 $P, Q$ 分别是正方体 $A B C D-A_{1} B_{1} C_{1} D_{1}$ 的面对角线 $A D_{1}, B D$ 的中点, 则异面直线 $P Q$ 和 $B C_{1}$ 所成的角为 $(\\quad)$", "options": "A. $30^{\\circ}$\nB. $45^{\\circ}$\nC. $60^{\\circ}$\nD. $90^{\\circ}$", "subject": "立体几何学", "analysis": "$A C, D_{1} C$.\n\n\n\n由 $P, Q$ 分别为 $A D_{1}, B D$ 的中点,\n\n得 $P Q / / C D_{1}$.\n\n又 $B C_{1} / / A D_{1}$,\n\n$\\therefore \\angle A D_{1} C$ 为异面直线 $P Q$ 和 $B C_{1}$ 所成的角.\n\n$\\because \\triangle A C D_{1}$ 为等边三角形,\n\n$\\therefore \\angle A D_{1} C=60^{\\circ}$.\n\n即异面直线 $P Q$ 和 $B C_{1}$ 所成的角为 $60^{\\circ}$."} {"id": "15441", "image": [], "answer": "D", "solution": "null", "level": "高一", "question": "直线在平面外是指( )", "options": "A. 直线与平面没有公共点\nB. 直线与平面相交\nC. 直线与平面平行\nD. 直线与平面最多只有一个公共点", "subject": "立体几何学", "analysis": "直线与平面的位置关系为:平行、相交、在平面内,其中平行和相交通称为直线在平面外,所以直线与平面最多只有一个公共点."} {"id": "15445", "image": [], "answer": "A", "solution": "null", "level": "高一", "question": "三棱台的一条侧棱所在直线与其对面所在的平面之间的关系是( )", "options": "A. 相交\nB. 平行\nC. 直线在平面内\nD. 平行或直线在平面内", "subject": "立体几何学", "analysis": "延长各侧棱可恢复成棱锥的形状,所以三棱台的一条侧棱所在直线与其对面所在的平面相交."} {"id": "15446", "image": [], "answer": "C", "solution": "null", "level": "高一", "question": "下列说法正确的是( )", "options": "A. 如果一条直线与一个平面内的无数条直线平行, 则这条直线与这个平面平行\n\nB. 两个平面相交于唯一的公共点\n\nC. 如果一条直线与一个平面有两个不同的公共点, 则它们必有无数个公共点\n\nD. 平面外的一条直线必与该平面内无数条直线平行", "subject": "立体几何学", "analysis": "在 A 中,如果一条直线与一个平面内的无数条直线平行,则这条直线与这个平面平行或这条直线在这个平面内,故 A 错误; 在 B 中,两个平面相交于一条直线,故 B 错误;在 C\n中,如果一条直线与一个平面有两个不同的公共点,则这条直线在平面内,它们必有无数个公共点,故 C 正确;在 D中,当平面外的一条直线与平面相交时,则平面外的这条直线必与该平面内的直线不平行,故 D 错误. 故选 C."} {"id": "15447", "image": [], "answer": "B", "solution": "null", "level": "高一", "question": "若平面 $\\alpha / /$ 平面 $\\beta, l \\subset \\alpha$, 则 $l$ 与 $\\beta$ 的位置关系是( )", "options": "A. $l$ 与 $\\beta$ 相交\nB. $l$ 与 $\\beta$ 平行\nC. $l$ 在 $\\beta$ 内\nD. 无法判定", "subject": "立体几何学", "analysis": "$\\because \\alpha / / \\beta, \\therefore \\alpha$ 与 $\\beta$ 无公共点.\n\n$\\because l \\subset \\alpha, \\therefore l$ 与 $\\beta$ 无公共点, $\\therefore l / / \\beta$."} {"id": "15448", "image": [], "answer": "C", "solution": "null", "level": "高一", "question": "若平面 $\\alpha$ 与 $\\beta$ 的公共点多于两个, 则( )", "options": "$\\alpha, \\beta$ 可能只有三个公共点\n\nB. $\\alpha, \\beta$ 可能有无数个公共点, 但这无数个公共点不在一条直线上\n\nC. $\\alpha, \\beta$ 一定有无数个公共点\n\nD. 以上均不正确", "subject": "立体几何学", "analysis": "若平面 $\\alpha$ 与 $\\beta$ 的公共点多于两个,则平面 $\\alpha$ 与 $\\beta$ 相交或重合,故选 C."} {"id": "15449", "image": [], "answer": "D", "solution": "null", "level": "高一", "question": "若直线 $a \\perp b$, 且直线 $a / /$ 平面 $\\alpha$, 则直线 $b$ 与平面 $\\alpha$ 的位置关系是 ( )", "options": "A. $b \\subset \\alpha$\n\nB. $b / / \\alpha$\n\nC. $b \\subset \\alpha$ 或 $b / / \\alpha$\n\nD. $b$ 与 $\\alpha$ 相交或 $b \\subset \\alpha$ 或 $b / / \\alpha$", "subject": "立体几何学", "analysis": "通过观察正方体,可知 $b$ 与 $\\alpha$ 相交或 $b \\subset \\alpha$ 或 $b / / \\alpha$.故选 D."} {"id": "15450", "image": [], "answer": "B", "solution": "null", "level": "高一", "question": "下列命题中, 正确的有 ( )\n\n(1)平行于同一直线的两条直线平行;\n\n(2)平行于同一个平面的两条直线平行;\n\n(3)平行于同一条直线的两个平面平行;\n\n(4)平行于同一个平面的两个平面平行.", "options": "1 个\nB. 2 个\nC. 3 个\nD. 4 个", "subject": "立体几何学", "analysis": "(2)中,也有可能是相交或异面,故(2)错误;(3)中,存在平行于两个相交平面的交线,\n且不在两个平面内的直线,故(3)错误."} {"id": "15451", "image": [], "answer": "B", "solution": "null", "level": "高一", "question": "下列说法中正确的个数是 ( )\n\n(1)平面 $\\alpha$ 与平面 $\\beta, \\gamma$ 都相交, 则这三个平面有 2 条或 3 条交线;\n\n(2)两个平面平行, 各任取两平面的一条直线, 它们不相交;\n\n(3)直线 $a$ 不平行于平面 $\\alpha$ ,则 $a$ 不平行于 $\\alpha$ 内任何一条直线;\n\n(4)如果 $\\alpha / / \\beta, a / / \\alpha$, 那么 $a / / \\beta$.", "options": "A. 0\nB. 1\nC. 2\nD. 3", "subject": "立体几何学", "analysis": ")错误,平面 $\\alpha$ 与平面 $\\beta , \\gamma$ 都相交,则这三个平面有可能有 2 条或 3 条交线,还有可能只有 1 条交线.\n\n(2)正确,两平行平面无公共点,任取的直线也无公共点,即不相交.\n\n(3)错误,直线 $a$ 不平行于平面 $\\alpha$ ,则 $a$ 有可能在平面 $\\alpha$ 内,此时可以与平面内无数条直线平行.\n\n(4)错误,如果 $\\alpha / / \\beta , a / / \\alpha$ ,那么 $a / / \\beta$ 或 $a \\subset \\beta$."} {"id": "15453", "image": [], "answer": "C", "solution": "null", "level": "高一", "question": "下列条件中能得出直线 $m$ 与平面 $\\alpha$ 平行的是 ( )", "options": "直线 $m$ 与平面 $\\alpha$ 内所有直线平行\n\nB. 直线 $m$ 与平面 $\\alpha$ 内无数条直线平行\n\nC. 直线 $m$ 与平面 $\\alpha$ 没有公共点\n\nD. 直线 $m$ 与平面 $\\alpha$ 内的一条直线平行", "subject": "立体几何学", "analysis": "A,本身说法错误;B,当直线 $m$ 在平面 $\\alpha$ 内时, $m$ 与 $\\alpha$ 不平行;C,能推出 $m$ 与 $\\alpha$平行;D,当直线 $m$ 在平面 $\\alpha$ 内时, $m$ 与 $\\alpha$ 不平行. 故选 C."} {"id": "15458", "image": [], "answer": "B", "solution": "null", "level": "高一", "question": "若直线 $l$ 不平行于平面 $\\alpha$, 且 $l 4 \\alpha$, 则 ( )", "options": "A. $\\alpha$ 内的所有直线与 $l$ 异面\n\nB. $\\alpha$ 内不存在与 $l$ 平行的直线\n\nC. $\\alpha$ 内存在唯一的直线与 $l$ 平行\n\nD. $\\alpha$ 内的直线与 $l$ 都相交", "subject": "立体几何学", "analysis": "若在平面 $\\alpha$ 内存在与直线 $l$ 平行的直线,因 $l \\Varangle \\alpha$ ,故 $l / / \\alpha$ ,这与题意矛盾."} {"id": "15459", "image": [], "answer": "D", "solution": "null", "level": "高一", "question": "过直线 $l$ 外两点, 作与 $l$ 平行的平面, 则这样的平面 ( )", "options": "不可能作出\nB. 只能作出一个\nC. 能作出无数个\nD. 上述三种情况都存在", "subject": "立体几何学", "analysis": "设直线外两点为 $A , B$ ,若直线 $A B / / l$ ,则过 $A , B$ 可作无数个平面与 $l$ 平行; 若直线 $A B$ 与 $l$ 异面,则只能作一个平面与 $l$ 平行; 若直线 $A B$ 与 $l$ 相交,则过 $A , B$ 没有平面与 $l$ 平行."} {"id": "15460", "image": ["7677.jpg"], "answer": "C", "solution": "null", "level": "高一", "question": "如图所示, $P$ 为矩形 $A B C D$ 所在平面外一点, 矩形对角线交点为 $O, M$ 为 $P B$ 的中点, 给出五个结论:\n\n\n\n(1) $O M / / P D$;\n\n(2) $O M / /$ 平面 $P C D$;\n\n(3) $O M / /$ 平面 $P D A$;\n\n(4) $O M / /$ 平面 $P B A$;\n\n(5) $O M / /$ 平面 $P B C$.\n\n其中正确的个数为 ( )", "options": "1\nB. 2\nC. 3\nD. 4", "subject": "立体几何学", "analysis": "由题意知, $O M$ 是 $\\triangle B P D$ 的中位线, $\\therefore O M / / P D$ ,故(1)正确; $P D \\subset$ 平面 $P C D , O M \\nsubseteq$平面 $P C D , \\therefore O M / /$ 平面 $P C D$ ,故(2)正确;同理可得: $O M / /$ 平面 $P D A ,$ 故(3)正确; $O M$ 与平面 $P B A$ 和平面 $P B C$ 都相交,故(4),(5)不正确. 故共有 3 个结论正确."} {"id": "15461", "image": [], "answer": "A", "solution": "null", "level": "高一", "question": "直线 $a, b$ 为异面直线, 过直线 $a$ 与直线 $b$ 平行的平面 ( )", "options": "A. 有且只有一个\n\nB. 有无数多个\n\nC. 有且只有一个或不存在\n\nD. 不存在", "subject": "立体几何学", "analysis": "在 $a$ 上任取一点 $A$ ,则过 $A$ 与 $b$ 平行的直线有且只有一条,设为 $b^{\\prime} , 又 ~ \\because a \\cap b^{\\prime}=$ $A , \\therefore a$ 与 $b^{\\prime}$ 确定一个平面 $\\alpha$ ,即为过 $a$ 与 $b$ 平行的平面,可知它是唯一的."} {"id": "15462", "image": ["7678.jpg", "7679.jpg", "7680.jpg", "7681.jpg", "7682.jpg"], "answer": "B", "solution": "null", "level": "高一", "question": "下列四个正方体图形中, $A, B$ 为正方体的两个顶点, $M, N, P$ 分别为其所在棱的中点,能得出 $A B / /$ 平面 $M N P$ 的图形的是( )\n\n\n\n(1)\n\n\n\n(2)\n\n\n\n(3)\n\n\n\n(4)", "options": "A. (1)(3)\nB. (1)(4)\nC. (2)(3)\nD. (2)(4)", "subject": "立体几何学", "analysis": "对于 (1),如图,连接 $B C$ 交 $P N$ 于点 $D$ ,连接 $M D$.\n\n\n\n由 $M D / / A B, A B 千$ 平面 $M N P , M D \\subset$ 平面 $M N P$ ,得 $A B / /$ 平面 $M N P$.\n\n对于(4),由 $A B / / N P , A B \\varangle$ 平面 $M N P , N P \\subset$ 平面 $M N P$ ,可得 $A B / /$ 平面 $M N P$."} {"id": "15463", "image": ["7683.jpg"], "answer": "C", "solution": "null", "level": "高一", "question": "如图所示, 四边形 $E F G H$ 为四面体 $A B C D$ 的一个截面, 若 $\\frac{A E}{C E}=\\frac{B F}{F C}=\\frac{B G}{G D}$, 则与平面 $E F G H$平行的直线有 $(\\quad)$\n\n", "options": "0 条\nB. 1 条\nC. 2 条\nD. 3 条", "subject": "立体几何学", "analysis": "$\\because \\frac{A E}{C E}=\\frac{B F}{F C} , \\therefore E F / / A B$.\n\n又 $E F \\subset$ 平面 $E F G H , A B \\nmid$ 平面 $E F G H$ ,\n\n$\\therefore A B / /$ 平面 $E F G H$.\n\n同理,由 $\\frac{B F}{F C}=\\frac{B G}{G D}$ ,\n\n可证 $C D / /$ 平面 $E F G H$.\n\n$\\therefore$ 与平面 $E F G H$ 平行的直线有 2 条."} {"id": "15464", "image": ["7684.jpg", "7685.jpg"], "answer": "B", "solution": "null", "level": "高一", "question": "如图, 已知三棱柱 $A B C-A_{1} B_{1} C_{1}$ 中, $E$ 是 $B C$ 的中点, $D$ 是 $A A_{1}$ 上的动点, 且 $\\frac{A D}{D A_{1}}=m$,若 $A E / /$ 平面 $D B_{1} C$, 则 $m$ 的值为 $($ )\n\n", "options": "$\\frac{1}{2}$\nB. $1 \\mathrm{C} \\cdot \\frac{3}{2}$\nD. 2", "subject": "立体几何学", "analysis": ",取 $C B_{1}$ 的中点 $G$ ,连接 $G E , D G$ ,当 $m=1$ 时, $A D=G E=\\frac{1}{2} B B_{1}$ 且 $A D / / G E$ ,\n\n$\\therefore$ 四边形 $A D G E$ 为平行四边形, 则 $A E / / D G$ ,又 $A E \\subset$ 平面 $D B_{1} C, D G \\subset$ 平面 $D B_{1} C$ ,可得 $A E / /$平面 $D B_{1} C$.\n\n"} {"id": "15471", "image": ["7691.jpg"], "answer": "A", "solution": "null", "level": "高一", "question": "如图所示, 设 $E, F, E_{1}, F_{1}$ 分别是长方体 $A B C D-A_{1} B_{1} C_{1} D_{1}$ 的棱 $A B, C D, A_{1} B_{1}, C_{1} D_{1}$的中点, 则平面 $E F D_{1} A_{1}$ 与平面 $B C F_{1} E_{1}$ 的位置关系是( )\n\n", "options": "A. 平行\nB. 相交\nC. 异面\nD. 不确定", "subject": "立体几何学", "analysis": "$\\because A_{1} E / / B E_{1} , A_{1} E$ 平面 $B C F_{1} E_{1} , B E_{1} \\subset$ 平面 $B C F_{1} E_{1}$ ,\n\n$\\therefore A_{1} E / /$ 平面 $B C F_{1} E_{1}$.\n\n同理, $A_{1} D_{1} / /$ 平面 $B C F_{1} E_{1}$.\n\n又 $A_{1} E \\cap A_{1} D_{1}=A_{1}, A_{1} E, A_{1} D_{1} \\subset$ 平面 $E F D_{1} A_{1}$ ,\n\n$\\therefore$ 平面 $E F D_{1} A_{1} / /$ 平面 $B C F_{1} E_{1}$."} {"id": "15472", "image": ["7692.jpg"], "answer": "D", "solution": "null", "level": "高一", "question": "六棱柱 $A B C D E F-A_{1} B_{1} C_{1} D_{1} E_{1} F_{1}$ 的底面是正六边形, 则此六棱柱的面中互相平行的有 ( )", "options": "1 对\nB. 2 对\nC. 3 对\nD. 4 对", "subject": "立体几何学", "analysis": "由图知平面 $A B B_{1} A_{1} / /$ 平面 $E D D_{1} E_{1}$ , 平面 $B C C_{1} B_{1} / /$ 平面 $F E E_{1} F_{1}$ ,平面 $A F F_{1} A_{1} / /$ 平面 $C D D_{1} C_{1}$ ,平面 $A B C D E F / /$ 平面 $A_{1} B_{1} C_{1} D_{1} E_{1} F_{1} ,$\n\n$\\therefore$ 此六棱柱的面中互相平行的有 4 对.\n\n"} {"id": "15473", "image": ["7693.jpg"], "answer": "C", "solution": "null", "level": "高一", "question": "在正方体 $A B C D-A_{1} B_{1} C_{1} D_{1}$ 中, $M$ 为棱 $A_{1} D_{1}$ 的动点, $O$ 为底面 $A B C D$ 的中心, 点 $E, F$分别是 $A_{1} B_{1}, C_{1} D_{1}$ 的中点, 下列平面中与 $O M$ 扫过的平面平行的是 $(\\quad)$", "options": "平面 $A B B_{1} A_{1}$\nB. 平面 $B C C_{1} B_{1}$\nC. 平面 $B C F E$\nD. 平面 $D C C_{1} D_{1}$", "subject": "立体几何学", "analysis": "取 $A B , D C$ 的中点分别为点 $E_{1}$ 和点 $F_{1}$ ,连接 $E_{1} F_{1}$ ,则 $E_{1} F_{1}$ 过点 $O , O M$ 扫过的平面即为平面 $A_{1} E_{1} F_{1} D_{1}$ (如图),\n\n\n\n故平面 $A_{1} E_{1} F_{1} D_{1} / /$ 平面 $B C F E$."} {"id": "15474", "image": [], "answer": "B", "solution": "null", "level": "高一", "question": "经过平面 $\\alpha$ 外两点, 作与 $\\alpha$ 平行的平面, 则这样的平面可以作 $($ )", "options": "A. 1 个或 2 个\nB. 0 个或 1 个\nC. 1 个\nD. 0 个", "subject": "立体几何学", "analysis": "(1)当经过两点的直线与平面 $\\alpha$ 平行时,可作出一个平面 $\\beta$ 使 $\\beta / / \\alpha$.\n\n(2)当经过两点的直线与平面 $\\alpha$ 相交时,由于作出的平面与平面 $\\alpha$ 至少有一个公共点,故经过两点的平面都与平面 $\\alpha$ 相交,不能作出与平面 $\\alpha$ 平行的平面. 故满足条件的平面有 0 个或 1 个."} {"id": "15475", "image": ["7694.jpg"], "answer": "D", "solution": "null", "level": "高一", "question": "已知立方体 $A B C D-A^{\\prime} B^{\\prime} C^{\\prime} D^{\\prime}$, 点 $E, F, G, H$ 分别是棱 $A D, B B^{\\prime}, B^{\\prime} C^{\\prime}, D D^{\\prime}$的中点, 从中任取两点确定的直线中, 与平面 $A B^{\\prime} D^{\\prime}$ 平行的条数是 ( )\n\n", "options": "0\nB. 2\nC. 4\nD. 6", "subject": "立体几何学", "analysis": "连接 $E G, E H, E F, F G, G H$ ,\n\n$\\because E H / / F G$ 且 $E H=F G ,$\n\n$\\therefore$ 四边形 $E F G H$ 为平行四边形,\n\n$\\therefore E, F, G, H$ 四点共面.\n\n由 $E G / / A B^{\\prime}, E H / / A D^{\\prime}, E G \\cap E H=E, A B^{\\prime} \\cap A D^{\\prime}=A, E G, E H \\subset$ 平面 $E F G H, A B^{\\prime}$ , $A D^{\\prime} \\subset$ 平面 $A B^{\\prime} D^{\\prime} ,$\n\n可得平面 $E F G H / /$ 平面 $A B^{\\prime} D^{\\prime}$.\n\n故平面 $E F G H$ 内的每条直线都符合条件.\n\n故选 D."} {"id": "15476", "image": ["7695.jpg"], "answer": "A", "solution": "null", "level": "高一", "question": "在正方体 $A B C D-A_{1} B_{1} C_{1} D_{1}$ 中, 点 $E, F, G$ 分别是棱 $A_{1} B_{1}, B_{1} C_{1}, B B_{1}$ 的中点, 给出下列四个推断:\n\n\n\n(1) $F G / /$ 平面 $A A_{1} D_{1} D$; (2) $E F / /$ 平面 $B C_{1} D_{1}$; (3) $F G / /$ 平面 $B C_{1} D_{1}$; (4) 平面 $E F G / /$ 平面 $B C_{1} D_{1}$.其中推断正确的序号是( )", "options": "A. (1)(3)\nB. (1)(4)\nC. (2)(3)\nD. (2)(4)", "subject": "立体几何学", "analysis": "$\\because$ 在正方体 $A B C D-A_{1} B_{1} C_{1} D_{1}$ 中,点 $E , F , G$ 分别是棱 $A_{1} B_{1} , B_{1} C_{1} , B B_{1}$ 的中点,\n\n$\\therefore F G / / B C_{1}$.\n\n$\\because B C_{1} / / A D_{1}, \\therefore F G / / A D_{1}$,\n\n又 $\\because F G \\Phi$ 平面 $A A_{1} D_{1} D, A D_{1} \\subset$ 平面 $A A_{1} D_{1} D$ ,\n\n$\\therefore F G / /$ 平面 $A A_{1} D_{1} D$ ,故(1)正确;\n\n$\\because E F / / A_{1} C_{1}, A_{1} C_{1}$ 与平面 $B C_{1} D_{1}$ 相交,\n\n$\\therefore E F$ 与平面 $B C_{1} D_{1}$ 相交,故(2)错误;\n\n$\\because F G / / B C_{1}, F G$ 平面 $B C_{1} D_{1}, B C_{1} \\subset$ 平面 $B C_{1} D_{1}$ ,\n$\\therefore F G / /$ 平面 $B C_{1} D_{1}$ ,故(3)正确;\n\n$\\because E F$ 与平面 $B C_{1} D_{1}$ 相交, $\\therefore$ 平面 $E F G$ 与平面 $B C_{1} D_{1}$ 相交,故(4)错误. 故选 A."} {"id": "15477", "image": ["7696.jpg", "7697.jpg"], "answer": "C", "solution": "null", "level": "高一", "question": "如图是四棱雉的平面展开图, 其中四边形 $A B C D$ 为正方形, 点 $E, F, G, H$ 分别为 $P A$, $P D, P C, P B$ 的中点, 在此几何体中, 给出下面四个结论:\n\n\n\n(1)平面 $E F G H / /$ 平面 $A B C D$;\n\n(2) $B C / /$ 平面 $P A D$;\n\n(3) $A B / /$ 平面 $P C D$;\n\n(4)平面 $P A D / /$ 平面 $P A B$.\n\n其中正确的有( )", "options": "A. (1)(3)\nB. (1)(4)\nC. (1)(2)(3)\nD. (2)(3)", "subject": "立体几何学", "analysis": "面展开图还原为四棱雉如图所示,则 $E H / / A B$ ,又 $E H \\Phi$ 平面 $A B C D , A B \\subset$ 平面 $A B C D$ ,所以 $E H / /$ 平面 $A B C D$. 同理可证 $E F / /$ 平面 $A B C D$ ,又 $E F \\cap E H=E , E F , E H \\subset$ 平面 $E F G H$ ,所以平面 $E F G H / /$ 平面 $A B C D$ ; 平面 $P A D \mathrm{~ 平 面 ~} P B C$ ,平面 $P A B \mathrm{~ 平 面 ~} P D C$ 均是四棱锥的四个侧面,则它们两两相交.\n\n\n\n$\\because A B / / C D , A B \\varangle$ 平面 $P C D, C D \\subset$ 平面 $P C D ,$\n\n$\\therefore A B / /$ 平面 $P C D$.\n\n同理 $B C / /$ 平面 $P A D$."} {"id": "15479", "image": ["7698.jpg"], "answer": "B", "solution": "null", "level": "高一", "question": "如图, 已知 $S$ 为四边形 $A B C D$ 外一点, 点 $G, H$ 分别为 $S B, B D$ 上的点, 若 $G H / /$ 平面 $S C D$, 则 ( )\n\n", "options": "$G H / / S A$\n\nB. $G H / / S D$\n\nC. $G H / / S C$\n\nD. 以上均有可能", "subject": "立体几何学", "analysis": "因为 $G H / /$ 平面 $S C D , G H \\subset$ 平面 $S B D$ ,平面 $S B D \\cap$ 平面 $S C D=S D$ ,所以 $G H / / S D \mathrm{~ , ~}$显然 $G H$ 与 $S A, S C$ 均不平行,故选 B."} {"id": "15483", "image": [], "answer": "C", "solution": "null", "level": "高一", "question": "直线 $a / /$ 平面 $\\alpha, P \\in \\alpha$, 过点 $P$ 平行于 $a$ 的直线( )", "options": "只有一条, 不在平面 $\\alpha$ 内\n\nB. 有无数条, 不一定在 $\\alpha$ 内\n\nC. 只有一条, 且在平面 $\\alpha$ 内\n\nD. 有无数条, 一定在 $\\alpha$ 内", "subject": "立体几何学", "analysis": "由线面平行性质定理知过点 $P$ 平行于 $a$ 的直线只有一条,且在平面 $\\alpha$ 内,故选 C."} {"id": "15484", "image": [], "answer": "C", "solution": "null", "level": "高一", "question": "对于直线 $m, n$ 和平面 $\\alpha$, 下列命题中正确的是 ( )", "options": "A. 如果 $m \\subset \\alpha, n \\nmid \\alpha, m, n$ 是异面直线, 那么 $n / / \\alpha$\n\nB. 如果 $m \\subset \\alpha, n \\Varangle \\alpha, m, n$ 是异面直线, 那么 $n$ 与 $\\alpha$ 相交\n\nC. 如果 $m \\subset \\alpha, n / / \\alpha, m, n$ 共面, 那么 $m / / n$\n\nD. 如果 $m / / \\alpha, n / / \\alpha, m, n$ 共面, 那么 $m / / n$", "subject": "立体几何学", "analysis": "由线面平行的性质定理知 C 正确."} {"id": "15485", "image": ["7704.jpg"], "answer": "A", "solution": "null", "level": "高一", "question": "如图, 在长方体 $A B C D-A_{1} B_{1} C_{1} D_{1}$ 中, 点 $E, F$ 分别是棱 $A A_{1}$ 和 $B B_{1}$ 的中点, 过 $E F$ 的平面 $E F G H$ 分别交 $B C$ 和 $A D$ 于点 $G, H$, 则 $G H$ 与 $A B$ 的位置关系是( )\n\n", "options": "A. 平行\nB. 相交\nC. 异面\nD. 平行或异面", "subject": "立体几何学", "analysis": "由长方体性质知: $E F / /$ 平面 $A B C D , \\because E F \\subset$ 平面 $E F G H$ ,平面 $E F G H \\cap$ 平面 $A B C D$ $=G H, \\therefore E F / / G H$.\n\n又 $\\because E F / / A B, \\therefore G H / / A B$."} {"id": "15486", "image": [], "answer": "D", "solution": "null", "level": "高一", "question": "在空间四边形 $A B C D$ 中, $E, F, G, H$ 分别是 $A B, B C, C D, D A$ 上的点, 当 $B D / /$ 平面 $E F G H$ 时, 下面结论正确的是( )", "options": "$E, F, G, H$ 一定是各边的中点\n\nB. $G, H$ 一定是 $C D, D A$ 的中点\n\nC. $B E: E A=B F: F C$, 且 $D H: H A=D G: G C$\n\nD. $A E: E B=A H: H D$, 且 $B F: F C=D G: G C$", "subject": "立体几何学", "analysis": "由于 $B D / /$ 平面 $E F G H$, 所以有 $B D / / E H, B D / / F G$, 则 $A E: E B=A H: H D$, 且 $B F: F C$ $=D G: G C$."} {"id": "15487", "image": ["7705.jpg", "7706.jpg"], "answer": "A", "solution": "null", "level": "高一", "question": "已知正方体 $A C_{1}$ 的棱长为 1 , 点 $P$ 是面 $A A_{1} D_{1} D$ 的中心, 点 $Q$ 是面 $A_{1} B_{1} C_{1} D_{1}$ 的对角线 $B_{1} D_{1}$ 上一点, 且 $P Q / /$ 平面 $A A_{1} B_{1} B$, 则线段 $P Q$ 的长为 $(\\quad)$\n\n", "options": "$\\frac{\\sqrt{2}}{2}$\nB. $\\frac{\\sqrt{3}}{2}$\nC. 1\nD. $\\sqrt{2}$", "subject": "立体几何学", "analysis": "如图,连接 $A D_{1}, A B_{1}$ ,\n\n\n\n$\\because P Q / /$ 平面 $A A_{1} B_{1} B$,\n\n平面 $A B_{1} D_{1} \\cap$ 平面 $A A_{1} B_{1} B=A B_{1}$ ,\n\n$P Q \\subset$ 平面 $A B_{1} D_{1}, \\therefore P Q / / A B_{1}$ ,\n\n$\\therefore P Q=\\frac{1}{2} A B_{1}=\\frac{1}{2} \\sqrt{1^{2}+1^{2}}=\\frac{\\sqrt{2}}{2}$."} {"id": "15488", "image": ["7707.jpg"], "answer": "C", "solution": "null", "level": "高一", "question": "如图, 四棱雉 $S-A B C D$ 的所有的棱长都等于 2 , 点 $E$ 是 $S A$ 的中点, 过 $C, D, E$ 三点的平面与 $S B$ 交于点 $F$, 则四边形 $D E F C$ 的周长为 $(\\quad)$\n\n", "options": "$2+\\sqrt{3}$\nB. $3+\\sqrt{3}$\nC. $3+2 \\sqrt{3}$\nD. $2+2 \\sqrt{3}$", "subject": "立体几何学", "analysis": "because C D / / A B, C D \\Phi$ 平面 $S A B, A B \\subset$ 平面 $S A B$ ,\n\n$\\therefore C D / /$ 平面 $S A B$.\n\n又平面 $C D E F \\cap$ 平面 $S A B=E F , \\therefore C D / / E F$ ,\n又 $C D / / A B, \\therefore A B / / E F$.\n\n$\\because S E=E A, \\therefore E F$ 为 $\\triangle A B S$ 的中位线,\n\n$\\therefore E F=\\frac{1}{2} A B=1$, 又 $D E=C F=\\sqrt{3}$,\n\n$\\therefore$ 四边形 $D E F C$ 的周长为 $3+2 \\sqrt{3}$."} {"id": "15491", "image": [], "answer": "A", "solution": "null", "level": "高一", "question": "两个平行平面与另两个平行平面相交所得四条直线的位置关系是 ( )", "options": "两两相互平行\n\nB. 两两相交于同一点\n\nC. 两两相交但不一定交于同一点\n\nD. 两两相互平行或交于同一点", "subject": "立体几何学", "analysis": "可以想象四棱柱. 由面面平行的性质定理可得."} {"id": "15496", "image": [], "answer": "C", "solution": "null", "level": "高一", "question": "下列命题:\n\n(1)一条直线与两个平行平面中的一个平面相交, 必与另外一个平面相交;\n\n(2)如果一个平面平行于两个平行平面中的一个平面, 必平行于另一个平面;\n\n(3)夹在两个平行平面间的平行线段相等.\n\n其中正确的命题的个数为 ( )", "options": "A. 1\nB. 2\nC. 3\nD. 0", "subject": "立体几何学", "analysis": "根据面面平行的性质知(1)(2)(3)正确,故选 C."} {"id": "15497", "image": [], "answer": "D", "solution": "null", "level": "高一", "question": "平面 $\\alpha / /$ 平面 $\\beta$, 点 $A, C \\in \\alpha, B, D \\in \\beta$, 则直线 $A C / /$ 直线 $B D$ 的充要条件是 $(\\quad)$", "options": "$A B / / C D$\nB. $A D / / C B$\nC. $A B$ 与 $C D$ 相交\nD. $A, B, C, D$ 四点共面", "subject": "立体几何学", "analysis": "充分性: $A, B, C, D$ 四点共面,由平面与平面平行的性质知 $A C / / B D$. 必要性显然成立."} {"id": "15498", "image": ["7713.jpg"], "answer": "B", "solution": "null", "level": "高一", "question": "如图所示, $P$ 是三角形 $A B C$ 所在平面外一点, 平面 $\\alpha / /$ 平面 $A B C, \\alpha$ 分别交线段 $P A, P B$, $P C$ 于 $A^{\\prime}, B^{\\prime}, C^{\\prime}$, 若 $P A^{\\prime}: A A^{\\prime}=2: 3$, 则 $S_{\\triangle A^{\\prime} B^{\\prime} C^{\\prime}}: S_{\\triangle A B C}$ 等于( )\n\n", "options": "A. $2: 25$\nB. $4: 25$\nC. $2: 5$\nD. $4: 5$", "subject": "立体几何学", "analysis": "$\\because$ 平面 $\\alpha / /$ 平面 $A B C$ ,平面 $P A B$ 与它们的交线分别为 $A^{\\prime} B^{\\prime} , A B , \\therefore A B / / A^{\\prime} B^{\\prime}$ ,同理 $B^{\\prime} C^{\\prime} / / B C$ ,易得 $\\triangle A B C \\sim \\triangle A^{\\prime} B^{\\prime} C^{\\prime}$ ,\n\n$S_{\\triangle A^{\\prime} B^{\\prime} C^{\\prime}}: S_{\\triangle A B C}=\\left(\\frac{A^{\\prime} B^{\\prime}}{A B}\\right)^{2}=\\left(\\frac{P A^{\\prime}}{P A}\\right)^{2}=\\frac{4}{25}$."} {"id": "15499", "image": [], "answer": "C", "solution": "null", "level": "高一", "question": "$\\alpha, \\beta, \\gamma$ 为三个不重合的平面, $a, b, c$ 为三条不同的直线, 则下列命题中不正确的是 ( )\n\n(1) $\\left.\\begin{array}{c}a / / c \\\\ b / / c\\end{array}\\right\\} \\Rightarrow a / / b$;\n\n(2) $\\left.\\begin{array}{c}a / / \\gamma \\\\ b / / \\gamma\\end{array}\\right\\} \\Rightarrow a / / b$;\n\n(3) $\\left.\\begin{array}{l}\\alpha / / c \\\\ \\beta / / c\\end{array}\\right\\} \\Rightarrow \\alpha / / \\beta$;\n\n(4) $\\left.\\begin{array}{l}\\alpha / / \\gamma \\\\ \\beta / / \\gamma\\end{array}\\right\\} \\Rightarrow \\alpha / / \\beta$;\n\n(5) $\\left.\\begin{array}{c}\\alpha / / c \\\\ a / / c\\end{array}\\right\\} \\Rightarrow \\alpha / / a$;\n\n(6) $\\left.\\begin{array}{c}\\alpha / / \\gamma \\\\ a / / \\gamma\\end{array}\\right\\} \\Rightarrow a / / \\alpha$.", "options": "A. (4)(6)\nB. (2)(3)(6)\nC. (2)(3)(5)(6)\nD. (2)(3)", "subject": "立体几何学", "analysis": "由公理 4 及平行平面的传递性知(1)(4)正确. 举反例知(2)(3)(5)(6)不正确. (2)中 $a, b$ 可以相交,还可以异面;(3)中 $\\alpha, \\beta$ 可以相交;(5)中 $a$ 可以在 $\\alpha$ 内;(6)中 $a$ 可以在 $\\alpha$ 内."} {"id": "15500", "image": ["7714.jpg"], "answer": "B", "solution": "null", "level": "高一", "question": "在棱长为 2 的正方体 $A B C D-A_{1} B_{1} C_{1} D_{1}$ 中, $M$ 是棱 $A_{1} D_{1}$ 的中点, 过 $C_{1}, B, M$ 作正方体的截面, 则这个截面的面积为( )", "options": "A. $\\frac{3 \\sqrt{5}}{2}$\nB. $\\frac{9}{2}$\nC. $\\frac{9}{8}$\nD. $\\frac{3 \\sqrt{5}}{8}$", "subject": "立体几何学", "analysis": "取 $A A_{1}$ 的中点 $N$ ,连接 $M N , N B , M C_{1} , B C_{1}$ ,\n\n\n\n由于截面被平行平面所截,所以截面为梯形,\n\n且 $M N=\\frac{1}{2} B C_{1}=\\sqrt{2}, M C_{1}=B N=\\sqrt{5}$,\n$\\therefore$ 梯形的高为 $\\frac{3}{\\sqrt{2}}$ ,\n\n$\\therefore$ 梯形的面积为 $\\frac{1}{2}(\\sqrt{2}+2 \\sqrt{2}) \\times \\frac{3}{\\sqrt{2}}=\\frac{9}{2}$."} {"id": "15501", "image": ["7715.jpg"], "answer": "D", "solution": "null", "level": "高一", "question": "设 $\\alpha / / \\beta, A \\in \\alpha, B \\in \\beta, C$ 是 $A B$ 的中点, 当 $A, B$ 分别在平面 $\\alpha, \\beta$ 内运动时, 得到无数个 $A B$ 的中点 $C$, 那么所有的动点 $C(\\quad)$", "options": "A. 不共面\n\nB. 当且仅当 $A, B$ 分别在两条直线上移动时才共面\n\nC. 当且仅当 $A, B$ 分别在两条给定的异面直线上移动时才共面\n\nD. 不论 $A, B$ 如何移动, 都共面", "subject": "立体几何学", "analysis": "如图所示, $A^{\\prime} , B^{\\prime}$ 分别是 $A, B$ 两点在 $\\alpha, \\beta$ 上运动后的两点,此时 $A B$ 的中点 $C$变成 $A^{\\prime} B^{\\prime}$ 的中点 $C^{\\prime}$ ,连接 $A^{\\prime} B$ ,取 $A^{\\prime} B$ 的中点 $E$. 连接 $C E , C^{\\prime} E , A A^{\\prime} , B B^{\\prime}$ , $C C^{\\prime}$ ,则 $C E / / A A^{\\prime}$ ,\n\n\n\n又 $C E \\Varangle \\alpha, A A^{\\prime} \\subset \\alpha, \\therefore C E / / \\alpha$.\n\n又 $C^{\\prime} E / / B B^{\\prime} , C^{\\prime} E \\not \\beta, B B^{\\prime} \\subset \\beta, \\therefore C^{\\prime} E / / \\beta$.\n\n又 $\\because \\alpha / / \\beta, C^{\\prime} E \\Varangle \\alpha, \\therefore C^{\\prime} E / / \\alpha$.\n\n$\\because C^{\\prime} E \\cap C E=E, C^{\\prime} E, C E \\subset$ 平面 $C C^{\\prime} E$ ,\n\n$\\therefore$ 平面 $C C^{\\prime} E / /$ 平面 $\\alpha$,\n\n$\\therefore C C^{\\prime} / /$ 平面 $\\alpha$.\n\n$\\therefore$ 不论 $A , B$ 如何移动,所有的动点 $C$ 都在过 $C$ 点且与 $\\alpha, \\beta$ 平行的平面上."} {"id": "15502", "image": ["7716.jpg"], "answer": "D", "solution": "null", "level": "高一", "question": "如图, 在正方体 $A B C D-A_{1} B_{1} C_{1} D_{1}$ 中, $E, F$ 分别为棱 $A B, C C_{1}$ 的中点, 在平面 $A D D_{1} A_{1}$内且与平面 $D_{1} E F$ 平行的直线( )\n\n", "options": "A. 不存在\nB. 有 1 条\nC. 有 2 条\nD. 有无数条", "subject": "立体几何学", "analysis": "平面 $D_{1} E F$ 与平面 $A D D_{1} A_{1}$ 相交,则在平面 $A D D_{1} A_{1}$ 内与这两个平面的交线平行且不重合的直线有无数条,这些直线都与平面 $D_{1} E F$ 平行."} {"id": "15504", "image": ["7718.jpg"], "answer": "B", "solution": "null", "level": "高一", "question": "如图, 在正方形 $A B C D$ 中, $E, F$ 分别是 $B C, C D$ 的中点, $G$ 是 $E F$ 的中点, 现在沿 $A E$, $A F$ 及 $E F$ 把这个正方形折成一个空间图形, 使 $B, C, D$ 三点重合, 重合后的点记为 $H$, 那么, 在这个空间图形中必有( )\n", "options": "$A G \\perp \\triangle E F H$ 所在平面\n\nB. $A H \\perp \\triangle E F H$ 所在平面\n\nC. $H F \\perp \\triangle A E F$ 所在平面\n\nD. $H G \\perp \\triangle A E F$ 所在平面", "subject": "立体几何学", "analysis": "根据折叠前、后 $A H \\perp H E , A H \\perp H F$ 不变,\n\n$\\therefore A H \\perp$ 平面 $E F H$ , B 正确;"} {"id": "15509", "image": ["7723.jpg"], "answer": "D", "solution": "null", "level": "高一", "question": "正方体 $A B C D-A_{1} B_{1} C_{1} D_{1}$ 中与 $A D_{1}$ 垂直的平面是( )", "options": "平面 $D D_{1} C_{1} C$\nB. 平面 $A_{1} D B$\nC. 平面 $A_{1} B_{1} C_{1} D_{1}$\nD. 平面 $A_{1} D B_{1}$\n\n", "subject": "立体几何学", "analysis": "$\\because A D_{1} \\perp A_{1} D , A D_{1} \\perp A_{1} B_{1} , A_{1} D \\cap A_{1} B_{1}=A_{1}$ ,\n\n$\\therefore A D_{1} \\perp$ 平面 $A_{1} D B_{1}$.\n\n故选 D."} {"id": "15510", "image": ["7724.jpg"], "answer": "C", "solution": "null", "level": "高一", "question": "如图所示, 如果 $M C \\perp$ 菱形 $A B C D$ 所在平面, 那么 $M A$ 与 $B D$ 的位置关系是 (\n\n", "options": "平行\nB. 垂直相交\nC. 垂直但不相交\nD. 相交但不垂直", "subject": "立体几何学", "analysis": "连接 $A C$. 因为 $A B C D$ 是菱形,所以 $B D \\perp A C$. 又 $M C \\perp$ 平面 $A B C D$ ,则 $B D \\perp M C$. 因为 $A C \\cap M C=C$ ,所以 $B D \\perp$ 平面 $A M C$. 又 $M A \\subset$ 平面 $A M C ,$ 所以 $M A \\perp B D$. 显然直线 $M A$ 与直线 $B D$ 不共面,因此直线 $M A$ 与 $B D$ 的位置关系是垂直但不相交."} {"id": "15511", "image": [], "answer": "C", "solution": "null", "level": "高一", "question": "在正方体 $A B C D-A_{1} B_{1} C_{1} D_{1}$ 中, 下面结论错误的是( )", "options": "$B D / /$ 平面 $C B_{1} D_{1}$\n\nB. $A C_{1} \\perp B D$\n\nC. $A C_{1} \\perp$ 平面 $C B_{1} D$\n\nD. 异面直线 $A D$ 与 $C B_{1}$ 所成的角为 $45^{\\circ}$", "subject": "立体几何学", "analysis": "由正方体的性质得 $B D / / B_{1} D_{1}$ ,且 $B D q$ 平面 $C B_{1} D_{1}$ ,所以 $B D / /$ 平面 $C B_{1} D_{1}$ ,故 $A$ 正确;因为 $B D \\perp$ 平面 $A C C_{1} A_{1}$ ,所以 $A C_{1} \\perp B D$ ,故 $\\mathrm{B}$ 正确;异面直线 $A D$ 与 $C B_{1}$ 所成的角即为 $A D$ 与 $D A_{1}$ 所成的角,故为 $45^{\\circ}$ ,所以 $\\mathrm{D}$ 正确."} {"id": "15512", "image": [], "answer": "B", "solution": "null", "level": "高一", "question": "下列说法中, 正确的有 ( )\n\n(1)如果一条直线垂直于平面内的两条直线,那么这条直线和这个平面垂直;\n\n(2)过直线 $l$ 外一点 $P$, 有且仅有一个平面与 $l$ 垂直;\n\n(3)如果三条共点直线两两垂直, 那么其中一条直线垂直于另两条直线确定的平面;\n\n(4)垂直于角的两边的直线必垂直角所在的平面;\n\n(5)过点 $A$ 垂直于直线 $a$ 的所有直线都在过点 $A$ 垂直于 $a$ 的平面内.", "options": "A. 2 个\nB. 3 个\nC. 4 个\nD. 5 个", "subject": "立体几何学", "analysis": "(1)(4)不正确,其他三项均正确."} {"id": "15513", "image": ["7725.jpg"], "answer": "D", "solution": "null", "level": "高一", "question": "如图, 在三棱雉 $P-A B C$ 中, $P A \\perp$ 平面 $A B C, A B \\perp B C, P A=A B, D$ 为 $P B$ 的中点, 则下列结论正确的有( )\n\n\n\n(1) $B C \\perp$ 平面 $P A B$;\n\n(2) $A D \\perp P C$;\n\n(3) $A D \\perp$ 平面 $P B C$;\n\n(4) $P B \\perp$ 平面 $A D C$.", "options": "A. 0 个\nB. 1 个\nC. 2 个\nD. 3 个", "subject": "立体几何学", "analysis": "$\\because P A \\perp$ 平面 $A B C$ ,\n\n$\\therefore P A \\perp B C$,\n\n又 $B C \\perp A B , P A \\cap A B=A$ ,\n\n$\\therefore B C \\perp$ 平面 $P A B$ ,\n\n故(1)确;\n\n由 $B C \\perp$ 平面 $P A B$ ,\n\n得 $B C \\perp A D$ ,\n\n又 $P A=A B, D$ 是 $P B$ 的中点,\n\n$\\therefore A D \\perp P B$ ,又 $P B \\cap B C=B , P B , B C \\subset$ 平面 $P B C$ ,\n\n$\\therefore A D \\perp$ 平面 $P B C$ ,\n\n$\\therefore A D \\perp P C$ ,故(2)正确;\n\n由 $A D \\perp$ 平面 $P B C$ ,\n\n$\\therefore$ (3)正确. 故选 D."} {"id": "15514", "image": ["7726.jpg"], "answer": "C", "solution": "null", "level": "高一", "question": "把正方形 $A B C D$ 沿对角线 $A C$ 折起, 当以 $A, B, C, D$ 四点为顶点棱雉体积最大时, 直线 $B D$ 和平面 $A B C$ 所成的角的大小为 $(\\quad)$", "options": "A. $90^{\\circ}$\nB. $60^{\\circ}$\nC. $45^{\\circ}$\nD. $30^{\\circ}$", "subject": "立体几何学", "analysis": ",当 $D O \\perp$ 平面 $A B C$ 时,三棱锥 $D-A B C$ 的体积最大.\n\n\n\n$\\therefore \\angle D B O$ 为直线 $B D$ 和平面 $A B C$ 所成的角,\n\n$\\because$ 在 Rt $\\triangle D O B$ 中, $O D=O B ,$\n$\\therefore$ 直线 $B D$ 和平面 $A B C$ 所成的角大小为 $45^{\\circ}$."} {"id": "15522", "image": [], "answer": "C", "solution": "null", "level": "高一", "question": "已知直线 $m, n$ 与平面 $\\alpha, \\beta$, 给出下列三个结论:\n\n(1)若 $m / / \\alpha, n / / \\alpha$, 则 $m / / n$; (2)若 $m / / \\alpha, n \\perp \\alpha$, 则 $m \\perp n$; (3)若 $m \\perp \\alpha, m / / \\beta$, 则 $\\alpha \\perp \\beta$.其中正确结论的个数是( )", "options": "0\nB. 1\nC. 2\nD. 3", "subject": "立体几何学", "analysis": "(1)若 $m / / \\alpha , n / / \\alpha$ ,则 $m$ 与 $n$ 可能平行、相交或异面,故(1)错误;易知(2)(3)正确. 所以正确结论的个数是 2 ."} {"id": "15523", "image": ["7733.jpg"], "answer": "C", "solution": "null", "level": "高一", "question": "如图所示, 在四面体 $D-A B C$ 中, 若 $A B=B C, A D=C D, E$ 是 $A C$ 的中点, 则下列命题中正确的是( )\n\n", "options": "平面 $A B C \\perp$ 平面 $A B D$\n\nB. 平面 $A B D \\perp$ 平面 $B D C$\n\nC. 平面 $A B C \\perp$ 平面 $B D E$, 且平面 $A D C \\perp$ 平面 $B D E$\n\nD. 平面 $A B C \\perp$ 平面 $A D C$, 且平面 $A D C \\perp$ 平面 $B D E$", "subject": "立体几何学", "analysis": "因为 $A B=B C$ ,且 $E$ 是 $A C$ 的中点,所以 $B E \\perp A C$. 同理, $D E \\perp A C$. 又 $B E \\cap D E=E$ ,所以 $A C \\perp$ 平面 $B D E$.\n\n因为 $A C \\subset$ 平面 $A B C$ ,所以平面 $A B C \\perp$ 平面 $B D E$.\n\n因为 $A C \\subset$ 平面 $A C D ,$ 所以平面 $A C D \\perp$ 平面 $B D E$."} {"id": "15524", "image": [], "answer": "C", "solution": "null", "level": "高一", "question": "过两点与一个已知平面垂直的平面( )", "options": "有且只有一个\nB. 有无数个\nC. 有且只有一个或无数个\nD. 可能不存在", "subject": "立体几何学", "analysis": "若过两点的直线与已知平面垂直时,此时过这两点有无数个平面与已知平面垂直,若过两点的直线与已知平面不垂直时,则有且只有一个过这两点的平面与已知平面垂直."} {"id": "15525", "image": ["7734.jpg"], "answer": "A", "solution": "null", "level": "高一", "question": "在四面体 $A-B C D$ 中, $A B=B C=C D=A D, \\angle B A D=\\angle B C D=90^{\\circ}, A-B D-C$ 为直二面角, $E$ 是 $C D$ 的中点, 则 $\\angle A E D$ 等于( )", "options": "A. $90^{\\circ}$\nB. $45^{\\circ}$\nC. $60^{\\circ}$\nD. $30^{\\circ}$", "subject": "立体几何学", "analysis": "如图,设 $A B=B C=C D=A D=a$ ,取 $B D$ 中点 $F$ ,连接 $A F , C F$.\n\n\n\n由题意可得 $A F=C F=\\frac{\\sqrt{2}}{2} a, \\angle A F C=90^{\\circ}$.\n\n在 Rt $\\triangle A F C$ 中,可得 $A C=a ,$\n\n$\\therefore \\triangle A C D$ 为正三角形.\n\n$\\because E$ 是 $C D$ 的中点,\n\n$\\therefore A E \\perp C D$,\n\n$\\therefore \\angle A E D=90^{\\circ}$ ,故选 $\\mathrm{A}$."} {"id": "15526", "image": ["7735.jpg"], "answer": "C", "solution": "null", "level": "高一", "question": "在正方体 $A B C D-A_{1} B_{1} C_{1} D_{1}$ 中, 截面 $A_{1} B D$ 与底面 $A B C D$ 所成二面角 $A_{1}-B D-A$ 的正切值为( )", "options": "A. $\\frac{\\sqrt{3}}{2}$\nB. $\\frac{\\sqrt{2}}{2}$\nC. $\\sqrt{2}$\nD. $\\sqrt{3}$", "subject": "立体几何学", "analysis": "如图所示,连接 $A C$ 交 $B D$ 于点 $O$ ,连接 $A_{1} O , O$ 为 $B D$ 中点,\n\n\n\n$\\because A_{1} D=A_{1} B$,\n\n$\\therefore$ 在 $\\triangle A_{1} B D$ 中, $A_{1} O \\perp B D$.\n又 $\\because$ 在正方形 $A B C D$ 中, $A C \\perp B D$.\n\n$\\therefore \\angle A_{1} O A$ 为二面角 $A_{1}-B D-A$ 的平面角.\n\n设 $A A_{1}=1$ ,则 $A O=\\frac{\\sqrt{2}}{2}$.\n\n$\\therefore \\tan \\angle A_{1} O A=\\frac{1}{\\frac{\\sqrt{2}}{2}}=\\sqrt{2}$."} {"id": "15527", "image": ["7736.jpg", "7737.jpg"], "answer": "B", "solution": "null", "level": "高一", "question": "如图, 在梯形 $A B C D$ 中, $A D / / B C, \\angle A B C=90^{\\circ}, A D: B C: A B=2: 3: 4, E, F$ 分别是 $A B, C D$ 的中点, 将四边形 $A D F E$ 沿直线 $E F$ 进行翻折. 给出四个结论:\n\n\n\n(1) $D F \\perp B C$; (2) $B D \\perp F C$; (3)平面 $D B F \\perp$ 平面 $B F C$; (4)平面 $D C F \\perp$ 平面 $B F C$.\n\n在翻折的过程中, 可能成立的结论是( )", "options": "A. (1)(3)\nB. (2)(3)\nC. (2)(4)\nD. (3)(4)", "subject": "立体几何学", "analysis": "对于(1),因为 $B C / / A D , A D$ 与 $D F$ 相交不垂直,所以 $B C$ 与 $D F$ 不垂直,故(1)不可能成立;对于(2),如图,设点 $D$ 在平面 $B C F$ 上的射影为点 $P$ ,当 $B P \\perp C F$ 时,有 $B D \\perp F C$ ,而 $A D: B C: A B=2: 3: 4$ 可使条件满足,故(2)可能成立; 对于(3),当点 $P$ 落在 $B F$ 上时, $D P$ $\\subset$ 平面 $B D F$ ,从而平面 $B D F \\perp$ 平面 $B C F$ ,故(3)可能成立;对于(4),因为点 $D$ 的射影不可能在 $F C$ 上,故(4)不可能成立,故选 B.\n\n"} {"id": "15528", "image": ["7738.jpg"], "answer": "C", "solution": "null", "level": "高一", "question": "在正四面体 $P-A B C$ 中, $D, E, F$ 分别是 $A B, B C, C A$ 的中点, 下面四个结论中不成立的是( )", "options": "$B C / /$ 平面 $P D F$\n\nB. $D F \\perp$ 平面 $P A E$\n\nC. 平面 $P D F \\perp$ 平面 $A B C$\n\nD. 平面 $P A E \\perp$ 平面 $A B C$", "subject": "立体几何学", "analysis": "所示, $\\because B C / / D F , B C 4$ 平面 $P D F , D F \\subset$ 平面 $P D F$ ,\n\n\n\n$\\therefore B C / /$ 平面 $P D F , \\therefore \\mathrm{A}$ 正确.\n\n由 $B C \\perp P E , B C \\perp A E , P E \\cap A E=E$ ,\n\n得 $B C \\perp$ 平面 $P A E$ ,\n\n$\\therefore D F \\perp$ 平面 $P A E , \\therefore B$ 正确.\n\n$\\therefore$ 平面 $A B C \\perp$ 平面 $P A E(B C \\perp$ 平面 $P A E)$,\n\n$\\therefore \\mathrm{D}$ 正确."} {"id": "15530", "image": [], "answer": "C", "solution": "null", "level": "高一", "question": "设平面 $\\alpha \\perp$ 平面 $\\beta$, 若平面 $\\alpha$ 内的一条直线 $a$ 垂直于平面 $\\beta$ 内的一条直线 $b$, 则 ()", "options": "直线 $a$ 必垂直于平面 $\\beta$\n\nB. 直线 $b$ 必垂直于平面 $\\alpha$\n\nC. 直线 $a$ 不一定垂直于平面 $\\beta$\n\nD. 过 $a$ 的平面与过 $b$ 的平面垂直", "subject": "立体几何学", "analysis": "当两个平面垂直时,在一个平面内只有垂直于交线的直线才垂直于另一个平面."} {"id": "15535", "image": [], "answer": "A", "solution": "null", "level": "高一", "question": "已知 $m, n$ 为两条不同直线, $\\alpha, \\beta$ 为两个不同平面, 给出下列命题:\n\n(1) $\\left\\{\\begin{array}{l}m \\perp \\alpha \\\\ m \\perp n\\end{array} \\Rightarrow n / / \\alpha \\quad\\right.$ (2) $\\left\\{\\begin{array}{l}m \\perp \\beta \\\\ n \\perp \\beta\\end{array} \\Rightarrow m / / n \\quad\\right.$ (3) $\\left\\{\\begin{array}{l}m \\perp \\alpha \\\\ m \\perp \\beta\\end{array} \\Rightarrow \\alpha / / \\beta \\quad\\right.$ (4) $\\begin{cases}m \\subset \\alpha \\\\ n \\subset \\beta & \\Rightarrow m / / n \\\\ \\alpha / / \\beta & \\end{cases}$\n\n其中正确命题的序号是( )", "options": "A. (2)(3)\nB. (3)(4)\nC. (1)(2)\nD. (1)(2)(3)(4)", "subject": "立体几何学", "analysis": "(1)中 $n , \\alpha$ 可能平行或 $n$ 在平面 $\\alpha$ 内; (2)(3)正确; (4)两直线 $m, n$ 平行或异面,故选 A."} {"id": "15536", "image": [], "answer": "D", "solution": "null", "level": "高一", "question": "已知 $l \\perp$ 平面 $\\alpha$, 直线 $m \\subset$ 平面 $\\beta$. 有下面四个命题:\n\n(1) $\\alpha / / \\beta \\Rightarrow l \\perp m$; (2) $\\alpha \\perp \\beta \\Rightarrow I / / m$; (3) $I / / m \\Rightarrow \\alpha \\perp \\beta$; (4) $l \\perp m \\Rightarrow \\alpha / / \\beta$.\n\n其中正确的两个命题是 (", "options": "A. (1)(2)\nB. (3)(4)\nC. (2)(4)\nD. (1)(3)", "subject": "立体几何学", "analysis": "$\\because l \\perp \\alpha , \\alpha / / \\beta \therefore l \\perp \\beta, \\because m \\subset \\beta, \\therefore l \\perp m$ ,故(1)正确; $\\because l / / m, l \\perp \\alpha , \\therefore m \\perp \\alpha$ ,又 $\\because m$ $\\subset \\beta , \\therefore \\alpha \\perp \\beta$ ,故(3)正确."} {"id": "15537", "image": ["7743.jpg"], "answer": "B", "solution": "null", "level": "高一", "question": "如图, 设平面 $\\alpha \\cap$ 平面 $\\beta=P Q, E G \\perp$ 平面 $\\alpha, F H \\perp$ 平面 $\\alpha$, 垂足分别为 $G, H$. 为使 $P Q \\perp G H$,则需增加的一个条件是( )\n\n", "options": "$E F \\perp$ 平面 $\\alpha$\n\nB. $E F \\perp$ 平面 $\\beta$\n\nC. $P Q \\perp G E$\n\nD. $P Q \\perp F H$", "subject": "立体几何学", "analysis": "因为 $E G \\perp$ 平面 $\\alpha, P Q \\subset$ 平面 $\\alpha$ ,所以 $E G \\perp P Q$. 若 $E F \\perp$ 平面 $\\beta$ ,则由 $P Q \\subset$ 平面 $\\beta$ ,得 $E F \\perp P Q$. 又 $E G$ 与 $E F$ 为相交直线,所以 $P Q \\perp$ 平面 $E F H G$ ,所以 $P Q \\perp G H$ ,故选 B."} {"id": "15538", "image": ["7744.jpg"], "answer": "B", "solution": "null", "level": "高一", "question": "如图所示, 三棱雉 $P A B C$ 中, 平面 $A B C \\perp$ 平面 $P A B, P A=P B, A D=D B$, 则( )\n\n", "options": "$P D \\subset$ 平面 $A B C$\n\nB. $P D \\perp$ 平面 $A B C$\n\nC. $P D$ 与平面 $A B C$ 相交但不垂直\n\nD. $P D / /$ 平面 $A B C$", "subject": "立体几何学", "analysis": ""} {"id": "15539", "image": ["7745.jpg"], "answer": "C", "solution": "null", "level": "高一", "question": "如图所示, 在长方体 $A B C D-A_{1} B_{1} C_{1} D_{1}$ 中, $A A_{1}=2 A B, A B=B C$, 则下列结论中正确的是 ( )\n\n", "options": "$B D_{1} / / B_{1} C$\n\nB. $A_{1} D_{1} / /$ 平面 $A B_{1} C$\n\nC. $B D_{1} \\perp A C$\n\nD. $B D_{1} \\perp$ 平面 $A B_{1} C$", "subject": "立体几何学", "analysis": "连接 $B D$.在长方体 $A B C D-A_{1} B_{1} C_{1} D_{1}$ 中, $A B=B C$ ,\n\n$\\therefore A C \\perp B D$.又 $A C \\perp D D_{1} , B D \\cap D D_{1}=D$ ,\n\n$\\therefore A C \\perp$ 平面 $B D D_{1}$.\n\n$\\because B D_{1} \\subset$ 平面 $B D D_{1}$ ,\n\n$\\therefore A C \\perp B D_{1}$.故选 $\\mathrm{C}$."} {"id": "15540", "image": ["7746.jpg", "7747.jpg"], "answer": "A", "solution": "null", "level": "高一", "question": "如图所示, 平面 $\\alpha \\perp$ 平面 $\\beta, A \\in \\alpha, B \\in \\beta, A B$ 与两平面 $\\alpha, \\beta$ 所成的角分别为 $45^{\\circ}$ 和 $30^{\\circ}$.过 $A, B$ 分别作两平面交线的垂线, 垂足分别为 $A^{\\prime}, B^{\\prime}$, 则 $A B: A^{\\prime} B^{\\prime}$ 等于( )\n\n", "options": "A. $2: 1$\nB. $3: 1$\nC. $3: 2$\nD. $4: 3$", "subject": "立体几何学", "analysis": "如图: 由已知得 $A A^{\\prime} \\perp$ 平面 $\\beta$ ,\n\n\n\n$\\angle A B A^{\\prime}=30^{\\circ}, B B^{\\prime} \\perp$ 平面 $\\alpha, \\angle B A B^{\\prime}=45^{\\circ}$.\n\n设 $A B=a$ ,则 $B A^{\\prime}=\\frac{\\sqrt{3}}{2} a B B^{\\prime}=\\frac{\\sqrt{2}}{2} a$ ,\n在 Rt $\\triangle B A^{\\prime} \\quad B^{\\prime}$ 中, $A^{\\prime} \\quad B^{\\prime}=\\frac{1}{2} a , \\therefore \\frac{A B}{A^{\\prime} \\quad B^{\\prime}}=2$."} {"id": "15541", "image": ["7748.jpg"], "answer": "D", "solution": "null", "level": "高一", "question": "如图, 四边形 $A B C D$ 中, $A D / / B C, A D=A B, \\angle B C D=45^{\\circ}, \\angle B A D=90^{\\circ}$, 将 $\\triangle A B D$ 沿 $B D$ 折起, 使平面 $A B D \\perp$ 平面 $B C D$, 构成四面体 $A-B C D$, 则在四面体 $A-B C D$ 中, 下列结论正确的是( )\n\n", "options": "平面 $A B D \\perp$ 平面 $A B C$\n\nB. 平面 $A D C \\perp$ 平面 $B D C$\n\nC. 平面 $A B C \\perp$ 平面 $B D C$\n\nD. 平面 $A D C \\perp$ 平面 $A B C$", "subject": "立体几何学", "analysis": "意得, $B D \\perp C D$ ,\n\n又平面 $A B D \\perp$ 平面 $B C D ,$\n\n且平面 $A B D \\cap$ 平面 $B C D=B D, C D \\subset$ 平面 $B C D ,$\n\n$\\therefore C D \\perp$ 平面 $A B D$ ,\n\n则 $C D \\perp A B$ ,又 $A D \\perp A B , A D \\cap C D=D , A D , C D \\subset$ 平面 $A C D$ ,\n\n$\\therefore A B \\perp$ 平面 $A D C$ ,\n\n$\\therefore$ 平面 $A B C \\perp$ 平面 $A D C$."} {"id": "17167", "image": [], "answer": "B", "solution": "null", "level": "高一", "question": "(2020 春 - 海珠区校级月考) 已知棱长为 2 的正方体 $A B C D-A_{1} B_{1} C_{1} D_{1}$, 在正方体 $A B C D-A_{1} B_{1} C_{1} D_{1}$内随机取一点 $P$, 则点 $P$ 与点 $A$ 距离大于 1 的概率为 ( )", "options": "A. $1-\\frac{\\pi}{24}$\n B. $1-\\frac{\\pi}{48}$\n C. $\\frac{\\pi}{6}$\n D. $\\frac{\\pi}{12}$", "subject": "立体几何学", "analysis": "解:根据题意, 分析可得,\n\n在正方体 $A B C D-A_{1} B_{1} C_{1} D_{1}$ 中, 与点 $A$ 距离小于等于 1 的点在以 $A$ 为球心, 半径为 1 的八分之一个球内,\n\n其体积为 $V_{1}=\\frac{1}{8} \\times \\frac{4 \\pi}{3} \\times 1^{3}=\\frac{\\pi}{6}$;\n\n正方体的体积为 $2^{3}=8$,\n\n则点 $P$ 到点 $A$ 的距离小于等于 1 的概率为: $\\frac{\\frac{\\pi}{6}}{8}=\\frac{\\pi}{48}$,\n\n故点 $P$ 到点 $A$ 的距离大于 1 的概率为 $1-\\frac{\\pi}{48}$,\n\n故选: B."} {"id": "17170", "image": ["8813.jpg", "8814.jpg"], "answer": "D", "solution": "null", "level": "高一", "question": "(2020・临川区校级模拟) 某同学自制了一套数学实验模型, 该模型三视图如图所示. 模型内置一个与其各个面都相切的球, 该模型及其内球在同一方向有开口装置. 实验的时候, 随机往模型中投掷大小相等, 形状相同的玻璃球, 通过计算落在球内的玻璃球数量, 来估算圆周率的近似值. 已知某次实验中, 某同学一次投郑了 1000 个玻璃球, 请你估算落在球内的玻璃球数量 $($ 其中 $\\pi \\approx 3$ )\n\n", "options": "A. 286\nB. 289\nC. 298\nD. 302", "subject": "立体几何学", "analysis": "解: 由三视图得该模型是一个棱长为 $a=50 \\sqrt{2}$ 的四面体及其内切球,\n\n正四面体体积 $V_{1}=\\frac{1}{3} \\times \\frac{\\sqrt{3}}{4} \\times a^{2} \\times \\sqrt{a^{2}-\\left(\\frac{\\sqrt{3}}{3} a\\right)^{2}}=\\frac{\\sqrt{2}}{12} a^{3}$,\n\n过球心及正四面体顶点作截图,如图所示,\n\n由题意得 $\\triangle B O D \\backsim \\triangle B E C, \\therefore \\frac{r}{E C}=\\frac{B D}{B E}$, 即 $\\frac{r}{\\frac{a}{2}}=\\frac{\\frac{\\sqrt{3}}{6} a}{\\frac{\\sqrt{2}}{2} a}$,\n解得 $r=\\frac{\\sqrt{6}}{12} a$,\n\n$\\therefore$ 内切球体积为 $V_{2}=\\frac{4 \\pi}{3} \\times\\left(\\frac{\\sqrt{6}}{12} a\\right)^{3}$,\n\n设落在球内的玻璃球数量为 $x$,\n\n则 $\\frac{x}{1000}=\\frac{V_{2}}{V_{1}}$, 即 $\\frac{x}{1000}=\\frac{\\sqrt{3}}{18} \\pi$,\n\n解得 $x \\approx 302$.\n\n故选: $D$.\n\n"} {"id": "14356", "image": ["7301.jpg", "7302.jpg", "7303.jpg", "7304.jpg", "7305.jpg"], "answer": "B", "solution": "null", "level": "高一", "question": "高为 $H$ 、满缸水量为 $V$ 的鱼缸的轴截面如图所示, 现底部有一个小洞, 满缸水从洞中流出, 若鱼缸水深为 $h$ 时水的体积为 $v$, 则函数 $v=f(h)$ 的大致图像是 ( )\n\n", "options": "A.\n\n\nB.\n\n\nC.\n\n\nD.\n\n", "subject": "立体几何学", "analysis": "\n\n根据题意知, 函数的自变量为水深 $h$, 函数值为鱼缸中水的体积, 所以当 $h=0$ 时, 体积 $v=0$, 所以函数图像过原点, 故排除 $\\mathrm{A} 、 \\mathrm{C}$;\n\n再根据鱼缸的形状, 下边较细, 中间较粗, 上边较细, 所以随着水深的增加, 体积的变化速度是先慢后快再慢的, 故选 B."} {"id": "15543", "image": [], "answer": "D", "solution": "null", "level": "高一", "question": "下列叙述中正确的是 ( )", "options": "圆柱是将矩形旋转一周所得到的几何体\n\nB. 棱柱中两个相互平行的平面一定是棱柱的底面\n\nC. 过圆锥侧面上的一点有无数条母线\n\nD. 球面上四个不同的点有可能在同一平面内", "subject": "立体几何学", "analysis": "【分析】\n\n利用圆柱的定义判断 $A$; 利用棱柱的定义判断 $B$; 利用圆雉母线的定义判断 $C$; 利用球体的性质判断 $D$.\n\n## 【详解】\n\n在 $A$ 中, 将矩形以矩形的一条对角线为轴, 旋转所得的就不是圆柱, 故 $A$ 错;\n\n在 $B$ 中,长方体中两个相互平行的平面不一定是棱柱的底面,故 $B$ 错误;\n\n在 $C$ 中, 两点确定一条直线, 圆锥过圆锥侧面上的一点只有一条母线, 故 $C$ 错误;\n\n在 $D$ 中,球面上四个不同的点有可能在同一平面内,故 $D$ 正确,故选 D.\n\n## 【点睛】\n\n本题考查命题真假的判断,考查圆柱、棱柱、圆雉、球等基础知识,意在考查对基本概念掌握的熟练程度,考查了空间想象能力,属于中档题."} {"id": "15125", "image": [], "answer": "D", "solution": "null", "level": "高一", "question": "为了得到函数 $y=\\left(\\frac{1}{2}\\right)^{x-3}-1$ 的图象, 只需把函数 $y=\\left(\\frac{1}{2}\\right)^{x}$ 的图像上所有的点 ( )", "options": "A. 向左平移 3 个单位长度, 再向上平移 1 个单位长度\n\nB. 向右平移 3 个单位长度, 再向上平移 1 个单位长度\n\nC. 向左平移 3 个单位长度, 再向下平移 1 个单位长度\n\nD. 向右平移 3 个单位长度, 再向下平移 1 个单位长度", "subject": "变换几何", "analysis": "首先 $y=\\left(\\frac{1}{2}\\right)^{x}$ 向右平移 3 个单位得到 $y=\\left(\\frac{1}{2}\\right)^{x-3}$,\n\n再由 $y=\\left(\\frac{1}{2}\\right)^{x-3}$ 向下平移 1 个单位得到 $y=\\left(\\frac{1}{2}\\right)^{x-3}-1$.\n\n故答案为: D\n\n【分析】根据平移规律“左十右一, 上十下一”, 得到变换过程."} {"id": "17012", "image": ["8769.jpg"], "answer": "D", "solution": "null", "level": "高一", "question": "(2020 秋-公主岭市期末)如图, 一颗豆子随机扔到桌面上, 假设豆子不落在线上, 则它落在阴影区域的概率为()\n\n", "options": "A. $\\frac{1}{9}$\nB. $\\frac{1}{6}$\nC. $\\frac{2}{3}$\nD. $\\frac{1}{3}$", "subject": "统计数学", "analysis": "解:由题意知本题是一个几何概型,试验发生包含的事件对应的图形是一个大正方形,若设正方形的边长是 3 , 则正方形的面积是 9 ,满足条件的事件是三个小正方形面积是 3 , $\\therefore$ 落在图中阴影部分中的概率是 $\\frac{3}{9}=\\frac{1}{3}$故选: D."} {"id": "17169", "image": ["8812.jpg"], "answer": "A", "solution": "null", "level": "高一", "question": "(2020 秋・广西期中)某小学要求下午放学后的 17: 00 - 18: 00 接学生回家, 该学生家长从下班后到达学校(随机)的时间为 17: 30 - 18: 30 , 则该学生家长从下班后, 在学校规定时间内接到孩子的概率为", "options": "A. $\\frac{7}{8}$\n B. $\\frac{3}{4}$\n C. $\\frac{1}{2}$\n D. $\\frac{1}{4}$", "subject": "统计数学", "analysis": "解: 根据题意, 设学生出来的时间为 $x$, 家长到达学校的时间为 $y$,学生出来的时间为 17: $00-18: 00$, 看作 $5 \\leqslant x \\leqslant 6$,\n\n\n\n家长到学校的时间为 17: $30-18: 30,5.5 \\leqslant y \\leqslant 6.5$,\n\n要使得家长从下班后, 在学校规定时间内接到孩子, 则需要 $y \\geqslant_{x}$,\n\n则相当于, 即求 $y \\geqslant x$ 的概率,\n\n如图所示:\n\n约束条件对应的可行域面积为: 1 ,\n\n则可行域中 $y \\geqslant x$ 的面积为阴影部分面积: $1-\\frac{1}{2} \\times \\frac{1}{2} \\times \\frac{1}{2}=\\frac{7}{8}$,\n\n所以对应的概率为: $\\frac{\\frac{7}{8}}{1}=\\frac{7}{8}$, 即学生家长从下班后, 在学校规定时间内接到孩子的概率为: $\\frac{7}{8}$.故选: A."} {"id": "17171", "image": ["8815.jpg"], "answer": "A", "solution": "null", "level": "高一", "question": "(2020 春・河南月考) 在矩形 $A B C D$ 中, $A B>A D$, 在 $C D$ 上任取一点 $P$, 使 $\\triangle A B P$ 的最大边是 $A B$的概率为 $\\frac{3}{5}$, 则在折线 $A-D-C-B$ 上任取一点 $Q$, 使 $\\triangle A B Q$ 是直角三角形的概率为 ( )", "options": "$\\frac{6}{11}$\n B. $\\frac{5}{11}$\n C. $\\frac{5}{9}$\n D. $\\frac{4}{9}$", "subject": "统计数学", "analysis": "如图: $P$ 所在位置使 $\\triangle A B P$ 的最大边是 $A B$ 的概率为 $\\frac{3}{5}$,\n\n可令 $A B=5$, 设 $P$ 在 $M N$ 之间运动时, $\\triangle A B P$ 的最大边是 $A B$, 易知 $A B=A M=B N=3, D N=M C$ $=1$, 由勾股定理得 $A D=B C=3$.\n\n又 $Q$ 在线段 $A D$ 或 $B C$ 上时, $\\triangle A B Q$ 是直角三角形, 故概率 $P=\\frac{6}{11}$.\n\n故选: A.\n\n"} {"id": "6353", "image": [], "answer": "A", "solution": "null", "level": "一年级", "question": "哪种图形多?()", "options": "A.\n\nB.\n0000", "subject": "计数", "analysis": "A"} {"id": "6373", "image": ["441.jpg", "442.jpg"], "answer": "A", "solution": "null", "level": "一年级", "question": "乐乐该坐哪个方向的车?()\n\n乐乐:\n\n\n4. _先数一数每一种水果各有多少个, 其中少。\n\n", "options": "A. (3) 1 1)\n\nB. (1)(2)", "subject": "计数", "analysis": "A"} {"id": "23187", "image": ["11430.jpg"], "answer": "A", "solution": "null", "level": "一年级", "question": "如图, 盒子里有()支铅笔。\n\n\n", "options": "A、 7\nB、 8\nC、9\n\n", "subject": "计数", "analysis": "解$A ; 12-5=7$ 支"} {"id": "23325", "image": ["11452.jpg", "11453.jpg", "11454.jpg"], "answer": "B", "solution": "null", "level": "一年级", "question": "一共有( )个橙子。\n\n\n\n000000000\n\n\n", "options": "A、 15\nB、 17\nC、19\n\n\n10 ._\n\n\n\nA、 5\n\nB、 6\n\nC、 7\n\n", "subject": "计数", "analysis": "解B; $8+9=17$, 篮子里有 8 个, 外面有 9 个橙子, 共有 $8+9=17$ 个橙\n子。"} {"id": "23326", "image": ["11455.jpg"], "answer": "A", "solution": "null", "level": "一年级", "question": "下列图片按数量分, 可以分为 ( ) 类。\n\n\n", "options": "A、 2\nB、 3\nC、 4\n\n", "subject": "计数", "analysis": "A; 有的图片是一个杯子, 有的是两个杯子, 所以按数量分可以分为 2 类。"} {"id": "23327", "image": ["11456.jpg"], "answer": "B", "solution": "null", "level": "一年级", "question": "按科目分,可以分为()类。\n\n\n", "options": "A、 2\nB、 3\nC、 4\n\n", "subject": "计数", "analysis": "B; 按照科目分, 可以分为语文、英语和数学三类。"} {"id": "23328", "image": ["11457.jpg"], "answer": "C", "solution": "null", "level": "一年级", "question": "以下这些水果不可以按()分类。\n\n\n", "options": "A、颜色\nB、数量\nC、价格\n\n", "subject": "计数", "analysis": "C; 因为没有给出水果的价格, 所以不能按照价格来分类。"} {"id": "6363", "image": ["428.jpg"], "answer": "C", "solution": "null", "level": "一年级", "question": "看图选数。()\n\n", "options": "A. 1\nB. 2\nC. 3\nD. 4", "subject": "算术", "analysis": "C"} {"id": "23163", "image": ["11429.jpg"], "answer": "B", "solution": "null", "level": "一年级", "question": "如图, 盒子里有()个苹果。\n\n\n", "options": "A、 14\nB、 15\nC、 16\n\n", "subject": "算术", "analysis": "解B; 19-4=15 个"} {"id": "23164", "image": [], "answer": "A", "solution": "null", "level": "一年级", "question": "小白兔和小灰兔一共采了 16 朵蘑菇, 小白兔采了 9 朵, 小灰兔采了()朵。\n ", "options": "A、 7\n B、 8\n C、9\n", "subject": "算术", "analysis": "解A; 16-9=7 朵"} {"id": "23165", "image": [], "answer": "B", "solution": "null", "level": "一年级", "question": "奶奶家有鸡和鸭共 12 只, 其中鸡有 4 只, 鸭有()只。\n ", "options": "A、 7\n B、 8\n C、9\n", "subject": "算术", "analysis": "解B; 12-4=8 只"} {"id": "23166", "image": [], "answer": "B", "solution": "null", "level": "一年级", "question": "一共有 14 只鸽子, 其中 8 只飞出去送信了, 鸽笼里还剩下()只鸽子。\n ", "options": "A、 5\n B、 6\n C、 7\n", "subject": "算术", "analysis": "解B; 14-8=6 只"} {"id": "23167", "image": [], "answer": "B", "solution": "null", "level": "一年级", "question": "乐乐有 10 个苹果和 4 个梨, 吃了 3 个苹果, 还剩下( ) 个苹果。\n ", "options": "A、 6\n B、 7\n C、8\n", "subject": "算术", "analysis": "解B; 只看苹果的数量, $10-3=7$ 个"} {"id": "23168", "image": [], "answer": "A", "solution": "null", "level": "一年级", "question": "一二班一共有 17 人参加植树, 要植 8 棵树, 其中女生有 8 人,男生有( )人。\n ", "options": "A、 9\n B、 10\n C、 11\n", "subject": "算术", "analysis": "解A; 17-8=9 人"} {"id": "23169", "image": [], "answer": "C", "solution": "null", "level": "一年级", "question": "YY有 18 朵花, 送给歆歆 7 朵, 还剩下()朵。\n ", "options": "A、 9\n B、 10\n C、 11\n", "subject": "算术", "analysis": "解C; 18-7=11 朵"} {"id": "23170", "image": [], "answer": "A", "solution": "null", "level": "一年级", "question": "小羽有 10 本故事书, 乐乐有 15 本故事书, 乐乐的故事书比小羽多( )本。\n ", "options": "A、 5\n B、6\n C、 7\n", "subject": "算术", "analysis": "解$A ; 15-10=5$ 本"} {"id": "23171", "image": [], "answer": "C", "solution": "null", "level": "一年级", "question": "小华今年 17 岁, 小红今年 14 岁, 小华比小红大()岁。\n ", "options": "A、 1\n B、2\n C、3\n", "subject": "算术", "analysis": "解C; 17-14=3 岁"} {"id": "23172", "image": [], "answer": "A", "solution": "null", "level": "一年级", "question": "飞机场上午和下午共起飞了 13 架飞机,其中上午起飞了 8 架,\n 下午起飞()架。\n ", "options": "A、 5\n B、 6\n C、 7\n", "subject": "算术", "analysis": "解A; 13-8=5 架"} {"id": "23173", "image": [], "answer": "B", "solution": "null", "level": "一年级", "question": "航模小组一共有 7 人, 象棋小组的人数和航模小组的人数一样多,两个兴趣小组一共有()人。\n ", "options": "A、 12\n B、 14\n C、16\n", "subject": "算术", "analysis": "解$B ; 7+7=14$ 人"} {"id": "23174", "image": [], "answer": "A", "solution": "null", "level": "一年级", "question": "小白兔一共拔了 11 根萝卜, 已经运回家了 5 根, 还剩下()根。\n ", "options": "A、 6\n B、 7\n C、8\n", "subject": "算术", "analysis": "解A; 11-5=6 根"} {"id": "23175", "image": [], "answer": "A", "solution": "null", "level": "一年级", "question": "小羽和小红一共折了 17 颗星星, 其中小红折了 8 颗, 小羽折了 ( ) 颗。\n ", "options": "A、 9\n B、 10\n C、 11\n", "subject": "算术", "analysis": "解A$ ; 17-8=9$ 颗"} {"id": "23176", "image": [], "answer": "A", "solution": "null", "level": "一年级", "question": "乐乐有 16 本故事书, 小羽借走了 12 本, 还剩下()本。\n ", "options": "A、 4\n B、 6\n C、8\n", "subject": "算术", "analysis": "解A; $16-12=4$ 本"} {"id": "23177", "image": [], "answer": "C", "solution": "null", "level": "一年级", "question": "下面三个算式中, ( )的结果最小。\n ", "options": "A、 $13-6$\n B、 $15-5$\n C、11-7\n", "subject": "算术", "analysis": "解$C ; 13-6=7,15-5,=10,11-7=4$"} {"id": "23178", "image": [], "answer": "B", "solution": "null", "level": "一年级", "question": "小羽和乐乐去果园摘苹果, 小羽摘了 7 个, 乐乐摘了 8 个, 两人一共摘了( )个苹果。\n ", "options": "A、 13\n B、 15\n C、 16\n", "subject": "算术", "analysis": "解$B ; 7+8=15$ 个"} {"id": "23179", "image": [], "answer": "C", "solution": "null", "level": "一年级", "question": "一盒巧克力有 16 颗, 小羽分给小伙伴 8 颗, 还剩下()颗。\n ", "options": "A、 4\n B、6\n C、8\n", "subject": "算术", "analysis": "解$C ; 16-8=8$ 颗"} {"id": "23180", "image": [], "answer": "B", "solution": "null", "level": "一年级", "question": "一本笔记本 6 元钱, 小羽有 13 元钱, 买完笔记本还剩下 ( ) 元。\n ", "options": "A、 6\n B、 7\n C、8\n", "subject": "算术", "analysis": "解B; 13-6=7 元"} {"id": "23181", "image": [], "answer": "A", "solution": "null", "level": "一年级", "question": "小羽原来有 8 元钱,今天做家务又挣了 3 元钱,他拿着自己挣来\n 的钱买了一个冰淇淋 7 元,小羽还剩下()元。\n ", "options": "A、 4\n B、 6\n C、8\n", "subject": "算术", "analysis": "解$A ; 8+3-7=4$ 元"} {"id": "23182", "image": [], "answer": "A", "solution": "null", "level": "一年级", "question": "鱼缸里一共有 18 条金鱼, 其中黑色的有 9 条, 剩下的是金色的,金色的有()条。\n ", "options": "A、 9\n B、 10\n C、 11\n", "subject": "算术", "analysis": "解A; 18-9=9 条"} {"id": "23183", "image": [], "answer": "C", "solution": "null", "level": "一年级", "question": "一共有 14 根木头, 大象运走了 6 根, 还剩下()根。\n ", "options": "A、 4\n B、6\n C、8\n", "subject": "算术", "analysis": "解C; 14-6=8 根"} {"id": "23184", "image": [], "answer": "C", "solution": "null", "level": "一年级", "question": "$13-8$ ( ) 16-4\n ", "options": "A、 $>$\n B、 $=$\n C、 $<$\n\n", "subject": "算术", "analysis": "解C; $13-8=5,16-4=12,5<12$"} {"id": "23185", "image": [], "answer": "C", "solution": "null", "level": "一年级", "question": "5+6 ( ) 19-7\n ", "options": "A、 $>$\n B、 $=$\n C、 $<$\n\n", "subject": "算术", "analysis": "解C; $5+6=11,19-7=12,11<12$"} {"id": "23186", "image": [], "answer": "A", "solution": "null", "level": "一年级", "question": "$13+0$ ( ) $12-5$\n ", "options": "A、 $>$\n B、 $=$\n C、 $<$\n\n", "subject": "算术", "analysis": "解A; $13+0=13,12-5=7,13>7$"} {"id": "23188", "image": [], "answer": "A", "solution": "null", "level": "一年级", "question": "农场里有 11 只小黄鸡, 走了 5 只, 剩下()只。\n ", "options": "A、 6\n B、 7\n C、8\n\n", "subject": "算术", "analysis": "解A; $11-5=6$ 只"} {"id": "23189", "image": [], "answer": "C", "solution": "null", "level": "一年级", "question": "小明家里有 15 个苹果, 吃了 6 个, 还有()个。\n ", "options": "A、 7\n B、 8\n C、9\n\n", "subject": "算术", "analysis": "解C; $15-6=9$ 个"} {"id": "23190", "image": [], "answer": "C", "solution": "null", "level": "一年级", "question": "原本有 15 朵小花, 现在剩下 7 朵小花, 被人摘取了()朵。\n", "options": "A、 6\nB、 7\nC、8\n\n", "subject": "算术", "analysis": "解C; 15-7=8 朵"} {"id": "23191", "image": [], "answer": "B", "solution": "null", "level": "一年级", "question": "春天到了, 老师带全班 14 个学生出去郊游, 一辆小巴车坐 9 人,剩下的坐小轿车,那么小轿车坐()人。\n ", "options": "A、 6\n B、 5\n C、 4\n\n", "subject": "算术", "analysis": "解$\\mathrm{B} ; 14-9=5$ 人"} {"id": "23192", "image": [], "answer": "B", "solution": "null", "level": "一年级", "question": "鱼缸里本来有 18 条小金鱼, 凡凡拿了 8 条送给了好朋友。凡凡的妈妈又买了 3 条小金鱼。现在鱼缸里有( )条小金鱼。\n", "options": "A、 10\nB、 13\nC、 15\n", "subject": "算术", "analysis": "解B; 18-8+3=13 条"} {"id": "23193", "image": ["11431.jpg"], "answer": "A", "solution": "null", "level": "一年级", "question": "图书角原本有 12 本故事书, 现在图书角有()本书。\n\n\n", "options": "A、 7\nB、 9\nC、11\n\n", "subject": "算术", "analysis": "解$A ; 12-8+3=7$ 本"} {"id": "23194", "image": ["11432.jpg"], "answer": "B", "solution": "null", "level": "一年级", "question": "农场里有 14 只兔子, 室内有()只。\n\n\n", "options": "A、 5\nB、 6\nC、 7\n\n", "subject": "算术", "analysis": "解B; $14-8=6$ 只"} {"id": "23195", "image": ["11433.jpg"], "answer": "A", "solution": "null", "level": "一年级", "question": "体育馆有 16 个篮球, 足球比篮球少 8 个, 体育馆有足球()个。\n\n\n", "options": "A、 8\nB、 9\nC、10\n\n", "subject": "算术", "analysis": "解$A ; 16-8=8$ 个"} {"id": "23196", "image": [], "answer": "B", "solution": "null", "level": "一年级", "question": "小红有 15 元钱, 买了一支铅笔 3 元, 买了一本笔记本 5 元, 还剩下()元。\n", "options": "A、 6\nB、 7\nC、8\n", "subject": "算术", "analysis": "解B; $15-3-5=7$ 元"} {"id": "23197", "image": [], "answer": "B", "solution": "null", "level": "一年级", "question": "11-8O4, 在○应该填上( )。\n", "options": "A、 $>$\nB、 $<$\nC、 $=$\n", "subject": "算术", "analysis": "解$\\mathrm{B} ; 11-8=3,3<4$"} {"id": "23198", "image": [], "answer": "C", "solution": "null", "level": "一年级", "question": "“19- $\\square=11$ ”, 在 $\\square$ 里应该填上()。\n", "options": "A、 6\nB、 7\nC、8\n", "subject": "算术", "analysis": "解$C ; 19-11=8$"} {"id": "23199", "image": [], "answer": "B", "solution": "null", "level": "一年级", "question": "“ $\\square-8=5$ ” , 在 $\\square$ 里应该填上( )。\n", "options": "A、 10\nB、 13\nC、 15\n", "subject": "算术", "analysis": "解$\\mathrm{B} ; 8+5=13$"} {"id": "23200", "image": [], "answer": "B", "solution": "null", "level": "一年级", "question": "店里一共有 12 个气球, 小朋友买了 8 个, 还剩下()个气球。\n\n", "options": "A、 3\nB、 4\nC、 5\n\n", "subject": "算术", "analysis": "解B; $12-8=4$ 个"} {"id": "23201", "image": [], "answer": "A", "solution": "null", "level": "一年级", "question": "一盒铅笔一共有 15 支铅笔, 被老师拿走了 8 支, 一盒铅笔里还有 ( )支铅笔?\n ", "options": "A、 7\n B、 8\n C、9\n\n", "subject": "算术", "analysis": "解$A ; 15-8=7$ 支"} {"id": "23202", "image": [], "answer": "C", "solution": "null", "level": "一年级", "question": "大猴子有 13 个桃子, 小猴子买了 9 个, 还剩()个桃子。\n ", "options": "A、 6\n B、 5\n C、 4\n\n", "subject": "算术", "analysis": "解C; $13-9=4$ 个"} {"id": "23203", "image": [], "answer": "A", "solution": "null", "level": "一年级", "question": "停车场里有小轿车和大客车共 11 辆, 其中大客车有 5 辆, 那么有 ( ) 辆小轿车。\n ", "options": "A、 6\n B、 7\n C、8\n\n", "subject": "算术", "analysis": "解$A ; 11-5=6$ 辆"} {"id": "23204", "image": [], "answer": "B", "solution": "null", "level": "一年级", "question": "一共有 15 个气球, 卖了 9 个, 还剩下()个气球。\n ", "options": "A、 5\n B、 6\n C、 7\n\n", "subject": "算术", "analysis": "解$\\mathrm{B} ; 15-9=6$ 个"} {"id": "23205", "image": [], "answer": "C", "solution": "null", "level": "一年级", "question": "一共有 12 只小老鼠, 3 只在玩跳绳, 剩下()只老鼠在转圈圈。\n ", "options": "A、 7\n B、 8\n C、 9\n\n", "subject": "算术", "analysis": "解$C ; 12-3=9$ 只"} {"id": "23206", "image": [], "answer": "A", "solution": "null", "level": "一年级", "question": "树上有 16 只小鸟, 被猎人打死了 9 只, 还剩()只。\n ", "options": "A、 7\n B、 8\n C、9\n\n", "subject": "算术", "analysis": "解$A ; 16-9=7$ 只"} {"id": "23207", "image": [], "answer": "A", "solution": "null", "level": "一年级", "question": "商店里有 14 个小狮子花灯, 卖去了 8 个, 还有()个。\n ", "options": "A、 6\n B、 7\n C、8\n\n", "subject": "算术", "analysis": "解$\\mathrm{A} ; 14-8=6$ 个"} {"id": "23208", "image": ["11434.jpg"], "answer": "B", "solution": "null", "level": "一年级", "question": "农场里共有羊和奶牛 17 只, 其中羊有 6 只, 奶牛有()只。\n\n\n", "options": "A、 10\nB、 11\nC、 12\n\n", "subject": "算术", "analysis": "解B; $17-6=11$ 只"} {"id": "23209", "image": [], "answer": "B", "solution": "null", "level": "一年级", "question": "小明有 15 支铅笔,送给小红 7 支,还剩下()支。\n", "options": "A、 7\nB、 8\nC、9\n", "subject": "算术", "analysis": "解$B ; 15-7=8$ 支"} {"id": "23210", "image": ["11435.jpg"], "answer": "A", "solution": "null", "level": "一年级", "question": "小红一共要花( )元。\n\n\n", "options": "A、 13\nB、 12\nC、 11\n\n", "subject": "算术", "analysis": "解$A ; 2+2+2+7=13$ 元"} {"id": "23211", "image": [], "answer": "A", "solution": "null", "level": "一年级", "question": "一辆公交车原本有 16 人, 下去了 9 个人, 上来了 5 个人。现在\n公交车上有()人。\n", "options": "A、 12\nB、 13\nC、 14\n\n\n", "subject": "算术", "analysis": "解A; 16-9+5=12 人"} {"id": "23212", "image": [], "answer": "C", "solution": "null", "level": "一年级", "question": "哥哥一共钓了 13 条鱼, 送给爷爷 7 条鱼, 还剩下 () 条。\n", "options": "A、 4\nB、 5\nC、 6\n", "subject": "算术", "analysis": "解C; 13-7=6 条"} {"id": "23213", "image": [], "answer": "B", "solution": "null", "level": "一年级", "question": "一共有 15 只兔子, 左边有 7 只兔子, 右边有( )只兔子。\n", "options": "A、 7\nB、 8\nC、9\n", "subject": "算术", "analysis": "解$B ; 15-7=8$ 只"} {"id": "23214", "image": [], "answer": "A", "solution": "null", "level": "一年级", "question": "原本有 12 只猴子, 跑走了 8 只, 又跑来了 13 只, 最后一共有 ( )只猴子。\n", "options": "A、 17\nB、 18\nC、19\n", "subject": "算术", "analysis": "解$A ; 12-8+13=17$ 只"} {"id": "23215", "image": [], "answer": "A", "solution": "null", "level": "一年级", "question": "原本有 15 把喇叭, 拿走了 8 把喇叭, 还剩( )把喇叭。\n", "options": "A、 7\nB、 8\nC、9\n", "subject": "算术", "analysis": "解A; 15-8=7 把"} {"id": "23216", "image": [], "answer": "C", "solution": "null", "level": "一年级", "question": "一盒有 12 支钢笔, 拿出了 4 支, 还有()支钢笔在盒子里。\n", "options": "A、 6\nB、 7\nC、8\n", "subject": "算术", "analysis": "解C; 12-4=8 支"} {"id": "6374", "image": ["443.jpg"], "answer": "D", "solution": "null", "level": "一年级", "question": "小象跑在最()面。\n", "options": "A. 上\nB. 下\nC. 前\nD. 后", "subject": "逻辑题", "analysis": "D"} {"id": "6394", "image": ["462.jpg"], "answer": "C", "solution": "null", "level": "一年级", "question": "小狗跑在最()面。\n", "options": "A. 上\nB. 下\nC. 前\nD. 后", "subject": "逻辑题", "analysis": "C"} {"id": "6395", "image": [], "answer": "D", "solution": "null", "level": "一年级", "question": "与其他三行不同的那一行是 ( )", "options": "A. 123\nB. 234\nC. 3\nD. 432", "subject": "逻辑题", "analysis": "D"} {"id": "6375", "image": [], "answer": "B", "solution": "null", "level": "一年级", "question": "硬币的上下两个面是()。", "options": "A. 长方形\nB. 圆形\nC.正方形", "subject": "立体几何学", "analysis": "B"} {"id": "6389", "image": ["451.jpg", "452.jpg"], "answer": "A", "solution": "null", "level": "一年级", "question": "下面图形是长方体的是()。", "options": "A.\n\n\nB.\n\n\nC. $\\bigoplus$", "subject": "立体几何学", "analysis": "A"} {"id": "6392", "image": ["457.jpg", "458.jpg", "459.jpg"], "answer": "A", "solution": "null", "level": "一年级", "question": "下图中哪个是长方体?", "options": "A.\n\n\nB.\n\n\nC.\n\n", "subject": "立体几何学", "analysis": "A"} {"id": "23336", "image": [], "answer": "A", "solution": "null", "level": "一年级", "question": "一元硬币表面的形状是()。\n", "options": "A、圆形\nB、三角形\nC、正方形\n", "subject": "立体几何学", "analysis": "解A; 硬币的表面是圆形的。"} {"id": "23345", "image": ["11476.jpg"], "answer": "B", "solution": "null", "level": "一年级", "question": "长方体的 “上面” 是()图形。\n\n\n", "options": "A、三角形\nB、长方形\nC、平行四边形\n\n", "subject": "立体几何学", "analysis": "解B; 长方体的最上面的那个面是长方形。"} {"id": "23351", "image": ["11480.jpg"], "answer": "A", "solution": "null", "level": "一年级", "question": "下图的上下两个面都是()。\n\n\n", "options": "A、圆形\nB、三角形\nC、正方形\n\n", "subject": "立体几何学", "analysis": "解A; 圆柱的上下两个底面都是圆形。"} {"id": "23357", "image": [], "answer": "A", "solution": "null", "level": "一年级", "question": "圆柱的上下两个面是( )。\n", "options": "A、圆形\nB、三角形\nC、正方形\n", "subject": "立体几何学", "analysis": "解A; 圆柱的上下两个底面都是圆形的。"} {"id": "23358", "image": [], "answer": "C", "solution": "null", "level": "一年级", "question": "长方体有()个面。\n", "options": "A、 4\nB、 5\nC、 6\n", "subject": "立体几何学", "analysis": "解C;每个长方体都有 6 个面。"} {"id": "6379", "image": [], "answer": "B", "solution": "null", "level": "一年级", "question": "的数量多? 还是 的数量多? ( )", "options": "A.\nB.\n$\\% \\% \\%$", "subject": "组合数学", "analysis": "B"} {"id": "23329", "image": ["11458.jpg"], "answer": "A", "solution": "null", "level": "一年级", "question": "以下交通工具按行驶的地方分类, 可以分为()类。\n\n\n", "options": "A、 2\nB、 3\nC、 4\n\n", "subject": "组合数学", "analysis": "A; 按行驶的地方, 可以分为陆地和天空 2 类。"} {"id": "23330", "image": ["11459.jpg"], "answer": "A", "solution": "null", "level": "一年级", "question": "下面图形按颜色分, 可以分为 ( ) 类。\n\n", "options": "A、 2\nB、 3\nC、 4\n", "subject": "组合数学", "analysis": "A; 按颜色可以分为蓝色和白色 2 类。"} {"id": "6393", "image": ["460.jpg", "461.jpg"], "answer": "A", "solution": "null", "level": "一年级", "question": "\n 的 ( ) 面。\n ", "options": "A. 左\n B. 右\n C. 前\n D.后", "subject": "画法几何学", "analysis": "A"} {"id": "23353", "image": ["11481.jpg", "11482.jpg"], "answer": "B", "solution": "null", "level": "一年级", "question": "将下图拼成一个\n\n\n\n“4” 的对面是()。\n\n\n", "options": "A、 1\nB、2\nC、 3\n\n", "subject": "画法几何学", "analysis": "解B; “ 4 ” 和 “ 2 ” 是对应的。"} {"id": "23331", "image": ["11460.jpg"], "answer": "B", "solution": "null", "level": "一年级", "question": "下面图形按形状分, 可以分为()类。\n\n", "options": "A、 2\nB、 3\nC、 4\n", "subject": "组合几何学", "analysis": "B; 按形状可以分为三角形、圆形和正方形三类。"} {"id": "23339", "image": ["11469.jpg"], "answer": "B", "solution": "null", "level": "一年级", "question": "图中一共有()个长方形。\n\n\n", "options": "A、 2\nB、 3\nC、 4\n\n", "subject": "组合几何学", "analysis": "解B; 两个灰色的长方形 “墙壁” 和一个白色的 “门”。"} {"id": "23341", "image": ["11471.jpg"], "answer": "B", "solution": "null", "level": "一年级", "question": "图中的屋顶是由()图形组成的。\n\n\n\n", "options": "A、平行四边形和正方形\n\nB、平行四边形和三角形\n\nC、正方形和三角形\n\n", "subject": "组合几何学", "analysis": "解B; 屋顶是由一个平行四边形和一个三角形组成的。"} {"id": "23342", "image": ["11472.jpg"], "answer": "A", "solution": "null", "level": "一年级", "question": "下图是由四个相同的()组成的一个正方形。\n\n\n", "options": "A、三角形\nB、长方形\nC、正方形\n\n", "subject": "组合几何学", "analysis": "解A; 正方形是由 4 个小三角形拼成的。"} {"id": "23344", "image": ["11475.jpg"], "answer": "B", "solution": "null", "level": "一年级", "question": "从右往左数, 第二个图形是( )。\n\n", "options": "A、三角形\nB、长方形\nC、平行四边形\n", "subject": "组合几何学", "analysis": "解B; 从右往左数, 第二个是长方形。"} {"id": "23346", "image": [], "answer": "B", "solution": "null", "level": "一年级", "question": "红领巾的形状是( )。\n", "options": "A、圆形\nB、三角形\nC、正方形\n", "subject": "组合几何学", "analysis": "解B; 红领巾是三角形的。"} {"id": "23348", "image": ["11477.jpg"], "answer": "A", "solution": "null", "level": "一年级", "question": "七巧板是由()种图形组成的。\n\n\n", "options": "A、 3\nB、 4\nC、 5\n\n", "subject": "组合几何学", "analysis": "解A; 七巧板是由三角形, 正方形和平行四边形三种图形拼成的。"} {"id": "23343", "image": ["11473.jpg", "11474.jpg"], "answer": "B", "solution": "null", "level": "一年级", "question": "$\\square$\n\n\n的()边。\n\n", "options": "A、左\nB、右\nC、上\n\n", "subject": "度量几何学", "analysis": "解B; 右图可以看出, 长方形在圆形的右边。"} {"id": "23347", "image": [], "answer": "B", "solution": "null", "level": "一年级", "question": "教室黑板的表面是()。\n", "options": "A、圆形\nB、长方形\nC、三角形\n", "subject": "度量几何学", "analysis": "解B; 教师的黑板表面是长方形的、"} {"id": "23359", "image": ["11483.jpg", "11484.jpg", "11485.jpg"], "answer": "C", "solution": "null", "level": "一年级", "question": "下图()是平行四边形。\n", "options": "A、\n\n\nB、\n\n\nC、\n\n\n\n", "subject": "解析几何", "analysis": "解$\\mathrm{C}$; $\\mathrm{A}$ 是圆形, $\\mathrm{B}$ 是三角形, $\\mathrm{C}$ 是平行四边形。"} {"id": "23366", "image": ["11489.jpg", "11490.jpg"], "answer": "B", "solution": "null", "level": "一年级", "question": "\n\n\n\n如图, 把它沿虚线折叠后的形状是()。\n", "options": "A、\n\n\nB、\n\n\nC. $\\square$\n\n", "subject": "变换几何", "analysis": "解B; 把这个正方形沿虚线对折两次,刚好是 4 个小正方形。"} {"id": "9038", "image": [], "answer": "C", "solution": "null", "level": "三年级", "question": "三名学生 50 米跑的成绩如下表, ()是第一名。\n\n| 姓名 | 李雷 | 张军 | 赵强 |\n| :---: | :---: | :---: | :---: |\n| 成绩/秒 | 8.5 | 8.3 | 8.1 |", "options": "A. 李雷\nB. 张军\nC. 赵强", "subject": "计数", "analysis": "解$\\mathrm{C}$\n\n50 米跑成绩, 用的时间越多, 成绩越差, 用的时间越少, 成绩越好, 再根据小数大小的比较方法比较他们的成绩,最小的就是第一名。\n\n因为 $8.1<8.3<8.5$,\n\n所以赵强是第一名,张军是第二名,李雷是第三名。\n\n故答案为: C\n\n此题考查小数大小的比较方法的灵活运用。要注意: 谁跑完 50 米用的时间最少, 谁就是第\n\n一名。"} {"id": "9095", "image": ["1493.jpg"], "answer": "C", "solution": "null", "level": "三年级", "question": "下面是某商场去年 5-7 月家电销售情况。下面说法错误的是()。\n\n", "options": "A. 空调的销量一个月比一个月多\n\nB. 6 月三种家电中冰箱销售的最多\n\nC. 每个月都是冰箱销售的最多", "subject": "计数", "analysis": "解$\\mathrm{C}$\n\n【分析】根据复式统计表中的数据, 对每个选项中的说法进行判断, 然后再进行选择即可;对 5 月、 6 月、 7 月空调的销量进行大小排序, 然后再判断;\n\n对 6 月三种家电销售的销量进行大小排序,然后再比较即可;\n\n分别对 5 月、 6 月、 7 月三种家电销售的销量进行大小排序, 然后再判断。\n\n【详解】A. $39<43<62$, 即空调的销量一个月比一个月多, 因此原说法正确;\n\nB. $46>43>38$, 即 6 月三种家电中冰箱销售的最多, 因此原说法正确;\n\nC. 5 月: $50>43>39$, 即 5 月是冰箱销售的最多; 6 月: $46>43>38$, 即 6 月是冰箱销售的最多; 7 月: $62>47>34$, 即 7 月是空调销售的最多,因此原说法错误。\n\n故答案为: C\n\n【点睛】熟练掌握复式统计表的特点是解答此题的关键。"} {"id": "9106", "image": [], "answer": "B", "solution": "null", "level": "三年级", "question": "下表是某校三 (2) 班全班同学喜欢吃蔬菜的情况, 统计如下表: (每人限选一种)\n\n| 人数 种类 | 茄子 | 西红柿 | 青菜 | 黄瓜 |\n| :---: | :---: | :---: | :---: | :---: |\n| 性别 | | 9 | 3 | |\n| 男生 | 8 | 6 | 9 | 8 |\n| 女生 | 2 | 7 | 8 | |\n\n喜欢吃黄瓜的人数比喜欢吃西红柿的少( )。", "options": "A. 6 人\nB. 2 人\nC. 8 人", "subject": "计数", "analysis": "解$\\mathrm{B}$\n\n【分析】先分别求出喜欢吃黄瓜、喜欢吃西红柿的总人数, 再根据求一个数比另一个数少几, 用减法解答。\n\n【详解】 $(6+7)-(3+8)$\n\n$=13-11$\n\n$=2(人)$\n\n喜欢吃黄瓜的人数比喜欢吃西红柿的少 2 人。\n\n故答案为: $\\mathrm{B}$\n\n【点睛】本题考查统计表的数据分析能力, 读懂统计表, 使用统计表提供的数据解决实际问题。"} {"id": "9110", "image": [], "answer": "C", "solution": "null", "level": "三年级", "question": "学校的图书室要购买一些新书, 想知道大多数学生最喜欢看什么类型的书, 下列收集数据的方法中合理的是 $(\\quad)$ 。", "options": "A. 找一个班统计每名学生一年读多少本书\n\nB. 找一个班统计每名学生最喜欢看什么类型的书\n\nC. 找每个年级的一部分学生统计他们最喜欢看什么类型的书", "subject": "计数", "analysis": "解$\\mathrm{C}$\n\n【分析】因为目的是知道大多数学生最喜欢看什么类型的书, 所以只需统计出每个年级一部分学生喜欢的类型。\n\n【详解】A. “找一个班”不具代表性, 还有其他年级的学生也需要统计一下, 也没必要统计“每一位学生”;\n\nB. “找一个班”不具代表性,还有其他年级的学生也需要统计一下,也没必要统计“每一位学生”;\n\nC. 找每个年级的一部分学生统计他们最喜欢看什么类型的书, 合理。\n\n故答案为: C\n\n【点睛】本题考查的是统计方法的选择, 要根据统计目的来确定。"} {"id": "9111", "image": [], "answer": "B", "solution": "null", "level": "三年级", "question": "这是小红家和小浩家近 3 个月用电量统计表,下面说法正确的是()。\n\n| 用电量 (度) 月份 | | | |\n| ---: | :---: | :---: | :---: |\n| 姓名 | 3月 | 4月 | 5月 |\n| 小红 | 75 | 73 | 70 |\n| 小浩 | 68 | 71 | 69 |", "options": "A. 小浩家用电量三月份最多\nB. 小红家用电量在减少\nC. 小浩家用电量在增加", "subject": "计数", "analysis": "$\\mathrm{B}$\n\n【分析】根据统计表中的数据, 对每个选项中的说法进行分析并判断;\n\n分别对小浩、小红家近 3 个月的用电量进行大小排序即可判断。\n\n【详解】A. $71>69>68$, 即小浩家用电量 4 月份最多, 因此原说法错误;\n\nB. $75>73>70$, 即小红家用电量在减少, 因此原说法正确;\n\nC. $71>69>68$, 即小浩家用电量 4 月份最多, 3 月份最少, 因此原说法错误。故答案为: B\n\n【点睛】熟练掌握复式统计表的特点是解答此题的关键。"} {"id": "9112", "image": [], "answer": "B", "solution": "null", "level": "三年级", "question": "下表是某饭店 1 3 月甲、乙两种品牌啤酒的销售情况 (单位: 箱) 。 3 月甲品牌啤酒比乙品牌啤酒多卖 $(\\quad)$ 箱。\n\n| 月份 | 1 月 | 2月 | 3月 |\n| :---: | :---: | :---: | :---: |\n| 甲品牌 | 150 | 100 | 190 |\n| 乙品牌 | 90 | 120 | 150 |", "options": "A. 60\nB. 40\nC. 30", "subject": "计数", "analysis": "解B\n\n【分析】用 3 月甲品牌啤酒的销售量减去 3 月乙品牌啤酒的销售量解答。\n\n【详解】 $190-150=40$ (箱)\n\n则 3 月甲品牌啤酒比乙品牌啤酒多卖 40 箱。\n\n故答案为: B\n\n【点睛】此题考查的目的是理解掌握统计表的特点及作用, 并且能够根据统计表提供的信息, 解决有关的实际问题。"} {"id": "9117", "image": [], "answer": "D", "solution": "null", "level": "三年级", "question": "据统计, 目前我国 $10-11$ 岁女生标准身高的范围是: $138-155 \\mathrm{~cm}$; 标准体重的范围是: $29-50 \\mathrm{~kg}$ 。下表是五年级 4 位女生的身高、体重情况。不符合标准的是()。\n\n| | 王媛 | 李想 | 刘菲 | 张奕 |\n| :--- | :---: | :---: | :---: | :---: |\n| 身高 $(\\mathrm{cm})$ | 145 | 151 | 143 | 153 |\n| 体重 $(\\mathrm{kg})$ | 50 | 46 | 43 | 55 |", "options": "A. 王媛\nB. 李想\nC. 刘菲\nD. 张奕", "subject": "计数", "analysis": "解$\\mathrm{D}$\n\n【分析】根据题意, 符合标准身高的范围是: $138-155 \\mathrm{~cm}$, 即比 $138 \\mathrm{~cm}$ 大或等于 $138 \\mathrm{~cm}$, 比 $155 \\mathrm{~cm}$小或等于 $155 \\mathrm{~cm}$; 标准体重的范围是: $29-50 \\mathrm{~kg}$, 即比 $29 \\mathrm{~kg}$ 大或等于 $29 \\mathrm{~kg}$, 比 $50 \\mathrm{~kg}$ 小或等于 $50 \\mathrm{~kg}$;据此对五年级 4 位女生的身高、体重分别进行对比, 即可得出结论。\n\n【详解】A. 王媛身高 $145 \\mathrm{~cm}$, 体重 $50 \\mathrm{~kg}$, 身高在 $138-155 \\mathrm{~cm}$ 范围内, 体重在 $29-50 \\mathrm{~kg}$ 范围内,符合标准;\n\nB. 李想身高 $151 \\mathrm{~cm}$, 体重 $46 \\mathrm{~kg}$, 身高在 $138-155 \\mathrm{~cm}$ 范围内, 体重在 $29-50 \\mathrm{~kg}$ 范围内, 符合标准;\n\nC. 刘菲身高 $143 \\mathrm{~cm}$, 体重 $43 \\mathrm{~kg}$, 身高在 $138-155 \\mathrm{~cm}$ 范围内, 体重在 $29-50 \\mathrm{~kg}$ 范围内, 符合标准;\n\nD. 张奕身高 $153 \\mathrm{~cm}$, 体重 $55 \\mathrm{~kg}$, 身高在 $138-155 \\mathrm{~cm}$ 范围内, 体重大于 $50 \\mathrm{~kg}$, 不符合标准;所以五年级 4 位女生的身高、体重情况。不符合标准的是张奕。\n\n故答案为: D\n\n【点睛】本题考查了学生从统计表中通过分析数据解答问题的能力。"} {"id": "9128", "image": [], "answer": "D", "solution": "null", "level": "三年级", "question": "要统计以下选项中的( ), 制成复式统计表更合适。", "options": "A. 三(1)班学生喜欢阅读的图书情况\n\nB. 三年级学生参加社团的情况\n\nC. 新华都超市 5 月份电器销售情况\n\nD. 三 (1) 班男生、女生 50 米跑步成绩", "subject": "计数", "analysis": "解$\\mathrm{D}$\n\n【分析】复式统计表的特点有: 复式统计表可表示多组数据; 可以更加清晰、明了地反映数据的情况,依此选择即可。\n\n【详解】A.三(1)班学生喜欢阅读的图书情况适合制成单式统计表;\n\nB. 三年级学生参加社团的情况适合制成单式统计表;\n\nC. 新华都超市 5 月份电器销售情况适合制成单式统计表;\n\nD. 三(1)班男生、女生 50 米跑步成绩适合制成复式统计表。故答案为: D\n\n【点睛】熟练掌握复式统计表的特点是解答此题的关键。"} {"id": "9049", "image": [], "answer": "B", "solution": "null", "level": "三年级", "question": "一本《童话故事》 17.9 元,一本《故事会》 12.5 元,小宁想各买一本, 30 元够吗?", "options": "A. 够\nB. 不够", "subject": "算术", "analysis": "解$\\mathrm{B}$\n\n把两本书的价钱相加, 再与 30 元进行比较即可解答。\n\n$17.9+12.5=30.4$ (元) $>30$ 元, 不够。\n\n故答案为: $B$\n\n本题主要考查学生对小数加法计算方法的掌握。"} {"id": "9060", "image": ["1470.jpg"], "answer": "A", "solution": "null", "level": "三年级", "question": "如图, 将正方形平均分成 10 份,如果正方形表示 1,则涂色部分应该表示()。\n\n", "options": "A. 0.4\nB. 0.6\nC. 4\nD. 6", "subject": "算术", "analysis": "解A\n\n根据题意可知,正方形表示 1 , 将正方形平均分成 10 份,涂色部分占其中的 4 份,小数表示为 0.4 , 据此即可解答。\n\n根据分析可知, 将正方形平均分成 10 份, 如果正方形表示 1 , 则涂色部分应该表示 0.4 。故答案为: A\n\n\n\n本题主要考查学生对小数意义的掌握和灵活运用。"} {"id": "9061", "image": [], "answer": "B", "solution": "null", "level": "三年级", "question": "张阿姨买葡萄花了 14.5 元, 比买苹果多花 2.8 元, 张阿姨买苹果花了()元。", "options": "A. 1.7\nB. 11.7\nC. 17.3", "subject": "算术", "analysis": "解$\\mathrm{B}$\n\n\n\n\n\n买葡萄花的钱减去 2.8 元等于买苹果花的钱。\n\n\n\n$14.5-2.8=11.7 ($ 元 $)$\n\n故答案为: $B$\n\n\n\n本题主要考查学生对小数减法计算方法的掌握。"} {"id": "9063", "image": ["1479.jpg", "1480.jpg", "1481.jpg"], "answer": "B", "solution": "null", "level": "三年级", "question": "下面水果中, 最便宜的水果是()。\n\n| 种类 | IN | | |\n| :--- | :---: | :---: | :---: |\n| 单价 | 2.5 元 | 2.1 元 | 3.4
元 |", "options": "A.\n\n\nB.\n\n\nC.\n\n", "subject": "算术", "analysis": "解B\n\n\n\n\n\n小数大小的比较, 先看小数的整数部分, 整数部分大的这个数就大, 整数部分相同的就看十分位, 十分位大的这个数就大, 十分位相同的, 再看百分位, 百分位大的这个数就大......以此类推。据此把这三种水果的价格进行比较即可得解。\n\n\n\n因为 2.1 元 $<2.5$ 元 $<3.4$ 元,\n\n所以最便宜的水果是火龙果。\n\n故答案为: B\n\n\n\n本题考查学生对小数比较大小的掌握和运用。"} {"id": "9064", "image": [], "answer": "A", "solution": "null", "level": "三年级", "question": "数学书的定价是 7 元 5 角, 用小数表示是()。", "options": "A. 7.50 元\nB. 7.05 元\nC. 75.0 元", "subject": "算术", "analysis": "解A\n\n元和角之间的进率是 10 , 据此将 7 元 5 角换算成元, 先用 5 角除以进率 10 化成元, 再加上 7 元即可。\n\n\n\n数学书的定价是 7 元 5 角, 用小数表示是 7.50 元。\n\n故答案为: A。\n\n\n\n单位换算首先要弄清是由高级单位化低级单位还是由低级单位化高级单位, 其次记住单位间的进率; 由高级单位化低级单位乘进率, 由低级单位化高级单位除以进率。"} {"id": "9065", "image": [], "answer": "A", "solution": "null", "level": "三年级", "question": "小数 30.25 读作 ( $\\quad$ )。", "options": "A. 三十点二五\nB. 三零点二五\nC. 三十点二十五", "subject": "算术", "analysis": "解A\n\n小数的读法: 小数的整数部分按照整数的读法读, 整数尾部的零不读。小数点读作点, 小数部分从左到右读数字, 中间的零也要读, 末尾的零也要读。\n\n小数 30.25 读作三十点二五;\n\n故答案为: A\n\n\n\n掌握小数的读法是解答此题的关键。"} {"id": "8374", "image": [], "answer": "C", "solution": "null", "level": "三年级", "question": "$69 \\times 96$ 的积的最高位是 $(\\quad)$ 位。", "options": "A. $十$\nB. 百\nC. 千", "subject": "算术", "analysis": "$\\mathrm{C}$\n\n【分析】 将 $69 \\times 96$ 计算出积即可解答。\n\n【详解】 $69 \\times 96=6624$\n\n故答案为:\n\n【点睛】此题主要考查学生对两位数乘两位数的应用。"} {"id": "8385", "image": [], "answer": "B", "solution": "null", "level": "三年级", "question": "下面各题的积最接近 5600 的是 ( ) 。", "options": "A. $58 \\times 65$\nB. $69 \\times 78$\nC. $79 \\times 88$", "subject": "算术", "analysis": "$\\mathrm{B}$\n\n【分析】根据题意, 把三个选项中的算式都估算出结果来, 然后再进一步解答即可。\n\n【详解】A. 把 58 看作 60,65 看作 $70,60 \\times 70=4200$, 所以, $58 \\times 65 \\approx 4200$;\n\nB. 把 69 看作 70,78 看作 $80,70 \\times 80=5600$, 所以, $69 \\times 78 \\approx 5600$;\n\nC. 把 79 看作 80,88 看作 $90,80 \\times 90=7200$, 所以, $79 \\times 88 \\approx 7200$;\n\n由以上分析可知, 只有 B 选项的结果最接近 5600 。\n\n故答案为: $\\mathrm{B}$\n\n【点睛】此题主要考查了学生对乘法估算方法知识的掌握情况, 在进行估算时, 我们首先应该对乘数进行“四舍五入”化成整十数相乘, 估算出答案, 再与给定的数进行比较即可。"} {"id": "8393", "image": [], "answer": "A", "solution": "null", "level": "三年级", "question": "36 个 12 相加的和是多少?列式是()。", "options": "A. $36 \\times 12$\nB. $36+12$\nC. $12+36$", "subject": "算术", "analysis": "A\n\n【分析】根据乘法和加法之间的关系进行解答, 求几个相同加数和的简便计算, 用乘法。\n\n【详解】根据题意列式: $36 \\times 12$\n\n故答案为: A\n\n【点睛】解决本题的关键是理解乘法的意义。"} {"id": "8394", "image": [], "answer": "B", "solution": "null", "level": "三年级", "question": "下列算式中, 积的末尾有 3 个 0 的是 ( )。", "options": "A. $50 \\times 30$\nB. $50 \\times 20$\nC. $50 \\times 70$", "subject": "算术", "analysis": "$\\mathrm{B}$\n\n【分析】整十数乘整十数的积的末尾至少有两个 0 , 如果两个因数的十位上的数相乘得到整十数, 那么积的末尾就有 3 个 0 ; 如十位上的数分别是 2 和 $5 、 5$ 和 $6 、 5$ 和 8 积的末尾就是 3 个 0 ; 由此解答。\n\n【详解】由分析可得, 积的末尾有 3 个 0 的算式是: $50 \\times 20$;\n\n故选 B。\n\n【点睛】此题主要考查整十数乘整十数积的末尾 0 的个数的判断方法。"} {"id": "8395", "image": [], "answer": "B", "solution": "null", "level": "三年级", "question": "下面表述正确的一句是()。", "options": "A. $56 \\times 50$ 积的末尾只有一个 0\nB. $76 \\times 35$ 的积是四位数\nC. $24 \\times 53$ 的得数比 1500 大, 比 1000 小", "subject": "算术", "analysis": "$\\mathrm{B}$\n\n【分析】逐项进行分析后, 再找出表述正确的一句即可。\n\n【详解】A、 $56 \\times 50=2800$, 所以积的末尾有两个 0 , 原说法错误;\n\nB、 $76 \\times 35=2660$, 所以说积是四位数的说法是正确的;\n\nC、 $24 \\times 53=1272$, 所以 $24 \\times 53$ 的得数比 1500 小, 比 1000 大, 原说法错误。\n故选 B。\n\n【点睛】解答此题关键是把每一个选项中的算式计算出得数, 再根据得数的情况进行判断并选择。"} {"id": "8396", "image": [], "answer": "B", "solution": "null", "level": "三年级", "question": "$\\square 3 \\times 27$ 的积是四位数, $\\square$ 里最小应该填 ( )", "options": "A. 3\nB. 4\nC. 5", "subject": "算术", "analysis": "$\\mathrm{B}$"} {"id": "8397", "image": [], "answer": "B", "solution": "null", "level": "三年级", "question": "比 36 的 18 倍少 129 的数是 ( )", "options": "A. 777\nB. 519\nC. 529", "subject": "算术", "analysis": "$\\mathrm{B}$"} {"id": "8398", "image": [], "answer": "C", "solution": "null", "level": "三年级", "question": "在计算 $35 \\times 28$ 时, 用 2 去乘 35 得到的是()", "options": "A. 70\nB. 70 个百\nC. 70 个十", "subject": "算术", "analysis": "$\\mathrm{C}$"} {"id": "9113", "image": [], "answer": "A", "solution": "null", "level": "三年级", "question": "估一估, 下边哪种水果平均每箱最轻?( )。\n\n王叔叔水果推进货的记录单\n\n| 品种 | 箱数 | 总质量/千克 |\n| :---: | :---: | :---: |\n| 草苞 | 6 | 144 |\n| 杏 | 4 | 128 |\n| 水蜜桃 | 5 | 175 |", "options": "A. 草莓\nB. 杏\nC. 水蜜桃", "subject": "算术", "analysis": "解A\n\n【分析】由表格中的数据, 可以知道每种水果的箱数和每种水果的总质量, 那么可以用每种水果总质量除以水果箱数, 就能得到每种水果每箱平均质量, 再来比较哪种的最轻。\n\n【详解】 草莓: $144 \\div 6=24$ (千克)\n\n杏: $128 \\div 4=32$ (千克)\n\n水蜜桃: $175 \\div 5=35$ (千克)\n\n24 千克 $<32$ 千克 $<35$ 千克, 所以草莓是最轻的。\n\n故答案为: A\n\n【点睛】本题考查学生对表格数据分析的掌握。在解决此题时, 要注意将每种水果的总质量和箱数对应上, 别对应错误, 导致计算结果不正确。"} {"id": "8400", "image": [], "answer": "B", "solution": "null", "level": "三年级", "question": "王老师给 40 名学生买奖品, 每件奖品的价格在 12 元到 19 元之间, 总价钱可能是 ( ) 元。", "options": "A. 400\nB. 600\nC. 900\nD. 1000", "subject": "算术", "analysis": "$\\mathrm{B}$\n\n【分析】根据题意可知, 用每件奖品的价钱乘需要买奖品的数量即可, 因此分别用 12 和 19 乘 40 即可选择, 依此计算并选择。\n\n【详解】 $12 \\times 40=480 ($ 元); $19 \\times 40=760 ($ 元)\n\nA. 400 元 $<480$ 元, 因此不满足;\n\nB. 480 元 $<600$ 元 $<760$ 元, 因此满足;\n\nC. 900 元 $>760$ 元, 因此不满足;\n\nD. 1000 元 $>760$ 元, 因此不满足;\n\n故答案为: B\n\n【点睛】熟练掌握两位数与两位数的乘法计算是解答此题的关键。"} {"id": "8411", "image": [], "answer": "D", "solution": "null", "level": "三年级", "question": "关于乘法计算,下列说法正确的是()。", "options": "A. 两位数乘两位数, 积一定是四位数\nB. $39 \\times 19$ 与 $36 \\times 10+9$ 结果相同\nC. 两个数的积一定大于这两个数的和\nD. $\\square 4 \\times \\square 5$ 积的末尾至少有 1 个 0", "subject": "算术", "analysis": "$\\mathrm{D}$\n\n【分析】根据乘法的计算法则, 然后举例说明即可解答。\n\n【详解】A. 例如 $10 \\times 10=100$, 积是三位数;\n\nB. $39 \\times 19=741,36 \\times 10+9=369,741>369$;\n\nC. 例如 $1 \\times 1=1,1+1=2,1<2$;\n\nD. $\\square 4 \\times \\square 5$ 个位上 $4 \\times 5=20$, 有 1 个零, 所以 $\\square 4 \\times \\square 5$ 积的末尾至少有 1 个 0 。\n\n故答案为: D\n\n【点睛】解答此题的关键是举出反例比较容易解答。"} {"id": "8420", "image": [], "answer": "B", "solution": "null", "level": "三年级", "question": "两位数乘两位数, $9 \\star \\times \\triangle 3=\\square 162$, 积的千位上的数是 ( )。", "options": "A. 1\nB. 2\nC. 3\nD. 4", "subject": "算术", "analysis": "$\\mathrm{B}$\n\n【分析】 $3 \\times 4=12$, 即 $\\star=4$, 因此分别计算出 $94 \\times 13 、 94 \\times 23 、 94 \\times 33 、 94 \\times 43$ 的积, 然后再进行选择即可。\n\n【详解】 $94 \\times 13=1222$;\n\n$94 \\times 23=2162 ;$\n\n$94 \\times 33=3102 ;$\n\n$94 \\times 43=4042$, 因此两位数乘两位数, $9 \\star \\times \\triangle 3=\\square 162$, 积的千位上的数是 2 。\n\n故答案为: $\\mathrm{B}$\n\n【点睛】熟练掌握两位数与两位数的乘法计算是解答此题的关键。"} {"id": "8421", "image": [], "answer": "C", "solution": "null", "level": "三年级", "question": "学校食堂买了 2 箱苹果, 每箱有 2 层, 每层有 15 个。每 4 个苹果约重 1 千克。每箱有多少个苹果?下面算式正确的是()。", "options": "A. $15 \\times 2 \\times 22$\nB. $15 \\times 2 \\times 4$\nC. $15 \\times 2$\nD. $15 \\times 2 \\times 22 \\times 1$", "subject": "算术", "analysis": "$\\mathrm{C}$\n\n【分析】根据题意可知, 用每层有苹果的个数 $\\times$ 每箱的层数 $=$ 每箱苹果的个数, 依此列式并选择即可。\n\n【详解】根据分析可知, 计算每箱苹果的个数可列式为: $15 \\times 2$ 。\n故答案为: $C$\n\n【点睛】此题考查的是根据题意列算式,应先找到题目中对应的关系再进行选择。"} {"id": "8422", "image": [], "answer": "D", "solution": "null", "level": "三年级", "question": "下面算式 ( ) 的积比 1000 大得多, 比 2000 少一些。", "options": "A. $45 \\times 18$\nB. $62 \\times 37$\nC. $21 \\times 52$\nD. $43 \\times 46$", "subject": "算术", "analysis": "$\\mathrm{D}$\n\n【分析】根据两位数乘两位数的计算法则, 分别计算出每个选项中的算式的积, 然后再选择即可。\n\n【详解】A. $45 \\times 18=810,810<1000$;\n\nB. $62 \\times 37=2294,2294>2000$;\n\nC. $21 \\times 52=1092,1000<1092<2000$;\n\nD. $43 \\times 46=1978,1000<1092<1978<2000$ 。\n\n故答案为: D\n\n【点睛】熟练掌握两位数与两位数的乘法计算是解答此题的关键。"} {"id": "8423", "image": [], "answer": "C", "solution": "null", "level": "三年级", "question": "妈妈在灵燕药店买了 12 盒儿童口罩, 每盒 27 元。妈妈说: “我买这些口罩要付 351 元”。李强认为妈妈算的钱数不对,并且说了道理。下面不能说明妈妈付错钱的理由是( )。", "options": "A. $27 \\times 12$ 的积的个位是 4 而不是 1 。\n\nB. $27 \\times 10=270,27 \\times 2=54,270+54=324,324$ 比 351 小。\n\nC. $27 \\times 10=270,27 \\times 12$ 的积比 270 大。\n\nD. 10 盒 270 元, 351 元减去 270 元等于 81 元, 81 元可以再买 3 盒, 351 元能买 13 盒。", "subject": "算术", "analysis": "$\\mathrm{C}$\n\n【分析】买儿童口罩的盒数 $\\times$ 每盒的价钱 $=$ 需要付的钱, 两位数乘两位数的计算方法: 先是第二个因数的个位上的数与第一个因数相乘, 所得的积末尾与个位对齐; 接着第二个因数的十位上的数与第一个乘数相乘, 所得的积末尾与十位对齐, 最后把两次乘得的积相加; 依此选择即可。\n\n【详解】A. $27 \\times 2=54$, 即 $27 \\times 12$ 的积的个位是 4 而不是 1 , 即此理由正确;\n\nB. $27 \\times 10=270,27 \\times 2=54,270+54=324,324$ 比 351 小, 即此理由正确;\n\nC. $27 \\times 10=270,27 \\times 12$ 的积比 270 大, 而 351 元 $>270$ 元, 即此理由错误;\n\nD. 10 盒 270 元, 351 元减去 270 元等于 81 元, 81 元可以再买 3 盒, 351 元能买 13 盒, 13 盒 $>12$盒, 即此理由正确。\n\n故答案为: C\n\n【点睛】此题考查的是经济问题的计算, 应熟练掌握经济问题中各部分之间的关系。"} {"id": "8424", "image": [], "answer": "B", "solution": "null", "level": "三年级", "question": "$50 \\times 80$ 的积末尾有 $(\\quad)$ 个 0 。", "options": "A. 4\nB. 3\nC. 2\nD. 1", "subject": "算术", "analysis": "$\\mathrm{B}$\n\n【分析】根据末尾有 0 的整数乘法的运算法则可知, 在计算 $50 \\times 80$ 时, 可先计算 $5 \\times 8,5 \\times 8$ 的结果是 40 , 然后再在 40 后边加上原来 50 和 80 后边的 0 , 积为 4000 , 即 $50 \\times 40$ 的积的末尾有 3 个零。\n\n【详解】 $50 \\times 80=4000$\n\n故答案为: B\n\n【点睛】解答此题根据因数末尾有 0 的乘法的简算方法计算。"} {"id": "8425", "image": ["1315.jpg", "1316.jpg", "1317.jpg", "1318.jpg", "1319.jpg"], "answer": "B", "solution": "null", "level": "三年级", "question": "在计算 $16 \\times 12$ 时, 小方是这样想的: $16 \\times 10=160,16 \\times 2=32,160+32=192$, 下面可以表示他的思考过程的图是 $(\\quad)$ 。\n\n", "options": "A.\n\n\nB.\n\nप\\#\\#\\#\\#\\#\\#\\#\n\n\n\nC.\n\n\nD.", "subject": "算术", "analysis": "B\n\n【分析】计算 $16 \\times 10=160$, 表示 16 个小正方形排一排, 10 排这样的小正方形就是 160 个, 再计算 $16 \\times 2=32$, 表示 16 个小正方形排一排, 2 排就是 32 个, 即 $160+32=192$, 据此解答。\n\n【详解】在计算 $16 \\times 32$ 时, 小方是这样想的: $16 \\times 10=160,16 \\times 2=32,160+32=192$, 下面可以表\n示他的思考过程的图是\n\n\n\n故答案为: B\n\n【点睛】熟练掌握两位数乘两位数的计算方法并灵活运用是解答本题的关键。"} {"id": "8427", "image": [], "answer": "B", "solution": "null", "level": "三年级", "question": "19 乘 45 的积的最高位是 ( )。", "options": "A. 十位\nB. 百位\nC. 千位", "subject": "算术", "analysis": "$\\mathrm{B}$\n\n【分析】根据两位数乘两位数的计算法则, 直接计算出 19 乘 45 的积, 然后根据计算出的结果进行选择即可。\n\n【详解】 $19 \\times 45=855$, 即 19 乘 45 的积的最高位是百位。故答案为: B\n\n【点睛】熟练掌握两位数与两位数的乘法计算是解答此题的关键。"} {"id": "8438", "image": [], "answer": "A", "solution": "null", "level": "三年级", "question": "在用坚式计算 $24 \\times 16$ 时, 用 1 去乘 24 得到的是( )。", "options": "A. 24 个十\nB. 24 个一\nC. 24 个百", "subject": "算术", "analysis": "A\n\n【分析】两位数乘两位数笔算法则:\n\n1、先用第二个因数的个位去乘第一个因数, 得数的末位与第一个因数的个位对齐;\n\n2、再用第二个因数的十位去乘第一个因数, 得数末位与第二个因数的十位对齐;\n\n3 、然后把两次乘的积加起来。\n\n【详解】根据整数乘法的计算方法, 16 中的 1 在十位上, 表示 1 个十和 24 相乘, 得到 24 个十。故答案为: A\n\n【点睛】熟练掌握两位数乘两位数笔算的法则是解决本题的关键。"} {"id": "8446", "image": [], "answer": "A", "solution": "null", "level": "三年级", "question": "与 $28 \\times 60$ 的计算结果相同的算式是 ( )。", "options": "A. $280 \\times 6$\nB. $208 \\times 6$\nC. $260 \\times 8$", "subject": "算术", "analysis": "A\n\n【分析】计算出各个算式的结果即可解答。\n\n【详解】 $28 \\times 60=1680$\n\nA. $280 \\times 6=1680$\n\nB. $208 \\times 6=1248$\n\nC. $260 \\times 8=2080$\n\n故答案为: A\n\n【点睛】本题主要考查学生对整数乘法计算方法的掌握。"} {"id": "8447", "image": [], "answer": "B", "solution": "null", "level": "三年级", "question": "$45 \\times 60$ 的积的末尾有 $(\\quad)$ 个 0 。", "options": "A. 1\nB. 2\nC. 3", "subject": "算术", "analysis": "B\n\n【分析】根据两位数乘两位数的计算法则, 计算出 $45 \\times 60$ 的积, 然后再选择即可。\n\n【详解】 $45 \\times 60=2700$, 即 $45 \\times 60$ 的积的末尾有 2 个 0 。\n\n故答案为: $\\mathrm{B}$\n\n【点睛】熟练掌握两位数与两位数的乘法计算是解答此题的关键。"} {"id": "8448", "image": ["1321.jpg", "1322.jpg"], "answer": "B", "solution": "null", "level": "三年级", "question": "悦耳音乐厅, 每排 62 个座位, 有 22 排, 一共多少个座位? 如下图坚式计算, 箭头所指的这一步表示 ( $\\quad$ ) 。\n\n", "options": "A. 2 排有 124 个座位\nB. 20 排有 1240 个座位\nC. 22 排有 1364 个座位", "subject": "算术", "analysis": "B\n\n【分析】两位数乘两位数, 坚式计算法则: 相同数位对齐, 从个位乘起; 先用第二个因数个位上的数去乘第一个因数, 得数的末位和因数的个位对齐; 再用第二个因数十位上的数去乘第一个因数, 得数\n的末位和十位对齐; 然后把两次乘得的积加起来。据此可知, 十位上的 2 与 62 的积是 124 个十, 表示 20 排有座位 1240 个; 据此解题即可。\n\n【详解】根据分析可知, 悦耳音乐厅, 每排 62 个座位, 有 22 排, 一共多少个座位? 如下图坚式计算,箭头所指的这一步表示 20 排有 1240 个座位。\n\n\n\n故答案为: B\n\n【点睛】熟练掌握两位数乘两位数的计算方法, 是解答此题的关键。"} {"id": "8449", "image": [], "answer": "C", "solution": "null", "level": "三年级", "question": "王老师买了 21 个篮球, 花的钱比 1200 元多比 1600 元少。篮球的价格可能是()。", "options": "A. 91 元/个\nB. 82 元/个\nC. 62 元/个\nD. 39 元/个", "subject": "算术", "analysis": "$\\mathrm{C}$\n\n【分析】A. 若 1 个篮球的价格是 91 元, 将 91 看成 90 , 则 21 个篮球的总钱数大于 1800 元;\n\nB. 若 1 个篮球的价格是 82 元, 将 82 看成 80 , 则 21 个篮球的总钱数大于 1600 元;\n\nC. 若 1 个篮球的价格是 62 元, 将 62 看成 60 , 则 21 个篮球的总钱数大于 1200 元而小于 1600 元;\n\nD. 若 1 个篮球的价格是 39 元, 将 39 看成 40 , 则 21 个篮球的总钱数大约是 840 元。\n\n【详解】A. $91 \\times 21 \\approx 90 \\times 21=1890$ (元);\n\n所以根据实际情况,付给店员超出 1890 元,不合题意;\n\nB. $82 \\times 21 \\approx 80 \\times 21=1680$ (元) ;\n\n因为 $80<82$, 所以根据实际情况, 付给店员超出 1680 元, 不符合题意;\n\nC. $62 \\times 21 \\approx 60 \\times 21=1260$ (元) ;\n\n因为 $60<62$, 所以所付钱数比 1200 元多比 1600 元少, 符合题意;\n\nD. $39 \\times 21 \\approx 40 \\times 21=840$ (元) ,\n\n因为 $39<40$, 所以根据实际情况, 付给店员少于 840 元, 不符合题意。\n\n故答案为: C\n\n【点睛】本题侧重考查知识点的理解能力和培养对知识点的理解能力。"} {"id": "8450", "image": [], "answer": "C", "solution": "null", "level": "三年级", "question": "$18 \\times 18$ 的积大约是 ( ) 。", "options": "A. 180\nB. 360\nC. 400", "subject": "算术", "analysis": "$\\mathrm{C}$\n\n【分析】把两位数看作与它接近的整十数, 然后相乘即可解答。\n\n【详解】 $18 \\times 18 \\approx 20 \\times 20=400$\n\n故答案为: C\n\n【点睛】熟练掌握整数乘法的估算方法是解答本题的关键。"} {"id": "8451", "image": [], "answer": "B", "solution": "null", "level": "三年级", "question": "$31 \\times \\square 2$ 的积是三位数, 嘬大填 ( )。", "options": "A. 2\nB. 3\nC. 4", "subject": "算术", "analysis": "$\\mathrm{B}$\n【分析】要使“ $31 \\times \\square 2$ ” 的积是三位数, 把 31 看作 30 , 因为 $30 \\times 30=900$; 再用 31 乘上 32 , 根据所得的积进一步解答即可。\n\n【详解】 $31 \\times 32=992$, 所以要使 $31 \\times \\square 2$ 的积是三位数, 口最大填 3 。\n\n故答案为: B\n\n【点睛】本题主要考查了学生运用估算的方法来解答问题的能力。"} {"id": "8453", "image": [], "answer": "C", "solution": "null", "level": "三年级", "question": "以下算式结果最接近 200 的是 ( )。", "options": "A. $691 \\div 3$\nB. $324 \\div 8$\nC. $19 \\times 11$", "subject": "算术", "analysis": "$\\mathrm{C}$\n\n【分析】根据题意, 先分别计算出各个选项中的算式, 再用所得的结果与 200 相减, 所得的差最小的那个算式的结果与 200 最接近。\n\n【详解】A. $691 \\div 3=230 \\ldots . .1,230-200=30$;\n\nB. $324 \\div 8=40 \\ldots . .4,200-40=160$;\n\nC. $19 \\times 11=209,209-200=9$;\n\n$160>30>9$\n\n故答案为: C\n\n【点睛】熟练掌握三位数除以一位数的计算方法、两位数乘两位数的计算方法, 是解答此题的关键。"} {"id": "8464", "image": [], "answer": "B", "solution": "null", "level": "三年级", "question": "在 $12 \\times 29$ 中, 因数 12 扩大到原数的 2 倍, 另一个因数不变, 积 ( )。", "options": "A. 不变\nB. 扩大到原数的 2 倍\nC. 扩大到原数的 4 倍", "subject": "算术", "analysis": "$\\mathrm{B}$\n\n【分析】在乘法算式里, 两个因数都不为 0 时, 一个因数不变, 另一个因数扩大到原数的几倍, 积就扩大到原数的几倍, 依此选择。\n\n【详解】根据分析可知: 在 $12 \\times 29$ 中, 因数 12 扩大到原数的 2 倍, 另一个因数不变, 积扩大到原数的 2 倍。\n\n故答案为: $\\mathrm{B}$\n\n【点睛】熟练掌握积的变化规律是解答此题的关键。"} {"id": "8469", "image": [], "answer": "B", "solution": "null", "level": "三年级", "question": "计算 $69 \\times \\square$ 口 $<6300$, 口中最大能填 ( )。", "options": "A. 90\nB. 91\nC. 92", "subject": "算术", "analysis": "$\\mathrm{B}$\n\n【分析】 将选项中的数据 90、91、92 分别代入算式和 69 相乘, 再和 6300 进行比较即可。\n\n【详解】A. $69 \\times 90=6210,6210<6300$;\n\nB. $69 \\times 91=6279,6279<6300$;\n\nC. $69 \\times 92=6348,6348>6300$ 。\n\n所以计算 $69 \\times \\square<6300$, 口中最大能填 91 。\n\n故答案为: $\\mathrm{B}$\n\n【点睛】此题主要考查两位数乘两位数的灵活运用。"} {"id": "8470", "image": [], "answer": "A", "solution": "null", "level": "三年级", "question": "下列算式中, 乘积小于 1500 的是( )。", "options": "A. $48 \\times 27$\nB. $31 \\times 52$\nC. $44 \\times 47$", "subject": "算术", "analysis": "A\n\n【分析】两位数乘两位数的估算, 先将给出的两位数看成与之接近的整十数, 再根据两位数乘两位数的方法进行计算即可, 计算的结果可与给出的式子的乘积比较, 判断大小, 如果乘数末尾有 0 的, 可以先将 0 之前的先乘, 再在乘积的末尾加 0 即可, 据此解答。\n\n【详解】A. $48 \\times 27$ 中把 48 近似看成 50 , 把 27 近似看成 30 , 乘积是 1500 , 乘积小于 1500 ;\n\nB. $31 \\times 52$ 中把 31 近似看成 30 , 把 52 近似看成 50 , 乘积是 1500 , 乘积大于 1500 ;\n\nC. $44 \\times 47$ 中把 44 近似看成 45 , 把 47 近似看成 50, 乘积是 2250 , 乘积大于 1500 。故答案为: A\n\n【点睛】熟练掌握两位数乘两位数的估算方法并灵活运用是解答本题的关键。"} {"id": "8471", "image": [], "answer": "C", "solution": "null", "level": "三年级", "question": "$25 \\times 32$ 的积的末尾", "options": "A. 没有 0\nB. 只有 1 个 0\nC. 有 2 个 0", "subject": "算术", "analysis": "C\n\n【分析】两位数乘两位数时, 先用第二个两位数的个位上的数分别与第一个两位数的每一位数相乘,乘得结果要与个位对齐, 再用两位数的十位上的数分别与第一个两位数的每一位数相乘, 乘得结果要与十位对齐, 然后两个结果相加就得到两位数乘两位数的结果。就求出 $25 \\times 32$ 的积, 再看积的末尾有几个 0 。\n\n【详解】 $25 \\times 32=800$\n\n则积的末尾有 2 个 0 。\n\n故答案为: $\\mathrm{C}$\n\n【点睛】熟练掌握两位数乘两位数的计算方法并正确计算是解决本题的关键。"} {"id": "8472", "image": [], "answer": "C", "solution": "null", "level": "三年级", "question": "笔算 $35 \\times 28,3$ 与 2 相乘得到的是 6 个 ( )。", "options": "A. -\nB. 十\nC. 百", "subject": "算术", "analysis": "$\\mathrm{C}$\n\n【分析】认清 3 在 35 中代表的是十位, 2 在 28 中代表的也是十位, 故 3 与 2 相乘相当于 $30 \\times 20$, 算出结果即可得到答案。\n\n【详解】 3 在 35 中代表的是十位, 即 30, 2 在 28 中代表的也是十位, 即 20, 故 3 与 2 相乘为: $30 \\times 20=600$, 为 6 个百,\n\n故答案为: C\n\n【点睛】本题主要考查两位数乘两位数的计算方法。"} {"id": "8476", "image": ["1329.jpg", "1330.jpg"], "answer": "D", "solution": "null", "level": "三年级", "question": "在计算 $13 \\times 12$ 时, 小东是这样想的:\n\n$$\n\\begin{gathered}\n13 \\times 10=13013 \\times 2=26 \\\\\n130+26=156\n\\end{gathered}\n$$\n\n下边能表示他思考过程的图是 $(\\quad)$ 。", "options": "A.\n", "subject": "算术", "analysis": "D\n\n【分析】小红计算 $13 \\times 12$ 时, 将 12 拆成 10 和 2 , 分别将这两个数与 13 相乘, 再将乘积加起来。\n\n【详解】能表示他思考过程的图是:\n\n\n\n故答案为: D。\n\n【点睛】计算两位数乘两位数数, 可以将其中一个两位数拆成整十数和一位数, 分别求出这两个数与另一个两位数相乘, 再将乘积加起来即可。"} {"id": "8487", "image": [], "answer": "C", "solution": "null", "level": "三年级", "question": "$13 \\times 25$ 的结果比 $14 \\times 25$ 的结果少 ( )。", "options": "A. 一个 1\nB. 一个 13\nC. 一个 25\nD. $一 个 14$", "subject": "算术", "analysis": "$\\mathrm{C}$\n\n【分析】计算出 $14 \\times 25$ 和 $13 \\times 25$ 的结果, 再相减即可解答。\n\n【详解】 $14 \\times 25=350$\n\n$13 \\times 25=325$\n\n$350-325=25$\n\n故答案为: $\\mathrm{C}$\n\n【点睛】本题主要考查学生对整数乘法计算方法的掌握和灵活运用。"} {"id": "8496", "image": [], "answer": "C", "solution": "null", "level": "三年级", "question": "王老师为同学们买了 29 本书, 每本书的价格在 22 元至 29 元之间, 这些书的总价格大约()。", "options": "A. 不到 300 元\nB. 在 300 元到 600 元之间\nC. 在 600 元到 900 元之间\nD. 超过 900 元", "subject": "算术", "analysis": "$\\mathrm{C}$\n\n【分析】如果每本书的价格都是 22 元的, 那么这些书的总价就是 29 个 22 元, 用 29 乘 22 求出书的总价; 如果每本书的价格都是 29 元, 这些书的总价就是 29 个 29 元, 用 29 乘 29 求出这些书的总价,计算时把 22 看成 20,29 看成 30 进行估算即可。\n\n【详解】 $29 \\times 22 \\approx 600$ (元)\n\n$29 \\times 29 \\approx 900($ 元 $)$\n\n即: 这些书的总价格大约在 600 元到 900 元之间。\n\n故答案为: C\n\n【点睛】解决本题根据乘法的意义列出算式, 再根据乘法估算的方法求解。"} {"id": "8498", "image": [], "answer": "A", "solution": "null", "level": "三年级", "question": "与 $20 \\times 30$ 的积相等的算式是()。", "options": "A. $2 \\times 3 \\times 100$\nB. $2 \\times 3 \\times 10$\nC. $2 \\times 30+10$\nD. $2 \\times 3+100$", "subject": "算术", "analysis": "A\n\n【分析】从题中可知 $20 \\times 30=600$, 将四个选项依次算出结果, 即可解答。\n\n【详解】A. $2 \\times 3 \\times 100=600$;\n\nB. $2 \\times 3 \\times 10=60$;\n\nC. $2 \\times 30+10=60+10=70$;\n\nD. $2 \\times 3+100=6+100=106$;\n\n故答案为: A。\n\n【点睛】本题主要考查学生对整数乘法的计算, 属于基础题。"} {"id": "8499", "image": [], "answer": "C", "solution": "null", "level": "三年级", "question": "$\\square 3 \\times 12$ 的积最大是 ( )。", "options": "A. 1563\nB. 143\nC. 1116\nD. 33", "subject": "算术", "analysis": "C\n\n【分析】要使 $\\square 3 \\times 12$ 的积最大, 因此 $\\square$ 内的数应最大, 即口内的数应为 9 , 也就是直接计算出 $93 \\times 12$ 的积即可选择。\n\n【详解】 $93 \\times 12=1116$\n\n故答案为: C\n\n【点睛】熟练掌握两位数与两位数的乘法计算是解答此题的关键。"} {"id": "8503", "image": [], "answer": "C", "solution": "null", "level": "三年级", "question": "浩浩每分钟能打 27 个字, 1 小时他能打()个字。", "options": "A. 27\nB. 270\nC. 1620", "subject": "算术", "analysis": "$\\mathrm{C}$\n\n【分析】 1 小时 $=60$ 分钟, 用每分钟打字的个数乘 60 即可解答。\n\n【详解】 $27 \\times 60=1620$ (个)\n\n故答案为: C\n\n【点睛】本题主要考查了基本的数量关系:工作量 $=$ 工作效率 $\\times$ 工作时间。"} {"id": "8514", "image": [], "answer": "C", "solution": "null", "level": "三年级", "question": "同学们去某地研学旅行, 每人应付车票和门票共计 58 元, 31 人大约需准备 ( ) 元钱。", "options": "A. 1500\nB. 1700\nC. 1800", "subject": "算术", "analysis": "$\\mathrm{C}$\n\n【分析】要求 31 人大约需准备多少钱, 这里需要用到估算, 把 58 看成 60,31 看成 30 , 用 $60 \\times 30$ 即可解答。\n\n【详解】 $58 \\times 31 \\approx 60 \\times 30=1800 ($ 元)\n\n故答案为: C\n\n【点睛】两位数与两位数的乘法估算时, 把两位数看成与它接近的整十数。"} {"id": "8524", "image": [], "answer": "B", "solution": "null", "level": "三年级", "question": "$19 \\times(\\quad)<600$, 括号里最大能填()。", "options": "A. 30\nB. 31\nC. 33", "subject": "算术", "analysis": "$\\mathrm{B}$\n\n【分析】根据两位数乘两位数的计算方法, 依次计算出每个选项与 19 的乘积即可解答。\n\n【详解】A. $19 \\times 30=570,570<600$;\n\nB. $19 \\times 31=589,589<600$;\n\nC. $19 \\times 32=608,608>600$;\n\n所以括号里最大能填 31 。\n\n故答案为: B\n\n【点睛】熟练掌握两位数乘两位数的计算方法是解答此题的关键。"} {"id": "8525", "image": [], "answer": "B", "solution": "null", "level": "三年级", "question": "如果一个因数乘 4 得 360, 那么这个数乘 30 得()。", "options": "A. 270\nB. 2700\nC. 120", "subject": "算术", "analysis": "$\\mathrm{B}$\n\n【分析】根据一个因数乘 4 得 360 , 先计算出这个因数用除法计算, 再用因数乘 30 计算最后的结果,据此解答。\n\n【详解】 $360 \\div 4=90$\n\n$90 \\times 30=2700$\n\n如果一个因数乘 4 得 360 , 那么这个数乘 30 得(2700)。\n\n故选: B\n\n【点睛】熟练掌握两位数乘两位数以及三位数除以一位数的计算方法是解答本题的关键。"} {"id": "8526", "image": [], "answer": "A", "solution": "null", "level": "三年级", "question": "乘积小于 1500 的是", "options": "A. $38 \\times 29$\nB. $31 \\times 53$\nC. $42 \\times 41$", "subject": "算术", "analysis": "A\n\n【分析】两位数乘两位数的计算方法, 先用乘数个位的数去乘被乘数, 得数的末位和乘数的个位对齐,再用乘数十位上的数去乘被乘数, 得数的末位和乘数的十位对齐, 然后把两次乘得的数加起来; 计算\n出每个选项的结果即可解答。\n\n【详解】A. $38 \\times 29=1102$, 积小于 1500 ;\n\nB. $31 \\times 53=1643$, 积大于 1500 ;\n\nC. $42 \\times 41=1722$, 积大于 1500 ;\n\n故答案为: A\n\n【点睛】熟练掌握两位数乘两位数的计算方法是解答此题的关键。"} {"id": "8527", "image": [], "answer": "C", "solution": "null", "level": "三年级", "question": "一个书架能放 80 本书, 15 个这样的书架能放()本书。", "options": "A. 95\nB. 800\nC. 1200", "subject": "算术", "analysis": "C\n\n【分析】要求 15 个这样的书架能放多少本书, 即求 15 个 80 的和, 用 15 乘 80 即可解答。\n\n【详解】 $15 \\times 80=1200($ 本 $)$\n\n故答案为: $\\mathrm{C}$\n\n【点睛】熟练掌握两位数乘两位数的计算方法是解答此题的关键。"} {"id": "8530", "image": [], "answer": "B", "solution": "null", "level": "三年级", "question": "两个因数都是整十数, 积的末尾至少有 ( )个 0 。", "options": "A. 1\nB. 2\nC. 3", "subject": "算术", "analysis": "$\\mathrm{B}$\n\n【分析】因数 $\\times$ 另一个因数 $=$ 积, 假设因数为 10 , 另一个因数为 50 , 那么积为 500 , 积末尾有 2 个 0 ;假设因数为 30 , 另一个因数为 20 , 那么积为 600 , 积末尾有 2 个 0 ;\n\n再假设因数为 20 , 另一个因数为 50 , 那么积为 1000 , 积末尾有 3 个 0 。\n\n【详解】两个因数都是整十数, 积的末尾至少有 2 个 0 。\n\n【点睛】熟练掌握两个乘数末尾都有 0 的乘法是解答本题的关键, 然后再举例进行解答。"} {"id": "8541", "image": [], "answer": "B", "solution": "null", "level": "三年级", "question": "阳光超市卖出 5 箱色拉油, 每箱 6 瓶, 每瓶色拉油的单价是 45 元, 表示每箱可卖多少元的算式是", "options": "A. $45 \\times 6 \\times 5$\nB. $45 \\times 6$\nC. $45 \\times 5$", "subject": "算术", "analysis": "$\\mathrm{B}$\n\n【分析】本题要求每箱可卖的价钱是多少, 只要用每瓶色拉油的单价, 乘每箱有色拉油的瓶数即可解答。\n\n【详解】根据分析可知, 每箱可卖的价钱 $=45 \\times 6=270$ (元);\n\n故答案为: B\n\n【点睛】此题考查的是经济问题的计算, 要根据问题来确定需要用到哪些条件, 这是解答本题的关键。"} {"id": "8550", "image": [], "answer": "A", "solution": "null", "level": "三年级", "question": "一个两位数乘 10 , 所得的积 ( )。", "options": "A. 一定是三位数\nB. 一定是四位数\nC. 可能是三位数也可能是四位数", "subject": "算术", "analysis": "A\n\n【分析】当最小的两位数乘 10 时, 乘积最小。当最大的两位数乘 10 时, 乘积最大。分别求出这两个乘积, 再进行判断。\n\n【详解】 $10 \\times 10=100,99 \\times 10=990$\n\n则最大和最小乘积均为三位数, 两位数乘 10 , 所得的积一定是三位数。\n\n故答案为: A。\n\n【点睛】解决本题时应先求出最大和最小的乘积, 再通过判断这两个乘积的位数进行解答。"} {"id": "8551", "image": [], "answer": "C", "solution": "null", "level": "三年级", "question": "计算 $32 \\times 58$ 时, 用“ 58 ”中十位上的 5 乘 32 的积是", "options": "A. 150\nB. 160\nC. 1600", "subject": "算术", "analysis": "C\n\n【分析】根据两位数乘两位数的计算方法可知, 用“ 58 ”中十位上的 5 乘 32 , 表示 5 个十乘 32 , 得 160 个十, 也就是 1600 。\n【详解】用“ 58 ”中十位上的 5 乘 32 的积是 1600 。\n\n故答案为: $\\mathrm{C}$ 。\n\n【点睛】本题考查两位数乘两位数的乘法计算方法, 注意“ 58 ”中十位上的 5 表示 5 个十。"} {"id": "8552", "image": [], "answer": "C", "solution": "null", "level": "三年级", "question": "$25 \\times 80$ 的积的末尾有 ( ) 个 0 。", "options": "A. 1\nB. 2\nC. 3", "subject": "算术", "analysis": "$\\mathrm{C}$\n\n【分析】计算因数末尾有 0 的乘法, 先把 0 前面的数相乘, 再看两个因数的末尾一共有几个 0 , 就在积的末尾添几个 0 。据此求出 $25 \\times 80$ 的积, 再进行解答。\n\n【详解】 $25 \\times 80=2000$\n\n则积的末尾有三个 0 。\n\n故答案为: C\n\n【点睛】求两个数的积的末尾 0 的个数, 可以先求出它们的乘积, 然后再进一步解答。"} {"id": "8556", "image": [], "answer": "B", "solution": "null", "level": "三年级", "question": "$412 \\div 4(\\quad) 972 \\div 9$ 。", "options": "A. $>$\nB. $<$\nC. $=$", "subject": "算术", "analysis": "$\\mathrm{B}$\n\n【分析】先求出两个算式的结果, 再进行比较。\n\n【详解】 $412 \\div 4=103$,\n\n$972 \\div 9=108$,\n\n因为 $103<108$,\n\n所以 $412 \\div 4<972 \\div 9$ 。\n\n故选 B。\n\n【点睛】此题先求出两个算式的结果, 然后比较大小。"} {"id": "8567", "image": [], "answer": "B", "solution": "null", "level": "三年级", "question": "下面各题商最接近 140 的算式是()", "options": "A. $20 \\times 7$\nB. $417 \\div 3$\nC. $580 \\div 5$", "subject": "算术", "analysis": "$\\mathrm{B}$\n\n【详解】试题分析:问商最接近 140 的算式是哪个, 因 A 是乘法算式不对, B、把 417 看成 420 再除以 3 得 140 , 商最接近 $140, C$ 、把 580 看成 600 除以 5 得 120 , 据此解答.\n\n解:由分析可知\n\nA、是乘法算式不对,\n\nB、 $417 \\div 3 \\approx 420 \\div 3=140$,\n\nC、 $580 \\div 5 \\approx 600 \\div 5=120$,\n\n所以 $417 \\div 3$ 的商最接近 140 ;\n\n故选 B.\n\n点评: 考查了商的估算方法, 可把被除数和除数分别看成最接近它的整百整十数试商."} {"id": "8576", "image": [], "answer": "B", "solution": "null", "level": "三年级", "question": "从 285 中连续减去 3 , 要减 $(\\quad)$ 次才能使结果为 0 .", "options": "A. 100\nB. 95\nC. 282", "subject": "算术", "analysis": "$\\mathrm{B}$\n\n【分析】求从 285 中连续减去 3 , 要减多少次才能使结果为 0 , 即求 285 中含有多少个 3 , 根据除法的意义, 用 285 除以 3 即得从 285 中连续减去 3 , 要减多少次才能使结果为 0 。\n\n【详解】 $285 \\div 3=95$\n\n即从 285 中连续减去 3 , 要减 95 次才能使结果为 0 。\n\n故答案为: B\n\n【点睛】根据题意明确本题即是求 285 中含有多少个 3 , 然后用除法计算是完成本题的关键。"} {"id": "8577", "image": [], "answer": "B", "solution": "null", "level": "三年级", "question": "四年级 (1) 班同学办手抄报作品展庆祝国庆节, 每份手抄报长 5 分米. 如果宣传板长 40 分米,每行最多能贴 $(\\quad)$ 张手抄报.", "options": "A. 7\nB. 8\nC. 9", "subject": "算术", "analysis": "$\\mathrm{B}$\n\n【详解】试题分析:要求每行最多能贴多少张手抄报, 也就是求 40 分米里面有几个 5 分米, 根整数除法的意义,用除法计算.\n\n解: $40 \\div 5=8($ 张 $) ;$\n\n答:每行最多能贴 8 张手抄报.\n故选 B.\n\n点评: 此题考查整数除法的意义: 明确求一个数里面有几个另一个数, 用除法计算."} {"id": "8578", "image": [], "answer": "C", "solution": "null", "level": "三年级", "question": "$6 \\longdiv { \\square 1 2 }$, 在口里填() 能使商是三位数.", "options": "A. $1 \\sim 4$\nB. $5 \\sim 9$\nC. $6 \\sim 9$", "subject": "算术", "analysis": "$\\mathrm{C}$\n\n【详解】试题分析:根据除数是一位数的计算方法知:当被除数的最高位大于或等于除数时, 商的位数和被除数的位数相同. 据此解答.\n\n解: 根据以上分析知: $\\square 12 \\div 6$, 要使商是三位数, 口里的数应大于或等于 6 .\n\n故选 C.\n\n点评: 本题主要考查了根据除法的计算方法判断商的位数的知识."} {"id": "8579", "image": [], "answer": "B", "solution": "null", "level": "三年级", "question": "642 除以 6 等于 107 , 表示 $(\\quad)$", "options": "A. 642 里面有 107 个 6\nB. 642 里面有 6 个 107\nC. 642 的 6 倍是 107", "subject": "算术", "analysis": "$\\mathrm{B}$\n\n【详解】试题分析:根据除法的意义, 642 除以 6 等于 107 , 表示 642 里面有 6 个 107.解:由分析知,\n\n642 除以 6 等于 107, 表示 642 里面有 6 个 $107 ;$\n\n故选 B.\n\n点评: 此题考查了除法的意义及运用."} {"id": "8583", "image": [], "answer": "A", "solution": "null", "level": "三年级", "question": "下列三句话中不正确的一句是 ( )。", "options": "A. $18 \\times 19$ 的积比 400 大\n\nB. 两位数乘两位数, 积不可能是五位数\n\nC. 一本书 356 页, 丁丁每天看 30 页, 12 天能看完", "subject": "算术", "analysis": "A\n\n【解析】A. 运用估算来分析;\n\nB. 赋值法。最大的两位数 $99,99 \\times 99$ 的积是四位数来判断;\n\nC. 求出 12 天看的页数和 356 页进行比较即可解答。\n\n【详解】A. $18 \\times 19 \\approx 20 \\times 20=400,18<20,19<20,18 \\times 19$ 的积比 400 小, 所以此选项说法错误;\n\nB. 最大的两位数是 $99,99 \\times 99=9801$, 积是四位数, 此选项说法正确;\n\nC. $12 \\times 30=360$ (页) , $360>356$, 能看完, 此选项说法正确。\n\n故答案为: A\n\n【点睛】解答此题要对每一个选项进行分析, 确定哪个选项是正确的。"} {"id": "8594", "image": [], "answer": "B", "solution": "null", "level": "三年级", "question": "$49 \\times 50$ 的积比 $50 \\times 50$ 的积少 $(\\quad)$ 。", "options": "A. 50\nB. 50\nC. 49", "subject": "算术", "analysis": "$\\mathrm{B}$\n\n【解析】两位数乘整十、整百数的口算方法: 先把整十、整百数 0 前面的数与两位数相乘, 计算出积后,再看整十、整百数的末尾有几个 0 , 就在积的末尾添上几个 0 。\n\n【详解】 $49 \\times 50=2450$\n\n$50 \\times 50=2500$\n\n$2500-2450=50$\n\n故答案为: $B$\n\n【点睛】此题运用两位数乘整十数和整十数乘整十数的口算方法进行计算, 注意计算时因数末尾的 0 。"} {"id": "8604", "image": [], "answer": "B", "solution": "null", "level": "三年级", "question": "$\\square \\times 30$ 的积最接近 1400 , $\\square$ 可能是 ( )。", "options": "A. 43\nB. 47\nC. 52", "subject": "算术", "analysis": "$\\mathrm{B}$\n\n【分析】 $\\square \\times 30$ 的积最接近 1400 , 把选项中的数代入 $\\square \\times 30$, 求出积, 找出最接近 1400 的即可求解。\n\n【详解】 $43 \\times 30=1290$\n\n$47 \\times 30=1410$\n\n$52 \\times 30=1560$\n\n显然 1410 最接近 1400, 所以口可能是 47。\n故选: B\n\n【点睛】解决本题也可以用 1400 除以 30 , 求出商, 找出与求出的商最接近的数。"} {"id": "8605", "image": [], "answer": "B", "solution": "null", "level": "三年级", "question": "计算 $43 \\times 25$ 时, 用 25 十位上的 2 去乘 43 , 得到的是 ( )。", "options": "A. 86\nB. 86 个十\nC. 86 个百", "subject": "算术", "analysis": "B\n\n【分析】计算 $43 \\times 25$ 时, 用 25 十位上的 2 去乘 43 , 也就是 $43 \\times 20=860$, 表示 86 个十, 由此求解。\n\n【详解】用 25 十位上的 2 去乘 43 , 是 2 个十与 43 相乘, 即: $43 \\times 2=86$, 得到是 86 个十, 即 860 。故答案为: B\n\n【点睛】解决本题关键是找清楚数字所在的数位, 表示的计数单位。"} {"id": "8606", "image": [], "answer": "A", "solution": "null", "level": "三年级", "question": "两位数乘两位数, 积最小是 $(\\quad)$ 。", "options": "A. 100\nB. 110\nC. 120", "subject": "算术", "analysis": "A\n\n【解析】两位数乘两位数当两位数都是最小的 10 , 它们的乘积最小, 据此解答。\n\n【详解】最小的两位数是 10 ;\n\n$10 \\times 10=100$\n\n所以两位数乘两位数积最小是 100 。\n\n故答案为: A。\n\n【点睛】本题关键是只有两位数都是最小的两位数, 所得的积才最小。"} {"id": "8610", "image": [], "answer": "C", "solution": "null", "level": "三年级", "question": "李阿姨买了 19 盆花, 每盆花的价格在 31 和 39 元之间, 这些盆花的总价钱()。", "options": "A. 不足 400 元\nB. 在 400 和 600 元之间\nC. 在 600 和 800 元之间\nD. 超过 800 元", "subject": "算术", "analysis": "$\\mathrm{C}$\n\n【分析】花的总价钱 $=$ 盆数 $\\times$ 每盆花的价格, 19 的近似数是 20,31 和 39 的近似数分别是 30 和 40 ,代入数据计算即可。\n\n【详解】 $19 \\approx 20,31 \\approx 30,39 \\approx 40$\n\n$19 \\times 31 \\approx 600($ 元 $)$\n\n$19 \\times 39 \\approx 800 ($ 元 $)$\n\n所以这些盆花的总价钱 600 元至 800 元之间。\n\n故答案为: C\n\n【点睛】本题考查数的估算, 解决本题的关键是准确找一个数的近似数, 并能正确计算。"} {"id": "8629", "image": ["1359.jpg"], "answer": "A", "solution": "null", "level": "三年级", "question": "下面是四位同学计算 $14 \\times 12$ 的思考过程, 其中合理的有( )个。\n\n", "options": "A. 4\nB. 3\nC. 2\nD. 1", "subject": "算术", "analysis": "A\n\n【分析】计算 $14 \\times 12$ 时, 可以将 14 拆成 2 个 7 , 分别求出 2 个 7 与 12 的积, 再相加, 即 $14 \\times 12=12 \\times 7$ $+12 \\times 7$ 。 计算 $14 \\times 12$ 时, 可以将 12 拆成 3 个 4 , 分别求出 3 个 4 与 14 的积, 再相加, 即 $14 \\times 12=14 \\times 4$ $+14 \\times 4+14 \\times 4$ 。计算 $14 \\times 12$ 时, 可以将 12 拆成 10 和 2 , 分别求出 $10 、 2$ 与 12 的积, 再相加, 即 $14 \\times 12$ $=14 \\times 10+14 \\times 2$ 。计算 $14 \\times 12$ 时, 可以将 14 拆成 10 和 4 , 将 12 拆成 10 和 2 , 分别求出 10 与 $10 、 2$以及 4 与 $10 、 2$ 的积, 再相加, 即 $14 \\times 12=10 \\times 10+10 \\times 4+2 \\times 10+2 \\times 4$ 。\n\n【详解】由分析得:\n\n这四位同学的思考过程均合理。\n\n故答案为: A\n\n【点睛】本题考查两位数乘两位数的计算方法, 需熟练掌握。"} {"id": "9062", "image": ["1471.jpg", "1472.jpg", "1473.jpg", "1474.jpg", "1475.jpg", "1476.jpg", "1477.jpg", "1478.jpg"], "answer": "A", "solution": "null", "level": "三年级", "question": "下面图形的阴影部分能用 0.3 表示的是()。", "options": "A.\n\n\nB.\n\n\nC.\n\n\nD.\n\n", "subject": "解析几何", "analysis": "解A\n\n写出各个选项表示的数即可解答。\n\nA.\n\n\n\nB.\n\n\n\nC.\n\n\n\n阴影表示 $\\frac{1}{3}$\n\nD.\n\n\n\n故答案为: A\n\n\n\n熟练掌握小数的意义是解答本题的关键。"} {"id": "9067", "image": [], "answer": "C", "solution": "null", "level": "三年级", "question": "小梅有三件上衣和两条裤子, 她有 ( $\\quad$ ) 种穿法。", "options": "A. 5\nB. 3\nC. 6\nD. 4", "subject": "组合数学", "analysis": "解C\n\n分两步进行, 先选上衣, 有 3 种选法, 每件上衣搭配不同的裤子有 2 种不同方法, 所以三件上衣,两条裤子搭配方法共有: $3 \\times 2=6$ (种),据此解答即可。\n\n$3 \\times 2=6$ (种)\n\n故答案为: C\n\n本题考查了乘法原理: 做一件事, 完成它需要分成 $\\mathrm{n}$ 个步骤, 做第一步有 $\\mathrm{m}_{1}$ 种不同的方法,做第二步有 $\\mathrm{m}_{2}$ 种不同的方法,做第 $\\mathrm{n}$ 步有 $\\mathrm{m}_{\\mathrm{n}}$ 种不同的方法, 那么完成这件事共有 $\\mathrm{N}$ $=\\mathrm{m}_{1} \\times \\mathrm{m}_{2} \\times \\mathrm{m}_{3} \\times \\ldots \\ldots \\times \\mathrm{m}_{\\mathrm{n}}$ 种不同的方法。"} {"id": "9078", "image": [], "answer": "C", "solution": "null", "level": "三年级", "question": "兰兰有两件上衣、三条裤子,每次上装和下装各穿一件,一共有()种不同的穿法。", "options": "A. 4\nB. 5\nC. 6", "subject": "组合数学", "analysis": "解C\n\n根据题意可知, 每件上衣都可以与 3 条裤子搭配, 所以有 3 种穿法; 每条裤子都可以与 2 件上衣搭配, 所以有 2 种穿法。则 2 件上衣搭配 3 条裤子一共就有 $2 \\times 3$ 种不同的穿法; 据此解答即可。\n\n$2 \\times 3=6$ (种)\n\n则一共有 6 种不同的穿法。\n\n故答案为: C。\n\n本题考查了排列组合中的乘法原理, 需要明确 2 件上衣和 3 条裤子各有几种选择, 然后相乘即可得解。"} {"id": "9088", "image": [], "answer": "C", "solution": "null", "level": "三年级", "question": "12 名同学参加象棋比赛, 如果每 2 名同学赛一局, 一共要赛 ( ) 局。", "options": "A. 24\nB. 48\nC. 66\nD. 132", "subject": "组合数学", "analysis": "解$\\mathrm{C}$\n\n因为每一名同学都要和其他 11 名同学比赛, 每名同学比赛 11 局, $12 \\times 11=132$ (局), 但\n每两名同学之间都重复了一次, 因此一共要赛 $132 \\div 2=66$ (局)。\n\n$12 \\times(12-1) \\div 2$\n\n$=132 \\div 2$\n\n$=66($ 局 $)$\n\n一共要赛 66 局。\n\n故选: C。\n\n在循环赛制中, 参赛人数和比赛场数的关系为: 比赛场数=参赛人数 $\\times$ (参赛人数 -1 ) $\\div 2$ 。"} {"id": "8549", "image": ["1339.jpg", "1340.jpg", "1341.jpg"], "answer": "C", "solution": "null", "level": "三年级", "question": "一个保险柜的密码是“1764”, 下面钥匙 ( ) 能打开它。", "options": "A.\n\n\nB.\n\n\nC.\n\n", "subject": "画法几何学", "analysis": "$\\mathrm{C}$\n\n【分析】两位数乘两位数时, 用第二个因数个位上的数乘第一个因数, 再用第二个因数十位上的数乘第一个因数, 再将乘得的数加起来; 据此解答即可。\n\n【详解】A. $45 \\times 34=1530$;\n\nB. $39 \\times 36=1404$;\n\nC. $63 \\times 28=1764$;\n\n故答案为: $\\mathrm{C}$ 。\n\n【点睛】熟练掌握两位数乘两位数的计算方法并正确计算是解决本题的关键。"} {"id": "9185", "image": [], "answer": "B", "solution": "null", "level": "三年级", "question": "早上国旗飘向太阳升起的方向, 这时吹的是 (\n\n) 风。", "options": "A. 东\nB. 西\nC. 南", "subject": "逻辑题", "analysis": "解$\\mathrm{B}$"} {"id": "9192", "image": ["1508.jpg", "1509.jpg", "1510.jpg", "1511.jpg", "1512.jpg", "1513.jpg"], "answer": "B", "solution": "null", "level": "三年级", "question": "下面三个图形中, ( )面积最大。(每个口代表 1 平方厘米。)", "options": "A.\n\n\nB.\n\n\nC.\n\n", "subject": "度量几何学", "analysis": "解$\\mathrm{B}$\n\n【分析】这个图形由多少个小方格组成, 则这个图形的面积就是几平方厘米, 依此计算出每个图形的面积,然后再选择即可。\n\n【详解】A.\n\n\n此图的面积是 8 平方厘米。\n\nB.\n\n\n\nC.\n\n\n\n10 平方厘米 $>9$ 平方厘米 $>8$ 平方厘米\n\n故答案为: $B$\n\n【点睛】熟练掌握对面积的认识是解答此题的关键。"} {"id": "9203", "image": [], "answer": "B", "solution": "null", "level": "三年级", "question": "表示教室面积的大小, 使用 ( )这个单位比较合适。", "options": "A. 米\nB. 平方米\nC. 平方分米", "subject": "度量几何学", "analysis": "解$\\mathrm{B}$\n\n【分析】根据面积单位的认识, 边长 1 米的正方形面积是 1 平方米, 以及生活经验进行选择即可。\n\n【详解】表示教室面积的大小, 使用平方米这个单位比较合适。\n\n故答案为: $\\mathrm{B}$\n\n【点睛】关键是建立单位标准, 可以利用身边熟悉的事物建立单位标准。"} {"id": "9210", "image": [], "answer": "C", "solution": "null", "level": "三年级", "question": "一块长方形的玻璃, 长是 10 分米, 宽是 6 分米, 如果长和宽都减少 2 分米, 则面积减少 ( )平方分米。", "options": "A. 32\nB. 4\nC. 28", "subject": "度量几何学", "analysis": "解$\\mathrm{C}$\n\n【分析】已知长是 10 分米, 宽是 6 分米, 如果长和宽都减少 2 分米, 则变化后的长和宽为 $(10-2)$分米、(6-2) 分米, 然后根据长方形的面积公式, 分别求出变化前后长方形的面积, 再相减即可。\n\n【详解】 $10 \\times 6=60$ (平方分米)\n\n$$\n(10-2) \\times(6-2)\n$$\n\n$=8 \\times 4$\n\n$=32($ 平方分米 $)$\n\n$60-32=28($ 平方分米 $)$\n\n面积减少 28 平方分米。\n\n故答案为: $\\mathrm{C}$\n【点睛】本题考查了长方形面积公式的应用。"} {"id": "9213", "image": ["1521.jpg"], "answer": "C", "solution": "null", "level": "三年级", "question": "一块长 10 米, 宽 8 米的长方形铁板上切下一个最大的正方形, 这个正方形的面积是 ( ) 平方米。\n\n", "options": "A. 32\nB. 100\nC. 64", "subject": "度量几何学", "analysis": "解C\n\n【分析】从长方形铁板上切下最大的正方形, 则这个正方形的边长等于长方形的宽。根据正方形的面积 $=$ 边长 $\\times$ 边长解答。\n\n【详解】 $8 \\times 8=64$ (平方米)\n\n这个正方形的面积是 64 平方米。\n\n故答案为: C\n\n【点睛】解决本题的关键是明确最大正方形的边长等于长方形的宽。"} {"id": "8763", "image": [], "answer": "A", "solution": "null", "level": "三年级", "question": "一个正方形的边长扩大到原来的 4 倍, 周长扩大到原来的 ( ) 倍。", "options": "A. 4\nB. 8\nC. 16", "subject": "度量几何学", "analysis": "A\n\n【分析】根据题意可知, 把一个正方形的边长扩大到原来的 4 倍, 假设原正方形的边长是 1 厘米, 那么现在的边长是 $(1 \\times 4)$ 厘米, 根据周长的计算公式及积的变化规律可知, 一个正方形的边长扩大到原来的 4 倍, 周长扩大到原来的 4 倍。\n\n【详解】根据分析可知,\n\n一个正方形的边长扩大到原来的 4 倍, 周长扩大到原来的 4 倍。故答案为: A\n\n【点睛】熟记: 正方形的周长 $=$ 边长 $\\times 4$, 正确理解积的变化规律, 是解答此题的关键。"} {"id": "9240", "image": [], "answer": "A", "solution": "null", "level": "三年级", "question": "用面积为 1 平方厘米的若干个小正方形, 拼成如下的图形, 周长最长的是 ( ), 面积最大的是\n\n(1) | | | $\\square$ |\n| :--- | :--- | :--- |\n| | | $\\square$ | $\\square \\square^{3}$", "options": "A. (3)(1)\nB. (1)(2)\nC. (2)(3)", "subject": "度量几何学", "analysis": "解A\n\n【分析】分别计算各图形的周长和面积, 比较即可完成填空。\n\n【详解】 (1)中图形的周长:\n\n$(3+2) \\times 2$\n\n$=5 \\times 2$\n\n$=10($ 厘米 $)$\n\n面积: $3 \\times 2=6$ (平方厘米)\n\n(2)中图形的周长可以看作长为 3 厘米、宽为 2 厘米的长方形的周长:\n\n$(3+2) \\times 2$\n\n$=5 \\times 2$\n\n$=10$ (厘米)\n\n面积: $1 \\times 4=4$ (平方厘米)\n\n(3)中图形的周长可以看作长为 4 厘米、宽为 2 厘米的长方形的周长:\n\n$(4+2) \\times 2$\n\n$=6 \\times 2$\n\n$=12($ 厘米 $)$\n\n$5 \\times 1=5$ (平方厘米)\n\n$10=10<12$\n\n$4<5<6$\n\n周长最长的是(3),面积最大的是(1)。\n\n故答案为: A\n\n【点睛】本题主要考查图形的拼组, 关键利用转化思想计算图形的周长和面积。"} {"id": "9251", "image": [], "answer": "C", "solution": "null", "level": "三年级", "question": "学校的占地面积约为 $3000(\\quad)$ 。", "options": "A. 平方分米\nB. 平方厘米\nC. 平方米", "subject": "度量几何学", "analysis": "解$\\mathrm{C}$\n\n【分析】根据生活经验, 以及对面积单位和数据大小的认识, 可知计量学校的占地面积应用“平方米”作单位; 据此解答。\n\n【详解】据分析可知:\n\n学校的占地面积约为 3000 平方米。\n\n故答案为: C\n\n【点睛】此题考查根据情景选择合适的计量单位, 要注意联系生活实际、计量单位和数据的大小, 灵活的选择。"} {"id": "9259", "image": [], "answer": "A", "solution": "null", "level": "三年级", "question": "两个一样大小的长方形, 长都是 $10 \\mathrm{~cm}$, 宽都是 $5 \\mathrm{~cm}$, 把这两个长方形拼成一个正方形。下面说法正确的是 $(\\quad)$ 。", "options": "A. 这个正方形的面积是 $100 \\mathrm{~cm}^{2}$\n\nB.这个正方形的周长是 $30 \\mathrm{~cm}$\n\nC. 这个正方形的边长是 $15 \\mathrm{~cm}$", "subject": "度量几何学", "analysis": "解A\n\n【分析】将这两个长方形拼成一个正方形, 正方形的边长等于长方形的长, 即 $10 \\mathrm{~cm}$ 。根据正方形的周长 $=$ 边长 $\\times 4$, 正方形的面积 $=$ 边长 $\\times$ 边长解答。\n\n【详解】A. $10 \\times 10=100\\left(\\mathrm{~cm}^{2}\\right)$, 则这个正方形的面积是 $100 \\mathrm{~cm}^{2}$, 说法正确。\n\nB. $10 \\times 4=40(\\mathrm{~cm})$, 则这个正方形的周长是 $40 \\mathrm{~cm}$, 说法错误。\n\nC. 这个正方形的边长是 $10 \\mathrm{~cm}$, 说法错误。\n\n故答案为: A\n\n【点睛】 本题考查正方形周长和面积公式的应用, 关键是明确正方形的边长等于长方形的长。"} {"id": "9260", "image": [], "answer": "C", "solution": "null", "level": "三年级", "question": "边长是 $5 \\mathrm{dm}$ 的正方形面积是()。", "options": "A. $5 \\mathrm{dm}^{2}$\nB. $20 \\mathrm{dm}^{2}$\nC. $25 \\mathrm{dm}^{2}$", "subject": "度量几何学", "analysis": "解C\n\n【分析】正方形的面积 $=$ 边长 $\\times$ 边长, 当边长为 $5 \\mathrm{dm}$ 时, 依此计算出它的面积即可。\n\n【详解】 $5 \\times 5=25\\left(\\mathrm{dm}^{2}\\right)$, 即边长是 $5 \\mathrm{dm}$ 的正方形面积是 $25 \\mathrm{dm}^{2}$ 。故答案为: $\\mathrm{C}$\n\n【点睛】熟练掌握正方形的面积的计算是解答此题的关键。"} {"id": "9276", "image": ["1536.jpg"], "answer": "B", "solution": "null", "level": "三年级", "question": "如图所示, 三边长度相等的三角形被分成两部分, 其中图(1)和图(2)相比, 表述正确的是( )。\n\n", "options": "A. 图(1)比图(2)周长长\nB. 图(1)比图(2)周长短\nC. 图(1)比图(2)面积小\nD. 图(1)和图(2)周长相等", "subject": "度量几何学", "analysis": "$\\mathrm{B}$\n\n【分析】根据直观可知: (1)的面积>(2)的面积; 图(1)的周长等于一条边的长度加上曲线的长度, 图(2)的周长等于两条边的长度加上曲线的长度; 因为三角形三条边的长度相等, 据此可知图(1)的周长小于图(2)的周长。\n\n【详解】 根据分析可知:\n\n图(1)比图(2)周长短;\n\n图(1)比图(2)面积大。\n\n故答案为: B\n\n【点睛】封闭图形一周的长度叫做周长; 物体所占的平面图形的大小, 叫做它们的面积。"} {"id": "9285", "image": ["1541.jpg"], "answer": "A", "solution": "null", "level": "三年级", "question": "图中每个小方格代表 1 平方厘米, 那么阴影部分的面积是( )平方厘米。\n\n", "options": "A. 18\nB. 19\nC. 20\nD. 21", "subject": "度量几何学", "analysis": "解A\n\n【分析】数出图形中阴影部分整格的个数, 再数出不是整格的个数, 不足整格的按半格进行计算。据此解答。\n\n【详解】整格的有 12 个, 半格的有 12 个。\n\n$12+12 \\div 2$\n\n$=12+6$\n\n$=18($ 个)\n\n$1 \\times 18=18($ 平方厘米 $)$\n阴影部分面积是 18 平方厘米。\n\n故答案为: A\n\n【点睛】本题主要考查了学生用数格子的方法来求阴影部分面积的能力, 注意不足一格的都按半格计算。"} {"id": "9286", "image": ["1542.jpg"], "answer": "B", "solution": "null", "level": "三年级", "question": "将 4 个边长是 1 厘米的小正方形拼成一个长方形或正方形(如图), 下面说法正确的是( )。\n\n", "options": "A. 长方形的面积比正方形的面积大\nB. 长方形的周长比正方形的周长长\nC. 长方形和正方形的周长和面积都相等\nD. 长方形和正方形的周长和面积都不相等", "subject": "度量几何学", "analysis": "解$\\mathrm{B}$\n\n【分析】正方形的周长 $=$ 边长 $\\times 4$, 正方形的面积 $=$ 边长 $\\times$ 边长, 长方形的周长 $=($ 长 + 宽 $) \\times 2$, 长方形的面积 $=$ 长 $\\times$ 宽。将拼成的正方形和长方形的周长、面积分别计算出来, 再进行比较即可。据此解答。\n\n【详解】拼成的正方形周长: $2 \\times 4=8$ (厘米)\n\n拼成的正方形面积: $2 \\times 2=4$ (平方厘米)\n\n拼成长方形的周长:\n\n$(4+1) \\times 2$\n\n$=5 \\times 2$\n\n$=10($ 厘米 $)$\n\n拼成长方形的面积: $4 \\times 1=4$ (平方厘米)\n\n8 厘米 $<10$ 厘米, 4 平方厘米 $=4$ 平方厘米。拼成的长方形周长大于正方形的周长, 拼成的长方形面积和正方形面积相等。\n\n故答案为: B\n\n【点睛】本题考查学生对长方形、正方形周长和面积的掌握。熟练掌握长方形、正方形周长和面积计算公式是解决此题的关键。"} {"id": "9287", "image": [], "answer": "A", "solution": "null", "level": "三年级", "question": "测量教学楼的占地面积, 下面合适的单位是 ( )。", "options": "A. 平方米\nB. 平方分米\nC. 米\nD. 分米", "subject": "度量几何学", "analysis": "解A\n\n【分析】1 平方厘米大约有大拇指指甲盖那么大; 1 平方分米大约有粉笔盒正面那么大; 一平方米大约有教师讲桌那么大。结合生活实际, 选择合适的面积单位即可。据此解答。\n\n【详解】根据分析, 结合生活实际可知, 测量教学楼的占地面积, 要选择平方米为单位。故答案为: A\n\n【点睛】本题考查学生对面积单位选择的掌握。熟练掌握每个面积单位的大小是解决此题的关键。解决此题时要注意区分面积单位和长度单位。"} {"id": "9288", "image": [], "answer": "C", "solution": "null", "level": "三年级", "question": "用边长 2 分米的方砖给一个长方形的通道铺地, 沿长边铺了 20 块, 沿宽边铺了 5 块, 这个通道的面积是()平方分米。", "options": "A. 100\nB. 200\nC. 400\nD. 320", "subject": "度量几何学", "analysis": "解$\\mathrm{C}$\n\n【分析】先算出一共铺了多少块方砖, 再乘每块方砖面积, 即可算出这个长方形通道的面积。据此解答。\n\n【详解】 $20 \\times 5=100$ (块)\n$2 \\times 2=4($ 平方分米 $)$\n\n$4 \\times 100=400$ (平方分米)\n\n这个长方形通道的面积是 400 平方分米。选项 C 符合题意。\n\n故答案为: C\n\n【点睛】本题考查了正方形面积公式的灵活应用, 关键是求出方砖的总块数。"} {"id": "9292", "image": ["1543.jpg"], "answer": "A", "solution": "null", "level": "三年级", "question": "在两个同样大小的长方形纸上, 分别剪去两个同样大的小长方形 (如图所示)。剩下部分的 ( )。\n", "options": "A. 面积相等, 周长不相等\nB. 周长相等, 面积不相等\nC. 周长相等,面积也相等", "subject": "度量几何学", "analysis": "解A\n\n【分析】通过观察图形可知, 第一个图形剩下部分的周长比原来增加了两条坚线的长, 第二个图形剩下部分的周长不变,剩下部分的面积相等,据此解答即可。\n\n【详解】第一个图形的周长 $=$ 长方形的周长+两条坚线的长,第二个图形的周长 $=$ 长方形的周长,所以它们的周长相等,在两个同样大小的长方形纸上, 分别剪去两个同样大的小长方形, 剩下部分的面积相等,所以剩下部分的面积相等,周长不相等。\n\n故答案为: A\n\n【点睛】本题主要考查面积的认识与比较, 以及不规则图形的周长的巧算。"} {"id": "9302", "image": [], "answer": "C", "solution": "null", "level": "三年级", "question": "会议室地面长 12 米, 宽 6 米。用面积是 9 平方分米的正方形地砖铺地面, 需要多少块?", "options": "A. 8 块\nB. 80 块\nC. 800 块", "subject": "度量几何学", "analysis": "解$\\mathrm{C}$\n\n【分析】根据长方形的面积 $=$ 长 $\\times$ 宽, 把数据代入公式求出会议室地面的面积, 然后根据“包含”除法的意义,用除法解答。\n\n【详解】 $12 \\times 6=72$ (平方米)\n\n72 平方米 $=7200$ 平方分米\n\n$7200 \\div 9=800$ (块)\n\n故答案为: C\n\n【点睛】此题主要考查长方形面积公式的灵活运用, 关键是熟记公式, 注意: 平方米与平方分米之间的进率及换算。"} {"id": "9310", "image": [], "answer": "B", "solution": "null", "level": "三年级", "question": "一块橡皮的面积大约是()。", "options": "A. 4 厘米\nB. 4 平方厘米\nC. 4 平方分米", "subject": "度量几何学", "analysis": "解$\\mathrm{B}$\n\n【分析】常用的面积单位有平方厘米、平方分米、平方米。测量较小物体的面积用平方厘米作单位,测量稍大物体的面积用平方分米作单位, 测量较大物体的面积用平方米作单位。根据生活经验以及对面积单位和数据大小的认识, 选择合适的计量单位, 即可解答。\n\n【详解】一块橡皮的面积大约是 4 平方厘米。\n\n故答案为: B\n\n【点睛】在判断物体面积的大小时, 要联系生活实际, 选择正确的面积单位进行表述。"} {"id": "9211", "image": [], "answer": "C", "solution": "null", "level": "三年级", "question": "一个长方形, 如果宽增加 3 厘米, 面积就增加 18 平方厘米, 如果长增加 2 厘米, 面积就增加 8 平方厘米。原来长方形的面积是 $(\\quad)$ 平方厘米。", "options": "A. 26\nB. 10\nC. 24", "subject": "代数", "analysis": "解$\\mathrm{C}$\n\n【分析】根据题意可知, 18 除以 3 等于原来的长, 8 除以 2 等于原来的宽, 原来的长乘原来的宽等于原来长方形的面积。\n\n【详解】 $(18 \\div 3) \\times(8 \\div 2)$\n\n$=6 \\times 4$\n\n$=24$ (平方厘米)\n\n原来长方形的面积是 24 平方厘米。\n\n故答案为: C\n\n【点睛】先计算出原来长方形的长、宽是解答本题的关键。"} {"id": "9258", "image": ["1531.jpg"], "answer": "A", "solution": "null", "level": "三年级", "question": "用两个同样的长方形分别拼成下面的形状, 说法正确的是()。\n\n", "options": "A. 三个图形的周长都不相等,面积都相等\n\nB. (3)的面积最大\n\nC. (1)的周长最小", "subject": "组合几何学", "analysis": "解A\n\n【分析】封闭图形一周的长度是这个图形的周长; 面积是指物体所占的平面图形的大小, 由此可知,在拼图中,不管怎么拼(不重叠),面积始终不变;周长可能会改变,依此选择。\n\n【详解】A. 三个图形的周长都不相等, 面积都相等, 即原说法正确。\n\nB. 由于拼图都没有重叠, 即(1)的面积=(2)的面积=(3)的面积, 即原说法错误。\n\nC. 由于长方形的长 $>$ 宽, 即(3)的周长 $>$ (1)的周长 $>$ (2)的周长, 即原说法错误。故答案为: A\n\n【点睛】熟练掌握对周长和面积的认识是解答此题的关键。"} {"id": "9261", "image": [], "answer": "B", "solution": "null", "level": "三年级", "question": "三年级办公室地面长 9 米, 宽 3 米, 用边长是 3 分米的正方形地砖铺办公室地面, 需要 ( ) 块。", "options": "A. 3\nB. 300\nC. 900", "subject": "立体几何学", "analysis": "解$\\mathrm{B}$\n\n【分析】根据“长方形面积 $=$ 长 $\\times$ 宽、正方形面积 $=$ 边长 $\\times$ 边长”, 分别求出三年级办公室的面积和地砖的面积, 统一单位后, 用办公室的面积除以地砖的面积即可。\n\n【详解】 $9 \\times 3=27$ (平方米)\n\n$3 \\times 3=9$ (平方分米)\n\n27 平方米 $=2700$ 平方分米\n\n$2700 \\div 9=300$ (块)\n\n所以, 需要 300 块地砖。\n\n故答案为: B\n\n【点睛】熟练掌握长方形面积、正方形面积的计算公式, 是解答此题的关键。"} {"id": "9339", "image": [], "answer": "B", "solution": "null", "level": "三年级", "question": "三年级办公室地面长 9 米, 宽 3 米, 用边长是 3 分米的正方形地砖铺办公室地面, 需要 ( ) 块。", "options": "A. 3\nB. 300\nC. 900", "subject": "立体几何学", "analysis": "解$\\mathrm{B}$\n\n【分析】根据“长方形面积 $=$ 长 $\\times$ 宽、正方形面积 $=$ 边长 $\\times$ 边长”, 分别求出三年级办公室的面积和地砖的面积, 统一单位后, 用办公室的面积除以地砖的面积即可。\n\n【详解】 $9 \\times 3=27$ (平方米)\n\n$3 \\times 3=9$ (平方分米)\n\n27 平方米 $=2700$ 平方分米\n\n$2700 \\div 9=300$ (块)\n\n所以, 需要 300 块地砖。\n\n故答案为: B\n\n【点睛】熟练掌握长方形面积、正方形面积的计算公式, 是解答此题的关键。"} {"id": "12277", "image": ["2568.jpg", "2569.jpg", "2570.jpg"], "answer": "C", "solution": "null", "level": "五年级", "question": "用同样大小的正方体摆成的物体, 从正面和上面看到的图形都是\n\n\n看到的可能是( $\\quad )$ 。", "options": "A.\n\n\nB.\n\n\nC.\n\n\nD. \\$ \\qquad \\$", "subject": "立体几何学", "analysis": "答案: $\\mathrm{C}$"} {"id": "12285", "image": [], "answer": "C", "solution": "null", "level": "五年级", "question": "一个几何体, 从正面看到的形状是 $\\square$ 济 $\\square$, 从左面看到的形状是 $\\square$, 要搭成这样的几何体最少要用 ( $\\quad )$ 个小正方体。", "options": "A. 4\nB. 5\nC. 6", "subject": "立体几何学", "analysis": "答案:$\\mathrm{C}$"} {"id": "12286", "image": ["2581.jpg", "2582.jpg"], "answer": "B", "solution": "null", "level": "五年级", "question": "一堆同样大小的正方体拼搭图形, 从不同方向看到的图形分别如下图, 那么至少有 ( )块同样的正方体。\n\n\n\n上面\n\n\n\n左面", "options": "A. 5\nB. 6\nC. 7\nD. 8", "subject": "立体几何学", "analysis": "答案:$\\mathrm{B}$"} {"id": "12287", "image": ["2583.jpg", "2584.jpg", "2585.jpg", "2586.jpg", "2587.jpg", "2588.jpg"], "answer": "B", "solution": "null", "level": "五年级", "question": "图中, 搭的这组积木, 从左面看是 ( )\n\n\n\n我搭的积木从上面香是\n\n", "options": "A.\n\n\nB.\n\n\nC.\n\n\nD.\n\n", "subject": "立体几何学", "analysis": "答案: B"} {"id": "12288", "image": ["2589.jpg", "2590.jpg", "2591.jpg", "2592.jpg", "2593.jpg"], "answer": "D", "solution": "null", "level": "五年级", "question": "一个立体图形, 从上面看到的图形是\n\n图形可能是())", "options": "A.\n\n\nB.\n\n\nC.\n\n\nD.\n\n", "subject": "立体几何学", "analysis": "答案:D"} {"id": "12289", "image": [], "answer": "B", "solution": "null", "level": "五年级", "question": "一个立体图形由 5 个小正方体组成, 从它的左面看到的形状是 $\\square$, 从它的上面看到的形状是 $\\square$, 共有 $(\\quad)$ 种搭法。", "options": "A. 1\nB. 2\nC. 3", "subject": "立体几何学", "analysis": "答案: $B$"} {"id": "24004", "image": ["11845.jpg", "11846.jpg", "11847.jpg", "11848.jpg", "11849.jpg", "11850.jpg", "11851.jpg", "11852.jpg", "11853.jpg", "11854.jpg"], "answer": "C", "solution": "null", "level": "五年级", "question": "小明用 5 个小正方体搭了一个立体图形, 从正面看是\n\n", "options": "A.\n\n\nB.\n\n\nC.\n\n\n", "subject": "立体几何学", "analysis": "解C\n\n【分析】结合从正面、左面、上面观察到的形状可知, 该立体图形下层 4 个小正方体分两行, 前面 3 个, 后面 1 个, 左齐; 上层 1 个, 在下层前排中间小正方体上; 据此解答。\n\n\n\nA.\n\n\n正面\n左面\n\n\n上面\nB.\n\n\n\n\n\nC.\n\n\n故答案为: $\\mathrm{C}$\n\n【点睛】本题考查从不同方向观察立体图形, 关键是培养学生的观察能力。"} {"id": "24012", "image": ["11883.jpg", "11884.jpg", "11885.jpg", "11886.jpg", "11887.jpg", "11888.jpg", "11889.jpg", "11890.jpg", "11891.jpg", "11892.jpg", "11893.jpg", "11894.jpg"], "answer": "D", "solution": "null", "level": "五年级", "question": "从上面观察\n\n\n的形状,与下面()几何体从上面看到的形状是一样的。\n", "options": "A.\n\n\nB.\n\n\nC.\n\n\nD.\n\n\n\n", "subject": "立体几何学", "analysis": "解$\\mathrm{D}$\n\n\n\n\n\n,从上面观察有 2 行, 前边 1 行 3 个小正方形, 后边中间 1 个小正方形, 再分别确定各选项从上面观察到的形状, 找到与题干一样的即可。\n\n\n从上面观察的形状是\n\n\n\nA. 从上面观察的形状是\n\n\n\nB. 从上面观察的形状是\n\n\n\nC. 从上面观察的形状是\n\n\n\nD. 从上面观察的形状是\n\n\n\n故答案为: D\n\n【点睛】解答本题的关键是具有一定的空间想象能力。"} {"id": "24016", "image": ["11901.jpg", "11902.jpg", "11903.jpg", "11904.jpg", "11905.jpg"], "answer": "B", "solution": "null", "level": "五年级", "question": "一个几何体由若干个相同的小正方体搭成, 下面是从三个方向看到的图形, 搭这个几何体需要( )个小正方体。\n\n\n\n", "options": "A. 9\n\n\nB. 10\n\n从左面看\n\n\n\n", "subject": "立体几何学", "analysis": "解$\\mathrm{B}$\n\n【分析】这个几何体从上面看, 至少有 8 个小正方体, 从左面看, 上层有 1 个小正方体, 从正面看,\n\n上层有 2 个小正方体,摆法如图:\n\n\n\n, 据此解答。\n\n【详解】根据分析得, $8+2=10$ (个)\n\n\n\n摆成这样需要 10 个小正方体。\n\n故答案为: B\n\n【点睛】此题的解题关键是根据三视图的认识来确认几何体的形状。"} {"id": "24017", "image": ["11906.jpg", "11907.jpg", "11908.jpg", "11909.jpg", "11910.jpg", "11911.jpg", "11912.jpg", "11913.jpg"], "answer": "B", "solution": "null", "level": "五年级", "question": "小虎用同样大的正方体摆成了一个长方体。如图分别是他从前面和上面看到的图形。那么从左面看到的是()。\n\n\n\n前面\n\n\n\n上面\n", "options": "A.\n\n\nB.\n\n\nC.\n\n\nD.\n\n\n\n", "subject": "立体几何学", "analysis": "解$\\mathrm{B}$\n\n【分析】假设一个小正方形的边长为 1 , 根据从前面看到的形状, 可知该长方体的高 2 , 长为 3 , 再\n\n从上面看到形状,可知该长方体的宽为 3 , 所以这个长方体如图所示\n\n\n\n, 再从左面观察,即可得解。\n\n【详解】根据分析得,从左面看到的图形是\n\n\n\n故答案为: B\n\n【点睛】本题是考查从不同方向观察物体和几何图形, 关键是培养学生的观察能力。"} {"id": "24018", "image": ["11914.jpg", "11915.jpg", "11916.jpg", "11917.jpg"], "answer": "C", "solution": "null", "level": "五年级", "question": "一个几何体, 从上面看是\n\n\n从右面看是 $\\square$摆成这个几何体,最多需要()个小正方体。\n", "options": "A. 6\nB. 7\nC. 8\nD. 9\n\n", "subject": "立体几何学", "analysis": "解$\\mathrm{C}$\n\n【分析】根据从上面和右面看到的形状可知,该几何体下层 5 个小正方体分两行,前面 3 个,后面 2 个,左齐;上层至多 3 个小正方体,在下层前排小正方体上。\n\n【详解】如图:\n\n\n\n一个几何体,从上面看是\n\n\n\n从右面看是\n\n\n摆成这个几何体, 最多需要 8 个小正方体。\n\n故答案为: $\\mathrm{C}$\n\n【点睛】本题是考查从不同方向观察物体和几何图形, 关键是培养学生的观察能力。"} {"id": "24019", "image": ["11918.jpg", "11919.jpg", "11920.jpg", "11921.jpg", "11922.jpg", "11923.jpg", "11924.jpg", "11925.jpg", "11926.jpg", "11927.jpg", "11928.jpg", "11929.jpg", "11930.jpg", "11931.jpg"], "answer": "A", "solution": "null", "level": "五年级", "question": "根据从三个面看到的图形摆几何体, 符合要求的是()。\n\n\n\n从正面看\n\n\n\n从左面看\n\n\n\n从上面看\n\n\n\n", "options": "A.\nB.\n\n\n\nC.\n\n\n\n\n", "subject": "立体几何学", "analysis": "解A\n\n【分析】逐项分析四个选项中的几何体从正面看、从左面看、从上面看到的平面图形, 找到符合要求的几何体即可。\n\nA.\n\n\n\nB.\n\n\n\nC.\n\n\n\nD.\n\n\n\n从正面看\n\n\n从左面看\n\n\n从上面看\n\n\n\n\n故答案为: A\n\n【点睛】掌握通过三视图确定几何体的方法, 培养学生的空间想象力。"} {"id": "24424", "image": ["12476.jpg"], "answer": "C", "solution": "null", "level": "五年级", "question": "(2022 春・双牌县期末)如图,一个长方体被挖掉一小块,则体积和表面积()\n\n\n", "options": "A. 体积、表面积不变\nB. 体积减少, 表面积增加\nC. 体积减少, 表面积不变\nD. 体积不变, 表面积减少\n\n", "subject": "立体几何学", "analysis": "解 在长方体顶点上的小正方体, 原来外露 3 个面, 从顶点上挖掉这个小正方体后, 又外露与原来相同的 3 个面, 所以表面积不变, 体积减少了。\n\n故选: $C$ 。"} {"id": "24435", "image": [], "answer": "C", "solution": "null", "level": "五年级", "question": "(2022 秋・汝州市校级期中)若大正方体的体积是小正方体体积的 8 倍, 则大正方体的表面积是小正方体的()倍。\n", "options": "A. 2\nB. 16\nC. 4\nD. 8\n", "subject": "立体几何学", "analysis": "解 设小正方体的棱长为 1 , 则大正方体的棱长为 2 。\n\n$$\n\\begin{aligned}\n& 2^{3} \\div 1^{3} \\\\\n& =8 \\div 1 \\\\\n& =8 \\\\\n& 2^{2} \\times 6 \\div\\left(1^{2} \\times 6\\right) \\\\\n& =4 \\times 6 \\div 6 \\\\\n& =24 \\div 6 \\\\\n& =4\n\\end{aligned}\n$$\n\n答: 大正方体的表面积是小正方体表面积的 4 倍。\n\n故选: $C$ 。"} {"id": "24443", "image": [], "answer": "D", "solution": "null", "level": "五年级", "question": "(2021 春・三台县期末)用()个棱长为 1 厘米的小正方体木块可以搭成一个棱长为 4 厘米的大正方体。\n", "options": "A. 4\nB. 8\nC. 32\nD. 64\n", "subject": "立体几何学", "analysis": "解 根据题干分析可得, 拼组后的大正方体每条棱长上都有 4 个小正方体:\n\n$$\n4 \\times 4 \\times 4=64 \\text { (个) }\n$$\n\n所以用棱长 $1 \\mathrm{~cm}$ 的小正方体木块拼成一个棱长 $4 \\mathrm{~cm}$ 的大正方体, 至少需要 64 个小正方体。\n\n故选: D。"} {"id": "24444", "image": ["12479.jpg", "12480.jpg", "12481.jpg"], "answer": "A", "solution": "null", "level": "五年级", "question": "(2021 秋・秦淮区期末)下面是一个长方体的四个面, 另两个面的面积和是()\n5. 米\n\n\n5速米\n2.\n\n\n7.舟米\n\n\n", "options": "A. 70 平方厘米\nB. 20 平方厘米\nC. 35 平方厘米\nD. 40 平方厘米\n\n", "subject": "立体几何学", "analysis": "解 $7 \\times 5 \\times 2$\n\n$=35 \\times 2$\n\n$=70$ (平方厘米)\n\n答:另外两个面的面积和是 70 平方厘米。\n\n故选: $A$ 。"} {"id": "24445", "image": [], "answer": "C", "solution": "null", "level": "五年级", "question": "(2022 春・播州区校级期中)一个装钙奶饼干的长方体盒子高 20 厘米, 底面是 $100 \\mathrm{~cm}^{2}$ 的正方形,一张商标纸正好贴满盒子四周, 这张商标纸至少有 $\\qquad$ $\\mathrm{cm}^{2}$ 。( $\\quad$ )\n", "options": "A. 200\nB. 400\nC. 800\nD. 2000\n", "subject": "立体几何学", "analysis": "解 因为 10 的平方是 100 , 所以长方体的底面边长是 10 厘米,\n\n$10 \\times 4 \\times 20$\n\n$=40 \\times 20$\n\n$=800$ (平方厘米)\n\n答: 这张商标纸的至少 800 平方厘米。\n故选: $C$ 。"} {"id": "24446", "image": [], "answer": "D", "solution": "null", "level": "五年级", "question": "(2021 秋・江宁区期末)一个长 $26 \\mathrm{~cm}$ 、宽 $18.5 \\mathrm{~cm}$ 、厚 $0.5 \\mathrm{~cm}$ 的物体,最有可能是( )\n", "options": "A. 普通手机\nB. 橡皮\nC. 新华字典\nD. 数学书\n", "subject": "立体几何学", "analysis": "解一个长 26 厘米、宽 18.5 厘米、高 0.5 厘米的物体, 最有可能是数学书.\n\n故选: D。"} {"id": "24448", "image": [], "answer": "C", "solution": "null", "level": "五年级", "question": "(2021 秋・鼓楼区期末)一个长方体形状的玻璃容器, 从里面量长为 50 厘米, 宽为 40 厘米, 高为 45 厘米。向容器里注水, 当容器内的水体第 1 次出现正方形面时, 容器里有水()升。\n", "options": "A. 90\nB. 100\nC. 80\nD. 81\n", "subject": "立体几何学", "analysis": "解 $50 \\times 40 \\times 40$\n\n$=2000 \\times 40$\n\n$=80000$ (立方厘米)\n\n80000 立方厘米 $=80$ 升\n\n答: 容器里有水 80 升。\n\n故选: $C$ 。"} {"id": "24084", "image": ["11988.jpg", "11989.jpg", "11990.jpg"], "answer": "C", "solution": "null", "level": "五年级", "question": "有一些大小相同的正方体木块堆成 - 堆, 从上面看是图 (1) 从前面看是图 (2), 从左面看是图 (3),这堆木块由()块小正方体木块堆成的.\n\n\n\n(1)\n\n\n\n图(2)\n\n\n\n图 (3)\n", "options": "A. 6\nB. 7\nC. 8\nD. 9\n\n", "subject": "立体几何学", "analysis": "解C\n\n【详解】由图(2)可知该图形有 3 层,由图(1)可知下层有 5 个正方体木块,由图(3)可知中间一层有 2 个正方体木块, 上面一层有 1 个正方体木块;\n\n共有: $5+2+1=8$ (个)\n\n故选 C."} {"id": "24086", "image": ["11994.jpg", "11995.jpg", "11996.jpg", "11997.jpg", "11998.jpg", "11999.jpg", "12000.jpg"], "answer": "B", "solution": "null", "level": "五年级", "question": "右图 1 由 7 个立方体叠加的几何体, 从上面观察, 可画出的平面图形是 ( )\n\n\n", "options": "A.\n\n\nB.\n\n\nC.\n\n\nD.\n\n\n", "subject": "立体几何学", "analysis": "解B\n\n【详解】从上面看到的图形是一个十字形\n\n\n\n, 从正面看到的是\n\n\n据此得解."} {"id": "24087", "image": ["12001.jpg", "12002.jpg", "12003.jpg", "12004.jpg", "12005.jpg", "12006.jpg", "12007.jpg", "12008.jpg", "12009.jpg", "12010.jpg", "12011.jpg", "12012.jpg", "12013.jpg"], "answer": "C", "solution": "null", "level": "五年级", "question": "用几个小正方体搭一个立体图形, 从上面看到的形状是\n\n\n", "options": "A.\n\n\nB.\n\n\nC.\n\n\nD.\n\n\n\n\n\n", "subject": "立体几何学", "analysis": "解C\n\n【分析】从上面看是 口说明此图是两行, 一行三个, 另一行只有中间一列, 据此选择即可。\n\n【详解】A.\n\n\nB.\n\n\n, 上面看到的形状是\n\n\n\nC.\n\n\n\n上面看到的形状是\n\n\n\nD.\n\n\n\n故答案为:\n\n【点睛】观察物体要从不同方向观察, 明白他们的构成, 是几行、几列、几层, 然后能根据看到的画\n出平面图。\n\n\n\n\n\n从上面看时, 看到的是圆柱的上底面, 是圆, 和长方体的上面的一个面, 如图\n\n\n\n【分析】另外, 选项 $\\mathrm{C}$ 的图形, 则是从左面观察到的图形. $\\mathrm{AB}$"} {"id": "24088", "image": ["12014.jpg", "12015.jpg"], "answer": "A", "solution": "null", "level": "五年级", "question": "一个立体图形由 6 个正方体组成,从左面看形状是\n\n\n\n从上面看形状是\n\n\n共有()种不同的搭法。\n", "options": "A. 3\nB. 6\nC. 7\nD. 8\n\n", "subject": "立体几何学", "analysis": "解A"} {"id": "24089", "image": ["12016.jpg", "12017.jpg", "12018.jpg", "12019.jpg", "12020.jpg", "12021.jpg", "12022.jpg"], "answer": "B", "solution": "null", "level": "五年级", "question": "给左边的立体图形添加一个 $\\square$ ,使得从上面看到的形状如右图,摆法正确的是()。\n\n", "options": "A.\n\n\nB.\n\n\nC.\n\n\nD.\n\n\n\n", "subject": "立体几何学", "analysis": "解$\\mathrm{B}$\n\n【分析】分析题意, 首先画出原立体图形从上面看得到的形状;\n\n\n\n,则所添小正方体在立体图形的前面, 据此解答即可。\n\n【详解】原立体图形从上面看得到的形状为\n\n\n\n要使从上面看的形状为 $\\square$, 则所添小正方体在立体图形的前面, 只有 $\\mathrm{B}$ 选项符合。\n\n故答案为: B\n\n【点睛】本题是从不同位置观察同一物体类型的题目解决本题的关键是观察立体图形的结构。"} {"id": "24090", "image": ["12023.jpg", "12024.jpg", "12025.jpg", "12026.jpg"], "answer": "C", "solution": "null", "level": "五年级", "question": "如图, 它是由 6 个同样大小的正方体摆成的几何体。将正方体(1)移走后, 从正面、上面和左面观察新几何体与从正面、上面和左面观察原几何体相比, 下列说法正确的是()。\n\n\n\n", "options": "A. 从正面看到的图形没有发生改变\n\nB. 从上面看到的图形没有发生改变\n\nC. 从左面看到的图形没有发生改变\n\nD. 从任何一面看到的图形都发生了改变\n", "subject": "立体几何学", "analysis": "解$\\mathrm{C}$\n\n【分析】对于选项 A, 将正方体(1)移走前从正面看到正方形的个数为 $1 、 2 、 1$, 移走后从正面看到正方体的个数为 1、2, 从而判断图形是否发生改变; 与上面的方法同理, 分析将正方体(1)移走前、后分别从上面、左面看到正方形的个数, 由此判断图形是否发生变化。\n\n【详解】将正方体(1)移走前, 从正面看到正方形的个数为 $1,2,1$;\n\n将正方体(1)移走后, 从正面看到正方形的个数为 1,2,发生改变, A 说法错误;\n\n将正方体(1)移走前, 从上面看到正方形的个数为 $1,3,1$;\n将正方体(1)移走后, 从上面看到正方形的个数 1,3 , 发生改变, B 说法错误;\n\n将正方体(1)移走前, 从左面看到正方形的个数为 $2,1,1$;\n\n将正方体(1)移走后, 从左面看到正方形的个数为 $2,1,1$, 没有发生改变, C 说法正确, D 说法错误。故答案为: C\n\n【点睛】本题考查了从不同方向观察立体图形,关键是明确移走正方体(1)前后看到的图形的特点。\n\n\n\n从左面看\n\n\n\n从正面看\n\n"} {"id": "24092", "image": ["12039.jpg", "12040.jpg", "12041.jpg", "12042.jpg", "12043.jpg", "12044.jpg", "12045.jpg", "12046.jpg", "12047.jpg", "12048.jpg"], "answer": "B", "solution": "null", "level": "五年级", "question": "(2022・海门市) 用 5 个相同的正方体摆成一个立体图形, 如果从上面看到的图形是\n\n\n从前面看到的图形是\n\n\n那么这个立体图形是()。\n", "options": "A.\n\n\nB.\n\n\nC.\n\n\nD.\n\n\n\n", "subject": "立体几何学", "analysis": "解B\n\n\n说明上边 1 层中间 1 个小正方体,据此确定几何体即可。\n\n【详解】用 5 个相同的正方体摆成一个立体图形,如果从上面看到的图形是\n\n\n\n到的图形是\n\n\n\n那么这个立体图形是\n\n\n故答案为: B\n\n【点睛】关键是具有一定的空间想象能力, 能根据三视图确认几何体形状。"} {"id": "24103", "image": ["12086.jpg", "12087.jpg", "12088.jpg", "12089.jpg", "12090.jpg", "12091.jpg", "12092.jpg", "12093.jpg", "12094.jpg", "12095.jpg", "12096.jpg", "12097.jpg"], "answer": "B", "solution": "null", "level": "五年级", "question": "(2022・淮安区)下图中是贝贝从三个方向看到的图形, 第()个几何体符合要求。\n\n\n从正面看\n\n\n从上面看\n\n\n从左面看\n", "options": "A.\n\n\nB.\n\n\nC.\n\n\nD.\n\n\n\n\n", "subject": "立体几何学", "analysis": "解$\\mathrm{B}$\n\n【分析】从上面看有 3 个小正方形, 可以确定底层有 3 个小正方体和摆放位置; 从正面和左面看到的形状相同, 说明第 2 层只有 1 个小正方体, 摆在靠前的位置, 据此确定几何体的形状。\n\n【详解】\n\n\n\n从正面看\n\n\n\n从上面看\n\n\n\n从左面看\n\n可以确定, 这个几何体的形状是\n\n\n\n故答案为: B\n\n【点睛】根据三视图可以确定几何体的形状, 关键是具有一定的空间想象能力。"} {"id": "24109", "image": ["12116.jpg", "12117.jpg", "12118.jpg", "12119.jpg", "12120.jpg"], "answer": "A", "solution": "null", "level": "五年级", "question": "(2022・海门市)小东用积木搭了一个几何体,从上面看到的形状是\n1\n\n(数字表示在这个位置上用的小正方体的个数),这个几何体从正面看是( )。\n", "options": "A.\n\n\nB.\n\n\nC.\n\n\nD.\n\n\n\n", "subject": "立体几何学", "analysis": "解A\n\n【分析】从正面看的图形有两列, 左边一列只有一个小正方形, 靠下, 右边一列有 3 个小正方形; 从上面看的图形有两列, 左边一列只有一个小正方形, 靠下, 右边一列有 2 个小正方形; 从左边看的图形有两列, 左边一列有 3 个小正方形, 右边一列有 2 个小正方形, 靠下。据此解答。\n\n【详解】据分析可知, 从正面看的图形有两列, 左边一列只有一个小正方形, 靠下, 右边一列有 3 个小正方形, 只有 $\\mathrm{A}$ 符合。这个几何体从正面看是\n\n\n故答案为: $\\mathrm{A}$\n\n【点睛】本题是考查从不同方向观察物体和几何图形, 关键是培养学生的观察能力。"} {"id": "24110", "image": ["12121.jpg", "12122.jpg", "12123.jpg"], "answer": "A", "solution": "null", "level": "五年级", "question": "(2022・南京)一个由小正方体摆成的几何体, 从左面和正面看到的形状如图, 这个几何体最少由()个小正方体摆成。\n\n\n\n从左面看\n\n\n\n从正面看\n", "options": "A. 4\nB. 5\nC. 6\nD. 7\n\n", "subject": "立体几何学", "analysis": "解A\n\n【分析】由图可知,从左面看左边一列至少有 1 个小正方体,中间一列至少有 2 个小正方体,右边一列至少有 1 个小正方体; 从正面看左边一列最高层数为 1 层, 右边一列最高层数为 2 层; 找出符合条件的几何体, 据此解答。\n\n【详解】\n\n\n(摆法不唯一)\n\n所以, 这个几何体最少由 4 个小正方体摆成。\n\n故答案为: A\n\n【点睛】本题主要考查根据从不同方向看到的平面图形确定几何体, 注意两个小正方体有一条棱重合的特殊情况是解答题目的关键。"} {"id": "24111", "image": ["12124.jpg", "12125.jpg", "12126.jpg", "12127.jpg", "12128.jpg", "12129.jpg", "12130.jpg", "12131.jpg"], "answer": "B", "solution": "null", "level": "五年级", "question": "(2022 ・淮安区)小虎用同样大的正方体摆成了一个长方体。如图分别是他从前面和上面看到的图形。那么从左面看到的是()。\n\n\n\n前面\n\n\n\n上面\n\n", "options": "A.\n\n\nB.\n\n\nC.\n\n\nD.\n\n\n\n", "subject": "立体几何学", "analysis": "解B\n\n【分析】假设一个小正方形的边长为 1 , 根据从前面看到的形状, 可知该长方体的高 2 , 长为 3 , 再\n\n从上面看到形状, 可知该长方体的宽为 3 , 所以这个长方体如图所示\n\n\n, 再从左面观察,即可得解。\n\n【详解】根据分析得, 从左面看到的图形是\n\n\n\n故答案为: B\n\n【点睛】本题是考查从不同方向观察物体和几何图形, 关键是培养学生的观察能力。"} {"id": "24112", "image": ["12132.jpg", "12133.jpg", "12134.jpg", "12135.jpg", "12136.jpg", "12137.jpg", "12138.jpg", "12139.jpg", "12140.jpg", "12141.jpg", "12142.jpg", "12143.jpg", "12144.jpg", "12145.jpg"], "answer": "B", "solution": "null", "level": "五年级", "question": "(2022・海门市)下面是从不同方向观察某个几何体时看到的图形,这个几何体是()。\n\n\n\n从正面看\n\n\n\n从上面看\n", "options": "A.\n\n\nB.\n\n\nC.\n\n\nD.\n\n\n\n", "subject": "立体几何学", "analysis": "解$\\mathrm{B}$\n\n【分析】根据观察物体的方法, 逐项画出三视图, 得出说法正确的一项即可解答问题。\n\n【详解】A. 从正面看到的图形是\n\n\n, 从上面看到的图形是:\n\n\n\nB. 从正面看到的图形是\n\n\n从上面看到的图形是:\n\n\n\nC. 从正面看到的图形是\n\n\n\n从上面看到的图形是:\n\n\n\nD. 从正面看到的图形是\n\n\n从上面看到的图形是:\n\n\n故答案为: B\n\n【点睛】本题考查作简单图形的三视图, 能正确辨认从正面、上面观察到的简单几何体的平面图形。"} {"id": "24113", "image": ["12146.jpg", "12147.jpg", "12148.jpg", "12149.jpg", "12150.jpg", "12151.jpg", "12152.jpg", "12153.jpg", "12154.jpg", "12155.jpg"], "answer": "A", "solution": "null", "level": "五年级", "question": "(2022・南京)一个用小正方体积木拼搭成的几何体, 下图是从三个不同的方向看到的图形。现在从正面看这个几何体,如果用数字表示这个位置上小正方体积木的个数,那么正确的是()。\n\n\n", "options": "A.\n\n\nB.\n\n\nC.\n\n\n$\\mathrm{D}$.\n\n\n\n\n\n\n\n从上面看\n\n", "subject": "立体几何学", "analysis": "解A\n\n\n\n【分析】根据从不同方向观察几何体的方法, 利用三视图, 从上面看到的图形是\n\n从上面看 , 说明第一层至少有 5 个小正方体, 结合从正面和左面看到的图形, 可确定第二层只有 1 个小正方体,总共\n\n有 $(5+1)$ 个小正方体, 摆法如图:\n\n\n\n【详解】根据分析得, $5+1=6$ (个)\n\n即这个几何体由 6 个小正方体组合而成。\n\n从正面看这个几何体, 用数字表示的图形应该是\n\n\n\n故答案为: A\n\n【点睛】此题的解题关键是利用三视图来确定几何体的形状。"} {"id": "24114", "image": ["12156.jpg", "12157.jpg", "12158.jpg", "12159.jpg", "12160.jpg", "12161.jpg", "12162.jpg"], "answer": "B", "solution": "null", "level": "五年级", "question": "(2022・南京\n\n\n是由 5 个小正方体搭成的几何体,小明从右面看到的图形是()。\n", "options": "A.\n\n\nB.\n\n\nC.\n\n\nD.\n\n\n", "subject": "立体几何学", "analysis": "解B\n\n【分析】由图可知, 从右面可以看到 2 列, 左边一列可以看到 2 个小正方形, 右边一列可以看到 1 个小正方形, 左右两列的小正方形底部对齐, 据此解答。\n\n【详解】分析可知,\n\n\n\n从右面看到的图形为\n\n\n\n故答案为: B\n\n【点睛】根据立体图形分析从右面看到小正方形的列数和每列小正方形的个数是解答题目的关键。"} {"id": "24116", "image": ["12167.jpg", "12168.jpg", "12169.jpg", "12170.jpg", "12171.jpg", "12172.jpg", "12173.jpg"], "answer": "A", "solution": "null", "level": "五年级", "question": "用 5 个同样的小正方体摆出一个几何体, 使它在左面看到的图形是 $\\square$, 摆法正确的是 ( )。\n", "options": "A.\n\n\nB.\n\n\nC.\n\n\nD.\n\n\n\n", "subject": "立体几何学", "analysis": "解A\n\n【分析】根据从不同方向观察几何体的方法, 逐项分析四个选项, 利用画出的三视图判断哪个几何体符合条件即可。\n\nB.\n\n\n\nC.\n\n\n\nD.\n\n\n\n故答案为: A\n\n【点睛】本题是考查通过三视图确认几何体,能正确辨认从正面、上面、左面(或右面)观察到的简单几何体的平面图形。"} {"id": "24127", "image": ["12237.jpg", "12238.jpg", "12239.jpg", "12240.jpg", "12241.jpg"], "answer": "B", "solution": "null", "level": "五年级", "question": "一个几何体从上面看到的图形是\n\n, 图形上的数字表示在这个位置上所用的小正方体的个数,这个几何体从正面看是(),从左面看是()。\n\n", "options": "A. (1)(3)\nB. (2)(4)\nC. (3)(4)\nD. (2)(3)\n\n", "subject": "立体几何学", "analysis": "解B\n\n【分析】一个几何体从上面看到的图形是\n\n\n\n可以确定底层小正方体的摆法, 以及层数,\n\n\n个小正方体; 从左面看有 3 层, 第 1 层和第 2 层分别 2 个小正方体, 第 3 层靠右 1 个小正方体, 据此分析。\n\n【详解】这个几何体从正面看是\n\n\n\n从左面看是\n\n\n\n故答案为: B\n\n【点睛】关键是确定几何体的形状, 具有一定的空间想象能力。"} {"id": "24133", "image": ["12263.jpg", "12264.jpg"], "answer": "B", "solution": "null", "level": "五年级", "question": "一个用同样的小正方体摆成的几何体,从正面、左面看到的图形都是\n\n\n摆这个几何体至少需要()个同样的小正方体。\n", "options": "A. 5\nB. 6\nC. 7\nD. 8\n\n", "subject": "立体几何学", "analysis": "解$\\mathrm{B}$\n\n【分析】根据从正面、左面看到的图形可知, 用小正方体摆出这个几何体, 确定至少用到小正方体的个数。\n【详解】结合从正面、左面看到的图形, 可以得出下面的几何体:\n\n\n\n摆这个几何体至少需要 6 个同样的小正方体。\n\n故答案为: B\n\n【点睛】本题考查根据部分视图还原立体图形的能力, 培养学生的空间想象力。"} {"id": "24135", "image": ["12275.jpg", "12276.jpg", "12277.jpg", "12278.jpg", "12279.jpg", "12280.jpg"], "answer": "B", "solution": "null", "level": "五年级", "question": "下图是一个由 5 个完全相同的小正方体组成的立体图形,从上面看到的是()。\n\n\n", "options": "A.\n\n\nB.\n\n\nC.\n\n\nD.\n\n\n\n", "subject": "立体几何学", "analysis": "解$\\mathrm{B}$\n\n【分析】从上面观察这个立体图形, 可以看到 2 层 4 个小正方形, 上层有 3 个, 下层有 1 个, 且居左;据此找到从上面看到的图形。\n【详解】如图:\n\n\n故答案为: B\n\n【点睛】本题考查从上面观察立体图形得到相应的平面图形。"} {"id": "24136", "image": ["12281.jpg", "12282.jpg", "12283.jpg", "12284.jpg", "12285.jpg", "12286.jpg", "12287.jpg", "12288.jpg", "12289.jpg", "12290.jpg", "12291.jpg", "12292.jpg"], "answer": "D", "solution": "null", "level": "五年级", "question": "下图是从三个不同方向观察小正方体拼成的几何体得到的图形,()摆法符合要求。\n\n\\section*{| | | |\n| :--- | :--- | :--- |\n\n\n\n上面 左面 前面\n", "options": "A.\n\n\nB.\n\n\nC.\n\n\nD.\n\n\n\n", "subject": "立体几何学", "analysis": "解D\n\n【分析】根据从上面、左面、前面看到的平面图形可知, 这个几何体由 4 个小正方体拼成, 2 层 1 行,上层有 1 个小正方体且居左, 下层有 3 个小正方体; 据此得出符合要求的几何体。\n\n【详解】如图:\n上面\n左面\n前面\nA.\n\n\n\n\n\nв.\n\nC.\n\n\n\n\n\n\n故答案为: D\n\n【点睛】本题考查根据从不同方向观察到几何体得到的平面图形, 确定几何体。"} {"id": "24137", "image": ["12293.jpg", "12294.jpg", "12295.jpg", "12296.jpg", "12297.jpg", "12298.jpg", "12299.jpg", "12300.jpg", "12301.jpg", "12302.jpg", "12303.jpg", "12304.jpg", "12305.jpg"], "answer": "B", "solution": "null", "level": "五年级", "question": "一个立体图形从上面看是图形 $\\square$, 从正面看是图形 $\\square$, 这个立体图形是()。\n", "options": "A.\n\n\nB.\n\n\nC.\n\n\nD.\n\n\n\n\n\n", "subject": "立体几何学", "analysis": "解$\\mathrm{B}$\n\n【分析】从上面看是 2 层 4 个小正方形, 上层 3 个, 下层 1 个且居左; 从正面看是 2 层 4 个小正方形,上层 1 个且居左, 下层 3 个; 可以得出这个立体图形的下层有 4 个小正方体, 上层有 1 个小正方体且居左。\n\n【详解】如图:\n正面\n上面\nA.\n\nB.\n\n\n\n\n\n\nC.\n\n\n\n\n\n\nD.\n\n\n故答案为: B\n\n【点睛】本题考查根据部分视图还原立体图形的能力,培养学生的空间想象力。"} {"id": "24138", "image": ["12306.jpg", "12307.jpg", "12308.jpg", "12309.jpg", "12310.jpg", "12311.jpg", "12312.jpg", "12313.jpg"], "answer": "B", "solution": "null", "level": "五年级", "question": "一个由小正方体搭成的立体图形, 从上面看到图形的是\n\n(图中数据表示在这个位置上小正方体的个数)。这个立体图形从右面看到的是()。\n", "options": "A.\n\n\nB.\n\n\nC.\n\n\nD.\n\n\n", "subject": "立体几何学", "analysis": "解$\\mathrm{B}$\n\n\n【分析】这个立体图形从上面看到的图形是\n\n,据此可确定这个立体图形的摆法如图:\n\n\n\n,根据三视图的认识,从右面观察这个立体图形,即可求出看到的图形。\n\n【详解】根据分析得,\n\n\n\n从右面看到的图形是\n\n\n\n故答案为: B\n\n【点睛】本题是考查通过三视图确定几何体的形状,能正确辨认从不同方向观察到的简单几何体的平面图形。"} {"id": "24140", "image": ["12316.jpg", "12317.jpg", "12318.jpg", "12319.jpg", "12320.jpg", "12321.jpg", "12322.jpg", "12323.jpg", "12324.jpg"], "answer": "B", "solution": "null", "level": "五年级", "question": "从正面看是 $\\square$ 的几何体是 ( ) 。\n", "options": "A.\n\n\nB.\n\n\nC.\n\n\nD.\n\n\n\n", "subject": "立体几何学", "analysis": "解$\\mathrm{B}$\n\n【分析】根据从正面观察几何体的方法, 逐项分析四个选项, 利用画出的三视图判断哪个几何体符合条件即可。\n\n【详解】A.\n\n\n从正面看的图形是\n\n\n\nB.\n\n\n\nC.\n\n\n\nD.\n\n\n\n故答案为: B\n\n【点睛】本题考查观察物体, 解答本题的关键是掌握根据物体三视图确定物体形状的方法。"} {"id": "24160", "image": ["12383.jpg", "12384.jpg", "12385.jpg", "12386.jpg", "12387.jpg", "12388.jpg", "12389.jpg", "12390.jpg", "12391.jpg"], "answer": "D", "solution": "null", "level": "五年级", "question": "一个立体图形, 从上面看到的图形是\n\n\n\n从左面看到的图形是\n\n\n这个立体图形可能是()。\n", "options": "A.\n\n\nB.\n\n\nC.\n\n\nD.\n\n\n", "subject": "立体几何学", "analysis": "解$\\mathrm{D}$\n\n【分析】根据从上面和左面观察到的形状可知, 该几何体下层 4 个小正方体, 分两行, 后排 3 个, 前排 1 个,右齐;上层至少 1 个,在后排右面的小正方体上。\n\n【详解】一个立体图形, 从上面看到的图形是\n\n\n\n从左面看到的图形是\n\n\n这个立\n\n体图形可能是\n\n\n\n故答案为:\n【点睛】本题是考查从不同方向观察物体和几何图形, 关键是培养学生的观察能力。"} {"id": "24161", "image": ["12392.jpg", "12393.jpg", "12394.jpg", "12395.jpg", "12396.jpg", "12397.jpg", "12398.jpg", "12399.jpg", "12400.jpg", "12401.jpg", "12402.jpg", "12403.jpg", "12404.jpg", "12405.jpg", "12406.jpg", "12407.jpg"], "answer": "B", "solution": "null", "level": "五年级", "question": "下面各立体图形中, 从正面看是\n\n,这个立体图形是()。\n", "options": "A.\n\n\nB.\n\n\nC.\n\n\nD.\n\n\n\n", "subject": "立体几何学", "analysis": "解$\\mathrm{B}$\n\n【分析】分别将每个图形从正面和上面看到的平面图画出来, 再进行选择即可。\n\n【详解】A.\n\n\n从正面看\n\n,从上面看\n\n\nB.\n\n\n从正面看\n\n\n从上面看\n\n\nC.\n\n\n从正面看\n\n\n\n\nD.\n\n\n\n\n\n从正面看\n\n, 从上面看\n\n\n\n故答案为: B\n\n【点睛】本题考查了观察物体的知识点, 关键是将从正面和上面看到的平面图想象出来或画出来。"} {"id": "24162", "image": ["12408.jpg"], "answer": "C", "solution": "null", "level": "五年级", "question": "一堆正方体摆放在一起, 从正面看、左面看如图, 这堆小正方体最多有()块。\n\n", "options": "A. 6\nB. 7\nC. 8\nD. 9\n", "subject": "立体几何学", "analysis": "解C\n\n【分析】根据图形的正面视图和左面视图可得, 此立方体图形分为前后两排, 上中下三层, 前面一排最多有三个小正方体, 后边一排最多有五个小正方体, 所以一共最多有八个小正方体, 从而求解。\n\n【详解】根据从正面和左面看到的图形, 可以知道这个立方体的前排第一层只有三个, 二三层没有,从左面和正面看到的图形, 可以知道这个立方体的后排第一层可以有三个也可以有一个, 第二层有一个, 第三层有一个, 所以最多一共有 8 个小正方体。\n\n故答案为: C\n\n【点睛】考查了从不同方向观察物体和几何体, 利用不同方向看到的平面图形来还原立体图形, 考查了学生的空间观念。"} {"id": "24163", "image": ["12409.jpg", "12410.jpg", "12411.jpg", "12412.jpg", "12413.jpg", "12414.jpg", "12415.jpg"], "answer": "A", "solution": "null", "level": "五年级", "question": "工厂仓库里有一堆存放货物的正方体纸箱, 从上面看到的形状是\n\n\n这个位置上所用正方体纸箱的个数。这堆纸箱从正面看是( )。\n", "options": "A.\n\n\nB.\n\n\nC.\n\n\nD.\n\n\n\n", "subject": "立体几何学", "analysis": "解A\n\n【分析】根据从上面看到的形状是\n\n么从正面看, 这个图形分为三层, 最下层有 3 个小正方形, 中间层在 2 个小正方形, 靠左对齐, 最上面居中只有 1 个小正方形, 据此解答。\n【详解】根据分析得, 这堆纸箱从正面看的图形是\n\n\n\n故答案为: $\\mathrm{A}$\n\n【点睛】此题主要考查通过三视图及正方体纸箱的个数确定这堆纸箱的摆法,再通过观察正面的图形找出正确的答案。"} {"id": "24165", "image": ["12421.jpg", "12422.jpg", "12423.jpg"], "answer": "C", "solution": "null", "level": "五年级", "question": "一个立体图形, 从上面看是\n\n\n\n从左面看是\n\n\n多能用()个小正方体。\n", "options": "A. 4\nB. 5\nC. 7\nD. 8\n", "subject": "立体几何学", "analysis": "解C\n\n【分析】根据从上面和左面看到的形状可知, 该几何体下层有 4 个小正方体, 分为 2 排, 前排 3 个,后排 1 个, 上层最多 3 个, 分别放在下层前排小正方体上, 据此解答即可。\n\n【详解】一个立体图形, 从上面看是\n\n要搭成这样的一个立体图形, 最多能用 7 个小正方体;\n\n故答案为:\n\n【点睛】本题考查了学生的空间思维能力, 也可以利用实物摆一摆。"} {"id": "24450", "image": ["12482.jpg", "12483.jpg", "12484.jpg", "12485.jpg", "12486.jpg", "12487.jpg", "12488.jpg", "12489.jpg", "12490.jpg", "12491.jpg"], "answer": "A", "solution": "null", "level": "五年级", "question": "下面图形中,()是正方体的表面展开图。\n", "options": "A.\n\n\nB.\n\n\nC.\n\n\n\n", "subject": "立体几何学", "analysis": "解$\\mathrm{A}$\n\n【分析】正方体的展开图有 11 种情况。\n\n(1)1-4-1 形: 上面有 1 个正方形, 中间有 4 个正方形, 下面有 1 个正方形, 如下图所示:\n\n\n(2) 2-3-1 形: 上面有 2 个正方形, 中间有 3 个正方形, 下面有 1 个正方形, 如下图所示:\n\n\n(3) 2-2-2 形: 上、中、下三行各有 2 个正方形, 如下图所示:\n\n\n\n(4)3-3 形:仅有 2 行, 每行有 3 个正方形, 如下图所示:\n\n\n\n根据上面正方形的展开图选择即可。\n\n\n\n是正方体展开图的“1-4-1”型, 是正方体的一种展开图;\n\nB.\n\n\n折成正方体时有两个面重叠, 不是正方体展开图;\n\nC.\n\n\n无法沿棱折起来,不是正方体展开图。\n\n故答案为: A\n\n【点睛】正方形出现“田”字型、“凹”字型排列, 都不能折成正方体。"} {"id": "24461", "image": [], "answer": "C", "solution": "null", "level": "五年级", "question": "一个长方体(不含正方体)最多有()条棱长度相等。\n", "options": "A. 4\nB. 6\nC. 8\n", "subject": "立体几何学", "analysis": "解$\\mathrm{C}$\n\n【分析】长方体(不含正方体)一般是由 6 个长方形(特征情况有两个相对的面是正方形)围成的立\n体图形。因为相对的棱长度相等, 所以长方体的 4 条长、 4 条宽、 4 条高分别相等; 特殊情况下, 若有两个相对的面是正方形, 比如左右面是正方形, 此时宽等高, 即有 8 条棱长度相等。\n\n【详解】有两个相对的面是正方形的长方体(不含正方体)中, 因为不含正方体, 所以不可能长、宽、高都相等, 最多是长、宽、高中的两者相等, $4 \\times 2=8$ (条), 所以最多有 8 条棱长度相等。\n\n故答案为: C\n\n【点睛】明确长方体中相对的棱长度相等是解决此题的关键。"} {"id": "24470", "image": [], "answer": "C", "solution": "null", "level": "五年级", "question": "一个长方体的长不变, 宽扩大到原来的 2 倍, 高缩小到原来的 $\\frac{1}{2}$, 体积()。\n", "options": "A. 增大\nB. 缩小\nC. 不变\n", "subject": "立体几何学", "analysis": "解$\\mathrm{C}$\n\n【分析】长方体的体积 $=$ 长 $\\times$ 宽 $\\times$ 高, 设长方体的长是 $\\mathrm{a}$, 宽是 $\\mathrm{b}$, 高是 $\\mathrm{h}$, 根据长方体的体积公式分别用含 $\\mathrm{a}, \\mathrm{b}, \\mathrm{h}$ 的式子表示长方体原来的体积和现在的体积, 并比较大小。\n\n【详解】设长方体的长是 $\\mathrm{a}$, 宽是 $\\mathrm{b}$, 高是 $\\mathrm{h}$ 。\n\n原来长方体的体积: $\\mathrm{abh}$\n\n现在长方体的体积: $\\mathrm{a} \\times(\\mathrm{b} \\times 2) \\times(\\mathrm{h} \\div 2)=\\mathrm{abh} \\times 2 \\div 2=\\mathrm{abh}$\n\n所以一个长方体的长不变, 宽扩大到原来的 2 倍, 高缩小到原来的 $\\frac{1}{2}$, 体积不变。\n\n故答案为: C\n\n【点睛】明确长方体的体积计算公式是解决此题的关键。"} {"id": "24471", "image": ["12498.jpg"], "answer": "C", "solution": "null", "level": "五年级", "question": "用 3 个棱长均为 $1 \\mathrm{~cm}$ 的正方体拼成一个长方体, 表面积减少()。\n", "options": "A. $2 \\mathrm{~cm}^{2}$\nB. $3 \\mathrm{~cm}^{2}$\nC. $4 \\mathrm{~cm}^{2}$\n", "subject": "立体几何学", "analysis": "解C\n\n【分析】用 3 个棱长均为 $1 \\mathrm{~cm}$ 的正方体拼成一个长方体, 长方体的表面积比 3 个正方体的表面积减少了 4 个正方形的面积; 根据正方形的面积=边长×边长, 求出一个面的面积, 再乘 4 即可求出减少的表面积。\n\n【详解】如图:\n\n\n\n$1 \\times 1 \\times 4=4\\left(\\mathrm{~cm}^{2}\\right)$\n\n表面积减少 $4 \\mathrm{~cm}^{2}$ 。\n\n故答案为: C\n\n【点睛】本题考查立体图形的拼接, 明确 3 个正方体拼成一个长方体, 表面积比原来减少正方体的 4 个面的面积。"} {"id": "24472", "image": [], "answer": "C", "solution": "null", "level": "五年级", "question": "一个长方体水箱容积是 100 升, 这个水箱底面是一个边长为 5 分米的正方形。水箱的高是 ( )。\n", "options": "A. 20 分米\nB. 10 分米\nC. 4 分米\n", "subject": "立体几何学", "analysis": "解C\n\n【分析】先根据“正方形的面积=边长 $\\times$ 边长”求出水箱的底面积; 再根据“长方体的体积:底面积=高”求出水箱的高。\n【详解】100 升 $=100$ 立方分米\n\n$100 \\div(5 \\times 5)$\n\n$=100 \\div 25$\n\n$=4$ (分米)\n\n所以水箱的高是 4 分米。\n\n故答案为: $\\mathrm{C}$\n\n【点睛】已知长方体的体积、底面积和高这三个量中的任意两个量, 都可以求出第三个量。"} {"id": "12281", "image": ["2572.jpg", "2573.jpg", "2574.jpg", "2575.jpg", "2576.jpg", "2577.jpg", "2578.jpg"], "answer": "D", "solution": "null", "level": "五年级", "question": "一个几何体从正面、上面、左面看到的图形如下。这个几何体是()\n\n\n\n从正面看\n\n\n\n从上面看\n\n\n\n从左面看", "options": "A.\n\n\nB.\n\n\nC.\n\n\nD.\n\n", "subject": "画法几何学", "analysis": "答案:D"} {"id": "24073", "image": ["11947.jpg", "11948.jpg", "11949.jpg", "11950.jpg", "11951.jpg"], "answer": "D", "solution": "null", "level": "五年级", "question": "从上面观察如图所示的物体, 看到的形状是 ( )\n\n\n", "options": "A.\n\n\nB.\n\n\nC.\n\n\nD.\n\n\n\n", "subject": "画法几何学", "analysis": "解D\n\n【详解】试题分析:观察图形可知, 从上面看到的图形是一行 3 个正方形, 据此即可选择.\n\n解:根据题干分析可得:这个物体从上面看到的图形是 $\\square \\square$.\n故选 D.\n\n点评: 此题考查了从不同方向观察问题和几何体, 锻炼了学生的空间想象力和抽象思维能力."} {"id": "24134", "image": ["12265.jpg", "12266.jpg", "12267.jpg", "12268.jpg", "12269.jpg", "12270.jpg", "12271.jpg", "12272.jpg", "12273.jpg", "12274.jpg"], "answer": "A", "solution": "null", "level": "五年级", "question": "从正面看到的图是\n\n\n的图形是()。\n", "options": "A.\n\n\nB.\n\n\nC.\n\n\nD.\n\n\n\n", "subject": "画法几何学", "analysis": "解A\n\n【分析】A. 从正面能看到 2 层 4 个小正方形, 上层 1 个且居左, 下层 3 个;\n\nB. 从正面能看到 2 层 4 个小正方形, 上层 1 个且居中, 下层 3 个;\n\nC. 从正面能看到 2 层 4 个小正方形, 上层 1 个且居右, 下层 3 个;\n\nD. 从正面能看到 1 层 3 个小正方形;\n\n据此选择符合题意的立体图形。\n\n【详解】如图:\n\n从正面看\n\nA.\n\nB.\n\nC.\n\n\n\n\nD.\n\n\n故答案为: A\n\n【点睛】本题考查从正面观察立体图形, 得出相应的平面图形。"} {"id": "23981", "image": [], "answer": "C", "solution": "null", "level": "五年级", "question": "一个数既是 6 的倍数, 又是 72 的因数, 这个数是 ( )。\n", "options": "A. 144\nB. 48\nC. 36\nD. 3\n", "subject": "算术", "analysis": "解C\n\n【分析】先求出 72 的因数, 然后再找出这些数里面 6 的倍数即可。\n\n【详解】 72 的因数有: $1 、 2 、 3 、 4 、 6 、 8 、 9 、 12 、 18 、 24 、 36 、 72$;\n\n这里面有 6 的倍数有: $6 、 12 、 18 、 24 、 36 、 72$ 。\n\n故答案为: C\n\n【点睛】解答此题应根据找一个数的因数的方法和找一个数倍数的方法进行分别列举, 进而得出结论。"} {"id": "23992", "image": [], "answer": "B", "solution": "null", "level": "五年级", "question": "既是质数又是偶数的数是 ( )。\n", "options": "A. 1\nB. 2\nC. 6\nD. 13\n", "subject": "算术", "analysis": "解B\n\n【分析】根据质数和偶数的概念, 直接解题即可。\n\n【详解】既是质数又是偶数的数是 2 。\n\n故答案为: B\n\n【点睛】本题考查了质数和偶数。一个数的因数只有 1 和本身, 那么它是质数。 2 的倍数是偶数。"} {"id": "23997", "image": [], "answer": "C", "solution": "null", "level": "五年级", "question": "小华在学习“质数和合数”时, 发现有一些合数, 它们的因数除了 1 和它本身之外, 都是质数, 于是小明把这些数称为 “理想合数”。比如: 6 的因数有 $1 、 2 、 3 、 6$, 这 4 个数中, 除 1 和 6 之外, 2 和 3 都是质数, 所以 6 是“理想合数”。那么, 下列数中不是“理想合数”的是( )。\n", "options": "A. 21\nB. 55\nC. 75\nD. 91\n", "subject": "算术", "analysis": "解C\n\n【分析】根据这些合数的因数除了 1 和它本身之外, 都是质数, 求出各选项中合数的因数, 再进行分析。\n\n【详解】A. 21 的因数有: $1 、 3 、 7 、 21$, 是理想合数;\n\nB. 55 的因数有: $1 、 5 、 11 、 55$, 是理想合数;\n\nC. 75 的因数有: $1 、 3 、 25 、 5 、 15 、 75$, 不是理想合数;\n\nD. 91 的因数有 $1 、 7 、 13 、 91$, 是理想合数。\n\n故答案为: C\n\n【点睛】关键是理解题意, 掌握质数、合数的分类标准。"} {"id": "23999", "image": [], "answer": "A", "solution": "null", "level": "五年级", "question": "如果“ $\\mathrm{a}+$ 偶数=偶数”, 那么 $\\mathrm{a}$ 一定是()。\n", "options": "A. 偶数\nB. 奇数\nC. 质数\nD. 合数\n", "subject": "算术", "analysis": "解A\n【分析】根据奇数十偶数=奇数, 偶数十偶数=偶数, 据此解答即可。\n\n【详解】由分析可知:\n\n如果“ $\\mathrm{a}+$ 偶数 $=$ 偶数”, 那么 $\\mathrm{a}$ 一定是偶数。\n\n故答案为: A\n\n【点睛】本题考查奇偶运算性质, 明确偶数十偶数=偶数是解题的关键。"} {"id": "24000", "image": [], "answer": "A", "solution": "null", "level": "五年级", "question": "要使 35 口同时是 2 和 5 的倍数, 口内应填 ( )。\n", "options": "A. 0\nB. 5\nC. 4\nD. 8\n", "subject": "算术", "analysis": "解A\n\n【分析】 2 的倍数的数的特征是:个位上是 $0 、 2 、 4 、 6 、 8$ 的数; 5 的倍数的数的特征是:个位上是 0 或 5 的数都是 5 的倍数; 由此解答。\n\n【详解】根据分析得, 要满足既是 2 的倍数又是 5 的倍数的数的特征:个位上必定是 0 , 所以 35 要同时是 2 和 5 的倍数, 口内应填 0 。\n\n故答案为: A\n\n【点睛】此题主要考查 $2 、 5$ 的倍数的特征。"} {"id": "24023", "image": [], "answer": "C", "solution": "null", "level": "五年级", "question": "在算式 $15 \\div 3=5$ 中, 3 和 5 是 15 的 ( )。\n", "options": "A. 倍数\nB. 合数\nC. 因数\n", "subject": "算术", "analysis": "解C\n\n【分析】只在自然数 (零除外) 范围内研究倍数和因数。如果 $\\mathrm{a} \\times \\mathrm{b}=\\mathrm{c}$ ( $\\mathrm{a} 、 \\mathrm{~b} 、 \\mathrm{c}$ 都是非 0 的自然数)那么 $\\mathrm{a}$ 和 $\\mathrm{b}$ 就是 $\\mathrm{c}$ 的因数, $\\mathrm{c}$ 就是 $\\mathrm{a}$ 和 $\\mathrm{b}$ 的倍数。\n\n【详解】在算式 $15 \\div 3=5$ 中, 3 和 5 是 15 的因数。\n\n故答案为: C\n\n【点睛】关键是理解因数和倍数的含义,因数和倍数两个不同的概念是相互依存的,不能单独存在。"} {"id": "24033", "image": [], "answer": "C", "solution": "null", "level": "五年级", "question": "两个质数的积一定是 ( )。\n", "options": "A. 质数\nB. 奇数\nC. 合数\n", "subject": "算术", "analysis": "解C\n\n【分析】因为质数 $\\times$ 质数 $=$ 积, 积是两个质数的倍数, 这两个质数也就是这个积的因数,所以积的因数除了 1 和它本身外还有这两个质数, 所以它们的积一定是合数。据此解答。\n\n【详解】这两个质数一定是它们积的因数, 积的因数还有 1 和它本身。所以积一定是合数。\n\n故答案为: C\n\n【点睛】掌握合数的概念是解答此题的关键。"} {"id": "24042", "image": [], "answer": "C", "solution": "null", "level": "五年级", "question": "如果 $\\mathrm{A}$ 和 B(A, B 均不为 0 )的和是偶数下面说法错误的是()。\n\n", "options": "A. 这两个数可能都是奇数\n\nB. 这两个数可能都是偶数\n\nC. 这两个数可能一个是奇数, 一个是偶数\n\n", "subject": "算术", "analysis": "解C\n\n【分析】分别举出两个或多个例子即可解答问题。\n\n【详解】假设 $\\mathrm{A} 、 \\mathrm{~B}$ 分别为奇数 3 和 $7,3+7=10$ (偶数), 所以 $\\mathrm{A}$ 正确;\n\n假设 A、B 分别为偶数 4 和 $6,4+6=10$ (偶数), 所以 $\\mathrm{B}$ 正确;\n\n假设 $\\mathrm{A} 、 \\mathrm{~B}$ 分别为 3 和 $4,3+4=7$ (奇数),所以 $\\mathrm{C}$ 错误。\n\n故答案为: C\n\n【点睛】本题主要考查两个奇数、两个偶数、一个奇数一个偶数和的奇偶性, 使用举例说明的方法即可解答问题。"} {"id": "24043", "image": [], "answer": "C", "solution": "null", "level": "五年级", "question": "如果 $\\mathrm{m}$ 是一个不为 0 的自然数, 那 $2 m-1$ 所表示的数一定是一个()。\n", "options": "A. 质数\nB. 合数\nC. 奇数\n", "subject": "算术", "analysis": "解C\n\n【分析】如果 $\\mathrm{m}$ 是一个不为 0 的自然数, $2 \\mathrm{~m}$ 表示是偶数, 根据偶数一奇数=奇数, 即可解答。\n\n【详解】如果 $\\mathrm{m}$ 是一个不为 0 的自然数, 那么 $2 \\mathrm{~m}$ 就是偶数, 1 是奇数, $2 \\mathrm{~m}-1$ 所表示的一定是一个奇数。\n\nA. 质数, 不符合题意;\n\nB. 合数, 不符合题意;\n\nC. 奇数, 符合题意。\n\n故答案选: C\n\n【点睛】本题考查奇数和偶数的含义和运算性质进行判断。"} {"id": "24044", "image": [], "answer": "C", "solution": "null", "level": "五年级", "question": "如果自然数 $a$ 是自然数 $b$ 的倍数, 那么 $a() b 。$\n", "options": "A. 一定大于\nB. 一定小于\nC. 大于或等于\n", "subject": "算术", "analysis": "解C\n【分析】根据一个数的倍数情况, 一个数的倍数个数是无限的, 最小的倍数是它本身, 没有最大的倍数, 进行分析。\n\n【详解】如果自然数 $a$ 是自然数 $b$ 的倍数, 如果是 $b$ 的最小倍数, $a=b$, 其它倍数 $a>b$, 所以 $a \\geq b$ 。故答案为: C\n\n【点睛】只在自然数 (零除外) 范围内研究倍数和因数。如果 $a \\times b=c ( a 、 b 、 c$ 都是非 0 的自然数)那么 $\\mathrm{a}$ 和 $\\mathrm{b}$ 就是 $\\mathrm{c}$ 的因数, $\\mathrm{c}$ 就是 $\\mathrm{a}$ 和 $\\mathrm{b}$ 的倍数。"} {"id": "24045", "image": [], "answer": "B", "solution": "null", "level": "五年级", "question": "$\\square a b$ 是一个三位数, 已知 $a+b=12$, 且 $\\square a b$ 是 3 的倍数, $\\square$ 里可以填 ( )。\n", "options": "A. $1,4,7$\nB. $3,6,9$\nC. $2,5,8$\n", "subject": "算术", "analysis": "解B\n\n【分析】 3 的倍数的特征: 各个数位上的数字的和是 3 的倍数, 这个数就是 3 的倍数。\n\n【详解】 $1+2=3,3+3=6,3+6=9,3+9=12$, $\\square$ 里可以填 $3,6,9$ 。\n\n故答案为: $\\mathrm{B}$\n\n【点睛】关键是掌握 3 的倍数的特征, 主要看各个数位上的数字和。"} {"id": "24046", "image": [], "answer": "C", "solution": "null", "level": "五年级", "question": "一个长方形的长是 $\\mathrm{a}$ 厘米, 宽是 $\\mathrm{b}$ 厘米( $\\mathrm{a} 、 \\mathrm{~b}$ 都是质数),这个长方形的面积是 $\\mathrm{c}$ 平方厘米, $\\mathrm{c}$不可能是()。\n", "options": "A. 合数\nB. 奇数\nC. 质数\n", "subject": "算术", "analysis": "解$\\mathrm{C}$\n\n【分析】根据长方形的面积 $=$ 长 $\\times$ 宽, 可知: $\\mathrm{c}=\\mathrm{a} \\times \\mathrm{b}$; 又因为 $\\mathrm{a} 、 \\mathrm{~b}$ 都是质数, 故 $\\mathrm{a} 、 \\mathrm{~b}$ 均不为 1 ; 由此可知 $\\mathrm{c}$ 一定不是质数。\n\n【详解】由分析可得: $\\mathrm{c}=\\mathrm{a} \\times \\mathrm{b}(\\mathrm{a} 、 \\mathrm{~b}$ 都是质数 $)$; 所以 $\\mathrm{c}$ 不可能是质数;\n\n故答案为: C\n\n【点睛】考查了长方形面积的计算公式以及质数、合数、奇数的概念和辨认。"} {"id": "24047", "image": [], "answer": "C", "solution": "null", "level": "五年级", "question": "一个合数 ( ) 因数。\n", "options": "A. 只有一个\nB. 只有两个\nC. 至少有三个\n", "subject": "算术", "analysis": "解C\n\n【分析】一个自然数, 如果除了 1 和它本身还有别的因数, 这样的数叫作合数, 据此解答即可。\n\n【详解】一个合数除了 1 和它本身外, 还有别的因数, 所以至少有三个因数;\n\n故答案为: C。\n\n【点睛】明确合数的意义是解答本题的关键。"} {"id": "24049", "image": [], "answer": "B", "solution": "null", "level": "五年级", "question": "(2022 秋・宁乡市期末)下列各数中, 既是 2 的倍数又是 5 的倍数的是 ( )\n", "options": "A. 185\nB. 2700\nC. 964\nD. 777\n", "subject": "算术", "analysis": "B既是 2 的倍数又是 5 的倍数的是 2700 。\n\n故选: $B$ 。"} {"id": "24060", "image": [], "answer": "D", "solution": "null", "level": "五年级", "question": "(2022 秋・宁乡市期末)关于合数, 下面说法不正确的是()\n\n", "options": "A. 一个合数至少有 3 个因数\n\nB. 最小的合数是 4\n\nC. 两个不同的合数的乘积一定还是合数\n\nD. 合数越大, 它的因数就越多\n\n", "subject": "算术", "analysis": "解 $A$ 选项一个合数至少有 3 个因数, 原题说法正确。\n\n$B$ 选项最小的合数是 4 , 原题说法正确。\n\n$C$ 选项两个不同的合数的乘积一定还是合数, 原题说法正确。\n\n$D$ 选项 25 是合数,因数只有 $1 、 5 、 25$ 共 3 个因数, 12 是合数,因数有 $1 、 2 、 3 、 4 、 6 、 12$ 共 6 个。原题说法错误。\n\n故选: $D$ 。"} {"id": "24066", "image": [], "answer": "B", "solution": "null", "level": "五年级", "question": "(2022 秋 - 运城期末)我们发现一些数有一个有趣的特点, 一个数所有因数(除了它本身)的和等于它本身。比如 6 的因数有 $1,2,3,6$, 这几个因数之间的关系是: $1+2+3=6$ 。像 6 这样的数叫做完全数(也叫完美数)。那么下面的数中也有这样的特点是()\n", "options": "A. 12\nB. 28\nC. 32\nD. 15\n", "subject": "算术", "analysis": "$A .12$ 的因数有: $1 、 2 、 3 、 4 、 6 、 12,1+2+3+4+6=16,16 \\neq 12$, 所以 12 不是完全数;\nB. 28 的因数有: $1 、 2 、 4 、 7 、 14 、 28,1+2+4+7+14=28$, 所以 28 是完全数;\n\n$C .32$ 的因数有: $1 、 2 、 4 、 8 、 16 、 32,1+2+4+8+16=31,31 \\neq 32$, 所以 32 不是完全数;\n\n$D .15$ 的因数有 $1 、 3 、 5 、 15,1+3+5=9,9 \\neq 15$, 所以 15 不是完全数。\n\n故选: $B$ 。"} {"id": "24067", "image": [], "answer": "A", "solution": "null", "level": "五年级", "question": "(2022 秋 ・ 浑南区期末)下列数中()既不是质数也不是合数。\n", "options": "A. 1\nB. 2\nC. 3\nD. 4\n\n", "subject": "算术", "analysis": "解根据质数与合数的意义, 1 的因数只有 1 , 所以 1 既不是质数也不是合数。\n\n故选: $A$ 。"} {"id": "24069", "image": [], "answer": "B", "solution": "null", "level": "五年级", "question": "(2022 秋・宜都市期末)小明妈妈今年的年龄既是 70 的因数, 又是 5 的倍数, 她今年是()岁。\n", "options": "A. 20\nB. 35\nC. 45\nD. 50\n", "subject": "算术", "analysis": "解70 的因数有 $1,2,5,7,10,14,35,70$,选项中都是 5 的倍数, 只有选项 $B$ 是 70 的因数。\n\n故选: $B$ 。"} {"id": "24070", "image": [], "answer": "B", "solution": "null", "level": "五年级", "question": "(2022 秋・陵城区期末)在 $\\square$ 里填一个数字, 使 “ $45 \\square$ ” 既是 3 的倍数又是 2 的倍数, 有 ( )种填法。\n", "options": "A. 1\nB. 2\nC. 3\nD. 4\n", "subject": "算术", "analysis": "解$\\square$ 里填一个数字必然是 3 的倍数, 才能保证 $45 \\square$ 是 3 的倍数, 所以可填 $0,3,6,9$,但还要满足 2 的倍数, 只有 0 和 2 符合题意, 共有 2 个。\n\n故选: $B$ 。"} {"id": "24178", "image": [], "answer": "C", "solution": "null", "level": "五年级", "question": "古希腊数学家认为:如果一个数恰好等于它的所有因数(本身除外)相加的和, 这个数就是“完全数”。例如: 6 有四个因数 $1 、 2 、 3 、 6$, 除本身 6 以外, 还有 $1 、 2 、 3$ 三个因数, $6=1+2+3$, 所以 6 就是“完全数”。下面数中是“完全数”的是()。\n", "options": "A. 16\nB. 20\nC. 28\nD. 36\n", "subject": "算术", "analysis": "解C\n\n【分析】先写出选项中各数的所有因数, 再把除它本身以外的所有因数相加, 观察结果是否等于这个数本身, 如果结果等于这个数本身它就是完全数, 据此解答。\n\n【详解】A. 16 的因数有: $1,2,4,8,16,1+2+4+8=15 \\neq 16$, 所以 16 不是完全数;\n\nB. 20 的因数有: $1,2,4,5,10,20,1+2+4+5+10=22 \\neq 20$, 所以 20 不是完全数;\n\nC. 28 的因数有: $1,2,4,7,14,28,1+2+4+7+14=28$, 所以 28 是完全数;\n\nD. 36 的因数有: $1,2,3,4,6,9,12,18,36,1+2+3+4+6+9+12+18=55 \\neq 36$, 所以 36 不是完全数。\n\n故答案为: C\n\n【点睛】准确找出各数的所有因数并掌握完全数的意义是解答题目的关键。"} {"id": "24187", "image": [], "answer": "A", "solution": "null", "level": "五年级", "question": "如果 $\\mathrm{a}$ 是 2 的倍数, 那么 $\\mathrm{a}+1$ 一定是()。\n", "options": "A. 奇数\nB. 质数\nC. 合数\nD. 偶数\n", "subject": "算术", "analysis": "解A\n\n【分析】整数中, 是 2 的倍数的数叫做偶数, 不是 2 的倍数的数叫做奇数。\n\n一个数, 如果只有 1 和它本身两个因数, 那么这样的数叫做质数; 一个数, 如果除了 1 和它本身还有\n别的因数, 那么这样的数叫做合数。\n\n根据题意, 如果 $\\mathrm{a}$ 是 2 的倍数, 说明 $\\mathrm{a}$ 是偶数, $\\mathrm{a}+1$ 即偶数十奇数 $=$ 奇数; 据此解答, 也可以举例说明。\n\n【详解】例如: $\\mathrm{a}=2,2+1=3,3$ 是奇数, 也是质数;\n\n$\\mathrm{a}=8,8+1=9,9$ 是奇数, 也是合数;\n\n所以如果 $\\mathrm{a}$ 是 2 的倍数, 那么 $\\mathrm{a}+1$ 一定是奇数。\n\n故答案为: A\n\n【点睛】掌握奇数与偶数、质数与合数的意义是解题的关键。"} {"id": "24188", "image": [], "answer": "B", "solution": "null", "level": "五年级", "question": "A、B 是两个连续的自然数, 且都不等于 0 , 它们相乘的积一定是()。\n", "options": "A. 奇数\nB. 偶数\nC. 质数\nD. 合数\n", "subject": "算术", "analysis": "解$\\mathrm{B}$\n\n【分析】可以令 A、B 等于某两个数值, 求出它们的积, 再与选项作对比即可。\n\n【详解】当 $\\mathrm{A} 、 \\mathrm{~B}$ 是两个连续的自然数, 且不为 0 时, 其中一个一定是偶数, 奇数 $\\times$ 偶数 $=$ 偶数, 例如: $\\mathrm{A}=1, \\mathrm{~B}=2,1 \\times 2=2$; 它们的积是偶数;\n\n所以 $\\mathrm{A}$ 选项不符合题意, $\\mathrm{B}$ 选项符合题意;\n\n例如: $\\mathrm{A}=2, \\mathrm{~B}=3,2 \\times 3=6$, 它们的积可能是质数, 也可能是合数;\n\n所以, C 选项和 D 选项不符合题意。\n\n故答案为: B\n\n【点睛】采用赋值法解答此题简便易行, 解题要掌握奇数、偶数、质数和合数的意义。"} {"id": "24189", "image": [], "answer": "D", "solution": "null", "level": "五年级", "question": "能同时被 2 和 5 整除,又有因数 3 的最大三位数是()。\n", "options": "A. 999\nB. 995\nC. 993\nD. 990\n", "subject": "算术", "analysis": "解D\n\n【分析】能同时被 2 和 5 整除的数的个位上的数字是 0 ; 一个数的各个数位上的数字之和是 3 的倍数,这个数就是 3 的倍数, 据此解答即可。\n\n【详解】分析各选项可知, 只有 D 项的个位数字是 0 。\n\n故答案为: $\\mathrm{D}$\n\n【点睛】本题考查 $2 、 3 、 5$ 的倍数特征, 明确它们的特征是解题的关键。"} {"id": "24190", "image": [], "answer": "B", "solution": "null", "level": "五年级", "question": "在 10 到 40 中, 7 的倍数共有()个。\n", "options": "A. 3\nB. 4\nC. 5\nD. 6\n", "subject": "算术", "analysis": "解B\n\n【分析】找一个数的倍数,直接把这个数分别乘 $1 、 2 、 3 、 4 、 5 、 6 \\ldots$ 从而选择与题意相符的数即可。\n\n【详解】 $7 \\times 2=14$\n\n$7 \\times 3=21$\n\n$7 \\times 4=28$\n\n$7 \\times 5=35$\n\n因此符合题意的 7 的倍数有 $14,21,28$ 和 35 这 4 个数。\n\n故答案为: B\n【点睛】本题考查了找 7 的倍数的方法。"} {"id": "24191", "image": [], "answer": "A", "solution": "null", "level": "五年级", "question": "在下面每组数中,有因数和倍数关系的是()。\n", "options": "A. 3 和 24\nB. 1.8 和 6\nC. 2 和 7\nD. 12 和 5\n", "subject": "算术", "analysis": "解A\n\n【分析】在整数除法中, 如果商是整数且没有余数, 我们就说被除数是除数的倍数, 除数是被除数的因数; 据此选择。\n\n【详解】A. $24 \\div 3=8$, 能整除, 所以 24 和 3 是因数和倍数关系;\n\nB. $1.8 \\div 6=0.3,1.8$ 和 0.3 都是小数, 所以 1.8 和 6 不是因数和倍数的关系;\n\nC. $7 \\div 2=3 \\ldots \\ldots 1$, 不能整除, 所以 2 和 7 不是因数和倍数关系;\n\nD. $12 \\div 5=2 \\ldots \\ldots .2$, 不能整除, 所以 12 和 5 不是因数和倍数关系。\n\n故答案为: A\n\n【点睛】本题考查因数和倍数的意义, 注意研究因数和倍数时, 所说的数是指非零的自然数。"} {"id": "24192", "image": [], "answer": "D", "solution": "null", "level": "五年级", "question": "亮亮家的电话号码是七位数 $\\mathrm{ABCDEFG}$, 其中: $\\mathrm{A}$ 是 6 的最小倍数, $\\mathrm{B}$ 是 6 的最大因数, $\\mathrm{C}$ 是最小的质数, $\\mathrm{D}$ 是最小的合数, $\\mathrm{E}$ 是 10 以内最大的奇数, $\\mathrm{F}$ 是 8 的最小因数, $\\mathrm{G}$ 是一位数中最大的偶数,则亮亮家的电话号码是()。\n", "options": "A. 6324928\nB. 6324918\nC. 6624928\nD. 6624918\n", "subject": "算术", "analysis": "解D\n\n【分析】根据质数与合数、奇数与偶数的意义, 一个自然数, 如果只有 1 和他本身两个因数, 这个数是质数;\n\n一个自然数如果除了 1 和它本身还有别的因数, 这样的数叫做合数;\n\n一个数最小因数是 1 , 最大因数是它自己;\n\n能被 2 整除的数叫做偶数, 不能被 2 整除的数叫做奇数;\n\n1 既不是质数也不是合数, 1 是最小的奇数。 2 是最小的质数, 4 是最小的合数。\n\n【详解】A 是 6 的最小倍数, $\\mathrm{A}$ 是 6\n\n$\\mathrm{B}$ 是 6 的最大因数, $\\mathrm{B}$ 是 6\n\n$\\mathrm{C}$ 是最小的质数, $\\mathrm{C}$ 是 2\n\n$\\mathrm{D}$ 是最小的合数, $\\mathrm{D}$ 是 4\n\n$\\mathrm{E}$ 是 10 以内最大的奇数, $\\mathrm{E}$ 是 9\n\n$\\mathrm{F}$ 是 8 的最小因数, $\\mathrm{F}$ 是 1\n\n$\\mathrm{G}$ 是一位数中最大的偶数, $\\mathrm{G}$ 是 8\n\n所以这个电话号码是 6624918 。\n\n故答案为: $\\mathrm{D}$\n\n【点睛】本题主要考查质数与合数、奇数与偶数、因数与倍数的概念和意义。\n\n9 . _详解$2 、 3 、 5 、 7 、 11 、 13 、 17$\n\n【分析】一个数只有 1 和它本身两个因数, 这个数叫做质数。据此解答。\n\n【详解】 18 以内的质数有 $2 、 3 、 5 、 7 、 11 、 13 、 17$ 。\n\n【点睛】本题考查了质数的认识, 需要对 20 以内的质数有所了解。"} {"id": "24193", "image": [], "answer": "B", "solution": "null", "level": "五年级", "question": "一个数既是 36 的因数, 又是 18 的倍数, 它是()。\n", "options": "A. 1\nB. 18\nC. 9\nD. 6\n", "subject": "算术", "analysis": "B"} {"id": "24204", "image": [], "answer": "C", "solution": "null", "level": "五年级", "question": "$a$ 的最小倍数正好等于 $b$ 的最大因数, 则 ( )\n", "options": "A. $\\mathrm{a}>\\mathrm{b}$\nB. $\\mathrm{a}<\\mathrm{b}$\nC. $a=b$\nD. 不能确定大小\n", "subject": "算术", "analysis": "解C"} {"id": "24211", "image": [], "answer": "B", "solution": "null", "level": "五年级", "question": "用 3、4、5 任意组成一个三位数, 这个三位数一定是()。\n", "options": "A. 2 的倍数\nB. 3 的倍数\nC. 4 的倍数\nD. 5 的倍数\n", "subject": "算术", "analysis": "解B"} {"id": "24212", "image": [], "answer": "B", "solution": "null", "level": "五年级", "question": "小莉说: “所有偶数都是合数。”下面选项 ( ) 说明小莉的说法是错误的。\n", "options": "A. 1\nB. 2\nC. 32\nD. 100\n", "subject": "算术", "analysis": "解B"} {"id": "24213", "image": [], "answer": "B", "solution": "null", "level": "五年级", "question": "下列说法中, 正确的是 ( )。\n\n", "options": "A. 两个质数的和一定是偶数。\nB. 1 是 $1,2,3,4,5$,的因数。\n\nC. 因为 $42 \\div 6=7$, 所以 42 是倍数, 7 和 6 是因数。\n\nD. 在 $1,2,3,4,5$, $\\qquad$中, 除了质数以外都是合数。\n\n", "subject": "算术", "analysis": "B"} {"id": "24214", "image": [], "answer": "A", "solution": "null", "level": "五年级", "question": "三个连续的偶数中, $\\mathrm{f}+1$ 是中间数, 其余两个偶数分别为()。\n", "options": "A. $\\mathrm{f}-1$ 和 $\\mathrm{f}+3$\nB. $\\mathrm{f}-2$ 和 $\\mathrm{f}+2$\nC. $\\mathrm{f}+3$ 和 $\\mathrm{f}+5$\nD. f-3 和 f-1\n", "subject": "算术", "analysis": "解A"} {"id": "24215", "image": [], "answer": "A", "solution": "null", "level": "五年级", "question": "100 以内是 3 和 5 的倍数的最大奇数是()。\n", "options": "A. 75\nB. 90\nC. 95\nD. 99\n", "subject": "算术", "analysis": "解A"} {"id": "24216", "image": [], "answer": "B", "solution": "null", "level": "五年级", "question": "8723 至少加(), 得到的数就同时是 $2 、 3 、 5$ 的倍数。\n", "options": "A. 2\nB. 7\nC. 1\nD. 4\n", "subject": "算术", "analysis": "解B"} {"id": "24218", "image": [], "answer": "D", "solution": "null", "level": "五年级", "question": "已知 $\\mathrm{a} \\div \\mathrm{b}=\\mathrm{c}$ (a, b, c 都大于 0 的自然数),那么下面说法正确的是()。\n", "options": "A. $\\mathrm{a}$ 是倍数\nB. $b$ 是因数\nC. c 是因数\nD. b, c 都是 $\\mathrm{a}$ 的因数\n", "subject": "算术", "analysis": "解$\\mathrm{D}$\n\n【分析】根据因数和倍数的意义, 如果数 $\\mathrm{a}$ 能被数 $\\mathrm{b}$ 整除 ( $\\mathrm{a}$ 和 $\\mathrm{b}$ 是非零自然数), $\\mathrm{a}$ 就叫做 $\\mathrm{b}$ 的倍数, $\\mathrm{b}$ 就叫做 $\\mathrm{a}$ 的因数; 据此解答即可。\n\n【详解】已知 $\\mathrm{a} \\div \\mathrm{b}=\\mathrm{c}(\\mathrm{a}, \\mathrm{b}, \\mathrm{c}$ 都是大于 0 的自然数 $)$ ,则 $\\mathrm{b} 、 \\mathrm{c}$ 都是 $\\mathrm{a}$ 的因数。\n\n故答案为: D\n\n【点睛】解答此题的关键是根据因数和倍数的意义进行分析、解答即可, 注意不能单独说这个数是因数或倍数。"} {"id": "24229", "image": [], "answer": "B", "solution": "null", "level": "五年级", "question": "连续两个非零自然数的乘积一定是()。\n", "options": "A. 奇数\nB. 偶数\nC. 质数\nD. 合数\n", "subject": "算术", "analysis": "解$\\mathrm{B}$\n\n【分析】两个连续的非零自然数相差 1, 所以其中一个一定是偶数, 另一个一定是奇数, 根据偶数 $\\times$奇数 $=$ 偶数, 连续两个非零自然数的乘积一定是偶数。据此解答。\n\n【详解】连续两个非零自然数的乘积一定是偶数。\n\n故答案为: B\n\n【点睛】本题主要运用奇数和偶数的运算性质解答, 明确相邻的自然数相差 1。"} {"id": "24238", "image": [], "answer": "A", "solution": "null", "level": "五年级", "question": "下面算式 ( $\\mathrm{a}$ 为任意自然数) 的结果, 得数一定是偶数的是 ( )。\n", "options": "A. $a+a$\nB. $3 a$\nC. $a^{2}$\nD. $a+5$\n", "subject": "算术", "analysis": "解A\n\n【分析】奇数 + 奇数 $=$ 偶数, 偶数 + 偶数 $=$ 偶数, 奇数 $\\times$ 奇数 $=$ 奇数, 奇数 $\\times$ 偶数 $=$ 偶数, 据此分析。\n\n【详解】A. $a+a$, 如果 $\\mathrm{a}$ 是奇数, 奇数 + 奇数 $=$ 偶数, 如果 $\\mathrm{a}$ 是偶数, 偶数+偶数=偶数, 所以 $\\mathrm{a}$ $+\\mathrm{a}$ 一定是偶数;\n\nB. $3 a$, 如果 $\\mathrm{a}$ 是奇数, 3 也是奇数, 奇数 $\\times$ 奇数 $=$ 奇数;\n\nC. $a^{2}$, 如果 $\\mathrm{a}$ 是奇数, 奇数 $\\times$ 奇数 $=$ 奇数;\n\nD. $a+5$, 如果 $\\mathrm{a}$ 是偶数, 5 是奇数, 奇数 + 偶数 $=$ 奇数。\n\n故答案为: A\n\n【点睛】关键是掌握奇数和偶数的运算性质。"} {"id": "24240", "image": [], "answer": "D", "solution": "null", "level": "五年级", "question": "要使四位数 425 口是 3 的倍数, 口里最小应填()。\n", "options": "A. 4\nB. 3\nC. 2\nD. 1\n", "subject": "算术", "analysis": "解D\n\n【分析】 3 的倍数特征: 各个位上数字相加的和是 3 的倍数, 从最小的自然数依次计算, 据此解答。\n\n【详解】口里为 0 时, $4+2+5=11,11$ 不是 3 的倍数;\n\n口里为 1 时, $4+2+5+1=12,12$ 是 3 的倍数。\n\n所以, $\\square$ 里最小应填 1 。\n\n故答案为: D\n\n【点睛】掌握 3 的倍数特征是解答题目的关键。"} {"id": "24241", "image": [], "answer": "D", "solution": "null", "level": "五年级", "question": "著名的哥德巴赫猜想中说: “任意一个大于 2 的偶数, 都可以表示成 2 个质数的和。”下面四组算式中, 可以验证这个猜想的是()。\n", "options": "A. $36=23+13 \\quad 2=1+1$\nB. $36=17+19 \\quad 26=17+9$\nC. $36=17+19 \\quad 28=15+13$\nD. $36=23+13 \\quad 28=11+17$\n", "subject": "算术", "analysis": "解D\n\n【分析】根据质数和合数的概念, 以及哥德巴赫猜想的内涵, 一一分析各个选项是否符合猜想即可。\n\n【详解】A. $2=1+1,1$ 不是质数, 所以不符合哥德巴赫猜想;\n\nB. $26=17+9,26$ 是大于 2 的偶数, 但是 9 是合数, 所以不符合哥德巴赫猜想;\n\nC. $28=15+13,28$ 是大于 2 的偶数, 但是 15 是合数, 所以不符合哥德巴赫猜想;\n\nD. $36=23+13,36$ 是大于 2 的偶数, 23 和 13 均为质数; $28=11+17,28$ 是大于 2 的偶数, 11 和 17 均为质数, 所以 $\\mathrm{D}$ 选项符合哥德巴赫猜想。\n\n故答案为: $\\mathrm{D}$\n【点睛】本题考查了偶数、质数和合数, 2 的倍数是偶数, 因数只有 1 和本身的数是质数, 合数的因数除了 1 和本身还有别的数。"} {"id": "24242", "image": [], "answer": "C", "solution": "null", "level": "五年级", "question": "下面说法中, 正确的有 ( ) 个。\n\n(1)一个数的因数的个数是有限的, 一个数的倍数的个数是无限的。(2)任何非零自然数都有因数 1 。 (3) $32 \\div 4=8$, 所以 32 是 4 的倍数。(4)奇数十奇数=奇数。(5) 1 既不是质数, 也不是合数。(6)自然数 $\\mathrm{a}$大于 $b, a$ 的因数个数一定比 $b$ 多。(7)非零自然数按照因数的个数可以分为质数、合数和 1。(8)个位是 $3 、 6 、 9$ 的数, 都是 3 的倍数。\n", "options": "A. 3\nB. 4\nC. 5\nD. 6\n", "subject": "算术", "analysis": "解C\n\n【分析】(1)一个数的因数的个数是有限的, 一个数的倍数的个数是无限的, 说法正确;\n\n②任何非零自然数都有因数 1, 说法正确;\n\n(3)根据因数和倍数的意义可知: $32 \\div 4=8$, 所以 32 是 4 的倍数, 说法正确;\n\n(4)因为奇数 + 奇数 $=$ 偶数, 所以奇数 + 奇数 $=$ 奇数的说法错误;\n\n(5)根据质数和合数的定义可知, 1 既不是质数, 也不是合数, 说法正确;\n\n(6)自然数 $\\mathrm{a}$ 大于 $\\mathrm{b}, \\mathrm{a}$ 的因数个数一定比 $\\mathrm{b}$ 多, 说法错误, 例如: 11 比 10 大, 11 只有 2 个因数, 10 有 4 个因数;\n\n(7)非零自然数按照因数的个数可以分为质数、合数和 1, 说法正确;\n\n(8)个位是 $3,6,9$ 的数, 都是 3 的倍数, 说法错误, 3 的倍数特征是各个数位上的数字和是 3 的倍数,例如: 103 不是 3 的倍数。据此解答。\n\n【详解】由分析可知, 说法正确的是(1)(2)(3)(5)(7), 一共有 5 个。\n\n故答案为: C\n\n【点睛】本题主要考查了因数和倍数、质数和合数、奇数和偶数的内容。逐一分析即可。"} {"id": "24244", "image": [], "answer": "A", "solution": "null", "level": "五年级", "question": "一个正方形的边长是一个质数, 这个正方形的周长一定是 ( )。\n", "options": "A. 合数\nB. 奇数\nC. 质数\n", "subject": "算术", "analysis": "解A\n\n【分析】一个数只有 1 和它本身两个因数, 这个数叫做质数。一个数除了 1 和它本身两个因数, 还有其他的因数, 这个数叫做合数。正方形的周长 $=$ 边长 $\\times 4$, 它的周长至少有的因数 (1、2、4、边长),所以说一定是合数。据此解答。\n\n【详解】根据分析可知, 一个正方形的边长是一个质数, 这个正方形的周长一定是合数。例如: 一个边长为 2 的正方形, $2 \\times 4=8$\n\n它的周长是 8,8 的因数有 $1 、 2 、 4 、 8$, 所以 8 是合数。\n\n故答案为: A\n\n【点睛】本题主要考查了质数和合数的认识以及应用。"} {"id": "24255", "image": [], "answer": "A", "solution": "null", "level": "五年级", "question": "要使 8270 是 3 的倍数, 至少要加上()。\n", "options": "A. 1\nB. 2\nC. 3\n", "subject": "算术", "analysis": "解A\n\n【分析】 3 的倍数的数的特征是:各位上的数字之和是 3 的倍数, 这个数就是 3 的倍数; 据此解答。\n\n【详解】 $8+2+7=17$\n\n$17+1=18$\n\n要使 8270 是 3 的倍数, 至少要加上 1 。\n\n故答案为: A\n\n【点睛】此题主要考查 3 的倍数的特征。"} {"id": "24262", "image": [], "answer": "A", "solution": "null", "level": "五年级", "question": "三位整数 $3 \\square 0$, 同时是 $2 、 3$ 和 5 的倍数, 口里有 ( ) 种填法。\n", "options": "A. 4\nB. 3\nC. 2\n", "subject": "算术", "analysis": "解A\n\n【分析】 2 的倍数特征: 末尾数字是 $0 、 2 、 4 、 6 、 8$ 的数是 2 的倍数;\n\n3 的倍数特征: 各个数位上的数字之和是 3 的倍数, 这个数字就是 3 的倍数;\n\n5 的倍数特征: 末尾数字是 0 或 5 的数是 5 的倍数;\n\n若一个数同时是 $2 、 3$ 和 5 的倍数, 则这个数的个位上的数字一定是 0 , 各个数位上的数字之和一定是 3 的倍数。据此选择即可。\n\n【详解】 $3 \\square 0$, 要使这个数三位数能同时被 2、3、5 整除, 十位上可以填 $0 、 3 、 6 、 9$; 即 300、330、 360、390 一共 4 种不同的填法。\n\n故答案为: A\n\n【点睛】本题考 $2 、 3 、 5$ 的倍数特征, 明确它们的特征是解题的关键。"} {"id": "24263", "image": [], "answer": "C", "solution": "null", "level": "五年级", "question": "24 的因数共有()个。\n", "options": "A. 4\nB. 6\nC. 8\n", "subject": "算术", "analysis": "解$\\mathrm{C}$\n\n【分析】列乘法算式找因数的方法: 把这个数写成两个整数相乘的形式, 算式中的每个整数都是这个数的因数。据此先写出 24 的因数, 再确定因数的个数。\n\n【详解】 $24=1 \\times 24=2 \\times 12=3 \\times 8=4 \\times 6$, 所以 24 的因数有: $1,2,3,4,6,8,12,24$ 。即 24 的因\n数共有 8 个。\n\n故答案为: C\n\n【点睛】用乘法找一个数的因数时, 一般要从自然数 1 开始一对一对地找, 这样不容易遗漏。"} {"id": "24264", "image": [], "answer": "C", "solution": "null", "level": "五年级", "question": "一个数的最大因数和这个数的最小倍数相比较 ( )。\n", "options": "A. 倍数大于因数\nB. 倍数小于因数\nC. 倍数等于因数\n", "subject": "算术", "analysis": "解C\n\n【分析】一个数的因数和倍数的特征: 一个数的倍数的个数是无限的, 最小的是它本身, 没有最大的倍数; 一个数的因数的个数是有限的, 最小的因数是 1, 最大的因数是它本身; 一个数最大的因数 $=$最小的倍数 $=$ 这个数本身。据此解答。\n\n【详解】根据分析得, 一个数的最大因数等于这个数的最小倍数。\n\n故答案为: C\n\n【点睛】此题主要考查因数与倍数的意义, 利用一个数的最小倍数是它的本身, 一个数的最大因数是它本身, 解决问题。"} {"id": "24265", "image": [], "answer": "B", "solution": "null", "level": "五年级", "question": "$\\mathrm{a}$ 是一个质数, 那么 $\\mathrm{a}$ ( )。\n", "options": "A. 有一个因数。\nB. 有两个因数。\nC. 至少有三个因数。\n", "subject": "算术", "analysis": "解$\\mathrm{B}$\n\n【详解】质数指的是在大于 1 的自然数中, 除了 1 和它本身以外不再有其他因数的自然数。根据质数的特征求解。质数只有两个因数, 比如 2 是质数, 只有 1 和 2 两个因数, 再比如 3 是质数, 只有 1 和 3 两个因数。\n\n故答案为: B"} {"id": "24266", "image": [], "answer": "C", "solution": "null", "level": "五年级", "question": "两个质数的积一定是 ( )。\n", "options": "A. 奇数\nB. 偶数\nC. 合数\n", "subject": "算术", "analysis": "解C\n\n【分析】一个数, 如果只有 1 和它本身两个因数, 那么这样的数叫做质数; 一个数, 如果除了 1 和它本身还有别的因数, 那么这样的数叫做合数。\n\n整数中, 是 2 的倍数的数叫做偶数, 不是 2 的倍数的数叫做奇数。\n\n【详解】A. 如: $2 \\times 3=6,6$ 是偶数, 所以两个质数的积不一定是奇数;\n\nB. 如: $5 \\times 7=35,35$ 是奇数, 所以两个质数的积不一定是偶数;\n\nC. 如: $2 \\times 3=6,6$ 是合数;\n\n$5 \\times 7=35,35$ 是合数;\n\n所以, 两个质数的积一定是合数。\n\n故答案为: C\n\n【点睛】本题考查质数与合数、奇数与偶数的意义及应用。"} {"id": "24267", "image": [], "answer": "C", "solution": "null", "level": "五年级", "question": "既是 2 的倍数又是 5 的倍数的最大三位数是 ( )。\n", "options": "A. 999\nB. 995\nC. 990\n", "subject": "算术", "analysis": "解C\n\n【分析】 2 的倍数特征:个位上是 $0 、 2 、 4 、 6 、 8$ 的数。\n\n5 的倍数特征: 个位上是 0 或 5 的数。\n\n2、 5 的倍数特征: 个位上是 0 的数。\n【详解】既是 2 的倍数又是 5 的倍数, 则这个三位数的个位一定是 0 , 而十位、百位最大都是 9 , 所以这个三位数最大是 990 。\n\n故答案为: C\n\n【点睛】本题考查 $2 、 5$ 的倍数的特征及应用。"} {"id": "24269", "image": [], "answer": "B", "solution": "null", "level": "五年级", "question": "10 以内(包括 10)的合数一共有()个。\n", "options": "A. 4\nB. 5\nC. 6\nD. 7\n", "subject": "算术", "analysis": "解$\\mathrm{B}$\n\n【分析】合数是指除了 1 和它本身的两个因数以外还有其他的因数。据此写出 10 以内(包括 10)所有的合数, 即可得解。\n\n【详解】 10 以内(包括 $10 )$ 的合数有:4、6、8、9、10, 共有 5 个。\n\n故答案为: B\n\n【点睛】此题的解题关键是理解掌握合数的定义, 学生要熟悉一些常用的合数。"} {"id": "24280", "image": [], "answer": "D", "solution": "null", "level": "五年级", "question": "下列说法错误的是()。\n", "options": "A. 合数有最小的, 但没有最大的\nB. 是 2 和 5 的倍数的数一定是偶数\nC. 在研究因数和倍数时没有考虑 0\nD. 所有的质数都是奇数\n", "subject": "算术", "analysis": "解D\n\n【分析】一个数, 如果只有 1 和它本身两个因数, 那么这样的数叫做质数; 一个数, 如果除了 1 和它本身还有别的因数, 那么这样的数叫做合数; 2、 5 的倍数特征: 个位上是 0 的数; 整数中, 是 2 的倍数的数叫做偶数, 不是 2 的倍数的数叫做奇数。\n\n【详解】A. 最小的合数是 4 , 没有最大的合数, 原题说法正确;\n\nB. 是 2 和 5 的倍数的数个位一定是 0 , 所以一定是偶数, 原题说法正确;\n\nC. 在研究因数和倍数时没有考虑 0 , 原题说法正确;\n\nD. 质数 2 是偶数, 原题说法错误。\n\n故答案为: D\n\n【点睛】掌握质数与合数、 2 和 5 的倍数特征、奇数与偶数的意义是解题的关键。"} {"id": "24289", "image": [], "answer": "C", "solution": "null", "level": "五年级", "question": "三位数 $6 \\square 9$ 是 3 的倍数, 口里有 ( ) 种不同的填法。\n", "options": "A. 2\nB. 3\nC. 4\nD. 5\n", "subject": "算术", "analysis": "解C\n\n【分析】能被 3 整除的数的特征: 各个数位上的数的和能被 3 整除, 也可以说各个数位上的数的和是 3 的倍数; 根据此特征计算后再选择。\n\n【详解】 $6 \\square 9$ 的个位和百位上的数的和: $6+9=15$\n\n因为 $15+0=15,15+3=18,15+6=21,15+9=24,15 、 18 、 21 、 24$ 都是 3 的倍数, 所以口里可以, $0 、 3 、 6 、 9,4$ 种填法;\n\n故答案为: C\n\n【点睛】此题考查能被 3 整除的数的特征。"} {"id": "24290", "image": [], "answer": "D", "solution": "null", "level": "五年级", "question": "$\\mathrm{a}+75$ 的和是奇数, $\\mathrm{a}$ 一定是 ( )。\n", "options": "A. 质数\nB. 合数\nC. 奇数\nD. 偶数\n", "subject": "算术", "analysis": "解D\n\n【分析】两个加数的和为奇数, 其中一个加数是奇数, 那么另一个加数一定是偶数, 举例说明即可。\n\n【详解】A. 当 $\\mathrm{a}$ 为质数时, 假设 $\\mathrm{a}$ 是 $3, \\mathrm{a}+75=3+75=78,78$ 是偶数, 错误;\n\nB. 当 $\\mathrm{a}$ 为合数时, 假设 $\\mathrm{a}$ 是 $9, \\mathrm{a}+75=9+75=84,84$ 是偶数, 错误;\n\nC. 当 $\\mathrm{a}$ 为奇数时, 假设 $\\mathrm{a}$ 是 $13, \\mathrm{a}+75=13+75=88,88$ 是偶数, 错误;\n\nD. 当 $\\mathrm{a}$ 为偶数时,假设 $\\mathrm{a}$ 是 $0, \\mathrm{a}+75=0+75=75,75$ 是奇数, 正确。\n故答案为: $\\mathrm{D}$\n\n【点睛】掌握奇数、偶数的运算性质是解答题目的关键。"} {"id": "24291", "image": [], "answer": "C", "solution": "null", "level": "五年级", "question": "下面各种说法, 有 ( ) 句是正确的。\n\n(1)一个数的最小倍数是它本身。\n\n(2)一个数有无数个倍数。\n\n(3)一个数的倍数大于它的因数。\n\n(4)一个数至少有两个因数。\n\n(5)两个不同质数的积一定是合数。\n", "options": "A. 1\nB. 2\nC. 3\nD. 4\n\n", "subject": "算术", "analysis": "解C\n\n【分析】根据题意, 对各选项进行依次分析、进而得出结论。\n\n【详解】(1)一个数的最小倍数是它本身, 说法正确;\n\n(2)一个数倍数的个数是无限的, 所以一个数的倍数有有无数个, 说法正确;\n\n(3)一个数至少有两个因数, 说法错误, 如 1 的因数只有 1 ;\n\n(4)两个不同质数的积一定是合数, 说法正确。\n\n故答案为: C\n\n【点睛】本题考查了因数、倍数、质数和合数的认识以及它们之间的区别和联系。"} {"id": "24292", "image": [], "answer": "C", "solution": "null", "level": "五年级", "question": "把 14 写成 2 个不同质数的和是 ( )。\n", "options": "A. $1+13$\nB. $7+7$\nC. $3+11$\nD. $9+5$\n", "subject": "算术", "analysis": "解C\n\n【分析】一个数, 如果只有 1 和它本身两个因数, 那么这样的数叫做质数; 一个数, 如果除了 1 和它本身还有别的因数, 那么这样的数叫做合数, 1 既不是质数也不是合数; 据此解答。\n\n【详解】A. $14=1+13,13$ 是质数, 1 既不是质数也不是合数, 错误;\n\nB. $14=7+7,7$ 是质数, 把 14 写成了 2 个相同质数的和, 错误;\n\nC. $14=3+11,3$ 和 11 都是质数, 正确;\n\nD. $14=9+5,9$ 是合数, 5 是质数, 错误。\n\n故答案为: C\n\n【点睛】掌握质数、合数的意义是解答题目的关键。"} {"id": "24293", "image": [], "answer": "B", "solution": "null", "level": "五年级", "question": "$2 \\square 60$ 这个四位数是 $2 、 3 、 5$ 的倍数, 它的百位上可能是()。\n", "options": "A. 0\nB. 1\nC. 3\nD. 5\n", "subject": "算术", "analysis": "解$\\mathrm{B}$\n\n【分析】同时是 $2 、 3 、 5$ 的倍数的数, 个位上必须是 0 且各位上的数字之和是 3 的倍数, 据此解答。\n\n【详解】 $2+6=8$\n\n$8+1=9$\n\n$8+4=12$\n\n$8+7=15$\n\n9、 $12 、 15$ 是 3 的倍数。\n\n它的百位上可能是 $1 、 4$ 或 7。\n\n故答案为: B\n\n【点睛】本题考查了同时是 $2 、 3 、 5$ 的倍数的数的特征, 要熟练掌握并运用。"} {"id": "24294", "image": [], "answer": "A", "solution": "null", "level": "五年级", "question": "20 以内既是奇数又是质数的数共有 ( ) 个。\n", "options": "A. 7\nB. 8\nC. 9\nD. 10\n", "subject": "算术", "analysis": "解A\n\n【分析】在整数中, 是 2 的倍数的数是偶数 ( 0 也是偶数); 不是 2 的倍数的数, 叫作奇数; 一个数,\n如果只有 1 和它本身两个因数, 这样的数就叫做质数; 据此解答。\n\n【详解】 20 以内:\n\n奇数有: $1 、 3 、 5 、 7 、 9 、 11 、 13 、 15 、 17 、 19$;\n\n质数有: $2 、 3 、 5 、 7 、 11 、 13 、 17 、 19$;\n\n2 是偶数, 所以 20 以内既是奇数又是质数的数共有 7 个。\n\n故答案为: A\n\n【点晴】本题同时结合奇数、质数, 这两种属性来做选择, 需要熟悉 20 以内奇数、质数的具体情况。"} {"id": "24296", "image": [], "answer": "B", "solution": "null", "level": "五年级", "question": "$a+6$ 是奇数, $a$ 一定是 ( )。\n", "options": "A. 偶数\nB. 奇数\nC. 合数\n", "subject": "算术", "analysis": "解$\\mathrm{B}$\n\n【分析】根据奇数十偶数 $=$ 奇数, 又因为 $a+6$ 是奇数, 6 是偶数, 则 $\\mathrm{a}$ 一定是奇数。\n\n【详解】结合奇偶数的运算性质可得:\n\n$a+6$ 是奇数, $a$ 一定是(奇数)。\n\n故答案为: B\n\n【点睛】考查了奇偶数的运算性质, 其中一个加数以字母的形式呈现, 又训练了抽象思维能力。"} {"id": "24307", "image": [], "answer": "A", "solution": "null", "level": "五年级", "question": "下列选项中一定表示偶数的是()。\n", "options": "A. $2 b$\nB. $\\mathrm{a}+4$\nC. $\\mathrm{m}-3$\n", "subject": "算术", "analysis": "解A\n\n【分析】根据奇数和偶数的定义可知, 不能被 2 整除的自然数叫奇数, 能被 2 整除的数叫偶数。逐一判断3 个选项, 可采用赋值法, 排除掉错误的答案。\n\n【详解】A. 因为 $2 \\mathrm{~b}$ 是 2 的倍数, 能被 2 整除, 所以 $2 b$ 可以表示偶数;\n\nB. 当 $\\mathrm{a}=1$ 时, $1+4=5,5$ 不是偶数, 所以 $\\mathrm{a}+4$ 不能表示偶数;\n\nC. 当 $\\mathrm{m}=4$ 时, $4-3=1,1$ 不是偶数, 所以 $\\mathrm{m}-3$ 不能表示偶数。\n\n故答案为: A\n\n【点睛】此题的解题关键是理解掌握偶数的定义。"} {"id": "24314", "image": [], "answer": "B", "solution": "null", "level": "五年级", "question": "著名的“哥德巴赫猜想”被喻为“数学皇冠上的明珠”, 猜想认为:任何大于 2 的偶数都是两个质数之和。下列算式中,符合这个猜想的是()。\n", "options": "A. $4=1+3$\nB. $16=3+13$\nC. $15=2+13$\n", "subject": "算术", "analysis": "解$\\mathrm{B}$\n\n【分析】整数中, 是 2 的倍数的数叫做偶数, 不是 2 的倍数的数叫做奇数。\n\n一个数, 如果只有 1 和它本身两个因数, 那么这样的数叫做质数; 一个数, 如果除了 1 和它本身还有别的因数, 那么这样的数叫做合数。\n\n根据题意, 看各选项中的算式是否符合“任何大于 2 的偶数都是两个质数之和”, 据此选择。\n\n【详解】A. $4=1+3$, 其中 1 不是质数, 不符合题意;\n\nB. $16=3+13$, 其中 16 是偶数, $3 、 13$ 是质数, 符合题意;\n\nC. $15=2+13$, 其中 15 是奇数, 不是偶数, 不符合题意。\n\n故答案为: B\n\n【点睛】掌握奇数与偶数、质数与合数的定义是解题的关键。"} {"id": "24315", "image": [], "answer": "B", "solution": "null", "level": "五年级", "question": "一个三位数, 百位是最小的合数, 个位是最小的质数且能被 3 整除, 它是()。\n", "options": "A. 204\nB. 402\nC. 452\n", "subject": "算术", "analysis": "解B\n\n【分析】最小的合数是 4 , 最小的质数是 2 , 然后根据 3 的倍数特征: 各个数位上数字之和是 3 的倍数, 这个数就是 3 的倍数。据此解答即可。\n\n【详解】由分析可知:\n\n结合选项可知 B 项符合要求。\n\n故答案为: B\n【点睛】本题考查质数和合数, 明确质数和合数的定义是解题的关键。"} {"id": "24316", "image": [], "answer": "C", "solution": "null", "level": "五年级", "question": "42 的因数一共有()个。\n", "options": "A. 6\nB. 7\nC. 8\n", "subject": "算术", "analysis": "解C\n\n【分析】利用求因数的方法, 先找出 42 的因数, 再统计出它的因数个数即可。\n\n【详解】 $42=1 \\times 42=2 \\times 21=3 \\times 14=6 \\times 7$\n\n所以, 42 的因数有 $1 、 2 、 3 、 6 、 7 、 14 、 21 、 42$, 一共有 8 个因数。\n\n故答案为: C\n\n【点睛】本题考查了因数, 掌握因数的求法是解题的关键。"} {"id": "24317", "image": [], "answer": "A", "solution": "null", "level": "五年级", "question": "一个三位数 $65 \\square$, 使它成为 3 的倍数, $\\square$ 里最大填()。\n", "options": "A. 7\nB. 8\nC. 9\n", "subject": "算术", "analysis": "解A\n\n【分析】根据 3 的倍数特征, 一个数的各个数位上数字之和是 3 的倍数, 这个数就是 3 的倍数。据此解答即可。\n\n【详解】A. $6+5+7=18,18$ 是 3 的倍数, 所以 657 是 3 的倍数;\n\nB. $6+5+8=19,19$ 不是 3 的倍数, 所以 658 不是 3 的倍数;\n\nC. $6+5+9=20,20$ 不是 3 的倍数, 所以 659 不是 3 的倍数。\n\n故答案为: A\n\n【点睛】本题考查 3 的倍数特征, 明确 3 的倍数特征是解题的关键。"} {"id": "24321", "image": [], "answer": "C", "solution": "null", "level": "五年级", "question": "15 的因数有 ( ) 个。\n", "options": "A. 1\nB. 3\nC. 4\n", "subject": "算术", "analysis": "解C\n\n【分析】列乘法算式找因数, 按照从小到大的顺序, 一组一组地写出所有积是这个数的乘法算式, 乘法算式中的两个因数就是这个数的因数。\n\n【详解】 15 的因数有: $1,3,5,15$; 一共有 4 个。\n\n故答案为: C\n\n【点睛】掌握找一个数的因数的方法是解题的关键。"} {"id": "24332", "image": [], "answer": "C", "solution": "null", "level": "五年级", "question": "任意两个奇数相乘, 积一定是()。\n", "options": "A. 合数\nB. 质数\nC. 奇数\n", "subject": "算术", "analysis": "解C\n\n【分析】根据奇数和偶数的定义:是 2 的倍数的数叫做偶数; 不是 2 的倍数的数叫做奇数; 奇数和偶数的运算性质: 奇数与奇数的积是奇数; 偶数与偶数的积是偶数; 奇数与偶数的积是偶数; 由此解答即可。\n\n【详解】根据上面的分析得: 奇数 $\\times$ 奇数 $=$ 奇数, 如: $3 \\times 5=15,3 \\times 7=21$\n\n两个奇数相乘, 积一定是奇数。\n\n故答案为: $\\mathrm{C}$\n\n【点睛】此题考查的目的是使学生理解偶数与奇数的意义, 掌握奇数和偶数的运算性质, 根据其性质进行分析解答。"} {"id": "24340", "image": [], "answer": "B", "solution": "null", "level": "五年级", "question": "哥德巴赫猜想被誉为“数学皇冠上的明珠”, 内容为“任何大于 2 的偶数都可以表示成两个质数的和”。下面算式中符合这个猜想的是()。\n", "options": "A. $8=1+7$\nB. $12=5+7$\nC. $24=3+21$\n", "subject": "算术", "analysis": "解$\\mathrm{B}$\n\n【分析】除了 1 和它本身以外不再有其他因数, 这样的数叫质数。据此选项中算式的加数是质数的即可。\n\n【详解】A. $8=1+7,1$ 既不是质数也不是合数, 不符合;\n\nB. $12=5+7,5$ 和 7 都是质数, 符合;\n\nC. $24=3+21,21$ 不是质数, 不符合。\n\n故答案为: B\n\n【点睛】关键是理解质数的含义,掌握质数、合数的分类标准。"} {"id": "24341", "image": [], "answer": "B", "solution": "null", "level": "五年级", "question": "五年一班有 37 人, 已知女生人数是偶数, 那男生人数一定是 ( )。\n", "options": "A. 偶数\nB. 奇数\nC. 质数\n", "subject": "算术", "analysis": "解$\\mathrm{B}$\n\n【分析】整数中, 是 2 的倍数的数叫做偶数, 不是 2 的倍数的数叫做奇数。\n\n已知五年一班总人数是奇数, 女生人数是偶数, 根据偶数 + 偶数 $=$ 偶数, 偶数 + 奇数 $=$ 奇数; 由此得出结论。\n\n【详解】偶数十奇数 $=$ 奇数\n\n五年一班有 37 人, 已知女生人数是偶数, 那男生人数一定是奇数。\n\n故答案为: B\n【点睛】掌握奇数与偶数的定义以及奇数、偶数的运算性质是解题的关键。"} {"id": "23998", "image": [], "answer": "C", "solution": "null", "level": "五年级", "question": "把分别写有 $1,2,3,4, \\ldots, 9,10$ 的 10 张卡片反扣在桌面上,打乱顺序后,任意摸出 1 张,摸到()的可能性最小。\n", "options": "A. 奇数\nB. 偶数\nC. 质数\nD. 合数\n", "subject": "组合数学", "analysis": "解C\n\n【分析】找出 $1,2,3,4, \\ldots, 9,10$ 中的奇数、偶数、质数、合数的数量, 再判断即可。\n\n【详解】在 $1,2,3,4, \\ldots, 9,10$ 中, 奇数有 $1 、 3 、 5 、 7 、 9$ 共 5 个; 偶数有 $2 、 4 、 6 、 8 、 10$ 共 5 个; 质数有 $2 、 3 、 5 、 7$ 共 4 个; 合数有 $4 、 6 、 8 、 9 、 10$ 共 5 个。\n\n因为 $4<5$, 所以摸到质数的可能性最小。\n\n故答案为: C\n\n【点睛】根据事件发生的可能性大小, 哪种情况发生的数量最多, 事件发生的可能性就最大; 哪种情况发生的数量最少, 事件发生的可能性就最小; 哪种情况发生的数量一样多, 事件发生的可能性就相等。"} {"id": "24239", "image": ["12456.jpg"], "answer": "D", "solution": "null", "level": "五年级", "question": "下面 代表一个不等于 0 的数字, $\\star$ 是 0 。下面组成的四位数中, 一定是 2、3、5 的公倍数的是 ( )。\nAS\nB. 人从\nC.\n\n\nD.\n\n5. _在 3 口里填上适当的数字, 使这个数既是 2 的倍数又是 3 的倍数, 有 ( ) 种填法。\n", "options": "A. 2\nB. 3\nC. 4\nD. 5\n", "subject": "组合数学", "analysis": "解D\n\n【分析】根据 $2 、 3 、 5$ 的倍数的特征, 个位上是 $0 、 2 、 4 、 6 、 8$ 的数是 2 的倍数; 各个位上的数字之和是 3 的倍数, 这个数一定是 3 的倍数; 个位上是 0 或 5 的数一定是 5 的倍数, 同时是 $2 、 3 、 5$ 的倍数的特征是:个位上必须是 0 且各位上的数字之和是 3 的倍数。据此解答。\n\n【详解】A. 假设 $\\bullet 1$, 这个数是 $1010,1+0+1+0=2$, 可得 1010 不是 3 的倍数,\n\nB. 假设 $\\bullet=1$, 这个数是 $1100,1+1+0+0=2$, 可得 1100 不是 3 的倍数;\n\nC. 个位上是 $\\bullet$, 不是 0 , 不满足同时是 $2 、 5$ 的倍数;\n\nD. 不管・是 $1 \\sim 9$ 中的哪一个数, 它都具备了 3 的倍数的特征, 而且个位上是 0 , 所以这个数一定是 2 、\n\n3、 5 的公倍数。\n\n故答案为: $\\mathrm{D}$\n\n【点睛】此题考查的目的是理解掌握 $2 、 3 、 5$ 的倍数的特征。"} {"id": "24343", "image": [], "answer": "B", "solution": "null", "level": "五年级", "question": "用 $1 、 3 、 5$ 三张数字卡片排成的所有三位数中, 下面说法正确的是()。\n", "options": "A. 一定是 5 的倍数\nB. 一定是 3 的倍数\nC. 无法确定\n", "subject": "组合数学", "analysis": "解B\n\n【分析】根据搭配方法, 先列出组成的所有三位数, 再根据 3 和 5 的倍数特征进行选项判断即可。\n\n【详解】可以排成的三位数:135、153、351、315、 513、531\n\nA. 0 和 5 结尾的整数才是 5 的倍数, 组成的数不完全是 5 的倍数, 说法错误;\n\nB. $1+3+5=9,9$ 能整除 3 , 所以一定是 3 的倍数, 说法正确;\n\nC. 有可以确定答案。\n\n故答案为: B\n\n【点晴】此题考查了搭配的方法以及 $3 、 5$ 的倍数特征。"} {"id": "24882", "image": [], "answer": "C", "solution": "null", "level": "五年级", "question": "有 10 张数字卡片, 分别写着 $1-10$, 从中任意抽取一张, 抽到 ( )可能性最小。\n", "options": "A. 奇数\nB. 偶数\nC. 质数\nD. 合数\n", "subject": "组合数学", "analysis": "解C\n\n【分析】 10 以内的数: $1 、 3 、 5 、 7 、 9$ 是奇数, $2 、 4 、 6 、 8 、 10$ 是偶数, $2 、 3 、 5 、 7$ 是质数, $4 、 6 、$ 8、9、10 是合数, 根据数字出现的数量解答。\n\n【详解】有 10 张数字卡片, 分别写着 $1-10$, 其中奇数有 5 个, 偶数有 5 个, 质数有 4 个, 合数有 5 个, 因此抽到质数的可能性最小。\n\n故答案为: C\n\n【点睛】本题考查了奇数、偶数、质数及合数的含义。"} {"id": "24917", "image": ["12679.jpg"], "answer": "D", "solution": "null", "level": "五年级", "question": "甲乙两人用一个转盘(如下图)做游戏, 如果指针停在符合要求的数上, 则获胜, 反之失败。若想使获胜的可能性最大,应该采用选项()。\n\n\n", "options": "A. 质数\nB. 合数\nC. 2 的倍数\nD. 36 的因数\n\n", "subject": "组合数学", "analysis": "解D\n\n【分析】可能性的大小与事件的基本条件和发展过程等许多因素有关。哪种数的数量多, 发生的可能性就大一些, 据此分析。\n\n【详解】A. 质数有 $2 、 3 、 5 、 7$, 共 4 个;\n\nB. 合数有 $4 、 6 、 8 、 9 、 10$, 共 5 个;\n\nC. 2 的倍数有 $2 、 4 、 6 、 8 、 10$, 共 5 个;\n\nD. 36 的因数有 $1 、 2 、 3 、 4 、 6 、 9$, 共 6 个。\n\n故答案为: D\n\n【点睛】关键是掌握 2 的倍数的特征, 理解质数、合数的分类标准, 会找一个数的因数, 再根据可能性大小的判断方法进行分析。"} {"id": "24068", "image": [], "answer": "C", "solution": "null", "level": "五年级", "question": "(2022 秋・怀柔区期末)把 48 块月饼装在盒子里, 每个盒子装同样多, 有()种装法。\n", "options": "A. 5\nB. 9\nC. 10\nD. 11\n\n", "subject": "计数", "analysis": "48 的因数有 $1,2,3,4,6,8,12,16,24,48$, 所以有 $1,2,3,4,6,8,12$, $16,24,48$ 这 10 种装法。"} {"id": "24071", "image": [], "answer": "B", "solution": "null", "level": "五年级", "question": "(2023・海淀区模拟) 聪聪用 $0 、 2 、 4 、 9$ 四张卡片摆出很多四位数, 他摆出的所有四位数是 ( )\n", "options": "A. 2 的倍数\nB. 3 的倍数\nC. 5 的倍数\nD. 无法确定\n", "subject": "计数", "analysis": "解摆出的四位数个位上的数字可以分别是 $0 、 2 、 4 、 9$; 当个位是 $2 、 4$ 时不是 5 的倍\n数; 当个位是 9 时既不是 2 的倍数也不是 5 的倍数; $0+2+4+9=15,15$ 是 3 的倍数, 所以组成的四位数一定是 3 的倍数。\n\n故选: $B$ 。"} {"id": "24167", "image": [], "answer": "B", "solution": "null", "level": "五年级", "question": "甲、乙两人玩掷骰子游戏, 下列游戏规则中, 公平的是()。\n", "options": "A. 质数甲赢, 合数乙赢\nB. 奇数甲赢,偶数乙赢\nC. 小于 4 的甲赢,大于 4 的乙赢\nD. 小于 3 的甲赢,大于 3 的乙赢\n", "subject": "计数", "analysis": "解$\\mathrm{B}$\n\n【分析】股子共有六个面, 标有六个数字: $1 、 2 、 3 、 4 、 5 、 6$; 找出每个选项要求的两种数, 哪种数的个数多, 赢的可能性就大, 游戏不公平; 两种数的个数相同, 赢的可能性一样大, 游戏公平。\n\n一个数, 如果只有 1 和它本身两个因数, 那么这样的数叫做质数;\n\n一个数, 如果除了 1 和它本身还有别的因数, 那么这样的数叫做合数;\n\n整数中, 是 2 的倍数的数叫做偶数, 不是 2 的倍数的数叫做奇数。\n\n【详解】A. 1〜6中, 质数有: $2 、 3 、 5$, 共 3 个; 合数有 $4 、 6$, 共 2 个;\n\n$3>2$, 甲赢的可能性比乙大, 游戏不公平;\n\nB. $1 \\sim 6$ 中, 奇数有: 1、3、5, 共 3 个; 偶数有 $2 、 4 、 6$, 共 3 个;\n\n甲、乙赢的可能性一样大, 游戏公平;\n\nC. 1 6 中, 小于 4 的数有: $1 、 2 、 3$, 共 3 个; 大于 4 的数有 $5 、 6$, 共 2 个; $3>2$, 甲赢的可能性比乙大, 游戏不公平;\n\nD. $1 \\sim 6$ 中, 小于 3 的数有: $1 、 2$, 共 2 个; 大于 3 的数有 $4 、 5 、 6$, 共 3 个; $2<3$, 乙赢的可能性比甲大, 游戏不公平。\n\n故答案为: B\n\n【点睛】本题考查根据可能性的大小确定游戏是否公平, 掌握质数与合数、奇数与偶数的意义是解题的关键。"} {"id": "24347", "image": [], "answer": "A", "solution": "null", "level": "五年级", "question": "(2022 秋・虎丘区期末)将 96 人进行分组, 其中(), 分的组数最多。\n", "options": "A. 每 4 人一组\nB. 每 6 人一组\nC. 每 8 人一组\n", "subject": "计数", "analysis": "解 $96 \\div 4=24$ (组)\n\n$96 \\div 6=16$ (组)\n\n$96 \\div 8=12$ (组)\n\n答: 将 96 人进行分组, 其中每 4 人一组, 分的组数最多。\n\n故选: $A$ 。"} {"id": "24393", "image": [], "answer": "C", "solution": "null", "level": "五年级", "question": "用 $1 、 2 、 4$ 这三个数字组成三位数, 组成的数是 2 的倍数的有 ( ) 个。\n", "options": "A. 2\nB. 3\nC. 4\nD. 6\n", "subject": "计数", "analysis": "解C\n\n【分析】一个数的个位上的数字是 $0 、 2 、 4 、 6 、 8$ 的数, 这样的数就是 2 的倍数, 据此解答即可。\n\n【详解】由分析可知:\n\n组成的数是 2 的倍数的三位数有: $124 、 142 、 214 、 412$ 共有 4 个。\n\n故答案为: C\n\n【点睛】本题考查 2 的倍数, 明确 2 的倍数特征是解题的关键。"} {"id": "24447", "image": [], "answer": "B", "solution": "null", "level": "五年级", "question": "(2020 春・通道县期中)把一根长 2 米的长方体木料锯成两段后,表面积增加了 100 平方厘米,它的体积是()\n", "options": "A. 200 立方厘米\nB. 10000 立方厘米\nC. 2 立方分米\nD. 1000 立方分米\n", "subject": "度量几何学", "analysis": "解 2 米 $=200$ 厘米\n\n$100 \\div 2 \\times 200$\n\n$=50 \\times 200$\n\n$=10000$ (立方厘米)\n\n答:它的体积是 10000 立方厘米。\n\n故选: $B$ 。"} {"id": "24474", "image": [], "answer": "A", "solution": "null", "level": "五年级", "question": "一个长方体水池, 长 10 米, 宽 8 米, 深 6 米, 占地()平方米。\n", "options": "A. 80\nB. 60\nC. 48\n", "subject": "度量几何学", "analysis": "解$\\mathrm{A}$\n\n【分析】求长方体水池的占地面积就是求长方体的底面积, 长方体水池的长是 10 米, 宽是 8 米, 利用“长方形的面积 $=$ 长 $\\times$ 宽”求出这个水池的占地面积, 据此解答。\n\n【详解】 $10 \\times 8=80$ (平方米)\n\n所以, 这个长方体水池的占地面积是 80 平方米。\n\n故答案为: $\\mathrm{A}$\n\n【点睛】理解求占地面积就是计算长方体的底面积是解答题目的关键。"} {"id": "24500", "image": [], "answer": "B", "solution": "null", "level": "五年级", "question": "一个长方体游泳池长 25 米, 宽 14 米, 高 2 米,它的占地面积是()平方米。\n", "options": "A. 50\nB. 350\nC. 28\nD. 856\n", "subject": "度量几何学", "analysis": "解B\n\n【分析】求游泳池的占地面积就是求长方体的底面积, 长方体的长是 25 米, 宽是 14 米, 利用“长方形的面积 $=$ 长 $\\times$ 宽”求出这个游泳池的占地面积, 据此解答。\n\n【详解】 $25 \\times 14=350$ (平方米)\n\n所以, 它的占地面积是 350 平方米。\n\n故答案为: B\n\n【点睛】理解求占地面积就是计算长方体的底面积是解答题目的关键。"} {"id": "24650", "image": [], "answer": "A", "solution": "null", "level": "五年级", "question": "一个长方体的长是 $8 \\mathrm{~cm}$, 宽是 $4 \\mathrm{~cm}$, 高是 $3 \\mathrm{~cm}$, 它的占地面积最大是 ( ) $\\mathrm{cm}^{2}$ 。\n", "options": "A. 32\nB. 12\nC. 24\nD. 15\n", "subject": "度量几何学", "analysis": "解A\n\n【分析】由题意可知, 要使该长方体的占地面积最大, 则用长方体最大的面积朝下即可, 最大的面也就是 $(8 \\times 4)$ 的面。\n\n【详解】 $8 \\times 4=32\\left(\\mathrm{~cm}^{2}\\right)$\n\n则它的占地面积最大是 $32 \\mathrm{~cm}^{2}$ 。\n\n故答案为: A\n\n【点睛】本题考查长方体的特征, 明确占地面积的定义是解题的关键。"} {"id": "24765", "image": ["12630.jpg", "12631.jpg", "12632.jpg"], "answer": "B", "solution": "null", "level": "五年级", "question": "把同样多的食盐溶解在下面三个长方体容器中,最咸的是()。\n\n\n\n(1)\n\n\n\n(2)\n\n\n\n(3)\n", "options": "A. 第(1)杯\nB. 第(2)杯\nC. 第(3)杯\nD. 无法确定\n\n", "subject": "度量几何学", "analysis": "解$\\mathrm{B}$\n【分析】加入同样多的食盐, 水越少越咸, 根据长方体体积 $=$ 长 $\\times$ 宽 $\\times$ 高, 分别求出三个容器中水的体积, 比较即可。\n\n【详解】 $12 \\times 6 \\times 3=216\\left(\\mathrm{~cm}^{3}\\right)$\n\n$14 \\times 5 \\times 3=210\\left(\\mathrm{~cm}^{3}\\right)$\n\n$10 \\times 7 \\times 4=280\\left(\\mathrm{~cm}^{3}\\right)$\n\n$210<216<280$\n\n故答案为: B\n\n【点睛】关键是掌握并灵活运用长方体体积公式。"} {"id": "24830", "image": [], "answer": "B", "solution": "null", "level": "五年级", "question": "小明对家里冰箱的描述正确的一项是()。\n", "options": "A. 高 1.5 米,容积 180 毫升, 重 60 千克\nB. 高 1.5 米, 容积 180 升, 重 60 千克\nC. 高 1.5 米, 容积 180 毫升, 重 60 克\nD. 高 1.5 米, 容积 180 升, 重 60 克\n", "subject": "度量几何学", "analysis": "解$\\mathrm{B}$\n\n【分析】根据对容积单位和数据大小的认识, 重量单位和数据大小的认识, 长度单位和数据大小的认识, 结合实际生活进行解答。\n\n【详解】小明对家里冰箱的描述正确的是高 1.5 米, 容积 180 升, 重 60 千克。\n\n故答案为: B\n\n【点睛】此题考查了容积单位、重量单位、长度单位与数据大小的认识, 关键是结合生活实际解答。"} {"id": "25079", "image": [], "answer": "B", "solution": "null", "level": "五年级", "question": "下面说法最合理的是 ( )。\n", "options": "A. 一座大桥限重 30 千克\nB. 数学书封面的长是 26 厘米\nC. 六年级同学跑 50 米最快用时 26 秒\nD. 一间仓库的容积是 300 立方分米\n", "subject": "度量几何学", "analysis": "解$\\mathrm{B}$\n\n【分析】据生活经验、对长度单位、重量单位、体积单位和时间单位和数据大小的认识, 可知计量一座大桥限重用“吨”作单位; 计量数学书封面的长用“厘米”作单位; 计量六年级同学跑 50 米用“秒”作单位; 计量一间仓库的容积用“立方分米”作单位。\n\n【详解】A. 一座大桥限重 30 吨, 原题干说法错误;\n\nB. 数学书封面的长是 26 厘米, 原题干说法正确;\n\nC. 六年级同学跑 50 米最快用时 26 秒, 26 秒不合实际, 与实际差别太大, 原题干说法错误;\n\nD. 一间仓库的容积是 300 立方米, 原题干说法错误。\n\n故答案为: B\n\n【点睛】此题考查根据情景选择合适的计量单位, 要注意联系生活实际、计量单位和数据的大小, 灵活的选择。"} {"id": "25106", "image": [], "answer": "C", "solution": "null", "level": "五年级", "question": "下面与 $90 \\mathrm{~L} 60 \\mathrm{~mL}$ 相等的是 ( ) L。\n", "options": "A. 9.06\nB. 9.6\nC. 90.06\nD. 90.6\n", "subject": "度量几何学", "analysis": "解$\\mathrm{C}$\n\n【分析】低级单位换高级单位除以进率,根据 $1 \\mathrm{~L}=1000 \\mathrm{~mL}$, 用 $60 \\div 1000$ 然后再加上 90 即可。\n\n【详解】 $60 \\div 1000+90$\n\n$=0.06+90$\n\n$=90.06(\\mathrm{~L})$\n\n故答案为: C\n\n【点睛】本题考查单位换算, 明确 $1 \\mathrm{~L}=1000 \\mathrm{~mL}$ 是解题的关键。"} {"id": "24151", "image": ["12352.jpg", "12353.jpg", "12354.jpg"], "answer": "A", "solution": "null", "level": "五年级", "question": "一张桌子上摆着若干个盘子,从三个方向看到的情况如图。这张桌子上共有()个盘子。\n\n\n\n\n从右面看\n\n\n", "options": "A. 12\nB. 13\nC. 14\nD. 17\n\n", "subject": "组合几何学", "analysis": "解A\n\n【分析】从上面看到的情况可知一共有三摞盘子, 由正面看到的情况可知右面一摞盘子的个数, 由右面看到的情况可知左面一列 2 摞盘子的个数。相加即可。\n\n【详解】 $3+5+4=12$ (个)\n\n这张桌子上共有 12 个盘子。\n\n故答案为: A\n\n【点睛】根据题干中的三视图判断出每摞盘子的个数是解题的关键。"} {"id": "24527", "image": ["12522.jpg", "12523.jpg", "12524.jpg", "12525.jpg", "12526.jpg", "12527.jpg"], "answer": "B", "solution": "null", "level": "五年级", "question": "下面各平面图中, 不能折成正方体的是()。\n", "options": "A.\n\n\nB.\n\n\nC.\n\n\n\n", "subject": "组合几何学", "analysis": "解B\n\n【分析】根据正方体展开图的类型, 1-4-1 型, 2-3-1 型, 2-2-2 型, 3-3 型, 据此判断解答即可。\n\n【详解】A.\n\n\n属于 1-4-1 型, 是正方体展开图类型, 能拼成正方体;\n\nB.\n\n\n\n不属于正方体展开图类型,不能拼成正方体;\n\n\n\n属于 2-2-2 型, 是正方体展开图类型, 能拼成正方体;\n\n故答案为: B\n\n【点睛】解决此题的关键是记住正方体展开图的基本类型。"} {"id": "24918", "image": [], "answer": "A", "solution": "null", "level": "五年级", "question": "哥德巴赫猜想被誉为“数学皇冠上的明珠”, 内容为“任何大于 2 的偶数都可以表示成两个质数的和”。下面符合这个猜想的算式是()。\n ", "options": "A. $56=13+23$\n B. $32=31+1$\n C. $18=2+16$\n D. $38=21+17$\n\n", "subject": "代数", "analysis": "解A\n\n【分析】能被 2 整除的数为偶数; 一个数( 0 除外)的因数只有 1 和它本身两个因数, 这样的数就是质数。据此解答即可。\n\n【详解】A. $56=13+23,56$ 是偶数, 13 和 23 都是质数, 符合题意。\n\nB. $32=31+1,32$ 是偶数, 1 不是质数, 不符合题意。\n\nC. $18=2+16,18$ 是偶数, 16 不是质数, 不符合题意。\n\nD. $38=21+17,38$ 是偶数, 21 不是质数, 不符合题意。\n\n故答案为: A\n\n【点睛】本题考查偶数和质数, 明确偶数和质数的定义是解题的关键。"} {"id": "12350", "image": ["2617.jpg"], "answer": "B", "solution": "null", "level": "五年级", "question": "旋转杆打开 (如下图所示) 的运动是绕 O 点 ( $\\quad$ )\n\n", "options": "A. 顺时针旋转 90 度\nB. 逆时针旋转 90 度\nC. 顺时针旋转 180 度", "subject": "变换几何", "analysis": "答案: $\\mathrm{B}$"} {"id": "12351", "image": ["2618.jpg"], "answer": "C", "solution": "null", "level": "五年级", "question": "如图, 指针按逆时针方向旋转 $180^{\\circ}$, 从 A 到 ( )。\n\n", "options": "A. A\nB. $B$\nC. C\nD. D", "subject": "变换几何", "analysis": "答案:C"} {"id": "12435", "image": [], "answer": "C", "solution": "null", "level": "五年级", "question": "用一根 ( ) cm 长的铁丝正好可以围成长 $6 \\mathrm{~cm}$ 、宽 $5 \\mathrm{~cm}$ 、高 $2 \\mathrm{~cm}$ 的长方体框架。", "options": "A. 26\nB. 117\nC. $\\quad 52$", "subject": "解析几何", "analysis": "答案: C$"} {"id": "11669", "image": [], "answer": "C", "solution": "null", "level": "五年级", "question": "在同一幅图上, 如果 $A$ 点用数对表示为 $(1,1), B$ 点用数对表示为 $(1$,\n\n4), $\\mathrm{C}$ 点用数对表示为( 3,1 ),那么三角形 $A B C$ 一定是()三角形。\nA .锐角\nB. 针角\nC. 直角", "options": "A. $(2,2)$\nB. $(5,4)$\nC. $(4,2)$", "subject": "解析几何", "analysis": "答案: C"} {"id": "11678", "image": [], "answer": "C", "solution": "null", "level": "五年级", "question": "下面数对中表示的位置和 $(3,5)$ 在同一行的是( )。", "options": "A. $(5,6)$\nB. $(6,3)$\nC. $(6,5)$", "subject": "解析几何", "analysis": "答案: C"} {"id": "11679", "image": [], "answer": "A", "solution": "null", "level": "五年级", "question": "小明的座位用数对表示是 $(4 , 3)$ ,小李的座位用数对表示是 $(6,3)$ ,下列说法正确的是()", "options": "A. 他们在同一行\nB. 他们不同列,也不同行\nC. 他们在同一列\nD. 他们既同列,又同行", "subject": "解析几何", "analysis": "答案: A"} {"id": "11747", "image": ["2974.jpg"], "answer": "A", "solution": "null", "level": "五年级", "question": "点 A (5.4) 向右移动三个单位后的位置是 ( )", "options": "A. $(5,7)$\nB. $(7,4)$\nC. $(8,4)$", "subject": "解析几何", "analysis": "答案:$\\mathrm{C}$\n\n【解析】略2.答案: $\\mathrm{B}$\n\n【分析】根据用数对表示点的位置的方法, 第一个数字表示列数, 第二个数字表示行数, 可在网格图中描出 $\\mathrm{A} 、 \\mathrm{~B} 、 \\mathrm{C}$ 三点, 然后连结成三角形, 根据这个三角形的形状即可确定按角分这个三角形属于什么三角形.\n\n【详解】如图:\n\n\n\n$\\triangle \\mathrm{ABC}$ 一定是直角三角形.\n\n故选: B."} {"id": "11754", "image": [], "answer": "B", "solution": "null", "level": "五年级", "question": "如果 A 点用数对表示为 $(2,5), \\mathrm{B}$ 点用数对表示为 $(2,1), \\mathrm{C}$ 点用数对表示为 $(4,1)$, 那么 $\\triangle \\mathrm{ABC}$ 一定是 $(\\quad)$ 三角形.", "options": "A. 锐角\nB. 直角\nC. 钝角", "subject": "解析几何", "analysis": ""} {"id": "11755", "image": ["2502.jpg"], "answer": "C", "solution": "null", "level": "五年级", "question": "在下图中, 如果的位置是 $(1,2)$, 则的位置是 ( ).\n\n", "options": "A. $(1,1)$\nB. $(1,3)$\nC. $(3,1)$", "subject": "解析几何", "analysis": "答案:$\\mathrm{C}$\n\n【分析】数对表示位置的方法是:第一个数字表示列, 第二个数字表示行, 由此即可解答问题。\n\n【解答】解: 的位置是 $(1,2)$, 是第 1 列第 2 行;\n\n位于第 3 列第 1 行,用数对表示为(3,1)。"} {"id": "11867", "image": ["2515.jpg"], "answer": "C", "solution": "null", "level": "五年级", "question": "如下图, 小亮从家到学校要穿过一个居民小区, 若小区的道路均沿南北或东西方向, 下面路线表示不正确的是( $\\quad )$ 。\n\n", "options": "A. $(1,4) \\rightarrow(1,1) \\rightarrow(4,1)$\nB. $(1,4) \\rightarrow(2,4) \\rightarrow(2,2) \\rightarrow(4,2) \\rightarrow(4,1)$\nC. $(1,4) \\rightarrow(3,4) \\rightarrow(4,2) \\rightarrow(4,1)$", "subject": "图论", "analysis": "答案: $\\mathrm{C}$"} {"id": "11691", "image": ["2480.jpg"], "answer": "A", "solution": "null", "level": "五年级", "question": "下图中转到()的可能性最大。\n\n", "options": "A. 偶数\nB. 小于 5 的数\nC. 质数\nD. 3 的倍数", "subject": "统计数学", "analysis": "答案:A"} {"id": "18438", "image": [], "answer": "$-2 \\sqrt{3}$", "solution": "null", "level": "高二", "question": "10. 在 $\\triangle A B C$ 中, $B C=1, B=\\frac{\\pi}{3}$, 当 $\\triangle A B C$ 的面积等于 $\\sqrt{3}$ 时, $\\tan C=$", "options": [], "subject": "解析几何", "analysis": "$S_{\\triangle A B C}=\\frac{1}{2} a c \\sin B=\\sqrt{3}, \\therefore c=4$. 由余弦定理得, $b^{2}=a^{2}+c^{2}-2 a c \\cos B=13$,\n\n$\\therefore \\cos C=\\frac{a^{2}+b^{2}-c^{2}}{2 a b}=-\\frac{1}{\\sqrt{13}}$, sin $C=\\frac{\\sqrt{12}}{\\sqrt{13}}$,\n\n$\\therefore \\tan C=-\\sqrt{12}=-2 \\sqrt{3}$."} {"id": "18441", "image": [], "answer": "$\\sqrt{3}$", "solution": "null", "level": "高二", "question": "13. 在 $\\triangle A B C$ 中, $A B=2, A C=\\sqrt{6}, B C=1+\\sqrt{3}, A D$ 为边 $B C$ 上的高,则 $A D$ 的长是", "options": [], "subject": "解析几何", "analysis": "$\\because \\cos C=\\frac{B C^{2}+A C^{2}-A B^{2}}{2 \\times B C \\times A C}=\\frac{\\sqrt{2}}{2}$,\n\n$\\therefore \\sin C=\\frac{\\sqrt{2}}{2}$.\n\n$\\therefore A D=A C \\cdot \\sin C=\\sqrt{3}$."} {"id": "18442", "image": [], "answer": "由余弦定理知\n\n$\\cos A=\\frac{b^{2}+c^{2}-a^{2}}{2 b c}, \\cos B=\\frac{a^{2}+c^{2}-b^{2}}{2 a c}$,\n\n$\\cos C=\\frac{a^{2}+b^{2}-c^{2}}{2 a b}$,\n\n代入已知条件得\n\n$a \\cdot \\frac{b^{2}+c^{2}-a^{2}}{2 b c}+b \\cdot \\frac{a^{2}+c^{2}-b^{2}}{2 a c}+c \\cdot \\frac{c^{2}-a^{2}-b^{2}}{2 a b}=0$,\n\n通分得 $a^{2}\\left(b^{2}+c^{2}-a^{2}\\right)+b^{2}\\left(a^{2}+c^{2}-b^{2}\\right)+c^{2}\\left(c^{2}-a^{2}-b^{2}\\right)=0$,\n\n展开整理得 $\\left(a^{2}-b^{2}\\right)^{2}=c^{4}$.\n\n$\\therefore a^{2}-b^{2}= \\pm c^{2}$, 即 $a^{2}=b^{2}+c^{2}$ 或 $b^{2}=a^{2}+c^{2}$.\n\n根据勾股定理知 $\\triangle A B C$ 是直角三角形.", "solution": "null", "level": "高二", "question": "14. 在 $\\triangle A B C$ 中, $a \\cos A+b \\cos B=c \\cos C$, 试判断三角形的形状.", "options": [], "subject": "解析几何", "analysis": "弦定理知\n\n$\\cos A=\\frac{b^{2}+c^{2}-a^{2}}{2 b c}, \\cos B=\\frac{a^{2}+c^{2}-b^{2}}{2 a c}$,\n\n$\\cos C=\\frac{a^{2}+b^{2}-c^{2}}{2 a b}$,\n\n代入已知条件得\n\n$a \\cdot \\frac{b^{2}+c^{2}-a^{2}}{2 b c}+b \\cdot \\frac{a^{2}+c^{2}-b^{2}}{2 a c}+c \\cdot \\frac{c^{2}-a^{2}-b^{2}}{2 a b}=0$,\n\n通分得 $a^{2}\\left(b^{2}+c^{2}-a^{2}\\right)+b^{2}\\left(a^{2}+c^{2}-b^{2}\\right)+c^{2}\\left(c^{2}-a^{2}-b^{2}\\right)=0$,\n\n展开整理得 $\\left(a^{2}-b^{2}\\right)^{2}=c^{4}$.\n\n$\\therefore a^{2}-b^{2}= \\pm c^{2}$, 即 $a^{2}=b^{2}+c^{2}$ 或 $b^{2}=a^{2}+c^{2}$.\n\n根据勾股定理知 $\\triangle A B C$ 是直角三角形."} {"id": "18448", "image": [], "answer": "$120^{\\circ}$", "solution": "null", "level": "高二", "question": "7. 在 $\\triangle A B C$ 中, 若 $a^{2}-b^{2}-c^{2}=b c$, 则 $A=$", "options": [], "subject": "解析几何", "analysis": "$120^{\\circ}$"} {"id": "18449", "image": [], "answer": "$30^{\\circ}$", "solution": "null", "level": "高二", "question": "8. $\\triangle A B C$ 中, 已知 $a=2, b=4, C=60^{\\circ}$, 则 $A=$", "options": [], "subject": "解析几何", "analysis": "$c^{2}=a^{2}+b^{2}-2 a b \\cos C$\n\n$=2^{2}+4^{2}-2 \\times 2 \\times 4 \\times \\cos 60^{\\circ}$\n\n$=12$\n\n$\\therefore c=2 \\sqrt{3}$.\n由正弦定理: $\\frac{a}{\\sin A}=\\frac{c}{\\sin C}$ 得 $\\sin A=\\frac{1}{2}$.\n\n$\\because a0, b>0)$, 则最大角为", "options": [], "subject": "解析几何", "analysis": "易知: $\\sqrt{a^{2}+a b+b^{2}}>a, \\sqrt{a^{2}+a b+b^{2}}>b$, 设最大角为 $\\theta$,\n\n则 $\\cos \\theta=\\frac{a^{2}+b^{2}-\\left(\\sqrt{a^{2}+a b+b^{2}}\\right)^{2}}{2 a b}=-\\frac{1}{2}$,\n\n$\\therefore \\theta=120^{\\circ}$."} {"id": "18462", "image": [], "answer": "$\\sqrt{19}$", "solution": "null", "level": "高二", "question": "7. 在 $\\triangle A B C$ 中, 边 $a, b$ 的长是方程 $x^{2}-5 x+2=0$ 的两个根, $C=60^{\\circ}$, 则边 $c=$", "options": [], "subject": "解析几何", "analysis": "由题意: $a+b=5, a b=2$.\n\n由余弦定理得: $c^{2}=a^{2}+b^{2}-2 a b \\cos C$\n\n$=a^{2}+b^{2}-a b=(a+b)^{2}-3 a b=5^{2}-3 \\times 2=19$,\n\n$\\therefore c=\\sqrt{19}$."} {"id": "18463", "image": [], "answer": "$20, \\therefore a>\\frac{1}{2}$, 最大边为 $2 a+1$.\n\n$\\because$ 三角形为钝角三角形, $\\therefore a^{2}+(2 a-1)^{2}<(2 a+1)^{2}$,\n\n化简得: $02 a+1$,\n\n$\\therefore a>2, \\quad \\therefore 20 , \\tan \\alpha<0$ ,\n\n$\\therefore \\sin \\alpha=\\sqrt{1-\\cos ^{2} \\alpha}=\\sqrt{1-\\left(-\\frac{3}{5}\\right)^{2}}=\\frac{4}{5}, \\tan \\alpha=\\frac{\\sin \\alpha}{\\cos \\alpha}=-\\frac{4}{3}$;\n\n当 $\\alpha$ 是第三象限角时, $\\sin \\alpha<0 , \\tan \\alpha>0$ ,\n\n$\\therefore \\sin \\alpha=-\\sqrt{1-\\cos ^{2} \\alpha}=-\\sqrt{1-\\left(-\\frac{3}{5}\\right)^{2}}=-\\frac{4}{5}$\n\n$\\tan \\alpha=\\frac{\\sin \\alpha}{\\cos \\alpha}=\\frac{4}{3}$.", "solution": "null", "level": "高二", "question": "10. 已知 $\\cos \\alpha=-\\frac{3}{5}$, 求 $\\sin \\alpha, \\tan \\alpha$ 的值.", "options": [], "subject": "解析几何", "analysis": "$\\because \\cos \\alpha=-\\frac{3}{5}<0 , \\therefore \\alpha$ 是第二或第三象限角.\n\n当 $\\alpha$ 是第二象限角时, $\\sin \\alpha>0 , \\tan \\alpha<0$ ,\n\n$\\therefore \\sin \\alpha=\\sqrt{1-\\cos ^{2} \\alpha}=\\sqrt{1-\\left(-\\frac{3}{5}\\right)^{2}}=\\frac{4}{5}, \\tan \\alpha=\\frac{\\sin \\alpha}{\\cos \\alpha}=-\\frac{4}{3}$;\n\n当 $\\alpha$ 是第三象限角时, $\\sin \\alpha<0 , \\tan \\alpha>0$ ,\n\n$\\therefore \\sin \\alpha=-\\sqrt{1-\\cos ^{2} \\alpha}=-\\sqrt{1-\\left(-\\frac{3}{5}\\right)^{2}}=-\\frac{4}{5}$\n\n$\\tan \\alpha=\\frac{\\sin \\alpha}{\\cos \\alpha}=\\frac{4}{3}$."} {"id": "17752", "image": [], "answer": "$\\frac{5}{3}$", "solution": "null", "level": "高二", "question": "11.直线 $2 x-y+1=0$ 的倾斜角为 $\\theta$, 则 $\\frac{1}{\\sin ^{2} \\theta-\\cos ^{2} \\theta}=$", "options": [], "subject": "解析几何", "analysis": "解析:由直线 $2 x-y+1=0$ 的倾斜角为 $\\theta$ ,得 $\\tan \\theta=2$ ,\n\n$\\therefore \\frac{1}{\\sin ^{2} \\theta-\\cos ^{2} \\theta}=\\frac{\\sin ^{2} \\theta+\\cos ^{2} \\theta}{\\sin ^{2} \\theta-\\cos ^{2} \\theta}=\\frac{\\tan ^{2} \\theta+1}{\\tan ^{2} \\theta-1}=\\frac{2^{2}+1}{2^{2}-1}=\\frac{5}{3}$."} {"id": "17753", "image": [], "answer": "2", "solution": "null", "level": "高二", "question": "14. 若 $\\cos \\alpha+2 \\sin \\alpha=-\\sqrt{5}$, 则 $\\tan \\alpha=$", "options": [], "subject": "解析几何", "analysis": "由 $\\cos \\alpha+2 \\sin \\alpha=-\\sqrt{5}$, 得 $\\cos ^{2} \\alpha+4 \\sin ^{2} \\alpha+4 \\sin \\alpha \\cos \\alpha=5\\left(\\cos ^{2} \\alpha+\\sin ^{2} \\alpha\\right)$ ,化简得 $\\sin ^{2} \\alpha$\n\n$-4 \\sin \\alpha \\cos \\alpha+4 \\cos ^{2} \\alpha=0$ ,即 $(\\sin \\alpha-2 \\cos \\alpha)^{2}=0$ ,则 $\\sin \\alpha=2 \\cos \\alpha$ ,故 $\\tan \\alpha=2$.\n\n答案: 2"} {"id": "17761", "image": [], "answer": "$\\pm \\frac{5}{12}$", "solution": "null", "level": "高二", "question": "7. 已知直线 $l$ 的倾斜角是 $\\theta$, 且 $\\sin \\theta=\\frac{5}{13}$, 则直线 $l$ 的斜率 $k=$", "options": [], "subject": "解析几何", "analysis": "解析:因为直线 $l$ 的倾斜角是 $\\theta$ ,所以 $\\theta \\in[0 , \\pi)$.\n\n因为 $\\sin \\theta=\\frac{5}{13}, \\sin ^{2} \\theta+\\cos ^{2} \\theta=1$ ,\n\n所以 $\\cos \\theta= \\pm \\sqrt{1-\\left(\\frac{5}{13}\\right)^{2}}= \\pm \\frac{12}{13}$ ,\n\n于是直线 $l$ 的斜率 $k=\\frac{\\sin \\theta}{\\cos \\theta}= \\pm \\frac{5}{12}$.\n\\pm \\frac{5}{12}$"} {"id": "17763", "image": [], "answer": "$-\\frac{12}{5}$", "solution": "null", "level": "高二", "question": "9. 已知 $\\sin \\alpha=\\frac{4-2 m}{m+5}, \\cos \\alpha=\\frac{m-3}{m+5}, \\alpha$ 是第四象限角, 求 $\\tan \\alpha$ 的值.", "options": [], "subject": "解析几何", "analysis": "$\\because \\sin ^{2} \\alpha+\\cos ^{2} \\alpha=1 , \\therefore\\left(\\frac{4-2 m}{m+5}\\right)^{2}+\\left(\\frac{m-3}{m+5}\\right)^{2}=1$.\n\n化简、整理,得 $m(m-8)=0$. 解得 $m=0$ 或 $m=8$.\n\n当 $m=0$ 时, $\\sin \\alpha=\\frac{4}{5} , \\cos \\alpha=-\\frac{3}{5}$ (此时 $\\alpha$ 不是第四象限角,故舍去);\n\n当 $m=8$ 时, $\\sin \\alpha=-\\frac{12}{13} , \\cos \\alpha=\\frac{5}{13} , \\therefore \\tan \\alpha=-\\frac{12}{5}$."} {"id": "17767", "image": [], "answer": "$\\sqrt{2}-2$", "solution": "null", "level": "高二", "question": "13. $\\frac{\\cos \\left(-585^{\\circ}\\right)}{\\sin 495^{\\circ}+\\sin \\left(-570^{\\circ}\\right)}$ 的值是", "options": [], "subject": "解析几何", "analysis": "解析 : $\\cos \\left(-585^{\\circ}\\right)=\\cos 225^{\\circ}=-\\cos 45^{\\circ}=-\\frac{\\sqrt{2}}{2}$,\n\n$\\sin 495^{\\circ}=\\sin 135^{\\circ}=\\frac{\\sqrt{2}}{2}$,\n\n$\\sin \\left(-570^{\\circ}\\right)=\\sin 150^{\\circ}=\\frac{1}{2}$,\n\n$\\therefore$ 原式 $=\\frac{-\\frac{\\sqrt{2}}{2}}{\\frac{\\sqrt{2}}{2}+\\frac{1}{2}}=\\frac{-\\sqrt{2}}{\\sqrt{2}+1}=\\sqrt{2}-2$."} {"id": "17775", "image": [], "answer": "2", "solution": "null", "level": "高二", "question": "6. $\\sin ^{2}(\\pi+\\alpha)-\\cos (\\pi+\\alpha) \\cdot \\cos (-\\alpha)+1=$", "options": [], "subject": "解析几何", "analysis": "解析 : 原式 $=(-\\sin \\alpha)^{2}-(-\\cos \\alpha) \\cdot \\cos \\alpha+1=\\sin ^{2} \\alpha+\\cos ^{2} \\alpha+1=2$."} {"id": "17787", "image": [], "answer": "$-\\frac{1}{3}$", "solution": "null", "level": "高二", "question": "6. 已知 $\\cos \\theta=\\frac{1}{3}$, 则 $\\sin \\left(\\theta-\\frac{\\pi}{2}\\right)=$", "options": [], "subject": "解析几何", "analysis": "解析 : $\\sin \\left(\\theta-\\frac{\\pi}{2}\\right)=-\\sin \\left(\\frac{\\pi}{2}-\\theta\\right)=-\\cos \\theta=-\\frac{1}{3}$."} {"id": "17788", "image": [], "answer": "$2 \\sqrt{2}$", "solution": "null", "level": "高二", "question": "7. 当 $\\theta=\\frac{5 \\pi}{4}$ 时, $\\frac{\\sin [\\theta+(2 k+1) \\pi]-\\sin [-\\theta-(2 k+1) \\pi]}{\\sin (\\theta+2 k \\pi) \\cos (\\theta-2 k \\pi)}(k \\in \\mathbf{Z})$ 的值等于", "options": [], "subject": "解析几何", "analysis": "解析 : 原式 $=\\frac{-\\sin \\theta-\\sin \\theta}{\\sin \\theta \\cos \\theta}=-\\frac{2}{\\cos \\theta}$.\n\n当 $\\theta=\\frac{5 \\pi}{4}$ 时, 原式 $=-\\frac{2}{\\cos \\frac{5 \\pi}{4}}=2 \\sqrt{2}$."} {"id": "17807", "image": [], "answer": ":$\\frac{13}{2}$", "solution": "null", "level": "高二", "question": "14. 设定义在 $\\mathbf{R}$ 上的函数 $f(x)$ 满足 $f(x) \\cdot f(x+2)=13$. 若 $f(1)=2$, 则 $f(99)=$", "options": [], "subject": "解析几何", "analysis": "由 $f(x) \\cdot f(x+2)=13$, 得 $f(x+2) \\cdot f(x+4)=13$,\n\n$\\therefore f(x)=f(x+4)$,\n\n$\\therefore T=4$,\n\n$\\therefore f(99)=f(24 \\times 4+3)=f(3)$.\n\n$\\because f(1) \\cdot f(3)=13, \\quad \\therefore f(3)=\\frac{13}{2}, \\quad \\therefore f(99)=\\frac{13}{2}$.\n答案: $\\frac{13}{2}$"} {"id": "17808", "image": [], "answer": "奇函数", "solution": "null", "level": "高二", "question": "15. 判断函数 $f(x)=\\ln \\left(\\sin x+\\sqrt{1+\\sin ^{2} x}\\right)$ 的奇偶性.", "options": [], "subject": "解析几何", "analysis": "$\\because \\sin x+\\sqrt{1+\\sin ^{2} x} \\geqslant \\sin x+1 \\geqslant 0$,\n\n若两处等号同时取到, 则 $\\sin x=0$ 且 $\\sin x=-1$ 矛盾,\n\n$\\therefore$ 对 $x \\in \\mathbf{R}$ 都有 $\\sin x+\\sqrt{1+\\sin ^{2} x}>0$.\n\n$\\because f(-x)=\\ln \\left(-\\sin x+\\sqrt{1+\\sin ^{2} x}\\right)$\n\n$=\\ln \\left(\\sqrt{1+\\sin ^{2} x}-\\sin x\\right)=\\ln \\left(\\sqrt{1+\\sin ^{2} x}+\\sin x\\right)^{-1}$\n\n$=-\\ln \\left(\\sin x+\\sqrt{1+\\sin ^{2} x}\\right)=-f(x)$,\n\n$\\therefore f(x)$ 为奇函数."} {"id": "17813", "image": [], "answer": "$\\frac{\\sqrt{2}}{2}$", "solution": "null", "level": "高二", "question": "6. 若函数 $f(x)$ 的定义域为 $\\mathbf{R}$, 最小正周期为 $\\frac{3 \\pi}{2}$, 且满足 $f(x)=\\left\\{\\begin{array}{ll}\\cos x, & -\\frac{\\pi}{2} \\leqslant x<0, \\\\ \\sin x, & 0 \\leqslant x<\\pi,\\end{array}\\right.$ 则 $f\\left(-\\frac{15 \\pi}{4}\\right)=$", "options": [], "subject": "解析几何", "analysis": "$\\because T=\\frac{3 \\pi}{2}, \\therefore f\\left(-\\frac{15}{4} \\pi\\right)=f\\left(-\\frac{15}{4} \\pi+\\frac{3}{2} \\pi \\times 3\\right)=f\\left(\\frac{3}{4} \\pi\\right)=\\sin \\frac{3}{4} \\pi=\\frac{\\sqrt{2}}{2}$.\n\n答案: $\\frac{\\sqrt{2}}{2}$"} {"id": "17826", "image": [], "answer": "$(-\\pi, 0]$", "solution": "null", "level": "高二", "question": "6. 函数 $y=\\cos x$ 在区间 $[-\\pi, a]$ 上为增函数, 则 $a$ 的取值范围是", "options": [], "subject": "解析几何", "analysis": "$\\because y=\\cos x$ 在 $[-\\pi, 0]$ 上是增函数, 在 $[0, \\pi]$ 上是减函数, $\\therefore$ 只有 $-\\pi
0)$ 的最小正周期为 $\\pi$, 为了得到函数 $g(x)=2 \\sin \\left(x+\\frac{\\pi}{4}\\right)$的图象, 只需将 $y=f(x)$ 的图象上各点的纵坐标变为原来的倍, 横坐标变为原来的 \\$ \\qquad \\$倍.", "options": [], "subject": "解析几何", "analysis": "由条件知 $\\omega=2$ ,所以只需将 $y=f(x)$ 的图象上各点的纵坐标变为原来的 2 倍,横坐标变为原来的 2 倍,即可得到 $y=g(x)$ 的图象,且两个变换没有先后顺序.\n\n答案: 2 2"} {"id": "17862", "image": [], "answer": "$\\frac{\\sqrt{3}}{2}$", "solution": "null", "level": "高二", "question": "12. 若函数 $f(x)=\\sin (\\omega x+\\varphi)\\left(\\omega>0\\right.$ 且 $\\left.|\\varphi|<\\frac{\\pi}{2}\\right)$ 在区间 $\\left[\\frac{\\pi}{6}, \\frac{2 \\pi}{3}\\right]$ 上是单调减函数, 且函数值从 1 减少到 -1 , 则 $f\\left(\\frac{\\pi}{4}\\right)$ 等于", "options": [], "subject": "解析几何", "analysis": "由题意知 $\\left\\{\\begin{array}{l}\\frac{\\pi}{6} \\omega+\\varphi=\\frac{\\pi}{2}+2 k \\pi \\\\ \\frac{2 \\pi}{3} \\omega+\\varphi=\\frac{3 \\pi}{2}+2 k \\pi\\end{array}, k \\in \\mathbf{Z}\\right.$ ,\n\n解之得 $\\omega=2 , \\varphi=\\frac{\\pi}{6}+2 k \\pi$ ,\n\n又因为 $|\\varphi|<\\frac{\\pi}{2}$ ,\n\n所以 $\\varphi=\\frac{\\pi}{6}$.\n\n所以 $f(x)=\\sin \\left(2 x+\\frac{\\pi}{6}\\right)$.\n\n所以 $f\\left(\\frac{\\pi}{4}\\right)=\\sin \\left(2 \\times \\frac{\\pi}{4}+\\frac{\\pi}{6}\\right)=\\cos \\frac{\\pi}{6}=\\frac{\\sqrt{3}}{2}$.\n答案: $\\frac{\\sqrt{3}}{2}$"} {"id": "17864", "image": [], "answer": "$\\left[k \\pi-\\frac{3 \\pi}{8}, k \\pi+\\frac{\\pi}{8}\\right], k \\in \\mathbf{Z}$", "solution": "null", "level": "高二", "question": "17.已知函数 $f(x)=-2 \\sin (2 x+\\varphi)(|\\varphi|<\\pi)$, 若 $f\\left(\\frac{\\pi}{8}\\right)=-2$, 则 $f(x)$ 的单调递减区间是", "options": [], "subject": "解析几何", "analysis": "由函数 $f(x)=-2 \\sin (2 x+\\varphi)(|\\varphi|<\\pi)$ ,且 $f\\left(\\frac{\\pi}{8}\\right)=-2$ ,故有 $-2 \\sin \\left(\\frac{\\pi}{4}+\\varphi\\right)=-2$ ,\n\n$\\therefore \\sin \\left(\\frac{\\pi}{4}+\\varphi\\right)=1, \\quad \\therefore \\varphi=\\frac{\\pi}{4}$,\n\n$\\therefore$ 函数 $f(x)=-2 \\sin \\left(2 x+\\frac{\\pi}{4}\\right)$.\n\n令 $2 k \\pi-\\frac{\\pi}{2} \\leqslant 2 x+\\frac{\\pi}{4} \\leqslant 2 k \\pi+\\frac{\\pi}{2}, k \\in \\mathbf{Z}$ ,\n\n得 $k \\pi-\\frac{3 \\pi}{8} \\leqslant x \\leqslant k \\pi+\\frac{\\pi}{8}, k \\in \\mathbf{Z}$.\n\n答案: $\\left[k \\pi-\\frac{3 \\pi}{8}, k \\pi+\\frac{\\pi}{8}\\right], k \\in \\mathbf{Z}$"} {"id": "18466", "image": [], "answer": "126", "solution": "null", "level": "高二", "question": "10. 在 $\\triangle A B C$ 中, $A=60^{\\circ}, a=6 \\sqrt{3}, b=12, S_{\\triangle A B C}=18 \\sqrt{3}$, 则 $\\frac{a+b+c}{\\sin A+\\sin B+\\sin C}=$", "options": [], "subject": "解析几何", "analysis": "frac{a+b+c}{\\sin A+\\sin B+\\sin C}=\\frac{a}{\\sin A}=\\frac{6 \\sqrt{3}}{\\frac{\\sqrt{3}}{2}}=12$.\n\n$\\because S_{\\triangle A B C}=\\frac{1}{2} a b \\sin C=\\frac{1}{2} \\times 6 \\sqrt{3} \\times 12 \\sin C=18 \\sqrt{3}$,\n\n$\\therefore \\sin C=\\frac{1}{2}, \\therefore \\frac{c}{\\sin C}=\\frac{a}{\\sin A}=12, \\quad \\therefore c=6$."} {"id": "18477", "image": [], "answer": "2", "solution": "null", "level": "高二", "question": "8. 在 $\\triangle A B C$ 中, 角 $A, B, C$ 的对边分别为 $a, b, c$, 已知 $A=60^{\\circ}, a=\\sqrt{3}, b=1$, 则 $c=$", "options": [], "subject": "解析几何", "analysis": "由正弦定理 $\\frac{a}{\\sin A}=\\frac{b}{\\sin B}$, 得 $\\frac{\\sqrt{3}}{\\sin 60^{\\circ}}=\\frac{1}{\\sin B}$,\n\n$\\therefore \\sin B=\\frac{1}{2}$, 故 $B=30^{\\circ}$ 或 $150^{\\circ}$. 由 $a>b$,\n\n得 $A>B, \\therefore B=30^{\\circ}$, 故 $C=90^{\\circ}$,\n\n由勾股定理得 $c=2$."} {"id": "18478", "image": [], "answer": "7", "solution": "null", "level": "高二", "question": "9. 在单位圆上有三点 $A, B, C$, 设 $\\triangle A B C$ 三边长分别为 $a, b, c$, 则 $\\frac{a}{\\sin A}+\\frac{b}{2 \\sin B}+\\frac{2 c}{\\sin C}$ $=$", "options": [], "subject": "解析几何", "analysis": "$\\because \\triangle A B C$ 的外接圆直径为 $2 R=2$,\n\n$\\therefore \\frac{a}{\\sin A}=\\frac{b}{\\sin B}=\\frac{c}{\\sin C}=2 R=2$,\n\n$\\therefore \\frac{a}{\\sin A}+\\frac{b}{2 \\sin B}+\\frac{2 c}{\\sin C}=2+1+4=7$."} {"id": "17896", "image": [], "answer": "$72^{\\circ}, 432^{\\circ}$", "solution": "null", "level": "高二", "question": "8.在 $0^{\\circ} \\sim 720^{\\circ}$ 中与 $\\frac{2 \\pi}{5}$ 终边相同的角为", "options": [], "subject": "解析几何", "analysis": "$\\because \\frac{2 \\pi}{5}=\\frac{2}{5} \\times 180^{\\circ}=72^{\\circ}$,\n\n$\\therefore$ 与角 $\\frac{2 \\pi}{5}$ 终边相同的角构成集合 $\\left\\{\\theta \\mid \\theta=72^{\\circ}+k \\cdot 360^{\\circ}, k \\in \\mathbf{Z}\\right\\}$.\n\n当 $k=0$ 时, $\\theta=72^{\\circ}$;\n\n当 $k=1$ 时, $\\theta=432^{\\circ}$\n\n$\\therefore$ 在 $0^{\\circ} \\sim 720^{\\circ}$ 范围内,\n\n与角 $\\frac{2 \\pi}{5}$ 终边相同的角为 $72^{\\circ}, 432^{\\circ}$.\n\n答案: $72^{\\circ}, 432^{\\circ}$."} {"id": "17910", "image": [], "answer": "$-\\sqrt{3}$", "solution": "null", "level": "高二", "question": "7. 若 $\\alpha$ 是第二象限角, $P(x, \\sqrt{5})$ 为其终边上一点, 且 $\\cos \\alpha=\\frac{\\sqrt{2}}{4} x$, 则 $x$ 的值为", "options": [], "subject": "解析几何", "analysis": "$\\because \\alpha$ 是第二象限角, $\\therefore x<0$. 又 $r=\\sqrt{x^{2}+5}$,\n\n$\\therefore \\cos \\alpha=\\frac{x}{r}=\\frac{x}{\\sqrt{x^{2}+5}}=\\frac{\\sqrt{2}}{4} x$, 解得 $x=-\\sqrt{3}$.\n\n答案: $-\\sqrt{3}$"} {"id": "17917", "image": [], "answer": "1", "solution": "null", "level": "高二", "question": "6. 在 $\\triangle A B C$ 中, $|\\overrightarrow{A B}|=|\\overrightarrow{B C}|=|\\overrightarrow{C A}|=1$, 则 $|\\overrightarrow{A B}-\\overrightarrow{A C}|$ 的值为", "options": [], "subject": "解析几何", "analysis": "1"} {"id": "17918", "image": [], "answer": "13", "solution": "null", "level": "高二", "question": "7. 已知 $\\overrightarrow{O A}=\\boldsymbol{a}, \\overrightarrow{O B}=\\boldsymbol{b}$, 若 $|\\overrightarrow{O A}|=12,|\\overrightarrow{O B}|=5$, 且 $\\angle A O B=90^{\\circ}$, 则 $|\\boldsymbol{a}-\\boldsymbol{b}|=$", "options": [], "subject": "解析几何", "analysis": "解析 : $|\\boldsymbol{a}-\\boldsymbol{b}|=\\sqrt{12^{2}+5^{2}}=13$."} {"id": "17919", "image": [], "answer": "(1)0 (2) $\\overrightarrow{A B}$", "solution": "null", "level": "高二", "question": "8. 化简: $(1) \\overrightarrow{P B}+\\overrightarrow{O P}-\\overrightarrow{O B}=$ \\$ \\qquad \\$\n(2) $\\overrightarrow{O B}-\\overrightarrow{O A}-\\overrightarrow{O C}-\\overrightarrow{C O}=$ \\$ \\qquad \\$ .", "options": [], "subject": "解析几何", "analysis": "(1)0 (2) $\\overrightarrow{A B}$"} {"id": "17924", "image": [], "answer": "$\\frac{1}{3}(\\boldsymbol{a}+\\boldsymbol{b}+\\boldsymbol{c})$", "solution": "null", "level": "高二", "question": "12. 设 $G$ 为 $\\triangle A B C$ 的重心, $O$ 为坐标原点, $\\overrightarrow{O A}=\\boldsymbol{a}, \\overrightarrow{O B}=\\boldsymbol{b}, \\overrightarrow{O C}=\\boldsymbol{c}$, 试用 $\\boldsymbol{a}, \\boldsymbol{b}, \\boldsymbol{c}$ 表示 $\\overrightarrow{O G}$,则 $\\overrightarrow{O G}=$", "options": [], "subject": "解析几何", "analysis": "$\\overrightarrow{O G}=\\overrightarrow{O C}+\\overrightarrow{C G}=\\overrightarrow{O C}+\\frac{1}{3}(\\overrightarrow{C A}+\\overrightarrow{C B})=\\overrightarrow{O C}+\\frac{1}{3}(\\overrightarrow{O A}-\\overrightarrow{O C}+\\overrightarrow{O B}-\\overrightarrow{O C})=\\frac{1}{3}(\\boldsymbol{a}+\\boldsymbol{b}+\\boldsymbol{c})$.\n\n答案: $\\frac{1}{3}(\\boldsymbol{a}+\\boldsymbol{b}+\\boldsymbol{c})$"} {"id": "17925", "image": ["9019.jpg"], "answer": "$-\\frac{2}{3} \\boldsymbol{e}_{1}+\\frac{5}{12} \\boldsymbol{e}_{2}$", "solution": "null", "level": "高二", "question": "13. 在平行四边形 $A B C D$ 中, $\\overrightarrow{A B}=\\boldsymbol{e}_{1}, \\overrightarrow{A C}=\\boldsymbol{e}_{2}, \\overrightarrow{N C}=\\frac{1}{4} \\overrightarrow{A C}, \\overrightarrow{B M}=\\frac{1}{2} \\overrightarrow{M C}$, 则 $\\overrightarrow{M N}=$ . (用 $\\boldsymbol{e}_{1}, \\boldsymbol{e}_{2}$ 表示)", "options": [], "subject": "解析几何", "analysis": "如图, $\\overrightarrow{M N}=\\overrightarrow{C N}-\\overrightarrow{C M}=\\overrightarrow{C N}+2 \\overrightarrow{B M}=\\overrightarrow{C N}+\\frac{2}{3} \\overrightarrow{B C}=-\\frac{1}{4} \\overrightarrow{A C}+\\frac{2}{3}(\\overrightarrow{A C}-\\overrightarrow{A B})=-\\frac{1}{4} e_{2}+\\frac{2}{3}\\left(\\boldsymbol{e}_{2}\\right.$\n$\\left.-\\boldsymbol{e}_{1}\\right)=-\\frac{2}{3} \\boldsymbol{e}_{1}+\\frac{5}{12} \\boldsymbol{e}_{2}$.\n\n\n\n答案: $-\\frac{2}{3} \\boldsymbol{e}_{1}+\\frac{5}{12} \\boldsymbol{e}_{2}$"} {"id": "17933", "image": [], "answer": "$\\frac{1}{2}$", "solution": "null", "level": "高二", "question": "7. 设 $D, E$ 分别是 $\\triangle A B C$ 的边 $A B, B C$ 上的点, $A D=\\frac{1}{2} A B, B E=\\frac{2}{3} B C$. 若 $\\overrightarrow{D E}=\\lambda_{1} \\overrightarrow{A B}+\\lambda_{2} \\overrightarrow{A C}\\left(\\lambda_{1}\\right.$, $\\lambda_{2}$ 为实数), 则 $\\lambda_{1}+\\lambda_{2}=$", "options": [], "subject": "解析几何", "analysis": "易知 $\\overrightarrow{D E}=\\frac{1}{2} \\overrightarrow{A B}+\\frac{2}{3} \\overrightarrow{B C}=\\frac{1}{2} \\overrightarrow{A B}+\\frac{2}{3}(\\overrightarrow{A C}-\\overrightarrow{A B})=-\\frac{1}{6} \\overrightarrow{A B}+\\frac{2}{3} \\overrightarrow{A C}$ ,所以 $\\lambda_{1}+\\lambda_{2}=\\frac{1}{2}$.\n\n答案: $\\frac{1}{2}$"} {"id": "17934", "image": ["9025.jpg", "9025.jpg"], "answer": "如图, $\\because \\overrightarrow{A B}=\\boldsymbol{e}_{2}$ ,且 $\\frac{D C}{A B}=k$ ,\n\n\n\n$\\therefore \\overrightarrow{D C}=k \\overrightarrow{A B}=k \\boldsymbol{e}_{2}$.\n\n又 $\\because \\overrightarrow{A B}+\\overrightarrow{B C}+\\overrightarrow{C D}+\\overrightarrow{D A}=0$ ,\n\n$\\therefore \\overrightarrow{B C}=-\\overrightarrow{A B}-\\overrightarrow{C D}-\\overrightarrow{D A}=-\\overrightarrow{A B}+\\overrightarrow{D C}+\\overrightarrow{A D}=-\\boldsymbol{e}_{2}+k \\boldsymbol{e}_{2}+\\boldsymbol{e}_{1}=\\boldsymbol{e}_{1}+(k-1) \\boldsymbol{e}_{2}$.\n\n$\\because \\overrightarrow{M N}+\\overrightarrow{N B}+\\overrightarrow{B A}+\\overrightarrow{A M}=0$,\n\n$\\therefore \\overrightarrow{M N}=-\\overrightarrow{N B}-\\overrightarrow{B A}-\\overrightarrow{A M}=\\overrightarrow{B N}+\\overrightarrow{A B}-\\overrightarrow{A M}=\\frac{1}{2} \\overrightarrow{B C}+\\boldsymbol{e}_{2}-\\frac{1}{2} \\overrightarrow{A D}=\\frac{1}{2}\\left[\\boldsymbol{e}_{1}+(k-1) \\boldsymbol{e}_{2}\\right]+\\boldsymbol{e}_{2}-\\frac{1}{2} \\boldsymbol{e}_{1}=$ $\\frac{k+1}{2} \\boldsymbol{e}_{2}$.", "solution": "null", "level": "高二", "question": "8. 在梯形 $A B C D$ 中, $A B / / C D, M, N$ 分别是 $D A, B C$ 的中点, 且 $\\frac{D C}{A B}=k(k \\neq 1)$. 设 $\\overrightarrow{A D}=\\boldsymbol{e}_{1}$, $\\overrightarrow{A B}=\\boldsymbol{e}_{2}$, 选择基底 $\\left\\{\\boldsymbol{e}_{1}, \\boldsymbol{e}_{2}\\right\\}$, 试写出下列向量在此基底下的分解式: $\\overrightarrow{D C}, \\overrightarrow{B C}, \\overrightarrow{M N}$.", "options": [], "subject": "解析几何", "analysis": "如图, $\\because \\overrightarrow{A B}=\\boldsymbol{e}_{2}$ ,且 $\\frac{D C}{A B}=k$ ,\n\n\n\n$\\therefore \\overrightarrow{D C}=k \\overrightarrow{A B}=k \\boldsymbol{e}_{2}$.\n\n又 $\\because \\overrightarrow{A B}+\\overrightarrow{B C}+\\overrightarrow{C D}+\\overrightarrow{D A}=0$ ,\n\n$\\therefore \\overrightarrow{B C}=-\\overrightarrow{A B}-\\overrightarrow{C D}-\\overrightarrow{D A}=-\\overrightarrow{A B}+\\overrightarrow{D C}+\\overrightarrow{A D}=-\\boldsymbol{e}_{2}+k \\boldsymbol{e}_{2}+\\boldsymbol{e}_{1}=\\boldsymbol{e}_{1}+(k-1) \\boldsymbol{e}_{2}$.\n\n$\\because \\overrightarrow{M N}+\\overrightarrow{N B}+\\overrightarrow{B A}+\\overrightarrow{A M}=0$,\n\n$\\therefore \\overrightarrow{M N}=-\\overrightarrow{N B}-\\overrightarrow{B A}-\\overrightarrow{A M}=\\overrightarrow{B N}+\\overrightarrow{A B}-\\overrightarrow{A M}=\\frac{1}{2} \\overrightarrow{B C}+\\boldsymbol{e}_{2}-\\frac{1}{2} \\overrightarrow{A D}=\\frac{1}{2}\\left[\\boldsymbol{e}_{1}+(k-1) \\boldsymbol{e}_{2}\\right]+\\boldsymbol{e}_{2}-\\frac{1}{2} \\boldsymbol{e}_{1}=$ $\\frac{k+1}{2} \\boldsymbol{e}_{2}$."} {"id": "17935", "image": ["9026.jpg", "9026.jpg"], "answer": "设 $\\overrightarrow{B M}=\\boldsymbol{a}, \\overrightarrow{C N}=\\boldsymbol{b}$ ,则 $\\overrightarrow{A M}=\\overrightarrow{A C}+\\overrightarrow{C M}=-\\boldsymbol{a}-3 \\boldsymbol{b} , \\overrightarrow{B N}=2 \\boldsymbol{a}+\\boldsymbol{b}$.\n\n$\\because A, P, M$ 和 $B, P, N$ 分别共线,\n\n$\\therefore$ 存在实数 $\\lambda, \\mu$ 使 $\\overrightarrow{A P}=\\lambda \\overrightarrow{A M}=-\\lambda \\boldsymbol{a}-3 \\lambda \\boldsymbol{b}$ ,\n\n$\\overrightarrow{B P}=\\mu \\overrightarrow{B N}=2 \\mu \\boldsymbol{a}+\\mu \\boldsymbol{b}$.\n\n$\\therefore \\overrightarrow{B A}=\\overrightarrow{B P}-\\overrightarrow{A P}=(\\lambda+2 \\mu) \\boldsymbol{a}+(3 \\lambda+\\mu) \\boldsymbol{b}$.\n\n又 $\\because \\overrightarrow{B A}=\\overrightarrow{B C}+\\overrightarrow{C A}=2 \\boldsymbol{a}+3 \\boldsymbol{b}$,\n\n\n\n解得 $\\left\\{\\begin{array}{l}\\lambda=\\frac{4}{5} } \\\\{\\mu=\\frac{3}{5},} \\end{array} \\text { 则 } \\overrightarrow{A P}=\\frac{4}{5} \\overrightarrow{A M} \\text {. }\\end{array}\\right.$\n\n$\\therefore A P$ 与 $A M$ 的比值为 $\\frac{4}{5}$.", "solution": "null", "level": "高二", "question": "9. 在 $\\triangle A B C$ 中, 点 $M$ 是 $B C$ 的中点, 点 $N$ 在 $A C$ 上且 $\\overrightarrow{A N}=2 \\overrightarrow{N C}, A M$ 交 $B N$ 于 $P$ 点, 求 $A P$与 $A M$ 的比值.", "options": [], "subject": "解析几何", "analysis": "设 $\\overrightarrow{B M}=\\boldsymbol{a}, \\overrightarrow{C N}=\\boldsymbol{b}$ ,则 $\\overrightarrow{A M}=\\overrightarrow{A C}+\\overrightarrow{C M}=-\\boldsymbol{a}-3 \\boldsymbol{b} , \\overrightarrow{B N}=2 \\boldsymbol{a}+\\boldsymbol{b}$.\n\n$\\because A, P, M$ 和 $B, P, N$ 分别共线,\n\n$\\therefore$ 存在实数 $\\lambda, \\mu$ 使 $\\overrightarrow{A P}=\\lambda \\overrightarrow{A M}=-\\lambda \\boldsymbol{a}-3 \\lambda \\boldsymbol{b}$ ,\n\n$\\overrightarrow{B P}=\\mu \\overrightarrow{B N}=2 \\mu \\boldsymbol{a}+\\mu \\boldsymbol{b}$.\n\n$\\therefore \\overrightarrow{B A}=\\overrightarrow{B P}-\\overrightarrow{A P}=(\\lambda+2 \\mu) \\boldsymbol{a}+(3 \\lambda+\\mu) \\boldsymbol{b}$.\n\n又 $\\because \\overrightarrow{B A}=\\overrightarrow{B C}+\\overrightarrow{C A}=2 \\boldsymbol{a}+3 \\boldsymbol{b}$,\n\n\n\n解得 $\\left\\{\\begin{array}{l}\\lambda=\\frac{4}{5} } \\\\{\\mu=\\frac{3}{5},} \\end{array} \\text { 则 } \\overrightarrow{A P}=\\frac{4}{5} \\overrightarrow{A M} \\text {. }\\end{array}\\right.$\n\n$\\therefore A P$ 与 $A M$ 的比值为 $\\frac{4}{5}$."} {"id": "17938", "image": [], "answer": "$(-3,6)$", "solution": "null", "level": "高二", "question": "11. 已知平面上三点 $A(2,-4), B(0,6), C(-8,10)$, 则 $\\frac{1}{2} \\overrightarrow{A C}-\\frac{1}{4} \\overrightarrow{B C}$ 的坐标是", "options": [], "subject": "解析几何", "analysis": "$(-3,6)$"} {"id": "17939", "image": [], "answer": "$\\frac{\\pi}{6}$ 或 $-\\frac{\\pi}{2}$", "solution": "null", "level": "高二", "question": "12. 已知 $A(2,3), B(1,4)$, 且 $\\frac{1}{2} \\overrightarrow{A B}=(\\sin \\alpha, \\cos \\beta), \\alpha, \\beta \\in\\left(-\\frac{\\pi}{2}, \\frac{\\pi}{2}\\right)$, 则 $\\alpha+\\beta=$", "options": [], "subject": "解析几何", "analysis": "$\\overrightarrow{A B}=(1,4)-(2,3)=(-1,1)$.\n$\\therefore(\\sin \\alpha, \\cos \\beta)=\\frac{1}{2}(-1,1)$ ,\n\n$\\therefore \\sin \\alpha=-\\frac{1}{2}, \\cos \\beta=\\frac{1}{2}$.\n\n又 $\\because \\alpha, \\beta \\in\\left(-\\frac{\\pi}{2}, \\frac{\\pi}{2}\\right)$,\n\n$\\therefore \\alpha=-\\frac{\\pi}{6} , \\beta=-\\frac{\\pi}{3}$ 或 $\\frac{\\pi}{3}, \\alpha+\\beta=-\\frac{\\pi}{2}$ 或 $\\frac{\\pi}{6}$.\n\n答案: $\\frac{\\pi}{6}$ 或 $-\\frac{\\pi}{2}$"} {"id": "17944", "image": [], "answer": "$(1,-1)(-4,-3)$", "solution": "null", "level": "高二", "question": "5. 若 $\\boldsymbol{a}+\\boldsymbol{b}=(-3,-4), \\boldsymbol{a}-\\boldsymbol{b}=(5,2)$, 则向量 $\\boldsymbol{a}=$ \\$ \\qquad \\$ ,向量 $\\boldsymbol{b}=$", "options": [], "subject": "解析几何", "analysis": "$\\boldsymbol{a}+\\boldsymbol{b}=(-3,-4)$, (1) $\\boldsymbol{a}-\\boldsymbol{b}=(5,2)$. (2)\n\n(1) + (2),得 $\\boldsymbol{a}=\\frac{1}{2}[(-3,-4)+(5,2)]=(1 ,-1)$ ;\n\n(1) - (2),得 $\\boldsymbol{b}=\\frac{1}{2}[(-3,-4)-(5,2)]=(-4,-3)$.\n\n答案: $(1,-1)(-4,-3)$"} {"id": "17945", "image": [], "answer": "-3", "solution": "null", "level": "高二", "question": "6. 已知向量 $\\boldsymbol{a}=(2,1), \\boldsymbol{b}=(1,-2)$. 若 $m \\boldsymbol{a}+n \\boldsymbol{b}=(9,-8)(m, n \\in \\mathbf{R})$, 则 $m-n$ 的值为 \\$ \\qquad \\$\n.", "options": [], "subject": "解析几何", "analysis": "解析 : 由题意得 $m \\boldsymbol{a}+n \\boldsymbol{b}=(2 m, m)+(n - 2 n)=(2 m+n, m-2 n)=(9 ,-8)$ ,即 $\\left\\{\\begin{array}{l}2 m+n=9 , \\\\ m-2 n=-8 ,\\end{array}\\right.$\n\n解得 $\\left\\{\\begin{array}{l}m=2 } \\\\{n=5 }\\end{array}\\end{array}\\right.$\n\n所以 $m-n=-3$"} {"id": "17956", "image": [], "answer": "10", "solution": "null", "level": "高二", "question": "5. 已知 $A, B, C$ 三点共线, $\\overrightarrow{B A}=-\\frac{3}{8} \\overrightarrow{A C}$, 点 $A, B$ 的纵坐标分别为 2,5 , 则点 $C$ 的纵坐标为", "options": [], "subject": "解析几何", "analysis": "设点 $C$ 的纵坐标为 $y . \\because A, B , C$ 三点共线, $\\overrightarrow{B A}=-\\frac{3}{8} \\overrightarrow{A C}, A , B$ 的纵坐标分别为 2 , $5, \\quad \\therefore 2-5=-\\frac{3}{8}(y-2), \\quad \\therefore y=10$\n答案: 10"} {"id": "17957", "image": ["9029.jpg"], "answer": "$(2,-4)$ 或 $(-2,4)$", "solution": "null", "level": "高二", "question": "6. 已知向量 $\\boldsymbol{a}=(1,-2),|\\boldsymbol{b}|=2 \\sqrt{5}$, 且 $\\boldsymbol{a} / / \\boldsymbol{b}$, 则 $\\boldsymbol{b}=$", "options": [], "subject": "解析几何", "analysis": "解析:\n\n\n\n- 4)或( - 2,4)."} {"id": "17958", "image": [], "answer": "$(8 ,-10)$", "solution": "null", "level": "高二", "question": "7. 已知 $\\boldsymbol{a}=\\overrightarrow{A B}$, 点 $B$ 的坐标为 $(1,0), \\boldsymbol{b}=(-3,4), \\boldsymbol{c}=(-1,1)$, 且 $\\boldsymbol{a}=3 \\boldsymbol{b}-2 \\boldsymbol{c}$, 求点 $A$ 的坐标.", "options": [], "subject": "解析几何", "analysis": "$\\because \\boldsymbol{b}=(-3,4), \\boldsymbol{c}=(-1,1), \\therefore 3 \\boldsymbol{b}-2 \\boldsymbol{c}=3(-3,4)-2(-1,1)=(-9,12)-(-2$ , $2)=(-7,10)$ ,即 $\\boldsymbol{a}=(-7,10)=\\overrightarrow{A B}$.\n\n又点 $B$ 的坐标为 $(1,0)$ ,设点 $A$ 的坐标为 $(x, y)$ ,则 $\\overrightarrow{A B}=(1-x, 0-y)=(-7,10)$ ,\n\n$\\therefore\\left\\{\\begin{array}{l}1-x=-7, \\\\ 0-y=10\\end{array} \\Rightarrow\\left\\{\\begin{array}{l}x=8, \\\\ y=-10,\\end{array}\\right.\\right.$\n\n即点 $A$ 的坐标为 $(8 ,-10)$."} {"id": "17965", "image": [], "answer": "$\\left(-\\infty, \\frac{-13-\\sqrt{133}}{6}\\right) \\cup\\left(\\frac{\\sqrt{133}-13}{6}, 1\\right) \\cup(1,+\\infty)$", "solution": "null", "level": "高二", "question": "14. 已知 $|\\boldsymbol{a}|=2,|\\boldsymbol{b}|=3, \\boldsymbol{a}$ 与 $\\boldsymbol{b}$ 的夹角为 $60^{\\circ}$. 若 $\\boldsymbol{a}+\\lambda \\boldsymbol{b}$ 与 $\\lambda \\boldsymbol{a}+\\boldsymbol{b}$ 的夹角为锐角, 则实数 $\\lambda$ 的取值范围为", "options": [], "subject": "解析几何", "analysis": "解析:由题意可得 $\\boldsymbol{a} \\cdot \\boldsymbol{b}=|\\boldsymbol{a}| \\boldsymbol{b} \\left\\lvert\\, \\cos 60^{\\circ}=2 \\times 3 \\times \\frac{1}{2}=3\\right.$.\n\n又 $\\because(\\boldsymbol{a}+\\lambda \\boldsymbol{b}) \\cdot(\\lambda \\boldsymbol{a}+\\boldsymbol{b})=\\lambda \\boldsymbol{a}^{2}+\\left(\\lambda^{2}+1\\right) \\boldsymbol{a} \\cdot \\boldsymbol{b}+\\lambda \\boldsymbol{b}^{2} ,$\n\n$\\boldsymbol{a}+\\lambda \\boldsymbol{b}$ 与 $\\lambda \\boldsymbol{a}+\\boldsymbol{b}$ 的夹角为锐角, $\\therefore \\lambda \\boldsymbol{a}^{2}+\\left(\\lambda^{2}+1\\right) \\boldsymbol{a} \\cdot \\boldsymbol{b}+\\lambda \\boldsymbol{b}^{2}>0$.\n\n$\\because \\boldsymbol{a}^{2}=|\\boldsymbol{a}|^{2}=4, \\quad \\boldsymbol{b}^{2}=|\\boldsymbol{b}|^{2}=9, \\quad \\boldsymbol{a} \\cdot \\boldsymbol{b}=3, \\quad \\therefore 3 \\lambda^{2}+13 \\lambda+3>0$,\n\n解得 $\\lambda>\\frac{\\sqrt{133}-13}{6}$ 或 $\\lambda<\\frac{-13-\\sqrt{133}}{6}$.\n\n当 $\\lambda=1$ 时, $\\boldsymbol{a}+\\lambda \\boldsymbol{b}$ 与 $\\lambda \\boldsymbol{a}+\\boldsymbol{b}$ 共线,其夹角不为锐角.\n\n故 $\\lambda$ 的取值范围是 $\\left(-\\infty, \\frac{-13-\\sqrt{133}}{6}\\right) \\cup\\left(\\frac{\\sqrt{133}-13}{6}, 1\\right) \\cup(1,+\\infty)$."} {"id": "17972", "image": [], "answer": "4", "solution": "null", "level": "高二", "question": "6. 已知 $\\boldsymbol{a} \\cdot \\boldsymbol{b}=16$, 若 $\\boldsymbol{a}$ 在 $\\boldsymbol{b}$ 方向上的投影为 4, 则 $|\\boldsymbol{b}|=$", "options": [], "subject": "解析几何", "analysis": "解析:设 $\\boldsymbol{a}$ 与 $\\boldsymbol{b}$ 的夹角为 $\\theta , \\because \\boldsymbol{a} \\cdot \\boldsymbol{b}=16 , \\therefore|\\boldsymbol{a}| \\boldsymbol{b} \\mid \\cos \\theta=16$.\n\n又 $\\because \\boldsymbol{a}$ 在 $\\boldsymbol{b}$ 方向上的投影为 $4 , \\therefore|\\boldsymbol{a}| \\cos \\theta=4 , \\therefore|\\boldsymbol{b}|=4$."} {"id": "17973", "image": [], "answer": "4", "solution": "null", "level": "高二", "question": "7. 若 $|\\boldsymbol{a}|=5, \\boldsymbol{a} \\cdot \\boldsymbol{b}=10$, 且 $\\boldsymbol{a}$ 与 $\\boldsymbol{b}$ 的夹角为 $60^{\\circ}$, 则 $|\\boldsymbol{b}|=$", "options": [], "subject": "解析几何", "analysis": "解析:因为 $\\boldsymbol{a} \\cdot \\boldsymbol{b}=10 ,|\\boldsymbol{a}|=5$ ,所以 $|\\boldsymbol{a}| \\cdot|\\boldsymbol{b}| \\cos 60^{\\circ}=10$ ,所以 $|\\boldsymbol{b}|=4$."} {"id": "17978", "image": [], "answer": "6", "solution": "null", "level": "高二", "question": "12. 已知向量 $\\boldsymbol{a}=(2,-4), \\boldsymbol{b}=(-3, m)$. 若 $|\\boldsymbol{a}||\\boldsymbol{b}|+\\boldsymbol{a} \\cdot \\boldsymbol{b}=0$, 则实数 $m=$\n\n.", "options": [], "subject": "解析几何", "analysis": "解析 : 由向量的数量积可知 $\\boldsymbol{a} \\cdot \\boldsymbol{b}=|\\boldsymbol{a}||\\boldsymbol{b}| \\cos \\theta$ ,又 $|\\boldsymbol{a}||\\boldsymbol{b}|+\\boldsymbol{a} \\cdot \\boldsymbol{b}=0$ ,所以 $\\cos \\theta=-1$ ,所以 $\\theta=\\pi$ ,\n即向量 $\\boldsymbol{a}=(2 ,-4)$ 与 $\\boldsymbol{b}=(-3, m)$ 的方向相反. 设 $\\boldsymbol{a}=\\lambda \\boldsymbol{b}$, 即 $(2 ,-4)=\\lambda(-3 , m)$ 可得 $\\left\\{\\begin{array}{l}2=-3 \\lambda , \\\\ -4=\\lambda m ,\\end{array}\\right.$ 解得 $m=6$"} {"id": "17979", "image": ["9033.jpg"], "answer": "$\\left[\\frac{1}{2}, \\frac{3}{2}\\right]$", "solution": "null", "level": "高二", "question": "13. 在边长为 1 的正方形 $A B C D$ 中, $M$ 为 $B C$ 的中点, 点 $E$ 在线段 $A B$ 上运动, 则 $\\overrightarrow{E C} \\cdot \\overrightarrow{E M}$ 的取值范围是", "options": [], "subject": "解析几何", "analysis": "将正方形放入如图所示的平面直角坐标系中,设 $E(x, 0) , 0 \\leqslant x \\leqslant 1$.\n\n\n\n因为 $M\\left(1, \\frac{1}{2}\\right), C(1,1)$, 所以 $\\overrightarrow{E M}=\\left(1-x, \\frac{1}{2}\\right), \\overrightarrow{E C}=(1-x, 1)$, 所以 $\\overrightarrow{E M} \\cdot \\overrightarrow{E C}=\\left(1-x, \\frac{1}{2}\\right) \\cdot(1$ $-x, 1)=(1-x)^{2}+\\frac{1}{2}$. 因为 $0 \\leqslant x \\leqslant 1$ ,所以 $\\frac{1}{2} \\leqslant(1-x)^{2}+\\frac{1}{2} \\leqslant \\frac{3}{2}$ ,即 $\\overrightarrow{E M} \\cdot \\overrightarrow{E C}$ 的取值范围是 $\\left[\\frac{1}{2} , \\frac{3}{2}\\right]$.答案: $\\left[\\frac{1}{2}, \\frac{3}{2}\\right]$"} {"id": "17985", "image": [], "answer": "$120^{\\circ}$", "solution": "null", "level": "高二", "question": "6. 已知向量 $\\boldsymbol{a}=(1,2), \\boldsymbol{b}=(-2,-4),|\\boldsymbol{c}|=\\sqrt{5}$. 若 $(\\boldsymbol{a}+\\boldsymbol{b}) \\cdot \\boldsymbol{c}=\\frac{5}{2}$, 则 $\\boldsymbol{a}$ 与 $\\boldsymbol{c}$ 的夹角的大小为", "options": [], "subject": "解析几何", "analysis": "设 $\\boldsymbol{a}$ 与 $\\boldsymbol{c}$ 的夹角为 $\\theta$ ,由 $\\boldsymbol{a}+\\boldsymbol{b}=(-1,-2)=-\\boldsymbol{a},|\\boldsymbol{a}|=\\sqrt{5}, \\cos \\theta=\\frac{\\boldsymbol{a} \\cdot \\boldsymbol{c}}{|\\boldsymbol{a}| \\boldsymbol{c} \\mid}=\\frac{-(\\boldsymbol{a}+\\boldsymbol{b}) \\cdot \\boldsymbol{c}}{\\sqrt{5} \\cdot \\sqrt{5}}$ $=\\frac{-\\frac{5}{2}}{5}=-\\frac{1}{2}$,\n\n$\\therefore \\theta=120^{\\circ}$.\n\n答案: $120^{\\circ}$"} {"id": "17986", "image": [], "answer": "2", "solution": "null", "level": "高二", "question": "7. 已知向量 $\\boldsymbol{a}=(1, \\sqrt{3})$, 向量 $\\boldsymbol{a}, \\boldsymbol{c}$ 的夹角是 $\\frac{\\pi}{3}, \\boldsymbol{a} \\cdot \\boldsymbol{c}=2$, 则 $\\mid \\boldsymbol{c}$ 等于", "options": [], "subject": "解析几何", "analysis": "解析:因为 $|\\boldsymbol{a}|=2 , \\boldsymbol{a} \\cdot \\boldsymbol{c}=2$ ,所以 $|\\boldsymbol{a}| \\cdot|\\boldsymbol{c}| \\cos 60^{\\circ}=2$ ,得 $\\boldsymbol{c} \\mid=2$."} {"id": "17987", "image": [], "answer": "$\\pm 2$", "solution": "null", "level": "高二", "question": "8. 已知 $\\triangle A B C$ 中, $|\\overrightarrow{A B}|=4,|\\overrightarrow{A C}|=1, S_{\\triangle A B C}=\\sqrt{3}$, 则 $\\overrightarrow{A B} \\cdot \\overrightarrow{A C}$ 的值为", "options": [], "subject": "解析几何", "analysis": "因为 $S_{\\triangle A B C}=\\frac{1}{2} \\times 4 \\times 1 \\times \\sin A=\\sqrt{3}$ ,所以 $\\sin A=\\frac{\\sqrt{3}}{2}$ ,得 $A=\\frac{\\pi}{3}$ 或 $A=\\frac{2 \\pi}{3} , \\overrightarrow{A B} \\cdot \\overrightarrow{A C}=$ $1 \\times 4 \\times \\cos A= \\pm 2$.\n\n答案: $\\pm 2$"} {"id": "18007", "image": ["9042.jpg"], "answer": "(1)(2)(3)", "solution": "null", "level": "高二", "question": "14. 设 $O$ 是正方形 $A B C D$ 的中心, 则(1) $\\overrightarrow{A O}=\\overrightarrow{O C}$; (2) $\\overrightarrow{A O} / / \\overrightarrow{A C}$; (3) $\\overrightarrow{A B}$ 与 $\\overrightarrow{C D}$ 共线; (4) $\\overrightarrow{A O}=\\overrightarrow{B O}$. 其中, 所有正确结论的序号为 \\$ \\qquad \\$ .\n\n", "options": [], "subject": "解析几何", "analysis": "正方形的对角线互相平分, 则 $\\overrightarrow{A O}=\\overrightarrow{O C}$, (1)正确; $\\overrightarrow{A O}$ 与 $\\overrightarrow{A C}$ 的方向相同, 所以 $\\overrightarrow{A O} / / \\overrightarrow{A C}$, (2)正确; $\\overrightarrow{A B}$ 与 $\\overrightarrow{C D}$ 的方向相反, 所以 $\\overrightarrow{A B}$ 与 $\\overrightarrow{C D}$ 共线, (3)正确; 尽管 $|\\overrightarrow{A O}|=|\\overrightarrow{B O}|$, 然而 $\\overrightarrow{A O}$ 与 $\\overrightarrow{B O}$ 的方向不相同, 所以 $\\overrightarrow{A O} \\neq \\overrightarrow{B O}$, (4)不正确.\n\n答案: (1)(2)(3)"} {"id": "18020", "image": [], "answer": "梯形", "solution": "null", "level": "高二", "question": "12. 在四边形 $A B C D$ 中, $\\overrightarrow{A B}=\\boldsymbol{a}+2 \\boldsymbol{b}, \\overrightarrow{B C}=-4 \\boldsymbol{a}-\\boldsymbol{b}, \\overrightarrow{C D}=-5 \\boldsymbol{a}-3 \\boldsymbol{b}$, 则四边形 $A B C D$ 的形状是", "options": [], "subject": "解析几何", "analysis": "$\\because \\overrightarrow{A D}=\\overrightarrow{A B}+\\overrightarrow{B C}+\\overrightarrow{C D}=(\\boldsymbol{a}+2 \\boldsymbol{b})+(-4 \\boldsymbol{a}-\\boldsymbol{b})+(-5 \\boldsymbol{a}-3 \\boldsymbol{b})=-8 \\boldsymbol{a}-2 \\boldsymbol{b}=2 \\overrightarrow{B C}$ , $\\therefore A D / / B C$ ,且 $A D=2 B C . \\therefore$ 四边形 $A B C D$ 是梯形.\n\n答案: 梯形"} {"id": "18029", "image": [], "answer": "$\\frac{2}{3}$", "solution": "null", "level": "高二", "question": "7. 已知 $\\overrightarrow{A D}=\\frac{2}{3} \\overrightarrow{A B}, \\overrightarrow{A E}=\\frac{2}{3} \\overrightarrow{A C}$, 则 $\\overrightarrow{D E}=$ \\$ \\qquad \\$ $\\overrightarrow{B C}$.", "options": [], "subject": "解析几何", "analysis": "$\\overrightarrow{D E}=\\overrightarrow{D A}+\\overrightarrow{A E}=-\\overrightarrow{A D}+\\overrightarrow{A E}=-\\frac{2}{3} \\overrightarrow{A B}+\\frac{2}{3} \\overrightarrow{A C}=\\frac{2}{3}(\\overrightarrow{A C}-\\overrightarrow{A B})=\\frac{2}{3} \\overrightarrow{B C}$.\n\n答案: $\\frac{2}{3}$"} {"id": "18035", "image": [], "answer": "$-\\frac{7}{9}$", "solution": "null", "level": "高二", "question": "13. 在平面直角坐标系 $x O y$ 中, 角 $\\alpha$ 与角 $\\beta$ 均以 $O x$ 为始边, 它们的终边关于 $y$ 轴对称. 若 $\\sin$\n$\\alpha=\\frac{1}{3}$, 则 $\\cos (\\alpha-\\beta)=$ \\$ \\qquad \\$ .", "options": [], "subject": "解析几何", "analysis": "由题意知 $\\alpha+\\beta=\\pi+2 k \\pi(k \\in \\mathbf{Z})$ ,\n\n$\\therefore \\beta=\\pi+2 k \\pi-\\alpha(k \\in \\mathbf{Z}), \\sin \\beta=\\sin \\alpha, \\cos \\beta=-\\cos \\alpha$.\n\n又 $\\sin \\alpha=\\frac{1}{3}$ ,\n\n$\\therefore \\cos (\\alpha-\\beta)=\\cos \\alpha \\cos \\beta+\\sin \\alpha \\sin \\beta$\n\n$=-\\cos ^{2} \\alpha+\\sin ^{2} \\alpha=2 \\sin ^{2} \\alpha-1$\n\n$=2 \\times \\frac{1}{9}-1=-\\frac{7}{9}$.\n\n答案: $-\\frac{7}{9}$"} {"id": "18036", "image": [], "answer": "$\\frac{\\pi}{4}$", "solution": "null", "level": "高二", "question": "14. 已知 $\\alpha, \\beta$ 均为锐角, 且 $\\sin \\alpha=\\frac{2 \\sqrt{5}}{5}, \\sin \\beta=\\frac{\\sqrt{10}}{10}$, 则 $\\alpha-\\beta=$ \\$ \\qquad \\$ .", "options": [], "subject": "解析几何", "analysis": "$\\because \\alpha, \\beta$ 均为锐角, $\\therefore \\cos \\alpha=\\frac{\\sqrt{5}}{5} , \\cos \\beta=\\frac{3 \\sqrt{10}}{10}$.\n\n$\\therefore \\cos (\\alpha-\\beta)=\\cos \\alpha \\cos \\beta+\\sin \\alpha \\sin \\beta=\\frac{\\sqrt{5}}{5} \\times \\frac{3 \\sqrt{10}}{10}+\\frac{2 \\sqrt{5}}{5} \\times \\frac{\\sqrt{10}}{10}=\\frac{\\sqrt{2}}{2}$.\n\n又 $\\because \\sin \\alpha>\\sin \\beta, \\quad \\therefore 0<\\beta<\\alpha<\\frac{\\pi}{2}$ ,\n\n$\\therefore 0<\\alpha-\\beta<\\frac{\\pi}{2}$ ,故 $\\alpha-\\beta=\\frac{\\pi}{4}$.\n\n## 答案: $\\frac{\\pi}{4}$"} {"id": "18452", "image": [], "answer": "$\\frac{13 \\pi}{3}$", "solution": "null", "level": "高二", "question": "10. 在 $\\triangle A B C$ 中, $A=60^{\\circ}, b=1, S_{\\triangle A B C}=\\sqrt{3}$, 则 $\\triangle A B C$ 外接圆的面积是", "options": [], "subject": "立体几何学", "analysis": "_{\\triangle A B C}=\\frac{1}{2} b c \\sin A=\\frac{\\sqrt{3}}{4} c=\\sqrt{3}$,\n\n$\\therefore c=4$,\n\n由余弦定理: $a^{2}=b^{2}+c^{2}-2 b c \\cos A$\n\n$=1^{2}+4^{2}-2 \\times 1 \\times 4 \\cos 60^{\\circ}=13$,\n\n$\\therefore a=\\sqrt{13}$.\n\n$\\therefore 2 R=\\frac{a}{\\sin A}=\\frac{\\sqrt{13}}{\\frac{\\sqrt{3}}{2}}=\\frac{2 \\sqrt{39}}{3}$,\n\n$\\therefore R=\\frac{\\sqrt{39}}{3} . \\therefore S_{\\text {外茹团 }}=\\pi R^{2}=\\frac{13 \\pi}{3}$."} {"id": "18005", "image": [], "answer": "$3 \\pi$", "solution": "null", "level": "高二", "question": "11. 把同一平面内所有模不小于 1 , 不大于 2 的向量的起点, 移到同一点 $O$, 则这些向量的终点构成的图形的面积等于 \\$ \\qquad \\$", "options": [], "subject": "立体几何学", "analysis": "这些向量的终点构成的图形是一个圆环, 其面积为 $\\pi \\times 2^{2}-\\pi \\times 1^{2}=3 \\pi$.\n\n答案: $3 \\pi$"} {"id": "18015", "image": [], "answer": "外", "solution": "null", "level": "高二", "question": "7. 设点 $O$ 是 $\\triangle A B C$ 所在平面上一点, 若 $|\\overrightarrow{O A}|=|\\overrightarrow{O B}|=|\\overrightarrow{O C}|$, 则点 $O$ 是 $\\triangle A B C$ 的()心.", "options": [], "subject": "立体几何学", "analysis": "外"} {"id": "17746", "image": [], "answer": "<", "solution": "null", "level": "高二", "question": "7. 比较大小: $\\sin 1 .2$ $\\sin 1 .5$ (填“〉”或““”).", "options": [], "subject": "算术", "analysis": "<"} {"id": "17762", "image": [], "answer": "1", "solution": "null", "level": "高二", "question": "8. 化简: $\\left(1+\\tan ^{2} \\alpha\\right)\\left(1-\\sin ^{2} \\alpha\\right)=$", "options": [], "subject": "算术", "analysis": "解析 : $\\left(1+\\tan ^{2} \\alpha\\right)\\left(1-\\sin ^{2} \\alpha\\right)=\\left(1+\\frac{\\sin ^{2} \\alpha}{\\cos ^{2} \\alpha}\\right) \\cdot \\cos ^{2} \\alpha=\\frac{\\cos ^{2} \\alpha+\\sin ^{2} \\alpha}{\\cos ^{2} \\alpha} \\cdot \\cos ^{2} \\alpha=1$."} {"id": "17768", "image": [], "answer": "$b>a>c$", "solution": "null", "level": "高二", "question": "14. 已知 $a=\\tan \\left(-\\frac{7 \\pi}{6}\\right), b=\\cos \\frac{23 \\pi}{4}, c=\\sin \\left(-\\frac{33 \\pi}{4}\\right)$, 则 $a, b, c$ 的大小关系是", "options": [], "subject": "算术", "analysis": "$b>a>c$"} {"id": "17780", "image": [], "answer": "$\\frac{91}{2}$", "solution": "null", "level": "高二", "question": "14. $\\sin ^{2} 1^{\\circ}+\\sin ^{2} 2^{\\circ}+\\sin ^{2} 3^{\\circ}+\\cdots+\\sin ^{2} 88^{\\circ}+\\sin ^{2} 89^{\\circ}+\\sin ^{2} 90^{\\circ}$ 的值为", "options": [], "subject": "算术", "analysis": "解析 : $\\because \\sin ^{2} 1^{\\circ}+\\sin ^{2} 89^{\\circ}=\\sin ^{2} 1^{\\circ}+\\cos ^{2} 1^{\\circ}=1$,\n\n$\\sin ^{2} 2^{\\circ}+\\sin ^{2} 88^{\\circ}=\\sin ^{2} 2^{\\circ}+\\cos ^{2} 2^{\\circ}=1$,\n\n$\\sin ^{2} x^{\\circ}+\\sin ^{2}\\left(90^{\\circ}-x^{\\circ}\\right)=\\sin ^{2} x^{\\circ}+\\cos ^{2} x^{\\circ}=1(1 \\leqslant x \\leqslant 44, x \\in \\mathbf{N})$,\n\n$\\therefore$ 原式 $=\\left(\\sin ^{2} 1^{\\circ}+\\sin ^{2} 89^{\\circ}\\right)+\\left(\\sin ^{2} 2^{\\circ}+\\sin ^{2} 88^{\\circ}\\right)+\\cdots+\\left(\\sin ^{2} 44^{\\circ}+\\sin ^{2} 46^{\\circ}\\right)+\\sin ^{2} 90^{\\circ}+\\sin ^{2} 45^{\\circ}=$\n\n$45+\\left(\\frac{\\sqrt{2}}{2}\\right)^{2}=\\frac{91}{2}$."} {"id": "17841", "image": [], "answer": "$(1, \\sqrt{3})$", "solution": "null", "level": "高二", "question": "7. 函数 $y=\\tan \\left(\\frac{x}{2}+\\frac{\\pi}{4}\\right), x \\in\\left(0, \\frac{\\pi}{6}\\right)$ 的值域是", "options": [], "subject": "算术", "analysis": "$\\because x \\in\\left(0 , \\frac{\\pi}{6}\\right) : \\therefore \\frac{x}{2}+\\frac{\\pi}{4} \\in\\left(\\frac{\\pi}{4}, \\frac{\\pi}{3}\\right)$ ,\n\n$\\therefore \\tan \\left(\\frac{x}{2}+\\frac{\\pi}{4}\\right) \\in(1, \\sqrt{3})$.\n\n答案: $(1, \\sqrt{3})$"} {"id": "17909", "image": [], "answer": "$\\frac{1}{2}$", "solution": "null", "level": "高二", "question": "6. $\\sin 750^{\\circ}=$", "options": [], "subject": "算术", "analysis": "$\\frac{1}{2}$"} {"id": "19706", "image": [], "answer": "$5+2 \\sqrt{6}$", "solution": "null", "level": "高二", "question": "已知实数 $\\mathrm{a}>0, \\mathrm{~b}>0, \\sqrt{2}$ 是 $8^{\\mathrm{a}}$ 与 $2^{\\mathrm{b}}$ 的等比中项, 则 $\\frac{1}{\\mathrm{a}}+\\frac{2}{\\mathrm{~b}}$ 的最小值是", "options": [], "subject": "算术", "analysis": "实数 $\\mathrm{a}>0, \\mathrm{~b}>0, \\sqrt{2}$ 是 $8^{\\mathrm{a}}$ 与 $2^{\\mathrm{b}}$ 的等比中项, $\\therefore 8^{a} \\cdot 2^{b}=2, \\therefore 2^{3 \\mathrm{a}+\\mathrm{b}}=2$, 解得 $3 \\mathrm{a}+\\mathrm{b}=1$. 则 $\\frac{1}{a}+\\frac{2}{b}=(3 a+b)\\left(\\frac{1}{a}+\\frac{2}{b}\\right)=5+\\frac{b}{a}+\\frac{6 a}{b} \\geq 5+2 \\sqrt{\\frac{b}{a} \\cdot \\frac{6 a}{b}}=5+2 \\sqrt{6}$, 当且仅当 $b=\\sqrt{6}, a=\\sqrt{6}-2$ 时取等号. 故答案为 $5+2 \\sqrt{6}$."} {"id": "19728", "image": [], "answer": "$\\frac{2^{n}}{2 n+1}$", "solution": "null", "level": "高二", "question": "数列 $\\frac{2}{3}, \\frac{4}{5}, \\frac{8}{7}, \\frac{16}{9}, \\cdots$ 的一个通项公式是", "options": [], "subject": "算术", "analysis": "分母是以 3 开头的奇数列, 分子是以 2 为首项, 以 2 为公比的等比数列,$\\therefore \\mathrm{a}_{\\mathrm{n}}=\\frac{2^{n}}{2 n+1}$, 故答案为: $\\frac{2^{n}}{2 n+1}$"} {"id": "19740", "image": [], "answer": "$\\frac{4}{5}$", "solution": "null", "level": "高二", "question": "设数列为公差不为零的等差数列, $2 a_{7}-a_{8}=0$, 则 $\\frac{a_{2}}{a_{1}}=$ \\$ \\qquad \\$ .", "options": [], "subject": "算术", "analysis": "由数列为公差不为零的等差数列, $2 a_{7}-a_{8}=0, \\therefore 2\\left(a_{1}+6 d\\right)=a_{1}+7 d$, $\\therefore \\mathrm{a}_{1}=-5 \\mathrm{~d}, \\therefore \\frac{a_{2}}{a_{1}}=\\frac{a_{1}+d}{a_{1}}=\\frac{-5 d+d}{-5 d}=\\frac{4}{5}$. 故答案为: $\\frac{4}{5}$."} {"id": "19750", "image": [], "answer": "$a_{n}=6 n-3$", "solution": "null", "level": "高二", "question": "设 $\\left\\{a_{n}\\right\\}$ 是等差数列, 且 $a_{1}=3, a_{2}+a_{5}=36$, 则 $\\left\\{a_{n}\\right\\}$ 的通项公式为", "options": [], "subject": "算术", "analysis": "$\\because\\left\\{a_{n}\\right\\}$ 是等差数列, 且 $a_{1}=3, a_{2}+a_{5}=36$,\n\n$\\therefore\\left\\{\\begin{array}{l}a_{1}=3 \\\\ a_{1}+d+a_{1}+4 d=36\\end{array}\\right.$, 解得 $\\mathrm{a}_{1}=3, \\mathrm{~d}=6, \\therefore \\mathrm{a}_{\\mathrm{n}}=\\mathrm{a}_{1}+(\\mathrm{n}-1) \\mathrm{d}=3+(\\mathrm{n}-1) \\times 6=6 \\mathrm{n}-3$.\n\n$\\therefore\\left\\{\\mathrm{a}_{\\mathrm{n}}\\right\\}$ 的通项公式为 $\\mathrm{a}_{\\mathrm{n}}=6 \\mathrm{n}-3$. 故答案为: $\\mathrm{a}_{\\mathrm{n}}=6 \\mathrm{n}-3$."} {"id": "19763", "image": [], "answer": "8", "solution": "null", "level": "高二", "question": "已知 $\\left\\{a_{n}\\right\\}$ 是等差数列, 其公差 $d<0$, 其前 $n$ 项和记为 $S_{n}$, 且 $S_{16}>0, S_{17}<0$, 则当 $S_{n}$取最大值时的 $n=$ \\$ \\qquad \\$ .", "options": [], "subject": "算术", "analysis": "$: \\because \\mathrm{S}_{16}>0, \\mathrm{~S}_{17}<0, \\therefore 16 a_{1}+\\frac{16 \\times 15}{2} d>0,17 \\mathrm{a}_{1}+\\frac{17 \\times 16}{2} d<0$,\n化为 $2 a_{1}+15 d>0, a_{1}+8 d<0$, 即 $a_{8}+a_{9}>0, a_{9}<0, \\therefore a_{8}>0, a_{9}<0$, 又公差 $d<0$,\n\n$\\therefore$ 数列 $\\left\\{\\mathrm{a}_{\\mathrm{n}}\\right\\}$ 是单调递减数列, $\\therefore$ 当 $\\mathrm{S}_{\\mathrm{n}}$ 取最大值时的 $\\mathrm{n}=8$. 故答案为 8 ."} {"id": "19772", "image": [], "answer": "63", "solution": "null", "level": "高二", "question": "设 $\\left\\{a_{n}\\right\\}$ 是等差数列, 若 $a_{4}+a_{5}+a_{6}=21$, 则 $S_{9}=$", "options": [], "subject": "算术", "analysis": "$\\because\\left\\{a_{n}\\right\\}$ 是等差数列, $a_{4}+a_{5}+a_{6}=21, \\therefore a_{4}+a_{5}+a_{6}=3 a_{5}=21$, 解得 $a_{5}=7$,\n\n$\\therefore S_{9}=\\frac{9}{2}\\left(a_{1}+a_{9}\\right)=9 a_{5}=63$. 故答案为 $: 63$."} {"id": "19783", "image": [], "answer": "42", "solution": "null", "level": "高二", "question": "在等差数列 $\\left\\{a_{n}\\right\\}$ 中, 已知 $a_{2}+a_{3}=13, a_{1}=2$, 则 $a_{4}+a_{5}+a_{6}=$ \\$ \\qquad \\$ .", "options": [], "subject": "算术", "analysis": "在等差数列 $\\left\\{a_{n}\\right\\}$ 中, 已知 $a_{1}=2, a_{2}+a_{3}=13$,\n\n$\\therefore 2+\\mathrm{d}+2+2 \\mathrm{~d}=13$, 解得 $\\mathrm{d}=3$,\n\n$\\therefore a_{4}+a_{5}+a_{6}=a_{1}=a_{1}+3 d+a_{1}+4 d+a_{1}+5 d=3 a_{1}+12 d=3 \\times 2+12 \\times 3=42$, 故答案为 42 ."} {"id": "19785", "image": [], "answer": "4", "solution": "null", "level": "高二", "question": "在正项等比数列 $\\left\\{a_{n}\\right\\}$ 中, 有 $a_{1} a_{3}+2 a_{2} a_{4}+a_{3} a_{5}=16$, 则 $a_{2}+a_{4}=$", "options": [], "subject": "算术", "analysis": "$:\\left\\{a_{n}\\right\\}$ 是等比数列, 且 $\\mathrm{a}_{\\mathrm{n}}>0 \\quad \\therefore \\mathrm{a}_{1} \\mathrm{a}_{3}=\\mathrm{a}_{2}{ }^{2}, \\mathrm{a}_{3} \\mathrm{a}_{5}=\\mathrm{a}_{4}{ }^{2}$\n\n$\\because a_{1} a_{3}+2 a_{2} a_{4}+a_{3} a_{5}=16 \\quad \\therefore a_{1} a_{3}+2 a_{2} a_{4}+a_{3} a_{5}=\\left(a_{2}+a_{4}\\right)^{2}=16$\n$\\because$ 正项等比数列 $\\left\\{\\mathrm{a}_{n}\\right\\}, \\quad \\therefore \\mathrm{a}_{2}+\\mathrm{a}_{4}=4$ 故答案为 4 ."} {"id": "19794", "image": [], "answer": "2", "solution": "null", "level": "高二", "question": "在等比数列 $\\left\\{a_{n}\\right\\}$ 中, 已知 $a_{2}=\\frac{1}{2}, a_{5}=4$, 则此数列的公比为", "options": [], "subject": "算术", "analysis": "解析 $\\because a_{2}=\\frac{1}{2}, a_{5}=4, \\therefore \\mathrm{a}_{5}=\\mathrm{a}_{2} \\mathrm{q}^{3}, \\therefore \\mathrm{q}^{3}=8, \\therefore \\mathrm{q}=2$, 故答案为 2"} {"id": "18042", "image": [], "answer": "$\\frac{\\sqrt{3}}{2}$", "solution": "null", "level": "高二", "question": "6. 计算 $\\cos 45^{\\circ} \\cdot \\cos 15^{\\circ}+\\sin 45^{\\circ} \\sin 15^{\\circ}=$\n\n.", "options": [], "subject": "算术", "analysis": "解析 : $\\cos 45^{\\circ} \\cos 15^{\\circ}+\\sin 45^{\\circ} \\sin 15^{\\circ}=\\cos \\left(45^{\\circ}-15^{\\circ}\\right)=\\cos 30^{\\circ}=\\frac{\\sqrt{3}}{2}$"} {"id": "18060", "image": [], "answer": "原式 $=\\sin (\\alpha+\\beta) \\cos \\alpha-\\frac{1}{2}[\\sin (\\alpha+\\alpha+\\beta)-\\sin (\\alpha+\\beta-\\alpha)]=\\sin (\\alpha+\\beta) \\cos \\alpha-\\frac{1}{2}[\\sin \\alpha \\cos (\\alpha$\n\n$+\\beta)+\\cos \\alpha \\sin (\\alpha+\\beta)-\\sin (\\alpha+\\beta) \\cos \\alpha+\\cos (\\alpha+\\beta) \\sin \\alpha]$\n\n$=\\sin (\\alpha+\\beta) \\cos \\alpha-\\frac{1}{2} \\times 2 \\sin \\alpha \\cos (\\alpha+\\beta)$\n\n$=\\sin (\\alpha+\\beta) \\cos \\alpha-\\cos (\\alpha+\\beta) \\sin \\alpha=\\sin (\\alpha+\\beta-\\alpha)=\\sin \\beta$.", "solution": "null", "level": "高二", "question": "9. 化简: $\\sin (\\alpha+\\beta) \\cos \\alpha-\\frac{1}{2}[\\sin (2 \\alpha+\\beta)-\\sin \\beta]$.", "options": [], "subject": "算术", "analysis": "原式 $=\\sin (\\alpha+\\beta) \\cos \\alpha-\\frac{1}{2}[\\sin (\\alpha+\\alpha+\\beta)-\\sin (\\alpha+\\beta-\\alpha)]=\\sin (\\alpha+\\beta) \\cos \\alpha-\\frac{1}{2}[\\sin \\alpha \\cos (\\alpha$\n\n$+\\beta)+\\cos \\alpha \\sin (\\alpha+\\beta)-\\sin (\\alpha+\\beta) \\cos \\alpha+\\cos (\\alpha+\\beta) \\sin \\alpha]$\n\n$=\\sin (\\alpha+\\beta) \\cos \\alpha-\\frac{1}{2} \\times 2 \\sin \\alpha \\cos (\\alpha+\\beta)$\n\n$=\\sin (\\alpha+\\beta) \\cos \\alpha-\\cos (\\alpha+\\beta) \\sin \\alpha=\\sin (\\alpha+\\beta-\\alpha)=\\sin \\beta$."} {"id": "18065", "image": [], "answer": "1", "solution": "null", "level": "高二", "question": "13.已知 $\\tan \\alpha=-2, \\tan (\\alpha+\\beta)=\\frac{1}{7}$, 则 $\\tan (\\alpha-\\beta)=$", "options": [], "subject": "算术", "analysis": "$\\tan \\beta=\\tan [(\\alpha+\\beta)-\\alpha]$\n\n$=\\frac{\\tan (\\alpha+\\beta)-\\tan \\alpha}{1+\\tan (\\alpha+\\beta) \\tan \\alpha}$\n\n$=\\frac{\\frac{1}{7}-(-2)}{1+\\frac{1}{7} \\times(-2)}$\n\n$=3$,\n\n$\\therefore \\tan (\\alpha-\\beta)=\\frac{\\tan \\alpha-\\tan \\beta}{1+\\tan \\alpha \\tan \\beta}=\\frac{-2-3}{1-2 \\times 3}=1$.\n\n答案: 1"} {"id": "18072", "image": [], "answer": "$\\frac{7}{5}$", "solution": "null", "level": "高二", "question": "6. 若 $\\tan \\left(\\alpha-\\frac{\\pi}{4}\\right)=\\frac{1}{6}$, 则 $\\tan \\alpha=$", "options": [], "subject": "算术", "analysis": "解析 : $\\because \\tan \\left(\\alpha-\\frac{\\pi}{4}\\right)=\\frac{\\tan \\alpha-\\tan \\frac{\\pi}{4}}{1+\\tan \\alpha \\tan \\frac{\\pi}{4}}=\\frac{\\tan \\alpha-1}{1+\\tan \\alpha}=\\frac{1}{6} , \\therefore \\tan \\alpha=\\frac{7}{5}$."} {"id": "18100", "image": [], "answer": "-3", "solution": "null", "level": "高二", "question": "6. 已知 $\\sin \\theta=-\\frac{3}{5}, 3 \\pi<\\theta<\\frac{7 \\pi}{2}$, 则 $\\tan \\frac{\\theta}{2}=$", "options": [], "subject": "算术", "analysis": "由 $\\sin \\theta=-\\frac{3}{5} , 3 \\pi<\\theta<\\frac{7 \\pi}{2}$ ,得 $\\cos \\theta=-\\frac{4}{5}$ ,\n\n从而 $\\tan \\frac{\\theta}{2}=\\frac{\\sin \\theta}{1+\\cos \\theta}=\\frac{-\\frac{3}{5}}{1-\\frac{4}{5}}=-3$.\n\n答案: -3"} {"id": "18101", "image": [], "answer": "$\\frac{7}{9}$", "solution": "null", "level": "高二", "question": "7. 已知 $\\sin \\frac{\\theta}{2}+\\cos \\frac{\\theta}{2}=\\frac{2 \\sqrt{3}}{3}$, 则 $\\cos 2 \\theta=$", "options": [], "subject": "算术", "analysis": "因为 $\\sin \\frac{\\theta}{2}+\\cos \\frac{\\theta}{2}=\\frac{2 \\sqrt{3}}{3}$ ,所以 $1+\\sin \\theta=\\frac{4}{3}$ ,\n\n即 $\\sin \\theta=\\frac{1}{3}$ , 所以 $\\cos 2 \\theta=1-2 \\sin ^{2} \\theta=1-\\frac{2}{9}=\\frac{7}{9}$.\n\n答案: $\\frac{7}{9}$"} {"id": "19857", "image": [], "answer": "$\\geqslant\\langle>$", "solution": "null", "level": "高二", "question": "用不等号填空:\n\n(1) 若 $a>b$, 则 $a c^{2}$ $b c^{2}$.\n\n(2)若 $a+b>0, b<0$, 则 $b$ $a$.\n\n(3)若 $a>b, c0$ 时, $a c^{2}>b c^{2}$\n\n$c^{2}=0$ 时, 则 $a c^{2}=b c^{2}$, 若 $a>b$, 则 $a c^{2} \\geq b c^{2}$\n\n(2)因为 $a+b>0, b<0$, 则 $a>0$, 所以 $b-d, \\quad \\because a>b$, 则 $a-c>b-d$"} {"id": "19843", "image": [], "answer": "$\\frac{a+m}{b+m}>\\frac{a}{b}$ 加糖之前糖水的浓度为 $\\frac{a}{b}$, 加糖之后糖水的浓度为 $\\frac{a+m}{b+m}$, 故 $\\frac{a+m}{b+m}>\\frac{a}{b}$.", "solution": "null", "level": "高二", "question": "$b \\mathrm{~g}$ 糖水中有 $a \\mathrm{~g}$ 糖 $(b>a>0)$, 若再添上 $m \\mathrm{~g}$ 糖 $(m>0)$, 则糖水变甜了,根据这个事实提炼的一个不等式为", "options": [], "subject": "算术", "analysis": "$\\frac{a+m}{b+m}>\\frac{a}{b}$ 加糖之前糖水的浓度为 $\\frac{a}{b}$, 加糖之后糖水的浓度为 $\\frac{a+m}{b+m}$, 故 $\\frac{a+m}{b+m}>\\frac{a}{b}$."} {"id": "18717", "image": [], "answer": "80", "solution": "null", "level": "高二", "question": "7. 已知数列 $\\left\\{a_{n}\\right\\}$ 的前 $n$ 项和 $S_{n}=n^{2}+n+1$, 则 $a_{6}+a_{7}+\\cdots+a_{10}$ 的值为", "options": [], "subject": "算术", "analysis": "$a_{6}+a_{7}+\\cdots+a_{10}=S_{10}-S_{5}=111-31=80$."} {"id": "18718", "image": [], "answer": "0", "solution": "null", "level": "高二", "question": "8. 设等差数列 $\\left\\{a_{n}\\right\\}$ 的前 $n$ 项和为 $S_{n}$, 若 $S_{p}=S_{q}\\left(p, q \\in \\mathbf{N}^{*}\\right.$ 且 $\\left.p \\neq q\\right)$, 则 $S_{p+q}=$", "options": [], "subject": "算术", "analysis": "设 $S_{n}=a n^{2}+b n$, 由 $S_{p}=S_{q}$.\n\n知 $a p^{2}+b p=a q^{2}+b q, \\therefore p+q=-\\frac{b}{a}$.\n\n$\\therefore S_{p+q}=a(p+q)^{2}+b(p+q)$\n\n$=a\\left(-\\frac{b}{a}\\right)^{2}+b\\left(-\\frac{b}{a}\\right)$\n\n$=\\frac{b^{2}}{a}-\\frac{b^{2}}{a}=0$."} {"id": "18719", "image": [], "answer": "5 或 6", "solution": "null", "level": "高二", "question": "9. 等差数列 $\\left\\{a_{n}\\right\\}$ 中, $\\left|a_{3}\\right|=\\left|a_{9}\\right|$, 公差 $d<0$, 则使前 $n$ 项和 $S_{n}$ 取得最大值的自然数 $n$ 是", "options": [], "subject": "算术", "analysis": "$d<0,\\left|a_{3}\\right|=\\left|a_{9}\\right|, \\therefore a_{3}>0, a_{9}<0$ 且 $a_{3}+a_{9}=0$,\n\n$\\therefore a_{6}=0, \\quad \\therefore a_{1}>a_{2}>\\cdots>a_{5}>0, \\quad a_{6}=0,0>a_{7}>a_{8}>\\cdots$.\n\n$\\therefore$ 当 $n=5$ 或 6 时, $S_{n}$ 取到最大值."} {"id": "18721", "image": [], "answer": "$\\left\\{\\begin{array}{l}1, \\\\ \\frac{1}{3} \\cdot\\left(\\frac{4}{3}\\right)^{n-2}, \\quad n \\geqslant 2\\end{array}\\right.$", "solution": "null", "level": "高二", "question": "10. 数列 $\\left\\{a_{n}\\right\\}$ 中, $S_{n}$ 是其前 $n$ 项和, 若 $a_{1}=1, a_{n+1}=\\frac{1}{3} S_{n}(n \\geqslant 1)$, 则 $a_{n}=$", "options": [], "subject": "算术", "analysis": "_{n+1}=\\frac{1}{3} S_{n}, a_{n+2}=\\frac{1}{3} S_{n+1}$,\n\n$\\therefore a_{n+2}-a_{n+1}=\\frac{1}{3}\\left(S_{n+1}-S_{n}\\right)=\\frac{1}{3} a_{n+1}$,\n\n$\\therefore a_{n+2}=\\frac{4}{3} a_{n+1}(n \\geqslant 1)$.\n\n$\\because a_{2}=\\frac{1}{3} S_{1}=\\frac{1}{3}, \\quad \\therefore a_{n}= \\begin{cases}1, & n=1 \\\\ \\frac{1}{3} \\cdot\\left(\\frac{4}{3}\\right)^{n-2}, \\quad n \\geqslant 2\\end{cases}$"} {"id": "18731", "image": [], "answer": "-6", "solution": "null", "level": "高二", "question": "7. 一个数列 $\\left\\{a_{n}\\right\\}$, 其中 $a_{1}=3, a_{2}=6, a_{n+2}=a_{n+1}-a_{n}$, 那么这个数列的第 5 项是", "options": [], "subject": "算术", "analysis": "-6"} {"id": "19505", "image": [], "answer": "$a<0\\frac{1}{a}$, 即 $\\frac{1}{a}<\\frac{1}{b}$;\n\n若 $a b>0$, 则 $\\frac{1}{a}>\\frac{1}{b}$, 所以 $a0,\\end{array}\\right.$ 即 $2 k \\pi \\leqslant x<2 k \\pi+\\frac{\\pi}{2}(k \\in \\mathbf{Z})$.\n\n故函数 $y=\\sqrt{\\sin x}+\\lg \\cos x$ 的定义域为 $\\left[2 k \\pi, 2 k \\pi+\\frac{\\pi}{2}\\right)(k \\in \\mathbf{Z})$.\n\n答案: $\\left[2 k \\pi, 2 k \\pi+\\frac{\\pi}{2}\\right)(k \\in \\mathbf{Z})$"} {"id": "17789", "image": [], "answer": "$-\\frac{2}{5}$", "solution": "null", "level": "高二", "question": "8. 已知 $\\sin (3 \\pi-\\alpha)=-2 \\sin \\left(\\frac{\\pi}{2}+\\alpha\\right)$, 则 $\\sin \\alpha \\cos \\alpha$ 等于", "options": [], "subject": "代数", "analysis": "因为 $\\sin (3 \\pi-\\alpha)=\\sin (\\pi-\\alpha)=-2 \\sin \\left(\\frac{\\pi}{2}+\\alpha\\right)$, 所以 $\\sin \\alpha=-2 \\cos \\alpha$, 所以 $\\tan \\alpha=-2$,所以 $\\sin \\alpha \\cos \\alpha=\\frac{\\sin \\alpha \\cos \\alpha}{\\sin ^{2} \\alpha+\\cos ^{2} \\alpha}=\\frac{\\tan \\alpha}{\\tan ^{2} \\alpha+1}=-\\frac{2}{5}$."} {"id": "17806", "image": [], "answer": "(1)(4)", "solution": "null", "level": "高二", "question": "13. 关于 $x$ 的函数 $f(x)=\\sin (x+\\varphi)$ 有以下命题:\n\n(1)对任意的 $\\varphi, f(x)$ 都是非奇非偶函数;\n\n(2)不存在 $\\varphi$, 使 $f(x)$ 既是奇函数, 又是偶函数;\n\n(3) 存在 $\\varphi$, 使 $f(x)$ 是奇函数;\n\n(4)对任意的 $\\varphi, f(x)$ 都不是偶函数.\n\n其中的假命题是 \\$ \\qquad \\$ . (写出所有假命题的序号)", "options": [], "subject": "代数", "analysis": "易知(2)(3)成立, 令 $\\varphi=\\frac{\\pi}{2}, f(x)=\\cos x$ 是偶函数, (1)(4)都不成立.\n\n答案: (1)(4)"} {"id": "17818", "image": [], "answer": "$\\because-\\frac{\\pi}{3} \\leqslant x \\leqslant \\frac{2 \\pi}{3}$,\n$\\therefore-\\frac{\\sqrt{3}}{2} \\leqslant \\sin x \\leqslant 1$.\n\n当 $a=0$, 不满足题意.\n\n若 $a>0$, 则 $\\left\\{\\begin{array}{l}2 a+b=1, \\\\ -\\sqrt{3} a+b=-5,\\end{array}\\right.$\n\n解得 $\\left\\{\\begin{array}{l}a=12-6 \\sqrt{3}, \\\\ b=-23+12 \\sqrt{3} .\\end{array}\\right.$\n\n若 $a<0$, 则 $\\left\\{\\begin{array}{l}2 a+b=-5, \\\\ -\\sqrt{3} a+b=1,\\end{array}\\right.$\n\n解得 $\\left\\{\\begin{array}{l}a=-12+6 \\sqrt{3}, \\\\ b=19-12 \\sqrt{3} .\\end{array}\\right.$\n\n故 $a=12-6 \\sqrt{3}, b=-23+12 \\sqrt{3}$ 或 $a=-12+6 \\sqrt{3}, b=19-12 \\sqrt{3}$.", "solution": "null", "level": "高二", "question": "10. 已知函数 $f(x)=2 a \\sin x+b$ 的定义域为 $\\left[-\\frac{\\pi}{3}, \\frac{2 \\pi}{3}\\right]$, 函数的最大值为 1 , 最小值为 -5 , 求 $a$和 $b$ 的值.", "options": [], "subject": "代数", "analysis": "$\\because-\\frac{\\pi}{3} \\leqslant x \\leqslant \\frac{2 \\pi}{3}$,\n$\\therefore-\\frac{\\sqrt{3}}{2} \\leqslant \\sin x \\leqslant 1$.\n\n当 $a=0$, 不满足题意.\n\n若 $a>0$, 则 $\\left\\{\\begin{array}{l}2 a+b=1, \\\\ -\\sqrt{3} a+b=-5,\\end{array}\\right.$\n\n解得 $\\left\\{\\begin{array}{l}a=12-6 \\sqrt{3}, \\\\ b=-23+12 \\sqrt{3} .\\end{array}\\right.$\n\n若 $a<0$, 则 $\\left\\{\\begin{array}{l}2 a+b=-5, \\\\ -\\sqrt{3} a+b=1,\\end{array}\\right.$\n\n解得 $\\left\\{\\begin{array}{l}a=-12+6 \\sqrt{3}, \\\\ b=19-12 \\sqrt{3} .\\end{array}\\right.$\n\n故 $a=12-6 \\sqrt{3}, b=-23+12 \\sqrt{3}$ 或 $a=-12+6 \\sqrt{3}, b=19-12 \\sqrt{3}$."} {"id": "17842", "image": [], "answer": "(1)(2)", "solution": "null", "level": "高二", "question": "8. 关于函数 $y=\\tan \\frac{X}{2}$ 的说法正确的是 \\$ \\qquad \\$ . (填所有正确答案的序号)\n(1) 在 $\\left(0, \\frac{\\pi}{2}\\right)$ 上单调递增;\n(2) 为奇函数;\n(3) 以 $\\pi$ 为最小正周期;\n(4) 定义域为\n$\\left\\{x \\left\\lvert\\, x \\neq \\frac{\\pi}{4}+\\frac{k \\pi}{2}\\right., \\quad k \\in \\mathbf{Z}\\right\\}$.", "options": [], "subject": "代数", "analysis": "令 $x \\in\\left(0 , \\frac{\\pi}{2}\\right)$ ,则 $\\frac{X}{2} \\in\\left(0 , \\frac{\\pi}{4}\\right)$ ,所以 $y=\\tan \\frac{\\underline{x}}{2}$ 在 $\\left(0 , \\frac{\\pi}{2}\\right)$ 上单调递增,(1)正确; $\\tan \\left(-\\frac{x}{2}\\right)$\n\n$=-\\tan \\frac{X}{2}$ ,故 $y=\\tan \\frac{X}{2}$ 为奇函数,(2)正确; $T=\\frac{\\pi}{\\omega}=2 \\pi$ ,所以(3)不正确; 由 $\\frac{X}{2} \\neq \\frac{\\pi}{2}+k \\pi , k \\in \\mathbf{Z}$ ,\n\n得 $x \\neq \\pi+2 k \\pi , k \\in \\mathbf{Z}$ ,即函数 $y=\\tan \\frac{\\underline{X}}{2}$ 的定义域为 $\\{x \\mid x \\neq \\pi+2 k \\pi , k \\in \\mathbf{Z}\\}$ ,所以(4)不正确.\n\n答案: (1)(2)"} {"id": "17863", "image": [], "answer": "1", "solution": "null", "level": "高二", "question": "13. 已知 $\\omega>0$, 函数 $f(x)=\\sin \\left(\\omega x+\\frac{\\pi}{4}\\right)$ 在 $\\left(\\frac{\\pi}{2}, \\pi\\right)$ 上单调递减, 则正整数 $\\omega$ 的值是", "options": [], "subject": "代数", "analysis": "因为函数 $f(x)=\\sin \\left(\\omega x+\\frac{\\pi}{4}\\right)$ 单调递减时, $\\omega x+\\frac{\\pi}{4} \\in\\left[2 k \\pi+\\frac{\\pi}{2}, 2 k \\pi+\\frac{3 \\pi}{2}\\right], k \\in \\mathbf{Z}$ ,又因为 $\\omega>0$ ,当 $k=0$ 时, $x \\in\\left[\\frac{\\pi}{4 \\omega}, \\frac{5 \\pi}{4 \\omega}\\right]$. 又已知 $x \\in\\left(\\frac{\\pi}{2} , \\pi\\right)$ 时,函数 $f(x)$ 单调递减, 所以 $\\left\\{\\begin{array}{l}\\frac{\\pi}{4 \\omega} \\leqslant \\frac{\\pi}{2}, \\\\ \\frac{5 \\pi}{4 \\omega} \\geqslant \\pi ,\\end{array}\\right.$解得 $\\frac{1}{2} \\leqslant \\omega \\leqslant \\frac{5}{4}$. 又因为 $\\omega$ 为正整数,所以 $\\omega=1$. 而当 $k \\neq 0$ 时, $\\omega$ 无解. 综上, $\\omega=1$.\n\n答案: 1"} {"id": "17883", "image": [], "answer": "25", "solution": "null", "level": "高二", "question": "6. 已知某种交流电电流 $i(A)$ 随时间 $t(s)$ 的变化规律可以用函数 $i=5 \\sqrt{2} \\sin \\left(100 \\pi t-\\frac{\\pi}{2}\\right), t \\in[0$, $+\\infty)$ 表示,则这种交流电电流在 $0,.5 \\mathrm{~s}$ 内往复运行 \\$ \\qquad \\$次.", "options": [], "subject": "代数", "analysis": "解析 : $\\because$ 周期 $T=\\frac{2 \\pi}{100 \\pi}=\\frac{1}{50}(\\mathrm{~s}) , \\therefore$ 频率为每秒 50 次,\n\n$\\therefore 0,.5$ 秒往复运行 25 次."} {"id": "17884", "image": [], "answer": "$2 \\sin \\left(\\frac{\\pi}{4} x-\\frac{\\pi}{4}\\right)+6$", "solution": "null", "level": "高二", "question": "7. 据市场调查, 某种商品每件的售价按月份 $x$ 呈 $f(x)=A \\sin (\\omega x+\\varphi)+B$ 的模型波动, 已知 3 月份达到最高价 8 千元, 7 月份价格最低为 4 千元, 则 $f(x)=$ \\$ \\qquad \\$ , $x=1,2, \\cdots, 12$", "options": [], "subject": "代数", "analysis": "解析 : 由题意得 $\\left\\{\\begin{array}{l}A+B=8 , \\\\ -A+B=4 ,\\end{array}\\right.$ 解得 $\\left\\{\\begin{array}{l}A=2 , \\\\ B=6 \\end{array}\\right.$ 周期 $T=2 \\times(7-3)=8$ ,\n\n$\\therefore \\omega=\\frac{2 \\pi}{T}=\\frac{\\pi}{4}, \\quad \\therefore f(x)=2 \\sin \\left(\\frac{\\pi}{4} x+\\varphi\\right)+6$\n\n又 $\\because$ 当 $x=3$ 时, $y=8, \\therefore 8=2 \\sin \\left(\\frac{3 \\pi}{4}+\\varphi\\right)+6$\n\n$\\therefore \\sin \\left(\\frac{3 \\pi}{4}+\\varphi\\right)=1 . \\because|\\varphi|<\\frac{\\pi}{2}, \\quad \\therefore \\varphi=-\\frac{\\pi}{4}$.\n\n$\\therefore f(x)=2 \\sin \\left(\\frac{\\pi}{4} x-\\frac{\\pi}{4}\\right)+6$"} {"id": "17885", "image": [], "answer": "$y=-4 \\cos \\frac{5 \\pi}{2} t, t \\geqslant 0$", "solution": "null", "level": "高二", "question": "8. 一物体相对于某一固定位置的位移 $y(\\mathrm{~cm})$ 和时间 $t(\\mathrm{~s})$ 之间的一组对应值如下表所示, 则可近似地描述该物体的位移 $y(\\mathrm{~cm})$ 和时间 $t(\\mathrm{~s})$ 之间的关系的一个三角函数关系式为 \\$ \\qquad \\$ .", "options": [], "subject": "代数", "analysis": "设 $y=A \\sin (\\omega t+\\varphi)$ ,则从表中可以得到 $A=4 , T=0,.8 , \\therefore \\omega=\\frac{2 \\pi}{T}=\\frac{2 \\pi}{0,.8}=\\frac{5 \\pi}{2} , \\therefore y=$ $4 \\sin \\left(\\frac{5 \\pi}{2} t+\\varphi\\right)$.\n\n又由 $4 \\sin \\varphi=-4,.0$ ,得 $\\sin \\varphi=-1$ ,取 $\\varphi=-\\frac{\\pi}{2}$ ,\n\n故 $y=4 \\sin \\left(\\frac{5 \\pi}{2} t-\\frac{\\pi}{2}\\right)=-4 \\cos \\frac{5 \\pi}{2} t, t \\geqslant 0$.\n\n答案: $y=-4 \\cos \\frac{5 \\pi}{2} t, t \\geqslant 0$"} {"id": "17901", "image": [], "answer": "$\\{-4,0,2\\}$", "solution": "null", "level": "高二", "question": "13. 函数 $y=\\frac{|\\sin x|}{\\sin x}+\\frac{|\\cos x|}{\\cos x}-\\frac{2|\\sin x \\cos x|}{\\sin x \\cos x}$ 的值域是", "options": [], "subject": "代数", "analysis": "解析:当 $x$ 在第一象限时, $\\sin x>0, \\cos x>0, y=0$;当 $x$ 在第二象限时, $\\sin x>0, \\cos x<0, y=2$;当 $x$ 在第三象限时, $\\sin x<0, \\cos x<0, y=-4$;当 $x$ 在第四象限时, $\\sin x<0, \\cos x>0, y=2$."} {"id": "17931", "image": ["9023.jpg"], "answer": "$-\\frac{7}{4} \\boldsymbol{m}+\\frac{13}{8} \\boldsymbol{n}$", "solution": "null", "level": "高二", "question": "5. 设向量 $m=2 a-3 b, n=4 a-2 b, p=3 a+2 b$, 试用 $\\boldsymbol{m}, \\boldsymbol{n}$ 表示 $\\boldsymbol{p}, \\boldsymbol{p}=$", "options": [], "subject": "代数", "analysis": "设 $\\boldsymbol{p}=x \\boldsymbol{m}+y \\boldsymbol{n}$ ,则 $3 \\boldsymbol{a}+2 \\boldsymbol{b}=x(2 \\boldsymbol{a}-3 \\boldsymbol{b})+y(4 \\boldsymbol{a}-2 \\boldsymbol{b})=(2 x+4 y) \\boldsymbol{a}+(-3 x-2 y) \\boldsymbol{b}$ ,得\n\n\n\n答案: $-\\frac{7}{4} \\boldsymbol{m}+\\frac{13}{8} \\boldsymbol{n}$"} {"id": "17932", "image": ["9024.jpg"], "answer": "3", "solution": "null", "level": "高二", "question": "6. 已知向量 $\\boldsymbol{e}_{1}, \\boldsymbol{e}_{2}$ 不共线, 实数 $x, y$ 满足 $(3 x-4 y) \\boldsymbol{e}_{1}+(2 x-3 y) \\boldsymbol{e}_{2}=6 \\boldsymbol{e}_{1}+3 \\boldsymbol{e}_{2}$, 则 $x-y=$​", "options": [], "subject": "代数", "analysis": "\n\n答案: 3"} {"id": "17946", "image": [], "answer": "-1", "solution": "null", "level": "高二", "question": "7. 若向量 $\\boldsymbol{a}=\\left(x+3, x^{2}-3 x-4\\right)$ 与 $\\overrightarrow{A B}$ 相等, 其中 $A(1,2), B(3,2)$, 则 $x=$ \\$ \\qquad \\$", "options": [], "subject": "代数", "analysis": "$\\because A(1,2) B(3,2), \\therefore \\overrightarrow{A B}=(2,0)$.\n\n又 $\\because \\boldsymbol{a}=\\overrightarrow{A B}$ ,即 $\\left(x+3 , x^{2}-3 x-4\\right)=(2 , 0)$ ,\n\n$\\therefore\\left\\{\\begin{array}{l}x+3=2, \\\\ x^{2}-3 x-4=0,\\end{array}\\right.$ 解得 $x=-1$.\n\n答案: -1"} {"id": "17947", "image": ["9027.jpg", "9027.jpg"], "answer": "$\\boldsymbol{p}=2 \\boldsymbol{a}+3 \\boldsymbol{b}+\\boldsymbol{c}$\n$=2(2,1)+3(-1,3)+(1,2)$\n\n$=(4,2)+(-3,9)+(1,2)=(2,13)$.\n\n设 $\\boldsymbol{p}=x \\boldsymbol{a}+y \\boldsymbol{b}=x(2,1)+y(-1,3)$\n\n$=(2 x-y, x+3 y)$,\n\n$\\boldsymbol{a}$ 与 $\\boldsymbol{b}$ 不共线,\n\n\n\n$\\therefore p=\\frac{19}{7} a+\\frac{24}{7} b$.", "solution": "null", "level": "高二", "question": "8. 已知 $\\boldsymbol{a}=(2,1), \\boldsymbol{b}=(-1,3), \\boldsymbol{c}=(1,2)$, 求 $\\boldsymbol{p}=2 \\boldsymbol{a}+3 \\boldsymbol{b}+\\boldsymbol{c}$, 并用基底 $\\boldsymbol{a}, \\boldsymbol{b}$ 表示 $\\boldsymbol{p}$.", "options": [], "subject": "代数", "analysis": "$\\boldsymbol{p}=2 \\boldsymbol{a}+3 \\boldsymbol{b}+\\boldsymbol{c}$\n$=2(2,1)+3(-1,3)+(1,2)$\n\n$=(4,2)+(-3,9)+(1,2)=(2,13)$.\n\n设 $\\boldsymbol{p}=x \\boldsymbol{a}+y \\boldsymbol{b}=x(2,1)+y(-1,3)$\n\n$=(2 x-y, x+3 y)$,\n\n$\\boldsymbol{a}$ 与 $\\boldsymbol{b}$ 不共线,\n\n\n\n$\\therefore p=\\frac{19}{7} a+\\frac{24}{7} b$."} {"id": "17955", "image": [], "answer": "-3", "solution": "null", "level": "高二", "question": "4. 已知向量 $\\boldsymbol{a}=(2,6), \\boldsymbol{b}=(-1, \\lambda)$. 若 $\\boldsymbol{a} / / \\boldsymbol{b}$, 则 $\\lambda=$", "options": [], "subject": "代数", "analysis": "$\\because \\boldsymbol{a}=(2,6), \\boldsymbol{b}=(-1, \\lambda), \\boldsymbol{a} / / \\boldsymbol{b}$,\n\n$\\therefore 2 \\lambda-6 \\times(-1)=0, \\quad \\therefore \\lambda=-3$.\n\n答案: -3"} {"id": "19715", "image": [], "answer": "$\\frac{1}{2}$ 或 2", "solution": "null", "level": "高二", "question": "已知在等比数列 $\\left\\{a_{n}\\right\\}$ 中, $a_{5}-a_{1}=15, a_{4}-a_{2}=6$, 则公比 $q$ 的所有可能的值为", "options": [], "subject": "代数", "analysis": "设等比数列 $\\left\\{a_{n}\\right\\}$ 的公比为 $q$,由 $\\mathrm{a}_{5}-\\mathrm{a}_{1}=15, \\mathrm{a}_{4}-\\mathrm{a}_{2}=6$, 得: $\\left\\{\\begin{array}{l}a_{1} q^{4}-a_{1}=15(1) \\ a_{1} q^{3}-a_{1} q=6(2)\\end{array},(1) \\div(2)\\right.$ 得: $\\frac{q^{2}+1}{q}=\\frac{5}{2}$,即 $2 q^{2}-5 q+2=0$. 解得: $q=\\frac{1}{2}$ 或 $q=2$. 故答案为: $\\frac{1}{2}$ 或 2 ."} {"id": "19738", "image": [], "answer": "14", "solution": "null", "level": "高二", "question": "已知数列 $\\sqrt{3}, 3, \\sqrt{15}, \\cdots, \\sqrt{3(2 n-1)}$, 那么 9 是数列的第项.", "options": [], "subject": "代数", "analysis": "由 $\\sqrt{3(2 n-1)}=9$. 解之得 $\\mathrm{n}=14$由此可知 9 是此数列的第 14 项. 故答案为: 14"} {"id": "18027", "image": [], "answer": "0", "solution": "null", "level": "高二", "question": "5. 化简 $\\frac{2}{5}(\\boldsymbol{a}-\\boldsymbol{b})-\\frac{1}{3}(2 \\boldsymbol{a}+4 \\boldsymbol{b})+\\frac{2}{15}(2 \\boldsymbol{a}+13 \\boldsymbol{b})=$ \\$ \\qquad \\$", "options": [], "subject": "代数", "analysis": "$\\frac{2}{5}(a-b)-\\frac{1}{3}(2 a+4 b)+\\frac{2}{15}(2 a+13 b)=\\frac{2}{5} a-\\frac{2}{5} b-\\frac{2}{3} a-\\frac{4}{3} b+\\frac{4}{15} a+\\frac{26}{15} b=\\left(\\frac{2}{5}-\\frac{2}{3}+\\frac{4}{15}\\right) a+$ $\\left(-\\frac{2}{5}-\\frac{4}{3}+\\frac{26}{15}\\right) \\boldsymbol{b}=0 \\boldsymbol{a}+0 \\boldsymbol{b}=0+0=0$.\n\n答案: 0"} {"id": "18030", "image": ["9047.jpg", "9048.jpg"], "answer": "3", "solution": "null", "level": "高二", "question": "8.如图, 在 $\\triangle A B C$ 中, $\\angle B A C=60^{\\circ}, \\angle B A C$ 的平分线交 $B C$ 于点 $D$. 若 $A B=4$, 且 $\\overrightarrow{A D}=\\frac{1}{4} \\overrightarrow{A C}+\\lambda \\overrightarrow{A B}$ $(\\lambda \\in \\mathbf{R})$, 求 $|\\overrightarrow{A D}|$.\n\n", "options": [], "subject": "代数", "analysis": "因为 $B , D , C$ 三点共线,所以 $\\frac{1}{4}+\\lambda=1$ ,解得 $\\lambda=\\frac{3}{4}$ 如图,过点 $D$ 分别作 $A C , A B$\n的平行线交 $A B, A C$ 于点 $M, N$, 则 $\\overrightarrow{A N}=\\frac{1}{4} \\overrightarrow{A C}, \\overrightarrow{A M}=\\frac{3}{4} \\overrightarrow{A B}$. 经计算得 $A N=A M=3, A D=3 \\sqrt{3}$,即 $|\\overrightarrow{A D}|=3 \\sqrt{3}$.\n\n"} {"id": "19774", "image": [], "answer": "$\\frac{1}{2}$", "solution": "null", "level": "高二", "question": "若数列 $\\left\\{a_{n}\\right\\}$ 满足 $a_{11}=\\frac{1}{52}, \\frac{1}{a_{n+1}}-\\frac{1}{a_{n}}=5\\left(n \\in N^{*}\\right)$, 则 $a_{1}=$ \\$ \\qquad \\$", "options": [], "subject": "代数", "analysis": "$\\frac{1}{a_{n+1}}-\\frac{1}{a_{n}}=5, \\therefore\\left\\{\\frac{1}{a_{n}}\\right\\}$ 是以 5 为公差的等差数列,\n\n$\\therefore \\frac{1}{a_{n}}=\\frac{1}{a_{1}}+5(\\mathrm{n}-1), \\quad \\because \\mathrm{a}_{11}=\\frac{1}{52}, \\quad \\therefore \\frac{1}{a_{11}}=\\frac{1}{a_{1}}+5(11-1)=52$, 即 $\\frac{1}{a_{1}}=2$,\n\n$\\therefore \\mathrm{a}_{1}=\\frac{1}{2}$. 故答案为: $\\frac{1}{2}$."} {"id": "18087", "image": [], "answer": "$\\frac{1}{2}$", "solution": "null", "level": "高二", "question": "7.$\\frac{\\sin 110^{\\circ} \\sin 20^{\\circ}}{\\cos ^{2} 155^{\\circ}-\\sin ^{2} 155^{\\circ}}=$", "options": [], "subject": "代数", "analysis": "解析 : $\\frac{\\sin 110^{\\circ} \\sin 20^{\\circ}}{\\cos ^{2} 155^{\\circ}-\\sin ^{2} 155^{\\circ}}=\\frac{\\sin 70^{\\circ} \\sin 20^{\\circ}}{\\cos 310^{\\circ}}=\\frac{\\cos 20^{\\circ} \\sin 20^{\\circ}}{\\cos 50^{\\circ}}=$\n\n$\\frac{\\frac{1}{2} \\sin 40^{\\circ}}{\\sin 40^{\\circ}}=\\frac{1}{2}$"} {"id": "18621", "image": [], "answer": "2300", "solution": "null", "level": "高二", "question": "6. 某公司租赁甲、乙两种设备生产 $A, B$ 两类产品, 甲种设备每天能生产 $A$ 类产品 5 件和 $B$ 类产品 10 件, 乙种设备每天能生产 $A$ 类产品 6 件和 $B$ 类产品 20 件. 已知设备甲每天的租赁费为 200 元, 设备乙每天的租赁费为 300 元, 现该公司至少要生产 $A$ 类产品 50 件, $B$ 类产品 140 件, 所需租赁费最少为 \\$ \\qquad \\$元.", "options": [], "subject": "代数", "analysis": "设需租低甲种设备 $x$ 台, 乙种设备 $y$ 台, 则 $\\left\\{\\begin{array}{l}5 x+6 y \\geqslant 50, \\\\ 10 x+20 y \\geqslant 140, \\\\ x \\in \\mathbf{N}^{*}, \\\\ y \\in \\mathbf{N}^{*} .\\end{array}\\right.$\n\n目标函数为 $z=200 x+300 y$.\n\n作出其可行域, 易知当 $x=4, y=5$ 时, $z=200 x+300 y$ 有最小值 2300 元."} {"id": "18622", "image": ["9131.jpg"], "answer": "90", "solution": "null", "level": "高二", "question": "7. 某公司招收男职员 $x$ 名, 女职员 $y$ 名, $x$ 和 $y$ 需满足约束条件 $\\left\\{\\begin{array}{l}5 x-11 y \\geqslant-22, \\\\ 2 x+3 y \\geqslant 9, \\\\ 2 x \\leqslant 11,\\end{array}\\right.$ 则 $z=10 x+10 y$ 的最大值是 \\$ \\qquad \\$ .", "options": [], "subject": "代数", "analysis": "\n\n该不等式组表示平面区域如图阴影所示, 由于 $x, y \\in \\mathbf{N}^{*}$, 计算区域内与点 $\\left(\\frac{11}{2}, \\frac{9}{2}\\right)$ 最近的整点为 $(5,4)$, 当 $x=5, y=4$ 时, $z$ 取得最大值为 90"} {"id": "18623", "image": ["10107.jpg"], "answer": "$20 \\quad 24$", "solution": "null", "level": "高二", "question": "8. 某工厂有甲、乙两种产品, 按计划每天各生产不少于 15 吨, 已知生产甲产品 1 吨需煤 9 吨, 电力 4 千瓦, 劳动力 3 个(按工作日计算); 生产乙产品 1 吨需煤 4 吨, 电力 5 千瓦,劳动力 10 个; 甲产品每吨价 7 万元, 乙产品每吨价 12 万元; 但每天用煤量不得超过 300 吨, 电力不得超过 200 千瓦, 劳动力只有 300 个, 当每天生产甲产品 \\$ \\qquad \\$吨,乙产品吨时, 既能保证完成生产任务, 又能使工厂每天的利润最大.", "options": [], "subject": "代数", "analysis": "\n\n设每天生产甲产品 $x$ 吨, 乙产品 $y$ 吨, 总利润为 $S$ 万元,依题意约束条件为:\n\n$$\n\\left\\{\\begin{array}{l}\n9 x+4 y \\leqslant 300 \\\\\n4 x+5 y \\leqslant 200 \\\\\n3 x+10 y \\leqslant 300 \\\\\nx \\geqslant 15 \\\\\ny \\geqslant 15\n\\end{array}\\right.\n$$\n\n目标函数为 $S=7 x+12 y$.\n\n从图中可以看出, 当直线 $S=7 x+12 y$ 经过点 $A$ 时, 直线的纵截距最大, 所以 $S$ 也取最大值.\n\n解方程组 $\\left\\{\\begin{array}{l}4 x+5 y-200=0, \\\\ 3 x+10 y-300=0,\\end{array}\\right.$\n\n得 $A(20,24)$, 故当 $x=20, y=24$ 时,\n\n$S_{\\text {max }}=7 \\times 20+12 \\times 24=428$ (万元)."} {"id": "18633", "image": ["9139.jpg"], "answer": "7", "solution": "null", "level": "高二", "question": "6. 设变量 $x, y$ 满足约束条件 $\\left\\{\\begin{array}{l}x+y \\geqslant 3, \\\\ x-y \\geqslant-1, \\\\ 2 x-y \\leqslant 3 .\\end{array}\\right.$ 则目标函数 $z=2 x+3 y$ 的最小值为\n\n## 答案 7", "options": [], "subject": "代数", "analysis": "作出可行域如图所示.\n\n\n\n由图可知, $z=2 x+3 y$ 经过点 $A(2,1)$ 时, $z$ 有最小值, $z$ 的最小值为 7 ."} {"id": "18634", "image": ["9140.jpg"], "answer": "$(3,8)$", "solution": "null", "level": "高二", "question": "7. 已知 $-1\n\n由 $\\left\\{\\begin{array}{l}x+y=-1, \\\\ x-y=3\\end{array}\\right.$ 得 $\\left\\{\\begin{array}{l}x=1, \\\\ y=-2 .\\end{array}\\right.$\n\n由 $\\left\\{\\begin{array}{l}x+y=4, \\\\ x-y=2\\end{array}\\right.$ 得 $\\left\\{\\begin{array}{l}x=3, \\\\ y=1 .\\end{array}\\right.$\n\n$\\therefore 2 \\times 3-3 \\times 1x z$; (2) $z(y$\n\n$-\\mathrm{x})>0$;\n\n(3) $z y^{2}0, \\\\ z<0, \\\\ zx z, \\therefore\\right.\\right.$ (1)正确.\n\n(2) $\\because\\left\\{\\begin{array}{c}z0, \\therefore\\right.\\right.$ (2)正确.\n\n(3) $\\because z0$ 且 $z<0$.\n\n当 $y=0$ 时, $z y^{2}=x y^{2}$; 当 $y \\neq 0$ 时, $z y^{2}\\mathrm{z}, \\quad \\therefore \\mathrm{x}-\\mathrm{z}>0 . \\because \\mathrm{xz}<0, \\therefore(\\mathrm{x}-\\mathrm{z}) \\mathrm{xz}<0 . \\therefore$ (4)正确.\n\n综上, (1)(2)(4)正确."} {"id": "19869", "image": ["9261.jpg"], "answer": "$\\{x \\mid 23\\}$, 则集合 $A \\cap B=$", "options": [], "subject": "代数", "analysis": "\n\n集合 $B$ 中的 $|2 x-1|>3$, 得到 $2 x-1>3$ 或 $2 x-1<-3$, 解得 $x>2$ 或 $x<-1$\n\n则 $A \\cap B=\\{x \\mid 2\n当 $z>0$, 且 $y=\\frac{z}{x}$ 的图象过点 $\\mathrm{C}$ 时, $z$ 取最大值 1 ; 当 $z<0$, 且 $y=\\frac{z}{x}$ 的图象与直线 $2 x-y+1=0$相切时, $z$ 取最小值 $-\\frac{1}{8}$; 当 $x=0$ 时, $z=0$. 综上, 目标函数 $z=x y$ 的取值范围为 $\\left[-\\frac{1}{8}, 1\\right]$.", "solution": "null", "level": "高二", "question": "设 $x, y$ 满足约束条件 $\\left\\{\\begin{array}{l}x \\leq 0, \\\\ x-y \\leq 0, \\\\ 2 x-y+1 \\geq 0,\\end{array}\\right.$ 则目标函数 $\\mathrm{z}=x y$ 的取值范围为 .", "options": [], "subject": "代数", "analysis": "\n当 $z>0$, 且 $y=\\frac{z}{x}$ 的图象过点 $\\mathrm{C}$ 时, $z$ 取最大值 1 ; 当 $z<0$, 且 $y=\\frac{z}{x}$ 的图象与直线 $2 x-y+1=0$相切时, $z$ 取最小值 $-\\frac{1}{8}$; 当 $x=0$ 时, $z=0$. 综上, 目标函数 $z=x y$ 的取值范围为 $\\left[-\\frac{1}{8}, 1\\right]$."} {"id": "19887", "image": ["9271.jpg", "9271.jpg"], "answer": "$a \\in\\left(0, \\frac{1}{2}\\right]$ 解析 作出不等式组所表示的平面区域, 如图所示, 由 $\\left\\{\\begin{array}{l}2 x-y+2=0, \\\\ 2 x+y=0\\end{array}\\right.$, 解得 $\\left\\{\\begin{array}{l}x=-\\frac{1}{2}, \\\\ y=1,\\end{array}\\right.$则 $A\\left(-\\frac{1}{2}, 1\\right)$, 当函数 $y=\\log _{a}(x+1)$ 的图象经过 $A$ 点时, $1=\\log _{a} \\frac{1}{2} \\Rightarrow a=\\frac{1}{2}$, 根据对数函数的图象与性质可知, 要使得函数 $y=\\log _{a}(x+1)$ 的图象经过不等式组所表示的平面区域, 则实数 $a$ 的取值范围是 $a \\in\\left(0, \\frac{1}{2}\\right]$.\n\n", "solution": "null", "level": "高二", "question": "若函数 $y=\\log _{a}(x+1)(a>0$ 且 $a \\neq 1)$ 的图象经过不等式组 $\\left\\{\\begin{array}{l}x \\geq-1, \\\\ 2 x-y+2 \\leq 0, \\\\ 2 x+y \\leq 0\\end{array}\\right.$ 所表示的平面区域,则 $a$ 的取值范围是", "options": [], "subject": "代数", "analysis": "$a \\in\\left(0, \\frac{1}{2}\\right]$ 解析 作出不等式组所表示的平面区域, 如图所示, 由 $\\left\\{\\begin{array}{l}2 x-y+2=0, \\\\ 2 x+y=0\\end{array}\\right.$, 解得 $\\left\\{\\begin{array}{l}x=-\\frac{1}{2}, \\\\ y=1,\\end{array}\\right.$则 $A\\left(-\\frac{1}{2}, 1\\right)$, 当函数 $y=\\log _{a}(x+1)$ 的图象经过 $A$ 点时, $1=\\log _{a} \\frac{1}{2} \\Rightarrow a=\\frac{1}{2}$, 根据对数函数的图象与性质可知, 要使得函数 $y=\\log _{a}(x+1)$ 的图象经过不等式组所表示的平面区域, 则实数 $a$ 的取值范围是 $a \\in\\left(0, \\frac{1}{2}\\right]$.\n\n"} {"id": "19905", "image": [], "answer": "$\\left[\\frac{1}{5},+\\infty\\right)$.", "solution": "null", "level": "高二", "question": "若对任意 $x>0, \\frac{x}{x^{2}+3 x+1} \\leq a$ 恒成立, 则 $a$ 的取值范围是", "options": [], "subject": "代数", "analysis": "因为 $x>0$, 所以 $\\frac{x}{x^{2}+3 x+1}=\\frac{1}{x+\\frac{1}{x}+3} \\leq \\frac{1}{2 \\sqrt{x \\cdot \\frac{1}{x}+3}}=\\frac{1}{5}$,当且仅当 $x=\\frac{1}{x}(x>0)$ 即 $x=1$ 时等号成立, 故 $a$ 的取值范围是 $\\frac{1}{5} \\leq a$,即 $a \\in\\left[\\frac{1}{5},+\\infty\\right)$"} {"id": "17760", "image": [], "answer": "$\\left[\\frac{\\pi}{2}, \\pi\\right]$", "solution": "null", "level": "高二", "question": "6. 若 $\\beta \\in[0,2 \\pi)$, 且 $\\sqrt{1-\\cos ^{2} \\beta}+\\sqrt{1-\\sin ^{2} \\beta}=\\sin \\beta-\\cos \\beta$, 则 $\\beta$ 的取值范围是", "options": [], "subject": "度量几何学", "analysis": "$\\because \\sqrt{1-\\cos ^{2} \\beta}+\\sqrt{1-\\sin ^{2} \\beta}=\\sqrt{\\sin ^{2} \\beta}+\\sqrt{\\cos ^{2} \\beta}=|\\sin \\beta|+|\\cos \\beta|=\\sin \\beta-\\cos \\beta , \\therefore \\sin$ $\\beta \\geqslant 0, \\cos \\beta \\leqslant 0, \\therefore \\beta$ 的终边在第二象限或在 $x$ 轴负半轴或在 $y$ 轴正半轴. $\\because 0 \\leqslant \\beta<2 \\pi$ , $\\therefore \\beta \\in\\left[\\frac{\\pi}{2}, \\pi\\right]$.\n\n答案: $\\left[\\frac{\\pi}{2}, \\pi\\right]$"} {"id": "17779", "image": [], "answer": "$\\frac{1}{2}$", "solution": "null", "level": "高二", "question": "13. 已知 $\\alpha$ 为第二象限角, $\\sin \\alpha=\\frac{2 \\sqrt{5}}{5}$, 则 $\\tan \\left(\\alpha+\\frac{\\pi}{2}\\right)=$", "options": [], "subject": "度量几何学", "analysis": "由题意知, $\\cos \\alpha=-\\sqrt{1-\\left(\\frac{2 \\sqrt{5}}{5}\\right)^{2}}=-\\frac{\\sqrt{5}}{5}$,\n$\\tan \\left(\\alpha+\\frac{\\pi}{2}\\right)=\\frac{\\sin \\left(\\alpha+\\frac{\\pi}{2}\\right)}{\\cos \\left(\\alpha+\\frac{\\pi}{2}\\right)}=\\frac{\\cos \\alpha}{-\\sin \\alpha}=-\\frac{-\\frac{\\sqrt{5}}{5}}{\\frac{2 \\sqrt{5}}{5}}=\\frac{1}{2}$.\n\n答案: $\\frac{1}{2}$"} {"id": "17814", "image": [], "answer": "$\\sqrt{2}+1$", "solution": "null", "level": "高二", "question": "7. 已知 $f(n)=\\sin \\frac{n \\pi}{4}(n \\in \\mathbf{Z})$, 则 $f(1)+f(2)+\\cdots+f(100)=$", "options": [], "subject": "度量几何学", "analysis": "$f(1)+f(2)+\\cdots+f(8)=0, f(9)+f(10)+\\cdots+f(16)=0$, 依次循环, $f(1)+f(2)+\\cdots+f(100)$\n\n$=0+f(97)+f(98)+f(99)+f(100)=\\sqrt{2}+1$.\n\n答案: $\\sqrt{2}+1$"} {"id": "18476", "image": [], "answer": "$2 \\sqrt{3}$", "solution": "null", "level": "高二", "question": "7. 在 $\\triangle A B C$ 中, 已知 $a=3 \\sqrt{2}, \\cos C=\\frac{1}{3}, S_{\\triangle A B C}=4 \\sqrt{3}$, 则 $b=$", "options": [], "subject": "度量几何学", "analysis": "$\\because \\cos C=\\frac{1}{3}, \\therefore \\sin C=\\frac{2 \\sqrt{2}}{3}$,\n\n$\\therefore \\frac{1}{2} a b \\sin C=4 \\sqrt{3}, \\therefore b=2 \\sqrt{3}$."} {"id": "17894", "image": [], "answer": "$-\\frac{23 \\pi}{3}$", "solution": "null", "level": "高二", "question": "6.时钟从 6 时 50 分走到 10 时 40 分, 分针旋转了弧度。", "options": [], "subject": "度量几何学", "analysis": "时钟共走了 3 小时 50 分钟, 分针旋转了 $-\\left(3 \\times 2 \\pi+\\frac{5}{6} \\times 2 \\pi\\right)=-\\frac{23 \\pi}{3}$\n\n答案: $-\\frac{23 \\pi}{3}$"} {"id": "17895", "image": ["9016.jpg"], "answer": "$47 \\mathrm{~m}$", "solution": "null", "level": "高二", "question": "如图, 公路弯道处 $\\widehat{A B}$ 的长 $l=$ (精确到 $1 \\mathrm{~m}$ )", "options": [], "subject": "度量几何学", "analysis": "$l=\\frac{\\pi}{3} \\times 45 \\approx 47(\\mathrm{~m})$."} {"id": "17897", "image": [], "answer": "(1) $\\because-1480^{\\circ}=-\\frac{74 \\pi}{9}=-10 \\pi+\\frac{16}{9} \\pi$,\n\n又 $0<\\frac{16}{9} \\pi<2 \\pi, \\quad \\therefore-1480^{\\circ}=\\frac{16}{9} \\pi+2 \\times(-5) \\pi$.\n\n(2) $\\because \\beta$ 与 $\\alpha$ 终边相同, $\\therefore \\beta=\\alpha+2 k \\pi=\\frac{16}{9} \\pi+2 k \\pi(k \\in \\mathbf{Z})$.\n当 $k=-1$ 时, $\\beta=\\frac{16}{9} \\pi-2 \\pi=-\\frac{2}{9} \\pi$,\n\n当 $k=-2$ 时, $\\beta=\\frac{16}{9} \\pi-4 \\pi=-\\frac{20}{9} \\pi$", "solution": "null", "level": "高二", "question": "9.(1)把 $-1480^{\\circ}$ 写成 $\\alpha+2 k \\pi(k \\in \\mathbf{Z})$ 的形式, 其中 $0 \\leqslant \\alpha<2 \\pi$;\n\n(2)若 $\\beta \\in[-4 \\pi, 0]$, 且 $\\beta$ 与(1)中 $\\alpha$ 的终边相同, 求 $\\beta$.", "options": [], "subject": "度量几何学", "analysis": "(1) $\\because-1480^{\\circ}=-\\frac{74 \\pi}{9}=-10 \\pi+\\frac{16}{9} \\pi$,\n\n又 $0<\\frac{16}{9} \\pi<2 \\pi, \\quad \\therefore-1480^{\\circ}=\\frac{16}{9} \\pi+2 \\times(-5) \\pi$.\n\n(2) $\\because \\beta$ 与 $\\alpha$ 终边相同, $\\therefore \\beta=\\alpha+2 k \\pi=\\frac{16}{9} \\pi+2 k \\pi(k \\in \\mathbf{Z})$.\n当 $k=-1$ 时, $\\beta=\\frac{16}{9} \\pi-2 \\pi=-\\frac{2}{9} \\pi$,\n\n当 $k=-2$ 时, $\\beta=\\frac{16}{9} \\pi-4 \\pi=-\\frac{20}{9} \\pi$."} {"id": "17902", "image": [], "answer": "-8", "solution": "null", "level": "高二", "question": "14. 已知角 $\\theta$ 的顶点为坐标原点, 始边为 $x$ 轴的正半轴, 若 $(4, y)$ 是角 $\\theta$ 终边上一点, 且 $\\sin \\theta$ $=-\\frac{2 \\sqrt{5}}{5}$, 则 $y=$", "options": [], "subject": "度量几何学", "analysis": "$\\because \\sin \\theta=\\frac{y}{\\sqrt{4^{2}+y^{2}}}=-\\frac{2 \\sqrt{5}}{5}<0$,\n$\\therefore y<0$, 且 $y^{2}=64$,\n\n$\\therefore y=-8$.\n\n答案: -8"} {"id": "17911", "image": [], "answer": "$(-2,3]$", "solution": "null", "level": "高二", "question": "8. 已知角 $\\alpha$ 的终边经过点 $(3 a-9, a+2)$, 且 $\\sin \\alpha>0, \\cos \\alpha \\leqslant 0$, 则实数 $a$ 的取值范围是\n\n解析:由 $\\sin \\alpha>0, \\cos \\alpha \\leqslant 0$ 可知 $\\alpha$ 的终边在第二象限或 $y$ 轴正半轴上,\n\n$\\therefore\\left\\{\\begin{array}{l}3 a-9 \\leqslant 0 \\\\ a+2>0\\end{array}\\right.$,\n\n$\\therefore-2\n\n\n\n又 $\\overrightarrow{O A}+\\overrightarrow{O C}=-2 \\overrightarrow{O B}$ ,所以 $\\overrightarrow{O D}=-\\overrightarrow{O B}$ ,即 $O$ 为 $B D$ 的中点,即 $\\triangle A O B$ 与 $\\triangle A O C$ 的面积之\n\n比为 $1: 2$.\n\n答案: $1: 2$"} {"id": "18028", "image": [], "answer": "$-\\frac{5}{7}$", "solution": "null", "level": "高二", "question": "6. 若 $|\\boldsymbol{a}|=5, \\boldsymbol{b}$ 与 $\\boldsymbol{a}$ 的方向相反, 且 $|\\boldsymbol{b}|=7$, 则 $\\boldsymbol{a}=$ \\$ \\qquad \\$ b.", "options": [], "subject": "度量几何学", "analysis": "$\\because \\boldsymbol{b}$ 与 $\\boldsymbol{a}$ 方向相反, $\\therefore$ 设 $\\boldsymbol{a}=\\lambda \\boldsymbol{b}(\\lambda<0) , \\therefore|\\boldsymbol{a}|=|\\lambda||\\boldsymbol{b}|$ ,\n\n$\\therefore 5=|\\lambda| \\times 7, \\therefore|\\lambda|=\\frac{5}{7}, \\quad \\therefore \\lambda= \\pm \\frac{5}{7}$.\n\n又 $\\because \\lambda<0, \\quad \\therefore \\lambda=-\\frac{5}{7}$.\n\n答案: $-\\frac{5}{7}$"} {"id": "18050", "image": [], "answer": "$\\frac{7 \\pi}{4}$", "solution": "null", "level": "高二", "question": "13. 已知 $A, B$ 均为针角且 $\\sin A=\\frac{\\sqrt{5}}{5}, \\sin B=\\frac{\\sqrt{10}}{10}$, 则 $A+B$ 的大小为", "options": [], "subject": "度量几何学", "analysis": "$\\because A, B$ 均为钝角且 $\\sin A=\\frac{\\sqrt{5}}{5} , \\sin B=\\frac{\\sqrt{10}}{10}$ ,\n\n$\\therefore \\cos A=-\\sqrt{1-\\sin ^{2} A}=-\\frac{2 \\sqrt{5}}{5}$,\n\n$\\cos B=-\\sqrt{1-\\sin ^{2} B}=-\\frac{3 \\sqrt{10}}{10}$.\n\n$\\therefore \\cos (A+B)=\\cos A \\cos B-\\sin A \\sin B=-\\frac{2 \\sqrt{5}}{5} \\times\\left(-\\frac{3 \\sqrt{10}}{10}\\right)-\\frac{\\sqrt{5}}{5} \\times \\frac{\\sqrt{10}}{10}=\\frac{\\sqrt{2}}{2}$.\n\n又 $\\because \\frac{\\pi}{2}0, \\cos (\\alpha+\\beta)<0$.\n\n$\\because \\cos \\left(\\beta-\\frac{\\pi}{4}\\right)=\\frac{1}{3}, \\sin (\\alpha+\\beta)=\\frac{4}{5}$,\n\n$\\therefore \\sin \\left(\\beta-\\frac{\\pi}{4}\\right)=\\frac{2 \\sqrt{2}}{3}, \\cos (\\alpha+\\beta)=-\\frac{3}{5}$ ,\n\n$\\therefore \\cos \\left(\\alpha+\\frac{\\pi}{4}\\right)=\\cos \\left[(\\alpha+\\beta)-\\left(\\beta-\\frac{\\pi}{4}\\right)\\right]$\n\n$=\\cos (\\alpha+\\beta) \\cos \\left(\\beta-\\frac{\\pi}{4}\\right)+\\sin (\\alpha+\\beta) \\cdot \\sin \\left(\\beta-\\frac{\\pi}{4}\\right)$\n\n$=-\\frac{3}{5} \\times \\frac{1}{3}+\\frac{4}{5} \\times \\frac{2 \\sqrt{2}}{3}=\\frac{8 \\sqrt{2}-3}{15}$."} {"id": "18062", "image": [], "answer": "$-\\frac{7 \\pi}{4}$", "solution": "null", "level": "高二", "question": "10. 已知 $\\tan (\\alpha-\\beta)=\\frac{1}{2}, \\tan \\beta=-\\frac{1}{7}$, 且 $\\alpha, \\beta \\in(-\\pi, 0)$, 求 $2 \\alpha-\\beta$ 的值.", "options": [], "subject": "度量几何学", "analysis": "$\\because \\alpha=(\\alpha-\\beta)+\\beta , \\tan (\\alpha-\\beta)=\\frac{1}{2} , \\tan \\beta=-\\frac{1}{7}$ ,\n\n$\\therefore \\tan \\alpha=\\tan [(\\alpha-\\beta)+\\beta]=\\frac{\\tan (\\alpha-\\beta)+\\tan \\beta}{1-\\tan (\\alpha-\\beta) \\tan \\beta}$\n\n$=\\frac{\\frac{1}{2}-\\frac{1}{7}}{1+\\frac{1}{2} \\times \\frac{1}{7}}=\\frac{1}{3}$.\n\n又 $2 \\alpha-\\beta=\\alpha+(\\alpha-\\beta)$\n$\\therefore \\tan (2 \\alpha-\\beta)=\\tan [\\alpha+(\\alpha-\\beta)]$\n\n$=\\frac{\\tan \\alpha+\\tan (\\alpha-\\beta)}{1-\\tan \\alpha \\tan (\\alpha-\\beta)}=\\frac{\\frac{1}{3}+\\frac{1}{2}}{1-\\frac{1}{3} \\times \\frac{1}{2}}=1$.\n\n$\\because \\tan \\alpha=\\frac{1}{3}>0, \\tan \\beta=-\\frac{1}{7}<0, \\alpha, \\beta \\in(-\\pi, 0)$,\n\n$\\therefore \\alpha \\in\\left(-\\pi,-\\frac{\\pi}{2}\\right), \\beta \\in\\left(-\\frac{\\pi}{2}, 0\\right)$,\n\n$\\therefore \\alpha-\\beta \\in(-\\pi, 0)$,\n\n而 $\\tan (\\alpha-\\beta)=\\frac{1}{2}>0$ ,则 $\\alpha-\\beta \\in\\left(-\\pi ,-\\frac{\\pi}{2}\\right)$ ,\n\n结合 $\\alpha \\in\\left(-\\pi ,-\\frac{\\pi}{2}\\right)$ ,则有 $2 \\alpha-\\beta \\in(-2 \\pi ,-\\pi)$ ,\n\n$\\therefore 2 \\alpha-\\beta=-\\frac{7 \\pi}{4}$."} {"id": "18079", "image": [], "answer": "0 或 $-\\frac{4}{3}$", "solution": "null", "level": "高二", "question": "13. 若 $\\sin \\frac{\\alpha}{2}=\\sqrt{1+\\sin \\alpha}-\\sqrt{1-\\sin \\alpha}, 0 \\leqslant \\alpha \\leqslant \\pi$, 则 $\\tan \\alpha$ 的值是", "options": [], "subject": "度量几何学", "analysis": "解析:两边平方得 $\\sin ^{2} \\frac{\\alpha}{2}=2-2 \\sqrt{1-\\sin ^{2} \\alpha}$ ,\n\n$\\therefore \\frac{1-\\cos \\alpha}{2}=2-2|\\cos \\alpha| \\cdot(1)$\n\n当 $0 \\leqslant \\alpha \\leqslant \\frac{\\pi}{2}$ 时,(1)式为 $\\frac{1-\\cos \\alpha}{2}=2-2 \\cos \\alpha$ ,\n\n$\\therefore \\cos \\alpha=1$ ,\n\n$\\therefore \\alpha=0, \\quad \\therefore \\tan \\alpha=0$.\n当 $\\frac{\\pi}{2}<\\alpha \\leqslant \\pi$ 时,(1)式为 $\\frac{1-\\cos \\alpha}{2}=2+2 \\cos \\alpha$ ,\n\n$\\therefore \\cos \\alpha=-\\frac{3}{5}$,\n\n$\\therefore \\sin \\alpha=\\frac{4}{5} . \\therefore \\tan \\alpha=-\\frac{4}{3}$.\n\n综上, $\\tan \\alpha$ 的值是 0 或 $-\\frac{4}{3}$."} {"id": "18086", "image": [], "answer": "2", "solution": "null", "level": "高二", "question": "6.$ \\frac{3-\\sin 70^{\\circ}}{2-\\cos ^{2} 10^{\\circ}}=$", "options": [], "subject": "度量几何学", "analysis": "$\\frac{3-\\sin 70^{\\circ}}{2-\\cos ^{2} 10^{\\circ}}=\\frac{3-\\sin 70^{\\circ}}{2-\\frac{1+\\cos 20^{\\circ}}{2}}=\\frac{2\\left(3-\\cos 20^{\\circ}\\right)}{3-\\cos 20^{\\circ}}=2$.\n\n答案: 2"} {"id": "18088", "image": [], "answer": "$-\\frac{1}{7}$", "solution": "null", "level": "高二", "question": "8. 已知 $\\alpha \\in\\left(\\frac{\\pi}{2}, \\pi\\right)$, 且 $\\sin \\alpha=\\frac{\\sqrt{5}}{5}$, 则 $\\tan \\left(2 \\alpha+\\frac{\\pi}{4}\\right)=$", "options": [], "subject": "度量几何学", "analysis": "因为 $\\alpha \\in\\left(\\frac{\\pi}{2} , \\pi\\right)$ ,且 $\\sin \\alpha=\\frac{\\sqrt{5}}{5}$ ,所以 $\\cos \\alpha=-\\frac{2 \\sqrt{5}}{5}$ ,所以 $\\tan \\alpha=-\\frac{1}{2}$ ,由二倍角公\n\n式得 $\\tan 2 \\alpha=\\frac{2 \\tan \\alpha}{1-\\tan ^{2} \\alpha}=-\\frac{4}{3} , \\tan \\left(2 \\alpha+\\frac{\\pi}{4}\\right)=\\frac{\\tan 2 \\alpha+1}{1-\\tan 2 \\alpha}=-\\frac{1}{7}$.\n\n答案: $-\\frac{1}{7}$"} {"id": "17855", "image": [], "answer": "$y=\\frac{1}{4} \\sin x$", "solution": "null", "level": "高二", "question": "6. 将函数 $y=\\frac{1}{2} \\sin 2 x$ 的图象上所有点的横坐标伸长为原来的 2 倍, 然后纵坐标缩短为原来的 $\\frac{1}{2}$,则所得图象的函数解析式为", "options": [], "subject": "变换几何", "analysis": "解析 : $y=\\frac{1}{2} \\sin 2 x$ 的图象 $\\frac{\\text { 横坐标伸长为 }}{\\text { 原来的 } 2 \\text { 倍 }}$\n\n$y=\\frac{1}{2} \\sin 2\\left(\\frac{1}{2} x\\right)=\\frac{1}{2} \\sin x$ 的图象 $\\xrightarrow{\\text { 纵坐标缩短为 }}$\n\n原来的 $\\frac{1}{2}$\n$y=\\frac{1}{4} \\sin x$ 的图象,即所得图象的解析式为 $y=\\frac{1}{4} \\sin x$."} {"id": "17926", "image": ["9020.jpg"], "answer": "$\\lambda=\\frac{1}{4}$ ,$\\mu=\\frac{1}{3}$.", "solution": "null", "level": "高二", "question": "14. 已知 $\\triangle A B C$ 内一点 $P$ 满足 $\\overrightarrow{A P}=\\lambda \\overrightarrow{A B}+\\mu \\overrightarrow{A C}$, 若 $\\triangle P A B$ 的面积与 $\\triangle A B C$ 的面积之比为 $1: 3$, $\\triangle P A C$ 的面积与 $\\triangle A B C$ 的面积之比为 $1: 4$, 求实数 $\\lambda, \\mu$ 的值.", "options": [], "subject": "组合几何学", "analysis": "如图,过点 $P$ 作 $P M / / A C , P N / / A B$ ,则 $\\overrightarrow{A P}=\\overrightarrow{A M}+\\overrightarrow{A N}$ ,所以 $\\overrightarrow{A M}=\\lambda \\overrightarrow{A B}, \\overrightarrow{A N}=\\mu \\overrightarrow{A C}$.\n\n\n\n作 $P G \\perp A C$ 于点 $G, B H \\perp A C$ 于点 $H$.\n\n因为 $\\frac{S_{\\triangle P A C}}{S_{\\triangle A B C}}=\\frac{1}{4}$, 所以 $\\frac{P G}{B H}=\\frac{1}{4}$.\n\n又因为 $\\triangle P N G \\sim \\triangle B A H$ ,所以 $\\frac{P G}{B H}=\\frac{P N}{A B}=\\frac{1}{4}$ ,\n\n即 $\\frac{A M}{A B}=\\frac{1}{4}$ ,所以 $\\lambda=\\frac{1}{4}$ ,同理 $\\mu=\\frac{1}{3}$."} {"id": "18016", "image": [], "answer": "0", "solution": "null", "level": "高二", "question": "8. 下列叙述中正确的个数是\n\n(1)若 $a=b$, 则 $3 a>2 b$;\n\n(2)若 $a / / b$, 则 $a$ 与 $b$ 的方向相同或相反;\n\n(3)对任一向量 $a, \\frac{a}{|a|}$ 是一个单位向量.", "options": [], "subject": "逻辑题", "analysis": "向量不能比较大小, (1)错误; 由于零向量与任一向量共线, 且零向量的方向是任意的, 故(2)错误; 对于(3), 当 $a=0$ 时, $\\frac{a}{|a|}$ 无意义, 故(3)错误.\n\n答案: 0"} {"id": "19844", "image": [], "answer": "(3)(1)同向不等式不具有可乘性, 如: $2>1,-1>-2$, 则 $2 \\times(-1)=1 \\times(-2)$,\n\n故错误. (2)若 $\\frac{c}{a}>\\frac{c}{b}$, 则 $\\mathrm{a}<\\mathrm{b}$, 错误, 如: $\\frac{1}{2}>\\frac{1}{-3}, 2>-3$. (3)若 $\\mathrm{a}>\\mathrm{b}$, 则 $\\mathrm{a} \\cdot 2^{\\mathrm{c}}>\\mathrm{b} \\cdot 2^{\\mathrm{c}}$, 根据不等式的性质可知正确. 故正确命题序号为(3).", "solution": "null", "level": "高二", "question": "已知 $\\mathrm{a}, \\mathrm{b} \\in \\mathrm{R}$, 有以下命题: (1)若 $\\mathrm{a}>\\mathrm{b}, \\mathrm{c}>d$, 则 $\\mathrm{ac}>\\mathrm{b} d$; (2) 若 $\\frac{c}{a}>\\frac{c}{b}$,则 $\\mathrm{a}<\\mathrm{b}$; (3)若 $\\mathrm{a}>\\mathrm{b}$ ,则 $\\mathrm{a} \\cdot 2^{\\mathrm{c}}>\\mathrm{b} \\cdot 2^{\\mathrm{c}}$. 则正确命题的序号为", "options": [], "subject": "逻辑题", "analysis": "(3)(1)同向不等式不具有可乘性, 如: $2>1,-1>-2$, 则 $2 \\times(-1)=1 \\times(-2)$,\n\n故错误. (2)若 $\\frac{c}{a}>\\frac{c}{b}$, 则 $\\mathrm{a}<\\mathrm{b}$, 错误, 如: $\\frac{1}{2}>\\frac{1}{-3}, 2>-3$. (3)若 $\\mathrm{a}>\\mathrm{b}$, 则 $\\mathrm{a} \\cdot 2^{\\mathrm{c}}>\\mathrm{b} \\cdot 2^{\\mathrm{c}}$, 根据不等式的性质可知正确. 故正确命题序号为(3)."} {"id": "20155", "image": [], "answer": ": 逆否", "solution": "null", "level": "高二", "question": "6. 若命题 $p$ 的逆命题是 $q$, 命题 $p$ 的否命题是 $v$, 则 $q$ 是 $v$ 的命题.", "options": [], "subject": "逻辑题", "analysis": ": 逆否"} {"id": "20156", "image": [], "answer": ":逆否命题", "solution": "null", "level": "高二", "question": "7. 给出命题: 若函数 $y=f(x)$ 是幂函数, 则函数 $y=f(x)$ 的图象不过第四象限. 在它的逆命题、否命题、逆否命题三个命题中, 是真命题的是", "options": [], "subject": "逻辑题", "analysis": ":逆否命题"} {"id": "20157", "image": [], "answer": ":(2)(4)", "solution": "null", "level": "高二", "question": "8. 有下列四个命题:\n\n(1) “若 $\\angle A=60^{\\circ}$, 则 $\\sin A=\\frac{\\sqrt{3}}{2}$ ” 的逆命题;\n\n(2) “若 $\\angle A=\\angle B$, 则 $\\sin A=\\sin B$ ” 的逆否命题;\n\n(3) “若 $a+b$ 是无理数, 则 $a, b$ 都是无理数” 的逆命题;\n\n(4) “若 $x^{2}<4$, 则 $-20$ 或 $x_{0}-2=0$", "solution": "null", "level": "高二", "question": "5. 若命题 $p: \\forall x \\in \\mathbf{R}, \\frac{1}{x-2}<0$, 则綈 $p$ :", "options": [], "subject": "逻辑题", "analysis": ": 綈 $p: \\exists x_{0} \\in \\mathbf{R}$, 使 $\\frac{1}{x_{0}-2}>0$ 或 $x_{0}-2=0$. 最易出现的错误答案是: $\\exists x_{0} \\in \\mathbf{R}, \\frac{1}{x_{0}-2} \\geqslant 0$.\n\n答案 :$\\exists x_{0} \\in \\mathbf{R}$, 使 $\\frac{1}{x_{0}-2}>0$ 或 $x_{0}-2=0$"} {"id": "20173", "image": [], "answer": ": $[-8,+\\infty)$", "solution": "null", "level": "高二", "question": "6. 已知命题: “存在 $x \\in[1,2]$, 使 $x^{2}+2 x+a \\geqslant 0$ ” 为真命题, 则 $a$ 的取值范围是", "options": [], "subject": "逻辑题", "analysis": ": $[-8,+\\infty)$"} {"id": "18733", "image": [], "answer": "1473", "solution": "null", "level": "高二", "question": "9. 在 100 内所有能被 3 整除但不能被 7 整除的正整数之和是", "options": [], "subject": "组合数学", "analysis": "100 内所有能被 3 整除的数的和为: $S_{1}=3+6+\\cdots+99=\\frac{33 \\times(3+99)}{2}=1683$.\n\n100 内所有能被 21 整除的数的和为: $S_{2}=21+42+63+84=210$.\n\n$\\therefore 100$ 内能被 3 整除不能被 7 整除的所有正整数之和为\n\n$S_{1}-S_{2}=1683-210=1473$."} {"id": "19173", "image": [], "answer": "55", "solution": "null", "level": "高二", "question": "3. 著名的斐波那契数列, 因数学家列昂纳多 - 斐波那契以兔子繁殖为例而引入, 故还称为 “兔子数列”,它满足: $a_{1}=, a_{2}=1$, 且 $a_{n+2}=a_{n+1}+a_{n}\\left(n \\in N^{*}\\right)$, 则 $a_{10}=$", "options": [], "subject": "组合数学", "analysis": "由递推关系 $a_{n+2}=a_{n+1}+a_{n}\\left(n \\in N^{*}\\right)$, 可知前 9 项分别是 $1,1,2,3,5,8,13,21,34$,所以 $a_{10}=55$"} {"id": "18603", "image": [], "answer": "$(1+q)^{12}-1$", "solution": "null", "level": "高二", "question": "10. 某工厂月生产总值的平均增长率为 $q$, 则该工厂的年平均增长率为", "options": [], "subject": "计数", "analysis": "设第一年第 1 个月的生产总值为 1 , 公比为 $(1+q)$, 该厂第一年的生产总值为 $S_{1}$ $=1+(1+q)+(1+q)^{2}+\\cdots+(1+q)^{11}$.\n\n则第 2 年第 1 个月的生产总值为 $(1+q)^{12}$,\n\n第 2 年全年生产总值 $S_{2}=(1+q)^{12}+(1+q)^{13}+\\cdots+(1+q)^{23}=(1+q)^{12} S_{1}$,\n\n$\\therefore$ 该厂生产总值的平均增长率为 $\\frac{S_{2}-S_{1}}{S_{1}}=\\frac{S_{2}}{S_{1}}-1=(1+q)^{12}-1$."} {"id": "1294", "image": [], "answer": "-30 元", "solution": "null", "level": "七年级", "question": "若银行账户余额增加 50 元, 记作“ +50 元”, 那么银行账户余额减少 30 元记作 \\$ \\qquad \\$ .", "options": [], "subject": "算术", "analysis": "如果收入 50 元,记作 +50 元,那么支出 30 元记作 -30 元.故答案为: -30 元."} {"id": "1318", "image": [], "answer": "-14", "solution": "null", "level": "七年级", "question": "已知 $a, b$ 均为有理数, 现我们定义一种新的运算, 规定: $a \\# b=a^{2}+a b-5$, 例如: $1 \\# 2=1^{2}+1 \\times 2$ $-5=-2$, 则 $(-3) \\# 6$ 的值是", "options": [], "subject": "算术", "analysis": "$(-3) \\# 6$,\n\n$=(-3)^{2}+(-3) \\times 6-5$,\n\n$=9-18-5$\n\n$=-14$\n\n故答案为: -14 ."} {"id": "1321", "image": [], "answer": "$\\pm 4$", "solution": "null", "level": "七年级", "question": "整数 $a, b, c, d$ 的绝对值均小于 5 , 且满 $1000 a+100 b^{2}+10 c^{3}+d^{4}=2021$, 则 $a b c d$的值为 \\$ \\qquad \\$ .", "options": [], "subject": "算术", "analysis": "$\\because 1000 a+100 b^{2}+10 c^{3}+d^{4}=2021$, 整数 $a 、 b 、 c 、 d$ 的绝对值均小于 5 ,\n\n$\\therefore$ 个位上的 1 一定为 $\\mathrm{d}^{4}$ 产生,\n\n$\\because( \\pm 3)^{4}=81,( \\pm 13)^{4}=81$,\n\n$\\therefore \\mathrm{d}= \\pm 3$ 或 $\\mathrm{d}= \\pm 1$,\n\n(1) 当 $\\mathrm{d}= \\pm 1, \\mathrm{~d}^{4}=1$,\n\n$\\therefore 1000 \\mathrm{a}+100 \\mathrm{~b}^{2}+10 \\mathrm{c}^{3}=2020$,\n\n$\\therefore 100 \\mathrm{a}+10 \\mathrm{~b}^{2}+\\mathrm{c}^{3}=202$,\n\n$\\therefore$ 个位上的 2 由 $\\mathrm{c}^{3}$ 产生,\n$\\therefore c^{3}=2$ 或 -8,\n\n$\\because c$ 的绝对值小于 5 ,\n\n$\\therefore \\mathrm{c}=-2$,\n\n$\\therefore 100 a+10 b^{2}-8=202$,\n\n$\\therefore 100 \\mathrm{a}+10 \\mathrm{~b}^{2}=210$,\n\n即 $10 a+b^{2}=21$,\n\n$\\therefore$ 此时个位上的 1 一定是 $\\mathrm{b}^{2}$ 产生的,\n\n$\\because$ 绝对值小于 5 的整数中, 只有 $( \\pm 1)^{2}=1$,\n\n$\\therefore \\mathrm{b}= \\pm 1$,\n\n将 $b= \\pm 1$ 代入 $10 a+b^{2}=21$,\n\n解得 $a=2$,\n\n$\\therefore \\mathrm{a}=2, \\mathrm{~b}= \\pm 1, \\mathrm{c}=-2, \\mathrm{~d}= \\pm 1$,\n\n$\\therefore$ abcd $=\\left\\{\\begin{array}{c}2 \\times 1 \\times(-2) \\times 1=-4 \\\\ 2 \\times(-1) \\times(-2) \\times 1=4 \\\\ 2 \\times 1 \\times(-2) \\times(-1)=4 \\\\ 2 \\times(-1) \\times(-2) \\times(-1)=-4\\end{array}\\right.$,\n\n$\\therefore \\mathrm{abcd}= \\pm 4$,\n\n当 $d= \\pm 3$ 时,$d^{4}=81$,\n\n$1000 a+100 b^{2}+10 c^{3}=1940$,\n\n即 $100 a+10 b+c^{3}=194$,\n\n$\\because$ 绝对值小于 5 的整数中, 只有 $4^{3}=64$,\n\n$\\therefore \\mathrm{c}=4$,\n\n$\\therefore 100 \\mathrm{a}+10 \\mathrm{~b}^{2}=130$,\n\n即 $10 a+b^{2}=13 ,$\n\n$\\because$ 绝对值小于 5 的整数中, 不存在某个数的平方的个位是 3 或 7 ,\n\n$\\therefore \\mathrm{d}= \\pm 3$ 不符合题意,\n\n综上所述, abcd 的值为 $\\pm 4$,\n\n故答案为: $\\pm 4$."} {"id": "1342", "image": [], "answer": "7", "solution": "null", "level": "七年级", "question": ": $3-3 \\div\\left(-\\frac{3}{2}\\right) \\times 2=$", "options": [], "subject": "算术", "analysis": "原式 $=3-3 \\times\\left(-\\frac{2}{3}\\right) \\times 2=3+2 \\times 2=7$,\n\n故答案为: 7 ."} {"id": "1344", "image": [], "answer": "-4", "solution": "null", "level": "七年级", "question": "$\\mathrm{a}$ 与 $\\mathrm{b}$ 互为倒数, $\\mathrm{c}$ 是最大的负整数. $|m|=2$, 则 $a b+c-m^{2}$ 的值为", "options": [], "subject": "算术", "analysis": "$\\because a$ 与 $b$ 互为倒数, $\\mathrm{c}$ 是最大的负整数, $|\\mathrm{m}|=2$,\n\n$\\therefore a b=1, \\quad c=-1, \\quad m= \\pm 2$,\n\n$\\therefore a b+c-m^{2}$\n\n$=1-1-4$\n\n$=-4$.\n\n故答案为: -4 ."} {"id": "1345", "image": [], "answer": "8", "solution": "null", "level": "七年级", "question": "昆虫从点 $\\mathrm{A}$ 处出发, 以每分钟 2 米的速度在一条直线上运动, 它先前进 1 米, 再后退 2 米, 又前进 3 米, 再后退 4 米, ... 依此规律继续走下去, 则运动 1 小时时这只昆虫与 $\\mathrm{A}$ 点相距米.", "options": [], "subject": "算术", "analysis": "1小时 $=60$ 分,\n\n规定昆虫每前进一次和后退一次为一运动周期, 则设昆虫的运动周期数为 $n$, 每一周期所用总时间为 $t$.\n\n设每周期前进的距离为 $S$ ,则 $s=2(n-1)+1=2 n-1$ ;\n\n由题意可得: $t=2(n-1)+1.5=2 n-0.5$;\n\n假设昆虫运动所用总时间为 $\\mathrm{T}$; 则 $T=(2 \\times 1-0.5)+(2 \\times 2-0.5)+(2 \\times 3-0.5)+\\cdots+(2 \\times n-$ $0.5)=2(1+2+3+\\cdots+n)-0.5 n=n^{2}+0.5 n$;\n\n当 $T=60$ 分时, 代入上式中可得 $n=7$ 但还剩余 7.5 分钟, 由公式 $t=2(n-1)+1.5=2 n-0.5$ 可得第 8 周需要 15.5 分钟, 但是每一周期中后退时间比前进时间多 0.5 分钟, 所以在第 8 周期中前进时间为 7.5 分钟, 后退时间为 8 分钟.\n\n由于运动一个周期后退一米, 所以运动 7 个周期就后退 7 米, 由于在 60 分钟内运动完 7 周期后正好剩余 7.5 分钟, 这样在第 8 周期就正好前进的距离 $S=2 \\times 8-1=15$ 米, 故运动 1 小时时这只昆虫与 $A$ 点相距为 $15-7=8$ 米.\n\n故答案为: 8"} {"id": "1346", "image": [], "answer": "0", "solution": "null", "level": "七年级", "question": "计算: $\\left|\\frac{1}{2018}-\\frac{1}{2017}\\right|+\\left|\\frac{1}{2017}-\\frac{1}{2016}\\right|-\\left|\\frac{1}{2018}-\\frac{1}{2016}\\right|=$", "options": [], "subject": "算术", "analysis": "原式 $=\\frac{1}{2017}-\\frac{1}{2018}+\\frac{1}{2016}-\\frac{1}{2017}-\\left(\\frac{1}{2016}-\\frac{1}{2018}\\right)$\n\n$=\\frac{1}{2017}-\\frac{1}{2018}+\\frac{1}{2016}-\\frac{1}{2017}-\\frac{1}{2016}+\\frac{1}{2018}=0$,\n\n故答案为: 0 ."} {"id": "1347", "image": [], "answer": "1970", "solution": "null", "level": "七年级", "question": "一个小球落在数轴上的某点 $P_{0}$, 第一次从点 $P_{0}$ 向左跳 1 个单位长度到点 $P_{1}$, 第二次从点 $P_{1}$ 向右跳 2 个单位长度到点 $P_{2}$, 第三次从点 $P_{2}$ 向左跳 3 个单位长度到点 $P_{3}$, 第四次从点 $P_{3}$\n向右跳 4 个单位长度到点 $P_{4}, \\ldots$, 按以上规律跳了 100 次时, 它落在数轴上的点 $P_{100}$ 所表示的数恰好是 2020, 则这个小球的初始位置点 $P_{0}$ 所表示的数是 \\$ \\qquad \\$ .", "options": [], "subject": "算术", "analysis": "设 $P_{0}$ 所表示的数是 $a$, 则 $a-1+2-3+4-\\cdots-99+100=2020$ ,\n\n则 $a+(-1+2)+(-3+4)+\\cdots+(-99+100)=2020$.\n\n$a+50=2020$,\n\n解得: $a=1970$.\n\n点 $P_{0}$ 表示的数是 1970 .\n\n故答案为: 1970 ."} {"id": "1365", "image": [], "answer": "$\\pm 2 ;-1$", "solution": "null", "level": "七年级", "question": "如果 $x^{2}=64$, 那么 $\\sqrt[3]{x}=$ \\$ \\qquad \\$ , 最小正整数与最大负整数的积等于 \\$ \\qquad \\$ .", "options": [], "subject": "算术", "analysis": "$\\because \\mathrm{x}^{2}=64$,\n\n$\\therefore \\mathrm{x}= \\pm 8$\n\n$\\therefore \\sqrt[3]{-8}=-2, \\sqrt[3]{8}=2$\n\n最小正整数与最大负整数的积等于 $1 \\times(-1)=-1$.\n\n故答案为: $\\pm 2,-1$."} {"id": "1368", "image": [], "answer": "$\\pm 3$", "solution": "null", "level": "七年级", "question": "对于有理数 $a, \\mathrm{~b}$, 定义 $\\min \\{a, \\mathrm{~b}\\}$ 的含义为: 当 $a<\\mathrm{b}$ 时, $\\min \\{a, \\mathrm{~b}\\}=a$ , 当 $a>b$ 时, $\\min \\{a, \\mathrm{~b}\\}=b$. 例如: $\\min \\{1,-2\\}=-2, \\min \\{3,-1\\}=-1$. 已知 $\\min \\{\\sqrt{21}, a\\}=\\sqrt{21}$, $\\min \\{\\sqrt{21}, \\mathrm{~b}\\}=\\mathrm{b}$, 且 $a$ 和 $\\mathrm{b}$ 为两个连续正整数, 则 $a+b$ 的平方根为 \\$ \\qquad \\$ .", "options": [], "subject": "算术", "analysis": "$\\because \\min \\{\\sqrt{21}, a\\}=\\sqrt{21}, \\min \\{\\sqrt{21}, b\\}=b, \\mathrm{a}, \\mathrm{b}$ 为两个连续的正整数, $\\therefore 4<\\sqrt{21}<5$\n$\\therefore \\mathrm{a}=4, \\mathrm{~b}=5$\n\n$\\therefore \\mathrm{a}+\\mathrm{b}=4+5=9$,\n\n$\\therefore \\mathrm{a}+\\mathrm{b}$ 的平方根为 $\\pm 3$.\n\n故答案为: $\\pm 3$."} {"id": "1369", "image": [], "answer": "(4) (5)", "solution": "null", "level": "七年级", "question": "给出下列判断: (1)若 $|a|=a$ ,则 $a>0$ ; (2)有理数包括整数、 0 和分数; (3)任何正数都大于它的倒数; (4) 代数式 $a^{2}+1$ 的值永远是正的; (5) $\\frac{\\sqrt{5}-1}{2}>\\frac{1}{2}$; 其中判断正确的有 \\$ \\qquad \\$ ( 填写序号即可)", "options": [], "subject": "算术", "analysis": "(1)若 $|a|=a ,$ 则 $a \\geqslant 0 ,$ 本说法错误;\n\n(2)有理数包括整数和分数, 本说法错误;\n\n(3)小于 1 的正数都大于它的倒数, 本说法错误;\n\n(4) $\\because a^{2} \\geqslant 0$\n\n$\\therefore$ 代数式 $a^{2}+1$ 的值永远是正的, 本说法正确;\n\n(5) $\\because 2<\\sqrt{5}<3$\n\n$\\therefore 1<\\sqrt{5}-1<2$\n\n$\\therefore \\frac{\\sqrt{5}-1}{2}>\\frac{1}{2}$, 本说法正确;\n\n故答案为: (4)(5)."} {"id": "1370", "image": [], "answer": "$\\frac{2}{3}$", "solution": "null", "level": "七年级", "question": "我们知道, 无限循环小数都可以转化为分数.例如: 将 $0 . \\dot{3}$ 转化为分数时, 可设 $0 . \\dot{3}=x$, 则 $x=$ $0.3+\\frac{1}{10} x$, 解得 $x=\\frac{1}{3}$, 即 $0 . \\dot{3}=\\frac{1}{3}$. 仿此方法, 将 $0 . \\dot{6}$ 化成分数是 \\$ \\qquad \\$", "options": [], "subject": "算术", "analysis": "设 $0 . \\dot{6}=x$, 则\n\n$x=0.6+\\frac{1}{10} x$\n\n解得: $x=\\frac{2}{3}$,\n\n故答案为: $\\frac{2}{3}$"} {"id": "1389", "image": [], "answer": "$9 ; 4 ; 2$", "solution": "null", "level": "七年级", "question": "计算: $(-3)^{2}=$ \\$ \\qquad \\$ , $\\sqrt{16}=$ , $\\sqrt[3]{8}=$", "options": [], "subject": "算术", "analysis": "$(-3)^{2}=9, \\sqrt{16}=4, \\sqrt[3]{8}=2$,\n\n故答案为: $9 ; 4 ; 2$."} {"id": "1390", "image": [], "answer": "例: $-\\sqrt{2}+\\sqrt{2}=0$ (答案不唯一)", "solution": "null", "level": "七年级", "question": "试举一例, 说明“两个无理数的和仍是无理数” 是错误的: \\$ \\qquad \\$", "options": [], "subject": "算术", "analysis": "如 $-\\sqrt{2}+\\sqrt{2}=0$, 答案不唯一.\n\n$\\therefore$ 两个无理数的和仍是无理数是错误的。\n\n故答案为: $-\\sqrt{2}+\\sqrt{2}=0$ (答案不唯一)."} {"id": "1391", "image": [], "answer": "$\\frac{2}{7}$", "solution": "null", "level": "七年级", "question": "两个不相等的实数 $\\mathrm{a}, \\mathrm{b}$, 定义一种新的运算: $a * b=\\frac{\\sqrt{a+b}}{a-b}$, 例如: $3 * 2=\\frac{\\sqrt{3+2}}{3-2}=\\sqrt{5}$, 那么 $15 *(6 * 3)=$ \\$ \\qquad \\$", "options": [], "subject": "算术", "analysis": "根据题中的新定义得: $15 *(6 * 3)=15 * \\frac{\\sqrt{6+3}}{6-3}=15 * 1=\\frac{\\sqrt{15+1}}{15-1}=\\frac{2}{7}$\n\n故答案为: $\\frac{2}{7}$."} {"id": "1393", "image": [], "answer": "), (4)", "solution": "null", "level": "七年级", "question": "left[x\\right.$ )表示小于 $x$ 的最大整数, 如 $[2.3)=2,[-4)=-5$, 则下列判断: (1) $\\left[-8 \\frac{3}{5}\\right)=-8$; (2) $[x)-x$ 有最大值是 0 ; (3) $[x)-x$ 有最小值是 -1 ; (4) $x-1 \\leq[x)2$,\n\n$\\therefore \\mathrm{m}>2$,\n\n$\\because \\sqrt{47}<\\sqrt{49}=7$,\n\n$\\therefore \\mathrm{m}<7-2=5$,\n\n$\\therefore 2<\\mathrm{m}<5$, 且 $\\mathrm{m}$ 为整数,\n\n$\\therefore$ 整数 $\\mathrm{m}$ 的值为: 3,4 .\n\n故答案为: 3,4 ."} {"id": "1415", "image": ["3123.jpg"], "answer": "$\\pm 10$", "solution": "null", "level": "七年级", "question": "一般地, 如果 $x^{4}=a(a \\geq 0)$, 则称 $x$ 为 $a$ 的四次方根, 一个正数 $a$ 的四次方根有两个, 它们互为相\n\n\\$ \\qquad \\$", "options": [], "subject": "算术", "analysis": "$\\quad 4 \\sqrt{\\mathrm{m}^{4}}=10, \\therefore \\mathrm{m}^{4}=10^{4}, \\therefore \\mathrm{m}= \\pm 10$."} {"id": "1416", "image": [], "answer": "pm 2 ;-1$", "solution": "null", "level": "七年级", "question": "如果 $x^{2}=64$, 那么 $\\sqrt[3]{x}=$ \\$ \\qquad \\$ , 最小正整数与最大负整数的积等于 \\$ \\qquad \\$ .", "options": [], "subject": "算术", "analysis": "$\\because \\mathrm{x}^{2}=64$,\n\n$\\therefore \\mathrm{x}= \\pm 8$\n\n$\\therefore \\sqrt[3]{-8}=-2, \\sqrt[3]{8}=2$\n\n最小正整数与最大负整数的积等于 $1 \\times(-1)=-1$.\n\n故答案为: $\\pm 2,-1$."} {"id": "1417", "image": [], "answer": "3", "solution": "null", "level": "七年级", "question": "若 $(m-2)^{2}+|n+7|=0$, 则 m-n 的算术平方根是 \\$ \\qquad \\$ .", "options": [], "subject": "算术", "analysis": "由题意得: $\\mathrm{m}-2=0, \\mathrm{n}+7=0$,\n\n$\\therefore \\mathrm{m}=2, \\mathrm{n}=-7$,\n\n$\\therefore \\mathrm{m}-\\mathrm{n}=2-(-7)=9$,\n\n$\\therefore \\mathrm{m}-\\mathrm{n}$ 的算术平方根是 3 .\n\n故答案为: 3 ."} {"id": "1437", "image": [], "answer": "$-\\sqrt{3}$", "solution": "null", "level": "七年级", "question": "这三个数 $-\\sqrt{3} 、-\\frac{1}{3} 、-1$ 中, 最小的数是 \\$ \\qquad \\$ .", "options": [], "subject": "算术", "analysis": "$\\because 1<3<4$,\n\n$\\therefore 1<\\sqrt{3}<2$,\n\n$\\therefore \\frac{1}{3}<1<\\sqrt{3}<2$,\n\n$\\therefore-\\sqrt{3} 、-\\frac{1}{3} 、-1$ 中最小的数是 $-\\sqrt{3}$.\n\n故答案为: $-\\sqrt{3}$."} {"id": "1438", "image": [], "answer": "-2 或 -18", "solution": "null", "level": "七年级", "question": "若有理数 $x, y$ 满足 $|y|=10, x^{2}=64$, 且 $|x-y|=x-y$, 则 $x+y$ 的值为 \\$ \\qquad \\$ .", "options": [], "subject": "算术", "analysis": "由 $|y|=10$ 得: $y= \\pm 10$ ,\n\n由 $x^{2}=64$ 得: $x= \\pm 8$,\n\n因为 $|x-y|=x-y$ ,\n\n所以 $x-y \\geq 0$ ,即 $x \\geq y$ ,\n\n所以 $\\mathrm{x}=8 、 \\mathrm{y}=-10$ 或 $\\mathrm{x}=-8 、 \\mathrm{y}=-10$,\n\n所以当 $x=8$ 时, $y=-10, x+y=8-10=-2$;\n\n当 $x=-8$ 时, $y=-10, x+y=-8-10=-18$.\n\n故答案为: -2 或 -18 ."} {"id": "1439", "image": [], "answer": "$5 ; \\sqrt[3]{10}$", "solution": "null", "level": "七年级", "question": "已知 $\\sqrt{x+4}=3$, 则 $x=$ \\$ \\qquad \\$ $;(-\\sqrt{10})^{2}$ 的立方根是 . \\$ \\qquad \\$", "options": [], "subject": "算术", "analysis": "$\\because \\sqrt{x+4}=3$ ,\n\n$\\therefore x+4=9$,\n\n解得 $x=5$;\n\n$\\because(-\\sqrt{10})^{2}=10$,\n\n$\\therefore 10$ 的立方根是 $\\sqrt[3]{10}$.\n\n故答案为: $5 ; \\sqrt[3]{10}$."} {"id": "1442", "image": [], "answer": "-4", "solution": "null", "level": "七年级", "question": "我们用符号 $[\\mathrm{x}]$ 表示一个不大于实数 $\\mathrm{x}$ 的最大整数, 如: [3.69]=3, $[-0.56]=-1$, 则按这个规律 $[-\\sqrt{5}-1]=$ \\$ \\qquad \\$ .", "options": [], "subject": "算术", "analysis": "$\\because 2<\\sqrt{5}<3$,\n\n$\\therefore-4<-\\sqrt{5}-1<-3$,\n\n$\\therefore[-\\sqrt{5}-1]=-4$.\n\n故答案为: -4 ."} {"id": "1295", "image": ["3090.jpg"], "answer": "$2 b$", "solution": "null", "level": "七年级", "question": "已知有理数 $a 、 b$ 在数轴上的位置如图所示, 化简 $|a-b|+|a+b|$ 的结果为 \\$ \\qquad \\$ .\n\n", "options": [], "subject": "度量几何学", "analysis": "由数轴知: $\\mathrm{b}>0, \\mathrm{a}<0,|\\mathrm{~b}|>|\\mathrm{a}|$\n\n$\\therefore \\mathrm{a}-\\mathrm{b}<0, \\mathrm{a}+\\mathrm{b}>0$.\n\n$\\therefore|a-b|+|a+b|$\n\n$=-(\\mathrm{a}-\\mathrm{b})+(\\mathrm{a}+\\mathrm{b})$\n\n$=-a+b+a+b$\n\n$=2 b$.\n\n故答案为: $2 b$."} {"id": "1297", "image": [], "answer": "1或-9", "solution": "null", "level": "七年级", "question": "数轴上 $\\mathrm{A}$ 点表示的数为 4 , 点 $\\mathrm{B}$ 与点 $\\mathrm{A}$ 位于原点两侧且到原点的距离相等, 点 $\\mathrm{C}$ 与点 $\\mathrm{B}$ 的距离为 5 , 则点 $\\mathrm{C}$ 表示的数为", "options": [], "subject": "度量几何学", "analysis": "$\\because$ 数轴上 $\\mathrm{A}$ 点表示的数为 4 ,\n\n$\\therefore A$ 到原点的距离是 4,\n\n又 $\\because$ 点 $\\mathrm{B}$ 与点 $\\mathrm{A}$ 位于原点两侧且到原点的距离相等,\n\n$\\therefore$ 点 B 所表示的数为 -4,\n\n$\\therefore$ 到 -4 的距离是 5 的点对应的数是 $-4+5=1$ 或 $-4-5=-9$.\n\n即点 C 所表示的有理数为 1 或 -9.\n\n故答案为: 1 或 -9 ."} {"id": "1298", "image": ["3092.jpg"], "answer": "$\\frac{5}{2}$ 或 $\\frac{35}{4}$", "solution": "null", "level": "七年级", "question": "如图, 已知 $A, B$ 两点在数轴上, 点 $A$ 表示的数为 -10 , 点 $B$ 表示的数为 30 , 点 $M$ 以每 6 个单位长度的速度从点 $\\mathrm{A}$ 向右运动, 点 $\\mathrm{N}$ 以每秒 2 个单位长度的速度从点 $\\mathrm{O}$ 向右运动, 其中点 $\\mathrm{M}$ 、点 $\\mathrm{N}$同时出发,经过秒, 点 $\\mathrm{M} 、$ 点 $\\mathrm{N}$ 分别到点 $\\mathrm{B}$ 的距离相等.\n\n", "options": [], "subject": "度量几何学", "analysis": "设经过 $\\mathrm{t}$ 秒, 点 $\\mathrm{M}$ 、点 $\\mathrm{N}$ 分别到点 $\\mathrm{B}$ 的距离相等, 则点 $\\mathrm{M}$ 所表示的数为 $(-10+6 \\mathrm{t})$, 点 $\\mathrm{N}$所表示的数为 $2 \\mathrm{t}$,\n\n(1) 当点 $\\mathrm{B}$ 是 $M N$ 的中点时, 有 $\\frac{-10+6 t+2 t}{2}=30$,\n\n解得: $t=\\frac{35}{4}$,\n\n(2)当点 $\\mathrm{M}$ 与点 $\\mathrm{N}$ 重合时, 有 $2 t=-10+6 t$,\n\n解得: $t=\\frac{5}{2}$,\n\n因此, $t=\\frac{5}{2}$ 或 $t=\\frac{35}{4}$,\n故答案为: $\\frac{5}{2}$ 或 $\\frac{35}{4}$."} {"id": "1299", "image": ["3093.jpg", "3094.jpg"], "answer": "-2", "solution": "null", "level": "七年级", "question": "1, 在一条可以折叠的数轴上有点 A, B, C, 其中点 A, 点 B 表示的数分别为 -16 和 9 ,现以点 $C$ 为折点, 将数轴向右对折, 点 $A$ 对应的点 $A_{1}$ 落在 $B$ 的右边; 如图 2, 再以点 $B$ 为折点,将数轴向左折叠, 点 $\\mathrm{A}_{1}$ 对应的点 $\\mathrm{A}_{2}$ 落在 $\\mathrm{B}$ 的左边. 若 $\\mathrm{A}_{2} 、 \\mathrm{~B}$ 之间的距离为 3 , 则点 $\\mathrm{C}$ 表示的数为 \\$ \\qquad \\$\n\n\n\n图 1\n\n\n\n图 2", "options": [], "subject": "度量几何学", "analysis": "由题意得: 点 $A_{1}, B$ 之间的距离与点 $A_{2}, B$ 之间的距离相等, 即为 3 ,\n\n因为点 $B$ 表示的数为 9 , 且点 $A_{1}$ 在点 $B$ 的右边,\n\n所以点 $A_{1}$ 表示的数为 $9+3=12$,\n\n因为点 $A$ 表示的数为 -16 , 点 $A_{1}$ 是点 $A$ 以点 $C$ 为折点的对应点,\n\n所以点 $C$ 表示的数为 $(-16+12) \\div 2=-2$.\n\n故答案为: -2 ."} {"id": "2431", "image": ["3521.jpg"], "answer": "\\frac{3+2 \\sqrt{2}}{4}", "solution": "null", "level": "七年级", "question": "如图, 扇形 $O A B, \\angle A O B=90^{\\circ}, \\odot P$ 与 $O A 、 O B$ 分别相切于点 $F 、 E$,并且与弧 $A B$ 切于点 $C$, 则扇形 $O A B$ 的面积与 $\\odot P$ 的面积比是\n\n\n第 15 题", "options": [], "subject": "度量几何学", "analysis": "\\frac{3+2 \\sqrt{2}}{4}"} {"id": "1392", "image": [], "answer": "11", "solution": "null", "level": "七年级", "question": "上点 $A, B$ 分别表示实数 $\\sqrt{5}-1$ 与 $\\sqrt{5}+10$, 则点 $A$ 距点 $B$ 的距离为 \\$ \\qquad \\$", "options": [], "subject": "度量几何学", "analysis": "$|\\sqrt{5}-1-(\\sqrt{5}+10)|=|\\sqrt{5}-1-\\sqrt{5}-10|=11$.\n\n所以答案为 11 ."} {"id": "2449", "image": [], "answer": "$-\\frac{\\sqrt{3}}{12}$.", "solution": "null", "level": "七年级", "question": "(3 分) 计算: $\\sin 30^{\\circ} \\cdot \\cos 30^{\\circ}-\\tan 30^{\\circ}=-\\frac{\\sqrt{3}}{12}$ -", "options": [], "subject": "度量几何学", "analysis": "根据 $\\sin 30^{\\circ}=\\frac{1}{2}, \\cos 30^{\\circ}=\\frac{\\sqrt{3}}{2}, \\tan 30^{\\circ}=\\frac{\\sqrt{3}}{3}$ 代入计算即可.\n\n解答: 解: 原式 $=\\frac{1}{2} \\times \\frac{\\sqrt{3}}{2}-\\frac{\\sqrt{3}}{3}=\\frac{\\sqrt{3}}{4}-\\frac{\\sqrt{3}}{3}=-\\frac{\\sqrt{3}}{12}$."} {"id": "2451", "image": ["3534.jpg"], "answer": "2", "solution": "null", "level": "七年级", "question": "(3 分) 如图, $\\mathrm{CD}$ 是 $\\mathrm{Rt} \\triangle \\mathrm{ABC}$ 斜边上的高线, 若 $\\sin \\mathrm{A}=\\frac{\\sqrt{3}}{3}, \\mathrm{BD}=1$, 则 $\\mathrm{AD}=\\underline{2}$.\n\n", "options": [], "subject": "度量几何学", "analysis": "证明 $\\angle \\mathrm{A}=\\angle \\mathrm{BCD}$, 求出 $\\mathrm{BC}$ 的长; 进而求 $\\mathrm{CD} 、 \\mathrm{AC}$, 运用勾股定理求 $\\mathrm{AD}$.\n\n解答: 解: $\\because \\mathrm{CD}$ 是 Rt $\\triangle \\mathrm{ABC}$ 斜边上的高线,\n\n$\\therefore \\angle \\mathrm{A}=\\angle \\mathrm{BCD}$.\n\n$\\because \\sin A=\\frac{\\sqrt{3}}{3}$\n\n$\\therefore \\sin \\angle B C D=\\frac{\\sqrt{3}}{3}=\\frac{B D}{B C}$.\n\n$\\because \\mathrm{BD}=1$\n\n$\\therefore \\mathrm{BC}=\\sqrt{3}$,\n\n$\\therefore \\mathrm{CD}=\\sqrt{2}$.\n\n$\\because \\sin A=\\frac{\\sqrt{3}}{3}=\\frac{C D}{A C}$\n\n$\\therefore \\mathrm{AC}=\\sqrt{6}$.\n\n$\\therefore \\mathrm{AD}=2$."} {"id": "2452", "image": ["3535.jpg"], "answer": "\\sqrt{2}", "solution": "null", "level": "七年级", "question": "(3 分) 如图, $A B$ 是半圆 $O$ 的直径, $A C=A D, O C=2, \\angle C A B=30^{\\circ}$, 则点 $O$ 到 $C D$ 的距离 $\\mathrm{OE}=$ $\\sqrt{2}$\n\n", "options": [], "subject": "度量几何学", "analysis": "在等腰 $\\triangle \\mathrm{ACD}$ 中, 顶角 $\\angle \\mathrm{A}=30^{\\circ}$, 易求得 $\\angle \\mathrm{ACD}=75^{\\circ}$; 根据等边对等角, 可得:\n\n$\\angle O C A=\\angle A=30^{\\circ}$, 由此可得, $\\angle O C D=45^{\\circ}$; 即 $\\triangle C O E$ 是等腰直角三角形, 则 $O E=\\sqrt{2}$.\n\n解答: 解: $\\because \\mathrm{AC}=\\mathrm{AD}, \\angle \\mathrm{A}=30^{\\circ}$;\n\n$\\therefore \\angle \\mathrm{ACD}=\\angle \\mathrm{ADC}=75^{\\circ}$\n\n$\\because \\mathrm{AO}=\\mathrm{OC}$\n\n$\\therefore \\angle \\mathrm{OCA}=\\angle \\mathrm{A}=30^{\\circ} ;$\n\n$\\therefore \\angle \\mathrm{OCD}=45^{\\circ}$, 即 $\\triangle \\mathrm{OCE}$ 是等腰直角三角形.\n\n在等腰 Rt $\\triangle \\mathrm{OCE}$ 中, $\\mathrm{OC}=2$; 因此 $\\mathrm{OE}=\\sqrt{2}$."} {"id": "2453", "image": ["3536.jpg", "3537.jpg"], "answer": "$3 \\sqrt{5}$.\n\n", "solution": "null", "level": "七年级", "question": "(3 分) 如图, 在坡度为 $1: 2$ 的山坡上种树, 要求株距 (相邻两树间的水平距离) 是 $6 \\mathrm{~m}$,则斜坡上相邻两树间的坡面距离是 $3 \\sqrt{5} \\_\\mathrm{m}$.\n\n", "options": [], "subject": "度量几何学", "analysis": "利用垂直距离: 水平宽度得到水平距离与斜坡的比, 把相应的数值代入即可.\n\n解答: 解: $\\because$ 坡度为 $1: 2, \\sqrt{1^{2}+2^{2}}=\\sqrt{5}$, 且株距为 6 米,\n\n$\\therefore$ 株距: 坡面距离 $=2: \\sqrt{5}$,\n\n$\\therefore$ 坡面距离 $=$ 株距 $\\times \\frac{\\sqrt{5}}{2}=3 \\sqrt{5}$ (米).\n\n另解: $\\because \\mathrm{CB}: \\mathrm{AB}=1: 2$,\n\n设 $\\mathrm{CB}=\\mathrm{x}, \\mathrm{AB}=2 \\mathrm{x}$,\n\n$\\therefore \\mathrm{AC}=\\sqrt{\\mathrm{x}^{2}+(2 \\mathrm{x})^{2}}=\\sqrt{5} \\mathrm{x}$,\n\n$\\therefore \\frac{A B}{A C}=\\frac{2}{\\sqrt{5}}$\n\n$\\because \\mathrm{AB}=6 \\mathrm{~m}$,\n\n$\\therefore A C=\\frac{\\sqrt{5}}{2} \\times 6=3 \\sqrt{5} \\mathrm{~m}$."} {"id": "2455", "image": ["3540.jpg"], "answer": "$60^{\\circ}$.\n\n", "solution": "null", "level": "七年级", "question": "(3 分) 在 $\\triangle \\mathrm{ABC}$ 中, $\\mathrm{AC}=\\sqrt{6}, \\mathrm{BC}=2, \\angle \\mathrm{A}=45^{\\circ}$, 则 $\\angle B=60^{\\circ}$.", "options": [], "subject": "度量几何学", "analysis": "作 $C D \\perp A B$ 于 $D$, 在 $R t \\triangle A C D$ 中, 根据 $\\angle A$ 的正弦可计算出 $C D=\\sqrt{3}$, 然后在 $R t \\triangle C D B$中, 计算出 $\\sin \\mathrm{B}=\\frac{\\sqrt{3}}{2}$, 再根据特殊角的三角函数值确定 $\\angle \\mathrm{B}=60^{\\circ}$.\n\n解答: 解: 如图, 作 $\\mathrm{CD} \\perp \\mathrm{AB}$ 于 $\\mathrm{D}$,\n\n在 Rt $\\triangle \\mathrm{ACD}$ 中, $\\mathrm{AC}=\\sqrt{6}, \\angle \\mathrm{A}=45^{\\circ}$,\n\n$\\therefore \\sin \\mathrm{A}=\\sin 45^{\\circ}=\\frac{C D}{A C}$,\n\n$\\therefore \\mathrm{CD}=\\sqrt{6} \\times \\frac{\\sqrt{2}}{2}=\\sqrt{3}$,\n\n在 Rt $\\triangle \\mathrm{CDB}$ 中, $\\sin \\mathrm{B}=\\frac{\\mathrm{CD}}{\\mathrm{BC}}=\\frac{\\sqrt{3}}{2}$,\n\n$\\therefore \\angle \\mathrm{B}=60^{\\circ}$."} {"id": "2458", "image": ["3545.jpg", "3546.jpg"], "answer": "$ 2\\pi $\n\n", "solution": "null", "level": "七年级", "question": "(3 分) 水管的外部需要包扎, 包扎时用带子缠绕在管道外部. 若要使带子全部包住管道且不重叠 (不考虑管道两端的情况), 需计算带子的缠绕角度 $\\alpha$ ( $\\alpha$ 指缠绕中将部分带子拉成图中所示的平面 $\\mathrm{ABCD}$ 时的 $\\angle \\mathrm{ABC}$, 其中 $\\mathrm{AB}$ 为管道侧面母线的一部分). 若带子宽度为 1 , 水管直径为 2 , 则 $\\alpha$ 的余弦值为 $-\\frac{1}{2 \\pi}-$.\n\n", "options": [], "subject": "度量几何学", "analysis": "本题使带子全部包住管道且不重叠 (不考虑管道两端的情况), 即斜边长为水管的周长为 $2 \\pi$.\n\n解答: 解: 其展开图如图所示.\n\n$\\because$ 水管直径为 2,\n\n$\\therefore$ 水管的周长为 $2 \\pi$,\n\n$\\therefore \\cos \\angle \\alpha=\\frac{1}{2 \\pi}$."} {"id": "1489", "image": ["3152.jpg"], "answer": "$3 n+7$", "solution": "null", "level": "七年级", "question": "如图, 有一种塑料杯子的高度是 $10 \\mathrm{~cm}$, 两个以及三个这种杯子叠放时高度如图所示, 第 $n$ 个这\n种杯子叠放在一起高度是 $\\mathrm{cm}$ (用含 $n$ 的式子表示).\n\n\n第 $n$ 个", "options": [], "subject": "度量几何学", "analysis": "由图可得,\n\n每增加一个杯子, 高度增加 $3 \\mathrm{~cm}$,\n\n则 $\\mathrm{n}$ 个这样的杯子叠放在一起高度是: $10+3(\\mathrm{n}-1)=(3 \\mathrm{n}+7) \\mathrm{cm}$,\n\n故答案为: $3 \\mathrm{n}+7$."} {"id": "1537", "image": ["3168.jpg"], "answer": "24", "solution": "null", "level": "七年级", "question": "如图所示, 大长方形 $A B C D$ 被分割成 3 个大小不同的正方形(1)、(2)、(3)和 2 个小长方形 (4)、(5), 其中阴影部分的周长之和为 20 , 且 $A B: B C=3: 2$, 则大长方形 $A B C D$ 的面积\n为\n\n", "options": [], "subject": "度量几何学", "analysis": "设(1)正方形的边长为 $\\mathrm{a}$, (3)正方形的边长为 $\\mathrm{b}$, (4)长方形的宽为 $\\mathrm{c}$,则(1)(2)(3)(4)(5)的长与宽分别表示为:\n\n(1)长为 $a$, 宽为 $a$, (2)长为 $a-b$, 宽为 $a-b$, (3)长为 $b$, 宽为 $b$, (4)长为 $\\mathrm{a}-2 \\mathrm{~b}$, 宽为 $\\mathrm{c}$,\n\n(5)长为 $a+b$, 宽为 $c-b$, 又 $\\because$ 大长方形的长为: $a+b+a-2 b=2 a-b$, 宽为: $a-b+c$,\n\n又 $\\because \\frac{2 a-b}{a-b+c}=\\frac{3}{2}$,\n\n$\\therefore 4 a-2 b=3 a-3 b+3 c$,\n\n$\\therefore \\mathrm{c}=\\frac{1}{3} \\quad(\\mathrm{a}+\\mathrm{b})$,\n\n又 $\\because$ (1)和(4)的周长和为: $4 a+2 c+2(a-b)=20$,\n\n$\\therefore 4 \\mathrm{a}+2 \\times \\frac{1}{3}(\\mathrm{a}+\\mathrm{b})+2(\\mathrm{a}-\\mathrm{b})=20$,\n\n化简可得: $2 a-b=6$, 即大长方形的长为 6,\n\n$\\therefore$ 长方形的宽为 $6 \\div \\frac{3}{2}=4$,\n\n$\\therefore$ 长方形的面积为 $6 \\times 4=24$,\n\n故答案为: 24 ."} {"id": "1557", "image": [], "answer": "$60^{\\circ}$", "solution": "null", "level": "七年级", "question": "已知 $\\angle \\alpha$ 的补角是它的余角的 4 倍, 则 $\\angle \\alpha=$", "options": [], "subject": "度量几何学", "analysis": "$\\angle \\alpha$ 的补角是 $180^{\\circ}-\\angle \\alpha, \\angle \\alpha$ 的余角是 $90^{\\circ}-\\angle \\alpha$,\n\n根据题意得: $180^{\\circ}-\\angle \\alpha=4\\left(90^{\\circ}-\\angle \\alpha\\right)$,\n\n解得: $\\angle \\alpha=60^{\\circ}$.\n\n故答案为: $60^{\\circ}$."} {"id": "1296", "image": ["3091.jpg"], "answer": "-2", "solution": "null", "level": "七年级", "question": "如图是一个正方体纸盒的展开图, 正方体的各面标有数 $1,2,3,-3, A, B$, 相对面上的两个数互为相反数, 则 $A=$\n\n", "options": [], "subject": "立体几何学", "analysis": "由正方体的平面展开图可得,\n\n1 的对面是 B,\n\n2 的对面是 A,\n\n3 的对面是-3,\n\n因为相对面上的两个数互为相反数,\n\n所以 $\\mathrm{B}$ 为 $-1, \\mathrm{~A}$ 为 -2 ,\n\n故答案为: -2."} {"id": "1824", "image": [], "answer": "sqrt[3]{341}$", "solution": "null", "level": "七年级", "question": "小红做了一个棱长为 $5 \\mathrm{~cm}$ 的正方体盒子, 小明说: “我做的盒子的体积比你的大 $216 \\mathrm{~cm}^{3}$,\"则小明的盒子的棱长为 cm.", "options": [], "subject": "立体几何学", "analysis": "析】略"} {"id": "1319", "image": [], "answer": "1", "solution": "null", "level": "七年级", "question": "$m$ 与 $n$ 互为相反数, $x 、 y$ 互为倒数, 则 $3 m+2 x y+3 n-1$ 的值为", "options": [], "subject": "代数", "analysis": "$\\because \\mathrm{m} 、 \\mathrm{n}$ 互为相反数, $\\mathrm{x} 、 \\mathrm{y}$ 互为倒数,\n\n$\\therefore \\mathrm{m}+\\mathrm{n}=0, x y=1$,\n\n$\\therefore 3 m+2 x y+3 n-1=3(m+n)+2 x y-1=0+2-1=1$,\n\n故答案为: 1 ."} {"id": "1322", "image": [], "answer": "4", "solution": "null", "level": "七年级", "question": "知道: $|5-2|$ 表示 5 与 2 的差的绝对值, 也可理解为 5 与 2 两数在数轴上所对应的两点之间的距离; $|5+2|$ 也可以看成 $|5-(-2)|$, 表示 5 与 -2 之差的绝对值, 也可理解为数轴上表示 5 与 -2 两数在数轴上所对应的两点之间的距离事实上, 数轴上表示有理数 $a, b$ 的点 $A, B$ 的距离均可以用 $|a-b|$ 来计算. 根据以上材料, 则使 $|x+3|+|x-4|=7$ 的所有整数 $\\mathrm{x}$ 的和是", "options": [], "subject": "代数", "analysis": "$|x+3|+|x-4|=7$, 表示在数轴上 $\\mathrm{x}$ 与 -3 和 $\\mathrm{x}$ 与 4 距离之和为 7 ,\n\n$\\because-3$ 与 4 之间的距离是 7,\n\n$\\therefore \\mathrm{x}$ 的取值范围为 $-3 \\leq \\mathrm{x} \\leq 4$,\n\n$\\therefore$ 符合条件的整数为 $-3,-2,-1,0,1,2,3,4$.\n\n$-3+(-2)+(-1)+0+1+2+3+4=4$\n\n故答案为: 4 ."} {"id": "1343", "image": [], "answer": "-1", "solution": "null", "level": "七年级", "question": "若 $|x-2|+(y+3)^{2}=0$, 则 $(x+y)^{2021}=$", "options": [], "subject": "代数", "analysis": "$\\because|x-2|+(y+3)^{2}=0$,\n\n$\\therefore \\mathrm{x}-2=0, \\mathrm{y}+3=0$,\n\n解得: $x=2, y=-3$,\n\n故 $(\\mathrm{x}+\\mathrm{y})^{2021}=(2-3)^{2021}=-1$.\n\n故答案为: -1 ."} {"id": "1366", "image": [], "answer": "$-3 ; 9$", "solution": "null", "level": "七年级", "question": "3 是 $m$ 的一个平方根, 则 $m$ 的另一个平方根是 \\$ \\qquad \\$ ,$m=$ \\$ \\qquad \\$ .", "options": [], "subject": "代数", "analysis": "$\\because 3$ 是 $m$ 的一个平方根,\n\n$\\therefore \\mathrm{m}$ 的另一个平方根为 -3 , 且 $\\mathrm{m}=3^{2}=9$.\n\n所以答案为 $-3,9$."} {"id": "1367", "image": [], "answer": "49", "solution": "null", "level": "七年级", "question": "如果一个正数 $\\mathrm{x}$ 的平方根是 $2 \\mathrm{a}-3$ 和 $5-\\mathrm{a}$, 那么 $\\mathrm{x}$ 的值是 \\$ \\qquad \\$ .", "options": [], "subject": "代数", "analysis": "$\\because$ 一个正数 $\\mathrm{x}$ 的平方根是 $2 \\mathrm{a}-3$ 和 $5-\\mathrm{a}$,\n\n$\\therefore 2 \\mathrm{a}-3+5-\\mathrm{a}=0$,\n\n解得 $a=-2$,\n\n当 $a=-2$ 时 $2 a-3=-2 \\times 2-3=-7$,\n\n$\\therefore \\mathrm{x}=(-7)^{2}=49$.\n\n故答案为: 49 .\n\n【分析】 根据平方根的性质可得 $2 a-3+5-a=0$, 求出 $a$ 的值, 再将 $a$ 的值代入计算即可。"} {"id": "1413", "image": [], "answer": "1", "solution": "null", "level": "七年级", "question": "若 $\\mathrm{y}=\\sqrt{5-x}+\\sqrt{x-5}+4$ 有意义, 则 $(\\mathrm{y}-\\mathrm{x})^{2020}=$", "options": [], "subject": "代数", "analysis": "由题意得 $\\left\\{\\begin{array}{l}5-x \\geq 0 \\\\ x-5 \\geq 0\\end{array}\\right.$\n\n解之: $\\left\\{\\begin{array}{l}x \\leq 5 \\\\ x \\geq 5\\end{array}\\right.$\n\n$\\therefore \\mathrm{x}=5$\n\n$\\therefore y=4$\n\n$\\therefore(y-x)^{2020}=(4-5)^{2020}=1$.\n\n故答案为: 1 ."} {"id": "1418", "image": [], "answer": "2", "solution": "null", "level": "七年级", "question": "$\\left(a^{2}+b^{2}+1\\right)^{2}=9$, 那么 $a^{2}+b^{2}$ 的值是", "options": [], "subject": "代数", "analysis": "$\\because\\left(\\mathrm{a}^{2}+\\mathrm{b}^{2}+1\\right)^{2}=9$,\n\n$\\therefore a^{2}+b^{2}+1= \\pm 3$,\n\n$\\because \\mathrm{a}^{2}+\\mathrm{b}^{2} \\geq 0$\n\n$\\therefore \\mathrm{a}^{2}+\\mathrm{b}^{2}=2$."} {"id": "1440", "image": [], "answer": "$4,0.32,0, \\frac{1}{2},(-3)^{2}$", "solution": "null", "level": "七年级", "question": "在数 $4,-7,0.32,0, \\frac{1}{2},(-3)^{2}$ 中, 有平方根的是", "options": [], "subject": "代数", "analysis": "$\\because 4>0$\n\n$\\therefore 4$ 有平方根\n\n$\\because-7<0$,\n\n$\\therefore-7$ 没有平方根;\n\n$\\because 0.32>0$\n\n$\\therefore 0.32$ 有平方根\n\n$\\because \\frac{1}{2}>0$\n\n$\\therefore \\frac{1}{2}$ 有平方根;\n\n$\\because(-3)^{2}=9>0$\n\n$\\therefore(-3)^{2}$ 有平方根;\n\n$\\therefore$ 有平方根的是 $4,0.32,0, \\frac{1}{2},(-3)^{2}$.\n\n故答案为: $4,0.32,0, \\frac{1}{2},(-3)^{2}$."} {"id": "1441", "image": [], "answer": "$3 ; 4$", "solution": "null", "level": "七年级", "question": "若 $\\sqrt{3}+12$,\n\n$\\therefore \\mathrm{m}>2$,\n\n$\\because \\quad \\sqrt{47}<\\sqrt{49}=7$,\n\n$\\therefore \\mathrm{m}<7-2=5$,\n\n$\\therefore 2<\\mathrm{m}<5$, 且 $\\mathrm{m}$ 为整数,\n\n$\\therefore$ 整数 $\\mathrm{m}$ 的值为: 3,4 .\n\n故答案为: 3,4 ."} {"id": "1461", "image": [], "answer": "9", "solution": "null", "level": "七年级", "question": "已知 $s-t=12,3 m+2 n=10$, 则多项式 $2 s-4.5 m-(3 n+2 t)$ 的值为", "options": [], "subject": "代数", "analysis": "$2 s-4.5 m-(3 n+2 t)$\n\n$$\n=(2 s-2 t)-(4.5 m+3 n)\n$$\n\n$=2(s-t)-\\frac{3}{2}(3 m+2 n)$,\n\n$\\because s-t=12,3 m+2 n=10$,\n\n$\\therefore$ 原式 $=2 \\times 12-\\frac{3}{2} \\times 10$\n\n$$\n=24-15\n$$\n\n$=9$,\n\n故答案为: 9 ."} {"id": "1462", "image": [], "answer": "-1 或 1", "solution": "null", "level": "七年级", "question": "若多项式 $3 x^{a+3}-x^{3-a}+4$ 是四次三项式, 则 $a=$", "options": [], "subject": "代数", "analysis": "由题意得: (1) $a+3=4,4 \\geq 3-a \\geq 0$,\n\n解得: $a=1$,\n\n(2) $3-a=4$, 且 $4 \\geq a+3 \\geq 0$,\n\n解得: $a=-1$.\n\n故答案为: -1 或 1 ."} {"id": "1463", "image": [], "answer": "(1) 81\n\n(2) 1", "solution": "null", "level": "七年级", "question": "如果 $\\mathrm{x}$ 取任意值, 等式 $(2 x+3)^{4}=a_{0} x^{4}+a_{1} x^{3}+a_{2} x^{2}+a_{3} x+a_{4}$ 都成立, 那么,\n\n(1) $a_{4}=$\n\n(2) $a_{0}-a_{1}+a_{2}-a_{3}+a_{4}=$ \\$ \\qquad \\$ .", "options": [], "subject": "代数", "analysis": "(1) 当 $x=0$ 时, $3^{4}=a_{4}$\n$\\therefore a_{4}=81$\n\n故答案为: 81\n\n(2) 取 $x=-1$, 则有 $[2 \\times(-1)+3]^{4}=a_{0} \\times(-1)^{4}+a_{1} \\times(-1)^{3}+a_{2} \\times(-1)^{2}+a_{3} \\times(-1)+a_{4}$即 $a_{0}-a_{1}+a_{2}-a_{3}+a_{4}=1^{4}=1$\n\n故答案为: 1"} {"id": "1464", "image": [], "answer": "( $2 \\mathrm{x}-2)$", "solution": "null", "level": "七年级", "question": "已知某快递公司的收费标准: 寄一件物品不超过 5 千克, 收费 8 元; 超过 5 千克时, 超过部分每千克收费 2 元. 如果小芳的妈妈在该快递公司寄一件 $\\mathrm{x}$ 千克 $(x>5)$ 的物品, 那么她需要付的费用为元. (用含 $\\mathrm{x}$ 的代数式表示)", "options": [], "subject": "代数", "analysis": "$8+(x-5) \\times 2=8+2 x-10=2 x-2$故答案为: $(2 \\mathrm{x}-2)$."} {"id": "1485", "image": [], "answer": "-1", "solution": "null", "level": "七年级", "question": "已知 $A=2 x^{2}+x+1, B=m x+1$, 若关于 $x$ 的多项式 $A+B$ 不含一次项, 则常数 $m=$", "options": [], "subject": "代数", "analysis": "$A+B=2 x^{2}+x+1+m x+1=2 x^{2}+(m+1) x+2$,\n\n$\\because$ 若关于 $\\mathrm{x}$ 的多项式 $\\mathrm{A}+\\mathrm{B}$ 不含一次项,\n\n$\\therefore m+1=0$,\n\n解得 $m=-1$.\n\n故答案为: -1 ."} {"id": "1487", "image": [], "answer": "-2019", "solution": "null", "level": "七年级", "question": "当 $x=1$ 时, $p x^{3}+q x+1=2021$, 则当 $x=-1$ 时, $p x^{3}+q x+1=$", "options": [], "subject": "代数", "analysis": "当 $x=1$ 时, $p x^{3}+q x+1=2021$,\n\n$\\therefore \\mathrm{p}+\\mathrm{q}+1=2021$\n\n$\\therefore \\mathrm{p}+\\mathrm{q}=2020$,\n\n当 $\\mathrm{x}=-1$ 时 $\\mathrm{px}^{3}+\\mathrm{qx}+1=-\\mathrm{p}-\\mathrm{q}+1=-(\\mathrm{p}+\\mathrm{q})+1=-2020+1=-2019$.\n\n故答案为: -2019 ."} {"id": "1488", "image": [], "answer": "-13", "solution": "null", "level": "七年级", "question": "已知关于 $x$ 的代数式 $2 x^{2}-\\frac{1}{2} b x^{2}-y+6$ 和 $a x+17 x-5 y-1$ 的值都与字母 $x$ 的取值无关.则 $a+b=$", "options": [], "subject": "代数", "analysis": "由题意得: $2 x^{2}-\\frac{1}{2} b x^{2}-y+6=\\left(2-\\frac{1}{2} b\\right) x^{2}-y+6, \\quad a x+17 x-5 y-1=(a+$ 17) $x-5 y-1$,\n\n$\\because$ 它们的值都与字母 $x$ 的取值无关,\n\n$\\therefore 2-\\frac{1}{2} b=0, a+17=0$,\n\n$\\therefore a=-17, b=4$,\n\n$\\therefore a+b=-17+4=-13$;\n\n故答案为: -13 ."} {"id": "1490", "image": [], "answer": "-1", "solution": "null", "level": "七年级", "question": "真奇妙: 两个有理数 $\\mathrm{a}$ 和 $\\mathrm{b}$, 如果分别计算 $\\mathrm{a}+\\mathrm{b}, \\mathrm{a}-\\mathrm{b}, \\mathrm{ab}, \\frac{a}{b}$ 的值, 发现有三个结果恰好相同,则 $b=$ \\$ \\qquad \\$", "options": [], "subject": "代数", "analysis": "$\\because \\frac{a}{b}$ 有意义\n\n$\\therefore \\mathrm{b} \\neq 0$,\n\n$\\therefore a+b \\neq a-b$,\n\n$\\because \\mathrm{a}+\\mathrm{b}, \\mathrm{a}-\\mathrm{b}, \\mathrm{ab}, \\frac{a}{b}$ 的值有三个结果恰好相同,\n\n$\\therefore \\mathrm{ab}=\\frac{a}{b}$,\n\n$\\therefore$ 当 $\\mathrm{a}=0, \\mathrm{ab}=\\frac{a}{b}$ 成立,\n\n当 $a \\neq 0$ 时, $b=\\frac{1}{b}$ 即 $b^{2}=1$,\n\n$\\therefore \\mathrm{b}= \\pm 1$,\n\n当 $a=0$ 时 $, a+b=b, a-b=-b, a b=0, \\frac{a}{b}=0$,\n\n$\\therefore$ 此时不能有三个结果恰好相同;\n\n当 $b=1$ 时, $a+b=a+1, a-b=a-1, a b=a, \\frac{a}{b}=a$,\n\n$\\therefore$ 此时不能有三个结果恰好相同;\n\n当 $\\mathrm{b}=-1$ 时, $a+b=a-1, a-b=a+1, a b=-a, \\frac{a}{b}=-a$,\n\n$\\therefore \\mathrm{a}-1=-\\mathrm{a}$ 或 $\\mathrm{a}+1=-\\mathrm{a}$,\n\n$\\therefore \\mathrm{a}=\\frac{1}{2}$ 或 $\\mathrm{a}=-\\frac{1}{2} ;$\n\n$\\therefore$ 能使三个结果恰好相同时, $\\mathrm{b}$ 的值为 -1 ,\n故答案为: -1 ."} {"id": "1509", "image": [], "answer": "$(2 \\mathrm{x}-2)$", "solution": "null", "level": "七年级", "question": "某快递公司的收费标准:寄一件物品不超过 5 千克, 收费 8 元; 超过 5 千克时, 超过部分每千克收费 2 元. 如果小芳的妈妈在该快递公司寄一件 $\\mathrm{x}$ 千克 $(x>5)$ 的物品, 那么她需要付的费用为元. (用含 $x$ 的代数式表示)", "options": [], "subject": "代数", "analysis": "$8+(x-5) \\times 2=8+2 x-10=2 x-2$\n\n故答案为: $(2 \\mathrm{x}-2)$."} {"id": "1510", "image": [], "answer": "$\\left(45-\\frac{3}{2} a-b\\right)$", "solution": "null", "level": "七年级", "question": "七年级举行一次数学基本功大赛, 某班 45 人全部参加, 有 $\\frac{1}{2} a$ 人获得一等奖, $a$ 人获得二等奖, $b$ 人获得三等奖, 该班没有获得奖项的同学有人. (用含 $a 、 b$ 的代数式表示)", "options": [], "subject": "代数", "analysis": "由题意得: $45-\\frac{1}{2} a-a-b=\\left(45-\\frac{3}{2} a-b\\right)$ 人故答案为: $\\left(45-\\frac{3}{2} a-b\\right)$."} {"id": "1512", "image": [], "answer": "-4041 或 1", "solution": "null", "level": "七年级", "question": "有理数 $\\mathrm{a} 、 \\mathrm{~b} 、 \\mathrm{c}$ 均不为 0 , 且 $\\mathrm{a}+\\mathrm{b}+\\mathrm{c}=0$, 设 $\\mathrm{x}=\\frac{|a|}{b+c}+\\frac{|b|}{c+a}+\\frac{|c|}{a+b}$, 则代数式 $\\mathrm{x}^{2021}+2021 \\mathrm{x}-2021$的值为 \\$ \\qquad \\$ .", "options": [], "subject": "代数", "analysis": "$\\because \\mathrm{a}+\\mathrm{b}+\\mathrm{c}=0$,\n\n$\\therefore \\mathrm{b}+\\mathrm{c}=-\\mathrm{a}, \\mathrm{c}+\\mathrm{a}=-\\mathrm{b}, \\mathrm{a}+\\mathrm{b}=-\\mathrm{c}$,\n\n当 $\\mathrm{a} 、 \\mathrm{~b} 、 \\mathrm{c}$ 有一个负数时, $\\mathrm{x}=\\frac{|a|}{-a}+\\frac{|b|}{-b}+\\frac{|c|}{-c}=-1-1+1=-1$,\n\n有两个负数时, $\\mathrm{x}=\\frac{|a|}{-a}+\\frac{|b|}{-b}+\\frac{|c|}{-c}=1+1-1=1$,\n\n$\\mathrm{x}=-1$ 时, $\\mathrm{x}^{2021}+2021 \\mathrm{x}-2021=(-1)^{2021+2021 \\times(-1)}-2021=-1-2021-2021=-4041$,\n\n$x=1$ 时, $x^{2021}+2021 x-2021=1^{2021}+2021 \\times 1-2021=1+2021-2021=1$.\n\n故答案为: -4041 或 1 ."} {"id": "1514", "image": [], "answer": "5", "solution": "null", "level": "七年级", "question": "$\\left(2 x^{2}-x-1\\right)^{3}=a_{0} x^{6}+a_{1} x^{5}+a_{2} x^{4}+a_{3} x^{3}+a_{4} x^{2}+a_{5} x+a_{6}$, 求 $a_{0}+a_{2}+a_{4}=$ \\$ \\qquad \\$", "options": [], "subject": "代数", "analysis": "$\\mathrm{x}=0$ 时, $\\mathrm{a}_{6}=-1$,\n\n当 $x=1$ 时, $a_{0}+a_{1}+a_{2}+a_{3}+a_{4}+a_{5}+a_{6}=0(1)$,\n\n当 $x=-1$ 时, $a_{0}-a_{1}+a_{2}-a_{3}+a_{4}-a_{5}+a_{6}=8(2)$,\n\n(1) + (2) 得, $2 \\mathrm{a}_{0}+2 \\mathrm{a}_{2}+2 \\mathrm{a}_{4}+2 \\mathrm{a}_{6}=8$,\n\n则 $\\mathrm{a}_{0}+\\mathrm{a}_{2}+\\mathrm{a}_{4}+\\mathrm{a}_{6}=4$,\n\n$\\mathrm{a}_{0}+\\mathrm{a}_{2}+\\mathrm{a}_{4}=4-\\mathrm{a}_{6}=4+1=5$,\n\n故答案为: 5 ."} {"id": "1536", "image": [], "answer": "9", "solution": "null", "level": "七年级", "question": "当 $x=2021$ 时, $a x^{3}-b x+5$ 的值为 1 ; 则当 $x=-2021$ 时, $a x^{3}-b x+5$ 的值是", "options": [], "subject": "代数", "analysis": "$\\because$ 当 $\\mathrm{x}=2021$ 时, $\\mathrm{ax}^{3}-\\mathrm{bx}+5$ 的值为 1 ;\n\n$\\therefore 2021^{3} a-2021 b+5=1$,\n\n$\\therefore 2021^{3} a-2021 b=-4$,\n\n当 $x=-2021$ 时, 有\n\n$a x^{3}-b x+5=(-2021)^{3} a-(-2021) b+5=-2021{ }_{1}^{3} a+2021 b+5=-(-4)+5=9 ;$\n\n故答案为: 9"} {"id": "1559", "image": [], "answer": "1", "solution": "null", "level": "七年级", "question": "关于 $\\mathrm{x}$ 的方程 $\\mathrm{a}(\\mathrm{x}-3)+\\mathrm{b}(3 \\mathrm{x}+1)=5(\\mathrm{x}+1)$ 有无穷多个解, 则 $\\mathrm{a}+\\mathrm{b}=$ \\$ \\qquad \\$ .", "options": [], "subject": "代数", "analysis": "移项, 得: $a(x-3)+b(3 x+1)-5(x+1)=0$,\n\n去括号, 得: $a x-3 a+3 b x+b-5 x-5=0$,\n\n整理关于 $x$ 的方程, 得: $(a+3 b-5) x-(3 a-b+5)=0$,\n\n$\\because$ 方程有无穷多解,\n\n$\\therefore\\left\\{\\begin{array}{l}a+3 b-5=0 \\\\ 3 a-b+5=0\\end{array}\\right.$,\n\n解得: $\\left\\{\\begin{array}{c}a=-1 \\\\ b=2\\end{array}\\right.$. 则 $\\mathrm{a}+\\mathrm{b}=1$.\n\n故答案为: 1 ."} {"id": "1560", "image": [], "answer": "$\\frac{6}{9}+\\frac{6-x}{12}=1$", "solution": "null", "level": "七年级", "question": "一项工程甲单独做 9 天完成, 乙单独做 12 天完成. 现甲、乙合作一段时间后乙休假, 结果共用了 6 天完成这项工程. 设乙休假 $\\mathrm{x}$ 天,可列方程为 \\$ \\qquad \\$ .", "options": [], "subject": "代数", "analysis": "若乙休假 $\\mathrm{x}$ 天, 乙工作的天数为 $(6-x)$ 天, 根据题意得:\n\n$\\frac{6}{9}+\\frac{6-x}{12}=1$,\n\n故答案为: $\\frac{6}{9}+\\frac{6-x}{12}=1$."} {"id": "1561", "image": [], "answer": "$-\\frac{1}{2}$", "solution": "null", "level": "七年级", "question": "已知代数式 $m x+2 m$, 当 $\\mathrm{x}$ 取一个值时, 代数式 $m x+2 m$ 对应的值如下表所示.\n\n| $\\mathrm{x}$ | -1 | -0.5 | 0 | 0.5 | 1 |\n| :--- | :--- | :--- | :--- | :--- | :--- |\n| $\\mathrm{mx}+2 \\mathrm{~m}$ | 1 | 1.5 | 2 | 2.5 | 3 |\n\n则关于 $x$ 的方程 $2 m x+4 m-3=0$ 的解为 \\$ \\qquad \\$ .", "options": [], "subject": "代数", "analysis": "$2 m x+4 m-3=0$,\n\n$\\therefore 2 \\mathrm{mx}+4 \\mathrm{~m}=3$,\n\n$\\therefore \\mathrm{mx}+2 \\mathrm{~m}=1.5$,\n\n看表可得: $x=-0.5$.\n\n即 $\\mathrm{x}=-\\frac{1}{2}$.\n\n故答案为: $-\\frac{1}{2}$."} {"id": "1562", "image": [], "answer": "$-\\frac{4}{3} ; \\frac{4}{9}$", "solution": "null", "level": "七年级", "question": "一般情况下, $\\frac{m}{2}+\\frac{n}{3}=\\frac{m+n}{2+3}$ 不成立, 但是, 有些数可以使它成立, 例如, $m=n=0$, 我们称使得 $\\frac{m}{2}+\\frac{n}{3}=\\frac{m+n}{2+3}$ 成立的一对数 $m 、 n$ 为“相伴数对”, 记作 ( $m, n$ ), 如果 ( $m, 3$ ) 是“相伴数对” 那\n么 $\\mathrm{m}$ 的值是 \\$ \\qquad \\$ ; 小明发现 ( $\\mathrm{x}, \\mathrm{y})$ 是“相伴数对”, 则式子 $\\frac{x}{y}$ 的值是 \\$ \\qquad \\$ $-$", "options": [], "subject": "代数", "analysis": "第 1 小题, 根据题意得: $\\frac{\\pi}{2}+1=\\frac{\\pi+3}{5}$,\n\n去分母得: $5 m+10=2 m+6$,\n\n移项合并得: $3 \\mathrm{~m}=-4$,\n\n解得: $m=-\\frac{4}{3}$;\n\n第 2 小题, 根据题意得: $\\frac{x}{2}+\\frac{y}{3}=\\frac{x+y}{5}$,\n\n去分母得: $15 x+10 y=6 x+6 y$,\n\n移项合并得: $9 x=-4 y$,\n\n解得: $\\frac{x}{y}=-\\frac{4}{9}$."} {"id": "1581", "image": [], "answer": "0", "solution": "null", "level": "七年级", "question": "$x=$ \\$ \\qquad \\$时, 整式 $3 x-1$ 与 $2 x+1$ 互为相反数;", "options": [], "subject": "代数", "analysis": "$\\because$ 代数式 $3 x-1$ 与 $2 x+1$ 互为相反数,\n\n$\\therefore 3 x-1+2 x+1=0$,\n\n解得 $x=0$."} {"id": "1582", "image": [], "answer": "0,2", "solution": "null", "level": "七年级", "question": "若关于 $x$ 的方程 $m x=3-x$ 的解为整数, 则非负整数 $m$ 的值为 \\$ \\qquad \\$ .", "options": [], "subject": "代数", "analysis": "$m x=3-x$,\n\n移项, 得\n\n$\\mathrm{mx}+\\mathrm{x}=3$,\n\n合并同类项, 得\n\n$(\\mathrm{m}+1) \\mathrm{x}=3$,\n\n系数化为 1 , 得\n\n$\\mathrm{x}=\\frac{3}{m+1}$,\n\n$\\because$ 方程的解是整数,\n\n$\\therefore \\mathrm{m}+1=-3,-1,1,3$,\n\n$\\therefore \\mathrm{m}=-4,-2,0,2$,\n\n$\\because \\mathrm{m}$ 是非负整数,\n\n$\\therefore \\mathrm{m}=0,2$,\n\n故答案为: 0,2 ."} {"id": "1602", "image": [], "answer": "$\\mathbf{x}=1$", "solution": "null", "level": "七年级", "question": "若关于 $\\mathrm{x}$ 的方程 $\\frac{a}{3} x+1=\\frac{b}{5} x-2$ 的解为 $\\mathrm{x}=-2$, 那么关于 $\\mathrm{x}$ 的方程 $\\frac{a(x-3)}{3}+1=\\frac{b(x-3)}{5}-2$\n的解是\n\n", "options": [], "subject": "代数", "analysis": "$\\because$ 关于 $\\mathrm{x}$ 的方程 $\\frac{a}{3} x+1=\\frac{b}{5} x-2$ 的解为 $\\mathrm{x}=-2$\n\n$\\therefore$ 关于 $\\mathrm{x}$ 的方程 $\\frac{a(x-3)}{3}+1=\\frac{b(x-3)}{5}-2$ 的解为\n\n$x-3=-2$\n\n解之: $\\mathrm{x}=1$.\n\n故答案为: $\\mathrm{x}=1$."} {"id": "1584", "image": [], "answer": "$x=1000 \\mathrm{a}$", "solution": "null", "level": "七年级", "question": "已知方程 $2021 x+m=184 x+n$ 的解为 $x=a$, 则方程 $2.021 x+m=0.184 x+n$ 的解为 (用含 $\\mathrm{a}$ 的式子表示).", "options": [], "subject": "代数", "analysis": "$\\because$ 方程 $2021 \\mathrm{x}+\\mathrm{m}=184 \\mathrm{x}+\\mathrm{n}$ 的解为 $\\mathrm{x}=\\mathrm{a}$,\n\n$\\therefore 2021 \\mathrm{a}+\\mathrm{m}=184 \\mathrm{a}+\\mathrm{n}$\n\n所以 $n-m=1837 a$,\n\n而 $2.021 \\mathrm{x}+\\mathrm{m}=0.184 \\mathrm{x}+\\mathrm{n}$ 的解为 $\\mathrm{x}=\\frac{n-m}{1.837}$,\n\n把 $\\mathrm{n}-\\mathrm{m}=1837 \\mathrm{a}$ 代入 $\\mathrm{x}=\\frac{n-m}{1.837}$ 得: $\\mathrm{x}=\\frac{1837 a}{1.837}=1000 a$,\n\n$\\therefore 2.021 \\mathrm{x}+\\mathrm{m}=0.184 \\mathrm{x}+\\mathrm{n}$ 的解为 $\\mathrm{x}=1000 \\mathrm{a}$,\n\n故答案为: $x=1000 a$."} {"id": "1585", "image": [], "answer": "(1)(2) (3)", "solution": "null", "level": "七年级", "question": "甲地到乙地, 长途汽车原需行驶 7 个小时, 开通高速公路后, 路程缩短了 30 千米, 车速平均每小时增加了 30 千米, 结果只需 4 小时即可到达, ”三位同学根据愿意, 分别获得如下数量关系: (1) 设汽车原来的速度为 $x$ 千米/小时, 则 $7 x-30=4(x+30)$; (2)设甲、乙两地之间的高速公路的路程为 $y$ 千米, 则 $\\frac{y}{4}-30=\\frac{y+30}{7}$; (3)设甲、乙两地之间的普通公路的路程为 $s$ 千米, 则 $\\frac{s}{7}=\\frac{s-30}{4}-30$. 你认为其中正确的数量关系序号为 \\$ \\qquad \\$ .", "options": [], "subject": "代数", "analysis": ")等量关系: 长途汽车原来行驶路程 - 缩短的路程=甲地到乙地的高速路程, 长途汽车高速公路上行驶的速度 $\\times$ 行驶的时间 $=$ 甲地到乙地的高速路程, 所以(1)正确;\n\n(2)等量关系: 长途汽车高速公路上行驶的速度 - 增加的速度 $=$ 长途汽车原来的行驶速度, 长途汽车原来行驶路程 $\\div$ 行驶时间 $=$ 长途汽车原来的行驶速度, 所以(2)正确;\n\n(3)等量关系: 长途汽车原来行驶路程 + 行驶时间 $=$ 长途汽车原来的行驶速度, 长途汽车高速公路上行驶的速度 - 增加的速度 $=$ 长途汽车原来的行驶速度, 所以③正确.\n\n故答案为: (1) (2)."} {"id": "1607", "image": [], "answer": "$2800 \\times \\frac{x}{10}-2000=2000 \\times 12 \\%$", "solution": "null", "level": "七年级", "question": "某商品的进价是 2000 元, 标价为 2800 元, 该商品打多少折才能获得 $12 \\%$ 的利润率? 设该商品需打 $\\mathrm{x}$ 折才能使利润率为 $12 \\%$ ,根据题意列出方程: \\$ \\qquad \\$ .", "options": [], "subject": "代数", "analysis": "由题意列出方程为 $2800 \\times \\frac{x}{10}-2000=2000 \\times 12 \\%$,\n\n故答案为: $2800 \\times \\frac{x}{10}-2000=2000 \\times 12 \\%$."} {"id": "1608", "image": [], "answer": "84", "solution": "null", "level": "七年级", "question": "两位数, 个位上的数与十位上的数之和是 12 , 若交换个位与十位的位置则得到的两位数为原来数字的 $\\frac{4}{7}$, 则原来的两位数是 \\$ \\qquad \\$ .", "options": [], "subject": "代数", "analysis": "设原两位数十位上的数字为 $\\mathrm{x}$, 那么个位数字为: $12-\\mathrm{x}$,由题意得, $\\frac{4}{7}[10 x+(12-x)]=10(12-x)+x$,解得 $x=8$, $12-x=4$,所以原来的两位数是 84,故答案为: 84 ."} {"id": "1610", "image": [], "answer": "$x=-\\frac{6}{5}$", "solution": "null", "level": "七年级", "question": "小明做作业时, 不小心将方程 $\\frac{x-2}{2}-1=\\frac{4 x}{3}+$ 中的一个常数污染了看不清楚, 小芳告诉他该方程的解是负数, 并且这个常数是负整数, 该方程的解是", "options": [], "subject": "代数", "analysis": "个常数为 $m, \\frac{x-2}{2}-1=\\frac{4 x}{3}+m$,\n\n$$\n3(x-2)-6=8 x+6 m\n$$\n\n解得 $x=-\\frac{6 m+12}{5}$,\n\n解是负数, $m$ 是负整数,\n\n所以 $6 m+12>0, m>-2$,\n\n$m$ 的值只有 -1 ,\n\n解 $\\frac{x-2}{2}-1=\\frac{4 x}{3}-1$ 得 $x=-\\frac{6}{5}$,\n\n故答案为: $x=-\\frac{6}{5}$."} {"id": "1629", "image": [], "answer": "2024", "solution": "null", "level": "七年级", "question": "已知 $\\mathrm{n}$ 是关于 $\\mathrm{x}$ 的方程 $\\frac{1}{2}(1-4 x)=-m$ 的解, 则 $2022-4 m+8 n$ 的值为", "options": [], "subject": "代数", "analysis": "把 $\\mathrm{x}=\\mathrm{n}$ 代入方程得: $\\frac{1}{2}(1-4 n)=-m$,\n\n变形得: $m-2 n=-\\frac{1}{2}$,\n\n$\\therefore 2022-4 m+8 n$\n\n$=2022-4(m-2 n)$\n\n$=2022-4 \\times\\left(-\\frac{1}{2}\\right)$\n\n$=2024$\n\n故答案为: 2024 ."} {"id": "1630", "image": [], "answer": "$\\frac{6}{9}+\\frac{6-x}{12}=1$", "solution": "null", "level": "七年级", "question": "一项工程甲单独做 9 天完成,乙单独做 12 天完成.现甲、乙合作一段时间后乙休假,结果共用了 6 天完成这项工程. 设乙休假 x 天,可列方程为", "options": [], "subject": "代数", "analysis": "若乙休假 $\\mathrm{x}$ 天, 乙工作的天数为 $(6-x)$ 天, 根据题意得:\n\n$\\frac{6}{9}+\\frac{6-x}{12}=1$,\n\n故答案为: $\\frac{6}{9}+\\frac{6-x}{12}=1$."} {"id": "1320", "image": [], "answer": "1.1", "solution": "null", "level": "七年级", "question": "今年的“十・一”黄金周是 7 天的长假, 青城山风景区在 7 天假期中每天旅游人数变化如表(正号表示人数比前一天多, 负号表示比前一天少)\n\n| 日期 | 1 日 | 2 日 | 3 日 | 4 日 | 5 日 | 6 日 | 7 日 |\n| :--- | :--- | :--- | :--- | :--- | :--- | :--- | :--- |\n| 人数变化
单位 $:$ 万人 | +1.1 | -0.6 | +0.2 | -0.4 | -0.2 | +0.4 | -0.5 |", "options": [], "subject": "计数", "analysis": "10 月 1 日有游客: $0.1+1.1=1.2$ (万),\n\n10 月 2 日有游客: $1.2-0.6=0.6$ (万),\n\n10 月 3 日有游客: $0.6+0.2=0.8$ (万),\n\n10 月 4 日有游客: $0.8-0.4=0.4$ (万),\n\n10 月 5 日有游客: $0.4-0.2=0.2 \\quad($ 万),\n\n10 月 6 日有游客: $0.2+0.4=0.6$ (万),\n\n10 月 7 日有游客: $0.6-0.5=0.1 \\quad($ 万 $)$;\n\n7 天中旅客最多的是 1 日为 1.2 万人,最少的是 7 日为 0.1 万人,\n\n则七天中旅客人数最多的一天比最少的一天多 1.2-0.1=1.1 (万人);\n\n故答案为: 1.1 ."} {"id": "2404", "image": [], "answer": "600 个.", "solution": "null", "level": "七年级", "question": "(3 分) 有一箱规格相同的红、黄两种颜色的小塑料球共 1000 个. 为了估计这两种颜色的球各有多少个, 小明将箱子里面的球搅匀后从中随机摸出一个球记下颜色, 再把它放回箱子中, 多次重复上述过程后. 发现摸到红球的频率约为 0.6 , 据此可以估计红球的个数约为 600 个", "options": [], "subject": "计数", "analysis": "因为多次重复上述过程后, 发现摸到红球的频率约为 0.6 , 所以红球所占的百分比也就是 $60 \\\\%$, 根据总数可求出红球个数.\\n\\n解答: 解: $\\\\because$ 摸到红球的频率约为 0.6 ,\\n\\n$\\\\therefore$ 红球所占的百分比是 $60 \\\\%$.\\n\\n$\\\\therefore 1000 \\\\times 60 \\\\%=600($ 个) ."} {"id": "2406", "image": ["3508.jpg"], "answer": "$\\frac{1}{4}$.", "solution": "null", "level": "七年级", "question": "(3 分) 如图所示, 圆盘被分成 8 个全等的小扇形, 分别写上数字 $1,2 , 3 , 4 , 5 , 6,7$, 8 , 自由转动圆盘, 指针指向的数字 $<3$ 的概率是 $-\\frac{1}{4}$ -\n\n", "options": [], "subject": "计数", "analysis": "根据题意分析可得: 指针指向的数字共 8 种情况, 其中有 2 种情况可使指针指向的数字 $<3$, 故其概率是 $\\frac{2}{8}=\\frac{1}{4}$.\n\n解答: 解: $\\mathrm{P}($ 数字 $<3)=\\frac{2}{8}=\\frac{1}{4}$."} {"id": "2407", "image": [], "answer": "$\\frac{1}{2}$.", "solution": "null", "level": "七年级", "question": "(3 分) 已知三角形的两边长分别为 $5 \\mathrm{~cm}, 6 \\mathrm{~cm}$, 从下列四条长分别为 $1 \\mathrm{~cm}, 2 \\mathrm{~cm}, 6 \\mathrm{~cm}$, $11 \\mathrm{~cm}$ 线段中任意抽出一条能与已知线段围成三角形的概率是 $-\\frac{1}{2}$.", "options": [], "subject": "计数", "analysis": "此题首先根据三角形的三边关系, 找到能够组成三角形的数值, 再根据概率公式进行求解.\n\n解答: 解: 根据三角形的三边关系, 得\n\n第三边的取值范围是大于 1 而小于 11,显然四个数中, 有两个数符合是 2,6 ."} {"id": "1631", "image": [], "answer": "4000", "solution": "null", "level": "七年级", "question": "某型号彩电每台标价为 5250 元, 按标价的八折销售, 此时每台彩电的利润率是 $5 \\%$, 则该型号彩电的进价为每台元.", "options": [], "subject": "计数", "analysis": "设该型号彩电的进价为每台 $\\mathrm{x}$ 元, 由题意得 $5250 \\times 0.8=(1+5 \\%) x$,解得 $x=4000$,\n\n故答案为: 4000 ."} {"id": "2503", "image": [], "answer": "0.12", "solution": "null", "level": "七年级", "question": "(2022$\\cdot$北大附属嘉兴实验学校七年级月考) “Lost time is never found again”(岁月既往, 一去不回),在这句谚语的所有英文字母中, 字母“e”出现的频率是", "options": [], "subject": "计数", "analysis": "找出字母“e”出现的次数, 及总的字母数, 再由频率=频数总数, 即可得出答案.\n\n【详解】由题意得, 总共有 25 个, 字母“e”出现的次数为: 3 次,\n\n故字母“伍”出现的频率是 $\\frac{3}{25}=0.12$. 故答案为: 0.12 .\n\n【点睛】此题考查了频数和频率的知识, 掌握频率=频数二总数是解答本题的关键, 注意在数字母频数的时候要细心."} {"id": "23713", "image": [], "answer": "【答案】 $28 \\%$\n\n【解析】\n\n【分析】\n\n本题主要考查分式的化简求值; 设设荤菜的成本为 $m$ 元, 素菜的成本 $n$ 元, 荤菜的利润率为 $x$, 素菜的利润率为 $y, A$ 套餐和 $B$ 套餐的数量为 $a$ 份, $C$ 套餐的数量为 $b$ 份, 根据 $A$, $B$ 套餐的利润和是 $C$ 套餐利润的两倍, 得出 $b=\\frac{3}{4} a$, 根据当天的总利润率是 $60 \\%$, 得出 $m x+n y=0.6(m+n)$, 根据 $C$ 套餐的售价打五折, $A, B$ 套餐的售价均不变, $A 、 B 、 C$ 三种套餐的销量相同, 列出分式, 再化简即可得出答案.\n\n【解答】\n\n解: 设荤菜的成本为 $m$ 元, 素菜的成本 $n$ 元, 荤菜的利润率为 $x$, 素菜的利润率为 $y, A$ 套\n餐和 $B$ 套餐的数量为 $a$ 份, $C$ 套餐的数量为 $b$ 份.\n\n$\\because A, B$ 套餐的利润和是 $C$ 套餐利润的两倍,\n\n$\\therefore 3(m x+n y) \\times a=2 \\times 2(m x+n y) \\times b$,\n\n整理得 $b=\\frac{3}{4} a$,\n\n$\\because$ 当天的总利润率是 $60 \\%$,\n\n$\\therefore 3(m x+n y) \\times a+2(m x+n y) \\times b=60 \\%[3(m+n) \\times a+2(m+n) \\times b]$,\n\n整理得 $m x+n y=0.6(m+n)$,\n\n第二天店内搞活动, $C$ 套餐的售价打五折, $A, B$ 套餐的售价均不变, $A 、 B 、 C$ 三种套餐的销量相同, 设均为 $c$ 份,\n\n则总利润率为:\n\n$$\n\\begin{aligned}\n& \\frac{3 c \\times[m(1+x)+n(1+y)]+2 c \\times[m(1+x)+n(1+y)] \\times 50 \\%}{3 c(m+n)+2 c \\times(m+n)}-1 \\\\\n& =\\frac{(3 c+c) \\times[m(1+x)+n(1+y)]}{(3 c+2 c) \\times(m+n)}-1 \\\\\n& =\\frac{4 c \\times(m+n+m x+n y)}{5 c(m+n)}-1 \\\\\n& =\\frac{4 \\times[(m+n)+0.6(m+n)]}{5(m+n)}-1 \\\\\n& =\\frac{4}{5} \\times 1.6-1 \\\\\n& =0.28\n\\end{aligned}\n$$\n\n$=28 \\%$.\n\n故答案为: $28 \\%$.", "solution": "null", "level": "七年级", "question": "一家快餐店销售 $A 、 B 、 C$ 三种套餐,其中 $A$ 套餐包含一荤两素, $B$ 套餐包含两荤一素, $C$ 套餐包含两荤两素, 每份套餐中一荤的成本相同, 一素的成本也相同, 已知一份 $C$ 套餐的售价是一份 $A$ 套餐和一份 $B$ 套餐售价之和的 $\\frac{2}{3}$, 一天下来, 店长发现 $A$ 套餐和 $B$ 套餐的销量相同, 且 $A 、 B$ 套餐的利润和是 $C$ 套餐利润的两倍, 当天的总利润率是60\\%.第二天店内搞活动, $C$ 套餐的售价打五折, $A 、 B$ 套餐的售价均不变, 当 $A$ 、 $B 、 C$ 三种套餐的销量相同时, 总利润率为 $\\qquad$ .", "options": [], "subject": "计数", "analysis": "【答案】 $28 \\%$\n\n【解析】\n\n【分析】\n\n本题主要考查分式的化简求值; 设设荤菜的成本为 $m$ 元, 素菜的成本 $n$ 元, 荤菜的利润率为 $x$, 素菜的利润率为 $y, A$ 套餐和 $B$ 套餐的数量为 $a$ 份, $C$ 套餐的数量为 $b$ 份, 根据 $A$, $B$ 套餐的利润和是 $C$ 套餐利润的两倍, 得出 $b=\\frac{3}{4} a$, 根据当天的总利润率是 $60 \\%$, 得出 $m x+n y=0.6(m+n)$, 根据 $C$ 套餐的售价打五折, $A, B$ 套餐的售价均不变, $A 、 B 、 C$ 三种套餐的销量相同, 列出分式, 再化简即可得出答案.\n\n【解答】\n\n解: 设荤菜的成本为 $m$ 元, 素菜的成本 $n$ 元, 荤菜的利润率为 $x$, 素菜的利润率为 $y, A$ 套\n餐和 $B$ 套餐的数量为 $a$ 份, $C$ 套餐的数量为 $b$ 份.\n\n$\\because A, B$ 套餐的利润和是 $C$ 套餐利润的两倍,\n\n$\\therefore 3(m x+n y) \\times a=2 \\times 2(m x+n y) \\times b$,\n\n整理得 $b=\\frac{3}{4} a$,\n\n$\\because$ 当天的总利润率是 $60 \\%$,\n\n$\\therefore 3(m x+n y) \\times a+2(m x+n y) \\times b=60 \\%[3(m+n) \\times a+2(m+n) \\times b]$,\n\n整理得 $m x+n y=0.6(m+n)$,\n\n第二天店内搞活动, $C$ 套餐的售价打五折, $A, B$ 套餐的售价均不变, $A 、 B 、 C$ 三种套餐的销量相同, 设均为 $c$ 份,\n\n则总利润率为:\n\n$$\n\\begin{aligned}\n& \\frac{3 c \\times[m(1+x)+n(1+y)]+2 c \\times[m(1+x)+n(1+y)] \\times 50 \\%}{3 c(m+n)+2 c \\times(m+n)}-1 \\\\\n& =\\frac{(3 c+c) \\times[m(1+x)+n(1+y)]}{(3 c+2 c) \\times(m+n)}-1 \\\\\n& =\\frac{4 c \\times(m+n+m x+n y)}{5 c(m+n)}-1 \\\\\n& =\\frac{4 \\times[(m+n)+0.6(m+n)]}{5(m+n)}-1 \\\\\n& =\\frac{4}{5} \\times 1.6-1 \\\\\n& =0.28\n\\end{aligned}\n$$\n\n$=28 \\%$.\n\n故答案为: $28 \\%$."} {"id": "23761", "image": ["11681.jpg"], "answer": "【答案】 20\n\n【解析】\n\n【分析】\n\n本题考查了条形统计图, 掌握求所占的百分比的正确的计算方法是解题的关键.\n根据统计图数据, 用 $A$ 等级的人数除以总人数, 计算即可得解.\n\n【解答】\n\n解: 达到 $A$ 等级的人数占总人数的百分比为:\n\n$\\frac{10}{10+15+12+10+3} \\times 100 \\%=\\frac{10}{50} \\times 100 \\%=20 \\%$.\n\n故答案为 20 .", "solution": "null", "level": "七年级", "question": "李老师对班上某次数学模拟考试成绩进行统计, 绘制了如图所示的统计图, 根据图中给出的信息,这次考试成绩达到 $A$ 等级的人数占总人数的 $\\qquad$ $\\%$.\n\n", "options": [], "subject": "计数", "analysis": "【答案】 20\n\n【解析】\n\n【分析】\n\n本题考查了条形统计图, 掌握求所占的百分比的正确的计算方法是解题的关键.\n根据统计图数据, 用 $A$ 等级的人数除以总人数, 计算即可得解.\n\n【解答】\n\n解: 达到 $A$ 等级的人数占总人数的百分比为:\n\n$\\frac{10}{10+15+12+10+3} \\times 100 \\%=\\frac{10}{50} \\times 100 \\%=20 \\%$.\n\n故答案为 20 ."} {"id": "1323", "image": ["3107.jpg", "3108.jpg", "3109.jpg", "3110.jpg", "3111.jpg"], "answer": "$321 \\times 123=39483$", "solution": "null", "level": "七年级", "question": "以下是利用图形计算正整数乘法的方法, 请根据图 1 图 4 所示的规律, 预测出图 5 所表达的算式为\n\n\n\n图 1\n\n\n\n图 2\n\n\n\n$21 \\times 12=252$\n\n图 3\n\n\n\n$31 \\times 12=372$\n\n图 4\n\n\n\n图 5", "options": [], "subject": "组合几何学", "analysis": "形知: 图 1 中标的数字从个位开始, 从右向左排列正是结果, 为 121 , 左下方的两组交点个数逆时针排列为 11 , 右下方的两组交点个数逆时针排列为 11 , 它们为两个因数, 即 $11 \\times 11=121$;\n\n图 2 中标的数字从个位开始, 从右向左排列正是结果, 为 231 , 左下方的两组交点个数逆时针排列为 21 , 右下方的两组交点个数逆时针排列为 11 , 它们为两个因数, 即 $21 \\times 11=231$;\n\n图 3 中标的数字从个位开始, 从右向左排列正是结果, 为 252 , 左下方的两组交点个数逆时针排列为 21 , 右下方的两组交点个数逆时针排列为 12 , 它们为两个因数, 即 $21 \\times 12=252$;\n\n图 4 中标的数字从个位开始, 从右向左排列正是结果, 为 372 , 左下方的两组交点个数逆时针排列为 31 , 右下方的两组交点个数逆时针排列为 12 , 它们为两个因数, 即 $31 \\times 12=372$;\n\n由此得出图 5 中标的数字从个位开始, 从右向左排列正是结果(超过 10 个点向上一位进 1 ), 为 39483 , 左下方的三组交点个数逆时针排列为 321 , 右下方的三组交点个数逆时针排列为 123 , 它们为两个因数,即 $321 \\times 123=39483$;\n\n故答案为: $321 \\times 123=39483$."} {"id": "2427", "image": ["3520.jpg"], "answer": "$\\sqrt{2}$", "solution": "null", "level": "七年级", "question": "如图, 在 $\\triangle A B C$ 中 $\\angle A=90^{\\circ}, A B=A C=2 \\mathrm{~cm}, \\odot A$ 与 $B C$ 相切于点 $D$, 则 $\\odot A$ 的半径为 $\\mathrm{cm}$.\n\n第 11 题", "options": [], "subject": "组合几何学", "analysis": "$\\sqrt{2}$"} {"id": "2405", "image": [], "answer": "$\\frac{1}{6}$.", "solution": "null", "level": "七年级", "question": "(3 分) 利用计算器产生 $1 \\sim 6$ 个随机数, 连续两次随机相同的概率是 $-\\frac{1}{6}$ -", "options": [], "subject": "组合数学", "analysis": "第一个数随机产生, 第二个数与第一个数相同的情况有一种, 而第二个数可能出现的情况有 6 种, 利用概率公式进行求解.\n\n解答: 解: $\\because$ 第一个数随机产生, 第二个数与第一个数相同的情况有一种, 而第二个数可能出现的情况有 6 种\n\n$\\therefore$ 连续两次随机相同的概率是 $\\frac{1}{6}$."} {"id": "2408", "image": [], "answer": "$\\frac{1}{3}$.", "solution": "null", "level": "七年级", "question": "(3 分) 在 $-1,1,2$ 这三个数中任选 2 个数分别作为 $P$ 点的横坐标和纵坐标, 过 $P$ 点画双曲线 $y=\\frac{\\mathrm{k}}{\\mathrm{x}}$, 该双曲线位于第一、三象限的概率是 $-\\frac{1}{3}$.", "options": [], "subject": "组合数学", "analysis": "根据概率求法直接列举出所有符合要求点的坐标, 再根据只有 $(1,2),(2,1)$ 符合 $x y=k>0$, 得出答案即可.\n\n解答: 解: $\\because$ 在 $-1,1,2$ 这三个数中任选 2 个数分别作为 $\\mathrm{P}$ 点的横坐标和纵坐标,\n\n$\\therefore$ 符合要求的点有 $(-1,1),(-1,2),(1,2),(1,-1),(2,1),(2,-1)$,\n\n$\\therefore$ 该双曲线位于第一、三象限时, $x y=k>0$,\n\n只有 $(1,2),(2,1)$ 符合 $x y=k>0$,\n\n$\\therefore$ 该双曲线位于第一、三象限的概率是: $2 \\div 6=\\frac{1}{3}$,"} {"id": "2409", "image": ["3509.jpg"], "answer": "$\\frac{2}{3}$.", "solution": "null", "level": "七年级", "question": "(3 分) 如图, 随机闭合开关 $\\mathrm{S}_{1}, \\mathrm{~S}_{2}, \\mathrm{~S}_{3}$ 中的两个, 能够让灯泡发光的概率为 $-\\frac{2}{3}$.\n\n", "options": [], "subject": "组合数学", "analysis": "根据题意可得:随机闭合开关 $S_{1}, S_{2}, S_{3}$ 中的两个,有 3 种方法,其中有两种能够让\n灯泡发光, 故其概率为 $\\frac{2}{3}$.\n\n解答: 解: $\\mathrm{P}$ (灯泡发光) $=\\frac{2}{3}$."} {"id": "2410", "image": [], "answer": "$\\frac{1}{9}$.", "solution": "null", "level": "七年级", "question": "(3 分) 一枚均匀的正方体骰子, 六个面分别标有数字 $1,2 , 3 , 4 , 5,6$, 连续抛掷两次, 朝上的数字分别是 $m$, $n$. 若把 $m, n$ 作为点 $\\mathrm{A}$ 的横、纵坐标, 那么点 $\\mathrm{A}(\\mathrm{m}, \\mathrm{n})$ 在函数 $\\mathrm{y}=\\frac{6}{\\mathrm{x}}$ 的图象上的概率是 $-\\frac{1}{9}$.", "options": [], "subject": "统计数学", "analysis": "列举出所有情况, 看所求的情况占总情况的多少即可.\n\n解答: 解:每次都有 6 种可能, 那么共有 $6 \\times 6=36$ 种可能, 其中 $(1,6)(2,3),(3,2),(6,1)在函数图象上, $\\therefore$ 点 $\\mathrm{A}(\\mathrm{m}, \\mathrm{n})$ 在函数 $\\mathrm{y}=\\frac{6}{\\mathrm{x}}$ 的图象上的概率是 $\\frac{1}{9}$."} {"id": "2413", "image": [], "answer": "$\\frac{3}{6}-\\frac{1}{2}$.", "solution": "null", "level": "七年级", "question": "(3 分) 对于平面内任意一个凸四边形 $\\mathrm{ABCD}$, 现从以下四个关系式(1) $\\mathrm{AB}=\\mathrm{CD}$; (2) $\\mathrm{AD}=\\mathrm{BC}$; (3) $\\mathrm{AB} / / \\mathrm{CD}$; (4) $\\angle \\mathrm{A}=\\angle \\mathrm{C}$ 中任取两个作为条件, 能够得出这个四边形 $\\mathrm{ABCD}$是平行四边形的概率是 $-\\frac{1}{2}$.", "options": [], "subject": "解析几何", "analysis": "本题是一道列举法求概率的问题, 属于基础题, 可以直接应用求概率的公式.\n\n解答: 解:从四个条件中选两个共有六种可能: (1)(2)、(1)(3)、(1)(4)、(2)(3)、(2)(4)、\n\n(3) (4),\n\n其中只有(1)(2)、 (1)(3)和(3) (4)可以判断 $\\mathrm{ABCD}$ 是平行四边形, 所以其概率为 $\\frac{3}{6}-\\frac{1}{2}$."} {"id": "2429", "image": [], "answer": "$ 或 3", "solution": "null", "level": "七年级", "question": "两圆内切, 其中一个圆的半径为 5 , 两圆的圆心距为 2 , 则另一个圆的半径为 \\$ \\qquad \\$", "options": [], "subject": "解析几何", "analysis": "$ 或 3"} {"id": "2430", "image": [], "answer": "(1)(3) $\\Rightarrow$ (2)或(2)(3) $\\Rightarrow$ (1) $", "solution": "null", "level": "七年级", "question": "(1) $O C$ 是 $\\odot O$ 的半径; (2) $A B \\perp O C$; (3)直线 $A B$ 切 $\\odot O$ 于点 $C$. 请以其中两个语句为条件,一个语句为结论, 写出一个真命题 \\$ \\qquad \\$ . (写标号,用推出符号表示)", "options": [], "subject": "解析几何", "analysis": "(1)(3) $\\Rightarrow$ (2)或(2)(3) $\\Rightarrow$ (1) $"} {"id": "2432", "image": ["3522.jpg"], "answer": "\\frac{1}{3}", "solution": "null", "level": "七年级", "question": "如图, $\\odot 0_{1}, \\odot 0_{2}$ 相互外切且都内切于半圆 0 , 且 $\\odot 0_{1}, \\odot 0_{2}$ 切半圆 0 的直径 $A B$ 于点 0 和 $C$, 则 $\\cos \\angle o O_{1} O_{2}$ 的值为 \\$ \\qquad \\$ .\n\n\n(第 $\\mathrm{O} 16$ 题)", "options": [], "subject": "解析几何", "analysis": "\\frac{1}{3}"} {"id": "2448", "image": [], "answer": "$\\frac{2 \\sqrt{5}}{5}$.", "solution": "null", "level": "七年级", "question": "(3 分) 在 $\\mathrm{Rt} \\triangle \\mathrm{ABC}$ 中, $\\angle C=90^{\\circ}$, 直角边 $\\mathrm{AC}$ 是 $\\mathrm{BC}$ 的 2 倍, 则 $\\cos \\mathrm{A}$ 的值是 $\\_\\frac{2 \\sqrt{5}}{5}$.", "options": [], "subject": "解析几何", "analysis": "设 $\\mathrm{BC}=\\mathrm{x}$, 则 $\\mathrm{AC}=2 \\mathrm{x}$, 利用勾股定理即可求得 $\\mathrm{AB}$ 的长, 然后利用余弦函数的定义即可求解.\n\n解答: 解: 设 $\\mathrm{BC}=\\mathrm{x}$, 则 $\\mathrm{AC}=2 \\mathrm{x}$,\n\n则 $A B=\\sqrt{A C^{2}+B C^{2}}=\\sqrt{5} x$,\n\n则 $\\cos A=\\frac{A C}{A B}=\\frac{2 x}{\\sqrt{5} x}=\\frac{2 \\sqrt{5}}{5}$."} {"id": "2450", "image": [], "answer": "$1<\\mathrm{m}<3$.", "solution": "null", "level": "七年级", "question": "(3 分) 若 $\\alpha$ 为锐角, 且 $\\sin \\alpha=\\frac{m-1}{2}$, 则 $m$ 的取值范围是 $1", "options": [], "subject": "画法几何学", "analysis": "(1) 过 $\\mathrm{E}$ 点作 $\\mathrm{EG} \\perp \\mathrm{DF}$, 由 $\\mathrm{E}$ 是 $\\mathrm{AB}$ 的中点, 得出 $\\mathrm{DG}=3$, 再根据 $\\angle \\mathrm{DEG}=60^{\\circ}$ 得出 $\\angle \\mathrm{DEF}=120^{\\circ}$, 由 $\\tan 60^{\\circ}=\\frac{\\mathrm{GF}}{\\sqrt{3}}$ 即可求出 $\\mathrm{GF}$ 的长, 进而得出结论;\n\n(2) 过点 $B$ 作 $B H \\perp D C$, 延长 $A B$ 至点 $M$, 过点 C 作 $C M \\perp A B$ 于 $F$, 则 $B H=A D=\\sqrt{3}$,再由锐角三角函数的定义求出 $\\mathrm{CH}$ 及 $\\mathrm{BC}$ 的长, 设 $\\mathrm{AE}=\\mathrm{x}$, 则 $\\mathrm{BE}=6-\\mathrm{x}$, 利用勾股定理用 $\\mathrm{x}$ 表示出 $\\mathrm{DE}$ 及 $\\mathrm{EF}$ 的长, 再判断出 $\\triangle \\mathrm{EDF} \\sim \\triangle \\mathrm{BCE}$, 由相似三角形的对应边成比例即可得出关于 $\\mathrm{x}$ 的方程, 求出 $\\mathrm{x}$ 的值即可.\n\n解答: 解: (1) 如图 1, 过 $\\mathrm{E}$ 点作 EG $\\perp \\mathrm{DF}$,\n\n$\\because \\mathrm{E}$ 是 $\\mathrm{AB}$ 的中点,\n\n$\\therefore$ DG $=3$,\n\n$\\therefore \\mathrm{EG}=\\mathrm{AD}=\\sqrt{3}$,\n\n$\\therefore \\angle \\mathrm{DEG}=60^{\\circ}$,\n\n$\\because \\angle \\mathrm{DEF}=120^{\\circ}$,\n\n$\\therefore \\tan 60^{\\circ}=\\frac{\\mathrm{GF}}{\\sqrt{3}}$,\n\n解得 $\\mathrm{GF}=3$,\n\n$\\therefore \\mathrm{DF}=6$;\n\n\n\n图1\n\n\n\n图2\n\n(2) 如图 2 所示:\n\n过点 $B$ 作 $B H \\perp D C$, 延长 $A B$ 至点 $M$, 过点 $C$ 作 $C M \\perp A B$ 于 $M$, 则 $B H=A D=M F=\\sqrt{3}$,\n\n$\\because \\angle \\mathrm{ABC}=120^{\\circ}, \\mathrm{AB} / / \\mathrm{CD}$,\n\n$\\therefore \\angle \\mathrm{BCH}=60^{\\circ}$,\n\n$\\therefore \\mathrm{CH}=\\mathrm{BM}=\\frac{\\mathrm{BH}}{\\tan 60^{\\circ}}=\\frac{\\sqrt{3}}{\\sqrt{3}}=1$,\n\n设 $\\mathrm{AE}=\\mathrm{x}$, 则 $\\mathrm{BE}=6-\\mathrm{x}$,\n\n在 Rt $\\triangle \\mathrm{EFM}$ 中,\n\n$\\mathrm{EF}=\\sqrt{(\\mathrm{EB}+\\mathrm{BM})^{2}+\\mathrm{MF}^{2}}=\\sqrt{(6-\\mathrm{x}+1)^{2}+(\\sqrt{3})^{2}}=\\sqrt{(7-\\mathrm{x})^{2}+3}$,\n\n$\\because \\mathrm{AB} / / \\mathrm{CD}$\n\n$\\therefore \\angle \\mathrm{EFD}=\\angle \\mathrm{BEC}$,\n\n$\\because \\angle \\mathrm{DEF}=\\angle \\mathrm{B}=120^{\\circ}$,\n\n$\\therefore \\triangle \\mathrm{EDF} \\sim \\triangle \\mathrm{BCE}$, 即 $\\triangle \\mathrm{EDF} \\sim \\triangle \\mathrm{BFE}$\n\n$\\therefore \\frac{\\mathrm{DF}}{\\mathrm{EF}}=\\frac{\\mathrm{EF}}{\\mathrm{BE}}$\n\n$\\therefore \\mathrm{EF}^{2}=\\mathrm{DF} \\cdot \\mathrm{BE}$, 即 $(7-\\mathrm{x})^{2}+3=7(6-\\mathrm{x})$\n\n解得 $x=2$ 或 5"} {"id": "1513", "image": ["3158.jpg", "3159.jpg"], "answer": "(1) $a+3 b$\n\n(2) $a+b+c=2 b+2$", "solution": "null", "level": "七年级", "question": "如图 1, 在一块长方形区域中布置了图中阴影部分所示的展区, 其中的展台有三种不同的形状,其规格如图 2 所示.\n\n\n\n图 1\n\n\n\n图 2\n\n(1) 该长方形区域的长可以用式子表示为\n\n(2)根据图中信息,用等式表示 $\\mathrm{a}, \\mathrm{b}, \\mathrm{c}$ 满足的关系为", "options": [], "subject": "画法几何学", "analysis": "解:(1)由图可知:\n\n长方形区域的长 $=$ 小长方形的长+小长方形的宽+小长方形的长+小长方形的长一小长方形的宽 +正方形的边长,\n\n即: $b+c+b+b-c+a=a+3 b$;\n\n故答案为: $a+3 b$;\n\n(2) 长方形区域左边宽度 $=b+2+b=2 b+2$, 右边宽度 $=a+b+c$,\n$\\therefore a+b+c=2 b+2$\n\n故答案为: $a+b+c=2 b+2$."} {"id": "23437", "image": ["11564.jpg", "11565.jpg", "11565.jpg"], "answer": "解【答案】 220\n\n【解析】解: 如图,\n\n$\\because$ 直线 $b$ 平移后得到直线 $a$,\n\n$\\therefore a / / b$,\n\n$\\therefore \\angle 1+\\angle 4=180^{\\circ}$, 即 $\\angle 4=180^{\\circ}-\\angle 1$,\n\n$\\because \\angle 5=\\angle 3=40^{\\circ}$,\n\n\n\n$\\therefore \\angle 2=\\angle 4+\\angle 5=180^{\\circ}-\\angle 1+40^{\\circ}$,\n\n$\\therefore \\angle 1+\\angle 2=220^{\\circ}$.\n\n故答案为 220 .\n\n如图, 利用平移的性质得 $a / / b$, 再根据平行线的性质得 $\\angle 4=180^{\\circ}-\\angle 1$, 加上对顶角相等得 $\\angle 5=\\angle 3=40^{\\circ}$, 则根据三角形外角性质得 $\\angle 2=\\angle 4+\\angle 5=180^{\\circ}-\\angle 1+40^{\\circ}$, 从而可计算出 $\\angle 1+\\angle 2$ 的度数.\n\n本题考查了平移的性质:把一个图形整体沿某一直线方向移动, 会得到一个新的图形,新图形与原图形的形状和大小完全相同.", "solution": "null", "level": "七年级", "question": "如图, $\\angle 3=40^{\\circ}$, 直线 $b$ 平移后得到直线 $a$, 则 $\\angle 1+$ $\\angle 2=$ $\\qquad$ $\\circ$.\n\n", "options": [], "subject": "变换几何", "analysis": "解【答案】 220\n\n【解析】解: 如图,\n\n$\\because$ 直线 $b$ 平移后得到直线 $a$,\n\n$\\therefore a / / b$,\n\n$\\therefore \\angle 1+\\angle 4=180^{\\circ}$, 即 $\\angle 4=180^{\\circ}-\\angle 1$,\n\n$\\because \\angle 5=\\angle 3=40^{\\circ}$,\n\n\n\n$\\therefore \\angle 2=\\angle 4+\\angle 5=180^{\\circ}-\\angle 1+40^{\\circ}$,\n\n$\\therefore \\angle 1+\\angle 2=220^{\\circ}$.\n\n故答案为 220 .\n\n如图, 利用平移的性质得 $a / / b$, 再根据平行线的性质得 $\\angle 4=180^{\\circ}-\\angle 1$, 加上对顶角相等得 $\\angle 5=\\angle 3=40^{\\circ}$, 则根据三角形外角性质得 $\\angle 2=\\angle 4+\\angle 5=180^{\\circ}-\\angle 1+40^{\\circ}$, 从而可计算出 $\\angle 1+\\angle 2$ 的度数.\n\n本题考查了平移的性质:把一个图形整体沿某一直线方向移动, 会得到一个新的图形,新图形与原图形的形状和大小完全相同."} {"id": "2480", "image": [], "answer": "c.", "solution": "null", "level": "七年级", "question": "(2021- 浙江七年级期末)三位先生 A、B、C 带着他们的妻子 a、b、c 到超市购物, 至于谁是谁的妻子现在只能从下列条件来推测: 他们 6 人, 每人花在买商品的钱数 (单位: 元) 正好等于商品数量的平方, 而且每位先生都比自己的妻子多花 48 元钱, 又知先生 A 比 b 多买 9 件商品, 先生 B 比 a 多买 7 件商品. 则先生 A 的妻子是 \\$ \\qquad \\$", "options": [], "subject": "逻辑题", "analysis": " $c$\n\n【分析】设一对夫妻, 丈夫买了 $\\mathrm{x}$ 件商品, 妻子买了 $\\mathrm{y}$ 件商品, 列出关于 $\\mathrm{x} 、 \\mathrm{y}$ 的二元二次方程, 再根据 $\\mathrm{x} 、 \\mathrm{y}$ 都是正整数, 且 $x+y$ 与 $x-y$ 有相同的奇偶性, 即可得出关于 $\\mathrm{x} 、 \\mathrm{y}$ 的二元一次方程组,求出 $\\mathrm{x} 、 \\mathrm{y}$ 的值, 再找出符合 $x-y=9$ 和 $x-y=7$ 的情况即可进行解答.\n\n【详解】设一对夫妻, 丈夫买了 $x$ 件商品, 则钱数为 $x^{2}$, 妻子买了 $y$ 件商品, 则钱数为 $y^{2}$,依题意有 $x^{2}-y^{2}=48$, 即 $(x+y)(x-y)=48$,\n\n$\\because \\mathrm{x} 、 \\mathrm{y}$ 都是正整数, 且 $x+y$ 与 $x-y$ 有相同的奇偶性,\n\n又 $\\because x+y>x-y, 48=24 \\times 2=12 \\times 4=8 \\times 6$, $\\therefore\\left\\{\\begin{array}{c}x+y=24 \\\\ x-y=2\\end{array}\\right.$ 或 $\\left\\{\\begin{array}{c}x+y=12 \\\\ x-y=4\\end{array}\\right.$ 或 $\\left\\{\\begin{array}{c}x+y=8 \\\\ x-y=6\\end{array}\\right.$, 解得 $x=13, y=11$ 或 $x=8, y=4$ 或 $x=7, y=1$,符合 $x-y=9$ 的只有一种, 可见 $\\mathrm{A}$ 买了 13 件商品, $\\mathrm{b}$ 买了 4 件,同时符合 $x-y=7$ 的也只有一种, 可知 $\\mathrm{B}$ 买了 8 件, $\\mathrm{a}$ 买了 1 件, $\\therefore \\mathrm{C}$ 买了 7 件, $\\mathrm{c}$ 买了 11 件.由此可知三对夫妻的组合是: A、c; B、b; C、a. 故答案为: c.\n\n【点睛】本题考查了不定方程组的解及数的奇偶性, 根据题意列出关于 $x 、 y$ 的不定方程是解答此题的关键."} {"id": "3574", "image": ["4869.jpg", "4870.jpg"], "answer": "30 或 60", "solution": "null", "level": "九年级", "question": "如图, 已知射线 $B P \\perp B A$, 点 $O$ 从 $B$ 点出发, 以每秒 1 个单位长度沿射线 $B A$ 向右运动; 同时射线 $B P$ 绕点 $B$ 顺时针旋转一周, 当射线 $B P$停止运动时, 点 $O$ 随之停止运动. 以 $O$ 为圆心, 1 个单位长度为半径画圆, 若运动两秒后, 射线 $B P$ 与 $\\odot O$ 恰好有且只有一个公共点, 则射线\n\n\n$B P$ 旋转的速度为每秒度.", "options": [], "subject": "度量几何学", "analysis": "设射线 $B P$ 旋转的速度为每秒 $a$ 度, 则两秒后, 射线 $B P$ 旋转 $2 a$ 度, 点 $O$ 运动到了距离 $B 2$ 个单位长度处,\n\n根据题意可得: 有两种情况使得射线 $B P$ 与 $\\odot O$ 恰好有且只有一个公共点, 即 $B P_{1} 、 B P_{2}$ 与圆相切, 如图,\n\n\n\n当 $B P$ 旋转到 $B P_{1}$ 时, $O^{\\prime} C=1, B O^{\\prime}=2, O^{\\prime} C \\perp B C$, 即 $\\angle C B O^{\\prime}=30^{\\circ}$,\n\n$\\therefore \\angle P B C=60^{\\circ}$,\n\n即 $2 a=60$,\n\n$a=30$,\n\n当 $B P$ 旋转到 $B P_{2}$ 时, $O^{\\prime} D=1, B O^{\\prime}=2, O^{\\prime} D \\perp B D$, 即 $\\angle D B O^{\\prime}=30^{\\circ}$,\n\n$\\therefore \\angle P B D=120^{\\circ}$,\n\n即 $2 a=120 ,$\n\n$a=60$,\n\n$\\therefore$ 射线 $B P$ 旋转的速度为每秒 30 或 60 度,\n\n故答案为 30 或 60 ."} {"id": "3576", "image": [], "answer": "9.4", "solution": "null", "level": "九年级", "question": "张明同学想利用树影测量校园内树的高度, 他在某一时刻测得某小树的高为 $1.5 \\mathrm{~m}$, 其影长为 $1.2 \\mathrm{~m}$. 同一时刻, 当他测量教学楼旁的一棵大树的影长时, 因大树靠近教学楼, 有一部分影子落\n在墙上. 经测量, 大树地面部分的影长为 $6.4 \\mathrm{~m}$, 墙上的影长为 $1.4 \\mathrm{~m}$, 则这棵大树高 $m$.", "options": [], "subject": "度量几何学", "analysis": "设这棵大树高为 $x m$,\n\n根据平行投影特点: 在同一时刻, 不同物体的物高和影长成比例可得:\n\n树高比影长为 1.5 . $=1.25$,\n\n则有 $\\frac{6.4}{x-1.4}=\\frac{1}{1.25}=0.8$,\n\n解得: $x=9.4$ 米.\n\n故答案为 9.4 ."} {"id": "3580", "image": ["4877.jpg"], "answer": "$(12,8 \\sqrt{3})$", "solution": "null", "level": "九年级", "question": "在 $\\triangle A B C$ 中, $a=12, A=60^{\\circ}$, 三角形有两解, 则边 $b$ 的取值范围为 \\$ \\qquad \\$\n\n", "options": [], "subject": "度量几何学", "analysis": "由题意得, $\\triangle A B C$ 有两解时需要: $b \\sin A", "options": [], "subject": "度量几何学", "analysis": "连接 $A M 、 A N$,\n\n$\\because M N$ 为 $\\odot A$ 的切线, 切点为点 $N$\n\n$$\n\\therefore A N \\perp M N \\therefore M N=\\sqrt{A M^{2}-A N^{2}} \\because A N=1\n$$\n\n所以当 $A M$ 最小时, $M N$ 取最小值,\n\n$\\because$ 点 $M$ 为 $B D$ 上一动点,\n$\\therefore$ 当 $A M \\perp B D$ 时, $A M$ 最小, $B D=\\sqrt{2} A B=3 \\sqrt{2}$\n\n所以此时 $A M=\\frac{1}{2} B D=\\frac{3 \\sqrt{2}}{2}$\n\n$\\therefore M N$ 的最小值为 $M N=\\sqrt{A M^{2}-A N^{2}}=\\sqrt{\\left(\\frac{3 \\sqrt{2}}{2}\\right)^{2}-1}=\\frac{\\sqrt{14}}{2}$.\n\n故答案为 $\\frac{\\sqrt{14}}{2}$."} {"id": "3597", "image": [], "answer": "$\\frac{3}{2}$", "solution": "null", "level": "九年级", "question": "$\\cos 30^{\\circ} \\cdot \\tan 60^{\\circ}$ 的值等于 \\$ \\qquad \\$", "options": [], "subject": "度量几何学", "analysis": "原式 $=\\frac{\\sqrt{3}}{2} \\times \\sqrt{3}$\n\n$=\\frac{3}{2}$.\n\n故答案为 $\\frac{3}{2}$."} {"id": "3599", "image": [], "answer": "2 或 $\\frac{8}{5}$", "solution": "null", "level": "九年级", "question": "等腰三角形的一边长为 10 , 面积为 40 , 则底角的正切值为 \\$ \\qquad \\$ .", "options": [], "subject": "度量几何学", "analysis": "在三角形 $A B C$ 中, $A B=A C, \\angle B=\\angle C=\\alpha$\n\n(1)当 $A B=10$ 时, 过 $A$ 作 $C D \\perp A B$ 于 $D$,\n\n$\\because S \\triangle A B C=\\frac{1}{2} A B \\cdot C D=40$,\n\n$\\therefore \\frac{1}{2} \\times 10 \\cdot C D=40$,\n\n$\\therefore C D=8$.\n\n在直角三角形 $A C D$ 中, 有勾股定理得\n\n$A D=\\sqrt{10^{2}-8^{2}}=6$,\n$B D=10-6=4$,\n\n$\\therefore$ 底角 $\\tan \\alpha=\\frac{C D}{B D}=2$;\n\n(2)当 $B C=10$ 时, 过 $A$ 作 $A D \\perp B C$ 于 $D$, 则 $B D=5$,\n\n$\\because S \\triangle A B C=\\frac{1}{2} B C \\cdot A D=40$,\n\n$\\therefore \\frac{1}{2} \\times 10 \\cdot A D=40$,\n\n$\\therefore A D=8$,\n\n$\\therefore$ 底角 $\\tan \\alpha=\\frac{A D}{B D}=\\frac{8}{5}$,\n\n綜上可知底角的正切值为 2 或 $\\frac{8}{5}$.\n\n故答案为 2 或 $\\frac{8}{5}$."} {"id": "3600", "image": ["4913.jpg"], "answer": "$\\sqrt{3}$", "solution": "null", "level": "九年级", "question": "在 $R t \\triangle A B C$ 中, $\\angle C=90^{\\circ}, \\sin A=\\frac{\\sqrt{3}}{2}, B C=3$, 则斜边上的中线长为 \\$ \\qquad \\$ .", "options": [], "subject": "度量几何学", "analysis": "如图, $C D$ 是 $A B$ 边上的中线,\n\n\n\n$\\because, \\angle C=90^{\\circ}, \\sin A=\\frac{\\sqrt{3}}{2}, B C=3$,\n\n又 $\\because \\sin A=\\frac{B C}{A B}$,\n\n$\\therefore \\frac{3}{A B}=\\frac{\\sqrt{3}}{2}$,\n\n$\\therefore A B=2 \\sqrt{3}$,\n\n$\\because C D$ 是 $A B$ 边上的中线,\n\n$\\therefore C D=\\frac{1}{2} A B=\\frac{1}{2} \\times 2 \\sqrt{3}=\\sqrt{3}$.\n\n故答案为 $\\sqrt{3}$."} {"id": "3601", "image": ["4914.jpg"], "answer": "", "solution": "null", "level": "九年级", "question": "平面直角坐标系 $x O y$ 中, 若 $P\\left(m, \\frac{4}{m}\\right), Q(3 m,-6 n+2)$ 是两个动点 $(m, n$ 是实数且 $m \\neq 0)$, 则 $P Q$长度的最小值为 \\$ \\qquad \\$ .", "options": [], "subject": "度量几何学", "analysis": "$\\because P\\left(m, \\frac{4}{m}\\right), Q(3 n,-6 n+2)$,\n\n$\\therefore$ 点 $P$ 在双曲线 $y=\\frac{4}{x}$ 上, 点 $Q$ 在直线 $y=-2 x+2$ 上,\n平移直线 $y=-2 x+2$ 使它与双曲线 $y=\\frac{4}{x}$ 只有一个交点,\n\n设平移后的直线为 $y=-2 x+2+b$,\n\n联立得方程 $-2 x+2+b=\\frac{4}{x}$,\n\n整理得 $2 x^{2}-(2+b) x+4=0$ ,\n\n$\\Delta=(2+b)^{2}-4 \\times 2 \\times 4=0$,\n\n解得 $b=-2+4 \\sqrt{2}$ 或 $-2-4 \\sqrt{2}$,\n\n即向上平移 $-2+4 \\sqrt{2}$ 或向下平移 $2+4 \\sqrt{2}$ 个单位,\n\n当向上平移 $-2+4 \\sqrt{2}$ 个单位即 $A B=-2+4 \\sqrt{2}$ 时, 平移前后两条直线之间的距离最小,\n\n$\\because \\tan \\angle B A C=\\tan \\angle O A D=\\frac{1}{2}$,\n\n$\\therefore \\frac{B C}{A C}=\\frac{1}{2}, \\frac{B C}{A B}=\\frac{1}{\\sqrt{5}}=\\frac{\\sqrt{5}}{5}$,\n\n解得 $B C=\\frac{\\sqrt{5}}{5} A B=\\frac{4 \\sqrt{10}-2 \\sqrt{5}}{5}$,\n\n即 $P Q$ 长度的最小值为 $\\frac{4 \\sqrt{10}-2 \\sqrt{5}}{5}$.\n\n"} {"id": "3603", "image": ["4916.jpg"], "answer": "$\\frac{3 \\sqrt{3}}{2}$", "solution": "null", "level": "九年级", "question": "将直角边长为 3 的等腰直角 $\\triangle A B C$ 绕点 $A$ 逆时针旋转 $15^{\\circ}$ 后得到 $\\triangle A B^{\\prime} C^{\\prime}$, 则图中阴影部分的面积是 \\$ \\qquad \\$\n\n", "options": [], "subject": "度量几何学", "analysis": ": 等腰直角 $\\triangle A B C$ 绕点 $A$ 逆时针旋转 $15^{\\circ}$ 后得到 $\\triangle A B^{\\prime} C^{\\prime}$,\n\n$\\because \\angle C A C^{\\prime}=15^{\\circ}$,\n\n$\\therefore \\angle C^{\\prime} A B=\\angle C A B-\\angle C A C^{\\prime}=45^{\\circ}-15^{\\circ}=30^{\\circ}, A C^{\\prime}=A C=3$,\n\n$\\therefore$ 阴影部分的面积 $=\\frac{1}{2} \\times 3 \\times \\tan 30^{\\circ} \\times 3=\\frac{3 \\sqrt{3}}{2}$.\n\n故答案为 $\\frac{3 \\sqrt{3}}{2}$."} {"id": "3621", "image": [], "answer": "4.5", "solution": "null", "level": "九年级", "question": "直角三角形的两条直角边分别为 5 和 12 , 则其外接圆半径与内切圆半径之差为", "options": [], "subject": "度量几何学", "analysis": "$\\because$ 直角边长分别为 5 和 12 ,\n\n$\\therefore$ 斜边 $=\\sqrt{5^{2}+12^{2}}=13$,\n\n$\\therefore$ :这个直角三角形的外接圆的半径为 6.5 .\n\n$\\because$ 直角三角形内切圆的半径等于两条直角边的和与斜边的差的一半,\n\n$\\therefore$ 其内切圆的半径是 $\\frac{12+5-13}{2}=2$,\n\n$\\therefore 6.5-2=4.5$\n\n故答案为: 4.5 ."} {"id": "3623", "image": ["4947.jpg", "4948.jpg"], "answer": "$28 \\mathrm{~cm}$", "solution": "null", "level": "九年级", "question": "如图, 半径为 $\\sqrt{3} \\mathrm{~cm}$ 的 $\\odot O$ 从斜坡上的 $A$ 点处沿斜坡滚动到平地上的 $C$ 点处, 已知 $\\angle A B C=120^{\\circ}$, $A B=10 \\mathrm{~cm}, B C=20 \\mathrm{~cm}$, 那么圆心 $O$ 运动所经过的路径长度为\n\n", "options": [], "subject": "度量几何学", "analysis": "如图: 当 $\\odot O$ 运动到 $\\odot D$ 位置时, 与 $A B, B C$ 都相切, 连接 $O A, O^{\\prime} C, D E, D F$,\n\n\n\n则 $O A \\perp A B, D E \\perp A B, D F \\perp B C, O^{\\prime} C \\perp B C$,\n\n$\\therefore$ 四边形 $O A E D$ 与四边形 $D F C O^{\\prime}$ 是矩形,\n\n$\\therefore O D=A E, O^{\\prime} D=C F$,\n\n$\\because \\angle A B C=120^{\\circ}$,\n\n$\\therefore \\angle D B F=\\frac{1}{2} \\angle A B C=60^{\\circ}$,\n\n$\\because D F=\\sqrt{3} \\mathrm{~cm}, \\quad \\therefore B F=\\frac{D F}{\\tan 60^{\\circ}}=1 \\mathrm{~cm}$,\n同理: $B E=1 \\mathrm{~cm}$,\n\n$\\therefore A E=A B-B E=9 \\mathrm{~cm}, C F=B C-B F=19 \\mathrm{~cm}$,\n\n$\\therefore O D=9 \\mathrm{~cm}, O^{\\prime} D=19 \\mathrm{~cm}$\n\n$\\therefore$ 圆心 $O$ 运动所经过的路径长度为: $O D+O^{\\prime} D=28 \\mathrm{~cm}$.\n\n故答案为 $28 \\mathrm{~cm}$."} {"id": "3628", "image": ["4956.jpg", "4957.jpg", "4958.jpg"], "answer": "$\\frac{4}{3} \\pi-\\sqrt{3}$", "solution": "null", "level": "九年级", "question": "如图, 有一张矩形纸片 $A B C D$, 其中 $A D=4 \\mathrm{~cm}$, 以 $A D$ 为直径的半圆, 正好与对边 $B C$ 相切, 将它沿 $D E$ 折叠, 使点 $A$ 落在 $B C$ 上, 这时, 半圆还露在外面的部分(阴影部分)的面积是 \\$ \\qquad \\$\n\n", "options": [], "subject": "度量几何学", "analysis": "设半圆的圆心是 $O$, 半圆与 $A D$ 交点是 $F$, 连接 $O F$, 作 $F G \\perp A^{\\prime} D$ 于 $G$.在Rt $\\triangle A C D$ 中, $\\because A D=4, C D \\perp B C$,\n\n$\\therefore C D=\\frac{1}{2} A D=2$,\n\n\n\n(甲)\n\n\n\n(乙)\n\n$\\therefore \\angle C A D=30^{\\circ}$,\n\n$\\therefore \\angle O D F=30^{\\circ}$,\n\n$\\therefore \\angle F O D=120^{\\circ}$.\n\n$\\therefore$ 扇形 $O D F$ 的面积 $=\\frac{120 \\pi \\times 4}{360}=\\frac{4}{3} \\pi$.\n\n在Rt $\\triangle O F G$ 中, $O F=2, \\angle F O G=60^{\\circ}$,\n\n$\\therefore F G=\\sqrt{3}$.\n\n$\\therefore \\triangle O F D$ 的面积 $=\\sqrt{3}$.\n$\\therefore$. 阴影部分的面积 $=\\frac{4}{3} \\pi-\\sqrt{3}$.\n\n故答案为: $\\frac{4}{3} \\pi-\\sqrt{3}$."} {"id": "3644", "image": [], "answer": "9.4", "solution": "null", "level": "九年级", "question": "小王同学想利用树影测量校园内的树高.他在某一时刻测得小树高为 1.5 米时, 其影长为 1.2 米, 当他测量教学楼旁的一棵大树的影长时, 因大树靠近教学楼, 有一部分影子在墙上.经测量, 地面部分影长为 6.4 米, 墙上影长为 1.4 米, 那么这棵大树高约为米.", "options": [], "subject": "度量几何学", "analysis": "设这棵大树高为 $x$,\n\n根据平行投影特点: 在同一时刻, 不同物体的物高和影长成比例.\n\n可得树高比影长为 $\\frac{1.5}{1.2}=1.25$,\n\n则有 $\\frac{6.4}{x-1.4}=\\frac{1}{1.25}=0.8$,\n\n解可得: $x=9.4$ 米."} {"id": "3645", "image": ["4989.jpg", "4990.jpg"], "answer": "4", "solution": "null", "level": "九年级", "question": "如图, 小明在 $A$ 时测得某树的影长为 $2 m, B$ 时又测得该树的影长为 $8 m$. 若两次日照的光线互相垂直, 则此树的高度为 $m$.\n\n", "options": [], "subject": "度量几何学", "analysis": "将实际问题转化为如图所示的模型.\n\n\n由题意, 得 $\\angle E C F=90^{\\circ}, \\angle E D C=\\angle C D F=90^{\\circ}$,\n\n所以 $\\angle E+\\angle E C D=\\angle E C D+\\angle D C F=90^{\\circ}$.\n\n所以 $\\angle E=\\angle D C F$.\n\n所以Rt $\\triangle E D C \\sim R t \\triangle C D F$,\n\n所以 $\\frac{E D}{C D}=\\frac{D C}{D F}$, 即 $\\frac{2}{C D}=\\frac{C D}{8}$, 解得 $D C=4 m$.\n\n则此树的高度为 $4 m$ 。"} {"id": "3672", "image": ["5047.jpg", "5048.jpg"], "answer": "2.7", "solution": "null", "level": "九年级", "question": "如图, 圆桌面正上方的灯泡发出的光线照射桌面后, 在地面上形成阴影(圆形).已知灯泡距离地面 $2.4 m$, 桌面距离地面 $0.8 m$ (桌面厚度不计算), 若桌面的面积是 $1.2 m^{2}$, 则地面上的阴影面积是 \\$ \\qquad \\$ $m^{2}$.\n\n", "options": [], "subject": "度量几何学", "analysis": "如图设 $C, D$ 分别是桌面和其地面影子的圆心, $C B / / A D$,\n\n\n\n$\\therefore \\triangle O B C \\sim \\triangle O A D$\n\n$\\therefore \\frac{S_{\\text {案面 }}}{S_{\\text {地而 }}}=\\left(\\frac{O C}{O D}\\right)^{2}$, 而 $O D=2.4(\\mathrm{~m}), C D=0.8(\\mathrm{~m})$,\n\n$\\therefore O C=O D-C D=1.6(m)$\n\n$\\therefore S_{\\text {地西 }}=2.7\\left(m^{2}\\right)$,\n\n这样地面上阴影部分的面积为 $2.7 \\mathrm{~m}^{2}$.\n故答案为 2.7 ."} {"id": "3673", "image": ["5049.jpg", "5050.jpg"], "answer": "(1)(3)", "solution": "null", "level": "九年级", "question": "如图, 一根直立于水平地面上的木杆 $A B$ 在灯光下形成影子, 当木杆绕 $A$ 按逆时针方向旋转直至到达地面时, 影子的长度发生变化. 设 $A B$垂直于地面时的影长为 $A C$ (假定 $A C>A B$ ), 影长的最大值为 $m$, 最小值为 $n$, 那么下列结论中: (1) $m>A C$; (2) $m=A C$;\n\n(3) $n=A B$;\n\n(4)\n\n\n影子的长度先增大后减小. 正确的结论序号是 \\$ \\qquad \\$ .\n\n(直角填写正确的结论的序号).", "options": [], "subject": "度量几何学", "analysis": "当木杆绕点 $A$ 按逆时针方向旋转时, 如图所示当 $A B$ 与光线 $B C$ 垂直时, $m$ 最大, 则 $m>A C$, (1)成立;\n\n\n\n(1)成立, 那么(2)不成立;\n\n最小值为 $A B$ 与底面重合, 故 $n=A B$, 故(3)成立;\n\n由上可知, 影子的长度先增大后减小, (4)成立;\n\n故答案为: (1) (3) (4)."} {"id": "3674", "image": [], "answer": "$(3.76,0)$", "solution": "null", "level": "九年级", "question": "小明家的客厅有一张直径为 1.1 米, 高 0.75 米的圆桌 $B C$, 在距地面 2 米的 $A$ 处有一或灯, 圆桌的影子为 $D E$, 依据题意建立平面直角坐标系, 其中点 $D$ 的坐标为 $(2,0)$, 则点 $E$ 的坐标是 \\$ \\qquad \\$", "options": [], "subject": "度量几何学", "analysis": "由题意得: $B C / / D E$,\n\n$\\therefore \\triangle A B C \\backsim \\triangle A D E$,\n\n$\\therefore \\frac{B C}{D E}=\\frac{2-0.75}{2}$,\n\n$\\because B C=1.1$,\n\n$\\therefore D E=1.76$,\n\n$\\therefore O E=O D+D E=2+1.76=3.76$.\n\n$\\therefore E(3.76,0)$.\n\n故答案为 $(3.76,0)$."} {"id": "3675", "image": ["5051.jpg"], "answer": "7.5", "solution": "null", "level": "九年级", "question": "如图是学校艺术馆中的柱子, 高 $4.5 \\mathrm{~m}$. 为迎接艺术节的到来, 工作人员用一条花带从柱底向柱顶均匀地缠绕 3 圈, 一直缠到起点的正上方为止. 若柱子的底面周长是 $2 m$, 则这条花带至少需要 \\$ \\qquad \\$ $m$.", "options": [], "subject": "度量几何学", "analysis": "将圆柱表面切开展开呈长方形,\n\n则有螺旋线长为三个长方形并排后的长方形的对角线长 $x$,\n\n\n\n$\\because$ 圆柱高 4.5 米, 底面周长 2 米,\n\n$x^{2}=(2 \\times 3)^{2}+4.5^{2}=56.25 m$,\n\n$x=7.5 m$,\n所以, 花带长至少是 $7.5 m$.\n\n故答案为: 7.5 ."} {"id": "3836", "image": ["5332.jpg"], "answer": "2 或 3", "solution": "null", "level": "九年级", "question": "已知一个圆柱的侧面展开图是如图所示的矩形, 长为 $6 \\pi$, 宽为 $4 \\pi$, 那么这个圆柱底面圆的半径为 \\$ \\qquad \\$ .\n\n", "options": [], "subject": "度量几何学", "analysis": "(1) 当底面圆的周长为 $6 \\pi$ 时, 底面圆的半径为 $6 \\pi \\div \\pi \\div 2=3$;\n\n(2) 当底面圆的周长为 $4 \\pi$ 时, 底面圆的半径为 $4 \\pi \\div \\pi \\div 2=2$.\n\n故答案为 2 或 3."} {"id": "3876", "image": ["5450.jpg", "5451.jpg"], "answer": "$40^{\\circ}$.", "solution": "null", "level": "九年级", "question": "如图, $A B$ 是 $\\odot O$ 的直径, 点 $D$ 在 $A B$ 的延长线上, $D C$ 切 $\\odot O$ 于点 $C$, 若 $\\angle A=25^{\\circ}$, 则 $\\angle D$ 等于 \\$ \\qquad \\$ .\n\n", "options": [], "subject": "度量几何学", "analysis": "连接 $\\mathrm{OC}$,\n\n\n\n$\\because \\mathrm{DC}$ 切 $\\odot \\mathrm{O}$ 于 $\\mathrm{C}, \\therefore \\angle \\mathrm{OCD}=90^{\\circ}, \\because$ 弧 $\\mathrm{BC}$ 对的圆周角是 $\\angle \\mathrm{A}$, 对的圆心角是 $\\angle \\mathrm{COB}$,\n\n$\\therefore \\angle \\mathrm{COB}=2 \\angle \\mathrm{A}=50^{\\circ}, \\quad \\therefore \\angle \\mathrm{D}=180^{\\circ}-\\angle \\mathrm{DCO}-\\angle \\mathrm{COB}=40^{\\circ}$"} {"id": "3877", "image": ["5452.jpg"], "answer": "4", "solution": "null", "level": "九年级", "question": "如图, $\\mathrm{PA}$ 切 $\\odot \\mathrm{O}$ 于 $\\mathrm{A}, \\mathrm{PBC}$ 交 $\\odot \\mathrm{O}$ 于 $\\mathrm{B}, \\mathrm{C}, \\mathrm{PA}=4 \\sqrt{3}, \\mathrm{PC}=12$, 则 $\\mathrm{PB}=$ \\$ \\qquad \\$\n\n", "options": [], "subject": "度量几何学", "analysis": "已知 $P A$ 切 $\\odot O$ 于 $A$,\n\n根据切割线定理, 得 $P A^{2}=P B \\cdot P C ; \\because \\quad P A=4 \\sqrt{3}, P C=12$,\n\n$\\therefore \\quad P B=\\frac{P A^{2}}{P C}=\\frac{48}{12}=4$"} {"id": "3880", "image": ["5457.jpg"], "answer": "122", "solution": "null", "level": "九年级", "question": "如图所示, $\\odot \\mathrm{O}$ 为 $\\triangle \\mathrm{ABC}$ 的内切圆, $\\angle \\mathrm{ABC}=80^{\\circ}, \\angle \\mathrm{ACB}=36^{\\circ}$, 则 $\\angle \\mathrm{BOC}=$ \\$ \\qquad \\$ ${ }^{\\circ}$.\n\n", "options": [], "subject": "度量几何学", "analysis": "$\\because O B 、 O C$ 是 $\\angle A B C 、 \\angle A C B$ 的角平分线,\n\n$\\therefore \\quad \\angle O B C+\\angle O C B=\\frac{1}{2}(\\angle A B C+\\angle A C B)=\\frac{1}{2}\\left(80^{\\circ}+36^{\\circ}\\right)=58^{\\circ}$,\n\n$\\therefore \\angle B O C=180^{\\circ}-58^{\\circ}=122^{\\circ}$. 故答案为 122 ."} {"id": "3900", "image": ["5491.jpg", "5492.jpg"], "answer": "$\\sqrt{3}$", "solution": "null", "level": "九年级", "question": "如图, 半径为 3 的 $\\odot O$ 与 Rt $\\triangle A O B$ 的斜边 $A B$ 切于点 $D$, 交 $O B$ 于点 $C$, 连接 $C D$ 交直线 $O A$ 于点 $\\mathrm{E}$, 若 $\\angle \\mathrm{B}=30^{\\circ}$, 则线段 $\\mathrm{AE}$ 的长为\n\n", "options": [], "subject": "度量几何学", "analysis": "连接 $\\mathrm{OD}$, 如图所示,\n\n由已知可得, $\\angle B O A=90^{\\circ}, \\mathrm{OD}=\\mathrm{OC}=3, \\angle \\mathrm{B}=30^{\\circ}, \\angle \\mathrm{ODB}=90^{\\circ}$,\n\n$\\therefore \\mathrm{BO}=2 \\mathrm{OD}=6, \\angle \\mathrm{BOD}=60^{\\circ}$,\n\n$\\therefore \\angle \\mathrm{ODC}=\\angle \\mathrm{OCD}=60^{\\circ}, \\quad \\mathrm{AO}=\\mathrm{BO} \\tan 30^{\\circ}=6 \\times \\frac{\\sqrt{3}}{3}=2 \\sqrt{3}$,\n\n$\\because \\angle \\mathrm{COE}=90^{\\circ}, \\mathrm{OC}=3$,\n\n$\\therefore \\mathrm{OE}=\\mathrm{OCtan} 60^{\\circ}=3 \\times \\sqrt{3}=3 \\sqrt{3}$,\n\n$\\therefore \\mathrm{AE}=\\mathrm{OE}-\\mathrm{OA}=3 \\sqrt{3}-2 \\sqrt{3}=\\sqrt{3}$,\n\n"} {"id": "3903", "image": ["5497.jpg", "5498.jpg"], "answer": "26", "solution": "null", "level": "九年级", "question": "如图, $A B$ 是 $\\odot O$ 的直径,, 是 $\\odot O$ 上的点, 过点 $C$ 作 $\\odot O$ 的切线交 $A B$ 的延长线于点 D. 若 $\\angle A=32^{\\circ}$,则 $\\angle \\mathrm{D}=$ \\$ \\qquad \\$度.\n\n", "options": [], "subject": "度量几何学", "analysis": ":连接 $\\mathrm{OC}$,\n\n\n\n由圆周角定理得, $\\angle \\mathrm{COD}=2 \\angle \\mathrm{A}=64^{\\circ}$,\n\n$\\because \\mathrm{CD}$ 为 $\\odot \\mathrm{O}$ 的切线,\n\n$\\therefore \\mathrm{OC} \\perp \\mathrm{CD}, \\therefore \\angle \\mathrm{D}=90^{\\circ}-\\angle \\mathrm{COD}=26^{\\circ}$, 故答案为: 26 ."} {"id": "3925", "image": ["5532.jpg"], "answer": "5.5", "solution": "null", "level": "九年级", "question": "如图, 小明同学用自制的直角三角形纸板 DEF 测量树的高度 AB, 他调整自己的位置, 设法使斜边 $\\mathrm{DF}$ 保持水平, 并且边 $\\mathrm{DE}$ 与点 $\\mathrm{B}$ 在同一直线上. 已知纸板的两条直角边 $\\mathrm{DE}=40 \\mathrm{~cm}$, $\\mathrm{EF}=20 \\mathrm{~cm}$, 测得边 $\\mathrm{DF}$ 离地面的高度 $\\mathrm{AC}=1.5 \\mathrm{~m}, \\mathrm{CD}=8 \\mathrm{~m}$, 则树高 $\\mathrm{AB}=$ \\$ \\qquad \\$ $\\mathrm{m}$.\n\n", "options": [], "subject": "度量几何学", "analysis": "在 $\\triangle \\mathrm{DEF}$ 和 $\\triangle \\mathrm{DBC}$ 中, $\\left\\{\\begin{array}{l}\\angle \\mathrm{D}=\\angle \\mathrm{D} \\\\ \\angle \\mathrm{DEF}=\\angle \\mathrm{DCB}\\end{array}\\right.$,\n$\\therefore \\triangle \\mathrm{DEF} \\sim \\triangle \\mathrm{DBC}$,\n\n$\\therefore \\frac{\\mathrm{DE}}{\\mathrm{EF}}=\\frac{\\mathrm{CD}}{\\mathrm{BC}}$, 即 $\\frac{40}{20}=\\frac{8}{\\mathrm{BC}}$\n\n解得 $\\mathrm{BC}=4, \\because \\mathrm{AC}=1.5 \\mathrm{~m}, \\therefore \\mathrm{AB}=\\mathrm{AC}+\\mathrm{BC}=1.5+4=5.5 \\mathrm{~m}$"} {"id": "3926", "image": ["5533.jpg"], "answer": "$\\frac{2 \\sqrt{5}}{5}$", "solution": "null", "level": "九年级", "question": "在 $\\triangle \\mathrm{ABC}$ 中, $\\angle \\mathrm{C}=90^{\\circ}$, 若 $\\tan \\mathrm{A}=\\frac{1}{2}$, 则 $\\sin B=$ \\$ \\qquad \\$ .", "options": [], "subject": "度量几何学", "analysis": "如图所示:\n\n\n\n$\\because \\angle \\mathrm{C}=90^{\\circ}, \\tan \\mathrm{A}=\\frac{1}{2}, \\therefore$ 设 $\\mathrm{BC}=\\mathrm{x}$, 则 $\\mathrm{AC}=2 \\mathrm{x}$, 故 $\\mathrm{AB}=\\sqrt{5} \\mathrm{x}$,\n\n则 $\\sin \\mathrm{B}=\\frac{A C}{A B}=\\frac{2 x}{\\sqrt{5} x}=\\frac{2 \\sqrt{5}}{5}$. 故答案为: $\\frac{2 \\sqrt{5}}{5}$."} {"id": "3927", "image": ["5534.jpg"], "answer": "$\\frac{\\sqrt{5}}{5}$", "solution": "null", "level": "九年级", "question": "如图。在 $4 \\times 4$ 的正方形方格图形中, 小正方形的顶点称为格点. $\\triangle A B C$ 的顶点都在格点上,则 $\\angle B A C$ 的正弦值是 \\$ \\qquad \\$ .\n\n", "options": [], "subject": "度量几何学", "analysis": "$\\because A B^{2}=3^{2}+4^{2}=25, A C^{2}=2^{2}+4^{2}=20, B C^{2}=1^{2}+2^{2}=5, \\therefore A C^{2}+B C^{2}=A B^{2}, \\therefore \\triangle A B C$ 为直角三角形, 且 $\\angle A C B=90^{\\circ}$, 则 $\\sin \\angle B A C=\\frac{B C}{A B}=\\frac{\\sqrt{5}}{5}$. 故答案为: $\\frac{\\sqrt{5}}{5}$."} {"id": "3302", "image": ["4278.jpg"], "answer": "12", "solution": "null", "level": "九年级", "question": "在测量旗杆高度的活动课中, 某小组学生于同一时刻在阳光下对一根直立于平地的竹竿及其影长和旗杆的影长进行了测量, 得到的数据如图所示, 根据这些数据计算出旗杆的高度为 $m$.\n\n", "options": [], "subject": "度量几何学", "analysis": "【分析】\n\n本题只要是把平行投影的问题抽象到相似三角形中, 利用相似三角形的相似比, 列出方程, 通过解方程求解即可, 体现了转化的思想. 此题的文字叙述比较多, 解题时要认真分析题意.\n\n利用平行投影的性质, 相似三角形的对应边成比例解答.\n\n【解答】\n\n解:设旗杆的高度为 $x m$,\n\n根据题意, 得: $\\frac{x}{9}=\\frac{0.8}{0.6}$,\n\n解得 $x=12$,\n\n即旗杆的高度为 $12 m$,\n\n故答案为: 12 ."} {"id": "3305", "image": [], "answer": "$\\frac{15}{\\pi}$", "solution": "null", "level": "九年级", "question": "将面积为 $225 \\mathrm{~cm}^{2}$ 的正方形硬纸片围成圆柱的侧面,则此圆柱的底面直径为 $c m$ (结果保留 $\\pi$.", "options": [], "subject": "度量几何学", "analysis": "解: 由面积为 $225 \\mathrm{~cm}^{2}$ 的正方形可知正方形的边长 $=\\sqrt{225}=15 \\mathrm{~cm}$, 即是圆柱底面的周长,所以用这硬纸片围成圆柱的侧面的直径 $=\\frac{15}{\\pi} \\mathrm{cm}$,\n\n故答案为: $\\frac{15}{\\pi}$.\n\n圆柱的底面直径 $=$ 底面周长 $\\div \\pi$.\n\n此题考查圆柱的计算, 本题要先由正方形的面积求出正方形的周长, 然后再求底面的直径."} {"id": "2983", "image": ["3887.jpg", "3888.jpg"], "answer": "8", "solution": "null", "level": "九年级", "question": "(本题3分)(2019$cdot$浙江杭州$cdot$九年级)如图,在$triangle{ABC}$中,${AB}={AC}=5,{BC}=4$$sqrt{5},{D}$为边${AB}$上一动点(${B}$点除外),以${CD}$为一边作正方形${CDEF}$,连接${BE}$,则$triangle{BDE}$面积的最大值为", "options": [], "subject": "度量几何学", "analysis": "【分析】如图,过点$A$作$AHperpBC$于$H$,过点$E$作$EMperpAB$于$M$,过点$C$作$CNperpAB$于$N$,根据等腰三角形的性质以及三角形的面积可求出${CN}=4$,继而根据勾股定理求出$AN=3$,从而求得$BN$的长,然后证明$triangleEDMcongtriangleDCN$,根据全等三角形的性质可得${EM}={DN}$,设${BD}={x}$,则${DN}=8-{x}$,继而根据三角形的面积公式可得${S}_{triangle{BDE}}=$$-frac{1}{2}(x-4)^{2}+8(0【点睛】本题考查了等腰三角形的性质,正方形的性质,全等三角形的判定与性质,二次函数的应用等,综合性质较强,有一定的难度,正确添加辅助线,熟练运用相关知识是解题的关键."} {"id": "3029", "image": ["3924.jpg"], "answer": "$frac{1}{3}$", "solution": "null", "level": "九年级", "question": "(本题3分)(2021浙江宁波$cdot$九年级期末)如图,点$D$在$triangleABC$的$BC$边上,且$CD=2BD$,点$E$是$AC$边的中点,连接$AD,DE$,假设可以随意在图中取点,那么这个点取在阴影部分的概率是", "options": [], "subject": "度量几何学", "analysis": "【分析】先设阴影部分的面积是$x$,得出整个图形的面积是$3x$,再根据几何概率的求法即可得出答案.【详解】解:设阴影部分的面积是$x$,$because$点$E$是$AC$边的中点,$thereforeS_{triangleACD}=2x$,$becauseCD=2BD$$thereforeS_{triangleACB}=3x$,则这个点取在阴影部分的概率是$frac{x}{3x}=frac{1}{3}$.故答案为:$frac{1}{3}$.【点睛】本题考查几何概率的求法:首先根据题意将代数关系用面积表示出来,一般用阴影区域表示所求事件(A);然后计算阴影区域的面积在总面积中占的比例,这个比例即事件(A)发生的概率."} {"id": "3045", "image": ["3936.jpg"], "answer": "10", "solution": "null", "level": "九年级", "question": "(本题3分)(2020$cdot$浙江温州$cdot$九年级期末)在Rt$triangleABC$中,两直角边的长分别为6和8,则这个三角形的外接圆的直径长为", "options": [], "subject": "度量几何学", "analysis": "【分析】根据题意,写出已知条件并画出图形,然后根据勾股定理即可求出${AB}$,再根据圆周角为直角所对的弦是直径即可得出结论.【详解】如图,已知:$AC=8,BC=6$,由勾股定理得:$AB=sqrt{{AC}^{2}+{BC}^{2}}=10$,$becauseangleACB=90^{circ}$,$thereforeAB$是$odotO$的直径,$therefore$这个三角形的外接圆直径是$10;$故答案为:10.【点睛】此题考查的是求三角形的外接圆的直径,掌握圆周角为直角所对的弦是直径是解决此题的关键."} {"id": "3046", "image": ["3937.jpg"], "answer": "$22.5^{circ}$", "solution": "null", "level": "九年级", "question": "(本题3分)($2020cdot$浙江$cdot$温州市第二中学九年级阶段练习)如图,正八边形度.", "options": [], "subject": "度量几何学", "analysis": "【分析】正八边形内接于圆,可求得${GF}$所对的圆心角为$45^{circ}$,进而可求得${GF}$所对的圆周角的度数.【详解】解:$because$多边形为正八边形$therefore$正八边形$ABCDEFGH$内接于圆$therefore{GF}$所对的圆心角为$45^{circ}$$therefore$GF所对的圆周角$angleGBF$为$22.5^{circ}$故答案为:$22.5^{circ}$.【点睛】本题主要考查了正多边形与圆,同弧所对的圆周角与圆心角的关系,解题的关键是掌握正多边形与圆的关系."} {"id": "3047", "image": [], "answer": "$80^{circ}80$度", "solution": "null", "level": "九年级", "question": "(本题3分)(2021浙江金华一模)一个圆被三条半径分成面积比为$2:3:4$的三个扇形,则最小扇形的圆心角为$qquad$.", "options": [], "subject": "度量几何学", "analysis": "【分析】因为一个圆被三条半径分成面积比为2:$3:4$的三个扇形,所以其圆心角之比也为2:$3:4$,则最小扇形的圆心角度数可求.【详解】解:由题意可得,三个圆心角的和为$360^{circ}$,又因为三个圆心角的度数比为$2:3:4$,所以最小的圆心角度数为:$360^{circ}timesfrac{2}{9}=80^{circ}$.故答案为:$80^{circ}$.【点睛】此题考查扇形统计图及相关计算,解题的关键是掌握圆心角的度数$=360^{circ}times$该部分占总体的百分比."} {"id": "3048", "image": ["3938.jpg", "3939.jpg", "3940.jpg"], "answer": "3", "solution": "null", "level": "九年级", "question": "(本题3分)(2022-浙江宁波$cdot$九年级期末)如图1,水车又称孔明车,是我国最古老的农业灌溉工具,是珍贵的历史文化遗产.如图2,圆心$O$在水面上方,且$odotO$被水面截得的弦$AB$长为8米,半径为5米,则圆心$O$到水面$AB$的距离为$qquad$米.图1$because$图2", "options": [], "subject": "度量几何学", "analysis": "【分析】过$O$作$ODperpAB$于$D$,连接$OA$,由垂径定理得$AD=BD=frac{1}{2}AB=4$(米),然后在Rt$triangleAOD$中,由勾股定理求出$OD$的长即可.【详解】解:过$O$作$ODperpAB$于$D$,连接$OA$,如图所示:图2则$AD=BD=frac{1}{2}AB=4$(米),在Rt$triangleAOD$中,由勾股定理得:$OD=sqrt{OA^{2}-AD^{2}}=sqrt{5^{2}-4^{2}}=3$(米),即圆心$O$到水面$AB$的距离为3米,故答案为:3.【点睛】本题考查了垂径定理的应用和勾股定理的应用,熟练掌握垂径定理和勾股定理是解题的关键."} {"id": "3049", "image": ["3941.jpg"], "answer": "$30^{circ}30$度", "solution": "null", "level": "九年级", "question": "(本题3分)(2022$cdot$浙江湖州$cdot$中考真题)如图,已知$AB$是$odotO$的弦,$angleAOB=$$120^{circ},OCperpAB$,垂足为$C,OC$的延长线交$odotO$于点${D}$.若$angleAPD$是$AD$所对的圆周角,则$angleAPD$的度数是", "options": [], "subject": "度量几何学", "analysis": "【分析】根据垂径定理得出$angleAOB=angleBOD$,进而求出$angleAOD=60^{circ}$,再根据圆周角定理可得$angleAPD=frac{1}{2}angleAOD=30^{circ}$.【详解】$becauseOCperpAB,OD$为直径,$thereforeBD=AD$,$thereforeangleAOB=angleBOD$,$becauseangleAOB=120^{circ}$$thereforeangleAOD=60^{circ}$$thereforeangleAPD=frac{1}{2}angleAOD=30^{circ}$,故答案为:$30^{circ}$.【点睛】本题考查了圆周角定理、垂径定理等知识,掌握垂径定理是解答本题的关键."} {"id": "3050", "image": ["3942.jpg", "3943.jpg"], "answer": "$30^{circ}30$度", "solution": "null", "level": "九年级", "question": "(本题3分)(2022$cdot$浙江温州$cdot$模拟预测)如图,四边形$ABCD$内接于$odotO$,已知$angleBAD=angleBCD=90^{circ},AD=CD$,且$angleADC=120^{circ}$,若点$E$为弧$BC$的中点,连接$DE$,则$angleCDE$的大小是$qquad$", "options": [], "subject": "度量几何学", "analysis": "【分析】连接$BD$,根据圆内接四边形的性质求出$angleABC$,根据弧、弦、圆心角之间的关系求出$angleABD=angleCBD=30^{circ}$,求出$angleBDC$,再求出答案即可.【详解】解:连接$BD$,$because$四边形$ABCD$是$odotO$的内接四边形,$thereforeangleABC+angleADC=180^{circ}$,$becauseangleADC=120^{circ}$,$thereforeangleABC=60^{circ}$,$becauseAD=CD$,$thereforeAD=CD$,$thereforeangleDBC=angleABD=frac{1}{2}times60^{circ}=30^{circ}$,$becauseangleBCD=90^{circ}$$thereforeangleBDC=90^{circ}-angleCBD=60^{circ}$,$becauseE$为$BC$的中点,$thereforeangleCDE=angleBDE=frac{1}{2}angleBDC=30^{circ}$;故答案为:$30^{circ}$.【点睛】本题考查了圆内接四边形的性质,圆周角定理,弧、弦、圆心角之间的关系等知识点,能熟记知识点是解此题的关键."} {"id": "3051", "image": ["3944.jpg", "3945.jpg"], "answer": "$frac{pi}{3}+frac{sqrt{3}}{2}$", "solution": "null", "level": "九年级", "question": "(本题3分)($2021cdot$浙江$cdot$温州市第二中学二模)如图,已知扇形$AOB$的圆心角为$90^{circ},C$是半径$OA$的中点,过$C$作$CDperpOA$交$AB$于点$D$.若$OA=2$,则阴影部分的面积为", "options": [], "subject": "度量几何学", "analysis": "【分析】连接$OD、AD$,根据线段垂直平分线的性质得出$OD=AD$,继而可得$triangleADO$为等边三角形,求出扇形$BOD$的面积,再加上$S_{triangle}COD$即可求出阴影部分的面积.【详解】解:连接$OD、AD$,$becauseC$是半径$OA$的中点,过$C$作$CDperpOA$交$AB$于点$D$.$thereforeOD=AD$,$thereforetriangleADO$为等边三角形,$thereforeangleDOC=60^{circ}$,$thereforeCD=frac{sqrt{3}}{2}OD=sqrt{3}$,$becauseangleAOB=90^{circ}$,$thereforeangleBOD=30^{circ}$,$thereforeS_{text{阴影}}=S_{text{扇形}}BOD+S_{triangle}COD=frac{30pitimes2^{2}}{360}+frac{1}{2}times1timessqrt{3}=frac{pi}{3}+frac{sqrt{3}}{2}$,故答案为:$frac{pi}{3}+frac{sqrt{3}}{2}$.【点睛】本题考查了扇形的面积计算,线段垂直平分线的性质,解答本题的关键是掌握扇形的面积公式."} {"id": "3069", "image": [], "answer": "$6pi$", "solution": "null", "level": "九年级", "question": "(本题3分)(2021$cdot$浙江$cdot$温州外国语学校九年级期中)已知扇形的弧长为$2pi{cm}$,半径为$6{~cm}$,则它的面积为$qquad$${cm}^{2}$.", "options": [], "subject": "度量几何学", "analysis": "【分析】直接根据扇形的面积公式即可得出结论.【详解】$because$扇形的弧度长为$2pi{cm}$,半径为$6{~cm}$$therefore$扇形的面积$S_{text{扇形}}=frac{1}{2}times2pitimes6=6pi{cm}^{2}$故答案为:$6pi$.【点睛】本题考查了扇形面积的计算,掌握扇形面积的计算是解题的关键."} {"id": "3070", "image": [], "answer": "$112.5^{circ}$", "solution": "null", "level": "九年级", "question": "(本题3分)(2020$cdot$浙江绍兴$cdot$模拟预测)圆内接四边形$ABCD$中,若$angleA=2angleB=3angleC$,则$angleD$的度数是$qquad$", "options": [], "subject": "度量几何学", "analysis": "【分析】先利用圆内接四边形对角互补得出$angle{A}+angle{C}=angle{B}+angle{D}=180^{circ}$,再把$angle{A}=3angle{C}$代入求出$angle{C}=45^{circ}$,那么$angle{A}=135^{circ}=2angle{B}$,再求出$angle{B}=67.5^{circ}$,则$angle{D}=180^{circ}-angle{B}=112.5^{circ}$.【详解】解:$because$四边形${ABCD}$是圆内接四边形,$thereforeangle{A}+angle{C}=angle{B}+angle{D}=180^{circ}$,$becauseangle{A}=2angle{B}=3angle{C}$,$therefore3angle{C}+angle{C}=180^{circ}$,$thereforeangle{C}=45^{circ}$$thereforeangle{A}=135^{circ}=2angle{B}$,$thereforeangle{B}=67.5^{circ}$,$thereforeangle{D}=180^{circ}-angle{B}=112.5^{circ}$.故答案为:$112.5^{circ}$.【点睛】此题主要考查了圆内接四边形的性质,掌握圆内接四边形对角互补是解题关键."} {"id": "3071", "image": ["3972.jpg", "3973.jpg"], "answer": "12", "solution": "null", "level": "九年级", "question": "(本题3分)(2021$cdot$浙江湖州$cdot$二模)如图所示一个圆柱体容器内装入一些水,截面$AB$在圆心$O$下方,若$odotO$的直径为$60{~cm}$,水面宽$AB=48{~cm}$,则水的最大深度为${cm}$.", "options": [], "subject": "度量几何学", "analysis": "【分析】连接$OB$,过点$O$作$OCperpAB$于点$D$,交$odotO$于点$C$,先由垂径定理求出$BD$的长,再根据勾股定理求出$OD$的长,进而得出$CD$的长即可.【详解】解:连接$OB$,过点$O$作$OCperpAB$于点$D$,交$odotO$于点$C$,如图所示:$becauseAB=48{~cm}$,$thereforeBD=frac{1}{2}AB=frac{1}{2}times48=24({~cm})$,$becauseodotO$的直径为$60{~cm}$,$thereforeOB=OC=30{~cm}$在Rt$triangleOBD$中,$OD=sqrt{OB^{2}-BD^{2}}=sqrt{30^{2}-24^{2}}=18({~cm})$,$thereforeCD=OC-OD=30-18=12({~cm})$,即水的最大深度为$12{~cm}$,故答案为:12.【点睛】本题考查了垂径定理、勾股定理等知识;根据题意作出辅助线,构造出直角三角形是解答此题的关键."} {"id": "3073", "image": ["3975.jpg", "3976.jpg"], "answer": "125", "solution": "null", "level": "九年级", "question": "(本题3分)(2021ㄱ九年级期中)如图,在$triangleABC$中,$angleB=70^{circ},odotO$截三边所得的", "options": [], "subject": "度量几何学", "analysis": "【分析】过点$O$作$OMperpDE$于$M,OKperpFG$于$K,OPperpHI$于$P$,如图,由于$DE=FG=HI$$=$,利用弦、圆心角和对应的弦心距的关系得到$OM=OK=OP$,则可判断$OA$平分$angleBAC$,$OC$平分$angleACB$,然后根据角平分线的定义和三角形内角和求解.【详解】解:过点$O$作$OMperpDE$于$M,OKperpFG$于$K,OPperpHI$于$P$,如图,$becauseDE=FG=HI$$thereforeOM=OK=OP$,$thereforeOA$平分$angleBAC,OC$平分$angleACB$,$thereforeangleOAC+angleOCA=frac{1}{2}(angleBAC+angleACB)=frac{1}{2}left(180^{circ}-angleBright)=90^{circ}-frac{1}{2}angleB=90^{circ}-frac{1}{2}times70^{circ}=55^{circ}$,$thereforeangleAOC=180^{circ}-(angleOAC+angleOCA)$$=180^{circ}-55^{circ}$$=125^{circ}$.故答案为:125.【点睛】本题考查了角平分线的性质:角的平分线上的点到角的两边的距离相等;在角的内部,到角的两边距离相等的点在这个角的平分线上.也考查了弦、弧、圆心角和弦心距的关系."} {"id": "3575", "image": ["4871.jpg", "4872.jpg"], "answer": "$2 \\sqrt{3}$", "solution": "null", "level": "九年级", "question": "如图 $\\odot P$ 与 $x$ 轴、直线 $l$ 相切于 $A 、 B$ 两点, $\\odot P$ 半径为 1 , 直线 $l$解析式为 $y=\\sqrt{3} x$, 在直线 1 和 $x$ 轴上有两个动点 $M 、 N($ 均不同于点 $O)$, 则 $\\triangle P M N$ 的周长最小值为 \\$ \\qquad \\$\n\n", "options": [], "subject": "解析几何", "analysis": "如图, 连接 $P A, P B$, 并延长使 $P B=B D, P A=A C$, 连接 $C D$, 和直线和, $x$ 轴分别交于点 $M$,点 $N$, 作 $P E \\perp C D$,\n\nD\n\n\n\n$\\because$ 直线 $l$ 相切于 $A, B$ 两点,\n\n$\\therefore P B \\perp l, P A \\perp x$ 轴,\n\n$\\therefore P M=D M, P N=C N$,\n\n$\\because y=\\sqrt{3} x$,\n\n$\\therefore \\angle A O B=60^{\\circ}$,\n\n$\\therefore \\angle B P A=120^{\\circ}$,\n\n$\\because P A=P B$\n\n$\\therefore P D=P C$,\n\n$\\therefore \\angle E P C=60^{\\circ}$,\n\n$\\therefore \\angle C=30^{\\circ}$,\n\n$\\therefore P E=\\frac{1}{2} P C=1$,\n\n$\\therefore E C=\\sqrt{3}$,\n\n$\\therefore D C=2 E C=2 \\sqrt{3}$,\n\n$\\therefore \\triangle P M N$ 的周长为 $P M+P N+M N=D C=2 \\sqrt{3}$,\n\n故答案为 $2 \\sqrt{3}$."} {"id": "3577", "image": ["4873.jpg", "4874.jpg"], "answer": "2", "solution": "null", "level": "九年级", "question": "如图, 在边长为 1 的小正方形组成的网格中, 点 $A, B, C, D$ 都在这些小正方形的顶点上, $A B, C D$ 相交于点 $O$, 则 $\\tan \\angle A O D=$\n\n", "options": [], "subject": "解析几何", "analysis": "如图, 连接 $B E$,\n\n\n\n$\\because$ 四边形 $B C E D$ 是正方形,\n\n$\\therefore D F=C F=\\frac{1}{2} C D, B F=\\frac{1}{2} B E, C D=B E, B E \\perp C D$,\n\n$\\therefore B F=C F$,\n\n根据题意得: $A C / / B D$,\n\n$\\therefore \\triangle A C P \\sim \\triangle B D P$,\n\n$\\therefore D P: C P=B D: A C=1: 3$,\n\n$\\therefore D P: D F=1: 2$,\n\n$\\therefore D P=P F=\\frac{1}{2} C F=\\frac{1}{2} B F$,\n\n在 $R t \\triangle P B F$ 中, $\\tan \\angle B P F=\\frac{B F}{P F}=2$,\n\n$\\because \\angle A P D=\\angle B P F$,\n\n$\\therefore \\tan \\angle A P D=2$,\n\n故答案为 2 ."} {"id": "3598", "image": [], "answer": "$\\frac{\\sqrt{3}}{2}$ 或 $\\frac{2 \\sqrt{5}}{5}$", "solution": "null", "level": "九年级", "question": "在直角三角形 $A B C$ 中, 若 $2 A B=A C$, 则 $\\cos C=$ \\$ \\qquad \\$", "options": [], "subject": "解析几何", "analysis": "若 $\\angle B=90^{\\circ}$, 设 $A B=x$, 则 $A C=2 x$,\n\n所以 $B C=\\sqrt{(2 x)^{2}-x^{2}}=\\sqrt{3} x$,\n\n所以 $\\cos C=\\frac{B C}{A C}=\\frac{\\sqrt{3} x}{2 x}=\\frac{\\sqrt{3}}{2}$;\n\n若 $\\angle A=90^{\\circ}$, 设 $A B=x$, 则 $A C=2 x$,\n\n所以 $B C=\\sqrt{(2 x)^{2}+x^{2}}=\\sqrt{5} x$,\n\n所以 $\\cos C=\\frac{A C}{B C}=\\frac{2 x}{\\sqrt{5} x}=\\frac{2 \\sqrt{5}}{5}$;\n\n综上所述, $\\cos C$ 的值为 $\\frac{\\sqrt{3}}{2}$ 或 $\\frac{2 \\sqrt{5}}{5}$.\n\n故答案为 $\\frac{\\sqrt{3}}{2}$ 或 $\\frac{2 \\sqrt{5}}{5}$."} {"id": "3622", "image": ["4945.jpg", "4946.jpg"], "answer": "$\\left(-\\frac{7}{3}, 0\\right)$ 或 $P\\left(-\\frac{17}{3}, 0\\right)$", "solution": "null", "level": "九年级", "question": "如图, 直线 $y=-\\frac{3}{4} x-3$ 交 $x$ 轴于点 $A$, 交 $y$ 轴于点 $B$, 点 $P$ 是 $x$ 轴上一动点, 以点 $P$ 为圆心,以 1 个单位长度为半径作 $\\odot P$, 当 $\\odot P$ 与直线 $A B$ 相切时, 点 $P$ 的坐标是 \\$ \\qquad \\$\n\n", "options": [], "subject": "解析几何", "analysis": "$\\because$ 直线 $y=-\\frac{3}{4} x-3$ 交 $x$ 轴于点 $A$, 交 $p$ 轴于点 $B$,\n\n$\\therefore$ 令 $x=0$, 得 $y=-3$, 令 $y=0$, 得 $x=-4$,\n\n$\\therefore A(-4,0), B(0 .-3)$,\n\n$\\therefore O A=4, O B=3$,\n$\\therefore A B=5$,\n\n设 $\\odot P$ 与直线 $A B$ 相切于 $D$, 连接 $P D$,\n\n\n\n则 $P D \\perp A B, P D=1$,\n\n$\\because \\angle A D P=\\angle A O B=90^{\\circ}, \\angle P A D=\\angle B A O$,\n\n$\\therefore \\triangle A P D \\sim \\triangle A B O$,\n\n$\\therefore \\frac{P D}{O B}=\\frac{A P}{A B}$,\n\n$\\therefore \\frac{1}{3}=\\frac{A P}{5}$,\n\n$\\therefore A P=\\frac{5}{3}$,\n\n$\\therefore O P=\\frac{7}{3}$ 或 $O P=\\frac{17}{3} ,$\n\n$\\therefore P\\left(-\\frac{7}{3}, 0\\right)$ 或 $P\\left(-\\frac{17}{3}, 0\\right)$,\n\n故答案为 $\\left(-\\frac{7}{3}, 0\\right)$ 或 $P\\left(-\\frac{17}{3}, 0\\right)$."} {"id": "3624", "image": ["4949.jpg", "4950.jpg"], "answer": "$\\sqrt{3}$", "solution": "null", "level": "九年级", "question": "如图, 菱形 $A B C D, \\angle B=60^{\\circ}, A B=4, \\odot O$ 内切于菱形 $A B C D$, 则 $\\odot O$ 的半径为\n\n", "options": [], "subject": "解析几何", "analysis": "设 $A B$ 和 $B C$ 上的切点分别为 $E 、 F$, 连接 $O A 、 O E 、 O B 、 O F$, 则 $O E \\perp A B, O F \\perp B C$,\n\n\n\n$\\because \\odot O$ 内切于菱形 $A B C D$,\n\n$\\therefore O E=O F$,\n\n$\\therefore O B$ 平分 $\\angle A B C$,\n\n$\\because \\angle A B C=60^{\\circ}$,\n\n$\\therefore \\angle A B O=30^{\\circ}$,\n\n同理得 $\\angle B A O=60^{\\circ}$,\n\n$\\therefore \\angle A O B=90^{\\circ}$,\n\n$\\therefore A O=\\frac{1}{2} A B=2, O B=2 \\sqrt{3}$,\n\n$\\therefore S_{\\triangle A O B}=\\frac{1}{2} A B \\cdot O E=\\frac{1}{2} A O \\cdot O B$,\n\n$4 O E=2 \\times 2 \\sqrt{3}$,\n\n$O E=\\sqrt{3}$,\n\n故答案为: $\\sqrt{3}$."} {"id": "3627", "image": ["4954.jpg", "4955.jpg"], "answer": "$\\pi$", "solution": "null", "level": "九年级", "question": "如图, 在 $R t \\triangle A B C$ 中, $\\angle C=90^{\\circ}, A B=4 \\sqrt{2}$, 以 $A B$ 的中点 $O$ 为圆心作圆, 圆 $O$ 分别与 $A C$ 、 $B C$ 相切于点 $D 、 E$ 两点, 则弧 $D E$ 的长为 \\$ \\qquad \\$ .\n\n", "options": [], "subject": "解析几何", "analysis": "连接 $O E, O D$,\n\n$\\because$ 圆 $O$ 分别与 $A C 、 B C$ 相切于点 $D 、 E$ 两点,\n\n$\\therefore O E \\perp B C, O D \\perp A C$,\n\n\n$\\because \\angle C=90^{\\circ}$,\n\n$\\therefore$ 四边形 $O E C D$ 是矩形,\n\n$\\because O E=O D$,\n\n$\\therefore$ 矩形 $O E C D$ 是正方形,\n\n$\\therefore C E=C D, \\angle E O D=90^{\\circ}$,\n\n$\\therefore \\angle B+\\angle B O E=\\angle B O E+\\angle A O D=90^{\\circ}$,\n\n$\\therefore \\angle B=\\angle A O D$,\n\n$\\because \\angle B E O=\\angle A D O=90^{\\circ}, O B=O A$,\n\n$\\therefore \\triangle B O E \\cong \\triangle O A D(A A S)$,\n\n$\\therefore B E=O D, O E=A D$,\n\n$\\therefore B E=O E=O D=A D$,\n\n$\\therefore \\angle B=\\angle A=45^{\\circ}$,\n\n$\\because A B=4 \\sqrt{2}$,\n\n$\\therefore O E=O D=2$,\n\n$\\therefore$. 弧 $D E$ 的长 $=\\frac{90 \\cdot \\pi \\times 2}{180}=\\pi$,\n\n故答案为: $\\pi$."} {"id": "3875", "image": ["5448.jpg", "5449.jpg"], "answer": "55", "solution": "null", "level": "九年级", "question": "如图, $\\mathrm{PA} 、 \\mathrm{~PB}$ 分别切 $\\odot \\mathrm{O}$ 于点 $\\mathrm{A} 、 \\mathrm{~B}$, 若 $\\angle \\mathrm{P}=70^{\\circ}$, 则 $\\angle \\mathrm{C}$ 的大小为 \\$ \\qquad \\$ (度) 。\n\n", "options": [], "subject": "解析几何", "analysis": "连接 $\\mathrm{OA}, \\mathrm{OB}$,\n\n\n\n$\\because \\mathrm{PA} 、 \\mathrm{~PB}$ 分别切 $\\odot \\mathrm{O}$ 于点 A、B, $\\therefore \\mathrm{OA} \\perp \\mathrm{PA}, \\mathrm{OB} \\perp \\mathrm{PB}$, 即 $\\angle \\mathrm{PAO}=\\angle \\mathrm{PBO}=90^{\\circ}$.\n\n$\\therefore \\angle \\mathrm{AOB}=360^{\\circ}-\\angle \\mathrm{PAO}-\\angle \\mathrm{P}-\\angle \\mathrm{PBO}=360^{\\circ}-90^{\\circ}-70^{\\circ}-90^{\\circ}=110^{\\circ}$.\n\n$\\therefore \\angle \\mathrm{C}$ 和 $\\angle \\mathrm{AOB}$ 是同弧所对的圆周角和圆心角, $\\therefore \\angle \\mathrm{C}=\\frac{1}{2} \\angle \\mathrm{AOB}=55^{\\circ}$."} {"id": "3899", "image": ["5489.jpg", "5490.jpg"], "answer": "$115^{\\circ}$", "solution": "null", "level": "九年级", "question": "如图, 四边形 $A B C D$ 内接于 $\\odot O, A B$ 是 $\\odot O$ 的直径, 过点 $C$ 作 $\\odot O$ 的切线交 $A B$ 的延长线于点 $P$,若 $\\angle P=40^{\\circ}$, 则 $\\angle A D C=$ ${ }^{\\circ}$.\n\n", "options": [], "subject": "解析几何", "analysis": "\n\n由题意可得, $\\angle O C P=90^{\\circ}, \\angle P=40^{\\circ}$,\n\n$\\therefore \\angle \\mathrm{COB}=50^{\\circ}, \\because \\mathrm{OC}=\\mathrm{OB}$,\n\n$\\therefore \\angle \\mathrm{OCB}=\\angle \\mathrm{OBC}=65^{\\circ}, \\because$ 四边形 $\\mathrm{ABCD}$ 是圆内接四边形,\n\n$\\therefore \\angle \\mathrm{D}+\\angle \\mathrm{ABC}=180^{\\circ}, \\therefore \\angle \\mathrm{D}=115^{\\circ}$, 故答案为: $115^{\\circ}$."} {"id": "3902", "image": ["5496.jpg"], "answer": "$(\\sqrt{6}, 2)$ 或 $(-\\sqrt{6}, 2)$", "solution": "null", "level": "九年级", "question": "如图, 已知 $\\odot P$ 的半径为 2 , 圆心 $P$ 在抛物线 $y=\\frac{1}{2} x^{2}-1$ 上运动, 当 $\\odot P$ 与 $x$ 轴相切时, 圆心 $P$的坐标为 \\$ \\qquad \\$ .\n\n", "options": [], "subject": "解析几何", "analysis": "根据直线和圆相切, 则圆心到直线的距离等于圆的半径, 得点 $\\mathrm{P}$ 的纵坐标是 2 或 -2 .\n\n当 $y=2$ 时, $\\frac{1}{2} x^{2}-1=2$, 解得 $x= \\pm \\sqrt{6}$\n\n当 $y=-2$ 时, $\\frac{1}{2} \\quad x^{2}-1=-2$, 方程无解\n\n故 $P$ 点的坐标为 $(\\sqrt{6}, 2)$ 或 $(-\\sqrt{6}, 2)$"} {"id": "3922", "image": [], "answer": "$60^{\\circ}$", "solution": "null", "level": "九年级", "question": "已知 $\\alpha$ 为锐角, 且 $2 \\cos ^{2} \\alpha-5 \\cos \\alpha+2=0$, 则 $\\alpha=$", "options": [], "subject": "解析几何", "analysis": "设 $\\cos \\alpha=\\mathrm{A}$,\n\n则原式 $=2 A^{2}-5 A+2=0$,\n\n解得, $A=2$ (因为 $\\cos \\alpha \\neq 2$, 舍) 或 $\\frac{1}{2}$,\n\n$\\therefore \\cos \\alpha=\\frac{1}{2}, \\therefore \\alpha=60^{\\circ}$. 故答案为: $60^{\\circ}$."} {"id": "3923", "image": [], "answer": "(2)", "solution": "null", "level": "九年级", "question": "在 Rt $\\triangle \\mathrm{ABC}$ 中, $\\angle \\mathrm{C}=90^{\\circ}, \\mathrm{a} 、 \\mathrm{~b} 、 \\mathrm{c}$ 分别是 $\\angle \\mathrm{A} 、 \\angle \\mathrm{B} 、 \\angle \\mathrm{C}$ 的对边, 下列式子: (1) $\\mathrm{a}=\\mathrm{c} \\cdot \\sin \\mathrm{B}$, (2) $\\mathrm{a}=\\mathrm{c} \\cdot \\cos \\mathrm{B}$, (3) $\\mathrm{a}=\\mathrm{c} \\cdot \\tan \\mathrm{B}$, (4) $\\mathrm{a}=\\frac{c}{\\tan \\mathrm{B}}$, 必定成立的是", "options": [], "subject": "解析几何", "analysis": "在 Rt $\\triangle A B C$ 中, $\\angle C=90^{\\circ}, a 、 b 、 c$ 分别是 $\\angle A 、 \\angle B 、 \\angle C$ 的对边,则: $\\sin B=\\frac{b}{c}, \\cos B=\\frac{a}{c}, \\tan B=\\frac{b}{a}$.\n\n即: $b=a \\cdot \\sin B, a=c \\cdot \\cos B, a=\\frac{b}{\\tan B}$. 必定成立的是(2).故答案为: (2)."} {"id": "3924", "image": [], "answer": "$45^{\\circ}$ 35", "solution": "null", "level": "九年级", "question": "在 $\\triangle A B C$ 中, $\\angle C=90^{\\circ}, a=35, c=35 \\sqrt{2}$, 则 $\\angle A=$ \\$ \\qquad \\$ ,$b=$ \\$ \\qquad \\$ .", "options": [], "subject": "解析几何", "analysis": "(1) $\\because$ 在 $\\triangle \\mathrm{ABC}$ 中, $\\angle \\mathrm{C}=90^{\\circ}, a=35, c=35 \\sqrt{2}$,\n\n$\\therefore \\sin \\mathrm{A}=\\frac{a}{c}=\\frac{35}{35 \\sqrt{2}}=\\frac{\\sqrt{2}}{2}, \\therefore \\angle \\mathrm{A}=45^{\\circ} ;$\n\n(2) $\\because$ 在 $\\triangle \\mathrm{ABC}$ 中, $\\angle \\mathrm{C}=90^{\\circ}, \\angle \\mathrm{A}=45^{\\circ}$,\n\n$\\therefore \\triangle \\mathrm{ABC}$ 是等腰直角三角形, $\\quad \\therefore b=a=35$."} {"id": "2977", "image": [], "answer": "$mgeq-1$", "solution": "null", "level": "九年级", "question": "(本题3分)(2018$cdot$浙江邵外九年级阶段练习)已知抛物线$y=-2(x+m)^{2}-3$,当$xgeq1$时,$y$随$x$的增大而减小,那么$m$的取值范围是", "options": [], "subject": "解析几何", "analysis": "【分析】可先求得抛物线的对称轴,再由条件可求得关于${m}$的不等式,可求得答案.【详解】解:$becausey=-2(x+m)^{2}-3$,$therefore$对称轴为${x}=-{m}$,$because{a}=-2<0$$therefore$抛物线开口向下,$therefore$在对称轴右侧${y}$随${x}$的增大而增大,$because$当$xgeq1$时,$y$随$x$的增大而减小,$therefore-mleq1$,解得$mgeq-1$,故答案为$mgeq-1$.【点睛】本题主要考查二次函数的性质,由函数的增减性得到关于${m}$的不等式是解题的关键."} {"id": "2978", "image": [], "answer": "${y}_{1}={y}_{2}>{y}_{3}$", "solution": "null", "level": "九年级", "question": "(本题3分)(2020浙江$cdot$台州市双语学校九年级期中)点${P}_{1}left(-1,{y}_{1}right),{P}_{2}left(3,{y}_{2}right)$,${P}_{3}left(5,{y}_{3}right)$均在二次函数${y}=-{x}^{2}+2{x}+{c}$的图象上,则${y}_{1},{y}_{2},{y}_{3}$的大小关系是", "options": [], "subject": "解析几何", "analysis": "【分析】根据函数解析式的特点,其对称轴为${x}=1$,图象开口向下,在对称轴的右侧,${y}$随${x}$的增大而减小,据二次函数图象的对称性可知,${P}_{1}left(-1,{y}_{1}right)$与$left(3,{y}_{1}right)$关于对称轴对称,可判断${y}_{1}={y}_{2}>{y}_{3}$.【详解】解:$becausey=-x^{2}+2x+c$,$therefore$对称轴为${x}=1$,$P_{2}left(3,y_{2}right),quadP_{3}left(5,y_{3}right)$在对称轴的右侧,$y$随$x$的增大而减小,$because3<5$$therefore{y}_{2}>{y}_{3}$,根据二次函数图象的对称性可知,${P}_{1}left(-1,{y}_{1}right)$与$left(3,{y}_{1}right)$关于对称轴对称,故${y}_{1}={y}_{2}>{y}_{3}$,故答案为$y_{1}=y_{2}>y_{3}$.【点睛】本题考查了函数图象上的点的坐标与函数解析式的关系,同时考查了函数的对称性及增减性."} {"id": "2979", "image": ["3885.jpg"], "answer": "4", "solution": "null", "level": "九年级", "question": "(本题3分)(2020:浙江$cdot$湖州市吴兴区城南实验学校九年级阶段练习)小明在某次投篮中,球的运动路线是抛物线$y=-frac{1}{5}x^{2}+3.5$的一部分(如图所示),若命中篮圈中心,则他与篮底的距离1是${m}$.", "options": [], "subject": "解析几何", "analysis": "【分析】根据题意可以求得当$y=3.05$时,抛物线$y=-frac{1}{5}x^{2}+3.5$中对应的$x$的值,从而可以解答本题.【详解】将$y=3.05$代入$y=-frac{1}{5}x^{2}+3.5$,得$3.05=-frac{1}{5}x^{2}+3.5$,解得,$x=-1.5$(舍去)或$x=1.5$,$therefore$若命中篮圈中心,则他与篮底的距离1是:$2.5+1.5=4({~m})$,故答案为:4【点睛】本题考查二次函数的应用."} {"id": "2980", "image": [], "answer": "9", "solution": "null", "level": "九年级", "question": "(本题3分)(2020浙江$cdot$九年级期中)若抛物线$y=x^{2}$过点$A(a,b),B(a+6,b)$,则$b=$", "options": [], "subject": "解析几何", "analysis": "【分析】由题意易得点$A、B$关于二次函数的对称轴对称,进而可得$frac{2a+6}{2}=0$,然后求解$a$的值,最后代入二次函数解析式求解$b$的值即可.【详解】解:由抛物线$y=x^{2}$过点$A(a,b),B(a+6,b)$,可得:该二次函数的对称轴为直线$x=0$,点$A、B$关于二次函数的对称轴对称,$thereforefrac{2a+6}{2}=0$,解得:$a=-3$,把$a=-3$代入抛物线解析式得:$y=(-3)^{2}=9$,$thereforeb=9$故答案为9.【点睛】本题主要考查二次函数的性质,熟练掌握二次函数的性质是解题的关键."} {"id": "2982", "image": ["3886.jpg"], "answer": "3", "solution": "null", "level": "九年级", "question": "(本题3分)(2019$cdot$浙江嘉兴$cdot$九年级阶段练习)如图,在平面直角坐标系中,抛物线$y=x^{2}+mx$交$x$轴的负半轴于点$A$.点$B$是$y$轴正半轴上一点,点$A$关于点$B$的对称点$A^{prime}$恰好落在抛物线上.过点$A^{prime}$作$x$轴的平行线交抛物线于另一点$C$.若点$A^{prime}$的横坐标为1,则$A^{prime}C$的长为", "options": [], "subject": "解析几何", "analysis": "【分析】解方程$x^{2}+mx=0$得$A(-m,0)$,再利用对称的性质得到点$A$的坐标为$(-1,0)$,所以抛物线解析式为$y=x^{2}+x$,再计算自变量为1的函数值得到$A^{prime}(1,2)$,接着利用$C$点的纵坐标为2求出$C$点的横坐标,然后计算$A^{prime}C$的长.【详解】解:当$y=0$时,$x^{2}+mx=0$,解得$x_{1}=0,x_{2}=-m$,则$A(-m,0)$,$because$点$A$关于点$B$的对称点为$A^{prime}$,点$A^{prime}$的横坐标为1,$therefore$点$A$的坐标为$(-1,0)$,$therefore$抛物线解析式为$y=x^{2}+x$,当$x=1$时,$y=x^{2}+x=2$,则$A^{prime}(1,2)$,当$y=2$时,$x^{2}+x=2$,解得$x_{1}=-2,x_{2}=1$,则$C(-2,1)$,$thereforeA^{prime}C$的长为$1-(-2)=3$,故答案为3。【点睛】本题考查了二次函数图象上点的坐标特征、坐标平面内关于某点对称的两点间坐标的关系以及抛物线与$x$轴的交点,解题的关键是把求二次函数$y=ax^{2}+bx+c(a,b,c$是常数,$aneq0)$与$x$轴的交点坐标问题转化为解关于$x$的一元二次方程."} {"id": "3000", "image": [], "answer": "$(3,1)$", "solution": "null", "level": "九年级", "question": "(本题3分)(2022$cdot$浙江疅州$cdot$九年级期末)二次函数$y=2(x-3)^{2}+1$图象的顶点坐标是$qquad$", "options": [], "subject": "解析几何", "analysis": "【分析】根据顶点式直接解答即可.【详解】解:二次函数$y=2(x-3)^{2}+1$的图象的顶点坐标是$(3,1)$.故答案为:$(3,1)$【点睛】本题考查了二次函数的性质,要熟悉顶点式的意义,并明确:$y=a(x-h)^{2}+k$$(aneq0)$的顶点坐标为$(h,k)$."} {"id": "3001", "image": [], "answer": "$(5,0)$", "solution": "null", "level": "九年级", "question": "(本题3分)(2021$cdot$浙江绍兴$cdot$年级期末)已知抛物线$y=ax^{2}+bx+c(aneq0)$的对称轴为直线$x=2$,且与$x$轴有两个交点,其中一个交点坐标为$-1,0$,则另一个交点坐标为$qquad$.", "options": [], "subject": "解析几何", "analysis": "【分析】根据二次函数的基本性质及对称轴求交点的对称点即可得.【详解】解:设另一个交点为$(a,0)$,根据对称轴及与$x$轴的交点可得:$frac{a+(-1)}{2}=2,$解得:$a=5$故答案为:$(5,0)$.【点睛】题目主要考查利用二次函数的基本性质求交点的对称点,理解题意,熟练掌握二次函数的基本性质是解题关键."} {"id": "3004", "image": [], "answer": "$-frac{3}{4}-0.75$", "solution": "null", "level": "九年级", "question": "(本题3分)(2022$cdot$浙江嘉兴$cdot$九年级期末)甲、乙两人研究二次函数$y=ax^{2}-4ax+3(aneq0)$与反比例函数$y=frac{k}{x}(kneq0)$,甲说:“二次函数图象一定过第一象限的一个定点.\"乙说:“二次函数图象的顶点及这个定点都在该反比例函数图象上.\"若甲、乙两人的描述正确,则$a$的值为", "options": [], "subject": "解析几何", "analysis": "【分析】根据二次函数过定点,则与$a$的取值无关,得出定点和顶点再进行解答.【详解】解:$becausey=ax^{2}-4ax+3=ax(x-4)+3$,$therefore$当$x=4$时,$y=3$,$therefore$二次函数图象一定过第一象限的一个定点$(4,3)$,$becausey=ax^{2}-4ax+3=a(x-2)^{2}+3-4a$,$therefore$顶点为$(2,3-4a)$,$because$二次函数的顶点及这个定点都在反比例函数图象上,$therefore2times(3-4a)=4times3$,$thereforea=-frac{3}{4}$.故答案为:$-frac{3}{4}$.【点睛】本题考查了二次函数和反比例函数的图象和性质,确定出二次函数过定点$(4,3)$是解答本题的关键."} {"id": "3005", "image": [], "answer": "$-frac{4}{5}-0.8$", "solution": "null", "level": "九年级", "question": "(本题3分)(2022$cdot$浙江温州$cdot$九年级期末)已知二次函数$y=ax^{2}+2ax-3a$(其中$x$是自变量)图象与$x$轴交于$A,B$两点,当$xgeq0$时,$y$随$x$的增大而减小,$P$为抛物线上一点,且横坐标为$m$,当$-2leqmleq2$时,$triangleABP$面积的最大值为8,则$a$的值为", "options": [], "subject": "解析几何", "analysis": "【分析】根据函数解析式可以求得与$x$轴的两个交点,然后根据当$xgeqslant0$时,$y$随$x$的增大而减小,$P$为抛物线上一点,且横坐标为$m$,当$-2leqslantmleqslant2$时,$triangleABP$面积的最大值为8,即可求得${a}$的值【详解】$becausey=ax^{2}+2ax-3a=a(x+3)(x-1)$,$therefore$当$y=0$时,$x=-3$或1,不妨设点${A}$的坐标为$(-3,0)$,点${B}(1,0)$,$therefore{AB}=1-(-3)=1+3=4$$therefore$该抛物线顶点的横坐标为$frac{-3+1}{2}=-1$,纵坐标为$y=a-2a-3a=-4a$,$because$当$xgeqslant0$时,$y$随$x$的增大而减小,$therefore{a}<0$,$becauseP$为抛物线上一点,且横坐标为$m$,当$-2leqslantmleqslant2$时,$triangleABP$面积的最大值为8,$therefore$当$x=2$时,$y=4a+4a-3a=5a$,当$x=-1$时,$y=-4a$,$because|5{a}|>|-4{a}|$$thereforefrac{ABcdot|5a|}{2}=8$,即$frac{4cdot(-5a)}{2}=8,$解得${a}=-frac{4}{5},$故答案为:$-frac{4}{5}$.【点睛】本题考查抛物线与${x}$轴的交点、二次函数的性质、二次函数的最值,解答本题的关键是判断${a}$的正负情况,求出${a}$的值."} {"id": "3006", "image": [], "answer": "(1)(3)(4)", "solution": "null", "level": "九年级", "question": "(本题3分)(2021$cdot$浙江$cdot$杭州外国语学校九年级期中)抛物线$y=ax^{2}+bx+c(a$,$b,c$是常数,$a<0)$经过$A(0,3),B(4,3)$.下列四个结论:(1)$4a+b=0$;(2)点$P_{1}left(x_{1},y_{1}right),P_{2}left(x_{2},y_{2}right)$在抛物线上,当$left|x_{1}-2right|-left|x_{2}-2right|>0$时,$y_{1}>y_{2}$;(3)若抛物线与$x$轴交于不同两点$C,D$,且$CDleq6$,则$aleq-frac{3}{5}$;(4)若$3leqxleq4$,对应的$y$的整数值有3个,则$-10$,即$P_{1}left(x_{1},y_{1}right)$离对称轴更远,$thereforey_{1}", "options": [], "subject": "解析几何", "analysis": "【分析】连接$OD$,交$AC$于$F$,根据垂径定理得出$ODperpAC,AF=CF$,进而证得$DF$$=BC$,根据三角形中位线定理求得$OF=frac{1}{2}BC=frac{1}{2}DF$,从而求得$BC=DF=2sqrt{2}$,利用勾股定理即可求得$AC$.【详解】解:连接$OD,$交$AC$于$F$,$becauseD$是$AC$的中点,$thereforeODperpAC,AF=CF$,$thereforeangleDFE=90^{circ}$,$becauseOA=OB,AF=CF$$thereforeOF=frac{1}{2}BC$,$becauseAB$是直径,$thereforeangleACB=90^{circ}$,在$triangleEFD$和$triangleECB$中,$left{begin{array}{l}angleDBE=angleBCE=90^{circ}angleDEF=angleBECDE=BEend{array}right.$,$thereforetriangleEFDcongtriangleECB(AAS)$,$thereforeDF=BC$,$thereforeOF=frac{1}{2}DF$,$becauseOD=3sqrt{2}$,$thereforeOF=sqrt{2}$,$thereforeBC=2sqrt{2}$,在Rt$triangleABC$中,$AC^{2}=AB^{2}-BC^{2}$,$thereforeAC=sqrt{AB^{2}-BC^{2}}=sqrt{(6sqrt{2})^{2}-(2sqrt{2})^{2}}=8$,故答案为8.【点睛】本题考查垂径定理、圆周角定理及推论、全等三角形的判定、勾股定理、灵活应用性质及定理是关键,熟练掌握垂径定理是重点."} {"id": "3119", "image": ["4049.jpg", "4050.jpg"], "answer": "9", "solution": "null", "level": "九年级", "question": "(本题3分)(2022-浙江$cdot$九年级专题练习)如图,$A$为双曲线$y=-frac{9}{x}$在第二象限分支上的一个动点,连接$AO$并延长交双曲线的另一分支于点$B$,以$AB$为边作等边三角形$ABC$,若点$C$的坐标为$(3,n)$,则$n=$", "options": [], "subject": "解析几何", "analysis": "【分析】连接$OC$,作$AMperpx$轴于$M,CNperpx$轴于$N$,根据题意得出$angleCAO=60^{circ}$,$COperpAB$,从而得出$frac{OC}{OA}=sqrt{3}$,通过证得$triangleAOMsimtriangleOCN$,得出$frac{S_{triangleCON}}{S_{triangleAOM}}=left(frac{OC}{OA}right)^{2}=3$,即可得出$frac{frac{3n}{2}}{frac{9}{2}}=3$,解方程即可求解.【详解】解:连接$OC$,作$AMperpx$轴于$M,CNperpx$轴于$N$,$becauseA、B$为双曲线$y=-frac{9}{x}$上的点,且$AB$经过原点$O$,$thereforeOA=OB$,$becausetriangleABC$是等边三角形,$thereforeangleCAO=60^{circ},quadCOperpAB$$thereforefrac{{OC}}{{OA}}=sqrt{3}$,$thereforeangleAOM+angleCON=90^{circ}$,$becauseangleAOM+angleOAM=90^{circ}$,$thereforeangleCON=angleOAM$$becauseangleAMO=angleCNO=90^{circ}$,$thereforetriangleAOMsimtriangleOCN$$thereforefrac{S_{triangleCON}}{S_{triangleAOM}}=left(frac{OC}{OA}right)^{2}=3$$becauseA$为双曲线$y=-frac{9}{x}$在第二象限分支上的一个动点,点$C$的坐标为$(3,n)$,$thereforeS_{triangleAOM}=frac{1}{2}times|-9|=frac{9}{2},quadS_{triangleCON}=frac{1}{2}timesONcdotCN=frac{3n}{2}$,$thereforefrac{S_{triangleCON}}{S_{triangleAOM}}=frac{frac{3n}{2}}{frac{9}{2}}=3$,$thereforen=9$.故答案为:9.【点睛】本题考查了反比例函数图象上点的坐标特征,反比例函数的比例系数的几何意义,等边三角形的性质,解直角三角形,相似三角形的性质和判定的应用.能综合运用相关知识进行推理和计算是解答关键."} {"id": "3142", "image": ["4095.jpg", "4096.jpg"], "answer": "1", "solution": "null", "level": "九年级", "question": "(本题3分)(2019$cdot$浙江湖州$cdot$九年级期中)如图,在平面直角坐标系中,$triangle{ABC}$的顶点在坐标轴上,${A},{B},{C}$三点的坐标分别为$(0,2),(1,0),(0,-0,5),{D}$为线段$AB$上-个动点(不与点$A,B$重合),过$B,D,0$三点的圆与直线$BC$交于点${E}$,当$triangleOED$面积取得最小值时,ED的长为$qquad$.", "options": [], "subject": "解析几何", "analysis": "【分析】如图,先证明$triangle{AOB}simtriangle{BOC}$得到$angle1=angle2$,再判断$angle{DBE}=90^{circ}$,利用圆周角定理可得到$DE$为过$B,D,O$三点的圆的直径,从而得到$angleDOE=90^{circ}$,接着证明$triangleAODsimtriangleBOE$,利用相似比得到$OD=2OE$,根据三角形面积公式得到$Striangle_{{ODE}}={OE}^{2}$,利用垂线段最短判断当$triangle{OED}$面积取得最小值时,${OE}perp{CB}$,然后计算${OE}、{OD}$,最后利用勾股定理计算对应的DE长.【详解】如图,$because{A},{B},{C}$三点的坐标分别为$(0,2),(1,0),(0,-0.5)$,$therefore{OA}=2,{OB}=1,{OC}=frac{1}{2}$,$becausefrac{OA}{OB}=frac{OB}{OC}=2$,而$angle{AOB}=angle{BOC}$,$thereforetriangle{AOB}simtriangle{BOC}$,$thereforeangle1=angle2$,$thereforeangle{ABC}=angle2+angle5=angle1+angle5=90^{circ}$,$becauseangle{DBE}=90^{circ}$,$therefore{DE}$为过${B},{D},{O}$三点的圆的直径,$thereforeangle{DOE}=90^{circ}$,$becauseangle3+angle{BOD}=angle4+angle{BOD}=90^{circ}$,$thereforeangle3=angle4$,$becauseangle1=angle2$$thereforetriangle{AOD}simtriangle{BOE}$,$thereforefrac{OD}{OE}=frac{OA}{OB}=frac{2}{1}$,即${OD}=2{OE}$,$because{S}triangle{ODE}=frac{1}{2}{OD}cdot{OE}=frac{1}{2}cdot2{OE}cdot{OE}={OE}^{2}$,当$triangle{OED}$面积取得最小值时,${OE}$最小,此时${OE}perp{CB}$,$because{BC}=sqrt{1^{2}+left(frac{1}{2}right)^{2}}=frac{sqrt{5}}{2}$,$therefore{OE}=frac{OCcdotOB}{BC}=frac{1timesfrac{1}{2}}{frac{sqrt{5}}{2}}=frac{sqrt{5}}{5}$,此时${OD}=2{OE}=frac{2sqrt{5}}{5}$,$therefore{DE}=sqrt{left(frac{sqrt{5}}{5}right)^{2}+left(frac{2sqrt{5}}{5}right)^{2}}=1$,即当$triangle{OED}$面积取得最小值时,ED的长为1.故答案为1.【点睛】本题考查了三角形的外接圆与外心:三角形外接圆的圆心是三角形三条边垂直平分线的交点,叫做三角形的外心.也考查了圆周角定理和菱形的判定与性质."} {"id": "3185", "image": [], "answer": "$25-2-2+25$", "solution": "null", "level": "九年级", "question": "(本题3分)(2022$cdot$浙江嘉兴$cdot$九年级期末)已知点$P$是线段$AB$的黄金分割点($AP$$>BP),AB=4$,那么$AP=$$qquad$", "options": [], "subject": "解析几何", "analysis": "【分析】根据黄金分割点的定义,知$AP$是较长线段;则$AP=frac{sqrt{5}-1}{2}AB$,代入数据即可得出$AP$的长.【详解】解:由于$P$为线段$AB$的黄金分割点,且$AP$是较长线段,$AB=4$,则$AP=frac{sqrt{5}-1}{2}AB=frac{sqrt{5}-1}{2}times4=2sqrt{5}-2$.故答案为$2sqrt{5}-2$.【点睛】本题考查了黄金分割的概念.解题关键是熟记黄金分割的公式:较短的线段$=$原线段的$frac{3-sqrt{5}}{2}$,较长的线段$=$原线段的$frac{sqrt{5}-1}{2}$."} {"id": "3188", "image": ["4160.jpg", "4161.jpg"], "answer": "$frac{sqrt{2}}{2}$", "solution": "null", "level": "九年级", "question": "(本题3分)(2019$cdot$浙江$cdot$龙泉市顺风实验学校九年级期中)如图,已知矩形$ABCD,AB:BC=1:2,P$为线段$AB$上的一点,以$BP$为边作矩形$EFBP$,使点$F$在线段$CB$的延长线上,矩形$ABCDbacksim$矩形$EFBP$,设$EF=a,AB=b$,当$EP$平分$angleAEC$时,则$frac{a}{b}=$$qquad$.", "options": [], "subject": "解析几何", "analysis": "【分析】设${AB}$与${CE}$交于点${G}$,根据等腰三角形三线合一的性质可表示出${PG}$,进而得到${BG}$,然后根据平行线分线段成比例定理列出比例式,变形求解即可.【详解】解:如图,${AB}$与${CE}$交于点${G}$,$because{EP}$平分$angle{AEC},{EP}perp{AG}$,$thereforeangle{AEP}=angle{GEP},angle{APE}=angle{GPE}=90^{circ}$$thereforeangle{EAP}=angle{EGP}$$therefore{EA}={EG}$$thereforeAP=PG=b-a,BG=a-(b-a)=2a-b$$because{PE}//{CF}$,$thereforefrac{PE}{BC}=frac{PG}{BG}=frac{b-a}{2a-b}$,$because$矩形${ABCD}sim$矩形EFBP,$thereforefrac{PE}{BC}=frac{EF}{AB}=frac{a}{b}$,$thereforefrac{a}{b}=frac{b-a}{2a-b}$,整理得:$b^{2}=2a^{2}$,$because{a}>0,{~b}>0$$thereforefrac{a}{b}=frac{sqrt{2}}{2}$,故答案是:$frac{sqrt{2}}{2}$.【点睛】本题考查了矩形的性质、等腰三角形的判定和性质、相似图形的性质以及平行线分线段成比例定理,根据三线合一表示出PG、BG的长是解答此题的关键."} {"id": "3190", "image": ["4164.jpg", "4165.jpg"], "answer": "$frac{1}{2}quadleft(frac{3sqrt{3}}{2},0right)$", "solution": "null", "level": "九年级", "question": "(本题3分)(2022-浙江宁波$cdot$年级专题练习)如图,四边形$OABC$为矩形,点$A$在第二象限,点$A$关于$OB$的对称点为点$D$,点$B,D$都在函数$y=frac{6sqrt{2}}{x}(x>0)$的图象上,$BEperpx$轴于点$E$.若$DC$的延长线交$x$轴于点$F$,当矩形$OABC$的面积为$9sqrt{2}$时,,点$F$的坐标为", "options": [], "subject": "解析几何", "analysis": "【分析】连接$OD$,作$DGperpx$轴,设点$Bleft(b,frac{6sqrt{2}}{b}right),Dleft(a,frac{6sqrt{2}}{a}right)$,根据矩形的面积得出三角形$BOD$的面积,将三角形$BOD$的面积转化为梯形$BEGD$的面积,从而得出$a,b$的等式,将其分解因式,从而得出$a,b$的关系,进而在直角三角形$BOD$中,根据勾股定理列出方程,进而求得$B,D$的坐标,进一步可求得结果.【详解】解:如图,作$DGperpx$轴于$G$,连接$OD$,设$BC$和$OD$交于$I$,设点$Bleft(b,frac{6sqrt{2}}{b}right),Dleft(a,frac{6sqrt{2}}{a}right)$,由对称性可得:$triangleBODcongtriangleBOAcongtriangleOBC$,$thereforeangleOBC=angleBOD,BC=OD$,$thereforeOI=BI$$thereforeDI=CI$$thereforefrac{DI}{OI}=frac{CI}{BI}$,$becauseangleCID=angleBIO$,$thereforetriangleCDIsimtriangleBOI$,$thereforeangleCDI=angleBOI$,$thereforeCD//OB$,$thereforeStriangleBOD=StriangleAOB=frac{1}{2}S_{text{矩形}}AOCB=frac{9sqrt{2}}{2}$,$becauseStriangleBOE=StriangleDOG=frac{1}{2}|k|=3sqrt{2},S_{text{四边形}}BOGD=StriangleBOD+StriangleDOG=S_{text{椾形}}$$BEGD+StriangleBOE$,$thereforeS_{text{梯形}}BEGD=StriangleBOD=frac{9sqrt{2}}{2}$,$thereforefrac{1}{2}left(frac{6sqrt{2}}{a}+frac{6sqrt{2}}{b}right)cdot(a-b)=frac{9sqrt{2}}{2}$,$therefore2a^{2}-3ab-2b^{2}=0$,$therefore(a-2b)cdot(2a+b)=0$,$thereforea=2b,a=-frac{b}{2}$(舍去),$thereforeDleft(2b,frac{6sqrt{2}}{2b}right)$,即:$left(2b,frac{3sqrt{2}}{b}right)$,在$RttriangleBOD$中,由勾股定理得,$OD^{2}+BD^{2}=OB^{2}$,$thereforeleft[(2b)^{2}+left(frac{3sqrt{2}}{b}right)^{2}right]+left[(2b-b)^{2}+left(frac{6sqrt{2}}{b}-frac{3sqrt{2}}{b}right)^{2}right]=b^{2}+left(frac{6sqrt{2}}{b}right)^{2}$,$thereforeb=sqrt{3}$,$thereforeB(sqrt{3},2sqrt{6}),D(2sqrt{3},sqrt{6})$,$because$直线$OB$的解析式为:$y=2sqrt{2}x$,$therefore$直线$DF$的解析式为:$y=2sqrt{2}x-3sqrt{6}$,当$y=0$时,$2sqrt{2}x-3sqrt{6}=0$,$thereforex=frac{3sqrt{3}}{2}$,$thereforeFleft(frac{3sqrt{3}}{2},0right)$,$becauseOE=sqrt{3},OF=frac{3sqrt{3}}{2}$,$thereforeEF=OF-OE=frac{sqrt{3}}{2}$,$thereforefrac{EF}{OE}=frac{1}{2}$,故答案为:$frac{1}{2},left(frac{3sqrt{3}}{2},0right)$.【点睛】本题考查了矩形性质,轴对称性质,反比例函数的“$k$”的几何含义,勾股定理,一次函数及其图象性质,分解因式等知识,解决问题的关键是变形等式,进行分解因式."} {"id": "3352", "image": [], "answer": "$\\frac{\\sqrt{5}}{3}$", "solution": "null", "level": "九年级", "question": "在Rt $\\triangle A B C$ 中, $\\angle C=90^{\\circ}, A B=3, B C=2$, 则 $\\cos A$ 的值是", "options": [], "subject": "解析几何", "analysis": "$\\because \\angle C=90^{\\circ}, A B=3, B C=2$,\n\n$\\therefore A C=\\sqrt{3^{2}-2^{2}}=\\sqrt{5}$,\n\n$\\therefore \\cos A=\\frac{A C}{A B}=\\frac{\\sqrt{5}}{3}$."} {"id": "3380", "image": ["4444.jpg"], "answer": "192", "solution": "null", "level": "九年级", "question": "如图, 在矩形 $A B C D$ 中, $E, F, G, H$ 分别为 $A B, B C, C D, D A$ 的中点, 若 $A H: A E=4: 3$,四边形 $E F G H$ 的周长是 $40 \\mathrm{~cm}$, 则矩形 $A B C D$ 的面积是 $\\mathrm{cm}^{2}$.\n\n", "options": [], "subject": "解析几何", "analysis": "解: 在 $\\triangle A H E$ 和 $\\triangle D H G$ 中,\n\n$\\because A H=D H=\\frac{1}{2} A D, \\angle A=\\angle D=90^{\\circ}, A E=D G=\\frac{1}{2} A B$,\n\n$\\therefore \\triangle A H E \\cong \\triangle D H G$,\n\n$\\therefore E H=G H$,\n\n同理 $E H=G H=G F=E F$,\n\n即四边形 $E F G H$ 为菱形.\n\n又 $\\because$ 四边形 $E F G H$ 的周长是 $40 \\mathrm{~cm}$,\n\n$\\therefore E H=10$.\n\n$\\because A H: A E=4: 3$,\n设 $A H=4 x$ ,则 $A E=3 x$.\n\n由勾股定理得, $E H^{2}=A E^{2}+A H^{2}$,\n\n$\\therefore x=2, A H=8, A E=6$,\n\n$\\therefore$ 矩形 $A B C D$ 的面积 $=16 \\times 12=192\\left(\\mathrm{~cm}^{2}\\right)$.\n\n由题意知, $\\triangle A E H, \\triangle D H G, \\triangle C G F, \\triangle E F B$ 是全等三角形, 所以 $E H=H G=F G=E F$, 即四边形 $E F G H$ 为菱形, 四边形 $E F G H$ 的周长是 $40 \\mathrm{~cm}$, 可知边长为 10 , 根据勾股定理可求得 $A H$ 和 $A E$, 即 $A D$ 和 $A B$ 的值就可求出, 从而求矩形面积.\n\n本题考查了矩形、菱形的性质及勾股定理, 有一定难度."} {"id": "3430", "image": ["4540.jpg", "4541.jpg", "4542.jpg", "4543.jpg"], "answer": "2秒或6秒或10秒", "solution": "null", "level": "九年级", "question": "如图, 在平面直角坐标系中, 直线 $y=x-4$ 与 $x$ 轴、 $y$ 轴分别交于点 $B 、 C$, 半径为 2 的 $\\odot P$的圆心 $P$ 从点 $A(8, m)$ (点 $A$ 在直线 $y=x-4$ 上) 出发以每秒 $\\sqrt{2}$ 个单位长度的速度沿射线 $A C$ 运动,设点 $P$ 运动的时间为 $t$ 秒,则当 $t=$ \\$ \\qquad \\$时, $\\odot P$ 与坐标轴相切.\n\n", "options": [], "subject": "解析几何", "analysis": "【分析】\n\n本题考查了切线的判定,等腰直角三角形的判定和性质,分类讨论是解题的关键.\n\n设 $\\odot P$ 与坐标轴的切点为 $D$, 根据已知条件得到 $A(8,4), B(4,0), C(0,-4)$, 求得 $A B=4 \\sqrt{2}, A C=$ $8 \\sqrt{2}, O B=O C=4$, 推出 $\\triangle O B C$ 是等腰直角三角形, $\\angle O B C=45^{\\circ}$, (1)当 $\\odot P$ 与 $x$ 轴相切时, (2)如图, $\\odot P$ 与 $x$ 轴和 $y$ 轴都相切时, (3)当点 $P$ 只与 $y$ 轴相切时, 根据等腰直角三角形的性质得到结论.\n\n【解答】\n\n解: 设 $\\odot P$ 与坐标轴的切点为 $D$,\n\n$\\because$ 直线 $y=x-4$ 与 $x$ 轴、 $y$ 轴分别交于点 $B 、 C$, 点 $A(8, m)$,\n\n$\\therefore x=0$ 时, $y=-4, y=0$ 时, $x=4, x=8$ 时, $y=4$,\n\n$\\therefore A(8,4), B(4,0), C(0,-4)$,\n\n$\\therefore A B=4 \\sqrt{2}, A C=8 \\sqrt{2}, O B=O C=4$,\n\n$\\therefore \\triangle O B C$ 是等腰直角三角形, $\\angle O B C=45^{\\circ}$,\n\n(1) 当 $\\odot P$ 与 $x$ 轴相切时,\n\n\n$\\because$ 点 $D$ 是切点, $\\odot P$ 的半径是 2 ,\n\n$\\therefore P D \\perp x$ 轴, $P D=2$,\n\n$\\therefore \\triangle B D P$ 是等腰直角三角形,\n\n$\\therefore B D=P D=2, P B=2 \\sqrt{2}$,\n\n$\\therefore A P=A B-P B=2 \\sqrt{2}$,\n\n$\\because$ 点 $P$ 的速度为每秒 $\\sqrt{2}$ 个单位长度,\n\n$\\therefore t=2$\n\n(2)如图, $\\odot P$ 与 $x$ 轴和 $y$ 轴都相切时,\n\n\n\n$\\because P B=2 \\sqrt{2}$,\n\n$\\therefore A P=A B+P B=6 \\sqrt{2}$,\n\n$\\because$ 点 $P$ 的速度为每秒 $\\sqrt{2}$ 个单位长度,\n\n$\\therefore t=6$\n\n(3)当点 $P$ 只与 $y$ 轴相切时,\n\n\n\n$\\because P B=2 \\sqrt{2}$,\n\n$\\therefore A P=A C+P B=10 \\sqrt{2}$,\n\n$\\because$ 点 $P$ 的速度为每秒 $\\sqrt{2}$ 个单位长度,\n\n$\\therefore t=10$.\n综上所述, 则当 $t=2$ 或6或 10 秒时, $\\odot P$ 与坐标轴相切,\n\n故答案为: 2 秒或6秒或 10 秒."} {"id": "3503", "image": ["4707.jpg", "4708.jpg"], "answer": "4", "solution": "null", "level": "九年级", "question": "如图, 已知 $\\angle A O B=30^{\\circ}, M$ 为 $O B$ 边上一点, 以 $M$ 为圆心, $2 \\mathrm{~cm}$ 为半径作 $\\odot M$. 若点 $M$ 在 $O B$边上运动, 则当 $O M=$ $c m$ 时, $\\odot M$ 与 $O A$ 相切.\n\n", "options": [], "subject": "解析几何", "analysis": "【分析】\n\n本题考查了切线的性质和判定,直角三角形的性质.根据直角三角形中, $30^{\\circ}$ 的角所对的直角边是斜边的一半解答. 连接 $M N, N$ 为切点, 根据 $M N \\perp O B$ 可知 $\\angle A O B=30^{\\circ}, 2 \\mathrm{~cm}$ 为半径, 利用直角\n三角形中 $30^{\\circ}$ 的角所对的直角边是斜边的一半解答.\n\n【解答】\n\n解:设 $\\odot M$ 与 $O A$ 相切时切点为 $N$, 连接 $M N$,\n\n\n\n$\\therefore M N \\perp O A, \\angle A O B=30^{\\circ}$,\n\n$\\because \\odot M$ 的半径为 $2 \\mathrm{~cm}$,\n\n$\\therefore O M=2 M N=2 \\times 2=4 \\mathrm{~cm}$.\n\n$\\therefore$ 当 $O M=4 \\mathrm{~cm}$ 时, $\\odot M$ 与 $O A$ 相切.\n\n故答案为 4 ."} {"id": "3553", "image": ["4812.jpg", "4813.jpg", "4814.jpg"], "answer": "$y=\\frac{5}{6} x^{2}+9 ; \\frac{81 \\sqrt{3}}{20}$", "solution": "null", "level": "九年级", "question": "以 $E F$ 所在直线为 $x$ 轴, $A B$ 所在直线为 $y$ 轴建立平面直角坐标系, 求抛物线的解析式_; 将高脚杯绕点 $F$ 缓缓倾斜倒出部分液体, 当 $\\angle E F H=30^{\\circ}$ 时停止,此时液面为 $G D$, 此时杯体内液体的最大深度为\n", "options": [], "subject": "解析几何", "analysis": "【分析】\n\n本题考查了二次函数在实际问题中的应用,数形结合并熟练掌握待定系数法、二次函数及解直角三角形等知识点是解题的关键.\n\n以 $A$ 为原点, 直线 $E F$ 为 $x$ 轴, 直线 $A B$ 为 $y$ 轴, 建立平面直角坐标系, 由待定系数法求得抛物线的解析式; 将高脚杯绕点 $F$ 倾斜后, 仍以 $A$ 为原点, 直线 $E F$ 为 $x$ 轴, 直线 $A B$ 为 $y$ 轴, 建立平面直角坐标系, 分别用待定系数法求得直线 $l$ 的解析式和直线 $G D$ 的解析式, 过点 $M$ 作 $M P \\perp l$ 于点 $P$, 用三角函数求得液面 $G D$ 到平面 $l$ 的距离; 过抛物线最低点 $Q$ 作 $Q L / / l$, 再将 $Q L$ 的解析式与抛物线的解析式联立, 得出关于 $x$ 的一元二次方程, 由判别式求得 $q$, 最后用三角函数求得答案.\n\n【解答】\n\n解: 以 $A$ 为原点, 直线 $E F$ 为 $x$ 轴, 直线 $A B$ 为 $y$ 轴, 建立平面直角坐标系, 如图:\n\n\n\n由题意得:\n\n$A(0,0), B(0,9), C(-2 \\sqrt{3}, 19), D(2 \\sqrt{3}, 19)$,\n\n设抛物线的解析式为: $y=a x^{2}+9$,\n\n将 $D(2 \\sqrt{3}, 19)$ 代入得:\n\n$19=a \\times(2 \\sqrt{3})^{2}+9$,\n\n解得: $a=\\frac{5}{6}$,\n\n$\\therefore y=\\frac{5}{6} x^{2}+9$;\n\n将高脚杯绕点 $F$ 倾斜后, 仍以 $A$ 为原点, 直线 $E F$ 为 $x$ 轴, 直线 $A B$ 为 $y$ 轴, 建立平面直角坐标系, 如图:\n\n\n\n由题意得: $A(0,0), F(\\sqrt{3}, 0), E(-\\sqrt{3}, 0), B(0,9), C(-2 \\sqrt{3}, 19), D(2 \\sqrt{3}, 19)$,\n\n由题可知, 直线 $l$ 与 $x$ 轴的夹角为 $30^{\\circ}, G D / / l$,\n\n$\\because l$ 经过点 $F(\\sqrt{3}, 0)$, 且 $\\angle E F H=30^{\\circ}$,\n\n$\\therefore$ 设直线 $l$ 的解析式为: $y=\\frac{\\sqrt{3}}{3} x+b$,\n\n将 $F(\\sqrt{3}, 0)$ 代入, 解得 $b=-1$,\n$\\therefore y=\\frac{\\sqrt{3}}{3} x-1$,\n\n又 $\\because G D / / l$,\n\n$\\therefore k_{G D}=k_{l}=\\frac{\\sqrt{3}}{3}$,\n\n$\\therefore$ 设直线 $G D$ 的解析式为 $y=\\frac{\\sqrt{3}}{3} x+p$,\n\n将 $D(2 \\sqrt{3}, 19)$ 代入,解得 $p=17$ ,\n\n$\\therefore y=\\frac{\\sqrt{3}}{3} x+17$,\n\n$\\therefore M(0,17), N(0,-1)$,\n\n过点 $M$ 作 $M P \\perp l$ 于点 $P$,\n\n$\\because \\angle E F H=30^{\\circ}, \\angle F A N=90^{\\circ}$,\n\n$\\therefore \\angle A N F=60^{\\circ}$,\n\n$\\therefore M P=M N \\cdot \\sin 60^{\\circ}=[17-(-1)] \\times \\frac{\\sqrt{3}}{2}=9 \\sqrt{3}$,\n\n过抛物线最低点 $Q$ 作 $Q L / / l, L$ 为 $Q L$ 于 $M P$ 的交点,\n\n设直线 $Q L$ 的解析式为,\n\n联立 $\\left\\{\\begin{array}{l}y=\\frac{5}{6} x^{2}+9 \\\\ y=\\frac{\\sqrt{3}}{3} x+q\\end{array}\\right.$ ,\n\n得: $\\frac{5}{6} x^{2}-\\frac{\\sqrt{3}}{3} x+9-q=0$,\n\n$\\because$ 只有一个交点 $Q$,\n\n$\\therefore \\Delta=0$,\n\n$\\therefore \\frac{1}{3}-4 \\times \\frac{5}{6}(9-q)=0$,\n\n$\\therefore q=\\frac{89}{10}$,\n\n$\\therefore M L=\\left(17-\\frac{89}{10}\\right) \\times \\sin 60^{\\circ}=\\frac{81 \\sqrt{3}}{20}$."} {"id": "3554", "image": ["4815.jpg", "4816.jpg", "4817.jpg"], "answer": "$\\left(-\\frac{7}{3}, 0\\right)$ 或 $\\left(-\\frac{17}{3}, 0\\right)$", "solution": "null", "level": "九年级", "question": "如图, 直线 $y=-\\frac{3}{4} x-3$ 交 $x$ 轴于点 $A$, 交 $y$ 轴于点 $B$, 点 $P$ 是 $x$ 轴上一动点, 以点 $P$ 为圆心,以 1 个单位长度为半径作 $\\odot P$, 当 $\\odot P$ 与直线 $A B$ 相切时, 点 $P$ 的坐标是\n\n", "options": [], "subject": "解析几何", "analysis": "【分析】\n\n本题考查了一次函数的图象与性质, 切线的性质, 三角形的面积, 先求出 $A 、 B$ 的坐标, 得到 $O A$ 、 $O B$ 的值, 进而得到 $A B=5$, 分两种情况讨论 (1)当点 $P$ 在点 $A$ 右侧时, (2)当点 $P$ 在点 $A$ 左侧时, 每种\n情况设 $O P=x$, 表示出 $A P$, 利用面积法即可求出 $x$ 的值, 进而得到答案.\n\n【解答】\n\n解: $\\because$ 直线 $y=-\\frac{3}{4} x-3$ 交 $x$ 轴于点 $A$, 交 $y$ 轴于点 $B$,\n\n$\\therefore$ 令 $x=0$, 得 $y=-3$, 令 $y=0$, 得 $x=-4$,\n\n$\\therefore A(-4,0), B(0,-3)$,\n\n$\\therefore O A=4, O B=3$\n\n$\\therefore A B=5$,\n\n(1) 如图, 当点 $P$ 在点 $A$ 右侧时,\n\n\n\n设 $\\odot P$ 与直线 $A B$ 相切于 $C$, 连接 $P C, P B$,\n\n则 $P C \\perp A B, P C=1$,\n\n设 $O P=x$, 则 $A P=O A-O P=4-x$,\n\n$\\therefore S_{\\triangle A P B}=\\frac{1}{2} A P \\cdot O B=\\frac{1}{2} A B \\cdot P C$\n\n即 $\\frac{1}{2} \\times 3(4-x)=\\frac{1}{2} \\times 5 \\times 1$ ,\n\n解得: $x=\\frac{7}{3}, \\therefore P\\left(-\\frac{7}{3}, 0\\right)$;\n\n(2)如图, 当点 $P$ 在点 $A$ 左侧时,\n\n\n\n设 $\\odot P$ 与直线 $A B$ 相切于 $D$, 连接 $P D, P B$,\n\n则 $P D \\perp A B, P D=1$,\n\n设 $O P=x$, 则 $A P=O P-A O=x-4$,\n\n$\\therefore S_{\\triangle A P B}=\\frac{1}{2} A P \\cdot O B=\\frac{1}{2} A B \\cdot P D$,\n\n即 $\\frac{1}{2} \\times 3(x-4)=\\frac{1}{2} \\times 5 \\times 1$,\n\n解得: $x=\\frac{17}{3}, \\therefore P\\left(-\\frac{17}{3}, 0\\right)$;\n\n综上所述, 点 $P$ 的坐标为 $\\left(-\\frac{7}{3}, 0\\right)$ 或 $\\left(-\\frac{17}{3}, 0\\right)$,\n\n故答案为 $\\left(-\\frac{7}{3}, 0\\right)$ 或 $\\left(-\\frac{17}{3}, 0\\right)$."} {"id": "2", "image": [], "answer": " ( $0,-3$ )\n【详解】抛物线 $\\mathrm{y}=-\\frac{1}{3} \\mathrm{x}^{2}-3$ 的顶点坐标是 $(0,-3)$, 故答案为 $(0,-3)$.", "solution": "null", "level": "九年级", "question": "拋物线 $\\mathrm{y}=-\\frac{1}{3} \\mathrm{x}^{2}-3$ 的顶点坐标是 $\\qquad$ .\n", "options": [], "subject": "解析几何", "analysis": "$0,-3$ )\n【详解】抛物线 $\\mathrm{y}=-\\frac{1}{3} \\mathrm{x}^{2}-3$ 的顶点坐标是 $(0,-3)$, 故答案为 $(0,-3)$.\n"} {"id": "3", "image": [], "answer": " 2 (答案不唯一).\n\n", "solution": "null", "level": "九年级", "question": "如果拋物线 $y=(m-1) x^{2}$ 的开口向上, 那么 $m$ 的值可以是 $\\qquad$ (写出一个满足条件的数即可).\n", "options": [], "subject": "解析几何", "analysis": "由二次函数图像的开口向上, 得到 $m-1>0$, 求出 $m$ 的取值范围, 即可得到答案.\n\n【详解】解: $\\because$ 抛物线 $y=(m-1) x^{2}$ 的开口向上,\n\n$\\therefore m-1>0$,\n\n$\\therefore m>1$,\n\n$\\therefore m$ 的值可以是 2 .\n\n故答案为: 2 (答案不唯一)。\n\n【点睛】本题考查了二次函数的图形和性质, 解题的关键是掌握抛物线开口向上, 则二次项系数大于 0 .\n\n"} {"id": "5", "image": [], "answer": " -2\n\n", "solution": "null", "level": "九年级", "question": "拋物线 $y=x^{2}+b x+2$ 与 $y$ 轴交于点 $\\mathrm{A}$, 如果点 $B(2,2)$ 和点 $\\mathrm{A}$ 关于该抛物线的对称轴对称,那么 $b$ 的值是 . $\\qquad$\n", "options": [], "subject": "解析几何", "analysis": "由点 $B(2,2)$ 和点 $\\mathrm{A}$ 关于该拋物线的对称轴对称,可知:抛物线的对称轴是:直线 $x=1$, 进而可得 $b$ 的值.\n\n【详解】 $\\because$ 拋物线 $y=x^{2}+b x+2$ 与 $y$ 轴交于点 $\\mathrm{A}$,\n\n$\\therefore$ 点 $\\mathrm{A}$ 的坐标是: $(0,2)$,\n\n$\\because$ 点 $B(2,2)$ 和点 $\\mathrm{A}$ 关于该抛物线的对称轴对称,\n\n$\\therefore$ 拋物线的对称轴是: 直线 $\\mathrm{x}=1$, 即: $-\\frac{b}{2 a}=1$,\n\n$\\therefore-\\frac{b}{2 \\times 1}=1$, 解得: $\\mathrm{b}=-2$.\n\n故答案是: -2 .\n\n【点睛】本题主要考查二次函数的对称轴公式, 理解二次函数图象的轴对称性, 是解题的关键.\n\n"} {"id": "6", "image": [], "answer": " $\\pm 4$\n\n", "solution": "null", "level": "九年级", "question": "抛物线 $y=x^{2}+b x+4$ 与 $\\mathrm{x}$ 轴有且只有 1 个公共点, 则 $\\mathrm{b}=$ $\\qquad$ .\n", "options": [], "subject": "解析几何", "analysis": "根据抛物线与 $\\mathrm{x}$ 轴有且只有 1 个公共点可知, 当 $y=0$ 时, 此方程有且有两个相等\n的实数根, 根据 $\\Delta=b^{2}-4 a c=0$ 算出 $\\mathrm{b}$ 的值即可.\n\n【详解】 $\\because$ 抛物线 $y=x^{2}+b x+4$ 与 $\\mathrm{x}$ 轴有且只有 1 个公共点,\n\n$\\therefore$ 令 $y=x^{2}+b x+4=0$,\n\n$\\therefore \\triangle=b^{2}-4 \\times 1 \\times 4=0$,\n\n$\\therefore b= \\pm 4$,\n\n故答案为: $\\pm 4$.\n\n【点睛】本题主要考查了抛物线与 $\\mathrm{x}$ 轴交点的知识, 正确把握抛物线与 $\\mathrm{x}$ 轴交点个数确定方法是解题的关键.\n\n"} {"id": "3578", "image": ["4875.jpg", "4876.jpg"], "answer": "$\\sqrt{2}$", "solution": "null", "level": "九年级", "question": "如图, 在 $\\triangle A B C$ 中, $\\angle A=90^{\\circ}, A B=A C=2 \\mathrm{~cm}, \\odot A$ 与 $B C$ 相切,则 $\\odot A$ 的半径为 \\$ \\qquad \\$ cm.\n\n", "options": [], "subject": "组合几何学", "analysis": "连接 $A D$;\n\n\n\n$\\because \\angle A=90^{\\circ}, A B=A C=2 \\mathrm{~cm}$,\n\n$\\therefore \\triangle A B C$ 是等腰直角三角形,\n\n$\\therefore B C=\\sqrt{2} A B=2 \\sqrt{2}$;\n\n$\\because$ 点 $D$ 是斜边的中点,\n\n$\\therefore A D=\\frac{1}{2} B C=\\sqrt{2} \\mathrm{~cm}$,\n\n故答案为 $\\sqrt{2}$."} {"id": "3626", "image": ["4953.jpg"], "answer": "$125^{\\circ}$", "solution": "null", "level": "九年级", "question": "如图, 在 $\\triangle A B C$ 中, $\\angle B O C=140^{\\circ}, I$ 是内心, $O$ 是外心, 则 $\\angle B I C=$\n\n", "options": [], "subject": "组合几何学", "analysis": "$\\because \\angle B O C=140^{\\circ}, O$ 为外心,\n\n$\\therefore \\angle A=\\frac{1}{2} \\angle B O C=70^{\\circ}$,\n\n$\\because I$ 为内心,\n\n$\\therefore \\angle I B C=\\frac{1}{2} \\angle A B C, \\angle I C B=\\frac{1}{2} \\angle A C B$,\n\n$$\n\\begin{aligned}\n& \\therefore \\angle I B C+\\angle I C B \\\\\n= & \\frac{1}{2}(\\angle A B C+\\angle A C B) \\\\\n= & \\frac{1}{2}\\left(180^{\\circ}-\\angle A\\right) \\\\\n= & \\frac{1}{2} \\times\\left(180^{\\circ}-70^{\\circ}\\right)\n\\end{aligned}\n$$\n\n$=55^{\\circ}$,\n\n$\\therefore \\angle B I C=180^{\\circ}-(\\angle I B C+\\angle I C B)=180^{\\circ}-55^{\\circ}=125^{\\circ}$,\n\n故答案为: $125^{\\circ}$."} {"id": "3668", "image": ["5042.jpg", "5043.jpg"], "answer": "$48+12 \\sqrt{3}$", "solution": "null", "level": "九年级", "question": "已知某几何体的三视图如图所示, 其中俯视图为正六边形, 则该几何体的表面积为 \\$ \\qquad \\$ .\n\n\n\n主视图 $\\quad$ 左视图\n\n\n\n俯视图", "options": [], "subject": "组合几何学", "analysis": "观察该几何体的三视图发现该几何体为正六棱柱, 其底面边长为 2, 高为 4 ,故其边心距为 $\\sqrt{3}$,\n\n所以其表面积为 $2 \\times 4 \\times 6+2 \\times \\frac{1}{2} \\times 6 \\times 2 \\times \\sqrt{3}=48+12 \\sqrt{3}$,\n\n故答案为: $48+12 \\sqrt{3}$."} {"id": "3671", "image": ["5046.jpg"], "answer": "6", "solution": "null", "level": "九年级", "question": "将正方体骰子 (相对面上的点数分别为 1 和 $6 、 2$ 和 $5 、 3$ 和 4 )放置于水平桌面上, 如图 1 .在图 2 中,将骰子向右翻滚 $90^{\\circ}$, 然后在桌面上按逆时针方向旋转 $90^{\\circ}$, 则完成一次变换. 若骰子的初始位置\n\n\n\n图 1\n\n图 2\n\n为图 1 所示的状态, 那么按上述规则连续完成 14 次变换后, 骰子朝上一面的点数是", "options": [], "subject": "组合几何学", "analysis": "根据题意可知连续 3 次变换是一循环. 所以 $14 \\div 3=4 \\ldots 2$. 所以是第 2 次变换后的图形, 即按上述规则连续完成 14 次变换后, 骰子朝上一面的点数是 6.\n\n故答案为: 6 ."} {"id": "3878", "image": ["5453.jpg", "5454.jpg"], "answer": "$20 \\quad 145 \\quad 72.5$", "solution": "null", "level": "九年级", "question": "如图, $\\mathrm{PA} 、 \\mathrm{~PB} 、 \\mathrm{EF}$ 分别切 $\\odot \\mathrm{O}$ 于 $\\mathrm{A} 、 \\mathrm{~B} 、 \\mathrm{D}$, 若 $\\mathrm{PA}=10 \\mathrm{~cm}$, 则 $\\triangle \\mathrm{PEF}$ 的周长是 \\$ \\qquad \\$ $\\mathrm{cm}$,若 $\\angle \\mathrm{P}=35^{\\circ}$, 则 $\\angle \\mathrm{AOB}=$ \\$ \\qquad \\$ (度),$\\angle \\mathrm{EOF}=$ \\$ \\qquad \\$ (度) 。\n\n", "options": [], "subject": "组合几何学", "analysis": "$\\because \\quad P A 、 P B 、 E F$ 分别切 $\\odot O$ 于 $A 、 B 、 D$,\n\n$\\therefore \\quad P A=P B=10 \\mathrm{~cm}, E D=E A, F D=D B$,\n\n$\\therefore \\quad P E+E F+P F=P E+E D+P F+F D$\n\n$=P A+P B=20(\\mathrm{~cm})$\n\n$\\because P A 、 P B$ 为 $\\odot O$ 的切线,\n\n$\\therefore \\quad \\angle P A O=\\angle P B O=90^{\\circ}$, 而 $\\angle P=35^{\\circ}$,\n\n\n\n$\\therefore \\quad \\angle A O B=360^{\\circ}-90^{\\circ}-90^{\\circ}-35^{\\circ}=145^{\\circ}$;\n\n连 $O D$, 如图, $\\therefore \\quad \\angle O D E=\\angle O D F=90^{\\circ}$,\n\n易证得 $R t \\triangle O A E \\cong R t \\triangle O D E, R t \\triangle O F D \\cong R t \\triangle O F B, \\therefore \\angle 1=\\angle 2, \\angle 3=\\angle 4$,\n\n$\\therefore \\quad \\angle 2+\\angle 3=\\frac{1}{2} \\angle A O B=72.5^{\\circ}$,\n\n$\\angle E O F=72.5^{\\circ}$. 故答案为 $20 ; 145 ; 72.5$"} {"id": "3579", "image": [], "answer": "$10 \\pi$", "solution": "null", "level": "九年级", "question": "如图所示, 把一个圆锥沿母线 $O A$ 剪开, 展开后得到扇形 $A O C$, 已知圆锥的高 $h$ 为 $12 \\mathrm{~cm}, O A=13 \\mathrm{~cm}$, 则扇形 $A O C$ 中 $\\overline{A C}$ 的长是 \\$ \\qquad \\$ cm. $($ 计算结果保留 $\\pi$ )", "options": [], "subject": "立体几何学", "analysis": ":圆锥的高 $h$ 为 $12 \\mathrm{~cm}, O A=13 \\mathrm{~cm}$,\n\n$\\therefore$ 圆锥的底面半径为 $\\sqrt{13^{2}-12^{2}}=5 \\mathrm{~cm}$,\n\n$\\therefore$ 圆锥的底面周长为 $10 \\pi c m$,\n\n$\\therefore$ 扇形 $A O C$ 中 $\\widehat{A} C$ 的长是 $10 \\pi c m$,\n\n故答案为 $10 \\pi$."} {"id": "3646", "image": ["4991.jpg", "4992.jpg", "4993.jpg", "4994.jpg"], "answer": "$(180 \\sqrt{3}+120)$", "solution": "null", "level": "九年级", "question": "如图, 礼盒的上下底面为全等的正六边形, 其主视图与左视图均由矩形构成, 主视图中大矩形边长如图所示, 左视图中包含两全等的矩形, 如果用彩色胶带如图包扎礼盒, 所需胶带长度至少为 \\$ \\qquad \\$厘米.\n\n\n\n实物图\n\n\n\n主视图\n\n\n\n左视图", "options": [], "subject": "立体几何学", "analysis": "根据题意, 作出实际图形的上底,\n\n如图: $A C, C D$ 是上底面的两边.\n\n则 $A C=60 \\div 2=30(\\mathrm{~cm}), \\angle A C D=120^{\\circ}$,\n\n作 $C B \\perp A D$ 于点 $B$,\n\n\n\n那么 $A B=A C \\times \\sin 60^{\\circ}=15 \\sqrt{3}(\\mathrm{~cm}) ,$\n\n所以 $A D=2 A B=30 \\sqrt{3}(\\mathrm{~cm})$,\n\n胶带的长至少 $=30 \\sqrt{3} \\times 6+20 \\times 6=(180 \\sqrt{3}+120)(\\mathrm{cm})$.\n\n故答案为: $(180 \\sqrt{3}+120)$."} {"id": "3647", "image": ["4995.jpg"], "answer": "27", "solution": "null", "level": "九年级", "question": "墙角处有若干大小相同的小正方体堆成如图所示的立体图形, 如果你打算搬走其中部分小正方体(不考虑操作技术的限制), 但希望搬完后从正面、从上面、从左面用平行光线照射时, 在墙面及地面上的影子不变, 那么你最多可以搬走个小正方体.\n\n", "options": [], "subject": "立体几何学", "analysis": "第 1 列最多可以搬走 9 个小正方体;\n\n第 2 列最多可以搬走 8 个小正方体;\n\n第 3 列最多可以搬走 3 个小正方体;\n\n第 4 列最多可以搬走 5 个小正方体;\n\n第 5 列最多可以搬走 2 个小正方体.\n\n$9+8+3+5+2=27$ 个.\n\n故最多可以搬走 27 个小正方体.\n\n故答案为: 27 ."} {"id": "3648", "image": ["4996.jpg"], "answer": "-2", "solution": "null", "level": "九年级", "question": "一个小立方块的六个面分别标有字母 $A 、 B 、 C 、 D 、 E 、 F$, 从三个不同方向看到的情形如图所示,其中 $A 、 B 、 C 、 D 、 E 、 F$ 分别代表数字 $-2 、-1 、 0 、 1 、 2 、 3$ ,则三个小立方块的下底面所标字母代表的数字的和为\n", "options": [], "subject": "立体几何学", "analysis": "由图形可知: $A$ 与 $B 、 D 、 E 、 F$ 是邻面,故 $A$ 和 $C$ 为对面;\n\n则 $B$ 与 $A 、 C 、 E 、 F$ 是邻面,故 $B$ 和 $D$ 为对面;\n\n故 $E$ 和 $F$ 为对面;\n\n则三个小立方块的下底面所标字母代表的数字的和为 $-1-2+1=-2$.\n故答案为: -2 .\n\n依据图形可知 $A$ 的邻面有 $B 、 D 、 E 、 F$, 故此点 $A$ 和 $C$ 为对面, 进一步得到 $B$ 和 $D$ 为对面; $E$ 和 $F$为对面; 从而可求得三个小立方块的下底面所标字母代表的数字的和.\n\n本题主要考查的是正方体相对两个面上的文字, 找出 $A$ 和 $C$ 为对面; $B$ 和 $D$ 为对面; $E$ 和 $F$ 为对面是解题的关键."} {"id": "3649", "image": ["4997.jpg"], "answer": "76", "solution": "null", "level": "九年级", "question": "如图, 一个 $5 \\times 5 \\times 5$ 的正方体, 先在它的前后方向正中央开丵一个 “十字形”的孔(打通), 再在它的上下方向正中央也开凿一个 “十字形” 的孔(打通), 最后在它的左右方向正中央开凿一个 “十字形” 的孔(打通), 这样得到一个被丵空了的几何体, 则所得几何体的体积为 \\$ \\qquad \\$\n\n", "options": [], "subject": "立体几何学", "analysis": "如图所示:该正方体可按如图方式分割,\n\n则体积为 $(1 \\times 1 \\times 1) \\times(8 \\times 8+12)$\n\n$$\n\\begin{gathered}\n=1 \\times 76 \\\\\n=76\n\\end{gathered}\n$$\n\n故所得几何体的体积为 76.\n\n故答案为: 76 ."} {"id": "3650", "image": [], "answer": "(1)(5)", "solution": "null", "level": "九年级", "question": "用一个平面截下列几何体: (1)长方体, (2)六棱柱, (3)球, (4)圆柱, (5)圆雉, 截面能得到三角形的是 \\$ \\qquad \\$ (填写序号即可)", "options": [], "subject": "立体几何学", "analysis": "1 长方体能截出三角形;\n\n(2)六棱柱沿对角线截几何体可以截出三角形;\n\n(3)球不能截出三角形;\n\n(4)圆柱不能截出三角形;\n\n(5)圆锥能截出三角形;\n\n故截面可能是三角形的有(1)(2)(5)共 3 个.\n\n故答案为: (1) (2)."} {"id": "3651", "image": ["4998.jpg", "4999.jpg"], "answer": "5126", "solution": "null", "level": "九年级", "question": "有同样大小的三个立方体股子, 每个股子的展开图如图 1 所示, 如果把每个骰子点数是 4 的一面放在桌子上, 那么其它五个可以看到的面上的数字的和是 17 , 现在把三个骰子放在桌子上(如图2), 凡是能看\n\n\n\n图1\n\n\n\n图2 得到的点数之和最大是 \\$ \\qquad \\$ , 最小是", "options": [], "subject": "立体几何学", "analysis": "根据题意, 得: 露在外面的数字之和最大是: $3+4+5+6+4+5+6+3+4+5+6=51$,\n\n最小值是: $1+2+3+4+1+2+3+1+2+3+4=26$,\n\n故答案为: 51,26 ."} {"id": "3832", "image": ["5328.jpg"], "answer": "6", "solution": "null", "level": "九年级", "question": "一个长方体的主视图和左视图如图所示(单位: cm), 则其俯视图的面积是 $\\mathrm{cm}^{2}$.\n\n", "options": [], "subject": "立体几何学", "analysis": "一个长方体的主视图和左视图如图所示, 这个长方体的高是 4, 底面长是 3 , 底面宽是 2; 长方体的俯视图就是其底面的图形, 是长是 3 , 宽是 2 的长方形, 它的面积 $=3 \\times 2=6$"} {"id": "3602", "image": ["4915.jpg"], "answer": "$3 \\sqrt{3}$", "solution": "null", "level": "九年级", "question": "以Rt $\\triangle A B C$ 的锐角顶点 $A$ 为圆心, 适当长为半径作弧, 与边 $A B, A C$ 各相交于一点, 再分别以这两个交点为圆心, 适当长为半径作弧, 过两弧的交点与点 $A$ 作直线, 与边 $B C$ 交于点 $D$. 若 $\\angle A D B=$ $60^{\\circ}$, 点 $D$ 到 $A C$ 的距离为 3 , 则 $A B$ 的长为 \\$ \\qquad \\$ .", "options": [], "subject": "画法几何学", "analysis": "如图, 作 $D E \\perp A C$ 于 $E$.\n\n\n\n由题意 $A D$ 平分 $\\angle B A C$,\n\n$\\because D B \\perp A B, D E \\perp A C$,\n\n$\\therefore D B=D E=3$,\n\n在Rt $\\triangle A D B$ 中, $\\because \\angle B=90^{\\circ}, \\angle B D A=60^{\\circ}, B D=3$,\n$\\therefore A B=B D \\cdot \\tan 60^{\\circ}=3 \\sqrt{3}$,\n\n故答案为 $3 \\sqrt{3}$."} {"id": "3604", "image": ["4917.jpg", "4918.jpg", "4919.jpg", "4920.jpg", "4921.jpg"], "answer": "300", "solution": "null", "level": "九年级", "question": "如图, 是一个液压升降机, 图中两个菱形的边长及等腰三角形的腰长都是定值且相等.如图 1, 载物台到水平导轨 $A B$ 的距离 $h_{1}$ 为 $468 \\mathrm{~cm}$, 此时 $\\tan \\angle O A B=\\frac{3}{4}$; 如图 2 , 当 $\\tan \\angle O A B=\\frac{5}{12}$ 时, 载物台到水平导轨 $A B$ 的距离 $h_{2}$ 为 cm.\n\n\n\n\n\n图 1\n\n\n\n图 2", "options": [], "subject": "画法几何学", "analysis": "如图所示, 在图 1 中做 $O M \\perp A B$, 在图 2 中做 $O N \\perp A B$,\n\n\n\n图 1\n\n\n\n图 2\n\n在Rt $\\triangle O A M$ 中,\n\n$\\because \\tan \\angle O A B=\\frac{3}{4}$, 故可设 $O M=3 x, A M=4 x$,\n\n$\\therefore O A=5 x$,\n\n$\\because h_{1}=468 \\mathrm{~cm}$, 两个菱形的边长及等腰三角形的腰长都是定值且相等,\n\n$\\therefore 60 M=468$, 即 $6 \\times 3 x=468$, 解得: $x=\\frac{234}{9}$,\n\n$\\therefore O A=5 x=5 \\times \\frac{234}{9}=\\frac{1170}{9}$,\n\n在Rt $\\triangle O A N$ 中,\n\n$\\because \\tan \\angle O A B=\\frac{5}{12}$, 故可设 $O N=5 y, A N=12 y$,\n\n$\\because O A=\\frac{1170}{9}$,\n\n$\\therefore O A^{2}=A N^{2}+O N^{2}$, 解得: $y=10$,\n\n$\\therefore O N=5 y=50$,\n$\\therefore h_{2}=6 \\times O N=300 \\mathrm{~cm}$,\n\n故答案为 300 ."} {"id": "3625", "image": ["4951.jpg", "4952.jpg"], "answer": "$219^{\\circ}$", "solution": "null", "level": "九年级", "question": "如图, $P A 、 P B$ 是 $\\odot O$ 的切线, $A 、 B$ 为切点, 点 $C 、 D$ 在 $\\odot O$ 上. 若 $\\angle P=102^{\\circ}$, 则 $\\angle A+\\angle C=$\n\n", "options": [], "subject": "画法几何学", "analysis": "连接 $A B$,\n\n\n\n$\\because P A 、 P B$ 是 $\\odot O$ 的切线,\n\n$\\therefore P A=P B$,\n\n$\\because \\angle P=102^{\\circ}$,\n\n$\\therefore \\angle P A B=\\angle P B A=\\frac{1}{2}\\left(180^{\\circ}-102^{\\circ}\\right)=39^{\\circ}$,\n\n$\\because \\angle D A B+\\angle C=180^{\\circ}$,\n\n$\\therefore \\angle P A D+\\angle C=\\angle P A B+\\angle D A B+\\angle C=180^{\\circ}+39^{\\circ}=219^{\\circ}$,\n\n故答案为: $219^{\\circ}$."} {"id": "3669", "image": ["5044.jpg"], "answer": "2.5", "solution": "null", "level": "九年级", "question": "如图是一正方体的表面展开图, 若 $A B=5$, 则该正方体上 $A 、 B$ 两点间的距离为 \\$ \\qquad \\$\n\n", "options": [], "subject": "画法几何学", "analysis": "由题意可得出:正方体上 $A 、 B$ 两点间的距离为正方形对角线长,\n则 $A 、 B$ 两点间的距离为 2.5 .\n\n故答案为 2.5 ."} {"id": "3858", "image": ["5397.jpg", "5398.jpg"], "answer": "24", "solution": "null", "level": "九年级", "question": "为解决楼房之间的挡光问题, 某地区规定: 两幢楼房间的距离至少为 40 米, 中午 12 时不能挡光. 如图, 某旧楼的一楼窗台高 1 米, 要在此楼正南方 40 米处再建一幢新楼. 已知该地区冬天中午 12 时阳光从正南方照射, 并且光线与水平线的夹角最小为 $30^{\\circ}$, 在不违反规定的情况下, 请问新建楼房最高米. (结果精确到 1 米. $\\sqrt{3} \\approx 1.732, \\sqrt{2} \\approx 1.414$ )\n\n", "options": [], "subject": "画法几何学", "analysis": "如图, 过点 $C$ 作 $C E \\perp B D$ 与点 $E$.\n\n在 Rt $\\triangle C D E$ 中, $\\angle D C E=30^{\\circ}, C E=A B=40$, 则 $D E=\\tan 30^{\\circ} \\cdot C E=\\frac{\\sqrt{3}}{3} \\times 40 \\approx 23$, 而 $E B=A C=1$,\n\n$\\therefore B D=D E+E B=23+1=24$ (米).\n\n"} {"id": "3879", "image": ["5455.jpg", "5456.jpg"], "answer": "$\\frac{2}{3}$", "solution": "null", "level": "九年级", "question": "如图, $\\mathrm{PA}, \\mathrm{PB}$ 切 $\\odot \\mathrm{O}$ 于 $\\mathrm{A}, \\mathrm{B}$ 两点, $\\mathrm{CD}$ 切 $\\odot \\mathrm{O}$ 于点 $\\mathrm{E}$, 交 $\\mathrm{PA}, \\mathrm{PB}$ 于 $C, \\mathrm{D}$, 若 $\\odot \\mathrm{O}$ 的半径为 $\\mathrm{r}, \\triangle \\mathrm{PCD}$ 的周长等于 $3 \\mathrm{r}$, 则 $\\tan \\frac{1}{2} \\angle \\mathrm{APB}$ 的值是 . \\$ \\qquad \\$\n\n", "options": [], "subject": "画法几何学", "analysis": "连接 $P O, A O$,\n$\\because \\quad P A, P B$ 切 $\\odot O$ 于 $A, B$ 两点, $C D$ 切 $\\odot O$ 于点 $E$, 交 $P A, P B$ 于 $C, D$,\n\n$\\therefore \\quad \\angle A P O=\\angle B P O, A C=E C, D E=B D, P A=P B$,\n\n$\\therefore \\quad P A+P B=\\triangle P C D$ 的周长 $=3 r, \\therefore \\quad P A=P B=1.5 r$,\n\n$\\therefore \\tan \\frac{1}{2} \\angle A P B=\\frac{A O}{P A}=\\frac{r}{1.5 r}=\\frac{2}{3}$, 故答案为: $\\frac{2}{3}$.\n\n"} {"id": "3670", "image": ["5045.jpg"], "answer": "$\\frac{1}{2}$", "solution": "null", "level": "九年级", "question": "如图是一个正方体的平面展开图, 若正方体每个面与它的对面所标注的代数式的值相等, 且 $x, y$ 满足 $4 x-3 y=8$, 则 $m=$ \\$ \\qquad \\$ .\n\n", "options": [], "subject": "算术", "analysis": "由题意可知 $\\left\\{\\begin{array}{l}2 x+y=3 m(1) \\\\ 2 x-y=7 m(2) \\\\ 4 x-3 y=8(3)\\end{array}\\right.$\n\n(1) + (2) 得 $x=\\frac{5}{2} m$ ,\n\n(1) - (2)得 $y=-2 m$,\n\n把 $x=\\frac{5}{2} m, y=-2 m$ 代入 (3)得 $4 \\times \\frac{5}{2} m-3 \\times(-2 m)=8$,\n\n解得 $m=\\frac{1}{2}$.\n\n故答案为 $\\frac{1}{2}$."} {"id": "3115", "image": [], "answer": "4", "solution": "null", "level": "九年级", "question": "(本题3分)(2022浙江绍兴.九年级期末)已知$frac{a}{b}=frac{3}{5}$,则$frac{b+a}{b-a}=$$qquad$.", "options": [], "subject": "算术", "analysis": "【分析】根据$frac{a}{b}=frac{3}{5}$,可设$a=3k,b=5k$,再代入,即可求解.【详解】解:$becausefrac{a}{b}=frac{3}{5}$,$therefore$可设$a=3k,b=5k$,$thereforefrac{b+a}{b-a}=frac{5k+3k}{5k-3k}=frac{8k}{2k}=4$,故答案为:4.【点睛】本题主要考查了比例的基本性质,熟练掌握比例的基本性质是解题的关键."} {"id": "3855", "image": ["5392.jpg"], "answer": "- 2", "solution": "null", "level": "九年级", "question": "如图是一个正方体纸盒的展开图, 正方体的各面标有数字 $1,2,3,-3, \\mathrm{~A}, \\mathrm{~B}$, 相对面上是两个数互为相反数, 则 $\\mathrm{A}=$ \\$ \\qquad \\$ .\n\n", "options": [], "subject": "图论", "analysis": "由图可知 $\\mathrm{A}=-2$."} {"id": "3856", "image": ["5393.jpg"], "answer": "5,3", "solution": "null", "level": "九年级", "question": "若要使图中平面展开图按虚线折叠成正方体后, 相对面上两个数之和为 $6, x=$ \\$ \\qquad \\$ , $y=$ \\$ \\qquad \\$ .\n\n", "options": [], "subject": "组合数学", "analysis": "这是一个正方体的平面展开图, 共有六个面, 其中面“\"”面“x”相对, 面“ 3 ”与面“ $y$ ”相对,则 $1+x=6,3+y=6$, 解得: $x=5, y=3$. 故答案是: 5,3 ."} {"id": "3027", "image": [], "answer": "3", "solution": "null", "level": "九年级", "question": "(本题3分)(2019$cdot$浙江$cdot$龙泉市顺风实验学校九年级期中)一个密码箱的密码,每个位数上的数都是从0到9的自然数,若要使不知道密码的一次就拨对密码的概率小于$frac{1}{999}$,则密码的位数至少需要$qquad$位.", "options": [], "subject": "组合数学", "analysis": "【分析】分别求出取一位数、两位数、三位数时一次就拨对密码的概率,再根据一次就拨对密码的概率小于$frac{1}{999}$解答即可.【详解】解:因为取一位数时一次就拨对密码的概率为$frac{1}{10}$,取两位数时一次就拨对密码的概率为$frac{1}{100}$,取三位数时一次就拨对密码的概率为$frac{1}{1000}$,故密码的位数至少需要3位.故答案为3.【点睛】本题考查概率的求法与运用,一般方法为:如果一个事件有${n}$种可能,而且这些事件的可能性相同,其中事件${A}$出现${m}$种结果,那么事件${A}$的概率${P}$(${A})=frac{m}{n}$."} {"id": "3028", "image": ["3922.jpg", "3923.jpg"], "answer": "$frac{5}{14}$", "solution": "null", "level": "九年级", "question": "(本题3分)(2021浙江$cdot$义乌市春晗学校九年级期中)如图,在$3times3$正方形网格中,$A、B$在格点上,在网格的其它格点上任取一点$C$,能使$triangleABC$为等腰三角形的概率是$qquad$.", "options": [], "subject": "组合数学", "analysis": "【分析】分三种情况:(1)点$A$为顶点;(2)点$B$为顶点;(3)点$C$为顶点;得到能使$triangleABC$为等腰三角形的点$C$的个数,再根据概率公式计算即可求解.【详解】如图,$becauseAB=sqrt{1^{2}+2^{2}}=sqrt{5}$,$therefore$(1)若$AB=AC$,符合要求的有3个点;(2)若$AB=BC$,符合要求的有2个点;(3)若$AC=BC$,不存在这样格点.$therefore$这样的$C$点有5个.$therefore$能使$triangleABC$为等腰三角形的概率是$frac{5}{14}$.故答案为:$frac{5}{14}$.【点睛】此题考查等腰三角形的判定和概率的求法:如果一个事件有$n$种可能,而且这些事件的可能性相同,其中事件$A$出现$m$种结果,那么事件${A}$的概率$P(A)=frac{m}{n}$."} {"id": "2981", "image": [], "answer": "4", "solution": "null", "level": "九年级", "question": "(本题3分)(2022$cdot$浙江金华$cdot$九年级期末)若二次函数$y=x^{2}-4x+n$的图象与$x$轴只有一个公共点,则实数$n=$", "options": [], "subject": "代数", "analysis": "【详解】解:$y=x^{2}-4x+n$中,$a=1,b=-4,c=n,b^{2}-4ac=16-4n=0$,解得$n=4$.故答案为4."} {"id": "3002", "image": [], "answer": "3.75", "solution": "null", "level": "九年级", "question": "(本题3分)(2020-浙江$cdot$九年级期末)加工爆米花时,爆开且不糊的颗粒的百分比称为“可食用率”,在特定条件下,可食用率$y$与加工时间$x$(单位:$min$)满足函数表达式$y=-0.2x^{2}+1.5x-2$,则最佳加工时间为$qquad$$min$.", "options": [], "subject": "代数", "analysis": "【分析】根据二次函数的对称轴公式$x=-frac{b}{2a}$直接计算即可.【详解】解:$becausey=-0.2x^{2}+1.5x-2$的对称轴为$x=-frac{b}{2a}=-frac{1.5}{2times(-0.2)}=3.75({~min})$,故:最佳加工时间为$3.75{~min},$故答案为:3.75.【点睛】此题主要考查了二次函数性质的应用,涉及求顶点坐标、对称轴方程等,记住抛物线顶点公式是解题关键."} {"id": "3003", "image": [], "answer": "4", "solution": "null", "level": "九年级", "question": "(本题3分)(2022:浙江杭州$cdot$九年级期末)当$xgeqm$时,两个函数$y_{l}=-(x-4)^{2}$+2和$y_{2}=-(x-3)^{2}+1$的函数值都随着$x$的增大而减小,则$m$的最小值为$qquad$.", "options": [], "subject": "代数", "analysis": "【分析】先确定两个函数的开口方向和对称轴,再得出符合条件的$x$的取值范围,从而得到$m$的最小值.【详解】解:函数$y_{l}=-(x-4)^{2}+2$开口向下,对称轴为直线$x=4$,函数$y_{2}=-(x-3)^{2}+1$开口向下,对称轴为直线$x=3$,当函数值都随着$x$的增大而减小,则$xgeq4$,即$m$的最小值为4,故答案为:4.【点睛】本题考查了二次函数的图像和性质,解题的关键是掌握二次函数的基本性质,"} {"id": "4", "image": [], "answer": " $y=(x-2)^{2}+1$\n\n", "solution": "null", "level": "九年级", "question": "将二次函数 $y=x^{2}-4 x+5$ 化成 $y=a(x-h)^{2}+k$ 的形式为 $\\qquad$\n", "options": [], "subject": "代数", "analysis": "利用配方法整理即可得解.\n\n【详解】解: $y=x^{2}-4 x+5=x^{2}-4 x+4+1=(x-2)^{2}+1$,\n\n所以 $y=(x-2)^{2}+1$.\n\n故答案为 $y=(x-2)^{2}+1$.\n\n【点睛】本题考查了二次函数的解析式有三种形式:\n\n(1)一般式: $y=a x^{2}+b x+c(a \\neq 0, a 、 b 、 c$ 为常数 $)$;\n\n(2)顶点式: $y=a(x-h)^{2}+k$;\n\n(3)交点式(与 $x$ 轴): $y=a\\left(x-x_{1}\\right)\\left(x-x_{2}\\right)$.\n\n"} {"id": "29", "image": [], "answer": " $x_{1}=-1, x_{2}=3$.\n\n【详解】试题解析: 根据题意, 得抛物线的对称轴为 $\\mathrm{x}=-\\frac{2 a}{2 \\times(-a)}=1$,\n\n又 $\\mathrm{x}=3$ 是方程 $-a x^{2}+2 a x+m=0$ 的一个根,\n\n一元二次方程 $-\\mathrm{x}^{2}+2 \\mathrm{x} 3=0$ 的解为: $\\mathrm{x}_{1}=-1, \\mathrm{x}_{2}=3$.\n\n考点: 抛物线与 $\\mathrm{x}$ 轴的交点.\n", "solution": "null", "level": "九年级", "question": "已知二次函数 $y=-a x^{2}+2 a x+m$ 的图像与 $\\mathrm{x}$ 轴的一个交点是 $(3,0)$, 则关于 $\\mathrm{x}$ 的\n一元二次方程 $-a x^{2}+2 a x+m=0$ 的解为 $\\qquad$\n", "options": [], "subject": "代数", "analysis": "_{1}=-1, x_{2}=3$.\n\n【详解】试题解析: 根据题意, 得抛物线的对称轴为 $\\mathrm{x}=-\\frac{2 a}{2 \\times(-a)}=1$,\n\n又 $\\mathrm{x}=3$ 是方程 $-a x^{2}+2 a x+m=0$ 的一个根,\n\n一元二次方程 $-\\mathrm{x}^{2}+2 \\mathrm{x} 3=0$ 的解为: $\\mathrm{x}_{1}=-1, \\mathrm{x}_{2}=3$.\n\n考点: 抛物线与 $\\mathrm{x}$ 轴的交点.\n\n"} {"id": "30", "image": [], "answer": " -3\n\n", "solution": "null", "level": "九年级", "question": "已知二次函数 $y=a x^{2}+b x+c(a, b, c$ 是常数, $a \\neq 0)$ 的 $y$ 与 $x$ 的部分对应值如表\n\n| $x$ | -1 | 1 | 4 |\n| :--- | :--- | :--- | :--- |\n| $y$ | 3 | -3 | 3 |\n\n当 $x=2$ 时, 函数值为 $\\qquad$\n.\n\n", "options": [], "subject": "代数", "analysis": "根据表格中的数据可以求得二次函数的解析式, 再将 $x=2$ 代入求 $y$ 的值即可;\n\n【详解】解: 由题意, 把 $(-1,3) 、(1,-3) 、(4,3)$ 、代入 $y=a^{2}+b x+c$ 得, $\\left\\{\\begin{array}{l}a-b+c=3 \\\\ a+b+c=-3 \\\\ 16 a+4 b+c=3\\end{array}\\right.$,\n解得: $\\left\\{\\begin{array}{l}a=1 \\\\ b=-3, \\\\ c=-1\\end{array}\\right.$\n\n$\\therefore$ 此二次函数关系式为: $y=x^{2}-3 x-1$,\n\n当 $x=2$ 时, $y=2^{2}-3 \\times 2-1=-3$,\n\n故答案为: -3 .\n\n【点睛】本题考查了待定系数法求二次函数的解析式以及给出自变量的值求函数值, 求出二次函数的解析式是解决问题的关键.\n\n"} {"id": "52", "image": [], "answer": " $-4 \\leqslant y \\leqslant 357$\n\n", "solution": "null", "level": "九年级", "question": "已知二次函数 $y=x^{2}+2 x-3$, 当 $-4 \\leq x \\leq 18$ 时, $y$ 的取值范围为 $\\qquad$\n", "options": [], "subject": "代数", "analysis": "先求出二次函数的对称轴和顶点坐标, 再利用二次函数的增减性即可得出结论.\n\n【详解】解: $\\because y=x^{2}+2 x-3=(x+1)^{2}-4$,\n\n$\\therefore$ 该拋物线的对称轴为直线 $x=-1$,\n\n当 $x=-4$ 时, $y=16-8-3=5$,\n\n当 $x=-1$ 时, 最小值为 $y=1-2-3=-4$,\n\n当 $x=18$ 时, $y=324+36-3=357$,\n$\\therefore-4 \\leqslant y \\leqslant 357$,\n\n故答案为: $-4 \\leqslant y \\leqslant 357$.\n\n【点睛】本题主要考查二次函数的增减性和最值, 关键是要牢记抛物线的对称轴的公式, 理解抛物线的增减性.\n\n"} {"id": "3024", "image": [], "answer": "$frac{1}{2}$", "solution": "null", "level": "九年级", "question": "(本题3分)(2019$cdot$浙江杭州$cdot$九年级期末)已经连续抛一枚质量均匀的硬币5次都是正面朝上,现再抛一次,正面朝上的概率是", "options": [], "subject": "计数", "analysis": "【分析】根据抛掷一枚硬币,正面朝上和反面朝上的可能性一样即可.【详解】解:连续抛一枚质量均匀的硬币5次都是正面朝上,现再抛一次,有可能正面朝上,也可能反面朝上,且可能性一样,$therefore$现再抛一次,正面朝上的概率是$frac{1}{2}$,故答案为:$frac{1}{2}$.【点睛】本题考查了事件可能性的大小,解题的关键是熟知抛郑一枚硬币,正面朝上和反面朝上的可能性一样."} {"id": "3025", "image": [], "answer": "$frac{1}{10}$", "solution": "null", "level": "九年级", "question": "(本题3分)(2022$cdot$浙江金华$cdot$九年级期末)20瓶饮料中有2瓶已过了保质期,从20瓶饮料中任取1瓶,取到已过保质期的饮料的概率是", "options": [], "subject": "计数", "analysis": "【分析】由有20瓶饮料,其中有2瓶已过保质期,直接利用概率公式求解即可求得答案.【详解】解:$because$有20瓶饮料,其中有2瓶已过保质期,$therefore$从20瓶饮料中任取1瓶,取到已过保质期的饮料的概率为:$frac{2}{20}=frac{1}{10}$.故答案为$frac{1}{10}$.【点睛】此题考查了概率公式的应用,用到的知识点为:概率$=$所求情况数与总情况数之比,熟知概率的求法是解题的关键."} {"id": "3026", "image": [], "answer": "0.95", "solution": "null", "level": "九年级", "question": "(本题3分)(2022$cdot$浙江宁波$cdot$九年级期末)下表是某种幼苗在一定条件下移植后成活率的试验结果|移植总数$n$|5|50|200|500|1000|3000||:---:|:---:|:---:|:---:|:---:|:---:|:---:||成活数$m$|4|45|188|476|951|285||||||||||成活的频率$frac{m}{n}$|0.8|0.9|0.94|0.952|0.951|0.95|则在相同条件下这种幼苗可成活的概率可估计为", "options": [], "subject": "计数", "analysis": "【分析】概率是大量重复实验的情况下,频率的稳定值可以作为概率的估计值,即次数越多的频率越接近于概率.【详解】解:$because$成活的频率的稳定值约为0.95,$therefore$这种幼苗可成活的概率可估计为0.95,故答案为:0.95.【点睛】此题主要考查了利用频率估计概率,大量反复试验下频率稳定值即概率.用到的知识点为:频率=所求情况数与总情况数之比."} {"id": "3030", "image": [], "answer": "0.68", "solution": "null", "level": "九年级", "question": "(本题3分)(2021$cdot$浙江温州$cdot$九年级期末)下表记录了一名篮球运动员在罚球线上投篮的结果:|投篮次数$n$|48|82|124|176|230|287|328||:---|:---|:---|:---|:---|:---|:---|:---||投中次数$m$|33|59|83|118|159|195|223||投中频率$frac{m}{n}$|0.69|0.72|0.67|0.67|0.69|0.68|0.68|根据表格,这名篮球运动员投篮一次,投中的概率约为.(结果精确到0.01)", "options": [], "subject": "计数", "analysis": "【分析】根据频率估计概率的方法结合表格数据可得答案.【详解】解:这名篮球运动员投篮一次,投中的概率约为0.68,故答案为:0.68.【点睛】本题考查了利用频率估计概率的知识,注意这种概率的得出是在大量实验的基础上得出的,不能单纯的依靠几次决定."} {"id": "237", "image": ["10286.jpg", "10287.jpg", "10288.jpg"], "answer": " $\\frac{1}{3}$\n\n", "solution": "null", "level": "九年级", "question": "现有三张正面印有 2023 年杭州亚运会吉祥物琮琮、宸宸和莲莲的不透明卡片, 卡片除正面图案不同外,其余均相同,将三张卡片正面向下洗匀,从中随机抽取一张卡片, 则抽出的卡片图案是琮琮的概率是 $\\qquad$ .\n\n\n\n琮琮\n\n\n\n宸宸\n\n\n\n莲莲\n\n(第 12 题)\n\n", "options": [], "subject": "计数", "analysis": "根据概率公式即可求解.\n\n【详解】解: 将三张卡片正面向下洗匀, 从中随机抽取一张卡片, 则抽出的卡片图案是琮琮的概率是 $\\frac{1}{3}$\n\n故答案为: $\\frac{1}{3}$.\n【点睛】本题考查了概率公式求概率, 熟练掌握概率公式是解题的关键.\n\n"} {"id": "3555", "image": ["4818.jpg"], "answer": "6", "solution": "null", "level": "九年级", "question": "将正方体骰子 (相对面上的点数分别为 1 和 $6 、 2$ 和 $5 、 3$ 和 4 )放置于水平桌面上, 如图 1 .在图2中, 将骰子向右翻滚 $90^{\\circ}$, 然后在桌面上按逆时针方向旋转 $90^{\\circ}$, 则完成一次变换. 若股子的初始位置为图1所示的状态,那么按上述规则连续完成14次变换后,骰子朝上一面的点数是 \\$ \\qquad \\$ .\n\n\n\n图 1\n\n图2", "options": [], "subject": "变换几何", "analysis": "解: 根据题意可知连续 3 次变换是一循环. 所以 $14 \\div 3=4 \\ldots 2$. 所以是第 2 次变换后的图形, 即按上述规则连续完成 14 次变换后, 骰子朝上一面的点数是 6.\n\n故答案为: 6 .\n\n先向右翻滚, 然后再逆时针旋转叫做一次变换, 那么连续 3 次变换是一个循环. 本题先要找出 3 次变换是一个循环, 然后再求 14 被 3 整除后余数是 2 , 从而确定出连续完成 14 次变换后, 股子朝上一面的点数.\n\n本题考查了正方体相对两个面上的文字, 是一道找规律的题目, 这类题型在中考中经常出现. 对于找规律的题目首先应找出哪些部分发生了变化,是按照什么规律变化的."} {"id": "265", "image": ["10312.jpg", "10313.jpg"], "answer": " 2\n\n", "solution": "null", "level": "九年级", "question": "如图, 在直角 $\\triangle D E F$ 中, $\\angle F=30^{\\circ}$, 将 $\\triangle D E F$ 绕点 $D$ 逆时针旋转一定的角度至\n\n$\\triangle D E^{\\prime} F^{\\prime}$ 处, 此时点 $E, D, F^{\\prime}$ 恰好在同一条直线上, 连接 $E E^{\\prime}$, 若 $E E^{\\prime}=\\sqrt{3}$,\n则 $D F=$ $\\qquad$ .\n\n\n\n(第 17 题)\n\n", "options": [], "subject": "变换几何", "analysis": "根据旋转的性质得出 $\\angle E^{\\prime} D F=60^{\\circ}$, 设 $E E^{\\prime}$ 与 $D F$ 交于点 $O$, 证明 $\\triangle E D O \\cong \\triangle E^{\\prime} D O(\\mathrm{SAS})$, 得到 $E O$ 和 $E F$ 的长度, 再利用勾股定理求解即可.\n\n【详解】解: $\\because$ Rt $\\triangle D E F$ 中, $\\angle F=30^{\\circ}$,\n\n$\\therefore \\angle E D F=60^{\\circ}$\n\n由旋转的性质可得 $\\angle E^{\\prime} D F^{\\prime}=\\angle E D F=60^{\\circ}$,\n\n$\\therefore \\angle E^{\\prime} D F=60^{\\circ}$,\n\n如图, 设 $E E^{\\prime}$ 与 $D F$ 交于点 $O$,\n\n\n\n$\\because D E=D E^{\\prime}, \\angle E D O=\\angle E^{\\prime} D O, \\quad D O=D O$,\n\n$\\therefore \\triangle E D O \\cong \\triangle E^{\\prime} D O(\\mathrm{SAS})$,\n\n$\\therefore \\angle D O E=\\angle D O E^{\\prime}=90^{\\circ}, E O=E O^{\\prime}=\\frac{1}{2} E E^{\\prime}=\\frac{\\sqrt{3}}{2}$,\n\n$\\because \\angle F=30^{\\circ}$\n\n$\\therefore E F=2 E O=\\sqrt{3}, \\quad D F=2 D E$,\n\n由勾股定理可得 $D F^{2}=D E^{2}+E F^{2}$,\n\n即 $4 D E^{2}=D E^{2}+3$,\n\n$\\therefore D E=1, \\quad D F=2$,\n故答案为: 2 .\n\n【点睛】本题考查了旋转图形的性质, 勾股定理, 含 30 度角直角三角形的性质等, 求出 $E F$的长是解题的关键.\n\n"} {"id": "311", "image": ["10389.jpg"], "answer": " $8 \\pi \\quad \\frac{4 \\sqrt{3}}{3} \\pi$\n\n", "solution": "null", "level": "九年级", "question": "如图, 在 Rt $\\triangle A B C$ 中, $\\angle A C B=90^{\\circ}, \\angle A=30^{\\circ}, A B=8$, 将 Rt $\\triangle A B C$ 绕点 $C$ 顺时针旋转得到 Rt $\\triangle A^{\\prime} B^{\\prime} C$, 点 $B$ 恰好在斜边 $A^{\\prime} B^{\\prime}$ 上, 则线段 $C A$ 扫过的面积为 $\\qquad$ .\n\n \n\n (第 15 题)\n\n\n\n\n\n", "options": [], "subject": "变换几何", "analysis": "解直角三角形求出 $B C 、 A C 、 \\angle A B C$ 的度数, 根据旋转的性质求出 $C B^{\\prime}=C B$,\n\n$\\angle B^{\\prime}=\\angle A B C=60^{\\circ}, \\angle A^{\\prime} C B^{\\prime}=\\angle A C B=90^{\\circ}$, 求出 $\\angle A C A^{\\prime}$, 再根据扇形的面积公式和弧长公式求出答案即可.\n\n【详解】解: $\\because \\angle A C B=90^{\\circ}, \\angle A=30^{\\circ}, A B=8$,\n\n$\\therefore B C=\\frac{1}{2} A B=4, \\quad \\angle A B C=90^{\\circ}-\\angle A=60^{\\circ}$,\n\n$\\therefore A C=\\sqrt{A B^{2}-B C^{2}}=\\sqrt{8^{2}-4^{2}}=4 \\sqrt{3}$,\n\n$\\because$ 将 Rt $\\triangle A B C$ 绕点 $C$ 顺时针旋转, 使斜边 $A^{\\prime} B^{\\prime}$ 过 $B$ 点,\n\n$\\therefore C B^{\\prime}=C B, \\angle B^{\\prime}=\\angle A B C=60^{\\circ}, \\angle A^{\\prime} C B^{\\prime}=\\angle A C B=90^{\\circ}$,\n\n$\\therefore \\triangle B C B^{\\prime}$ 是等边三角形,\n\n$\\therefore \\angle B C B^{\\prime}=60^{\\circ}$,\n\n$\\therefore \\angle A^{\\prime} C B=90^{\\circ}-60^{\\circ}=30^{\\circ}$,\n\n$\\therefore \\angle A C A^{\\prime}=\\angle A C B-\\angle A^{\\prime} C B=90^{\\circ}-30^{\\circ}=60^{\\circ}$,\n\n$\\therefore$ 线段 $C A$ 扫过的面积为 $\\frac{60 \\pi \\times(4 \\sqrt{3})^{2}}{360}=8 \\pi$,\n\n点 $A$ 经过的路径的长为 $\\frac{60 \\pi \\times 4 \\sqrt{3}}{180}=\\frac{4 \\sqrt{3}}{3} \\pi$,\n\n故答案为: $8 \\pi ; \\frac{4 \\sqrt{3}}{3} \\pi$.\n\n【点睛】本题考查了旋转的性质, 等腰直角三角形, 解直角三角形和扇形的面积以及弧长计算等知识点, 能综合运用知识点进行推理和计算是解此题的关键.\n\n"} {"id": "236", "image": [], "answer": " 随机事件\n\n", "solution": "null", "level": "九年级", "question": "“八月十五云遮月,正月十五雪打灯”是一句谚语,意思是说如果八月十五晚上阴天的话,正月十五晚上就下雪,你认为农谚说的是 $\\qquad$ (填写“必然事件”或“不可能事件”或“随机事件”).\n", "options": [], "subject": "逻辑题", "analysis": "根据确定事件和随机事件的定义来区分判断即可, 必然事件和不可能事件统称确定性事件; 必然事件:在一定条件下,一定会发生的事件称为必然事件; 不可能事件:在一定条件下,一定不会发生的事件称为不可能事件; 随机事件:在一定条件下,可能发生也可能不发生的事件称为随机事件.\n\n【详解】“八月十五云遮月,正月十五雪打灯”是一句谚语,意思是说如果八月十五晚上阴天的话,正月十五晚上就下雪,说的是随机事件.故答案为: 随机事件.\n\n【点睛】本题考查了确定事件和随机事件的定义, 熟悉定义是解题的关键.\n"} {"id": "240", "image": [], "answer": " $\\frac{1}{2}$\n\n", "solution": "null", "level": "九年级", "question": "某人随意投郑一枚均匀的骰子, 六个面分别写有 $1 , 2 , 3 , 4 , 5 , 6$ 六个数字, 投掷了 $n$ 次, 其中有 $m$ 次掷出的点数是偶数, 即掷出的点数是偶数的频率为 $\\frac{m}{n}$. 若投掷的次数足够多, 则 $\\frac{m}{n}$ 的值会稳定在 . $\\qquad$\n\n", "options": [], "subject": "统计数学", "analysis": "根据在相同的条件下, 大量重复试验时, 随机事件发生的频率逐渐稳定在概率附近解题即可.\n\n【详解】解: 某人随意投掷一枚均匀的股子, 投郑了 $n$ 次, 其中有 $m$ 次郑出的点数是偶数,即掷出的点数是偶数的频率为 $\\frac{m}{n}$, 若投掷的次数足够多, 则 $\\frac{m}{n}$ 的值会稳定在 $\\frac{1}{2}$.\n\n故答案为: $\\frac{1}{2}$.\n\n【点睛】本题考查频率估计概率, 掌握频率与概率间的关系是解题的关键.\n\n"} {"id": "13155", "image": [], "answer": "答案: $-7 \\quad+5$", "solution": "null", "level": "六年级", "question": "某宾馆大厅有 8 根一样大小的圆柱形大理石柱, 每根柱子的半径是 5 分米,高6 米, 如果要清洗这些柱子, 清洗的面积是( )平方米。", "options": [], "subject": "立体几何学", "analysis": "答案: $-7 \\quad+5$"} {"id": "13158", "image": [], "answer": "答案: $X$", "solution": "null", "level": "六年级", "question": "圆柱的侧面积一定, 它的高和底面半径成反比例。", "options": [], "subject": "立体几何学", "analysis": "答案: $X$"} {"id": "13042", "image": [], "answer": "答案:错误", "solution": "null", "level": "六年级", "question": "两个圆柱的侧面积相等,则它们的体积也一定相等.", "options": [], "subject": "立体几何学", "analysis": "答案:错误"} {"id": "13043", "image": [], "answer": "2", "solution": "null", "level": "六年级", "question": "一个圆柱的高不变, 底面积扩大 2 倍, 圆柱的体积扩大 \\$ \\qquad \\$ .", "options": [], "subject": "立体几何学", "analysis": ""} {"id": "13044", "image": [], "answer": "答案: 40", "solution": "null", "level": "六年级", "question": "把 20 分米长的圆柱形木棒锯成三段,分成三个小圆柱,表面积增加了 8 平方分米,原来木棒的体积是 \\$ \\qquad \\$立方分米.", "options": [], "subject": "立体几何学", "analysis": "答案: 40"} {"id": "13045", "image": [], "answer": "4", "solution": "null", "level": "六年级", "question": "数学老师的教具里有一个圆柱和一个圆锥, 老师告诉陈明, 圆柱和圆锥的体积相等, 底面积也相等, 圆雉的高是 $12 \\mathrm{~cm}$, 这个圆柱的高是 \\$ \\qquad \\$ $\\mathrm{cm}$ 。", "options": [], "subject": "立体几何学", "analysis": ""} {"id": "13046", "image": [], "answer": "$28.26 ; 18.84$", "solution": "null", "level": "六年级", "question": "一个近似圆锥的煤堆, 底面半径是 $3 \\mathrm{~m}$, 高 $2 \\mathrm{~m}$, 它的占地面积是 $\\mathrm{m}^{2} \\quad$, 体积是 $m^{3}$ 。", "options": [], "subject": "立体几何学", "analysis": "$28.26 ; 18.84$"} {"id": "13057", "image": [], "answer": "答案:错误", "solution": "null", "level": "六年级", "question": "圆柱的高度不变, 底面半径扩大 2 倍, 圆柱的体积也扩大 2 倍. ( )", "options": [], "subject": "立体几何学", "analysis": "答案:错误"} {"id": "13058", "image": [], "answer": "答案:错误", "solution": "null", "level": "六年级", "question": "如果圆柱的侧面展开后是正方形,那么圆柱的底面直径和高相等.", "options": [], "subject": "立体几何学", "analysis": "答案:错误"} {"id": "13060", "image": [], "answer": "B", "solution": "null", "level": "六年级", "question": "圆雉的体积一定等于圆柱体积的三分之一。", "options": [], "subject": "立体几何学", "analysis": ""} {"id": "13139", "image": [], "answer": "答案:120", "solution": "null", "level": "六年级", "question": "把一个 360 立方厘米的圆柱加工成与它底面相等的最大圆锥, 圆锥的体积是()立方厘米。", "options": [], "subject": "立体几何学", "analysis": "答案:120"} {"id": "27008", "image": [], "answer": "$\\sqrt{ }$", "solution": "null", "level": "六年级", "question": " 底面积和高都相等的长方体、正方体、圆柱体, 它们的体积也一定相等。", "options": [], "subject": "立体几何学", "analysis": "$\\sqrt{ }$"} {"id": "27010", "image": [], "answer": "0.3", "solution": "null", "level": "六年级", "question": " 一个圆雉和一个圆柱等底等高, 圆锥的体积比圆柱小 0.6 立方分米, 圆雉的体积是 $\\qquad$立方分米。", "options": [], "subject": "立体几何学", "analysis": "0.3"} {"id": "27012", "image": [], "answer": "6", "solution": "null", "level": "六年级", "question": " 一个棱长是 2 分米的正方体容器装满水, 将水倒入一个底面积是 4 平方分米的圆锥形容器中正好装满, 这个圆雉的高是 $\\qquad$分米。", "options": [], "subject": "立体几何学", "analysis": "6"} {"id": "27013", "image": [], "answer": "$1 ; 28.26$", "solution": "null", "level": "六年级", "question": "一个圆雉的体积是 9.42 立方分米, 底面半径是 3 分米, 它的高是 $\\qquad$分米,和它等底等高的圆柱的体积是 $\\qquad$立方分米。", "options": [], "subject": "立体几何学", "analysis": "$1 ; 28.26$"} {"id": "27014", "image": [], "answer": "20", "solution": "null", "level": "六年级", "question": " 一个圆柱体和一个圆雉体等底等高, 体积相差 40 立方厘米, 圆锥体的体积是 $\\qquad$立方厘米。", "options": [], "subject": "立体几何学", "analysis": "20"} {"id": "27015", "image": [], "answer": "28. 26; 75.36; 113.04", "solution": "null", "level": "六年级", "question": " 一个圆柱的底面半径是 3 厘米, 高是 4 厘米, 它的底面积是 $\\qquad$平方厘米,侧面积是 $\\qquad$平方厘米,体积是 $\\qquad$立方厘米。", "options": [], "subject": "立体几何学", "analysis": "28. 26; 75.36; 113.04"} {"id": "27016", "image": [], "answer": "14.4", "solution": "null", "level": "六年级", "question": " 一个圆柱和一个圆雉的体积相等, 它们底面积的比是 $3: 5$, 圆柱的高是 8 厘米, 圆雉的高是 $\\qquad$厘米。", "options": [], "subject": "立体几何学", "analysis": "14.4"} {"id": "27017", "image": [], "answer": "略", "solution": "null", "level": "六年级", "question": " 把一个圆柱削成一个最大的圆雉,体积减少了 1.8 立方分米,这个圆柱的体积是 $\\qquad$立方厘米。", "options": [], "subject": "立体几何学", "analysis": "略"} {"id": "27019", "image": [], "answer": "45", "solution": "null", "level": "六年级", "question": " 把一个圆柱形木料加工成一个最大的圆雉体, 需要削去 30 立方分米的木料, 则原来这根木料的体积是 $\\qquad$立方分米。", "options": [], "subject": "立体几何学", "analysis": "45"} {"id": "27020", "image": [], "answer": "$30 ; 2$", "solution": "null", "level": "六年级", "question": " 将一块体积是 $90 \\mathrm{~m}^{3}$ 的圆柱形铁块, 削成一个最大的圆雉, 削成的圆雉的体积是 $\\qquad$ $\\mathrm{m}^{3}$ 。削下的铁屑还可熔铸成 $\\qquad$个这样的圆锥。", "options": [], "subject": "立体几何学", "analysis": "$30 ; 2$"} {"id": "27021", "image": ["13226.jpg"], "answer": "9. $42 ; 226.08$", "solution": "null", "level": "六年级", "question": " 如图, 把底面周长 $18.84 \\mathrm{~cm}$ 、高 $8 \\mathrm{~cm}$ 的圆柱切成若干等份, 拼成一个近似的长方体。这个长方体的长是 $\\qquad$ $\\mathrm{cm}$, 体积是 $\\qquad$ $\\mathrm{cm}^{3}$ 。\n", "options": [], "subject": "立体几何学", "analysis": "9. $42 ; 226.08$"} {"id": "27022", "image": [], "answer": "37. 68; 56.52", "solution": "null", "level": "六年级", "question": " 一个圆柱形鼓, 底面直径是 6 分米, 高是 2 分米, 它的侧面由铝皮围成,上、下底面蒙的是羊皮。做一个这样的鼓, 需要铝皮 $\\qquad$平方分米,羊皮 $\\qquad$平方分米。", "options": [], "subject": "立体几何学", "analysis": "37. 68; 56.52"} {"id": "27023", "image": [], "answer": "1.5; 0.5", "solution": "null", "level": "六年级", "question": " 一个圆柱和一个圆雉容器等底等高, 张老师把 $2 \\mathrm{~L}$ 药水倒入这两个容器中, 正好倒满, 圆柱形容器的容积是 $\\qquad$ $\\mathrm{L}$, 圆雉形容器的容积是 $\\qquad$ L。", "options": [], "subject": "立体几何学", "analysis": "1.5; 0.5"} {"id": "27024", "image": [], "answer": "600", "solution": "null", "level": "六年级", "question": " 将一根长 4 米的圆柱形木料锯成 3 段, 表面积增加了 60 平方分米。这根木料的体积是 $\\qquad$立方分米。", "options": [], "subject": "立体几何学", "analysis": "600"} {"id": "27025", "image": ["13227.jpg", "13228.jpg", "13228.jpg"], "answer": "", "solution": "null", "level": "六年级", "question": " 下图所示长方形、半圆形、梯形、三角形快速旋转一周, 能形成什么图形?请你连一连。\n", "options": [], "subject": "立体几何学", "analysis": ""} {"id": "27027", "image": ["13232.jpg", "13233.jpg", "13234.jpg"], "answer": "(1) $12 \\times(4 \\div 2)^{2} \\times 3.14$\n\n$=12 \\times 4 \\times 3.14$\n\n$=48 \\times 3.14$\n\n$=150.72$ (立方厘来)\n\n(2) $\\frac{1}{3} \\times 12 \\times(8 \\div 2)^{2} \\times 3.14$\n\n$=4 \\times 50.24$\n\n$=200.96$ (立方厘米)\n\n(3) $12 \\times 2 \\times 5+\\frac{1}{3} \\times 2^{2} \\times 3.14 \\times 9$\n\n$=120+4 \\times 3.14 \\times 3$\n\n$=157.68$ (立方厘米)", "solution": "null", "level": "六年级", "question": " 计算下面图形的体积。(单位:厘米)\n\n\n\n(1)\n\n(2)\n\n\n\n(3)\n\n", "options": [], "subject": "立体几何学", "analysis": "(1) $12 \\times(4 \\div 2)^{2} \\times 3.14$\n\n$=12 \\times 4 \\times 3.14$\n\n$=48 \\times 3.14$\n\n$=150.72$ (立方厘来)\n\n(2) $\\frac{1}{3} \\times 12 \\times(8 \\div 2)^{2} \\times 3.14$\n\n$=4 \\times 50.24$\n\n$=200.96$ (立方厘米)\n\n(3) $12 \\times 2 \\times 5+\\frac{1}{3} \\times 2^{2} \\times 3.14 \\times 9$\n\n$=120+4 \\times 3.14 \\times 3$\n\n$=157.68$ (立方厘米)"} {"id": "27038", "image": [], "answer": "$\\times$", "solution": "null", "level": "六年级", "question": " 两个圆柱的底面周长相等,它们的侧面积也相等。", "options": [], "subject": "立体几何学", "analysis": "$\\times$"} {"id": "27039", "image": [], "answer": "$\\sqrt{ }$", "solution": "null", "level": "六年级", "question": " 等底等高的正方体体积比圆雉的体积大。", "options": [], "subject": "立体几何学", "analysis": "$\\sqrt{ }$"} {"id": "27040", "image": [], "answer": "$\\times$", "solution": "null", "level": "六年级", "question": " 圆雉的体积是圆柱体积的 $\\frac{1}{3}$, 那么圆雉和圆柱一定等底等高。", "options": [], "subject": "立体几何学", "analysis": "$\\times$"} {"id": "27042", "image": [], "answer": "$\\sqrt{ }$", "solution": "null", "level": "六年级", "question": " 正方体的表面积和它的一个面的面积成正比例关系。", "options": [], "subject": "立体几何学", "analysis": "$\\sqrt{ }$"} {"id": "27165", "image": [], "answer": "$\\times$", "solution": "null", "level": "六年级", "question": " 两个圆柱的体积相等,那么它们的表面积也相等。()", "options": [], "subject": "立体几何学", "analysis": "$\\times$"} {"id": "27166", "image": [], "answer": "3.6a; 2.4a", "solution": "null", "level": "六年级", "question": " 有一圆柱形材料, 体积约是 3.6 立方米, 每立方米材料约重 0.8 千克, 这个圆柱形材料约重千克。把这个圆柱形材料削成最大的圆雉,重量减少了 $\\qquad$千克。", "options": [], "subject": "立体几何学", "analysis": "3.6a; 2.4a"} {"id": "27167", "image": ["13273.jpg"], "answer": "$32 ; 25.12$", "solution": "null", "level": "六年级", "question": " 两位同学对同一圆柱的截面进行研究。如图, 两种不同的截法(平均分成两部分), 甲同学切分后表面积比原来增加 $\\qquad$ $\\mathrm{cm}^{2} ;$ 乙同学切分后, 表面积比原来增加 $\\qquad$ $\\mathrm{cm}^{2}$ 。\n\n", "options": [], "subject": "立体几何学", "analysis": "$32 ; 25.12$"} {"id": "13156", "image": [], "answer": "$\frac{1}{10} 0.2$", "solution": "null", "level": "六年级", "question": "把 2 支红色铅笔和 8 支粉色铅笔放进一个布袋里, 每次从布袋里摸出一支,摸出 ( ) 色铅笔的可能性大。", "options": [], "subject": "组合数学", "analysis": ""} {"id": "13171", "image": [], "answer": "5: 4$", "solution": "null", "level": "六年级", "question": "$\\mathrm{O} O$ 改记的 $\\mathrm{O} O$ 记於的……左起第 21 个图形是 ( ) , 前 50 个图形中 $O$ 有 ( ) 个, 改有 ( )个。", "options": [], "subject": "组合数学", "analysis": ""} {"id": "13123", "image": [], "answer": "答案:3", "solution": "null", "level": "六年级", "question": "只涻子飞回了 3 个鸽舍, 总有 1 个鸽舍至少飞进只鸽子。", "options": [], "subject": "组合数学", "analysis": "答案:3"} {"id": "13124", "image": [], "answer": "答案:3", "solution": "null", "level": "六年级", "question": "本书放进 3 个抽屉中. 无论怎么放总有一个抽屉至少放进本。", "options": [], "subject": "组合数学", "analysis": "答案:3"} {"id": "13125", "image": [], "answer": "答案:8", "solution": "null", "level": "六年级", "question": "位同学练习投篮, 投进 30 个球, 那么总有 1 人至少投进了个球。", "options": [], "subject": "组合数学", "analysis": "答案:8"} {"id": "13134", "image": [], "answer": "答案: $\\times$", "solution": "null", "level": "六年级", "question": "只小鸡装入 4 个笼子,至少有一个笼子放小鸡 3 只。", "options": [], "subject": "组合数学", "analysis": "答案: $\\times$"} {"id": "13135", "image": [], "answer": "答案:$\\sqrt{ }$", "solution": "null", "level": "六年级", "question": "六年级共有学生 370 人, 其中至少有 2 人是同一天出生的。", "options": [], "subject": "组合数学", "analysis": "答案:$\\sqrt{ }$"} {"id": "27073", "image": [], "answer": "$\\sqrt{ }$", "solution": "null", "level": "六年级", "question": " 把 7 本书放进 3 个抽屉, 不管怎么放, 总有一个抽屉里至少放进 3 本书。", "options": [], "subject": "组合数学", "analysis": "$\\sqrt{ }$"} {"id": "27074", "image": [], "answer": "$4 ; 11$", "solution": "null", "level": "六年级", "question": " 有红、黄、蓝三种颜色的小球各 5 个, 放入一个布袋里。至少\n\n取 $\\qquad$个球,可以保证取到两个颜色相同的球; 至少取出 $\\qquad$个球, 可以保证取出的球中一定有黄色的球。", "options": [], "subject": "组合数学", "analysis": "$4 ; 11$"} {"id": "27075", "image": [], "answer": "2", "solution": "null", "level": "六年级", "question": " 从扑克牌中取出两张王牌,在剩下的 52 张中任意抽出 5 张, 至少有 $\\qquad$张是同花色的。", "options": [], "subject": "组合数学", "analysis": "2"} {"id": "27076", "image": [], "answer": "$\\frac{2}{5} ; 12$", "solution": "null", "level": "六年级", "question": " 袋中有 4 个红球、 5 个黄球和 6 个黑球,那么,任意摸出 1 个球,摸到黑球的可能性是 $\\qquad$ ,至少摸出 $\\qquad$个球, 才能保证有一个是红球。", "options": [], "subject": "组合数学", "analysis": "$\\frac{2}{5} ; 12$"} {"id": "27077", "image": [], "answer": "2", "solution": "null", "level": "六年级", "question": " 生日在 3 月份的任意 32 名同学中, 至少有 $\\qquad$人的生日是同一天。", "options": [], "subject": "组合数学", "analysis": "2"} {"id": "27078", "image": [], "answer": "2", "solution": "null", "level": "六年级", "question": " 要给一个正方体木块的 6 个面涂上三种不同的颜色, 至少 $\\qquad$个有面颜色相同。", "options": [], "subject": "组合数学", "analysis": "2"} {"id": "13157", "image": [], "answer": "答案: 24,20", "solution": "null", "level": "六年级", "question": "用 50 粒大豆做发芽试验, 有 2 粒没发芽, 大豆的发芽率是 ( )。", "options": [], "subject": "计数", "analysis": "答案: 24,20"} {"id": "13083", "image": [], "answer": "答案: 正确", "solution": "null", "level": "六年级", "question": "某班有学生 50 人, 今天有 2 人请假, 该班今天的出勤率是 $96 \\%$. ( )", "options": [], "subject": "计数", "analysis": "答案: 正确"} {"id": "13141", "image": [], "answer": "答案:\\times$", "solution": "null", "level": "六年级", "question": "兴趣小组做发芽实验, 浸泡了 20 粒种子, 结果 16 课发芽了, 发芽率是 $16 \\%$ 。 $\\quad(\\quad)$", "options": [], "subject": "计数", "analysis": "答案:\\times$"} {"id": "13143", "image": ["2753.jpg", "2753.jpg"], "answer": "答案: $\\times$22.答案: $\\times$\n\n\n\n", "solution": "null", "level": "六年级", "question": "联合国在调查 200 个国家中,发现缺水的国家有 100 个,严重缺水的国家有 40 个, 严重缺水的国家占调查国家的 $40 \\%$ 。()", "options": [], "subject": "计数", "analysis": "答案: $\\times$22.答案: $\\times$\n\n\n\n"} {"id": "27080", "image": ["13243.jpg"], "answer": "5", "solution": "null", "level": "六年级", "question": " 将 18 枚棋子放入下图的 4 个小方格中, 那么一定有一个小方格内至少放 $\\qquad$枚棋子。\n\n", "options": [], "subject": "计数", "analysis": "5"} {"id": "27086", "image": [], "answer": "30", "solution": "null", "level": "六年级", "question": " 育才小学六年级共有学生 356 人, 至少有 $\\qquad$名学生是同月出生的。", "options": [], "subject": "计数", "analysis": "30"} {"id": "27117", "image": ["13259.jpg", "13260.jpg", "13261.jpg", "13262.jpg", "13263.jpg", "13264.jpg", "13265.jpg", "13266.jpg", "13263.jpg", "13264.jpg", "13265.jpg", "13266.jpg"], "answer": "\n\n石家庄\n\n$-6^{\\circ} \\mathrm{C}$\n\n\n\n武汉\n\n$0^{\\circ} \\mathrm{C}$\n\n\n\n厦门\n\n$11^{\\circ} \\mathrm{C}$\n\n\n\n北京\n\n$-4^{\\circ} \\mathrm{C}$", "solution": "null", "level": "六年级", "question": " (4 分) 下面是我国几个城市某年一月份的日均最低气温, 你能在温度计上涂一涂表示出这些气温吗?\n\n\n石家庄\n$-6^{\\circ} \\mathrm{C}$\n\n\n武汉\n\n\n厦门\n\n\n北京\n$-4^{\\circ} \\mathrm{C}$\n$11^{\\circ} \\mathrm{C}$", "options": [], "subject": "计数", "analysis": "\n\n石家庄\n\n$-6^{\\circ} \\mathrm{C}$\n\n\n\n武汉\n\n$0^{\\circ} \\mathrm{C}$\n\n\n\n厦门\n\n$11^{\\circ} \\mathrm{C}$\n\n\n\n北京\n\n$-4^{\\circ} \\mathrm{C}$"} {"id": "27228", "image": [], "answer": "$1 ; 3 ; 4$", "solution": "null", "level": "六年级", "question": " 盒子中装有 8 个红球、 8 个黑球和 8 个白球, 任意摸出 1 个球, 是红球的可能性是 $\\qquad$ ;\n\n至少摸出 $\\qquad$个球, 就可以保证其中至少有 2 个球的颜色相同。", "options": [], "subject": "计数", "analysis": "$1 ; 3 ; 4$"} {"id": "27230", "image": [], "answer": "红; 6", "solution": "null", "level": "六年级", "question": " 盒子里有同样大小的 5 个红球, 4 个白球。任意摸一个球, 摸出 $\\qquad$球的可能性大。\n如果保证至少要摸出一个白球, 至少要摸 $\\qquad$个球。", "options": [], "subject": "计数", "analysis": "红; 6"} {"id": "27231", "image": [], "answer": "2", "solution": "null", "level": "六年级", "question": " 6 个小朋友乘 5 只小船游玩, 至少要有 $\\qquad$个小朋友坐在同一只小船里。", "options": [], "subject": "计数", "analysis": "2"} {"id": "27238", "image": [], "answer": "$2 ; 2$", "solution": "null", "level": "六年级", "question": " 某小区 2019 年共新增加了 13 辆电动清洁能源小客车, 一定有 $\\qquad$辆或 $\\qquad$辆以上的小客车是在同一个月内购买的。", "options": [], "subject": "计数", "analysis": "$2 ; 2$"} {"id": "26185", "image": [], "answer": "1", "solution": "null", "level": "六年级", "question": " 六(1)班学生少于 50 人, 在一次考试中, $\\frac{1}{7}$ 学生得优, $\\frac{1}{3}$ 学生得良, $\\frac{1}{2}$ 学生及格,剩下的学生不及格, 那么有( )人不及格。", "options": [], "subject": "计数", "analysis": "1"} {"id": "26202", "image": [], "answer": "$\\quad 96 \\%$", "solution": "null", "level": "六年级", "question": " 六(1)班今天出勤 48 人,有 2 人请假,六(1)班这一天的出勤率是()。", "options": [], "subject": "计数", "analysis": "$\\quad 96 \\%$"} {"id": "26207", "image": [], "answer": "近视人数 总人数", "solution": "null", "level": "六年级", "question": " 某校六年级学生的近视率为 $18 \\%$, 就是说() 占 ()的 $\\frac{18}{100}$ 。", "options": [], "subject": "计数", "analysis": "近视人数 总人数"} {"id": "26211", "image": [], "answer": "条形 折线扇形", "solution": "null", "level": "六年级", "question": " ()统计图能很容易地看出各种数量的多少;统计图能清楚地表示出数量的增减变化。如果要更清楚地了解各部分数量同总数之间的关系,则用()统计图表示。", "options": [], "subject": "计数", "analysis": "条形 折线扇形"} {"id": "26243", "image": [], "answer": "98 %", "solution": "null", "level": "六年级", "question": "一批零件, 不合格产品数是合格产品数的 $\\frac{1}{49}$, 这批零件的合格率是()。", "options": [], "subject": "计数", "analysis": "98 %"} {"id": "26248", "image": [], "answer": "总数 每个扇形", "solution": "null", "level": "六年级", "question": " 扇形统计图用整个圆表示( ), 用( 的大小表示各部分占总数的百分数。", "options": [], "subject": "计数", "analysis": "总数 每个扇形"} {"id": "26256", "image": [], "answer": "(1)6\n(2) 36\n(3) 48\n(4)30", "solution": "null", "level": "六年级", "question": "六年级有 120 人参加了学校的 4 个课外小组。(每人只参加一个小组)\n\n(1)参加美术小组的有()人;\n\n(2)参加歌咏小组的有( )人;\n\n(3)参加书法小组的有( )人;\n\n(4)参加科技小组的有( )人;", "options": [], "subject": "计数", "analysis": "(1)6\n(2) 36\n(3) 48\n(4)30"} {"id": "26257", "image": [], "answer": "(1)60 45\n\n(2) 1365", "solution": "null", "level": "六年级", "question": "有 $300 \\mathrm{~m}^{2}$ 的菜地, 4 种蔬菜的种植面积分布情况如右图。\n\n(1)油菜的种植面积是 ( ) m², 芹菜的种植面积是 $(\\quad) \\mathrm{m}^{2}$ 。\n\n(2)如果黄瓜和西红柿平均每平方米的产量都是 $7 \\mathrm{~kg}$, 黄瓜和西红柿一共能产 ( ) kg。", "options": [], "subject": "计数", "analysis": "(1)60 45\n\n(2) 1365"} {"id": "13159", "image": [], "answer": "答案: $\\times$", "solution": "null", "level": "六年级", "question": "一件商品打八折出售, 就是便宜 $80 \\%$ 。( )", "options": [], "subject": "算术", "analysis": "答案: $\\times$"} {"id": "13067", "image": [], "answer": "答案: $3 ; 20 ; 75 ;", "solution": "null", "level": "六年级", "question": "七五折 $=$ \\$ \\qquad \\$ $: 4=15 \\div$ \\$ \\qquad \\$ $=\\square=$ \\$ \\qquad \\$ (填小数)", "options": [], "subject": "算术", "analysis": "答案: $3 ; 20 ; 75 ;"} {"id": "13068", "image": [], "answer": "答案: 16", "solution": "null", "level": "六年级", "question": "一件儿童毛衣原价 80 元, 现在打八折出售, 现价比原价便宜 \\$ \\qquad \\$元。", "options": [], "subject": "算术", "analysis": "答案: 16"} {"id": "13069", "image": [], "answer": "答案: 10420", "solution": "null", "level": "六年级", "question": "李伯伯把 10000 元钱存入银行, 存 2 年, 到期后, 李伯伯可以拿回 \\$ \\qquad \\$钱。\n\n| 存期 | 一年 | 二年 | 三年 |\n| :--- | :--- | :--- | :--- |\n| 年利率 $(\\%)$ |", "options": [], "subject": "算术", "analysis": "答案: 10420"} {"id": "13070", "image": [], "answer": "答案:二", "solution": "null", "level": "六年级", "question": "春节期间, 原价 100 元 / 件的某商品按以下两种方式促销: 第一种方式: 减价 20 元后再打八折;第二种方式: 打八折后再减价 20 元. 那么, 能使消费者少花钱的方式是第 \\$ \\qquad \\$种。", "options": [], "subject": "算术", "analysis": "答案:二"} {"id": "13071", "image": [], "answer": "$\frac{4.5}{10} ; 45 \\%$", "solution": "null", "level": "六年级", "question": "四成五就是 \\$ \\qquad \\$ , 改写成百分数就是 \\$ \\qquad \\$一.", "options": [], "subject": "算术", "analysis": "$\frac{4.5}{10} ; 45 \\%$"} {"id": "13081", "image": [], "answer": "答案:错误", "solution": "null", "level": "六年级", "question": "丹丹把 500 元存入银行, 定期一年, 年利率是 $", "options": [], "subject": "算术", "analysis": "答案:错误"} {"id": "13082", "image": [], "answer": "答案:错误", "solution": "null", "level": "六年级", "question": "一件商品先提高 $20 \\%$ 的价格后打八折出售, 这件商品的价格不变。()", "options": [], "subject": "算术", "analysis": "答案:错误"} {"id": "13085", "image": [], "answer": "答案:错误", "solution": "null", "level": "六年级", "question": "整数可以分成正整数和负整数。( )", "options": [], "subject": "算术", "analysis": "答案:错误"} {"id": "13086", "image": [], "answer": "答案:减", "solution": "null", "level": "六年级", "question": "写出与下面量具有相反意义的量.\n\n在环保知识竟赛中, 甲队加 10 分.\n\n10 分", "options": [], "subject": "算术", "analysis": "答案:减"} {"id": "13088", "image": [], "answer": "向西 $80 \\mathrm{~m} ; 20$", "solution": "null", "level": "六年级", "question": "如果把小明向东走 $100 \\mathrm{~m}$ 记作 $+100 \\mathrm{~m}$, 那么他又走- $80 \\mathrm{~m}$, 表示小明 \\$ \\qquad \\$ ,这时他距离出发点有 $\\mathrm{m}$ 远。", "options": [], "subject": "算术", "analysis": ""} {"id": "13089", "image": [], "answer": "答案: -1700", "solution": "null", "level": "六年级", "question": "张大伯玩股票, 上月赚了 3800 元, 表示为+3800 元, 本月亏了 1700 元, 表示为 \\$ \\qquad \\$元。", "options": [], "subject": "算术", "analysis": "答案: -1700"} {"id": "13091", "image": [], "answer": "答案:向西走 60 米", "solution": "null", "level": "六年级", "question": "如果 60 米表示向东走 60 米, 那么 -60 米表示", "options": [], "subject": "算术", "analysis": "答案:向西走 60 米"} {"id": "13092", "image": [], "answer": "答案:下车 8 位乘客", "solution": "null", "level": "六年级", "question": "如果上车的 10 位乘客用 +10 表示, 那么-8 表示", "options": [], "subject": "算术", "analysis": "答案:下车 8 位乘客"} {"id": "13103", "image": [], "answer": "答案: 不成", "solution": "null", "level": "六年级", "question": "圆的周长一定, 圆周率和直径成 \\$ \\qquad \\$比例。", "options": [], "subject": "算术", "analysis": "答案: 不成"} {"id": "13110", "image": [], "answer": "答案: 24", "solution": "null", "level": "六年级", "question": "一个比例的两个内项分别是 3 和 8 . 那么这个比例的两个外项的积是 \\$ \\qquad \\$ -", "options": [], "subject": "算术", "analysis": "答案: 24"} {"id": "13137", "image": [], "answer": "$20, \frac{1}{3}$", "solution": "null", "level": "六年级", "question": "爷爷养了 20 只公鸡, 30 只母鸡, 公鸡只数是母鸡的 $\\frac{(\\quad)}{(\\quad)}$, 母鸡只数比公鸡多()%。", "options": [], "subject": "算术", "analysis": ""} {"id": "13138", "image": [], "answer": "280", "solution": "null", "level": "六年级", "question": "学校有图书 630 本。按 $2: 3: 4$ 的比例分给三四五年级, 五年级分到()本。", "options": [], "subject": "算术", "analysis": ""} {"id": "13152", "image": [], "answer": "答案: 970200000 九亿七千零荎拾万 10", "solution": "null", "level": "六年级", "question": "9 个亿. 7 个千万. 2 个十万组成的数写作(),读作 ),四舍五入到亿位约是()亿。", "options": [], "subject": "算术", "analysis": "答案: 970200000 九亿七千零荎拾万 10"} {"id": "13153", "image": [], "answer": "答案: $16+\\mathrm{a}+\\mathrm{b}$", "solution": "null", "level": "六年级", "question": "小林今年 16 岁,父亲的年龄比他大 a 岁,再过 $\\mathrm{b}$ 年,父亲的年龄是 $(\\quad)$ 岁。", "options": [], "subject": "算术", "analysis": "答案: $16+\\mathrm{a}+\\mathrm{b}$"} {"id": "26953", "image": [], "answer": "$\\times$", "solution": "null", "level": "六年级", "question": " 北京某天的气温是 $-3^{\\circ} \\mathrm{C}$ 到 $8^{\\circ} \\mathrm{C}$, 这天的温差是 $8^{\\circ} \\mathrm{C}$ 。", "options": [], "subject": "算术", "analysis": "$\\times$"} {"id": "26954", "image": [], "answer": "$+36 、 59 ;-4 、-18 、-290$", "solution": "null", "level": "六年级", "question": " 在 $-4 、+36 、-18 、 59 、 0 、-290$ 中, 正数有 $\\qquad$ , 负数有 $\\qquad$ -", "options": [], "subject": "算术", "analysis": "$+36 、 59 ;-4 、-18 、-290$"} {"id": "26955", "image": [], "answer": "-16", "solution": "null", "level": "六年级", "question": " 12 月某城市的最高气温是零上 $5^{\\circ} \\mathrm{C}$, 记作 $+5^{\\circ} \\mathrm{C}$, 最低气温是零下 $16^{\\circ} \\mathrm{C}$,记作 $\\qquad$ ${ }^{\\circ} \\mathrm{C}$ 。", "options": [], "subject": "算术", "analysis": "-16"} {"id": "26956", "image": [], "answer": "$-2 \\mathrm{~F}$", "solution": "null", "level": "六年级", "question": " 某商场地上 5 层表示为 $+5 \\mathrm{~F}$, 那么地下 2 层表示为 $\\qquad$。", "options": [], "subject": "算术", "analysis": "$-2 \\mathrm{~F}$"} {"id": "26957", "image": [], "answer": "西; 6", "solution": "null", "level": "六年级", "question": " 东西为两个相反方向, 如果 $+5 m$ 表示向东走 $5 \\mathrm{~m}$, 则 $-6 \\mathrm{~m}$ 表示向 $\\qquad$走 $\\qquad$ $\\mathrm{m}$ 。", "options": [], "subject": "算术", "analysis": "西; 6"} {"id": "26958", "image": [], "answer": "西; $55 ;-5$", "solution": "null", "level": "六年级", "question": " 红红向东走 102 米, 记作 +102 米, 那么-55 米表示红红向 $\\qquad$走\n\n了 $\\qquad$米。如果电梯上升 10 层记作 +10 层, 那么下降 5 层应记作 $\\qquad$层。", "options": [], "subject": "算术", "analysis": "西; $55 ;-5$"} {"id": "26959", "image": [], "answer": "逆时针转动 6 圈", "solution": "null", "level": "六年级", "question": " 转动转盘, 如果把顺时针转动 5 圈记作 +5 圈, 那么 -6 圈表示 $\\qquad$。", "options": [], "subject": "算术", "analysis": "逆时针转动 6 圈"} {"id": "26960", "image": [], "answer": "$-1200 ;-60$", "solution": "null", "level": "六年级", "question": " 如果妈妈领取工资 7500 元记作 +7500 元,那么给 “希望工程” 捐款 1200 元,可记作 $\\qquad$元,一艘潜艇所处高度是海拔-100 米,一条鲨鱼在潜艇上方 40 米处,鲨鱼所处位置是海拔 $\\qquad$米。", "options": [], "subject": "算术", "analysis": "$-1200 ;-60$"} {"id": "26961", "image": [], "answer": "$+5000 ;-2400$", "solution": "null", "level": "六年级", "question": " 妈妈 10 月 28 日存入 5000 元,在存折上应记作 $\\qquad$元,11 月 2 日取出 2400 元,在存折上应记作 $\\qquad$元。", "options": [], "subject": "算术", "analysis": "$+5000 ;-2400$"} {"id": "26962", "image": [], "answer": "$-5 ; 154.6$", "solution": "null", "level": "六年级", "question": " 实验小学六 (1) 班学生的平均身高是 153 厘米, 记为 “ 0 ”,那么小红身高 148 厘米,记作 $\\qquad$厘米; 小强身高记作 +1.6 厘米, 小强身高是 $\\qquad$厘米。", "options": [], "subject": "算术", "analysis": "$-5 ; 154.6$"} {"id": "26964", "image": [], "answer": "$+1.5 ; 1.5 ;-2$", "solution": "null", "level": "六年级", "question": " 如果河水的警戒水位记为 $0 \\mathrm{~m}$, 汛期水位高于警戒水位 $1.5 \\mathrm{~m}$, 记为 $\\qquad$ $\\mathrm{m}$, 或是 $\\qquad$ $\\mathrm{m}$, 旱季水位低于警戒水位 $2 \\mathrm{~m}$, 记\n\n为 $\\qquad$ m。", "options": [], "subject": "算术", "analysis": "$+1.5 ; 1.5 ;-2$"} {"id": "26965", "image": [], "answer": "17", "solution": "null", "level": "六年级", "question": " 唐山市区某日的气温显示为 $-5^{\\circ} \\mathrm{C}^{\\sim} 12^{\\circ} \\mathrm{C}$, 那么这一天内昼夜温差最大\n是 $\\qquad$ ${ }^{\\circ} \\mathrm{C}$ 。", "options": [], "subject": "算术", "analysis": "17"} {"id": "26966", "image": [], "answer": "$+7 \\mathrm{~m} ;-2 \\mathrm{~kg}$", "solution": "null", "level": "六年级", "question": " 如果下降 $5 \\mathrm{~m}$ 记作 $-5 \\mathrm{~m}$, 那么上升 $7 \\mathrm{~m}$ 记作 $\\qquad$ ; 如果体重增加 $2 \\mathrm{~kg}$ ,记作+2kg,那么体重减少 $2 \\mathrm{~kg}$ 记作 $\\qquad$。", "options": [], "subject": "算术", "analysis": "$+7 \\mathrm{~m} ;-2 \\mathrm{~kg}$"} {"id": "26975", "image": [], "answer": "$\\times$", "solution": "null", "level": "六年级", "question": " 所有的数不是正数、就是负数。", "options": [], "subject": "算术", "analysis": "$\\times$"} {"id": "26976", "image": [], "answer": "$\\sqrt{ }$", "solution": "null", "level": "六年级", "question": " 某水库的警戒水位是 $18 \\mathrm{~m}$, 如果把水位 $18.5 \\mathrm{~m}$ 记作 $+0.5 \\mathrm{~m}$, 则水位 $17.5 \\mathrm{~m}$记作 $-0.5 \\mathrm{~m}$ 。", "options": [], "subject": "算术", "analysis": "$\\sqrt{ }$"} {"id": "26978", "image": [], "answer": "$\\sqrt{ }$", "solution": "null", "level": "六年级", "question": " 没有最大的正数, 也没有最大的负数。", "options": [], "subject": "算术", "analysis": "$\\sqrt{ }$"} {"id": "26980", "image": [], "answer": "$\\sqrt{ }$", "solution": "null", "level": "六年级", "question": " 一种商品八折出售正好保本, 如果不打折, 那么会获得 $25 \\%$ 的利润。( )", "options": [], "subject": "算术", "analysis": "$\\sqrt{ }$"} {"id": "26981", "image": [], "answer": "原价; 40", "solution": "null", "level": "六年级", "question": " 一本书六折出售, 是把 $\\qquad$看作单位 “1”,现价比原价降低\n了 $\\qquad$ $\\%$ 。\n\n12 . $\\qquad$ $=$ $\\qquad$ $\\div 8=0.75=$ $\\qquad$ $\\%=$ $\\qquad$折。", "options": [], "subject": "算术", "analysis": "原价; 40"} {"id": "26982", "image": [], "answer": "100", "solution": "null", "level": "六年级", "question": " 一个毛线玩具按八折出售, 便宜了 20 元, 这个玩具原价是 $\\qquad$元。", "options": [], "subject": "算术", "analysis": "100"} {"id": "26983", "image": [], "answer": "24", "solution": "null", "level": "六年级", "question": " 一件衣服原价 120 元,按八折出售,便宜了 $\\qquad$元。", "options": [], "subject": "算术", "analysis": "24"} {"id": "26984", "image": [], "answer": "120", "solution": "null", "level": "六年级", "question": " “六一”期间, 某商场举行促销活动, 所有商品九折出售, 小丽买一件上衣花去了 108 元, 这件大衣原价是 $\\qquad$元。", "options": [], "subject": "算术", "analysis": "120"} {"id": "26985", "image": [], "answer": "21764", "solution": "null", "level": "六年级", "question": " 张阿姨将 2 万元存入温州银行, 年利率为 $2.94 \\%$, 存期为 3 年, 到期时张阿姨一共能取回 $\\qquad$元钱。", "options": [], "subject": "算术", "analysis": "21764"} {"id": "26986", "image": [], "answer": "八", "solution": "null", "level": "六年级", "question": " 腾达超市某品牌酸牛奶举行 “买四送一” 促销活动。妈妈一次购买了 5 瓶这种品牌的酸牛奶, 那么, 每瓶相当于 $\\qquad$折优惠。", "options": [], "subject": "算术", "analysis": "八"} {"id": "26987", "image": [], "answer": "七五; 买四赠一", "solution": "null", "level": "六年级", "question": " 开学时, 小华学校附近文具店实行买三赠一促销, 实际上是打 $\\qquad$折出售, 如果实行 $\\qquad$促销, 就是打八折出售。", "options": [], "subject": "算术", "analysis": "七五; 买四赠一"} {"id": "26988", "image": [], "answer": "$160 ; 80$", "solution": "null", "level": "六年级", "question": " 一件衣服原价 200 元, 打八折出售, 现价 $\\qquad$元,妈妈买 2 件这样的衣服, 比原来便宜 $\\qquad$元。", "options": [], "subject": "算术", "analysis": "$160 ; 80$"} {"id": "26990", "image": [], "answer": "90", "solution": "null", "level": "六年级", "question": " 王阿姨月工资 8000 元,扣除 5000 元个税免征额后的部分需按 $3 \\%$ 的税率缴纳个人所得税,她应缴纳个人所得税 $\\qquad$元。", "options": [], "subject": "算术", "analysis": "90"} {"id": "26991", "image": [], "answer": "去年收成; 116", "solution": "null", "level": "六年级", "question": " 今年的收成比去年增加了一成六, 是把 $\\qquad$看作单位 “ 1 ” ,今年的收成是去年的 $\\qquad$ $\\%$ 。", "options": [], "subject": "算术", "analysis": "去年收成; 116"} {"id": "26992", "image": [], "answer": "54", "solution": "null", "level": "六年级", "question": " 小李每月的收入减去五险一金和个税专项附加扣除后是 6800 元, 这些收入再减去个人所得税的起征点 5000 元, 然后按照 $3 \\%$ 的税率计算。他这个月要缴纳个人所得税 $\\qquad$元。", "options": [], "subject": "算术", "analysis": "54"} {"id": "26993", "image": [], "answer": "| 小数 | 0.28 | 0.75 | 0.24 |\n| :--- | :--- | :--- | :--- |\n| 分数 | $\\frac{7}{25}$ | $\\frac{3}{4}$ | $\\frac{6}{25}$ |\n| 百分数 | $28 \\%$ | $75 \\%$ | $24 \\%$ |\n| 成数 | 二成八 | 七成五 | 二成四 |\n| 折扣 | 二八折 | 七五折 | 二四折 |", "solution": "null", "level": "六年级", "question": " 把表格填写完整。\n\n| 小数 | | | 0.24 |\n| :--- | :--- | :--- | :--- |\n| 分数 | | $\\frac{3}{4}$ | |\n| 百分数 | $28 \\%$ | | |\n| 成数 | | | |\n| 折扣 | | | |", "options": [], "subject": "算术", "analysis": "| 小数 | 0.28 | 0.75 | 0.24 |\n| :--- | :--- | :--- | :--- |\n| 分数 | $\\frac{7}{25}$ | $\\frac{3}{4}$ | $\\frac{6}{25}$ |\n| 百分数 | $28 \\%$ | $75 \\%$ | $24 \\%$ |\n| 成数 | 二成八 | 七成五 | 二成四 |\n| 折扣 | 二八折 | 七五折 | 二四折 |"} {"id": "27003", "image": [], "answer": "$\\sqrt{ }$", "solution": "null", "level": "六年级", "question": " 一种商品打七五折出售, 就是现价比原价降低了 $25 \\%$ 。", "options": [], "subject": "算术", "analysis": "$\\sqrt{ }$"} {"id": "27004", "image": [], "answer": "$\\times$", "solution": "null", "level": "六年级", "question": " 购买一辆 12 万元的汽车, 按规定缴纳 $10 \\%$ 的车辆购置税, 应缴纳 1200 元。", "options": [], "subject": "算术", "analysis": "$\\times$"} {"id": "27005", "image": [], "answer": "$\\sqrt{ }$", "solution": "null", "level": "六年级", "question": " 一本书原价 42 元, 如果按九折出售, 现价比原价便宜了 4.2 元。( )", "options": [], "subject": "算术", "analysis": "$\\sqrt{ }$"} {"id": "27006", "image": [], "answer": "$\\sqrt{ }$", "solution": "null", "level": "六年级", "question": " 一种商品先打六五折, 再提价 $\\frac{1}{2}$, 则现价比原价相低。", "options": [], "subject": "算术", "analysis": "$\\sqrt{ }$"} {"id": "27046", "image": [], "answer": "$\\frac{5}{2}$", "solution": "null", "level": "六年级", "question": " 在一个比例里, 两个外项的积是 $\\frac{5}{8}$, 其中一个内项是 $\\frac{1}{4}$, 另一个内项是 $\\qquad$。", "options": [], "subject": "算术", "analysis": "$\\frac{5}{2}$"} {"id": "27047", "image": [], "answer": "$12 : 2=6 : 1$", "solution": "null", "level": "六年级", "question": " 用 12 的因数组成一个比值最大的比例式是 $\\qquad$。", "options": [], "subject": "算术", "analysis": "$12 : 2=6 : 1$"} {"id": "27048", "image": [], "answer": "1、2、3、4、6、8、12、24; 2: $3=4: 6$ (最后一空答案不唯一)", "solution": "null", "level": "六年级", "question": " 24 的因数有 $\\qquad$ , 从中选择 4 个数, 其中 2 个是质数, 组成一个比例是 $\\qquad$。", "options": [], "subject": "算术", "analysis": "1、2、3、4、6、8、12、24; 2: $3=4: 6$ (最后一空答案不唯一)"} {"id": "27050", "image": [], "answer": "正; 8;$\\frac{8}{5}$", "solution": "null", "level": "六年级", "question": " 已知 $4 a=5 b ( a 、 b$ 不为 $0 ), a$ 和 $b$ 成 $\\qquad$比例。如果 $a=2$, 则 $b=$ $\\qquad$。", "options": [], "subject": "算术", "analysis": "正; 8;$\\frac{8}{5}$"} {"id": "27056", "image": ["13829.jpg"], "answer": "正; 4; 5", "solution": "null", "level": "六年级", "question": " , x:\n\n$\\mathrm{y}=$ $\\qquad$ $:$ $\\qquad$。", "options": [], "subject": "算术", "analysis": "正; 4; 5"} {"id": "27058", "image": [], "answer": "(1) $x=\\frac{1}{4}$\n\n(2) $x=1$\n\n(3) $\\quad \\mathrm{x}=90$\n\n(4) $\\mathrm{x}=\\frac{1}{2}$", "solution": "null", "level": "六年级", "question": " 解比例。\n(1)\n$\\frac{5}{7}: x=\\frac{2}{3}: \\frac{7}{30}$\n(2)\n$\\frac{4}{15}: 16=x: 60$\n\n(3)\n\n$\\frac{x}{7}: 15=\\frac{2}{3}: \\frac{7}{9}$\n(4)\n\n$\\frac{5}{7}: x=\\frac{2}{3}: \\frac{7}{15}$", "options": [], "subject": "算术", "analysis": "(1) $x=\\frac{1}{4}$\n\n(2) $x=1$\n\n(3) $\\quad \\mathrm{x}=90$\n\n(4) $\\mathrm{x}=\\frac{1}{2}$"} {"id": "27069", "image": [], "answer": "$\\sqrt{ }$", "solution": "null", "level": "六年级", "question": " 出盐率一定, 盐的质量和海水质量成正比例。", "options": [], "subject": "算术", "analysis": "$\\sqrt{ }$"} {"id": "27070", "image": [], "answer": "$\\sqrt{ }$", "solution": "null", "level": "六年级", "question": " 一个比例的两个外项互为倒数, 那么两个内项也一定互为倒数。", "options": [], "subject": "算术", "analysis": "$\\sqrt{ }$"} {"id": "27103", "image": [], "answer": "$\\sqrt{ }$", "solution": "null", "level": "六年级", "question": " 大于 0 的数是正数, 小于 0 的数是负数。", "options": [], "subject": "算术", "analysis": "$\\sqrt{ }$"} {"id": "27104", "image": [], "answer": "$<$; $=$", "solution": "null", "level": "六年级", "question": " 在横线上填上“ $>$ \"“<”或“=”。\n$-10$ $\\qquad$ $-1$\n$11.11 \\div 0.3$ $\\qquad$ $111.1 \\div 3$", "options": [], "subject": "算术", "analysis": "$<$; $=$"} {"id": "27107", "image": [], "answer": "67", "solution": "null", "level": "六年级", "question": " 在学校运动会的跳绳测试中, 如果每分钟 70 个为合格, 小红每分钟跳 80 个, 记作 +10 个。\n小亮的成绩记作-3 个,他跳了 $\\qquad$个。", "options": [], "subject": "算术", "analysis": "67"} {"id": "27108", "image": [], "answer": "13", "solution": "null", "level": "六年级", "question": " 某日北京气温是 $-5^{\\circ} \\mathrm{C} \\sim 8^{\\circ} \\mathrm{C}$, 这天的最大温差是 $\\qquad$ ${ }^{\\circ} \\mathrm{C}$ 。", "options": [], "subject": "算术", "analysis": "13"} {"id": "27109", "image": [], "answer": "144", "solution": "null", "level": "六年级", "question": " 以小明身高 145 厘米为标准, 把小强身高记为 +3 , 小丽身高记为 -6 , 则三个小朋友的平均身高是 $\\qquad$厘米。", "options": [], "subject": "算术", "analysis": "144"} {"id": "27110", "image": [], "answer": "$-2^{\\circ} \\mathrm{C} ; 4^{\\circ} \\mathrm{C}$", "solution": "null", "level": "六年级", "question": " 某市 1 月份的平均气温是零下 $2^{\\circ} \\mathrm{C}$, 写作 $\\qquad$。 2 月份的平均气温比 1 月份升高了 $6^{\\circ} \\mathrm{C}$,该市 2 月份的平均气温是 $\\qquad$。", "options": [], "subject": "算术", "analysis": "$-2^{\\circ} \\mathrm{C} ; 4^{\\circ} \\mathrm{C}$"} {"id": "27111", "image": [], "answer": "$3^{\\circ} \\mathrm{C} ;-3^{\\circ} \\mathrm{C}$", "solution": "null", "level": "六年级", "question": " 某市 2022 年 2 月 5 日最高气温为 $3^{\\circ} \\mathrm{C}$, 记作 $\\qquad$ , 最低气温为零下 $3^{\\circ} \\mathrm{C}$, 记作 $\\qquad$。", "options": [], "subject": "算术", "analysis": "$3^{\\circ} \\mathrm{C} ;-3^{\\circ} \\mathrm{C}$"} {"id": "27112", "image": ["13253.jpg", "13254.jpg", "13255.jpg"], "answer": "-8 ; 不合适; 因为北京和巴黎相差 $12-5=7$ (小时),10-7=3(时)是晚上 3 点, 时间太晚。", "solution": "null", "level": "六年级", "question": " 如下图所示, 与北京时间比, 东京时间早 1 时, 记作 +1 时, 伦敦时间晚 8 时, 记作 $\\qquad$时。北京时间上午 10: 00, 张红想和巴黎的爸爸视频通话, 你认为合适吗? $\\qquad$ , 理由是 $\\qquad$\n\n\n伦軲\n\n\n巴然\n\n\n北京", "options": [], "subject": "算术", "analysis": "-8 ; 不合适; 因为北京和巴黎相差 $12-5=7$ (小时),10-7=3(时)是晚上 3 点, 时间太晚。"} {"id": "27114", "image": [], "answer": "$+1.2 ;-0.5$", "solution": "null", "level": "六年级", "question": " 如果某地防汛的警戒水位记作 0m, 正数表示高于警戒水位的水面高度, 那么高于警戒水位 $1.2 \\mathrm{~m}$, 记为 $\\qquad$ $\\mathrm{m}$, 低于警戒水位 $0.5 \\mathrm{~m}$, 记为 $\\qquad$ $\\mathrm{m}$ 。", "options": [], "subject": "算术", "analysis": "$+1.2 ;-0.5$"} {"id": "27115", "image": [], "answer": "$5 ;-3$", "solution": "null", "level": "六年级", "question": " 某地一天的最高气温是五摄氏度, 记作 $\\qquad$ ${ }^{\\circ} \\mathrm{C}$; 最低气温是零下三摄氏度, 记作 $\\qquad$ ${ }^{\\circ} \\mathrm{C}$ 。", "options": [], "subject": "算术", "analysis": "$5 ;-3$"} {"id": "27126", "image": [], "answer": "$\\times$", "solution": "null", "level": "六年级", "question": " 在-1 与 1 之间, 没有整数。", "options": [], "subject": "算术", "analysis": "$\\times$"} {"id": "27127", "image": [], "answer": "$\\times$", "solution": "null", "level": "六年级", "question": " 在 $6,-5,0.2,0,-9.6,30 \\%$ 中, 正数有 4 个。( )", "options": [], "subject": "算术", "analysis": "$\\times$"} {"id": "27128", "image": [], "answer": "$\\times$", "solution": "null", "level": "六年级", "question": " 气温升高 $3^{\\circ} \\mathrm{C}$, 记作 $+3^{\\circ} \\mathrm{C}$ : 降低 $2^{\\circ} \\mathrm{C}$, 则记作 $2^{\\circ} \\mathrm{C}$ 。( )", "options": [], "subject": "算术", "analysis": "$\\times$"} {"id": "27131", "image": [], "answer": "$\\sqrt{ }$", "solution": "null", "level": "六年级", "question": " “买三赠一”实际上是打七五折。()", "options": [], "subject": "算术", "analysis": "$\\sqrt{ }$"} {"id": "27132", "image": [], "answer": "$\\times$", "solution": "null", "level": "六年级", "question": " 本金与利息的比率叫做利率。()", "options": [], "subject": "算术", "analysis": "$\\times$"} {"id": "27133", "image": [], "answer": "$\\times$", "solution": "null", "level": "六年级", "question": " 李晓芳在 2012 年 7 月 6 日把 1500 元钱存入银行,存期二年,年利率是 3.75\\%。到期她一共可以取出 112.5 元。", "options": [], "subject": "算术", "analysis": "$\\times$"} {"id": "27134", "image": [], "answer": "$85 ; 75$", "solution": "null", "level": "六年级", "question": " 一种商品打八五折销售, “八五折”表示现价是原价的 $\\qquad$ $\\%$ 。如果这种商品的原价是 500 元,现在便宜了 $\\qquad$元。", "options": [], "subject": "算术", "analysis": "$85 ; 75$"} {"id": "27135", "image": [], "answer": "5412.5", "solution": "null", "level": "六年级", "question": " 小雪爸爸在 2022 年 4 月 10 日把 5000 元钱存入银行, 定期三年, 年利率为 $2.75 \\%$, 到期时\n爸爸可以取回本息共 $\\qquad$元。", "options": [], "subject": "算术", "analysis": "5412.5"} {"id": "27136", "image": [], "answer": "$18 ; 25 ; 10 ; 4$", "solution": "null", "level": "六年级", "question": " $\\qquad$ $\\div 24=$ 七成五 $=1-$ $\\qquad$ $\\%=$ $\\frac{12-3}{2+()}=3$ :", "options": [], "subject": "算术", "analysis": "$18 ; 25 ; 10 ; 4$"} {"id": "27137", "image": [], "answer": "九; 10", "solution": "null", "level": "六年级", "question": " 周末,妈妈带安安去看电影, 电影票原价 40 元,支付宝购买只需支付 36 元,从支付宝购票相当于打 $\\qquad$折, 比原价便宜 $\\qquad$ $\\%$ 。", "options": [], "subject": "算术", "analysis": "九; 10"} {"id": "27138", "image": [], "answer": "七; 30", "solution": "null", "level": "六年级", "question": " 一件商品原价是 100 元, 打折后卖 70 元, 这是打 $\\qquad$折, 比原价便宜了 $\\qquad$ $\\%$ 。", "options": [], "subject": "算术", "analysis": "七; 30"} {"id": "27139", "image": [], "answer": "90", "solution": "null", "level": "六年级", "question": " 妈妈将 2000 元存入中国银行, 存期二年, 年利率为 $2.25 \\%$, 那么到期时可得到利息 $\\qquad$元。", "options": [], "subject": "算术", "analysis": "90"} {"id": "27140", "image": [], "answer": "八", "solution": "null", "level": "六年级", "question": " 一本书现价 6.4 元比原价便宜 1.6 元, 这本书是打 $\\qquad$折出售。", "options": [], "subject": "算术", "analysis": "八"} {"id": "27142", "image": [], "answer": "4200", "solution": "null", "level": "六年级", "question": " 李阿姨得到一笔 5000 元的劳务费,按国家税法规定,其中 1000 元是免税的,其余部分要按 $20 \\%$ 的税率缴税。这笔劳务费交税后, 李阿姨最终能得到 $\\qquad$元。", "options": [], "subject": "算术", "analysis": "4200"} {"id": "27143", "image": [], "answer": "5038.75", "solution": "null", "level": "六年级", "question": " 大宝的奶奶将 5000 元存入银行, 存期是 6 个月, 年利率是 $1.55 \\%$, 到期后奶奶一共能取回 $\\qquad$元。", "options": [], "subject": "算术", "analysis": "5038.75"} {"id": "27144", "image": [], "answer": "$8000 ; 8400$", "solution": "null", "level": "六年级", "question": " 爸爸想买一台标价是 10000 元的电脑, 他对经理说: “八折可以吗?”爸爸希望这台电脑的售价是 $\\qquad$元。经理说: “你说的价再加 5\\%吧!”这样爸爸买这台电脑实际花了 $\\qquad$元。", "options": [], "subject": "算术", "analysis": "$8000 ; 8400$"} {"id": "27145", "image": [], "answer": "$80 ; 20$", "solution": "null", "level": "六年级", "question": " 一件毛衣打八折销售, 现价是原价的 $\\qquad$ $\\%$, 现价比原价降低了 $\\qquad$ $\\%$ 。", "options": [], "subject": "算术", "analysis": "$80 ; 20$"} {"id": "27146", "image": [], "answer": "3600", "solution": "null", "level": "六年级", "question": " 一家饭店三月份营业额中应纳税部分是 12 万元,如果按应纳税部分的 $3 \\%$ 缴纳增值税, 这家饭店三月份应缴纳 $\\qquad$元。", "options": [], "subject": "算术", "analysis": "3600"} {"id": "27147", "image": [], "answer": "90", "solution": "null", "level": "六年级", "question": " 小东将 2000 元压岁钱存入银行, 存期两年, 年利率为 $2.25 \\%$ 。到期时可得利息 $\\qquad$元。", "options": [], "subject": "算术", "analysis": "90"} {"id": "27161", "image": [], "answer": "$\\times$", "solution": "null", "level": "六年级", "question": " 某种商品先提价 $10 \\%$, 又打九折销售, 现价与原价相等。()", "options": [], "subject": "算术", "analysis": "$\\times$"} {"id": "27163", "image": [], "answer": "$\\times$", "solution": "null", "level": "六年级", "question": " 某商品先打七折再提价 $30 \\%$, 结果与原价相同。()", "options": [], "subject": "算术", "analysis": "$\\times$"} {"id": "27196", "image": [], "answer": "$\\times$", "solution": "null", "level": "六年级", "question": " 比例 5: 3=15: 9 的内项 3 增加 6, 要使比例成立, 外项 9 应该增加 6。", "options": [], "subject": "算术", "analysis": "$\\times$"} {"id": "27207", "image": [], "answer": "$9 ; 10$", "solution": "null", "level": "六年级", "question": " 有两堆煤, 甲堆煤的质量的 $\\frac{2}{3}$ 正好等于乙堆煤质量的 $\\frac{3}{5}$, 甲乙两堆煤的质量比是 $\\qquad$ $:$ $\\qquad$ )", "options": [], "subject": "算术", "analysis": "$9 ; 10$"} {"id": "27208", "image": [], "answer": "正", "solution": "null", "level": "六年级", "question": " 如果 $4 n=5 m$, 那么 $n$ 和 $m$ 成 $\\qquad$比例。", "options": [], "subject": "算术", "analysis": "正"} {"id": "27221", "image": [], "answer": "$\\sqrt{ }$", "solution": "null", "level": "六年级", "question": " 小麦每公顷产量一定, 小麦的总产量与公顷数成正比例关系。()", "options": [], "subject": "算术", "analysis": "$\\sqrt{ }$"} {"id": "27223", "image": [], "answer": "$\\sqrt{ }$", "solution": "null", "level": "六年级", "question": " $15 : 5$ 和 $1.2: 0.4$ 能组成比例。()", "options": [], "subject": "算术", "analysis": "$\\sqrt{ }$"} {"id": "27255", "image": [], "answer": "$\\sqrt{ }$", "solution": "null", "level": "六年级", "question": " 张叔叔参加飞镖比赛, 投了 4 镖, 总成绩是 33 环, 且每一镖的成绩都是整数环。张叔叔至少有一镖不低于 9 环。", "options": [], "subject": "算术", "analysis": "$\\sqrt{ }$"} {"id": "26076", "image": [], "answer": "$\\begin{array}{lllll}\\frac{3}{8} & 4 & \\frac{3}{2}\\end{array} $", "solution": "null", "level": "六年级", "question": " $\\frac{3}{8}+\\frac{3}{8}+\\frac{3}{8}+\\frac{3}{8}=(\\quad) \\times(\\quad)=(\\quad)$", "options": [], "subject": "算术", "analysis": "$\\begin{array}{lllll}\\frac{3}{8} & 4 & \\frac{3}{2}\\end{array} $"} {"id": "26077", "image": [], "answer": "v", "solution": "null", "level": "六年级", "question": " $1 \\mathrm{t}$ 的 $\\frac{4}{5}$ 和 $4 \\mathrm{t}$ 的 $\\frac{1}{5}$ 一样重。", "options": [], "subject": "算术", "analysis": "v"} {"id": "26078", "image": [], "answer": "x", "solution": "null", "level": "六年级", "question": " 一件商品先降价 $\\frac{1}{5}$, 再涨价 $\\frac{1}{5}$, 现价和原价相等。", "options": [], "subject": "算术", "analysis": "x"} {"id": "26079", "image": [], "answer": "v", "solution": "null", "level": "六年级", "question": " 求 $\\frac{1}{2}$ 公顷的 $\\frac{3}{5}$ 是多少,就是把 $\\frac{1}{2}$ 公顷平均分成 5 份, 取其中的 3 份。也就是把 1 公顷平均分成 (2x5) 份, 取其中的 $(1 \\times 3)$ 份。", "options": [], "subject": "算术", "analysis": "v"} {"id": "26080", "image": [], "answer": "x", "solution": "null", "level": "六年级", "question": " 男生比女生多 $\\frac{1}{5}$, 那么女生就比男生少 $\\frac{4}{5}$ 。()", "options": [], "subject": "算术", "analysis": "x"} {"id": "26087", "image": [], "answer": "$10 $ 16", "solution": "null", "level": "六年级", "question": " 12 个 $\\frac{5}{6}$ 是 ( ) ; 24 的 $\\frac{2}{3}$ 是 ( )。", "options": [], "subject": "算术", "analysis": "$10 $ 16"} {"id": "26099", "image": [], "answer": "$>=<$", "solution": "null", "level": "六年级", "question": " 在 $\\bigcirc$ 里填上 “ $>$ ” “ $<$ ”或 “ $=$ ”。\n\n$\\frac{5}{6} \\times 4 \\bigcirc \\frac{5}{6} \\quad 9 \\times \\frac{2}{3} \\bigcirc \\frac{2}{3} \\times 9 \\quad \\frac{3}{8} \\times \\frac{1}{2} \\bigcirc \\frac{3}{8}$", "options": [], "subject": "算术", "analysis": "$>=<$"} {"id": "26100", "image": [], "answer": "$2 $$ \\quad \\frac{1}{4}$", "solution": "null", "level": "六年级", "question": " 边长为 $\\frac{1}{2} \\mathrm{dm}$ 的正方形周长是 ( ) dm, 面积是 ( ) $\\mathrm{dm}^{2}$ 。", "options": [], "subject": "算术", "analysis": "$2 $$ \\quad \\frac{1}{4}$"} {"id": "26101", "image": [], "answer": "$35 \\quad 9$", "solution": "null", "level": "六年级", "question": " 比 $30 \\mathrm{~m}$ 多 $\\frac{1}{6}$ 是 $(\\quad) \\mathrm{m}$; 比 $36 \\mathrm{~m}$ 少 $\\frac{3}{4}(\\quad) \\mathrm{m}$ 。", "options": [], "subject": "算术", "analysis": "$35 \\quad 9$"} {"id": "26102", "image": [], "answer": "$3 \\quad 3 \\frac{3}{4}$", "solution": "null", "level": "六年级", "question": " 一条绳子长 $4 \\mathrm{~m}$, 用去 $\\frac{1}{4}$, 还剩 ( ) m; 一条绳子长 $4 \\mathrm{~m}$, 用去 $\\frac{1}{4} \\mathrm{~m}$, 还剩 ( )m.", "options": [], "subject": "算术", "analysis": "$3 \\quad 3 \\frac{3}{4}$"} {"id": "26103", "image": [], "answer": "$\\frac{1}{5} \\quad 10 \\frac{2}{3}$", "solution": "null", "level": "六年级", "question": " 10 的 $\\frac{2}{5}$ 相当与 20 的 ( ); 比 $15 \\mathrm{~kg}$ 的的 $\\frac{2}{3}$ 多 $\\frac{2}{3} \\mathrm{~kg}$ 是() kg。", "options": [], "subject": "算术", "analysis": "$\\frac{1}{5} \\quad 10 \\frac{2}{3}$"} {"id": "26104", "image": [], "answer": "x", "solution": "null", "level": "六年级", "question": " 一个数乘分数, 积一定小于这个数。", "options": [], "subject": "算术", "analysis": "x"} {"id": "26105", "image": [], "answer": "$18 \\quad 32$", "solution": "null", "level": "六年级", "question": " 24 的 $\\frac{3}{4}$ 是 $\\left(\\right.$, ( ) 的 $\\frac{3}{4}$ 是 24 。", "options": [], "subject": "算术", "analysis": "$18 \\quad 32$"} {"id": "26111", "image": [], "answer": "$81 \\quad \\frac{1}{4}$", "solution": "null", "level": "六年级", "question": " 45 是 ( ) 的 $\\frac{5}{9},\\left(\\quad \\mathrm{~m}\\right.$ 是 $\\frac{3}{4} \\mathrm{~m}^{2}$ 的 $\\frac{1}{3}$ 。", "options": [], "subject": "算术", "analysis": "$81 \\quad \\frac{1}{4}$"} {"id": "26120", "image": [], "answer": "$\\frac{1}{4} \\quad \\frac{2}{9}$", "solution": "null", "level": "六年级", "question": " 把 $\\frac{8}{9} m$ 长的铁丝平均分成 4 段, 每段是全长的(),每段长()m。", "options": [], "subject": "算术", "analysis": "$\\frac{1}{4} \\quad \\frac{2}{9}$"} {"id": "26121", "image": [], "answer": "$\\frac{7}{12} \\quad \\frac{1}{2} \\quad \\frac{3}{4} \\quad \\frac{2}{5}$", "solution": "null", "level": "六年级", "question": "用分数表示。\n35 分 $=(\\quad)$ 时\n50 公顷 $=(\\quad) k^{2}$\n$75 \\mathrm{dm}^{2}=(\\quad) \\mathrm{m}^{2}$\n$400 \\mathrm{~mL}=(\\quad) \\mathrm{L}$", "options": [], "subject": "算术", "analysis": "$\\frac{7}{12} \\quad \\frac{1}{2} \\quad \\frac{3}{4} \\quad \\frac{2}{5}$"} {"id": "26122", "image": [], "answer": "$\\frac{1}{2} 4$", "solution": "null", "level": "六年级", "question": " 最小质数的倒数是( ), ( ) 和 $\\frac{1}{4}$ 互为倒数。", "options": [], "subject": "算术", "analysis": "$\\frac{1}{2} 4$"} {"id": "26123", "image": [], "answer": "足球 $\\frac{5}{4}$", "solution": "null", "level": "六年级", "question": " 篮球的数量比足球多 $\\frac{1}{4}$, 是把()的数量看作单位 “ 1 ”,篮球的数量是足球的 ( )", "options": [], "subject": "算术", "analysis": "足球 $\\frac{5}{4}$"} {"id": "26124", "image": [], "answer": "8", "solution": "null", "level": "六年级", "question": " 一桶油倒出的 $20 \\mathrm{~kg}$ 恰好占全桶油的 $\\frac{5}{7}$, 这桶油还剩下( )kg。", "options": [], "subject": "算术", "analysis": "8"} {"id": "26125", "image": [], "answer": "$\\frac{2}{3}$", "solution": "null", "level": "六年级", "question": " 一个数的 $\\frac{3}{4}$ 是 $\\frac{7}{8}$ 。,这个数的 $\\frac{4}{7}$ 是()。", "options": [], "subject": "算术", "analysis": "$\\frac{2}{3}$"} {"id": "26137", "image": [], "answer": "除以 $\\frac{1}{5}$", "solution": "null", "level": "六年级", "question": " 比的前项除以 $\\frac{1}{5}$, 要使比值不变, 比的后项应该 ( )", "options": [], "subject": "算术", "analysis": "除以 $\\frac{1}{5}$"} {"id": "13170", "image": [], "answer": "答案:0 .44 & 440000 &", "solution": "null", "level": "六年级", "question": "44 公顷 $=(\\quad)$ 平方千米 $=(\\quad)$ 平方米 4080 米 $=(\\quad)$ 千米 3 立方分米 40 立方厘米 $=(\\quad)$ 立方分米 $\\quad 3_{-}^{4}$ 吨 $=(\\quad)$ 吨 ( ) 千克", "options": [], "subject": "度量几何学", "analysis": "答案:0 .44 & 440000 &"} {"id": "13062", "image": [], "answer": "88$ 12", "solution": "null", "level": "六年级", "question": "大圆的直径是 4 厘米, 小圆的直径是 2 厘米, 大圆和小圆面积最简单的整数比是", "options": [], "subject": "度量几何学", "analysis": ""} {"id": "13107", "image": [], "answer": "答案: 90", "solution": "null", "level": "六年级", "question": "在比例尺是 1:600000 的地图上,量的甲、乙两地之间的距离是 15 厘米,甲乙两地的实际距离是 \\$ \\qquad \\$千米。", "options": [], "subject": "度量几何学", "analysis": "答案: 90"} {"id": "13140", "image": [], "answer": "向西走 50 米", "solution": "null", "level": "六年级", "question": "如果 $50 \\mathrm{~m}$ 表示向东走 50m,那么一50m 表示()。", "options": [], "subject": "度量几何学", "analysis": ""} {"id": "12488", "image": ["2664.jpg"], "answer": "答案:50.24", "solution": "null", "level": "六年级", "question": "一个圆形杯垫的直径是 $8 \\mathrm{~cm}$ ,这个杯垫的面积是 ) $\\mathrm{cm}^{2}$ 。", "options": [], "subject": "度量几何学", "analysis": ""} {"id": "12490", "image": [], "answer": "答案: $\\times$", "solution": "null", "level": "六年级", "question": "圆心角是 $60^{\\circ}$ 的扇形比圆心角是 $50^{\\circ}$ 的扇形面积大。", "options": [], "subject": "度量几何学", "analysis": "答案: $\\times$"} {"id": "12491", "image": [], "answer": "答案:$\\times$", "solution": "null", "level": "六年级", "question": "圆的直径与正方形边长相等,圆的周长大于正方形周长。 $\\left(\\_\\_\\right)$", "options": [], "subject": "度量几何学", "analysis": "答案:$\\times$"} {"id": "12493", "image": [], "answer": "答案:$x$", "solution": "null", "level": "六年级", "question": "大圆的周长是小圆周长的 2 倍,大圆的面积与小圆面积的比是 $2: 1$ 。 $\\left(\\_\\_\\right)$ _) )", "options": [], "subject": "度量几何学", "analysis": "答案:$x$"} {"id": "27009", "image": [], "answer": "48", "solution": "null", "level": "六年级", "question": " 把一个底面积为 24 平方厘米的圆柱锯成两段小圆柱,表面积增加了 $\\qquad$平方厘米。", "options": [], "subject": "度量几何学", "analysis": "48"} {"id": "27011", "image": [], "answer": "$50.24 ; 37680$", "solution": "null", "level": "六年级", "question": " 一个圆柱形水池, 量得底面周长是 25.12 米, 深 0.75 米。这个水池的占地面积是 $\\qquad$平方米,最多能蓄水 $\\qquad$升。", "options": [], "subject": "度量几何学", "analysis": "$50.24 ; 37680$"} {"id": "27043", "image": [], "answer": "60", "solution": "null", "level": "六年级", "question": " 在比例尺是 1: 500000 的地图上, 量得甲、乙两地的距离约为 12 厘米, 两地之间的实际距离大约是 $\\qquad$千米。", "options": [], "subject": "度量几何学", "analysis": "60"} {"id": "27045", "image": ["13240.jpg"], "answer": "东; $45 ; 120$", "solution": "null", "level": "六年级", "question": " 观察下图, 学校在小冬家北偏 $\\qquad$ $\\qquad$ )ㅇ的方向上, 距离小冬家约 $\\qquad$米。 (比例尺 1: 8000)\n\n", "options": [], "subject": "度量几何学", "analysis": "东; $45 ; 120$"} {"id": "27049", "image": [], "answer": "$1: 600 ; 180$", "solution": "null", "level": "六年级", "question": " 在一张校园平面图上, 量得长方形操场长 9 厘米, 宽 6 厘米, 而操场的实际宽为 36 米, 这张平面图的比例尺是 $\\qquad$ , 沿操场走一周, 要走 $\\qquad$米。", "options": [], "subject": "度量几何学", "analysis": "$1: 600 ; 180$"} {"id": "27051", "image": [], "answer": "336", "solution": "null", "level": "六年级", "question": " 在比例尺是 1: 30000000 地图上, 量得甲地到乙地的距离是 5.6 厘米, 一辆汽车按 $3: 2$ 的比分两天行完全程, 两天行的路程差是 $\\qquad$千米。", "options": [], "subject": "度量几何学", "analysis": "336"} {"id": "27054", "image": [], "answer": "比例; 正", "solution": "null", "level": "六年级", "question": " 用数学的眼光来看成语“立竿见影”, 是应用了本学期所学的 $\\qquad$知识。即同一时间,同一地点, 竿高和影长成 $\\qquad$比例。", "options": [], "subject": "度量几何学", "analysis": "比例; 正"} {"id": "27055", "image": [], "answer": "1: 25000000; 3550", "solution": "null", "level": "六年级", "question": " 一幅地图的线段比例尺是 $0 \\quad 250 \\quad 500 \\mathrm{~km}$, 改写成数值比例尺是 $\\qquad$ ,\n\n在这幅地图上量得 $\\mathrm{A}$ 地到 B 地的距离是 $14.2 \\mathrm{~cm}, \\mathrm{~A}$ 地到 B 地的实际距离是 $\\qquad$ $\\mathrm{km}$ 。", "options": [], "subject": "度量几何学", "analysis": "1: 25000000; 3550"} {"id": "27071", "image": [], "answer": "$\\sqrt{ }$", "solution": "null", "level": "六年级", "question": " 圆的周长和它的直径成正比例。", "options": [], "subject": "度量几何学", "analysis": "$\\sqrt{ }$"} {"id": "27105", "image": ["13252.jpg"], "answer": "$-1.5 ; 2.25$", "solution": "null", "level": "六年级", "question": " 如下图, 直线上 $\\mathrm{A}$ 表示的数是 $\\qquad$。如果 A 向东走 $\\qquad$米, 到达 B 的位置。\n\n", "options": [], "subject": "度量几何学", "analysis": "$-1.5 ; 2.25$"} {"id": "27116", "image": ["13256.jpg", "13257.jpg", "13258.jpg", "13257.jpg", "13258.jpg"], "answer": "(1)\n\n\n\n(2)\n\n\n\n(3) 12 ; 8", "solution": "null", "level": "六年级", "question": " (8 分)下面每格代表 $4 \\mathrm{~m}$, 小兔的起始位置在 0 点处。\n\n\n\n(1)小兔先向东跳 3 格到 $\\mathrm{A}$ 点, 在图上标出 $\\mathrm{A}$ 点。\n\n(2)小兔再从 $\\mathrm{A}$ 点向西跳了 $20 \\mathrm{~m}$ 到了 $\\mathrm{B}$ 点, 在图上标出 B 点。\n\n(3) $\\mathrm{A}$ 点和 $\\mathrm{B}$ 点离 0 点的距离分别是 $\\qquad$米和 $\\qquad$米。", "options": [], "subject": "度量几何学", "analysis": "(1)\n\n\n\n(2)\n\n\n\n(3) 12 ; 8"} {"id": "27129", "image": [], "answer": "$\\sqrt{ }$", "solution": "null", "level": "六年级", "question": " 所有的正数都比负数大, 在数轴上, 左边的数比右边的数小。( )", "options": [], "subject": "度量几何学", "analysis": "$\\sqrt{ }$"} {"id": "27174", "image": ["13276.jpg"], "answer": "$55 ; 12 ; 100.48$", "solution": "null", "level": "六年级", "question": " 下面是一个直角三角形, 已知 $\\angle 1$ 是 $35^{\\circ}$, 那么 $\\angle 2$ 是 $\\qquad$ ㅇ 这个三角形的面积是 $\\mathrm{cm}^{2}$ 。绕三角形中 $6 \\mathrm{~cm}$ 长的边旋转 1 周, 形成的立体图形的体积是 $\\qquad$ $\\mathrm{cm}^{3}$ 。\n\n", "options": [], "subject": "度量几何学", "analysis": "$55 ; 12 ; 100.48$"} {"id": "27177", "image": [], "answer": "122.46", "solution": "null", "level": "六年级", "question": " 用铁皮制作一个底面半径是 3 分米, 高是 5 分米的圆柱形水桶 (无盖), 至少要用铁皮 $\\mathrm{cm}$ 。", "options": [], "subject": "度量几何学", "analysis": "122.46"} {"id": "27178", "image": [], "answer": "5.652", "solution": "null", "level": "六年级", "question": " 将一个底面周长是 $9.42 \\mathrm{dm}$ 的圆柱形木料, 沿着底面直径垂直切一刀, 切成两个半圆柱, 表面积增加 $4.8 \\mathrm{dm}^{2}$, 原来这个圆柱形木料的体积是 $\\qquad$立方分米。", "options": [], "subject": "度量几何学", "analysis": "5.652"} {"id": "27199", "image": [], "answer": "正; $\\mathrm{x}$ 和 $\\mathrm{y}$ 的商一定", "solution": "null", "level": "六年级", "question": " $\\mathrm{AB}$ 两城间的铁路长 170 千米, 在一幅比例尺是 1:5000000 的地图上,这条铁路的图上距离是 $\\qquad$厘米。一列动车沿此铁路从 $\\mathrm{A}$ 城开往 $\\mathrm{B}$ 城, 所用的时间与行驶的平均速度成比例关系。", "options": [], "subject": "度量几何学", "analysis": "正; $\\mathrm{x}$ 和 $\\mathrm{y}$ 的商一定"} {"id": "27200", "image": [], "answer": "3.2; 反", "solution": "null", "level": "六年级", "question": " 在比例尺是 1:3000000 的地图上,量得 A、B 两地之间的距离是 5 厘米, A、B 两地实际相距 $\\qquad$千米。一辆轿车和一辆客车同时从 A 地出发开往 B 地, 轿车每小时行驶 60 千米,客车每小时行驶 45 千米, 当轿车到达 $\\mathrm{B}$ 地时, 客车距离 $\\mathrm{B}$ 地还有 $\\qquad$千米。\n\nA. 在比例尺是 1:3000000 的地图上,量得", "options": [], "subject": "度量几何学", "analysis": "3.2; 反"} {"id": "27201", "image": [], "answer": "1: $4000000 ; 80$", "solution": "null", "level": "六年级", "question": " 一幅地图上, 1.5 厘米长的线段表示 60 千米的实际距离, 这幅地图的比例尺是 $\\qquad$ ,在这幅地图上,甲、乙两地之间的图上距离是 2 厘米,那么甲、乙两地的实际距离是 $\\qquad$千米。", "options": [], "subject": "度量几何学", "analysis": "1: $4000000 ; 80$"} {"id": "27202", "image": [], "answer": "$810 ; 3$", "solution": "null", "level": "六年级", "question": " 郑万高铁(郑州-万州)预计今年全线通车, 人们出行越来越方便。在一幅比例尺是 1:10000000 的地图上, 量得郑万高铁的长度约是 8.1 厘米, 郑万高铁的实际长度是 $\\qquad$千米。若一列动车以 270 千米/时的速度从万州出发, $\\qquad$小时后可到达郑州。", "options": [], "subject": "度量几何学", "analysis": "$810 ; 3$"} {"id": "27204", "image": [], "answer": "135", "solution": "null", "level": "六年级", "question": " 把一个长 $5 \\mathrm{~cm}$ 、宽 $3 \\mathrm{~cm}$ 的长方形按 3: 1 放大,得到的图形的面积是 $\\qquad$ $\\mathrm{cm}^{2}$ 。", "options": [], "subject": "度量几何学", "analysis": "135"} {"id": "27206", "image": [], "answer": "反", "solution": "null", "level": "六年级", "question": " 面积一定, 平行四边形的底和高成 $\\qquad$比例。", "options": [], "subject": "度量几何学", "analysis": "反"} {"id": "27222", "image": [], "answer": "$\\times$", "solution": "null", "level": "六年级", "question": " 圆的半径与面积成正比例关系。( )", "options": [], "subject": "度量几何学", "analysis": "$\\times$"} {"id": "27224", "image": [], "answer": "$\\times$", "solution": "null", "level": "六年级", "question": " 在比例尺中, 图上距离都比实际距离小。()", "options": [], "subject": "度量几何学", "analysis": "$\\times$"} {"id": "26161", "image": [], "answer": "12.56\n 12.56", "solution": "null", "level": "六年级", "question": "圆规画圆时, 两脚间距离是 $2 \\mathrm{~cm}$, 画出的圆的周长是 ( ) cm, 面积是 $1 \\quad \\mathrm{~cm}^{2}$ 。", "options": [], "subject": "度量几何学", "analysis": "12.56\n 12.56"} {"id": "26165", "image": [], "answer": "半径\n 圆心", "solution": "null", "level": "六年级", "question": " ( ) 决定圆的大小, (决定圆的位置。", "options": [], "subject": "度量几何学", "analysis": "半径\n 圆心"} {"id": "26166", "image": [], "answer": "无数", "solution": "null", "level": "六年级", "question": "圆有()条对称轴。", "options": [], "subject": "度量几何学", "analysis": "无数"} {"id": "13048", "image": [], "answer": "32", "solution": "null", "level": "六年级", "question": "一管净含量为 100 立方厘米的牙膏, 它的圆形出口的直径是 1 厘米。如果早晚各刷牙一次, 每次挤出的牙膏长约 2 厘米。照这样计算, 这管牙膏估计能用 \\$ \\qquad \\$天。", "options": [], "subject": "组合几何学", "analysis": ""} {"id": "13100", "image": [], "answer": "答案:正确", "solution": "null", "level": "六年级", "question": "判断对错.\n\n今天最低气温是 $-2^{\\circ} \\mathrm{C}$, 明天最低气温是 $-4^{\\circ} \\mathrm{C}$, 所以明天比今天冷.", "options": [], "subject": "逻辑题", "analysis": "答案:正确"} {"id": "13101", "image": [], "answer": "错误", "solution": "null", "level": "六年级", "question": "一个数如果不是正数, 那么它就一定是负数。", "options": [], "subject": "逻辑题", "analysis": "错误"} {"id": "13105", "image": [], "answer": "答案:$\\frac{1}{1000} ; 10$", "solution": "null", "level": "六年级", "question": "比例尺 1:1000,表示图上距离是实际距离的 \\$ \\qquad \\$ , 根据比例尺,可以知道图上距离 1 厘米表示实际距离 \\$ \\qquad \\$米。", "options": [], "subject": "画法几何学", "analysis": "答案:$\\frac{1}{1000} ; 10$"} {"id": "13106", "image": [], "answer": "答案: $4: 1$", "solution": "null", "level": "六年级", "question": "有一个机器零件长 $5 \\mathrm{~mm}$, 画在设计图纸上长 $2 \\mathrm{~cm}$ 。这幅图的比例尺是 \\$ \\qquad \\$ $\\circ$", "options": [], "subject": "画法几何学", "analysis": "答案: $4: 1$"} {"id": "12499", "image": [], "answer": "答案:3", "solution": "null", "level": "六年级", "question": "要画一个直径是 6 厘米的圆,圆规两脚之间的距离是 ) 厘米.", "options": [], "subject": "画法几何学", "analysis": "答案:3"} {"id": "27026", "image": ["13229.jpg", "13230.jpg", "13231.jpg", "13231.jpg"], "answer": "(1)\n\n(2)\n\n", "solution": "null", "level": "六年级", "question": " 画出下面各圆雉的高。\n\n(1)\n\n\n\n(2)\n\n", "options": [], "subject": "画法几何学", "analysis": "(1)\n\n(2)\n\n"} {"id": "27044", "image": [], "answer": "6", "solution": "null", "level": "六年级", "question": " 一种精密零件的长是 5 毫米, 把它画在比例尺是 12: 1 的零件图上, 长应画 $\\qquad$厘米。", "options": [], "subject": "画法几何学", "analysis": "6"} {"id": "13109", "image": [], "answer": "答案: $2 ; 5$", "solution": "null", "level": "六年级", "question": "若 $5 x=2 y$, 则 $x: y=$ \\$ \\qquad \\$ \\$ \\qquad \\$。", "options": [], "subject": "代数", "analysis": "答案: $2 ; 5$"} {"id": "13121", "image": [], "answer": "答案:$\\times$", "solution": "null", "level": "六年级", "question": "若 $7 a=5 b$, 则 $a b$ 成反比例。", "options": [], "subject": "代数", "analysis": "答案:$\\times$"} {"id": "27053", "image": [], "answer": "正; 反", "solution": "null", "level": "六年级", "question": " 4: $a=3: b, a$ 和 $b$ 成 $\\qquad$比例。 $\\frac{3}{x}=4 y, x$ 和 $\\mathrm{y}$ 成 $\\qquad$比例。", "options": [], "subject": "代数", "analysis": "正; 反"} {"id": "27068", "image": [], "answer": "$\\sqrt{ }$", "solution": "null", "level": "六年级", "question": " 如果 $\\mathrm{a}-\\mathrm{b}=0$, 那么 $\\mathrm{a}$ 和 $\\mathrm{b}$ 成正比例。", "options": [], "subject": "代数", "analysis": "$\\sqrt{ }$"} {"id": "13119", "image": [], "answer": "答案:、", "solution": "null", "level": "六年级", "question": "一个图形放大或缩小后,大小改变,形状不变。", "options": [], "subject": "变换几何", "analysis": "答案:、"} {"id": "27057", "image": ["13241.jpg", "13242.jpg", "13242.jpg"], "answer": "", "solution": "null", "level": "六年级", "question": " 按 2:1的比画出三角形放大后的图形, 按 2: 3 的比画出正方形缩小后的图形。\n\n", "options": [], "subject": "变换几何", "analysis": ""} {"id": "12489", "image": [], "answer": "4", "solution": "null", "level": "六年级", "question": "把一个直径是 4 厘米的圆分成若干等份,然后把它剪开,照下图的样子拼起来,拼成的图形的周长比原来圆的周长增加厘米.", "options": [], "subject": "解析几何", "analysis": ""} {"id": "26977", "image": [], "answer": "$\\sqrt{ }$", "solution": "null", "level": "六年级", "question": " 在直线上 -1 在 +1 的左边。", "options": [], "subject": "解析几何", "analysis": "$\\sqrt{ }$"} {"id": "27037", "image": [], "answer": "$\\times$", "solution": "null", "level": "六年级", "question": " 把一个圆雉的侧面展开就能得到一个圆形。", "options": [], "subject": "解析几何", "analysis": "$\\times$"} {"id": "26188", "image": [], "answer": "(4,3)", "solution": "null", "level": "六年级", "question": " 音乐课上, 聪聪坐在音乐教室的第 4 列第 2 行, 用数对 $(4,2)$ 表示。明明坐在聪聪正后方的第一个位置上, 明明的位置用数对表示是( )。", "options": [], "subject": "解析几何", "analysis": "(4,3)"} {"id": "12553", "image": [], "answer": "答案:正确", "solution": "null", "level": "六年级", "question": "想要准确描述路线,既要确定方向,又要确定距离和途经的地方。", "options": [], "subject": "解析几何", "analysis": "答案:正确"} {"id": "12554", "image": [], "answer": "答案: $\\times$", "solution": "null", "level": "六年级", "question": "以不同学生的家为观测点,所描述的学校的位置是相同的.", "options": [], "subject": "解析几何", "analysis": "答案: $\\times$"} {"id": "25921", "image": ["13027.jpg"], "answer": "$X$", "solution": "null", "level": "六年级", "question": " 商店在中心广场的南偏西方向上, 距离中心广场 1000m。\n\n", "options": [], "subject": "解析几何", "analysis": "$X$"} {"id": "26805", "image": [], "answer": "可能", "solution": "null", "level": "六年级", "question": " “明天降雨的可能性是 $90 \\%$ \" 的意思是明天 $\\qquad$ )降雨。(填 “可能” 或 “不可能”)。", "options": [], "subject": "统计数学", "analysis": "可能"} {"id": "5687", "image": ["7142.jpg"], "answer": "18", "solution": "null", "level": "八年级", "question": "初 2018 级某班文娱委员, 对该班“肆月”学习小组同学购买不同单价的毕业照 (单位: 元) 情况进行了统计,绘制了如图所示的条形统计图,则所购毕业照平均每张的单价是 \\$ \\qquad \\$元.\n\n", "options": [], "subject": "计数", "analysis": "所购毕业照平均每张的单价是 $\\frac{10 \\times 5+15 \\times 1+20 \\times 4+25 \\times 5}{5+1+4+5}=18$ (元),\n\n故答案为: 18 ."} {"id": "5689", "image": [], "answer": "6", "solution": "null", "level": "八年级", "question": "一组数据: $3,8,6,7,6,5$ 的中位数是 \\$ \\qquad \\$", "options": [], "subject": "计数", "analysis": "把给出的此组数据中的数按从小到大的顺序排列为:3、5、6、6、7、8,\n\n最中间的两个数的平均数是: $(6+6) \\div 2=12 \\div 2=6$;\n\n故答案为: 6 ."} {"id": "5690", "image": [], "answer": "23", "solution": "null", "level": "八年级", "question": "武汉市某气象观测点记录了 5 天的平均气温(单位: ${ }^{\\circ} \\mathrm{C}$ ), 分别是 $25 、 20 、 18 、 23 、 27$, 这组数据的中位数是 \\$ \\qquad \\$", "options": [], "subject": "计数", "analysis": "排序为 $18 , 20 , 23 , 25,27$\n\n最中间的是 23\n\n$\\therefore$ 这组数据的中位数是 23\n\n故答案为: 23"} {"id": "5691", "image": ["7143.jpg", "7144.jpg"], "answer": "(1) $\\mathrm{B}$\n\n(2) 4", "solution": "null", "level": "八年级", "question": "有 $A 、 B$ 两个班级,每个班级各有 45 名学生参加一次测验. 每名参加者可获 $0,1,2,3,4,5,6,7,8,9$ 这几种不同的分值中的一种.测试结果 A 班的成绩如下表所示,B 班的成绩如右图所示.\n\n\n\nA 班\n\n\n\n(1) 由观察所得, \\$ \\qquad \\$班的方差大;\n\n(2) 若两班合计共有 60 人及格, 问参加者最少获 \\$ \\qquad \\$分才可以及格.", "options": [], "subject": "计数", "analysis": "(1)观察图象可知, B 班成绩分布集中,故可得 A 班的方差较大;\n\n(2)据统计表可知: 两个班的成绩从高到低排到 60 名时, 为 4 分;若两班合计共有 60 人及格, 参加者最少获 4 分才可以及格. 故答案为 B; 4 ."} {"id": "5695", "image": [], "answer": "4.8 或 5 或 5.2", "solution": "null", "level": "八年级", "question": "某学校九 (1) 班 40 名同学的期中测试成绩分别为 $a_{1}, a_{2} , a_{3} , \\ldots \\ldots, a_{40}$. 已知 $a_{1}+$ $a_{2}+a_{3}+\\ldots \\ldots+a_{40}=4800, \\mathrm{y}=\\left(a-a_{1}\\right)^{2}+\\left(a-a_{2}\\right)^{2}+\\left(a-a_{2}\\right)^{2}+\\ldots \\ldots+\\left(a-a_{40}\\right)^{2}$ ,当 $\\mathrm{y}$ 取最小值时, $\\quad a$ 的值为", "options": [], "subject": "计数", "analysis": "若 $\\mathrm{a}$ 为数据的中位数, 则数字排列在数据中第三个位置上。\n\n当 $a=3$ 时, 平均数为\n\n$$\n\\frac{1+3+3+5+12}{5}=4.8\n$$\n\n$1+3+4+5+12$\n\n当 $a=4$ 时, 平均数为 $5=5$\n\n$1+3+5+5+12$\n\n当 $a=5$ 时, 平均数为 $=5.2$\n\n故答案为: 4.8 或 5 或 5.2 。"} {"id": "5697", "image": [], "answer": "1.6", "solution": "null", "level": "八年级", "question": "已知一组数据 $4, \\mathrm{x}, 5, \\mathrm{y}, 7,9$ 的平均数为 6 , 众数为 5 , 则这组数据的中位数为 \\$ \\qquad \\$ .", "options": [], "subject": "计数", "analysis": "$0.1 \\times 4^{2}=1.6$."} {"id": "5806", "image": [], "answer": "1.35", "solution": "null", "level": "八年级", "question": "小明的爸爸是个“健步走”运动爱好者, 他用手机软件记录了某个月 (30 天) 每天健步走的步数,并将记录结果绘制成了如下统计表:\n\n| 步数 (万步) | 1.1 | 1.2 | 1.3 | 1.4 | 1.5 |\n| :--- | :--- | :--- | :--- | :--- | :--- | :--- | :--- |\n| 天数 | 3 | 7 | 5 | 12 | 3 |\n\n在每天所走的步数这组数据中, 中位数是 (万步)", "options": [], "subject": "计数", "analysis": "要求一组数据的中位数,\n\n把这组数据按照从小到大的顺序排列,第 $15 、 16$ 个两个数的平均数是 $(1.3+1.4) \\div 2=1.35$,所以中位数是 1.35 ,\n\n故答案为: 1.35 ."} {"id": "5688", "image": [], "answer": "乙", "solution": "null", "level": "八年级", "question": "某校欲招聘一名教师, 计划将面试成绩与笔试成绩按 6: 4 计算总分并择优录取. 下面是两名候选人的测试成绩,则该校应录取的是 \\$ \\qquad \\$ . (填“甲”或“乙”)\n\n| 侯选人 | | 甲 | 乙 |\n| :--- | :--- | :---: | :---: |\n| 测论成绩
(百分䏀) | 面试 | 86 | 92 |\n| | 笔试 | 90 | 83 |", "options": [], "subject": "算术", "analysis": "甲的成绩: $86 \\times 60 \\%+90 \\times 40 \\%=51.6+36=87.6$; 乙的成绩: $92 \\times 60 \\%+83 \\times 40 \\%=55.2+33.2=88.4$,根据成绩应录取乙。"} {"id": "5694", "image": ["7146.jpg"], "answer": "16", "solution": "null", "level": "八年级", "question": "已知: 一组自然数 $1,2,3 \\ldots \\mathrm{k}$, 去掉其中一个数后剩下的数的平均数为 16 , 则去掉的数是 \\$ \\qquad \\$ . ", "options": [], "subject": "算术", "analysis": "设去掉的数为 $\\mathrm{x}$,\n\n$\\because$ 一组自然数 $1,2,3 \\ldots \\mathrm{k}$, 去掉其中一个数后剩下的数的平均数为 16 ,\n\n\n\n$\\therefore \\mathrm{x}={ }^{\\frac{k(k+1)}{2}}-16(\\mathrm{k}-1)=^{\\frac{1}{2}}\\left(\\mathrm{k}^{2}-31 \\mathrm{k}+32\\right) \\leq \\mathrm{k}$,\n\n即: $\\mathrm{k}^{2}-33 \\mathrm{k}+32 \\leq 0$,\n\n解得: $1 \\leq k \\leq 32$,\n\n$\\therefore \\mathrm{k}=31, \\mathrm{x}=16$,\n\n$\\therefore$ 去掉的数是 16,\n\n故答案为: 16 ."} {"id": "5746", "image": [], "answer": "$2 \\sqrt{2}$", "solution": "null", "level": "八年级", "question": "计算 $\\sqrt{40} \\div \\sqrt{5}$ 的结果是", "options": [], "subject": "算术", "analysis": "$\\sqrt{40} \\div \\sqrt{5}=\\sqrt{40 \\div 5}=\\sqrt{8}=2 \\sqrt{2}$,\n\n故答案为 $2 \\sqrt{2}$."} {"id": "5751", "image": [], "answer": "$x \\geq-1$", "solution": "null", "level": "八年级", "question": "若式子 $\\sqrt{x+1}$ 在实数范围内有意义, 则 $\\mathrm{x}$ 的取值范围是", "options": [], "subject": "算术", "analysis": "根据题意得: $x+1 \\geq 0$,\n\n解得 $x \\geq-1$,\n\n故答案为: $x \\geq-1$."} {"id": "5752", "image": [], "answer": "$\\frac{\\sqrt{101}-1}{2}$", "solution": "null", "level": "八年级", "question": "观察下列等式:\n\n(1) $\\frac{1}{\\sqrt{3}+1}=\\frac{\\sqrt{3}-1}{(\\sqrt{3}+1)(\\sqrt{3}-1)}=\\frac{\\sqrt{3}-1}{2}$\n\n(2) $\\frac{1}{\\sqrt{5}+\\sqrt{3}}=\\frac{\\sqrt{5}-\\sqrt{3}}{(\\sqrt{5}+\\sqrt{3})(\\sqrt{5}-\\sqrt{3})}=\\frac{\\sqrt{5}-\\sqrt{3}}{2}$\n\n(3) $\\frac{1}{\\sqrt{7}+\\sqrt{5}}=\\frac{\\sqrt{7}-\\sqrt{5}}{(\\sqrt{7}+\\sqrt{5})(\\sqrt{7}-\\sqrt{5})}=\\frac{\\sqrt{7}-\\sqrt{5}}{2}$\n\n参照上面等式计算方法计算:\n\n$\\frac{1}{1+\\sqrt{3}}+\\frac{1}{\\sqrt{3}+\\sqrt{5}}+\\frac{1}{\\sqrt{5}+\\sqrt{7}}+\\cdots+\\frac{1}{3 \\sqrt{11}+\\sqrt{101}}=$ \\$ \\qquad \\$ .", "options": [], "subject": "算术", "analysis": "$\\frac{1}{1+\\sqrt{3}}+\\frac{1}{\\sqrt{3}+\\sqrt{5}}+\\frac{1}{\\sqrt{5}+\\sqrt{7}}+\\cdots+\\frac{1}{3 \\sqrt{11}+\\sqrt{101}}$,\n\n$=\\frac{\\sqrt{3}-1}{2}+\\frac{\\sqrt{5}-\\sqrt{3}}{2}+\\frac{\\sqrt{7}-\\sqrt{5}}{2}+\\cdots+\\frac{\\sqrt{101}-\\sqrt{99}}{2}$\n\n$=\\frac{\\sqrt{3}-1+\\sqrt{5}-\\sqrt{3}+\\sqrt{7}-\\sqrt{5}+\\ldots+\\sqrt{101}-\\sqrt{99}}{2}$\n\n$=\\frac{\\sqrt{101}-1}{2}$\n\n故答案为: $\\frac{\\sqrt{101}-1}{2}$"} {"id": "5014", "image": [], "answer": "根据二次根式的被开方数与算术平方根的关系即可直接求解.【解析】解:0.00236是由23.6小数点向左移动4位得到,则$sqrt{0.00236}=0.04858$;0.4858是由4.858向左移动一位得到,则$x=0.236$.故答案是:$0.04858,0.236$.【点睛】本题考查了二次根式的性质与化简,理解被开方数与算术平方根的关系:被开方数向一个方向移动2位,对应的算术平方根的小数点向相同的方向移动1位.", "solution": "null", "level": "八年级", "question": "已知$sqrt{2.36}=1.536,sqrt{23.6}=4.858$.则$sqrt{0.00236}=0.04858$.若$sqrt{{x}}=0.4858$,则$x=$0.236.", "options": [], "subject": "算术", "analysis": "根据二次根式的被开方数与算术平方根的关系即可直接求解.【解析】解:0.00236是由23.6小数点向左移动4位得到,则$sqrt{0.00236}=0.04858$;0.4858是由4.858向左移动一位得到,则$x=0.236$.故答案是:$0.04858,0.236$.【点睛】本题考查了二次根式的性质与化简,理解被开方数与算术平方根的关系:被开方数向一个方向移动2位,对应的算术平方根的小数点向相同的方向移动1位."} {"id": "5082", "image": [], "answer": "根据加权平均数定义可得.【解析】解:这个人的面试成绩是$85times20%+80times30%+88times50%=85$(分).故答案为:85.【点睛】本题主要考查加权平均数的计算,掌握加权平均数的定义是解题的关键.", "solution": "null", "level": "八年级", "question": "面试时,某人的基本知识、表达能力、工作态度的得分分别是85分,80分,88分,若依次按$20%,30%,50%$的比例确定成绩,则这个人的面试成绩是85分.", "options": [], "subject": "算术", "analysis": "根据加权平均数定义可得.【解析】解:这个人的面试成绩是$85times20%+80times30%+88times50%=85$(分).故答案为:85.【点睛】本题主要考查加权平均数的计算,掌握加权平均数的定义是解题的关键."} {"id": "5692", "image": ["7145.jpg"], "answer": "$$\n\\frac{4}{3}\n$$\n\n(2) $-2 \\leq x \\leq 4$", "solution": "null", "level": "八年级", "question": "某中学数学兴趣小组在一次课外学习与探究中遇到一些新的数学符号, 他们将其中某些材料摘录如下:\n\n对于三个实数 $\\mathrm{a}, \\mathrm{b}, \\mathrm{c}$, 用 $\\mathrm{M}\\{\\mathrm{a}, \\mathrm{b}, \\mathrm{c}\\}$ 表示这三个数的平均数, 用 $\\min \\{\\mathrm{a}, \\mathrm{b}, \\mathrm{c}\\}$ 表示这三个数中最小的数.例如: $\\mathrm{M}\\{1,2,9\\}=\\frac{1+2+9}{3}=4, \\min \\{1,2,-3\\}=-3, \\min \\{3,1,1\\}=1$.请结合上述材料,解决下列问题:\n\n(1) $\\mathrm{M}\\left\\{(-2)^{2}, 2^{2},-2^{2}\\right\\}=$ \\$ \\qquad \\$ ;\n\n(2) 若 $\\min \\{3-2 x, 1+3 x,-5\\}=-5$, 则 $x$ 的取值范围为 \\$ \\qquad \\$ .", "options": [], "subject": "代数", "analysis": "(1) $\\mathrm{M}\\left\\{(-2)^{2}, 2^{2},-2^{2}\\right\\}=\\frac{(-2)^{2}+2^{2}-2^{2}}{3}={ }^{\\frac{4}{3}}$;\n\n( 2 ) $\\because \\min \\{3-2 x, 1+3 x,-5\\}=-5$,\n\n$\\therefore\\left\\{\\begin{array}{l}3-2 x \\geq 5 \\\\ 1+3 x \\geq 5\\end{array}\\right.$\n\n解得 $-2 \\leq x \\leq 4$.\n\n故 $\\mathrm{x}$ 的取值范围为 $-2 \\leq \\mathrm{x} \\leq 4$.\n\n"} {"id": "5715", "image": [], "answer": "$4 ;-3 ;-7$", "solution": "null", "level": "八年级", "question": "方程 $4 x^{2}=3 x+7$ 的二次项系数是 \\$ \\qquad \\$ , 一次项系数是 \\$ \\qquad \\$ , 常数项是 \\$ \\qquad \\$", "options": [], "subject": "代数", "analysis": "由 $4 x^{2}=3 x+7$ 得 $4 x^{2}-3 x-7=0$,\n\n所以, 二次项系数是 4, 一次项系数 -3 , 常数项 -7 .\n\n故答案为 $4,-3,-7$."} {"id": "5716", "image": [], "answer": "$m>-\\frac{49}{4}$", "solution": "null", "level": "八年级", "question": "若关于 $x$ 的一元二次方程 $x^{2}-7 x-m=0$ 有两个不相等的实数根, 则实数 $m$ 的取值范围是", "options": [], "subject": "代数", "analysis": "$\\because$ 关于 $\\mathrm{x}$ 的一元二次方程 $\\mathrm{x}^{2}-7 \\mathrm{x}-\\mathrm{m}=0$ 有两个不相等的实数根\n\n$\\therefore \\Delta=b^{2}-4 a c=(-7)^{2}-4 \\times(-m)>0$\n\n解得: $m>-\\frac{45}{4}$\n\n故答案为: $m>-\\frac{4 s}{4}$."} {"id": "5717", "image": [], "answer": "$\\mathrm{m}<-1$", "solution": "null", "level": "八年级", "question": "若一元二次方程 $x^{2}-2 x-m=0$ 无实根, 则 $m$ 的取值范围是 \\$ \\qquad \\$ .", "options": [], "subject": "代数", "analysis": "$\\because$ 一元二次方程 $\\mathrm{x}^{2}-2 \\mathrm{x}-\\mathrm{m}=0$ 无实根,\n\n$\\therefore \\triangle=4+4 \\mathrm{~m}<0$, 解得: $\\mathrm{m}<-1$."} {"id": "5718", "image": ["7158.jpg"], "answer": "$0 \\leq m \\leq 2$ 且 $m \\neq 1$", "solution": "null", "level": "八年级", "question": "已知关于的 $x$ 方程 $(m-1) x^{2}-\\sqrt{2-m} x-\\frac{1}{2}=0$ 有两个实数根, 则 $m$ 的取值范围是", "options": [], "subject": "代数", "analysis": "$\\because$ 关于 $\\mathrm{x}$ 的一元二次方程 $(m-1) x^{2}-\\sqrt{2-m} x-\\frac{1}{2}=0$ 有两个实数根,\n\n\n\n解得: $0 \\leq m \\leq 2$ 且 $m \\neq 1$,\n\n故答案为: $0 \\leq m \\leq 2$ 且 $m \\neq 1$."} {"id": "5719", "image": [], "answer": "2019", "solution": "null", "level": "八年级", "question": "设 $\\mathrm{m}, \\mathrm{n}$ 分别为一元二次方程 $x^{2}+x-2020=0$ 的两个实数根, 则 $m^{2}+2 m+n=$", "options": [], "subject": "代数", "analysis": "$\\because \\mathrm{m}, \\mathrm{n}$ 分别为一元二次方程 $x^{2}+x-2020=0$ 的两个实数根,\n\n$\\therefore \\mathrm{m}+\\mathrm{n}=-1, m^{2}+m=2020$\n\n$$\nm^{2}+2 m+n=\\left(m^{2}+m\\right)+(m+n)=2020-1=2019\n$$\n\n故答案是: 2019"} {"id": "5720", "image": [], "answer": "$x=0$ 或 $x=1$", "solution": "null", "level": "八年级", "question": "方程 $\\mathrm{x}(\\mathrm{x}-1)=0$ 的根是 \\$ \\qquad \\$", "options": [], "subject": "代数", "analysis": "依题意得:\n\n$\\mathrm{x}=0$ 或 $\\mathrm{x}-1=0$\n\n$\\therefore \\mathrm{x}=0$ 或 $\\mathrm{x}=1$"} {"id": "5721", "image": [], "answer": "10", "solution": "null", "level": "八年级", "question": "若 $\\mathrm{a}, \\mathrm{b}$ 是方程 $\\mathrm{x}^{2}-2 \\mathrm{x}-3=0$ 的两个实数根, 则 $\\mathrm{a}^{2}+\\mathrm{b}^{2}=$ \\$ \\qquad \\$ .", "options": [], "subject": "代数", "analysis": "$\\because a, b$ 是方程 $x^{2}-2 x-3=0$ 的两个实数根,\n\n$\\therefore \\mathrm{a}+\\mathrm{b}=2, \\mathrm{ab}=-3$,\n\n$\\therefore \\mathrm{a}^{2}+\\mathrm{b}^{2}=(\\mathrm{a}+\\mathrm{b})^{2}-2 \\mathrm{ab}=\\mathrm{2}^{2}-2 \\times(-3)=10$.\n\n故答案为: 10 ."} {"id": "5722", "image": [], "answer": "$3 ;-4$", "solution": "null", "level": "八年级", "question": "已知方程 $x^{2}+m x+3=0$ 的一个根是 1 , 则它的另一个根是 \\$ \\qquad \\$ , $\\mathrm{m}$ 的值是 \\$ \\qquad \\$ .", "options": [], "subject": "代数", "analysis": "设方程的另一个解是 $a$, 则 $1+a=-m \\quad, 1 \\times a=3$,\n\n解得: $m=-4, a=3$.\n\n故答案是: $3,-4$"} {"id": "5723", "image": [], "answer": "10", "solution": "null", "level": "八年级", "question": "设 $\\mathrm{x}_{1}, \\mathrm{x}_{2}$ 是一元二次方程 $\\mathrm{x}^{2}+5 \\mathrm{x}-3=0$ 的两根, 且 $2 x_{1}\\left(x_{2}{ }^{2}+6 x_{2}-3\\right)+a=4$, 则 $a=$", "options": [], "subject": "代数", "analysis": "$\\because \\mathrm{x}_{2}$ 是一元二次方程 $\\mathrm{x}^{2}+5 \\mathrm{x}-3=0$ 的根,\n\n$\\therefore \\mathrm{x}_{2}{ }^{2}+5 \\mathrm{x}_{2}-3=0$,\n\n$\\therefore \\mathrm{x}_{2}{ }^{2}+5 \\mathrm{x}_{2}=3$,\n\n$\\because 2 \\mathrm{x}_{1}\\left(\\mathrm{x}_{2}^{2}+6 \\mathrm{x}_{2}-3\\right)+\\mathrm{a}=4$,\n\n$\\therefore 2 \\mathrm{x}_{1} \\cdot \\mathrm{x}_{2}+\\mathrm{a}=4$,\n\n$\\because \\mathrm{x}_{1}, \\quad \\mathrm{x}_{2}$ 是一元二次方程 $\\mathrm{x}^{2}+5 \\mathrm{x}-3=0$ 的两根,\n\n$\\therefore \\mathrm{x}_{1} \\mathrm{x}_{2}=-3$,\n\n$\\therefore 2 \\times(-3)+a=4$,\n\n$\\therefore \\mathrm{a}=10$.\n\n故答案为: 10 ."} {"id": "5725", "image": [], "answer": "39", "solution": "null", "level": "八年级", "question": "已知 $(x-2016)^{2}+(x-2018)^{2}=80$, 则 $(x-2017)^{2}=$ \\$ \\qquad \\$ .", "options": [], "subject": "代数", "analysis": "$\\because(x-2016)^{2+}(x-2018)^{2}=80$,\n\n$\\therefore(x-2017+1)^{2}+(x-2017-1)^{2}=80$,\n\n$(x-2017)^{2}+2(x-2017)+1+(x-2017)^{2}-2(x-2017)+1=80$,\n$2(x-2017)^{2}+2=80$,\n\n$2(x-2017)^{2}=78$,\n\n$(x-2017)^{2}=39$.\n\n故答案为: 39"} {"id": "5745", "image": [], "answer": "2", "solution": "null", "level": "八年级", "question": "已知最简二次根式 $\\sqrt{4 a+3 b}$ 与 $\\sqrt[b+1]{2 a-b+6}$ 可以合并, 则 $a+b$ 的值为", "options": [], "subject": "代数", "analysis": "由题意得: $b+1=2,4 a+3 b=2 a-b+6$; 解得 $b=1 , a=1$; 所以 $a+b=2$所以答案为 2"} {"id": "5747", "image": ["7164.jpg"], "answer": "$\\frac{3}{7}$", "solution": "null", "level": "八年级", "question": "化简: $\\sqrt{\\frac{9}{49}}=$ \\$ \\qquad \\$ .", "options": [], "subject": "代数", "analysis": "$\\sqrt{\\frac{9}{49}}=\\frac{3}{7}$\n\n"} {"id": "5748", "image": [], "answer": "$-\\sqrt{-m}$", "solution": "null", "level": "八年级", "question": "化简 $\\sqrt[m]{-\\frac{1}{m}}$ 的结果为", "options": [], "subject": "代数", "analysis": "依题意可知 $m<0$,\n\n$\\therefore \\sqrt[m]{-\\frac{1}{m}}=-\\sqrt{-m}$"} {"id": "5749", "image": [], "answer": "3", "solution": "null", "level": "八年级", "question": "若 $\\sqrt{(m+2)^{2}}+|5-n|=0$, 则 $m+n=$ \\$ \\qquad \\$ .", "options": [], "subject": "代数", "analysis": "根据题意得, $m+2=0,5-n=0$,\n\n解得 $m=-2, n=5$,\n\n则 $m+n=-2+5=3$.\n\n故答案为: 3 ."} {"id": "5753", "image": [], "answer": "3", "solution": "null", "level": "八年级", "question": "若实数 $a=\\frac{1}{2-\\sqrt{3}}$, 则代数式 $a^{2}-4 a+4$ 的值为", "options": [], "subject": "代数", "analysis": "$\\because^{a=\\frac{1}{2-\\sqrt{3}}}=\\frac{2+\\sqrt{3}}{(2-\\sqrt{3}) \\times(2+\\sqrt{3})}=2+\\sqrt{3}$,\n\n$\\therefore a^{2}-4 a+4=(\\mathrm{a}-2)^{2}=(2+\\sqrt{3}-2)^{2}=3$,\n\n故答案为: 3 ."} {"id": "5804", "image": [], "answer": "$x_{1}=0, x_{2}=2020$", "solution": "null", "level": "八年级", "question": "方程 $x^{2}=2020 x$ 的解是 \\$ \\qquad \\$ .", "options": [], "subject": "代数", "analysis": "移项得: $x^{2}-2020 x=0$,\n\n$\\therefore \\mathrm{x}(\\mathrm{x}-2020)=0$,\n\n则 $x=0$ 或 $x-2020=0$,\n\n解得 $x_{1}=0, x_{2}=2020$,\n\n故答案为: $x_{1}=0, x_{2}=2020$."} {"id": "5805", "image": [], "answer": "-2", "solution": "null", "level": "八年级", "question": "如果 $a$ 是一元二次方程 $x^{2}-3 x-3=0$ 的一个解, 那么代数式 $2 a^{2}-6 a-8$ 的值为", "options": [], "subject": "代数", "analysis": "把 $a_{\\text {代入 }} x^{2}-3 x-3=0$ 得, $a^{2}-3 a-3=0, \\therefore 2 a^{2}-6 a=6, \\therefore 2 a^{2}-6 a-8=6-8=-2$."} {"id": "5808", "image": [], "answer": "$168(1-x)^{2}=128$", "solution": "null", "level": "八年级", "question": "某药品经过两次降价, 每瓶零售价由 168 元降为 128 元, 已知两次降价的百分率相同, 每次降价的百分率为 $x$, 根据题意列方程得 \\$ \\qquad \\$", "options": [], "subject": "代数", "analysis": "根据题意得: $168(1-x)^{2}=128$.\n\n故答案为: $168(1-x)^{2}=128$."} {"id": "5810", "image": [], "answer": " 1\n\n解. 原式 $=2 \\times \\sqrt{1-2 \\sqrt{2}+2}+\\sqrt{9-12 \\sqrt{2}+8}$\n\n$=2 \\times \\sqrt{(\\sqrt{2}-1)^{2}}+\\sqrt{(3-2 \\sqrt{2})^{2}}$,\n\n$=1$\n\n故答案为: 1", "solution": "null", "level": "八年级", "question": "$2 \\sqrt{3-2 \\sqrt{2}}+\\sqrt{17-12 \\sqrt{2}}=$ \\$ \\qquad \\$ .", "options": [], "subject": "代数", "analysis": " 1\n\n解. 原式 $=2 \\times \\sqrt{1-2 \\sqrt{2}+2}+\\sqrt{9-12 \\sqrt{2}+8}$\n\n$=2 \\times \\sqrt{(\\sqrt{2}-1)^{2}}+\\sqrt{(3-2 \\sqrt{2})^{2}}$,\n\n$=1$\n\n故答案为: 1"} {"id": "4989", "image": [], "answer": "根据二次根式的被开方数是非负数、分母不为0列出不等式,解不等式得到答案.【解析】解:由题意得:$2-3xgeqslant0$且$2x+1neq0$,解得:$xleqslantfrac{2}{3}$且$xneq-frac{1}{2}$,故答案为:$xleqslantfrac{2}{3}$且$xneq-frac{1}{2}$.【点睛】本题考查的是二次根式有意义的条件,掌握二次根式的被开方数是非负数、分母不为0是解题的关键.", "solution": "null", "level": "八年级", "question": "若代数式$frac{sqrt{2-3x}}{2x+1}$有意义,则$x$的取值范围是$xleqslantfrac{2}{3}$且$xneq-frac{1}{2}$一", "options": [], "subject": "代数", "analysis": "根据二次根式的被开方数是非负数、分母不为0列出不等式,解不等式得到答案.【解析】解:由题意得:$2-3xgeqslant0$且$2x+1neq0$,解得:$xleqslantfrac{2}{3}$且$xneq-frac{1}{2}$,故答案为:$xleqslantfrac{2}{3}$且$xneq-frac{1}{2}$.【点睛】本题考查的是二次根式有意义的条件,掌握二次根式的被开方数是非负数、分母不为0是解题的关键."} {"id": "4990", "image": [], "answer": "根据二次根式的性质计算即可.【解析】解:$6sqrt{frac{1}{3}}=sqrt{36timesfrac{1}{3}}=sqrt{12}=2sqrt{3}$.故答案为:$2sqrt{3}$.【点睛】本题考查的是最简二次根式,二次根式的化简,熟记二次根式的性质是解题的关键.", "solution": "null", "level": "八年级", "question": "$6sqrt{frac{1}{3}}$化简为最简二次根式的结果是$2sqrt{3}ldots$.", "options": [], "subject": "代数", "analysis": "根据二次根式的性质计算即可.【解析】解:$6sqrt{frac{1}{3}}=sqrt{36timesfrac{1}{3}}=sqrt{12}=2sqrt{3}$.故答案为:$2sqrt{3}$.【点睛】本题考查的是最简二次根式,二次根式的化简,熟记二次根式的性质是解题的关键."} {"id": "5693", "image": [], "answer": "19", "solution": "null", "level": "八年级", "question": "某地某日最高气温为 $12^{\\circ} \\mathrm{C}$, 最低气温为 $-7^{\\circ} \\mathrm{C}$, 该日气温的极差是 \\$ \\qquad \\$\n${ }^{\\circ} \\mathrm{C}$.", "options": [], "subject": "度量几何学", "analysis": "极差=12- $(-7)=12+7=19$.\n\n故答案为: 19 ."} {"id": "5755", "image": [], "answer": "$75^{\\circ}$", "solution": "null", "level": "八年级", "question": "已知 $\\alpha 、 \\beta$ 均为锐角, 且满足 $\\left|\\sin \\alpha-{ }^{\\frac{1}{2}}\\right|+\\sqrt{(\\tan \\beta-1)^{2}}=0$, 则 $\\alpha+\\beta=$ \\$ \\qquad \\$ .", "options": [], "subject": "度量几何学", "analysis": "$\\because\\left|\\sin \\alpha--^{\\frac{1}{2}}\\right|+\\sqrt{(\\tan \\beta-1)^{2}}=0$,\n\n$\\therefore \\sin \\alpha==^{\\frac{1}{2}}, \\quad \\tan \\beta=1$,\n\n$\\therefore \\alpha=30^{\\circ}, \\beta=45^{\\circ}$,\n\n则 $\\alpha+\\beta=30^{\\circ}+45^{\\circ}=75^{\\circ}$.\n\n故答案为: $75^{\\circ}$."} {"id": "5776", "image": ["7181.jpg"], "answer": "15", "solution": "null", "level": "八年级", "question": "如图, $\\triangle \\mathrm{ABCD}$ 的对角线相交于点 $\\mathrm{O}, \\mathrm{BC}=7, \\mathrm{BD}=10, \\mathrm{AC}=6$, 则 $\\triangle \\mathrm{AOD}$ 的周长是\n\n", "options": [], "subject": "度量几何学", "analysis": "已知四边形 $\\mathrm{ABCD}$ 是平行四边形, 根据平行四边形的性质可得 $\\mathrm{AD}=\\mathrm{BC}, \\mathrm{OA}=\\mathrm{OC}, \\mathrm{OB}=\\mathrm{OD}$, 再由 $B C=7, B D=10, A C=6$, 可得 $A D=7, O A=3, O D=5$, 即可得 $\\triangle A O D$ 的周长 $=A D+O A+O D=15$."} {"id": "5777", "image": [], "answer": "$120^{\\circ}$", "solution": "null", "level": "八年级", "question": "在平行四边形 $\\mathrm{ABCD}$ 中, 已知 $\\angle \\mathrm{A}-\\angle \\mathrm{B}=60^{\\circ}$, 则 $\\angle \\mathrm{C}=$ \\$ \\qquad \\$ .", "options": [], "subject": "度量几何学", "analysis": "$\\because$ 四边形 $\\mathrm{ABCD}$ 是平行四边形, $\\therefore \\angle \\mathrm{A}+\\angle \\mathrm{B}=180^{\\circ}$, 又 $\\angle \\mathrm{A}-\\angle \\mathrm{B}=60^{\\circ}$, 故可知 $\\angle \\mathrm{A}=120^{\\circ}, \\therefore \\angle \\mathrm{C}$\n\n$=\\angle \\mathrm{A}=120^{\\circ}$,\n\n故答案为: $120^{\\circ}$."} {"id": "5781", "image": ["7185.jpg"], "answer": "32", "solution": "null", "level": "八年级", "question": "如图, 在 $\\Rightarrow \\mathrm{ABCD}$ 中, $\\mathrm{DB}=\\mathrm{DC}, \\angle \\mathrm{C}=58^{\\circ}, \\mathrm{AE} \\perp \\mathrm{BD}$ 于 $\\mathrm{E}$, 则 $\\angle \\mathrm{DAE}=$\n\n度.\n\n", "options": [], "subject": "度量几何学", "analysis": "$\\because \\mathrm{DC}=\\mathrm{BD}, \\quad \\therefore \\angle \\mathrm{C}=\\angle \\mathrm{DBC}=58^{\\circ}$,\n\n$\\because$ 四边形 $\\mathrm{ABCD}$ 是平行四边形,\n\n$\\therefore \\mathrm{AD} / / \\mathrm{BC}$\n\n$\\therefore \\angle \\mathrm{ADB}=\\angle \\mathrm{CBD}=58^{\\circ}$,\n\n$\\because \\mathrm{AE} \\perp \\mathrm{BD}$ 于 $\\mathrm{E}$,\n\n$\\therefore \\angle \\mathrm{AED}=90^{\\circ}$,\n\n$\\therefore \\angle \\mathrm{DAE}=32^{\\circ}$,\n\n故答案为: 32 ."} {"id": "5785", "image": ["7188.jpg"], "answer": "10", "solution": "null", "level": "八年级", "question": "如图, 在 $\\triangle A B C D$ 中, 连结 $B D$, 且 $B D=C D$, 过点 $A$ 作 $A M \\perp B D$ 于点 $M$, 过点 $D$ 作 $D N \\perp A B$ 于点 $N$, 且 $D N=5 \\sqrt{2}$, 在 $\\mathrm{DB}$ 的延长线上取一点 $\\mathrm{P}$, 满足 $\\angle \\mathrm{ABD}=\\angle \\mathrm{MAP}+\\angle \\mathrm{PAB}$, 则 $\\mathrm{AP}=$", "options": [], "subject": "度量几何学", "analysis": "$\\because$ 四边形 $\\mathrm{ABCD}$ 为平行四边形,\n\n$\\therefore \\mathrm{AB}=\\mathrm{CD}$,\n\n又 $\\because B D=C D$,\n\n$\\therefore \\mathrm{AB}=\\mathrm{BD}$,\n\n$\\because \\mathrm{S}_{\\triangle \\mathrm{ABD}}={ }^{\\frac{1}{2}} \\mathrm{AB} \\times \\mathrm{DN}=={ }^{\\frac{1}{2}} \\mathrm{BD} \\times \\mathrm{AM}$,\n\n$\\therefore \\mathrm{AM}=\\mathrm{DN}=5 \\sqrt{2}$,\n\n$\\because \\angle \\mathrm{ABD}=\\angle \\mathrm{APB}+\\angle \\mathrm{PAB}$,\n\n又 $\\because \\angle \\mathrm{ABD}=\\angle \\mathrm{MAP}+\\angle \\mathrm{PAB}$,\n\n$\\therefore \\angle \\mathrm{APB}=\\angle \\mathrm{MAP}$\n\n$\\therefore \\mathrm{MA}=\\mathrm{MB}, \\triangle \\mathrm{AMP}$ 为等腰直角三角形,\n\n$A P=\\sqrt{M A^{2}+M P^{2}}=\\sqrt{2 M A^{2}}=\\sqrt{2} M A=\\sqrt{2} \\times 5 \\sqrt{2}=10$.\n\n故答案为: 10 ."} {"id": "5812", "image": ["7211.jpg", "7212.jpg", "7213.jpg"], "answer": "6 或 $^{\\frac{15}{8}}$", "solution": "null", "level": "八年级", "question": "如图, 在矩形 $\\mathrm{ABCD}$ 中, $\\mathrm{AD}=3 \\mathrm{AB}=3 \\sqrt{10}$, 点 $\\mathrm{P}$ 是 $\\mathrm{AD}$ 的中点, 点 $\\mathrm{E}$ 在 $\\mathrm{BC}$ 上, $\\mathrm{CE}=2 \\mathrm{BE}$,点 $\\mathrm{M} 、 \\mathrm{~N}$ 在线段 $\\mathrm{BD}$ 上.若 $\\triangle \\mathrm{PMN}$ 是等腰三角形且底角与 $\\angle \\mathrm{DEC}$ 相等, 则 $\\mathrm{MN}=$ \\$ \\qquad \\$ .\n\n", "options": [], "subject": "度量几何学", "analysis": "分两种情况:\n\n(1)MN 为等腰 $\\triangle \\mathrm{PMN}$ 的底边时, 作 $\\mathrm{PF} \\perp \\mathrm{MN}$ 于 $\\mathrm{F}$, 如图 1 所示:\n\n\n\n图 1\n\n则 $\\angle \\mathrm{PFM}=\\angle \\mathrm{PFN}=90^{\\circ}$,\n\n$\\because$ 四边形 $\\mathrm{ABCD}$ 是矩形,\n\n$\\therefore A B=C D, B C=A D=3 A B=3 \\sqrt{10}, \\angle A=\\angle C=90^{\\circ}$,\n\n$\\therefore \\mathrm{AB}=\\mathrm{CD}=\\sqrt{10}, \\mathrm{BD}=\\sqrt{A B^{2}+A D^{2}}=10$,\n\n$\\because$ 点 $\\mathrm{P}$ 是 $\\mathrm{AD}$ 的中点,\n\n$\\therefore \\mathrm{PD}=\\frac{1}{2} \\mathrm{AD}=\\frac{3 \\sqrt{10}}{2}$\n\n$\\because \\angle \\mathrm{PDF}=\\angle \\mathrm{BDA}$,\n\n$\\therefore \\triangle \\mathrm{PDF} \\sim \\triangle \\mathrm{BDA}$,\n$\\therefore \\frac{P F}{A B}=\\frac{P D}{B D}$ ,即 $\\frac{P F}{\\sqrt{10}}=\\frac{\\frac{3 \\sqrt{10}}{2}}{10}$ ,\n\n解得: $\\mathrm{PF}=\\frac{3}{2}$,\n\n$\\because \\mathrm{CE}=2 \\mathrm{BE}$,\n\n$\\therefore \\mathrm{BC}=\\mathrm{AD}=3 \\mathrm{BE}$,\n\n$\\therefore \\mathrm{BE}=\\mathrm{CD}$,\n\n$\\therefore \\mathrm{CE}=2 \\mathrm{CD}$\n\n$\\because \\triangle \\mathrm{PMN}$ 是等腰三角形且底角与 $\\angle \\mathrm{DEC}$ 相等, $\\mathrm{PF} \\perp \\mathrm{MN}$,\n\n$\\therefore \\mathrm{MF}=\\mathrm{NF}, \\angle \\mathrm{PNF}=\\angle \\mathrm{DEC}$,\n\n$\\because \\angle \\mathrm{PFN}=\\angle \\mathrm{C}=90^{\\circ}$,\n\n$\\therefore \\triangle \\mathrm{PNF} \\sim \\triangle \\mathrm{DEC}$\n\n$\\therefore \\frac{N F}{P F}=\\frac{C E}{C D}=2$\n\n$\\therefore \\mathrm{MF}=\\mathrm{NF}=2 \\mathrm{PF}=3$,\n\n$\\therefore \\mathrm{MN}=2 \\mathrm{NF}=6$;\n\n(2) $M N$ 为等腰 $\\triangle \\mathrm{PMN}$ 的腰时, 作 $\\mathrm{PF} \\perp \\mathrm{BD}$ 于 $\\mathrm{F}$, 如图 2 所示:\n\n\n\n图 2\n\n由(1)得: $\\mathrm{PF}=\\frac{3}{2}, \\mathrm{MF}=3$,\n\n设 $\\mathrm{MN}=\\mathrm{PN}=\\mathrm{x}$, 则 $\\mathrm{FN}=3-\\mathrm{x}$,\n\n在 Rt $\\triangle$ PNF 中, $\\left({ }^{\\frac{3}{2}}\\right)^{2}+(3-x)^{2}=x^{2}$,\n\n解得: $\\mathrm{x}^{=\\frac{15}{8}}$, 即 $\\mathrm{MN}=\\frac{15}{8}$;\n\n综上所述, $\\mathrm{MN}$ 的长为 6 或 ${ }^{\\frac{15}{8}}$;\n\n故答案为: 6 或 $^{\\frac{15}{8}}$"} {"id": "4994", "image": ["6610.jpg"], "answer": "直接利用正方形的性质得出两个小正方形的边长,进而得出大正方形的边长,即可得出答案.【解析】解:$because$两个小正方形面积为$8{~cm}^{2}$和$18{~cm}^{2}$,$therefore$大正方形边长为:$sqrt{8}+sqrt{18}=2sqrt{2}+3sqrt{2}=5sqrt{2}({~cm})$,$therefore$大正方形面积为$(5sqrt{2})^{2}=50left({~cm}^{2}right)$,$therefore$留下的阴影部分面积和为:$50-8-18=24left({~cm}^{2}right)$.故答案为:24.【点睛】此题主要考查了二次根式的应用,正确得出大正方形的边长是解题关键.", "solution": "null", "level": "八年级", "question": "如图,从一个大正方形中裁去面积为$8{~cm}^{2}$和$18{~cm}^{2}$的两个小正方形,则留下的阴影部分面积和为$24{~cm}^{2}$.", "options": [], "subject": "度量几何学", "analysis": "直接利用正方形的性质得出两个小正方形的边长,进而得出大正方形的边长,即可得出答案.【解析】解:$because$两个小正方形面积为$8{~cm}^{2}$和$18{~cm}^{2}$,$therefore$大正方形边长为:$sqrt{8}+sqrt{18}=2sqrt{2}+3sqrt{2}=5sqrt{2}({~cm})$,$therefore$大正方形面积为$(5sqrt{2})^{2}=50left({~cm}^{2}right)$,$therefore$留下的阴影部分面积和为:$50-8-18=24left({~cm}^{2}right)$.故答案为:24.【点睛】此题主要考查了二次根式的应用,正确得出大正方形的边长是解题关键."} {"id": "5126", "image": [], "answer": "由平行四边形的性质得出$angleA=angleC,angleA+angleB=180^{circ}$,再由已知条件求出$angleA$,即可得出$angleB$.【解析】解:$because$四边形$ABCD$是平行四边形,$thereforeangleA=angleC,angleA+angleB=180^{circ}$,$becauseangleA+angleC=200^{circ}$$thereforeangleA=100^{circ}$,$thereforeangleB=80^{circ}$,故答案为:$80^{circ}$.【点睛】本题考查了平行四边形的性质,熟练掌握平行四边形的性质,并能进行推理计算是解决问题的关键.", "solution": "null", "level": "八年级", "question": "已知$squareABCD$中,$angleA+angleC=200^{circ}$,则$angleB$的度数是$80^{circ}$", "options": [], "subject": "度量几何学", "analysis": "由平行四边形的性质得出$angleA=angleC,angleA+angleB=180^{circ}$,再由已知条件求出$angleA$,即可得出$angleB$.【解析】解:$because$四边形$ABCD$是平行四边形,$thereforeangleA=angleC,angleA+angleB=180^{circ}$,$becauseangleA+angleC=200^{circ}$$thereforeangleA=100^{circ}$,$thereforeangleB=80^{circ}$,故答案为:$80^{circ}$.【点睛】本题考查了平行四边形的性质,熟练掌握平行四边形的性质,并能进行推理计算是解决问题的关键."} {"id": "5129", "image": ["6646.jpg", "6647.jpg", "6647.jpg"], "answer": "由平行四边形的性质和周长得$BC+CD=15$,设$BC$为$x$,则$CD=15-x$,再由平行四边形的面积求出$x=9$,即可解决问题.【解析】解:$because$平行四边形$ABCD$的周长为30,$thereforeAB=CD,AD=BC,BC+CD=15$,设$BC$为$x$,则$CD=15-x$,$therefore4x=(15-x)times6$,解得:$x=9$,$thereforeBC=9$,$thereforeS_{text{平行四边形}ABCD}=BCcdotAE=9times4=36$,故答案为:36.【点睛】本题主要考查了平行四边形的性质,熟记平行四边形的对边相等,面积$=$底$times$高是解题的关键.", "solution": "null", "level": "八年级", "question": "如图,平行四边形$ABCD$的周长为$30,AEperpBC$于$E,AFperpDC$的延长线于点$F,AE=4,AF=$6,则平行四边形$ABCD$的面积是36.", "options": [], "subject": "度量几何学", "analysis": "由平行四边形的性质和周长得$BC+CD=15$,设$BC$为$x$,则$CD=15-x$,再由平行四边形的面积求出$x=9$,即可解决问题.【解析】解:$because$平行四边形$ABCD$的周长为30,$thereforeAB=CD,AD=BC,BC+CD=15$,设$BC$为$x$,则$CD=15-x$,$therefore4x=(15-x)times6$,解得:$x=9$,$thereforeBC=9$,$thereforeS_{text{平行四边形}ABCD}=BCcdotAE=9times4=36$,故答案为:36.【点睛】本题主要考查了平行四边形的性质,熟记平行四边形的对边相等,面积$=$底$times$高是解题的关键."} {"id": "5130", "image": ["6648.jpg", "6649.jpg", "6649.jpg"], "answer": "当$CMperpAB$时,$CM$的值最小,此时$DE$的值也最小,根据勾股定理求出$AB$,根据三角形的面积求出$CM$,再求出答案即可.【解析】解:连接$CM$,$because$点$D、E$分别为$CN,MN$的中点,$thereforeDE=frac{1}{2}CM$,当$CMperpAB$时,$CM$的值最小,此时$DE$的值也最小,由勾股定理得:$AB=sqrt{AC^{2}+BC^{2}}=sqrt{6^{2}+8^{2}}=10$,$becauseS_{triangleABC}=frac{1}{2}timesABtimesCM=frac{1}{2}timesACtimesBC$,$thereforeCM=frac{24}{5}$,$thereforeDE=frac{1}{2}CM=frac{12}{5}$,故答案为:$frac{12}{5}$.【点睛】本题考查了三角形的面积,勾股定理,三角形的中位线,垂线段最短等知识点,注意:三角形的中位线等于第三边的一半.", "solution": "null", "level": "八年级", "question": "在Rt$triangleABC$中,$angleC=90^{circ},AC=6,BC=8$,点$N$是$BC$边上一点,点$M$为$AB$边上的动点,点$D、E$分别为$CN,MN$的中点,则$DE$的最小值是$-frac{12}{5}-$.", "options": [], "subject": "度量几何学", "analysis": "当$CMperpAB$时,$CM$的值最小,此时$DE$的值也最小,根据勾股定理求出$AB$,根据三角形的面积求出$CM$,再求出答案即可.【解析】解:连接$CM$,$because$点$D、E$分别为$CN,MN$的中点,$thereforeDE=frac{1}{2}CM$,当$CMperpAB$时,$CM$的值最小,此时$DE$的值也最小,由勾股定理得:$AB=sqrt{AC^{2}+BC^{2}}=sqrt{6^{2}+8^{2}}=10$,$becauseS_{triangleABC}=frac{1}{2}timesABtimesCM=frac{1}{2}timesACtimesBC$,$thereforeCM=frac{24}{5}$,$thereforeDE=frac{1}{2}CM=frac{12}{5}$,故答案为:$frac{12}{5}$.【点睛】本题考查了三角形的面积,勾股定理,三角形的中位线,垂线段最短等知识点,注意:三角形的中位线等于第三边的一半."} {"id": "5150", "image": ["6677.jpg", "6678.jpg", "6678.jpg"], "answer": "根据三角形的内外角之间的关系可得$angle1+angle2=240^{circ}$.【解析】解:$because$三角形的内角和等于$180^{circ},angleD=60^{circ}$,$thereforeangle1=angleD+angleDFE$,$angle2=angleD+angleDEF$,$becauseangleDEF+angleDFE+angleD=180^{circ}$,$thereforeangle1+angle2=angleDEF+angleDFE+angleD+angleD=180^{circ}+60^{circ}=240^{circ}$.故答案为:$240^{circ}$.【点睛】本题考查了多边形的内角与外角.解题的关键是明确三角形的内外角之间的关系和三角形的内角和等于$180^{circ}$的知识点.", "solution": "null", "level": "八年级", "question": "如图,在四边形$ABCD$中,$angleD=60^{circ}$,若沿图中虚线剪去$angleD$,则$angle1+angle2=240^{circ}$", "options": [], "subject": "度量几何学", "analysis": "根据三角形的内外角之间的关系可得$angle1+angle2=240^{circ}$.【解析】解:$because$三角形的内角和等于$180^{circ},angleD=60^{circ}$,$thereforeangle1=angleD+angleDFE$,$angle2=angleD+angleDEF$,$becauseangleDEF+angleDFE+angleD=180^{circ}$,$thereforeangle1+angle2=angleDEF+angleDFE+angleD+angleD=180^{circ}+60^{circ}=240^{circ}$.故答案为:$240^{circ}$.【点睛】本题考查了多边形的内角与外角.解题的关键是明确三角形的内外角之间的关系和三角形的内角和等于$180^{circ}$的知识点."} {"id": "5151", "image": ["6679.jpg"], "answer": "由平行四边形的性质得$AB//CD,AD//BC,AB=CD,AD=BC$,再证$angleBAE=angleDAF=$$30^{circ}$,然后由含$30^{circ}$角的直角三角形的性质得$AB=2BE=4,AD=2DF=6$,即可解决问题.【解析】解:$because$四边形$ABCD$是平行四边形,$thereforeAB//CD,AD//BC,AB=CD,AD=BC$$becauseAEperpBC,AFperpCD$,$thereforeangleAEB=angleAFD=90^{circ},AFperpAB,AEperpAD$,$thereforeangleBAF=angleDAE=90^{circ}$,$becauseangleEAF=60^{circ}$,$thereforeangleBAE=angleDAF=90^{circ}-60^{circ}=30^{circ}$,$thereforeAB=2BE,AD=2DF$$becauseBE=2,DF=3$,$thereforeCD=AB=4,BC=AD=6$,$thereforesquareABCD$的周长$=2(AB+BC)=2times(4+6)=20$,故答案为:20.【点睛】此题考查了平行四边形的性质、含$30^{circ}$角的直角三角形的性质等知识,熟练掌握平行四边形的性质是解题的关键.", "solution": "null", "level": "八年级", "question": "如图,平行四边形$ABCD$中,$AEperpBC,AFperpCD$,垂足分别是$E、F,angleEAF=60^{circ},BE=2$,$DF=3$,则平行四边形$ABCD$的周长为20", "options": [], "subject": "度量几何学", "analysis": "由平行四边形的性质得$AB//CD,AD//BC,AB=CD,AD=BC$,再证$angleBAE=angleDAF=$$30^{circ}$,然后由含$30^{circ}$角的直角三角形的性质得$AB=2BE=4,AD=2DF=6$,即可解决问题.【解析】解:$because$四边形$ABCD$是平行四边形,$thereforeAB//CD,AD//BC,AB=CD,AD=BC$$becauseAEperpBC,AFperpCD$,$thereforeangleAEB=angleAFD=90^{circ},AFperpAB,AEperpAD$,$thereforeangleBAF=angleDAE=90^{circ}$,$becauseangleEAF=60^{circ}$,$thereforeangleBAE=angleDAF=90^{circ}-60^{circ}=30^{circ}$,$thereforeAB=2BE,AD=2DF$$becauseBE=2,DF=3$,$thereforeCD=AB=4,BC=AD=6$,$thereforesquareABCD$的周长$=2(AB+BC)=2times(4+6)=20$,故答案为:20.【点睛】此题考查了平行四边形的性质、含$30^{circ}$角的直角三角形的性质等知识,熟练掌握平行四边形的性质是解题的关键."} {"id": "5152", "image": ["6680.jpg", "6680.jpg"], "answer": "根据平行四边形的性质求出$OA、OB$,根据三角形的三边关系定理得到$OA-OB【点睛】本题考查对平行四边形的性质,三角形的三边关系定理等知识点的理解和掌握,求出$OA、$$OB$后得出$OA-OB【点睛】本题考查对平行四边形的性质,三角形的三边关系定理等知识点的理解和掌握,求出$OA、$$OB$后得出$OA-OB【点睛】本题考查了平行四边形的性质,掌握平行四边形的性质,勾股定理等知识是解题的关键.", "solution": "null", "level": "八年级", "question": "平行四边形$ABCD$的面积为$10sqrt{77}$,其中$angleA$为锐角,$AE、AF$分别为$BC、CD$上的高,若$AB$$=5sqrt{11},BC=2sqrt{11}$,则$CE+CF$的长为$7sqrt{11}+14$", "options": [], "subject": "度量几何学", "analysis": "先利用平行四边形的面积公式求出$AE$和$AF$,再利用勾股定理求出$BE$和$DF$,即可求解.【解析】解:如图,作$AEperpBC,AFperpCD$,垂足分别为$E、F$,$because$四边形$ABCD$是平行四边形,$therefore{AB}={CD}=5sqrt{11},{AD}={BC}=2sqrt{11}$,$because$平行四边形的面积为$S={AB}cdot{AF}={BC}cdot{AE}$,$therefore5sqrt{11}{AF}=2sqrt{11}{AE}=10sqrt{77}$,$therefore{AF}=2sqrt{7},quad{AE}=5sqrt{7}$,$becauseAEperpBC,AFperpCD$,$therefore{BE}=sqrt{{AB}^{2}-{AE}^{2}}=10,quad{DF}=sqrt{{AD}^{2}-{AF}^{2}}=4$,$therefore{CE}={BC}+{BE}=2sqrt{11}+10,{CF}={CD}+{DF}=5sqrt{11}+4$,$therefore{CE}+{CF}=7sqrt{11}+14$,故答案为:$7sqrt{11}+14$.【点睛】本题考查了平行四边形的性质,掌握平行四边形的性质,勾股定理等知识是解题的关键."} {"id": "5172", "image": ["6719.jpg"], "answer": "由菱形的性质可得$AO=CO=8{~cm},ACperpBD$,由勾股定理可求$BO$的长,即可求解.【解析】解:$because$四边形$ABCD$是菱形,$thereforeAO=CO=8{~cm},ACperpBD$,$thereforeBO=sqrt{{AB}^{2}-{AO}^{2}}=sqrt{100-64}=6({~cm})$,$thereforeBD=12{~cm}$,$therefore$菱形$ABCD$的面积$=frac{{AC}times{BD}}{2}=frac{16times12}{2}=96left({~cm}^{2}right),$故答案为:96.【点睛】本题考查了菱形的性质,勾股定理,掌握菱形的对角线互相垂直平分是解题的关键.", "solution": "null", "level": "八年级", "question": "如图,菱形$ABCD$的边长为$10{~cm}$,其中对角线$AC$的长为$16{~cm}$,则菱形$ABCD$的面积为${cm}^{2}$.", "options": [], "subject": "度量几何学", "analysis": "由菱形的性质可得$AO=CO=8{~cm},ACperpBD$,由勾股定理可求$BO$的长,即可求解.【解析】解:$because$四边形$ABCD$是菱形,$thereforeAO=CO=8{~cm},ACperpBD$,$thereforeBO=sqrt{{AB}^{2}-{AO}^{2}}=sqrt{100-64}=6({~cm})$,$thereforeBD=12{~cm}$,$therefore$菱形$ABCD$的面积$=frac{{AC}times{BD}}{2}=frac{16times12}{2}=96left({~cm}^{2}right),$故答案为:96.【点睛】本题考查了菱形的性质,勾股定理,掌握菱形的对角线互相垂直平分是解题的关键."} {"id": "5173", "image": ["6720.jpg"], "answer": "根据矩形的性质推出$OA=OB$,根据角平分线求出$AB=BE$,得到等边三角形$OAB$,推出$angleOBC=angleOCB=30^{circ},OB=BE$,求出$angleBOE$的度数即可求出答案.【解析】解:在矩形$ABCD$中,$AO=BO=CO=DO,angleABC=90^{circ}$,$becauseangleCAE=15^{circ},AE$平分$angleBAD$,$thereforeangleBAE=angleBEA=45^{circ}$,$thereforeAB=BE$,$thereforeangleBAC=60^{circ},OA=OB$,$thereforetriangleAOB$是等边三角形,$thereforeangleBAC=60^{circ}$,$thereforeangleBCA=30^{circ},AB=frac{1}{2}AC=BO$,$thereforeBE=BO$,又$becauseangleDBC=angleACB=30^{circ}$,在$triangleBOE$中,$angleBOE=left(180^{circ}-angleDBCright)div2=75^{circ}$.故答案为:75.【点睛】本题主要考查对三角形的内角和定理,角平分线性质,等腰三角形的性质,等边三角形的性质和判定,矩形的性质,平行线的性质等知识点的理解和掌握,能求出$angleBOE$和$angleAOB$的度数是解此题的关键.", "solution": "null", "level": "八年级", "question": "如图,矩形$ABCD$的对角线相交于点$O,AE$平分$angleBAD$交$BC$于点$E$,若$angleCAE=15^{circ}$,则$angle$$BOE=underline{75}$度.", "options": [], "subject": "度量几何学", "analysis": "根据矩形的性质推出$OA=OB$,根据角平分线求出$AB=BE$,得到等边三角形$OAB$,推出$angleOBC=angleOCB=30^{circ},OB=BE$,求出$angleBOE$的度数即可求出答案.【解析】解:在矩形$ABCD$中,$AO=BO=CO=DO,angleABC=90^{circ}$,$becauseangleCAE=15^{circ},AE$平分$angleBAD$,$thereforeangleBAE=angleBEA=45^{circ}$,$thereforeAB=BE$,$thereforeangleBAC=60^{circ},OA=OB$,$thereforetriangleAOB$是等边三角形,$thereforeangleBAC=60^{circ}$,$thereforeangleBCA=30^{circ},AB=frac{1}{2}AC=BO$,$thereforeBE=BO$,又$becauseangleDBC=angleACB=30^{circ}$,在$triangleBOE$中,$angleBOE=left(180^{circ}-angleDBCright)div2=75^{circ}$.故答案为:75.【点睛】本题主要考查对三角形的内角和定理,角平分线性质,等腰三角形的性质,等边三角形的性质和判定,矩形的性质,平行线的性质等知识点的理解和掌握,能求出$angleBOE$和$angleAOB$的度数是解此题的关键."} {"id": "5174", "image": ["6721.jpg", "6722.jpg", "6722.jpg"], "answer": "由勾股定理求出$BC$的长,再证明四边形$DMAN$是矩形,可得$MN=AD$,根据垂线段最短和三角形面积即可解决问题.【解析】解:连接$AD$,$becauseangleBAC=90^{circ}$,且$BA=5,AC=12$,$therefore{BC}=sqrt{{BA}^{2}+{AC}}{}^{2}=13$,$becauseDMperpAB,DNperpAC$,$thereforeangleDMA=angleDNA=angleBAC=90^{circ}$,$therefore$四边形$DMAN$是矩形,$thereforeMN=AD$,$therefore$当$ADperpBC$时,$AD$的值最小,此时,$triangleABC$的面积$=frac{1}{2}{AB}timesAC=frac{1}{2}BCtimesAD$,$thereforeAD=frac{ABtimesAC}{BC}=frac{60}{13}$,$thereforeMN$的最小值为$frac{60}{13}$;故答案为:$frac{60}{13}$.【点睛】本题考查了矩形的判定和性质、勾股定理、三角形面积、垂线段最短等知识,解题的关键是熟练掌握基本知识,属于中考常考题型.", "solution": "null", "level": "八年级", "question": "如图,在Rt$triangleABC$中,$angleBAC=90^{circ}$,且$BA=5,AC=12$,点$D$是斜边$BC$上的一个动点,过点$D$分别作$DMperpAB$于点$M,DNperpAC$于点$N$,连接$MN$,则线段$MN$的最小值为$-frac{60}{13}-$.", "options": [], "subject": "度量几何学", "analysis": "由勾股定理求出$BC$的长,再证明四边形$DMAN$是矩形,可得$MN=AD$,根据垂线段最短和三角形面积即可解决问题.【解析】解:连接$AD$,$becauseangleBAC=90^{circ}$,且$BA=5,AC=12$,$therefore{BC}=sqrt{{BA}^{2}+{AC}}{}^{2}=13$,$becauseDMperpAB,DNperpAC$,$thereforeangleDMA=angleDNA=angleBAC=90^{circ}$,$therefore$四边形$DMAN$是矩形,$thereforeMN=AD$,$therefore$当$ADperpBC$时,$AD$的值最小,此时,$triangleABC$的面积$=frac{1}{2}{AB}timesAC=frac{1}{2}BCtimesAD$,$thereforeAD=frac{ABtimesAC}{BC}=frac{60}{13}$,$thereforeMN$的最小值为$frac{60}{13}$;故答案为:$frac{60}{13}$.【点睛】本题考查了矩形的判定和性质、勾股定理、三角形面积、垂线段最短等知识,解题的关键是熟练掌握基本知识,属于中考常考题型."} {"id": "5195", "image": ["6751.jpg", "6752.jpg", "6752.jpg"], "answer": "如图,作$DH//MN$,先证明$triangleADHcongtriangleBAE$推出$MNperpAE$,在$RTtriangleAFM$中求出$AM$即可,再根据对称性求出$AM^{prime}$,由此即可解决问题.【解析】解:如图,作$DH//MN$,$because$四边形$ABCD$是正方形,$thereforeAD=AB,angleDAB=angleB=90^{circ},AB//CD$,$therefore$四边形$DHMN$是平行四边形,$thereforeDH=MN=AE$,在$RTtriangleADH$和$RTtriangleBAE$中,$left{begin{array}{l}{AD}={AB}{DH}={AE}end{array}right.$$thereforetriangleADHcongtriangleBAE$,$thereforeangleADH=angleBAE$,$thereforeangleADH+angleAHD=angleADH+angleAMN=90^{circ}$$thereforeangleBAE+angleAMN=90^{circ}$,$thereforeangleAFM=90^{circ}$,在$RTtriangleABE$中,$becauseangleB=90^{circ},AB=sqrt{3},angleBAE=30^{circ}$,$thereforeAEcdotcos30^{circ}=AB$,$thereforeAE=2$,在$RTtriangleAFM$中,$becauseangleAFM=90^{circ},AF=1,angleFAM=30^{circ}$,$thereforeAMcdotcos30^{circ}=AF$$thereforeAM=frac{2sqrt{3}}{3}$,根据对称性当$M^{prime}N^{prime}=AE$时,$BM^{prime}=frac{2sqrt{3}}{3},AM^{prime}=AB-BM^{prime}=sqrt{3}-frac{2sqrt{3}}{3}=frac{sqrt{3}}{3}$故答案为$frac{sqrt{3}}{3}$或$frac{2sqrt{3}}{3}$.【点睛】本题科学正方形的性质、全等三角形的判定和性质、锐角三角函数、勾股定理等知识,解题的关键是学会添加常用辅助线,构造全等三角形,属于中考常考题型.", "solution": "null", "level": "八年级", "question": "如图,正方形$ABCD$的面积为$3{~cm}^{2},E$为$BC$边上一点,$angleBAE=30^{circ},F$为$AE$的中点,过点$F$作直线分别与$AB,DC$相交于点$M,N$.若$MN=AE$,则$AM$的长等于$-frac{sqrt{3}}{3}$或$frac{2sqrt{3}}{3}_cm$.", "options": [], "subject": "度量几何学", "analysis": "如图,作$DH//MN$,先证明$triangleADHcongtriangleBAE$推出$MNperpAE$,在$RTtriangleAFM$中求出$AM$即可,再根据对称性求出$AM^{prime}$,由此即可解决问题.【解析】解:如图,作$DH//MN$,$because$四边形$ABCD$是正方形,$thereforeAD=AB,angleDAB=angleB=90^{circ},AB//CD$,$therefore$四边形$DHMN$是平行四边形,$thereforeDH=MN=AE$,在$RTtriangleADH$和$RTtriangleBAE$中,$left{begin{array}{l}{AD}={AB}{DH}={AE}end{array}right.$$thereforetriangleADHcongtriangleBAE$,$thereforeangleADH=angleBAE$,$thereforeangleADH+angleAHD=angleADH+angleAMN=90^{circ}$$thereforeangleBAE+angleAMN=90^{circ}$,$thereforeangleAFM=90^{circ}$,在$RTtriangleABE$中,$becauseangleB=90^{circ},AB=sqrt{3},angleBAE=30^{circ}$,$thereforeAEcdotcos30^{circ}=AB$,$thereforeAE=2$,在$RTtriangleAFM$中,$becauseangleAFM=90^{circ},AF=1,angleFAM=30^{circ}$,$thereforeAMcdotcos30^{circ}=AF$$thereforeAM=frac{2sqrt{3}}{3}$,根据对称性当$M^{prime}N^{prime}=AE$时,$BM^{prime}=frac{2sqrt{3}}{3},AM^{prime}=AB-BM^{prime}=sqrt{3}-frac{2sqrt{3}}{3}=frac{sqrt{3}}{3}$故答案为$frac{sqrt{3}}{3}$或$frac{2sqrt{3}}{3}$.【点睛】本题科学正方形的性质、全等三角形的判定和性质、锐角三角函数、勾股定理等知识,解题的关键是学会添加常用辅助线,构造全等三角形,属于中考常考题型."} {"id": "5196", "image": ["6753.jpg", "6753.jpg"], "answer": "如图当点$E$在$BD$右侧时,求出$angleEBD,angleDBC$即可解决问题,当点$E$在$BD$左侧时,求出$angleDBE^{prime}$即可解决问题.【解析】解:如图,$because$四边形$ABCD$是菱形,$thereforeAB=AD=BC=CD,angleA=angleC=30^{circ}$,$angleABC=angleADC=150^{circ}$,$thereforeangleDBA=angleDBC=75^{circ}$,$becauseED=EB,angleDEB=120^{circ}$,$thereforeangleEBD=angleEDB=30^{circ}$,$thereforeangleEBC=angleEBD+angleDBC=105^{circ}$,当点$E^{prime}$在$BD$右侧时,$becauseangleDBE^{prime}=30^{circ}$,$thereforeangleE^{prime}BC=angleDBC-angleDBE^{prime}=45^{circ}$,$thereforeangleEBC=105^{circ}$或$45^{circ}$,故答案为$105^{circ}$或$45^{circ}$.【点睛】本题考查菱形的性质、等腰三角形的性质等知识,解题的关键是正确画出图形,考虑问题要全面,属于中考常考题型.", "solution": "null", "level": "八年级", "question": "在菱形$ABCD$中,$angleA=30^{circ}$,在同一平面内,以对角线$BD$为底边作顶角为$120^{circ}$的等腰三角形$BDE$,则$angleEBC$的度数为$45^{circ}$或$105^{circ}$", "options": [], "subject": "度量几何学", "analysis": "如图当点$E$在$BD$右侧时,求出$angleEBD,angleDBC$即可解决问题,当点$E$在$BD$左侧时,求出$angleDBE^{prime}$即可解决问题.【解析】解:如图,$because$四边形$ABCD$是菱形,$thereforeAB=AD=BC=CD,angleA=angleC=30^{circ}$,$angleABC=angleADC=150^{circ}$,$thereforeangleDBA=angleDBC=75^{circ}$,$becauseED=EB,angleDEB=120^{circ}$,$thereforeangleEBD=angleEDB=30^{circ}$,$thereforeangleEBC=angleEBD+angleDBC=105^{circ}$,当点$E^{prime}$在$BD$右侧时,$becauseangleDBE^{prime}=30^{circ}$,$thereforeangleE^{prime}BC=angleDBC-angleDBE^{prime}=45^{circ}$,$thereforeangleEBC=105^{circ}$或$45^{circ}$,故答案为$105^{circ}$或$45^{circ}$.【点睛】本题考查菱形的性质、等腰三角形的性质等知识,解题的关键是正确画出图形,考虑问题要全面,属于中考常考题型."} {"id": "5198", "image": ["6756.jpg", "6757.jpg", "6758.jpg", "6757.jpg", "6758.jpg"], "answer": "连接$AH$,由四边形$ABCD$是正方形与点$E、F、H$分别是$AB、BC、CD$的中点,易证得$triangleBCEcongtriangleCDF$与$triangleADHcongtriangleDCF$,根据全等三角形的性质,易证得$CEperpDF$与$AHperpDF$,根据垂直平分线的性质,即可证得$AG=AD$,由直角三角形斜边上的中线等于斜边的一半,即可证得$HG=frac{1}{2}AD$,根据等腰三角形的性质,即可得$angleCHG=angleDAG$.则问题得解.【解析】解:$because$四边形$ABCD$是正方形,$thereforeAB=BC=CD=AD,angleB=angleBCD=90^{circ}$,$because$点$E、F、H$分别是$AB、BC、CD$的中点,$thereforeBE=CF$,在$triangleBCE$与$triangleCDF$中,$thereforetriangleBCEcongtriangleCDF$,(SAS)$thereforeangleECB=angleCDF$,$becauseangleBCE+angleECD=90^{circ}$$thereforeangleECD+angleCDF=90^{circ}$,$thereforeangleCGD=90^{circ}$,$thereforeCEperpDF$,故(1)正确;在Rt$triangleCGD$中,$H$是$CD$边的中点,$thereforeHG=frac{1}{2}CD=frac{1}{2}AD$,故(4)正确;如图,连接${A}H$,同理可得:$AHperpDF$,$becauseHG=HD=frac{1}{2}CD$,$thereforeDK=GK$,$thereforeAH$垂直平分$DG$,$thereforeAG=AD$,故(2)正确;$thereforeangleDAG=2angleDAH$,同理:$triangleADHcongtriangleDCF$,$thereforeangleDAH=angleCDF$,$becauseGH=DH$,$thereforeangleHDG=angleHGD$,$thereforeangleGHC=angleHDG+angleHGD=2angleCDF$,$thereforeangleCHG=angleDAG$.故(3)正确.综上所述:正确的有:(1)(2)(3)(4).故答案为:(1)(2)(4).【点睛】此题考查了正方形的性质,全等三角形的判定与性质,等腰三角形的性质以及垂直平分线的性质等知识.此题综合性很强,难度较大,解题的关键是注意数形结合思想的应用.", "solution": "null", "level": "八年级", "question": "如图,正方形$ABCD$中,点$E、F、H$分别是$AB、BC、CD$的中点,$CE、DF$交于$G$,连接$AG、$$HG$.下列结论:(1)$CEperpDF$;(2)$AG=AD$;(3)$angleCHG=angleDAG$;(4)$HG=frac{1}{2}AD$.其中正确的有(2)(3)(4)", "options": [], "subject": "度量几何学", "analysis": "连接$AH$,由四边形$ABCD$是正方形与点$E、F、H$分别是$AB、BC、CD$的中点,易证得$triangleBCEcongtriangleCDF$与$triangleADHcongtriangleDCF$,根据全等三角形的性质,易证得$CEperpDF$与$AHperpDF$,根据垂直平分线的性质,即可证得$AG=AD$,由直角三角形斜边上的中线等于斜边的一半,即可证得$HG=frac{1}{2}AD$,根据等腰三角形的性质,即可得$angleCHG=angleDAG$.则问题得解.【解析】解:$because$四边形$ABCD$是正方形,$thereforeAB=BC=CD=AD,angleB=angleBCD=90^{circ}$,$because$点$E、F、H$分别是$AB、BC、CD$的中点,$thereforeBE=CF$,在$triangleBCE$与$triangleCDF$中,$thereforetriangleBCEcongtriangleCDF$,(SAS)$thereforeangleECB=angleCDF$,$becauseangleBCE+angleECD=90^{circ}$$thereforeangleECD+angleCDF=90^{circ}$,$thereforeangleCGD=90^{circ}$,$thereforeCEperpDF$,故(1)正确;在Rt$triangleCGD$中,$H$是$CD$边的中点,$thereforeHG=frac{1}{2}CD=frac{1}{2}AD$,故(4)正确;如图,连接${A}H$,同理可得:$AHperpDF$,$becauseHG=HD=frac{1}{2}CD$,$thereforeDK=GK$,$thereforeAH$垂直平分$DG$,$thereforeAG=AD$,故(2)正确;$thereforeangleDAG=2angleDAH$,同理:$triangleADHcongtriangleDCF$,$thereforeangleDAH=angleCDF$,$becauseGH=DH$,$thereforeangleHDG=angleHGD$,$thereforeangleGHC=angleHDG+angleHGD=2angleCDF$,$thereforeangleCHG=angleDAG$.故(3)正确.综上所述:正确的有:(1)(2)(3)(4).故答案为:(1)(2)(4).【点睛】此题考查了正方形的性质,全等三角形的判定与性质,等腰三角形的性质以及垂直平分线的性质等知识.此题综合性很强,难度较大,解题的关键是注意数形结合思想的应用."} {"id": "4079", "image": ["5756.jpg"], "answer": "3", "solution": "null", "level": "八年级", "question": ", 在 $\\triangle A B C$ 中, $\\angle C=90^{\\circ}, B D$ 平分 $\\angle A B C, C D=3$, 则点 $D$ 到 $A B$ 的距离是\n\n", "options": [], "subject": "度量几何学", "analysis": ""} {"id": "4080", "image": ["5757.jpg"], "answer": "$A B=A C($ 不唯一 $)$", "solution": "null", "level": "八年级", "question": "如图, 已知 $\\angle 1=\\angle 2$, 请你添加一个条件 \\$ \\qquad \\$ ,使得 $\\triangle A B D \\cong \\triangle A C D . \\quad($ 添一个即可 $)$\n\n", "options": [], "subject": "度量几何学", "analysis": ""} {"id": "4081", "image": ["5758.jpg"], "answer": "75.", "solution": "null", "level": "八年级", "question": "副三角板如图表示摆放, 那么图中 $\\angle \\alpha=$ \\$ \\qquad \\$度\n\n", "options": [], "subject": "度量几何学", "analysis": ""} {"id": "4083", "image": ["5760.jpg"], "answer": "$80^{\\circ}$", "solution": "null", "level": "八年级", "question": "如图, 已知 $\\angle B D C=142^{\\circ}, \\angle B=34^{\\circ}, \\angle C=28^{\\circ}$, 则 $\\angle A=$\n\n", "options": [], "subject": "度量几何学", "analysis": ""} {"id": "4084", "image": ["5761.jpg"], "answer": "$\\frac{9}{4}$", "solution": "null", "level": "八年级", "question": "如图, 在 $\\triangle A B C$ 中, 点 $D$ 为 $B C$ 上一点, $E, F$ 分别为 $A D, B E$ 的中点, 且 $S \\triangle A B C=9$,则图中阴影部分 $\\triangle C E F$ 的面积是\n\n", "options": [], "subject": "度量几何学", "analysis": ""} {"id": "4086", "image": ["5763.jpg"], "answer": "2^{\\circ}$", "solution": "null", "level": "八年级", "question": ", 锐角 $\\triangle A B C$ 中, 直线 $l$ 为 $B C$ 的中垂线, $B M$ 为的 $\\angle A B C$ 角平分线, $l$ 与 $B M$ 相交于点 $P$.若 $\\angle A=60^{\\circ}, \\angle A C P=24^{\\circ}$, 则 $\\angle A B P$ 的度数为 \\$ \\qquad \\$ .\n\n", "options": [], "subject": "度量几何学", "analysis": ""} {"id": "4105", "image": ["5789.jpg"], "answer": "14", "solution": "null", "level": "八年级", "question": "如图, 在高为 6 米, 坡面长度 $A B$ 为 10 米的楼梯表面铺上地毯, 则至少需要地毯米.\n\n", "options": [], "subject": "度量几何学", "analysis": ""} {"id": "4110", "image": ["5792.jpg"], "answer": "16", "solution": "null", "level": "八年级", "question": "如图, 在 $\\triangle A B C$ 中, $\\angle C=90^{\\circ}, A C=2, B C=4$.\n\n以 $A B$ 为一边在 $\\triangle A B C$ 的同侧作正方形 $A B D E$, 则图中阴影部分的面积为\n\n", "options": [], "subject": "度量几何学", "analysis": ""} {"id": "4128", "image": [], "answer": "$\\sqrt{3}$", "solution": "null", "level": "八年级", "question": "边三角形的边长为 $2 \\mathrm{~cm}$, 则该等边三角形的面积是 $\\mathrm{cm}^{2}$.", "options": [], "subject": "度量几何学", "analysis": ""} {"id": "4130", "image": ["5810.jpg"], "answer": "$A B=D C$", "solution": "null", "level": "八年级", "question": "如图, 在 $R t \\triangle A B C$ 与 $R t \\triangle D C B$ 中, 已知 $\\angle A=\\angle D=90^{\\circ}$, 请你添加一个条件 (不添加字母和辅助线),使 Rt $\\triangle A B C \\xlongequal{\\text { 尺 }}$ R $\\triangle D C B$, 你添加的条件是\n\n", "options": [], "subject": "度量几何学", "analysis": ""} {"id": "4131", "image": ["5811.jpg"], "answer": "$250^{\\circ}$", "solution": "null", "level": "八年级", "question": "如图, 在 $\\triangle A B C$ 中, $\\angle C=70^{\\circ}$, 沿图中虚线截去 $\\angle C$, 则 $\\angle 1+\\angle 2=$\n\n", "options": [], "subject": "度量几何学", "analysis": ""} {"id": "4134", "image": ["5813.jpg"], "answer": "9", "solution": "null", "level": "八年级", "question": ", 在 $\\triangle A B C$ 中, $A B=A C, D E$ 是 $A B$ 的垂直平分线, $\\triangle B C E$ 的周长为 $16, B C=7$, 则 $A B$ 的长为\n\n", "options": [], "subject": "度量几何学", "analysis": ""} {"id": "4151", "image": ["5837.jpg"], "answer": "40", "solution": "null", "level": "八年级", "question": "如图, 一般轮船按箭头所示方向行驶, $C$ 处有一灯塔, 当轮船从 $A$ 点行驶到 $B$ 点时, $\\angle A C B=$\n\n", "options": [], "subject": "度量几何学", "analysis": ""} {"id": "5750", "image": [], "answer": "-2", "solution": "null", "level": "八年级", "question": "已知 $x, y$ 为实数, $y=\\frac{\\sqrt{x^{2}-9}+\\sqrt{9-x^{2}+1}}{x-3}$, 则 $x-6 y$ 的值", "options": [], "subject": "解析几何", "analysis": "要使 $y=\\frac{\\sqrt{x^{2}-9}+\\sqrt{9-x^{2}+1}}{x-3}$ 有意义, 则有\n\n$\\binom{x^{2}=9}{x-3 \\neq 0}$, 解得 $x=-3$,\n\n则 $\\mathrm{y}=-{ }^{\\frac{1}{6}}$\n\n将 $x=-3, y=-\\frac{1}{\\frac{1}{6}}$ 代入 $x-6 y$ 得,\n\n$x-6 y=-3-6 \\times\\left(-\\frac{1}{\\frac{1}{6}}\\right)=-2$."} {"id": "5775", "image": [], "answer": "8", "solution": "null", "level": "八年级", "question": "已知凸 $\\mathrm{n}$ 边形的每一个外角均为 $45^{\\circ}$, 则 $\\mathrm{n}=$", "options": [], "subject": "解析几何", "analysis": "$\\mathrm{n}=360^{\\circ} \\div 45^{\\circ}=8$.\n\n故答案为: 8 ."} {"id": "5778", "image": ["7182.jpg"], "answer": "$(2,-1)$", "solution": "null", "level": "八年级", "question": "以 $A B C D$ 对角线的交点 $O$ 为原点, 平行于 $B C$ 边的直线为 $x$ 轴, 建立如图所示的平面直角坐标系. 若 $\\mathrm{A}$ 点坐标为 $(-2,1)$, 则 $\\mathrm{C}$ 点坐标为 \\$ \\qquad \\$ .\n\n", "options": [], "subject": "解析几何", "analysis": "$\\because \\mathrm{ABCD}$ 对角线的交点 $\\mathrm{O}$ 为原点, $\\mathrm{A}$ 点坐标为 $(-2,1)$,\n\n$\\therefore$ 点 C 的坐标为 $(2,-1)$,\n\n故答案为: $(2,-1)$."} {"id": "5779", "image": ["7183.jpg"], "answer": "", "solution": "null", "level": "八年级", "question": "在平面直角坐标系中, $A 、 B 、 C$ 三点的坐标分别为 $(0,0),(0,-5),(-2,2)$, 以这三点为平行四边形的三个顶点, 则第四个顶点不可能在第 \\$ \\qquad \\$象限.", "options": [], "subject": "解析几何", "analysis": "如图,\n\n以 $\\mathrm{AB}$ 为对角线时, 第四个顶点 $(2,-3)$ 在第四象限,\n\n以 $\\mathrm{AC}$ 为对角线时, 第四个顶点在 $(-2,3)$ 第二象限,\n\n以 $\\mathrm{BC}$ 为对角线时, 第四个顶点 $(-2,-7)$ 在第三象限,\n\n故第四个顶点不可能在第一象限,\n\n故答案为: 一.\n\n"} {"id": "5780", "image": ["7184.jpg"], "answer": "矩形", "solution": "null", "level": "八年级", "question": "顺次连接对角线互相垂直的四边形各边中点, 得到的四边形是", "options": [], "subject": "解析几何", "analysis": "如图: $\\because \\mathrm{E} 、 \\mathrm{~F}$ 分别为 $\\mathrm{AB} 、 \\mathrm{BC}$ 的中点,\n$\\therefore \\mathrm{EF}$ 是 $\\triangle \\mathrm{ABC}$ 的中位线,\n\n$\\therefore \\mathrm{EF} / / \\mathrm{AC}, \\mathrm{EF}={ }^{\\frac{1}{2}} \\mathrm{AC}$,\n\n同理: $\\mathrm{GH} / / \\mathrm{AC}, \\mathrm{GH}={ }^{\\frac{1}{2}} \\mathrm{AC}$,\n\n$\\therefore \\mathrm{EF}-\\mathrm{GH}$,\n\n$\\therefore$ 四边形 $\\mathrm{EFGH}$ 是平行四边形,\n\n又 $\\mathrm{EH} / / \\mathrm{BD}, \\mathrm{AC} \\perp \\mathrm{BD}$,\n\n$\\therefore \\mathrm{EF} \\perp \\mathrm{EH}$,\n\n$\\therefore$ 平行四边形 $\\mathrm{EFGH}$ 是矩形.\n\n"} {"id": "5782", "image": [], "answer": "10", "solution": "null", "level": "八年级", "question": "如果正 $n$ 边形的每一个内角都等于 $144^{\\circ}$, 那么 $n=$", "options": [], "subject": "解析几何", "analysis": "首先求得外角的度数, 然后利用 360 度除以外角的度数即可求得.\n\n外角的度数是: $180^{\\circ}-144^{\\circ}=36^{\\circ}$,\n\n则 $n=\" 360 \\div 36 \"=10$."} {"id": "5809", "image": [], "answer": "7 .", "solution": "null", "level": "八年级", "question": "一个多边形的内角和为 $900^{\\circ}$, 则这个多边形的边数为 \\$ \\qquad \\$ .", "options": [], "subject": "解析几何", "analysis": "设这个多边形的边数为 $n$, 则有\n\n$(\\mathrm{n}-2) \\times 180^{\\circ}=900^{\\circ}$,\n\n解得: $\\mathrm{n}=7$,\n\n$\\therefore$ 这个多边形的边数为 7 .\n\n故答案为: 7 ."} {"id": "5814", "image": ["7214.jpg", "7215.jpg"], "answer": "$\\frac{96 \\sqrt{5}-192}{5}$", "solution": "null", "level": "八年级", "question": "在正方形 $A B C D$ 中, $A B=4 \\sqrt{5}, E$ 为 $B C$ 的中点, 连接 $A E$, 点 $F$ 为 $A E$ 上一点, 且 $E F=2 . F G \\perp A E$交 $D C$ 于 $G$, 将 $F G$ 绕着点 $G$ 顺时针旋转, 使得点 $F$ 恰好落在 $A D$ 上的点 $H$ 处, 过点 $H$ 作 $H N \\perp H G$,交 $\\mathrm{AB}$ 于 $\\mathrm{N}$, 交 $\\mathrm{AE}$ 于 $\\mathrm{M}$, 则 $\\mathrm{S}_{\\triangle \\mathrm{MNF}}=$\n\n", "options": [], "subject": "解析几何", "analysis": "如图, $\\because$ 四边形 $\\mathrm{ABCD}$ 是正方形,\n\n\n\n$\\therefore \\angle \\mathrm{B}=90^{\\circ}, \\mathrm{AB}=\\mathrm{BC}=4 \\sqrt{5}$,\n\n$\\because \\mathrm{E}$ 为 $\\mathrm{BC}$ 的中点,\n\n$\\therefore B E=\\frac{1}{2} B C=2 \\sqrt{5}$\n\n由勾股定理得:\n\n$$\nA E=\\sqrt{(4 \\sqrt{5})^{2}+(2 \\sqrt{5})^{2}}=10\n$$\n\n$\\because \\mathrm{EF}=2$\n\n$\\therefore \\mathrm{AF}=10-2=8$,\n\n过 $F$ 作 $P Q / / B C$, 交 $A B$ 于 $P$, 交 $C D$ 于 $Q$,\n\n$\\therefore \\triangle \\mathrm{APF} \\sim \\triangle \\mathrm{ABE}$,\n\n$\\therefore \\frac{P F}{B E}=\\frac{A F}{A E}=\\frac{A P}{A B}$\n\n$\\therefore \\frac{P F}{2 \\sqrt{5}}=\\frac{8}{10}=\\frac{A P}{4 \\sqrt{5}}$\n\n$\\therefore P F=\\frac{8 \\sqrt{5}}{5}, A P=\\frac{16 \\sqrt{5}}{5}$\n\n$\\therefore P B=C Q=4 \\sqrt{5}-\\frac{16 \\sqrt{5}}{5}=\\frac{4 \\sqrt{5}}{5}$\n\n$F Q=4 \\sqrt{5}-\\frac{8 \\sqrt{5}}{5}=\\frac{12 \\sqrt{5}}{5}$\n\n$\\because \\angle \\mathrm{AFG}=90^{\\circ}$,\n\n易得 $\\triangle A P F \\backsim \\triangle F Q G$,\n\n$\\therefore G Q=\\frac{6 \\sqrt{5}}{5}$\n\n$\\therefore C G=C Q+Q G=2 \\sqrt{5}=D G$\n\n由旋转得: $F G=H G$,\n\n$\\therefore \\mathrm{DH}^{2}+\\mathrm{DG}^{2}=\\mathrm{QG}^{2}+\\mathrm{FQ}^{2}$,\n\n$\\therefore D H^{2}+(2 \\sqrt{5})^{2}=\\left(\\frac{6 \\sqrt{5}}{5}\\right)^{2}+\\left(\\frac{12 \\sqrt{5}}{5}\\right)^{2}$\n\n$\\therefore \\mathrm{DH}=4$,\n\n$\\therefore A H=4 \\sqrt{5}-4$\n\n$\\because \\angle \\mathrm{NHG}=90^{\\circ}$,\n\n同理 $\\triangle \\mathrm{NAH} \\backsim \\triangle \\mathrm{HDG}$,\n\n$\\therefore \\frac{A N}{A H}=\\frac{D H}{D G}$\n\n$\\therefore \\frac{A N}{4 \\sqrt{5}-4}=\\frac{4}{2 \\sqrt{5}}$\n$\\therefore A N=\\frac{40-8 \\sqrt{5}}{5}$\n\n过 $\\mathrm{M}$ 作 $\\mathrm{MK} \\perp \\mathrm{AB}$ 于 $\\mathrm{K}$,\n\n$\\because \\angle \\mathrm{ANH}=\\angle \\mathrm{GHD}$,\n\n$\\therefore \\tan \\angle A N H=\\tan \\angle G H D=\\frac{M K}{N K}=\\frac{D G}{D H}=\\frac{2 \\sqrt{5}}{4}$\n\n设 $M K=2 \\sqrt{5} x, \\mathrm{NK}=4 \\mathrm{x}$ ,\n\n$\\because \\mathrm{MK} / / \\mathrm{BE}$,\n\n$\\therefore \\frac{M K}{A K}=\\frac{B E}{A B}=\\frac{1}{2}$\n\n$\\therefore A K=4 \\sqrt{5} x$\n\n$\\because \\mathrm{AN}=\\mathrm{AK}+\\mathrm{KN}$,\n\n$\\therefore \\frac{40-8 \\sqrt{5}}{5}=4 \\sqrt{5} x+4 x$\n\n$x=\\frac{3 \\sqrt{5}}{5}-1$\n\n$\\therefore S_{\\triangle M N F}=S_{\\triangle A N F}-S_{\\triangle A M N}$\n\n$=\\frac{1}{2} A N \\cdot P F-\\frac{1}{2} A N \\cdot M K$\n\n$=\\frac{1}{2} \\cdot \\frac{40-8 \\sqrt{5}}{5} \\cdot\\left[\\frac{8 \\sqrt{5}}{5}-2 \\sqrt{5}\\left(\\frac{3 \\sqrt{5}}{5}-1\\right)\\right]$\n\n$=\\frac{96 \\sqrt{5}-192}{5}$\n\n故答案为: $\\frac{96 \\sqrt{5}-192}{5}$"} {"id": "5038", "image": [], "answer": "每个人都要与其它$(x-1)$个人握手一次,则$x$个人可握手$x(x-1)$次,但其中每两人的握手重复计算了一次,则总的握手次数为:$frac{1}{2}x(x-1)$,由握手的次数10即可得方程.【解析】解:由题意得:$frac{1}{2}x(x-1)=10$;故答案为:$frac{1}{2}x(x-1)=10$.【点睛】本题考查了列一元二次方程解应用问题,用代数式表示出握手的总次数是关键.", "solution": "null", "level": "八年级", "question": "根据下列问题列方程.问题:参加一次聚会的每两人都握了一次手,所有人共握手10次,共有多少人参加聚会?设有$x$人参加聚会,所列方程为:$-frac{1}{2}x(x-1)=10_$.", "options": [], "subject": "解析几何", "analysis": "每个人都要与其它$(x-1)$个人握手一次,则$x$个人可握手$x(x-1)$次,但其中每两人的握手重复计算了一次,则总的握手次数为:$frac{1}{2}x(x-1)$,由握手的次数10即可得方程.【解析】解:由题意得:$frac{1}{2}x(x-1)=10$;故答案为:$frac{1}{2}x(x-1)=10$.【点睛】本题考查了列一元二次方程解应用问题,用代数式表示出握手的总次数是关键."} {"id": "5060", "image": [], "answer": "设$t=x^{2}+y^{2}(tgeqslant0)$,则原方程转化为$t(t-7)=8$,然后利用因式分解法解方程求得$t$的值即可.【解析】解:设$t=x^{2}+y^{2}(tgeqslant0)$,则:$t(t-7)=8$,整理,得$(t-8)(t+1)=0$.所以$t=8$或$t=-1$(舍去).所以$x^{2}+y^{2}=8$.故答案为:8.【点睛】本题主要考查了换元法解一元二次方程,换元的实质是转化,关键是构造元和设元,理论依据是等量代换,目的是变换研究对象,将问题移至新对象的知识背景中去研究,从而使非标准型问题标准化、复杂问题简单化,变得容易处理.", "solution": "null", "level": "八年级", "question": "已知实数$x,y$满足$left(x^{2}+y^{2}right)left(x^{2}+y^{2}-7right)=8$,那么$x^{2}+y^{2}=8$.", "options": [], "subject": "解析几何", "analysis": "设$t=x^{2}+y^{2}(tgeqslant0)$,则原方程转化为$t(t-7)=8$,然后利用因式分解法解方程求得$t$的值即可.【解析】解:设$t=x^{2}+y^{2}(tgeqslant0)$,则:$t(t-7)=8$,整理,得$(t-8)(t+1)=0$.所以$t=8$或$t=-1$(舍去).所以$x^{2}+y^{2}=8$.故答案为:8.【点睛】本题主要考查了换元法解一元二次方程,换元的实质是转化,关键是构造元和设元,理论依据是等量代换,目的是变换研究对象,将问题移至新对象的知识背景中去研究,从而使非标准型问题标准化、复杂问题简单化,变得容易处理."} {"id": "5175", "image": ["6723.jpg"], "answer": "首先证明$triangleABEcongtriangleDAF$,然后结合全等三角形的性质推导$DF=AE=2,angleBGF=90^{circ}$,因为点$H$为$BF$的中点,由“直角三角形斜边上的中线等于斜边的一半”可知${GH}=frac{1}{2}{BF}$,再由勾股定理计算${BF}=sqrt{34}$,即可获得答案.【解析】解:$because$四边形$ABCD$是正方形,$thereforeAB=DA,angleBAD=angleC=90^{circ}$,$becauseBE=AF$,$therefore$Rt$triangleABEcong$Rt$triangleDAF(HL)$,$thereforeDF=AE,angleABE=angleDAF$,$becauseangleBAD=90^{circ}$,即$angleABE+angleAEB=90^{circ},$$thereforeangleDAF+angleAEB=90^{circ}$,$thereforeangleAGE=90^{circ}$,$thereforeangleBGF=angleAGE=90^{circ}$,$because$点$H$为$BF$的中点,$therefore{GH}=frac{1}{2}{BF}$,$because$正方形$ABCD$的边长为5,即$BC=DC=5$,$becauseAE=2$,$thereforeDF=AE=2$,$thereforeCF=DC-DF=5-2=3$,在Rt$triangleBCF$中,$therefore{BF}=sqrt{{BC}^{2}+{CF}^{2}}=sqrt{5^{2}+3^{2}}=sqrt{34}$,$therefore{GH}=frac{1}{2}{BF}=frac{sqrt{34}}{2}$,故答案为:$frac{sqrt{34}}{2}$.【点睛】本题主要考查了全等三角形的判定与性质、正方形的性质、勾股定理、直角三角形斜边上的中线的性质等知识,熟练掌握相关性质并灵活运用是解题的关键.", "solution": "null", "level": "八年级", "question": "如图,已知四边形$ABCD$是正方形,点$E、F$分别在$AD、DC$上,$BE$与$AF$相交于点$G$,且$BE$$=AF$.正方形$ABCD$的边长为$5,AE=2$,点$H$为$BF$的中点,$GH$的长$-frac{sqrt{34}}{2}-$.", "options": [], "subject": "解析几何", "analysis": "首先证明$triangleABEcongtriangleDAF$,然后结合全等三角形的性质推导$DF=AE=2,angleBGF=90^{circ}$,因为点$H$为$BF$的中点,由“直角三角形斜边上的中线等于斜边的一半”可知${GH}=frac{1}{2}{BF}$,再由勾股定理计算${BF}=sqrt{34}$,即可获得答案.【解析】解:$because$四边形$ABCD$是正方形,$thereforeAB=DA,angleBAD=angleC=90^{circ}$,$becauseBE=AF$,$therefore$Rt$triangleABEcong$Rt$triangleDAF(HL)$,$thereforeDF=AE,angleABE=angleDAF$,$becauseangleBAD=90^{circ}$,即$angleABE+angleAEB=90^{circ},$$thereforeangleDAF+angleAEB=90^{circ}$,$thereforeangleAGE=90^{circ}$,$thereforeangleBGF=angleAGE=90^{circ}$,$because$点$H$为$BF$的中点,$therefore{GH}=frac{1}{2}{BF}$,$because$正方形$ABCD$的边长为5,即$BC=DC=5$,$becauseAE=2$,$thereforeDF=AE=2$,$thereforeCF=DC-DF=5-2=3$,在Rt$triangleBCF$中,$therefore{BF}=sqrt{{BC}^{2}+{CF}^{2}}=sqrt{5^{2}+3^{2}}=sqrt{34}$,$therefore{GH}=frac{1}{2}{BF}=frac{sqrt{34}}{2}$,故答案为:$frac{sqrt{34}}{2}$.【点睛】本题主要考查了全等三角形的判定与性质、正方形的性质、勾股定理、直角三角形斜边上的中线的性质等知识,熟练掌握相关性质并灵活运用是解题的关键."} {"id": "5215", "image": [], "answer": "利用反比例函数图象上点的坐标特征得到$-2timesm=2times3$,然后解关于$m$的方程即可.【解析】解:$because$反比例函数$y=frac{{k}}{{x}}(kneq0)$的图象经过点$A(2,3)$和点$B(-2,m)$,$therefore-2timesm=3times2$,解得$m=-3$,即$m$的值为-3,故答案为:-3.【点睛】本题考查了反比例函数图象上点的坐标特征,掌握反比例函数$y=frac{{k}}{{x}}(k$为常数,$kneq0)$的图象是双曲线,图象上的点$(x,y)$的横纵坐标的积是定值$k$,即$xy=k$是解题的关键.", "solution": "null", "level": "八年级", "question": "在平面直角坐标系$xOy$中,若反比例函数${y}=frac{{k}}{{x}}({k}neq0)$的图象经过点$A(2,3)$和点$B(-2$,$m)$,则$m$的值为$qquad$", "options": [], "subject": "解析几何", "analysis": "利用反比例函数图象上点的坐标特征得到$-2timesm=2times3$,然后解关于$m$的方程即可.【解析】解:$because$反比例函数$y=frac{{k}}{{x}}(kneq0)$的图象经过点$A(2,3)$和点$B(-2,m)$,$therefore-2timesm=3times2$,解得$m=-3$,即$m$的值为-3,故答案为:-3.【点睛】本题考查了反比例函数图象上点的坐标特征,掌握反比例函数$y=frac{{k}}{{x}}(k$为常数,$kneq0)$的图象是双曲线,图象上的点$(x,y)$的横纵坐标的积是定值$k$,即$xy=k$是解题的关键."} {"id": "5216", "image": [], "answer": "根据反比例函数的增减性解答即可.【解析】解:$becausek=1>0$,$therefore$反比例函数$y=frac{1}{{x}}$的图象在一、三象限,$because2>0>-1$,$thereforeleft(2,y_{1}right)$在第一象限,$left(-1,y_{2}right)$在第四象限,$thereforey_{1}>y_{2}$.故答案为:$>$.【点睛】本题主要考查反比例函数图象上点的坐标特征.注意:反比例函数的增减性只指在同一象限内.", "solution": "null", "level": "八年级", "question": "已知点$left(2,y_{1}right),left(-1,y_{2}right)$都在反比例函数${y}=frac{1}{{x}}$的图象上,则$y_{1}geqy_{2}$(填“>”、“<”或$“=”)$.", "options": [], "subject": "解析几何", "analysis": "根据反比例函数的增减性解答即可.【解析】解:$becausek=1>0$,$therefore$反比例函数$y=frac{1}{{x}}$的图象在一、三象限,$because2>0>-1$,$thereforeleft(2,y_{1}right)$在第一象限,$left(-1,y_{2}right)$在第四象限,$thereforey_{1}>y_{2}$.故答案为:$>$.【点睛】本题主要考查反比例函数图象上点的坐标特征.注意:反比例函数的增减性只指在同一象限内."} {"id": "5219", "image": ["6790.jpg"], "answer": "根据$triangleAOB$的面积确定$k$的绝对值为2,再由点$A$所在象限确定$k$为-2.【解析】解:设点$A$的坐标为$(x,y)$,且$xy=k$.$because$点$A$在第二象限,$thereforex<0,y>0$.$AB=-x,OB=y$.$thereforeS_{triangleAOB}=frac{1}{2}ABtimesOB$$=frac{1}{2}(-x)timesy=-frac{1}{2}xy=1$.$thereforexy=k=-2$.故答案为:-2.【点睛】本题考查了反比例函数系数$k$的几何意义,易错点:分清是矩形的面积还是三角形的面积;看双曲线所在象限确定$k$的符号.", "solution": "null", "level": "八年级", "question": "如图,已知点$A$在反比例函数$y=frac{{k}}{{x}}$($k$是常数,$x<0$)的图象上,过点$A$作$ABperpy$轴于$B$,若$triangleAOB$的面积为1,则$k$的值是$qquad$", "options": [], "subject": "解析几何", "analysis": "根据$triangleAOB$的面积确定$k$的绝对值为2,再由点$A$所在象限确定$k$为-2.【解析】解:设点$A$的坐标为$(x,y)$,且$xy=k$.$because$点$A$在第二象限,$thereforex<0,y>0$.$AB=-x,OB=y$.$thereforeS_{triangleAOB}=frac{1}{2}ABtimesOB$$=frac{1}{2}(-x)timesy=-frac{1}{2}xy=1$.$thereforexy=k=-2$.故答案为:-2.【点睛】本题考查了反比例函数系数$k$的几何意义,易错点:分清是矩形的面积还是三角形的面积;看双曲线所在象限确定$k$的符号."} {"id": "5238", "image": [], "answer": "由点$A$与点$A_{1}$关于$y$轴的对称,可得到$A_{1}(4,m)$,代入$y=frac{1}{2}x$即可求得$m$的值,从而求得$A(-4,2)$,进而即可求出$k$的值.【解析】解:$because$点$A$与点$A_{1}$关于$y$轴的对称,点$A(-4,m)$,$thereforeA_{1}(4,m)$,$because$点$A_{1}$恰好在直线$y=frac{1}{2}x$上,$thereforem=2$,$thereforeA(-4,2)$,$because$点$A(-4,m)$在反比例函数${y}=frac{{k}}{{x}}$的图象上,$thereforek=-4times2=-8$,故答案为:-8.【点睛】本题考查反比例函数图象上的点坐标的特征,一次函数图象上点的坐标特征,关于$y$轴对称的点的坐标的特征,掌握方程思想是解题的关键.", "solution": "null", "level": "八年级", "question": "已知点$A(-4,m)$在反比例函数$y=frac{k}{x}$的图象上,点$A$关于$y$轴的对称点$A_{1}$恰好在直线$y=frac{1}{2}x$上,那么$k$的值为-8.", "options": [], "subject": "解析几何", "analysis": "由点$A$与点$A_{1}$关于$y$轴的对称,可得到$A_{1}(4,m)$,代入$y=frac{1}{2}x$即可求得$m$的值,从而求得$A(-4,2)$,进而即可求出$k$的值.【解析】解:$because$点$A$与点$A_{1}$关于$y$轴的对称,点$A(-4,m)$,$thereforeA_{1}(4,m)$,$because$点$A_{1}$恰好在直线$y=frac{1}{2}x$上,$thereforem=2$,$thereforeA(-4,2)$,$because$点$A(-4,m)$在反比例函数${y}=frac{{k}}{{x}}$的图象上,$thereforek=-4times2=-8$,故答案为:-8.【点睛】本题考查反比例函数图象上的点坐标的特征,一次函数图象上点的坐标特征,关于$y$轴对称的点的坐标的特征,掌握方程思想是解题的关键."} {"id": "5239", "image": [], "answer": "根据反比例函数的增减性判断即可.【解析】解:对于函数${y}=frac{{k}}{{x}}$的图象上有两点$P_{1}left(-1,y_{1}right),P_{2}left(-2,y_{2}right)$,$because$当$-1>-2$时,$y_{1}0$,$therefore$符合题意的$k$值可以为大于0的任意数,故答案为:2(答案不唯一)。【点睛】本题考查反比例函数的图象与性质,熟知反比例函数的增减性是解答的关键.", "solution": "null", "level": "八年级", "question": "函数${y}=frac{{k}}{{x}}$的图象上有两点$P_{1}left(-1,y_{1}right),P_{2}left(-2,y_{2}right)$,若$y_{1}-2$时,$y_{1}0$,$therefore$符合题意的$k$值可以为大于0的任意数,故答案为:2(答案不唯一)。【点睛】本题考查反比例函数的图象与性质,熟知反比例函数的增减性是解答的关键."} {"id": "5240", "image": [], "answer": "把点$(m,n)$分别代入直线$y=-2x+4$与双曲线$y=frac{2}{{x}}$,整理后整体代入$frac{{n}+2{~m}}{{~m}n}$即可.【解析】解:$because$直线$y=-2x+4$与双曲线$y=frac{2}{{x}}$相交于点$(m,n)$,$thereforen=-2m+4,n=frac{2}{m}$,$thereforen+2m=4,mn=2$,则$frac{1}{{~m}}+frac{2}{{n}}=frac{{n}+2{~m}}{{mn}}=frac{4}{2}=2$.故答案为:2.【点睛】本题考查了反比例函数与一次函数的交点问题,知道交点坐标符合函数解析式是解题的关键,", "solution": "null", "level": "八年级", "question": "已知直线$y=-2x+4$与双曲线$y=frac{2}{x}$相交于点$(m,n)$,则$frac{1}{m}+frac{2}{n}$的值等于2.", "options": [], "subject": "解析几何", "analysis": "把点$(m,n)$分别代入直线$y=-2x+4$与双曲线$y=frac{2}{{x}}$,整理后整体代入$frac{{n}+2{~m}}{{~m}n}$即可.【解析】解:$because$直线$y=-2x+4$与双曲线$y=frac{2}{{x}}$相交于点$(m,n)$,$thereforen=-2m+4,n=frac{2}{m}$,$thereforen+2m=4,mn=2$,则$frac{1}{{~m}}+frac{2}{{n}}=frac{{n}+2{~m}}{{mn}}=frac{4}{2}=2$.故答案为:2.【点睛】本题考查了反比例函数与一次函数的交点问题,知道交点坐标符合函数解析式是解题的关键,"} {"id": "5241", "image": [], "answer": "根据中,$k>0$,则在每一个象限内,$y$随$x$的增大而减小.判断出AB.C三点横坐标的范围,从而确定$y_{1},y_{2},y_{3}$的大小关系.【解析】解:$becausek>0$,$therefore{y}=frac{{k}}{{x}}$分布在第一,三象限,且在每一个象限内,$y$随$x$的增大而减小.$becausea>b>0>c$$thereforec-a<0$,即$C$点在第三象限,则$y_{3}<0$;$becausea>b>0>c$,$thereforea-by_{2}>0$,故答案为:$y_{1}>y_{2}>y_{3}$.【点睛】本题主要考查了反比例函数的图象与性质,当$k>0$,在每一个象限内,$y$随$x$的增大而减小.", "solution": "null", "level": "八年级", "question": "已知$a>b>0>c$,点$Aleft(a-b,y_{1}right),Bleft(a-c,y_{2}right),Cleft(c-a,y_{3}right)$在反比例函数${y}=frac{{k}}{{x}}(k$为常数,$k>0$)的图象上,则$y_{1},y_{2},y_{3}$的大小关系是$y_{1}geqy_{2}geqy_{3}$.(用“>”连接)", "options": [], "subject": "解析几何", "analysis": "根据中,$k>0$,则在每一个象限内,$y$随$x$的增大而减小.判断出AB.C三点横坐标的范围,从而确定$y_{1},y_{2},y_{3}$的大小关系.【解析】解:$becausek>0$,$therefore{y}=frac{{k}}{{x}}$分布在第一,三象限,且在每一个象限内,$y$随$x$的增大而减小.$becausea>b>0>c$$thereforec-a<0$,即$C$点在第三象限,则$y_{3}<0$;$becausea>b>0>c$,$thereforea-by_{2}>0$,故答案为:$y_{1}>y_{2}>y_{3}$.【点睛】本题主要考查了反比例函数的图象与性质,当$k>0$,在每一个象限内,$y$随$x$的增大而减小."} {"id": "5242", "image": ["6804.jpg"], "answer": "先根据$Aleft(x_{1},y_{1}right),Bleft(x_{2},y_{2}right)$双曲线$y=-frac{2}{{x}}$上的点可知$x_{1}y_{1}=-2,x_{2}y_{2}=-2$,再根据反比例函数与正比例函数均关于原点对称可知$x_{1}=-x_{2},y_{1}=-y_{2}$,故可知$x_{1}y_{2}=-x_{1}y_{1},x_{2}y_{1}$$=-x_{2}y_{2}$,把此关系式代入所求代数式求解即可.【解析】解:$becauseAleft(x_{1},y_{1}right),Bleft(x_{2},y_{2}right)$双曲线$y=-frac{2}{{x}}$上的点,$thereforex_{1}y_{1}=-2,x_{2}y_{2}=-2$,$because$直线$y=kx(k<0)$与双曲线$y=-frac{2}{{x}}$交于$Aleft(x_{1},y_{1}right),Bleft(x_{2},y_{2}right)$两点,$thereforex_{1}=-x_{2},y_{1}=-y_{2}$,$thereforex_{1}y_{2}=-x_{1}y_{1},x_{2}y_{1}=-x_{2}y_{2}$,$therefore2x_{1}y_{2}-5x_{2}y_{1}=-2x_{1}y_{1}+5x_{2}y_{2}=(-2)times(-2)+5times(-2)=-6$.故答案为:-6.【点睛】本题考查的是反比例函数与一次函数的交点问题,根据题意得出$x_{1}y_{2}=-x_{1}y_{1},x_{2}y_{1}=-$$x_{2}y_{2}$是解答此题的关键.", "solution": "null", "level": "八年级", "question": "如图,直线$y=kx(k<0)$与双曲线${y}=frac{2}{{x}}$交于$Aleft(x_{1},y_{1}right)、Bleft(x_{2},y_{2}right)$两点,则$2x_{1}y_{2}-5y_{2}x_{1}$的值为$qquad$", "options": [], "subject": "解析几何", "analysis": "先根据$Aleft(x_{1},y_{1}right),Bleft(x_{2},y_{2}right)$双曲线$y=-frac{2}{{x}}$上的点可知$x_{1}y_{1}=-2,x_{2}y_{2}=-2$,再根据反比例函数与正比例函数均关于原点对称可知$x_{1}=-x_{2},y_{1}=-y_{2}$,故可知$x_{1}y_{2}=-x_{1}y_{1},x_{2}y_{1}$$=-x_{2}y_{2}$,把此关系式代入所求代数式求解即可.【解析】解:$becauseAleft(x_{1},y_{1}right),Bleft(x_{2},y_{2}right)$双曲线$y=-frac{2}{{x}}$上的点,$thereforex_{1}y_{1}=-2,x_{2}y_{2}=-2$,$because$直线$y=kx(k<0)$与双曲线$y=-frac{2}{{x}}$交于$Aleft(x_{1},y_{1}right),Bleft(x_{2},y_{2}right)$两点,$thereforex_{1}=-x_{2},y_{1}=-y_{2}$,$thereforex_{1}y_{2}=-x_{1}y_{1},x_{2}y_{1}=-x_{2}y_{2}$,$therefore2x_{1}y_{2}-5x_{2}y_{1}=-2x_{1}y_{1}+5x_{2}y_{2}=(-2)times(-2)+5times(-2)=-6$.故答案为:-6.【点睛】本题考查的是反比例函数与一次函数的交点问题,根据题意得出$x_{1}y_{2}=-x_{1}y_{1},x_{2}y_{1}=-$$x_{2}y_{2}$是解答此题的关键."} {"id": "5783", "image": ["7186.jpg", "7187.jpg"], "answer": "$12 \\sqrt{3}$", "solution": "null", "level": "八年级", "question": "如图在平行四边形 $\\mathrm{ABCD}$ 中, $\\angle \\mathrm{ABC}=60^{\\circ}, \\mathrm{AB}=4$, 四条内角平分线围成四边形 $\\mathrm{EFGH}$ 面积为 $\\sqrt{3}$, 则平行四边形 $\\mathrm{ABCD}$ 面积为\n\n", "options": [], "subject": "组合几何学", "analysis": "过点 $A$ 作 $A M \\perp B C$ 交 $B C$ 于 $M$, 延长 $A F$ 交 $B C$ 于 $N$, 连接 $E F$\n\n\n\n$\\because A B C D$ 为平行四边形,$A N$ 平分 $\\angle B A D$\n\n$\\therefore \\angle \\mathrm{BNA}=\\angle \\mathrm{DAN}, \\angle \\mathrm{BAN}=\\angle \\mathrm{DAN}$\n\n$\\therefore \\angle \\mathrm{BNA}=\\angle \\mathrm{BAN}$\n\n$\\because \\angle \\mathrm{ABC}=60^{\\circ}$\n\n$\\therefore \\triangle \\mathrm{ABN}$ 为等边三角形\n\n$\\therefore \\mathrm{AN}=\\mathrm{NB}=\\mathrm{AB}=4$\n\n$\\because \\mathrm{AM} \\perp \\mathrm{BC}$\n\n$\\therefore \\mathrm{AM}=\\sqrt{4^{2}-2^{2}}=2 \\sqrt{3}$\n\n$\\because B E$ 平分 $\\angle A B C, C G$ 平分 $\\angle B C D$\n\n$\\therefore \\angle \\mathrm{EBC}=30^{\\circ}, \\angle \\mathrm{NCG}=60^{\\circ}$\n\n$\\because \\angle \\mathrm{BNA}=60^{\\circ}$\n\n$\\therefore \\angle \\mathrm{BEN}=90^{\\circ}, \\mathrm{EN} / / \\mathrm{HC}$\n\n同理可得 $\\mathrm{BH} / / \\mathrm{DF}$\n\n$\\therefore$ 四边形 $\\mathrm{EFGH}$ 为矩形\n\n$\\because$ 四边形 EFGH 面积为 $\\sqrt{3}$\n\n$\\therefore \\mathrm{EF}=1, \\quad \\mathrm{FG}=\\sqrt{3}$\n\n$\\therefore \\mathrm{EG}=2$\n\n$\\because \\mathrm{EN} / / \\mathrm{GC}, \\mathrm{EN}=\\mathrm{GC}$\n\n$\\therefore$ 四边形 ENCG 为平行四边形\n\n$\\therefore \\mathrm{NC}=\\mathrm{EG}=2$\n\n$\\therefore B C=4+2=6$\n\n$\\therefore$ 平行四边形 $A B C D$ 面积 $=B C \\times A M=6 \\times 2 \\sqrt{3}=12 \\sqrt{3}$\n\n故答案为: $12 \\sqrt{3}$"} {"id": "5811", "image": ["7209.jpg", "7210.jpg"], "answer": "$\\sqrt{3}$", "solution": "null", "level": "八年级", "question": "如图, 在平行四边形 $A B C D$ 中, $A C$ 与 $B D$ 相交于点 $O, \\angle A O B=60^{\\circ}, B D=4$, 将 $\\triangle A B C$ 沿直线 $\\mathrm{AC}$ 翻折后, 点 $\\mathrm{B}$ 落在点 $\\mathrm{E}$ 处, 那么 $\\mathrm{S}_{\\triangle \\mathrm{AED}}=$ \\$ \\qquad \\$\n\n", "options": [], "subject": "组合几何学", "analysis": "如图, $\\triangle \\mathrm{AEC}$ 是 $\\triangle \\mathrm{ABC}$ 沿 $\\mathrm{AC}$ 翻折后的图形, 连接 $\\mathrm{OE} 、 \\mathrm{DE}$,\n\n$\\because$ 四边形 $\\mathrm{ABCD}$ 是平行四边形,\n\n$\\therefore \\mathrm{OB}=\\mathrm{OD}={ }^{\\frac{1}{2}} \\mathrm{BD}=2$,\n\n$\\because \\triangle \\mathrm{AEC}$ 是 $\\triangle \\mathrm{ABC}$ 沿 $\\mathrm{AC}$ 翻折后的图形, $\\angle \\mathrm{AOB}=60^{\\circ}$,\n\n$\\therefore \\angle \\mathrm{AOE}=60^{\\circ}, \\mathrm{OE}=\\mathrm{OB}$,\n\n$\\therefore \\angle \\mathrm{EOD}=60^{\\circ}, \\mathrm{OE}=\\mathrm{OD}$,\n$\\therefore \\triangle \\mathrm{OED}$ 是等边三角形,\n\n$\\therefore \\angle \\mathrm{DEO}=\\angle \\mathrm{AOE}=60^{\\circ}, \\mathrm{ED}=\\mathrm{OD}=2$,\n\n$\\therefore \\mathrm{ED} / / \\mathrm{AC}$\n\n$\\therefore \\mathrm{S}_{\\triangle \\mathrm{AED}}=\\mathrm{S}_{\\triangle \\mathrm{OED}}$,\n\n作 $\\mathrm{OF} \\perp \\mathrm{ED}$ 于 $\\mathrm{F}, \\mathrm{DF}={ }^{\\frac{1}{2}} \\mathrm{ED}=1$,\n\n$\\therefore \\mathrm{OF}=\\sqrt{\\mathrm{OD}^{2}-\\mathrm{DF}^{2}}=\\sqrt{3}$,\n\n$\\therefore \\mathrm{S}_{\\triangle \\mathrm{OED}}={ }^{\\frac{1}{2}} \\mathrm{ED} \\cdot \\mathrm{DF}=\\sqrt{3}$\n\n$\\therefore \\mathrm{S}_{\\triangle \\mathrm{AED}}=\\sqrt{3}$\n\n\n\n故答案为: $\\sqrt{3}$."} {"id": "5128", "image": ["6645.jpg"], "answer": "先证$triangleDOEcongtriangleBOF(ASA)$,得$DE=BF,OE=OF=2$,则$EF=4$,再求出$AB+AD=$12,然后由四边形$ABFE$的周长$=EF+AE+AB+BF=EF+AB+AD$,即可得出结论.【解析】解:$because$四边形$ABCD$是平行四边形,$thereforeAB=CD,AD=BC,AD//BC,OD=OB$,$thereforeangleEDO=angleFBO$,在$triangleDOE$和$triangleBOF$中,$left{begin{array}{l}angle{EDO}=angle{FBO}{OD}={OB}angle{DOE}=angle{BOF}end{array}right.$,$thereforetriangleDOEcongtriangleBOF(ASA)$,$thereforeDE=BF,OE=OF=2$,$thereforeEF=4$,$because$平行四边形$ABCD$的周长为24,$thereforeAB+AD=12$,$therefore$四边形$ABFE$的周长$=EF+AE+AB+BF=EF+AE+AB+DE=EF+AB+AD=4+12=16$,故答案为:16.【点睛】本题考查了平行四边形的性质、全等三角形的判定与性质等知识,熟练掌握平行四边形的性质,证明三角形全等是解题的关键.", "solution": "null", "level": "八年级", "question": "如图,点$O$是平行四边形$ABCD$对角线$BD$的中点,$EF$过点$O$分别与$AD、BC$相交于点$E、F$,若平行四边形$ABCD$的周长为$24,OE=2$,那么四边形$ABFE$的周长为16.", "options": [], "subject": "组合几何学", "analysis": "先证$triangleDOEcongtriangleBOF(ASA)$,得$DE=BF,OE=OF=2$,则$EF=4$,再求出$AB+AD=$12,然后由四边形$ABFE$的周长$=EF+AE+AB+BF=EF+AB+AD$,即可得出结论.【解析】解:$because$四边形$ABCD$是平行四边形,$thereforeAB=CD,AD=BC,AD//BC,OD=OB$,$thereforeangleEDO=angleFBO$,在$triangleDOE$和$triangleBOF$中,$left{begin{array}{l}angle{EDO}=angle{FBO}{OD}={OB}angle{DOE}=angle{BOF}end{array}right.$,$thereforetriangleDOEcongtriangleBOF(ASA)$,$thereforeDE=BF,OE=OF=2$,$thereforeEF=4$,$because$平行四边形$ABCD$的周长为24,$thereforeAB+AD=12$,$therefore$四边形$ABFE$的周长$=EF+AE+AB+BF=EF+AE+AB+DE=EF+AB+AD=4+12=16$,故答案为:16.【点睛】本题考查了平行四边形的性质、全等三角形的判定与性质等知识,熟练掌握平行四边形的性质,证明三角形全等是解题的关键."} {"id": "5131", "image": [], "answer": "根据中心对称定义以及轴对称图形的定义可得答案.【解析】解:把标有序号(1)或(6)的小正方形涂上阴影,可以与图中阴影部分组成的新图形是中心对称图形但不是轴对称图形.故答案为:(1)或(6.【点睛】本题主要考查轴对称图形和中心对称图形,掌握轴对称图形和中心对称图形定义是解题的关键.", "solution": "null", "level": "八年级", "question": "如图,把标有序号(1)、(2)、(3)、(4)、(5)、(6)中某个小正方形涂上阴影,使它与图中阴影部分组成的新图形是中心对称图形但不是轴对称图形,那么该小正方形的序号是写出所有符合条件的序号)|(1)||(2)||:---|:---|:---||(3)||||(4)|(5)|(6)|", "options": [], "subject": "组合几何学", "analysis": "根据中心对称定义以及轴对称图形的定义可得答案.【解析】解:把标有序号(1)或(6)的小正方形涂上阴影,可以与图中阴影部分组成的新图形是中心对称图形但不是轴对称图形.故答案为:(1)或(6.【点睛】本题主要考查轴对称图形和中心对称图形,掌握轴对称图形和中心对称图形定义是解题的关键."} {"id": "5149", "image": [], "answer": "根据多边形的外角和$360^{circ}div$外角的度数求出多边形的边数,然后根据多边形的对角线条数公式$frac{n(n-3)}{2}$即可解答.【解析】解:多边形的边数:$360^{circ}div60^{circ}=6$,对角线条数:$frac{6times(6-3)}{2}=9$.故答案为:9.【点睛】本题主要考查了多边形的外角和,多边形的对角线的条数等知识点,掌捯对角线总条数的计算公式是解答本题的关键.", "solution": "null", "level": "八年级", "question": "一个多边形的每个外角都是$60^{circ}$,则这个多边形的对角线共有9条.", "options": [], "subject": "组合几何学", "analysis": "根据多边形的外角和$360^{circ}div$外角的度数求出多边形的边数,然后根据多边形的对角线条数公式$frac{n(n-3)}{2}$即可解答.【解析】解:多边形的边数:$360^{circ}div60^{circ}=6$,对角线条数:$frac{6times(6-3)}{2}=9$.故答案为:9.【点睛】本题主要考查了多边形的外角和,多边形的对角线的条数等知识点,掌捯对角线总条数的计算公式是解答本题的关键."} {"id": "5153", "image": ["6681.jpg", "6682.jpg", "6682.jpg"], "answer": "连接$AC、AG、$作$AHperpBC$于$H$,根据三角形中位线定理得$AG=2EF$,再求出$AH$和$AC$的长即可.【解析】解:连接$AC、AG、$作$AHperpBC$于$H$,$because$点$E$为$AH$的中点,点$F$为$GH$的中点,$thereforeEF$为$triangleAGH$的中位线,$thereforeAG=2EF$,在平行四边形$ABCD$中,$becauseangleC=135^{circ}$,$thereforeangleB=45^{circ}$,$thereforeAH=BH=1$,$thereforeCH=2$,由勾股定理得$AC=sqrt{5}$,$thereforeAG$的最大值为$sqrt{5}$,最小值为1,$thereforeEF$的最大值与最小值的差为$frac{sqrt{5}-1}{2}$,故答案为:$frac{sqrt{5}-1}{2}$.【点睛】本题主要考查了平行四边形的性质,勾股定理,三角形中位线定理等知识,作辅助线构造三角形中位线是解题的关键.", "solution": "null", "level": "八年级", "question": "如图,在平行四边形$ABCD$中,$angleC=135^{circ},AD=3,AB=sqrt{2}$,点$H、G$分别是边$CD、BC$上的动点,连接$AH、GH$,点$E$为$AH$的中点,点$F$为$GH$的中点,连接$EF$,则$EF$的最大值与最小值的差为$-frac{sqrt{5}-1}{2}-$.", "options": [], "subject": "组合几何学", "analysis": "连接$AC、AG、$作$AHperpBC$于$H$,根据三角形中位线定理得$AG=2EF$,再求出$AH$和$AC$的长即可.【解析】解:连接$AC、AG、$作$AHperpBC$于$H$,$because$点$E$为$AH$的中点,点$F$为$GH$的中点,$thereforeEF$为$triangleAGH$的中位线,$thereforeAG=2EF$,在平行四边形$ABCD$中,$becauseangleC=135^{circ}$,$thereforeangleB=45^{circ}$,$thereforeAH=BH=1$,$thereforeCH=2$,由勾股定理得$AC=sqrt{5}$,$thereforeAG$的最大值为$sqrt{5}$,最小值为1,$thereforeEF$的最大值与最小值的差为$frac{sqrt{5}-1}{2}$,故答案为:$frac{sqrt{5}-1}{2}$.【点睛】本题主要考查了平行四边形的性质,勾股定理,三角形中位线定理等知识,作辅助线构造三角形中位线是解题的关键."} {"id": "5177", "image": ["6726.jpg", "6726.jpg"], "answer": "根据题意画出图形,然后根据垂直平分线的性质以及菱形的性质:对角线互相垂直平分,对角线平分对角进行分情况讨论即可.【解析】解:$because$在菱形$ABCD$中,$angleABC=100^{circ}$,对角线$AC$和$BD$相交于点$O$,$thereforeAC,BD$互相垂直平分,$becauseangleABC=angleADC=100^{circ}$,$thereforeangle{ABO}=angle{CBO}=angle{ADO}=angle{CDO}=frac{1}{2}times100^{circ}=50^{circ}$,当点$P$如下图$P$点所在位置时:$becausePB=PD$,$thereforeanglePBD=anglePDB=20^{circ}$,$thereforeanglePDC=50^{circ}-20^{circ}=30^{circ}$;当点$P$如下图$P^{prime}$点所在位置时:$becauseP^{prime}B=P^{prime}D$,$thereforeangleP^{prime}BD=angleP^{prime}DB=20^{circ}$$thereforeangleP^{prime}DC=angleP^{prime}DB^{prime}angleCDO=70^{circ}$综上:$anglePDC$的度数为$30^{circ}$或$70^{circ}$,故答案为:$30^{circ}$或$70^{circ}$.【点睛】本题考查了菱形的性质以及线段垂直平分线的性质,熟练掌握菱形的性质是解本题的关键,注意分类讨论.", "solution": "null", "level": "八年级", "question": "已知,在菱形$ABCD$中,$angleABC=100^{circ}$,对角线$AC$和$BD$相交于点$O$,在$AC$上取点$P$,连接$PB、PD$,若$anglePBD=20^{circ}$,则$anglePDC$的度数为$30^{circ}$或$70^{circ}$.", "options": [], "subject": "组合几何学", "analysis": "根据题意画出图形,然后根据垂直平分线的性质以及菱形的性质:对角线互相垂直平分,对角线平分对角进行分情况讨论即可.【解析】解:$because$在菱形$ABCD$中,$angleABC=100^{circ}$,对角线$AC$和$BD$相交于点$O$,$thereforeAC,BD$互相垂直平分,$becauseangleABC=angleADC=100^{circ}$,$thereforeangle{ABO}=angle{CBO}=angle{ADO}=angle{CDO}=frac{1}{2}times100^{circ}=50^{circ}$,当点$P$如下图$P$点所在位置时:$becausePB=PD$,$thereforeanglePBD=anglePDB=20^{circ}$,$thereforeanglePDC=50^{circ}-20^{circ}=30^{circ}$;当点$P$如下图$P^{prime}$点所在位置时:$becauseP^{prime}B=P^{prime}D$,$thereforeangleP^{prime}BD=angleP^{prime}DB=20^{circ}$$thereforeangleP^{prime}DC=angleP^{prime}DB^{prime}angleCDO=70^{circ}$综上:$anglePDC$的度数为$30^{circ}$或$70^{circ}$,故答案为:$30^{circ}$或$70^{circ}$.【点睛】本题考查了菱形的性质以及线段垂直平分线的性质,熟练掌握菱形的性质是解本题的关键,注意分类讨论."} {"id": "5193", "image": ["6747.jpg", "6748.jpg", "6748.jpg"], "answer": "作$AEperpBC$于点$E,AFperpDC$于点$F$,根据两张等宽的长方形纸条交叉叠放在一起可得$AE=AF$,再根据等面积法证明$BC=DC$,进而证明四边形$ABCD$的形状一定是菱形.【解析】解:如图,$becauseAB//CD,AD//BC$$therefore$四边形$ABCD$是平行四边形,作$AEperpBC$于点$E,AFperpDC$于点$F$,$because$两张等宽的长方形纸条交叉叠放在一起,$thereforeAE=AF$,$thereforeS_{text{平行四边形}ABCD}=BCcdotAE=DCcdotAF$,$thereforeBC=DC$,$thereforesquareABCD$是菱形.故答案为:是.【点睛】本题考查了菱形的判定与性质,利用等面积法解决本题是关键.", "solution": "null", "level": "八年级", "question": "如图,将两张对边平行且等宽的纸条交叉叠放在一起,则重合部分构成的四边形$ABCD$$qquad$菱形(填“是”或“不是”)。", "options": [], "subject": "组合几何学", "analysis": "作$AEperpBC$于点$E,AFperpDC$于点$F$,根据两张等宽的长方形纸条交叉叠放在一起可得$AE=AF$,再根据等面积法证明$BC=DC$,进而证明四边形$ABCD$的形状一定是菱形.【解析】解:如图,$becauseAB//CD,AD//BC$$therefore$四边形$ABCD$是平行四边形,作$AEperpBC$于点$E,AFperpDC$于点$F$,$because$两张等宽的长方形纸条交叉叠放在一起,$thereforeAE=AF$,$thereforeS_{text{平行四边形}ABCD}=BCcdotAE=DCcdotAF$,$thereforeBC=DC$,$thereforesquareABCD$是菱形.故答案为:是.【点睛】本题考查了菱形的判定与性质,利用等面积法解决本题是关键."} {"id": "4082", "image": ["5759.jpg"], "answer": "120", "solution": "null", "level": "八年级", "question": "如图所示, 在 $\\triangle A B C$ 中, $\\angle A=60^{\\circ}, B D 、 C E$ 分别是 $A C 、 A B$ 上的高, $H$ 是 $B D 、 C E$ 的交点,则 $\\angle \\mathrm{BHC}=$ \\$ \\qquad \\$度.\n\n", "options": [], "subject": "组合几何学", "analysis": ""} {"id": "4085", "image": ["5762.jpg"], "answer": "$30^{\\circ}$", "solution": "null", "level": "八年级", "question": "如图, $C D$ 是 Rt $\\triangle A B C$ 斜边 $A B$ 上的高, 将 $\\triangle B C D$ 沿 CD 折叠, $B$ 点恰好落在 $A B$ 的中点 $E$ 处,则 $\\angle A$ 等于 \\$ \\qquad \\$ .\n\n", "options": [], "subject": "组合几何学", "analysis": ""} {"id": "4108", "image": ["5791.jpg"], "answer": "$\\angle A B C=\\angle D B C$ 或 $A C=D C$, 写出一个即可", "solution": "null", "level": "八年级", "question": "如图, $A B=D B$, 要得到 $\\triangle A B C \\cong \\triangle \\triangle D B C$, 可以添加的一个条件是 \\$ \\qquad \\$ . (写出一个即可)\n\n", "options": [], "subject": "组合几何学", "analysis": ""} {"id": "5039", "image": ["6617.jpg"], "answer": "根据题意和图形,可以得到裁剪后的底面的长是$(11-2x){cm}$,宽为$(7-2x){cm}$,然后根据长方形的面积$=$长$times$宽,可以列出相应的方程.【解析】解:由题意可得:$(11-2x)(7-2x)=21$,故答案为:$(11-2x)(7-2x)=21$.【点睛】本题考查由实际问题抽象出一元二次方程,解答本题的关键是写出裁剪后的底面的长和宽.", "solution": "null", "level": "八年级", "question": "如图,小明同学用一张长$11{~cm}$,宽$7{~cm}$的矩形纸板制作一个底面积为$21{~cm}^{2}$的无盖长方体纸盒,他将纸板的四个角各剪去一个同样大小的正方形,将四周向上折叠即可(损耗不计).设剪去的正方形边长为$xcm$,则可列出关于$x$的方程为$(11-2x)(7-2x)=21$.", "options": [], "subject": "画法几何学", "analysis": "根据题意和图形,可以得到裁剪后的底面的长是$(11-2x){cm}$,宽为$(7-2x){cm}$,然后根据长方形的面积$=$长$times$宽,可以列出相应的方程.【解析】解:由题意可得:$(11-2x)(7-2x)=21$,故答案为:$(11-2x)(7-2x)=21$.【点睛】本题考查由实际问题抽象出一元二次方程,解答本题的关键是写出裁剪后的底面的长和宽."} {"id": "5127", "image": ["6643.jpg", "6644.jpg", "6644.jpg"], "answer": "连接$BD$交$AC$于点$O$,由平行四边形的性质得到$OA=OC,OB=OD$,添加$AE=CF$,得出$OE=OF$,即可得出结论.【解析】解:增加条件:$AE=CF$,理由如下:如图,连接$BD$交$AC$于点$O$,$because$四边形$ABCD$为平行四边形,$thereforeOB=OD,OA=OC$,若$AE=CF$,则有$AO-AE=CO-CF$,即$OE=OF$,$therefore$四边形$BEDF$为平行四边形,故答案为:$AE=CF($答案不唯一$)$.【点睛】本题主要考查平行四边形的判定,掌握平行四边形的判定与性质是解题的关键.", "solution": "null", "level": "八年级", "question": "如图,在平行四边形$ABCD$中,$E,F$两点均在对角线$AC$上.要使四边形$BEDF$为平行四边形,在不添加辅助线的情况下,需要增加的一个条件是$AE=CF$(答案不唯一)(写出一个即可).", "options": [], "subject": "画法几何学", "analysis": "连接$BD$交$AC$于点$O$,由平行四边形的性质得到$OA=OC,OB=OD$,添加$AE=CF$,得出$OE=OF$,即可得出结论.【解析】解:增加条件:$AE=CF$,理由如下:如图,连接$BD$交$AC$于点$O$,$because$四边形$ABCD$为平行四边形,$thereforeOB=OD,OA=OC$,若$AE=CF$,则有$AO-AE=CO-CF$,即$OE=OF$,$therefore$四边形$BEDF$为平行四边形,故答案为:$AE=CF($答案不唯一$)$.【点睛】本题主要考查平行四边形的判定,掌握平行四边形的判定与性质是解题的关键."} {"id": "5176", "image": ["6724.jpg", "6725.jpg", "6725.jpg"], "answer": "由正方形的性质得出$angleDCB=90^{circ},angleDBC=angleDBA=45^{circ},AB=BC$,证明$triangleABEcongtriangle$$CBE$,得出$EA=EC$,由等腰直角三角形的性质求出$EF=1$,证明四边形$GEFC$是矩形,由矩形性质得出$CF=EG=2$,由勾股定理求出$EC=sqrt{5}$,即可得出$EA=sqrt{5}$.【解析】解:如图,连接$EC$,$because$四边形$ABCD$是正方形,$thereforeangleDCB=90^{circ},angleDBC=angleDBA=45^{circ},AB=BC$,在$triangleABE$和$triangleCBE$中,$left{begin{array}{l}{AB}={CB}angle{ABE}=angle{CBE}=45^{circ},{BE}={BE}end{array}right.$,$thereforetriangleABEcongtriangleCBE(SAS)$,$thereforeEA=EC$,$becauseEFperpBC,angleDBC=45^{circ}$,$thereforetriangleBEF$是等輻直角三角形,$thereforeEF=frac{BE}{sqrt{2}}$,$becauseBE=sqrt{2}$,$thereforeEF=1$,$becauseEGperpCD,EFperpBC,angleDCB=90^{circ}$,$thereforeangleEGC=angleGCF=angleCFE=90^{circ}$,$therefore$四边形$GEFC$是矩形,$thereforeCF=GE$,$thereforeGE=2$,$thereforeCF=2$,$thereforeEC=sqrt{EF^{2}+CF^{2}}=sqrt{1^{2}+2^{2}}=sqrt{5}$$thereforeEA=sqrt{5}$,故答案为:$sqrt{5}$.【点睛】本题考查了正方形的性质,全等三角形的判定与性质,勾股定理,掌握正方形的性质,全等三角形的判定与性质,矩形的判定与性质,勾股定理是解决问题的关键.", "solution": "null", "level": "八年级", "question": "如图,正方形$ABCD$中,点$E$在$BD$上,$EFperpBC$于$F,EGperpDC$于$G$,连接$AE$,若$EB=sqrt{2}$,$EG=2$,则$EA$的长是$_sqrt{5}ldots$.", "options": [], "subject": "画法几何学", "analysis": "由正方形的性质得出$angleDCB=90^{circ},angleDBC=angleDBA=45^{circ},AB=BC$,证明$triangleABEcongtriangle$$CBE$,得出$EA=EC$,由等腰直角三角形的性质求出$EF=1$,证明四边形$GEFC$是矩形,由矩形性质得出$CF=EG=2$,由勾股定理求出$EC=sqrt{5}$,即可得出$EA=sqrt{5}$.【解析】解:如图,连接$EC$,$because$四边形$ABCD$是正方形,$thereforeangleDCB=90^{circ},angleDBC=angleDBA=45^{circ},AB=BC$,在$triangleABE$和$triangleCBE$中,$left{begin{array}{l}{AB}={CB}angle{ABE}=angle{CBE}=45^{circ},{BE}={BE}end{array}right.$,$thereforetriangleABEcongtriangleCBE(SAS)$,$thereforeEA=EC$,$becauseEFperpBC,angleDBC=45^{circ}$,$thereforetriangleBEF$是等輻直角三角形,$thereforeEF=frac{BE}{sqrt{2}}$,$becauseBE=sqrt{2}$,$thereforeEF=1$,$becauseEGperpCD,EFperpBC,angleDCB=90^{circ}$,$thereforeangleEGC=angleGCF=angleCFE=90^{circ}$,$therefore$四边形$GEFC$是矩形,$thereforeCF=GE$,$thereforeGE=2$,$thereforeCF=2$,$thereforeEC=sqrt{EF^{2}+CF^{2}}=sqrt{1^{2}+2^{2}}=sqrt{5}$$thereforeEA=sqrt{5}$,故答案为:$sqrt{5}$.【点睛】本题考查了正方形的性质,全等三角形的判定与性质,勾股定理,掌握正方形的性质,全等三角形的判定与性质,矩形的判定与性质,勾股定理是解决问题的关键."} {"id": "5194", "image": ["6749.jpg", "6750.jpg", "6749.jpg", "6750.jpg"], "answer": "分$angleB$为钝角和锐角两种情况,在Rt$triangleABE$中求得$BE$,则可求得$EC$,在Rt$triangleAEC$中利用勾股定理可求得$AC$,再利用等积法可求得$BD$的长.【解析】解:当$angleB$为针角时,如图1,图1$becauseAB=5,AE=4$,且$AEperpBC$,$thereforeBE=3$,$thereforeCE=BC+BE=5+3=8$,在Rt$triangleACE$中,由勾股定理可得$AC=sqrt{{AE}^{2}+C{E}^{2}}=sqrt{4^{2}+8^{2}}=4sqrt{5}$,$becauseS_{text{数}ABCD}ABCBcdotAE=frac{1}{2}BDcdotAC$,$therefore5times4=frac{1}{2}times4sqrt{5}BD$,解得$BD=2sqrt{5}$;当$angleB$为锐角时,如图2,图2同理可求得$BE=3$,则$CE=5-3=2$,在Rt$triangleACE$中,可求得$AC=sqrt{4^{2}+2^{2}}=2sqrt{5}$,同理可求得$BD=4sqrt{5}$,综上可知$BD$的长为$2sqrt{5}$或$4sqrt{5}$,故答案为:$2sqrt{5}$或$4sqrt{5}$.【点睛】本题主要考查菱形的性质,求得对角线$AC$的长是解题的关键,注意等积法的应用.", "solution": "null", "level": "八年级", "question": "菱形$ABCD$中,$AB=5,AE$是$BC$边上的高,$AE=4$,则对角线$BD$的长为$2sqrt{5}$或$4sqrt{5}ldots$.", "options": [], "subject": "立体几何学", "analysis": "分$angleB$为钝角和锐角两种情况,在Rt$triangleABE$中求得$BE$,则可求得$EC$,在Rt$triangleAEC$中利用勾股定理可求得$AC$,再利用等积法可求得$BD$的长.【解析】解:当$angleB$为针角时,如图1,图1$becauseAB=5,AE=4$,且$AEperpBC$,$thereforeBE=3$,$thereforeCE=BC+BE=5+3=8$,在Rt$triangleACE$中,由勾股定理可得$AC=sqrt{{AE}^{2}+C{E}^{2}}=sqrt{4^{2}+8^{2}}=4sqrt{5}$,$becauseS_{text{数}ABCD}ABCBcdotAE=frac{1}{2}BDcdotAC$,$therefore5times4=frac{1}{2}times4sqrt{5}BD$,解得$BD=2sqrt{5}$;当$angleB$为锐角时,如图2,图2同理可求得$BE=3$,则$CE=5-3=2$,在Rt$triangleACE$中,可求得$AC=sqrt{4^{2}+2^{2}}=2sqrt{5}$,同理可求得$BD=4sqrt{5}$,综上可知$BD$的长为$2sqrt{5}$或$4sqrt{5}$,故答案为:$2sqrt{5}$或$4sqrt{5}$.【点睛】本题主要考查菱形的性质,求得对角线$AC$的长是解题的关键,注意等积法的应用."} {"id": "4231", "image": ["5921.jpg"], "answer": "169", "solution": "null", "level": "八年级", "question": "如图, $B C=3, A B=4, A F=12$. 则正方形 $C D E F$ 的面积为\n\n", "options": [], "subject": "立体几何学", "analysis": ""} {"id": "5265", "image": ["6834.jpg", "6835.jpg"], "answer": "$75^{\\circ}$", "solution": "null", "level": "八年级", "question": "如图, 菱形纸片 $A B C D$ 中, $\\angle A=60^{\\circ}$, 点 $P$ 是 $A B$ 边的中点, 折叠纸片, 使点 $C$ 落在直线 $D P$ 上的 $C$ 处, 折痕为经过点 $D$ 的线段 $D E$.则 $\\angle D E C$ 的度数为 \\$ \\qquad \\$ .\n\n", "options": [], "subject": "变换几何", "analysis": "连接 $B D$, 如图所示:\n\n$\\because$ 四边形 $A B C D$ 为菱形,\n\n$\\therefore A B=A D, \\angle C=\\angle A=60^{\\circ}$,\n\n$\\therefore \\triangle A B D$ 为等边三角形, $\\angle A D C=120^{\\circ}$,\n\n\n$\\because P$ 为 $A B$ 的中点,\n\n$\\therefore D P$ 为 $\\angle A D B$ 的平分线, 即 $\\angle A D P=\\angle B D P=30^{\\circ}$,\n\n$\\therefore \\angle P D C=90^{\\circ}$,\n\n$\\therefore$ 由折叠的性质得: $\\angle C D E=\\angle P D E=45^{\\circ}$,\n\n在 $\\triangle D E C$ 中, $\\angle D E C=180^{\\circ}-(\\angle C D E+\\angle C)=75^{\\circ}$.\n\n故答案为: $75^{\\circ}$."} {"id": "4176", "image": ["5864.jpg"], "answer": "$105^{\\circ}$", "solution": "null", "level": "八年级", "question": "如图, $\\triangle A B C$ 与 $\\triangle A^{\\prime} B^{\\prime} C^{\\prime}$ 关于直线对称, 则 $\\angle B$ 的度数为\n\n", "options": [], "subject": "变换几何", "analysis": ""} {"id": "4514", "image": [], "answer": "$(4,3)$", "solution": "null", "level": "八年级", "question": "如果将电影院里的 3 排 4 号记作 $(3,4)$ 。那么 4 排 3 号应记作", "options": [], "subject": "组合数学", "analysis": ""} {"id": "5575", "image": ["7042.jpg"], "answer": "8", "solution": "null", "level": "八年级", "question": "如图是一张月历表, 在此月历表上可以用一个矩形任意圈出 $2 \\times 2$ 个位置上相邻的数 (如 $2,3,9$,\n\n\n\n10). 如果圈出的 4 个数中最大数与最小数的积为 128 , 则这 4 个数中最小的数是 \\$ \\qquad \\$", "options": [], "subject": "组合数学", "analysis": "设这 4 个数中最小数是 $x$, 则最大数为: $x+8$, 根据题意可得:\n\n$x(x+8)=128$\n\n整理得: $x^{2}+8 x-128=0$,\n\n$(x-8)(x+16)=0$\n\n解得: $x_{1}=8, x_{2}=-16$,\n\n则这 4 个数中最小的数是 8 .\n\n故答案为: 8 ."} {"id": "4543", "image": [], "answer": "$4<\\mathrm{a} \\leqslant 5$", "solution": "null", "level": "八年级", "question": "已知不等式组 $\\left\\{\\begin{array}{l}x>1 \\\\ x", "options": [], "subject": "组合数学", "analysis": "答案: 4"} {"id": "14102", "image": ["2910.jpg"], "answer": "答案:轴对称图形 对称轴", "solution": "null", "level": "四年级", "question": "汽车在笔直的公路上行驶, 车身的运动是 ( ) 现象, 车轮的运动是 ( ) 现象。", "options": [], "subject": "组合数学", "analysis": ""} {"id": "14092", "image": [], "answer": "答案: $\\times$", "solution": "null", "level": "四年级", "question": "每个三角形都有 3 条对称轴。", "options": [], "subject": "图论", "analysis": "答案: $\\times$"} {"id": "14101", "image": ["2912.jpg", "2913.jpg", "2914.jpg", "2915.jpg"], "answer": "答案: $\\bigcirc \\triangle$\nO\n$\\triangle$", "solution": "null", "level": "四年级", "question": "下面哪些现象是平移? 是平移的在()里面画 “ $\\bigcirc$ ”, 不是的画 “ $\\triangle$ ”。\n\n\n\n$(\\quad)$\n\n\n\n$(\\quad)$\n\n\n\n$(\\quad)$\n\n\n\n)", "options": [], "subject": "变换几何", "analysis": "答案: $\\bigcirc \\triangle$\nO\n$\\triangle$"} {"id": "14103", "image": [], "answer": "20.26; 二十点二六", "solution": "null", "level": "四年级", "question": "一个数由 2 个十、 2 个十分之一和 6 个百分之一组成, 这个数写作( ),读作 ( ).", "options": [], "subject": "算术", "analysis": "20.26; 二十点二六"} {"id": "14104", "image": [], "answer": "答案: \\times \\quad", "solution": "null", "level": "四年级", "question": "$125 \\times 8 \\div 125 \\times 8=1$ 。", "options": [], "subject": "算术", "analysis": "答案: \\times \\quad"} {"id": "14117", "image": [], "answer": "42; 19; 加法交换; 加法结合", "solution": "null", "level": "四年级", "question": "$58+19+42=(58+$ )+ ,运用了 ) 律。", "options": [], "subject": "算术", "analysis": ""} {"id": "14118", "image": [], "answer": "$246.67 ; 246.7$", "solution": "null", "level": "四年级", "question": "2022 年珠海市常住人口约 2466700 人, 改写成用 “万” 作单位的数是 ( ) 万人,精确到十分位约是 $(\\quad)$ 万人。", "options": [], "subject": "算术", "analysis": ""} {"id": "14119", "image": [], "answer": "$8.524 ;; 8.515$", "solution": "null", "level": "四年级", "question": "一个三位小数四舍五入后是", "options": [], "subject": "算术", "analysis": "$8.524 ;; 8.515$"} {"id": "27257", "image": [], "answer": "解$42+300 \\div(150-100)$\n\n【分析】根据题意, 原式先算除法, 再算加减法, 要想先减法, 应把减法用小括号括起来, 再进一步解答。\n\n【详解】如果要把算式 $42+300 \\div 150-100$ 的运算顺序改成先算减法, 那么算式中的减法 $150-100$ 应用小括号括起来, 所以应改为: $42+300 \\div(150-100)$ 。\n\n【点睛】本题主要是考查了括号的作用:改变运算的顺序。", "solution": "null", "level": "四年级", "question": "如果要把算式 $42+300 \\div 150-100$ 的运算顺序改成先算减法, 那么算式应改为( )。", "options": [], "subject": "算术", "analysis": "解$42+300 \\div(150-100)$\n\n【分析】根据题意, 原式先算除法, 再算加减法, 要想先减法, 应把减法用小括号括起来, 再进一步解答。\n\n【详解】如果要把算式 $42+300 \\div 150-100$ 的运算顺序改成先算减法, 那么算式中的减法 $150-100$ 应用小括号括起来, 所以应改为: $42+300 \\div(150-100)$ 。\n\n【点睛】本题主要是考查了括号的作用:改变运算的顺序。"} {"id": "27258", "image": [], "answer": "解$0 \\quad 0 \\quad 0$\n\n【分析】 0 乘任何数都得 0,0 除以任何不为 0 的数, 都得 0 ; 根据 0 的运算规律, 结合商 $\\times$ 除数 $=$ 被除数,即可解答。\n【详解】一个数除以 8 等于 0 , 根据除法各部分间关系可知, $8 \\times 0=0$, 所以被除数是 0 。\n\n( 0 ) $\\div 8=0$, 根据乘、除互逆关系可得 $8 \\times 0=(0)$ ,所以( $0 ) \\div 8=0$ 。\n\n【点睛】本题主要考查有关 0 的运算,以及乘除法间各部分的关系,属于基础知识,要熟练掌握。", "solution": "null", "level": "四年级", "question": "( ) $\\div 8=0$, 根据乘、除互逆关系可得 $8 \\times 0=(\\quad) \\div 8=0$ 。", "options": [], "subject": "算术", "analysis": "解$0 \\quad 0 \\quad 0$\n\n【分析】 0 乘任何数都得 0,0 除以任何不为 0 的数, 都得 0 ; 根据 0 的运算规律, 结合商 $\\times$ 除数 $=$ 被除数,即可解答。\n【详解】一个数除以 8 等于 0 , 根据除法各部分间关系可知, $8 \\times 0=0$, 所以被除数是 0 。\n\n( 0 ) $\\div 8=0$, 根据乘、除互逆关系可得 $8 \\times 0=(0)$ ,所以( $0 ) \\div 8=0$ 。\n\n【点睛】本题主要考查有关 0 的运算,以及乘除法间各部分的关系,属于基础知识,要熟练掌握。"} {"id": "27259", "image": [], "answer": "解加 减 除\n\n【分析】混合算式中有小括号应该先算小括号里的; 据此解答。\n\n【详解】根据分析, 应先算加法和减法, 最后算除法。\n\n【点睛】本题主要考查的是带小括号的混合算式的运算顺序。", "solution": "null", "level": "四年级", "question": "计算 $(230+48) \\div(200-61)$ 时,应先算 ( ) 法和( 法,最后算 $($ 法。", "options": [], "subject": "算术", "analysis": "解加 减 除\n\n【分析】混合算式中有小括号应该先算小括号里的; 据此解答。\n\n【详解】根据分析, 应先算加法和减法, 最后算除法。\n\n【点睛】本题主要考查的是带小括号的混合算式的运算顺序。"} {"id": "27277", "image": [], "answer": "解$19 \\quad 493$\n\n【分析】在有余数的除法中, 余数总比除数小, 即余数最大为: 除数 -1 ; 除数最小为: 余数 +1 , 进而根据“被除数=商×除数十余数”解答即可。\n【详解】除数最小是: $18+1=19$\n\n$25 \\times 19+18$\n\n$=475+18$\n\n$=493$\n\n除数最小是 19 , 当除数最小时, 被除数是 493\n\n【点睛】解答此题的关键是: 根据在有余数的除法中, 余数总比除数小, 得出除数最小为: 余数 +1 ,余数最大为: 除数 -1 , 然后再根据被除数、除数、商和余数四个量之间的关系进行解答即可。", "solution": "null", "level": "四年级", "question": "在除法算式 $\\square \\div \\square=25 \\ldots . .18$ 中,除数最小是(),当除数最小时,被除数是()。", "options": [], "subject": "算术", "analysis": "解$19 \\quad 493$\n\n【分析】在有余数的除法中, 余数总比除数小, 即余数最大为: 除数 -1 ; 除数最小为: 余数 +1 , 进而根据“被除数=商×除数十余数”解答即可。\n【详解】除数最小是: $18+1=19$\n\n$25 \\times 19+18$\n\n$=475+18$\n\n$=493$\n\n除数最小是 19 , 当除数最小时, 被除数是 493\n\n【点睛】解答此题的关键是: 根据在有余数的除法中, 余数总比除数小, 得出除数最小为: 余数 +1 ,余数最大为: 除数 -1 , 然后再根据被除数、除数、商和余数四个量之间的关系进行解答即可。"} {"id": "27278", "image": [], "answer": "解$3 \\times 4+2 \\times 6$\n\n【分析】在这几个数中加入“+”、“一”、“ $\\times$ \"、“符号, 以及括号, 数字位置可交换, 只要使计算结果等于 24 即可, $12+12=24$, 因此最后一步算加法, 只要前面两步的计算结果都等于 12 即可, $3 \\times 4$ $=12,2 \\times 6=12$, 根据混合运算的计算顺序填空即可。\n\n【详解】 $3 \\times 4+2 \\times 6$\n\n$=12+12$\n\n$=24$\n\n即用 2、3、4、6 这 4 个数, 经过怎样的运算才能得到 $24: 3 \\times 4+2 \\times 6=24$ 。(答案不唯一)\n\n【点睛】此题考查的是对运算符号的熟练运用, 熟练掌握混合运算的计算顺序是解答此题的关键。", "solution": "null", "level": "四年级", "question": "用 2、3、4、6 这 4 个数, 经过怎样的运算才能得到 24: ( ) $=24$ (每个数都用, 但不重复,用综合算式表示)。", "options": [], "subject": "算术", "analysis": "解$3 \\times 4+2 \\times 6$\n\n【分析】在这几个数中加入“+”、“一”、“ $\\times$ \"、“符号, 以及括号, 数字位置可交换, 只要使计算结果等于 24 即可, $12+12=24$, 因此最后一步算加法, 只要前面两步的计算结果都等于 12 即可, $3 \\times 4$ $=12,2 \\times 6=12$, 根据混合运算的计算顺序填空即可。\n\n【详解】 $3 \\times 4+2 \\times 6$\n\n$=12+12$\n\n$=24$\n\n即用 2、3、4、6 这 4 个数, 经过怎样的运算才能得到 $24: 3 \\times 4+2 \\times 6=24$ 。(答案不唯一)\n\n【点睛】此题考查的是对运算符号的熟练运用, 熟练掌握混合运算的计算顺序是解答此题的关键。"} {"id": "27279", "image": [], "answer": "解$180 \\div(20 \\times 8-115)=4$\n\n【分析】先求出 $20 \\times 8$ 的积, 然后减去 115 求出差, 最后再用 180 除以求出的差, 据此列综合算式即可。\n\n【详解】根据分析可得: 把 $20 \\times 8=160,160-115=45,180 \\div 45=4$, 这三个算式合并成一个综合算式为: $180 \\div(20 \\times 8-115)=4$ 。\n\n【点睛】本题考查的目的是让学生熟练掌握整数四则混合运算的顺序。", "solution": "null", "level": "四年级", "question": "$20 \\times 8=160,160-115=45,180 \\div 45=4$, 这三个算式合并成一个综合算式为 $(\\quad$ 。", "options": [], "subject": "算术", "analysis": "解$180 \\div(20 \\times 8-115)=4$\n\n【分析】先求出 $20 \\times 8$ 的积, 然后减去 115 求出差, 最后再用 180 除以求出的差, 据此列综合算式即可。\n\n【详解】根据分析可得: 把 $20 \\times 8=160,160-115=45,180 \\div 45=4$, 这三个算式合并成一个综合算式为: $180 \\div(20 \\times 8-115)=4$ 。\n\n【点睛】本题考查的目的是让学生熟练掌握整数四则混合运算的顺序。"} {"id": "13968", "image": [], "answer": "答案:400 \\quad", "solution": "null", "level": "四年级", "question": "右图的游泳池长 25 米, 游 8 个来回, 一共要游 $(\\quad)$ 米。", "options": [], "subject": "算术", "analysis": "答案:400 \\quad"} {"id": "13974", "image": ["3079.jpg"], "answer": "答案:减法", "solution": "null", "level": "四年级", "question": "648-289-11=648-(289+11)$, 这里运用了 $(\\quad)$ 的性质。", "options": [], "subject": "算术", "analysis": "答案:减法"} {"id": "13978", "image": [], "answer": "答案:70^{\\circ} \\quad 100^{\\circ}$", "solution": "null", "level": "四年级", "question": "一个等腰三角形的两条边的长度分别是 3 厘米和 7 厘米, 这个三角\n形的周长是 $(\\quad)$ 厘米。", "options": [], "subject": "算术", "analysis": "答案:70^{\\circ} \\quad 100^{\\circ}$"} {"id": "14005", "image": [], "answer": "答案: \n\n| 大船/条 | 9 | 8 | 7 | 6 | 5 | 4 | 3 | 2 |\n| :---: | :---: | :---: | :---: | :---: | :---: | :---: | :---: | :---: |\n| 小船/条 | 0 | 1 | 2 | 3 | 4 | 5 | 6 | 7 |\n| 乘坐人数 | 72 | 68 | 64 | 60 | 56 | 52 | 48 | 44 |\n\n$6 \\quad 3$", "solution": "null", "level": "四年级", "question": "博爱小学 60 名师生去划船, 恰好坐满了大船、小船共 9 条, 大船每条坐 8 人,小船每条坐 4 人。(先按顺序填表再写结果)\n\n| 大船/条 | 9 | 8 | 7 | 6 | | | | |\n| :--- | :---: | :---: | :---: | :---: | :---: | :---: | :---: | :---: |\n| 小船/条 | 0 | 1 | 2 | 3 | | | | |\n| 乘坐人数 | 72 | 68 | | | | | | |\n\n租了大船( )条,小船( )条。", "options": [], "subject": "算术", "analysis": "答案: \n\n| 大船/条 | 9 | 8 | 7 | 6 | 5 | 4 | 3 | 2 |\n| :---: | :---: | :---: | :---: | :---: | :---: | :---: | :---: | :---: |\n| 小船/条 | 0 | 1 | 2 | 3 | 4 | 5 | 6 | 7 |\n| 乘坐人数 | 72 | 68 | 64 | 60 | 56 | 52 | 48 | 44 |\n\n$6 \\quad 3$"} {"id": "14006", "image": [], "answer": "答案:12 \\quad 8$", "solution": "null", "level": "四年级", "question": "幸福餐厅有 2 人桌和 4 人桌共 20 张, 可以坐满 56 人用餐, 幸福餐厅 2 人桌有 $(\\quad)$ 张, 4 人桌有 $(\\quad)$ 张。", "options": [], "subject": "算术", "analysis": "答案:12 \\quad 8$"} {"id": "14013", "image": [], "answer": "答案:12", "solution": "null", "level": "四年级", "question": "明明家上半年用水 78 吨, 下半年用水 66 吨, 全年平均每月用水 $(\\quad)$ 吨。", "options": [], "subject": "算术", "analysis": "答案:12"} {"id": "14014", "image": [], "answer": "答案:33", "solution": "null", "level": "四年级", "question": "典典、华华、天天三人参加“我是阅读小达人”打卡集赞活动。他们三人分别集赞 30 个, 35 个, 34 个,他们平均每人集赞( )个。", "options": [], "subject": "算术", "analysis": "答案:33"} {"id": "14026", "image": [], "answer": "1489", "solution": "null", "level": "四年级", "question": "年 7 月 7 日 1 元人民币可以换", "options": [], "subject": "算术", "analysis": ""} {"id": "14029", "image": [], "answer": "答案:0 .68 \\quad", "solution": "null", "level": "四年级", "question": "至少减去( )才能得到一个整数, 至少加上( )才能得到一个整数。", "options": [], "subject": "算术", "analysis": "答案:0 .68 \\quad"} {"id": "14030", "image": [], "answer": "864 & 5.36 & 712 & 4.25 & 102 & 6.11\\end{array}$", "solution": "null", "level": "四年级", "question": "在 $\\quad \\quad)$ 里填上合适的数。\n\n$$\n\\begin{array}{lll}\n(\\quad) \\div 24=36 & (\\quad)-", "options": [], "subject": "算术", "analysis": ""} {"id": "14031", "image": [], "answer": "答案:3.10(答案不唯一)", "solution": "null", "level": "四年级", "question": "下面是一张跳远比赛成绩单。小亮获得了第 2 名, 他的成绩可能是\n\n( ) 米。\n\n| 姓名 | 小明 | 小军 | 小亮 | 小强 |\n| :---: | :---: | :---: | :---: | :---: |\n| 成绩/米 |", "options": [], "subject": "算术", "analysis": "答案:"} {"id": "14032", "image": [], "answer": "0", "solution": "null", "level": "四年级", "question": "丁丁在计算 $300-\\square \\times 3$ 时先算了减法, 结果是 600 , 正确的结果是 ( )。", "options": [], "subject": "算术", "analysis": ""} {"id": "14041", "image": [], "answer": "$(64-30) \times 20+13=693$", "solution": "null", "level": "四年级", "question": "根据 $64-30=34,34 \\times 20=680,680+13=693$ 写成一道综合算式是 (", "options": [], "subject": "算术", "analysis": ""} {"id": "14064", "image": [], "answer": "imes \\quad 100$", "solution": "null", "level": "四年级", "question": "在 里填上运算符号, 在 $\\square$ 里填上合适的数。 $300 \\div 5 \\div 2=300 \\div(5 \\bigcirc 2)$ $23 \\times 99=23 \\times \\square-23$", "options": [], "subject": "算术", "analysis": "imes \\quad 100$"} {"id": "14066", "image": [], "answer": "答案:50 \\times(9+15 \\div 15)$", "solution": "null", "level": "四年级", "question": "请添上括号改变 $50 \\times 9+15 \\div 15$ 的运算顺序, 使结果是 500 , 这个算式应改为 (", "options": [], "subject": "算术", "analysis": "答案:50 \\times(9+15 \\div 15)$"} {"id": "14074", "image": [], "answer": "答案:不同 左边的“6”表示 6 个", "solution": "null", "level": "四年级", "question": "中的两个“6”表示的含义( )(填“相同”或“不同”), 理由是 ( )。", "options": [], "subject": "算术", "analysis": "答案:不同 左边的“6”表示 6 个"} {"id": "14077", "image": [], "answer": "答案:270 & 310 & 15 & 345\\end{array}$", "solution": "null", "level": "四年级", "question": "在 $(\\quad)$ 里填上合适的数。\n\n$370+(\\quad)=640 \\quad(\\quad)-96=214$\n\n$72 \\times(\\quad)=1080 \\quad(\\quad) \\div 21=16 \\ldots \\ldots 9$", "options": [], "subject": "算术", "analysis": "答案:270 & 310 & 15 & 345\\end{array}$"} {"id": "14085", "image": [], "answer": "5.2、0.08、7.02、3.0", "solution": "null", "level": "四年级", "question": "港珠澳大桥是中国境内一座连接香港、珠海和澳门的桥隧工程, 其中主桥长 29600 米,将横线上的数改写成用“万”作单位的数是 $(\\quad)$万,精确到十分位约是 $(\\quad)$ 万。", "options": [], "subject": "算术", "analysis": ""} {"id": "14086", "image": [], "answer": "答案:4.05 $\\quad 56000 \\quad 7 \\quad 300 \\quad 0.8$", "solution": "null", "level": "四年级", "question": "给算式 $35 \\times 120+40 \\div 40$ 添上括号使它的运算顺序变为先算加法, 再算除法, 最后算乘法, 新算式为 ( )。", "options": [], "subject": "算术", "analysis": "答案:"} {"id": "27281", "image": [], "answer": "解【解答】解: 因为被减数 $=$ 减数 + 差, 根据被减数 + 减数 + 差 $=400$, 可得:\n\n被减数+被减数 $=4,0$\n\n所以被减数 $=400 \\div 2=200$\n\n故答案为: 200 。", "solution": "null", "level": "四年级", "question": "(2022 春・鄱阳县期末)在一道减法算式中, 被减数、减数与差的和是 400 , 被减数是 $\\qquad$", "options": [], "subject": "算术", "analysis": "解【解答】解: 因为被减数 $=$ 减数 + 差, 根据被减数 + 减数 + 差 $=400$, 可得:\n\n被减数+被减数 $=4,0$\n\n所以被减数 $=400 \\div 2=200$\n\n故答案为: 200 。"} {"id": "27282", "image": [], "answer": "解【解答】解: 要使算式 $35 \\times 78 \\div 654-615$ 变成先算减法, 再算除法, 最后算乘法, 添上括号后的算式是: $35 \\times[78 \\div(654-615)]$ 。\n\n$35 \\times[78 \\div(654-615)]$\n\n$=35 \\times[78 \\div 39]$\n\n$=35 \\times 2$\n\n$=70$\n\n即最后结果是 70 。\n\n故答案: $35 \\times[78 \\div(654-615)], 70$ 。", "solution": "null", "level": "四年级", "question": "(2022 春・潼南区期末)要使算式 $35 \\times 78 \\div 654-615$ 先算减法, 再算除法, 最后算乘法, 添上括号后的算式是 $\\qquad$ , 最后结果是 $\\qquad$。", "options": [], "subject": "算术", "analysis": "解【解答】解: 要使算式 $35 \\times 78 \\div 654-615$ 变成先算减法, 再算除法, 最后算乘法, 添上括号后的算式是: $35 \\times[78 \\div(654-615)]$ 。\n\n$35 \\times[78 \\div(654-615)]$\n\n$=35 \\times[78 \\div 39]$\n\n$=35 \\times 2$\n\n$=70$\n\n即最后结果是 70 。\n\n故答案: $35 \\times[78 \\div(654-615)], 70$ 。"} {"id": "27283", "image": [], "answer": "解【解答】解: $24 \\times 18=432$\n\n$432 \\div 9=48$\n\n$\\triangle \\times O=\\square, \\square \\div 18=24$, 其中 $\\square$ 是 $432, \\bigcirc$ 是 $9, \\triangle$ 是 48 。\n\n故答案为: 48 。", "solution": "null", "level": "四年级", "question": "(2022 春・盘州市期末) $\\triangle \\times O=\\square, \\square \\div 18=24$, 其中 $O$ 是最大的一位数, $\\triangle=$ $\\qquad$。", "options": [], "subject": "算术", "analysis": "解【解答】解: $24 \\times 18=432$\n\n$432 \\div 9=48$\n\n$\\triangle \\times O=\\square, \\square \\div 18=24$, 其中 $\\square$ 是 $432, \\bigcirc$ 是 $9, \\triangle$ 是 48 。\n\n故答案为: 48 。"} {"id": "27284", "image": [], "answer": "解【解答】解: $46 \\times 9=36$\n$36 \\div 6=9$\n\n答: 正确的商是 9 。\n\n故答案为: 9。", "solution": "null", "level": "四年级", "question": "(2022 春・固始县期中)小明在计算一道除法算式时, 把除数的 “ 6 ” 错误的看成了 “ 9 ”, 结果\n算得的商是 4 , 正确的商应该是 $\\qquad$。", "options": [], "subject": "算术", "analysis": "解【解答】解: $46 \\times 9=36$\n$36 \\div 6=9$\n\n答: 正确的商是 9 。\n\n故答案为: 9。"} {"id": "27285", "image": [], "answer": "【解答】解:\n\n$90 \\div(3 \\times 3)<90 \\div 3+3$\n\n$90 \\div(3 \\times 3)<90 \\div(3+3)$\n\n$360 \\div 2 \\div 3=360 \\div(2 \\times 3)$\n\n$80 \\times(7+1)>80 \\times 7+1$\n\n故答案为: $<;<;=;>$ 。", "solution": "null", "level": "四年级", "question": "(2022 春・大方县期中)不计算,在横线上填 “>”“<”“=”。\n$90 \\div(3 \\times 3)$ $\\qquad$ $90 \\div 3+3$\n$90 \\div(3 \\times 3)$ $\\qquad$ $90 \\div(3+3)$\n$360 \\div 2 \\div 3$ $\\qquad$ $360 \\div(2 \\times 3)$\n$80 \\times(7+1)$ $\\qquad$ $80 \\times 7+1$", "options": [], "subject": "算术", "analysis": "【解答】解:\n\n$90 \\div(3 \\times 3)<90 \\div 3+3$\n\n$90 \\div(3 \\times 3)<90 \\div(3+3)$\n\n$360 \\div 2 \\div 3=360 \\div(2 \\times 3)$\n\n$80 \\times(7+1)>80 \\times 7+1$\n\n故答案为: $<;<;=;>$ 。"} {"id": "27286", "image": [], "answer": "解【解答】解: 若 $A \\div B=C \\cdots \\cdots D$, 则 $A=B \\times C+D$ 。\n\n故答案为: $B \\times C+D$ 。", "solution": "null", "level": "四年级", "question": "(2022 春 ・ 沂南县期中)若 $A \\div B=C \\cdots \\cdots D$, 则 $A=$ $\\qquad$。", "options": [], "subject": "算术", "analysis": "解【解答】解: 若 $A \\div B=C \\cdots \\cdots D$, 则 $A=B \\times C+D$ 。\n\n故答案为: $B \\times C+D$ 。"} {"id": "27287", "image": [], "answer": "解【解答】解:另一个因数: $1632 \\div 24=68$\n\n正确的积: $68 \\times 42=2856$\n\n故答案为: 68,2856 。", "solution": "null", "level": "四年级", "question": "(2022 秋 - 东湖区期末)小马虎在计算乘法时, 将其中一个因数 42 看成了 24 , 结果得到的积是 1632。另一个因数是 $\\qquad$ , 正确的积是 $\\qquad$ .", "options": [], "subject": "算术", "analysis": "解【解答】解:另一个因数: $1632 \\div 24=68$\n\n正确的积: $68 \\times 42=2856$\n\n故答案为: 68,2856 。"} {"id": "27309", "image": [], "answer": "解【解答】解: $(50-14) \\div 6$\n\n$=36 \\div 6$\n\n$=6$ (元)\n\n答: 平均每支钢笔 6 元。\n\n故答案为: 6。", "solution": "null", "level": "四年级", "question": "(2022 秋・茂名期末)笑笑拿了 50 元去买文具, 买了 6 支钢笔, 售货员找回 14 元。平均每支钢笔 $\\qquad$元。", "options": [], "subject": "算术", "analysis": "解【解答】解: $(50-14) \\div 6$\n\n$=36 \\div 6$\n\n$=6$ (元)\n\n答: 平均每支钢笔 6 元。\n\n故答案为: 6。"} {"id": "27311", "image": [], "answer": "解$25 \\times(200-160) \\div 40 \\quad 25$\n\n【分析】根据混合运算的计算顺序进行计算并填空, 混合运算的计算顺序是: 先算乘、除法, 再算加、减法, 有括号时应先算括号里面的, 再算括号外面的。\n\n【详解】 $25 \\times(200-160) \\div 40$\n\n$=25 \\times 40 \\div 40$\n\n$=1000 \\div 40$\n\n$=25$\n\n即正确的算式是 $25 \\times(200-160) \\div 40$, 结果是 25 。\n\n【点睛】熟练掌握混合运算的计算顺序是解答此题的关键。", "solution": "null", "level": "四年级", "question": "如果要改变 $25 \\times 200-160 \\div 40$ 的运算顺序, 先算减法, 再算乘法, 最后算除法, 那么正确的算式是 ( ), 结果是 ( )。", "options": [], "subject": "算术", "analysis": "解$25 \\times(200-160) \\div 40 \\quad 25$\n\n【分析】根据混合运算的计算顺序进行计算并填空, 混合运算的计算顺序是: 先算乘、除法, 再算加、减法, 有括号时应先算括号里面的, 再算括号外面的。\n\n【详解】 $25 \\times(200-160) \\div 40$\n\n$=25 \\times 40 \\div 40$\n\n$=1000 \\div 40$\n\n$=25$\n\n即正确的算式是 $25 \\times(200-160) \\div 40$, 结果是 25 。\n\n【点睛】熟练掌握混合运算的计算顺序是解答此题的关键。"} {"id": "27312", "image": [], "answer": "解$96 \\div[(12+4) \\times 2]$\n\n【分析】先用 12 加上 4 求出和, 再用求出的和乘 2 求出积, 最后用 96 除以求出的积即可。\n\n【详解】 $96 \\div[(12+4) \\times 2]$\n\n$=96 \\div[16 \\times 2]$\n\n$=96 \\div 32$\n\n$=3$\n\n按照(1) $12+4=16$, (2) $16 \\times 2=32$, (3) $96 \\div 32=3$ 的运算顺序, 把这三个算式写成一个综合算式是 $96 \\div[$ (12 $+4) \\times 2]$ 。\n\n【点睛】解决这类题目, 要分清楚先算什么, 再算什么, 哪些数是运算出的结果, 这些数不要在算式中出现。", "solution": "null", "level": "四年级", "question": "按照(1) $12+4=16$, (2) $16 \\times 2=32$, (3) $96 \\div 32=3$ 的运算顺序, 把这三个算式写成一个综合算式是 ( )。", "options": [], "subject": "算术", "analysis": "解$96 \\div[(12+4) \\times 2]$\n\n【分析】先用 12 加上 4 求出和, 再用求出的和乘 2 求出积, 最后用 96 除以求出的积即可。\n\n【详解】 $96 \\div[(12+4) \\times 2]$\n\n$=96 \\div[16 \\times 2]$\n\n$=96 \\div 32$\n\n$=3$\n\n按照(1) $12+4=16$, (2) $16 \\times 2=32$, (3) $96 \\div 32=3$ 的运算顺序, 把这三个算式写成一个综合算式是 $96 \\div[$ (12 $+4) \\times 2]$ 。\n\n【点睛】解决这类题目, 要分清楚先算什么, 再算什么, 哪些数是运算出的结果, 这些数不要在算式中出现。"} {"id": "27314", "image": [], "answer": "解300\n\n【分析】首先根据工作量 $=$ 工作效率 $\\times$ 工作时间, 求出 5 天修了多少米, 再求出还剩多少米没有修,\n然后根据工作效率 $=$ 工作量 $\\div$ 工作时间, 列式解答即可。\n\n【详解】 $(2100-240 \\times 5) \\div 3$\n\n$=(2100-1200) \\div 3$\n\n$=900 \\div 3$\n\n$=300$ (米)\n\n所以余下的平均每天修 300 米。\n\n【点睛】此题考查的目的是理解掌握工作量、工作效率、工作时间三者之间的关系及应用。", "solution": "null", "level": "四年级", "question": "一个修路队修筑一条长 2100 米的公路, 前 5 天平均每天修 240 米, 余下的要求 3 天完成, 余下的平均每天修( )米。", "options": [], "subject": "算术", "analysis": "解300\n\n【分析】首先根据工作量 $=$ 工作效率 $\\times$ 工作时间, 求出 5 天修了多少米, 再求出还剩多少米没有修,\n然后根据工作效率 $=$ 工作量 $\\div$ 工作时间, 列式解答即可。\n\n【详解】 $(2100-240 \\times 5) \\div 3$\n\n$=(2100-1200) \\div 3$\n\n$=900 \\div 3$\n\n$=300$ (米)\n\n所以余下的平均每天修 300 米。\n\n【点睛】此题考查的目的是理解掌握工作量、工作效率、工作时间三者之间的关系及应用。"} {"id": "27315", "image": [], "answer": "解$24 \\quad 24 \\quad 864$\n\n【分析】除法各部分间的关系: 被除数 $\\div$ 除数 $=$ 商, 被除数 $=$ 商 $\\times$ 除数, 除数 $=$ 被除数 $\\div$ 商, 据此解答即可。\n\n【详解】根据分析可知,\n\n根据 $864 \\div 24=36$, 那么 $864 \\div 36=24,24 \\times 36=864$ 。\n\n【点睛】此题根据被除数、除数、商的关系进行解答。", "solution": "null", "level": "四年级", "question": "根据 $864 \\div 24=36$, 那么 $864 \\div 36=(\\quad)$ , $(\\quad) \\times 36=(\\quad$", "options": [], "subject": "算术", "analysis": "解$24 \\quad 24 \\quad 864$\n\n【分析】除法各部分间的关系: 被除数 $\\div$ 除数 $=$ 商, 被除数 $=$ 商 $\\times$ 除数, 除数 $=$ 被除数 $\\div$ 商, 据此解答即可。\n\n【详解】根据分析可知,\n\n根据 $864 \\div 24=36$, 那么 $864 \\div 36=24,24 \\times 36=864$ 。\n\n【点睛】此题根据被除数、除数、商的关系进行解答。"} {"id": "27316", "image": ["13306.jpg"], "answer": "解$4 \\times(9-3) \\times 1=24$\n\n【分析】本题可以把 24 先分解成两个数的和、差、商、积的形式, 然后再通过给的已知数字, 尝试调整凑成得数是 24 即可。\n\n【详解】 $4 \\times(9-3) \\times 1$\n\n$=4 \\times 6 \\times 1$\n\n$=24 \\times 1$\n\n$=24$\n\n用上面 4 张扑克牌算出 24 点, 算式为: $4 \\times(9-3) \\times 1=24$ 。(答案不唯一)\n\n【点睛】此题考查对运算符号的熟练运用,有一定的技巧性,关键是掌握整数的四则混合运算。", "solution": "null", "level": "四年级", "question": "用下面 4 张扑克牌算出 24 点, 并写出算式: ( )。\n\n", "options": [], "subject": "算术", "analysis": "解$4 \\times(9-3) \\times 1=24$\n\n【分析】本题可以把 24 先分解成两个数的和、差、商、积的形式, 然后再通过给的已知数字, 尝试调整凑成得数是 24 即可。\n\n【详解】 $4 \\times(9-3) \\times 1$\n\n$=4 \\times 6 \\times 1$\n\n$=24 \\times 1$\n\n$=24$\n\n用上面 4 张扑克牌算出 24 点, 算式为: $4 \\times(9-3) \\times 1=24$ 。(答案不唯一)\n\n【点睛】此题考查对运算符号的熟练运用,有一定的技巧性,关键是掌握整数的四则混合运算。"} {"id": "27317", "image": [], "answer": "解60\n\n【分析】因为先计算的减法, 后计算的乘法, 所以用 660 先除以 3 , 再进一步用 300 减去所除得商,就得到了ם,再代入原式子,按照正确的运算顺序,先计算乘法再计算减法, 求得正确的结果即可,据此解答。\n\n【详解】 $300-660 \\div 3$\n\n$=300-220$\n\n$=80$\n\n$300-80 \\times 3$\n\n$=300-240$\n$=60$\n\n丁丁在计算 $300-\\square \\times 3$ 时先算了减法, 结果得出 660 。那么这个算式的正确答案应该是(60)。\n\n【点睛】计算注意抓住运算顺序是正确得出结果的前提。", "solution": "null", "level": "四年级", "question": "丁丁在计算 $300-\\square \\times 3$ 时先算了减法, 结果得出 660 。那么这个算式的正确答案应该是 ( )。", "options": [], "subject": "算术", "analysis": "解60\n\n【分析】因为先计算的减法, 后计算的乘法, 所以用 660 先除以 3 , 再进一步用 300 减去所除得商,就得到了ם,再代入原式子,按照正确的运算顺序,先计算乘法再计算减法, 求得正确的结果即可,据此解答。\n\n【详解】 $300-660 \\div 3$\n\n$=300-220$\n\n$=80$\n\n$300-80 \\times 3$\n\n$=300-240$\n$=60$\n\n丁丁在计算 $300-\\square \\times 3$ 时先算了减法, 结果得出 660 。那么这个算式的正确答案应该是(60)。\n\n【点睛】计算注意抓住运算顺序是正确得出结果的前提。"} {"id": "27338", "image": [], "answer": "解减 除 加\n\n【分析】四则混合运算的计算顺序是:先算乘、除法,再算加、减法,有括号时应先算括号里面的,再算括号外面的; 依此计算并填空即可。\n\n【详解】 $75+360 \\div(20-5)$\n\n$=75+360 \\div 15$\n\n$=75+24$\n\n$=99$\n\n计算 $75+360 \\div(20-5)$ 时, 要先算括号里面的减法, 再算括号外面的除法, 最后计算括号外面的加法。\n\n【点睛】熟练掌握混合运算的计算顺序是解答此题的关键。", "solution": "null", "level": "四年级", "question": "计算 $75+360 \\div(20-5)$ 时, 先算 $($ 法, 再算 $($ 法, 最后算 $($ 法。", "options": [], "subject": "算术", "analysis": "解减 除 加\n\n【分析】四则混合运算的计算顺序是:先算乘、除法,再算加、减法,有括号时应先算括号里面的,再算括号外面的; 依此计算并填空即可。\n\n【详解】 $75+360 \\div(20-5)$\n\n$=75+360 \\div 15$\n\n$=75+24$\n\n$=99$\n\n计算 $75+360 \\div(20-5)$ 时, 要先算括号里面的减法, 再算括号外面的除法, 最后计算括号外面的加法。\n\n【点睛】熟练掌握混合运算的计算顺序是解答此题的关键。"} {"id": "27340", "image": [], "answer": "解$270 \\div(9 \\times 31-234) \\quad 6$\n\n【分析】混合运算的计算顺序是:先算乘、除法,再算加、减法,有括号时应先算括号里面的,再算括号外面的, 依此解答即可。\n\n【详解】 $270 \\div(9 \\times 31-234)$\n\n$=270 \\div(279-234)$\n\n$=270 \\div 45$\n\n$=6$\n\n即在数字顺序不变的前提条件下,如果把 $270 \\div 9 \\times 31-234$ 的运算顺序改成先算乘法, 再算减法, 最后算除法,那么算式应改成 $270 \\div(9 \\times 31-234 )$ ,结果是 6 。\n\n【点睛】熟练掌握混合运算的计算顺序是解答此题的关键。", "solution": "null", "level": "四年级", "question": "在数字顺序不变的前提条件下, 如果把 $270 \\div 9 \\times 31-234$ 的运算顺序改成先算乘法, 再算减法, 最后算除法,那么算式应改成( ),结果是( )。", "options": [], "subject": "算术", "analysis": "解$270 \\div(9 \\times 31-234) \\quad 6$\n\n【分析】混合运算的计算顺序是:先算乘、除法,再算加、减法,有括号时应先算括号里面的,再算括号外面的, 依此解答即可。\n\n【详解】 $270 \\div(9 \\times 31-234)$\n\n$=270 \\div(279-234)$\n\n$=270 \\div 45$\n\n$=6$\n\n即在数字顺序不变的前提条件下,如果把 $270 \\div 9 \\times 31-234$ 的运算顺序改成先算乘法, 再算减法, 最后算除法,那么算式应改成 $270 \\div(9 \\times 31-234 )$ ,结果是 6 。\n\n【点睛】熟练掌握混合运算的计算顺序是解答此题的关键。"} {"id": "27341", "image": [], "answer": "解$314 \\quad 21$\n\n【分析】加数 $=$ 和 - 另一个加数, 因数 $=$ 积 $\\div$ 另一个因数, 依此计算。\n\n【详解】 $530-216=314$, 即 $216+314=530$;\n\n$882 \\div 42=21$ ,即 $21 \\times 42=882$ 。\n\n【点睛】熟练掌握加减法、乘除法的意义和各部分之间的关系是解答此题的关键。\n\n12 ._详解\n\n【分析】由算式可知: 最后的结果等于 1 , 最后一个数为 3 , 根据算式 $3 \\div 3=1$, 即前面 3 个数使用加减乘除符号使结果变成 3 即可, 因为 $3 \\times 3 \\div 3=3$, 所以 $3 \\times 3 \\div 3 \\div 3=1$; 同理, 第二个算式的结果为 8 ,可知算式中有 $9-1=8$, 即前面两个 3 的结果变成 9 , 后面两个 3 的结果变成 1 , 由此解答即可。\n\n【详解】 $3 \\times 3 \\div 3 \\div 3=1 \\quad 3 \\times 3-3 \\div 3=8$ (答案不唯一)\n\n【点睛】此题考查对运算符号的熟练运用,根据数的特点,进行分析即可。", "solution": "null", "level": "四年级", "question": "在括号里填上合适的数。\n\n$216+(\\quad)=530 \\quad(\\quad) \\times 42=882$", "options": [], "subject": "算术", "analysis": "解$314 \\quad 21$\n\n【分析】加数 $=$ 和 - 另一个加数, 因数 $=$ 积 $\\div$ 另一个因数, 依此计算。\n\n【详解】 $530-216=314$, 即 $216+314=530$;\n\n$882 \\div 42=21$ ,即 $21 \\times 42=882$ 。\n\n【点睛】熟练掌握加减法、乘除法的意义和各部分之间的关系是解答此题的关键。\n\n12 ._详解\n\n【分析】由算式可知: 最后的结果等于 1 , 最后一个数为 3 , 根据算式 $3 \\div 3=1$, 即前面 3 个数使用加减乘除符号使结果变成 3 即可, 因为 $3 \\times 3 \\div 3=3$, 所以 $3 \\times 3 \\div 3 \\div 3=1$; 同理, 第二个算式的结果为 8 ,可知算式中有 $9-1=8$, 即前面两个 3 的结果变成 9 , 后面两个 3 的结果变成 1 , 由此解答即可。\n\n【详解】 $3 \\times 3 \\div 3 \\div 3=1 \\quad 3 \\times 3-3 \\div 3=8$ (答案不唯一)\n\n【点睛】此题考查对运算符号的熟练运用,根据数的特点,进行分析即可。"} {"id": "27342", "image": [], "answer": "【分析】由算式可知: 最后的结果等于 1 , 最后一个数为 3 , 根据算式 $3 \\div 3=1$, 即前面 3 个数使用加减乘除符号使结果变成 3 即可, 因为 $3 \\times 3 \\div 3=3$, 所以 $3 \\times 3 \\div 3 \\div 3=1$; 同理, 第二个算式的结果为 8 ,可知算式中有 $9-1=8$, 即前面两个 3 的结果变成 9 , 后面两个 3 的结果变成 1 , 由此解答即可。【详解】 $3 \\times 3 \\div 3 \\div 3=1 \\quad 3 \\times 3-3 \\div 3=8$ (答案不唯一)【点睛】此题考查对运算符号的熟练运用,根据数的特点,进行分析即可。", "solution": "null", "level": "四年级", "question": "在括号里填上适当的运算符号, 使等号两边相等。\n\n$3(\\quad) 3(\\quad) 3(\\quad 3=1 \\quad 3(\\quad) 3(\\quad) 3=8$", "options": [], "subject": "算术", "analysis": "【分析】由算式可知: 最后的结果等于 1 , 最后一个数为 3 , 根据算式 $3 \\div 3=1$, 即前面 3 个数使用加减乘除符号使结果变成 3 即可, 因为 $3 \\times 3 \\div 3=3$, 所以 $3 \\times 3 \\div 3 \\div 3=1$; 同理, 第二个算式的结果为 8 ,可知算式中有 $9-1=8$, 即前面两个 3 的结果变成 9 , 后面两个 3 的结果变成 1 , 由此解答即可。【详解】 $3 \\times 3 \\div 3 \\div 3=1 \\quad 3 \\times 3-3 \\div 3=8$ (答案不唯一)【点睛】此题考查对运算符号的熟练运用,根据数的特点,进行分析即可。"} {"id": "27345", "image": [], "answer": "解$9 \\quad 1$\n\n【分析】先根据“单价=总价数量”分别计算出租大车和小车时每人需要的钱, 要使租车最省钱, 则应尽量租最便宜的一种车型, 并且使每辆车都坐满, 没有空位; 因此用总人数除以最便宜的一种车型可坐的人数, 再根据计算出的结果进行解答即可。\n\n【详解】 $800 \\div 40=20$ (元 $/ 人$ )\n\n$500 \\div 20=25$ (元/人)\n\n20 元 $<25$ 元, 即租大车较便宜\n\n$15+365=380$ (人)\n\n$380 \\div 40=9$ (辆)\n\n剩下的 20 人刚好可以租 1 辆小车, 即租 9 辆大车和 1 辆小车最省钱。\n\n【点睛】此题考查的是经济问题的计算, 要使租车最省钱, 则应尽量多租最便宜的一种车型, 并且使每辆车都坐满。", "solution": "null", "level": "四年级", "question": "学校开展“红色足迹”革命教育,15 名教师带领 365 名学生参观革命纪念馆。大车可以坐 40 人,租金 800 元; 小车可以坐 20 人,租金 500 元。租( )辆大车和 ( 辆小车最省钱。", "options": [], "subject": "算术", "analysis": "解$9 \\quad 1$\n\n【分析】先根据“单价=总价数量”分别计算出租大车和小车时每人需要的钱, 要使租车最省钱, 则应尽量租最便宜的一种车型, 并且使每辆车都坐满, 没有空位; 因此用总人数除以最便宜的一种车型可坐的人数, 再根据计算出的结果进行解答即可。\n\n【详解】 $800 \\div 40=20$ (元 $/ 人$ )\n\n$500 \\div 20=25$ (元/人)\n\n20 元 $<25$ 元, 即租大车较便宜\n\n$15+365=380$ (人)\n\n$380 \\div 40=9$ (辆)\n\n剩下的 20 人刚好可以租 1 辆小车, 即租 9 辆大车和 1 辆小车最省钱。\n\n【点睛】此题考查的是经济问题的计算, 要使租车最省钱, 则应尽量多租最便宜的一种车型, 并且使每辆车都坐满。"} {"id": "27346", "image": [], "answer": "解300\n\n【分析】首先根据工作量 $=$ 工作效率 $\\times$ 工作时间, 求出 5 天修了多少米, 再求出还剩多少米没有修,然后根据工作效率=工作量:工作时间,列式解答即可。\n\n【详解】 $(2100-240 \\times 5) \\div 3$\n\n$=(2100-1200) \\div 3$\n\n$=900 \\div 3$\n\n$=300$ (米)\n\n所以余下的平均每天修 300 米。\n\n【点睛】此题考查的目的是理解掌握工作量、工作效率、工作时间三者之间的关系及应用。", "solution": "null", "level": "四年级", "question": "一个修路队修筑一条长 2100 米的公路, 前 5 天平均每天修 240 米, 余下的要求 3 天完成, 余下的平均每天修( )米。", "options": [], "subject": "算术", "analysis": "解300\n\n【分析】首先根据工作量 $=$ 工作效率 $\\times$ 工作时间, 求出 5 天修了多少米, 再求出还剩多少米没有修,\n然后根据工作效率 $=$ 工作量 $\\div$ 工作时间, 列式解答即可。\n\n【详解】 $(2100-240 \\times 5) \\div 3$\n\n$=(2100-1200) \\div 3$\n\n$=900 \\div 3$\n\n$=300$ (米)\n\n所以余下的平均每天修 300 米。\n\n【点睛】此题考查的目的是理解掌握工作量、工作效率、工作时间三者之间的关系及应用。"} {"id": "27366", "image": [], "answer": "解$(160+80) \\times 20=4800$\n\n【分析】根据题意可知, 此题是先算加法, 再算乘法, 根据混合运算的计算顺序列式即可。\n混合运算的计算顺序是:先算乘、除法,再算加、减法,有括号时应先算括号里面的,再算括号外面的, 依此解答。\n\n【详解】 $(160+80) \\times 20$\n\n$=240 \\times 20$\n\n$=4800$\n\n即 $160+80=240,240 \\times 20=4800$, 把它改写成综合算式是 $(160+80 ) \\times 20=4800$ 。\n\n【点睛】熟练掌握混合运算的计算顺序是解答此题的关键。", "solution": "null", "level": "四年级", "question": "$160+80=240,240 \\times 20=4800$, 把它改写成综合算式是 $(\\quad) 。$", "options": [], "subject": "算术", "analysis": "解$(160+80) \\times 20=4800$\n\n【分析】根据题意可知, 此题是先算加法, 再算乘法, 根据混合运算的计算顺序列式即可。\n混合运算的计算顺序是:先算乘、除法,再算加、减法,有括号时应先算括号里面的,再算括号外面的, 依此解答。\n\n【详解】 $(160+80) \\times 20$\n\n$=240 \\times 20$\n\n$=4800$\n\n即 $160+80=240,240 \\times 20=4800$, 把它改写成综合算式是 $(160+80 ) \\times 20=4800$ 。\n\n【点睛】熟练掌握混合运算的计算顺序是解答此题的关键。"} {"id": "27368", "image": [], "answer": "解(1)正确", "solution": "null", "level": "四年级", "question": "把算式 $(201 \\div 3)+( 12 \\times 14 )$ 的括号去掉后,运算顺序不变。()", "options": [], "subject": "算术", "analysis": "解(1)正确"} {"id": "27370", "image": [], "answer": "解(1)正确", "solution": "null", "level": "四年级", "question": "除数=被除数 $\\div$ 商()", "options": [], "subject": "算术", "analysis": "解(1)正确"} {"id": "27371", "image": [], "answer": "解(1)错误", "solution": "null", "level": "四年级", "question": "四则运算的运算顺序都是从左往右依次计算。", "options": [], "subject": "算术", "analysis": "解(1)错误"} {"id": "27392", "image": [], "answer": "(1)错误", "solution": "null", "level": "四年级", "question": "$15 \\times 6 \\div 15 \\times 6=90 \\div 90=1 \\quad(\\quad)$", "options": [], "subject": "算术", "analysis": "(1)错误"} {"id": "27394", "image": [], "answer": "解小括号 $23 \\times(320-180) \\div 60$\n\n【分析】如果要先算减法, 再算乘法, 最后算除法, 应运用小括号来改变运算顺序, 把 320-180用括号括起来,即 $23 \\times ( 320-160 ) \\div 60$, 然后再进一步解答。\n\n【详解】 $23 \\times 300-180 \\div 60$, 如果要改变运算顺序, 先算减法, 再算乘法, 最后算除法, 那么必须使用小括号括, 把(300-180)用小括号括起来,即: $23 \\times(320-180 ) \\div 60$ 。\n\n【点睛】四则混合运算的顺序:1、一个算式里, 如果只有加减法或者只有乘除法, 按照从左到右的顺序依次计算; 2、如果既有加减法、又有乘除法, 先算乘除法、再算加减法; 3、如果有括号, 先算括号里面的。", "solution": "null", "level": "四年级", "question": "$23 \\times 300-180 \\div 60$, 如果要改变运算顺序, 先算减法, 再算乘法, 最后算除法, 那么必须使用\n\n( ), 列式为 ( )。", "options": [], "subject": "算术", "analysis": "解小括号 $23 \\times(320-180) \\div 60$\n\n【分析】如果要先算减法, 再算乘法, 最后算除法, 应运用小括号来改变运算顺序, 把 320-180用括号括起来,即 $23 \\times ( 320-160 ) \\div 60$, 然后再进一步解答。\n\n【详解】 $23 \\times 300-180 \\div 60$, 如果要改变运算顺序, 先算减法, 再算乘法, 最后算除法, 那么必须使用小括号括, 把(300-180)用小括号括起来,即: $23 \\times(320-180 ) \\div 60$ 。\n\n【点睛】四则混合运算的顺序:1、一个算式里, 如果只有加减法或者只有乘除法, 按照从左到右的顺序依次计算; 2、如果既有加减法、又有乘除法, 先算乘除法、再算加减法; 3、如果有括号, 先算括号里面的。"} {"id": "27395", "image": [], "answer": "解403\n\n【分析】把减数 49 错写成了 94 , 多减了 $94-49=45$, 用得到的差加上 45 即可。\n\n【详解】 $94-49=45$\n$358+45=403$\n\n正确的差是 403。\n\n【点睛】解决本题还可以先用 94 加上 358 求出被减数, 再减去 49 求出差。", "solution": "null", "level": "四年级", "question": "小明在做一道减法题时, 把减数 49 错写成了 94 , 这时得到的差是 358 , 正确的差是 ( )。", "options": [], "subject": "算术", "analysis": "解403\n\n【分析】把减数 49 错写成了 94 , 多减了 $94-49=45$, 用得到的差加上 45 即可。\n\n【详解】 $94-49=45$\n$358+45=403$\n\n正确的差是 403。\n\n【点睛】解决本题还可以先用 94 加上 358 求出被减数, 再减去 49 求出差。"} {"id": "27396", "image": [], "answer": "解$3588 \\quad 23$\n\n【分析】乘法各部分间的关系:因数 $\\times$ 因数 $=$ 积,一个因数 $=$ 积 $\\div$ 另一个因数; 据此解题即可。\n\n【详解】由“ $156 \\times 23=3588$ ”可知,\n\n$23 \\times 156=3588$\n\n$3588 \\div 156=23$\n\n所以, 根据 $156 \\times 23=3588$, 直接写出 $23 \\times 156=3588,3588 \\div 156=23$ 。\n\n【点睛】此题主要考查的是乘法各部分间的关系,应熟练掌握并灵活运用。", "solution": "null", "level": "四年级", "question": "根据 $156 \\times 23=3588$, 直接写出 $23 \\times 156=($\n\n), $3588 \\div 156=(\\quad) 。$", "options": [], "subject": "算术", "analysis": "解$3588 \\quad 23$\n\n【分析】乘法各部分间的关系:因数 $\\times$ 因数 $=$ 积,一个因数 $=$ 积 $\\div$ 另一个因数; 据此解题即可。\n\n【详解】由“ $156 \\times 23=3588$ ”可知,\n\n$23 \\times 156=3588$\n\n$3588 \\div 156=23$\n\n所以, 根据 $156 \\times 23=3588$, 直接写出 $23 \\times 156=3588,3588 \\div 156=23$ 。\n\n【点睛】此题主要考查的是乘法各部分间的关系,应熟练掌握并灵活运用。"} {"id": "27397", "image": [], "answer": "解$(9-5) \\times(8-2)=24$\n\n【分析】将 9 和 5 相减得到 4,8 和 2 相减得到 6 , 再将两个差相乘, 即可得到 24。\n\n【详解】 $(9-5) \\times(8-2)$\n\n$=4 \\times 6$\n\n$=24$\n\n(答案不唯一)\n\n【点睛】本题考查“ 24 点”, 利用 $2 \\times 12=3 \\times 8=4 \\times 6=24$, 将已知数字通过加减乘除法得到 $2 、 12 、 3 、$\n\n8、4、6, 再进行计算。列算式时, 要根据运算顺序判断是否需要添上小括号。", "solution": "null", "level": "四年级", "question": "用 2、5、8、9 经过怎样的运算才能得到 24?可以这样列式计算: ( )。", "options": [], "subject": "算术", "analysis": "解$(9-5) \\times(8-2)=24$\n\n【分析】将 9 和 5 相减得到 4,8 和 2 相减得到 6 , 再将两个差相乘, 即可得到 24。\n\n【详解】 $(9-5) \\times(8-2)$\n\n$=4 \\times 6$\n\n$=24$\n\n(答案不唯一)\n\n【点睛】本题考查“ 24 点”, 利用 $2 \\times 12=3 \\times 8=4 \\times 6=24$, 将已知数字通过加减乘除法得到 $2 、 12 、 3 、$\n\n8、4、6, 再进行计算。列算式时, 要根据运算顺序判断是否需要添上小括号。"} {"id": "27419", "image": [], "answer": "解$270 \\div(9 \\times 31-234)$\n\n【分析】 $270 \\div 9 \\times 31-234$ 是先算除法, 再算乘法, 最后算减法。改成先算乘法, 再算减法, 最后算除法,是把乘法和减法提前了,把乘法和减法加上小括号即可。据此解答。\n\n【详解】 $270 \\div 9 \\times 31-234$ 的运算顺序改成先算乘法, 再算减法, 最后算除法, 那么算式应改为 $270 \\div$ $(9 \\times 31-234)$ 。\n\n【点睛】本题考查学生对四则运算的掌握。解决本题注意找清楚计算顺序的变化, 合理利用小括号进行求解。", "solution": "null", "level": "四年级", "question": "如果把 $270 \\div 9 \\times 31-234$ 的运算顺序改成先算乘法, 再算减法, 最后算除法, 那么算式应改成\n\n( )。", "options": [], "subject": "算术", "analysis": "解$270 \\div(9 \\times 31-234)$\n\n【分析】 $270 \\div 9 \\times 31-234$ 是先算除法, 再算乘法, 最后算减法。改成先算乘法, 再算减法, 最后算除法,是把乘法和减法提前了,把乘法和减法加上小括号即可。据此解答。\n\n【详解】 $270 \\div 9 \\times 31-234$ 的运算顺序改成先算乘法, 再算减法, 最后算除法, 那么算式应改为 $270 \\div$ $(9 \\times 31-234)$ 。\n\n【点睛】本题考查学生对四则运算的掌握。解决本题注意找清楚计算顺序的变化, 合理利用小括号进行求解。"} {"id": "27493", "image": [], "answer": "解$99 \\quad 60$\n\n【分析】(1)利用乘法分配律, 把右边算式改写成左边算式的形式; (2) 连续除以两个数等于除以两个数的积。\n\n【详解】 $263 \\times 100=263 \\times(99+1)=263 \\times 99+263$, 所以 $263 \\times(\\quad)+263=263 \\times 100$ 的括号里填\n\n99 ;\n\n$1200 \\div 15 \\div 4=1200 \\div(15 \\times 4)=1200 \\div 60$ 。\n\n【点睛】本题主要考查学生对运算定律的掌握和灵活运用。", "solution": "null", "level": "四年级", "question": "在括号里填合适的数。\n\n$263 \\times($\n\n)$+263=263 \\times 100$\n\n$1200 \\div 15 \\div 4=1200 \\div(\\quad)$", "options": [], "subject": "算术", "analysis": "解$99 \\quad 60$\n\n【分析】(1)利用乘法分配律, 把右边算式改写成左边算式的形式; (2) 连续除以两个数等于除以两个数的积。\n\n【详解】 $263 \\times 100=263 \\times(99+1)=263 \\times 99+263$, 所以 $263 \\times(\\quad)+263=263 \\times 100$ 的括号里填\n\n99 ;\n\n$1200 \\div 15 \\div 4=1200 \\div(15 \\times 4)=1200 \\div 60$ 。\n\n【点睛】本题主要考查学生对运算定律的掌握和灵活运用。"} {"id": "27494", "image": [], "answer": "解$125 \\times 8$ 乘法结合\n\n【分析】 $125 \\times 8=1000$, 因此可根据乘法结合律的特点进行解答, 乘法结合律的特点是三个数相乘,先把前两个数相乘, 或先把后两个数相乘, 积不变, 依此填空。\n\n【详解】 $(77 \\times 125) \\times 8$\n\n$=77 \\times(125 \\times 8)$\n\n$=77 \\times 1000$\n\n$=77000$\n\n由此可知, 计算 $(77 \\times 125) \\times 8$ 时, 为了计算简便, 可以先算 $125 \\times 8$, 这样计算是根据整数乘法结合律的特点。\n【点睛】熟练掌握乘法结合律的特点是解答此题的关键。", "solution": "null", "level": "四年级", "question": "计算 $(77 \\times 125) \\times 8$ 时, 为了计算简便, 可以先算 ( ), 这样计算是根据(\n\n)律的特点。", "options": [], "subject": "算术", "analysis": "解$125 \\times 8$ 乘法结合\n\n【分析】 $125 \\times 8=1000$, 因此可根据乘法结合律的特点进行解答, 乘法结合律的特点是三个数相乘,先把前两个数相乘, 或先把后两个数相乘, 积不变, 依此填空。\n\n【详解】 $(77 \\times 125) \\times 8$\n\n$=77 \\times(125 \\times 8)$\n\n$=77 \\times 1000$\n\n$=77000$\n\n由此可知, 计算 $(77 \\times 125) \\times 8$ 时, 为了计算简便, 可以先算 $125 \\times 8$, 这样计算是根据整数乘法结合律的特点。\n【点睛】熟练掌握乘法结合律的特点是解答此题的关键。"} {"id": "27496", "image": [], "answer": "解44\n\n【分析】首先根据乘法分配律, 可得 $12 \\times(\\mathrm{A}+4)=12 \\times \\mathrm{A}+12 \\times 4$, 然后再减去 $12 \\times \\mathrm{A}+4$, 求出他的计算结果与正确结果相差多少即可。\n\n【详解】 $12 \\times(\\mathrm{A}+4)-(12 \\times \\mathrm{A}+4)$\n\n$=12 \\mathrm{~A}+12 \\times 4-12 \\mathrm{~A}-4$\n\n$=12 \\mathrm{~A}-12 \\mathrm{~A}+48-4$\n\n$=44$\n\n小马虎把 $12 \\times(\\mathrm{A}+4)$ 错算成 $12 \\times \\mathrm{A}+4$, 计算结果与正确答案相差 44 。\n\n【点睛】此题考查的目的是理解掌握乘法分配律的意义, 并且能够灵活运用乘法分配律进行简便计算。", "solution": "null", "level": "四年级", "question": "小马虎把 $12 \\times(\\mathrm{A}+4)$ 错算成 $12 \\times \\mathrm{A}+4$, 计算结果与正确答案相差 $($ 。", "options": [], "subject": "算术", "analysis": "解44\n\n【分析】首先根据乘法分配律, 可得 $12 \\times(\\mathrm{A}+4)=12 \\times \\mathrm{A}+12 \\times 4$, 然后再减去 $12 \\times \\mathrm{A}+4$, 求出他的计算结果与正确结果相差多少即可。\n\n【详解】 $12 \\times(\\mathrm{A}+4)-(12 \\times \\mathrm{A}+4)$\n\n$=12 \\mathrm{~A}+12 \\times 4-12 \\mathrm{~A}-4$\n\n$=12 \\mathrm{~A}-12 \\mathrm{~A}+48-4$\n\n$=44$\n\n小马虎把 $12 \\times(\\mathrm{A}+4)$ 错算成 $12 \\times \\mathrm{A}+4$, 计算结果与正确答案相差 44 。\n\n【点睛】此题考查的目的是理解掌握乘法分配律的意义, 并且能够灵活运用乘法分配律进行简便计算。"} {"id": "27497", "image": [], "answer": "解$>=<$\n\n【分析】(1) 大数比较大小的方法: 位数相同的两个数比较大小, 从最高位比起, 最高位上的数大的那个数就大, 如果最高位上的数相同, 就比较下一个数位上的数, 一直到比较出大小为止; 位数不同的两个数比较大小, 位数多的数就大;\n\n(2) 先分别将 70000 万、 7 亿改写成整数, 然后再比较;\n\n(3) 计算 $999 \\times 99$ 时, 可将 99 写成 100-1, 然后根据乘法分配律的特点进行比较。\n\n【详解】 (1) 60000 与 59988, 万位上的数 $6>5$, 即 $60000>59988$;\n\n(2) 70000 万 $=700000000,7$ 亿 $=700000000$, 即 70000 万 $=7$ 亿;\n\n(3) $999 \\times 99=999 \\times(100-1)=999 \\times 100-999 \\times 1=999 \\times 100-999$, 即 $999 \\times 99<999 \\times 100-1$ 。\n\n【点睛】此题考查的是大数的比较, 整数的改写与大小比较, 以及乘法分配律的特点, 应熟练掌握。", "solution": "null", "level": "四年级", "question": "在括号里填上“ $>$ ”“<”或“=”。\n\n60000( ) 59988 ) 70000 万 亿 $999 \\times 99$ 亿 $999 \\times 100-1$", "options": [], "subject": "算术", "analysis": "解$>=<$\n\n【分析】(1) 大数比较大小的方法: 位数相同的两个数比较大小, 从最高位比起, 最高位上的数大的那个数就大, 如果最高位上的数相同, 就比较下一个数位上的数, 一直到比较出大小为止; 位数不同的两个数比较大小, 位数多的数就大;\n\n(2) 先分别将 70000 万、 7 亿改写成整数, 然后再比较;\n\n(3) 计算 $999 \\times 99$ 时, 可将 99 写成 100-1, 然后根据乘法分配律的特点进行比较。\n\n【详解】 (1) 60000 与 59988, 万位上的数 $6>5$, 即 $60000>59988$;\n\n(2) 70000 万 $=700000000,7$ 亿 $=700000000$, 即 70000 万 $=7$ 亿;\n\n(3) $999 \\times 99=999 \\times(100-1)=999 \\times 100-999 \\times 1=999 \\times 100-999$, 即 $999 \\times 99<999 \\times 100-1$ 。\n\n【点睛】此题考查的是大数的比较, 整数的改写与大小比较, 以及乘法分配律的特点, 应熟练掌握。"} {"id": "27517", "image": [], "answer": "解加法交换律 加法结合律\n\n【分析】改变加数 24 与加数 78 的位置, 需要用到加法交换律。通过添加小括号改变运算顺序, 需要用到加法结合律。\n\n【详解】 $122+24+78+76=(122+78)+(24+76)$ 运用了加法交换律和加法结合律。\n\n【点睛】解答本题的关键是熟练掌握整数的加法运算定律。", "solution": "null", "level": "四年级", "question": "$122+24+78+76=(122+78)+(24+76)$ 运用了 $(\\quad)$ 和 $($ 运算定律。", "options": [], "subject": "算术", "analysis": "解加法交换律 加法结合律\n\n【分析】改变加数 24 与加数 78 的位置, 需要用到加法交换律。通过添加小括号改变运算顺序, 需要用到加法结合律。\n\n【详解】 $122+24+78+76=(122+78)+(24+76)$ 运用了加法交换律和加法结合律。\n\n【点睛】解答本题的关键是熟练掌握整数的加法运算定律。"} {"id": "27519", "image": [], "answer": "解$(1000-352) \\div 9 \\times 6=432$\n\n【分析】分析算式的关系, 发现 $72 \\times 6=432$ 中的 72 是由算式 $648 \\div 9=72$ 得到的, 而算式 $648 \\div 9=72$这个算式中的 648 是由算式 $1000-352=648$ 得到的。所以这个综合算式应该是先算减法, 再算除法,最后算乘法。据此解答。\n\n【详解】减法和乘、除法在一起, 要先算减法, 需要添加小括号, 所以把 $72 \\times 6=432,648 \\div 9=72$, $1000-352=648$ 这三个算式合并成一道综合算式是: $(1000-352) \\div 9 \\times 6=432$ 。\n\n【点睛】本题考查学生对四则混合运算的掌握。解决此题的关键是弄清楚运算顺序。", "solution": "null", "level": "四年级", "question": "根据 $72 \\times 6=432,648 \\div 9=72,1000-352=648$, 把三个算式合并成一道综合算式是 $(\\quad) 。$", "options": [], "subject": "算术", "analysis": "解$(1000-352) \\div 9 \\times 6=432$\n\n【分析】分析算式的关系, 发现 $72 \\times 6=432$ 中的 72 是由算式 $648 \\div 9=72$ 得到的, 而算式 $648 \\div 9=72$这个算式中的 648 是由算式 $1000-352=648$ 得到的。所以这个综合算式应该是先算减法, 再算除法,最后算乘法。据此解答。\n\n【详解】减法和乘、除法在一起, 要先算减法, 需要添加小括号, 所以把 $72 \\times 6=432,648 \\div 9=72$, $1000-352=648$ 这三个算式合并成一道综合算式是: $(1000-352) \\div 9 \\times 6=432$ 。\n\n【点睛】本题考查学生对四则混合运算的掌握。解决此题的关键是弄清楚运算顺序。"} {"id": "27520", "image": [], "answer": "解$648 \\div 18=36 \\# \\# 648 \\div 36=18 \\quad 648 \\div 36=18 \\# \\# 648 \\div 18=36$\n\n【分析】在乘法中, 因数 $\\times$ 因数 $=$ 积, 那么“积 $\\div$ 一个因数 $=$ 另一个因数”, 据此解答。\n\n【详解】根据 $36 \\times 18=648$, 写出两个除法算式: $(648 \\div 18=36)$ 和 $(648 \\div 36=18) 。$\n\n【点睛】根据乘与除的互逆关系解答即可。", "solution": "null", "level": "四年级", "question": "根据 $36 \\times 18=648$, 写出两个除法算式: $(\\quad)$ 和 $(\\quad)$", "options": [], "subject": "算术", "analysis": "解$648 \\div 18=36 \\# \\# 648 \\div 36=18 \\quad 648 \\div 36=18 \\# \\# 648 \\div 18=36$\n\n【分析】在乘法中, 因数 $\\times$ 因数 $=$ 积, 那么“积 $\\div$ 一个因数 $=$ 另一个因数”, 据此解答。\n\n【详解】根据 $36 \\times 18=648$, 写出两个除法算式: $(648 \\div 18=36)$ 和 $(648 \\div 36=18) 。$\n\n【点睛】根据乘与除的互逆关系解答即可。"} {"id": "27521", "image": [], "answer": "解$84 \\div[(8+6) \\times 2] ; 3$\n\n【分析】 $84 \\div 8+6 \\times 2$ 是先算乘除法, 最后算加法, 要求先算加法, 再算乘法, 最后算除法, 就需要把 $8+6$ 加上小括号, 再把 $8+6 \\times 2$ 加上中括号即可。\n\n【详解】 $84 \\div 8+6 \\times 2$ 改为: 加法 $\\rightarrow$ 乘法 $\\rightarrow$ 除法\n\n$84 \\div[(8+6) \\times 2]$\n\n$=84 \\div[14 \\times 2]$\n\n$=84 \\div 28$\n\n$=3$\n\n即: $84 \\div[(8+6) \\times 2]=3$ 。\n\n【点睛】解决本题要注意找清楚计算顺序的变化, 合理利用小括号、中括号求解。", "solution": "null", "level": "四年级", "question": "根据指定的运算顺序, 在下面的算式中添上适当的括号, 并计算出得数。\n\n加法 $\\rightarrow$ 乘法 $\\rightarrow$ 除法: $84 \\div 8+6 \\times 2=(\\quad)$", "options": [], "subject": "算术", "analysis": "解$84 \\div[(8+6) \\times 2] ; 3$\n\n【分析】 $84 \\div 8+6 \\times 2$ 是先算乘除法, 最后算加法, 要求先算加法, 再算乘法, 最后算除法, 就需要把 $8+6$ 加上小括号, 再把 $8+6 \\times 2$ 加上中括号即可。\n\n【详解】 $84 \\div 8+6 \\times 2$ 改为: 加法 $\\rightarrow$ 乘法 $\\rightarrow$ 除法\n\n$84 \\div[(8+6) \\times 2]$\n\n$=84 \\div[14 \\times 2]$\n\n$=84 \\div 28$\n\n$=3$\n\n即: $84 \\div[(8+6) \\times 2]=3$ 。\n\n【点睛】解决本题要注意找清楚计算顺序的变化, 合理利用小括号、中括号求解。"} {"id": "27523", "image": ["13508.jpg", "13509.jpg"], "answer": "解(1) $2 \\times 5+6+8=24$ (答案不唯一)\n\n(2) $9 \\times 8 \\div(7-4)=24$ (答案不唯一)\n\n【分析】(1)要使结果为 24,4 个数加起来等于 21 , 还差 3 , 就想 $2 \\times 5=10$ 比 $2+5=7$ 多 3 再加 6 加 8 正好得 24 ;\n\n(2) 4 个数加起来的 28 , 多 4 , 不能只用加减法, 还需用乘、除法。\n\n【详解】(1) $2 \\times 5+6+8$\n\n$=10+6+8$\n\n$=16+8$\n\n$=24$\n\n(答案不唯一)\n\n(2) $9 \\times 8 \\div(7-4)$\n\n$=72 \\div 3$\n\n$=24$\n\n(答案不唯一)\n\n【点睛】解决本题的关键是熟练掌握整数四则运算的计算方法。", "solution": "null", "level": "四年级", "question": "算 24 点是一种益智游戏。下面每组中四张扑克牌上的点数, 经过运算可以得到 24 请你写出综合算式。\n\n\n\n综合算式: $\\qquad$\n\n\n综合算式: $\\qquad$", "options": [], "subject": "算术", "analysis": "解(1) $2 \\times 5+6+8=24$ (答案不唯一)\n\n(2) $9 \\times 8 \\div(7-4)=24$ (答案不唯一)\n\n【分析】(1)要使结果为 24,4 个数加起来等于 21 , 还差 3 , 就想 $2 \\times 5=10$ 比 $2+5=7$ 多 3 再加 6 加 8 正好得 24 ;\n\n(2) 4 个数加起来的 28 , 多 4 , 不能只用加减法, 还需用乘、除法。\n\n【详解】(1) $2 \\times 5+6+8$\n\n$=10+6+8$\n\n$=16+8$\n\n$=24$\n\n(答案不唯一)\n\n(2) $9 \\times 8 \\div(7-4)$\n\n$=72 \\div 3$\n\n$=24$\n\n(答案不唯一)\n\n【点睛】解决本题的关键是熟练掌握整数四则运算的计算方法。"} {"id": "27545", "image": [], "answer": "解减 加\n\n【分析】根据整数四则混合运算的顺序, 有括号的算式, 先算括号里面的, 再算括号外面的, 没有括号的算式, 有乘除法、又有加减法, 先算乘除法、再算加减法。据此解答即可。\n\n【详解】计算 $137+[480 \\div(657-637)]$ 时, 最先算减法, 最后算加法。\n\n【点睛】查了整数四则混合运算的顺序, 熟练掌握运算顺序是解答的关键。", "solution": "null", "level": "四年级", "question": "计算 $137+[480 \\div(657-637)]$ 时, 最先算 $($ 法, 最后算 $($ 法。", "options": [], "subject": "算术", "analysis": "解减 加\n\n【分析】根据整数四则混合运算的顺序, 有括号的算式, 先算括号里面的, 再算括号外面的, 没有括号的算式, 有乘除法、又有加减法, 先算乘除法、再算加减法。据此解答即可。\n\n【详解】计算 $137+[480 \\div(657-637)]$ 时, 最先算减法, 最后算加法。\n\n【点睛】查了整数四则混合运算的顺序, 熟练掌握运算顺序是解答的关键。"} {"id": "27547", "image": [], "answer": "解【解答】解: $454-318=136$\n\n$$\n\\begin{aligned}\n& 306-69=237 \\\\\n& 204 \\div 12=17 \\\\\n& 345 \\div 15=23 \\\\\n& 318+136=454 \\quad 306-237=69 \\quad 345 \\div 23=15\n\\end{aligned}\n$$\n\n故答案为: $136,237,17,23$ 。", "solution": "null", "level": "四年级", "question": "(2021 春・平山县期中)在横线里填上合适的数。\n\n$318+$ $\\qquad$ $=454 \\quad 306-$ $\\qquad$ $=69 \\quad 12 \\times$ $\\qquad$ $=204 \\quad 345 \\div$ $\\qquad$ $=15$", "options": [], "subject": "算术", "analysis": "解【解答】解: $454-318=136$\n\n$$\n\\begin{aligned}\n& 306-69=237 \\\\\n& 204 \\div 12=17 \\\\\n& 345 \\div 15=23 \\\\\n& 318+136=454 \\quad 306-237=69 \\quad 345 \\div 23=15\n\\end{aligned}\n$$\n\n故答案为: $136,237,17,23$ 。"} {"id": "27548", "image": [], "answer": "解【解答】解: $107-34=73$\n\n$34 \\times 73=2482$\n\n答: 正确的结果应该是 2482 。\n\n故答案为: 2482 。", "solution": "null", "level": "四年级", "question": "(2022 春・襄垣县月考)一位同学在用计算器算 $34 \\times \\square$ 时, 把乘号键误按成加号键, 得到的结果是 107 , 正确的结果应该是 $\\qquad$", "options": [], "subject": "算术", "analysis": "解【解答】解: $107-34=73$\n\n$34 \\times 73=2482$\n\n答: 正确的结果应该是 2482 。\n\n故答案为: 2482 。"} {"id": "27549", "image": [], "answer": "【解答】解: 一个数与它自己相加是这个数的 2 倍, 相减是 0 , 相除是 1 ;\n\n所以这个数是:\n\n$$\n\\begin{aligned}\n& (421-0-1) \\div 2 \\\\\n= & 420 \\div 2 \\\\\n= & 210\n\\end{aligned}\n$$\n\n答: 这个数是 210 。\n\n故答案为: 210 。", "solution": "null", "level": "四年级", "question": "(2022 春・固始县期中)一个数与自己相加、相减、相除所得的和、差、商的和是 421 , 这个数是。 $\\qquad$", "options": [], "subject": "算术", "analysis": "【解答】解: 一个数与它自己相加是这个数的 2 倍, 相减是 0 , 相除是 1 ;\n\n所以这个数是:\n\n$$\n\\begin{aligned}\n& (421-0-1) \\div 2 \\\\\n= & 420 \\div 2 \\\\\n= & 210\n\\end{aligned}\n$$\n\n答: 这个数是 210 。\n\n故答案为: 210 。"} {"id": "27550", "image": [], "answer": "解【解答】解:如果要改变 $600-300 \\div 50$ 的运算顺序, 先算减法, 那么必须使用括号, 算式是: $(600-300) \\div 50$ 。\n\n故答案为: $(600-300) \\div 50$ 。", "solution": "null", "level": "四年级", "question": "(2022 春・湖北期中)如果要改变 $600-300 \\div 50$ 的运算顺序,先算减法,那么必须使用括号,算式是 $\\qquad$", "options": [], "subject": "算术", "analysis": "解【解答】解:如果要改变 $600-300 \\div 50$ 的运算顺序, 先算减法, 那么必须使用括号, 算式是: $(600-300) \\div 50$ 。\n\n故答案为: $(600-300) \\div 50$ 。"} {"id": "27551", "image": [], "answer": "解【解答】解: $120 \\div 3 \\times 16$\n\n$=40 \\times 16$\n\n$=640$\n\n故答案为: 640 。", "solution": "null", "level": "四年级", "question": "(2022 春・铜梁区期末)一个数的 3 倍是 120 , 这个数的 16 倍是 $\\qquad$", "options": [], "subject": "算术", "analysis": "解【解答】解: $120 \\div 3 \\times 16$\n\n$=40 \\times 16$\n\n$=640$\n\n故答案为: 640 。"} {"id": "27552", "image": [], "answer": "【解答】解: 改写成综合算式是 $183 \\div 3+8 \\times 15=181$ 。\n\n故答案为: $183 \\div 3+8 \\times 15=181$ 。", "solution": "null", "level": "四年级", "question": "(2022 春・红花岗区期末)把下面几个分步式改写成综合算式是 $\\qquad$。\n\n$8 \\times 15=120$\n\n$61+120=181$\n\n$183 \\div 3=61$", "options": [], "subject": "算术", "analysis": "【解答】解: 改写成综合算式是 $183 \\div 3+8 \\times 15=181$ 。\n\n故答案为: $183 \\div 3+8 \\times 15=181$ 。"} {"id": "27573", "image": [], "answer": "【解答】解: $3 \\times 2=6$ (本)\n\n$15-6=9$ (本)\n\n答:第二小组原来有 9 本图书。\n\n故答案为: 9 。", "solution": "null", "level": "四年级", "question": "(2022 秋・顺义区期末)第一小组有 15 本图书, 给第二小组 3 本后, 两组的图书就一样多了。第二小组原来有 $\\qquad$本图书。", "options": [], "subject": "算术", "analysis": "【解答】解: $3 \\times 2=6$ (本)\n\n$15-6=9$ (本)\n\n答:第二小组原来有 9 本图书。\n\n故答案为: 9 。"} {"id": "27575", "image": [], "answer": "【解答】解: $a+352+148=a+(352+148)$ ,运用的运算定律是加法结合律。\n\n故答案为:加法结合律。", "solution": "null", "level": "四年级", "question": "(2022 秋 $\\cdot$ 合川区期末) $a+352+148=a+( 352+148 )$, 运用的运算定律是 $\\qquad$。", "options": [], "subject": "算术", "analysis": "【解答】解: $a+352+148=a+(352+148)$ ,运用的运算定律是加法结合律。\n\n故答案为:加法结合律。"} {"id": "27576", "image": ["13831.jpg"], "answer": "【解答】解: (1) $98+74+26=98+(74+26 )$\n\n(2) $(80+70) \\times 5=80 \\times 5+70 \\times 5$\n\n故答案为: 98,$26 ; 5,5$ 。", "solution": "null", "level": "四年级", "question": "(2021 秋・高青县期末)在横线上填上适当的数\n\n$ $\\qquad$ $\\qquad$", "options": [], "subject": "算术", "analysis": "【解答】解: (1) $98+74+26=98+(74+26 )$\n\n(2) $(80+70) \\times 5=80 \\times 5+70 \\times 5$\n\n故答案为: 98,$26 ; 5,5$ 。"} {"id": "27577", "image": [], "answer": "解【解答】解: $25 \\times 44$\n\n$=25 \\times(40+4)$\n\n$=25 \\times 40+25 \\times 4$\n\n$=1000+100=1100$\n\n这是运用了乘法分配律;\n\n$25 \\times 44$\n\n$=25 \\times(4 \\times 11)$\n\n$=25 \\times 4 \\times 11$\n\n$=100 \\times 11$\n\n$=1100$\n\n这是运用了乘法结合律。\n\n故答案为: 乘法分配, 乘法结合。", "solution": "null", "level": "四年级", "question": "(2021 秋 ・高青县期末)在计算 $25 \\times 44$ 时, 果果是这样想的: $25 \\times 44=25 \\times 40+25 \\times 4=1000+100$ $=1100$, 这是运用了 $\\qquad$律; 豆豆是这样想的: $25 \\times 44=25 \\times 4 \\times 11=100 \\times 11=1100$, 这是运用了 $\\qquad$律。", "options": [], "subject": "算术", "analysis": "解【解答】解: $25 \\times 44$\n\n$=25 \\times(40+4)$\n\n$=25 \\times 40+25 \\times 4$\n\n$=1000+100=1100$\n\n这是运用了乘法分配律;\n\n$25 \\times 44$\n\n$=25 \\times(4 \\times 11)$\n\n$=25 \\times 4 \\times 11$\n\n$=100 \\times 11$\n\n$=1100$\n\n这是运用了乘法结合律。\n\n故答案为: 乘法分配, 乘法结合。"} {"id": "27578", "image": [], "answer": "解【解答】解: $240 \\div 12 \\div 5$\n\n$=20 \\div 5$\n\n$=4$\n\n或 $240 \\div 12 \\div 5$\n\n$=240 \\div(12 \\times 5)$\n\n$=240 \\div 60$\n\n$=4$\n\n故答案为: 从左到右, $4 ; 240 \\div(12 \\times 5)$, 乘, 除。", "solution": "null", "level": "四年级", "question": "(2022 秋・汝州市校级期中) $240 \\div 12 \\div 5$, 既可以按照 $\\qquad$的顺序计算, 结果是 $\\qquad$ ;也可以加括号, 将原式变为 $\\qquad$ , 先算 $\\qquad$法, 再算 $\\qquad$法, 结果相同, 这样算更简便。", "options": [], "subject": "算术", "analysis": "解【解答】解: $240 \\div 12 \\div 5$\n\n$=20 \\div 5$\n\n$=4$\n\n或 $240 \\div 12 \\div 5$\n\n$=240 \\div(12 \\times 5)$\n\n$=240 \\div 60$\n\n$=4$\n\n故答案为: 从左到右, $4 ; 240 \\div(12 \\times 5)$, 乘, 除。"} {"id": "27598", "image": [], "answer": " 乘法结合律, 乘法分配律", "solution": "null", "level": "四年级", "question": "(2022 秋 - 芝罘区期末)计算 $88 \\times 125$ 时,可以采用不同的方法进行简便运算:\n$88 \\times 125=11 \\times 8 \\times 125=11 \\times(8 \\times 125)$, 运用的运算律是 $\\qquad$ ;\n\n$88 \\times 125=80 \\times 125+8 \\times 125$, 运用的运算律是 $\\qquad$。", "options": [], "subject": "算术", "analysis": "【解答】解:计算 $88 \\times 125$ 时, 可以采用不同的方法进行简便运算:\n\n$88 \\times 125=11 \\times 8 \\times 125=11 \\times(8 \\times 125)$, 运用的运算律是:乘法结合律;\n\n$88 \\times 125=80 \\times 125+8 \\times 125$, 运用的运算律是:乘法分配律。\n\n故答案为: 乘法结合律, 乘法分配律。"} {"id": "27600", "image": [], "answer": "解正确", "solution": "null", "level": "四年级", "question": "计算 $420 \\div(80-12 \\times 5)$ 时, 应先算乘法, 再算减法, 最后算除法。()", "options": [], "subject": "算术", "analysis": "解正确"} {"id": "27601", "image": [], "answer": "解正确", "solution": "null", "level": "四年级", "question": "在没有余数的除法里, 除数乘商等于被除数。()", "options": [], "subject": "算术", "analysis": "解正确"} {"id": "27603", "image": [], "answer": "解错误", "solution": "null", "level": "四年级", "question": "一个算式里, 既有小括号又有中括号, 要先算中括号里面的。()", "options": [], "subject": "算术", "analysis": "解错误"} {"id": "27625", "image": [], "answer": "正确", "solution": "null", "level": "四年级", "question": "$25 \\times[$ (356-270)%43]去掉中括号, 计算结果不变。()", "options": [], "subject": "算术", "analysis": "正确"} {"id": "27627", "image": ["13523.jpg", "13523.jpg"], "answer": "解54\n\n【分析】减少部分长方形的面积是 18 平方厘米, 宽是 3 厘米, 利用长方形的面积求出原来长方形的宽,原来长方形的长 $=$ 原来长方形的宽 +3 厘米,最后利用“长方形的面积 $=$ 长 $\\times$ 宽”,即可求得。\n\n【详解】\n\n\n\n$(18 \\div 3+3) \\times(18 \\div 3)$\n\n$=(6+3) \\times 6$\n\n$=9 \\times 6$\n\n$=54$ (平方厘米)\n\n所以, 原来长方形的面积是 54 平方厘米。\n\n【点睛】画图分析求出原来长方形的长和宽是解答题目的关键。", "solution": "null", "level": "四年级", "question": "有一个长方形, 它的长减少 3 厘米, 面积就减少 18 平方厘米, 剩下的部分正好是正方形, 原来长方形的面积是 ( ) 平方厘米。", "options": [], "subject": "算术", "analysis": "解54\n\n【分析】减少部分长方形的面积是 18 平方厘米, 宽是 3 厘米, 利用长方形的面积求出原来长方形的宽,原来长方形的长 $=$ 原来长方形的宽 +3 厘米,最后利用“长方形的面积 $=$ 长 $\\times$ 宽”,即可求得。\n\n【详解】\n\n\n\n$(18 \\div 3+3) \\times(18 \\div 3)$\n\n$=(6+3) \\times 6$\n\n$=9 \\times 6$\n\n$=54$ (平方厘米)\n\n所以, 原来长方形的面积是 54 平方厘米。\n\n【点睛】画图分析求出原来长方形的长和宽是解答题目的关键。"} {"id": "27628", "image": [], "answer": "解98\n\n【分析】乘法各部分间的关系:因数 $\\times$ 因数 $=$ 积,一个因数 $=$ 积 $\\div$ 另一个因数; 据此解题即可。\n\n【详解】 $67 \\times 98=6566$\n\n所以, $67 \\times 98=6566$, 那么 $6566 \\div 67=98$ 。\n\n【点睛】正确理解乘法各部分间的关系, 是解答此题的关键。", "solution": "null", "level": "四年级", "question": "$67 \\times 98=6566$, 那么, $6566 \\div 67=(\\quad)$ 。", "options": [], "subject": "算术", "analysis": "解98\n\n【分析】乘法各部分间的关系:因数 $\\times$ 因数 $=$ 积,一个因数 $=$ 积 $\\div$ 另一个因数; 据此解题即可。\n\n【详解】 $67 \\times 98=6566$\n\n所以, $67 \\times 98=6566$, 那么 $6566 \\div 67=98$ 。\n\n【点睛】正确理解乘法各部分间的关系, 是解答此题的关键。"} {"id": "27629", "image": [], "answer": "解$12 \\times(89+61)=1800$\n\n【分析】根据题意可知, 此题是先算加法, 再算乘法, 根据混合运算的计算顺序列出综合算式即可。\n\n【详解】 $12 \\times(89+61)$\n\n$=12 \\times 150$\n\n$=1800$\n\n即把 $89+61=150$ 与 $12 \\times 150=1800$ 合并成一个混合算式是 $12 \\times(89+61)=1800$ 。\n\n【点睛】熟练掌握混合运算的计算顺序是解答此题的关键。", "solution": "null", "level": "四年级", "question": "把 $89+61=150$ 与 $12 \\times 150=1800$ 合并成一个混合算式是 $(\\quad)$ 。", "options": [], "subject": "算术", "analysis": "解$12 \\times(89+61)=1800$\n\n【分析】根据题意可知, 此题是先算加法, 再算乘法, 根据混合运算的计算顺序列出综合算式即可。\n\n【详解】 $12 \\times(89+61)$\n\n$=12 \\times 150$\n\n$=1800$\n\n即把 $89+61=150$ 与 $12 \\times 150=1800$ 合并成一个混合算式是 $12 \\times(89+61)=1800$ 。\n\n【点睛】熟练掌握混合运算的计算顺序是解答此题的关键。"} {"id": "27630", "image": [], "answer": "解$8 \\times 8 \\div 2-8$\n\n【分析】解答此题应根据数的特点, 四则混合运算的运算顺序, 进行尝试凑数即可解决问题。\n\n【详解】 $8 \\times 8 \\div 2-8$\n\n$=64 \\div 2-8$\n\n$=32-8$\n\n$=24$\n\n所以, 用 $8 、 8 、 8 、 2$ “算 24 点”, 算式是: $8 \\times 8 \\div 2-8$ 。\n\n【点睛】此题考查对运算符号的熟练运用, 有一定的技巧性, 关键是掌握整数的四则混合运算。", "solution": "null", "level": "四年级", "question": "用 $8 、 8 、 8 、 2$ “算 24 点”, 算式是 ( )。", "options": [], "subject": "算术", "analysis": "解$8 \\times 8 \\div 2-8$\n\n【分析】解答此题应根据数的特点, 四则混合运算的运算顺序, 进行尝试凑数即可解决问题。\n\n【详解】 $8 \\times 8 \\div 2-8$\n\n$=64 \\div 2-8$\n\n$=32-8$\n\n$=24$\n\n所以, 用 $8 、 8 、 8 、 2$ “算 24 点”, 算式是: $8 \\times 8 \\div 2-8$ 。\n\n【点睛】此题考查对运算符号的熟练运用, 有一定的技巧性, 关键是掌握整数的四则混合运算。"} {"id": "27631", "image": [], "answer": "解$30 \\times[(16+24) \\div 2] \\quad$ 加\n\n【分析】16 与 24 的和为 $16+24$, 所以求 30 乘 16 与 24 的和的一半,积是多少, 即可写出综合算式;根据四则混合运算的运算顺序可知, 这个算式要先算加法。\n\n【详解】求“ 30 乘 16 与 24 的和的一半, 积是多少?”写成综合算式是 $30 \\times[(16+24) \\div 2]$\n\n这个算式要先算加法。\n\n【点睛】本题考查了学生根据文字述列出缩合算式的能力,完成此类问题要注意题目中“乘、和、一半、积”等此类体现运算顺序及运算符号的词语。", "solution": "null", "level": "四年级", "question": "30 乘 16 与 24 的和的一半, 积是多少?列成综合算式是( ), 这个算式先算( 法。", "options": [], "subject": "算术", "analysis": "解$30 \\times[(16+24) \\div 2] \\quad$ 加\n\n【分析】16 与 24 的和为 $16+24$, 所以求 30 乘 16 与 24 的和的一半,积是多少, 即可写出综合算式;根据四则混合运算的运算顺序可知, 这个算式要先算加法。\n\n【详解】求“ 30 乘 16 与 24 的和的一半, 积是多少?”写成综合算式是 $30 \\times[(16+24) \\div 2]$\n\n这个算式要先算加法。\n\n【点睛】本题考查了学生根据文字述列出缩合算式的能力,完成此类问题要注意题目中“乘、和、一半、积”等此类体现运算顺序及运算符号的词语。"} {"id": "27648", "image": [], "answer": "解938\n\n【分析】根据在有余数的除法中, 余数总比除数小, 即余数最大为: 除数 -1 , 当余数最大时, 被除数最大, 进而根据“被除数=商×除数+余数”解答即可。\n\n【详解】余数最大为: $3-1=2$\n\n被除数最大为: $3 \\times 312+2$\n\n$=936+2$\n\n$=938$\n\n【点睛】解答此题的关键: 根据在有余数的除法中, 余数总比除数小, 得出余数最大为: 除数 -1 ,然后被除数、除数、商和余数四个量之间的关系进行解答。", "solution": "null", "level": "四年级", "question": "在 $\\square \\div 3=312 \\ldots \\ldots$ 中,被除数最大是 $($ 。", "options": [], "subject": "算术", "analysis": "解938\n\n【分析】根据在有余数的除法中, 余数总比除数小, 即余数最大为: 除数 -1 , 当余数最大时, 被除数最大, 进而根据“被除数=商×除数+余数”解答即可。\n\n【详解】余数最大为: $3-1=2$\n\n被除数最大为: $3 \\times 312+2$\n\n$=936+2$\n\n$=938$\n\n【点睛】解答此题的关键: 根据在有余数的除法中, 余数总比除数小, 得出余数最大为: 除数 -1 ,然后被除数、除数、商和余数四个量之间的关系进行解答。"} {"id": "27649", "image": [], "answer": "解$540 \\quad 1$\n\n【分析】根据除法各部分间的关系: 被除数 $\\because$ 除数 $=$ 商, 被除数 $=$ 商 $\\times$ 余数, 除数 $=$ 被除数 $\\div$ 商, 据此解题即可。\n\n【详解】 $\\times \\triangle \\triangle=$ ॐ $=540$\n\n$\\hat{\\sim} \\div(0 \\times \\triangle)=\\hat{\\sim} \\div \\dot{\\psi}=1$\n已知 $\\div \\circ=\\triangle, \\nLeftarrow=540$, 那么 $\\times \\triangle=540$, $\\div(\\circ \\times \\triangle)=1$ 。\n\n【点睛】熟练掌握除法各部分间的关系以及等量代换, 是解答此题的关键。", "solution": "null", "level": "四年级", "question": "已知 $\\div 0=\\triangle, \\rightsquigarrow=540$, 那么 $\\times \\triangle \\triangle(\\quad), \\rightsquigarrow \\div(0 \\times \\triangle)=(\\quad)$", "options": [], "subject": "算术", "analysis": "解$540 \\quad 1$\n\n【分析】根据除法各部分间的关系: 被除数 $\\because$ 除数 $=$ 商, 被除数 $=$ 商 $\\times$ 余数, 除数 $=$ 被除数 $\\div$ 商, 据此解题即可。\n\n【详解】 $\\times \\triangle \\triangle=$ ॐ $=540$\n\n$\\hat{\\sim} \\div(0 \\times \\triangle)=\\hat{\\sim} \\div \\dot{\\psi}=1$\n已知 $\\div \\circ=\\triangle, \\nLeftarrow=540$, 那么 $\\times \\triangle=540$, $\\div(\\circ \\times \\triangle)=1$ 。\n\n【点睛】熟练掌握除法各部分间的关系以及等量代换, 是解答此题的关键。"} {"id": "27650", "image": [], "answer": "解1680\n\n【分析】在没有余数的除法中, 被除数 $=$ 除数 $\\times$ 商, 据此即可解答。\n\n【详解】根据 $1680 \\div 35=48$, 可知 $35 \\times 48=1680$\n\n【点睛】熟练掌握除法各部分间关系是解答本题的关键。", "solution": "null", "level": "四年级", "question": "根据 $1680 \\div 35=48$, 可知 $35 \\times 48$ 的得数是 $(\\quad)$ 。", "options": [], "subject": "算术", "analysis": "解1680\n\n【分析】在没有余数的除法中, 被除数 $=$ 除数 $\\times$ 商, 据此即可解答。\n\n【详解】根据 $1680 \\div 35=48$, 可知 $35 \\times 48=1680$\n\n【点睛】熟练掌握除法各部分间关系是解答本题的关键。"} {"id": "27652", "image": [], "answer": "解$(9-5) \\times(8-2)=24$\n\n【分析】将 9 和 5 相减得到 4,8 和 2 相减得到 6 , 再将两个差相乘, 即可得到 24。\n\n【详解】 $(9-5) \\times(8-2)$\n\n$=4 \\times 6$\n\n$=24$\n\n(答案不唯一)\n\n【点睛】本题考查“ 24 点”, 利用 $2 \\times 12=3 \\times 8=4 \\times 6=24$, 将已知数字通过加减乘除法得到 $2 、 12 、 3 、$\n\n8、4、6, 再进行计算。列算式时, 要根据运算顺序判断是否需要添上小括号。", "solution": "null", "level": "四年级", "question": "用 2、5、8、9 经过怎样的运算才能得到 24?可以这样列式计算: ( )。", "options": [], "subject": "算术", "analysis": "解$(9-5) \\times(8-2)=24$\n\n【分析】将 9 和 5 相减得到 4,8 和 2 相减得到 6 , 再将两个差相乘, 即可得到 24。\n\n【详解】 $(9-5) \\times(8-2)$\n\n$=4 \\times 6$\n\n$=24$\n\n(答案不唯一)\n\n【点睛】本题考查“ 24 点”, 利用 $2 \\times 12=3 \\times 8=4 \\times 6=24$, 将已知数字通过加减乘除法得到 $2 、 12 、 3 、$\n\n8、4、6, 再进行计算。列算式时, 要根据运算顺序判断是否需要添上小括号。"} {"id": "27653", "image": [], "answer": "解$112 \\times[(15+67) \\div 41]$\n\n【分析】四则混合运算的计算顺序是先算乘除法, 再算加减法; 当有中括号时, 应先算小括号里面的,再算中括号里面的, 最后算中括号外面的, 依此将括号加在合适的位置即可。\n\n【详解】 $112 \\times[(15+67) \\div 41]$\n\n$=112 \\times[82 \\div 41]$\n\n$=112 \\times 2$\n$=224$\n\n在算式 $112 \\times 15+67 \\div 41$ 内加上括号, 使之先算加法, 再算除法, 最后算乘法, 算式应该为:112×[(15 $+67) \\div 41]$ 。\n\n【点睛】熟练掌握混合运算的计算顺序是解答此题的关键。", "solution": "null", "level": "四年级", "question": "请在算式 $112 \\times 15+67 \\div 41$ 内加上括号, 使之先算加法, 再算除法, 最后算乘法, 算式应该为:", "options": [], "subject": "算术", "analysis": "解$112 \\times[(15+67) \\div 41]$\n\n【分析】四则混合运算的计算顺序是先算乘除法, 再算加减法; 当有中括号时, 应先算小括号里面的,再算中括号里面的, 最后算中括号外面的, 依此将括号加在合适的位置即可。\n\n【详解】 $112 \\times[(15+67) \\div 41]$\n\n$=112 \\times[82 \\div 41]$\n\n$=112 \\times 2$\n$=224$\n\n在算式 $112 \\times 15+67 \\div 41$ 内加上括号, 使之先算加法, 再算除法, 最后算乘法, 算式应该为:112×[(15 $+67) \\div 41]$ 。\n\n【点睛】熟练掌握混合运算的计算顺序是解答此题的关键。"} {"id": "27654", "image": [], "answer": "解商 $\\times$ 除数 + 余数\n\n【分析】在有余数的除法里, 根据“被除数 $\\div$ 除数 $=$ 商.....余数”可得, 余数 $=$ 被除数一商 $\\times$ 除数, 被除数 $=$ 商 $\\times$ 除数 + 余数; 据此解题即可。\n\n【详解】根据分析可知,\n\n根据除法各部分之间的关系,在有余数的除法里,被除数=商 $\\times$ 除数十余数。\n\n【点睛】本题主要考查了除法各部分之间的关系,正确理解被除数、除数、商和余数之间的关系,是解答此题的关键。", "solution": "null", "level": "四年级", "question": "根据除法各部分之间的关系, 在有余数的除法里, 被除数 $=(\\quad) 。$", "options": [], "subject": "算术", "analysis": "解商 $\\times$ 除数 + 余数\n\n【分析】在有余数的除法里, 根据“被除数 $\\div$ 除数 $=$ 商.....余数”可得, 余数 $=$ 被除数一商 $\\times$ 除数, 被除数 $=$ 商 $\\times$ 除数 + 余数; 据此解题即可。\n\n【详解】根据分析可知,\n\n根据除法各部分之间的关系,在有余数的除法里,被除数=商 $\\times$ 除数十余数。\n\n【点睛】本题主要考查了除法各部分之间的关系,正确理解被除数、除数、商和余数之间的关系,是解答此题的关键。"} {"id": "27655", "image": [], "answer": "解5\n\n653\n\n【分析】余数小于除数,余数最大为除数减 1 , 再根据“被除数 $=$ 除数 $\\times$ 商 + 余数”即可求出这个整数。\n\n【详解】 $6-1=5$\n\n$108 \\times 6+5$\n\n$=648+5$\n\n$=653$\n\n一个整数除以 6 , 商是 108 , 余数最大是 5 , 这个整数最大是 653 。\n\n【点睛】本题主要考查学生对除法各部分间关系的掌握和灵活运用。", "solution": "null", "level": "四年级", "question": "一个整数除以 6 , 商是 108 , 余数最大是 (\n) , 这个整数最大是 ( )。", "options": [], "subject": "算术", "analysis": "解5\n\n653\n\n【分析】余数小于除数,余数最大为除数减 1 , 再根据“被除数 $=$ 除数 $\\times$ 商 + 余数”即可求出这个整数。\n\n【详解】 $6-1=5$\n\n$108 \\times 6+5$\n\n$=648+5$\n\n$=653$\n\n一个整数除以 6 , 商是 108 , 余数最大是 5 , 这个整数最大是 653 。\n\n【点睛】本题主要考查学生对除法各部分间关系的掌握和灵活运用。"} {"id": "27656", "image": ["13532.jpg", "13533.jpg", "13533.jpg"], "answer": "解$50 ; 74 ; 626 ; 700-(300 \\div 6+24)=626$\n\n【分析】先计算除法, 再计算加法, 最后计算减法, 所以需要给除法和加法加一个小括号,据此解答。\n\n\n\n【详解】\n\n综合算式: $700-(300 \\div 6+24)=626$\n\n【点睛】本题考查四则混合运算的运算顺序,应熟练掌握并灵活运用。", "solution": "null", "level": "四年级", "question": "在 $\\square$ 里填上适当的数, 再列出综合算式。\n\n\n\n综合算式: $\\qquad$。", "options": [], "subject": "算术", "analysis": "解$50 ; 74 ; 626 ; 700-(300 \\div 6+24)=626$\n\n【分析】先计算除法, 再计算加法, 最后计算减法, 所以需要给除法和加法加一个小括号,据此解答。\n\n\n\n【详解】\n\n综合算式: $700-(300 \\div 6+24)=626$\n\n【点睛】本题考查四则混合运算的运算顺序,应熟练掌握并灵活运用。"} {"id": "27677", "image": [], "answer": "解$216 \\div(18-6) \\times 2=36$\n\n【分析】根据题意, 先算 18 减 6 的差, 再用 216 除以所得的差, 最后用所得的商乘 2 , 据此列式解答即可。\n\n【详解】 $216 \\div(18-6) \\times 2$\n\n$=216 \\div 12 \\times 2$\n\n$=18 \\times 2$\n\n$=36$\n\n所以把“ $18-6=12,216 \\div 12=18,18 \\times 2=36$ ”合并成一道综合算式是 $216 \\div(18-6) \\times 2=36$ 。\n\n【点睛】列综合算式, 关键是弄清运算顺序, 然后再列式解答。", "solution": "null", "level": "四年级", "question": "把“ $18-6=12,216 \\div 12=18,18 \\times 2=36$ ”合并成一道综合算式是 $($ 。", "options": [], "subject": "算术", "analysis": "解$216 \\div(18-6) \\times 2=36$\n\n【分析】根据题意, 先算 18 减 6 的差, 再用 216 除以所得的差, 最后用所得的商乘 2 , 据此列式解答即可。\n\n【详解】 $216 \\div(18-6) \\times 2$\n\n$=216 \\div 12 \\times 2$\n\n$=18 \\times 2$\n\n$=36$\n\n所以把“ $18-6=12,216 \\div 12=18,18 \\times 2=36$ ”合并成一道综合算式是 $216 \\div(18-6) \\times 2=36$ 。\n\n【点睛】列综合算式, 关键是弄清运算顺序, 然后再列式解答。"} {"id": "27679", "image": [], "answer": "解$>>=<$\n\n【分析】第一小题, $180-30 \\div 5=174,(180-30) \\div 5=30$, 所以, $180-30 \\div 5>(180-30) \\div 5$;第二小题, 根据整数减法性质可知, $703-(75+25)=703-75-25$, 所以, 703-75+25>703-( 75 $+25)$;\n\n第三小题, 根据乘法分配律可知, $98 \\times 25=100 \\times 25-2 \\times 25$;\n\n第四小题, 根据整数除法性质可知, $1600 \\div 80 \\div 5=1600 \\div(80 \\times 5)$, 所以 $1600 \\div 80 \\div 5<1600 \\div 80 \\times 5$ 。\n\n【详解】 $180-30 \\div 5>(180-30) \\div 5$\n\n$703-75+25>703-(75+25)$\n\n$98 \\times 25=100 \\times 25-2 \\times 25$\n\n$1600 \\div 80 \\div 5<1600 \\div 80 \\times 5$\n\n【点睛】解答此题时, 先计算出各个算式的结果, 再比较到达即可。", "solution": "null", "level": "四年级", "question": "在下面的括号里填上\" $>\" \\times<$ \"或\" $=\"$ 。\n$180-30 \\div 5($\n) $(180-30) \\div 5$\n$703-75+25($\n) $703-(75+25)$\n$98 \\times 25(\\quad) 100 \\times 25-2 \\times 25 \\quad 1600 \\div 80 \\div 5(\\quad) 1600 \\div 80 \\times 5$", "options": [], "subject": "算术", "analysis": "解$>>=<$\n\n【分析】第一小题, $180-30 \\div 5=174,(180-30) \\div 5=30$, 所以, $180-30 \\div 5>(180-30) \\div 5$;第二小题, 根据整数减法性质可知, $703-(75+25)=703-75-25$, 所以, 703-75+25>703-( 75 $+25)$;\n\n第三小题, 根据乘法分配律可知, $98 \\times 25=100 \\times 25-2 \\times 25$;\n\n第四小题, 根据整数除法性质可知, $1600 \\div 80 \\div 5=1600 \\div(80 \\times 5)$, 所以 $1600 \\div 80 \\div 5<1600 \\div 80 \\times 5$ 。\n\n【详解】 $180-30 \\div 5>(180-30) \\div 5$\n\n$703-75+25>703-(75+25)$\n\n$98 \\times 25=100 \\times 25-2 \\times 25$\n\n$1600 \\div 80 \\div 5<1600 \\div 80 \\times 5$\n\n【点睛】解答此题时, 先计算出各个算式的结果, 再比较到达即可。"} {"id": "27680", "image": [], "answer": "解$125 \\div[(572+78) \\div 26]=5$\n\n【分析】根据题意可知, 此题是先算加法, $650 \\div 26=25$, 最后算 $125 \\div 25=5$, 根据混合运算的计算顺序合并成一道综合算式即可, 四则混合运算的计算顺序是先算乘除法, 再算加减法; 当有中括号时,\n应先算小括号里面的,再算中括号里面的,最后算中括号外面的。\n\n【详解】根据混合运算的计算顺序将 $572+78=650,650 \\div 26=25,125 \\div 25=5$, 合并成一道综合算式是: $125 \\div[(572+78) \\div 26]=5$ 。\n\n【点睛】熟练掌握混合运算的计算顺序是解答此题的关键。", "solution": "null", "level": "四年级", "question": "把 $572+78=650,650 \\div 26=25,125 \\div 25=5$, 合并成一道综合算式是 $(\\quad)$ 。", "options": [], "subject": "算术", "analysis": "解$125 \\div[(572+78) \\div 26]=5$\n\n【分析】根据题意可知, 此题是先算加法, $650 \\div 26=25$, 最后算 $125 \\div 25=5$, 根据混合运算的计算顺序合并成一道综合算式即可, 四则混合运算的计算顺序是先算乘除法, 再算加减法; 当有中括号时,\n应先算小括号里面的,再算中括号里面的,最后算中括号外面的。\n\n【详解】根据混合运算的计算顺序将 $572+78=650,650 \\div 26=25,125 \\div 25=5$, 合并成一道综合算式是: $125 \\div[(572+78) \\div 26]=5$ 。\n\n【点睛】熟练掌握混合运算的计算顺序是解答此题的关键。"} {"id": "27681", "image": [], "answer": "解18\n\n【分析】题干中被减数、减数、差的和为 100 , 根据被减数、减数、差之间的关系, 被减数=减数十差, 可知被减数 + 被减数 $=100$, 即可求出被减数; 根据减数比差大 14 , 差 $+14=$ 减数, 可知差 + 差 $+14=50$; 据此解答。\n\n【详解】被减数 $=$ 减数十差\n\n因为被减数、减数、差的和是 100 ;\n\n被减数 + 被减数 $=100$, 被减数 $=50$;\n\n因为减数比差大 14 , 差 $+14+$ 差 $=50$;\n\n差: $(50-14) \\div 2$\n\n$=36 \\div 2$\n\n$=18$\n\n【点睛】本题考查减法运算中被减数、减数与差之间关系的灵活应用。", "solution": "null", "level": "四年级", "question": "在一道减法算式中, 如果被减数、减数、差的和为 100 , 减数比差大 14 , 那么差是 ( )。", "options": [], "subject": "算术", "analysis": "解18\n\n【分析】题干中被减数、减数、差的和为 100 , 根据被减数、减数、差之间的关系, 被减数=减数十差, 可知被减数 + 被减数 $=100$, 即可求出被减数; 根据减数比差大 14 , 差 $+14=$ 减数, 可知差 + 差 $+14=50$; 据此解答。\n\n【详解】被减数 $=$ 减数十差\n\n因为被减数、减数、差的和是 100 ;\n\n被减数 + 被减数 $=100$, 被减数 $=50$;\n\n因为减数比差大 14 , 差 $+14+$ 差 $=50$;\n\n差: $(50-14) \\div 2$\n\n$=36 \\div 2$\n\n$=18$\n\n【点睛】本题考查减法运算中被减数、减数与差之间关系的灵活应用。"} {"id": "27701", "image": [], "answer": "解积 减数\n\n【分析】本题根据乘法算式和减法算式中各部分的关系: “一个因数 $\\times$ 另一个因数 $=$ 积”可知, 一个因数=积 $\\div$ 另一个因数; “被减数一减数=差”可知, 被减数=差十减数。\n\n【详解】根据分析可得: 一个因数=积:另一个因数; 被减数=差+减数。\n\n【点睛】本题考查了乘法和减法算式中各部分的关系, 要能根据其中的两个量表示出第三个量。", "solution": "null", "level": "四年级", "question": "一个因数 $=(\\quad) \\div$ 另一个因数; 被减数 $=$ 差 $+($ 。", "options": [], "subject": "算术", "analysis": "解积 减数\n\n【分析】本题根据乘法算式和减法算式中各部分的关系: “一个因数 $\\times$ 另一个因数 $=$ 积”可知, 一个因数=积 $\\div$ 另一个因数; “被减数一减数=差”可知, 被减数=差十减数。\n\n【详解】根据分析可得: 一个因数=积:另一个因数; 被减数=差+减数。\n\n【点睛】本题考查了乘法和减法算式中各部分的关系, 要能根据其中的两个量表示出第三个量。"} {"id": "27702", "image": [], "answer": "解$(200+400) \\div 4-2 \\quad(200+400) \\div(4-2) \\quad 200+400 \\div(4-2)$\n\n【分析】根据四则混合运算顺序: 如果没有括号, 先算乘除, 再算加减; 如果有括号, 就先算括号里\n的计算。\n\n【详解】 (1) 先算加法, 再算除法, 最后算减法的算式: (200+400) $\\div 4-2$;\n\n(2) 最后算除法的算式: $(200+400) \\div(4-2)$;\n\n(3) $200+400 \\div(4-2)$\n\n$=200+400 \\div 2$\n\n$=200+200$\n\n$=400$\n\n则得数是 400 的算式: $200+400 \\div(4-2)$ 。\n\n【点睛】此题考查了整数的混合运算的应用, 关键是明确运算顺序即可。", "solution": "null", "level": "四年级", "question": "根据要求在算式 $200+400 \\div 4-2$ 中添上括号。\n\n(1) 先加, 再除, 最后减的算式( )。\n\n(2)最后算除法的算式( )。\n\n(3) 得数为 400 的算式( )。", "options": [], "subject": "算术", "analysis": "解$(200+400) \\div 4-2 \\quad(200+400) \\div(4-2) \\quad 200+400 \\div(4-2)$\n\n【分析】根据四则混合运算顺序: 如果没有括号, 先算乘除, 再算加减; 如果有括号, 就先算括号里\n的计算。\n\n【详解】 (1) 先算加法, 再算除法, 最后算减法的算式: (200+400) $\\div 4-2$;\n\n(2) 最后算除法的算式: $(200+400) \\div(4-2)$;\n\n(3) $200+400 \\div(4-2)$\n\n$=200+400 \\div 2$\n\n$=200+200$\n\n$=400$\n\n则得数是 400 的算式: $200+400 \\div(4-2)$ 。\n\n【点睛】此题考查了整数的混合运算的应用, 关键是明确运算顺序即可。"} {"id": "27703", "image": [], "answer": "解$36 \\quad 420$\n\n【分析】(被除数一余数) $\\div$ 商 $=$ 除数, 商 $\\times$ 除数 + 余数 $=$ 被除数, 据此即可解答。\n\n【详解】 $(478-10) \\div 13$\n\n$=468 \\div 13$\n\n$=36$\n\n$18 \\times 23+6$\n\n$=420$\n\n【点睛】熟练掌握除法各部分间关系是解答本题的关键。", "solution": "null", "level": "四年级", "question": "填表。\n\n| 被除数 | 除数 | 商 | 余数 |\n| :---: | :---: | :---: | :---: |\n| 478 | | 13 | 10 |\n| | 18 | 23 | 6 |", "options": [], "subject": "算术", "analysis": "解$36 \\quad 420$\n\n【分析】(被除数一余数) $\\div$ 商 $=$ 除数, 商 $\\times$ 除数 + 余数 $=$ 被除数, 据此即可解答。\n\n【详解】 $(478-10) \\div 13$\n\n$=468 \\div 13$\n\n$=36$\n\n$18 \\times 23+6$\n\n$=420$\n\n【点睛】熟练掌握除法各部分间关系是解答本题的关键。"} {"id": "27705", "image": [], "answer": "解$<>=>$\n\n【详解】略", "solution": "null", "level": "四年级", "question": "在 $\\qquad$填\">\"、\"<\"或“=\".\n\n$72 \\times 308$ $\\qquad$ $380 \\times 72$\n\n$64 \\times(30+50)$ $\\qquad$ $64 \\times 30+50$\n\n$480 \\div 12 \\div 4$ $\\qquad$ $480 \\div(12 \\times 4) \\quad 18 \\times 6 \\times 7$ $\\qquad$ $18 \\times 13$", "options": [], "subject": "算术", "analysis": "解$<>=>$\n\n【详解】略"} {"id": "14105", "image": [], "answer": "答案:$\\sqrt{ }$", "solution": "null", "level": "四年级", "question": "有一个角是 $60^{\\circ}$ 的等腰三角形一定是等边三角形。", "options": [], "subject": "度量几何学", "analysis": "答案:$\\sqrt{ }$"} {"id": "14120", "image": [], "answer": "28", "solution": "null", "level": "四年级", "question": "图中每个小方格的面积是 $1 \\mathrm{~cm}^{2}$, 那么涂色部分图形的面积是 ( $\\quad$ ) 平方厘米。", "options": [], "subject": "度量几何学", "analysis": "28"} {"id": "13976", "image": [], "answer": "答案:2614 【点拨】最长: $7+20-1=26$ (厘米), 最短: $20-7+1=$ 14 (厘米)。", "solution": "null", "level": "四年级", "question": "一个三角形的三条边的长度都是整厘米数, 其中的两条边的长度分别是 7 厘米和 20 厘米, 第三条边最长是( )厘米, 最短是( )厘米。", "options": [], "subject": "度量几何学", "analysis": "答案:2614 【点拨】最长: $7+20-1=26$ (厘米), 最短: $20-7+1=$ 14 (厘米)。"} {"id": "13986", "image": ["3059.jpg"], "answer": "答案:稳定", "solution": "null", "level": "四年级", "question": "法国的埃菲尔铁塔、自行车上的三角架上都有三角形,这是运用了三角形的 $(\\quad)$ 性。9.如右图,典典去爷爷家走第(2)条路最近。第(2)条路比第(1)条路近, 是因为两点间所有连线中 $(\\quad)$ 最短;\n\n\n\n图书馆第(2)条路比第(3)条路近, 是因为三角形任意两边之和( $\\quad$ )第三边。", "options": [], "subject": "度量几何学", "analysis": "答案:稳定"} {"id": "14055", "image": ["2894.jpg"], "answer": "105 , 75", "solution": "null", "level": "四年级", "question": "11. 如右图, $\\angle 1=40^{\\circ}, \\angle 3=65^{\\circ}$, 则 $\\angle 2=(\\quad)$, $\\angle 4=(\\quad)$ 。\n\n", "options": [], "subject": "度量几何学", "analysis": ""} {"id": "14067", "image": ["3080.jpg"], "answer": "9.048 右移动两位", "solution": "null", "level": "四年级", "question": "把14.3 。\n\n\n左图阴影部分的面积占整个图形的( )。", "options": [], "subject": "度量几何学", "analysis": "9.048 右移动两位"} {"id": "28084", "image": [], "answer": "$\\begin{array}{lllll} \\quad 5.06 & 806 & 9.02 & 7 & 3\\end{array}$\n\n【分析】 1 吨 $=1000$ 千克, 1 平方米 $=100$ 平方分米, 1 米 $=100$ 厘米, 1 元 $=10$ 角, 单位之间的换算,大单位换算成小单位要乘它们之间的进率; 小单位换算成大单位要除以它们之间的进率。\n\n【详解】 5060 千克 $=5.06$ 吨\n\n8.06 平方米 $=806$ 平方分米\n\n9 米 2 厘米 $=9.02$ 米\n\n7.3 元 $=7$ 元 3 角\n\n【点睛】此题考查名数的换算, 把高级单位的名数换算成低级单位的名数, 就乘单位间的进率; 把低级单位的名数换算成高级单位的名数, 就除以单位间的进率。", "solution": "null", "level": "四年级", "question": "填一填。\n\n5060 千克 $=(\\quad)$ 吨 8.06 平方米 $=(\\quad)$ 平方分米\n\n9 米 2 厘米 $=(\\quad)$ 米 7.3 元 $=(\\quad)$ 元 $($ 角", "options": [], "subject": "度量几何学", "analysis": "$\\begin{array}{lllll} \\quad 5.06 & 806 & 9.02 & 7 & 3\\end{array}$\n\n【分析】 1 吨 $=1000$ 千克, 1 平方米 $=100$ 平方分米, 1 米 $=100$ 厘米, 1 元 $=10$ 角, 单位之间的换算,大单位换算成小单位要乘它们之间的进率; 小单位换算成大单位要除以它们之间的进率。\n\n【详解】 5060 千克 $=5.06$ 吨\n\n8.06 平方米 $=806$ 平方分米\n\n9 米 2 厘米 $=9.02$ 米\n\n7.3 元 $=7$ 元 3 角\n\n【点睛】此题考查名数的换算, 把高级单位的名数换算成低级单位的名数, 就乘单位间的进率; 把低级单位的名数换算成高级单位的名数, 就除以单位间的进率。"} {"id": "28114", "image": ["13594.jpg"], "answer": "解300\n\n【分析】题图中的涂色部分可以看成是大长方形去掉长 $15 \\mathrm{~m}$ 、宽 $2 \\mathrm{~m}$ 的小长方形后剩余的部分, 其面积为 $22 \\times 15-2 \\times 15=(22-2) \\times 15=20 \\times 15=300\\left(\\mathrm{~m}^{2}\\right)$ 。\n\n【详解】 $22 \\times 15-2 \\times 15$\n\n$=(22-2) \\times 15$\n\n$=20 \\times 15$\n\n$=300\\left(\\mathrm{~m}^{2}\\right)$\n\n【点睛】本题既是一个有关面积计算的实际问题, 也是乘法分配律的一种几何模型, 主要考查学生对乘法分配律的掌握和灵活运用。", "solution": "null", "level": "四年级", "question": "如图中涂色部分的面积是 ( ) m²\n\n", "options": [], "subject": "度量几何学", "analysis": "解300\n\n【分析】题图中的涂色部分可以看成是大长方形去掉长 $15 \\mathrm{~m}$ 、宽 $2 \\mathrm{~m}$ 的小长方形后剩余的部分, 其面积为 $22 \\times 15-2 \\times 15=(22-2) \\times 15=20 \\times 15=300\\left(\\mathrm{~m}^{2}\\right)$ 。\n\n【详解】 $22 \\times 15-2 \\times 15$\n\n$=(22-2) \\times 15$\n\n$=20 \\times 15$\n\n$=300\\left(\\mathrm{~m}^{2}\\right)$\n\n【点睛】本题既是一个有关面积计算的实际问题, 也是乘法分配律的一种几何模型, 主要考查学生对乘法分配律的掌握和灵活运用。"} {"id": "28167", "image": [], "answer": "解$78 \\quad 7 \\quad 5$\n\n【分析】高级单位换低级单位乘进率,根据 1 平方千米 $=100$ 公顷,用 $0.78 \\times 100$ 即可; 把 7.05 拆成 7 +0.05 , 用 $0.05 \\times 100$ 即可。\n\n【详解】 0.78 平方千米 $=0.78 \\times 100$ 公顷 $=78$ 公顷\n\n7.05 平方千米 $=7$ 平方千米 +0.05 平方千米 $=7$ 平方千米 $+0.05 \\times 100$ 公顷 $=7$ 平方千米 5 公顷\n【点睛】本题考查单位换算, 明确各单位之间的进率是解题的关键。", "solution": "null", "level": "四年级", "question": "0.78 平方千米 $=($\n)公顷\n7.05 平方千米 $=($\n)平方千米\n)公顷", "options": [], "subject": "度量几何学", "analysis": "解$78 \\quad 7 \\quad 5$\n\n【分析】高级单位换低级单位乘进率,根据 1 平方千米 $=100$ 公顷,用 $0.78 \\times 100$ 即可; 把 7.05 拆成 7 +0.05 , 用 $0.05 \\times 100$ 即可。\n\n【详解】 0.78 平方千米 $=0.78 \\times 100$ 公顷 $=78$ 公顷\n\n7.05 平方千米 $=7$ 平方千米 +0.05 平方千米 $=7$ 平方千米 $+0.05 \\times 100$ 公顷 $=7$ 平方千米 5 公顷\n【点睛】本题考查单位换算, 明确各单位之间的进率是解题的关键。"} {"id": "28169", "image": [], "answer": "解(1) 0.15\n\n(2)6.05\n\n【分析】 1 米 $=100$ 厘米; 1 千克 $=1000$ 克; 高级单位转化为低级单位要乘进率, 低级单位转化为高级单位要除以进率。据此解答。\n\n【详解】(1) $15 \\div 100=0.15,15$ 厘米 $=0.15$ 米。\n\n一支铅笔长 15 厘米, 是 0.15 米。\n\n(2) 6 千克 50 克可以看成 6 千克和 50 克, $50 \\div 1000=0.05,50$ 克 $=0.05$ 千克, 所以 6 千克 50 克 $=$ 6.05 千克。\n\n一袋苹果重 6 千克 50 克, 是 6.05 千克。\n\n【点睛】本题考查学生对运用小数点移动规律进行单位换算的掌握。熟练掌握单位间的进率是解决此题的关键。", "solution": "null", "level": "四年级", "question": "在括号里填上适当的小数。\n\n(1)一支铅笔长 15 厘米, 是( )米。\n\n(2)一袋苹果重 6 千克 50 克, 是( )千克。", "options": [], "subject": "度量几何学", "analysis": "解(1) 0.15\n\n(2)6.05\n\n【分析】 1 米 $=100$ 厘米; 1 千克 $=1000$ 克; 高级单位转化为低级单位要乘进率, 低级单位转化为高级单位要除以进率。据此解答。\n\n【详解】(1) $15 \\div 100=0.15,15$ 厘米 $=0.15$ 米。\n\n一支铅笔长 15 厘米, 是 0.15 米。\n\n(2) 6 千克 50 克可以看成 6 千克和 50 克, $50 \\div 1000=0.05,50$ 克 $=0.05$ 千克, 所以 6 千克 50 克 $=$ 6.05 千克。\n\n一袋苹果重 6 千克 50 克, 是 6.05 千克。\n\n【点睛】本题考查学生对运用小数点移动规律进行单位换算的掌握。熟练掌握单位间的进率是解决此题的关键。"} {"id": "28188", "image": ["13601.jpg", "13602.jpg", "13603.jpg", "13604.jpg", "13602.jpg", "13603.jpg", "13604.jpg"], "answer": "解$\\quad 0.8 \\quad 1.43 \\quad 0.39$\n\n【分析】第一幅图,把一个圆看作整体“1”,平均分成 10 份, 涂色部分为 8 份, 用小数表示是 0.8 。第二幅图,可以分为两部分:左面的正方形看作整体“1”,把右面的正方形看作整体“1”,平均分成 100 份, 涂色部分表示其中的 43 份, 用小数表示是 0.43 ; 与左面的整体“1\"合起来为 1.43 。第三幅图,把整体“ 1 ”平均分成 100 份,涂色部分是 39 份,用小数表示是 0.39 。\n\n【详解】用小数表示下面各图涂色部分; 如下:\n\n\n\n0.8\n\n\n\n1.43\n\n\n\n0.39\n\n【点睛】熟练掌握小数的意义是解答本题的关键。", "solution": "null", "level": "四年级", "question": "用小数表示下面各图涂色部分。\n\n\n( )", "options": [], "subject": "度量几何学", "analysis": "解$\\quad 0.8 \\quad 1.43 \\quad 0.39$\n\n【分析】第一幅图,把一个圆看作整体“1”,平均分成 10 份, 涂色部分为 8 份, 用小数表示是 0.8 。第二幅图,可以分为两部分:左面的正方形看作整体“1”,把右面的正方形看作整体“1”,平均分成 100 份, 涂色部分表示其中的 43 份, 用小数表示是 0.43 ; 与左面的整体“1\"合起来为 1.43 。第三幅图,把整体“ 1 ”平均分成 100 份,涂色部分是 39 份,用小数表示是 0.39 。\n\n【详解】用小数表示下面各图涂色部分; 如下:\n\n\n\n0.8\n\n\n\n1.43\n\n\n\n0.39\n\n【点睛】熟练掌握小数的意义是解答本题的关键。"} {"id": "28238", "image": [], "answer": "解$\\quad 6500 \\quad 0.8 \\quad 0.24 \\quad 2070$\n\n【分析】1 公顷 $=10000$ 平方米, 1 平方千米 $=100$ 公顷, 1 千克 $=1000$ 克, 1 千米 $=1000$ 米, 高级单位换算低级单位乘进率,低级单位换算高级单位除以进率,据此解答。\n\n【详解】 (1) $0.65 \\times 10000=6500$ (平方米)\n\n(2) $80 \\div 100=0.8$ (平方千米)\n\n(3) $240 \\div 1000=0.24$ (千克)\n\n(4) $2.07 \\times 1000=2070$ (米)\n\n所以, 0.65 公顷 $=6500$ 平方米, 80 公顷 $=0.8$ 平方千米, 240 克 $=0.24$ 千克, 2.07 千米 $=2070$ 米。\n\n【点睛】熟记单位之间的进率并掌握高低级单位之间转化的方法是解答题目的关键。", "solution": "null", "level": "四年级", "question": "0.65 公顷 $=(\\quad 80$ 公顷 $=(\\quad$ 平方米 $\\quad$ 平方千米 240 克 $=(\\quad 2.07$ 千米 $=(\\quad$ 千克 $\\quad$ 米", "options": [], "subject": "度量几何学", "analysis": "解$\\quad 6500 \\quad 0.8 \\quad 0.24 \\quad 2070$\n\n【分析】1 公顷 $=10000$ 平方米, 1 平方千米 $=100$ 公顷, 1 千克 $=1000$ 克, 1 千米 $=1000$ 米, 高级单位换算低级单位乘进率,低级单位换算高级单位除以进率,据此解答。\n\n【详解】 (1) $0.65 \\times 10000=6500$ (平方米)\n\n(2) $80 \\div 100=0.8$ (平方千米)\n\n(3) $240 \\div 1000=0.24$ (千克)\n\n(4) $2.07 \\times 1000=2070$ (米)\n\n所以, 0.65 公顷 $=6500$ 平方米, 80 公顷 $=0.8$ 平方千米, 240 克 $=0.24$ 千克, 2.07 千米 $=2070$ 米。\n\n【点睛】熟记单位之间的进率并掌握高低级单位之间转化的方法是解答题目的关键。"} {"id": "28262", "image": [], "answer": "$\\begin{array}{lllllll} & 0.95 & 3.04 & 132 & 7.08 & 3.01 & 3.06\\end{array}$\n\n【分析】 $1 \\mathrm{~km}=1000 \\mathrm{~m}, 1 \\mathrm{~m}=100 \\mathrm{~cm}, 1 \\mathrm{~m}^{2}=100 \\mathrm{dm}^{2}, 1 \\mathrm{~kg}=1000 \\mathrm{~g}, 1 \\mathrm{t}=1000 \\mathrm{~kg}$, 据此即可解答。\n\n【详解】 $950 \\mathrm{~m}=0.95 \\mathrm{~km} \\quad 3 \\mathrm{~kg} 40 \\mathrm{~g}=3.04 \\mathrm{~kg}$\n\n$1.32 \\mathrm{~m}=132 \\mathrm{~cm} \\quad 7 \\mathrm{~m}^{2} 8 \\mathrm{dm}^{2}=7.08 \\mathrm{~m}^{2}$\n\n$3 \\mathrm{~km} 10 \\mathrm{~m}=3.01 \\mathrm{~km} \\quad 360 \\mathrm{~kg}=3.06 \\mathrm{t}$\n\n【点睛】单位换算题, 熟练掌握长度、面积和质量单位间的进率是解答本题的关键。", "solution": "null", "level": "四年级", "question": "$950 \\mathrm{~m}=($\n)km\n$3 \\mathrm{~kg} 40 \\mathrm{~g}=($\n) $\\mathrm{kg}$\n\n| $1.32 \\mathrm{~m}=($ | ) $\\mathrm{cm}$ | $7 \\mathrm{~m}^{2} 8 \\mathrm{dm}^{2}=($ | ) $\\mathrm{m}^{2}$ |\n| :--- | :--- | ---: | :--- |\n| $3 \\mathrm{~km} 10 \\mathrm{~m}=($ | ) $\\mathrm{km}$ | $3 \\mathrm{t} 60 \\mathrm{~kg}=($ | ) $\\mathrm{t}$ |", "options": [], "subject": "度量几何学", "analysis": "$\\begin{array}{lllllll} & 0.95 & 3.04 & 132 & 7.08 & 3.01 & 3.06\\end{array}$\n\n【分析】 $1 \\mathrm{~km}=1000 \\mathrm{~m}, 1 \\mathrm{~m}=100 \\mathrm{~cm}, 1 \\mathrm{~m}^{2}=100 \\mathrm{dm}^{2}, 1 \\mathrm{~kg}=1000 \\mathrm{~g}, 1 \\mathrm{t}=1000 \\mathrm{~kg}$, 据此即可解答。\n\n【详解】 $950 \\mathrm{~m}=0.95 \\mathrm{~km} \\quad 3 \\mathrm{~kg} 40 \\mathrm{~g}=3.04 \\mathrm{~kg}$\n\n$1.32 \\mathrm{~m}=132 \\mathrm{~cm} \\quad 7 \\mathrm{~m}^{2} 8 \\mathrm{dm}^{2}=7.08 \\mathrm{~m}^{2}$\n\n$3 \\mathrm{~km} 10 \\mathrm{~m}=3.01 \\mathrm{~km} \\quad 360 \\mathrm{~kg}=3.06 \\mathrm{t}$\n\n【点睛】单位换算题, 熟练掌握长度、面积和质量单位间的进率是解答本题的关键。"} {"id": "28289", "image": [], "answer": "解$\\begin{array}{lllll}0.3 & 148 & 0.25 & 780 & 7.5\\end{array}$\n\n【分析】根据 1 吨 $=1000$ 千克, 1 米 $=100$ 厘米, 1 小时 $=60$ 分钟, 1 公顷 $=10000$ 平方米, 1 平方米 $=100$ 平方分米, 高级单位换算成低级单位, 乘进率, 低级单位换算成高级单位, 除以进率, 据此解答。\n\n【详解】 300 千克 $=0.3$ 吨\n\n1.48 米 $=148$ 厘米\n\n15 分 $=0.25$ 时\n\n0.078 公顷 $=780$ 平方米\n\n7 平方米 50 平方分米 $=7.5$ 平方米\n\n【点睛】本题考查单位之间的互化, 关键是熟记进率。", "solution": "null", "level": "四年级", "question": "300 千克 $=(\\quad$ 吨 1.48 米 $=(\\quad)$ 厘米 15 分 $=(\\quad$ 时 0.078 公顷 $=(\\quad)$ 平方米 7 平方米 50 平方分米 $=(\\quad)$ 平方米", "options": [], "subject": "度量几何学", "analysis": "解$\\begin{array}{lllll}0.3 & 148 & 0.25 & 780 & 7.5\\end{array}$\n\n【分析】根据 1 吨 $=1000$ 千克, 1 米 $=100$ 厘米, 1 小时 $=60$ 分钟, 1 公顷 $=10000$ 平方米, 1 平方米 $=100$ 平方分米, 高级单位换算成低级单位, 乘进率, 低级单位换算成高级单位, 除以进率, 据此解答。\n\n【详解】 300 千克 $=0.3$ 吨\n\n1.48 米 $=148$ 厘米\n\n15 分 $=0.25$ 时\n\n0.078 公顷 $=780$ 平方米\n\n7 平方米 50 平方分米 $=7.5$ 平方米\n\n【点睛】本题考查单位之间的互化, 关键是熟记进率。"} {"id": "28293", "image": [], "answer": "解$84000 \\quad 24 \\quad 0.24$\n\n【分析】(1)高级单位公顷化低级单位平方米乘进率 10000。\n\n(2)低级单位平方厘米化高级单位平方分米除以进率 100 ; 化高级单位平方米除以进率 10000 。\n\n【详解】(1) 8.4 公顷 $=84000$ 平方米\n\n(2) 2400 平方厘米 $=24$ 平方分米 $=0.24$ 平方米\n\n【点睛】平方米、平方分米、平方厘米相邻单位间的进率是 100 , 平方米与公顷间的进率是 10000 ,公顷与平方千米间的进率是 100, 由高级单位化低级单位乘进率,反之除以进率。", "solution": "null", "level": "四年级", "question": "8.4 公顷 $=(\\quad)$ 平方米 2400 平方厘米 $=(\\quad)$ 平方分米 $=(\\quad$ 平方米", "options": [], "subject": "度量几何学", "analysis": "解$84000 \\quad 24 \\quad 0.24$\n\n【分析】(1)高级单位公顷化低级单位平方米乘进率 10000。\n\n(2)低级单位平方厘米化高级单位平方分米除以进率 100 ; 化高级单位平方米除以进率 10000 。\n\n【详解】(1) 8.4 公顷 $=84000$ 平方米\n\n(2) 2400 平方厘米 $=24$ 平方分米 $=0.24$ 平方米\n\n【点睛】平方米、平方分米、平方厘米相邻单位间的进率是 100 , 平方米与公顷间的进率是 10000 ,公顷与平方千米间的进率是 100, 由高级单位化低级单位乘进率,反之除以进率。"} {"id": "28318", "image": [], "answer": "解$<><>==$\n\n【分析】1 米 $=100$ 厘米, 1 米 $=1000$ 毫米, 1 千克 $=1000$ 克, 1 平方米 $=100$ 平方分米, 1 元 $=10$ 角,根据进率移动小数点, 统一单位后再进行比较; $17.8 \\div 100$ 也就是把小数点向左移动两位; 据此解答。\n\n【详解】根据分析:\n\n(1)米和厘米之间的进率是 100, 大单位米转换成小单位厘米,把小数点向右移动两位; 6.4 米=640 厘米, 64 厘米 $<640$ 厘米, 所以 64 厘米 $<6.4$ 米;\n\n(2)米和毫米之间的进率是 1000, 大单位米转换成小单位毫米, 把小数点向右移动三位; 0.67 米=670 毫米, 670 毫米 $>67$ 毫米, 所以 0.67 米 $>67$ 毫米;\n\n(3)克和千克之间的进率是 1000, 大单位千克转换成小单位克, 把小数点向右移动三位; 1.02 千克 $=$ 1020 克, 580 克 $<1020$ 克, 所以 580 克 $<1.02$ 千克;\n\n(4)平方米和平方分米之间的进率是 100 , 大单位平方米转换成小单位平方分米, 把小数点向右移动两位; 2.5 平方米 $=250$ 平方分米, 250 平方分米 $>205$ 平方分米,所以 2.5 平方米 $>205$ 平方分米; (5)元和角之间的进率是 10 , 大单位元转换成小单位角, 把小数点向右移动一位; 5 元 $=50$ 角, $50+4$\n$=54$ (角) ; 5.4 元 $=54$ 角, 54 角 $=54$ 角, 所以 5 元 4 角 $=5.4$ 元;\n\n(6) $17.8 \\div 100=0.178,0.178=0.178$, 所以 $17.8 \\div 100=0.178$ 。\n\n【点睛】掌握小数点移动引起的小数的变化是解答本题的关键。", "solution": "null", "level": "四年级", "question": "比较大小。\n\n64 厘米 $\\quad 6.4$ 米 0.67 米( 67 毫米\n580 克 (\n)1.02 千克\n2.5 平方米 (\n)205 平方分米\n\n)千克; 1 吨小麦可磨出面粉 $\\qquad$\n5 元 4 角 (\n)5.4 元\n$17.8 \\div 100($\n0.178", "options": [], "subject": "度量几何学", "analysis": "解$<><>==$\n\n【分析】1 米 $=100$ 厘米, 1 米 $=1000$ 毫米, 1 千克 $=1000$ 克, 1 平方米 $=100$ 平方分米, 1 元 $=10$ 角,根据进率移动小数点, 统一单位后再进行比较; $17.8 \\div 100$ 也就是把小数点向左移动两位; 据此解答。\n\n【详解】根据分析:\n\n(1)米和厘米之间的进率是 100, 大单位米转换成小单位厘米,把小数点向右移动两位; 6.4 米=640 厘米, 64 厘米 $<640$ 厘米, 所以 64 厘米 $<6.4$ 米;\n\n(2)米和毫米之间的进率是 1000, 大单位米转换成小单位毫米, 把小数点向右移动三位; 0.67 米=670 毫米, 670 毫米 $>67$ 毫米, 所以 0.67 米 $>67$ 毫米;\n\n(3)克和千克之间的进率是 1000, 大单位千克转换成小单位克, 把小数点向右移动三位; 1.02 千克 $=$ 1020 克, 580 克 $<1020$ 克, 所以 580 克 $<1.02$ 千克;\n\n(4)平方米和平方分米之间的进率是 100 , 大单位平方米转换成小单位平方分米, 把小数点向右移动两位; 2.5 平方米 $=250$ 平方分米, 250 平方分米 $>205$ 平方分米,所以 2.5 平方米 $>205$ 平方分米; (5)元和角之间的进率是 10 , 大单位元转换成小单位角, 把小数点向右移动一位; 5 元 $=50$ 角, $50+4$\n$=54$ (角) ; 5.4 元 $=54$ 角, 54 角 $=54$ 角, 所以 5 元 4 角 $=5.4$ 元;\n\n(6) $17.8 \\div 100=0.178,0.178=0.178$, 所以 $17.8 \\div 100=0.178$ 。\n\n【点睛】掌握小数点移动引起的小数的变化是解答本题的关键。"} {"id": "28338", "image": [], "answer": "解0.682000\n\n【分析】 1 公顷 $=10000$ 平方米, 1 平方千米 $=100$ 公顷, 根据这两个进率进行单位换算即可。\n\n【详解】 $6800 \\div 10000=0.68$ (公顷)\n\n$20 \\times 100=2000$ (公顷)\n\n所以, 6800 平方米 $=0.68$ 公顷, 20 平方千米 $=2000$ 公顷。\n\n【点睛】本题考查了单位换算, 掌握平方千米、平方米和公顷之间的进率是解题的关键。", "solution": "null", "level": "四年级", "question": "6800 平方米 $=($\n\n)公顷 20 平方千米 $=($\n\n)公顷", "options": [], "subject": "度量几何学", "analysis": "解0.682000\n\n【分析】 1 公顷 $=10000$ 平方米, 1 平方千米 $=100$ 公顷, 根据这两个进率进行单位换算即可。\n\n【详解】 $6800 \\div 10000=0.68$ (公顷)\n\n$20 \\times 100=2000$ (公顷)\n\n所以, 6800 平方米 $=0.68$ 公顷, 20 平方千米 $=2000$ 公顷。\n\n【点睛】本题考查了单位换算, 掌握平方千米、平方米和公顷之间的进率是解题的关键。"} {"id": "28396", "image": [], "answer": "解$180.95 \\quad 181.0$\n\n【分析】改成以“万”为单位的数: 先找到万位, 再在万位后面点“.”, 然后根据实际情况进行化简, 最后在数的末尾加一个“万”字;\n精确到十分位, 也就是保留一位小数, 保留一位小数时, 就把十分位后面的数省略, 当百分位上的数等于或大于 5 时, 应向十分位上进 1 后再省略; 当百分位上的数小于 5 时, 就直接省略, 在表示近似数时, 小数末尾的 0 不能去掉; 依此计算。\n\n【详解】1809500 万位上的数是 0 , 即 1809500 改写用“万”作单位是 180.95 万。\n\n180.95 百分位上的数是 5 , 即 180.95 万精确到十分位约是 181.0 万。\n\n【点睛】熟练掌握小数近似数的计算, 以及整数的改写方法, 是解答本题的关键。", "solution": "null", "level": "四年级", "question": "长江是亚洲第一大河, 流域面积达 1809500 平方千米, 改写用“万”作单位是 ( )万平方千米, 精确到十分位约是 ( )万平方千米。", "options": [], "subject": "度量几何学", "analysis": "解$180.95 \\quad 181.0$\n\n【分析】改成以“万”为单位的数: 先找到万位, 再在万位后面点“.”, 然后根据实际情况进行化简, 最后在数的末尾加一个“万”字;\n精确到十分位, 也就是保留一位小数, 保留一位小数时, 就把十分位后面的数省略, 当百分位上的数等于或大于 5 时, 应向十分位上进 1 后再省略; 当百分位上的数小于 5 时, 就直接省略, 在表示近似数时, 小数末尾的 0 不能去掉; 依此计算。\n\n【详解】1809500 万位上的数是 0 , 即 1809500 改写用“万”作单位是 180.95 万。\n\n180.95 百分位上的数是 5 , 即 180.95 万精确到十分位约是 181.0 万。\n\n【点睛】熟练掌握小数近似数的计算, 以及整数的改写方法, 是解答本题的关键。"} {"id": "28415", "image": [], "answer": "解$527 \\quad 1.48$\n\n【分析】根据进率: 1 平方米 $=100$ 平方分米, 1 米 $=100$ 厘米; 从高级单位向低级单位转换, 乘进率;从低级单位向高级单位转换, 除以进率; 据此解答。\n\n【详解】(1) $5.27 \\times 100=527$ (平方分米)\n\n5.27 平方米 $=527$ 平方分米\n\n(2) $148 \\div 100=1.48$ (米)\n\n148 厘米 $=1.48$ 米\n\n【点睛】掌握各单位之间的进率以及转换方向是单位换算的关键。", "solution": "null", "level": "四年级", "question": "5.27 平方米 $=($\n)平方分米\n148 厘米 $=($\n)米", "options": [], "subject": "度量几何学", "analysis": "解$527 \\quad 1.48$\n\n【分析】根据进率: 1 平方米 $=100$ 平方分米, 1 米 $=100$ 厘米; 从高级单位向低级单位转换, 乘进率;从低级单位向高级单位转换, 除以进率; 据此解答。\n\n【详解】(1) $5.27 \\times 100=527$ (平方分米)\n\n5.27 平方米 $=527$ 平方分米\n\n(2) $148 \\div 100=1.48$ (米)\n\n148 厘米 $=1.48$ 米\n\n【点睛】掌握各单位之间的进率以及转换方向是单位换算的关键。"} {"id": "28420", "image": ["13634.jpg"], "answer": "解$\\mathrm{ac}$ bc $\\mathrm{ac}+\\mathrm{bc} \\quad \\mathrm{a}+\\mathrm{b}$ c $(\\mathrm{a}+\\mathrm{b}) \\mathrm{c} \\quad \\mathrm{ac}+\\mathrm{bc \\# \\#}(\\mathrm{a}+\\mathrm{b}) \\mathrm{c} \\quad(\\mathrm{a}+\\mathrm{b}) \\mathrm{c} \\# \\# \\mathrm{ac}+\\mathrm{bc}$ 相乘 $\\quad$ 相加 $\\quad(\\mathrm{a}+\\mathrm{b}) \\mathrm{c}=\\mathrm{ac}+\\mathrm{bc}$\n\n【分析】观察图示, 种西红柿和黄瓜的菜地, 长分别为 $\\mathrm{a}$ 和 $\\mathrm{b}$, 宽是 $\\mathrm{c}$, 根据长方形面积 $=$ 长 $\\times$ 宽进行解答; 两种方法求菜地总面积, 用 2 个长方形面积相加或看成一个长方形进行计算。方法不同, 但结果相同, 再通过两个结果的不同表达方式寻找规律即可。\n\n【详解】方法一:西红柿的面积是 $\\mathrm{ac}$, 黄瓜的面积是 $\\mathrm{bc}$, 所以这块菜地的面积是 $\\mathrm{ac}+\\mathrm{bc}$ 。方法二:菜地的面积是大长方形,它的长是 $\\mathrm{a}+\\mathrm{b}$, 宽是 $\\mathrm{c}$, 面积也可以这样计算: $(a+b) c$ 。综合方法一、二,菜地面积 $=\\mathrm{ac}+\\mathrm{bc}=(\\mathrm{a}+\\mathrm{b}) \\mathrm{c}$ 。\n\n通过不同的计算方法可发现,两个数的和与一个数相乘,可以先把它们与这个数分别相乘,再相加,这叫乘法分配律,用字母可以表示为 $(\\mathrm{a}+\\mathrm{b}) \\mathrm{c}=\\mathrm{ac}+\\mathrm{bc}$ 。\n\n【点睛】本题主要考查乘法分配律的应用, 便于学生熟练掌握。", "solution": "null", "level": "四年级", "question": "王伯伯家有一块菜地, 种着西红柿和黄瓜(如图), 这块菜地的面积是多少平方米?\n\n\n\n方法一:西红柿的面积是( ), 黄瓜的面积是( ), 所以这块菜地的面积是( )。方法二: 莱地的面积是大长方形, 它的长是( ), 宽是( ), 面积也可以这样计算( )。通过比较: 菜地的面积 $=(\\quad)=(\\quad)$ 。\n\n我发现: 两个数的和与一个数相乘, 可以先把它们与这个数分别( ), 再( ), 这叫乘法分配律,用字母可以表示为( )。", "options": [], "subject": "度量几何学", "analysis": "解$\\mathrm{ac}$ bc $\\mathrm{ac}+\\mathrm{bc} \\quad \\mathrm{a}+\\mathrm{b}$ c $(\\mathrm{a}+\\mathrm{b}) \\mathrm{c} \\quad \\mathrm{ac}+\\mathrm{bc \\# \\#}(\\mathrm{a}+\\mathrm{b}) \\mathrm{c} \\quad(\\mathrm{a}+\\mathrm{b}) \\mathrm{c} \\# \\# \\mathrm{ac}+\\mathrm{bc}$ 相乘 $\\quad$ 相加 $\\quad(\\mathrm{a}+\\mathrm{b}) \\mathrm{c}=\\mathrm{ac}+\\mathrm{bc}$\n\n【分析】观察图示, 种西红柿和黄瓜的菜地, 长分别为 $\\mathrm{a}$ 和 $\\mathrm{b}$, 宽是 $\\mathrm{c}$, 根据长方形面积 $=$ 长 $\\times$ 宽进行解答; 两种方法求菜地总面积, 用 2 个长方形面积相加或看成一个长方形进行计算。方法不同, 但结果相同, 再通过两个结果的不同表达方式寻找规律即可。\n\n【详解】方法一:西红柿的面积是 $\\mathrm{ac}$, 黄瓜的面积是 $\\mathrm{bc}$, 所以这块菜地的面积是 $\\mathrm{ac}+\\mathrm{bc}$ 。方法二:菜地的面积是大长方形,它的长是 $\\mathrm{a}+\\mathrm{b}$, 宽是 $\\mathrm{c}$, 面积也可以这样计算: $(a+b) c$ 。综合方法一、二,菜地面积 $=\\mathrm{ac}+\\mathrm{bc}=(\\mathrm{a}+\\mathrm{b}) \\mathrm{c}$ 。\n\n通过不同的计算方法可发现,两个数的和与一个数相乘,可以先把它们与这个数分别相乘,再相加,这叫乘法分配律,用字母可以表示为 $(\\mathrm{a}+\\mathrm{b}) \\mathrm{c}=\\mathrm{ac}+\\mathrm{bc}$ 。\n\n【点睛】本题主要考查乘法分配律的应用, 便于学生熟练掌握。"} {"id": "28679", "image": [], "answer": "解$40 \\quad 60$\n\n【分析】等腰三角形的两个底角相等, 三角形的内角和为 $180^{\\circ}$, 因此用 $180^{\\circ}$ 减去 $100^{\\circ}$ 后, 再除以 2 ,即可得到其中一个底角的度数, 依此计算。\n\n三条边都相等的三角形是等边三角形, 等边三角形的三个角都相等, 因此用 $180^{\\circ}$ 除以 3 , 即可计算出它的一个内角度数, 依此计算。\n\n【详解】 $180^{\\circ}-100^{\\circ}=80^{\\circ}$\n\n$80^{\\circ} \\div 2=40^{\\circ}$\n\n$180^{\\circ} \\div 3=60^{\\circ}$\n\n即 $\\angle \\mathrm{B}=40^{\\circ}$; 一个三角形的三条边的长度都是 $6 \\mathrm{~cm}$, 它的一个内角是 $60^{\\circ}$ 。\n\n【点睛】解答此题的关键是要熟练掌握等边三角形、等腰三角形的特点, 以及熟记三角形的内角和度\n数。", "solution": "null", "level": "四年级", "question": "在等腰三角形 $\\mathrm{ABC}$ 中, 顶角 $\\angle \\mathrm{A}=100^{\\circ}$, 则 $\\angle \\mathrm{B}=(\\quad)^{\\circ}$; 一个三角形的三条边的长度都是 $6 \\mathrm{~cm}$, 它的一个内角是 $(\\quad)^{\\circ}$ 。", "options": [], "subject": "度量几何学", "analysis": "解$40 \\quad 60$\n\n【分析】等腰三角形的两个底角相等, 三角形的内角和为 $180^{\\circ}$, 因此用 $180^{\\circ}$ 减去 $100^{\\circ}$ 后, 再除以 2 ,即可得到其中一个底角的度数, 依此计算。\n\n三条边都相等的三角形是等边三角形, 等边三角形的三个角都相等, 因此用 $180^{\\circ}$ 除以 3 , 即可计算出它的一个内角度数, 依此计算。\n\n【详解】 $180^{\\circ}-100^{\\circ}=80^{\\circ}$\n\n$80^{\\circ} \\div 2=40^{\\circ}$\n\n$180^{\\circ} \\div 3=60^{\\circ}$\n\n即 $\\angle \\mathrm{B}=40^{\\circ}$; 一个三角形的三条边的长度都是 $6 \\mathrm{~cm}$, 它的一个内角是 $60^{\\circ}$ 。\n\n【点睛】解答此题的关键是要熟练掌握等边三角形、等腰三角形的特点, 以及熟记三角形的内角和度\n数。"} {"id": "28680", "image": [], "answer": "解钝角 等腰\n\n【分析】有一个角是钝角的三角形是钝角三角形, 三个角都是锐角的三角形是锐角三角形, 有一个角是直角的三角形是直角三角形; 两腰相等, 两个底角相等的三角形是等腰三角形; 三条边都相等, 三个角都相等的三角形是等边三角形,依此填空。\n\n【详解】根据三角形的分类标准可知, 少先队员佩戴的红领巾, 按角分属于钝角三角形, 按边分属于等腰三角形。\n\n【点睛】熟练掌握三角形的分类标准是解答此题的关键。", "solution": "null", "level": "四年级", "question": "“红领巾心向党, 争做新时代好少年。”少先队员佩戴的红领巾, 按角分属于 ( )三角形,按边分属于 ( )三角形。", "options": [], "subject": "度量几何学", "analysis": "解钝角 等腰\n\n【分析】有一个角是钝角的三角形是钝角三角形, 三个角都是锐角的三角形是锐角三角形, 有一个角是直角的三角形是直角三角形; 两腰相等, 两个底角相等的三角形是等腰三角形; 三条边都相等, 三个角都相等的三角形是等边三角形,依此填空。\n\n【详解】根据三角形的分类标准可知, 少先队员佩戴的红领巾, 按角分属于钝角三角形, 按边分属于等腰三角形。\n\n【点睛】熟练掌握三角形的分类标准是解答此题的关键。"} {"id": "28681", "image": ["13709.jpg"], "answer": "解20\n\n【分析】直角三角形中有一个角是 90 度 $(\\angle \\mathrm{B}=90$ 度), 三角形的内角和为 180 度, 因此用 180 度减去另外两个角的度数之和, 即可得到 $\\angle \\mathrm{C}$ 的度数。\n\n【详解】 $90+70=160$ (度)\n\n$180-160=20$ (度)\n\n即 $\\angle \\mathrm{C}=20$ 度。\n\n【点睛】解答此题的关键是要熟练掌握直角三角形的特点, 以及熟记三角形的内角和度数。", "solution": "null", "level": "四年级", "question": "下边的三角形中, $\\angle \\mathrm{A}=70$ 度, $\\angle \\mathrm{C}=(\\quad)$ 度。\n\n", "options": [], "subject": "度量几何学", "analysis": "解20\n\n【分析】直角三角形中有一个角是 90 度 $(\\angle \\mathrm{B}=90$ 度), 三角形的内角和为 180 度, 因此用 180 度减去另外两个角的度数之和, 即可得到 $\\angle \\mathrm{C}$ 的度数。\n\n【详解】 $90+70=160$ (度)\n\n$180-160=20$ (度)\n\n即 $\\angle \\mathrm{C}=20$ 度。\n\n【点睛】解答此题的关键是要熟练掌握直角三角形的特点, 以及熟记三角形的内角和度数。"} {"id": "28682", "image": ["13710.jpg"], "answer": "解$55 \\quad 35$\n\n【分析】根据三角形的内角和是 $180^{\\circ}$, 观察图可知, $\\angle 2=180^{\\circ}-90^{\\circ}-\\angle 1 ; \\angle 3$ 与 $\\angle 2$ 组成一个直角,所以, $\\angle 3=90^{\\circ}-\\angle 2$; 据此解题即可。\n\n【详解】 $\\angle 2=180^{\\circ}-90^{\\circ}-\\angle 1$\n\n$=180^{\\circ}-90^{\\circ}-35^{\\circ}$\n\n$=90^{\\circ}-35^{\\circ}$\n\n$=55^{\\circ}$\n\n$\\angle 3=90^{\\circ}-\\angle 2$\n\n$=90^{\\circ}-55^{\\circ}$\n\n$=35^{\\circ}$\n\n所以, $\\angle 2=55^{\\circ} 、 \\angle 3=35^{\\circ}$ 。\n\n【点睛】熟记三角形内角和是 $180^{\\circ}$ 、直角是 $90^{\\circ}$, 是解答此题的关键。", "solution": "null", "level": "四年级", "question": "下图, 已知 $\\angle 1=35^{\\circ}$, 那么 $\\angle 2=(\\quad)^{\\circ}, \\angle 3=(\\quad)^{\\circ}$ 。\n\n", "options": [], "subject": "度量几何学", "analysis": "解$55 \\quad 35$\n\n【分析】根据三角形的内角和是 $180^{\\circ}$, 观察图可知, $\\angle 2=180^{\\circ}-90^{\\circ}-\\angle 1 ; \\angle 3$ 与 $\\angle 2$ 组成一个直角,所以, $\\angle 3=90^{\\circ}-\\angle 2$; 据此解题即可。\n\n【详解】 $\\angle 2=180^{\\circ}-90^{\\circ}-\\angle 1$\n\n$=180^{\\circ}-90^{\\circ}-35^{\\circ}$\n\n$=90^{\\circ}-35^{\\circ}$\n\n$=55^{\\circ}$\n\n$\\angle 3=90^{\\circ}-\\angle 2$\n\n$=90^{\\circ}-55^{\\circ}$\n\n$=35^{\\circ}$\n\n所以, $\\angle 2=55^{\\circ} 、 \\angle 3=35^{\\circ}$ 。\n\n【点睛】熟记三角形内角和是 $180^{\\circ}$ 、直角是 $90^{\\circ}$, 是解答此题的关键。"} {"id": "28471", "image": [], "answer": "解$\\quad 0.45 \\quad 0.23 \\quad 8.2 \\quad 2450$\n\n【分析】根据 1 米 $=100$ 厘米, 1 千米 $=1000$ 米, 1 米 $=10$ 分米, 1 千克 $=1000$ 克, 进行换算即可。\n\n【详解】 45 厘米 $\\div 100=0.45$ 米; 230 米 $\\div 1000=0.23$ 千米\n\n82 分米 $\\div 10=8.2$ 米; 2.45 千克 $\\times 1000=2450$ 克\n\n【点睛】单位大变小乘进率, 单位小变大除以进率。", "solution": "null", "level": "四年级", "question": "在括号里填上合适的数。\n45 厘米 $=($\n)米 230 米 $=($\n)千米\n\n82 分米 $=(\\quad)$ 米 2.45 千克 $=(\\quad$ 克", "options": [], "subject": "度量几何学", "analysis": "解$\\quad 0.45 \\quad 0.23 \\quad 8.2 \\quad 2450$\n\n【分析】根据 1 米 $=100$ 厘米, 1 千米 $=1000$ 米, 1 米 $=10$ 分米, 1 千克 $=1000$ 克, 进行换算即可。\n\n【详解】 45 厘米 $\\div 100=0.45$ 米; 230 米 $\\div 1000=0.23$ 千米\n\n82 分米 $\\div 10=8.2$ 米; 2.45 千克 $\\times 1000=2450$ 克\n\n【点睛】单位大变小乘进率, 单位小变大除以进率。"} {"id": "28474", "image": [], "answer": "解$><=$\n\n【分析】(1) 1 米 $=10$ 分米, 依此将 0.5 米化成分米后再比较。\n\n(2)小数比较大小的方法: 先看整数部分, 整数部分大的, 这个小数就大; 整数部分相同, 就看十分位上的数, 十分位上的数大, 这个小数就大, 十分位上的数相同, 再比较百分位上的数, 百分位上的数大, 这个小数就大, 以此类推。\n\n(3) 两个数单位相同, 小数的末尾添上“ 0 ”或去掉“ 0 ”, 小数的大小不变, 依此比较。\n\n【详解】(1) 0.5 米 $=5$ 分米, 即 0.5 米 $>3$ 分米。\n\n(2) 0.78 与 0.8 , 整数部分都是 0, 十分位上的数 $7<8$, 即 $0.78<0.8$ 。\n\n(3) $2.60=2.6$, 即 2.60 元 $=2.6$ 元。\n\n【点睛】此题考查了小数的性质、小数的大小比较, 以及米与分米之间的换算、比较。", "solution": "null", "level": "四年级", "question": "比较大小。\n0.5 米 (\n)3 分米\n0.78(\n)0.8 2.60 元 (\n)2.6 元", "options": [], "subject": "度量几何学", "analysis": "解$><=$\n\n【分析】(1) 1 米 $=10$ 分米, 依此将 0.5 米化成分米后再比较。\n\n(2)小数比较大小的方法: 先看整数部分, 整数部分大的, 这个小数就大; 整数部分相同, 就看十分位上的数, 十分位上的数大, 这个小数就大, 十分位上的数相同, 再比较百分位上的数, 百分位上的数大, 这个小数就大, 以此类推。\n\n(3) 两个数单位相同, 小数的末尾添上“ 0 ”或去掉“ 0 ”, 小数的大小不变, 依此比较。\n\n【详解】(1) 0.5 米 $=5$ 分米, 即 0.5 米 $>3$ 分米。\n\n(2) 0.78 与 0.8 , 整数部分都是 0, 十分位上的数 $7<8$, 即 $0.78<0.8$ 。\n\n(3) $2.60=2.6$, 即 2.60 元 $=2.6$ 元。\n\n【点睛】此题考查了小数的性质、小数的大小比较, 以及米与分米之间的换算、比较。"} {"id": "14121", "image": [], "answer": "等腰 三条边正", "solution": "null", "level": "四年级", "question": "两条边相等的三角形叫做 ( ) 三角形, ( ) 相等的三角形叫做等边三角形, 等边三角形又叫做 $(\\quad)$ 三角形。", "options": [], "subject": "度量几何学", "analysis": ""} {"id": "14123", "image": [], "answer": "$\times$", "solution": "null", "level": "四年级", "question": "两个形状不同的三角形的内角和不相等。", "options": [], "subject": "度量几何学", "analysis": ""} {"id": "14124", "image": [], "answer": "$\times$", "solution": "null", "level": "四年级", "question": "有一个直角的三角形是直角三角形, 有一个锐角的三角形就是锐角三角形。", "options": [], "subject": "度量几何学", "analysis": ""} {"id": "14125", "image": [], "answer": "$\\sqrt{ }$", "solution": "null", "level": "四年级", "question": "等边三角形也是锐角三角形。", "options": [], "subject": "度量几何学", "analysis": ""} {"id": "14126", "image": [], "answer": "99 钝角", "solution": "null", "level": "四年级", "question": "一个等边三角形, 每个角的度数都 ( ), 都是 ( $\\quad$ 。。", "options": [], "subject": "度量几何学", "analysis": ""} {"id": "14127", "image": [], "answer": "80 或 65", "solution": "null", "level": "四年级", "question": "在一个三角形中, $\\angle 1=32^{\\circ}, \\angle 2=49^{\\circ}, \\angle 3=(\\quad)^{\\circ}$, 这是一个 ( ) 三角形。", "options": [], "subject": "度量几何学", "analysis": ""} {"id": "14128", "image": [], "answer": "5", "solution": "null", "level": "四年级", "question": "有一个角是 $60^{\\circ}$ 的等腰三角形, 这个等腰三角形一定是 ( ) 三角形。", "options": [], "subject": "度量几何学", "analysis": ""} {"id": "14147", "image": [], "answer": "答案:$\\sqrt{ }$", "solution": "null", "level": "四年级", "question": "用一个放大 10 倍的放大镜看一个 $30^{\\circ}$ 的角, 看到的角还是 $30^{\\circ}$ 。", "options": [], "subject": "度量几何学", "analysis": "答案:$\\sqrt{ }$"} {"id": "14155", "image": [], "answer": "答案:18", "solution": "null", "level": "四年级", "question": "丽丽想用三根长度都是整厘米数的小棒围三角形, 其中两根小棒的长度分别是 $8 \\mathrm{~cm}$ 和 $11 \\mathrm{~cm}$, 第三根小棒的长度最长是 $(\\quad) \\mathrm{cm}$ 。", "options": [], "subject": "度量几何学", "analysis": "答案:18"} {"id": "14156", "image": [], "answer": "答案:360", "solution": "null", "level": "四年级", "question": "$\\square$ 这个多边形的内角和是 ( )。", "options": [], "subject": "度量几何学", "analysis": "答案:360"} {"id": "14194", "image": [], "answer": "答案: \\times$", "solution": "null", "level": "四年级", "question": "身高 134 厘米的菲菲在平均水深 1 米的泳池站立时双脚可以完全接触池底。", "options": [], "subject": "度量几何学", "analysis": "答案: \\times$"} {"id": "14245", "image": [], "answer": "答案: 三条线段\n\n边", "solution": "null", "level": "四年级", "question": "由 ( ) 围成的图形叫做三角形, 任何一个三角形都有3条()、3个()和()个顶点。", "options": [], "subject": "度量几何学", "analysis": "答案: 三条线段\n\n边"} {"id": "14246", "image": [], "answer": "答案: $\\times$", "solution": "null", "level": "四年级", "question": "一个锐角三角形中最大的内角是 $59^{\\circ}$ 。", "options": [], "subject": "度量几何学", "analysis": "答案: $\\times$"} {"id": "14247", "image": [], "answer": "答案: $\\sqrt{ }$", "solution": "null", "level": "四年级", "question": "一个四边形中有 3 个角都是直角, 那么第 4 个角也是直角。", "options": [], "subject": "度量几何学", "analysis": "答案: $\\sqrt{ }$"} {"id": "14248", "image": [], "answer": "答案:$\\sqrt{ }$", "solution": "null", "level": "四年级", "question": "直角三角形可以是等腰三角形。", "options": [], "subject": "度量几何学", "analysis": "答案:$\\sqrt{ }$"} {"id": "14258", "image": [], "answer": "答案: 锐角", "solution": "null", "level": "四年级", "question": "在一个三角形的三个角中, 一个是 $50^{\\circ}$, 一个是 $80^{\\circ}$, 这个三角形既是 ( ) 三角形, 又是 $(\\quad)$ 三角形。", "options": [], "subject": "度量几何学", "analysis": "答案: 锐角"} {"id": "14106", "image": [], "answer": "答案: $\\times$", "solution": "null", "level": "四年级", "question": "三根分别长 $25 \\mathrm{~cm}, 25 \\mathrm{~cm}, 9 \\mathrm{~cm}$ 的小棒不能围成一个三角形。", "options": [], "subject": "立体几何学", "analysis": "答案: $\\times$"} {"id": "27450", "image": ["13325.jpg", "13326.jpg", "13326.jpg"], "answer": "解$3 \\quad 3$\n\n【分析】观察题图可知,从正面可以看到两行, 下面一行是 2 个正方形, 上面一行是 1 个正方形, 左对齐, 则可以看到 3 个正方形。若拿去一个小正方体, 从正面看到的形状不变, 可以拿去下面一层中的 1 个正方体,除了上面有一个正方形的那个正方体,其余 3 个正方体中任意一个被拿去都可以,即有 3 种不同的拿法。\n\n【详解】如图\n\n\n如图中的立体图形从正面看可以看到 3 个正方形。若拿去一个小正方体, 从正面看到的形状不变, 则有 3 种不同的拿法。\n\n【点睛】本题考查观察物体,旨在考查学生的空间想象和推理能力,可以亲自拿出几个正方体摆一摆,即可得出结论。", "solution": "null", "level": "四年级", "question": "如图中的立体图形从正面看可以看到( )个正方形。若拿去一个小正方体, 从正面看到的形状不变, 则有 ( )种不同的拿法。\n\n", "options": [], "subject": "立体几何学", "analysis": "解$3 \\quad 3$\n\n【分析】观察题图可知,从正面可以看到两行, 下面一行是 2 个正方形, 上面一行是 1 个正方形, 左对齐, 则可以看到 3 个正方形。若拿去一个小正方体, 从正面看到的形状不变, 可以拿去下面一层中的 1 个正方体,除了上面有一个正方形的那个正方体,其余 3 个正方体中任意一个被拿去都可以,即有 3 种不同的拿法。\n\n【详解】如图\n\n\n如图中的立体图形从正面看可以看到 3 个正方形。若拿去一个小正方体, 从正面看到的形状不变, 则有 3 种不同的拿法。\n\n【点睛】本题考查观察物体,旨在考查学生的空间想象和推理能力,可以亲自拿出几个正方体摆一摆,即可得出结论。"} {"id": "27451", "image": ["13327.jpg"], "answer": "解上\n\n【分析】左边是一个圆柱体和一个正方体, 仔细观察这两个立体图形可知, 从正面可以看到一个圆形和一个正方形; 从上面可以看到一个长方形和一个正方形, 从左面可以看到一个长方形, 据此解答。\n\n【详解】由分析得:\n\n立体图右边的图形是从上面看到的。\n【点睛】本题考查了从不同的方向观察物体, 需要学生有较强的空间想象和推理能力。", "solution": "null", "level": "四年级", "question": "立体图右边的图形是从( )面看到的。\n", "options": [], "subject": "立体几何学", "analysis": "解上\n\n【分析】左边是一个圆柱体和一个正方体, 仔细观察这两个立体图形可知, 从正面可以看到一个圆形和一个正方形; 从上面可以看到一个长方形和一个正方形, 从左面可以看到一个长方形, 据此解答。\n\n【详解】由分析得:\n\n立体图右边的图形是从上面看到的。\n【点睛】本题考查了从不同的方向观察物体, 需要学生有较强的空间想象和推理能力。"} {"id": "27467", "image": ["13375.jpg"], "answer": "解43\n\n【分析】观察图形可知:\n\n从前面看, 是 4 个正方形, 分两层 , 上边一层 1 个, 下边一层 3 个, 左边对齐;从左面看, 是 3 个正方形, 分两层, 上边一层 1 个, 下边一层 2 个, 左边对齐。\n\n【详解】从前面看, 是 4 个正方形, 所以是 4 个正方形是红色的;\n\n从左面看, 是 3 个正方形, 所以是 3 个正方形是蓝色的。\n\n【点睛】此题考查了从不同方向观察几何体,可以锻炼学生的空间想象力和抽象思维力。", "solution": "null", "level": "四年级", "question": "下图是由 6 个同样大的正方体摆成的物体。如果将它的前面涂上红色, 有 ( )个正方形是红色的; 如果将它的左面涂上蓝色, 有( )个正方形是蓝色的。\n\n", "options": [], "subject": "立体几何学", "analysis": "解43\n\n【分析】观察图形可知:\n\n从前面看, 是 4 个正方形, 分两层 , 上边一层 1 个, 下边一层 3 个, 左边对齐;从左面看, 是 3 个正方形, 分两层, 上边一层 1 个, 下边一层 2 个, 左边对齐。\n\n【详解】从前面看, 是 4 个正方形, 所以是 4 个正方形是红色的;\n\n从左面看, 是 3 个正方形, 所以是 3 个正方形是蓝色的。\n\n【点睛】此题考查了从不同方向观察几何体,可以锻炼学生的空间想象力和抽象思维力。"} {"id": "27468", "image": ["13376.jpg"], "answer": "解(1)(4)\n\n(2)(1)\n\n(3)(2)\n\n【分析】(1)(2)(3)根据所给图形,结合几何体从前面、上面和左面看到的形状选择即可。 (1)\n\n从前面看是图 A 的是(4)。\n\n(2)\n\n从上面看是图 B 的是(1)。\n\n(3)\n\n从左面看是图 C 的是(2)。\n\n【点睛】本题主要考查从不同方向观察物体和几何图形, 关键是培养学生的观察能力。", "solution": "null", "level": "四年级", "question": "根据图回答问题。(填序号)\n\n\n\n(1)从前面看是图 A 的是( )。\n\n(2)从上面看是图 B 的是( )。\n\n(3)从左面看是图 C 的是( )。", "options": [], "subject": "立体几何学", "analysis": "解(1)(4)\n\n(2)(1)\n\n(3)(2)\n\n【分析】(1)(2)(3)根据所给图形,结合几何体从前面、上面和左面看到的形状选择即可。 (1)\n\n从前面看是图 A 的是(4)。\n\n(2)\n\n从上面看是图 B 的是(1)。\n\n(3)\n\n从左面看是图 C 的是(2)。\n\n【点睛】本题主要考查从不同方向观察物体和几何图形, 关键是培养学生的观察能力。"} {"id": "14016", "image": [], "answer": "答案:35 【点拨】 $7 \\mathrm{~cm} 、 14 \\mathrm{~cm}$ 都可能是这个等腰三角形的腰长, 如果 $7 \\mathrm{~cm}$ 是腰长, 那么三条边长分别是 $7 \\mathrm{~cm}, 7 \\mathrm{~cm}, 14 \\mathrm{~cm}$, 此时无法围成一个三角形,所以腰长只能是 $14 \\mathrm{~cm}$ 。", "solution": "null", "level": "四年级", "question": "若一个等腰三角形的两条边长分别是 $7 \\mathrm{~cm}$ 和 $14 \\mathrm{~cm}$, 那么它的周长是 $(\\quad) \\mathrm{cm}$ 。", "options": [], "subject": "立体几何学", "analysis": "答案:35 【点拨】 $7 \\mathrm{~cm} 、 14 \\mathrm{~cm}$ 都可能是这个等腰三角形的腰长, 如果 $7 \\mathrm{~cm}$ 是腰长, 那么三条边长分别是 $7 \\mathrm{~cm}, 7 \\mathrm{~cm}, 14 \\mathrm{~cm}$, 此时无法围成一个三角形,所以腰长只能是 $14 \\mathrm{~cm}$ 。"} {"id": "14075", "image": [], "answer": "前 左", "solution": "null", "level": "四年级", "question": "如图, 从( )面和( )面看到的形状是完全相同的。 (填“前”“左”或“上”)", "options": [], "subject": "立体几何学", "analysis": "前 左"} {"id": "27449", "image": ["13323.jpg", "13324.jpg"], "answer": "解$6 \\quad 14$\n\n【分析】(1)要想组成图形的小正方体个数最少, 先把小正方体分 3 个、 2 个、 1 个坚直放在一起,再把 3 个坚直放置的小正方体与 2 个坚直放置的小正方体棱对棱的交错放置, 同理 2 个坚直放置的小正方体与那一个小正方体棱对棱的放置, 最少需要 $3+2+1=6$ 个小正方体。\n\n(2)要想组成图形的小正方体最多, 在第一层有 3 行 3 列, 可排列 9 个小正方体; 第二层有 2 行 2 列, 可排列 4 个小正方体; 第三层有 1 个小正方体, 放在最左侧角上, 这样最多有 $9+4+1=14$ 个小正方体。\n\n【详解】由分析得:\n\n组成这样的图形最少有 6 个小正方体, 最多有 14 个小正方体。\n\n【点睛】本题考查根据三视图确认几何体, 旨在考查学生的空间思维和想象能力。", "solution": "null", "level": "四年级", "question": "一些正方体堆放在一起, 从正面看是\n\n\n\n从左面看是\n\n\n\n组成这样的图形最少有\n\n( ) 个小正方体, 最多有( )个小正方体。", "options": [], "subject": "组合几何学", "analysis": "解$6 \\quad 14$\n\n【分析】(1)要想组成图形的小正方体个数最少, 先把小正方体分 3 个、 2 个、 1 个坚直放在一起,再把 3 个坚直放置的小正方体与 2 个坚直放置的小正方体棱对棱的交错放置, 同理 2 个坚直放置的小正方体与那一个小正方体棱对棱的放置, 最少需要 $3+2+1=6$ 个小正方体。\n\n(2)要想组成图形的小正方体最多, 在第一层有 3 行 3 列, 可排列 9 个小正方体; 第二层有 2 行 2 列, 可排列 4 个小正方体; 第三层有 1 个小正方体, 放在最左侧角上, 这样最多有 $9+4+1=14$ 个小正方体。\n\n【详解】由分析得:\n\n组成这样的图形最少有 6 个小正方体, 最多有 14 个小正方体。\n\n【点睛】本题考查根据三视图确认几何体, 旨在考查学生的空间思维和想象能力。"} {"id": "13977", "image": [], "answer": "答案: 平行四边 三角形 长方形 平行四边形", "solution": "null", "level": "四年级", "question": "用两个完全一样的普通三角形可以拼成一个( )形, 用两个完全一样的直角三角形可以拼成 ()$,(),($ 。", "options": [], "subject": "组合几何学", "analysis": "答案: 平行四边 三角形 长方形 平行四边形"} {"id": "27469", "image": ["13377.jpg", "13378.jpg", "13379.jpg", "13380.jpg", "13381.jpg", "13382.jpg", "13383.jpg", "13379.jpg", "13380.jpg", "13381.jpg", "13382.jpg", "13383.jpg"], "answer": "解上\n\n【分析】观察图形可知,图形从正面看是\n\n\n从右面看是\n\n\n从左面看是\n\n\n\n从上面看是\n\n\n\n; 据此解答即可。\n\n【详解】由分析得:\n图形\n\n\n\n是从上面看到的。\n\n【点睛】本题考查了从不同方向观察物体, 需要学生有较强的空间想象和推理能力。", "solution": "null", "level": "四年级", "question": "如图, 图形\n\n\n\n是从( )面看到的。\n\n", "options": [], "subject": "画法几何学", "analysis": "解上\n\n【分析】观察图形可知,图形从正面看是\n\n\n从右面看是\n\n\n从左面看是\n\n\n\n从上面看是\n\n\n\n; 据此解答即可。\n\n【详解】由分析得:\n图形\n\n\n\n是从上面看到的。\n\n【点睛】本题考查了从不同方向观察物体, 需要学生有较强的空间想象和推理能力。"} {"id": "14025", "image": [], "answer": "1.27", "solution": "null", "level": "四年级", "question": "龙龙和秘聪一起参加学校的跳高比赛。龙龙获得第一名, 成绩为", "options": [], "subject": "计数", "analysis": "1.27"} {"id": "14027", "image": [], "answer": "答案:97 \\quad", "solution": "null", "level": "四年级", "question": "同同参加学校“才艺达人”比赛, 五位评委的打分分别为 96 分、 100 分、 92 分、 98 分、 97 分, 去掉一个最低分, 去掉一个最高分, 她的平均成绩是 $(\\quad)$ 分。", "options": [], "subject": "计数", "analysis": "答案:97 \\quad"} {"id": "14040", "image": [], "answer": "答案:0.01 1240", "solution": "null", "level": "四年级", "question": "截止到 2022 年 7 月 6 日, 我国的“玉兔二号”月球车在月球背面累计行驶里程", "options": [], "subject": "计数", "analysis": "答案:"} {"id": "14063", "image": [], "answer": "8.92 $\\quad 9 \\quad", "solution": "null", "level": "四年级", "question": "中国 5G 应用引领全球, 预计到 2025 年, 中国 5G 连接总数将从 2021 年的", "options": [], "subject": "计数", "analysis": "8.92 $\\quad 9 \\quad"} {"id": "27579", "image": [], "answer": "解【解答】解: $395 \\approx 400$\n\n$412 \\approx 400$\n\n$402 \\approx 400$\n\n$400 \\times 30=12000$ (千克)\n\n答:这个批发部一个月(按 30 天计算)大约售出水果 12000 千克。\n\n故答案为: 12000 。", "solution": "null", "level": "四年级", "question": "(2021 秋・梅县区期末)一个水果批发部 3 天售出水果的重量分别是 395 千克、 412 千克、 402 千克。照这样的销售情况, 这个批发部一个月(按 30 天计算)大约售出水果 $\\qquad$千克。", "options": [], "subject": "计数", "analysis": "解【解答】解: $395 \\approx 400$\n\n$412 \\approx 400$\n\n$402 \\approx 400$\n\n$400 \\times 30=12000$ (千克)\n\n答:这个批发部一个月(按 30 天计算)大约售出水果 12000 千克。\n\n故答案为: 12000 。"} {"id": "28011", "image": [], "answer": "解$6.52 \\quad 0.625$\n\n【分析】小数比较大小的方法: 先看整数部分, 整数部分大的, 这个小数就大; 整数部分相同, 就看十分位上的数, 十分位上的数大, 这个小数就大, 十分位上的数相同, 再比较百分位上的数, 百分位上的数大, 这个小数就大, 以此类推。此题依此比较即可。\n\n【详解】在数据 $0.625 、 6.250 、 6.52 、 6.025$ 中, 整数部分 $6>0$, 即 0.625 最小。\n\n$6.250 、 6.52 、 6.025$ 中, 整数部分的数都是 6 , 十分位上的数 $5>2>0$, 即 6.52 最大。\n\n【点睛】熟练掌握多位小数的大小比较是解答此题的关键。", "solution": "null", "level": "四年级", "question": "在数据 $0.625 、 6.250 、 6.52 、 6.025$ 中, 最大的数是 (\n\n), 最小的数是( )。", "options": [], "subject": "计数", "analysis": "解$6.52 \\quad 0.625$\n\n【分析】小数比较大小的方法: 先看整数部分, 整数部分大的, 这个小数就大; 整数部分相同, 就看十分位上的数, 十分位上的数大, 这个小数就大, 十分位上的数相同, 再比较百分位上的数, 百分位上的数大, 这个小数就大, 以此类推。此题依此比较即可。\n\n【详解】在数据 $0.625 、 6.250 、 6.52 、 6.025$ 中, 整数部分 $6>0$, 即 0.625 最小。\n\n$6.250 、 6.52 、 6.025$ 中, 整数部分的数都是 6 , 十分位上的数 $5>2>0$, 即 6.52 最大。\n\n【点睛】熟练掌握多位小数的大小比较是解答此题的关键。"} {"id": "28083", "image": [], "answer": "解$29.5 \\quad 29$\n\n【分析】改写成用“亿”作单位的数, 就是在亿位数的右下角点上小数点, 然后把小数末尾的 0 去掉,再在数的后面写上“亿”字; 精确到十分位, 就是把百分位上的数进行四舍五入; 保留整数, 就是把十分位上的数进行四舍五入, 据此解答。\n\n【详解】 2948030760 改写为用“亿”为单位的数是 29.4803076 亿;\n\n29.4803076 亿的百分位是“8”, 要“五入”, 所以 29.4803076 亿精确到十分位约是 29.5 亿;\n\n29.4803076 亿的十分位是“4”, 要“四舍”, 所以 29.4803076 亿保留整数约是 29 亿。\n\n【点睛】本题主要考查学生对小数的改写和求近似数的掌握。注意改写和求近似数时, 要带计数单位。", "solution": "null", "level": "四年级", "question": "把 2948030760 改写成用“亿”为单位, 并用“四舍五入法”精确到十分位约是( )亿, 保留整数约是 $($ 亿。", "options": [], "subject": "计数", "analysis": "解$29.5 \\quad 29$\n\n【分析】改写成用“亿”作单位的数, 就是在亿位数的右下角点上小数点, 然后把小数末尾的 0 去掉,再在数的后面写上“亿”字; 精确到十分位, 就是把百分位上的数进行四舍五入; 保留整数, 就是把十分位上的数进行四舍五入, 据此解答。\n\n【详解】 2948030760 改写为用“亿”为单位的数是 29.4803076 亿;\n\n29.4803076 亿的百分位是“8”, 要“五入”, 所以 29.4803076 亿精确到十分位约是 29.5 亿;\n\n29.4803076 亿的十分位是“4”, 要“四舍”, 所以 29.4803076 亿保留整数约是 29 亿。\n\n【点睛】本题主要考查学生对小数的改写和求近似数的掌握。注意改写和求近似数时, 要带计数单位。"} {"id": "28603", "image": [], "answer": "解$12 \\quad 18$\n\n【分析】假设 30 枝全是百合花, 则可获利 $30 \\times 3=90$ (元), 比实际多 $90-72=18$ (元),一枝玫瑰花看作百合花增加 $3-2=1$ (元), 所以玫瑰花有 $18 \\div 1=18$ (枝), 百合花有 $30-18=12$ (枝),据此即可解答。\n\n【详解】 $(30 \\times 3-72) \\div(3-2)$\n\n$=(90-72) \\div 1$\n\n$=18$ (枝)\n\n$30-18=12$ (枝)\n\n【点睛】本题是鸡兔同笼问题, 可以用假设法来进行解答。", "solution": "null", "level": "四年级", "question": "星星花店卖出一枝百合花可获利 3 元, 卖出一枝玫瑰花可获利 2 元。该花店昨天卖出百合花和玫瑰花一共 30 枝,一共获利 72 元,该花店昨天卖出百合花( )枝,玫瑰花()枝。", "options": [], "subject": "计数", "analysis": "解$12 \\quad 18$\n\n【分析】假设 30 枝全是百合花, 则可获利 $30 \\times 3=90$ (元), 比实际多 $90-72=18$ (元),一枝玫瑰花看作百合花增加 $3-2=1$ (元), 所以玫瑰花有 $18 \\div 1=18$ (枝), 百合花有 $30-18=12$ (枝),据此即可解答。\n\n【详解】 $(30 \\times 3-72) \\div(3-2)$\n\n$=(90-72) \\div 1$\n\n$=18$ (枝)\n\n$30-18=12$ (枝)\n\n【点睛】本题是鸡兔同笼问题, 可以用假设法来进行解答。"} {"id": "28220", "image": [], "answer": "解小浩 小轩 小宇\n\n【分析】将他们每个人的成绩进行大小排序, 时间用得最少的人跑得最快; 小数比较大小的方法: 先看整数部分, 整数部分大的, 这个小数就大; 整数部分相同, 就看十分位上的数, 十分位上的数大,这个小数就大, 十分位上的数相同, 再比较百分位上的数, 百分位上的数大, 这个小数就大, 以此类推。此题依此比较即可。\n\n【详解】 9.25 秒、 8.87 秒、 8.5 秒、 10.02 秒中, 整数部分 $10>9>8$,\n\n8.87 秒、 8.5 秒中, 整数部分都是 8 , 十分位上的数 $8>5$, 即: 10.02 秒 $>9.25$ 秒 $>8.87$ 秒 $>8.5$ 秒;因此: 第一名: 小浩, 第二名: 小轩, 第三名: 小宇。\n\n【点睛】此题考查的是时间快慢的比较, 熟练掌握多位小数的大小比较是解答此题的关键。", "solution": "null", "level": "四年级", "question": "下面是几位同学 $50 \\mathrm{~m}$ 跑比赛的成绩:小宇 9.25 秒, 小轩 8.87 秒, 小浩 8.5 秒, 小扬 10.02 秒。请按成绩排出他们本次比赛的前三名(填姓名)。\n第一名:\n), 第二名: (\n),第三名:()。", "options": [], "subject": "计数", "analysis": "解小浩 小轩 小宇\n\n【分析】将他们每个人的成绩进行大小排序, 时间用得最少的人跑得最快; 小数比较大小的方法: 先看整数部分, 整数部分大的, 这个小数就大; 整数部分相同, 就看十分位上的数, 十分位上的数大,这个小数就大, 十分位上的数相同, 再比较百分位上的数, 百分位上的数大, 这个小数就大, 以此类推。此题依此比较即可。\n\n【详解】 9.25 秒、 8.87 秒、 8.5 秒、 10.02 秒中, 整数部分 $10>9>8$,\n\n8.87 秒、 8.5 秒中, 整数部分都是 8 , 十分位上的数 $8>5$, 即: 10.02 秒 $>9.25$ 秒 $>8.87$ 秒 $>8.5$ 秒;因此: 第一名: 小浩, 第二名: 小轩, 第三名: 小宇。\n\n【点睛】此题考查的是时间快慢的比较, 熟练掌握多位小数的大小比较是解答此题的关键。"} {"id": "27343", "image": [], "answer": "解$\\triangle$\n\n【分析】加数 + 加数 $=$ 和, 则加数 $=$ 和 - 另一个加数, 依此填空。\n\n【详解】已知○ $+\\triangle=\\square$, 则 $\\square \\circ=\\triangle$ 。\n【点睛】熟练掌握加、减法的意义和各部分之间的关系是解答此题的关键。", "solution": "null", "level": "四年级", "question": "已知 $\\circ+\\triangle=\\square$, 则 $\\square 一 \\circ=(\\quad)$ 。", "options": [], "subject": "代数", "analysis": "解$\\triangle$\n\n【分析】加数 + 加数 $=$ 和, 则加数 $=$ 和 - 另一个加数, 依此填空。\n\n【详解】已知○ $+\\triangle=\\square$, 则 $\\square \\circ=\\triangle$ 。\n【点睛】熟练掌握加、减法的意义和各部分之间的关系是解答此题的关键。"} {"id": "27495", "image": [], "answer": "解乘法分配 3600\n\n【分析】乘法分配律: $\\mathrm{a} \\times(\\mathrm{b}+\\mathrm{c})=\\mathrm{a} \\times \\mathrm{b}+\\mathrm{a} \\times \\mathrm{c}$ ,据此即可解答。\n\n【详解】 $\\square+\\triangle=100$\n\n$36 \\times \\square+36 \\times \\triangle$\n\n$=36 \\times(\\square+\\triangle)$\n\n$=36 \\times 100$\n\n$=3600$\n\n$17 \\times 425+3 \\times 425=(17+3) \\times 425$, 这里运用了乘法分配律; 如果 $\\square+\\triangle=100$, 那么 $36 \\times \\square+36 \\times \\triangle=$ 3600 。\n\n【点睛】本题主要考查学生对乘法分配律的掌握和灵活运用。", "solution": "null", "level": "四年级", "question": "$17 \\times 425+3 \\times 425=(17+3) \\times 425$, 这里运用了 $(\\quad)$ 律; 如果 $\\square+\\triangle=100$, 那么 $36 \\times \\square+36 \\times \\triangle$ $=(\\quad)$ 。", "options": [], "subject": "代数", "analysis": "解乘法分配 3600\n\n【分析】乘法分配律: $\\mathrm{a} \\times(\\mathrm{b}+\\mathrm{c})=\\mathrm{a} \\times \\mathrm{b}+\\mathrm{a} \\times \\mathrm{c}$ ,据此即可解答。\n\n【详解】 $\\square+\\triangle=100$\n\n$36 \\times \\square+36 \\times \\triangle$\n\n$=36 \\times(\\square+\\triangle)$\n\n$=36 \\times 100$\n\n$=3600$\n\n$17 \\times 425+3 \\times 425=(17+3) \\times 425$, 这里运用了乘法分配律; 如果 $\\square+\\triangle=100$, 那么 $36 \\times \\square+36 \\times \\triangle=$ 3600 。\n\n【点睛】本题主要考查学生对乘法分配律的掌握和灵活运用。"} {"id": "27553", "image": ["13512.jpg"], "answer": "解【解答】解:如果 $\\star \\times \\triangle=\\square$, 那么, $\\star=\\square \\div \\triangle, \\triangle=\\square \\div \\star$ (从 “ $\\star 、 \\triangle 、 \\square 、+、-、 \\times 、$\n\n”中选填);这样填的依据是乘与除的互逆关系(填文字)。\n\n故答案为: $\\square \\div \\triangle$; $\\square \\div \\star$; 。乘与除的互逆关系", "solution": "null", "level": "四年级", "question": "(2022 春・梁平区期末)如果 $\\star \\times \\triangle=\\square$, 那么, $\\star=$ $\\qquad$ , $\\triangle=$ $\\qquad$\n\n\n$+、-、 \\times 、 \\div$ 中选填); 这样填的依据是 $\\qquad$ (填文字)。", "options": [], "subject": "代数", "analysis": "解【解答】解:如果 $\\star \\times \\triangle=\\square$, 那么, $\\star=\\square \\div \\triangle, \\triangle=\\square \\div \\star$ (从 “ $\\star 、 \\triangle 、 \\square 、+、-、 \\times 、$\n\n”中选填);这样填的依据是乘与除的互逆关系(填文字)。\n\n故答案为: $\\square \\div \\triangle$; $\\square \\div \\star$; 。乘与除的互逆关系"} {"id": "14144", "image": [], "answer": "斜", "solution": "null", "level": "四年级", "question": "在直角三角形中, 最长的边是 ( $\\quad$ ) 边。", "options": [], "subject": "解析几何", "analysis": "斜"} {"id": "11022", "image": [], "answer": "千克 克 千克 克", "solution": "null", "level": "二年级", "question": "我们学过的质量单位有 ( ) 和 ( ), 在称一般的物品时, 通常用 ( ) 作单位, 在称比较轻的物品时, 常用 ( $\\quad$ ) 作单位。", "options": [], "subject": "算术", "analysis": "千克 克 千克 克"} {"id": "11023", "image": [], "answer": "$54 \\quad 40$", "solution": "null", "level": "二年级", "question": "比 18 千克多 36 千克是()千克, 65 千克比 25 千克多()千克。", "options": [], "subject": "算术", "analysis": "$54 \\quad 40$"} {"id": "11033", "image": [], "answer": "$1000 \\quad 1$", "solution": "null", "level": "二年级", "question": "一袋小米重 500 克, 2 袋小米重 ( ) 克, 合 ( ) 千克。", "options": [], "subject": "算术", "analysis": "$1000 \\quad 1$"} {"id": "11042", "image": [], "answer": "55", "solution": "null", "level": "二年级", "question": "3 千克鸡蛋 15 元钱, 1 千克鸡蛋要 ( ) 元, 25 元可以买()千克鸡蛋。", "options": [], "subject": "算术", "analysis": "55"} {"id": "11044", "image": ["2291.jpg", "2292.jpg", "2293.jpg", "2294.jpg"], "answer": "$>=<>$", "solution": "null", "level": "二年级", "question": "在O里填上 “>”、“<”或 “=”。\n\n\n\n\n\n$0 \\bigcirc 0$\n\n\n\n\n\n$\\otimes \\bigcirc \\otimes$", "options": [], "subject": "算术", "analysis": "$>=<>$"} {"id": "11045", "image": [], "answer": "千克 克 克 千克", "solution": "null", "level": "二年级", "question": "填上合适的质量单位。\n250\n19(", "options": [], "subject": "算术", "analysis": "千克 克 克 千克"} {"id": "11046", "image": [], "answer": "$1000 \\quad 6 \\quad 9$", "solution": "null", "level": "二年级", "question": "填上合适的数。\n1 千克 $=($\n) 克\n6000 克= $=($\n) 千克\n9000 克= $=($\n) 千克", "options": [], "subject": "算术", "analysis": "$1000 \\quad 6 \\quad 9$"} {"id": "11047", "image": [], "answer": "2 千克\n1 千克 500 克\n1400 克\n250 克\n80 克", "solution": "null", "level": "二年级", "question": "将下列质量按从大到小顺序排列。\n\n$$\n\\begin{gathered}\n80 \\text { 克 } \\quad 1400 \\text { 克 } \\quad 250 \\text { 克 } \\quad 2 \\text { 千克 } \\quad 1 \\text { 千克 } 500 \\text { 克 } \\\\\n(\\quad)>(\\quad)>(\\quad)>(\\quad)>(\\quad)\n\\end{gathered}\n$$", "options": [], "subject": "算术", "analysis": "2 千克\n1 千克 500 克\n1400 克\n250 克\n80 克"} {"id": "11048", "image": [], "answer": "4", "solution": "null", "level": "二年级", "question": "1 块香白重 250 克, 1 个苹果重 200 克, 5 个苹果的重量相当于 ( ) 块香㿝的重量。", "options": [], "subject": "算术", "analysis": "4"} {"id": "10269", "image": [], "answer": "$\\sqrt{ }$", "solution": "null", "level": "二年级", "question": "比 1 米短 40 厘米是 60 厘米。", "options": [], "subject": "算术", "analysis": "$\\sqrt{ }$"} {"id": "10272", "image": [], "answer": "$\\mathrm{x}$", "solution": "null", "level": "二年级", "question": "小明步行去上学, 他 1 分钟走了 50 厘米。", "options": [], "subject": "算术", "analysis": "$\\mathrm{x}$"} {"id": "10274", "image": [], "answer": "$\\sqrt{ }$", "solution": "null", "level": "二年级", "question": "两位数加两位数的和可能是三位数, 也可能是两位数。", "options": [], "subject": "算术", "analysis": "$\\sqrt{ }$"} {"id": "10297", "image": [], "answer": "$\\sqrt{ }$", "solution": "null", "level": "二年级", "question": "笔算加减法时, 都要把相同数位对齐, 从个位算起。", "options": [], "subject": "算术", "analysis": "$\\sqrt{ }$"} {"id": "10298", "image": [], "answer": "$\\mathrm{x}$", "solution": "null", "level": "二年级", "question": "两个数相加的和一定比它们的差大。", "options": [], "subject": "算术", "analysis": "$\\mathrm{x}$"} {"id": "10299", "image": [], "answer": "$\\mathrm{x}$", "solution": "null", "level": "二年级", "question": "明明有 2 张 10 元纸币, 买一套尺子用了 6 元, 还剩下 4 元。", "options": [], "subject": "算术", "analysis": "$\\mathrm{x}$"} {"id": "10300", "image": [], "answer": "$\\mathrm{x}$", "solution": "null", "level": "二年级", "question": "商店里进了一批货, 卖了 48 件后还剩 27 件, 则进了 65 件。", "options": [], "subject": "算术", "analysis": "$\\mathrm{x}$"} {"id": "10325", "image": [], "answer": "$\\sqrt{ }$", "solution": "null", "level": "二年级", "question": "比直角小的角是锐角。", "options": [], "subject": "算术", "analysis": "$\\sqrt{ }$"} {"id": "10330", "image": [], "answer": "$\\sqrt{ }$", "solution": "null", "level": "二年级", "question": "个 5 相加的和是 10 。", "options": [], "subject": "算术", "analysis": "$\\sqrt{ }$"} {"id": "10357", "image": [], "answer": "$\\mathrm{x}$", "solution": "null", "level": "二年级", "question": "$6+6+6+6+6+4$ 改写成乘法算式是 $6 \\times 6$ 。", "options": [], "subject": "算术", "analysis": "$\\mathrm{x}$"} {"id": "10358", "image": [], "answer": "$\\sqrt{ }$", "solution": "null", "level": "二年级", "question": "两个数的积不一定比它们的和大。", "options": [], "subject": "算术", "analysis": "$\\sqrt{ }$"} {"id": "10359", "image": [], "answer": "$\\mathrm{x}$", "solution": "null", "level": "二年级", "question": "两个因数都是 5 , 它们的积是 10 。", "options": [], "subject": "算术", "analysis": "$\\mathrm{x}$"} {"id": "10360", "image": [], "answer": "$\\sqrt{ }$", "solution": "null", "level": "二年级", "question": "两个数相乘, 积可能比乘数大。", "options": [], "subject": "算术", "analysis": "$\\sqrt{ }$"} {"id": "10362", "image": [], "answer": "$\\times$", "solution": "null", "level": "二年级", "question": "小明步行去上学, 他 1 分钟走了 50 厘米。", "options": [], "subject": "算术", "analysis": "$\\mathrm{x}$"} {"id": "10391", "image": [], "answer": "$\\sqrt{ }$", "solution": "null", "level": "二年级", "question": "两个数的积不一定比它们的和大。", "options": [], "subject": "算术", "analysis": "$\\sqrt{ }$"} {"id": "10394", "image": [], "answer": "$\\times$", "solution": "null", "level": "二年级", "question": "公共汽车上原有 43 人, 到站后下去了 18 人, 又上来 20 人。现在公共汽车上的人数比原来少了。", "options": [], "subject": "算术", "analysis": "$\\times$"} {"id": "10396", "image": [], "answer": "$\\mathrm{x}$", "solution": "null", "level": "二年级", "question": "莉莉有两种不同的笔, 一种 3 支, 另一种 7 支, 一共 21 支笔。()", "options": [], "subject": "算术", "analysis": "$\\mathrm{x}$"} {"id": "10422", "image": [], "answer": "$\\mathrm{x}$", "solution": "null", "level": "二年级", "question": "每一句乘法口诀都能写出两道乘法算式。", "options": [], "subject": "算术", "analysis": "$\\mathrm{x}$"} {"id": "10423", "image": [], "answer": "$\\sqrt{ }$", "solution": "null", "level": "二年级", "question": "$4 \\times 9=6 \\times 6$ 。", "options": [], "subject": "算术", "analysis": "$\\sqrt{ }$"} {"id": "10424", "image": [], "answer": "$\\mathrm{x}$", "solution": "null", "level": "二年级", "question": "求 5 个 8 相加是多少? 列式是 $5+8$ 。", "options": [], "subject": "算术", "analysis": "$\\mathrm{x}$"} {"id": "10425", "image": [], "answer": "$\\mathrm{x}$", "solution": "null", "level": "二年级", "question": "( ) $\\times 7<49$, 括号里最大能填 7 。", "options": [], "subject": "算术", "analysis": "$\\mathrm{x}$"} {"id": "10451", "image": [], "answer": "$\\sqrt{ }$", "solution": "null", "level": "二年级", "question": "5 分也可以说成是一刻。", "options": [], "subject": "算术", "analysis": "$\\sqrt{ }$"} {"id": "10454", "image": [], "answer": "$\\mathrm{x}$", "solution": "null", "level": "二年级", "question": "分针走 1 小格是 5 分钟。", "options": [], "subject": "算术", "analysis": "$\\mathrm{x}$"} {"id": "11439", "image": ["2398.jpg"], "answer": "3 \\times 5=15, \\quad 5 \\times 3=15$,\n\n$15 \\div 3=5,15 \\div 5=3$\n\n| 被除数 | 10 | 15 | 24 | 24 | 8 | 12 | 30 |\n| :---: | :---: | :---: | :---: | :---: | :---: | :---: | :---: |\n| 除数 | 2 | 3 | 4 | 8 | 2 | 3 | 5 |\n| 商 | 5 | 5 | 6 | 3 | 4 | 4 | 6 |\n\n$\\left\\{\\begin{array}{l}4 \\times 6=24 \\\\ 24 \\div 4=6 \\\\ 24 \\div 6=4\\end{array} \\quad\\left\\{\\begin{array}{l}5 \\times 4=20 \\\\ 20 \\div 4=5 \\\\ 20 \\div 5=4\\end{array} \\quad\\left\\{\\begin{array}{l}5 \\times 3=15 \\\\ 15 \\div 5=3 \\\\ 15 \\div 3=5\\end{array}\\right.\\right.\\right.$", "solution": "null", "level": "二年级", "question": "\n\n乘法算式: $(\\quad) \\mathrm{X}(\\quad)=(\\quad),(\\quad) \\mathrm{X}(\\quad)=(\\quad)$\n\n除法算式: $(\\quad) \\div(\\quad)=(\\quad),(\\quad) \\div(\\quad)=(\\quad)$", "options": [], "subject": "算术", "analysis": "3 \\times 5=15, \\quad 5 \\times 3=15$,\n\n$15 \\div 3=5,15 \\div 5=3$\n\n| 被除数 | 10 | 15 | 24 | 24 | 8 | 12 | 30 |\n| :---: | :---: | :---: | :---: | :---: | :---: | :---: | :---: |\n| 除数 | 2 | 3 | 4 | 8 | 2 | 3 | 5 |\n| 商 | 5 | 5 | 6 | 3 | 4 | 4 | 6 |\n\n$\\left\\{\\begin{array}{l}4 \\times 6=24 \\\\ 24 \\div 4=6 \\\\ 24 \\div 6=4\\end{array} \\quad\\left\\{\\begin{array}{l}5 \\times 4=20 \\\\ 20 \\div 4=5 \\\\ 20 \\div 5=4\\end{array} \\quad\\left\\{\\begin{array}{l}5 \\times 3=15 \\\\ 15 \\div 5=3 \\\\ 15 \\div 3=5\\end{array}\\right.\\right.\\right.$"} {"id": "11451", "image": [], "answer": "30 \\div 5=6$\n\n被除数 除号 除数 商", "solution": "null", "level": "二年级", "question": "30 \\div 5=6$\n\n( ) ( ) ( ) ( )", "options": [], "subject": "算术", "analysis": "30 \\div 5=6$\n\n被除数 除号 除数 商"} {"id": "11452", "image": ["2402.jpg"], "answer": "20 \\div 5=4 \\quad 20 \\div 4=5$\n\n$12 \\div 4=3 \\quad 12 \\div 3=4$\n\n$18 \\div 3=6 \\quad 18 \\div 6=3$", "solution": "null", "level": "二年级", "question": "下面卡片上的数摆成学过的除法算式。\n\n\n$\\square \\div \\square=\\square$\n$\\square \\div \\square=\\square$\n$\\square \\div \\square=\\square$\n$\\square \\div \\square=\\square$\n$\\square \\div \\square=\\square$\n$\\square \\div \\square=\\square$", "options": [], "subject": "算术", "analysis": "20 \\div 5=4 \\quad 20 \\div 4=5$\n\n$12 \\div 4=3 \\quad 12 \\div 3=4$\n\n$18 \\div 3=6 \\quad 18 \\div 6=3$"} {"id": "11453", "image": ["2403.jpg"], "answer": "6 \\div 2=3$ 2, 3", "solution": "null", "level": "二年级", "question": "\n\n( )只分为一组, 可以分成这样的 ( )组。", "options": [], "subject": "算术", "analysis": "6 \\div 2=3$ 2, 3"} {"id": "11454", "image": [], "answer": "6,3", "solution": "null", "level": "二年级", "question": "果 $\\square+\\square+※+※+※=21$,\n\n$\\square+\\square+※+※+※+※+※=27$\n\n那么, $\\square=(\\quad), \\mathbb{N}=(\\quad)$", "options": [], "subject": "算术", "analysis": "6,3"} {"id": "10466", "image": [], "answer": "被除数 除数 商", "solution": "null", "level": "二年级", "question": "在 $63 \\div 9=7$ 中, 63 是 (\n), 9 是 (\n), 7 是 ( $\\quad$ 。", "options": [], "subject": "算术", "analysis": "被除数 除数 商"} {"id": "10467", "image": [], "answer": "$(1) 63$\n\n(2) 9\n\n【分析】(1) 3 天读了 27 页, 则平均每天读 $27 \\div 3=9$ 页。用平均每天读书页数乘 9 , 求出 7 天读书页数。\n\n(2) 用全书总页数除以平均每天读书页数, 求出读完这本书需要的天数。\n\n(1)\n\n$27 \\div 3=9$ (页)\n\n$7 \\times 9=63$ (页)\n\n照这种速度, 7 天可以读 63 页。\n\n(2)\n\n$81 \\div 9=9$ (天)\n\n全书 81 页, 9 天读完。\n\n【点睛】本题考查工程问题和归一问题, 关键是明确归一问题的解题方法, 应先求单一量, 再根据题意用乘法或除法解答。", "solution": "null", "level": "二年级", "question": "小方读一本故事书, 3 天读了 27 页。\n\n(1)照这种速度, 7 天可以读( )页。\n\n(2)全书 81 页, $(\\quad)$ 天读完。", "options": [], "subject": "算术", "analysis": "$(1) 63$\n\n(2) 9\n\n【分析】(1) 3 天读了 27 页, 则平均每天读 $27 \\div 3=9$ 页。用平均每天读书页数乘 9 , 求出 7 天读书页数。\n\n(2) 用全书总页数除以平均每天读书页数, 求出读完这本书需要的天数。\n\n(1)\n\n$27 \\div 3=9$ (页)\n\n$7 \\times 9=63$ (页)\n\n照这种速度, 7 天可以读 63 页。\n\n(2)\n\n$81 \\div 9=9$ (天)\n\n全书 81 页, 9 天读完。\n\n【点睛】本题考查工程问题和归一问题, 关键是明确归一问题的解题方法, 应先求单一量, 再根据题意用乘法或除法解答。"} {"id": "10468", "image": [], "answer": "$><>>=<$\n\n【详解】略", "solution": "null", "level": "二年级", "question": "在括号里填上“ $>\" “<$ ”或“=”。\n$33+16($\n) $24+19$\n$0 \\times 7($\n) $7+0$\n25(\n) $5 \\times 2$\n\n2 米 $\\quad) 20$ 厘米 60 厘米 +40 厘米 $\\quad$ )1 米 $36 \\div 9(6 \\times 1$", "options": [], "subject": "算术", "analysis": "$><>>=<$\n\n【详解】略"} {"id": "10470", "image": [], "answer": "34", "solution": "null", "level": "二年级", "question": "24 块蛋糕, 每 8 块装一盒, 可以装(\n\n)盒; 如果每 6 块装一盒, 可以装(\n\n)盒。", "options": [], "subject": "算术", "analysis": "34"} {"id": "10488", "image": [], "answer": "4 四八三十二 84\n\n【详解】略", "solution": "null", "level": "二年级", "question": "$32 \\div 8=(\\quad)$, 想乘法口诀 $(\\quad)$ 求商, 这个算式表示 32 里面有 $(\\quad)$ 个 $(\\quad)$ 。", "options": [], "subject": "算术", "analysis": "4 四八三十二 84\n\n【详解】略"} {"id": "10489", "image": [], "answer": "$\\quad 7 \\quad 1$\n\n【详解】略", "solution": "null", "level": "二年级", "question": "$\\triangle+\\circ=8, \\triangle-\\circ=6, \\triangle \\times 0=7, \\triangle \\div o=7, \\triangle=(\\quad), \\circ=(\\quad)$ 。", "options": [], "subject": "算术", "analysis": "$\\quad 7 \\quad 1$\n\n【详解】略"} {"id": "10490", "image": [], "answer": "9", "solution": "null", "level": "二年级", "question": "妈妈把 18 元钱平均分给她的 2 个孩子,每个孩子分到( )元。", "options": [], "subject": "算术", "analysis": "9"} {"id": "10492", "image": ["2146.jpg"], "answer": "36 除以 4 等于 $9 \\quad 36 \\quad 4 \\quad 9 \\quad 4 \\quad 9 \\quad 9 \\quad 4$\n\n【分析】根据除法算式的各部分名称, 读出这个算式, 再根据平均除法与包含除法的意义解答此题。\n\n【详解】 $36 \\div 4=9$ 这个算式读作: 36 除以 4 等于 9 。被除数是 36 , 除数是 4 , 商是 9 。把 36 平均分成 4 份, 每份是 9 。还可以表示 36 里有 9 个 4 。\n\n【点睛】熟练掌握除法算式的读写法, 再根据平均除法与除法的意义完成填空是此题的关键。", "solution": "null", "level": "二年级", "question": "$36 \\div 4=9$ 这个算式读作 (\n\n)。其中被除数是 (\n\n) , 除数是 (\n\n\n\n) , 商是 (\n\n表示把 36 平均分成 ( ) 份。每份是 (\n\n)。还可以表示 36 里面有 (\n\n) 个 (\n\n)。", "options": [], "subject": "算术", "analysis": "36 除以 4 等于 $9 \\quad 36 \\quad 4 \\quad 9 \\quad 4 \\quad 9 \\quad 9 \\quad 4$\n\n【分析】根据除法算式的各部分名称, 读出这个算式, 再根据平均除法与包含除法的意义解答此题。\n\n【详解】 $36 \\div 4=9$ 这个算式读作: 36 除以 4 等于 9 。被除数是 36 , 除数是 4 , 商是 9 。把 36 平均分成 4 份, 每份是 9 。还可以表示 36 里有 9 个 4 。\n\n【点睛】熟练掌握除法算式的读写法, 再根据平均除法与除法的意义完成填空是此题的关键。"} {"id": "10493", "image": [], "answer": "$6 \\quad 6$\n\n【分析】求 30 里面有几个 5 , 用除法解答; 根据平均分的意义, 用除法即可求出把 45 平均分成 9 份,每份是多少。\n\n【详解】 $30 \\div 5=6$\n\n$45 \\div 9=5$\n\n【点睛】本题考查除法, 明确除法的意义是解题的关键。", "solution": "null", "level": "二年级", "question": "30 里面有 ( )个 5 , 把 45 平均分成 9 份,每份是 $(\\quad)$ 。", "options": [], "subject": "算术", "analysis": "$6 \\quad 6$\n\n【分析】求 30 里面有几个 5 , 用除法解答; 根据平均分的意义, 用除法即可求出把 45 平均分成 9 份,每份是多少。\n\n【详解】 $30 \\div 5=6$\n\n$45 \\div 9=5$\n\n【点睛】本题考查除法, 明确除法的意义是解题的关键。"} {"id": "10494", "image": [], "answer": "$7 \\quad 1$\n\n【分析】用促销的总价除以总个数即可求出现价, 用原价减去现价即可求出现价比原来便宜的价格。\n\n【详解】 $21 \\div 3=7$ (元)\n\n现在优惠促销, 3 个 21 元。现在 1 个鸡腿 7 元\n\n$8-7=1 ($ 元 $)$\n\n鸡腿原来 8 元一个, 现在优惠促销, 3 个 21 元。比原来便宜 1 元。\n\n【点睛】本题主要考查了根据 7 的乘法口诀求商的实际应用。", "solution": "null", "level": "二年级", "question": "鸡腿原来 8 元一个, 现在优惠促销, 3 个 21 元。现在 1 个鸡腿( )元,比原来便宜 $(\\quad)$\n元。", "options": [], "subject": "算术", "analysis": "$7 \\quad 1$\n\n【分析】用促销的总价除以总个数即可求出现价, 用原价减去现价即可求出现价比原来便宜的价格。\n\n【详解】 $21 \\div 3=7$ (元)\n\n现在优惠促销, 3 个 21 元。现在 1 个鸡腿 7 元\n\n$8-7=1 ($ 元 $)$\n\n鸡腿原来 8 元一个, 现在优惠促销, 3 个 21 元。比原来便宜 1 元。\n\n【点睛】本题主要考查了根据 7 的乘法口诀求商的实际应用。"} {"id": "10495", "image": [], "answer": "9\n\n【分析】根据平均分的意义,用 36 除以 4 即可解答。\n\n【详解】 $36 \\div 4=9$ (段)\n\n【点睛】本题考查平均分, 明确平均分用除法是解题的关键。", "solution": "null", "level": "二年级", "question": "一根 36 米的绳子每 4 米分成一段, 可以分成( )段。", "options": [], "subject": "算术", "analysis": "9\n\n【分析】根据平均分的意义,用 36 除以 4 即可解答。\n\n【详解】 $36 \\div 4=9$ (段)\n\n【点睛】本题考查平均分, 明确平均分用除法是解题的关键。"} {"id": "10496", "image": [], "answer": "$\\quad 6 \\div 2=3 \\quad 18 \\div 6=3 \\quad 27 \\div 9=3 \\quad 12 \\div 4=3$\n\n【分析】根据表内乘法口诀, 进行解答即可。(答案不唯一)\n【详解】商是 3 的四个算式: $6 \\div 2=3 、 18 \\div 6=3 、 27 \\div 9=3 、 12 \\div 4=3$ 。\n\n【点睛】本题考查表内除法的计算。注意计算的准确性。", "solution": "null", "level": "二年级", "question": "写出商是 3 的四个算式: \\$ \\qquad \\$ \\$ \\qquad \\$ \\$ \\qquad \\$ \\$ \\qquad \\$ .", "options": [], "subject": "算术", "analysis": "$\\quad 6 \\div 2=3 \\quad 18 \\div 6=3 \\quad 27 \\div 9=3 \\quad 12 \\div 4=3$\n\n【分析】根据表内乘法口诀, 进行解答即可。(答案不唯一)\n【详解】商是 3 的四个算式: $6 \\div 2=3 、 18 \\div 6=3 、 27 \\div 9=3 、 12 \\div 4=3$ 。\n\n【点睛】本题考查表内除法的计算。注意计算的准确性。"} {"id": "10514", "image": [], "answer": "六九五十四\n\n【分析】分别计算出两个算式得数可以看出这两个算式用了同一句乘法口诀。\n\n【详解】 $54 \\div 9=6$ 和 $54 \\div 6=9$ 都可以用乘法口决(六九五十四)计算。\n\n【点睛】本题主要考查了两个因数不同的乘法口诀可以写出两道除法算式。", "solution": "null", "level": "二年级", "question": "$54 \\div 9$ 和 $54 \\div 6$ 都可以用乘法口决( )计算。", "options": [], "subject": "算术", "analysis": "六九五十四\n\n【分析】分别计算出两个算式得数可以看出这两个算式用了同一句乘法口诀。\n\n【详解】 $54 \\div 9=6$ 和 $54 \\div 6=9$ 都可以用乘法口决(六九五十四)计算。\n\n【点睛】本题主要考查了两个因数不同的乘法口诀可以写出两道除法算式。"} {"id": "10515", "image": [], "answer": "$<=>$\n\n【分析】分别计算出括号左右两边算式的得数, 然后比较大小即可。\n\n【详解】 $35 \\div 7=5,35 \\div 5=7,35 \\div 7<35 \\div 5$\n\n$63 \\div 9=7,56 \\div 8=7,63 \\div 9=56 \\div 8$\n\n$24 \\div 3=8,24 \\div 4=6,24 \\div 3>24 \\div 4$\n\n【点睛】本题主要考查了用表内乘法口诀求商, 能够正确的计算出各个算式的结果。", "solution": "null", "level": "二年级", "question": "在括号里填上“>”“<”或“=”。\n\n$35 \\div 7(\\quad) 35 \\div 5 \\quad 63 \\div 9(\\quad) 56 \\div 8 \\quad 24 \\div 3(\\quad) 24 \\div 4$", "options": [], "subject": "算术", "analysis": "$<=>$\n\n【分析】分别计算出括号左右两边算式的得数, 然后比较大小即可。\n\n【详解】 $35 \\div 7=5,35 \\div 5=7,35 \\div 7<35 \\div 5$\n\n$63 \\div 9=7,56 \\div 8=7,63 \\div 9=56 \\div 8$\n\n$24 \\div 3=8,24 \\div 4=6,24 \\div 3>24 \\div 4$\n\n【点睛】本题主要考查了用表内乘法口诀求商, 能够正确的计算出各个算式的结果。"} {"id": "10518", "image": [], "answer": "$35 \\quad 9$", "solution": "null", "level": "二年级", "question": "一个星期有 7 天, 5 个星期有 ( ) 天; 63 天是 ( ) 个星期。", "options": [], "subject": "算术", "analysis": "$35 \\quad 9$"} {"id": "10519", "image": [], "answer": "2", "solution": "null", "level": "二年级", "question": "鸡腿原来 9 元一个, 现在优惠促销, 3 个 21 元。现在每个鸡腿比原来便宜 ( ) 元。", "options": [], "subject": "算术", "analysis": "2"} {"id": "10520", "image": [], "answer": "$6 \\times 9=54 \\# \\# 9 \\times 6=54 \\quad 54 \\div 6=9 \\# \\# 54 \\div 9=6$\n\n【详解】略", "solution": "null", "level": "二年级", "question": "根据口诀“六九五十四”写出一道乘法算式和一道除法算式: (\n\n)、( $\\quad$ )。", "options": [], "subject": "算术", "analysis": "$6 \\times 9=54 \\# \\# 9 \\times 6=54 \\quad 54 \\div 6=9 \\# \\# 54 \\div 9=6$\n\n【详解】略"} {"id": "10521", "image": [], "answer": "9\n\n【解析】略", "solution": "null", "level": "二年级", "question": "一道除法算式的被除数是 27 , 除数是 3 , 则商是 ( )。", "options": [], "subject": "算术", "analysis": "9\n\n【解析】略"} {"id": "10540", "image": [], "answer": "428 除以 7 等于 4 四七二十八", "solution": "null", "level": "二年级", "question": "$28 \\div 7=(\\quad)$, 读作 $\\quad$ ), 可以用口诀 ( )计算。", "options": [], "subject": "算术", "analysis": "428 除以 7 等于 4 四七二十八"} {"id": "10541", "image": [], "answer": "7", "solution": "null", "level": "二年级", "question": "28 个〇平均分成 4 份, 每份是 ( ) 个。", "options": [], "subject": "算术", "analysis": "7"} {"id": "10542", "image": [], "answer": "86", "solution": "null", "level": "二年级", "question": "从 64 里连续减去 $\\quad$ )个 8 , 结果为 $0 ; 54$ 里面有 ( ) 个 9 。", "options": [], "subject": "算术", "analysis": "86"} {"id": "10544", "image": [], "answer": "$\\quad 9 \\quad 1$\n\n【分析】把 36 元平均分成 4 份, 每份多少元就是促销面包每个的价钱, 用除法; 用面包原来的价钱和现在每个面包的价钱相减, 可以求出每个比原来便宜多少元, 据此解答。\n\n【详解】 $36 \\div 4=9$ (元), 知道促销面包每个 9 元;\n\n$10-9=1$ (元), 知道每个比原来便宜 1 元。\n\n【点睛】 掌握用 9 的乘法口诀求商是解答此题的关键。", "solution": "null", "level": "二年级", "question": "面包原来 10 元一个, 现在一次买 4 个 36 元, 促销面包每个 ( )元, 每个比原来便宜 ( )元。", "options": [], "subject": "算术", "analysis": "$\\quad 9 \\quad 1$\n\n【分析】把 36 元平均分成 4 份, 每份多少元就是促销面包每个的价钱, 用除法; 用面包原来的价钱和现在每个面包的价钱相减, 可以求出每个比原来便宜多少元, 据此解答。\n\n【详解】 $36 \\div 4=9$ (元), 知道促销面包每个 9 元;\n\n$10-9=1$ (元), 知道每个比原来便宜 1 元。\n\n【点睛】 掌握用 9 的乘法口诀求商是解答此题的关键。"} {"id": "10545", "image": [], "answer": "9\n\n【分析】根据平均分的意义, 用 36 除以 4 即可解答。\n\n【详解】 $36 \\div 4=9$ (段)\n\n【点睛】本题考查平均分, 明确平均分用除法是解题的关键。", "solution": "null", "level": "二年级", "question": "一根 36 米的绳子每 4 米分成一段, 可以分成()段。", "options": [], "subject": "算术", "analysis": "9\n\n【分析】根据平均分的意义, 用 36 除以 4 即可解答。\n\n【详解】 $36 \\div 4=9$ (段)\n\n【点睛】本题考查平均分, 明确平均分用除法是解题的关键。"} {"id": "10546", "image": [], "answer": "$64 \\quad 9 \\quad 8$\n\n【分析】思考每个除法算式所对应的乘法口诀,依次补充空缺的数字。\n\n【详解】 $32 \\div 4=8 \\quad 27 \\div 3=9 \\quad 8 \\times 8=64$\n\n| 被除数 | 32 | 27 | 64 |\n| :---: | :---: | :---: | :---: |\n| 除数 | 4 | 9 | 8 |\n| 商 | 8 | 3 | 8 |\n\n【点睛】本题考查表内除法的计算, 熟练掌握利用乘法口诀计算表内除法。", "solution": "null", "level": "二年级", "question": "填表。\n\n| 被除数 | 32 | 27 | $(\\quad)$ |\n| :---: | :---: | :---: | :---: |\n| 除数 | 4 | $(\\quad)$ | 8 |\n| 商 | $(\\quad)$ | 3 | 8 |", "options": [], "subject": "算术", "analysis": "$64 \\quad 9 \\quad 8$\n\n【分析】思考每个除法算式所对应的乘法口诀,依次补充空缺的数字。\n\n【详解】 $32 \\div 4=8 \\quad 27 \\div 3=9 \\quad 8 \\times 8=64$\n\n| 被除数 | 32 | 27 | 64 |\n| :---: | :---: | :---: | :---: |\n| 除数 | 4 | 9 | 8 |\n| 商 | 8 | 3 | 8 |\n\n【点睛】本题考查表内除法的计算, 熟练掌握利用乘法口诀计算表内除法。"} {"id": "10547", "image": [], "answer": "【分析】根据 100 以内的加减法以及表内乘、除法填空即可。\n\n【详解】 $9 \\times 8=72$\n\n$5+5=10$\n\n$36 \\div 6=6$\n\n$28-4=24$\n\n【点睛】熟练掌握整数四则计算的方法是解题关键。", "solution": "null", "level": "二年级", "question": "在括号里填上“+”“一”“×”或“$\\cdot$”。\n\n$$\n9(\\quad) 8=72 \\quad 5(\\quad) 5=10 \\quad 36(\\quad) 6=6 \\quad 28(\\quad) 4=24\n$$", "options": [], "subject": "算术", "analysis": "【分析】根据 100 以内的加减法以及表内乘、除法填空即可。\n\n【详解】 $9 \\times 8=72$\n\n$5+5=10$\n\n$36 \\div 6=6$\n\n$28-4=24$\n\n【点睛】熟练掌握整数四则计算的方法是解题关键。"} {"id": "10548", "image": [], "answer": "$6 \\quad 6$\n\n【分析】把 24 人按每份 4 人分组,可分几组,即 24 里面有几个 4 ,用除法计算; 把 24 人平均分成 3 组, 即把 24 平均分成 3 份, 每份是多少,用除法计算。据此解答。\n\n【详解】 $24 \\div 4=6$ (个) $; 24 \\div 3=8$ (人)。\n\n所以 24 个同学做游戏。如果每 4 人分成一组, 可以分成( 6 ) 个组; 如果把 24 个同学平均分成 3 个组,每组分( 8 ) 人。\n\n【点睛】本题考查平均分及表内除法的应用, 解题的关键是: 把总人数按每份 4 人分几组或按 3 组平均分几人都用除法计算。", "solution": "null", "level": "二年级", "question": "24 个同学做游戏。如果每 4 人分成一组, 可以分成(\n\n)个组;如果把 24 个同学平均分成 3 个组, 每组分 $(\\quad)$ 人。", "options": [], "subject": "算术", "analysis": "$6 \\quad 6$\n\n【分析】把 24 人按每份 4 人分组,可分几组,即 24 里面有几个 4 ,用除法计算; 把 24 人平均分成 3 组, 即把 24 平均分成 3 份, 每份是多少,用除法计算。据此解答。\n\n【详解】 $24 \\div 4=6$ (个) $; 24 \\div 3=8$ (人)。\n\n所以 24 个同学做游戏。如果每 4 人分成一组, 可以分成( 6 ) 个组; 如果把 24 个同学平均分成 3 个组,每组分( 8 ) 人。\n\n【点睛】本题考查平均分及表内除法的应用, 解题的关键是: 把总人数按每份 4 人分几组或按 3 组平均分几人都用除法计算。"} {"id": "10568", "image": [], "answer": "$\\quad 1 \\quad 64$\n\n【分析】根据题目意思, 列出算式计算即可。\n\n【详解】 $9 \\div 9=1$\n\n$8 \\times 8=64$\n\n【点睛】本题主要考查利用表内乘法口诀求商、求积, 注意计算要仔细认真。", "solution": "null", "level": "二年级", "question": "被除数和除数都是 9 , 商是 $(\\quad)$ 。两个 8 相乘, 积是 ( )。", "options": [], "subject": "算术", "analysis": "$\\quad 1 \\quad 64$\n\n【分析】根据题目意思, 列出算式计算即可。\n\n【详解】 $9 \\div 9=1$\n\n$8 \\times 8=64$\n\n【点睛】本题主要考查利用表内乘法口诀求商、求积, 注意计算要仔细认真。"} {"id": "10569", "image": [], "answer": "$\\quad=><$\n\n【分析】先算出得数, 在比较大小, 据此解答。\n\n【详解】 $48 \\div 8=6,54 \\div 9=6,6=6$, 故 $48 \\div 8=4 \\div 9$;\n\n$36 \\div 6=6,2+3=5,6>5$, 故 $36 \\div 6>2+3$;\n\n$3+6 \\times 5$\n\n$=3+30$\n\n$=33,33<45$, 故 $3+6 \\times 5<45$\n\n【点睛】本题考查表内除法及大小的比较。", "solution": "null", "level": "二年级", "question": "在括号里填上“>”“<”或“=”。 $48 \\div 8(\\quad) 54 \\div 9 \\quad 36 \\div 6(\\quad) 2+3 \\quad 3+6 \\times 5(\\quad) 45$", "options": [], "subject": "算术", "analysis": "$\\quad=><$\n\n【分析】先算出得数, 在比较大小, 据此解答。\n\n【详解】 $48 \\div 8=6,54 \\div 9=6,6=6$, 故 $48 \\div 8=4 \\div 9$;\n\n$36 \\div 6=6,2+3=5,6>5$, 故 $36 \\div 6>2+3$;\n\n$3+6 \\times 5$\n\n$=3+30$\n\n$=33,33<45$, 故 $3+6 \\times 5<45$\n\n【点睛】本题考查表内除法及大小的比较。"} {"id": "10570", "image": [], "answer": "36 除以 4 等于 $9 \\quad 36 \\quad 4 \\quad 9 \\quad 4 \\quad 9$ 四九三十六 $36 \\div 9=4$\n\n【分析】 根据平均分的意义以及用乘法口诀求商的方法进行解答即可。\n\n【详解】 $36 \\div 4=9$ 读作: 36 除以 4 等于 9 , 可以表示把 36 平均分成 4 , 每份是 9 ; 也可以表示 36 里面有 4 个 9 。计算时用到的乘法口诀是四九三十六, 根据这个乘法口诀还可以写出另一个除法算式是 $36 \\div 9$ $=4$ 。\n\n【点睛】熟练掌握乘法的意义以及乘法和除法的关系是解题关键。", "solution": "null", "level": "二年级", "question": "$36 \\div 4=9$ 读作: ( ), 可以表示把 $(\\quad)$ 平均分成( ), 每份是 $\\quad$ ); 也可以表示 36 里面有 ( ) 个( )。计算时用到的乘法口诀是( ), 根据这个乘法口诀还可以写出另一个除法算式是 $(\\quad)$ 。", "options": [], "subject": "算术", "analysis": "36 除以 4 等于 $9 \\quad 36 \\quad 4 \\quad 9 \\quad 4 \\quad 9$ 四九三十六 $36 \\div 9=4$\n\n【分析】 根据平均分的意义以及用乘法口诀求商的方法进行解答即可。\n\n【详解】 $36 \\div 4=9$ 读作: 36 除以 4 等于 9 , 可以表示把 36 平均分成 4 , 每份是 9 ; 也可以表示 36 里面有 4 个 9 。计算时用到的乘法口诀是四九三十六, 根据这个乘法口诀还可以写出另一个除法算式是 $36 \\div 9$ $=4$ 。\n\n【点睛】熟练掌握乘法的意义以及乘法和除法的关系是解题关键。"} {"id": "10572", "image": [], "answer": "$5 \\quad 32 \\quad 64$\n\n【分析】先根据乘法口诀求出等号左边算式的结果也就是等号右边算式的商, 再根据乘法口诀填空即可。\n\n【详解】 $56 \\div 8=7=35 \\div 5$\n\n$48 \\div 6=8=32 \\div 4$\n\n$4 \\times 2=8=64 \\div 8$\n\n【点睛】解答本题关键是熟练掌握乘法口诀正确进行计算。", "solution": "null", "level": "二年级", "question": "填一填。\n\n$56 \\div 8=35 \\div(\\quad) \\quad 48 \\div 6=(\\quad) \\div 4 \\quad 4 \\times 2=(\\quad) \\div 8$", "options": [], "subject": "算术", "analysis": "$5 \\quad 32 \\quad 64$\n\n【分析】先根据乘法口诀求出等号左边算式的结果也就是等号右边算式的商, 再根据乘法口诀填空即可。\n\n【详解】 $56 \\div 8=7=35 \\div 5$\n\n$48 \\div 6=8=32 \\div 4$\n\n$4 \\times 2=8=64 \\div 8$\n\n【点睛】解答本题关键是熟练掌握乘法口诀正确进行计算。"} {"id": "10573", "image": [], "answer": "$9 \\quad 6$\n\n【分析】有 36 个气球, 挂 4 行, 求每行挂几个, 用除法计算; 如果每行挂 6 个, 求可以挂几行, 用除法计算。\n\n【详解】 $36 \\div 6=9$ (个)\n\n$36 \\div 6=6($ 行 $)$\n\n【点睛】本题主要考查表内除法的计算。", "solution": "null", "level": "二年级", "question": "有 36 个气球, 挂 4 行, 平均每行挂 ( )个; 如果每行挂 6 个, 可以挂 ( )行。", "options": [], "subject": "算术", "analysis": "$9 \\quad 6$\n\n【分析】有 36 个气球, 挂 4 行, 求每行挂几个, 用除法计算; 如果每行挂 6 个, 求可以挂几行, 用除法计算。\n\n【详解】 $36 \\div 6=9$ (个)\n\n$36 \\div 6=6($ 行 $)$\n\n【点睛】本题主要考查表内除法的计算。"} {"id": "10574", "image": [], "answer": "$56 \\div 8=7$\n\n【解析】略", "solution": "null", "level": "二年级", "question": "被除数是 56 , 除数是 8 ,商是 7 。算式是 (", "options": [], "subject": "算术", "analysis": "$56 \\div 8=7$\n\n【解析】略"} {"id": "10575", "image": [], "answer": "$6 \\quad 6 \\div 3=6$\n\n【解析】略", "solution": "null", "level": "二年级", "question": "18 里面连续减去 $\\quad$ ) 个 3 , 结果等于 0 。列式为 $(\\quad)$ 。", "options": [], "subject": "算术", "analysis": "$6 \\quad 6 \\div 3=6$\n\n【解析】略"} {"id": "10576", "image": [], "answer": "8\n\n【解析】略", "solution": "null", "level": "二年级", "question": "现在有 32 个鸡蛋, 要把这 32 个鸡蛋放到 4 个篮子里, 每个篮子里可以放( ) 个鸡蛋。", "options": [], "subject": "算术", "analysis": "8\n\n【解析】略"} {"id": "10594", "image": [], "answer": "三九二十七\n\n【分析】根据与 9 有关的乘法口诀进行解答即可。\n\n【详解】由分析得:计算 $27 \\div 3$ 时, 用到的乘法口决是三九二十七。\n\n【点睛】此题的关键是要熟练掌握乘法口诀。", "solution": "null", "level": "二年级", "question": "计算 $27 \\div 3$ 时, 用到的乘法口决是 ( )。", "options": [], "subject": "算术", "analysis": "三九二十七\n\n【分析】根据与 9 有关的乘法口诀进行解答即可。\n\n【详解】由分析得:计算 $27 \\div 3$ 时, 用到的乘法口决是三九二十七。\n\n【点睛】此题的关键是要熟练掌握乘法口诀。"} {"id": "10595", "image": [], "answer": "36 除以 4 等于 $9 \\quad 36 \\quad 4 \\quad 9 \\quad 4 \\quad 9 \\quad 9 \\quad 4$\n\n【分析】根据除法算式的各部分名称, 读出这个算式, 再根据平均除法与包含除法的意义解答此题。\n\n【详解】 $36 \\div 4=9$ 这个算式读作: 36 除以 4 等于 9 。被除数是 36 , 除数是 4 , 商是 9 。把 36 平均分成 4 份, 每份是 9 。还可以表示 36 里有 9 个 4 。\n\n【点睛】熟练掌握除法算式的读写法, 再根据平均除法与除法的意义完成填空是此题的关键。", "solution": "null", "level": "二年级", "question": "$36 \\div 4=9$ 这个算式读作 (\n )。其中被除数是(\n ) , 除数是 (\n ) , 商是 (\n 表示把 36 平均分成 () 份。每份是 (\n )。还可以表示 36 里面有 (\n ) 个(", "options": [], "subject": "算术", "analysis": "36 除以 4 等于 $9 \\quad 36 \\quad 4 \\quad 9 \\quad 4 \\quad 9 \\quad 9 \\quad 4$\n\n【分析】根据除法算式的各部分名称, 读出这个算式, 再根据平均除法与包含除法的意义解答此题。\n\n【详解】 $36 \\div 4=9$ 这个算式读作: 36 除以 4 等于 9 。被除数是 36 , 除数是 4 , 商是 9 。把 36 平均分成 4 份, 每份是 9 。还可以表示 36 里有 9 个 4 。\n\n【点睛】熟练掌握除法算式的读写法, 再根据平均除法与除法的意义完成填空是此题的关键。"} {"id": "10596", "image": [], "answer": "$28 \\div 4=7 \\quad 28 \\div 7=4$\n\n【分析】先根据“四七二十八”可知乘法算式: $4 \\times 7=28$, 用积除以其中的一个因数就会得到另一个因数。\n\n【详解】根据“四七二十八”这句口诀写出两个除法算式是 $28 \\div 4=7$ 和 $28 \\div 7=4$ 。\n\n【点睛】本题考查了运用乘法口诀计算表内乘除法, 注意要熟记口诀。", "solution": "null", "level": "二年级", "question": "用口诀“四七二十八”写出两个除法算式是( )和( )。", "options": [], "subject": "算术", "analysis": "$28 \\div 4=7 \\quad 28 \\div 7=4$\n\n【分析】先根据“四七二十八”可知乘法算式: $4 \\times 7=28$, 用积除以其中的一个因数就会得到另一个因数。\n\n【详解】根据“四七二十八”这句口诀写出两个除法算式是 $28 \\div 4=7$ 和 $28 \\div 7=4$ 。\n\n【点睛】本题考查了运用乘法口诀计算表内乘除法, 注意要熟记口诀。"} {"id": "10598", "image": [], "answer": "$6 \\quad 3$", "solution": "null", "level": "二年级", "question": "学校买了 24 个足球, 如果把这些足球平均分给 4 个队, 那么每队可以分到( )。如果每队分 8 个足球,那么可以分给( $\\quad$ )队。", "options": [], "subject": "算术", "analysis": "$6 \\quad 3$"} {"id": "10599", "image": [], "answer": "6", "solution": "null", "level": "二年级", "question": "有 48 个苹果 ,每 8 个装一袋, 可以装 $(\\quad)$ 袋。", "options": [], "subject": "算术", "analysis": "6"} {"id": "10600", "image": [], "answer": "$7 \\quad 4$\n\n【分析】根据题意, 把一根 32 米长的绳子, 平均截成 8 段, 截的次数比平均截成的段 8 少 1 次, 即 7 次; 要求每段是多少米, 用绳子的长度 32 米除以截成的段数 8 即可。\n\n【详解】根据题意可得:\n\n截的次数是: $8-1=7$ (次)\n\n每段的长度是: $32 \\div 8=4$ (米)\n\n所以需要截 ( 7 ) 次, 每段是 ( 4 ) 米。\n\n【点睛】截绳子问题, 截的次数比截成的段数少 1 , 然后再根据题意进一步解答即可。", "solution": "null", "level": "二年级", "question": "一根 32 米长的绳子, 平均截成 8 段, 需要截 ) 次,每段是 $(\\quad)$ 米。", "options": [], "subject": "算术", "analysis": "$7 \\quad 4$\n\n【分析】根据题意, 把一根 32 米长的绳子, 平均截成 8 段, 截的次数比平均截成的段 8 少 1 次, 即 7 次; 要求每段是多少米, 用绳子的长度 32 米除以截成的段数 8 即可。\n\n【详解】根据题意可得:\n\n截的次数是: $8-1=7$ (次)\n\n每段的长度是: $32 \\div 8=4$ (米)\n\n所以需要截 ( 7 ) 次, 每段是 ( 4 ) 米。\n\n【点睛】截绳子问题, 截的次数比截成的段数少 1 , 然后再根据题意进一步解答即可。"} {"id": "10601", "image": [], "answer": "356", "solution": "null", "level": "二年级", "question": "一个乘数是 5 , 另一个乘数是 7 , 积是 $(\\quad)$; 被除数是 42 , 除数是 7 , 商是 $(\\quad)$ 。", "options": [], "subject": "算术", "analysis": "356"} {"id": "10602", "image": [], "answer": "$35 \\quad 6$", "solution": "null", "level": "二年级", "question": "猜一猜, $\\triangle \\div 5=7, \\triangle$ 代表( ); $42 \\div \\square=7$, $\\square$ 表示 $(\\quad)$ 。", "options": [], "subject": "算术", "analysis": "$35 \\quad 6$"} {"id": "10620", "image": [], "answer": "$6 \\times 8=48 \\quad 48 \\div 6=8$\n\n【分析】口诀“六八四十八”中, 6 和 8 是两个因数, 积是 48 , 由此写出两个乘法算式, 再根据乘法和除法的互逆关系, 写出两道除法算式。\n\n【详解】 口诀“六八四十八”可以写成的乘法算式是: $6 \\times 8=48$ 或 $8 \\times 6=48$可以写成的除法算式是: $48 \\div 6=8$ 或 $48 \\div 8=6$\n\n【点睛】本题考查了乘除法最基础的内容乘法口诀, 要熟记口诀, 并能根据口诀写出乘法和除法算式。", "solution": "null", "level": "二年级", "question": "根据口块“六八四十八”可以写出两道算式为( )或( )。", "options": [], "subject": "算术", "analysis": "$6 \\times 8=48 \\quad 48 \\div 6=8$\n\n【分析】口诀“六八四十八”中, 6 和 8 是两个因数, 积是 48 , 由此写出两个乘法算式, 再根据乘法和除法的互逆关系, 写出两道除法算式。\n\n【详解】 口诀“六八四十八”可以写成的乘法算式是: $6 \\times 8=48$ 或 $8 \\times 6=48$可以写成的除法算式是: $48 \\div 6=8$ 或 $48 \\div 8=6$\n\n【点睛】本题考查了乘除法最基础的内容乘法口诀, 要熟记口诀, 并能根据口诀写出乘法和除法算式。"} {"id": "10623", "image": [], "answer": "$\\quad 3 \\times 7=21 \\quad 21 \\div 3=7$\n\n【分析】根据三七二十一这句乘法口诀可以写出两道乘法算式和两道除法算式。\n\n【详解】三七二十一\n$3 \\times 7=21$\n\n$7 \\times 3=21$\n\n$21 \\div 3=7$\n\n$21 \\div 7=3$\n\n【点睛】本题主要考查了一句乘法口诀可以写出乘法算式和除法算式。", "solution": "null", "level": "二年级", "question": "根据乘法口块“三七二十一”写出一个乘法算式是(\n\n), 再写出一个除法算式是 $(\\quad)$ 。", "options": [], "subject": "算术", "analysis": "$\\quad 3 \\times 7=21 \\quad 21 \\div 3=7$\n\n【分析】根据三七二十一这句乘法口诀可以写出两道乘法算式和两道除法算式。\n\n【详解】三七二十一\n$3 \\times 7=21$\n\n$7 \\times 3=21$\n\n$21 \\div 3=7$\n\n$21 \\div 7=3$\n\n【点睛】本题主要考查了一句乘法口诀可以写出乘法算式和除法算式。"} {"id": "10624", "image": [], "answer": "5\n\n【分析】根据“被除数 $\\div$ 除数 $=$ 商”, 再根据 8 的乘法口诀即可得出答案。\n\n【详解】 $40 \\div 8=5$, 所以商是 5 。\n\n【点睛】本题考查学生对用 8 的乘法口诀求商的掌握和运用。", "solution": "null", "level": "二年级", "question": "被除数是 40 ,除数是 8 ,商是 $(\\quad)$ 。", "options": [], "subject": "算术", "analysis": "5\n\n【分析】根据“被除数 $\\div$ 除数 $=$ 商”, 再根据 8 的乘法口诀即可得出答案。\n\n【详解】 $40 \\div 8=5$, 所以商是 5 。\n\n【点睛】本题考查学生对用 8 的乘法口诀求商的掌握和运用。"} {"id": "10625", "image": [], "answer": "$<=<>$\n\n【分析】计算出左边或右边式子的结果, 再进行比较即可。\n\n【详解】 $10 \\div 2=5,5<10$, 则 $10 \\div 2<10 ; 28 \\div 7=4$, 则 $4=28 \\div 7$;\n\n$3+3=6,3 \\times 3=9,6<9$, 则 $3+3<3 \\times 3 ; 48 \\div 6 \\div 2=4,4>3$, 则 $48 \\div 6 \\div 2>3$\n\n【点睛】此题的关键是要灵活运用乘法口诀, 并掌握 20 以内的数的大小比较。", "solution": "null", "level": "二年级", "question": "在 ( ) 里填上“ $>” “>”$ 或“ $=”$ 。\n\n$10 \\div 2(\\quad) 104(\\quad) 28 \\div 7 \\quad 3+3(\\quad) 3 \\times 3 \\quad 48 \\div 6 \\div 2(\\quad) 3$", "options": [], "subject": "算术", "analysis": "$<=<>$\n\n【分析】计算出左边或右边式子的结果, 再进行比较即可。\n\n【详解】 $10 \\div 2=5,5<10$, 则 $10 \\div 2<10 ; 28 \\div 7=4$, 则 $4=28 \\div 7$;\n\n$3+3=6,3 \\times 3=9,6<9$, 则 $3+3<3 \\times 3 ; 48 \\div 6 \\div 2=4,4>3$, 则 $48 \\div 6 \\div 2>3$\n\n【点睛】此题的关键是要灵活运用乘法口诀, 并掌握 20 以内的数的大小比较。"} {"id": "10626", "image": [], "answer": "$\\begin{array}{lllll}8 & 7 & 36 & 8 & 2\\end{array}$\n\n【分析】 根据 $8 、 6 、 4$ 的乘法口决来解题即可。\n\n【详解】 (1) 三八二十四, 所以 $3 \\times 8=24$;\n\n(2) 七八五十六, 所以 $56 \\div 7=8$;\n\n$(3)$ 六六三十六, 所以 $36 \\div 6=6$;\n\n(4) 二四得八, 所以 $8 \\div 2=4$ (答案不唯一)。\n\n【点睛】掌握表内乘、除法是解题的关键。", "solution": "null", "level": "二年级", "question": "在 $()$ 里填上适当的数。\n\n$3 \\times(\\quad)=24 \\quad 56 \\div(\\quad)=8 \\quad(\\quad) \\div 6=6 \\quad(\\quad) \\div(\\quad)=4$", "options": [], "subject": "算术", "analysis": "$\\begin{array}{lllll}8 & 7 & 36 & 8 & 2\\end{array}$\n\n【分析】 根据 $8 、 6 、 4$ 的乘法口决来解题即可。\n\n【详解】 (1) 三八二十四, 所以 $3 \\times 8=24$;\n\n(2) 七八五十六, 所以 $56 \\div 7=8$;\n\n$(3)$ 六六三十六, 所以 $36 \\div 6=6$;\n\n(4) 二四得八, 所以 $8 \\div 2=4$ (答案不唯一)。\n\n【点睛】掌握表内乘、除法是解题的关键。"} {"id": "10627", "image": ["2185.jpg"], "answer": "$\\quad 21 \\quad 7 \\quad 3 \\quad 21 \\quad 3 \\quad 7 \\quad 21 \\div 7=3$\n\n【分析】数清楚星星的总数和圆圈中星星的数量, 再根据除法的意义进行解答即可。\n\n【详解】有 21 个凸, 按每 7 个分一份, 可以分成 3 份, 也就是 21 里面有 3 个 7 ; 可以列式为: $21 \\div 7$ $=3$ 。\n\n【点睛】本题考查学生对表内除法以及除法意义的掌握和运用。", "solution": "null", "level": "二年级", "question": "\n\n有\n\n) 个出, 按每 (\n\n)个分一份,可以分成(\n\n) 份,也就是 (\n\n)里面有\n\n( )个( ) 算式: ( )。", "options": [], "subject": "算术", "analysis": "$\\quad 21 \\quad 7 \\quad 3 \\quad 21 \\quad 3 \\quad 7 \\quad 21 \\div 7=3$\n\n【分析】数清楚星星的总数和圆圈中星星的数量, 再根据除法的意义进行解答即可。\n\n【详解】有 21 个凸, 按每 7 个分一份, 可以分成 3 份, 也就是 21 里面有 3 个 7 ; 可以列式为: $21 \\div 7$ $=3$ 。\n\n【点睛】本题考查学生对表内除法以及除法意义的掌握和运用。"} {"id": "10645", "image": [], "answer": "7\n\n【分析】 35 里面有几个 5 就可以买几块这样的手帕, 用除法解答即可。\n\n【详解】 $35 \\div 5=7$ (块)\n\n【点睛】本题考查除法, 明确除法的意义是解题的关键。", "solution": "null", "level": "二年级", "question": "一块手帕 5 元, 35 元钱可以买 ()块这样的手帕。", "options": [], "subject": "算术", "analysis": "7\n\n【分析】 35 里面有几个 5 就可以买几块这样的手帕, 用除法解答即可。\n\n【详解】 $35 \\div 5=7$ (块)\n\n【点睛】本题考查除法, 明确除法的意义是解题的关键。"} {"id": "10646", "image": [], "answer": "9 七九六十三 $7 \\times 9=63 \\quad 9 \\times 7=63 \\quad 63 \\div 9=7$\n\n【分析】算除法和乘法时, 我们一般运用乘法口诀进行计算。\n\n【详解】 $63 \\div 7=9$, 想口决: 七九六十三, 用这句口决还可以算的算式有 $7 \\times 9=63,9 \\times 7=63,63 \\div 9$ $=7$ 。\n\n【点睛】本题考查除法, 明确根据乘法口诀算除法是解题的关键。", "solution": "null", "level": "二年级", "question": "$63 \\div 7=(\\quad)$, 想口诀: ( ), 用这句口决还可以算的算式有", "options": [], "subject": "算术", "analysis": "9 七九六十三 $7 \\times 9=63 \\quad 9 \\times 7=63 \\quad 63 \\div 9=7$\n\n【分析】算除法和乘法时, 我们一般运用乘法口诀进行计算。\n\n【详解】 $63 \\div 7=9$, 想口决: 七九六十三, 用这句口决还可以算的算式有 $7 \\times 9=63,9 \\times 7=63,63 \\div 9$ $=7$ 。\n\n【点睛】本题考查除法, 明确根据乘法口诀算除法是解题的关键。"} {"id": "10670", "image": [], "answer": "$\\begin{array}{llll}10 & 8 & 7\\end{array}$\n\n【详解】略", "solution": "null", "level": "二年级", "question": "在括号里填上合适的数。\n$(\\quad)+4 \\times 7=38$\n$64 \\div 8=($\n$2 \\times($\n)$=14$", "options": [], "subject": "算术", "analysis": "$\\begin{array}{llll}10 & 8 & 7\\end{array}$\n\n【详解】略"} {"id": "10671", "image": [], "answer": "$\\quad 12 \\quad 20 \\quad 8$\n\n【分析】一张桌子有 4 条腿, 求 3 张桌子和 5 张桌子分别有多少条腿, 用乘法计算; 32 条腿有多少张桌子, 用除法计算。\n\n【详解】 $3 \\times 4=12$ (条)\n\n$5 \\times 4=20$ (条)\n\n$32 \\div 4=8$ (张)\n\n【点睛】此题考查表内乘除法的简单应用。", "solution": "null", "level": "二年级", "question": "一张桌子 4 条腿, 3 张桌子 (\n\n)条腿, 5 张桌子 (\n\n)条腿\n\n)张桌子 32\n\n条腿。", "options": [], "subject": "算术", "analysis": "$\\quad 12 \\quad 20 \\quad 8$\n\n【分析】一张桌子有 4 条腿, 求 3 张桌子和 5 张桌子分别有多少条腿, 用乘法计算; 32 条腿有多少张桌子, 用除法计算。\n\n【详解】 $3 \\times 4=12$ (条)\n\n$5 \\times 4=20$ (条)\n\n$32 \\div 4=8$ (张)\n\n【点睛】此题考查表内乘除法的简单应用。"} {"id": "10672", "image": [], "answer": "$48 \\quad 6 \\quad 8$\n\n【详解】略", "solution": "null", "level": "二年级", "question": "二(2)班有男生 25 人,女生 23 人,共有学生( )人,如果每组 8 人,可分成(\n组, 如果每组 6 人, 可分成( )组。", "options": [], "subject": "算术", "analysis": "$48 \\quad 6 \\quad 8$\n\n【详解】略"} {"id": "10673", "image": [], "answer": "34\n\n【详解】略", "solution": "null", "level": "二年级", "question": "24 块蛋糕, 每 8 块装一盒, 可以装( )盒; 如果每 6 块装一盒, 可以装( )盒。", "options": [], "subject": "算术", "analysis": "34\n\n【详解】略"} {"id": "10674", "image": [], "answer": "$5 \\quad 35 \\div 7=5$\n\n【详解】略", "solution": "null", "level": "二年级", "question": "35 天有 ( )个星期。算式是( )。", "options": [], "subject": "算术", "analysis": "$5 \\quad 35 \\div 7=5$\n\n【详解】略"} {"id": "10693", "image": [], "answer": "$<><>=>$", "solution": "null", "level": "二年级", "question": "在括号里填上“>”“<”或“=”。\n(1) $5 \\mathrm{~cm}(\\quad) 5 \\mathrm{~m}$\n(2) $45 \\div 5(\\quad) 45 \\div 9$\n(3) $4 \\times 7(\\quad) 5 \\times 6$\n(4) $1 \\mathrm{~m}(\\quad) 99 \\mathrm{~cm}$\n(5) $18 \\div 3(\\quad) 2 \\times 3$\n(6) $3 \\times 5(\\quad) 3+5$", "options": [], "subject": "算术", "analysis": "$<><>=>$"} {"id": "10694", "image": [], "answer": "9\n\n【分析】用每排站的人数乘排数可以计算出做课间操的同学一共有多少人, 再用做课间操的同学总数\n除以实际站的排数, 就可以计算出平均每排站多少人。\n\n【详解】 $6 \\times 6 \\div 4$\n\n$=36 \\div 4$\n\n$=9$ (人)\n\n平均每排站 9 人。\n\n【点睛】本题考查归总问题的解题方法, 解题关键是抓住归总问题总数不变, 再利用每排站的人数、站的排数、做课间操的同学总数之间的关系, 列式计算。", "solution": "null", "level": "二年级", "question": "同学们排队做课间操。如果每排站 6 人,可以站 6 排;如果站成 4 排,平均每排站( )人。", "options": [], "subject": "算术", "analysis": "9\n\n【分析】用每排站的人数乘排数可以计算出做课间操的同学一共有多少人, 再用做课间操的同学总数\n除以实际站的排数, 就可以计算出平均每排站多少人。\n\n【详解】 $6 \\times 6 \\div 4$\n\n$=36 \\div 4$\n\n$=9$ (人)\n\n平均每排站 9 人。\n\n【点睛】本题考查归总问题的解题方法, 解题关键是抓住归总问题总数不变, 再利用每排站的人数、站的排数、做课间操的同学总数之间的关系, 列式计算。"} {"id": "10695", "image": [], "answer": "$\\quad 6 \\quad 9 \\quad 9 \\quad 54 \\quad 54 \\quad 6 \\quad 9$\n\n【分析】根据表内乘法口诀即可解答。\n\n【详解】 $6 \\times 9=54 \\quad 54 \\div 6=9$\n\n【点睛】本题主要考查表内乘法口诀的灵活运用。", "solution": "null", "level": "二年级", "question": "根据口诀“六九五十四”写算式。\n\n)$\\times(\\quad)=(\\quad)(\\quad) \\div(\\quad)=(\\quad)$", "options": [], "subject": "算术", "analysis": "$\\quad 6 \\quad 9 \\quad 9 \\quad 54 \\quad 54 \\quad 6 \\quad 9$\n\n【分析】根据表内乘法口诀即可解答。\n\n【详解】 $6 \\times 9=54 \\quad 54 \\div 6=9$\n\n【点睛】本题主要考查表内乘法口诀的灵活运用。"} {"id": "11043", "image": ["2289.jpg", "2290.jpg"], "answer": "$2 \\mathrm{~kg} \\quad 400 \\mathrm{~g} \\quad 30 \\mathrm{~kg}$", "solution": "null", "level": "二年级", "question": "下面的物品有多重, 请填在括号里。\n\n\n$(\\quad)$\n\n\n\n(", "options": [], "subject": "度量几何学", "analysis": "$2 \\mathrm{~kg} \\quad 400 \\mathrm{~g} \\quad 30 \\mathrm{~kg}$"} {"id": "10247", "image": [], "answer": "$\\mathrm{x}$", "solution": "null", "level": "二年级", "question": "长 1 米的木棒比 100 厘米的铁丝短。", "options": [], "subject": "度量几何学", "analysis": "$\\mathrm{x}$"} {"id": "10270", "image": [], "answer": "$\\mathrm{x}$", "solution": "null", "level": "二年级", "question": "数学课本封面的长是 20 米。", "options": [], "subject": "度量几何学", "analysis": "$\\mathrm{x}$"} {"id": "10271", "image": [], "answer": "$\\sqrt{ }$", "solution": "null", "level": "二年级", "question": "我沿着学校操场的跑道跑了一圈, 刚好跑了 400 厘米。", "options": [], "subject": "度量几何学", "analysis": "$\\sqrt{ }$"} {"id": "10302", "image": ["2055.jpg"], "answer": "$\\sqrt{ }$", "solution": "null", "level": "二年级", "question": "\n这个图形中有 1 个角。", "options": [], "subject": "度量几何学", "analysis": "$\\sqrt{ }$"} {"id": "10326", "image": [], "answer": "$x$", "solution": "null", "level": "二年级", "question": "一个角的两边越长, 角的度数越大。", "options": [], "subject": "度量几何学", "analysis": "$x$"} {"id": "10327", "image": [], "answer": "$\\sqrt{ }$", "solution": "null", "level": "二年级", "question": "角都有一个顶点, 两条边。", "options": [], "subject": "度量几何学", "analysis": "$\\sqrt{ }$"} {"id": "10392", "image": [], "answer": "$\\times$", "solution": "null", "level": "二年级", "question": "一个角的两边越长, 角的度数越大。", "options": [], "subject": "度量几何学", "analysis": "$x$"} {"id": "10328", "image": [], "answer": "$\\mathrm{x}$", "solution": "null", "level": "二年级", "question": "直角比钝角和锐角都大。", "options": [], "subject": "立体几何学", "analysis": "$\\mathrm{x}$"} {"id": "9957", "image": [], "answer": "$\\sqrt{ }$", "solution": "null", "level": "二年级", "question": "观察球时,不管从哪个方向看,看到的形状都是圆。( )", "options": [], "subject": "立体几何学", "analysis": "$\\sqrt{ }$"} {"id": "9958", "image": ["1867.jpg", "1868.jpg", "1869.jpg", "1870.jpg"], "answer": "1)", "solution": "null", "level": "二年级", "question": "是在上面看到的。\n\n\n\n\n\n(1)\n\n\n\n(2)\n\n\n\n(3)", "options": [], "subject": "画法几何学", "analysis": "1)"} {"id": "9975", "image": ["1906.jpg", "1907.jpg", "1908.jpg", "1909.jpg", "1910.jpg", "1911.jpg"], "answer": "2)", "solution": "null", "level": "二年级", "question": "果看到的是\n\n\n\n\n\n(1)\n\n\n\n(2)\n\n\n\n(3)\n\n\n\n(2)\n\n\n\n(3)", "options": [], "subject": "画法几何学", "analysis": "2)"} {"id": "9972", "image": [], "answer": "$\\sqrt{ }$", "solution": "null", "level": "二年级", "question": "从不同的角度观察物体, 看到的形状不一定相同。", "options": [], "subject": "变换几何", "analysis": "$\\sqrt{ }$"} {"id": "9974", "image": ["1905.jpg"], "answer": "(3)", "solution": "null", "level": "二年级", "question": "面的照片是在房子后边拍的。\n\n\n(1)", "options": [], "subject": "变换几何", "analysis": "(3)"} {"id": "9977", "image": ["1915.jpg"], "answer": "2)", "solution": "null", "level": "二年级", "question": "到的照片正好是左右相反的。\n\n(1)其琪和果果\n\n(2) 果果和乔乔\n\n(3)其琪和乔乔\n\n", "options": [], "subject": "变换几何", "analysis": "2)"} {"id": "11455", "image": ["2404.jpg"], "answer": "[](本地图片-3673.jpg)", "solution": "null", "level": "二年级", "question": "画出下面各图形的对称轴。\n\n", "options": [], "subject": "变换几何", "analysis": "[](本地图片-3673.jpg)"} {"id": "11462", "image": ["2414.jpg"], "answer": "$\\square \\quad \\square \\quad \\square \\quad \\circ \\quad \\circ$", "solution": "null", "level": "二年级", "question": "下列现象是平移的在括号里画 “ $\\bigcirc$ ”, 是旋转的在括号里画 “ $\\square$ ”。\n \n\n$(\\quad)$", "options": [], "subject": "变换几何", "analysis": "$\\square \\quad \\square \\quad \\square \\quad \\circ \\quad \\circ$"} {"id": "9976", "image": ["1912.jpg", "1913.jpg", "1914.jpg"], "answer": "3)", "solution": "null", "level": "二年级", "question": "个物体, 从某个位置看到的图形是长方形, 这个物体的形状不可能是\n\n\n\n(1)\n\n\n\n(2)\n\n\n\n(3)", "options": [], "subject": "组合几何学", "analysis": "3)"} {"id": "10427", "image": [], "answer": "$\\mathrm{x}$", "solution": "null", "level": "二年级", "question": "个同学站成一排合影, 一共有 3 种站法。", "options": [], "subject": "组合数学", "analysis": "$\\mathrm{x}$"} {"id": "10452", "image": [], "answer": "$\\mathrm{x}$", "solution": "null", "level": "二年级", "question": "爸爸有 4 件上衣, 3 条裤子。每次只穿 1 件上衣和 1 条裤子, 一共有 7 种不同的穿法。", "options": [], "subject": "组合数学", "analysis": "$\\mathrm{x}$"} {"id": "10453", "image": [], "answer": "$\\mathrm{x}$", "solution": "null", "level": "二年级", "question": "有三个同学, 每两人握一次手, 一共要握 6 次手。", "options": [], "subject": "组合数学", "analysis": "$\\mathrm{x}$"} {"id": "10469", "image": ["2144.jpg"], "answer": "(1) $364 \\quad 24$\n\n(2) 4\n\n【解析】(1)\n\n略\n\n(2)\n\n略", "solution": "null", "level": "二年级", "question": "\n\n(1)有 $(\\quad)$ 束气球,每束有 (\n\n) 个,一共(\n\n)个。\n\n(2)把这些气球平均分给 6 个小朋友, 每人分得 (\n\n)个。", "options": [], "subject": "组合数学", "analysis": "(1) $364 \\quad 24$\n\n(2) 4\n\n【解析】(1)\n\n略\n\n(2)\n\n略"} {"id": "10621", "image": ["2183.jpg", "2184.jpg"], "answer": "$\\quad 8 \\quad 5$\n\n【详解】略\n\n9.", "solution": "null", "level": "二年级", "question": "用 48 个 $O$ 可以拼摆成 (\n\n$$\n\\text { )朵 }\n$$\n\n\n\n用 25 个口可以摆成(\n\n\n\n9.", "options": [], "subject": "组合数学", "analysis": "$\\quad 8 \\quad 5$\n\n【详解】略\n\n9."} {"id": "10647", "image": ["2189.jpg"], "answer": "$\\quad 8 \\quad 6$\n\n【分析】共有 24 根小棒, 摆一个三角形需要 3 根, 可以摆的个数用总根数除以一个三角形需要的根数; 摆一个正方形需要 4 根小棒, 可以摆的个数用总根数除以一个正方形需要的根数即可解答。\n\n【详解】 $24 \\div 3=8$ (个)\n\n$24 \\div 4=6$ (个)\n\n用 24 根小棒可以摆成 8 个三角形或 6 个正方形。\n\n【点睛】本题主要考查了表内除法的应用。", "solution": "null", "level": "二年级", "question": "用 24 根小棒可以摆成(\n\n\n)个廿。", "options": [], "subject": "计数", "analysis": "$\\quad 8 \\quad 6$\n\n【分析】共有 24 根小棒, 摆一个三角形需要 3 根, 可以摆的个数用总根数除以一个三角形需要的根数; 摆一个正方形需要 4 根小棒, 可以摆的个数用总根数除以一个正方形需要的根数即可解答。\n\n【详解】 $24 \\div 3=8$ (个)\n\n$24 \\div 4=6$ (个)\n\n用 24 根小棒可以摆成 8 个三角形或 6 个正方形。\n\n【点睛】本题主要考查了表内除法的应用。"} {"id": "11488", "image": [], "answer": "63", "solution": "null", "level": "二年级", "question": "18 根小棒可以摆成 ( ) 个 , 如果是摆 , 可以摆 ( ) 个。", "options": [], "subject": "计数", "analysis": "63"} {"id": "11513", "image": [], "answer": "2", "solution": "null", "level": "二年级", "question": "摆一个用 3 根小棒, 6 根小棒能摆 ( ) 个』。", "options": [], "subject": "计数", "analysis": "2"} {"id": "11230", "image": [], "answer": "$ \\times$", "solution": "null", "level": "二年级", "question": "39 里面最多有 7 个 6 。", "options": [], "subject": "计数", "analysis": "$ \\times$"} {"id": "11176", "image": ["2319.jpg"], "answer": "数学", "solution": "null", "level": "二年级", "question": "他们分别拿着语文、数学、英语书, 根据对话内容可知, 小兰拿着 ( )书。\n\n", "options": [], "subject": "逻辑题", "analysis": "数学"} {"id": "11192", "image": [], "answer": "灰 白", "solution": "null", "level": "二年级", "question": "小花兔、小灰兔和小白兔三只兔子在赛跑。小花兔说: “我跑得不是最快的, 但比小白兔快。”请你说说, 小 ( ) 兔跑得最快, 小 ( ) 兔跑得最慢。", "options": [], "subject": "逻辑题", "analysis": "灰 白"} {"id": "11560", "image": [], "answer": "(1) $11 \\quad 101 \\quad$ (2) 合 喜 天", "solution": "null", "level": "二年级", "question": "仔细观察, 可以发现一些数字与汉字是轴对称图形, 请按要求写一写。\n\n$$\n\\begin{array}{lllllllllll}\n11 & 38 & 54 & 66 & 79 & 101 & \\text { 合家欢喜连天 }\n\\end{array}\n$$\n\n(1) 找出是轴对称图形的数字: (\n\n(2) 找出是轴对称图形的汉字: (\n\n)。", "options": [], "subject": "图论", "analysis": "(1) $11 \\quad 101 \\quad$ (2) 合 喜 天"} {"id": "9841", "image": [], "answer": "直\n\n【分析】根据直角、钝角的意义, 90 度角是直角, 大于 90 度小于 180 度的角叫做钝角。由生活经验可知: 钟面上 3 时整, 时针与分针所成的角是直角。\n\n【详解】据分析可知: 3 时整,时针与分针所成的角是直角。\n\n【点睛】此题考查的目的是理解掌握直角、钝角的意义及应用。", "solution": "null", "level": "二年级", "question": "3 时整, 时针与分针所成的角是 $(\\quad)$ 角。", "options": [], "subject": "解析几何", "analysis": "直\n\n【分析】根据直角、钝角的意义, 90 度角是直角, 大于 90 度小于 180 度的角叫做钝角。由生活经验可知: 钟面上 3 时整, 时针与分针所成的角是直角。\n\n【详解】据分析可知: 3 时整,时针与分针所成的角是直角。\n\n【点睛】此题考查的目的是理解掌握直角、钝角的意义及应用。"} {"id": "21053", "image": [], "answer": "\\quad 1,2$.", "solution": "null", "level": "高三", "question": "已知命题 $\\mathrm{p}:|\\mathrm{x}-1| \\geq 2$ 和命题 $\\mathrm{q}: \\mathrm{x} \\in \\mathrm{Z}$. 若 “ $p$ 且 $q$ ” 与 “非 $\\mathrm{q}$ ” 同时为假命题, 求实数 $x$ 的值", "options": [], "subject": "逻辑题", "analysis": "由命题 $p:|x-1| \\geqslant 2$, 得到命题 $P: x-1 \\geqslant 2$ 或 $x-1 \\leqslant-2$, 即命题 $P: x \\geqslant 3$ 或 $x \\leqslant-1$; $\because \neg q$ 为假命题, $\therefore$ 命题 $q: x \\in Z$ 为真命题, 再由 “ $p$ 且 $q$ ” 为假命题, 知命题 $P: x \\geqslant 3$或 $x \\leqslant-1$ 是假命题, 故 $-10$ 恒成立, 命题 $Q: \\exists x_{0} \\in[-2,2]$, 使得 $2^{\\mathrm{a}} \\leqslant$ $2^{x_{0}}$, 若命题 $P \\wedge Q$ 为真命题, 则实数 $a$ 的取值范围为", "options": [], "subject": "逻辑题", "analysis": "因为命题P: $\forall x \\in R, \\log _{2}\\left(x^{2}+x+a\right)>0$ 恒成立,所以 $x^{2}+x+a-1>0$ 对于 $\forall x \\in R$ 恒成立, 即 $\\Delta=1-4(a-1)<0$, 解得 $a>$ $\frac{5}{4}$;因为命题 $Q: \\exists x_{0} \\in[-2,2]$, 使得 $2^{a} \\leq 2^{x_{0}}$, 所以 $a \\leq 2$. 因为命题 $P \\wedge Q$ 为真命题,"} {"id": "21066", "image": [], "answer": "$\forall x<1, \\quad x^{2}+2 x+1>0$", "solution": "null", "level": "高三", "question": "命题 “ $\\exists x<1, \\quad x^{2}+2 x+1 \\leq 0$ ” 的否定是", "options": [], "subject": "逻辑题", "analysis": "特称命题的否定是全称命题, 命题 “ $\\exists x<1, x^{2}+2 x+1 \\leq 0$ ” 的否定是: “ $\forall x$ $<1, x^{2}+2 x+1>0$ ”.故答案为 $\forall x<1, x^{2}+2 x+1>0$."} {"id": "21079", "image": [], "answer": "", "solution": "null", "level": "高三", "question": "“若 $x<0$, 则 $x^{2}>x$ ” 的否命题是 \\$ \\qquad \\$命题. (请在 “真” 或 “假”中选择恰当的一个填在横线上)", "options": [], "subject": "逻辑题", "analysis": "命题 “若 $\\mathrm{x}<0$, 则 $\\mathrm{x} 2>\\mathrm{x}$ ” 的否命题是: “若 $\\mathrm{x} \\geq 0$, 则 $\\mathrm{x} 2 \\leq \\mathrm{x}$ ”,这是一个假命题,如 $x=3$,则 $x^{2}>x$. 故答案为假."} {"id": "21092", "image": [], "answer": "1", "solution": "null", "level": "高三", "question": "命题 “若 $a>-3$, 则 $a>-6$ ” 的逆命题、否命题、逆否命题中, 真命题的个数为", "options": [], "subject": "逻辑题", "analysis": "因为在命题的四种形式中原命题和逆否命题互为逆否命题, 同真同假,否命题和逆命题互为逆否命题同真同假.$\because$ 原命题 “若 $a>-3$, 则 $a>-6$ ” 为真命题; 逆命题 “若 $a>-6$, 则 $a>-$ 3 ” 是假命题, $\therefore$ 命题的逆否命题为真命题, 否命题为假命题. 故答案为 1 ."} {"id": "22988", "image": [], "answer": "答案: $m>2$", "solution": "null", "level": "高三", "question": "17. 已知命题 $p:\\left|1-\\frac{x+1}{2}\\right| \\leq 1$, 命题 $q: x^{2}-2 x+1-m^{2}<0(m>0)$, 若 $p$ 是 $q$ 的充分不必要条件, 则实数 $m$ 的范围是", "options": [], "subject": "逻辑题", "analysis": ": 解答: 命题 $p$ 首先化简为 $-1 \\leq x \\leq 3$, 命题 $q$ 是二次不等式, $p$ 是 $q$ 的充分不必要条件说 明当 $-1 \\leq x \\leq 3$ 时不等式 $x^{2}-2 x+1-m^{2}<0$ 恒成立, 故 $\\left\\{\\begin{array}{l}(-1)^{2}-2 \\times(-1)+1-m^{2}<0, \\\\ 3^{2}-2 \\times 3+1-m^{2}<0,\\end{array}\\right.$ 又 $m>0$, 故可解得 $m>2$\n分析: 本题主要考查了绝对值不等式的解法, 解决问题的关键是根据绝对值不等式的解法结合命题的关系分析计算即可"} {"id": "20499", "image": [], "answer": "答案: $\\left(a_{1}-1\\right)+\\left(a_{2}-2\\right)+\\cdots+\\left(a_{7}-7\\right)=\\left(a_{1}+a_{2}+\\cdots+a_{7}\\right)-(1+2+\\cdots+7)$", "solution": "null", "level": "高三", "question": "16. 完成反证法证题的全过程. 设 $\\mathrm{a}_{1}, \\mathrm{a}_{2}, \\ldots, \\mathrm{a}_{7}$ 是 $1,2, \\ldots, 7$ 的一个排列,求证: 乘积 $\\mathrm{p}=\\left(\\mathrm{a}_{1}-1\\right)\\left(\\mathrm{a}_{2}-2\\right) \\ldots\\left(\\mathrm{a}_{2}-7\\right)$ 为偶数. 证明:假设 $\\mathrm{p}$ 为奇数, 则 $\\mathrm{a}_{1}-1, \\mathrm{a}_{2}-2, \\ldots, \\mathrm{a}_{7}-7$ 均为奇数. 因奇数个奇数之和为奇数, 故有奇数 $=$ \\$ \\qquad \\$ $=0$.但 $0 \\neq$ 奇数, 这一矛盾说明 $\\mathrm{p}$ 为偶数.", "options": [], "subject": "逻辑题", "analysis": ": 解答: 因为 $a_{1}, a_{2}, \\cdots, a_{7}$ 是 $1,2,3, \\cdots$ 的一个排列, 所以 $\\left(a_{1}-1\\right)+\\left(a_{2}-2\\right)+\\cdots+\\left(a_{7}-7\\right)$ $=\\left(a_{1}+a_{2}+\\cdots+a_{7}\\right)-(1+2+\\cdots+7)=0$, 所以答案为\n$\\left(a_{1}-1\\right)+\\left(a_{2}-2\\right)+\\cdots+\\left(a_{7}-7\\right)=\\left(a_{1}+a_{2}+\\cdots+a_{7}\\right)-(1+2+\\cdots+7)$.\n\n分析: 本题主要考查了反证法的应用, 解决问题的关键是根据所给证明步骤进行分析即可."} {"id": "23028", "image": [], "answer": ":答案: (3)(1)(2)", "solution": "null", "level": "高三", "question": "10. 用反证法证明“一个三角形不能有两个直角”有三个步骤:\n\n(1) $\\angle A+\\angle B+\\angle C=90^{\\circ}+90^{\\circ}+\\angle C>180^{\\circ}$, 这与三角形内角和为 $180^{\\circ}$ 矛盾, 故假设错误.\n\n(2)所以一个三角形不能有两个直角.\n\n(3)假设 $\\triangle A B C$ 中有两个直角, 不妨设 $\\angle A=90^{\\circ}, \\angle B=90^{\\circ}$.\n\n上述步骤的正确顺序为 \\$ \\qquad \\$ .(填序号)", "options": [], "subject": "逻辑题", "analysis": ":【解答】根据反证法知, 上述步骤的正确顺序应为(3)(1)(2)\n\n【分析】本题主要考查了反证法与放缩法, 解决问题的关键是根据反证法的步骤分析判断即可."} {"id": "21054", "image": [], "answer": "$\\left(\frac{1}{2}, 2\right)$", "solution": "null", "level": "高三", "question": "命题 $p: a^{2}-a-2<0$, 命题 $q:$ 函数 $y=1 g\\left(a x^{2}-x+a\\right)$ 的定义域为 $R$, 若 “ $\\mathrm{p}$ 且 $q$ ” 为真命题, 则 $\\mathrm{a}$ 的取值范围是", "options": [], "subject": "代数", "analysis": "因为当命题 $p: a^{2}-a-2<0$ 为真时, $-10 \\ 1-4 a^{2}<0\\end{array}\right.$, 解析得 $a>\frac{1}{2}$. 要 “ $p$ 且 $q$ ” 为真命题, 则 $p$ 与 $q$ 都是真命题, 所以 $a$ 的取值范围是 $\frac{1}{2}<$ $a<2$. 故答案为 $\\left(\frac{1}{2}, 2\right)$."} {"id": "22970", "image": [], "answer": "答案: $a b \\geq 9$", "solution": "null", "level": "高三", "question": "20. 若正数 $\\mathrm{a} . \\mathrm{~b}$ 满足 $a b=a+b+3$, 则 $a b$ 的取值范围是", "options": [], "subject": "代数", "analysis": ": 解答: 由 $\\mathrm{a} . \\mathrm{~b}$ 均为正数, 有 $a+b \\geq 2 \\sqrt{a b}$, 则 $a b \\geq 2 \\sqrt{a b}+3$, 利用换元法设 $t=\\sqrt{a b}$ $(t>0)$, 解得 $t \\leq-1$ (舍), 或 $t \\geq 3$, 即 $a b \\geq 9$.\n\n分析: 本题主要考查了基本不等式在最值问题中的应用, 解决问题的关键是根据基本不等式分析进行即可"} {"id": "22971", "image": [], "answer": "答案: 9", "solution": "null", "level": "高三", "question": "21. 已知 $00 \\\\ 1-x>0\\end{array}\\right.$,而\n\n$y=\\frac{4}{x}+\\frac{1}{1-x}=\\frac{4}{x}-4+\\frac{1}{1-x}-1+5=\\frac{4(1-x)}{x}+\\frac{x}{1-x}+5 \\geq 2 \\sqrt{4}+5=9$, 当且仅当 $x=\\frac{2}{3}$\n\n时, 上式取“=”,所以 $y_{\\text {min }}=9$.\n\n分析: 本题主要考查了基本不等式在最值问题中的应用, 解决问题的关键是根据所给不等式分析计算即可"} {"id": "21762", "image": ["9660.jpg"], "answer": "$\\left[x_{3}, x_{4}\\right]$.", "solution": "null", "level": "高三", "question": "15.如图 1-1-5 所示, 函数 $y=f(x)$ 在 $\\left[x_{1}, x_{2}\\right],\\left[x_{2}, x_{3}\\right],\\left[x_{3}, x_{4}\\right]$ 这几个区间内, 平均变化率最大的一个区间是 \\$ \\qquad \\$ .\n\n\n\n图 1-1-5", "options": [], "subject": "代数", "analysis": ": $\\left[x_{3}, x_{4}\\right]$\n\n由平均变化率的定义可知, 函数 $y=f(x)$ 在区间 $\\left[x_{1}, x_{2}\\right],\\left[x_{2}, x_{3}\\right],\\left[x_{3}, x_{4}\\right]$ 上的平均变化率分别为: $\\frac{f\\left(x_{2}\\right)-f\\left(x_{1}\\right)}{x_{2}-x_{1}}, \\frac{f\\left(x_{3}\\right)-f\\left(x_{2}\\right)}{x_{3}-x_{2}}, \\frac{f\\left(x_{4}\\right)-f\\left(x_{3}\\right)}{x_{4}-x_{3}}$, 结合图象可以发现函数 $y$ $=f(x)$ 的平均变化率最大的一个区间是 $\\left[x_{3}, x_{4}\\right]$."} {"id": "22986", "image": [], "answer": "答案: $[-1,3]$", "solution": "null", "level": "高三", "question": "15. 对于任意实数 $a(a \\neq 0)$ 和 $b$ 不等式 $|a+b|+|a-b| \\geq|a||x-1|$ 恒成立, 则实数 $\\mathrm{x}$ 的取值范围是", "options": [], "subject": "代数", "analysis": ":解答:依题意可得 $|x-1| \\leq\\left|1-\\frac{b}{a}\\right|+\\left|1+\\frac{b}{a}\\right|$ 恒成立, 等价于 $|x-1|$ 小于或等于 $\\left|1-\\frac{b}{a}\\right|+\\left|1+\\frac{b}{a}\\right|$ 的 最 小 值. 因 为 $\\left|1-\\frac{b}{a}\\right|+\\left|1+\\frac{b}{a}\\right| \\geq\\left|\\left(1-\\frac{b}{a}\\right)+\\left(1+\\frac{b}{a}\\right)\\right|=2$. 所 以 $|x-1| \\leq 2, \\therefore x \\in[-1,3]$.\n\n分析: 本题主要考查了绝对值不等式的解法, 解决问题的关键是根据绝对值不等式的性质分析计算即可"} {"id": "22989", "image": [], "answer": "答案: $[-2,5]$", "solution": "null", "level": "高三", "question": "18. 若不等式 $|x+3|+|x-7| \\geq a^{2}-3 a$ 的解集为 $R$, 则实数 $a$ 的取值范围是 $—$.", "options": [], "subject": "代数", "analysis": ": 解答: 不等式 $|x+3|+|x-7| \\geq a^{2}-3 a$ 的解集为 $R$, 所以 $a^{2}-3 a \\leq(|x+3|+|x-7|)_{\\text {min }}$. $|x+3|+|x-7| \\geq|x+3-(x-7)|=10$, 所以 $a^{2}-3 a \\leq 10, a^{2}-3 a-10 \\leq 0,-2 \\leq a \\leq 5$分析: 本题主要考查了绝对值不等式的解法, 解决问题的关键是"} {"id": "22990", "image": [], "answer": "答案: $a>10$", "solution": "null", "level": "高三", "question": "19. 如果关于 $\\mathrm{x}$ 的不等式 $|x-10|+|x-20|10$.\n\n分析: 本题主要考查了绝对值不等式的解法, 解决问题的关键是根据绝对值的性质分析计算即可"} {"id": "22992", "image": ["10095.jpg"], "answer": "答案: $(-7,-1] \\cup[5,11)$", "solution": "null", "level": "高三", "question": "20. 不等式 $3 \\leq|x-2|<9$ 的解集为 \\$ \\qquad \\$", "options": [], "subject": "代数", "analysis": ":解答: 不等式 $3 \\leq x-2 \\mid<9$ 可化为 $\\left\\{\\begin{array}{l}|x-2| \\geq 3 \\\\ |x-2|<9\\end{array}\\right.$\n\n由 $|x-2| \\geq 3$ 得 $x-2 \\geq 3$ 或 $x-2 \\leq-3$. 解得 $x \\geq 5$ 或 $x \\leq-1$\n\n由 $|x-2|<9$ 得 $-9\n\n分析: 本题主要考查了绝对值不等式的解法, 解决问题的关键是根据绝对值的几何意义结合数轴分析计算即可"} {"id": "22399", "image": [], "answer": "答案: $-x^{5}$\n\n【解析】 $\\left(x-\\frac{1}{7 x}\\right)^{7}$ 的展开式的第 2 项为 $T_{2}=C_{7} \\cdot\\left(-\\frac{1}{7}\\right)^{1} \\cdot x^{5}=-x^{5}$,\n\n故答案为: $-x^{5}$.", "solution": "null", "level": "高三", "question": "$\\left(x-\\frac{1}{7 x}\\right)^{7}$ 的展开式的第 2 项为", "options": [], "subject": "代数", "analysis": "答案: $-x^{5}$\n\n【解析】 $\\left(x-\\frac{1}{7 x}\\right)^{7}$ 的展开式的第 2 项为 $T_{2}=C_{7} \\cdot\\left(-\\frac{1}{7}\\right)^{1} \\cdot x^{5}=-x^{5}$,\n\n故答案为: $-x^{5}$."} {"id": "23011", "image": [], "answer": ":答案: 充要", "solution": "null", "level": "高三", "question": "15. 设 $p, q$ 均为实数, 则 “ $q<0$ ” 是“方程 $x^{2}+p x+q=0$ 有一个正实根和一个负实根” 的条件.(选填:充要.必要不充分.充分不必要.既不充分也不必要)", "options": [], "subject": "代数", "analysis": ":【解答】 $\\because q<0, \\therefore \\Delta=p^{2}-4 q>0$.\n\n$\\therefore$ “方程 $x^{2}+p x+q=0$ 有一个正实根和一个负实根”成立.\n\n$\\because “$ 方程 $x^{2}+p x+q=0$ 有一个正实根和一个负实根”成立, $\\therefore q<0$\n\n【分析】本题主要考查了分析法与综合法, 解决问题的关键是根据所给条件 $q<0$, 可得 $\\Delta$ $=p^{2}-4 q>0$ ,然后分析判断即可."} {"id": "23012", "image": [], "answer": "答案: $a^{2}+b^{2}-2 a b \\geq 0 \\quad(a-b)^{2} \\geq 0 \\quad(a-b)^{2} \\geq 0$", "solution": "null", "level": "高三", "question": "16. 补足下面用分析法证明基本不等式 $\\frac{a^{2}+b^{2}}{2} \\geq a b$ 的步骤:\n\n要证明 $\\frac{a^{2}+b^{2}}{2} \\geq a b$,\n\n只需证明 $a^{2}+b^{2} \\geq 2 a b$,\n只需证\n\n只需证 \\$ \\qquad \\$\n由于显然成立, 因此原不等式成立.", "options": [], "subject": "代数", "analysis": ":【解答】要证明\n\n$$\n\\frac{a^{2}+b^{2}}{2} \\geq a b\n$$\n\n只需证明 $a^{2}+b^{2} \\geq 2 a b$,\n\n只需证 $a^{2}+b^{2}-2 a b \\geq 0$,\n\n只需证 $(a-b)^{2} \\geq 0$,\n\n由于 $(a-b)^{2} \\geq 0$ 显然成立, 因此原不等式成立\n\n【分析】本题主要考查了分析法与综合法, 解决问题的关键是根据分析法的怎么步骤分析计算即可."} {"id": "20500", "image": [], "answer": "答案: 偶", "solution": "null", "level": "高三", "question": "17. 设 $y=f(x)(x \\in \\mathbf{R}, x \\neq 0)$ 对任意非零实数 $x_{1}, x_{2}$ 均满足 $f\\left(x_{1} x_{2}\\right)=f\\left(x_{1}\\right)+f\\left(x_{2}\\right)$, 则 $f(x)$ 为函数. (填 “奇” 或 “偶”)", "options": [], "subject": "代数", "analysis": ":解答:因为 $\\mathrm{f}(1 \\times 1)=\\mathrm{f}(1)+\\mathrm{f}(1)$, 所以 $\\mathrm{f}(1)=0$,\n\n$\\mathrm{f}[(-1) \\times(-1)]=\\mathrm{f}(-1)+\\mathrm{f}(-1)=0$, 所以 $\\mathrm{f}(-1)=0, \\mathrm{f}[(-1) \\mathrm{x}]=\\mathrm{f}(\\mathrm{x})+\\mathrm{f}(-1)$,\n\n即 $\\mathrm{f}(-\\mathrm{x})=\\mathrm{f}(\\mathrm{x})$ 所以 $\\mathrm{f}(\\mathrm{x})$ 是偶函数.\n\n分析: 本题主要考查了综合法, 解决问题的关键是根据所给条件和所求结论运用综合法分析计算即可证明问题."} {"id": "20502", "image": [], "answer": "答案: $a_{1}+a_{2}+\\ldots+a_{n} \\leq \\sqrt{n}$", "solution": "null", "level": "高三", "question": "19. 请阅读下列材料: 若两个正实数 $\\mathrm{a}_{1}, \\mathrm{a}_{2}$ 满足 $\\mathrm{a}_{1}{ }^{2}+\\mathrm{a}_{2}{ }^{2}=1$, 那么 $\\mathrm{a}_{1}+\\mathrm{a}_{2} \\leq \\sqrt{2}$.\n\n证明: 构造函数 $\\mathrm{f}(x)=\\left(x-\\mathrm{a}_{1}\\right)^{2}+\\left(x-\\mathrm{a}_{2}\\right)^{2}=2 x^{2}-2\\left(\\mathrm{a}_{1}+\\mathrm{a}_{2}\\right) x+1$, 因为对一切实数 $x$, 恒有 $\\mathrm{f}(x) \\geq 0$,所以 $\\Delta \\leq 0$, 从而得 $4\\left(a_{1}+a_{2}\\right)^{2}-8 \\leq 0$, 所以 $a_{1}+a_{2} \\leq \\sqrt{2}$.\n\n根据上述证明方法, 若 $\\mathrm{n}$ 个正实数满足 $\\mathrm{a}_{1}{ }^{2}+\\mathrm{a}_{2}{ }^{2}+\\ldots+\\mathrm{a}_{\\mathrm{n}}{ }^{2}=1$ 时, 你能得到的结论为 \\$ \\qquad \\$ .", "options": [], "subject": "代数", "analysis": ": 解答: 构造函数 $\\mathrm{f}(x)=\\left(x-\\mathrm{a}_{1}\\right)^{2}+\\left(x-\\mathrm{a}_{2}\\right)^{2}+\\ldots+\\left(x-\\mathrm{a}_{n}\\right)^{2}=\\mathrm{n} x^{2}-2\\left(\\mathrm{a}_{1}+\\mathrm{a}_{2}+\\ldots+\\mathrm{a}_{n}\\right) x+1$,因为对一切实数 $x$, 恒有 $\\mathrm{f}(x) \\geq 0$, 所以 $\\Delta \\leq 0$, 从而得 $4\\left(\\mathrm{a}_{1}+\\mathrm{a}_{2}+\\ldots+\\mathrm{a}_{\\mathrm{n}}\\right)^{2}-4 \\mathrm{n} \\leq 0$,所以 $\\mathrm{a}_{1}+\\mathrm{a}_{2}+\\ldots+\\mathrm{a}_{\\mathrm{n}} \\leq \\sqrt{n}$.\n\n分析: 本题主要考查了公理化思想, 解决问题的关键是根据所给条件与结论之间的联系进行构造函数, 运用公理化思想进行分析计算即可得到所求结论."} {"id": "20504", "image": [], "answer": "答案: 9", "solution": "null", "level": "高三", "question": "20. 设 $\\mathrm{a}, \\mathrm{b}, \\mathrm{c}$ 为正数, 若 $a+b+c=1$, 则 $\\frac{1}{a}+\\frac{1}{b}+\\frac{1}{c} \\geq$ \\$ \\qquad \\$ .", "options": [], "subject": "代数", "analysis": ": 解答: 因为 $a+b+c=1$, 所以 $\\frac{1}{a}+\\frac{1}{b}+\\frac{1}{c}=(a+b+c)\\left(\\frac{1}{a}+\\frac{1}{b}+\\frac{1}{c}\\right)$ $=3+\\frac{b}{a}+\\frac{a}{b}+\\frac{a}{c}+\\frac{c}{a}+\\frac{c}{b}+\\frac{b}{c} \\geq 3++2+2+2=9$.\n\n分析: 本题主要考查了综合法, 解决问题的关键是根据所给条件结合所求命题进行分析计算即可."} {"id": "22447", "image": [], "answer": "答案: $\\frac{3}{16}$\n\n解析:由题意得 $P(X=4)=\\mathrm{C}_{5}^{+}\\left(\\frac{1}{2}\\right)^{+} \\times\\left(1-\\frac{1}{2}\\right)=\\frac{5}{32}, \\quad P(X=5)=\\mathrm{C}_{5}^{3}\\left(\\frac{1}{2}\\right)^{3}=\\frac{1}{32}$,\n\n$\\therefore P(X \\ldots 4)=P(X=4)+P(X=5)=\\frac{5}{32}+\\frac{1}{32}=\\frac{3}{16}$", "solution": "null", "level": "高三", "question": "若 $X \\sim B\\left(5, \\frac{1}{2}\\right)$, 则 $P(X \\geq 4)=$", "options": [], "subject": "代数", "analysis": "答案: $\\frac{3}{16}$\n\n解析:由题意得 $P(X=4)=\\mathrm{C}_{5}^{+}\\left(\\frac{1}{2}\\right)^{+} \\times\\left(1-\\frac{1}{2}\\right)=\\frac{5}{32}, \\quad P(X=5)=\\mathrm{C}_{5}^{3}\\left(\\frac{1}{2}\\right)^{3}=\\frac{1}{32}$,\n\n$\\therefore P(X \\ldots 4)=P(X=4)+P(X=5)=\\frac{5}{32}+\\frac{1}{32}=\\frac{3}{16}$"} {"id": "20816", "image": [], "answer": "$\\sqrt{3}$", "solution": "null", "level": "高三", "question": "若复数 $z$ 满足 $z \\cdot z=3$ 则 $|z|=$", "options": [], "subject": "代数", "analysis": "设 $z=a+b \\mathrm{i}(a, b \\in \\mathrm{R})$ 有 $z \\cdot z=a^{2}+b^{2}=3, \\mid z \\models \\sqrt{3}$"} {"id": "20823", "image": ["9487.jpg"], "answer": "$-\frac{5}{4}$", "solution": "null", "level": "高三", "question": "执行如图所示的程序框图, 若输入 $x=10$, 则输出 $\\mathrm{y}$ 的值为\n\n\n\n结束", "options": [], "subject": "代数", "analysis": "本题程序框图实际上是迭代求函数值, 条件是函数值与自变量的差的绝对值小于 1 时就停止, 输出 $^{y}$.把初始值 10 代入计算,最后可得结论 ${ }^{-\frac{5}{4}}$"} {"id": "20844", "image": [], "answer": "答案: $(0,1)$", "solution": "null", "level": "高三", "question": "已知复数 ${ }^{z=\\frac{1}{i} \\text {, 则 }}{ }^{z}$ 的共轭复数 $\\bar{z}$ 在复平面内对应的点的坐标为", "options": [], "subject": "代数", "analysis": "答案: $(0,1)$\n\n解析: 复数 $z=\\frac{1}{\\mathrm{i}}=\\frac{\\mathrm{i}}{\\mathrm{i}^{2}}=-\\mathrm{i}$, 故 $\\bar{z}=\\mathrm{i}$, 得 $\\bar{z}$ 在复平面内对应的点的坐标为 $(0,1)$."} {"id": "21328", "image": ["9546.jpg"], "answer": "$\\{a \\mid a<-27$ 或 $a>5\\}$", "solution": "null", "level": "高三", "question": "已知曲线 $f(x)=-x^{3}+3 x^{2}+9 x+a$ 与 $x$ 轴只有一个交点, 则实数 $a$ 的取值范围为", "options": [], "subject": "代数", "analysis": "$f^{\\prime}(x)=-3 x^{2}+6 x+9$, 令 $f^{\\prime}(x)=0$, 解得 $x=-1$ 或 $x=3$.当 $x$ 发生变化时, $f^{\\prime}(x), f(x)$ 的变化情况如下表:| $x$ | $(-\\infty,-1)$ | -1 | $(-1,3)$ | 3 | $(3,+\\infty)$ || :---: | :---: | :---: | :---: | :---: | :---: || $f^{\\prime}(x)$ | - | 0 | + | 0 | - || $f(x)$ | $\\searrow$ | 极小值 | $\nearrow$ | 极大值 | $\\searrow$ |所以当 $x=-1$ 时, $f(x)$ 有极小值, 且极小值为 $f(-1)=a-5$;当 $x=3$ 时, $f(x)$ 有极大值, 且极大值为 $f(3)=a+27$.画出大致图象, 要使 $f(x)$ 的图象与 $x$ 轴只有一个交点, 只需极大值小于 0 (如图 1) 或极小值大于 0 (如图 2),图1![](https://cdn.mathpix.com/cropped/2024_04_19_28d10227961faf2becddg-26.jpg?height=321&width=380&top_left_y=2497&top_left_x=1049)图2所以 $a+27<0$ 或 $a-5>0$, 解得 $a<-27$ 或 $a>5$,故实数 $a$ 的取值范围为 $\\{a \\mid a<-27$ 或 $a>5\\}$.故答案为 $\\{a \\mid a<-27$ 或 $a>5\\}$ ,"} {"id": "20501", "image": [], "answer": "答案: $a_{5}>b_{5}$", "solution": "null", "level": "高三", "question": "18. 在等比数列 $\\left\\{a_{n}\\right\\}$ 和等差数列 $\\left\\{b_{n}\\right\\}$ 中, $a_{1}=b_{1}>0, a_{3}=b_{3}>0, a_{1} \\neq a_{3}$, 则 $a_{5}$ 与 $b_{5}$ 的大小关系为", "options": [], "subject": "算术", "analysis": ":解答:设公比为 $q$, 公差为 $d$. 则 $a_{3}=a_{1} q^{2}, b_{3}=b_{1}+2 d=a_{1}+2 d$,\n\n由 $\\mathrm{a}_{3}=\\mathrm{b}_{3}, \\therefore 2 \\mathrm{~d}=\\mathrm{a}_{1}\\left(\\mathrm{q}^{2}-1\\right)$.\n\n又 $\\because \\mathrm{a}_{1} \\neq \\mathrm{a}_{3}, \\quad \\therefore \\mathrm{q}^{2} \\neq 1$.\n\n$\\therefore \\mathrm{a}_{5}-\\mathrm{b}_{5}=\\mathrm{a}_{1} \\mathrm{q}^{4}-\\left(\\mathrm{a}_{1}+4 \\mathrm{~d}\\right)=\\mathrm{a}_{1}\\left(\\mathrm{q}^{2}-1\\right)^{2}>0 . \\therefore \\mathrm{a}_{5}>\\mathrm{b}_{5}$.\n\n分析: 本题主要考查了综合法的思考过程、特点及应用, 解决问题的关键是根据所求命题与所给条件综合分析计算比较大小即可."} {"id": "23029", "image": [], "answer": ":答案:质数只有有限多个除 $p_{1}, p_{2}, \\ldots, p_{n}$ 之外", "solution": "null", "level": "高三", "question": "11. 用反证法证明质数有无限多个的过程如下:假设 \\$ \\qquad \\$ . 设全体质数为 $p_{1}, p_{2}, \\ldots, p_{n}$, 令 $p=p_{1} p_{2} \\ldots p_{n}+1$.\n\n显然, $p$ 不含因数 $p_{1}, p_{2}, \\ldots, p_{n}$ 故 $p$ 要么是质数, 要么含有 \\$ \\qquad \\$的质因数. 这表明, 除质数 $p_{1}, p_{2}, \\ldots, p_{n}$ 之外, 还有质数, 因此原假设不成立.于是, 质数有无限多个。", "options": [], "subject": "算术", "analysis": ":【解答】由反证法的步骤可得, 应假设质数只有有限多个, 故 $p$ 要么是质数, 要么含有除 $p_{1}, p_{2}, \\ldots, p_{n}$ 之外的质因数\n\n【分析】本题主要考查了反证法与放缩法, 解决问题的关键是根据反证法的证明原理进行具体分析判断步骤即可."} {"id": "20807", "image": ["9478.jpg"], "answer": " \n\n$$\nk \\pi-\\frac{\\pi}{4}(k \\in \\mathbf{Z})\n$$", "solution": "null", "level": "高三", "question": "若复数 $\\cos \\theta-i \\sin \\theta$ 与 $-\\sin \\theta+i \\cos \\theta(\\theta \\in \\mathbf{R})$ 相等, 则 $\\theta=$ \\$ \\qquad \\$。", "options": [], "subject": "算术", "analysis": "答案:\n\n$$\nk \\pi-\\frac{\\pi}{4}(k \\in \\mathbf{Z})\n$$\n\n解析:由复数相等的条件得 $\\cos \\theta=-\\sin \\theta$,\n\n$$\n\\theta=k \\pi-\\frac{\\pi}{4}(k \\in \\mathbf{Z})\n$$"} {"id": "22375", "image": [], "answer": "答案: 24\n\n【解析】分三步来完成:第一步,放第一个小球,有 4 种放法;\n\n第二步,放第二个小球, 有 3 种放法;\n\n第三步,放第三个小球, 有 2 种放法.\n\n根据分步乘法计数原理. 得共有 $4 \\times 3 \\times 2=24$ 种放法.", "solution": "null", "level": "高三", "question": "个不同的小球放入 4 个不同的盒子,每个盒子至多放 1 个小球,共有种放法.", "options": [], "subject": "组合数学", "analysis": "答案: 24\n\n【解析】分三步来完成:第一步,放第一个小球,有 4 种放法;\n\n第二步,放第二个小球, 有 3 种放法;\n\n第三步,放第三个小球, 有 2 种放法.\n\n根据分步乘法计数原理. 得共有 $4 \\times 3 \\times 2=24$ 种放法."} {"id": "22383", "image": [], "answer": "答案: 192\n\n【解析】 不能被 5 整除实质上是末位数字不是 0 或 5 ,则可以在全部符合条件的四位数中排除末位数字是 0 或 5 的即可; 所有 4 位数有 $A_{5}^{3} \\cdot A_{5}^{3}=300$ 个, 末位为 0 时有 $A_{5}^{3}=60$ 个, 末位为 5 时有 $A_{1}^{2} \\cdot A_{4}^{2}=4 \\times 12=48$ 个, 则不能被 5 整除的数共有有 $300-60-48=192$ 个;", "solution": "null", "level": "高三", "question": "在由数字 $0,1,2,3,4,5$ 所组成的没有重复数字的四位数中, 不能被 5 整除的数共有个. (用数字作答)\n\n![](https://cdn.mathpix.com/cropped/2024_04_19_7da760c0cd38339a5113g-", "options": [], "subject": "组合数学", "analysis": "答案: 192\n\n【解析】 不能被 5 整除实质上是末位数字不是 0 或 5 ,则可以在全部符合条件的四位数中排除末位数字是 0 或 5 的即可; 所有 4 位数有 $A_{5}^{3} \\cdot A_{5}^{3}=300$ 个, 末位为 0 时有 $A_{5}^{3}=60$ 个, 末位为 5 时有 $A_{1}^{2} \\cdot A_{4}^{2}=4 \\times 12=48$ 个, 则不能被 5 整除的数共有有 $300-60-48=192$ 个;"} {"id": "22405", "image": [], "answer": "253", "solution": "null", "level": "高三", "question": "杨辉, 字谦光, 南宋时期杭州人, 在他 1261 年所著的一书中, 辑录了如图所示的角形数表, 称之为“开方作法本源”图, 并说明此表引自 11 世纪中叶(约公元 1050 年)贾宪的 “释锁算术》并绘画了“古法七乘方图”, 故此, 杨辉三角又被称为“贾宪三角”, 杨辉三角是一个由数字排列成的三角形数表, 一般形式如图: 基于上述规律, 可以推测, 当 $n=23$ 时, 从左往右第 22 个数为 \\$ \\qquad \\$ .\n\n![](https://cdn.mathpix.com/cropped/2024_04_19_7da760c0cd38339a5113g-", "options": [], "subject": "组合数学", "analysis": "当 $n=23$ 时, 共有 24 个数, 从左往右第 22 个数即为这一行的倒数第 3 个数, 观察可知, 其规律为 1,31,61,101,151,211,281,361,451,551,661,781,911,1051, $1201,1361,1531,1711,1901,2101,2311,253$, 故所求数字为 253."} {"id": "22406", "image": [], "answer": "1", "solution": "null", "level": "高三", "question": "将三项式 $\\left(x^{2}+x+1\\right)^{n}$ 展开,当 $n=0,1,2,3, \\cdots$ 时,得到以下等式:\n\n$\\left(x^{2}+x+1\\right)^{0}=1$\n\n$\\left(x^{2}+x+1\\right)^{1}=x^{2}+x+1$\n\n$\\left(x^{2}+x+1\\right)^{2}=x^{4}+2 x^{3}+3 x^{2}+2 x+1$\n\n$\\left(x^{2}+x+1\\right)^{3}=x^{6}+3 x^{5}+6 x^{4}+7 x^{3}+6 x^{2}+3 x+1$\n\n观察多项式系数之间的关系, 可以仿照杨辉三角构造如图所示的广义杨辉三角形, 其构造方法为: 第 0 行为 1 , 以下各行每个数是它正头顶上与左右两肩上 3 个数 (不足 3 个数的, 缺少的数记为 0 )的和, 第 $k$ 行共有 $2 k+1$ 个数, $\\left(x^{2}+x+1\\right)^{5}(1+a x)$ 在的展开式中, $x^{7}$ 项的系数为 75 ,则实数 $a$ 的值为\n\n![](https://cdn.mathpix.com/cropped/2024_04_19_7da760c0cd38339a5113g-", "options": [], "subject": "组合数学", "analysis": "根据题意可得广义杨辉三角第 5 行为 1,5,15,30,45,51,45,30,15,5,1,故 $(1+a x)\\left(x^{2}+x+1\right)^{5}$ 的展开式中, $x^{7}$ 项的系数为 $30+45 a=75$, 得 $a=1$.故答案为 1 ."} {"id": "22407", "image": [], "answer": "答案:2\n\n【解析】由图(2)可得: $S_{1}=1, S_{2}=2, S_{3}=2, S_{4}=4, S_{5}=2, S_{6}=4, S_{7}=4, S_{8}=8, \\ldots$,由此可以发现: 第一行 1 个 1 , 即第 $2^{1-1}$ 行有 $2^{1-1}$ 个 1 , 第二行 2 个 1 , 即第 $2^{2-1}$ 行有 $2^{2-1}$ 个 1 , 第四行 4 个 1 , 即第 $2^{3-1}$ 行有 $2^{3-1}$ 个 1 , 第八行 8 个 1 , 即第 $2^{4-1}$ 行有 $2^{4-1}$ 个 $1, \\ldots$, 因\n为 $32=2^{6-1}$, 所以第 $2^{6-1}$ 行有 $2^{6-1}$ 个 1 , 即第 32 行的数全为奇数, 由杨辉三角形的特点可得第 33 行应为 2 个奇数, 即 $S_{33}=2$.", "solution": "null", "level": "高三", "question": "我国南宋数学家杨辉所著的《详解九章算术》一书中, 用图(1)的数表列出了一些正整数在三角形中的一种几何排列, 俗称“杨辉三角形”, 该数表的规律是每行首尾数字均为 1 , 从第三行开始, 其余的数字是它 “上方” 左右两个数字之和。现将杨辉三角形中的奇数换成 1 , 偶数换成 0 , 得到图(2)所示的由数字 0 和 1 组成的三角形数表, 由上往下数, 记第 $n$ 行各数字的和为 $S_{n}$, 如 $S_{1}=1, S_{2}=2, S_{3}=2, S_{4}=4, S_{5}=2, \\ldots \\ldots$, 则 $S_{33}=$\n\n![](https://cdn.mathpix.com/cropped/2024_04_19_7da760c0cd38339a5113g-", "options": [], "subject": "组合数学", "analysis": "答案:2\n\n【解析】由图(2)可得: $S_{1}=1, S_{2}=2, S_{3}=2, S_{4}=4, S_{5}=2, S_{6}=4, S_{7}=4, S_{8}=8, \\ldots$,由此可以发现: 第一行 1 个 1 , 即第 $2^{1-1}$ 行有 $2^{1-1}$ 个 1 , 第二行 2 个 1 , 即第 $2^{2-1}$ 行有 $2^{2-1}$ 个 1 , 第四行 4 个 1 , 即第 $2^{3-1}$ 行有 $2^{3-1}$ 个 1 , 第八行 8 个 1 , 即第 $2^{4-1}$ 行有 $2^{4-1}$ 个 $1, \\ldots$, 因\n为 $32=2^{6-1}$, 所以第 $2^{6-1}$ 行有 $2^{6-1}$ 个 1 , 即第 32 行的数全为奇数, 由杨辉三角形的特点可得第 33 行应为 2 个奇数, 即 $S_{33}=2$."} {"id": "22408", "image": [], "answer": "答案:$n^{2}+2$\n\n解析:", "solution": "null", "level": "高三", "question": "如图, 一个类似杨辉三角的数阵, 请写出第 $n(n \\geq 2)$ 行的第 2 个数为\n\n![](https://cdn.mathpix.com/cropped/2024_04_19_7da760c0cd38339a5113g-", "options": [], "subject": "组合数学", "analysis": "答案:$n^{2}+2$\n\n解析:"} {"id": "21078", "image": [], "answer": "(1)(4)", "solution": "null", "level": "高三", "question": "已知 $\\alpha, \\beta, \\gamma$ 是三个平面, $\\mathrm{m}, \\mathrm{n}$ 是两条直线, 有下列四个命题:\n\n(1)如果 $\\mathrm{m} \\perp \\alpha, \\mathrm{m} \\subset \\beta$ ,那么 $\\alpha \\perp \\beta$;\n\n(2)如果 $\\mathrm{m} \\perp \\mathrm{n}, \\mathrm{m} \\perp \\alpha$ ,那么 $\\mathrm{n} / / \\alpha$;\n\n(3)如果 $\\alpha \\perp \\beta, \\mathrm{m} / / \\alpha$, 那么 $\\mathrm{m} \\perp \\beta$;\n\n(4)如果 $\\alpha / / \\beta, \\alpha \\cap \\gamma=m, \\beta \\cap \\gamma=n$, 那么 $m / / n$.\n\n其中正确的命题有 . (写出所有正确命题的序号)", "options": [], "subject": "立体几何学", "analysis": "(1)由面面垂直的判定定理可知, 如果 $m \\perp \\alpha, m \\subset \beta$, 那么 $\\alpha \\perp \beta$; 故(1)正确.(2)如果 $\\mathrm{m} \\perp \\mathrm{n}, \\mathrm{m} \\perp \\alpha$ ,那么 $\\mathrm{n} / / \\alpha$ 或 $\\mathrm{n} \\subset \\alpha$; 故(2)错误;(3)如果 $\\alpha \\perp \beta, m / / \\alpha$, 那么 $m$, $\beta$ 位置关系不确定; 故(3)错误;(4)如果 $\\alpha / / \beta, \\alpha \\cap \\gamma=m, \beta \\cap \\gamma=n$, 那么 $m / / n$. 正确.故答案为(1)(4)."} {"id": "21091", "image": [], "answer": "(1)(4)", "solution": "null", "level": "高三", "question": "已知 $\\alpha, \\beta, \\gamma$ 是三个平面, $m, n$ 是两条直线, 有下列四个命题:\n\n(1)如果 $\\mathrm{m} \\perp \\alpha, \\mathrm{m} \\subset \\beta$, 那么 $\\alpha \\perp \\beta$;\n\n(2)如果 $\\mathrm{m} \\perp \\mathrm{n}, \\mathrm{m} \\perp \\alpha$, 那么 $\\mathrm{n} / / \\alpha$;\n\n(3)如果 $\\alpha \\perp \\beta, \\mathrm{m} / / \\alpha$, 那么 $\\mathrm{m} \\perp \\beta$;\n\n(4)如果 $\\alpha / / \\beta, \\alpha \\cap \\gamma=m, \\beta \\cap \\gamma=n$, 那么 $m / / n$.\n\n其中正确的命题有 . (写出所有正确命题的序号)", "options": [], "subject": "立体几何学", "analysis": "(1)由面面垂直的判定定理可知, 如果 $\\mathrm{m} \\perp \\alpha, \\mathrm{m} \\subset \beta$, 那么 $\\alpha \\perp \beta$; 故(1)正确.(2)如果 $\\mathrm{m} \\perp \\mathrm{n}, \\mathrm{m} \\perp \\alpha$ ,那么 $\\mathrm{n} / / \\alpha$ 或 $\\mathrm{n} \\subset \\alpha$; 故(2)错误;(3)如果 $\\alpha \\perp \beta, \\mathrm{m} / / \\alpha$ ,那么 $\\mathrm{m}, \beta$ 位置关系不确定; 故(3)错误;(4)如果 $\\alpha / / \beta, \\alpha \\cap \\gamma=m, \beta \\cap \\gamma=n$, 那么 $m / / n$. 正确.故答案为(1)(4)."} {"id": "22540", "image": ["9860.jpg"], "answer": "答案: (1)(4)", "solution": "null", "level": "高三", "question": "15. 如图, 在正方体 $A B C D-A_{1} B_{1} C_{1} D_{1}$ 中, $P$ 为 $B D_{1}$ 的中点, 则 $\\triangle P A C$ 在该正方体各个面上的射影可能是 \\$ \\qquad \\$ .\n\n", "options": [], "subject": "立体几何学", "analysis": ":解答:由所给的正方体知, $\\triangle \\mathrm{PAC}$ 在该正方体上下面上的射影是(1), $\\triangle \\mathrm{PAC}$ 在该正方体左右面上的射影是(4), $\\triangle \\mathrm{PAC}$ 在该正方体前后面上的射影是(4)故答案为: (1) (4)\n\n分析: 本题主要考查了直角三角形的射影定理, 解决问题的关键是根据直角三角形的射影定理根据点的投影的做法, 做出 $\\triangle \\mathrm{PAC}$ 在该正方体各个面上的射影, 这里应该有三种情况, 做出在前后面上的投影, 在上下面上的投影, 在左右面上的投影, 得到结果."} {"id": "22862", "image": [], "answer": "答案: 3", "solution": "null", "level": "高三", "question": "7. 已知柱坐标系 $O x y z$ 中, 点 $M$ 的柱坐标为 $\\left(2, \\frac{\\pi}{3}, \\sqrt{5}\\right)$, 则 $|O M|=$", "options": [], "subject": "立体几何学", "analysis": ":解答:因为 $(\\rho, \\theta, z)=\\left(2, \\frac{\\pi}{3}, \\sqrt{5}\\right)$,\n\n设 $M$ 的直角坐标为 $(x, y, z)$, 则 $x^{2}+y^{2}=\\rho^{2}=4$, 所以\n\n$|O M|=\\sqrt{x^{2}+y^{2}+z^{2}}=\\sqrt{4+(\\sqrt{5})^{2}}=3$.\n\n分析: 本题主要考查了柱.球坐标系与空间直角坐标系的区别, 解决问题的关键是根据柱.球坐标系与空间直角坐标系的区别转化计算即可"} {"id": "22863", "image": [], "answer": "答案: $(3,3 \\sqrt{3}, 0)$", "solution": "null", "level": "高三", "question": "8. 点 $M$ 的球坐标为 $\\left(6, \\frac{\\pi}{2}, \\frac{\\pi}{3}\\right)$, 则它的直角坐标为", "options": [], "subject": "立体几何学", "analysis": ": 解答: $x=6 \\cdot \\sin \\frac{\\pi}{2} \\cdot \\cos \\frac{\\pi}{3}=3, y=6 \\sin \\frac{\\pi}{2} \\cdot \\sin \\frac{\\pi}{3}=3 \\sqrt{3}, z=6 \\cos \\frac{\\pi}{2}=0, \\therefore$ 它的直角坐标为 $(3,3 \\sqrt{3}, 0)$.\n\n分析: 本题主要考查了球坐标刻画点的位置, 解决问题的关键是转化为直角坐标即可"} {"id": "23009", "image": [], "answer": "答案: $\\frac{5 \\sqrt{3}}{6} \\pi$", "solution": "null", "level": "高三", "question": "13. 正方体 $A B C D-A_{1} B_{1} C_{1} D_{1}$ 的棱长为 1 , 在正方体的表面上与点 $A$ 距离为 $\\frac{2 \\sqrt{3}}{3}$ 的点形成一条曲线, 这条曲线的长度为", "options": [], "subject": "立体几何学", "analysis": ":【解答】这条曲线在面 $A D D_{1} A_{1}$ 上的一段是以 $A$ 为圆心, $\\frac{2 \\sqrt{3}}{3}$ 为半径, 管为圆心角的一段圆弧, 在面 $A_{1} B_{1} C_{1} D_{1}$ 上的一段是以 $A_{1}$ 为圆心, $\\frac{\\sqrt{3} \\sqrt{3}}{3} \\frac{1}{3}$ 为半径, $\\frac{\\pi}{2}$ 为圆心角的一段圆弧, 由正方体的对称性知, 这条曲线的长度为 $3\\left(\\frac{\\pi}{6} \\cdot \\frac{2 \\sqrt{3}}{3}+\\frac{\\pi}{2} \\cdot \\frac{\\sqrt{3}}{3}\\right)=\\frac{5 \\sqrt{3}}{6} \\pi$\n\n【分析】本题主要考查了分析法与综合法, 解决问题的关键是根据所给条件分析得到曲线在\n面 $A D D_{1} A_{1}$ 上的一段是以 $A$ 为圆心, $\\frac{2 \\sqrt{3}}{3}$ 为半径, $\\frac{\\pi}{6}$ 为圆心角的一段圆弧, 然后计算即可."} {"id": "22494", "image": ["9803.jpg", "9804.jpg"], "answer": "答案: $\\frac{10}{3}$\n\n.", "solution": "null", "level": "高三", "question": "16. 如图, $\\mathrm{AB} / / \\mathrm{CD} / / \\mathrm{EF}, \\mathrm{AF}, \\mathrm{BE}$ 相交于点 $\\mathrm{O}$, 若 $\\mathrm{AO}=\\mathrm{OD}=\\mathrm{DF}, \\mathrm{BE}=10 \\mathrm{~cm}$, 则 $\\mathrm{BO}=$ $\\mathrm{cm}$.\n\n", "options": [], "subject": "组合几何学", "analysis": ": 解答: 如图,过点 $\\mathrm{O}$ 作 $1 / / \\mathrm{AB}$, 则 $1 / / \\mathrm{AB} / / \\mathrm{CD} / / \\mathrm{EF} \\because \\mathrm{AO}=\\mathrm{OD}=\\mathrm{DF}, \\therefore \\mathrm{BO}=\\mathrm{OC}=\\mathrm{CE}, \\therefore$ $\\mathrm{BO}=\\frac{1}{3} \\mathrm{BE}=\\frac{10}{3} \\mathrm{~cm}$.\n\n\n\n分析: 本题主要考查了平行线等分线段定理, 解决问题的关键是根据平行线等分线段定理计算即可."} {"id": "22495", "image": ["9805.jpg"], "answer": "答案: 6", "solution": "null", "level": "高三", "question": "17. 如图, 在正方形 $A^{\\prime} B^{\\prime} C^{\\prime} D^{\\prime}$ 中, $O^{\\prime}$ 是两条对角线 $A^{\\prime} C^{\\prime}$ 与 $B^{\\prime} D^{\\prime}$ 的交点, 作 $O^{\\prime} F^{\\prime} / / C^{\\prime} D^{\\prime}$ 交 $A^{\\prime} D^{\\prime}$ 于点 $F^{\\prime}$, 且正方形边长等于 12 , 则 $A^{\\prime} F^{\\prime}=$\n\n", "options": [], "subject": "组合几何学", "analysis": ": 解答: 因为四边形 $A^{\\prime} B^{\\prime} C^{\\prime} D^{\\prime}$ 是正方形,$O^{\\prime}$ 是 $A^{\\prime} C^{\\prime}$ 与 $B^{\\prime} D^{\\prime}$ 的交点, 所以 $A^{\\prime} O^{\\prime}=O^{\\prime} C^{\\prime}$. 又因为 $O^{\\prime} F^{\\prime} / / C^{\\prime} D^{\\prime}$, 所以 $A^{\\prime} F^{\\prime}=F^{\\prime} D^{\\prime}$, 即 $A^{\\prime} F^{\\prime}=\\frac{1}{2} A^{\\prime} D^{\\prime}=\\frac{1}{2} \\times 12=6$.\n\n分析: 本题主要考查了平行线等分线段定理, 解决问题的关键是根据平行线等分线段定理计算即可."} {"id": "22496", "image": ["9806.jpg"], "answer": "答案: 8", "solution": "null", "level": "高三", "question": "18. 在 $\\triangle \\mathrm{ABC}$ 中, $\\mathrm{AD}$ 是 $\\mathrm{BC}$ 边上的中线, $\\mathrm{M}$ 是 $\\mathrm{AD}$ 的中点, $\\mathrm{BM}$ 的延长线交 $\\mathrm{AC}$ 于点 $\\mathrm{N}, \\mathrm{AN}=4 \\mathrm{~cm}$,\n则 $\\mathrm{CN}=$ $\\mathrm{cm}$.", "options": [], "subject": "度量几何学", "analysis": ":解答:如图, 过点 $\\mathrm{D}$ 作 $\\mathrm{DE} / / \\mathrm{BN}$, 交 $\\mathrm{AC}$ 于点 $\\mathrm{E} . \\because \\mathrm{D}$ 为 $\\mathrm{BC}$ 的中点, $\\therefore \\mathrm{NE}=\\mathrm{EC}$. 又 $\\because \\mathrm{M}$ 为 $\\mathrm{AD}$ 的中点, $\\mathrm{MN} / / \\mathrm{DE}, \\therefore \\mathrm{AN}=\\mathrm{NE}, \\therefore \\mathrm{AN}=\\mathrm{NE}=\\mathrm{EC} . \\therefore \\mathrm{CN}=2 \\mathrm{AN}=8 \\mathrm{~cm}$.\n\n\n\n分析: 本题主要考查了平行线等分线段定理, 解决问题的关键是根据平行线等分线段定理方向计算即可."} {"id": "22533", "image": ["9854.jpg", "9855.jpg"], "answer": "7", "solution": "null", "level": "高三", "question": "9. 如图,在梯形 $\\mathrm{ABCD}$ 中, $\\mathrm{AD} / / \\mathrm{EF} / / \\mathrm{BC}, \\mathrm{BE}=3 \\mathrm{AE}, \\mathrm{AD}=3, \\mathrm{EF}=4$,则 $\\mathrm{BC}=$\n\n", "options": [], "subject": "度量几何学", "analysis": ": 解答: 如图,\n\n\n\n分别取 $\\mathrm{AB}, \\mathrm{CD}$ 的中点 $\\mathrm{G}, \\mathrm{H}$, 连接 $\\mathrm{GH}$,\n\n则 $\\mathrm{GH}$ 为梯形 $\\mathrm{ABCD}$ 的中位线, $\\mathrm{EF}$ 为梯形 $\\mathrm{AGHD}$ 的中位线,\n\n故 $\\mathrm{GH}=2 \\mathrm{EF}-\\mathrm{AD}=2 \\times 4-3=5, \\mathrm{BC}=2 \\mathrm{GH}-\\mathrm{AD}=2 \\times 5-3=7$.\n\n分析: 本题主要考查了平行线分线段成比例定理, 解决问题的关键是根据平行线分线段成比例定理分析计算即可"} {"id": "22783", "image": ["10074.jpg"], "answer": "3", "solution": "null", "level": "高三", "question": "11. 在极坐标系中, 已知两点 $A, B$ 的极坐标分别为 $\\left(3, \\frac{\\pi}{3}\\right),\\left(4, \\frac{\\pi}{6}\\right)$, 则 $\\triangle A O B$ (其中 0 为极点)的面积为", "options": [], "subject": "度量几何学", "analysis": ": 解答: 如图所示, $|O A|=3,|O B|=4, \\angle A O B=\\frac{\\pi}{6}$ ,所以 $S_{\\triangle A O B}=\\frac{1}{2} \\times 3 \\times 4 \\times \\frac{1}{2}=3$.\n\n\n\n分析: 本题主要考查了点的极坐标和直角坐标的互化, 解决问题的关键是根据极坐标与直角坐标系关系计算即可"} {"id": "22535", "image": ["9856.jpg"], "answer": "答案: $\\frac{1}{3}$", "solution": "null", "level": "高三", "question": "10. 在 Rt $\\triangle \\mathrm{ACB}$ 中, $\\angle \\mathrm{C}=90^{\\circ}, \\mathrm{CD} \\perp \\mathrm{AB}$ 于 $\\mathrm{D}$, 若 $\\mathrm{BD}: \\mathrm{AD}=1: 9$, 则 $\\tan \\angle \\mathrm{BCD}=$\n\n", "options": [], "subject": "度量几何学", "analysis": ":解答:在 Rt $\\triangle \\mathrm{ACB}$ 中, $\\mathrm{CD} \\perp \\mathrm{AB}$, 由射影定理得 $\\mathrm{CD}^{2}=\\mathrm{AD} \\cdot \\mathrm{BD}$.\n\n又 $B D : A D=1: 9$, 令 $B D=x$, 则 $A D=9 x ( x>0 ) . \\therefore D^{2}=9 x^{2} . \\therefore C D=3 x$.\n\n在 Rt $\\triangle \\mathrm{CDB}$ 中, $\\tan \\angle \\mathrm{BCD}=\\frac{\\mathrm{BD}}{\\mathrm{CD}}=\\frac{\\mathrm{x}}{3 \\mathrm{x}}=\\frac{1}{3}$\n\n分析: 本题主要考查了直角三角形的射影定理, 解决问题的关键是根据直角三角形的射影定\n理结合所给条件计算即可"} {"id": "22536", "image": ["9857.jpg"], "answer": ": 答案: $2 \\sqrt{3}$\n\n", "solution": "null", "level": "高三", "question": "11. 如图, Rt $\\triangle A B P$ 中, $\\angle A B P=90^{\\circ}, B C \\perp A P$, 垂足为点 $C$, 且 $A B=2 \\sqrt{6}, A C=4$, 则 $P B=$", "options": [], "subject": "度量几何学", "analysis": ": 解答: $\\because$ 在 Rt $\\triangle \\mathrm{ABP}$ 中, $\\angle \\mathrm{ABP}=90^{\\circ}, \\mathrm{BC} \\perp \\mathrm{AP}, \\therefore \\mathrm{AB}=\\mathrm{AC} \\cdot \\mathrm{AP}$, 即 $(2 \\sqrt{6})^{2}=4 \\mathrm{AP}$, 解得 $\\mathrm{AP}=6$.\n\n在 Rt $\\triangle \\mathrm{ABP}$ 中,由勾股定理, 得 $\\mathrm{BP}=\\sqrt{\\mathrm{AP}^{2}-\\mathrm{AB}^{2}}=\\sqrt{6^{2}-(2 \\sqrt{6})^{2}}=2 \\sqrt{3}$.\n\n分析: 本题主要考查了直角三角形的射影定理, 解决问题的关键是根据直角三角形的射影定理集合所给条件运用勾股定理计算即可"} {"id": "22538", "image": ["9858.jpg", "9859.jpg"], "answer": "8", "solution": "null", "level": "高三", "question": "13. 如图, 圆 $O$ 上一点 $C$ 在直径 $A B$ 上的射影为 $D$, 点 $D$ 在半径 $O C$ 上的射影为 $E$. 若 $A B=3 A D$,则 $\\frac{\\mathrm{CE}}{\\mathrm{EO}}$ 的值为\n\n", "options": [], "subject": "度量几何学", "analysis": ":\n\n解答: 设 $A D=2$, 则 $A B=6$,于是 $B D=4, O D=1$.\n\n\n\n如图,由射影定理得\n$\\mathrm{CD}^{2}=\\mathrm{AD} \\cdot \\mathrm{BD}=8$,\n\n则 $\\mathrm{CD}=2 \\sqrt{2}$.\n\n在 Rt $\\triangle \\mathrm{OCD}$ 中, $\\mathrm{DE}=\\frac{\\mathrm{OD} \\cdot \\mathrm{CD}}{\\mathrm{OC}}=\\frac{1 \\times 2 \\sqrt{2}}{3}=\\frac{2 \\sqrt{2}}{3}$则 $\\mathrm{CE}=\\sqrt{\\mathrm{DC}^{2}-\\mathrm{DE}^{2}}=\\sqrt{8-\\frac{8}{9}}=\\frac{8}{3}$, $\\mathrm{EO}=\\mathrm{OC}-\\mathrm{CE}=3-\\frac{8}{3}=\\frac{1}{3}$.因此 $\\frac{\\mathrm{CE}}{\\mathrm{EO}}=\\frac{\\frac{8}{3}}{\\frac{1}{3}}=8$.\n\n分析: 本题主要考查了直角三角形的射影定理, 解决问题的关键是根据直角三角形的射影定理结合所给三角形满足的条件分析计算即可"} {"id": "22541", "image": ["9861.jpg"], "answer": "答案: $\\frac{\\sqrt{3}}{2}$", "solution": "null", "level": "高三", "question": "16. 如图, Rt $\\triangle A B C$ 中, $\\angle C=90^{\\circ}, C D \\perp A B$ 于 $D, B C=\\sqrt{3}, A C=3$, 则 $B D=$ \\$ \\qquad \\$ .\n\n", "options": [], "subject": "度量几何学", "analysis": ": 解答: 由勾股定理得 $A B=\\sqrt{B C^{2}+A C^{2}}=\\sqrt{3+9}=2 \\sqrt{3}$. 由直角三角形射影定理, $\\mathrm{BC}^{2}=\\mathrm{BD} \\times \\mathrm{BA}, \\quad 3=2 \\sqrt{3} \\times \\mathrm{BD}, \\quad \\mathrm{BD}=\\frac{\\sqrt{3}}{2}$\n\n故答案为: $\\frac{\\sqrt{3}}{2}$\n\n分析: 本题主要考查了直角三角形的射影定理, 解决问题的关键是根据直角三角形的射影定理应用勾股定理先求出 $\\mathrm{AB}$, 再由直角三角形射影定理, $\\mathrm{BC}^{2}=\\mathrm{BD} \\times \\mathrm{BA}$ 代入数据求出 $\\mathrm{BD}$."} {"id": "22542", "image": ["9862.jpg"], "answer": "答案: 5", "solution": "null", "level": "高三", "question": "17. 如图所示, 圆 0 上一点 $C$ 在直径 $A B$ 上的射影为 $D, C D=4, B D=8$, 则圆 0 的半径等于\n\n", "options": [], "subject": "度量几何学", "analysis": ": 解答: $\\mathrm{AB}$ 为圆的直径, $\\therefore \\angle \\mathrm{ACB}=90^{\\circ}$\n\n在 Rt $\\triangle \\mathrm{ABC}$ 中由射影定理可知 $\\mathrm{CD}^{2}=\\mathrm{BD} \\times \\mathrm{AD}, \\therefore 16=8 \\times \\mathrm{AD}$,\n\n$\\therefore \\mathrm{AD}=2, \\quad \\therefore$ 半径 $=5$\n\n故答案为: 5\n\n分析: 本题主要考查了直角三角形的射影定理, 解决问题的关键是根据直角三角形的射影定理先利用 $A B$ 为圆的直径, 判断出 $\\triangle A B C$ 为直角三角形, 进而利用射影定理求得 $A D$, 最后根据 $\\mathrm{AB}=\\mathrm{AD}+\\mathrm{BD}$ 求得 $\\mathrm{AB}$, 则圆的半径可求."} {"id": "22558", "image": ["9878.jpg"], "answer": "答案: $\\frac{9}{2}$", "solution": "null", "level": "高三", "question": "9. 如图, 在 Rt $\\triangle A B C$ 中, $A C \\perp B C$, 点 $C$ 在 $A B$ 上的正射影为点 $D$, 且 $A C=3, A D=2$, 则 $A B=$\n\n", "options": [], "subject": "度量几何学", "analysis": ": 解答: $\\because \\mathrm{AC} \\perp \\mathrm{CB}$, 又 $\\because$ 点 $\\mathrm{D}$ 是点 $\\mathrm{C}$ 在 $\\mathrm{AB}$ 上的正射影, $\\therefore \\mathrm{CD} \\perp \\mathrm{AB}, \\therefore \\mathrm{AC}=\\mathrm{AD} \\cdot \\mathrm{AB}$.\n\n$$\n\\text { 又 } \\because A C=3, A D=2, \\therefore A B=\\frac{A C^{2}}{A D}=\\frac{9}{2}\n$$\n\n分析: 本题主要考查了直角三角形的射影定理, 解决问题的关键是根据直角三角形的射影定理结合所给几何关系分析计算即可"} {"id": "20480", "image": [], "answer": "答案: $\\frac{1}{16}$", "solution": "null", "level": "高三", "question": "20. 已知 $\\cos \\frac{\\pi}{3}=\\frac{1}{2}, \\cos \\frac{\\pi}{5} \\cos \\frac{2 \\pi}{5}=\\frac{1}{4}, \\cos \\frac{\\pi}{7} \\cos \\frac{2 \\pi}{7} \\cos \\frac{3 \\pi}{7}=\\frac{1}{8}, \\ldots$, 根据这些结果,猜想 $\\cos \\frac{\\pi}{9} \\cos \\frac{2 \\pi}{9} \\cos \\frac{3 \\pi}{9} \\cos \\frac{4 \\pi}{9}=$", "options": [], "subject": "度量几何学", "analysis": ": 解答: 根据题意, 分析所给的等式可得: $\\cos \\frac{\\pi}{3}=\\frac{1}{2}$, 可化为 $\\cos \\frac{1 \\times \\pi}{2 \\times 1+1}=\\frac{1}{2^{1}}$, $\\cos \\frac{\\pi}{5} \\cos \\frac{2 \\pi}{5}=\\frac{1}{4}$, 可化为 $\\cos \\frac{1 \\times \\pi}{2 \\times 2+1} \\cos \\frac{2 \\times \\pi}{2 \\times 2+1}=\\frac{1}{2^{2}}, \\cos \\frac{\\pi}{7} \\cos \\frac{2 \\pi}{7} \\cos \\frac{3 \\pi}{7}=\\frac{1}{8}$,可化为 $\\cos \\frac{1 \\times \\pi}{2 \\times 3+1} \\cos \\frac{2 \\times \\pi}{2 \\times 3+1} \\cos \\frac{3 \\times \\pi}{2 \\times 3+1}=\\frac{1}{2^{3}}$;\n\n则一般的结论为 $\\cos \\frac{\\pi}{2 n+1} \\cos \\frac{2 \\pi}{2 n+1} \\ldots \\cos \\frac{n \\pi}{2 n+1}=\\frac{1}{2^{n}}$ ;\n\n$\\therefore \\cos \\frac{\\pi}{9} \\cos \\frac{2 \\pi}{9} \\cos \\frac{3 \\pi}{9} \\cos \\frac{4 \\pi}{9}=\\cos \\frac{\\pi}{2 \\times 4+1} \\cos \\frac{2 \\pi}{2 \\times 4+1} \\cos \\frac{3 \\pi}{2 \\times 4+1} \\cos \\frac{4 \\pi}{2 \\times 4+1}=$\n\n$$\n\\frac{1}{2^{4}}=\\frac{1}{16}\n$$\n\n分析: 本题主要考查了归纳推理, 解决问题的关键是根据所给式子的特征进行归纳得到对应的规律即可."} {"id": "22560", "image": ["9880.jpg"], "answer": "答案: 105", "solution": "null", "level": "高三", "question": "10. 如图, 若 $\\angle \\mathrm{BAD}=75^{\\circ}$, 则 $\\angle \\mathrm{BCD}=$\n\n", "options": [], "subject": "度量几何学", "analysis": ":解答: $\\mathrm{BAD}$ 是 $\\mathrm{BCD}$ 所对的圆周角, $\\angle \\mathrm{BCD}$ 是 $\\mathrm{BAD}$ 所对的圆周角, 则 $\\mathrm{BCD}$ 所对的圆心角为 $2 \\times 75^{\\circ}=150^{\\circ}$.\n\n又 $\\because \\mathrm{BCD}$ 和 $\\mathrm{BAD}$ 所对圆心角的和是周角 $360^{\\circ}$,\n\n$\\therefore \\mathrm{BAD}$ 所对圆心角是 $360^{\\circ}-150^{\\circ}=210^{\\circ}$,\n\n$\\therefore \\mathrm{BAD}$ 所对圆周角 $\\angle \\mathrm{BCD}=\\frac{1}{2} \\times 210^{\\circ}=105^{\\circ}$.\n\n分析: 本题主要考查了圆周角定理, 解决问题的关键是根据圆周角定理结合所给条件计算即可"} {"id": "22763", "image": [], "answer": "答案: $\\frac{x^{2}}{2}-y^{2}=1$", "solution": "null", "level": "高三", "question": "15. 已知 $A(2,1), B(-1,1), O$ 为坐标原点,动点 $M$ 满足 $\\overrightarrow{O M}=m \\overrightarrow{O A}+n \\overrightarrow{O B}$, 其中 $m, n \\in \\mathbf{R}$, 且 $2 m^{2}-n^{2}=2$, 则 $M$ 的轨迹方程为", "options": [], "subject": "解析几何", "analysis": ":解答:设 $M(x, y)$ ,\n\n则 $(x, y)=m(2,-1)+n(-1,1)=(2 m-n, n-m)$,\n\n$\\therefore\\left\\{\\begin{array}{l}x=2 m-n_{s} \\\\ y=n-m .\\end{array}\\right.$ 又 $2 m^{2}-n^{2}=2$, 消去 $m, n$ 得 $\\frac{x^{2}}{2}-y^{2}=1$.\n\n分析: 本题主要考查了平面直角坐标系与曲线方程, 解决问题的关键是根据所给条件求解曲线方程即可"} {"id": "22784", "image": [], "answer": "答案: $\\theta=\\frac{\\pi}{2}+\\alpha$", "solution": "null", "level": "高三", "question": "12. 直线 $x \\cos a+y \\sin a=0$ 的极坐标方程为", "options": [], "subject": "解析几何", "analysis": ":解答: $\\rho \\cos \\theta \\cos \\alpha+\\rho \\sin \\theta \\sin \\alpha=0, \\cos (\\theta-\\alpha)=0$, 取 $\\theta-\\alpha=\\frac{\\pi}{2}$\n\n分析: 本题主要考查了极坐标系, 解决问题的关键是根据所给方程化为极坐标即可"} {"id": "22785", "image": [], "answer": ":答案: $(\\sqrt{2}, \\sqrt{2})$", "solution": "null", "level": "高三", "question": "13. 在直角坐标系 $\\mathrm{xOy}$ 中, 以 $\\mathrm{O}$ 为极点, $\\mathrm{x}$ 正半轴为极轴建立极坐标系,且在两种坐标系中取相同的单位长度,将点 $\\mathrm{P}$ 的极坐标 $\\left(2, \\frac{\\pi}{4}\\right)$ 化成直角坐标", "options": [], "subject": "解析几何", "analysis": ": 解答: 本题主要考查了极坐标与直角坐标的互化, 由点 $\\mathrm{P}$ 的极坐标为 $\\left(2, \\frac{\\pi}{4}\\right)$, 设点 $\\mathrm{P}$的直角坐标为 $(\\mathrm{x}, \\mathrm{y})$,所以 $x=2 \\cos \\frac{\\pi}{4}=\\sqrt{2}, y=2 \\sin \\frac{\\pi}{4}=\\sqrt{2}$, 故填 $(\\sqrt{2}, \\sqrt{2})$\n\n分析: 本题主要考查了点的极坐标和直角坐标的互化, 解决问题的关键是根据点的极坐标和直角坐标的互化分析计算即可"} {"id": "22786", "image": [], "answer": "答案: $2 \\sqrt{3}$", "solution": "null", "level": "高三", "question": "14. 在极坐标系中, 若过点 $(3,0)$ 且与极轴垂直的直线交曲线 $\\rho=4 \\cos \\theta$ 于 $\\mathrm{A} . \\mathrm{~B}$ 两点, 则 $|A B|=$ \\$ \\qquad \\$ .", "options": [], "subject": "解析几何", "analysis": ": 解答: 本题考查极坐标系与直角坐标系的互化, 意在考查考生的运算求解能力. 曲线 $\\rho=4 \\cos \\theta$ 化为直角坐标方程为 $x^{2}+y^{2}-4 x=0$, 过点 $(3,0)$ 且与极轴垂直的直线为 $x=3$ 直线与圆联立得 $A(3, \\sqrt{3}), B(3,-\\sqrt{3})$, 故 $|A B|=2 \\sqrt{3}$. 故本题正确答案为 $2 \\sqrt{3}$.\n\n分析: 本题主要考查了点的极坐标和直角坐标的互化, 解决问题的关键是根据点的极坐标和直角坐标的互化分析计算即可"} {"id": "22987", "image": [], "answer": "答案: $-2 \\leq a \\leq 4$", "solution": "null", "level": "高三", "question": "16. 若存在实数 $x$ 使 $|x-a|+|x-1| \\leq 3$ 成立, 则实数 $a$ 的取值范围是", "options": [], "subject": "解析几何", "analysis": ": 解答: 在数轴上, $|x-a|$ 表示横坐标为 $X$ 的点 $P$ 到横坐标为 $\\mathrm{a}$ 的点 $\\mathrm{A}$ 距离, $|x-1|$ 就表示点 $P$ 到横坐标为 1 的点 $\\mathrm{B}$ 的距离, 所以 $(|P A|+|P B|)_{\\text {min }}=|a-1|$,\n\n从而 $|a-1| \\leq 3$, 解得 $-2 \\leq a \\leq 4$. 故答案为 $-2 \\leq a \\leq 4$.\n\n分析: 本题主要考查了绝对值不等式的解法, 解决问题的关键是为使存在实数 $X$ 使 $|x-a|+|x-1| \\leq 3$ 成立, 只需 $|\\mathrm{x}-\\mathrm{a}|+|\\mathrm{x}-1|$ 的最小值满足不大于"} {"id": "22804", "image": [], "answer": "答案: $\\theta=\\frac{\\pi}{6}$ 或 $\\theta=\\frac{7 \\pi}{6}$", "solution": "null", "level": "高三", "question": "10. 直线 $\\frac{\\sqrt{3}}{3} x-y=0$ 的极坐标方程为", "options": [], "subject": "解析几何", "analysis": ": 解答: 直线方程 $\\frac{\\sqrt{3}}{3} x-y=0$ 变为 $\\frac{\\sqrt{3}}{3} \\rho \\cos \\theta-\\rho \\sin \\theta=0$, 即 $\\frac{\\sqrt{3}}{3} \\cos \\theta-\\sin \\theta=0$,故 $\\tan \\theta=\\frac{\\sqrt{3}}{3}$, 故 $\\theta=\\frac{\\pi}{6}$ 或 $\\theta=\\frac{7 \\pi}{6}$.所以直线 $\\frac{\\sqrt{3}}{3} x-y=0$ 的极坐标方程为 $\\theta=\\frac{\\pi}{6}$ 或 $\\theta=\\frac{7 \\pi}{6}$.\n\n分析: 本题主要考查了简单曲线的极坐标方程, 解决问题的关键是将所给普通方程转化即可"} {"id": "22805", "image": [], "answer": "答案: $a=\\frac{\\sqrt{2}}{2}$", "solution": "null", "level": "高三", "question": "11. 在极坐标系中, 曲线 $C_{1}: \\rho(\\sqrt{2} \\cos \\theta+\\sin \\theta)=1$ 与曲线 $C_{2}: \\rho=a(a>0)$ 的一个交点在极轴上, 则 $a=$", "options": [], "subject": "解析几何", "analysis": ": 解答: 曲线 $C_{1}$ 的直角坐标方程是 $\\sqrt{2} x+y=1$, 曲线 $C_{2}$ 的直角坐标方程是 $x^{2}+y^{2}=a^{2}$,因为曲线 $C_{1}$ 与曲线 $C_{2}$ 的一个交点在极轴上, 所以 $C_{1}$ 与 $\\mathrm{x}$ 轴交点的横坐标与 $\\mathrm{a}$ 值相等, 由 $y=0, x=\\frac{\\sqrt{2}}{2}$, 知 $a=\\frac{\\sqrt{2}}{2}$.\n\n分析: 本题主要考查了简单曲线的极坐标方程, 解决问题的关键是转化为普通方程分析判定即可"} {"id": "22806", "image": [], "answer": "答案: $\\rho \\cos \\theta=\\frac{3}{2}$", "solution": "null", "level": "高三", "question": "12. 在极坐标系中, 过点 $P\\left(3, \\frac{\\pi}{3}\\right)$ 且垂直于极轴的直线的极坐标方程是", "options": [], "subject": "解析几何", "analysis": ": 解答: 因 $3 \\times \\cos \\frac{\\pi}{3}=\\frac{3}{2}$, 故过点 $P\\left(3, \\frac{\\pi}{3}\\right)$ 且垂直于极轴的直线与极轴的交点 $\\left(\\frac{3}{2}, 0\\right)$,因为过 $\\left(\\frac{3}{2}, 0\\right)$ 且垂直于极轴的直线的极坐标方程为 $\\rho \\cos \\theta=\\frac{3}{2}$.故过 $P\\left(3, \\frac{\\pi}{3}\\right)$ 且垂直于极轴的直线的极坐标方程是 $\\rho \\cos \\theta=\\frac{3}{2}$分析: 本题主要考查了简单曲线的极坐标方程, 解决问题的关键是根据所给极坐标关系分析\n即可"} {"id": "22807", "image": [], "answer": "答案: $\\rho \\cos \\theta=2$", "solution": "null", "level": "高三", "question": "13. 在极坐标系中, 过点 $\\left(2 \\sqrt{2}, \\frac{\\pi}{4}\\right)$ 作圆 $\\rho=4 \\sin \\theta$ 的切线, 则切线的极坐标方程为", "options": [], "subject": "解析几何", "analysis": ": 解答: 点 $\\left(2 \\sqrt{2}, \\frac{\\pi}{4}\\right)$ 的直角坐标为 $(2,2)$, 圆 $\\rho=4 \\sin \\theta$ 的直角坐标方程为 $x^{2}+y^{2}-4 y=0$, 配方, 得 $x^{2}+(y-2)^{2}=4$, 所以过点 $(2,2)$ 的圆的切线方程为 $x=2$,化为极坐标方程为 $\\rho \\cos \\theta=2$.\n\n分析: 本题主要考查了简单曲线的极坐标方程, 解决问题的关键是转化为普通方程分析即可"} {"id": "22830", "image": [], "answer": "答案: $(x-1)^{2}+y^{2}=1$", "solution": "null", "level": "高三", "question": "16. 已知圆的极坐标方程是 $\\rho=2 \\cos \\theta$, 那么该圆的直角坐标方程是 \\$ \\qquad \\$", "options": [], "subject": "解析几何", "analysis": ": 解答: $\\because \\rho=2 \\cos \\theta, \\therefore \\rho^{2}=2 \\rho \\cos \\theta, \\therefore x^{2}+y^{2}=2 x$, 即 $(x-1)^{2}+y^{2}=1$分析: 本题主要考查了圆的极坐标方程, 解决问题的关键是根据所给极坐标方程转化即可"} {"id": "22831", "image": [], "answer": "答案: $\\rho=8 \\cos (\\theta-\\pi)$", "solution": "null", "level": "高三", "question": "17. 设有半径为 4 的圆, 在极坐标系内它的圆心坐标为 $(4, \\pi)$, 则这个圆的极坐标方程是", "options": [], "subject": "解析几何", "analysis": ": 解答: 根据公式圆心为 $\\left(a, \\theta_{0}\\right)$, 半径 $a$ 的圆的极坐标方程是 $\\rho=2 a \\cos \\left(\\theta-\\theta_{0}\\right)$ 得此圆的极坐标方程是 $\\rho=8 \\cos (\\theta-\\pi)$\n\n分析: 本题主要考查了圆的极坐标方程, 解决问题的关键是根据所给普通方程转化即可"} {"id": "22832", "image": [], "answer": "答案: $y=x-2$", "solution": "null", "level": "高三", "question": "18. 圆 $O_{1}$ 和圆 $O_{2}$ 的极坐标方程分别为 $\\rho=4 \\cos \\theta, \\rho=-4 \\sin \\theta$, 则经过两圆圆心的直线的直角坐标方程为 \\$ \\qquad \\$ .", "options": [], "subject": "解析几何", "analysis": ": 解答: 圆 $O_{1}: \\rho=4 \\cos \\theta$ 的直角坐标方程为 $(x-2)^{2}+y^{2}=4$, 其圆心为 $O_{1}(2,0)$;圆 $O_{1}: \\rho=-4 \\sin \\theta$ 的直角坐标方程为 $x^{2}+(y+2)^{2}=4$, 其圆心为 $O_{2}(0,-2)$, 由直线两点式方程求得经过两圆圆心的直线的直角坐标方程为 $y=x-2$ 。\n\n分析: 本题主要考查了圆的极坐标方程, 解决问题的关键是要解决坐标系与参数方程的问题,需将问题转化为直角坐标系的问题"} {"id": "22833", "image": [], "answer": "答案: $2 \\sqrt{3}$", "solution": "null", "level": "高三", "question": "19. 已知圆的极坐标方程为 $\\rho=4 \\cos \\theta$, 圆心为 $\\mathrm{C}$, 点 $\\mathrm{P}$ 的极坐标为 $\\left(4, \\frac{\\pi}{3}\\right)$, 则 $|\\mathrm{CP}|=$ \\$ \\qquad \\$ .", "options": [], "subject": "解析几何", "analysis": ": 解答: 由题意知, 圆心坐标为 $(2,0)$, 设点 $\\mathrm{P}(x, y)$, 则 $\\rho=\\sqrt{x^{2}+y^{2}}, x=\\rho \\cos \\theta=2$,\n$y=\\rho \\sin \\theta=2 \\sqrt{3}$,\n\n所以点 $\\mathrm{P}$ 的坐标为 $(2,2 \\sqrt{3})$, 所以 $|\\mathrm{CP}|=2 \\sqrt{3}$.\n\n分析: 本题主要考查了圆的极坐标方程, 解决问题的关键是化为普通方程分析计算即可"} {"id": "22835", "image": [], "answer": "答案: $2+\\sqrt{3}$", "solution": "null", "level": "高三", "question": "20. 已知某圆的极坐标方程为 $\\rho^{2}-4 \\sqrt{2} \\rho \\cos \\left(\\theta-\\frac{\\pi}{4}\\right)+6=0$, 若点 $P(x, y)$ 在该圆上, 则 $\\frac{y}{x}$ 的最大值是", "options": [], "subject": "解析几何", "analysis": ": 解答: 极坐标方程 $\\rho^{2}-4 \\sqrt{2} \\rho \\cos \\left(\\theta-\\frac{\\pi}{4}\\right)+6=0$, 整理的 $(x-2)^{2}+(y-2)^{2}=2$,圆心 $(2,2)$ 半径 $r=\\sqrt{2}, \\frac{y}{x}=\\frac{y-0}{x-0}$ 看作连接 $(x, y),(0,0)$ 的直线斜率, 当直线与圆相切时,斜率取得最值, 设直线为 $k x-y=0 \\therefore \\frac{|2 k-2|}{\\sqrt{k^{2}+1}}=\\sqrt{2} \\therefore k=2+\\sqrt{3}$\n\n分析: 本题主要考查了圆的极坐标方程, 解决问题的关键是数形结合法将所求 $\\frac{y}{x}$ 转化为切线斜率, 进而利用直线与圆相切得到 $\\boldsymbol{d}=\\boldsymbol{r}$ 求解, 此题用到了数形结合法, 此法解题时经常用到, 本题难度适中"} {"id": "22537", "image": [], "answer": "答案: $A D=2$ 或 3", "solution": "null", "level": "高三", "question": "12. 在 Rt $\\triangle \\mathrm{ABC}$ 中, $\\angle \\mathrm{ACB}=90^{\\circ}, \\mathrm{CD} \\perp \\mathrm{AB}$ 于点 $\\mathrm{D}, \\mathrm{CD}=\\sqrt{6}, \\mathrm{AB}=5$, 则 $\\mathrm{AD}=$", "options": [], "subject": "解析几何", "analysis": ":解答: $\\because \\angle \\mathrm{ACB}=90^{\\circ}, \\mathrm{CD} \\perp \\mathrm{AB}, \\therefore \\mathrm{CD}^{2}=\\mathrm{AD} \\cdot \\mathrm{DB}$.\n\n$\\because \\mathrm{CD}=\\sqrt{6}, \\therefore \\mathrm{AD} \\cdot \\mathrm{DB}=6$. 又 $\\because \\mathrm{AB}=5, \\therefore \\mathrm{DB}=5-\\mathrm{AD}$.\n\n$\\therefore A D(5-A D)=6$,解得 $A D=2$ 或 3 .\n\n分析: 本题主要考查了直角三角形的射影定理, 解决问题的关键是根据直角三角形的射影定理结合所给条件计算即可"} {"id": "22539", "image": [], "answer": "10", "solution": "null", "level": "高三", "question": "14. 直角三角形斜边上的高把斜边分成了 $3: 2$ 两段, 且斜边上的高为 $2 \\sqrt{6}$ 厘米, 则斜边长为厘米.", "options": [], "subject": "解析几何", "analysis": ":解答:设斜边长为 $5 \\mathrm{acm}$,则斜边上的高把斜边分成两段的长分别为 $3 \\mathrm{a} \\mathrm{cm}, 2 \\mathrm{a} \\mathrm{cm}$, 则由射影定理得 $3 a \\cdot 2 a=(2 \\sqrt{6})^{2}$, 解得 $a=2$, 则斜边长为 $5 a=5 \\times 2=10(\\mathrm{~cm})$.\n\n分析: 本题主要考查了直角三角形的射影定理, 解决问题的关键是根据直角三角形的射影定理结合所给三角形满足条件分析计算即可"} {"id": "22861", "image": [], "answer": "答案: $(1, \\sqrt{3}, 8)$", "solution": "null", "level": "高三", "question": "6. 点 $M$ 的柱坐标为 $\\left(2, \\frac{\\pi}{3}, 8\\right)\\left(2, \\frac{\\pi}{3}, 8\\right)$, 则它的直角坐标为", "options": [], "subject": "解析几何", "analysis": ": 解答: $\\because x=2 \\cos \\frac{\\pi}{3}=1, y=2 \\sin \\frac{\\pi}{3}=\\sqrt{3}, z=8 . \\therefore$ 它的直角坐标为 $(1, \\sqrt{3}, 8)$分析: 本题主要考查了柱坐标刻画点的位置, 解决问题的关键是转化即可"} {"id": "21104", "image": [], "answer": "(4, 5)", "solution": "null", "level": "高三", "question": "已知椭圆的标准方程为 $\\frac{x^{2}}{m-3}+\\frac{y^{2}}{5-m}=1$, 若其焦点在 $x$ 轴上, 则 $m$ 的取值范围是", "options": [], "subject": "解析几何", "analysis": "解析: $\frac{x^{2}}{m-3}+\frac{y^{2}}{5-m}=1$ 为焦点在 $x$ 轴上的椭圆, 则 $m-3>5-m>0$, 解得 $4|\\mathrm{AB}|$ (定值) 因此, 动点 $P$ 的轨迹为以 $A 、 B$ 为焦点的椭圆, $2 a=6, c=2$, 可得 $b=\\sqrt{3^{2}-2^{2}}=\\sqrt{5}:$.椭圆的方程为 $\frac{\\mathrm{x}^{2}}{9}+\frac{\\mathrm{y}^{2}}{5}=1$, 即为动圆圆心的轨迹方程."} {"id": "21117", "image": [], "answer": "$\frac{\\mathrm{x}^{2}}{25}+\frac{\\mathrm{y}^{2}}{20}=1$ 由粗圆 $\frac{\\mathrm{x}^{2}}{9}+\frac{\\mathrm{y}^{2}}{4}=1$, 得 $\\mathrm{a}^{2}=9, \\quad \\mathrm{~b}^{2}=4, \therefore \\mathrm{c}^{2}=\\mathrm{a}^{2}-\\mathrm{b}^{2}=5$, $\therefore$ 该粗圆的焦点坐标为 $( \\pm \\sqrt{5}, 0)$,设所求粗圆方程为 $\frac{x^{2}}{a^{2}}+\frac{y^{2}}{b^{2}}=1, a>b>0$, 则 $c=\\sqrt{5}$, 又 $\frac{c}{a}=\frac{\\sqrt{5}}{5}$, 得 $a=5, \therefore b^{2}=25-5=20$, $\therefore$ 所求椭圆方程为: $\frac{\\mathrm{x}^{2}}{25}+\frac{\\mathrm{y}^{2}}{20}=1$.", "solution": "null", "level": "高三", "question": "与粗圆 $\\frac{x^{2}}{9}+\\frac{y^{2}}{4}=1$ 有相同的焦点, 且离心率为 $\\frac{\\sqrt{5}}{5}$ 的椭圆标准方程为", "options": [], "subject": "解析几何", "analysis": ""} {"id": "21118", "image": [], "answer": "", "solution": "null", "level": "高三", "question": "已知椭圆 $\\frac{x^{2}}{\\mathrm{~m}+2}+\\frac{\\mathrm{y}^{2}}{4}=1$ 的离心率 $\\mathrm{e}=\\frac{1}{3}$, 则 $\\mathrm{m}$ 的值等于", "options": [], "subject": "解析几何", "analysis": "$\frac{5}{2}$ 或 $\frac{14}{9}$: $\because$ 椭圆 $\frac{x^{2}}{m+2}+\frac{y^{2}}{4}=1, \therefore$ (1)当椭圆焦点在 $x$ 轴上时, $a^{2}=m+2, \\quad b^{2}=4$, 可得 $c=\\sqrt{m-2}$,离心率 $e=\frac{\\sqrt{m-2}}{\\sqrt{m+2}}=\frac{1}{3}$, 得 $m=\frac{5}{2}$; (2) 当椭圆焦点在 $y$ 轴上时, $a^{2}=4, b^{2}=m+2$, 可得 $c=$ $\\sqrt{2-\\mathrm{m}}$, 离心率 $\\mathrm{e}=\frac{\\sqrt{2-\\mathrm{m}}}{2}=\frac{1}{3}$, 得 $\\mathrm{m}=\frac{14}{9}$. 综上所述 $\\mathrm{m}=\frac{5}{2}$ 或 $\frac{14}{9}$."} {"id": "21129", "image": [], "answer": "8", "solution": "null", "level": "高三", "question": "粗圆 $\\frac{x^{2}}{8}+y^{2}=1$ 的左、右焦点为 $F_{1}, F_{2}$, 点 $P$ 在椭圆上, 则 $\\left|P F_{1}\\right| \\cdot\\left|P F_{2}\\right|$ 最大值为", "options": [], "subject": "解析几何", "analysis": "由椭圆方程 $\frac{x^{2}}{8}+y^{2}=1$ 可知: $a=2 \\sqrt{2},\\left|P F_{1}\right|+\\left|P F_{2}\right|=2 a=4 \\sqrt{2}$, 则基本不等式的性质可知: $\\left|P F_{1}\right| \\cdot\\left|P F_{2}\right| \\leq\\left(\frac{\\left|P F_{1}\right|+\\left|P F_{2}\right|}{2}\right)^{2}=8$, 当且仅当 $\\left|P F_{1}\right|=\\left|P F_{2}\right|=2 \\sqrt{2}$ 时, 取等号, $\\left|P F_{1}\right| \\cdot\\left|P F_{2}\right|$最大值为 8 ,"} {"id": "21130", "image": [], "answer": "$\frac{1}{2}$", "solution": "null", "level": "高三", "question": "已知椭圆 $\\frac{x^{2}}{a^{2}}+\\frac{y^{2}}{b^{2}}=1(a>b>0)$ 的左焦点为 $F, A(-a, 0), B(0, b)$ 为椭圆的两个顶点, 若 $F$ 到 $A B$ 的距离等于 $\\frac{b}{\\sqrt{7}}$, 则粗圆的离心率为", "options": [], "subject": "解析几何", "analysis": "$A B$ 的方程为 $\frac{x}{-a}+\frac{y}{b}=1$, 即: $b x-a y+a b=0$, 点 $F(-c, 0)$ 到直线 $A B$ 的距离 $d=\frac{|-b c+a b|}{\\sqrt{a^{2}+b^{2}}}=\frac{b}{\\sqrt{7}}$, $\therefore 5 a^{2}-14 a c+8 c^{2}=0, \therefore 8 e^{2}-14 e+5=0, \because e \\in(0,1) \therefore e=\frac{1}{2}$ 或 $e=\frac{5}{4}$ (舍)."} {"id": "21141", "image": [], "answer": "2 \\sqrt{5}$", "solution": "null", "level": "高三", "question": "椭圆 $\\frac{x^{2}}{9}+\\frac{y^{2}}{4}=1$ 的焦距是 \\$ \\qquad \\$", "options": [], "subject": "解析几何", "analysis": "因为粗圆的标准方程为 $\frac{x^{2}}{9}+\frac{y^{2}}{4}=1$, 所以 $a=3, b=2$, 所以 $c=\\sqrt{a^{2}-b^{2}}=\\sqrt{5}$, 所以椭圆 $\frac{x^{2}}{9}+\frac{y^{2}}{4}=1$ 的焦距为 $2 \\sqrt{5}$, 故答案为 $2 \\sqrt{5}$."} {"id": "21142", "image": [], "answer": "$\frac{5}{4}$", "solution": "null", "level": "高三", "question": "在平面直角坐标系 $x O y$ 中, 已知 $\\triangle A B C$ 顶点 $A(-4,0)$ 和 $C(4,0)$, 顶点 $B$ 在椭圆 $\\frac{x^{2}}{25}+\\frac{y^{2}}{9}=1$ 上, 则 $\\frac{\\sin A+\\sin C}{\\sin B}=$ \\$ \\qquad \\$", "options": [], "subject": "解析几何", "analysis": "利用椭圆定义得三角形的三边存在 $a+c=2 \times 5=10, b=2 \times 4=8$, 由正弦定理得 $\frac{\\sin A+\\sin C}{\\sin B}=\frac{a+c}{b}=\frac{10}{8}=\frac{5}{4}$. 故答案为 $\frac{5}{4}$."} {"id": "21154", "image": [], "answer": "28", "solution": "null", "level": "高三", "question": "如果 $F_{1}, F_{2}$ 分别是双曲线 $\\frac{x^{2}}{16}-\\frac{y^{2}}{9}=1$ 的左、右焦点, $A B$ 是双曲线左支上过点 $F_{1}$ 的弦, 且 $|A B|=6$,则 $\\triangle A B F_{2}$ 的周长是", "options": [], "subject": "解析几何", "analysis": "由题意知: $a=4, b=3$, 故 $c=5$. 由双曲线的定义知 $\\left|A F_{2}\right|-\\left|A F_{1}\right|=8(1),\\left|B F_{2}\right|-\\left|B F_{1}\right|=8(2)$, (1)+(2)得: $\\left|A F_{2}\right|+\\left|B F_{2}\right|-|A B|=16$, 所以 $\\left|A F_{2}\right|+\\left|B F_{2}\right|=22$, 所以 $\triangle A B F_{2}$ 的周长是 $\\left|A F_{2}\right|+\\left|B F_{2}\right|+|A B|=28$"} {"id": "21155", "image": [], "answer": "$\frac{x^{2}}{2}-\frac{y^{2}}{2}=1$", "solution": "null", "level": "高三", "question": "以椭圆 $\\frac{x^{2}}{25}+\\frac{y^{2}}{9}=1$ 长轴的端点为焦点, 以陏圆的焦点为顶点的双曲线方程为", "options": [], "subject": "解析几何", "analysis": "由粗圆方程可知所求双曲线的焦点为 $(-2,0),(2,0)$, 顶点为 $(-\\sqrt{2}, 0),(\\sqrt{2}, 0)$.则设双曲线方程为 $\frac{x^{2}}{a^{2}}-\frac{y^{2}}{b^{2}}=1(a>0, b>0)$, 所以 $a=\\sqrt{2}, c=2$, 则 $b=\\sqrt{c^{2}-a^{2}}=\\sqrt{2}$. 所以所求双曲线方程为 $\frac{x^{2}}{2}-\frac{y^{2}}{2}=1$."} {"id": "22759", "image": ["10065.jpg", "10065.jpg"], "answer": ": $\\left\\{\\begin{array}{l}x^{s}=\\frac{x}{2} \\\\ y^{\\prime}=\\frac{y}{3}\\end{array}\\right.$\n\n\n\n分析: 本题主要考查了平面直角坐标轴中的伸缩变换, 解决问题的关键是根据伸缩变换计算即可", "solution": "null", "level": "高三", "question": "11. 将点 $P(2,3)$ 变换为点 $(1,1)$ 的一个伸缩变换公式为", "options": [], "subject": "变换几何", "analysis": ": $\\left\\{\\begin{array}{l}x^{s}=\\frac{x}{2} \\\\ y^{\\prime}=\\frac{y}{3}\\end{array}\\right.$\n\n\n\n分析: 本题主要考查了平面直角坐标轴中的伸缩变换, 解决问题的关键是根据伸缩变换计算即可"} {"id": "22760", "image": [], "answer": "答案: $\\left\\{\\begin{array}{c}x^{s}=3 x, \\\\ y^{\\prime}=\\frac{1}{2} y\\end{array}\\right.$", "solution": "null", "level": "高三", "question": "12. 将点 $P(-2,2)$ 变换为 $P^{\\prime}(-6,1)$ 的伸缩变换公式为", "options": [], "subject": "变换几何", "analysis": ": 解答: 由伸缩变换公式 $\\left\\{\\begin{array}{c}x^{3}=\\lambda x(\\lambda>0), \\\\ y^{3}=\\mu y(\\mu>0)\\end{array}\\right.$ 得 $\\left\\{\\begin{array}{c}-6=\\lambda \\times(-2), \\\\ 1=\\mu \\times 2,\\end{array}\\right.$ 所以 $\\lambda=3, \\mu=\\frac{1}{2}$.\n\n故伸缩变换公式为 $\\left\\{\\begin{array}{l}x^{*}=3 x, \\\\ y^{\\prime}=\\frac{1}{2} y .\\end{array}\\right.$\n分析: 本题主要考查了平面直角坐标轴中的伸缩变换, 解决问题的关键是根据伸缩变换的原理计算即可"} {"id": "22384", "image": [], "answer": "答案: 1560\n\n【解析】 依题两两彼此给对方写一条毕业留言相当于从 40 人中任选两人的排列数,所以全班共写了 $A^{2}=40 \\times 39=1560$ 条毕业留言,故应填入 1560 .", "solution": "null", "level": "高三", "question": "某高三毕业班有 40 人,同学之间两两彼此给对方仅写一条毕业留言,那么全班共写了条毕业留言.(用数字作答)", "options": [], "subject": "计数", "analysis": "答案: 1560\n\n【解析】 依题两两彼此给对方写一条毕业留言相当于从 40 人中任选两人的排列数,所以全班共写了 $A^{2}=40 \\times 39=1560$ 条毕业留言,故应填入 1560 ."} {"id": "22392", "image": [], "answer": "答案: 112\n\n【解析】 由分层抽样可得, 应从 8 名女生中抽取 2 人, 从 4 名男生中抽取 1 人, 所以不同的抽取方法共有 $\\mathrm{C}_{8}^{2} \\mathrm{C}_{+}^{1}=112$ 种.", "solution": "null", "level": "高三", "question": "从 8 名女生和 4 名男生中, 抽取 3 名学生参加某档电视节目, 如果按性别比例分层抽样,则不同的抽取方法数为", "options": [], "subject": "计数", "analysis": "答案: 112\n\n【解析】 由分层抽样可得, 应从 8 名女生中抽取 2 人, 从 4 名男生中抽取 1 人, 所以不同的抽取方法共有 $\\mathrm{C}_{8}^{2} \\mathrm{C}_{+}^{1}=112$ 种."} {"id": "22416", "image": [], "answer": "答案:(2)\n\n解析:(1)(3)(4)中的随机变量 $X$ 的所有取值, 我们都可以按照一定的次序一一列出, 因此它们是离散型随机变量; (2)中随机变量 $X$ 可以取某一区间内的一切值, 但无法按一定次序一一列出, 故不是离散型随机变量.", "solution": "null", "level": "高三", "question": "下列随机变量中不是离散型随机变量的有 .(填序号)\n\n(1)某宾馆每天入住的旅客数量 $X$;\n\n(2)广州某水文站观测到一天中珠江的水位 $X$;\n\n(3) 深圳欢乐谷一日接待游客的数量 $X$;\n\n(4)虎门大桥一天经过的车辆数 $X$.", "options": [], "subject": "计数", "analysis": "答案:(2)\n\n解析:(1)(3)(4)中的随机变量 $X$ 的所有取值, 我们都可以按照一定的次序一一列出, 因此它们是离散型随机变量; (2)中随机变量 $X$ 可以取某一区间内的一切值, 但无法按一定次序一一列出, 故不是离散型随机变量."} {"id": "22424", "image": [], "answer": "$\frac{2}{9}$", "solution": "null", "level": "高三", "question": "若随机变量 $X$ 的分布列为\n\n| $X$ | 0 | 1 |\n| :---: | :---: | :---: |\n| $P$ | $\\frac{2}{3}$ | $m$ |\n\n则 $D(X)=$", "options": [], "subject": "计数", "analysis": "由分布列的性质可得 $\frac{2}{3}+m=1, \therefore m=\frac{1}{3}$, 由两点分布的方差可得 $D(X)=\frac{1}{3} \times\\left(1-\frac{1}{3}\right)=\frac{2}{9}$."} {"id": "22455", "image": [], "answer": "答案: $2 p-3$\n\n【解析】$E(X)=0 \\cdot(1-p)+1 \\cdot p=p$ 则 $E(2 X-3)=2 E(X)-3=2 p-3$ 故答案是 $2 p-3$", "solution": "null", "level": "高三", "question": "若 $P(X=0)=1-P, P(X=1)=P$, 则 $E(2 X-3)=$", "options": [], "subject": "计数", "analysis": "答案: $2 p-3$\n\n【解析】$E(X)=0 \\cdot(1-p)+1 \\cdot p=p$ 则 $E(2 X-3)=2 E(X)-3=2 p-3$ 故答案是 $2 p-3$"} {"id": "22463", "image": [], "answer": "答案: $\\frac{2}{5}$\n\n【解析】 本题主要考查方差与概率.\n\n已知 $P(\\xi=0)=\\frac{1}{5}$, 设 $P(\\xi=1)=x, P(\\xi=2)=y$.\n\n故 $E(\\xi)=0 \\times \\frac{1}{5}+1 \\times x+2 \\times y=x+2 y=1$ ,\n\n又因为 $\\frac{1}{5}+x+y=1$, 所以 $x=\\frac{3}{5}, y=\\frac{1}{5}$,故 $D(\\xi)=\\frac{1}{5} \\times(0-1)^{2}+\\frac{3}{5} \\times(1-1)^{2}+\\frac{1}{5} \\times(2-1)^{2}=\\frac{2}{5}$.", "solution": "null", "level": "高三", "question": "随机变量 $\\xi^{\\xi}$ 的取值为 $0,1,2$, 若 $P(\\xi=0)=\\frac{1}{5}, E(\\xi)=1$, 则 $D(\\xi)=$", "options": [], "subject": "计数", "analysis": "答案: $\\frac{2}{5}$\n\n【解析】 本题主要考查方差与概率.\n\n已知 $P(\\xi=0)=\\frac{1}{5}$, 设 $P(\\xi=1)=x, P(\\xi=2)=y$.\n\n故 $E(\\xi)=0 \\times \\frac{1}{5}+1 \\times x+2 \\times y=x+2 y=1$ ,\n\n又因为 $\\frac{1}{5}+x+y=1$, 所以 $x=\\frac{3}{5}, y=\\frac{1}{5}$,故 $D(\\xi)=\\frac{1}{5} \\times(0-1)^{2}+\\frac{3}{5} \\times(1-1)^{2}+\\frac{1}{5} \\times(2-1)^{2}=\\frac{2}{5}$."} {"id": "22471", "image": [], "answer": "0.6", "solution": "null", "level": "高三", "question": "已知随机变量 $\\xi$ 服从正态分布 $N\\left(40, a^{2}\\right)$, 若 $P(\\xi<30)=", "options": [], "subject": "计数", "analysis": "根据随机变量 $\\xi$ 服从正态分布, 知正态曲线的对称轴是 $x=40$,利用正态分布的对称性可得 $P(\\xi>50)=P(\\xi<30)=0.2$, 所以 $P(30<\\xi<50)=1-[P(\\xi>50)+P(\\xi<30)]=1-0.4=0.6$故答案为: 0.6"} {"id": "22472", "image": [], "answer": "0.3", "solution": "null", "level": "高三", "question": "随机变量 $X \\sim N\\left(\\mu, \\sigma^{2}\\right)$, 且 $P(X<-3)=P(X>1)=", "options": [], "subject": "计数", "analysis": "$X \\sim N\\left(\\mu, \\sigma^{2}\right)$, 且 $P(X<-3)=P(X>1)=0.2$, 可得 $\\mu=-1, P(-1", "options": [], "subject": "图论", "analysis": ": 解答: 由题意可知: 工序(1) $\\rightarrow$ 工序(4)工时数为 2 , 工序(4) $\\rightarrow$ 工序(6)工时数为 2 , 工序 (6) $\\rightarrow$ 工序(7)工时数为 5 , 工序(7) $\\rightarrow$ 工序(8)工时数为 1 , 所以所用工程总时数为: $2+2+5+1=10$天.\n\n故答案为: 10 .\n\n分析: 本题主要考查了工序流程图 (即统筹图), 解决问题的关键是仔细观察工序流程图,寻找关键路线, 注意利用优选法对重复的供需选择用时较多的. 进而问题即可获得解答."} {"id": "20577", "image": ["9379.jpg"], "answer": "答案: (1)2", "solution": "null", "level": "高三", "question": "19. 下面的图示中, 是流程图的是\n\n", "options": [], "subject": "图论", "analysis": ": 解答: (1)(2)为流程图, (3)(4)为结构图\n\n分析: 本题主要考查了绘制简单实际问题的流程图, 解决问题的关键是根据流程图与结构图的联系与区别进行分析解决."} {"id": "14955", "image": [], "answer": "-2 或 1", "solution": "null", "level": "高一", "question": "若 $2 \\in\\left\\{1, x^{2}+x\\right\\}$, 则 $x$ 的值为", "options": [], "subject": "算术", "analysis": "$2 \\in\\left\\{1, x^{2}+x\\right\\}$, 故 $2=x^{2}+x \\therefore x=1$ 或- 2 , 经检验满足元素的互异性.\n\n故填 -2 或 1\n\n【分析】利用元素和集合的关系结合一元二次方程求根的方法求出 $\\mathrm{x}$ 的值, 再用元素的互异性得出满足要求的 $\\mathrm{x}$ 的值。"} {"id": "14956", "image": [], "answer": "6", "solution": "null", "level": "高一", "question": "已知 $P=\\{x \\mid 25 \\\\ a \\leq 6,\\end{array}\\right.$ 即 $5
0$ 且 $k \\neq 1), \\frac{2}{a}+\\frac{1}{b}=2$, 则 $k=$", "options": [], "subject": "算术", "analysis": "$2 \\log _{5} 25+3 \\log _{2} 64-8 \\log _{7} 1=2 \\times 2+3 \\times 6-8 \\times 0=4+18-0=22$\n\n故答案为: 22\n\n【分析】先算出每一个对数式的值, 再代入进行乘法和加减运算即可."} {"id": "15155", "image": [], "answer": "$\\left\\{x \\left\\lvert\\, 00 \\\\ \\lg \\frac{2}{x}-1 \\geqslant 0, \\text { 解得, } 00 \\\\ \\lg \\frac{2}{x}-1 \\geqslant 0, \\text { 解不等式可求. }\\end{array}\\right.$"} {"id": "16908", "image": [], "answer": "4.9", "solution": "null", "level": "高一", "question": "(2017 春 -大丰市校级期中)阅读如图所示的伪代码:若输入 $x$ 的值为 12 , 则 $p=$\n\nRead $x$\n\nIf $x \\leqslant 10$ Then\n\n$$\np \\leftarrow 0.35 \\mathrm{x}\n$$\n\nElse\n\n$p \\leftarrow 3.5+0.7(x-10)$\n\n## End If\n\n## Print $p$", "options": [], "subject": "算术", "analysis": "解: 由已知中伪代码, 可知:\n\n该程序的功能是计算并输出分段函数 $p=\\left\\{\\begin{array}{l}0.35 x, x \\leq 10 \\\\ 3.5+0.7(x-10), x>10\\end{array}\\right.$ 的函数值,\n\n当 $x=12$ 时, $p=3.5+0.7(12-10)=4.9$,\n\n故答案为: 4.9"} {"id": "16912", "image": [], "answer": ".", "solution": "null", "level": "高一", "question": "018 春・惠山区校级期中)执行算法代码“ForIFrom 1 To50 Step 2”, 共执行的循环次数为", "options": [], "subject": "算术", "analysis": "算法代码是 “ForIFrom 1 To 50 Step 2”,\n\n$I$ 的取值构成等差数列, 等差 $d=2, a_{1}=1, a_{n}=50$,\n\n根据等差数列的通项公式: $a_{n}=a_{1}+(n-1) d$, 可得: $50=1+(n-1) \\times 2$\n\n$\\therefore$ 可解得: $n=25.5$, 共执行 25 次.\n\n故答案为: 25 ."} {"id": "16933", "image": [], "answer": "6 .", "solution": "null", "level": "高一", "question": "(2018 秋・怀仁市校级期末)用秦九韶算法求函数 $f(x)=1+2 x+x^{2}-3 x^{3}+2 x^{4}$, 当 $x=-1$ 时的值时, $v_{2}$ 的结果是 \\$ \\qquad \\$ .", "options": [], "subject": "算术", "analysis": "解: $v_{1}=2 \\times(-1)-3=-5$;\n\n$\\therefore v_{2}=(-5) \\times(-1)+1=6$,\n\n故答案为: 6 ."} {"id": "16934", "image": [], "answer": ".", "solution": "null", "level": "高一", "question": "019 春 $・$ 乐山期中) 若二进制数 $10 b 1_{(2)}$ 和三进制数 $a 02_{(3)}$ 相等, $a, b$ 为正整数, 则 $2 a+b=$ \\$ \\qquad \\$ .", "options": [], "subject": "算术", "analysis": "$\\because 10 b 1_{(2)}=1 \\times 2^{3}+0 \\times 2^{2}+b \\times 2^{1}+1 \\times 2^{0}=8+0+2 b+1=9+2 b, \\quad(b=0,1)$, $a 02_{(3)}=a \\times 3^{2}+0 \\times 3^{1}+2 \\times 3^{0}=9 a+2, \\quad(a=0,1,2)$,\n\n$\\therefore$ 根据题意, 可得: $9+2 b=9 a+2$,\n\n$\\therefore$ 解得: $a=1, b=1$,\n\n$\\therefore 2 a+b=3$.\n\n故答案为: 3 ."} {"id": "16947", "image": [], "answer": "56 .", "solution": "null", "level": "高一", "question": "(2020 秋・黑龙江期中)二进制数 101110 转化为等值的八进制数为", "options": [], "subject": "算术", "analysis": "解: $101110(2)=0 \\times 2^{0}+1 \\times 2^{1}+1 \\times 2^{2}+1 \\times 2^{3}+1 \\times 2^{5}=46$\n\n$46 \\div 8=5 \\cdots 6$\n\n$5 \\div 8=0 \\cdots 5$\n\n故 $46{ }_{(10)}=56(8)$\n\n故答案为: 56 ."} {"id": "16948", "image": [], "answer": "20 .", "solution": "null", "level": "高一", "question": "(2020 春・宁县校级期末)用秦九韶算法求多项式 $f(x)=x^{6}-5 x^{5}+6 x^{4}+x^{2}+3 x+2$ 的值, 当 $x=-2$\n 时, $v_{2}$ 的值为 \\$ \\qquad \\$", "options": [], "subject": "算术", "analysis": "解: 函数 $\\left.f(x)=x^{6}-5 x^{5}+6 x^{4}+x^{2}+3 x+2=((((x-5) x+6) x) x+1) x+3\\right) x+2$,\n\n当 $x=-2$ 时, 分别算出 $v_{0}=1$,\n\n$v_{1}=-1 \\times 2-5=-7$,\n\n$v_{2}=-7 \\times(-2)+6=20$,\n\n故得 $v_{2}$ 的值为 20 .\n\n故答案为: 20 ."} {"id": "14958", "image": [], "answer": "14", "solution": "null", "level": "高一", "question": "定义集合 $A, B$ 的一种运算“*”, $A * B=\\{p \\mid p=x+y, x \\in A, y \\in B\\}$. 若 $A=\\{1,2,3\\}, B=\\{1,2\\}$, 则集合 $A * B$ 中所有元素的和", "options": [], "subject": "代数", "analysis": "$\\because A * B=\\{p \\mid p=x+y, x \\in A, y \\in B\\}, A=\\{1,2,3\\}, B=\\{1,2\\}$,\n\n$\\therefore A * B=\\{2,3,4,5\\}, 2+3+4+5=14$.\n\n故答案为: 14 .\n\n【分析】由 $\\mathrm{A} * \\mathrm{~B}=\\{\\mathrm{p} \\mid \\mathrm{p}=\\mathrm{x}+\\mathrm{y}, \\mathrm{x} \\in \\mathrm{A}, \\mathrm{y} \\in \\mathrm{B}\\}, \\mathrm{A}=\\{1,2,3\\}, \\mathrm{B}=\\{1,2\\}$, 知 $\\mathrm{A} * \\mathrm{~B}=\\{2,3,4,5\\}$, 由此能求出集合 $\\mathrm{A} * \\mathrm{~B}$ 中所有元素的和."} {"id": "14976", "image": [], "answer": "$a \\geq \\frac{9}{8}$ 或 $a=0$ 。", "solution": "null", "level": "高一", "question": "已知集合 $A=\\left\\{x \\mid a x^{2}-3 x+2=0\\right\\}$ 至多有一个元素, 则 $a$ 的取值范围", "options": [], "subject": "代数", "analysis": "$\\because$ 集合 $A$ 中至多有一个元素, $\\therefore$ 当 $a=0$ 时, $A=\\left\\{x \\mid a x^{2}-3 x+2=0\\right\\}=\\left\\{\\frac{2}{3}\\right\\}$, 合题意;\n\n当 $a \\neq 0$ 时, $\\triangle=9-8 a \\leq 0$ 解得 $a \\geq \\frac{9}{8}$, 总之 $\\left\\{d a \\geq \\frac{9}{8}\\right.$ 或 $\\left.a=0\\right\\}$,\n\n故答案为 $\\left\\{d a \\geq \\frac{9}{8}\\right.$ 或 $\\left.a=0\\right\\}$ 。\n\n【分析】由已知分两种情况讨论 $\\mathrm{a}$, 当 $a=0$ 时, $A=\\left\\{\\frac{2}{3}\\right\\}$, 符合题意, 当 $a \\neq 0$ 时, 由 $\\triangle=9-8 a \\leq 0$,\n\n可得 $a \\geq \\frac{9}{8}$, 即可求出 $a$ 的取值范围."} {"id": "14977", "image": ["7468.jpg"], "answer": "$2 \\leq a \\leq 4$", "solution": "null", "level": "高一", "question": "设集合 $A=\\{x|x-a|<1, x \\in R\\}, B=\\{x \\mid 1\n\n又当 $a=2$ 时, $\\mathrm{A}=\\{x \\mid 1\n\n【分析】 $f(x)=-3 x^{2}+6 x$ 配方求出顶点, 作出图像, 求出 $f(x)=-9$ 对应的自变量, 结合函数图像, 即可求解."} {"id": "15016", "image": [], "answer": "$[4,9]$", "solution": "null", "level": "高一", "question": "设函数 $f(x)$ 的定义域为 $[0,1]$, 则函数 $f(\\sqrt{x}-2)$ 的定义域为", "options": [], "subject": "代数", "analysis": "解: 因为函数 $f(x)$ 的定义域为 $[0,1]$,\n\n由 $0 \\leqslant \\sqrt{x}-2 \\leqslant 1$, 得: $\\left\\{\\begin{array}{l}\\sqrt{x}-2 \\geqslant 0(1) \\\\ \\sqrt{x}-2 \\leqslant 1(2)\\end{array}\\right.$\n\n解(1)得: $x \\geqslant 4$, 解(2)得: $x \\leqslant 9$.\n\n所以, 函数 $f(\\sqrt{x}-2)$ 的定义域为 $\\lfloor 4,9\\rfloor$.\n\n故答案为: $\\lfloor 4,9\\rfloor$.\n\n【分析】根据函数 $f(x)$ 的定义域为 $[0,1]$, 由 $\\sqrt{x}-2 \\in[0,1]$, 求出 $x$ 的取值集合即可得函数 $f(\\sqrt{x}-2)$ 的定义域."} {"id": "15019", "image": [], "answer": "$\\lfloor 0,4\\rfloor$", "solution": "null", "level": "高一", "question": "若函数 $y=\\sqrt{a x^{2}+a x+1}$ 的定义域为 $R$, 则 $a$ 的取值范围为", "options": [], "subject": "代数", "analysis": "由题意得 $a x^{2}+a x+1 \\geq 0$ 在 $R$ 上恒成立.\n\n(1)当 $a=0$ 时, 则 $1 \\geq 0$ 恒成立,\n\n$\\therefore a=0$ 符合题意;\n\n(2)当 $a \\neq 0$ 时,\n\n则 $\\left\\{\\begin{array}{l}a>0 \\\\ a^{2}-4 a \\leq 0,\\end{array}\\right.$ 解得 $00$ 的解是全体实数(或恒成立)的条件是当 $a=0$ 时, $b=0, c>0$; 当\n\n$a \\neq 0$ 时, $\\left\\{\\begin{array}{l}a>0 \\\\ -<0 ;\\end{array}\\right.$ 不等式 $a x^{2}+b x+c<0$ 的解是全体实数(或恒成立)的条件是当 $a=0$ 时, $b=0, c<0$; 当 $a \\neq 0$ 时, $\\left\\{\\begin{array}{l}a<0 \\\\ \\Delta<0\\end{array}\\right.$."} {"id": "15032", "image": [], "answer": "$$\nf(x)=x^{2}-2, x \\in(-\\infty,-2] \\cup[2,+\\infty)\n$$", "solution": "null", "level": "高一", "question": "已知 $f\\left(x+\\frac{1}{x}\\right)=x^{2}+\\frac{1}{x^{2}}$, 则 $f(x)=$", "options": [], "subject": "代数", "analysis": "(配凑法) (1) $f\\left(x+\\frac{1}{x}\\right)=x^{2}+\\frac{1}{x^{2}}=\\left(x+\\frac{1}{x}\\right)^{2}-2$,\n\n又 $x+\\frac{1}{x} \\in(-\\infty,-2] \\cup[2,+\\infty)$,\n\n$\\therefore f(x)=x^{2}-2, x \\in(-\\infty,-2] \\cup[2,+\\infty)$.\n\n故答案为: $f(x)=x^{2}-2, x \\in(-\\infty,-2] \\cup[2,+\\infty)$\n\n【分析】利用配凑法求出函数的解析式即可."} {"id": "15033", "image": [], "answer": "$3 x$", "solution": "null", "level": "高一", "question": "已知函数 $f(x)$ 满足 $2 f(x)+f(-x)=3 x \\quad, \\quad$ 则 $f(x)=$", "options": [], "subject": "代数", "analysis": "因为 $2 f(x)+f(-x)=3 x$,\n\n所以将 $x$ 用 $-x$ 替换,得 $2 f(-x)+f(x)=-3 x$ ,\n\n解由(1)(2)组成的方程组得 $f(x)=3 x$.\n\n故答案为: $3 x$\n\n【分析】由 $2 \\mathrm{f}(\\mathrm{x})+\\mathrm{f}(-\\mathrm{x})=3 \\mathrm{x}$, 用 $-\\mathrm{x}$ 代入可得 $2 \\mathrm{f}(-\\mathrm{x})+\\mathrm{f}(\\mathrm{x})=-3 \\mathrm{x}$, 由两式联立解方程组消去 $\\mathrm{f}(-\\mathrm{x})$即可求解出 $f(x)=3 x$ 。"} {"id": "15034", "image": [], "answer": "$f(x)=x-\\frac{1}{2}$", "solution": "null", "level": "高一", "question": "已知一次函数 $f(x)$ 满足条件 $f(x+1)+f(x)=2 x$, 则函数 $f(x)$ 的解析式为 $f(x)=$", "options": [], "subject": "代数", "analysis": "设 $f(x)=k x+b, k \\neq 0$,\n\n$\\because f(x+1)+f(x)=2 x$,\n\n$\\therefore k(x+1)+b+k x+b=2 x$,\n\n即 $2 k x+k+2 b=2 x$ ,\n\n$\\{2 k=2$\n\n$\\therefore k+2 b=0$,\n\n解可得, $k=1, \\quad b=-\\frac{1}{2}$,\n\n$\\therefore f(x)=x-\\frac{1}{2}$\n\n故答案为: $f(x)=x-\\frac{1}{2}$\n\n【分析】先设 $f(x)=k x+b, k \\neq 0$, 然后根据 $f(x+1)+f(x)=2 x$, 代入后根据对应系数相等可求 $k, b$, 即可求解.\n\n三、解答题"} {"id": "15050", "image": [], "answer": "$k<-2$", "solution": "null", "level": "高一", "question": "函数 $y=(k+2) x+1$ 在实数集上是减函数, 则 $\\mathrm{k}$ 的范围是", "options": [], "subject": "代数", "analysis": "$\\because$ 函数 $y=(k+2) x+1$ 在实数集上是减函数,\n\n当 $k+2=0$ 时, $y=1$ 是常函数, 不满足题意,\n\n$\\therefore k+2<0, \\quad \\therefore k<-2$\n\n故答案为: $(-\\infty,-2)$\n\n【分析】先验证当 $k+2=0$ 时函数 $y=(k+2) x+1=1$ 为常函数不满足条件, 然后根据一次函数是减函数时斜率必为小于 0 的数从而可求出 $k$ 的值, 确定答案."} {"id": "15051", "image": [], "answer": "$-\\frac{5}{4}$", "solution": "null", "level": "高一", "question": "函数 $f(x)=x^{2}+x-1$ 的最小值是", "options": [], "subject": "代数", "analysis": "$\\because f(x)=x^{2}+x-1=\\left(x+\\frac{1}{2}\\right)^{2}-\\frac{5}{4}$,\n\n因此, 函数 $f(x)=x^{2}+x-1$ 的最小值是 $-\\frac{5}{4}$.\n故答案为: $-\\frac{5}{4}$.\n\n【分析】将二次函数 $y=f(x)$ 的解析式进行配方, 可得出该函数的最小值."} {"id": "15053", "image": [], "answer": "9", "solution": "null", "level": "高一", "question": "函数 $f(x)=9 x^{2}+\\sqrt{x-1}$ 的最小值为", "options": [], "subject": "代数", "analysis": "$\\because f(x)$ 的定义域为 $\\lfloor 1,+\\infty)$, 且 $f(x)$ 在定义域上单调递增,\n\n$\\therefore f(x)_{\\min }=f(1)=9$.\n\n故答案为: 9\n\n【分析】结合 $f(x)$ 的定义域, 判断出 $f(x)$ 的单调性, 由此求得 $f(x)$ 的最小值."} {"id": "15054", "image": [], "answer": "$\\left[\\frac{1}{2}, 1\\right]$", "solution": "null", "level": "高一", "question": "已知函数 $f(x)=\\sqrt{a x^{2}-2 x-5 a+5}$ 在 $\\lfloor 2,+\\infty)$ 是增函数, 则实数 $a$ 的取值范围是", "options": [], "subject": "代数", "analysis": "当 $a=0$ 时, $f(x)=\\sqrt{-2 x+5}$, 显然在 $\\lfloor 2,+\\infty)$ 不是增函数, 所以舍去;\n\n当 $a \\neq 0$ 时, 由题得 $\\left\\{\\begin{array}{l}a>0 \\\\ -\\frac{-2}{2 a} \\leq 2 \\\\ a \\cdot 2^{2}-4-5 a+5 \\geq 0, \\text { 所以 } \\frac{1}{2} \\leq a \\leq 1\\end{array}\\right.$.\n\n所以实数 $\\mathrm{a}$ 的取值范围为 $\\left[\\frac{1}{2}, 1\\right]$.\n\n故答案为:\n\n$\\left[\\frac{1}{2}, 1\\right]$\n\n【分析】对 $a$ 分 $a=0$ 和 $a \\neq 0$ 两种情况结合复合函数的单调性讨论得解."} {"id": "15055", "image": [], "answer": "$\\left(-\\frac{1}{2}, \\frac{2}{3}\\right)$", "solution": "null", "level": "高一", "question": "已知 $\\mathrm{f}(\\mathrm{x})$ 是定义在 $(-2,2)$ 上的减函数, 并且 $\\mathrm{f}(\\mathrm{m}-1)-\\mathrm{f}(1-2 \\mathrm{~m})>0$, 则实数 $m$ 的取值范围是", "options": [], "subject": "代数", "analysis": "由 $f(m-1)-f(1-2 m)>0$ 得 $f(m-1)>f(1-2 m)$,\n\n由于函数 $f(x)$ 在 $(-2,2)$ 上递减,\n故 $\\left\\{\\begin{array}{l}-20$ 得 $f(m-1)>f(1-2 m)$, 再利用函数的单调性结合函数的定义域求出 $\\mathrm{m}$ 的取值范围。\n\n三、解答题"} {"id": "15069", "image": [], "answer": "$\\frac{1}{2}$", "solution": "null", "level": "高一", "question": "已知一个奇函数的定义域为 $[\\mathrm{a}+1, \\mathrm{~b}-2]$, 则 $\\frac{a+b}{2}=$", "options": [], "subject": "代数", "analysis": "根据题意, 函数 $\\mathrm{f}(\\mathrm{x})$ 是奇函数, 且其定义域为 $[\\mathrm{a}+1, \\mathrm{~b}-2]$,\n\n则有 $a+1+b-2=0$, 解得: $a+b=1$,\n\n即 $\\frac{a+b}{2}=\\frac{1}{2}$,\n\n故答案为: $\\frac{1}{2}$.\n\n【分析】根据题意, 由函数奇偶性的定义可得 $\\mathrm{a}+1+\\mathrm{b}-2=0$, 即可得答案."} {"id": "15070", "image": [], "answer": "1", "solution": "null", "level": "高一", "question": "若函数 $y=(x+1)(x-a)$ 为偶函数, 则 $a=$", "options": [], "subject": "代数", "analysis": "函数 $y=(x+1)(x-a)=x^{2}+(1-a) x-a$\n\n$\\because$ 函数 $y=(x+1)(x-a)$ 为偶函数,\n\n$\\therefore(-x)^{2}+(1-a)(-x)-a=x^{2}+(1-a) x-a$\n\n$\\therefore 1-a=0, \\quad \\therefore a=1$\n\n【分析】根据偶函数的定义, 可得一次项系数为 0 , 从而可得结论."} {"id": "15071", "image": [], "answer": "$(-\\infty,-4) \\cup(0,+\\infty)$", "solution": "null", "level": "高一", "question": "设函数 $f(x)$ 是定义在 $R$ 上的偶函数, 记 $g(x)=f(x)-x^{2}$, 且函数 $g(x)$ 在区间 $[0,+\\infty)$ 上是增函数, 则不等式 $f(x+2)-f(2)>x^{2}+4 x$ 的解集为", "options": [], "subject": "代数", "analysis": "根据题意 $g(x)$, 且 $f(x)$ 是定义在 $R$ 上的偶函数,\n\n则 $g(-x)=f(-x)-(-x)^{2}=f(x)-x^{2}=g(x)$, 则函数 $g(x)$ 为偶函数,\n\n$f(x+2)-f(2)>x^{2}+4 x \\Rightarrow f(x+2)-(x+2)^{2}>f(2)-4 \\Rightarrow g(x+2)>g(2)$,\n\n又由 $g(x)$ 为增函数且在区间 $[0,+\\infty)$ 上是增函数,\n\n则 $|x+2|>2$, 解得: $x<-4$ 或 $x>0$,\n\n即 $x$ 的取值范围为 $(-\\infty,-4) \\cup(0,+\\infty)$,\n\n故答案为 $(-\\infty,-4) \\cup(0,+\\infty)$;\n\n【分析】根据题意, 分析可得 $g(x)$ 为偶函数, 进而分析可得原不等式转化为 $g(x+2)>g(2)$, 结合函数的奇偶性与单调性分析可得 $|x+2|>2$, 解可得 $x$ 的取值范围."} {"id": "15072", "image": ["7506.jpg"], "answer": "$(-1,0) \\cup(1,+\\infty)$", "solution": "null", "level": "高一", "question": "已知函数 $f(x)$ 满足 $f(x)+f(-x)=0$, 对任 意的 $x_{1}, x_{2} \\in(0,+\\infty)$ 都 有 $\\frac{x_{2} f\\left(x_{2}\\right)-x_{1} f\\left(x_{1}\\right)}{x_{1}-x_{2}}<0$ 恒成立, 且 $f(1)=0$, 则关于 $x$ 的不等式 $f(x)<0$ 的解集为", "options": [], "subject": "代数", "analysis": "由题意, 设函数 $g(x)=x f(x)$,\n\n因为函数 $f(x)$ 满足 $f(x)+f(-x)=0$, 即 $f(-x)=-f(x)$,\n\n则 $g(-x)=(-x) f(-x)=x f(x)=g(x)$, 所以函数 $g(x)$ 为 $R$ 上的偶函数,\n\n又由 $f(1)=0$, 则 $g(-1)=g(1)=1 \\times f(1)=0$,\n\n因为对任意的 $x_{1}, x_{2} \\in(0,+\\infty)$ 都有 $\\frac{x_{2} f\\left(x_{2}\\right)-x_{1} f\\left(x_{1}\\right)}{x_{1}-x_{2}}<0$ 恒成立,\n\n则函数 $g(x)$ 在 $(0,+\\infty)$ 为单调递增函数,\n\n所以当 $x \\in(-1,0)$ 时, $g(x)=x f(x)<0$, 此时 $f(x)>0$,\n\n当 $x \\in(1,+\\infty)$ 时, $g(x)=x f(x)>0$, 此时 $f(x)>0$,\n所以 $f(x)>0$ 的解集为 $(-1,0) \\cup(1,+\\infty)$.\n\n故答案为: $(-1,0) \\cup(1,+\\infty)$.\n\n【分析】构造新函数 $g(x)=x f(x)$, 求得函数 $g(x)$ 为 $R$ 上的偶函数, 得出 $g(-1)=g(1)=0$, 在\n\n\n单调递增函数, 结合函数 $g(x)$ 的取值, 即可求解.\n\n三、解答题"} {"id": "15092", "image": [], "answer": "47", "solution": "null", "level": "高一", "question": "已知 $a^{\\frac{1}{2}}+a^{-\\frac{1}{2}}=3$, 求值 $\\mathrm{a}^{2}+\\mathrm{a}^{-2}=$", "options": [], "subject": "代数", "analysis": "因为 $a+a^{-1}=\\left(a_{2}^{\\frac{1}{2}}+a^{\\frac{1}{2}}\\right)^{2}-2=7$,\n\n所以 $a^{2}+a^{-2}=\\left(a+a^{-1}\\right)^{2}-2=49-2=47$.\n\n【分析】考虑 $a^{\\frac{1}{2}}+a^{-\\frac{1}{2}}$ 和 $a+a^{-1} 、 a^{2}+a^{-2}$ 之间的关系."} {"id": "15093", "image": [], "answer": "$2 \\sqrt{2}$", "solution": "null", "level": "高一", "question": "$\\sqrt{5+2 \\sqrt{6}}-\\sqrt{6-4 \\sqrt{2}}+\\sqrt{7-4 \\sqrt{3}}=$", "options": [], "subject": "代数", "analysis": "$$\n\\sqrt{5+2 \\sqrt{6}}=\\sqrt{\\frac{10+4 \\sqrt{6}}{2}}=\\sqrt{\\frac{(\\sqrt{6}+2)^{2}}{2}}=\\frac{\\sqrt{6}+2}{\\sqrt{2}}=\\frac{2 \\sqrt{3}+2 \\sqrt{2}}{2}=\\sqrt{3}+\\sqrt{2}\n$$\n\n$\\sqrt{6-4 \\sqrt{2}}=\\sqrt{(2-\\sqrt{2})^{2}}=2-\\sqrt{2}, \\quad \\sqrt{7-4 \\sqrt{3}}=\\sqrt{(2-\\sqrt{3})^{2}}=2-\\sqrt{3}$\n\n$\\therefore \\sqrt{5+2 \\sqrt{6}}-\\sqrt{6-4 \\sqrt{2}}+\\sqrt{7-4 \\sqrt{3}}=\\sqrt{3}+\\sqrt{2}-2+\\sqrt{2}+2-\\sqrt{3}=2 \\sqrt{2}$\n\n故答案为: $2 \\sqrt{2}$\n\n【分析】将每个被开方数化为完全平方式的形式, 从而开方整理得到结果.\n\n三、解答题"} {"id": "15110", "image": [], "answer": "$(-\\infty,-1)$", "solution": "null", "level": "高一", "question": "不等式 $0.5^{2 x}>0.5^{x-1}$ 的解集为 (用区间表示).", "options": [], "subject": "代数", "analysis": "根据函数单调性可有: $2 x", "solution": "null", "level": "高一", "question": "已知函数 $f(x)$ 是指数函数, 如果 $f(3)=9 f(1)$, 那么 $f(8)$ $f(4)$\n\n(请在横线上填写“ >”, “ =”或“ < ”)", "options": [], "subject": "代数", "analysis": "因为函数 $f(x)$ 是指数函数,\n\n设 $f(x)=a^{x}$,\n\n则 $f(3)=a^{3}=9 a^{1}=9 f(1)$,\n\n解得 $a=3$ 或 $a=-3$ (舍去)\n\n所以 $f(x)=3^{x}$ 是增函数, 所以 $f(8)>f(4)$,\n\n故答案为:\n\n【分析】由题意设 $f(x)=a^{x}$, 根据 $f(3)=9 f(1)$ 求出解析式, 即可比较 $f(8), f(4)$ 的大小."} {"id": "15113", "image": ["7517.jpg"], "answer": "(1)(2)", "solution": "null", "level": "高一", "question": "已知实数 $a, b$ 满足等式 $2019^{a}=2020^{b}$, 下列五个关系式:\n(1) $0\n根据实数 $a, b$ 满足等式 $2019^{a}=2020^{b}$, 结合图象可知,\n下列五个关系式:\n(1) $00$ 且 $a \\neq 1)$ 在定义域 $[m, n]$ 上的值域是 $\\left[m^{2}, n^{2}\\right](1\\log _{3} 27=3$.\n\n故所有正确结论的编号是(2)(3).\n\n故答案为: (2)(3)\n\n【分析】将指数式转化为对数式, 再根据对数的运算性质验证."} {"id": "15154", "image": ["7531.jpg"], "answer": "$\\frac{1}{3}$", "solution": "null", "level": "高一", "question": "\\text{如果函数 }y=\\log_ax_\\text{的图象过点}\\left(\\frac19,2\\right),\\text{则 }a=", "options": [], "subject": "代数", "analysis": "\n\n$\\because a>0$ 且 $a \\neq 1$, 因此, $a=\\frac{1}{3}$.\n\n故答案为: $\\frac{1}{3}$.将点的坐标代入对数函数解析式, 利用对数式化指数式可求出实数 $a$ 的值."} {"id": "15156", "image": ["7532.jpg"], "answer": "1", "solution": "null", "level": "高一", "question": "设函数 $f(x)=\\log _{a}(x-1)+1(a>0$ 且 $a \\neq 1)$ 恒过点 $(m, n)$, 则 $m-n=$", "options": [], "subject": "代数", "analysis": "\n\n故答案为: 1根据函数过定点得到 $m=2, n=1$, 计算得到答案."} {"id": "15157", "image": ["7533.jpg"], "answer": "$(-\\infty, 0) \\cup(0,+\\infty)$", "solution": "null", "level": "高一", "question": "函数 $f(x)=\\ln \\frac{x+1}{x-1}$ 的值域为", "options": [], "subject": "代数", "analysis": "$\\ln \\frac{x+1}{x-1}=\\ln \\frac{x-1+2}{x-1}=\\ln \\left(1+\\frac{2}{x-1}\\right)$\n\n$\\because 1+\\frac{2}{x-1}>0$ 且 $1+\\frac{2}{x-1} \\neq 1 \\quad \\therefore \\ln \\left(1+\\frac{2}{x-1}\\right) \\neq 0$\n\n$\\therefore f(x)$ 值域为: $(-\\infty, 0) \\cup(0,+\\infty)$\n\n故答案为: $(-\\infty, 0) \\cup(0,+\\infty)$\n\n【分析】把已知对数函数变形整理, 得到 $f(x)=\\ln \\left(1+\\frac{2}{x-1}\\right)$, 利用 ${ }^{1+\\frac{2}{x-1}>0}$ 且 $1+\\frac{2}{x-1} \\neq 1$,即可求出函数的值域."} {"id": "15176", "image": ["7545.jpg"], "answer": "$x^{-\\frac{1}{3}}$", "solution": "null", "level": "高一", "question": "已知幂函数 $f(x)$ 的图象过点 $\\left(8, \\frac{1}{2}\\right)$, 则此幂函数的解析式是 $f(x)=$", "options": [], "subject": "代数", "analysis": "设 $f(x)=x^{\\alpha}$, 因为 $y=f(x)$ 的图象过点 $\\left(8, \\frac{1}{2}\\right)$,\n\n所以 $8^{\\alpha}=\\frac{1}{2}$, 即 $2^{3 \\alpha}=2^{-1}$,\n\n$\\therefore 3 \\alpha=-1$, 解得 $\\alpha=-\\frac{1}{3}$, 因此 $f(x)=x^{-\\frac{1}{3}}$.\n\n故答案为 $x^{-\\frac{1}{3}}$.\n\n【分析】设函数 $f(x)=x^{\\alpha}$, 将点 $\\left(8, \\frac{1}{2}\\right)$ 的坐标代入函数 $y=f(x)$ 的解析式, 求出 $\\alpha$ 的值, 即可得\n\n"} {"id": "15177", "image": [], "answer": "$\\pm 2$", "solution": "null", "level": "高一", "question": "已知关于 $x$ 的函数 $y=\\left(m^{2}-3\\right) x^{2 m}$ 是幂函数, 则 $m=$", "options": [], "subject": "代数", "analysis": "关于 $x$ 的函数 $y=\\left(m^{2}-3\\right) x^{2 m}$ 是幂函数, 则 $m^{2}-3=1 \\Rightarrow m= \\pm 2$.\n\n【分析】由已知利用幂函数的概念, 得到 $m^{2}-3=1$, 即可求出 $\\mathrm{m}$ 的值."} {"id": "15178", "image": [], "answer": "-2", "solution": "null", "level": "高一", "question": "设 $\\alpha \\in\\left\\{\\frac{1}{3}, \\frac{1}{2},-1,-2,3\\right\\}$, 若 $f(x)=x^{\\alpha}$ 为偶函数, 则 $\\alpha=$", "options": [], "subject": "代数", "analysis": "由题可知, $\\alpha=-2$ 时, $f(x)=x^{-2}$, 满足 $\\mathrm{f}(-\\mathrm{x})=\\mathrm{f}(\\mathrm{x})$, 所以是偶函数;\n\n当 $\\alpha=\\frac{1}{3}, \\frac{1}{2},-13$ 时, 不满足 $\\mathrm{f}(-\\mathrm{x})=\\mathrm{f}(\\mathrm{x})$,\n\n$\\therefore \\alpha=-2$.\n\n故答案为: -2 .\n\n【分析】将不同的 $\\alpha$ 值代入,逐一验证即可."} {"id": "15181", "image": [], "answer": "2", "solution": "null", "level": "高一", "question": "幕函数 $y=\\left(m^{2}-m-1\\right) x^{-5 m-3}$ 在 $x \\in(0,+\\infty)$ 时为减函数, 则 $\\mathrm{m}=$", "options": [], "subject": "代数", "analysis": "因为 $y=\\left(m^{2}-m-1\\right) x^{-2 m-3}$ 是幕函数,\n\n所以 $m^{2}-m-1=1$, 故 $\\mathrm{m}=2$ 或 $\\mathrm{m}=-1$,\n\n又幂函数 $y=\\left(m^{2}-m-1\\right) x^{-5 m-3}$ 在 $x \\in(0,+\\infty)$ 时为减函数,\n\n所以 $-5 \\mathrm{~m}-3<0$, 所以 $\\mathrm{m}=2$.\n\n【分析】由已知函数 $y=\\left(m^{2}-m-1\\right) x^{-2 m-3}$ 是幂函数列式, 得到 $\\mathrm{m}=2$ 或 $\\mathrm{m}=-1$, 结合函数的单调性,即可判断 $\\mathrm{m}$ 的值.\n\n三、解答题"} {"id": "16909", "image": ["8730.jpg"], "answer": "2", "solution": "null", "level": "高一", "question": "(2020 - 吴江区模拟) 如图是一个算法的程序框图, 当输入的值 $x$ 为 8 时, 则其输出的结果是\n\n", "options": [], "subject": "代数", "analysis": "解: $x=8>0$, 执行循环体, $x=x-3=5-3=2>0$, 继续执行循环体,\n\n$x=x-3=2-3=-1<0$, 满足条件, 退出循环体, 故输出 $y=0.5^{-1}=\\left(\\frac{1}{2}\\right)-1=2$.\n\n故答案为: 2"} {"id": "16910", "image": ["8731.jpg"], "answer": "14 .", "solution": "null", "level": "高一", "question": "(2019$\\cdot$苏州模拟)如图所示是一个算法的伪代码,输出结果是\n\n", "options": [], "subject": "代数", "analysis": "解:由程序语句得程序的流程为:\n\n$$\n\\begin{aligned}\n& a=2, \\quad S=0+2=2 \\\\\n& a=2 \\times 2=4, \\quad S=2+4=6 \\\\\n& a=2 \\times 4=8, \\quad S=8+6=14\n\\end{aligned}\n$$\n\n故输出 $S=14$.\n\n故答案为: 14 ."} {"id": "16911", "image": ["8732.jpg"], "answer": "1 .", "solution": "null", "level": "高一", "question": "(2019 春・鄂州期末)运行如图所示的程序, 其输出的结果为\n\n", "options": [], "subject": "代数", "analysis": "解: 由程序语句知, 第一次运行 $s=0+5, n=5-1=4$;\n第二次运行 $s=0+5+4=9, n=4-1=3$;\n\n第三次运行 $s=9+3=12, n=3-1=2$;\n\n第四次运行 $s=12+2=14, n=2-1=1$, 不满足条件 $s<14$, 输出 $n=1$.\n\n故答案为: 1 ."} {"id": "16928", "image": [], "answer": "859.", "solution": "null", "level": "高一", "question": "(2019 秋 -内江期末)秦九韶是我国南宋时期的数学家, 普州(现四川省安岳县)人, 他在所著的《数书九章》中提出的多项式求值的秦九韶算法, 至今仍是比较先进的算法. 已知一个 5 次多项式为 $f(x)=4 x^{5}-3 x^{3}-2 x^{2}-5 x+1$, 用秦九韶算法求这个多项式当 $x=3$ 时的值为 \\$ \\qquad \\$ .", "options": [], "subject": "代数", "analysis": "解: $f(x)=4 x^{5}-3 x^{3}-2 x^{2}-5 x+1=((((4 x) x-3) x-2) x-5) x+1$,\n\n则 $f(3)=((((4 \\times 3) \\times 3-3) \\times 3-2) \\times 3-5) \\times 3+1=859$,\n\n故答案为: 859."} {"id": "16930", "image": [], "answer": "24 .", "solution": "null", "level": "高一", "question": "(2020 秋・广西期中)已知 $f(x)=x^{5}-2 x^{3}+3 x^{2}-x+1$, 应用秦九韶算法计算 $x=3$ 时的值时, $v_{3}$的值为 \\$ \\qquad \\$ .", "options": [], "subject": "代数", "analysis": "解: 由秦九韶算法可得 $f(x)=x^{5}-2 x^{3}+3 x^{2}-x+1=((((x+0) x-2) x+3) x-1)$\n\n$$\n\\begin{aligned}\n& x+1 \\\\\n& \\therefore v_{0}=1 \\\\\n& v_{1}=1 \\times 3+0=3 \\\\\n& v_{2}=3 \\times 3-2=7\n\\end{aligned}\n$$\n\n$v_{3}=7 \\times 3+3=24$.\n\n故答案为: 24 ."} {"id": "16931", "image": [], "answer": "4485 .", "solution": "null", "level": "高一", "question": "(2020 春・黄山期末) 已知函数 $f(x)=x^{5}+2 x^{4}+x^{3}-x^{2}+3 x-5$, 用秦九韶算法计算 $f(5)=$ \\$ \\qquad \\$", "options": [], "subject": "代数", "analysis": "解: $f(x)=x^{5}+2 x^{4}+x^{3}-x^{2}+3 x-5=((((x+2) x+1) x-1) x+3) x-5$\n\n则 $f(5)=((((5+2) 5+1) 5-1) 5+3) 5-5$\n\n$=4485$.\n\n故答案为: 4485 ."} {"id": "16932", "image": [], "answer": "1209.4 .", "solution": "null", "level": "高一", "question": "(2019 春・息县期中)用秦九韶算法求多项式 $f(x)=4 x^{5}+2 x^{4}+3.5 x^{3}-2.6 x^{2}+1.7 x-0.8$ 当 $x=3$ 时的值为 \\$ \\qquad \\$ ;", "options": [], "subject": "代数", "analysis": "解: $\\because v_{0}=4, v_{1}=4 \\times 3+2=14, v_{2}=14 \\times 3+3.5=45.5, v_{3}=45.5 \\times 3-2.6=133.9, v_{4}=$ $133.9 \\times 3+1.7=403.4, \\quad v_{5}=403.4 \\times 3-0.8=1209.4$.\n\n故答案为: 1209.4 ."} {"id": "16952", "image": ["8741.jpg"], "answer": "18 .", "solution": "null", "level": "高一", "question": "(2020$\\cdot$江苏模拟)如图是某算法的伪代码, 输出的结果 $S$ 的值为\n\n", "options": [], "subject": "代数", "analysis": "解:模拟算法的运行过程, 如下;\n\n$$\ni=1, \\quad S=2\n$$\n\n$S=2+1=3, \\quad i=3$,\n\n$S=3+3=6, \\quad i=5$,\n\n$S=6+5=11, \\quad i=7$,\n\n$S=11+7=18, \\quad i=9 ;$\n\n终止循环, 输出 $S=18$.\n\n故答案为: 18 ."} {"id": "16955", "image": ["8743.jpg"], "answer": "sqrt{e}$.", "solution": "null", "level": "高一", "question": "020$\\cdot$ 陵区校级模拟) 根据如图所示的伪代码, 当输出 $y$ 的值为 $\\frac{1}{2}$ 时, 则输入的 $x$ 的值为 \\$ \\qquad \\$ .\n\n", "options": [], "subject": "代数", "analysis": "根据如图所示的伪代码知, 程序的功能是输出函数 $y=\\left\\{\\begin{array}{l}x^{2}+1, x \\leq 0 \\\\ \\ln x, x>0\\end{array}\\right.$;\n\n当输出 $y$ 的值为 $\\frac{1}{2}$ 时, 若 $x \\leqslant 0$, 则 $x^{2}+1=\\frac{1}{2}$, 此时方程无解;\n\n若 $x>0$, 则 $\\ln x=\\frac{1}{2}$, 解得 $x=\\sqrt{e}$;\n\n所以输入 $x$ 的值为 $\\sqrt{e}$.\n\n故答案为: $\\sqrt{e}$."} {"id": "16970", "image": ["8752.jpg"], "answer": "32 .", "solution": "null", "level": "高一", "question": "(2020 春・商丘期末)如图所示的程序执行后输出的结果为\n\n", "options": [], "subject": "代数", "analysis": "解:模拟程序的运行过程知,\n\n$i=1$ 时, 满足条件 $S<25$, 得 $S=0+2 \\times 1=2, i=3$;\n\n$i=3$ 时, 满足条件 $S<25$, 得 $S=2+2 \\times 3=8, i=5$;\n\n$i=5$ 时, 满足条件 $S<25$, 得 $S=8+2 \\times 5=18, i=7$;\n\n$i=7$ 时, 满足条件 $S<25$, 得 $S=18+2 \\times 7=32, i=9$;\n\n$i=9$ 时, 不满足条件 $S<25$, 退出循环, 输出 $S=32$.\n\n故答案为: 32 ."} {"id": "16971", "image": ["8753.jpg"], "answer": "16 .", "solution": "null", "level": "高一", "question": "(2020$\\cdot$鼓楼区校级模拟) 一个算法的伪代码如图所示, 执行此算法, 最后输出的 $S$ 的值为\n\n", "options": [], "subject": "代数", "analysis": "解:模拟程序的运行过程,如下;\n\n$k=1, S=1$,\n\n进入循环;\n\n$k=3, S=1+3=4$,\n\n$k=5, \\quad S=4+5=9$,\n\n$k=7, \\quad S=9+7=16$,\n\n终止循环, 输出 $S=16$.\n\n故答案为: 16 ."} {"id": "17018", "image": ["8773.jpg", "8773.jpg"], "answer": "$\\frac{3}{4}$.\n\n", "solution": "null", "level": "高一", "question": "(2020 春 - 金牛区校级月考)在区间 $[0,1]$ 上任取两个数 $a 、 b$, 则函数 $f(x)=x^{2}+a x+b^{2}$ 无零点的概率为 \\$ \\qquad \\$ .", "options": [], "subject": "代数", "analysis": "解: 若 $f(x)=x^{2}+a x+b^{2}$ 无零点, 则 $\\triangle=a^{2}-4 b^{2}<0$,\n\n又 $a, b \\in[0,1]$, 故 $a<2 b$,\n在平面直角坐标系 $a O b$ 中作出正方形 $O A B C$,\n\n设直线 $a=2 b$ 与 $A B$ 交点为 $D$, 则 $D$ 为 $A B$ 的中点,\n\n故当点( $a, b )$ 在梯形 $D B C O$ (不含边界 $O D )$ 时, 函数 $f(x)$ 无零点.\n\n故所求概率 $P=\\frac{S_{\\text {梯柇 } B C O D}}{S_{\\text {正方形 } O A B C}}=\\frac{3}{4}$.\n\n故答案为: $\\frac{3}{4}$.\n\n"} {"id": "14975", "image": [], "answer": "-3", "solution": "null", "level": "高一", "question": "已知实数集合 $\\{1,2,3, x\\}$ 的最大元素等于该集合的所有元素之和, 则 $x=$", "options": [], "subject": "逻辑题", "analysis": "因为实数集合 $\\{1,2,3, x\\}$ 的最大元素等于该集合的所有元素之和,\n\n所以 $1+2+3+x=x$ (无解) 或者 $1+2+3+x=3$, 解得: $x=-3$.\n\n故答案为: -3 .\n\n【分析】根据题意求元素的关系."} {"id": "14993", "image": [], "answer": "-1", "solution": "null", "level": "高一", "question": "若集合 $A=\\{x \\mid x \\leq m\\}, B=\\{x \\mid x \\geq-1\\}$, 且 $A \\cap B=\\{m\\}$, 则实数 $\\mathrm{m}$ 的值为", "options": [], "subject": "逻辑题", "analysis": "解: $\\because A=\\{x \\mid x \\leq m\\}, B=\\{x \\mid x \\geq-1\\}$, 且 $A \\cap B=\\{m\\}$,\n\n$\\therefore m=-1$,\n\n故答案为: -1 .\n\n【分析】直接根据交集运算的定义求解即可."} {"id": "15518", "image": [], "answer": "平行", "solution": "null", "level": "高一", "question": "如果规定: $x=y, y=z$, 则 $x=z$, 叫作 $x, y, z$ 关于相等关系具有传递性, 那么空间三个平面 $\\alpha, \\beta, \\gamma$ 关于相交、垂直、平行这三种关系中具有传递性的是", "options": [], "subject": "逻辑题", "analysis": "由平面与平面的位置关系及两个平面平行、垂直的定义、判定定理,知平面平行具有传递性,相交、垂直都不具有传递性."} {"id": "17111", "image": ["8780.jpg"], "answer": "52,1 处.", "solution": "null", "level": "高一", "question": "(2017 秋・云岩区校级月考)下面程序执行后输出的结果是 \\$ \\qquad \\$ , 若要求画出对应的猜想框\n\n图, 则选择程序框有 \\$ \\qquad \\$ .\n\n", "options": [], "subject": "逻辑题", "analysis": "解:模拟执行程序语言,如下;\n\n$T=1, \\quad S=0$,\n\n第 1 次循环, $S=0+1=1, T=1+1=2$;\n\n第 2 次循环, $S=1+1=2, T=2+1=3$;\n\n第 3 次循环, $S=2+1=3, T=3+1=4$;\n\n$\\cdots ;$\n\n第 51 次循环, $S=50+1=51, T=51+1+52$;\n\n此时不满足条件 $S \\leqslant 50$, 终止循环, 输出 $T=52$.\n\n若要求画出对应的猜想框图, 则选择程序框有 1 处.\n\n故答案为: 52,1 处."} {"id": "14978", "image": [], "answer": "49", "solution": "null", "level": "高一", "question": "已知集合 $P=\\{1,2,3,4,5\\}$, 若 $A, B$ 是 $P$ 的两个非空子集, 则所有满足 $A$ 中的最大数小于 $B$ 中的最小数的集合对 $(A, B)$ 的个数为", "options": [], "subject": "组合数学", "analysis": "当 $A$ 中的最大数为 1 , 即 $A=\\{1\\}$ 时, $B=\\{2\\},\\{3\\},\\{4\\},\\{5\\},\\{2,3\\},\\{2,4\\},\\{2,5\\}$, $\\{3,4\\},\\{3,5\\},\\{4,5\\},\\{2,3,4\\},\\{2,3,5\\},\\{2,4,5\\},\\{3,4,5\\},\\{2,3,4,5\\} ;$\n\n所以满足题意的集合对 $(A, B)$ 的个数为 15 个;\n\n当 $A$ 中的最大数为 2 , 即 $A=\\{2\\},\\{1,2\\}$ 时, $B=\\{3\\},\\{4\\},\\{5\\},\\{3,4\\},\\{3,5\\},\\{4,5\\},\\{3,4,5\\}$;即满足题意的集合对 $(A, B)$ 的个数为 $2 \\times 7=14$ 个;\n\n当 $A$ 中的最大数为 3 , 即 $A=\\{3\\},\\{1,3\\},\\{2,3\\},\\{1,2,3\\}$ 时, $B=\\{4\\},\\{5\\},\\{4,5\\}$, 即满足题意的集合对 $(A, B)$ 的个数 $4 \\times 3=12$ 个;\n\n当 $A$ 中的最大数为 4 , 即 $A=\\{4\\},\\{1,4\\},\\{2,4\\},\\{3,4\\},\\{1,2,4\\},\\{1,3,4\\},\\{2,3,4\\},\\{1,2,3,4\\}$ 时, $B=\\{5\\}$, 即满足题意的集合对 $(A, B)$ 的个数为 8 个;\n\n所以总共个数为 49 个.\n\n【分析】分 $A$ 中的最大数为 $1, A$ 中的最大数为 $2, A$ 中的最大数为 $3, A$ 中的最大数为 4 , 四种情况, 根据题意列举出满足条件的集合 $A, B$, 即可得出结果."} {"id": "14994", "image": [], "answer": "4", "solution": "null", "level": "高一", "question": "已知集合 $A=\\{1,2,3\\}, B=\\{2,3,4\\}$, 则集合 $A \\cup B$ 中元素的个数为", "options": [], "subject": "组合数学", "analysis": "解: 因为集合 $A=\\{1,2,3\\}, B=\\{2,3,4\\}$,\n\n所以 $A \\cup B=\\{1,2,3,4\\}$.\n\n所以集合 $A \\cup B$ 中元素的个数为 4 ,\n\n故答案为 4 。\n\n【分析】本题首先可以通过题意得出集合 $\\mathrm{A}$ 以及集合 $\\mathrm{B}$ 所包含的元素,然后利用并集定义写出 $A \\cup B$, 即可得出结果。"} {"id": "14995", "image": ["7472.jpg"], "answer": "$\\{x \\mid 0 \\leq x \\leq 1$, 或 $x>3\\}$", "solution": "null", "level": "高一", "question": "设 $A, B$ 是非空集合, 定义 $A \\times B=\\{x \\mid x \\in A \\cup B$ 且 $x \\notin A \\cap B\\}$, 已知 $A=\\{x \\mid 0 \\leq x \\leq 3\\}, B=\\{x \\mid x>1\\}$, 则 $A \\times B=$", "options": [], "subject": "组合数学", "analysis": "解:根据题意可知 $A \\times B$ 表示的图像如下图阴影部分所示,\n\n\n而 $A=\\{x \\mid 0 \\leq x \\leq 3\\}, B=\\{x \\mid x>1\\}, \\quad A \\cap B=\\{x \\mid 13\\}$.\n\n故填: $\\{x \\mid 0 \\leq x \\leq 1$, 或 $x>3\\}$.\n\n【分析】由已知给出的定义, 得到 $A \\times B$ 所表示的韦恩图, 利用交集,并集的运算即可求出 $A \\times B$."} {"id": "16489", "image": [], "answer": "抽签法", "solution": "null", "level": "高一", "question": "福利彩票的中奖号码是从 1-36 的号码中, 依次选出 7 个号码来确定的, 这种从 36 个号码中选 7 个号码的抽样方法是", "options": [], "subject": "组合数学", "analysis": "抽签法,故答案为:抽签法"} {"id": "16500", "image": [], "answer": "不正确", "solution": "null", "level": "高一", "question": "在平时的生活中, 对于抽签, 有人认为先抽白人会占便宜, 这种想法 (填“正确”或 “不正确”)", "options": [], "subject": "组合数学", "analysis": "不正确 抽签时每个个体被抽到的机会是相等的, 与先后顺序无关.故答案为:不正确"} {"id": "16501", "image": [], "answer": "1", "solution": "null", "level": "高一", "question": "一个布袋中有 10 个同样质地的小球,从中不放回地依次抽取 3 个小球,则某一特定小球被抽到的可能性是 \\$ \\qquad \\$ , 第三次抽取时, 剩余每个小球被抽到的可能性是", "options": [], "subject": "组合数学", "analysis": "因为简单随机抽样过程中每个个体被抽到的可能性均为, 所以第一个空填因\n本题中的抽样是不放回抽样, 所以第一次抽取时, 每个小球被抽到的可能性为 1 , 第二次抽取时, 剩余 9 个小球,每个小球被抽到的可能性为 2 , 第三次抽取时, 剩余 8 个小球,每个小球被抽到的可能性为 1 .故答案为:1"} {"id": "16929", "image": [], "answer": "1010 (2).", "solution": "null", "level": "高一", "question": "(2019 秋・南充期末) 把十进制数 10 化为二进制数为 \\$ \\qquad \\$ .", "options": [], "subject": "组合数学", "analysis": "解: $10 \\div 2=5 \\cdots 0$\n\n$5 \\div 2=2 \\cdots 1$\n\n$2 \\div 2=1 \\cdots 0$\n\n$1 \\div 2=0 \\cdots 1$\n\n故 $10{ }_{(10)}=1010(2)$\n\n故答案为: 1010 (2)."} {"id": "16973", "image": [], "answer": "$\\frac{7}{10}$.", "solution": "null", "level": "高一", "question": "(2020 秋・天津期末)从 11 至 14 世纪涌现出一批著名的数学家和其创作的数学著作, 如秦九韶的《数书九章》, 李冶的《测圆海镜》, 杨辉的《详解九章算法》、《日用算法》和《杨辉算法》. 某学校团委为拓展学生课外学习兴趣, 现从上述五部著作中任意选择两部作为学生课外拓展学习的参考书目,则所选的两部中至少有一部不是杨辉著作的概率为", "options": [], "subject": "组合数学", "analysis": "解: 秦九韶的《数书九章》, 李冶的《测圆海镜》, 杨辉的《详解九章算法》、《日用算法》和《杨辉算法》.\n\n某学校团委为拓展学生课外学习兴趣, 现从上述五部著作中任意选择两部作为学生课外拓展学习的参考书目,\n\n基本事件总数 $n=C_{5}^{2}=10$,\n\n所选的两部中至少有一部不是杨辉著作包含的基本事件个数 $m=C_{2}^{2}+C_{3}^{1} C_{2}^{1}=7$,\n\n则所选的两部中至少有一部不是杨辉著作的概率 $P=\\frac{m}{n}=\\frac{7}{10}$.\n\n故答案为: $\\frac{7}{10}$."} {"id": "16974", "image": [], "answer": "$\\frac{3}{10}$.", "solution": "null", "level": "高一", "question": "(2020 秋 - 沙依巴克区校级期末)一个袋中装有大小相同的 5 个球, 现将这 5 个球分别编号为 1 、 $2 、 3 、 4 、 5$, 从袋中取出两个球, 每次只取出一个, 并且取出的球不放回, 求取出的两个球上编号之积为奇数的概率为", "options": [], "subject": "组合数学", "analysis": "解: 一个袋中装有大小相同的 5 个球, 现将这 5 个球分别编号为 $1 、 2 、 3 、 4 、 5$,从袋中取出两个球, 每次只取出一个, 并且取出的球不放回,基本事件总数 $n=5 \\times 4=20$,取出的两个球上编号之积为奇数包含的基本事件个数 $m=3 \\times 2=6$.\n\n$\\therefore$ 取出的两个球上编号之积为奇数的概率 $P=\\frac{m}{n}=\\frac{6}{20}=\\frac{3}{10}$.\n\n故答案为: $\\frac{3}{10}$."} {"id": "15017", "image": [], "answer": "$\\left[-\\frac{3}{4}, \\frac{3}{7}\\right)$", "solution": "null", "level": "高一", "question": "若 $f(x)=\\frac{2 x-5}{x+3}, x \\in[1,4)$, 则 $f(x)$ 的值域是 (请用区间表示)", "options": [], "subject": "解析几何", "analysis": "$f(x)=\\frac{2 x-5}{x+3}=\\frac{2 x+6-11}{x+3}=2-\\frac{11}{x+3}$,\n\n函数 $f(x)$ 在 $[1,4)$ 上为增函数,\n\n而 $f(1)=-\\frac{3}{4}, \\quad f(4)=\\frac{3}{7}$,\n\n则函数 $f(x)$ 的值域为 $\\left[-\\frac{3}{4}, \\frac{3}{7}\\right)$.\n\n【分析】由已知得到 $f(x)=2-\\frac{11}{x+3}$, 判断函数 $f(x)$ 在 $[1,4)$ 上为增函数, 计算 $f(1)=-\\frac{3}{4}$, $f(4)=\\frac{3}{7}$, 即可求出函数 $f(x)$ 的值域."} {"id": "15031", "image": [], "answer": "0", "solution": "null", "level": "高一", "question": "已知 $f(\\sqrt{x}-1)=x-2 \\sqrt{x}$, 则 $f(1)=$", "options": [], "subject": "解析几何", "analysis": "令 $t=\\sqrt{x}-1 \\geq-1$, 得 $x=(t+1)^{2}$,\n\n$\\therefore f(t)=(t+1)^{2}-2(t+1)=t^{2}-1$,\n$\\therefore f(x)=x^{2}-1(x \\geq-1)$, 因此, $f(1)=1^{2}-1=0$.\n\n故答案为: 0 .\n\n【分析】先利用换元法求出函数 $y=f(x)$ 的解析式, 然后可计算出 $f(1)$ 的值"} {"id": "15052", "image": [], "answer": "$(-\\infty,-3]$", "solution": "null", "level": "高一", "question": "函数 $y=\\sqrt{x^{2}+2 x-3}$ 的单调递减区间是", "options": [], "subject": "解析几何", "analysis": "因为函数 $y=\\sqrt{x^{2}+2 x-3}$ 有意义,\n\n则满足 $x^{2}+2 x-3 \\geq 0 \\Leftrightarrow(x+3)(x-1) \\geq 0 \\Leftrightarrow x \\geq 1, x \\leq-3$,\n\n而二次函数 $y=x^{2}+2 x-3$ 开口向上, 对称轴为 $x=1$,\n\n那么根据复合函数的单调性, 可知当 $x \\leq-3$ 时, 函数是递减的,\n\n因此答案为 $(-\\infty,-3]$.\n\n【分析】首先求出函数的定义域,结合二次函数的图像求出其单调区间,再利用符合函数的单调性:单调性一致为增不一致为减, 即可求出结果。"} {"id": "15111", "image": [], "answer": "$(2,-3)$", "solution": "null", "level": "高一", "question": "已知函数 $\\mathrm{f}(\\mathrm{x})=\\mathrm{a}^{\\mathrm{x}-2}-4(\\mathrm{a}>0, \\mathrm{a} \\neq 1)$ 的图象恒过定点 $\\mathrm{A}$, 则 $\\mathrm{A}$ 的坐标为", "options": [], "subject": "解析几何", "analysis": "$\\because$ 函数 $f(x)=a^{x-2}-4$, 其中 $a>0, a \\neq 1$,\n\n令 $x-2=0$ 可得 $x=2, \\quad a^{x-2}=1$,\n\n$\\therefore f(x)=1-4=-3$,\n\n$\\therefore$ 点 $A$ 的坐标为 $(2,-3)$,\n\n故答案为: $(2,-3)$.\n\n【分析】根据指数函数的图像恒过点 $(0,1)$, 令 $x-2=0$ 可得 $x=2, \\quad a^{x-2}=1$, 可得 $f(x)=1-4=-3$, 从而得恒过点的坐标."} {"id": "15179", "image": [], "answer": "$\\frac{3}{2}$", "solution": "null", "level": "高一", "question": "设幂函数 $f(x)=k x^{a}$ 的图像经过点 $(4,2)$, 则 $k+\\alpha=$", "options": [], "subject": "解析几何", "analysis": "由题意得 $k=1,2=4^{\\alpha} \\Rightarrow \\alpha=\\frac{1}{2} \\therefore k+\\alpha=\\frac{3}{2}$\n\n【分析】利用已知条件幂函数的图象经过点 $(4,2)$, 借助代入法和幂函数的定义求出 $\\mathrm{k}$ 和 $\\mathrm{a}$ 的值, 从而求出 $\\mathrm{k}+\\mathrm{a}$ 的值。"} {"id": "16997", "image": [], "answer": "$\\frac{1}{2}$.", "solution": "null", "level": "高一", "question": "(2020 春・双流区校级月考)关于 $x$ 的方程 $x^{2}-4 x+m=0(-1 \\leqslant m \\leqslant 7)$ 有两个正实根的概率是 \\$ \\qquad \\$", "options": [], "subject": "解析几何", "analysis": "解: 由 $x^{2}-4 x+m=0$, 得 $m=-x^{2}+4 x=-(x-2)^{2}+4,(x>0)$,\n\n解得 $m \\in(0,4]$,\n\n由 $-7 \\leqslant m \\leqslant 1$, 方程 $x^{2}-4 x+m=0(-1 \\leqslant m \\leqslant 7)$ 有两个正实根的概率为 $\\frac{1}{2}$.\n\n故答案为: $\\frac{1}{2}$."} {"id": "14365", "image": [], "answer": "9", "solution": "null", "level": "高一", "question": "函数 $y=\\log _{a}(2 x-3)+4$ 的图像恒过定点 $A$, 且点 $A$ 在幂函数 $f(x)$ 的图像上, 则 $f(3)=$ \\$ \\qquad \\$ .", "options": [], "subject": "解析几何", "analysis": "\n\n$\\because \\log _{a} 1=0, \\therefore$ 当 $2 x-3=1$, 即 $x=2$ 时, $y=4, \\therefore$ 点定点 $A$ 的坐标是 $P(2,4)$, 幂函数 $f(x)=x^{\\alpha}$ 图象过点 $A(2,4), \\therefore 4=2^{\\alpha}$, 解得 $\\alpha=2, \\therefore$ 幂函数为 $f(x)=x^{2}$, 则 $f(3)=9$, 故答案为 9 ."} {"id": "14380", "image": [], "answer": "2", "solution": "null", "level": "高一", "question": "定义区间 $\\left[x_{1}, x_{2}\\right]$ 的长度为 $x_{2}-x_{1}$, 已知函数 $f(x)=3^{|x|}$ 的定义域为 $[a, b]$, 值域为 $[1,9]$, 则区间 $[a, b]$长度的最小值为 \\$ \\qquad \\$", "options": [], "subject": "解析几何", "analysis": "【详解】\n\n$\\because$ 函数 $f(x)=3^{|x|}$ 的定义域为 $[a, b]$, 值域为 $[1,9]$, 又 $3^{0}=1,3^{2}=3^{|2|}=9, \\therefore 0 \\in[a, b] .2$ 和 -2 至少有一个属于区间 $[a, b]$,\n\n故区间 $[a, b]$ 的长度最小时为 $[-2,0]$ 或 $[0,2]$, 即区间 $[a, b]$ 长度的最小值为 2 .\n\n故答案为: 2 .\n\n【点睛】\n\n本题主要考查指数函数的图象和性质, 考查绝对值不等式的解法, 属于中档题."} {"id": "14307", "image": [], "answer": "2", "solution": "null", "level": "高一", "question": "设 $[t]$ 表示不超过实数 $t$ 的最大整数 (如 $[-1.3]=-2,[2.6=2$ ), 则函数 $f(x)=|2 x-1|-|x|$ 的零点个数为", "options": [], "subject": "解析几何", "analysis": "函数 $f(x)=|2 x-1|-\\lfloor x\\rfloor$ 的零点即方程 $|2 x-1|=\\lfloor x\\rfloor$ 的根,\n\n$\\therefore$ 函数 $f(x)$ 的零点个数, 即方程 $|2 x-1|=\\lfloor x\\rfloor$ 的根的个数.\n\n$\\because|2 x-1| \\geq 0, \\therefore|x| \\geq 0, \\therefore x \\geq 0$.\n\n当 $0 \\leq x<1$ 时, $[x]=0, \\therefore|2 x-1|=0, \\therefore x=\\frac{1}{2}$.\n\n当 $x=1$ 时, $\\lfloor x\\rfloor=1 , \\therefore|2 x-1|=1, \\therefore 2 x-1=1$ 或 $2 x-1=-1 \\therefore x=1$ 或 $x=0$ (舍).\n\n当 $x>1$ 时, $|2 x-1|=2 x-1>x \\geq\\lfloor x\\rfloor, \\therefore$ 方程 $|2 x-1|=\\lfloor x\\rfloor_{\\text {无解. }}$.\n\n综上, 方程 $|2 x-1|=\\lfloor x\\rfloor_{\\text {的根为 }} \\frac{1}{2}, 1$.\n\n所以方程 $|2 x-1|=\\lfloor x\\rfloor$ 有 2 个根, 即函数 $f(x)=|2 x-1|-\\lfloor x\\rfloor$ 有 2 个零点.\n\n故答案为: 2 .\n\n【分析】函数 $f(x)=|2 x-1|-\\lfloor x\\rfloor$ 的零点即方程 $|2 x-1|=\\lfloor x\\rfloor$ 的根, 由 $\\mid 2 x-1 \\geq \\geq 0$ 可得 $x \\geq 0$. 分\n$0 \\leq x<1 、 x=1$ 和 $x>1$ 讨论, 求出方程 $|2 x-1|=\\lfloor x \\mid$ 的根, 即得函数 $f(x)$ 的零点个数."} {"id": "16560", "image": ["8575.jpg"], "answer": "\\left(x_{1}+x_{2}\\right) / 2\\left(y_{1}+y_{2}\\right) / 2", "solution": "null", "level": "高一", "question": "已知 $A\\left(x_{1}, y_{1}\\right), B\\left(x_{2}, y_{2}\\right)$ 是平面上的两点, 试根据平面几何中的中点坐标公式设计一个程序, 要求输入 $A, B$ 两点的坐标, 输出它们连线中点的坐标. 现已给出程序的一部分,请在横线处把程序补充完整: \\$ \\qquad \\$\n\n\n\n$\\mathrm{y}=$\n\nPRINT $\\mathrm{x}, \\mathrm{y}$\n\nEND", "options": [], "subject": "解析几何", "analysis": "故答案为:\\left(x_{1}+x_{2}\\right) / 2\\left(y_{1}+y_{2}\\right) / 2"} {"id": "15679", "image": [], "answer": "\\not \\subset \\alpha, b \\subset \\alpha, a / / b$", "solution": "null", "level": "高一", "question": "请你正确地使用符号写出直线与平面平行的判定定理条件", "options": [], "subject": "解析几何", "analysis": "【分析】\n\n直线与平面平行的判定定理为: 如果平面外一条直线与这个平面内的一条直线平行, 那么这条直线与这个平面平行,将文字语言转化为符号语言即可.\n\n## 【详解】\n\n直线与平面平行的判定定理的条件为“平面外一条直线与这个平面内的一条直线平行”,转化为符号语言为: “ $a \\not \\subset \\alpha, b \\subset \\alpha, a / / b$ ”\n\n故答案为: $a \\not \\subset \\alpha, b \\subset \\alpha, a / / b$\n\n## 【点睛】\n\n本题主要考查直线与平面平行的判定定理, 考查学生正确运用符号表示立体几何位置关系的能力,属于基础题."} {"id": "15334", "image": ["7618.jpg"], "answer": "(1) 由图可知, $f(2)=0$, 所以 $f[f(2)]=f(0)=4$\n\n(2) 由图可知, 函数值小于等于 2 的解集为 $[1,4]$", "solution": "null", "level": "高一", "question": "如图, 函数 $f(x)$ 的图象是折线段 $A B C$, 其中 $A, B, C$ 的坐标分别为 $(0,4),(2,0),(6,4)$, 则 $f[f(2)]=$ ; 不等式 $f(x) \\leq 2$ 的解集为\n\n", "options": [], "subject": "解析几何", "analysis": "(1) 由图可知, $f(2)=0$, 所以 $f[f(2)]=f(0)=4$\n\n(2) 由图可知, 函数值小于等于 2 的解集为 $[1,4]$"} {"id": "14473", "image": [], "answer": "$[3,+\\infty)$", "solution": "null", "level": "高一", "question": "函数 $f(x)=\\sqrt{x^{2}-2 x-3}$ 的单调递增区间为", "options": [], "subject": "解析几何", "analysis": "【详解】\n\n令 $x^{2}-2 x-3 \\geq 0$, 解得 $x \\leq-1$ 或 $x \\geq 3$,\n\n函数 $f(x)=\\sqrt{x^{2}-2 x-3}$ 的定义域为 $(-\\infty,-1] \\cup[3,+\\infty)$.\n\n内层函数 $u=x^{2}-2 x-3$ 的减区间为 $(-\\infty,-1]$, 增区间为 $[3,+\\infty)$.\n\n外层函数 $y=\\sqrt{u}$ 在 $[0,+\\infty)$ 上为增函数,\n\n由复合函数法可知, 函数 $f(x)=\\sqrt{x^{2}-2 x-3}$ 的单调递增区间为 $[3,+\\infty)$.\n\n故答案为 $[3,+\\infty)$.\n\n【点睛】\n\n本题考查函数单调区间的求解, 常用的方法有复合函数法、图象法, 另外在求单调区间时, 首先应求函数的定义域, 考查分析问题和解决问题的能力, 属于中等题."} {"id": "15357", "image": [], "answer": "-4", "solution": "null", "level": "高一", "question": "函数 $f(x)=\\frac{a}{x}+b x+3$ (a, b 均为正数), 若 $\\mathbf{f}(\\mathbf{x})$ 在 $(\\mathbf{0},+\\infty)$ 上有最小值 $\\mathbf{1 0}$, 则 $\\mathbf{f}(\\mathrm{x})$ 在 $(-\\infty, 0)$ 上的最大值为", "options": [], "subject": "解析几何", "analysis": "函数 ( $\\mathrm{a}, \\mathrm{b}$ 均为正数),\n\n$$\nf(x)=\\frac{a}{x}+b x+3\n$$\n\n可设 $g(x)=\\frac{\\underline{a}}{\\underline{a}}+b x$, 可得 $g(-x)=-\\left(\\frac{a}{x}+b x\\right)=-g(x)$ ,\n\n即 $\\mathrm{g}(\\mathrm{x})$ 为奇函数,\n\n设 $g(x)$ 在 $x>0$ 的最小值为 $m$, 在 $x<0$ 的最大值为 $n$,\n\n且 $m+n=0$,\n\n由 $\\mathrm{f}(\\mathrm{x})$ 在 $(0,+\\infty)$ 上有最小值 10 ,\n\n可得 $m+3=10$,\n\n即 $m=7$, 可得 $n=-7$,\n\n则 $f(x)$ 在 $(-\\infty, 0)$ 上的最大值为 $-7+3=-4$.\n\n故答案为: -4 ."} {"id": "15416", "image": [], "answer": "3", "solution": "null", "level": "高一", "question": "三条平行直线最多能确定的平面的个数为", "options": [], "subject": "度量几何学", "analysis": "当三条平行直线在一个平面内时,可以确定 1 个平面;当三条平行直线不在同一平面上时,可以确定 3 个平面. 综上最多可确定 3 个平面."} {"id": "15467", "image": [], "answer": "平行", "solution": "null", "level": "高一", "question": "已知平面 $\\alpha$ 和 $\\beta$, 在平面 $\\alpha$ 内任取一条直线 $a$, 在 $\\beta$ 内总存在直线 $b / / a$, 则 $\\alpha$ 与 $\\beta$ 的位置关系是 \\$ \\qquad \\$ . (填 “平行” 或 “相交”)", "options": [], "subject": "度量几何学", "analysis": "若 $\\alpha \\cap \\beta=l$ ,则在平面 $\\alpha$ 内,与 $l$ 相交的直线 $a$ ,\n\n设 $a \\cap l=A$ ,对于 $\\beta$ 内的任意直线 $b$ ,\n\n若 $b$ 过点 $A$ ,则 $a$ 与 $b$ 相交,若 $b$ 不过点 $A$ ,则 $a$ 与 $b$ 异面,即 $\\beta$ 内不存在直线 $b / / a$ ,矛盾.\n\n故 $\\alpha / / \\beta$."} {"id": "17000", "image": ["8761.jpg", "8762.jpg", "8762.jpg"], "answer": "$\\frac{1}{4 \\pi}$.\n\n", "solution": "null", "level": "高一", "question": "(2021・十五模拟)如图是一个圆形射击靶的示意图, 靶心为圆心 $O$, 半径为 2 分米. 一名运动员在练习射击的时候, 在靶上画了一个标志胜利的 “ $V$ ” 形轴对称图案 $A O B C$, 其中 $\\angle A O B=60^{\\circ}$,点 $A, B$ 在圆形靶的边缘上, 点 $C$ 与靶的边缘的最短距离为 1 分米. 该运动员朝靶上任意射击一次,没有脱靶,则命中靶中 “ $V$ ” 形图案的概率为 \\$ \\qquad \\$\n\n", "options": [], "subject": "度量几何学", "analysis": "解: 连接 $A B$, 由题意可得 $\\triangle A O B$ 是边长为 2 的等边三角形,\n\n则靶中 “ $V$ ” 形图案的面积为 $S_{\\triangle A B C}-S_{\\triangle A O B}=\\frac{1}{2} \\times 2 \\times(1+\\sqrt{3})-\\frac{1}{2} \\times 2 \\times \\sqrt{3}=1$,\n\n又圆形靶的面积为 $4 \\pi$,\n\n根据几何概型的概率计算公式得命中靶中 “ $V$ ” 形图案的概率为 $\\frac{1}{4 \\pi}$,\n\n故答案为: $\\frac{1}{4 \\pi}$.\n\n"} {"id": "17001", "image": ["8763.jpg", "8763.jpg"], "answer": "frac{5 \\sqrt{6} \\pi}{96}$.\n\n", "solution": "null", "level": "高一", "question": "020 秋・二道区校级期末)在矩形 $A B C D$ 中, $A B=5, A C=7$, 现向该矩形 $A B C D$ 内随机投一点 $P$, 则 $\\angle A P B>90^{\\circ}$ 的概率为 \\$ \\qquad \\$ .", "options": [], "subject": "度量几何学", "analysis": "由题意, $A D=\\sqrt{A C^{2}-A B^{2}}=2 \\sqrt{6}$, 矩形的面积为 $10 \\sqrt{6}$, 如图而使 $\\angle A P B>90^{\\circ}$ 成立的区域为以 $A B$ 为直径的半圆, 面积为 $\\frac{1}{2} \\pi\\left(\\frac{5}{2}\\right)^{2}=\\frac{25 \\pi}{8}$,由几何概型公式得到向该矩形 $A B C D$ 内随机投一点 $P$, 则 $\\angle A P B>90^{\\circ}$ 的概率为: $\\frac{\\frac{25 \\pi}{8}}{10 \\sqrt{6}}=\\frac{5 \\sqrt{6} \\pi}{96}$;故答案为: $\\frac{5 \\sqrt{6} \\pi}{96}$.\n\n"} {"id": "15417", "image": [], "answer": "1 或 4", "solution": "null", "level": "高一", "question": "已知空间四点中无任何三点共线, 那么这四点可以确定平面的个数是", "options": [], "subject": "立体几何学", "analysis": "其中三个点可确定唯一的平面,当第四个点在此平面内时,可确定 1 个平面,当第四个点不在此平面内时,则可确定 4 个平面."} {"id": "15418", "image": ["7655.jpg"], "answer": "三点共线", "solution": "null", "level": "高一", "question": "若直线 $l$ 与平面 $\\alpha$ 相交于点 $O, A, B \\in l, C, D \\in \\alpha$, 且 $A C / / B D$, 则 $O, C, D$ 三点的位置关系是", "options": [], "subject": "立体几何学", "analysis": "because A C / / B D$ ,\n\n$\\therefore A C$ 与 $B D$ 确定一个平面,记作平面 $\\beta$ ,则 $\\alpha \\cap \\beta=$ 直线 $C D$.\n\n\n\n$\\because l \\cap \\alpha=O, \\quad \\therefore O \\in \\alpha$.\n\n又 $\\because O \\in A B \\subset \\beta$,\n$\\therefore O \\in$ 直线 $C D$ ,\n\n$\\therefore O, C, D$ 三点共线."} {"id": "15427", "image": [], "answer": "(3)", "solution": "null", "level": "高一", "question": "线 $l$ 上有两个点在平面 $\\alpha$ 内, 则下列说法中正确的序号为\n\n(1)直线 $l$ 上至少有一个点在平面 $\\alpha$ 外;\n\n(2)直线 $l$ 上有无穷多个点在平面 $\\alpha$ 外;\n\n(3)直线 $l$ 上所有点都在平面 $\\alpha$ 内;\n\n(4)直线 $l$ 上至多有两个点在平面 $\\alpha$ 内", "options": [], "subject": "立体几何学", "analysis": ""} {"id": "15429", "image": ["7659.jpg", "7660.jpg"], "answer": "5", "solution": "null", "level": "高一", "question": "如图, 空间四边形 $A B C D$ 的对角线 $A C=8, B D=6, M, N$ 分别为 $A B, C D$ 的中点, 并且异面直线 $A C$ 与 $B D$ 所成的角为 $90^{\\circ}$, 则 $M N=$\n\n", "options": [], "subject": "立体几何学", "analysis": "取 $A D$ 的中点 $P$ ,连接 $P M , P N$ ,\n\n\n\n则 $B D / / P M , A C / / P N$ ,\n\n$\\therefore \\angle M P N$ 即为异面直线 $A C$ 与 $B D$ 所成的角,\n\n$\\therefore \\angle M P N=90^{\\circ}$,\n\n$P N=\\frac{1}{2} A C=4, \\quad P M=\\frac{1}{2} B D=3$,\n\n$\\therefore M N=5$."} {"id": "15430", "image": ["7661.jpg"], "answer": "(1)(3)", "solution": "null", "level": "高一", "question": "正方体纸盒展开后如图所示, 在原正方体纸盒中有如下结论:\n\n\n\n(1) $A B \\perp E F$;\n\n(2) $A B$ 与 $C M$ 所成的角为 $60^{\\circ}$;\n\n(3)EF 与 $M N$ 是异面直线;\n\n(4) $M N / / C D$.\n\n以上结论正确的为 \\$ \\qquad \\$ . (填序号)", "options": [], "subject": "立体几何学", "analysis": "方体的平面展开图还原成原来的正方体可知, $A B \\perp E F , E F$ 与 $M N$ 是异面直线, $A B / / C M , M N \\perp C D$ ,只有(1)(3)正确."} {"id": "15440", "image": ["7671.jpg"], "answer": "(3) (4)", "solution": "null", "level": "高一", "question": "如图所示, 在正方体 $A B C D-A_{1} B_{1} C_{1} D_{1}$ 中, $M, N$ 分别为棱 $C_{1} D_{1}, C_{1} C$ 的中点, 有以下四个结论:\n\n\n\n(1)直线 $A M$ 与 $C C_{1}$ 是相交直线;\n\n(2)直线 $A M$ 与 $B N$ 是平行直线;\n\n(3)直线 $B N$ 与 $M B_{1}$ 是异面直线;\n\n(4)直线 $A M$ 与 $D D_{1}$ 是异面直线.\n\n其中正确的结论为 . (填序号)", "options": [], "subject": "立体几何学", "analysis": "直线 $A M$ 与 $C C_{1}$ 是异面直线,直线 $A M$ 与 $B N$ 也是异面直线,故(1)(2)错误; (3)(4)正确."} {"id": "15442", "image": [], "answer": "相交", "solution": "null", "level": "高一", "question": "若点 $A \\in \\alpha, B \\notin \\alpha, C \\notin \\alpha$, 则平面 $A B C$ 与平面 $\\alpha$ 的位置关系是", "options": [], "subject": "立体几何学", "analysis": "$\\because$ 点 $A \\in \\alpha , B \\notin \\alpha , C \\notin \\alpha$ ,\n\n$\\therefore$ 平面 $A B C$ 与平面 $\\alpha$ 有公共点,且不重合,\n$\\therefore$ 平面 $A B C$ 与平面 $\\alpha$ 的位置关系是相交."} {"id": "15443", "image": [], "answer": "(3) (4)", "solution": "null", "level": "高一", "question": "下列说法中正确的是 \\$ \\qquad \\$ . (填序号)\n\n(1)若直线 $a$ 不在平面 $\\alpha$ 内,则 $a / / \\alpha$;\n\n(2)若直线 $l$ 上有无数个点不在平面 $\\alpha$ 内, 则 $l / /$;\n\n(3)若直线 $l$ 与平面 $\\alpha$ 平行, 则 $l$ 与 $\\alpha$ 内任何一条直线都没有公共点;\n\n(4)平行于同一平面的两条直线可以相交.", "options": [], "subject": "立体几何学", "analysis": "当 $a \\cap \\alpha=A$ 时, $a \\downarrow \\alpha$ ,故(1)错; 当直线 $l$ 与 $\\alpha$ 相交时, $l$ 上有无数个点不在 $\\alpha$ 内,故(2)错;若 $l / / \\alpha$ ,则 $l$ 与 $\\alpha$ 无公共点,则 $l$ 与 $\\alpha$ 内任何一条直线都无公共点,故(3)正确;在长方体 $A B C D-A_{1} B_{1} C_{1} D_{1}$ 中, $A_{1} C_{1}$ 与 $B_{1} D_{1}$ 相交,且都与平面 $A B C D$ 平行,故(4)正确. 故答案为 (3) (4)."} {"id": "15444", "image": [], "answer": "8", "solution": "null", "level": "高一", "question": "互不重合的三个平面最多可以把空间分成 \\$ \\qquad \\$个部分.", "options": [], "subject": "立体几何学", "analysis": "重合的三个平面将空间分成五种情形: 当三个平面互相平行时,将空间分成四部分;当两个平面平行,第三个平面与它们相交时,将空间分成六部分;当三个平面相交于同一条直线时,将空间分成六部分; 当三个平面相交于三条直线时,且三条交线交于同一点时,将空间分成八个部分;当三个平面相交于三条直线,且三条交线互相平行时,将空间分成七部分. 即不重合的三个平面可以将空间分成四部分或六部分或七部分或八部分. 所以最多将空间分成 8 部分."} {"id": "15452", "image": [], "answer": "(3)", "solution": "null", "level": "高一", "question": "已知下列说法:\n\n(1)若两个平面 $\\alpha / / \\beta, a \\subset \\alpha, b \\subset \\beta$, 则 $a / / b$;\n\n(2)若两个平面 $\\alpha / / \\beta, a \\subset \\alpha, b \\subset \\beta$, 则 $a$ 与 $b$ 是异面直线;\n\n(3)若两个平面 $\\alpha / / \\beta, a \\subset \\alpha, b \\subset \\beta$, 则 $a$ 与 $b$ 平行或异面;\n\n(4)若两个平面 $\\alpha \\cap \\beta=b, a \\subset \\alpha$, 则 $a$ 与 $\\beta$ 一定相交.\n\n其中正确的序号是", "options": [], "subject": "立体几何学", "analysis": "(1)错, $a$ 与 $b$ 也可能异面;\n\n(2)错, $a$ 与 $b$ 也可能平行;\n\n(3)对, $\\because \\alpha / / \\beta , \\therefore \\alpha$ 与 $\\beta$ 无公共点,又 $\\because a \\subset \\alpha , b \\subset \\beta , \\therefore a$ 与 $b$ 无公共点,那么 $a / / b$ 或 $a$ 与 $b$ 异面;\n\n(4)错, $a$ 与 $\\beta$ 也可能平行."} {"id": "15454", "image": ["7672.jpg"], "answer": "平行", "solution": "null", "level": "高一", "question": "三棱雉 $S A B C$ 中, $G$ 为 $\\triangle A B C$ 的重心, $E$ 在棱 $S A$ 上, 且 $A E=2 E S$, 则 $E G$ 与平面 $S B C$的关系为", "options": [], "subject": "立体几何学", "analysis": "如图, 延长 $A G$ 交 $B C$ 于 $F$, 连接 $S F$, 则由 $G$ 为 $\\triangle A B C$ 的重心知 $A G: G F=2$, 又 $A E: E S$ $=2 , \\therefore E G / / S F$ ,又 $S F \\subset$ 平面 $S B C , E G$ 平面 $S B C$ ,\n\n\n\n$\\therefore E G / /$ 平面 $S B C$."} {"id": "15455", "image": ["7673.jpg"], "answer": "平行", "solution": "null", "level": "高一", "question": "如图, 在五面体 $F E-A B C D$ 中, 四边形 $C D E F$ 为矩形, $M, N$ 分别是 $B F, B C$ 的中点,则 $M N$ 与平面 $A D E$ 的位置关系是 \\$ \\qquad \\$ .\n\n", "options": [], "subject": "立体几何学", "analysis": "because M , N$ 分别是 $B F , B C$ 的中点, $\\therefore M N / / C F$.\n\n## 又四边形 $C D E F$ 为矩形,\n\n$\\therefore C F / / D E$,\n\n$\\therefore M N / / D E$.\n\n又 $M N \\nmid$ 平面 $A D E , D E \\subset$ 平面 $A D E , \\therefore M N / /$ 平面 $A D E$."} {"id": "15465", "image": [], "answer": "$l \\Varangle \\alpha$", "solution": "null", "level": "高一", "question": "已知 $l, m$ 是两条直线, $\\alpha$ 是平面, 若要得到 “ $l / / \\alpha$ ”, 则需要在条件 “ $m \\subset \\alpha, l / / m$ ” 中另外添加的一个条件是", "options": [], "subject": "立体几何学", "analysis": ""} {"id": "15468", "image": ["7686.jpg", "7687.jpg"], "answer": "(1)(2)(3)", "solution": "null", "level": "高一", "question": "如图是正方体的平面展开图. 在这个正方体中,\n\n\n\n(1) $B M / /$ 平面 $D E$; (2) $C N / /$ 平面 $A F$; (3)平面 $B D M / /$ 平面 $A F N$; (4) 平面 $B D E / /$ 平面 $N C F$.以上四个命题中, 正确命题的序号是 \\$ \\qquad \\$ .", "options": [], "subject": "立体几何学", "analysis": "$A B C D$ 为下底面还原正方体,如图:\n\n\n\n则易判定四个命题都是正确的."} {"id": "15478", "image": [], "answer": "相交或平行", "solution": "null", "level": "高一", "question": "已知平面 $\\alpha, \\beta$ 和直线 $a, b, c$, 且 $a / / b / / c, a \\subset \\alpha, b, c \\subset \\beta$, 则 $\\alpha$ 与 $\\beta$ 的关系是", "options": [], "subject": "立体几何学", "analysis": "$b , c \\subset \\beta a \\subset \\alpha , a / / b / / c$ ,若 $\\alpha / / \\beta$ ,满足要求; 若 $\\alpha$ 与 $\\beta$ 相交,交线为 $l, b / / c / / l , a / / l$ ,\n满足要求,故答案为相交或平行."} {"id": "15480", "image": ["7699.jpg"], "answer": "平行四边形", "solution": "null", "level": "高一", "question": "如图, 已知 $A, B, C, D$ 四点不共面, 且 $A B / / \\alpha, C D / / \\alpha, A C \\cap \\alpha=E, A D \\cap \\alpha=F, B D \\cap \\alpha$ $=H, B C \\cap \\alpha=G$, 则四边形 $E F H G$ 的形状是 \\$ \\qquad \\$ .\n\n", "options": [], "subject": "立体几何学", "analysis": "$\\because A B / / \\alpha$,\n\n平面 $A B C \\cap \\alpha=E G ,$\n\n$\\therefore E G / / A B$.同理 $F H / / A B$ ,\n\n$\\therefore E G / / F H$.又 $C D / / \\alpha$ ,平面 $B C D \\cap \\alpha=G H$ ,\n\n$\\therefore G H / / C D$.同理 $E F / / C D$ ,\n\n$\\therefore G H / / E F$ ,\n\n$\\therefore$ 四边形 $E F H G$ 是平行四边形."} {"id": "15481", "image": ["7700.jpg"], "answer": "$4 \\sqrt{5}+6 \\sqrt{2}$", "solution": "null", "level": "高一", "question": "如图所示的正方体的棱长为 4 , 点 $E, F$ 分别为 $A_{1} D_{1}, A A_{1}$ 的中点, 则过 $C_{1}, E, F$ 的截面的周长为 \\$ \\qquad \\$\n\n", "options": [], "subject": "立体几何学", "analysis": "$E F / /$ 平面 $B C C_{1} B_{1}$ 可知平面 $B C C_{1} B_{1}$ 与平面 $E F C_{1}$ 的交线为 $B C_{1}$ ,平面 $E F C_{1}$ 与平面 $A B B_{1} A_{1}$ 的交线为 $B F$ ,所以截面周长为 $E F+F B+B C_{1}+C_{1} E=4 \\sqrt{5}+6 \\sqrt{2}$."} {"id": "15489", "image": ["7708.jpg"], "answer": "$\\frac{2 \\sqrt{2}}{3} a$", "solution": "null", "level": "高一", "question": "如图所示, $A B C D-A_{1} B_{1} C_{1} D_{1}$ 是棱长为 $a$ 的正方体, $M, N$ 分别是下底面的棱 $A_{1} B_{1}, B_{1} C_{1}$的中点, $P$ 是上底面的棱 $A D$ 上的一点, $A P=\\frac{a}{3}$, 过 $P, M, N$ 的平面交上底面于 $P Q, Q$ 在 $C D$ 上,则 $P Q=$ \\$ \\qquad \\$ .\n\n", "options": [], "subject": "立体几何学", "analysis": "$\\because M N / /$ 平面 $A C$ ,平面 $P M N \\cap$ 平面 $A C=P Q$ ,\n\n$\\therefore M N / / P Q$ ,易知 $D P=D Q=\\frac{2 a}{3}$ ,\n\n故 $P Q=\\sqrt{P D^{2}+D Q^{2}}=\\sqrt{2} D P=\\frac{2 \\sqrt{2} a}{3}$."} {"id": "15490", "image": [], "answer": "0 或 1", "solution": "null", "level": "高一", "question": "直线 $a / /$ 平面 $\\alpha, \\alpha$ 内有 $n$ 条直线交于一点, 则这 $n$ 条直线中与直线 $a$ 平行的直线有_条.", "options": [], "subject": "立体几何学", "analysis": "过直线 $a$ 与交点作平面 $\\beta$ ,设平面 $\\beta$ 与 $\\alpha$ 交于直线 $b$ ,则 $a / / b$ ,若所给 $n$ 条直线中有 1 条是与 $b$ 重合的,则此直线与直线 $a$ 平行,若没有与 $b$ 重合的,则与直线 $a$ 平行的直线有 0 条."} {"id": "15492", "image": ["7709.jpg"], "answer": "$\\frac{4 \\sqrt{3}}{9}$", "solution": "null", "level": "高一", "question": "如图所示, 平面 $\\alpha / /$ 平面 $\\beta, \\triangle A B C, \\triangle A^{\\prime} B^{\\prime} C^{\\prime}$ 分别在 $\\alpha, \\beta$ 内, 线段 $A A^{\\prime}, B B^{\\prime}$, $C C^{\\prime}$ 共点于 $O, O$ 在平面 $\\alpha$ 和平面 $\\beta$ 之间, 若 $A B=2, A C=2, \\angle B A C=60^{\\circ}, O A: O A^{\\prime}=$ $3: 2$, 则 $\\triangle A^{\\prime} B^{\\prime} C^{\\prime}$ 的面积为 \\$ \\qquad \\$ .\n\n", "options": [], "subject": "立体几何学", "analysis": "$A A^{\\prime} , B B^{\\prime}$ 相交于 $O$ ,所以 $A A^{\\prime} , B B^{\\prime}$ 确定的平面与平面 $\\alpha$ ,平面 $\\beta$ 的交线分别为 $A B , A^{\\prime} B^{\\prime}$ ,有 $A B / / A^{\\prime} B^{\\prime}$ ,\n\n且 $\\frac{O A}{O A^{\\prime}}=\\frac{A B}{A^{\\prime} B^{\\prime}}=\\frac{3}{2}$ ,同理可得 $\\frac{O A}{O A^{\\prime}}=\\frac{A C}{A^{\\prime} C^{\\prime}}=\\frac{3}{2}, \\frac{O A}{O A^{\\prime}}=\\frac{B C}{B^{\\prime} C^{\\prime}}=\\frac{3}{2}$, 所以 $\\triangle A B C$ ,\n$\\triangle A^{\\prime} B^{\\prime} C^{\\prime}$ 面积的比为 $9: 4$, 又 $\\triangle A B C$ 的面积为 $\\sqrt{3}$, 所以 $\\triangle A^{\\prime} B^{\\prime} C^{\\prime}$ 的面积为 $\\frac{4 \\sqrt{3}}{9}$."} {"id": "15493", "image": ["7710.jpg"], "answer": "(1)", "solution": "null", "level": "高一", "question": "如图, 在多面体 $A B C-D E F G$ 中, 平面 $A B C / /$ 平面 $D E F G, A D / / B E, A C / / D G / / E F$,且 $A B=D E, D G=2 E F$, 则下列说法中正确的是 \\$ \\qquad \\$ . (填序号)\n\n\n\n(1) $B F / /$ 平面 $A C G D$;\n\n(2) $C F / /$ 平面 $A B E D$;\n\n(3) $B C / / F G$;\n\n(4)平面 $A B E D / /$ 平面 $C G F$.", "options": [], "subject": "立体几何学", "analysis": "$\\because E F / / D G, B E / / A D, B E \\cap E F=E$ ,\n\n$A D \\cap D G=D ,$\n\n$\\therefore$ 平面 $B E F / /$ 平面 $A D G C$.\n\n$\\because B F \\subset$ 平面 $B E F , \\therefore B F / /$ 平面 $A C G D$ ,故(1)正确;\n\n由于 $D G=2 E F$ ,则四边形 $E F G D$ 是梯形,\n\n$G F$ 的延长线必与直线 $D E$ 相交,\n\n故(4)不正确;\n\n选项(2)(3)不能推出."} {"id": "15494", "image": ["7711.jpg"], "answer": "$\\frac{\\sqrt{3}}{2}$", "solution": "null", "level": "高一", "question": "如图, 四棱雉 $P-A B C D$ 的底面是平行四边形, $P A=P B=A B=2, E, F$ 分别是 $A B, C D$的中点, 平面 $A G F / /$ 平面 $P E C, P D \\cap$ 平面 $A G F=G, E D$ 与 $A F$ 相交于点 $H$, 则 $G H=$\n\n", "options": [], "subject": "立体几何学", "analysis": "$A B C D$ 是平行四边形,\n\n$\\therefore A B / / C D$ ,且 $A B=C D ,$\n又 $E, F$ 分别是 $A B, C D$ 的中点,\n\n$\\therefore A E=F D$ ,\n\n又 $\\angle E A H=\\angle D F H$ ,\n\n$\\angle A E H=\\angle F D H$,\n\n$\\therefore \\triangle A E H \\cong \\triangle F D H$,\n\n$\\therefore E H=D H$.\n\n$\\because$ 平面 $A G F / /$ 平面 $P E C$ ,\n\n又平面 $P E D \\cap$ 平面 $A G F=G H$ ,\n\n平面 $P E D \\cap$ 平面 $P E C=P E$ ,\n\n$\\therefore G H / / P E$ ,\n\n则 $G$ 是 $P D$ 的中点.\n\n$\\because P A=P B=A B=2$,\n\n$\\therefore P E=2 \\times \\sin 60^{\\circ}=\\sqrt{3}$,\n\n$\\therefore G H=\\frac{1}{2} P E=\\frac{\\sqrt{3}}{2}$."} {"id": "15503", "image": ["7717.jpg"], "answer": "$\\frac{1}{2}$", "solution": "null", "level": "高一", "question": "如图, 在长方体 $A B C D-A_{1} B_{1} C_{1} D_{1}$ 中, 过 $B B_{1}$ 的中点 $E$ 作一个与平面 $A C B_{1}$ 平行的平面交 $A B$ 于 $M$, 交 $B C$ 于 $N$, 则 $\\frac{M N}{A C}=$ \\$ \\qquad \\$ .\n\n", "options": [], "subject": "立体几何学", "analysis": "$\\because$ 平面 $M N E / /$ 平面 $A C B_{1}$,\n\n由面面平行的性质定理可得 $E N / / B_{1} C , E M / / B_{1} A$ ,\n\n又 $\\because E$ 为 $B B_{1}$ 的中点, $\\therefore M, N$ 分别为 $B A , B C$ 的中点,\n\n$\\therefore M N=\\frac{1}{2} A C$ ,即 $\\frac{M N}{A C}=\\frac{1}{2}$."} {"id": "15505", "image": ["7719.jpg", "7720.jpg"], "answer": "$\\angle A_{1} C_{1} B_{1}=90^{\\circ}$", "solution": "null", "level": "高一", "question": "如图, 在直三棱柱 $A B C-A_{1} B_{1} C_{1}$ 中, $B C=C C_{1}$, 当底面 $A_{1} B_{1} C_{1}$ 满足条件 \\$ \\qquad \\$时,有 $A B_{1} \\perp B C_{1}$. (注: 填上你认为正确的一种条件即可, 不必考虑所有可能的情况)\n\n", "options": [], "subject": "立体几何学", "analysis": "如图所示,连接 $B_{1} C$ ,由 $B C=C C_{1}$ ,可得 $B C_{1} \\perp B_{1} C$ ,因此,要证 $A B_{1} \\perp B C_{1}$ ,则只要证明 $B C_{1} \\perp$ 平面 $A B_{1} C$ ,即只要证 $A C \\perp B C_{1}$ 即可,由直三棱柱可知,只要证 $A C \\perp B C$ 即可. 因为 $A_{1} C_{1} / / A C , B_{1} C_{1} / / B C$ ,故只要证 $A_{1} C_{1} \\perp B_{1} C_{1}$ 即可. (或者能推出 $A_{1} C_{1} \\perp B_{1} C_{1}$ 的条件,如 $\\angle A_{1} C_{1} B_{1}=90^{\\circ}$ 等)\n\n"} {"id": "15506", "image": [], "answer": "$90^{\\circ}$\n\n#", "solution": "null", "level": "高一", "question": "在正方体 $A B C D-A_{1} B_{1} C_{1} D_{1}$ 中, $M, N$ 分别是棱 $A A_{1}$ 和 $A B$ 上的点, 若 $\\angle B_{1} M N$ 是直角,则 $\\angle C_{1} M N=$", "options": [], "subject": "立体几何学", "analysis": "体 $A B C D-A_{1} B_{1} C_{1} D_{1}$ 中, $M, N$ 分别是棱 $A A_{1}$ 和 $A B$ 上的点, 若 $\\angle B_{1} M N$ 是直角,则 $\\angle C_{1} M N=$\n\n答案 $90^{\\circ}$"} {"id": "15515", "image": ["7727.jpg"], "answer": "4", "solution": "null", "level": "高一", "question": "如图, 已知四棱雉 $P-A B C D$ 中, 底面 $A B C D$ 为矩形, $P A \\perp$ 平面 $A B C D$,则图中共有直角三角形的个数为 \\$ \\qquad \\$\n\n", "options": [], "subject": "立体几何学", "analysis": "$\\because P A \\perp$ 平面 $A B C D ,$\n\n$\\therefore P A \\perp B C$,\n\n又 $B C \\perp A B$ ,\n\n$\\therefore B C \\perp$ 平面 $P A B$.\n\n$\\therefore B C \\perp P B$,\n\n同理得 $C D \\perp P D$ ,\n\n## 故共有 4 个直角三角形."} {"id": "15516", "image": ["7728.jpg"], "answer": "2", "solution": "null", "level": "高一", "question": "所示, $A B$ 是 $\\odot O$ 的直径, $P A \\perp \\odot O$ 所在的平面, $C$ 是圆上一点, 且 $\\angle A B C=30^{\\circ}$, $P A=A B$ ,则直线 $P C$ 与平面 $A B C$ 所成角的正切值为 \\$ \\qquad \\$ .\n\n", "options": [], "subject": "立体几何学", "analysis": "因为 $P A \\perp$ 平面 $A B C$ ,所以 $A C$ 为斜线 $P C$ 在平面 $A B C$ 上的射影,所以 $\\angle P C A$ 即为 $P C$与平面 $A B C$ 所成的角. 在 Rt $\\triangle P A C$ 中, $A C=\\frac{1}{2} A B=\\frac{1}{2} P A$, 所以 $\\tan \\angle P C A=\\frac{P A}{A C}=2$."} {"id": "15519", "image": ["7730.jpg"], "answer": "$D M \\perp P C$ (或 $B M \\perp P C$ 等)", "solution": "null", "level": "高一", "question": "如图所示, 在四棱雉 $P-A B C D$ 中, $P A \\perp$ 底面 $A B C D$, 且底面各边都相等, $M$ 是 $P C$ 上的一动点, 当点 $M$ 满足 \\$ \\qquad \\$时,平面 $M B D \\perp$ 平面 $P C D$. (只要填写一个你认为是正确的条件即可)\n\n", "options": [], "subject": "立体几何学", "analysis": "意得 $B D \\perp A C$ ,\n\n$\\because P A \\perp$ 平面 $A B C D ,$\n\n$\\therefore P A \\perp B D$.\n\n又 $P A \\cap A C=A$ ,\n\n$\\therefore B D \\perp$ 平面 $P A C$ ,\n\n$\\therefore B D \\perp P C$.\n\n$\\therefore$ 当 $D M \\perp P C($ 或 $B M \\perp P C)$ 时,\n\n即有 $P C \\perp$ 平面 $M B D ,$\n\n而 $P C \\subset$ 平面 $P C D$,\n\n$\\therefore$ 平面 $M B D \\perp$ 平面 $P C D$."} {"id": "15529", "image": [], "answer": "(1)(3)(4) $\\Rightarrow$ (2)", "solution": "null", "level": "高一", "question": "$\\alpha, \\beta$ 是两个不同的平面, $m, n$ 是平面 $\\alpha$ 及 $\\beta$ 之外的两条不同直线, 给出四个论断:\n\n(1) $m \\perp n$; (2) $\\alpha \\perp \\beta$; (3) $n \\perp \\beta$; (4) $m \\perp \\alpha$.\n\n以其中三个论断作为条件, 余下一个论断作为结论, 写出你认为正确的一个命题", "options": [], "subject": "立体几何学", "analysis": "$m \\perp n$ ,将 $m$ 和 $n$ 平移到一起,则确定一平面,\n\n$\\because n \\perp \\beta, m \\perp \\alpha$,\n\n$\\therefore$ 该平面与平面 $\\alpha$ 和平面 $\\beta$ 的交线也互相垂直,\n\n从而平面 $\\alpha$ 和平面 $\\beta$ 的二面角的平面角为 $90^{\\circ} , \\therefore \\alpha \\perp \\beta$.\n\n故答案为(1)(3)(4) $\\Rightarrow$ (2)."} {"id": "15531", "image": ["7739.jpg"], "answer": "6", "solution": "null", "level": "高一", "question": "如图, 平面 $A B C \\perp$ 平面 $A B D, \\angle A C B=90^{\\circ}, C A=C B, \\triangle A B D$ 是正三角形, $O$ 为 $A B$中点, 则图中直角三角形的个数为 \\$ \\qquad \\$ .\n\n", "options": [], "subject": "立体几何学", "analysis": "$\\because C A=C B, O$ 为 $A B$ 的中点, $\\therefore C O \\perp A B$.\n又平面 $A B C \\perp$ 平面 $A B D ,$ 平面 $A B C \\cap$ 平面 $A B D=A B, C O \\subset$ 平面 $A B C$ ,\n\n$\\therefore C O \\perp$ 平面 $A B D$.\n\n$\\because O D \\subset$ 平面 $A B D , \\therefore C O \\perp O D , \\therefore \\triangle C O D$ 为直角三角形.\n\n$\\therefore$ 图中的直角三角形有 $\\triangle A O C , \\triangle C O B , \\triangle A B C , \\triangle A O D , \\triangle B O D , \\triangle C O D$ 共 6 个."} {"id": "15532", "image": ["7740.jpg"], "answer": "(2)(4)", "solution": "null", "level": "高一", "question": "如图所示, $A B$ 为圆 $O$ 的直径, 点 $C$ 在圆周上(异于点 $A, B$ ), 直线 $P A$ 垂直于圆 $O$ 所在的平面, 点 $M$ 为线段 $P B$ 的中点. 有以下四个命题: (1) $P A / /$ 平面 $M O B$; (2) $M O / /$ 平面 $P A C$; (3) $O C \\perp$ 平面 $P A C$; (4)平面 $P A C \\perp$ 平面 $P B C$.其中正确的命题是 . (填上所有正确命题的序号)\n\n", "options": [], "subject": "立体几何学", "analysis": "$P A \\subset$ 平面 $M O B$ ,所以(1)不正确;因为 $M O / / P A$ ,而且 $M O \\Varangle$ 平面 $P A C$ ,所以(2)正确; $O C$ 不垂直于 $A C$ ,所以(3)不正确; 因为 $B C \\perp A C , B C \\perp P A , A C \\cap P A=A$ ,所以 $B C \\perp$ 平面 $P A C$ ,所以平面 $P A C \\perp$ 平面 $P B C$ ,所以(4)正确."} {"id": "15542", "image": [], "answer": "平行", "solution": "null", "level": "高一", "question": "$a, b$ 是异面直线, 直线 $l \\perp a, l \\perp b$, 直线 $m \\perp a, m \\perp b$, 则 $l$ 与 $m$ 的位置关系是", "options": [], "subject": "立体几何学", "analysis": "由线面垂直的性质定理可得."} {"id": "17159", "image": ["8806.jpg", "8806.jpg"], "answer": "frac{1}{3}$\n\n", "solution": "null", "level": "高一", "question": "020 春-桃城区校级月考)有一个底面圆的半径为 1 , 高为 2 的圆柱, 点 $O_{1}, O_{2}$ 分别为这个圆柱上底面和下底面的圆心,在这个圆柱内随机取一点 $P$, 则点 $P$ 到点 $O_{1}, O_{2}$ 的距离都大于 1 的概\n 率为 \\$ \\qquad \\$ .", "options": [], "subject": "立体几何学", "analysis": "$\\because$ 到点 $O_{1}$ 的距离等于 1 的点构成一个半个球面, 到点 $O_{2}$ 的距离等于 1 的点构成一个半个球面,两个半球构成一个整球,如图,\n\n点 $P$ 到点 $O_{1}, O_{2}$ 的距离都大于 1 的概率为:\n\n$P=\\frac{\\text { 球外的体积 }}{\\text { 圆柱体柱体积 }- \\text { 球体积 }}=1-\\frac{\\frac{4}{3} \\pi \\times 1^{3}}{\\pi \\times 1^{2} \\times 2}=\\frac{1}{3}$;\n\n故答案为: $\\frac{1}{3}$\n\n"} {"id": "16694", "image": ["8638.jpg"], "answer": "\\frac{19}{27}", "solution": "null", "level": "高一", "question": "四棱雉 $\\mathrm{S}-\\mathrm{ABCD}$ 内任取一点 $\\mathrm{P}$, 则 $V_{P-A B C D}<\\frac{1}{3} V_{S-A B C D}$ 的概率是", "options": [], "subject": "立体几何学", "analysis": "因为四棱雉 $S-A B C D$ 与四棱雉 $\\mathrm{P}-\\mathrm{ABCD}$ 的底面相同, 所以 $V_{P-A B C D}<\\frac{1}{3} V_{S-A B C D}$ 就是四棱雉 $\\mathrm{P}-\\mathrm{ABCD}$ 的高小于四棱雉 $\\mathrm{S}-\\mathrm{ABCD}$ 的高的 $\\frac{1}{3}$, 过四棱雉高 $\\frac{1}{3}$ 分点(靠近底面)作一个平行于底面的截面, 这个截面下任取一点都符合题意, 且\n\n$S_{\\text {截面 }}=\\left(\\frac{2}{3}\\right)^{2} S_{\\text {底面 } A B C D}=\\frac{4}{9} S_{\\text {底面 } A B C D}$\n\n\n\n故答案为:\\frac{19}{27}"} {"id": "15545", "image": [], "answer": "\\pi$", "solution": "null", "level": "高一", "question": "已知 $A, B, C$ 三点在球 $O$ 的表面上, $A B=B C=C A=2$, 且球心 $O$ 到平面 $A B C$ 的距离等于球半径的 $\\frac{1}{3}$, 则球 $O$ 的表面积为", "options": [], "subject": "立体几何学", "analysis": "【分析】\n\n设出球的半径,小圆半径,通过已知条件求出两个半径,再求球的表面积.\n\n## 【详解】\n\n解: 设球的半径为 $r, O^{\\prime}$ 是 $\\triangle A B C$ 的外心, 外接圆半径为 $R=\\frac{2 \\sqrt{3}}{3}$,\n\n$\\because$ 球心 $O$ 到平面 $A B C$ 的距离等于球半径的 $\\frac{1}{3}$,\n\n$\\therefore$ 得 $r^{2}-\\frac{1}{9} r^{2}=\\frac{4}{3}$, 得 $r^{2}=\\frac{3}{2}$.\n\n球的表面积 $S=4 \\pi r^{2}=4 \\pi \\times \\frac{3}{2}=6 \\pi$.\n\n故答案为 $6 \\pi$.\n\n【点睛】\n\n本题考查球 $O$ 的表面积, 考查学生分析问题解决问题能力, 空间想象能力, 是中档题."} {"id": "15546", "image": [], "answer": "frac{3}{4}$", "solution": "null", "level": "高一", "question": "若一圆柱与圆雉的高相等, 且轴截面面积也相等, 那么圆柱与圆雉的体积之比为", "options": [], "subject": "立体几何学", "analysis": "【分析】\n\n根据圆柱与圆雉轴截面面积相等计算出两几何体底面半径之比, 然后利用雉体和柱体的体积公式可计算出这两个几何体的体积之比.\n\n## 【详解】\n\n设圆柱与圆雉的底面半径分别为 $r 、 R$, 高均为 $h$, 圆柱和圆锥的体积分别为 $V_{1} 、 V_{2}$,\n则 $2 r h=\\frac{1}{2} \\times 2 R h, \\therefore R=2 r$,\n\n所以, 圆柱和圆锥的体积之比为 $\\frac{V_{1}}{V_{2}}=\\frac{\\pi r^{2} h}{\\frac{1}{3} \\pi R^{2} h}=\\frac{\\pi r^{2} h}{\\frac{1}{3} \\pi \\times 4 r^{2} h}=\\frac{3}{4}$.\n\n故答案为: $\\frac{3}{4}$.\n\n## 【点睛】\n\n本题考查圆柱和圆雉体积比的计算, 涉及轴截面的计算, 解题的关键就是计算出这两个几何体的底面半径之比, 考查计算能力, 属于基础题."} {"id": "16502", "image": [], "answer": "0", "solution": "null", "level": "高一", "question": "某中学高一有 400 人, 高二有 320 人, 高三有 280 人, 用简单随机抽样的方法抽取一个容量为 $\\mathrm{n}$ 的样本, 已知每个人被抽到的可能性为 0.2 , 则 $\\mathrm{n}=$", "options": [], "subject": "计数", "analysis": "中的个体数 $\\mathrm{N}=1000, \\frac{n}{N}=0.2$ 即可求出样本容量为 200 .故答案为:200"} {"id": "16504", "image": [], "answer": "49", "solution": "null", "level": "高一", "question": "已知某种型号的产品共有 $N$ 件, 且 $40", "options": [], "subject": "计数", "analysis": "由题图知, 底部周长的众数是 $\\frac{100+110}{2}=105$, 中位数是 $\\frac{0.05-0.015-0.025}{0.03} \\times 10+100=\\frac{10}{3}+100=\\frac{310}{3}$故答案为:105,\\frac{310}{3}"} {"id": "16543", "image": [], "answer": "x+3,m+3,n+3", "solution": "null", "level": "高一", "question": "若一组数据的平均数是 $\\mathrm{x}$, 众数是 $\\mathrm{m}$, 中位数是 $\\mathrm{n}$, 那么将每个数据加上 3 后得到一组新数据, 则这组新数据的平均数、众数、中位数分别为", "options": [], "subject": "计数", "analysis": "因为原众数为 $\\mathrm{m}$, 原中位数为 $\\mathrm{n}$, 每个数据加上 3 后, $\\mathrm{m}$ 会变为 $\\mathrm{m}+3, \\mathrm{n}$ 会变为 $n+3$, 所以众数变为 $m+3$, 中位数变为 $n+3$;故答案为:x+3,m+3,n+3"} {"id": "16544", "image": [], "answer": "3.2", "solution": "null", "level": "高一", "question": "某老师从星期一到星期五收到的信件数分别为 $10,6,8,5,6$, 则该组数据的方差 $s^{2}=$", "options": [], "subject": "计数", "analysis": "该组数据的平均数 $\\bar{x}=\\frac{10+6+8+5+6}{5}=7$, . 该组数掘的方差 $s^{2}=\\frac{1}{5} \\times\\left[(10-7)^{2}+(6-7)^{2}+(8-7)^{2}+(5-7)^{2}+(6-7)^{2}\\right]=\\frac{16}{5}=3.2$.故答案为:3.2"} {"id": "16545", "image": [], "answer": "3", "solution": "null", "level": "高一", "question": "一组数据按从小到大的顺序排列为 $1,2,2, \\mathrm{x}, 5,10$, 其中 $x \\neq 5$, 已知该组数据的中位数是众数的 $\\frac{3}{2}$ 倍, 则该组数据的标准差为", "options": [], "subject": "计数", "analysis": "由题意, 可得该组数据的众数为 2 , 所以 $\\frac{2+x}{2}=\\frac{3}{2} \\times 2=3$, 解得 $x=4$, 故该组数据的平均数为 $\\frac{1+2+2+4+5+10}{6}=4$. 所以该组数据 6 的方差为 $\\frac{1}{6} \\times\\left[(1-4)^{2}+(2-4)^{2}+(2-4)^{2}+(4-4)^{2}+(5-4)^{2}+(10-4)^{2}\\right]=9$\n\n, 即标准差为 3 .故答案为:3"} {"id": "16906", "image": [], "answer": "(4)(1)(3)(2)(5).", "solution": "null", "level": "高一", "question": "(2018 秋-阳泉期末)用抽签法进行抽样有以下几个步骤: (1)制签; (2)抽签; (3)将签匀; (4)编号; (5)将抽取的号码对应的个体取出, 组成样本. 这些步骤的正确顺序为", "options": [], "subject": "计数", "analysis": "解: 利用抽签法第一步要进行编号, 然后制签, 然后做放入容器将签匀, 然后抽签, 最后将抽取的号码对应的个体取出, 组成样本,\n\n故这些步骤的先后顺序应为: (4)(1)(3)(2)(5).\n\n故答案为: (4)(1)(3)(2)(5)."} {"id": "16907", "image": ["8729.jpg"], "answer": "5 .", "solution": "null", "level": "高一", "question": "(2019 秋$\\cdot$赣州月考)如下算法中,输出 $i$ 的值为 \\$ \\qquad \\$\n\n", "options": [], "subject": "计数", "analysis": "解:执行算法和框图, 有\n\n$a=10, i=1$\n\n不满足条件 $a=4$, 不满足条件 $a$ 是奇数, 有 $a=5, i=2$,\n\n不满足条件 $a=4$, 满足条件 $a$ 是奇数, 有 $a=16, i=3$,\n\n不满足条件 $a=4$, 不满足条件 $a$ 是奇数, 有 $a=8, i=4$,\n\n不满足条件 $a=4$, 不满足条件 $a$ 是奇数, 有 $a=4, i=5$,\n\n满足条件 $a=4$, 输出 $i$ 的值为 5 .\n\n故答案为: 5 ."} {"id": "16978", "image": ["8754.jpg"], "answer": "frac{1}{2}$.", "solution": "null", "level": "高一", "question": "020 秋 -渝中区校级月考)算盘是中国传统的计算工具, 其形为长方形, 周为木框, 内贯直柱,俗称 “档” , 档中横以梁, 梁上两珠, 每珠作数五, 梁下五珠, 每珠作数一, 运算时定位后拨珠\n 计算,算珠梁上部分叫上珠,梁下部分叫下珠,如图,若拨珠的三档从左至右依次定位:百位档、十位档、个位档, 则表示数字 518.若在千、百、十、个位档中随机选择一档拨一颗上珠, 再随机选择两个档位各拨一颗下珠,则所拨数字能被 5 整除的概率为 \\$ \\qquad \\$ .\n\n", "options": [], "subject": "计数", "analysis": "在千、百、十、个位档中随机选择一档拨一颗上珠, 再随机选择两个档位各拨一颗下珠,\n\n基本事件总数 $n=C_{4}^{1} C_{4}^{2}=24$,\n\n所拨数字能被 5 整除的情况有两种,\n\n(1) 个位档拨一颗上珠, 千、百、十档中拨 2 颗下珠, 包含的基本事件个数有 $m_{1}=C_{3}^{2}=3$,\n\n(2)千、百、十档中拨 1 颗下珠, 同时千、百、十档中拨 2 颗下珠, 包含的基本事件个数有 $m_{2}=C_{3}^{1} C_{3}^{2}=9$,\n\n$\\therefore$ 所拨数字能被 5 整除的概率为 $p=\\frac{m_{1}+m_{2}}{n}=\\frac{3+9}{24}=\\frac{1}{2}$.\n\n故答案为: $\\frac{1}{2}$."} {"id": "16994", "image": [], "answer": "$\\frac{15}{56}$.", "solution": "null", "level": "高一", "question": "(2020 秋・天津期末) 某科技小组有 5 名男生、 3 名女生, 从中任选 3 名同学参加活动, 若 $X$ 表示选出女生的人数, 则 $P(X=2)=$", "options": [], "subject": "计数", "analysis": "解: 某科技小组有 5 名男生、 3 名女生, 从中任选 3 名同学参加活动,\n\n基本事件总数 $n=C_{8}^{3}=56$,\n\n若 $X$ 表示选出女生的人数, 则 $X=2$ 包含的基本事件个数 $m=C_{5}^{1} C_{3}^{2}=15$,\n\n则 $P(X=2)=\\frac{m}{n}=\\frac{15}{56}$.\n\n故答案为: $\\frac{15}{56}$."} {"id": "17020", "image": [], "answer": "0.48 .", "solution": "null", "level": "高一", "question": "(2020 秋-青铜峡市校级月考)某射手一次射击中, 击中 10 环、 9 环、 8 环的概率分别是 0.24 , $0.28,0.19$, 则这射手一次射击中不够 9 环的概率为", "options": [], "subject": "计数", "analysis": "解: 射手射中 9 环及以上的概率为 $0.24+0.28=0.52$,\n\n所以射手一次射击中不够 9 环的概率为 $1-0.52=0.48$,\n\n故答案为: 0.48 ."} {"id": "17023", "image": [], "answer": "$\\frac{1}{6}, \\frac{2}{3}$.", "solution": "null", "level": "高一", "question": "(2020 $\\cdot$津)已知甲、乙两球落入盒子的概率分别为 $\\frac{1}{2}$ 和 $\\frac{1}{3}$. 假定两球是否落入盒子互不影响,则甲、乙两球都落入盒子的概率为 \\$ \\qquad \\$ ; 甲、乙两球至少有一个落入盒子的概率为", "options": [], "subject": "计数", "analysis": "解: 甲、乙两球落入盒子的概率分别为 $\\frac{1}{2}$ 和 $\\frac{1}{3}$, 则 $\\frac{1}{2} \\times \\frac{1}{3}=\\frac{1}{6}$,甲、乙两球至少有一个落入盒子的概率为 $1-\\left(1-\\frac{1}{2}\\right) \\quad\\left(1-\\frac{1}{3}\\right)=1-\\frac{1}{3}=\\frac{2}{3}$,故答案为: $\\frac{1}{6}, \\frac{2}{3}$."} {"id": "17041", "image": [], "answer": "0.39 .", "solution": "null", "level": "高一", "question": "(2020 秋・榆林期末)某商店的有奖促销活动中仅有一等奖、二等奖、鼓励奖三个奖项,其中中一等奖的概率为 0.05 , 中二等奖的概率为 0.16 , 中鼓励奖的概率为 0.40 , 则不中奖的概率为", "options": [], "subject": "计数", "analysis": "解:某商店的有奖促销活动中仅有一等奖、二等奖、鼓励奖三个奖项,其中中一等奖的概率为 0.05 , 中二等奖的概率为 0.16 , 中鼓励奖的概率为 0.40 ,则不中奖的概率为 $P=1-0.05-0.16-0.40=0.39$.\n\n故答案为: 0.39 ."} {"id": "17043", "image": [], "answer": "15 .", "solution": "null", "level": "高一", "question": "(2020 秋・南岗区校级期末)某单位有职工 160 人,其中有业务人员 120 人,管理人员 16 人,后勤人员 24 人. 为了了解职工的某种情况, 要从中抽取一个容量为 20 的样本. 用分层抽样的方法抽取的业务人员的人数是 \\$ \\qquad \\$ .", "options": [], "subject": "计数", "analysis": "解:分层抽样应按各层所占的比例从总体中抽取,\n\n120: 16: $24=15: 2: 3$,\n\n所以抽取的业务人员的人数是 $20 \\times \\frac{15}{20}=15$,\n\n故答案为: 15 ."} {"id": "17044", "image": [], "answer": "40 .", "solution": "null", "level": "高一", "question": "(2020 秋・齐齐哈尔期末)某市共有 800 家企业, 其中中外合资企业 160 家, 私营企业 320 家,国有企业 240 家, 其他性质的企业 80 家. 为了了解企业的管理情况, 现用分层抽样的方法从这 800 家企业中抽取一个容量为 $n$ 的样本, 已知从国有企业中抽取了 12 家, 那么 $n=$ \\$ \\qquad \\$", "options": [], "subject": "计数", "analysis": "解:某市共有 800 家企业, 其中中外合资企业 160 家, 私营企业 320 家, 国有企业 240 家,其他性质的企业 80 家.\n\n用分层抽样的方法从这 800 家企业中抽取一个容量为 $n$ 的样本,\n\n已知从国有企业中抽取了 12 家,则 $\\frac{n}{800}=\\frac{12}{240} ,$\n\n解得 $n=40$.\n\n故答案为: 40 ."} {"id": "16998", "image": ["8758.jpg", "8759.jpg"], "answer": "$1-\\frac{2}{\\pi}$.", "solution": "null", "level": "高一", "question": "(2020 秋・朝阳区校级期末)如图, 在圆心角为直角的扇形 $O A B$ 中, 分别以 $O A, O B$ 为直径作两个半圆。在扇形 $O A B$ 内随机取一点,则此点取自阴影部分的概率是 \\$ \\qquad \\$\n\n", "options": [], "subject": "组合几何学", "analysis": "解:如图所示:\n\n\n\n设 $O A$ 的中点为 $D$, 两半圆交于点 $C$, 连接 $C D$, 则 $C D \\perp O A$, 设扇形 $O A B$ 的半径为 $r$,\n\n$\\therefore S_{\\text {㲏形 } O A B}=\\frac{1}{4} \\pi r^{2}, S_{\\text {半 } O A C}=\\frac{1}{8} \\pi r^{2}, S_{\\triangle C O D}=\\frac{1}{2} \\times \\frac{1}{2} r \\times \\frac{1}{2} r=\\frac{1}{8} r^{2}$,\n\n$\\therefore S_{\\text {纸 } O C}=\\frac{1}{2} S_{\\text {半图OAC }}-S_{\\triangle C O D}=\\frac{1}{16} \\pi r^{2}-\\frac{1}{8} r^{2}$,\n$\\therefore$ 两个圆的弧 $O C$ 围成的阴影部分的面积为 $\\frac{1}{8} \\pi r^{2}-\\frac{1}{4} r^{2}$,\n\n$\\therefore$ 图中阴影部分的面积为 $\\frac{1}{4} \\pi r^{2}-2 \\times \\frac{1}{8} \\pi r^{2}+2\\left(\\frac{1}{8} \\pi r^{2}-\\frac{1}{4} r^{2}\\right)=\\frac{1}{4} \\pi r^{2}-\\frac{1}{2} r^{2}$,\n\n$\\therefore$ 此点取自阴影部分的概率是 $\\frac{\\frac{1}{4} \\pi r^{2}-\\frac{1}{2} r^{2}}{\\frac{1}{4} \\pi r^{2}}=1-\\frac{2}{\\pi}$.\n\n故答案为: $1-\\frac{2}{\\pi}$."} {"id": "16999", "image": ["8760.jpg"], "answer": "$\\frac{15}{32}$.", "solution": "null", "level": "高一", "question": "(2020 秋・哈尔滨期末)明朝著名易学家来知德以其太极图解释一年、一日之象的图式, 一年气象图将二十四节气配以太极图, 说明一年之气象, 来氏认为 “万古之人事, 一年之气象也, 春作夏长秋收冬藏, 一年不过如此” . 如图是来氏太极图, 其大圆半径为 4, 大圆内部的同心小圆半径为 1 , 两圆之间的图案是对称的, 若在大圆内随机取一点, 则该点落在黑色区域的概率为 \\$ \\qquad \\$ .\n\n", "options": [], "subject": "统计数学", "analysis": "解:设大圆的面积为 $S_{1}$, 小圆的面积为 $S_{2}$,\n\n则 $S_{1}=16 \\pi, S_{2}=\\pi$,\n\n$\\therefore$ 黑色区域的面积为 $\\frac{1}{2}\\left(S_{1}-S_{2}\\right)=\\frac{15 \\pi}{2}$,\n\n$\\therefore$ 点落在黑色区域的概率为 $\\frac{\\frac{15 \\pi}{2}}{16 \\pi}=\\frac{15}{32}$.\n\n故答案为: $\\frac{15}{32}$."} {"id": "15628", "image": ["7919.jpg", "7920.jpg", "7921.jpg", "7922.jpg"], "answer": ")(2)(3)(4)", "solution": "null", "level": "高一", "question": "一个等腰直角三角形在一个平面内的平行投影可能是下列图形中的 (把你认为正确的选项代号都填上).(1)等腰直角三角形; (2)直角非等腰三角形; (3)钝角三角形; (4)锐角三角形; (5)线段.", "options": [], "subject": "画法几何学", "analysis": "【分析】\n\n一个等腰直角三角形在一个平面内的平行投影的形状取决于该等腰直角三角形所在平面放置的位置, 可分等腰直角三角形所在的平面与投影面平行、等腰直角三角形的一条直角边与投影面平行时、等腰直角三角形的斜边在投影面内或与投影面平行时、等腰直角三角形的任何一边都不与投影面平行时、等腰直角三角形所在平面与投影面垂直共五种情况讨论即可。\n\n## 【详解】\n\n(1) 当等腰直角三角形所在的平面与投影面平行时, 如图, 它的正投影是与它全等的等腰直角三角形。\n\n\n\n(2) 当等腰直角三角形的一条直角边与投影面平行时, 如图, 它的正投影是直角非等腰三角形。\n\n\n\n(3)当等腰直角三角形的斜边在投影面内或与投影面平行时, 如图, 改变等腰直角三角形所在平面与投影面所成角的大小, 它的正投影为钝角三角形。\n\n\n\n令 $A B=A C=1$, 则 $B C=\\sqrt{2},-A O \\perp \\alpha$, 则 $\\triangle B C O$ 就是 $\\triangle B C A$ 在 $\\alpha$ 内的正投影. 当 $A O=\\frac{\\sqrt{3}}{2}$ 时, $B O=\\frac{1}{2}, C O=\\frac{1}{2}$, 且有 $B O^{2}+C O^{2}=\\frac{1}{4}+\\frac{1}{4}<2=B C^{2}$, 所以 $\\angle B O C$ 为针角. 所以 $\\triangle B O C$ 为钝角三角形。\n\n(4)当等腰直角三角形的任何一边都不与投影面平行时, 如图, 等腰直角三角形的正投影为锐角三\n\n角形。\n\n\n\n若令 $A B=A C=2, \\angle B A C=90^{\\circ}$, 则 $B C=2 \\sqrt{2}$. 又令 $A A^{\\prime}=1, B B^{\\prime}=2$, 则在 Rt $\\triangle A A^{\\prime} C$ 中, $A^{\\prime} C=\\sqrt{3}$; 在 Rt $\\triangle B B^{\\prime} C$ 中, $B^{\\prime} C=2$. 过点 $\\mathrm{A}$ 作 $A D \\perp B B^{\\prime}$, 则有 $B^{\\prime} D=A A^{\\prime}=1$, 故 $B D=1$. 在 Rt $\\triangle A B D$ 中, $A D=\\sqrt{3}$, 即 $A^{\\prime} B^{\\prime}=A D=\\sqrt{3}$. 在 $\\triangle A^{\\prime} B^{\\prime} C$ 中, $A^{\\prime} B^{\\prime 2}+A^{\\prime} C^{2}=(\\sqrt{3})^{2}+(\\sqrt{3})^{2}=6>4=B^{\\prime} C^{2}$, 所以 $\\triangle A^{\\prime} B^{\\prime} C^{\\prime}$ 是锐角三角形。\n\n(5)当等腰直角三角形所在平面与投影面垂直时,易知它的正投影是一条线段。故应填(1)(2)(3)(4)(5).\n\n## 【点睛】\n\n本题应全面考虑等腰直角三角形相对投影面的各种情况, 如等腰直角三角形所在平面与投影面相交时,需分别考虑直角边、斜边相对投影面的位置关系。"} {"id": "6354", "image": [], "answer": "\\sqrt{ }$", "solution": "null", "level": "一年级", "question": "可以表示一个也没有。()", "options": [], "subject": "逻辑题", "analysis": "\\sqrt{ }$"} {"id": "6376", "image": ["444.jpg"], "answer": "\\times$", "solution": "null", "level": "一年级", "question": "壸在热水瓶的右面,杯子在热水壸的左面。( )\n\n", "options": [], "subject": "逻辑题", "analysis": "\\times$"} {"id": "6382", "image": ["447.jpg"], "answer": "\\times$", "solution": "null", "level": "一年级", "question": "蜓在荷花的下面, 荷花在蛙蜓的上面。()\n\n", "options": [], "subject": "逻辑题", "analysis": "\\times$"} {"id": "6396", "image": [], "answer": "\\sqrt{ }$", "solution": "null", "level": "一年级", "question": "\\triangle$ 在 $\\square$ 的前面。( )", "options": [], "subject": "逻辑题", "analysis": "\\sqrt{ }$"} {"id": "6710", "image": [], "answer": "$\\sqrt{ }$", "solution": "null", "level": "一年级", "question": "两位数一定比三位数小。", "options": [], "subject": "逻辑题", "analysis": "$\\sqrt{ }$"} {"id": "6356", "image": ["424.jpg"], "answer": "\\times$", "solution": "null", "level": "一年级", "question": "图是正方体。\n\n", "options": [], "subject": "立体几何学", "analysis": "\\times$"} {"id": "6398", "image": [], "answer": "\\times$", "solution": "null", "level": "一年级", "question": "两个同样大小的正方体可以拼成一个大的正方形。", "options": [], "subject": "立体几何学", "analysis": "\\times$"} {"id": "6143", "image": [], "answer": "$\\times$", "solution": "null", "level": "一年级", "question": "至少用 2 个 可以拼成一个大正方体。()", "options": [], "subject": "立体几何学", "analysis": "$\\times$"} {"id": "6162", "image": [], "answer": "$\\sqrt{ }$", "solution": "null", "level": "一年级", "question": "搭积木时, 球最容易滚动。", "options": [], "subject": "立体几何学", "analysis": "$\\sqrt{ }$"} {"id": "6377", "image": [], "answer": "\\sqrt{ }$", "solution": "null", "level": "一年级", "question": "右边起,第一位是个位,第二位是十位。()", "options": [], "subject": "算术", "analysis": "\\sqrt{ }$"} {"id": "6381", "image": [], "answer": "\\times$", "solution": "null", "level": "一年级", "question": "5 大 1 的数是 4 , 比 5 小 1 的数是 6 。", "options": [], "subject": "算术", "analysis": "\\times$"} {"id": "5856", "image": ["1.jpg"], "answer": "解12, 13, 15, 20;", "solution": "null", "level": "一年级", "question": "看图写数。\n", "options": [], "subject": "算术", "analysis": "解12, 13, 15, 20;"} {"id": "5857", "image": [], "answer": "解9, 11, 13, 12, 8, 6, 14, 15, 16, 10, 11, 12;", "solution": "null", "level": "一年级", "question": "找规律数一数, 填一填。\n\n(1) $1,3,5,7,(\\quad),(\\quad),(\\quad), 15$ 。\n\n(2) $20,18,16,14,(\\quad), 10,(\\quad),(\\quad) 。$\n\n(3) $10,11,12,13,(\\quad),(\\quad),(\\quad) 。$\n\n(4) $7,8,9,(\\quad),(\\quad),(\\quad), 13$ 。", "options": [], "subject": "算术", "analysis": "解9, 11, 13, 12, 8, 6, 14, 15, 16, 10, 11, 12;"} {"id": "5858", "image": ["2.jpg"], "answer": "详解大于 $1: 6,12,14,20$ 一位数: 1,6 大于 $10: 12,14,20$ 大于 13: 14,20 ;", "solution": "null", "level": "一年级", "question": "选数, 填一填。\n\n\n\n大于 13 的数", "options": [], "subject": "算术", "analysis": "详解大于 $1: 6,12,14,20$ 一位数: 1,6 大于 $10: 12,14,20$ 大于 13: 14,20 ;"} {"id": "5859", "image": [], "answer": "详解5", "solution": "null", "level": "一年级", "question": "班上排队, 一队有 16 人, 从左往右数, 小红排第 12 号, 从右往左数, 小红排第()号。", "options": [], "subject": "算术", "analysis": "详解5"} {"id": "5861", "image": ["3.jpg"], "answer": "解12<15 ;", "solution": "null", "level": "一年级", "question": "图填空, 再比大小。\n\n\n$(\\quad) \\circ(\\quad)$", "options": [], "subject": "算术", "analysis": "解12<15 ;"} {"id": "5862", "image": [], "answer": "解$\\quad>,<,=,<,<,<,>,=$", "solution": "null", "level": "一年级", "question": "20 \\bigcirc 19$\n$12-10 \\bigcirc 12$\n$13 \\bigcirc 10+3$\n$10 \\bigcirc 15$\n$17 \\bigcirc 18$\n$18-7 \\bigcirc 13$\n$16 \\bigcirc 10+5$\n$10+3 \\bigcirc 13$", "options": [], "subject": "算术", "analysis": "解$\\quad>,<,=,<,<,<,>,=$"} {"id": "5867", "image": ["4.jpg", "5.jpg", "6.jpg"], "answer": "解四, 一, 14, 2, 20 ,1, 6, 16;", "solution": "null", "level": "一年级", "question": "数一数, 填一填。\n\n\n\n( ) 个一和 ( ) 个十是 ( )\n\n\n\n$(\\quad)$ 个十是 ( $\\quad$ )\n\n", "options": [], "subject": "算术", "analysis": "解四, 一, 14, 2, 20 ,1, 6, 16;"} {"id": "5868", "image": [], "answer": "解1, 9;", "solution": "null", "level": "一年级", "question": "19 里面有 ( ) 个十和( ) 个一。", "options": [], "subject": "算术", "analysis": "解1, 9;"} {"id": "5869", "image": [], "answer": "解20;", "solution": "null", "level": "一年级", "question": "2 个十组成()。", "options": [], "subject": "算术", "analysis": "解20;"} {"id": "5870", "image": [], "answer": "解14;", "solution": "null", "level": "一年级", "question": "1 个十和 4 个一合起来是()。", "options": [], "subject": "算术", "analysis": "解14;"} {"id": "5871", "image": [], "answer": "解15;", "solution": "null", "level": "一年级", "question": "5 个一和 1 个十合起来是()。", "options": [], "subject": "算术", "analysis": "解15;"} {"id": "5872", "image": [], "answer": "解十, 十, 个, 一;", "solution": "null", "level": "一年级", "question": "18 中的 “1” 在()位,表示 1 个(); “8”在()位,表示 8 个()。", "options": [], "subject": "算术", "analysis": "解十, 十, 个, 一;"} {"id": "5873", "image": [], "answer": "解17, 19;", "solution": "null", "level": "一年级", "question": "18 的前面一个数是(),18 的后面一个数是()。", "options": [], "subject": "算术", "analysis": "解17, 19;"} {"id": "5874", "image": [], "answer": "解9 个;", "solution": "null", "level": "一年级", "question": "小丽踢建踢了 6 个, 小华踢的个数比小丽多一些。小华可能踢了()个。 $(4$ 个, 9 个, 17 个)", "options": [], "subject": "算术", "analysis": "解9 个;"} {"id": "6059", "image": [], "answer": "$\\mathrm{x}$", "solution": "null", "level": "一年级", "question": "最大的一位数是 10 。", "options": [], "subject": "算术", "analysis": "$\\mathrm{x}$"} {"id": "6084", "image": [], "answer": "$\\sqrt{ }$", "solution": "null", "level": "一年级", "question": "2 和 6 之间有 $3 、 4 、 5$ 。", "options": [], "subject": "算术", "analysis": "$\\sqrt{ }$"} {"id": "6085", "image": [], "answer": "$\\mathrm{x}$", "solution": "null", "level": "一年级", "question": "与 8 相邻的两个数是 7 和 6 。", "options": [], "subject": "算术", "analysis": "$\\mathrm{x}$"} {"id": "6086", "image": [], "answer": "$\\sqrt{ }$", "solution": "null", "level": "一年级", "question": "小东读一本故事书, 第 1 天从第 1 页读到第 8 页, 第二天该从第 9 页开始读。", "options": [], "subject": "算术", "analysis": "$\\sqrt{ }$"} {"id": "6087", "image": [], "answer": "$\\mathrm{x}$", "solution": "null", "level": "一年级", "question": "比 3 多 4 的数是 6 。", "options": [], "subject": "算术", "analysis": "$\\mathrm{x}$"} {"id": "6654", "image": [], "answer": "详解6", "solution": "null", "level": "一年级", "question": "两个数的和是 12, 一个加数是 6 , 另一个加数是\n", "options": [], "subject": "算术", "analysis": "详解6"} {"id": "6655", "image": ["642.jpg"], "answer": "详解$6 ; 6 ; 6$", "solution": "null", "level": "一年级", "question": "\n\n小红\n\n一共有 15 个梨, 小红吃了 9 个, 还剩个。因为 $9+$ $=15$,所以 $15-9=$ \\$ \\qquad \\$。", "options": [], "subject": "算术", "analysis": "详解$6 ; 6 ; 6$"} {"id": "6656", "image": [], "answer": "详解7", "solution": "null", "level": "一年级", "question": "被减数是 16 , 减数是 9 , 差是 \\$ \\qquad \\$ ○", "options": [], "subject": "算术", "analysis": "详解7"} {"id": "6657", "image": ["643.jpg"], "answer": "详解$<$; =; =; <; <; =", "solution": "null", "level": "一年级", "question": "在横线上填上 “>”“<”或 “=”。\n\n", "options": [], "subject": "算术", "analysis": "详解$<$; =; =; <; <; ="} {"id": "6658", "image": [], "answer": "详解$19 ; 5$", "solution": "null", "level": "一年级", "question": "12 与 7 的和是 \\$ \\qquad \\$ ,差是 \\$ \\qquad \\$ .", "options": [], "subject": "算术", "analysis": "详解$19 ; 5$"} {"id": "6659", "image": [], "answer": "详解$7 ; 7$", "solution": "null", "level": "一年级", "question": "小熊摘了 7 个苹果, 小猴摘了 14 个苹果, 小熊比小猴少摘了 \\$ \\qquad \\$个苹果,小猴比小熊多摘了 \\$ \\qquad \\$个苹果。", "options": [], "subject": "算术", "analysis": "详解$7 ; 7$"} {"id": "6660", "image": [], "answer": "详解3", "solution": "null", "level": "一年级", "question": "美术课上, 丁丁做了 15 朵纸花, 敏敏做了 9 朵纸花。丁丁给敏敏朵后, 两人的纸花就一样多。", "options": [], "subject": "算术", "analysis": "详解3"} {"id": "6663", "image": [], "answer": "详解6", "solution": "null", "level": "一年级", "question": "妈妈给明明买来 14 个练习本, 明明用了一些以后还剩 8 个, 明明一共用了 \\$ \\qquad \\$个练习本。", "options": [], "subject": "算术", "analysis": "详解6"} {"id": "6664", "image": [], "answer": "详解$9 ; 9 ; 9 ; 5 ; 6 ; 8$", "solution": "null", "level": "一年级", "question": "填一填。\n$7+$ $=16$\n$4^{+}$ $=13$\n$3+$ $=12$\n$14-9=$ \\$ \\qquad \\$\n$15-9=$ \\$ \\qquad \\$\n$17-9=$ \\$ \\qquad \\$", "options": [], "subject": "算术", "analysis": "详解$9 ; 9 ; 9 ; 5 ; 6 ; 8$"} {"id": "6665", "image": [], "answer": "详解7", "solution": "null", "level": "一年级", "question": "小红有 15 支彩笔,小明有 9 支彩笔。小明至少再买 \\$ \\qquad \\$支彩笔就\n超过小红了。", "options": [], "subject": "算术", "analysis": "详解7"} {"id": "7217", "image": [], "answer": "$>;>;<;<;<;<$", "solution": "null", "level": "一年级", "question": "$10-2 \\mathrm{O} 10-7 \\quad 15+4 \\mathrm{O} 15-4$\n\n$13-5 \\mathrm{O} 14-5 \\quad 12-2 \\mathrm{O} 13-2$\n\n$4+8 \\mathrm{O} 5+9 \\quad 14-5 \\mathrm{O} 15-5$", "options": [], "subject": "算术", "analysis": "$>;>;<;<;<;<$"} {"id": "7229", "image": ["973.jpg"], "answer": "涂色略 提示:第四个涂红色,给第三个涂蓝色。", "solution": "null", "level": "一年级", "question": "给得数最大的涂上红色,给得数最小的涂上蓝色。", "options": [], "subject": "算术", "analysis": "涂色略 提示:第四个涂红色,给第三个涂蓝色。"} {"id": "7246", "image": [], "answer": "两 个 6 个一 十 6 个十", "solution": "null", "level": "一年级", "question": "66 是( )位数,右边的 6 在( )位上,表示( ),左边的 6 在( )位上,表示\n\n( $\\quad$ )。", "options": [], "subject": "算术", "analysis": "两 个 6 个一 十 6 个十"} {"id": "6378", "image": ["445.jpg"], "answer": "\\times$", "solution": "null", "level": "一年级", "question": "子宝宝既在梨宝宝的下面, 又在西瓜的左面。( )\n\n", "options": [], "subject": "组合几何学", "analysis": "\\times$"} {"id": "6632", "image": ["619.jpg"], "answer": "详解4; 3; 4; 2", "solution": "null", "level": "一年级", "question": "\n\n长方形有个,正方形有个, 三角形有个,\n\n圆有 \\$ \\qquad \\$个。", "options": [], "subject": "组合几何学", "analysis": "详解4; 3; 4; 2"} {"id": "6380", "image": ["446.jpg"], "answer": "\\times$", "solution": "null", "level": "一年级", "question": "数量比 a 多。( $\\quad )$\n\n", "options": [], "subject": "计数", "analysis": "\\times$"} {"id": "6634", "image": ["621.jpg"], "answer": "详解9; 7; 1; 2", "solution": "null", "level": "一年级", "question": "数一数, 填一填。\n\n\n图中有 \\$ \\qquad \\$个长方形, \\$ \\qquad \\$个三角形, \\$ \\qquad \\$个正方形,\n\n个圆。", "options": [], "subject": "计数", "analysis": "详解9; 7; 1; 2"} {"id": "7245", "image": [], "answer": "一 二百", "solution": "null", "level": "一年级", "question": "计数器上, 从右边起, 个位是第( )位, 十位是第( )位, 第三位是( ) 位。", "options": [], "subject": "计数", "analysis": "一 二百"} {"id": "6780", "image": ["682.jpg"], "answer": "4", "solution": "null", "level": "一年级", "question": "照这样的规律排列, 盒子里一共有 \\$ \\qquad \\$颗珠子。\n\n", "options": [], "subject": "计数", "analysis": "4"} {"id": "6397", "image": [], "answer": "\\times$", "solution": "null", "level": "一年级", "question": "在水边, 人的倒影与人上下相反, 左右相同。", "options": [], "subject": "变换几何", "analysis": "\\times$"} {"id": "5860", "image": ["2944.jpg"], "answer": "解$2,10,13,3$", "solution": "null", "level": "一年级", "question": "", "options": [], "subject": "图论", "analysis": ":分析:(1)由点 $\\mathrm{A}$ 在抛物线上, 将 $\\mathrm{A}$ 点坐标代入, 求出参数 $\\mathrm{P}$, 求解即可 (2) 由于 $F(8,0)$ 是 $\\triangle A B C$ 的重心, 则重心与焦点重合, 由重心坐标公式可求 $M$ 是 $B C$ 的中点。\n\n(3) 由于线段 $\\mathrm{BC}$ 的中点 $\\mathrm{M}$ 不在 $x$ 轴上, 所以 $\\mathrm{BC}$ 所在的直线不垂直于 $x$ 轴. 设 $\\mathrm{BC}$ 所在直线的方程为: $y+4=k(x-11)(k \\neq 0)$, 解出 $k$ 即可。"} {"id": "6661", "image": [], "answer": "详解6", "solution": "null", "level": "一年级", "question": "有 13 个小朋友排成一行, 其中男生有 7 人,女生有 \\$ \\qquad \\$人。", "options": [], "subject": "组合数学", "analysis": "详解6"} {"id": "6779", "image": [], "answer": "$\\triangle ; \\square ; \\leftarrow ; \\downarrow$", "solution": "null", "level": "一年级", "question": "按照规律接着画。\n\n$\\square 、 \\square 、 \\triangle 、 \\square 、 \\square 、 \\triangle 、 \\square 、 \\square 、$ \\$ \\qquad \\$ \\$ \\qquad \\$\n$\\rightarrow 、 \\uparrow 、 \\leftarrow 、 \\downarrow 、 、 \\uparrow 、$ \\$ \\qquad \\$", "options": [], "subject": "组合数学", "analysis": "$\\triangle ; \\square ; \\leftarrow ; \\downarrow$"} {"id": "6017", "image": [], "answer": "$\\times$", "solution": "null", "level": "一年级", "question": "$\\left(\\begin{array}{c}11^{12} 1 \\\\ 0 \\\\ 3 \\\\ 0\\end{array}\\right)$\n\n钟面上分针走过 12 时针刚走过 10 , 表示快到 10 时了。( )", "options": [], "subject": "组合数学", "analysis": "$\\times$"} {"id": "6057", "image": ["165.jpg"], "answer": "\\times$", "solution": "null", "level": "一年级", "question": "一共有 16 只鸭。\n\n7 . _这些小棒一共有 13 根。\n\n\n\n8 . _最大的一位数加 1 得 10 。\n\n9 ._ 一个加数是 7 , 另一个加数是 5 , 和是 13 。\n\n10 ._ 树上有 8 只鸟, 又飞来 4 只, 现在一共有 13 只鸟。( )", "options": [], "subject": "组合数学", "analysis": "\\times$"} {"id": "6038", "image": [], "answer": "V", "solution": "null", "level": "一年级", "question": "分针和时针在一条直线上的时候是 6 点。", "options": [], "subject": "度量几何学", "analysis": "V"} {"id": "6830", "image": [], "answer": "$\\sqrt{ }$", "solution": "null", "level": "一年级", "question": "正方形有四条边, 并且都是相等的。", "options": [], "subject": "度量几何学", "analysis": "$\\sqrt{ }$"} {"id": "6829", "image": [], "answer": "$\\sqrt{ }$", "solution": "null", "level": "一年级", "question": "两个相同的三角形可以拼成一个平行四边形 。", "options": [], "subject": "解析几何", "analysis": "$\\sqrt{ }$"} {"id": "9039", "image": [], "answer": "解1.6\n\n根据可知, 爸爸走了 0.8 千米后返回去拿材料, 返回时又走了 0.8 千米, 这样就比平时多走了 2 个 0.8 千米,据此解题即可。\n\n$0.8+0.8=1.6$ (千米)\n\n所以, 爸爸比平时要多走 1.6 千米。\n\n要明确, 爸爸走了 0.8 千米后返回再去上班, 比平时多走了 2 个 0.8 千米; 是解答此题的关键。", "solution": "null", "level": "三年级", "question": "爸爸从家到单位要走 2.7 千米, 他走了 0.8 千米后发现有一份文件忘了拿, 于是又回家取文件, 这样他比平时上班要多走( $\\quad$ 千米。", "options": [], "subject": "算术", "analysis": "解1.6\n\n根据可知, 爸爸走了 0.8 千米后返回去拿材料, 返回时又走了 0.8 千米, 这样就比平时多走了 2 个 0.8 千米,据此解题即可。\n\n$0.8+0.8=1.6$ (千米)\n\n所以, 爸爸比平时要多走 1.6 千米。\n\n要明确, 爸爸走了 0.8 千米后返回再去上班, 比平时多走了 2 个 0.8 千米; 是解答此题的关键。"} {"id": "9040", "image": [], "answer": "解$\\quad 2.3 \\quad 4.2$\n\n要求 3.8 比什么数多 1.5 , 用 3.8 减去 1.5 即可;\n\n要求 5.7 比 9.9 少多少, 用 9.9 减去 5.7 即可。\n\n$3.8-1.5=2.3$\n\n$9.9-5.7=4.2$\n\n3.8 比 2.3 多 $1.5 ; 5.7$ 比 9.9 少 4.2 。\n\n\n\n本题主要考查了小数加减法的意义和计算方法, 关键是要分清楚谁多谁少。", "solution": "null", "level": "三年级", "question": "3.8 比( )多 1.5; 5.7 比 9.9 少( )。", "options": [], "subject": "算术", "analysis": "解$\\quad 2.3 \\quad 4.2$\n\n要求 3.8 比什么数多 1.5 , 用 3.8 减去 1.5 即可;\n\n要求 5.7 比 9.9 少多少, 用 9.9 减去 5.7 即可。\n\n$3.8-1.5=2.3$\n\n$9.9-5.7=4.2$\n\n3.8 比 2.3 多 $1.5 ; 5.7$ 比 9.9 少 4.2 。\n\n\n\n本题主要考查了小数加减法的意义和计算方法, 关键是要分清楚谁多谁少。"} {"id": "9041", "image": [], "answer": "解5\n\n一位小数比较大小的方法是: 先比较整数部分, 整数部分大, 这个小数就大; 整数部分相同,就比较小数点右边的数, 小数点右边的数大, 这个小数就大; 依此填空即可。\n\n9. 元与 9.6 元, 整数部分都是 $9,9$. . 是一位小数, 要使 9. 口元 $<9.6$ 元成立, 口里最大可以填\n\n5 。\n\n熟练掌握一位小数的大小比较是解答此题的关键。", "solution": "null", "level": "三年级", "question": "9 . 是一位小数, 要使 9. . 元 $<9.6$ 元成立, 口里最大可以填 ( 。", "options": [], "subject": "算术", "analysis": "解5\n\n一位小数比较大小的方法是: 先比较整数部分, 整数部分大, 这个小数就大; 整数部分相同,就比较小数点右边的数, 小数点右边的数大, 这个小数就大; 依此填空即可。\n\n9. 元与 9.6 元, 整数部分都是 $9,9$. . 是一位小数, 要使 9. 口元 $<9.6$ 元成立, 口里最大可以填\n\n5 。\n\n熟练掌握一位小数的大小比较是解答此题的关键。"} {"id": "9042", "image": [], "answer": "解0.8\n\n$\\frac{8}{10}$ 米表示把 1 米平均分成 10 份, 1 份是 0.1 米, 8 份就是 8 个 0.1 米, 8 个 0.1 米是 0.8 米,据此解答。\n\n\n\n$\\frac{8}{10}=0.8$, 用小数表示是 0.8 米。\n\n\n\n分数表示把 1 个整体平均分成若干份, 取其中的 1 份或几份。", "solution": "null", "level": "三年级", "question": "一根绳子用去 $\\frac{8}{10}$ 米, 用小数表示是 $(\\quad)$ 米。", "options": [], "subject": "算术", "analysis": "解0.8\n\n$\\frac{8}{10}$ 米表示把 1 米平均分成 10 份, 1 份是 0.1 米, 8 份就是 8 个 0.1 米, 8 个 0.1 米是 0.8 米,据此解答。\n\n\n\n$\\frac{8}{10}=0.8$, 用小数表示是 0.8 米。\n\n\n\n分数表示把 1 个整体平均分成若干份, 取其中的 1 份或几份。"} {"id": "9043", "image": [], "answer": "解$\\quad 1.5 \\quad 1.6$\n\n\n\n\n\n根据所给数据发现: 后一个数等于前一个数加 0.1 , 据此解答。\n\n\n\n$1.1+0.1=1.2$\n\n$1.2+0.1=1.3$\n\n$1.3+0.1=1.4$\n\n$1.4+0.1=1.5$\n\n$1.5+0.1=1.6$\n\n所以 $1.1,1.2,1.3,1.4,1.5,1.6$ 。\n\n\n\n关键是根据已知的数得出前后数之间的变化关系的规律,然后再利用这个变化规律再回到问题中去解决问题。", "solution": "null", "level": "三年级", "question": "按规律接着往下写: 1.1, 1.2, 1.3, 1.4, ( ), ( )", "options": [], "subject": "算术", "analysis": "解$\\quad 1.5 \\quad 1.6$\n\n\n\n\n\n根据所给数据发现: 后一个数等于前一个数加 0.1 , 据此解答。\n\n\n\n$1.1+0.1=1.2$\n\n$1.2+0.1=1.3$\n\n$1.3+0.1=1.4$\n\n$1.4+0.1=1.5$\n\n$1.5+0.1=1.6$\n\n所以 $1.1,1.2,1.3,1.4,1.5,1.6$ 。\n\n\n\n关键是根据已知的数得出前后数之间的变化关系的规律,然后再利用这个变化规律再回到问题中去解决问题。"} {"id": "9045", "image": [], "answer": "解$\\quad$ 一点二\n$0.3 \\quad 0.9$\n\n(1) 读小数时, 先读整数部分, 再读小数点, 最后读小数部分, 小数部分是几就读几;\n\n(2) 根据 1 分米 $=0.1$ 米, 将 3 分米的单位化为米即可;\n\n(3) 先将 3 分米的单位化为米, 再用 1.2 减这个数即可解答。\n\n(1) 1.2 米读作 (一点二) 米。\n\n(2) 3 分米写成小数是 $(0.3)$ 米。\n\n(3) 3 分米 $=0.3$ 米\n\n$1.2-0.3=0.9$ (米)\n\n小林跳的高度是 $(0.9)$ 米。\n\n\n\n计算小数减法时, 按照整数减法的计算法则计算, 再将小数点对齐即可。", "solution": "null", "level": "三年级", "question": "小刚和小林比赛跳高, 小刚跳的高度是 1.2 米, 比小林跳的高度高 3 分米。\n\n(1) 1.2 米读作 $(\\quad)$ 米。\n\n(2) 3 分米写成小数是 $(\\quad)$ 米。\n\n(3) 小林跳的高度是 $(\\quad)$ 米。", "options": [], "subject": "算术", "analysis": "解$\\quad$ 一点二\n$0.3 \\quad 0.9$\n\n(1) 读小数时, 先读整数部分, 再读小数点, 最后读小数部分, 小数部分是几就读几;\n\n(2) 根据 1 分米 $=0.1$ 米, 将 3 分米的单位化为米即可;\n\n(3) 先将 3 分米的单位化为米, 再用 1.2 减这个数即可解答。\n\n(1) 1.2 米读作 (一点二) 米。\n\n(2) 3 分米写成小数是 $(0.3)$ 米。\n\n(3) 3 分米 $=0.3$ 米\n\n$1.2-0.3=0.9$ (米)\n\n小林跳的高度是 $(0.9)$ 米。\n\n\n\n计算小数减法时, 按照整数减法的计算法则计算, 再将小数点对齐即可。"} {"id": "9046", "image": [], "answer": "解三点五\n\n\n\n小数的读法:先读整数部分,小数点读作“点”,小数部分从左往右依次读出数字即可;小数的写法: 先写整数部分, 小数点用圆点表示, 小数部分从左往右写出数字即可。\n\n3.5 读作: 三点五;\n\n五点二五写作: 5.25 。\n\n\n\n本题考查的是小数的读法和写法, 属于基础题。", "solution": "null", "level": "三年级", "question": "陆地上最大的动物是非洲象, 它的高度可达 $\\underline{3.5}$ 米, 横线上的数读作 ( ), 一头非洲象的体重可达五点二五吨横线上的数写作( $\\quad$ 。", "options": [], "subject": "算术", "analysis": "解三点五\n\n\n\n小数的读法:先读整数部分,小数点读作“点”,小数部分从左往右依次读出数字即可;小数的写法: 先写整数部分, 小数点用圆点表示, 小数部分从左往右写出数字即可。\n\n3.5 读作: 三点五;\n\n五点二五写作: 5.25 。\n\n\n\n本题考查的是小数的读法和写法, 属于基础题。"} {"id": "8375", "image": [], "answer": "$5 \\quad 2$\n\n【分析】 $55 \\times 18=990,65 \\times 18=1170$, 积要是三位数, 据此得 $\\square$ 最大填多少; $7 \\square 8 \\div 7$, 被除数百位上是 7 , 能被 7 整除, 要使商的中间是 0 , 且没有余数, 首先口 $<7$, 并且被除数十位和个位组成的数能被 7 整除, 据此可得到口填多少。\n\n【详解】根据分析可知, $\\square 5 \\times 18$, 如果积是三位数, 那么 $\\square$ 里最大能填 $5 ; 7 \\square 8 \\div 7$ 没有余数, 商的中间有 0 , 哩应填 2 。\n\n【点睛】本题考查的是两位数乘两位、三位数除以一位数的计算, 注意根据题目要求来填空。", "solution": "null", "level": "三年级", "question": "$\\square 5 \\times 18$, 如果积是三位数, 那么 $\\square$ 里最大能填 $($ ); $7 \\square 8 \\div 7$ 没有余数, 商的中间有 0 , $\\square$ 里应填( )。", "options": [], "subject": "算术", "analysis": "$5 \\quad 2$\n\n【分析】 $55 \\times 18=990,65 \\times 18=1170$, 积要是三位数, 据此得 $\\square$ 最大填多少; $7 \\square 8 \\div 7$, 被除数百位上是 7 , 能被 7 整除, 要使商的中间是 0 , 且没有余数, 首先口 $<7$, 并且被除数十位和个位组成的数能被 7 整除, 据此可得到口填多少。\n\n【详解】根据分析可知, $\\square 5 \\times 18$, 如果积是三位数, 那么 $\\square$ 里最大能填 $5 ; 7 \\square 8 \\div 7$ 没有余数, 商的中间有 0 , 哩应填 2 。\n\n【点睛】本题考查的是两位数乘两位、三位数除以一位数的计算, 注意根据题目要求来填空。"} {"id": "8376", "image": [], "answer": "$473 \\quad 902$\n\n【分析】根据所给数据发现这组数的规律, 一个两位数乘 11 , 积为三位数, 个位为这个两位数的个位; 十位为这个两位数的个位上的数与十位上的数的和; 百位为这个两位数的十位上的数; 据此解答。\n\n【详解】 $12 \\times 11=132,24 \\times 11=264,32 \\times 11=352,43 \\times 11=473,82 \\times 11=902$ 。\n\n【点睛】本题主要考查“式”的规律, 关键根据所给算式发现这组算式的规律, 并运用规律做题。", "solution": "null", "level": "三年级", "question": "想一想,填一填。\n\n$12 \\times 11=132,24 \\times 11=264,32 \\times 11=352,43 \\times 11=(\\quad), 82 \\times 11=(\\quad)$ 。", "options": [], "subject": "算术", "analysis": "$473 \\quad 902$\n\n【分析】根据所给数据发现这组数的规律, 一个两位数乘 11 , 积为三位数, 个位为这个两位数的个位; 十位为这个两位数的个位上的数与十位上的数的和; 百位为这个两位数的十位上的数; 据此解答。\n\n【详解】 $12 \\times 11=132,24 \\times 11=264,32 \\times 11=352,43 \\times 11=473,82 \\times 11=902$ 。\n\n【点睛】本题主要考查“式”的规律, 关键根据所给算式发现这组算式的规律, 并运用规律做题。"} {"id": "8377", "image": [], "answer": "$\\quad 3 \\quad 81$\n\n【分析】结合估算的方法, 将 19 估成 20 , 将 69 估成 70 , 然后找出另一个乘数的大概值, 然后在第一个括号填一个最大的数使它乘 19 的积比 60 小; 在第二个括号里填入一个最大数, 使它与 69 的积比 5600 小即可。\n\n【详解】 $20 \\times 3=60,4 \\times 19=76,76>60$, 括号里最大填 3 ;\n\n$70 \\times 80=5600,69 \\times 81=5589,5589<5600,69 \\times 82=5658,5658>5600$, 因此括号里最大填 81 ;\n【点睛】熟练掌握整数乘法的计算是解答此题的关键。", "solution": "null", "level": "三年级", "question": "在下面的括号里最大能填几?\n\n$$\n(\\quad) \\times 19<60 \\quad 69 \\times(\\quad)<5600\n$$", "options": [], "subject": "算术", "analysis": "$\\quad 3 \\quad 81$\n\n【分析】结合估算的方法, 将 19 估成 20 , 将 69 估成 70 , 然后找出另一个乘数的大概值, 然后在第一个括号填一个最大的数使它乘 19 的积比 60 小; 在第二个括号里填入一个最大数, 使它与 69 的积比 5600 小即可。\n\n【详解】 $20 \\times 3=60,4 \\times 19=76,76>60$, 括号里最大填 3 ;\n\n$70 \\times 80=5600,69 \\times 81=5589,5589<5600,69 \\times 82=5658,5658>5600$, 因此括号里最大填 81 ;\n【点睛】熟练掌握整数乘法的计算是解答此题的关键。"} {"id": "8378", "image": [], "answer": "$56 \\times 78=4368 \\quad 75 \\times 86=6450$\n\n【分析】积的个位数是 $8,6 \\times 8=48$, 因此一个乘数的个位是 6 , 一个乘数的个位上是 8 ;积的末尾至少有一个 $0,5 \\times 6=30$, 因此一个乘数的个位是 5 , 一个乘数的个位上是 6 ; 因此列式即可。\n\n【详解】 $56 \\times 78=4368$;\n\n$75 \\times 86=6450 ;$\n\n【点睛】熟练掌握两位数乘两位数的计算是解答此题的关键。", "solution": "null", "level": "三年级", "question": "用 $5 、 6 、 7 、 8$ 这四个数字组成两位数乘两位数的算式, 写出一个积的个位数是 8 的算式( ),写出一个积的末尾至少有一个 0 的算式( 。", "options": [], "subject": "算术", "analysis": "$56 \\times 78=4368 \\quad 75 \\times 86=6450$\n\n【分析】积的个位数是 $8,6 \\times 8=48$, 因此一个乘数的个位是 6 , 一个乘数的个位上是 8 ;积的末尾至少有一个 $0,5 \\times 6=30$, 因此一个乘数的个位是 5 , 一个乘数的个位上是 6 ; 因此列式即可。\n\n【详解】 $56 \\times 78=4368$;\n\n$75 \\times 86=6450 ;$\n\n【点睛】熟练掌握两位数乘两位数的计算是解答此题的关键。"} {"id": "8379", "image": [], "answer": "3599\n\n【分析】与 60 相邻的两个数, 前面一个数就用: $60-1=59$, 后面一个数是: $60+1=61$; 然后运用整数的乘法进行解答。\n\n【详解】与 60 相邻的两个数是 59 和 61 。\n\n$59 \\times 61=3599$\n\n【点睛】本题主要考查了学生对数的顺序知识的掌握情况及整数的乘法。", "solution": "null", "level": "三年级", "question": "与 60 相邻的两个数的乘积是 ( $\\quad$ 。", "options": [], "subject": "算术", "analysis": "3599\n\n【分析】与 60 相邻的两个数, 前面一个数就用: $60-1=59$, 后面一个数是: $60+1=61$; 然后运用整数的乘法进行解答。\n\n【详解】与 60 相邻的两个数是 59 和 61 。\n\n$59 \\times 61=3599$\n\n【点睛】本题主要考查了学生对数的顺序知识的掌握情况及整数的乘法。"} {"id": "8399", "image": [], "answer": "1200\n\n【分析】每行有 38 个字, 每页有 32 行, 即 $32 \\times 38$, 把 32 看作 30,38 看作 40 , 然后再进一步解答。\n\n【详解】 $32 \\times 38 \\approx 30 \\times 40=1200($ 个)\n\n一份稿件, 每行有 38 个字, 每页有 32 行。这份稿件每页约有 1200 个字。\n\n【点睛】考查了整数乘法的估算, 把两个因数看作与它接近的整十数、整百数, 然后再进一步解答。", "solution": "null", "level": "三年级", "question": "一份稿件, 每行有 38 个字, 每页有 32 行。这份稿件每页大约有 ( ) 个字。", "options": [], "subject": "算术", "analysis": "1200\n\n【分析】每行有 38 个字, 每页有 32 行, 即 $32 \\times 38$, 把 32 看作 30,38 看作 40 , 然后再进一步解答。\n\n【详解】 $32 \\times 38 \\approx 30 \\times 40=1200($ 个)\n\n一份稿件, 每行有 38 个字, 每页有 32 行。这份稿件每页约有 1200 个字。\n\n【点睛】考查了整数乘法的估算, 把两个因数看作与它接近的整十数、整百数, 然后再进一步解答。"} {"id": "8401", "image": [], "answer": "$\\quad 3500 \\quad 3$\n\n【分析】乘法的估算,一般要根据“四舍五入”法把因数看作是整十、整百、整千.......的数来进行计算,然后按表内乘法的计算方法计算, 再在乘积的末尾添上相应的 0 即可;\n\n根据末尾有 0 的整数乘法的运算法则可知, 在计算 $25 \\times 40$ 时, 可先计算 $25 \\times 4,25 \\times 4$ 的结果是 100,然后再在 100 后边加上原来 40 后边的 0 , 积为 1000 , 即 $25 \\times 40$ 的积的末尾有 3 个零。\n\n【详解】 $69 \\times 48 \\approx 70 \\times 50=3500$;\n\n$25 \\times 40=1000$, 积的末尾有 3 个 0 。\n\n【点睛】在乘法计算过程中, 个位上是 5 与偶数相乘时积的末尾会有 0 出现。", "solution": "null", "level": "三年级", "question": "$69 \\times 48$ 的积大约是 (\n), $25 \\times 40$ 的积的末尾有 (\n)个 0 。", "options": [], "subject": "算术", "analysis": "$\\quad 3500 \\quad 3$\n\n【分析】乘法的估算,一般要根据“四舍五入”法把因数看作是整十、整百、整千.......的数来进行计算,然后按表内乘法的计算方法计算, 再在乘积的末尾添上相应的 0 即可;\n\n根据末尾有 0 的整数乘法的运算法则可知, 在计算 $25 \\times 40$ 时, 可先计算 $25 \\times 4,25 \\times 4$ 的结果是 100,然后再在 100 后边加上原来 40 后边的 0 , 积为 1000 , 即 $25 \\times 40$ 的积的末尾有 3 个零。\n\n【详解】 $69 \\times 48 \\approx 70 \\times 50=3500$;\n\n$25 \\times 40=1000$, 积的末尾有 3 个 0 。\n\n【点睛】在乘法计算过程中, 个位上是 5 与偶数相乘时积的末尾会有 0 出现。"} {"id": "8402", "image": ["1310.jpg", "1311.jpg", "1310.jpg", "1311.jpg"], "answer": "7; 336;\n\n$30 ; 1440$;\n\n37; 1776;\n\n【分析】平均每班有多少人, 则平均每班一顿午餐就需要多少盒酸奶, 因此用平均每班需要酸奶的盒数乘这个小学班级的总数即可, 两位数乘两位数的计算方法: 先是第二个因数的个位上的数与第一个因数相乘, 所得的积末尾与个位对齐; 接着第二个因数的十位上的数与第一个乘数相乘, 所得的积末尾与十位对齐,最后把两次乘得的积相加;依此填空。\n\n【详解】 $48 \\times 7=336$ (盒) $; 48 \\times 30=1440$ (盒); $336+1440=1776$ (盒);即填空如下:\n\n```\n48\n` 37\n\n```\n\n\n\n```\n\n```\n\n\n【点睛】熟练掌握两位数与两位数的乘法计算是解答此题的关键。", "solution": "null", "level": "三年级", "question": "春风小学有 37 个班, 平均每班 48 人。一顿午餐要为每人配备一盒酸奶, 一共需要多少盒酸奶?小刚是这样计算的:\n\n| | | | | |\n| ---: | ---: | ---: | :--- | :--- |\n| | 4 | | | |\n| $\\times \\quad 37$ | | | | |", "options": [], "subject": "算术", "analysis": "7; 336;\n\n$30 ; 1440$;\n\n37; 1776;\n\n【分析】平均每班有多少人, 则平均每班一顿午餐就需要多少盒酸奶, 因此用平均每班需要酸奶的盒数乘这个小学班级的总数即可, 两位数乘两位数的计算方法: 先是第二个因数的个位上的数与第一个因数相乘, 所得的积末尾与个位对齐; 接着第二个因数的十位上的数与第一个乘数相乘, 所得的积末尾与十位对齐,最后把两次乘得的积相加;依此填空。\n\n【详解】 $48 \\times 7=336$ (盒) $; 48 \\times 30=1440$ (盒); $336+1440=1776$ (盒);即填空如下:\n\n```\n48\n` 37\n\n```\n\n\n\n```\n\n```\n\n\n【点睛】熟练掌握两位数与两位数的乘法计算是解答此题的关键。"} {"id": "8403", "image": [], "answer": "132\n\n【分析】根据题意可知, 一盒有铅笔的支数 $\\times$ 铅笔的盒数 $=$ 铅笔的总支数, 依此列式并计算即可。\n\n【详解】 $12 \\times 11=132$ (支)\n\n【点睛】熟练掌握两位数与两位数的乘法计算是解答此题的关键。", "solution": "null", "level": "三年级", "question": "如下图, 一盒有 12 支铅笔, 11 盒共有 $(\\quad)$ 支铅笔。", "options": [], "subject": "算术", "analysis": "132\n\n【分析】根据题意可知, 一盒有铅笔的支数 $\\times$ 铅笔的盒数 $=$ 铅笔的总支数, 依此列式并计算即可。\n\n【详解】 $12 \\times 11=132$ (支)\n\n【点睛】熟练掌握两位数与两位数的乘法计算是解答此题的关键。"} {"id": "8404", "image": [], "answer": "9900\n\n【分析】最大的两位数是 99 , 最小的三位数是 100 , 则要求两个数的积, 列式为 $99 \\times 100$ 。\n\n【详解】 $99 \\times 100=9900$\n\n则最大的两位数与最小的三位数相乘的积是 9900 。\n\n【点睛】本题考查三位数乘两位数的计算方法, 旨在考查学生的计算能力。", "solution": "null", "level": "三年级", "question": "最大的两位数与最小的三位数相乘的积是 ( )。", "options": [], "subject": "算术", "analysis": "9900\n\n【分析】最大的两位数是 99 , 最小的三位数是 100 , 则要求两个数的积, 列式为 $99 \\times 100$ 。\n\n【详解】 $99 \\times 100=9900$\n\n则最大的两位数与最小的三位数相乘的积是 9900 。\n\n【点睛】本题考查三位数乘两位数的计算方法, 旨在考查学生的计算能力。"} {"id": "8405", "image": [], "answer": "1200 元\n\n【分析】用全系列的枚数乘每枚的价钱, 求出一套的价钱, 再乘购买的套数, 即可求出明明买了 2 套需要支付的钱数。\n\n【详解】 $40 \\times 15 \\times 2$\n\n$=600 \\times 2$\n\n$=1200($ 元 $)$\n\n明明买了 2 套需要支付 1200 元。\n\n【点睛】本题考查两位数乘两位数的计算及应用。理解题意, 找出数量关系, 列式计算即可。", "solution": "null", "level": "三年级", "question": "冬奥会的“一墩难求”传递出如今中国无以伦比的文化自信。冰墩墩运动造型陶瓷纪念章每枚 40 元,全系列共 15 枚,明明买了 2 套需要支付 $(\\quad) 。$", "options": [], "subject": "算术", "analysis": "1200 元\n\n【分析】用全系列的枚数乘每枚的价钱, 求出一套的价钱, 再乘购买的套数, 即可求出明明买了 2 套需要支付的钱数。\n\n【详解】 $40 \\times 15 \\times 2$\n\n$=600 \\times 2$\n\n$=1200($ 元 $)$\n\n明明买了 2 套需要支付 1200 元。\n\n【点睛】本题考查两位数乘两位数的计算及应用。理解题意, 找出数量关系, 列式计算即可。"} {"id": "8426", "image": [], "answer": "不能\n\n【分析】先计算出 25 张 20 元的总钱数, 用乘法计算, 然后与 549 元比较即可。\n\n【详解】 $20 \\times 25=500$ (元)\n\n500 元<549 元, 不能\n\n【点睛】熟练掌握两位数与两位数的乘法计算是解答此题的关键。", "solution": "null", "level": "三年级", "question": "一件羽线服 549 元,刘老师带 25 张 20 元能买到吗?()。(填“能”或“不能”)", "options": [], "subject": "算术", "analysis": "不能\n\n【分析】先计算出 25 张 20 元的总钱数, 用乘法计算, 然后与 549 元比较即可。\n\n【详解】 $20 \\times 25=500$ (元)\n\n500 元<549 元, 不能\n\n【点睛】熟练掌握两位数与两位数的乘法计算是解答此题的关键。"} {"id": "8428", "image": [], "answer": "3\n\n【分析】要使 $\\square 2 \\times 42$ 的积是四位数, 则 $\\square \\times 4$ 的积是三位数, 口内如果填写 $3,3 \\times 4=12$, 即 $32 \\times 42=1344$,积是四位数, 由此求解。\n\n【详解】 据分析可知:\n\n$32 \\times 42=1344$\n\n所以, 在 $\\square 2 \\times 42$ 这个算式中, 如果积是四位数, 口里最小的数是 3 。\n\n【点睛】本题运用整数的乘法口诀进行初步的估算, 然后再代入式子进行解答计算验证。", "solution": "null", "level": "三年级", "question": "在 $\\square 2 \\times 42$ 这个算式中, 如果积是四位数, 哩最小的数是 ( )。", "options": [], "subject": "算术", "analysis": "3\n\n【分析】要使 $\\square 2 \\times 42$ 的积是四位数, 则 $\\square \\times 4$ 的积是三位数, 口内如果填写 $3,3 \\times 4=12$, 即 $32 \\times 42=1344$,积是四位数, 由此求解。\n\n【详解】 据分析可知:\n\n$32 \\times 42=1344$\n\n所以, 在 $\\square 2 \\times 42$ 这个算式中, 如果积是四位数, 口里最小的数是 3 。\n\n【点睛】本题运用整数的乘法口诀进行初步的估算, 然后再代入式子进行解答计算验证。"} {"id": "8429", "image": [], "answer": "$><<=$\n\n【分析】第一小题, $24 \\times 50=1200,25 \\times 40=1000$, 所以, $24 \\times 50>25 \\times 40$;\n\n第二小题, $178 \\div 6=29 \\ldots . .4$, 所以, $178 \\div 6<30$ 。\n\n第三小题, $33<34$, 所以, $15 \\times 33<34 \\times 15$;\n\n第四小题, $240 \\div 4=60,420 \\div 7=60$, 所以, $240 \\div 4=420 \\div 7$ 。\n\n【详解】 $24 \\times 50>25 \\times 40$\n\n$178 \\div 6<30$\n\n$15 \\times 33<34 \\times 15$\n\n$240 \\div 4=420 \\div 7$\n\n【点睛】先计算出各个算式的结果, 再比较大小即可。", "solution": "null", "level": "三年级", "question": "在括号里填上“>”“<”或“=”。\n\n| $24 \\times 50($ | ) $25 \\times 40$ | $178 \\div 6($ | ) 30 |\n| :---: | :---: | :---: | :---: |\n| $15 \\times 33($ | ) $34 \\times 15$ | $240 \\div 4($ | ) $420 \\div 7$ |", "options": [], "subject": "算术", "analysis": "$><<=$\n\n【分析】第一小题, $24 \\times 50=1200,25 \\times 40=1000$, 所以, $24 \\times 50>25 \\times 40$;\n\n第二小题, $178 \\div 6=29 \\ldots . .4$, 所以, $178 \\div 6<30$ 。\n\n第三小题, $33<34$, 所以, $15 \\times 33<34 \\times 15$;\n\n第四小题, $240 \\div 4=60,420 \\div 7=60$, 所以, $240 \\div 4=420 \\div 7$ 。\n\n【详解】 $24 \\times 50>25 \\times 40$\n\n$178 \\div 6<30$\n\n$15 \\times 33<34 \\times 15$\n\n$240 \\div 4=420 \\div 7$\n\n【点睛】先计算出各个算式的结果, 再比较大小即可。"} {"id": "8430", "image": [], "answer": "$20 \\quad 620$\n\n【分析】下边竖式虚线方框中的“ 62 ”是用另一个乘数的十位上的数乘得的结果, 故表示 20 个 31 即 620, 据此解答。\n【详解】 $31 \\times 20=620$ (个)\n\n一个工业机器人平均每分钟做 31 个零件, 它工作 22 分钟能做多少个零件? 下边坚式虚线方框中的“ 62 ”表示机器人工作 $(20)$ 分钟做了 $(620)$ 个零件。\n\n【点睛】熟练掌握两位数乘两位数的计算方法并灵活运用是解答本题的关键。", "solution": "null", "level": "三年级", "question": "一个工业机器人平均每分钟做 31 个零件, 它工作 22 分钟能做多少个零件?下边坚式虚线方框中的“62”表示机器人工作 $(\\quad)$ 分钟做了 $(\\quad)$ 个零件。\n\n| 3 | 1 | |\n| ---: | ---: | ---: |\n| $\\times \\quad 2$ | 2 | |\n| | 6 | 2 |\n| | 2 | |\n| 6 | 8 | 1 |", "options": [], "subject": "算术", "analysis": "$20 \\quad 620$\n\n【分析】下边竖式虚线方框中的“ 62 ”是用另一个乘数的十位上的数乘得的结果, 故表示 20 个 31 即 620, 据此解答。\n【详解】 $31 \\times 20=620$ (个)\n\n一个工业机器人平均每分钟做 31 个零件, 它工作 22 分钟能做多少个零件? 下边坚式虚线方框中的“ 62 ”表示机器人工作 $(20)$ 分钟做了 $(620)$ 个零件。\n\n【点睛】熟练掌握两位数乘两位数的计算方法并灵活运用是解答本题的关键。"} {"id": "8431", "image": [], "answer": "$38 \\quad 2736$\n\n【分析】先用除法计算出可以装多少盒, 再用除法计算出可以装多少箱; 每箱风梨卖 72 元, 用乘法计算一共可以卖多少钱,据此解答。\n\n【详解】 $304 \\div 2 \\div 4$\n\n$=152 \\div 4$\n\n$=38$ (箱)\n\n$38 \\times 72=2736($ 元 $)$\n\n超市运来 304 个凤梨, 准备 2 个装一盒, 4 盒装一箱, 这些苹果可以装(38)箱。如果每箱凤梨卖 72 元,这些凤梨能卖(2736)元。\n\n【点睛】本题考查连除的实际应用以及两位数乘两位数的计算方法是解答本题的关键。", "solution": "null", "level": "三年级", "question": "超市运来 304 个凤梨,准备 2 个装一盒, 4 盒装一箱, 这些凤梨可以装( )箱。如果每箱凤梨卖 72 元,这些凤梨能卖( $\\quad$ 元。", "options": [], "subject": "算术", "analysis": "$38 \\quad 2736$\n\n【分析】先用除法计算出可以装多少盒, 再用除法计算出可以装多少箱; 每箱风梨卖 72 元, 用乘法计算一共可以卖多少钱,据此解答。\n\n【详解】 $304 \\div 2 \\div 4$\n\n$=152 \\div 4$\n\n$=38$ (箱)\n\n$38 \\times 72=2736($ 元 $)$\n\n超市运来 304 个凤梨, 准备 2 个装一盒, 4 盒装一箱, 这些苹果可以装(38)箱。如果每箱凤梨卖 72 元,这些凤梨能卖(2736)元。\n\n【点睛】本题考查连除的实际应用以及两位数乘两位数的计算方法是解答本题的关键。"} {"id": "8432", "image": [], "answer": "1440\n\n【分析】用一圈所用的时间乘 16 , 就是 16 圈所用的时间, 据此解答。\n\n【详解】 $90 \\times 16=1440$ (分钟)\n\n中国空间站绕地球一圈的时间大约是 90 分钟, 绕地球 16 圈, 需要 (1440) 分钟。\n\n【点睛】本题考查乘法的意义以及两位数乘两位数的计算方法, 应熟练掌握并灵活运用。", "solution": "null", "level": "三年级", "question": "中国空间站绕地球一圈的时间大约是 90 分钟, 绕地球 16 圈, 需要 (\n\n)分钟。", "options": [], "subject": "算术", "analysis": "1440\n\n【分析】用一圈所用的时间乘 16 , 就是 16 圈所用的时间, 据此解答。\n\n【详解】 $90 \\times 16=1440$ (分钟)\n\n中国空间站绕地球一圈的时间大约是 90 分钟, 绕地球 16 圈, 需要 (1440) 分钟。\n\n【点睛】本题考查乘法的意义以及两位数乘两位数的计算方法, 应熟练掌握并灵活运用。"} {"id": "8452", "image": [], "answer": "三3 四4\n\n【分析】根据两位数乘两位数的计算法则, 分别计算出 $39 \\times 18 、 52 \\times 21$ 的积, 然后再根据计算出的结果填空即可。\n\n【详解】 $39 \\times 18=702$, 即 $39 \\times 18$ 的积是三位数;\n\n$52 \\times 21=1092$, 即 $52 \\times 21$ 的积是四位数。\n\n【点睛】熟练掌握两位数与两位数的乘法计算是解答此题的关键。", "solution": "null", "level": "三年级", "question": "$39 \\times 18$ 的积是 $\\quad$ )位数; $52 \\times 21$ 的积是 $(\\quad)$ 位数。", "options": [], "subject": "算术", "analysis": "三3 四4\n\n【分析】根据两位数乘两位数的计算法则, 分别计算出 $39 \\times 18 、 52 \\times 21$ 的积, 然后再根据计算出的结果填空即可。\n\n【详解】 $39 \\times 18=702$, 即 $39 \\times 18$ 的积是三位数;\n\n$52 \\times 21=1092$, 即 $52 \\times 21$ 的积是四位数。\n\n【点睛】熟练掌握两位数与两位数的乘法计算是解答此题的关键。"} {"id": "8454", "image": [], "answer": "16 或 17\n\n【分析】每箱的重量比 15 千克重, 卖出 20 箱, 所以卖出的总质量要大于 $(15 \\times 20)$ 千克, 然后求出剩下的质量, 再进一步解答即可。\n\n【详解】 $15 \\times 20=300$ (千克)\n\n$360-300=60$ (千克)\n\n所以剩下质量要小于 60 千克,\n\n20 个 3 千克是 60 千克\n\n所以每箱的重量比 15 千克重可以多 1 千克或 2 千克,\n\n$15+1=16$ (千克)\n\n$15+2=17$ (千克)\n\n即每箱苹果可能重 16 或 17 千克。\n\n【点睛】解答本题关键是求出余下的质量的取值范围。", "solution": "null", "level": "三年级", "question": "水果店进了 360 千克苹果, 每箱的重量比 15 千克重, 卖出 20 箱后, 还有剩余。每箱苹果可能重 )千克。", "options": [], "subject": "算术", "analysis": "16 或 17\n\n【分析】每箱的重量比 15 千克重, 卖出 20 箱, 所以卖出的总质量要大于 $(15 \\times 20)$ 千克, 然后求出剩下的质量, 再进一步解答即可。\n\n【详解】 $15 \\times 20=300$ (千克)\n\n$360-300=60$ (千克)\n\n所以剩下质量要小于 60 千克,\n\n20 个 3 千克是 60 千克\n\n所以每箱的重量比 15 千克重可以多 1 千克或 2 千克,\n\n$15+1=16$ (千克)\n\n$15+2=17$ (千克)\n\n即每箱苹果可能重 16 或 17 千克。\n\n【点睛】解答本题关键是求出余下的质量的取值范围。"} {"id": "8455", "image": [], "answer": "4500\n\n【分析】用运动服的价格乘购买的套数, 求出购买 50 套需要的钱数即可, 注意计算时用估算的方法计算即可。\n\n【详解】 $89 \\times 50$\n\n$\\approx 90 \\times 50$\n\n$=4500 ($ 元 $)$\n\n所以,大约需要 4500 元。\n\n【点睛】熟练掌握两位数乘两位数的估算方法, 是解答此题的关键。", "solution": "null", "level": "三年级", "question": "每套运动服 89 元, 买 50 套这样的衣服, 大约需要 $($ 元。", "options": [], "subject": "算术", "analysis": "4500\n\n【分析】用运动服的价格乘购买的套数, 求出购买 50 套需要的钱数即可, 注意计算时用估算的方法计算即可。\n\n【详解】 $89 \\times 50$\n\n$\\approx 90 \\times 50$\n\n$=4500 ($ 元 $)$\n\n所以,大约需要 4500 元。\n\n【点睛】熟练掌握两位数乘两位数的估算方法, 是解答此题的关键。"} {"id": "8456", "image": [], "answer": "四\\#\\# 4\n\n【分析】要知道 $94 \\times 3$ 口的积是几位数, 那么直接计算出 $94 \\times 30$ 或 $94 \\times 39$ 的积即可填空;\n三位数除以一位数, 要使商中间有 0 , 那么被除数百位上的数要等于或大于除数, 并且要是除数的倍数, 被除数十位上的数还要比除数小, 依此填空。\n\n【详解】 $94 \\times 30=2820,94 \\times 39=3666$, 即 $94 \\times 3$ 口的积是四位数;要使 5 口 $9 \\div 5$ 的商中间有 $0,5=5, \\square<5$, 即 $\\square$ 里最大能填 4 。\n\n【点睛】此题考查的是两位数与两位数的乘法计算, 以及三位数除以一位数的试商, 应熟练掌握。", "solution": "null", "level": "三年级", "question": "$94 \\times 3$ 口的积是 $(\\quad)$ 位数; 要使 $5 \\square 9 \\div 5$ 的商中间有 0 , 哩最大能填 $(\\quad) 。$", "options": [], "subject": "算术", "analysis": "四\\#\\# 4\n\n【分析】要知道 $94 \\times 3$ 口的积是几位数, 那么直接计算出 $94 \\times 30$ 或 $94 \\times 39$ 的积即可填空;\n三位数除以一位数, 要使商中间有 0 , 那么被除数百位上的数要等于或大于除数, 并且要是除数的倍数, 被除数十位上的数还要比除数小, 依此填空。\n\n【详解】 $94 \\times 30=2820,94 \\times 39=3666$, 即 $94 \\times 3$ 口的积是四位数;要使 5 口 $9 \\div 5$ 的商中间有 $0,5=5, \\square<5$, 即 $\\square$ 里最大能填 4 。\n\n【点睛】此题考查的是两位数与两位数的乘法计算, 以及三位数除以一位数的试商, 应熟练掌握。"} {"id": "8457", "image": ["1323.jpg", "1324.jpg", "1324.jpg"], "answer": "4; 180; 20; 900; 24; 1080\n\n【分析】根据两位数乘两位数乘法的笔算法则: 先用一个乘数个位上的数去乘另一个乘数, 再用一个乘数十位上的数去乘另一个乘数, 最后把它们的积加起来, 然后直接进行填空即可。\n\n\n\n【点睛】此题主要考查两位数乘两位数的乘法, 明确乘法各步的意义是解答本题的关键。", "solution": "null", "level": "三年级", "question": "每个篮球的价格是 45 元, 王老师买了 24 个篮球, 一共需要多少元? 在下面的括号里填上合适的数。\n\n", "options": [], "subject": "算术", "analysis": "4; 180; 20; 900; 24; 1080\n\n【分析】根据两位数乘两位数乘法的笔算法则: 先用一个乘数个位上的数去乘另一个乘数, 再用一个乘数十位上的数去乘另一个乘数, 最后把它们的积加起来, 然后直接进行填空即可。\n\n\n\n【点睛】此题主要考查两位数乘两位数的乘法, 明确乘法各步的意义是解答本题的关键。"} {"id": "8473", "image": [], "answer": "$34 \\quad 450$\n\n【分析】计算一个数是另一个数的几倍, 用除法计算; 最小的两位数是 10 , 因此直接计算出 10 与 45 的积即可。\n\n【详解】 $136 \\div 4=34$, 即 136 是 4 的 34 倍;\n\n$10 \\times 45=450$, 即最小的两位数与 45 的积是 450 。\n\n【点睛】此题考查的是三位数与一位数的除法计算, 以及整十数与两位数的乘法口算, 应熟练掌握。", "solution": "null", "level": "三年级", "question": "136 是 4 的 ( )倍; 最小的两位数与 45 的积是 ( )。", "options": [], "subject": "算术", "analysis": "$34 \\quad 450$\n\n【分析】计算一个数是另一个数的几倍, 用除法计算; 最小的两位数是 10 , 因此直接计算出 10 与 45 的积即可。\n\n【详解】 $136 \\div 4=34$, 即 136 是 4 的 34 倍;\n\n$10 \\times 45=450$, 即最小的两位数与 45 的积是 450 。\n\n【点睛】此题考查的是三位数与一位数的除法计算, 以及整十数与两位数的乘法口算, 应熟练掌握。"} {"id": "8474", "image": [], "answer": "$\\quad=><$\n\n【分析】 (1) 根据整数乘法计算方法, 分别求出两个算式的积, 再比较大小;\n\n(2) 千克和吨之间的进率是 1000 , 据此将 9 吨换算成千克, 再和 9 千克比较大小;\n\n(3) $7 \\square \\square \\div 8$ 中, 被除数最高位上的数小于除数, 商是两位数; $7 \\square \\square \\div 6$ 中, 被除数最高位上的数大于除数, 商是三位数; 任何一个两位数都小于三位数, 则 $7 \\square \\square \\div 8<7 \\square \\square \\div 6$ 。\n\n【详解】 $26 \\times 30=780,260 \\times 3=780$, 则 $26 \\times 30=260 \\times 3$\n\n9 吨 $=9000$ 千克, 9000 千克 $>9$ 千克, 则 9 吨 $>9$ 千克\n\n$7 \\square \\square \\div 8$ 的商是两位数, $7 \\square \\square \\div 6$ 的商是三位数, 则 $7 \\square \\square \\div 8<7 \\square \\square \\div 6$\n\n【点睛】不同单位的数比较大小, 要先换算成同一单位的数, 再进行比较。三位数除以一位数, 如果被除数的最高位小于除数, 则商是两位数; 如果被除数的最高位大于或等于除数, 则商是三位数。", "solution": "null", "level": "三年级", "question": "在括号里填上“>”“惐“=”。\n\n$26 \\times 30(\\quad) 260 \\times 3 \\quad 9$ 吨( ) 千克 7 7口ם $\\div 8(\\quad 7 \\square \\square \\div 6$", "options": [], "subject": "算术", "analysis": "$\\quad=><$\n\n【分析】 (1) 根据整数乘法计算方法, 分别求出两个算式的积, 再比较大小;\n\n(2) 千克和吨之间的进率是 1000 , 据此将 9 吨换算成千克, 再和 9 千克比较大小;\n\n(3) $7 \\square \\square \\div 8$ 中, 被除数最高位上的数小于除数, 商是两位数; $7 \\square \\square \\div 6$ 中, 被除数最高位上的数大于除数, 商是三位数; 任何一个两位数都小于三位数, 则 $7 \\square \\square \\div 8<7 \\square \\square \\div 6$ 。\n\n【详解】 $26 \\times 30=780,260 \\times 3=780$, 则 $26 \\times 30=260 \\times 3$\n\n9 吨 $=9000$ 千克, 9000 千克 $>9$ 千克, 则 9 吨 $>9$ 千克\n\n$7 \\square \\square \\div 8$ 的商是两位数, $7 \\square \\square \\div 6$ 的商是三位数, 则 $7 \\square \\square \\div 8<7 \\square \\square \\div 6$\n\n【点睛】不同单位的数比较大小, 要先换算成同一单位的数, 再进行比较。三位数除以一位数, 如果被除数的最高位小于除数, 则商是两位数; 如果被除数的最高位大于或等于除数, 则商是三位数。"} {"id": "8475", "image": [], "answer": "3 四\n\n【分析】 直接计算出 $25 \\times 40$ 的积即可解答。\n\n【详解】 $25 \\times 40=1000$, 积的末尾有 3 个 0 , 积是四位数。\n\n【点睛】熟练掌握整数乘法的计算方法是解答本题的关键。", "solution": "null", "level": "三年级", "question": "$25 \\times 40$ 的积的末尾有 $(\\quad)$ 个 0 , 积是 ( ) 位数。", "options": [], "subject": "算术", "analysis": "3 四\n\n【分析】 直接计算出 $25 \\times 40$ 的积即可解答。\n\n【详解】 $25 \\times 40=1000$, 积的末尾有 3 个 0 , 积是四位数。\n\n【点睛】熟练掌握整数乘法的计算方法是解答本题的关键。"} {"id": "8477", "image": [], "answer": "456\n\n【分析】根据题意可知, 用学校买牛奶的箱数乘每箱的瓶数即可, 一共有多少瓶, 则就够分给多少名学生。\n\n【详解】 $38 \\times 12=456$ (瓶)\n\n一共有 456 瓶, 则够分给 456 名学生。\n\n【点睛】熟练掌握两位数与两位数的乘法计算是解答此题的关键。", "solution": "null", "level": "三年级", "question": "光明学校买来 38 箱牛奶, 每箱有 12 瓶, 若每个学生 1 瓶, 则够分给 (\n\n)名学生。", "options": [], "subject": "算术", "analysis": "456\n\n【分析】根据题意可知, 用学校买牛奶的箱数乘每箱的瓶数即可, 一共有多少瓶, 则就够分给多少名学生。\n\n【详解】 $38 \\times 12=456$ (瓶)\n\n一共有 456 瓶, 则够分给 456 名学生。\n\n【点睛】熟练掌握两位数与两位数的乘法计算是解答此题的关键。"} {"id": "8478", "image": [], "answer": "32\n\n【分析】 $15 \\times 32=480,25 \\times 32=800,35 \\times 32=1120$, 则要使 $\\square 5 \\times 32$ 的积是四位数, $\\square$ 里的数应大于等于 3 。要使积是三位数, 哩的数小于 3 。\n\n【详解】由分析得:\n\n要使 $\\square 5 \\times 32$ 的积是四位数, 口里最小填 3 , 要使积是三位数, 口里最大填 2 。\n\n【点睛】本题考查两位数乘两位数的计算, 采用赋值法解答更简便。", "solution": "null", "level": "三年级", "question": "要使 $\\square 5 \\times 32$ 的积是四位数, 口里最小填 (\n\n) , 要使积是三位数, 哩最大填 ( $\\quad)$", "options": [], "subject": "算术", "analysis": "32\n\n【分析】 $15 \\times 32=480,25 \\times 32=800,35 \\times 32=1120$, 则要使 $\\square 5 \\times 32$ 的积是四位数, $\\square$ 里的数应大于等于 3 。要使积是三位数, 哩的数小于 3 。\n\n【详解】由分析得:\n\n要使 $\\square 5 \\times 32$ 的积是四位数, 口里最小填 3 , 要使积是三位数, 口里最大填 2 。\n\n【点睛】本题考查两位数乘两位数的计算, 采用赋值法解答更简便。"} {"id": "8479", "image": [], "answer": "三 四\n\n【分析】用最大的两位数乘最大的两位数, 乘积最大。用最小的两位数乘最小的两位数, 乘积最小。据此求出最大和最小的乘积, 进而判断乘积是几位数。\n\n【详解】 $99 \\times 99=9801$\n\n$10 \\times 10=100$\n\n则两位数乘两位数, 积最少是三位数, 最多是四位数。\n【点睛】本题考查两位数乘两位数的计算方法, 明确最大乘积是 $99 \\times 99$, 最小乘积是 $10 \\times 10$ 。", "solution": "null", "level": "三年级", "question": "两位数乘两位数, 积最少是 ( ) 位数, 最多是 ( )位数。", "options": [], "subject": "算术", "analysis": "三 四\n\n【分析】用最大的两位数乘最大的两位数, 乘积最大。用最小的两位数乘最小的两位数, 乘积最小。据此求出最大和最小的乘积, 进而判断乘积是几位数。\n\n【详解】 $99 \\times 99=9801$\n\n$10 \\times 10=100$\n\n则两位数乘两位数, 积最少是三位数, 最多是四位数。\n【点睛】本题考查两位数乘两位数的计算方法, 明确最大乘积是 $99 \\times 99$, 最小乘积是 $10 \\times 10$ 。"} {"id": "8480", "image": [], "answer": "900\n\n【分析】根据速度 $\\times$ 时间 $=$ 路程, 把速度与时间的数量代入, 即可求出路程。\n\n【详解】 $60 \\times 15=900$ (米)\n\n【点睛】此题考查了速度、时间、路程之间的关系, 需要熟练掌握并应用。", "solution": "null", "level": "三年级", "question": "小林每分钟走 60 米,他 15 分钟走 ( )米。", "options": [], "subject": "算术", "analysis": "900\n\n【分析】根据速度 $\\times$ 时间 $=$ 路程, 把速度与时间的数量代入, 即可求出路程。\n\n【详解】 $60 \\times 15=900$ (米)\n\n【点睛】此题考查了速度、时间、路程之间的关系, 需要熟练掌握并应用。"} {"id": "8481", "image": [], "answer": "$109 \\quad 990$\n\n【分析】最大的两位数是 99 , 最小的两位数是 10 , 把两数相加即是和, 把两数相乘即是积。\n\n【详解】最大的两位数是 99 , 最小的两位数是 10 。\n\n它们的和是: $99+10=109$\n\n它们的积是: $99 \\times 10=990$\n\n【点睛】本题关键是求出最大的两位数与最小的两位数, 然后再进一步解答。", "solution": "null", "level": "三年级", "question": "最大的两位数与最小的两位数的和是 ( ), 乘积是( $\\quad$ 。", "options": [], "subject": "算术", "analysis": "$109 \\quad 990$\n\n【分析】最大的两位数是 99 , 最小的两位数是 10 , 把两数相加即是和, 把两数相乘即是积。\n\n【详解】最大的两位数是 99 , 最小的两位数是 10 。\n\n它们的和是: $99+10=109$\n\n它们的积是: $99 \\times 10=990$\n\n【点睛】本题关键是求出最大的两位数与最小的两位数, 然后再进一步解答。"} {"id": "8500", "image": [], "answer": "648\n\n【分析】乘数 24 个位上的 4 看成 9, 则这个乘数增加 $29-24=5$, 乘积就增加 5 个另一个乘数, 另一个乘数是 $135 \\div 5=27$ 。算式就是 $24 \\times 27$, 求出正确的积即可。\n\n【详解】 $135 \\div(29-24)$\n\n$=135 \\div 5$\n\n$=27$\n\n$24 \\times 27=648$\n\n正确的积应该是 648 。\n\n【点睛】本题关键是明确乘数增加几或者减少几, 另一个乘数不变, 积就增加或者减少几个另一个乘数。", "solution": "null", "level": "三年级", "question": "东东在计算两位数乘两位数时, 把其中一个乘数 24 个位上的 4 看成 9 , 结果比正确的乘积多了 135 ,正确的积应该是 ( $\\quad$ 。", "options": [], "subject": "算术", "analysis": "648\n\n【分析】乘数 24 个位上的 4 看成 9, 则这个乘数增加 $29-24=5$, 乘积就增加 5 个另一个乘数, 另一个乘数是 $135 \\div 5=27$ 。算式就是 $24 \\times 27$, 求出正确的积即可。\n\n【详解】 $135 \\div(29-24)$\n\n$=135 \\div 5$\n\n$=27$\n\n$24 \\times 27=648$\n\n正确的积应该是 648 。\n\n【点睛】本题关键是明确乘数增加几或者减少几, 另一个乘数不变, 积就增加或者减少几个另一个乘数。"} {"id": "8501", "image": [], "answer": "$960 \\quad 160$\n\n【分析】用小学数量乘平均每所小学老师数量, 求出老师总数量。再用老师总数量除以批次数量, 求出平均每批老师人数。\n\n【详解】 $12 \\times 80=960$ (人)\n\n$960 \\div 6=160($ 人)\n\n该区一共有 960 名老师,平均每批有 160 人。\n\n【点睛】本题考查学生对乘法意义和除法意义的掌握情况, 据此列出算式解答。", "solution": "null", "level": "三年级", "question": "接种新冠病毒疫苗是预防新冠肺炎最经济有效的手段。某区有 12 所小学, 平均每所小学有 80 名老师, 该区一共有 $($ ) 名老师。为配合国家防疫要求, 将这些老师分成 6 批接种新冠疫苗, 平均每批有 $(\\quad)$ 人。", "options": [], "subject": "算术", "analysis": "$960 \\quad 160$\n\n【分析】用小学数量乘平均每所小学老师数量, 求出老师总数量。再用老师总数量除以批次数量, 求出平均每批老师人数。\n\n【详解】 $12 \\times 80=960$ (人)\n\n$960 \\div 6=160($ 人)\n\n该区一共有 960 名老师,平均每批有 160 人。\n\n【点睛】本题考查学生对乘法意义和除法意义的掌握情况, 据此列出算式解答。"} {"id": "8504", "image": ["2954.jpg"], "answer": "20\n\n【分析】 利用估算的方法, 把 29 估成 $30,20 \\times 30=600,20 \\times 29<20 \\times 30=600$, 据此填空即可。\n\n【详解】 $\\times 29<600$, 山最大是 $(20)$ (填整数)。\n\n【点睛】在乘法估算中一般要根据“四舍五入”法,把因数看作是整十、整百、整千 . 的数来进行计算。", "solution": "null", "level": "三年级", "question": "", "options": [], "subject": "算术", "analysis": ":分析:(1)由点 $\\mathrm{A}$ 在抛物线上, 将 $\\mathrm{A}$ 点坐标代入, 求出参数 $\\mathrm{P}$, 求解即可 (2) 由于 $F(8,0)$ 是 $\\triangle A B C$ 的重心, 则重心与焦点重合, 由重心坐标公式可求 $M$ 是 $B C$ 的中点。\n\n(3) 由于线段 $\\mathrm{BC}$ 的中点 $\\mathrm{M}$ 不在 $x$ 轴上, 所以 $\\mathrm{BC}$ 所在的直线不垂直于 $x$ 轴. 设 $\\mathrm{BC}$ 所在直线的方程为: $y+4=k(x-11)(k \\neq 0)$, 解出 $k$ 即可。"} {"id": "8505", "image": [], "answer": "450\n\n【分析】一个数去掉末尾的 0 后, 则原来的数为新数的 10 倍, 因此将新数看成 1 份, 则原来的数为 10 份, 依此计算出得到的新数比原数少的份数, 并用 405 除以得到的新数比原数少的份数即可计算出新数, 最后用新数乘 10 即可。\n\n【详解】 $10-1=9($ 份 $)$\n\n$405 \\div 9=45$\n\n$45 \\times 10=450$\n\n【点睛】此题考查的是和差倍问题, 先计算出得到的新数比原数少的份数是解答此题的关键。", "solution": "null", "level": "三年级", "question": "一个数去掉末尾的 0 后, 得到的新数比原数少了 405 , 原数是 $(\\quad) 。$", "options": [], "subject": "算术", "analysis": "450\n\n【分析】一个数去掉末尾的 0 后, 则原来的数为新数的 10 倍, 因此将新数看成 1 份, 则原来的数为 10 份, 依此计算出得到的新数比原数少的份数, 并用 405 除以得到的新数比原数少的份数即可计算出新数, 最后用新数乘 10 即可。\n\n【详解】 $10-1=9($ 份 $)$\n\n$405 \\div 9=45$\n\n$45 \\times 10=450$\n\n【点睛】此题考查的是和差倍问题, 先计算出得到的新数比原数少的份数是解答此题的关键。"} {"id": "8506", "image": ["1333.jpg", "1334.jpg"], "answer": "$1440 \\quad 12$\n\n【分析】书包的单价乘 36 即等于 36 个书包的总价; 108 元除以文具盒的单价即等于能买的个数。\n\n【详解】 $40 \\times 36=1440 ($ 元)\n\n$108 \\div 9=12($ 个)\n\n【点睛】熟练掌握总价、单价和数量之间的关系是解答本题的关键。", "solution": "null", "level": "三年级", "question": "买 36 个书包, 一共花(\n)元。李老师有 108 元, 能买(\n)个文具盒。\n\n\n\n9 元\n\n", "options": [], "subject": "算术", "analysis": "$1440 \\quad 12$\n\n【分析】书包的单价乘 36 即等于 36 个书包的总价; 108 元除以文具盒的单价即等于能买的个数。\n\n【详解】 $40 \\times 36=1440 ($ 元)\n\n$108 \\div 9=12($ 个)\n\n【点睛】熟练掌握总价、单价和数量之间的关系是解答本题的关键。"} {"id": "8507", "image": ["1335.jpg"], "answer": "20\n\n【分析】箭头所指“ 28 ”是 14 与 23 十位上 2 的乘积, 表示 20 套书的本数, 据此即可解答。\n\n【详解】根据分析可知, 箭头所指“ 28 ” 表示 20 套书的本数。\n\n【点睛】本题主要考查学生对整数乘法计算方法的掌握。", "solution": "null", "level": "三年级", "question": "王老师买了 23 套课外书, 每套 14 本。求买来课外书的总本数, 可以这样计算:\n\n", "options": [], "subject": "算术", "analysis": "20\n\n【分析】箭头所指“ 28 ”是 14 与 23 十位上 2 的乘积, 表示 20 套书的本数, 据此即可解答。\n\n【详解】根据分析可知, 箭头所指“ 28 ” 表示 20 套书的本数。\n\n【点睛】本题主要考查学生对整数乘法计算方法的掌握。"} {"id": "8528", "image": [], "answer": "$>>=<=<$\n\n【分析】根据除数是一位数的除法计算方法以及两位数乘两位数的计算方法、三位数乘一位数的计算方法, 分别求出各个算式的得数, 再比较大小。\n\n【详解】 $408 \\div 8=51,51>50$, 则 $408 \\div 8>50$\n\n$15 \\times 30=450,450>400$, 则 $15 \\times 30>400$\n\n$70 \\times 15=1050,150 \\times 7=1050$, 则 $70 \\times 15=150 \\times 7$\n\n$96 \\div 6=16,16<18$, 则 $96 \\div 6<18$\n\n$24 \\times 16=384$\n\n$560 \\div 4=140,560 \\div 2=280,140<280$, 则 $560 \\div 4<560 \\div 2$\n\n【点睛】熟练掌握整数乘除法计算方法并正确计算是解决本题的关键。", "solution": "null", "level": "三年级", "question": "在括号里填上“>”“<”或“=”。\n\n| $408 \\div 8($ | ) 50 | $15 \\times 30($ | ) 400 | $70 \\times 15($ | ) $150 \\times 7$ |\n| :---: | :---: | :---: | :---: | :---: | :---: |\n| $96 \\div 6($ | ) 18 | $24 \\times 16($ | ) 384 | $560 \\div 4($ | ) $560 \\div 2$ |\n\n8 . _若 $\\square 36 \\div 5$ 的商是两位数, 则 $\\square$ 里最大填 $(\\quad)$; 若 $\\square 4 \\times 23$ 的积是四位数, 则 $\\square$ 里最小填 $(\\quad)$ 。", "options": [], "subject": "算术", "analysis": "$>>=<=<$\n\n【分析】根据除数是一位数的除法计算方法以及两位数乘两位数的计算方法、三位数乘一位数的计算方法, 分别求出各个算式的得数, 再比较大小。\n\n【详解】 $408 \\div 8=51,51>50$, 则 $408 \\div 8>50$\n\n$15 \\times 30=450,450>400$, 则 $15 \\times 30>400$\n\n$70 \\times 15=1050,150 \\times 7=1050$, 则 $70 \\times 15=150 \\times 7$\n\n$96 \\div 6=16,16<18$, 则 $96 \\div 6<18$\n\n$24 \\times 16=384$\n\n$560 \\div 4=140,560 \\div 2=280,140<280$, 则 $560 \\div 4<560 \\div 2$\n\n【点睛】熟练掌握整数乘除法计算方法并正确计算是解决本题的关键。"} {"id": "8529", "image": [], "answer": "$\\quad 22$ 6 $\\quad 6 \\quad 700 \\quad 80 \\quad 101 \\quad 4 \\quad 900 \\quad 4 \\quad 600$\n\n【分析】 (1) 根据积 $\\div$ 一个因数 $=$ 另一个因数解答。\n\n(2)计算因数末尾有 0 的乘法, 先把 0 前面的数相乘, 再看两个因数的末尾一共有几个 0 , 就在积的末尾添几个 0 。据此解答。\n\n【详解】 $66 \\div 3=22$, 则 $22 \\times 3=66$\n$6 \\times 7=42$, 则 $6 \\times 700=4200 ($ 答案不唯一)\n\n$640 \\div 8=80$, 则 $8 \\times 80=640$\n\n$404 \\div 4=101$, 则 $101 \\times 4=404$\n\n$4 \\times 9=36$, 则 $4 \\times 900=3600 ($ 答案不唯一)\n\n$4 \\times 6=24$, 则 $4 \\times 600=2400 ($ 答案不唯一)\n\n【点睛】本题考查乘法各部分之间的关系以及因数末尾有 0 的乘法, 要求哪两个数的积是几千几百,先看积的前两位数是由哪两个数相乘得到的, 再在这两个数其中一个的末尾添上 2 个零即可。", "solution": "null", "level": "三年级", "question": "在括号里填上合适的数。\n\n| $(\\quad) \\times 3=66$ | $(\\quad) \\times(\\quad)=4200$ | $8 \\times(\\quad)=640$ | | |\n| :--- | :--- | :--- | :--- | :--- | :--- |\n| $(\\quad) \\times 4=404$ | ( | )$\\times(\\quad)=3600$ | ( | )$\\times(\\quad)=2400$ |", "options": [], "subject": "算术", "analysis": "$\\quad 22$ 6 $\\quad 6 \\quad 700 \\quad 80 \\quad 101 \\quad 4 \\quad 900 \\quad 4 \\quad 600$\n\n【分析】 (1) 根据积 $\\div$ 一个因数 $=$ 另一个因数解答。\n\n(2)计算因数末尾有 0 的乘法, 先把 0 前面的数相乘, 再看两个因数的末尾一共有几个 0 , 就在积的末尾添几个 0 。据此解答。\n\n【详解】 $66 \\div 3=22$, 则 $22 \\times 3=66$\n$6 \\times 7=42$, 则 $6 \\times 700=4200 ($ 答案不唯一)\n\n$640 \\div 8=80$, 则 $8 \\times 80=640$\n\n$404 \\div 4=101$, 则 $101 \\times 4=404$\n\n$4 \\times 9=36$, 则 $4 \\times 900=3600 ($ 答案不唯一)\n\n$4 \\times 6=24$, 则 $4 \\times 600=2400 ($ 答案不唯一)\n\n【点睛】本题考查乘法各部分之间的关系以及因数末尾有 0 的乘法, 要求哪两个数的积是几千几百,先看积的前两位数是由哪两个数相乘得到的, 再在这两个数其中一个的末尾添上 2 个零即可。"} {"id": "8531", "image": [], "answer": "【分析】 $80 \\times 90=7200$, 因此只要这四个数组成的两个因数分别比 $80 、 90$ 大即可, 依此填空;要使这几个数组成的两位数乘两位数积最小, 两个乘数的十位上应分别为 2 和 4 , 因此直接计算出 $28 \\times 49$ 和 $29 \\times 48$ 的积, 然后再比较即可。\n\n【详解】 $84>80,92>90$, 因此 $84 \\times 92$ 的积大于 7000 ;\n\n$28 \\times 49=1372 ; 29 \\times 48=1392 ; 1372<1392$, 即把 $2,4,8,9$ 组成的两位数乘两位数中, $28 \\times 49$ 的积最小。\n\n【点睛】熟练掌握两位数与两位数的乘法计算是解答此题的关键。", "solution": "null", "level": "三年级", "question": "把 $2,4,8,9$ 按要求填入下面的括号内, 且括号内的数不相同。积大于 $7000:$积最小: $(\\quad)(\\quad) \\times(\\quad)$", "options": [], "subject": "算术", "analysis": "【分析】 $80 \\times 90=7200$, 因此只要这四个数组成的两个因数分别比 $80 、 90$ 大即可, 依此填空;要使这几个数组成的两位数乘两位数积最小, 两个乘数的十位上应分别为 2 和 4 , 因此直接计算出 $28 \\times 49$ 和 $29 \\times 48$ 的积, 然后再比较即可。\n\n【详解】 $84>80,92>90$, 因此 $84 \\times 92$ 的积大于 7000 ;\n\n$28 \\times 49=1372 ; 29 \\times 48=1392 ; 1372<1392$, 即把 $2,4,8,9$ 组成的两位数乘两位数中, $28 \\times 49$ 的积最小。\n\n【点睛】熟练掌握两位数与两位数的乘法计算是解答此题的关键。"} {"id": "8532", "image": [], "answer": "7\n\n【分析】 $24 \\times 32=768,34 \\times 32=1088$, 据此即可解答。\n\n【详解】根据分析可知, 要使 $\\square 4 \\times 32$ 的积是四位数, 则 $\\square \\geq 3$, $\\square$ 中可填的数有 $3 、 4 、 5 、 6 、 7 、 8 、 9$,共有 7 个。\n\n【点睛】熟练掌握两位数乘两位数的乘法计算方法是解答本题的关键。", "solution": "null", "level": "三年级", "question": "$\\square 4$ 是一个两位数, 要使 $\\square 4 \\times 32$ 的积是四位数, 口中可填的数有 $(\\quad)$ 个。", "options": [], "subject": "算术", "analysis": "7\n\n【分析】 $24 \\times 32=768,34 \\times 32=1088$, 据此即可解答。\n\n【详解】根据分析可知, 要使 $\\square 4 \\times 32$ 的积是四位数, 则 $\\square \\geq 3$, $\\square$ 中可填的数有 $3 、 4 、 5 、 6 、 7 、 8 、 9$,共有 7 个。\n\n【点睛】熟练掌握两位数乘两位数的乘法计算方法是解答本题的关键。"} {"id": "8533", "image": [], "answer": "$\\quad 660 \\quad 9$\n\n【分析】把 66 扩大 10 倍, 也就是用 66 乘 10 ; 所得的数为 10 个 66 , 原数为 1 个 66 , 因此用 10 个 66 减去 1 个 66 即可。\n\n【详解】 $66 \\times 10=660$\n\n10 个 66 减去 1 个 66 得 9 个 66 , 即所得的数比原来增加 9 个 66 。\n\n【点睛】熟练掌握两位数与整十数的乘法计算是解答此题的关键。", "solution": "null", "level": "三年级", "question": "把 66 扩大 10 倍得 $(\\quad)$, 所得的数比原来增加 $(\\quad)$ 个 66 。", "options": [], "subject": "算术", "analysis": "$\\quad 660 \\quad 9$\n\n【分析】把 66 扩大 10 倍, 也就是用 66 乘 10 ; 所得的数为 10 个 66 , 原数为 1 个 66 , 因此用 10 个 66 减去 1 个 66 即可。\n\n【详解】 $66 \\times 10=660$\n\n10 个 66 减去 1 个 66 得 9 个 66 , 即所得的数比原来增加 9 个 66 。\n\n【点睛】熟练掌握两位数与整十数的乘法计算是解答此题的关键。"} {"id": "8534", "image": [], "answer": "768\n\n【分析】因数 $\\times$ 因数=积, 因此用 32 减去 23 计算出少算了另一个因数的个数, 然后用 216 除以少算了另一个因数的个数计算出另一个因数, 最后用另一个因数乘 32 即可, 依此计算。\n\n【详解】 $32-23=9$\n\n$216 \\div 9=24$\n\n$24 \\times 32=768$\n\n【点睛】此图考查的是两位数与两位数的乘法计算, 先计算出另一个因数是解答此题的关键。", "solution": "null", "level": "三年级", "question": "明明在做一道乘法计算题的时候, 把一个因数 32 错看成了 23 , 这样得到的积比正确的积少了 216 ,正确的积是 $(\\quad)$ 。", "options": [], "subject": "算术", "analysis": "768\n\n【分析】因数 $\\times$ 因数=积, 因此用 32 减去 23 计算出少算了另一个因数的个数, 然后用 216 除以少算了另一个因数的个数计算出另一个因数, 最后用另一个因数乘 32 即可, 依此计算。\n\n【详解】 $32-23=9$\n\n$216 \\div 9=24$\n\n$24 \\times 32=768$\n\n【点睛】此图考查的是两位数与两位数的乘法计算, 先计算出另一个因数是解答此题的关键。"} {"id": "8535", "image": [], "answer": "能\n\n【分析】根据题意, 用每个班的人数乘班级个数, 求出全校的学生人数, 再与礼堂的座位数进行比较即可解题。\n\n【详解】 $28 \\times 50=1400($ 个)\n\n$1532>1400$\n\n所以, 能全部坐下。\n\n【点睛】熟练掌握整数乘法的计算方法, 是解答此题的关键。", "solution": "null", "level": "三年级", "question": "光明小学礼堂有 1532 个座位。全校 28 个班一起观看演出, 每个班大约有 50 人, ( )全部坐下。(填“能”或者“不能”)", "options": [], "subject": "算术", "analysis": "能\n\n【分析】根据题意, 用每个班的人数乘班级个数, 求出全校的学生人数, 再与礼堂的座位数进行比较即可解题。\n\n【详解】 $28 \\times 50=1400($ 个)\n\n$1532>1400$\n\n所以, 能全部坐下。\n\n【点睛】熟练掌握整数乘法的计算方法, 是解答此题的关键。"} {"id": "8553", "image": [], "answer": "【分析】(1) 0 乘任何数都得 0 , 任何数加上 0 都得原数, 据此求出 $0 \\times 43$ 和 $0+43$ 的得数, 再比较大小。\n\n(2) 根据两位数乘两位数的计算方法, 求出 $25 \\times 45$ 和 $27 \\times 43$ 的积, 再比较大小。\n\n(3) 根据三位数除以一位数的计算方法, 求出 $238 \\div 6$ 和 $165 \\div 4$ 的商, 再比较大小。\n\n【详解】 $0 \\times 43=0,0+43=43,0<43$, 则 $0 \\times 43<0+43$\n\n$25 \\times 45=1125,27 \\times 43=1161,1125<1161$, 则 $25 \\times 45<27 \\times 43$\n\n$238 \\div 6=39 \\ldots \\ldots .4,165 \\div 4=41 \\ldots \\ldots .1,39<41$, 则 $238 \\div 6<165 \\div 4$\n\n【点睛】比较两个算式的大小, 可以先求出算式的得数, 再进行比较。", "solution": "null", "level": "三年级", "question": "在括号里填上“>”“惐“=”。\n$0 \\times 43($\n) $0+43$\n$25 \\times 45($\n) $27 \\times 43$\n$238 \\div 6($\n) $165 \\div 4$", "options": [], "subject": "算术", "analysis": "【分析】(1) 0 乘任何数都得 0 , 任何数加上 0 都得原数, 据此求出 $0 \\times 43$ 和 $0+43$ 的得数, 再比较大小。\n\n(2) 根据两位数乘两位数的计算方法, 求出 $25 \\times 45$ 和 $27 \\times 43$ 的积, 再比较大小。\n\n(3) 根据三位数除以一位数的计算方法, 求出 $238 \\div 6$ 和 $165 \\div 4$ 的商, 再比较大小。\n\n【详解】 $0 \\times 43=0,0+43=43,0<43$, 则 $0 \\times 43<0+43$\n\n$25 \\times 45=1125,27 \\times 43=1161,1125<1161$, 则 $25 \\times 45<27 \\times 43$\n\n$238 \\div 6=39 \\ldots \\ldots .4,165 \\div 4=41 \\ldots \\ldots .1,39<41$, 则 $238 \\div 6<165 \\div 4$\n\n【点睛】比较两个算式的大小, 可以先求出算式的得数, 再进行比较。"} {"id": "8554", "image": [], "answer": "$\\quad 1360 \\quad 1624$\n\n【分析】求几个相同加数的和用乘法; 求一个数的几倍是多少用乘法。\n\n【详解】 $16 \\times 85=1360$\n\n$29 \\times 56=1624$\n\n【点睛】本题主要考查学生对整数乘法计算方法的掌握。", "solution": "null", "level": "三年级", "question": "85 个 16 的和是 (\n); 29 的 56 倍是 (", "options": [], "subject": "算术", "analysis": "$\\quad 1360 \\quad 1624$\n\n【分析】求几个相同加数的和用乘法; 求一个数的几倍是多少用乘法。\n\n【详解】 $16 \\times 85=1360$\n\n$29 \\times 56=1624$\n\n【点睛】本题主要考查学生对整数乘法计算方法的掌握。"} {"id": "8555", "image": ["1342.jpg", "1343.jpg", "1343.jpg"], "answer": "$5 ; 290$\n\n$10 ; 580$\n\n【分析】由题意可知, 5 与 58 相乘表示 5 个吉祥物一共需要 290 元, 十位上的 1 与 58 相乘表示 10 个吉祥物一共需要 580 元, 由此进行解答即可。\n\n\n\n【点睛】本题主要考查了整数乘法运算, 关键是熟练掌握计算法则正确进行计算。", "solution": "null", "level": "三年级", "question": "“扎比瓦卡”是本次世界杯足球赛的吉祥物, 每个 58 元, 李叔叔买了 15 个, 下面的坚式在计算一共付的钱。\n", "options": [], "subject": "算术", "analysis": "$5 ; 290$\n\n$10 ; 580$\n\n【分析】由题意可知, 5 与 58 相乘表示 5 个吉祥物一共需要 290 元, 十位上的 1 与 58 相乘表示 10 个吉祥物一共需要 580 元, 由此进行解答即可。\n\n\n\n【点睛】本题主要考查了整数乘法运算, 关键是熟练掌握计算法则正确进行计算。"} {"id": "8557", "image": [], "answer": "$12 \\quad 4$\n\n【分析】花店进来 100 朵玫瑰, 每 8 朵扎成一束, 就是求 100 里面有几个 8 , 用除法计算。\n\n【详解】 $100 \\div 8=12$ (束) $\\ldots . . .4$ (朵)\n\n【点睛】本题主要考查了除法的意义。", "solution": "null", "level": "三年级", "question": "花店进来 100 朵玫瑰, 每 8 朵扎成一束, 可以扎( )束, 还剩( 朵。", "options": [], "subject": "算术", "analysis": "$12 \\quad 4$\n\n【分析】花店进来 100 朵玫瑰, 每 8 朵扎成一束, 就是求 100 里面有几个 8 , 用除法计算。\n\n【详解】 $100 \\div 8=12$ (束) $\\ldots . . .4$ (朵)\n\n【点睛】本题主要考查了除法的意义。"} {"id": "8558", "image": [], "answer": "$\\equiv$ 两\n\n【分析】三位数除以一位数, 被除数的百位上的数字和除数比较大小, 如果比除数大或相等, 商就是三位数, 比除数小, 商就是两位数。\n\n【详解】 $409 \\div 4$ 商的最高位是百位, 商是三位数。 $409 \\div 7$ 商的最高位是十位, 商是两位数。\n\n【点睛】解答本题的关键是熟练掌握三位数除以一位数的计算法则。", "solution": "null", "level": "三年级", "question": "$409 \\div 4$ 的商是 $(\\quad)$ 位数, $409 \\div 7$ 的商是 $(\\quad)$ 位数。", "options": [], "subject": "算术", "analysis": "$\\equiv$ 两\n\n【分析】三位数除以一位数, 被除数的百位上的数字和除数比较大小, 如果比除数大或相等, 商就是三位数, 比除数小, 商就是两位数。\n\n【详解】 $409 \\div 4$ 商的最高位是百位, 商是三位数。 $409 \\div 7$ 商的最高位是十位, 商是两位数。\n\n【点睛】解答本题的关键是熟练掌握三位数除以一位数的计算法则。"} {"id": "8559", "image": [], "answer": "$6 \\quad 5$\n\n【分析】要使 $\\square 41 \\div 6$ 的商是三位数, 应大于等于除数 6 ;\n\n要使 $\\square 41 \\div 6$ 的商是两位数, 口应小于除数 6 ; 然后再进一步解答。\n\n【详解】要使 $\\square 41 \\div 6$ 的商是三位数, 呵以填 $6 、 7 、 8 、 9$, 最小填 $6 ;$要使 $\\square 41 \\div 6$ 的商是两位数, $\\square$ 可以填 $1 、 2 、 3 、 4 、 5$, 最大填 5 。\n\n【点睛】三位数除以一位数, 被除数最高位上的数大于除数, 商是三位数, 否则商是两位数。", "solution": "null", "level": "三年级", "question": "如果 $\\square 41 \\div 6$ 的商是三位数, 口里最小能填 $($ )。要使 $\\square 41 \\div 6$ 的商是两位数, 口里最大能填", "options": [], "subject": "算术", "analysis": "$6 \\quad 5$\n\n【分析】要使 $\\square 41 \\div 6$ 的商是三位数, 应大于等于除数 6 ;\n\n要使 $\\square 41 \\div 6$ 的商是两位数, 口应小于除数 6 ; 然后再进一步解答。\n\n【详解】要使 $\\square 41 \\div 6$ 的商是三位数, 呵以填 $6 、 7 、 8 、 9$, 最小填 $6 ;$要使 $\\square 41 \\div 6$ 的商是两位数, $\\square$ 可以填 $1 、 2 、 3 、 4 、 5$, 最大填 5 。\n\n【点睛】三位数除以一位数, 被除数最高位上的数大于除数, 商是三位数, 否则商是两位数。"} {"id": "8560", "image": [], "answer": "5\n\n【分析】第一空, 要使得商是两位数, 那么被除数的最高位上的数要小于 6 , 可以填 $1 \\sim 5$;第二空, 要使得商是三位数, 那么被除数的最高位要大于等于 6 , 可以填 $6 \\sim 9$;第三空, 要使得商中间有 0 , 被除数的最高位上被除尽了, 十位上的数要小于 4 , 可以填 $0 \\sim 3$;第四空, 要使得商的中间没有 0 , 被除数的最高位除尽了, 十位上的数要大于等于 4 , 可填 $4 \\sim 9$, 据此解答。\n\n【详解】 (5) $54 \\div 6$ (商是两位数); (6) $54 \\div 6$ (商是三位数);\n\n$8(3) 4 \\div 4$ (商的中间有 $0 ) ; 8(4) 4 \\div 4$ (商的中间没有 0 )\n\n【点睛】熟练掌握除法中各个部分之间的关系并灵活运用是解答本题的关键。", "solution": "null", "level": "三年级", "question": "在括号里填上合适的数字。\n\n) $54 \\div 6$ (商是两位数)\n\n) $54 \\div 6$ (商是三位数)\n$8(\\quad) 4 \\div 4$ (商的中间有 $0 ) \\quad 8(\\quad) 4 \\div 4$ (商的中间没有 0 )", "options": [], "subject": "算术", "analysis": "5\n\n【分析】第一空, 要使得商是两位数, 那么被除数的最高位上的数要小于 6 , 可以填 $1 \\sim 5$;第二空, 要使得商是三位数, 那么被除数的最高位要大于等于 6 , 可以填 $6 \\sim 9$;第三空, 要使得商中间有 0 , 被除数的最高位上被除尽了, 十位上的数要小于 4 , 可以填 $0 \\sim 3$;第四空, 要使得商的中间没有 0 , 被除数的最高位除尽了, 十位上的数要大于等于 4 , 可填 $4 \\sim 9$, 据此解答。\n\n【详解】 (5) $54 \\div 6$ (商是两位数); (6) $54 \\div 6$ (商是三位数);\n\n$8(3) 4 \\div 4$ (商的中间有 $0 ) ; 8(4) 4 \\div 4$ (商的中间没有 0 )\n\n【点睛】熟练掌握除法中各个部分之间的关系并灵活运用是解答本题的关键。"} {"id": "8561", "image": [], "answer": "$12 \\times 2 \\times 6 \\quad 12 \\times 6 \\div 3$\n【分析】每盒的支数 $x$ 每支的价钱 $=$ 每盒的价钱, 每盒的价钱 $x$ 卖的盒数 $=$ 一共卖的钱; 每盒的支数 $\\times$卖的盒数 $=$ 一共卖的支数, 一共卖的支数 - 卖的天数 $=$ 王阿姨平均每天卖的支数, 依此根据混合运算的计算顺序列出综合算式即可。\n\n【详解】 $12 \\times 2=24$ (元) ; $24 \\times 6=144$ (元), 即王阿姨 3 天卖了多少钱? 列综合算式是: $12 \\times 2 \\times 6$; $12 \\times 6=72$ (支), $72 \\div 3=24$ (支),即王阿姨平均每天卖了多少支? 列综合算式是 $12 \\times 6 \\div 3$ 。\n\n【点睛】此题考查的是经济问题的计算, 应先根据题意找到对应的关系后再列式。", "solution": "null", "level": "三年级", "question": "王阿姨 3 天卖了 6 盒中性笔,每盒 12 支,每支 2 元,王阿姨 3 天卖了多少钱?列综合算式是\n\n( ), 王阿姨平均每天卖了多少支?列综合算式是( )。", "options": [], "subject": "算术", "analysis": "$12 \\times 2 \\times 6 \\quad 12 \\times 6 \\div 3$\n【分析】每盒的支数 $x$ 每支的价钱 $=$ 每盒的价钱, 每盒的价钱 $x$ 卖的盒数 $=$ 一共卖的钱; 每盒的支数 $\\times$卖的盒数 $=$ 一共卖的支数, 一共卖的支数 - 卖的天数 $=$ 王阿姨平均每天卖的支数, 依此根据混合运算的计算顺序列出综合算式即可。\n\n【详解】 $12 \\times 2=24$ (元) ; $24 \\times 6=144$ (元), 即王阿姨 3 天卖了多少钱? 列综合算式是: $12 \\times 2 \\times 6$; $12 \\times 6=72$ (支), $72 \\div 3=24$ (支),即王阿姨平均每天卖了多少支? 列综合算式是 $12 \\times 6 \\div 3$ 。\n\n【点睛】此题考查的是经济问题的计算, 应先根据题意找到对应的关系后再列式。"} {"id": "8580", "image": ["1346.jpg", "1346.jpg"], "answer": "6 百 206\n\n【分析】三位数除以一位数笔算法则, 先看被除数的最高位上的数字, 如果最高位上的数字比除数小,就看被除数的前两位, 商写在十位上; 除到被除数的哪一位, 商就写在哪一位的上面。\n\n【详解】根据题目意思列出除法算式:\n\n$3 \\longdiv { 6 1 5 }$\n\n\n\n$\\frac{15}{0}$\n\n$615 \\div 3$, 可知: 6 个百除以 3 , 在百位上商 2 , 十位上 1 除以 3 不够商在十位上商 0 , 余下的 15 在个位上商 5 。\n\n【点睛】本题要求熟练掌握三位数除以一位数笔算除法。", "solution": "null", "level": "三年级", "question": "笔算 $615 \\div 3$, 想: ( )个百除以 3 , 在 $(\\quad)$ 位上商 $\\quad$ ), 十位上 1 除以 3 不够商在十位上商( ), 余下的 15 在个位上商 $(\\quad)$ 。", "options": [], "subject": "算术", "analysis": "6 百 206\n\n【分析】三位数除以一位数笔算法则, 先看被除数的最高位上的数字, 如果最高位上的数字比除数小,就看被除数的前两位, 商写在十位上; 除到被除数的哪一位, 商就写在哪一位的上面。\n\n【详解】根据题目意思列出除法算式:\n\n$3 \\longdiv { 6 1 5 }$\n\n\n\n$\\frac{15}{0}$\n\n$615 \\div 3$, 可知: 6 个百除以 3 , 在百位上商 2 , 十位上 1 除以 3 不够商在十位上商 0 , 余下的 15 在个位上商 5 。\n\n【点睛】本题要求熟练掌握三位数除以一位数笔算除法。"} {"id": "8581", "image": [], "answer": "$\\quad 130 \\quad 38$\n\n【分析】求一个数的几倍用乘法, 求一倍数用除法。\n\n【详解】 $26 \\times 5=130$\n\n$228 \\div 6=38$\n\n【点睛】本题考查求一倍数和几倍数的问题。", "solution": "null", "level": "三年级", "question": "6 的 5 倍是 $(\\quad)$, 一个数的 6 倍是 228 , 这个数是 $(\\quad)$ 。", "options": [], "subject": "算术", "analysis": "$\\quad 130 \\quad 38$\n\n【分析】求一个数的几倍用乘法, 求一倍数用除法。\n\n【详解】 $26 \\times 5=130$\n\n$228 \\div 6=38$\n\n【点睛】本题考查求一倍数和几倍数的问题。"} {"id": "8582", "image": [], "answer": "520\n\n【分析】根据题意, 可以计算出除数是多少, 再根据除法算式中各部分之间的关系, 即“被除数=除数 $\\times$ 商十余数”, 即可解答。\n\n【详解】除数: $8 \\times 8=64$\n\n被除数: $64 \\times 8+8=520$\n\n【点睛】灵活运用除法各部分之间的关系解决问题。", "solution": "null", "level": "三年级", "question": "一道除法算式中, 商和余数都是 8 , 除数正好是余数的 8 倍, 被除数是 $($ 。", "options": [], "subject": "算术", "analysis": "520\n\n【分析】根据题意, 可以计算出除数是多少, 再根据除法算式中各部分之间的关系, 即“被除数=除数 $\\times$ 商十余数”, 即可解答。\n\n【详解】除数: $8 \\times 8=64$\n\n被除数: $64 \\times 8+8=520$\n\n【点睛】灵活运用除法各部分之间的关系解决问题。"} {"id": "8584", "image": [], "answer": "275\n\n【分析】买了 11 袋面粉, 每袋 25 千克, 那么面粉的总质量就是 11 个 25 千克, 用 25 乘 11 即可求解。\n\n【详解】 $25 \\times 11=275$ (千克)\n\n【点睛】本题考查了整数乘法的意义: 求几个几是多少,用乘法求解;注意大米的数量是多余条件。", "solution": "null", "level": "三年级", "question": "学校食堂购回一批粮食,其中 11 袋面粉,每袋 25 千克; 20 袋大米,每袋 20 千克。购回面粉 (千克。", "options": [], "subject": "算术", "analysis": "275\n\n【分析】买了 11 袋面粉, 每袋 25 千克, 那么面粉的总质量就是 11 个 25 千克, 用 25 乘 11 即可求解。\n\n【详解】 $25 \\times 11=275$ (千克)\n\n【点睛】本题考查了整数乘法的意义: 求几个几是多少,用乘法求解;注意大米的数量是多余条件。"} {"id": "8585", "image": [], "answer": "$1 、 2 、 3$\n\n【分析】最小的四位数是 $1000,1000 \\div 24=41 \\ldots . .16$, 要使 $\\square 9 \\times 24$ 的积是三位数, 只要 $\\square 9 \\leq 41$, 然后再进一步解答。\n\n【详解】 $1000 \\div 24=41 \\ldots . .16$\n\n要使 $\\square \\times 24$ 的积是三位数, 只要 $\\square 9 \\leq 41$; 那么 $\\square$ 里面可以填 $1 、 2 、 3$ 。\n\n【点睛】两位数乘两位数, 要使积是三位数, 关键是求出未知因数的取值范围, 然后再进一步解答。", "solution": "null", "level": "三年级", "question": "$\\square 9 \\times 24$ 的积是三位数, 哩可以填 ( )。", "options": [], "subject": "算术", "analysis": "$1 、 2 、 3$\n\n【分析】最小的四位数是 $1000,1000 \\div 24=41 \\ldots . .16$, 要使 $\\square 9 \\times 24$ 的积是三位数, 只要 $\\square 9 \\leq 41$, 然后再进一步解答。\n\n【详解】 $1000 \\div 24=41 \\ldots . .16$\n\n要使 $\\square \\times 24$ 的积是三位数, 只要 $\\square 9 \\leq 41$; 那么 $\\square$ 里面可以填 $1 、 2 、 3$ 。\n\n【点睛】两位数乘两位数, 要使积是三位数, 关键是求出未知因数的取值范围, 然后再进一步解答。"} {"id": "8586", "image": [], "answer": "$5963 \\quad 160$\n\n【分析】根据“乘数×乘数=积”“被除数 $\\div$ 除数=商”分别算出积和商。\n\n【详解】 $67 \\times 89=5963$\n\n$960 \\div 6=160$\n\n则一个乘数是 67 , 另一个乘数是 89 , 积是 5963 ; 被除数是 960 , 除数是 6 , 商是 160 。\n\n【点睛】本题考查两位数乘两位数以及除数是一位数的除法计算方法, 属于基础题, 应熟练掌握。", "solution": "null", "level": "三年级", "question": "一个乘数是 67 , 另一个乘数是 89 , 积是 $($ ) ; 被除数是 960 , 除数是 6 , 商是 ( )", "options": [], "subject": "算术", "analysis": "$5963 \\quad 160$\n\n【分析】根据“乘数×乘数=积”“被除数 $\\div$ 除数=商”分别算出积和商。\n\n【详解】 $67 \\times 89=5963$\n\n$960 \\div 6=160$\n\n则一个乘数是 67 , 另一个乘数是 89 , 积是 5963 ; 被除数是 960 , 除数是 6 , 商是 160 。\n\n【点睛】本题考查两位数乘两位数以及除数是一位数的除法计算方法, 属于基础题, 应熟练掌握。"} {"id": "8587", "image": [], "answer": "20\n\n【分析】先将 1 吨换算成千克。再求出 1 吨里面有几个 50 千克, 也就是有几只这样的山羊重 1 吨。\n\n【详解】 1 吨 $=1000$ 千克\n\n$20 \\times 50=1000$ (千克)\n\n则 1000 千克里面有 20 个 50 千克, 也就是 20 只这样的山羊重 1 吨。\n\n【点睛】解决本题的关键是明确 1 吨 $=1000$ 千克, 再进行解答即可。", "solution": "null", "level": "三年级", "question": "如果一只山羊重 50 千克, 那么( )只这样的山羊重 1 吨。", "options": [], "subject": "算术", "analysis": "20\n\n【分析】先将 1 吨换算成千克。再求出 1 吨里面有几个 50 千克, 也就是有几只这样的山羊重 1 吨。\n\n【详解】 1 吨 $=1000$ 千克\n\n$20 \\times 50=1000$ (千克)\n\n则 1000 千克里面有 20 个 50 千克, 也就是 20 只这样的山羊重 1 吨。\n\n【点睛】解决本题的关键是明确 1 吨 $=1000$ 千克, 再进行解答即可。"} {"id": "8588", "image": ["1347.jpg", "1348.jpg", "1348.jpg"], "answer": "2310\n\n【分析】根据两位数乘两位数的计算方法可知, 用第二个因数十位上的数乘第一个因数个位上的数时,乘积的个位是 $8 。 3 \\times 6=18$, 则第一个因数个位上的数应是 6 。则这个算式为 $66 \\times 35$ 。\n【详解】 $66 \\times 35=2310$\n\n\n\n2030\n\n【点睛】两位数乘两位数时, 用第二个因数每一位上的数分别去乘第一个因数, 然后把各次乘得的数加起来。", "solution": "null", "level": "三年级", "question": "下图数字谜中的积是 ( $\\quad$ 。\n\n", "options": [], "subject": "算术", "analysis": "2310\n\n【分析】根据两位数乘两位数的计算方法可知, 用第二个因数十位上的数乘第一个因数个位上的数时,乘积的个位是 $8 。 3 \\times 6=18$, 则第一个因数个位上的数应是 6 。则这个算式为 $66 \\times 35$ 。\n【详解】 $66 \\times 35=2310$\n\n\n\n2030\n\n【点睛】两位数乘两位数时, 用第二个因数每一位上的数分别去乘第一个因数, 然后把各次乘得的数加起来。"} {"id": "8607", "image": [], "answer": "$\\quad 6 \\quad 6 \\quad 8 \\quad 48 \\quad 2 \\quad 4800$\n\n【分析】口算 $60 \\times 80$ 时, 先用 6 乘 8 , 再根据乘数中末尾 0 的个数填上即可。所以末尾要填 2 个 0 ,据此解答。\n\n【详解】 算 $60 \\times 80$ 时, 先算 $6 \\times 8=48$; 因乘数末尾中共有 2 个 0 , 所以再在末尾填 2 个 0 , 结果为: 4800 。\n\n【点睛】本题主要考查乘数末尾有 0 的乘法的口算方法, 熟练掌握这一方法是解答此题的关键。", "solution": "null", "level": "三年级", "question": "口算 $60 \\times 80$ 时, 可以先算 $\\quad) \\times(\\quad)=(\\quad)$, 再在末尾添( )个 0 , 得 $(\\quad) 。$", "options": [], "subject": "算术", "analysis": "$\\quad 6 \\quad 6 \\quad 8 \\quad 48 \\quad 2 \\quad 4800$\n\n【分析】口算 $60 \\times 80$ 时, 先用 6 乘 8 , 再根据乘数中末尾 0 的个数填上即可。所以末尾要填 2 个 0 ,据此解答。\n\n【详解】 算 $60 \\times 80$ 时, 先算 $6 \\times 8=48$; 因乘数末尾中共有 2 个 0 , 所以再在末尾填 2 个 0 , 结果为: 4800 。\n\n【点睛】本题主要考查乘数末尾有 0 的乘法的口算方法, 熟练掌握这一方法是解答此题的关键。"} {"id": "8608", "image": [], "answer": "396\n\n【分析】求 $25 \\times 40$ 的积的末尾有几个 0 , 依据整数乘法的计算方法求出算式的积即可解答;求几个 8 是 768 , 也就是求 768 里面有几个 8 , 用 768 除以 8 即可求解。\n\n【详解】 $25 \\times 40=1000$, 积的末尾有 3 个 0 , $768 \\div 8=96,96$ 个 8 是 768 。\n\n【点睛】解答本题的关键是依据整数乘法计算方法, 求出题干中算式的积; 以及求一个数里面有几个另一个数, 用除法求解。", "solution": "null", "level": "三年级", "question": "$25 \\times 40$ 的积的末尾有 $(\\quad)$ 个 $0,(\\quad)$ 个 8 是 768 。", "options": [], "subject": "算术", "analysis": "396\n\n【分析】求 $25 \\times 40$ 的积的末尾有几个 0 , 依据整数乘法的计算方法求出算式的积即可解答;求几个 8 是 768 , 也就是求 768 里面有几个 8 , 用 768 除以 8 即可求解。\n\n【详解】 $25 \\times 40=1000$, 积的末尾有 3 个 0 , $768 \\div 8=96,96$ 个 8 是 768 。\n\n【点睛】解答本题的关键是依据整数乘法计算方法, 求出题干中算式的积; 以及求一个数里面有几个另一个数, 用除法求解。"} {"id": "8609", "image": [], "answer": "$<<<<<<$\n\n【分析】先根据整数加法、乘除法的计算方法算出各个算式的得数, 再按照整数比较大小的方法进行解答。\n\n【详解】 $114 \\div 3=38,38<40$, 所以 $114 \\div 3<40 ;$\n$6 \\times 68=408,408<435$, 所以 $6 \\times 68<435$;\n\n$505 \\div 5=101,550 \\div 5=110,101<110$, 所以 $505 \\div 5<550 \\div 5$;\n\n$68 \\times 10=680,34 \\times 20=680,680=680$, 所以 $68 \\times 10=34 \\times 20$;\n\n$7 \\times 80=560,560<563$, 所以 $7 \\times 80<563 ;$\n\n$32+30=62,32 \\times 3=96,62<96$, 所以 $32+30<32 \\times 3$ 。\n\n【点睛】掌握整数四则计算的计算法则是解决问题的关键。", "solution": "null", "level": "三年级", "question": "在括号里面填上“>”“””“=”\n\n$114 \\div 3(\\quad) 40 \\quad 6 \\times 68(\\quad) 435 \\quad 505 \\div 5(\\quad) 550 \\div 5$\n\n$68 \\times 10(\\quad) 34 \\times 20 \\quad 7 \\times 80(\\quad) 563 \\quad 32+30(\\quad) 32 \\times 3$", "options": [], "subject": "算术", "analysis": "$<<<<<<$\n\n【分析】先根据整数加法、乘除法的计算方法算出各个算式的得数, 再按照整数比较大小的方法进行解答。\n\n【详解】 $114 \\div 3=38,38<40$, 所以 $114 \\div 3<40 ;$\n$6 \\times 68=408,408<435$, 所以 $6 \\times 68<435$;\n\n$505 \\div 5=101,550 \\div 5=110,101<110$, 所以 $505 \\div 5<550 \\div 5$;\n\n$68 \\times 10=680,34 \\times 20=680,680=680$, 所以 $68 \\times 10=34 \\times 20$;\n\n$7 \\times 80=560,560<563$, 所以 $7 \\times 80<563 ;$\n\n$32+30=62,32 \\times 3=96,62<96$, 所以 $32+30<32 \\times 3$ 。\n\n【点睛】掌握整数四则计算的计算法则是解决问题的关键。"} {"id": "8611", "image": ["1351.jpg", "1352.jpg", "1352.jpg"], "answer": "$50 \\quad 800$\n【分析】用每套明信片的价钱乘购买套数, 求出一共卖的钱数。计算 $16 \\times 56$ 时, 用第二个因数十位上的 5 乘第一个因数 16 , 表示 5 个十乘 16 , 得到 80 个十, 即表示 50 套明信片卖了 800 元。据此解答。\n\n\n\n卖了 50 套共 800 元。\n\n【点睛】本题考查经济问题和两位数乘两位数的计算, 关键是明确积是由哪两个数相乘得到的, 以及这两个数表示的意义,进而明确积表示的意义。", "solution": "null", "level": "三年级", "question": "商店今天卖出 56 套明信片, 每套 16 元, 一共卖了多少元? 请根据坚式填上合适的数。\n\n\n\n卖了( $\\quad$ 套共 $(\\quad)$ 元", "options": [], "subject": "算术", "analysis": "$50 \\quad 800$\n【分析】用每套明信片的价钱乘购买套数, 求出一共卖的钱数。计算 $16 \\times 56$ 时, 用第二个因数十位上的 5 乘第一个因数 16 , 表示 5 个十乘 16 , 得到 80 个十, 即表示 50 套明信片卖了 800 元。据此解答。\n\n\n\n卖了 50 套共 800 元。\n\n【点睛】本题考查经济问题和两位数乘两位数的计算, 关键是明确积是由哪两个数相乘得到的, 以及这两个数表示的意义,进而明确积表示的意义。"} {"id": "8612", "image": [], "answer": "315\n\n【分析】用每天练字页数乘每页字数, 求出每天练字字数。再乘练字天数, 求出练字总字数。\n\n【详解】 $3 \\times 5 \\times 21$\n\n$=15 \\times 21$\n\n$=315($ 个)\n\n则小红 21 天练了 315 个字。\n\n【点睛】本题考查两步连乘解决实际问题, 也可以先求出练字总页数, 再求出练字总字数, 列式为 $3 \\times 21 \\times 5$ 。", "solution": "null", "level": "三年级", "question": "小红在暑假练习书法, 每天练 3 页, 每页 5 个字, 小红 21 天练了 ( ) 个字。", "options": [], "subject": "算术", "analysis": "315\n\n【分析】用每天练字页数乘每页字数, 求出每天练字字数。再乘练字天数, 求出练字总字数。\n\n【详解】 $3 \\times 5 \\times 21$\n\n$=15 \\times 21$\n\n$=315($ 个)\n\n则小红 21 天练了 315 个字。\n\n【点睛】本题考查两步连乘解决实际问题, 也可以先求出练字总页数, 再求出练字总字数, 列式为 $3 \\times 21 \\times 5$ 。"} {"id": "8613", "image": [], "answer": "1440\n\n【分析】绕地球一圈需要 90 分钟,绕地球 16 圈,则需要 16 个 90 分钟,根据乘法的意义可以看成 $90 \\times 16$,再根据两位数乘两位数的乘法计算可以得到答案。\n\n【详解】 $90 \\times 16=1440$\n\n【点睛】此题的关键点是灵活运用乘法的意义进行乘法计算, 求几个相同加数的和的简便运算就是用这个加数乘加数的个数。", "solution": "null", "level": "三年级", "question": "神舟十三号载人飞船绕地球一圈需要 90 分钟, 绕地球 16 圈, 需要 ( )分钟。", "options": [], "subject": "算术", "analysis": "1440\n\n【分析】绕地球一圈需要 90 分钟,绕地球 16 圈,则需要 16 个 90 分钟,根据乘法的意义可以看成 $90 \\times 16$,再根据两位数乘两位数的乘法计算可以得到答案。\n\n【详解】 $90 \\times 16=1440$\n\n【点睛】此题的关键点是灵活运用乘法的意义进行乘法计算, 求几个相同加数的和的简便运算就是用这个加数乘加数的个数。"} {"id": "8614", "image": [], "answer": "$\\quad 15 \\quad 750$\n\n【分析】根据题意可知, 用到书店的时刻减去从家出发的时刻即可; 再用王芳从家到书店走的时间长乘她每分钟走的路程即可。\n\n【详解】 9 时 40 分 -9 时 25 分 $=15$ (分钟)\n\n$15 \\times 50=750$ (米)\n\n【点睛】此题考查的是普通的行程问题, 熟练掌握经过时间的计算是解答此题的关键。", "solution": "null", "level": "三年级", "question": "王芳步行去书店, 9: 25 从家出发, 9: 40 到书店, 王芳从家到书店走了 ()分钟; 她每分钟大约走 50 米, 王芳家到书店大约有 $(\\quad)$ 米。", "options": [], "subject": "算术", "analysis": "$\\quad 15 \\quad 750$\n\n【分析】根据题意可知, 用到书店的时刻减去从家出发的时刻即可; 再用王芳从家到书店走的时间长乘她每分钟走的路程即可。\n\n【详解】 9 时 40 分 -9 时 25 分 $=15$ (分钟)\n\n$15 \\times 50=750$ (米)\n\n【点睛】此题考查的是普通的行程问题, 熟练掌握经过时间的计算是解答此题的关键。"} {"id": "8615", "image": ["1353.jpg", "1354.jpg", "1355.jpg", "1355.jpg"], "answer": "见详解\n\n【分析】这是一个两位数乘两位数、积为三位数的乘法坚式, 首先第二个因数的十位数乘 8 的积的十\n位数上是 6 , 在乘法口诀中, $2 \\times 8=16 ; 7 \\times 8=56$ 这两种情况积的末尾是 6 , 所以第二个因数的十位上是 2 或 7 , 当十位上是 2 时, 积是三位数, 当第二个因数的十位上是 7 时, 积是四位数, 不符合条件,排除。然后依次进行计算即可。\n\n\n\n【点睛】解答这类题目, 要分析算式的特点, 运用加、减、乘、除的运算法则来安排每一个数。一个算式中填几个数时, 要选好先填什么, 再填什么, 选准“突破口”, 其他就好填了。", "solution": "null", "level": "三年级", "question": "小聪邀请你玩填数游戏, 现在小聪已经完成了个位数的填写, 余下的请你来完成, 在图 2 的口里填上合适的数字。\n\n\n\n图 1\n\n\n\n图 2", "options": [], "subject": "算术", "analysis": "见详解\n\n【分析】这是一个两位数乘两位数、积为三位数的乘法坚式, 首先第二个因数的十位数乘 8 的积的十\n位数上是 6 , 在乘法口诀中, $2 \\times 8=16 ; 7 \\times 8=56$ 这两种情况积的末尾是 6 , 所以第二个因数的十位上是 2 或 7 , 当十位上是 2 时, 积是三位数, 当第二个因数的十位上是 7 时, 积是四位数, 不符合条件,排除。然后依次进行计算即可。\n\n\n\n【点睛】解答这类题目, 要分析算式的特点, 运用加、减、乘、除的运算法则来安排每一个数。一个算式中填几个数时, 要选好先填什么, 再填什么, 选准“突破口”, 其他就好填了。"} {"id": "8633", "image": [], "answer": "$\\quad 20 \\quad 60 \\quad 1200$\n\n【分析】根据整数乘法的估算方法: 利用“四舍五入法”, 把两个乘数分别看作与它接近的整十数、整百数. ;然后进行口算即可。\n\n【详解】 $21 \\times 59 \\approx 20 \\times 60=1200$\n\n估算 $21 \\times 59$, 可以把 21 看作 20 , 把 59 看作 60 , 估算结果是 1200 。\n\n【点睛】此题考查的目的是理解掌握整数乘法的估算方法。", "solution": "null", "level": "三年级", "question": "估算 $21 \\times 59$, 可以把 21 看作 (\n) , 把 59 看作 (\n) , 估算结果是 (", "options": [], "subject": "算术", "analysis": "$\\quad 20 \\quad 60 \\quad 1200$\n\n【分析】根据整数乘法的估算方法: 利用“四舍五入法”, 把两个乘数分别看作与它接近的整十数、整百数. ;然后进行口算即可。\n\n【详解】 $21 \\times 59 \\approx 20 \\times 60=1200$\n\n估算 $21 \\times 59$, 可以把 21 看作 20 , 把 59 看作 60 , 估算结果是 1200 。\n\n【点睛】此题考查的目的是理解掌握整数乘法的估算方法。"} {"id": "8634", "image": [], "answer": "$\\quad$ 百\n\n【分析】根据整数乘法的计算方法, 先算出 $36 \\times 23$ 的积是多少, 再进一步解答即可。\n\n【详解】 $36 \\times 23=828$, 所以 $36 \\times 23$ 的积是三位数, 最高位是百位。\n\n【点睛】此题考查了整数乘法的计算方法。", "solution": "null", "level": "三年级", "question": "$36 \\times 23$ 的积是 (\n\n) 位数, 积的最高位在(\n\n)位上。", "options": [], "subject": "算术", "analysis": "$\\quad$ 百\n\n【分析】根据整数乘法的计算方法, 先算出 $36 \\times 23$ 的积是多少, 再进一步解答即可。\n\n【详解】 $36 \\times 23=828$, 所以 $36 \\times 23$ 的积是三位数, 最高位是百位。\n\n【点睛】此题考查了整数乘法的计算方法。"} {"id": "8635", "image": [], "answer": "14000\n\n【分析】先把 14 亿改写成以 1 为单位的数, 再用乘法计算出 14 亿人节约米饭的克数, 再将克换算成吨。\n\n【详解】 14 亿人 $=14000000000$ 人\n\n$1400000000 \\times 10=14000000000(\\mathrm{~g})$\n\n$14000000000 \\mathrm{~g}=14000 \\mathrm{t}$\n\n【点睛】解决本题的关键是明确 1 千克 $=1000$ 克, 1 吨 $=1000$ 千克。", "solution": "null", "level": "三年级", "question": "点滴事小, 节约为大。我国约有 14 亿人, 如果每人每天节约 $10 \\mathrm{~g}$ 米饭, 那么全国每天可节约\n\n$(\\quad)$ 米饭。", "options": [], "subject": "算术", "analysis": "14000\n\n【分析】先把 14 亿改写成以 1 为单位的数, 再用乘法计算出 14 亿人节约米饭的克数, 再将克换算成吨。\n\n【详解】 14 亿人 $=14000000000$ 人\n\n$1400000000 \\times 10=14000000000(\\mathrm{~g})$\n\n$14000000000 \\mathrm{~g}=14000 \\mathrm{t}$\n\n【点睛】解决本题的关键是明确 1 千克 $=1000$ 克, 1 吨 $=1000$ 千克。"} {"id": "9169", "image": [], "answer": "解$\\times$\n\n【分析】其中一个因数末尾有 0 , 积的末尾也一定有 0 ; 两个因数的末尾都没有 0 , 积的末尾可能有 0 ;据此解答。\n\n【详解】根据分析举例: $256 \\times 35=8960$, 这两个因数的末尾都没有 0 , 但是积的末尾有 0 。故答案为: $\\times$\n\n【点睛】本题主要考查的是积的末尾的 0 和因数的关系。", "solution": "null", "level": "三年级", "question": "两个因数的末尾都没有 0 , 积的末尾也一定没有 0 。( $)$", "options": [], "subject": "算术", "analysis": "解$\\times$\n\n【分析】其中一个因数末尾有 0 , 积的末尾也一定有 0 ; 两个因数的末尾都没有 0 , 积的末尾可能有 0 ;据此解答。\n\n【详解】根据分析举例: $256 \\times 35=8960$, 这两个因数的末尾都没有 0 , 但是积的末尾有 0 。故答案为: $\\times$\n\n【点睛】本题主要考查的是积的末尾的 0 和因数的关系。"} {"id": "9170", "image": [], "answer": "解$\\times$\n\n【分析】 23 乘 51 , 可以把 51 写成 50 加 1 , 然后利用乘法分配律, 就可以得到 $23 \\times 50+23$, 据此解答。\n\n【详解】 $23 \\times(50+1)$\n\n$=23 \\times 50+23$\n\n$23 \\times 50$ 的结果不等于 $23 \\times 50+23$ 的结果。\n\n故答案为: $\\times$\n\n【点睛】此题考查乘法分配律, 熟练掌握并灵活运用。", "solution": "null", "level": "三年级", "question": "$23 \\times 50$ 的结果等于 $23 \\times 50+23$ 的结果。", "options": [], "subject": "算术", "analysis": "解$\\times$\n\n【分析】 23 乘 51 , 可以把 51 写成 50 加 1 , 然后利用乘法分配律, 就可以得到 $23 \\times 50+23$, 据此解答。\n\n【详解】 $23 \\times(50+1)$\n\n$=23 \\times 50+23$\n\n$23 \\times 50$ 的结果不等于 $23 \\times 50+23$ 的结果。\n\n故答案为: $\\times$\n\n【点睛】此题考查乘法分配律, 熟练掌握并灵活运用。"} {"id": "9172", "image": [], "answer": "解$\\times$\n\n【分析】根据题意, 美术组的人数是书法组的 2 倍, 也就是多少的 2 倍是 24 , 用除法计算, 据此解答。\n\n【详解】 $24 \\div 2=12$ (人)\n\n美术组有 24 人, 是书法组人数的 2 倍, 书法组有 12 人。\n\n故答案为: $\\times$\n\n【点睛】熟练掌握对倍的认识并灵活运用是解答本题的关键。", "solution": "null", "level": "三年级", "question": "美术组有 24 人,是书法组人数的 2 倍, 书法组有 48 人。( )", "options": [], "subject": "算术", "analysis": "解$\\times$\n\n【分析】根据题意, 美术组的人数是书法组的 2 倍, 也就是多少的 2 倍是 24 , 用除法计算, 据此解答。\n\n【详解】 $24 \\div 2=12$ (人)\n\n美术组有 24 人, 是书法组人数的 2 倍, 书法组有 12 人。\n\n故答案为: $\\times$\n\n【点睛】熟练掌握对倍的认识并灵活运用是解答本题的关键。"} {"id": "9173", "image": [], "answer": "解$\\sqrt{ }$\n\n【分析】在乘法算式里, 两个乘数都不为 0 时, 一个乘数乘 10 , 另一个乘数除以 10 , 积的大小不变,\n依此判断。\n\n【详解】 $27 \\times 10=270 ; 30 \\div 10=3$, 即 $27 \\times 30=270 \\times 3$ 。\n\n故答案为: $\\sqrt{ }$\n\n【点睛】熟练掌握积的变化规律是解答此题的关键。", "solution": "null", "level": "三年级", "question": "$27 \\times 30$ 和 $270 \\times 3$ 的结果是相同的。( )", "options": [], "subject": "算术", "analysis": "解$\\sqrt{ }$\n\n【分析】在乘法算式里, 两个乘数都不为 0 时, 一个乘数乘 10 , 另一个乘数除以 10 , 积的大小不变,\n依此判断。\n\n【详解】 $27 \\times 10=270 ; 30 \\div 10=3$, 即 $27 \\times 30=270 \\times 3$ 。\n\n故答案为: $\\sqrt{ }$\n\n【点睛】熟练掌握积的变化规律是解答此题的关键。"} {"id": "8637", "image": [], "answer": "640\n\n【分析】用 32 减去 23 , 计算出第二个因数减少了多少, 再用计算结果减少的部分除以第二个因数减少的部分, 可以计算出第一个因数是多少, 再把两个正确的因数相乘, 计算出这道题的正确结果是多少, 据此解答。\n\n【详解】 $180 \\div(32-23) \\times 32$\n\n$=180 \\div 9 \\times 32$\n\n$=20 \\times 32$\n\n$=640$\n\n小精灵在计算一道两位数乘两位数的题时, 把第二个乘数 32 错看成了 23 , 这样计算出的结果比原来少了 180 。这道题的正确结果是 $(640)$ 。\n\n【点睛】本题解题关键是先用减法计算出第二个因数减少了多少, 再用除法计算出第一个因数是多少,最后算出正确的结果。", "solution": "null", "level": "三年级", "question": "小精灵在计算一道两位数乘两位数的题时, 把第二个乘数 32 错看成了 23 , 这样计算出的结果比原来少了 180 。这道题的正确结果是 ( )。", "options": [], "subject": "算术", "analysis": "640\n\n【分析】用 32 减去 23 , 计算出第二个因数减少了多少, 再用计算结果减少的部分除以第二个因数减少的部分, 可以计算出第一个因数是多少, 再把两个正确的因数相乘, 计算出这道题的正确结果是多少, 据此解答。\n\n【详解】 $180 \\div(32-23) \\times 32$\n\n$=180 \\div 9 \\times 32$\n\n$=20 \\times 32$\n\n$=640$\n\n小精灵在计算一道两位数乘两位数的题时, 把第二个乘数 32 错看成了 23 , 这样计算出的结果比原来少了 180 。这道题的正确结果是 $(640)$ 。\n\n【点睛】本题解题关键是先用减法计算出第二个因数减少了多少, 再用除法计算出第一个因数是多少,最后算出正确的结果。"} {"id": "8638", "image": [], "answer": "$>=><$\n\n【分析】利用两位数乘两位数的计算方法和整数四则混合运算的计算方法, 分别将括号左右两边的算式计算出结果, 即可比较。据此解答。\n\n【详解】 $24 \\times 50=1200,25 \\times 40=1000,1200>1000$, 所以 $24 \\times 50>25 \\times 40$;\n\n$30 \\times 21=630,30 \\times 20+30=600+30=630,630=630$, 所以 $30 \\times 21=30 \\times 20+30$;\n\n$60-40 \\div 5=60-8=52,(60-40) \\div 5=20 \\div 5=4,52>4$, 所以 $60-40 \\div 5>(60-40) \\div 5$;\n\n$60 \\times 2+30=120+30=150,60 \\times(2+3)=60 \\times 5=300,150<300$, 所以 $60 \\times 2+30<60 \\times(2+3)$ 。\n\n【点睛】本题考查了算式之间的大小比较。熟练掌握两位数乘两位数、除数是一位数的除法的计算方法是解决此题的关键。", "solution": "null", "level": "三年级", "question": "在括号里填“>”“<”或“=”。\n\n$24 \\times 50(\\quad) 25 \\times 40 \\quad 30 \\times 21(\\quad) 30 \\times 20+30$\n\n$60-40 \\div 5(\\quad)(60-40) \\div 5 \\quad 60 \\times 2+30(\\quad) 60 \\times(2+3)$", "options": [], "subject": "算术", "analysis": "$>=><$\n\n【分析】利用两位数乘两位数的计算方法和整数四则混合运算的计算方法, 分别将括号左右两边的算式计算出结果, 即可比较。据此解答。\n\n【详解】 $24 \\times 50=1200,25 \\times 40=1000,1200>1000$, 所以 $24 \\times 50>25 \\times 40$;\n\n$30 \\times 21=630,30 \\times 20+30=600+30=630,630=630$, 所以 $30 \\times 21=30 \\times 20+30$;\n\n$60-40 \\div 5=60-8=52,(60-40) \\div 5=20 \\div 5=4,52>4$, 所以 $60-40 \\div 5>(60-40) \\div 5$;\n\n$60 \\times 2+30=120+30=150,60 \\times(2+3)=60 \\times 5=300,150<300$, 所以 $60 \\times 2+30<60 \\times(2+3)$ 。\n\n【点睛】本题考查了算式之间的大小比较。熟练掌握两位数乘两位数、除数是一位数的除法的计算方法是解决此题的关键。"} {"id": "8639", "image": [], "answer": "$24 \\times 51 \\# \\# 51 \\times 24$\n\n【分析】两个乘法算式中都有 51, 说明其中一个因数是 51; 51 分别与 20 和 4 相乘, 说明另一个因数十位上是 2 , 个位上是 4 , 据此解答。\n\n【详解】 $24 \\times 51=1224$\n\n笑笑计算了一个两位数乘两位数的算式。她的计算过程是: $20 \\times 51=1020,4 \\times 51=204$, $1020+204=1224$ ,笑笑原来计算的乘法算式是( $24 \\times 51 ) 。$\n\n【点睛】本题主要考查了两位数乘两位数的运算, 要正确理解算理。", "solution": "null", "level": "三年级", "question": "笑笑计算了一个两位数乘两位数的算式。她的计算过程是: $20 \\times 51=1020,4 \\times 51=204$, $1020+204=1224$, 笑笑原来计算的乘法算式是 $($ 。", "options": [], "subject": "算术", "analysis": "$24 \\times 51 \\# \\# 51 \\times 24$\n\n【分析】两个乘法算式中都有 51, 说明其中一个因数是 51; 51 分别与 20 和 4 相乘, 说明另一个因数十位上是 2 , 个位上是 4 , 据此解答。\n\n【详解】 $24 \\times 51=1224$\n\n笑笑计算了一个两位数乘两位数的算式。她的计算过程是: $20 \\times 51=1020,4 \\times 51=204$, $1020+204=1224$ ,笑笑原来计算的乘法算式是( $24 \\times 51 ) 。$\n\n【点睛】本题主要考查了两位数乘两位数的运算, 要正确理解算理。"} {"id": "8640", "image": [], "answer": "1800\n\n【分析】根据题意: 一套有 4 本, 每本 25 元, 首先用乘法求出每一套多少钱, 再求出 18 套多少钱即可。\n\n【详解】 $4 \\times 25 \\times 18$\n\n$=100 \\times 18$\n\n$=1800 ($ 元),所以一共要花 1800 元\n\n【点睛】解答此题的关键是掌握总价、数量和单价三者之间的关系, 总价 $\\div$ 数量 $=$ 单价, 单价 $\\times$ 数量 $=$ 总价。总价 $\\div$ 单价 $=$ 数量。", "solution": "null", "level": "三年级", "question": "一套《百科全书》有 4 本书, 每本书 25 元。王老师准备买 18 套《百科全书》奖励给优秀学生,一共要花 $(\\quad)$ 元。", "options": [], "subject": "算术", "analysis": "1800\n\n【分析】根据题意: 一套有 4 本, 每本 25 元, 首先用乘法求出每一套多少钱, 再求出 18 套多少钱即可。\n\n【详解】 $4 \\times 25 \\times 18$\n\n$=100 \\times 18$\n\n$=1800 ($ 元),所以一共要花 1800 元\n\n【点睛】解答此题的关键是掌握总价、数量和单价三者之间的关系, 总价 $\\div$ 数量 $=$ 单价, 单价 $\\times$ 数量 $=$ 总价。总价 $\\div$ 单价 $=$ 数量。"} {"id": "8641", "image": ["1360.jpg", "1361.jpg", "1362.jpg", "1363.jpg", "1361.jpg", "1362.jpg", "1363.jpg"], "answer": "4\n\n$20 ; 900$\n\n24\n\n【分析】两位数乘两位数: 先用第 2 个因数个位上的数去乘第 1 个因数的每一位, 得数的末位和第 2 个因数的个位对齐; 再用第 2 个因数十位上的数去乘第 1 个因数的每一位, 得数的末位和第 2 个因数的十位对齐; 然后把两次乘得的数加起来。\n\n【详解】如图所示:\n\n```\n\n45\n\n* 24\n\n```\n\n```\n\n```\n\n```\n\n```\n\n\n【点睛】此题主要考查了两位数乘两位数乘法, 明确乘法各步的意义是解答本题的关键。", "solution": "null", "level": "三年级", "question": "每个篮球的价钱是 45 元, 李老师为同学们买了 24 个篮球。\n\n", "options": [], "subject": "算术", "analysis": "4\n\n$20 ; 900$\n\n24\n\n【分析】两位数乘两位数: 先用第 2 个因数个位上的数去乘第 1 个因数的每一位, 得数的末位和第 2 个因数的个位对齐; 再用第 2 个因数十位上的数去乘第 1 个因数的每一位, 得数的末位和第 2 个因数的十位对齐; 然后把两次乘得的数加起来。\n\n【详解】如图所示:\n\n```\n\n45\n\n* 24\n\n```\n\n```\n\n```\n\n```\n\n```\n\n\n【点睛】此题主要考查了两位数乘两位数乘法, 明确乘法各步的意义是解答本题的关键。"} {"id": "8660", "image": [], "answer": "$720 \\quad 50$\n\n【分析】求 24 的 30 倍是多少, 用 24 乘 30 列式解答; 求 450 是 9 的多少倍, 用 450 除以 9 列式解答。\n\n【详解】 $24 \\times 30=720$\n\n$450 \\div 9=50$\n\n24 的 30 倍是 720,450 是 9 的 50 倍。\n\n【点睛】本题主要考查了求一个数的几倍是多少, 用乘法解答; 求一个数是另一个数的几倍, 用除法解答。", "solution": "null", "level": "三年级", "question": "24 的 30 倍是 ( ), 450 是 9 的 ( )倍。", "options": [], "subject": "算术", "analysis": "$720 \\quad 50$\n\n【分析】求 24 的 30 倍是多少, 用 24 乘 30 列式解答; 求 450 是 9 的多少倍, 用 450 除以 9 列式解答。\n\n【详解】 $24 \\times 30=720$\n\n$450 \\div 9=50$\n\n24 的 30 倍是 720,450 是 9 的 50 倍。\n\n【点睛】本题主要考查了求一个数的几倍是多少, 用乘法解答; 求一个数是另一个数的几倍, 用除法解答。"} {"id": "8661", "image": [], "answer": "0\n\n【详解】 0 乘任何数都得 0 。如 $0 \\times 0=0,0 \\times 3=0,0 \\times 158=0,0 \\times 1000=0$ 。", "solution": "null", "level": "三年级", "question": "0 乘任何数都得 ( $\\quad$ 。", "options": [], "subject": "算术", "analysis": "0\n\n【详解】 0 乘任何数都得 0 。如 $0 \\times 0=0,0 \\times 3=0,0 \\times 158=0,0 \\times 1000=0$ 。"} {"id": "8662", "image": [], "answer": "3\n\n【分析】最大的三位数是 999 , 可从 5 开始, 用 56、46、36 分别乘 22, 结合积的具体情况作出判断。\n\n【详解】 $56 \\times 22=1232$, 积是四位数;\n\n$46 \\times 22=1012$, 积是四位数;\n\n$36 \\times 22=792$, 积是三位数;\n要使 $\\square 6 \\times 22$ 的积是三位数, $\\square$ 里最大填 $(3) 。$\n\n【点睛】此题需要学生熟练掌握两位数乘两位数的计算并灵活运用。", "solution": "null", "level": "三年级", "question": "要使 $\\square 6 \\times 22$ 的积是三位数, 口里最大填 $(\\quad)$ 。", "options": [], "subject": "算术", "analysis": "3\n\n【分析】最大的三位数是 999 , 可从 5 开始, 用 56、46、36 分别乘 22, 结合积的具体情况作出判断。\n\n【详解】 $56 \\times 22=1232$, 积是四位数;\n\n$46 \\times 22=1012$, 积是四位数;\n\n$36 \\times 22=792$, 积是三位数;\n要使 $\\square 6 \\times 22$ 的积是三位数, $\\square$ 里最大填 $(3) 。$\n\n【点睛】此题需要学生熟练掌握两位数乘两位数的计算并灵活运用。"} {"id": "8664", "image": [], "answer": "30\n\n【分析】先计算出 $30 \\times 51$ 的积, 然后再用 $30 \\times 51$ 的积减 1500 即可。\n\n【详解】 $30 \\times 51=1530,1530-1500=30$, 即 $30 \\times 51$ 的积比 1500 多 30 。\n\n【点睛】熟练掌握两位数与两位数的乘法计算是解答此题的关键。", "solution": "null", "level": "三年级", "question": "$30 \\times 51$ 的积比 1500 多 ( )。", "options": [], "subject": "算术", "analysis": "30\n\n【分析】先计算出 $30 \\times 51$ 的积, 然后再用 $30 \\times 51$ 的积减 1500 即可。\n\n【详解】 $30 \\times 51=1530,1530-1500=30$, 即 $30 \\times 51$ 的积比 1500 多 30 。\n\n【点睛】熟练掌握两位数与两位数的乘法计算是解答此题的关键。"} {"id": "8665", "image": [], "answer": "19\n\n【分析】 $20 \\times 20=400$, 要使 $\\square \\times 21<400,21>20$, 则 $\\square$ 里的数要小于 20 , 因此分别计算出 $18 \\times 21 、 19 \\times 21$的积, 然后再根据计算出的积, 确定出口里的最大数即可。\n\n【详解】 $18 \\times 21=378,19 \\times 21=399$, 即口里最大能填 19 。\n\n【点睛】熟练掌握两位数与两位数的乘法计算是解答此题的关键。", "solution": "null", "level": "三年级", "question": "$\\square \\times 21<400$, $\\square$ 里最大能填 ( )。", "options": [], "subject": "算术", "analysis": "19\n\n【分析】 $20 \\times 20=400$, 要使 $\\square \\times 21<400,21>20$, 则 $\\square$ 里的数要小于 20 , 因此分别计算出 $18 \\times 21 、 19 \\times 21$的积, 然后再根据计算出的积, 确定出口里的最大数即可。\n\n【详解】 $18 \\times 21=378,19 \\times 21=399$, 即口里最大能填 19 。\n\n【点睛】熟练掌握两位数与两位数的乘法计算是解答此题的关键。"} {"id": "8666", "image": [], "answer": "两 三\n\n【分析】三位数除以一位数, 如果被除数百位上的数大于等于除数, 则商是三位数, 否则, 商是两位数; 最大两位数是 99 , 最小两位数是 10 , 求出两数相乘的积即可知道积的位数; 据此即可解答。\n\n【详解】 $617 \\div 7$ 中, $6<7$, 商是两位数。 $99 \\times 10=990$, 所以最大的两位数乘最小的两位数, 积是三位数。\n\n【点睛】本题主要考查学生对整数乘除法知识的掌握和灵活运用。", "solution": "null", "level": "三年级", "question": "$617 \\div 7$ 的商是 $(\\quad)$ 位数。最大的两位数乘最小的两位数, 积是 $(\\quad)$ 位数。", "options": [], "subject": "算术", "analysis": "两 三\n\n【分析】三位数除以一位数, 如果被除数百位上的数大于等于除数, 则商是三位数, 否则, 商是两位数; 最大两位数是 99 , 最小两位数是 10 , 求出两数相乘的积即可知道积的位数; 据此即可解答。\n\n【详解】 $617 \\div 7$ 中, $6<7$, 商是两位数。 $99 \\times 10=990$, 所以最大的两位数乘最小的两位数, 积是三位数。\n\n【点睛】本题主要考查学生对整数乘除法知识的掌握和灵活运用。"} {"id": "8667", "image": ["1371.jpg"], "answer": "$750 \\quad 205$\n\n【分析】根据总价=单价 $\\times$ 数量, 求出 10 件上衣的价钱。根据单价 $=$ 总价 $\\div$ 数量, 求出每条裤子的价钱。\n\n【详解】 $75 \\times 10=750 ($ 元)\n\n则 10 件上衣 750 元。\n\n$820 \\div 4=205 ($ 元)\n\n则每条裤子 205 元。\n\n【点睛】本题考查经济问题, 关键是熟记总价、单价和数量之间的关系。", "solution": "null", "level": "三年级", "question": "每件上衣 75 元, 10 件上衣( )元; 820 元买了 4 条裤子,每条裤子( )元。\n\n14 . _在 $\\square$里填上合适的数, 使坚式成立。\n\n", "options": [], "subject": "算术", "analysis": "$750 \\quad 205$\n\n【分析】根据总价=单价 $\\times$ 数量, 求出 10 件上衣的价钱。根据单价 $=$ 总价 $\\div$ 数量, 求出每条裤子的价钱。\n\n【详解】 $75 \\times 10=750 ($ 元)\n\n则 10 件上衣 750 元。\n\n$820 \\div 4=205 ($ 元)\n\n则每条裤子 205 元。\n\n【点睛】本题考查经济问题, 关键是熟记总价、单价和数量之间的关系。"} {"id": "8686", "image": [], "answer": "吨 t\n\n【分析】 1 袋大米的重量 $\\times$ 大米的袋数 $=$ 这些大米的总重量, 依此计算并将单位化成吨即可, 1000 千克 $=1$ 吨, 依此换算。\n\n【详解】 $50 \\times 20=1000$ (千克)\n\n1000 千克 $=1$ 吨\n\n即 1 袋大米重 50 千克, 这样的 20 袋大米重 1 吨。\n\n【点睛】此题考查的是吨与千克之间的换算, 熟记它们之间的进率是解答本题的关键。", "solution": "null", "level": "三年级", "question": "1 袋大米重 50 千克, 这样的 20 袋大米重 $1($ )。", "options": [], "subject": "算术", "analysis": "吨 t\n\n【分析】 1 袋大米的重量 $\\times$ 大米的袋数 $=$ 这些大米的总重量, 依此计算并将单位化成吨即可, 1000 千克 $=1$ 吨, 依此换算。\n\n【详解】 $50 \\times 20=1000$ (千克)\n\n1000 千克 $=1$ 吨\n\n即 1 袋大米重 50 千克, 这样的 20 袋大米重 1 吨。\n\n【点睛】此题考查的是吨与千克之间的换算, 熟记它们之间的进率是解答本题的关键。"} {"id": "9044", "image": [], "answer": "解东东 贝贝\n\n\n\n\n\n比较三人跳远成绩大小, 最大的那个小数则那人跳的最远。比较三人的跑步成绩, 用时最短的那人跑得最快。\n\n\n\n1.5 米 $>1.4$ 米 $>1.2$ 米\n\n则东东跳得最远。\n\n17.6 秒 $<18.2$ 秒 $<18.3$ 秒\n\n则贝贝跑得最快。\n\n\n\n小数比较大小的方法与整数基本相同, 都是从高位起, 依次把相同数位上的数进行比较。", "solution": "null", "level": "三年级", "question": "在跳远比赛中, 明明跳 1.2 米, 东东跳 1.5 米, 贝贝跳 1.4 米, (跳得最远, 在百米跑比赛中, 贝贝跑 17.6 秒, 明明跑 18.3 秒, 东东跑 18.2 秒, ( )跑得最快。", "options": [], "subject": "计数", "analysis": "解东东 贝贝\n\n\n\n\n\n比较三人跳远成绩大小, 最大的那个小数则那人跳的最远。比较三人的跑步成绩, 用时最短的那人跑得最快。\n\n\n\n1.5 米 $>1.4$ 米 $>1.2$ 米\n\n则东东跳得最远。\n\n17.6 秒 $<18.2$ 秒 $<18.3$ 秒\n\n则贝贝跑得最快。\n\n\n\n小数比较大小的方法与整数基本相同, 都是从高位起, 依次把相同数位上的数进行比较。"} {"id": "9099", "image": [], "answer": "解$x$\n\n【分析】单式统计表和复式统计表的区别: 单式统计表用于表示一组数据, 而复式统计表用于表示多组的数据。任意两个统计表不一定能合成一个复式统计表, 只有两个有联系的统计表才可以合成一个复式统计表;据此解答。\n\n【详解】据分析可知:\n\n复式统计表是由几个有联系的单式统计表合并而成的,可以更简洁地表示信息,所以任意两个统计表不一定可以合成一个复式统计表, 故原题说法错误。故答案为: $x$\n\n【点睛】本题主要考查复式统计表的特点, 应熟练掌握。", "solution": "null", "level": "三年级", "question": "任意两个统计表都可以合成一个复式统计表。( $)$", "options": [], "subject": "计数", "analysis": "解$x$\n\n【分析】单式统计表和复式统计表的区别: 单式统计表用于表示一组数据, 而复式统计表用于表示多组的数据。任意两个统计表不一定能合成一个复式统计表, 只有两个有联系的统计表才可以合成一个复式统计表;据此解答。\n\n【详解】据分析可知:\n\n复式统计表是由几个有联系的单式统计表合并而成的,可以更简洁地表示信息,所以任意两个统计表不一定可以合成一个复式统计表, 故原题说法错误。故答案为: $x$\n\n【点睛】本题主要考查复式统计表的特点, 应熟练掌握。"} {"id": "9100", "image": [], "answer": "解$\\sqrt{ }$\n\n【分析】统计表按照表头的形式, 可分为单式统计表和复式统计表, 据此解答。\n\n【详解】我们学过了统计表, 知道统计表分单式统计表和复式统计表。故答案为: $\\sqrt{ }$\n\n【点睛】本题考查统计表的分类, 正确记忆并灵活运用。", "solution": "null", "level": "三年级", "question": "我们学过了统计表,知道统计表分单式统计表和复式统计表。( )", "options": [], "subject": "计数", "analysis": "解$\\sqrt{ }$\n\n【分析】统计表按照表头的形式, 可分为单式统计表和复式统计表, 据此解答。\n\n【详解】我们学过了统计表, 知道统计表分单式统计表和复式统计表。故答案为: $\\sqrt{ }$\n\n【点睛】本题考查统计表的分类, 正确记忆并灵活运用。"} {"id": "9101", "image": [], "answer": "解$\\sqrt{ }$\n\n【分析】复式统计表可表示多组数据, 可以更加清晰、明了地反映数据的情况, 依此判断。\n\n【详解】把两个有关联的统计表合成一个复式统计表,便于数据分析和比较。例如:四一班男生最喜欢的活动\n\n| 活动 | 看书 | 踢球 | 看电视 | 画画 | 跳绳 |\n| :---: | :---: | :---: | :---: | :---: | :---: |\n| 人数 | 5 | 10 | 6 | 1 | 4 |\n\n四一班女生最喜欢的活动\n\n| 活动 | 看书 | 踢球 | 看电视 | 画画 | 跳绳 |\n| :---: | :---: | :---: | :---: | :---: | :---: |\n\n\n| 人数 | 9 | 2 | 5 | 4 | 3 |\n| :--- | :--- | :--- | :--- | :--- | :--- |\n\n四一班男生、女生最喜欢的活动统计表\n\n| 人数 | 看书 | 踢球 | 看电视 | 画画 | 跳绳 |\n| :---: | :---: | :---: | :---: | :---: | :---: |\n| 男生 | 5 | 10 | 6 | 1 | 4 |\n| 女生 | 9 | 2 | 5 | 4 | 3 |\n\n故答案为: $\\sqrt{ }$\n\n【点睛】熟练掌握复式统计表的特点是解答本题的关键。", "solution": "null", "level": "三年级", "question": "把两个有关联的统计表合成一个复式统计表,便于数据分析和比较。( )", "options": [], "subject": "计数", "analysis": "解$\\sqrt{ }$\n\n【分析】复式统计表可表示多组数据, 可以更加清晰、明了地反映数据的情况, 依此判断。\n\n【详解】把两个有关联的统计表合成一个复式统计表,便于数据分析和比较。例如:四一班男生最喜欢的活动\n\n| 活动 | 看书 | 踢球 | 看电视 | 画画 | 跳绳 |\n| :---: | :---: | :---: | :---: | :---: | :---: |\n| 人数 | 5 | 10 | 6 | 1 | 4 |\n\n四一班女生最喜欢的活动\n\n| 活动 | 看书 | 踢球 | 看电视 | 画画 | 跳绳 |\n| :---: | :---: | :---: | :---: | :---: | :---: |\n\n\n| 人数 | 9 | 2 | 5 | 4 | 3 |\n| :--- | :--- | :--- | :--- | :--- | :--- |\n\n四一班男生、女生最喜欢的活动统计表\n\n| 人数 | 看书 | 踢球 | 看电视 | 画画 | 跳绳 |\n| :---: | :---: | :---: | :---: | :---: | :---: |\n| 男生 | 5 | 10 | 6 | 1 | 4 |\n| 女生 | 9 | 2 | 5 | 4 | 3 |\n\n故答案为: $\\sqrt{ }$\n\n【点睛】熟练掌握复式统计表的特点是解答本题的关键。"} {"id": "9102", "image": [], "answer": "解$\\times$\n\n【分析】其它种类的树品种不确定,举例说明即可。\n\n【详解】其它种类的可能有柏树,数量可能比 10 小,所以原题说法错误。\n\n【点睛】关键是注意其它类, 可能有多种为区分的种类。", "solution": "null", "level": "三年级", "question": "从统计表中可看出, 槐树棵数一定最少。( $\\quad)$\n\n| 树种 | 柳树 | 杨树 | 槐树 | 其他 |\n| :---: | :---: | :---: | :---: | :---: |\n| 棵数/棵 | 40 | 30 | 10 | 20 |", "options": [], "subject": "计数", "analysis": "解$\\times$\n\n【分析】其它种类的树品种不确定,举例说明即可。\n\n【详解】其它种类的可能有柏树,数量可能比 10 小,所以原题说法错误。\n\n【点睛】关键是注意其它类, 可能有多种为区分的种类。"} {"id": "9103", "image": [], "answer": "解$\\sqrt{ }$\n\n【分析】复式统计表可表示多组数据, 还可以更加清晰、明了地反映数据的情况。此题依此判断即可。\n\n【详解】复式统计表的优点是简洁明了, 信息多, 易于比较。\n\n故答案为: $\\sqrt{ }$\n\n【点睛】灵活掌握复式统计表的特点, 是解答本题的关键。", "solution": "null", "level": "三年级", "question": "复式统计表的优点是信息多、易比较。", "options": [], "subject": "计数", "analysis": "解$\\sqrt{ }$\n\n【分析】复式统计表可表示多组数据, 还可以更加清晰、明了地反映数据的情况。此题依此判断即可。\n\n【详解】复式统计表的优点是简洁明了, 信息多, 易于比较。\n\n故答案为: $\\sqrt{ }$\n\n【点睛】灵活掌握复式统计表的特点, 是解答本题的关键。"} {"id": "9125", "image": [], "answer": "解$\\sqrt{ }$\n\n【详解】把多个统计内容 (调查内容) 一样的单式统计表合并在一起, 统称为复式统计表。据此可知,复式统计表是由若干个单式统计表合并而成的。此说法正确。\n\n故答案为: $\\sqrt{ }$ 。", "solution": "null", "level": "三年级", "question": "复式统计表是由许多个单式统计表合成的。( )", "options": [], "subject": "计数", "analysis": "解$\\sqrt{ }$\n\n【详解】把多个统计内容 (调查内容) 一样的单式统计表合并在一起, 统称为复式统计表。据此可知,复式统计表是由若干个单式统计表合并而成的。此说法正确。\n\n故答案为: $\\sqrt{ }$ 。"} {"id": "9076", "image": [], "answer": "解3 种\n\n3 张 5 元、 4 张 2 元、 4 张 1 元, 总共 27 元, 不够 33 元, 所以 10 元肯定是要用的, 然后再分析 5 元、2 元、1 元各用了几张。\n\n从金额大的开始考虑:\n\n$33=10 \\times 1+5 \\times 3+2 \\times 4$\n\n$33=10 \\times 1+5 \\times 3+2 \\times 3+1 \\times 2$\n\n$33=10 \\times 1+5 \\times 3+2 \\times 2+1 \\times 4$\n\n答: 他有 3 种付钱方式。\n\n本题考查的是搭配问题, 可以把 5 元、 2 元、 1 元的张数设为未知数, 列不定方程求解。", "solution": "null", "level": "三年级", "question": "小明有 1 张 10 元、 3 张 5 元、 4 张 2 元、 4 张 1 元,买笔花掉了 33 元钱,他有几种付钱方式?", "options": [], "subject": "组合数学", "analysis": "解3 种\n\n3 张 5 元、 4 张 2 元、 4 张 1 元, 总共 27 元, 不够 33 元, 所以 10 元肯定是要用的, 然后再分析 5 元、2 元、1 元各用了几张。\n\n从金额大的开始考虑:\n\n$33=10 \\times 1+5 \\times 3+2 \\times 4$\n\n$33=10 \\times 1+5 \\times 3+2 \\times 3+1 \\times 2$\n\n$33=10 \\times 1+5 \\times 3+2 \\times 2+1 \\times 4$\n\n答: 他有 3 种付钱方式。\n\n本题考查的是搭配问题, 可以把 5 元、 2 元、 1 元的张数设为未知数, 列不定方程求解。"} {"id": "9077", "image": [], "answer": "解3 种\n\n由于两个盒子是同样的, 且允许有的盒子空着不放, 那么可以从一个盒子取 0 开始枚举, 求出所有的可能。\n\n分类枚举如下:\n\n$4=4+0=3+1=2+2$\n\n答: 一共有 3 种。\n\n由于不考虑顺序, 所以相当于是数的分拆的问题, 可以按照一定的顺序进行分拆, 注意不能重复。", "solution": "null", "level": "三年级", "question": "小明想把 4 块巧克力放入两个同样的盒子里, 允许有的盒子空着不放, 你知道一共有多少种不同的放法么?", "options": [], "subject": "组合数学", "analysis": "解3 种\n\n由于两个盒子是同样的, 且允许有的盒子空着不放, 那么可以从一个盒子取 0 开始枚举, 求出所有的可能。\n\n分类枚举如下:\n\n$4=4+0=3+1=2+2$\n\n答: 一共有 3 种。\n\n由于不考虑顺序, 所以相当于是数的分拆的问题, 可以按照一定的顺序进行分拆, 注意不能重复。"} {"id": "9079", "image": [], "answer": "解80 束\n\n【分析】\n\n一束鲜花需要 2 枝康乃馨, 204 枝康乃馨可以扎 $204 \\div 2=102$ (束) ; 同理, 满天星可以扎 80 束还剩 5 枝; 玫瑰花可以扎 92 束还剩 2 枝。取 $102 、 80$ 和 92 中最小的数, 因此最多可以扎 80 束鲜花。\n\n【详解】\n\n$204 \\div 2=102($ 束 $)$\n\n$485 \\div 6=80$ (束) $\\ldots . . .5$ (枝)\n\n$278 \\div 3=92($ 束 $) \\ldots . . .2($ 枝 $)$\n\n$102>92>80$\n\n答: 这些花最多可以扎成 80 束鲜花。\n\n【点睛】\n\n本题考查除数是一位数的除法的应用和搭配问题, 注意除不尽时, 余数小于除数。", "solution": "null", "level": "三年级", "question": "鲜花店进了一批鲜花, 康乃馨 204 枝, 满天星 485 枝, 玫瑰花 278 枝。如果每 2 枝康乃馨, 6 枝满天星和 3 枝玫瑰花扎成一束鲜花, 那么这些花最多可以扎成多少束鲜花?", "options": [], "subject": "组合数学", "analysis": "解80 束\n\n【分析】\n\n一束鲜花需要 2 枝康乃馨, 204 枝康乃馨可以扎 $204 \\div 2=102$ (束) ; 同理, 满天星可以扎 80 束还剩 5 枝; 玫瑰花可以扎 92 束还剩 2 枝。取 $102 、 80$ 和 92 中最小的数, 因此最多可以扎 80 束鲜花。\n\n【详解】\n\n$204 \\div 2=102($ 束 $)$\n\n$485 \\div 6=80$ (束) $\\ldots . . .5$ (枝)\n\n$278 \\div 3=92($ 束 $) \\ldots . . .2($ 枝 $)$\n\n$102>92>80$\n\n答: 这些花最多可以扎成 80 束鲜花。\n\n【点睛】\n\n本题考查除数是一位数的除法的应用和搭配问题, 注意除不尽时, 余数小于除数。"} {"id": "9171", "image": [], "answer": "解$\\times$\n\n【分析】根据方向的相对性, 东和西相对, 南和北相对, 所以向南走 300 米, 对应返回时应该向北走 300 米,据此解答。\n\n【详解】聪聪去超市时从家向南走 300 米, 回来时要向北走 300 米。\n\n故答案为: $\\times$\n\n【点睛】本题主要考查方向的相对性, 方向相反, 距离不变。", "solution": "null", "level": "三年级", "question": "聪聪去超市时从家向南走 300 米, 回来时要向西走 300 米。( )", "options": [], "subject": "度量几何学", "analysis": "解$\\times$\n\n【分析】根据方向的相对性, 东和西相对, 南和北相对, 所以向南走 300 米, 对应返回时应该向北走 300 米,据此解答。\n\n【详解】聪聪去超市时从家向南走 300 米, 回来时要向北走 300 米。\n\n故答案为: $\\times$\n\n【点睛】本题主要考查方向的相对性, 方向相反, 距离不变。"} {"id": "9198", "image": [], "answer": "解$\\sqrt{ }$\n\n【分析】根据 1 平方分米 $=100$ 平方厘米, 高级单位化成低级单位乘它们之间的进率, 算出 45 平方厘米 +55 平方厘米的和, 统一单位后进行比较即可。\n\n【详解】 45 平方厘米 +55 平方厘米 $=100$ 平方厘米 $=1$ 平方分米\n\n故答案为: $\\sqrt{ }$\n\n【点睛】此题是考查面积的单位换算、面积的名数加减, 需熟练掌握。", "solution": "null", "level": "三年级", "question": "45 平方厘米 +55 平方厘米 $=1$ 平方分米。 $(\\quad)$", "options": [], "subject": "度量几何学", "analysis": "解$\\sqrt{ }$\n\n【分析】根据 1 平方分米 $=100$ 平方厘米, 高级单位化成低级单位乘它们之间的进率, 算出 45 平方厘米 +55 平方厘米的和, 统一单位后进行比较即可。\n\n【详解】 45 平方厘米 +55 平方厘米 $=100$ 平方厘米 $=1$ 平方分米\n\n故答案为: $\\sqrt{ }$\n\n【点睛】此题是考查面积的单位换算、面积的名数加减, 需熟练掌握。"} {"id": "9199", "image": [], "answer": "解$\\sqrt{ }$\n\n【分析】根据正方形的周长 $=$ 边长 $\\times 4$, 那么边长 $=$ 周长 $\\div 4$, 据此求出边长, 再根据正方形的面积 $=$ 边长 $\\times$ 边长, 把数据代入公式解答。\n\n【详解】 $20 \\div 4=5$ (厘米)\n\n$5 \\times 5=25($ 平方厘米 $)$\n\n所以一个周长是 20 厘米的正方形, 它的面积是 25 厘米, 此说法正确。\n\n故答案为: $\\sqrt{ }$\n\n【点睛】此题主要考查正方形的周长公式、面积公式的灵活运用, 关键是熟记公式。", "solution": "null", "level": "三年级", "question": "一个周长是 20 厘米的正方形, 它的面积是 25 厘米。( )", "options": [], "subject": "度量几何学", "analysis": "解$\\sqrt{ }$\n\n【分析】根据正方形的周长 $=$ 边长 $\\times 4$, 那么边长 $=$ 周长 $\\div 4$, 据此求出边长, 再根据正方形的面积 $=$ 边长 $\\times$ 边长, 把数据代入公式解答。\n\n【详解】 $20 \\div 4=5$ (厘米)\n\n$5 \\times 5=25($ 平方厘米 $)$\n\n所以一个周长是 20 厘米的正方形, 它的面积是 25 厘米, 此说法正确。\n\n故答案为: $\\sqrt{ }$\n\n【点睛】此题主要考查正方形的周长公式、面积公式的灵活运用, 关键是熟记公式。"} {"id": "9200", "image": [], "answer": "解$\\sqrt{ }$\n\n【分析】正方形的面积 $=$ 边长 $\\times$ 边长, 依此计算出边长 20 厘米的正方形的面积, 然后根据“100 平方厘米 $=1$ 平方分米”将单位化成平方分米即可, 依此计算并判断。\n\n【详解】 $20 \\times 20=400$ (平方厘米)\n\n400 平方厘米 $=4$ 平方分米\n\n即边长 20 厘米的正方形的面积是 4 平方分米。\n\n故答案为: $\\sqrt{ }$\n\n【点睛】熟练掌握正方形的面积的计算, 以及面积单位之间的换算, 是解答此题的关键。", "solution": "null", "level": "三年级", "question": "边长 20 厘米的正方形的面积是 4 平方分米。( )", "options": [], "subject": "度量几何学", "analysis": "解$\\sqrt{ }$\n\n【分析】正方形的面积 $=$ 边长 $\\times$ 边长, 依此计算出边长 20 厘米的正方形的面积, 然后根据“100 平方厘米 $=1$ 平方分米”将单位化成平方分米即可, 依此计算并判断。\n\n【详解】 $20 \\times 20=400$ (平方厘米)\n\n400 平方厘米 $=4$ 平方分米\n\n即边长 20 厘米的正方形的面积是 4 平方分米。\n\n故答案为: $\\sqrt{ }$\n\n【点睛】熟练掌握正方形的面积的计算, 以及面积单位之间的换算, 是解答此题的关键。"} {"id": "9201", "image": [], "answer": "解$\\times$\n\n【分析】正方形的面积 $=$ 边长 $\\times$ 边长, 依此分别计算出边长是 $1 \\mathrm{dm} 、 2 \\mathrm{dm}$ 的正方形的面积, 然后再判断即可。\n\n【详解】 $1 \\times 1=1\\left(\\mathrm{dm}^{2}\\right), 2 \\times 2=4\\left(\\mathrm{dm}^{2}\\right)$\n\n即边长是 $1 \\mathrm{dm}$ 的正方形的面积是 $1 \\mathrm{dm}^{2}$ ,边长是 $2 \\mathrm{dm}$ 的正方形的面积是 $4 \\mathrm{dm}^{2}$ 。\n\n故答案为: $\\times$\n\n【点睛】熟练掌握正方形的面积的计算是解答此题的关键。", "solution": "null", "level": "三年级", "question": "边长是 $1 \\mathrm{dm}$ 的正方形的面积是 $1 \\mathrm{dm}^{2}$ ,边长是 $2 \\mathrm{dm}$ 的正方形的面积是 $2 \\mathrm{dm}^{2}$ 。( )", "options": [], "subject": "度量几何学", "analysis": "解$\\times$\n\n【分析】正方形的面积 $=$ 边长 $\\times$ 边长, 依此分别计算出边长是 $1 \\mathrm{dm} 、 2 \\mathrm{dm}$ 的正方形的面积, 然后再判断即可。\n\n【详解】 $1 \\times 1=1\\left(\\mathrm{dm}^{2}\\right), 2 \\times 2=4\\left(\\mathrm{dm}^{2}\\right)$\n\n即边长是 $1 \\mathrm{dm}$ 的正方形的面积是 $1 \\mathrm{dm}^{2}$ ,边长是 $2 \\mathrm{dm}$ 的正方形的面积是 $4 \\mathrm{dm}^{2}$ 。\n\n故答案为: $\\times$\n\n【点睛】熟练掌握正方形的面积的计算是解答此题的关键。"} {"id": "9202", "image": [], "answer": "解$\\times$\n\n【分析】正方形的周长是指围成正方形四条边的总长度, 正方形的面积是指围成正方形的大小。据此判断。\n\n【详解】 $4 \\times 4=16$ (分米)\n\n$4 \\times 4=16$ (平方分米)\n\n所以边长为 4 分米的正方形, 周长与面积的值相等, 单位不同, 周长和面积是不同的两个量, 不能比较。\n\n故答案为: $\\times$\n\n【点睛】本题考查学生对正方形周长和面积的认识。周长与面积单位不同, 不能比较大小。", "solution": "null", "level": "三年级", "question": "边长 4 分米的正方形,面积和周长相等。( $)$", "options": [], "subject": "度量几何学", "analysis": "解$\\times$\n\n【分析】正方形的周长是指围成正方形四条边的总长度, 正方形的面积是指围成正方形的大小。据此判断。\n\n【详解】 $4 \\times 4=16$ (分米)\n\n$4 \\times 4=16$ (平方分米)\n\n所以边长为 4 分米的正方形, 周长与面积的值相等, 单位不同, 周长和面积是不同的两个量, 不能比较。\n\n故答案为: $\\times$\n\n【点睛】本题考查学生对正方形周长和面积的认识。周长与面积单位不同, 不能比较大小。"} {"id": "9247", "image": [], "answer": "解$\\times$\n\n【分析】正方形的面积 $=$ 边长 $\\times$ 边长, 可知边长为 1 米的正方形, 面积是 1 平方米。 1 米 $=10$ 分米,边长为 10 分米的正方形, 面积是 100 平方分米, 则 1 平方米 $=100$ 平方分米。边长为 1 分米的正方形, 面积是 1 平方分米。 1 分米 $=10$ 厘米, 边长为 10 厘米的正方形, 面积是 100 平方厘米, 则 1 平方分米 $=100$ 平方厘米。\n\n【详解】 $1 \\times 1=1$ (平方米)\n\n1 米 $=10$ 分米\n\n$10 \\times 10=100$ (平方分米)\n\n则 1 平方米 $=100$ 平方分米\n\n$1 \\times 1=1$ (平方分米)\n\n1 分米 $=10$ 厘米\n\n$10 \\times 10=100$ (平方厘米)\n\n则 1 平方分米 $=100$ 平方厘米\n\n故答案为: $\\times$\n\n【点睛】本题考查面积单位进率的推导, 同时需要熟记长度单位的进率。", "solution": "null", "level": "三年级", "question": "1 平方米 $=100$ 平方分米, 1 平方分米 $=10$ 平方厘米。 $(\\quad)$", "options": [], "subject": "度量几何学", "analysis": "解$\\times$\n\n【分析】正方形的面积 $=$ 边长 $\\times$ 边长, 可知边长为 1 米的正方形, 面积是 1 平方米。 1 米 $=10$ 分米,边长为 10 分米的正方形, 面积是 100 平方分米, 则 1 平方米 $=100$ 平方分米。边长为 1 分米的正方形, 面积是 1 平方分米。 1 分米 $=10$ 厘米, 边长为 10 厘米的正方形, 面积是 100 平方厘米, 则 1 平方分米 $=100$ 平方厘米。\n\n【详解】 $1 \\times 1=1$ (平方米)\n\n1 米 $=10$ 分米\n\n$10 \\times 10=100$ (平方分米)\n\n则 1 平方米 $=100$ 平方分米\n\n$1 \\times 1=1$ (平方分米)\n\n1 分米 $=10$ 厘米\n\n$10 \\times 10=100$ (平方厘米)\n\n则 1 平方分米 $=100$ 平方厘米\n\n故答案为: $\\times$\n\n【点睛】本题考查面积单位进率的推导, 同时需要熟记长度单位的进率。"} {"id": "9248", "image": [], "answer": "解$\\sqrt{ }$\n【分析】长方形的面积 $=$ 长 $\\times$ 宽, 因此当长不变时, 宽增加到原来的几倍, 则面积就增加到原来的几倍, 依此计算。\n\n【详解】 $400 \\times 2=800$ (平方米)\n\n即一个长方形的面积是 400 平方米, 现在长不变, 宽增加到原来的 2 倍, 现在的面积是 800 平方米。故答案为: $\\sqrt{ }$\n\n【点睛】此题考查的是长方形的面积的计算, 熟练掌握积的变化规律是解答此题的关键。", "solution": "null", "level": "三年级", "question": "一个长方形的面积是 400 平方米, 现在长不变, 宽增加到原来的 2 倍, 现在的面积是 800 平方米。", "options": [], "subject": "度量几何学", "analysis": "解$\\sqrt{ }$\n【分析】长方形的面积 $=$ 长 $\\times$ 宽, 因此当长不变时, 宽增加到原来的几倍, 则面积就增加到原来的几倍, 依此计算。\n\n【详解】 $400 \\times 2=800$ (平方米)\n\n即一个长方形的面积是 400 平方米, 现在长不变, 宽增加到原来的 2 倍, 现在的面积是 800 平方米。故答案为: $\\sqrt{ }$\n\n【点睛】此题考查的是长方形的面积的计算, 熟练掌握积的变化规律是解答此题的关键。"} {"id": "9271", "image": [], "answer": "解$\\times$\n\n【分析】米是长度单位, 平方米是面积单位, 依此判断。\n\n【详解】 1 米指的是长度, 2 平方米指的是面积, 因此无法比较它们的大小。故答案为: $\\times$\n\n【点睛】熟练掌握对长度单位和面积单位的认识是解答此题的关键。", "solution": "null", "level": "三年级", "question": "1 米比 2 平方米小。( $\\quad)$", "options": [], "subject": "度量几何学", "analysis": "解$\\times$\n\n【分析】米是长度单位, 平方米是面积单位, 依此判断。\n\n【详解】 1 米指的是长度, 2 平方米指的是面积, 因此无法比较它们的大小。故答案为: $\\times$\n\n【点睛】熟练掌握对长度单位和面积单位的认识是解答此题的关键。"} {"id": "9272", "image": [], "answer": "解$\\sqrt{ }$\n\n【分析】根据正方形的面积 $=$ 边长 $\\times$ 边长, 把数据代入公式, 求出正方形的边长, 在乘法口诀中, 八八六十四, 所以 $8 \\times 8=64$, 仅此一种可能性, 据此解答。\n\n【详解】因为正方形的面积 $=$ 边长 $\\times$ 边长,\n\n$64=8 \\times 8$\n\n面积是 64 平方分米的正方形的边长一定是 8 分米。\n\n故答案为: $\\sqrt{ }$\n\n【点睛】熟练掌握正方形的面积公式是解决本题的关键。", "solution": "null", "level": "三年级", "question": "面积是 64 平方分米的正方形的边长一定是 8 分米。( )", "options": [], "subject": "度量几何学", "analysis": "解$\\sqrt{ }$\n\n【分析】根据正方形的面积 $=$ 边长 $\\times$ 边长, 把数据代入公式, 求出正方形的边长, 在乘法口诀中, 八八六十四, 所以 $8 \\times 8=64$, 仅此一种可能性, 据此解答。\n\n【详解】因为正方形的面积 $=$ 边长 $\\times$ 边长,\n\n$64=8 \\times 8$\n\n面积是 64 平方分米的正方形的边长一定是 8 分米。\n\n故答案为: $\\sqrt{ }$\n\n【点睛】熟练掌握正方形的面积公式是解决本题的关键。"} {"id": "9273", "image": [], "answer": "解$\\times$\n\n【分析】先设定一个正方形的边长, 然后计算出边长增加 2 厘米后的正方形的面积和原正方形的面积差, 据此即可解答。\n\n【详解】假设一个正方形的边长为 3 厘米, 面积就为 9 平方厘米, 边长增加 2 厘米后就变成 5 厘米,面积就为 25 平方厘米, 面积就增加了 $25-9=16$ 平方厘米, 原说法错误。故答案为: $\\times$\n\n【点睛】可以通过举例来判断原说法的对错。", "solution": "null", "level": "三年级", "question": "一个正方形, 它的边长增加 2 厘米, 面积就增加 4 平方厘米。( )", "options": [], "subject": "度量几何学", "analysis": "解$\\times$\n\n【分析】先设定一个正方形的边长, 然后计算出边长增加 2 厘米后的正方形的面积和原正方形的面积差, 据此即可解答。\n\n【详解】假设一个正方形的边长为 3 厘米, 面积就为 9 平方厘米, 边长增加 2 厘米后就变成 5 厘米,面积就为 25 平方厘米, 面积就增加了 $25-9=16$ 平方厘米, 原说法错误。故答案为: $\\times$\n\n【点睛】可以通过举例来判断原说法的对错。"} {"id": "9275", "image": [], "answer": "解$\\times$\n\n【分析】根据面积的意义, 围成平面的大小叫做图形的面积。此题是没有告诉两个图形的边长, 没法\n比较面积的大小, 只有在特定的条件下如: 周长相等的正方形和长方形, 正方形的面积比长方形的面积大。\n\n【详解】据分析可知: 在没有确定正方形的边长与长方形的长、宽是否相等的前提条件下, 无法求出长方形和正方形面积的大小;所以原题的说法错误。\n\n故答案为: $\\times$\n\n【点睛】此题考查的目的是理解掌握长方形、正方形面积的意义及应用。", "solution": "null", "level": "三年级", "question": "正方形的面积比长方形的面积大。", "options": [], "subject": "度量几何学", "analysis": "解$\\times$\n\n【分析】根据面积的意义, 围成平面的大小叫做图形的面积。此题是没有告诉两个图形的边长, 没法\n比较面积的大小, 只有在特定的条件下如: 周长相等的正方形和长方形, 正方形的面积比长方形的面积大。\n\n【详解】据分析可知: 在没有确定正方形的边长与长方形的长、宽是否相等的前提条件下, 无法求出长方形和正方形面积的大小;所以原题的说法错误。\n\n故答案为: $\\times$\n\n【点睛】此题考查的目的是理解掌握长方形、正方形面积的意义及应用。"} {"id": "9299", "image": [], "answer": "解$\\times$\n\n【分析】封闭图形一周的长度, 是它的周长; 物体的表面或围成的平面图形的大小, 叫面积。根据周长、面积的意义可知, 因为周长和面积是不同的两个量, 所以无法比较。\n\n【详解】正方形的周长是指围成正方形四条边的总长度, 正方形的面积是指围成正方形的大小, 意义不同;\n\n正方形的周长是边长 $\\times 4$, 正方形的面积是边长 $\\times$ 边长, 计算方法不同;\n\n周长的计量单位是长度单位, 面积的计量单位是面积单位, 计量单位不同;\n\n故无法比较, 所以原题说法错误。\n\n故答案为: $\\times$\n\n【点睛】此题考查的目的是理解掌握周长、面积的意义及应用。周长和面积是不同的两个量, 无法比较大小。", "solution": "null", "level": "三年级", "question": "边长 4 米的正方形, 它的周长和面积相等。(", "options": [], "subject": "度量几何学", "analysis": "解$\\times$\n\n【分析】封闭图形一周的长度, 是它的周长; 物体的表面或围成的平面图形的大小, 叫面积。根据周长、面积的意义可知, 因为周长和面积是不同的两个量, 所以无法比较。\n\n【详解】正方形的周长是指围成正方形四条边的总长度, 正方形的面积是指围成正方形的大小, 意义不同;\n\n正方形的周长是边长 $\\times 4$, 正方形的面积是边长 $\\times$ 边长, 计算方法不同;\n\n周长的计量单位是长度单位, 面积的计量单位是面积单位, 计量单位不同;\n\n故无法比较, 所以原题说法错误。\n\n故答案为: $\\times$\n\n【点睛】此题考查的目的是理解掌握周长、面积的意义及应用。周长和面积是不同的两个量, 无法比较大小。"} {"id": "9301", "image": [], "answer": "解$\\sqrt{ }$\n\n【分析】周长相等的长方形和正方形, 正方形的面积大于长方形的面积, 可以通过举例证明。\n\n【详解】比如:长方形和正方形的周长都是 16 厘米,\n\n长方形的长是 5 厘米, 宽是 3 厘米, 面积是: $5 \\times 3=15$ (平方厘米),\n\n正方形的边长是 4 厘米,面积是: $4 \\times 4=16$ (平方厘米),\n\n因此, 长方形和正方形的周长相等时, 正方形的面积大于长方形的面积, 这种说法是正确的。\n故答案为: $\\sqrt{ }$\n\n【点睛】此题考查的目的是理解掌握长方形、正方形的周长、面积的意义及应用。", "solution": "null", "level": "三年级", "question": "长方形和正方形的周长相等时, 正方形的面积大于长方形的面积。( )\n\n19._拇指指甲盖大约有 1 平方厘米那么大。", "options": [], "subject": "度量几何学", "analysis": "解$\\sqrt{ }$\n\n【分析】周长相等的长方形和正方形, 正方形的面积大于长方形的面积, 可以通过举例证明。\n\n【详解】比如:长方形和正方形的周长都是 16 厘米,\n\n长方形的长是 5 厘米, 宽是 3 厘米, 面积是: $5 \\times 3=15$ (平方厘米),\n\n正方形的边长是 4 厘米,面积是: $4 \\times 4=16$ (平方厘米),\n\n因此, 长方形和正方形的周长相等时, 正方形的面积大于长方形的面积, 这种说法是正确的。\n故答案为: $\\sqrt{ }$\n\n【点睛】此题考查的目的是理解掌握长方形、正方形的周长、面积的意义及应用。"} {"id": "9323", "image": [], "answer": "解$\\times$\n\n【详解】略", "solution": "null", "level": "三年级", "question": "一个长方形的长是 6 米,宽是 5 米,面积是 30 米.", "options": [], "subject": "度量几何学", "analysis": "解$\\times$\n\n【详解】略"} {"id": "9324", "image": [], "answer": "解$\\times$\n\n【详解】略", "solution": "null", "level": "三年级", "question": "长方形的面积越大, 长越长. ( )", "options": [], "subject": "度量几何学", "analysis": "解$\\times$\n\n【详解】略"} {"id": "9325", "image": [], "answer": "解$\\times$\n\n【分析】根据长方形的面积公式和周长公式, 判断即可。\n\n【详解】长方形的面积 $=$ 长 $\\times$ 宽, 长方形的周长 $=2 \\times($ 长 + 宽 $)=$ 长 $\\times 2+$ 宽 $\\times 2$, 所以题干说法错误。故答案为: $\\times$\n\n【点睛】本题主要考查的是长方形的周长公式和面积公式。熟练掌握长方形的周长和面积公式是解题的关键。", "solution": "null", "level": "三年级", "question": "长方形的面积公式是长 $\\times$ 宽, 周长公式是长 + 宽 $\\times 2 。(\\quad)$", "options": [], "subject": "度量几何学", "analysis": "解$\\times$\n\n【分析】根据长方形的面积公式和周长公式, 判断即可。\n\n【详解】长方形的面积 $=$ 长 $\\times$ 宽, 长方形的周长 $=2 \\times($ 长 + 宽 $)=$ 长 $\\times 2+$ 宽 $\\times 2$, 所以题干说法错误。故答案为: $\\times$\n\n【点睛】本题主要考查的是长方形的周长公式和面积公式。熟练掌握长方形的周长和面积公式是解题的关键。"} {"id": "9326", "image": [], "answer": "解$\\times$\n\n【分析】 正方形的面积 $=$ 边长 $\\times$ 边长。\n\n【详解】 正方形的边长扩大到原来的 4 倍, 面积也扩大到原来的 16 倍。\n故答案为: 错误。", "solution": "null", "level": "三年级", "question": "正方形的边长扩大到原来的 4 倍, 面积扩大到原来的 4 倍。( )", "options": [], "subject": "度量几何学", "analysis": "解$\\times$\n\n【分析】 正方形的面积 $=$ 边长 $\\times$ 边长。\n\n【详解】 正方形的边长扩大到原来的 4 倍, 面积也扩大到原来的 16 倍。\n故答案为: 错误。"} {"id": "9327", "image": [], "answer": "解$\\times$\n\n【分析】设这两根同样长度的铁丝长为 12 厘米, 则它们可以分别围成长 5 厘米, 宽为 1 厘米的长方形和长为 4 厘米, 宽为 2 厘米的长方形, 根据长方形的面积=长 $\\times$ 宽, 求得它们的面积即可进行判断。\n\n【详解】设这两根同样长度的铁丝长为 12 厘米, 它们可以分别围成长 5 厘米, 宽为 1 厘米的长方形和长为 4 厘米, 宽为 2 厘米的长方形, 则围成的长方形的面积分别为: $1 \\times 5=5$ (平方厘米); $2 \\times 4=8$ (平方厘米);所以原题说法错误。\n\n故答案为错误。\n\n【点睛】此题是利用举实例的方法进行判断正误: 周长相等的长方形, 面积不一定相等。", "solution": "null", "level": "三年级", "question": "用两根同样长的铁丝围成两个不同形状的长方形, 它们的面积相等。( )", "options": [], "subject": "度量几何学", "analysis": "解$\\times$\n\n【分析】设这两根同样长度的铁丝长为 12 厘米, 则它们可以分别围成长 5 厘米, 宽为 1 厘米的长方形和长为 4 厘米, 宽为 2 厘米的长方形, 根据长方形的面积=长 $\\times$ 宽, 求得它们的面积即可进行判断。\n\n【详解】设这两根同样长度的铁丝长为 12 厘米, 它们可以分别围成长 5 厘米, 宽为 1 厘米的长方形和长为 4 厘米, 宽为 2 厘米的长方形, 则围成的长方形的面积分别为: $1 \\times 5=5$ (平方厘米); $2 \\times 4=8$ (平方厘米);所以原题说法错误。\n\n故答案为错误。\n\n【点睛】此题是利用举实例的方法进行判断正误: 周长相等的长方形, 面积不一定相等。"} {"id": "9346", "image": [], "answer": "平方米; 平方分米; 平方厘米; 平方分米", "solution": "null", "level": "三年级", "question": "在横线上填上合适的单位名称, 用字母表示。\n\n一个会议室的占地面积是 70\n\n一张报纸的面积约为 20\n\n橡皮一个面的面积大约是 4\n\n电视屏幕的面积大约是 68", "options": [], "subject": "度量几何学", "analysis": "平方米; 平方分米; 平方厘米; 平方分米"} {"id": "9347", "image": [], "answer": " $200 ; 1000 ; 7 ; 10$", "solution": "null", "level": "三年级", "question": "2 平方分米 $=$ \\$ \\qquad \\$平方厘米\n\n10 平方米 $=$ \\$ \\qquad \\$平方分米\n\n700 平方厘米 $=$ \\$ \\qquad \\$平方分米\n\n100 分米 $=$ \\$ \\qquad \\$米", "options": [], "subject": "度量几何学", "analysis": " $200 ; 1000 ; 7 ; 10$"} {"id": "9351", "image": [], "answer": " $24 ; 96$", "solution": "null", "level": "三年级", "question": "梦梦家客厅长 6 米, 宽 4 米, 客厅的面积是 \\$ \\qquad \\$平方米。用边长为 5 分米的正方形地砖来铺地面, 需要 \\$ \\qquad \\$块。", "options": [], "subject": "度量几何学", "analysis": " $24 ; 96$"} {"id": "8714", "image": [], "answer": "$\\quad 50 \\quad 100$\n\n【分析】根据题意可知, 用周老师买球的总钱数减去王老师买球的总钱数, 就是 2 个网球的钱数, 除以 2 就是一个网球的价格, 再用王老师买球的总钱数减去 2 个网球的钱数除以 4 就是一个篮球的价格,据此解答。\n\n【详解】网球:\n\n$(600-500) \\div 2$\n\n$=100 \\div 2$\n\n$=50 ($ 元 $)$\n\n篮球:\n\n$(500-2 \\times 50) \\div 4$\n\n$=(500-100) \\div 4$\n\n$=400 \\div 4$\n\n$=100 ($ 元 $)$\n\n王老师买 2 个网球和 4 个篮球一共需要 500 元, 周老师同样的买 4 个网球和 4 个篮球一共需要 600 元。那么, 一个网球要 50 元, 一个篮球要 100 元。\n\n【点睛】本题考查三位数除以一位数的计算, 找出数量关系, 正确计算是解答本题的关键。", "solution": "null", "level": "三年级", "question": "王老师买 2 个网球和 4 个篮球一共需要 500 元, 周老师同样的买 4 个网球和 4 个篮球一共需要 600 元。那么, 一个网球要( )元, 一个篮球要( 元。", "options": [], "subject": "代数", "analysis": "$\\quad 50 \\quad 100$\n\n【分析】根据题意可知, 用周老师买球的总钱数减去王老师买球的总钱数, 就是 2 个网球的钱数, 除以 2 就是一个网球的价格, 再用王老师买球的总钱数减去 2 个网球的钱数除以 4 就是一个篮球的价格,据此解答。\n\n【详解】网球:\n\n$(600-500) \\div 2$\n\n$=100 \\div 2$\n\n$=50 ($ 元 $)$\n\n篮球:\n\n$(500-2 \\times 50) \\div 4$\n\n$=(500-100) \\div 4$\n\n$=400 \\div 4$\n\n$=100 ($ 元 $)$\n\n王老师买 2 个网球和 4 个篮球一共需要 500 元, 周老师同样的买 4 个网球和 4 个篮球一共需要 600 元。那么, 一个网球要 50 元, 一个篮球要 100 元。\n\n【点睛】本题考查三位数除以一位数的计算, 找出数量关系, 正确计算是解答本题的关键。"} {"id": "9249", "image": [], "answer": "解$\\times$\n\n【分析】长方形的面积 $=$ 长 $\\times$ 宽, 长方形的周长 $=($ 长 + 宽 $) \\times 2$; 依此可以假设出两个长方形的面积都是 16 平方厘米, 从而计算出它们的周长, 然后再判断。\n\n【详解】假设两个长方形的面积都是 18 平方厘米\n\n$9 \\times 2=18$ (平方厘米), 此时长方形的长为 9 厘米, 宽为 2 厘米\n\n$(9+2) \\times 2$\n\n$=11 \\times 2$\n\n$=22($ 厘米 $)$\n\n$6 \\times 3=18$ (平方厘米), 此时长方形的长为 6 厘米, 宽为 3 厘米\n\n$(6+3) \\times 2$\n\n$=9 \\times 2$\n\n$=18$ (厘米)\n\n22 厘米 $>18$ 厘米\n\n当两个长方形的长和宽相等时, 它们的面积相等,周长也相等,\n\n由此可知, 两个面积相等的长方形, 它们的周长不一定相等。\n\n故答案为: $\\times$\n\n【点睛】此题考查的是比较相同面积的长方形的周长, 应通过举例的方法进行判断。", "solution": "null", "level": "三年级", "question": "两个面积相等的长方形,它们的周长也一定相等。( )", "options": [], "subject": "解析几何", "analysis": "解$\\times$\n\n【分析】长方形的面积 $=$ 长 $\\times$ 宽, 长方形的周长 $=($ 长 + 宽 $) \\times 2$; 依此可以假设出两个长方形的面积都是 16 平方厘米, 从而计算出它们的周长, 然后再判断。\n\n【详解】假设两个长方形的面积都是 18 平方厘米\n\n$9 \\times 2=18$ (平方厘米), 此时长方形的长为 9 厘米, 宽为 2 厘米\n\n$(9+2) \\times 2$\n\n$=11 \\times 2$\n\n$=22($ 厘米 $)$\n\n$6 \\times 3=18$ (平方厘米), 此时长方形的长为 6 厘米, 宽为 3 厘米\n\n$(6+3) \\times 2$\n\n$=9 \\times 2$\n\n$=18$ (厘米)\n\n22 厘米 $>18$ 厘米\n\n当两个长方形的长和宽相等时, 它们的面积相等,周长也相等,\n\n由此可知, 两个面积相等的长方形, 它们的周长不一定相等。\n\n故答案为: $\\times$\n\n【点睛】此题考查的是比较相同面积的长方形的周长, 应通过举例的方法进行判断。"} {"id": "9274", "image": [], "answer": "解$\\sqrt{ }$\n\n【分析】因为 $9 \\times 1=9$, 所以用 9 个小正方形拼成一个长方形, 可以拼成 1 行, 这一行有 9 个小正方形, 即拼成宽为 1 个正方形的边长, 长为 9 个正方形的边长的长方形, 据此判断。\n\n【详解】由分析得:\n\n用同样大的 9 个小正方形拼成一个长方形, 只有一种拼法。\n\n故答案为: $\\sqrt{ }$\n\n【点睛】本题考查图形的拼接, 小正方形的个数可以由两个不相同的数相乘得到, 这样的两个数有几组, 就可以拼成几个长方形。", "solution": "null", "level": "三年级", "question": "用同样大的 9 个小正方形拼成一个长方形,只有一种拼法。( )", "options": [], "subject": "组合几何学", "analysis": "解$\\sqrt{ }$\n\n【分析】因为 $9 \\times 1=9$, 所以用 9 个小正方形拼成一个长方形, 可以拼成 1 行, 这一行有 9 个小正方形, 即拼成宽为 1 个正方形的边长, 长为 9 个正方形的边长的长方形, 据此判断。\n\n【详解】由分析得:\n\n用同样大的 9 个小正方形拼成一个长方形, 只有一种拼法。\n\n故答案为: $\\sqrt{ }$\n\n【点睛】本题考查图形的拼接, 小正方形的个数可以由两个不相同的数相乘得到, 这样的两个数有几组, 就可以拼成几个长方形。"} {"id": "9375", "image": [], "answer": "解$\\sqrt{ }$\n\n【分析】根据面积的意义: 面积是图形所占平面的大小可知, 无论拼成什么图形, 图形的面积都等于 4 个小正方形的面积和, 据此判断。\n\n【详解】由分析得:\n\n用 4 个相同的小正方形拼成不同形状的图形,这些图形的面积都相等,都等于 4 个小正方形的面积和,原说法正确。\n\n故答案为: $\\sqrt{ }$\n\n【点睛】此题考查的目的是理解面积的意义。", "solution": "null", "level": "三年级", "question": "用 4 个相同的小正方形拼成不同形状的图形,这些图形的面积都相等。( )", "options": [], "subject": "组合几何学", "analysis": "解$\\sqrt{ }$\n\n【分析】根据面积的意义: 面积是图形所占平面的大小可知, 无论拼成什么图形, 图形的面积都等于 4 个小正方形的面积和, 据此判断。\n\n【详解】由分析得:\n\n用 4 个相同的小正方形拼成不同形状的图形,这些图形的面积都相等,都等于 4 个小正方形的面积和,原说法正确。\n\n故答案为: $\\sqrt{ }$\n\n【点睛】此题考查的目的是理解面积的意义。"} {"id": "9300", "image": [], "answer": "解$\\sqrt{ }$\n\n【分析】根据正方形的面积 $=$ 边长 $\\times$ 边长, 把数据代入公式求出这块手拍的面积, 再根据面积单位间的进率, 化成用平方分米作单位, 然后与 4 平方分米进行比较即可。\n\n【详解】 $20 \\times 20=400$ (平方厘米)\n\n400 平方厘米 $=4$ 平方分米\n\n4 平方分米 $=4$ 平方分米\n\n所以一块正方形手帕的边长是 20 厘米, 它的面积是 4 平方分米。\n\n故答案为: $\\sqrt{ }$\n\n【点睛】此题主要考查正方形面积公式的灵活运用, 关键是熟记公式。注意单位的换算。", "solution": "null", "level": "三年级", "question": "一块正方形手帕的边长是 20 厘米, 它的面积是 4 平方分米。( )", "options": [], "subject": "立体几何学", "analysis": "解$\\sqrt{ }$\n\n【分析】根据正方形的面积 $=$ 边长 $\\times$ 边长, 把数据代入公式求出这块手拍的面积, 再根据面积单位间的进率, 化成用平方分米作单位, 然后与 4 平方分米进行比较即可。\n\n【详解】 $20 \\times 20=400$ (平方厘米)\n\n400 平方厘米 $=4$ 平方分米\n\n4 平方分米 $=4$ 平方分米\n\n所以一块正方形手帕的边长是 20 厘米, 它的面积是 4 平方分米。\n\n故答案为: $\\sqrt{ }$\n\n【点睛】此题主要考查正方形面积公式的灵活运用, 关键是熟记公式。注意单位的换算。"} {"id": "7522", "image": [], "answer": "1)错误", "solution": "null", "level": "三年级", "question": "海象是大型食肉动物, 它的体重约是 2 千克。", "options": [], "subject": "立体几何学", "analysis": "1)错误"} {"id": "7867", "image": ["1154.jpg"], "answer": "6630\n\n【分析】根据图示, 从小凡家到学校有 3 条路可以选择, 从学校到公园共有 2 条路可以选择, 根据乘法原理可知, 共有 $3 \\times 2=6$ 条路可以选择, 分别计算出每条路的长度, 比较得出最长的一条路即可,据此解答。\n\n【详解】(1) $180+390=570$ (米)\n\n(2) $180+450=630$ (米)\n\n(3) $100+390=490$ (米)\n\n(4) $100+450=550$ (米)\n\n(5) $160+190=350$ (米)\n\n(6) $160+450=610$ (米)\n\n630 米 $>610$ 米 $>570$ 米 $>550$ 米 $>490$ 米 $>350$ 米\n\n如图, 小凡从家经过学校去公园, 有 (6) 条路可以选择, 最远的一条路是(630)米。\n\n【点睛】本题考查三位数的加法计算, 熟练掌握并正确计算。", "solution": "null", "level": "三年级", "question": "如图, 小凡从家经过学校去公园, 有\n\n) 条路可以选择, 最远的一条路是(\n\n)米。\n\n", "options": [], "subject": "图论", "analysis": "6630\n\n【分析】根据图示, 从小凡家到学校有 3 条路可以选择, 从学校到公园共有 2 条路可以选择, 根据乘法原理可知, 共有 $3 \\times 2=6$ 条路可以选择, 分别计算出每条路的长度, 比较得出最长的一条路即可,据此解答。\n\n【详解】(1) $180+390=570$ (米)\n\n(2) $180+450=630$ (米)\n\n(3) $100+390=490$ (米)\n\n(4) $100+450=550$ (米)\n\n(5) $160+190=350$ (米)\n\n(6) $160+450=610$ (米)\n\n630 米 $>610$ 米 $>570$ 米 $>550$ 米 $>490$ 米 $>350$ 米\n\n如图, 小凡从家经过学校去公园, 有 (6) 条路可以选择, 最远的一条路是(630)米。\n\n【点睛】本题考查三位数的加法计算, 熟练掌握并正确计算。"} {"id": "7871", "image": [], "answer": "$\\times$\n\n【分析】计量比较重的物品, 常用“千克”作单位, 因此计量 1 只大公鸡的重量以“千克”为单位, 依此判断。\n\n【详解】根据分析可知, 王爷爷家的 1 只大公鸡有 2 千克重。\n\n故答案为: $\\times$\n【点睛】此题考查的是结合数据选择合适的单位名称, 熟练掌握对质量单位的认识是解答此题的关键。", "solution": "null", "level": "三年级", "question": "王爷爷家的 1 只大公鸡有 2 吨重。( )", "options": [], "subject": "逻辑题", "analysis": "$\\times$\n\n【分析】计量比较重的物品, 常用“千克”作单位, 因此计量 1 只大公鸡的重量以“千克”为单位, 依此判断。\n\n【详解】根据分析可知, 王爷爷家的 1 只大公鸡有 2 千克重。\n\n故答案为: $\\times$\n【点睛】此题考查的是结合数据选择合适的单位名称, 熟练掌握对质量单位的认识是解答此题的关键。"} {"id": "7987", "image": ["1202.jpg"], "answer": "正方 36\n\n【分析】如图的折叠方法可以得到一个最大的正方形和一个小长方形, 用原来长方形的长减去长方形的宽计算出剩下的长方形的长, 剩下的长方形的宽等于原长方形的宽, 长方形的周长 $=($ 长 + 宽 $) \\times 2$,依此计算出小长方形的周长即可。\n\n【详解】 $18-12=6$ (厘米)\n\n$(6+12) \\times 2$\n\n$=18 \\times 2$\n\n$=36($ 厘米 $)$\n\n由此可知, 将一张长 18 厘米、宽 12 厘米的长方形纸先折叠, 再剪一剪, 可以得到一个正方形和一个小长方形。小长方形的周长是 36 厘米。\n\n【点睛】此题考查的是平面图形的分割, 以及长方形的周长的计算, 应熟记: 长方形的周长 $=($ 长 +宽 $) \\times 2$ 。", "solution": "null", "level": "三年级", "question": "如图, 将一张长 18 厘米、宽 12 厘米的长方形纸先折叠, 再剪一剪, 可以得到一个(\n形和一个小长方形。小长方形的周长是 (\n\n)厘米。\n\n", "options": [], "subject": "变换几何", "analysis": "正方 36\n\n【分析】如图的折叠方法可以得到一个最大的正方形和一个小长方形, 用原来长方形的长减去长方形的宽计算出剩下的长方形的长, 剩下的长方形的宽等于原长方形的宽, 长方形的周长 $=($ 长 + 宽 $) \\times 2$,依此计算出小长方形的周长即可。\n\n【详解】 $18-12=6$ (厘米)\n\n$(6+12) \\times 2$\n\n$=18 \\times 2$\n\n$=36($ 厘米 $)$\n\n由此可知, 将一张长 18 厘米、宽 12 厘米的长方形纸先折叠, 再剪一剪, 可以得到一个正方形和一个小长方形。小长方形的周长是 36 厘米。\n\n【点睛】此题考查的是平面图形的分割, 以及长方形的周长的计算, 应熟记: 长方形的周长 $=($ 长 +宽 $) \\times 2$ 。"} {"id": "12278", "image": [], "answer": "答案:$\\sqrt{ }$", "solution": "null", "level": "五年级", "question": "根据三个方向观察到的图形摆小正方体, 结果只有一种。()", "options": [], "subject": "立体几何学", "analysis": "答案:$\\sqrt{ }$"} {"id": "12280", "image": ["2571.jpg"], "answer": "答案: $5 ; 6$", "solution": "null", "level": "五年级", "question": "一个用小正方体摆成的几何体。从正面、上面看到的都是 $\\square$, 那么摆成这样的几何体至少用 \\$ \\qquad \\$个小正方体,最多用 \\$ \\qquad \\$个小正方体。", "options": [], "subject": "立体几何学", "analysis": "答案: $5 ; 6$"} {"id": "12283", "image": [], "answer": "答案:$6 ; 8$", "solution": "null", "level": "五年级", "question": "由同样大小的小正方体搭成的一个立体图形, 从正面和左面看到的形状都是 $\\square 。$ 摆这样的立体图形,最少需要 \\$ \\qquad \\$个小正方体,最多需要 \\$ \\qquad \\$个小正方体。", "options": [], "subject": "立体几何学", "analysis": "答案:$6 ; 8$"} {"id": "12284", "image": [], "answer": "答案:$5 ; 7$", "solution": "null", "level": "五年级", "question": "一个立体图形从上面看到的形状是 $\\square \\square$,从左面看到的形状是 $\\square$,摆这个立体图形最少需要 \\$ \\qquad \\$个小正方体,最多需要 \\$ \\qquad \\$个小正方体。", "options": [], "subject": "立体几何学", "analysis": "答案:$5 ; 7$"} {"id": "12290", "image": [], "answer": "答案: $\\sqrt{ }$", "solution": "null", "level": "五年级", "question": "从一个方向看到的图形, 无法确定几何体的形状。", "options": [], "subject": "立体几何学", "analysis": "答案: $\\sqrt{ }$"} {"id": "24005", "image": ["11855.jpg", "11856.jpg", "11857.jpg", "11858.jpg", "11859.jpg", "11860.jpg"], "answer": "解(1)(2)(3)(4)\n\n(2) 1)(3)(6)\n\n【分析】(1) 观察发现, (2)(3)(4)从正面看, 都是 3 个小正方形排一行;\n\n(2) 观察发现, (1)(3)(6)从左面看, 都是 3 个小正方形排一行。\n\n(1)从正面看到的形状是 $\\square \\square$ 的物体有(2)(3)(4)。\n\n(2)从左面看到的形状是 $\\square \\square$ 的物体有(1)(3)(6)\n\n【点睛】本题考查观察物体, 解答本题的关键是掌握物体三视图的画法。", "solution": "null", "level": "五年级", "question": "根据要求, 选择适当的序号填在下面的括号里。\n\n\n\n(1)\n\n\n\n(2)\n\n\n\n(3)\n\n\n\n(4)\n\n\n\n(5)\n\n\n\n(6)\n\n(1)从正面看到的形状是 $\\square \\square$的物体有 ( )。\n\n(2)从左面看到的形状是 $\\square$ 口的物体有()。", "options": [], "subject": "立体几何学", "analysis": "解(1)(2)(3)(4)\n\n(2) 1)(3)(6)\n\n【分析】(1) 观察发现, (2)(3)(4)从正面看, 都是 3 个小正方形排一行;\n\n(2) 观察发现, (1)(3)(6)从左面看, 都是 3 个小正方形排一行。\n\n(1)从正面看到的形状是 $\\square \\square$ 的物体有(2)(3)(4)。\n\n(2)从左面看到的形状是 $\\square \\square$ 的物体有(1)(3)(6)\n\n【点睛】本题考查观察物体, 解答本题的关键是掌握物体三视图的画法。"} {"id": "24007", "image": ["11866.jpg", "11867.jpg", "11868.jpg", "11869.jpg", "11870.jpg", "11871.jpg", "11872.jpg", "11870.jpg", "11871.jpg", "11872.jpg"], "answer": "解(1) (2)\n\n【分析】根据从上面看到的几何体的平面图, 可以得出:从正面看有 3 列共 6 个小正方形; 从左往右分别是 1 个、 2 个、 3 个, 下齐; 从左面看有 3 列共 7 个小正方形, 从左往右分别是 2 个、 2 个、 3 个,下齐; 据此得出从正面、左面看的平面图形。\n\n【详解】结合从上面看到的平面图,可以得出几何体以及从正面、左面看的图形:\n\n\n\n\n\n正面\n\n\n\n左 面\n\n【点睛】具备根据部分视图还原立体图形的能力, 从而得出其他视图。", "solution": "null", "level": "五年级", "question": "小明搭的积木从上面看的形状是\n\n3 ,上面的数字表示在这个位置上所用的小正方体的个数。搭这组积木, 从正面看是( ), 从左面看是 $($ 。\n(1)\n\n\n(2)\n\n\n(3)\n\n\n(4.)\n\n", "options": [], "subject": "立体几何学", "analysis": "解(1) (2)\n\n【分析】根据从上面看到的几何体的平面图, 可以得出:从正面看有 3 列共 6 个小正方形; 从左往右分别是 1 个、 2 个、 3 个, 下齐; 从左面看有 3 列共 7 个小正方形, 从左往右分别是 2 个、 2 个、 3 个,下齐; 据此得出从正面、左面看的平面图形。\n\n【详解】结合从上面看到的平面图,可以得出几何体以及从正面、左面看的图形:\n\n\n\n\n\n正面\n\n\n\n左 面\n\n【点睛】具备根据部分视图还原立体图形的能力, 从而得出其他视图。"} {"id": "24020", "image": ["11932.jpg"], "answer": "解10\n\n【分析】通过观察发现,这个立体图形有三层,第一层有 6 个小正方体,第二层有 3 个小正方体,第三层有 1 个小正方体, 总共有 $(6+3+1)$ 个小正方体。\n\n【详解】根据分析得, $6+3+1=10$ (个)\n\n【点睛】此题考查了从不同方向观察物体的方法,意在培养学生的观察能力和空间思维能力。", "solution": "null", "level": "五年级", "question": "下面立体图形是由( )个小正方体搭成的。\n\n", "options": [], "subject": "立体几何学", "analysis": "解10\n\n【分析】通过观察发现,这个立体图形有三层,第一层有 6 个小正方体,第二层有 3 个小正方体,第三层有 1 个小正方体, 总共有 $(6+3+1)$ 个小正方体。\n\n【详解】根据分析得, $6+3+1=10$ (个)\n\n【点睛】此题考查了从不同方向观察物体的方法,意在培养学生的观察能力和空间思维能力。"} {"id": "24021", "image": ["11933.jpg", "11934.jpg"], "answer": "解$6 \\quad 9$\n\n【分析】根据从上面看到的图形可得, 这个图形的下层是 5 个小正方体, 后面一行 4 个小正方体, 前面一行 1 个小正方体靠右边; 根据从左面看到的图形可知,后面一行是 2 层,所以后面一行的 4 个小正方体的上面最少有 1 个小正方体,最多是 4 个小正方体,据此即可解答问题。\n\n【详解】 $5+1=6$ (个)\n\n$8+1=9$ (个)\n\n【点睛】本题是考查从不同方向观察物体和几何图形, 主要是培养学生的观察力和想象力。", "solution": "null", "level": "五年级", "question": "用同样大小的小正方体搭成的立体图形, 从上面看是\n\n\n\n, 从左面看是\n\n\n搭这个立体图形最少用( )个小正方体,最多用( )个小正方体。", "options": [], "subject": "立体几何学", "analysis": "解$6 \\quad 9$\n\n【分析】根据从上面看到的图形可得, 这个图形的下层是 5 个小正方体, 后面一行 4 个小正方体, 前面一行 1 个小正方体靠右边; 根据从左面看到的图形可知,后面一行是 2 层,所以后面一行的 4 个小正方体的上面最少有 1 个小正方体,最多是 4 个小正方体,据此即可解答问题。\n\n【详解】 $5+1=6$ (个)\n\n$8+1=9$ (个)\n\n【点睛】本题是考查从不同方向观察物体和几何图形, 主要是培养学生的观察力和想象力。"} {"id": "24022", "image": ["11935.jpg", "11936.jpg", "11937.jpg", "11938.jpg", "11939.jpg", "11940.jpg", "11941.jpg", "11942.jpg", "11943.jpg", "11944.jpg", "11945.jpg", "11938.jpg", "11939.jpg", "11940.jpg", "11941.jpg", "11942.jpg", "11943.jpg", "11944.jpg", "11945.jpg"], "answer": "解13 正\n\n【分析】分别画出三个几何体从正面、上面、左面看到的平面图形, 再找出从左面看到形状相同的两个几何体; 由图可知, 三个几何体从正面都可以看到三列, 左边一列最高层数为 2 层, 中间一列和右边一列都只有 1 层, 则这三个几何体从正面看到的形状相同; 据此解答。\n\n\n\n图 1\n\n\n\n图 2\n\n\n\n\n\n从左面看\n\n\n\n从上面看\n\n\n\n\n从正面看\n\n\n【详解】图 3\n\n由上可知, 从左面看, 图 1 和图 3 的形状相同; 从正面看, 这三个几何体的形状都相同。\n\n【点睛】本题主要考查从不同方向观察物体, 根据立体图形画出从不同方向看到的平面图形是解答题目的关键。", "solution": "null", "level": "五年级", "question": "把 5 个同样大小的正方体分别摆成下面的样子, 从左面看, 图(状相同;从( )面看, 这三个几何体的形状都相同。\n\n\n\n图 1\n\n\n\n图2\n\n\n\n图 3", "options": [], "subject": "立体几何学", "analysis": "解13 正\n\n【分析】分别画出三个几何体从正面、上面、左面看到的平面图形, 再找出从左面看到形状相同的两个几何体; 由图可知, 三个几何体从正面都可以看到三列, 左边一列最高层数为 2 层, 中间一列和右边一列都只有 1 层, 则这三个几何体从正面看到的形状相同; 据此解答。\n\n\n\n图 1\n\n\n\n图 2\n\n\n\n\n\n从左面看\n\n\n\n从上面看\n\n\n\n\n从正面看\n\n\n【详解】图 3\n\n由上可知, 从左面看, 图 1 和图 3 的形状相同; 从正面看, 这三个几何体的形状都相同。\n\n【点睛】本题主要考查从不同方向观察物体, 根据立体图形画出从不同方向看到的平面图形是解答题目的关键。"} {"id": "24425", "image": [], "answer": "解 $72 \\div 12=6$ (厘米)\n\n$$\n\\begin{aligned}\n& 6 \\times 6 \\times 6 \\\\\n& =36 \\times 6 \\\\\n& =216 \\text { (平方厘米 })\n\\end{aligned}\n$$\n\n答: 这个正方体的棱长是 6 厘米, 至少需要彩纸 216 平方厘米。\n\n故答案为: 6,216 。", "solution": "null", "level": "五年级", "question": "(2022 春・黄陵县期末) 小虎用 72 厘米长的铁丝正好焊接成一个正方体框架 (接头处忽略不计),这个正方体的棱长是 $\\qquad$厘米, 在框架的外面糊上彩纸, 至少需要彩纸 $\\qquad$平方厘米。", "options": [], "subject": "立体几何学", "analysis": "解 $72 \\div 12=6$ (厘米)\n\n$$\n\\begin{aligned}\n& 6 \\times 6 \\times 6 \\\\\n& =36 \\times 6 \\\\\n& =216 \\text { (平方厘米 })\n\\end{aligned}\n$$\n\n答: 这个正方体的棱长是 6 厘米, 至少需要彩纸 216 平方厘米。\n\n故答案为: 6,216 。"} {"id": "24426", "image": [], "answer": "解$(8 \\times 6+8 \\times 5+6 \\times 5) \\times 2$\n\n$=(48+40+30) \\times 2$\n\n$=118 \\times 2$\n$=236$ (平方厘米)\n\n$8 \\times 6 \\times 5$\n\n$=48 \\times 5$\n\n$=240$ (立方厘米)\n\n答: 这个长方体的表面积是 236 平方厘米, 体积是 240 立方厘米。\n\n故答案为: 236,240 。", "solution": "null", "level": "五年级", "question": "(2021 秋・洛宁县期末)一个长方体,相交于一个顶点的 3 条棱的长度分别是 8 厘米、 6 厘米和 5 厘米。这个长方体的表面积是 $\\qquad$平方厘米,体积是 $\\qquad$立方厘米。", "options": [], "subject": "立体几何学", "analysis": "解$(8 \\times 6+8 \\times 5+6 \\times 5) \\times 2$\n\n$=(48+40+30) \\times 2$\n\n$=118 \\times 2$\n$=236$ (平方厘米)\n\n$8 \\times 6 \\times 5$\n\n$=48 \\times 5$\n\n$=240$ (立方厘米)\n\n答: 这个长方体的表面积是 236 平方厘米, 体积是 240 立方厘米。\n\n故答案为: 236,240 。"} {"id": "24427", "image": [], "answer": "解$(2+4+4) \\times 4$\n\n$=10 \\times 4$\n\n$=40$ (厘米)\n\n$2 \\times 4 \\times 4=32$ (立方厘米)\n\n答: 这个长方体的棱长总和是 40 厘米, 体积是 32 立方厘米。\n\n故答案为: 40,32 。", "solution": "null", "level": "五年级", "question": "(2022 春・余杭区期中)明明有 4 根长 2 厘米, 3 根 3 厘米, 9 根 4 厘米的小棒, 选取 12 根搭成一个长方体, 这个长方体的棱长总和是 $\\qquad$ $\\mathrm{cm} ;$ 体积是 $\\qquad$ $\\mathrm{cm}^{3}$ 。", "options": [], "subject": "立体几何学", "analysis": "解$(2+4+4) \\times 4$\n\n$=10 \\times 4$\n\n$=40$ (厘米)\n\n$2 \\times 4 \\times 4=32$ (立方厘米)\n\n答: 这个长方体的棱长总和是 40 厘米, 体积是 32 立方厘米。\n\n故答案为: 40,32 。"} {"id": "24428", "image": [], "answer": "解 $10 \\times 10 \\times(10-3)$\n\n$=100 \\times 7$\n\n$=700$ (立方厘米)\n\n700 立方厘米 $=0.7$ 立方分米\n\n答:原来长方体的体积是 0.7 立方分米。\n\n故答案为: 0.7 。", "solution": "null", "level": "五年级", "question": "(2022 春・濮阳期末)一个长方体, 如果把它的高增加 $3 \\mathrm{~cm}$, 长和宽不变, 就变成一个棱长 $10 \\mathrm{~cm}$的正方体。原来长方体的体积是 $\\qquad$ $d m^{3}$ 。", "options": [], "subject": "立体几何学", "analysis": "解 $10 \\times 10 \\times(10-3)$\n\n$=100 \\times 7$\n\n$=700$ (立方厘米)\n\n700 立方厘米 $=0.7$ 立方分米\n\n答:原来长方体的体积是 0.7 立方分米。\n\n故答案为: 0.7 。"} {"id": "24429", "image": ["12477.jpg"], "answer": "解16 厘米 $=1.6$ 分米\n\n$4 \\times 1 \\times 1.6$\n\n$=4 \\times 1.6$\n\n$=6.4$ (立方分米 )\n\n$6.4 \\div(2 \\times 1)$\n\n$=6.4 \\div 2$\n\n$=3.2$ (分米)\n\n3.2 分米 $=32$ 厘米\n\n答:这时水深 32 厘米。\n\n故答案为: 32 。", "solution": "null", "level": "五年级", "question": "(2022・揭东区) 一个密封长方体容器长 4 分米, 宽 1 分米, 高 2 分米, 里面水深 16 厘米 (如图)。现在把这个容器的左侧面放在桌面上,这时水深 $\\qquad$厘米。\n\n", "options": [], "subject": "立体几何学", "analysis": "解16 厘米 $=1.6$ 分米\n\n$4 \\times 1 \\times 1.6$\n\n$=4 \\times 1.6$\n\n$=6.4$ (立方分米 )\n\n$6.4 \\div(2 \\times 1)$\n\n$=6.4 \\div 2$\n\n$=3.2$ (分米)\n\n3.2 分米 $=32$ 厘米\n\n答:这时水深 32 厘米。\n\n故答案为: 32 。"} {"id": "24430", "image": [], "answer": "解 $72 \\div 2=36$ (平方厘米)\n\n因为 $6 \\times 6=36$ (平方厘米), 可知正方体的棱长是 6 厘米。\n\n$6 \\times 6 \\times 6 \\times 2$\n$=216 \\times 2$\n\n$=432$ (立方厘米)\n\n答: 正方体的棱长是 $6 \\mathrm{~cm}$, 拼成的长方体的体积是 432 立方厘米。\n\n故答案为: $6 ; 432$ 。", "solution": "null", "level": "五年级", "question": "(2022・金湾区)如果把两个正方体拼成一个长方体,表面积减少了 $72 \\mathrm{~cm}^{2}$, 那么一个正方体的棱长是 $\\qquad$ cm,拼成的长方体的体积是 $\\qquad$ $\\mathrm{cm}^{3}$ 。", "options": [], "subject": "立体几何学", "analysis": "解 $72 \\div 2=36$ (平方厘米)\n\n因为 $6 \\times 6=36$ (平方厘米), 可知正方体的棱长是 6 厘米。\n\n$6 \\times 6 \\times 6 \\times 2$\n$=216 \\times 2$\n\n$=432$ (立方厘米)\n\n答: 正方体的棱长是 $6 \\mathrm{~cm}$, 拼成的长方体的体积是 432 立方厘米。\n\n故答案为: $6 ; 432$ 。"} {"id": "24431", "image": [], "answer": "解 $56 \\div 4-(6+5)$\n\n$=14-11$\n\n$=3(\\mathrm{~cm})$\n\n答:它的高是 $3 \\mathrm{~cm}$ 。\n\n故答案为: 3 。", "solution": "null", "level": "五年级", "question": "(2020 春・玉门市期末)用一根长 $56 \\mathrm{~cm}$ 的铁丝做成一个长方体框架, 长 $6 \\mathrm{~cm}$, 宽 $5 \\mathrm{~cm}$, 它的高是 $\\qquad$ $\\mathrm{cm}$ 。", "options": [], "subject": "立体几何学", "analysis": "解 $56 \\div 4-(6+5)$\n\n$=14-11$\n\n$=3(\\mathrm{~cm})$\n\n答:它的高是 $3 \\mathrm{~cm}$ 。\n\n故答案为: 3 。"} {"id": "24449", "image": [], "answer": "解$(15 \\times 12+15 \\times 8+12 \\times 8) \\times 2$\n\n$=(180+120+96) \\times 2$\n\n$=396 \\times 2$\n\n$=792$ (平方厘米)\n\n答: 作这个纸盒至少要用 792 平方厘米的硬纸板。\n\n故答案为: 792。", "solution": "null", "level": "五年级", "question": "(2021 秋・玄武区期末)一个长方体纸盒, 长 15 厘米, 宽 12 厘米, 高 8 厘米。做这个纸盒至少要用 $\\qquad$平方厘米的硬纸板。", "options": [], "subject": "立体几何学", "analysis": "解$(15 \\times 12+15 \\times 8+12 \\times 8) \\times 2$\n\n$=(180+120+96) \\times 2$\n\n$=396 \\times 2$\n\n$=792$ (平方厘米)\n\n答: 作这个纸盒至少要用 792 平方厘米的硬纸板。\n\n故答案为: 792。"} {"id": "24075", "image": ["11954.jpg", "11955.jpg", "11956.jpg", "11957.jpg", "11958.jpg", "11959.jpg", "11960.jpg", "11961.jpg", "11961.jpg"], "answer": "\n(1)\n(5)\n(6)\n(5)\n(4)\n\n【分析】观察图形,找出符合题意的序号即可。\n\n【详解】(1)从正面和左面看都是 $\\square$ 的有(1)。\n\n(2) (5)和(6)从上面看是 $\\square \\square$ 。\n\n(3) 从正面看(5)和从上面看(4)都是\n\n\n\n【点睛】此题考查了观察图形, 同时培养了学生的观察能力和空间想象能力。", "solution": "null", "level": "五年级", "question": "填一填。\n\n\n\n(1)\n\n\n\n(2)\n\n\n\n(3)\n\n\n\n(4)\n\n\n\n(5)\n\n\n\n(6)\n\n(1) 从正面和左面看都是 $\\square$ 的有( )。\n\n(2)( )和( ) 从上面看是 $\\square \\square$ 。\n\n(3) 从正面看( ) 和从上面看( ) 都是\n\n", "options": [], "subject": "立体几何学", "analysis": "\n(1)\n(5)\n(6)\n(5)\n(4)\n\n【分析】观察图形,找出符合题意的序号即可。\n\n【详解】(1)从正面和左面看都是 $\\square$ 的有(1)。\n\n(2) (5)和(6)从上面看是 $\\square \\square$ 。\n\n(3) 从正面看(5)和从上面看(4)都是\n\n\n\n【点睛】此题考查了观察图形, 同时培养了学生的观察能力和空间想象能力。"} {"id": "24076", "image": ["11962.jpg", "11963.jpg", "11964.jpg"], "answer": "解8", "solution": "null", "level": "五年级", "question": "有几堆摆好的小方块, 从三个不同的方向观察看到的形状如下图, 这里至少有 ( )个小方块.\n\n\n\n从正面看\n\n\n\n从右面看\n\n\n\n从上面看", "options": [], "subject": "立体几何学", "analysis": "解8"} {"id": "24077", "image": [], "answer": "解520", "solution": "null", "level": "五年级", "question": "一个几何体, 从正面看到的图形是 $\\square \\quad \\square$ ,我们可以初步判断这个几何体最少由()个小正方体组成,如果它是从由 6 个小正方体组成的几何体的正面看到的图形,该几何体只有两行,由( )种不同的摆法.", "options": [], "subject": "立体几何学", "analysis": "解520"} {"id": "24078", "image": ["11965.jpg", "11966.jpg", "11967.jpg", "11968.jpg", "11969.jpg", "11970.jpg", "11971.jpg", "11970.jpg", "11971.jpg"], "answer": "解(3) (2)\n\n【分析】根据从上面看到的图形,可以确定这个立体图形,如图所示:\n\n\n\n从左面看到的是三层, 第一层是 2 个正方形,第二层是 2 个正方形, 第 3 层靠左边有 1 个正方形, 据此解答。\n\n从正面看到的是三层,第一层是 3 个正方形, 第二层是 2 个正方形居左, 第 3 层靠有 1 个正方形居中,据此解答。\n\n\n则这个几何体从正面看是(3)),从左面看是(22)。\n\n【点睛】本题考查根据给出的三视图确定几何体以及从不同的方向观察立体图形。", "solution": "null", "level": "五年级", "question": "一个几何体从上面看是\n\n\n\n图中的数字表示在这个位置上所用的小正方体的个数, 则这个几何体从正面看是( ), 从左面看是( )。(填序号)\n\n(1)\n\n\n\n(2)\n\n\n\n(3)\n\n\n\n(4)\n\n", "options": [], "subject": "立体几何学", "analysis": "解(3) (2)\n\n【分析】根据从上面看到的图形,可以确定这个立体图形,如图所示:\n\n\n\n从左面看到的是三层, 第一层是 2 个正方形,第二层是 2 个正方形, 第 3 层靠左边有 1 个正方形, 据此解答。\n\n从正面看到的是三层,第一层是 3 个正方形, 第二层是 2 个正方形居左, 第 3 层靠有 1 个正方形居中,据此解答。\n\n\n则这个几何体从正面看是(3)),从左面看是(22)。\n\n【点睛】本题考查根据给出的三视图确定几何体以及从不同的方向观察立体图形。"} {"id": "24091", "image": ["12027.jpg", "12028.jpg", "12029.jpg", "12030.jpg", "12031.jpg", "12032.jpg", "12033.jpg", "12034.jpg", "12035.jpg", "12036.jpg", "12037.jpg", "12038.jpg", "12032.jpg", "12033.jpg", "12034.jpg", "12035.jpg", "12036.jpg", "12037.jpg", "12038.jpg"], "answer": "解\n\n\n\n\n\n( )\n\n\n\n$(\\sqrt{ } 1$\n\n【详解】试题分析:第一个图形从左面看是 2 层, 下面一层 2 个正方形, 上面一层 1 个正方形靠左;从正面看是 2 层, 下面一层 3 个正方形, 上面一层 1 个正方形靠左;\n\n第二个图形从左面看是 2 层, 下面一层 3 个正方形, 上面一层 1 个正方形靠左; 从正面看是 2 层, 下面一层 3 个正方形, 上面一层 1 个正方形靠左;\n\n第三个图形从左面看是 2 层, 下面一层 2 个正方形, 上面一层 1 个正方形靠左; 从正面看是 2 层, 下面一层 3 个正方形, 上面一层 1 个正方形靠左, 据此解答即可.\n\n解答: 解: 由分析可得:\n\n\n\n从左面看\n\n\n\n从正面看\n\n\n\n( )\n\n\n\n$(\\sqrt{ } 1$\n\n点评: 从不同的方向观察立体图形时, 因观察的方向不同, 观察到物体的形状也不相同.", "solution": "null", "level": "五年级", "question": "在符合要求的几何体下的括号里画“ $\\sqrt{ }$ ”.\n\n\n\n从左面看\n\n\n\n从正面看\n\n\n\n\n\n( )\n\n\n\n( )", "options": [], "subject": "立体几何学", "analysis": "解\n\n\n\n\n\n( )\n\n\n\n$(\\sqrt{ } 1$\n\n【详解】试题分析:第一个图形从左面看是 2 层, 下面一层 2 个正方形, 上面一层 1 个正方形靠左;从正面看是 2 层, 下面一层 3 个正方形, 上面一层 1 个正方形靠左;\n\n第二个图形从左面看是 2 层, 下面一层 3 个正方形, 上面一层 1 个正方形靠左; 从正面看是 2 层, 下面一层 3 个正方形, 上面一层 1 个正方形靠左;\n\n第三个图形从左面看是 2 层, 下面一层 2 个正方形, 上面一层 1 个正方形靠左; 从正面看是 2 层, 下面一层 3 个正方形, 上面一层 1 个正方形靠左, 据此解答即可.\n\n解答: 解: 由分析可得:\n\n\n\n从左面看\n\n\n\n从正面看\n\n\n\n( )\n\n\n\n$(\\sqrt{ } 1$\n\n点评: 从不同的方向观察立体图形时, 因观察的方向不同, 观察到物体的形状也不相同."} {"id": "24094", "image": ["12052.jpg", "12053.jpg", "12052.jpg", "12053.jpg"], "answer": "解$5 \\quad 6$\n\n【分析】根据从正面和上面看到的平面图形, 用小正方体摆出这个几何体, 确定最多和最少用到小正方体的个数。\n\n【详解】如图:\n\n\n\n最少 5 个小正方体\n\n\n\n最多6个小正方体\n\n【点睛】本题考查根据部分视图还原立体图形的能力, 培养学生的空间想象力。", "solution": "null", "level": "五年级", "question": "(2022・南京)一个由小正方体摆成的立体图形从正面和上面看到的都是 $\\square$, 这个立体图形最少由( ) 个小正方体摆成, 最多由( )个小正方体摆成。", "options": [], "subject": "立体几何学", "analysis": "解$5 \\quad 6$\n\n【分析】根据从正面和上面看到的平面图形, 用小正方体摆出这个几何体, 确定最多和最少用到小正方体的个数。\n\n【详解】如图:\n\n\n\n最少 5 个小正方体\n\n\n\n最多6个小正方体\n\n【点睛】本题考查根据部分视图还原立体图形的能力, 培养学生的空间想象力。"} {"id": "24095", "image": ["12054.jpg", "12055.jpg", "12056.jpg", "12057.jpg", "12058.jpg", "12059.jpg", "12060.jpg", "12061.jpg"], "answer": "解(1)(2)(4)(6)\n\n(2)(5)(7)(8)(9)\n【分析】本题将视角想象到每个图形的正面和左面,找到正面看是 $\\square$, 左面看是 $\\square$ 的即可。从正面看是 $\\square$ 的, 首先排除(5)(7)(8)(9), 它们都有 2 行, 再从(1)(2)(3)(4)(6)中找到正面看是 $\\square$ 的即可; 从左面看是 $\\square$ 的,首先排除只有 1 层的,剩下有 2 层的通过观察都可以。\n\n(1)\n\n从正面看是 $\\square$ 的有: (2)(4)(6)。\n\n(2)\n\n从左面看是 $\\square$ 的有: (5)(7)(8)(9)。\n\n【点睛】观察一个用小正方体搭建的立方体图形, 发现从不同的位置观察到图形的形状可能是不同的,也可能是相同的。\n13 ._详解\n(1)(3)\n(2) (1)(3)\n\n【分析】(1)从右边看到的图形有两层, 第一层有两个正方形, 第二层有 1 个正方形靠最左边; 从正面看到的图形有有两层, 第一层有两个正方形, 第二层有 1 个正方形靠最左边;\n\n(2)从右面看到的图形有两层,第一层有一个正方形,第二层有一个正方形; 从正面看到的图形有有两层, 第一层有三个正方形, 第二层有一个正方形靠中间; (3)从右边看到的图形有两层, 第一层有两个正方形, 第二层有 1 个正方形靠最左边;从正面看到的图形有有两层,第一层有两个正方形,第二层有 1 个正方形靠最左边; 据此解答即可。\n\n【详解】由分析可知:\n\n从右边看是图 A 的有(1)(3); 从右边看是图 B 的有(2); 从正面看是图 A 的有(1)(3)。\n\n【点睛】本题考查从不同方向观察物体,明确从不同方向观察到的形状是解题的关键。", "solution": "null", "level": "五年级", "question": "(2022・海门市)下面是用一些小正方体摆成的几何体。\n\n\n\n(1)\n\n\n\n(6)\n\n\n\n(2)\n\n\n\n(7)\n\n\n\n(3)\n\n\n\n(8)\n\n\n\n(4)\n\n\n\n(5)\n\n(1)从正面看是 $\\square$ 的有: $\\qquad$。\n\n(2)从左面看是 $\\square_{\\text {的有: }}$ $\\qquad$", "options": [], "subject": "立体几何学", "analysis": "解(1)(2)(4)(6)\n\n(2)(5)(7)(8)(9)\n【分析】本题将视角想象到每个图形的正面和左面,找到正面看是 $\\square$, 左面看是 $\\square$ 的即可。从正面看是 $\\square$ 的, 首先排除(5)(7)(8)(9), 它们都有 2 行, 再从(1)(2)(3)(4)(6)中找到正面看是 $\\square$ 的即可; 从左面看是 $\\square$ 的,首先排除只有 1 层的,剩下有 2 层的通过观察都可以。\n\n(1)\n\n从正面看是 $\\square$ 的有: (2)(4)(6)。\n\n(2)\n\n从左面看是 $\\square$ 的有: (5)(7)(8)(9)。\n\n【点睛】观察一个用小正方体搭建的立方体图形, 发现从不同的位置观察到图形的形状可能是不同的,也可能是相同的。\n13 ._详解\n(1)(3)\n(2) (1)(3)\n\n【分析】(1)从右边看到的图形有两层, 第一层有两个正方形, 第二层有 1 个正方形靠最左边; 从正面看到的图形有有两层, 第一层有两个正方形, 第二层有 1 个正方形靠最左边;\n\n(2)从右面看到的图形有两层,第一层有一个正方形,第二层有一个正方形; 从正面看到的图形有有两层, 第一层有三个正方形, 第二层有一个正方形靠中间; (3)从右边看到的图形有两层, 第一层有两个正方形, 第二层有 1 个正方形靠最左边;从正面看到的图形有有两层,第一层有两个正方形,第二层有 1 个正方形靠最左边; 据此解答即可。\n\n【详解】由分析可知:\n\n从右边看是图 A 的有(1)(3); 从右边看是图 B 的有(2); 从正面看是图 A 的有(1)(3)。\n\n【点睛】本题考查从不同方向观察物体,明确从不同方向观察到的形状是解题的关键。"} {"id": "24096", "image": ["12062.jpg", "12063.jpg", "12064.jpg", "12065.jpg", "12066.jpg"], "answer": "(1)(3)(2) (1)(3)【分析】(1)从右边看到的图形有两层, 第一层有两个正方形, 第二层有 1 个正方形靠最左边; 从正面看到的图形有有两层, 第一层有两个正方形, 第二层有 1 个正方形靠最左边;(2)从右面看到的图形有两层,第一层有一个正方形,第二层有一个正方形; 从正面看到的图形有有两层, 第一层有三个正方形, 第二层有一个正方形靠中间; (3)从右边看到的图形有两层, 第一层有两个正方形, 第二层有 1 个正方形靠最左边;从正面看到的图形有有两层,第一层有两个正方形,第二层有 1 个正方形靠最左边; 据此解答即可。【详解】由分析可知:从右边看是图 A 的有(1)(3); 从右边看是图 B 的有(2); 从正面看是图 A 的有(1)(3)。【点睛】本题考查从不同方向观察物体,明确从不同方向观察到的形状是解题的关键。", "solution": "null", "level": "五年级", "question": "(2022・海门市)从右边看是图 A 的有( ); 从右边看是图 B 的有( ); 从正面看是图 A 的有( )。\n\n\n\n(1)\n\n\n\n(2)\n\n\n\n(3)\n\n\n\nA\n\n\n\nB", "options": [], "subject": "立体几何学", "analysis": "(1)(3)(2) (1)(3)【分析】(1)从右边看到的图形有两层, 第一层有两个正方形, 第二层有 1 个正方形靠最左边; 从正面看到的图形有有两层, 第一层有两个正方形, 第二层有 1 个正方形靠最左边;(2)从右面看到的图形有两层,第一层有一个正方形,第二层有一个正方形; 从正面看到的图形有有两层, 第一层有三个正方形, 第二层有一个正方形靠中间; (3)从右边看到的图形有两层, 第一层有两个正方形, 第二层有 1 个正方形靠最左边;从正面看到的图形有有两层,第一层有两个正方形,第二层有 1 个正方形靠最左边; 据此解答即可。【详解】由分析可知:从右边看是图 A 的有(1)(3); 从右边看是图 B 的有(2); 从正面看是图 A 的有(1)(3)。【点睛】本题考查从不同方向观察物体,明确从不同方向观察到的形状是解题的关键。"} {"id": "24115", "image": ["12163.jpg", "12164.jpg", "12165.jpg", "12166.jpg", "12166.jpg"], "answer": "解见详解\n\n【分析】从左往右, 图一从左面看是 2 层 3 个小正方形, 上层 1 个且居左, 下层 2 个; 从上面看是 2 层 3 个小正方形, 上层 2 个, 下层 1 个且居左;\n\n图二从左面看是 2 层 3 个小正方形, 上层 1 个且居右, 下层 2 个; 从上面看是 2 层 3 个小正方形, 上\n层 1 个且居右, 下层 2 个;\n\n图三从左面看是 2 层 3 个小正方形, 上层 1 个且居右, 下层 2 个; 从上面看是 2 层 3 个小正方形, 上层 2 个, 下层 1 个且居右;\n\n据此解答。\n\n\n\n【详解】\n\n【点睛】本题考查从不同的方向观察几何体, 培养学生的空间想象力。", "solution": "null", "level": "五年级", "question": "2022・海门市)哪个几何体符合要求?在对的括号里打“ $\\sqrt{ }$ ”。\n\n\n\n从左面看\n\n\n\n从上面看\n\n\n( )", "options": [], "subject": "立体几何学", "analysis": "解见详解\n\n【分析】从左往右, 图一从左面看是 2 层 3 个小正方形, 上层 1 个且居左, 下层 2 个; 从上面看是 2 层 3 个小正方形, 上层 2 个, 下层 1 个且居左;\n\n图二从左面看是 2 层 3 个小正方形, 上层 1 个且居右, 下层 2 个; 从上面看是 2 层 3 个小正方形, 上\n层 1 个且居右, 下层 2 个;\n\n图三从左面看是 2 层 3 个小正方形, 上层 1 个且居右, 下层 2 个; 从上面看是 2 层 3 个小正方形, 上层 2 个, 下层 1 个且居右;\n\n据此解答。\n\n\n\n【详解】\n\n【点睛】本题考查从不同的方向观察几何体, 培养学生的空间想象力。"} {"id": "24117", "image": ["12174.jpg", "12175.jpg", "12176.jpg", "12177.jpg", "12178.jpg", "12179.jpg", "12180.jpg", "12179.jpg", "12180.jpg"], "answer": "(1)(5)(2)(4)7【分析】从正面看是 $\\square$, 说明这个几何体是 1 层 3 列, 从正面能看到 3 个小正方形;从左面看是 $\\square$, 说明这个几何体是 1 层 3 排, 从左面能看到 3 个小正方形;从正面看到的形状和(3)一样, 先确定(3)从正面看到的形状, 再用 5 个小正方体搭一搭, 找出不同的搭法。【详解】从正面看是 $\\square$ 的有(1)(5), 从左面看是 $\\square$ 的有(2)(4), 如果用 5 个小正方体搭出来的几何体, 从正面看到的形状和(3)一样, 有 7 种搭法。从正面看到的形状和(3)一样:搭法如下:【点睛】本题考查根据部分视图确定几何体, 培养学生的观察能力。", "solution": "null", "level": "五年级", "question": "下面的几何体中, 从正面看是 $\\square$ 的有( ), 从左面看是 $\\square \\square$ 的有 ( ), 如果用 5 个小正方体搭出来的几何体, 从正面看到的形状和(3)一样, 有 $($ 种搭法。\n\n\n\n(1)\n\n\n\n(2)\n\n\n\n(3)\n\n\n\n(4)\n\n\n\n(5)", "options": [], "subject": "立体几何学", "analysis": "(1)(5)(2)(4)7【分析】从正面看是 $\\square$, 说明这个几何体是 1 层 3 列, 从正面能看到 3 个小正方形;从左面看是 $\\square$, 说明这个几何体是 1 层 3 排, 从左面能看到 3 个小正方形;从正面看到的形状和(3)一样, 先确定(3)从正面看到的形状, 再用 5 个小正方体搭一搭, 找出不同的搭法。【详解】从正面看是 $\\square$ 的有(1)(5), 从左面看是 $\\square$ 的有(2)(4), 如果用 5 个小正方体搭出来的几何体, 从正面看到的形状和(3)一样, 有 7 种搭法。从正面看到的形状和(3)一样:搭法如下:【点睛】本题考查根据部分视图确定几何体, 培养学生的观察能力。"} {"id": "24118", "image": ["12181.jpg", "12182.jpg", "12183.jpg", "12184.jpg", "12185.jpg", "12186.jpg", "12187.jpg", "12188.jpg", "12189.jpg", "12190.jpg", "12191.jpg", "12192.jpg", "12193.jpg", "12194.jpg", "12195.jpg", "12196.jpg", "12197.jpg", "12198.jpg", "12199.jpg", "12200.jpg", "12201.jpg", "12187.jpg", "12188.jpg", "12189.jpg", "12190.jpg", "12191.jpg", "12192.jpg", "12193.jpg", "12194.jpg", "12195.jpg", "12196.jpg", "12197.jpg", "12198.jpg", "12199.jpg", "12200.jpg", "12201.jpg"], "answer": "解(1)(1)(3)(5)(7)(8)\n\n(2)(1)(2)(5)(6)8\n\n【分析】从不同位置观察由小正方体拼摆成的物体的形状的方法: 从哪一位置观察物体, 就从哪一面数出小正方形的数量, 并确定形状。\n\n【详解】(1) 从正面看, (1)看到的是\n\n\n(2)看到的是\n\n\n(3)看到的是\n\n\n(4)看到的是\n\n\n\n(5)看到的是\n\n\n(6)看到的是\n\n\n(7)看到的是 $\\square$,\n(8)看到的是\n\n\n所以从正面看到的是 $\\square$ 的有(1)(3)(5)(7)。\n\n(2)从左面看,(1)看到的是\n\n\n(2)看到的是\n\n\n\n(3)看到的是\n\n\n\n(4)看到的是\n\n\n看到的是\n\n\n(6)看到的是\n\n\n(7)看到的是\n\n\n(8)看到的是\n\n\n所以从左面看到的图形是 $\\square$ 的有(1)(2)(5)(6)(8)。\n\n【点睛】从不同方向观察物体时, 观察到的图形可能相同, 也可能不同。", "solution": "null", "level": "五年级", "question": "下面是用小正方体搭建的一些几何体。\n\n\n\n(2)\n(5)\n\n\n(3)\n\n\n(7)\n\n\n(4)\n\n\n(8)\n\n(1)从正面看到的是 $\\square$ 的有( )(填序号);\n\n(2)从左面看到的图形是 $\\square$ 的有()(填序号)。", "options": [], "subject": "立体几何学", "analysis": "解(1)(1)(3)(5)(7)(8)\n\n(2)(1)(2)(5)(6)8\n\n【分析】从不同位置观察由小正方体拼摆成的物体的形状的方法: 从哪一位置观察物体, 就从哪一面数出小正方形的数量, 并确定形状。\n\n【详解】(1) 从正面看, (1)看到的是\n\n\n(2)看到的是\n\n\n(3)看到的是\n\n\n(4)看到的是\n\n\n\n(5)看到的是\n\n\n(6)看到的是\n\n\n(7)看到的是 $\\square$,\n(8)看到的是\n\n\n所以从正面看到的是 $\\square$ 的有(1)(3)(5)(7)。\n\n(2)从左面看,(1)看到的是\n\n\n(2)看到的是\n\n\n\n(3)看到的是\n\n\n\n(4)看到的是\n\n\n看到的是\n\n\n(6)看到的是\n\n\n(7)看到的是\n\n\n(8)看到的是\n\n\n所以从左面看到的图形是 $\\square$ 的有(1)(2)(5)(6)(8)。\n\n【点睛】从不同方向观察物体时, 观察到的图形可能相同, 也可能不同。"} {"id": "24119", "image": ["12202.jpg", "12203.jpg", "12204.jpg", "12205.jpg", "12206.jpg", "12207.jpg", "12208.jpg", "12209.jpg", "12207.jpg", "12208.jpg", "12209.jpg"], "answer": "解见详解\n\n【分析】当我们从某一角度观察一个实物时, 根据给出的从一个方向看到的形状图, 辨认从不同方向 (前面、侧面、上面)看到的物体的形状图。\n\n【详解】根据从左面看的图形, 判断 3 个几何体都符合;\n\n从上面看的图形, 前两个都不符合, 只有最后一个符合。\n\n由分析得,\n\n\n\n( )\n\n\n\n( )\n\n\n\n( $\\sqrt{2}$ )\n\n【点睛】本题主要考查观察物体, 关键是区分从不同方向观察的物体形状。", "solution": "null", "level": "五年级", "question": "哪个几何体符合要求? (在符合要求的几何体下打 $\\sqrt{ }$ )\n\n\n\n从左面看\n\n\n\n从上面看\n\n\n\n( )\n\n\n\n( )\n\n\n\n( )", "options": [], "subject": "立体几何学", "analysis": "解见详解\n\n【分析】当我们从某一角度观察一个实物时, 根据给出的从一个方向看到的形状图, 辨认从不同方向 (前面、侧面、上面)看到的物体的形状图。\n\n【详解】根据从左面看的图形, 判断 3 个几何体都符合;\n\n从上面看的图形, 前两个都不符合, 只有最后一个符合。\n\n由分析得,\n\n\n\n( )\n\n\n\n( )\n\n\n\n( $\\sqrt{2}$ )\n\n【点睛】本题主要考查观察物体, 关键是区分从不同方向观察的物体形状。"} {"id": "24121", "image": ["12214.jpg", "12215.jpg", "12216.jpg", "12217.jpg", "12218.jpg", "12219.jpg", "12220.jpg", "12221.jpg", "12222.jpg", "12223.jpg", "12224.jpg", "12225.jpg", "12226.jpg", "12227.jpg", "12228.jpg", "12220.jpg", "12221.jpg", "12222.jpg", "12223.jpg", "12224.jpg", "12225.jpg", "12226.jpg", "12227.jpg", "12228.jpg"], "answer": "解(1) (3) (4)\n\n(2) (1) (2)和(3)\n\n(3)上\n\n【分析】观察图形可知, (1)图形从正面看是\n\n| $\\square$ | |\n| :--- | :--- |\n| | |\n| 是 | |\n\n从左面看是 $\\qquad$从上面看是\n\n\n\n(2)图形从正面看是\n\n\n\n从左面看是 $\\square$ 题 $\\square$,从上面看 $\\square$;\n\n(3)图形从正面看是\n\n\n\n从左面看是\n\n\n从上面看是\n\n\n\n(4)图形从正面看是\n\n\n从左面看是 $\\square$,从上面看是 $\\qquad$据此回答问题。\n\n【详解】(1)根据分析可知, 从正面看是\n\n\n的积木是(3)号, 从左面是 $\\square$ 的积木是(4)号。\n\n(2)根据分析可知,从上面看是\n\n\n的积木是(1)号,从左面是\n\n\n的积木是(2)和(3)号。\n\n(3) 根据分析可知, (1)号积木从正面看和(2)号积木从上面看图形一样。\n\n【点睛】本题是考查从不同方向观察物体和几何图形, 是培养学生的观察能力。", "solution": "null", "level": "五年级", "question": "先观察下面的积木形状, 再填空。\n\n\n\n(1)\n\n\n(3)\n\n\n\n(4)\n\n(1)从正面看是㖇的木是(\n\n\n\n(2)从上面看是\n\n\n\n的积木是 (\n\n(3)(1)号积木从正面看和(2)号积木从(\n\n\n\n)面看图形一样。", "options": [], "subject": "立体几何学", "analysis": "解(1) (3) (4)\n\n(2) (1) (2)和(3)\n\n(3)上\n\n【分析】观察图形可知, (1)图形从正面看是\n\n| $\\square$ | |\n| :--- | :--- |\n| | |\n| 是 | |\n\n从左面看是 $\\qquad$从上面看是\n\n\n\n(2)图形从正面看是\n\n\n\n从左面看是 $\\square$ 题 $\\square$,从上面看 $\\square$;\n\n(3)图形从正面看是\n\n\n\n从左面看是\n\n\n从上面看是\n\n\n\n(4)图形从正面看是\n\n\n从左面看是 $\\square$,从上面看是 $\\qquad$据此回答问题。\n\n【详解】(1)根据分析可知, 从正面看是\n\n\n的积木是(3)号, 从左面是 $\\square$ 的积木是(4)号。\n\n(2)根据分析可知,从上面看是\n\n\n的积木是(1)号,从左面是\n\n\n的积木是(2)和(3)号。\n\n(3) 根据分析可知, (1)号积木从正面看和(2)号积木从上面看图形一样。\n\n【点睛】本题是考查从不同方向观察物体和几何图形, 是培养学生的观察能力。"} {"id": "24139", "image": ["12314.jpg", "12315.jpg", "12179.jpg", "12180.jpg", "12315.jpg", "12179.jpg", "12180.jpg"], "answer": "解4\n\n【分析】从上面看的形状可以确定底层摆法; 再通过正面和左面看到的形状确定第 2 层的摆法, 即可确定小正方体的个数, 据此分析。\n\n【详解】如图\n\n\n\n,是由 4 个小正方体组成的。\n【点睛】关键是具有一定的空间想象能力, 能根据三视图确定几何体的形状。\n10 ._详解\n(1)(5)\n(2)(4)\n7\n\n【分析】从正面看是 $\\square$, 说明这个几何体是 1 层 3 列, 从正面能看到 3 个小正方形;\n\n从左面看是 $\\square$, 说明这个几何体是 1 层 3 排, 从左面能看到 3 个小正方形;\n\n从正面看到的形状和(3)一样, 先确定(3)从正面看到的形状, 再用 5 个小正方体搭一搭, 找出不同的搭法。\n\n【详解】从正面看是 $\\square$ 的有(1)(5), 从左面看是 $\\square$ 的有(2)(4), 如果用 5 个小正方体搭出来的几何体, 从正面看到的形状和(3)一样, 有 7 种搭法。\n\n从正面看到的形状和(3)一样:\n\n\n\n搭法如下:\n\n\n【点睛】本题考查根据部分视图确定几何体, 培养学生的观察能力。", "solution": "null", "level": "五年级", "question": "这个立体图形是由( )个小正方体组成的。\n\n", "options": [], "subject": "立体几何学", "analysis": "解4\n\n【分析】从上面看的形状可以确定底层摆法; 再通过正面和左面看到的形状确定第 2 层的摆法, 即可确定小正方体的个数, 据此分析。\n\n【详解】如图\n\n\n\n,是由 4 个小正方体组成的。\n【点睛】关键是具有一定的空间想象能力, 能根据三视图确定几何体的形状。\n10 ._详解\n(1)(5)\n(2)(4)\n7\n\n【分析】从正面看是 $\\square$, 说明这个几何体是 1 层 3 列, 从正面能看到 3 个小正方形;\n\n从左面看是 $\\square$, 说明这个几何体是 1 层 3 排, 从左面能看到 3 个小正方形;\n\n从正面看到的形状和(3)一样, 先确定(3)从正面看到的形状, 再用 5 个小正方体搭一搭, 找出不同的搭法。\n\n【详解】从正面看是 $\\square$ 的有(1)(5), 从左面看是 $\\square$ 的有(2)(4), 如果用 5 个小正方体搭出来的几何体, 从正面看到的形状和(3)一样, 有 7 种搭法。\n\n从正面看到的形状和(3)一样:\n\n\n\n搭法如下:\n\n\n【点睛】本题考查根据部分视图确定几何体, 培养学生的观察能力。"} {"id": "24143", "image": ["12328.jpg", "12329.jpg", "12330.jpg", "12330.jpg"], "answer": "解9\n【分析】从正面看有 2 层, 下边 1 层 4 个小正方形, 上边从左数第二个位置 1 个小正方形; 从左面看有 2 层, 下边 1 层 2 个小正方形, 上边 1 层靠右 1 个小正方形。据此可以确定层数有 2 层, 上下 2 层,前后也是 2 层, 再根据遮挡关系, 尽可能的多用小正方体, 拼搭成符合题干观察形状的立体图形即可。\n\n\n体。\n\n【点睛】关键是具有一定的空间想象能力,可以画一画示意图。", "solution": "null", "level": "五年级", "question": "用一些同样大小的小正方体拼搭, 从正面看是\n\n\n\n, 从左面看是\n\n\n\n,最多可以用( ) 个小正方体。", "options": [], "subject": "立体几何学", "analysis": "解9\n【分析】从正面看有 2 层, 下边 1 层 4 个小正方形, 上边从左数第二个位置 1 个小正方形; 从左面看有 2 层, 下边 1 层 2 个小正方形, 上边 1 层靠右 1 个小正方形。据此可以确定层数有 2 层, 上下 2 层,前后也是 2 层, 再根据遮挡关系, 尽可能的多用小正方体, 拼搭成符合题干观察形状的立体图形即可。\n\n\n体。\n\n【点睛】关键是具有一定的空间想象能力,可以画一画示意图。"} {"id": "24144", "image": ["12331.jpg", "12332.jpg", "12333.jpg", "12334.jpg", "12332.jpg", "12333.jpg", "12334.jpg"], "answer": "解4\n\n【分析】从三个不同的位置观察这个几何体,看到的形状都是\n\n\n\n可以确定这个几何体的摆法\n\n如下:\n\n\n\n, 即可确定这个几何体用的小正方体的数量。\n\n【详解】根据分析得,\n\n\n\n搭这个几何体用了 4 个小正方体。\n\n【点睛】此题的解题关键是根据从不同位置观察得到的三视图来确定几何体的形状。", "solution": "null", "level": "五年级", "question": "酷酷搭建了一个几何体,从正面、上面和左面看到的形状都是\n\n\n他搭这个几何体用了 ( )个小正方体。", "options": [], "subject": "立体几何学", "analysis": "解4\n\n【分析】从三个不同的位置观察这个几何体,看到的形状都是\n\n\n\n可以确定这个几何体的摆法\n\n如下:\n\n\n\n, 即可确定这个几何体用的小正方体的数量。\n\n【详解】根据分析得,\n\n\n\n搭这个几何体用了 4 个小正方体。\n\n【点睛】此题的解题关键是根据从不同位置观察得到的三视图来确定几何体的形状。"} {"id": "24147", "image": ["12344.jpg", "12345.jpg", "12346.jpg", "12347.jpg", "12348.jpg"], "answer": "解(2)\n\n【分析】分别画出每个物体从正面和上面看到的图形, 然后再填空即可。\n\n(1)从正面看有 2 层, 下层 4 个, 上层 2 个靠右; 从上面看有 3 排, 1 排 2 个靠右, 1 排 4 个, 1 排 1 个,与中间 1 排右起的第 2 个对齐;\n\n(2)从正面看有 2 层, 下层 4 个, 上层 2 个靠左; 从上面看有 2 排, 1 排 3 个左齐, 1 排 4 个;\n\n(3)从正面看有 2 层, 下层 4 个, 上层 3 个靠左; 从上面看有 2 排, 每排 4 个。\n\n【详解】\n\n| 从正面看 | 从上面看 | |\n| :--- | :--- | :--- | :--- | :--- | :--- | :--- | :--- | :--- | :--- |\n\n因此, (2)符合要求。\n\n【点睛】此题考查的是根据三视图确定几何体, 熟练掌握对物体三视图的认识是解答此题的关键。", "solution": "null", "level": "五年级", "question": "一个几何体, 从正面看是\n\n\n\n从上面看是\n\n\n\n, 下列搭法中, (符合要求。\n\n\n\n(1)\n\n\n\n(2)\n\n\n\n(3)", "options": [], "subject": "立体几何学", "analysis": "解(2)\n\n【分析】分别画出每个物体从正面和上面看到的图形, 然后再填空即可。\n\n(1)从正面看有 2 层, 下层 4 个, 上层 2 个靠右; 从上面看有 3 排, 1 排 2 个靠右, 1 排 4 个, 1 排 1 个,与中间 1 排右起的第 2 个对齐;\n\n(2)从正面看有 2 层, 下层 4 个, 上层 2 个靠左; 从上面看有 2 排, 1 排 3 个左齐, 1 排 4 个;\n\n(3)从正面看有 2 层, 下层 4 个, 上层 3 个靠左; 从上面看有 2 排, 每排 4 个。\n\n【详解】\n\n| 从正面看 | 从上面看 | |\n| :--- | :--- | :--- | :--- | :--- | :--- | :--- | :--- | :--- | :--- |\n\n因此, (2)符合要求。\n\n【点睛】此题考查的是根据三视图确定几何体, 熟练掌握对物体三视图的认识是解答此题的关键。"} {"id": "24166", "image": ["12424.jpg", "12425.jpg", "12426.jpg", "12427.jpg", "12428.jpg", "12429.jpg", "12430.jpg", "12431.jpg", "12432.jpg", "12433.jpg", "12434.jpg", "12435.jpg", "12436.jpg", "12437.jpg", "12438.jpg", "12439.jpg", "12440.jpg", "12441.jpg", "12442.jpg", "12443.jpg", "12444.jpg", "12445.jpg", "12446.jpg", "12447.jpg", "12448.jpg", "12431.jpg", "12432.jpg", "12433.jpg", "12434.jpg", "12435.jpg", "12436.jpg", "12437.jpg", "12438.jpg", "12439.jpg", "12440.jpg", "12441.jpg", "12442.jpg", "12443.jpg", "12444.jpg", "12445.jpg", "12446.jpg", "12447.jpg", "12448.jpg"], "answer": "解见详解\n\n【分析】根据从不同方向观察几何体的方法, 仔细观察四个几何体, 利用画出的三视图判断哪个几何体符合条件即可。\n\n【详解】\n\n\n\n从正面看到的图形是\n\n\n\n,从上面看到的图形是\n\n\n\n\n\n从正面看到的图形是\n\n\n\n, 从上面看到的图形是\n\n\n\n, 从左面看\n\n从正面看到的图形是\n\n\n\n, 从上面看到的图形是\n\n\n\n, 从左面看\n\n\n从正面看到的图形是\n\n\n\n, 从上面看到的图形是\n\n\n\n, 从左\n面看到的图形是\n\n\n\n综上, 只有\n\n\n\n符合要求。\n\n\n( )\n\n\n$(\\sqrt{ })$\n\n\n( )\n\n\n( )\n\n【点睛】本题考查从不同的方向观察物体, 解答本题的关键是掌握根据物体三视图确定物体形状的方法。", "solution": "null", "level": "五年级", "question": "哪个几何体符合要求?在正确图形下面的括号里画“ $\\sqrt{ }$ ”。\n\n\n\n从正面看\n\n\n\n从上面看\n\n\n\n从左面看\n\n\n\n\n\n( )\n\n\n\n( )\n\n\n\n( )", "options": [], "subject": "立体几何学", "analysis": "解见详解\n\n【分析】根据从不同方向观察几何体的方法, 仔细观察四个几何体, 利用画出的三视图判断哪个几何体符合条件即可。\n\n【详解】\n\n\n\n从正面看到的图形是\n\n\n\n,从上面看到的图形是\n\n\n\n\n\n从正面看到的图形是\n\n\n\n, 从上面看到的图形是\n\n\n\n, 从左面看\n\n从正面看到的图形是\n\n\n\n, 从上面看到的图形是\n\n\n\n, 从左面看\n\n\n从正面看到的图形是\n\n\n\n, 从上面看到的图形是\n\n\n\n, 从左\n面看到的图形是\n\n\n\n综上, 只有\n\n\n\n符合要求。\n\n\n( )\n\n\n$(\\sqrt{ })$\n\n\n( )\n\n\n( )\n\n【点睛】本题考查从不同的方向观察物体, 解答本题的关键是掌握根据物体三视图确定物体形状的方法。"} {"id": "24451", "image": [], "answer": "解$6 \\quad 292 \\quad 336$\n\n【分析】长方体的高 $=$ 棱长总和 $\\div 4$ 一长一宽, 长方体表面积 $=($ 长 $\\times$ 宽 + 长 $\\times$ 高 + 宽 $\\times$ 高 $) \\times 2$, 长方体体积 $=$ 长 $\\times$ 宽 $\\times$ 高, 据此列式计算。\n\n【详解】 $84 \\div 4-8-7$\n\n$=21-8-7$\n\n$=6(\\mathrm{~cm})$\n\n$(8 \\times 7+8 \\times 6+7 \\times 6) \\times 2$\n\n$=(56+48+42) \\times 2$\n\n$=146 \\times 2$\n\n$=292\\left(\\mathrm{~cm}^{2}\\right)$\n\n$8 \\times 7 \\times 6=336\\left(\\mathrm{dm}^{3}\\right)$\n\n一个长方体棱长之和是 $84 \\mathrm{~cm}$, 它的长是 $8 \\mathrm{~cm}$, 宽是 $7 \\mathrm{~cm}$, 高是 $6 \\mathrm{~cm}$, 它的表面积是 $292 \\mathrm{~cm}^{2}$, 体积是 $336 \\mathrm{dm}^{3}$ 。\n\n【点睛】关键是熟悉长方体特征, 掌握并灵活运用长方体表面积和体积公式。", "solution": "null", "level": "五年级", "question": "一个长方体棱长之和是 $84 \\mathrm{~cm}$, 它的长是 $8 \\mathrm{~cm}$, 宽是 $7 \\mathrm{~cm}$, 高是 $(\\quad) \\mathrm{cm}$, 它的表面积是\n\n( ) $\\mathrm{cm}^{2}$, 体积是 $(\\quad) \\mathrm{dm}^{3}$ 。", "options": [], "subject": "立体几何学", "analysis": "解$6 \\quad 292 \\quad 336$\n\n【分析】长方体的高 $=$ 棱长总和 $\\div 4$ 一长一宽, 长方体表面积 $=($ 长 $\\times$ 宽 + 长 $\\times$ 高 + 宽 $\\times$ 高 $) \\times 2$, 长方体体积 $=$ 长 $\\times$ 宽 $\\times$ 高, 据此列式计算。\n\n【详解】 $84 \\div 4-8-7$\n\n$=21-8-7$\n\n$=6(\\mathrm{~cm})$\n\n$(8 \\times 7+8 \\times 6+7 \\times 6) \\times 2$\n\n$=(56+48+42) \\times 2$\n\n$=146 \\times 2$\n\n$=292\\left(\\mathrm{~cm}^{2}\\right)$\n\n$8 \\times 7 \\times 6=336\\left(\\mathrm{dm}^{3}\\right)$\n\n一个长方体棱长之和是 $84 \\mathrm{~cm}$, 它的长是 $8 \\mathrm{~cm}$, 宽是 $7 \\mathrm{~cm}$, 高是 $6 \\mathrm{~cm}$, 它的表面积是 $292 \\mathrm{~cm}^{2}$, 体积是 $336 \\mathrm{dm}^{3}$ 。\n\n【点睛】关键是熟悉长方体特征, 掌握并灵活运用长方体表面积和体积公式。"} {"id": "24452", "image": [], "answer": "解6\n\n【分析】石头完全浸没在水里后, 石头的体积=水面上升的体积, 水面上升的体积可看作长为 75 厘米, 宽为 40 厘米, 高为 2 厘米的长方体的体积, 根据长方体的体积公式, 把数据代入即可得解。\n\n【详解】 $75 \\times 40 \\times 2$\n\n$=75 \\times 80$\n\n$=6000$ (立方厘米)\n\n6000 立方厘米 $=6$ 立方分米\n即这块石头的体积是立方分米。\n\n【点睛】此题的解题关键是掌握不规则物体的体积的计算方法, 通过转化的数学思想, 灵活运用长方体的体积公式, 解决问题。", "solution": "null", "level": "五年级", "question": "在一个长 75 厘米, 宽 40 厘米, 高 50 厘米长方体鱼缸内放入一块石头, 水面上升了 2 厘米。这\n块石头的体积是 ( ) 立方分米。", "options": [], "subject": "立体几何学", "analysis": "解6\n\n【分析】石头完全浸没在水里后, 石头的体积=水面上升的体积, 水面上升的体积可看作长为 75 厘米, 宽为 40 厘米, 高为 2 厘米的长方体的体积, 根据长方体的体积公式, 把数据代入即可得解。\n\n【详解】 $75 \\times 40 \\times 2$\n\n$=75 \\times 80$\n\n$=6000$ (立方厘米)\n\n6000 立方厘米 $=6$ 立方分米\n即这块石头的体积是立方分米。\n\n【点睛】此题的解题关键是掌握不规则物体的体积的计算方法, 通过转化的数学思想, 灵活运用长方体的体积公式, 解决问题。"} {"id": "24453", "image": [], "answer": "解16\n\n【分析】这根长方体木料长 2 米, 宽和高都是 2 分米, 要求的至少增加的面积, 可沿着 $2 \\times 2$ 的截面来锯; 长方体木材锯一次则增加两个面的面积, 根据题意可知长方体木材锯了两次, 增加了四个面的面积。\n\n【详解】 $2 \\times 2 \\times 4$\n\n$=4 \\times 4$\n\n$=16$ (平方分米)\n\n【点睛】此题考查了长方体的表面积, 解答本题的关键是掌握长方体表面积的计算公式。", "solution": "null", "level": "五年级", "question": "一根长方体木料长 2 米, 宽和高都是 2 分米, 把它锯成 3 段, 表面积至少增加 ( )平方分米。", "options": [], "subject": "立体几何学", "analysis": "解16\n\n【分析】这根长方体木料长 2 米, 宽和高都是 2 分米, 要求的至少增加的面积, 可沿着 $2 \\times 2$ 的截面来锯; 长方体木材锯一次则增加两个面的面积, 根据题意可知长方体木材锯了两次, 增加了四个面的面积。\n\n【详解】 $2 \\times 2 \\times 4$\n\n$=4 \\times 4$\n\n$=16$ (平方分米)\n\n【点睛】此题考查了长方体的表面积, 解答本题的关键是掌握长方体表面积的计算公式。"} {"id": "24454", "image": [], "answer": "解2 分米\\#\\#2dm\n\n【分析】根据正方体体积 $=$ 棱长 $\\times$ 棱长 $\\times$ 棱长, 求出钢材体积, 即长方体体积, 再根据长方体的高 $=$ 体积 $\\div$ 底面积, 列式计算即可。\n\n【详解】 $4 \\times 4 \\times 4 \\div(8 \\times 4)$\n\n$=64 \\div 32$\n\n$=2(\\mathrm{dm})$\n\n这个长方体的高是 $2 \\mathrm{dm}$ 。\n\n【点睛】关键是掌握并灵活运用正方体和长方体体积公式。", "solution": "null", "level": "五年级", "question": "把棱长为 $4 \\mathrm{dm}$ 的正方体钢材锻造成一块长是 $8 \\mathrm{dm}$, 宽是 $4 \\mathrm{dm}$ 的长方体。那么长方体的高是\n\n( )。", "options": [], "subject": "立体几何学", "analysis": "解2 分米\\#\\#2dm\n\n【分析】根据正方体体积 $=$ 棱长 $\\times$ 棱长 $\\times$ 棱长, 求出钢材体积, 即长方体体积, 再根据长方体的高 $=$ 体积 $\\div$ 底面积, 列式计算即可。\n\n【详解】 $4 \\times 4 \\times 4 \\div(8 \\times 4)$\n\n$=64 \\div 32$\n\n$=2(\\mathrm{dm})$\n\n这个长方体的高是 $2 \\mathrm{dm}$ 。\n\n【点睛】关键是掌握并灵活运用正方体和长方体体积公式。"} {"id": "24478", "image": ["12500.jpg", "12501.jpg", "12501.jpg"], "answer": "解毫升\\#\\#mL 立方米\\#\\#m³ 立方厘米\\#\\# $\\mathrm{cm}^{3}$\n\n【分析】根据生活经验、对体积单位、容积单位和数据大小的认识,可知计量牛奶盒的容积应用“毫升\"作单位; 计量车厢的体积应用“立方米”作单位; 计量橡皮的体积应用“立方厘米”作单位; 据此填空即可。\n\n【详解】由分析可知:\n\n牛奶盒的容积约是 250 毫升。\n\n\n\n橡皮的体积约是 8 立方厘米。\n\n【点睛】此题考查根据情景选择合适的计量单位, 要注意联系生活实际、计量单位和数据的大小, 灵活的选择。", "solution": "null", "level": "五年级", "question": "在括号里填上适当的体积单位或容积单位。\n\n牛奶盒的容积约是 250 ( )。\n\n\n\n橡皮的体积约是 8( )。", "options": [], "subject": "立体几何学", "analysis": "解毫升\\#\\#mL 立方米\\#\\#m³ 立方厘米\\#\\# $\\mathrm{cm}^{3}$\n\n【分析】根据生活经验、对体积单位、容积单位和数据大小的认识,可知计量牛奶盒的容积应用“毫升\"作单位; 计量车厢的体积应用“立方米”作单位; 计量橡皮的体积应用“立方厘米”作单位; 据此填空即可。\n\n【详解】由分析可知:\n\n牛奶盒的容积约是 250 毫升。\n\n\n\n橡皮的体积约是 8 立方厘米。\n\n【点睛】此题考查根据情景选择合适的计量单位, 要注意联系生活实际、计量单位和数据的大小, 灵活的选择。"} {"id": "24479", "image": [], "answer": "解6144\n\n【分析】根据长方体的体积公式: $\\mathrm{V}=\\mathrm{abh}$, 把数据代入到公式中, 求出砖墙的体积, 再乘每立方米用砖的数量, 即可求出一共需要砖的数量。\n\n【详解】 $20 \\times 0.24 \\times 8=38.4$ (立方米)\n\n$38.4 \\times 160=6144$ (块)\n\n即一共需要 6144 块砖。\n\n【点睛】此题的解题关键是灵活运用长方体的体积公式求解。", "solution": "null", "level": "五年级", "question": "清水河小区要砌一道长 20 米、厚 0.24 米、高 8 米的砖墙, 如果每立方米用砖 160 块, 一共需要砖( )块。", "options": [], "subject": "立体几何学", "analysis": "解6144\n\n【分析】根据长方体的体积公式: $\\mathrm{V}=\\mathrm{abh}$, 把数据代入到公式中, 求出砖墙的体积, 再乘每立方米用砖的数量, 即可求出一共需要砖的数量。\n\n【详解】 $20 \\times 0.24 \\times 8=38.4$ (立方米)\n\n$38.4 \\times 160=6144$ (块)\n\n即一共需要 6144 块砖。\n\n【点睛】此题的解题关键是灵活运用长方体的体积公式求解。"} {"id": "24480", "image": [], "answer": "解96\n\n【分析】根据正方体的棱长总和 $=$ 棱长 $\\times 12$, 已知正方体棱长是 8 厘米, 代入到公式中, 即可求出正方体的棱长总和。\n\n【详解】 $12 \\times 8=96$ (厘米)\n\n即正方体的棱长总和是 96 厘米。\n\n【点睛】此题的解题关键是灵活运用正方体的棱长总和公式求解。", "solution": "null", "level": "五年级", "question": "有一个正方体棱长是 8 厘米, 它的棱长总和是( )厘米。", "options": [], "subject": "立体几何学", "analysis": "解96\n\n【分析】根据正方体的棱长总和 $=$ 棱长 $\\times 12$, 已知正方体棱长是 8 厘米, 代入到公式中, 即可求出正方体的棱长总和。\n\n【详解】 $12 \\times 8=96$ (厘米)\n\n即正方体的棱长总和是 96 厘米。\n\n【点睛】此题的解题关键是灵活运用正方体的棱长总和公式求解。"} {"id": "24481", "image": [], "answer": "解126\n\n【分析】根据长方体的体积公式: $\\mathrm{V}=\\mathrm{abh}$, 已知长方体的长 7 厘米, 宽 4.5 厘米,高 4 厘米,代入到\n公式中, 即可求出长方体的体积。\n\n【详解】 $7 \\times 4.5 \\times 4$\n\n$=31.5 \\times 4$\n\n$=126$ (立方厘米)\n\n即它的体积是 126 立方厘米。\n\n【点睛】此题的解题关键是根据长方体的体积公式解决问题。", "solution": "null", "level": "五年级", "question": "一个长方体木块, 长 7 厘米,宽 4.5 厘米,高 4 厘米,它的体积是 ( )立方厘米。", "options": [], "subject": "立体几何学", "analysis": "解126\n\n【分析】根据长方体的体积公式: $\\mathrm{V}=\\mathrm{abh}$, 已知长方体的长 7 厘米, 宽 4.5 厘米,高 4 厘米,代入到\n公式中, 即可求出长方体的体积。\n\n【详解】 $7 \\times 4.5 \\times 4$\n\n$=31.5 \\times 4$\n\n$=126$ (立方厘米)\n\n即它的体积是 126 立方厘米。\n\n【点睛】此题的解题关键是根据长方体的体积公式解决问题。"} {"id": "24506", "image": [], "answer": "解$50 \\quad 74 \\quad 60$\n\n【分析】长方体棱长总和 $=($ 长 + 宽 + 高 $) \\times 4$, 鱼缸表面积 $=$ 长 $\\times$ 宽 + 长 $\\times$ 高 $\\times 2+$ 宽 $\\times$ 高 $\\times 2$, 长方体体积 $=$ 长 $\\times$ 宽 $\\times$ 高, 据此列式计算。\n\n【详解】 $(6+4+2.5) \\times 4$\n\n$=12.5 \\times 4$\n\n$=50(\\mathrm{dm})$\n\n$6 \\times 4+6 \\times 2.5 \\times 2+4 \\times 2.5 \\times 2$\n\n$=24+30+20$\n\n$=74\\left(\\mathrm{dm}^{2}\\right)$\n\n$6 \\times 4 \\times 2.5=60\\left(\\mathrm{dm}^{3}\\right)=60(\\mathrm{~L})$\n\n至少要 $50 \\mathrm{dm}$ 的角钢, 至少要玻璃 $74 \\mathrm{dm}^{2}$, 最多可盛水 $60 \\mathrm{~L}$ 。\n\n【点睛】关键是熟悉长方体特征,掌握并灵活运用长方体表面积和体积公式。", "solution": "null", "level": "五年级", "question": "做一个长 $6 \\mathrm{dm}$, 宽 $4 \\mathrm{dm}$, 高 $2.5 \\mathrm{dm}$ 的无盖鱼缸, 用角钢做它的框架, 至少要 $($ dm 的角钢, 至少要玻璃( ) dm², 最多可盛水( )L。", "options": [], "subject": "立体几何学", "analysis": "解$50 \\quad 74 \\quad 60$\n\n【分析】长方体棱长总和 $=($ 长 + 宽 + 高 $) \\times 4$, 鱼缸表面积 $=$ 长 $\\times$ 宽 + 长 $\\times$ 高 $\\times 2+$ 宽 $\\times$ 高 $\\times 2$, 长方体体积 $=$ 长 $\\times$ 宽 $\\times$ 高, 据此列式计算。\n\n【详解】 $(6+4+2.5) \\times 4$\n\n$=12.5 \\times 4$\n\n$=50(\\mathrm{dm})$\n\n$6 \\times 4+6 \\times 2.5 \\times 2+4 \\times 2.5 \\times 2$\n\n$=24+30+20$\n\n$=74\\left(\\mathrm{dm}^{2}\\right)$\n\n$6 \\times 4 \\times 2.5=60\\left(\\mathrm{dm}^{3}\\right)=60(\\mathrm{~L})$\n\n至少要 $50 \\mathrm{dm}$ 的角钢, 至少要玻璃 $74 \\mathrm{dm}^{2}$, 最多可盛水 $60 \\mathrm{~L}$ 。\n\n【点睛】关键是熟悉长方体特征,掌握并灵活运用长方体表面积和体积公式。"} {"id": "24507", "image": [], "answer": "解125\n\n【分析】根据题意可知, 从这个长方体木块上切下一个最大的正方体, 这个最大的正方体的棱长等于长方体的高, 正方体的体积公式是 $\\mathrm{V}=\\mathrm{a}^{3}$, 直接列式解答。\n\n【详解】 $5 \\times 5 \\times 5$\n\n$=25 \\times 5$\n\n$=125$ (立方分米)\n\n即这个正方体的的体积是 125 立方分米。\n\n【点睛】此题主要考查正方体体积公式的灵活运用, 关键是熟记公式。", "solution": "null", "level": "五年级", "question": "把一个长 8 分米, 高 6 分米, 宽 5 分米的长方体木块削出一个最大的正方体, 这个正方体的的体积是 ( ) 立方分米。", "options": [], "subject": "立体几何学", "analysis": "解125\n\n【分析】根据题意可知, 从这个长方体木块上切下一个最大的正方体, 这个最大的正方体的棱长等于长方体的高, 正方体的体积公式是 $\\mathrm{V}=\\mathrm{a}^{3}$, 直接列式解答。\n\n【详解】 $5 \\times 5 \\times 5$\n\n$=25 \\times 5$\n\n$=125$ (立方分米)\n\n即这个正方体的的体积是 125 立方分米。\n\n【点睛】此题主要考查正方体体积公式的灵活运用, 关键是熟记公式。"} {"id": "24508", "image": [], "answer": "解48\n\n【分析】正方体表面积 $=$ 棱长 $\\times$ 棱长 $\\times 6$, 正方体体积 $=$ 棱长 $\\times$ 棱长 $\\times$ 棱长, 正方体的棱长扩大到原来的几倍, 表面积扩大到原来的倍数 $\\times$ 倍数, 体积扩大到原来的倍数 $\\times$ 倍数 $\\times$ 倍数, 据此分析。\n\n【详解】 $2 \\times 2=4$\n\n$2 \\times 2 \\times 2=8$\n\n一个正方体的棱长扩大到原来的 2 倍, 表面积就扩大到原来 4 倍, 体积扩大到原来的 8 倍。\n\n【点睛】关键是掌握并灵活运用正方体表面积和体积公式, 根据积的变化规律进行分析。", "solution": "null", "level": "五年级", "question": "一个正方体的棱长扩大到原来的 2 倍, 表面积就扩大到原来(\n\n)倍,体积扩大到原来的\n\n( )倍。", "options": [], "subject": "立体几何学", "analysis": "解48\n\n【分析】正方体表面积 $=$ 棱长 $\\times$ 棱长 $\\times 6$, 正方体体积 $=$ 棱长 $\\times$ 棱长 $\\times$ 棱长, 正方体的棱长扩大到原来的几倍, 表面积扩大到原来的倍数 $\\times$ 倍数, 体积扩大到原来的倍数 $\\times$ 倍数 $\\times$ 倍数, 据此分析。\n\n【详解】 $2 \\times 2=4$\n\n$2 \\times 2 \\times 2=8$\n\n一个正方体的棱长扩大到原来的 2 倍, 表面积就扩大到原来 4 倍, 体积扩大到原来的 8 倍。\n\n【点睛】关键是掌握并灵活运用正方体表面积和体积公式, 根据积的变化规律进行分析。"} {"id": "24509", "image": ["12514.jpg"], "answer": "解4\n【分析】由 4 个棱长 1 厘米的正方体拼成长方体后, 长方体的长为 $(1 \\times 4)$ 厘米, 宽和高为 1 厘米,根据长方体的体积公式: $V=\\mathrm{abh}$, 代入数据即可求出长方体的体积。\n\n【详解】 $1 \\times 4=4$ (厘米)\n\n$4 \\times 1 \\times 1=4$ (立方厘米)\n\n即长方体的体积是 4 立方厘米。\n\n【点睛】本题考查立体图形的拼接, 灵活运用长方体的体积公式是解题的关键。", "solution": "null", "level": "五年级", "question": "如图是 4 个由棱长 1 厘米的正方体拼成的长方体,它的体积是 $($ )立方厘米。\n\n", "options": [], "subject": "立体几何学", "analysis": "解4\n【分析】由 4 个棱长 1 厘米的正方体拼成长方体后, 长方体的长为 $(1 \\times 4)$ 厘米, 宽和高为 1 厘米,根据长方体的体积公式: $V=\\mathrm{abh}$, 代入数据即可求出长方体的体积。\n\n【详解】 $1 \\times 4=4$ (厘米)\n\n$4 \\times 1 \\times 1=4$ (立方厘米)\n\n即长方体的体积是 4 立方厘米。\n\n【点睛】本题考查立体图形的拼接, 灵活运用长方体的体积公式是解题的关键。"} {"id": "24510", "image": [], "answer": "解3\n\n【分析】根据正方体的体积 $=$ 棱长 $\\times$ 棱长 $\\times$ 棱长, 代入数据求出正方体钢坏的体积, 锻造成一个占地面积是 72 平方分米的长方体后, 体积不变, 利用长方体的体积 $=$ 底面积 $\\times$ 高, 用体积除以 72 即可求出这个长方体的高。\n\n【详解】 $6 \\times 6 \\times 6 \\div 72$\n\n$=216 \\div 72$\n\n$=3$ (分米)\n\n即这个长方体的高是 3 分米。\n\n【点睛】此题主要根据等积变形, 灵活运用正方体和长方体的体积公式求解。", "solution": "null", "level": "五年级", "question": "把一个棱长 6 分米的正方体钢坏锻造成一个占地面积是 72 平方分米的长方体, 这个长方体的高是( )分米。", "options": [], "subject": "立体几何学", "analysis": "解3\n\n【分析】根据正方体的体积 $=$ 棱长 $\\times$ 棱长 $\\times$ 棱长, 代入数据求出正方体钢坏的体积, 锻造成一个占地面积是 72 平方分米的长方体后, 体积不变, 利用长方体的体积 $=$ 底面积 $\\times$ 高, 用体积除以 72 即可求出这个长方体的高。\n\n【详解】 $6 \\times 6 \\times 6 \\div 72$\n\n$=216 \\div 72$\n\n$=3$ (分米)\n\n即这个长方体的高是 3 分米。\n\n【点睛】此题主要根据等积变形, 灵活运用正方体和长方体的体积公式求解。"} {"id": "24511", "image": [], "answer": "解3\n\n【分析】由长方体体积: $V=S h$ 可知: $h=V \\div S$, 将数值代入即可求得长方体木箱的高。据此解答。\n\n【详解】 $27 \\div 9=3$ (分米)\n\n高是 3 分米。\n\n【点睛】本题考查了长方体体积公式的灵活运用。已知体积和底面积, 求高, 用体积除以底面积即可。", "solution": "null", "level": "五年级", "question": "一个长方体木箱的体积是 27 立方分米,它的底面积是 9 平方分米,高是(\n\n)分米。", "options": [], "subject": "立体几何学", "analysis": "解3\n\n【分析】由长方体体积: $V=S h$ 可知: $h=V \\div S$, 将数值代入即可求得长方体木箱的高。据此解答。\n\n【详解】 $27 \\div 9=3$ (分米)\n\n高是 3 分米。\n\n【点睛】本题考查了长方体体积公式的灵活运用。已知体积和底面积, 求高, 用体积除以底面积即可。"} {"id": "24533", "image": [], "answer": "解396\n\n【分析】根据长方体的特点, 相对的面面积相等,从上部截去高 5 厘米的长方体后剩下是正方体, 可知底面是正方形, 表面积减少了 120 平方厘米, 减少的只是前后左右的 4 个相等侧面积, 用 $120 \\div 4=$ 30 厘米, 求出减少的一个侧面, 再除以截去部分的高, 就可以求出剩下部分正方体的棱长, 据此求出底面积, 再用底面棱长加上截去的高, 就是原来长方体的高, 据此解答。\n\n【详解】由题意可知底面是正方形, 底面边长为:\n\n$120 \\div 4 \\div 5$\n\n$=30 \\div 5$\n\n$=6$ (厘米)\n\n原来的体积:\n\n$6 \\times 6 \\times(5+6)$\n\n$=36 \\times 11$\n\n$=396$ (立方厘米)\n\n【点睛】本题考查长方体的体积计算, 解答的关键是理解表面积减少的只是侧面积, 只要求出剩下部分正方体的棱长, 再根据长方体体积公式, 即可解答。", "solution": "null", "level": "五年级", "question": "一条长方体木块, 从上部截去高 5 厘米的长方体, 剩下部分是正方体。表面积减少了 120 平方厘米, 那么原来长方体的体积是( ) 立方厘米。", "options": [], "subject": "立体几何学", "analysis": "解396\n\n【分析】根据长方体的特点, 相对的面面积相等,从上部截去高 5 厘米的长方体后剩下是正方体, 可知底面是正方形, 表面积减少了 120 平方厘米, 减少的只是前后左右的 4 个相等侧面积, 用 $120 \\div 4=$ 30 厘米, 求出减少的一个侧面, 再除以截去部分的高, 就可以求出剩下部分正方体的棱长, 据此求出底面积, 再用底面棱长加上截去的高, 就是原来长方体的高, 据此解答。\n\n【详解】由题意可知底面是正方形, 底面边长为:\n\n$120 \\div 4 \\div 5$\n\n$=30 \\div 5$\n\n$=6$ (厘米)\n\n原来的体积:\n\n$6 \\times 6 \\times(5+6)$\n\n$=36 \\times 11$\n\n$=396$ (立方厘米)\n\n【点睛】本题考查长方体的体积计算, 解答的关键是理解表面积减少的只是侧面积, 只要求出剩下部分正方体的棱长, 再根据长方体体积公式, 即可解答。"} {"id": "24535", "image": ["12533.jpg", "12534.jpg"], "answer": "解大于 等于\n\n【分析】物体所占空间的大小就是体积;乙比甲少了一小块正方体,所以甲的体积大于乙的体积;乙比完整的正方体少了 3 个面但又增加了 3 个面的面积,所以乙的表面积与甲的表面积相等。\n\n【详解】由分析可知:\n甲的体积大于乙的体积,甲的表面积等于乙的表面积。\n\n【点睛】本题考查体积和表面积, 明确体积和表面积的定义是解题的关键。", "solution": "null", "level": "五年级", "question": "下图中, 甲的体积 ( )乙的体积, 甲的表面积( )乙的表面积。(图中所有小正方体大小一样)\n\n\n甲\n\n\n乙", "options": [], "subject": "立体几何学", "analysis": "解大于 等于\n\n【分析】物体所占空间的大小就是体积;乙比甲少了一小块正方体,所以甲的体积大于乙的体积;乙比完整的正方体少了 3 个面但又增加了 3 个面的面积,所以乙的表面积与甲的表面积相等。\n\n【详解】由分析可知:\n甲的体积大于乙的体积,甲的表面积等于乙的表面积。\n\n【点睛】本题考查体积和表面积, 明确体积和表面积的定义是解题的关键。"} {"id": "24536", "image": [], "answer": "解$64 \\quad 192$\n\n【分析】把三个棱长都是 4 厘米的正方体拼成一个长方体, 那么减少了 4 个面, 算出这 4 个面的面积即可; 求拼成的体积,根据正方体的体积公式,只要求出一个正方体的体积,乘 3 即可得出体积。\n\n【详解】 $4 \\times 4 \\times 4$\n\n$=16 \\times 4$\n\n$=64$ (平方厘米)\n\n$4 \\times 4 \\times 4 \\times 3$\n\n$=64 \\times 3$\n\n$=192$ (立方厘米)\n\n表面积减少了 64 平方厘米,拼成的长方体的体积是 192 立方厘米。\n\n【点睛】此题考查了立体图形的切拼,正方体的表面积以及体积的计算,同时考查了学生的空间想象力。", "solution": "null", "level": "五年级", "question": "把三个棱长都是 4 厘米的正方体拼成一个长方体,表面积减少了( )平方厘米,它的体积是( ) 立方厘米。", "options": [], "subject": "立体几何学", "analysis": "解$64 \\quad 192$\n\n【分析】把三个棱长都是 4 厘米的正方体拼成一个长方体, 那么减少了 4 个面, 算出这 4 个面的面积即可; 求拼成的体积,根据正方体的体积公式,只要求出一个正方体的体积,乘 3 即可得出体积。\n\n【详解】 $4 \\times 4 \\times 4$\n\n$=16 \\times 4$\n\n$=64$ (平方厘米)\n\n$4 \\times 4 \\times 4 \\times 3$\n\n$=64 \\times 3$\n\n$=192$ (立方厘米)\n\n表面积减少了 64 平方厘米,拼成的长方体的体积是 192 立方厘米。\n\n【点睛】此题考查了立体图形的切拼,正方体的表面积以及体积的计算,同时考查了学生的空间想象力。"} {"id": "24555", "image": [], "answer": "解263400\n\n【分析】(1)首先明确单位间的进率; (2) 再根据低级单位化高级单位除以进率, 高级单位化低级单位乘进率, 来进行单位换算。\n\n【详解】(1) $1 \\mathrm{dm}^{3}=1000 \\mathrm{~cm}^{3} ; 26000 \\div 1000=26$, 所以 $26000 \\mathrm{~cm}^{3}=26 \\mathrm{dm}^{3}$;\n\n(2) $1 \\mathrm{~L}=1000 \\mathrm{~mL}, 3.4 \\times 1000=3400$, 所以 $3.4 \\mathrm{~L}=3400 \\mathrm{~mL}$ 。\n\n【点睛】只有相邻的两个体积单位之间的进率才是 1000。判断和互化时首先要看这两个体积单位是不是相邻的。", "solution": "null", "level": "五年级", "question": "$26000 \\mathrm{~cm}^{3}=($\n) $\\mathrm{dm}^{3} \\quad 3.4 \\mathrm{~L}=($\n) $\\mathrm{mL}$", "options": [], "subject": "立体几何学", "analysis": "解263400\n\n【分析】(1)首先明确单位间的进率; (2) 再根据低级单位化高级单位除以进率, 高级单位化低级单位乘进率, 来进行单位换算。\n\n【详解】(1) $1 \\mathrm{dm}^{3}=1000 \\mathrm{~cm}^{3} ; 26000 \\div 1000=26$, 所以 $26000 \\mathrm{~cm}^{3}=26 \\mathrm{dm}^{3}$;\n\n(2) $1 \\mathrm{~L}=1000 \\mathrm{~mL}, 3.4 \\times 1000=3400$, 所以 $3.4 \\mathrm{~L}=3400 \\mathrm{~mL}$ 。\n\n【点睛】只有相邻的两个体积单位之间的进率才是 1000。判断和互化时首先要看这两个体积单位是不是相邻的。"} {"id": "24556", "image": [], "answer": "解8.556\n\n【分析】鱼缸的容积需要从里面进行测量, 从里面量得鱼缸的长是 $32-0.5 \\times 2=31$ 厘米, 宽是 $24-0.5 \\times 2$ $=23$ 厘米, 高是 $12.5-0.5=12$ 厘米, 根据长方体的容积公式: $\\mathrm{V}=\\mathrm{abh}$, 据此进行计算即可。\n\n【详解】 $(32-0.5 \\times 2) \\times(24-0.5 \\times 2) \\times(12.5-0.5)$\n$=31 \\times 23 \\times 12$\n\n$=713 \\times 12$\n\n$=8556$ (立方厘米)\n\n$=8.556$ (升)\n\n则这个鱼缸的容积是 8.556 升。\n\n【点睛】本题考查长方体的容积, 明确容积的计算是从里面进行测量的是解题的关键。", "solution": "null", "level": "五年级", "question": "李爷爷用厚 0.5 厘米的玻璃板自制了一个无盖的鱼缸。鱼缸从外面量长 32 厘米, 宽 24 厘米, 高 12.5 厘米,这个鱼缸的容积是( ) 升。", "options": [], "subject": "立体几何学", "analysis": "解8.556\n\n【分析】鱼缸的容积需要从里面进行测量, 从里面量得鱼缸的长是 $32-0.5 \\times 2=31$ 厘米, 宽是 $24-0.5 \\times 2$ $=23$ 厘米, 高是 $12.5-0.5=12$ 厘米, 根据长方体的容积公式: $\\mathrm{V}=\\mathrm{abh}$, 据此进行计算即可。\n\n【详解】 $(32-0.5 \\times 2) \\times(24-0.5 \\times 2) \\times(12.5-0.5)$\n$=31 \\times 23 \\times 12$\n\n$=713 \\times 12$\n\n$=8556$ (立方厘米)\n\n$=8.556$ (升)\n\n则这个鱼缸的容积是 8.556 升。\n\n【点睛】本题考查长方体的容积, 明确容积的计算是从里面进行测量的是解题的关键。"} {"id": "24557", "image": ["12548.jpg"], "answer": "解$4 \\quad 7$\n\n【分析】找出正方体展开图中三个数字的相对面, 找相对面时, 先找同行, 同行中间隔一个正方形的两个面是相对面, 再找异行, 异行中间隔两个正方形的两个面是相对面, 最后求出 $\\mathrm{A} 、 \\mathrm{~B}$ 代表的数字,据此解答。\n\n【详解】分析可知, B 和“ 3 ”是相对面, $\\mathrm{C}$ 和“ 8 ”是相对面, $\\mathrm{A}$ 和“ 6 ”是相对面, 则 $\\mathrm{A}=10-6=4, \\mathrm{~B}=$ $10-3=7$ 。\n\n【点睛】掌握正方体的展开图中找相对面的方法是解答题目的关键。", "solution": "null", "level": "五年级", "question": "将一个正方体纸盒展开(如图), 现有三个正方形分别填着 3、6、8, 如果要使相对面上两个数的和都为 10 。那么 $A=(\\quad), B=(\\quad)$ 。\n\n", "options": [], "subject": "立体几何学", "analysis": "解$4 \\quad 7$\n\n【分析】找出正方体展开图中三个数字的相对面, 找相对面时, 先找同行, 同行中间隔一个正方形的两个面是相对面, 再找异行, 异行中间隔两个正方形的两个面是相对面, 最后求出 $\\mathrm{A} 、 \\mathrm{~B}$ 代表的数字,据此解答。\n\n【详解】分析可知, B 和“ 3 ”是相对面, $\\mathrm{C}$ 和“ 8 ”是相对面, $\\mathrm{A}$ 和“ 6 ”是相对面, 则 $\\mathrm{A}=10-6=4, \\mathrm{~B}=$ $10-3=7$ 。\n\n【点睛】掌握正方体的展开图中找相对面的方法是解答题目的关键。"} {"id": "24558", "image": [], "answer": "解$27 \\quad 972$\n\n【分析】正方体的体积 $=$ 棱长 $\\times$ 棱长 $\\times$ 棱长, 得到小正方体的个数 $=$ 大正方体的体积 $\\div$ 小正方体的体积,正方体的表面积 $=$ 棱长 $\\times$ 棱长 $\\times 6$, 先求出一个小正方体的表面积, 再乘小正方体的个数, 最后减去原来正方体的表面积, 据此解答。\n\n【详解】大正方体的体积: $9 \\times 9 \\times 9$\n\n$=81 \\times 9$\n\n$=729\\left(\\mathrm{~cm}^{3}\\right)$\n\n小正方体的体积: $3 \\times 3 \\times 3$\n\n$=9 \\times 3$\n\n$=27\\left(\\mathrm{~cm}^{3}\\right)$\n\n$729 \\div 27=27$ (个)\n\n大正方体的表面积: $9 \\times 9 \\times 6$\n\n$=81 \\times 6$\n\n$=486\\left(\\mathrm{~cm}^{2}\\right)$\n\n小正方体的表面积之和: $3 \\times 3 \\times 6 \\times 27$\n\n$=9 \\times 6 \\times 27$\n\n$=54 \\times 27$\n\n$=1458\\left(\\mathrm{~cm}^{2}\\right)$\n$1458-486=972\\left(\\mathrm{~cm}^{2}\\right)$\n\n【点睛】掌握正方体的表面积和体积计算公式是解答题目的关键。", "solution": "null", "level": "五年级", "question": "把一个棱长 $9 \\mathrm{~cm}$ 的正方体切成棱长 $3 \\mathrm{~cm}$ 的小正方体, 可以得到( )个小正方体, 它们的表面积之和比原来大正方体的表面积增加了 ( $) \\mathrm{cm}^{2}$ 。", "options": [], "subject": "立体几何学", "analysis": "解$27 \\quad 972$\n\n【分析】正方体的体积 $=$ 棱长 $\\times$ 棱长 $\\times$ 棱长, 得到小正方体的个数 $=$ 大正方体的体积 $\\div$ 小正方体的体积,正方体的表面积 $=$ 棱长 $\\times$ 棱长 $\\times 6$, 先求出一个小正方体的表面积, 再乘小正方体的个数, 最后减去原来正方体的表面积, 据此解答。\n\n【详解】大正方体的体积: $9 \\times 9 \\times 9$\n\n$=81 \\times 9$\n\n$=729\\left(\\mathrm{~cm}^{3}\\right)$\n\n小正方体的体积: $3 \\times 3 \\times 3$\n\n$=9 \\times 3$\n\n$=27\\left(\\mathrm{~cm}^{3}\\right)$\n\n$729 \\div 27=27$ (个)\n\n大正方体的表面积: $9 \\times 9 \\times 6$\n\n$=81 \\times 6$\n\n$=486\\left(\\mathrm{~cm}^{2}\\right)$\n\n小正方体的表面积之和: $3 \\times 3 \\times 6 \\times 27$\n\n$=9 \\times 6 \\times 27$\n\n$=54 \\times 27$\n\n$=1458\\left(\\mathrm{~cm}^{2}\\right)$\n$1458-486=972\\left(\\mathrm{~cm}^{2}\\right)$\n\n【点睛】掌握正方体的表面积和体积计算公式是解答题目的关键。"} {"id": "12279", "image": [], "answer": "答案:V", "solution": "null", "level": "五年级", "question": "根据从一个方向看到的形状图拼摆几何组合体, 摆法不止一种。()", "options": [], "subject": "组合几何学", "analysis": "答案:V"} {"id": "12282", "image": ["2579.jpg", "2580.jpg"], "answer": "答案: $7 ; 11$", "solution": "null", "level": "五年级", "question": "一个几何体, 从上面看到的图形是 $\\square$ , 从正面看到的图形是 , 搭这样的几何体最少需要 \\$ \\qquad \\$个小正方体,最多可以摆 \\$ \\qquad \\$个小正方体。", "options": [], "subject": "组合几何学", "analysis": "答案: $7 ; 11$"} {"id": "23982", "image": [], "answer": "解38\n\n【分析】一个大于 1 的自然数, 除了 1 和它自身外, 不能整除其他自然数的数, 即除了 1 和它本身以外不再有其他因数, 这样的数叫质数。据此确定这个质数, 再求积即可。\n\n【详解】 $21=2+19$\n\n$2 \\times 19=38$\n\n两个质数的和是 21 , 这两个质数的积是 38 。\n\n【点睛】关键是理解质数、合数的分类标准, 通过两个质数的和确定这两个质数。", "solution": "null", "level": "五年级", "question": "两个质数的和是 21 , 这两个质数的积是 ( )。", "options": [], "subject": "算术", "analysis": "解38\n\n【分析】一个大于 1 的自然数, 除了 1 和它自身外, 不能整除其他自然数的数, 即除了 1 和它本身以外不再有其他因数, 这样的数叫质数。据此确定这个质数, 再求积即可。\n\n【详解】 $21=2+19$\n\n$2 \\times 19=38$\n\n两个质数的和是 21 , 这两个质数的积是 38 。\n\n【点睛】关键是理解质数、合数的分类标准, 通过两个质数的和确定这两个质数。"} {"id": "23983", "image": [], "answer": "解120\n\n【分析】 2 的倍数特征: 个位数是 $0 、 2 、 4 、 6$ 或 $8 ; 5$ 的倍数特征: 个位数是 0 或 $5 ; 3$ 的倍数特征:各个数位上的数字和是 3 的倍数。同时是 $2 、 3$ 和 5 的倍数特征: 个位是 0 , 且各个数位上的数字和是 3 的倍数。\n\n【详解】同时为 $2 、 3 、 5$ 的倍数的最小三位数, 个位上是 0 , 要使数最小, 则百位上最小为 1 , $3-1=2$\n\n所以要使这个数是 3 的倍数, 则十位上是 2 , 所以这个同时为 $2 、 3 、 5$ 的倍数的最小三位数是 120 ,那么小乐平均每天跑步产生 120 克能量。\n\n【点睛】熟练掌握 $2 、 3 、 5$ 的倍数的特征是解决此题的关键。", "solution": "null", "level": "五年级", "question": "为了在蚂蚁森林种一棵白杨树, 小乐坚持每天跑步锻炼, 他平均每天跑步产生的能量 (单位: 克)是一个同时为 $2 、 3 、 5$ 的倍数的最小三位数, 那么小乐平均每天跑步产生 ( ) 克能量。", "options": [], "subject": "算术", "analysis": "解120\n\n【分析】 2 的倍数特征: 个位数是 $0 、 2 、 4 、 6$ 或 $8 ; 5$ 的倍数特征: 个位数是 0 或 $5 ; 3$ 的倍数特征:各个数位上的数字和是 3 的倍数。同时是 $2 、 3$ 和 5 的倍数特征: 个位是 0 , 且各个数位上的数字和是 3 的倍数。\n\n【详解】同时为 $2 、 3 、 5$ 的倍数的最小三位数, 个位上是 0 , 要使数最小, 则百位上最小为 1 , $3-1=2$\n\n所以要使这个数是 3 的倍数, 则十位上是 2 , 所以这个同时为 $2 、 3 、 5$ 的倍数的最小三位数是 120 ,那么小乐平均每天跑步产生 120 克能量。\n\n【点睛】熟练掌握 $2 、 3 、 5$ 的倍数的特征是解决此题的关键。"} {"id": "23984", "image": [], "answer": "解$0 、 5$ 0、2、4、6、8\n\n【分析】个位是 0 或 5 的数是 5 的倍数; 个位是 $0 、 2 、 4 、 6 、 8$ 的数是 2 的倍数; 据此, 要使 “ 4 口”是 5 的倍数, 口里可以填 $0 、 5$, 要使“13口”是 2 的倍数, 口里可以填 $0 、 2 、 4 、 6 、 8$ 。\n\n【详解】由分析得:\n\n“ 4 ”是 5 的倍数, 口里可以填 $(0 、 5)$ 。“ 13 口”是 2 的倍数, $\\square$ 里可以填 $(0 、 2 、 4 、 6 、 8) 。$\n\n【点睛】熟悉 2、5 倍数的特征, 且能够结合题意来推断口里应该填的数, 同时训练了对于因数倍数的含义的理解。", "solution": "null", "level": "五年级", "question": "“4口”是 5 的倍数, 口里可以填( )。“13口”是 2 的倍数, 口里可以填( )。", "options": [], "subject": "算术", "analysis": "解$0 、 5$ 0、2、4、6、8\n\n【分析】个位是 0 或 5 的数是 5 的倍数; 个位是 $0 、 2 、 4 、 6 、 8$ 的数是 2 的倍数; 据此, 要使 “ 4 口”是 5 的倍数, 口里可以填 $0 、 5$, 要使“13口”是 2 的倍数, 口里可以填 $0 、 2 、 4 、 6 、 8$ 。\n\n【详解】由分析得:\n\n“ 4 ”是 5 的倍数, 口里可以填 $(0 、 5)$ 。“ 13 口”是 2 的倍数, $\\square$ 里可以填 $(0 、 2 、 4 、 6 、 8) 。$\n\n【点睛】熟悉 2、5 倍数的特征, 且能够结合题意来推断口里应该填的数, 同时训练了对于因数倍数的含义的理解。"} {"id": "24001", "image": ["11844.jpg", "11844.jpg"], "answer": "\n\n【分析】根据求一个因数的方法, 求出 60 的所有因数, 再结合奇数、偶数、质数和合数的意义: 是 2 的倍数的数叫做偶数, 不是 2 的倍数的数叫做奇数; 一个自然数如果只有 1 和它本身两个因数, 那么这个自然数叫做质数; 一个自然数如果除了 1 和它本身还有其它的因数, 那么这个自然数叫做合数; 1 既不是质数也不是合数。据此填空即可。\n\n【详解】 60 的所有因数有: $1 、 2 、 3 、 4 、 5 、 6 、 10 、 12 、 15 、 20 、 30 、 60$, 其中 2 是最小的质数, 4 是最小的合数, 15 既是合数又是奇数, 2 既是质数又是偶数, 1 既不是质数也不是合数。\n\n【点睛】本题考查奇数、偶数、质数和合数, 明确它们的定义是解题的关键。", "solution": "null", "level": "五年级", "question": "0 的所有因数有 ( ), 其中 ( )是最小的质数, ( )是最小的合数, $(\\quad$ )既是合数又是奇数, ( ) 既是质数又是偶数, ( )既不是质数也不是合数。", "options": [], "subject": "算术", "analysis": "\n\n【分析】根据求一个因数的方法, 求出 60 的所有因数, 再结合奇数、偶数、质数和合数的意义: 是 2 的倍数的数叫做偶数, 不是 2 的倍数的数叫做奇数; 一个自然数如果只有 1 和它本身两个因数, 那么这个自然数叫做质数; 一个自然数如果除了 1 和它本身还有其它的因数, 那么这个自然数叫做合数; 1 既不是质数也不是合数。据此填空即可。\n\n【详解】 60 的所有因数有: $1 、 2 、 3 、 4 、 5 、 6 、 10 、 12 、 15 、 20 、 30 、 60$, 其中 2 是最小的质数, 4 是最小的合数, 15 既是合数又是奇数, 2 既是质数又是偶数, 1 既不是质数也不是合数。\n\n【点睛】本题考查奇数、偶数、质数和合数, 明确它们的定义是解题的关键。"} {"id": "24002", "image": [], "answer": "解$237 \\quad 267$\n\n【分析】各个数位上的数字的和是 3 的倍数的数是 3 的倍数, $2+7=9$ 是 3 的倍数, $9+0=9,9+3$ $=12,9+6=15,9+9=18,9 、 12 、 15 、 18$ 都是 3 的倍数, 所以十位上的数字 “ 5 ”可以改为 $0 、 3 、 6$ 、 9, 任取两种填空即可。\n\n【详解】改动 257 十位上的数字“ 5 ”, 使它成为 3 的倍数, 请写出两种不同的答案 237, 267。(答案不唯一)\n\n【点睛】掌握 3 的倍数的特征是解决此题的关键。", "solution": "null", "level": "五年级", "question": "改动 257 十位上的数字“ 5 ”, 使它成为 3 的倍数, 请写出两种不同的答案 ( ), ( )。", "options": [], "subject": "算术", "analysis": "解$237 \\quad 267$\n\n【分析】各个数位上的数字的和是 3 的倍数的数是 3 的倍数, $2+7=9$ 是 3 的倍数, $9+0=9,9+3$ $=12,9+6=15,9+9=18,9 、 12 、 15 、 18$ 都是 3 的倍数, 所以十位上的数字 “ 5 ”可以改为 $0 、 3 、 6$ 、 9, 任取两种填空即可。\n\n【详解】改动 257 十位上的数字“ 5 ”, 使它成为 3 的倍数, 请写出两种不同的答案 237, 267。(答案不唯一)\n\n【点睛】掌握 3 的倍数的特征是解决此题的关键。"} {"id": "24003", "image": [], "answer": "解$40 \\quad 48$\n\n【分析】同时是 2 和 5 倍数的倍数特征: 个位数字为 0 ; 同时是 2 和 3 倍数的倍数特征: 个位数字为 $0 、 2 、 4 、 6 、 8$, 各个位上数字相加的和是 3 的倍数, 据此解答。\n\n【详解】分析可知, 如果这个两位数同时是 2 和 5 的倍数, 那么这个数的个位数字为 0 , 这个数是 40 。\n十位数字是 4 , 离 4 最近的 3 的倍数是 $6,6-4=2$, 所以个位上可以填 $2 、 5 、 8$, 最大是 8 , 所以这个两位数个位数字为 8 。\n\n由上可知, 一个两位数 $4 \\square$, 它既是 2 的倍数, 又是 5 的倍数, 这个数是 40 , 若它既是 2 的倍数, 又有因数 3 , 则这个数最大是 48 。\n\n【点睛】掌握 $2 、 3 、 5$ 的倍数特征是解答题目的关键。", "solution": "null", "level": "五年级", "question": "一个两位数 $4 \\square$, 它既是 2 的倍数, 又是 5 的倍数, 这个数是 ( ), 若它既是 2 的倍数, 又有因数 3, 则这个数最大是 ( )。", "options": [], "subject": "算术", "analysis": "解$40 \\quad 48$\n\n【分析】同时是 2 和 5 倍数的倍数特征: 个位数字为 0 ; 同时是 2 和 3 倍数的倍数特征: 个位数字为 $0 、 2 、 4 、 6 、 8$, 各个位上数字相加的和是 3 的倍数, 据此解答。\n\n【详解】分析可知, 如果这个两位数同时是 2 和 5 的倍数, 那么这个数的个位数字为 0 , 这个数是 40 。\n十位数字是 4 , 离 4 最近的 3 的倍数是 $6,6-4=2$, 所以个位上可以填 $2 、 5 、 8$, 最大是 8 , 所以这个两位数个位数字为 8 。\n\n由上可知, 一个两位数 $4 \\square$, 它既是 2 的倍数, 又是 5 的倍数, 这个数是 40 , 若它既是 2 的倍数, 又有因数 3 , 则这个数最大是 48 。\n\n【点睛】掌握 $2 、 3 、 5$ 的倍数特征是解答题目的关键。"} {"id": "24024", "image": [], "answer": "解810\n\n【分析】 2 的倍数特征: 末尾数字是 $0 、 2 、 4 、 6 、 8$ 的数是 2 的倍数; 3 的倍数特征: 各个数位上的数字之和是 3 的倍数, 这个数字就是 3 的倍数; 5 的倍数特征: 末尾数字是 0 或 5 的数是 5 的倍数;最小的奇数是 1 , 据此解答即可。\n\n【详解】由分析可知:\n\n有一个三位数, 它的十位上的数字是最小的奇数, 如果这个三位数能同时被 $2 、 3 、 5$ 整除, 这个三位数最大是 810。\n\n【点睛】本题考查 $2 、 3 、 5$ 的倍数特征, 明确 $2 、 3 、 5$ 的倍数特征是解题的关键。", "solution": "null", "level": "五年级", "question": "有一个三位数, 它的十位上的数字是最小的奇数, 如果这个三位数能同时被 $2 、 3 、 5$ 整除, 这个三位数最大是 ( )。", "options": [], "subject": "算术", "analysis": "解810\n\n【分析】 2 的倍数特征: 末尾数字是 $0 、 2 、 4 、 6 、 8$ 的数是 2 的倍数; 3 的倍数特征: 各个数位上的数字之和是 3 的倍数, 这个数字就是 3 的倍数; 5 的倍数特征: 末尾数字是 0 或 5 的数是 5 的倍数;最小的奇数是 1 , 据此解答即可。\n\n【详解】由分析可知:\n\n有一个三位数, 它的十位上的数字是最小的奇数, 如果这个三位数能同时被 $2 、 3 、 5$ 整除, 这个三位数最大是 810。\n\n【点睛】本题考查 $2 、 3 、 5$ 的倍数特征, 明确 $2 、 3 、 5$ 的倍数特征是解题的关键。"} {"id": "24025", "image": [], "answer": "解50\n\n【分析】既是 3 的倍数又是 5 的倍数的特征:个位上的数字是 0 或 5 , 各个数位上的数字的和是 3 的倍数的数。\n\n既是 2 的倍数又是 5 的倍数的特征: 个位上的数字是 0 的数, 既是 2 的倍数, 又是 5 的倍数。\n\n【详解】一个四位数 103口, 它既是 3 的倍数, 又是 5 的倍数, 口里应填 5 ;一个三位数 15 口, 当口里填 0 时, 这个数既是 2 的倍数, 又是 5 的倍数。\n\n【点睛】关键是掌握 $2 、 3 、 5$ 的倍数的特征。", "solution": "null", "level": "五年级", "question": "一个四位数 103口, 它既是 3 的倍数, 又是 5 的倍数, 口里应填 ( ); 一个三位数 $15 \\square$, 当口里填 $($ )时, 这个数既是 2 的倍数, 又是 5 的倍数。", "options": [], "subject": "算术", "analysis": "解50\n\n【分析】既是 3 的倍数又是 5 的倍数的特征:个位上的数字是 0 或 5 , 各个数位上的数字的和是 3 的倍数的数。\n\n既是 2 的倍数又是 5 的倍数的特征: 个位上的数字是 0 的数, 既是 2 的倍数, 又是 5 的倍数。\n\n【详解】一个四位数 103口, 它既是 3 的倍数, 又是 5 的倍数, 口里应填 5 ;一个三位数 15 口, 当口里填 0 时, 这个数既是 2 的倍数, 又是 5 的倍数。\n\n【点睛】关键是掌握 $2 、 3 、 5$ 的倍数的特征。"} {"id": "24026", "image": [], "answer": "解$\\quad 5 \\quad 9 \\quad 45 \\quad 45 \\quad 5 \\quad 9$\n\n【分析】在乘法算式 $\\mathrm{a} \\times \\mathrm{b}=\\mathrm{c}(\\mathrm{a} 、 \\mathrm{~b} 、 \\mathrm{c}$ 均为非 0 的自然数 $)$ 中, $\\mathrm{a} 、 \\mathrm{~b}$ 就是 $\\mathrm{c}$ 的因数, $\\mathrm{c}$ 就是 $\\mathrm{a} 、 \\mathrm{~b}$ 的倍数。\n\n【详解】在 $45 \\div 9=5$ 中, 5 和 9 是 45 的因数, 45 是 5 和 9 的倍数。\n\n【点睛】因数和倍数两个不同的概念是相互依存的, 不能单独存在。", "solution": "null", "level": "五年级", "question": "在 $45 \\div 9=5$ 中, $($ 和 $($ 是 $($ 的因数, $($ 是 $($ 和( )的倍数。", "options": [], "subject": "算术", "analysis": "解$\\quad 5 \\quad 9 \\quad 45 \\quad 45 \\quad 5 \\quad 9$\n\n【分析】在乘法算式 $\\mathrm{a} \\times \\mathrm{b}=\\mathrm{c}(\\mathrm{a} 、 \\mathrm{~b} 、 \\mathrm{c}$ 均为非 0 的自然数 $)$ 中, $\\mathrm{a} 、 \\mathrm{~b}$ 就是 $\\mathrm{c}$ 的因数, $\\mathrm{c}$ 就是 $\\mathrm{a} 、 \\mathrm{~b}$ 的倍数。\n\n【详解】在 $45 \\div 9=5$ 中, 5 和 9 是 45 的因数, 45 是 5 和 9 的倍数。\n\n【点睛】因数和倍数两个不同的概念是相互依存的, 不能单独存在。"} {"id": "24027", "image": [], "answer": "解$2 、 3$ 6, $9 、 18 \\quad 1,3,9$\n\n【分析】列乘法算式找因数, 按照从小到大的顺序, 一组一组地写出所有积是这个数的乘法算式, 乘法算式中的两个因数就是这个数的因数。\n\n除了 1 和它本身以外不再有其他因数, 这样的数叫质数; 除了 1 和它本身以外还有其他因数, 这样的数叫合数。\n\n整数中, 是 2 的倍数的数叫偶数, 不是 2 的倍数的数叫奇数。\n\n【详解】18 的因数有: $1 、 2 、 3 、 6 、 9 、 18$, 是质数的有 $2 、 3$, 是合数的有 $6 、 9 、 18$, 是奇数的有 1 、 3、9。\n\n【点睛】关键是先找出 18 的因数, 再根据质数、合数、奇数的分类标准进行填空。", "solution": "null", "level": "五年级", "question": "在 18 的因数中, 是质数的有 $($, 是合数的有 $(\\quad)$, 是奇数的有 $($ 。", "options": [], "subject": "算术", "analysis": "解$2 、 3$ 6, $9 、 18 \\quad 1,3,9$\n\n【分析】列乘法算式找因数, 按照从小到大的顺序, 一组一组地写出所有积是这个数的乘法算式, 乘法算式中的两个因数就是这个数的因数。\n\n除了 1 和它本身以外不再有其他因数, 这样的数叫质数; 除了 1 和它本身以外还有其他因数, 这样的数叫合数。\n\n整数中, 是 2 的倍数的数叫偶数, 不是 2 的倍数的数叫奇数。\n\n【详解】18 的因数有: $1 、 2 、 3 、 6 、 9 、 18$, 是质数的有 $2 、 3$, 是合数的有 $6 、 9 、 18$, 是奇数的有 1 、 3、9。\n\n【点睛】关键是先找出 18 的因数, 再根据质数、合数、奇数的分类标准进行填空。"} {"id": "24028", "image": ["11946.jpg", "11946.jpg"], "answer": "\n\n【分析】(1)整数中, 是 2 的倍数的数叫做偶数( 0 也是偶数), 不是 2 的倍数的数叫做奇数;\n\n(2) 3 的倍数特征:各个位上数字相加的和是 3 的倍数; 5 的倍数特征:个位数字是 0 或 5 的数是 5 的倍数; 据此解答。\n\n【详解】(1)是偶数的有:18、70;\n\n(2)是奇数的有: $15 、 29 、 35 、 39 、 41 、 47 、 87$;\n\n(3)有因数 3 的是: $15 、 18 、 39 、 87$;\n\n(4) 5 的倍数有: $15 、 35 、 70$ 。\n\n【点睛】掌握奇数、偶数的意义和 3、5 的倍数特征是解答题目的关键。", "solution": "null", "level": "五年级", "question": "在 $15 、 18 、 29 、 35 、 39 、 41 、 47 、 70 、 87$ 这些数中。\n\n(1)是偶数的有 ( );\n\n(2)是奇数的有 ( );\n\n(3)有因数 3 的是 ( );\n\n(4) 5 的倍数有 ( )。", "options": [], "subject": "算术", "analysis": "\n\n【分析】(1)整数中, 是 2 的倍数的数叫做偶数( 0 也是偶数), 不是 2 的倍数的数叫做奇数;\n\n(2) 3 的倍数特征:各个位上数字相加的和是 3 的倍数; 5 的倍数特征:个位数字是 0 或 5 的数是 5 的倍数; 据此解答。\n\n【详解】(1)是偶数的有:18、70;\n\n(2)是奇数的有: $15 、 29 、 35 、 39 、 41 、 47 、 87$;\n\n(3)有因数 3 的是: $15 、 18 、 39 、 87$;\n\n(4) 5 的倍数有: $15 、 35 、 70$ 。\n\n【点睛】掌握奇数、偶数的意义和 3、5 的倍数特征是解答题目的关键。"} {"id": "24029", "image": [], "answer": "解$\\quad 3 \\quad \\begin{array}{lllll}7 & 5 & 7 & 3 & 7\\end{array}$\n\n【分析】一个数, 如果只有 1 和它本身两个因数, 那么这样的数叫作质数, 20 以内的质数有: 2,3 ,\n\n$5,7,11,13,17,19$, 据此把合适的质数填在括号里。\n\n【详解】 $10=(3)+(7) \\quad 12=(5)+(7) 21=(3) \\times\\left(\\begin{array}{l}7\\end{array}\\right)$\n\n【点睛】掌握质数的意义并熟记 100 以内的质数是解答题目的关键。", "solution": "null", "level": "五年级", "question": "在括号里填上合适的质数。\n\n$10=(\\quad)+(\\quad) 12=(\\quad)+(\\quad) 21=(\\quad) \\times(\\quad)$\n\n", "options": [], "subject": "算术", "analysis": "解$\\quad 3 \\quad \\begin{array}{lllll}7 & 5 & 7 & 3 & 7\\end{array}$\n\n【分析】一个数, 如果只有 1 和它本身两个因数, 那么这样的数叫作质数, 20 以内的质数有: 2,3 ,\n\n$5,7,11,13,17,19$, 据此把合适的质数填在括号里。\n\n【详解】 $10=(3)+(7) \\quad 12=(5)+(7) 21=(3) \\times\\left(\\begin{array}{l}7\\end{array}\\right)$\n\n【点睛】掌握质数的意义并熟记 100 以内的质数是解答题目的关键。"} {"id": "24048", "image": [], "answer": "解$a-2 \\quad a-4$\n\n【分析】根据连续两个奇数之间相差 2, 进而用含字母的式子表示出另外两个奇数。\n\n【详解】由分析可知:\n\n有三个连续奇数, 最大的数是 $\\mathrm{a}$, 另外两个数可以表示为 $\\mathrm{a}-2$ 和 $\\mathrm{a}-4$ 。\n\n【点睛】解决此题关键是知道奇数的意义, 以及连续两个奇数之间相差 2 。", "solution": "null", "level": "五年级", "question": "有三个连续奇数, 最大的数是 $\\mathrm{a}$, 另外两个数可以表示为 ( ) 和( )。", "options": [], "subject": "算术", "analysis": "解$a-2 \\quad a-4$\n\n【分析】根据连续两个奇数之间相差 2, 进而用含字母的式子表示出另外两个奇数。\n\n【详解】由分析可知:\n\n有三个连续奇数, 最大的数是 $\\mathrm{a}$, 另外两个数可以表示为 $\\mathrm{a}-2$ 和 $\\mathrm{a}-4$ 。\n\n【点睛】解决此题关键是知道奇数的意义, 以及连续两个奇数之间相差 2 。"} {"id": "24050", "image": [], "answer": "解:因为这个数是 2 的倍数, 所以个位上只能是 $0 、 2 、 4 、 6 、 8$;\n\n$2+0=2$, 不能被 3 整除;\n\n$2+2=4$ ,不能被 3 整除;\n\n$2+4=6$, 能被 3 整除;\n\n$2+6=8 ,$ 不能被 3 整除;\n\n$2+8=10$, 不能被 3 整除;\n\n所以, 这个数是 24 。\n\n故答案为: 24 。", "solution": "null", "level": "五年级", "question": "(2022 秋・市中区期末)一个两位数, 十位上的数是 2 , 这个数即是 2 的倍数也是 3 的倍数, 它是 $\\qquad$", "options": [], "subject": "算术", "analysis": "解:因为这个数是 2 的倍数, 所以个位上只能是 $0 、 2 、 4 、 6 、 8$;\n\n$2+0=2$, 不能被 3 整除;\n\n$2+2=4$ ,不能被 3 整除;\n\n$2+4=6$, 能被 3 整除;\n\n$2+6=8 ,$ 不能被 3 整除;\n\n$2+8=10$, 不能被 3 整除;\n\n所以, 这个数是 24 。\n\n故答案为: 24 。"} {"id": "24051", "image": [], "answer": "解 在 $246 \\square 5$ 中, $2+4+6+5=17,17$ 不是 3 的倍数, 17 至少加上 1 才是 3 的倍数, 所以 $\\square$ 里最小填 1 。\n\n故答案为: 1 。", "solution": "null", "level": "五年级", "question": "(2022 秋 $\\cdot$ 宁乡市期末)已知 $246 \\square 5$ 是 3 的倍数, $\\square$ 里最小可填 $\\qquad$。", "options": [], "subject": "算术", "analysis": "解 在 $246 \\square 5$ 中, $2+4+6+5=17,17$ 不是 3 的倍数, 17 至少加上 1 才是 3 的倍数, 所以 $\\square$ 里最小填 1 。\n\n故答案为: 1 。"} {"id": "24052", "image": [], "answer": " $21=2+19$\n\n$18=5+13=7+11$\n\n故答案为: 2,$19 ; 5,13 ; 7,11$ 。", "solution": "null", "level": "五年级", "question": "(2022 秋・定州市期末)把下面的合数写成两个质数和的形式。\n\n$21=$ $\\qquad$ $+$\n\n$18=$ $\\qquad$ $+$ $\\qquad$ $=$ $\\qquad$ $+$", "options": [], "subject": "算术", "analysis": " $21=2+19$\n\n$18=5+13=7+11$\n\n故答案为: 2,$19 ; 5,13 ; 7,11$ 。"} {"id": "24053", "image": [], "answer": "解 在 $1 \\sim 10$ 中, 质数是 $2 、 3 、 5 、 7$, 合数是 $4 、 6 、 8 、 9 、 10$, 既是奇数又是合数的是 9, 既是偶数又是质数的是 2,1 既不是质数又不是合数。\n\n故答案为: $2 、 3 、 5 、 7 ; 4 、 6 、 8 、 9 、 10 ; 9 ; 2 ; 1$ 。", "solution": "null", "level": "五年级", "question": "(2022 秋・市中区期末)在 $1 1 0$ 中,质数是 $\\qquad$ , 合数是 $\\qquad$ , 既是奇数又是合数的是 $\\qquad$ ,既是偶数又是质数的是 $\\qquad$ , $\\qquad$既不是质数又不是合数。", "options": [], "subject": "算术", "analysis": "解 在 $1 \\sim 10$ 中, 质数是 $2 、 3 、 5 、 7$, 合数是 $4 、 6 、 8 、 9 、 10$, 既是奇数又是合数的是 9, 既是偶数又是质数的是 2,1 既不是质数又不是合数。\n\n故答案为: $2 、 3 、 5 、 7 ; 4 、 6 、 8 、 9 、 10 ; 9 ; 2 ; 1$ 。"} {"id": "24054", "image": [], "answer": "解 一个数比 50 大, 比 70 小。如果这个数是 3 的倍数, 那么它是 $51 、 54 、 57 、 60$ 、 63、 $66 、 69$; 如果这个数同时是 2 和 3 的倍数, 那么它是 $54 、 60$; 如果这个数同时是 2 和 5 的倍数, 那么它是 60 ; 如果这个数是质数, 那么它是 $53 、 59 、 61 、 67$ 。\n\n故答案为: $51 、 54 、 57 、 60 、 63 、 66 、 69 ; 54 、 60 ; 60 ; 53 、 59 、 61 、 67$ 。", "solution": "null", "level": "五年级", "question": "(2022 秋・茂名期中)一个数比 50 大, 比 70 小。如果这个数是 3 的倍数, 那么它是 $\\qquad$ ;如果这个数同时是 2 和 3 的倍数, 那么它_是 $\\qquad$ ; 如果这个数同时是 2 和 5 的倍数, 那么它是 $\\qquad$ ; 如果这个数是质数, 那么它是 $\\qquad$。", "options": [], "subject": "算术", "analysis": "解 一个数比 50 大, 比 70 小。如果这个数是 3 的倍数, 那么它是 $51 、 54 、 57 、 60$ 、 63、 $66 、 69$; 如果这个数同时是 2 和 3 的倍数, 那么它是 $54 、 60$; 如果这个数同时是 2 和 5 的倍数, 那么它是 60 ; 如果这个数是质数, 那么它是 $53 、 59 、 61 、 67$ 。\n\n故答案为: $51 、 54 、 57 、 60 、 63 、 66 、 69 ; 54 、 60 ; 60 ; 53 、 59 、 61 、 67$ 。"} {"id": "24072", "image": [], "answer": ":一个数的最小倍数是 12 , 这个数是 12 。\n\n$6=1 \\times 6=2 \\times 3$\n\n所以 6 的因数有: $1 、 2 、 3 、 6$, 即有 4 个。\n\n故答案为: $12 ; 4$ 。", "solution": "null", "level": "五年级", "question": "(2022 秋・宁乡市期末)一个数最小的倍数是 12 , 这个数是 $\\qquad$ ; 6 的因数有 $\\qquad$个。", "options": [], "subject": "算术", "analysis": ":一个数的最小倍数是 12 , 这个数是 12 。\n\n$6=1 \\times 6=2 \\times 3$\n\n所以 6 的因数有: $1 、 2 、 3 、 6$, 即有 4 个。\n\n故答案为: $12 ; 4$ 。"} {"id": "24168", "image": [], "answer": "解1 和它本身\n\n【详解】一个数只有 1 和它本身两个因数, 这个数叫做质数。一个数除了 1 和它本身两个因数, 还有其他的因数, 这个数叫做合数。\n\n例如: 9 的因数有: $1 、 3 、 9$, 是合数,\n\n2 的因数有: $1 、 2$, 是质数。", "solution": "null", "level": "五年级", "question": "一个数, 如果除了( )还有别的因数, 那么这样的数叫做合数。", "options": [], "subject": "算术", "analysis": "解1 和它本身\n\n【详解】一个数只有 1 和它本身两个因数, 这个数叫做质数。一个数除了 1 和它本身两个因数, 还有其他的因数, 这个数叫做合数。\n\n例如: 9 的因数有: $1 、 3 、 9$, 是合数,\n\n2 的因数有: $1 、 2$, 是质数。"} {"id": "24169", "image": [], "answer": "解$2 、 5 、 13 \\quad 2 、 6 、 18 、 20$\n\n【分析】一个数只有 1 和它本身两个因数, 这个数叫做质数。一个数除了 1 和它本身两个因数, 还有其他的因数, 这个数叫做合数。在自然数中, 是 2 的倍数的数叫做偶数, 不是 2 的倍数的数叫做奇数。\n\n【详解】在 $1 、 2 、 5 、 6 、 13 、 15 、 18 、 20$ 这些数中, 质数有 $2 、 5 、 13$, 偶数有 $2 、 6 、 18 、 20$ 。\n\n【点睛】本题考查了质数与偶数的意义; 完成时要注意质数与奇数的区别。", "solution": "null", "level": "五年级", "question": "在 $1 、 2 、 5 、 6 、 13 、 15 、 18 、 20$ 这些数中, 质数有 $($, 偶数有 $($ 。", "options": [], "subject": "算术", "analysis": "解$2 、 5 、 13 \\quad 2 、 6 、 18 、 20$\n\n【分析】一个数只有 1 和它本身两个因数, 这个数叫做质数。一个数除了 1 和它本身两个因数, 还有其他的因数, 这个数叫做合数。在自然数中, 是 2 的倍数的数叫做偶数, 不是 2 的倍数的数叫做奇数。\n\n【详解】在 $1 、 2 、 5 、 6 、 13 、 15 、 18 、 20$ 这些数中, 质数有 $2 、 5 、 13$, 偶数有 $2 、 6 、 18 、 20$ 。\n\n【点睛】本题考查了质数与偶数的意义; 完成时要注意质数与奇数的区别。"} {"id": "24170", "image": [], "answer": "解214\n\n【分析】最小的质数是 2 , 最小的奇数是 1 , 最小的合数是 4 , 据此解答。\n\n【详解】一个三位数, 百位上是最小的质数, 十位上是最小的奇数, 个位上是最小的合数, 这个三位数是 214 。\n\n【点睛】掌握质数、合数、奇数的概念是解答此题的关键。", "solution": "null", "level": "五年级", "question": "一个三位数, 百位上是最小的质数, 十位上是最小的奇数, 个位上是最小的合数, 这个三位数是 ( ) 。", "options": [], "subject": "算术", "analysis": "解214\n\n【分析】最小的质数是 2 , 最小的奇数是 1 , 最小的合数是 4 , 据此解答。\n\n【详解】一个三位数, 百位上是最小的质数, 十位上是最小的奇数, 个位上是最小的合数, 这个三位数是 214 。\n\n【点睛】掌握质数、合数、奇数的概念是解答此题的关键。"} {"id": "24171", "image": [], "answer": "解31\n\n【分析】 2 的倍数特征:个位上的数字是 $0 、 2 、 4 、 6 、 8$ 的数是 2 的倍数。\n\n5 的倍数特征:个位上的数字是 0 或 5 的数是 5 的倍数。\n\n【详解】离 313 最近且比 313 小的 5 的倍数是 310 。\n\n$313-310=3$\n\n离 313 最近的且比 313 大的 2 的倍数是 314 。\n\n$314-313=1$\n\n313 至少减去 3 是 5 的倍数, 至少加上 1 是 2 的倍数。\n\n【点睛】关键是掌握 2 和 5 的倍数的特征。", "solution": "null", "level": "五年级", "question": "313 至少减去 ( )是 5 的倍数, 至少加上( )是 2 的倍数。", "options": [], "subject": "算术", "analysis": "解31\n\n【分析】 2 的倍数特征:个位上的数字是 $0 、 2 、 4 、 6 、 8$ 的数是 2 的倍数。\n\n5 的倍数特征:个位上的数字是 0 或 5 的数是 5 的倍数。\n\n【详解】离 313 最近且比 313 小的 5 的倍数是 310 。\n\n$313-310=3$\n\n离 313 最近的且比 313 大的 2 的倍数是 314 。\n\n$314-313=1$\n\n313 至少减去 3 是 5 的倍数, 至少加上 1 是 2 的倍数。\n\n【点睛】关键是掌握 2 和 5 的倍数的特征。"} {"id": "24172", "image": [], "answer": "解倍数 因数\n\n【分析】在整数除法中, 如果商是整数而且没有余数, 我们就说被除数是除数和商的倍数, 除数和商是被除数的因数, 据此填空即可。\n\n【详解】据分析可知:\n\n如果 $\\mathrm{a} \\div \\mathrm{b}=5(\\mathrm{a}, \\mathrm{b}$ 是非零自然数),那么 $\\mathrm{a}$ 是 $\\mathrm{b}$ 的倍数, 5 是 $\\mathrm{a}$ 的因数。\n\n【点睛】掌握因数和倍数的概念是解答此题的关键。", "solution": "null", "level": "五年级", "question": "如果 $a \\div b=5$ ( $a, b$ 是非零自然数), 那么 $a$ 是 $b$ 的 $($ ), 5 是 $a$ 的 $($ 。(填“因数”或“倍数”)", "options": [], "subject": "算术", "analysis": "解倍数 因数\n\n【分析】在整数除法中, 如果商是整数而且没有余数, 我们就说被除数是除数和商的倍数, 除数和商是被除数的因数, 据此填空即可。\n\n【详解】据分析可知:\n\n如果 $\\mathrm{a} \\div \\mathrm{b}=5(\\mathrm{a}, \\mathrm{b}$ 是非零自然数),那么 $\\mathrm{a}$ 是 $\\mathrm{b}$ 的倍数, 5 是 $\\mathrm{a}$ 的因数。\n\n【点睛】掌握因数和倍数的概念是解答此题的关键。"} {"id": "24173", "image": [], "answer": "解2914\n【分析】千位上的数字既是偶数也是质数, 这个数是 2 ; 百位上的数字既是奇数又是合数, 这个数是 9 ; 十位上的数字既不是质数也不是合数, 这个数是 1 ; 个位上的数字是最小的合数, 这个数是 4 , 据此解答。\n\n【详解】根据分析可知,\n\n有一个四位数, 它千位上的数字既是偶数也是质数, 百位上的数字既是奇数又是合数, 十位上的数字既不是质数也不是合数, 个位上的数字是最小的合数, 这个数是 2914 。\n\n【点睛】本题是考查万以内整数的写法, 属于基础知识; 应用的知识点有质数与合数的意义,奇数与偶数的意义等。", "solution": "null", "level": "五年级", "question": "一个四位数, 它千位上的数字既是偶数又是质数, 百位上的数字既是奇数又是合数, 十位上的数字既不是质数也不是合数, 个位上的数字是最小的合数, 这个四位数是( )。", "options": [], "subject": "算术", "analysis": "解2914\n【分析】千位上的数字既是偶数也是质数, 这个数是 2 ; 百位上的数字既是奇数又是合数, 这个数是 9 ; 十位上的数字既不是质数也不是合数, 这个数是 1 ; 个位上的数字是最小的合数, 这个数是 4 , 据此解答。\n\n【详解】根据分析可知,\n\n有一个四位数, 它千位上的数字既是偶数也是质数, 百位上的数字既是奇数又是合数, 十位上的数字既不是质数也不是合数, 个位上的数字是最小的合数, 这个数是 2914 。\n\n【点睛】本题是考查万以内整数的写法, 属于基础知识; 应用的知识点有质数与合数的意义,奇数与偶数的意义等。"} {"id": "24174", "image": [], "answer": "解$19 \\quad 11 \\quad 13 \\quad 5$\n\n【分析】一个数(0 除外)的因数只有 1 和它本身两个因数, 这样的数就是质数。据此填空即可。\n\n【详解】由分析可知:\n\n$30=19+11 \\quad 18=13+5$\n\n【点睛】本题考查质数, 明确质数的定义是解题的关键。", "solution": "null", "level": "五年级", "question": "在括号里填上适当的质数。\n\n$30=(\\quad)+(\\quad) \\quad 18=(\\quad)+(\\quad)$", "options": [], "subject": "算术", "analysis": "解$19 \\quad 11 \\quad 13 \\quad 5$\n\n【分析】一个数(0 除外)的因数只有 1 和它本身两个因数, 这样的数就是质数。据此填空即可。\n\n【详解】由分析可知:\n\n$30=19+11 \\quad 18=13+5$\n\n【点睛】本题考查质数, 明确质数的定义是解题的关键。"} {"id": "24194", "image": [], "answer": "解(1)错误", "solution": "null", "level": "五年级", "question": "在 $9 \\div 6=1.5$ 的算式中, 6 和 1.5 是 9 的因数, 9 是 6 和 1.5 的倍数。( )", "options": [], "subject": "算术", "analysis": "解(1)错误"} {"id": "24195", "image": [], "answer": "(1)正确", "solution": "null", "level": "五年级", "question": "一个数最大的因数就是这个数最小的倍数。", "options": [], "subject": "算术", "analysis": "(1)正确"} {"id": "24196", "image": [], "answer": "(1)错误", "solution": "null", "level": "五年级", "question": "一个数越大, 它的因数个数越多, 一个数越小, 它的因数个数就越少。( )", "options": [], "subject": "算术", "analysis": "(1)错误"} {"id": "24197", "image": [], "answer": "(1)错误", "solution": "null", "level": "五年级", "question": "奇数加上 1 后是偶数, 质数减去 1 后是合数。()", "options": [], "subject": "算术", "analysis": "(1)错误"} {"id": "24217", "image": [], "answer": "(1)正确", "solution": "null", "level": "五年级", "question": "两个质数的积一定能被这两个质数同时整除。( )", "options": [], "subject": "算术", "analysis": "(1)正确"} {"id": "24219", "image": [], "answer": "解0\n\n【分析】既是 2 的倍数又是 5 的倍数的特征: 个位上的数字是 0 的数, 既是 2 的倍数, 又是 5 的倍数。\n\n【详解】59口既是 2 的倍数, 又是 5 的倍数, 口里可填 0 。\n\n【点睛】关键是掌握 2 和 5 的倍数的特征。", "solution": "null", "level": "五年级", "question": "59 口既是 2 的倍数, 又是 5 的倍数, 口里可填( )。", "options": [], "subject": "算术", "analysis": "解0\n\n【分析】既是 2 的倍数又是 5 的倍数的特征: 个位上的数字是 0 的数, 既是 2 的倍数, 又是 5 的倍数。\n\n【详解】59口既是 2 的倍数, 又是 5 的倍数, 口里可填 0 。\n\n【点睛】关键是掌握 2 和 5 的倍数的特征。"} {"id": "24220", "image": [], "answer": "解30\n【分析】根据偶数、奇数的意义: 是 2 的倍数的数叫做偶数; 不是 2 的倍数的数叫做奇数; 3 的倍数的数的特征是: 各位上的数字之和是 3 的倍数, 这个数就是 3 的倍数; 5 的倍数的数的特征是:个位上是 0 或 5 的数都是 5 的倍数; 根据 5 的倍数的特征,从 $5 、 10 、 15 、 20 、 25 、 30 、 35 \\cdots \\cdots$ 依次检验,找出是 3 的倍数, 并且是偶数的数即可。由此解答。\n\n【详解】 5 不是 3 的倍数;\n\n10 不是 3 的倍数;\n\n15 是 3 的倍数, 但 15 不是偶数;\n\n20 不是 3 的倍数;\n\n25 不是 3 的倍数;\n\n30 既是 3 的倍数, 又是偶数, 符合要求。\n\n所以既是 3 的倍数又是 5 的倍数的最小偶数是 30 。\n\n【点睛】本题主要考查 $3 、 5$ 倍数的特征及偶数的认识。\n12 ._详解\n$44,94,82,178$\n37,61\n$25,39,44,94,82,178,111$\n\n【分析】(1) 整数中, 是 2 的倍数的数叫做偶数 ( 0 也是偶数), 不是 2 的倍数的数叫做奇数, 最小的奇数是 1 ;\n\n(2)一个数, 如果只有 1 和它本身两个因数, 那么这样的数叫做质数; 一个数, 如果除了 1 和它本身还有别的因数, 那么这样的数叫做合数, 1 既不是质数也不是合数, 据此解答。\n\n【详解】偶数有: $44,94,82,178$;\n\n质数有: 37,61 ;\n\n合数有: $25,39,44,94,82,178,111$ 。\n\n【点睛】掌握奇数、偶数、质数、合数的意义是解答题目的关键。", "solution": "null", "level": "五年级", "question": "既是 3 的倍数又是 5 的倍数的最小偶数是 ( )。", "options": [], "subject": "算术", "analysis": "解30\n【分析】根据偶数、奇数的意义: 是 2 的倍数的数叫做偶数; 不是 2 的倍数的数叫做奇数; 3 的倍数的数的特征是: 各位上的数字之和是 3 的倍数, 这个数就是 3 的倍数; 5 的倍数的数的特征是:个位上是 0 或 5 的数都是 5 的倍数; 根据 5 的倍数的特征,从 $5 、 10 、 15 、 20 、 25 、 30 、 35 \\cdots \\cdots$ 依次检验,找出是 3 的倍数, 并且是偶数的数即可。由此解答。\n\n【详解】 5 不是 3 的倍数;\n\n10 不是 3 的倍数;\n\n15 是 3 的倍数, 但 15 不是偶数;\n\n20 不是 3 的倍数;\n\n25 不是 3 的倍数;\n\n30 既是 3 的倍数, 又是偶数, 符合要求。\n\n所以既是 3 的倍数又是 5 的倍数的最小偶数是 30 。\n\n【点睛】本题主要考查 $3 、 5$ 倍数的特征及偶数的认识。\n12 ._详解\n$44,94,82,178$\n37,61\n$25,39,44,94,82,178,111$\n\n【分析】(1) 整数中, 是 2 的倍数的数叫做偶数 ( 0 也是偶数), 不是 2 的倍数的数叫做奇数, 最小的奇数是 1 ;\n\n(2)一个数, 如果只有 1 和它本身两个因数, 那么这样的数叫做质数; 一个数, 如果除了 1 和它本身还有别的因数, 那么这样的数叫做合数, 1 既不是质数也不是合数, 据此解答。\n\n【详解】偶数有: $44,94,82,178$;\n\n质数有: 37,61 ;\n\n合数有: $25,39,44,94,82,178,111$ 。\n\n【点睛】掌握奇数、偶数、质数、合数的意义是解答题目的关键。"} {"id": "24221", "image": [], "answer": "$44,94,82,178$37,61$25,39,44,94,82,178,111$【分析】(1) 整数中, 是 2 的倍数的数叫做偶数 ( 0 也是偶数), 不是 2 的倍数的数叫做奇数, 最小的奇数是 1 ;(2)一个数, 如果只有 1 和它本身两个因数, 那么这样的数叫做质数; 一个数, 如果除了 1 和它本身还有别的因数, 那么这样的数叫做合数, 1 既不是质数也不是合数, 据此解答。【详解】偶数有: $44,94,82,178$;质数有: 37,61 ;合数有: $25,39,44,94,82,178,111$ 。【点睛】掌握奇数、偶数、质数、合数的意义是解答题目的关键。", "solution": "null", "level": "五年级", "question": "分一分, 把下列各数填入相应的括号内。\n\n$1,25,37,39,44,61,94,82,178,111$\n\n偶数: ); 质数:( ); 合数:(", "options": [], "subject": "算术", "analysis": "$44,94,82,178$37,61$25,39,44,94,82,178,111$【分析】(1) 整数中, 是 2 的倍数的数叫做偶数 ( 0 也是偶数), 不是 2 的倍数的数叫做奇数, 最小的奇数是 1 ;(2)一个数, 如果只有 1 和它本身两个因数, 那么这样的数叫做质数; 一个数, 如果除了 1 和它本身还有别的因数, 那么这样的数叫做合数, 1 既不是质数也不是合数, 据此解答。【详解】偶数有: $44,94,82,178$;质数有: 37,61 ;合数有: $25,39,44,94,82,178,111$ 。【点睛】掌握奇数、偶数、质数、合数的意义是解答题目的关键。"} {"id": "24222", "image": [], "answer": "解奇 合\n\n【分析】整数中, 是 2 的倍数的数叫做偶数, 不是 2 的倍数的数叫做奇数。\n\n一个数, 如果只有 1 和它本身两个因数, 那么这样的数叫做质数; 一个数, 如果除了 1 和它本身还有别的因数, 那么这样的数叫做合数。\n\n【详解】如: 奇数 1 和 $3,1 \\times 3=3,3$ 是奇数;\n\n奇数 5 和 $7,5 \\times 7=35,35$ 是奇数;\n\n合数 4 和 $6,4 \\times 6=24,24$ 是合数;\n\n合数 $9,9 \\times 9=81,81$ 是合数;\n\n所以, 两个奇数的积一定是奇数; 两个合数的积一定是合数。\n\n【点睛】本题考查奇数、合数的意义, 以及奇数与偶数的运算性质。", "solution": "null", "level": "五年级", "question": "两个奇数的积一定是 $($ 数; 两个合数的积一定是 ( )数。", "options": [], "subject": "算术", "analysis": "解奇 合\n\n【分析】整数中, 是 2 的倍数的数叫做偶数, 不是 2 的倍数的数叫做奇数。\n\n一个数, 如果只有 1 和它本身两个因数, 那么这样的数叫做质数; 一个数, 如果除了 1 和它本身还有别的因数, 那么这样的数叫做合数。\n\n【详解】如: 奇数 1 和 $3,1 \\times 3=3,3$ 是奇数;\n\n奇数 5 和 $7,5 \\times 7=35,35$ 是奇数;\n\n合数 4 和 $6,4 \\times 6=24,24$ 是合数;\n\n合数 $9,9 \\times 9=81,81$ 是合数;\n\n所以, 两个奇数的积一定是奇数; 两个合数的积一定是合数。\n\n【点睛】本题考查奇数、合数的意义, 以及奇数与偶数的运算性质。"} {"id": "24223", "image": [], "answer": "解偶\n\n【分析】整数中, 是 2 的倍数的数叫做偶数, 不是 2 的倍数的数叫做奇数。\n\n根据奇数 + 奇数 $=$ 偶数, 偶数 + 奇数 $=$ 奇数, 偶数 + 偶数 $=$ 偶数, 据此解答。\n\n【详解】 5 是奇数, 和是奇数, 因为偶数 + 奇数 $=$ 奇数, 所以 $\\mathrm{a}+5$ 的和是奇数, $\\mathrm{a}$ 一定是偶数。\n\n【点睛】本题考查奇数与偶数的认识, 以及奇数与偶数的运算性质。", "solution": "null", "level": "五年级", "question": "$a+5$ 的和是奇数, $a$ 一定是 $(\\quad)$ 数。", "options": [], "subject": "算术", "analysis": "解偶\n\n【分析】整数中, 是 2 的倍数的数叫做偶数, 不是 2 的倍数的数叫做奇数。\n\n根据奇数 + 奇数 $=$ 偶数, 偶数 + 奇数 $=$ 奇数, 偶数 + 偶数 $=$ 偶数, 据此解答。\n\n【详解】 5 是奇数, 和是奇数, 因为偶数 + 奇数 $=$ 奇数, 所以 $\\mathrm{a}+5$ 的和是奇数, $\\mathrm{a}$ 一定是偶数。\n\n【点睛】本题考查奇数与偶数的认识, 以及奇数与偶数的运算性质。"} {"id": "24224", "image": [], "answer": "解倍数 因数\n\n【分析】在整数除法中, 如果商是整数而没有余数, 我们就说被除数是除数和商的倍数, 除数和商是被除数的因数, 据此解答。\n\n【详解】分析可知, $\\mathrm{a} \\div \\mathrm{b}=\\mathrm{c}$ ( $\\mathrm{a} 、 \\mathrm{~b}$ 、 $\\mathrm{c}$ 都是不为 0 的自然数 $)$, 那么 $\\mathrm{a}$ 是 $\\mathrm{b}$ 和 $\\mathrm{c}$ 的倍数, $\\mathrm{b}$ 和 $\\mathrm{c}$ 是 $\\mathrm{a}$的因数。\n\n【点睛】本题主要考查因数和倍数的认识, 理解因数和倍数的含义是解答题目的关键。", "solution": "null", "level": "五年级", "question": "已知 $a \\div b=c$ ( $a 、 b 、 c$ 都是不为 0 的自然数),那么 $a$ 是 $b$ 和 $c$ 的 ( ),", "options": [], "subject": "算术", "analysis": "解倍数 因数\n\n【分析】在整数除法中, 如果商是整数而没有余数, 我们就说被除数是除数和商的倍数, 除数和商是被除数的因数, 据此解答。\n\n【详解】分析可知, $\\mathrm{a} \\div \\mathrm{b}=\\mathrm{c}$ ( $\\mathrm{a} 、 \\mathrm{~b}$ 、 $\\mathrm{c}$ 都是不为 0 的自然数 $)$, 那么 $\\mathrm{a}$ 是 $\\mathrm{b}$ 和 $\\mathrm{c}$ 的倍数, $\\mathrm{b}$ 和 $\\mathrm{c}$ 是 $\\mathrm{a}$的因数。\n\n【点睛】本题主要考查因数和倍数的认识, 理解因数和倍数的含义是解答题目的关键。"} {"id": "24225", "image": [], "answer": "解$100 \\quad 95$\n\n【分析】整数中, 是 2 的倍数的数叫偶数, 不是 2 的倍数的数叫奇数。除了 1 和它本身以外不再有其他因数, 这样的数叫质数; 除了 1 和它本身以外还有其他因数, 这样的数叫合数。\n\n据此确定 100 以内最大奇数和最小奇数就和, 最大质数和最小质数就差即可。\n\n【详解】 100 以内最大的奇数是 99 、最小奇数是 1 ; 最大的质数是 97 、最小的质数是 2 。\n\n$99+1=100$\n\n$97-2=95$\n\n100 以内最大的奇数与最小奇数的和是 100,100 以内最大的质数与最小的质数的差是 95 。\n\n【点睛】关键是理解奇数、偶数、质数、合数的分类标准。", "solution": "null", "level": "五年级", "question": "100 以内最大的奇数与最小奇数的和是 (\n\n), 100 以内最大的质数与最小的质数的差是\n\n( )。", "options": [], "subject": "算术", "analysis": "解$100 \\quad 95$\n\n【分析】整数中, 是 2 的倍数的数叫偶数, 不是 2 的倍数的数叫奇数。除了 1 和它本身以外不再有其他因数, 这样的数叫质数; 除了 1 和它本身以外还有其他因数, 这样的数叫合数。\n\n据此确定 100 以内最大奇数和最小奇数就和, 最大质数和最小质数就差即可。\n\n【详解】 100 以内最大的奇数是 99 、最小奇数是 1 ; 最大的质数是 97 、最小的质数是 2 。\n\n$99+1=100$\n\n$97-2=95$\n\n100 以内最大的奇数与最小奇数的和是 100,100 以内最大的质数与最小的质数的差是 95 。\n\n【点睛】关键是理解奇数、偶数、质数、合数的分类标准。"} {"id": "24243", "image": [], "answer": "$1,3,5,7,9 、 11 、 13 、 15,17,19 \\quad 4,6,8,9 、 10 、 12,14,15,16,18,20 \\quad 2 \\quad 9 、$ 15 2, 3、5、7、11、13、17、19\n\n【分析】一个数不能被 2 整除, 这样的数就是奇数; 一个数可以被 2 整除, 这样的数就是偶数; 一个数 ( 0 除外) 的因数只有 1 和它本身两个, 这样的数就是质数; 一个数 ( 0 除外) 的因数除了 1 和它本身以外, 还有其他的因数, 这样的数就是合数; 据此填空即可。\n\n【详解】由分析可知:\n\n自然数 $1-20$ 中, 奇数有 $1 、 3 、 5 、 7 、 9 、 11 、 13 、 15 、 17 、 19$, 合数有 $4 、 6 、 8 、 9 、 10 、 12 、 14 、$ $15 、 16 、 18 、 20$, 是偶数又是质数的是 2 , 是奇数又是合数的有 $9 、 15$, 质数有 $2 、 3 、 5 、 7 、 11 、 13$ 、 17、19。\n\n【点睛】本题考查奇数、偶数、质数和合数, 明确它们的定义是解题的关键。", "solution": "null", "level": "五年级", "question": "自然数 $1-20$ 中, 奇数有 (\n), 合数有(\n), 是偶数又是质数的是(\n), 是奇\n数又是合数的有 ( ), 质数有 ( )。", "options": [], "subject": "算术", "analysis": "$1,3,5,7,9 、 11 、 13 、 15,17,19 \\quad 4,6,8,9 、 10 、 12,14,15,16,18,20 \\quad 2 \\quad 9 、$ 15 2, 3、5、7、11、13、17、19\n\n【分析】一个数不能被 2 整除, 这样的数就是奇数; 一个数可以被 2 整除, 这样的数就是偶数; 一个数 ( 0 除外) 的因数只有 1 和它本身两个, 这样的数就是质数; 一个数 ( 0 除外) 的因数除了 1 和它本身以外, 还有其他的因数, 这样的数就是合数; 据此填空即可。\n\n【详解】由分析可知:\n\n自然数 $1-20$ 中, 奇数有 $1 、 3 、 5 、 7 、 9 、 11 、 13 、 15 、 17 、 19$, 合数有 $4 、 6 、 8 、 9 、 10 、 12 、 14 、$ $15 、 16 、 18 、 20$, 是偶数又是质数的是 2 , 是奇数又是合数的有 $9 、 15$, 质数有 $2 、 3 、 5 、 7 、 11 、 13$ 、 17、19。\n\n【点睛】本题考查奇数、偶数、质数和合数, 明确它们的定义是解题的关键。"} {"id": "24245", "image": [], "answer": "解249\n\n【分析】一个数, 如果只有 1 和它本身两个因数, 那么这样的数叫做质数; 一个数, 如果除了 1 和它本身还有别的因数, 那么这样的数叫做合数。\n\n整数中, 是 2 的倍数的数叫做偶数, 不是 2 的倍数的数叫做奇数。\n\n【详解】一个三位数, 百位上的数既是质数又是偶数, 即 2 ;\n\n十位上的数是最小的合数, 即 4;\n\n个位上的数是最大的一位数, 即 9 ;\n\n这个数是 249 。\n\n【点睛】本题考查质数与合数、奇数与偶数的意义及应用。", "solution": "null", "level": "五年级", "question": "一个三位数, 百位上的数既是质数又是偶数, 十位上的数是最小的合数, 个位上的数是最大的一位数, 这个数是 ( )。", "options": [], "subject": "算术", "analysis": "解249\n\n【分析】一个数, 如果只有 1 和它本身两个因数, 那么这样的数叫做质数; 一个数, 如果除了 1 和它本身还有别的因数, 那么这样的数叫做合数。\n\n整数中, 是 2 的倍数的数叫做偶数, 不是 2 的倍数的数叫做奇数。\n\n【详解】一个三位数, 百位上的数既是质数又是偶数, 即 2 ;\n\n十位上的数是最小的合数, 即 4;\n\n个位上的数是最大的一位数, 即 9 ;\n\n这个数是 249 。\n\n【点睛】本题考查质数与合数、奇数与偶数的意义及应用。"} {"id": "24246", "image": [], "answer": "解$96 \\quad 12$\n\n【分析】个位上是 $0,2,4,6,8$ 的数都是 2 的倍数。一个数各位上的数的和是 3 的倍数, 这个数就是 3 的倍数。要想得到最大两位数, 十位上可先考虑是 9 ; 要想得到最小两位数, 十位上可先考虑是 1。再根据 2、3 的倍数的特征解答。\n【详解】根据 2 的倍数的特征可知: 这个两位数的个位上是 $0,2,4,6,8$ 。\n\n最大两位数: 十位上最大是 $9,9+8=17,17$ 不是 3 的倍数, 所以 98 不是 3 的倍数; $9+6=15,15$是 3 的倍数, 所以 96 是 3 的倍数。\n\n最小两位数: 十位上最小是 $1,1+0=1,1$ 不是 3 的倍数, 所以 10 不是 3 的倍数; $1+1=2,2$ 不是 3 的倍数, 所以 11 不是 3 的倍数; $1+2=3,3$ 是 3 的倍数, 所以 12 是 3 的倍数。\n\n所以既是 2 的倍数, 又是 3 的倍数的最大两位数是 96 , 最小两位数是 12 。\n\n【点睛】要想组成最大的数, 就要把尽可能大的数字填在高位; 要想组成最小的数, 就要把尽可能小的数字填在高位,同时注意是否具备倍数的特征。", "solution": "null", "level": "五年级", "question": "既是 2 的倍数, 又是 3 的倍数的最大两位数是 ( ), 最小两位数是 $($ 。", "options": [], "subject": "算术", "analysis": "解$96 \\quad 12$\n\n【分析】个位上是 $0,2,4,6,8$ 的数都是 2 的倍数。一个数各位上的数的和是 3 的倍数, 这个数就是 3 的倍数。要想得到最大两位数, 十位上可先考虑是 9 ; 要想得到最小两位数, 十位上可先考虑是 1。再根据 2、3 的倍数的特征解答。\n【详解】根据 2 的倍数的特征可知: 这个两位数的个位上是 $0,2,4,6,8$ 。\n\n最大两位数: 十位上最大是 $9,9+8=17,17$ 不是 3 的倍数, 所以 98 不是 3 的倍数; $9+6=15,15$是 3 的倍数, 所以 96 是 3 的倍数。\n\n最小两位数: 十位上最小是 $1,1+0=1,1$ 不是 3 的倍数, 所以 10 不是 3 的倍数; $1+1=2,2$ 不是 3 的倍数, 所以 11 不是 3 的倍数; $1+2=3,3$ 是 3 的倍数, 所以 12 是 3 的倍数。\n\n所以既是 2 的倍数, 又是 3 的倍数的最大两位数是 96 , 最小两位数是 12 。\n\n【点睛】要想组成最大的数, 就要把尽可能大的数字填在高位; 要想组成最小的数, 就要把尽可能小的数字填在高位,同时注意是否具备倍数的特征。"} {"id": "24247", "image": ["12457.jpg", "12458.jpg", "12458.jpg"], "answer": "解$12 \\quad 60$\n\n【分析】找一个数的因数的方法:列乘法算式找因数,按照从小到大的顺序,一组一组地写出所有积是这个数的乘法算式,乘法算式中的两个因数就是这个数的因数。找一个数的倍数的方法:列乘法算式找倍数, 按照从小到大的顺序, 一组一组地写出这个数与非 0 自然数的乘法算式, 乘法算式中的积就是这个数的倍数。据此写出 60 的所有因数, 再写出 60 以内 4 和 6 的倍数, 同时满足这 3 个条件的数即可。\n\n\n\n4 的倍数: $4 、 8 、 12 、 16 、 20 、 24 、 28 、 32 、 36 、 40 、 44 、 48 、 52 、 56 、 60$ 。\n\n6 的倍数: $6 、 12 、 18 、 24 、 30 、 36 、 42 、 48 、 54 、 60$ 。\n\n所以这样的数有 $12 、 60$ 。\n\n【点睛】此题的解题关键是掌握求一个数的因数和倍数的方法。", "solution": "null", "level": "五年级", "question": "你能写出这样的数吗?\n\n\n\n这样的数有 ( ) 和( 。", "options": [], "subject": "算术", "analysis": "解$12 \\quad 60$\n\n【分析】找一个数的因数的方法:列乘法算式找因数,按照从小到大的顺序,一组一组地写出所有积是这个数的乘法算式,乘法算式中的两个因数就是这个数的因数。找一个数的倍数的方法:列乘法算式找倍数, 按照从小到大的顺序, 一组一组地写出这个数与非 0 自然数的乘法算式, 乘法算式中的积就是这个数的倍数。据此写出 60 的所有因数, 再写出 60 以内 4 和 6 的倍数, 同时满足这 3 个条件的数即可。\n\n\n\n4 的倍数: $4 、 8 、 12 、 16 、 20 、 24 、 28 、 32 、 36 、 40 、 44 、 48 、 52 、 56 、 60$ 。\n\n6 的倍数: $6 、 12 、 18 、 24 、 30 、 36 、 42 、 48 、 54 、 60$ 。\n\n所以这样的数有 $12 、 60$ 。\n\n【点睛】此题的解题关键是掌握求一个数的因数和倍数的方法。"} {"id": "24248", "image": ["12459.jpg", "12459.jpg"], "answer": "解$\\quad 3 \\quad 23 \\quad 7 \\quad 19 \\quad 13 \\quad 13$\n\n【分析】质数是指除了 1 和它本身的两个因数以外再没有其他的因数的数。合数是指就除了 1 和它本身的两个因数以外还有其他的因数的数。据此可以把合数 26 分成两个质数的和。\n\n\n\n$26=3+23=7+19=13+13$ 。\n\n【点睛】根据质数的意义进行解答, 需要熟悉一些常用的质数。", "solution": "null", "level": "五年级", "question": "在括号里填上合适的质数。\n\n$26=(\\quad)+(\\quad)=(\\quad)+(\\quad)+(\\quad)$", "options": [], "subject": "算术", "analysis": "解$\\quad 3 \\quad 23 \\quad 7 \\quad 19 \\quad 13 \\quad 13$\n\n【分析】质数是指除了 1 和它本身的两个因数以外再没有其他的因数的数。合数是指就除了 1 和它本身的两个因数以外还有其他的因数的数。据此可以把合数 26 分成两个质数的和。\n\n\n\n$26=3+23=7+19=13+13$ 。\n\n【点睛】根据质数的意义进行解答, 需要熟悉一些常用的质数。"} {"id": "24249", "image": [], "answer": "解3\n\n【分析】一个数各位上的数的和是 3 的倍数, 这个数就是 3 的倍数。根据 3 的倍数的特征解答即可。\n\n【详解】 $5+6=11,11+1=12,12$ 是 3 的倍数, 所以个位上的数字可以是 $1 \\circ 1+3=4,4+3=7$,所以个位上的数字还可以是 4,7 。即口里可以填 $1,4,7$ 。所以共有 3 种填法。\n\n【点睛】明确 3 的倍数的特征是解决此题的关键。", "solution": "null", "level": "五年级", "question": "在“ 56 \"”这个三位数的 $\\square$ 里填上一个数字,使这个三位数是 3 的倍数, 共有 $($ 种填法。", "options": [], "subject": "算术", "analysis": "解3\n\n【分析】一个数各位上的数的和是 3 的倍数, 这个数就是 3 的倍数。根据 3 的倍数的特征解答即可。\n\n【详解】 $5+6=11,11+1=12,12$ 是 3 的倍数, 所以个位上的数字可以是 $1 \\circ 1+3=4,4+3=7$,所以个位上的数字还可以是 4,7 。即口里可以填 $1,4,7$ 。所以共有 3 种填法。\n\n【点睛】明确 3 的倍数的特征是解决此题的关键。"} {"id": "24268", "image": [], "answer": "解(1) 200\n\n(2) 0 0 $\\quad 0$\n\n(3) $00 \\quad 1 \\quad 1 \\quad 0$\n\n【分析】 2 的倍数特征:个位上是 $0 、 2 、 4 、 6 、 8$ 的数。\n\n5 的倍数特征: 个位上是 0 或 5 的数。\n\n2、 5 的倍数特征: 个位上是 0 的数。\n\n3 的倍数特征: 各个数位上的数字相加, 和要能被 3 整除。\n\n【详解】(1) 是 3 的倍数: $72 、 102 、 30$ 。\n\n(答案不唯一)\n\n(2)既是 2 的倍数, 又是 5 的倍数:30、20、140。\n\n(3) 同时是 $2 、 3 、 5$ 的倍数: $180 、 210 、 510 、 360$ 。\n\n(第 $2 、 4$ 空的答案不唯一)\n\n【点睛】掌握 $2 、 3 、 5$ 的倍数特征是解题的关键。", "solution": "null", "level": "五年级", "question": "按要求填上适当的数字。\n\n(1)是 3 的倍数: $7($ )、 $1($ )\n\n(2)既是 2 的倍数, 又是 5 的倍数: $3($ )、 $2($ ) 14(\n(3)同时是 2、3、5 的倍数: 18 (\n)、2(\n)0、5(\n)0、36( )。", "options": [], "subject": "算术", "analysis": "解(1) 200\n\n(2) 0 0 $\\quad 0$\n\n(3) $00 \\quad 1 \\quad 1 \\quad 0$\n\n【分析】 2 的倍数特征:个位上是 $0 、 2 、 4 、 6 、 8$ 的数。\n\n5 的倍数特征: 个位上是 0 或 5 的数。\n\n2、 5 的倍数特征: 个位上是 0 的数。\n\n3 的倍数特征: 各个数位上的数字相加, 和要能被 3 整除。\n\n【详解】(1) 是 3 的倍数: $72 、 102 、 30$ 。\n\n(答案不唯一)\n\n(2)既是 2 的倍数, 又是 5 的倍数:30、20、140。\n\n(3) 同时是 $2 、 3 、 5$ 的倍数: $180 、 210 、 510 、 360$ 。\n\n(第 $2 、 4$ 空的答案不唯一)\n\n【点睛】掌握 $2 、 3 、 5$ 的倍数特征是解题的关键。"} {"id": "24505", "image": [], "answer": "解12\n\n【分析】由于 $1 \\mathrm{~L}=1000 \\mathrm{~mL}$, 则 $3 \\mathrm{~L}=3000 \\mathrm{~mL}$, 由题意可知, 用水的量除以水瓶的容积即可求解。\n\n【详解】 3 升 $=3000$ 毫升\n$3000 \\div 250=12$ (瓶)\n\n则能装 12 瓶。\n\n【点睛】本题考查容积单位, 明确容积单位之间的进率是解题的关键。", "solution": "null", "level": "五年级", "question": "将 3 升水装入容积是 250 毫升的水瓶里, 能装(\n)瓶。", "options": [], "subject": "算术", "analysis": "解12\n\n【分析】由于 $1 \\mathrm{~L}=1000 \\mathrm{~mL}$, 则 $3 \\mathrm{~L}=3000 \\mathrm{~mL}$, 由题意可知, 用水的量除以水瓶的容积即可求解。\n\n【详解】 3 升 $=3000$ 毫升\n$3000 \\div 250=12$ (瓶)\n\n则能装 12 瓶。\n\n【点睛】本题考查容积单位, 明确容积单位之间的进率是解题的关键。"} {"id": "24534", "image": [], "answer": "解300\n\n【分析】由题意可知, 两个完全一样的正方体拼成一个长方体后, 表面积比原来减少了两个正方形的面积, 也就是 60 平方厘米, 据此求出一个正方形的面积;拼成长方体后原来的两个正方体变成了 10 个面,用一个正方形的面积乘 10 即可求出这个长方体的表面积。\n\n【详解】 $60 \\div 2 \\times 10$\n\n$=30 \\times 10$\n\n$=300$ (平方厘米)\n\n则这个长方体的表面积是 300 平方厘米。\n\n【点睛】本题考查长方体和正方体的表面积, 明确两个完全一样的正方体拼成一个长方体后, 表面积比原来减少了两个正方形的面积是解题的关键。", "solution": "null", "level": "五年级", "question": "两个完全一样的正方体拼成一个长方体后, 表面积比原来减少了 60 平方厘米。这个长方体的表面积是 $(\\quad)$ 平方厘米。", "options": [], "subject": "算术", "analysis": "解300\n\n【分析】由题意可知, 两个完全一样的正方体拼成一个长方体后, 表面积比原来减少了两个正方形的面积, 也就是 60 平方厘米, 据此求出一个正方形的面积;拼成长方体后原来的两个正方体变成了 10 个面,用一个正方形的面积乘 10 即可求出这个长方体的表面积。\n\n【详解】 $60 \\div 2 \\times 10$\n\n$=30 \\times 10$\n\n$=300$ (平方厘米)\n\n则这个长方体的表面积是 300 平方厘米。\n\n【点睛】本题考查长方体和正方体的表面积, 明确两个完全一样的正方体拼成一个长方体后, 表面积比原来减少了两个正方形的面积是解题的关键。"} {"id": "24270", "image": [], "answer": "解$8 \\quad 2$\n\n【分析】质数是指除了 1 和它本身的两个因数以外再没有其他的因数的数。合数是指就除了 1 和它本身的两个因数以外还有其他的因数的数。不能被 2 整除的自然数叫奇数, 能被 2 整除的自然数叫偶数。据此写出 20 以内所有的质数即可得解; 再写出 20 以内所有的奇数和所有的合数, 找出既是奇数又是合数的数即可。\n\n【详解】 20 以内的质数有: $2 、 3 、 5 、 7 、 11 、 13 、 17 、 19$; 共有 8 个。\n\n20 以内的奇数有: $1 、 3 、 5 、 7 、 9 、 11 、 13 、 15 、 17 、 19$;\n\n20 以内的合数有: $4 、 6 、 8 、 9 、 10 、 12 、 14 、 15 、 16 、 18 、 20$;\n\n所以既是奇数又是合数的数有: $9 、 15$; 共有 2 个。\n\n【点睛】此题主要明确奇数与偶数、质数与合数的定义, 以及奇数与质数、偶数与合数的区别, 才能做出正确的解答。", "solution": "null", "level": "五年级", "question": "20 以内的质数共有 ( ) 个。在 20 以内(包含 20 ), 既是奇数又是合数的数有 ( )\n个。", "options": [], "subject": "算术", "analysis": "解$8 \\quad 2$\n\n【分析】质数是指除了 1 和它本身的两个因数以外再没有其他的因数的数。合数是指就除了 1 和它本身的两个因数以外还有其他的因数的数。不能被 2 整除的自然数叫奇数, 能被 2 整除的自然数叫偶数。据此写出 20 以内所有的质数即可得解; 再写出 20 以内所有的奇数和所有的合数, 找出既是奇数又是合数的数即可。\n\n【详解】 20 以内的质数有: $2 、 3 、 5 、 7 、 11 、 13 、 17 、 19$; 共有 8 个。\n\n20 以内的奇数有: $1 、 3 、 5 、 7 、 9 、 11 、 13 、 15 、 17 、 19$;\n\n20 以内的合数有: $4 、 6 、 8 、 9 、 10 、 12 、 14 、 15 、 16 、 18 、 20$;\n\n所以既是奇数又是合数的数有: $9 、 15$; 共有 2 个。\n\n【点睛】此题主要明确奇数与偶数、质数与合数的定义, 以及奇数与质数、偶数与合数的区别, 才能做出正确的解答。"} {"id": "24271", "image": [], "answer": "解$\\mathrm{n}-1 \\quad \\mathrm{n}+1$\n\n【分析】不能被 2 整除的自然数叫奇数, 能被 2 整除的自然数叫偶数。据此可知, 相邻的三个自然数,要么有两个是奇数, 一个是偶数, 要么有两个是偶数, 一个是奇数; 所以与奇数相邻的两个偶数, 其中一个比奇数小 1 , 另一个比奇数大 1 , 据此解答。\n\n【详解】根据分析得, 如果 $\\mathrm{n}$ 表示一个奇数, 那么与 $\\mathrm{n}$ 相邻的两个偶数分别是: $\\mathrm{n}-1$ 和 $\\mathrm{n}+1$ 。\n\n【点睛】此题的解题关键是理解掌握奇数和偶数的意义以及用字母表示数的方法。", "solution": "null", "level": "五年级", "question": "如果 $\\mathrm{n}$ 表示一个奇数, 那么与它相邻的两个偶数可以表示为( ) 和( )。", "options": [], "subject": "算术", "analysis": "解$\\mathrm{n}-1 \\quad \\mathrm{n}+1$\n\n【分析】不能被 2 整除的自然数叫奇数, 能被 2 整除的自然数叫偶数。据此可知, 相邻的三个自然数,要么有两个是奇数, 一个是偶数, 要么有两个是偶数, 一个是奇数; 所以与奇数相邻的两个偶数, 其中一个比奇数小 1 , 另一个比奇数大 1 , 据此解答。\n\n【详解】根据分析得, 如果 $\\mathrm{n}$ 表示一个奇数, 那么与 $\\mathrm{n}$ 相邻的两个偶数分别是: $\\mathrm{n}-1$ 和 $\\mathrm{n}+1$ 。\n\n【点睛】此题的解题关键是理解掌握奇数和偶数的意义以及用字母表示数的方法。"} {"id": "24272", "image": [], "answer": "解2410\n【分析】质数是指除了 1 和它本身的两个因数以外再没有其他的因数的数。合数是指除了 1 和它本身的两个因数以外还有其他的因数的数。不能被 2 整除的自然数叫奇数, 能被 2 整除的自然数叫偶数。最小的质数是 2 , 最小的合数是 4 , 最小的奇数是 1 , 最小的偶数是 0 , 再按照整数的写法, 写出这个四位数, 据此解答。\n\n【详解】根据分析得, 千位上的数是 2 , 百位上的数是 4 , 十位上的数是 1 , 个位上的数是 0 , 这个数是 2410。\n\n【点睛】此题主要明确奇数与偶数、质数与合数的定义, 熟悉一些常用的数, 通过整数的写法, 做出正确的解答。", "solution": "null", "level": "五年级", "question": "一个四位数, 千位上的数是最小的质数, 百位上的数是最小的合数, 十位上的数是最小的奇数,个位上的数是最小的偶数, 这个数是( )。", "options": [], "subject": "算术", "analysis": "解2410\n【分析】质数是指除了 1 和它本身的两个因数以外再没有其他的因数的数。合数是指除了 1 和它本身的两个因数以外还有其他的因数的数。不能被 2 整除的自然数叫奇数, 能被 2 整除的自然数叫偶数。最小的质数是 2 , 最小的合数是 4 , 最小的奇数是 1 , 最小的偶数是 0 , 再按照整数的写法, 写出这个四位数, 据此解答。\n\n【详解】根据分析得, 千位上的数是 2 , 百位上的数是 4 , 十位上的数是 1 , 个位上的数是 0 , 这个数是 2410。\n\n【点睛】此题主要明确奇数与偶数、质数与合数的定义, 熟悉一些常用的数, 通过整数的写法, 做出正确的解答。"} {"id": "24273", "image": [], "answer": "解90\n\n【分析】根据 2 的倍数的特征, 一个数的个位如果是偶数, 这个数就是 2 的倍数; 根据 5 的倍数的特征, 一个数的个位是 0 或 5 , 这个数就是 5 的倍数; 要想同时是 $2 、 5$ 的倍数, 这个数的个位一定是 0 ,个位上是 0 的最大两位数是 90 。\n\n【详解】一个两位数, 既是 2 的倍数又是 5 的倍数, 这个数最大是 90 。\n\n【点睛】本题是考查 $2 、 5$ 的倍数特征, 关键是抓住同时是 $2 、 5$ 的倍数个位一定是 0 这一特征。", "solution": "null", "level": "五年级", "question": "一个两位数, 既是 2 的倍数又是 5 的倍数, 这个数最大是 ( )。", "options": [], "subject": "算术", "analysis": "解90\n\n【分析】根据 2 的倍数的特征, 一个数的个位如果是偶数, 这个数就是 2 的倍数; 根据 5 的倍数的特征, 一个数的个位是 0 或 5 , 这个数就是 5 的倍数; 要想同时是 $2 、 5$ 的倍数, 这个数的个位一定是 0 ,个位上是 0 的最大两位数是 90 。\n\n【详解】一个两位数, 既是 2 的倍数又是 5 的倍数, 这个数最大是 90 。\n\n【点睛】本题是考查 $2 、 5$ 的倍数特征, 关键是抓住同时是 $2 、 5$ 的倍数个位一定是 0 这一特征。"} {"id": "24274", "image": [], "answer": "解$1 \\quad 1,4 、 7$\n\n【分析】 3 的倍数特征:各个数位上的数字和是 3 的倍数; 2 的倍数特征:个位是 $0 、 2 、 4 、 6$ 或 8 ; 5 的倍数特征: 个位是 0 或 5 ; 同时是 $2 、 3 、 5$ 的倍数特征: 个位上是 0 , 且各个数位上的数字和是 3 的倍数。据此解答。\n\n【详解】 $2+5+1=8$\n\n比 8 大的 3 的倍数有 $9 、 12 、 15 \\ldots$\n\n$9-8=1$\n\n所以如果 251口是 3 的倍数, 口里最小能填 1 ;\n\n$2 \\square 0$ 的个位上是 0 , 方框里的只能填小于 10 的自然数;\n\n比 2 大的 3 的倍数有 $3 、 6 、 9 、 12 \\ldots$\n\n$3-2=1$\n\n$6-2=4$\n\n$9-2=7$\n\n$2 \\square 0$ 是 $2 、 3 、 5$ 的倍数, $\\square$ 里能填 $1 、 4 、 7$ 。\n\n【点睛】本题考查了 $2 、 3 、 5$ 的倍数特征。", "solution": "null", "level": "五年级", "question": "如果 251口是 3 的倍数, 口里最小能填( ); $2 \\square 0$ 是 2、3、 5 的倍数, $\\square$ 里能填 ( )。", "options": [], "subject": "算术", "analysis": "解$1 \\quad 1,4 、 7$\n\n【分析】 3 的倍数特征:各个数位上的数字和是 3 的倍数; 2 的倍数特征:个位是 $0 、 2 、 4 、 6$ 或 8 ; 5 的倍数特征: 个位是 0 或 5 ; 同时是 $2 、 3 、 5$ 的倍数特征: 个位上是 0 , 且各个数位上的数字和是 3 的倍数。据此解答。\n\n【详解】 $2+5+1=8$\n\n比 8 大的 3 的倍数有 $9 、 12 、 15 \\ldots$\n\n$9-8=1$\n\n所以如果 251口是 3 的倍数, 口里最小能填 1 ;\n\n$2 \\square 0$ 的个位上是 0 , 方框里的只能填小于 10 的自然数;\n\n比 2 大的 3 的倍数有 $3 、 6 、 9 、 12 \\ldots$\n\n$3-2=1$\n\n$6-2=4$\n\n$9-2=7$\n\n$2 \\square 0$ 是 $2 、 3 、 5$ 的倍数, $\\square$ 里能填 $1 、 4 、 7$ 。\n\n【点睛】本题考查了 $2 、 3 、 5$ 的倍数特征。"} {"id": "24276", "image": [], "answer": "解25\n\n【分析】 5 的倍数特征:个位上是 0 或 5 的数; 可据此结合题意具体分析。\n\n【详解】因为是 5 的倍数, 个位只能是 0 或者 5 ;当个位是 0 时, 十位是: $7-0=7$, 这个数是 70 ;当个位是 5 时, 十位是: $7-5=2$, 这个数是 25 ; $25<70$, 因此这个两位数最小是 25 。\n\n【点睛】此题需要学生熟练掌握 5 的倍数特征并灵活运用。", "solution": "null", "level": "五年级", "question": "有一个两位数, 它是 5 的倍数, 个位和十位上的数字之和是 7 , 这个两位数最小是 ( )。", "options": [], "subject": "算术", "analysis": "解25\n\n【分析】 5 的倍数特征:个位上是 0 或 5 的数; 可据此结合题意具体分析。\n\n【详解】因为是 5 的倍数, 个位只能是 0 或者 5 ;当个位是 0 时, 十位是: $7-0=7$, 这个数是 70 ;当个位是 5 时, 十位是: $7-5=2$, 这个数是 25 ; $25<70$, 因此这个两位数最小是 25 。\n\n【点睛】此题需要学生熟练掌握 5 的倍数特征并灵活运用。"} {"id": "24295", "image": [], "answer": "解$\\quad 9 \\quad 7 \\quad 63$\n\n【分析】在整除除法中, 商是整除且没有余数, 我们就说商和除数是被除数的因数, 被除数是商和除数的因数, 据此填空即可。\n\n【详解】由分析可知:\n\n$63 \\div 9=7,9$ 和 7 是 63 的因数, 63 是 9 的倍数。\n\n【点睛】本题考查因数和倍数, 明确因数和倍数的定义是解题的关键。", "solution": "null", "level": "五年级", "question": "$63 \\div 9=7$, ( ) 和( 是 63 的因数, ( )是 9 的倍数。", "options": [], "subject": "算术", "analysis": "解$\\quad 9 \\quad 7 \\quad 63$\n\n【分析】在整除除法中, 商是整除且没有余数, 我们就说商和除数是被除数的因数, 被除数是商和除数的因数, 据此填空即可。\n\n【详解】由分析可知:\n\n$63 \\div 9=7,9$ 和 7 是 63 的因数, 63 是 9 的倍数。\n\n【点睛】本题考查因数和倍数, 明确因数和倍数的定义是解题的关键。"} {"id": "24297", "image": ["12460.jpg"], "answer": "解(1) $17,31 \\quad 25 、 60 、 24 、 9 \\quad 60 、 24 \\quad 60$\n\n(2)奇\n\n【分析】(1)质数:在大于 1 的自然数中, 除了 1 和它本身外, 不能再被其他数整除的数, 叫质数;如: 2、3、5、7...都是质数, 也叫素数;\n\n合数:自然数中除了能被 1 和本身整除外, 还能被其他的数整除的数叫合数, 如: $4 、 6 、 8 \\ldots$ 都是合数;\n\n2 的倍数: 个位上是 $0 、 2 、 4 、 6 、 8$ 的数;\n\n能同时被 3 和 5 整除的数:个位上是 0 或 5 , 且各个数位上的数的和是 3 的倍数。据此解答。\n\n(2)奇数: 不能被 2 整除的自然数叫奇数。如:1、3、5、7、9...\n\n偶数:能被 2 整除的自然数叫偶数。如: $2 、 4 、 6 、 8 、 10 \\ldots$\n\n可据此找出奇数、偶数, 然后看哪类数的个数较多,摸到这类数的可能性就大。\n\n(1)\n\n质数有: $17 、 31$\n\n合数有: $25 、 60 、 24 、 9$\n\n2 的倍数有: $60 、 24$\n\n能同时被 3 和 5 整除的数有: 60\n\n奇数有: $17 、 25 、 9 、 31$, 共计 4 个;\n\n偶数有: 60、24, 共计 2 个;\n\n$4>2$, 所以摸到奇数的可能性大。\n\n【点睛】熟悉质数合数、奇偶数的相关概念,同时明确,事件发生的可能性大小与参加的个体数量有\n关, 个体在总数中所占数量越多, 出现的可能性就越大; 反之, 个体在总数中所占数量越少, 出现的可能性就越小。", "solution": "null", "level": "五年级", "question": "看下图填空。\n\n\n\n24\n\n(1)上面数字卡片上的数中, (\n\n同时能被 3 和 5 整除。\n\n(2)把这几张数字卡片反扣在桌子上, 从中任意摸出 1 张, 摸到(\n\n)数的可能性大。(填“奇”或 “偶”)", "options": [], "subject": "算术", "analysis": "解(1) $17,31 \\quad 25 、 60 、 24 、 9 \\quad 60 、 24 \\quad 60$\n\n(2)奇\n\n【分析】(1)质数:在大于 1 的自然数中, 除了 1 和它本身外, 不能再被其他数整除的数, 叫质数;如: 2、3、5、7...都是质数, 也叫素数;\n\n合数:自然数中除了能被 1 和本身整除外, 还能被其他的数整除的数叫合数, 如: $4 、 6 、 8 \\ldots$ 都是合数;\n\n2 的倍数: 个位上是 $0 、 2 、 4 、 6 、 8$ 的数;\n\n能同时被 3 和 5 整除的数:个位上是 0 或 5 , 且各个数位上的数的和是 3 的倍数。据此解答。\n\n(2)奇数: 不能被 2 整除的自然数叫奇数。如:1、3、5、7、9...\n\n偶数:能被 2 整除的自然数叫偶数。如: $2 、 4 、 6 、 8 、 10 \\ldots$\n\n可据此找出奇数、偶数, 然后看哪类数的个数较多,摸到这类数的可能性就大。\n\n(1)\n\n质数有: $17 、 31$\n\n合数有: $25 、 60 、 24 、 9$\n\n2 的倍数有: $60 、 24$\n\n能同时被 3 和 5 整除的数有: 60\n\n奇数有: $17 、 25 、 9 、 31$, 共计 4 个;\n\n偶数有: 60、24, 共计 2 个;\n\n$4>2$, 所以摸到奇数的可能性大。\n\n【点睛】熟悉质数合数、奇偶数的相关概念,同时明确,事件发生的可能性大小与参加的个体数量有\n关, 个体在总数中所占数量越多, 出现的可能性就越大; 反之, 个体在总数中所占数量越少, 出现的可能性就越小。"} {"id": "24298", "image": [], "answer": "解1294\n\n【分析】质数是因数只有 1 和它本身的数; 合数是因数除了 1 和它本身之外还有其他因数的数; 是 2 的倍数的数是偶数, 不是 2 的倍数的数是奇数, 据此分析。\n\n【详解】既不是质数也不是合数的数是 1,2 是最小的质数也是唯一的偶质数, 10 以内的是奇数又是合数的数是 9 , 最小的合数是 4 。\n\n所以这个四位数的手机密码是: 1294。\n\n【点睛】此题考查质数合数以及奇数偶数的应用, 掌握这些数的特点是解题的关键。", "solution": "null", "level": "五年级", "question": "李阿姨的手机屏幕解锁密码提示: 第一个数字既不是质数又不是合数, 第二个数字既是偶数又是质数, 第三个数字既是奇数又是合数, 最后一个数字是最小的合数。解锁密码是 ( )。", "options": [], "subject": "算术", "analysis": "解1294\n\n【分析】质数是因数只有 1 和它本身的数; 合数是因数除了 1 和它本身之外还有其他因数的数; 是 2 的倍数的数是偶数, 不是 2 的倍数的数是奇数, 据此分析。\n\n【详解】既不是质数也不是合数的数是 1,2 是最小的质数也是唯一的偶质数, 10 以内的是奇数又是合数的数是 9 , 最小的合数是 4 。\n\n所以这个四位数的手机密码是: 1294。\n\n【点睛】此题考查质数合数以及奇数偶数的应用, 掌握这些数的特点是解题的关键。"} {"id": "24299", "image": [], "answer": "解$90 \\quad 120$\n\n【分析】根据能被 $2 、 5$ 整除的数的特征, 可以得出: 该两位数的个位数是 0 ; 进而根据能被 3 整除的数的特征: 即该数各个数位上数的和能被 3 整除, 得出: 十位上的数最大是 9 ; 同理可得: 最小三位数是 120 , 解答即可。\n\n【详解】由分析知: 含有因数 $2 、 3 、 5$ 的最大的两位数是 90 , 最小三位数是 120 。\n\n【点睛】本题主要根据 $2 、 3 、 5$ 倍数的特征可知, 要先确定个位满足是 2 和 5 的倍数, 再确定百位、十位, 使 3 位数是 3 的倍数。", "solution": "null", "level": "五年级", "question": "含有因数 $2 、 3 、 5$ 的最大的两位数是 $($ ), 最小的三位数是 $($ 。", "options": [], "subject": "算术", "analysis": "解$90 \\quad 120$\n\n【分析】根据能被 $2 、 5$ 整除的数的特征, 可以得出: 该两位数的个位数是 0 ; 进而根据能被 3 整除的数的特征: 即该数各个数位上数的和能被 3 整除, 得出: 十位上的数最大是 9 ; 同理可得: 最小三位数是 120 , 解答即可。\n\n【详解】由分析知: 含有因数 $2 、 3 、 5$ 的最大的两位数是 90 , 最小三位数是 120 。\n\n【点睛】本题主要根据 $2 、 3 、 5$ 倍数的特征可知, 要先确定个位满足是 2 和 5 的倍数, 再确定百位、十位, 使 3 位数是 3 的倍数。"} {"id": "24300", "image": [], "answer": "解偶\n\n【分析】根据偶数、奇数的意义: 是 2 的倍数的数叫做偶数; 不是 2 的倍数的数叫做奇数; 然后化简带有字母的算式, 根据奇数和偶数概念判断即可。\n\n【详解】 $2 \\mathrm{~m}+\\mathrm{m}+\\mathrm{n}+\\mathrm{m}+\\mathrm{n}$\n\n$=4 \\mathrm{~m}+2 \\mathrm{n}$\n\n$=2(2 \\mathrm{~m}+\\mathrm{n})$\n\n$\\mathrm{m}$ 和 $\\mathrm{n}$ 不管等于多少, $2(2 \\mathrm{~m}+\\mathrm{n})$ 始终是 2 的倍数, 所以 $2 \\mathrm{~m}+\\mathrm{m}+\\mathrm{n}+\\mathrm{m}+\\mathrm{n}$ 的和是偶数。\n\n【点睛】此题的解题关键是理解掌握偶数与奇数的意义。", "solution": "null", "level": "五年级", "question": "$2 \\mathrm{~m}+\\mathrm{m}+\\mathrm{n}+\\mathrm{m}+\\mathrm{n}$ 的和是 ( )数。(填“奇”或“偶”)", "options": [], "subject": "算术", "analysis": "解偶\n\n【分析】根据偶数、奇数的意义: 是 2 的倍数的数叫做偶数; 不是 2 的倍数的数叫做奇数; 然后化简带有字母的算式, 根据奇数和偶数概念判断即可。\n\n【详解】 $2 \\mathrm{~m}+\\mathrm{m}+\\mathrm{n}+\\mathrm{m}+\\mathrm{n}$\n\n$=4 \\mathrm{~m}+2 \\mathrm{n}$\n\n$=2(2 \\mathrm{~m}+\\mathrm{n})$\n\n$\\mathrm{m}$ 和 $\\mathrm{n}$ 不管等于多少, $2(2 \\mathrm{~m}+\\mathrm{n})$ 始终是 2 的倍数, 所以 $2 \\mathrm{~m}+\\mathrm{m}+\\mathrm{n}+\\mathrm{m}+\\mathrm{n}$ 的和是偶数。\n\n【点睛】此题的解题关键是理解掌握偶数与奇数的意义。"} {"id": "24301", "image": [], "answer": "解$12 \\quad 14$\n\n【分析】根据平均数=总数量-总个数, 先求出平均数; 由于是连续的偶数, 则每相邻的两个偶数相差 2, 则平均数是中间的数, 最大的数比平均数大 2 。\n\n【详解】平均数为: $36 \\div 3=12$\n\n最大的数为: $12+2=14$\n\n【点睛】此题考查的是平均数的应用, 解答本题关键是要先求出平均数, 再结合 3 个连续偶数的特点解答。", "solution": "null", "level": "五年级", "question": "三个连续偶数的和是 36 , 这三个数的平均数是 ( ), 其中最大的数是 ( )。", "options": [], "subject": "算术", "analysis": "解$12 \\quad 14$\n\n【分析】根据平均数=总数量-总个数, 先求出平均数; 由于是连续的偶数, 则每相邻的两个偶数相差 2, 则平均数是中间的数, 最大的数比平均数大 2 。\n\n【详解】平均数为: $36 \\div 3=12$\n\n最大的数为: $12+2=14$\n\n【点睛】此题考查的是平均数的应用, 解答本题关键是要先求出平均数, 再结合 3 个连续偶数的特点解答。"} {"id": "24318", "image": [], "answer": "解23 4 $6 、 12,18 、 36$\n\n【分析】先求出 36 的因数, 再根据奇数、偶数、质数和合数的定义解答即可; 能被 2 整除的数就是偶数, 不能被 2 整除的数就是奇数; 一个数的因数只有 1 和它本身两个因数, 这样的数就是质数; 一个数的因数除了 1 和它本身以外还有其他的因数, 这样的数就是合数。\n\n【详解】 36 的因数有: $1 、 2 、 3 、 4 、 6 、 9 、 12 、 18 、 36$ 。\n\n既是偶数, 又是质数的是 2 , 既是奇数, 又是质数的是 3 , 既是偶数, 又是合数的是 $4 、 6 、 12 、 18$ 、 36 。\n\n【点睛】本题考查奇数、偶数、质数和合数, 明确它们的定义是解题的关键。", "solution": "null", "level": "五年级", "question": "36 的因数中, 既是偶数, 又是质数的是 $($ ), 既是奇数, 又是质数的是 $($, 既是偶数, 又是合数的是( )。", "options": [], "subject": "算术", "analysis": "解23 4 $6 、 12,18 、 36$\n\n【分析】先求出 36 的因数, 再根据奇数、偶数、质数和合数的定义解答即可; 能被 2 整除的数就是偶数, 不能被 2 整除的数就是奇数; 一个数的因数只有 1 和它本身两个因数, 这样的数就是质数; 一个数的因数除了 1 和它本身以外还有其他的因数, 这样的数就是合数。\n\n【详解】 36 的因数有: $1 、 2 、 3 、 4 、 6 、 9 、 12 、 18 、 36$ 。\n\n既是偶数, 又是质数的是 2 , 既是奇数, 又是质数的是 3 , 既是偶数, 又是合数的是 $4 、 6 、 12 、 18$ 、 36 。\n\n【点睛】本题考查奇数、偶数、质数和合数, 明确它们的定义是解题的关键。"} {"id": "24319", "image": [], "answer": "解$420 、 450$ 或 480\n\n【分析】 2 的倍数特征: 个位上是 $0 、 2 、 4 、 6 、 8$ 的数; 5 的倍数特征: 个位上是 0 或 5 的数; 3 的倍数特征: 各个数位上的数字相加, 和要能被 3 整除。\n\n一个数, 如果除了 1 和它本身还有别的因数, 那么这样的数叫做合数。\n\n【详解】百位上是最小的合数, 即 4 ;\n\n同时是 2、5 的倍数, 则个位是 0 ;\n\n$4+2+0=6$, 是 3 的倍数;\n\n$4+5+0=9$, 是 3 的倍数;\n$4+8+0=12$, 是 3 的倍数;\n\n这个数可能是 $420 、 450$ 或 480 。\n\n【点睛】本题考查 $2 、 3 、 5$ 的倍数特征以及合数的意义是解题的关键, 注意既是 2 的倍数又是 5 的倍数的数, 个位上一定是 0 。", "solution": "null", "level": "五年级", "question": "小健家的电脑开机密码是一个三位数, 百位上是最小的合数, 它同时是 $2 、 3 、 5$ 的倍数, 这个数可能是( )。", "options": [], "subject": "算术", "analysis": "解$420 、 450$ 或 480\n\n【分析】 2 的倍数特征: 个位上是 $0 、 2 、 4 、 6 、 8$ 的数; 5 的倍数特征: 个位上是 0 或 5 的数; 3 的倍数特征: 各个数位上的数字相加, 和要能被 3 整除。\n\n一个数, 如果除了 1 和它本身还有别的因数, 那么这样的数叫做合数。\n\n【详解】百位上是最小的合数, 即 4 ;\n\n同时是 2、5 的倍数, 则个位是 0 ;\n\n$4+2+0=6$, 是 3 的倍数;\n\n$4+5+0=9$, 是 3 的倍数;\n$4+8+0=12$, 是 3 的倍数;\n\n这个数可能是 $420 、 450$ 或 480 。\n\n【点睛】本题考查 $2 、 3 、 5$ 的倍数特征以及合数的意义是解题的关键, 注意既是 2 的倍数又是 5 的倍数的数, 个位上一定是 0 。"} {"id": "24320", "image": [], "answer": "解因数\n\n【分析】在乘法算式 $\\mathrm{a} \\times \\mathrm{b}=\\mathrm{c}(\\mathrm{a} 、 \\mathrm{~b} 、 \\mathrm{c}$ 均为非 0 的自然数 $)$ 中, $\\mathrm{a} 、 \\mathrm{~b}$ 就是 $\\mathrm{c}$ 的因数, $\\mathrm{c}$ 就是 $\\mathrm{a} 、 \\mathrm{~b}$ 的倍数。\n\n【详解】根据 $63 \\div 7=9$ 可得, $9 \\times 7=63$, 被除数 63 是除数 7 的倍数, 7 是 63 的因数。\n\n【点睛】因数和倍数两个不同的概念是相互依存的, 不能单独存在。", "solution": "null", "level": "五年级", "question": "$63 \\div 7=9$, 被除数 63 是除数 7 的倍数, 7 是 63 的 $($ 。", "options": [], "subject": "算术", "analysis": "解因数\n\n【分析】在乘法算式 $\\mathrm{a} \\times \\mathrm{b}=\\mathrm{c}(\\mathrm{a} 、 \\mathrm{~b} 、 \\mathrm{c}$ 均为非 0 的自然数 $)$ 中, $\\mathrm{a} 、 \\mathrm{~b}$ 就是 $\\mathrm{c}$ 的因数, $\\mathrm{c}$ 就是 $\\mathrm{a} 、 \\mathrm{~b}$ 的倍数。\n\n【详解】根据 $63 \\div 7=9$ 可得, $9 \\times 7=63$, 被除数 63 是除数 7 的倍数, 7 是 63 的因数。\n\n【点睛】因数和倍数两个不同的概念是相互依存的, 不能单独存在。"} {"id": "24322", "image": [], "answer": "解1 它本身 偶数\n\n【分析】根据质数的定义可知, 一个数, 如果只有 1 和它本身两个因数, 那么这样的数叫做质数。整数中, 是 2 的倍数的数叫做偶数 ( 0 也是偶数), 不是 2 的倍数的数叫做奇数, 自然数按照是不是 2 的倍数分为奇数和偶数, 据此解答。\n\n【详解】一个数, 如果只有 1 和它本身两个因数, 那么这样的数叫做质数。在全部整数里, 不是奇数就是偶数。\n\n【点睛】此题的解题关键是理解掌握质数、奇数和偶数的定义。", "solution": "null", "level": "五年级", "question": "一个数, 如果只有 ( )和( )两个因数, 那么这样的数叫做质数。在全部整数里,不是奇数就是 $\\quad$ 。", "options": [], "subject": "算术", "analysis": "解1 它本身 偶数\n\n【分析】根据质数的定义可知, 一个数, 如果只有 1 和它本身两个因数, 那么这样的数叫做质数。整数中, 是 2 的倍数的数叫做偶数 ( 0 也是偶数), 不是 2 的倍数的数叫做奇数, 自然数按照是不是 2 的倍数分为奇数和偶数, 据此解答。\n\n【详解】一个数, 如果只有 1 和它本身两个因数, 那么这样的数叫做质数。在全部整数里, 不是奇数就是偶数。\n\n【点睛】此题的解题关键是理解掌握质数、奇数和偶数的定义。"} {"id": "24323", "image": [], "answer": "解870\n\n【分析】同时是 $2,3,5$ 的倍数的特征: 个位上的数字是 0 , 各个数位上的数字的和是 3 的倍数的数。\n\n【详解】要使三位数 $8 \\square$ 同时是 $2 、 3 、 5$ 的倍数, 则这个三位数最大是 870 。\n\n【点睛】关键是掌握 $2 、 3 、 5$ 的倍数的特征。", "solution": "null", "level": "五年级", "question": "要使三位数 8 口同时是 $2 、 3 、 5$ 的倍数, 则这个三位数最大是 $($ 。", "options": [], "subject": "算术", "analysis": "解870\n\n【分析】同时是 $2,3,5$ 的倍数的特征: 个位上的数字是 0 , 各个数位上的数字的和是 3 的倍数的数。\n\n【详解】要使三位数 $8 \\square$ 同时是 $2 、 3 、 5$ 的倍数, 则这个三位数最大是 870 。\n\n【点睛】关键是掌握 $2 、 3 、 5$ 的倍数的特征。"} {"id": "24324", "image": [], "answer": "解奇\n\n【分析】根据奇数和偶数的运算性质: 奇数 + 奇数 $=$ 偶数, 偶数 + 奇数 $=$ 奇数, 偶数 + 偶数 $=$ 偶数,偶数 - 偶数 $=$ 偶数, 奇数 - 奇数 $=$ 偶数, 偶数 - 奇数 $=$ 奇数, 奇数 - 偶数 $=$ 奇数, 据此解答。\n\n【详解】 28 颗糖果要分别装在甲、乙两个礼品盒里, 甲盒里的数量为奇数,\n\n所以乙盆的数量 $=28$-甲盆的数量,\n\n因为 28 为偶数, 所以根据偶数 - 奇数 $=$ 奇数可得,\n\n乙盒里的数量为奇数。\n\n【点睛】本题考查了奇数和偶数的运算性质。", "solution": "null", "level": "五年级", "question": "28 颗糖果要分别装在甲、乙两个礼品盒里, 如果甲盒里的数量为奇数, 那么乙盒里的数量为 ( ) 数。", "options": [], "subject": "算术", "analysis": "解奇\n\n【分析】根据奇数和偶数的运算性质: 奇数 + 奇数 $=$ 偶数, 偶数 + 奇数 $=$ 奇数, 偶数 + 偶数 $=$ 偶数,偶数 - 偶数 $=$ 偶数, 奇数 - 奇数 $=$ 偶数, 偶数 - 奇数 $=$ 奇数, 奇数 - 偶数 $=$ 奇数, 据此解答。\n\n【详解】 28 颗糖果要分别装在甲、乙两个礼品盒里, 甲盒里的数量为奇数,\n\n所以乙盆的数量 $=28$-甲盆的数量,\n\n因为 28 为偶数, 所以根据偶数 - 奇数 $=$ 奇数可得,\n\n乙盒里的数量为奇数。\n\n【点睛】本题考查了奇数和偶数的运算性质。"} {"id": "24325", "image": [], "answer": "解2 9、15 1\n\n【分析】质数是指除了 1 和它本身的两个因数以外再没有其他的因数。合数是指就除了 1 和它本身的两个因数以外还有其他的因数。 1 既不是质数也不是合数。不能被 2 整除的自然数叫奇数, 能被 2 整\n除的数叫偶数。据此解答。\n\n【详解】在 $8 、 9 、 1 、 2 、 5 、 15 、 17$ 中,\n\n偶数有: $8 、 2$;\n\n奇数有: $9 、 1 、 5 、 15 、 17$;\n\n质数有: 2、5、17;\n\n合数有: 8、9、15;\n\n所以既是偶数又是质数的有 2 ;\n\n既是奇数又是合数的有 $9 、 15$;\n\n1 既不是质数也不是合数。\n\n【点睛】此题主要明确奇数与偶数、质数与合数的定义, 以及奇数与质数、偶数与合数的区别, 才能做出正确的解答。", "solution": "null", "level": "五年级", "question": "在 $8 、 9 、 1 、 2 、 5 、 15 、 17$ 中, 既是偶数又是质数的有 $($ ), 既是奇数又是合数的有 $($,\n\n( )既不是质数也不是合数。", "options": [], "subject": "算术", "analysis": "解2 9、15 1\n\n【分析】质数是指除了 1 和它本身的两个因数以外再没有其他的因数。合数是指就除了 1 和它本身的两个因数以外还有其他的因数。 1 既不是质数也不是合数。不能被 2 整除的自然数叫奇数, 能被 2 整\n除的数叫偶数。据此解答。\n\n【详解】在 $8 、 9 、 1 、 2 、 5 、 15 、 17$ 中,\n\n偶数有: $8 、 2$;\n\n奇数有: $9 、 1 、 5 、 15 、 17$;\n\n质数有: 2、5、17;\n\n合数有: 8、9、15;\n\n所以既是偶数又是质数的有 2 ;\n\n既是奇数又是合数的有 $9 、 15$;\n\n1 既不是质数也不是合数。\n\n【点睛】此题主要明确奇数与偶数、质数与合数的定义, 以及奇数与质数、偶数与合数的区别, 才能做出正确的解答。"} {"id": "24326", "image": [], "answer": "解$\\quad 17 \\quad 19 \\quad 21$\n\n【分析】中间的奇数等于三个连续奇数和的平均数, 最大的奇数比中间的奇数多 2 , 最小的奇数比中间的奇数少 2 , 据此解答。\n\n【详解】中间的奇数: $57 \\div 3=19$\n\n最小的奇数: $19-2=17$\n\n最大的奇数: $19+2=21$\n\n所以, 这三个数分别是 $17 、 19 、 21$ 。\n\n【点睛】相邻的两个奇数 (偶数) 相差 2 , 利用平均数的计算方法求出中间的奇数是解答题目的关键。", "solution": "null", "level": "五年级", "question": "三个连续奇数的和是 57 , 这三个数分别是 $(\\quad) 、(\\quad$ 。", "options": [], "subject": "算术", "analysis": "解$\\quad 17 \\quad 19 \\quad 21$\n\n【分析】中间的奇数等于三个连续奇数和的平均数, 最大的奇数比中间的奇数多 2 , 最小的奇数比中间的奇数少 2 , 据此解答。\n\n【详解】中间的奇数: $57 \\div 3=19$\n\n最小的奇数: $19-2=17$\n\n最大的奇数: $19+2=21$\n\n所以, 这三个数分别是 $17 、 19 、 21$ 。\n\n【点睛】相邻的两个奇数 (偶数) 相差 2 , 利用平均数的计算方法求出中间的奇数是解答题目的关键。"} {"id": "24344", "image": [], "answer": "解2609\n\n【分析】质数是指除了 1 和它本身的两个因数以外再没有其他的因数。最小的质数是 2 ; 一个数的因数的个数是有限的, 最小的因数是 1 , 最大的因数是它本身; 所以 6 的最大因数是 6 ; 最小的自然数是 0 , 没有最大的自然数; 最大的一位数是 9 。据此写出苗苗的学号。\n\n【详解】(1)最小的质数是 2 ;\n\n(2)6的最大因数是 6 ;\n\n(3)最小的自然数是 0 ;\n\n(4) 最大的一位数是 9 。\n\n所以苗苗的学号是 2609 。\n\n【点睛】此题的解题关键是理解掌握质数、自然数、整数的定义以及一个数的因数的求法。", "solution": "null", "level": "五年级", "question": "苗苗的学号是一个四位数, 从左向右依次是: (1)最小的质数; (2)6的最大因数; (3)最小的自然数;\n\n(4)最大的一位数。苗苗的学号是 ( )。", "options": [], "subject": "算术", "analysis": "解2609\n\n【分析】质数是指除了 1 和它本身的两个因数以外再没有其他的因数。最小的质数是 2 ; 一个数的因数的个数是有限的, 最小的因数是 1 , 最大的因数是它本身; 所以 6 的最大因数是 6 ; 最小的自然数是 0 , 没有最大的自然数; 最大的一位数是 9 。据此写出苗苗的学号。\n\n【详解】(1)最小的质数是 2 ;\n\n(2)6的最大因数是 6 ;\n\n(3)最小的自然数是 0 ;\n\n(4) 最大的一位数是 9 。\n\n所以苗苗的学号是 2609 。\n\n【点睛】此题的解题关键是理解掌握质数、自然数、整数的定义以及一个数的因数的求法。"} {"id": "24345", "image": [], "answer": "解$30 \\quad 90$\n\n【分析】既是 3 的倍数又是 5 的倍数的特征: 个位上的数字是 0 或 5 , 各个数位上的数字的和是 3 的倍数的数。\n\n2 的倍数特征:个位上的数字是 $0 、 2 、 4 、 6 、 8$ 的数是 2 的倍数。整数中,是 2 的倍数的数叫偶数,不是 2 的倍数的数叫奇数。\n\n【详解】一个两位数既是 5 的倍数, 又有因数 3 , 而且是偶数, 这个数的个位一定是 0 , 这个数最小\n是 30 , 最大是 90 。\n\n【点睛】关键是掌握 $2 、 3 、 5$ 的倍数的特征。", "solution": "null", "level": "五年级", "question": "一个两位数既是 5 的倍数, 又有因数 3, 而且是偶数, 这个数最小是 ( ), 最大是 ( )。", "options": [], "subject": "算术", "analysis": "解$30 \\quad 90$\n\n【分析】既是 3 的倍数又是 5 的倍数的特征: 个位上的数字是 0 或 5 , 各个数位上的数字的和是 3 的倍数的数。\n\n2 的倍数特征:个位上的数字是 $0 、 2 、 4 、 6 、 8$ 的数是 2 的倍数。整数中,是 2 的倍数的数叫偶数,不是 2 的倍数的数叫奇数。\n\n【详解】一个两位数既是 5 的倍数, 又有因数 3 , 而且是偶数, 这个数的个位一定是 0 , 这个数最小\n是 30 , 最大是 90 。\n\n【点睛】关键是掌握 $2 、 3 、 5$ 的倍数的特征。"} {"id": "24006", "image": ["11861.jpg", "11862.jpg", "11863.jpg", "11864.jpg", "11865.jpg", "11863.jpg", "11864.jpg", "11865.jpg"], "answer": "解(1)(1)或(2)\n\n(2)(2)或(3)\n\n【分析】从左面看小明搭的积木, 看到的形状是\n\n\n\n,从左面看小兵搭的积木,找出看到的形状与小明搭的积木相同的即可; 从正面看小兵搭的积木,找出看到的形状相同的即可。\n\n(1)从左面看小明搭的积木, 看到的形状是\n\n\n, 小兵搭的积木中(1)和(2)看到的形状也是\n\n\n(2)小兵搭的积木中, 从正面看, 形状相同的是(2)和(3), 都是\n\n【点睛】本题主要考查的是观察物体,从左面看,观测点在左边;从正面看,观测点要在正前面。", "solution": "null", "level": "五年级", "question": "观察物体填空。\n\n(1)从左面看,小兵搭的积木中( )号的形状和小明搭的是相同的。\n\n(2)小兵搭的积木中, 从正面看, 形状相同的是( )号。\n\n\n\n1\n\n\n\n3", "options": [], "subject": "画法几何学", "analysis": "解(1)(1)或(2)\n\n(2)(2)或(3)\n\n【分析】从左面看小明搭的积木, 看到的形状是\n\n\n\n,从左面看小兵搭的积木,找出看到的形状与小明搭的积木相同的即可; 从正面看小兵搭的积木,找出看到的形状相同的即可。\n\n(1)从左面看小明搭的积木, 看到的形状是\n\n\n, 小兵搭的积木中(1)和(2)看到的形状也是\n\n\n(2)小兵搭的积木中, 从正面看, 形状相同的是(2)和(3), 都是\n\n【点睛】本题主要考查的是观察物体,从左面看,观测点在左边;从正面看,观测点要在正前面。"} {"id": "24142", "image": ["12327.jpg"], "answer": "解56\n\n【分析】观察图形, 最上面一层规定为第一层, 从上到下数出各个图形所有的小正方体数量; 第一个图形有两层, 第一层有 1 个小正方体, 第二层比第一层多 2 个小正方体, 有 3 个小正方体, 共有(1 +3)个, 第二个图形有三层, 第一层有 1 个小正方体, 第二层比第一层多 2 个小正方体, 第三层比第二层多 3 个小正方体, 共有 $(1+3+6)$ 个, 依次类推, 第 5 个图形有五层, 第一层有 1 个小正方体,第二层比第一层多 2 个小正方体,第三层比第二层多 3 个小正方体,第四层比第三层多 4 个小正方体,第五层比第四层多 5 个小正方体,第六层比第五层多 6 个小正方体,即第一层有 1 个,第二层有 $1+2=3$ (个), 第三层有 $3+3=6$ (个), 第四层有 $6+4=10$ (个), 第五层有 $10+5=15$ (个),第六层有 $15+6=21$ (个), 全部加起来即是需要的小正方体的总数。\n\n【详解】根据分析得, $1+3+6+10+15+21=10+10+15+21=56$ (个)\n\n【点睛】本题主要考查归纳推理, 其基本思路是先分析具体, 观察, 总结其内在联系, 得到一般性的结论。", "solution": "null", "level": "五年级", "question": "如下图, 按这样的规律摆下去, 第 5 个图形要(\n\n)个小正方体。\n", "options": [], "subject": "计数", "analysis": "解56\n\n【分析】观察图形, 最上面一层规定为第一层, 从上到下数出各个图形所有的小正方体数量; 第一个图形有两层, 第一层有 1 个小正方体, 第二层比第一层多 2 个小正方体, 有 3 个小正方体, 共有(1 +3)个, 第二个图形有三层, 第一层有 1 个小正方体, 第二层比第一层多 2 个小正方体, 第三层比第二层多 3 个小正方体, 共有 $(1+3+6)$ 个, 依次类推, 第 5 个图形有五层, 第一层有 1 个小正方体,第二层比第一层多 2 个小正方体,第三层比第二层多 3 个小正方体,第四层比第三层多 4 个小正方体,第五层比第四层多 5 个小正方体,第六层比第五层多 6 个小正方体,即第一层有 1 个,第二层有 $1+2=3$ (个), 第三层有 $3+3=6$ (个), 第四层有 $6+4=10$ (个), 第五层有 $10+5=15$ (个),第六层有 $15+6=21$ (个), 全部加起来即是需要的小正方体的总数。\n\n【详解】根据分析得, $1+3+6+10+15+21=10+10+15+21=56$ (个)\n\n【点睛】本题主要考查归纳推理, 其基本思路是先分析具体, 观察, 总结其内在联系, 得到一般性的结论。"} {"id": "24350", "image": [], "answer": "解 $2 、 3$ 的倍数有 $2 、 3 、 4 、 6 、 8 、 9 、 10 、 12 、 14 、 15 、 16 、 18 、 20 、 21 、 22 、 24$ 、 $26 、 27 、 28 、 30 、 32 、 33 、 34 \\ldots .$.\n\n因为 2 和 3 的最小公倍数是 $2 \\times 3=6$, 。\n\n所以这些数中既是 3 的倍数又是 2 的倍数的有 $6 、 12 、 18 、 24 、 30$, 共 5 个。故答案为: 5 。", "solution": "null", "level": "五年级", "question": "(2022 秋 ・永定区期末)田老师让同学们在黑板上写 2 和 3 的倍数, 同学们一共写了 23 个, 其中有 16 个是 2 的倍数, 14 个是 3 的倍数。这些数中既是 3 的倍数又是 2 的倍数的有 $\\qquad$个。", "options": [], "subject": "计数", "analysis": "解 $2 、 3$ 的倍数有 $2 、 3 、 4 、 6 、 8 、 9 、 10 、 12 、 14 、 15 、 16 、 18 、 20 、 21 、 22 、 24$ 、 $26 、 27 、 28 、 30 、 32 、 33 、 34 \\ldots .$.\n\n因为 2 和 3 的最小公倍数是 $2 \\times 3=6$, 。\n\n所以这些数中既是 3 的倍数又是 2 的倍数的有 $6 、 12 、 18 、 24 、 30$, 共 5 个。故答案为: 5 。"} {"id": "24870", "image": [], "answer": "解 $2000 \\div 400=5$ (杯)\n\n$500 \\times 6=3000$ (毫升)\n\n3000 毫升 $=3$ 升。\n\n故答案为: $5 ; 3$ 。", "solution": "null", "level": "五年级", "question": "(2022 秋 $\\cdot$ 通江县期末)把 $2000 m L$ 水倒入 400 毫升的量杯,可以倒 $\\qquad$杯。用 500 毫升的量杯往茶杯里水, 6 次正好倒满。这个茶杯能盛 $\\qquad$升水。", "options": [], "subject": "计数", "analysis": "解 $2000 \\div 400=5$ (杯)\n\n$500 \\times 6=3000$ (毫升)\n\n3000 毫升 $=3$ 升。\n\n故答案为: $5 ; 3$ 。"} {"id": "25188", "image": [], "answer": "解6\n\n【分析】先根据“长方体的体积 $=$ 长 $\\times$ 宽 $\\times$ 高”求出水池的容积, 需要的时间 $=$ 水池的容积 $\\div$ 每分钟注水的体积, 据此解答。\n\n【详解】 $3 \\times 1.5 \\times 0.8$\n\n$=4.5 \\times 0.8$\n\n$=3.6$ (立方米)\n\n3.6 立方米 $=3600$ 立方分米 $=3600$ 升\n\n$3600 \\div 600=6$ (分钟)\n\n所以, 需要 6 分钟才能把水池注满。\n\n【点睛】掌握长方体的体积(容积)计算公式是解答题目的关键。", "solution": "null", "level": "五年级", "question": "往一个长 3 米, 宽 1.5 米, 深 0.8 米的水池里注水, 如果每分钟注水 600 升, 需要\n\n)分钟才能把水池注满。", "options": [], "subject": "计数", "analysis": "解6\n\n【分析】先根据“长方体的体积 $=$ 长 $\\times$ 宽 $\\times$ 高”求出水池的容积, 需要的时间 $=$ 水池的容积 $\\div$ 每分钟注水的体积, 据此解答。\n\n【详解】 $3 \\times 1.5 \\times 0.8$\n\n$=4.5 \\times 0.8$\n\n$=3.6$ (立方米)\n\n3.6 立方米 $=3600$ 立方分米 $=3600$ 升\n\n$3600 \\div 600=6$ (分钟)\n\n所以, 需要 6 分钟才能把水池注满。\n\n【点睛】掌握长方体的体积(容积)计算公式是解答题目的关键。"} {"id": "25024", "image": [], "answer": "解(1) 一 四\n\n(2)二 三\n\n【分析】(1) 5 的倍数特征:个位上是 0 或 5 的数; 如果每组 5 人, 能正好分完的班级总人数是 5 的倍数;\n\n(2)因为 $6=2 \\times 3$, 所以 6 的倍数特征: 各数位上的数字和能被 3 整除的偶数; 也可以说既是 2 的倍数又是 3 的倍数的数, 一定是 6 的倍数;\n\n如果每组 6 人, 能正好分完的班级总人数是 6 的倍数, 据此解答。\n\n(1)\n\n5 的倍数是: $40 、 45$;\n\n如果每组 5 人,一班和四班能正好分完。\n\n2 的倍数是: $40 、 42 、 48$;\n\n3 的倍数是: $42 、 48 、 45$;\n\n既是 2 的倍数又是 3 的倍数(即 6 的倍数): $42 、 48$;\n\n如果每组 6 人,二班和三班能正好分完。\n\n【点睛】本题考查 5 的倍数、 6 的倍数特征的应用, 关键是由 $2 、 3$ 的倍数特征得出 6 的倍数特征。", "solution": "null", "level": "五年级", "question": "某校五年级各班人数情况统计如表:现各班要划分活动小组。\n\n| 班别 | 一班 | 二班 | 三班 | 四班 |\n| :---: | :---: | :---: | :---: | :---: |\n| 人数 | 40 | 42 | 48 | 45 |\n\n(1)如果每组 5 人,( )班和( 班能正好分完;\n(2)如果每组 6 人,( )班和( 班能正好分完。", "options": [], "subject": "计数", "analysis": "解(1) 一 四\n\n(2)二 三\n\n【分析】(1) 5 的倍数特征:个位上是 0 或 5 的数; 如果每组 5 人, 能正好分完的班级总人数是 5 的倍数;\n\n(2)因为 $6=2 \\times 3$, 所以 6 的倍数特征: 各数位上的数字和能被 3 整除的偶数; 也可以说既是 2 的倍数又是 3 的倍数的数, 一定是 6 的倍数;\n\n如果每组 6 人, 能正好分完的班级总人数是 6 的倍数, 据此解答。\n\n(1)\n\n5 的倍数是: $40 、 45$;\n\n如果每组 5 人,一班和四班能正好分完。\n\n2 的倍数是: $40 、 42 、 48$;\n\n3 的倍数是: $42 、 48 、 45$;\n\n既是 2 的倍数又是 3 的倍数(即 6 的倍数): $42 、 48$;\n\n如果每组 6 人,二班和三班能正好分完。\n\n【点睛】本题考查 5 的倍数、 6 的倍数特征的应用, 关键是由 $2 、 3$ 的倍数特征得出 6 的倍数特征。"} {"id": "12329", "image": [], "answer": "答案: 14", "solution": "null", "level": "五年级", "question": "下图有一部分 $\\circ$ 被覆盖, 已知露出的 $\\circ$ 是 $\\circ$ 总数的 $\\frac{3}{7}$, 一共有 \\$ \\qquad \\$个०。\n\n$0 \\circ$ $\\circ \\circ \\circ$", "options": [], "subject": "计数", "analysis": "答案: 14"} {"id": "12356", "image": [], "answer": "答案:$\\sqrt{ }$", "solution": "null", "level": "五年级", "question": "折线统计图不仅能表示数量的多少,还能清楚地反映数量的增减情况。()", "options": [], "subject": "计数", "analysis": "答案:$\\sqrt{ }$"} {"id": "24455", "image": [], "answer": "解340\n\n【分析】商标纸的面积就是长方体罐头盒前后面和左右面的面积和, 即 $($ 长 $\\times$ 高 + 宽 $\\times$ 高 $) \\times 2$, 据此列式计算即可。\n\n【详解】 $(13 \\times 8.5+7 \\times 8.5) \\times 2$\n\n$=(110.5+59.5) \\times 2$\n\n$=170 \\times 2$\n\n$=340$ (平方厘米)\n\n所以这张商标纸的面积至少 340 平方厘米。\n\n【点睛】此题考查长方体的表面积, 注意需要计算哪些面的面积和。", "solution": "null", "level": "五年级", "question": "一个长方体罐头盒, 它长 13 厘米、宽 7 厘米, 高 8.5 厘米, 如果在盒的四周贴上商标纸 (上、下面不贴),这张商标纸的面积至少( )平方厘米。", "options": [], "subject": "度量几何学", "analysis": "解340\n\n【分析】商标纸的面积就是长方体罐头盒前后面和左右面的面积和, 即 $($ 长 $\\times$ 高 + 宽 $\\times$ 高 $) \\times 2$, 据此列式计算即可。\n\n【详解】 $(13 \\times 8.5+7 \\times 8.5) \\times 2$\n\n$=(110.5+59.5) \\times 2$\n\n$=170 \\times 2$\n\n$=340$ (平方厘米)\n\n所以这张商标纸的面积至少 340 平方厘米。\n\n【点睛】此题考查长方体的表面积, 注意需要计算哪些面的面积和。"} {"id": "24476", "image": [], "answer": "解$0.0045 \\quad 0.00075 \\quad 12000 \\quad 4 \\quad 70$\n\n【分析】根据 1 立方分米 $=1000$ 立方厘米, 1 立方米 $=1000000$ 立方厘米, 1 立方米 $=1000$ 立方分米,进行换算即可。\n\n【详解】 4.5 立方厘米 $\\div 1000=0.0045$ 立方分米; 750 立方厘米 $\\div 1000000=0.00075$ 立方米\n\n12 立方分米 $\\times 1000=12000$ 立方厘米; 0.07 立方米 $\\times 1000=70$ 立方分米, 4.07 立方米 $=4$ 立方米 70 立方分米\n\n【点睛】单位大变小乘进率,单位小变大除以进率。", "solution": "null", "level": "五年级", "question": "单位换算。\n\n4.5 立方厘米 $=(\\quad)$ 立方分米 $\\quad 750$ 立方厘米 $=(\\quad)$ 立方米\n\n12 立方分米 $=(\\quad)$ 立方厘米 4.07 立方米 $=(\\quad$ 立方米 $(\\quad$ 立方分米", "options": [], "subject": "度量几何学", "analysis": "解$0.0045 \\quad 0.00075 \\quad 12000 \\quad 4 \\quad 70$\n\n【分析】根据 1 立方分米 $=1000$ 立方厘米, 1 立方米 $=1000000$ 立方厘米, 1 立方米 $=1000$ 立方分米,进行换算即可。\n\n【详解】 4.5 立方厘米 $\\div 1000=0.0045$ 立方分米; 750 立方厘米 $\\div 1000000=0.00075$ 立方米\n\n12 立方分米 $\\times 1000=12000$ 立方厘米; 0.07 立方米 $\\times 1000=70$ 立方分米, 4.07 立方米 $=4$ 立方米 70 立方分米\n\n【点睛】单位大变小乘进率,单位小变大除以进率。"} {"id": "24482", "image": ["12502.jpg"], "answer": "解2 B 60\n\n【分析】根据长方体的特征, 长方体的 4 条长、 4 条宽和 4 条高分别相等, 长方体共有 6 个面; 小芳选了 2 块 $\\mathrm{A}$ 纸板和 2 块 $\\mathrm{D}$ 纸板, 则需要选一条边长为 $4 \\mathrm{~cm}$, 另一条边长为 $3 \\mathrm{~cm}$ 的长方形, 也就是需要选 2 块 $\\mathrm{B}$ 纸板; 再根据长方体的体积公式: $\\mathrm{V}=\\mathrm{abh}$, 据此进行计算即可。\n\n【详解】她应该再选 2 块 B 纸板\n\n$5 \\times 4 \\times 3$\n\n$=20 \\times 3$\n\n$=60$ (立方厘米)\n\n则她应该再选 2 块 $\\mathrm{B}$ 纸板, 做成的长方体体积是 60 立方厘米。\n\n【点睛】本题考查长方体的特征和体积, 明确长方体的特征是解题的关键。", "solution": "null", "level": "五年级", "question": "有 4 种不同规格的纸板, 每种纸板的数量都足够多。要从中选六张做成一个长方体。小芳选了 2 块 $\\mathrm{A}$ 纸板和 2 块 $\\mathrm{D}$ 纸板, 她应该再选( )块(纸板才能做成一个长方体, 做成的长方体体积是( ) 立方厘米。\n", "options": [], "subject": "度量几何学", "analysis": "解2 B 60\n\n【分析】根据长方体的特征, 长方体的 4 条长、 4 条宽和 4 条高分别相等, 长方体共有 6 个面; 小芳选了 2 块 $\\mathrm{A}$ 纸板和 2 块 $\\mathrm{D}$ 纸板, 则需要选一条边长为 $4 \\mathrm{~cm}$, 另一条边长为 $3 \\mathrm{~cm}$ 的长方形, 也就是需要选 2 块 $\\mathrm{B}$ 纸板; 再根据长方体的体积公式: $\\mathrm{V}=\\mathrm{abh}$, 据此进行计算即可。\n\n【详解】她应该再选 2 块 B 纸板\n\n$5 \\times 4 \\times 3$\n\n$=20 \\times 3$\n\n$=60$ (立方厘米)\n\n则她应该再选 2 块 $\\mathrm{B}$ 纸板, 做成的长方体体积是 60 立方厘米。\n\n【点睛】本题考查长方体的特征和体积, 明确长方体的特征是解题的关键。"} {"id": "24503", "image": [], "answer": "解$45000 \\quad 980$\n\n【分析】根据 1 平方米 $=100$ 平方分米 $=10000$ 平方厘米, 1 立方米 $=1000$ 立方分米, 高级单位换算\n成低级单位, 乘进率, 低级单位换算成高级单位, 除以进率, 据此解答。\n\n【详解】 4.5 平方米 $=45000$ 平方厘米 $\\quad 0.98$ 立方米 $=980$ 立方分米\n\n【点睛】本题考查单位之间的互化, 关键是熟记进率。", "solution": "null", "level": "五年级", "question": "4.5 平方米 $=(\\quad)$ 平方厘米 $\\quad 0.98$ 立方米 $=(\\quad)$ 立方分米", "options": [], "subject": "度量几何学", "analysis": "解$45000 \\quad 980$\n\n【分析】根据 1 平方米 $=100$ 平方分米 $=10000$ 平方厘米, 1 立方米 $=1000$ 立方分米, 高级单位换算\n成低级单位, 乘进率, 低级单位换算成高级单位, 除以进率, 据此解答。\n\n【详解】 4.5 平方米 $=45000$ 平方厘米 $\\quad 0.98$ 立方米 $=980$ 立方分米\n\n【点睛】本题考查单位之间的互化, 关键是熟记进率。"} {"id": "24531", "image": [], "answer": "解米\\#\\# $m$ 平方分米\\#\\# $d m^{2}$ 毫升\\#\\# $\\mathrm{mL}$ 立方米 $\\# \\# m^{3}$\n\n【分析】根据生活经验以及数据的大小, 选择合适的计量单位, 即可解答。.\n\n【详解】教室的讲台的高大约 1(米)。\n\n数学书的封面的面积大约 5(平方分米)。\n\n一盒牛奶的容积大约有 250(毫升)。\n\n教室里面的空间约是 180(立方米)。\n\n【点睛】此题考查根据情景选择合适的计量单位, 要注意联系生活实际、计量单位和数据的大小,灵活地选择。", "solution": "null", "level": "五年级", "question": "在括号里填上合适的单位。\n\n教室的讲台的高大约 1( )。数学书的封面的面积大约 5( )。\n\n一盒牛奶的容积大约有 250( )。教室里面的空间约是 180( )。", "options": [], "subject": "度量几何学", "analysis": "解米\\#\\# $m$ 平方分米\\#\\# $d m^{2}$ 毫升\\#\\# $\\mathrm{mL}$ 立方米 $\\# \\# m^{3}$\n\n【分析】根据生活经验以及数据的大小, 选择合适的计量单位, 即可解答。.\n\n【详解】教室的讲台的高大约 1(米)。\n\n数学书的封面的面积大约 5(平方分米)。\n\n一盒牛奶的容积大约有 250(毫升)。\n\n教室里面的空间约是 180(立方米)。\n\n【点睛】此题考查根据情景选择合适的计量单位, 要注意联系生活实际、计量单位和数据的大小,灵活地选择。"} {"id": "24562", "image": [], "answer": "解$20 \\quad 20000 \\quad 610 \\quad 610$\n\n【分析】高级单位换低级单位乘进率, 根据 $1 \\mathrm{~m}^{3}=1000 \\mathrm{dm}^{3}$, 用 $0.02 \\times 1000$ 即可; 根据 $1 \\mathrm{~m}^{3}=1000000 \\mathrm{~cm}^{3}$,用 $0.02 \\times 1000000$ 即可; 低级单位换高级单位除以进率, 根据 $1 \\mathrm{~L}=1000 \\mathrm{~mL}$, 用 $610000 \\div 1000$ 即可; 根据 $1 \\mathrm{~L}=1 \\mathrm{~m}^{3}$ ,据此填空即可。\n\n【详解】 $0.02 \\mathrm{~m}^{3}=0.02 \\times 1000 \\mathrm{dm}^{3}=20 \\mathrm{dm}^{3}=0.02 \\times 1000000 \\mathrm{~cm}^{3}=20000 \\mathrm{~cm}^{3}$\n\n$610000 \\mathrm{~mL}=610000 \\div 1000 \\mathrm{~L}=610 \\mathrm{~L}=610 \\mathrm{~m}^{3}$\n\n【点睛】本题考查单位换算, 明确各单位之间的进率是解题的关键。", "solution": "null", "level": "五年级", "question": "在括号里填上合适的数。\n\n$0.02 \\mathrm{~m}^{3}=(\\quad) \\mathrm{dm}^{3}=(\\quad) \\mathrm{cm}^{3} \\quad 610000 \\mathrm{~mL}=(\\quad) \\mathrm{L}=(\\quad) \\mathrm{m}^{3}$", "options": [], "subject": "度量几何学", "analysis": "解$20 \\quad 20000 \\quad 610 \\quad 610$\n\n【分析】高级单位换低级单位乘进率, 根据 $1 \\mathrm{~m}^{3}=1000 \\mathrm{dm}^{3}$, 用 $0.02 \\times 1000$ 即可; 根据 $1 \\mathrm{~m}^{3}=1000000 \\mathrm{~cm}^{3}$,用 $0.02 \\times 1000000$ 即可; 低级单位换高级单位除以进率, 根据 $1 \\mathrm{~L}=1000 \\mathrm{~mL}$, 用 $610000 \\div 1000$ 即可; 根据 $1 \\mathrm{~L}=1 \\mathrm{~m}^{3}$ ,据此填空即可。\n\n【详解】 $0.02 \\mathrm{~m}^{3}=0.02 \\times 1000 \\mathrm{dm}^{3}=20 \\mathrm{dm}^{3}=0.02 \\times 1000000 \\mathrm{~cm}^{3}=20000 \\mathrm{~cm}^{3}$\n\n$610000 \\mathrm{~mL}=610000 \\div 1000 \\mathrm{~L}=610 \\mathrm{~L}=610 \\mathrm{~m}^{3}$\n\n【点睛】本题考查单位换算, 明确各单位之间的进率是解题的关键。"} {"id": "24582", "image": [], "answer": "解立方分米\\#\\#dm ${ }^{3}$ 毫升\\#\\#mL\n\n【分析】根据生活经验、对体积单位、容积单位和数据大小的认识, 可知计量粉笔盒的体积应用“立方分米”作单位; 计量保温杯的容积应用“毫升”作单位。\n\n【详解】由分析可知:\n\n粉笔盒的体积约是 1 立方分米; 保温杯的容积约是 500 毫升。\n\n【点睛】此题考查根据情景选择合适的计量单位, 要注意联系生活实际、计量单位和数据的大小,灵活的选择。", "solution": "null", "level": "五年级", "question": "填写合适的单位名称。\n\n粉笔盒的体积约是 1( ); 保温杯的容积约是 500( )。", "options": [], "subject": "度量几何学", "analysis": "解立方分米\\#\\#dm ${ }^{3}$ 毫升\\#\\#mL\n\n【分析】根据生活经验、对体积单位、容积单位和数据大小的认识, 可知计量粉笔盒的体积应用“立方分米”作单位; 计量保温杯的容积应用“毫升”作单位。\n\n【详解】由分析可知:\n\n粉笔盒的体积约是 1 立方分米; 保温杯的容积约是 500 毫升。\n\n【点睛】此题考查根据情景选择合适的计量单位, 要注意联系生活实际、计量单位和数据的大小,灵活的选择。"} {"id": "24612", "image": [], "answer": "解\n 42 平方分米 $=4200$ 平方厘米 3050 升 $=3.05$ 立方米\n 50 立方厘米 $=0.05$ 立方分米 0.64 立方分米 $=640$ 毫升\n\n故答案为: $4200,3.05,0.05,640$ 。", "solution": "null", "level": "五年级", "question": "(2022 秋 $\\cdot$ 锡山区期末)\n\n42 平方分米 $=$ $\\qquad$平方厘米\n\n3050 升 $=$ $\\qquad$立方米\n\n50 立方厘米 $=$ $\\qquad$立方分米\n\n0.64 立方分米 $=$ $\\qquad$毫升", "options": [], "subject": "度量几何学", "analysis": "解\n 42 平方分米 $=4200$ 平方厘米 3050 升 $=3.05$ 立方米\n 50 立方厘米 $=0.05$ 立方分米 0.64 立方分米 $=640$ 毫升\n\n故答案为: $4200,3.05,0.05,640$ 。"} {"id": "24663", "image": [], "answer": "解$\\begin{array}{llllll} 502 & 813 & 4.905 & 1.06 & 1.06\\end{array}$\n\n【分析】 $1 \\mathrm{~m}=100 \\mathrm{~cm}, 1 \\mathrm{~m}^{2}=100 \\mathrm{dm}^{2}, 1 \\mathrm{dm}^{3}=1000 \\mathrm{~cm}^{3}, 1 \\mathrm{~L}=1 \\mathrm{dm}^{3}=1000 \\mathrm{~mL}$, 根据这几个进率进行单\n位换算即可。\n\n【详解】 $5.02 \\mathrm{~m}=502 \\mathrm{~cm} ; 8.13 \\mathrm{~m}^{2}=813 \\mathrm{dm}^{2}$\n\n$4905 \\mathrm{~cm}^{3}=4.905 \\mathrm{dm}^{3} ; 1060 \\mathrm{~mL}=1.06 \\mathrm{~L}=1.06 \\mathrm{dm}^{3}$\n\n【点睛】本题考查了单位换算,掌握单位间的进率是解题的关键。", "solution": "null", "level": "五年级", "question": "$5.02 \\mathrm{~m}=($\n)cm\n$8.13 \\mathrm{~m}^{2}=($\n$\\mathrm{dm}^{2}$\n$4905 \\mathrm{~cm}^{3}=(\\quad) \\mathrm{dm}^{3} \\quad 1060 \\mathrm{~mL}=(\\quad) \\mathrm{L}=(\\quad) \\mathrm{dm}^{3}$", "options": [], "subject": "度量几何学", "analysis": "解$\\begin{array}{llllll} 502 & 813 & 4.905 & 1.06 & 1.06\\end{array}$\n\n【分析】 $1 \\mathrm{~m}=100 \\mathrm{~cm}, 1 \\mathrm{~m}^{2}=100 \\mathrm{dm}^{2}, 1 \\mathrm{dm}^{3}=1000 \\mathrm{~cm}^{3}, 1 \\mathrm{~L}=1 \\mathrm{dm}^{3}=1000 \\mathrm{~mL}$, 根据这几个进率进行单\n位换算即可。\n\n【详解】 $5.02 \\mathrm{~m}=502 \\mathrm{~cm} ; 8.13 \\mathrm{~m}^{2}=813 \\mathrm{dm}^{2}$\n\n$4905 \\mathrm{~cm}^{3}=4.905 \\mathrm{dm}^{3} ; 1060 \\mathrm{~mL}=1.06 \\mathrm{~L}=1.06 \\mathrm{dm}^{3}$\n\n【点睛】本题考查了单位换算,掌握单位间的进率是解题的关键。"} {"id": "24666", "image": [], "answer": "解$3040 \\quad 3 \\quad 600$\n\n【分析】根据 $1 \\mathrm{~m}^{3}=1000 \\mathrm{dm}^{3}, 1 \\mathrm{~L}=1000 \\mathrm{~mL}$ ,高级单位换算成低级单位,乘进率,低级单位换算成高级单位, 除以进率, 据此解答。\n\n【详解】 $3.04 \\mathrm{~m}^{3}=3040 \\mathrm{dm}^{3}$;\n\n因为 $3.6 \\mathrm{~L}=3 \\mathrm{~L}+0.6 \\mathrm{~L}, 0.6 \\mathrm{~L}=600 \\mathrm{~mL}$, 所以 $3.6 \\mathrm{~L}=3 \\mathrm{~L} 600 \\mathrm{~mL}$ 。\n\n【点睛】本题考查单位之间的互化,关键是熟记进率。", "solution": "null", "level": "五年级", "question": "$3.04 \\mathrm{~m}^{3}=($\n) $\\mathrm{dm}^{3} \\quad 3.6 \\mathrm{~L}=($\n) $\\mathrm{L}($\n$\\mathrm{mL}$", "options": [], "subject": "度量几何学", "analysis": "解$3040 \\quad 3 \\quad 600$\n\n【分析】根据 $1 \\mathrm{~m}^{3}=1000 \\mathrm{dm}^{3}, 1 \\mathrm{~L}=1000 \\mathrm{~mL}$ ,高级单位换算成低级单位,乘进率,低级单位换算成高级单位, 除以进率, 据此解答。\n\n【详解】 $3.04 \\mathrm{~m}^{3}=3040 \\mathrm{dm}^{3}$;\n\n因为 $3.6 \\mathrm{~L}=3 \\mathrm{~L}+0.6 \\mathrm{~L}, 0.6 \\mathrm{~L}=600 \\mathrm{~mL}$, 所以 $3.6 \\mathrm{~L}=3 \\mathrm{~L} 600 \\mathrm{~mL}$ 。\n\n【点睛】本题考查单位之间的互化,关键是熟记进率。"} {"id": "24822", "image": ["12657.jpg"], "answer": "解$13 \\quad 300$\n\n【分析】根据题意可知, 这个书柜的占地面积等于这个长方体的底面积, 靠墙部分(仅书柜背面靠墙)的面积等于这个长方体的后面的面积, 根据长方形的面积公式: $\\mathrm{S}=\\mathrm{ab}$, 把数据代入公式解答, 再根据 1 平方米 $=100$ 平方分米,换算成平方分米即可。\n\n【详解】 $2 \\times 0.5=1$ (平方米)\n\n$2 \\times 1.5=3$ (平方米)\n\n3 平方米 $=300$ 平方分米\n\n即这个长方体书柜的占地面积是 1 平方米,靠墙部分(仅书柜背面靠墙)的面积是 3 平方米。合 300 平方分米。\n\n【点睛】此题主要考查长方体的表面积, 关键要弄清求的是哪个面的面积。", "solution": "null", "level": "五年级", "question": "如图, 靠墙摆放一个长 2 米, 宽 0.5 米, 高 1.5 米的长方体书柜, 这个长方体书柜的占地面积是 ( )平方米,靠墙部分(仅书柜背面靠墙)的面积是( )平方米。合( )平方分米。\n\n", "options": [], "subject": "度量几何学", "analysis": "解$13 \\quad 300$\n\n【分析】根据题意可知, 这个书柜的占地面积等于这个长方体的底面积, 靠墙部分(仅书柜背面靠墙)的面积等于这个长方体的后面的面积, 根据长方形的面积公式: $\\mathrm{S}=\\mathrm{ab}$, 把数据代入公式解答, 再根据 1 平方米 $=100$ 平方分米,换算成平方分米即可。\n\n【详解】 $2 \\times 0.5=1$ (平方米)\n\n$2 \\times 1.5=3$ (平方米)\n\n3 平方米 $=300$ 平方分米\n\n即这个长方体书柜的占地面积是 1 平方米,靠墙部分(仅书柜背面靠墙)的面积是 3 平方米。合 300 平方分米。\n\n【点睛】此题主要考查长方体的表面积, 关键要弄清求的是哪个面的面积。"} {"id": "24847", "image": [], "answer": "解\n\n5.5 平方分米 $=550$ 平方厘米 90 升 $=90$ 立方分米 $=0.09$ 立方米\n\n1650 立方厘米 $=1.65$ 立方分米\n\n3.9 升 $=3900$ 毫升 $=3900$ 立方厘米\n\n0.76 立方米 $=760$ 立方分米\n\n8.06 升 $=8$ 升 60 毫升\n\n故答案为: $550 ; 90,0.09 ; 1.65 ; 3900,3900 ; 760 ; 8,60$ 。", "solution": "null", "level": "五年级", "question": "(2022 秋・鹿邑县期末)\n5.5 平方分米 $=$ $\\qquad$平方厘米\n90 升 $=$ $\\qquad$立方分米 $=$ $\\qquad$立方米\n1650 立方厘米 $=$ $\\qquad$立方分米\n3.9 升 $=$ $\\qquad$毫升 $=$ $\\qquad$立方厘米\n0.76 立方米 $=$ $\\qquad$立方分米\n8.06 升 $=$ $\\qquad$升 $\\qquad$毫升", "options": [], "subject": "度量几何学", "analysis": "解\n\n5.5 平方分米 $=550$ 平方厘米 90 升 $=90$ 立方分米 $=0.09$ 立方米\n\n1650 立方厘米 $=1.65$ 立方分米\n\n3.9 升 $=3900$ 毫升 $=3900$ 立方厘米\n\n0.76 立方米 $=760$ 立方分米\n\n8.06 升 $=8$ 升 60 毫升\n\n故答案为: $550 ; 90,0.09 ; 1.65 ; 3900,3900 ; 760 ; 8,60$ 。"} {"id": "24275", "image": [], "answer": "解329\n\n【分析】一个数, 如果只有 1 和它本身两个因数, 那么这样的数叫做质数, 32 以内的质数有: $2 、 3$ 、\n$5 、 7 、 11 、 13 、 17 、 19 、 23 、 29 、 31$, 据此找出两个质数和为 32 的数即可。\n\n【详解】分析可知, $32=3+29=13+19$ 。\n\n【点睛】掌握质数的意义, 根据 32 以内的质数找出符合条件的两个质数是解答题目的关键。", "solution": "null", "level": "五年级", "question": "“哥德巴赫猜想”认为, 每一个不小于 4 的偶数都可以写成两个质数之和, 如 $4=2+2,6=3+3$, $8=3+5,12=5+7,20=13+7 \\ldots \\ldots$ 那么, $32=(\\quad)+(\\quad$ 。", "options": [], "subject": "逻辑题", "analysis": "解329\n\n【分析】一个数, 如果只有 1 和它本身两个因数, 那么这样的数叫做质数, 32 以内的质数有: $2 、 3$ 、\n$5 、 7 、 11 、 13 、 17 、 19 、 23 、 29 、 31$, 据此找出两个质数和为 32 的数即可。\n\n【详解】分析可知, $32=3+29=13+19$ 。\n\n【点睛】掌握质数的意义, 根据 32 以内的质数找出符合条件的两个质数是解答题目的关键。"} {"id": "12411", "image": [], "answer": "答案:X", "solution": "null", "level": "五年级", "question": "偶数都是合数, 奇数都是质数。( )", "options": [], "subject": "逻辑题", "analysis": "答案:X"} {"id": "11567", "image": [], "answer": "答案:", "solution": "null", "level": "五年级", "question": "住在中国香港的姑姑为美美寄来一个", "options": [], "subject": "逻辑题", "analysis": "答案:"} {"id": "24614", "image": [], "answer": "解256199\n\n【分析】 $2 、 5 、 3$ 的倍数的特征:\n\n2 的倍数: 个位上是 $0 、 2 、 4 、 6 、 8$ 的数;\n\n5 的倍数:个位上是的 0 或 5 数;\n\n3 的倍数:各个数位上的数字的和是 3 的倍数的数。\n\n奇数和偶数, 自然数中, 是 2 的倍数的数叫做偶数(0 也是偶数),不是 2 的倍数的数叫做奇数。质数和合数, 一个数如果只有 1 和它本身两个因数, 这样的数叫做质数; 一个数如果除了 1 和它本身还有别的因数, 这样的数叫做合数。\n\n【详解】第一位数:既是偶数又是质数的数是 2 ;\n\n第二位数:既是 5 的倍数, 又是 5 的因数的数是 5 ;\n\n第三位数: 既是 2 的倍数, 又是 3 的倍数的一位数是 6 ;\n\n第四位数:既不是质数,也不是合数的数是 1 ;\n\n第五位数:既是奇数, 又是合数是一位数是 9 ;\n\n第六位数: 一位数中最大的合数是 9 ;\n\n所以这个银行密码: 256199 。\n\n【点睛】本题把 $2 、 5 、 3$ 的倍数的特征、奇数偶数、质数合数与银行卡六位密码相结合, 充满趣味又不失严谨的风范。只是思考量稍大,解题时注意别马虎。", "solution": "null", "level": "五年级", "question": "妈妈的银行卡密码是一个六位数。根据下面信息, 银行卡密码是()。第一位数: 既是偶数, 又是质数; 第二位数: 既是 5 的倍数, 又是 5 的因数;\n第三位数:既是 2 的倍数, 又是 3 的倍数; 第四位数:既不是质数, 也不是合数;\n\n第五位数:既是奇数, 又是合数; 第六位数:一位数中最大的合数。", "options": [], "subject": "组合数学", "analysis": "解256199\n\n【分析】 $2 、 5 、 3$ 的倍数的特征:\n\n2 的倍数: 个位上是 $0 、 2 、 4 、 6 、 8$ 的数;\n\n5 的倍数:个位上是的 0 或 5 数;\n\n3 的倍数:各个数位上的数字的和是 3 的倍数的数。\n\n奇数和偶数, 自然数中, 是 2 的倍数的数叫做偶数(0 也是偶数),不是 2 的倍数的数叫做奇数。质数和合数, 一个数如果只有 1 和它本身两个因数, 这样的数叫做质数; 一个数如果除了 1 和它本身还有别的因数, 这样的数叫做合数。\n\n【详解】第一位数:既是偶数又是质数的数是 2 ;\n\n第二位数:既是 5 的倍数, 又是 5 的因数的数是 5 ;\n\n第三位数: 既是 2 的倍数, 又是 3 的倍数的一位数是 6 ;\n\n第四位数:既不是质数,也不是合数的数是 1 ;\n\n第五位数:既是奇数, 又是合数是一位数是 9 ;\n\n第六位数: 一位数中最大的合数是 9 ;\n\n所以这个银行密码: 256199 。\n\n【点睛】本题把 $2 、 5 、 3$ 的倍数的特征、奇数偶数、质数合数与银行卡六位密码相结合, 充满趣味又不失严谨的风范。只是思考量稍大,解题时注意别马虎。"} {"id": "24378", "image": [], "answer": "解570、750 570、975 570、750 750、570\n\n【分析】 2 的倍数特征:个位上的数字是 $0 、 2 、 4 、 6 、 8$ 的数是 2 的倍数。\n\n5 的倍数特征:个位上的数字是 0 或 5 的数是 5 的倍数。\n\n既是 2 的倍数又是 5 的倍数的特征:个位上的数字是 0 的数, 既是 2 的倍数, 又是 5 的倍数。\n\n$2,3,5$ 的倍数的特征: 个位上的数字是 0 , 各个数位上的数字的和是 3 的倍数的数。\n\n【详解】 2 的倍数:570、750;\n\n有因数 5: 570、975;\n同时是 2 和 5 的倍数: $570 、 750$;\n\n同时有因数 2, 3, 5: 750、570。\n\n【点睛】关键是掌握 $2 、 3 、 5$ 的倍数的特征。", "solution": "null", "level": "五年级", "question": "从数字 $0,7,5,9$ 中任取 3 个按要求组成三位数。(每项写两个数)\n\n2 的倍数: ( );\n\n有因数 5: ( );\n\n同时是 2 和 5 的倍数: ( );\n\n同时有因数 2, 3, 5: ( )。", "options": [], "subject": "组合数学", "analysis": "解570、750 570、975 570、750 750、570\n\n【分析】 2 的倍数特征:个位上的数字是 $0 、 2 、 4 、 6 、 8$ 的数是 2 的倍数。\n\n5 的倍数特征:个位上的数字是 0 或 5 的数是 5 的倍数。\n\n既是 2 的倍数又是 5 的倍数的特征:个位上的数字是 0 的数, 既是 2 的倍数, 又是 5 的倍数。\n\n$2,3,5$ 的倍数的特征: 个位上的数字是 0 , 各个数位上的数字的和是 3 的倍数的数。\n\n【详解】 2 的倍数:570、750;\n\n有因数 5: 570、975;\n同时是 2 和 5 的倍数: $570 、 750$;\n\n同时有因数 2, 3, 5: 750、570。\n\n【点睛】关键是掌握 $2 、 3 、 5$ 的倍数的特征。"} {"id": "24921", "image": [], "answer": "解$874 \\quad 470 \\quad 870$\n\n【分析】一个数的个位数字是 $0 、 2 、 4 、 6 、 8$ 的数就是 2 的倍数, 要使这个三位数是 2 的倍数且最大则让百位、十位和个位的数字最大即可, 也就是 874 ; 一个数的个位数字是 0 或 5 的数字是 5 的倍数,则这个三位数的个位数字一定是 0 , 然后让百位和十位上的数字最小即可, 也就是 470 ; 要使这个三位数同时是 $2 、 3 、 5$ 的倍数, 则这个三位数的个位数字一定是 0 , 且百位和十位上的数字之和是 3 的倍数, 也就是这个数字是 870 。\n【详解】由分析可知:\n\n在 $0 、 7 、 8 、 4$ 中选 3 个数字组成三位数, 使它成为 2 的倍数, 最大是 874 ; 成为 5 的倍数, 最小是 470 ; 成为 $2 、 3 、 5$ 的公倍数, 最大是 870 。\n\n【点睛】本题考查 $2 、 3 、 5$ 的数特征, 明确它们的特征是解题的关键。", "solution": "null", "level": "五年级", "question": "在 $0 、 7 、 8 、 4$ 中选 3 个数字组成三位数, 使它成为 2 的倍数, 最大是 $(\\quad)$; 成为 5 的倍数,最小是 $($ ) ; 成为 $2 、 3 、 5$ 的公倍数, 最大是 $($ 。", "options": [], "subject": "组合数学", "analysis": "解$874 \\quad 470 \\quad 870$\n\n【分析】一个数的个位数字是 $0 、 2 、 4 、 6 、 8$ 的数就是 2 的倍数, 要使这个三位数是 2 的倍数且最大则让百位、十位和个位的数字最大即可, 也就是 874 ; 一个数的个位数字是 0 或 5 的数字是 5 的倍数,则这个三位数的个位数字一定是 0 , 然后让百位和十位上的数字最小即可, 也就是 470 ; 要使这个三位数同时是 $2 、 3 、 5$ 的倍数, 则这个三位数的个位数字一定是 0 , 且百位和十位上的数字之和是 3 的倍数, 也就是这个数字是 870 。\n【详解】由分析可知:\n\n在 $0 、 7 、 8 、 4$ 中选 3 个数字组成三位数, 使它成为 2 的倍数, 最大是 874 ; 成为 5 的倍数, 最小是 470 ; 成为 $2 、 3 、 5$ 的公倍数, 最大是 870 。\n\n【点睛】本题考查 $2 、 3 、 5$ 的数特征, 明确它们的特征是解题的关键。"} {"id": "24926", "image": [], "answer": "解$3 \\# \\#$ 三\n【分析】对于任何一㙉灯, 由于原来都不亮, 那么, 当它的开关被按了奇数次时, 灯是亮着的; 当它的开关被按了偶数次时, 灯是关着的; 一典灯的开关被按的次数, 恰等于这軖灯的编号的因数的个数;要求哪些灯还亮着, 就是问哪些灯的编号的因数有奇数个。据此解答即可。\n\n【详解】 1 的因数只有 $1 ; 2$ 的因数有: $1 、 2 ; 3$ 的因数有:1、3;4 的因数有: $1 、 2 、 4 ; 5$ 的因数有: $1 、 5 ; 6$ 的因数有: $1 、 2 、 3 、 6 ; 7$ 的因数有: $1 、 7 ; 8$ 的因数有: $1 、 2 、 4 、 8 ; 9$ 的因数有: $1 、 3$ 、 $9 ; 10$ 的因数有: $1 、 2 、 5 、 10$; 则编号的因数有奇数个的有: $1 、 4 、 9$ 共 3 血, 所以还有 3 㙉灯是亮着的。\n\n【点睛】本题考查求一个数的因数, 明确开关被按了奇数次时, 灯是亮着的是解题的关键。", "solution": "null", "level": "五年级", "question": "有个房间里有 10 盏灯, 都是关着的。房间外面有 10 位学生, 都是编号 1-10, 现在要求学生排队逐个进入房间, 将灯的号码是自己的编号的整数倍的灯按一下, 那么 10 个人全部做完之后, 还有\n\n( )监灯是亮着的。", "options": [], "subject": "组合数学", "analysis": "解$3 \\# \\#$ 三\n【分析】对于任何一㙉灯, 由于原来都不亮, 那么, 当它的开关被按了奇数次时, 灯是亮着的; 当它的开关被按了偶数次时, 灯是关着的; 一典灯的开关被按的次数, 恰等于这軖灯的编号的因数的个数;要求哪些灯还亮着, 就是问哪些灯的编号的因数有奇数个。据此解答即可。\n\n【详解】 1 的因数只有 $1 ; 2$ 的因数有: $1 、 2 ; 3$ 的因数有:1、3;4 的因数有: $1 、 2 、 4 ; 5$ 的因数有: $1 、 5 ; 6$ 的因数有: $1 、 2 、 3 、 6 ; 7$ 的因数有: $1 、 7 ; 8$ 的因数有: $1 、 2 、 4 、 8 ; 9$ 的因数有: $1 、 3$ 、 $9 ; 10$ 的因数有: $1 、 2 、 5 、 10$; 则编号的因数有奇数个的有: $1 、 4 、 9$ 共 3 血, 所以还有 3 㙉灯是亮着的。\n\n【点睛】本题考查求一个数的因数, 明确开关被按了奇数次时, 灯是亮着的是解题的关键。"} {"id": "24927", "image": [], "answer": "解公平\n\n【分析】一个数, 如果只有 1 和它本身两个因数, 那么这样的数叫做质数; 一个数, 如果除了 1 和它本身还有别的因数, 那么这样的数叫做合数; 先找出质数、合数的个数, 如果两种数字的个数相同,那么摸到质数和合数的可能性相同, 游戏公平, 如果两种数字的个数不相同, 哪种数字的球个数越多摸到的可能性越大, 游戏不公平。\n\n【详解】-9 中, 质数有 $2,3,5,7$, 一共 4 个, 合数有 $4,6,8,9$, 一共 4 个, 质数球的个数与合数球的个数相同, 他们得分的可能性相同, 游戏公平。\n\n【点睛】掌握质数、合数的意义, 准确找出质数、合数的个数是解答题目的关键。", "solution": "null", "level": "五年级", "question": "把 9 个完全相同的球, 先按照 1-9 编号, 然后放进盒子里。小红和小明玩摸球游戏, 规定二人轮流摸球, 每次任意摸出一个球再放回, 如果摸到的球的编号是质数, 小红得 1 分; 如果摸到的球的编号是合数, 小明得 1 分; 如果摸到的球的编号既不是质数也不是合数, 两人都不得分。你认为这个游戏规则公平吗? ( )。", "options": [], "subject": "组合数学", "analysis": "解公平\n\n【分析】一个数, 如果只有 1 和它本身两个因数, 那么这样的数叫做质数; 一个数, 如果除了 1 和它本身还有别的因数, 那么这样的数叫做合数; 先找出质数、合数的个数, 如果两种数字的个数相同,那么摸到质数和合数的可能性相同, 游戏公平, 如果两种数字的个数不相同, 哪种数字的球个数越多摸到的可能性越大, 游戏不公平。\n\n【详解】-9 中, 质数有 $2,3,5,7$, 一共 4 个, 合数有 $4,6,8,9$, 一共 4 个, 质数球的个数与合数球的个数相同, 他们得分的可能性相同, 游戏公平。\n\n【点睛】掌握质数、合数的意义, 准确找出质数、合数的个数是解答题目的关键。"} {"id": "24974", "image": [], "answer": "解$874 \\quad 470 \\quad 870$\n\n【分析】一个数的个位数字是 $0 、 2 、 4 、 6 、 8$ 的数就是 2 的倍数, 要使这个三位数是 2 的倍数且最大则让百位、十位和个位的数字最大即可, 也就是 874 ; 一个数的个位数字是 0 或 5 的数字是 5 的倍数,则这个三位数的个位数字一定是 0 , 然后让百位和十位上的数字最小即可, 也就是 470 ; 要使这个三位数同时是 $2 、 3 、 5$ 的倍数, 则这个三位数的个位数字一定是 0 , 且百位和十位上的数字之和是 3 的倍数, 也就是这个数字是 870 。\n【详解】由分析可知:\n\n在 $0 、 7 、 8 、 4$ 中选 3 个数字组成三位数, 使它成为 2 的倍数, 最大是 874 ; 成为 5 的倍数, 最小是 470 ; 成为 $2 、 3 、 5$ 的公倍数, 最大是 870 。\n\n【点睛】本题考查 $2 、 3 、 5$ 的数特征, 明确它们的特征是解题的关键。", "solution": "null", "level": "五年级", "question": "在 $0 、 7 、 8 、 4$ 中选 3 个数字组成三位数, 使它成为 2 的倍数, 最大是 $($; 成为 5 的倍数,最小是 ( ); 成为 $2 、 3 、 5$ 的公倍数, 最大是 $($ 。", "options": [], "subject": "组合数学", "analysis": "解$874 \\quad 470 \\quad 870$\n\n【分析】一个数的个位数字是 $0 、 2 、 4 、 6 、 8$ 的数就是 2 的倍数, 要使这个三位数是 2 的倍数且最大则让百位、十位和个位的数字最大即可, 也就是 874 ; 一个数的个位数字是 0 或 5 的数字是 5 的倍数,则这个三位数的个位数字一定是 0 , 然后让百位和十位上的数字最小即可, 也就是 470 ; 要使这个三位数同时是 $2 、 3 、 5$ 的倍数, 则这个三位数的个位数字一定是 0 , 且百位和十位上的数字之和是 3 的倍数, 也就是这个数字是 870 。\n【详解】由分析可知:\n\n在 $0 、 7 、 8 、 4$ 中选 3 个数字组成三位数, 使它成为 2 的倍数, 最大是 874 ; 成为 5 的倍数, 最小是 470 ; 成为 $2 、 3 、 5$ 的公倍数, 最大是 870 。\n\n【点睛】本题考查 $2 、 3 、 5$ 的数特征, 明确它们的特征是解题的关键。"} {"id": "24998", "image": [], "answer": "解120\n\n【详解】略", "solution": "null", "level": "五年级", "question": "幼儿园的钟老师买来一些糖果分给小朋友, 糖果的数量是一个三位数, 有因数 2 , 同时又是 3 和 5 的倍数, 则钟老师最少买来( )颗糖果.", "options": [], "subject": "组合数学", "analysis": "解120\n\n【详解】略"} {"id": "25029", "image": [], "answer": "解$372 \\quad 732$\n\n【分析】质数是指除了 1 和它本身的两个因数以外再没有其他的因数的数。 2 的倍数的数的特征是:个位上是 $0 、 2 、 4 、 6 、 8$ 的数; 3 的倍数的数的特征是: 各位上的数字之和是 3 的倍数, 这个数就是 3 的倍数; 先写出 10 以内所有的质数; 再根据 2,3 的倍数的特征找出符合要求的三位数, 据此解答。\n\n【详解】 10 以内的质数有: $2 、 3 、 5 、 7$ 。\n\n根据 2 的倍数特征可知, 这个三位数个位上是 2 。\n\n$2+3+7=12,12$ 是 3 的倍数;\n\n$2+3+5=10,10$ 不是 3 的倍数;\n\n$2+5+7=14,14$ 不是 3 的倍数;\n\n可见这个三位数只能由 $2 、 3 、 7$ 组成。\n\n所以这个三位数是 372 和 732。\n\n【点睛】此题的解题关键是理解掌握质数的意义以及 $2 、 3$ 的倍数的特征。", "solution": "null", "level": "五年级", "question": "从 10 以内的质数中选出 3 个数, 这三个数组成既是 2 的倍数, 又是 3 的倍数的三位数是 $($ )和( )。", "options": [], "subject": "组合数学", "analysis": "解$372 \\quad 732$\n\n【分析】质数是指除了 1 和它本身的两个因数以外再没有其他的因数的数。 2 的倍数的数的特征是:个位上是 $0 、 2 、 4 、 6 、 8$ 的数; 3 的倍数的数的特征是: 各位上的数字之和是 3 的倍数, 这个数就是 3 的倍数; 先写出 10 以内所有的质数; 再根据 2,3 的倍数的特征找出符合要求的三位数, 据此解答。\n\n【详解】 10 以内的质数有: $2 、 3 、 5 、 7$ 。\n\n根据 2 的倍数特征可知, 这个三位数个位上是 2 。\n\n$2+3+7=12,12$ 是 3 的倍数;\n\n$2+3+5=10,10$ 不是 3 的倍数;\n\n$2+5+7=14,14$ 不是 3 的倍数;\n\n可见这个三位数只能由 $2 、 3 、 7$ 组成。\n\n所以这个三位数是 372 和 732。\n\n【点睛】此题的解题关键是理解掌握质数的意义以及 $2 、 3$ 的倍数的特征。"} {"id": "25056", "image": [], "answer": "解公平\n\n【分析】一个数只有 1 和它本身两个因数, 这个数叫做质数。一个数除了 1 和它本身两个因数, 还有其他的因数, 这个数叫做合数。据此分别找出 $1 \\sim 9$ 里面质数和合数的个数, 如果质数的个数多, 则甲赢的可能性大, 如果合数的个数多, 则乙赢的可能性大, 如果质数和合数的个数相同, 则甲乙赢的可能性相等。\n\n【详解】在 1 9 中, $2 、 3 、 5 、 7$ 是质数, 一共 4 个, $4 、 6 、 8 、 9$ 是合数, 一共 4 个, 所以甲乙赢的可能性相等, 这个规则是公平的。\n\n【点睛】本题考查游戏的公平性以及可能性大小的判断, 理解不确定事件发生的可能性的大小与事物的数量有关, 数量越多, 可能性越大, 反之则越小, 数量相同, 可能性也相同。", "solution": "null", "level": "五年级", "question": "布袋里有 9 张卡片分别写着 $1 \\sim 9$, 摸出的卡片上如果是质数表示甲赢, 如果是合数表示乙赢,这个规则是 ( )的。(填公平或不公平)", "options": [], "subject": "组合数学", "analysis": "解公平\n\n【分析】一个数只有 1 和它本身两个因数, 这个数叫做质数。一个数除了 1 和它本身两个因数, 还有其他的因数, 这个数叫做合数。据此分别找出 $1 \\sim 9$ 里面质数和合数的个数, 如果质数的个数多, 则甲赢的可能性大, 如果合数的个数多, 则乙赢的可能性大, 如果质数和合数的个数相同, 则甲乙赢的可能性相等。\n\n【详解】在 1 9 中, $2 、 3 、 5 、 7$ 是质数, 一共 4 个, $4 、 6 、 8 、 9$ 是合数, 一共 4 个, 所以甲乙赢的可能性相等, 这个规则是公平的。\n\n【点睛】本题考查游戏的公平性以及可能性大小的判断, 理解不确定事件发生的可能性的大小与事物的数量有关, 数量越多, 可能性越大, 反之则越小, 数量相同, 可能性也相同。"} {"id": "12391", "image": [], "answer": "答案: $\\times$", "solution": "null", "level": "五年级", "question": "一箱橙子有 6 袋, 其中有 5 袋质量相同, 另外有一袋质量不足, 要找出较轻的一袋, 比较合适的分法是 $1,1,4$.", "options": [], "subject": "组合数学", "analysis": "答案: $\\times$"} {"id": "12392", "image": [], "answer": "答案:2", "solution": "null", "level": "五年级", "question": "8 瓶水中 7 瓶是纯净水, 另外 1 瓶是盐水, 盐水稍微重一点, 用天平至少称 \\$ \\qquad \\$次,能保证找到这瓶盐水。", "options": [], "subject": "组合数学", "analysis": "答案:2"} {"id": "24980", "image": [], "answer": "解(1)3、43、11、31、47\n\n(2) $25,12 、 18 、 81 、 33 、 82 、 34 、 57,116,87,1101,645,14,417$\n\n(3)12、18、82、34、116、14\n$(4) 3 、 25 、 43 、 81 、 33 、 11 、 57 、 31 、 47 、 87 、 1101 、 645 、 417$\n\n【分析】根据奇数、偶数、质数和合数的意义: 是 2 的倍数的数叫做偶数, 不是 2 的倍数的数叫做奇数; 一个数如果只有 1 和它本身两个因数, 那么这个数叫做质数; 一个数如果除了 1 和它本身还有其它的因数, 那么这个数叫做合数。据此填空即可。\n\n【详解】(1)质数:3、43、11、31、47\n\n(2) 合数: $25 、 12 、 18 、 81 、 33 、 82 、 34 、 57,116 、 87,1101 、 645 、 14 、 417$\n\n(3)偶数: $12 、 18 、 82 、 34 、 116 、 14$\n\n(4) 奇数: $3 、 25 、 43 、 81 、 33 、 11 、 57,31 、 47,87 、 1101 、 645 、 417$\n\n【点睛】本题考查奇数、偶数、质数和合数, 明确它们的定义是解题的关键。", "solution": "null", "level": "五年级", "question": "分一分。\n\n$\\begin{array}{lllllllllllllllllll}3 & 25 & 12 & 18 & 43 & 81 & 33 & 11 & 82 & 34 & 57 & 31 & 47 & 116 & 87 & 1101 & 645 & 14 & 417\\end{array}$\n\n(1)质数: ( );\n\n(2)合数: ( );\n\n(3)偶数: ( );\n\n(4)奇数: ( )。", "options": [], "subject": "代数", "analysis": "解(1)3、43、11、31、47\n\n(2) $25,12 、 18 、 81 、 33 、 82 、 34 、 57,116,87,1101,645,14,417$\n\n(3)12、18、82、34、116、14\n$(4) 3 、 25 、 43 、 81 、 33 、 11 、 57 、 31 、 47 、 87 、 1101 、 645 、 417$\n\n【分析】根据奇数、偶数、质数和合数的意义: 是 2 的倍数的数叫做偶数, 不是 2 的倍数的数叫做奇数; 一个数如果只有 1 和它本身两个因数, 那么这个数叫做质数; 一个数如果除了 1 和它本身还有其它的因数, 那么这个数叫做合数。据此填空即可。\n\n【详解】(1)质数:3、43、11、31、47\n\n(2) 合数: $25 、 12 、 18 、 81 、 33 、 82 、 34 、 57,116 、 87,1101 、 645 、 14 、 417$\n\n(3)偶数: $12 、 18 、 82 、 34 、 116 、 14$\n\n(4) 奇数: $3 、 25 、 43 、 81 、 33 、 11 、 57,31 、 47,87 、 1101 、 645 、 417$\n\n【点睛】本题考查奇数、偶数、质数和合数, 明确它们的定义是解题的关键。"} {"id": "11674", "image": [], "answer": "答案: $(5,4) ;(4,4)$", "solution": "null", "level": "五年级", "question": "李明坐在教室的第 5 列第 4 行, 用数对表示是 \\$ \\qquad \\$王丽坐在李明的左边用数对表示是 \\$ \\qquad \\$ $\\circ$", "options": [], "subject": "代数", "analysis": "答案: $(5,4) ;(4,4)$"} {"id": "11681", "image": [], "answer": "答案: \\times$", "solution": "null", "level": "五年级", "question": "茸茸在班级的位置是第 5 行、第 2 列,用数对表示是 $(5,2)$ 。", "options": [], "subject": "代数", "analysis": "答案: \\times$"} {"id": "11760", "image": [], "answer": "答案:\n$(3,6)$\n$(3,7)$\n\n【分析】数对中第一个数字表示列, 第二个数字表示行, 由此写出王强的位置即可; 根据“明明坐在王强正后方的第一个位置”可知, 他们在同一列, 明明所在的行数是王强所在行数加 1 , 据此解答即可。\n\n【详解】王强在班里的位置是第 3 列第 6 行, 用数对表示是 $(3,6)$;明明的位置用数对表示是 $(3,7)$ 。\n\n【点睛】明确数对表示位置时的特点是解答本题的关键。", "solution": "null", "level": "五年级", "question": "王强在班里的位置是第 3 列第 6 行, 用数对表示是 (); 明明坐在王强正后方的第一个位置,明明的位置用数对表示是 $(\\quad)$ 。", "options": [], "subject": "代数", "analysis": "答案:\n$(3,6)$\n$(3,7)$\n\n【分析】数对中第一个数字表示列, 第二个数字表示行, 由此写出王强的位置即可; 根据“明明坐在王强正后方的第一个位置”可知, 他们在同一列, 明明所在的行数是王强所在行数加 1 , 据此解答即可。\n\n【详解】王强在班里的位置是第 3 列第 6 行, 用数对表示是 $(3,6)$;明明的位置用数对表示是 $(3,7)$ 。\n\n【点睛】明确数对表示位置时的特点是解答本题的关键。"} {"id": "12344", "image": [], "answer": "答案: $\\times$", "solution": "null", "level": "五年级", "question": "一个图形绕一点顺时针旋转 $90^{\\circ}$ 和逆时针旋转 $90^{\\circ}$, 所得到的两个图形正好重合。", "options": [], "subject": "变换几何", "analysis": "答案: $\\times$"} {"id": "12353", "image": [], "answer": "答案: $\\times$", "solution": "null", "level": "五年级", "question": "等边三角形绕三条高的交点旋转 $60^{\\circ}$ 后能与原来的图形重合。( )", "options": [], "subject": "变换几何", "analysis": "答案: $\\times$"} {"id": "11670", "image": [], "answer": "答案: \\cdot \\times$", "solution": "null", "level": "五年级", "question": "小芳的位置是 $(5,6)$ ,那么她同桌的位置可以用 $(5,5)$ 表示。( )", "options": [], "subject": "解析几何", "analysis": "答案: \\cdot \\times$"} {"id": "11671", "image": [], "answer": "答案: $7 ; 5$", "solution": "null", "level": "五年级", "question": "乐乐的位置用数对表示是 $(7,5)$, 它表示第 \\$ \\qquad \\$列第 \\$ \\qquad \\$行。", "options": [], "subject": "解析几何", "analysis": "答案: $7 ; 5$"} {"id": "11673", "image": [], "answer": "答案: 直角", "solution": "null", "level": "五年级", "question": "在同一幅图上, 如果点 A 用数对表示为 $(2,5)$, 点 B 用数对表示为\n\n$(2,1)$ ,点 C 用数对表示为 $(5,1)$ ,那么三角形 $A B C$ 一定是 \\$ \\qquad \\$三角形。", "options": [], "subject": "解析几何", "analysis": "答案: 直角"} {"id": "11675", "image": [], "answer": "答案: ( 13,17$)$", "solution": "null", "level": "五年级", "question": "莫茸在电影院看电影, 她坐在第 19 列第 12 排用数对表示是 $(19,12)$,洋洋坐在第 13 列第 17 排用数对表示是 \\$ \\qquad \\$。", "options": [], "subject": "解析几何", "analysis": "答案: ( 13,17$)$"} {"id": "11680", "image": [], "answer": "答案: \\times$", "solution": "null", "level": "五年级", "question": "用(5,5)来表示位置, 两个 5 表示的意思是一样的。( )", "options": [], "subject": "解析几何", "analysis": "答案: \\times$"} {"id": "11748", "image": ["2492.jpg"], "answer": "$(3,5)$$(1,4)$$(6,4)$$(2,2)$$(3,0)$", "solution": "null", "level": "五年级", "question": "11. 用数对标出下面场馆的位置。\n\n动物园示意图\n\n\n\n熊猫馆( ), 大象馆( ), 海洋馆( ), 猴山( ), 大门( )。", "options": [], "subject": "解析几何", "analysis": ""} {"id": "14090", "image": [], "answer": "错", "solution": "null", "level": "四年级", "question": "在一个轴对称图形上, 两点到对称轴的距离相等, 如果将这个图形沿对称轴对折, 这两点一定重合。", "options": [], "subject": "变换几何", "analysis": ""} {"id": "14091", "image": [], "answer": "对", "solution": "null", "level": "四年级", "question": "林、百、人、日、目、亩这些都是轴对称图形。", "options": [], "subject": "组合几何学", "analysis": ""} {"id": "6631", "image": [], "answer": "错", "solution": "null", "level": "一年级", "question": "两个一样的三角形可以拼成一个长方形。", "options": [], "subject": "组合几何学", "analysis": "详解$\\times$"} {"id": "6649", "image": [], "answer": "对", "solution": "null", "level": "一年级", "question": "正方形的四条边都一样长。", "options": [], "subject": "度量几何学", "analysis": "_详解$\\sqrt{ }$"} {"id": "6678", "image": [], "answer": "错", "solution": "null", "level": "一年级", "question": "哥哥今年 15 岁, 比弟弟多 7 岁, 弟弟今年 7 岁。", "options": [], "subject": "算术", "analysis": "详解$\\times$"} {"id": "6679", "image": [], "answer": "对", "solution": "null", "level": "一年级", "question": "15 比 7 大 8 , 也可以说 7 比 15 少 8 。", "options": [], "subject": "算术", "analysis": "详解$\\sqrt{ }$"} {"id": "6680", "image": [], "answer": "对", "solution": "null", "level": "一年级", "question": "$1 、 3 、 5$ 这三个数的和与 18 减去 9 的差相等。", "options": [], "subject": "算术", "analysis": "详解$\\sqrt{ }$"} {"id": "6681", "image": [], "answer": "对", "solution": "null", "level": "一年级", "question": "被减数是 14 , 减数是 9 , 差是 5", "options": [], "subject": "算术", "analysis": "详解$\\sqrt{ }$"} {"id": "6902", "image": [], "answer": "对", "solution": "null", "level": "一年级", "question": "66 是由 6 个十和 6 个一组成的。", "options": [], "subject": "算术", "analysis": "详解$\\sqrt{}$"} {"id": "6903", "image": [], "answer": "错", "solution": "null", "level": "一年级", "question": "88 减去 5 个十等于 83 。", "options": [], "subject": "算术", "analysis": "详解$\\times$"} {"id": "6904", "image": [], "answer": "错", "solution": "null", "level": "一年级", "question": "$27+4=$ 改, 得数改的个位上的数是 7 。", "options": [], "subject": "算术", "analysis": "详解$\\times$"} {"id": "6905", "image": [], "answer": "错", "solution": "null", "level": "一年级", "question": "买一个书包需要 58 元,小明付了 50 元还找回 8 元。", "options": [], "subject": "算术", "analysis": "详解$\\times$"} {"id": "6906", "image": [], "answer": "错", "solution": "null", "level": "一年级", "question": "有 18 个红球, 黄球比红球多 6 个, 白球比黄球少 3 个。白球比红球多 9 个。", "options": [], "subject": "算术", "analysis": "详解$\\times$"} {"id": "6930", "image": [], "answer": "错", "solution": "null", "level": "一年级", "question": "8 个十减去 5 个十是 3 个十, 这个数是 3 。", "options": [], "subject": "算术", "analysis": "详解$\\times$"} {"id": "6983", "image": [], "answer": "详解(2)", "solution": "null", "level": "一年级", "question": "按 “ $\\uparrow \\rightarrow \\downarrow ”$ 的顺序排列, 接下来的第 10 个应该是 ( )。\n\n(1) $\\uparrow$\n\n(2) $\\rightarrow$\n\n(3) $\\downarrow$", "options": [], "subject": "组合数学", "analysis": "详解(2)"} {"id": "7040", "image": ["870.jpg", "871.jpg", "871.jpg"], "answer": "详解", "solution": "null", "level": "一年级", "question": "把同一类的两个事物用线连起来。\n\n", "options": [], "subject": "图论", "analysis": "详解"} {"id": "7041", "image": ["872.jpg", "873.jpg", "873.jpg"], "answer": "解", "solution": "null", "level": "一年级", "question": "把下列事物中是动物的用 $\\bigcirc$ 圈起来, 是植物的用 $\\square$ 圈起来, 是食物的用 $\\triangle$ 圈起来。\n\n", "options": [], "subject": "逻辑题", "analysis": "解"} {"id": "11035", "image": [], "answer": "$94000 \\quad 5$\n\n$\\begin{array}{lll}6 & 1400 \\quad 8\\end{array}$", "solution": "null", "level": "二年级", "question": "看谁算得快。(每空 2 分, 共 12 分)\n\n| 4 千克 +5000 克 $=(\\quad)$ 千克 | 6 千克 -2000 克 $=(\\quad)$ 克 |\n| :--- | :--- |\n| 1500 克 +3500 克 $=(\\quad)$ 千克 | 3000 克 +3 千克 $=(\\quad)$ 千克 |\n| 6 千克 -4600 克 $=(\\quad)$ 克 | $7500+500$ 克 $=(\\quad)$ 千克 |", "options": [], "subject": "算术", "analysis": "$94000 \\quad 5$\n\n$\\begin{array}{lll}6 & 1400 \\quad 8\\end{array}$"} {"id": "11036", "image": [], "answer": "$>\\quad><$", "solution": "null", "level": "二年级", "question": "在 $\\bigcirc$ 里填上 “>”、“<”或 “=”。(每空 1 分, 共 6 分) 5900 克 $\\bigcirc 6$ 千克 $\\quad 2000$ 克 $\\bigcirc 2$ 千克 $\\quad 900$ 克 +900 克 $\\bigcirc 2$ 千克 50 千克 $\\bigcirc 5000$ 克 $\\quad 7$ 千克 $\\bigcirc 777$ 克 $\\quad 8$ 千克 $\\bigcirc 6$ 千克 +3000 克", "options": [], "subject": "算术", "analysis": "$>\\quad><$"} {"id": "11037", "image": ["2283.jpg"], "answer": "2 千克 $=2000$ 克 $\\quad 2300-2000=300$ (克)\n\n答: 这个饼干盒子重 300 克。", "solution": "null", "level": "二年级", "question": "一盒饼干连盒重 2300 克, 这个饼干盒子重多少克?\n\n\n\n答:这个饼干盒子重()克。", "options": [], "subject": "算术", "analysis": "2 千克 $=2000$ 克 $\\quad 2300-2000=300$ (克)\n\n答: 这个饼干盒子重 300 克。"} {"id": "11038", "image": [], "answer": "$50-15=35$ (元) $\\quad 35 \\div 7=5$ (元)\n\n答: 每千克桃子 5 元。", "solution": "null", "level": "二年级", "question": "小明用 50 元买了 7 千克桃子,找回 15 元,每千克桃子多少元?", "options": [], "subject": "算术", "analysis": "$50-15=35$ (元) $\\quad 35 \\div 7=5$ (元)\n\n答: 每千克桃子 5 元。"} {"id": "11039", "image": ["2284.jpg"], "answer": "(1) $1500+500=2000$ (克)\n\n2000 克 $=2$ 千克\n\n答: 雪梨和葡萄一共 2000 克, 合 2 千克。\n\n(2) 2 千克 $=2000$ 克 $\\quad 2000-300=1700$ (克)\n\n答: 西瓜比草莓多买 1700 克。", "solution": "null", "level": "二年级", "question": "妈妈在水果商店买了一些水果。\n\n\n\n(1)雪梨和葡萄一共多少克?合多少千克?\n\n(2) 西瓜比草莓多买多少克?", "options": [], "subject": "算术", "analysis": "(1) $1500+500=2000$ (克)\n\n2000 克 $=2$ 千克\n\n答: 雪梨和葡萄一共 2000 克, 合 2 千克。\n\n(2) 2 千克 $=2000$ 克 $\\quad 2000-300=1700$ (克)\n\n答: 西瓜比草莓多买 1700 克。"} {"id": "10460", "image": [], "answer": "答案】(1) 解: $4 \\times 2 \\times 2$\n\n$=8 \\times 2$\n\n$=16$ (元)\n\n$9 \\times 2=18$ (元)\n\n$16+18=34($ 元 $)$\n\n答: 一共要花 34 元。\n\n(2) 解: 答案不唯一, 如: 买 1 千克果冻和 500 克饼干需要花多少钱?\n\n$5 \\times 2+8$\n\n$=10+8$\n\n$=18$ (元)\n\n答: 买 1 千克果冻和 500 克饼干需要花 18 元。\n\n【知识点】千克与克之间的换算与比较", "solution": "null", "level": "二年级", "question": "下面是宏达超市部分商品的价格表。\n\n| 名称 | 果冻 | 软面包 | 饼干 | 蛋卷 |\n| :---: | :---: | :---: | :---: | :---: |\n| 价格 | 5 元 $/ 500$ 克 | 4 元 $/ 500$ 克 | 8 元 $/ 500$ 克 | 9 元 $/ 500$ 克 |\n\n(1)李阿姨买了 2 千克软面包和 1 千克蛋卷, 一共要花多少元?\n\n(2) 你还能提出什么数学问题?解答出来。", "options": [], "subject": "算术", "analysis": "答案】(1) 解: $4 \\times 2 \\times 2$\n\n$=8 \\times 2$\n\n$=16$ (元)\n\n$9 \\times 2=18$ (元)\n\n$16+18=34($ 元 $)$\n\n答: 一共要花 34 元。\n\n(2) 解: 答案不唯一, 如: 买 1 千克果冻和 500 克饼干需要花多少钱?\n\n$5 \\times 2+8$\n\n$=10+8$\n\n$=18$ (元)\n\n答: 买 1 千克果冻和 500 克饼干需要花 18 元。\n\n【知识点】千克与克之间的换算与比较"} {"id": "10461", "image": [], "answer": "答案】(1) 解: $16 \\div 4=4$\n\n$500+500=1000$ (克)\n\n$1000+500=1500$ (克)\n\n$1500+500=2000$ (克)\n\n2000 克 $=2$ 千克\n\n答: 买 500 克山竹的价钱可以买 2 千克苹果。\n\n(2) 解: $7+7=14($ 元)\n\n$14+16=30$ (元)\n\n答: 一共要花 30 元。\n\n(3) 解:榴莲的价格是龙眼的几倍?\n\n$21 \\div 7=3$\n\n答: 榴莲的价格是龙眼的 3 倍。\n\n【知识点】千克与克之间的换算与比较", "solution": "null", "level": "二年级", "question": "下图是“三果演义”水果店的水果标价牌。\n\n苹果: 4 元 500 克\n\n橘子: 2 元 500 克\n\n龙眼: 7 元 500 克\n\n山竹: 16 元 500 克\n\n榴莲: 21 元 500 克\n\n(1) 买 500 克山竹的价钱可以买多少千克苹果?\n\n(2) 妈妈买了 1 千克龙眼和 500 克山竹, 一共要花多少钱?\n\n(3) 你还能提出哪些数学问题? 解答出来。", "options": [], "subject": "算术", "analysis": "答案】(1) 解: $16 \\div 4=4$\n\n$500+500=1000$ (克)\n\n$1000+500=1500$ (克)\n\n$1500+500=2000$ (克)\n\n2000 克 $=2$ 千克\n\n答: 买 500 克山竹的价钱可以买 2 千克苹果。\n\n(2) 解: $7+7=14($ 元)\n\n$14+16=30$ (元)\n\n答: 一共要花 30 元。\n\n(3) 解:榴莲的价格是龙眼的几倍?\n\n$21 \\div 7=3$\n\n答: 榴莲的价格是龙眼的 3 倍。\n\n【知识点】千克与克之间的换算与比较"} {"id": "10462", "image": [], "answer": "答案】解: $5000-2000=3000$ (克)\n\n3000 克 $=3$ 千克\n\n答: 苹果重 3 千克。\n\n【知识点】千克与克之间的换算与比较", "solution": "null", "level": "二年级", "question": "梨和苹果共重 5000 克, 其中梨重 2000 克, 苹果重多少千克?\n\n\n\n【知识点】千克与克之间的换算与比较", "options": [], "subject": "算术", "analysis": "答案】解: $5000-2000=3000$ (克)\n\n3000 克 $=3$ 千克\n\n答: 苹果重 3 千克。\n\n【知识点】千克与克之间的换算与比较"} {"id": "10255", "image": [], "answer": "$64 ; 57 ; 67$", "solution": "null", "level": "二年级", "question": "( 6 分 )算一算。\n\n$24 \\mathrm{~cm}+40 \\mathrm{~cm}=(\\quad) \\mathrm{cm}$\n\n$27 \\mathrm{~cm}+30 \\mathrm{~cm}=(\\quad) \\mathrm{cm}$\n\n$37 \\mathrm{~cm}+30 \\mathrm{~cm}=(\\quad) \\mathrm{cm}$", "options": [], "subject": "算术", "analysis": "$64 ; 57 ; 67$"} {"id": "10256", "image": [], "answer": "$27 ; 43 ; 44$", "solution": "null", "level": "二年级", "question": "(6 分 )算一算。\n\n$47 \\mathrm{~cm}-20 \\mathrm{~cm}=(\\quad) \\mathrm{cm}$\n\n$83 \\mathrm{~cm}-40 \\mathrm{~cm}=(\\quad) \\mathrm{cm}$\n\n$74 \\mathrm{~cm}-30 \\mathrm{~cm}=(\\quad) \\mathrm{cm}$\n100 厘米()1米 10 厘米\n\n4 米( ) 400 厘米", "options": [], "subject": "算术", "analysis": "$27 ; 43 ; 44$"} {"id": "10285", "image": [], "answer": "$\\quad 5338 \\quad 38 \\quad 90 \\quad 50 \\quad 92$\n\n$\\begin{array}{lllll}90 & 55 & 24 & 80 & 80\\end{array}$", "solution": "null", "level": "二年级", "question": "( 5 分 )口算。\n\n| $65-12=$ | $27+11=$ | $61+29=$ | $92-42=$ | $12+80=$ |\n| :--- | ---: | ---: | ---: | :--- |\n| $33+57=$ | $75-20=$ | $35-11=$ | $28+52=$ | $96-16=$ |", "options": [], "subject": "算术", "analysis": "$\\quad 5338 \\quad 38 \\quad 90 \\quad 50 \\quad 92$\n\n$\\begin{array}{lllll}90 & 55 & 24 & 80 & 80\\end{array}$"} {"id": "10286", "image": [], "answer": "(1) $86-59=27$\n\n(2) $17+28=45$\n\n(3) $6+57=63$\n\n(4) $53+28-35=46$\n\n(5) $76-8+19=87$\n\n(6) $50+27-9=68$​", "solution": "null", "level": "二年级", "question": "( 18 分)列坚式计算。\n(1) $86-59=$\n(2) $17+28=$\n(3) $6+57=$\n(4) $53+28-35=$\n(5) $76-8+19=$\n(6) $50+27-9=$", "options": [], "subject": "算术", "analysis": "(1) $86-59=27$\n\n(2) $17+28=45$\n\n(3) $6+57=63$\n\n(4) $53+28-35=46$\n\n(5) $76-8+19=87$\n\n(6) $50+27-9=68$​"} {"id": "10287", "image": ["2050.jpg"], "answer": "[](本地图片-2661.jpg)", "solution": "null", "level": "二年级", "question": "8分)填一填。\n\n", "options": [], "subject": "算术", "analysis": "[](本地图片-2661.jpg)"} {"id": "10344", "image": ["2081.jpg", "2082.jpg", "2082.jpg"], "answer": "(1) $2 \\times 4=8$\n\n(2) $3 \\times 3=9$\n\n", "solution": "null", "level": "二年级", "question": "( 6 分)看图列式。\n\n(1)\n\n\n\n$\\square \\times \\square=\\square$\n\n(2) 要要要\n\n$\\square \\times \\square=\\square$", "options": [], "subject": "算术", "analysis": "(1) $2 \\times 4=8$\n\n(2) $3 \\times 3=9$\n\n"} {"id": "10345", "image": ["2083.jpg"], "answer": "(10) (20) (25) (15)(5)", "solution": "null", "level": "二年级", "question": "( 10 分 )", "options": [], "subject": "算术", "analysis": "(10) (20) (25) (15)(5)"} {"id": "10378", "image": ["2101.jpg", "2102.jpg", "2102.jpg"], "answer": "解:\n\n", "solution": "null", "level": "二年级", "question": "( 4 分 )小兔回家。(连一连)\n", "options": [], "subject": "算术", "analysis": "解:\n\n"} {"id": "10409", "image": [], "answer": "$2 \\times 9=18$\n\n$2 \\times 7=14$\n\n$6 \\times 8=48$\n\n$8 \\times 4=32$\n\n$5 \\times 6=30$\n\n$6 \\times 7=42$\n\n$4 \\times 9=36$\n\n$3 \\times 5=15$\n\n$39-9=30$\n\n$82-50=32$\n\n$27+30=57$\n\n$45-5=40$\n\n$13-9=4$\n\n$22+32=54$\n\n$12+24=36$\n\n$25-4=21$", "solution": "null", "level": "二年级", "question": "8分)看谁算的又对又快。\n\n| $39-9=$ | $82-50=$ | $27+30=$ | $45-5=$ |\n| :--- | ---: | :--- | :--- |\n| $13-9=$ | $22+32=$ | $12+24=$ | $25-4=$ |", "options": [], "subject": "算术", "analysis": "$2 \\times 9=18$\n\n$2 \\times 7=14$\n\n$6 \\times 8=48$\n\n$8 \\times 4=32$\n\n$5 \\times 6=30$\n\n$6 \\times 7=42$\n\n$4 \\times 9=36$\n\n$3 \\times 5=15$\n\n$39-9=30$\n\n$82-50=32$\n\n$27+30=57$\n\n$45-5=40$\n\n$13-9=4$\n\n$22+32=54$\n\n$12+24=36$\n\n$25-4=21$"} {"id": "10410", "image": [], "answer": "(1) $7 \\times 7=49$\n\n(2) $9 \\times 4=36$\n\n(3) $55+26=81$", "solution": "null", "level": "二年级", "question": "( 9 分 )列式计算。\n\n(1)两个因数都是 7 ,积是多少?\n\n(2) 4 个 9 相加的和是多少?\n\n(3) 比 55 多 26 的数是多少?", "options": [], "subject": "算术", "analysis": "(1) $7 \\times 7=49$\n\n(2) $9 \\times 4=36$\n\n(3) $55+26=81$"} {"id": "10411", "image": ["2117.jpg", "2118.jpg", "2119.jpg", "2120.jpg", "2121.jpg", "2117.jpg", "2118.jpg", "2119.jpg", "2120.jpg", "2121.jpg"], "answer": "(1)\n\n\n\n(2)\n\n\n\n(3)\n\n\n\n(4)\n\n\n\n(5)\n\n", "solution": "null", "level": "二年级", "question": "( 10 分 )用坚式计算。\n(1) $\\begin{array}{r}9 \\\\ \\times \\quad 4 \\\\ \\hline\\end{array}$\n(2) $\\begin{array}{r}5 \\\\ \\times \\quad 8 \\\\ \\hline\\end{array}$\n(3) $\\begin{array}{r}7 \\\\ \\times \\quad 6 \\\\ \\hline\\end{array}$\n(4) $\\begin{array}{r}4 \\\\ \\times \\quad 4 \\\\ \\hline\\end{array}$\n(5) $\\begin{array}{r}3 \\\\ \\times \\quad 9 \\\\ \\hline\\end{array}$", "options": [], "subject": "算术", "analysis": "(1)\n\n\n\n(2)\n\n\n\n(3)\n\n\n\n(4)\n\n\n\n(5)\n\n"} {"id": "10443", "image": ["2127.jpg", "2128.jpg", "2128.jpg"], "answer": "", "solution": "null", "level": "二年级", "question": "(5 分) 写出下面各个钟面上所指的时刻, 并算一算从上一时刻到下一时刻所经过的时间。\n\n", "options": [], "subject": "算术", "analysis": ""} {"id": "11440", "image": [], "answer": "乓球便宜。每个便宜 l 元钱。", "solution": "null", "level": "二年级", "question": "个乒乓球 2 元钱, 4 个羽毛球 12 元钱, 哪种球比较便宜?便宜多少钱?", "options": [], "subject": "算术", "analysis": "乓球便宜。每个便宜 l 元钱。"} {"id": "11441", "image": [], "answer": "2 元\n\n提示: 方法一: 先求单价, 再求总量。 $6 \\div 3 \\times 6=12$ (元)\n\n方法二: 6 瓶中包含着几个 3 瓶, 就包含着几个 6 元。\n\n6 瓶 $\\div 3$ 瓶 $\\times 6$ 元 $=12($ 元 $)$", "solution": "null", "level": "二年级", "question": "市饮料降价, 买 3 瓶鲜橙汁 6 元钱, 买 6 瓶多少钱?", "options": [], "subject": "算术", "analysis": "2 元\n\n提示: 方法一: 先求单价, 再求总量。 $6 \\div 3 \\times 6=12$ (元)\n\n方法二: 6 瓶中包含着几个 3 瓶, 就包含着几个 6 元。\n\n6 瓶 $\\div 3$ 瓶 $\\times 6$ 元 $=12($ 元 $)$"} {"id": "11442", "image": [], "answer": "36 \\div 6=6$ (天), $36 \\div 4=9$ (页)", "solution": "null", "level": "二年级", "question": "本书有 36 页, 小明每天看 6 页, 几天可以看完?如果 4 天看完, 平均每天看多少页?", "options": [], "subject": "算术", "analysis": "36 \\div 6=6$ (天), $36 \\div 4=9$ (页)"} {"id": "10477", "image": [], "answer": "$4 ; 6 ; 9 ; 28$\n\n$6 ; 40 ; 40 ; 38$\n\n$40 ; 100 ; 9 ; 62$", "solution": "null", "level": "二年级", "question": "直接写出得数。\n\n$$\n\\begin{aligned}\n& 12 \\div 3=\\quad 30 \\div 5=\\quad 81 \\div 9=7 \\times 4= \\\\\n& 48 \\div 8=\\quad 13+27=5 \\times 8=18+20= \\\\\n& 62-22=72+8+20=58-9-40=64+6-8=\n\\end{aligned}\n$$", "options": [], "subject": "算术", "analysis": "$4 ; 6 ; 9 ; 28$\n\n$6 ; 40 ; 40 ; 38$\n\n$40 ; 100 ; 9 ; 62$"} {"id": "10478", "image": ["2145.jpg"], "answer": "$3 \\times 9=27 ; 9 \\times 3=27$\n\n$27 \\div 3=9 ; 27 \\div 9=3$\n\n【分析】根据图意, 共有 9 组, 每组 3 个, 求一共的个数用乘法计算, 由此列出两个乘法算式; 再根据乘除法的互逆关系, 列出两个除法算式。\n\n【详解】 $3 \\times 9=27 ; 9 \\times 3=27$\n\n$27 \\div 3=9 ; \\quad 27 \\div 9=3$", "solution": "null", "level": "二年级", "question": "列算式。\n\n", "options": [], "subject": "算术", "analysis": "$3 \\times 9=27 ; 9 \\times 3=27$\n\n$27 \\div 3=9 ; 27 \\div 9=3$\n\n【分析】根据图意, 共有 9 组, 每组 3 个, 求一共的个数用乘法计算, 由此列出两个乘法算式; 再根据乘除法的互逆关系, 列出两个除法算式。\n\n【详解】 $3 \\times 9=27 ; 9 \\times 3=27$\n\n$27 \\div 3=9 ; \\quad 27 \\div 9=3$"} {"id": "10503", "image": [], "answer": "$16 ; 8 ; 54 ; 9 ; 16$\n\n4; 36; 48; 9; 36", "solution": "null", "level": "二年级", "question": "直接写得数。\n\n| $2 \\times 8=$ | $56 \\div 7=$ | $6 \\times 9=$ | $81 \\div 9=$ | $9 \\times 2-2=$ |\n| :--- | :--- | :--- | :--- | :--- |\n| $12 \\div 3=$ | $9 \\times 4=$ | $8 \\times 6=$ | $63 \\div 7=$ | $4 \\times 8+4=$ |", "options": [], "subject": "算术", "analysis": "$16 ; 8 ; 54 ; 9 ; 16$\n\n4; 36; 48; 9; 36"} {"id": "10463", "image": ["2139.jpg"], "answer": "答案】解:店主的主意不好。\n\n因为带骨的肉是 8 元 1 千克, 那就是说肉和骨头都是 8 元 1 千克, 如果骨头 2 元 1 千克, 肉 6 元 1 千克, 店主就亏了。\n\n【知识点】千克的认识与使用", "solution": "null", "level": "二年级", "question": "你觉得店主的主意好吗?\n\n", "options": [], "subject": "逻辑题", "analysis": "答案】解:店主的主意不好。\n\n因为带骨的肉是 8 元 1 千克, 那就是说肉和骨头都是 8 元 1 千克, 如果骨头 2 元 1 千克, 肉 6 元 1 千克, 店主就亏了。\n\n【知识点】千克的认识与使用"} {"id": "10258", "image": ["2046.jpg", "2046.jpg"], "answer": "解: 4 厘米 +2 厘米 $=6$ 厘米\n\n如图所示:\n\n", "solution": "null", "level": "二年级", "question": "6 分)画一条比 4 厘米多 2 厘米的线段。", "options": [], "subject": "度量几何学", "analysis": "解: 4 厘米 +2 厘米 $=6$ 厘米\n\n如图所示:\n\n"} {"id": "10259", "image": ["2047.jpg", "2047.jpg"], "answer": "解:如图:\n\n", "solution": "null", "level": "二年级", "question": "( 6 分 )在 $ֹ$ 左边 5 厘米处画一个 $\\triangle, 4$ 厘米处画一个 $\\square, 8$ 厘米处画一个ค。", "options": [], "subject": "画法几何学", "analysis": "解:如图:\n\n"} {"id": "10316", "image": ["2065.jpg", "2066.jpg", "2066.jpg"], "answer": "", "solution": "null", "level": "二年级", "question": "(6 分) 在下面的方格中, 分别画出一个锐角、一个直角和一个针角。\n", "options": [], "subject": "画法几何学", "analysis": ""} {"id": "10315", "image": ["2064.jpg"], "answer": "$(\\mathrm{V})$\n$(\\mathrm{V})$\n\n( )\n\n$( \\mathrm{V} )$\n\n$(\\mathrm{V})$", "solution": "null", "level": "二年级", "question": "(5 分 )下列物体表面有直角的在()里画“ $\\checkmark$ ”。\n \n\n$(\\quad)$\n\n$(\\quad)$\n\n$(\\quad)$\n\n$(\\quad)$", "options": [], "subject": "立体几何学", "analysis": "$(\\mathrm{V})$\n$(\\mathrm{V})$\n\n( )\n\n$( \\mathrm{V} )$\n\n$(\\mathrm{V})$"} {"id": "11456", "image": [], "answer": "错", "solution": "null", "level": "二年级", "question": "长方形、正方形、圆和平行四边形都是轴对称图形。( )", "options": [], "subject": "变换几何", "analysis": "$ 错 $"} {"id": "11459", "image": ["2409.jpg", "2410.jpg", "2410.jpg"], "answer": "", "solution": "null", "level": "二年级", "question": "下面的图案分别是从哪张对折后的纸上剪下来的?连一连。\n", "options": [], "subject": "组合几何学", "analysis": ""} {"id": "11317", "image": ["2366.jpg", "2367.jpg", "2367.jpg"], "answer": "", "solution": "null", "level": "二年级", "question": "规律填空。(共 6 分)\n", "options": [], "subject": "组合数学", "analysis": ""} {"id": "5863", "image": [], "answer": "解\n\n| $10-7$ |\n| :---: |\n| $13-3$ |\n| $6+10$ |\n| $19-10$ |", "solution": "null", "level": "一年级", "question": "线。\n\n| $10-7$ |\n| :---: |\n| $13-3$ |\n| $6+10$ |\n| $19-10$ |\n\n\n| 16 |\n| :---: |\n| 9 |\n| 3 |\n| 10 |\n\n\n| $20-10$ |\n| :--- |\n| $10-1$ |\n| $13-10$ |\n| $18-2$ |", "options": [], "subject": "算术", "analysis": "解\n\n| $10-7$ |\n| :---: |\n| $13-3$ |\n| $6+10$ |\n| $19-10$ |"} {"id": "5864", "image": [], "answer": "详解$11,15,16,3, \\quad 19,10,19,12, \\quad 13,19,10,19, \\quad 11,13,17,4, \\quad 17,18,13,6$", "solution": "null", "level": "一年级", "question": "算。\n\n| $14-3=$ | $5+10=$ | $10+6=$ | $9-6=$ |\n| :--- | :--- | :--- | :--- |\n| $7+12=$ | $18-8=$ | $13+6=$ | $10+2=$ |\n| $15-2=$ | $12+7=$ | $17-7=$ | $14+5=$ |\n| $18-7=$ | $19-6=$ | $12+5=$ | $14-10=$ |\n| $13+4=$ | $16+2=$ | | |\n| | | $17-4=$ | $16-10=$ |", "options": [], "subject": "算术", "analysis": "详解$11,15,16,3, \\quad 19,10,19,12, \\quad 13,19,10,19, \\quad 11,13,17,4, \\quad 17,18,13,6$"} {"id": "5865", "image": [], "answer": "解$13,12,9,19,18,9,10,12,19,11,12,11$", "solution": "null", "level": "一年级", "question": "接写出得数。\n\n| $18-7+2=$ | $15+4-7=$ | $6+1+2=$ |\n| :--- | :--- | :--- |\n| $12-2+9=$ | $15-4+7=$ | $6-5+8=$ |\n| $18+1-9=$ | $11+4-3=$ | |\n| | | $16+3+0=$ |\n| $0+16-5=$ | $17+2-7=$ | $19+0-8=$ |", "options": [], "subject": "算术", "analysis": "解$13,12,9,19,18,9,10,12,19,11,12,11$"} {"id": "6072", "image": ["173.jpg"], "answer": "$7+3=10,3+7=10$,", "solution": "null", "level": "一年级", "question": "8 分)看图列式并计算。\n\n\n\n$\\square \\mathrm{O} \\square=\\square$\n\n$\\square \\bigcirc \\square=\\square$\n\n$\\square \\bigcirc \\square=\\square$\n\n$\\square \\bigcirc \\square=\\square$", "options": [], "subject": "计数", "analysis": "$7+3=10,3+7=10$,"} {"id": "6635", "image": ["622.jpg", "622.jpg"], "answer": "解\n\n", "solution": "null", "level": "一年级", "question": "(6 分)\n正方形\n三角形\n长方形\n圆", "options": [], "subject": "组合几何学", "analysis": "解\n\n"} {"id": "6636", "image": ["623.jpg", "624.jpg", "624.jpg"], "answer": "详解\n\n", "solution": "null", "level": "一年级", "question": "(4 分) 用上面的哪个物体可以画出下面的图形, 请你连一连。\n", "options": [], "subject": "画法几何学", "analysis": "详解\n\n"} {"id": "6666", "image": ["644.jpg", "645.jpg", "646.jpg", "645.jpg", "646.jpg"], "answer": "详解\n\n\n\n24 ._详解\n\n", "solution": "null", "level": "一年级", "question": "(5 分)信鸽送信。\n", "options": [], "subject": "组合数学", "analysis": "详解\n\n\n\n24 ._详解\n\n"} {"id": "6148", "image": ["232.jpg", "233.jpg", "234.jpg", "234.jpg"], "answer": "", "solution": "null", "level": "一年级", "question": "(4 分) 连一连。\n\n\n(G)\n\n\n$\\theta$", "options": [], "subject": "立体几何学", "analysis": ""} {"id": "23415", "image": ["11538.jpg"], "answer": "【答案】解: 此题答案不唯一, 只要答案正确即可得分.\n\n(1)如: $D E / / C B, D F / / C B, F E / / C B$.\n\n(2)如: $E D \\perp A C, F D \\perp A C, F D \\perp A D$.\n\n(3)如: 钝角: $\\angle G F D=135^{\\circ}, \\angle C G B=\\angle F G E=105^{\\circ}$.\n\n直角有: $\\angle A D E=90^{\\circ}$.\n\n如: 锐角 $\\angle G C B=30^{\\circ}, \\angle A F D=45^{\\circ}, \\angle C G F=75^{\\circ}$.\n\n【解析】(1)直线 $D E / / B C$, 故直线 $D E$ 上的线段都与 $B C$ 平行.\n\n(2)根据 $\\angle C D E$ 和 $\\angle A C B$ 都是直角, 即可找出互相垂直的线段.\n\n(3)根据角的概念进行解答.\n\n本题考查平行线, 垂线以及角的概念, 难度不大.", "solution": "null", "level": "七年级", "question": "一副透明的直角三角尺, 按如图所示的位置摆放. 如果把三角尺的每条边看成线段,请根据图形解答下列问题:\n\n(1)找出图中一对互相平行的线段, 并用符号表示出来;\n\n(2)找出图中一对互相垂直的线段, 并用符号表示出来;\n\n(3)找出图中的一个钝角、一个直角和一个锐角, 用符号把它们表示出来, 并求出它们的度数. (不包括直角尺自身所成的角)\n\n", "options": [], "subject": "画法几何学", "analysis": "【答案】解: 此题答案不唯一, 只要答案正确即可得分.\n\n(1)如: $D E / / C B, D F / / C B, F E / / C B$.\n\n(2)如: $E D \\perp A C, F D \\perp A C, F D \\perp A D$.\n\n(3)如: 钝角: $\\angle G F D=135^{\\circ}, \\angle C G B=\\angle F G E=105^{\\circ}$.\n\n直角有: $\\angle A D E=90^{\\circ}$.\n\n如: 锐角 $\\angle G C B=30^{\\circ}, \\angle A F D=45^{\\circ}, \\angle C G F=75^{\\circ}$.\n\n【解析】(1)直线 $D E / / B C$, 故直线 $D E$ 上的线段都与 $B C$ 平行.\n\n(2)根据 $\\angle C D E$ 和 $\\angle A C B$ 都是直角, 即可找出互相垂直的线段.\n\n(3)根据角的概念进行解答.\n\n本题考查平行线, 垂线以及角的概念, 难度不大."} {"id": "23421", "image": ["11544.jpg", "11545.jpg", "11545.jpg"], "answer": "【答案】解: $\\because \\angle 1=\\angle B A C$,\n\n$\\therefore A B / / E F$.\n\n$\\therefore \\angle B+(\\angle 2+\\angle 3)=180^{\\circ}$.\n\n$\\because \\angle 2=45^{\\circ}, \\angle 3=20^{\\circ}$,\n\n\n\n$\\therefore \\angle B=115^{\\circ}$.\n\n$\\because E F / / A B$,\n\n$\\therefore \\angle F A B=\\angle 3=20^{\\circ}$,\n\n$\\because \\angle C A B=\\angle C A F+\\angle F A B$, 且 $\\angle C A F=15^{\\circ}$,\n\n$\\therefore \\angle C A B=20^{\\circ}+15^{\\circ}=35^{\\circ}$.\n\n$\\because A B / / C D$,\n\n$\\therefore \\angle A C D=\\angle C A B=35^{\\circ}$.\n\n【解析】由 $\\angle 1=\\angle B A C$ 说明 $A B$ 与 $E F$ 的关系, 再由平行线的性质求出 $\\angle B$; 先由 $A B$ 与 $E F$ 的关系及角的和差关系求出 $\\angle C A B$, 再利用平行线的判定和性质求出 $\\angle A C D$.\n\n本题考查了平行线的性质与判定、角的和差计算, 掌握平行线的性质和判定是解决本题的关键.", "solution": "null", "level": "七年级", "question": "如图, $A B / / C D$, 点 $E 、 F$ 分别在线段 $A D 、 B C$ 上, 连结 $A C$ 交 $E F$ 于 $G, \\angle 1=\\angle B A C$. 若 $\\angle C A F=15^{\\circ}, \\angle 2=45^{\\circ}, \\angle 3=20^{\\circ}$, 求 $\\angle B$ 和 $\\angle A C D$ 的度数.\n\n", "options": [], "subject": "画法几何学", "analysis": "【答案】解: $\\because \\angle 1=\\angle B A C$,\n\n$\\therefore A B / / E F$.\n\n$\\therefore \\angle B+(\\angle 2+\\angle 3)=180^{\\circ}$.\n\n$\\because \\angle 2=45^{\\circ}, \\angle 3=20^{\\circ}$,\n\n\n\n$\\therefore \\angle B=115^{\\circ}$.\n\n$\\because E F / / A B$,\n\n$\\therefore \\angle F A B=\\angle 3=20^{\\circ}$,\n\n$\\because \\angle C A B=\\angle C A F+\\angle F A B$, 且 $\\angle C A F=15^{\\circ}$,\n\n$\\therefore \\angle C A B=20^{\\circ}+15^{\\circ}=35^{\\circ}$.\n\n$\\because A B / / C D$,\n\n$\\therefore \\angle A C D=\\angle C A B=35^{\\circ}$.\n\n【解析】由 $\\angle 1=\\angle B A C$ 说明 $A B$ 与 $E F$ 的关系, 再由平行线的性质求出 $\\angle B$; 先由 $A B$ 与 $E F$ 的关系及角的和差关系求出 $\\angle C A B$, 再利用平行线的判定和性质求出 $\\angle A C D$.\n\n本题考查了平行线的性质与判定、角的和差计算, 掌握平行线的性质和判定是解决本题的关键."} {"id": "23416", "image": ["11539.jpg", "11540.jpg", "11540.jpg"], "answer": "【答案】解: $\\angle B C D=\\angle B-\\angle D$.\n\n理由如下:\n\n如图, 过点 $C$ 作 $C F / / A B$,\n\n\n\n$\\therefore \\angle B=\\angle B C F$.\n\n$\\because A B / / D E, C F / / A B$,\n\n$\\therefore C F / / D E$.\n\n$\\therefore \\angle D C F=\\angle D$.\n\n$\\therefore \\angle B-\\angle D=\\angle B C F-\\angle D C F$.\n\n$\\because \\angle B C D=\\angle B C F-\\angle D C F$,\n\n$\\therefore \\angle B C D=\\angle B-\\angle D$.\n【解析】本题考查了平行线的判定与性质, 解决本题的关键是准确区分平行线的判定与性质, 并熟练运用. 先添加辅助线, 然后根据 $C F / / A B$ 可得内错角相等, 再根据平行可得 $C F / / D E$, 根据平行线的性质可得内错角相等, 然后根据角的关系可得结论.", "solution": "null", "level": "七年级", "question": "如图, $A B / / D E$, 则 $\\angle B C D, \\angle B, \\angle D$ 有何关系? 为什么?\n\n", "options": [], "subject": "组合几何学", "analysis": "【答案】解: $\\angle B C D=\\angle B-\\angle D$.\n\n理由如下:\n\n如图, 过点 $C$ 作 $C F / / A B$,\n\n\n\n$\\therefore \\angle B=\\angle B C F$.\n\n$\\because A B / / D E, C F / / A B$,\n\n$\\therefore C F / / D E$.\n\n$\\therefore \\angle D C F=\\angle D$.\n\n$\\therefore \\angle B-\\angle D=\\angle B C F-\\angle D C F$.\n\n$\\because \\angle B C D=\\angle B C F-\\angle D C F$,\n\n$\\therefore \\angle B C D=\\angle B-\\angle D$.\n【解析】本题考查了平行线的判定与性质, 解决本题的关键是准确区分平行线的判定与性质, 并熟练运用. 先添加辅助线, 然后根据 $C F / / A B$ 可得内错角相等, 再根据平行可得 $C F / / D E$, 根据平行线的性质可得内错角相等, 然后根据角的关系可得结论."} {"id": "23417", "image": ["11541.jpg"], "answer": "【答案】解: $\\angle A E D=\\angle C$; 理由如下:\n\n证明: $\\because \\angle 1+\\angle 4=180^{\\circ}$ (邻补角定义)\n\n$\\angle 1+\\angle 2=180^{\\circ}$ (已知)\n\n$\\therefore \\angle 2=\\angle 4$ (同角的补角相等)\n\n$\\therefore E F / / A B$ (内错角相等, 两直线平行)\n\n$\\therefore \\angle 3=\\angle A D E$ (两直线平行, 内错角相等)\n\n又 $\\because \\angle B=\\angle 3$ (已知),\n\n$\\therefore \\angle A D E=\\angle B$ (等量代换),\n\n$\\therefore D E / / B C$ (同位角相等, 两直线平行)\n\n$\\therefore \\angle A E D=\\angle C$ (两直线平行, 同位角相等).\n\n【解析】本题考查了平行线的判定和性质; 由图中题意可先猜测 $\\angle A E D=\\angle C$, 那么需证明 $D E / / B C$. 题中说 $\\angle 1+\\angle 2=180^{\\circ}$, 而 $\\angle 1+\\angle 4=180^{\\circ}$ 所以 $\\angle 2=\\angle 4$, 那么可得到 $A B / / E F$, 题中有 $\\angle 3=\\angle B$, 所以应根据平行得到 $\\angle 3$ 与 $\\angle A D E$ 之间的关系为相等. 就得到了 $\\angle B$ 与 $\\angle A D E$ 之间的关系为相等, 那么 $D E / / B C$ 。", "solution": "null", "level": "七年级", "question": "如图, 已知 $\\angle 1+\\angle 2=180, \\angle 3=\\angle B$, 试判断 $\\angle A E D$ 与 $\\angle C$ 的大小关系, 并对结论\n\n进行说理。\n\n", "options": [], "subject": "度量几何学", "analysis": "【答案】解: $\\angle A E D=\\angle C$; 理由如下:\n\n证明: $\\because \\angle 1+\\angle 4=180^{\\circ}$ (邻补角定义)\n\n$\\angle 1+\\angle 2=180^{\\circ}$ (已知)\n\n$\\therefore \\angle 2=\\angle 4$ (同角的补角相等)\n\n$\\therefore E F / / A B$ (内错角相等, 两直线平行)\n\n$\\therefore \\angle 3=\\angle A D E$ (两直线平行, 内错角相等)\n\n又 $\\because \\angle B=\\angle 3$ (已知),\n\n$\\therefore \\angle A D E=\\angle B$ (等量代换),\n\n$\\therefore D E / / B C$ (同位角相等, 两直线平行)\n\n$\\therefore \\angle A E D=\\angle C$ (两直线平行, 同位角相等).\n\n【解析】本题考查了平行线的判定和性质; 由图中题意可先猜测 $\\angle A E D=\\angle C$, 那么需证明 $D E / / B C$. 题中说 $\\angle 1+\\angle 2=180^{\\circ}$, 而 $\\angle 1+\\angle 4=180^{\\circ}$ 所以 $\\angle 2=\\angle 4$, 那么可得到 $A B / / E F$, 题中有 $\\angle 3=\\angle B$, 所以应根据平行得到 $\\angle 3$ 与 $\\angle A D E$ 之间的关系为相等. 就得到了 $\\angle B$ 与 $\\angle A D E$ 之间的关系为相等, 那么 $D E / / B C$ 。"} {"id": "23440", "image": ["11569.jpg", "11570.jpg", "11571.jpg", "11570.jpg", "11571.jpg"], "answer": "解【答案】解: (1) $\\because \\alpha=30^{\\circ}, A C / / B D$,\n\n$\\therefore \\angle C B D=30^{\\circ}$,\n\n$\\because B C$ 平分 $\\angle A B D$,\n$\\therefore \\angle A B E=\\angle C B D=30^{\\circ}$,\n\n$\\therefore \\angle B A C=180^{\\circ}-\\angle A B E-\\alpha=180^{\\circ}-30^{\\circ}-30^{\\circ}=120^{\\circ}$,\n\n又 $\\because \\angle B A E=\\angle C A E$,\n\n$\\therefore \\angle C A E=\\frac{1}{2} \\angle B A C=\\frac{1}{2} \\times 120^{\\circ}=60^{\\circ}$;\n\n(2)根据题意画图,如图1所示,\n\n$\\because \\angle B A E=100^{\\circ}, \\angle B A E: \\angle C A E=5: 1$,\n\n$\\therefore \\angle C A E=20^{\\circ}$,\n\n$\\therefore \\angle B A C=\\angle B A E-\\angle C A E=100^{\\circ}-20^{\\circ}=80^{\\circ}$,\n\n$\\because A C / / B D$,\n\n$\\therefore \\angle A B D=180^{\\circ}-\\angle B A C=100^{\\circ}$,\n\n又 $\\because B C$ 平分 $\\angle A B D$,\n\n$\\therefore \\angle C B D=\\frac{1}{2} \\angle A B D=\\frac{1}{2} \\times 100^{\\circ}=50^{\\circ}$,\n\n$\\therefore \\alpha=\\angle C B D=50^{\\circ}$;\n\n(3)11如图2所示,\n\n$\\because A C / / B D$\n\n$\\therefore \\angle C B D=\\angle A C B=\\alpha$,\n\n$\\because B C$ 平分 $\\angle A B D$,\n\n\n\n图 2\n\n$\\therefore \\angle A B D=2 \\angle C B D=2 \\alpha$,\n\n$\\therefore \\angle B A C=180^{\\circ}-\\angle A B D=180^{\\circ}-2 \\alpha$,\n\n又 $\\because \\angle B A E: \\angle C A E=n$,\n\n$\\therefore(\\angle B A C+\\angle C A E): \\angle C A E=n$,\n\n$\\left(180^{\\circ}-2 \\alpha+\\angle C A E\\right): \\angle C A E=n$,\n\n解得 $\\angle C A E=\\frac{180^{\\circ}-2 \\alpha}{n-1}$;\n\n(2)如图3所示,\n\n$\\because A C / / B D$,\n\n$\\therefore \\angle C B D=\\angle A C B=\\alpha$,\n\n$\\because B C$ 平分 $\\angle A B D$,\n\n\n\n图 3\n\n$\\therefore \\angle A B D=2 \\angle C B D=2 \\alpha$,\n\n$\\therefore \\angle B A C=180^{\\circ}-\\angle A B D=180^{\\circ}-2 \\alpha$,\n\n又 $\\because \\angle B A E: \\angle C A E=n$,\n$\\therefore(\\angle B A C-\\angle C A E): \\angle C A E=n$,\n\n$\\left(180^{\\circ}-2 \\alpha-\\angle C A E\\right): \\angle C A E=n$,\n\n解得 $\\angle C A E=\\frac{180^{\\circ}-2 \\alpha}{n+1}$.\n\n综上 $\\angle C A E$ 的度数为 $\\frac{180^{\\circ}-2 \\alpha}{n-1}$ 或 $\\frac{180^{\\circ}-2 \\alpha}{n+1}$.\n\n【解析】(1)根据平行线的性质可得 $\\angle C B D$ 的度数, 再根据角平分线的性质可得 $A B E$ 的度数, 应用三角形内角和计算 $\\angle B A C$ 的度数, 由已知条件 $\\angle B A E=\\angle C A E$, 可计算出 $\\angle C A E$ 的度数;\n\n(2)根据题意画出图形, 先根据 $\\angle B A E: \\angle C A E=5: 1$ 可计算出 $\\angle C A E$ 的度数, 由 $\\angle B A E=$ $100^{\\circ}$ 可计算出 $\\angle B A C$ 的度数, 再根据平行线的性质和角平分线的性质, 计算出 $\\angle C B D$ 的度数,即可得出结论;\n\n(3)根据题意可分两种情况,\n\n(1)若点 $E$ 运动到 $l_{1}$ 上方, 根据平行线的性质由 $\\alpha$ 可计算出 $\\angle C B D$ 的度数, 再根据角平分线的性质和平行线的性质, 计算出 $\\angle B A C$ 的度数, 再 $\\angle B A E: \\angle C A E=n, \\angle B A E=\\angle B A C+$ $\\angle C A E$, 列出等量关系求解即可等处结论;\n\n(2)若点 $E$ 运动到 $l_{1}$ 下方, 根据平行线的性质由 $\\alpha$ 可计算出 $\\angle C B D$ 的度数, 再根据角平分线的性质和平行线的性质, 计算出 $\\angle B A C$ 的度数, 再 $\\angle B A E: \\angle C A E=n, \\angle B A E=\\angle B A C-$ $\\angle C A E$ 列出等量关系求解即可等处结论.\n\n本题主要考查平行线的性质和角平分线的性质, 两直线平行, 同位角相等.两直线平行,同旁内角互补. 两直线平行,内错角相等. 合理应用平行线的性质是解决本题的关键.", "solution": "null", "level": "七年级", "question": "如图, $A C / / B D, B C$ 平分 $\\angle A B D$, 设 $\\angle A C B$ 为 $\\alpha$, 点 $E$ 是射线 $B C$ 上的一个动点.\n\n(1)若 $\\alpha=30^{\\circ}$ 时, 且 $\\angle B A E=\\angle C A E$, 求 $\\angle C A E$ 的度数;\n\n(2)若点 $E$ 运动到 $l_{1}$ 上方, 且满足 $\\angle B A E=100^{\\circ}, \\angle B A E: \\angle C A E=5: 1$, 求 $a$ 的值;\n\n(3)若 $\\angle B A E: \\angle C A E=n(n>1)$, 求 $\\angle C A E$ 的度数(用含 $n$ 和 $\\alpha$ 的代数式表示).\n\n", "options": [], "subject": "度量几何学", "analysis": "解【答案】解: (1) $\\because \\alpha=30^{\\circ}, A C / / B D$,\n\n$\\therefore \\angle C B D=30^{\\circ}$,\n\n$\\because B C$ 平分 $\\angle A B D$,\n$\\therefore \\angle A B E=\\angle C B D=30^{\\circ}$,\n\n$\\therefore \\angle B A C=180^{\\circ}-\\angle A B E-\\alpha=180^{\\circ}-30^{\\circ}-30^{\\circ}=120^{\\circ}$,\n\n又 $\\because \\angle B A E=\\angle C A E$,\n\n$\\therefore \\angle C A E=\\frac{1}{2} \\angle B A C=\\frac{1}{2} \\times 120^{\\circ}=60^{\\circ}$;\n\n(2)根据题意画图,如图1所示,\n\n$\\because \\angle B A E=100^{\\circ}, \\angle B A E: \\angle C A E=5: 1$,\n\n$\\therefore \\angle C A E=20^{\\circ}$,\n\n$\\therefore \\angle B A C=\\angle B A E-\\angle C A E=100^{\\circ}-20^{\\circ}=80^{\\circ}$,\n\n$\\because A C / / B D$,\n\n$\\therefore \\angle A B D=180^{\\circ}-\\angle B A C=100^{\\circ}$,\n\n又 $\\because B C$ 平分 $\\angle A B D$,\n\n$\\therefore \\angle C B D=\\frac{1}{2} \\angle A B D=\\frac{1}{2} \\times 100^{\\circ}=50^{\\circ}$,\n\n$\\therefore \\alpha=\\angle C B D=50^{\\circ}$;\n\n(3)11如图2所示,\n\n$\\because A C / / B D$\n\n$\\therefore \\angle C B D=\\angle A C B=\\alpha$,\n\n$\\because B C$ 平分 $\\angle A B D$,\n\n\n\n图 2\n\n$\\therefore \\angle A B D=2 \\angle C B D=2 \\alpha$,\n\n$\\therefore \\angle B A C=180^{\\circ}-\\angle A B D=180^{\\circ}-2 \\alpha$,\n\n又 $\\because \\angle B A E: \\angle C A E=n$,\n\n$\\therefore(\\angle B A C+\\angle C A E): \\angle C A E=n$,\n\n$\\left(180^{\\circ}-2 \\alpha+\\angle C A E\\right): \\angle C A E=n$,\n\n解得 $\\angle C A E=\\frac{180^{\\circ}-2 \\alpha}{n-1}$;\n\n(2)如图3所示,\n\n$\\because A C / / B D$,\n\n$\\therefore \\angle C B D=\\angle A C B=\\alpha$,\n\n$\\because B C$ 平分 $\\angle A B D$,\n\n\n\n图 3\n\n$\\therefore \\angle A B D=2 \\angle C B D=2 \\alpha$,\n\n$\\therefore \\angle B A C=180^{\\circ}-\\angle A B D=180^{\\circ}-2 \\alpha$,\n\n又 $\\because \\angle B A E: \\angle C A E=n$,\n$\\therefore(\\angle B A C-\\angle C A E): \\angle C A E=n$,\n\n$\\left(180^{\\circ}-2 \\alpha-\\angle C A E\\right): \\angle C A E=n$,\n\n解得 $\\angle C A E=\\frac{180^{\\circ}-2 \\alpha}{n+1}$.\n\n综上 $\\angle C A E$ 的度数为 $\\frac{180^{\\circ}-2 \\alpha}{n-1}$ 或 $\\frac{180^{\\circ}-2 \\alpha}{n+1}$.\n\n【解析】(1)根据平行线的性质可得 $\\angle C B D$ 的度数, 再根据角平分线的性质可得 $A B E$ 的度数, 应用三角形内角和计算 $\\angle B A C$ 的度数, 由已知条件 $\\angle B A E=\\angle C A E$, 可计算出 $\\angle C A E$ 的度数;\n\n(2)根据题意画出图形, 先根据 $\\angle B A E: \\angle C A E=5: 1$ 可计算出 $\\angle C A E$ 的度数, 由 $\\angle B A E=$ $100^{\\circ}$ 可计算出 $\\angle B A C$ 的度数, 再根据平行线的性质和角平分线的性质, 计算出 $\\angle C B D$ 的度数,即可得出结论;\n\n(3)根据题意可分两种情况,\n\n(1)若点 $E$ 运动到 $l_{1}$ 上方, 根据平行线的性质由 $\\alpha$ 可计算出 $\\angle C B D$ 的度数, 再根据角平分线的性质和平行线的性质, 计算出 $\\angle B A C$ 的度数, 再 $\\angle B A E: \\angle C A E=n, \\angle B A E=\\angle B A C+$ $\\angle C A E$, 列出等量关系求解即可等处结论;\n\n(2)若点 $E$ 运动到 $l_{1}$ 下方, 根据平行线的性质由 $\\alpha$ 可计算出 $\\angle C B D$ 的度数, 再根据角平分线的性质和平行线的性质, 计算出 $\\angle B A C$ 的度数, 再 $\\angle B A E: \\angle C A E=n, \\angle B A E=\\angle B A C-$ $\\angle C A E$ 列出等量关系求解即可等处结论.\n\n本题主要考查平行线的性质和角平分线的性质, 两直线平行, 同位角相等.两直线平行,同旁内角互补. 两直线平行,内错角相等. 合理应用平行线的性质是解决本题的关键."} {"id": "23461", "image": ["11599.jpg"], "answer": "【答案】证明: $\\because \\angle 1=\\angle C G D, \\angle 1$ 与 $\\angle 2$ 互补,\n\n$\\therefore \\angle C G D+\\angle 2=180^{\\circ}$,\n\n$\\therefore A F / / E D$,\n\n$\\therefore \\angle A+\\angle A E D=180^{\\circ}$,\n\n$\\because \\angle A=\\angle D$,\n$\\therefore \\angle D+\\angle A E D=180^{\\circ}$,\n\n$\\therefore A B / / C D$.\n\n【解析】由对顶角相等得到一对角相等,根据已知一对角互补,得到同旁内角互补,利用同旁内角互补两直线平行得到 $A F$ 与 $E D$ 平行, 由两直线平行同旁内角互补得到一对角互补,等量代换得到 $\\angle D$ 与 $\\angle A E D$ 互补,利用同旁内角互补两直线平行即可得证.\n\n此题考查了平行线的判定, 熟练掌握平行线的判定方法是解本题的关键.", "solution": "null", "level": "七年级", "question": "如图, 已知: $\\angle 1$ 与 $\\angle 2$ 互补, $\\angle A=\\angle D$, 求证: $A B / / C D$.\n\n", "options": [], "subject": "度量几何学", "analysis": "【答案】证明: $\\because \\angle 1=\\angle C G D, \\angle 1$ 与 $\\angle 2$ 互补,\n\n$\\therefore \\angle C G D+\\angle 2=180^{\\circ}$,\n\n$\\therefore A F / / E D$,\n\n$\\therefore \\angle A+\\angle A E D=180^{\\circ}$,\n\n$\\because \\angle A=\\angle D$,\n$\\therefore \\angle D+\\angle A E D=180^{\\circ}$,\n\n$\\therefore A B / / C D$.\n\n【解析】由对顶角相等得到一对角相等,根据已知一对角互补,得到同旁内角互补,利用同旁内角互补两直线平行得到 $A F$ 与 $E D$ 平行, 由两直线平行同旁内角互补得到一对角互补,等量代换得到 $\\angle D$ 与 $\\angle A E D$ 互补,利用同旁内角互补两直线平行即可得证.\n\n此题考查了平行线的判定, 熟练掌握平行线的判定方法是解本题的关键."} {"id": "23419", "image": ["11542.jpg"], "answer": "【答案】解: 设 $A E=x$, 根据题意列出方程: $10(6-x)=20$,\n\n解得 $x=4$,\n\n$\\because A$ 的对应点为 $E, \\therefore$ 平移距离为 $A E$ 的长,\n\n故向下平移 $4 \\mathrm{~cm}$.\n\n【解析】设线段 $A E=x$, 则 $E D=A D-A E=6-x$, 因为 $B C=6$, 所以矩形 $E F C D$ 的面积为 $E D \\cdot A B=20 \\mathrm{~cm}^{2}$, 就可以列出方程, 解方程即可.\n\n本题综合考查了平移的性质和一元一次方程的应用, 关键是扣住矩形 $E F C D$ 的面积为 $20 \\mathrm{~cm}^{2}$, 运用方程思想求解", "solution": "null", "level": "七年级", "question": "如图, 已知: 长方形 $A B C D$ 中, $A B=10 \\mathrm{~cm}, B C=6 \\mathrm{~cm}$, 试问将长方形 $A B C D$ 沿着 $B C$ 方向平移多少才能够使平移后的长方形与原来的长方形 $A B C D$ 重叠部分的面积为 $20 \\mathrm{~cm}^{2}$ ?\n\n", "options": [], "subject": "变换几何", "analysis": "【答案】解: 设 $A E=x$, 根据题意列出方程: $10(6-x)=20$,\n\n解得 $x=4$,\n\n$\\because A$ 的对应点为 $E, \\therefore$ 平移距离为 $A E$ 的长,\n\n故向下平移 $4 \\mathrm{~cm}$.\n\n【解析】设线段 $A E=x$, 则 $E D=A D-A E=6-x$, 因为 $B C=6$, 所以矩形 $E F C D$ 的面积为 $E D \\cdot A B=20 \\mathrm{~cm}^{2}$, 就可以列出方程, 解方程即可.\n\n本题综合考查了平移的性质和一元一次方程的应用, 关键是扣住矩形 $E F C D$ 的面积为 $20 \\mathrm{~cm}^{2}$, 运用方程思想求解"} {"id": "23420", "image": ["11543.jpg"], "answer": "【答案】解: $\\because$ 阴影部分的面积等于梯形 $A B C D$ 的面积减去梯形 $E F M D$ 的面积,等于梯形 $E F G H$ 的面积减去梯形 $E F D M$ 的面积,\n\n$\\therefore$ 阴影部分的面积等于梯形 $D H G M$ 的面积,\n\n$\\because H G=10, M C=2, M G=4$,\n\n$\\therefore S_{\\text {阴 }}=S_{D H G M}=\\frac{1}{2} \\times(8+10) \\times 4=36$.\n【解析】根据图形可知图中阴影部分的面积等于梯形 $A B C D$ 的面积减去梯形 $E F M D$ 的面积, 恰好等于梯形 $E F G H$ 的面积减去梯形 $E F D M$ 的面积.\n\n主要考查了梯形的性质和平移的性质. 要注意: 平移前后图形的形状和大小不变. 本题的关键是能得到: 图中阴影部分的面积等于梯形 $A B C D$ 的面积减去梯形 $E F M D$ 的面积,恰好等于梯形 $E F G H$ 的面积减去梯形 EFDM的面积.", "solution": "null", "level": "七年级", "question": "将直角梯形 $A B C D$ 平移得梯形 $E F G H$, 若 $H G=10, M C=2, M G=4$, 则图中阴影部分的面积为 . $\\qquad$\n\n", "options": [], "subject": "解析几何", "analysis": "【答案】解: $\\because$ 阴影部分的面积等于梯形 $A B C D$ 的面积减去梯形 $E F M D$ 的面积,等于梯形 $E F G H$ 的面积减去梯形 $E F D M$ 的面积,\n\n$\\therefore$ 阴影部分的面积等于梯形 $D H G M$ 的面积,\n\n$\\because H G=10, M C=2, M G=4$,\n\n$\\therefore S_{\\text {阴 }}=S_{D H G M}=\\frac{1}{2} \\times(8+10) \\times 4=36$.\n【解析】根据图形可知图中阴影部分的面积等于梯形 $A B C D$ 的面积减去梯形 $E F M D$ 的面积, 恰好等于梯形 $E F G H$ 的面积减去梯形 $E F D M$ 的面积.\n\n主要考查了梯形的性质和平移的性质. 要注意: 平移前后图形的形状和大小不变. 本题的关键是能得到: 图中阴影部分的面积等于梯形 $A B C D$ 的面积减去梯形 $E F M D$ 的面积,恰好等于梯形 $E F G H$ 的面积减去梯形 EFDM的面积."} {"id": "23422", "image": ["11546.jpg", "11547.jpg", "11548.jpg", "11549.jpg", "11550.jpg", "11551.jpg", "11549.jpg", "11550.jpg", "11551.jpg"], "answer": "【答案】解: (1)过点 $E$ 作 $E F / / A B$,\n\n$\\because \\angle A B E=30^{\\circ}$,\n\n$\\therefore \\angle B E F=\\angle A B E=30^{\\circ}$.\n\n\n\n图 1\n\n$\\because E F / / A B 、 A B / / C D$,\n\n$\\therefore E F / / C D$.\n\n$\\because \\angle C D E=35^{\\circ}$,\n\n$\\therefore \\angle D E F=\\angle C D E=35^{\\circ}$.\n\n$\\therefore \\angle B E D=\\angle B E F+\\angle D E F$\n$=65^{\\circ}$.\n\n(2)过点 $F$ 作 $E F / / A B$\n\n$\\because \\angle A B F=150^{\\circ}$,\n\n$\\therefore \\angle E F B=180^{\\circ}-\\angle A B F$\n\n$=180^{\\circ}-150^{\\circ}$\n\n\n\n图 2\n\n$=30^{\\circ}$.\n\n$\\because E F / / A B 、 A B / / C D$,\n\n$\\therefore E F / / C D$.\n\n$\\because \\angle C D F=130^{\\circ}$.\n\n$\\therefore \\angle E F D=180^{\\circ}-\\angle C D F$\n\n$=180^{\\circ}-130^{\\circ}$\n\n$=50^{\\circ}$.\n\n$\\therefore \\angle B F D=\\angle E F D-\\angle E F B$\n\n$=50^{\\circ}-30^{\\circ}$\n\n$=20^{\\circ}$.\n\n(3) $\\angle B G D=\\alpha-\\beta$.\n\n延长 $A B$ 交 $G D$ 于点 $E$.\n\n$\\because A B / / C D$,\n\n$\\therefore \\angle G E B=\\angle D=\\beta$.\n\n\n\n图 3\n\n在 $\\triangle B G E$ 中,\n\n$\\angle B G D=\\angle A B G-\\angle G E B$\n\n$=\\alpha-\\beta$.\n\n【解析】(1)过点 $E$ 作 $E F / / A B$, 通过平行线的判定和性质, 把 $\\angle B 、 \\angle D$ 搬至 $\\angle B E D$, 求出 $\\angle B E D$ 的度数;\n\n(2)过点 $F$ 作 $E F / / A B$, 通过平行线的判定和性质, 把 $\\angle B 、 \\angle D$ 与 $\\angle E F D 、 \\angle E F B$ 联系起来,利用角的和差关系求出 $\\angle B E D$ 的度数;\n\n(3)延长 $A B$ 交 $G D$ 于点 $E$, 利用三角形的外角和内角的关系, 易得结论.\n\n本题考查了平行线的性质和判定,掌握平行线的性质和判定,添加适当辅助线把分散的角集中起来是解决本题的关键.", "solution": "null", "level": "七年级", "question": "小华同学探究平行线的性质:\n\n(1)如图(1)在平面上画出两条平行线 $A B / / C D$, 在平行线之间取一点 $E$, 连接 $B E$ 和 $D E$, 已知 $\\angle A B E=30^{\\circ}, \\angle C D E=35^{\\circ}$, 求: $\\angle B E D$ 的度数.\n\n(2)如图(2)在平面上画出两条平行线 $A B / / C D$, 在平行线右上方取一点 $F$, 连接 $B F$ 和 $D F$, 已知 $\\angle A B F=150^{\\circ}, \\angle C D F=130^{\\circ}$, 求: $\\angle B F D$ 的度数.\n\n(3)如图(3)在平面上画出两条平行线 $A B / / C D$, 在平行线正上方取一点 $G$, 连接 $B G$ 和 $D G$, 已知 $\\angle A B G=\\alpha, \\angle C D G=\\beta(\\alpha>\\beta)$, 直接写出 $\\angle B G D$ 的度数(用含有 $\\alpha 、 \\beta$ 的式子表示).\n\n\n\n图 1\n\n\n\n图 2\n\n\n\n图 3", "options": [], "subject": "解析几何", "analysis": "【答案】解: (1)过点 $E$ 作 $E F / / A B$,\n\n$\\because \\angle A B E=30^{\\circ}$,\n\n$\\therefore \\angle B E F=\\angle A B E=30^{\\circ}$.\n\n\n\n图 1\n\n$\\because E F / / A B 、 A B / / C D$,\n\n$\\therefore E F / / C D$.\n\n$\\because \\angle C D E=35^{\\circ}$,\n\n$\\therefore \\angle D E F=\\angle C D E=35^{\\circ}$.\n\n$\\therefore \\angle B E D=\\angle B E F+\\angle D E F$\n$=65^{\\circ}$.\n\n(2)过点 $F$ 作 $E F / / A B$\n\n$\\because \\angle A B F=150^{\\circ}$,\n\n$\\therefore \\angle E F B=180^{\\circ}-\\angle A B F$\n\n$=180^{\\circ}-150^{\\circ}$\n\n\n\n图 2\n\n$=30^{\\circ}$.\n\n$\\because E F / / A B 、 A B / / C D$,\n\n$\\therefore E F / / C D$.\n\n$\\because \\angle C D F=130^{\\circ}$.\n\n$\\therefore \\angle E F D=180^{\\circ}-\\angle C D F$\n\n$=180^{\\circ}-130^{\\circ}$\n\n$=50^{\\circ}$.\n\n$\\therefore \\angle B F D=\\angle E F D-\\angle E F B$\n\n$=50^{\\circ}-30^{\\circ}$\n\n$=20^{\\circ}$.\n\n(3) $\\angle B G D=\\alpha-\\beta$.\n\n延长 $A B$ 交 $G D$ 于点 $E$.\n\n$\\because A B / / C D$,\n\n$\\therefore \\angle G E B=\\angle D=\\beta$.\n\n\n\n图 3\n\n在 $\\triangle B G E$ 中,\n\n$\\angle B G D=\\angle A B G-\\angle G E B$\n\n$=\\alpha-\\beta$.\n\n【解析】(1)过点 $E$ 作 $E F / / A B$, 通过平行线的判定和性质, 把 $\\angle B 、 \\angle D$ 搬至 $\\angle B E D$, 求出 $\\angle B E D$ 的度数;\n\n(2)过点 $F$ 作 $E F / / A B$, 通过平行线的判定和性质, 把 $\\angle B 、 \\angle D$ 与 $\\angle E F D 、 \\angle E F B$ 联系起来,利用角的和差关系求出 $\\angle B E D$ 的度数;\n\n(3)延长 $A B$ 交 $G D$ 于点 $E$, 利用三角形的外角和内角的关系, 易得结论.\n\n本题考查了平行线的性质和判定,掌握平行线的性质和判定,添加适当辅助线把分散的角集中起来是解决本题的关键."} {"id": "23484", "image": [], "answer": "【答案】解: 把方程 $x+2 y=2,2 x+y=7$ 组成二元一次方程组, 可得\n$$\n\\left\\{\\begin{array}{l}\nx+2 y=2(1) \\\\\n2 x+y=7(2)\n\\end{array}\\right.\n$$\n\n(2) $\\times 2-(1)$, 可得 $3 x=12$,\n\n解得 $x=4$,\n\n把 $x=4$ 代入(1), 可得 $4+2 y=2$,\n\n解得 $y=-1$,\n\n$\\therefore\\left\\{\\begin{array}{l}x+2 y=2(1) \\\\ 2 x+y=7(2)\\end{array}\\right.$ 的解是 $\\left\\{\\begin{array}{l}x=4 \\\\ y=-1\\end{array}\\right.$,\n\n$\\therefore 4-(-1)=2 k-1$,\n\n解得 $k=3$.\n\n【解析】首先把方程 $x+2 y=2,2 x+y=7$ 组成二元一次方程组, 应用加减消元法,求出 $x, y$ 的值, 然后把 $x, y$ 的值代入 $x-y=2 k-1$ 即可.\n\n此题主要考查了二元一次方程组的解, 要熟练掌握, 采用加减消元法即可.", "solution": "null", "level": "七年级", "question": "已知关于 $x, y$ 的方程 $x+2 y=2,2 x+y=7, x-y=2 k-1$ 有公共解, 求 $k$ 的值.", "options": [], "subject": "代数", "analysis": "【答案】解: 把方程 $x+2 y=2,2 x+y=7$ 组成二元一次方程组, 可得\n$$\n\\left\\{\\begin{array}{l}\nx+2 y=2(1) \\\\\n2 x+y=7(2)\n\\end{array}\\right.\n$$\n\n(2) $\\times 2-(1)$, 可得 $3 x=12$,\n\n解得 $x=4$,\n\n把 $x=4$ 代入(1), 可得 $4+2 y=2$,\n\n解得 $y=-1$,\n\n$\\therefore\\left\\{\\begin{array}{l}x+2 y=2(1) \\\\ 2 x+y=7(2)\\end{array}\\right.$ 的解是 $\\left\\{\\begin{array}{l}x=4 \\\\ y=-1\\end{array}\\right.$,\n\n$\\therefore 4-(-1)=2 k-1$,\n\n解得 $k=3$.\n\n【解析】首先把方程 $x+2 y=2,2 x+y=7$ 组成二元一次方程组, 应用加减消元法,求出 $x, y$ 的值, 然后把 $x, y$ 的值代入 $x-y=2 k-1$ 即可.\n\n此题主要考查了二元一次方程组的解, 要熟练掌握, 采用加减消元法即可."} {"id": "23485", "image": [], "answer": "【答案】解: 由题意可将 $x+y=5$ 与 $2 x-y=1$ 组成方程组 $\\left\\{\\begin{array}{l}x+y=5 \\\\ 2 x-y=1\\end{array}\\right.$,解得: $\\left\\{\\begin{array}{l}x=2 \\\\ y=3\\end{array}\\right.$,\n\n把 $\\left\\{\\begin{array}{l}x=2 \\\\ y=3\\end{array}\\right.$ 代入 $4 a x+5 b y=-22$, 得 $8 a+15 b=-22$ (1),\n\n把 $\\left\\{\\begin{array}{l}x=2 \\\\ y=3\\end{array}\\right.$ 代入 $a x-b y-8=0$, 得 $2 a-3 b-8=0$ (2),\n\n(1)与(2)组成方程组, 得 $\\left\\{\\begin{array}{l}8 a+15 b=-22 \\\\ 2 a-3 b-8=0\\end{array}\\right.$,\n解得: $\\left\\{\\begin{array}{l}a=1 \\\\ b=-2\\end{array}\\right.$.\n\n【解析】联立不含 $a$ 与 $b$ 的方程求出 $x$ 与 $y$ 的值, 代入剩下的方程求出 $a$ 与 $b$ 的值即可.此题考查了二元一次方程组的解, 方程组的解即为能使方程组中两方程都成立的未知数的值.", "solution": "null", "level": "七年级", "question": "已知关于 $x, y$ 的方程组 $\\left\\{\\begin{array}{l}x+y=5 \\\\ 4 a x+5 b y=-22\\end{array}\\right.$ 与 $\\left\\{\\begin{array}{l}2 x-y=1 \\\\ a x-b y-8=0\\end{array}\\right.$ 有相同的解, 求 $a, b$的值.", "options": [], "subject": "代数", "analysis": "【答案】解: 由题意可将 $x+y=5$ 与 $2 x-y=1$ 组成方程组 $\\left\\{\\begin{array}{l}x+y=5 \\\\ 2 x-y=1\\end{array}\\right.$,解得: $\\left\\{\\begin{array}{l}x=2 \\\\ y=3\\end{array}\\right.$,\n\n把 $\\left\\{\\begin{array}{l}x=2 \\\\ y=3\\end{array}\\right.$ 代入 $4 a x+5 b y=-22$, 得 $8 a+15 b=-22$ (1),\n\n把 $\\left\\{\\begin{array}{l}x=2 \\\\ y=3\\end{array}\\right.$ 代入 $a x-b y-8=0$, 得 $2 a-3 b-8=0$ (2),\n\n(1)与(2)组成方程组, 得 $\\left\\{\\begin{array}{l}8 a+15 b=-22 \\\\ 2 a-3 b-8=0\\end{array}\\right.$,\n解得: $\\left\\{\\begin{array}{l}a=1 \\\\ b=-2\\end{array}\\right.$.\n\n【解析】联立不含 $a$ 与 $b$ 的方程求出 $x$ 与 $y$ 的值, 代入剩下的方程求出 $a$ 与 $b$ 的值即可.此题考查了二元一次方程组的解, 方程组的解即为能使方程组中两方程都成立的未知数的值."} {"id": "23486", "image": [], "answer": "【答案】解: 将 $x=-2, y=1$ 代入方程组得: $\\left\\{\\begin{array}{l}4 a+3 b=4(1) \\\\ 2 a+b=-5(2)\\end{array}\\right.$,\n\n(2) $\\times 3$ - (1) 得: $2 a=-19$,\n\n解得: $a=-\\frac{19}{2}$,\n\n将 $a=-\\frac{19}{2}$ 代入(2)得: $-19+b=-5$,\n\n解得: $b=14$,\n\n则 $4 a^{2}-4 a b-3 b^{2}=(2 a+b)(2 a-3 b)=(-19+14) \\times(-19-42)=305$.\n\n【解析】将 $x=-2, y=1$ 代入已知方程组中, 得到关于 $a$ 与 $b$ 的二元一次方程组, 求出方程组的解即可得到 $a$ 与 $b$ 的值.\n\n此题考查了二元一次方程组的解, 方程组的解即为能使方程组中两方程左右两边相等的未知数的值.", "solution": "null", "level": "七年级", "question": "如果关于 $x 、 y$ 的方程组 $\\left\\{\\begin{array}{l}2 a x-3 b y=-4 \\\\ a x-b y=5\\end{array}\\right.$ 的解是 $\\left\\{\\begin{array}{l}x=-2 \\\\ y=1\\end{array}\\right.$, 求 $4 a^{2}-4 a b-3 b^{2}$ 的值.", "options": [], "subject": "代数", "analysis": "【答案】解: 将 $x=-2, y=1$ 代入方程组得: $\\left\\{\\begin{array}{l}4 a+3 b=4(1) \\\\ 2 a+b=-5(2)\\end{array}\\right.$,\n\n(2) $\\times 3$ - (1) 得: $2 a=-19$,\n\n解得: $a=-\\frac{19}{2}$,\n\n将 $a=-\\frac{19}{2}$ 代入(2)得: $-19+b=-5$,\n\n解得: $b=14$,\n\n则 $4 a^{2}-4 a b-3 b^{2}=(2 a+b)(2 a-3 b)=(-19+14) \\times(-19-42)=305$.\n\n【解析】将 $x=-2, y=1$ 代入已知方程组中, 得到关于 $a$ 与 $b$ 的二元一次方程组, 求出方程组的解即可得到 $a$ 与 $b$ 的值.\n\n此题考查了二元一次方程组的解, 方程组的解即为能使方程组中两方程左右两边相等的未知数的值."} {"id": "23488", "image": [], "answer": "【答案】解: 由 (1), 得 $2 x-3 y=2$. (3)\n\n把 (3)代入 (2), 得 $\\frac{2+5}{7}+2 y=9$.\n\n解得 $y=4$.\n\n把 $y=4$ 代入 (3), 得 $2 x-3 \\times 4=2$.\n\n解得 $x=7$.\n\n$\\therefore$ 原方程组的解为 $\\left\\{\\begin{array}{l}x=7, \\\\ y=4 .\\end{array}\\right.$\n\n【解析】见答案", "solution": "null", "level": "七年级", "question": " 【注重阅读理解】\n\n先阅读材料, 然后解方程组.\n\n材料: 解方程组: $\\left\\{\\begin{array}{l}x-y-1=0, \\text { (1) } \\\\ 4(x-y)-y=5 . \\text { (2) }\\end{array}\\right.$\n\n由 (1), 得 $x-y=1$. (3)\n\n把 (3)代入 (2), 得 $4 \\times 1-y=5$, 解得 $y=-1$.\n\n把 $y=-1$ 代入 (3), 得 $x=0$.\n\n$\\therefore$ 原方程组的解为 $\\left\\{\\begin{array}{l}x=0, \\\\ y=-1 .\\end{array}\\right.$\n\n这种方法称为 “整体代入法” . 你若留心观察, 有很多方程组可采用此方法解答,\n\n请用这种方法解方程组: $\\left\\{\\begin{array}{l}2 x-3 y-2=0 \\text {, (1) } \\\\ \\frac{2 x-3 y+5}{7}+2 y=9 \\text {. (2) }\\end{array}\\right.$", "options": [], "subject": "代数", "analysis": "【答案】解: 由 (1), 得 $2 x-3 y=2$. (3)\n\n把 (3)代入 (2), 得 $\\frac{2+5}{7}+2 y=9$.\n\n解得 $y=4$.\n\n把 $y=4$ 代入 (3), 得 $2 x-3 \\times 4=2$.\n\n解得 $x=7$.\n\n$\\therefore$ 原方程组的解为 $\\left\\{\\begin{array}{l}x=7, \\\\ y=4 .\\end{array}\\right.$\n\n【解析】见答案"} {"id": "23489", "image": [], "answer": "【答案】解: (1) $\\left\\{\\begin{array}{l}x-y=1 \\text { (1) } \\\\ 2 x+y=5 \\text { (2) }\\end{array}\\right.$\n\n(1) + (2)得: $3 x=6$,\n$x=2$,\n\n代入(1)得: $y=1$.\n\n故原方程组的解为 $\\left\\{\\begin{array}{l}x=2 \\\\ y=1\\end{array}\\right.$;\n\n(2)原方程组可化为 $\\left\\{\\begin{array}{l}2 x-5 y+17=0 \\\\ 3 x+4 y=32 \\text { (2) }\\end{array}\\right.$,\n\n(1) $\\times 3-(2 \\times 2$ 得: $-23 y=-115$,\n\n$y=5$,\n\n代入(1)得 $2 x=8$,\n\n$x=4$.\n\n故原方程组的解为 $\\left\\{\\begin{array}{l}x=4 \\\\ y=5\\end{array}\\right.$.\n\n【解析】根据各方程组的特点选用相应的方法: (1)用加减消元法(2)先去分母, 再用加减消元法.\n\n这类题目的解题关键是掌握方程组解法中的加减消元法.\n\n根据未知数系数的特点, 选择合适的方法.", "solution": "null", "level": "七年级", "question": "解方程组:\n(1) $\\left\\{\\begin{array}{l}x-y=1 \\\\ 2 x+y=5\\end{array}\\right.$\n(2) $\\left\\{\\begin{array}{l}\\frac{x+1}{5}=\\frac{y-3}{2} \\\\ 3 x+4 y=32\\end{array}\\right.$.", "options": [], "subject": "代数", "analysis": "【答案】解: (1) $\\left\\{\\begin{array}{l}x-y=1 \\text { (1) } \\\\ 2 x+y=5 \\text { (2) }\\end{array}\\right.$\n\n(1) + (2)得: $3 x=6$,\n$x=2$,\n\n代入(1)得: $y=1$.\n\n故原方程组的解为 $\\left\\{\\begin{array}{l}x=2 \\\\ y=1\\end{array}\\right.$;\n\n(2)原方程组可化为 $\\left\\{\\begin{array}{l}2 x-5 y+17=0 \\\\ 3 x+4 y=32 \\text { (2) }\\end{array}\\right.$,\n\n(1) $\\times 3-(2 \\times 2$ 得: $-23 y=-115$,\n\n$y=5$,\n\n代入(1)得 $2 x=8$,\n\n$x=4$.\n\n故原方程组的解为 $\\left\\{\\begin{array}{l}x=4 \\\\ y=5\\end{array}\\right.$.\n\n【解析】根据各方程组的特点选用相应的方法: (1)用加减消元法(2)先去分母, 再用加减消元法.\n\n这类题目的解题关键是掌握方程组解法中的加减消元法.\n\n根据未知数系数的特点, 选择合适的方法."} {"id": "23507", "image": [], "answer": "【答案】解:(1)去分母得: $3(2 x+1)-12=12 x-(10 x+1)$\n\n去括号得: $6 x+3-12=12 x-10 x-1$,\n\n移项得, $6 x-12 x+10 x=-1-3+12$\n\n合并同类项得: $4 x=8$,\n\n解得: $x=2$;\n\n(2): 方程组整理得 $\\left\\{\\begin{array}{l}x-2 y=-1 \\\\ 2 x-y=6\\end{array}\\right.$ (2)\n\n(1) $\\times 2$ 得: $2 x-4 y=-2(3)$,\n\n(2) - (3) 得: $3 y=8$, 即 $y=\\frac{8}{3}$,\n\n将 $y=\\frac{8}{3}$ 代入(1)得: $x=\\frac{13}{3}$,\n\n则原方程组的解为 $\\left\\{\\begin{array}{l}x=\\frac{13}{3} \\\\ y=\\frac{8}{3}\\end{array}\\right.$.\n【解析】本题主要考查了解一元一次方程和解二元一次方程组, 解一元一次方程的一般步骤为: 去分母, 去括号, 移项, 合并同类项, 化系数为 1 ; 解二元一次方程组的关键是消元.\n\n(1)去分母, 去括号, 移项, 合并同类项, 最后化系数为1即可求出方程的解;\n\n(2)先整理方程组, 然后再用加减消元法解之即可.", "solution": "null", "level": "七年级", "question": "解方程:\n\n(1) $\\frac{2 x+1}{4}-1=x-\\frac{10 x+1}{12}$\n\n(2) $\\left\\{\\begin{array}{c}x+1=2 y \\\\ 2(x+1)-y=8\\end{array}\\right.$.", "options": [], "subject": "代数", "analysis": "【答案】解:(1)去分母得: $3(2 x+1)-12=12 x-(10 x+1)$\n\n去括号得: $6 x+3-12=12 x-10 x-1$,\n\n移项得, $6 x-12 x+10 x=-1-3+12$\n\n合并同类项得: $4 x=8$,\n\n解得: $x=2$;\n\n(2): 方程组整理得 $\\left\\{\\begin{array}{l}x-2 y=-1 \\\\ 2 x-y=6\\end{array}\\right.$ (2)\n\n(1) $\\times 2$ 得: $2 x-4 y=-2(3)$,\n\n(2) - (3) 得: $3 y=8$, 即 $y=\\frac{8}{3}$,\n\n将 $y=\\frac{8}{3}$ 代入(1)得: $x=\\frac{13}{3}$,\n\n则原方程组的解为 $\\left\\{\\begin{array}{l}x=\\frac{13}{3} \\\\ y=\\frac{8}{3}\\end{array}\\right.$.\n【解析】本题主要考查了解一元一次方程和解二元一次方程组, 解一元一次方程的一般步骤为: 去分母, 去括号, 移项, 合并同类项, 化系数为 1 ; 解二元一次方程组的关键是消元.\n\n(1)去分母, 去括号, 移项, 合并同类项, 最后化系数为1即可求出方程的解;\n\n(2)先整理方程组, 然后再用加减消元法解之即可."} {"id": "23508", "image": [], "answer": "【答案】解:根据题意可联立 $\\left\\{\\begin{array}{l}3 x-y=5(1) \\\\ 2 x+3 y=-4(2)\\end{array}\\right.$,\n\n(1) $\\times 3+$ (2), 得 $11 x=11$,\n\n解得 $x=1$.\n\n把 $x=1$ 代入(1), 得 $y=-2$.\n\n$\\therefore$ 方程组的解为 $\\left\\{\\begin{array}{l}x=1 \\\\ y=-2\\end{array}\\right.$;\n\n根据题意, 得 $\\left\\{\\begin{array}{l}4 a-10 b=-22 \\\\ a+2 b=8\\end{array}\\right.$ 即 $\\left\\{\\begin{array}{l}2 a-5 b=-11(1) \\\\ a+2 b=8(2)\\end{array}\\right.$,\n\n(2) $\\times 2$ - (1), 得 $9 b=27$,\n\n解得 $b=3$.\n\n把 $b=3$ 代入(2), 得 $a=2$.\n\n则 $a+b=3+2=5$.\n\n【解析】根据题意可联立已知方程组, 求出 $x$ 和 $y$ 的值, 再将 $x$ 和 $y$ 的值代入含 $a$ 和 $b$ 的方程中组成关于 $a$ 和 $b$ 的方程组, 求出 $a$ 和 $b$ 的值, 进而得解.\n\n本题考查了二元一次方程组的解, 解决本题的关键是掌握二元一次方程组的解法.", "solution": "null", "level": "七年级", "question": "关于 $x, y$ 的方程组 $\\left\\{\\begin{array}{l}3 x-y=5 \\\\ 4 a x+5 b y=-22\\end{array}\\right.$ 与 $\\left\\{\\begin{array}{l}2 x+3 y=-4 \\\\ a x-b y=8\\end{array}\\right.$ 有相同的解, 求 $a+b$ 的值.", "options": [], "subject": "代数", "analysis": "【答案】解:根据题意可联立 $\\left\\{\\begin{array}{l}3 x-y=5(1) \\\\ 2 x+3 y=-4(2)\\end{array}\\right.$,\n\n(1) $\\times 3+$ (2), 得 $11 x=11$,\n\n解得 $x=1$.\n\n把 $x=1$ 代入(1), 得 $y=-2$.\n\n$\\therefore$ 方程组的解为 $\\left\\{\\begin{array}{l}x=1 \\\\ y=-2\\end{array}\\right.$;\n\n根据题意, 得 $\\left\\{\\begin{array}{l}4 a-10 b=-22 \\\\ a+2 b=8\\end{array}\\right.$ 即 $\\left\\{\\begin{array}{l}2 a-5 b=-11(1) \\\\ a+2 b=8(2)\\end{array}\\right.$,\n\n(2) $\\times 2$ - (1), 得 $9 b=27$,\n\n解得 $b=3$.\n\n把 $b=3$ 代入(2), 得 $a=2$.\n\n则 $a+b=3+2=5$.\n\n【解析】根据题意可联立已知方程组, 求出 $x$ 和 $y$ 的值, 再将 $x$ 和 $y$ 的值代入含 $a$ 和 $b$ 的方程中组成关于 $a$ 和 $b$ 的方程组, 求出 $a$ 和 $b$ 的值, 进而得解.\n\n本题考查了二元一次方程组的解, 解决本题的关键是掌握二元一次方程组的解法."} {"id": "23509", "image": [], "answer": "【答案】 $1-2 y \\frac{1-m}{4} \\frac{m+1}{2}\\left\\{\\begin{array}{l}x=\\frac{m+1}{2} \\\\ y=\\frac{1-m}{4}\\end{array}\\right.$\n\n【解析】解: (1)若用代入法求解, 可由(1)得: $x=1-2 y$ (3),\n\n把(3)代入(2)解得: $y=\\frac{1-m}{4}$,\n\n将其代入(3)解得: $x=\\frac{m+1}{2}$,\n\n$\\therefore$ 原方程组的解为 $\\left\\{\\begin{array}{l}x=\\frac{m+1}{2} \\\\ y=\\frac{1-m}{4}\\end{array}\\right.$,\n故答案为: $1-2 y ; \\frac{1-m}{4} ; \\frac{m+1}{2} ;\\left\\{\\begin{array}{l}x=\\frac{m+1}{2} \\\\ y=\\frac{1-m}{4} ;\\end{array}\\right.$\n\n(2) $\\because$ 方程组的解 $x 、 y$ 互为相反数,\n\n$\\therefore x=-y 3$,\n\n(3)代入(1)得, $-y+2 y=1$,\n\n$\\therefore y=1$,\n\n$x=-1$,\n\n$m=-1-2=-3$,\n\n$\\therefore$ 方程组的解是 $\\left\\{\\begin{array}{l}x=-1 \\\\ y=1\\end{array}\\right.$,\n\n$m=-3$.\n\n(1)根据代入消元法的求解方法解答即可;\n\n(2)根据方程组的解互为相反数可得 $x=-y$, 代入方程(1)求出 $y$, 再代入方程(2)求出 $m$ 即可.\n\n本题考查了代入消元法解二元一次方程组, 熟练掌握代入法的操作方法是解题的关键.", "solution": "null", "level": "七年级", "question": "已知关于 $x 、 y$ 的方程组 $\\left\\{\\begin{array}{l}x+2 y=1(1) \\\\ x-2 y=m(2)\\end{array}\\right.$,\n\n(1)若用代入法求解,可由(1)得: $x=$ $\\qquad$ (3)\n\n把(3)代入(2)解得: $y=$ $\\qquad$\n将其代入(3)解得: $x=$ $\\qquad$\n$\\therefore$ 原方程组的解为 $\\qquad$\n(2)若此方程组的解 $x 、 y$ 互为相反数, 求这个方程组的解及 $m$ 的值.", "options": [], "subject": "代数", "analysis": "【答案】 $1-2 y \\frac{1-m}{4} \\frac{m+1}{2}\\left\\{\\begin{array}{l}x=\\frac{m+1}{2} \\\\ y=\\frac{1-m}{4}\\end{array}\\right.$\n\n【解析】解: (1)若用代入法求解, 可由(1)得: $x=1-2 y$ (3),\n\n把(3)代入(2)解得: $y=\\frac{1-m}{4}$,\n\n将其代入(3)解得: $x=\\frac{m+1}{2}$,\n\n$\\therefore$ 原方程组的解为 $\\left\\{\\begin{array}{l}x=\\frac{m+1}{2} \\\\ y=\\frac{1-m}{4}\\end{array}\\right.$,\n故答案为: $1-2 y ; \\frac{1-m}{4} ; \\frac{m+1}{2} ;\\left\\{\\begin{array}{l}x=\\frac{m+1}{2} \\\\ y=\\frac{1-m}{4} ;\\end{array}\\right.$\n\n(2) $\\because$ 方程组的解 $x 、 y$ 互为相反数,\n\n$\\therefore x=-y 3$,\n\n(3)代入(1)得, $-y+2 y=1$,\n\n$\\therefore y=1$,\n\n$x=-1$,\n\n$m=-1-2=-3$,\n\n$\\therefore$ 方程组的解是 $\\left\\{\\begin{array}{l}x=-1 \\\\ y=1\\end{array}\\right.$,\n\n$m=-3$.\n\n(1)根据代入消元法的求解方法解答即可;\n\n(2)根据方程组的解互为相反数可得 $x=-y$, 代入方程(1)求出 $y$, 再代入方程(2)求出 $m$ 即可.\n\n本题考查了代入消元法解二元一次方程组, 熟练掌握代入法的操作方法是解题的关键."} {"id": "23511", "image": [], "answer": "【答案】解:设甲种货车每辆运 $x$ 吨, 乙种货车每辆运 $y$ 吨,\n\n$\\left\\{\\begin{array}{l}2 x+3 y=15.5 \\\\ 5 x+6 y=35\\end{array}\\right.$, 得 $\\left\\{\\begin{array}{l}x=4 \\\\ y=2.5\\end{array}\\right.$,\n\n$\\therefore 3 x+5 y=3 \\times 4+5 \\times 2.5=24.5$,\n\n$30 \\times 24.5=735($ 元 $)$,\n\n答: 如果每吨货物的运费为 30 元, 这批货物应该付运费 735 元.\n\n【解析】根据题意和表格可以得到相应的方程组, 从而可以求得甲乙货车每辆可以运输多少吨货物, 从而可以解答本题.\n\n本题考查二元一次方程组的应用, 解答本题的关键是明确题意, 列出相应的方程组.", "solution": "null", "level": "七年级", "question": "某公司要把一批货物运往 $A$ 地, 准备租用汽车运输公司的甲乙两种货车.过去曾两次租用这两种货车的情况如表:\n\n| | 第一次 | 第二次 |\n| :---: | :---: | :---: |\n| 租用甲种货车(辆) | 2 | 5 |\n| 租用乙种货车(辆) | 3 | 6 |\n| 合计运货吨数(吨) | 15.5 | 35 |\n\n现租用该公司甲种货车3辆, 乙种货车5辆, 正好运完这批货, 如果每吨货物的运费为 30 元,这批货物应该付运费多少元?", "options": [], "subject": "代数", "analysis": "【答案】解:设甲种货车每辆运 $x$ 吨, 乙种货车每辆运 $y$ 吨,\n\n$\\left\\{\\begin{array}{l}2 x+3 y=15.5 \\\\ 5 x+6 y=35\\end{array}\\right.$, 得 $\\left\\{\\begin{array}{l}x=4 \\\\ y=2.5\\end{array}\\right.$,\n\n$\\therefore 3 x+5 y=3 \\times 4+5 \\times 2.5=24.5$,\n\n$30 \\times 24.5=735($ 元 $)$,\n\n答: 如果每吨货物的运费为 30 元, 这批货物应该付运费 735 元.\n\n【解析】根据题意和表格可以得到相应的方程组, 从而可以求得甲乙货车每辆可以运输多少吨货物, 从而可以解答本题.\n\n本题考查二元一次方程组的应用, 解答本题的关键是明确题意, 列出相应的方程组."} {"id": "23513", "image": [], "answer": "【答案】解:(1)设 1 辆 $A$ 型车和 1 辆 $B$ 型车都载满货物一次分别可以运货 $x$ 吨, $y$ 吨,根据题意得: $\\left\\{\\begin{array}{l}3 x+2 y=19 \\\\ 2 x+3 y=21\\end{array}\\right.$,\n\n解得: $\\left\\{\\begin{array}{l}x=3 \\\\ y=5\\end{array}\\right.$.\n答: 1 辆 $A$ 型车一次可以运货 3 吨, 1 辆 $B$ 型车一次可以运货 5 吨.\n\n(2)(1)由(1)和题意得: $3 m+5 n=49$,\n\n$\\therefore m=\\frac{49-5 n}{3}$,\n\n$\\because m 、 n$ 都是正整数,\n\n$\\therefore\\left\\{\\begin{array}{l}m=13 \\\\ n=2\\end{array}\\right.$ 或 $\\left\\{\\begin{array}{l}m=8 \\\\ n=5\\end{array}\\right.$ 或 $\\left\\{\\begin{array}{l}m=3 \\\\ n=8\\end{array}\\right.$.\n\n(2) $\\because A$ 型车每辆需租金 150 元 $/$ 次, $B$ 型车每辆需租金 200 元 $/$ 次, $\\therefore$ 当 $m=13, n=2$ 时, 需租金: $130 \\times 13+200 \\times 2=2090$ (元),\n\n当 $m=8, n=5$ 时, 需租金: $130 \\times 8+200 \\times 5=2040$ (元),\n\n当 $m=3, n=8$ 时, 需租金: $130 \\times 3+200 \\times 8=1990$ (元),\n\n$\\because 2090>2040>1990$,\n\n所以租车费用最少的是 1990 元.\n\n【解析】(1)根据3辆 $A$ 型车和 2 辆 $B$ 型车载满货物一次可运货 $=10$ 吨; 2 辆 $A$ 型车和 3 辆 $B$ 型车载满货物一次可运货 $=11$ 吨, 列出方程组即可解决问题.\n\n(2)(1)由题意得到 $3 m+5 n=49$, 根据 $m 、 n$ 均为正整数, 即可求出 $m 、 n$ 的值.\n\n(2)求出每种方案下的租金数, 经比较、分析, 即可解决问题.\n\n主要考查了列二元一次方程组或二元一次方程来解决现实生活中的实际应用问题; 解题的关键是深入把握题意, 准确找出命题中隐含的数量关系, 正确列出方程或方程组来分析、推理、解答.", "solution": "null", "level": "七年级", "question": "已知: 用 3 辆 $A$ 型车和 2 辆 $B$ 型车载满货物一次可运货共 19 吨; 用 2 辆 $A$ 型车和 3 辆 $B$ 型车载满货物一次可运货共 21 吨.\n\n(1)1辆 $A$ 型车和 1 辆 $B$ 型车都载满货物一次分别可以运货多少吨?\n\n(2)某物流公司现有49吨货物, 计划同时租用 $A$ 型车 $m$ 辆, $B$ 型车 $n$ 辆, 一次运完,且恰好每辆车都载满货物.\n\n(1)求 $m 、 n$ 的值;\n\n(2)若 $A$ 型车每辆需租金 130 元/次, $B$ 型车每辆需租金 200 元/次. 请求出租车费用最少是多少元?", "options": [], "subject": "代数", "analysis": "【答案】解:(1)设 1 辆 $A$ 型车和 1 辆 $B$ 型车都载满货物一次分别可以运货 $x$ 吨, $y$ 吨,根据题意得: $\\left\\{\\begin{array}{l}3 x+2 y=19 \\\\ 2 x+3 y=21\\end{array}\\right.$,\n\n解得: $\\left\\{\\begin{array}{l}x=3 \\\\ y=5\\end{array}\\right.$.\n答: 1 辆 $A$ 型车一次可以运货 3 吨, 1 辆 $B$ 型车一次可以运货 5 吨.\n\n(2)(1)由(1)和题意得: $3 m+5 n=49$,\n\n$\\therefore m=\\frac{49-5 n}{3}$,\n\n$\\because m 、 n$ 都是正整数,\n\n$\\therefore\\left\\{\\begin{array}{l}m=13 \\\\ n=2\\end{array}\\right.$ 或 $\\left\\{\\begin{array}{l}m=8 \\\\ n=5\\end{array}\\right.$ 或 $\\left\\{\\begin{array}{l}m=3 \\\\ n=8\\end{array}\\right.$.\n\n(2) $\\because A$ 型车每辆需租金 150 元 $/$ 次, $B$ 型车每辆需租金 200 元 $/$ 次, $\\therefore$ 当 $m=13, n=2$ 时, 需租金: $130 \\times 13+200 \\times 2=2090$ (元),\n\n当 $m=8, n=5$ 时, 需租金: $130 \\times 8+200 \\times 5=2040$ (元),\n\n当 $m=3, n=8$ 时, 需租金: $130 \\times 3+200 \\times 8=1990$ (元),\n\n$\\because 2090>2040>1990$,\n\n所以租车费用最少的是 1990 元.\n\n【解析】(1)根据3辆 $A$ 型车和 2 辆 $B$ 型车载满货物一次可运货 $=10$ 吨; 2 辆 $A$ 型车和 3 辆 $B$ 型车载满货物一次可运货 $=11$ 吨, 列出方程组即可解决问题.\n\n(2)(1)由题意得到 $3 m+5 n=49$, 根据 $m 、 n$ 均为正整数, 即可求出 $m 、 n$ 的值.\n\n(2)求出每种方案下的租金数, 经比较、分析, 即可解决问题.\n\n主要考查了列二元一次方程组或二元一次方程来解决现实生活中的实际应用问题; 解题的关键是深入把握题意, 准确找出命题中隐含的数量关系, 正确列出方程或方程组来分析、推理、解答."} {"id": "23514", "image": [], "answer": "【答案】解: 由题意可将 $x+y=5$ 与 $2 x-y=1$ 组成方程组 $\\left\\{\\begin{array}{l}x+y=5 \\\\ 2 x-y=1\\end{array}\\right.$,\n\n解得: $\\left\\{\\begin{array}{l}x=2 \\\\ y=3\\end{array}\\right.$,\n\n把 $\\left\\{\\begin{array}{l}x=2 \\\\ y=3\\end{array}\\right.$ 代入 $4 a x+5 b y=-22$, 得 $8 a+15 b=-22(1$,\n\n把 $\\left\\{\\begin{array}{l}x=2 \\\\ y=3\\end{array}\\right.$ 代入 $a x-b y-8=0$, 得 $2 a-3 b-8=0$ (2),\n\n(1)与(2)组成方程组, 得 $\\left\\{\\begin{array}{l}8 a+15 b=-22 \\\\ 2 a-3 b-8=0\\end{array}\\right.$,\n\n解得: $\\left\\{\\begin{array}{l}a=1 \\\\ b=-2\\end{array}\\right.$.\n【解析】联立不含 $a$ 与 $b$ 的方程求出 $x$ 与 $y$ 的值, 代入剩下的方程求出 $a$ 与 $b$ 的值即可.\n\n此题考查了二元一次方程组的解, 方程组的解即为能使方程组中两方程都成立的未知数的值.", "solution": "null", "level": "七年级", "question": "已知关于 $x, y$ 的方程组 $\\left\\{\\begin{array}{l}x+y=5 \\\\ 4 a x+5 b y=-22\\end{array}\\right.$ 与 $\\left\\{\\begin{array}{l}2 x-y=1 \\\\ a x-b y-8=0\\end{array}\\right.$ 有相同的解, 求 $a, b$的值.", "options": [], "subject": "代数", "analysis": "【答案】解: 由题意可将 $x+y=5$ 与 $2 x-y=1$ 组成方程组 $\\left\\{\\begin{array}{l}x+y=5 \\\\ 2 x-y=1\\end{array}\\right.$,解得: $\\left\\{\\begin{array}{l}x=2 \\\\ y=3\\end{array}\\right.$,\n\n把 $\\left\\{\\begin{array}{l}x=2 \\\\ y=3\\end{array}\\right.$ 代入 $4 a x+5 b y=-22$, 得 $8 a+15 b=-22$ (1),\n\n把 $\\left\\{\\begin{array}{l}x=2 \\\\ y=3\\end{array}\\right.$ 代入 $a x-b y-8=0$, 得 $2 a-3 b-8=0$ (2),\n\n(1)与(2)组成方程组, 得 $\\left\\{\\begin{array}{l}8 a+15 b=-22 \\\\ 2 a-3 b-8=0\\end{array}\\right.$,\n解得: $\\left\\{\\begin{array}{l}a=1 \\\\ b=-2\\end{array}\\right.$.\n\n【解析】联立不含 $a$ 与 $b$ 的方程求出 $x$ 与 $y$ 的值, 代入剩下的方程求出 $a$ 与 $b$ 的值即可.此题考查了二元一次方程组的解, 方程组的解即为能使方程组中两方程都成立的未知数的值."} {"id": "23530", "image": [], "answer": "【答案】解: 整理得: $\\begin{cases}x+4 y=14 & 1 \\\\ 3 x-4 y=-2 & \\text { (2) }\\end{cases}$\n\n(1) + (2)得: $4 x=12$,\n\n$\\therefore x=3$,\n\n把 $x=3$ 代入(1)得: $3+4 y=14$,\n\n$\\therefore y=\\frac{11}{4}$.\n\n$\\therefore$ 此方程组的解是 $\\left\\{\\begin{array}{l}x=3 \\\\ y=\\frac{11}{4}\\end{array}\\right.$.\n\n【解析】本题考查了二元一次方程组的解法, 二元一次方程组解法有代入消元法和加减消元法两种. 利用消元的思想 “化二元为一元” 是解题关键. 解题时, 根据方程组的特征本题选择加减消元法, 先把原方程组整理后, (1)+ (2)可以求出 $x$ 的值, 再把 $x$ 的值代入(1)求出 $y$ 的值, 即可求出方程组的解.", "solution": "null", "level": "七年级", "question": "解方程组: $\\left\\{\\begin{array}{l}x+4 y=14 \\\\ \\frac{x-3}{4}-\\frac{y-3}{3}=\\frac{1}{12}\\end{array}\\right.$.", "options": [], "subject": "代数", "analysis": "【答案】解: 整理得: $\\begin{cases}x+4 y=14 & 1 \\\\ 3 x-4 y=-2 & \\text { (2) }\\end{cases}$\n\n(1) + (2)得: $4 x=12$,\n\n$\\therefore x=3$,\n\n把 $x=3$ 代入(1)得: $3+4 y=14$,\n\n$\\therefore y=\\frac{11}{4}$.\n\n$\\therefore$ 此方程组的解是 $\\left\\{\\begin{array}{l}x=3 \\\\ y=\\frac{11}{4}\\end{array}\\right.$.\n\n【解析】本题考查了二元一次方程组的解法, 二元一次方程组解法有代入消元法和加减消元法两种. 利用消元的思想 “化二元为一元” 是解题关键. 解题时, 根据方程组的特征本题选择加减消元法, 先把原方程组整理后, (1)+ (2)可以求出 $x$ 的值, 再把 $x$ 的值代入(1)求出 $y$ 的值, 即可求出方程组的解."} {"id": "23531", "image": [], "answer": "【答案】解: (1) $A=\\frac{1}{2} x-2 x+\\frac{4}{3} y-\\frac{1}{2} x+\\frac{2}{3} y$\n\n$=-2 x+2 y$,\n\n当 $x=-2, y=1$ 时,\n\n原式 $=-2 \\times(-2)+2 \\times 1$\n\n$=4+2$\n\n$=6$,\n\n即 $A$ 的值为 6 ;\n\n(2)由题意可得 $-2 x+2 y=6$,\n\n则当 $\\left\\{\\begin{array}{l}x=1 \\\\ y=4\\end{array}\\right.$ 时, $-2 x+2 y=6$ 也成立,\n\n$\\therefore$ 若使求得的 $A$ 的值与 (1)中的结果相同, 则给出的 $x, y$ 的值还能够是 $x=1, y=4$ (答案不唯一).\n【解析】(1)原式去括号, 合并同类项进行化简, 然后代入求值;\n\n(2)根据二元一次方程的解的概念分析求值.\n\n本题考查整式的加减一化简求值, 二元一次方程的解, 理解方程的解的概念, 掌握合并同类项(系数相加,字母及其指数不变)和去括号的运算法则(括号前面是 “ + ” 号, 去掉 “+” 号和括号, 括号里的各项不变号; 括号前面是 “-” 号, 去掉 “一” 号和括号,括号里的各项都变号)是解题关键.", "solution": "null", "level": "七年级", "question": "设 $A=\\frac{1}{2} x-2\\left(x-\\frac{2}{3} y\\right)+\\left(-\\frac{1}{2} x+\\frac{2}{3} y\\right)$.\n\n(1)当 $x=-2, y=1$ 时, 求 $A$ 的值;\n\n(2)若使求得的 $A$ 的值与(1)中的结果相同,则给出的 $x, y$ 的值还能够是什么?", "options": [], "subject": "代数", "analysis": "【答案】解: (1) $A=\\frac{1}{2} x-2 x+\\frac{4}{3} y-\\frac{1}{2} x+\\frac{2}{3} y$\n\n$=-2 x+2 y$,\n\n当 $x=-2, y=1$ 时,\n\n原式 $=-2 \\times(-2)+2 \\times 1$\n\n$=4+2$\n\n$=6$,\n\n即 $A$ 的值为 6 ;\n\n(2)由题意可得 $-2 x+2 y=6$,\n\n则当 $\\left\\{\\begin{array}{l}x=1 \\\\ y=4\\end{array}\\right.$ 时, $-2 x+2 y=6$ 也成立,\n\n$\\therefore$ 若使求得的 $A$ 的值与 (1)中的结果相同, 则给出的 $x, y$ 的值还能够是 $x=1, y=4$ (答案不唯一).\n【解析】(1)原式去括号, 合并同类项进行化简, 然后代入求值;\n\n(2)根据二元一次方程的解的概念分析求值.\n\n本题考查整式的加减一化简求值, 二元一次方程的解, 理解方程的解的概念, 掌握合并同类项(系数相加,字母及其指数不变)和去括号的运算法则(括号前面是 “ + ” 号, 去掉 “+” 号和括号, 括号里的各项不变号; 括号前面是 “-” 号, 去掉 “一” 号和括号,括号里的各项都变号)是解题关键."} {"id": "23532", "image": [], "answer": "【答案】解: 联立得: $\\left\\{\\begin{array}{l}3 x+4 y=2(1) \\\\ 2 x-y=5(2)\\end{array}\\right.$,\n\n(1) + (2) $\\times$ 4得: $11 x=22$, 即 $x=2$,\n\n将 $x=2$ 代入(2)得: $4-y=5$, 即 $y=-1$,\n\n$\\therefore$ 方程组的解为 $\\left\\{\\begin{array}{l}x=2 \\\\ y=-1\\end{array}\\right.$,\n\n代入得: $\\left\\{\\begin{array}{l}2 a-\\frac{b}{2}=5 \\\\ \\frac{2 a}{3}-b=4\\end{array}\\right.$,\n\n解得: $a=\\frac{9}{5}, b=-\\frac{14}{5}$.\n\n【解析】第一个方程组的第一个方程与第二个方程的第二个方程联立组成方程组, 求出方程组的解得到 $x$ 与 $y$ 的值, 代入剩下的方程即可求出 $a$ 与 $b$ 的值.\n\n此题考查了二元一次方程组的解, 方程组的解即为能使方程组中两方程成立的未知数的值.", "solution": "null", "level": "七年级", "question": "若方程组 $\\left\\{\\begin{array}{l}3 x+4 y=2 \\\\ a x+\\frac{b}{2} y=5\\end{array}\\right.$ 与 $\\left\\{\\begin{array}{l}\\frac{a}{3} x+b y=4 \\\\ 2 x-y=5\\end{array}\\right.$ 有相同的解, 则 $a 、 b$ 的值为多少?", "options": [], "subject": "代数", "analysis": "【答案】解: 联立得: $\\left\\{\\begin{array}{l}3 x+4 y=2(1) \\\\ 2 x-y=5(2)\\end{array}\\right.$,\n\n(1) + (2) $\\times$ 4得: $11 x=22$, 即 $x=2$,\n\n将 $x=2$ 代入(2)得: $4-y=5$, 即 $y=-1$,\n\n$\\therefore$ 方程组的解为 $\\left\\{\\begin{array}{l}x=2 \\\\ y=-1\\end{array}\\right.$,\n\n代入得: $\\left\\{\\begin{array}{l}2 a-\\frac{b}{2}=5 \\\\ \\frac{2 a}{3}-b=4\\end{array}\\right.$,\n\n解得: $a=\\frac{9}{5}, b=-\\frac{14}{5}$.\n\n【解析】第一个方程组的第一个方程与第二个方程的第二个方程联立组成方程组, 求出方程组的解得到 $x$ 与 $y$ 的值, 代入剩下的方程即可求出 $a$ 与 $b$ 的值.\n\n此题考查了二元一次方程组的解, 方程组的解即为能使方程组中两方程成立的未知数的值."} {"id": "23534", "image": [], "answer": "【答案】解:(1)把 $x=3, y=3$ 与 $x=-1, y=1$ 代入 $y=k x+b$ 得: $\\left\\{\\begin{array}{l}3 k+b=3 \\\\ -k+b=1\\end{array}\\right.$,\n\n解得 $\\left\\{\\begin{array}{l}k=\\frac{1}{2} \\\\ b=\\frac{3}{2}\\end{array}\\right.$,\n\n$\\therefore k=\\frac{1}{2}, \\quad b=\\frac{3}{2}$.\n\n(2)由(1)得 $y=\\frac{1}{2} x+\\frac{3}{2}$,\n\n$\\therefore$ 当 $x=-2$ 时, $y=-1+\\frac{3}{2}=\\frac{1}{2}$.\n【解析】(1)把 $x$ 与 $y$ 的值代入 $y=k x+b$ 中, 求出 $k$ 与 $b$ 的值;\n\n(2)将 $x$ 的值代入(1)所求的关系式计算即可求出 $y$ 的值\n\n此题考查了解二元一次方程组, 利用了消元的思想, 消元的方法有: 代入消元法与加减消元法.", "solution": "null", "level": "七年级", "question": "在等式 $y=k x+b$ 中, 当 $x=3$ 时, $y=3$; 当 $x=-1$ 时, $y=1$.\n\n(1)求 $k 、 b$ 的值;\n\n(2)求当 $x=-2$ 时 $y$ 的值.", "options": [], "subject": "代数", "analysis": "【答案】解:(1)把 $x=3, y=3$ 与 $x=-1, y=1$ 代入 $y=k x+b$ 得: $\\left\\{\\begin{array}{l}3 k+b=3 \\\\ -k+b=1\\end{array}\\right.$,\n\n解得 $\\left\\{\\begin{array}{l}k=\\frac{1}{2} \\\\ b=\\frac{3}{2}\\end{array}\\right.$,\n\n$\\therefore k=\\frac{1}{2}, \\quad b=\\frac{3}{2}$.\n\n(2)由(1)得 $y=\\frac{1}{2} x+\\frac{3}{2}$,\n\n$\\therefore$ 当 $x=-2$ 时, $y=-1+\\frac{3}{2}=\\frac{1}{2}$.\n【解析】(1)把 $x$ 与 $y$ 的值代入 $y=k x+b$ 中, 求出 $k$ 与 $b$ 的值;\n\n(2)将 $x$ 的值代入(1)所求的关系式计算即可求出 $y$ 的值\n\n此题考查了解二元一次方程组, 利用了消元的思想, 消元的方法有: 代入消元法与加减消元法."} {"id": "23490", "image": [], "answer": "【答案】解: 设原来预定 $x$ 天完成, 共定做 $y$ 个零件, 则 $\\{40 x=y$\n\n$\\{(1+20 \\%) \\times 40 \\times(x-16)=y+32 ,$\n\n解之得 $\\left\\{\\begin{array}{l}x=100 \\\\ y=4000\\end{array}\\right.$\n\n答:原来预定100天完成,共定做 4000 个零件.\n\n【解析】效率提高后, 每天加工 $(1+20 \\%) \\times 40$, 用了 $(x-16)$ 天完成任务. 本题的等量关系为: $40 \\times$ 原来的定额零件个数=原来共定做零件个数; $(1+20 \\%) \\times 40 \\times(x-$ 16) $=$ 原来共定做零件个数 +32 , 根据这两个等量关系可列出方程组.\n\n此题考查二元一次方程组的问题, 属工作量问题, 注意用公式: 工作量 $=$ 工作效率 $\\times$ 工作时间.", "solution": "null", "level": "七年级", "question": "某人承做一批零件, 原计划每天做 40 个, 可按期完成任务, 由于改进工艺, 工作效率提高了 $20 \\%$, 结果不但提前了 16 天完成, 而且超额完成了 32 件, 求原来预定几天完成? 原计划共做多少零件?", "options": [], "subject": "算术", "analysis": "【答案】解: 设原来预定 $x$ 天完成, 共定做 $y$ 个零件, 则 $\\{40 x=y$\n\n$\\{(1+20 \\%) \\times 40 \\times(x-16)=y+32 ,$\n\n解之得 $\\left\\{\\begin{array}{l}x=100 \\\\ y=4000\\end{array}\\right.$\n\n答:原来预定100天完成,共定做 4000 个零件.\n\n【解析】效率提高后, 每天加工 $(1+20 \\%) \\times 40$, 用了 $(x-16)$ 天完成任务. 本题的等量关系为: $40 \\times$ 原来的定额零件个数=原来共定做零件个数; $(1+20 \\%) \\times 40 \\times(x-$ 16) $=$ 原来共定做零件个数 +32 , 根据这两个等量关系可列出方程组.\n\n此题考查二元一次方程组的问题, 属工作量问题, 注意用公式: 工作量 $=$ 工作效率 $\\times$ 工作时间."} {"id": "23491", "image": ["11624.jpg", "11624.jpg"], "answer": "【答案】解: (1)设 1 辆 $A$ 型车和 1 辆 $B$ 型车都装满货物一次可分别运货 $\\lambda$ 吨、 $\\mu$ 吨,由题意得: $\\left\\{\\begin{array}{l}2 \\lambda+\\mu=10 \\\\ \\lambda+2 \\mu=11\\end{array}\\right.$,\n解得: $\\lambda=3, \\mu=4$.\n\n故1辆 $A$ 型车和1辆 $B$ 型车都装满货物一次可分别运货3吨、4吨.\n\n(2)由题意和(1)得: $3 a+4 b=26$,\n\n$\\because a 、 b$ 均为非负整数,\n\n\n\n$\\therefore$ 共有 2 种租车方案:\n\n(1) 租 $A$ 型车 6 辆, $B$ 型车 2 辆,\n\n(2) 租 $A$ 型车 2 辆, $B$ 型车 5 辆.\n\n(3)方案(1)的租金为: $6 \\times 100+2 \\times 120=840$ (元),\n\n方案(2)的租金为: $2 \\times 100+5 \\times 120=800$ (元),\n\n$\\because 840>800$,\n\n$\\therefore$ 最省钱的租车方案为方案(2), 租车费用为 800 元.\n\n【解析】(1)根据 2 辆 $A$ 型车和 1 辆 $B$ 型车装满货物 $=10$ 吨; 1 辆 $A$ 型车和 2 辆 $B$ 型车装满货物 $=11$ 吨, 列出方程组即可解决问题.\n\n(2)由题意得到 $3 a+4 b=26$, 根据 $a 、 b$ 均为正整数, 即可求出 $a 、 b$ 的值.\n\n(3)求出每种方案下的租金数, 经比较、分析, 即可解决问题.\n\n该题主要考查了列二元一次方程组或二元一次方程来解决现实生活中的实际应用问题;解题的关键是深入把握题意, 准确找出命题中隐含的数量关系, 正确列出方程或方程组来分析、推理、解答.", "solution": "null", "level": "七年级", "question": "已知: 用 2 辆 $A$ 型车和 1 辆 $B$ 型车载满货物一次可运货 10 吨; 用 1 辆 $A$ 型车和 2 辆 $B$ 型车载满货物一次可运货11吨, 某物流公司现有26吨货物, 计划 $A$ 型车 $a$ 辆, $B$ 型车 $b$ 辆,一次运完, 且恰好每辆车都载满货物.\n\n根据以上信息,解答下列问题:\n\n(1) $l$ 辆 $A$ 型车和 $l$ 辆车 $B$ 型车都载满货物一次可分别运货多少吨?\n\n(2)请你帮该物流公司设计租车方案:\n\n(3)若 $A$ 型车每辆需租金 100 元/次, $B$ 型车每辆需租金 120 元/次. 请选出最省钱车方案, 并求出最少租车费.", "options": [], "subject": "算术", "analysis": "【答案】解: (1)设 1 辆 $A$ 型车和 1 辆 $B$ 型车都装满货物一次可分别运货 $\\lambda$ 吨、 $\\mu$ 吨,由题意得: $\\left\\{\\begin{array}{l}2 \\lambda+\\mu=10 \\\\ \\lambda+2 \\mu=11\\end{array}\\right.$,\n解得: $\\lambda=3, \\mu=4$.\n\n故1辆 $A$ 型车和1辆 $B$ 型车都装满货物一次可分别运货3吨、4吨.\n\n(2)由题意和(1)得: $3 a+4 b=26$,\n\n$\\because a 、 b$ 均为非负整数,\n\n\n\n$\\therefore$ 共有 2 种租车方案:\n\n(1) 租 $A$ 型车 6 辆, $B$ 型车 2 辆,\n\n(2) 租 $A$ 型车 2 辆, $B$ 型车 5 辆.\n\n(3)方案(1)的租金为: $6 \\times 100+2 \\times 120=840$ (元),\n\n方案(2)的租金为: $2 \\times 100+5 \\times 120=800$ (元),\n\n$\\because 840>800$,\n\n$\\therefore$ 最省钱的租车方案为方案(2), 租车费用为 800 元.\n\n【解析】(1)根据 2 辆 $A$ 型车和 1 辆 $B$ 型车装满货物 $=10$ 吨; 1 辆 $A$ 型车和 2 辆 $B$ 型车装满货物 $=11$ 吨, 列出方程组即可解决问题.\n\n(2)由题意得到 $3 a+4 b=26$, 根据 $a 、 b$ 均为正整数, 即可求出 $a 、 b$ 的值.\n\n(3)求出每种方案下的租金数, 经比较、分析, 即可解决问题.\n\n该题主要考查了列二元一次方程组或二元一次方程来解决现实生活中的实际应用问题;解题的关键是深入把握题意, 准确找出命题中隐含的数量关系, 正确列出方程或方程组来分析、推理、解答."} {"id": "23512", "image": [], "answer": "【答案】 $\\frac{3}{10}$\n【解析】解: (1)设第一组有 $x$ 人, 第二组有 $y$ 人, 第三组有 $z$ 人, 根据题意得, $\\left\\{\\begin{array}{l}x=\\frac{3}{7}(y+z) \\\\ y=\\frac{1}{3}(x+z)\\end{array}\\right.$,\n\n整理, 得\n\n$\\left\\{\\begin{array}{l}7 x-3 y-3 z=01 \\\\ 3 y-x-z=0\\end{array}\\right.$\n\n(1) + (2)得,\n\n$x=\\frac{2}{3} z(3$,\n\n把(3)代入(2)得,\n\n$y=\\frac{5}{9} z$\n\n$\\therefore x+y+z=\\frac{2}{3} z+\\frac{5}{9} z+z=\\frac{20}{9} z$,\n\n$\\therefore \\frac{2 z}{3} \\div \\frac{20 z}{9}=\\frac{3}{10}$.\n\n$\\therefore$ 第一小组人数占三个小组总人数的 $\\frac{3}{10}$;\n\n故答案为: $\\frac{3}{10}$;\n\n(2)设第一组有 $x$ 人,第二组有 $y$ 人,\n\n依题意, 得: $\\left\\{\\begin{array}{l}x=\\frac{3}{7}(y+18) \\\\ y=\\frac{1}{3}(x+18)\\end{array}\\right.$\n\n解得: $\\left\\{\\begin{array}{l}x=12 \\\\ y=10\\end{array}\\right.$,\n\n答: 第一小组和第二小组分别有 12 人和 10 人.\n\n(1)设第一组有 $x$ 人, 第二组有 $y$ 人,第三组有 $z$ 人,根据题意用含 $z$ 的式子表示出 $x$ 和 $y$,进而可得第一小组人数占三个小组总人数的几分之几;\n\n(2)设第一组有 $x$ 人, 第二组有 $y$ 人, 根据题意即可得出关于 $x, y$ 的二元一次方程组, 解之即可得出 $x, y$ 的值.\n\n本题考查了二元一次方程组的应用, 找准等量关系, 正确列出二元一次方程组是解题的关键.", "solution": "null", "level": "七年级", "question": "三个小组, 第一小组人数是第二、第三小组人数和的 $\\frac{3}{7}$, 第二小组人数是第一、第三小组人数和的 $\\frac{1}{3}$.\n\n(1)第一小组人数占三个小组总人数的 $\\qquad$ (填几分之几).\n\n(2)如果第三小组有18人, 那么第一小组和第二小组分别有多少人?", "options": [], "subject": "算术", "analysis": "【答案】 $\\frac{3}{10}$\n【解析】解: (1)设第一组有 $x$ 人, 第二组有 $y$ 人, 第三组有 $z$ 人, 根据题意得, $\\left\\{\\begin{array}{l}x=\\frac{3}{7}(y+z) \\\\ y=\\frac{1}{3}(x+z)\\end{array}\\right.$,\n\n整理, 得\n\n$\\left\\{\\begin{array}{l}7 x-3 y-3 z=01 \\\\ 3 y-x-z=0\\end{array}\\right.$\n\n(1) + (2)得,\n\n$x=\\frac{2}{3} z(3$,\n\n把(3)代入(2)得,\n\n$y=\\frac{5}{9} z$\n\n$\\therefore x+y+z=\\frac{2}{3} z+\\frac{5}{9} z+z=\\frac{20}{9} z$,\n\n$\\therefore \\frac{2 z}{3} \\div \\frac{20 z}{9}=\\frac{3}{10}$.\n\n$\\therefore$ 第一小组人数占三个小组总人数的 $\\frac{3}{10}$;\n\n故答案为: $\\frac{3}{10}$;\n\n(2)设第一组有 $x$ 人,第二组有 $y$ 人,\n\n依题意, 得: $\\left\\{\\begin{array}{l}x=\\frac{3}{7}(y+18) \\\\ y=\\frac{1}{3}(x+18)\\end{array}\\right.$\n\n解得: $\\left\\{\\begin{array}{l}x=12 \\\\ y=10\\end{array}\\right.$,\n\n答: 第一小组和第二小组分别有 12 人和 10 人.\n\n(1)设第一组有 $x$ 人, 第二组有 $y$ 人,第三组有 $z$ 人,根据题意用含 $z$ 的式子表示出 $x$ 和 $y$,进而可得第一小组人数占三个小组总人数的几分之几;\n\n(2)设第一组有 $x$ 人, 第二组有 $y$ 人, 根据题意即可得出关于 $x, y$ 的二元一次方程组, 解之即可得出 $x, y$ 的值.\n\n本题考查了二元一次方程组的应用, 找准等量关系, 正确列出二元一次方程组是解题的关键."} {"id": "23537", "image": [], "answer": "【答案】解: (1)设每瓶免洗手消毒液和每瓶84消毒液的价格分别是 $a$ 元、 $b$ 元, $\\left\\{\\begin{array}{l}30 a+60 b=930 \\\\ 40 a+90 b=1320\\end{array}\\right.$\n\n解得 $\\left\\{\\begin{array}{l}a=15 \\\\ b=8\\end{array}\\right.$,\n\n即每瓶免洗手消毒液和每瓶84消毒液的价格分别是 15 元、8元;\n\n(2)方案一的花费为: $(15 \\times 100+8 \\times 60) \\times 0.8=1584$ (元),\n\n方案二的花费为: $15 \\times 100+8 \\times(60-100 \\div 10 \\times 5)=1580$ (元),\n\n$1584-1580=4$ (元), $1584>1580$,\n\n答: 学校选用方案二更节约钱, 节约 4 元.\n\n【解析】(1)根据购买 30 瓶免洗手消毒液和60瓶84消毒液, 共需花费 930 元, 如果购买 40 瓶免洗手消毒液和90瓶84消毒液, 共需花费1320元, 可以列出相应的二元一次方程组,从而可以求出每瓶免洗手消毒液和每瓶84消毒液的价格分别是多少元;\n\n(2)根据题意, 可以求出方案一和方案二的花费情况, 然后比较大小并作差即可解答本题.\n\n本题考查二元一次方程组的应用, 解答本题的关键是明确题意, 利用方程组的知识解答.", "solution": "null", "level": "七年级", "question": "在疫情防控期间, 某中学为保障广大师生生命健康安全, 欲从商场购进一批免洗手消毒液和84消毒液. 如果购买30瓶免洗手消毒液和60瓶84消毒液, 共需花费930元,如果购买 40 瓶免洗手消毒液和90瓶84消毒液, 共需花费1320元.\n\n(1)每瓶免洗手消毒液和每瓶84消毒液的价格分别是多少元?\n\n(2)若商场有两种促销方案: 方案一, 所有购买商品均打八折; 方案二, 购买 10 瓶免洗手消毒液送5瓶84消毒液, 学校打算购进免洗手消毒液100瓶, 84消毒液60瓶,请问学校选用哪种方案更节约钱?节约多少钱?", "options": [], "subject": "算术", "analysis": "【答案】解: (1)设每瓶免洗手消毒液和每瓶84消毒液的价格分别是 $a$ 元、 $b$ 元, $\\left\\{\\begin{array}{l}30 a+60 b=930 \\\\ 40 a+90 b=1320\\end{array}\\right.$\n\n解得 $\\left\\{\\begin{array}{l}a=15 \\\\ b=8\\end{array}\\right.$,\n\n即每瓶免洗手消毒液和每瓶84消毒液的价格分别是 15 元、8元;\n\n(2)方案一的花费为: $(15 \\times 100+8 \\times 60) \\times 0.8=1584$ (元),\n\n方案二的花费为: $15 \\times 100+8 \\times(60-100 \\div 10 \\times 5)=1580$ (元),\n\n$1584-1580=4$ (元), $1584>1580$,\n\n答: 学校选用方案二更节约钱, 节约 4 元.\n\n【解析】(1)根据购买 30 瓶免洗手消毒液和60瓶84消毒液, 共需花费 930 元, 如果购买 40 瓶免洗手消毒液和90瓶84消毒液, 共需花费1320元, 可以列出相应的二元一次方程组,从而可以求出每瓶免洗手消毒液和每瓶84消毒液的价格分别是多少元;\n\n(2)根据题意, 可以求出方案一和方案二的花费情况, 然后比较大小并作差即可解答本题.\n\n本题考查二元一次方程组的应用, 解答本题的关键是明确题意, 利用方程组的知识解答."} {"id": "23558", "image": [], "answer": "【答案】解: 原式 $=-8 x^{6}+3 x^{6}-x^{6}$\n\n$=-6 x^{6}$.\n\n【解析】根据同底数幂乘法, 积的乘方, 幂的乘方,同底数幂除法法则进行计算即可.本题考查同底数幂乘法, 积的乘方, 幂的乘方, 同底数幂除法, 解题关键是熟知同底数幂乘法, 积的乘方, 幂的乘方, 同底数幂除法法则.", "solution": "null", "level": "七年级", "question": "$\\left(-2 x^{2}\\right)^{3}+3 x^{2} \\cdot x^{4}-x^{8} \\div x^{2}$.", "options": [], "subject": "算术", "analysis": "【答案】解: 原式 $=-8 x^{6}+3 x^{6}-x^{6}$\n\n$=-6 x^{6}$.\n\n【解析】根据同底数幂乘法, 积的乘方, 幂的乘方,同底数幂除法法则进行计算即可.本题考查同底数幂乘法, 积的乘方, 幂的乘方, 同底数幂除法, 解题关键是熟知同底数幂乘法, 积的乘方, 幂的乘方, 同底数幂除法法则."} {"id": "23560", "image": [], "answer": "【答案】解: 原式 $=-1-2+1-(-5)=3$.\n\n【解析】本题主要考查实数的运算, 负整数指数幂和零指数幂, 掌握负整数指数幂和零指数幂是解答本题的关键.\n\n先分别算出立方根, 负整数指数幂和零指数幂, 然后相加减解答即可.", "solution": "null", "level": "七年级", "question": "计算: $(-1)^{2021}-\\sqrt[3]{8}+(\\pi-3.14)^{0}-\\left(-\\frac{1}{5}\\right)^{-1}$", "options": [], "subject": "算术", "analysis": "【答案】解: 原式 $=-1-2+1-(-5)=3$.\n\n【解析】本题主要考查实数的运算, 负整数指数幂和零指数幂, 掌握负整数指数幂和零指数幂是解答本题的关键.\n\n先分别算出立方根, 负整数指数幂和零指数幂, 然后相加减解答即可."} {"id": "23578", "image": [], "answer": "【答案】解: $\\because 1+2+3+\\cdots+n=m$,\n\n$\\therefore\\left(a b^{n}\\right) \\cdot\\left(a^{2} b^{n-1}\\right) \\ldots\\left(a^{n-1} b^{2}\\right) \\cdot\\left(a^{n} b\\right)$,\n\n$=a^{1+2+\\cdots n} b^{n+n-1+\\cdots+1}$,\n\n$=a^{m} b^{m}$.\n\n【解析】根据单项式的乘法法则, 同底数幂相乘, 底数不变, 指数相加的性质, $\\left(a b^{n}\\right)$. $\\left(a^{2} b^{n-1}\\right) \\ldots\\left(a^{n-1} b^{2}\\right) \\cdot\\left(a^{n} b\\right)=a^{1+2+\\cdots n} b^{n+n-1+\\cdots+1}=a^{m} b^{m}$.\n\n本题考查单项式的乘法法则和同底数幂的乘法的性质.", "solution": "null", "level": "七年级", "question": "若 $1+2+3+\\cdots+n=m$, 求 $\\left(a b^{n}\\right) \\cdot\\left(a^{2} b^{n-1}\\right) \\ldots\\left(a^{n-1} b^{2}\\right) \\cdot\\left(a^{n} b\\right)$ 的值.", "options": [], "subject": "算术", "analysis": "【答案】解: $\\because 1+2+3+\\cdots+n=m$,\n\n$\\therefore\\left(a b^{n}\\right) \\cdot\\left(a^{2} b^{n-1}\\right) \\ldots\\left(a^{n-1} b^{2}\\right) \\cdot\\left(a^{n} b\\right)$,\n\n$=a^{1+2+\\cdots n} b^{n+n-1+\\cdots+1}$,\n\n$=a^{m} b^{m}$.\n\n【解析】根据单项式的乘法法则, 同底数幂相乘, 底数不变, 指数相加的性质, $\\left(a b^{n}\\right)$. $\\left(a^{2} b^{n-1}\\right) \\ldots\\left(a^{n-1} b^{2}\\right) \\cdot\\left(a^{n} b\\right)=a^{1+2+\\cdots n} b^{n+n-1+\\cdots+1}=a^{m} b^{m}$.\n\n本题考查单项式的乘法法则和同底数幂的乘法的性质."} {"id": "23536", "image": [], "answer": "【答案】解: (1)第二天的账目有误, 理由如下:\n\n设甲、乙商品的单价分别为 $x, y$ 元, 可得:\n\n第一天: $39 x+21 y=321$ (1);\n\n第二天: $26 x+14 y=204$ (2);\n\n第三天: $39 x+25 y=345$ (3),\n\n由(1) $\\div 3$, 得: $13 x+7 y=107$,\n\n由(2) $\\div 2$, 得: $13 x+7 y=102$,\n$\\because$ 第一天的账目正确,\n\n$\\therefore$ 第二天的账目错误;\n\n(2)由(1)得: 第二天的账目错误,\n\n$\\therefore\\left\\{\\begin{array}{l}39 x+21 y=321 \\text { (1) } \\\\ 39 x+25 y=345 \\text { (3) }\\end{array}\\right.$,\n\n(3) - (1)得: $y=6$,\n\n把 $y=6$ 代入(1)得: $x=5$,\n\n所以方程组的解为: $\\left\\{\\begin{array}{l}x=5 \\\\ y=6\\end{array}\\right.$,\n\n答: 甲、乙商品的单价分别为 5 元, 6 元.\n\n【解析】(1)设甲、乙商品的单价分别为 $x, y$ 元, 根据题意列出方程组进行解答即可;\n\n(2)根据题意列出方程组进行解答即可.\n\n此题考查二元一次方程组的应用, 关键是根据题意列出方程组解答.", "solution": "null", "level": "七年级", "question": "某商店甲、乙两种商品三天销售情况的账目记录如下表:\n\n| 日期 | 买出甲商品的数量(
个) | | 卖出乙商品的数量 (
个) |\n| :--- | :---: | :---: | :--- |\n| 第一天 | 收入(元) | | |\n| 第二天 | 26 | 21 | 321 |\n| 第三天 | 39 | 14 | 204 |\n\n(1)财务主管在核查时发现: 第一天的账目正确,但其它两天的账目有一天有误,请你判断第几天的账目有误, 并说明理由;\n\n(2)求甲、乙商品的单价.", "options": [], "subject": "计数", "analysis": "【答案】解: (1)第二天的账目有误, 理由如下:\n\n设甲、乙商品的单价分别为 $x, y$ 元, 可得:\n\n第一天: $39 x+21 y=321$ (1);\n\n第二天: $26 x+14 y=204$ (2);\n\n第三天: $39 x+25 y=345$ (3),\n\n由(1) $\\div 3$, 得: $13 x+7 y=107$,\n\n由(2) $\\div 2$, 得: $13 x+7 y=102$,\n$\\because$ 第一天的账目正确,\n\n$\\therefore$ 第二天的账目错误;\n\n(2)由(1)得: 第二天的账目错误,\n\n$\\therefore\\left\\{\\begin{array}{l}39 x+21 y=321 \\text { (1) } \\\\ 39 x+25 y=345 \\text { (3) }\\end{array}\\right.$,\n\n(3) - (1)得: $y=6$,\n\n把 $y=6$ 代入(1)得: $x=5$,\n\n所以方程组的解为: $\\left\\{\\begin{array}{l}x=5 \\\\ y=6\\end{array}\\right.$,\n\n答: 甲、乙商品的单价分别为 5 元, 6 元.\n\n【解析】(1)设甲、乙商品的单价分别为 $x, y$ 元, 根据题意列出方程组进行解答即可;\n\n(2)根据题意列出方程组进行解答即可.\n\n此题考查二元一次方程组的应用, 关键是根据题意列出方程组解答."} {"id": "23765", "image": [], "answer": "【答案】调查\n\n【解析】略", "solution": "null", "level": "七年级", "question": "下表是某地区1000名居民 “获取信息的主要途径” 的统计数据, 这里的数据是通过什么方法收集得到的.\n\n| 途径 | 电脑 | 手机 | 电视 | 广播 | 其他 |\n| :---: | :---: | :---: | :---: | :---: | :---: |\n| 人数 | 260 | 400 | 150 | 100 | 90 |", "options": [], "subject": "计数", "analysis": "【答案】调查\n\n【解析】略"} {"id": "23766", "image": [], "answer": "【答案】解:(1)总体:该种家用空调工作1小时的用电量;\n\n个体:每一台该种家用空调工作1小时的用电量;\n\n样本:抽取的10台该种家用空调每台工作1小时的用电量;\n\n样本容量: 10 ;\n\n(2)总体: 初二年级270名学生的视力情况;\n\n个体:每一名学生的视力情况;\n\n样本: 抽取的50名学生的视力情况;\n\n样本容量: 50 .\n\n【解析】考查了总体、个体、样本、样本容量.\n\n(1)根据总体、个体、样本、样本容量的定义判断即可;\n\n(2)根据总体、个体、样本、样本容量的定义判断即可.", "solution": "null", "level": "七年级", "question": "请指出下列抽样调查的总体、个体、样本、样本容量分别是什么.\n\n(1)为了了解某种家用空调工作 1 小时的用电量, 调查 10 台该种空调每台工作 1 小时的用电量.\n\n(2)为了了解初二年级270名学生的视力情况, 从中抽取 50 名学生进行视力检查.", "options": [], "subject": "计数", "analysis": "【答案】解:(1)总体:该种家用空调工作1小时的用电量;\n\n个体:每一台该种家用空调工作1小时的用电量;\n\n样本:抽取的10台该种家用空调每台工作1小时的用电量;\n\n样本容量: 10 ;\n\n(2)总体: 初二年级270名学生的视力情况;\n\n个体:每一名学生的视力情况;\n\n样本: 抽取的50名学生的视力情况;\n\n样本容量: 50 .\n\n【解析】考查了总体、个体、样本、样本容量.\n\n(1)根据总体、个体、样本、样本容量的定义判断即可;\n\n(2)根据总体、个体、样本、样本容量的定义判断即可."} {"id": "23767", "image": ["11684.jpg", "11685.jpg", "11685.jpg"], "answer": "【答案】解:(1) $40 ; 30$;\n\n(2)补全条形统计图为:\n\n莱中学 \"珛是喜受的职业\" 拥量的统计固\n\n\n\n【解析】\n\n【分析】\n\n本题考查了条形统计图, 通过条形统计图获取需要的数据, 然后求得教师和医生的人数之和, 再利用已知比例得出各自的人数, 最后补全条形统计图.本题难度不大, 是基础题.\n\n(1)由统计图可知: 喜欢的职业是公务员的有 40 人,军人 20 人,其他 70 人,这样教师和医生的和还有 $200-40-20-70=70$ 人, 根据 $4: 3$ 得出各自的人数, 然后将图形补充完整即可.\n\n(2)由(1)中得到的数据补全条形统计图即可.\n\n【解答】\n\n解: (1) $\\because$ 选择 “教师” 的人数与选择 “医生” 的人数比为 $4: 3$, $\\therefore$ 设选择 “教师” 的有 $4 x$ 人,选择 “医生” 的有 $3 x$ 人,\n\n由题意知: “教师” 和选择 “医生” 的共有:\n\n$200-40-20-70=70$ (人),\n\n$\\therefore 4 x+3 x=70$,\n\n解得 $x=10$,\n\n$\\therefore 4 x=40,3 x=30$,\n\n即选择 “教师” 的有 40 人,选择“医生” 的有 30 人.\n\n故答案为: $40 ; 30$;\n\n(2)见答案.", "solution": "null", "level": "七年级", "question": "某中学开展以 “我最喜爱的职业” 为主题的调查活动. 通过对 200 名学生的随机调查得到一组数据,并绘制成条形统计图(不完整).根据统计图回答:\n\n\n\n(1)若选择 “教师” 的人数与选择 “医生” 的人数比为 $4: 3$, 则选择 “教师” 的有 $\\qquad$人,选择 “医生” 的有 $\\qquad$人.\n\n(2)根据第(1)题的结论补全条形统计图.", "options": [], "subject": "计数", "analysis": "【答案】解:(1) $40 ; 30$;\n\n(2)补全条形统计图为:\n\n莱中学 \"珛是喜受的职业\" 拥量的统计固\n\n\n\n【解析】\n\n【分析】\n\n本题考查了条形统计图, 通过条形统计图获取需要的数据, 然后求得教师和医生的人数之和, 再利用已知比例得出各自的人数, 最后补全条形统计图.本题难度不大, 是基础题.\n\n(1)由统计图可知: 喜欢的职业是公务员的有 40 人,军人 20 人,其他 70 人,这样教师和医生的和还有 $200-40-20-70=70$ 人, 根据 $4: 3$ 得出各自的人数, 然后将图形补充完整即可.\n\n(2)由(1)中得到的数据补全条形统计图即可.\n\n【解答】\n\n解: (1) $\\because$ 选择 “教师” 的人数与选择 “医生” 的人数比为 $4: 3$, $\\therefore$ 设选择 “教师” 的有 $4 x$ 人,选择 “医生” 的有 $3 x$ 人,\n\n由题意知: “教师” 和选择 “医生” 的共有:\n\n$200-40-20-70=70$ (人),\n\n$\\therefore 4 x+3 x=70$,\n\n解得 $x=10$,\n\n$\\therefore 4 x=40,3 x=30$,\n\n即选择 “教师” 的有 40 人,选择“医生” 的有 30 人.\n\n故答案为: $40 ; 30$;\n\n(2)见答案."} {"id": "23769", "image": ["11686.jpg", "11687.jpg", "11688.jpg", "11687.jpg", "11688.jpg"], "answer": "【答案】 50\n\n【解析】解: (1) $\\because B 、 E$ 两组发言人数的比为 $5: 2, E$ 组发言人数占 $8 \\%$,\n\n$\\therefore B$ 组发言的人数占 $20 \\%$,\n\n由直方图可知 $B$ 组人数为 10 人,\n\n所以, 被抽查的学生人数为: $10 \\div 20 \\%=50$ 人,\n\n故答案为: 50 人;\n\n(2) $F$ 组人数为: $50 \\times(1-6 \\%-20 \\%-30 \\%-26 \\%-8 \\%)$\n\n$=50 \\times(1-90 \\%)$\n\n$=50 \\times 10 \\%$,\n\n$=5$ (人),\n\n$C$ 组人数为: $50 \\times 30 \\%=15$ (人),\n$E$ 组人数为: $50 \\times 8 \\%=4$ (人),\n\n补全的直方图如图;\n\n\n\n(3) $F$ 组发言的人数所占的百分比为: $10 \\%$,\n\n所以, 估计全年级在这天里发言次数不少于 12 次的人数为: $500 \\times(8 \\%+10 \\%)=90$ (人 );\n\n(4) $\\because A$ 组发言的学生为: $50 \\times 6 \\%=3$ 人, 有 1 位女生,\n\n$\\therefore A$ 组发言的有2位男生,\n\n$\\because E$ 组发言的学生: 4 人,\n\n$\\therefore$ 有2位女生, 2位男生.\n\n$\\therefore$ 由题意可画树状图为:\n\n\n\n$\\therefore$ 共有12种情况, 所抽的两位学生恰好是一男一女的情况有6种,\n\n$\\therefore$ 所抽的两位学生恰好是一男一女的概率为 $\\frac{1}{2}$.\n\n(1)求得 $B$ 组所占的百分比, 然后根据 $B$ 组有 10 人即可求得总人数;\n\n(2)求得 $C$ 组的人数, 从而补全直方图;\n\n(3)利用总人数乘以对应的百分比即可求解;\n\n(4)分别求出 $A 、 E$ 两组的人数, 确定出各组的男女生人数, 然后列表或画树状图, 再根据概率公式计算即可得解.\n\n本题考查读频数分布直方图的能力和利用统计图获取信息的能力; 利用统计图获取信息时, 必须认真观察、分析、研究统计图, 才能作出正确的判断和解决问题, 本题根据B组\n的人数与所占的百分比求解是解题的关键, 也是本题的突破口.", "solution": "null", "level": "七年级", "question": "为了解某校九年级学生课堂发言情况, 随机抽取该年级部分学生, 对他们某天在课堂上发言的次数进行统计, 结果如下表, 并绘制了如下尚不完整的统计图, 已知 $B$, $E$ 两组发言的人数比为 $5: 2$, 请结合图表中相关数据回答下列问题:\n\n\n发言人数扇形统计图\n\n| 组别 | 课堂发言次数 $n$ |\n| :---: | :---: |\n| $A$ | $0 \\leq n<3$ |\n| $B$ | $3 \\leq n<6$ |\n| $C$ | $6 \\leq n<9$ |\n| $D$ | $9 \\leq n<12$ |\n| $E$ | $12 \\leq n<15$ |\n| $F$ | $15 \\leq n<18$ |\n\n(1)本次抽样的学生人数为 $\\qquad$ ;\n\n(2)补全条形统计图;\n\n(3)该年级共有学生 500 人, 请估计这天全年级发言次数不少于 12 的人数;\n\n(4)已知 $A$ 组发言的学生中有1位女生, $E$ 组发言的学生中有 2 位男生, 现从 $A$ 组与 $E$ 组中分别抽一位学生写报告, 请用树状图或列表法, 求所抽到的两位学生恰好是一男\n\n一女的概率.", "options": [], "subject": "计数", "analysis": "【答案】 50\n\n【解析】解: (1) $\\because B 、 E$ 两组发言人数的比为 $5: 2, E$ 组发言人数占 $8 \\%$,\n\n$\\therefore B$ 组发言的人数占 $20 \\%$,\n\n由直方图可知 $B$ 组人数为 10 人,\n\n所以, 被抽查的学生人数为: $10 \\div 20 \\%=50$ 人,\n\n故答案为: 50 人;\n\n(2) $F$ 组人数为: $50 \\times(1-6 \\%-20 \\%-30 \\%-26 \\%-8 \\%)$\n\n$=50 \\times(1-90 \\%)$\n\n$=50 \\times 10 \\%$,\n\n$=5$ (人),\n\n$C$ 组人数为: $50 \\times 30 \\%=15$ (人),\n$E$ 组人数为: $50 \\times 8 \\%=4$ (人),\n\n补全的直方图如图;\n\n\n\n(3) $F$ 组发言的人数所占的百分比为: $10 \\%$,\n\n所以, 估计全年级在这天里发言次数不少于 12 次的人数为: $500 \\times(8 \\%+10 \\%)=90$ (人 );\n\n(4) $\\because A$ 组发言的学生为: $50 \\times 6 \\%=3$ 人, 有 1 位女生,\n\n$\\therefore A$ 组发言的有2位男生,\n\n$\\because E$ 组发言的学生: 4 人,\n\n$\\therefore$ 有2位女生, 2位男生.\n\n$\\therefore$ 由题意可画树状图为:\n\n\n\n$\\therefore$ 共有12种情况, 所抽的两位学生恰好是一男一女的情况有6种,\n\n$\\therefore$ 所抽的两位学生恰好是一男一女的概率为 $\\frac{1}{2}$.\n\n(1)求得 $B$ 组所占的百分比, 然后根据 $B$ 组有 10 人即可求得总人数;\n\n(2)求得 $C$ 组的人数, 从而补全直方图;\n\n(3)利用总人数乘以对应的百分比即可求解;\n\n(4)分别求出 $A 、 E$ 两组的人数, 确定出各组的男女生人数, 然后列表或画树状图, 再根据概率公式计算即可得解.\n\n本题考查读频数分布直方图的能力和利用统计图获取信息的能力; 利用统计图获取信息时, 必须认真观察、分析、研究统计图, 才能作出正确的判断和解决问题, 本题根据B组\n的人数与所占的百分比求解是解题的关键, 也是本题的突破口."} {"id": "23770", "image": ["11689.jpg"], "answer": "【答案】解: (1) $550 \\div 55 \\%=1000$ (只) $1000-400-550-30=20$ (只)\n\n即: $m=20$,\n\n$360^{\\circ} \\times \\frac{400}{1000}=144^{\\circ}$,\n\n答: 表中 $m$ 的值为 20 , 图中 $B$ 组扇形的圆心角的度数为 $144^{\\circ}$;\n\n(2) $\\frac{400}{1000}+\\frac{550}{1000}=\\frac{950}{1000}=95 \\%$,\n\n$12 \\times 10 \\times(1-95 \\%)=120 \\times 5 \\%=6$ (只),\n\n答: 这次抽样检验的合格率是 $95 \\%$, 所购得的羽毛球中, 非合格品的羽毛球有 6 只.\n\n【解析】(1)图表中 “ $C$ 组” 的频数为 550 只, 占抽查总数的 $55 \\%$, 可求出抽查总数, 进而求出 “ $A$ 组” 的频数, 即 $m$ 的值; 求出 “ $B$ 组” 所占总数的百分比, 即可求出相应的圆心角的度数;\n\n(2)计算 “ $B$ 组” “ $C$ 组” 的频率的和即为合格率, 求出 “不合格” 所占的百分比, 即可求出不合格的数量.\n\n本题考查了统计表、扇形统计图的意义和制作方法, 理解图表中的数量和数量之间的关系, 是正确计算的前提.", "solution": "null", "level": "七年级", "question": "一只羽毛球的重量合格标准是5.0克 $\\sim 5.2$ 克(含5.0克, 不含5.2克), 某厂对4月份生产的羽毛球重量进行抽样检验. 并将所得数据绘制成如图统计图表.\n\n4月份生产的羽毛球重量统计表\n\n| 组别 | 重量 $x$ (克) | 数量(只) |\n| :---: | :---: | :---: |\n| $A$ | $x<5.0$ | $m$ |\n| $B$ | $5.0 \\leq x<5.1$ | 400 |\n| $C$ | $5.1 \\leq x<5.2$ | 550 |\n| $D$ | $x \\geq 5.2$ | 30 |\n\n(1)求表中 $m$ 的值及图中 $B$ 组扇形的圆心角的度数.\n\n(2)问这些抽样检验的羽毛球中, 合格率是多少?如果购得4月份生产的羽毛球10筒 (每筒12只), 估计所购得的羽毛球中, 非合格品的羽毛球有多少只?\n\n4月份生产的羽毛球重量统计图\n\n", "options": [], "subject": "计数", "analysis": "【答案】解: (1) $550 \\div 55 \\%=1000$ (只) $1000-400-550-30=20$ (只)\n\n即: $m=20$,\n\n$360^{\\circ} \\times \\frac{400}{1000}=144^{\\circ}$,\n\n答: 表中 $m$ 的值为 20 , 图中 $B$ 组扇形的圆心角的度数为 $144^{\\circ}$;\n\n(2) $\\frac{400}{1000}+\\frac{550}{1000}=\\frac{950}{1000}=95 \\%$,\n\n$12 \\times 10 \\times(1-95 \\%)=120 \\times 5 \\%=6$ (只),\n\n答: 这次抽样检验的合格率是 $95 \\%$, 所购得的羽毛球中, 非合格品的羽毛球有 6 只.\n\n【解析】(1)图表中 “ $C$ 组” 的频数为 550 只, 占抽查总数的 $55 \\%$, 可求出抽查总数, 进而求出 “ $A$ 组” 的频数, 即 $m$ 的值; 求出 “ $B$ 组” 所占总数的百分比, 即可求出相应的圆心角的度数;\n\n(2)计算 “ $B$ 组” “ $C$ 组” 的频率的和即为合格率, 求出 “不合格” 所占的百分比, 即可求出不合格的数量.\n\n本题考查了统计表、扇形统计图的意义和制作方法, 理解图表中的数量和数量之间的关系, 是正确计算的前提."} {"id": "23771", "image": [], "answer": "【答案】解: $(1) 2+4+21+13+8+4=52$ (人);\n\n(2)组距: $80-60=20$,\n\n组数是 6 ;\n\n(3)跳绳次数 $x$ 在 $120 \\leq x<160$ 范围的学生有: $13+8=21$ (人).\n\n【解析】此题主要考查了频数分布表, 在统计数据时, 经常把数据按照不同的范围分成几个组, 分成的组的个数称为组数, 每一组两个端点的差称为组距, 称这样画出的统计图表为频数分布表.\n\n(1)根据频数分布表可得把每个小组的频数加起来就是全班的学生数;\n\n(2)组距就是每个小组的最大值和最小值之差; 根据表格可直接得到组数为 6 ;\n\n(3)跳绳次数 $x$ 在 $120 \\leq x<160$ 就是求 $120 \\leq x<140,140 \\leq x<160$ 两组的频数和.", "solution": "null", "level": "七年级", "question": "体育委员统计了全班同学 $60 s$ 跳绳的次数, 并列出频数表如下:\n\n| 次数 | $60 \\leq x$
$<80$ | $80 \\leq x$
$<100$ | $100 \\leq x$
$<120$ | $120 \\leq x$
$<140$ | $140 \\leq x$
$<160$ | $160 \\leq x$
$<180$ |\n| :---: | :---: | :---: | :---: | :---: | :---: | :---: |\n| 频数 | 2 | 4 | 21 | 13 | 8 | 4 |\n\n(1)全班共有多少名学生?\n\n(2)组距是多少? 组数是多少?\n\n(3)跳绳次数在 $120 \\leq x<160$ 范围内的学生有多少?", "options": [], "subject": "计数", "analysis": "【答案】解: $(1) 2+4+21+13+8+4=52$ (人);\n\n(2)组距: $80-60=20$,\n\n组数是 6 ;\n\n(3)跳绳次数 $x$ 在 $120 \\leq x<160$ 范围的学生有: $13+8=21$ (人).\n\n【解析】此题主要考查了频数分布表, 在统计数据时, 经常把数据按照不同的范围分成几个组, 分成的组的个数称为组数, 每一组两个端点的差称为组距, 称这样画出的统计图表为频数分布表.\n\n(1)根据频数分布表可得把每个小组的频数加起来就是全班的学生数;\n\n(2)组距就是每个小组的最大值和最小值之差; 根据表格可直接得到组数为 6 ;\n\n(3)跳绳次数 $x$ 在 $120 \\leq x<160$ 就是求 $120 \\leq x<140,140 \\leq x<160$ 两组的频数和."} {"id": "23629", "image": ["11655.jpg"], "answer": "【答案】解: $\\pi\\left[\\left(\\frac{1}{2} D\\right)^{2}-\\left(\\frac{1}{2} d\\right)^{2}\\right] h$\n$$\n\\begin{aligned}\n& =\\frac{1}{4} \\pi(D+d)(D-d) h \\\\\n& =\\frac{1}{4} \\pi \\times(88+68)(88-68) \\times 200\n\\end{aligned}\n$$\n\n$=156000 \\pi\\left(\\mathrm{cm}^{3}\\right)$,\n\n$156000 \\pi \\mathrm{cm}^{3}=0.156 \\pi \\mathrm{m}^{3}$.\n\n答: 浇制一节这样的排水管需要 $0.156 \\pi m^{3}$ 的混凝土.\n\n【解析】利用圆柱的体积公式, 浇制一节这样的排水管需要的混凝土为 $\\pi\\left[\\left(\\frac{1}{2} D\\right)^{2}-\\right.$ $\\left.\\left(\\frac{1}{2} d\\right)^{2}\\right] h$, 然后把 $d=68 \\mathrm{~cm}$, 外径 $D=88 \\mathrm{~cm}$, 长 $h=200 \\mathrm{~cm}$ 代入计算, 最后结果保留 $\\pi$.本题考查了因式分解的应用, 利用因式分解简化计算问题.", "solution": "null", "level": "七年级", "question": "一种混凝土排水管, 其形状为空心的圆柱体, 它的内径 $d=68 \\mathrm{~cm}$, 外径 $D=88 \\mathrm{~cm}$,长 $h=200 \\mathrm{~cm}$.浇制一节这样的排水管需要多少立方米的混凝土 $($ 结果保留 $\\pi)$ ?怎样计算较简便?\n\n", "options": [], "subject": "立体几何学", "analysis": "【答案】解: $\\pi\\left[\\left(\\frac{1}{2} D\\right)^{2}-\\left(\\frac{1}{2} d\\right)^{2}\\right] h$\n$$\n\\begin{aligned}\n& =\\frac{1}{4} \\pi(D+d)(D-d) h \\\\\n& =\\frac{1}{4} \\pi \\times(88+68)(88-68) \\times 200\n\\end{aligned}\n$$\n\n$=156000 \\pi\\left(\\mathrm{cm}^{3}\\right)$,\n\n$156000 \\pi \\mathrm{cm}^{3}=0.156 \\pi \\mathrm{m}^{3}$.\n\n答: 浇制一节这样的排水管需要 $0.156 \\pi m^{3}$ 的混凝土.\n\n【解析】利用圆柱的体积公式, 浇制一节这样的排水管需要的混凝土为 $\\pi\\left[\\left(\\frac{1}{2} D\\right)^{2}-\\right.$ $\\left.\\left(\\frac{1}{2} d\\right)^{2}\\right] h$, 然后把 $d=68 \\mathrm{~cm}$, 外径 $D=88 \\mathrm{~cm}$, 长 $h=200 \\mathrm{~cm}$ 代入计算, 最后结果保留 $\\pi$.本题考查了因式分解的应用, 利用因式分解简化计算问题."} {"id": "23816", "image": [], "answer": "【答案】答案test\n\n【解析】解析test", "solution": "null", "level": "七年级", "question": "某玩具生产公司每天计划生产卫兵、骑兵、伞兵这三种玩具共100个, 生产一个卫兵需 $5 \\mathrm{~min}$, 生产一个骑兵需 $7 \\mathrm{~min}$, 生产一个伞兵需 $4 \\mathrm{~min}$, 已知总生产时间不超过 $10 h$. 若生产一个卫兵可获利5元, 生产一个骑兵可获利6元, 生产一个冞兵可获利3元.\n\n(1)试用每天生产的卫兵个数 $x$ 与骑兵个数 $y$ 表示每天的利润 $w$.\n\n(2)怎样分配生产任务才能使每天的利润最大?最大利润是多少?", "options": [], "subject": "组合数学", "analysis": "【答案】答案test\n\n【解析】解析test"} {"id": "9047", "image": [], "answer": "解$\\times$\n\n用米作单位表示物体的长度时,整数部分表示几米,小数点后面第一位表示几分米,第二位表示几厘米;据此解答即可。\n\n7.3 米就是 7 米 3 分米,所以原题的说法错误。\n\n故答案为: $x$\n\n此题主要是考查长度单位的换算, 记住: 单位是米时, 小数部分十分位上的数表示分米数,百分位上的数表示厘米数......。", "solution": "null", "level": "三年级", "question": "7.3 米就是 7 米 3 厘米。( $\\quad)$", "options": [], "subject": "度量几何学", "analysis": "解$\\times$\n\n用米作单位表示物体的长度时,整数部分表示几米,小数点后面第一位表示几分米,第二位表示几厘米;据此解答即可。\n\n7.3 米就是 7 米 3 分米,所以原题的说法错误。\n\n故答案为: $x$\n\n此题主要是考查长度单位的换算, 记住: 单位是米时, 小数部分十分位上的数表示分米数,百分位上的数表示厘米数......。"} {"id": "9252", "image": ["1527.jpg"], "answer": "解周长 $92 \\mathrm{~cm}$; 面积 $368 \\mathrm{~cm}^{2}$\n\n【分析】根据图示, 图形的周长等于大长方形的周长加 2 个 $4 \\mathrm{~cm}$, 根据长方形周长 $=($ 长 + 宽 $) \\times 2$,代入数据计算即可解答;\n\n根据图示, 图形的面积等于大长方形的面积减去长 $12 \\mathrm{~cm}$, 宽 $4 \\mathrm{~cm}$ 的小长方形的面积, 根据长方形面积 $=$ 长 $\\times$ 宽, 代入数据计算即可解答。\n\n【详解】周长: $(16+26) \\times 2+4 \\times 2$\n\n$=42 \\times 2+4 \\times 2$\n\n$=84+8$\n\n$=92(\\mathrm{~cm})$\n\n面积: $16 \\times 26-12 \\times 4$\n\n$=416-48$\n\n$=368\\left(\\mathrm{~cm}^{2}\\right)$", "solution": "null", "level": "三年级", "question": "计算下图所示图形的周长及面积。(单位: $\\mathrm{cm}$ )\n\n", "options": [], "subject": "度量几何学", "analysis": "解周长 $92 \\mathrm{~cm}$; 面积 $368 \\mathrm{~cm}^{2}$\n\n【分析】根据图示, 图形的周长等于大长方形的周长加 2 个 $4 \\mathrm{~cm}$, 根据长方形周长 $=($ 长 + 宽 $) \\times 2$,代入数据计算即可解答;\n\n根据图示, 图形的面积等于大长方形的面积减去长 $12 \\mathrm{~cm}$, 宽 $4 \\mathrm{~cm}$ 的小长方形的面积, 根据长方形面积 $=$ 长 $\\times$ 宽, 代入数据计算即可解答。\n\n【详解】周长: $(16+26) \\times 2+4 \\times 2$\n\n$=42 \\times 2+4 \\times 2$\n\n$=84+8$\n\n$=92(\\mathrm{~cm})$\n\n面积: $16 \\times 26-12 \\times 4$\n\n$=416-48$\n\n$=368\\left(\\mathrm{~cm}^{2}\\right)$"} {"id": "9277", "image": ["1537.jpg"], "answer": "解121 平方厘米\n\n【分析】 此图为正方形, 正方形的面积 $=$ 边长 $\\times$ 边长, 依此计算即可。\n\n【详解】 $11 \\times 11=121$ (平方厘米)", "solution": "null", "level": "三年级", "question": "求下面正方形的面积。\n\n", "options": [], "subject": "度量几何学", "analysis": "解121 平方厘米\n\n【分析】 此图为正方形, 正方形的面积 $=$ 边长 $\\times$ 边长, 依此计算即可。\n\n【详解】 $11 \\times 11=121$ (平方厘米)"} {"id": "9278", "image": ["1538.jpg"], "answer": "解72 平方分米\n\n【分析】根据图形的特点, 可以利用“填衴”法, 也就是用大长方形的面积减去一个长是 4 分米, 宽是 3 分米的小长方形的面积。根据长方形的面积 $=$ 长 $\\times$ 宽, 把数据代入公式解答。\n\n【详解】 $12 \\times 7-4 \\times 3$\n\n$=84-12$\n\n$=72($ 平方分米 $)$\n\n图形面积是 72 平方分米。", "solution": "null", "level": "三年级", "question": "计算下图图形的面积。\n\n", "options": [], "subject": "度量几何学", "analysis": "解72 平方分米\n\n【分析】根据图形的特点, 可以利用“填衴”法, 也就是用大长方形的面积减去一个长是 4 分米, 宽是 3 分米的小长方形的面积。根据长方形的面积 $=$ 长 $\\times$ 宽, 把数据代入公式解答。\n\n【详解】 $12 \\times 7-4 \\times 3$\n\n$=84-12$\n\n$=72($ 平方分米 $)$\n\n图形面积是 72 平方分米。"} {"id": "9048", "image": [], "answer": "解$\\times$\n\n比 15 大比 18 小的小数有一位小数, 还有两位小数、三位小数、四位小数......, 所以比 15 大而比 18 小的小数有无数个; 据此进行判断即可。\n\n比 15 大比 18 小的小数有无数个;\n\n故答案为: $x$\n\n\n\n做此类型的判断题一定要注意“数”的取值范围。", "solution": "null", "level": "三年级", "question": "比 15 大比 18 小的小数只有 2 个。 $(\\quad)$", "options": [], "subject": "算术", "analysis": "解$\\times$\n\n比 15 大比 18 小的小数有一位小数, 还有两位小数、三位小数、四位小数......, 所以比 15 大而比 18 小的小数有无数个; 据此进行判断即可。\n\n比 15 大比 18 小的小数有无数个;\n\n故答案为: $x$\n\n\n\n做此类型的判断题一定要注意“数”的取值范围。"} {"id": "9050", "image": ["1453.jpg", "1454.jpg"], "answer": "对", "solution": "null", "level": "三年级", "question": "红红带 20 元钱足够买一本《安徒生通话》和一本《我们爱科学》. ( )\n\n\n\n\n\n7.5 元 $/$ 本 4.5 元 $/$ 本", "options": [], "subject": "算术", "analysis": "解对rt{ }$\n\n略"} {"id": "9051", "image": ["1455.jpg"], "answer": "对", "solution": "null", "level": "三年级", "question": "用坚式计算小数加减法时, 小数点要对齐。( $)$\n\n", "options": [], "subject": "算术", "analysis": "解对rt{ }$\n\n依据小数坚式计算方法: 把小数点对齐, 相同的数位也就对齐了, 然后按照整数加减法计算方法计算解答。\n\n根据分析, 用坚式计算小数加减法时, 小数点要对齐。\n\n故答案为: 对rt{ }$\n\n本题主要考查学生对于小数坚式计算方法的掌握情况。"} {"id": "8386", "image": [], "answer": "$120 ; 780 ; 57 ; 90 ; 950$;\n\n$960 ; 690 ; 780 ; 720 ; 80$;", "solution": "null", "level": "三年级", "question": "计算。\n\n| $20 \\times 6=$ | $130 \\times 6=$ | $19 \\times 3=$ | $45 \\times 2=$ | $190 \\times 5=$ |\n| :--- | :--- | :--- | :--- | ---: |\n| $240 \\times 4=$ | $230 \\times 3=$ | $390 \\times 2=$ | $180 \\times 4=$ | $16 \\times 5=$ |", "options": [], "subject": "算术", "analysis": "$120 ; 780 ; 57 ; 90 ; 950$;\n\n$960 ; 690 ; 780 ; 720 ; 80$;"} {"id": "8387", "image": ["1308.jpg", "1308.jpg"], "answer": "$418 ; 759 ; 882 ; 504$\n\n【分析】笔算两位数乘两位数的进位乘法时, 用第二个乘数每一位上的数分别去乘第一个乘数, 用哪一位上的数去乘, 积的末尾就和那一位对齐, 哪一位乘得的积满几十, 就要向前一位进几, 再把两次乘得的积相加,据此解题即可。\n【详解】 $38 \\times 11=418$\n$33 \\times 23=759$\n$42 \\times 21=882$\n$21 \\times 24=504$\n\n", "solution": "null", "level": "三年级", "question": "列坚式计算。\n\n$38 \\times 11=\\quad 33 \\times 23=\\quad 42 \\times 21=\\quad 21 \\times 24=$", "options": [], "subject": "算术", "analysis": "$418 ; 759 ; 882 ; 504$\n\n【分析】笔算两位数乘两位数的进位乘法时, 用第二个乘数每一位上的数分别去乘第一个乘数, 用哪一位上的数去乘, 积的末尾就和那一位对齐, 哪一位乘得的积满几十, 就要向前一位进几, 再把两次乘得的积相加,据此解题即可。\n【详解】 $38 \\times 11=418$\n$33 \\times 23=759$\n$42 \\times 21=882$\n$21 \\times 24=504$\n\n"} {"id": "8412", "image": [], "answer": "$90 ; 96 ; 56 ; 36 ;$\n\n$900 ; 960 ; 560 ; 360 ;$", "solution": "null", "level": "三年级", "question": "比一比,算一算。\n\n| $18 \\times 5=$ | $24 \\times 4=$ | $28 \\times 2=$ | $12 \\times 3=$ |\n| :--- | :--- | :--- | :--- |\n| $180 \\times 5=$ | $240 \\times 4=$ | $280 \\times 2=$ | $120 \\times 3=$ |", "options": [], "subject": "算术", "analysis": "$90 ; 96 ; 56 ; 36 ;$\n\n$900 ; 960 ; 560 ; 360 ;$"} {"id": "8413", "image": ["1312.jpg", "1313.jpg", "1312.jpg", "1313.jpg"], "answer": "$2888 ; 92 \\ldots . .6$\n\n$759 ; 83$\n\n【分析】两位数乘两位数的坚式计算方法: 先用下面因数个位的数去乘上面因数, 得数的末位和下面因数的个位对齐, 再用下面因数十位上的数去乘上面的因数, 得数的末位和下面因数的十位对齐, 然后把两次乘得的数加起来;\n\n三位数除以一位数的坚式计算方法: 从百位数开始除, 如果百位上的数不够除, 就用被除数前两位数除, 当除不尽有余数的时候, 把余数和被除数下一位上的数合在一起除以除数, 每次除得的余数必须比除数小;\n\n除法验算方法: 商 $\\times$ 除数 $=$ 被除数, 或者被除数 $\\div$ 商 $=$ 除数;\n\n有余数除法的验算方法: 商 $\\times$ 除数+余数=被除数。\n【详解】 $76 \\times 38=2888$\n$* 650 \\div 7=92 \\ldots . .6$\n\n$\\begin{array}{r}76 \\\\ \\times \\quad 38 \\\\ \\hline 608\\end{array}$\n\n\n\n$23 \\times 33=759$\n\n23
$\\begin{array}{r}\\times 3 \\\\ \\hline 69\\end{array}$\n\n$\\frac{69}{759}$\n\n\n\n$* 415 \\div 5=83$\n\n$5 \\longdiv { \\begin{array} { r r } { 8 } & { 3 } \\\\ { 4 } & { 1 5 } \\\\ { 4 } & { 0 } \\\\ { \\hline 1 5 } \\end{array} }$\n\n$\\frac{15}{0}$ 验算: $\\frac{\\times 15}{415}$", "solution": "null", "level": "三年级", "question": "列坚式计算, 带*的需要验算。\n\n$76 \\times 38=$\n\n$* 650 \\div 7=$\n\n$23 \\times 33=$\n\n* $415 \\div 5=$", "options": [], "subject": "算术", "analysis": "$2888 ; 92 \\ldots . .6$\n\n$759 ; 83$\n\n【分析】两位数乘两位数的坚式计算方法: 先用下面因数个位的数去乘上面因数, 得数的末位和下面因数的个位对齐, 再用下面因数十位上的数去乘上面的因数, 得数的末位和下面因数的十位对齐, 然后把两次乘得的数加起来;\n\n三位数除以一位数的坚式计算方法: 从百位数开始除, 如果百位上的数不够除, 就用被除数前两位数除, 当除不尽有余数的时候, 把余数和被除数下一位上的数合在一起除以除数, 每次除得的余数必须比除数小;\n\n除法验算方法: 商 $\\times$ 除数 $=$ 被除数, 或者被除数 $\\div$ 商 $=$ 除数;\n\n有余数除法的验算方法: 商 $\\times$ 除数+余数=被除数。\n【详解】 $76 \\times 38=2888$\n$* 650 \\div 7=92 \\ldots . .6$\n\n$\\begin{array}{r}76 \\\\ \\times \\quad 38 \\\\ \\hline 608\\end{array}$\n\n\n\n$23 \\times 33=759$\n\n23
$\\begin{array}{r}\\times 3 \\\\ \\hline 69\\end{array}$\n\n$\\frac{69}{759}$\n\n\n\n$* 415 \\div 5=83$\n\n$5 \\longdiv { \\begin{array} { r r } { 8 } & { 3 } \\\\ { 4 } & { 1 5 } \\\\ { 4 } & { 0 } \\\\ { \\hline 1 5 } \\end{array} }$\n\n$\\frac{15}{0}$ 验算: $\\frac{\\times 15}{415}$"} {"id": "8414", "image": [], "answer": "$315 ; 304$;\n\n$383 ; 2990$\n\n【分析】 $910-17 \\times 35$ 此题先算乘法, 再算减法。\n\n$16 \\times(57 \\div 3)$ 此题先算除法, 再算乘法。\n\n$48 \\div 6+375$ 此题先算除法, 再算加法。\n$5 \\times 13 \\times 46$ 此题应依次计算。\n\n【详解】 $910-17 \\times 35$\n\n$=910-595$\n\n$=315$\n\n$16 \\times(57 \\div 3)$\n\n$=16 \\times 19$\n\n$=304$\n\n$48 \\div 6+375$\n\n$=8+375$\n\n$=383$\n\n$5 \\times 13 \\times 46$\n\n$=65 \\times 46$\n\n$=2990$", "solution": "null", "level": "三年级", "question": "脱式计算。\n\n$910-17 \\times 35$\n\n$16 \\times(57 \\div 3)$\n\n$48 \\div 6+375$\n\n$5 \\times 13 \\times 46$", "options": [], "subject": "算术", "analysis": "$315 ; 304$;\n\n$383 ; 2990$\n\n【分析】 $910-17 \\times 35$ 此题先算乘法, 再算减法。\n\n$16 \\times(57 \\div 3)$ 此题先算除法, 再算乘法。\n\n$48 \\div 6+375$ 此题先算除法, 再算加法。\n$5 \\times 13 \\times 46$ 此题应依次计算。\n\n【详解】 $910-17 \\times 35$\n\n$=910-595$\n\n$=315$\n\n$16 \\times(57 \\div 3)$\n\n$=16 \\times 19$\n\n$=304$\n\n$48 \\div 6+375$\n\n$=8+375$\n\n$=383$\n\n$5 \\times 13 \\times 46$\n\n$=65 \\times 46$\n\n$=2990$"} {"id": "8439", "image": [], "answer": "$240 ; 2000 ; 24 ; 32 ;$\n\n42;70;500;46", "solution": "null", "level": "三年级", "question": "直接写得数。\n\n| $480 \\div 2=$ | $20 \\times 100=$ | $12 \\times 2=$ | $96 \\div 3=$ |\n| :--- | :--- | :--- | :--- |\n| $14 \\times 3=$ | $7 \\times 10=$ | $3000 \\div 6=$ | $23 \\times 2=$ |\n\n21._列坚式计算。(带 $\\boldsymbol{k}$ 要验算)。\n\n$43 \\times 21=32 \\times 30=\\star 37 \\times 52=$", "options": [], "subject": "算术", "analysis": "$240 ; 2000 ; 24 ; 32 ;$\n\n42;70;500;46"} {"id": "8440", "image": [], "answer": "240; 4332; 2700;\n\n【分析】 $14 \\times 37-278$ 此题先算乘法, 再算减法;\n\n57× (304;4) 此题先算除法, 再算乘法;\n\n$(25+35) \\times 45$ 此题先算加法, 再算乘法。\n\n【详解】 $14 \\times 37-278$\n\n$=518-278$\n\n$=240$\n\n$57 \\times(304 \\div 4)$\n\n$=57 \\times 76$\n\n$=4332$\n\n$(25+35) \\times 45$\n\n$=60 \\times 45$\n\n$=2700$", "solution": "null", "level": "三年级", "question": "脱式计算。\n\n$14 \\times 37-278$\n\n$57 \\times(304 \\div 4)$\n\n$(25+35) \\times 45$", "options": [], "subject": "算术", "analysis": "240; 4332; 2700;\n\n【分析】 $14 \\times 37-278$ 此题先算乘法, 再算减法;\n\n57× (304;4) 此题先算除法, 再算乘法;\n\n$(25+35) \\times 45$ 此题先算加法, 再算乘法。\n\n【详解】 $14 \\times 37-278$\n\n$=518-278$\n\n$=240$\n\n$57 \\times(304 \\div 4)$\n\n$=57 \\times 76$\n\n$=4332$\n\n$(25+35) \\times 45$\n\n$=60 \\times 45$\n\n$=2700$"} {"id": "8465", "image": [], "answer": "$200 ; 0 ; 360$\n\n$800 ; 260 ; 21$\n\n【详解】略", "solution": "null", "level": "三年级", "question": "直接写出得数。\n\n| $800 \\div 4=$ | $0 \\div 2=$ | $120 \\times 3=$ |\n| :--- | :--- | ---: |\n| $4800 \\div 6=$ | $13 \\times 20=$ | $84 \\div 4=$ |", "options": [], "subject": "算术", "analysis": "$200 ; 0 ; 360$\n\n$800 ; 260 ; 21$\n\n【详解】略"} {"id": "8466", "image": ["1325.jpg", "1326.jpg", "1327.jpg", "1328.jpg", "1325.jpg", "1326.jpg", "1327.jpg", "1328.jpg"], "answer": "$416 ; 1600 ; 1222$\n\n$61 ; 156 ; 210 \\ldots . .1$\n\n【分析】两位数乘两位数时, 先用第二个因数的个位上的数分别与第一个因数的每一位数相乘, 乘得结果要与个位对齐, 再用第二个因数的十位上的数分别与第一个因数的每一位数相乘, 乘得结果要与十位对齐, 然后两个结果相加就得到两位数乘两位数的结果。\n\n乘法验算时, 交换两个因数的位置, 看积是不是相等。\n\n除数是一位数的除法计算时, 从被除数的最高位除起, 每次先用除数试除被除数的前一位数, 如果它比除数小, 再试除前两位数。除到被除数的哪一位, 就把商写在那一位上面。每求出一位商, 余下的数必须比除数小。\n除法验算时用商乘除数看是不是等于被除数; 有余数时, 用商乘除数加上余数看是不是等于被除数。\n\n【详解】 $32 \\times 13=416$ $64 \\times 25=1600$\n\n$\\begin{array}{r}32 \\\\ \\times \\quad 13 \\\\ \\hline 96\\end{array}$\n\n$\\begin{array}{r}64 \\\\ \\times \\quad 25 \\\\ \\hline 320\\end{array}$\n\n| 32 |\n| :--- | :--- |\n| 416 |\n\n\n\n$\\star 26 \\times 47=1222$\n\n\n\n\n\n$305 \\div 5=61$\n\n$624 \\div 4=156$\n\n$5 \\longdiv { 3 6 1 }$\n\n$4 \\longdiv { 1 5 6 }$\n\n| 4 |\n| :--- | :--- |\n| 22 |\n\n\n| 20 |\n| :--- | :--- |\n| 24 |\n\n$\\begin{array}{r}24 \\\\ \\hline 0\\end{array}$\n\n$\\star 631 \\div 3=210$.\n\n.1\n\n$3 \\begin{array}{r}21 \\\\ 3 \\longdiv { 6 3 1 }\\end{array}$\n\n6\n\n3 验算: 630\n\n$\\frac{3}{1}$\n\n", "solution": "null", "level": "三年级", "question": "笔算下面各题(带 $\\star$ 的要验算)。\n\n$32 \\times 13=\\quad 64 \\times 25=\\quad \\star 26 \\times 47=$\n\n$305 \\div 5=\\quad 624 \\div 4=\\quad \\star 631 \\div 3=$", "options": [], "subject": "算术", "analysis": "$416 ; 1600 ; 1222$\n\n$61 ; 156 ; 210 \\ldots . .1$\n\n【分析】两位数乘两位数时, 先用第二个因数的个位上的数分别与第一个因数的每一位数相乘, 乘得结果要与个位对齐, 再用第二个因数的十位上的数分别与第一个因数的每一位数相乘, 乘得结果要与十位对齐, 然后两个结果相加就得到两位数乘两位数的结果。\n\n乘法验算时, 交换两个因数的位置, 看积是不是相等。\n\n除数是一位数的除法计算时, 从被除数的最高位除起, 每次先用除数试除被除数的前一位数, 如果它比除数小, 再试除前两位数。除到被除数的哪一位, 就把商写在那一位上面。每求出一位商, 余下的数必须比除数小。\n除法验算时用商乘除数看是不是等于被除数; 有余数时, 用商乘除数加上余数看是不是等于被除数。\n\n【详解】 $32 \\times 13=416$ $64 \\times 25=1600$\n\n$\\begin{array}{r}32 \\\\ \\times \\quad 13 \\\\ \\hline 96\\end{array}$\n\n$\\begin{array}{r}64 \\\\ \\times \\quad 25 \\\\ \\hline 320\\end{array}$\n\n| 32 |\n| :--- | :--- |\n| 416 |\n\n\n\n$\\star 26 \\times 47=1222$\n\n\n\n\n\n$305 \\div 5=61$\n\n$624 \\div 4=156$\n\n$5 \\longdiv { 3 6 1 }$\n\n$4 \\longdiv { 1 5 6 }$\n\n| 4 |\n| :--- | :--- |\n| 22 |\n\n\n| 20 |\n| :--- | :--- |\n| 24 |\n\n$\\begin{array}{r}24 \\\\ \\hline 0\\end{array}$\n\n$\\star 631 \\div 3=210$.\n\n.1\n\n$3 \\begin{array}{r}21 \\\\ 3 \\longdiv { 6 3 1 }\\end{array}$\n\n6\n\n3 验算: 630\n\n$\\frac{3}{1}$\n\n"} {"id": "8467", "image": [], "answer": "$955 ; 107 ; 90$;\n\n【分析】 $78 \\times 21-683$ 此题先算乘法, 再算减法;\n\n(855-106) $\\div 7$ 此题先算减法, 再算除法;\n\n$25 \\times 18 \\div 5$ 此题先算乘法, 再算除法。\n\n【详解】 $78 \\times 21-683$\n\n$=1638-683$\n\n$=955$\n$(855-106) \\div 7$\n\n$=749 \\div 7$\n\n$=107$\n\n$25 \\times 18 \\div 5$\n\n$=450 \\div 5$\n\n$=90$", "solution": "null", "level": "三年级", "question": "脱式计算。\n$78 \\times 21-683$\n$(855-106) \\div 7$\n$25 \\times 18 \\div 5$", "options": [], "subject": "算术", "analysis": "$955 ; 107 ; 90$;\n\n【分析】 $78 \\times 21-683$ 此题先算乘法, 再算减法;\n\n(855-106) $\\div 7$ 此题先算减法, 再算除法;\n\n$25 \\times 18 \\div 5$ 此题先算乘法, 再算除法。\n\n【详解】 $78 \\times 21-683$\n\n$=1638-683$\n\n$=955$\n$(855-106) \\div 7$\n\n$=749 \\div 7$\n\n$=107$\n\n$25 \\times 18 \\div 5$\n\n$=450 \\div 5$\n\n$=90$"} {"id": "8488", "image": [], "answer": "$20 ; 0 ; 100 ; 360 ; 1200$;\n\n$101 ; 320 ; 120 ; 2000 ; 90 ;$", "solution": "null", "level": "三年级", "question": "口算。\n\n| $120 \\div 6=$ | $0 \\div 55=$ | $300 \\div 3=$ | $12 \\times 30=$ | $29 \\times 39 \\approx$ |\n| :--- | :--- | ---: | :--- | :--- |\n| $505 \\div 5=$ | $960 \\div 3=$ | $24 \\times 5=$ | $25 \\times 80=$ | $816 \\div 9 \\approx$ |", "options": [], "subject": "算术", "analysis": "$20 ; 0 ; 100 ; 360 ; 1200$;\n\n$101 ; 320 ; 120 ; 2000 ; 90 ;$"} {"id": "8489", "image": ["1331.jpg", "1332.jpg", "1331.jpg", "1332.jpg"], "answer": "$782 ; 702 ; 1053 ; 476$\n\n【分析】两位数乘两位数的笔算方法: 先用第二个因数个位上的数依次去乘第一个因数, 积的末尾与个位对齐; 再用第二个因数十位上的数依次去乘第一个因数, 积的末尾与十位对齐; 最后把两次乘得的积加起来, 据此解答即可。\n【详解】 $23 \\times 34=782$\n$54 \\times 13=702$\n$39 \\times 27=1053$\n$17 \\times 28=476$\n\n\n\n$\\frac{69}{782}$\n54\n\n\n$\\frac{78}{1053}$\n17\n\n| $\\times \\quad 28$ |\n| ---: | ---: |\n| 136 |\n\n$\\frac{34}{476}$", "solution": "null", "level": "三年级", "question": "列坚式计算。\n\n$23 \\times 34=$\n\n$54 \\times 13=$\n\n$39 \\times 27=$\n\n$17 \\times 28=$", "options": [], "subject": "算术", "analysis": "$782 ; 702 ; 1053 ; 476$\n\n【分析】两位数乘两位数的笔算方法: 先用第二个因数个位上的数依次去乘第一个因数, 积的末尾与个位对齐; 再用第二个因数十位上的数依次去乘第一个因数, 积的末尾与十位对齐; 最后把两次乘得的积加起来, 据此解答即可。\n【详解】 $23 \\times 34=782$\n$54 \\times 13=702$\n$39 \\times 27=1053$\n$17 \\times 28=476$\n\n\n\n$\\frac{69}{782}$\n54\n\n\n$\\frac{78}{1053}$\n17\n\n| $\\times \\quad 28$ |\n| ---: | ---: |\n| 136 |\n\n$\\frac{34}{476}$"} {"id": "8515", "image": [], "answer": "$80 ; 500 ; 0 ; 90 ; 300$\n\n$2800 ; 840 ; 60 ; 90 ; 80$", "solution": "null", "level": "三年级", "question": "直接写出得数。\n\n| $560 \\div 7=$ | $3000 \\div 6=$ | $0 \\div 39=$ | $810 \\div 9=$ | $15 \\times 20=$ |\n| :--- | ---: | :--- | :--- | :--- |\n| $40 \\times 70=$ | $20 \\times 42=$ | $300 \\div 5=$ | $181 \\div 2 \\approx$ | $323 \\div 4 \\approx$ |", "options": [], "subject": "算术", "analysis": "$80 ; 500 ; 0 ; 90 ; 300$\n\n$2800 ; 840 ; 60 ; 90 ; 80$"} {"id": "8516", "image": ["1336.jpg", "1337.jpg", "1336.jpg", "1337.jpg"], "answer": "$650 ; 102 ;$\n\n2491;66\n\n【分析】两位数乘两位数, 先用第二个乘数的个位上的数去乘第一个乘数, 得数的末尾和个位对齐,再用第二个乘数的十位去乘第一个乘数, 得数的末尾和十位对齐, 然后将两次的积相加即可; 三位数除以一位数, 先用被除数的百位上的数去除, 如果百位上的数大于除数, 商就在百位, 如果小于, 商就在十位, 每次所得的余数一定要比除数小; 据此解答。\n\n【详解】 $26 \\times 25=650$\n\n$$\n408 \\div 4=102\n$$\n\n| 26 | |\n| ---: | ---: |\n| $\\times \\quad 25$ | |\n| 130 | |\n| 5 | 2 |\n| 650 | |\n\n$47 \\times 53=2491$\n\n| | 47 | |\n| ---: | ---: | ---: |\n| $\\times \\quad 5$ | 3 | |\n| | 14 | 4 |\n| 2 | 3 | 5 |\n| 2 | 4 | 91 |\n\n\n\n$528 \\div 8=66$\n\n", "solution": "null", "level": "三年级", "question": "用坚式计算。\n\n$26 \\times 25=408 \\div 4=$\n\n$47 \\times 53=\\quad 528 \\div 8=$", "options": [], "subject": "算术", "analysis": "$650 ; 102 ;$\n\n2491;66\n\n【分析】两位数乘两位数, 先用第二个乘数的个位上的数去乘第一个乘数, 得数的末尾和个位对齐,再用第二个乘数的十位去乘第一个乘数, 得数的末尾和十位对齐, 然后将两次的积相加即可; 三位数除以一位数, 先用被除数的百位上的数去除, 如果百位上的数大于除数, 商就在百位, 如果小于, 商就在十位, 每次所得的余数一定要比除数小; 据此解答。\n\n【详解】 $26 \\times 25=650$\n\n$$\n408 \\div 4=102\n$$\n\n| 26 | |\n| ---: | ---: |\n| $\\times \\quad 25$ | |\n| 130 | |\n| 5 | 2 |\n| 650 | |\n\n$47 \\times 53=2491$\n\n| | 47 | |\n| ---: | ---: | ---: |\n| $\\times \\quad 5$ | 3 | |\n| | 14 | 4 |\n| 2 | 3 | 5 |\n| 2 | 4 | 91 |\n\n\n\n$528 \\div 8=66$\n\n"} {"id": "8517", "image": [], "answer": "$315 ; 62$\n\n【分析】(1)先算乘法, 再算减法; (2)先算加法, 再算除法。\n【详解】 $910-17 \\times 35$\n\n$=910-595$\n\n$=315$\n\n$(263+109) \\div 6$\n\n$=372 \\div 6$\n\n$=62$", "solution": "null", "level": "三年级", "question": "脱式计算。\n\n$910-17 \\times 35$\n\n$$\n(263+109) \\div 6\n$$", "options": [], "subject": "算术", "analysis": "$315 ; 62$\n\n【分析】(1)先算乘法, 再算减法; (2)先算加法, 再算除法。\n【详解】 $910-17 \\times 35$\n\n$=910-595$\n\n$=315$\n\n$(263+109) \\div 6$\n\n$=372 \\div 6$\n\n$=62$"} {"id": "8542", "image": [], "answer": "$4000 ; 80 ; 40 ; 50$", "solution": "null", "level": "三年级", "question": "直接写出得数。\n$80 \\times 50=$\n$560 \\div 7=$\n$356 \\div 9 \\approx$\n$203 \\div 4 \\approx$", "options": [], "subject": "算术", "analysis": "$4000 ; 80 ; 40 ; 50$"} {"id": "8543", "image": ["1338.jpg", "1338.jpg"], "answer": "$416 ; 1600 ; 1222$\n\n$61 ; 156 ; 210 \\ldots . .1$\n\n【分析】两位数乘两位数时, 先用第二个因数的个位上的数分别与第一个因数的每一位数相乘, 乘得结果要与个位对齐, 再用第二个因数的十位上的数分别与第一个因数的每一位数相乘, 乘得结果要与十位对齐, 然后两个结果相加就得到两位数乘两位数的结果。\n\n乘法验算时, 交换两个因数的位置, 看积是不是相等。\n\n除数是一位数的除法计算时, 从被除数的最高位除起, 每次先用除数试除被除数的前一位数, 如果它比除数小, 再试除前两位数。除到被除数的哪一位, 就把商写在那一位上面。每求出一位商, 余下的数必须比除数小。\n\n除法验算时用商乘除数看是不是等于被除数; 有余数时, 用商乘除数加上余数看是不是等于被除数。\n\n【详解】 $32 \\times 13=416$ $64 \\times 25=1600$\n$\\begin{array}{r}32 \\\\ \\times \\quad 13 \\\\ \\hline 96\\end{array}$\n\n| 3 | 2 |\n| :--- | :--- |\n| 4 | 16 |\n\n$\\begin{array}{r}64 \\\\ \\times \\quad 25 \\\\ \\hline 320\\end{array}$\n\n| 1 | 2 | 8 | |\n| :--- | :--- | :--- | :--- |\n| 1 | 6 | 0 | 0 |\n\n$\\star 26 \\times 47=1222$\n\n\n\n\\$\\begin{array}{lll}1 \\& 0 \\& 4
\n\n1222 \\& 22\\end{array} \\quad\\$| 94 |\n| :--- |\n| 1222 |\n\n$305 \\div 5=61$\n\n$624 \\div 4=156$\n\n$5 \\longdiv { 3 \\quad 6 \\quad }$\n\n$4 \\begin{array}{r}156 \\\\ 624\\end{array}$\n\n4\n\n$3 \\quad 0$\n\n5\n5\n\n0 $\\quad$\\begin{tabular}{rr}\n20
\n\\hline\n\n$\\quad$\n\n2 \\& 2
\n24
\n\\hline \\& 0\n\\end{tabular}\n\n$\\star 631 \\div 3=210$.\n\n1\n\n$3 \\begin{array}{rrr}2 & 1 & 0 \\\\ 6 & 3 & \\end{array}$\n\n$\\frac{6}{3}$ 验算: $\\frac{\\times 3}{630}$\n\n$\\frac{3}{1} \\quad \\frac{+\\quad 1}{631}$", "solution": "null", "level": "三年级", "question": "笔算下面各题(带 $\\boldsymbol{n}$ 的要验算)。\n\n$32 \\times 13=$\n\n$64 \\times 25=$\n\n$\\star 26 \\times 47=$\n$305 \\div 5=\\quad 624 \\div 4=\\quad \\star 631 \\div 3=$", "options": [], "subject": "算术", "analysis": "$416 ; 1600 ; 1222$\n\n$61 ; 156 ; 210 \\ldots . .1$\n\n【分析】两位数乘两位数时, 先用第二个因数的个位上的数分别与第一个因数的每一位数相乘, 乘得结果要与个位对齐, 再用第二个因数的十位上的数分别与第一个因数的每一位数相乘, 乘得结果要与十位对齐, 然后两个结果相加就得到两位数乘两位数的结果。\n\n乘法验算时, 交换两个因数的位置, 看积是不是相等。\n\n除数是一位数的除法计算时, 从被除数的最高位除起, 每次先用除数试除被除数的前一位数, 如果它比除数小, 再试除前两位数。除到被除数的哪一位, 就把商写在那一位上面。每求出一位商, 余下的数必须比除数小。\n\n除法验算时用商乘除数看是不是等于被除数; 有余数时, 用商乘除数加上余数看是不是等于被除数。\n\n【详解】 $32 \\times 13=416$ $64 \\times 25=1600$\n$\\begin{array}{r}32 \\\\ \\times \\quad 13 \\\\ \\hline 96\\end{array}$\n\n| 3 | 2 |\n| :--- | :--- |\n| 4 | 16 |\n\n$\\begin{array}{r}64 \\\\ \\times \\quad 25 \\\\ \\hline 320\\end{array}$\n\n| 1 | 2 | 8 | |\n| :--- | :--- | :--- | :--- |\n| 1 | 6 | 0 | 0 |\n\n$\\star 26 \\times 47=1222$\n\n\n\n\\$\\begin{array}{lll}1 \\& 0 \\& 4
\n\n1222 \\& 22\\end{array} \\quad\\$| 94 |\n| :--- |\n| 1222 |\n\n$305 \\div 5=61$\n\n$624 \\div 4=156$\n\n$5 \\longdiv { 3 \\quad 6 \\quad }$\n\n$4 \\begin{array}{r}156 \\\\ 624\\end{array}$\n\n4\n\n$3 \\quad 0$\n\n5\n5\n\n0 $\\quad$\\begin{tabular}{rr}\n20
\n\\hline\n\n$\\quad$\n\n2 \\& 2
\n24
\n\\hline \\& 0\n\\end{tabular}\n\n$\\star 631 \\div 3=210$.\n\n1\n\n$3 \\begin{array}{rrr}2 & 1 & 0 \\\\ 6 & 3 & \\end{array}$\n\n$\\frac{6}{3}$ 验算: $\\frac{\\times 3}{630}$\n\n$\\frac{3}{1} \\quad \\frac{+\\quad 1}{631}$"} {"id": "8568", "image": [], "answer": "$5 ; 40 ; 140 ; 53$\n\n$69 ; 64 ; 23 ; 0$", "solution": "null", "level": "三年级", "question": "口算。\n\n| $25 \\div 5=$ | $240 \\div 6=$ | $28 \\times 5=$ | $98-45=$ |\n| :--- | :--- | :---: | :---: |\n| $23 \\times 3=$ | $46+18=$ | $92 \\div 4=$ | $0 \\div 7=$ |", "options": [], "subject": "算术", "analysis": "$5 ; 40 ; 140 ; 53$\n\n$69 ; 64 ; 23 ; 0$"} {"id": "8569", "image": ["1344.jpg", "1345.jpg", "1344.jpg", "1345.jpg"], "answer": "$231 ; 137 ; 118$\n\n【分析】笔算除数是一位数的除法, 先用除数去除被除数的首位, 如果不够除, 再去除被除数的前两位数, 除到哪位商就写在那位上, 除的过程中要注意, 余数总比除数小。没有余数的除法可以用商 $\\times$除数=被除数来验算。\n\n【详解】 $693 \\div 3=231$\n\n$274 \\div 2=137$\n\nॐ $472 \\div 4=118$\n$3 \\begin{gathered}231 \\\\ 693\\end{gathered}$\n6 9\n$2 \\longdiv { 1 3 7 }$\n$\\frac{9}{3}$\n2\n7\n$4 \\begin{array}{r}118 \\\\ 4 \\quad 72\\end{array}$\n$\\frac{6}{14}$\n4\n\n\n$\\begin{array}{lll}1 & 1 & 8\\end{array}$\n验算:\n\n| 3 |\n| :--- |\n| 0 |\n\n\n| $1 \\quad 4$ | |\n| ---: | ---: |\n| | 0 |\n\n", "solution": "null", "level": "三年级", "question": "用坚式计算, 带的的要验算。\n\n$693 \\div 3$\n\n$274 \\div 2$\n\nऊ $472 \\div 4$", "options": [], "subject": "算术", "analysis": "$231 ; 137 ; 118$\n\n【分析】笔算除数是一位数的除法, 先用除数去除被除数的首位, 如果不够除, 再去除被除数的前两位数, 除到哪位商就写在那位上, 除的过程中要注意, 余数总比除数小。没有余数的除法可以用商 $\\times$除数=被除数来验算。\n\n【详解】 $693 \\div 3=231$\n\n$274 \\div 2=137$\n\nॐ $472 \\div 4=118$\n$3 \\begin{gathered}231 \\\\ 693\\end{gathered}$\n6 9\n$2 \\longdiv { 1 3 7 }$\n$\\frac{9}{3}$\n2\n7\n$4 \\begin{array}{r}118 \\\\ 4 \\quad 72\\end{array}$\n$\\frac{6}{14}$\n4\n\n\n$\\begin{array}{lll}1 & 1 & 8\\end{array}$\n验算:\n\n| 3 |\n| :--- |\n| 0 |\n\n\n| $1 \\quad 4$ | |\n| ---: | ---: |\n| | 0 |\n\n"} {"id": "8570", "image": [], "answer": "$81 ; 251 ; 70$\n\n【分析】 $114-99 \\div 3$ 此题先算除法, 再算减法;\n\n$136+23 \\times 5$ 此题先算乘法, 再算加法;\n\n$630 \\div(75-66)$ 此题先算减法, 再算除法。\n\n【详解】 $114-99 \\div 3$\n\n$=114-33$\n\n$=81$\n$136+23 \\times 5$\n\n$=136+115$\n\n$=251$\n\n$630 \\div(75-66)$\n\n$=630 \\div 9$\n\n$=70$", "solution": "null", "level": "三年级", "question": "脱式计算。\n\n$114-99 \\div 3 \\quad 136+23 \\times 5 \\quad 630 \\div(75-66)$", "options": [], "subject": "算术", "analysis": "$81 ; 251 ; 70$\n\n【分析】 $114-99 \\div 3$ 此题先算除法, 再算减法;\n\n$136+23 \\times 5$ 此题先算乘法, 再算加法;\n\n$630 \\div(75-66)$ 此题先算减法, 再算除法。\n\n【详解】 $114-99 \\div 3$\n\n$=114-33$\n\n$=81$\n$136+23 \\times 5$\n\n$=136+115$\n\n$=251$\n\n$630 \\div(75-66)$\n\n$=630 \\div 9$\n\n$=70$"} {"id": "8595", "image": [], "answer": "$90 ; 400 ; 450 ; 90$\n\n$91 ; 320 ; 0 ; 80$\n\n【详解】1略", "solution": "null", "level": "三年级", "question": "口算。\n\n| $270 \\div 3=$ | $3200 \\div 8=$ | $30 \\times 15=$ | $809 \\div 9 \\approx$ |\n| :--- | :--- | :--- | :--- |\n| $637 \\div 7=$ | $160 \\times 2=$ | $0 \\div 8=$ | $329 \\div 4 \\approx$ |", "options": [], "subject": "算术", "analysis": "$90 ; 400 ; 450 ; 90$\n\n$91 ; 320 ; 0 ; 80$\n\n【详解】1略"} {"id": "8596", "image": ["1350.jpg", "1350.jpg"], "answer": "2610, 1472;\n\n5850,1541 。\n\n【分析】笔算两位数乘两位数的进位乘法时, 用第二个乘数每一位上的数分别去乘第一个乘数, 用哪一位上的数去乘, 积的末尾就和那一位对齐, 哪一位乘得的积满几十, 就要向前一位进几, 再把两次乘得的积相加。\n\n乘法算式的验算方法有: 方法一, 交换式中因数的位置进行验算; 方法二, 根据积 $=$ 其中一个因数 $=$另一个因数进行验算。\n\n【详解】 $45 \\times 58=2610 \\quad \\times 32 \\times 46=1472$\n\n\n\n| 22 | 5 | | 2 | 8 | |\n| :---: | :---: | :---: | :---: | :---: | :---: |\n| | 10 | | | | |\n\n$90 \\times 65=5850 \\quad ※ 67 \\times 23=1541$\n\n| | | 0 | | | | 7 | | | 2 |\n| :---: | :---: | :---: | :---: | :---: | :---: | :---: | :---: | :---: | :---: |\n| | $k 6$ | 5 | | $x$ | | 3 | | $x$ | 6 |\n| | 45 | 0 | | 2 | D | 1验算: | | 1 | 6 |\n| 5 | 4 | | 1 | 3 | | | 1 | 3 | 8 |\n| | 8 | $\\overline{0}$ | 1 | 5 | 4 | 1 | | 5 | 4 |", "solution": "null", "level": "三年级", "question": "用坚式计算, 带 $※$ 的要验算。\n\n$45 \\times 58=\\quad ※ 32 \\times 46=\\quad 90 \\times 65=\\quad ※ 67 \\times 23=$", "options": [], "subject": "算术", "analysis": "2610, 1472;\n\n5850,1541 。\n\n【分析】笔算两位数乘两位数的进位乘法时, 用第二个乘数每一位上的数分别去乘第一个乘数, 用哪一位上的数去乘, 积的末尾就和那一位对齐, 哪一位乘得的积满几十, 就要向前一位进几, 再把两次乘得的积相加。\n\n乘法算式的验算方法有: 方法一, 交换式中因数的位置进行验算; 方法二, 根据积 $=$ 其中一个因数 $=$另一个因数进行验算。\n\n【详解】 $45 \\times 58=2610 \\quad \\times 32 \\times 46=1472$\n\n\n\n| 22 | 5 | | 2 | 8 | |\n| :---: | :---: | :---: | :---: | :---: | :---: |\n| | 10 | | | | |\n\n$90 \\times 65=5850 \\quad ※ 67 \\times 23=1541$\n\n| | | 0 | | | | 7 | | | 2 |\n| :---: | :---: | :---: | :---: | :---: | :---: | :---: | :---: | :---: | :---: |\n| | $k 6$ | 5 | | $x$ | | 3 | | $x$ | 6 |\n| | 45 | 0 | | 2 | D | 1验算: | | 1 | 6 |\n| 5 | 4 | | 1 | 3 | | | 1 | 3 | 8 |\n| | 8 | $\\overline{0}$ | 1 | 5 | 4 | 1 | | 5 | 4 |"} {"id": "8597", "image": [], "answer": "$714 ; 566 ; 208$\n\n【分析】按照四则混合运算的顺序, 先算乘除法, 再算加减法, 有括号的要先算括号里的。\n\n【详解】 $68 \\div 2 \\times 21$\n\n$=34 \\times 21$\n\n$=714$\n\n$16+22 \\times 25$\n\n$=16+550$\n\n$=566$\n\n$16 \\times(65 \\div 5)$\n\n$=16 \\times 13$\n\n$=208$", "solution": "null", "level": "三年级", "question": "脱式计算。\n\n$68 \\div 2 \\times 21$\n\n$16+22 \\times 25$\n\n$16 \\times(65 \\div 5)$", "options": [], "subject": "算术", "analysis": "$714 ; 566 ; 208$\n\n【分析】按照四则混合运算的顺序, 先算乘除法, 再算加减法, 有括号的要先算括号里的。\n\n【详解】 $68 \\div 2 \\times 21$\n\n$=34 \\times 21$\n\n$=714$\n\n$16+22 \\times 25$\n\n$=16+550$\n\n$=566$\n\n$16 \\times(65 \\div 5)$\n\n$=16 \\times 13$\n\n$=208$"} {"id": "9052", "image": ["1456.jpg", "1457.jpg", "1458.jpg", "1458.jpg"], "answer": "解\n\n\n\n略", "solution": "null", "level": "三年级", "question": "稌一涂。\n\n\n0.23\n\n", "options": [], "subject": "组合数学", "analysis": "解\n\n\n\n略"} {"id": "9081", "image": [], "answer": "解6 个, $\\frac{1}{3}, \\frac{1}{5}, \\frac{1}{7}, \\frac{3}{5}, \\frac{3}{7}, \\frac{5}{7}$\n\n【解析】\n\n【分析】\n\n分子要小于分母, 按一定顺序排列, 做到不重复、不遗漏。当 1 作为分子时, 分母可以是 3 、 5 和 7, 三种情况。当 3 作为分子时, 分母可以是 5 和 7 , 两种情况。当 5 作为分子时, 分母可以是 7, 一种情况。一共有 6 种情况。\n\n【详解】\n\n根据分析可知, 从 $1 、 3 、 5 、 7$ 中选出两个数组成一个分数 (分子要小于分母), 一共能组成 6 个, 分别是 $\\frac{1}{3}, \\frac{1}{5}, \\frac{1}{7}, \\frac{3}{5}, \\frac{3}{7}, \\frac{5}{7}$ 。\n\n答: 一共能组成 6 个, 分别是 $\\frac{1}{3}, \\frac{1}{5}, \\frac{1}{7}, \\frac{3}{5}, \\frac{3}{7}, \\frac{5}{7}$ 。\n\n【点睛】\n\n写出的的分数一定要分子小于分母, 这是解答本题的关键。", "solution": "null", "level": "三年级", "question": "从 $1 、 3 、 5 、 7$ 中选出两个数组成一个分数(分子要小于分母),一共能组成多少个?请把这些分数都写出来。", "options": [], "subject": "组合数学", "analysis": "解6 个, $\\frac{1}{3}, \\frac{1}{5}, \\frac{1}{7}, \\frac{3}{5}, \\frac{3}{7}, \\frac{5}{7}$\n\n【解析】\n\n【分析】\n\n分子要小于分母, 按一定顺序排列, 做到不重复、不遗漏。当 1 作为分子时, 分母可以是 3 、 5 和 7, 三种情况。当 3 作为分子时, 分母可以是 5 和 7 , 两种情况。当 5 作为分子时, 分母可以是 7, 一种情况。一共有 6 种情况。\n\n【详解】\n\n根据分析可知, 从 $1 、 3 、 5 、 7$ 中选出两个数组成一个分数 (分子要小于分母), 一共能组成 6 个, 分别是 $\\frac{1}{3}, \\frac{1}{5}, \\frac{1}{7}, \\frac{3}{5}, \\frac{3}{7}, \\frac{5}{7}$ 。\n\n答: 一共能组成 6 个, 分别是 $\\frac{1}{3}, \\frac{1}{5}, \\frac{1}{7}, \\frac{3}{5}, \\frac{3}{7}, \\frac{5}{7}$ 。\n\n【点睛】\n\n写出的的分数一定要分子小于分母, 这是解答本题的关键。"} {"id": "9083", "image": [], "answer": "解132 封信; 66 次电话\n\n【解析】\n\n【分析】\n\n12 位朋友互通一封信, 相当于是双循环比赛, 互通一次电话相当于是单循环比赛。\n\n【详解】\n\n$12 \\times(12-1)$\n\n$=12 \\times 11$\n\n$=132$\n\n$12 \\times(12-1) \\div 2$\n\n$=12 \\times 11 \\div 2$\n\n$=66$\n\n答: 互通一封信需写 132 封信; 互通一次电话需打 66 次电话。\n\n【点睛】\n\n$\\mathrm{n}$ 的选手进行比赛, 单循环比赛场次 $n \\times(n-1) \\div 2$, 双循环比赛场次 $n \\times(n-1)$ 。", "solution": "null", "level": "三年级", "question": "12 位朋友互通一封信需写多少封信?他们互通一次电话需打多少次电话?", "options": [], "subject": "计数", "analysis": "解132 封信; 66 次电话\n\n【解析】\n\n【分析】\n\n12 位朋友互通一封信, 相当于是双循环比赛, 互通一次电话相当于是单循环比赛。\n\n【详解】\n\n$12 \\times(12-1)$\n\n$=12 \\times 11$\n\n$=132$\n\n$12 \\times(12-1) \\div 2$\n\n$=12 \\times 11 \\div 2$\n\n$=66$\n\n答: 互通一封信需写 132 封信; 互通一次电话需打 66 次电话。\n\n【点睛】\n\n$\\mathrm{n}$ 的选手进行比赛, 单循环比赛场次 $n \\times(n-1) \\div 2$, 双循环比赛场次 $n \\times(n-1)$ 。"} {"id": "9104", "image": [], "answer": "解(1) $783 ; 107$;\n\n(2)一年级一共有多少人?230 人;(答案不唯一)\n\n【分析】(1) 直接将每个年级的男生人数加起来即可。通过统计表即可知道三年级的女生有多少人,依此解答。\n\n(2)根据统计表中的信息提出问题并解答, 符合题意即可。例如: 一年级一共有多少人? 计算时用一年级的男生人数加女生人数即可。\n\n【详解】 (1) $120+125+118+130+140+150$\n\n$=245+118+130+140+150$\n\n$=363+130+140+150$\n\n$=493+140+150$\n\n$=633+150$\n$=783$ (人)\n\n即全校男生有 783 人; 通过统计表可知, 三年级女生有 107 人。\n\n(2) 一年级一共有多少人?\n\n$120+110=230$ (人)\n\n答: 一年级一共有 230 人。\n\n【点睛】熟练掌握复式统计表的特点是解答此题的关键。", "solution": "null", "level": "三年级", "question": "实验小学各年级学生人数统计如下:\n\n| 人 | 年
性 | 一年级 | 二年级 | 三年级 | 四年级 | 五年级 | 六年级 |\n| :---: | :---: | :---: | :---: | :---: | :---: | :---: | :---: |\n| 性 | 别 | 120 | 125 | 118 | 130 | 140 | 150 |\n| 女 | 生 | 110 | 105 | 107 | 108 | 113 | 110 |\n\n(1)全校男生有()人,三年级女生有()人。\n\n(2) 提出其它数学问题并解答。", "options": [], "subject": "计数", "analysis": "解(1) $783 ; 107$;\n\n(2)一年级一共有多少人?230 人;(答案不唯一)\n\n【分析】(1) 直接将每个年级的男生人数加起来即可。通过统计表即可知道三年级的女生有多少人,依此解答。\n\n(2)根据统计表中的信息提出问题并解答, 符合题意即可。例如: 一年级一共有多少人? 计算时用一年级的男生人数加女生人数即可。\n\n【详解】 (1) $120+125+118+130+140+150$\n\n$=245+118+130+140+150$\n\n$=363+130+140+150$\n\n$=493+140+150$\n\n$=633+150$\n$=783$ (人)\n\n即全校男生有 783 人; 通过统计表可知, 三年级女生有 107 人。\n\n(2) 一年级一共有多少人?\n\n$120+110=230$ (人)\n\n答: 一年级一共有 230 人。\n\n【点睛】熟练掌握复式统计表的特点是解答此题的关键。"} {"id": "9175", "image": ["1502.jpg", "1503.jpg", "1503.jpg"], "answer": "解见详解。\n\n【分析】依据地图上的方向别别方法, 即“上北下南, 左西右东”,以及题目所给的各种信息, 即可进行解答。\n\n【详解】\n\n\n\n【点睛】本题考查方向与位置的实际应用能力。", "solution": "null", "level": "三年级", "question": "这次客人们到学校里来参观科技成果展, 请你把科教楼的展厅平面图补充完整。展厅的入口在东南角; 出口在西南角; 声光科技厅在花坛的东北角; 动手操作室在花坛的西北角; 学生成果展示厅在动手操作室的南面, 休息室在花坛的南面。\n\n", "options": [], "subject": "画法几何学", "analysis": "解见详解。\n\n【分析】依据地图上的方向别别方法, 即“上北下南, 左西右东”,以及题目所给的各种信息, 即可进行解答。\n\n【详解】\n\n\n\n【点睛】本题考查方向与位置的实际应用能力。"} {"id": "9204", "image": ["1516.jpg", "1517.jpg", "1517.jpg"], "answer": "解33 平方米\n\n【分析】组合图形的面积是由一个长为 4 米, 宽为 3 米的长方形面积和一个长为 7 米, 宽为 $(6-3)$米的长方形面积组合而成, 利用长方形的面积公式分别求出这两个图形的面积, 再相加即可求出组合图形的面积。\n\n【详解】根据分析, 如图所示:\n\n\n\n$4 \\times 3+7 \\times(6-3)$\n$=12+7 \\times 3$\n\n$=12+21$\n\n$=33$ (平方米)\n\n即组合图形的面积是 33 平方米。", "solution": "null", "level": "三年级", "question": "求组合图形的面积。 (在求解过程中, 要求画出对应的分割线)\n\n", "options": [], "subject": "组合几何学", "analysis": "解33 平方米\n\n【分析】组合图形的面积是由一个长为 4 米, 宽为 3 米的长方形面积和一个长为 7 米, 宽为 $(6-3)$米的长方形面积组合而成, 利用长方形的面积公式分别求出这两个图形的面积, 再相加即可求出组合图形的面积。\n\n【详解】根据分析, 如图所示:\n\n\n\n$4 \\times 3+7 \\times(6-3)$\n$=12+7 \\times 3$\n\n$=12+21$\n\n$=33$ (平方米)\n\n即组合图形的面积是 33 平方米。"} {"id": "8140", "image": [], "answer": "错", "solution": "null", "level": "三年级", "question": "四边形的周长一定大于三角形的周长。", "options": [], "subject": "立体几何学", "analysis": "解$错$"} {"id": "27264", "image": [], "answer": "解$6300 ; 8 ; 12 ; 0$;\n\n$50 ; 88 ; 1000 ; 29$\n\n【详解】略", "solution": "null", "level": "四年级", "question": "直接写出得数。\n\n| $90 \\times 70=$ | $400 \\div 50=$ | $28 \\div 7 \\times 3=$ | $300 \\times 24 \\times 0=$ |\n| :--- | :--- | :--- | :--- |\n| $700 \\div 14=$ | $174-86=$ | $350+650=$ | $36-36 \\div 4=$ |", "options": [], "subject": "算术", "analysis": "解$6300 ; 8 ; 12 ; 0$;\n\n$50 ; 88 ; 1000 ; 29$\n\n【详解】略"} {"id": "27265", "image": [], "answer": "解$550 ; 149$\n\n【分析】(1)先算小括号里面的减法, 再从左向右进行计算;\n\n(2)先算乘法, 再从左向右进行计算。\n\n【详解】 $350 \\div 14 \\times(276-254)$\n\n$=350 \\div 14 \\times 22$\n\n$=25 \\times 22$\n\n$=550$\n\n$810-54 \\times 13+41$\n\n$=810-702+41$\n\n$=108+41$\n\n$=149$", "solution": "null", "level": "四年级", "question": "脱式计算。\n\n$350 \\div 14 \\times(276-254) \\quad 810-54 \\times 13+41$", "options": [], "subject": "算术", "analysis": "解$550 ; 149$\n\n【分析】(1)先算小括号里面的减法, 再从左向右进行计算;\n\n(2)先算乘法, 再从左向右进行计算。\n\n【详解】 $350 \\div 14 \\times(276-254)$\n\n$=350 \\div 14 \\times 22$\n\n$=25 \\times 22$\n\n$=550$\n\n$810-54 \\times 13+41$\n\n$=810-702+41$\n\n$=108+41$\n\n$=149$"} {"id": "27267", "image": [], "answer": "解7\n\n【分析】根据题目意思, 先计算差, 再除以 7 即可。\n\n【详解】 (118-69) $\\div 7$\n$=49 \\div 7$\n\n$=7$", "solution": "null", "level": "四年级", "question": "118 与 69 的差是 7 的多少倍?", "options": [], "subject": "算术", "analysis": "解7\n\n【分析】根据题目意思, 先计算差, 再除以 7 即可。\n\n【详解】 (118-69) $\\div 7$\n$=49 \\div 7$\n\n$=7$"} {"id": "27294", "image": [], "answer": "解【解答】解:\n$82+258=340$\n$860-403=457$\n$24 \\times 60=1440$\n$75-6 \\times(1+4)=45$\n$58 \\times 0=0$\n$320 \\div 20=16$\n$125-25 \\times 0=125$\n$12 \\times 4 \\div 12 \\times 4=16$", "solution": "null", "level": "四年级", "question": "(2015 春・衡阳月考)口算\n$82+258=$\n$860-403=$\n$24 \\times 60=$\n$75-6 \\times(1+4)=$\n$58 \\times 0=$\n$320 \\div 20=$\n$125-25 \\times 0=$\n$12 \\times 4 \\div 12 \\times 4=$", "options": [], "subject": "算术", "analysis": "解【解答】解:\n$82+258=340$\n$860-403=457$\n$24 \\times 60=1440$\n$75-6 \\times(1+4)=45$\n$58 \\times 0=0$\n$320 \\div 20=16$\n$125-25 \\times 0=125$\n$12 \\times 4 \\div 12 \\times 4=16$"} {"id": "27295", "image": [], "answer": "解【解答】解: (1) $480 \\div(2 \\times \\square)=16$\n\n$$\n2 \\times \\square=30\n$$\n\n$\\square=15$\n\n(2) $(3079+\\square) \\div 76=89$\n\n$$\n3079+\\square=6764\n$$\n\n$\\square=3685$\n\n故答案为: 15,3685 .", "solution": "null", "level": "四年级", "question": "(2013 秋 $\\cdot$ 海安县期中)在方框内填上合适的数.\n\n$$\n\\begin{array}{r}\n480 \\div(2 \\times \\square)=16 \\\\\n(3079+\\square) \\div 76=89\n\\end{array}\n$$", "options": [], "subject": "算术", "analysis": "解【解答】解: (1) $480 \\div(2 \\times \\square)=16$\n\n$$\n2 \\times \\square=30\n$$\n\n$\\square=15$\n\n(2) $(3079+\\square) \\div 76=89$\n\n$$\n3079+\\square=6764\n$$\n\n$\\square=3685$\n\n故答案为: 15,3685 ."} {"id": "27296", "image": [], "answer": "解【解答】解: (1) $540 \\div 6+3 \\times 2$\n\n$$\n\\begin{aligned}\n& 90+6 \\\\\n& =96 \\\\\n& (2) \\quad(540 \\div 6+3) \\times 2 \\\\\n& =(90+3) \\times 2 \\\\\n& =93 \\times 2 \\\\\n& =186\n\\end{aligned}\n$$\n\n(3) $540 \\div[(6+3) \\times 2]$\n\n$=540 \\div[9 \\times 2]$\n\n$=540 \\div 18$\n\n$=30$", "solution": "null", "level": "四年级", "question": "算一算, 比一比。\n$540 \\div 6+3 \\times 2$\n$(540 \\div 6+3) \\times 2$\n$540 \\div[(6+3) \\times 2]$", "options": [], "subject": "算术", "analysis": "解【解答】解: (1) $540 \\div 6+3 \\times 2$\n\n$$\n\\begin{aligned}\n& 90+6 \\\\\n& =96 \\\\\n& (2) \\quad(540 \\div 6+3) \\times 2 \\\\\n& =(90+3) \\times 2 \\\\\n& =93 \\times 2 \\\\\n& =186\n\\end{aligned}\n$$\n\n(3) $540 \\div[(6+3) \\times 2]$\n\n$=540 \\div[9 \\times 2]$\n\n$=540 \\div 18$\n\n$=30$"} {"id": "27324", "image": [], "answer": "解$150 ; 30 ; 492 ; 68$;\n\n20;960;225;5\n\n【详解】略", "solution": "null", "level": "四年级", "question": "直接写得数。\n\n| $15 \\times 10=$ | $600 \\div 20=$ | $430+62=$ | $8+2 \\times 30=$ |\n| :--- | :--- | :--- | :--- |\n| $800 \\div 40=$ | $32 \\times 30=$ | $350-125=$ | $16 \\times 5 \\div 16=$ |", "options": [], "subject": "算术", "analysis": "解$150 ; 30 ; 492 ; 68$;\n\n20;960;225;5\n\n【详解】略"} {"id": "27325", "image": [], "answer": "解$536 ; 5$\n\n【分析】整数四则混合运算,先乘除后加减; 同级运算,从左往右算; 有括号的,先算括号里面的,再算括号外面的。\n\n【详解】 $36+64 \\div 16 \\times 125$\n\n$=36+500$\n\n$=536$\n\n$3000 \\div[(50-26) \\times 25]$\n\n$=3000 \\div[24 \\times 25]$\n\n$=3000 \\div 600$\n\n$=5$", "solution": "null", "level": "四年级", "question": "计算下面各题。\n\n$36+64 \\div 16 \\times 125 \\quad 3000 \\div[(50-26) \\times 25]$", "options": [], "subject": "算术", "analysis": "解$536 ; 5$\n\n【分析】整数四则混合运算,先乘除后加减; 同级运算,从左往右算; 有括号的,先算括号里面的,再算括号外面的。\n\n【详解】 $36+64 \\div 16 \\times 125$\n\n$=36+500$\n\n$=536$\n\n$3000 \\div[(50-26) \\times 25]$\n\n$=3000 \\div[24 \\times 25]$\n\n$=3000 \\div 600$\n\n$=5$"} {"id": "27326", "image": [], "answer": "解23\n\n【分析】根据题意要求, 计算顺序是先求出和与差, 最后求商, 列式 $(85+260) \\div(50-35)$ 据此解答。\n\n【详解】 $(85+260) \\div(50-35)$\n\n$=345 \\div 15$\n\n$=23$", "solution": "null", "level": "四年级", "question": "85 与 260 的和除以 50 减去 35 的差, 商是多少?", "options": [], "subject": "算术", "analysis": "解23\n\n【分析】根据题意要求, 计算顺序是先求出和与差, 最后求商, 列式 $(85+260) \\div(50-35)$ 据此解答。\n\n【详解】 $(85+260) \\div(50-35)$\n\n$=345 \\div 15$\n\n$=23$"} {"id": "27353", "image": [], "answer": "解$150 ; 1500 ; 1955$;\n\n$3 ; 126 ; 840$\n\n【分析】第一小题,先算乘法,再算加法, 最后计算减法;\n\n第二小题,先算小括号里面的减法,再算中括号里面的加法,最后计算中括号外面的乘法;第三小题, 先算括号里面的除法, 再算括号里面的加法, 最后计算括号外面的乘法;\n\n第四小题, 先算小括号里面的减法, 再算中括号里面的乘法, 最后计算中括号外面的除法;第五小题, 先算括号里面的加法, 再算括号外面的除法, 最后计算括号外面的加法;第六小题, 先算乘法, 再算减法。\n\n【详解】 $560+40-30 \\times 15$\n\n$=560+40-450$\n\n$=600-450$\n\n$=150$\n\n$[228+(84-12)] \\times 5$\n\n$=[228+72] \\times 5$\n\n$=300 \\times 5$\n\n$=1500$\n\n$17 \\times(35+480 \\div 6)$\n\n$=17 \\times(35+80)$\n\n$=17 \\times 115$\n\n$=1955$\n\n$270 \\div[18 \\times(51-46)]$\n\n$$\n\\begin{aligned}\n&= 270 \\div[18 \\times 5] \\\\\n&= 270 \\div 90 \\\\\n&= 3 \\\\\n&(135+415) \\div 5+16 \\\\\n&= 550 \\div 5+16 \\\\\n&= 110+16 \\\\\n&= 126 \\\\\n& 1200-20 \\times 18 \\\\\n&= 1200-360 \\\\\n&=840\n\\end{aligned}\n$$", "solution": "null", "level": "四年级", "question": "用你喜欢的方法计算。\n\n$560+40-30 \\times 15 \\quad[228+(84-12)] \\times 5 \\quad 17 \\times(35+480 \\div 6)$\n$270 \\div[18 \\times(51-46)] \\quad(135+415) \\div 5+16 \\quad 1200-20 \\times 18$", "options": [], "subject": "算术", "analysis": "解$150 ; 1500 ; 1955$;\n\n$3 ; 126 ; 840$\n\n【分析】第一小题,先算乘法,再算加法, 最后计算减法;\n\n第二小题,先算小括号里面的减法,再算中括号里面的加法,最后计算中括号外面的乘法;第三小题, 先算括号里面的除法, 再算括号里面的加法, 最后计算括号外面的乘法;\n\n第四小题, 先算小括号里面的减法, 再算中括号里面的乘法, 最后计算中括号外面的除法;第五小题, 先算括号里面的加法, 再算括号外面的除法, 最后计算括号外面的加法;第六小题, 先算乘法, 再算减法。\n\n【详解】 $560+40-30 \\times 15$\n\n$=560+40-450$\n\n$=600-450$\n\n$=150$\n\n$[228+(84-12)] \\times 5$\n\n$=[228+72] \\times 5$\n\n$=300 \\times 5$\n\n$=1500$\n\n$17 \\times(35+480 \\div 6)$\n\n$=17 \\times(35+80)$\n\n$=17 \\times 115$\n\n$=1955$\n\n$270 \\div[18 \\times(51-46)]$\n\n$$\n\\begin{aligned}\n&= 270 \\div[18 \\times 5] \\\\\n&= 270 \\div 90 \\\\\n&= 3 \\\\\n&(135+415) \\div 5+16 \\\\\n&= 550 \\div 5+16 \\\\\n&= 110+16 \\\\\n&= 126 \\\\\n& 1200-20 \\times 18 \\\\\n&= 1200-360 \\\\\n&=840\n\\end{aligned}\n$$"} {"id": "27354", "image": [], "answer": "解5\n\n【分析】根据题意可知, 此题是先算加法和减法, 再算除法, 根据混合运算的计算顺序列出综合算式并计算即可, 混合运算的计算顺序是:先算乘、除法, 再算加、减法, 有括号时应先算括号里面的,再算括号外面的, 依此解答。\n\n【详解】 $(91+9) \\div(375-355)$\n\n$=100 \\div 20$\n\n$=5$", "solution": "null", "level": "四年级", "question": "用 91 与 9 的和除以 375 与 355 的差,商是多少?", "options": [], "subject": "算术", "analysis": "解5\n\n【分析】根据题意可知, 此题是先算加法和减法, 再算除法, 根据混合运算的计算顺序列出综合算式并计算即可, 混合运算的计算顺序是:先算乘、除法, 再算加、减法, 有括号时应先算括号里面的,再算括号外面的, 依此解答。\n\n【详解】 $(91+9) \\div(375-355)$\n\n$=100 \\div 20$\n\n$=5$"} {"id": "27379", "image": [], "answer": "解 $25 \\times 8=200$\n\n$125 \\div 100=1.25$\n\n$28 \\div 4 \\times 7=49$\n\n46- $(29+17)=0$\n\n$57+43+27=127$\n\n$36-0 \\times 7=36$\n\n$25 \\times 9 \\times 4=900$\n\n$75-25 \\div 5=70$\n\n$(24-8) \\div 4=4$", "solution": "null", "level": "四年级", "question": "直接写出得数\n$25 \\times 8=$\n$125 \\div 100=$\n$28 \\div 4 \\times 7=$\n46- $(29+17)=$\n$57+43+27=$\n$36-0 \\times 7=$\n$25 \\times 9 \\times 4=$\n$75-25 \\div 5=$\n$(24-8) \\div 4=$", "options": [], "subject": "算术", "analysis": "解 $25 \\times 8=200$\n\n$125 \\div 100=1.25$\n\n$28 \\div 4 \\times 7=49$\n\n46- $(29+17)=0$\n\n$57+43+27=127$\n\n$36-0 \\times 7=36$\n\n$25 \\times 9 \\times 4=900$\n\n$75-25 \\div 5=70$\n\n$(24-8) \\div 4=4$"} {"id": "27380", "image": [], "answer": "解 (1) 解: $(120-85) \\times 12 \\div 7$\n\n$=(120-85) \\times 12 \\div 7$\n\n$=35 \\times 12 \\div 7$\n\n$=420 \\div 7$\n\n$=60$\n\n(2)解: ( $900-16 \\times 35) \\div 34$\n\n$=(900-560) \\div 34$\n$=340 \\div 34$\n\n$=10$\n\n(3)解: $22+[(72+28) \\div 4]$\n\n$=22+[100 \\div 4]$\n\n$=22+25$\n\n$=47$", "solution": "null", "level": "四年级", "question": "脱式计算\n(1) $(120-85) \\times 12 \\div 7$\n(2) $(900-16 \\times 35) \\div 34$\n(3) $22+[(72+28) \\div 4]$", "options": [], "subject": "算术", "analysis": "解 (1) 解: $(120-85) \\times 12 \\div 7$\n\n$=(120-85) \\times 12 \\div 7$\n\n$=35 \\times 12 \\div 7$\n\n$=420 \\div 7$\n\n$=60$\n\n(2)解: ( $900-16 \\times 35) \\div 34$\n\n$=(900-560) \\div 34$\n$=340 \\div 34$\n\n$=10$\n\n(3)解: $22+[(72+28) \\div 4]$\n\n$=22+[100 \\div 4]$\n\n$=22+25$\n\n$=47$"} {"id": "27405", "image": [], "answer": "解$150 ; 30 ; 492 ; 68$;\n\n$20 ; 960 ; 225 ; 5$", "solution": "null", "level": "四年级", "question": "直接写得数。\n\n| $15 \\times 10=$ | $600 \\div 20=$ | $430+62=$ | $8+2 \\times 30=$ |\n| :--- | :--- | :--- | :--- |\n| $800 \\div 40=$ | $32 \\times 30=$ | $350-125=$ | $16 \\times 5 \\div 16=$ |", "options": [], "subject": "算术", "analysis": "解$150 ; 30 ; 492 ; 68$;\n\n20;960;225;5\n\n【详解】略"} {"id": "27406", "image": [], "answer": "解$414 ; 35$\n\n【分析】(1)从左到右依次计算;\n\n(2)先算括号里面的加法,再算括号外的除法。\n\n【详解】 $774 \\div 43 \\times 23$\n\n$=18 \\times 23$\n\n$=414$\n\n$(589+146) \\div 21$\n\n$=735 \\div 21$\n\n$=35$", "solution": "null", "level": "四年级", "question": "脱式计算。\n\n$774 \\div 43 \\times 23 \\quad(589+146) \\div 21$", "options": [], "subject": "算术", "analysis": "解$414 ; 35$\n\n【分析】(1)从左到右依次计算;\n\n(2)先算括号里面的加法,再算括号外的除法。\n\n【详解】 $774 \\div 43 \\times 23$\n\n$=18 \\times 23$\n\n$=414$\n\n$(589+146) \\div 21$\n\n$=735 \\div 21$\n\n$=35$"} {"id": "27407", "image": [], "answer": "解420\n\n【分析】先用 38 乘 5 求出积, 再用 230 加上求出的积即可;\n\n【详解】 $230+38 \\times 5$\n\n$=230+190$\n$=420$\n\n所以所得的和是 420 。\n\n【点睛】列式计算, 关键是弄清运算顺序,然后再列式解答。", "solution": "null", "level": "四年级", "question": "列式计算。\n\n230 加上 38 的 5 倍, 所得的和是多少?", "options": [], "subject": "算术", "analysis": "解420\n\n【分析】先用 38 乘 5 求出积, 再用 230 加上求出的积即可;\n\n【详解】 $230+38 \\times 5$\n\n$=230+190$\n$=420$\n\n所以所得的和是 420 。\n\n【点睛】列式计算, 关键是弄清运算顺序,然后再列式解答。"} {"id": "27969", "image": [], "answer": "解760; 360; 60; 3600\n\n200; 1000; 250; 200\n\n【解析】略", "solution": "null", "level": "四年级", "question": "直接写出得数。\n\n$\\begin{array}{llll}60+700= & 72 \\times 5= & 420 \\div 7= & 36 \\times 100= \\\\ 800-600= & 125 \\times 8= & 25 \\times 5 \\times 2= & 360-134-26=\\end{array}$", "options": [], "subject": "算术", "analysis": "解760; 360; 60; 3600\n\n200; 1000; 250; 200\n\n【解析】略"} {"id": "28024", "image": [], "answer": "解12 个\n\n【分析】可以组成的小数有: $2.34 、 2.43 、 24.3 、 23.4 、 3.24 、 3.42 、 32.4 、 34.2 、 4.23 、 4.32 、 42.3 、$ 43.2, 共有 12 个小数, 据此解答。\n\n【详解】用数字卡片 $、 3$, $、 4$ 和小数点 $\\square$, 能够组成多少个不同的小数(每张卡片都要用上)?\n\n答: 能够组成 12 个不同的小数。\n\n【点睛】本题考查小数组成的实际应用, 熟知小数是由整数部分、小数部分、小数点组成是解答本题的关键。", "solution": "null", "level": "四年级", "question": "用数字卡片 $、 2$, $、 4$ 和小数点 $\\square$, 能够组成多少个不同的小数(每张卡片都要用上)?", "options": [], "subject": "组合数学", "analysis": "解12 个\n\n【分析】可以组成的小数有: $2.34 、 2.43 、 24.3 、 23.4 、 3.24 、 3.42 、 32.4 、 34.2 、 4.23 、 4.32 、 42.3 、$ 43.2, 共有 12 个小数, 据此解答。\n\n【详解】用数字卡片 $、 3$, $、 4$ 和小数点 $\\square$, 能够组成多少个不同的小数(每张卡片都要用上)?\n\n答: 能够组成 12 个不同的小数。\n\n【点睛】本题考查小数组成的实际应用, 熟知小数是由整数部分、小数部分、小数点组成是解答本题的关键。"} {"id": "28690", "image": ["13713.jpg", "13714.jpg"], "answer": "解(1) $55^{\\circ}$\n\n(2) $120^{\\circ}$\n\n【分析】(1) 如图一个直角三角形, 那么一个角是 90 度, 另两个角的和是 90 度, 用 90 度减去给出的一个角的度数就是所求的角的度数;\n\n(2)根据三角形内角和度数是 180 度, 减去所给的两个角的度数, 就是所求角的度数, 据此解答。\n\n【详解】(1) $90^{\\circ}-35^{\\circ}=55^{\\circ}$\n\n(2) $180^{\\circ}-36^{\\circ}-24$\n\n$=144^{\\circ}-24^{\\circ}$\n\n$=120^{\\circ}$", "solution": "null", "level": "四年级", "question": "求出图中未知角的度数。\n(1)\n\n\n(2)\n\n", "options": [], "subject": "度量几何学", "analysis": "解(1) $55^{\\circ}$\n\n(2) $120^{\\circ}$\n\n【分析】(1) 如图一个直角三角形, 那么一个角是 90 度, 另两个角的和是 90 度, 用 90 度减去给出的一个角的度数就是所求的角的度数;\n\n(2)根据三角形内角和度数是 180 度, 减去所给的两个角的度数, 就是所求角的度数, 据此解答。\n\n【详解】(1) $90^{\\circ}-35^{\\circ}=55^{\\circ}$\n\n(2) $180^{\\circ}-36^{\\circ}-24$\n\n$=144^{\\circ}-24^{\\circ}$\n\n$=120^{\\circ}$"} {"id": "28765", "image": ["13737.jpg"], "answer": "解$\\angle 1=30^{\\circ} ; \\angle 1=56^{\\circ}$;\n\n【分析】三角形的内角和为 $180^{\\circ}$, 直角 $=90^{\\circ}$, 平角 $=180^{\\circ}$, 即用 $180^{\\circ}$ 减去 $120^{\\circ}$ 即可得到图 1 中与 $120^{\\circ}$相邻的角的度数, 然后用 $180^{\\circ}$ 减去三角形用另外两个角的度数之和即可;\n\n等腰三角形的两个底角相等, 因此用 $180^{\\circ}$ 减去 $68^{\\circ}$ 后, 再除以 2 即可得到 $\\angle 1$ 的度数, 依此计算。\n\n【详解】 $180^{\\circ}-120^{\\circ}=60^{\\circ}$\n\n$60^{\\circ}+90^{\\circ}=150^{\\circ}$\n\n$180^{\\circ}-150^{\\circ}=30^{\\circ}$\n\n即图 1 中, $\\angle 1=30^{\\circ}$\n\n$180^{\\circ}-68^{\\circ}=112^{\\circ}$\n\n$112^{\\circ} \\div 2=56^{\\circ}$\n\n即图 2 中, $\\angle 1=56^{\\circ}$ 。", "solution": "null", "level": "四年级", "question": "下列图形中 $\\angle 1$ 的度数。\n", "options": [], "subject": "度量几何学", "analysis": "解$\\angle 1=30^{\\circ} ; \\angle 1=56^{\\circ}$;\n\n【分析】三角形的内角和为 $180^{\\circ}$, 直角 $=90^{\\circ}$, 平角 $=180^{\\circ}$, 即用 $180^{\\circ}$ 减去 $120^{\\circ}$ 即可得到图 1 中与 $120^{\\circ}$相邻的角的度数, 然后用 $180^{\\circ}$ 减去三角形用另外两个角的度数之和即可;\n\n等腰三角形的两个底角相等, 因此用 $180^{\\circ}$ 减去 $68^{\\circ}$ 后, 再除以 2 即可得到 $\\angle 1$ 的度数, 依此计算。\n\n【详解】 $180^{\\circ}-120^{\\circ}=60^{\\circ}$\n\n$60^{\\circ}+90^{\\circ}=150^{\\circ}$\n\n$180^{\\circ}-150^{\\circ}=30^{\\circ}$\n\n即图 1 中, $\\angle 1=30^{\\circ}$\n\n$180^{\\circ}-68^{\\circ}=112^{\\circ}$\n\n$112^{\\circ} \\div 2=56^{\\circ}$\n\n即图 2 中, $\\angle 1=56^{\\circ}$ 。"} {"id": "29053", "image": ["13817.jpg"], "answer": "解(1) $2018 ; 2021$;\n(2) 2021 ;\n\n(3)男生和女生的近视人数都在逐年减少。(答案不唯一)\n\n【分析】(1)男生对应的灰色条状中,哪一年对应的条状最高,则男生这一年近视人数最多;男生对应的灰色条状中,哪一年对应的条状最矮,则男生这一年近视人数最少。\n\n(2)哪一年中,男生和女生对应的条状相差最多,则这一年男女近视人数相差数量最大。\n\n(3)根据统计图中的信息进行解答,言之合理即可。\n\n【详解】(1)根据统计图可知,男生 2018 年近视人数最多,2021 年最少。\n\n(2)根据统计图可知,2021 年男女近视人数相差数量最大。\n\n(3)根据统计图可知,男生和女生的近视人数都在逐年减少。\n\n【点睛】此题是考查如何从复式条形统计图中获取信息,并根据所获取的信息解决实际问题。", "solution": "null", "level": "四年级", "question": "上海路小学近四年男、女生近视人数如下图。\n\n\n\n(1)男生()年近视人数最多,()年最少。\n\n(2)()年男女近视人数相差数量最大。\n\n(3) 你还能得到哪些信息?", "options": [], "subject": "计数", "analysis": "解(1) $2018 ; 2021$;\n(2) 2021 ;\n\n(3)男生和女生的近视人数都在逐年减少。(答案不唯一)\n\n【分析】(1)男生对应的灰色条状中,哪一年对应的条状最高,则男生这一年近视人数最多;男生对应的灰色条状中,哪一年对应的条状最矮,则男生这一年近视人数最少。\n\n(2)哪一年中,男生和女生对应的条状相差最多,则这一年男女近视人数相差数量最大。\n\n(3)根据统计图中的信息进行解答,言之合理即可。\n\n【详解】(1)根据统计图可知,男生 2018 年近视人数最多,2021 年最少。\n\n(2)根据统计图可知,2021 年男女近视人数相差数量最大。\n\n(3)根据统计图可知,男生和女生的近视人数都在逐年减少。\n\n【点睛】此题是考查如何从复式条形统计图中获取信息,并根据所获取的信息解决实际问题。"} {"id": "24438", "image": [], "answer": "解 $144 \\div 4 \\div 3$\n\n$$\n=36 \\div 3\n$$\n\n$=12$ (厘米)\n\n$12-3=9$ (厘米)\n\n$12 \\times 12 \\times 9$\n\n$=144 \\times 9$\n\n$=1296$ (立方厘米)\n\n答: 原来长方体的体积是 1296 立方厘米。", "solution": "null", "level": "五年级", "question": "(2022 春・峄城区期末)一个长方体,高增加 $3 \\mathrm{~cm}$ ,就变成了一个正方体,这时表面积比原来增加 $144 \\mathrm{~cm}^{2}$ 。原来长方体的体积是多少立方厘米?", "options": [], "subject": "立体几何学", "analysis": "解 $144 \\div 4 \\div 3$\n\n$$\n=36 \\div 3\n$$\n\n$=12$ (厘米)\n\n$12-3=9$ (厘米)\n\n$12 \\times 12 \\times 9$\n\n$=144 \\times 9$\n\n$=1296$ (立方厘米)\n\n答: 原来长方体的体积是 1296 立方厘米。"} {"id": "24439", "image": ["12478.jpg"], "answer": "解 $4 \\times 4 \\times 4-6 \\times 5 \\times(4-3.2)$\n\n$=16 \\times 4-3 \\times 0.8$\n\n$=64-24$\n\n$=40$ (立方分米)\n\n40 立方分米 $=40$ 升\n\n答: 缸里的水溢出 40 升。", "solution": "null", "level": "五年级", "question": "(2022 春・兴平市期末)一个长方体的玻璃缸, 从里面量长是 $6 \\mathrm{dm}$, 宽是 $5 \\mathrm{dm}$, 高是 $4 \\mathrm{dm}$, 水深是 $3.2 d m$ 。如果投入一块棱长是 $4 d m$​ 的正方体铁块(如图), 缸里的水溢出多少升?\n ", "options": [], "subject": "立体几何学", "analysis": "解 $4 \\times 4 \\times 4-6 \\times 5 \\times(4-3.2)$\n\n$=16 \\times 4-3 \\times 0.8$\n\n$=64-24$\n\n$=40$ (立方分米)\n\n40 立方分米 $=40$ 升\n\n答: 缸里的水溢出 40 升。"} {"id": "24440", "image": [], "answer": "解 $10 \\times 10 \\times 10=1000$ (立方厘米) $=1$ (立方分米)\n\n$1 \\div(2.5 \\times 2)$\n\n$=1 \\div 5$\n\n$=0.2$ (分米)\n\n答: 这块钢板有 0.2 分米厚。", "solution": "null", "level": "五年级", "question": "(2022・天津模拟)把一块棱长为 10 厘米的正方体钢坏, 锻造成一个长 2.5 分米, 宽 2 分米的长方体钢板,这块钢板有多厚?(损耗不计)", "options": [], "subject": "立体几何学", "analysis": "解 $10 \\times 10 \\times 10=1000$ (立方厘米) $=1$ (立方分米)\n\n$1 \\div(2.5 \\times 2)$\n\n$=1 \\div 5$\n\n$=0.2$ (分米)\n\n答: 这块钢板有 0.2 分米厚。"} {"id": "24462", "image": ["12492.jpg"], "answer": "解96 平方厘米, 64 立方厘米; 136 平方厘米, 96 立方厘米\n\n【分析】根据长方体的表面积公式: $\\mathrm{S}=(\\mathrm{ab}+\\mathrm{ah}+\\mathrm{bh}) \\times 2$, 长方体的体积公式: $\\mathrm{V}=\\mathrm{abh}$; 正方体的表面积公式: $S=6 a^{2}$, 正方体的体积公式: $V=a^{3}$; 据此代入数值进行计算即可。\n\n【详解】 $4 \\times 4 \\times 6$\n\n$=16 \\times 6$\n\n$=96$ (平方厘米)\n\n$4 \\times 4 \\times 4$\n\n$=16 \\times 4$\n\n$=64$ (立方厘米)\n\n$(8 \\times 4+8 \\times 3+4 \\times 3) \\times 2$\n\n$=(32+24+12) \\times 2$\n\n$=68 \\times 2$\n\n$=136$ (平方厘米)\n\n$8 \\times 4 \\times 3$\n$=32 \\times 3$\n\n$=96$ (立方厘米)", "solution": "null", "level": "五年级", "question": "求下面长方体和正方体的表面积和体积。(单位:厘米。)\n", "options": [], "subject": "立体几何学", "analysis": "解96 平方厘米, 64 立方厘米; 136 平方厘米, 96 立方厘米\n\n【分析】根据长方体的表面积公式: $\\mathrm{S}=(\\mathrm{ab}+\\mathrm{ah}+\\mathrm{bh}) \\times 2$, 长方体的体积公式: $\\mathrm{V}=\\mathrm{abh}$; 正方体的表面积公式: $S=6 a^{2}$, 正方体的体积公式: $V=a^{3}$; 据此代入数值进行计算即可。\n\n【详解】 $4 \\times 4 \\times 6$\n\n$=16 \\times 6$\n\n$=96$ (平方厘米)\n\n$4 \\times 4 \\times 4$\n\n$=16 \\times 4$\n\n$=64$ (立方厘米)\n\n$(8 \\times 4+8 \\times 3+4 \\times 3) \\times 2$\n\n$=(32+24+12) \\times 2$\n\n$=68 \\times 2$\n\n$=136$ (平方厘米)\n\n$8 \\times 4 \\times 3$\n$=32 \\times 3$\n\n$=96$ (立方厘米)"} {"id": "24463", "image": ["12493.jpg"], "answer": "解216 平方厘米; 204 立方厘米\n\n【分析】组合体的表面积 $=$ 大正方体表面积, 正方体表面积 $=$ 棱长 $\\times$ 棱长 $\\times 6$; 组合体体积 $=$ 大正方体体积一小长方体体积, 正方体体积 $=$ 棱长 $\\times$ 棱长 $\\times$ 棱长, 长方体体积 $=$ 棱长 $\\times$ 棱长 $\\times$ 棱长, 据此列式计\n\n算。\n\n【详解】 $6 \\times 6 \\times 6=216$ (平方厘米)\n\n$6 \\times 6 \\times 6-2 \\times 2 \\times 3$\n\n$=216-12$\n\n$=204$ (立方厘米)", "solution": "null", "level": "五年级", "question": "求下面图形的表面积和体积。(单位: 厘米)\n\n", "options": [], "subject": "立体几何学", "analysis": "解216 平方厘米; 204 立方厘米\n\n【分析】组合体的表面积 $=$ 大正方体表面积, 正方体表面积 $=$ 棱长 $\\times$ 棱长 $\\times 6$; 组合体体积 $=$ 大正方体体积一小长方体体积, 正方体体积 $=$ 棱长 $\\times$ 棱长 $\\times$ 棱长, 长方体体积 $=$ 棱长 $\\times$ 棱长 $\\times$ 棱长, 据此列式计\n\n算。\n\n【详解】 $6 \\times 6 \\times 6=216$ (平方厘米)\n\n$6 \\times 6 \\times 6-2 \\times 2 \\times 3$\n\n$=216-12$\n\n$=204$ (立方厘米)"} {"id": "24489", "image": ["12504.jpg"], "answer": "解216 平方米, 180 立方米; 384 平方米, 512 立方米\n\n【分析】根据长方体的表面积公式: $\\mathrm{S}=( \\mathrm{ab}+\\mathrm{ah}+\\mathrm{bh}) \\times 2$; 长方体的体积公式: $\\mathrm{V}=\\mathrm{abh}$; 正方体的表面积公式: $S=6 \\mathrm{a}^{2}$; 正方体的体积公式: $V=\\mathrm{a}^{3}$; 据此代入数值进行计算即可。\n\n【详解】 $(10 \\times 3+10 \\times 6+3 \\times 6) \\times 2$\n\n$=(30+60+18) \\times 2$\n\n$=108 \\times 2$\n\n$=216$ (平方米)\n$10 \\times 3 \\times 6$\n\n$=30 \\times 6$\n\n$=180$ (立方米)\n\n$8 \\times 8 \\times 6$\n\n$=64 \\times 6$\n\n$=384$ (平方米)\n\n$8 \\times 8 \\times 8$\n\n$=64 \\times 8$\n\n$=512$ (立方米)", "solution": "null", "level": "五年级", "question": "计算下面各图形的表面积和体积。(单位:米)\n", "options": [], "subject": "立体几何学", "analysis": "解216 平方米, 180 立方米; 384 平方米, 512 立方米\n\n【分析】根据长方体的表面积公式: $\\mathrm{S}=( \\mathrm{ab}+\\mathrm{ah}+\\mathrm{bh}) \\times 2$; 长方体的体积公式: $\\mathrm{V}=\\mathrm{abh}$; 正方体的表面积公式: $S=6 \\mathrm{a}^{2}$; 正方体的体积公式: $V=\\mathrm{a}^{3}$; 据此代入数值进行计算即可。\n\n【详解】 $(10 \\times 3+10 \\times 6+3 \\times 6) \\times 2$\n\n$=(30+60+18) \\times 2$\n\n$=108 \\times 2$\n\n$=216$ (平方米)\n$10 \\times 3 \\times 6$\n\n$=30 \\times 6$\n\n$=180$ (立方米)\n\n$8 \\times 8 \\times 6$\n\n$=64 \\times 6$\n\n$=384$ (平方米)\n\n$8 \\times 8 \\times 8$\n\n$=64 \\times 8$\n\n$=512$ (立方米)"} {"id": "24490", "image": ["12505.jpg"], "answer": "解$150 \\mathrm{~cm}^{2} ; 109 \\mathrm{~cm}^{3}$\n\n【分析】把图形缺口处露出来的 3 个面向外平移, 正好补全缺口, 这样图形的表面积就是正方体的表面积; 根据正方体的表面积 $=$ 棱长 $\\times$ 棱长 $\\times 6$, 代入数据计算即可求出图形的表面积;\n\n把图形的缺口处补上, 补成一个完整的正方体, 那么图形的体积=正方体的体积一小长方体的体积;根据正方体的体积 $=$ 棱长 $\\times$ 棱长 $\\times$ 棱长, 小长方体的体积 $=$ 长 $\\times$ 宽 $\\times$ 高, 代入数据计算即可。\n\n【详解】图形的表面积:\n\n$5 \\times 5 \\times 6$\n\n$=25 \\times 6$\n\n$=150\\left(\\mathrm{~cm}^{2}\\right)$\n\n图形的体积:\n\n$5 \\times 5 \\times 5-4 \\times 2 \\times 2$\n\n$=25 \\times 5-8 \\times 2$\n\n$=125-16$\n\n$=109\\left(\\mathrm{~cm}^{3}\\right)$", "solution": "null", "level": "五年级", "question": "计算下面图形的表面积和体积。(单位: cm)\n\n", "options": [], "subject": "立体几何学", "analysis": "解$150 \\mathrm{~cm}^{2} ; 109 \\mathrm{~cm}^{3}$\n\n【分析】把图形缺口处露出来的 3 个面向外平移, 正好补全缺口, 这样图形的表面积就是正方体的表面积; 根据正方体的表面积 $=$ 棱长 $\\times$ 棱长 $\\times 6$, 代入数据计算即可求出图形的表面积;\n\n把图形的缺口处补上, 补成一个完整的正方体, 那么图形的体积=正方体的体积一小长方体的体积;根据正方体的体积 $=$ 棱长 $\\times$ 棱长 $\\times$ 棱长, 小长方体的体积 $=$ 长 $\\times$ 宽 $\\times$ 高, 代入数据计算即可。\n\n【详解】图形的表面积:\n\n$5 \\times 5 \\times 6$\n\n$=25 \\times 6$\n\n$=150\\left(\\mathrm{~cm}^{2}\\right)$\n\n图形的体积:\n\n$5 \\times 5 \\times 5-4 \\times 2 \\times 2$\n\n$=25 \\times 5-8 \\times 2$\n\n$=125-16$\n\n$=109\\left(\\mathrm{~cm}^{3}\\right)$"} {"id": "24518", "image": ["12518.jpg"], "answer": "解85 平方厘米; 50 立方厘米\n\n【分析】把长方体的长、宽、高的数据代入长方体的表面积公式: $\\mathrm{S}=(\\mathrm{a} \\times \\mathrm{b}+\\mathrm{a} \\times \\mathrm{h}+\\mathrm{b} \\times \\mathrm{h}) \\times 2$, 和长方体的体积公式: $V=\\mathrm{a} \\times \\mathrm{b} \\times \\mathrm{h}$ 中, 计算出长方体的表面积和体积。\n\n【详解】 $(5 \\times 4+5 \\times 2.5+4 \\times 2.5) \\times 2$\n\n$=(20+12.5+10) \\times 2$\n\n$=42.5 \\times 2$\n\n$=85$ (平方厘米)\n\n$5 \\times 4 \\times 2.5=50$ (立方厘米)\n\n即长方体的表面积是 85 平方厘米,体积是 50 立方厘米。", "solution": "null", "level": "五年级", "question": "计算长方体的表面积和体积。(单位:厘米)\n\n", "options": [], "subject": "立体几何学", "analysis": "解85 平方厘米; 50 立方厘米\n\n【分析】把长方体的长、宽、高的数据代入长方体的表面积公式: $\\mathrm{S}=(\\mathrm{a} \\times \\mathrm{b}+\\mathrm{a} \\times \\mathrm{h}+\\mathrm{b} \\times \\mathrm{h}) \\times 2$, 和长方体的体积公式: $V=\\mathrm{a} \\times \\mathrm{b} \\times \\mathrm{h}$ 中, 计算出长方体的表面积和体积。\n\n【详解】 $(5 \\times 4+5 \\times 2.5+4 \\times 2.5) \\times 2$\n\n$=(20+12.5+10) \\times 2$\n\n$=42.5 \\times 2$\n\n$=85$ (平方厘米)\n\n$5 \\times 4 \\times 2.5=50$ (立方厘米)\n\n即长方体的表面积是 85 平方厘米,体积是 50 立方厘米。"} {"id": "24519", "image": ["12519.jpg"], "answer": "解$1220 \\mathrm{~m}^{2} ; 2445 \\mathrm{~m}^{3}$\n\n【分析】长方体的表面积 $=($ 长 $\\times$ 宽十+宽 $\\times$ 高 + 长 $\\times$ 高 $) \\times 2$, 长方体的体积 $=$ 长 $\\times$ 宽 $\\times$ 高, 几何体的表面积 $=$ 大长方体的表面积 + 小长方体 4 个侧面的面积, 几何体的体积 $=$ 大长方体的体积 + 小长方体的体积, 据此解答。\n\n【详解】表面积: $(20 \\times 8+20 \\times 15+8 \\times 15 ) \\times 2+5 \\times 3 \\times 4$\n\n$=(160+300+120) \\times 2+5 \\times 3 \\times 4$\n\n$=580 \\times 2+5 \\times 3 \\times 4$\n\n$=1160+60$\n\n$=1220\\left(\\mathrm{~m}^{2}\\right)$\n\n体积: $20 \\times 8 \\times 15+5 \\times 3 \\times 3$\n\n$=160 \\times 15+15 \\times 3$\n\n$$\n\\begin{aligned}\n& =2400+45 \\\\\n& =2445\\left(\\mathrm{~m}^{3}\\right)\n\\end{aligned}\n$$\n\n所以,这个几何体的表面积是 $1220 \\mathrm{~m}^{2}$, 体积是 $2445 \\mathrm{~m}^{3}$ 。", "solution": "null", "level": "五年级", "question": "计算下面几何体的表面积和体积。\n\n", "options": [], "subject": "立体几何学", "analysis": "解$1220 \\mathrm{~m}^{2} ; 2445 \\mathrm{~m}^{3}$\n\n【分析】长方体的表面积 $=($ 长 $\\times$ 宽十+宽 $\\times$ 高 + 长 $\\times$ 高 $) \\times 2$, 长方体的体积 $=$ 长 $\\times$ 宽 $\\times$ 高, 几何体的表面积 $=$ 大长方体的表面积 + 小长方体 4 个侧面的面积, 几何体的体积 $=$ 大长方体的体积 + 小长方体的体积, 据此解答。\n\n【详解】表面积: $(20 \\times 8+20 \\times 15+8 \\times 15 ) \\times 2+5 \\times 3 \\times 4$\n\n$=(160+300+120) \\times 2+5 \\times 3 \\times 4$\n\n$=580 \\times 2+5 \\times 3 \\times 4$\n\n$=1160+60$\n\n$=1220\\left(\\mathrm{~m}^{2}\\right)$\n\n体积: $20 \\times 8 \\times 15+5 \\times 3 \\times 3$\n\n$=160 \\times 15+15 \\times 3$\n\n$$\n\\begin{aligned}\n& =2400+45 \\\\\n& =2445\\left(\\mathrm{~m}^{3}\\right)\n\\end{aligned}\n$$\n\n所以,这个几何体的表面积是 $1220 \\mathrm{~m}^{2}$, 体积是 $2445 \\mathrm{~m}^{3}$ 。"} {"id": "24542", "image": ["12537.jpg"], "answer": "解表面积: $64 \\mathrm{~cm}^{2}$\n\n体积: $30 \\mathrm{~cm}^{3}$\n【分析】根据长方体的表面积公式: $\\mathrm{S}=(\\mathrm{ab}+\\mathrm{ah}+\\mathrm{bh}) \\times 2$, 长方体的体积公式: $\\mathrm{V}=\\mathrm{abh}$, 据此代入数值进行计算即可。\n\n【详解】 $(6 \\times 2.5+6 \\times 2+2.5 \\times 2) \\times 2$\n\n$=(15+12+5) \\times 2$\n\n$=32 \\times 2$\n\n$=64\\left(\\mathrm{~cm}^{2}\\right)$\n\n$6 \\times 2.5 \\times 2$\n\n$=15 \\times 2$\n\n$=30\\left(\\mathrm{~cm}^{3}\\right)$", "solution": "null", "level": "五年级", "question": "计算下图物体的表面积和体积。(单位: $\\mathrm{cm}$ )\n\n", "options": [], "subject": "立体几何学", "analysis": "解表面积: $64 \\mathrm{~cm}^{2}$\n\n体积: $30 \\mathrm{~cm}^{3}$\n【分析】根据长方体的表面积公式: $\\mathrm{S}=(\\mathrm{ab}+\\mathrm{ah}+\\mathrm{bh}) \\times 2$, 长方体的体积公式: $\\mathrm{V}=\\mathrm{abh}$, 据此代入数值进行计算即可。\n\n【详解】 $(6 \\times 2.5+6 \\times 2+2.5 \\times 2) \\times 2$\n\n$=(15+12+5) \\times 2$\n\n$=32 \\times 2$\n\n$=64\\left(\\mathrm{~cm}^{2}\\right)$\n\n$6 \\times 2.5 \\times 2$\n\n$=15 \\times 2$\n\n$=30\\left(\\mathrm{~cm}^{3}\\right)$"} {"id": "24544", "image": ["12538.jpg"], "answer": "解表面积是 184 平方厘米, 体积是 152 立方厘米\n\n【分析】图形的表面积等于正方体的侧面积加长方体的表面积, 根据正方体的侧面积 $=$ 棱长 $\\times$ 棱长 $\\times 4$,长方体的表面积 $=($ 长 $\\times$ 宽 + 长 $\\times$ 高 + 宽 $\\times$ 高 $) \\times 2$, 代入数据计算即可解答; 组合图形的体积等于正方体的体积加上长方体的体积, 根据正方体的体积 $=$ 棱长 $\\times$ 棱长 $\\times$ 棱长, 长方体的体积 $=$ 长 $\\times$ 宽 $\\times$ 高, 代入数据解答即可。\n\n【详解】 $2 \\times 2 \\times 4+(6 \\times 6+6 \\times 4+6 \\times 4) \\times 2$\n\n$=16+(36+24+24) \\times 2$\n\n$=16+84 \\times 2$\n\n$=16+168$\n\n$=184$ (平方厘米)\n\n$2 \\times 2 \\times 2+6 \\times 6 \\times 4$\n\n$=8+144$\n\n$=152$ (立方厘米)\n\n表面积是 184 平方厘米, 体积是 152 立方厘米。", "solution": "null", "level": "五年级", "question": "计算下面组合图形的表面积和体积。\n\n", "options": [], "subject": "立体几何学", "analysis": "解表面积是 184 平方厘米, 体积是 152 立方厘米\n\n【分析】图形的表面积等于正方体的侧面积加长方体的表面积, 根据正方体的侧面积 $=$ 棱长 $\\times$ 棱长 $\\times 4$,长方体的表面积 $=($ 长 $\\times$ 宽 + 长 $\\times$ 高 + 宽 $\\times$ 高 $) \\times 2$, 代入数据计算即可解答; 组合图形的体积等于正方体的体积加上长方体的体积, 根据正方体的体积 $=$ 棱长 $\\times$ 棱长 $\\times$ 棱长, 长方体的体积 $=$ 长 $\\times$ 宽 $\\times$ 高, 代入数据解答即可。\n\n【详解】 $2 \\times 2 \\times 4+(6 \\times 6+6 \\times 4+6 \\times 4) \\times 2$\n\n$=16+(36+24+24) \\times 2$\n\n$=16+84 \\times 2$\n\n$=16+168$\n\n$=184$ (平方厘米)\n\n$2 \\times 2 \\times 2+6 \\times 6 \\times 4$\n\n$=8+144$\n\n$=152$ (立方厘米)\n\n表面积是 184 平方厘米, 体积是 152 立方厘米。"} {"id": "24545", "image": ["12539.jpg"], "answer": "解$392 \\mathrm{~cm}^{2}$\n\n【分析】由长方体的展开图可知, 这个长方体的长是 $12 \\mathrm{~cm}$, 宽是 $8 \\mathrm{~cm}$, 高是 $5 \\mathrm{~cm}$; 根据长方体的表面积 $=($ 长 $\\times$ 宽 + 长 $\\times$ 高 + 宽 $\\times$ 高 $) \\times 2$, 代入数据计算即可。\n\n【详解】 $(12 \\times 8+12 \\times 5+8 \\times 5) \\times 2$\n\n$=(96+60+40) \\times 2$\n\n$=196 \\times 2$\n\n$=392\\left(\\mathrm{~cm}^{2}\\right)$", "solution": "null", "level": "五年级", "question": "下图是一个长方体的展开图, 计算这个长方体的表面积。\n\n", "options": [], "subject": "立体几何学", "analysis": "解$392 \\mathrm{~cm}^{2}$\n\n【分析】由长方体的展开图可知, 这个长方体的长是 $12 \\mathrm{~cm}$, 宽是 $8 \\mathrm{~cm}$, 高是 $5 \\mathrm{~cm}$; 根据长方体的表面积 $=($ 长 $\\times$ 宽 + 长 $\\times$ 高 + 宽 $\\times$ 高 $) \\times 2$, 代入数据计算即可。\n\n【详解】 $(12 \\times 8+12 \\times 5+8 \\times 5) \\times 2$\n\n$=(96+60+40) \\times 2$\n\n$=196 \\times 2$\n\n$=392\\left(\\mathrm{~cm}^{2}\\right)$"} {"id": "24571", "image": ["12557.jpg"], "answer": "解940 平方厘米\n\n【分析】长方体的表面积 $=($ 长 $\\times$ 宽 + 长 $\\times$ 高 + 宽 $\\times$ 高 $) \\times 2$, 图中, 长方体的长是 $(10+10)$ 厘米, 宽是 9 厘米,高是 10 厘米,将数据代入公式解答即可。\n\n【详解】长: $10+10=20$ (厘米)\n\n$(20 \\times 9+20 \\times 10+9 \\times 10) \\times 2$\n\n$=(180+200+90) \\times 2$\n\n$=470 \\times 2$\n$=940$ (平方厘米)", "solution": "null", "level": "五年级", "question": "计算下面图形的表面积。(单位: 厘米)\n\n", "options": [], "subject": "立体几何学", "analysis": "解940 平方厘米\n\n【分析】长方体的表面积 $=($ 长 $\\times$ 宽 + 长 $\\times$ 高 + 宽 $\\times$ 高 $) \\times 2$, 图中, 长方体的长是 $(10+10)$ 厘米, 宽是 9 厘米,高是 10 厘米,将数据代入公式解答即可。\n\n【详解】长: $10+10=20$ (厘米)\n\n$(20 \\times 9+20 \\times 10+9 \\times 10) \\times 2$\n\n$=(180+200+90) \\times 2$\n\n$=470 \\times 2$\n$=940$ (平方厘米)"} {"id": "24572", "image": ["12558.jpg", "12559.jpg", "12559.jpg"], "answer": "解表面积 150; 体积 113\n\n【分析】图形右上角缺口处露出了 3 个面, 分别向外平移,正好补成一个棱长是 5 的大正方体, 根据正方体的表面积 $=$ 棱长 $\\times$ 棱长 $\\times 6$, 代入数据计算即可求出这个图形的表面积;\n\n补好图形右上角的缺口, 那么图形的体积 $=$ 棱长为 5 的正方体的体积一长 2 、宽 2 、高 3 的长方体的体积; 根据长方体的体积 $=$ 长 $\\times$ 宽 $\\times$ 高, 代入数据计算求出这个图形的体积。\n\n【详解】如图:\n\n\n\n表面积:\n\n$5 \\times 5 \\times 6$\n\n$=25 \\times 6$\n\n$=150$\n\n体积:\n\n$5 \\times 5 \\times 5-2 \\times 2 \\times 3$\n\n$=125-12$\n\n$=113$", "solution": "null", "level": "五年级", "question": "分别求出下面图形的表面积和体积。\n\n", "options": [], "subject": "立体几何学", "analysis": "解表面积 150; 体积 113\n\n【分析】图形右上角缺口处露出了 3 个面, 分别向外平移,正好补成一个棱长是 5 的大正方体, 根据正方体的表面积 $=$ 棱长 $\\times$ 棱长 $\\times 6$, 代入数据计算即可求出这个图形的表面积;\n\n补好图形右上角的缺口, 那么图形的体积 $=$ 棱长为 5 的正方体的体积一长 2 、宽 2 、高 3 的长方体的体积; 根据长方体的体积 $=$ 长 $\\times$ 宽 $\\times$ 高, 代入数据计算求出这个图形的体积。\n\n【详解】如图:\n\n\n\n表面积:\n\n$5 \\times 5 \\times 6$\n\n$=25 \\times 6$\n\n$=150$\n\n体积:\n\n$5 \\times 5 \\times 5-2 \\times 2 \\times 3$\n\n$=125-12$\n\n$=113$"} {"id": "24599", "image": ["12565.jpg"], "answer": "解(1)长方体的表面积:\n\n$(5 \\times 5+12 \\times 5+12 \\times 5) \\times 2$\n\n$=(25+60+60) \\times 2$\n\n$=145 \\times 2$\n\n$=290\\left(\\mathrm{~cm}^{2}\\right)$\n\n长方体的体积:\n\n$12 \\times 5 \\times 5$\n\n$=60 \\times 5$\n\n$=300\\left(\\mathrm{~cm}^{3}\\right)$\n\n(2)正方体的表面积:\n\n$0.25 \\times 6=1.5\\left(\\mathrm{dm}^{2}\\right)$\n\n正方体的体积:\n\n$0.5 \\times 0.5 \\times 0.5$\n\n$=0.25 \\times 0.5$\n\n$=0.125\\left(d m^{3}\\right)$\n\n(3)表面积:\n\n$10 \\times 8 \\times 2+5 \\times 10 \\times 2+8 \\times 5 \\times 2+5 \\times 5 \\times 2+5 \\times(12-8) \\times 4-5 \\times 5 \\times 2$\n\n$=160+100+80+80$\n\n$=260+160$\n$=420$ (平方厘米)\n\n体积:\n\n$12 \\times 10 \\times 5-5 \\times(12-8) \\times 5$\n\n$=600-100$\n\n$=500$ (立方厘米)\n\n答:表面积 420 平方厘米;体积 500 立方厘米。", "solution": "null", "level": "五年级", "question": "(2022 春・乐清市校级月考)求出如图图形的表面积和体积。\n", "options": [], "subject": "立体几何学", "analysis": "解(1)长方体的表面积:\n\n$(5 \\times 5+12 \\times 5+12 \\times 5) \\times 2$\n\n$=(25+60+60) \\times 2$\n\n$=145 \\times 2$\n\n$=290\\left(\\mathrm{~cm}^{2}\\right)$\n\n长方体的体积:\n\n$12 \\times 5 \\times 5$\n\n$=60 \\times 5$\n\n$=300\\left(\\mathrm{~cm}^{3}\\right)$\n\n(2)正方体的表面积:\n\n$0.25 \\times 6=1.5\\left(\\mathrm{dm}^{2}\\right)$\n\n正方体的体积:\n\n$0.5 \\times 0.5 \\times 0.5$\n\n$=0.25 \\times 0.5$\n\n$=0.125\\left(d m^{3}\\right)$\n\n(3)表面积:\n\n$10 \\times 8 \\times 2+5 \\times 10 \\times 2+8 \\times 5 \\times 2+5 \\times 5 \\times 2+5 \\times(12-8) \\times 4-5 \\times 5 \\times 2$\n\n$=160+100+80+80$\n\n$=260+160$\n$=420$ (平方厘米)\n\n体积:\n\n$12 \\times 10 \\times 5-5 \\times(12-8) \\times 5$\n\n$=600-100$\n\n$=500$ (立方厘米)\n\n答:表面积 420 平方厘米;体积 500 立方厘米。"} {"id": "24623", "image": [], "answer": "解$5 ; 0 ; 5 ; 16.2 ; 9.4$\n\n$0.06 ; 22 ; 4 ; 30 ; 0.16$\n\n【分析】根据小数乘除法的计算方法,直接进行口算即可。\n\n【详解】 $25 \\times 0.2=5 \\quad 0 \\times 5.8=0 \\quad 1.25 \\times 4=5 \\quad 4.05 \\times 4=16.2 \\quad 0.6+4.4 \\times 2=0.6+8.8=9.4$ $6 \\div 100=0.06 \\quad 2.2 \\div 0.1=22 \\quad 2.4 \\div 0.6=4 \\quad 0.6 \\div 0.02=30 \\quad 5 \\times 0.4 \\div 5 \\times 0.4=5 \\div 5 \\times 0.4 \\times 0.4=0.16$\n\n【点睛】本题考查了小数乘除法的口算, 计算时要认真。", "solution": "null", "level": "五年级", "question": "直接写得数。\n\n$25 \\times 0.2=0 \\times 5.8=1.25 \\times 4=4.05 \\times 4=0.6+4.4 \\times 2=$ $6 \\div 100=2.2 \\div 0.1=2.4 \\div 0.6=0.6 \\div 0.02=5 \\times 0.4 \\div 5 \\times 0.4=$", "options": [], "subject": "算术", "analysis": "解$5 ; 0 ; 5 ; 16.2 ; 9.4$\n\n$0.06 ; 22 ; 4 ; 30 ; 0.16$\n\n【分析】根据小数乘除法的计算方法,直接进行口算即可。\n\n【详解】 $25 \\times 0.2=5 \\quad 0 \\times 5.8=0 \\quad 1.25 \\times 4=5 \\quad 4.05 \\times 4=16.2 \\quad 0.6+4.4 \\times 2=0.6+8.8=9.4$ $6 \\div 100=0.06 \\quad 2.2 \\div 0.1=22 \\quad 2.4 \\div 0.6=4 \\quad 0.6 \\div 0.02=30 \\quad 5 \\times 0.4 \\div 5 \\times 0.4=5 \\div 5 \\times 0.4 \\times 0.4=0.16$\n\n【点睛】本题考查了小数乘除法的口算, 计算时要认真。"} {"id": "24625", "image": [], "answer": "解\n (1) 77.77 ;\n (2) 10.1 ;\n (3) 1\n\n【分析】(1)先将 101 写成 $100+1$, 再根据乘法分配律, 展开计算即可;\n\n(2)根据乘法分配律将 1.01 先提出来,再计算;\n\n(3) 将 32 写成 $4 \\times 8$, 再根据乘法交换律和结合律, 分别计算 $0.125 \\times 8$ 和 $4 \\times 0.25$, 再计算括号外的乘法。\n\n【详解】(1) $0.77 \\times 101$\n\n$=0.77 \\times(100+1)$\n\n$=0.77 \\times 100+0.77 \\times 1$\n\n$=77+0.77$\n\n$=77.77$\n\n(2) $1.01+9 \\times 1.01$\n\n$=1.01 \\times 1+9 \\times 1.01$\n\n$=(1+9) \\times 1.01$\n\n$=10 \\times 1.01$\n\n$=10.1$;\n\n(3) $0.125 \\times 32 \\times 0.25$\n\n$=0.125 \\times(8 \\times 4) \\times 0.25$\n\n$=(0.125 \\times 8) \\times(4 \\times 0.25)$\n\n$=1 \\times 1$\n$=1$", "solution": "null", "level": "五年级", "question": "简便计算。\n(1) $0.77 \\times 101$\n(2) $1.01+9 \\times 1.01$\n(3) $0.125 \\times 32 \\times 0.25$", "options": [], "subject": "算术", "analysis": "解\n (1) 77.77 ;\n (2) 10.1 ;\n (3) 1\n\n【分析】(1)先将 101 写成 $100+1$, 再根据乘法分配律, 展开计算即可;\n\n(2)根据乘法分配律将 1.01 先提出来,再计算;\n\n(3) 将 32 写成 $4 \\times 8$, 再根据乘法交换律和结合律, 分别计算 $0.125 \\times 8$ 和 $4 \\times 0.25$, 再计算括号外的乘法。\n\n【详解】(1) $0.77 \\times 101$\n\n$=0.77 \\times(100+1)$\n\n$=0.77 \\times 100+0.77 \\times 1$\n\n$=77+0.77$\n\n$=77.77$\n\n(2) $1.01+9 \\times 1.01$\n\n$=1.01 \\times 1+9 \\times 1.01$\n\n$=(1+9) \\times 1.01$\n\n$=10 \\times 1.01$\n\n$=10.1$;\n\n(3) $0.125 \\times 32 \\times 0.25$\n\n$=0.125 \\times(8 \\times 4) \\times 0.25$\n\n$=(0.125 \\times 8) \\times(4 \\times 0.25)$\n\n$=1 \\times 1$\n$=1$"} {"id": "24780", "image": [], "answer": "解35 平方米\n【分析】一个长方形的长和宽都是以米为单位的质数, 周长是 24 米, 又长方形周长 $=($ 长+宽 $) \\times 2$ $=24$ 米, 则长十宽 $=24 \\div 2=12$ 米, 在自然数中, 除了 1 和它本身外, 没有别的因数的数为质数, 据此即可确定此长方形的长与宽是多少米, 然后求出面积即可。\n\n【详解】由于(长+宽) $\\times 2=24$ 米\n\n即长 + 宽 $=24 \\div 2=12$ 米\n\n长与宽分别是以米为单位的质数,\n\n$5+7=12$\n\n即宽是 5 米, 长是 7 米\n\n则长方形面积是 $5 \\times 7=35$ 平方米\n\n答:长方形的面积是 35 平方米。\n\n【点睛】此题考查的是已知长方形周长求面积, 解答此题关键是根据长方形周长公式及质数的意义确定长方形的长与宽是完成本题的关键。", "solution": "null", "level": "五年级", "question": "一个长方形的长和宽都是以米为单位的质数, 周长是 24 米, 这个长方形的面积是多少?", "options": [], "subject": "算术", "analysis": "解35 平方米\n【分析】一个长方形的长和宽都是以米为单位的质数, 周长是 24 米, 又长方形周长 $=($ 长+宽 $) \\times 2$ $=24$ 米, 则长十宽 $=24 \\div 2=12$ 米, 在自然数中, 除了 1 和它本身外, 没有别的因数的数为质数, 据此即可确定此长方形的长与宽是多少米, 然后求出面积即可。\n\n【详解】由于(长+宽) $\\times 2=24$ 米\n\n即长 + 宽 $=24 \\div 2=12$ 米\n\n长与宽分别是以米为单位的质数,\n\n$5+7=12$\n\n即宽是 5 米, 长是 7 米\n\n则长方形面积是 $5 \\times 7=35$ 平方米\n\n答:长方形的面积是 35 平方米。\n\n【点睛】此题考查的是已知长方形周长求面积, 解答此题关键是根据长方形周长公式及质数的意义确定长方形的长与宽是完成本题的关键。"} {"id": "24782", "image": [], "answer": "解见详解\n\n【分析】根据偶数的性质: 偶数的倍数是偶数, 偶数的和是偶数, 所以洋洋买了一些甜甜圈和三明治,花费的钱数仍是偶数, 偶数一偶数=偶数, 所以找回的钱数是偶数, 不能是 11; 由此即可判断。\n\n【详解】偶数+偶数=偶数, 偶数一偶数=偶数\n\n所以小明买了一些甜甜圈和三明治, 花费的钱数仍是偶数, 所以找回的钱数是偶数, 不能是 11。答:他付给营业员 50 元,找回 11 元,找得不对, 11 是奇数。\n\n【点睛】此题考查了奇数、偶数的性质, 明确数的奇偶性特点, 是解答此题的关键。", "solution": "null", "level": "五年级", "question": "洋洋到蛋糕店买面包。甜甜圈 2 元一个,奶油面包 3 元一个,三明治 10 元一个。如果买了一些甜甜圈和三明治,他付给营业员 50 元,找回了 11 元,找的对吗?写出你的理由。", "options": [], "subject": "算术", "analysis": "解见详解\n\n【分析】根据偶数的性质: 偶数的倍数是偶数, 偶数的和是偶数, 所以洋洋买了一些甜甜圈和三明治,花费的钱数仍是偶数, 偶数一偶数=偶数, 所以找回的钱数是偶数, 不能是 11; 由此即可判断。\n\n【详解】偶数+偶数=偶数, 偶数一偶数=偶数\n\n所以小明买了一些甜甜圈和三明治, 花费的钱数仍是偶数, 所以找回的钱数是偶数, 不能是 11。答:他付给营业员 50 元,找回 11 元,找得不对, 11 是奇数。\n\n【点睛】此题考查了奇数、偶数的性质, 明确数的奇偶性特点, 是解答此题的关键。"} {"id": "24781", "image": [], "answer": "解不亮; 亮\n\n【分析】弟弟拉第一下灯, 灯亮, 再拉一下, 灯不亮, 再拉一下, 灯亮, 可见灯是按亮、不亮、亮、不亮的顺序循环出现的, 所以拉奇数下灯亮, 偶数下灯不亮; 所以,拉了 8 下开关灯是不亮的,同种方法可求出拉 13 下灯的状态。\n\n【详解】拉奇数下灯变亮, 偶数下变为不亮;\n\n拉 8 下, 为偶数次, 所以这时灯是不亮的;\n\n拉 13 下, 为奇数次, 所以这时灯是亮的;\n\n答: 这时灯是不亮的, 13 下后这时灯是亮的。\n\n【点睛】完成本题的关键是明确开关拉偶数次开关的状态与原来相比不变,拉奇数次状态变化。", "solution": "null", "level": "五年级", "question": "傍晚弟弟开灯, 一连开了 8 下。请你说说这时灯亮了还是没亮。13 下呢?", "options": [], "subject": "逻辑题", "analysis": "解不亮; 亮\n\n【分析】弟弟拉第一下灯, 灯亮, 再拉一下, 灯不亮, 再拉一下, 灯亮, 可见灯是按亮、不亮、亮、不亮的顺序循环出现的, 所以拉奇数下灯亮, 偶数下灯不亮; 所以,拉了 8 下开关灯是不亮的,同种方法可求出拉 13 下灯的状态。\n\n【详解】拉奇数下灯变亮, 偶数下变为不亮;\n\n拉 8 下, 为偶数次, 所以这时灯是不亮的;\n\n拉 13 下, 为奇数次, 所以这时灯是亮的;\n\n答: 这时灯是不亮的, 13 下后这时灯是亮的。\n\n【点睛】完成本题的关键是明确开关拉偶数次开关的状态与原来相比不变,拉奇数次状态变化。"} {"id": "25038", "image": ["12691.jpg", "12692.jpg", "12693.jpg", "12693.jpg"], "answer": "解4 种 如图\n\n\n\n【解析】略", "solution": "null", "level": "五年级", "question": "在下面的方格纸上画面积是 $24 \\mathrm{~cm}^{2}$ 的长方形(边长是整数, 每个小方格的边长是 $1 \\mathrm{~cm}$ ), 你有几种不同的画法?\n\n\n\n", "options": [], "subject": "组合数学", "analysis": "解4 种 如图\n\n\n\n【解析】略"} {"id": "18439", "image": [], "answer": "由条件知: $\\cos A=\\frac{A B^{2}+A C^{2}-B C^{2}}{2 \\cdot A B \\cdot A C}=\\frac{9^{2}+8^{2}-7^{2}}{2 \\times 9 \\times 8}=\\frac{2}{3}$, 设中线长为 $x$, 由余弦定理知: $x^{2}=\\left(\\frac{A C}{2}\\right)^{2}+A B^{2}-2 \\cdot \\frac{A C}{2} \\cdot A B \\cos A=4^{2}+9^{2}-2 \\times 4 \\times 9 \\times \\frac{2}{3}=49$ $\\Rightarrow x=7$.\n\n所以, 所求中线长为 7 .", "solution": "null", "level": "高二", "question": "11. 在 $\\triangle A B C$ 中, 已知 $C B=7, A C=8, A B=9$, 试求 $A C$ 边上的中线长.", "options": [], "subject": "解析几何", "analysis": "由条件知: $\\cos A=\\frac{A B^{2}+A C^{2}-B C^{2}}{2 \\cdot A B \\cdot A C}=\\frac{9^{2}+8^{2}-7^{2}}{2 \\times 9 \\times 8}=\\frac{2}{3}$, 设中线长为 $x$, 由余弦定理知: $x^{2}=\\left(\\frac{A C}{2}\\right)^{2}+A B^{2}-2 \\cdot \\frac{A C}{2} \\cdot A B \\cos A=4^{2}+9^{2}-2 \\times 4 \\times 9 \\times \\frac{2}{3}=49$ $\\Rightarrow x=7$.\n\n所以, 所求中线长为 7 ."} {"id": "18440", "image": [], "answer": "(1) $\\cos C=\\cos [\\pi-(A+B)]$\n\n$=-\\cos (A+B)=-\\frac{1}{2}$,\n\n又 $\\because C \\in\\left(0^{\\circ}, 180^{\\circ}\\right), \\therefore C=120^{\\circ}$.\n\n(2) $\\because a, b$ 是方程 $x^{2}-2 \\sqrt{3} x+2=0$ 的两根,\n\n$\\therefore\\left\\{\\begin{array}{l}a+b=2 \\sqrt{3}, \\\\ a b=2 .\\end{array}\\right.$\n\n$\\therefore A B^{2}=b^{2}+a^{2}-2 a b \\cos 120^{\\circ}=(a+b)^{2}-a b=10$,\n\n$\\therefore A B=\\sqrt{10}$.\n\n(3) $S_{\\triangle A B C}=\\frac{1}{2} a b \\sin C=\\frac{\\sqrt{3}}{2}$.", "solution": "null", "level": "高二", "question": "12. 在 $\\triangle A B C$ 中, $B C=a, A C=b$, 且 $a, b$ 是方程 $x^{2}-2 \\sqrt{3} x+2=0$ 的两根, $2 \\cos (A+$ $B)=1$.\n\n(1)求角 $C$ 的度数;\n\n(2)求 $A B$ 的长;\n\n(3)求 $\\triangle A B C$ 的面积.", "options": [], "subject": "解析几何", "analysis": "(1) $\\cos C=\\cos [\\pi-(A+B)]$\n\n$=-\\cos (A+B)=-\\frac{1}{2}$,\n\n又 $\\because C \\in\\left(0^{\\circ}, 180^{\\circ}\\right), \\therefore C=120^{\\circ}$.\n\n(2) $\\because a, b$ 是方程 $x^{2}-2 \\sqrt{3} x+2=0$ 的两根,\n\n$\\therefore\\left\\{\\begin{array}{l}a+b=2 \\sqrt{3}, \\\\ a b=2 .\\end{array}\\right.$\n\n$\\therefore A B^{2}=b^{2}+a^{2}-2 a b \\cos 120^{\\circ}=(a+b)^{2}-a b=10$,\n\n$\\therefore A B=\\sqrt{10}$.\n\n(3) $S_{\\triangle A B C}=\\frac{1}{2} a b \\sin C=\\frac{\\sqrt{3}}{2}$."} {"id": "18453", "image": [], "answer": "右边 $=\\frac{\\sin A \\cos B-\\cos A \\sin B}{\\sin C}=\\frac{\\sin A}{\\sin C} \\cdot \\cos B-\\frac{\\sin B}{\\sin C} \\cdot \\cos A$\n\n$=\\frac{a}{c} \\cdot \\frac{a^{2}+c^{2}-b^{2}}{2 a c}-\\frac{b}{c} \\cdot \\frac{b^{2}+c^{2}-a^{2}}{2 b c}=\\frac{a^{2}+c^{2}-b^{2}}{2 c^{2}}-\\frac{b^{2}+c^{2}-a^{2}}{2 c^{2}}=\\frac{a^{2}-b^{2}}{c^{2}}=$ 左边.\n\n所以 $\\frac{a^{2}-b^{2}}{c^{2}}=\\frac{\\sin (A-B)}{\\sin C}$.", "solution": "null", "level": "高二", "question": "11. 在 $\\triangle A B C$ 中, 求证: $\\frac{a^{2}-b^{2}}{c^{2}}=\\frac{\\sin (A-B)}{\\sin C}$.", "options": [], "subject": "解析几何", "analysis": "右边 $=\\frac{\\sin A \\cos B-\\cos A \\sin B}{\\sin C}=\\frac{\\sin A}{\\sin C} \\cdot \\cos B-\\frac{\\sin B}{\\sin C} \\cdot \\cos A$\n\n$=\\frac{a}{c} \\cdot \\frac{a^{2}+c^{2}-b^{2}}{2 a c}-\\frac{b}{c} \\cdot \\frac{b^{2}+c^{2}-a^{2}}{2 b c}=\\frac{a^{2}+c^{2}-b^{2}}{2 c^{2}}-\\frac{b^{2}+c^{2}-a^{2}}{2 c^{2}}=\\frac{a^{2}-b^{2}}{c^{2}}=$ 左边.\n\n所以 $\\frac{a^{2}-b^{2}}{c^{2}}=\\frac{\\sin (A-B)}{\\sin C}$."} {"id": "18454", "image": [], "answer": "(1) $\\because \\overrightarrow{A B} \\cdot \\overrightarrow{B C}=-21, \\therefore \\overrightarrow{B A} \\cdot \\overrightarrow{B C}=21$.\n\n$\\therefore \\overrightarrow{B A} \\cdot \\overrightarrow{B C}=|\\overrightarrow{B A}| \\cdot|\\overrightarrow{B C}| \\cdot \\cos \\mathrm{B}=\\operatorname{accos} \\mathrm{B}=21$.\n\n$\\therefore \\mathrm{ac}=35, \\because \\cos \\mathrm{B}=\\frac{3}{5}, \\therefore \\quad \\sin \\mathrm{B}=\\frac{4}{5}$.\n\n$\\therefore \\mathrm{S} \\triangle \\mathrm{ABC}=\\frac{1}{2} \\mathrm{acsin} \\mathrm{B}=\\frac{1}{2} \\times 35 \\times \\frac{4}{5}=14$.\n\n(2) $a c=35, a=7, \\therefore c=5$.\n\n由余弦定理得, $b^{2}=a^{2}+c^{2}-2 a c \\cos B=32$,\n\n$\\therefore b=4 \\sqrt{2}$. 由正弦定理: $\\frac{c}{\\sin C}=\\frac{b}{\\sin B}$.\n\n$\\therefore \\sin C=\\frac{c}{b} \\sin B=\\frac{5}{4 \\sqrt{2}} \\times \\frac{4}{5}=\\frac{\\sqrt{2}}{2}$.\n\n$\\because c\\sin \\frac{7 \\pi}{3}$, 利用三角函数线得角 $\\theta$ 的取值范围是\n\n解析 : $\\cos \\theta>\\sin \\frac{7}{3} \\pi=\\sin \\frac{\\pi}{3}=\\frac{\\sqrt{3}}{2}$.\n$\\therefore 2 k \\pi-\\frac{\\pi}{6}<\\theta<2 k \\pi+\\frac{\\pi}{6}, \\quad k \\in \\mathbf{Z}$.", "options": [], "subject": "解析几何", "analysis": "$\\left(2 k \\pi-\\frac{\\pi}{6}, 2 k \\pi+\\frac{\\pi}{6}\\right)(k \\in \\mathbf{Z})$"} {"id": "17740", "image": ["8961.jpg", "8961.jpg"], "answer": "如图, 单位圆 $O$ 与 $x$ 轴正半轴交于点 $A$, 与角 $\\alpha, \\beta$ 的终边分别交于点 $Q, P$, 过 $P$, $Q$ 分别作 $O A$ 的垂线,设垂足分别为点 $M, N$, 则由三角函数线定义可知:\n\n$\\sin \\alpha=N Q, \\sin \\beta=M P$, 过点 $Q$ 作 $Q H \\perp M P$ 于点 $H$, 于是 $M H=N Q$, 则 $H P=M P-M H=$ $\\sin \\beta-\\sin \\alpha$.\n\n由图可知, $H P<\\overparen{P Q}=\\overparen{A P}-\\overparen{A Q}=\\beta-\\alpha$,\n\n即 $\\beta-\\alpha>\\sin \\beta-\\sin \\alpha$.", "solution": "null", "level": "高二", "question": "16. 利用三角函数线证明: 若 $0<\\alpha<\\beta<\\frac{\\pi}{2}$, 则 $\\beta-\\alpha>\\sin \\beta-\\sin \\alpha$.", "options": [], "subject": "解析几何", "analysis": "如图, 单位圆 $O$ 与 $x$ 轴正半轴交于点 $A$, 与角 $\\alpha, \\beta$ 的终边分别交于点 $Q, P$, 过 $P$, $Q$ 分别作 $O A$ 的垂线,设垂足分别为点 $M, N$, 则由三角函数线定义可知:\n\n$\\sin \\alpha=N Q, \\sin \\beta=M P$, 过点 $Q$ 作 $Q H \\perp M P$ 于点 $H$, 于是 $M H=N Q$, 则 $H P=M P-M H=$ $\\sin \\beta-\\sin \\alpha$.\n\n由图可知, $H P<\\overparen{P Q}=\\overparen{A P}-\\overparen{A Q}=\\beta-\\alpha$,\n\n即 $\\beta-\\alpha>\\sin \\beta-\\sin \\alpha$."} {"id": "17748", "image": ["8963.jpg", "8964.jpg"], "answer": "", "solution": "null", "level": "高二", "question": "9. 在单位圆中画出适合下列条件的角 $\\alpha$ 的终边.\n\n(1) $\\sin \\alpha=\\frac{2}{3}$;\n\n(2) $\\cos \\alpha=-\\frac{3}{5}$.", "options": [], "subject": "解析几何", "analysis": "(1)作直线 $y=\\frac{2}{3}$ 交单位圆于 $P, Q$ 两点, 则 $O P, O Q$ 为角 $\\alpha$ 的终边, 如图甲.\n\n(2)作直线 $x=-\\frac{3}{5}$ 交单位圆于 $M, N$ 两点, 则 $O M, O N$ 为角 $\\alpha$ 的终边, 如图乙."} {"id": "17777", "image": [], "answer": "$\\frac{5}{13}$", "solution": "null", "level": "高二", "question": "10. 已知 $\\sin \\left(-\\frac{\\pi}{2}-\\alpha\\right) \\cdot \\cos \\left(-\\frac{5 \\pi}{2}-\\alpha\\right)=\\frac{60}{169}$, 且 $\\frac{\\pi}{4}<\\alpha<\\frac{\\pi}{2}$, 求 $\\sin \\alpha$ 与 $\\cos \\alpha$ 的值.", "options": [], "subject": "解析几何", "analysis": "$\\because \\sin \\left(-\\frac{\\pi}{2}-\\alpha\\right)=-\\cos \\alpha$, $\\cos \\left(-\\frac{5 \\pi}{2}-\\alpha\\right)=\\cos \\left(2 \\pi+\\frac{\\pi}{2}+\\alpha\\right)=-\\sin \\alpha$,\n\n$\\therefore \\sin \\alpha \\cdot \\cos \\alpha=\\frac{60}{169}$,\n\n即 $2 \\sin \\alpha \\cdot \\cos \\alpha=\\frac{120}{169}$. 1\n\n又 $\\because \\sin ^{2} \\alpha+\\cos ^{2} \\alpha=1$,\n\n(1) + (2) 得 $(\\sin \\alpha+\\cos \\alpha)^{2}=\\frac{289}{169}$,\n\n(2) $-(1)$ 得 $(\\sin \\alpha-\\cos \\alpha)^{2}=\\frac{49}{169}$.\n\n又 $\\because \\alpha \\in\\left(\\frac{\\pi}{4}, \\frac{\\pi}{2}\\right), \\therefore \\sin \\alpha>\\cos \\alpha>0$,\n\n即 $\\sin \\alpha+\\cos \\alpha>0, \\sin \\alpha-\\cos \\alpha>0$,\n$\\therefore \\sin \\alpha+\\cos \\alpha=\\frac{17}{13}$,\n\n$\\sin \\alpha-\\cos \\alpha=\\frac{7}{13}$,\n\n(3)+(4)得 $\\sin \\alpha=\\frac{12}{13}$,\n\n(3)-(4) 得 $\\cos \\alpha=\\frac{5}{13}$."} {"id": "17781", "image": [], "answer": "$f(\\alpha)=\\frac{\\sin (\\pi-\\alpha) \\cos (2 \\pi-\\alpha)}{\\cos \\left(\\frac{\\pi}{2}-\\alpha\\right) \\sin (-\\pi-\\alpha) \\cos (2 \\pi+\\alpha)}$\n\n$=\\frac{\\sin \\alpha \\cos \\alpha}{-\\sin \\alpha \\sin (\\pi+\\alpha) \\cos \\alpha}=\\frac{1}{\\sin \\alpha}$.\n\n(1) $\\because \\cos \\left(\\alpha-\\frac{3 \\pi}{2}\\right)=\\frac{1}{5}$,\n\n$\\therefore \\cos \\left(\\alpha-\\frac{3 \\pi}{2}+2 \\pi\\right)=\\frac{1}{5}$,\n\n$\\therefore \\cos \\left(\\frac{\\pi}{2}+\\alpha\\right)=\\frac{1}{5}$,\n\n$\\therefore \\sin \\alpha=-\\frac{1}{5}, \\therefore f(\\alpha)=\\frac{1}{\\sin \\alpha}=-5$.\n\n(2)当 $\\alpha=-1860^{\\circ}$ 时, $f(\\alpha)=\\frac{1}{\\sin \\alpha}$\n\n$=\\frac{1}{\\sin \\left(-1860^{\\circ}\\right)}=\\frac{1}{-\\sin 1860^{\\circ}}$\n\n$=\\frac{1}{-\\sin \\left(5 \\times 360^{\\circ}+60^{\\circ}\\right)}=\\frac{1}{-\\sin 60^{\\circ}}=-\\frac{2 \\sqrt{3}}{3}$.", "solution": "null", "level": "高二", "question": "15. 已知 $\\alpha$ 是第四象限角, 且\n\n$f(\\alpha)=\\frac{\\sin (\\pi-\\alpha) \\cos (2 \\pi-\\alpha)}{\\cos \\left(\\frac{\\pi}{2}-\\alpha\\right) \\sin (-\\pi-\\alpha) \\cos (2 \\pi+\\alpha)}$.\n\n(1)若 $\\cos \\left(\\alpha-\\frac{3 \\pi}{2}\\right)=\\frac{1}{5}$, 求 $f(\\alpha)$ 的值;\n\n(2)若 $\\alpha=-1860^{\\circ}$, 求 $f(\\alpha)$ 的值.", "options": [], "subject": "解析几何", "analysis": "$f(\\alpha)=\\frac{\\sin (\\pi-\\alpha) \\cos (2 \\pi-\\alpha)}{\\cos \\left(\\frac{\\pi}{2}-\\alpha\\right) \\sin (-\\pi-\\alpha) \\cos (2 \\pi+\\alpha)}$\n\n$=\\frac{\\sin \\alpha \\cos \\alpha}{-\\sin \\alpha \\sin (\\pi+\\alpha) \\cos \\alpha}=\\frac{1}{\\sin \\alpha}$.\n\n(1) $\\because \\cos \\left(\\alpha-\\frac{3 \\pi}{2}\\right)=\\frac{1}{5}$,\n\n$\\therefore \\cos \\left(\\alpha-\\frac{3 \\pi}{2}+2 \\pi\\right)=\\frac{1}{5}$,\n\n$\\therefore \\cos \\left(\\frac{\\pi}{2}+\\alpha\\right)=\\frac{1}{5}$,\n\n$\\therefore \\sin \\alpha=-\\frac{1}{5}, \\therefore f(\\alpha)=\\frac{1}{\\sin \\alpha}=-5$.\n\n(2)当 $\\alpha=-1860^{\\circ}$ 时, $f(\\alpha)=\\frac{1}{\\sin \\alpha}$\n\n$=\\frac{1}{\\sin \\left(-1860^{\\circ}\\right)}=\\frac{1}{-\\sin 1860^{\\circ}}$\n\n$=\\frac{1}{-\\sin \\left(5 \\times 360^{\\circ}+60^{\\circ}\\right)}=\\frac{1}{-\\sin 60^{\\circ}}=-\\frac{2 \\sqrt{3}}{3}$."} {"id": "17790", "image": [], "answer": "$-\\frac{2}{9}$", "solution": "null", "level": "高二", "question": "9. 已知 $\\cos \\left(\\frac{\\pi}{4}-\\alpha\\right)=\\frac{1}{3}$, 求 $\\cos ^{2}\\left(\\frac{3 \\pi}{4}+\\alpha\\right)-\\sin \\left(\\alpha+\\frac{\\pi}{4}\\right)$ 的值.", "options": [], "subject": "解析几何", "analysis": "$\\because \\cos \\left(\\frac{3 \\pi}{4}+\\alpha\\right)$\n\n$=\\cos \\left[\\pi-\\left(\\frac{\\pi}{4}-\\alpha\\right)\\right]$\n\n$=-\\cos \\left(\\frac{\\pi}{4}-\\alpha\\right)=-\\frac{1}{3}$,\n\n$\\therefore \\cos ^{2}\\left(\\frac{3 \\pi}{4}+\\alpha\\right)=\\frac{1}{9}$.\n\n又 $\\because \\sin \\left(\\alpha+\\frac{\\pi}{4}\\right)=\\sin \\left[\\frac{\\pi}{2}-\\left(\\frac{\\pi}{4}-\\alpha\\right)\\right]=\\cos \\left(\\frac{\\pi}{4}-\\alpha\\right)=\\frac{1}{3}$,\n\n$\\therefore \\cos ^{2}\\left(\\frac{3 \\pi}{4}+\\alpha\\right)-\\sin \\left(\\alpha+\\frac{\\pi}{4}\\right)=\\frac{1}{9}-\\frac{1}{3}=-\\frac{2}{9}$."} {"id": "17809", "image": [], "answer": "$\\left\\{\\left.x\\right|_{x=k \\pi-\\frac{\\pi}{2}}\\right.$ 或 $\\left.x=k \\pi-\\frac{\\pi}{6}, k \\in \\mathbf{Z}\\right\\}$.", "solution": "null", "level": "高二", "question": "16. 已知函数 $f(x)=\\cos \\left(2 x+\\frac{\\pi}{3}\\right)$, 若函数 $g(x)$ 的最小正周期是 $\\pi$, 且当 $x \\in\\left[-\\frac{\\pi}{2}, \\frac{\\pi}{2}\\right]$ 时, $g(x)=$ $f\\left(\\frac{x}{2}\\right)$, 求关于 $x$ 的方程 $g(x)=\\frac{\\sqrt{3}}{2}$ 的解集.", "options": [], "subject": "解析几何", "analysis": "当 $x \\in\\left[-\\frac{\\pi}{2}, \\frac{\\pi}{2}\\right]$ 时, $g(x)=f\\left(\\frac{x}{2}\\right)=\\cos \\left(x+\\frac{\\pi}{3}\\right)$.\n\n因为 $x+\\frac{\\pi}{3} \\in\\left[-\\frac{\\pi}{6}, \\frac{5 \\pi}{6}\\right]$,\n\n所以由 $g(x)=\\frac{\\sqrt{3}}{2}$ 解得 $x+\\frac{\\pi}{3}=-\\frac{\\pi}{6}$ 或 $\\frac{\\pi}{6}$,\n\n即 $x=-\\frac{\\pi}{2}$ 或 $-\\frac{\\pi}{6}$.\n\n又因为 $g(x)$ 的最小正周期为 $\\pi$.\n\n所以 $g(x)=\\frac{\\sqrt{3}}{2}$ 的解集为 $\\left\\{\\left.x\\right|_{x=k \\pi-\\frac{\\pi}{2}}\\right.$ 或 $\\left.x=k \\pi-\\frac{\\pi}{6}, k \\in \\mathbf{Z}\\right\\}$.\n\n##第一章 三角函数"} {"id": "17815", "image": [], "answer": "$f(x)=\\sin |x|$", "solution": "null", "level": "高二", "question": "8. 若 $f(x)$ 是 $\\mathbf{R}$ 上的偶函数, 当 $x \\geqslant 0$ 时, $f(x)=\\sin x$, 则 $f(x)$ 的解析式是", "options": [], "subject": "解析几何", "analysis": "解析:当 $x<0$ 时, $-x>0, f(-x)=\\sin (-x)=-\\sin x$.\n\n$\\because f(-x)=f(x), \\quad \\therefore x<0$ 时, $f(x)=-\\sin x$.\n\n$\\therefore f(x)=\\sin |x|$."} {"id": "17821", "image": [], "answer": "令 $\\cos x=t, t \\in[-1,1]$, 则 $y=2 t^{2}-2 a t-(2 a+1)$, 函数图象的对称轴为直线 $t=\\frac{a}{2}$.当 $\\frac{a}{2}<-1$, 即 $a<-2$ 时, 函数 $y$ 在 $[-1,1]$ 上单调递增, $y_{\\text {min }}=1 \\neq \\frac{1}{2}$;\n\n当 $\\frac{a}{2}>1$, 即 $a>2$ 时, 函数 $y$ 在 $[-1,1]$ 上单调递减, $y_{\\min }=-4 a+1=\\frac{1}{2}$, 解得 $a=\\frac{1}{8}$, 与 $a>2$矛盾;\n\n当 $-1 \\leqslant \\frac{a}{2} \\leqslant 1$, 即 $-2 \\leqslant a \\leqslant 2$ 时, $y_{\\text {min }}=-\\frac{a^{2}}{2}-2 a-1=\\frac{1}{2}, a^{2}+4 a+3=0$, 解得 $a=-1$ 或 $a$ $=-3, \\quad \\therefore a=-1$.\n\n此时 $y_{\\text {max }}=-4 a+1=5$.\n\n综上可知, 满足 $f(a)=\\frac{1}{2}$ 的 $a$ 的值为 -1 , 此时 $y$ 的最大值为 5 .", "solution": "null", "level": "高二", "question": "16. 关于 $x$ 的函数 $y=2 \\cos ^{2} x-2 a \\cos x-(2 a+1)$ 的最小值为 $f(a)$, 试确定满足 $f(a)=\\frac{1}{2}$ 的 $a$ 的值,并对此时的 $a$ 值求 $y$ 的最大值.", "options": [], "subject": "解析几何", "analysis": "令 $\\cos x=t, t \\in[-1,1]$, 则 $y=2 t^{2}-2 a t-(2 a+1)$, 函数图象的对称轴为直线 $t=\\frac{a}{2}$.当 $\\frac{a}{2}<-1$, 即 $a<-2$ 时, 函数 $y$ 在 $[-1,1]$ 上单调递增, $y_{\\text {min }}=1 \\neq \\frac{1}{2}$;\n\n当 $\\frac{a}{2}>1$, 即 $a>2$ 时, 函数 $y$ 在 $[-1,1]$ 上单调递减, $y_{\\min }=-4 a+1=\\frac{1}{2}$, 解得 $a=\\frac{1}{8}$, 与 $a>2$矛盾;\n\n当 $-1 \\leqslant \\frac{a}{2} \\leqslant 1$, 即 $-2 \\leqslant a \\leqslant 2$ 时, $y_{\\text {min }}=-\\frac{a^{2}}{2}-2 a-1=\\frac{1}{2}, a^{2}+4 a+3=0$, 解得 $a=-1$ 或 $a$ $=-3, \\quad \\therefore a=-1$.\n\n此时 $y_{\\text {max }}=-4 a+1=5$.\n\n综上可知, 满足 $f(a)=\\frac{1}{2}$ 的 $a$ 的值为 -1 , 此时 $y$ 的最大值为 5 ."} {"id": "17834", "image": [], "answer": "(1)", "solution": "null", "level": "高二", "question": "14.关于函数 $f(x)=\\tan \\left(2 x-\\frac{\\pi}{4}\\right)$ 有以下命题:\n\n(1)函数 $f(x)$ 的周期是 $\\frac{\\pi}{2}$;\n\n(2)函数 $f(x)$ 的定义域是 $\\left\\{x \\mid x \\in \\mathbf{R}\\right.$ 且 $\\left.x \\neq \\frac{k \\pi}{2}+\\frac{\\pi}{8}, k \\in \\mathbf{Z}\\right\\}$;\n\n(3) $y=f(x)$ 是奇函数;\n\n(4) $y=f(x)$ 的一个单调递增区间为 $\\left(-\\frac{\\pi}{2}, \\frac{\\pi}{2}\\right)$.\n\n其中,正确的命题是", "options": [], "subject": "解析几何", "analysis": "解析: $f(x)=\\tan \\left(2 x-\\frac{\\pi}{4}\\right)$ 的周期 $T=\\frac{\\pi}{2}$ ,故 (1) 正确; $f(x)$ 的定义域为 $\\left\\{x \\mid x \\in \\mathbf{R}\\right.$ 且 $\\left.x \\neq \\frac{k \\pi}{2}+\\frac{3 \\pi}{8} , k \\in \\mathbf{Z}\\right\\}$ ,故(2)不正确; $f(x)$ 是非奇非偶函数,故(3)不正确; $f(x)$ 的单调递增区间为 $\\left(\\frac{k \\pi}{2}-\\frac{\\pi}{8}, \\frac{k \\pi}{2}+\\frac{3}{8} \\pi\\right) , k \\in \\mathbf{Z}$ ,故(4)不正确."} {"id": "17835", "image": [], "answer": "设 $t=\\tan x$. 因为 $|x| \\leqslant \\frac{\\pi}{4}$ ,所以 $\\tan x \\in[-1,1]$ ,则原函数化为 $y=t^{2}-a t=\\left(t-\\frac{a}{2}\\right)^{2}$ $\\frac{a^{2}}{4}$, 对称轴方程为 $t=\\frac{a}{2}$.\n\n(1) 若 $-1 \\leqslant \\frac{a}{2} \\leqslant 1$ ,即 $-2 \\leqslant a \\leqslant 2$ ,\n\n则当 $t=\\frac{a}{2}$ 时, $y_{\\text {min }}=-\\frac{a^{2}}{4}=-6$ ,\n\n所以 $a^{2}=24$ ,不符合题意,舍去;\n\n(2)若 $\\frac{a}{2}<-1$ ,即 $a<-2$ ,则二次函数在 $[-1 , 1]$ 上单调递增,当 $t=-1$ 时, $y_{\\text {min }}=1+a=-$\n\n6, 所以 $a=-7$;\n\n(3) 若 ${ }_{2}^{a}>1$ ,即 $a>2$ ,则二次函数在 $[-1 , 1]$ 上单调递减,\n\n当 $t=1$ 时, $y_{\\text {min }}=1-a=-6$, 所以 $a=7$.\n\n综上所述,实数 $a$ 的值为 -7 或 7", "solution": "null", "level": "高二", "question": "16. 若函数 $y=\\tan ^{2} x-a \\tan x\\left(|x| \\leqslant \\frac{\\pi}{4}\\right)$ 的最小值为 -6 , 求实数 $a$ 的值.", "options": [], "subject": "解析几何", "analysis": "设 $t=\\tan x$. 因为 $|x| \\leqslant \\frac{\\pi}{4}$ ,所以 $\\tan x \\in[-1,1]$ ,则原函数化为 $y=t^{2}-a t=\\left(t-\\frac{a}{2}\\right)^{2}$ $\\frac{a^{2}}{4}$, 对称轴方程为 $t=\\frac{a}{2}$.\n\n(1) 若 $-1 \\leqslant \\frac{a}{2} \\leqslant 1$ ,即 $-2 \\leqslant a \\leqslant 2$ ,\n\n则当 $t=\\frac{a}{2}$ 时, $y_{\\text {min }}=-\\frac{a^{2}}{4}=-6$ ,\n\n所以 $a^{2}=24$ ,不符合题意,舍去;\n\n(2)若 $\\frac{a}{2}<-1$ ,即 $a<-2$ ,则二次函数在 $[-1 , 1]$ 上单调递增,当 $t=-1$ 时, $y_{\\text {min }}=1+a=-$\n\n6, 所以 $a=-7$;\n\n(3) 若 ${ }_{2}^{a}>1$ ,即 $a>2$ ,则二次函数在 $[-1 , 1]$ 上单调递减,\n\n当 $t=1$ 时, $y_{\\text {min }}=1-a=-6$, 所以 $a=7$.\n\n综上所述,实数 $a$ 的值为 -7 或 7 ."} {"id": "17850", "image": [], "answer": "(1)根据表中已知数据,解得 $A=5 , \\omega=2 , \\varphi=-\\frac{\\pi}{6}$ 数据补全如下表:\n\n且函数解析式为 $f(x)=5 \\sin \\left(2 x-\\frac{\\pi}{6}\\right)$.\n\n(2)由(1)知 $f(x)=5 \\sin \\left(2 x-\\frac{\\pi}{6}\\right)$ ,\n\n则 $g(x)=5 \\sin \\left(2 x+2 \\theta-\\frac{\\pi}{6}\\right)$.\n\n因为函数 $y=\\sin x$ 图象的对称中心为 $(k \\pi , 0), k \\in \\mathbf{Z}$.\n\n令 $2 x+2 \\theta-\\frac{\\pi}{6}=k \\pi , k \\in \\mathbf{Z}$ ,解得 $x=\\frac{k \\pi}{2}+\\frac{\\pi}{12}-\\theta , k \\in \\mathbf{Z}$.\n\n由于函数 $y=g(x)$ 的图象关于点 $\\left(\\frac{5 \\pi}{12} , 0\\right)$ 成中心对称,所以令 $\\frac{k \\pi}{2}+\\frac{\\pi}{12}-\\theta=\\frac{5 \\pi}{12} , k \\in \\mathbf{Z}$ ,\n\n解得 $\\theta=\\frac{k \\pi}{2}-\\frac{\\pi}{3} , k \\in \\mathbf{Z}$.由 $\\theta>0$ 可知,当 $k=1$ 时, $\\theta$ 取得最小值 $\\frac{\\pi}{6}$.", "solution": "null", "level": "高二", "question": "16. 某同学用 “五点法” 画函数 $f(x)=A \\sin (\\omega x+\\varphi)\\left(\\omega>0,|\\varphi|<\\frac{\\pi}{2}\\right)$ 在某一个周期内的图象时, 列表并填入了部分数据, 如下表:\n\n(1)请将上表数据补充完整, 并直接写出函数 $f(x)$ 的解析式;\n\n(2) 将 $y=f(x)$ 图象上所有点向左平行移动 $\\theta(\\theta>0)$ 个单位长度, 得到 $y=g(x)$ 的图象. 若 $y=g(x)$图象的一个对称中心为 $\\left(\\frac{5 \\pi}{12}, 0\\right)$, 求 $\\theta$ 的最小值.", "options": [], "subject": "解析几何", "analysis": "(1)根据表中已知数据,解得 $A=5 , \\omega=2 , \\varphi=-\\frac{\\pi}{6}$ 数据补全如下表:\n\n且函数解析式为 $f(x)=5 \\sin \\left(2 x-\\frac{\\pi}{6}\\right)$.\n\n(2)由(1)知 $f(x)=5 \\sin \\left(2 x-\\frac{\\pi}{6}\\right)$ ,\n\n则 $g(x)=5 \\sin \\left(2 x+2 \\theta-\\frac{\\pi}{6}\\right)$.\n\n因为函数 $y=\\sin x$ 图象的对称中心为 $(k \\pi , 0), k \\in \\mathbf{Z}$.\n\n令 $2 x+2 \\theta-\\frac{\\pi}{6}=k \\pi , k \\in \\mathbf{Z}$ ,解得 $x=\\frac{k \\pi}{2}+\\frac{\\pi}{12}-\\theta , k \\in \\mathbf{Z}$.\n\n由于函数 $y=g(x)$ 的图象关于点 $\\left(\\frac{5 \\pi}{12} , 0\\right)$ 成中心对称,所以令 $\\frac{k \\pi}{2}+\\frac{\\pi}{12}-\\theta=\\frac{5 \\pi}{12} , k \\in \\mathbf{Z}$ ,\n\n解得 $\\theta=\\frac{k \\pi}{2}-\\frac{\\pi}{3} , k \\in \\mathbf{Z}$.由 $\\theta>0$ 可知,当 $k=1$ 时, $\\theta$ 取得最小值 $\\frac{\\pi}{6}$."} {"id": "17870", "image": [], "answer": "解析 : (1) $\\frac{T}{2}=\\frac{11}{12} \\pi-\\frac{5}{12} \\pi=\\frac{1}{2} \\pi$ ,\n\n$\\therefore T=\\pi, \\omega=\\frac{2 \\pi}{T}=2$,\n\n又 $\\sin \\left(2 \\cdot \\frac{5 \\pi}{12}+\\varphi\\right)=1 ,|\\varphi|<\\frac{\\pi}{2}$ ,\n\n$\\therefore \\varphi=-\\frac{\\pi}{3}, \\therefore f(x)=\\sin \\left(2 x-\\frac{\\pi}{3}\\right)$,\n\n$\\therefore g(x)=\\sin \\left(4 x+\\frac{\\pi}{6}\\right)$.\n\n(2) $g(x)$ 在 $\\left[0 , \\frac{\\pi}{12}\\right]$ 上为增函数,在 $\\left[\\frac{\\pi}{12} , \\frac{\\pi}{4}\\right]$ 上为减函数,所以 $g(x)_{\\text {max }}=g\\left(\\frac{\\pi}{12}\\right)=1$ ,\n\n又 $g(0)=\\frac{1}{2} , g\\left(\\frac{\\pi}{4}\\right)=-\\frac{1}{2}$ ,\n\n所以 $g(x)_{\\min }=-\\frac{1}{2}$,\n\n故函数 $g(x)$ 在区间 $\\left[0 , \\frac{\\pi}{4}\\right]$ 上的最大值和最小值分别为 1 和 $-\\frac{1}{2}$.", "solution": "null", "level": "高二", "question": "9. 函数 $f(x)=\\sin (\\omega x+\\varphi)\\left(\\omega>0,|\\varphi|<\\frac{\\pi}{2}\\right)$ 在它的某一个周期内的单调递减区间是 $\\left[\\frac{5 \\pi}{12}, \\frac{11 \\pi}{12}\\right]$ 将 $y=$ $f(x)$ 的图象先向左平移 $\\frac{\\pi}{4}$ 个单位长度, 再将图象上所有点的横坐标变为原来的 $\\frac{1}{2}$ ( 纵坐标不变), 所得到的图象对应的函数记为 $g(x)$.\n\n(1)求 $g(x)$ 的解析式;\n\n(2)求 $g(x)$ 在区间 $\\left[0, \\frac{\\pi}{4}\\right]$ 上的最大值和最小值.", "options": [], "subject": "解析几何", "analysis": "解析 : (1) $\\frac{T}{2}=\\frac{11}{12} \\pi-\\frac{5}{12} \\pi=\\frac{1}{2} \\pi$ ,\n\n$\\therefore T=\\pi, \\omega=\\frac{2 \\pi}{T}=2$,\n\n又 $\\sin \\left(2 \\cdot \\frac{5 \\pi}{12}+\\varphi\\right)=1 ,|\\varphi|<\\frac{\\pi}{2}$ ,\n\n$\\therefore \\varphi=-\\frac{\\pi}{3}, \\therefore f(x)=\\sin \\left(2 x-\\frac{\\pi}{3}\\right)$,\n\n$\\therefore g(x)=\\sin \\left(4 x+\\frac{\\pi}{6}\\right)$.\n\n(2) $g(x)$ 在 $\\left[0 , \\frac{\\pi}{12}\\right]$ 上为增函数,在 $\\left[\\frac{\\pi}{12} , \\frac{\\pi}{4}\\right]$ 上为减函数,所以 $g(x)_{\\text {max }}=g\\left(\\frac{\\pi}{12}\\right)=1$ ,\n\n又 $g(0)=\\frac{1}{2} , g\\left(\\frac{\\pi}{4}\\right)=-\\frac{1}{2}$ ,\n\n所以 $g(x)_{\\min }=-\\frac{1}{2}$,\n\n故函数 $g(x)$ 在区间 $\\left[0 , \\frac{\\pi}{4}\\right]$ 上的最大值和最小值分别为 1 和 $-\\frac{1}{2}$."} {"id": "17872", "image": ["9004.jpg"], "answer": "(1)由已知可设 $y=40.5-40 \\cos \\omega t , t \\geqslant 0$ ,由周期为 12 分钟可知,当 $t=6$ 时,摩天轮第 1 次到达最高点,即此函数第 1 次取得最大值,所以 $6 \\omega=\\pi$ ,即 $\\omega=\\frac{\\pi}{6}$ , 所以 $y=40.5$ - $40 \\cos \\frac{\\pi}{6} t(t \\geqslant 0)$.\n\n(2)设转第 1 圈时,第 $t_{0}$ 分钟时距离地面 60.5 米. 由 $60.5=40.5-40 \\cos \\frac{\\pi}{6} t_{0}$, 得 $\\cos \\frac{\\pi}{6} t_{0}=-\\frac{1}{2}$ ,\n所以 $\\frac{\\pi}{6} t_{0}=\\frac{2 \\pi}{3}$ 或 $\\frac{\\pi}{6} t_{0}=\\frac{4 \\pi}{3}$ ,解得 $t_{0}=4$ 或 $t_{0}=8$ ,所以 $t=8$ (分钟)时,第 2 次距地面 60.5 米,故第 4 次距离地面 60.5 米时,用了 $12+8=20$ (分钟)", "solution": "null", "level": "高二", "question": "10. 如图所示, 游乐场中的摩天轮匀速转动, 每转一圈需要 12 分钟, 其中心 $O$ 距离地面 40.5 米, 半径为 40 米. 如果你从最低处登上摩天轮, 那么你与地面的距离将随时间的变化而变化, 以你登上摩天轮的时刻开始计时, 请解答下列问题:\n\n\n\n(1)求出你与地面的距离 $y$ (米)与时间 $t$ (分钟)的函数关系式;\n\n(2)当你第 4 次距离地面 60.5 米时, 用了多长时间?", "options": [], "subject": "解析几何", "analysis": "解析: (1)由已知可设 $y=40.5-40 \\cos \\omega t , t \\geqslant 0$ ,由周期为 12 分钟可知,当 $t=6$ 时,摩天轮第 1 次到达最高点,即此函数第 1 次取得最大值,所以 $6 \\omega=\\pi$ ,即 $\\omega=\\frac{\\pi}{6}$ , 所以 $y=40.5$ - $40 \\cos \\frac{\\pi}{6} t(t \\geqslant 0)$.\n\n(2)设转第 1 圈时,第 $t_{0}$ 分钟时距离地面 60.5 米. 由 $60.5=40.5-40 \\cos \\frac{\\pi}{6} t_{0}$, 得 $\\cos \\frac{\\pi}{6} t_{0}=-\\frac{1}{2}$ ,\n所以 $\\frac{\\pi}{6} t_{0}=\\frac{2 \\pi}{3}$ 或 $\\frac{\\pi}{6} t_{0}=\\frac{4 \\pi}{3}$ ,解得 $t_{0}=4$ 或 $t_{0}=8$ ,所以 $t=8$ (分钟)时,第 2 次距地面 60.5 米,故第 4 次距离地面 60.5 米时,用了 $12+8=20$ (分钟)."} {"id": "17877", "image": ["9012.jpg", "9013.jpg", "9013.jpg"], "answer": "解析: (1)以圆心 $O$ 为原点,建立如图所示的坐标系,则以 $O x$ 为始边, $O B$ 为终边的角为 $\\theta-\\frac{\\pi}{2} ,$ 故点 $B$ 坐标为\n\n\n\n$\\left(4,.8 \\cos \\left(\\theta-\\frac{\\pi}{2}\\right), 4,.8 \\sin \\left(\\theta-\\frac{\\pi}{2}\\right)\\right)$.\n\n$\\therefore h=5,.6+4,.8 \\sin \\left(\\theta-\\frac{\\pi}{2}\\right)$.\n\n(2)点 $A$ 在圆上转动的角速度是 $\\frac{\\pi}{30}$ ,故 $t s$ 转过的弧度数为 $\\frac{\\pi t}{30}$.\n\n$\\therefore h=5,.6+4,.8 \\sin \\left(\\frac{\\pi}{30} t-\\frac{\\pi}{2}\\right), t \\in[0,+\\infty)$.\n\n到达最高点时, $h=10,.4 \\mathrm{~m}$.\n\n由 $\\sin \\left(\\frac{\\pi}{30} t-\\frac{\\pi}{2}\\right)=1$ ,得 $\\frac{\\pi}{30} t-\\frac{\\pi}{2}=\\frac{\\pi}{2} , \\therefore t=30$\n\n$\\therefore$ 缆车到达最高点时,用的最少时间是 $30 \\mathrm{~s}$.", "solution": "null", "level": "高二", "question": "如图, 为一个缆车示意图, 该缆车半径为 $4,.8 \\mathrm{~m}$, 圆上最低点与地面的距离为 $0,.8 \\mathrm{~m}, 60 \\mathrm{~s}$\n转动一圈, 图中 $O A$ 与地面垂直, 以 $O A$ 为始边, 逆时针转动 $\\theta$ 角到 $O B$, 设点 $B$ 与地面距离是 $h$.\n\n\n\n(1)求 $h$ 与 $\\theta$ 之间的函数解析式;\n\n(2)设从 $O A$ 开始转动, 经过 $t \\mathrm{~s}$ 后到达 $O B$, 求 $h$ 与 $t$ 之间的函数解析式, 并求缆车到达最高点时用的最少时间是多少.", "options": [], "subject": "解析几何", "analysis": "解析: (1)以圆心 $O$ 为原点,建立如图所示的坐标系,则以 $O x$ 为始边, $O B$ 为终边的角为 $\\theta-\\frac{\\pi}{2} ,$ 故点 $B$ 坐标为\n\n\n\n$\\left(4,.8 \\cos \\left(\\theta-\\frac{\\pi}{2}\\right), 4,.8 \\sin \\left(\\theta-\\frac{\\pi}{2}\\right)\\right)$.\n\n$\\therefore h=5,.6+4,.8 \\sin \\left(\\theta-\\frac{\\pi}{2}\\right)$.\n\n(2)点 $A$ 在圆上转动的角速度是 $\\frac{\\pi}{30}$ ,故 $t s$ 转过的弧度数为 $\\frac{\\pi t}{30}$.\n\n$\\therefore h=5,.6+4,.8 \\sin \\left(\\frac{\\pi}{30} t-\\frac{\\pi}{2}\\right), t \\in[0,+\\infty)$.\n\n到达最高点时, $h=10,.4 \\mathrm{~m}$.\n\n由 $\\sin \\left(\\frac{\\pi}{30} t-\\frac{\\pi}{2}\\right)=1$ ,得 $\\frac{\\pi}{30} t-\\frac{\\pi}{2}=\\frac{\\pi}{2} , \\therefore t=30$\n\n$\\therefore$ 缆车到达最高点时,用的最少时间是 $30 \\mathrm{~s}$."} {"id": "17886", "image": [], "answer": "(1)由函数易知,当 $x=14$ 时,函数取得最大值,此时最高温度为 $30{ }^{\\circ} \\mathrm{C}$ ,当 $x=6$ 时,函数取得最小值,此时最低温度为 $10{ }^{\\circ} \\mathrm{C}$ ,所以最大温差为 $30-10=20$\n\n(2)令 $10 \\sin \\left(\\frac{\\pi}{8} x-\\frac{5 \\pi}{4}\\right)+20=15$ ,得 $\\sin \\left(\\frac{\\pi}{8} x-\\frac{5 \\pi}{4}\\right)=-\\frac{1}{2}$ ,\n\n因为 $x \\in[4,16]$ ,所以 $x=\\frac{26}{3}$\n\n令 $10 \\sin \\left(\\frac{\\pi}{8} x-\\frac{5 \\pi}{4}\\right)+20=25$ ,得 $\\sin \\left(\\frac{\\pi}{8} x-\\frac{5 \\pi}{4}\\right)=\\frac{1}{2}$\n\n因为 $x \\in[4,16]$ ,所以 $x=\\frac{34}{3}$\n\n故该细菌能存活的最长时间为 $\\frac{34}{3}-\\frac{26}{3}=\\frac{8}{3}$ (小时).", "solution": "null", "level": "高二", "question": "9. 已知某地一天从 $4 \\sim 16$ 时的温度变化曲线近似满足函数 $y=10 \\sin \\left(\\frac{\\pi}{8} x-\\frac{5 \\pi}{4}\\right)+20, x \\in[4,16]$.\n\n(1)求该地区这一段时间内温度的最大温差;\n\n(2)若有一种细菌在 $15^{\\circ} \\mathrm{C}$ 到 $25^{\\circ} \\mathrm{C}$ 之间可以生存, 则在这段时间内, 该细菌最多能存活多长时间?", "options": [], "subject": "解析几何", "analysis": "解析: (1)由函数易知,当 $x=14$ 时,函数取得最大值,此时最高温度为 $30{ }^{\\circ} \\mathrm{C}$ ,当 $x=6$ 时,函数取得最小值,此时最低温度为 $10{ }^{\\circ} \\mathrm{C}$ ,所以最大温差为 $30-10=20$\n\n(2)令 $10 \\sin \\left(\\frac{\\pi}{8} x-\\frac{5 \\pi}{4}\\right)+20=15$ ,得 $\\sin \\left(\\frac{\\pi}{8} x-\\frac{5 \\pi}{4}\\right)=-\\frac{1}{2}$ ,\n\n因为 $x \\in[4,16]$ ,所以 $x=\\frac{26}{3}$\n\n令 $10 \\sin \\left(\\frac{\\pi}{8} x-\\frac{5 \\pi}{4}\\right)+20=25$ ,得 $\\sin \\left(\\frac{\\pi}{8} x-\\frac{5 \\pi}{4}\\right)=\\frac{1}{2}$\n\n因为 $x \\in[4,16]$ ,所以 $x=\\frac{34}{3}$\n\n故该细菌能存活的最长时间为 $\\frac{34}{3}-\\frac{26}{3}=\\frac{8}{3}$ (小时)."} {"id": "18467", "image": [], "answer": "因为在 $\\triangle A B C$ 中, $\\frac{a}{\\sin A}=\\frac{b}{\\sin B}=\\frac{c}{\\sin C}=2 R$,\n\n所以左边 $=\\frac{2 R \\sin A-2 R \\sin C \\cos B}{2 R \\sin B-2 R \\sin C \\cos A}$\n\n$=\\frac{\\sin (B+C)-\\sin C \\cos B}{\\sin (A+C)-\\sin C \\cos A}=\\frac{\\sin B \\cos C}{\\sin A \\cos C}=\\frac{\\sin B}{\\sin A}=$ 右边.\n\n所以等式成立, 即 $\\frac{a-c \\cos B}{b-c \\cos A}=\\frac{\\sin B}{\\sin A}$.", "solution": "null", "level": "高二", "question": "11. 在 $\\triangle A B C$ 中, 求证: $\\frac{a-c \\cos B}{b-c \\cos A}=\\frac{\\sin B}{\\sin A}$.", "options": [], "subject": "解析几何", "analysis": "因为在 $\\triangle A B C$ 中, $\\frac{a}{\\sin A}=\\frac{b}{\\sin B}=\\frac{c}{\\sin C}=2 R$,\n\n所以左边 $=\\frac{2 R \\sin A-2 R \\sin C \\cos B}{2 R \\sin B-2 R \\sin C \\cos A}$\n\n$=\\frac{\\sin (B+C)-\\sin C \\cos B}{\\sin (A+C)-\\sin C \\cos A}=\\frac{\\sin B \\cos C}{\\sin A \\cos C}=\\frac{\\sin B}{\\sin A}=$ 右边.\n\n所以等式成立, 即 $\\frac{a-c \\cos B}{b-c \\cos A}=\\frac{\\sin B}{\\sin A}$."} {"id": "18468", "image": [], "answer": "设三角形外接圆半径为 $R$, 则 $a^{2} \\tan B=b^{2} \\tan A$\n\n$\\Leftrightarrow \\frac{a^{2} \\sin B}{\\cos B}=\\frac{b^{2} \\sin A}{\\cos A}$\n\n$\\Leftrightarrow \\frac{4 R^{2} \\sin ^{2} A \\sin B}{\\cos B}=\\frac{4 R^{2} \\sin ^{2} B \\sin A}{\\cos A}$\n\n$\\Leftrightarrow \\sin A \\cos A=\\sin B \\cos B$\n\n$\\Leftrightarrow \\sin 2 A=\\sin 2 B$\n\n$\\Leftrightarrow 2 A=2 B$ 或 $2 A+2 B=\\pi$\n\n$\\Leftrightarrow A=B$ 或 $A+B=\\frac{\\pi}{2}$.\n\n$\\therefore \\triangle A B C$ 为等腰三角形或直角三角形.", "solution": "null", "level": "高二", "question": "12. 在 $\\triangle A B C$ 中, 已知 $a^{2} \\tan B=b^{2} \\tan A$, 试判断 $\\triangle A B C$ 的形状.", "options": [], "subject": "解析几何", "analysis": "设三角形外接圆半径为 $R$, 则 $a^{2} \\tan B=b^{2} \\tan A$\n\n$\\Leftrightarrow \\frac{a^{2} \\sin B}{\\cos B}=\\frac{b^{2} \\sin A}{\\cos A}$\n\n$\\Leftrightarrow \\frac{4 R^{2} \\sin ^{2} A \\sin B}{\\cos B}=\\frac{4 R^{2} \\sin ^{2} B \\sin A}{\\cos A}$\n\n$\\Leftrightarrow \\sin A \\cos A=\\sin B \\cos B$\n\n$\\Leftrightarrow \\sin 2 A=\\sin 2 B$\n\n$\\Leftrightarrow 2 A=2 B$ 或 $2 A+2 B=\\pi$\n\n$\\Leftrightarrow A=B$ 或 $A+B=\\frac{\\pi}{2}$.\n\n$\\therefore \\triangle A B C$ 为等腰三角形或直角三角形."} {"id": "18470", "image": [], "answer": "$\\cos B=2 \\cos ^{2} \\frac{B}{2}-1=\\frac{3}{5}$,\n\n故 $B$ 为锐角, $\\sin B=\\frac{4}{5}$.\n所以 $\\sin A=\\sin (\\pi-B-C)=\\sin \\left(\\frac{3 \\pi}{4}-B\\right)=\\frac{7 \\sqrt{2}}{10}$.\n\n由正弦定理得 $c=\\frac{a \\sin C}{\\sin A}=\\frac{10}{7}$,\n\n所以 $S_{\\triangle A B C}=\\frac{1}{2} a c \\sin B=\\frac{1}{2} \\times 2 \\times \\frac{10}{7} \\times \\frac{4}{5}=\\frac{8}{7}$.", "solution": "null", "level": "高二", "question": "14. 在 $\\triangle A B C$ 中, $a, b, c$ 分别是三个内角 $A, B, C$ 的对边, 若 $a=2, C=\\frac{\\pi}{4}$,\n\n$\\cos \\frac{B}{2}=\\frac{2 \\sqrt{5}}{5}$, 求 $\\triangle A B C$ 的面积 $S$.", "options": [], "subject": "解析几何", "analysis": "$\\cos B=2 \\cos ^{2} \\frac{B}{2}-1=\\frac{3}{5}$,\n\n故 $B$ 为锐角, $\\sin B=\\frac{4}{5}$.\n所以 $\\sin A=\\sin (\\pi-B-C)=\\sin \\left(\\frac{3 \\pi}{4}-B\\right)=\\frac{7 \\sqrt{2}}{10}$.\n\n由正弦定理得 $c=\\frac{a \\sin C}{\\sin A}=\\frac{10}{7}$,\n\n所以 $S_{\\triangle A B C}=\\frac{1}{2} a c \\sin B=\\frac{1}{2} \\times 2 \\times \\frac{10}{7} \\times \\frac{4}{5}=\\frac{8}{7}$.哈哈"} {"id": "17903", "image": [], "answer": "设角 $\\alpha$ 的终边与单位圆的交点为 $P(x, y)$, 则有 $x^{2}+y^{2}=1$.\n\n由三角函数的定义, 得\n\n$\\frac{1}{\\cos \\alpha} \\cdot \\sqrt{1+\\tan ^{2} \\alpha}+\\tan \\alpha \\cdot \\sqrt{\\frac{1}{\\sin ^{2} \\alpha}-1}$\n\n$=\\frac{1}{x} \\cdot \\sqrt{1+\\left(\\frac{y}{x}\\right)^{2}}+\\frac{y}{x} \\cdot \\sqrt{\\frac{1}{y^{2}}-1}$\n\n$=\\frac{1}{x} \\cdot \\sqrt{\\frac{x^{2}+y^{2}}{x^{2}}}+\\frac{y}{x} \\cdot \\sqrt{\\frac{1-y^{2}}{y^{2}}}=\\frac{1}{x} \\cdot \\frac{1}{|x|}+\\frac{y}{x} \\cdot \\frac{|x|}{|y|}$.\n\n$\\because \\alpha$ 为第四象限角, $\\therefore x>0, y<0$,\n\n$\\therefore$ 原式 $=\\frac{1}{x^{2}}-1=\\frac{1-x^{2}}{x^{2}}=\\left(\\frac{y}{x}\\right)^{2}=\\tan ^{2} \\alpha$", "solution": "null", "level": "高二", "question": "15. 化简 $\\frac{1}{\\cos \\alpha} \\cdot \\sqrt{1+\\tan ^{2} \\alpha}+\\tan \\alpha \\cdot \\sqrt{\\frac{1}{\\sin ^{2} \\alpha}-1}$ (其中 $\\alpha$ 为第四象限角)", "options": [], "subject": "解析几何", "analysis": "设角 $\\alpha$ 的终边与单位圆的交点为 $P(x, y)$, 则有 $x^{2}+y^{2}=1$.\n\n由三角函数的定义, 得\n\n$\\frac{1}{\\cos \\alpha} \\cdot \\sqrt{1+\\tan ^{2} \\alpha}+\\tan \\alpha \\cdot \\sqrt{\\frac{1}{\\sin ^{2} \\alpha}-1}$\n\n$=\\frac{1}{x} \\cdot \\sqrt{1+\\left(\\frac{y}{x}\\right)^{2}}+\\frac{y}{x} \\cdot \\sqrt{\\frac{1}{y^{2}}-1}$\n\n$=\\frac{1}{x} \\cdot \\sqrt{\\frac{x^{2}+y^{2}}{x^{2}}}+\\frac{y}{x} \\cdot \\sqrt{\\frac{1-y^{2}}{y^{2}}}=\\frac{1}{x} \\cdot \\frac{1}{|x|}+\\frac{y}{x} \\cdot \\frac{|x|}{|y|}$.\n\n$\\because \\alpha$ 为第四象限角, $\\therefore x>0, y<0$,\n\n$\\therefore$ 原式 $=\\frac{1}{x^{2}}-1=\\frac{1-x^{2}}{x^{2}}=\\left(\\frac{y}{x}\\right)^{2}=\\tan ^{2} \\alpha$."} {"id": "17927", "image": ["9021.jpg"], "answer": "(1) $\\because B, E, C$ 三点共线,\n\n$\\therefore \\overrightarrow{O E}=x \\overrightarrow{O C}+(1-x) \\overrightarrow{O B}=2 x \\boldsymbol{a}+(1-x) \\boldsymbol{b}$. (1)\n同理, $\\because A, E, D$ 三点共线, $\\therefore \\overrightarrow{O E}=y \\boldsymbol{a}+3(1-y) \\boldsymbol{b}$.(2)\n\n比较(1)(2),得 $\\left\\{\\begin{array}{l}2 x=y , \\\\ 1-x=3(1-y) ,\\end{array}\\right.$\n\n解得 $x=\\frac{2}{5} y=\\frac{4}{5} , \\therefore \\overrightarrow{O E}=\\frac{4}{5} a+\\frac{3}{5} \\boldsymbol{b}$.\n\n(2)证明: $\\because \\overrightarrow{O L}=\\frac{\\boldsymbol{a}+\\boldsymbol{b}}{2}, \\overrightarrow{O M}=\\frac{1}{2} \\overrightarrow{O E}=\\frac{4 \\boldsymbol{a}+3 \\boldsymbol{b}}{10}, \\overrightarrow{O N}=\\frac{1}{2}(\\overrightarrow{O C}+\\overrightarrow{O D})=\\frac{2 \\boldsymbol{a}+3 \\boldsymbol{b}}{2}, \\therefore \\overrightarrow{M N}=\\overrightarrow{O N}-\\overrightarrow{O M}$ $=\\frac{6 \\boldsymbol{a}+12 \\boldsymbol{b}}{10}, \\overrightarrow{M L}=\\overrightarrow{O L}-\\overrightarrow{O M}=\\frac{\\boldsymbol{a}+2 \\boldsymbol{b}}{10}$,\n\n$\\therefore \\overrightarrow{M N}=6 \\overrightarrow{M L}$,\n\n又 $\\overrightarrow{M N}$ 与 $\\overrightarrow{M L}$ 有公共点 $M$ ,\n\n$\\therefore L, M, N$ 三点共线.", "solution": "null", "level": "高二", "question": "15. 如图, 已知三点 $O, A, B$ 不共线, 且 $\\overrightarrow{O C}=2 \\overrightarrow{O A}, \\overrightarrow{O D}=3 \\overrightarrow{O B}$, 设 $\\overrightarrow{O A}=\\boldsymbol{a}, \\overrightarrow{O B}=\\boldsymbol{b}$.\n\n(1)试用 $\\boldsymbol{a}, \\boldsymbol{b}$ 表示向量 $\\overrightarrow{O E}$;\n\n(2)设线段 $A B, O E, C D$ 的中点分别为 $L, M, N$, 试证明: $L, M, N$ 三点共线.\n\n", "options": [], "subject": "解析几何", "analysis": "(1) $\\because B, E, C$ 三点共线,\n\n$\\therefore \\overrightarrow{O E}=x \\overrightarrow{O C}+(1-x) \\overrightarrow{O B}=2 x \\boldsymbol{a}+(1-x) \\boldsymbol{b}$. (1)\n同理, $\\because A, E, D$ 三点共线, $\\therefore \\overrightarrow{O E}=y \\boldsymbol{a}+3(1-y) \\boldsymbol{b}$.(2)\n\n比较(1)(2),得 $\\left\\{\\begin{array}{l}2 x=y , \\\\ 1-x=3(1-y) ,\\end{array}\\right.$\n\n解得 $x=\\frac{2}{5} y=\\frac{4}{5} , \\therefore \\overrightarrow{O E}=\\frac{4}{5} a+\\frac{3}{5} \\boldsymbol{b}$.\n\n(2)证明: $\\because \\overrightarrow{O L}=\\frac{\\boldsymbol{a}+\\boldsymbol{b}}{2}, \\overrightarrow{O M}=\\frac{1}{2} \\overrightarrow{O E}=\\frac{4 \\boldsymbol{a}+3 \\boldsymbol{b}}{10}, \\overrightarrow{O N}=\\frac{1}{2}(\\overrightarrow{O C}+\\overrightarrow{O D})=\\frac{2 \\boldsymbol{a}+3 \\boldsymbol{b}}{2}, \\therefore \\overrightarrow{M N}=\\overrightarrow{O N}-\\overrightarrow{O M}$ $=\\frac{6 \\boldsymbol{a}+12 \\boldsymbol{b}}{10}, \\overrightarrow{M L}=\\overrightarrow{O L}-\\overrightarrow{O M}=\\frac{\\boldsymbol{a}+2 \\boldsymbol{b}}{10}$,\n\n$\\therefore \\overrightarrow{M N}=6 \\overrightarrow{M L}$,\n\n又 $\\overrightarrow{M N}$ 与 $\\overrightarrow{M L}$ 有公共点 $M$ ,\n\n$\\therefore L, M, N$ 三点共线."} {"id": "17940", "image": [], "answer": "(1) $\\overrightarrow{O P}=\\overrightarrow{O A}+t \\overrightarrow{A B}=(1+3 t, 2+3 t)$ ,\n若 $P$ 在 $x$ 轴上,则 $2+3 t=0 , \\therefore t=-\\frac{2}{3}$; 若 $P$ 在 $y$ 轴上,则 $1+3 t=0 , \\therefore t=-\\frac{1}{3}$.\n\n(2)不能. 由题意知 $\\overrightarrow{O A}=(1,2), \\overrightarrow{P B}=(3-3 t, 3-3 t)$.\n\n若四边形 $O A B P$ 为平行四边形,则 $\\overrightarrow{O A}=\\overrightarrow{P B}$.\n\n$\\because\\left\\{\\begin{array}{l}3-3 t=1 \\\\{3-3 t=2} \\end{array} \\text { 无解, } \\therefore \\text { 四边形 } O A B P \\text { 不能成为平行四边形. }\\end{array}\\right.$", "solution": "null", "level": "高二", "question": "14. 已知 $O(0,0), A(1,2), B(4,5)$ 及 $\\overrightarrow{O P}=\\overrightarrow{O A}+t \\overrightarrow{A B}$, 则:\n\n(1) $t$ 为何值时, $P$ 在 $x$ 轴上? $P$ 在 $y$ 轴上?\n\n(2)四边形 $O A B P$ 能否成为平行四边形? 若能, 求出相应的 $t$ 值; 若不能, 请说明理由.", "options": [], "subject": "解析几何", "analysis": "(1) $\\overrightarrow{O P}=\\overrightarrow{O A}+t \\overrightarrow{A B}=(1+3 t, 2+3 t)$ ,\n若 $P$ 在 $x$ 轴上,则 $2+3 t=0 , \\therefore t=-\\frac{2}{3}$; 若 $P$ 在 $y$ 轴上,则 $1+3 t=0 , \\therefore t=-\\frac{1}{3}$.\n\n(2)不能. 由题意知 $\\overrightarrow{O A}=(1,2), \\overrightarrow{P B}=(3-3 t, 3-3 t)$.\n\n若四边形 $O A B P$ 为平行四边形,则 $\\overrightarrow{O A}=\\overrightarrow{P B}$.\n\n$\\because\\left\\{\\begin{array}{l}3-3 t=1 \\\\{3-3 t=2} \\end{array} \\text { 无解, } \\therefore \\text { 四边形 } O A B P \\text { 不能成为平行四边形. }\\end{array}\\right.$"} {"id": "17951", "image": [], "answer": "设 $P(x, y)$ ,则由 $|\\overrightarrow{A P}|=2|\\overrightarrow{P B}|$ ,得 $\\overrightarrow{A P}=2 \\overrightarrow{P B}$ 或 $\\overrightarrow{A P}=-2 \\overrightarrow{P B}$.\n\n若 $\\overrightarrow{A P}=2 \\overrightarrow{P B}$ ,则 $(x-3, y+4)=2(-1-x, 2-y)$.\n\n所以 $\\left\\{\\begin{array}{l}x-3=-2-2 x , \\\\ y+4=4-2 y ,\\end{array}\\right.$ 解得 $\\left\\{\\begin{array}{l}x=\\frac{1}{3} } \\\\{y=0 ,} \\end{array} \\text { 故 } P\\left(\\frac{1}{3} , 0\\right) \\text {. }\\end{array}\\right.$\n若 $\\overrightarrow{A P}=-2 \\overrightarrow{P B}$ ,同理可解得 $\\left\\{\\begin{array}{l}x=-5 , \\\\ y=8 ,\\end{array}\\right.$ 故 $P(-5 , 8)$.\n\n综上,点 $P$ 的坐标为 $\\left(\\frac{1}{3} , 0\\right)$ 或 $(-5 , 8)$.\n\n答案: $\\left(\\frac{1}{3}, 0\\right)$ 或 $(-5,8)$", "solution": "null", "level": "高二", "question": "11. 已知点 $A(3,-4)$ 与点 $B(-1,2)$, 点 $P$ 在直线 $A B$ 上, 且 $|\\overrightarrow{A P}|=2|\\overrightarrow{P B}|$, 则点 $P$ 的坐标为", "options": [], "subject": "解析几何", "analysis": "设 $P(x, y)$ ,则由 $|\\overrightarrow{A P}|=2|\\overrightarrow{P B}|$ ,得 $\\overrightarrow{A P}=2 \\overrightarrow{P B}$ 或 $\\overrightarrow{A P}=-2 \\overrightarrow{P B}$.\n\n若 $\\overrightarrow{A P}=2 \\overrightarrow{P B}$ ,则 $(x-3, y+4)=2(-1-x, 2-y)$.\n\n所以 $\\left\\{\\begin{array}{l}x-3=-2-2 x , \\\\ y+4=4-2 y ,\\end{array}\\right.$ 解得 $\\left\\{\\begin{array}{l}x=\\frac{1}{3} } \\\\{y=0 ,} \\end{array} \\text { 故 } P\\left(\\frac{1}{3} , 0\\right) \\text {. }\\end{array}\\right.$\n若 $\\overrightarrow{A P}=-2 \\overrightarrow{P B}$ ,同理可解得 $\\left\\{\\begin{array}{l}x=-5 , \\\\ y=8 ,\\end{array}\\right.$ 故 $P(-5 , 8)$.\n\n综上,点 $P$ 的坐标为 $\\left(\\frac{1}{3} , 0\\right)$ 或 $(-5 , 8)$.\n\n答案: $\\left(\\frac{1}{3}, 0\\right)$ 或 $(-5,8)$"} {"id": "17952", "image": ["9028.jpg"], "answer": "$\\left(\\frac{8}{3},-7\\right)$", "solution": "null", "level": "高二", "question": "12. 平面上有 $A(2,-1), B(1,4), D(4,-3)$ 三点, 点 $C$ 在直线 $A B$ 上, 且 $\\overrightarrow{A C}=\\frac{1}{2} \\overrightarrow{B C}$, 连接 $D C$ 并延长至点 $E$, 使 $|\\overrightarrow{C E}|=\\frac{1}{4}|\\overrightarrow{E D}|$, 则点 $E$ 的坐标为", "options": [], "subject": "解析几何", "analysis": "$\\because \\overrightarrow{A C}=\\frac{1}{2} \\overrightarrow{B C} , \\therefore \\overrightarrow{O C}-\\overrightarrow{O A}=\\frac{1}{2}(\\overrightarrow{O C}-\\overrightarrow{O B})$.\n\n$\\therefore \\overrightarrow{O C}=2 \\overrightarrow{O A}-\\overrightarrow{O B}=(3,-6), \\therefore$ 点 $C$ 的坐标为 $(3,-6)$.\n\n又 $\\because|\\overrightarrow{C E}|=\\frac{1}{4}|\\overrightarrow{E D}|$ ,且 $E$ 在 $D C$ 的延长线上, $\\therefore \\overrightarrow{C E}=-\\frac{1}{4} \\overrightarrow{E D}$.\n\n设 $E(x, y)$ ,则 $(x-3, y+6)=-\\frac{1}{4}(4-x,-3-y)$ ,\n\n\n\n$\\therefore$ 点 $E$ 的坐标为 $\\left(\\frac{8}{3} ,-7\\right)$.\n\n答案: $\\left(\\frac{8}{3},-7\\right)$"} {"id": "17959", "image": [], "answer": "$(1) \\because \\boldsymbol{a}=(1,0), \\boldsymbol{b}=(2,1), \\therefore k \\boldsymbol{a}-\\boldsymbol{b}=k(1,0)-(2,1)=(k-2 ,-1), \\boldsymbol{a}+2 \\boldsymbol{b}=(1$ , $0)+2(2,1)=(5,2)$.\n\n$\\because k a-b$ 与 $\\boldsymbol{a}+2 \\boldsymbol{b}$ 共线,\n\n$\\therefore 2(k-2)-(-1) \\times 5=0$,\n\n$\\therefore k=-\\frac{1}{2}$.\n\n$(2) \\overrightarrow{A B}=2(1,0)+3(2,1)=(8,3)$,\n\n$\\overrightarrow{B C}=(1,0)+m(2,1)=(2 m+1, m)$.\n\n$\\because A, B, C$ 三点共线, $\\therefore \\overrightarrow{A B} / / \\overrightarrow{B C}$ ,\n\n$\\therefore 8 m-3(2 m+1)=0, \\quad \\therefore m=\\frac{3}{2}$.", "solution": "null", "level": "高二", "question": "8. 已知 $\\boldsymbol{a}=(1,0), \\boldsymbol{b}=(2,1)$.\n\n(1)当 $k$ 为何值时, $k \\boldsymbol{a}-\\boldsymbol{b}$ 与 $\\boldsymbol{a}+2 \\boldsymbol{b}$ 共线;\n\n(2)若 $\\overrightarrow{A B}=2 \\boldsymbol{a}+3 \\boldsymbol{b}, \\overrightarrow{B C}=\\boldsymbol{a}+m \\boldsymbol{b}$, 且 $A, B, C$ 三点共线, 求 $m$ 的值.", "options": [], "subject": "解析几何", "analysis": "$(1) \\because \\boldsymbol{a}=(1,0), \\boldsymbol{b}=(2,1), \\therefore k \\boldsymbol{a}-\\boldsymbol{b}=k(1,0)-(2,1)=(k-2 ,-1), \\boldsymbol{a}+2 \\boldsymbol{b}=(1$ , $0)+2(2,1)=(5,2)$.\n\n$\\because k a-b$ 与 $\\boldsymbol{a}+2 \\boldsymbol{b}$ 共线,\n\n$\\therefore 2(k-2)-(-1) \\times 5=0$,\n\n$\\therefore k=-\\frac{1}{2}$.\n\n$(2) \\overrightarrow{A B}=2(1,0)+3(2,1)=(8,3)$,\n\n$\\overrightarrow{B C}=(1,0)+m(2,1)=(2 m+1, m)$.\n\n$\\because A, B, C$ 三点共线, $\\therefore \\overrightarrow{A B} / / \\overrightarrow{B C}$ ,\n\n$\\therefore 8 m-3(2 m+1)=0, \\quad \\therefore m=\\frac{3}{2}$."} {"id": "17962", "image": [], "answer": "因为 $\\boldsymbol{a}^{2}=\\left(3 \\boldsymbol{e}_{1}-2 \\boldsymbol{e}_{2}\\right)^{2}=9-2 \\times 3 \\times 2 \\times \\cos \\alpha+4=9$ ,所以 $|\\boldsymbol{a}|=3 , \\boldsymbol{b}^{2}=\\left(3 \\boldsymbol{e}_{1}-\\boldsymbol{e}_{2}\\right)^{2}=9-$ $2 \\times 3 \\times 1 \\times \\cos \\alpha+1=8$ ,所以 $|\\boldsymbol{b}|=2 \\sqrt{2}, \\boldsymbol{a} \\cdot \\boldsymbol{b}=\\left(3 \\boldsymbol{e}_{1}-2 \\boldsymbol{e}_{2}\\right) \\cdot\\left(3 \\boldsymbol{e}_{1}-\\boldsymbol{e}_{2}\\right)=9 \\boldsymbol{e}_{1}^{2}-9 \\boldsymbol{e}_{1} \\cdot \\boldsymbol{e}_{2}+2 \\boldsymbol{e}_{2}^{2}=9-$ $9 \\times 1 \\times 1 \\times \\frac{1}{3}+2=8$ ,所以 $\\cos \\beta=\\frac{\\boldsymbol{a} \\cdot \\boldsymbol{b}}{|\\boldsymbol{a}| \\cdot|\\boldsymbol{b}|}=\\frac{8}{3 \\times 2 \\sqrt{2}}=\\frac{2 \\sqrt{2}}{3}$.", "solution": "null", "level": "高二", "question": "10. 已知单位向量 $\\boldsymbol{e}_{1}$ 与 $\\boldsymbol{e}_{2}$ 的夹角为 $\\alpha$, 且 $\\cos \\alpha=\\frac{1}{3}$, 向量 $\\boldsymbol{a}=3 \\boldsymbol{e}_{1}-2 \\boldsymbol{e}_{2}$ 与 $\\boldsymbol{b}=3 \\boldsymbol{e}_{1}-\\boldsymbol{e}_{2}$ 的夹角为 $\\beta$, 求 $\\beta$ 的余弦值.", "options": [], "subject": "解析几何", "analysis": "因为 $\\boldsymbol{a}^{2}=\\left(3 \\boldsymbol{e}_{1}-2 \\boldsymbol{e}_{2}\\right)^{2}=9-2 \\times 3 \\times 2 \\times \\cos \\alpha+4=9$ ,所以 $|\\boldsymbol{a}|=3 , \\boldsymbol{b}^{2}=\\left(3 \\boldsymbol{e}_{1}-\\boldsymbol{e}_{2}\\right)^{2}=9-$ $2 \\times 3 \\times 1 \\times \\cos \\alpha+1=8$ ,所以 $|\\boldsymbol{b}|=2 \\sqrt{2}, \\boldsymbol{a} \\cdot \\boldsymbol{b}=\\left(3 \\boldsymbol{e}_{1}-2 \\boldsymbol{e}_{2}\\right) \\cdot\\left(3 \\boldsymbol{e}_{1}-\\boldsymbol{e}_{2}\\right)=9 \\boldsymbol{e}_{1}^{2}-9 \\boldsymbol{e}_{1} \\cdot \\boldsymbol{e}_{2}+2 \\boldsymbol{e}_{2}^{2}=9-$ $9 \\times 1 \\times 1 \\times \\frac{1}{3}+2=8$ ,所以 $\\cos \\beta=\\frac{\\boldsymbol{a} \\cdot \\boldsymbol{b}}{|\\boldsymbol{a}| \\cdot|\\boldsymbol{b}|}=\\frac{8}{3 \\times 2 \\sqrt{2}}=\\frac{2 \\sqrt{2}}{3}$."} {"id": "17966", "image": [], "answer": "(1) $\\because(2 \\boldsymbol{a}-3 \\boldsymbol{b}) \\cdot(2 \\boldsymbol{a}+\\boldsymbol{b})=61$ ,\n\n$\\therefore 4|\\boldsymbol{a}|^{2}-4 \\boldsymbol{a} \\cdot \\boldsymbol{b}-3|\\boldsymbol{b}|^{2}=61$.\n\n又 $\\because|\\boldsymbol{a}|=4 ,|\\boldsymbol{b}|=3 , \\therefore 64-4 \\boldsymbol{a} \\cdot \\boldsymbol{b}-27=61$ ,\n\n$\\therefore \\boldsymbol{a} \\cdot \\boldsymbol{b}=-6$,\n\n$\\therefore \\cos \\theta=\\frac{\\boldsymbol{a} \\cdot \\boldsymbol{b}}{|\\boldsymbol{a}||\\boldsymbol{b}|}=\\frac{-6}{4 \\times 3}=-\\frac{1}{2}$.\n\n又 $\\because 0 \\leqslant \\theta \\leqslant \\pi , \\therefore \\theta=\\frac{2 \\pi}{3}$.\n\n(2) $|\\boldsymbol{a}+\\boldsymbol{b}|^{2}=(\\boldsymbol{a}+\\boldsymbol{b})^{2}=|\\boldsymbol{a}|^{2}+2 \\boldsymbol{a} \\cdot \\boldsymbol{b}+|\\boldsymbol{b}|^{2}$\n\n$=4^{2}+2 \\times(-6)+3^{2}=13, \\therefore|\\boldsymbol{a}+\\boldsymbol{b}|=\\sqrt{13}$.", "solution": "null", "level": "高二", "question": "15. 已知 $|\\boldsymbol{a}|=4,|\\boldsymbol{b}|=3,(2 \\boldsymbol{a}-3 \\boldsymbol{b}) \\cdot(2 \\boldsymbol{a}+\\boldsymbol{b})=61$.\n\n(1)求 $\\boldsymbol{a}$ 与 $\\boldsymbol{b}$ 的夹角 $\\theta$;\n\n(2)求 $|\\boldsymbol{a}+\\boldsymbol{b}|$.", "options": [], "subject": "解析几何", "analysis": "(1) $\\because(2 \\boldsymbol{a}-3 \\boldsymbol{b}) \\cdot(2 \\boldsymbol{a}+\\boldsymbol{b})=61$ ,\n\n$\\therefore 4|\\boldsymbol{a}|^{2}-4 \\boldsymbol{a} \\cdot \\boldsymbol{b}-3|\\boldsymbol{b}|^{2}=61$.\n\n又 $\\because|\\boldsymbol{a}|=4 ,|\\boldsymbol{b}|=3 , \\therefore 64-4 \\boldsymbol{a} \\cdot \\boldsymbol{b}-27=61$ ,\n\n$\\therefore \\boldsymbol{a} \\cdot \\boldsymbol{b}=-6$,\n\n$\\therefore \\cos \\theta=\\frac{\\boldsymbol{a} \\cdot \\boldsymbol{b}}{|\\boldsymbol{a}||\\boldsymbol{b}|}=\\frac{-6}{4 \\times 3}=-\\frac{1}{2}$.\n\n又 $\\because 0 \\leqslant \\theta \\leqslant \\pi , \\therefore \\theta=\\frac{2 \\pi}{3}$.\n\n(2) $|\\boldsymbol{a}+\\boldsymbol{b}|^{2}=(\\boldsymbol{a}+\\boldsymbol{b})^{2}=|\\boldsymbol{a}|^{2}+2 \\boldsymbol{a} \\cdot \\boldsymbol{b}+|\\boldsymbol{b}|^{2}$\n\n$=4^{2}+2 \\times(-6)+3^{2}=13, \\therefore|\\boldsymbol{a}+\\boldsymbol{b}|=\\sqrt{13}$."} {"id": "17967", "image": [], "answer": "(1) $\\because|\\boldsymbol{a}|=2|\\boldsymbol{b}|=2 , \\therefore|\\boldsymbol{a}|=2 ,|\\boldsymbol{b}|=1$.\n\n又 $\\boldsymbol{a}$ 在 $\\boldsymbol{b}$ 方向上的投影为 $|\\boldsymbol{a}| \\cos \\theta=-1$ ,\n\n$\\therefore \\boldsymbol{a} \\cdot \\boldsymbol{b}=|\\boldsymbol{a}||\\boldsymbol{b}| \\cos \\theta=-1$.\n\n又 $\\because|\\boldsymbol{a}|=2,|\\boldsymbol{b}|=1, \\quad \\therefore \\cos \\theta=-\\frac{1}{2}, \\quad \\therefore \\theta=\\frac{2 \\pi}{3}$.\n\n(2) $(\\boldsymbol{a}-2 \\boldsymbol{b}) \\cdot \\boldsymbol{b}=\\boldsymbol{a} \\cdot \\boldsymbol{b}-2 \\boldsymbol{b}^{2}=-1-2=-3$.\n\n(3) $\\because \\lambda \\boldsymbol{a}+\\boldsymbol{b}$ 与 $\\boldsymbol{a}-3 \\boldsymbol{b}$ 互相垂直,\n\n$\\therefore(\\lambda \\boldsymbol{a}+\\boldsymbol{b}) \\cdot(\\boldsymbol{a}-3 \\boldsymbol{b})=\\lambda \\boldsymbol{a}^{2}-3 \\lambda \\boldsymbol{a} \\cdot \\boldsymbol{b}+\\boldsymbol{b} \\cdot \\boldsymbol{a}-3 \\boldsymbol{b}^{2}$\n\n$=4 \\lambda+3 \\lambda-1-3=7 \\lambda-4=0, \\quad \\therefore \\lambda=\\frac{4}{7}$.", "solution": "null", "level": "高二", "question": "16. 已知 $|\\boldsymbol{a}|=2|\\boldsymbol{b}|=2$, 且 $\\boldsymbol{a}$ 在 $\\boldsymbol{b}$ 方向上的投影为 -1 .\n\n(1)求 $\\boldsymbol{a}$ 与 $\\boldsymbol{b}$ 的夹角 $\\theta$;\n\n(2)求 $(\\boldsymbol{a}-2 \\boldsymbol{b}) \\cdot \\boldsymbol{b}$;\n\n(3)当 $\\lambda$ 为何值时, 向量 $\\lambda \\boldsymbol{a}+\\boldsymbol{b}$ 与向量 $\\boldsymbol{a}-3 \\boldsymbol{b}$ 互相垂直?", "options": [], "subject": "解析几何", "analysis": "(1) $\\because|\\boldsymbol{a}|=2|\\boldsymbol{b}|=2 , \\therefore|\\boldsymbol{a}|=2 ,|\\boldsymbol{b}|=1$.\n\n又 $\\boldsymbol{a}$ 在 $\\boldsymbol{b}$ 方向上的投影为 $|\\boldsymbol{a}| \\cos \\theta=-1$ ,\n\n$\\therefore \\boldsymbol{a} \\cdot \\boldsymbol{b}=|\\boldsymbol{a}||\\boldsymbol{b}| \\cos \\theta=-1$.\n\n又 $\\because|\\boldsymbol{a}|=2,|\\boldsymbol{b}|=1, \\quad \\therefore \\cos \\theta=-\\frac{1}{2}, \\quad \\therefore \\theta=\\frac{2 \\pi}{3}$.\n\n(2) $(\\boldsymbol{a}-2 \\boldsymbol{b}) \\cdot \\boldsymbol{b}=\\boldsymbol{a} \\cdot \\boldsymbol{b}-2 \\boldsymbol{b}^{2}=-1-2=-3$.\n\n(3) $\\because \\lambda \\boldsymbol{a}+\\boldsymbol{b}$ 与 $\\boldsymbol{a}-3 \\boldsymbol{b}$ 互相垂直,\n\n$\\therefore(\\lambda \\boldsymbol{a}+\\boldsymbol{b}) \\cdot(\\boldsymbol{a}-3 \\boldsymbol{b})=\\lambda \\boldsymbol{a}^{2}-3 \\lambda \\boldsymbol{a} \\cdot \\boldsymbol{b}+\\boldsymbol{b} \\cdot \\boldsymbol{a}-3 \\boldsymbol{b}^{2}$\n\n$=4 \\lambda+3 \\lambda-1-3=7 \\lambda-4=0, \\quad \\therefore \\lambda=\\frac{4}{7}$."} {"id": "17974", "image": ["9031.jpg"], "answer": "$(1) \\overrightarrow{E F}=\\overrightarrow{C F}-\\overrightarrow{C E}=\\frac{2}{3} \\overrightarrow{C D}-\\frac{1}{3} \\overrightarrow{C B}$\n\n$=-\\frac{2}{3} \\overrightarrow{A B}+\\frac{1}{3} \\overrightarrow{A D}=-\\frac{2}{3} \\boldsymbol{a}+\\frac{1}{3} \\boldsymbol{b}$.\n\n(2)因为 $|\\boldsymbol{a}|=1 ,|\\boldsymbol{b}|=4 , \\angle D A B=60^{\\circ}$ ,\n\n所以 $|\\overrightarrow{E F}|^{2}=\\left(\\frac{1}{3} \\boldsymbol{b}-\\frac{2}{3} \\boldsymbol{a}\\right)^{2}$\n\n$=\\frac{1}{9}|\\boldsymbol{b}|^{2}-\\frac{4}{9} \\boldsymbol{a} \\cdot \\boldsymbol{b}+\\frac{4}{9}|\\boldsymbol{a}|^{2}$\n$=\\frac{16}{9}-\\frac{4}{9} \\times 1 \\times 4 \\times \\cos 60^{\\circ}+\\frac{4}{9}=\\frac{4}{3}$.\n\n所以 $|\\overrightarrow{E F}|=\\frac{2 \\sqrt{3}}{3}$.\n\n$\\overrightarrow{A C} \\cdot \\overrightarrow{F E}=(\\boldsymbol{a}+\\boldsymbol{b}) \\cdot\\left(\\frac{2}{3} \\boldsymbol{a}-\\frac{1}{3} \\boldsymbol{b}\\right)$\n\n$=\\frac{2}{3}|\\boldsymbol{a}|^{2}+\\frac{1}{3} \\boldsymbol{a} \\cdot \\boldsymbol{b}-\\frac{1}{3}|\\boldsymbol{b}|^{2}$\n\n$=\\frac{2}{3}+\\frac{1}{3} \\times 1 \\times 4 \\times \\cos 60^{\\circ}-\\frac{16}{3}=-4$.", "solution": "null", "level": "高二", "question": "9.如图, 在 $\\square A B C D$ 中, $\\overrightarrow{A B}=\\boldsymbol{a}, \\overrightarrow{A D}=\\boldsymbol{b}, \\overrightarrow{C E}=\\frac{1}{3} \\overrightarrow{C B}, \\overrightarrow{C F}=\\frac{2}{3} \\overrightarrow{C D}$.\n\n\n\n(1)用 $\\boldsymbol{a}, \\boldsymbol{b}$ 表示 $\\overrightarrow{E F}$;\n\n(2)若 $|\\boldsymbol{a}|=1,|\\boldsymbol{b}|=4, \\angle D A B=60^{\\circ}$, 分别求 $|\\overrightarrow{E F}| \\vec{F}$ 和 $\\overrightarrow{A C} \\cdot \\overrightarrow{F E}$ 的值.", "options": [], "subject": "解析几何", "analysis": "$(1) \\overrightarrow{E F}=\\overrightarrow{C F}-\\overrightarrow{C E}=\\frac{2}{3} \\overrightarrow{C D}-\\frac{1}{3} \\overrightarrow{C B}$\n\n$=-\\frac{2}{3} \\overrightarrow{A B}+\\frac{1}{3} \\overrightarrow{A D}=-\\frac{2}{3} \\boldsymbol{a}+\\frac{1}{3} \\boldsymbol{b}$.\n\n(2)因为 $|\\boldsymbol{a}|=1 ,|\\boldsymbol{b}|=4 , \\angle D A B=60^{\\circ}$ ,\n\n所以 $|\\overrightarrow{E F}|^{2}=\\left(\\frac{1}{3} \\boldsymbol{b}-\\frac{2}{3} \\boldsymbol{a}\\right)^{2}$\n\n$=\\frac{1}{9}|\\boldsymbol{b}|^{2}-\\frac{4}{9} \\boldsymbol{a} \\cdot \\boldsymbol{b}+\\frac{4}{9}|\\boldsymbol{a}|^{2}$\n$=\\frac{16}{9}-\\frac{4}{9} \\times 1 \\times 4 \\times \\cos 60^{\\circ}+\\frac{4}{9}=\\frac{4}{3}$.\n\n所以 $|\\overrightarrow{E F}|=\\frac{2 \\sqrt{3}}{3}$.\n\n$\\overrightarrow{A C} \\cdot \\overrightarrow{F E}=(\\boldsymbol{a}+\\boldsymbol{b}) \\cdot\\left(\\frac{2}{3} \\boldsymbol{a}-\\frac{1}{3} \\boldsymbol{b}\\right)$\n\n$=\\frac{2}{3}|\\boldsymbol{a}|^{2}+\\frac{1}{3} \\boldsymbol{a} \\cdot \\boldsymbol{b}-\\frac{1}{3}|\\boldsymbol{b}|^{2}$\n\n$=\\frac{2}{3}+\\frac{1}{3} \\times 1 \\times 4 \\times \\cos 60^{\\circ}-\\frac{16}{3}=-4$."} {"id": "17980", "image": [], "answer": "(1)因为 $\\boldsymbol{a}=(\\sqrt{3},-1), \\boldsymbol{b}=\\left(\\frac{1}{2}, \\frac{\\sqrt{3}}{2}\\right)$ ,所以 $\\boldsymbol{a} \\cdot \\boldsymbol{b}=0$ ,且 $|\\boldsymbol{a}|=2 ,|\\boldsymbol{b}|=1$.\n\n又因为 $\\boldsymbol{x} \\perp \\boldsymbol{y}$ ,所以 $x \\cdot y=0$ ,即 $[\\boldsymbol{a}+(t-3) \\boldsymbol{b}] \\cdot(-k \\boldsymbol{a}+\\boldsymbol{t} \\boldsymbol{b})=0$ ,\n\n所以 $-k \\boldsymbol{a}^{2}-k(t-3) \\boldsymbol{a} \\cdot \\boldsymbol{b}+t \\boldsymbol{a} \\cdot \\boldsymbol{b}+t(t-3) \\boldsymbol{b}^{2}=0$.\n\n因为 $|\\boldsymbol{a}|=2 ,|\\boldsymbol{b}|=1, \\boldsymbol{a} \\cdot \\boldsymbol{b}=0$ ,\n\n所以 $-4 k+t^{2}-3 t=0$, 所以 $k=\\frac{1}{4}\\left(t^{2}-3 t\\right)$.\n\n(2)由(1)知, $k=\\frac{1}{4}\\left(t^{2}-3 t\\right)=\\frac{1}{4}\\left(t-\\frac{3}{2}\\right)^{2}-\\frac{9}{16}$ ,\n所以函数 $k=f(t)$ 的最小值为 $-\\frac{9}{16}$", "solution": "null", "level": "高二", "question": "14. 设向量 $\\boldsymbol{a}=(\\sqrt{3},-1), \\boldsymbol{b}=\\left(\\frac{1}{2}, \\frac{\\sqrt{3}}{2}\\right), k, t$ 是两个不同时为零的实数. 若向量 $\\boldsymbol{x}=\\boldsymbol{a}+(t-$ 3) $\\boldsymbol{b}$ 与 $\\boldsymbol{y}=-k \\boldsymbol{a}+\\boldsymbol{t} \\boldsymbol{b}$ 垂直.\n\n(1)求 $k$ 关于 $t$ 的函数关系式;\n\n(2)求函数 $k=f(t)$ 的最小值.", "options": [], "subject": "解析几何", "analysis": "(1)因为 $\\boldsymbol{a}=(\\sqrt{3},-1), \\boldsymbol{b}=\\left(\\frac{1}{2}, \\frac{\\sqrt{3}}{2}\\right)$ ,所以 $\\boldsymbol{a} \\cdot \\boldsymbol{b}=0$ ,且 $|\\boldsymbol{a}|=2 ,|\\boldsymbol{b}|=1$.\n\n又因为 $\\boldsymbol{x} \\perp \\boldsymbol{y}$ ,所以 $x \\cdot y=0$ ,即 $[\\boldsymbol{a}+(t-3) \\boldsymbol{b}] \\cdot(-k \\boldsymbol{a}+\\boldsymbol{t} \\boldsymbol{b})=0$ ,\n\n所以 $-k \\boldsymbol{a}^{2}-k(t-3) \\boldsymbol{a} \\cdot \\boldsymbol{b}+t \\boldsymbol{a} \\cdot \\boldsymbol{b}+t(t-3) \\boldsymbol{b}^{2}=0$.\n\n因为 $|\\boldsymbol{a}|=2 ,|\\boldsymbol{b}|=1, \\boldsymbol{a} \\cdot \\boldsymbol{b}=0$ ,\n\n所以 $-4 k+t^{2}-3 t=0$, 所以 $k=\\frac{1}{4}\\left(t^{2}-3 t\\right)$.\n\n(2)由(1)知, $k=\\frac{1}{4}\\left(t^{2}-3 t\\right)=\\frac{1}{4}\\left(t-\\frac{3}{2}\\right)^{2}-\\frac{9}{16}$ ,\n所以函数 $k=f(t)$ 的最小值为 $-\\frac{9}{16}$."} {"id": "17988", "image": [], "answer": "2", "solution": "null", "level": "高二", "question": "9. 已知平面向量 $\\boldsymbol{a}=(1, x), \\boldsymbol{b}=(2 x+3,-x)(x \\in \\mathbf{R})$.\n\n(1)若 $\\boldsymbol{a} \\perp \\boldsymbol{b}$, 求 $x$ 的值;\n\n(2)若 $\\boldsymbol{a} / / \\boldsymbol{b}$, 求 $|\\boldsymbol{a}-\\boldsymbol{b}|$.", "options": [], "subject": "解析几何", "analysis": "(1) 由 $\\boldsymbol{a} \\perp \\boldsymbol{b}$ 得 $2 x+3-x^{2}=0$ ,即 $(x-3)(x+1)=0$. 解得 $x=3$ 或 $x=-1$.\n\n(2)由 $\\boldsymbol{a} / / \\boldsymbol{b}$ , 得 $2 x^{2}+3 x+x=0$ ,即 $2 x^{2}+4 x=0$ ,解得 $x=0$ 或 $x=-2$.\n\n当 $x=0$ 时, $\\boldsymbol{a}=(1,0), \\boldsymbol{b}=(3,0)$ ,\n\n所以 $\\boldsymbol{a}-\\boldsymbol{b}=(-2,0)$.\n\n此时 $|\\boldsymbol{a}-\\boldsymbol{b}|=2$.\n\n当 $x=-2$ 时, $\\boldsymbol{a}=(1,-2), \\boldsymbol{b}=(-1,2)$,\n\n则 $\\boldsymbol{a}-\\boldsymbol{b}=(2 ,-4)$. 故 $|\\boldsymbol{a}-\\boldsymbol{b}|=\\sqrt{2^{2}+(-4)^{2}}=2 \\sqrt{5}$."} {"id": "17993", "image": ["9036.jpg"], "answer": "$\\left(-\\frac{\\sqrt{10}}{5}, \\frac{3 \\sqrt{10}}{5}\\right)$", "solution": "null", "level": "高二", "question": "13. 在平面直角坐标系 $x O y$ 中, 已知点 $A(0,1)$ 和点 $B(-3,4)$. 若点 $C$ 在 $\\angle A O B$ 的平分线上且 $|\\overrightarrow{O C}|=2$, 则 $\\overrightarrow{O C}=$", "options": [], "subject": "解析几何", "analysis": "如图,已知 $A(0,1), B(-3 , 4)$ ,设 $E(0,5) , D(-3 , 9)$ ,\n\n\n\n$\\therefore$ 四边形 $O B D E$ 为菱形, $\\therefore \\angle A O B$ 的平分线是菱形 $O B D E$ 的对角线 $O D$.\n\n设 $C\\left(x_{1}, y_{1}\\right) , \\because|\\overrightarrow{O C}|=2 ,|\\overrightarrow{O D}|=3 \\sqrt{10} , \\therefore \\overrightarrow{O C}=\\frac{2}{3 \\sqrt{10}} \\overrightarrow{O D}$.\n\n$\\therefore \\overrightarrow{O C}=\\left(x_{1}, y_{1}\\right)=\\frac{2}{3 \\sqrt{10}} \\times(-3,9)=\\left(-\\frac{\\sqrt{10}}{5}, \\frac{3 \\sqrt{10}}{5}\\right)$.\n\n答案: $\\left(-\\frac{\\sqrt{10}}{5}, \\frac{3 \\sqrt{10}}{5}\\right)$"} {"id": "18009", "image": [], "answer": "(1) 当 $\\overrightarrow{A B}$ 为零向量时, 点 $B$ 与点 $A$ 重合, 此时 $x=0, y=-\\frac{\\sqrt{2}}{2}$.\n\n(2) 当 $\\vec{B} B$ 为单位向量时, $|\\overrightarrow{A B}|=1$, 即 $A$ 与 $B$ 两点间的距离为 1 ,所以 $\\sqrt{(x-0)^{2}+\\left(y+\\frac{\\sqrt{2}}{2}\\right)^{2}}=1$, 即 $x^{2}+\\left(y+\\frac{\\sqrt{2}}{2}\\right)^{2}=1$,将 $y=x-\\frac{\\sqrt{2}}{2}$ 代入, 得 $2 x^{2}=1$,所以 $x=\\frac{\\sqrt{2}}{2}, y=0$ 或 $x=-\\frac{\\sqrt{2}}{2}, y=-\\sqrt{2}$.", "solution": "null", "level": "高二", "question": "16. 已知直线 $I: y=x_{x}-\\frac{\\sqrt{2}}{2}$, 点 $A\\left(0,-\\frac{\\sqrt{2}}{2}\\right), B(x, y)$ 是直线 $l$ 上的两点.\n\n(1) 若 $\\overrightarrow{A B}$ 为零向量, 求 $x, y$ 的值;\n\n(2) 若 $\\overrightarrow{A B}$ 为单位向量, 求 $x, y$ 的值.", "options": [], "subject": "解析几何", "analysis": "(1) 当 $\\overrightarrow{A B}$ 为零向量时, 点 $B$ 与点 $A$ 重合, 此时 $x=0, y=-\\frac{\\sqrt{2}}{2}$.\n\n(2) 当 $\\vec{B} B$ 为单位向量时, $|\\overrightarrow{A B}|=1$, 即 $A$ 与 $B$ 两点间的距离为 1 ,所以 $\\sqrt{(x-0)^{2}+\\left(y+\\frac{\\sqrt{2}}{2}\\right)^{2}}=1$, 即 $x^{2}+\\left(y+\\frac{\\sqrt{2}}{2}\\right)^{2}=1$,将 $y=x-\\frac{\\sqrt{2}}{2}$ 代入, 得 $2 x^{2}=1$,所以 $x=\\frac{\\sqrt{2}}{2}, y=0$ 或 $x=-\\frac{\\sqrt{2}}{2}, y=-\\sqrt{2}$."} {"id": "18023", "image": ["9046.jpg", "9046.jpg"], "answer": "(1)证明: 如图, $\\overrightarrow{O B}+\\overrightarrow{O C}=2 \\overrightarrow{O D} , \\overrightarrow{O A}+\\overrightarrow{O C}=2 \\overrightarrow{O E} , \\because \\overrightarrow{O A}+2 \\overrightarrow{O B}+3 \\overrightarrow{O C}=(\\overrightarrow{O A}+\\overrightarrow{O C})$ $+2(\\overrightarrow{O B}+\\overrightarrow{O C})=2(2 \\overrightarrow{O D}+\\overrightarrow{O E})=0$,\n\n\n\n即 $2 \\overrightarrow{O D}+\\overrightarrow{O E}=0$ ,\n\n$\\therefore \\overrightarrow{O D}$ 与 $\\overrightarrow{O E}$ 共线,\n\n又 $\\overrightarrow{O D}$ 与 $\\overrightarrow{O E}$ 有公共点 0 ,\n\n$\\therefore D, E, O$ 三点共线.\n\n(2)由(1)知 $2|\\overrightarrow{O D}|=|\\overrightarrow{O E}|$ ,\n\n$\\therefore S_{\\triangle A O C}=2 S_{\\triangle C O E}=2 \\times \\frac{2}{3} S_{\\triangle C D E}=2 \\times \\frac{2}{3} \\times \\frac{1}{4} S_{\\triangle A B C}=\\frac{1}{3} S_{\\triangle A B C}, \\quad \\therefore \\frac{S_{\\triangle A B C}}{S_{\\triangle A O C}}=3$.", "solution": "null", "level": "高二", "question": "15. 设 $O$ 为 $\\triangle A B C$ 内任一点, 且满足 $\\overrightarrow{O A}+2 \\overrightarrow{O B}+3 \\overrightarrow{O C}=0$.\n\n(1)若 $D, E$ 分别是 $B C, C A$ 的中点, 求证: $D, E, O$ 共线;\n\n(2)求 $\\triangle A B C$ 与 $\\triangle A O C$ 的面积之比.", "options": [], "subject": "解析几何", "analysis": "(1)证明: 如图, $\\overrightarrow{O B}+\\overrightarrow{O C}=2 \\overrightarrow{O D} , \\overrightarrow{O A}+\\overrightarrow{O C}=2 \\overrightarrow{O E} , \\because \\overrightarrow{O A}+2 \\overrightarrow{O B}+3 \\overrightarrow{O C}=(\\overrightarrow{O A}+\\overrightarrow{O C})$ $+2(\\overrightarrow{O B}+\\overrightarrow{O C})=2(2 \\overrightarrow{O D}+\\overrightarrow{O E})=0$,\n\n\n\n即 $2 \\overrightarrow{O D}+\\overrightarrow{O E}=0$ ,\n\n$\\therefore \\overrightarrow{O D}$ 与 $\\overrightarrow{O E}$ 共线,\n\n又 $\\overrightarrow{O D}$ 与 $\\overrightarrow{O E}$ 有公共点 0 ,\n\n$\\therefore D, E, O$ 三点共线.\n\n(2)由(1)知 $2|\\overrightarrow{O D}|=|\\overrightarrow{O E}|$ ,\n\n$\\therefore S_{\\triangle A O C}=2 S_{\\triangle C O E}=2 \\times \\frac{2}{3} S_{\\triangle C D E}=2 \\times \\frac{2}{3} \\times \\frac{1}{4} S_{\\triangle A B C}=\\frac{1}{3} S_{\\triangle A B C}, \\quad \\therefore \\frac{S_{\\triangle A B C}}{S_{\\triangle A O C}}=3$."} {"id": "18031", "image": ["9049.jpg", "9050.jpg", "9050.jpg"], "answer": "(1)如图,延长 $A D$ 到 $G$ ,使 $\\overrightarrow{A G}=2 \\overrightarrow{A D}$ ,连接 $B G , C G$ ,得到平行四边形 $A B G C$.\n\n\n\n则 $\\overrightarrow{A G}=\\boldsymbol{a}+\\boldsymbol{b}, \\overrightarrow{A D}=\\frac{1}{2} \\overrightarrow{A G}=\\frac{1}{2}(\\boldsymbol{a}+\\boldsymbol{b}), \\overrightarrow{A E}=\\frac{2}{3} \\overrightarrow{A D}=\\frac{1}{3}(\\boldsymbol{a}+\\boldsymbol{b})$,\n\n$\\overrightarrow{A F}=\\frac{1}{2} \\overrightarrow{A C}=\\frac{1}{2} \\boldsymbol{b}$,\n\n$\\overrightarrow{B E}=\\overrightarrow{A E}-\\overrightarrow{A B}=\\frac{1}{3}(\\boldsymbol{a}+\\boldsymbol{b})-\\boldsymbol{a}=\\frac{1}{3}(\\boldsymbol{b}-2 \\boldsymbol{a})$,\n\n$\\overrightarrow{B F}=\\overrightarrow{A F}-\\overrightarrow{A B}=\\frac{1}{2} \\boldsymbol{b}-\\boldsymbol{a}=\\frac{1}{2}(\\boldsymbol{b}-2 \\boldsymbol{a})$.\n\n(2)证明: 由(1)知, $\\overrightarrow{B E}=\\frac{2}{3} \\overrightarrow{B F} , \\therefore \\overrightarrow{B E} , \\overrightarrow{B F}$ 共线.\n\n又 $\\because \\overrightarrow{B E} , \\overrightarrow{B F}$ 有公共点 $B , \\therefore B, E , F$ 三点共线.", "solution": "null", "level": "高二", "question": "9.如图所示, 在 $\\triangle A B C$ 中, $D, F$ 分别是 $B C, A C$ 的中点, $\\overrightarrow{A E}=\\frac{2}{3} \\overrightarrow{A D}, \\overrightarrow{A B}=\\boldsymbol{a}, \\overrightarrow{A C}=\\boldsymbol{b}$.\n\n\n\n(1)用 $\\boldsymbol{a}, \\boldsymbol{b}$ 表示 $\\overrightarrow{A D}, \\overrightarrow{A E}, \\overrightarrow{A F}, \\overrightarrow{B E}, \\overrightarrow{B F}$;\n\n(2)求证: $B, E, F$ 三点共线.", "options": [], "subject": "解析几何", "analysis": "(1)如图,延长 $A D$ 到 $G$ ,使 $\\overrightarrow{A G}=2 \\overrightarrow{A D}$ ,连接 $B G , C G$ ,得到平行四边形 $A B G C$.\n\n\n\n则 $\\overrightarrow{A G}=\\boldsymbol{a}+\\boldsymbol{b}, \\overrightarrow{A D}=\\frac{1}{2} \\overrightarrow{A G}=\\frac{1}{2}(\\boldsymbol{a}+\\boldsymbol{b}), \\overrightarrow{A E}=\\frac{2}{3} \\overrightarrow{A D}=\\frac{1}{3}(\\boldsymbol{a}+\\boldsymbol{b})$,\n\n$\\overrightarrow{A F}=\\frac{1}{2} \\overrightarrow{A C}=\\frac{1}{2} \\boldsymbol{b}$,\n\n$\\overrightarrow{B E}=\\overrightarrow{A E}-\\overrightarrow{A B}=\\frac{1}{3}(\\boldsymbol{a}+\\boldsymbol{b})-\\boldsymbol{a}=\\frac{1}{3}(\\boldsymbol{b}-2 \\boldsymbol{a})$,\n\n$\\overrightarrow{B F}=\\overrightarrow{A F}-\\overrightarrow{A B}=\\frac{1}{2} \\boldsymbol{b}-\\boldsymbol{a}=\\frac{1}{2}(\\boldsymbol{b}-2 \\boldsymbol{a})$.\n\n(2)证明: 由(1)知, $\\overrightarrow{B E}=\\frac{2}{3} \\overrightarrow{B F} , \\therefore \\overrightarrow{B E} , \\overrightarrow{B F}$ 共线.\n\n又 $\\because \\overrightarrow{B E} , \\overrightarrow{B F}$ 有公共点 $B , \\therefore B, E , F$ 三点共线."} {"id": "18037", "image": [], "answer": "$\\beta=\\frac{\\pi}{3}$", "solution": "null", "level": "高二", "question": "15. 已知 $\\cos \\alpha=\\frac{1}{7}, \\sin (\\alpha+\\beta)=\\frac{5 \\sqrt{3}}{14}, \\alpha 、 \\beta \\in\\left(0, \\frac{\\pi}{2}\\right)$, 求 $\\beta$ 的值.", "options": [], "subject": "解析几何", "analysis": "$\\because \\alpha, \\beta \\in\\left(0 , \\frac{\\pi}{2}\\right) , \\therefore \\alpha+\\beta \\in(0 , \\pi)$ ,\n\n$\\because \\cos \\alpha=\\frac{1}{7}, \\sin (\\alpha+\\beta)=\\frac{5 \\sqrt{3}}{14}$,\n\n$\\therefore \\sin \\alpha=\\frac{4 \\sqrt{3}}{7}, \\cos (\\alpha+\\beta)= \\pm \\frac{11}{14}$,\n\n当 $\\cos (\\alpha+\\beta)=-\\frac{11}{14}$ 时,\n\n$\\cos \\beta=\\cos [(\\alpha+\\beta)-\\alpha]=\\cos (\\alpha+\\beta) \\cos \\alpha+\\sin (\\alpha+\\beta) \\sin \\alpha=\\left(-\\frac{11}{14}\\right) \\times \\frac{1}{7}+\\frac{5 \\sqrt{3}}{14} \\times \\frac{4 \\sqrt{3}}{7}=\\frac{1}{2} ,$\n\n$\\because \\beta \\in\\left(0, \\frac{\\pi}{2}\\right), \\quad \\therefore \\beta=\\frac{\\pi}{3}$;\n当 $\\cos (\\alpha+\\beta)=\\frac{11}{14}$ 时,\n\n$\\cos \\beta=\\cos [(\\alpha+\\beta)-\\alpha]=\\cos (\\alpha+\\beta) \\cos \\alpha+\\sin (\\alpha+\\beta) \\sin \\alpha=\\frac{11}{14} \\times \\frac{1}{7}+\\frac{5 \\sqrt{3}}{14} \\times \\frac{4 \\sqrt{3}}{7}=\\frac{71}{98}<\\frac{11}{14}=$ $\\cos (\\alpha+\\beta)$,\n\n且 $\\alpha+\\beta \\in\\left(0, \\frac{\\pi}{2}\\right), \\beta \\in\\left(0, \\frac{\\pi}{2}\\right)$,\n\n所以 $\\beta>\\alpha+\\beta$ ,即 $\\alpha<0$ ,与已知矛盾,舍去,所以 $\\beta=\\frac{\\pi}{3}$."} {"id": "18047", "image": [], "answer": "(1)因为 $\\alpha, \\beta \\in\\left(0 , \\frac{\\pi}{2}\\right)$ ,所以 $\\alpha-\\beta \\in\\left(-\\frac{\\pi}{2} , \\frac{\\pi}{2}\\right)$.\n\n又因为 $\\sin (\\alpha-\\beta)=\\frac{\\sqrt{10}}{10}>0$ ,所以 $0<\\alpha-\\beta<\\frac{\\pi}{2}$.\n\n所以 $\\sin \\alpha=\\sqrt{1-\\cos ^{2} \\alpha}=\\frac{2 \\sqrt{5}}{5}$ ,\n\n$\\cos (\\alpha-\\beta)=\\sqrt{1-\\sin ^{2}(\\alpha-\\beta)}=\\frac{3 \\sqrt{10}}{10}$.\n$\\cos (2 \\alpha-\\beta)=\\cos [\\alpha+(\\alpha-\\beta)]$\n\n$=\\cos \\alpha \\cos (\\alpha-\\beta)-\\sin \\alpha \\sin (\\alpha-\\beta)$\n\n$=\\frac{\\sqrt{5}}{5} \\times \\frac{3 \\sqrt{10}}{10}-\\frac{2 \\sqrt{5}}{5} \\times \\frac{\\sqrt{10}}{10}=\\frac{\\sqrt{2}}{10}$.\n\n(2) $\\cos \\beta=\\cos [\\alpha-(\\alpha-\\beta)]$\n\n$=\\cos \\alpha \\cos (\\alpha-\\beta)+\\sin \\alpha \\sin (\\alpha-\\beta)$\n\n$=\\frac{\\sqrt{5}}{5} \\times \\frac{3 \\sqrt{10}}{10}+\\frac{2 \\sqrt{5}}{5} \\times \\frac{\\sqrt{10}}{10}=\\frac{\\sqrt{2}}{2}$,\n\n又因为 $\\beta \\in\\left(0, \\frac{\\pi}{2}\\right)$ ,所以 $\\beta=\\frac{\\pi}{4}$.", "solution": "null", "level": "高二", "question": "10. 已知 $\\cos \\alpha=\\frac{\\sqrt{5}}{5}, \\sin (\\alpha-\\beta)=\\frac{\\sqrt{10}}{10}$, 且 $\\alpha, \\beta \\in\\left(0, \\frac{\\pi}{2}\\right)$. 求:\n\n(1) $\\cos (2 \\alpha-\\beta)$ 的值;\n\n(2) $\\beta$ 的值.", "options": [], "subject": "解析几何", "analysis": "(1)因为 $\\alpha, \\beta \\in\\left(0 , \\frac{\\pi}{2}\\right)$ ,所以 $\\alpha-\\beta \\in\\left(-\\frac{\\pi}{2} , \\frac{\\pi}{2}\\right)$.\n\n又因为 $\\sin (\\alpha-\\beta)=\\frac{\\sqrt{10}}{10}>0$ ,所以 $0<\\alpha-\\beta<\\frac{\\pi}{2}$.\n\n所以 $\\sin \\alpha=\\sqrt{1-\\cos ^{2} \\alpha}=\\frac{2 \\sqrt{5}}{5}$ ,\n\n$\\cos (\\alpha-\\beta)=\\sqrt{1-\\sin ^{2}(\\alpha-\\beta)}=\\frac{3 \\sqrt{10}}{10}$.\n$\\cos (2 \\alpha-\\beta)=\\cos [\\alpha+(\\alpha-\\beta)]$\n\n$=\\cos \\alpha \\cos (\\alpha-\\beta)-\\sin \\alpha \\sin (\\alpha-\\beta)$\n\n$=\\frac{\\sqrt{5}}{5} \\times \\frac{3 \\sqrt{10}}{10}-\\frac{2 \\sqrt{5}}{5} \\times \\frac{\\sqrt{10}}{10}=\\frac{\\sqrt{2}}{10}$.\n\n(2) $\\cos \\beta=\\cos [\\alpha-(\\alpha-\\beta)]$\n\n$=\\cos \\alpha \\cos (\\alpha-\\beta)+\\sin \\alpha \\sin (\\alpha-\\beta)$\n\n$=\\frac{\\sqrt{5}}{5} \\times \\frac{3 \\sqrt{10}}{10}+\\frac{2 \\sqrt{5}}{5} \\times \\frac{\\sqrt{10}}{10}=\\frac{\\sqrt{2}}{2}$,\n\n又因为 $\\beta \\in\\left(0, \\frac{\\pi}{2}\\right)$ ,所以 $\\beta=\\frac{\\pi}{4}$."} {"id": "18052", "image": [], "answer": "(1) $\\because f(x)=A \\sin \\left(x+\\frac{\\pi}{3}\\right)$ ,且 $f\\left(\\frac{5 \\pi}{12}\\right)=\\frac{3 \\sqrt{2}}{2}$ ,\n\n$\\therefore A \\sin \\left(\\frac{5 \\pi}{12}+\\frac{\\pi}{3}\\right)=\\frac{3 \\sqrt{2}}{2}$,\n\n即 $A \\sin \\frac{3 \\pi}{4}=\\frac{3 \\sqrt{2}}{2} , \\therefore A=3$.\n\n(2)由(1)知 $f(x)=3 \\sin \\left(x+\\frac{\\pi}{3}\\right)$ ,\n\n$\\because f(\\theta)-f(-\\theta)=\\sqrt{3}$,\n\n$\\therefore 3 \\sin \\left(\\theta+\\frac{\\pi}{3}\\right)-3 \\sin \\left(-\\theta+\\frac{\\pi}{3}\\right)=\\sqrt{3}$,\n\n展开得 $3\\left(\\frac{1}{2} \\sin \\theta+\\frac{\\sqrt{3}}{2} \\cos \\theta+\\frac{1}{2} \\sin \\theta-\\frac{\\sqrt{3}}{2} \\cos \\theta\\right)=\\sqrt{3}$ ,\n\n化简得 $\\sin \\theta=\\frac{\\sqrt{3}}{3}$.\n\n$$\n\\begin{aligned}\n& \\because \\theta \\in\\left(0, \\frac{\\pi}{2}\\right), \\therefore \\cos \\theta=\\frac{\\sqrt{6}}{3} \\\\\n& \\therefore\\left(\\frac{\\pi}{6}-\\theta\\right)=3 \\sin \\left[\\left(\\frac{\\pi}{6}-\\theta\\right)+\\frac{\\pi}{3}\\right]=3 \\sin \\left(\\frac{\\pi}{2}-\\theta\\right)=3 \\cos \\theta=\\sqrt{6}\n\\end{aligned}\n$$", "solution": "null", "level": "高二", "question": "15. 已知函数 $f(x)=A \\sin \\left(x+\\frac{\\pi}{3}\\right), x \\in \\mathbf{R}$, 且 $f\\left(\\frac{5 \\pi}{12}\\right)=\\frac{3 \\sqrt{2}}{2}$.\n\n(1)求 $A$ 的值;\n\n(2)若 $f(\\theta)-f(-\\theta)=\\sqrt{3}, \\theta \\in\\left(0, \\frac{\\pi}{2}\\right)$, 求 $f\\left(\\frac{\\pi}{6}-\\theta\\right)$.", "options": [], "subject": "解析几何", "analysis": "(1) $\\because f(x)=A \\sin \\left(x+\\frac{\\pi}{3}\\right)$ ,且 $f\\left(\\frac{5 \\pi}{12}\\right)=\\frac{3 \\sqrt{2}}{2}$ ,\n\n$\\therefore A \\sin \\left(\\frac{5 \\pi}{12}+\\frac{\\pi}{3}\\right)=\\frac{3 \\sqrt{2}}{2}$,\n\n即 $A \\sin \\frac{3 \\pi}{4}=\\frac{3 \\sqrt{2}}{2} , \\therefore A=3$.\n\n(2)由(1)知 $f(x)=3 \\sin \\left(x+\\frac{\\pi}{3}\\right)$ ,\n\n$\\because f(\\theta)-f(-\\theta)=\\sqrt{3}$,\n\n$\\therefore 3 \\sin \\left(\\theta+\\frac{\\pi}{3}\\right)-3 \\sin \\left(-\\theta+\\frac{\\pi}{3}\\right)=\\sqrt{3}$,\n\n展开得 $3\\left(\\frac{1}{2} \\sin \\theta+\\frac{\\sqrt{3}}{2} \\cos \\theta+\\frac{1}{2} \\sin \\theta-\\frac{\\sqrt{3}}{2} \\cos \\theta\\right)=\\sqrt{3}$ ,\n\n化简得 $\\sin \\theta=\\frac{\\sqrt{3}}{3}$.\n\n$$\n\\begin{aligned}\n& \\because \\theta \\in\\left(0, \\frac{\\pi}{2}\\right), \\therefore \\cos \\theta=\\frac{\\sqrt{6}}{3} \\\\\n& \\therefore\\left(\\frac{\\pi}{6}-\\theta\\right)=3 \\sin \\left[\\left(\\frac{\\pi}{6}-\\theta\\right)+\\frac{\\pi}{3}\\right]=3 \\sin \\left(\\frac{\\pi}{2}-\\theta\\right)=3 \\cos \\theta=\\sqrt{6}\n\\end{aligned}\n$$"} {"id": "18066", "image": [], "answer": "(1) $\\because \\tan \\left(\\frac{\\pi}{4}+\\alpha\\right)=\\frac{\\tan \\frac{\\pi}{4}+\\tan \\alpha}{1-\\tan \\frac{\\pi}{4} \\tan \\alpha}=\\frac{1+\\tan \\alpha}{1-\\tan \\alpha}=2$ ,\n\n$\\therefore \\tan \\alpha=\\frac{1}{3}$.\n\n(2) $\\frac{\\sin (\\alpha+\\beta)-2 \\sin \\alpha \\cos \\beta}{2 \\sin \\alpha \\sin \\beta+\\cos (\\alpha+\\beta)}$\n\n$=\\frac{\\sin \\alpha \\cos \\beta+\\cos \\alpha \\sin \\beta-2 \\sin \\alpha \\cos \\beta}{2 \\sin \\alpha \\sin \\beta+\\cos \\alpha \\cos \\beta-\\sin \\alpha \\sin \\beta}$\n\n$=\\frac{\\sin \\beta \\cos \\alpha-\\cos \\beta \\sin \\alpha}{\\cos \\alpha \\cos \\beta+\\sin \\alpha \\sin \\beta}$\n\n$=\\frac{\\sin (\\beta-\\alpha)}{\\cos (\\beta-\\alpha)}$\n\n$=\\tan (\\beta-\\alpha)=\\frac{\\tan \\beta-\\tan \\alpha}{1+\\tan \\beta \\tan \\alpha}$\n\n$=\\frac{\\frac{1}{2}-\\frac{1}{3}}{1+\\frac{1}{2} \\times \\frac{1}{3}}=\\frac{1}{7}$.", "solution": "null", "level": "高二", "question": "9. 已知 $\\tan \\left(\\frac{\\pi}{4}+\\alpha\\right)=2, \\tan \\beta=\\frac{1}{2}$.\n\n求: (1)tan $\\alpha$ 的值;\n\n(2) $\\frac{\\sin (\\alpha+\\beta)-2 \\sin \\alpha \\cos \\beta}{2 \\sin \\alpha \\sin \\beta+\\cos (\\alpha+\\beta)}$ 的值.", "options": [], "subject": "解析几何", "analysis": "(1) $\\because \\tan \\left(\\frac{\\pi}{4}+\\alpha\\right)=\\frac{\\tan \\frac{\\pi}{4}+\\tan \\alpha}{1-\\tan \\frac{\\pi}{4} \\tan \\alpha}=\\frac{1+\\tan \\alpha}{1-\\tan \\alpha}=2$ ,\n\n$\\therefore \\tan \\alpha=\\frac{1}{3}$.\n\n(2) $\\frac{\\sin (\\alpha+\\beta)-2 \\sin \\alpha \\cos \\beta}{2 \\sin \\alpha \\sin \\beta+\\cos (\\alpha+\\beta)}$\n\n$=\\frac{\\sin \\alpha \\cos \\beta+\\cos \\alpha \\sin \\beta-2 \\sin \\alpha \\cos \\beta}{2 \\sin \\alpha \\sin \\beta+\\cos \\alpha \\cos \\beta-\\sin \\alpha \\sin \\beta}$\n\n$=\\frac{\\sin \\beta \\cos \\alpha-\\cos \\beta \\sin \\alpha}{\\cos \\alpha \\cos \\beta+\\sin \\alpha \\sin \\beta}$\n\n$=\\frac{\\sin (\\beta-\\alpha)}{\\cos (\\beta-\\alpha)}$\n\n$=\\tan (\\beta-\\alpha)=\\frac{\\tan \\beta-\\tan \\alpha}{1+\\tan \\beta \\tan \\alpha}$\n\n$=\\frac{\\frac{1}{2}-\\frac{1}{3}}{1+\\frac{1}{2} \\times \\frac{1}{3}}=\\frac{1}{7}$."} {"id": "18067", "image": [], "answer": "$\\because \\alpha+2 \\beta=\\frac{2 \\pi}{3} , \\therefore \\frac{\\alpha}{2}+\\beta=\\frac{\\pi}{3}$,\n\n$\\therefore \\tan \\left(\\frac{\\alpha}{2}+\\beta\\right)=\\frac{\\tan \\frac{\\alpha}{2}+\\tan \\beta}{1-\\tan \\frac{\\alpha}{2} \\tan \\beta}=\\sqrt{3}$.\n\n$\\because \\tan \\frac{\\alpha}{2} \\tan \\beta=2-\\sqrt{3}, \\therefore \\tan \\frac{\\alpha}{2}+\\tan \\beta=3-\\sqrt{3}$.\n\n故 $\\tan \\frac{\\alpha}{2}, \\tan \\beta$ 是方程 $x^{2}-(3-\\sqrt{3}) x+2-\\sqrt{3}=0$ 的两根,解得 $x_{1}=2-\\sqrt{3} , x_{2}=1$.\n\n$\\because \\alpha, \\beta$ 都为锐角, $\\therefore 0<\\frac{\\alpha}{2}<\\frac{\\pi}{4} , \\therefore 0<\\tan \\frac{\\alpha}{2}<1$ ,\n\n$\\therefore \\tan \\frac{\\alpha}{2}=2-\\sqrt{3}, \\tan \\beta=1$,\n\n$\\therefore$ 存在满足条件的锐角 $\\alpha, \\beta$ ,且 $\\alpha=\\frac{\\pi}{6} , \\beta=\\frac{\\pi}{4}$.", "solution": "null", "level": "高二", "question": "15. 是否存在锐角 $\\alpha$ 和 $\\beta$, 使得 $\\alpha+2 \\beta=\\frac{2 \\pi}{3}, \\tan \\frac{\\alpha}{2} \\tan \\beta=2-\\sqrt{3}$ 同时成立? 若存在, 求出 $\\alpha$ 和 $\\beta$的值; 若不存在, 说明理由.", "options": [], "subject": "解析几何", "analysis": "$\\because \\alpha+2 \\beta=\\frac{2 \\pi}{3} , \\therefore \\frac{\\alpha}{2}+\\beta=\\frac{\\pi}{3}$,\n\n$\\therefore \\tan \\left(\\frac{\\alpha}{2}+\\beta\\right)=\\frac{\\tan \\frac{\\alpha}{2}+\\tan \\beta}{1-\\tan \\frac{\\alpha}{2} \\tan \\beta}=\\sqrt{3}$.\n\n$\\because \\tan \\frac{\\alpha}{2} \\tan \\beta=2-\\sqrt{3}, \\therefore \\tan \\frac{\\alpha}{2}+\\tan \\beta=3-\\sqrt{3}$.\n\n故 $\\tan \\frac{\\alpha}{2}, \\tan \\beta$ 是方程 $x^{2}-(3-\\sqrt{3}) x+2-\\sqrt{3}=0$ 的两根,解得 $x_{1}=2-\\sqrt{3} , x_{2}=1$.\n\n$\\because \\alpha, \\beta$ 都为锐角, $\\therefore 0<\\frac{\\alpha}{2}<\\frac{\\pi}{4} , \\therefore 0<\\tan \\frac{\\alpha}{2}<1$ ,\n\n$\\therefore \\tan \\frac{\\alpha}{2}=2-\\sqrt{3}, \\tan \\beta=1$,\n\n$\\therefore$ 存在满足条件的锐角 $\\alpha, \\beta$ ,且 $\\alpha=\\frac{\\pi}{6} , \\beta=\\frac{\\pi}{4}$."} {"id": "18075", "image": [], "answer": "(1) $\\because \\tan \\left(\\frac{\\pi}{4}+\\alpha\\right)=\\frac{\\tan \\frac{\\pi}{4}+\\tan \\alpha}{1-\\tan \\frac{\\pi}{4} \\tan \\alpha}=\\frac{1+\\tan \\alpha}{1-\\tan \\alpha}=2$ ,\n\n$\\therefore \\tan \\alpha=\\frac{1}{3}$.\n\n(2) $\\frac{\\sin (\\alpha+\\beta)-2 \\sin \\alpha \\cos \\beta}{2 \\sin \\alpha \\sin \\beta+\\cos (\\alpha+\\beta)}$\n\n$=\\frac{\\sin \\alpha \\cos \\beta+\\cos \\alpha \\sin \\beta-2 \\sin \\alpha \\cos \\beta}{2 \\sin \\alpha \\sin \\beta+\\cos \\alpha \\cos \\beta-\\sin \\alpha \\sin \\beta}$\n\n$=\\frac{\\sin \\beta \\cos \\alpha-\\cos \\beta \\sin \\alpha}{\\cos \\alpha \\cos \\beta+\\sin \\alpha \\sin \\beta}$\n\n$=\\frac{\\sin (\\beta-\\alpha)}{\\cos (\\beta-\\alpha)}$\n\n$=\\tan (\\beta-\\alpha)=\\frac{\\tan \\beta-\\tan \\alpha}{1+\\tan \\beta \\tan \\alpha}$\n\n$=\\frac{\\frac{1}{2}-\\frac{1}{3}}{1+\\frac{1}{2} \\times \\frac{1}{3}}=\\frac{1}{7}$.", "solution": "null", "level": "高二", "question": "9. 已知 $\\tan \\left(\\frac{\\pi}{4}+\\alpha\\right)=2, \\tan \\beta=\\frac{1}{2}$.\n\n求: (1)tan $\\alpha$ 的值;\n\n(2) $\\frac{\\sin (\\alpha+\\beta)-2 \\sin \\alpha \\cos \\beta}{2 \\sin \\alpha \\sin \\beta+\\cos (\\alpha+\\beta)}$ 的值.", "options": [], "subject": "解析几何", "analysis": "(1) $\\because \\tan \\left(\\frac{\\pi}{4}+\\alpha\\right)=\\frac{\\tan \\frac{\\pi}{4}+\\tan \\alpha}{1-\\tan \\frac{\\pi}{4} \\tan \\alpha}=\\frac{1+\\tan \\alpha}{1-\\tan \\alpha}=2$ ,\n\n$\\therefore \\tan \\alpha=\\frac{1}{3}$.\n\n(2) $\\frac{\\sin (\\alpha+\\beta)-2 \\sin \\alpha \\cos \\beta}{2 \\sin \\alpha \\sin \\beta+\\cos (\\alpha+\\beta)}$\n\n$=\\frac{\\sin \\alpha \\cos \\beta+\\cos \\alpha \\sin \\beta-2 \\sin \\alpha \\cos \\beta}{2 \\sin \\alpha \\sin \\beta+\\cos \\alpha \\cos \\beta-\\sin \\alpha \\sin \\beta}$\n\n$=\\frac{\\sin \\beta \\cos \\alpha-\\cos \\beta \\sin \\alpha}{\\cos \\alpha \\cos \\beta+\\sin \\alpha \\sin \\beta}$\n\n$=\\frac{\\sin (\\beta-\\alpha)}{\\cos (\\beta-\\alpha)}$\n\n$=\\tan (\\beta-\\alpha)=\\frac{\\tan \\beta-\\tan \\alpha}{1+\\tan \\beta \\tan \\alpha}$\n\n$=\\frac{\\frac{1}{2}-\\frac{1}{3}}{1+\\frac{1}{2} \\times \\frac{1}{3}}=\\frac{1}{7}$."} {"id": "18081", "image": [], "answer": "(1) $f(x)=\\frac{1}{2} \\cos 2 x-\\frac{\\sqrt{3}}{2} \\sin 2 x-\\cos 2 x+\\sqrt{3} \\sin 2 x=\\frac{\\sqrt{3}}{2} \\sin 2 x-\\frac{1}{2} \\cos 2 x=\\sin \\left(2 x-\\frac{\\pi}{6}\\right)$.\n\n$(2) f(\\alpha)=\\sin \\left(2 \\alpha-\\frac{\\pi}{6}\\right)=\\frac{1}{7} , 2 \\alpha$ 是第一象限角,即 $2 k \\pi<2 \\alpha<\\frac{\\pi}{2}+2 k \\pi(k \\in \\mathbf{Z})$ ,\n\n$\\therefore 2 k \\pi-\\frac{\\pi}{6}<2 \\alpha-\\frac{\\pi}{6}<\\frac{\\pi}{3}+2 k \\pi, \\quad k \\in \\mathbf{Z}$ ,\n\n$\\therefore \\cos \\left(2 \\alpha-\\frac{\\pi}{6}\\right)=\\frac{4 \\sqrt{3}}{7}$,\n\n$\\therefore \\sin 2 \\alpha=\\sin \\left[\\left(2 \\alpha-\\frac{\\pi}{6}\\right)+\\frac{\\pi}{6}\\right]$\n\n$=\\sin \\left(2 \\alpha-\\frac{\\pi}{6}\\right) \\cdot \\cos \\frac{\\pi}{6}+\\cos \\left(2 \\alpha-\\frac{\\pi}{6}\\right) \\cdot \\sin \\frac{\\pi}{6}$\n\n$=\\frac{1}{7} \\times \\frac{\\sqrt{3}}{2}+\\frac{4 \\sqrt{3}}{7} \\times \\frac{1}{2}=\\frac{5 \\sqrt{3}}{14}$.", "solution": "null", "level": "高二", "question": "15. 已知函数 $f(x)=\\cos \\left(2 x+\\frac{\\pi}{3}\\right)+\\sin ^{2} x-\\cos ^{2} x+2 \\sqrt{3} \\sin x \\cos x$.\n\n(1) 化简 $f(x)$;\n\n(2)若 $f(\\alpha)=\\frac{1}{7}, 2 \\alpha$ 是第一象限角, 求 $\\sin 2 \\alpha$.", "options": [], "subject": "解析几何", "analysis": "(1) $f(x)=\\frac{1}{2} \\cos 2 x-\\frac{\\sqrt{3}}{2} \\sin 2 x-\\cos 2 x+\\sqrt{3} \\sin 2 x=\\frac{\\sqrt{3}}{2} \\sin 2 x-\\frac{1}{2} \\cos 2 x=\\sin \\left(2 x-\\frac{\\pi}{6}\\right)$.\n\n$(2) f(\\alpha)=\\sin \\left(2 \\alpha-\\frac{\\pi}{6}\\right)=\\frac{1}{7} , 2 \\alpha$ 是第一象限角,即 $2 k \\pi<2 \\alpha<\\frac{\\pi}{2}+2 k \\pi(k \\in \\mathbf{Z})$ ,\n\n$\\therefore 2 k \\pi-\\frac{\\pi}{6}<2 \\alpha-\\frac{\\pi}{6}<\\frac{\\pi}{3}+2 k \\pi, \\quad k \\in \\mathbf{Z}$ ,\n\n$\\therefore \\cos \\left(2 \\alpha-\\frac{\\pi}{6}\\right)=\\frac{4 \\sqrt{3}}{7}$,\n\n$\\therefore \\sin 2 \\alpha=\\sin \\left[\\left(2 \\alpha-\\frac{\\pi}{6}\\right)+\\frac{\\pi}{6}\\right]$\n\n$=\\sin \\left(2 \\alpha-\\frac{\\pi}{6}\\right) \\cdot \\cos \\frac{\\pi}{6}+\\cos \\left(2 \\alpha-\\frac{\\pi}{6}\\right) \\cdot \\sin \\frac{\\pi}{6}$\n\n$=\\frac{1}{7} \\times \\frac{\\sqrt{3}}{2}+\\frac{4 \\sqrt{3}}{7} \\times \\frac{1}{2}=\\frac{5 \\sqrt{3}}{14}$."} {"id": "19832", "image": [], "answer": "( I )由题知 1 和 2 是方程式 $\\mathrm{x} 2-3 \\mathrm{ax}+\\mathrm{b}=0$ 的根,\n\n由根与系数关系得 $\\left\\{\\begin{array}{l}1+2=3 a \\\\ 1 \\times 2=b\\end{array}\\right.$,\n\n解得 $a=1, b=2$.\n\n( II ) 方程 ( $\\mathrm{x}-\\mathrm{b})(\\mathrm{x}-\\mathrm{m})=0$ 两根为 $\\mathrm{x} 1=2, \\mathrm{x} 2=\\mathrm{m}$,\n\n当 $\\mathrm{m}<2$ 时, 所求不等式的解集为 $\\{\\mathrm{m} \\mid \\mathrm{m}<\\mathrm{x}<2\\}$,\n\n当 $\\mathrm{m}=2$ 时, 所求不等式的解集为 $\\emptyset$,\n\n当 $\\mathrm{m}>2$ 时, 所求不等式的解集为 $\\{\\mathrm{x} \\mid 2<\\mathrm{x}<\\mathrm{m}\\}$.", "solution": "null", "level": "高二", "question": "已知不等式 $x^{2}-3 a x+b>0$ 的解集为 $\\{x \\mid x<1$ 或 $x>2\\}$.\n\n(I) 求 $a, b$ 的值;\n\n(II) 解不等式 $(x-b)(x-m)<0$.", "options": [], "subject": "解析几何", "analysis": "( I ) 由题知 1 和 2 是方程式 $\\mathrm{x} 2-3 \\mathrm{ax}+\\mathrm{b}=0$ 的根,\n\n由根与系数关系得 $\\left\\{\\begin{array}{l}1+2=3 a \\\\ 1 \\times 2=b\\end{array}\\right.$,\n\n解得 $a=1, b=2$.\n\n( II ) 方程 ( $\\mathrm{x}-\\mathrm{b})(\\mathrm{x}-\\mathrm{m})=0$ 两根为 $\\mathrm{x} 1=2, \\mathrm{x} 2=\\mathrm{m}$,\n\n当 $\\mathrm{m}<2$ 时, 所求不等式的解集为 $\\{\\mathrm{m} \\mid \\mathrm{m}<\\mathrm{x}<2\\}$,\n\n当 $\\mathrm{m}=2$ 时, 所求不等式的解集为 $\\emptyset$,\n\n当 $\\mathrm{m}>2$ 时, 所求不等式的解集为 $\\{\\mathrm{x} \\mid 2<\\mathrm{x}<\\mathrm{m}\\}$."} {"id": "18095", "image": [], "answer": "(1) $\\because \\cos \\beta=\\frac{\\sqrt{5}}{5} , \\beta \\in(0, \\pi) , \\therefore \\sin \\beta=\\frac{2 \\sqrt{5}}{5}$ ,\n\n$\\tan \\beta=2$.\n\n$\\therefore \\tan (\\alpha+\\beta)=\\frac{\\tan \\alpha+\\tan \\beta}{1-\\tan \\alpha \\tan \\beta}=1$.\n\n(2) $\\because \\tan \\alpha=-\\frac{1}{3}, \\alpha \\in(0, \\pi), \\therefore \\sin \\alpha=\\frac{\\sqrt{10}}{10}, \\cos \\alpha=-\\frac{3 \\sqrt{10}}{10}$.\n\n$f(x)=\\sqrt{2}(\\sin x \\cos \\alpha-\\cos x \\sin \\alpha)+\\cos x \\cos \\beta-\\sin x \\sin \\beta$\n\n$=-\\frac{3 \\sqrt{5}}{5} \\sin x-\\frac{\\sqrt{5}}{5} \\cos x+\\frac{\\sqrt{5}}{5} \\cos x-\\frac{2 \\sqrt{5}}{5} \\sin x$\n\n$=-\\sqrt{5} \\sin x$.\n\n又 $\\because-1 \\leqslant \\sin x \\leqslant 1 , \\therefore f(x)$ 的最大值为 $\\sqrt{5}$.", "solution": "null", "level": "高二", "question": "16. 已知 $\\tan \\alpha=-\\frac{1}{3}, \\cos \\beta=\\frac{\\sqrt{5}}{5}, \\alpha, \\beta \\in(0, \\pi)$.\n\n(1)求 $\\tan (\\alpha+\\beta)$ 的值;\n\n(2) 求函数 $f(x)=\\sqrt{2} \\sin (x-\\alpha)+\\cos (x+\\beta)$ 的最大值.", "options": [], "subject": "解析几何", "analysis": "(1) $\\because \\cos \\beta=\\frac{\\sqrt{5}}{5} , \\beta \\in(0, \\pi) , \\therefore \\sin \\beta=\\frac{2 \\sqrt{5}}{5}$ ,\n\n$\\tan \\beta=2$.\n\n$\\therefore \\tan (\\alpha+\\beta)=\\frac{\\tan \\alpha+\\tan \\beta}{1-\\tan \\alpha \\tan \\beta}=1$.\n\n(2) $\\because \\tan \\alpha=-\\frac{1}{3}, \\alpha \\in(0, \\pi), \\therefore \\sin \\alpha=\\frac{\\sqrt{10}}{10}, \\cos \\alpha=-\\frac{3 \\sqrt{10}}{10}$.\n\n$f(x)=\\sqrt{2}(\\sin x \\cos \\alpha-\\cos x \\sin \\alpha)+\\cos x \\cos \\beta-\\sin x \\sin \\beta$\n\n$=-\\frac{3 \\sqrt{5}}{5} \\sin x-\\frac{\\sqrt{5}}{5} \\cos x+\\frac{\\sqrt{5}}{5} \\cos x-\\frac{2 \\sqrt{5}}{5} \\sin x$\n\n$=-\\sqrt{5} \\sin x$.\n\n又 $\\because-1 \\leqslant \\sin x \\leqslant 1 , \\therefore f(x)$ 的最大值为 $\\sqrt{5}$."} {"id": "18624", "image": [], "answer": "将已知数据列成下表:\n\n设甲、乙两种原料分别用 $10 \\mathrm{xg}$ 和 $10 \\mathrm{~g} \\mathrm{~g}$, 总费用为 $\\mathrm{z}$, 那么 $\\left\\{\\begin{array}{l}5 x+7 y \\geqslant 35, \\\\ 10 x+4 y \\geqslant 40, \\\\ x \\geqslant 0, y \\geqslant 0,\\end{array}\\right.$\n\n目标函数为 $z=3 x+2 y$, 作出可行域如图所示:\n\n![](https://cdn.mathpix.com/cropped/2024_04_19_3540a5c85e1968e614e9g-098\\.jpg?height=486&width=508&top_left_y=2047&top_left_x=817)\n\n把 $z=3 x+2 y$ 变形为 $y=-\\frac{3}{2} x+\\frac{z}{2}$, 得到斜率为 $-\\frac{3}{2}$, 在 $y$ 轴上的截距为 $\\frac{Z}{2}$, 随 $z$ 变化的一\n族平行直线.\n\n由图可知, 当直线 $y=-\\frac{3}{2} x+\\frac{Z}{2}$ 经过可行域上的点 $A$ 时, 截距 $\\frac{Z}{2}$ 最小, 即 $z$ 最小.\n\n由 $\\left\\{\\begin{array}{l}10 x+4 y=40, \\\\ 5 x+7 y=35,\\end{array}\\right.$ 得 $A\\left(\\frac{14}{5}, 3\\right)$,\n\n$\\therefore z_{\\min }=3 \\times \\frac{14}{5}+2 \\times 3=", "solution": "null", "level": "高二", "question": "9. 医院用甲、乙两种原料为手术后的病人配营养餐. 甲种原料每 $10 \\mathrm{~g}$ 含 5 单位蛋白质和 10 单位铁质, 售价 3 元; 乙种原料每 $10 \\mathrm{~g}$ 含 7 单位蛋白质和 4 单位铁质, 售价 2 元. 若病人每餐至少需要 35 单位蛋白质和 40 单位铁质. 试问: 应如何使用甲、乙原料, 才能既满足营养,又使费用最省?", "options": [], "subject": "解析几何", "analysis": "将已知数据列成下表:\n\n设甲、乙两种原料分别用 $10 \\mathrm{xg}$ 和 $10 \\mathrm{~g} \\mathrm{~g}$, 总费用为 $\\mathrm{z}$, 那么 $\\left\\{\\begin{array}{l}5 x+7 y \\geqslant 35, \\\\ 10 x+4 y \\geqslant 40, \\\\ x \\geqslant 0, y \\geqslant 0,\\end{array}\\right.$\n\n目标函数为 $z=3 x+2 y$, 作出可行域如图所示:\n\n![](https://cdn.mathpix.com/cropped/2024_04_19_3540a5c85e1968e614e9g-098\\.jpg?height=486&width=508&top_left_y=2047&top_left_x=817)\n\n把 $z=3 x+2 y$ 变形为 $y=-\\frac{3}{2} x+\\frac{z}{2}$, 得到斜率为 $-\\frac{3}{2}$, 在 $y$ 轴上的截距为 $\\frac{Z}{2}$, 随 $z$ 变化的一\n族平行直线.\n\n由图可知, 当直线 $y=-\\frac{3}{2} x+\\frac{Z}{2}$ 经过可行域上的点 $A$ 时, 截距 $\\frac{Z}{2}$ 最小, 即 $z$ 最小.\n\n由 $\\left\\{\\begin{array}{l}10 x+4 y=40, \\\\ 5 x+7 y=35,\\end{array}\\right.$ 得 $A\\left(\\frac{14}{5}, 3\\right)$,\n\n$\\therefore z_{\\min }=3 \\times \\frac{14}{5}+2 \\times 3="} {"id": "18626", "image": ["9133.jpg", "9133.jpg"], "answer": "作出不等式组\n\n$$\n\\left\\{\\begin{array}{l}\n2 x+y-5 \\geqslant 0 \\\\\n3 x-y-5 \\leqslant 0 \\\\\nx-2 y+5 \\geqslant 0\n\\end{array}\\right. \\text { 的可行域如图所示, }\n$$\n\n\n\n由 $\\left\\{\\begin{array}{l}x-2 y+5=0 \\\\ 2 x+y-5=0\\end{array}\\right.$, 得 $A(1,3)$,\n\n由 $\\left\\{\\begin{array}{l}x-2 y+5=0 \\\\ 3 x-y-5=0\\end{array}\\right.$, 得 $B(3,4)$,\n\n由 $\\left\\{\\begin{array}{l}3 x-y-5=0 \\\\ 2 x+y-5=0\\end{array}\\right.$, 得 $C(2,1)$,\n\n设 $z=x^{2}+y^{2}$, 则它表示可行域内的点到原点的距离的平方, 结合图形知, 原点到点 $B$的距离最大, 注意到 $O C \\perp A C, \\therefore$ 原点到点 $C$ 的距离最小.\n\n故 $z_{\\text {max }}=|O B|^{2}=25, z_{\\min }=|O C|^{2}=5$.", "solution": "null", "level": "高二", "question": "10. 已知 $\\left\\{\\begin{array}{l}2 x+y-5 \\geqslant 0 \\\\ 3 x-y-5 \\leqslant 0 \\\\ x-2 y+5 \\geqslant 0\\end{array}\\right.$, 求 $x^{2}+y^{2}$ 的最小值和最大值.", "options": [], "subject": "解析几何", "analysis": "作出不等式组\n\n$$\n\\left\\{\\begin{array}{l}\n2 x+y-5 \\geqslant 0 \\\\\n3 x-y-5 \\leqslant 0 \\\\\nx-2 y+5 \\geqslant 0\n\\end{array}\\right. \\text { 的可行域如图所示, }\n$$\n\n\n\n由 $\\left\\{\\begin{array}{l}x-2 y+5=0 \\\\ 2 x+y-5=0\\end{array}\\right.$, 得 $A(1,3)$,\n\n由 $\\left\\{\\begin{array}{l}x-2 y+5=0 \\\\ 3 x-y-5=0\\end{array}\\right.$, 得 $B(3,4)$,\n\n由 $\\left\\{\\begin{array}{l}3 x-y-5=0 \\\\ 2 x+y-5=0\\end{array}\\right.$, 得 $C(2,1)$,\n\n设 $z=x^{2}+y^{2}$, 则它表示可行域内的点到原点的距离的平方, 结合图形知, 原点到点 $B$的距离最大, 注意到 $O C \\perp A C, \\therefore$ 原点到点 $C$ 的距离最小.\n\n故 $z_{\\text {max }}=|O B|^{2}=25, z_{\\min }=|O C|^{2}=5$."} {"id": "18636", "image": ["9141.jpg", "9141.jpg"], "answer": "如图作出线性约束条件\n\n\n\n$\\left\\{\\begin{array}{l}x+3 y \\geqslant 12 \\\\ x+y \\leqslant 10 \\\\ 3 x+y \\geqslant 12\\end{array}\\right.$ 下的可行域, 包含边界: 其中三条直线中 $x+3 y=12$ 与 $3 x+y=12$ 交于点 $A(3,3)$,\n\n$x+y=10$ 与 $x+3 y=12$ 交于点 $B(9,1)$,\n\n$x+y=10$ 与 $3 x+y=12$ 交于点 $C(1,9)$,\n\n作一组与直线 $2 x-y=0$ 平行的直线 $l: 2 x-y=z$,\n\n即 $y=2 x-z$, 然后平行移动直线 $l$, 直线 $l$ 在 $y$ 轴上的截距为 $-z$, 当 $l$ 经过点 $B$ 时, 一 $z$ 取最小值, 此时 $z$ 最大, 即 $Z_{\\text {max }}=2 \\times 9-1=17$; 当 $l$ 经过点 $C$ 时, $-z$ 取最大值, 此时 $z$最小, 即 $z_{\\text {min }}=2 \\times 1-9=-7$.\n\n$\\therefore z_{\\text {max }}=17, z_{\\text {min }}=-7$.", "solution": "null", "level": "高二", "question": "9. 线性约束条件 $\\left\\{\\begin{array}{l}x+3 y \\geqslant 12 \\\\ x+y \\leqslant 10 \\\\ 3 x+y \\geqslant 12\\end{array}\\right.$ 下, 求 $z=2 x-y$ 的最大值和最小值.", "options": [], "subject": "解析几何", "analysis": "如图作出线性约束条件\n\n\n\n$\\left\\{\\begin{array}{l}x+3 y \\geqslant 12 \\\\ x+y \\leqslant 10 \\\\ 3 x+y \\geqslant 12\\end{array}\\right.$ 下的可行域, 包含边界: 其中三条直线中 $x+3 y=12$ 与 $3 x+y=12$ 交于点 $A(3,3)$,\n\n$x+y=10$ 与 $x+3 y=12$ 交于点 $B(9,1)$,\n\n$x+y=10$ 与 $3 x+y=12$ 交于点 $C(1,9)$,\n\n作一组与直线 $2 x-y=0$ 平行的直线 $l: 2 x-y=z$,\n\n即 $y=2 x-z$, 然后平行移动直线 $l$, 直线 $l$ 在 $y$ 轴上的截距为 $-z$, 当 $l$ 经过点 $B$ 时, 一 $z$ 取最小值, 此时 $z$ 最大, 即 $Z_{\\text {max }}=2 \\times 9-1=17$; 当 $l$ 经过点 $C$ 时, $-z$ 取最大值, 此时 $z$最小, 即 $z_{\\text {min }}=2 \\times 1-9=-7$.\n\n$\\therefore z_{\\text {max }}=17, z_{\\text {min }}=-7$."} {"id": "19870", "image": [], "answer": "(1) 将 $x 2-7 x+12>0$ 化为 $(x+3)(x+4)>0$,解得 $x<-4$ 或 $x>-3$,\n\n所以不等式的解集是 $(-\\infty,-4) \\cup(-3,+\\infty)$;\n\n(2) 将 $-x 2-2 x+3 \\geqslant 0$ 化为 $x 2+2 x-3 \\leqslant 0$,\n\n即 $(x+3) \\quad(x-1) \\leqslant 0$, 解得 $-3 \\leqslant x \\leqslant 1$,所以不等式的解集是 $[-3,1]$;\n\n(3) 将 $x 2-2 x+1<0$ 化为 $(x-1) 2<0$,所以不等式的解集是 $\\varnothing$;\n\n(4) 将 $x 2-2 x+2>0$ 化为 $(x-1) 2+1>0$,所以不等式的解集是 $\\mathrm{R}$.", "solution": "null", "level": "高二", "question": "解下列不等式:\n\n(1) $x^{2}-7 x+12>0$;\n\n(2) $-x^{2}-2 x+3 \\geq 0$;\n(3) $x^{2}-2 x+1<0$;\n(4) $x^{2}-2 x+2>0$.", "options": [], "subject": "解析几何", "analysis": "(1) 将 $x 2-7 x+12>0$ 化为 $(x+3)(x+4)>0$,解得 $x<-4$ 或 $x>-3$,\n\n所以不等式的解集是 $(-\\infty,-4) \\cup(-3,+\\infty)$;\n\n(2) 将 $-x 2-2 x+3 \\geqslant 0$ 化为 $x 2+2 x-3 \\leqslant 0$,\n\n即 $(x+3) \\quad(x-1) \\leqslant 0$, 解得 $-3 \\leqslant x \\leqslant 1$,所以不等式的解集是 $[-3,1]$;\n\n(3) 将 $x 2-2 x+1<0$ 化为 $(x-1) 2<0$,所以不等式的解集是 $\\varnothing$;\n\n(4) 将 $x 2-2 x+2>0$ 化为 $(x-1) 2+1>0$,所以不等式的解集是 $\\mathrm{R}$."} {"id": "19888", "image": ["9272.jpg", "9273.jpg", "9274.jpg", "9275.jpg", "9272.jpg", "9273.jpg", "9274.jpg", "9275.jpg"], "answer": "(1)最大值 17 , 最小值 8 ; (2) 最大值 4 , 最小值 -3 ; (3) $[5,25]$; (4) $\\left[\\frac{2}{3}, 2\\right]$.\n\n 解析 解约束条件中不等式组表示的平面区域如图所示, 阴影部分 $\\triangle A B C$ (含边界) 为可行域.\n\n(1) 由图可知, 当直线 $z=3 x+2 y$, 即直线 $y=-\\frac{3}{2} x+\\frac{1}{2} z$ 过点 $B$ 时, 该直线在 $X$ 轴上的截距最大,此时, $z$ 取最大值 $Z_{\\text {max }}=3 \\times 3+2 \\times 4=17$, 当直线 $y=-\\frac{3}{2} x+\\frac{1}{2} z$ 过点 $A$ 时, 该直线在 $X$ 轴上的截距最小, 此时, $z$ 取最小值 $z_{\\text {min }}=3 \\times 2+2 \\times 1=8$;\n\n\n\n(2) 由图可知, 当直线 $z=3 x-2 y$, 即直线 $y=\\frac{3}{2} x-\\frac{1}{2} z$ 过点 $A$ 时, 该直线在 $X$ 轴上的截距最大,此时, $z$ 取最大值 $Z_{\\text {max }}=3 \\times 2-2 \\times 1=4$, 当直线 $y=\\frac{3}{2} x-\\frac{1}{2} z$ 过点 $C$ 时, 该直线在 $X$ 轴上的截距最小, 此时, $z$ 取最小值 $z_{\\text {min }}=3 \\times 1-2 \\times 3=-3$;\n\n\n\n(3) 设点 $P(x, y)$, 则 $z=x^{2}+y^{2}=|O P|^{2}$ 表示可行域内任一点 $P(x, y)$ 到原点距离的平方.由图可知,其最大值为 $z_{\\text {max }}=|O B|^{2}=3^{2}+4^{2}=25$.\n\n直线 $O A$ 的斜率为 $k_{O A}=\\frac{1}{2}$, 直线 $A C$ 的斜率为 $k_{A C}=-2, k_{O A} \\cdot k_{A C}=-1, \\therefore O A \\perp A C$, 故其最小值为 $z_{\\text {min }}=|O A|^{2}=2^{2}+1^{2}=5$, 因此, $z$ 的取值范围为 $[5,25]$;\n\n\n\n(4) 令点 $P(x, y)$, 则 $z=\\frac{y+1}{x+1}$ 表示可行域内任一点 $P(x, y)$ 与点 $M(-1,-1)$ 连线的斜率 $k_{P M}$.当直线 $P M$ 过点 $A$ 时, 此时直线 $P M$ 的倾斜角取得最小值, 此时, $Z$ 取最小值 $Z_{\\text {min }}=\\frac{1+1}{2+1}=\\frac{2}{3}$, 当直线 $P M$ 过点 $C$ 时, 此时, 直线 $P M$ 的倾斜角最大, 此时, $z$ 取最大值 $z_{\\text {max }}=\\frac{3+1}{1+1}=2$, 因此, $z$ 的取值范围是 $\\left[\\frac{2}{3}, 2\\right]$.\n\n", "solution": "null", "level": "高二", "question": "已知 $X 、 y$ 满足约束条件 $\\left\\{\\begin{array}{l}2 x+y-5 \\geq 0 \\\\ 3 x-y-5 \\leq 0 \\\\ x-2 y+5 \\geq 0\\end{array}\\right.$.\n\n(1) 求目标函数 $z=3 x+2 y$ 的最大值与最小值;\n\n(2) 求目标函数 $z=3 x-2 y$ 的最大值与最小值;\n\n(3) 求目标函数 $z=x^{2}+y^{2}$ 的取值范围;\n\n(4) 求目标函数 $z=\\frac{y+1}{x+1}$ 的取值范围.", "options": [], "subject": "解析几何", "analysis": "(1)最大值 17 , 最小值 8 ; (2) 最大值 4 , 最小值 -3 ; (3) $[5,25]$; (4) $\\left[\\frac{2}{3}, 2\\right]$.\n\n 解析 解约束条件中不等式组表示的平面区域如图所示, 阴影部分 $\\triangle A B C$ (含边界) 为可行域.\n\n(1) 由图可知, 当直线 $z=3 x+2 y$, 即直线 $y=-\\frac{3}{2} x+\\frac{1}{2} z$ 过点 $B$ 时, 该直线在 $X$ 轴上的截距最大,此时, $z$ 取最大值 $Z_{\\text {max }}=3 \\times 3+2 \\times 4=17$, 当直线 $y=-\\frac{3}{2} x+\\frac{1}{2} z$ 过点 $A$ 时, 该直线在 $X$ 轴上的截距最小, 此时, $z$ 取最小值 $z_{\\text {min }}=3 \\times 2+2 \\times 1=8$;\n\n\n\n(2) 由图可知, 当直线 $z=3 x-2 y$, 即直线 $y=\\frac{3}{2} x-\\frac{1}{2} z$ 过点 $A$ 时, 该直线在 $X$ 轴上的截距最大,此时, $z$ 取最大值 $Z_{\\text {max }}=3 \\times 2-2 \\times 1=4$, 当直线 $y=\\frac{3}{2} x-\\frac{1}{2} z$ 过点 $C$ 时, 该直线在 $X$ 轴上的截距最小, 此时, $z$ 取最小值 $z_{\\text {min }}=3 \\times 1-2 \\times 3=-3$;\n\n\n\n(3) 设点 $P(x, y)$, 则 $z=x^{2}+y^{2}=|O P|^{2}$ 表示可行域内任一点 $P(x, y)$ 到原点距离的平方.由图可知,其最大值为 $z_{\\text {max }}=|O B|^{2}=3^{2}+4^{2}=25$.\n\n直线 $O A$ 的斜率为 $k_{O A}=\\frac{1}{2}$, 直线 $A C$ 的斜率为 $k_{A C}=-2, k_{O A} \\cdot k_{A C}=-1, \\therefore O A \\perp A C$, 故其最小值为 $z_{\\text {min }}=|O A|^{2}=2^{2}+1^{2}=5$, 因此, $z$ 的取值范围为 $[5,25]$;\n\n\n\n(4) 令点 $P(x, y)$, 则 $z=\\frac{y+1}{x+1}$ 表示可行域内任一点 $P(x, y)$ 与点 $M(-1,-1)$ 连线的斜率 $k_{P M}$.当直线 $P M$ 过点 $A$ 时, 此时直线 $P M$ 的倾斜角取得最小值, 此时, $Z$ 取最小值 $Z_{\\text {min }}=\\frac{1+1}{2+1}=\\frac{2}{3}$, 当直线 $P M$ 过点 $C$ 时, 此时, 直线 $P M$ 的倾斜角最大, 此时, $z$ 取最大值 $z_{\\text {max }}=\\frac{3+1}{1+1}=2$, 因此, $z$ 的取值范围是 $\\left[\\frac{2}{3}, 2\\right]$.\n\n"} {"id": "19889", "image": [], "answer": "(1) 不存在 $m$ 使不等式 $m x^{2}-2 x-m+1<0$ 恒成立 (2) $\\left\\{x \\left\\lvert\\, \\frac{-1+\\sqrt{7}}{2}\n\n(2)作出直线 $l: x+3 y=0$, 将直线 $l$ 向上平移至 $l_{1}$ 与 $y$ 轴的交点 $M$ 位置时, 此时可行域内 $M$ 点与直线 $l$ 的距离最大, 而直线 $\\mathrm{x}+\\mathrm{y}-3=0$ 与 $\\mathrm{y}$ 轴交于点 $\\mathrm{M}(0,3)$, 所以 $\\mathrm{z}_{\\max }=0+3 \\times 3=9$", "solution": "null", "level": "高二", "question": "已知非负实数 $\\mathrm{x}, \\mathrm{y}$ 满足 $\\left\\{\\begin{array}{l}2 x+y-4 \\leq 0 \\\\ x+y-3 \\leq 0\\end{array}\\right.$ 。\n\n(1)在所给坐标系中画出不等式组所表示的平面区域;\n\n\n\n(2)求 $\\mathrm{z}=\\mathrm{x}+3 \\mathrm{y}$ 的最大值.", "options": [], "subject": "解析几何", "analysis": "(1)由 $\\mathrm{x}, \\mathrm{y}$ 取非负实数, 根据线性约束条件作出可行域, 如图所示阴影部分.\n\n\n\n(2)作出直线 $l: x+3 y=0$, 将直线 $l$ 向上平移至 $l_{1}$ 与 $y$ 轴的交点 $M$ 位置时, 此时可行域内 $M$ 点与直线 $l$ 的距离最大, 而直线 $\\mathrm{x}+\\mathrm{y}-3=0$ 与 $\\mathrm{y}$ 轴交于点 $\\mathrm{M}(0,3)$, 所以 $\\mathrm{z}_{\\max }=0+3 \\times 3=9$."} {"id": "19909", "image": [], "answer": "(1) 64\n,(2) $\\mathrm{x}+\\mathrm{y}$ 的最小值为 18", "solution": "null", "level": "高二", "question": "已知 $x>0, y>0,2 x+8 y-x y=0$.\n\n(1) 求 $x y$ 的最小值;\n\n(2) 求 $x+y$ 的最小值.", "options": [], "subject": "解析几何", "analysis": "(1)利用基本不等式构建不等式即可得出;\n\n(2)由 $2 x+8 y=x y$ ,变形得 $\\frac{8}{x}+\\frac{2}{y}=1$ ,利用“乘 1 法”和基本不等式即可得出.试题解析: (1)由 $2 x+8 y-x y=0$, 得 $\\frac{8}{x}+\\frac{2}{y}=1$, 又 $x>0, y>0$, 故 $1=\\frac{8}{x}+\\frac{2}{y} \\geq 2 \\sqrt{\\frac{8}{x} \\cdot \\frac{2}{y}}=\\frac{8}{\\sqrt{x y}}$,故 $x y \\geq 64$, 当且仅当 $\\left\\{\\begin{array}{c}\\frac{8}{x}+\\frac{2}{y}=1, \\\\ \\frac{8}{x}=\\frac{2}{y}\\end{array}\\right.$ 即 $\\left\\{\\begin{array}{l}x=16 \\\\ y=4\\end{array}\\right.$ 时等号成立, $\\therefore(x y) \\min =64$\n\n(2)由 $22 x+8 y-x y=0$, 得 $\\frac{8}{x}+\\frac{2}{y}=1$, 则 $x+y=\\left(\\frac{8}{x}+\\frac{2}{y}\\right) \\cdot(x+y)$\n$=10+\\frac{2 x}{y}+\\frac{8 y}{x} \\geq 10+2 \\sqrt{\\frac{2 x}{y} \\cdot \\frac{8 y}{x}}=18$. 当且仅当 $\\left\\{\\begin{array}{l}\\frac{8}{x}+\\frac{2}{y}=1, \\\\ \\frac{2 x}{y}=\\frac{8 y}{x}\\end{array}\\right.$ 即 $\\left\\{\\begin{array}{l}x=12 \\\\ y=6\\end{array}\\right.$ 时等号成\n\n立. $\\therefore(x+y) \\min =18$"} {"id": "19476", "image": [], "answer": "", "solution": "null", "level": "高二", "question": "在 $\\triangle A B C$ 中, $\\frac{\\tan A}{\\tan B}=\\frac{a^{2}}{b^{2}}$, 试判断 $\\triangle A B C$ 的形状。", "options": [], "subject": "解析几何", "analysis": "答案:\n$\\frac{\\tan A}{\\tan B}=\\frac{a^{2}}{b^{2}} \\Leftrightarrow \\frac{\\sin A \\cos B}{\\cos A \\sin B}=\\frac{\\sin ^{2} A}{\\sin ^{2} B} \\Leftrightarrow \\frac{\\cos B}{\\cos A}=\\frac{\\sin A}{\\sin B} \\Leftrightarrow$\n\n$\\sin A \\cos A=\\sin B \\cos B \\Leftrightarrow \\sin 2 A=\\sin 2 B \\Leftrightarrow 2 A=2 B$ 或 $2 A+2 B=\\pi 。 \\therefore A=B$ 或 $A+B=\\frac{\\pi}{2}$ 。\n\n$\\therefore \\triangle A B C$ 是等腰三角形或直角三角形。\n\n解析:"} {"id": "19477", "image": [], "answer": "", "solution": "null", "level": "高二", "question": "在 $\\triangle A B C$ 中, $a, b, c$ 分别表示三个内角 $A, B, C$ 的对边, 如果 $\\left(a^{2}+b^{2}\\right) \\sin (A-B)=\\left(a^{2}-b^{2}\\right) \\sin (A+B)$,试判断该三角形的形状。", "options": [], "subject": "解析几何", "analysis": "答案: 方法一: 由已知得 $a^{2}[\\sin (A-B)-\\sin (A+B)]=b^{2}[-\\sin (A+B)-\\sin (A-B)]$ 。\n\n$\\therefore 2 a^{2} \\cos A \\sin B=2 b^{2} \\cos B \\sin A$ 。\n\n由正弦定理,得 $\\sin ^{2} A \\cos A \\sin B=\\sin ^{2} B \\cos B \\sin A$ 。\n\n$\\therefore \\sin A \\sin B(\\sin A \\cos A-\\sin B \\cos B)=0$ 。\n\n$\\therefore \\sin 2 A=\\sin 2 B$ 。由 $0<2 A, 2 B<2 \\pi$,\n\n得 $2 A=2 B$ 或 $2 A=\\pi-2 B$ 。\n\n即 $\\triangle A B C$ 是等腰三角形或直角三角形。\n\n方法二:同方法一可得 $2 a^{2} \\cos A \\sin B=2 b^{2} \\cos B \\sin A$ 。\n\n由正、余弦定理,得\n\n$a^{2} b \\frac{b^{2}+c^{2}-a^{2}}{2 b c}=b^{2} a \\frac{a^{2}+c^{2}-b^{2}}{2 a c}$ 。\n\n$\\therefore a^{2}\\left(b^{2}+c^{2}-a^{2}\\right)=b^{2}\\left(a^{2}+c^{2}-b^{2}\\right)$ 。\n\n即 $\\left(a^{2}-b^{2}\\right)\\left(c^{2}-a^{2}-b^{2}\\right)=0$ 。\n\n$\\therefore a=b$ 或 $c^{2}=a^{2}+b^{2}$ 。\n\n$\\therefore$ 三角形为等腰三角形或直角三角形。"} {"id": "19478", "image": [], "answer": "", "solution": "null", "level": "高二", "question": "在 $\\triangle A B C$ 中, 内角 $A, B, C$ 所对的边分别为 $a, b, c$, 且 $a=b \\cos C+\\frac{1}{2} c$.\n\n(1) 求角 $B$.\n\n(2) 若 $b=3$, 求 $\\triangle A B C$ 面积的最大值.", "options": [], "subject": "解析几何", "analysis": "答案: (1) 由正弦定理得 $\\sin A=\\sin B \\cos C+\\frac{1}{2} \\sin C$.\n\n$\\because A+B+C=\\pi, \\therefore A=\\pi-(B+C)$,\n\n$\\therefore \\sin [\\pi-(B+C)]=\\sin (B+C)=\\sin B \\cos C+\\frac{1}{2} \\sin C$, 即 $\\cos B \\sin C=\\frac{1}{2} \\sin C$.\n\n又 $\\because 00, \\therefore \\cos B=\\frac{1}{2}$.\n\n又 $\\because 00$\n\n$\\therefore$ 要使不等式 $\\frac{x^{2}-8 x+20}{m x^{2}+2(m+1) x+9 m+4}>0$ 对任意实数 $x$ 恒成立\n\n只要 $m x^{2}+2(m+1) x+9 m+4>0$ 对任意实数 $x$ 恒成立\n\n当 $m=0$ 时, $2 x+4>0, x>-2$, 此时原不等式只对于 $x>-2$ 的实数 $x$ 成立\n\n$\\therefore m=0$ 不符合题意\n\n当 $m \\neq 0$ 时, 要使不等式对任意实数 $x$ 恒成立, 必须满足 $\\left\\{\\begin{array}{l}m>0 \\\\ \\Delta<0\\end{array}\\right.$ 解得 $m>\\frac{1}{4}$\n\n综上所述, $m$ 的取值范围是 $m>\\frac{1}{4}$", "solution": "null", "level": "高二", "question": "若不等式 $\\frac{x^{2}-8 x+20}{m x^{2}+2(m+1) x+9 m+4}>0$ 对任意实数 $x$ 恒成立, 求 $m$ 的取值范围", "options": [], "subject": "解析几何", "analysis": "$\\because x^{2}-8 x+20=(x-4)^{2}+4>0$\n\n$\\therefore$ 要使不等式 $\\frac{x^{2}-8 x+20}{m x^{2}+2(m+1) x+9 m+4}>0$ 对任意实数 $x$ 恒成立\n\n只要 $m x^{2}+2(m+1) x+9 m+4>0$ 对任意实数 $x$ 恒成立\n\n当 $m=0$ 时, $2 x+4>0, x>-2$, 此时原不等式只对于 $x>-2$ 的实数 $x$ 成立\n\n$\\therefore m=0$ 不符合题意\n\n当 $m \\neq 0$ 时, 要使不等式对任意实数 $x$ 恒成立, 必须满足 $\\left\\{\\begin{array}{l}m>0 \\\\ \\Delta<0\\end{array}\\right.$ 解得 $m>\\frac{1}{4}$\n\n综上所述, $m$ 的取值范围是 $m>\\frac{1}{4}$"} {"id": "19509", "image": [], "answer": "(I)当 $a=2$ 时, 不等式为 $x^{2}-2 x<0$,\n\n方程 $x^{2}-2 x=0$ 的根为 $x_{1}=0, x_{2}=2$,\n不等式 $x^{2}-2 x<0$ 的解集为 $(0,2)$.\n\n( II ) $f(x)>2 a^{2}$, 即 $x^{2}-a x-2 a^{2}>0$\n\n分方程 $x^{2}-a x-2 a^{2}=0$ 的根为 $x_{1}=-a, x_{2}=2 a$\n\n(1)当 $-a<2 a$, 即 $a>0$ 时, 不等式的解集为 $(-\\infty,-a) \\cup(2 a,+\\infty)$;\n\n(2) 当 $-a=2 a$, 即 $a=0$ 时, 不等式的解集为 $(-\\infty, 0) \\cup(0,+\\infty)$;\n\n(3)当 $-a>2 a$, 即 $a<0$ 时, 不等式的解集为 $(-\\infty, 2 a) \\cup(-a,+\\infty)$.\n\n综上: (1) 当 $a>0$ 时, 不等式的解集为 $(-\\infty,-a) \\cup(2 a,+\\infty)$;\n\n(2) 当 $a=0$ 时, 不等式的解集为 $(-\\infty, 0) \\cup(0,+\\infty)$;\n\n(3) 当 $a<0$ 时, 不等式的解集为 $(-\\infty, 2 a) \\cup(-a,+\\infty)$\n\n19.答案: 证明 (1)由于 $\\frac{b}{a}-\\frac{a}{b}=\\frac{b^{2}-a^{2}}{a b}=\\frac{(b+a)(b-a)}{a b}$\n\n$\\because a0, a b>0$\n\n$\\therefore \\frac{(b+a)(b-a)}{a b}<0$, 故 $\\frac{b}{a}<\\frac{a}{b}$\n\n(2) $\\because \\frac{1}{a}<\\frac{1}{b}$\n\n$\\therefore \\frac{1}{a}-\\frac{1}{b}<0$, 即 $\\frac{b-a}{a b}<0$\n\n而 $a>b$\n\n$\\therefore b-a<0$\n\n$\\therefore a b>0$", "solution": "null", "level": "高二", "question": "已知函数 $f(x)=x^{2}-a x(a \\in \\mathrm{R})$.\n\n(I) 当 $a=2$ 时, 求满足 $f(x)<0$ 的 $x$ 的取值范围;\n\n(II) 解关于 $x$ 的不等式 $f(x)>2 a^{2}$", "options": [], "subject": "解析几何", "analysis": "(I)当 $a=2$ 时, 不等式为 $x^{2}-2 x<0$,\n\n方程 $x^{2}-2 x=0$ 的根为 $x_{1}=0, x_{2}=2$,\n不等式 $x^{2}-2 x<0$ 的解集为 $(0,2)$.\n\n( II ) $f(x)>2 a^{2}$, 即 $x^{2}-a x-2 a^{2}>0$\n\n分方程 $x^{2}-a x-2 a^{2}=0$ 的根为 $x_{1}=-a, x_{2}=2 a$\n\n(1)当 $-a<2 a$, 即 $a>0$ 时, 不等式的解集为 $(-\\infty,-a) \\cup(2 a,+\\infty)$;\n\n(2) 当 $-a=2 a$, 即 $a=0$ 时, 不等式的解集为 $(-\\infty, 0) \\cup(0,+\\infty)$;\n\n(3)当 $-a>2 a$, 即 $a<0$ 时, 不等式的解集为 $(-\\infty, 2 a) \\cup(-a,+\\infty)$.\n\n综上: (1) 当 $a>0$ 时, 不等式的解集为 $(-\\infty,-a) \\cup(2 a,+\\infty)$;\n\n(2) 当 $a=0$ 时, 不等式的解集为 $(-\\infty, 0) \\cup(0,+\\infty)$;\n\n(3) 当 $a<0$ 时, 不等式的解集为 $(-\\infty, 2 a) \\cup(-a,+\\infty)$\n\n19.答案: 证明 (1)由于 $\\frac{b}{a}-\\frac{a}{b}=\\frac{b^{2}-a^{2}}{a b}=\\frac{(b+a)(b-a)}{a b}$\n\n$\\because a0, a b>0$\n\n$\\therefore \\frac{(b+a)(b-a)}{a b}<0$, 故 $\\frac{b}{a}<\\frac{a}{b}$\n\n(2) $\\because \\frac{1}{a}<\\frac{1}{b}$\n\n$\\therefore \\frac{1}{a}-\\frac{1}{b}<0$, 即 $\\frac{b-a}{a b}<0$\n\n而 $a>b$\n\n$\\therefore b-a<0$\n\n$\\therefore a b>0$"} {"id": "18456", "image": [], "answer": "(1) 由 $\\cos B=\\frac{3}{4}$, 得 $\\sin B=\\sqrt{1-\\left(\\frac{3}{4}\\right)}=\\frac{\\sqrt{7}}{4}$.\n\n由 $b^{2}=a c$ 及正弦定理得 $\\sin ^{2} B=\\sin A \\sin C$.\n\n于是 $\\frac{1}{\\tan A}+\\frac{1}{\\tan C}=\\frac{\\cos A}{\\sin A}+\\frac{\\cos C}{\\sin C}$\n\n$=\\frac{\\sin C \\cos A+\\cos C \\sin A}{\\sin A \\sin C}=\\frac{\\sin (A+C)}{\\sin ^{2} B}$\n\n$=\\frac{\\sin B}{\\sin ^{2} B}=\\frac{1}{\\sin B}=\\frac{4 \\sqrt{7}}{7}$.\n\n(2) 由 $\\overrightarrow{B A} \\cdot \\overrightarrow{B C}=\\frac{3}{2}$ 得 $c a \\cdot \\cos B=\\frac{3}{2}$\n\n由 $\\cos B=\\frac{3}{4}$, 可得 $c a=2$, 即 $b^{2}=2$.\n\n由余弦定理: $b^{2}=a^{2}+c^{2}-2 a c \\cdot \\cos B$,\n\n得 $a^{2}+c^{2}=b^{2}+2 a c \\cdot \\cos B=5$,\n\n$\\therefore(a+c)^{2}=a^{2}+c^{2}+2 a c=5+4=9, \\quad \\therefore a+c=3$.", "solution": "null", "level": "高二", "question": "14. $\\triangle A B C$ 中, 内角 $A 、 B 、 C$ 的对边分别为 $a 、 b 、 c$, 已知 $b^{2}=a c$ 且 $\\cos B=\\frac{3}{4}$.\n\n(1)求 $\\frac{1}{\\tan A}+\\frac{1}{\\tan C}$ 的值;\n\n(2) 设 $\\overrightarrow{B A} \\cdot \\overrightarrow{B C}=\\frac{3}{2}$, 求 $\\mathrm{a}+\\mathrm{c}$ 的值.", "options": [], "subject": "度量几何学", "analysis": "(1) 由 $\\cos B=\\frac{3}{4}$, 得 $\\sin B=\\sqrt{1-\\left(\\frac{3}{4}\\right)}=\\frac{\\sqrt{7}}{4}$.\n\n由 $b^{2}=a c$ 及正弦定理得 $\\sin ^{2} B=\\sin A \\sin C$.\n\n于是 $\\frac{1}{\\tan A}+\\frac{1}{\\tan C}=\\frac{\\cos A}{\\sin A}+\\frac{\\cos C}{\\sin C}$\n\n$=\\frac{\\sin C \\cos A+\\cos C \\sin A}{\\sin A \\sin C}=\\frac{\\sin (A+C)}{\\sin ^{2} B}$\n\n$=\\frac{\\sin B}{\\sin ^{2} B}=\\frac{1}{\\sin B}=\\frac{4 \\sqrt{7}}{7}$.\n\n(2) 由 $\\overrightarrow{B A} \\cdot \\overrightarrow{B C}=\\frac{3}{2}$ 得 $c a \\cdot \\cos B=\\frac{3}{2}$\n\n由 $\\cos B=\\frac{3}{4}$, 可得 $c a=2$, 即 $b^{2}=2$.\n\n由余弦定理: $b^{2}=a^{2}+c^{2}-2 a c \\cdot \\cos B$,\n\n得 $a^{2}+c^{2}=b^{2}+2 a c \\cdot \\cos B=5$,\n\n$\\therefore(a+c)^{2}=a^{2}+c^{2}+2 a c=5+4=9, \\quad \\therefore a+c=3$.哈哈"} {"id": "17738", "image": ["8959.jpg"], "answer": "$\\cos \\frac{5}{7} \\pi<\\sin \\frac{\\pi}{12}<\\sin \\frac{5}{12} \\pi<\\tan \\frac{5}{12} \\pi$", "solution": "null", "level": "高二", "question": "14. 把 $\\sin \\frac{\\pi}{12}, \\sin \\frac{5}{12} \\pi, \\cos \\frac{5}{7} \\pi, \\tan \\frac{5}{12} \\pi$ 按由小到大的顺序排列: \\$ \\qquad \\$ .", "options": [], "subject": "度量几何学", "analysis": "解析 : 如图, $\\sin \\frac{\\pi}{12}=M_{1} P_{1}>0, \\sin \\frac{5}{12} \\pi=M_{2} P_{2}>0, \\tan \\frac{5}{12} \\pi=A T>0, \\cos \\frac{5}{7} \\pi=O M_{3}<0$.\n\n而 $00, \\omega>0,|\\varphi|<\\frac{\\pi}{2}\\right)$ 的部分图象如图所示.\n\n\n\n(1) 求函数 $f(x)$ 的解析式;\n\n(2) 设 $\\frac{1}{12} \\pi0, \\quad \\therefore 8 m^{2}=1, \\quad \\therefore m=\\frac{\\sqrt{2}}{4}$", "solution": "null", "level": "高二", "question": "16. 在平面直角坐标系 $x O y$ 中, 角 $\\alpha$ 与角 $\\beta$ 均以 $O x$ 为始边, 它们的终边关于 $y$ 轴对称, 若 $\\sin$ $\\alpha=\\frac{1}{3}$, 求:\n\n(1)sin $\\beta$;\n\n(2) $\\beta$ 的终边过点 $(-1, m)$, 求 $m$ 的值.", "options": [], "subject": "度量几何学", "analysis": "(1) $\\because$ 角 $\\alpha$ 的始边为 $O X$, 且 $\\sin \\alpha=\\frac{1}{3}$,\n\n$\\therefore$ 角 $\\alpha$ 的终边与单位圆交于点 $\\left(x_{0}, \\frac{1}{3}\\right)$.\n\n又角 $\\beta$ 的终边与角 $\\alpha$ 的终边关于 $y$ 轴对称,\n\n$\\therefore$ 角 $\\beta$ 的终边与单位圆的交点为 $\\left(-x_{0}, \\frac{1}{3}\\right)$,\n\n由三角函数的定义可知, $\\sin \\beta=\\frac{1}{3}$.\n\n(2)由于 $\\beta$ 的终边过点 $(-1, m)$,\n\n$\\therefore \\sin \\beta=\\frac{m}{\\sqrt{1+m^{2}}}=\\frac{1}{3}$.\n\n显然 $m>0, \\quad \\therefore 8 m^{2}=1, \\quad \\therefore m=\\frac{\\sqrt{2}}{4}$."} {"id": "17990", "image": ["9034.jpg", "9034.jpg"], "answer": "以 $C$ 为坐标原点,以边 $C B, C A$ 所在的直线分别为 $x$ 轴, $y$ 轴建立平面直角坐标系,如图,\n\n\n\n则 $A(0, m), B(n, 0)$.\n\n(1)证明: $\\because D$ 为斜边 $A B$ 的中点,\n\n$\\therefore D\\left(\\frac{n}{2}, \\frac{m}{2}\\right)$,\n$\\therefore|\\overrightarrow{C D}|=\\frac{1}{2} \\sqrt{n^{2}+m^{2}},|\\overrightarrow{A B}|=\\sqrt{m^{2}+n^{2}}$,\n\n$\\therefore|\\overrightarrow{C D}|=\\frac{1}{2}|\\overrightarrow{A B}|$ ,即 $C D=\\frac{1}{2} A B$.\n\n(2) $\\because E$ 为 $C D$ 的中点, $\\therefore E\\left(\\frac{n}{4}, \\frac{m}{4}\\right)$.\n\n设 $F(x, 0)$ ,则 $\\overrightarrow{A E}=\\left(\\frac{n}{4} - \\frac{3}{4} m\\right) , \\overrightarrow{A F}=(x,-m)$.\n\n$\\because$ 点 $A, E , F$ 共线, $\\therefore$ 存在实数 $\\lambda$ ,使 $\\overrightarrow{A F}=\\lambda \\overrightarrow{A E}$ ,\n\n即 $(x,-m)=\\lambda\\left(\\frac{n}{4},-\\frac{3}{4} m\\right), \\therefore\\left\\{\\begin{array}{l}x=\\frac{n}{4} \\lambda, \\\\ -m=-\\frac{3}{4} m \\lambda .\\end{array}\\right.$\n\n解得 $x=\\frac{n}{3} , \\therefore F\\left(\\frac{n}{3} , 0\\right) . \\therefore|\\overrightarrow{A F}|=\\frac{1}{3} \\sqrt{n^{2}+9 m^{2}}$ ,\n\n即 $A F=\\frac{1}{3} \\sqrt{n^{2}+9 m^{2}}$", "solution": "null", "level": "高二", "question": "10. 已知 Rt $\\triangle A B C$ 中, $\\angle C=90^{\\circ}$, 设 $A C=m, B C=n$.\n\n(1)若 $D$ 为斜边 $A B$ 的中点, 求证: $C D=\\frac{1}{2} A B$;\n\n(2)在(1)的条件下, 若 $E$ 为 $C D$ 的中点, 连接 $A E$ 并延长交 $B C$ 于点 $F$, 求 $A F$ 的长(用 $m, n$表示).", "options": [], "subject": "度量几何学", "analysis": "以 $C$ 为坐标原点,以边 $C B, C A$ 所在的直线分别为 $x$ 轴, $y$ 轴建立平面直角坐标系,如图,\n\n\n\n则 $A(0, m), B(n, 0)$.\n\n(1)证明: $\\because D$ 为斜边 $A B$ 的中点,\n\n$\\therefore D\\left(\\frac{n}{2}, \\frac{m}{2}\\right)$,\n$\\therefore|\\overrightarrow{C D}|=\\frac{1}{2} \\sqrt{n^{2}+m^{2}},|\\overrightarrow{A B}|=\\sqrt{m^{2}+n^{2}}$,\n\n$\\therefore|\\overrightarrow{C D}|=\\frac{1}{2}|\\overrightarrow{A B}|$ ,即 $C D=\\frac{1}{2} A B$.\n\n(2) $\\because E$ 为 $C D$ 的中点, $\\therefore E\\left(\\frac{n}{4}, \\frac{m}{4}\\right)$.\n\n设 $F(x, 0)$ ,则 $\\overrightarrow{A E}=\\left(\\frac{n}{4} - \\frac{3}{4} m\\right) , \\overrightarrow{A F}=(x,-m)$.\n\n$\\because$ 点 $A, E , F$ 共线, $\\therefore$ 存在实数 $\\lambda$ ,使 $\\overrightarrow{A F}=\\lambda \\overrightarrow{A E}$ ,\n\n即 $(x,-m)=\\lambda\\left(\\frac{n}{4},-\\frac{3}{4} m\\right), \\therefore\\left\\{\\begin{array}{l}x=\\frac{n}{4} \\lambda, \\\\ -m=-\\frac{3}{4} m \\lambda .\\end{array}\\right.$\n\n解得 $x=\\frac{n}{3} , \\therefore F\\left(\\frac{n}{3} , 0\\right) . \\therefore|\\overrightarrow{A F}|=\\frac{1}{3} \\sqrt{n^{2}+9 m^{2}}$ ,\n\n即 $A F=\\frac{1}{3} \\sqrt{n^{2}+9 m^{2}}$."} {"id": "17994", "image": ["9037.jpg"], "answer": ":$-\\frac{25}{2}$", "solution": "null", "level": "高二", "question": "14. 设点 $O$ 是 $\\triangle A B C$ 的外心, $A B=13, A C=12$, 则 $\\overrightarrow{B C} \\cdot \\overrightarrow{A O}=$", "options": [], "subject": "度量几何学", "analysis": "设 $\\{\\overrightarrow{A B}, \\overrightarrow{A C}\\}$ 为平面 $A B C$ 内的一组基底. 如图所示,设 $M$ 为 $B C$ 的中点,连接 $O M$ , $A M , O A ,$ 则 $O M \\perp B C$.\n\n\n\n又 $\\because \\overrightarrow{B C}=\\overrightarrow{A C}-\\overrightarrow{A B}, \\overrightarrow{A O}=\\overrightarrow{A M}+\\overrightarrow{M O}=\\frac{1}{2}(\\overrightarrow{A B}+\\overrightarrow{A C})+\\overrightarrow{M O}, \\therefore \\overrightarrow{B C} \\cdot \\overrightarrow{A O}=\\overrightarrow{B C} \\cdot(\\overrightarrow{A M}+\\overrightarrow{M O})=\\overrightarrow{B C} \\cdot \\overrightarrow{A M}$ $=(\\overrightarrow{A C}-\\overrightarrow{A B}) \\cdot \\frac{1}{2}(\\overrightarrow{A B}+\\overrightarrow{A C})=\\frac{1}{2}\\left(\\overrightarrow{A C}^{2}-\\overrightarrow{A B^{2}}\\right)=\\frac{1}{2} \\times\\left(12^{2}-13^{2}\\right)=-\\frac{25}{2}$.\n答案: $-\\frac{25}{2}$"} {"id": "19892", "image": ["9279.jpg"], "answer": "(1) 18 (2) -9", "solution": "null", "level": "高二", "question": "已知实数 $x 、 y$ 满足 $\\left\\{\\begin{array}{c}y \\leq 2 x \\\\ y \\geq-2 x \\\\ x \\leq 3\\end{array}\\right.$\n\n(1)求不等式组表示的平面区域的面积;\n\n(2) 若目标函数为 $z=x-2 y$, 求 $z$ 的最小值.", "options": [], "subject": "度量几何学", "analysis": "画出满足不等式组的可行域如图所示:\n\n\n\n(1)易求点 $A 、 B$ 的坐标为: $A(3,6), B(3,-6)$,\n\n所以三角形 $O A B$ 的面积为:\n\n$S_{\\triangle O A B}=\\frac{1}{2} \\times 12 \\times 3=18$.\n\n(2)目标函数化为: $y=\\frac{1}{2} x-\\frac{z}{2}$, 画直线 $y=\\frac{1}{2} x$ 及其平行线, 当此直线经过 $A$ 时, $-\\frac{z}{2}$ 的值最大, $z$ 的值最小, 易求 $A$ 点坐标为 $(3,6)$, 所以, $z$ 的最小值为 $3-2 \\times 6=-9$."} {"id": "18984", "image": ["9181.jpg", "9181.jpg"], "answer": "(1)在 $\\triangle A B C$ 中, $\\because \\cos B=-\\frac{1}{7}, \\therefore B \\in\\left(\\frac{\\pi}{2}, \\pi\\right), \\therefore \\sin B=\\sqrt{1-\\cos ^{2} B}=\\frac{4 \\sqrt{3}}{7}$.\n\n由正弦定理得 $\\frac{a}{\\sin A}=\\frac{b}{\\sin B} \\Rightarrow \\frac{7}{\\sin A}=\\frac{8}{\\frac{4 \\sqrt{3}}{7}}, \\therefore \\sin A=\\frac{\\sqrt{3}}{2}$.\n\n$\\because B \\in\\left(\\frac{\\pi}{2}, \\pi\\right), \\therefore A \\in\\left(0, \\frac{\\pi}{2}\\right), \\quad \\therefore \\angle A=\\frac{\\pi}{3}$.\n\n(2)在 $\\triangle A B C$ 中, $\\because \\sin C=\\sin (A+B)=\\sin A \\cos B+\\cos A \\sin B=\\frac{\\sqrt{3}}{2} \\times\\left(-\\frac{1}{7}\\right)+\\frac{1}{2} \\times \\frac{4 \\sqrt{3}}{7}=\\frac{3 \\sqrt{3}}{14}$.\n\n如图所示, 在 $\\triangle A B C$ 中, $\\because \\sin C=\\frac{h}{B C}, \\therefore h=B C \\cdot \\sin C=7 \\times \\frac{3 \\sqrt{3}}{14}=\\frac{3 \\sqrt{3}}{2}, \\therefore A C$ 边上的高为\n$\\frac{3 \\sqrt{3}}{2}$.\n\n", "solution": "null", "level": "高二", "question": "21. 在 $\\triangle A B C$ 中, $a=7, b=8, \\cos B=-\\frac{1}{7}$.\n\n(1)求 $\\angle A$; (2)求 $A C$ 边上的高.", "options": [], "subject": "度量几何学", "analysis": "(1)在 $\\triangle A B C$ 中, $\\because \\cos B=-\\frac{1}{7}, \\therefore B \\in\\left(\\frac{\\pi}{2}, \\pi\\right), \\therefore \\sin B=\\sqrt{1-\\cos ^{2} B}=\\frac{4 \\sqrt{3}}{7}$.\n\n由正弦定理得 $\\frac{a}{\\sin A}=\\frac{b}{\\sin B} \\Rightarrow \\frac{7}{\\sin A}=\\frac{8}{\\frac{4 \\sqrt{3}}{7}}, \\therefore \\sin A=\\frac{\\sqrt{3}}{2}$.\n\n$\\because B \\in\\left(\\frac{\\pi}{2}, \\pi\\right), \\therefore A \\in\\left(0, \\frac{\\pi}{2}\\right), \\quad \\therefore \\angle A=\\frac{\\pi}{3}$.\n\n(2)在 $\\triangle A B C$ 中, $\\because \\sin C=\\sin (A+B)=\\sin A \\cos B+\\cos A \\sin B=\\frac{\\sqrt{3}}{2} \\times\\left(-\\frac{1}{7}\\right)+\\frac{1}{2} \\times \\frac{4 \\sqrt{3}}{7}=\\frac{3 \\sqrt{3}}{14}$.\n\n如图所示, 在 $\\triangle A B C$ 中, $\\because \\sin C=\\frac{h}{B C}, \\therefore h=B C \\cdot \\sin C=7 \\times \\frac{3 \\sqrt{3}}{14}=\\frac{3 \\sqrt{3}}{2}, \\therefore A C$ 边上的高为\n$\\frac{3 \\sqrt{3}}{2}$.\n\n"} {"id": "19005", "image": [], "answer": "$6 \\sqrt{3}$", "solution": "null", "level": "高二", "question": "19.【2019 年高考全国 II 卷理数 $】 \\triangle A B C$ 的内角 $A, B, C$ 的对边分别为 $a, b, c$. 若 $b=6, a=2 c, B=\\frac{\\pi}{3}$,则\n\n$\\triangle A B C$ 的面积为 \\$ \\qquad \\$ .", "options": [], "subject": "度量几何学", "analysis": "由余弦定理得 $b^{2}=a^{2}+c^{2}-2 a c \\cos B$, 所以 $(2 c)^{2}+c^{2}-2 \\times 2 c \\times c \\times \\frac{1}{2}=6^{2}$, 即 $c^{2}=12$,解得 $c=2 \\sqrt{3}, c=-2 \\sqrt{3}$ (舍去),\n\n所以 $a=2 c=4 \\sqrt{3}, S_{\\triangle A B C}=\\frac{1}{2} a c \\sin B=\\frac{1}{2} \\times 4 \\sqrt{3} \\times 2 \\sqrt{3} \\times \\frac{\\sqrt{3}}{2}=6 \\sqrt{3}$.\n\n【名师点睛】本题易错点往往是余弦定理应用有误或是开方导致错误. 解答此类问题, 关键是在明确方法的基础上,准确记忆公式,细心计算. 本题首先应用余弦定理,建立关于 $c$ 的方程,应用 $a, c$ 的关系、三角形面积公式计算求解, 本题属于常见题目, 难度不大, 注重了基础知识、基本方法、数学式子的变形及运算求解能力的考查."} {"id": "19007", "image": ["9183.jpg"], "answer": "$\\frac{12 \\sqrt{2}}{5}, \\frac{7 \\sqrt{2}}{10}$", "solution": "null", "level": "高二", "question": "20.【2019 年高考浙江卷】在 $\\triangle A B C$ 中, $\\angle A B C=90^{\\circ}, A B=4, B C=3$, 点 $D$ 在线段 $A C$ 上,若 $\\angle B D C=45^{\\circ}$, 则 $B D=$ \\$ \\qquad \\$ , $\\cos \\angle A B D=$ \\$ \\qquad \\$", "options": [], "subject": "度量几何学", "analysis": "如图, 在 $\\triangle A B D$ 中, 由正弦定理有: $\\frac{A B}{\\sin \\angle A D B}=\\frac{B D}{\\sin \\angle B A C}$, 而 $A B=4, \\angle A D B=\\frac{3 \\pi}{4}$, $A C=\\sqrt{A B^{2}+B C^{2}}=5, \\sin \\angle B A C=\\frac{B C}{A C}=\\frac{3}{5}, \\cos \\angle B A C=\\frac{A B}{A C}=\\frac{4}{5}$, 所以 $B D=\\frac{12 \\sqrt{2}}{5}$.\n\n$\\cos \\angle A B D=\\cos (\\angle B D C-\\angle B A C)=\\cos \\frac{\\pi}{4} \\cos \\angle B A C+\\sin \\frac{\\pi}{4} \\sin \\angle B A C=\\frac{7 \\sqrt{2}}{10}$.\n\n\n\n【名师点睛】本题主要考查解三角形问题, 即正弦定理、三角恒等变换、数形结合思想及函数方程思"} {"id": "19028", "image": [], "answer": "( I ) $\\frac{\\pi}{3}$; (II) 1 .", "solution": "null", "level": "高二", "question": "18. (2019$\\cdot$北京高考模拟 (理) ) 已知在 $\\triangle A B C$ 中, $a^{2}+c^{2}-a c=b^{2}$.\n\n(I) 求角 $B$ 的大小;\n\n(II) 求 $\\cos A+\\cos C$ 的最大值.", "options": [], "subject": "度量几何学", "analysis": "(I) 由余弦定理得 $\\cos B=\\frac{a^{2}+c^{2}-b^{2}}{2 a \\cdot c}=\\frac{a \\cdot c}{2 a \\cdot c}=\\frac{1}{2}$, 因为角 $B$ 为三角形内角 $\\therefore \\angle B=\\frac{\\pi}{3}$\n\n(II) 由(I)可得 $\\angle A+\\angle C=\\pi-\\angle B=\\frac{2 \\pi}{3}$\n\n$\\therefore \\angle A=\\frac{2 \\pi}{3}-\\angle C \\therefore \\cos A+\\cos C=\\cos \\left(\\frac{2 \\pi}{3}-C\\right)+\\cos C=\\cos \\frac{2 \\pi}{3} \\cdot \\cos C+\\sin \\frac{2 \\pi}{3} \\cdot \\sin C+\\cos C$\n\n$=-\\frac{1}{2} \\cdot \\cos C+\\frac{\\sqrt{3}}{2} \\cdot \\sin C+\\cos C=\\frac{\\sqrt{3}}{2} \\cdot \\sin C+\\frac{1}{2} \\cdot \\cos C=\\cos \\frac{\\pi}{6} \\cdot \\sin C+\\sin \\frac{\\pi}{6} \\cdot \\cos C=\\sin \\left(C+\\frac{\\pi}{6}\\right)$\n\n$\\because 0", "solution": "null", "level": "高二", "question": "20. 如图, 点 $P$ 在直径 $A B=1$ 的半圆上移动, 过 $P$ 作圆的切线 $P T$ 且 $P T=1, \\angle P A B=\\alpha$,问 $\\alpha$ 为何值时, 四边形 $A B T P$ 的面积最大?\n\n", "options": [], "subject": "度量几何学", "analysis": "$\\because A B$ 为直径,\n\n$\\therefore \\angle A P B=90^{\\circ}, A B=1, P A=\\cos \\alpha, P B=\\sin \\alpha$.\n\n又 $\\because P T$ 切圆于 $P$ 点, $\\angle T P B=\\angle P A B=\\alpha ,$\n\n$\\therefore S_{\\text {四边形 } A B T P}=S_{\\triangle P A B}+S_{\\triangle T P B}$\n\n$=\\frac{1}{2} P A \\cdot P B+\\frac{1}{2} P T \\cdot P B \\cdot \\sin \\alpha$\n\n$=\\frac{1}{2} \\sin \\alpha \\cos \\alpha+\\frac{1}{2} \\sin ^{2} \\alpha=\\frac{1}{4} \\sin 2 \\alpha+\\frac{1}{4}(1-\\cos 2 \\alpha)$\n\n$=\\frac{1}{4}(\\sin 2 \\alpha-\\cos 2 \\alpha)+\\frac{1}{4}=\\frac{\\sqrt{2}}{4} \\sin \\left(2 \\alpha-\\frac{\\pi}{4}\\right)+\\frac{1}{4}$.\n\n$\\because 0<\\alpha<\\frac{\\pi}{2}, \\quad \\therefore-\\frac{\\pi}{4}<2 \\alpha-\\frac{\\pi}{4}<\\frac{3}{4} \\pi$,\n\n"} {"id": "18139", "image": ["9056.jpg", "9056.jpg"], "answer": "$(1) f(x)=4 \\sin \\left(x-\\frac{\\pi}{3}\\right) \\cos x+\\sqrt{3}$\n\n$=4\\left(\\frac{1}{2} \\sin x-\\frac{\\sqrt{3}}{2} \\cos x\\right) \\cos x+\\sqrt{3}$\n\n$=2 \\sin x \\cos x-2 \\sqrt{3} \\cos ^{2} x+\\sqrt{3}$\n\n$=\\sin 2 x-\\sqrt{3} \\cos 2 x=2 \\sin \\left(2 x-\\frac{\\pi}{3}\\right)$,\n\n$\\therefore$ 函数 $f(x)$ 的最小正周期为 $T=\\pi$.\n\n由 $2 k \\pi-\\frac{\\pi}{2} \\leqslant 2 x-\\frac{\\pi}{3} \\leqslant 2 k \\pi+\\frac{\\pi}{2}$ ,\n\n得 $k \\pi-\\frac{\\pi}{12} \\leqslant x \\leqslant k \\pi+\\frac{5}{12} \\pi(k \\in \\mathbf{Z})$.\n\n$\\therefore f(x)$ 的递增区间为 $\\left[k \\pi-\\frac{\\pi}{12}, k \\pi+\\frac{5 \\pi}{12}\\right](k \\in \\mathbf{Z})$.\n\n(2) $\\because$ 方程 $g(x)=f(x)-m=0$ 同解于 $f(x)=m$ ,\n\n\n\n在直角坐标系中画出函数 $y=f(x)=2 \\sin \\left(2 x-\\frac{\\pi}{3}\\right)$ 在 $\\left[0 , \\frac{\\pi}{2}\\right]$ 上的图象如图,由图象可知,\n\n当且仅当 $m \\in[\\sqrt{3} , 2)$ 时,方程 $f(x)=m$ 有两个不同的解 $x_{1} , x_{2}$ ,且 $x_{1}+x_{2}=2 \\times \\frac{5 \\pi}{12}=\\frac{5 \\pi}{6}$ ,\n\n故 $\\tan \\left(x_{1}+x_{2}\\right)=\\tan \\frac{5 \\pi}{6}=-\\tan \\frac{\\pi}{6}=-\\frac{\\sqrt{3}}{3}$.", "solution": "null", "level": "高二", "question": "21. 已知函数 $f(x)=4 \\sin \\left(x-\\frac{\\pi}{3}\\right) \\cos x+\\sqrt{3}$.\n\n(1)求函数 $f(x)$ 的最小正周期和单调递增区间;\n\n(2)若函数 $g(x)=f(x)-m$ 在区间 $\\left[0, \\frac{\\pi}{2}\\right]$ 上有两个不同的零点 $x_{1}, x_{2}$, 求实数 $m$ 的取值范围,并计算 $\\tan \\left(x_{1}+x_{2}\\right)$ 的值.", "options": [], "subject": "度量几何学", "analysis": "$(1) f(x)=4 \\sin \\left(x-\\frac{\\pi}{3}\\right) \\cos x+\\sqrt{3}$\n\n$=4\\left(\\frac{1}{2} \\sin x-\\frac{\\sqrt{3}}{2} \\cos x\\right) \\cos x+\\sqrt{3}$\n\n$=2 \\sin x \\cos x-2 \\sqrt{3} \\cos ^{2} x+\\sqrt{3}$\n\n$=\\sin 2 x-\\sqrt{3} \\cos 2 x=2 \\sin \\left(2 x-\\frac{\\pi}{3}\\right)$,\n\n$\\therefore$ 函数 $f(x)$ 的最小正周期为 $T=\\pi$.\n\n由 $2 k \\pi-\\frac{\\pi}{2} \\leqslant 2 x-\\frac{\\pi}{3} \\leqslant 2 k \\pi+\\frac{\\pi}{2}$ ,\n\n得 $k \\pi-\\frac{\\pi}{12} \\leqslant x \\leqslant k \\pi+\\frac{5}{12} \\pi(k \\in \\mathbf{Z})$.\n\n$\\therefore f(x)$ 的递增区间为 $\\left[k \\pi-\\frac{\\pi}{12}, k \\pi+\\frac{5 \\pi}{12}\\right](k \\in \\mathbf{Z})$.\n\n(2) $\\because$ 方程 $g(x)=f(x)-m=0$ 同解于 $f(x)=m$ ,\n\n\n\n在直角坐标系中画出函数 $y=f(x)=2 \\sin \\left(2 x-\\frac{\\pi}{3}\\right)$ 在 $\\left[0 , \\frac{\\pi}{2}\\right]$ 上的图象如图,由图象可知,\n\n当且仅当 $m \\in[\\sqrt{3} , 2)$ 时,方程 $f(x)=m$ 有两个不同的解 $x_{1} , x_{2}$ ,且 $x_{1}+x_{2}=2 \\times \\frac{5 \\pi}{12}=\\frac{5 \\pi}{6}$ ,\n\n故 $\\tan \\left(x_{1}+x_{2}\\right)=\\tan \\frac{5 \\pi}{6}=-\\tan \\frac{\\pi}{6}=-\\frac{\\sqrt{3}}{3}$."} {"id": "17734", "image": ["8957.jpg", "8957.jpg"], "answer": "由图可知 $\\sin \\alpha=M P, \\cos \\alpha=O M$,\n\n在 $\\triangle O M P$ 中, $\\because O M+M P>O P$,\n\n$\\therefore \\sin \\alpha+\\cos \\alpha>1$.", "solution": "null", "level": "高二", "question": "10. 已知 $0<\\alpha<\\frac{\\pi}{2}$, 求证: $\\sin \\alpha+\\cos \\alpha>1$.", "options": [], "subject": "算术", "analysis": "由图可知 $\\sin \\alpha=M P, \\cos \\alpha=O M$,\n\n在 $\\triangle O M P$ 中, $\\because O M+M P>O P$,\n\n$\\therefore \\sin \\alpha+\\cos \\alpha>1$."} {"id": "19729", "image": [], "answer": "数列 $\\left\\{a_{n}\\right\\}$ 的通项公式是 $a_{n}=(-1)^{n}+n$,$\\therefore a_{1}=-1+1=0, \\quad a_{2}=1+2=3, \\quad a_{3}=-1+3=2, \\quad a_{4}=1+4=5, \\quad a_{5}=-1+5=4$.", "solution": "null", "level": "高二", "question": "已知数列 $\\left\\{a_{n}\\right\\}$ 的通项公式是 $a_{n}=(-1)^{n}+n$, 写出数列 $\\left\\{a_{n}\\right\\}$ 的前 5 项.", "options": [], "subject": "算术", "analysis": "数列 $\\left\\{a_{n}\\right\\}$ 的通项公式是 $a_{n}=(-1)^{n}+n$,$\\therefore a_{1}=-1+1=0, \\quad a_{2}=1+2=3, \\quad a_{3}=-1+3=2, \\quad a_{4}=1+4=5, \\quad a_{5}=-1+5=4$."} {"id": "19741", "image": [], "answer": "(1) 由题意得 $5 a_{3} a_{1}=\\left(2 a_{2}+2\\right)^{2}$, 即 $d^{2}-3 d-4=0$,\n\n解得: $d=-1$ 或 $d=4$,所以 $a_{n}=11-n, n \\in N^{*}$ 或 $a_{n}=4 n+6, n \\in N^{*}$.", "solution": "null", "level": "高二", "question": "在公差为 $d$ 的等差数列 $\\left\\{a_{n}\\right\\}$ 中, 已知 $a_{1}=10$, 且 $5 a_{3} \\cdot a_{1}=\\left(2 a_{2}+2\\right)^{2}$.\n\n求 $d$ 和 $a_{n}$;", "options": [], "subject": "算术", "analysis": "(1) 由题意得 $5 a_{3} a_{1}=\\left(2 a_{2}+2\\right)^{2}$, 即 $d^{2}-3 d-4=0$,\n\n解得: $d=-1$ 或 $d=4$,所以 $a_{n}=11-n, n \\in N^{*}$ 或 $a_{n}=4 n+6, n \\in N^{*}$."} {"id": "19742", "image": [], "answer": "(1) $\\because$ 等差数列 $\\left\\{a_{n}\\right\\}$ 的前 $n$ 项的和记为 $S_{n} . a_{4}=-12, a_{8}=-4$, $\\therefore\\left\\{\\begin{array}{c}a_{1}+3 d=-12 \\\\ a_{1}+7 d=-4\\end{array}\\right.$, 解得 $a_{1}=-18, d=2, \\therefore$ 数列 $\\left\\{a_{n}\\right\\}$ 的通项公式 $a_{n}=2 n-20$.", "solution": "null", "level": "高二", "question": "已知等差数列 $\\left\\{a_{n}\\right\\}$ 的前 $n$ 项的和记为 $S_{n}$. 如果 $a_{4}=-12, a_{8}=-4$求数列 $\\left\\{a_{n}\\right\\}$ 的通项公式;", "options": [], "subject": "算术", "analysis": "(1) $\\because$ 等差数列 $\\left\\{a_{n}\\right\\}$ 的前 $n$ 项的和记为 $S_{n} . a_{4}=-12, a_{8}=-4$, $\\therefore\\left\\{\\begin{array}{c}a_{1}+3 d=-12 \\\\ a_{1}+7 d=-4\\end{array}\\right.$, 解得 $a_{1}=-18, d=2, \\therefore$ 数列 $\\left\\{a_{n}\\right\\}$ 的通项公式 $a_{n}=2 n-20$."} {"id": "19775", "image": [], "answer": "(1) $\\mathrm{a}_{2}+\\mathrm{a}_{6}=\\mathrm{a}_{3}+\\mathrm{a}_{5}=16$, 又 $\\mathrm{a}_{3} \\cdot \\mathrm{a}_{5}=63$,\n所以 $a_{3}$ 与 $a_{5}$ 是方程 $x^{2}-16 x+63=0$ 的两根,\n\n解得 $\\left\\{\\begin{array}{l}a_{3}=7 \\\\ a_{5}=9\\end{array}\\right.$ 或 $\\left\\{\\begin{array}{l}a_{3}=9 \\\\ a_{5}=7\\end{array}\\right.$,\n\n又该等差数列递减, 所以 $\\left\\{\\begin{array}{l}a_{3}=9 \\\\ a_{5}=7\\end{array}\\right.$,\n\n则公差 $\\mathrm{d}=\\frac{\\mathrm{a}_{5}-\\mathrm{a}_{3}}{2}=-1, \\mathrm{a}_{1}=11$,\n\n所以 $\\mathrm{a}_{\\mathrm{n}}=11+(\\mathrm{n}-1) \\quad(-1)=12-\\mathrm{n}$;\n\n(2)易知,当 $n \\leq 12$ 时 $a_{n} \\geq 0$, 当 $n>12$ 时 $a_{n}<0$,\n\n(1)当 $\\mathrm{n} \\leq 12$ 时,\n\n$\\left|a_{1}\\right|+\\left|a_{2}\\right|+\\left|a_{3}\\right|+\\cdots+\\left|a_{n}\\right|=a_{1}+a_{2}+a_{3}+\\cdots+a_{n}$\n\n$=\\mathrm{S}_{\\mathrm{n}}=\\frac{\\mathrm{n}\\left(\\mathrm{a}_{1}+\\mathrm{a}_{\\mathrm{n}}\\right)}{2} \\mathrm{n}\\left(\\frac{(11+12-\\mathrm{n})}{2}=-\\frac{1}{2} \\mathrm{n}^{2}+\\frac{23}{2} \\mathrm{n} ;\\right.$\n\n(2)当 $n>12$ 时,\n\n$\\left|a_{1}\\right|+\\left|a_{2}\\right|+\\left|a_{3}\\right|+\\cdots+\\left|a_{n}\\right|=\\left(a_{1}+a_{2}+a_{3}+\\cdots+a_{12}\\right)-\\left(a_{13}+a_{14}+\\cdots+a_{n}\\right)$\n\n$=-\\mathrm{S}_{\\mathrm{n}}+2 \\mathrm{~S}_{12}=\\frac{1}{2} \\mathrm{n}^{2}-\\frac{23}{2} \\mathrm{n}+2 \\times 66=\\frac{1}{2} \\mathrm{n}^{2}-\\frac{23}{2} \\mathrm{n}+132 ;$\n\n所以 $\\left|a_{1}\\right|+\\left|a_{2}\\right|+\\left|a_{3}\\right|+\\cdots+\\left|a_{n}\\right|=\\left\\{\\begin{array}{c}-\\frac{1}{2} n^{2}+\\frac{23}{2} n, n \\leq 12 \\\\ \\frac{1}{2} n^{2}-\\frac{23}{2} n+132, n>12\\end{array}\\right.$.", "solution": "null", "level": "高二", "question": "递减的等差数列 $\\left\\{a_{n}\\right\\}$ 的前 $n$ 项和为 $S_{n}$. 若 $a_{3} a_{5}=63, a_{2}+a_{6}=16$,\n\n(1) 求 $\\left\\{\\mathrm{a}_{n}\\right\\}$ 的通项公式\n\n(2) 求 $\\left|a_{1}\\right|+\\left|a_{2}\\right|+\\left|a_{3}\\right|+\\cdots+\\left|a_{n}\\right|$.", "options": [], "subject": "算术", "analysis": "(1) $\\mathrm{a}_{2}+\\mathrm{a}_{6}=\\mathrm{a}_{3}+\\mathrm{a}_{5}=16$, 又 $\\mathrm{a}_{3} \\cdot \\mathrm{a}_{5}=63$,\n所以 $a_{3}$ 与 $a_{5}$ 是方程 $x^{2}-16 x+63=0$ 的两根,\n\n解得 $\\left\\{\\begin{array}{l}a_{3}=7 \\\\ a_{5}=9\\end{array}\\right.$ 或 $\\left\\{\\begin{array}{l}a_{3}=9 \\\\ a_{5}=7\\end{array}\\right.$,\n\n又该等差数列递减, 所以 $\\left\\{\\begin{array}{l}a_{3}=9 \\\\ a_{5}=7\\end{array}\\right.$,\n\n则公差 $\\mathrm{d}=\\frac{\\mathrm{a}_{5}-\\mathrm{a}_{3}}{2}=-1, \\mathrm{a}_{1}=11$,\n\n所以 $\\mathrm{a}_{\\mathrm{n}}=11+(\\mathrm{n}-1) \\quad(-1)=12-\\mathrm{n}$;\n\n(2)易知,当 $n \\leq 12$ 时 $a_{n} \\geq 0$, 当 $n>12$ 时 $a_{n}<0$,\n\n(1)当 $\\mathrm{n} \\leq 12$ 时,\n\n$\\left|a_{1}\\right|+\\left|a_{2}\\right|+\\left|a_{3}\\right|+\\cdots+\\left|a_{n}\\right|=a_{1}+a_{2}+a_{3}+\\cdots+a_{n}$\n\n$=\\mathrm{S}_{\\mathrm{n}}=\\frac{\\mathrm{n}\\left(\\mathrm{a}_{1}+\\mathrm{a}_{\\mathrm{n}}\\right)}{2} \\mathrm{n}\\left(\\frac{(11+12-\\mathrm{n})}{2}=-\\frac{1}{2} \\mathrm{n}^{2}+\\frac{23}{2} \\mathrm{n} ;\\right.$\n\n(2)当 $n>12$ 时,\n\n$\\left|a_{1}\\right|+\\left|a_{2}\\right|+\\left|a_{3}\\right|+\\cdots+\\left|a_{n}\\right|=\\left(a_{1}+a_{2}+a_{3}+\\cdots+a_{12}\\right)-\\left(a_{13}+a_{14}+\\cdots+a_{n}\\right)$\n\n$=-\\mathrm{S}_{\\mathrm{n}}+2 \\mathrm{~S}_{12}=\\frac{1}{2} \\mathrm{n}^{2}-\\frac{23}{2} \\mathrm{n}+2 \\times 66=\\frac{1}{2} \\mathrm{n}^{2}-\\frac{23}{2} \\mathrm{n}+132 ;$\n\n所以 $\\left|a_{1}\\right|+\\left|a_{2}\\right|+\\left|a_{3}\\right|+\\cdots+\\left|a_{n}\\right|=\\left\\{\\begin{array}{c}-\\frac{1}{2} n^{2}+\\frac{23}{2} n, n \\leq 12 \\\\ \\frac{1}{2} n^{2}-\\frac{23}{2} n+132, n>12\\end{array}\\right.$."} {"id": "19786", "image": [], "answer": "(1) 设等差数列 $\\left\\{a_{n}\\right\\}$ 的公差为 $d, \\because a_{3}=2$, 前 3 项和 $S_{3}=\\frac{9}{2}$.\n\n$\\therefore a_{1}+2 d=2, \\quad 3 a_{1}+3 d=\\frac{9}{2}$, 解得 $a_{1}=1, d=\\frac{1}{2} . \\therefore a_{n}=1+\\frac{1}{2}(n-1)=\\frac{n+1}{2}$.\n\n(2) $b_{1}=a_{1}=1, b_{4}=a_{15}=8$, 可得等比数列 $\\left\\{b_{n}\\right\\}$ 的公比 $q$ 满足 $q^{3}=8$, 解得 $q=2$.\n\n$\\therefore\\left\\{b_{n}\\right\\}$ 前 $n$ 项和 $T_{n}=\\frac{2^{n}-1}{2-1}=2^{n}-1$.", "solution": "null", "level": "高二", "question": "已知等差数列 $\\left\\{a_{n}\\right\\}$ 满足 $a_{3}=2$, 前 3 项和为 $S_{3}=\\frac{9}{2}$.\n\n(1)求 $\\left\\{a_{n}\\right\\}$ 的通项公式;\n\n(2)设等比数列 $\\left\\{b_{n}\\right\\}$ 满足 $b_{1}=a_{1}, b_{4}=a_{15}$, 求 $\\left\\{b_{n}\\right\\}$ 的前 $n$ 项和 $T_{n}$.", "options": [], "subject": "算术", "analysis": "(1) 设等差数列 $\\left\\{a_{n}\\right\\}$ 的公差为 $d, \\because a_{3}=2$, 前 3 项和 $S_{3}=\\frac{9}{2}$.\n\n$\\therefore a_{1}+2 d=2, \\quad 3 a_{1}+3 d=\\frac{9}{2}$, 解得 $a_{1}=1, d=\\frac{1}{2} . \\therefore a_{n}=1+\\frac{1}{2}(n-1)=\\frac{n+1}{2}$.\n\n(2) $b_{1}=a_{1}=1, b_{4}=a_{15}=8$, 可得等比数列 $\\left\\{b_{n}\\right\\}$ 的公比 $q$ 满足 $q^{3}=8$, 解得 $q=2$.\n\n$\\therefore\\left\\{b_{n}\\right\\}$ 前 $n$ 项和 $T_{n}=\\frac{2^{n}-1}{2-1}=2^{n}-1$."} {"id": "19858", "image": [], "answer": "因为 $a-\\frac{1}{a}=\\frac{a^{2}-1}{a}=\\frac{(a-1)(a+1)}{a}$, 因为 $a>0$, 所以当 $a>1$ 时, $\\frac{(a-1)(a+1)}{a}>0$, 有 $a>\\frac{1}{a}$;\n\n当 $a=1$ 时, $\\frac{(a-1)(a+1)}{a}=0$, 有 $a=\\frac{1}{a}$;\n\n当 $01$ 时, $a>\\frac{1}{a}$; 当 $a=1$ 时, $a=\\frac{1}{a}$; 当 $00$, 试比较 $a$ 与 $\\frac{1}{\\underline{1}}$ 的大小.", "options": [], "subject": "算术", "analysis": "因为 $a-\\frac{1}{a}=\\frac{a^{2}-1}{a}=\\frac{(a-1)(a+1)}{a}$, 因为 $a>0$, 所以当 $a>1$ 时, $\\frac{(a-1)(a+1)}{a}>0$, 有 $a>\\frac{1}{a}$;\n\n当 $a=1$ 时, $\\frac{(a-1)(a+1)}{a}=0$, 有 $a=\\frac{1}{a}$;\n\n当 $01$ 时, $a>\\frac{1}{a}$; 当 $a=1$ 时, $a=\\frac{1}{a}$; 当 $00$.\n\n$\\because a_{3}+a_{4}=a_{2}+a_{5}=22$, 又 $a_{3} \\cdot a_{4}=117$,\n\n又公差 $d>0, \\therefore a_{3}0$ 且 $\\left|a_{10}\\right|0$,\n\n$\\therefore S_{20}=\\frac{20\\left(a_{1}+a_{20}\\right)}{2}=10\\left(a_{10}+a_{11}\\right)>0$.\n\n又 $\\because d=a_{11}-a_{10}>0$.\n\n$\\therefore S_{n}>0(n \\geqslant 20)$.", "solution": "null", "level": "高二", "question": "12. 已知公差大于零的等差数列 $\\left\\{a_{n}\\right\\}$ 的前 $n$ 项和为 $S_{n}$, 且满足: $a_{3} \\cdot a_{4}=117, a_{2}+a_{5}=22$.\n\n(1)求数列 $\\left\\{a_{n}\\right\\}$ 的通项公式 $a_{n}$;\n\n(2)若数列 $\\left\\{b_{n}\\right\\}$ 是等差数列, 且 $b_{n}=\\frac{S_{n}}{n+c}$, 求非零常数 $c$.", "options": [], "subject": "算术", "analysis": "(1)设等差数列 $\\left\\{a_{n}\\right\\}$ 的公差为 $d$, 且 $d>0$.\n\n$\\because a_{3}+a_{4}=a_{2}+a_{5}=22$, 又 $a_{3} \\cdot a_{4}=117$,\n\n又公差 $d>0, \\therefore a_{3}0$ 且 $\\left|a_{10}\\right|0$,\n\n$\\therefore S_{20}=\\frac{20\\left(a_{1}+a_{20}\\right)}{2}=10\\left(a_{10}+a_{11}\\right)>0$.\n\n又 $\\because d=a_{11}-a_{10}>0$.\n\n$\\therefore S_{n}>0(n \\geqslant 20)$."} {"id": "18722", "image": [], "answer": "(1)设等差数列 $\\left\\{a_{n}\\right\\}$ 的首项为 $a_{1}$, 公差为 $d$.\n\n因为 $a_{3}=7, a_{5}+a_{7}=26$, 所以 $\\left\\{\\begin{array}{l}a_{1}+2 d=7, \\\\ 2 a_{1}+10 d=26,\\end{array}\\right.$\n\n解得 $\\left\\{\\begin{array}{l}a_{1}=3, \\\\ d=2 .\\end{array}\\right.$ 所以 $a_{n}=3+2(n-1)=2 n+1, S_{n}=3 n+\\frac{n(n-1)}{2} \\times 2=n^{2}+2 n$.\n\n所以, $a_{n}=2 n+1, S_{n}=n^{2}+2 n$.\n\n(2) 由(1)知 $a_{n}=2 n+1$,\n\n所以 $b_{n}=\\frac{1}{a_{n}^{2}-1}=\\frac{1}{(2 n+1)^{2}-1}=\\frac{1}{4} \\cdot \\frac{1}{n(n+1)}$\n\n$=\\frac{1}{4} \\cdot\\left(\\frac{1}{n}-\\frac{1}{n+1}\\right)$,\n\n所以 $T_{n}=\\frac{1}{4} \\cdot\\left(1-\\frac{1}{2}+\\frac{1}{2}-\\frac{1}{3}+\\cdots+\\frac{1}{n}-\\frac{1}{n+1}\\right)$\n\n$=\\frac{1}{4} \\cdot\\left(1-\\frac{1}{n+1}\\right)=\\frac{n}{4(n+1)}$,\n\n即数列 $\\left\\{b_{n}\\right\\}$ 的前 $n$ 项和 $T_{n}=\\frac{n}{4(n+1)}$", "solution": "null", "level": "高二", "question": "11. 已知等差数列 $\\left\\{a_{n}\\right\\}$ 满足: $a_{3}=7, a_{5}+a_{7}=26,\\left\\{a_{n}\\right\\}$ 的前 $n$ 项和为 $S_{n}$.\n\n(1)求 $a_{n}$ 及 $S_{n}$;\n\n(2) 令 $b_{n}=\\frac{1}{a_{n}^{2}-1}\\left(n \\in \\mathbf{N}^{*}\\right)$, 求数列 $\\left\\{b_{n}\\right\\}$ 的前 $n$ 项和 $T_{n}$.", "options": [], "subject": "算术", "analysis": "(1)设等差数列 $\\left\\{a_{n}\\right\\}$ 的首项为 $a_{1}$, 公差为 $d$.\n\n因为 $a_{3}=7, a_{5}+a_{7}=26$, 所以 $\\left\\{\\begin{array}{l}a_{1}+2 d=7, \\\\ 2 a_{1}+10 d=26,\\end{array}\\right.$\n\n解得 $\\left\\{\\begin{array}{l}a_{1}=3, \\\\ d=2 .\\end{array}\\right.$ 所以 $a_{n}=3+2(n-1)=2 n+1, S_{n}=3 n+\\frac{n(n-1)}{2} \\times 2=n^{2}+2 n$.\n\n所以, $a_{n}=2 n+1, S_{n}=n^{2}+2 n$.\n\n(2) 由(1)知 $a_{n}=2 n+1$,\n\n所以 $b_{n}=\\frac{1}{a_{n}^{2}-1}=\\frac{1}{(2 n+1)^{2}-1}=\\frac{1}{4} \\cdot \\frac{1}{n(n+1)}$\n\n$=\\frac{1}{4} \\cdot\\left(\\frac{1}{n}-\\frac{1}{n+1}\\right)$,\n\n所以 $T_{n}=\\frac{1}{4} \\cdot\\left(1-\\frac{1}{2}+\\frac{1}{2}-\\frac{1}{3}+\\cdots+\\frac{1}{n}-\\frac{1}{n+1}\\right)$\n\n$=\\frac{1}{4} \\cdot\\left(1-\\frac{1}{n+1}\\right)=\\frac{n}{4(n+1)}$,\n\n即数列 $\\left\\{b_{n}\\right\\}$ 的前 $n$ 项和 $T_{n}=\\frac{n}{4(n+1)}$."} {"id": "18723", "image": [], "answer": "(1) 由已知, 当 $n \\geqslant 1$ 时, $a_{n+1}=\\left[\\left(a_{n+1}-a_{n}\\right)+\\left(a_{n}-a_{n-1}\\right)+\\ldots+\\left(a_{2}-a_{1}\\right)\\right]+a_{1}=3\\left(2^{2 n^{-1}}\\right.$ $\\left.+2^{2 n^{-3}}+\\cdots+2\\right)+2=2^{2\\left(n^{+}+1\\right)^{-1}}$.\n\n而 $a_{1}=2$, 符合上式, 所以数列 $\\left\\{a_{n}\\right\\}$ 的通项公式为 $a_{n}=2^{2 n^{-1}}$.\n\n(2) 由 $b_{n}=n a_{n}=n \\cdot 2^{2 n^{-1}}$ 知\n\n$S_{n}=1 \\cdot 2+2 \\cdot 2^{3}+3 \\cdot 2^{5}+\\cdots+n \\cdot 2^{2 n^{-1}}$,\n\n从而 $2^{2} \\cdot S_{n}=1 \\cdot 2^{3}+2 \\cdot 2^{5}+3 \\cdot 2^{7}+\\cdots+n \\cdot 2^{2 n+1}$.\n\n(1) - (2) 得 $\\left(1-2^{2}\\right) S_{n}=2+2^{3}+2^{5}+\\cdots+2^{2 n^{-1}}-n \\cdot 2^{2 n^{+} 1}$,\n\n即 $S_{n}=\\frac{1}{9}\\left[(3 n-1) 2^{2 n+1}+2\\right]$.", "solution": "null", "level": "高二", "question": "12. 设数列 $\\left\\{a_{n}\\right\\}$ 满足 $a_{1}=2, a_{n+1}-a_{n}=3 \\cdot 2^{2 n^{-}}$.\n\n(1)求数列 $\\left\\{a_{n}\\right\\}$ 的通项公式;\n\n(2) 令 $b_{n}=n a_{n}$, 求数列 $\\left\\{b_{n}\\right\\}$ 的前 $n$ 项和 $S_{n}$.", "options": [], "subject": "算术", "analysis": ") 由已知, 当 $n \\geqslant 1$ 时, $a_{n+1}=\\left[\\left(a_{n+1}-a_{n}\\right)+\\left(a_{n}-a_{n-1}\\right)+\\ldots+\\left(a_{2}-a_{1}\\right)\\right]+a_{1}=3\\left(2^{2 n^{-1}}\\right.$ $\\left.+2^{2 n^{-3}}+\\cdots+2\\right)+2=2^{2\\left(n^{+}+1\\right)^{-1}}$.\n\n而 $a_{1}=2$, 符合上式, 所以数列 $\\left\\{a_{n}\\right\\}$ 的通项公式为 $a_{n}=2^{2 n^{-1}}$.\n\n(2) 由 $b_{n}=n a_{n}=n \\cdot 2^{2 n^{-1}}$ 知\n\n$S_{n}=1 \\cdot 2+2 \\cdot 2^{3}+3 \\cdot 2^{5}+\\cdots+n \\cdot 2^{2 n^{-1}}$,\n\n从而 $2^{2} \\cdot S_{n}=1 \\cdot 2^{3}+2 \\cdot 2^{5}+3 \\cdot 2^{7}+\\cdots+n \\cdot 2^{2 n+1}$.\n\n(1) - (2) 得 $\\left(1-2^{2}\\right) S_{n}=2+2^{3}+2^{5}+\\cdots+2^{2 n^{-1}}-n \\cdot 2^{2 n^{+} 1}$,\n\n即 $S_{n}=\\frac{1}{9}\\left[(3 n-1) 2^{2 n+1}+2\\right]$."} {"id": "18725", "image": [], "answer": "当 $n=1$ 时, $a_{1}=S_{1}$, 所以 $a_{1}=\\frac{1}{4}\\left(a_{1}+1\\right)^{2}$,\n\n解得 $a_{1}=1$.\n\n当 $n \\geqslant 2$ 时, $a_{n}=S_{n}-S_{n-1}=\\frac{1}{4}\\left(a_{n}+1\\right)^{2}-\\frac{1}{4}\\left(a_{n-1}+1\\right)^{2}=\\frac{1}{4}\\left(a_{n}^{2}-a_{n-1}^{2}+2 a_{n}-2 a_{n-1}\\right)$,\n\n$\\therefore a_{n}^{2}-a_{n-1}^{2}-2\\left(a_{n}+a_{n-1}\\right)=0$,\n\n$\\therefore\\left(a_{n}+a_{n-1}\\right)\\left(a_{n}-a_{n-1}-2\\right)=0$.\n\n$\\because a_{n}+a_{n-1}>0, \\quad \\therefore a_{n}-a_{n-1}-2=0$.\n\n$\\therefore a_{n}-a_{n-1}=2$.\n\n$\\therefore\\left\\{a_{n}\\right\\}$ 是首项为 1 ,公差为 2 的等差数列.\n\n$\\therefore a_{n}=1+2(n-1)=2 n-1$.", "solution": "null", "level": "高二", "question": "14.已知正项数列 $\\left\\{a_{n}\\right\\}$ 的前 $n$ 项和 $S_{n}=\\frac{1}{4}\\left(a_{n}+1\\right)^{2}$, 求 $\\left\\{a_{n}\\right\\}$ 的通项公式.", "options": [], "subject": "算术", "analysis": "当 $n=1$ 时, $a_{1}=S_{1}$, 所以 $a_{1}=\\frac{1}{4}\\left(a_{1}+1\\right)^{2}$,\n\n解得 $a_{1}=1$.\n\n当 $n \\geqslant 2$ 时, $a_{n}=S_{n}-S_{n-1}=\\frac{1}{4}\\left(a_{n}+1\\right)^{2}-\\frac{1}{4}\\left(a_{n-1}+1\\right)^{2}=\\frac{1}{4}\\left(a_{n}^{2}-a_{n-1}^{2}+2 a_{n}-2 a_{n-1}\\right)$,\n\n$\\therefore a_{n}^{2}-a_{n-1}^{2}-2\\left(a_{n}+a_{n-1}\\right)=0$,\n\n$\\therefore\\left(a_{n}+a_{n-1}\\right)\\left(a_{n}-a_{n-1}-2\\right)=0$.\n\n$\\because a_{n}+a_{n-1}>0, \\quad \\therefore a_{n}-a_{n-1}-2=0$.\n\n$\\therefore a_{n}-a_{n-1}=2$.\n\n$\\therefore\\left\\{a_{n}\\right\\}$ 是首项为 1 ,公差为 2 的等差数列.\n\n$\\therefore a_{n}=1+2(n-1)=2 n-1$.哈哈"} {"id": "19910", "image": [], "answer": "(1) 2 ; (2) $a=b=\\frac{3}{2}$", "solution": "null", "level": "高二", "question": "已知 $a>0, b>0$, 且 $\\frac{1}{a}+\\frac{2}{b}=2$.\n\n(1) 求 $a b$ 的最小值;\n\n(2) 求 $\\boldsymbol{a}+2 \\boldsymbol{b}$ 的最小值, 并求出 $a 、 b$ 相应的取值.", "options": [], "subject": "算术", "analysis": "(1) 由 $a>0, b>0, \\frac{1}{a}+\\frac{2}{b}=2$ 得: $2 \\sqrt{\\frac{1}{a} \\cdot \\frac{2}{b}} \\leq \\frac{1}{a}+\\frac{2}{b}=2$, 即: $a b \\geq 2$;\n\n等号成立的充要条件是 $\\left\\{\\frac{1}{a}+\\frac{2}{b}=2\\right.$\n\n$$\n\\frac{1}{a}=\\frac{2}{b}\n$$\n\n(2) $a+2 b=\\frac{1}{2}(a+2 b)\\left(\\frac{1}{a}+\\frac{2}{b}\\right)=\\frac{1}{2}\\left(5+\\frac{2 b}{a}+\\frac{2 a}{b}\\right) \\geq \\frac{1}{2}\\left(5+2 \\sqrt{\\frac{2 b}{a} \\cdot \\frac{2 a}{b}}\\right)=\\frac{9}{2}$;\n\n等号成立的充要条件是 $\\left\\{\\begin{array}{l}\\frac{1}{a}+\\frac{2}{b}=2 \\\\ \\frac{2 b}{a}=\\frac{2 a}{b}\\end{array}\\right.$ 且 $a>0, b>0$, 即: $a=b=\\frac{3}{2}$;\n\n$\\therefore \\boldsymbol{a}+2 \\boldsymbol{b}$ 的最小值为 $\\frac{9}{2}$; 此时 $a=b=\\frac{3}{2}$."} {"id": "19914", "image": [], "answer": "(1) 1 ; (2) 16", "solution": "null", "level": "高二", "question": "(1) 已知 $x<\\frac{5}{4}$, 求函数 $y=4 x-2+\\frac{1}{4 x-5}$ 的最大值;\n\n(2) 已知 $\\mathrm{x}>0, \\mathrm{y}>0$ 且 $\\frac{1}{x}+\\frac{9}{y}=1$, 求 $\\mathrm{x}+\\mathrm{y}$ 的最小值.", "options": [], "subject": "算术", "analysis": "$(1) \\mathrm{x}<\\frac{5}{4}, \\therefore 4 \\mathrm{x}-5<0$.\n\n$\\therefore y=4 x-5+\\frac{1}{4 x-5}+3=-\\left[(5-4 x)+\\frac{1}{(5-4 x)}\\right]+3$\n\n$\\leq-2 \\sqrt{(5-4 x) \\frac{1}{(5-4 x)}}+3=1, \\quad y_{\\max }=1$.\n\n(2) $\\because x>0, \\quad y>0$ 且 $\\frac{1}{x}+\\frac{9}{y}=1$,\n\n$\\therefore \\mathrm{x}+\\mathrm{y}=(\\mathrm{x}+\\mathrm{y})\\left(\\frac{1}{x}+\\frac{9}{y}\\right)=10+\\frac{y}{x}+\\frac{9 x}{y} \\geq 10+2 \\sqrt{\\frac{y}{x} \\frac{9 x}{y}}=16$, 即 $\\mathrm{x}+\\mathrm{y}$ 的最小值为 16"} {"id": "19529", "image": [], "answer": "(1)-6,(2)是第 16 项", "solution": "null", "level": "高二", "question": "数列 $\\left\\{a_{n}\\right\\}$ 的通项公式是 $a_{n}=n^{2}-7 n+6$.\n\n(1)这个数列的第 4 项是多少?\n\n(2) 150 是不是这个数列的项? 若是这个数列的项, 它是第几项?", "options": [], "subject": "算术", "analysis": "答案: (1)-6,(2)是第 16 项"} {"id": "19530", "image": [], "answer": "$1 \\because$ 数列 $\\left\\{a_{n}\\right\\}$ 中, $a_{1}=1$, 当 $n \\geq 2$ 时, $a_{n}=\\frac{3 a_{n-1}}{a_{n-1}+3}$,\n\n$\\therefore a_{2}=\\frac{3}{4}, a_{3}=\\frac{3}{5}, a_{4}=\\frac{1}{2}$;\n\n猜想 $a_{n}=\\frac{3}{n+2}$\n\n答案: 解析: 法一: 将 $n=1,2,3,4$ 依次代入递推公式得 $a_{2}=\\frac{2}{3}, a_{3}=\\frac{2}{4}, a_{4}=\\frac{2}{5}$, 又 $a_{1}=\\frac{2}{2}$,\n\n$\\therefore$ 可猜想 $a_{n}=\\frac{2}{n+1}$ 应有 $a_{n+1}=\\frac{2}{n+2}$, 将其代入递推关系式验证成立, $\\therefore a_{n}=\\frac{2}{n+2}$,\n\n法二: $\\because a_{n+1}=\\frac{2 a_{n}}{a_{n}+2}, \\therefore a_{n}+1 a_{n}=2 a_{n}-2 a_{n+1}$.\n\n两边同除以 $2 a_{n}+1 a_{n}$, 得 $\\frac{1}{a_{n+1}}-\\frac{1}{a_{n}}=\\frac{1}{2}$.\n\n$\\therefore \\frac{1}{a_{2}}-\\frac{1}{a_{1}}=\\frac{1}{2}, \\frac{1}{a_{3}}-\\frac{1}{a_{2}}=\\frac{1}{2}, \\cdots, \\frac{1}{a_{n}}-\\frac{1}{a_{n-1}}=\\frac{1}{2}$.\n\n把以上各式累加得 $\\frac{1}{a_{n}}-\\frac{1}{a_{1}}=\\frac{n-1}{2}$.\n\n又 $a_{1}=1, \\therefore a_{n}=\\frac{2}{n+1}$\n故数列 $\\left\\{a_{n}\\right\\}$ 的通项公式为 $a_{n}=\\frac{2}{n+1}\\left(n \\in N^{*}\\right)$", "solution": "null", "level": "高二", "question": "在数列 $\\left\\{a_{n}\\right\\}$ 中, $a_{1}=1$, 当 $n \\geq 2$ 时, $a_{n}=\\frac{3 a_{n-1}}{a_{n-1}+3}$\n\n(1) 求 $a_{2}, a_{3}, a_{4}$;\n\n(2) 猜想数列 $\\left\\{a_{n}\\right\\}$ 的通项 $a_{n}$, 并证明你的结论", "options": [], "subject": "算术", "analysis": "答案: $1 \\because$ 数列 $\\left\\{a_{n}\\right\\}$ 中, $a_{1}=1$, 当 $n \\geq 2$ 时, $a_{n}=\\frac{3 a_{n-1}}{a_{n-1}+3}$,\n\n$\\therefore a_{2}=\\frac{3}{4}, a_{3}=\\frac{3}{5}, a_{4}=\\frac{1}{2}$;\n\n猜想 $a_{n}=\\frac{3}{n+2}$\n\n答案: 解析: 法一: 将 $n=1,2,3,4$ 依次代入递推公式得 $a_{2}=\\frac{2}{3}, a_{3}=\\frac{2}{4}, a_{4}=\\frac{2}{5}$, 又 $a_{1}=\\frac{2}{2}$,\n\n$\\therefore$ 可猜想 $a_{n}=\\frac{2}{n+1}$ 应有 $a_{n+1}=\\frac{2}{n+2}$, 将其代入递推关系式验证成立, $\\therefore a_{n}=\\frac{2}{n+2}$,\n\n法二: $\\because a_{n+1}=\\frac{2 a_{n}}{a_{n}+2}, \\therefore a_{n}+1 a_{n}=2 a_{n}-2 a_{n+1}$.\n\n两边同除以 $2 a_{n}+1 a_{n}$, 得 $\\frac{1}{a_{n+1}}-\\frac{1}{a_{n}}=\\frac{1}{2}$.\n\n$\\therefore \\frac{1}{a_{2}}-\\frac{1}{a_{1}}=\\frac{1}{2}, \\frac{1}{a_{3}}-\\frac{1}{a_{2}}=\\frac{1}{2}, \\cdots, \\frac{1}{a_{n}}-\\frac{1}{a_{n-1}}=\\frac{1}{2}$.\n\n把以上各式累加得 $\\frac{1}{a_{n}}-\\frac{1}{a_{1}}=\\frac{n-1}{2}$.\n\n又 $a_{1}=1, \\therefore a_{n}=\\frac{2}{n+1}$\n故数列 $\\left\\{a_{n}\\right\\}$ 的通项公式为 $a_{n}=\\frac{2}{n+1}\\left(n \\in N^{*}\\right)$"} {"id": "18894", "image": [], "answer": "(1) $a_{n}=1-n(n \\geq 1)$ (2) $a=-\\frac{4}{3}, b=\\frac{11}{6}$.", "solution": "null", "level": "高二", "question": "18. (2020-四川省成都市树德中学高三二诊 (理) )已知数列 $\\left\\{a_{n}\\right\\}$ 的前 $n$ 项和为 $S_{n}$, 且满足 $2 S_{n}=n-n^{2}$ $\\left(n \\in \\mathbf{N}^{*}\\right)$.\n\n(1) 求数列 $\\left\\{a_{n}\\right\\}$ 的通项公式;\n\n(2) 设 $b_{n}=\\left\\{\\begin{array}{ll}2^{a_{n}}, & (n=2 k-1) \\\\ \\frac{2}{\\left(1-a_{n}\\right)\\left(1-a_{n+2}\\right)}, & (n=2 k)\\end{array}\\left(k \\in \\mathbf{N}^{*}\\right)\\right.$, 数列 $\\left\\{b_{n}\\right\\}$ 的前 $n$ 项和 $T_{n}$. 若\n$T_{2 n}=a\\left(\\frac{1}{4}\\right)^{n}-\\frac{1}{2 n+2}+b$ 对 $n \\in \\mathbf{N}^{*}$ 恒成立, 求实数 $a, b$ 的值.", "options": [], "subject": "算术", "analysis": "(1) 由题意, 当 $n=1$ 时, 由 $2 S_{1}=1-1^{2}$, 解得 $a_{1}=0$;\n\n当 $n \\geq 2$ 时, 可得 $2 a_{n}=2 S_{n}-2 S_{n-1}=n-n^{2}-\\left[(n-1)-(n-1)^{2}\\right]=2-2 n$,\n\n即 $a_{n}=1-n,(n \\geq 2)$,\n\n显然当 $n=1$ 时上式也适合, 所以数列的通项公式为 $a_{n}=1-n$.\n\n(2) 由(1)可得 $\\frac{2}{\\left(1-a_{n}\\right)\\left(1-a_{n+2}\\right)}=\\frac{2}{n(n+2)}=\\frac{1}{n}-\\frac{1}{n+2}$,\n\n所以 $T_{2 n}=\\left(b_{1}+b_{3}+\\cdots+b_{2 n-1}\\right)+\\left(b_{2}+b_{4}+\\cdots+b_{2 n}\\right)$\n\n$=\\left(2^{0}+2^{-2}+\\cdots+2^{2-2 n}\\right)+\\left[\\left(\\frac{1}{2}-\\frac{1}{4}\\right)+\\left(\\frac{1}{4}-\\frac{1}{6}\\right)+\\cdots+\\left(\\frac{1}{2 n}-\\frac{1}{2 n+2}\\right)\\right]$\n\n$=\\frac{1-\\left(\\frac{1}{4}\\right)^{n}}{1-\\frac{1}{4}}+\\frac{1}{2}-\\frac{1}{2 n+2}=\\frac{11}{6}-\\frac{4}{3}\\left(\\frac{1}{4}\\right)^{n}-\\frac{1}{2 n+2}$.\n\n因为 $T_{2 n}=a\\left(\\frac{1}{4}\\right)^{n}-\\frac{1}{2 n+2}+b$ 对 $n \\in \\mathbf{N}^{*}$ 恒成立,\n\n所以 $a=-\\frac{4}{3}, b=\\frac{11}{6}$."} {"id": "18915", "image": [], "answer": "(I) $a_{n}=2 n+1$. (II) 8 .", "solution": "null", "level": "高二", "question": "17. 已知等差数列 $\\left\\{a_{n}\\right\\}$ 满足 $a_{6}=6+a_{3}$, 且 $a_{3}-1$ 是 $a_{2}-1, a_{4}$ 的等比中项.\n\n(I) 求数列 $\\left\\{a_{n}\\right\\}$ 的通项公式;\n\n(II) 设 $b_{n}=\\frac{1}{a_{n} a_{n+1}}\\left(n \\in \\mathbf{N}^{*}\\right)$, 数列 $\\left\\{b_{n}\\right\\}$ 的前项和为 $T_{n}$, 求使 $T_{n}<1$ 成立的最大正整数 $n$ 的值", "options": [], "subject": "算术", "analysis": "(I) 设等差数列 $\\left\\{a_{n}\\right\\}$ 的公差为 $d, \\because a_{6}-a_{3}=3 d=6$, 即 $d=2$,\n\n$\\therefore a_{3}-1=a_{1}+3, \\quad a_{2}-1=a_{1}+1, \\quad a_{4}=a_{1}+6$,\n\n$\\because a_{3}-1$ 是 $a_{2}-1, a_{4}$ 的等比中项,\n\n$\\therefore\\left(a_{3}-1\\right)^{2}=\\left(a_{2}-1\\right) \\cdot a_{4}$, 即 $\\left(a_{1}+3\\right)^{2}=\\left(a_{1}+1\\right)\\left(a_{1}+6\\right)$, 解得 $a_{1}=3$.\n\n$\\therefore$ 数列 $\\left\\{a_{n}\\right\\}$ 的通项公式为 $a_{n}=2 n+1$.\n\n(II) 由(I) 得 $b_{n}=\\frac{1}{a_{n} a_{n+1}}=\\frac{1}{(2 n+1)(2 n+3)}=\\frac{1}{2}\\left(\\frac{1}{2 n+1}-\\frac{1}{2 n+3}\\right)$.\n\n$\\therefore T_{n}=b_{1}+b_{2}+\\cdots+b_{n}=\\frac{1}{2}\\left(\\frac{1}{3}-\\frac{1}{5}+\\frac{1}{5}-\\frac{1}{7}+\\cdots+\\frac{1}{2 n+1}-\\frac{1}{2 n+3}\\right)=\\frac{1}{2}\\left(\\frac{1}{3}-\\frac{1}{2 n+3}\\right)=\\frac{n}{3(2 n+3)}$,\n\n由 $\\frac{n}{3(2 n+3)}<\\frac{1}{7}$, 得 $n<9 . \\therefore$ 使得 $T_{n}<1$ 成立的最大正整数 $n$ 的值为 8 ."} {"id": "19195", "image": [], "answer": "(I) 设 $\\left\\{a_{n}\\right\\}$ 的公差为 $d$, 则 $S_{n}=n a_{1}+\\frac{n(n-1)}{2} d$ 。\n\n由已知可得 $\\left\\{\\begin{array}{l}3 a_{1}+3 d=0, \\\\ 5 a_{1}+10 d=-5,\\end{array}\\right.$ 解得 $a_{1}=1, d=-1$.\n\n故 $\\left\\{a_{n}\\right\\}$ 的通项公式为 $a_{n}=2-n$.\n\n(II) 由 (I ) 知 $\\frac{1}{a_{2 n-1} a_{2 n+1}}=\\frac{1}{(3-2 n)(1-2 n)}=\\frac{1}{2}\\left(\\frac{1}{2 n-3}-\\frac{1}{2 n-1}\\right)$,\n\n从而数列 $\\left\\{\\frac{1}{a_{2 n-1} a_{2 n+1}}\\right\\}$ 的前 $n$ 项和为 $\\frac{1}{2}\\left(\\frac{1}{-1}-\\frac{1}{1}+\\frac{1}{1}-\\frac{1}{3}+\\cdots+\\frac{1}{2 n-3}-\\frac{1}{2 n-1}\\right)=\\frac{n}{1-2 n}$", "solution": "null", "level": "高二", "question": "23. (2013新课标1) 已知等差数列 $\\left\\{a_{n}\\right\\}$ 的前 $n$ 项和 $S_{n}$ 满足 $S_{3}=0, S_{5}=-5$.\n\n(I) 求 $\\left\\{a_{n}\\right\\}$ 的通项公式;\n\n(II) 求数列 $\\left\\{\\frac{1}{a_{2 n-1} a_{2 n+1}}\\right\\}$ 的前 $n$ 项和.", "options": [], "subject": "算术", "analysis": "(I) 设 $\\left\\{a_{n}\\right\\}$ 的公差为 $d$, 则 $S_{n}=n a_{1}+\\frac{n(n-1)}{2} d$ 。\n\n由已知可得 $\\left\\{\\begin{array}{l}3 a_{1}+3 d=0, \\\\ 5 a_{1}+10 d=-5,\\end{array}\\right.$ 解得 $a_{1}=1, d=-1$.\n\n故 $\\left\\{a_{n}\\right\\}$ 的通项公式为 $a_{n}=2-n$.\n\n(II) 由 (I ) 知 $\\frac{1}{a_{2 n-1} a_{2 n+1}}=\\frac{1}{(3-2 n)(1-2 n)}=\\frac{1}{2}\\left(\\frac{1}{2 n-3}-\\frac{1}{2 n-1}\\right)$,\n\n从而数列 $\\left\\{\\frac{1}{a_{2 n-1} a_{2 n+1}}\\right\\}$ 的前 $n$ 项和为 $\\frac{1}{2}\\left(\\frac{1}{-1}-\\frac{1}{1}+\\frac{1}{1}-\\frac{1}{3}+\\cdots+\\frac{1}{2 n-3}-\\frac{1}{2 n-1}\\right)=\\frac{n}{1-2 n}$."} {"id": "19196", "image": [], "answer": "(1) 由已知 $s_{n}=2 a_{n}-a_{1}$ 有 $a_{n}=s_{n}-s_{n-1}=2 a_{n}-2 a_{n-1}(n \\geq 2)$,即 $a_{n}=2 a_{n-1}(n \\geq 2)$, 从而 $a_{2}=2 a_{1}, a_{3}=4 a_{1}$.又因为 $a_{1}, a_{2}+1, a_{3}$ 成等差数列, 即 $a_{1}+a_{3}=2\\left(a_{2}+1\\right)$.所以 $a_{1}+4 a_{1}=2\\left(2 a_{1}+1\\right)$, 解得 $a_{1}=2$.\n所以, 数列 $\\left\\{a_{n}\\right\\}$ 是首项为 2 , 公比为 2 的等比数列. 故 $a_{n}=2^{n}$.\n\n(2) 由(1) 得 $\\frac{1}{a_{n}}=\\frac{1}{2^{n}}$.\n\n所以 $T_{n}=\\frac{1}{2}+\\frac{1}{2^{2}}+\\frac{1}{2^{3}}+\\cdots+\\frac{1}{2^{n}}=\\frac{\\frac{1}{2}\\left[1-\\left(\\frac{1}{2}\\right)^{n}\\right]}{1-\\frac{1}{2}}=1-\\frac{1}{2^{n}}$.\n\n由 $\\left|T_{n}-1\\right|<\\frac{1}{1000}$, 得 $\\left|1-\\frac{1}{2^{n}}-1\\right|<\\frac{1}{1000}$, 即 $2^{n}>1000$ 。\n\n因为 $2^{9}=512<1000<1024=2^{10}$,\n\n所以 $n \\geq 10$.\n\n于是, 使 $\\left|T_{n}-1\\right|<\\frac{1}{1000}$ 成立的 $n$ 的最小值为 10", "solution": "null", "level": "高二", "question": "24.设数列 $\\left\\{a_{n}\\right\\}$ 的前 $n$ 项和 $S_{n}=2 a_{n}-a_{1}$, 且 $a_{1}, a_{2}+1, a_{3}$ 成等差数列\n\n(1) 求数列 $\\left\\{a_{n}\\right\\}$ 的通项公式;\n\n(2) 记数列 $\\left\\{\\frac{1}{a_{n}}\\right\\}$ 的前 $n$ 项和 $T_{n}$, 求得 $\\left\\lvert\\, T_{n}-1<\\frac{1}{1000}\\right.$ 成立的 $n$ 的最小值。", "options": [], "subject": "算术", "analysis": "(1) 由已知 $s_{n}=2 a_{n}-a_{1}$ 有 $a_{n}=s_{n}-s_{n-1}=2 a_{n}-2 a_{n-1}(n \\geq 2)$,即 $a_{n}=2 a_{n-1}(n \\geq 2)$, 从而 $a_{2}=2 a_{1}, a_{3}=4 a_{1}$.又因为 $a_{1}, a_{2}+1, a_{3}$ 成等差数列, 即 $a_{1}+a_{3}=2\\left(a_{2}+1\\right)$.所以 $a_{1}+4 a_{1}=2\\left(2 a_{1}+1\\right)$, 解得 $a_{1}=2$.\n所以, 数列 $\\left\\{a_{n}\\right\\}$ 是首项为 2 , 公比为 2 的等比数列. 故 $a_{n}=2^{n}$.\n\n(2) 由(1) 得 $\\frac{1}{a_{n}}=\\frac{1}{2^{n}}$.\n\n所以 $T_{n}=\\frac{1}{2}+\\frac{1}{2^{2}}+\\frac{1}{2^{3}}+\\cdots+\\frac{1}{2^{n}}=\\frac{\\frac{1}{2}\\left[1-\\left(\\frac{1}{2}\\right)^{n}\\right]}{1-\\frac{1}{2}}=1-\\frac{1}{2^{n}}$.\n\n由 $\\left|T_{n}-1\\right|<\\frac{1}{1000}$, 得 $\\left|1-\\frac{1}{2^{n}}-1\\right|<\\frac{1}{1000}$, 即 $2^{n}>1000$ 。\n\n因为 $2^{9}=512<1000<1024=2^{10}$,\n\n所以 $n \\geq 10$.\n\n于是, 使 $\\left|T_{n}-1\\right|<\\frac{1}{1000}$ 成立的 $n$ 的最小值为 10"} {"id": "17739", "image": ["8960.jpg", "8960.jpg"], "answer": "由二次根式的被开方数为非负数, 得 $\\left\\{\\begin{array}{l}\\sin x \\geqslant 0, \\\\ \\tan x \\leqslant 1\\end{array}\\right.$ 如图所示, 其中 $\\sin x \\geqslant 0$ 时, 角 $x$的终边落在图中横线阴影部分内; $\\tan x \\leqslant 1$ 时, 角 $x$ 的终边落在图中坚线阴影部分内.\n\n$\\therefore$ 满足 $\\left\\{\\begin{array}{l}\\sin x \\geqslant 0, \\\\ \\tan x \\leqslant 1\\end{array}\\right.$ 的角 $x$ 的终边落在图中双重阴影部分内.\n$\\therefore$ 该函数的定义域为 $\\left\\{x \\left\\lvert\\, 2 k \\pi \\leqslant x \\leqslant 2 k \\pi+\\frac{\\pi}{4}\\right., k \\in \\mathbf{Z}\\right\\}$\n\n$\\cup\\left\\{x \\left\\lvert\\, 2 k \\pi+\\frac{\\pi}{2} $\\tan x \\leqslant 1$ 时, 角 $x$ 的终边落在图中坚线阴影部分内.\n\n$\\therefore$ 满足 $\\left\\{\\begin{array}{l}\\sin x \\geqslant 0, \\\\ \\tan x \\leqslant 1\\end{array}\\right.$ 的角 $x$ 的终边落在图中双重阴影部分内.\n$\\therefore$ 该函数的定义域为 $\\left\\{x \\left\\lvert\\, 2 k \\pi \\leqslant x \\leqslant 2 k \\pi+\\frac{\\pi}{4}\\right., k \\in \\mathbf{Z}\\right\\}$\n\n$\\cup\\left\\{x \\left\\lvert\\, 2 k \\pi+\\frac{\\pi}{2}0, \\omega>0,-\\frac{\\pi}{2}<\\varphi<\\frac{\\pi}{2}\\right)$ 一个周期的图象如图所示.\n\n\n\n(1)求函数 $f(x)$ 的最小正周期 $T$ 及最大值、最小值;\n\n(2)求函数 $f(x)$ 的解析式及单调递增区间.", "options": [], "subject": "代数", "analysis": "(1)由题图知 $\\frac{1}{4} T=\\frac{\\pi}{12}-\\left(-\\frac{\\pi}{6}\\right)=\\frac{\\pi}{4} , \\therefore T=\\pi$ ,最大值为 1 ,最小值为 -1 .\n\n(2)由(1)知 $\\omega=\\frac{2 \\pi}{T}=2$.又 $2 \\times\\left(-\\frac{\\pi}{6}\\right)+\\varphi=2 k \\pi , k \\in \\mathbf{Z}$ ,解得 $\\varphi=2 k \\pi+\\frac{\\pi}{3} , k \\in \\mathbf{Z}$ ,又 $-\\frac{\\pi}{2}<\\varphi<\\frac{\\pi}{2}$ , $\\therefore \\varphi=\\frac{\\pi}{3} , A=1$. 则 $f(x)=\\sin \\left(2 x+\\frac{\\pi}{3}\\right)$, 由图知 $f(x)$ 的单调递增区间是 $\\left[k \\pi-\\frac{5 \\pi}{12}, k \\pi+\\frac{\\pi}{12}\\right](k \\in \\mathbf{Z})$."} {"id": "19707", "image": [], "answer": "(I) 解: 设等比数列 $\\left\\{\\mathrm{a}_{n}\\right\\}$ 的公比为 $\\mathrm{q}$, 依题意 $\\mathrm{q}>0$. $\\because \\mathrm{a}_{2}=8, \\quad \\mathrm{a}_{3}+\\mathrm{a}_{4}=48, \\quad \\therefore \\mathrm{a}_{1} \\mathrm{q}=8, \\quad \\mathrm{a}_{1} \\mathrm{q}^{2}+\\mathrm{a}_{1} \\mathrm{q}^{3}=48$.两式相除得 $\\mathrm{q}^{2}+\\mathrm{q}-6=0$, 解得 $\\mathrm{q}=2$, 舍去 $\\mathrm{q}=-3$. $: \\mathrm{a}_{1}=\\frac{\\mathrm{a}_{2}}{\\mathrm{q}}=4$.:数列 $\\left\\{a_{n}\\right\\}$ 的通项公式为 $\\mathrm{a}_{\\mathrm{n}}=\\mathrm{a}_{1} \\cdot \\mathrm{q}^{\\mathrm{n}-1}=2^{\\mathrm{n}+1}$.(II) 证明: 由 (I ) 得 $b_{n}=\\log _{4} a_{n}=\\frac{n+1}{2} \\cdot: b_{n+1}-b_{n}=\\frac{n+2}{2}-\\frac{n+1}{2}=\\frac{1}{2}$,$\\therefore$ 数列 $\\left\\{\\mathrm{b}_{\\mathrm{n}}\\right\\}$ 是首项为 1 , 公差为 $\\mathrm{d}=\\frac{1}{2}$ 的等差数列. $\\therefore \\mathrm{S}_{\\mathrm{n}}=\\mathrm{nb}_{1}+\\frac{\\mathrm{n}(\\mathrm{n}-1)}{2} \\mathrm{~d}=\\frac{\\mathrm{n}^{2}+3 \\mathrm{n}}{4}$.", "solution": "null", "level": "高二", "question": "已知等比数列 $\\left\\{a_{n}\\right\\}$ 的各项均为正数, $a_{2}=8, a_{3}+a_{4}=48$.\n\n(I) 求数列 $\\left\\{a_{n}\\right\\}$ 的通项公式;\n\n(II) 设 $\\mathrm{b}_{\\mathrm{n}}=\\log _{4} \\mathrm{a}_{\\mathrm{n}}$. 证明: $\\left\\{\\mathrm{b}_{\\mathrm{n}}\\right\\}$ 为等差数列, 并求 $\\left\\{\\mathrm{b}_{\\mathrm{n}}\\right\\}$ 的前 $\\mathrm{n}$ 项和 $\\mathrm{S}_{\\mathrm{n}}$", "options": [], "subject": "代数", "analysis": "(I) 解: 设等比数列 $\\left\\{\\mathrm{a}_{n}\\right\\}$ 的公比为 $\\mathrm{q}$, 依题意 $\\mathrm{q}>0$. $\\because \\mathrm{a}_{2}=8, \\quad \\mathrm{a}_{3}+\\mathrm{a}_{4}=48, \\quad \\therefore \\mathrm{a}_{1} \\mathrm{q}=8, \\quad \\mathrm{a}_{1} \\mathrm{q}^{2}+\\mathrm{a}_{1} \\mathrm{q}^{3}=48$.两式相除得 $\\mathrm{q}^{2}+\\mathrm{q}-6=0$, 解得 $\\mathrm{q}=2$, 舍去 $\\mathrm{q}=-3$. $: \\mathrm{a}_{1}=\\frac{\\mathrm{a}_{2}}{\\mathrm{q}}=4$.:数列 $\\left\\{a_{n}\\right\\}$ 的通项公式为 $\\mathrm{a}_{\\mathrm{n}}=\\mathrm{a}_{1} \\cdot \\mathrm{q}^{\\mathrm{n}-1}=2^{\\mathrm{n}+1}$.(II) 证明: 由 (I ) 得 $b_{n}=\\log _{4} a_{n}=\\frac{n+1}{2} \\cdot: b_{n+1}-b_{n}=\\frac{n+2}{2}-\\frac{n+1}{2}=\\frac{1}{2}$,$\\therefore$ 数列 $\\left\\{\\mathrm{b}_{\\mathrm{n}}\\right\\}$ 是首项为 1 , 公差为 $\\mathrm{d}=\\frac{1}{2}$ 的等差数列. $\\therefore \\mathrm{S}_{\\mathrm{n}}=\\mathrm{nb}_{1}+\\frac{\\mathrm{n}(\\mathrm{n}-1)}{2} \\mathrm{~d}=\\frac{\\mathrm{n}^{2}+3 \\mathrm{n}}{4}$."} {"id": "19730", "image": [], "answer": "(1) 符号可通过 $(-1)^{n}$ 表示, 后面的数的绝对值总比前面的数的绝对值大 6 ,故通项公式为 $a_{n}=(-1)^{n} \\bullet(6 n-5)$.(2) 将数列变形为 $\\frac{8}{9}(1-0,.1), \\frac{8}{9}(1-0,.01), \\frac{8}{9}(1-0,.001), \\cdots$, $\\therefore a_{n}=\\frac{8}{9}\\left(1-\\frac{1}{10^{n}}\\right)$.(3)各项的分母分别为 $2^{1}, 2^{2}, 2^{3}, 2^{4}, \\cdots$, 易看出第 $2,3,4$ 项的分子分别比分母少 3. 因此把第 1 项变为 $\\frac{2-3}{2}$.原数列可化为 $-\\frac{2^{1}-3}{2^{1}}, \\frac{2^{2}-3}{2^{2}},-\\frac{2^{3}-3}{2^{3}}, \\frac{2^{4}-3}{2^{4}}, \\cdots, \\therefore a_{n}=(-1)^{n} \\cdot \\frac{2^{n}-3}{2^{n}}$.", "solution": "null", "level": "高二", "question": "根据数列的前几项, 写出下列各数列的一个通项公式.\n\n(1) $-1,7,-13,19, \\cdots$\n\n(2) $0,.8,0,.88,0,.888$,\n\n(3) $-\\frac{1}{2}, \\frac{1}{4},-\\frac{5}{8}, \\frac{13}{16},-\\frac{29}{32}, \\frac{61}{64}, \\ldots$", "options": [], "subject": "代数", "analysis": "(1) 符号可通过 $(-1)^{n}$ 表示, 后面的数的绝对值总比前面的数的绝对值大 6 ,故通项公式为 $a_{n}=(-1)^{n} \\bullet(6 n-5)$.(2) 将数列变形为 $\\frac{8}{9}(1-0,.1), \\frac{8}{9}(1-0,.01), \\frac{8}{9}(1-0,.001), \\cdots$, $\\therefore a_{n}=\\frac{8}{9}\\left(1-\\frac{1}{10^{n}}\\right)$.(3)各项的分母分别为 $2^{1}, 2^{2}, 2^{3}, 2^{4}, \\cdots$, 易看出第 $2,3,4$ 项的分子分别比分母少 3. 因此把第 1 项变为 $\\frac{2-3}{2}$.原数列可化为 $-\\frac{2^{1}-3}{2^{1}}, \\frac{2^{2}-3}{2^{2}},-\\frac{2^{3}-3}{2^{3}}, \\frac{2^{4}-3}{2^{4}}, \\cdots, \\therefore a_{n}=(-1)^{n} \\cdot \\frac{2^{n}-3}{2^{n}}$."} {"id": "18022", "image": [], "answer": "(1)证明: $\\because \\overrightarrow{B D}=\\overrightarrow{B C}+\\overrightarrow{C D}=4 \\boldsymbol{e}_{1}+\\boldsymbol{e}_{2}+8 \\boldsymbol{e}_{1}-9 \\boldsymbol{e}_{2}$\n\n$=12 \\boldsymbol{e}_{1}-8 \\boldsymbol{e}_{2}=4\\left(3 \\boldsymbol{e}_{1}-2 \\boldsymbol{e}_{2}\\right)=4 \\overrightarrow{A B}, \\therefore \\overrightarrow{A B}$ 与 $\\overrightarrow{B D}$ 共线.\n\n$\\because \\overrightarrow{A B}$ 与 $\\overrightarrow{B D}$ 有公共点 $B, \\therefore A, B, D$ 三点共线.\n\n(2) $\\because 2 \\lambda e_{1}+e_{2}$ 与 $e_{1}+\\lambda e_{2}$ 共线,\n\n$\\therefore$ 存在实数 $\\mu$ ,使 $2 \\lambda \\boldsymbol{e}_{1}+\\boldsymbol{e}_{2}=\\mu\\left(\\boldsymbol{e}_{1}+\\lambda \\boldsymbol{e}_{2}\\right)$.\n\n$\\because \\boldsymbol{e}_{1}, \\boldsymbol{e}_{2}$ 不共线, $\\therefore\\left\\{\\begin{array}{l}2 \\lambda=\\mu, \\\\ 1=\\lambda \\mu,\\end{array} \\quad \\therefore \\lambda= \\pm \\frac{\\sqrt{2}}{2}\\right.$.\n\n(3)假设 $e_{1}+\\lambda e_{2}$ 与 $\\lambda e_{1}+e_{2}$ 共线,则存在实数 $\\mu$ ,使 $e_{1}+\\lambda e_{2}=\\mu\\left(\\lambda e_{1}+e_{2}\\right)$.\n\n$\\because \\boldsymbol{e}_{1}, \\boldsymbol{e}_{2}$ 不共线, $\\therefore\\left\\{\\begin{array}{l}1=\\lambda \\mu, \\\\ \\lambda=\\mu,\\end{array} \\quad \\therefore \\lambda= \\pm 1\\right.$.\n\n$\\therefore$ 当 $\\lambda \\neq \\pm 1$ 时, $\\boldsymbol{e}_{1}+\\lambda \\boldsymbol{e}_{2}$ 与 $\\lambda \\boldsymbol{e}_{1}+\\boldsymbol{e}_{2}$ 不共线.", "solution": "null", "level": "高二", "question": "14. 设 $\\boldsymbol{e}_{1}, \\boldsymbol{e}_{2}$ 是两个不共线的向量, 如果 $\\overrightarrow{A B}=3 \\boldsymbol{e}_{1}-2 \\boldsymbol{e}_{2}, \\overrightarrow{B C}=4 \\boldsymbol{e}_{1}+\\boldsymbol{e}_{2}, \\overrightarrow{C D}=8 \\boldsymbol{e}_{1}-9 \\boldsymbol{e}_{2}$.\n\n(1)求证: $A, B, D$ 三点共线;\n\n(2)试确定 $\\lambda$ 的值, 使 $2 \\lambda \\boldsymbol{e}_{1}+\\boldsymbol{e}_{2}$ 和 $\\boldsymbol{e}_{1}+\\lambda \\boldsymbol{e}_{2}$ 共线;\n\n(3)若 $\\boldsymbol{e}_{1}+\\lambda \\boldsymbol{e}_{2}$ 与 $\\lambda \\boldsymbol{e}_{1}+\\boldsymbol{e}_{2}$ 不共线, 试求 $\\lambda$ 的取值范围.", "options": [], "subject": "代数", "analysis": "(1)证明: $\\because \\overrightarrow{B D}=\\overrightarrow{B C}+\\overrightarrow{C D}=4 \\boldsymbol{e}_{1}+\\boldsymbol{e}_{2}+8 \\boldsymbol{e}_{1}-9 \\boldsymbol{e}_{2}$\n\n$=12 \\boldsymbol{e}_{1}-8 \\boldsymbol{e}_{2}=4\\left(3 \\boldsymbol{e}_{1}-2 \\boldsymbol{e}_{2}\\right)=4 \\overrightarrow{A B}, \\therefore \\overrightarrow{A B}$ 与 $\\overrightarrow{B D}$ 共线.\n\n$\\because \\overrightarrow{A B}$ 与 $\\overrightarrow{B D}$ 有公共点 $B, \\therefore A, B, D$ 三点共线.\n\n(2) $\\because 2 \\lambda e_{1}+e_{2}$ 与 $e_{1}+\\lambda e_{2}$ 共线,\n\n$\\therefore$ 存在实数 $\\mu$ ,使 $2 \\lambda \\boldsymbol{e}_{1}+\\boldsymbol{e}_{2}=\\mu\\left(\\boldsymbol{e}_{1}+\\lambda \\boldsymbol{e}_{2}\\right)$.\n\n$\\because \\boldsymbol{e}_{1}, \\boldsymbol{e}_{2}$ 不共线, $\\therefore\\left\\{\\begin{array}{l}2 \\lambda=\\mu, \\\\ 1=\\lambda \\mu,\\end{array} \\quad \\therefore \\lambda= \\pm \\frac{\\sqrt{2}}{2}\\right.$.\n\n(3)假设 $e_{1}+\\lambda e_{2}$ 与 $\\lambda e_{1}+e_{2}$ 共线,则存在实数 $\\mu$ ,使 $e_{1}+\\lambda e_{2}=\\mu\\left(\\lambda e_{1}+e_{2}\\right)$.\n\n$\\because \\boldsymbol{e}_{1}, \\boldsymbol{e}_{2}$ 不共线, $\\therefore\\left\\{\\begin{array}{l}1=\\lambda \\mu, \\\\ \\lambda=\\mu,\\end{array} \\quad \\therefore \\lambda= \\pm 1\\right.$.\n\n$\\therefore$ 当 $\\lambda \\neq \\pm 1$ 时, $\\boldsymbol{e}_{1}+\\lambda \\boldsymbol{e}_{2}$ 与 $\\lambda \\boldsymbol{e}_{1}+\\boldsymbol{e}_{2}$ 不共线."} {"id": "19764", "image": [], "answer": "(1) 由等差数列的性质可得 $\\left\\{\\begin{array}{l}3 a_{1}+3 d=0 \\\\ 5 a_{1}+\\frac{5 \\times 4 d}{2}=-5\\end{array}\\right.$,\n\n解得 $\\mathrm{a}_{1}=1, \\mathrm{~d}=-1$,\n\n则 $\\left\\{a_{n}\\right\\}$ 的通项公式 $a_{n}=1-(n-1)=2-n$;\n\n(2) $\\because\\left\\{\\mathrm{a}_{\\mathrm{a}}\\right\\}$ 为等差数列,\n\n$\\therefore a_{1}+a_{4}+a_{7}+\\cdots+a_{3 n+1}$ 以 1 为首项, 以 -3 为公差的等差数列,\n\n$\\therefore \\mathrm{a}_{1}+\\mathrm{a}_{4}+\\mathrm{a}_{7}+\\cdots+\\mathrm{a}_{3 \\mathrm{n}+1}=\\mathrm{n}+1+\\frac{(\\mathrm{n}+1)(\\mathrm{n}+1-1) \\times(-3)}{2}=\\frac{(\\mathrm{n}+1)(2-3 \\mathrm{n})}{2}$", "solution": "null", "level": "高二", "question": "已知等差数列 $\\left\\{a_{n}\\right\\}$ 的前 $n$ 项和 $S_{n}$ 满足 $S_{3}=0, S_{5}=-5$.\n\n(1) 求 $\\left\\{a_{n}\\right\\}$ 的通项公式;\n\n(2) 求 $a_{1}+a_{4}+a_{7}+\\cdots+a_{3 n+1}$.", "options": [], "subject": "代数", "analysis": "(1) 由等差数列的性质可得 $\\left\\{\\begin{array}{l}3 a_{1}+3 d=0 \\\\ 5 a_{1}+\\frac{5 \\times 4 d}{2}=-5\\end{array}\\right.$,\n\n解得 $\\mathrm{a}_{1}=1, \\mathrm{~d}=-1$,\n\n则 $\\left\\{a_{n}\\right\\}$ 的通项公式 $a_{n}=1-(n-1)=2-n$;\n\n(2) $\\because\\left\\{\\mathrm{a}_{\\mathrm{a}}\\right\\}$ 为等差数列,\n\n$\\therefore a_{1}+a_{4}+a_{7}+\\cdots+a_{3 n+1}$ 以 1 为首项, 以 -3 为公差的等差数列,\n\n$\\therefore \\mathrm{a}_{1}+\\mathrm{a}_{4}+\\mathrm{a}_{7}+\\cdots+\\mathrm{a}_{3 \\mathrm{n}+1}=\\mathrm{n}+1+\\frac{(\\mathrm{n}+1)(\\mathrm{n}+1-1) \\times(-3)}{2}=\\frac{(\\mathrm{n}+1)(2-3 \\mathrm{n})}{2}$"} {"id": "18615", "image": ["9125.jpg", "9125.jpg"], "answer": "由题意可画表格如下:\n\n(1)设只生产书桌 $x$ 个, 可获得利润 $z$ 元,\n\n则 $\\left\\{\\begin{array}{l}0\\.1 x \\leqslant 90 \\\\ 2 x \\leqslant 600 \\\\ z=80 x\\end{array} \\Rightarrow\\left\\{\\begin{array}{l}x \\leqslant 900 \\\\ x \\leqslant 300\\end{array} \\Rightarrow x \\leqslant 300\\right.\\right.$\n\n所以当 $x=300$ 时, $z_{\\max }=80 \\times 300=24000$ (元),\n\n即如果只安排生产书桌, 最多可生产 300 张书桌, 获得利润 24000 元.\n\n(2)设只生产书楖 $y$ 个, 可获利润 $z$ 元,\n\n则 $\\left\\{\\begin{array}{l}0\\.2 y \\leqslant 90 \\\\ 1 \\cdot y \\leqslant 600 \\\\ z=120 y\\end{array} \\Rightarrow\\left\\{\\begin{array}{l}y \\leqslant 450 \\\\ y \\leqslant 600\\end{array} \\Rightarrow y \\leqslant 450\\right.\\right.$.\n\n所以当 $y=450$ 时, $z_{\\text {max }}=120 \\times 450=54000$ (元),\n\n即如果只安排生产书楖, 最多可生产 450 个书根, 获得利润 54000 元.\n\n(3)设生产书桌 $x$ 张, 书根 $y$ 个, 利润总额为 $z$ 元, 则 $\\left\\{\\begin{array}{l}0.1 x+0.2 y \\leqslant 90 \\\\ 2 x+y \\leqslant 600 \\\\ x \\geqslant 0 \\\\ y \\geqslant 0\\end{array} \\Rightarrow\\right.$\n\n$\\left\\{\\begin{array}{l}x+2 y \\leqslant 900 \\\\ 2 x+y \\leqslant 600 \\\\ x \\geqslant 0 \\\\ y \\geqslant 0\\end{array}\\right.$\n\n$z=80 x+120 y$.\n\n在直角坐标平面内作出上面不等式组所表示的平面区域, 即可行域.\n\n\n\n作直线 $l: 80 x+120 y=0$, 即直线 $l: 2 x+3 y=0$.\n\n把直线 $l$ 向右上方平移至 $l_{1}$ 的位置时, 直线经过可行域上的点 $M$, 此时 $z=80 x+120 y$取得最大值.\n\n由 $\\left\\{\\begin{array}{l}x+2 y=900, \\\\ 2 x+y=600\\end{array}\\right.$ 解得点 $M$ 的坐标为 $(100,400)$.\n\n所以当 $x=100, y=400$ 时,\n\n$Z_{\\max }=80 \\times 100+120 \\times 400=56000$ (元).\n\n因此, 生产书桌 100 张、书橱 400 个,\n\n可使所得利润最大.", "solution": "null", "level": "高二", "question": "10. 某家具厂有方木料 $90 \\mathrm{~m}^{3}$, 五合板 $600 \\mathrm{~m}^{2}$, 准备加工成书桌和书梪出售. 已知生产每张书桌需要方木料 $0.1 \\mathrm{~m}^{3}$, 五合板 $2 \\mathrm{~m}^{2}$, 生产每个书相需要方木料 $0.2 \\mathrm{~m}^{3}$, 五合板 $1 \\mathrm{~m}^{2}$,出售一张方桌可获利润 80 元, 出售一个书榞可获利润 120 元.\n\n(1)如果只安排生产书桌, 可获利润多少?\n\n(2)如果只安排生产书桭, 可获利润多少?\n\n(3)怎样安排生产可使所得利润最大?", "options": [], "subject": "代数", "analysis": "由题意可画表格如下:\n\n(1)设只生产书桌 $x$ 个, 可获得利润 $z$ 元,\n\n则 $\\left\\{\\begin{array}{l}0\\.1 x \\leqslant 90 \\\\ 2 x \\leqslant 600 \\\\ z=80 x\\end{array} \\Rightarrow\\left\\{\\begin{array}{l}x \\leqslant 900 \\\\ x \\leqslant 300\\end{array} \\Rightarrow x \\leqslant 300\\right.\\right.$\n\n所以当 $x=300$ 时, $z_{\\max }=80 \\times 300=24000$ (元),\n\n即如果只安排生产书桌, 最多可生产 300 张书桌, 获得利润 24000 元.\n\n(2)设只生产书楖 $y$ 个, 可获利润 $z$ 元,\n\n则 $\\left\\{\\begin{array}{l}0\\.2 y \\leqslant 90 \\\\ 1 \\cdot y \\leqslant 600 \\\\ z=120 y\\end{array} \\Rightarrow\\left\\{\\begin{array}{l}y \\leqslant 450 \\\\ y \\leqslant 600\\end{array} \\Rightarrow y \\leqslant 450\\right.\\right.$.\n\n所以当 $y=450$ 时, $z_{\\text {max }}=120 \\times 450=54000$ (元),\n\n即如果只安排生产书楖, 最多可生产 450 个书根, 获得利润 54000 元.\n\n(3)设生产书桌 $x$ 张, 书根 $y$ 个, 利润总额为 $z$ 元, 则 $\\left\\{\\begin{array}{l}0.1 x+0.2 y \\leqslant 90 \\\\ 2 x+y \\leqslant 600 \\\\ x \\geqslant 0 \\\\ y \\geqslant 0\\end{array} \\Rightarrow\\right.$\n\n$\\left\\{\\begin{array}{l}x+2 y \\leqslant 900 \\\\ 2 x+y \\leqslant 600 \\\\ x \\geqslant 0 \\\\ y \\geqslant 0\\end{array}\\right.$\n\n$z=80 x+120 y$.\n\n在直角坐标平面内作出上面不等式组所表示的平面区域, 即可行域.\n\n\n\n作直线 $l: 80 x+120 y=0$, 即直线 $l: 2 x+3 y=0$.\n\n把直线 $l$ 向右上方平移至 $l_{1}$ 的位置时, 直线经过可行域上的点 $M$, 此时 $z=80 x+120 y$取得最大值.\n\n由 $\\left\\{\\begin{array}{l}x+2 y=900, \\\\ 2 x+y=600\\end{array}\\right.$ 解得点 $M$ 的坐标为 $(100,400)$.\n\n所以当 $x=100, y=400$ 时,\n\n$Z_{\\max }=80 \\times 100+120 \\times 400=56000$ (元).\n\n因此, 生产书桌 100 张、书橱 400 个,\n\n可使所得利润最大."} {"id": "18627", "image": ["9134.jpg", "9134.jpg"], "answer": "作出可行域如图,\n\n\n\n由 $x^{2}+y^{2}=(x-0)^{2}+(y-0)^{2}$,\n\n可以看作区域内的点与原点的距离的平方,\n\n最小值为原点到直线 $x+y-6=0$ 的距离的平方,\n\n即 $|O P|^{2}$, 最大值为 $|O A|^{2}$,\n\n其中 $A(4,10),|O P|=\\frac{|0+0-6|}{\\sqrt{1^{2}+1^{2}}}=\\frac{6}{\\sqrt{2}}=3 \\sqrt{2}$,\n\n$|O A|=\\sqrt{4^{2}+10^{2}}=\\sqrt{116}$,\n\n$\\therefore\\left(x^{2}+y^{2}-2\\right)_{\\text {min }}=(3 \\sqrt{2})^{2}-2=18-2=16$,\n\n$\\left(x^{2}+y^{2}-2\\right)_{\\max }=(\\sqrt{116})^{2}-2=116-2=114$,\n\n$\\therefore 16 \\leqslant x^{2}+y^{2}-2 \\leqslant 114$.\n\n即 $x^{2}+y^{2}-2$ 的取值范围为 $16 \\leqslant x^{2}+y^{2}-2 \\leqslant 114$", "solution": "null", "level": "高二", "question": "11. 已知实数 $x, y$ 满足 $\\left\\{\\begin{array}{l}(x-y+6)(x+y-6) \\geqslant 0 \\\\ 1 \\leqslant x \\leqslant 4\\end{array}\\right.$, 求 $x^{2}+y^{2}-2$ 的取值范围.", "options": [], "subject": "代数", "analysis": "作出可行域如图,\n\n\n\n由 $x^{2}+y^{2}=(x-0)^{2}+(y-0)^{2}$,\n\n可以看作区域内的点与原点的距离的平方,\n\n最小值为原点到直线 $x+y-6=0$ 的距离的平方,\n\n即 $|O P|^{2}$, 最大值为 $|O A|^{2}$,\n\n其中 $A(4,10),|O P|=\\frac{|0+0-6|}{\\sqrt{1^{2}+1^{2}}}=\\frac{6}{\\sqrt{2}}=3 \\sqrt{2}$,\n\n$|O A|=\\sqrt{4^{2}+10^{2}}=\\sqrt{116}$,\n\n$\\therefore\\left(x^{2}+y^{2}-2\\right)_{\\text {min }}=(3 \\sqrt{2})^{2}-2=18-2=16$,\n\n$\\left(x^{2}+y^{2}-2\\right)_{\\max }=(\\sqrt{116})^{2}-2=116-2=114$,\n\n$\\therefore 16 \\leqslant x^{2}+y^{2}-2 \\leqslant 114$.\n\n即 $x^{2}+y^{2}-2$ 的取值范围为 $16 \\leqslant x^{2}+y^{2}-2 \\leqslant 114$."} {"id": "18628", "image": ["9135.jpg", "9135.jpg"], "answer": "由于 $\\mathrm{z}=\\frac{y+1}{x+1}=\\frac{y-(-1)}{x-(-1)}$,\n\n所以 $z$ 的几何意义是点 $(x, y)$ 与点 $M(-1,-1)$ 连线的斜率,\n\n因此 $\\frac{y+1}{x+1}$ 的最值就是点 $(x, y)$ 与点 $M(-1,-1)$ 连线的斜率的最值,\n\n结合图可知, 直线 $M B$ 的斜率最大, 直线 $M C$ 的斜率最小, 即\n\n$z_{\\text {max }}=k_{M B}=3$, 此时 $x=0, y=2$;\n$z_{\\text {min }}=k_{M C}=\\frac{1}{2}$, 此时 $x=1, y=0$.\n\n$\\therefore z$ 的最大值为 3 , 最小值为 $\\frac{1}{2}$.\n\n", "solution": "null", "level": "高二", "question": "12. 已知实数 $x 、 y$ 满足 $\\left\\{\\begin{array}{l}2 x+y-2 \\geqslant 0 \\\\ x-2 y+4 \\geqslant 0 \\\\ 3 x-y-3 \\leqslant 0\\end{array}\\right.$, 试求 $z=\\frac{y+1}{x+1}$ 的最大值和最小值.", "options": [], "subject": "代数", "analysis": "由于 $\\mathrm{z}=\\frac{y+1}{x+1}=\\frac{y-(-1)}{x-(-1)}$,\n\n所以 $z$ 的几何意义是点 $(x, y)$ 与点 $M(-1,-1)$ 连线的斜率,\n\n因此 $\\frac{y+1}{x+1}$ 的最值就是点 $(x, y)$ 与点 $M(-1,-1)$ 连线的斜率的最值,\n\n结合图可知, 直线 $M B$ 的斜率最大, 直线 $M C$ 的斜率最小, 即\n\n$z_{\\text {max }}=k_{M B}=3$, 此时 $x=0, y=2$;\n$z_{\\text {min }}=k_{M C}=\\frac{1}{2}$, 此时 $x=1, y=0$.\n\n$\\therefore z$ 的最大值为 3 , 最小值为 $\\frac{1}{2}$.\n\n哈哈"} {"id": "19845", "image": [], "answer": "(1) $x^{2}+3>3 x$; (2) $a^{3}+b^{3}>a^{2} b+a b^{2}$.\n\n(1) $\\left(x^{2}+3\\right)-3 x=x^{2}-3 x+3=\\left(x-\\frac{3}{2}\\right)^{2}+\\frac{3}{4} \\geqslant \\frac{3}{4}>0, \\quad \\therefore x^{2}+3>3 x$.\n\n(2) $\\left(a^{3}+b^{3}\\right)-\\left(a^{2} b+a b^{2}\\right)=a^{3}+b^{3}-a^{2} b-a b^{2}$\n\n$=a^{2}(a-b)-b^{2}(a-b)=(a-b)\\left(a^{2}-b^{2}\\right)=(a-b)^{2}(a+b)$,\n\n$\\because a>0, b>0$ 且 $a \\neq b$,\n\n$\\therefore(a-b)^{2}>0, a+b>0$,\n\n$\\therefore\\left(a^{3}+b^{3}\\right)-\\left(a^{2} b+a b^{2}\\right)>0$, 即 $a^{3}+b^{3}>a^{2} b+a b^{2}$.", "solution": "null", "level": "高二", "question": "比较下列各组中两个代数式的大小:\n\n(1) $x^{2}+3$ 与 $3 x$; (2)已知 $a, b$ 为正数, 且 $a \\neq b$, 比较 $a^{3}+b^{3}$ 与 $a^{2} b+a b^{2}$.", "options": [], "subject": "代数", "analysis": "(1) $x^{2}+3>3 x$; (2) $a^{3}+b^{3}>a^{2} b+a b^{2}$.\n\n(1) $\\left(x^{2}+3\\right)-3 x=x^{2}-3 x+3=\\left(x-\\frac{3}{2}\\right)^{2}+\\frac{3}{4} \\geqslant \\frac{3}{4}>0, \\quad \\therefore x^{2}+3>3 x$.\n\n(2) $\\left(a^{3}+b^{3}\\right)-\\left(a^{2} b+a b^{2}\\right)=a^{3}+b^{3}-a^{2} b-a b^{2}$\n\n$=a^{2}(a-b)-b^{2}(a-b)=(a-b)\\left(a^{2}-b^{2}\\right)=(a-b)^{2}(a+b)$,\n\n$\\because a>0, b>0$ 且 $a \\neq b$,\n\n$\\therefore(a-b)^{2}>0, a+b>0$,\n\n$\\therefore\\left(a^{3}+b^{3}\\right)-\\left(a^{2} b+a b^{2}\\right)>0$, 即 $a^{3}+b^{3}>a^{2} b+a b^{2}$."} {"id": "19912", "image": [], "answer": "(1) 1\n\n(2) 2", "solution": "null", "level": "高二", "question": "已知 $\\lg (3 x)+\\lg y=\\lg (x+y+1)$.\n\n(1)求 $\\mathrm{xy}$ 的最小值;\n\n(2)求 $x+y$ 的最小值.", "options": [], "subject": "代数", "analysis": "由 $\\lg (3 \\mathrm{x})+\\lg y=\\lg (\\mathrm{x}+\\mathrm{y}+1)$ 得 $\\{y>0$\n\n$$\n3 x y=x+y+1\n$$\n\n(1) $\\because x>0, y>0$,\n\n$\\therefore 3 x y=x+y+1 \\geq 2 \\sqrt{x y}+1$,\n\n$\\therefore 3 x y-2 \\sqrt{x y}-1 \\geq 0$,\n\n即 $3(\\sqrt{x y})^{2}-2 \\sqrt{x y}-1 \\geq 0$,\n\n$\\therefore(3 \\sqrt{x y}+1)(\\sqrt{x y}-1) \\geq 0$,\n\n$\\therefore \\sqrt{x y} \\geq 1, \\quad \\therefore x y \\geq 1$,\n\n当且仅当 $x=y=1$ 时, 等号成立.\n$\\therefore \\mathrm{xy}$ 的最小值为 1 .\n\n(2) $\\because x>0, \\quad y>0$,\n\n$\\therefore \\mathrm{x}+\\mathrm{y}+1=3 \\mathrm{xy} \\leq 3 \\cdot\\left(\\frac{x+y}{2}\\right)^{2}$,\n\n$\\therefore 3(\\mathrm{x}+\\mathrm{y})^{2}-4(\\mathrm{x}+\\mathrm{y})-4 \\geq 0$,\n\n$\\therefore[3(\\mathrm{x}+\\mathrm{y})+2][(\\mathrm{x}+\\mathrm{y})-2] \\geq 0$,\n\n$\\therefore \\mathrm{x}+\\mathrm{y} \\geq 2$,\n\n当且仅当 $x=y=1$ 时取等号,\n\n$\\therefore \\mathrm{x}+\\mathrm{y}$ 的最小值为 2 ."} {"id": "19913", "image": [], "answer": "(1) 4\n(2) $\\frac{1}{12}$", "solution": "null", "level": "高二", "question": "求解下列问题:\n\n(1) 若 $x>0$, 求 $f(x)=\\frac{4}{x}+x$ 的最小值;\n\n(2) 已知 $00, \\frac{4}{x}>0$,\n\n$\\therefore f(x)=x+\\frac{4}{x} \\geq 2 \\sqrt{x \\cdot \\frac{4}{x}}=4$,\n\n当且仅当 $x=\\frac{4}{x}$, 即 $x=2$ 时取等号. 故 $f(x)_{\\min }=4$.\n\n(2) $\\because 00$,\n\n$\\therefore y=x(1-3 x)=\\frac{1}{3} \\cdot 3 x \\cdot(1-3 x) \\leq \\frac{1}{3} \\cdot\\left(\\frac{3 x+1-3 x}{2}\\right)^{2}=\\frac{1}{12}$,\n\n当且仅当 $3 x=1-3 x$, 即 $x=\\frac{1}{6}$ 时取等号. $\\therefore y_{\\text {max }}=\\frac{1}{12}$."} {"id": "19510", "image": [], "answer": "证明 (1)由于 $\\frac{b}{a}-\\frac{a}{b}=\\frac{b^{2}-a^{2}}{a b}=\\frac{(b+a)(b-a)}{a b}$$\\because a0, a b>0$$\\therefore \\frac{(b+a)(b-a)}{a b}<0$, 故 $\\frac{b}{a}<\\frac{a}{b}$(2) $\\because \\frac{1}{a}<\\frac{1}{b}$$\\therefore \\frac{1}{a}-\\frac{1}{b}<0$, 即 $\\frac{b-a}{a b}<0$", "solution": "null", "level": "高二", "question": "解答\n\n(1) $ab, \\frac{1}{a}<\\frac{1}{b}$, 求证: $a b>0$", "options": [], "subject": "代数", "analysis": "证明 (1)由于 $\\frac{b}{a}-\\frac{a}{b}=\\frac{b^{2}-a^{2}}{a b}=\\frac{(b+a)(b-a)}{a b}$$\\because a0, a b>0$$\\therefore \\frac{(b+a)(b-a)}{a b}<0$, 故 $\\frac{b}{a}<\\frac{a}{b}$(2) $\\because \\frac{1}{a}<\\frac{1}{b}$$\\therefore \\frac{1}{a}-\\frac{1}{b}<0$, 即 $\\frac{b-a}{a b}<0$"} {"id": "19528", "image": [], "answer": "(1) 因为 $a_{n}=1+\\frac{1}{a+2(n-1)}\\left(n \\in \\mathbf{N}^{*}, a \\in \\mathbf{R}\\right.$ 且 $\\left.a \\neq 0\\right)$, 又 $a=-7$,\n\n所以 $a_{n}=1+\\frac{1}{2 n-9}\\left(n \\in \\mathbf{N}^{*}\\right)$.\n\n结合函数 $f(x)=1+\\frac{1}{2 x-9}$ 的单调性,\n\n可知 $1>a_{1}>a_{2}>a_{3}>a_{4}, a_{5}>a_{6}>a_{7}>\\cdots>a_{n}>1\\left(n \\in \\mathbf{N}^{*}\\right)$.\n\n所以数列 $\\left\\{a_{n}\\right\\}$ 中的最大项为 $a_{5}=2$, 最小项为 $a_{4}=0$.\n\n(2) $a_{n}=1+\\frac{1}{a+2(n-1)}=1+\\frac{\\frac{1}{2}}{n-\\frac{2-a}{2}}$, 已知对任意的 $n \\in \\mathbf{N}^{*}$, 都有 $a_{n} \\leqslant a_{6}$ 成立, 结合函数\n\n$f(x)=1+\\frac{\\frac{1}{2}}{x-\\frac{2-a}{2}}$ 的单调性, 可知 $5<\\frac{2-a}{2}<6$, 即 $-10a_{1}>a_{2}>a_{3}>a_{4}, a_{5}>a_{6}>a_{7}>\\cdots>a_{n}>1\\left(n \\in \\mathbf{N}^{*}\\right)$.\n\n所以数列 $\\left\\{a_{n}\\right\\}$ 中的最大项为 $a_{5}=2$, 最小项为 $a_{4}=0$.\n\n(2) $a_{n}=1+\\frac{1}{a+2(n-1)}=1+\\frac{\\frac{1}{2}}{n-\\frac{2-a}{2}}$, 已知对任意的 $n \\in \\mathbf{N}^{*}$, 都有 $a_{n} \\leqslant a_{6}$ 成立, 结合函数\n\n$f(x)=1+\\frac{\\frac{1}{2}}{x-\\frac{2-a}{2}}$ 的单调性, 可知 $5<\\frac{2-a}{2}<6$, 即 $-100$, 设 $\\left\\{a_{n}\\right\\}$ 公比为 $q$,\n\n$\\therefore a_{1} q^{2}=8, \\quad \\therefore q=2$\n\n$\\therefore a_{n}=2 \\times 2^{n-1}=2^{n}$\n$\\therefore 2 b_{n}=2^{1}+2^{2}+2^{3}+\\cdots+2^{n}=\\frac{2\\left(1-2^{n}\\right)}{1-2}=2^{n+1}-2$,\n\n$\\therefore b_{n}=2^{n}-1$.\n\n(2) 证明: 由已知: $c_{n}=\\frac{1}{\\log _{2} a_{n} \\log _{2} a_{n+1}}=\\frac{1}{n(n+1)}=\\frac{1}{\\mathrm{n}}-\\frac{1}{\\mathrm{n}+1}$.\n\n$\\therefore c_{1}+c_{2}+c_{3}+\\cdots+c_{n}=\\frac{1}{1}-\\frac{1}{2}+\\frac{1}{2}-\\frac{1}{3}+\\cdots+\\frac{1}{\\mathrm{n}}-\\frac{1}{\\mathrm{n}+1}=1-\\frac{1}{n+1}<1$"} {"id": "18942", "image": [], "answer": "(1) 由 $a_{n+1}=\\lambda S_{n}+1$ 知 $a_{n}=\\lambda S_{n-1}+1(n \\geq 2)$,\n\n两式相减, 得 $a_{n+1}=(\\lambda+1) a_{n}$, 于是公比 $q=\\lambda+1$,\n\n所以 $a_{2}=\\lambda a_{1}+1=(\\lambda+1) a_{1}$,\n\n解得 $a_{1}=1$.\n\n(2) 由 (1) 可得 $q=5, a_{n+1}=5 a_{n}$, 所以 $a_{n}=5^{n-1}$.\n\n所以 $\\frac{1}{a_{n}}=\\frac{1}{5^{n-1}}$,\n\n所以 $\\frac{1}{a_{1}}+\\frac{1}{a_{2}}+\\cdots+\\frac{1}{a_{n}}=\\frac{1}{1}+\\frac{1}{5}+\\cdots+\\frac{1}{5^{n-1}}=\\frac{1\\left(1-\\frac{1}{5^{n}}\\right)}{1-\\frac{1}{5}}=\\frac{5}{4}\\left(1-\\frac{1}{5^{n}}\\right)<\\frac{5}{4}$,\n因为 $\\forall n \\in \\mathbf{N}^{*}, \\frac{1}{a_{1}}+\\frac{1}{a_{2}}+\\cdots+\\frac{1}{a_{n}}0, \\therefore S_{1}=2$, 即 $a_{1}=2$.\n\n( II ) 由 $S_{n}^{2}-\\left(n^{2}+n-3\\right) S_{n}-3\\left(n^{2}+n\\right)=0$, 得: $\\left(S_{n}+3\\right)\\left[S_{n}-\\left(n^{2}+n\\right)\\right]=0$,\n\n$\\because a_{n}>0\\left(n \\in N^{*}\\right), \\therefore S_{n}>0$, 从而 $S_{n}+3>0, \\therefore S_{n}=n^{2}+n$,\n\n$\\therefore$ 当 $n \\geq 2$ 时, $a_{n}=S_{n}-S_{n-1}=n^{2}+n-\\left[(n-1)^{2}+(n-1)\\right]=2 n$, 又 $a_{1}=2=2 \\times 1, \\therefore a_{n}=2 n\\left(n \\in N^{*}\\right)$.\n\n(III) 当 $k \\in \\mathbf{N}^{*}$ 时, $k^{2}+\\frac{k}{2}>k^{2}+\\frac{k}{2}-\\frac{3}{16}=\\left(k-\\frac{1}{4}\\right)\\left(k+\\frac{3}{4}\\right)$\n\n$\\therefore \\frac{1}{a_{k}\\left(a_{k}+1\\right)}=\\frac{1}{2 k(2 k+1)}=\\frac{1}{4} \\cdot \\frac{1}{k\\left(k+\\frac{1}{2}\\right)}<\\frac{1}{4} \\cdot \\frac{1}{\\left(k-\\frac{1}{4}\\right)\\left(k+\\frac{3}{4}\\right)}$\n\n$=\\frac{1}{4} \\cdot \\frac{1}{\\left(k-\\frac{1}{4}\\right) \\cdot\\left[(k+1)-\\frac{1}{4}\\right]}=\\frac{1}{4} \\cdot\\left[\\frac{1}{k-\\frac{1}{4}}-\\frac{1}{(k+1)-\\frac{1}{4}}\\right]$\n\n$\\therefore \\frac{1}{a_{1}\\left(a_{1}+1\\right)}+\\frac{1}{a_{2}\\left(a_{2}+1\\right)}+\\cdots+\\frac{1}{a_{n}\\left(a_{n}+1\\right)}<\\frac{1}{3}$", "solution": "null", "level": "高二", "question": "22. (2014广东) 设各项均为正数的数列 $\\left\\{a_{n}\\right\\}$ 的前 $n$ 项和为 $S_{n}$, 且 $S_{n}$ 满足\n\n$$\nS_{n}^{2}-\\left(n^{2}+n-3\\right) S_{n}-3\\left(n^{2}+n\\right)=0, n \\in N^{*}\n$$\n\n(I) 求 $a_{1}$ 的值;\n\n(II) 求数列 $\\left\\{a_{n}\\right\\}$ 的通项公式;\n\n(III) 证明: 对一切正整数 $n$, 有 $\\frac{1}{a_{1}\\left(a_{1}+1\\right)}+\\frac{1}{a_{2}\\left(a_{2}+1\\right)}+\\cdots \\frac{1}{a_{n}\\left(a_{n}+1\\right)}<\\frac{1}{3}$.", "options": [], "subject": "代数", "analysis": "(I) 令 $n=1$ 得: $S_{1}^{2}-(-1) S_{1}-3 \\times 2=0$, 即 $S_{1}^{2}+S_{1}-6=0$,\n\n所以 $\\left(S_{1}+3\\right)\\left(S_{1}-2\\right)=0, \\because S_{1}>0, \\therefore S_{1}=2$, 即 $a_{1}=2$.\n\n( II ) 由 $S_{n}^{2}-\\left(n^{2}+n-3\\right) S_{n}-3\\left(n^{2}+n\\right)=0$, 得: $\\left(S_{n}+3\\right)\\left[S_{n}-\\left(n^{2}+n\\right)\\right]=0$,\n\n$\\because a_{n}>0\\left(n \\in N^{*}\\right), \\therefore S_{n}>0$, 从而 $S_{n}+3>0, \\therefore S_{n}=n^{2}+n$,\n\n$\\therefore$ 当 $n \\geq 2$ 时, $a_{n}=S_{n}-S_{n-1}=n^{2}+n-\\left[(n-1)^{2}+(n-1)\\right]=2 n$, 又 $a_{1}=2=2 \\times 1, \\therefore a_{n}=2 n\\left(n \\in N^{*}\\right)$.\n\n(III) 当 $k \\in \\mathbf{N}^{*}$ 时, $k^{2}+\\frac{k}{2}>k^{2}+\\frac{k}{2}-\\frac{3}{16}=\\left(k-\\frac{1}{4}\\right)\\left(k+\\frac{3}{4}\\right)$\n\n$\\therefore \\frac{1}{a_{k}\\left(a_{k}+1\\right)}=\\frac{1}{2 k(2 k+1)}=\\frac{1}{4} \\cdot \\frac{1}{k\\left(k+\\frac{1}{2}\\right)}<\\frac{1}{4} \\cdot \\frac{1}{\\left(k-\\frac{1}{4}\\right)\\left(k+\\frac{3}{4}\\right)}$\n\n$=\\frac{1}{4} \\cdot \\frac{1}{\\left(k-\\frac{1}{4}\\right) \\cdot\\left[(k+1)-\\frac{1}{4}\\right]}=\\frac{1}{4} \\cdot\\left[\\frac{1}{k-\\frac{1}{4}}-\\frac{1}{(k+1)-\\frac{1}{4}}\\right]$\n\n$\\therefore \\frac{1}{a_{1}\\left(a_{1}+1\\right)}+\\frac{1}{a_{2}\\left(a_{2}+1\\right)}+\\cdots+\\frac{1}{a_{n}\\left(a_{n}+1\\right)}<\\frac{1}{3}$版权声明"} {"id": "19194", "image": [], "answer": "(I) 由题设, $a_{n} a_{n+1}=\\lambda S_{n}-1, a_{n+1} a_{n+2}=\\lambda S_{n+1}-1$. 两式相减得 $a_{n+1}\\left(a_{n+2}-a\\right)=\\lambda a_{n+1}$.由于 $a_{n+1} \\neq 0$, 所以 $a_{n+2}-a_{n}=\\lambda$.\n\n(II) 由题设, $a_{1}=1, a_{1} a_{2}=\\lambda S_{1}-1$, 可得 $a_{2}=\\lambda-1$. 由( I) 知, $a_{3}=\\lambda+1$.\n\n令 $2 a_{2}=a_{1}+a_{3}$, 解得 $\\lambda=4$. 故 $a_{n+2}-a_{n}=4$, 由此可得\n\n$\\left\\{a_{2 n-1}\\right\\}$ 是首项为 1 , 公差为 4 的等差数列, $a_{2 n-1}=4 n-3$;\n\n$\\left\\{a_{2 n}\\right\\}$ 是首项为 3 , 公差为 4 的等差数列, $a_{2 n}=4 n-1$.\n\n所以 $a_{n}=2 n-1, a_{n-1}-a_{n}=2$.\n\n因此存在 $\\lambda=4$, 使得数列 $\\left\\{a_{n}\\right\\}$ 为等差数列.", "solution": "null", "level": "高二", "question": "22. (2014 新课标 1) 已知数列 $\\left\\{a_{n}\\right\\}$ 的前 $n$ 项和为 $S_{n}, a_{1}=1, a_{n} \\neq 0, a_{n} a_{n+1}=\\lambda S_{n}-1$, 其中 $\\lambda$ 为常数.\n\n( I )证明: $a_{n+2}-a_{n}=\\lambda$;\n\n( II) 是否存在 $\\lambda$, 使得 $\\left\\{a_{n}\\right\\}$ 为等差数列? 并说明理由.", "options": [], "subject": "代数", "analysis": "(I) 由题设, $a_{n} a_{n+1}=\\lambda S_{n}-1, a_{n+1} a_{n+2}=\\lambda S_{n+1}-1$. 两式相减得 $a_{n+1}\\left(a_{n+2}-a\\right)=\\lambda a_{n+1}$.由于 $a_{n+1} \\neq 0$, 所以 $a_{n+2}-a_{n}=\\lambda$.\n\n(II) 由题设, $a_{1}=1, a_{1} a_{2}=\\lambda S_{1}-1$, 可得 $a_{2}=\\lambda-1$. 由( I) 知, $a_{3}=\\lambda+1$.\n\n令 $2 a_{2}=a_{1}+a_{3}$, 解得 $\\lambda=4$. 故 $a_{n+2}-a_{n}=4$, 由此可得\n\n$\\left\\{a_{2 n-1}\\right\\}$ 是首项为 1 , 公差为 4 的等差数列, $a_{2 n-1}=4 n-3$;\n\n$\\left\\{a_{2 n}\\right\\}$ 是首项为 3 , 公差为 4 的等差数列, $a_{2 n}=4 n-1$.\n\n所以 $a_{n}=2 n-1, a_{n-1}-a_{n}=2$.\n\n因此存在 $\\lambda=4$, 使得数列 $\\left\\{a_{n}\\right\\}$ 为等差数列."} {"id": "19210", "image": [], "answer": "(I) 当 $n \\geq 2$ 时, $S_{n}-S_{n-1}=\\frac{2 S_{n}^{2}}{2 S_{n}-1}, S_{n-1}-S_{n}=2 S_{n} S_{n-1}, \\frac{1}{S_{n}}-\\frac{1}{S_{n-1}}=2$,从而 $\\left\\{\\frac{1}{S_{n}}\\right\\}$ 构成以 4 为首项, 2 为公差的等差数列.\n\n(II) 由 (1) 可知, $\\frac{1}{S_{n}}=\\frac{1}{S_{1}}+(n-1) \\times 2=2 n+2 \\quad \\therefore S_{n}=\\frac{1}{2(n+1)}$.\n\n$\\frac{1}{n} S_{n}=\\frac{1}{2 n(n+1)}=\\frac{1}{2}\\left(\\frac{1}{n}-\\frac{1}{n+1}\\right)$\n\n$S_{1}+\\frac{1}{2} S_{2}+\\frac{1}{3}+\\cdots+\\frac{1}{n} S_{n}=\\frac{1}{2}\\left(1-\\frac{1}{2}+\\frac{1}{2}-\\frac{1}{3}+\\cdots+\\frac{1}{n}-\\frac{1}{n+1}\\right)=\\frac{1}{2}\\left(1-\\frac{1}{n+1}\\right)<\\frac{1}{2}$.", "solution": "null", "level": "高二", "question": "16. (2018 届湖北省华师一附中高三 9 月调研)已知数列 $\\left\\{a_{n}\\right\\}$ 中, $a_{1}=\\frac{1}{4}$, 其前 $n$ 项的和为 $S_{n}$, 且满足 $a_{n}=\\frac{2 S_{n}{ }^{2}}{2 S_{n}-1}(n \\geq 2)$.\n\n(I) 求证: 数列 $\\left\\{\\frac{1}{S_{n}}\\right\\}$ 是等差数列;\n\n(II) 证明: $S_{1}+\\frac{1}{2} S_{2}+\\frac{1}{3}+\\cdots+\\frac{1}{n} S_{n}<\\frac{1}{2}$", "options": [], "subject": "代数", "analysis": "(I) 当 $n \\geq 2$ 时, $S_{n}-S_{n-1}=\\frac{2 S_{n}^{2}}{2 S_{n}-1}, S_{n-1}-S_{n}=2 S_{n} S_{n-1}, \\frac{1}{S_{n}}-\\frac{1}{S_{n-1}}=2$,从而 $\\left\\{\\frac{1}{S_{n}}\\right\\}$ 构成以 4 为首项, 2 为公差的等差数列.\n\n(II) 由 (1) 可知, $\\frac{1}{S_{n}}=\\frac{1}{S_{1}}+(n-1) \\times 2=2 n+2 \\quad \\therefore S_{n}=\\frac{1}{2(n+1)}$.\n\n$\\frac{1}{n} S_{n}=\\frac{1}{2 n(n+1)}=\\frac{1}{2}\\left(\\frac{1}{n}-\\frac{1}{n+1}\\right)$\n\n$S_{1}+\\frac{1}{2} S_{2}+\\frac{1}{3}+\\cdots+\\frac{1}{n} S_{n}=\\frac{1}{2}\\left(1-\\frac{1}{2}+\\frac{1}{2}-\\frac{1}{3}+\\cdots+\\frac{1}{n}-\\frac{1}{n+1}\\right)=\\frac{1}{2}\\left(1-\\frac{1}{n+1}\\right)<\\frac{1}{2}$."} {"id": "19220", "image": [], "answer": "(1)由 $\\left(a_{n}+1\\right) \\cdot a_{n+1}=a_{n}$ 得 $a_{n+1}=\\frac{a_{n}}{a_{n}+1}$,\n两边取倒数得 $\\frac{1}{a_{n+1}}=\\frac{a_{n}+1}{a_{n}}=\\frac{1}{a_{n}}+1$,\n\n故 $\\frac{1}{a_{n+1}}-\\frac{1}{a_{n}}=1$, 又 $\\frac{1}{a_{1}}=1$,\n\n所以数列 $\\left\\{\\frac{1}{a_{n}}\\right\\}$ 是以 1 为首项, 以 1 为公差的等差数列.\n\n(2) 由(1)可知知 $\\frac{1}{a_{n}}=n$, 故 $a_{n}=\\frac{1}{n}$,\n\n因此 $b_{n}=\\frac{\\sqrt{a_{n} a_{n+1}}}{\\sqrt{n+1}+\\sqrt{n}}=\\frac{\\sqrt{\\frac{1}{n(n+1)}}}{\\sqrt{n+1}+\\sqrt{n}}=\\frac{\\sqrt{n+1}-\\sqrt{n}}{\\sqrt{n(n+1)}}=\\frac{1}{\\sqrt{n}}-\\frac{1}{\\sqrt{n+1}}$,\n\n所以 $T_{n}=b_{1}+b_{2}+b_{3}+\\cdots+b_{n}$\n\n$$\n\\begin{aligned}\n& =\\left(1-\\frac{1}{\\sqrt{2}}\\right)+\\left(\\frac{1}{\\sqrt{2}}-\\frac{1}{\\sqrt{3}}\\right)+\\left(\\frac{1}{\\sqrt{3}}-\\frac{1}{\\sqrt{4}}\\right)+\\cdots+\\left(\\frac{1}{\\sqrt{n}}-\\frac{1}{\\sqrt{n+1}}\\right) \\\\\n& =1-\\frac{1}{\\sqrt{n+1}}\n\\end{aligned}\n$$", "solution": "null", "level": "高二", "question": "25. 已知数列 $\\left\\{a_{n}\\right\\}$ 的首项为 $a_{1}=1$, 且 $\\left(a_{n}+1\\right) \\cdot a_{n+1}=a_{n}, n \\in \\mathbf{N}^{*}$.\n\n(1) 求证: 数列 $\\left\\{\\frac{1}{a_{n}}\\right\\}$ 是等差数列;\n\n(2) 设 $b_{n}=\\frac{\\sqrt{a_{n} a_{n+1}}}{\\sqrt{n+1}+\\sqrt{n}}$, 求数列 $\\left\\{b_{n}\\right\\}$ 的前 $n$ 项和 $T_{n}$.", "options": [], "subject": "代数", "analysis": "(1)由 $\\left(a_{n}+1\\right) \\cdot a_{n+1}=a_{n}$ 得 $a_{n+1}=\\frac{a_{n}}{a_{n}+1}$,\n两边取倒数得 $\\frac{1}{a_{n+1}}=\\frac{a_{n}+1}{a_{n}}=\\frac{1}{a_{n}}+1$,\n\n故 $\\frac{1}{a_{n+1}}-\\frac{1}{a_{n}}=1$, 又 $\\frac{1}{a_{1}}=1$,\n\n所以数列 $\\left\\{\\frac{1}{a_{n}}\\right\\}$ 是以 1 为首项, 以 1 为公差的等差数列.\n\n(2) 由(1)可知知 $\\frac{1}{a_{n}}=n$, 故 $a_{n}=\\frac{1}{n}$,\n\n因此 $b_{n}=\\frac{\\sqrt{a_{n} a_{n+1}}}{\\sqrt{n+1}+\\sqrt{n}}=\\frac{\\sqrt{\\frac{1}{n(n+1)}}}{\\sqrt{n+1}+\\sqrt{n}}=\\frac{\\sqrt{n+1}-\\sqrt{n}}{\\sqrt{n(n+1)}}=\\frac{1}{\\sqrt{n}}-\\frac{1}{\\sqrt{n+1}}$,\n\n所以 $T_{n}=b_{1}+b_{2}+b_{3}+\\cdots+b_{n}$\n\n$$\n\\begin{aligned}\n& =\\left(1-\\frac{1}{\\sqrt{2}}\\right)+\\left(\\frac{1}{\\sqrt{2}}-\\frac{1}{\\sqrt{3}}\\right)+\\left(\\frac{1}{\\sqrt{3}}-\\frac{1}{\\sqrt{4}}\\right)+\\cdots+\\left(\\frac{1}{\\sqrt{n}}-\\frac{1}{\\sqrt{n+1}}\\right) \\\\\n& =1-\\frac{1}{\\sqrt{n+1}}\n\\end{aligned}\n$$"} {"id": "19221", "image": [], "answer": "$$\nb_{n+1}-b_{n}=\\frac{1}{a_{n+1}-1}-\\frac{1}{a_{n}-1}=\\frac{1}{2-\\frac{1}{a_{n}}-1}-\\frac{1}{a_{n}-1}=\\frac{a_{n}}{a_{n}-1}-\\frac{1}{a_{n}-1}=1\n$$\n\n$\\therefore$ 数列 $\\left\\{b_{n}\\right\\}$ 是公差为 1 的等差数列;\n\n(II) 由题意可得 $b_{2}^{2}=b_{1} b_{4}$, 即 $\\left(b_{1}+1\\right)^{2}=b_{1}\\left(b_{1}+3\\right)$, 所以 $b_{1}=1$, 所以 $b_{n}=1$,\n\n$$\n\\therefore S_{n}=\\frac{n(n+1)}{2}, \\therefore \\frac{1}{S_{n}}=\\frac{2}{n(n+1)}=2\\left(\\frac{1}{n}-\\frac{1}{n+1}\\right)\n$$\n\n$T_{n}=2 \\times\\left(1-\\frac{1}{2}+\\frac{1}{2}-\\frac{1}{3}+\\ldots+\\frac{1}{n}-\\frac{1}{n+1}\\right)=2 \\times\\left(1-\\frac{1}{n+1}\\right)=\\frac{2 n}{n+1}$.", "solution": "null", "level": "高二", "question": "26. 已知数列 $\\left\\{a_{n}\\right\\}$ 满足: $a_{n} \\neq 1, a_{n+1}=2-\\frac{1}{a_{n}}\\left(n \\in \\mathbf{N}^{*}\\right)$, 数列 $\\left\\{b_{n}\\right\\}$ 中, $b_{n}=\\frac{1}{a_{n}-1}$, 且 $b_{1}, b_{2}$, $b_{4}$ 成\n\n等比数列.\n\n(I) 求证: 数列 $\\left\\{b_{n}\\right\\}$ 是等差数列;\n\n(II) 若 $S_{n}$ 是数列 $\\left\\{b_{n}\\right\\}$ 的前 $n$ 项和, 求数列 $\\left\\{\\frac{1}{S_{n}}\\right\\}$ 的前 $n$ 项和 $T_{n}$.", "options": [], "subject": "代数", "analysis": "$$\nb_{n+1}-b_{n}=\\frac{1}{a_{n+1}-1}-\\frac{1}{a_{n}-1}=\\frac{1}{2-\\frac{1}{a_{n}}-1}-\\frac{1}{a_{n}-1}=\\frac{a_{n}}{a_{n}-1}-\\frac{1}{a_{n}-1}=1\n$$\n\n$\\therefore$ 数列 $\\left\\{b_{n}\\right\\}$ 是公差为 1 的等差数列;\n\n(II) 由题意可得 $b_{2}^{2}=b_{1} b_{4}$, 即 $\\left(b_{1}+1\\right)^{2}=b_{1}\\left(b_{1}+3\\right)$, 所以 $b_{1}=1$, 所以 $b_{n}=1$,\n\n$$\n\\therefore S_{n}=\\frac{n(n+1)}{2}, \\therefore \\frac{1}{S_{n}}=\\frac{2}{n(n+1)}=2\\left(\\frac{1}{n}-\\frac{1}{n+1}\\right)\n$$\n\n$T_{n}=2 \\times\\left(1-\\frac{1}{2}+\\frac{1}{2}-\\frac{1}{3}+\\ldots+\\frac{1}{n}-\\frac{1}{n+1}\\right)=2 \\times\\left(1-\\frac{1}{n+1}\\right)=\\frac{2 n}{n+1}$."} {"id": "19232", "image": [], "answer": "(1)由已知, $a_{1} b_{2}+b_{2}=b_{1}, b_{1}=1, b_{2}=\\frac{1}{3}$, 得 $a_{1}=2$.\n\n$\\therefore$ 数列 $\\left\\{a_{n}\\right\\}$ 是首项为 2 , 公差为 3 的等差数列, 通项公式为 $a_{n}=3 n-1$.\n\n(2)由(1)知 $a_{n} b_{n-1}+b_{n-1}=n b_{n}$, 得 $b_{n-1}=\\frac{b_{n}}{3}$,\n\n因此 $\\left\\{b_{n}\\right\\}$ 是首项为 1 ,公比为 $\\frac{1}{3}$ 的等比数列.\n\n记 $\\left\\{b_{n}\\right\\}$ 的前 $n$ 项和为 $S_{n}$ ,\n\n则 $S_{x}=\\frac{1-\\left(\\frac{1}{3}\\right)^{n}}{1-\\frac{1}{3}}=\\frac{3}{2}-\\frac{1}{2 \\times 3^{n-1}}$.", "solution": "null", "level": "高二", "question": "12. 已知 $\\left\\{a_{n}\\right\\}$ 是公差为 3 的等差数列, 数列 $\\left\\{b_{n}\\right\\}$ 满足 $b_{1}=1, b_{2}=\\frac{1}{3}, a_{n} b_{n+1}+b_{n+1}=n b_{n}$.\n\n(1)求 $\\left\\{a_{n}\\right\\}$ 的通项公式;\n\n(2)求 $\\left\\{b_{n}\\right\\}$ 的前 $n$ 项和.", "options": [], "subject": "代数", "analysis": "(1)由已知, $a_{1} b_{2}+b_{2}=b_{1}, b_{1}=1, b_{2}=\\frac{1}{3}$, 得 $a_{1}=2$.\n\n$\\therefore$ 数列 $\\left\\{a_{n}\\right\\}$ 是首项为 2 , 公差为 3 的等差数列, 通项公式为 $a_{n}=3 n-1$.\n\n(2)由(1)知 $a_{n} b_{n-1}+b_{n-1}=n b_{n}$, 得 $b_{n-1}=\\frac{b_{n}}{3}$,\n\n因此 $\\left\\{b_{n}\\right\\}$ 是首项为 1 ,公比为 $\\frac{1}{3}$ 的等比数列.\n\n记 $\\left\\{b_{n}\\right\\}$ 的前 $n$ 项和为 $S_{n}$ ,\n\n则 $S_{x}=\\frac{1-\\left(\\frac{1}{3}\\right)^{n}}{1-\\frac{1}{3}}=\\frac{3}{2}-\\frac{1}{2 \\times 3^{n-1}}$."} {"id": "19236", "image": [], "answer": "(1)由 $a_{4}+2$ 是 $a_{3}, a_{5}$ 的等差中项得 $a_{3}+a_{5}=2 a_{4}+4$, 所以 $a_{3}+a_{4}+a_{5}=3 a_{4}+4=28$,解得 $a_{4}=8$.\n由 $a_{3}+a_{5}=20$ 得 $8\\left(q+\\frac{1}{q}\\right)=20$, 因为 $q>1$, 所以 $q=2$.\n\n(2) 设 $c_{n}=\\left(b_{n+1}-b_{n}\\right) a_{n}$, 数列 $\\left\\{c_{n}\\right\\}$ 前 $n$ 项和为 $S_{n}$. 由 $c_{n}=\\left\\{\\begin{array}{l}S_{1}, n=1 \\\\ S_{n}-S_{n-1}, n \\geqslant 2\\end{array}\\right.$, 解得 $c_{n}=4 n-1$.\n\n由(1)可知 $a_{n}=2^{n-1}$, 所以 $b_{n+1}-b_{n}=(4 n-1) \\cdot\\left(\\frac{1}{2}\\right)^{n-1}$,\n\n故 $b_{n}-b_{n-1}=(4 n-5) \\cdot\\left(\\frac{1}{2}\\right)^{n-2}, n \\geqslant 2$,\n\n$b_{n}-b_{1}=\\left(b_{n}-b_{n-1}\\right)+\\left(b_{n-1}-b_{n-2}\\right)+\\cdots+\\left(b_{3}-b_{2}\\right)+\\left(b_{2}-b_{1}\\right)$\n\n$=(4 n-5) \\cdot\\left(\\frac{1}{2}\\right)^{n-2}+(4 n-9) \\cdot\\left(\\frac{1}{2}\\right)^{n-3}+\\cdots+7 \\cdot \\frac{1}{2}+3$.\n\n设 $T_{n}=3+7 \\cdot \\frac{1}{2}+11 \\cdot\\left(\\frac{1}{2}\\right)^{2}+\\cdots+(4 n-5) \\cdot\\left(\\frac{1}{2}\\right)^{n-2}, \\quad n \\geqslant 2$,\n\n$\\frac{1}{2} T_{n}=3 \\cdot \\frac{1}{2}+7 \\cdot\\left(\\frac{1}{2}\\right)^{2}+11 \\cdot\\left(\\frac{1}{2}\\right)^{3}+\\cdots+(4 n-5) \\cdot\\left(\\frac{1}{2}\\right)^{n-1}$\n\n所以 $\\frac{1}{2} T_{n}=3+4 \\cdot \\frac{1}{2}+4 \\cdot\\left(\\frac{1}{2}\\right)^{2}+\\cdots+4 \\cdot\\left(\\frac{1}{2}\\right)^{n-2}-(4 n-5) \\cdot\\left(\\frac{1}{2}\\right)^{n-1}$,\n\n因此 $T_{n}=14-(4 n-3) \\cdot\\left(\\frac{1}{2}\\right)^{n-2}, n \\geqslant 2$,\n\n又 $b_{1}=1$, 所以 $b_{n}=15-(4 n-3) \\cdot\\left(\\frac{1}{2}\\right)^{n-2}$.", "solution": "null", "level": "高二", "question": "16. (2018 浙江) 已知等比数列 $\\left\\{a_{1}\\right\\}$ 的公比 $q>1$, 且 $a_{3}+a_{4}+a_{5}=28, a_{4}+2$ 是 $a_{3}, a_{5}$ 的等差中项. 数\n\n列 $\\left\\{b_{n}\\right\\}$ 满足 $b_{1}=1$, 数列 $\\left\\{\\left(b_{n+1}-b_{n}\\right) a_{n}\\right\\}$ 的前 $n$ 项和为 $2 n^{2}+n$.\n\n(1)求 $q$ 的值;\n\n(2)求数列 $\\left\\{b_{n}\\right\\}$ 的通项公式.", "options": [], "subject": "代数", "analysis": "(1)由 $a_{4}+2$ 是 $a_{3}, a_{5}$ 的等差中项得 $a_{3}+a_{5}=2 a_{4}+4$, 所以 $a_{3}+a_{4}+a_{5}=3 a_{4}+4=28$,解得 $a_{4}=8$.\n由 $a_{3}+a_{5}=20$ 得 $8\\left(q+\\frac{1}{q}\\right)=20$, 因为 $q>1$, 所以 $q=2$.\n\n(2) 设 $c_{n}=\\left(b_{n+1}-b_{n}\\right) a_{n}$, 数列 $\\left\\{c_{n}\\right\\}$ 前 $n$ 项和为 $S_{n}$. 由 $c_{n}=\\left\\{\\begin{array}{l}S_{1}, n=1 \\\\ S_{n}-S_{n-1}, n \\geqslant 2\\end{array}\\right.$, 解得 $c_{n}=4 n-1$.\n\n由(1)可知 $a_{n}=2^{n-1}$, 所以 $b_{n+1}-b_{n}=(4 n-1) \\cdot\\left(\\frac{1}{2}\\right)^{n-1}$,\n\n故 $b_{n}-b_{n-1}=(4 n-5) \\cdot\\left(\\frac{1}{2}\\right)^{n-2}, n \\geqslant 2$,\n\n$b_{n}-b_{1}=\\left(b_{n}-b_{n-1}\\right)+\\left(b_{n-1}-b_{n-2}\\right)+\\cdots+\\left(b_{3}-b_{2}\\right)+\\left(b_{2}-b_{1}\\right)$\n\n$=(4 n-5) \\cdot\\left(\\frac{1}{2}\\right)^{n-2}+(4 n-9) \\cdot\\left(\\frac{1}{2}\\right)^{n-3}+\\cdots+7 \\cdot \\frac{1}{2}+3$.\n\n设 $T_{n}=3+7 \\cdot \\frac{1}{2}+11 \\cdot\\left(\\frac{1}{2}\\right)^{2}+\\cdots+(4 n-5) \\cdot\\left(\\frac{1}{2}\\right)^{n-2}, \\quad n \\geqslant 2$,\n\n$\\frac{1}{2} T_{n}=3 \\cdot \\frac{1}{2}+7 \\cdot\\left(\\frac{1}{2}\\right)^{2}+11 \\cdot\\left(\\frac{1}{2}\\right)^{3}+\\cdots+(4 n-5) \\cdot\\left(\\frac{1}{2}\\right)^{n-1}$\n\n所以 $\\frac{1}{2} T_{n}=3+4 \\cdot \\frac{1}{2}+4 \\cdot\\left(\\frac{1}{2}\\right)^{2}+\\cdots+4 \\cdot\\left(\\frac{1}{2}\\right)^{n-2}-(4 n-5) \\cdot\\left(\\frac{1}{2}\\right)^{n-1}$,\n\n因此 $T_{n}=14-(4 n-3) \\cdot\\left(\\frac{1}{2}\\right)^{n-2}, n \\geqslant 2$,\n\n又 $b_{1}=1$, 所以 $b_{n}=15-(4 n-3) \\cdot\\left(\\frac{1}{2}\\right)^{n-2}$."} {"id": "17844", "image": ["8994.jpg", "8995.jpg", "8995.jpg"], "answer": "解析 : (1)函数 $f(x)$ 的周期 $T=\\frac{2 \\pi}{\\frac{1}{2}}=4 \\pi$.\n\n由 $\\frac{1}{2} x-\\frac{\\pi}{4}=0, \\frac{\\pi}{2} , \\pi, \\frac{3 \\pi}{2} , 2 \\pi$ ,\n\n解得 $x=\\frac{\\pi}{2} , \\frac{3 \\pi}{2} , \\frac{5 \\pi}{2}, \\frac{7 \\pi}{2}, \\frac{9 \\pi}{2}$.\n\n列表如下:\n\n描出五个关键点并光滑连线,得到一个周期的简图,图象如下:\n\n\n\n(2)先把 $y=\\sin x$ 的图象向右平移 $\\frac{\\pi}{4}$ 个单位长度,然后把所有点的横坐标扩大为原来的 2 倍 (纵坐标不变),再把所有点的纵坐标扩大为原来的 3 倍(横坐标不变),得到 $f(x)$ 的图象.", "solution": "null", "level": "高二", "question": "10. 已知函数 $f(x)=3 \\sin \\left(\\frac{1}{2} x-\\frac{\\pi}{4}\\right), x \\in \\mathbf{R}$.\n\n(1)列表并画出函数 $f(x)$ 在长度为一个周期的闭区间上的简图;\n\n(2)将函数 $y=\\sin x$ 的图象作怎样的变换可得到 $f(x)$ 的图象?\n\n", "options": [], "subject": "变换几何", "analysis": "解析 : (1)函数 $f(x)$ 的周期 $T=\\frac{2 \\pi}{\\frac{1}{2}}=4 \\pi$.\n\n由 $\\frac{1}{2} x-\\frac{\\pi}{4}=0, \\frac{\\pi}{2} , \\pi, \\frac{3 \\pi}{2} , 2 \\pi$ ,\n\n解得 $x=\\frac{\\pi}{2} , \\frac{3 \\pi}{2} , \\frac{5 \\pi}{2}, \\frac{7 \\pi}{2}, \\frac{9 \\pi}{2}$.\n\n列表如下:\n\n描出五个关键点并光滑连线,得到一个周期的简图,图象如下:\n\n\n\n(2)先把 $y=\\sin x$ 的图象向右平移 $\\frac{\\pi}{4}$ 个单位长度,然后把所有点的横坐标扩大为原来的 2 倍 (纵坐标不变),再把所有点的纵坐标扩大为原来的 3 倍(横坐标不变),得到 $f(x)$ 的图象."} {"id": "17847", "image": [], "answer": "$2 \\sin \\left(4 x+\\frac{13 \\pi}{12}\\right)-1$", "solution": "null", "level": "高二", "question": "13. 将函数 $f(x)$ 的图象向右平移 $\\frac{\\pi}{3}$ 个单位长度后, 再向上平移 1 个单位长度得函数 $y=2 \\sin \\left(4 x-\\frac{\\pi}{4}\\right)$的图象, 则 $f(x)=$ \\$ \\qquad \\$ .", "options": [], "subject": "变换几何", "analysis": "将 $y=2 \\sin \\left(4 x-\\frac{\\pi}{4}\\right)$ 的图象向左平移 $\\frac{\\pi}{3}$ 个单位长度,得函数 $y=2 \\sin \\left[4\\left(x+\\frac{\\pi}{3}\\right)-\\frac{\\pi}{4}\\right]=$ $2 \\sin \\left(4 x+\\frac{13 \\pi}{12}\\right)$ 的图象,再向下平移 1 个单位长度,得函数 $y=2 \\sin \\left(4 x+\\frac{13 \\pi}{12}\\right)-1$ 的图象,即 $f(x)=2 \\sin \\left(4 x+\\frac{13 \\pi}{12}\\right)-1$.\n\n答案: $2 \\sin \\left(4 x+\\frac{13 \\pi}{12}\\right)-1$"} {"id": "17848", "image": [], "answer": "$f(x)=\\sqrt{3} \\cos 2 x$ 纵坐标伸长到原来的 2 倍,得到 $g(x)=2 \\sqrt{3} \\cos 2 x$ ,向左平移 $\\frac{\\pi}{6}$ 个单位,得到 $g(x)=2 \\sqrt{3} \\cos \\left[2\\left(x+\\frac{\\pi}{6}\\right)\\right]=2 \\sqrt{3} \\cos \\left(2 x+\\frac{\\pi}{3}\\right)$ ,\n$\\therefore g\\left(\\frac{\\pi}{3}\\right)=2 \\sqrt{3} \\cos \\left(\\frac{2}{3} \\pi+\\frac{\\pi}{3}\\right)=-2 \\sqrt{3}$.\n\n答案: $-2 \\sqrt{3}$", "solution": "null", "level": "高二", "question": "14. 将函数 $f(x)=\\sqrt{3} \\cos 2 x$ 的图象纵坐标伸长到原来的 2 倍(横坐标不变), 再向左平移 $\\frac{\\pi}{6}$ 个单位长度后得到函数 $g(x)$ 的图象, 则 $g\\left(\\frac{\\pi}{3}\\right)=$ \\$ \\qquad \\$ .", "options": [], "subject": "变换几何", "analysis": "$f(x)=\\sqrt{3} \\cos 2 x$ 纵坐标伸长到原来的 2 倍,得到 $g(x)=2 \\sqrt{3} \\cos 2 x$ ,向左平移 $\\frac{\\pi}{6}$ 个单位,得到 $g(x)=2 \\sqrt{3} \\cos \\left[2\\left(x+\\frac{\\pi}{6}\\right)\\right]=2 \\sqrt{3} \\cos \\left(2 x+\\frac{\\pi}{3}\\right)$ ,\n$\\therefore g\\left(\\frac{\\pi}{3}\\right)=2 \\sqrt{3} \\cos \\left(\\frac{2}{3} \\pi+\\frac{\\pi}{3}\\right)=-2 \\sqrt{3}$.\n\n答案: $-2 \\sqrt{3}$"} {"id": "17876", "image": [], "answer": "根据题意得 $28=a+A , 18=a+A \\cos \\left[\\frac{\\pi}{6} \\times(12-6)\\right]=a-A$ ,解得 $a=23 , A=5$ ,所以 $y=23+5 \\cos \\left[\\frac{\\pi}{6}(x-6)\\right]$. 令 $x=10$ ,得 $y=23+5 \\cos \\left[\\frac{\\pi}{6} \\times(10-6)\\right]=23+5 \\cos \\frac{2 \\pi}{3}=$ 20.5 .", "solution": "null", "level": "高二", "question": "14. 某城市一年中 12 个月的平均气温 $y$ 与月份 $x$ 的关系可近似地用函数 $y=a+\\operatorname{Acos}\\left[\\frac{\\pi}{6}(x-6)\\right]$ $(x=1,2,3, \\cdots, 12)$ 来表示. 已知 6 月份的月平均气温最高, 为 $28{ }^{\\circ} \\mathrm{C}, 12$ 月份的月平均气温最低, 为 $18{ }^{\\circ} \\mathrm{C}$, 则 10 月份的平均气温为 \\$ \\qquad \\$ ${ }^{\\circ} \\mathrm{C}$.", "options": [], "subject": "计数", "analysis": "解析: 根据题意得 $28=a+A , 18=a+A \\cos \\left[\\frac{\\pi}{6} \\times(12-6)\\right]=a-A$ ,解得 $a=23 , A=5$ ,所以 $y=23+5 \\cos \\left[\\frac{\\pi}{6}(x-6)\\right]$. 令 $x=10$ ,得 $y=23+5 \\cos \\left[\\frac{\\pi}{6} \\times(10-6)\\right]=23+5 \\cos \\frac{2 \\pi}{3}=$ 20.5 ."} {"id": "17995", "image": [], "answer": "设 $\\overrightarrow{O A}=\\boldsymbol{a}, \\overrightarrow{O B}=\\boldsymbol{b}, \\overrightarrow{O C}=\\boldsymbol{c} ,$\n\n则 $\\overrightarrow{B C}=\\boldsymbol{c}-\\boldsymbol{b}, \\overrightarrow{C A}=\\boldsymbol{a}-\\boldsymbol{c}, \\overrightarrow{A B}=\\boldsymbol{b}-\\boldsymbol{a}$.\n\n$\\because|\\overrightarrow{O A}|^{2}+|\\overrightarrow{B C}|^{2}=|\\overrightarrow{O B}|^{2}+|\\overrightarrow{C A}|^{2}=|\\overrightarrow{O C}|^{2}+|\\overrightarrow{A B}|^{2}$,\n\n$\\therefore a^{2}+(c-b)^{2}=b^{2}+(a-c)^{2}=c^{2}+(b-a)^{2}$.\n\n$\\therefore c \\cdot b=a \\cdot c=b \\cdot a$.\n\n故 $\\overrightarrow{A B} \\cdot \\overrightarrow{O C}=(\\boldsymbol{b}-\\boldsymbol{a}) \\cdot \\boldsymbol{c}=\\boldsymbol{b} \\cdot \\boldsymbol{c}-\\boldsymbol{a} \\cdot \\boldsymbol{c}=0$,\n\n$\\overrightarrow{B C} \\cdot \\overrightarrow{O A}=(\\boldsymbol{c}-\\boldsymbol{b}) \\cdot \\boldsymbol{a}=\\boldsymbol{c} \\cdot \\boldsymbol{a}-\\boldsymbol{b} \\cdot \\boldsymbol{a}=0$.\n\n$\\therefore \\overrightarrow{A B} \\perp \\overrightarrow{O C} , \\overrightarrow{B C} \\perp \\overrightarrow{O A}$ ,即 $A B \\perp O C , B C \\perp O A$.\n\n$\\therefore$ 点 $O$ 是 $\\triangle A B C$ 的垂心.", "solution": "null", "level": "高二", "question": "15. 已知 $O$ 为 $\\triangle A B C$ 所在平面内一点, 且满足 $|\\overrightarrow{O A}|^{2}+|\\overrightarrow{B C}|^{2}=|\\overrightarrow{O B}|^{2}+|\\overrightarrow{C A}|^{2}=|\\overrightarrow{O C}|^{2}+|\\overrightarrow{A B}|^{2}$. 求证:点 $O$ 是 $\\triangle A B C$ 的垂心.", "options": [], "subject": "立体几何学", "analysis": "设 $\\overrightarrow{O A}=\\boldsymbol{a}, \\overrightarrow{O B}=\\boldsymbol{b}, \\overrightarrow{O C}=\\boldsymbol{c} ,$\n\n则 $\\overrightarrow{B C}=\\boldsymbol{c}-\\boldsymbol{b}, \\overrightarrow{C A}=\\boldsymbol{a}-\\boldsymbol{c}, \\overrightarrow{A B}=\\boldsymbol{b}-\\boldsymbol{a}$.\n\n$\\because|\\overrightarrow{O A}|^{2}+|\\overrightarrow{B C}|^{2}=|\\overrightarrow{O B}|^{2}+|\\overrightarrow{C A}|^{2}=|\\overrightarrow{O C}|^{2}+|\\overrightarrow{A B}|^{2}$,\n\n$\\therefore a^{2}+(c-b)^{2}=b^{2}+(a-c)^{2}=c^{2}+(b-a)^{2}$.\n\n$\\therefore c \\cdot b=a \\cdot c=b \\cdot a$.\n\n故 $\\overrightarrow{A B} \\cdot \\overrightarrow{O C}=(\\boldsymbol{b}-\\boldsymbol{a}) \\cdot \\boldsymbol{c}=\\boldsymbol{b} \\cdot \\boldsymbol{c}-\\boldsymbol{a} \\cdot \\boldsymbol{c}=0$,\n\n$\\overrightarrow{B C} \\cdot \\overrightarrow{O A}=(\\boldsymbol{c}-\\boldsymbol{b}) \\cdot \\boldsymbol{a}=\\boldsymbol{c} \\cdot \\boldsymbol{a}-\\boldsymbol{b} \\cdot \\boldsymbol{a}=0$.\n\n$\\therefore \\overrightarrow{A B} \\perp \\overrightarrow{O C} , \\overrightarrow{B C} \\perp \\overrightarrow{O A}$ ,即 $A B \\perp O C , B C \\perp O A$.\n\n$\\therefore$ 点 $O$ 是 $\\triangle A B C$ 的垂心."} {"id": "17996", "image": ["9038.jpg", "9039.jpg", "9039.jpg"], "answer": "如图,建立平面直角坐标系,设正方形的边长为 1 ,则 $A(-1 , 1) , B(0 , 1)$.\n\n\n\n设 $E(x, y)$ ,则 $\\overrightarrow{B E}=(x, y-1) , \\overrightarrow{A C}=(1 ,-1)$.\n\n又 $\\because \\overrightarrow{A C} / / \\overrightarrow{B E}, \\therefore x(-1)-1 \\times(y-1)=0$ ,\n\n$\\therefore x+y-1=0$.\n\n又 $\\because|\\overrightarrow{C E}|=|\\overrightarrow{A C}|, \\quad \\therefore x^{2}+y^{2}-2=0$.\n由 $\\left\\{\\begin{array}{l}x^{2}+y^{2}-2=0 , \\\\ x+y-1=0 ,\\end{array}\\right.$ 得 $\\left\\{\\begin{array}{l}x=\\frac{1+\\sqrt{3}}{2} , \\\\ y=\\frac{1-\\sqrt{3}}{2}\\end{array}\\right.$ 或 $\\left\\{\\begin{array}{l}x=\\frac{1-\\sqrt{3}}{2} , \\\\ y=\\frac{1+\\sqrt{3}}{2}\\end{array}\\right.$ (舍).\n\n$\\therefore E\\left(\\frac{1+\\sqrt{3}}{2}, \\frac{1-\\sqrt{3}}{2}\\right)$.\n\n又设 $F\\left(x^{\\prime} , 1\\right)$ ,由 $\\overrightarrow{C F}=\\left(x^{\\prime} , 1\\right)$ 和 $\\overrightarrow{C E}=\\left(\\frac{1+\\sqrt{3}}{2} , \\frac{1-\\sqrt{3}}{2}\\right)$ 共线得 $\\frac{1-\\sqrt{3}}{2} x^{\\prime}-\\frac{1+\\sqrt{3}}{2}=0$ ,得 $x^{\\prime}$\n\n$=-2-\\sqrt{3}$,\n\n$\\therefore F(-2-\\sqrt{3}, 1)$,\n\n$\\therefore \\overrightarrow{A F}=(-1-\\sqrt{3}, 0), \\overrightarrow{A E}=\\left(\\frac{3+\\sqrt{3}}{2},-\\frac{1+\\sqrt{3}}{2}\\right)$,\n\n$\\therefore|\\overrightarrow{A E}|=\\sqrt{\\left(\\frac{3+\\sqrt{3}}{2}\\right)^{2}+\\left(\\frac{-1-\\sqrt{3}}{2}\\right)^{2}}=1+\\sqrt{3}=|\\overrightarrow{A F}|$,\n\n$\\therefore A F=A E$.", "solution": "null", "level": "高二", "question": "16.如图所示, 四边形 $A B C D$ 是正方形, $B E / / A C, A C=C E, E C$ 的延长线交 $B A$ 的延长线于点 $F$.求证: $A F=A E$.\n\n", "options": [], "subject": "组合几何学", "analysis": "如图,建立平面直角坐标系,设正方形的边长为 1 ,则 $A(-1 , 1) , B(0 , 1)$.\n\n\n\n设 $E(x, y)$ ,则 $\\overrightarrow{B E}=(x, y-1) , \\overrightarrow{A C}=(1 ,-1)$.\n\n又 $\\because \\overrightarrow{A C} / / \\overrightarrow{B E}, \\therefore x(-1)-1 \\times(y-1)=0$ ,\n\n$\\therefore x+y-1=0$.\n\n又 $\\because|\\overrightarrow{C E}|=|\\overrightarrow{A C}|, \\quad \\therefore x^{2}+y^{2}-2=0$.\n由 $\\left\\{\\begin{array}{l}x^{2}+y^{2}-2=0 , \\\\ x+y-1=0 ,\\end{array}\\right.$ 得 $\\left\\{\\begin{array}{l}x=\\frac{1+\\sqrt{3}}{2} , \\\\ y=\\frac{1-\\sqrt{3}}{2}\\end{array}\\right.$ 或 $\\left\\{\\begin{array}{l}x=\\frac{1-\\sqrt{3}}{2} , \\\\ y=\\frac{1+\\sqrt{3}}{2}\\end{array}\\right.$ (舍).\n\n$\\therefore E\\left(\\frac{1+\\sqrt{3}}{2}, \\frac{1-\\sqrt{3}}{2}\\right)$.\n\n又设 $F\\left(x^{\\prime} , 1\\right)$ ,由 $\\overrightarrow{C F}=\\left(x^{\\prime} , 1\\right)$ 和 $\\overrightarrow{C E}=\\left(\\frac{1+\\sqrt{3}}{2} , \\frac{1-\\sqrt{3}}{2}\\right)$ 共线得 $\\frac{1-\\sqrt{3}}{2} x^{\\prime}-\\frac{1+\\sqrt{3}}{2}=0$ ,得 $x^{\\prime}$\n\n$=-2-\\sqrt{3}$,\n\n$\\therefore F(-2-\\sqrt{3}, 1)$,\n\n$\\therefore \\overrightarrow{A F}=(-1-\\sqrt{3}, 0), \\overrightarrow{A E}=\\left(\\frac{3+\\sqrt{3}}{2},-\\frac{1+\\sqrt{3}}{2}\\right)$,\n\n$\\therefore|\\overrightarrow{A E}|=\\sqrt{\\left(\\frac{3+\\sqrt{3}}{2}\\right)^{2}+\\left(\\frac{-1-\\sqrt{3}}{2}\\right)^{2}}=1+\\sqrt{3}=|\\overrightarrow{A F}|$,\n\n$\\therefore A F=A E$."} {"id": "18008", "image": [], "answer": "12", "solution": "null", "level": "高二", "question": "15. 如图所示, 平行四边形 $A B C D$ 中, $O$ 是两对角线 $A C, B D$ 的交点, 设点集 $S=\\{A, B, C, D$, $0\\}$, 向量集合 $T=\\{\\overrightarrow{M N} \\mid M, N \\in S$, 且 $M, N$ 不重合 $\\}$, 试求集合 $T$ 中元素的个数.\n\n![](https://cdn.mathpix.com/cropped/2024_04_19_f934fd3200dbf4af6dd7g-135,.jpg?height=206&width=488&top_left_y=751&top_left_x=818)", "options": [], "subject": "组合数学", "analysis": "由题可知, 集合 $T$ 中的元素实质上是 $S$ 中任意两点连成的有向线段, 共有 20 个,即 $\\overrightarrow{A B}, \\overrightarrow{A C}, \\overrightarrow{A D}, \\overrightarrow{A O}, \\overrightarrow{B A}, \\overrightarrow{B C}, \\overrightarrow{B D}, \\overrightarrow{B O}, \\overrightarrow{C A}, \\overrightarrow{C B}, \\overrightarrow{C D}, \\overrightarrow{C O}, \\overrightarrow{D A}, \\overrightarrow{D B}, \\overrightarrow{D C}, \\overrightarrow{D O}, \\overrightarrow{O A}, \\overrightarrow{O B}, \\overrightarrow{O C}$, $\\overrightarrow{O D}$. 由平行四边形的性质可知, 共有 8 对向量相等, 即 $\\overrightarrow{A B}=\\overrightarrow{D C}, \\overrightarrow{A D}=\\overrightarrow{B C}, \\overrightarrow{D A}=\\overrightarrow{C B}, \\overrightarrow{B A}=\\overrightarrow{C D}$, $\\overrightarrow{A O}=\\overrightarrow{O C}, \\overrightarrow{O A}=\\overrightarrow{C O}, \\overrightarrow{D O}=\\overrightarrow{O B}, \\overrightarrow{O D}=\\overrightarrow{B O}$. 又集合元素具有互异性, 故集合 $T$ 中的元素共有 12 个."} {"id": "19893", "image": ["9280.jpg", "9280.jpg"], "answer": "平面区域见解析, 面积为 10\n\n 解析 $x-y-2 \\leq 0$ 不等式组表示直线 $x-y-2=0$ 左上方的点和该直线, $x+y+3 \\leq 0$ 表示直线 $x+y+3=0$ 左下方的点和该直线, 由 $\\left\\{\\begin{array}{l}x-y-2=0, \\\\ x+y+3=0,\\end{array}\\right.$ 得 $C\\left(-\\frac{1}{2},-\\frac{5}{2}\\right)$. 由不等式 $1 \\leq|x+3| \\leq 2$ 可得 $-2 \\leq x \\leq-1$ 或 $-5 \\leq x \\leq-4$, 则不等式组表示的区域如图阴影部分.\n\n\n\n由 $\\left\\{\\begin{array}{l}x=-1, \\\\ x+y+3=0,\\end{array}\\right.$ 得点 $(-1,-2)$. 由 $\\left\\{\\begin{array}{l}x=-1, \\\\ x-y-2=0,\\end{array}\\right.$ 得 $(-1,-3)$. 同样的可以求出直线 $x-y-2=0$, $x+y+3=0$ 与 $x=-2$ 的交点为 $(-2,-4),(-2,-1)$, 所以小梯形的面积为 $S_{1}=\\frac{1}{2}[(-2)-(-3)+(-1)-(-4)] \\times 1=2$, 同理可以求出大梯形的面积 $S_{2}=8$, 所以不等式组围成的平面区域的面积为 $S=S_{1}+S_{2}=2+8=10$.", "solution": "null", "level": "高二", "question": "画出不等式组 $\\left\\{\\begin{array}{l}x-y-2 \\leq 0, \\\\ x+y+3 \\leq 0, \\\\ 1 \\leq|x+3| \\leq 2\\end{array}\\right.$ 表示的平面区域, 并求其面积.", "options": [], "subject": "画法几何学", "analysis": "平面区域见解析, 面积为 10\n\n 解析 $x-y-2 \\leq 0$ 不等式组表示直线 $x-y-2=0$ 左上方的点和该直线, $x+y+3 \\leq 0$ 表示直线 $x+y+3=0$ 左下方的点和该直线, 由 $\\left\\{\\begin{array}{l}x-y-2=0, \\\\ x+y+3=0,\\end{array}\\right.$ 得 $C\\left(-\\frac{1}{2},-\\frac{5}{2}\\right)$. 由不等式 $1 \\leq|x+3| \\leq 2$ 可得 $-2 \\leq x \\leq-1$ 或 $-5 \\leq x \\leq-4$, 则不等式组表示的区域如图阴影部分.\n\n\n\n由 $\\left\\{\\begin{array}{l}x=-1, \\\\ x+y+3=0,\\end{array}\\right.$ 得点 $(-1,-2)$. 由 $\\left\\{\\begin{array}{l}x=-1, \\\\ x-y-2=0,\\end{array}\\right.$ 得 $(-1,-3)$. 同样的可以求出直线 $x-y-2=0$, $x+y+3=0$ 与 $x=-2$ 的交点为 $(-2,-4),(-2,-1)$, 所以小梯形的面积为 $S_{1}=\\frac{1}{2}[(-2)-(-3)+(-1)-(-4)] \\times 1=2$, 同理可以求出大梯形的面积 $S_{2}=8$, 所以不等式组围成的平面区域的面积为 $S=S_{1}+S_{2}=2+8=10$."} {"id": "20151", "image": [], "answer": ":解 设 $a=2 m-1, b=2 n-1, c=2 p-1(m, n, p \\in \\mathbf{Z})$,则 $b^{2}-4 a c=(2 n-1)^{2}-4(2 m-1)(2 p-1)$\n\n$=4\\left[n^{2}-n-(2 m-1)(2 p-1)\\right]+1$ 为奇数.\n\n$\\therefore b^{2}-4 a c \\neq 0$.\n$\\therefore$ 方程 $a x^{2}+b x+c=0(a \\neq 0)$ 没有等根.\n\n即原命题是真命题.\n\n它的逆否命题 “若方程 $a x^{2}+b x+c=0(a \\neq 0)$ 有等根, 则 $a, b, c$不全为奇数”也是真命题.\n\n它的逆命题为 “若方程 $a x^{2}+b x+c=0(a \\neq 0)$ 没有等根, 则 $a, b$, $c$ 均为奇数”\n\n当 $a=1, b=0, c=-1$ 时, 方程 $x^{2}-1=0$ 没有等根, 其中 $b=0$不是奇数.\n\n所以它的逆命题是假命题.\n\n它的否命题 “如果 $a, b, c$ 不全为奇数, 那么方程 $a x^{2}+b x+c=$ $0(a \\neq 0)$ 有等根” 也是假命题.", "solution": "null", "level": "高二", "question": "10. 设命题 “如果 $a, b, c$ 均为奇数, 那么方程 $a x^{2}+b x+c=0(a \\neq 0)$没有等根” . 试判断它的四种命题的真假.", "options": [], "subject": "逻辑题", "analysis": ":解 设 $a=2 m-1, b=2 n-1, c=2 p-1(m, n, p \\in \\mathbf{Z})$,则 $b^{2}-4 a c=(2 n-1)^{2}-4(2 m-1)(2 p-1)$\n\n$=4\\left[n^{2}-n-(2 m-1)(2 p-1)\\right]+1$ 为奇数.\n\n$\\therefore b^{2}-4 a c \\neq 0$.\n$\\therefore$ 方程 $a x^{2}+b x+c=0(a \\neq 0)$ 没有等根.\n\n即原命题是真命题.\n\n它的逆否命题 “若方程 $a x^{2}+b x+c=0(a \\neq 0)$ 有等根, 则 $a, b, c$不全为奇数”也是真命题.\n\n它的逆命题为 “若方程 $a x^{2}+b x+c=0(a \\neq 0)$ 没有等根, 则 $a, b$, $c$ 均为奇数”\n\n当 $a=1, b=0, c=-1$ 时, 方程 $x^{2}-1=0$ 没有等根, 其中 $b=0$不是奇数.\n\n所以它的逆命题是假命题.\n\n它的否命题 “如果 $a, b, c$ 不全为奇数, 那么方程 $a x^{2}+b x+c=$ $0(a \\neq 0)$ 有等根” 也是假命题."} {"id": "20158", "image": [], "answer": ":解 原命题的逆否命题为:\n\n已知 $a, x \\in \\mathbf{R}$, 如果 $a<1$, 那么关于 $x$ 的不等式 $x^{2}+(2 a+1) x+$ $a^{2}+2 \\leqslant 0$ 的解集为空集.\n\n判断如下:\n\n抛物线 $y=x^{2}+(2 a+1) x+a^{2}+2$ 开口向上,\n\n判别式 $\\Delta=(2 a+1)^{2}-4\\left(a^{2}+2\\right)=4 a-7$\n\n$\\because a<1, \\quad \\therefore 4 a-7<0$.\n\n即抛物线 $y=x^{2}+(2 a+1) x+a^{2}+2$ 与 $x$ 轴无交点.\n\n$\\therefore$ 关于 $x$ 的不等式 $x^{2}+(2 a+1) x+a^{2}+2 \\leqslant 0$ 的解集为空集.\n\n故逆否命题为真.", "solution": "null", "level": "高二", "question": "9. 判断命题 “已知 $a, x \\in \\mathbf{R}$, 若关于 $x$ 的不等式 $x^{2}+(2 a+1) x$ $+a^{2}+2 \\leqslant 0$ 的解集非空, 则 $a \\geqslant 1$ ” 的逆否命题的真假.", "options": [], "subject": "逻辑题", "analysis": ":解 原命题的逆否命题为:\n\n已知 $a, x \\in \\mathbf{R}$, 如果 $a<1$, 那么关于 $x$ 的不等式 $x^{2}+(2 a+1) x+$ $a^{2}+2 \\leqslant 0$ 的解集为空集.\n\n判断如下:\n\n抛物线 $y=x^{2}+(2 a+1) x+a^{2}+2$ 开口向上,\n\n判别式 $\\Delta=(2 a+1)^{2}-4\\left(a^{2}+2\\right)=4 a-7$\n\n$\\because a<1, \\quad \\therefore 4 a-7<0$.\n\n即抛物线 $y=x^{2}+(2 a+1) x+a^{2}+2$ 与 $x$ 轴无交点.\n\n$\\therefore$ 关于 $x$ 的不等式 $x^{2}+(2 a+1) x+a^{2}+2 \\leqslant 0$ 的解集为空集.\n\n故逆否命题为真."} {"id": "20175", "image": [], "answer": ":解 (1)綈 $p: \\forall x, y \\in \\mathbf{Z}, \\sqrt{2} x+y \\neq 3$,\n\n当 $x=0, y=3$ 时, $\\sqrt{2} x+y=3$,\n\n因此綈 $p$ 是假命题.\n\n(2)綈 $q: \\exists x \\in \\mathbf{R}, x^{2}+x-4 \\leqslant 0$,\n当 $x=0$ 时, $x^{2}+x-4=-4 \\leqslant 0$,\n\n因此綈 $q$ 是真命题.", "solution": "null", "level": "高二", "question": "8. 写出下列命题的否定, 并判断其真假.\n\n(1)p: $\\exists x_{0}, y_{0} \\in \\mathbf{Z}$, 使得 $\\sqrt{2} x_{0}+y_{0}=3$;\n\n(2) $q: \\forall x \\in \\mathbf{R}, x^{2}+x-4>0$.", "options": [], "subject": "逻辑题", "analysis": ":解 (1)綈 $p: \\forall x, y \\in \\mathbf{Z}, \\sqrt{2} x+y \\neq 3$,\n\n当 $x=0, y=3$ 时, $\\sqrt{2} x+y=3$,\n\n因此綈 $p$ 是假命题.\n\n(2)綈 $q: \\exists x \\in \\mathbf{R}, x^{2}+x-4 \\leqslant 0$,\n当 $x=0$ 时, $x^{2}+x-4=-4 \\leqslant 0$,\n\n因此綈 $q$ 是真命题."} {"id": "20176", "image": [], "answer": ":解 $\\because$ “存在 $x \\in \\mathbf{R}$, 使 $2 x^{2}-3 a x+9<0$ ” 为假命题.\n\n$\\therefore$ 它的否定 “对任意的 $x \\in \\mathbf{R}, 2 x^{2}-3 a x+9 \\geqslant 0$ ” 为真命题.\n\n$\\therefore$ 只要 $\\Delta=9 a^{2}-4 \\times 2 \\times 9 \\leqslant 0$ 即可.\n\n解得 $-2 \\sqrt{2} \\leqslant a \\leqslant 2 \\sqrt{2}$.\n\n故 $a$ 的取值范围是 $[-2 \\sqrt{2}, 2 \\sqrt{2}]$.", "solution": "null", "level": "高二", "question": "9. 命题 “存在 $x \\in \\mathbf{R}$, 使 $2 x^{2}-3 a x+9<0$ ” 为假命题, 求实数 $a$的取值范围.", "options": [], "subject": "逻辑题", "analysis": ":解 $\\because$ “存在 $x \\in \\mathbf{R}$, 使 $2 x^{2}-3 a x+9<0$ ” 为假命题.\n\n$\\therefore$ 它的否定 “对任意的 $x \\in \\mathbf{R}, 2 x^{2}-3 a x+9 \\geqslant 0$ ” 为真命题.\n\n$\\therefore$ 只要 $\\Delta=9 a^{2}-4 \\times 2 \\times 9 \\leqslant 0$ 即可.\n\n解得 $-2 \\sqrt{2} \\leqslant a \\leqslant 2 \\sqrt{2}$.\n\n故 $a$ 的取值范围是 $[-2 \\sqrt{2}, 2 \\sqrt{2}]$."} {"id": "20202", "image": [], "answer": ":解 $p: x>1$ 或 $x<-\\frac{1}{5}$, 则綈 $p:-\\frac{1}{5} \\leqslant x \\leqslant 1$.\n\n$q: \\frac{1}{x^{2}+4 x-5}>0$, 即 $x^{2}+4 x-5>0$, 则綈 $q: x^{2}+4 x-5 \\leqslant 0$, 即一 $5 \\leqslant x \\leqslant 1$.\n\n$\\because\\left\\{x \\left\\lvert\\,-\\frac{1}{5} \\leqslant x \\leqslant 1\\right.\\right\\} \\quad\\{x \\mid-5 \\leqslant x \\leqslant 1\\}$,\n\n$\\therefore$ 綈 $p$ 是綈 $q$ 的充分不必要条件.", "solution": "null", "level": "高二", "question": "10. 已知 $p: x>1$ 或 $x<-\\frac{1}{5}, q: \\frac{1}{x^{2}+4 x-5}>0$, 则綈 $p$ 是綈 $q$ 的什么条件?", "options": [], "subject": "逻辑题", "analysis": ":解 $p: x>1$ 或 $x<-\\frac{1}{5}$, 则綈 $p:-\\frac{1}{5} \\leqslant x \\leqslant 1$.\n\n$q: \\frac{1}{x^{2}+4 x-5}>0$, 即 $x^{2}+4 x-5>0$, 则綈 $q: x^{2}+4 x-5 \\leqslant 0$, 即一 $5 \\leqslant x \\leqslant 1$.\n\n$\\because\\left\\{x \\left\\lvert\\,-\\frac{1}{5} \\leqslant x \\leqslant 1\\right.\\right\\} \\quad\\{x \\mid-5 \\leqslant x \\leqslant 1\\}$,\n\n$\\therefore$ 綈 $p$ 是綈 $q$ 的充分不必要条件."} {"id": "20208", "image": ["9310.jpg", "9310.jpg"], "answer": ":解 由 $\\lg \\left(x^{2}-2 x-2\\right) \\geqslant 0$, 得 $x^{2}-2 x-2 \\geqslant 1$,\n\n$\\therefore x \\geqslant 3$, 或 $x \\leqslant-1$. 即 $p: x \\geqslant 3$, 或 $x \\leqslant-1$.\n\n$\\therefore$ 綈 $p:-1\n\n得 $x \\geqslant 4$, 或 $x \\leqslant-1$;\n\n当 $p$ 假 $q$ 真时, 由 $\\left\\{\\begin{array}{l}-1\n\n得 $x \\geqslant 4$, 或 $x \\leqslant-1$;\n\n当 $p$ 假 $q$ 真时, 由 $\\left\\{\\begin{array}{l}-1", "solution": "null", "level": "七年级", "question": "如图, 一只甲虫在 $5 \\times 5$ 的方格 (每小格边长为 1 ) 上沿着网格线运动. 它从 $\\mathrm{A}$ 处出发看望 $\\mathrm{B} 、$ C、 $\\mathrm{D}$ 处的其他甲虫, 规定:向上向右走为正,向下向左走为负,如果从 A 到 B 记为: $\\mathrm{A} \\rightarrow \\mathrm{B}(+1$, $+4)$, 从 $\\mathrm{B}$ 到 $\\mathrm{A}$ 记为: $\\mathrm{B} \\rightarrow \\mathrm{A}(-1,-4)$, 其中第一个数表示左右方向, 第二个数表示上下方向,那么图中.\n\n\n(2)若这只甲虫的行走路线为 $\\mathrm{A} \\rightarrow \\mathrm{B} \\rightarrow \\mathrm{C} \\rightarrow \\mathrm{D}$ ,请计算该甲虫走过的路程;\n\n(3)若这只甲虫从 $\\mathrm{A}$ 处去甲虫 $\\mathrm{P}$ 处的行走路线依次为 $(+2,+2),(+1,-1),(-2,+3),(-$ $1,-2)$, 请在图中标出 $\\mathrm{P}$ 的位置.", "options": [], "subject": "组合数学", "analysis": "(1) 规定:向上向右走为正,向下向左走为负,\n\n则 $\\mathrm{A} \\rightarrow \\mathrm{C}(+3,+4), \\mathrm{B} \\rightarrow \\mathrm{D}(+3,-2), \\mathrm{C} \\rightarrow \\mathrm{D}(+1,-2)$,\n\n故答案为: $+3,+4 ;+3,-2 ; \\mathrm{D},-2$"} {"id": "1352", "image": [], "answer": "(1) $9 ;-6$\n\n(2) 解: 前 $\\mathrm{m}$ 个格子中所填整数之和可能为 2023 ,\n\n$\\because 9+(-6)+2=5,2023 \\div 5=404 \\ldots 3,9+(-6)=3$,\n\n故前 $\\mathrm{m}$ 个格子中所填数字之和可能为 2023 ;\n\n$\\mathrm{m}$ 的值为: $404 \\times 3+2=1214$\n\n(3) 解: 由于是三个数重复出现, 那么前 7 个格子中, 这三个数中, 9 出现了三次,\n\n-6 和 2 都出现了 2 次,\n\n原式 $=[|9-(-6)| \\times 2+|9-2| \\times 2] \\times 3+[|-6-9| \\times 3+|-6-2| \\times 2] \\times 2+[|2-9| \\times 3+|2-(-6)| \\times 2] \\times 2$ $=[15 \\times 2+14] \\times 3+[15 \\times 3+16] \\times 2+[21+16] \\times 2=328$.", "solution": "null", "level": "七年级", "question": "如图, 从左到右, 在每个小格子都填入一个整数, 使得其中任意三个相邻格子中所填整数之和都相等.\n\n| 9 | $\\&$ | $\\#$ | $\\mathrm{x}$ | -6 | | | | 2 | | $\\ldots$ |\n| :--- | :--- | :--- | :--- | :--- | :--- | :--- | :--- | :--- | :--- | :--- |\n\n(1) 可求得 $x=$ \\$ \\qquad \\$ , 第 2021 个格子中的数为 \\$ \\qquad \\$ ;\n\n(2)判断: 前 $m$ 个格子中所填整数之和是否可能为 2023? 若能, 求出 $m$ 的值; 若不能, 请说明理由.\n\n(3)如果 $\\mathrm{a} 、 \\mathrm{~b}$ 为前三个格子中的任意两个数, 那么所有的 $|\\mathrm{a}-\\mathrm{b}|$ 的和可以通过计算: $|9-\\&|+\\mid 9-$ $\\#+|\\&-\\#++\\&-9|+|\\#-9|+|\\#-\\&|$ 得到, 若 $\\mathrm{a}, \\mathrm{b}$ 为前 7 个格子中的任意两个数, 则所有的 $|\\mathrm{a}-\\mathrm{b}|$ 的和为多少?", "options": [], "subject": "组合数学", "analysis": "(1) $\\because$ 任意三个相邻格子中所填整数之和都相等,\n\n$\\therefore 9+\\&+\\#=\\&+\\#+x=\\#+x+(-6)$,\n\n$\\therefore \\mathrm{x}=9, \\&=-6$,\n\n由格子中后面有个数字 2, 可知\\#=2,\n\n故这个表格中的数据以 $9,-6,2$ 循环出现,\n\n$\\because 2021 \\div 3=674 \\ldots 2$,\n\n$\\therefore$ 第 2021 个格子中的数为 -6,\n\n故答案为: $9,-6$;"} {"id": "1354", "image": [], "answer": "(1) 解: 数组 $1,2,3,4$ 是“运算平衡”数组, $+1-2-3+4=0$;\n\n(2)解: 要使数组 $1,4,6, m$ 是“运算平衡”数组, 有以下情况: $1+4+6+m=0 ;-1+4+6+m=0 ; 1-4+6+m=0 ; \\quad 1+4-6+m=0 ; 1+4+6-m=0 ;-1-4+6+m=0 ;-1+4-6+m=0 ;-$ $1+4+6-\\mathrm{m}=0 ; \\quad 1-4-6+\\mathrm{m}=0 ; \\quad 1-4+6-\\mathrm{m}=0 ; \\quad 1+4-6-\\mathrm{m}=0 ; \\quad-1-4-6+\\mathrm{m}=0 ; \\quad-1-4+6-\\mathrm{m}=0, \\quad-1+4-6-\\mathrm{m}=0, \\quad 1-4-6-$ $\\mathrm{m}=0 ;-1-4-6-\\mathrm{m}=0$; 共 16 中情况,\n\n经计算得 $\\mathrm{m}= \\pm 1, \\pm 3, \\pm 9, \\pm 11$;\n\n(3) 解: 这 $n$ 个整数互不相同, 在这 $n$ 个数字前任意添加“+”或“-”号后运算结果为 0 .", "solution": "null", "level": "七年级", "question": "将 $n$ 个互不相同的整数置于一排, 构成一个数组. 在这 $n$ 个数字前任意添加“+”或“-”号, 可以得到一个算式. 若运算结果可以为 0 , 我们就将这个数组称为“运算平衡”数组.\n\n(1)数组 $1,2,3,4$ 是否是“运算平衡”数组? 若是, 请在以下数组中填上相应的符号, 并完成运算;\n\n$1234=$\n\n(2) 若数组 $1,4,6, m$ 是“运算平衡”数组, 则 $m$ 的值可以是多少?\n\n(3)若某“运算平衡”数组中共含有 $n$ 个整数, 则这 $n$ 个整数需要具备什么样的规律?", "options": [], "subject": "组合数学", "analysis": "(1) 解: 数组 $1,2,3,4$ 是“运算平衡”数组, $+1-2-3+4=0$;\n\n(2)解: 要使数组 $1,4,6, m$ 是“运算平衡”数组, 有以下情况: $1+4+6+m=0 ;-1+4+6+m=0 ; 1-4+6+m=0 ; \\quad 1+4-6+m=0 ; 1+4+6-m=0 ;-1-4+6+m=0 ;-1+4-6+m=0 ;-$ $1+4+6-\\mathrm{m}=0 ; \\quad 1-4-6+\\mathrm{m}=0 ; \\quad 1-4+6-\\mathrm{m}=0 ; \\quad 1+4-6-\\mathrm{m}=0 ; \\quad-1-4-6+\\mathrm{m}=0 ; \\quad-1-4+6-\\mathrm{m}=0, \\quad-1+4-6-\\mathrm{m}=0, \\quad 1-4-6-$ $\\mathrm{m}=0 ;-1-4-6-\\mathrm{m}=0$; 共 16 中情况,\n\n经计算得 $\\mathrm{m}= \\pm 1, \\pm 3, \\pm 9, \\pm 11$;\n\n(3) 解: 这 $n$ 个整数互不相同, 在这 $n$ 个数字前任意添加“+”或“-”号后运算结果为 0 ."} {"id": "1301", "image": [], "answer": "(1) 解: $(-0.8) \\times 1+(-0.5) \\times 4+(-0.3) \\times 2+0 \\times 3+0.4 \\times 2+0.5 \\times 8$, $=-0.8-2-0.6+0+0.8+4$ $=1.4$ (千克),所以这 20 筐药材总计超过 1.4 千克.\n\n(2) 解: $(10 \\times 20+1.4) \\times 15$,\n\n$=201.4 \\times 15$,\n\n$=3021($ 元 $)$,\n\n所以这 20 筐药材可卖 3021 元.", "solution": "null", "level": "七年级", "question": "城固资源富集, 享有“天然药库”的美誉, 现有 20 筐药材, 以每筐 10 千克为标准质量, 超过的质量用正数表示,不足的质量用负数表示,结果记录如下:\n\n| 与标准质量的差值(单位: 千克) | -0.8 | -0.5 | -0.3 | 0 | 0.4 | 0.5 |\n| :--- | :--- | :--- | :--- | :--- | :--- | :--- |\n| 筐数 | 1 | 4 | 2 | 3 | 2 | 8 |\n\n(1)与标准质量相比, 这 20 筐药材总计超过或不足多少千克?\n\n(2)若这些药材平均以每千克 15 元的价格出售, 则这 20 筐药材可卖多少元?", "options": [], "subject": "计数", "analysis": "(1) 解: $(-0.8) \\times 1+(-0.5) \\times 4+(-0.3) \\times 2+0 \\times 3+0.4 \\times 2+0.5 \\times 8$, $=-0.8-2-0.6+0+0.8+4$ $=1.4$ (千克),所以这 20 筐药材总计超过 1.4 千克.\n\n(2) 解: $(10 \\times 20+1.4) \\times 15$,\n\n$=201.4 \\times 15$,\n\n$=3021($ 元 $)$,\n\n所以这 20 筐药材可卖 3021 元."} {"id": "2416", "image": ["3511.jpg", "3512.jpg", "3513.jpg", "3513.jpg"], "answer": "解: (1) $\\because$ 结合折线图与扇形图得出参加乒乓球队的人数为 20 , 占总数的 $20 \\%$,\n\n$\\therefore$ 总人数为: $20 \\div 20 \\%=100$ 人,\n\n$\\therefore$ 参加篮球队的有: $100 \\times 40 \\%=40$ 人,\n\n参加足球队的人数占全部参加人数的: $30 \\div 100 \\times 100 \\%=30 \\%$,故答案为: 40,30 ;\n\n(2)喜欢排球队的人数在扇形统计图中所占的圆心角是百分比为: 1 $(40 \\%+30 \\%+20 \\%)=10 \\%$,圆心角度数 $=360 \\times 10 \\%=36^{\\circ} ;$ 正确补全折线图中篮球、排球折线;\n\n\n\n(3) 用列表法\n\n| 小虎
小明 | 1 | 2 | 3 | 4 |\n| :--- | :--- | :--- | :--- | :--- |\n| 1 | 1,1 | 1,2 | 1,3 | 1,4 |\n\n\n| 2 | 2,1 | 2,2 | 2,3 | 2,4 |\n| :---: | :---: | :---: | :---: | :---: |\n| 3 | 3,1 | 3,2 | 3,3 | 3,4 |\n| 4 | 4,1 | 4,2 | 4,3 | 4,4 |\n\n共有 16 种可能的结果, 且每种结果的可能性相同, 其中小明可能获得参加权的结果是六种,\n\n分别是 $2,1 3 , 1 3 , 2 ; 4 , 1 4 , 2 ; 4 , 3$;\n\n$\\therefore$ 小明获参加权的概率 $\\mathrm{P}_{1}=\\frac{6}{16}=\\frac{3}{8}$, 小虎获参加权的概率 $\\mathrm{P}_{2}=\\frac{10}{16}=\\frac{5}{8}$, 或小虎获参加权的概率 $\\mathrm{P}_{2}=1-\\frac{3}{8}=\\frac{5}{8}$,\n\n$\\because \\mathrm{P}_{1}<\\mathrm{P}_{2}, \\therefore$ 这个规则对双方不公平.", "solution": "null", "level": "七年级", "question": "(7 分) 学校为了响应国家阳光体育活动, 选派部分学生参加足球、乒乓球、篮球、排球队集训. 根据参加项目制成如下两幅不完整的统计图 (如图 1 和如图 2, 要求每位同学只能选择一种自己喜欢的球类, 图中用足球、乒乓球、篮球、排球代表喜欢这四种球类某种球类的学生人数)\n\n\n\n图1\n\n\n\n图2\n\n请你根据图中提供的信息解答下列问题:\n\n(1)参加篮球队的有 40 人, 参加足球队的人数占全部参加人数的 $30 \\ldots \\%$.\n\n(2)喜欢排球队的人数在扇形统计图中所占的圆心角是多少度? 并补全频数分布折线统计图.\n\n(3)若足球队只剩一个集训名额, 学生小明和小虎都想参加足球队, 决定采用随机摸球的方式确定参加权, 具体规则如下: 一个不透明的袋子中装着标有数字 1、2、3、4 的四个完全相同的小球,小明随机地从四个小球中摸出一球然后放回,小虎再随机地摸出一球,若小明摸出的小球标有数字比小虎摸出的小球标有的数字大, 则小明参加, 否则小虎参加, 试分析这种规则对双方是否公平?", "options": [], "subject": "计数", "analysis": "解: (1) $\\because$ 结合折线图与扇形图得出参加乒乓球队的人数为 20 , 占总数的 $20 \\%$,\n\n$\\therefore$ 总人数为: $20 \\div 20 \\%=100$ 人,\n\n$\\therefore$ 参加篮球队的有: $100 \\times 40 \\%=40$ 人,\n\n参加足球队的人数占全部参加人数的: $30 \\div 100 \\times 100 \\%=30 \\%$,故答案为: 40,30 ;\n\n(2)喜欢排球队的人数在扇形统计图中所占的圆心角是百分比为: 1 $(40 \\%+30 \\%+20 \\%)=10 \\%$,圆心角度数 $=360 \\times 10 \\%=36^{\\circ} ;$ 正确补全折线图中篮球、排球折线;\n\n\n\n(3) 用列表法\n\n| 小虎
小明 | 1 | 2 | 3 | 4 |\n| :--- | :--- | :--- | :--- | :--- |\n| 1 | 1,1 | 1,2 | 1,3 | 1,4 |\n\n\n| 2 | 2,1 | 2,2 | 2,3 | 2,4 |\n| :---: | :---: | :---: | :---: | :---: |\n| 3 | 3,1 | 3,2 | 3,3 | 3,4 |\n| 4 | 4,1 | 4,2 | 4,3 | 4,4 |\n\n共有 16 种可能的结果, 且每种结果的可能性相同, 其中小明可能获得参加权的结果是六种,\n\n分别是 $2,1 3 , 1 3 , 2 ; 4 , 1 4 , 2 ; 4 , 3$;\n\n$\\therefore$ 小明获参加权的概率 $\\mathrm{P}_{1}=\\frac{6}{16}=\\frac{3}{8}$, 小虎获参加权的概率 $\\mathrm{P}_{2}=\\frac{10}{16}=\\frac{5}{8}$, 或小虎获参加权的概率 $\\mathrm{P}_{2}=1-\\frac{3}{8}=\\frac{5}{8}$,\n\n$\\because \\mathrm{P}_{1}<\\mathrm{P}_{2}, \\therefore$ 这个规则对双方不公平."} {"id": "2514", "image": ["3592.jpg"], "answer": "(1) 20,20 ;\n(2) 50,10 ;\n(3) 72 人\n\n【分析】(1)根据统计表和扇形统计图给出的数据即可得出答案;\n\n(2) 根据良好的人数和所占的百分比求出总人数, 再用“不及格”的女生人数除以总人数即可得出所\n占的百分比; (3) 用总人数乘以等级为“优秀”的学生人数所占的百分比即可.\n\n【详解】解: (1) 被测试女生中, 成绩等级为“良好”的女生人数为 20 ;\n\n成绩等级为“及格”的女生人数占被测试女生总人数的百分比为 $20 \\%$, 故答案为: 20,20 ;\n\n(2) 被测试女生总数是 $20 \\div 0.4=50$ (人),\n\n成绩等级为“不及格”的女生人数占被测试女生总人数的百分比为 $\\frac{5}{50} \\times 100 \\%=10 \\%$; 故答案为: 50,10 ;\n\n(3)及格人数有 $50 \\times 20 \\%=10$ (人),优秀人数有: $50-20-10-5=15$ (人), $240 \\times \\frac{15}{50}=72$ (人),\n\n答: 该校八年级女生成绩等级为“优秀”的学生人数有 72 人.\n\n【点睛】本题考查的是表格统计图和扇形统计图的综合运用. 读獚统计图, 从不同的统计图中得到必要的信息是解决问题的关键. 表格统计图能清楚地表示出每个项目的数据; 扇形统计图直接反映部分占总体的百分比大小.", "solution": "null", "level": "七年级", "question": "(2021-浙江宁波市$\\cdot$七年级期中)某校为了解九年级女生“仰卧起坐”成绩的情况,随机选取该年级部分女生进行测试. 以下是根据测试成绩绘制的统计图表的一部分.\n\n| 成绩等级 | 频数 (人) | 频率 |\n| :---: | :---: | :---: |\n| 优秀 | | |\n| 良好 | 20 | 0.4 |\n| 及格 | | |\n| 不及格 | 5 | |\n\n\n\n根据以上信息, 解答下列问题: (1)被测试女生中,成绩等级为“良好”的女生人数为人,成绩等级为“及格”的女生人数占被测试女生总人数的百分比为 \\$ \\qquad \\$ \\%; (2)被测试女生的总人数为人, 成绩等级为“不及格”的女生人数占被测试女生总人数的百分比为 \\$ \\qquad \\$ $\\% ; \\quad(3)$若该校九年级共有 240 名女生,根据调查结果,估计该校八年级女生成绩等级为“优秀”的学生人数.", "options": [], "subject": "计数", "analysis": "(1)根据统计表和扇形统计图给出的数据即可得出答案;\n\n(2) 根据良好的人数和所占的百分比求出总人数, 再用“不及格”的女生人数除以总人数即可得出所\n占的百分比; (3) 用总人数乘以等级为“优秀”的学生人数所占的百分比即可.\n\n【详解】解: (1) 被测试女生中, 成绩等级为“良好”的女生人数为 20 ;\n\n成绩等级为“及格”的女生人数占被测试女生总人数的百分比为 $20 \\%$, 故答案为: 20,20 ;\n\n(2) 被测试女生总数是 $20 \\div 0.4=50$ (人),\n\n成绩等级为“不及格”的女生人数占被测试女生总人数的百分比为 $\\frac{5}{50} \\times 100 \\%=10 \\%$; 故答案为: 50,10 ;\n\n(3)及格人数有 $50 \\times 20 \\%=10$ (人),优秀人数有: $50-20-10-5=15$ (人), $240 \\times \\frac{15}{50}=72$ (人),\n\n答: 该校八年级女生成绩等级为“优秀”的学生人数有 72 人.\n\n【点睛】本题考查的是表格统计图和扇形统计图的综合运用. 读獚统计图, 从不同的统计图中得到必要的信息是解决问题的关键. 表格统计图能清楚地表示出每个项目的数据; 扇形统计图直接反映部分占总体的百分比大小."} {"id": "2540", "image": ["3618.jpg", "3619.jpg"], "answer": "(1) $5 、 12$; (2) $10 、 C$; (3) 541 人\n\n【分析】(1) 根据组距的定义结合表格可得组距, 求出男生总人数, 再用女生总人数乘以 $B$ 组的百分比可得; (2) 将位于这一小组内的频数相加, 分别计算出各组人数之和即可求得结果;\n\n(3)分别用男、女生的人数乘以对应的百分比, 相加即可得解.\n\n【详解】解: (1) 在样本中, 组距是 5 , 男生共有 $2+4+8+12+14=40$ 人,\n\n$\\because$ 男、女生的人数相同, 女生身高在 B 组的人数有 $40 \\times(1-35 \\%-20 \\%-15 \\%-5 \\%)=12$ 人,故答案为: $5 、 12$;\n\n(2) 在样本中, 身高在 $170 \\leq x<175$ 之间的人数共有 $8+40 \\times 5 \\%=10$ 人,\n\n$\\because A$ 组人数为 $2+40 \\times 20 \\%=10$ 人, $B$ 组人数为 $4+12=16$ 人, $C$ 组人数为 $12+40 \\times 35 \\%=26$ 人, $D$ 组人数为 $14+40 \\times 10 \\%=18$ 人, $E$ 组人数为 $8+40 \\times 5 \\%=10$ 人, $\\therefore C$ 组人数最多, 故答案为: $10 、 C$;\n\n(3) $500 \\times \\frac{12+14}{40}+480 \\times(35 \\%+10 \\%)=541$ (人),\n\n故估计身高在 $160 \\leq x<170$ 之间的学生约有 541 人.\n\n【点睛】本题考查读频数分布直方图的能力和利用统计图获取信息的能力; 利用统计图获取信息时,必须认真观察、分析、研究统计图, 才能作出正确的判断和解决问题.", "solution": "null", "level": "七年级", "question": "(2022-浙江) 为了了解某中学学生的身高情况, 随机对该校男、女生的身高进行抽样调查. 抽取的样本中, 男、女生的人数相同, 根据所得数据绘制成如图所示的统计图表.\n\n| 组别 | 男女生身高 $(\\mathrm{cm})$ |\n| :--- | :--- |\n| $A$ | $150 \\leqslant x<155$ |\n\n\n| $B$ | $155 \\leqslant x<160$ |\n| :--- | :--- |\n| $C$ | $160 \\leqslant x<165$ |\n| $D$ | $165 \\leqslant x<170$ |\n| $E$ | $170 \\leqslant x<175$ |\n\n\n\n男生身高直方图\n\n\n\n女生身高扇形图\n\n根据图表中提供的信息, 回答下列问题: (1) 在样本中, 组距是 \\$ \\qquad \\$ ,女生身高在 $B$ 组的有\n\n人; (2) 在样本中, 身高在 $170 \\leqslant x<175$ 之间的共有人, 人数最多的是 \\$ \\qquad \\$组 $($ 填组别序号) ; (3) 已知该校共有男生 500 人, 女生 480 人, 请估计身高在 $160 \\leqslant x<170$ 之间的学生有多少人?\n\n", "options": [], "subject": "计数", "analysis": "(1) 根据组距的定义结合表格可得组距, 求出男生总人数, 再用女生总人数乘以 $B$ 组的百分比可得; (2) 将位于这一小组内的频数相加, 分别计算出各组人数之和即可求得结果;\n\n(3)分别用男、女生的人数乘以对应的百分比, 相加即可得解.\n\n【详解】解: (1) 在样本中, 组距是 5 , 男生共有 $2+4+8+12+14=40$ 人,\n\n$\\because$ 男、女生的人数相同, 女生身高在 B 组的人数有 $40 \\times(1-35 \\%-20 \\%-15 \\%-5 \\%)=12$ 人,故答案为: $5 、 12$;\n\n(2) 在样本中, 身高在 $170 \\leq x<175$ 之间的人数共有 $8+40 \\times 5 \\%=10$ 人,\n\n$\\because A$ 组人数为 $2+40 \\times 20 \\%=10$ 人, $B$ 组人数为 $4+12=16$ 人, $C$ 组人数为 $12+40 \\times 35 \\%=26$ 人, $D$ 组人数为 $14+40 \\times 10 \\%=18$ 人, $E$ 组人数为 $8+40 \\times 5 \\%=10$ 人, $\\therefore C$ 组人数最多, 故答案为: $10 、 C$;\n\n(3) $500 \\times \\frac{12+14}{40}+480 \\times(35 \\%+10 \\%)=541$ (人),\n\n故估计身高在 $160 \\leq x<170$ 之间的学生约有 541 人.\n\n【点睛】本题考查读频数分布直方图的能力和利用统计图获取信息的能力; 利用统计图获取信息时,必须认真观察、分析、研究统计图, 才能作出正确的判断和解决问题."} {"id": "1877", "image": [], "answer": "(1)图略.\n\n$(2)|+6|+|-9|+|+7|+|-5|=27(\\mathrm{~km})$,\n\n所以耗油 $27 \\times", "solution": "null", "level": "七年级", "question": "(本小题 8.0 分)\n\n某电力公司检修小组乘汽车沿一条东西走向的公路检修线路, 向东行驶记为正.某天从 $A$ 地出发到收工时,行走记录为(单位: $k m):+6,-9,+7,-5$.\n\n(1)画一条数轴, 以原点表示 $A$ 地, 在数轴上表示出检修小组每次行走的终点位置.\n\n(2)估计汽车行驶 $100 \\mathrm{~km}$ 的耗油量为 $12.5 \\mathrm{~L}$, 则这天检修小组从出发到收工共耗油多少升?", "options": [], "subject": "计数", "analysis": ""} {"id": "1302", "image": ["3095.jpg", "3096.jpg", "3096.jpg"], "answer": "(1) 解: $-1,2,-\\frac{1}{2},-(-3),|-3.5|$ 的绝对值分别为;\n\n$|-1|=1,|2|=2,\\left|-\\frac{1}{2}\\right|=\\frac{1}{2},|-(-3)|=3,|-3.5|=3.5$,\n\n并分别把它们和它们的绝对值在数轴上表示如下图:\n\n\n\n(2)解:由题意可知: $a+b=0, c d=1, m= \\pm 2$,\n\n$\\therefore$ 原式 $=0+4-2$\n$=2 ;$", "solution": "null", "level": "七年级", "question": "(1)写出下列各数的绝对值, 并分别把它们和它们的绝对值在数轴上表示出来.\n\n$-1,2,-\\frac{1}{2},-(-3),|-3.5|$.\n\n\n\n(2) 已知 $\\mathrm{a}, \\mathrm{b}$ 互为相反数, $\\mathrm{c}, \\mathrm{d}$ 互为倒数, $\\mathrm{m}$ 绝对值等于 2 的数, 求 $\\frac{a+b}{a+b+c}+m^{2}-2 c d$ 的值.", "options": [], "subject": "算术", "analysis": "(1) 解: $-1,2,-\\frac{1}{2},-(-3),|-3.5|$ 的绝对值分别为;\n\n$|-1|=1,|2|=2,\\left|-\\frac{1}{2}\\right|=\\frac{1}{2},|-(-3)|=3,|-3.5|=3.5$,\n\n并分别把它们和它们的绝对值在数轴上表示如下图:\n\n\n\n(2)解:由题意可知: $a+b=0, c d=1, m= \\pm 2$,\n\n$\\therefore$ 原式 $=0+4-2$\n$=2 ;$"} {"id": "1324", "image": [], "answer": "(1) 解: $10-(-5)+(-9)$\n\n$=10+5-9$\n\n$=6$\n\n(2) 解: $\\left(-\\frac{1}{6}+\\frac{2}{3}-\\frac{5}{12}\\right) \\div\\left(-\\frac{1}{48}\\right)$\n\n$=\\left(\\frac{1}{6}-\\frac{2}{3}+\\frac{5}{12}\\right) \\times 48$\n\n$=\\frac{1}{6} \\times 48-\\frac{2}{3} \\times 48+\\frac{5}{12} \\times 48$\n\n$=8-32+20$\n\n$=-4$\n\n(3) 解: $-1^{2021}-\\frac{1}{6} \\div\\left(-3 \\times \\frac{1}{9}\\right)+|-2|$\n\n$=-1-\\frac{1}{6} \\div\\left(-\\frac{1}{3}\\right)+2$\n\n$=-1+\\frac{1}{6} \\times 3+2$\n\n$=-1+\\frac{1}{2}+2$\n\n$=\\frac{3}{2}$", "solution": "null", "level": "七年级", "question": "计算:\n\n(1) $10-(-5)+(-9)$\n\n(2) $\\left(-\\frac{1}{6}+\\frac{2}{3}-\\frac{5}{12}\\right) \\div\\left(-\\frac{1}{48}\\right)$\n\n(3) $-1^{2021}-\\frac{1}{6} \\div\\left(-3 \\times \\frac{1}{9}\\right)+|-2|$", "options": [], "subject": "算术", "analysis": "(1) 解: $10-(-5)+(-9)$\n\n$=10+5-9$\n\n$=6$\n\n(2) 解: $\\left(-\\frac{1}{6}+\\frac{2}{3}-\\frac{5}{12}\\right) \\div\\left(-\\frac{1}{48}\\right)$\n\n$=\\left(\\frac{1}{6}-\\frac{2}{3}+\\frac{5}{12}\\right) \\times 48$\n\n$=\\frac{1}{6} \\times 48-\\frac{2}{3} \\times 48+\\frac{5}{12} \\times 48$\n\n$=8-32+20$\n\n$=-4$\n\n(3) 解: $-1^{2021}-\\frac{1}{6} \\div\\left(-3 \\times \\frac{1}{9}\\right)+|-2|$\n\n$=-1-\\frac{1}{6} \\div\\left(-\\frac{1}{3}\\right)+2$\n\n$=-1+\\frac{1}{6} \\times 3+2$\n\n$=-1+\\frac{1}{2}+2$\n\n$=\\frac{3}{2}$"} {"id": "1325", "image": [], "answer": "解: (1)原式 $=1-\\frac{1}{2}+\\frac{1}{2}-\\frac{1}{3}+\\cdots+\\frac{1}{2004}-\\frac{1}{2005}=1-\\frac{1}{2005}=\\frac{2004}{2005}$;\n\n(2) 原式 $=\\frac{1}{2} \\times\\left(1-\\frac{1}{3}+\\frac{1}{3}-\\frac{1}{5}+\\frac{1}{5}-\\frac{1}{7}+\\cdots+\\frac{1}{49}-\\frac{1}{51}\\right)=\\frac{1}{2} \\times\\left(1-\\frac{1}{51}\\right)=\\frac{25}{51}$.", "solution": "null", "level": "七年级", "question": "请先阅读下列一组内容, 然后解答问题:\n\n因为: $\\frac{1}{1 \\times 2}=1-\\frac{1}{2}, \\frac{1}{2 \\times 3}=\\frac{1}{2}-\\frac{1}{3}, \\frac{1}{3 \\times 4}=\\frac{1}{3}-\\frac{1}{4} \\cdots \\frac{1}{9 \\times 10}=\\frac{1}{9}-\\frac{1}{10}$\n\n所以: $\\frac{1}{1 \\times 2}+\\frac{1}{2 \\times 3}+\\frac{1}{3 \\times 4}+\\cdots+\\frac{1}{9 \\times 10}$\n\n$=\\left(1-\\frac{1}{2}\\right)+\\left(\\frac{1}{2}-\\frac{1}{3}\\right)+\\left(\\frac{1}{3}-\\frac{1}{4}\\right)+\\cdots+\\left(\\frac{1}{9}-\\frac{1}{10}\\right)$\n\n$=1-\\frac{1}{2}+\\frac{1}{2}-\\frac{1}{3}+\\frac{1}{3}-\\frac{1}{4}+\\cdots+\\frac{1}{9}-\\frac{1}{10}=1-\\frac{1}{10}=\\frac{9}{10}$\n\n问题:\n\n计算: (1) $\\frac{1}{1 \\times 2}+\\frac{1}{2 \\times 3}+\\frac{1}{3 \\times 4}+\\cdots+\\frac{1}{2004 \\times 2005}$;\n\n(2) $\\frac{1}{1 \\times 3}+\\frac{1}{3 \\times 5}+\\frac{1}{5 \\times 7}+\\cdots+\\frac{1}{49 \\times 51}$", "options": [], "subject": "算术", "analysis": "解: (1)原式 $=1-\\frac{1}{2}+\\frac{1}{2}-\\frac{1}{3}+\\cdots+\\frac{1}{2004}-\\frac{1}{2005}=1-\\frac{1}{2005}=\\frac{2004}{2005}$;\n\n(2) 原式 $=\\frac{1}{2} \\times\\left(1-\\frac{1}{3}+\\frac{1}{3}-\\frac{1}{5}+\\frac{1}{5}-\\frac{1}{7}+\\cdots+\\frac{1}{49}-\\frac{1}{51}\\right)=\\frac{1}{2} \\times\\left(1-\\frac{1}{51}\\right)=\\frac{25}{51}$."} {"id": "1326", "image": [], "answer": "解:解法一:原式 $=\\frac{1}{12} \\div \\frac{1}{3}-\\frac{1}{12} \\div \\frac{1}{4}+\\frac{1}{12} \\div \\frac{1}{12}=\\frac{1}{4}-\\frac{1}{3}+1=\\frac{11}{12}$ ,\n\n解法二: 原式 $=\\frac{1}{12} \\div\\left(\\frac{1}{3}-\\frac{1}{4}+\\frac{1}{12}\\right)=\\frac{1}{12} \\div\\left(\\frac{4}{12}-\\frac{3}{12}+\\frac{1}{12}\\right)=\\frac{1}{12} \\div \\frac{1}{6}=\\frac{1}{12} \\times 6=\\frac{1}{2}$,\n\n解法三: 原式的倒数为\n\n$\\left(\\frac{1}{3}-\\frac{1}{4}+\\frac{1}{12}\\right) \\div \\frac{1}{12}=\\left(\\frac{1}{3}-\\frac{1}{4}+\\frac{1}{12}\\right) \\times 12=\\frac{1}{3} \\times 12-\\frac{1}{4} \\times 12+\\frac{1}{12} \\times 12=4-3+1=2$,\n\n$\\therefore \\frac{1}{12} \\div\\left(\\frac{1}{3}-\\frac{1}{4}+\\frac{1}{12}\\right)=\\frac{1}{2} ;$\n\n上述得到的结果不同, 所以我认为解法一是错误的;\n\n故答案为一;\n\n$\\left(-\\frac{1}{30}\\right) \\div\\left(\\frac{2}{3}-\\frac{1}{10}+\\frac{1}{6}-\\frac{2}{5}\\right)$ 的倒数为 $\\left(\\frac{2}{3}-\\frac{1}{10}+\\frac{1}{6}-\\frac{2}{5}\\right) \\div\\left(-\\frac{1}{30}\\right)=\\left(\\frac{2}{3}-\\frac{1}{10}+\\frac{1}{6}-\\frac{2}{5}\\right) \\times(-30)$\n\n$$\n=\\frac{2}{3} \\times(-30)-\\frac{1}{10} \\times(-30)+\\frac{1}{6} \\times(-30)-\\frac{2}{5} \\times(-30)=-20+3-5+12\n$$\n\n$=-10 ;$\n\n$\\therefore\\left(-\\frac{1}{30}\\right) \\div\\left(\\frac{2}{3}-\\frac{1}{10}+\\frac{1}{6}-\\frac{2}{5}\\right)=-\\frac{1}{10}$", "solution": "null", "level": "七年级", "question": "请你仔细阅读下列材料, 计算:\n\n阅读下列材料: 计算 $\\frac{1}{12} \\div\\left(\\frac{1}{3}-\\frac{1}{4}+\\frac{1}{12}\\right)$.\n解法一: 原式 $==\\frac{1}{12} \\div \\frac{1}{3}-\\frac{1}{12} \\div \\frac{1}{4}+\\frac{1}{12} \\div \\frac{1}{12}$\n\n解法二: 原式 $=\\frac{1}{12} \\div\\left(\\frac{1}{3}-\\frac{1}{4}+\\frac{1}{12}\\right)$\n\n解法三: 原式的倒数为 $\\left(\\frac{1}{3}-\\frac{1}{4}+\\frac{1}{12}\\right) \\div \\frac{1}{12}=\\left(\\frac{1}{3}-\\frac{1}{4}+\\frac{1}{12}\\right) \\times 12$\n\n上述得出的结果不同, 肯定有错误的解法, 你认为解法 $\\_$_ _是错误的.\n\n请你选择合适的解法解答下列问题: 计算: $\\left(-\\frac{1}{30}\\right) \\div\\left(\\frac{2}{3}-\\frac{1}{10}+\\frac{1}{6}-\\frac{2}{5}\\right)$", "options": [], "subject": "算术", "analysis": "解:解法一:原式 $=\\frac{1}{12} \\div \\frac{1}{3}-\\frac{1}{12} \\div \\frac{1}{4}+\\frac{1}{12} \\div \\frac{1}{12}=\\frac{1}{4}-\\frac{1}{3}+1=\\frac{11}{12}$ ,\n\n解法二: 原式 $=\\frac{1}{12} \\div\\left(\\frac{1}{3}-\\frac{1}{4}+\\frac{1}{12}\\right)=\\frac{1}{12} \\div\\left(\\frac{4}{12}-\\frac{3}{12}+\\frac{1}{12}\\right)=\\frac{1}{12} \\div \\frac{1}{6}=\\frac{1}{12} \\times 6=\\frac{1}{2}$,\n\n解法三: 原式的倒数为\n\n$\\left(\\frac{1}{3}-\\frac{1}{4}+\\frac{1}{12}\\right) \\div \\frac{1}{12}=\\left(\\frac{1}{3}-\\frac{1}{4}+\\frac{1}{12}\\right) \\times 12=\\frac{1}{3} \\times 12-\\frac{1}{4} \\times 12+\\frac{1}{12} \\times 12=4-3+1=2$,\n\n$\\therefore \\frac{1}{12} \\div\\left(\\frac{1}{3}-\\frac{1}{4}+\\frac{1}{12}\\right)=\\frac{1}{2} ;$\n\n上述得到的结果不同, 所以我认为解法一是错误的;\n\n故答案为一;\n\n$\\left(-\\frac{1}{30}\\right) \\div\\left(\\frac{2}{3}-\\frac{1}{10}+\\frac{1}{6}-\\frac{2}{5}\\right)$ 的倒数为 $\\left(\\frac{2}{3}-\\frac{1}{10}+\\frac{1}{6}-\\frac{2}{5}\\right) \\div\\left(-\\frac{1}{30}\\right)=\\left(\\frac{2}{3}-\\frac{1}{10}+\\frac{1}{6}-\\frac{2}{5}\\right) \\times(-30)$\n\n$$\n=\\frac{2}{3} \\times(-30)-\\frac{1}{10} \\times(-30)+\\frac{1}{6} \\times(-30)-\\frac{2}{5} \\times(-30)=-20+3-5+12\n$$\n\n$=-10 ;$\n\n$\\therefore\\left(-\\frac{1}{30}\\right) \\div\\left(\\frac{2}{3}-\\frac{1}{10}+\\frac{1}{6}-\\frac{2}{5}\\right)=-\\frac{1}{10}$"} {"id": "1328", "image": [], "answer": "(1) $1+2+3+4+5 ($ 或 $15 ; 225$\n\n(2) $\\frac{n(n+1)}{2}$\n\n(3) 解: 由 (2) 得,\n\n$11^{3}+12^{3}+13^{3}+\\ldots+19^{3}+20^{3}$\n\n$=1^{3}+2^{3}+3^{3}+\\ldots+19^{3}+20^{3}-\\left(1^{3}+2^{3}+3^{3}+\\ldots+9^{3}+10^{3}\\right)$\n\n$=\\left(\\frac{20 \\times 21}{2}\\right)^{2}-\\left(\\frac{10 \\times 11}{2}\\right)^{2}$\n\n$=44100-3025$\n\n$=41075$", "solution": "null", "level": "七年级", "question": "观察下列一组算式的特征, 并探索规律:\n\n(1) $\\sqrt{1^{3}}=1=1$;\n\n(2) $\\sqrt{1^{3}+2^{3}}=1+2=3$;\n\n(3) $\\sqrt{1^{3}+2^{3}+3^{3}}=1+2+3=6$;\n\n(4) $\\sqrt{1^{3}+2^{3}+3^{3}+4^{3}}=1+2+3+4=10$.\n\n根据以上算式的规律, 解答下列问题:\n\n(1) $1^{3}+2^{3}+3^{3}+4^{3}+5^{3}=($ )$^{2}=$\n\n(2) $\\sqrt{1^{3}+2^{3}+3^{3}+\\cdots+(n-1)^{3}+n^{3}}=$\n\n(3)简便计算: $11^{3}+12^{3}+13^{3}+\\ldots+19^{3}+20^{3}$.", "options": [], "subject": "算术", "analysis": "(1) $1+2+3+4+5 ($ 或 $15 ; 225$\n\n(2) $\\frac{n(n+1)}{2}$\n\n(3) 解: 由 (2) 得,\n\n$11^{3}+12^{3}+13^{3}+\\ldots+19^{3}+20^{3}$\n\n$=1^{3}+2^{3}+3^{3}+\\ldots+19^{3}+20^{3}-\\left(1^{3}+2^{3}+3^{3}+\\ldots+9^{3}+10^{3}\\right)$\n\n$=\\left(\\frac{20 \\times 21}{2}\\right)^{2}-\\left(\\frac{10 \\times 11}{2}\\right)^{2}$\n\n$=44100-3025$\n\n$=41075$"} {"id": "1329", "image": [], "answer": "(1) 结果为正, 并将两数的绝对值相加; 结果为负, 并用较大绝对值减去较小绝对值\n\n(2) $45 ;-17$\n\n(3) $-2020 ;-2022$", "solution": "null", "level": "七年级", "question": "定义 $\\oplus$ 运算\n\n观察下列运算:\n\n$(+4) \\oplus(+16)=+20, \\quad(-5) \\oplus(-7)=+12, \\quad(-6) \\oplus(-17)=+23, \\quad(-2) \\oplus(+14)=-12$, $(-15) \\oplus(+9)=-6, \\quad(+25) \\oplus(-10)=-15, \\quad 0 \\oplus(-15)=-15, \\quad(+12) \\oplus 0=+12$.\n\n(1) 请你认真思考上述运算,归纳 $\\oplus$ 运算的法则:两数进行 $\\oplus$ 运算时, 同号两数运算算 ,异号两数运\n\n特别地, 0 和任何数进行 $\\oplus$ 运算, 或任何数和 0 进行 $\\oplus$ 运算, 仍得这个数.\n\n(2) 计算: $(-16) \\oplus(-29)=$ , $(+18) \\oplus[0 \\oplus(-35)]=$\n\n(3) 若 $-1 \\oplus a=2021$, 则 $a=$ \\$ \\qquad \\$ , 若 $1 \\oplus b=-2021$, 则 $b=$", "options": [], "subject": "算术", "analysis": "(1) 由题意知, 两数进行 $\\oplus$ 运算时, 同号两数运算结果为正, 并将两数的绝对值相加, 异号两数运算结果为负, 并用较大绝对值减去较小绝对值.\n\n特别地, 0 和任何数进行 $\\oplus$ 运算, 或任何数和 0 进行 $\\oplus$ 运算, 仍得这个数.\n\n故答案为: 结果为正, 并将两数的绝对值相加; 结果为负, 并用较大绝对值减去较小绝对值;\n\n(2) $(-16) \\oplus(-29)=+(16+29)=45$,\n\n$(+18) \\oplus[0 \\oplus(-35)]$\n\n$=(+18) \\oplus(-35)$\n\n$=-(35-18)$\n$=-17$;\n\n故答案为: $45,-17$;\n\n(3) $\\because-1 \\oplus \\mathrm{a}=2021$,\n\n$\\therefore \\mathrm{a}=-(2021-1)=-2020$;\n\n$\\because 1 \\oplus \\mathrm{b}=-2021$,\n\n$\\therefore \\mathrm{b}=-(2021+1)=-2022$,\n\n故答案为: $-2020,-2022$."} {"id": "1330", "image": [], "answer": "(1) 解: $-3 \\times(-1) \\times(5+3)=24$ (答案不唯一)\n\n(2) 解: $3 \\times(10-6+4)=24$ 或 $3 \\times(10-4)-(-6)=24$ (答案不唯一)\n\n(3) 解: $2-[8 \\times 11 \\div(-4)]=24$ (答案不唯一)", "solution": "null", "level": "七年级", "question": "“ 24 ”点游戏的规则是这样的: 在整数范围内任意取四个数, 然后进行加、减、乘、除四则运算(每个数只能用一次,可使用小括号、中括号), 使其结果等于 24 . 例如, 取 $2,3,6$, 9 这四个数进行运算, 得: $2 \\times 6+3+9=24$, 或 $6 \\times 9 \\div 2-3=24$, 或 $3 \\times 9-6 \\div 2=24$等.\n\n(1) 用-3, -1, 5, 3 这四个整数, 写出 1 种算式, 使其运算结果为 24 ;\n\n(2) 用 $-6,3,4,10$ 这四个整数, 写出 2 种不同的算式, 使其运算结果为 24 ;\n\n(3) 用 $-4,2,8,11$ 这四个整数, 写出 1 种算式, 使其运算结果为 24 .", "options": [], "subject": "算术", "analysis": "(1) 解: $-3 \\times(-1) \\times(5+3)=24$ (答案不唯一)\n\n(2) 解: $3 \\times(10-6+4)=24$ 或 $3 \\times(10-4)-(-6)=24$ (答案不唯一)\n\n(3) 解: $2-[8 \\times 11 \\div(-4)]=24$ (答案不唯一)"} {"id": "1332", "image": [], "answer": "(1) $225 ; 5 ; 6$\n\n(2) $\\frac{1}{4} \\times n^{2} \\times(n+1)^{2}$\n\n(3) 解: 利用 (2) 中的结论计算:\n\n$11^{3}+12^{3}+13^{3}+14^{3}+15^{3}+16^{3}+\\ldots+39^{3}+40^{3}$.\n\n解: 原式 $=1^{3}+2^{3}+3^{3}+\\ldots+39^{3}+40^{3}-\\left(1^{3}+2^{3}+3^{3}+\\ldots+10^{3}\\right)$\n\n21 世纪教育网(www.21cnjy.com)\n$=\\frac{1}{4} \\times 40^{2} \\times 41^{2}-\\frac{1}{4} \\times 10^{2} \\times 11^{2}$\n\n$=672400-3025$\n\n$=669375$.", "solution": "null", "level": "七年级", "question": "已知 $1^{3}=\\frac{1}{4} \\times 1^{2} \\times 2^{2}, 1^{3}+2^{3}=9=\\frac{1}{4} \\times 2^{2} \\times 3^{2}, 1^{3}+2^{3}+3^{3}=36=\\frac{1}{4} \\times 3^{2} \\times 4^{2}, \\ldots$,按照这个规完成下列问题:\n\n(1) $1^{3}+2^{3}+3^{3}+4^{3}+5^{3}=$ \\$ \\qquad \\$ $=\\frac{1}{4} \\times$ \\$ \\qquad \\$ in $^{2} \\times$ \\$ \\qquad \\$ ! $^{2}$\n\n(2)猜想: $1^{3}+2^{3}+3^{3}+\\ldots+n^{3}=$ \\$ \\qquad \\$\n(3)利用 (2) 中的结论计算: (写出计算过程) $11^{3}+12^{3}+13^{3}+14^{3}+15^{3}+16^{3}+\\ldots+39^{3}+$ $40^{3}$", "options": [], "subject": "算术", "analysis": ") $1^{3}+2^{3}+3^{3}+4^{3}+5^{3}=225=\\frac{1}{4} \\times 5^{2} \\times 6^{2}$;\n\n故答案为: $225,5,6$;\n\n(2)猜想: $1^{3}+2^{3}+3^{3}+\\ldots+n^{3}=\\frac{1}{4} \\times n^{2} \\times(n+1)^{2}$;\n\n故答案为: $\\frac{1}{4} \\times n^{2} \\times(n+1)^{2}$."} {"id": "1348", "image": [], "answer": "(1) 解: $(-5.3)+(-3.2)-(-5.3)-(+4.8)$\n\n$=(-5.3)+(-3.2)+5.3+(-4.8)$\n\n$=(-5.3+5.3)+(-3.2-4.8)$\n\n$=0+(-8)$\n\n$=-8 ;$\n\n(2) 解: $9 \\frac{17}{18} \\times(-9)$\n\n$=\\left(10-\\frac{1}{18}\\right) \\times(-9)$\n\n$=-10 \\times 9+\\frac{1}{18} \\times 9$\n\n$=-90+0.5$\n\n$=-89.5$\n\n(3) 解: $\\left(-\\frac{3}{4}-\\frac{5}{9}+\\frac{7}{12}\\right) \\div \\frac{1}{36}$\n$=\\left(-\\frac{3}{4}-\\frac{5}{9}+\\frac{7}{12}\\right) \\times 36$\n\n$=-\\frac{3}{4} \\times 36-\\frac{5}{9} \\times 36+\\frac{7}{12} \\times 36$\n\n$=-27-20+21$\n\n$=-26$\n\n(4) 解: $\\left|-\\frac{7}{9}\\right| \\div\\left(\\frac{2}{3}-\\frac{1}{5}\\right)-\\frac{1}{3} \\times(-4)^{2}$\n\n$=\\frac{7}{9} \\div \\frac{7}{15}-\\frac{1}{3} \\times 16$\n\n$=\\frac{7}{9} \\times \\frac{15}{7}-\\frac{1}{3} \\times 16$\n\n$=\\frac{5}{3}-\\frac{16}{3}$\n\n$=-\\frac{11}{3}$.", "solution": "null", "level": "七年级", "question": "(1) $(-5.3)+(-3.2)-(-5.3)-(+4.8)$.\n\n(2) $9 \\frac{17}{18} \\times(-9)$.\n\n(3) $\\left(-\\frac{3}{4}-\\frac{5}{9}+\\frac{7}{12}\\right) \\div \\frac{1}{36}$.\n\n(4) $\\left|-\\frac{7}{9}\\right| \\div\\left(\\frac{2}{3}-\\frac{1}{5}\\right)-\\frac{1}{3} \\times(-4)^{2}$.", "options": [], "subject": "算术", "analysis": "(1) 解: $(-5.3)+(-3.2)-(-5.3)-(+4.8)$\n\n$=(-5.3)+(-3.2)+5.3+(-4.8)$\n\n$=(-5.3+5.3)+(-3.2-4.8)$\n\n$=0+(-8)$\n\n$=-8 ;$\n\n(2) 解: $9 \\frac{17}{18} \\times(-9)$\n\n$=\\left(10-\\frac{1}{18}\\right) \\times(-9)$\n\n$=-10 \\times 9+\\frac{1}{18} \\times 9$\n\n$=-90+0.5$\n\n$=-89.5$\n\n(3) 解: $\\left(-\\frac{3}{4}-\\frac{5}{9}+\\frac{7}{12}\\right) \\div \\frac{1}{36}$\n$=\\left(-\\frac{3}{4}-\\frac{5}{9}+\\frac{7}{12}\\right) \\times 36$\n\n$=-\\frac{3}{4} \\times 36-\\frac{5}{9} \\times 36+\\frac{7}{12} \\times 36$\n\n$=-27-20+21$\n\n$=-26$\n\n(4) 解: $\\left|-\\frac{7}{9}\\right| \\div\\left(\\frac{2}{3}-\\frac{1}{5}\\right)-\\frac{1}{3} \\times(-4)^{2}$\n\n$=\\frac{7}{9} \\div \\frac{7}{15}-\\frac{1}{3} \\times 16$\n\n$=\\frac{7}{9} \\times \\frac{15}{7}-\\frac{1}{3} \\times 16$\n\n$=\\frac{5}{3}-\\frac{16}{3}$\n\n$=-\\frac{11}{3}$."} {"id": "1349", "image": [], "answer": "(1) $\\frac{1}{n \\times(n+1)}-\\frac{1}{(n+1) \\times(n+2)}=\\frac{2}{n \\times(n+1) \\times(n+2)}$\n\n(2) 解:原式 $=\\frac{2}{6}+\\frac{2}{24}+\\frac{2}{60}+\\frac{2}{120}$\n\n$=\\frac{1}{1 \\times 2}-\\frac{1}{2 \\times 3}+\\frac{1}{2 \\times 3}-\\frac{1}{3 \\times 4}+\\frac{1}{3 \\times 4}-\\frac{1}{4 \\times 5}+\\frac{1}{4 \\times 5}-\\frac{1}{5 \\times 6}$\n\n$=\\frac{1}{1 \\times 2}-\\frac{1}{5 \\times 6}$\n\n$=\\frac{1}{2}-\\frac{1}{30}$\n\n$=\\frac{7}{15}$\n\n(3) 解: $\\frac{2}{1 \\times 2 \\times 3}+\\frac{2}{2 \\times 3 \\times 4}+\\frac{2}{3 \\times 4 \\times 5}+\\ldots+\\frac{2}{n(n+1)(n+2)}=\\frac{22}{45}$ $\\frac{1}{1 \\times 2}-\\frac{1}{2 \\times 3}+\\frac{1}{2 \\times 3}-\\frac{1}{3 \\times 4}+\\frac{1}{3 \\times 4}-\\frac{1}{4 \\times 5}+\\ldots+\\frac{1}{n(n+1)}-\\frac{1}{(n+1)(n+2)}=\\frac{22}{45}$\n\n$$\n\\begin{gathered}\n\\frac{1}{1 \\times 2}-\\frac{1}{(n+1)(n+2)}=\\frac{22}{45} \\\\\n-\\frac{1}{(n+1)(n+2)}=\\frac{22}{45}-\\frac{1}{1 \\times 2} \\\\\n\\frac{1}{(n+1)(n+2)}=\\frac{1}{90}\n\\end{gathered}\n$$\n\n$\\because \\mathrm{n}$ 为正整数, 所以 $\\mathrm{n}+1=9, \\mathrm{n}+2=10$, 则 $\\mathrm{n}=8$.", "solution": "null", "level": "七年级", "question": "认真观察, 寻找规律\n\n第 1 个算式: $\\frac{1}{1 \\times 2}-\\frac{1}{2 \\times 3}=\\frac{2}{6}$;\n\n第 2 个算式: $\\frac{1}{2 \\times 3}-\\frac{1}{3 \\times 4}=\\frac{2}{24}$\n\n第 3 个算式: $\\frac{1}{3 \\times 4}-\\frac{1}{4 \\times 5}=\\frac{2}{60}$;\n\n第 4 个算式: $\\frac{1}{4 \\times 5}-\\frac{1}{5 \\times 6}=\\frac{2}{120}$\n\n用你发现的规律解答问题:\n\n(1)第 $\\mathrm{n}$ 个算式为:\n\n(2)计算: $\\frac{1}{3}+\\frac{1}{12}+\\frac{1}{30}+\\frac{1}{60}$;\n\n(3) 若 $\\frac{1}{1 \\times 2 \\times 3}+\\frac{1}{2 \\times 3 \\times 4}+\\frac{1}{3 \\times 4 \\times 5}+\\ldots+\\frac{1}{n(n+1)(n+2)}=\\frac{11}{45}$, 求 $\\mathrm{n}$ 的值.", "options": [], "subject": "算术", "analysis": "(1) $\\frac{1}{n \\times(n+1)}-\\frac{1}{(n+1) \\times(n+2)}=\\frac{2}{n \\times(n+1) \\times(n+2)}$\n\n(2) 解:原式 $=\\frac{2}{6}+\\frac{2}{24}+\\frac{2}{60}+\\frac{2}{120}$\n\n$=\\frac{1}{1 \\times 2}-\\frac{1}{2 \\times 3}+\\frac{1}{2 \\times 3}-\\frac{1}{3 \\times 4}+\\frac{1}{3 \\times 4}-\\frac{1}{4 \\times 5}+\\frac{1}{4 \\times 5}-\\frac{1}{5 \\times 6}$\n\n$=\\frac{1}{1 \\times 2}-\\frac{1}{5 \\times 6}$\n\n$=\\frac{1}{2}-\\frac{1}{30}$\n\n$=\\frac{7}{15}$\n\n(3) 解: $\\frac{2}{1 \\times 2 \\times 3}+\\frac{2}{2 \\times 3 \\times 4}+\\frac{2}{3 \\times 4 \\times 5}+\\ldots+\\frac{2}{n(n+1)(n+2)}=\\frac{22}{45}$ $\\frac{1}{1 \\times 2}-\\frac{1}{2 \\times 3}+\\frac{1}{2 \\times 3}-\\frac{1}{3 \\times 4}+\\frac{1}{3 \\times 4}-\\frac{1}{4 \\times 5}+\\ldots+\\frac{1}{n(n+1)}-\\frac{1}{(n+1)(n+2)}=\\frac{22}{45}$\n\n$$\n\\begin{gathered}\n\\frac{1}{1 \\times 2}-\\frac{1}{(n+1)(n+2)}=\\frac{22}{45} \\\\\n-\\frac{1}{(n+1)(n+2)}=\\frac{22}{45}-\\frac{1}{1 \\times 2} \\\\\n\\frac{1}{(n+1)(n+2)}=\\frac{1}{90}\n\\end{gathered}\n$$\n\n$\\because \\mathrm{n}$ 为正整数, 所以 $\\mathrm{n}+1=9, \\mathrm{n}+2=10$, 则 $\\mathrm{n}=8$."} {"id": "1350", "image": [], "answer": "解: 原式的倒数为: $\\left(\\frac{1}{6}-\\frac{3}{14}+\\frac{2}{3}-\\frac{2}{7}\\right) \\div\\left(-\\frac{1}{42}\\right)=\\left(\\frac{1}{6}-\\frac{3}{14}+\\frac{2}{3}-\\frac{2}{7}\\right) \\times(-42)$\n\n$$\n=-7+9-28+12=-14\n$$\n\n$$\n\\therefore \\text { 原式 }=-\\frac{1}{14} \\text {. }\n$$\n\n20.(1)尝试: 比较下列各式的大小关系:(用“ $>,<,=, \\geq$ 或 $<$ ”填空)\n(1) $|-2|+|3|$ $|-2+3|$\n(2) $|-6|+|4|$ $|-6+4|$\n(3) $|-3|+|-4|$ $|-3-4|$\n(4) $|0|+|-7|$ $|0-7|$.\n\n(2) 归纳: 观察上面的数量关系, 可以得到: $|a|+|b|$填空) \\$ \\qquad \\$ $|\\mathrm{a}+\\mathrm{b}| \\quad($ 用“ $>,<,=, \\geq$ 或 $\\leq ”$\n\n(3)应用: 利用上面得到的结论解决下面问题:若 $|m|+|n|=10,|m+n|=4$, 则 $m$ $=$ \\$ \\qquad \\$\n(4)拓展:当 $|a|+|b|+|c|>|a+b+c|$ 成立时, $a 、 b 、 c$ 应满足的条件是 \\$ \\qquad \\$ ( 填写所有正确选项的序号)\n(1) 1 个正数, 2 个负数;\n(2) 2 个正数, 1 个负数;\n(3) 3 个正数;\n(4) 3 个负数; (5) 1 个 0,2 个正数;\n\n(6) 1 个 0,2 个负数;\n\n(7) 1 个 0,1 个正数, 1 个负数.\n\n【答案】 $(1)>;>;=;=$\n\n(2) $\\geq$\n\n(3) $\\pm 3$ 或 $\\pm 7$\n\n(4) (1) (2) (7)", "solution": "null", "level": "七年级", "question": "请你先认真阅读材料:\n\n计算 $\\left(-\\frac{1}{30}\\right) \\div\\left(\\frac{2}{3}-\\frac{1}{10}+\\frac{1}{6}-\\frac{2}{5}\\right)$\n\n解法 1 :\n\n$$\n\\begin{aligned}\n& \\left(-\\frac{1}{30}\\right) \\div\\left(\\frac{2}{3}-\\frac{1}{10}+\\frac{1}{6}-\\frac{2}{5}\\right) \\\\\n= & \\left(-\\frac{1}{30}\\right) \\div\\left[\\left(\\frac{2}{3}+\\frac{1}{6}\\right)-\\left(\\frac{1}{10}+\\frac{2}{5}\\right)\\right] \\\\\n= & \\left(-\\frac{1}{30}\\right) \\div\\left(\\frac{5}{6}-\\frac{1}{2}\\right) \\\\\n= & \\left(-\\frac{1}{30}\\right) \\div \\frac{1}{3} \\\\\n= & -\\frac{1}{30} \\times 3 \\\\\n= & -\\frac{1}{10}\n\\end{aligned}\n$$\n\n解法 2 :\n\n原式的倒数为:\n\n$$\n\\begin{aligned}\n& \\left(\\frac{2}{3}-\\frac{1}{10}+\\frac{1}{6}-\\frac{2}{5}\\right) \\div\\left(-\\frac{1}{30}\\right) \\\\\n= & \\left(\\frac{2}{3}-\\frac{1}{10}+\\frac{1}{6}-\\frac{2}{5}\\right) \\times(-30) \\\\\n= & -20+3-5+12 \\\\\n= & (-20-5)+(3+12) \\\\\n= & -10\n\\end{aligned}\n$$\n\n再根据你对所提供材料的理解,选择合适的方法计算:\n$\\left(-\\frac{1}{42}\\right) \\div\\left(\\frac{1}{6}-\\frac{3}{14}+\\frac{2}{3}-\\frac{2}{7}\\right)$.", "options": [], "subject": "算术", "analysis": "(1) (1) $|-2|+|3|=2+3=5,|-2+3|=1$, 则 $|-2|+|3|>|-2+3|$;\n\n(2) $|-6|+|4|=6+4=10,|-6+4|=2$, 则 $|-6|+|4|>|-6+4|$;\n\n(3) $|-3|+|-4|=3+4=7,|-3-4|=7$, 则 $|-3|+|-4|=|-3-4|$;\n\n(4) $|0|+|-7|=0+7=7,|0-7|=7$, 则 $|0|+|-7|=|0-7|$;\n\n故答案为: $>;>;=;=$;\n\n(2)由(1)中的结果可知: 当 $a 、 b$ 异号时, 则有 $|a|+|b|>|a+b|$, 当 $a 、 b$ 同号或者其中有一个为零时, 则有 $|a|+|b|=|a+b|$,\n\n$\\therefore$ 综上所述: $|a|+|b| \\geq|a+b|$;\n\n故答案为: $\\geq$;\n\n(3) $\\because|\\mathrm{m}|+|\\mathrm{n}|=10,|\\mathrm{~m}+\\mathrm{n}|=4$,\n\n$\\therefore|\\mathrm{m}|+|\\mathrm{n}|>|\\mathrm{m}+\\mathrm{n}|$,\n由上述结论可得: $\\mathrm{m} 、 \\mathrm{n}$ 异号,\n\n(1) 当 $\\mathrm{m}$ 为正数, $\\mathrm{n}$ 为负数时, 则 $|m|+|n|=m-n=10$, 即 $n=m-10$, 代入 $|\\mathrm{m}+\\mathrm{n}|=4$ 得: $|m+m-10|=4$, 解得: $m=7$ 或 $m=3$;\n\n(2)当 $\\mathrm{m}$ 为负数, $\\mathrm{n}$ 为正数时, 则 $|m|+|n|=-m+n=10$, 即 $n=m+10$, 代入 $|\\mathrm{m}+\\mathrm{n}|=4$ 得: $|m+m+10|=4$, 解得: $m=-7$ 或 $m=-3$;\n\n$\\therefore$ 综上所述: $m= \\pm 7$ 或 $\\pm 3 ;$\n\n故答案为: $\\pm 3$ 或 $\\pm 7$;\n\n(4) 由题意, 分以下四类:\n\n第一类: 当 $a 、 b 、 c$ 三个数都不等于 0 时,\n\n(1) 1 个正数, 2 个负数, 此时 $|a|+|b|+|c|>|a+b+c|$,\n\n(2) 两个正数, 一个负数, 此时 $|a|+|b|+|c|>|a+b+c|$,\n\n(3)三个正数, 此时 $|a|+|b|+|c|=|a+b+c|$, 不符合题意, 舍去,\n\n(4)三个负数, 此时 $|a|+|b|+|c|=|a+b+c|$, 不符合题意, 舍去;\n\n第二类: 当 $a 、 b 、 c$ 三个数中有一个等于零时,\n\n(1) 1 个 0,2 个正数, 此时 $|a|+|b|+|c|=|a+b+c|$, 不符合题意, 舍去,\n\n(2) 1 个 0,2 个负数, 此时 $|a|+|b|+|c|=|a+b+c|$, 不符合题意, 舍去,\n\n(3) 1 个 0,1 个正数, 1 个负数, 此时 $|a|+|b|+|c|>|a+b+c|$;\n\n第三类: 当 $a 、 b 、 c$ 三个数中有 2 个等于 0 时,\n\n(1) 2 个 0,1 个正数, 此时 $|\\mathbf{a}|+|\\mathbf{b}|+|c|=|\\mathbf{a}+\\mathrm{b}+\\mathrm{c}|$, 不符合题意, 舍去,\n\n(2) 2 个 0,1 个负数, 此时 $|a|+|b|+|c|=|a+b+c|$, 不符合题意, 舍去,\n\n第四类: 当 $a 、 b 、 c$ 三个数都等于 0 时, 此时 $|a|+|b|+|c|=|a+b+c|$, 不符合题意, 舍去,\n\n综上所述: 当 $|a|+|b|+|c|>|a+b+c|$ 时, 符合条件的有(1)(2)(7);\n\n故答案为: (1) (2)."} {"id": "1353", "image": [], "answer": "(1) $\\frac{1}{2}$\n\n(2) (2) (3)\n\n(3) $\\left(-\\frac{1}{3}\\right)^{n-2} ; 7^{3}$\n\n(4) $\\left(\\frac{1}{a}\\right)^{n-2}$\n\n(5) $-\\frac{31}{4}$", "solution": "null", "level": "七年级", "question": "概念学习\n\n规定: 求若干个相同的有理数(均不等于 0 )的除法运算叫做除方, 如 $2 \\div 2 \\div 2,(-3) \\div(-3) \\div$ $(-3) \\div(-3)$ 等, 类比有理数的乘方, 我们把 $2 \\div 2 \\div 2$ 记作 $2_{3}$, 读作“ 2 的 3 次商”, $(-3) \\div(-$ $3) \\div(-3) \\div(-3)$ 记作 $(-3) 4$, 读作“ -3 的 4 次商”,一般地, 我们把 $\\mathrm{n}$ 个 a ( $\\mathrm{a} \\neq 0$ ) 相除记作 $\\mathrm{a}_{\\mathrm{n}}$, 读作“ $\\mathrm{a}$ 的 $\\mathrm{n}$ 次商\".\n\n(1) 初步探究直接写出结果: $2_{3}=$ \\$ \\qquad \\$ $;$\n\n(2) 关于除方,下列说法错误的是 \\$ \\qquad \\$ ;\n\n(1)任何非零数的 2 次商都等于 1 ; (2)对于任何正整数 $\\mathrm{n},(-1){ }_{\\mathrm{n}}=-1$; (3) $3_{4}=4_{3}$; (4) 负数的奇数次商结果是负数, 负数的偶数次商结果是正数.\n\n$(3)$ 深入思考\n我们知道, 有理数的减法运算可以转化为加法运算, 除法运算能够转化为乘法运算, 那么有理数的除方运算如何转化为乘方运算呢? 例: $2_{4}=2 \\div 2 \\div 2 \\div 2=2 \\times \\frac{1}{2} \\times \\frac{1}{2} \\times \\frac{1}{2}=\\left(\\frac{1}{2}\\right)^{2}$.试一试: 仿照上面的算式,将下列运算结果直接写成乘方(幂)的形式 $(-3)_{4}=$ \\$ \\qquad \\$ $;$ $\\left(\\frac{1}{7}\\right)_{5}=$ \\$ \\qquad \\$\n(4) 想一想: 将一个非零有理数 $\\mathrm{a}$ 的 $\\mathrm{n}$ 次商写成幂的形式等于 \\$ \\qquad \\$ ;\n\n(5) 算一算: $5_{2} \\div\\left(-\\frac{1}{2}\\right)_{4} \\times\\left(-\\frac{1}{3}\\right)_{5}+\\left(-\\frac{1}{4}\\right)_{3} \\times \\frac{1}{4}=$ \\$ \\qquad \\$ .", "options": [], "subject": "算术", "analysis": "(1) $2_{3}=2 \\div 2 \\div 2=\\frac{1}{2}$;\n\n故答案为: $\\frac{1}{2}$;\n\n(2) $\\because$ 任何非零数的 2 次商等于这个数与它本身相除, 结果为 1 ,\n\n$\\therefore$ 任何非零数的 2 次商都等于 1,\n\n故(1)正确;\n\n$\\because$ 对于任何正整数 $n$, 当 $n$ 为奇数时, $(-1)_{n}=-1$, 当 $n$ 为偶数时, $(-1)_{n}=1$,\n\n$\\therefore$ (2)错误;\n\n$\\because 3_{4}=3 \\div 3 \\div 3 \\div 3=\\frac{1}{9}, 4_{3}=4 \\div 4 \\div 4=\\frac{1}{4}$,\n\n$\\therefore 3_{4} \\neq 4_{3}$.\n\n$\\therefore$ (3)错误;\n\n$\\because$ 负数的奇数次商结果是负数, 负数的偶数次商结果是正数,\n\n$\\therefore$ (4) 正确;\n\n综上, 说法错误的是: (2) (3),\n\n故答案为: (2) (3);\n\n$$\n\\text { (3) }(-3)_{4}=(-3) \\div(-3) \\div(-3) \\div(-3)=(-3) \\times\\left(-\\frac{1}{3}\\right) \\times\\left(-\\frac{1}{3}\\right) \\times\\left(-\\frac{1}{3}\\right)=\n$$\n\n$\\left(-\\frac{1}{3}\\right)^{2}$,\n\n$\\left(\\frac{1}{7}\\right)_{5}=\\frac{1}{7} \\div \\frac{1}{7} \\div \\frac{1}{7} \\div \\frac{1}{7} \\div \\frac{1}{7}=\\frac{1}{7} \\times 7 \\times 7 \\times 7 \\times 7=7^{3}$,\n\n故答案为: $\\left(-\\frac{1}{3}\\right)^{2} ; 7^{3}$;\n\n(4) $\\because \\mathrm{a}_{\\mathrm{n}}=\\underset{n \\uparrow a}{a \\div a \\underset{\\sim}{\\div} \\div \\div a}=\\mathrm{a} \\times \\frac{1}{a} \\times \\underset{(n-2) \\uparrow \\frac{1}{a}}{\\frac{1}{a} \\times \\frac{1}{a} \\times \\cdots \\times \\frac{1}{a}}=\\left(\\frac{1}{a}\\right)^{n-2}$,\n\n$\\therefore$ 将一个非零有理数 $\\mathrm{a}$ 的 $\\mathrm{n}$ 次商写成幂的形式等于 $\\left(\\frac{1}{a}\\right)^{n-2}$.\n\n故答案为: $\\left(\\frac{1}{a}\\right)^{n-2}$.\n\n(5) 原式 $=1 \\div(-2)^{2} \\times(-3)^{3}+(-4)^{1} \\times \\frac{1}{4}=1 \\times \\frac{1}{4} \\times(-27)+(-1)=-\\frac{31}{4}$."} {"id": "1355", "image": [], "answer": ") 解: $\\mathrm{M}(1)=-2, \\mathrm{M}(2)=-1, \\mathrm{M}(3)=0, \\mathrm{M}(4)=1 \\ldots$则 $M$ 对整数 $n$ 运算规律是 $n-3$,\n\n$M\\left(\\frac{1}{2}\\right)=-\\frac{1}{4}, M\\left(\\frac{1}{3}\\right)=-\\frac{1}{9}, M\\left(\\frac{1}{4}\\right)=-\\frac{1}{16}, \\ldots$\n\n则 $\\mathrm{M}$ 对分数 $\\frac{1}{m}$ 的运算规律是 $-\\frac{1}{m^{2}}$ ,\n\n$\\therefore \\mathrm{M}(28) \\times \\mathrm{M}\\left(\\frac{1}{5}\\right)=(28-3) \\times\\left(-\\frac{1}{5^{2}}\\right)=-25 \\times \\frac{1}{25}=-1$\n\n(2) 解: $-1 \\div \\mathrm{M}(39) \\div\\left[-\\mathrm{M}\\left(\\frac{1}{6}\\right)\\right]$.\n\n$=-1 \\div(39-3) \\div\\left[-\\left(-\\frac{1}{6^{2}}\\right)\\right]$,\n\n$=-1 \\div 36 \\div\\left(\\frac{1}{36}\\right)$,\n\n$=-\\frac{1}{36} \\times 36$,\n\n$=-1$.", "solution": "null", "level": "七年级", "question": "用符号 $M$ 表示一种运算, 它对整数和分数的运算结果分别如下:\n\n$\\mathrm{M}(1)=-2, \\mathrm{M}(2)=-1, \\mathrm{M}(3)=0, \\mathrm{M}(4)=1 \\ldots$\n\n$\\mathrm{M}\\left(\\frac{1}{2}\\right)=-\\frac{1}{4}, \\mathrm{M}\\left(\\frac{1}{3}\\right)=-\\frac{1}{9}, \\mathrm{M}\\left(\\frac{1}{4}\\right)=-\\frac{1}{16}, \\ldots$\n\n利用以上规律计算:\n\n(1) $\\mathrm{M}(28) \\times \\mathrm{M}\\left(\\frac{1}{5}\\right)$;\n\n(2) $-1 \\div \\mathrm{M}(39) \\div\\left[-\\mathrm{M}\\left(\\frac{1}{6}\\right)\\right]$.", "options": [], "subject": "算术", "analysis": ") 解: $\\mathrm{M}(1)=-2, \\mathrm{M}(2)=-1, \\mathrm{M}(3)=0, \\mathrm{M}(4)=1 \\ldots$则 $M$ 对整数 $n$ 运算规律是 $n-3$,\n\n$M\\left(\\frac{1}{2}\\right)=-\\frac{1}{4}, M\\left(\\frac{1}{3}\\right)=-\\frac{1}{9}, M\\left(\\frac{1}{4}\\right)=-\\frac{1}{16}, \\ldots$\n\n则 $\\mathrm{M}$ 对分数 $\\frac{1}{m}$ 的运算规律是 $-\\frac{1}{m^{2}}$ ,\n\n$\\therefore \\mathrm{M}(28) \\times \\mathrm{M}\\left(\\frac{1}{5}\\right)=(28-3) \\times\\left(-\\frac{1}{5^{2}}\\right)=-25 \\times \\frac{1}{25}=-1$\n\n(2) 解: $-1 \\div \\mathrm{M}(39) \\div\\left[-\\mathrm{M}\\left(\\frac{1}{6}\\right)\\right]$.\n\n$=-1 \\div(39-3) \\div\\left[-\\left(-\\frac{1}{6^{2}}\\right)\\right]$,\n\n$=-1 \\div 36 \\div\\left(\\frac{1}{36}\\right)$,\n\n$=-\\frac{1}{36} \\times 36$,\n\n$=-1$."} {"id": "1371", "image": [], "answer": "(3) (4) (8); (1) (6) (7)", "solution": "null", "level": "七年级", "question": "下列 8 个实数:\n\n(1) $-3 ;$ (2) $0 ;$ (3) $\\sqrt{3}$\n\n(4) $\\sqrt[3]{10} ; \\quad(5) \\frac{1}{3}$\n\n(6) $-2.4 ;$ ( $7-\\frac{10}{7}$;\n\n(8) $2 \\pi$\n\n属于无理数的有: \\$ \\qquad \\$ . (填序号)\n\n属于负数的有: \\$ \\qquad \\$ . (填序号)", "options": [], "subject": "算术", "analysis": "属于无理数的是: $\\sqrt{3}, \\sqrt[3]{10}$ 和 $2 \\pi$,属于负数的是: $-3,-2.4,--\\frac{10}{7}$.\n故答案为: (3)(4) (8), (1) (6) (7)."} {"id": "1372", "image": [], "answer": "(1) 解: $\\pm 4$\n\n(2) 解: $\\pm \\frac{7}{9}$\n\n(3) 解: $\\pm 9$", "solution": "null", "level": "七年级", "question": "求下列各数的平方根.\n\n(1) $\\sqrt{256}$;\n\n(2) $\\frac{49}{81}$;\n\n(3) $(-\\sqrt{81})^{2}$.", "options": [], "subject": "算术", "analysis": "(1) $\\because \\sqrt{256}=16, \\therefore \\sqrt{256}$ 的平方根为 $\\pm \\sqrt{14}= \\pm 4$;\n\n(2) $\\pm \\sqrt{\\frac{49}{81}}= \\pm \\frac{7}{9}$,\n\n(3) $\\because(-\\sqrt{81})^{2}=81, \\therefore(-\\sqrt{81})^{2}=$ 的平方根为 $\\pm \\sqrt{81}= \\pm 9$"} {"id": "1373", "image": [], "answer": "(1) 解: $4-3.3+(-1.7)$\n\n$=4-3.3-1.7$\n\n$=4-5$\n\n$=-1$;\n\n(2) 解: $\\frac{1}{18} \\div\\left(\\frac{1}{2}-\\frac{2}{3}+\\frac{4}{9}\\right)=\\frac{1}{18} \\div \\frac{5}{18}=\\frac{1}{5}$;\n\n(3) 解: $\\left(-\\frac{1}{2}\\right) \\div 8 \\times(-2)^{3}=-\\frac{1}{2} \\times \\frac{1}{8} \\times(-8)=\\frac{1}{2} \\times \\frac{1}{8} \\times 8=\\frac{1}{2}$;\n\n(4) 解: $\\sqrt[3]{-27}+|\\sqrt{3}-6|-(-\\sqrt{3})=-3+6-\\sqrt{3}+\\sqrt{3}=3$.", "solution": "null", "level": "七年级", "question": "计算:\n\n(1) $4-3.3+(-1.7)$\n\n(2) $\\frac{1}{18} \\div\\left(\\frac{1}{2}-\\frac{2}{3}+\\frac{4}{9}\\right)$\n\n(3) $\\left(-\\frac{1}{2}\\right) \\div 8 \\times(-2)^{3}$\n\n(4) $\\sqrt[3]{-27}+|\\sqrt{3}-6|-(-\\sqrt{3})$", "options": [], "subject": "算术", "analysis": "(1) 解: $4-3.3+(-1.7)$\n\n$=4-3.3-1.7$\n\n$=4-5$\n\n$=-1$;\n\n(2) 解: $\\frac{1}{18} \\div\\left(\\frac{1}{2}-\\frac{2}{3}+\\frac{4}{9}\\right)=\\frac{1}{18} \\div \\frac{5}{18}=\\frac{1}{5}$;\n\n(3) 解: $\\left(-\\frac{1}{2}\\right) \\div 8 \\times(-2)^{3}=-\\frac{1}{2} \\times \\frac{1}{8} \\times(-8)=\\frac{1}{2} \\times \\frac{1}{8} \\times 8=\\frac{1}{2}$;\n\n(4) 解: $\\sqrt[3]{-27}+|\\sqrt{3}-6|-(-\\sqrt{3})=-3+6-\\sqrt{3}+\\sqrt{3}=3$."} {"id": "1377", "image": [], "answer": "(1) 两\n\n(2) 9\n\n(3) $3 ; 39$\n\n(4) -48", "solution": "null", "level": "七年级", "question": "我国数学家华罗庚在一次出国访问途中, 看到飞机上邻座的乘客阅读的杂志上有一道智力题, 求 59319 的立方根, 华罗庚脱口而出, 你知道怎样迅速准确地计算出结果的吗? 请按照下面的问题试一试:\n\n(1) 由 $10^{3}=1000,100^{3}=1000000$, 确定 59319 的立方根是 \\$ \\qquad \\$位数;\n\n(2) 由 59319 的个位数是 9 , 确定 59319 的立方根的个位数是 \\$ \\qquad \\$\n(3)如果划去 59319 后面的三位 319 得到数 59 , 而 $3^{3}=27,4^{3}=64$, 由此能确定 59319 的立方根的十位数是 \\$ \\qquad \\$ ; 所以 59319 的立方根是 \\$ \\qquad \\$ $;$\n\n(4) 用类似的方法, 请说出 -110592 的立方根是 \\$ \\qquad \\$", "options": [], "subject": "算术", "analysis": "(1) $\\because 1000<59319<1000000$,\n\n$\\therefore 10<\\sqrt[3]{59319}<100$\n\n$\\therefore 59319$ 的立方根是两位数,\n\n故答案为: 两; (2) 只有个位数是 9 的立方数的个位数依然是 9 ,\n\n$\\therefore 59319$ 的立方根的个位数是 9 ,\n\n故答案为: 9 ; ( 3 ) $\\because 27<59<64$,\n\n$\\therefore 3<\\sqrt[3]{59}<4$\n\n$\\therefore \\sqrt[3]{59319}$ 的十位数是 3 ,\n\n$\\therefore \\sqrt[3]{59319}=39$,\n\n故答案为: 3,$39 ;(4)$ 根据上述知识可知,\n\n$\\therefore \\sqrt[3]{-110592}$ 是个负两位数, 十位上的数是 4 , 个位上的数是 8 , 则\n\n$\\sqrt[3]{-110592}=-48$\n\n故答案为: -48 ;"} {"id": "1378", "image": [], "answer": "(1) 解: $4<5<9 . .2<\\sqrt{5}<3$, $\\sqrt{5}$ 的小数部分 $\\mathrm{a}=\\sqrt{5}-2$.\n\n$9<13<16$,\n\n$3<\\sqrt{13}<4$,\n\n$\\sqrt{13}$ 的整数部分 $b=3$.\n\n$\\therefore \\mathrm{a}+6-\\sqrt{5}=\\sqrt{5}-2+3-\\sqrt{5}=1$\n\n(2) 解: $\\because 1<3<4$. $\\therefore 1<\\sqrt{3}<2$,\n\n$\\because$ 的整数部分是 1 , 小数部分是 $\\sqrt{3}-1$,\n\n$\\therefore \\mathrm{x}+\\mathrm{y}=10+\\sqrt{3}=10+1+(\\sqrt{3}-1)=11+(\\sqrt{3}-1)$.\n\n又 $\\because \\mathrm{x}$ 是整数, 且 $0<\\mathrm{y}<1, \\therefore \\mathrm{x}=11, \\mathrm{y}=\\sqrt{3}-1$.\n\n$\\therefore x-y=11-(\\sqrt{3}-1)=12-\\sqrt{3}$,\n\n$\\therefore \\mathrm{x}-\\mathrm{y}$ 的相反数为 $\\sqrt{3}-12$.", "solution": "null", "level": "七年级", "question": "先阅读下面的文字, 再回答问题:\n大家知道 $\\sqrt{2}$ 是无理数, 而无理数是无限不循环小数, 因此 $\\sqrt{2}$ 的小数部分我们不可能全部写出来,于是小明用 $\\sqrt{2}-1$ 来表示、 $\\sqrt{2}$ 的小数部分, 你同意小明的表示方法吗?\n\n事实上, 小明的表示方法是有道理的. 因为 $\\sqrt{2}$ 的整数部分是 1 , 所以将 $\\sqrt{2}$ 减去其整数部分, 所得的差就是 $\\sqrt{2}$ 的小数部分.\n\n例如: $\\sqrt{4}<\\sqrt{7}<\\sqrt{9}$, 即 $2<\\sqrt{7}<3$.\n\n$\\sqrt{7}$ 的整数部分为 2 , 小数部分为 $\\sqrt{7}-2$.\n\n(1) 如果 $\\sqrt{5}$ 的小数部分为 $\\mathrm{a}, \\sqrt{13}$ 的整数部分为 $\\mathrm{b}$, 求 $\\mathrm{a}+\\mathrm{b}-\\sqrt{5}$ 的值;\n\n(2) 已知 $10+\\sqrt{3}=x+y$, 其中 $x$ 是整数, 且 $0\n\n(2) 原式 $=\\frac{\\sqrt{2} \\times \\frac{\\sqrt{2}}{2}}{\\sqrt{3}-\\frac{\\sqrt{3}}{2}}=\\frac{2 \\sqrt{3}}{3}$.", "solution": "null", "level": "七年级", "question": "(6 分) 计算:\n\n(1) $4 \\sin ^{2} 60^{\\circ}-2 \\tan 45^{\\circ}+4 \\cos ^{2} 30^{\\circ}$\n\n(2) $\\frac{\\sqrt{2} \\cos 45^{\\circ}}{\\tan 60^{\\circ}-\\sin 60^{\\circ}}$.", "options": [], "subject": "算术", "analysis": "分) 计算:\n\n(1) $4 \\sin ^{2} 60^{\\circ}-2 \\tan 45^{\\circ}+4 \\cos ^{2} 30^{\\circ}$\n\n(2) $\\frac{\\sqrt{2} \\cos 45^{\\circ}}{\\tan 60^{\\circ}-\\sin 60^{\\circ}}$.\n\n考点: 特殊角的三角函数值.\n\n分析: 将特殊角的三角函数值代入运算即可.\n\n\n\n(2) 原式 $=\\frac{\\sqrt{2} \\times \\frac{\\sqrt{2}}{2}}{\\sqrt{3}-\\frac{\\sqrt{3}}{2}}=\\frac{2 \\sqrt{3}}{3}$."} {"id": "1419", "image": [], "answer": "(1) 解: $\\sqrt{25}+\\sqrt[3]{-8}-\\sqrt{6^{2}}$\n\n$$\n\\begin{gathered}\n=5+(-2)-6 \\\\\n=-3\n\\end{gathered}\n$$\n\n(2) 解: $\\sqrt{(-3)^{2}}+\\sqrt{6}+|\\sqrt{6}-3|$\n\n$$\n\\begin{gathered}\n=3+\\sqrt{6}+3-\\sqrt{6} \\\\\n=6\n\\end{gathered}\n$$", "solution": "null", "level": "七年级", "question": "计算:\n\n(1) $\\sqrt{25}+\\sqrt[3]{-8}-\\sqrt{6^{2}}$\n\n(2) $\\sqrt{(-3)^{2}}+\\sqrt{6}+|\\sqrt{6}-3|$", "options": [], "subject": "算术", "analysis": "(1) 解: $\\sqrt{25}+\\sqrt[3]{-8}-\\sqrt{6^{2}}$\n\n$$\n\\begin{gathered}\n=5+(-2)-6 \\\\\n=-3\n\\end{gathered}\n$$\n\n(2) 解: $\\sqrt{(-3)^{2}}+\\sqrt{6}+|\\sqrt{6}-3|$\n\n$$\n\\begin{gathered}\n=3+\\sqrt{6}+3-\\sqrt{6} \\\\\n=6\n\\end{gathered}\n$$"} {"id": "1421", "image": [], "answer": "解: $\\because x+1$ 的平方根是 $\\pm 2$,\n\n$\\therefore \\mathrm{x}+1=4$,\n\n$\\therefore \\mathrm{x}=3$,\n$\\because 2 x+y-2$ 的立方根是 2 ,\n\n$\\therefore 2 \\mathrm{x}+\\mathrm{y}-2=8$,\n\n把 $x$ 的值代入解得:\n\n$\\mathrm{y}=4$,\n\n$\\therefore \\mathrm{x}^{2}+\\mathrm{y}^{2}=25$,\n\n$\\therefore \\mathrm{x}^{2}+\\mathrm{y}^{2}$ 的算术平方根为 5 .", "solution": "null", "level": "七年级", "question": "已知 $x+1$ 的平方根是 $\\pm 2,2 x+y-2$ 的立方根是 2 , 求 $x^{2}+y^{2}$ 的算术平方根.", "options": [], "subject": "算术", "analysis": "解: $\\because x+1$ 的平方根是 $\\pm 2$,\n\n$\\therefore \\mathrm{x}+1=4$,\n\n$\\therefore \\mathrm{x}=3$,\n$\\because 2 x+y-2$ 的立方根是 2 ,\n\n$\\therefore 2 \\mathrm{x}+\\mathrm{y}-2=8$,\n\n把 $x$ 的值代入解得:\n\n$\\mathrm{y}=4$,\n\n$\\therefore \\mathrm{x}^{2}+\\mathrm{y}^{2}=25$,\n\n$\\therefore \\mathrm{x}^{2}+\\mathrm{y}^{2}$ 的算术平方根为 5 ."} {"id": "1443", "image": [], "answer": "(1) 解: 原式 $=5-4+1$\n\n$=2 ;$\n\n(2) 解: 原式 $=2-\\sqrt{3}-3-(-3)$\n\n$=2-\\sqrt{3}$.", "solution": "null", "level": "七年级", "question": "计算:\n\n(1) $\\sqrt{25}-\\sqrt[3]{64}-(-1)^{2021}$\n\n(2) $|\\sqrt{3}-2|-\\sqrt{(-3)^{2}}-\\sqrt[3]{-27}$.", "options": [], "subject": "算术", "analysis": "(1) 解: 原式 $=5-4+1$\n\n$=2 ;$\n\n(2) 解: 原式 $=2-\\sqrt{3}-3-(-3)$\n\n$=2-\\sqrt{3}$."} {"id": "1444", "image": [], "answer": "解:正分数集合: $\\{|-3.5|, 10 \\%$\n\n负有理数集合:\\{ - ( +4$),+(-5) \\quad \\ldots\\}$;\n\n非负整数集合: $\\{0,2018$\n\n无理数集合: $\\left\\{\\frac{\\pi}{3},-2.030030003 \\ldots\\right.$", "solution": "null", "level": "七年级", "question": "请把下列各数填入相应的集合中:\n\n$-(+4),|-3.5|, 0, \\frac{\\pi}{3}, 10 \\%, 2018,+(-5),-2.030030003 \\ldots$ (每两个 3 之间逐次加一个 $0)$.\n\n正分数集合:\\{ $\\ldots\\}$\n\n负有理数集合:\\{ $\\cdots\\}$\n\n非负整数集合: \\{\n\n无理数集合: \\{", "options": [], "subject": "算术", "analysis": "解:正分数集合: $\\{|-3.5|, 10 \\%$\n\n负有理数集合:\\{ - ( +4$),+(-5) \\quad \\ldots\\}$;\n\n非负整数集合: $\\{0,2018$\n\n无理数集合: $\\left\\{\\frac{\\pi}{3},-2.030030003 \\ldots\\right.$"} {"id": "1450", "image": [], "answer": "(1) $1+2+3+4+5$ (或 15 );225\n\n(2) $\\frac{n(n+1)}{2}$\n\n(3) 解: 由 (2) 得,\n\n$11^{3}+12^{3}+13^{3}+\\ldots+19^{3}+20^{3}$\n\n$=1^{3}+2^{3}+3^{3}+\\ldots+19^{3}+20^{3}-\\left(1^{3}+2^{3}+3^{3}+\\ldots+9^{3}+10^{3}\\right)$\n\n$=\\left(\\frac{20 \\times 21}{2}\\right)^{2}-\\left(\\frac{10 \\times 11}{2}\\right)^{2}$\n\n$=44100-3025$\n\n$=41075$", "solution": "null", "level": "七年级", "question": "观察下列一组算式的特征, 并探索规律:\n\n(1) $\\sqrt{1^{3}}=1=1$;\n\n(2) $\\sqrt{1^{3}+2^{3}}=1+2=3$;\n\n(3) $\\sqrt{1^{3}+2^{3}+3^{3}}=1+2+3=6$;\n\n(4) $\\sqrt{1^{3}+2^{3}+3^{3}+4^{3}}=1+2+3+4=10$.\n根据以上算式的规律, 解答下列问题:\n\n(1) $1^{3}+2^{3}+3^{3}+4^{3}+5^{3}=($ \\$ \\qquad \\$ )$^{2}=$ \\$ \\qquad \\$ ;\n\n(2) $\\sqrt{1^{3}+2^{3}+3^{3}+\\cdots+(n-1)^{3}+n^{3}}=$ \\$ \\qquad \\$ ; (用含 $\\mathrm{n}$ 的代数式表示)\n\n(3)简便计算: $11^{3}+12^{3}+13^{3}+\\ldots+19^{3}+20^{3}$.", "options": [], "subject": "算术", "analysis": "(1) $1+2+3+4+5$ (或 15 );225\n\n(2) $\\frac{n(n+1)}{2}$\n\n(3) 解: 由 (2) 得,\n\n$11^{3}+12^{3}+13^{3}+\\ldots+19^{3}+20^{3}$\n\n$=1^{3}+2^{3}+3^{3}+\\ldots+19^{3}+20^{3}-\\left(1^{3}+2^{3}+3^{3}+\\ldots+9^{3}+10^{3}\\right)$\n\n$=\\left(\\frac{20 \\times 21}{2}\\right)^{2}-\\left(\\frac{10 \\times 11}{2}\\right)^{2}$\n\n$=44100-3025$\n\n$=41075$"} {"id": "1451", "image": [], "answer": "(1) $\\frac{1}{\\sqrt{5}+\\sqrt{4}}=\\sqrt{5}-\\sqrt{4}$\n\n(2) $\\frac{1}{\\sqrt{n+1}+\\sqrt{n}}=\\sqrt{n+1}-\\sqrt{n} \\quad(n \\geq 1$ 的整数)\n\n(3) 解: 原式 $=\\sqrt{2}-1+\\sqrt{3}-\\sqrt{2}+\\sqrt{4}-\\sqrt{3}+\\cdots+\\sqrt{2021}-\\sqrt{2020}$ $=\\sqrt{2021}-1$.", "solution": "null", "level": "七年级", "question": "观察下列各式: $\\frac{1}{\\sqrt{2}+1}=\\sqrt{2}-1 ; \\frac{1}{\\sqrt{3}+\\sqrt{2}}=\\sqrt{3}-\\sqrt{2} ; \\frac{1}{\\sqrt{4}+\\sqrt{3}}=\\sqrt{4}-\\sqrt{3}$.\n\n(1)请根据以上规律,写出第 4 个式子: \\$ \\qquad \\$\n(2)请根据以上规律,写出第 $\\mathrm{n}$ 个式子:\n\n(3) 根据以上规律计算: $\\frac{1}{\\sqrt{2}+1}+\\frac{1}{\\sqrt{3}+\\sqrt{2}}+\\frac{1}{\\sqrt{4}+\\sqrt{3}}+\\cdots+\\frac{1}{\\sqrt{2021}+\\sqrt{2020}}$ 的值.", "options": [], "subject": "算术", "analysis": ") 第 4 个式子为: $\\frac{1}{\\sqrt{5}+\\sqrt{4}}=\\sqrt{5}-\\sqrt{4}$;\n\n故答案为: $\\frac{1}{\\sqrt{5}+\\sqrt{4}}=\\sqrt{5}-\\sqrt{4}$;\n\n(2) 第 $\\mathrm{n}$ 个式子为: $\\frac{1}{\\sqrt{n+1}+\\sqrt{n}}=\\sqrt{n+1}-\\sqrt{n}$ ( $\\mathrm{n} \\geq 1$ 的整数);\n\n故答案为: $\\frac{1}{\\sqrt{n+1}+\\sqrt{n}}=\\sqrt{n+1}-\\sqrt{n} \\quad(n \\geq 1$ 的整数 $)$;"} {"id": "1472", "image": [], "answer": "(1) $1.8 \\mathrm{x} ;(1.6 \\mathrm{x}+8)$\n\n(2) 解: 当 $x=25$ 时,\n\n在甲文具店需要花费 $1.8 \\times 25=45($ 元 $)$,\n\n在乙文具店需要花费 $1.6 \\times 25+8=48($ 元 $)$,\n\n$\\because 45<48$,\n\n$\\therefore$ 选择在甲文具店购买更优惠.", "solution": "null", "level": "七年级", "question": "某某种圆珠笔的售价是每支 2 元, 甲、乙两家文具店均有促销活动: 甲文具店全部九折; 乙文具店 20 支及以内不打折, 比 20 支多的部分打八折. 设小明需要购买的圆珠笔的数量为 $\\mathrm{x}$, 根据题意回答下列问题:\n\n(1) 若购买多于 20 支的圆珠笔, 则在甲文具店需要花费元, 在乙文具店需要花费元; (用含 $\\mathrm{x}$ 的式子表示)\n\n(2) 当 $x=25$ 时, 选择哪家文具店更优惠?", "options": [], "subject": "算术", "analysis": "(1) 解:若购买多于 20 支的圆珠笔,\n\n在甲文具店需要花费 $2 \\times 0.9 \\mathrm{x}=1.8 \\mathrm{x}($ 元 $)$,\n\n在乙文具店需要花费 $2 \\times 20+2 \\times 0.8(x-20)=40+1.6 x-32=(1.6 x+8)$ 元,\n\n故答案为: $1.8 x ;(1.6 x+8)$;"} {"id": "1474", "image": [], "answer": "(1) $(3600+60 x) ;(2250+75 x)$\n\n(2) 当 $x=40$ 时,\n\n方案(1)购买, 需付款: $3600+60 x=3600+60 \\times 40=6000$ (元);\n\n方案(2)购买. 需付款: $2250+75 x=2250+75 \\times 40=5250$ (元);\n\n按照方案 (2)购买较为合算.\n\n(3)先按方案(2)购买 30 只极品母蟹, 再按方案(1)购买 10 只至尊公蟹, 需付款: $30 \\times 150+75 \\times 10 \\times 80 \\%=5100($ 元 $)$,\n\n$\\because 5100<5250<6000$\n\n$\\therefore$ 先按方案 (2)购买 30 只极品母蟹, 再按方案 (1)购买 10 只至尊公蟹较为合算.", "solution": "null", "level": "七年级", "question": "秋风起, 桂花飘香, 也就进入了吃螃蟹的最好季节清代文人李渔把秋天称作“蟹秋”, 意为错过了螃蟹,便是错过了整个秋季,小贤去水产市场采购大闸蟹,极品母蟹每只 150 元,至尊公蟹每只 75 元. 商家在开展促销活动期间, 向客户提供以下两种优惠方案. 方案 (1):极品母蟹和至尊公蟹都按定价的 $80 \\%$ 付款; 方案(2): 买一只极品母蟹送一只至尊公蟹. 现小贤要购买极品母蟹 30 只, 至尊公蟹 $x(x>30)$ 只.\n\n(1)按方案(1)购买极品母蟹和至尊公蟹共需付款 \\$ \\qquad \\$ (用含 $\\mathrm{x}$ 的式子表示) 元;按方案(2)购买极品母蟹和至尊公蟹共需付款 \\$ \\qquad \\$ (用含 $\\mathrm{x}$ 的式子表示)元.\n\n(2)当 $x=40$ 时, 通过计算说明此时按哪种方案购买较合算.\n\n(3)若两种优惠方案可同时使用, 当 $x=40$ 时, 你能给出一种更为省钱的购买方案吗?试写出你的购买方案, 并说明理由.", "options": [], "subject": "算术", "analysis": "(1) 按方案 (1)购买, 需付款: $(150 \\times 30+75 \\mathrm{x}) \\times 80 \\%=3600+60 \\mathrm{x}$ (元)\n\n按方案(2)购买, 需付款: $150 \\times 30+75(x-30)=2250+75 x$ (元);\n\n故答案是: $(3600+60 x) ;(2250+75 x)$;"} {"id": "1495", "image": [], "answer": "(1) 解: $3\\left(x-\\frac{1}{3}\\right)=2 x+7$,\n\n去括号得: $3 x-1=2 x+7$,\n\n移项合并得: $x=8$,\n\n把 $x=8$ 代入 $3 a-8=2(x+a)-a$ 中得: $3 a-8=2(8+a)-a$,\n\n$\\mathrm{a}=12$;\n\n(2) 解: 由题意得: $b=-12, c= \\pm 1$,\n\n$\\therefore(a+b-c)^{2022}=(0 \\pm 1)^{2022}=1$.", "solution": "null", "level": "七年级", "question": "已知方程 $3\\left(x-\\frac{1}{3}\\right)=2 x+7$ 与关于 $\\mathrm{x}$ 的方程 $3 \\mathrm{a}-8=2$ ( $\\left.\\mathrm{x}+\\mathrm{a}\\right)-\\mathrm{a}$ 的解相同.\n\n(1) 求 $\\mathrm{a}$ 的值;\n\n(2) 若 $a 、 b$ 在数轴上对应的点在原点的两侧, 且到原点的距离相等, $c$ 是倒数等于本身的数,求 $(a+b-c)^{2022}$ 的值.", "options": [], "subject": "算术", "analysis": "(1) 解: $3\\left(x-\\frac{1}{3}\\right)=2 x+7$,\n\n去括号得: $3 x-1=2 x+7$,\n\n移项合并得: $x=8$,\n\n把 $x=8$ 代入 $3 a-8=2(x+a)-a$ 中得: $3 a-8=2(8+a)-a$,\n\n$\\mathrm{a}=12$;\n\n(2) 解: 由题意得: $b=-12, c= \\pm 1$,\n\n$\\therefore(a+b-c)^{2022}=(0 \\pm 1)^{2022}=1$."} {"id": "1497", "image": [], "answer": "(1) 不是\n\n(2) 解: $\\because a=3$,\n\n$\\therefore \\mathrm{x}=\\mathrm{b}-3$,\n\n$\\therefore \\mathrm{b}-3=\\frac{b}{3}$,\n\n$\\therefore \\mathrm{b}=\\frac{9}{2}$,\n\n即 $\\mathrm{b}=\\frac{9}{2}$ 时有符合要求的“定值方程”;\n\n(3) 解: 由题可知, $m n+m=4(1)$,\n\n设 $-2 x=c$, 则 $x=-\\frac{c}{2}=c+2$, 解得: $c=-\\frac{4}{3}$,\n\n$\\therefore m n+n=-\\frac{4}{3}(2)$,\n\n(1)-(2)得: $m-n=\\frac{16}{3}$,\n\n$\\therefore 5-3 \\mathrm{~m}+3 \\mathrm{n}=5-3(m-n)=5-3 \\times \\frac{16}{3}=-11$.", "solution": "null", "level": "七年级", "question": "我们规定, 若关于 $x$ 的一元一次方程 $\\mathrm{ax}=\\mathrm{b}$ 的解为 $\\mathrm{x}=\\mathrm{b}-\\mathrm{a}$, 则称该方程为 “定值方程”,例如: $2 \\mathrm{x}=4$的解为 $x=2=4-2$, 则该方程 $2 x=4$ 是“定值方程”. 请根据上述规定解答下列问题:\n\n(1) 判断方程 $4 x=6$ (回答“是”或“不是”) “定值方程”;\n\n(2)若 $\\mathrm{a}=3$, 有符合要求的“定值方程”吗? 若有, 求 $\\mathrm{b}$ 的值; 若没有, 请说明理由.\n\n(3) 若关于 $x$ 的一元一次方程 $2 x=m n+m$ 和 $-2 x=m n+n$ 都是“定值方程”, 求代数式 $5-3 m+3 n$ 的值.", "options": [], "subject": "算术", "analysis": "(1) $\\because 4 \\mathrm{x}=6$,\n\n$\\therefore x=\\frac{3}{2}$,\n\n$\\because 6-4=2, \\quad 2 \\neq \\frac{3}{2}$,\n\n$\\therefore 4 \\mathrm{x}=6$ 不是定值方程;\n\n故答案为: 不是;"} {"id": "1498", "image": [], "answer": "(1) $(100 x+3000) ;(80 x+4800)$\n\n(2) 当 $x=40$ 时, 按方案 (1)购买所需费用: $100 x+3000=7000$ (元);\n\n当 $x=40$ 时, 按方案 (2)购买所需费用: $80 x+4800=8000$ (元),\n\n因为 $7000<8000$,\n\n所以按方案 (1) 购买较为合算;\n\n(3)先按方案(1)购买夹克 30 件, 再按方案(2)购买 $\\mathrm{T}$ 恤 10 件更为省钱. 理由如下:先按方案①购买夹克 30 件所需费用 $=6000$ (元),\n\n按方案(2)购买 T 恤 10 件的费用 $=100 \\times 80 \\% \\times 10=800$ (元),\n\n所以总费用为 $6000+800=6800$ (元), 小于 7000 元,\n\n所以此种购买方案更为省钱.", "solution": "null", "level": "七年级", "question": "前进服装厂生产一种夹克和 $T$ 恤,夹克每件定价 200 元, $T$ 恤每件定价 100 元. 厂方在开展促销活动期间,向客户提供两种优惠方案:\n\n(1) 买一件夹克送一件 $T$ 恤;\n\n(2)夹克和 $T$ 恤都按定价的 $80 \\%$ 付款.\n\n现某客户要到该服装厂购买夹克 30 件, $T$ 恤 $x$ 件 $(x>30)$.\n\n(1) 若该客户按方案(1)购买, 夹克和 $T$ 恤共需付款元(用含 $x$ 的式子表示); 若该客户按方案(2)购买, 夹克和 $T$ 恤共需付款\n\n(2) 若 $x=40$, 按方案(1)购买夹克和 $T$ 恤共需付款多少元? 按方案 (2)购买夹克和 $T$ 恤共需付款多少元, 哪一种方案合算?\n\n(3)若两种优惠方案可同时使用, 当 $x=40$ 时, 你能给出一种更为省钱的购买方案吗? 试写出你的购买方案, 并说明理由.", "options": [], "subject": "算术", "analysis": "(1) 该客户按方案 (1)购买,\n\n夹克需付款 $30 \\times 200=6000($ 元 $)$,\n\n$\\mathrm{T}$ 恤需付款 $100(x-30)$ 元,\n\n夹克和 T 恤共需付款 $(100 x+3000)$ 元;\n\n若该客户按方案 购买,\n\n夹克需付款 $30 \\times 200 \\times 80 \\%=4800($ 元 $)$,\n\n$\\mathrm{T}$ 恤需付款 $100 \\times 80 \\% x=80 x$ (元),\n\n夹克和 T 恤共需付款 $(80 x+4800)$ 元;\n\n故答案为: $(100 x+3000) ;(80 x+4800)$;"} {"id": "1304", "image": ["3097.jpg"], "answer": "(1) $6 ; 4$\n\n(2) $5 t-10 ; 3 t-4$\n\n(3) 解: 根据题意得: $5 t-10=3 t-4$,\n\n解得: $t=3$;\n\n答: 当 $t=3$ 时, 点 $\\mathrm{A}$ 与点 $\\mathrm{B}$ 恰好重合.\n\n(4) 解: 存在.\n\n当 $\\mathrm{A}$ 没追上 $\\mathrm{B}$ 时, 可得由题意:\n\n$(3 t-4)-(5 t-10)=5$\n\n解得: $t=\\frac{1}{2}$;\n\n当 $A, B$ 错开后, 可得 $(5 t-10)-(3 t-4)=5$,\n\n解得: $t=\\frac{11}{2}$,\n\n$\\therefore \\mathrm{t}$ 的值为 $\\frac{1}{2}$ 或 $\\frac{11}{2}$ 秒时, 线段 $\\mathrm{AB}$ 的长为 5 .", "solution": "null", "level": "七年级", "question": "已知如图, 在数轴上有 $\\mathrm{A}, \\mathrm{B}$ 两点, 所表示的数分别为 $-10,-4$, 点 $\\mathrm{A}$ 以每秒 5 个单位长度的速度向右运动, 同时点 $\\mathrm{B}$ 以每秒 3 个单位长度的速度也向右运动, 如果设运动时间为 $\\mathrm{t}$ 秒, 解答下列问题:\n\n\n\n(1) 运动前线段 $\\mathrm{AB}$ 的长为 \\$ \\qquad \\$ ;运动 1 秒后线段 $\\mathrm{AB}$ 的长为 \\$ \\qquad \\$ ;\n\n(2) 运动 $\\mathrm{t}$ 秒后, 点 $\\mathrm{A}$, 点 $\\mathrm{B}$ 在数轴上表示的数分别为 \\$ \\qquad \\$和 \\$ \\qquad \\$ ; (用含 $\\mathrm{t}$ 的代数式表示)\n\n(3) 求 $t$ 为何值时, 点 $\\mathrm{A}$ 与点 $\\mathrm{B}$ 恰好重合;\n\n(4) 在上述运动的过程中, 是否存在某一时刻 $t$, 使得线段 $A B$ 的长为 5 , 若存在, 求 $t$ 的值; 若不存在, 请说明理由.", "options": [], "subject": "度量几何学", "analysis": "(1) 解:运动前线段 $\\mathrm{AB}$ 的长为 $(-4)-(-10)=6$; 运动 1 秒后线段 $\\mathrm{AB}$ 的长为 $(-$ 1) - $(-5)=4$;\n\n故答案为: $6 ; 4$.\n\n(2)\n\n解: 运动 $\\mathrm{t}$ 秒后, 用 $\\mathrm{t}$ 表示 $\\mathrm{A}, \\mathrm{B}$ 分别为 $5 \\mathrm{t}-10,3 \\mathrm{t}-4$;\n\n故答案为: $5 t-10,3 t-4$."} {"id": "1305", "image": ["3098.jpg"], "answer": "(1) $-6 ; 1 ; 4$\n\n(2) $7 ; 10$\n\n(3) 解: $\\because A C=10$,\n\n$\\therefore$ 点 $B$ 到点 $\\mathrm{A}$ 和点 $C$ 的距离都是 5,\n\n此时将点 $B$ 向左移动 2 个单位即可.", "solution": "null", "level": "七年级", "question": "如图, 在数轴上有 $A 、 B 、 C$ 这三个点.\n\n\n\n回答:\n(1) $A$ 、\n$B$ 、\n$C$ 这三个点表示的数各是多少?\n\n$A:$ \\$ \\qquad \\$ ; $B$ : \\$ \\qquad \\$ ; $\\quad C$ : \\$ \\qquad \\$ ;\n\n(2) $A 、 B$ 两点间的距离是 \\$ \\qquad \\$ , $A 、 C$ 两点间的距离是 \\$ \\qquad \\$ ;\n\n(3) 应怎样移动点 $B$ 的位置, 使点 $B$ 到点 $A$ 和点 $C$ 的距离相等?", "options": [], "subject": "度量几何学", "analysis": "(1) 根据图示, 知 A、 $B 、 C$ 这三个点表示的数各是 $-6 、 1 、 4$,故答案为 $-6 、 1 、 4 ;$\n\n(2) 根据图示知 $A B=|-6-1|=7 ; A C=|-6-4|=10$,故答案为: 7 ; 10 ;"} {"id": "1308", "image": ["3102.jpg", "3103.jpg", "3103.jpg"], "answer": ") $-1 ; 3$\n\n$\\because \\mathrm{AB}=4$, 且 $\\mathrm{OB}=3 \\mathrm{OA}, \\mathrm{A} 、 \\mathrm{~B}$ 对应的数分别是 $\\mathrm{a} 、 \\mathrm{~b}, \\quad \\therefore a=-1, b=3$故答案为: $-1,3$\n\n\n\n(2) 解: (1) 当 $P$ 点在 $A$ 点左侧时, $P A$\n\n(2)当 $P$ 点位于 $A 、 B$ 两点之间\n\n$$\n\\begin{gathered}\n\\because P A=2 P B \\\\\n\\therefore x+1=2(3-x)\n\\end{gathered}\n$$\n\n解得 $x=\\frac{5}{3}$\n\n(3) 当 $P$ 点在 $B$ 点右侧时\n\n$$\n\\begin{gathered}\n\\because P A=2 P B \\\\\n\\therefore x+1=2(x-3)\n\\end{gathered}\n$$\n\n解得 $x=7$\n\n故 $\\mathrm{x}$ 的值为解得 $\\frac{5}{3}$ 或 7 .\n\n(3) 解: $\\mathrm{t}$ 秒后, $\\mathrm{A}$ 点的值为 $(-1-t), \\mathrm{P}$ 点的值为 $2 \\mathrm{t}, \\mathrm{B}$ 点的值为 $(3+3 t)$\n\n$$\n\\begin{gathered}\n\\therefore 3(3+3 t-2 t)-[2 t-(-1-t)] \\\\\n=9-3 t-(2 t+1+t) \\\\\n=9+3 t-3 t-1 \\\\\n=8\n\\end{gathered}\n$$\n\n所以 3PB-PA 的值为定值, 不随着时间 $\\mathrm{t}$ 的变化而改变.", "solution": "null", "level": "七年级", "question": "数轴上两点 $A 、 B, A$ 在 $B$ 左边, 原点 $O$ 是线段 $A B$ 上的一点, 已知 $A B=4$, 且 $O B=3 O A$. $A$ 、 $B$ 对应的数分别是 $a 、 b$, 点 $P$ 为数轴上的一动点, 其对应的数为 $x$.\n\n\n\n(1) $\\mathrm{a}=$ \\$ \\qquad \\$ , $b=$ \\$ \\qquad \\$ , 并在数轴上面标出 $\\mathrm{A} 、 \\mathrm{~B}$ 两点;\n\n(2) 若 $\\mathrm{PA}=2 \\mathrm{~PB}$, 求 $\\mathrm{x}$ 的值;\n\n(3) 若点 $\\mathrm{P}$ 以每秒 2 个单位长度的速度从原点 $\\mathrm{O}$ 向右运动, 同时点 $\\mathrm{A}$ 以每秒 1 个单位长度的速度向左运动, 点 $\\mathrm{B}$ 以每秒 3 个单位长度的速度向右运动, 设运动时间为 $\\mathrm{t}$ 秒, 请问在运动过程中, 3PB-PA 的值是否随着时间 $\\mathrm{t}$ 的变化而改变? 若变化, 请说明理由若不变, 请求其值.", "options": [], "subject": "度量几何学", "analysis": ") $-1 ; 3$\n\n$\\because \\mathrm{AB}=4$, 且 $\\mathrm{OB}=3 \\mathrm{OA}, \\mathrm{A} 、 \\mathrm{~B}$ 对应的数分别是 $\\mathrm{a} 、 \\mathrm{~b}, \\quad \\therefore a=-1, b=3$故答案为: $-1,3$\n\n\n\n(2) 解: (1) 当 $P$ 点在 $A$ 点左侧时, $P A$\n\n(2)当 $P$ 点位于 $A 、 B$ 两点之间\n\n$$\n\\begin{gathered}\n\\because P A=2 P B \\\\\n\\therefore x+1=2(3-x)\n\\end{gathered}\n$$\n\n解得 $x=\\frac{5}{3}$\n\n(3) 当 $P$ 点在 $B$ 点右侧时\n\n$$\n\\begin{gathered}\n\\because P A=2 P B \\\\\n\\therefore x+1=2(x-3)\n\\end{gathered}\n$$\n\n解得 $x=7$\n\n故 $\\mathrm{x}$ 的值为解得 $\\frac{5}{3}$ 或 7 .\n\n(3) 解: $\\mathrm{t}$ 秒后, $\\mathrm{A}$ 点的值为 $(-1-t), \\mathrm{P}$ 点的值为 $2 \\mathrm{t}, \\mathrm{B}$ 点的值为 $(3+3 t)$\n\n$$\n\\begin{gathered}\n\\therefore 3(3+3 t-2 t)-[2 t-(-1-t)] \\\\\n=9-3 t-(2 t+1+t) \\\\\n=9+3 t-3 t-1 \\\\\n=8\n\\end{gathered}\n$$\n\n所以 3PB-PA 的值为定值, 不随着时间 $\\mathrm{t}$ 的变化而改变."} {"id": "2433", "image": ["3523.jpg"], "answer": "略", "solution": "null", "level": "七年级", "question": "(本题 6 分) 如图, $\\triangle A B C$ 中, $\\angle B C A=90^{\\circ}, \\angle A=30^{\\circ}$, 以 $A B$ 为直径画 $\\odot O$,延长 $A B$ 到 $D$, 使 $B D$ 等于 $\\odot O$ 的半径. 求证: $C D$ 是 $\\odot O$ 的切线.\n\n", "options": [], "subject": "度量几何学", "analysis": "略"} {"id": "2438", "image": ["3527.jpg", "3528.jpg"], "answer": "(1) 2(2) ~ s=\\frac{1}{2} \\mathrm{~L} \\cdot \\mathrm{r}(3) \\mathrm{s}=\\frac{1}{2} \\mathrm{~L} \\cdot \\mathrm{r} \\quad", "solution": "null", "level": "七年级", "question": "(本题 8 分) 阅读材料: 如图 (1), $\\triangle A B C$ 的周长为 $L$, 内切圆 $O$ 的半径为 $r$, 连结 $O A$, $O B, \\triangle A B C$ 被划分为三个小三角形, 用 $\\mathrm{S}_{\\triangle \\mathrm{ABC}}$ 表示 $\\triangle A B C$ 的面积.\n\n$\\because S_{\\triangle A B C}=S_{\\triangle O A B}+S_{\\triangle O B C}+S_{\\triangle O C A}$\n\n$$\n\\text { 又 } \\because S_{\\triangle O A B}=\\frac{1}{2} A B \\cdot r, S_{\\triangle O B C}=\\frac{1}{2} B C \\cdot r, S_{\\triangle O C A}\n$$\n\n$=\\frac{1}{2} A C \\cdot \\mathrm{r}$\n\n$\\therefore S_{\\triangle A B C}=\\frac{1}{2} A B \\cdot r+\\frac{1}{2} B C \\cdot r+\\frac{1}{2} C A \\cdot r=\\frac{1}{2} L \\cdot r($ 可作为三角形内切圆半径公式 $)$\n\n(1) 理解与应用: 利用公式计算边长分为 $5,12,13$ 的三角形内切圆半径;\n\n(2)类比与推理:若四边形 $A B C D$ 存在内切圆(与各边都相切的圆,如图 (2) 且面积为 $S$, 各边长分别为 $a, b, c, d$, 试推导四边形的内切圆半径公式;\n\n(3) 拓展与延伸: 若一个 $n$ 边形 $(n$ 为不小于 3 的整数) 存在内切圆, 且面积为 $S$,各边长分别为 $a_{1}, a_{2}, a_{3}, \\cdots a_{n}$, 合理猜想其内切圆半径公式 (不需说明理由).\n\n\n\n(1)\n\n\n\n$(2)$", "options": [], "subject": "度量几何学", "analysis": "(1) 2(2) ~ s=\\frac{1}{2} \\mathrm{~L} \\cdot \\mathrm{r}(3) \\mathrm{s}=\\frac{1}{2} \\mathrm{~L} \\cdot \\mathrm{r} \\quad"} {"id": "1376", "image": ["3114.jpg", "3115.jpg", "3116.jpg", "3117.jpg", "3115.jpg", "3116.jpg", "3117.jpg"], "answer": "(1) $C_{1}, C_{4}$\n\n(2) (1) 设点 $P$ 表示的数为 $x$,\n\n\n\n如图, 当点 $P_{1}$ 在点 $\\mathrm{A}$ 左侧时, $P_{1} B=2 P_{1} A$,\n\n则 $30-x=2(-10-x)$,\n\n解得 $x=-50$.\n\n所以点 $P_{1}$ 表示的数为 -50 ;\n\n如图, 当点 $P_{2}$ 在线段 $\\mathrm{AB}$ 上且 $P_{2} B=2 P_{2} A$ 时,\n\n\n\n则 $30-x=2(x+10)$,\n\n解得 $x=\\frac{10}{3}$.\n\n所以点 $P_{2}$ 表示的数为 $\\frac{10}{3} ;$\n\n如图, 当点 $P_{3}$ 在线段 $\\mathrm{AB}$ 上且 $P_{3} A=2 P_{3} B$ 时,\n\n\n\n则 $\\mathrm{x}+10=2(30-\\mathrm{x})$,\n解得 $x=\\frac{50}{3}$.\n\n所以点 $P_{3}$ 表示的数为 $\\frac{50}{3}$.\n\n综上所述, 当点 $\\mathrm{P}$ 在点 $\\mathrm{B}$ 的左侧时, 点 $\\mathrm{P}$ 表示的数为 -50 或 $\\frac{10}{3}$ 或 $\\frac{50}{3}$.\n\n(2)若点 $P$ 在点 $B$ 的右侧, 点 $P, A, B$ 中, 有一个点恰好是其它两个点的“联盟点”, 写出此时点 $P$ 表示的数 \\$ \\qquad \\$\n50 或 70 或 110", "solution": "null", "level": "七年级", "question": "对于数轴上的 $A, B, C$ 三点, 给出如下定义: 若其中一个点与其它两个点的距离恰好满足 2 倍的数量关系, 则称该点是其它两个点的“联盟点”。\n\n例如数轴上点 A,B,C 所表示的数分别为 $1,3,4$, 此时点 B 是点 $A, C$ 的“联盟点”.\n\n\n\n(1) 若点 $\\mathrm{A}$ 表示数 -2 ,点 $\\mathrm{B}$ 表示的数 2 , 下列各数 $-\\frac{2}{3}, 0,4,6$ 所对应的点分别 $\\mathrm{C}_{1}, \\mathrm{C}_{2}, \\mathrm{C}_{3}, \\mathrm{C}_{4}$,其中是点 $A, B$ 的“联盟点”的是\n\n(2) 点 $A$ 表示数 -10, 点 $B$ 表示的数 $30, P$ 在为数轴上一个动点:\n\n(1) 若点 $P$ 在点 $B$ 的左侧, 且点 $P$ 是点 $A, B$ 的“联盟点”, 求此时点 $P$ 表示的数;", "options": [], "subject": "度量几何学", "analysis": "(1) $C_{1}, C_{4}$\n\n(2) (1) 设点 $P$ 表示的数为 $x$,\n\n\n\n如图, 当点 $P_{1}$ 在点 $\\mathrm{A}$ 左侧时, $P_{1} B=2 P_{1} A$,\n\n则 $30-x=2(-10-x)$,\n\n解得 $x=-50$.\n\n所以点 $P_{1}$ 表示的数为 -50 ;\n\n如图, 当点 $P_{2}$ 在线段 $\\mathrm{AB}$ 上且 $P_{2} B=2 P_{2} A$ 时,\n\n\n\n则 $30-x=2(x+10)$,\n\n解得 $x=\\frac{10}{3}$.\n\n所以点 $P_{2}$ 表示的数为 $\\frac{10}{3} ;$\n\n如图, 当点 $P_{3}$ 在线段 $\\mathrm{AB}$ 上且 $P_{3} A=2 P_{3} B$ 时,\n\n\n\n则 $\\mathrm{x}+10=2(30-\\mathrm{x})$,\n解得 $x=\\frac{50}{3}$.\n\n所以点 $P_{3}$ 表示的数为 $\\frac{50}{3}$.\n\n综上所述, 当点 $\\mathrm{P}$ 在点 $\\mathrm{B}$ 的左侧时, 点 $\\mathrm{P}$ 表示的数为 -50 或 $\\frac{10}{3}$ 或 $\\frac{50}{3}$.\n\n(2)若点 $P$ 在点 $B$ 的右侧, 点 $P, A, B$ 中, 有一个点恰好是其它两个点的“联盟点”, 写出此时点 $P$ 表示的数 \\$ \\qquad \\$\n50 或 70 或 110"} {"id": "1379", "image": ["3118.jpg"], "answer": ") 解: $\\because$ 点 $\\mathrm{C}$ 表示的数是 $6, B C=4$\n\n$\\therefore$ 点 $\\mathrm{B}$ 表示的数为 $6-4=2 \\because A B=12 \\therefore$ 点 $\\mathrm{A}$ 表示的数为 $2-12=-10$\n\n$\\therefore \\mathrm{A}$ 表示的数是-10, B 表示的数是 2 .\n\n(2) 解: $\\because A B=12, M$ 是 $A B$ 的中点.\n$\\because \\mathrm{CN}=3$\n\n当点 $N$ 在点 $C$ 的左侧时, $B N=B C-C N=1$, 此时 $M N=B M+B N=6+1=7$\n\n当点 $N$ 在点 $C$ 的右侧时, $B N=B C+C N=7$, 此时 $M N=B M+B N=6+9=13$\n\n综上所述, MN 的值为 7 或 13\n\n(3) 解: $\\because \\mathrm{A}$ 表示的数是 -10 , 即 $\\mathrm{OA}=10$\n\n$\\mathrm{C}$ 表示的数是 6, 即 $\\mathrm{OC}=6$\n\n又 $\\because$ 点 $P 、$ 点 $Q$ 同时出发, 且运动的时间为 $t$\n\n$\\therefore \\mathrm{AP}=6 \\mathrm{t}, \\mathrm{CQ}=3 \\mathrm{t}$,\n\n$\\therefore \\mathrm{OP}=\\mathrm{OA}-\\mathrm{AP}=10-6 \\mathrm{t}, \\mathrm{OQ}=\\mathrm{OC}-\\mathrm{CQ}=6-3 \\mathrm{t}$\n\n当原点 $O$ 为 $P Q$ 的中点时, $O P=O Q$\n\n$\\therefore 10-6 \\mathrm{t}=6-3 \\mathrm{t}$.\n\n解得 $\\mathrm{t}=\\frac{4}{3}$\n\n$\\therefore$ 当 $\\mathrm{t}=\\frac{4}{3}$ 时, 原点 $\\mathrm{O}$ 为 $\\mathrm{PQ}$ 的中点.", "solution": "null", "level": "七年级", "question": "如图, 已知 $A, B, C$ 是数轴上的三点, 点 $\\mathrm{C}$ 表示的数是 $6, \\quad B C=4, A B=12$.\n\n\n\n(1) 写出数轴上点 $A$, 点 $B$ 表示的数;\n\n(2) 点 $\\mathrm{M}$ 为线段 $A B$ 的中点, $C N=3$, 求 $M N$ 的长;\n\n(3) 动点 $P, Q$ 分别从 $A, C$ 同时出发, 点 $\\mathrm{P}$ 以每秒 6 个单位长度的速度沿数轴向右匀速运动, 点 $\\mathrm{Q}$以每秒 3 个单位长度的速度沿数轴向左匀速运动, 求 $\\mathrm{t}$ 为何值时, 原点 $\\mathrm{O}$ 恰好为线段 $P Q$ 的中点.", "options": [], "subject": "度量几何学", "analysis": ") 解: $\\because$ 点 $\\mathrm{C}$ 表示的数是 $6, B C=4$\n\n$\\therefore$ 点 $\\mathrm{B}$ 表示的数为 $6-4=2 \\because A B=12 \\therefore$ 点 $\\mathrm{A}$ 表示的数为 $2-12=-10$\n\n$\\therefore \\mathrm{A}$ 表示的数是-10, B 表示的数是 2 .\n\n(2) 解: $\\because A B=12, M$ 是 $A B$ 的中点.\n$\\because \\mathrm{CN}=3$\n\n当点 $N$ 在点 $C$ 的左侧时, $B N=B C-C N=1$, 此时 $M N=B M+B N=6+1=7$\n\n当点 $N$ 在点 $C$ 的右侧时, $B N=B C+C N=7$, 此时 $M N=B M+B N=6+9=13$\n\n综上所述, MN 的值为 7 或 13\n\n(3) 解: $\\because \\mathrm{A}$ 表示的数是 -10 , 即 $\\mathrm{OA}=10$\n\n$\\mathrm{C}$ 表示的数是 6, 即 $\\mathrm{OC}=6$\n\n又 $\\because$ 点 $P 、$ 点 $Q$ 同时出发, 且运动的时间为 $t$\n\n$\\therefore \\mathrm{AP}=6 \\mathrm{t}, \\mathrm{CQ}=3 \\mathrm{t}$,\n\n$\\therefore \\mathrm{OP}=\\mathrm{OA}-\\mathrm{AP}=10-6 \\mathrm{t}, \\mathrm{OQ}=\\mathrm{OC}-\\mathrm{CQ}=6-3 \\mathrm{t}$\n\n当原点 $O$ 为 $P Q$ 的中点时, $O P=O Q$\n\n$\\therefore 10-6 \\mathrm{t}=6-3 \\mathrm{t}$.\n\n解得 $\\mathrm{t}=\\frac{4}{3}$\n\n$\\therefore$ 当 $\\mathrm{t}=\\frac{4}{3}$ 时, 原点 $\\mathrm{O}$ 为 $\\mathrm{PQ}$ 的中点."} {"id": "1400", "image": ["3121.jpg"], "answer": "(1) 无理; $\\pi$\n\n(2) $4 \\pi$ 或 $-4 \\pi$\n\n(3) $4 ; 3 ; 26 \\pi ;-6 \\pi$", "solution": "null", "level": "七年级", "question": "如图, 半径为 1 个单位的圆片上有一点 $A$ 与数轴上的原点重合, $A B$ 是圆片的直径. (注: 结果保留 $\\pi)$\n\n\n\n(1) 把圆片沿数轴向右滚动半周, 点 $B$ 到达数轴上点 $C$ 的位置, 点 $C$ 表示的数是数 (填“无理”或“有理”), 这个数是\n\n(2)把圆片沿数轴滚动 2 周, 点 $A$ 到达数轴上点 $D$ 的位置, 点 $D$ 表示的数是\n\n(3) 圆片在数轴上向右滚动的周数记为正数, 圆片在数轴上向左滚动的周数记为负数, 依次运动情况记录如下: $+2,-1,+3,-4,-3$.\n\n(1) 第 \\$ \\qquad \\$次滚动后, $A$ 点距离原点最近, 第 \\$ \\qquad \\$次滚动后, $A$ 点距离原点最远.\n\n(2) 当圆片结束运动时, $A$ 点运动的路程共有 \\$ \\qquad \\$ , 此时点 $A$ 所表示的数是 . \\$ \\qquad \\$", "options": [], "subject": "度量几何学", "analysis": "(1) 把圆片沿数轴向左滚动半周, 点 $\\mathrm{B}$ 到达数轴上点 $\\mathrm{C}$ 的位置, 点 $\\mathrm{C}$ 表示的数是无理数, 这个数是 $\\pi$;\n\n故答案为: 无理, $\\pi$; (2) 把圆片沿数轴滚动 2 周, 点 A 到达数轴上点 D 的位置, 点 D 表示的数是 $4 \\pi$ 或 $-4 \\pi$\n\n故答案为: $4 \\pi$ 或 $-4 \\pi$; (3) (1) $\\because$ 圆片在数轴上向右滚动的周数记为正数, 圆片在数轴上向左滚动的周数记为负数, 依次运动情况记录如下: $+2,-1,+3,-4,-3$,\n\n$\\therefore$ 第 4 次滚动后, $\\mathrm{A}$ 点距离原点最近, 第 3 次滚动后, $\\mathrm{A}$ 点距离原点最远,故答案为: 4,3 ;\n\n(2) $\\because|+2|+|-1|+|+3|+|-4|+|-3|=13$,\n\n$\\therefore 13 \\times 2 \\pi \\times 1=26 \\pi$,\n\n$\\therefore \\mathrm{A}$ 点运动的路程共有 $26 \\pi ;$\n\n$\\because(+2)+(-1)+(+3)+(-4)+(-3)=-3$,\n\n$(-3) \\times 2 \\pi=-6 \\pi$,\n\n$\\therefore$ 此时点 A 所表示的数是: $-6 \\pi$,\n\n故答案为: $26 \\pi,-6 \\pi$."} {"id": "1401", "image": [], "answer": "(1) $9 ; \\sqrt{83}-9$\n\n(2) 解: 正确;\n\n理由: $\\because \\sqrt{83}$ 的整数部分为 $\\mathrm{x}, \\sqrt{97}$ 的小数部分记为 $\\mathrm{y}$,\n\n$\\therefore \\mathrm{x}=9, \\mathrm{y}=\\sqrt{97}-9$,\n\n则 $\\mathrm{x}+\\mathrm{y}=\\sqrt{97}$\n\n(3) 解: $\\because \\sqrt{35}$ 的整数部分为 $\\mathrm{a}, \\sqrt{35}$ 的小数部分为 $\\mathrm{b}$,\n\n$\\therefore \\mathrm{a}=5, \\mathrm{~b}=\\sqrt{35}-5$\n\n$\\therefore \\mathrm{a}-2 \\mathrm{~b}+2 \\sqrt{35}=5-2(\\sqrt{35}-5)+2 \\sqrt{35}=15$.", "solution": "null", "level": "七年级", "question": "阅读下面的文字, 解答问题\n\n大家知道, $\\sqrt{2}$ 是无理数, 而无理数是无限不循环小数, 因此 $\\sqrt{2}$ 的小数部分我们不可能全部地写出来, 于是小明用 $\\sqrt{2}-1$ 来表示 $\\sqrt{2}$ 的小数部分, 你同意小明的表示方法吗?\n\n事实上,小明的表示方法是有道理的,因为 $\\sqrt{2}$ 的整数部分是 1 , 将这个数减去其整数部分, 差就是小数部分.\n\n又例如因为 $\\sqrt{4}<\\sqrt{7}<\\sqrt{9}$, 即 $2<\\sqrt{7}<3$, 所以行的整数部分为 2 , 小数部分为 $\\sqrt{7}-2$.\n\n请解答\n\n(1) $\\sqrt{83}$ 的整数部分为 \\$ \\qquad \\$ ; 小数部分为 \\$ \\qquad \\$ ;\n\n(2) 有人说, 如果 $\\sqrt{83}$ 的整数部分为 $x, \\sqrt{97}$ 的小数部分记为 $\\mathrm{y}$, 则 $\\mathrm{x}+\\mathrm{y}=\\sqrt{97}$, 你认为对吗?为什么?\n\n(3) 如果 $\\sqrt{35}$ 的整数部分为 $a, \\sqrt{35}$ 的小数部分为 $\\mathrm{b}$, 求 $\\mathrm{a}-2 \\mathrm{~b}+2 \\sqrt{35}$ 的值.", "options": [], "subject": "度量几何学", "analysis": "(1) $\\because 9<\\sqrt{83}<10$,\n\n$\\therefore \\sqrt{83}$ 的整数部分为 9 ;\n\n小数部分为: $\\sqrt{83}-9$;\n\n故答案为: $9 ; \\sqrt{83}-9$"} {"id": "1402", "image": [], "answer": "(1) 解:由题意可知 $\\mathrm{a}^{2}=69(a>0)$,解得 $a=\\sqrt{69}$\n\n(2) $\\because \\mathrm{a}^{2}=69$\n\n$64<69<91$\n\n$\\sqrt{64}<\\sqrt{69}<\\sqrt{81}$\n即 $8<\\sqrt{69}<9$\n\n$\\mathrm{m}=8, \\mathrm{n}=9$\n\n(3)\n\n$\\sqrt[3]{-8}+\\sqrt{9}$\n\n$=-2-3$\n\n$=-5$", "solution": "null", "level": "七年级", "question": "如果一个正方形 $\\mathrm{ABCD}$ 的面积为 69 .\n\n(1) 求正方形 $\\mathrm{ABCD}$ 的边长 $\\mathrm{a}$.\n\n(2) 正方形 $\\mathrm{ABCD}$ 的边长满足 $\\mathrm{m}<\\mathrm{a}<\\mathrm{n}, \\mathrm{m}, \\mathrm{n}$ 表示两个连续正整数, 求 $\\mathrm{m}, \\mathrm{n}$ 的值.\n\n(3) $\\mathrm{m}, \\mathrm{n}$ 在满足(2)的条件下, 求 $\\sqrt{3} \\sqrt{-m}-\\sqrt{n}$ 的值.", "options": [], "subject": "度量几何学", "analysis": "(1) 解:由题意可知 $\\mathrm{a}^{2}=69(a>0)$,解得 $a=\\sqrt{69}$\n\n(2) $\\because \\mathrm{a}^{2}=69$\n\n$64<69<91$\n\n$\\sqrt{64}<\\sqrt{69}<\\sqrt{81}$\n即 $8<\\sqrt{69}<9$\n\n$\\mathrm{m}=8, \\mathrm{n}=9$\n\n(3)\n\n$\\sqrt[3]{-8}+\\sqrt{9}$\n\n$=-2-3$\n\n$=-5$"} {"id": "1403", "image": [], "answer": "(1) $2 ; \\sqrt{7}-2$\n\n(2) $-3 ; 3-\\sqrt{7}$\n\n(3) 解: $\\because 5<3+\\sqrt{7}<6$,\n\n$\\therefore \\mathrm{m}=5, \\mathrm{n}=3+\\sqrt{7}-5=\\sqrt{7}-2$,\n\n$\\therefore|m-n|$\n\n$=|5-(\\sqrt{7}-2)|$\n\n$=|7-\\sqrt{7}|$\n\n$=7-\\sqrt{7}$.", "solution": "null", "level": "七年级", "question": "阅读下面文字,然后回答问题.\n\n给出定义: 一个实数的整数部分是不大于这个数的最大数, 这个实数的小数部分为这个数与它的整数部分的差的绝对值. 例如: 2.4 的整数部分为 2 , 小数部分为 $2.4-2=0.4 ; \\sqrt{2}$ 的整数部分为 1 , 小数部分可用 $\\sqrt{2}-1$ 表示; 再如, -2.6 的整数部分为 -3 , 小数部分为 $|-2.6-(-3)|=0.4$. 由此我们得到一个真命题: 如果 $\\sqrt{2}=x+y$, 其中 $x$ 是整数, 且 $0", "solution": "null", "level": "七年级", "question": "(8 分) 如图, 已知斜坡 $\\mathrm{AB}$ 长 60 米, 坡角 (即 $\\angle \\mathrm{BAC}$ ) 为 $30^{\\circ}, \\mathrm{BC} \\perp \\mathrm{AC}$, 现计划在斜坡中点 D 处挖去部分坡体 (用阴影表示) 修建一个平行于水平线 CA 的平台 DE 和一条新的斜坡 BE. (请将下面 2 小题的结果都精确到 0.1 米, 参考数据: $\\sqrt{3} \\approx 1.732$ ).\n\n(1) 若修建的斜坡 $\\mathrm{BE}$ 的坡角(即 $\\angle \\mathrm{BEF}$ ) 不大于 $45^{\\circ}$, 则平台 $\\mathrm{DE}$ 的长最多为 11.0 米;\n\n(2) 一座建筑物 $\\mathrm{GH}$ 距离坡角 $\\mathrm{A}$ 点 27 米远(即 $\\mathrm{AG}=27$ 米), 小明在 $\\mathrm{D}$ 点测得建筑物顶部 $\\mathrm{H}$ 的仰角 (即 $\\angle H D M$ ) 为 $30^{\\circ}$. 点 B、C、A、G、H 在同一个平面内, 点 C、A、G 在同一条直线上,且 $\\mathrm{HG} \\perp \\mathrm{CG}$, 问建筑物 $\\mathrm{GH}$ 高为多少米?\n\n", "options": [], "subject": "度量几何学", "analysis": "解: (1) $\\because$ 修建的斜坡 $\\mathrm{BE}$ 的坡角(即 $\\angle \\mathrm{BEF})$ 不大于 $45^{\\circ}$,\n\n$\\therefore \\angle \\mathrm{BEF}$ 最大为 $45^{\\circ}$,\n\n当 $\\angle \\mathrm{BEF}=45^{\\circ}$ 时, $\\mathrm{EF}$ 最短, 此时 $\\mathrm{ED}$ 最长,\n\n$\\because \\angle \\mathrm{DAC}=\\angle \\mathrm{BDF}=30^{\\circ}, \\mathrm{AD}=\\mathrm{BD}=30$,\n\n$\\therefore \\mathrm{BF}=\\mathrm{EF}=\\frac{1}{2} \\mathrm{BD}=15$\n\n$\\mathrm{DF}=15 \\sqrt{3}$,\n\n故: $\\mathrm{DE}=\\mathrm{DF}-\\mathrm{EF}=15(\\sqrt{3}-1) \\approx 11.0$;\n\n(2) 过点 $\\mathrm{D}$ 作 $\\mathrm{DP} \\perp \\mathrm{AC}$, 垂足为 $\\mathrm{P}$.\n\n在 Rt $\\triangle \\mathrm{DPA}$ 中, $\\mathrm{DP}=\\frac{1}{2} \\mathrm{AD}=\\frac{1}{2} \\times 30=15$,\n\n$\\mathrm{PA}=\\mathrm{AD} \\cdot \\cos 30^{\\circ}=\\frac{\\sqrt{3}}{2} \\times 30=15 \\sqrt{3}$.\n\n在矩形 DPGM 中, $\\mathrm{MG}=\\mathrm{DP}=15, \\mathrm{DM}=\\mathrm{PG}=15 \\sqrt{3}+27$,在 Rt $\\triangle \\mathrm{DMH}$ 中, $\\mathrm{HM}=\\mathrm{DM} \\cdot \\tan 30^{\\circ}=\\frac{\\sqrt{3}}{3} \\times(15 \\sqrt{3}+27)=15+9 \\sqrt{3}$.\n\n$\\mathrm{GH}=\\mathrm{HM}+\\mathrm{MG}=15+15+9 \\sqrt{3} \\approx 45.6$.\n\n答: 建筑物 $\\mathrm{GH}$ 高约为 45.6 米.\n\n"} {"id": "2464", "image": ["3554.jpg", "3555.jpg", "3555.jpg"], "answer": "解: (1) 过点 $\\mathrm{B}$ 作 $\\mathrm{BF} \\perp \\mathrm{AD}$, 交 $\\mathrm{DA}$ 的延长线于点 $\\mathrm{F}$.\n\n由题意得: $\\angle \\mathrm{BAF}=\\angle \\mathrm{ABD}+\\angle \\mathrm{ADB}=15^{\\circ}+45^{\\circ}=60^{\\circ}$,在 Rt $\\triangle \\mathrm{BFA}$ 中, $\\mathrm{BF}=\\mathrm{AB} \\sin 60^{\\circ}=4 \\sqrt{3} \\times \\frac{\\sqrt{3}}{2}=6$ (千米),\n\n$\\mathrm{AF}=\\mathrm{AB} \\cos 60^{\\circ}=4 \\sqrt{3} \\times \\frac{1}{2}=2 \\sqrt{3}$ (千米).\n\n$\\because \\mathrm{CD} \\perp \\mathrm{AD}, \\angle \\mathrm{BDC}=45^{\\circ}$,\n\n$\\therefore \\angle \\mathrm{BDF}=45^{\\circ}$,\n\n在 Rt $\\triangle \\mathrm{BFD}$ 中, $\\because \\angle \\mathrm{BDF}=45^{\\circ}$,\n\n$\\therefore \\mathrm{DF}=\\mathrm{BF}=6$ 千米.\n\n$\\therefore \\mathrm{AD}=\\mathrm{DF}-\\mathrm{AF}=6-2 \\sqrt{3}$ (千米).\n即河宽 $\\mathrm{AD}$ 为 $(6-2 \\sqrt{3})$ 千米;\n\n(2) 过点 $B$ 作 $B G \\perp C D$ 于 $G$, 易证四边形 $B F D G$ 是正方形,\n\n$\\therefore \\mathrm{BG}=\\mathrm{BF}=6$ 千米.\n\n在 Rt $\\triangle \\mathrm{BGC}$ 中, $\\mathrm{CG}=\\sqrt{\\mathrm{BC}^{2}-\\mathrm{BG}^{2}}=\\sqrt{10^{2}-6^{2}}=8$ (千米),\n\n$\\therefore \\mathrm{CD}=\\mathrm{CG}+\\mathrm{GD}=14$ 千米.\n\n即公路 CD 的长为 14 千米;\n\n(3) 方案一的铺设电缆费用低.\n\n由 (2) 得 $\\mathrm{DE}=\\mathrm{CD}-\\mathrm{CE}=8$ 千米.\n\n$\\therefore$ 方案一的铺设费用为: $2(\\mathrm{DE}+\\mathrm{AB})+4 \\mathrm{AD}=40$ 万元,\n\n方案二的铺设费用为: $2(\\mathrm{CE}+\\mathrm{BC}+\\mathrm{AB})=(32+8 \\sqrt{3})$ 万元.\n\n$\\because 40<32+8 \\sqrt{3}$\n\n$\\therefore$ 方案一的铺设电缆费用低.\n\n", "solution": "null", "level": "七年级", "question": "(12 分) 如图所示, $A 、 B$ 为两个村庄, $A B 、 B C 、 C D$ 为公路, $B D$ 为田地, $A D$ 为河宽,且 $C D$ 与 $A D$ 互相垂直. 现在要从 $\\mathrm{E}$ 处开始铺设通往村庄 $\\mathrm{A}$ 、村庄 B 的一条电缆, 共有如下两种铺设方案:\n\n方案一: $E \\Rightarrow D \\Rightarrow A \\Rightarrow B ;$\n\n方案二: $E \\Rightarrow C \\Rightarrow B \\Rightarrow A$.\n\n经测量得 $\\mathrm{AB}=4 \\sqrt{3}$ 千米, $\\mathrm{BC}=10$ 千米, $\\mathrm{CE}=6$ 千米, $\\angle \\mathrm{BDC}=45^{\\circ}, \\angle \\mathrm{ABD}=15$ 度. 已知: 地下电缆的修建费为 2 万元/千米, 水下电缆的修建费为 4 万元/千米.\n\n(1) 求出河宽 AD (结果保留根号);\n\n(2) 求出公路 $\\mathrm{CD}$ 的长;\n\n(3)哪种方案铺设电缆的费用低? 请说明你的理由.\n\n", "options": [], "subject": "度量几何学", "analysis": "解: (1) 过点 $\\mathrm{B}$ 作 $\\mathrm{BF} \\perp \\mathrm{AD}$, 交 $\\mathrm{DA}$ 的延长线于点 $\\mathrm{F}$.\n\n由题意得: $\\angle \\mathrm{BAF}=\\angle \\mathrm{ABD}+\\angle \\mathrm{ADB}=15^{\\circ}+45^{\\circ}=60^{\\circ}$,在 Rt $\\triangle \\mathrm{BFA}$ 中, $\\mathrm{BF}=\\mathrm{AB} \\sin 60^{\\circ}=4 \\sqrt{3} \\times \\frac{\\sqrt{3}}{2}=6$ (千米),\n\n$\\mathrm{AF}=\\mathrm{AB} \\cos 60^{\\circ}=4 \\sqrt{3} \\times \\frac{1}{2}=2 \\sqrt{3}$ (千米).\n\n$\\because \\mathrm{CD} \\perp \\mathrm{AD}, \\angle \\mathrm{BDC}=45^{\\circ}$,\n\n$\\therefore \\angle \\mathrm{BDF}=45^{\\circ}$,\n\n在 Rt $\\triangle \\mathrm{BFD}$ 中, $\\because \\angle \\mathrm{BDF}=45^{\\circ}$,\n\n$\\therefore \\mathrm{DF}=\\mathrm{BF}=6$ 千米.\n\n$\\therefore \\mathrm{AD}=\\mathrm{DF}-\\mathrm{AF}=6-2 \\sqrt{3}$ (千米).\n即河宽 $\\mathrm{AD}$ 为 $(6-2 \\sqrt{3})$ 千米;\n\n(2) 过点 $B$ 作 $B G \\perp C D$ 于 $G$, 易证四边形 $B F D G$ 是正方形,\n\n$\\therefore \\mathrm{BG}=\\mathrm{BF}=6$ 千米.\n\n在 Rt $\\triangle \\mathrm{BGC}$ 中, $\\mathrm{CG}=\\sqrt{\\mathrm{BC}^{2}-\\mathrm{BG}^{2}}=\\sqrt{10^{2}-6^{2}}=8$ (千米),\n\n$\\therefore \\mathrm{CD}=\\mathrm{CG}+\\mathrm{GD}=14$ 千米.\n\n即公路 CD 的长为 14 千米;\n\n(3) 方案一的铺设电缆费用低.\n\n由 (2) 得 $\\mathrm{DE}=\\mathrm{CD}-\\mathrm{CE}=8$ 千米.\n\n$\\therefore$ 方案一的铺设费用为: $2(\\mathrm{DE}+\\mathrm{AB})+4 \\mathrm{AD}=40$ 万元,\n\n方案二的铺设费用为: $2(\\mathrm{CE}+\\mathrm{BC}+\\mathrm{AB})=(32+8 \\sqrt{3})$ 万元.\n\n$\\because 40<32+8 \\sqrt{3}$\n\n$\\therefore$ 方案一的铺设电缆费用低.\n\n"} {"id": "1426", "image": [], "answer": "(1) $2 ; \\sqrt{7}-2$\n\n(2) $-3 ; 3-\\sqrt{7}$\n\n(3) 解: $\\because 5<3+\\sqrt{7}<6$,\n\n$\\therefore \\mathrm{m}=5, \\mathrm{n}=3+\\sqrt{7}-5=\\sqrt{7}-2$,\n\n$\\therefore|m-n|$\n\n$=|5-(\\sqrt{7}-2)|$\n\n$=|7-\\sqrt{7}|$\n\n$=7-\\sqrt{7}$.", "solution": "null", "level": "七年级", "question": "阅读下面文字, 然后回答问题.\n\n给出定义: 一个实数的整数部分是不大于这个数的最大数, 这个实数的小数部分为这个数与它的整数部分的差的绝对值. 例如: 2.4 的整数部分为 2 , 小数部分为 $2.4-2=0.4 ; \\sqrt{2}$ 的整数部分为 1 , 小数部分可用 $\\sqrt{2}-1$ 表示; 再如, -2.6 的整数部分为 -3 , 小数部分为 $|-2.6-(-3)|=$ 0.4 . 由此我们得到一个真命题: 如果 $\\sqrt{2}=x+y$, 其中 $x$ 是整数, 且 $0\n\n(1) 操作思考:\n\n勤学小组的对折方案是: 使表示 -5 的点与表示 5 的点重合.\n\n(1)对折后数轴上表示 7 的点与表示 \\$ \\qquad \\$的点重合;\n\n(2)对折后数轴上表示有理数 $m$ 的点与表示 \\$ \\qquad \\$的点重合 (用含 $m$ 的式子表示);\n\n(2)善思小组的对折方案是: 使表示- 5 的点与表示 7 的点重合.\n\n(1)对折后数轴上表示 \\$ \\qquad \\$的点与原点重合; 对折后表示- 25 的点与表示 \\$ \\qquad \\$的点重合;\n\n(2)对折后数轴上表示有理数 $m$ 的点与表示 \\$ \\qquad \\$的点重合 (用含 $m$ 的式子表示);\n\n(3) 拓展探究:\n好问小组的对折方案是: 使表示有理数 $m$ 的点与表示有理数 $n$ 的点重合 (其中 $m\n\n(1) 用含 $\\mathrm{t}$ 的代数式表示 $\\mathrm{P}$ 到点 $\\mathrm{A}$ 和点 $\\mathrm{C}$ 的距离: $\\mathrm{PA}=$ \\$ \\qquad \\$ , $\\mathrm{PC}=$ \\$ \\qquad \\$ —.\n\n(2) 当点 $P$ 运动到 $B$ 点时, 点 $Q$ 从 $A$ 点出发, 以每秒 3 个单位的速度向 $C$ 点运动, $Q$ 点到达 $C$点后, 再立即以同样的速度返回, 当点 $\\mathrm{P}$ 运动到点 $\\mathrm{C}$ 时, $\\mathrm{P} 、 \\mathrm{Q}$ 两点运动停止,\n\n(1) 当 $P 、 Q$ 两点运动停止时, 求点 $P$ 和点 $Q$ 的距离;\n\n(2)求当 $t$ 为何值时 $P 、 Q$ 两点恰好在途中相遇.", "options": [], "subject": "度量几何学", "analysis": ") $\\mathrm{PA}=\\mathrm{t}, \\mathrm{PC}=36-\\mathrm{t}$;\n故答案是: $t ; 36-t ;$\n\n21 世纪教育网 (www.21cnjy.com)"} {"id": "1306", "image": ["3099.jpg"], "answer": "(1) 解:到点 $A 、$ 、点 $B$ 的距离相等的点位于 $A 、 B$ 的中点, 即 $x=1$ 的点\n\n(2) 解: 若 $\\mathrm{P}$ 向数轴负方向运动,\n\n使 $\\mathrm{PB}=3 \\mathrm{AB}, \\mathrm{AB}=4$\n\n则 $\\mathrm{PB}=12$\n\n所以 $\\mathrm{P}$ 点对应的数是 $3-12=-9$,\n从原点到- 9 对应点的距离是 $9, \\mathrm{P}$ 移动的速度是 3 个单位 $/ \\mathrm{s}$\n\n所以到达- 9 处需要时间 $=9 \\div 3=3 s$;\n\n若 $\\mathrm{P}$ 向数轴正方向运动,\n\n使 $\\mathrm{PB}=3 \\mathrm{AB}, \\mathrm{AB}=4$\n\n则 $\\mathrm{PB}=12$\n\n所以 $\\mathrm{P}$ 点对应的数是 $3+12=15$\n\n从原点到 15 对应点的距离是 $15, \\mathrm{P}$ 移动的速度是 3 个单位 $/ \\mathrm{s}$\n\n所以到达 15 处需要时间 $=15 \\div 3=5 s$.\n\n综上, 当以数轴负向运动时, 3 秒后可使 $\\mathrm{PB}=3 \\mathrm{AB}$; 当以数轴正向运动时, 5 秒后可使 $\\mathrm{PB}=3 \\mathrm{AB}$\n\n(3) 解: 由题可得, 要找出与 $A 、 B$ 两点距离之和为 6 的点, 因为 $A B=4$, 所以必定在线段 $A B$ 两侧\n\n在线段 $\\mathrm{AB}$ 右侧的点为 $\\mathrm{x}=4$ 的点, 与 $\\mathrm{B}$ 距离为 1 , 与 $\\mathrm{A}$ 距离为 5 ;\n\n在线段 $A B$ 左侧的点为 $x=-2$ 的点, 与 $A$ 距离为 1 , 与 $B$ 距离为 5", "solution": "null", "level": "七年级", "question": "已知数轴上两点 $A 、 B$ 对应的数分别为 $-1 、 3$, 点 $\\mathrm{P}$ 为数轴上一动点, 其对应的数为 $\\mathrm{x}$.\n\n\n\n(1) 如果点 $\\mathrm{P}$ 到点 A、点 $\\mathrm{B}$ 的距离相等, 直接写出 $\\mathrm{x}$ 的值;\n\n(2) 当点 $\\mathrm{P}$ 以每秒 3 个单位长的速度从数轴的原点出发, 几秒后可使 $\\mathrm{PB}=3 \\mathrm{AB}$ ?\n\n(3) 利用数轴, 根据绝对值的几何意义, 找出满足 $|x+1|+|x-3|=6$ 的所有 $x$ 的值.", "options": [], "subject": "解析几何", "analysis": "(1) 解:到点 $A 、$ 、点 $B$ 的距离相等的点位于 $A 、 B$ 的中点, 即 $x=1$ 的点\n\n(2) 解: 若 $\\mathrm{P}$ 向数轴负方向运动,\n\n使 $\\mathrm{PB}=3 \\mathrm{AB}, \\mathrm{AB}=4$\n\n则 $\\mathrm{PB}=12$\n\n所以 $\\mathrm{P}$ 点对应的数是 $3-12=-9$,\n从原点到- 9 对应点的距离是 $9, \\mathrm{P}$ 移动的速度是 3 个单位 $/ \\mathrm{s}$\n\n所以到达- 9 处需要时间 $=9 \\div 3=3 s$;\n\n若 $\\mathrm{P}$ 向数轴正方向运动,\n\n使 $\\mathrm{PB}=3 \\mathrm{AB}, \\mathrm{AB}=4$\n\n则 $\\mathrm{PB}=12$\n\n所以 $\\mathrm{P}$ 点对应的数是 $3+12=15$\n\n从原点到 15 对应点的距离是 $15, \\mathrm{P}$ 移动的速度是 3 个单位 $/ \\mathrm{s}$\n\n所以到达 15 处需要时间 $=15 \\div 3=5 s$.\n\n综上, 当以数轴负向运动时, 3 秒后可使 $\\mathrm{PB}=3 \\mathrm{AB}$; 当以数轴正向运动时, 5 秒后可使 $\\mathrm{PB}=3 \\mathrm{AB}$\n\n(3) 解: 由题可得, 要找出与 $A 、 B$ 两点距离之和为 6 的点, 因为 $A B=4$, 所以必定在线段 $A B$ 两侧\n\n在线段 $\\mathrm{AB}$ 右侧的点为 $\\mathrm{x}=4$ 的点, 与 $\\mathrm{B}$ 距离为 1 , 与 $\\mathrm{A}$ 距离为 5 ;\n\n在线段 $A B$ 左侧的点为 $x=-2$ 的点, 与 $A$ 距离为 1 , 与 $B$ 距离为 5"} {"id": "2435", "image": ["3525.jpg"], "answer": "(1) $\\angle \\mathrm{ADB}=\\angle \\mathrm{ABC}=90^{\\circ} \\quad \\angle \\mathrm{DAB}=\\angle \\mathrm{COB} \\quad$ (2) $\\mathrm{AD}=\\frac{\\sqrt{6}}{3}$", "solution": "null", "level": "七年级", "question": "(本题 6 分) 如图, $A B$ 是 $\\odot O$ 的直径, $B C$ 是 $\\odot O$ 的切线, $D$ 是 $\\odot O$ 上一点, 且 $A D / /$ $O C$\n\n(1) 求证: $\\triangle A D B \\backsim \\triangle O B C \\quad$ (2) 若 $A B=2, B C=\\sqrt{5}$, 求 $A D$ 的长 (结果保留根号)\n\n", "options": [], "subject": "解析几何", "analysis": "(1) $\\angle \\mathrm{ADB}=\\angle \\mathrm{ABC}=90^{\\circ} \\quad \\angle \\mathrm{DAB}=\\angle \\mathrm{COB} \\quad$ (2) $\\mathrm{AD}=\\frac{\\sqrt{6}}{3}$"} {"id": "2440", "image": [], "answer": "(1) $A(4,0)\\left[(6,3)=(2) \\frac{35}{6} s\\right.$ 或 $\\frac{85}{6} s$ (3) $\\frac{20}{3} s$", "solution": "null", "level": "七年级", "question": "(本题 10 分) 如图, 已知直线 $l$ 的解析式为 $\\mathrm{y}=\\frac{3}{4} \\mathrm{x}-3$, 且与 $\\mathrm{x}$ 轴、 $\\mathrm{y}$ 轴分别交于点 $A, B$.\n\n(1)求 $A, B$ 两点的坐标;\n\n(2)一个圆心在坐标原点、半径为 1 的圆, 以 $\\frac{2}{5}$ 个单位/秒的速度向 $x$ 轴正方向运动,问在什么时刻圆与直线 $l$ 相切?\n\n(3)在题 (2) 中, 若在圆开始运动的同时, 一动点 $P$ 从 $B$ 点出发, 沿 $B A$ 方向以 $\\frac{1}{2}$ 个单位/秒的速度运动, 问: 在整个运动过程中, 点 $P$ 在动圆的圆面上 (包括圆上和圆内部) 一共运动了多长时间?", "options": [], "subject": "解析几何", "analysis": "(1) $A(4,0)\\left[(6,3)=(2) \\frac{35}{6} s\\right.$ 或 $\\frac{85}{6} s$ (3) $\\frac{20}{3} s$"} {"id": "1375", "image": ["3113.jpg"], "answer": "(1) 解: 大正方形的边长是 $\\sqrt{2 \\times(15 \\sqrt{2})^{2}}=30(\\mathrm{~cm})$\n\n(2) 解: 设长方形纸片的长为 $4 \\mathrm{xcm}$, 宽为 $3 \\mathrm{xcm}$, 则 $4 \\mathrm{x} \\cdot 3 \\mathrm{x}=720$, 解得: $\\mathrm{x}=\\sqrt{60}, 4 \\mathrm{x}=$ $\\sqrt{4 \\times 4 \\times 60}=\\sqrt{960}>30$, 所以沿此大正方形边的方向剪出一个长方形, 不能使剪出的长方形纸片的长宽之比为 $4: 3$, 且面积为 $720 \\mathrm{~cm}^{2}$", "solution": "null", "level": "七年级", "question": "如图, 用两个边长为 $15 \\sqrt{2} \\mathrm{~cm}$ 的小正方形拼成一个大的正方形.\n\n\n\n(1) 求大正方形的边长?\n\n(2) 若沿此大正方形边的方向剪出一个长方形, 能否使剪出的长方形纸片的长宽之比为 4: 3 且面积\n为 $720 \\mathrm{~cm}^{2}$.若能, 试求出剪出的长方形纸片的长与宽; 若不能, 试说明理由?", "options": [], "subject": "解析几何", "analysis": "(1) 解: 大正方形的边长是 $\\sqrt{2 \\times(15 \\sqrt{2})^{2}}=30(\\mathrm{~cm})$\n\n(2) 解: 设长方形纸片的长为 $4 \\mathrm{xcm}$, 宽为 $3 \\mathrm{xcm}$, 则 $4 \\mathrm{x} \\cdot 3 \\mathrm{x}=720$, 解得: $\\mathrm{x}=\\sqrt{60}, 4 \\mathrm{x}=$ $\\sqrt{4 \\times 4 \\times 60}=\\sqrt{960}>30$, 所以沿此大正方形边的方向剪出一个长方形, 不能使剪出的长方形纸片的长宽之比为 $4: 3$, 且面积为 $720 \\mathrm{~cm}^{2}$"} {"id": "1593", "image": ["3181.jpg"], "answer": "(1) 解:若设图中最大正方形 $\\mathrm{B}$ 的边长是 $\\mathrm{x}$ 米, 最小的正方形的边长是 1 米.\n\n$\\therefore \\mathrm{F}$ 的边长为: $(\\mathrm{x}-1)$ 米,\n\n$\\therefore \\mathrm{C}$ 的边长为: $\\frac{x+1}{2}$ 米,\n\n$\\therefore \\mathrm{E}$ 的边长为: $\\mathrm{x}-1-1=(\\mathrm{x}-2)$ 米;\n\n(2) 解: $\\because \\mathrm{MQ}=\\mathrm{PN}$,\n\n$\\therefore \\mathrm{x}-1+\\mathrm{x}-2=\\mathrm{x}+\\frac{x+1}{2}$,\n\n解得: $x=7$,\n\n$\\therefore \\mathrm{x}$ 的值为 7 ;\n\n(3) 解: 设余下的工程由乙队单独施工, 还要 $\\mathrm{x}$ 天完成.\n\n$\\therefore\\left(\\frac{1}{10}+\\frac{1}{15}\\right) \\times 2+\\frac{1}{15} \\mathrm{x}=1$,\n解得: $x=10$.\n\n答: 余下的工程由乙队单独施工, 还要 10 天完成.", "solution": "null", "level": "七年级", "question": "如图是某市民健身广场的平面示意图, 它是由 6 个正方形拼成的长方形, 已知中间最小的正方形 $A$ 的边长是 1 米;\n\n\n\n(1)若设图中最大正方形 $B$ 的边长是 $x$ 米, 请用含 $x$ 的代数式分别表示出正方形 $F 、 E 、 C$的边长\n\n(2) 观察图形的特点可知, 长方形相对的两边是相等的(即 $M N=P Q, \\quad M Q=P N$ ) 请根据以上结论, 求出 $x$ 的值\n\n(3)现沿着长方形广场的四条边铺设下水管道,由甲、乙工程队单独铺设分别需要 10 天、 15 天完成, 如果两队从同一位置开始, 沿相反的方向同时施工 2 天后, 因甲队另有任务, 余下的工程由乙队单独施工,还要多少天完成?", "options": [], "subject": "解析几何", "analysis": "(1) 若设图中最大正方形 $\\mathrm{B}$ 的边长是 $\\mathrm{x}$ 米, 最小的正方形的边长是 1 米, 从图中可看出 $\\mathrm{F}$的边长为 ( $\\mathrm{x}-1$ ) 米, $\\mathrm{C}$ 的边长为 $\\frac{x+1}{2}, \\mathrm{E}$ 的边长为 ( $\\mathrm{x}-1-1$ ), 即可得到答案; (2) 根据长方形相对的两边是相等的 (如图中的 $\\mathrm{MN}$ 和 PQ). 请根据这个等量关系, 求出 $\\mathrm{x}$ 的值; (3) 根据工作效率 $\\times$ 工作时间 $=$ 工作量这个等量关系且完成工作, 工作量就为 1, 可列方程求解."} {"id": "1848", "image": ["3292.jpg"], "answer": "解:(1) $\\pm 2$ 是 4 的平方根, 3 是 27 的立方根,\n\n$\\therefore a=4, b=3$\n\n(2) $\\because A B / / O N$,\n\n$\\therefore \\angle B A O+\\angle A O N=180^{\\circ}$,\n\n$\\because$ 点 $Q$ 是 $\\angle B A N$ 的平分线与 $\\angle P O N$ 的平分线的交点,\n$\\therefore \\angle B A Q=\\frac{1}{2} \\angle B A N, \\quad \\angle Q O N=\\frac{1}{2} \\angle P O N$,\n\n$\\therefore \\angle B A Q+\\angle Q O N=\\frac{1}{2}(\\angle B A N+\\angle P O N)=40^{\\circ}$,\n\n$\\therefore \\angle Q A O+\\angle Q O A=140^{\\circ}$,\n\n$\\therefore \\angle A Q O=180^{\\circ}-140^{\\circ}=40^{\\circ}$;\n\n(3)设线段 $B C$ 运动时间为 $t$ ,\n\n$\\because A B / / M N$,\n\n$\\therefore$ 当 $A C^{\\prime}=O N^{\\prime}$ 时, 四边形 $A O N^{\\prime} C^{\\prime}$ 是平行四边形,\n\n$\\therefore N^{\\prime} C^{\\prime} / / y$ 轴,\n\n当点 $C$ 在点 $B$ 的右边时, $A C^{\\prime}=2+1+t, O N^{\\prime}=6-2 t$,\n\n由题意得, $2+1+t=6-2 t$,\n\n解得, $t=1$,\n\n点 $B^{\\prime}$ 的坐标为 $(3,0) 、 M^{\\prime}$ 的坐标为 $(-1,0)$,\n\n当点 $C$ 在点 $B$ 的左边时, $A C^{\\prime}=t-1, O N^{\\prime}=2 t-6$,\n\n由题意得, $t-1=2 t-6$,\n\n解得, $t=5$,\n\n点 $B^{\\prime}$ 的坐标为 $(-3,0) 、 M^{\\prime}$ 的坐标为 $(-9,0)$,\n\n则点 $B^{\\prime} 、 M^{\\prime}$ 的坐标为 $(-3,4) 、(-9,0)$ 或 $(3,4) 、(-1,0) .$.", "solution": "null", "level": "七年级", "question": "(本小题8.0分)\n\n在平面直角坐标系中有四个点 $A(0, a) 、 B(b-1, a) 、 M(b-2,0) 、 N(a+b-1,0)$ 且 $\\pm 2$ 是 $a$ 的平方根, $b$ 是 27 的立方根.\n\n(1)求 $a, b$ 的值;\n\n(2) 设点 $P$ 是线段 $A N$ 上任意一点, 点 $Q$ 是 $\\angle B A N$ 的平分线与 $\\angle P O N$ 的平分线的交点, 若 $\\angle B A N+$ $\\angle P O N=80^{\\circ}$, 求 $\\angle A Q O$ 的度数;\n\n(3) 已知点 $C$ 在直线 $A B$ 上, 且 $B C=1$.线段 $B C$ 以每秒 1 个单位长度的速度在直线 $A B$ 上沿射线 $B C$方向运动, 同时线段 $M N$ 以每秒 2 个单位长度的速度沿 $x$ 轴向左运动. 在运动过程中, 点 $B^{\\prime} 、 C^{\\prime} 、$ $M^{\\prime} 、 N^{\\prime}$ 分别是点 $B 、 C 、 M 、 N$ 的对应点, 连接 $N^{\\prime} C^{\\prime}$, 当 $N^{\\prime} C^{\\prime}$ 平行于 $y$ 轴时, 求点 $B^{\\prime} 、 M^{\\prime}$ 的坐标.\n\n\n备用图", "options": [], "subject": "解析几何", "analysis": "(1)根据平方根的定义、立方根的定义分别求出 $a 、 b$;\n\n(2)根据平行线的性质得到 $\\angle B A O+\\angle A O N=180^{\\circ}$, 根据角平分线的定义、三角形内角和定理计算即可;\n\n(3)根据平行四边形的判定和性质得到当 $A C^{\\prime}=O N^{\\prime}$ 时, 四边形 $A O N^{\\prime} C^{\\prime}$ 是平行四边形, $N^{\\prime} C^{\\prime} / / y$ 轴,分点 $C$ 在点 $B$ 的右边、点 $C$ 在点 $B$ 的左边两种情况列式计算即可.\n\n本题考查的是平方根、立方根的定义、角平分线的定义、三角形内角和定理, 掌握相关的判定定理和性质定理、定义是解题的关键."} {"id": "1331", "image": [], "answer": "(1) $\\frac{1}{3} ; \\frac{1}{25}$\n\n(2)解:因为 $x>0$,\n\n所以 $x<2 x$,\n\n$\\because 2^{x} \\div 2^{2 x}=\\frac{1}{2^{x}}$\n\n$\\because \\frac{1}{8}=\\frac{1}{2^{3}}$,\n\n所以 $x=3$,\n\n(3) $5 、 3 、 1$", "solution": "null", "level": "七年级", "question": "本学期我们学习了 “有理数乘方” 运算, 知道乘方的结果叫做“幂”, 下面介绍一种有关“幂”的新运算.\n\n定义: $a^{m}$ 与 $a^{n}(a \\neq 0, m, n$ 都是正整数 $)$ 叫做同底数幂, 同底数幂除法记作 $a^{m} \\div$ $a^{n}$.运算法则如下: $a^{m} \\div a^{n}=\\left\\{\\begin{array}{c}\\text { 当 } m>n \\text { 时, } a^{m} \\div a^{n}=a^{m-n} \\\\ \\text { 当 } m=n \\text { 时, } a^{m} \\div a^{n}=1 \\\\ \\text { 当 } m0$, 且 $2^{x} \\div 2^{2 x}=\\frac{1}{8}$, 求出 $x$ 的值;\n\n(3) 如果 $(x-2)^{2 x+2} \\div(x-2)^{12}=1$, 则 $x=$ \\$ \\qquad \\$ .", "options": [], "subject": "代数", "analysis": "(1) $\\left(\\frac{1}{3}\\right)^{3} \\div\\left(\\frac{1}{3}\\right)^{2}=\\frac{1}{3}, \\quad 5^{2} \\div 5^{4}=\\frac{1}{5^{2}}=\\frac{1}{25}$,\n\n故答案为: $\\frac{1}{3}, \\frac{1}{25}$;\n\n(3) 由题意知, (1) $2 x+2-12=0$,\n\n解得: $x=5$;\n\n(2) $x-2=1$,\n\n解得: $x=3$;\n\n(3) $x-2=-1$ 且 $2 x+2$ 为整数,\n\n解得: $x=1$ ;\n\n综上, $\\mathrm{x}=5, \\mathrm{x}=3, \\mathrm{x}=1$.\n\n故答案为: 5 或 3 或 1 ."} {"id": "2418", "image": ["3514.jpg", "3514.jpg"], "answer": "解:(1)把 $a=-2$ 代入不等式得\n\n$-2 x+3>0$\n\n解得 $x<\\frac{3}{2}$.\n\n\n\n(2) $\\because a x+3>0(a \\neq 0)$ 的解集为 $x<-\\frac{3}{a}$,\n\n若要使不等式没有正整数解, 则 $-\\frac{3}{a} \\leq 1$, 即 $a \\leq-3$,\n\n$\\therefore$ 在这 10 个数中,能使不等式没有正整数解的 a 的值有 $-10,-9,-8,-7,-6$, $-5,-4,-3$ 共 8 个\n\n$\\therefore$ 该不等式没有正整数解的概率为 $\\frac{8}{10}=\\frac{4}{5}$.", "solution": "null", "level": "七年级", "question": "(8 分) 已知关于 $x$ 的不等式 $a x+3>0$ (其中 $a \\neq 0$ ).\n\n(1)当 $a=-2$ 时, 求此不等式的解, 并在数轴上表示此不等式的解集;\n\n(2)小明准备了十张形状、大小完全相同的不透明卡片, 上面分别写有整数: $-10,-9$, $-8,-7,-6,-5,-4,-3,-2,-1$, 将这 10 张卡片写有整数的一面向下放在桌面上. 从中任意抽取一张, 以卡片上的数作为不等式中的系数 a, 求使该不等式没有正整数解的概率.", "options": [], "subject": "代数", "analysis": "解:(1)把 $a=-2$ 代入不等式得\n\n$-2 x+3>0$\n\n解得 $x<\\frac{3}{2}$.\n\n\n\n(2) $\\because a x+3>0(a \\neq 0)$ 的解集为 $x<-\\frac{3}{a}$,\n\n若要使不等式没有正整数解, 则 $-\\frac{3}{a} \\leq 1$, 即 $a \\leq-3$,\n\n$\\therefore$ 在这 10 个数中,能使不等式没有正整数解的 a 的值有 $-10,-9,-8,-7,-6$, $-5,-4,-3$ 共 8 个\n\n$\\therefore$ 该不等式没有正整数解的概率为 $\\frac{8}{10}=\\frac{4}{5}$."} {"id": "1420", "image": [], "answer": "解: $\\sqrt{4}=2, \\sqrt[3]{27}=3$,\n\n正整数 $\\{(3) \\sqrt{4},(9) \\sqrt[3]{27} \\ldots \\ldots\\} ;$\n\n负分数 $\\left\\{\\right.$ (2) $-\\frac{17}{15}$, (8) $\\left.-3.1 \\ldots ..\\right\\}$;\n\n无理数 $\\{1) \\sqrt[3]{2}$, (5) $-\\sqrt{0.9}$, ( 7 ) $\\left.-\\frac{\\pi}{4} \\cdots \\cdots\\right\\}$.", "solution": "null", "level": "七年级", "question": "用序号将下列各数填入相应的大括号内.\n$(1) \\sqrt[3]{2}$,\n(2) $-\\frac{17}{15}$,\n(3) $\\sqrt{4}$,\n(4) 0 ,\n(5) $-\\sqrt{0.9}$,\n(6) 3.14, (7) $-\\frac{\\pi}{4}$,\n(8) -3.1, (9) $\\sqrt[3]{27}$\n\n正整数 \\{\n\n负分数 \\{\n\n\\$.\n\n无理数\\{", "options": [], "subject": "代数", "analysis": "解: $\\sqrt{4}=2, \\sqrt[3]{27}=3$,\n\n正整数 $\\{(3) \\sqrt{4},(9) \\sqrt[3]{27} \\ldots \\ldots\\} ;$\n\n负分数 $\\left\\{\\right.$ (2) $-\\frac{17}{15}$, (8) $\\left.-3.1 \\ldots ..\\right\\}$;\n\n无理数 $\\{1) \\sqrt[3]{2}$, (5) $-\\sqrt{0.9}$, ( 7 ) $\\left.-\\frac{\\pi}{4} \\cdots \\cdots\\right\\}$."} {"id": "1423", "image": [], "answer": "解: $\\because 4<5<9,36<37<49$,\n\n$\\therefore 2<\\sqrt{5}<3,6<\\sqrt{37}<7$,\n\n$\\therefore \\mathrm{a}=\\sqrt{5}-2, \\mathrm{~b}=6$,\n\n$\\therefore \\mathrm{a}+\\mathrm{b}-\\sqrt{5}=\\sqrt{5}-2+6-\\sqrt{5}=4$.", "solution": "null", "level": "七年级", "question": "数学老师在课堂上提出一个问题: “通过探究知道: $\\sqrt{2}=1.414 \\ldots$, 它是个无限不循环小数, 也叫无理数, 它的整数部分是 1 , 那么有谁能说出它的小数部分是多少? ”小明举手回答: “它的小数部分我们无法全部写出来, 但可以用 $\\sqrt{2}-1$ 来表示它的小数部分.”张老师肯定了他的说法.现请你根据小明的说法解答: 若 $\\sqrt{5}$ 的小数部分是 $\\mathrm{a}, \\sqrt{37}$ 的整数部分是 $\\mathrm{b}$, 求 $\\mathrm{a}+\\mathrm{b}-\\sqrt{5}$ 的值.", "options": [], "subject": "代数", "analysis": "解: $\\because 4<5<9,36<37<49$,\n\n$\\therefore 2<\\sqrt{5}<3,6<\\sqrt{37}<7$,\n\n$\\therefore \\mathrm{a}=\\sqrt{5}-2, \\mathrm{~b}=6$,\n\n$\\therefore \\mathrm{a}+\\mathrm{b}-\\sqrt{5}=\\sqrt{5}-2+6-\\sqrt{5}=4$."} {"id": "1424", "image": [], "answer": "(1) 解: $\\therefore \\sqrt{16}<\\sqrt{17}<\\sqrt{25}$ ,\n\n$\\therefore 4<\\sqrt{17}<5$,\n\n$\\therefore 1<\\sqrt{17}-3<2$,\n\n$\\therefore \\mathrm{a}=1, \\mathrm{~b}=\\sqrt{17}-4$\n\n(2) 解: $(-a)^{3}+(b+4)^{2}=(-1)^{3}+(\\sqrt{17}-4+4)^{2}=-1+17=16$,\n\n$\\therefore(-a)^{3}+(b+4)^{2}$ 的平方根是 $\\pm \\sqrt{16}= \\pm 4$.", "solution": "null", "level": "七年级", "question": "阅读理解.\n\n$\\because \\sqrt{4}<\\sqrt{5}<\\sqrt{9}$ 即 $2<\\sqrt{5}<3$\n\n$$\n\\therefore 1<\\sqrt{5}-1<2\n$$\n\n$\\therefore \\sqrt{5}-1$ 的整数部分为 1,\n\n$\\therefore \\sqrt{5}-1$ 的小数部分为 $\\sqrt{5}-2$\n\n解决问题: 已知 $a$ 是 $\\sqrt{17}-3$ 的整数部分, $b$ 是 $\\sqrt{17}-3$ 的小数部分.\n\n(1) 求 $\\mathrm{a}, \\mathrm{b}$ 的值;\n\n(2) 求 $(-a)^{3}+(b+4)^{2}$ 的平方根, 提示: $(\\sqrt{17})^{2}=17$.", "options": [], "subject": "代数", "analysis": "(1) 解: $\\therefore \\sqrt{16}<\\sqrt{17}<\\sqrt{25}$ ,\n\n$\\therefore 4<\\sqrt{17}<5$,\n\n$\\therefore 1<\\sqrt{17}-3<2$,\n\n$\\therefore \\mathrm{a}=1, \\mathrm{~b}=\\sqrt{17}-4$\n\n(2) 解: $(-a)^{3}+(b+4)^{2}=(-1)^{3}+(\\sqrt{17}-4+4)^{2}=-1+17=16$,\n\n$\\therefore(-a)^{3}+(b+4)^{2}$ 的平方根是 $\\pm \\sqrt{16}= \\pm 4$."} {"id": "1427", "image": [], "answer": "1)<;<$\n\n(2) $\\sqrt{2}-1 ; \\sqrt{3}-\\sqrt{2}$\n\n(3) 解: 原式 $=\\sqrt{2}-1+\\sqrt{3}-\\sqrt{2}+\\sqrt{4}-\\sqrt{3}+\\ldots+\\sqrt{2021}-\\sqrt{2020}=\\sqrt{2020}-1$", "solution": "null", "level": "七年级", "question": "(1) 用“<”“\"或“=”填空: 1 \\$ \\qquad \\$ $\\sqrt{2}, \\sqrt{2}$ $\\sqrt{3} ;$\n\n(2) 由以上可知:\n\n(1) $|1-\\sqrt{2}|=$ \\$ \\qquad \\$ (2) $|\\sqrt{2}-\\sqrt{3}|=$ \\$ \\qquad \\$\n(3) 计算: $|1-\\sqrt{2}+| \\sqrt{2}-\\sqrt{3}|+| \\sqrt{3}-\\sqrt{4}|+\\ldots+| \\sqrt{2020}-\\sqrt{2021} \\mid$ (结果保留根号).", "options": [], "subject": "代数", "analysis": "1)<;<$\n\n(2) $\\sqrt{2}-1 ; \\sqrt{3}-\\sqrt{2}$\n\n(3) 解: 原式 $=\\sqrt{2}-1+\\sqrt{3}-\\sqrt{2}+\\sqrt{4}-\\sqrt{3}+\\ldots+\\sqrt{2021}-\\sqrt{2020}=\\sqrt{2020}-1$"} {"id": "1445", "image": [], "answer": "解: $|1-n|=2, n=-1$ 或 3 ,\n\n$\\because \\mathrm{m}^{2}=25, \\quad \\therefore \\mathrm{m}= \\pm 5$,\n\n$\\because \\mathrm{m}<\\mathrm{n}, \\quad \\therefore \\mathrm{m}=-5$,\n\n$\\therefore \\mathrm{m}-\\mathrm{n}=-4$ 或 -8 .", "solution": "null", "level": "七年级", "question": "已知 $m^{2}=25,|1-n|=2$, 且 $m\n\n图1\n\n(1) $\\mathrm{a}=$ \\$ \\qquad \\$ $\\mathrm{b}=$ \\$ \\qquad \\$ ;\n\n(2) 计算: $6 x^{2}-3 y+1$ 的值;\n\n(3)图 2 所示是“积幻方”,其每行、每列、每条对角线上的三个数字之积均相等,则 $\\mathrm{m}^{\\mathrm{n}}=$ \\$ \\qquad \\$", "options": [], "subject": "代数", "analysis": "(1)根据“和幻方”每行、每列、每条对角线上的三个数字之和均相等,得:\n\n$\\left\\{\\begin{array}{l}8+b+7=8+3+(-2) \\\\ a+3+b=8+3+(-2)\\end{array}\\right.$\n\n$a+3+b=8+3+(-2)$\n\n$\\therefore\\left\\{\\begin{array}{l}a=12 \\\\ b=-6\\end{array}\\right.$,\n\n故答案为: $12 ;-6$;\n\n(3)根据“积幻方”每行、每列、每条对角线上的三个数字之积均相等,得:\n\n$\\left\\{\\begin{array}{l}2 \\times 2 n=(-3) \\times\\left(-\\frac{4}{3}\\right) \\times 2 \\\\ -3 m n=(-3) \\times\\left(-\\frac{4}{3}\\right) \\times 2\\end{array}\\right.$,\n\n$\\therefore\\left\\{\\begin{array}{c}m=-\\frac{4}{3}, \\\\ n=2\\end{array}\\right.$\n\n$\\therefore \\mathrm{m}^{\\mathrm{n}}=\\left(-\\frac{4}{3}\\right)^{2}=\\frac{16}{9}$,\n\n故答案为: $\\frac{16}{9}$."} {"id": "1491", "image": [], "answer": "解: 原式 $=\\frac{1}{2} x-2 x+\\frac{2}{3} y^{2}-\\frac{3}{2} x+\\frac{1}{3} y^{2}$\n\n$=-3 x+y^{2}$,\n\n由 $(x-2)^{2}+|y-3|=0$ 得,\n\n$x=2, y=3$,\n\n当 $y=3, x=2$ 时,\n\n原式 $=-3 x+y^{2}$\n\n$$\n=-3 \\times 2+3^{2}\n$$\n\n$=3$.", "solution": "null", "level": "七年级", "question": "先化简, 再求值: $\\frac{1}{2} x-2\\left(x-\\frac{1}{3} y^{2}\\right)+\\left(-\\frac{3}{2} x+\\frac{1}{3} y^{2}\\right)$, 其中 $x 、 y$ 满足 $(x-2)^{2}+|y-3|=0$.", "options": [], "subject": "代数", "analysis": "解: 原式 $=\\frac{1}{2} x-2 x+\\frac{2}{3} y^{2}-\\frac{3}{2} x+\\frac{1}{3} y^{2}$\n\n$=-3 x+y^{2}$,\n\n由 $(x-2)^{2}+|y-3|=0$ 得,\n\n$x=2, y=3$,\n\n当 $y=3, x=2$ 时,\n\n原式 $=-3 x+y^{2}$\n\n$$\n=-3 \\times 2+3^{2}\n$$\n\n$=3$."} {"id": "1492", "image": [], "answer": "(1) 解: $[3 @(-2)]-[(-2) \\oplus(-1)]$\n\n$$\n\\begin{gathered}\n=\\frac{3-2}{2}-\\frac{-2+1}{2} \\\\\n=\\frac{1}{2}+\\frac{1}{2} \\\\\n=1\n\\end{gathered}\n$$\n\n(2) 解: $A=[3 b @(-a)]+[a \\oplus(2-3 b)]$\n\n$$\n\\begin{gathered}\n=\\frac{3 b-a}{2}+\\frac{a-(2-3 b)}{2} \\\\\n=3 b-1 \\\\\nB=[a @(-3 b)]+[(-a) \\oplus(-2-9 b)] \\\\\n=\\frac{a-3 b}{2}+\\frac{-a-(-2-9 b)}{2} \\\\\n=3 b+1\n\\end{gathered}\n$$\n\n则 $A+B=3 b-1+3 b+1=6 b$", "solution": "null", "level": "七年级", "question": "定义新运算“ @ ”与“ $\\oplus \": \\quad a @ b=\\frac{a+b}{2}, a \\oplus b=\\frac{a-b}{2}$\n\n(1) 计算的值 $[3 @(-2)]-[(-2) \\oplus(-1)]$;\n\n(2) 若 $A=[3 b @(-a)]+[a \\oplus(2-3 b)], \\quad B=[a @(-3 b)]+[(-a) \\oplus(-2-9 b)]$, 求 $A+$ $B$ 的值.", "options": [], "subject": "代数", "analysis": "(1) 解: $[3 @(-2)]-[(-2) \\oplus(-1)]$\n\n$$\n\\begin{gathered}\n=\\frac{3-2}{2}-\\frac{-2+1}{2} \\\\\n=\\frac{1}{2}+\\frac{1}{2} \\\\\n=1\n\\end{gathered}\n$$\n\n(2) 解: $A=[3 b @(-a)]+[a \\oplus(2-3 b)]$\n\n$$\n\\begin{gathered}\n=\\frac{3 b-a}{2}+\\frac{a-(2-3 b)}{2} \\\\\n=3 b-1 \\\\\nB=[a @(-3 b)]+[(-a) \\oplus(-2-9 b)] \\\\\n=\\frac{a-3 b}{2}+\\frac{-a-(-2-9 b)}{2} \\\\\n=3 b+1\n\\end{gathered}\n$$\n\n则 $A+B=3 b-1+3 b+1=6 b$"} {"id": "1493", "image": [], "answer": "(1) 解: $A=\\frac{1}{2} x-2 x+\\frac{4}{3} y-\\frac{1}{2} x+\\frac{2}{3} y$\n\n$=-2 x+2 y$,\n\n当 $x=-2, y=1$ 时,\n\n原式 $=-2 \\times(-2)+2 \\times 1$\n\n$$\n=4+2\n$$\n\n$=6$,\n\n即 $A$ 的值为 6 ;\n\n(2) 解: 由题意可得 $-2 x+2 y=6$,\n\n则当 $\\left\\{\\begin{array}{l}x=1 \\\\ y=4\\end{array}\\right.$ 时, $-2 x+2 y=6$ 也成立,\n\n$\\therefore$ :若使求得的 $A$ 的值与 (1) 中的结果相同, 则给出的 $x, y$ 的值还能够是 $x=1, y=4$ (答案不唯一).", "solution": "null", "level": "七年级", "question": "设 $\\mathrm{A}=\\frac{1}{2} x-2\\left(x-\\frac{2}{3} y\\right)+\\left(-\\frac{1}{2} x+\\frac{2}{3} y\\right)$.\n\n(1) 当 $\\mathrm{x}=-2, \\mathrm{y}=1$ 时, 求 $\\mathrm{A}$ 的值;\n\n(2) 若使求得的 $\\mathrm{A}$ 的值与 (1) 中的结果相同, 则给出的 $x, y$ 的值还能够是什么?", "options": [], "subject": "代数", "analysis": "(1) 解: $A=\\frac{1}{2} x-2 x+\\frac{4}{3} y-\\frac{1}{2} x+\\frac{2}{3} y$\n\n$=-2 x+2 y$,\n\n当 $x=-2, y=1$ 时,\n\n原式 $=-2 \\times(-2)+2 \\times 1$\n\n$$\n=4+2\n$$\n\n$=6$,\n\n即 $A$ 的值为 6 ;\n\n(2) 解: 由题意可得 $-2 x+2 y=6$,\n\n则当 $\\left\\{\\begin{array}{l}x=1 \\\\ y=4\\end{array}\\right.$ 时, $-2 x+2 y=6$ 也成立,\n\n$\\therefore$ :若使求得的 $A$ 的值与 (1) 中的结果相同, 则给出的 $x, y$ 的值还能够是 $x=1, y=4$ (答案不唯一)."} {"id": "1496", "image": [], "answer": "(1) 解: $\\because \\mathrm{A}-\\mathrm{B}=\\mathrm{x}^{2}+\\mathrm{x}-4$,\n\n$\\therefore A=B+x^{2}+x-4=3 x^{2}-2 x+1+x^{2}+x-4$,\n\n解得: $A=4 x^{2}-x-3$ ,\n\n$\\therefore \\mathrm{A}$ 所表示的代数式为 $4 x^{2}-x-3$;\n\n(2) 解: $\\because \\mathrm{A}=4 \\mathrm{x}^{2}-\\mathrm{x}-3, \\mathrm{~B}=3 \\mathrm{x}^{2}-2 \\mathrm{x}+1$\n\n$\\therefore 2 A-B=2\\left(4 \\mathrm{x}^{2}-\\mathrm{x}-3\\right)-\\left(3 \\mathrm{x}^{2}-2 \\mathrm{x}+1\\right)$\n\n$=5 \\mathrm{x}^{2}-7$\n\n当 $x=-1$ 时, 原式 $=5 \\times(-1)^{2}-7=5 \\times 1-7=5-7=-2$.", "solution": "null", "level": "七年级", "question": "小刚同学由于粗心, 把“ $2 A-B$ ”看成了“ $A-B$ ”, 算出 $A-B$ 的结果为 $x^{2}+x-4$, 其中 $B=3 x^{2}-$ $2 \\mathrm{x}+1$.\n\n(1) 求 A 所表示的代数式;\n\n(2) 若 $x=-1$, 求代数式 $2 \\mathrm{~A}-\\mathrm{B}$ 的值.", "options": [], "subject": "代数", "analysis": "(1) 解: $\\because \\mathrm{A}-\\mathrm{B}=\\mathrm{x}^{2}+\\mathrm{x}-4$,\n\n$\\therefore A=B+x^{2}+x-4=3 x^{2}-2 x+1+x^{2}+x-4$,\n\n解得: $A=4 x^{2}-x-3$ ,\n\n$\\therefore \\mathrm{A}$ 所表示的代数式为 $4 x^{2}-x-3$;\n\n(2) 解: $\\because \\mathrm{A}=4 \\mathrm{x}^{2}-\\mathrm{x}-3, \\mathrm{~B}=3 \\mathrm{x}^{2}-2 \\mathrm{x}+1$\n\n$\\therefore 2 A-B=2\\left(4 \\mathrm{x}^{2}-\\mathrm{x}-3\\right)-\\left(3 \\mathrm{x}^{2}-2 \\mathrm{x}+1\\right)$\n\n$=5 \\mathrm{x}^{2}-7$\n\n当 $x=-1$ 时, 原式 $=5 \\times(-1)^{2}-7=5 \\times 1-7=5-7=-2$."} {"id": "1499", "image": ["3153.jpg", "3154.jpg"], "answer": ") 解: $(2 x-3) m+2 m^{2}-3 x=2 m x-3 m+2 m^{2}-3 x$ $=(2 m-3) x-3 m+2 m^{2}$,\n\n$\\because$ 关于 $x$ 的多项式 $(2 x-3) m+2 m^{2}-3 x$ 的值与 $x$ 的取值无关,\n\n$\\therefore 2 m-3=0$,\n\n解得 $m=\\frac{3}{2}$.\n\n(2) 解: $\\because ~ A=2 x^{2}+3 x y-2 x-1, B=-x^{2}+x y-1$,\n\n$$\n\\begin{gathered}\n\\therefore 3 A+6 B=3\\left(2 x^{2}+3 x y-2 x-1\\right)+6\\left(-x^{2}+x y-1\\right) \\\\\n=6 x^{2}+9 x y-6 x-3-6 x^{2}+6 x y-6 \\\\\n=15 x y-6 x-9\n\\end{gathered}\n$$\n\n$=(15 y-6) x-9$,\n\n$\\because 3 A+6 B$ 的值与 $x$ 无关,\n\n$\\therefore 15 y-6=0$,\n\n解得 $y=\\frac{2}{5}$.\n\n(3) 解: 设 $A B=x$,\n\n由图可知, $S_{1}=a(x-3 b)=a x-3 a b, S_{2}=2 b(x-2 a)=2 b x-4 a b$,\n\n则 $S_{1}-S_{2}=a x-3 a b-(2 b x-4 a b)$\n\n$$\n=a x-3 a b-2 b x+4 a b\n$$\n\n$=(a-2 b) x+a b$,\n$\\because$ 当 $A B$ 的长变化时, $S_{1}-S_{2}$ 的值始终保持不变,\n\n$\\therefore S_{1}-S_{2}$ 的值与 $x$ 的值无关,\n\n$\\therefore a-2 b=0$,\n\n$\\therefore a=2 b$.", "solution": "null", "level": "七年级", "question": "七年级学习代数式求值时, 遇到这样一类题“代数式 $a x-y+6+3 x-5 y-1$ 的值与 $x$ 的取值无\n关, 求 $a$ 的值”, 通常的解题方法是: 把 $\\mathrm{x} 、 \\mathrm{y}$ 看作字母, $\\mathrm{a}$ 看作系数合并同类项, 因为代数式的值与 $\\mathrm{x}$ 的取值无关, 所以含 $x$ 项的系数为 0 ,\n\n即原式 $=(a+3) x-6 y+5$, 所以 $a+3=0$, 则 $a=-3$.\n\n\n\n图1\n\n\n\n(1) 若关于 $x$ 的多项式 $(2 x-3) m+2 m^{2}-3 x$ 的值与 $x$ 的取值无关, 求 $m$ 值;\n\n(2) 已知 $\\mathrm{A}=2 x^{2}+3 x y-2 x-1, \\mathrm{~B}=-x^{2}+x y-1$; 且 $3 \\mathrm{~A}+6 \\mathrm{~B}$ 的值与 $x$ 无关, 求 $y$ 的值;\n\n(3) 7 张如图 1 的小长方形, 长为 $\\mathrm{a}$, 宽为 $\\mathrm{b}$, 按照图 2 方式不重叠地放在大长方形 $\\mathrm{ABCD}$ 内,大长方形中未被覆盖的两个部分 (图中阴影部分), 设右上角的面积为 $S_{1}$, 左下角的面积为 $S_{2}$, 当 $\\mathrm{AB}$ 的长变化时, $S_{1}-S_{2}$ 的值始终保持不变, 求 $a$ 与 $b$ 的等量关系.", "options": [], "subject": "代数", "analysis": ") 解: $(2 x-3) m+2 m^{2}-3 x=2 m x-3 m+2 m^{2}-3 x$ $=(2 m-3) x-3 m+2 m^{2}$,\n\n$\\because$ 关于 $x$ 的多项式 $(2 x-3) m+2 m^{2}-3 x$ 的值与 $x$ 的取值无关,\n\n$\\therefore 2 m-3=0$,\n\n解得 $m=\\frac{3}{2}$.\n\n(2) 解: $\\because ~ A=2 x^{2}+3 x y-2 x-1, B=-x^{2}+x y-1$,\n\n$$\n\\begin{gathered}\n\\therefore 3 A+6 B=3\\left(2 x^{2}+3 x y-2 x-1\\right)+6\\left(-x^{2}+x y-1\\right) \\\\\n=6 x^{2}+9 x y-6 x-3-6 x^{2}+6 x y-6 \\\\\n=15 x y-6 x-9\n\\end{gathered}\n$$\n\n$=(15 y-6) x-9$,\n\n$\\because 3 A+6 B$ 的值与 $x$ 无关,\n\n$\\therefore 15 y-6=0$,\n\n解得 $y=\\frac{2}{5}$.\n\n(3) 解: 设 $A B=x$,\n\n由图可知, $S_{1}=a(x-3 b)=a x-3 a b, S_{2}=2 b(x-2 a)=2 b x-4 a b$,\n\n则 $S_{1}-S_{2}=a x-3 a b-(2 b x-4 a b)$\n\n$$\n=a x-3 a b-2 b x+4 a b\n$$\n\n$=(a-2 b) x+a b$,\n$\\because$ 当 $A B$ 的长变化时, $S_{1}-S_{2}$ 的值始终保持不变,\n\n$\\therefore S_{1}-S_{2}$ 的值与 $x$ 的值无关,\n\n$\\therefore a-2 b=0$,\n\n$\\therefore a=2 b$."} {"id": "1515", "image": [], "answer": "(1) 解: $\\because A=3 b^{2}-2 a^{2}+5 a b, B=4 a b+2 b^{2}-a^{2}$, $\\therefore 2 A-3 B=2\\left(3 b^{2}-2 a^{2}+5 a b\\right)-3\\left(4 a b+2 b^{2}-a^{2}\\right)=-a^{2}-2 a b ;$\n\n(2) 解:当 $\\mathrm{a}=-1, \\mathrm{~b}=2$ 时, $2 \\mathrm{~A}-3 \\mathrm{~B}=-(-1)^{2}-2 \\times(-1) \\times 2=3$.", "solution": "null", "level": "七年级", "question": "已知 $\\mathrm{A}=3 b^{2}-2 a^{2}+5 a b, \\mathrm{~B}=4 a b+2 b^{2}-a^{2}$\n\n(1) 化简: $2 \\mathrm{~A}-3 \\mathrm{~B}$;\n\n(2)当 $a=-1, b=2$ 时,求 $2 A-3 B$ 的值.\n\n", "options": [], "subject": "代数", "analysis": "(1) 解: $\\because A=3 b^{2}-2 a^{2}+5 a b, B=4 a b+2 b^{2}-a^{2}$, $\\therefore 2 A-3 B=2\\left(3 b^{2}-2 a^{2}+5 a b\\right)-3\\left(4 a b+2 b^{2}-a^{2}\\right)=-a^{2}-2 a b ;$\n\n(2) 解:当 $\\mathrm{a}=-1, \\mathrm{~b}=2$ 时, $2 \\mathrm{~A}-3 \\mathrm{~B}=-(-1)^{2}-2 \\times(-1) \\times 2=3$."} {"id": "1516", "image": [], "answer": "(1) 解:填表如下:\n\n| | 一等奖奖品 | 二等奖奖品 | 三等奖奖品 |\n| :--- | :--- | :--- | :--- |\n| 单价/元 | 20 | 15 | 10 |\n\n\n| 数量 $/$ 件 | $\\mathrm{x}$ | $2 x+10$ | $50-3 x$ |\n| :--- | :--- | :---: | :---: |\n\n(2) 解: $20 x+15(2 x+10)+10(50-3 x)$,\n\n$=20 x+30 x+150+500-30 x$,\n\n$=20 x+650 ($ 元).\n\n所以购买这 60 件奖品所需的总费用为 $(20 x+650)$ 元.", "solution": "null", "level": "七年级", "question": "电影 《长津湖》真实生动地诠释了中国人民伟大的抗美援朝精神, 某校为了对学生进行爱国主义教育,开展了“爱我中华”经典诵读活动,并设立了一、二、三等奖,根据需要一共购买了 60 件奖品, 其中二等奖的奖品的件数比一等奖的奖品件数的 2 倍多 10 件, 各种奖品的单价如下表所示:\n\n| | 一等奖奖品 | 二等奖奖品 | 三等奖奖品 |\n| :--- | :--- | :--- | :--- |\n| 单价/元 | 20 | 15 | 10 |\n| 数量/件 | $\\mathrm{x}$ | $\\underline{(1)}$ | $\\underline{(2)}$ |\n\n(1)用含 $\\mathrm{x}$ 的代数式补全表格;\n\n(2)用含 $\\mathrm{x}$ 的代数式表示购买这 60 件奖品所需的总费用.\n\n", "options": [], "subject": "代数", "analysis": "(1) 解:填表如下:\n\n| | 一等奖奖品 | 二等奖奖品 | 三等奖奖品 |\n| :--- | :--- | :--- | :--- |\n| 单价/元 | 20 | 15 | 10 |\n\n\n| 数量 $/$ 件 | $\\mathrm{x}$ | $2 x+10$ | $50-3 x$ |\n| :--- | :--- | :---: | :---: |\n\n(2) 解: $20 x+15(2 x+10)+10(50-3 x)$,\n\n$=20 x+30 x+150+500-30 x$,\n\n$=20 x+650 ($ 元).\n\n所以购买这 60 件奖品所需的总费用为 $(20 x+650)$ 元."} {"id": "1517", "image": [], "answer": "(1)当 $x=1, y=2$ 时, $(x+y)(x-y)=(1+2) \\times(1-2)=-3, x^{2}-y^{2}=1^{2}-2^{2}=-3$; 当 $x=1, y=3$时, $(x+y)(x-y)=(1+3) \\times(1-3)=-8, x^{2}-y^{2}=1^{2}-3^{2}=-8$; 当 $x=0, y=3$ 时, $(x+y)(x-y)=$ $(0+3) \\times(0-3)=-9, x^{2}-y^{2}=0^{2}-3^{2}=-9$; 当 $x=2, y=1$ 时, $(x+y)(x-y)=(2+1) \\times(2-1)=3, x^{2}-$ $y^{2}=2^{2}-1^{2}=3$; 填写表格如下:\n\n| $x$ | 1 | 1 | 0 | 2 |\n| :--- | :--- | :--- | :--- | :--- |\n| $y$ | 2 | 3 | 3 | 1 |\n| $(x+y)(x-y)$ | -3 | -8 | -9 | 3 |\n| $x^{2}-y^{2}$ | -3 | -8 | -9 | 3 |\n\n观察可发现: $(x+y)(x-y)=x^{2}-y^{2}$ 。故答案为: $(x+y)(x-y)=x^{2}-y^{2}$.\n\n(2) 解: 由 (1) 得: $x^{2}-y^{2}=(x+y)(x-y)$,\n\n$\\therefore 2022^{2}-2021^{2}=(2022+2021) \\times(2022-2021)=4043$.", "solution": "null", "level": "七年级", "question": "填表并回答问题:\n\n| $x$ | 1 | 1 | 0 | 2 |\n| :--- | :--- | :--- | :--- | :--- |\n| $y$ | 2 | 3 | 3 | 1 |\n| $(x+y)(x-y)$ | | | | |\n| $x^{2}-y^{2}$ | | | | |\n\n(1)观察并填出表, 你有何发现, 将你的发现写在横线\n\n上:\n\n(2) 利用你发现的结果计算: $2022^{2}-2021^{2}$.\n\n", "options": [], "subject": "代数", "analysis": "(1)当 $x=1, y=2$ 时, $(x+y)(x-y)=(1+2) \\times(1-2)=-3, x^{2}-y^{2}=1^{2}-2^{2}=-3$; 当 $x=1, y=3$时, $(x+y)(x-y)=(1+3) \\times(1-3)=-8, x^{2}-y^{2}=1^{2}-3^{2}=-8$; 当 $x=0, y=3$ 时, $(x+y)(x-y)=$ $(0+3) \\times(0-3)=-9, x^{2}-y^{2}=0^{2}-3^{2}=-9$; 当 $x=2, y=1$ 时, $(x+y)(x-y)=(2+1) \\times(2-1)=3, x^{2}-$ $y^{2}=2^{2}-1^{2}=3$; 填写表格如下:\n\n| $x$ | 1 | 1 | 0 | 2 |\n| :--- | :--- | :--- | :--- | :--- |\n| $y$ | 2 | 3 | 3 | 1 |\n| $(x+y)(x-y)$ | -3 | -8 | -9 | 3 |\n| $x^{2}-y^{2}$ | -3 | -8 | -9 | 3 |\n\n观察可发现: $(x+y)(x-y)=x^{2}-y^{2}$ 。故答案为: $(x+y)(x-y)=x^{2}-y^{2}$.\n\n(2) 解: 由 (1) 得: $x^{2}-y^{2}=(x+y)(x-y)$,\n\n$\\therefore 2022^{2}-2021^{2}=(2022+2021) \\times(2022-2021)=4043$."} {"id": "1520", "image": [], "answer": "(1) (1) (2) (3)\n\n(2) 解: $\\because$ 只含有字母 $x, y$, 单项式是对称式, 且次数为 6 ,\n\n$\\therefore$ 单项式可以是: $\\mathrm{x}^{3} \\mathrm{y}^{3}$ (答案不唯一);\n\n(3) 解: $\\because A=a^{2} b-3 b^{2} a+\\frac{4}{3} c^{2} a, B=a^{2} b-5 b^{2} a+2 c^{2} a$,\n\n$\\therefore 3 A-2 B=3\\left(a^{2} b-3 b^{2} a+\\frac{4}{3} c^{2} a\\right)-2\\left(a^{2} b-5 b^{2} a+2 c^{2} a\\right)$\n\n$=3 a^{2} b-9 b^{2} a+4 c^{2} a-2 a^{2} b+10 b^{2} a-4 c^{2} a$\n\n$=a^{2} b+b^{2} c$,\n\n根据对称式的定义可知, $\\mathrm{a}^{2} \\mathrm{~b}+\\mathrm{b}^{2} \\mathrm{c}$ 不是对称式,\n\n$\\therefore 3 \\mathrm{~A}-2 \\mathrm{~B}$ 不是对称式.", "solution": "null", "level": "七年级", "question": "阅读下列材料, 完成相应的任务:\n\n对称式\n\n一个含有多个字母的代数式中, 如果任意交换两个字母的位置, 代数式的值都不变, 这样的代数式就叫做对称式.\n\n例如: 代数式 $a b c$ 中任意两个字母交换位置, 可得到代数式 $b a c, a c b, c b a$, 因为 $a b c=$ $b a c=a c b=c b a$, 所以 $a b c$ 是对称式; 而代数式 $a-b$ 中字母 $\\mathrm{a}, \\mathrm{b}$ 交换位置, 得到代数式 $b-a$, 因为 $a-b \\neq b-a$, 所以 $a-b$ 不是对称式.\n\n任务:\n\n(1) 下列四个代数式中,是对称式的是 (填序号);\n(1) $a+b+c$\n(2) $a+b$;\n(3) $a b$\n(4) $\\frac{a}{b}$.\n\n(2) 写出一个只含有字母 $x, y$ 的单项式, 使该单项式是对称式, 且次数为 6 ;\n\n(3) 已知 $A=a^{2} b-3 b^{2} a+\\frac{4}{3} c^{2} a, B=a^{2} b-5 b^{2} a+2 c^{2} a$, 求 $3 A-2 B$, 并直接判断所得结果是否为对称式.\n\n", "options": [], "subject": "代数", "analysis": "(1) 解:根据对称式的定义可知: $\\mathrm{a}+\\mathrm{b}+\\mathrm{c} 、 \\mathrm{a}+\\mathrm{b} 、 \\mathrm{ab}$ 是对称式, $\\frac{a}{b}$ 不是对称式,\n故答案为: (1) (2);"} {"id": "1521", "image": [], "answer": "(1) 是\n\n(2) 解: 根据题中的新定义得: $\\frac{1}{2}+\\frac{b}{3}=\\frac{1+b}{2+3}$,\n\n去分母得: $15+10 b=6+6 b$,\n\n解得: $b=-\\frac{9}{4}$;\n\n(3) 解: 将 $\\mathrm{a}=\\mathrm{m}, \\mathrm{b}=\\mathrm{n}$, 代入 $\\frac{a}{2}+\\frac{b}{3}=\\frac{a+b}{2+3}$ 有, $\\frac{m}{2}+\\frac{n}{3}=\\frac{m+n}{2+3}$,\n\n$\\therefore 9 \\mathrm{~m}+4 \\mathrm{n}=0$,\n\n$\\therefore 4 \\mathrm{n}=-9 \\mathrm{~m}$,\n\n把 $\\mathrm{a}=\\mathrm{m}+1, \\mathrm{~b}=\\mathrm{n}-\\frac{9}{4}$ 代入 $\\frac{a}{2}+\\frac{b}{3}$ 和 $\\frac{a+b}{2+3}$,\n\n$\\therefore \\frac{a}{2}+\\frac{b}{3}=\\frac{m+1}{2}+\\frac{n-\\frac{9}{4}}{3}=-\\frac{m+1}{4}$,\n\n$\\frac{a+b}{2+3}=\\frac{m+1+n-\\frac{9}{4}}{2+3}=-\\frac{m+1}{4}$,\n\n$\\therefore \\frac{m+1}{2}+\\frac{n-\\frac{9}{4}}{3}=\\frac{m+1+n-\\frac{9}{4}}{2+3}$,\n\n$\\therefore\\left(\\mathrm{m}+1, \\mathrm{n}-\\frac{9}{4}\\right)$ 也是“双语数对”.", "solution": "null", "level": "七年级", "question": "一般情况下 $\\frac{a}{2}+\\frac{b}{3}=\\frac{a+b}{2+3}$ 不成立, 但有些数可以使得它成立, 例如: $\\mathrm{a}=\\mathrm{b}=0$, 我们称使得 $\\frac{a}{2}+$ $\\frac{b}{3}=\\frac{a+b}{2+3}$ 成立的一对数 $\\mathrm{a}, \\mathrm{b}$ 为“双语数对”, 记为 $(\\mathrm{a}, \\mathrm{b})$.\n\n(1) 填空: $(-4,9)$ \\$ \\qquad \\$ “双语数对” (填“是”或“否”);\n\n(2) 若 $(1, b)$ 是“双语数对”, 求 $b$ 的值;\n\n(3) 已知 $(m, n)$ 是“双语数对”, 试说明 $\\left(m+1, n-\\frac{9}{4}\\right)$ 也是“双语数对”.\n\n", "options": [], "subject": "代数", "analysis": "(1) $\\because \\frac{-4}{2}+\\frac{9}{3}=1, \\frac{-4+9}{2+3}=1$,\n\n$\\therefore \\frac{-4}{2}+\\frac{9}{3}=\\frac{-4+9}{2+3}=1$,\n\n$\\therefore(-4,9)$ 是“双语数对”,\n\n故答案为: 是;"} {"id": "1522", "image": [], "answer": "(1) 2027\n\n(2) $3(a-b)-7 a+11 b+5$\n\n$$\n\\begin{gathered}\n=3 a-3 b-7 a+11 b+5 \\\\\n=-4 a+8 b+5 \\\\\n=-4(a-2 b)+5 \\\\\n=-4 \\times(-3)+5 \\\\\n=17\n\\end{gathered}\n$$\n\n(3) $2 a^{2}+\\frac{5}{2} a b+3 b^{2}$\n\n$=2\\left(a^{2}+2 a b\\right)-\\frac{3}{2}\\left(a b-2 b^{2}\\right)=2 \\times(-5)-\\frac{3}{2} \\times(-3)=-10+4.5=-5.5$", "solution": "null", "level": "七年级", "question": "有这样一道题“如果代数式 $5 a+3 b$ 的值为-4, 那么代数式 $2(a+b)+4(2 a+b)$ 的值是多少? \"爱动脑筋的汤同学解题过程如下:\n\n$$\n\\text { 原式 }=2 a+2 b+8 a+4 b=10 a+6 b=2(5 a+3 b)=2 \\times(-4)=-8 \\text {. }\n$$\n\n汤同学把 $5 a+3 b$ 作为一个整体求解整体思想是中学数学解题中的一种重要思想方法, 请仿照上面的解题方法,完成下面问题:\n\n(简单应用)\n\n(1) 已知 $a^{2}+a=3$ ,则 $2 a^{2}+2 a+2021=$\n\n(2) 已知 $a-2 b=-3$, 求 $3(a-b)-7 a+11 b+5$ 的值;\n\n(3))(拓展提高)\n\n已知 $a^{2}+2 a b=-5, a b-2 b^{2}=-3$, 求代数式 $2 a^{2}+\\frac{5}{2} a b+3 b^{2}$ 的值.\n\n", "options": [], "subject": "代数", "analysis": "(1) 2027\n\n(2) $3(a-b)-7 a+11 b+5$\n\n$$\n\\begin{gathered}\n=3 a-3 b-7 a+11 b+5 \\\\\n=-4 a+8 b+5 \\\\\n=-4(a-2 b)+5 \\\\\n=-4 \\times(-3)+5 \\\\\n=17\n\\end{gathered}\n$$\n\n(3) $2 a^{2}+\\frac{5}{2} a b+3 b^{2}$\n\n$=2\\left(a^{2}+2 a b\\right)-\\frac{3}{2}\\left(a b-2 b^{2}\\right)=2 \\times(-5)-\\frac{3}{2} \\times(-3)=-10+4.5=-5.5$"} {"id": "1539", "image": [], "answer": "(1) 解: $3 a+2 b-5 a-b$\n\n$$\n=(3-5) a+(2-1) b\n$$\n\n$=-2 a+b$,\n\n当 $a=-2, b=1$ 时, 原式 $=-2 \\times(-2)+1=5$\n\n(2) 解: $\\frac{1}{3} x-3\\left(x-\\frac{1}{5} y^{2}\\right)+\\left(-\\frac{4}{3} x+\\frac{2}{5} y^{2}\\right)$\n\n$$\n=\\frac{1}{3} x-3 x+\\frac{3}{5} y^{2}-\\frac{4}{3} x+\\frac{2}{5} y^{2}\n$$\n\n$=-4 x+y^{2}$,\n\n当 $x=-3, y=\\frac{3}{5}$ 时,\n\n原式 $=-4 \\times(-3)+\\left(\\frac{3}{5}\\right)^{2}=12+\\frac{9}{25}=12 \\frac{9}{25}$", "solution": "null", "level": "七年级", "question": "先化简, 再求值.\n\n(1) $3 a+2 b-5 a-b$ ,其中 $a=-2, b=1$ ;\n\n(2) $\\frac{1}{3} x-3\\left(x-\\frac{1}{5} y^{2}\\right)+\\left(-\\frac{4}{3} x+\\frac{2}{5} y^{2}\\right)$, 其中 $x=-3, y=\\frac{3}{5}$.", "options": [], "subject": "代数", "analysis": "(1) 解: $3 a+2 b-5 a-b$\n\n$$\n=(3-5) a+(2-1) b\n$$\n\n$=-2 a+b$,\n\n当 $a=-2, b=1$ 时, 原式 $=-2 \\times(-2)+1=5$\n\n(2) 解: $\\frac{1}{3} x-3\\left(x-\\frac{1}{5} y^{2}\\right)+\\left(-\\frac{4}{3} x+\\frac{2}{5} y^{2}\\right)$\n\n$$\n=\\frac{1}{3} x-3 x+\\frac{3}{5} y^{2}-\\frac{4}{3} x+\\frac{2}{5} y^{2}\n$$\n\n$=-4 x+y^{2}$,\n\n当 $x=-3, y=\\frac{3}{5}$ 时,\n\n原式 $=-4 \\times(-3)+\\left(\\frac{3}{5}\\right)^{2}=12+\\frac{9}{25}=12 \\frac{9}{25}$"} {"id": "1540", "image": [], "answer": "(1) 解: 由题意可知: $A=6 x^{2} y+12 x y-2 x-9+\\left(3 x^{2} y-5 x y+x+7\\right)$\n\n$=6 x^{2} y+12 x y-2 x-9+3 x^{2} y-5 x y+x+7$\n\n$=9 x^{2} y+7 x y-x-2$.\n\n$\\therefore \\mathrm{A}+\\mathrm{B}=9 \\mathrm{x}^{2} \\mathrm{y}+7 \\mathrm{xy}-\\mathrm{x}-2+\\left(3 \\mathrm{x}^{2} \\mathrm{y}-5 \\mathrm{xy}+\\mathrm{x}+7\\right)$\n\n$=9 x^{2} y+7 x y-x-2+3 x^{2} y-5 x y+x+7$\n\n$=12 x^{2} y+2 x y+5$\n\n(2) 解: $A-3 B=9 x^{2} y+7 x y-x-2-3\\left(3 x^{2} y-5 x y+x+7\\right)$\n\n$=9 x^{2} y+7 x y-x-2-9 x^{2} y+15 x y-3 x-21$\n\n$=22 x y-4 x-23$\n\n$=(22 y-4) x-23$.\n\n$\\because$ 当 $\\mathrm{x}$ 取任意数值, $\\mathrm{A}-3 \\mathrm{~B}$ 的值是一个定值,\n\n$\\therefore 22 \\mathrm{y}-4=0$.\n\n解得: $y=\\frac{2}{11}$.\n\n故答案为 $(1) 12 x^{2} y+2 x y+5 ;(2) \\frac{2}{11}$", "solution": "null", "level": "七年级", "question": "某同学做一道数学题, 已知两个多项式 A、B, B $3 x 2 y-5 x y+x+7$, 试求 $A+B$, 这位同学把 $\\mathrm{A}+\\mathrm{B}$ 看成 $\\mathrm{A}-\\mathrm{B}$, 结果求出的答案为 $6 x 2 y+12 x y-2 x-9$.\n\n(1)请你替这位同学求出的符合题意答案;\n\n(2) 当 $x$ 取任意数值, $A-3 B$ 的值是一个定值, 求 $y$ 的值.", "options": [], "subject": "代数", "analysis": "(1) 解: 由题意可知: $A=6 x^{2} y+12 x y-2 x-9+\\left(3 x^{2} y-5 x y+x+7\\right)$\n\n$=6 x^{2} y+12 x y-2 x-9+3 x^{2} y-5 x y+x+7$\n\n$=9 x^{2} y+7 x y-x-2$.\n\n$\\therefore \\mathrm{A}+\\mathrm{B}=9 \\mathrm{x}^{2} \\mathrm{y}+7 \\mathrm{xy}-\\mathrm{x}-2+\\left(3 \\mathrm{x}^{2} \\mathrm{y}-5 \\mathrm{xy}+\\mathrm{x}+7\\right)$\n\n$=9 x^{2} y+7 x y-x-2+3 x^{2} y-5 x y+x+7$\n\n$=12 x^{2} y+2 x y+5$\n\n(2) 解: $A-3 B=9 x^{2} y+7 x y-x-2-3\\left(3 x^{2} y-5 x y+x+7\\right)$\n\n$=9 x^{2} y+7 x y-x-2-9 x^{2} y+15 x y-3 x-21$\n\n$=22 x y-4 x-23$\n\n$=(22 y-4) x-23$.\n\n$\\because$ 当 $\\mathrm{x}$ 取任意数值, $\\mathrm{A}-3 \\mathrm{~B}$ 的值是一个定值,\n\n$\\therefore 22 \\mathrm{y}-4=0$.\n\n解得: $y=\\frac{2}{11}$.\n\n故答案为 $(1) 12 x^{2} y+2 x y+5 ;(2) \\frac{2}{11}$"} {"id": "1541", "image": [], "answer": "(1) 解: $\\because$ 一班植树 $\\mathrm{a}$ 棵,\n\n$\\therefore$ 二班植树的棵数为 $(3 a-20)$ 棵, 三班植树的棵数为 $\\left[\\frac{1}{2}(3 a-20)+15\\right]$ 棵,\n\n则三个班共植树的棵数为:\n\n$a+3 a-20+\\frac{1}{2}(3 a-20)+15$\n\n$=4 a-20+\\frac{3}{2} a-10+15$\n\n$=(5.5 \\mathrm{a}-15)$ 棵,\n\n答: 三个班共植树为 $(5.5 \\mathrm{a}-15)$ 棵.\n\n(2) 解: 二班比三班多植的棵数为:\n\n$3 a-20-\\left[\\frac{1}{2}(3 a-20)+15\\right]$\n$=(1.5 a-25)$ 棵\n\n当 $\\mathrm{a}=50$ 时\n\n$1.5 \\mathrm{a}-25=1.5 \\times 50-25=50$ (棵)\n\n答: 二班比三班多植 50 棵.", "solution": "null", "level": "七年级", "question": "某校七年级三个班级的学生在植树节当天义务植树,一班植树 a 棵, 二班植树的棵数比一班的 3 倍少 20 棵, 三班植树的棵数比二班的一半多 15 棵.\n\n(1) 求三个班共植树多少棵 (用含 $\\alpha$ 的式子表示);\n\n(2) 当 $a=50$ 时, 求二班比三班多植多少棵?", "options": [], "subject": "代数", "analysis": "(1) 解: $\\because$ 一班植树 $\\mathrm{a}$ 棵,\n\n$\\therefore$ 二班植树的棵数为 $(3 a-20)$ 棵, 三班植树的棵数为 $\\left[\\frac{1}{2}(3 a-20)+15\\right]$ 棵,\n\n则三个班共植树的棵数为:\n\n$a+3 a-20+\\frac{1}{2}(3 a-20)+15$\n\n$=4 a-20+\\frac{3}{2} a-10+15$\n\n$=(5.5 \\mathrm{a}-15)$ 棵,\n\n答: 三个班共植树为 $(5.5 \\mathrm{a}-15)$ 棵.\n\n(2) 解: 二班比三班多植的棵数为:\n\n$3 a-20-\\left[\\frac{1}{2}(3 a-20)+15\\right]$\n$=(1.5 a-25)$ 棵\n\n当 $\\mathrm{a}=50$ 时\n\n$1.5 \\mathrm{a}-25=1.5 \\times 50-25=50$ (棵)\n\n答: 二班比三班多植 50 棵."} {"id": "1543", "image": [], "answer": "(1) 3\n\n(2) 解: $\\frac{a}{2}-3 b+\\left(-\\frac{1}{2} a+2 b\\right)$\n\n$=\\frac{a}{2}-3 \\mathrm{~b}-\\frac{1}{2} \\mathrm{a}+2 \\mathrm{~b}$\n\n$=\\frac{a}{2}-\\frac{1}{2} a-3 b+2 b$\n\n$=-b$.", "solution": "null", "level": "七年级", "question": "已知下面 5 个式子: 观察下列五个式子, 解答问题: $\\frac{1}{3} a b^{2}, \\frac{1}{a}+b, \\frac{a}{2}-3 b,-a+b,-\\frac{1}{2} a+$ $2 b$.\n\n(1) 这五个式子中, 多项式有 \\$ \\qquad \\$个;\n\n(2) 选择两个多项式进行加法运算, 要求计算结果为单项式.", "options": [], "subject": "代数", "analysis": "(1) 解: 式子 $\\frac{1}{3} \\mathrm{ab}^{2}$ 是单项式, 由于 $\\frac{1}{a}$ 不是单项式, 所以式子 $\\frac{1}{a}+\\mathrm{b}$ 不是多项式,式子 $\\frac{a}{2}-3 b,-a+b,-\\frac{1}{2} a+2 b$ 是多项式, 共 3 个;故答案为: 3 ;"} {"id": "1545", "image": ["3174.jpg", "3175.jpg"], "answer": "(1) 解: $(2 x-3) m+2 m^{2}-3 x=2 m x-3 m+2 m^{2}-3 x$\n\n$=(2 m-3) x-3 m+2 m^{2}$,\n\n$\\because$ 关于 $x$ 的多项式 $(2 x-3) m+2 m^{2}-3 x$ 的值与 $x$ 的取值无关,\n\n$\\therefore 2 m-3=0$,\n\n解得 $m=\\frac{3}{2}$.\n\n(2) 解: $\\because A=2 x^{2}+3 x y-2 x-1, B=-x^{2}+x y-1$,\n\n$$\n\\begin{gathered}\n\\therefore 3 A+6 B=3\\left(2 x^{2}+3 x y-2 x-1\\right)+6\\left(-x^{2}+x y-1\\right) \\\\\n=6 x^{2}+9 x y-6 x-3-6 x^{2}+6 x y-6 \\\\\n=15 x y-6 x-9\n\\end{gathered}\n$$\n\n$=(15 y-6) x-9$,\n\n$\\because 3 A+6 B$ 的值与 $x$ 无关,\n\n$\\therefore 15 y-6=0$,\n\n解得 $y=\\frac{2}{5}$.\n\n(3) 解: 设 $A B=x$,\n由图可知, $S_{1}=a(x-3 b)=a x-3 a b, S_{2}=2 b(x-2 a)=2 b x-4 a b$,\n\n则 $S_{1}-S_{2}=a x-3 a b-(2 b x-4 a b)$\n\n$$\n=a x-3 a b-2 b x+4 a b\n$$\n\n$=(a-2 b) x+a b$,\n\n$\\because$ 当 $A B$ 的长变化时, $S_{1}-S_{2}$ 的值始终保持不变,\n\n$\\therefore S_{1}-S_{2}$ 的值与 $x$ 的值无关,\n\n$\\therefore a-2 b=0$,\n\n$\\therefore a=2 b$.", "solution": "null", "level": "七年级", "question": "七年级学习代数式求值时, 遇到这样一类题“代数式 $a x-y+6+3 x-5 y-1$ 的值与 $x$ 的取值无关, 求 $a$ 的值”, 通常的解题方法是: 把 $\\mathrm{x} 、 \\mathrm{y}$ 看作字母, $\\mathrm{a}$ 看作系数合并同类项, 因为代数式的值与 $\\mathrm{x}$ 的取值无关, 所以含 $x$ 项的系数为 0 ,\n\n即原式 $=(a+3) x-6 y+5$, 所以 $a+3=0$, 则 $a=-3$.\n\n\n\n图1\n\n\n\n(1) 若关于 $x$ 的多项式 $(2 x-3) m+2 m^{2}-3 x$ 的值与 $x$ 的取值无关, 求 $m$ 值;\n\n(2) 已知 $\\mathrm{A}=2 x^{2}+3 x y-2 x-1, \\mathrm{~B}=-x^{2}+x y-1$; 且 $3 \\mathrm{~A}+6 \\mathrm{~B}$ 的值与 $x$ 无关, 求 $y$ 的值;\n\n(3) 7 张如图 1 的小长方形, 长为 $\\mathrm{a}$, 宽为 $\\mathrm{b}$, 按照图 2 方式不重叠地放在大长方形 $\\mathrm{ABCD}$ 内,大长方形中未被覆盖的两个部分 (图中阴影部分), 设右上角的面积为 $S_{1}$, 左下角的面积为 $S_{2}$, 当 $\\mathrm{AB}$ 的长变化时, $S_{1}-S_{2}$ 的值始终保持不变, 求 $a$ 与 $b$ 的等量关系.", "options": [], "subject": "代数", "analysis": "(1) 解: $(2 x-3) m+2 m^{2}-3 x=2 m x-3 m+2 m^{2}-3 x$\n\n$=(2 m-3) x-3 m+2 m^{2}$,\n\n$\\because$ 关于 $x$ 的多项式 $(2 x-3) m+2 m^{2}-3 x$ 的值与 $x$ 的取值无关,\n\n$\\therefore 2 m-3=0$,\n\n解得 $m=\\frac{3}{2}$.\n\n(2) 解: $\\because A=2 x^{2}+3 x y-2 x-1, B=-x^{2}+x y-1$,\n\n$$\n\\begin{gathered}\n\\therefore 3 A+6 B=3\\left(2 x^{2}+3 x y-2 x-1\\right)+6\\left(-x^{2}+x y-1\\right) \\\\\n=6 x^{2}+9 x y-6 x-3-6 x^{2}+6 x y-6 \\\\\n=15 x y-6 x-9\n\\end{gathered}\n$$\n\n$=(15 y-6) x-9$,\n\n$\\because 3 A+6 B$ 的值与 $x$ 无关,\n\n$\\therefore 15 y-6=0$,\n\n解得 $y=\\frac{2}{5}$.\n\n(3) 解: 设 $A B=x$,\n由图可知, $S_{1}=a(x-3 b)=a x-3 a b, S_{2}=2 b(x-2 a)=2 b x-4 a b$,\n\n则 $S_{1}-S_{2}=a x-3 a b-(2 b x-4 a b)$\n\n$$\n=a x-3 a b-2 b x+4 a b\n$$\n\n$=(a-2 b) x+a b$,\n\n$\\because$ 当 $A B$ 的长变化时, $S_{1}-S_{2}$ 的值始终保持不变,\n\n$\\therefore S_{1}-S_{2}$ 的值与 $x$ 的值无关,\n\n$\\therefore a-2 b=0$,\n\n$\\therefore a=2 b$."} {"id": "1546", "image": [], "answer": "(1) (1) 21 ; (2)-3;\n\n(3) $x \\otimes \\frac{3}{2}=-3$,\n\n当 $x \\geq \\frac{3}{2}$ 时,\n\n$x \\otimes \\frac{3}{2}=3 x-2 \\times \\frac{3}{2}=-3$\n\n解得: $x=0<\\frac{3}{2}$, 不符合题意, 舍去;\n\n当 $x<\\frac{3}{2}$ 时,\n\n$x \\otimes \\frac{3}{2}=x-\\frac{2}{3} \\times \\frac{3}{2}=-3$,\n\n解得: $x=-2$, 符合题意;\n\n综上可得: $x=-2$;\n\n(2) 解: $A=-2 x^{3}+\\frac{2}{3} x^{2}-x+1, B=-2 x^{3}+x^{2}-x+\\frac{3}{2}$,\n$B-A=\\frac{1}{3} x^{2}+\\frac{1}{2}>0$, 即 $A\\frac{x}{3}+1$,\n\n$\\therefore\\left(\\frac{k}{3} x+k\\right) \\otimes\\left(\\frac{x}{3}+1\\right)=\\frac{4}{3} x+12$,\n\n即 $3\\left(\\frac{k}{3} x+k\\right)-2\\left(\\frac{x}{3}+1\\right)=\\frac{4}{3} x+12$,\n\n去括号得: $k x+3 k-\\frac{2}{3} x-2=\\frac{4}{3} x+12$,\n\n移项合并同类项可得: $(x+3) k=2 x+14$,\n\n$\\therefore k=\\frac{2 x+14}{x+3}=\\frac{2(x+3)+8}{x+3}=2+\\frac{8}{x+3}$,\n\n当 $x=1$ 时, $k=2+\\frac{8}{1+3}=4$,\n\n当 $x=5$ 时, $k=2+\\frac{8}{5+3}=3$,\n\n$\\therefore k=4$ 或 $k=3$.", "solution": "null", "level": "七年级", "question": "对于有理数 $\\mathrm{a} 、 \\mathrm{~b}$ 定义一种新运算 $\\mathrm{a} \\otimes \\mathrm{b}=\\left\\{\\begin{array}{l}3 a-2 b(a \\geq b) \\\\ a-\\frac{2}{3} b(a", "options": [], "subject": "组合几何学", "analysis": "$\\frac{9}{4} \\sqrt{3}$"} {"id": "2437", "image": ["3526.jpg"], "answer": "(1) $\\frac{2}{3}$ $y=-\\frac{1}{8} x^{2}+x \\quad", "solution": "null", "level": "七年级", "question": "(本题 8 分) 如图, 在 $\\triangle A B C$ 中, $\\angle C=90^{\\circ}, A C+B C=8$, 点 $O$ 是斜边 $A B$ 上一点,以 $O$ 为圆心的 $\\odot O$ 分别与 $A C 、 B C$ 相切于点 $D 、 E$.\n\n(1) 当 $A C=2$ 时, 求 $\\odot O$ 的半径; (2) 设 $A C=x, \\odot O$ 的半径为 $y$, 求 $y$ 与 $x$ 的函数关系式.\n\n", "options": [], "subject": "组合几何学", "analysis": "(1) $\\frac{2}{3}$ $y=-\\frac{1}{8} x^{2}+x \\quad"} {"id": "1396", "image": [], "answer": "解: $\\because \\sqrt[3]{8}=2,|-3|=3$, 可得:\n\n正整数: ( (6), (7), )\n\n负分数: ( (3), (5), )\n\n无理数: ( (2), (4), (8), )", "solution": "null", "level": "七年级", "question": "把下列各数对应的序号填在相应的括号里.\n\n(1) 0 , (2) $\\sqrt{5}$, (3) -2.5 , (4) $\\pi$, (5) $-\\frac{12}{7}$, (6) $\\sqrt[3]{8}$, (7) $-3 \\mid$, (8) $1.202002 \\cdots$ (每两个“ “ ”之间依次多一个“ 0 \").\n\n正整数:\n\n负分数:\n\n无理数:", "options": [], "subject": "组合几何学", "analysis": "解: $\\because \\sqrt[3]{8}=2,|-3|=3$, 可得:\n\n正整数: ( (6), (7), )\n\n负分数: ( (3), (5), )\n\n无理数: ( (2), (4), (8), )"} {"id": "1425", "image": ["3124.jpg", "3125.jpg", "3126.jpg", "3127.jpg", "3128.jpg", "3129.jpg"], "answer": "(1) $5 ; \\sqrt{5}$\n\n(2) 解:如图所示 :\n\n\n\n图3\n\n(3) 解: 如图所示:\n\n\n\n图4\n\n其边长为 $\\sqrt{10}$.", "solution": "null", "level": "七年级", "question": "如图, 纸上有五个边长为 1 的小正方形组成的图形纸 (图 1), 我们可以把它剪开拼成一个正方形 (图 2).\n\n\n\n图1\n\n\n\n图2\n\n\n\n图3\n\n\n\n图4\n\n(1) 图 2 中拼成的正方形的面积是 \\$ \\qquad \\$ ;边长是 \\$ \\qquad \\$ ; (填实数)\n\n(2)请你在图 3 中画一个面积为 5 的正方形, 要求所画正方形的顶点都在格点上.请用虚线画出.\n\n(3)你能把十个小正方形组成的图形纸(图 4), 剪开并拼成正方形吗? 若能, 请仿照图 2 的形式把它重新拼成一个正方形.并求出它的边长.", "options": [], "subject": "组合几何学", "analysis": "(1) $\\because$ 剪拼前后, 图形的面积不变,\n\n$\\therefore$ 正方形面积仍然为 5\n\n$\\therefore$ 边长为 $\\sqrt{5}$\n\n故答案为: $5, \\sqrt{5}$;"} {"id": "1469", "image": ["3143.jpg", "3144.jpg"], "answer": "(1) 解: 根据图形, 1 个I型窗框用料 $(3 x+2 y)$ 米;\n\n1 个II型窗框用料 $(2 x+3 y)$ 米;\n\n2 个I型窗框和 3 个II型窗框共需这种材料 (单位: 米)\n\n$$\n\\begin{gathered}\n2(3 x+2 y)+3(2 x+3 y) \\\\\n=6 x+4 y+6 x+9 y\n\\end{gathered}\n$$\n\n$=12 x+13 y$\n\n(2) 解: 1 个II型窗框和 1 个I型窗框多用这种材料 (单位: 米)\n\n$$\n\\begin{aligned}\n& (2 x+3 y)-(3 x+2 y) \\\\\n& =2 x+3 y-3 x-2 y\n\\end{aligned}\n$$\n\n$=y-x$.", "solution": "null", "level": "七年级", "question": "现在有一种既隔热又耐老化的新型窗框材料—“断桥铝”, 下图是这种材料做成的两种长方形窗框, 已知窗框的长都是 $y$ 米, 宽都是 $x$ 米.\n\n\n\nI型\n\n\n\nII型\n\n(1)若一用户需I型的窗框 2 个, II型的窗框 3 个, 求共需这种材料多少米(接㖓忽略不计)?\n\n(2) 已知 $y>x$, 求一个I型的窗框比一个II型的窗框节约这种材料多少米?", "options": [], "subject": "组合几何学", "analysis": "(1) 解: 根据图形, 1 个I型窗框用料 $(3 x+2 y)$ 米;\n\n1 个II型窗框用料 $(2 x+3 y)$ 米;\n\n2 个I型窗框和 3 个II型窗框共需这种材料 (单位: 米)\n\n$$\n\\begin{gathered}\n2(3 x+2 y)+3(2 x+3 y) \\\\\n=6 x+4 y+6 x+9 y\n\\end{gathered}\n$$\n\n$=12 x+13 y$\n\n(2) 解: 1 个II型窗框和 1 个I型窗框多用这种材料 (单位: 米)\n\n$$\n\\begin{aligned}\n& (2 x+3 y)-(3 x+2 y) \\\\\n& =2 x+3 y-3 x-2 y\n\\end{aligned}\n$$\n\n$=y-x$."} {"id": "2439", "image": ["3529.jpg"], "answer": "(1) 由三角形相似得到 $\\frac{E H}{B F}=\\frac{A E}{A F}=\\frac{C E}{D F}$, 又 $\\because \\mathrm{E}$ 是 $\\mathrm{CH}$ 的中点, $\\therefore \\mathrm{F}$ 是 $\\mathrm{BD}$ 的中点 (2) 连接 $\\mathrm{OC} 、 \\mathrm{OF}$,在直角三角形 $\\mathrm{BCD}$ 中, $\\mathrm{F}$ 是 $\\mathrm{BD}$ 的中点, $\\therefore \\mathrm{BF}=\\mathrm{CF}$ 再证得 $\\triangle \\mathrm{OCF}=\\triangle \\mathrm{OBF} \\therefore \\angle \\mathrm{OCF}=$ $\\angle \\mathrm{OBF}=90^{\\circ} \\therefore \\mathrm{FC}$ 是圆 $\\mathrm{O}$ 的切线", "solution": "null", "level": "七年级", "question": "(本题 8 分) 如图, 已知点 $\\mathrm{C}$ 是以 $\\mathrm{AB}$ 为直径的 $\\odot \\mathrm{O}$ 上一点, $\\mathrm{CH} \\perp \\mathrm{AB}$ 于点 $\\mathrm{H}$,过点 $\\mathrm{B}$ 作 $\\odot O$ 的切线交直线 $\\mathrm{AC}$ 于点 $\\mathrm{D}$, 点 $\\mathrm{E}$ 为 $\\mathrm{CH}$ 的中点, 连接 $\\mathrm{AE}$ 并延长交 $B D$ 于点 $F$, 直线 $C F$ 交 $A B$ 的延长线于 $G$.\n\n(1) 求证: $\\mathrm{F}$ 是 $\\mathrm{BD}$ 的中点\n\n(2) 求证: FC 是圆 $O$ 的切线\n\n", "options": [], "subject": "画法几何学", "analysis": "(1) 由三角形相似得到 $\\frac{E H}{B F}=\\frac{A E}{A F}=\\frac{C E}{D F}$, 又 $\\because \\mathrm{E}$ 是 $\\mathrm{CH}$ 的中点, $\\therefore \\mathrm{F}$ 是 $\\mathrm{BD}$ 的中点 (2) 连接 $\\mathrm{OC} 、 \\mathrm{OF}$,在直角三角形 $\\mathrm{BCD}$ 中, $\\mathrm{F}$ 是 $\\mathrm{BD}$ 的中点, $\\therefore \\mathrm{BF}=\\mathrm{CF}$ 再证得 $\\triangle \\mathrm{OCF}=\\triangle \\mathrm{OBF} \\therefore \\angle \\mathrm{OCF}=$ $\\angle \\mathrm{OBF}=90^{\\circ} \\therefore \\mathrm{FC}$ 是圆 $\\mathrm{O}$ 的切线"} {"id": "2460", "image": ["3548.jpg"], "answer": "解: $(1) 30$;\n\n(2) 由题意得: $\\angle \\mathrm{PBH}=60^{\\circ}$,\n\n$\\because \\angle \\mathrm{ABC}=30^{\\circ}$,\n\n$\\therefore \\angle \\mathrm{ABP}=90^{\\circ}$, 又 $\\angle \\mathrm{APB}=45^{\\circ}$,\n\n$\\therefore \\triangle \\mathrm{PAB}$ 为等腰直角三角形,\n\n在直角 $\\triangle \\mathrm{PHB}$ 中, $\\mathrm{PB}=\\frac{\\mathrm{PH}}{\\sin \\angle \\mathrm{PBH}}=\\frac{30}{\\frac{\\sqrt{3}}{2}}=20 \\sqrt{3}$.\n\n在直角 $\\triangle \\mathrm{PBA}$ 中, $\\mathrm{AB}=\\mathrm{PB}=20 \\sqrt{3} \\approx 34.6$ 米.\n\n答: $\\mathrm{A}, \\mathrm{B}$ 两点间的距离是 34.6 米.", "solution": "null", "level": "七年级", "question": "(6 分) 如图, 小明在大楼 30 米高 (即 $\\mathrm{PH}=30$ 米) 的窗口 $\\mathrm{P}$ 处进行观测, 测得山坡上 $\\mathrm{A}$处的俯角为 $15^{\\circ}$, 山脚 $\\mathrm{B}$ 处的俯角为 $60^{\\circ}$, 巳知该山坡的坡度 $\\mathrm{i}$ (即 $\\tan \\angle \\mathrm{ABC}$ ) 为 $1: \\sqrt{3}$,点 $\\mathrm{P}, \\mathrm{H}, \\mathrm{B}, \\mathrm{C}, \\mathrm{A}$ 在同一个平面上, 点 $\\mathrm{H} 、 \\mathrm{~B} 、 \\mathrm{C}$ 在同一条直线上, 且 $\\mathrm{PH} \\perp \\mathrm{HC}$.\n\n(1)山坡坡角(即 $\\angle \\mathrm{ABC}$ )的度数等于 30 度;\n\n(2)求 $\\mathrm{A} 、 \\mathrm{~B}$ 两点间的距离(结果精确到 0.1 米,参考数据: $\\sqrt{3} \\approx 1.732$ ).\n\n", "options": [], "subject": "画法几何学", "analysis": "解: $(1) 30$;\n\n(2) 由题意得: $\\angle \\mathrm{PBH}=60^{\\circ}$,\n\n$\\because \\angle \\mathrm{ABC}=30^{\\circ}$,\n\n$\\therefore \\angle \\mathrm{ABP}=90^{\\circ}$, 又 $\\angle \\mathrm{APB}=45^{\\circ}$,\n\n$\\therefore \\triangle \\mathrm{PAB}$ 为等腰直角三角形,\n\n在直角 $\\triangle \\mathrm{PHB}$ 中, $\\mathrm{PB}=\\frac{\\mathrm{PH}}{\\sin \\angle \\mathrm{PBH}}=\\frac{30}{\\frac{\\sqrt{3}}{2}}=20 \\sqrt{3}$.\n\n在直角 $\\triangle \\mathrm{PBA}$ 中, $\\mathrm{AB}=\\mathrm{PB}=20 \\sqrt{3} \\approx 34.6$ 米.\n\n答: $\\mathrm{A}, \\mathrm{B}$ 两点间的距离是 34.6 米."} {"id": "2462", "image": ["3551.jpg"], "answer": "解: (1) 线段 $\\mathrm{BQ}$ 与 $\\mathrm{PQ}$ 相等.\n\n证明: $\\because \\angle \\mathrm{PQB}=90^{\\circ}-41^{\\circ}=49^{\\circ}$, $\\angle \\mathrm{BPQ}=90^{\\circ}-24.5^{\\circ}=65.5^{\\circ}$,\n\n$\\therefore \\angle \\mathrm{PBQ}=180^{\\circ}-49^{\\circ}-65.5^{\\circ}=65.5^{\\circ}$,\n\n$\\therefore \\angle \\mathrm{BPQ}=\\angle \\mathrm{PBQ}$,\n\n$\\therefore \\mathrm{BQ}=\\mathrm{PQ}$\n\n(2) $\\angle \\mathrm{AQB}=180^{\\circ}-49^{\\circ}-41^{\\circ}=90^{\\circ}$, $\\angle \\mathrm{PQA}=90^{\\circ}-49^{\\circ}=41^{\\circ}$,\n\n$\\therefore \\mathrm{AQ}=\\frac{\\mathrm{PQ}}{\\cos 41^{\\circ}}=\\frac{1200}{0.75}=1600$,\n\n$\\mathrm{BQ}=\\mathrm{PQ}=1200$,\n\n$\\therefore \\mathrm{AB}^{2}=\\mathrm{AQ}^{2}+\\mathrm{BQ}^{2}=1600^{2}+1200^{2}$,\n\n$\\therefore A B=2000$,\n\n答: A、B 的距离为 $2000 \\mathrm{~m}$.", "solution": "null", "level": "七年级", "question": "(8 分) 如图, 自来水厂 A 和村庄 $\\mathrm{B}$ 在小河 $\\mathrm{l}$ 的两侧, 现要在 $\\mathrm{A}, \\mathrm{B}$ 间铺设一条输水管道. 为了搞好工程预算, 需测算出 $A, B$ 间的距离. 一小船在点 $\\mathrm{P}$ 处测得 $\\mathrm{A}$ 在正北方向, $\\mathrm{B}$位于南偏东 $24.5^{\\circ}$ 方向, 前行 $1200 \\mathrm{~m}$, 到达点 $\\mathrm{Q}$ 处, 测得 $\\mathrm{A}$ 位于北偏西 $49^{\\circ}$ 方向, $\\mathrm{B}$ 位于南偏西 $41^{\\circ}$ 方向.\n\n(1) 线段 $\\mathrm{BQ}$ 与 $\\mathrm{PQ}$ 是否相等 $?$ 请说明理由;\n\n(2) 求 $\\mathrm{A}, \\mathrm{B}$ 间的距离. (参考数据 $\\cos 41^{\\circ} \\approx 0.75$ )\n\n", "options": [], "subject": "画法几何学", "analysis": "解: (1) 线段 $\\mathrm{BQ}$ 与 $\\mathrm{PQ}$ 相等.\n\n证明: $\\because \\angle \\mathrm{PQB}=90^{\\circ}-41^{\\circ}=49^{\\circ}$, $\\angle \\mathrm{BPQ}=90^{\\circ}-24.5^{\\circ}=65.5^{\\circ}$,\n\n$\\therefore \\angle \\mathrm{PBQ}=180^{\\circ}-49^{\\circ}-65.5^{\\circ}=65.5^{\\circ}$,\n\n$\\therefore \\angle \\mathrm{BPQ}=\\angle \\mathrm{PBQ}$,\n\n$\\therefore \\mathrm{BQ}=\\mathrm{PQ}$\n\n(2) $\\angle \\mathrm{AQB}=180^{\\circ}-49^{\\circ}-41^{\\circ}=90^{\\circ}$, $\\angle \\mathrm{PQA}=90^{\\circ}-49^{\\circ}=41^{\\circ}$,\n\n$\\therefore \\mathrm{AQ}=\\frac{\\mathrm{PQ}}{\\cos 41^{\\circ}}=\\frac{1200}{0.75}=1600$,\n\n$\\mathrm{BQ}=\\mathrm{PQ}=1200$,\n\n$\\therefore \\mathrm{AB}^{2}=\\mathrm{AQ}^{2}+\\mathrm{BQ}^{2}=1600^{2}+1200^{2}$,\n\n$\\therefore A B=2000$,\n\n答: A、B 的距离为 $2000 \\mathrm{~m}$."} {"id": "2463", "image": ["3552.jpg", "3553.jpg", "3553.jpg"], "answer": "解: (1) 测量图案 (示意图) 如图示.\n\n(2) 测量步骤:\n\n第一步: 在地面上选择点 C 安装测角仪, 测得此时树尖 A 的仰角 $\\angle A H E=\\alpha$,\n\n第二步: 沿 $\\mathrm{CB}$ 前进到点 $\\mathrm{D}$, 用皮尺量出 $\\mathrm{C}, \\mathrm{D}$ 之间的距离 $\\mathrm{CD}=\\mathrm{m}$,\n\n第三步: 在点 $\\mathrm{D}$ 安装测角仪, 测得此时树尖 $\\mathrm{A}$ 的仰角 $\\angle A F E=\\beta$,\n\n第四步: 用皮尺测出测角仪的高 $h$.\n\n(3) 计算: 令 $A E=x$, 则 $\\tan \\alpha=\\frac{x}{H E}$, 得 $H E=\\frac{x}{\\tan \\alpha}$, 又 $\\tan \\beta=\\frac{x}{E F}$, 得 $E F=\\frac{x}{\\tan \\beta}$,\n\n$\\because \\mathrm{HE}-\\mathrm{FE}=\\mathrm{HF}=\\mathrm{CD}=\\mathrm{m}$\n\n$\\therefore \\frac{x}{\\tan \\alpha}-\\frac{x}{\\tan \\beta}=\\pi$\n\n解得 $x=\\frac{m \\tan \\alpha \\cdot \\tan \\beta}{\\tan \\beta-\\tan \\alpha}$,\n\n$\\therefore A B=\\frac{m \\tan \\alpha \\cdot \\tan \\beta}{\\tan \\beta-\\tan \\alpha}+h$.\n\n", "solution": "null", "level": "七年级", "question": "(10 分) 如图, 小山上有一棵树. 现有测角仪和皮尺两种测量工具, 请你设计一种测量方案, 在山脚水平地面上测出小树顶端 $\\mathrm{A}$ 到水平地面的距离 $\\mathrm{AB}$.\n\n要求:\n\n(1) 画出测量示意图;\n\n(2) 写出测量步骤 (测量数据用字母表示);\n\n(3) 根据(2)中的数据计算 $\\mathrm{AB}$.\n\n", "options": [], "subject": "画法几何学", "analysis": "解: (1) 测量图案 (示意图) 如图示.\n\n(2) 测量步骤:\n\n第一步: 在地面上选择点 C 安装测角仪, 测得此时树尖 A 的仰角 $\\angle A H E=\\alpha$,\n\n第二步: 沿 $\\mathrm{CB}$ 前进到点 $\\mathrm{D}$, 用皮尺量出 $\\mathrm{C}, \\mathrm{D}$ 之间的距离 $\\mathrm{CD}=\\mathrm{m}$,\n\n第三步: 在点 $\\mathrm{D}$ 安装测角仪, 测得此时树尖 $\\mathrm{A}$ 的仰角 $\\angle A F E=\\beta$,\n\n第四步: 用皮尺测出测角仪的高 $h$.\n\n(3) 计算: 令 $A E=x$, 则 $\\tan \\alpha=\\frac{x}{H E}$, 得 $H E=\\frac{x}{\\tan \\alpha}$, 又 $\\tan \\beta=\\frac{x}{E F}$, 得 $E F=\\frac{x}{\\tan \\beta}$,\n\n$\\because \\mathrm{HE}-\\mathrm{FE}=\\mathrm{HF}=\\mathrm{CD}=\\mathrm{m}$\n\n$\\therefore \\frac{x}{\\tan \\alpha}-\\frac{x}{\\tan \\beta}=\\pi$\n\n解得 $x=\\frac{m \\tan \\alpha \\cdot \\tan \\beta}{\\tan \\beta-\\tan \\alpha}$,\n\n$\\therefore A B=\\frac{m \\tan \\alpha \\cdot \\tan \\beta}{\\tan \\beta-\\tan \\alpha}+h$.\n\n"} {"id": "1397", "image": [], "answer": "(1) 解: $\\because\\left(-\\frac{5}{8}\\right)^{3}=-\\frac{125}{512}$, $\\therefore-\\frac{125}{512}$ 的立方根是: $-\\frac{5}{8}$;\n\n(2) 解: $\\because 3^{3}=27$,\n\n$\\therefore 27$ 的立方根是: 3 ;\n\n(3) 解: $\\quad(-0.09)^{3}=-0.000729$,\n\n$\\therefore-0.000729$ 的立方根是: -0.09 .", "solution": "null", "level": "七年级", "question": "求下列各数的立方根:\n\n(1) $-\\frac{125}{512}$\n\n(2) 27\n\n(3) -0.000729", "options": [], "subject": "立体几何学", "analysis": "(1) 解: $\\because\\left(-\\frac{5}{8}\\right)^{3}=-\\frac{125}{512}$, $\\therefore-\\frac{125}{512}$ 的立方根是: $-\\frac{5}{8}$;\n\n(2) 解: $\\because 3^{3}=27$,\n\n$\\therefore 27$ 的立方根是: 3 ;\n\n(3) 解: $\\quad(-0.09)^{3}=-0.000729$,\n\n$\\therefore-0.000729$ 的立方根是: -0.09 ."} {"id": "2021", "image": ["3350.jpg"], "answer": "解: 设这个长方体容器的长 $x \\mathrm{~cm}$,\n\n则 $5 x^{2}=350$,\n\n所以 $x=\\sqrt{70}$ (长方体的长不能为负),\n\n因为", "solution": "null", "level": "七年级", "question": "(本小题 8.0 分)\n\n一个容积为 $350 \\mathrm{~cm}^{3}$ 的正方体容器中装满水, 现要将其中的水全部倒入到另一个长方体容器中, 若长方体容器的长与宽相等且高是 $5 \\mathrm{~cm}$, 则这个长方体容器的长与宽至少是多少? (结果精确到 $0.1 \\mathrm{~cm})$\n\n", "options": [], "subject": "立体几何学", "analysis": "$"} {"id": "1878", "image": ["3305.jpg", "3306.jpg"], "answer": "解: (1)因为 $m+p=0$, 所以 $m$ 与 $p$ 互为相反数,所以原点的大致位置如图,\n\n\n所以绝对值最小的数是 $q$,\n\n故答案为: $q$;\n\n(2)由(1)数轴可知, $q<0,-n<0$,\n\n故答案为: $<,<$;\n\n(3)因为 $m+p=0, M P=10$, 则 $m=-5, p=5$.\n\n因为 $M Q=4$, 则 $q=-1$.\n\n因为 $Q N=8$, 则 $n=7$.", "solution": "null", "level": "七年级", "question": "(本小题8.0分)\n\n如图, 已知四个有理数 $m, n, p, q$ 在一条缺失了原点和刻度的数轴上对应的点分别为 $M, N$, $P, Q$, 且 $m+p=0$, 则\n\n\n\n(1) 在 $m, n, p, q$ 这四个有理数中, 绝对值最小的一个数是\n\n(2) 比较大小: $q$ $0,-n$ ", "options": [], "subject": "图论", "analysis": "略"} {"id": "1952", "image": [], "answer": "解: 根据题意得, $a+1=0, b-2=0$,\n\n解得 $a=-1, b=2$,\n\n$\\therefore(-a-b)^{2020}+(a+b)^{2019}=(1-2)^{2020}+(-1+2)^{2019}=1+1=2$.", "solution": "null", "level": "七年级", "question": "(本小题8.0分)\n\n已知 $|a+1|+(b-2)^{2}=0$, 求 $(-a-b)^{2020}+(a+b)^{2019}$ 的值.", "options": [], "subject": "逻辑题", "analysis": "根据非负数的性质列式求出 $a 、 b$ 的值, 然后代入代数式进行计算即可求解.本题考查了非负数的性质: 几个非负数的和为 0 时, 这几个非负数都为 0 ."} {"id": "2562", "image": ["3637.jpg", "3638.jpg", "3638.jpg", "3637.jpg", "3638.jpg"], "answer": "解:(1)如图, $\\triangle D E F$ 即为所求. (2) 平行四边形 $A B E D$ 的面积 $=$ 平行四边形 $A B M N$ 的面积 $=7 \\times 4=28$. 故答案为: 28 .\n\n(3) 如图, 点 $P$ 即为所求 (答案不唯一).\n\n", "solution": "null", "level": "七年级", "question": "(2021 春・连云港期末)在正方形的网格中, 每个小正方形的边长为 1 个单位长度, $\\triangle A B C$ 的三个顶点 $A, B, C$ 都在格点 (正方形网格的交点称为格点). 现将 $\\triangle A B C$ 平移. 使点 $A$ 平移到点 $D$,点 $E 、 F$ 分别是 $B 、 C$ 的对应点. (1) 在图中请画出平移后的 $\\triangle D E F ;$ (2) 四边形 $A B E D$ 的面积为\n\n(3) 在网格中画出一个格点 $P$, 使得 $S_{\\triangle B C P}=\\frac{1}{2} S_{\\triangle D E F}$. (画出一个即可)\n\n\n\n【思路点拨】(1)利用平移变换的性质分别作出 $B, C$ 的对应点 $E, F$ 即可.\n\n(2) 平行四边形 $A B E D$ 的面积 $=$ 平行四边形 $A B M N$ 的面积, 利用轴的思想解决问题.\n\n(3) 取 $A B$ 的中点 $P$ 即可(答案不唯一).\n\n【答案】解:(1)如图, $\\triangle D E F$ 即为所求. (2) 平行四边形 $A B E D$ 的面积 $=$ 平行四边形 $A B M N$ 的面积 $=7 \\times 4=28$. 故答案为: 28 .\n\n(3) 如图, 点 $P$ 即为所求 (答案不唯一).\n\n\n\n【点睛】本题考查作图 - 平移变换, 平行四边形的面积, 三角形的面积等知识, 解题的关键是正确作出图形,学会利用转化的思想解决问题", "options": [], "subject": "变换几何", "analysis": "春・连云港期末)在正方形的网格中, 每个小正方形的边长为 1 个单位长度, $\\triangle A B C$ 的三个顶点 $A, B, C$ 都在格点 (正方形网格的交点称为格点). 现将 $\\triangle A B C$ 平移. 使点 $A$ 平移到点 $D$,点 $E 、 F$ 分别是 $B 、 C$ 的对应点. (1) 在图中请画出平移后的 $\\triangle D E F ;$ (2) 四边形 $A B E D$ 的面积为\n\n(3) 在网格中画出一个格点 $P$, 使得 $S_{\\triangle B C P}=\\frac{1}{2} S_{\\triangle D E F}$. (画出一个即可)\n\n\n\n【思路点拨】(1)利用平移变换的性质分别作出 $B, C$ 的对应点 $E, F$ 即可.\n\n(2) 平行四边形 $A B E D$ 的面积 $=$ 平行四边形 $A B M N$ 的面积, 利用轴的思想解决问题.\n\n(3) 取 $A B$ 的中点 $P$ 即可(答案不唯一).\n\n【答案】解:(1)如图, $\\triangle D E F$ 即为所求. (2) 平行四边形 $A B E D$ 的面积 $=$ 平行四边形 $A B M N$ 的面积 $=7 \\times 4=28$. 故答案为: 28 .\n\n(3) 如图, 点 $P$ 即为所求 (答案不唯一).\n\n\n\n【点睛】本题考查作图 - 平移变换, 平行四边形的面积, 三角形的面积等知识, 解题的关键是正确作出图形,学会利用转化的思想解决问题."} {"id": "3582", "image": ["4879.jpg", "4880.jpg", "4881.jpg", "4879.jpg", "4880.jpg", "4881.jpg"], "answer": "解: (1)原式 $=-1+3 \\times\\left(\\frac{\\sqrt{2}}{2}\\right)^{-1}+1-(3 \\sqrt{2}-4)$\n\n$=-1+3 \\sqrt{2}+1-3 \\sqrt{2}+4$\n\n$=4$;\n\n(2) 原式 $=\\frac{x y+2 x+2 y}{x+y} \\times(x+y)$\n\n$=x y+2 x+2 y$,\n\n$\\because x=8 \\cos 60^{\\circ}-\\tan 45^{\\circ}=8 \\times \\frac{1}{2}-1=3, y=-1$,\n\n$\\therefore$ 原式 $=3 \\times(-1)+2 \\times 3+2 \\times(-1)=1$\n\n20、【答案】(1)如图所示:\n\n\n\n主视图\n\n\n\n左视图\n\n\n\n俯视图\n\n(2) 21 ;\n\n(3)", "solution": "null", "level": "九年级", "question": "(10 分) (1)计算: $(-1)^{2017}+3(\\cos 45 \\text { ٪ })^{-1}+\\left(\\frac{\\sqrt{2}}{2}\\right)^{0}-|4-\\sqrt{18}|$\n\n(2)求式子 $\\left(\\frac{x y}{x+y}+2\\right) \\div \\frac{1}{x+y}$ 的值, 其中 $x=8 \\cos 60^{\\circ}-\\tan 45^{\\circ}, y=-1$.", "options": [], "subject": "算术", "analysis": "解: (1)原式 $=-1+3 \\times\\left(\\frac{\\sqrt{2}}{2}\\right)^{-1}+1-(3 \\sqrt{2}-4)$\n\n$=-1+3 \\sqrt{2}+1-3 \\sqrt{2}+4$\n\n$=4$;\n\n(2) 原式 $=\\frac{x y+2 x+2 y}{x+y} \\times(x+y)$\n\n$=x y+2 x+2 y$,\n\n$\\because x=8 \\cos 60^{\\circ}-\\tan 45^{\\circ}=8 \\times \\frac{1}{2}-1=3, y=-1$,\n\n$\\therefore$ 原式 $=3 \\times(-1)+2 \\times 3+2 \\times(-1)=1$\n\n20、【答案】(1)如图所示:\n\n\n\n主视图\n\n\n\n左视图\n\n\n\n俯视图\n\n(2) 21 ;\n\n(3)"} {"id": "3605", "image": [], "answer": "解: $(1)|2-\\sqrt{3}|+2 \\sin 60^{\\circ}+\\left(\\frac{1}{3}\\right)^{-1}-(\\sqrt{2019}-1)^{0}$\n\n$$\n\\begin{aligned}\n= & 2-\\sqrt{3}+2 \\times \\frac{\\sqrt{3}}{2}+\\frac{1}{\\frac{1}{3}}-1 \\\\\n= & 2-\\sqrt{3}+\\sqrt{3}+3-1\n\\end{aligned}\n$$\n\n$=4$;\n\n$$\n\\begin{gathered}\n\\text { (2) } \\frac{x^{2}-2 x+1}{x^{2}-1} \\div\\left(\\frac{x-1}{x+1}-x+1\\right) \\\\\n=\\frac{x^{2}-2 x+1}{x^{2}-1} \\div \\frac{x-1-(x-1)(x+1)}{x+1} \\\\\n=\\frac{(x-1)^{2}}{(x-1)(x+1)} \\div \\frac{-x(x-1)}{x+1} \\\\\n=\\frac{(x-1)^{2}}{(x-1)(x+1)} \\times \\frac{x+1}{-x(x-1)}\n\\end{gathered}\n$$\n\n$=-\\frac{1}{x}$,\n\n$\\because x^{2}-3 x+2=0$,\n\n$\\therefore(x-2)(x-1)=0$,\n\n$\\therefore x=2$ 或 $x=1$,\n\n当 $x=1$ 时, 分式无意义,则 $x=1$ 舍去;\n\n当 $x=2$ 时, 原式 $=-\\frac{1}{2}$.", "solution": "null", "level": "九年级", "question": "(每小题 3 分, 合计 6 分) $(1)|2-\\sqrt{3}|+2 \\sin 60 \\cdots+\\left(\\frac{1}{3}\\right)^{-1}-(\\sqrt{2019}-1)^{0}$\n\n(2)化简求值 $\\frac{x^{2}-2 x+1}{x^{2}-1} \\div\\left(\\frac{x-1}{x+1}-x+1\\right)$, 其中 $x$ 是方程 $x^{2}-3 x+2=0$ 的根.", "options": [], "subject": "算术", "analysis": "解: $(1)|2-\\sqrt{3}|+2 \\sin 60^{\\circ}+\\left(\\frac{1}{3}\\right)^{-1}-(\\sqrt{2019}-1)^{0}$\n\n$$\n\\begin{aligned}\n= & 2-\\sqrt{3}+2 \\times \\frac{\\sqrt{3}}{2}+\\frac{1}{\\frac{1}{3}}-1 \\\\\n= & 2-\\sqrt{3}+\\sqrt{3}+3-1\n\\end{aligned}\n$$\n\n$=4$;\n\n$$\n\\begin{gathered}\n\\text { (2) } \\frac{x^{2}-2 x+1}{x^{2}-1} \\div\\left(\\frac{x-1}{x+1}-x+1\\right) \\\\\n=\\frac{x^{2}-2 x+1}{x^{2}-1} \\div \\frac{x-1-(x-1)(x+1)}{x+1} \\\\\n=\\frac{(x-1)^{2}}{(x-1)(x+1)} \\div \\frac{-x(x-1)}{x+1} \\\\\n=\\frac{(x-1)^{2}}{(x-1)(x+1)} \\times \\frac{x+1}{-x(x-1)}\n\\end{gathered}\n$$\n\n$=-\\frac{1}{x}$,\n\n$\\because x^{2}-3 x+2=0$,\n\n$\\therefore(x-2)(x-1)=0$,\n\n$\\therefore x=2$ 或 $x=1$,\n\n当 $x=1$ 时, 分式无意义,则 $x=1$ 舍去;\n\n当 $x=2$ 时, 原式 $=-\\frac{1}{2}$."} {"id": "3584", "image": ["4882.jpg", "4883.jpg", "4883.jpg"], "answer": "(1)证明:连接 $O A$,\n\n\n\n$\\because A D$ 是 $\\odot O$ 的切线,\n$\\therefore O A \\perp A D$,\n\n$\\therefore \\angle O A D=\\angle C A D+\\angle O A C=90^{\\circ}$,\n\n$\\because \\angle C=90^{\\circ}$,\n\n$\\therefore \\angle O A C+\\angle A O C=90^{\\circ}$,\n\n$\\therefore \\angle C A D=\\angle A O C$,\n\n$\\because O A=O B$,\n\n$\\therefore \\angle B=\\angle O A B$,\n\n$\\therefore \\angle C A D=\\angle A O C=\\angle B+\\angle O A B=2 \\angle B ;$\n\n(2)解:设 $O A=r$, 由 $O B=r$,\n\nRt $\\triangle C A B$ 中, $\\sin B=\\frac{3}{5}=\\frac{A C}{A B}$,\n\n$\\because A C=6$,\n\n$\\therefore A B=10$,\n\n$\\therefore B C=8$,\n\nRt $\\triangle A C O$ 中, 由勾股定理得: $A C^{2}+C O^{2}=A O^{2}$,\n\n$\\therefore 6^{2}+(8-r)^{2}=r^{2}$,\n\n解得: $r=\\frac{25}{4}$,\n\n答: $\\odot O$ 的半径是 $\\frac{25}{4}$.", "solution": "null", "level": "九年级", "question": "(10 分) 如图, 以 $\\odot O$ 的弦 $A B$ 为斜边作 $R t \\triangle A B C, \\angle C=90^{\\circ}, C$ 点在圆内, 边 $B C$ 经过圆心 $O$,过 $A$ 点作 $\\odot O$ 的切线 $A D$.\n\n\n\n(1)求证: $\\angle D A C=2 \\angle B$;\n\n(2)若 $\\sin B=\\frac{3}{5}, A C=6$, 求 $\\odot O$ 的半径.", "options": [], "subject": "画法几何学", "analysis": "(1)证明:连接 $O A$,\n\n\n\n$\\because A D$ 是 $\\odot O$ 的切线,\n$\\therefore O A \\perp A D$,\n\n$\\therefore \\angle O A D=\\angle C A D+\\angle O A C=90^{\\circ}$,\n\n$\\because \\angle C=90^{\\circ}$,\n\n$\\therefore \\angle O A C+\\angle A O C=90^{\\circ}$,\n\n$\\therefore \\angle C A D=\\angle A O C$,\n\n$\\because O A=O B$,\n\n$\\therefore \\angle B=\\angle O A B$,\n\n$\\therefore \\angle C A D=\\angle A O C=\\angle B+\\angle O A B=2 \\angle B ;$\n\n(2)解:设 $O A=r$, 由 $O B=r$,\n\nRt $\\triangle C A B$ 中, $\\sin B=\\frac{3}{5}=\\frac{A C}{A B}$,\n\n$\\because A C=6$,\n\n$\\therefore A B=10$,\n\n$\\therefore B C=8$,\n\nRt $\\triangle A C O$ 中, 由勾股定理得: $A C^{2}+C O^{2}=A O^{2}$,\n\n$\\therefore 6^{2}+(8-r)^{2}=r^{2}$,\n\n解得: $r=\\frac{25}{4}$,\n\n答: $\\odot O$ 的半径是 $\\frac{25}{4}$."} {"id": "3608", "image": ["4922.jpg", "4923.jpg", "4923.jpg"], "answer": "答案 解: 过点 $D$ 作 $D M \\perp C E$, 交 $C E$ 于点 $M$, 作 $D N \\perp A B$,\n\n交 $A B$ 于点 $N$, 如图所示.\n\n在Rt $\\triangle C M D$ 中, $C D=20 m, \\angle D C M=40^{\\circ}, \\angle C M D=90^{\\circ}$,\n\n\n$\\therefore C M=C D \\cdot \\cos 40^{\\circ} \\approx 15.4 m, D M=C D \\cdot \\sin 40^{\\circ} \\approx 12.8 m$,\n\n$\\therefore D N=M F=C M+C G+G F=60 m$.\n\n在Rt $\\triangle B D N$ 中, $\\angle B D N=10^{\\circ}, \\angle B N D=90^{\\circ}, D N=60 \\mathrm{~m}$,\n\n$\\therefore B N=D N \\cdot \\tan 10^{\\circ} \\approx 10.8 \\mathrm{~m}$.\n\n在Rt $\\triangle A D N$ 中, $\\angle A D N=30^{\\circ}, \\angle A N D=90^{\\circ}, D N=60 \\mathrm{~m}$,\n\n$\\therefore A N=D N \\cdot \\tan 30^{\\circ} \\approx 34.6 \\mathrm{~m}$.\n\n$\\therefore A B=A N+B N=45.4 m$.\n\n答: 瀑布 $A B$ 的高度约为 45.4 米.", "solution": "null", "level": "九年级", "question": "(6 分) 随着人们生活水平的不断提高, 旅游已成为人们的一种生活时尚. 为开发新的旅游项目,我市对某山区进行调查, 发现一瀑布. 为测量它的高度, 测量人员在瀑布的对面山上 $D$ 点处测\n得瀑布顶端 $A$ 点的仰角是 $30^{\\circ}$, 测得瀑布底端 $B$ 点的俯角是 $10^{\\circ}, A B$ 与水平面垂直. 又在瀑布下的水平面测得 $C G=27 m, G F=17.6 m$ (注: $C 、 G 、 F$ 三点在同一直线上, $C F \\perp A B$ 于点 $F$ ).斜坡 $C D=20 \\mathrm{~m}$, 坡角 $\\angle E C D=40^{\\circ}$. 求瀑布 $A B$ 的高度.\n\n(参考数据: $\\sqrt{3} \\approx 1.73, \\sin 40^{\\circ} \\approx 0.64, \\cos 40^{\\circ} \\approx 0.77, \\tan 40^{\\circ} \\approx 0.84, \\sin 10^{\\circ} \\approx 0.17, \\cos 10^{\\circ} \\approx$\n\n\n\n$\\left.0.98, \\tan 10^{\\circ} \\approx 0.18\\right)$", "options": [], "subject": "画法几何学", "analysis": "答案 解: 过点 $D$ 作 $D M \\perp C E$, 交 $C E$ 于点 $M$, 作 $D N \\perp A B$,\n\n交 $A B$ 于点 $N$, 如图所示.\n\n在Rt $\\triangle C M D$ 中, $C D=20 m, \\angle D C M=40^{\\circ}, \\angle C M D=90^{\\circ}$,\n\n\n$\\therefore C M=C D \\cdot \\cos 40^{\\circ} \\approx 15.4 m, D M=C D \\cdot \\sin 40^{\\circ} \\approx 12.8 m$,\n\n$\\therefore D N=M F=C M+C G+G F=60 m$.\n\n在Rt $\\triangle B D N$ 中, $\\angle B D N=10^{\\circ}, \\angle B N D=90^{\\circ}, D N=60 \\mathrm{~m}$,\n\n$\\therefore B N=D N \\cdot \\tan 10^{\\circ} \\approx 10.8 \\mathrm{~m}$.\n\n在Rt $\\triangle A D N$ 中, $\\angle A D N=30^{\\circ}, \\angle A N D=90^{\\circ}, D N=60 \\mathrm{~m}$,\n\n$\\therefore A N=D N \\cdot \\tan 30^{\\circ} \\approx 34.6 \\mathrm{~m}$.\n\n$\\therefore A B=A N+B N=45.4 m$.\n\n答: 瀑布 $A B$ 的高度约为 45.4 米."} {"id": "3611", "image": ["4928.jpg", "4929.jpg", "4930.jpg"], "answer": "(1)证明:由题意可得: $\\angle A=\\angle A D M=30^{\\circ}$,\n\n$\\therefore M A=M D$,\n\n又 $\\because M G \\perp A D$ 于点 $G$,\n\n$\\therefore A G=D G$,\n\n$\\because \\angle B D C=180^{\\circ}-\\angle A D E-\\angle E D F=180^{\\circ}-30^{\\circ}-90^{\\circ}=60^{\\circ}=\\angle B$,\n\n$\\therefore C B=C D$,\n$\\therefore C$ 与 $N$ 重叠,\n\n又 $\\because N H \\perp D B$ 于点 $H$,\n\n$\\therefore D H=B H$,\n\n$\\because A D=D B$,\n\n$\\therefore A G=D H$\n\n(2)解:当 $0^{\\circ}<\\alpha<90^{\\circ}$ 时,(1)中的结论成立.\n\n如图(3), 在Rt $\\triangle A M G$ 中, $\\angle A=30^{\\circ}$,\n\n$\\therefore \\angle A M G=60^{\\circ}=\\angle B$,\n\n又 $\\because \\angle A G M=\\angle N H B=90^{\\circ}$,\n\n$\\therefore \\triangle A G M \\sim \\triangle N H B$,\n\n$\\therefore \\frac{A G}{N H}=\\frac{M G}{B H}$\n\n$\\because \\angle M D G=\\alpha$,\n\n$\\therefore \\angle D M G=90^{\\circ}-\\alpha=\\angle N D H$\n\n又 $\\because \\angle M G D=\\angle D H N=90^{\\circ}$,\n\n$\\therefore$ Rt $\\triangle M G D \\sim R t \\triangle D H N$,\n\n$\\therefore \\frac{D H}{M G}=\\frac{N H}{D G}$,\n\n(1) $\\times$ (2), 得 $\\frac{D G}{A G}=\\frac{B H}{D H}$,\n\n由比例的性质, 得 $\\frac{D G+A G}{A G}=\\frac{B H+D H}{D H}$,\n\n即 $\\frac{A D}{A G}=\\frac{B D}{D H}$,\n\n$\\because A D=D B$,\n\n$\\therefore A G=D H$\n\n(3)在 $R t \\triangle D E F$ 绕点 $D$ 顺时针方向旋转过程中, $\\frac{D M}{D N}$ 值没有改变,\n\n$\\because$ Rt $\\triangle M G D \\sim R t \\triangle D H N$,\n\n$\\therefore \\frac{D M}{D N}=\\frac{M G}{D H}$,\n\n$\\because A G=D H$,\n$\\therefore \\frac{D M}{D N}=\\frac{M G}{A G}=\\tan \\angle A=\\tan 30^{\\circ}=\\frac{\\sqrt{3}}{3}$,\n\n$\\therefore \\frac{D M}{D N}=\\frac{\\sqrt{3}}{3}$.", "solution": "null", "level": "九年级", "question": "(10 分) 已知: 将一副三角板 (Rt $\\triangle A B C$ 和 $R t \\triangle D E F$ ) 如图 (1)摆放, 点 $E 、 A 、 D 、 B$ 在一条直线上, 且 $D$ 是 $A B$ 的中点. 将 $R t \\triangle D E F$ 绕点 $D$ 顺时针方向旋转角 $\\alpha\\left(0^{\\circ}<\\alpha<90^{\\circ}\\right)$, 在旋转过程中,直线 $D E 、 A C$ 相交于点 $M$, 直线 $D F 、 B C$ 相交于点 $N$, 分别过点 $M 、 N$ 作直线 $A B$ 的垂线, 垂足\n为 $G 、 H$.\n\n\n\n图(1)\n\n\n\n图(2)\n\n\n\n图(3)\n\n(1)当 $\\alpha=30^{\\circ}$ 时(如图(2)), 求证: $A G=D H$;\n\n(2)当 $0^{\\circ}<\\alpha<90^{\\circ}$ 时, (1)中的结论是否成立?请根据图 (3)说明理由.\n\n(3)在 $R t \\triangle D E F$ 绕点 $D$ 顺时针方向旋转过程中, $D M$ 与 $D N$ 的比值是否发生改变? 如果不改变,请求出比值; 如果改变, 请说明理由.", "options": [], "subject": "画法几何学", "analysis": "(1)证明:由题意可得: $\\angle A=\\angle A D M=30^{\\circ}$,\n\n$\\therefore M A=M D$,\n\n又 $\\because M G \\perp A D$ 于点 $G$,\n\n$\\therefore A G=D G$,\n\n$\\because \\angle B D C=180^{\\circ}-\\angle A D E-\\angle E D F=180^{\\circ}-30^{\\circ}-90^{\\circ}=60^{\\circ}=\\angle B$,\n\n$\\therefore C B=C D$,\n$\\therefore C$ 与 $N$ 重叠,\n\n又 $\\because N H \\perp D B$ 于点 $H$,\n\n$\\therefore D H=B H$,\n\n$\\because A D=D B$,\n\n$\\therefore A G=D H$\n\n(2)解:当 $0^{\\circ}<\\alpha<90^{\\circ}$ 时,(1)中的结论成立.\n\n如图(3), 在Rt $\\triangle A M G$ 中, $\\angle A=30^{\\circ}$,\n\n$\\therefore \\angle A M G=60^{\\circ}=\\angle B$,\n\n又 $\\because \\angle A G M=\\angle N H B=90^{\\circ}$,\n\n$\\therefore \\triangle A G M \\sim \\triangle N H B$,\n\n$\\therefore \\frac{A G}{N H}=\\frac{M G}{B H}$\n\n$\\because \\angle M D G=\\alpha$,\n\n$\\therefore \\angle D M G=90^{\\circ}-\\alpha=\\angle N D H$\n\n又 $\\because \\angle M G D=\\angle D H N=90^{\\circ}$,\n\n$\\therefore$ Rt $\\triangle M G D \\sim R t \\triangle D H N$,\n\n$\\therefore \\frac{D H}{M G}=\\frac{N H}{D G}$,\n\n(1) $\\times$ (2), 得 $\\frac{D G}{A G}=\\frac{B H}{D H}$,\n\n由比例的性质, 得 $\\frac{D G+A G}{A G}=\\frac{B H+D H}{D H}$,\n\n即 $\\frac{A D}{A G}=\\frac{B D}{D H}$,\n\n$\\because A D=D B$,\n\n$\\therefore A G=D H$\n\n(3)在 $R t \\triangle D E F$ 绕点 $D$ 顺时针方向旋转过程中, $\\frac{D M}{D N}$ 值没有改变,\n\n$\\because$ Rt $\\triangle M G D \\sim R t \\triangle D H N$,\n\n$\\therefore \\frac{D M}{D N}=\\frac{M G}{D H}$,\n\n$\\because A G=D H$,\n$\\therefore \\frac{D M}{D N}=\\frac{M G}{A G}=\\tan \\angle A=\\tan 30^{\\circ}=\\frac{\\sqrt{3}}{3}$,\n\n$\\therefore \\frac{D M}{D N}=\\frac{\\sqrt{3}}{3}$."} {"id": "3632", "image": ["4964.jpg", "4965.jpg", "4966.jpg", "4965.jpg", "4966.jpg"], "answer": "(1)证明:如图 1 中, 连接 $O C$.\n\n\n\n图1\n\n$\\because O A=O C$,\n\n$\\therefore \\angle O A C=\\angle O C A$,\n\n$\\because A C$ 平分 $\\angle D A B$,\n\n$\\therefore \\angle D A C=\\angle C A B=\\angle A C O$,\n\n$\\therefore A D / / O C$,\n\n$\\because A D \\perp C D$,\n\n$\\therefore O C \\perp D C$\n\n$\\therefore C D$ 是 $\\odot O$ 的切线.\n\n(2)解: 如图 2 中, 连接 $B E 、 B C 、 O C, B E$ 交 $A C$ 于 $F$ 交 $O C$ 于 $H$.\n\n\n\n图2\n\n$\\because A B$ 是直径,\n\n$\\therefore \\angle A E B=\\angle D E H=\\angle D=\\angle D C H=90^{\\circ}$,\n\n$\\therefore$ 四边形 $D E H C$ 是矩形,\n\n$\\therefore \\angle E H C=90^{\\circ}$ 即 $O C \\perp E B$,\n\n$\\therefore D C=E H=H B, D E=H C$,\n\n$\\because \\angle D C E=\\angle D A C, \\angle D=\\angle A E F=90^{\\circ}$,\n\n$\\therefore \\triangle C D E \\sim \\triangle A E F$,\n\n$\\therefore \\frac{D E}{E F}=\\frac{D C}{A E}=\\frac{E C}{A F}=2$, 设 $A E=a, E F=b$, 则 $C D=2 a, D E=C H=2 b, F H=2 a-b$,\n\n$\\because A E / / C H$\n\n$\\therefore \\frac{A E}{C H}=\\frac{E F}{F H}$,\n\n$\\therefore \\frac{a}{2 b}=\\frac{b}{2 a-b}$,\n\n$\\therefore 2 a^{2}-a b-2 b^{2}=0$,\n\n$\\therefore a=\\frac{1+\\sqrt{17}}{4} b$ 或 $\\frac{1-\\sqrt{17}}{4} b($ 舍弃),\n\n在Rt $\\triangle A E F$ 中, $A F=\\sqrt{A E^{2}+E F^{2}}=\\frac{\\sqrt{34+2 \\sqrt{34}}}{4} b$,\n\n$\\therefore \\cos \\angle C A D=\\frac{A E}{A F}=\\frac{\\frac{1+\\sqrt{17}}{4} b}{\\frac{\\sqrt{34+2 \\sqrt{34}}}{4} b}=\\frac{1+\\sqrt{17}}{\\sqrt{34+2 \\sqrt{34}}}$.", "solution": "null", "level": "九年级", "question": "如图, 点 $C$ 在以 $A B$ 为直径的 $\\odot O$ 上, $A C$ 平分 $\\angle D A B, A D$ 与过点 $C$ 的直线垂直, 垂足为点 $D, A D$ 交 $\\odot O$ 于点 $E$.\n\n(1)求证: $C D$ 与 $\\odot O$ 相切;\n\n(2)连接 $B E$ 交 $A C$ 于点 $F$, 若 $\\frac{A F}{C E}=\\frac{1}{2}$, 求 $\\cos \\angle C A D$ 的值.\n\n", "options": [], "subject": "画法几何学", "analysis": "(1)证明:如图 1 中, 连接 $O C$.\n\n\n\n图1\n\n$\\because O A=O C$,\n\n$\\therefore \\angle O A C=\\angle O C A$,\n\n$\\because A C$ 平分 $\\angle D A B$,\n\n$\\therefore \\angle D A C=\\angle C A B=\\angle A C O$,\n\n$\\therefore A D / / O C$,\n\n$\\because A D \\perp C D$,\n\n$\\therefore O C \\perp D C$\n\n$\\therefore C D$ 是 $\\odot O$ 的切线.\n\n(2)解: 如图 2 中, 连接 $B E 、 B C 、 O C, B E$ 交 $A C$ 于 $F$ 交 $O C$ 于 $H$.\n\n\n\n图2\n\n$\\because A B$ 是直径,\n\n$\\therefore \\angle A E B=\\angle D E H=\\angle D=\\angle D C H=90^{\\circ}$,\n\n$\\therefore$ 四边形 $D E H C$ 是矩形,\n\n$\\therefore \\angle E H C=90^{\\circ}$ 即 $O C \\perp E B$,\n\n$\\therefore D C=E H=H B, D E=H C$,\n\n$\\because \\angle D C E=\\angle D A C, \\angle D=\\angle A E F=90^{\\circ}$,\n\n$\\therefore \\triangle C D E \\sim \\triangle A E F$,\n\n$\\therefore \\frac{D E}{E F}=\\frac{D C}{A E}=\\frac{E C}{A F}=2$, 设 $A E=a, E F=b$, 则 $C D=2 a, D E=C H=2 b, F H=2 a-b$,\n\n$\\because A E / / C H$\n\n$\\therefore \\frac{A E}{C H}=\\frac{E F}{F H}$,\n\n$\\therefore \\frac{a}{2 b}=\\frac{b}{2 a-b}$,\n\n$\\therefore 2 a^{2}-a b-2 b^{2}=0$,\n\n$\\therefore a=\\frac{1+\\sqrt{17}}{4} b$ 或 $\\frac{1-\\sqrt{17}}{4} b($ 舍弃),\n\n在Rt $\\triangle A E F$ 中, $A F=\\sqrt{A E^{2}+E F^{2}}=\\frac{\\sqrt{34+2 \\sqrt{34}}}{4} b$,\n\n$\\therefore \\cos \\angle C A D=\\frac{A E}{A F}=\\frac{\\frac{1+\\sqrt{17}}{4} b}{\\frac{\\sqrt{34+2 \\sqrt{34}}}{4} b}=\\frac{1+\\sqrt{17}}{\\sqrt{34+2 \\sqrt{34}}}$."} {"id": "3654", "image": ["5002.jpg"], "answer": " $65 x y 2(x y+65 y+65 x)$\n\n解: (1) 由题意, 知该长方体的长为 $y$ 毫米, 宽为 $x$ 毫米, 高为 65 毫米,\n\n则长方体包装盒的体积为: $65 x y$ 立方毫米.\n\n故答案为 $65 x y$;\n\n(2)长方体的表面积(不含内部粘贴角料)为: $2(x y+65 y+65 x)$ 立方毫米;\n\n故答案为: $2(x y+65 y+65 x)$;\n\n(3) $\\because$ 长方体的长为 $y$ 毫米, 宽为 $x$ 毫米, 高为 65 毫米,\n\n$\\therefore$ 长方体的表面积 $=2(x y+65 y+65 x)$ 平方毫米,\n\n又 $\\because$ 内部粘贴角料的面积占长方体表面纸板面积的 $\\frac{1}{5}$,\n\n$\\therefore$ 制作这样一个长方体共需要纸板的面积 $=\\left(1+\\frac{1}{5}\\right) \\times 2(x y+65 y+65 x)=\\frac{12}{5}(x y+65 y+65 x)=$ $\\frac{12}{5} x y+156 y+156 x$ (平方毫米)\n\n$\\because x=40, y=70$\n\n$\\therefore$ :制作这样一个长方体共需要纸板 $\\frac{12}{5} \\times 40 \\times 70+156 \\times 70+156 \\times 40=23880$ 平方毫米.", "solution": "null", "level": "九年级", "question": "某班数学活动小组的同学用纸板制作长方体包装盒, 其平面展开图和相关尺寸如图所示, 其中阴影部分为内部粘贴角料. (单位: 毫米)\n\n(1)此长方体包装盒的体积为 \\$ \\qquad \\$立方毫米; (用含 $x 、 y$ 的式子表示)\n\n(2)此长方体的表面积(不含内部粘贴角料)为 \\$ \\qquad \\$平方毫米; (用含 $x 、 y$ 的式子表示)\n\n(3)若内部粘贴角料的面积占长方体表面纸板面积的 $\\frac{1}{5}$, 求当 $x=40$ 毫米, $y=70$ 毫米时, 制作这\n\n\n\n样一个长方体共需要纸板多少平方毫米.", "options": [], "subject": "画法几何学", "analysis": " $65 x y 2(x y+65 y+65 x)$\n\n解: (1) 由题意, 知该长方体的长为 $y$ 毫米, 宽为 $x$ 毫米, 高为 65 毫米,\n\n则长方体包装盒的体积为: $65 x y$ 立方毫米.\n\n故答案为 $65 x y$;\n\n(2)长方体的表面积(不含内部粘贴角料)为: $2(x y+65 y+65 x)$ 立方毫米;\n\n故答案为: $2(x y+65 y+65 x)$;\n\n(3) $\\because$ 长方体的长为 $y$ 毫米, 宽为 $x$ 毫米, 高为 65 毫米,\n\n$\\therefore$ 长方体的表面积 $=2(x y+65 y+65 x)$ 平方毫米,\n\n又 $\\because$ 内部粘贴角料的面积占长方体表面纸板面积的 $\\frac{1}{5}$,\n\n$\\therefore$ 制作这样一个长方体共需要纸板的面积 $=\\left(1+\\frac{1}{5}\\right) \\times 2(x y+65 y+65 x)=\\frac{12}{5}(x y+65 y+65 x)=$ $\\frac{12}{5} x y+156 y+156 x$ (平方毫米)\n\n$\\because x=40, y=70$\n\n$\\therefore$ :制作这样一个长方体共需要纸板 $\\frac{12}{5} \\times 40 \\times 70+156 \\times 70+156 \\times 40=23880$ 平方毫米."} {"id": "3656", "image": ["5006.jpg", "5007.jpg", "5008.jpg", "5009.jpg", "5010.jpg", "5011.jpg", "5012.jpg", "5013.jpg", "5014.jpg", "5015.jpg", "5012.jpg", "5013.jpg", "5014.jpg", "5015.jpg"], "answer": " 4\n\n【解析】解: (1)这个几何体三个视图如图所示:\n\n\n主视图\n\n\n\n左视图\n\n\n\n俯视图\n\n$(2)(6+6+6) \\times 2+2=38$,\n\n(3)这个几何体的左视图和俯视图不变, 在俯视图上, 标上该位置放小立方体的个数, $(+$ 后面的数是可以增加的数 $)$\n\n\n\n俯视图\n\n因此最多可以增加 4 个,\n\n故答案为: 4 .", "solution": "null", "level": "九年级", "question": "把边长为 1 的 10 个相同正方体摆成如图的形式.\n\n(1)画出该几何体的主视图、左视图、俯视图;\n\n(2)试求出其表面积(包括向下的面);\n\n(3)如果在这个几何体上再添加一些相同的小正方体, 并保持这个几何体的左视图和俯视图不变,\n\n\n\n那么最多可以再添加 \\$ \\qquad \\$个小正方体.\n\n\n\n左视图\n\n\n\n\n\n俯视图\n\n\n\n\n\n主视图", "options": [], "subject": "画法几何学", "analysis": " 4\n\n【解析】解: (1)这个几何体三个视图如图所示:\n\n\n主视图\n\n\n\n左视图\n\n\n\n俯视图\n\n$(2)(6+6+6) \\times 2+2=38$,\n\n(3)这个几何体的左视图和俯视图不变, 在俯视图上, 标上该位置放小立方体的个数, $(+$ 后面的数是可以增加的数 $)$\n\n\n\n俯视图\n\n因此最多可以增加 4 个,\n\n故答案为: 4 ."} {"id": "3678", "image": ["5054.jpg", "5055.jpg", "5055.jpg"], "answer": "解: 作 $D E \\perp A B$ 于 $E$,\n\n\n\n$\\because D C \\perp B C$ 于 $C, A B \\perp B C$ 于 $B$,\n\n$\\therefore$ 四边形 $B C D E$ 为矩形,\n\n$\\therefore D E=B C=20 m, B E=D C=4 m$,\n\n$\\because$ 同一时刻物高与影长所组成的三角形相似,\n\n$\\therefore \\frac{1}{0.8}=\\frac{A E}{20}$,\n\n解得 $A E=25 m$,\n\n$\\therefore A B=25+4=29 \\mathrm{~m}$.\n\n答: 旗杆的高度为 $29 \\mathrm{~m}$.", "solution": "null", "level": "九年级", "question": "如图, 在阳光下, 某一时刻, 旗杆 $A B$ 的影子一部分在地面上, 另一部分在建筑物的墙面上. 小明测得旗杆 $A B$ 在地面上的影长 $B C$ 为 $20 m$ ,在墙面上的影长 $C D$ 为 $4 m$ ,同一时刻,小明又测得坚立于地面长 $1 m$ 的标杆的影长为 $0.8 m$, 求旗杆 $A B$ 的高度\n\n", "options": [], "subject": "画法几何学", "analysis": "解: 作 $D E \\perp A B$ 于 $E$,\n\n\n\n$\\because D C \\perp B C$ 于 $C, A B \\perp B C$ 于 $B$,\n\n$\\therefore$ 四边形 $B C D E$ 为矩形,\n\n$\\therefore D E=B C=20 m, B E=D C=4 m$,\n\n$\\because$ 同一时刻物高与影长所组成的三角形相似,\n\n$\\therefore \\frac{1}{0.8}=\\frac{A E}{20}$,\n\n解得 $A E=25 m$,\n\n$\\therefore A B=25+4=29 \\mathrm{~m}$.\n\n答: 旗杆的高度为 $29 \\mathrm{~m}$."} {"id": "3680", "image": ["5057.jpg", "5058.jpg", "5059.jpg", "5060.jpg", "5060.jpg"], "answer": "解: 设灯泡的位置为点 $P$, 横向影子 $A^{\\prime} B, D^{\\prime} C$ 的长度和为 $x c m$,\n\n$\\because A D / / A^{\\prime} D^{\\prime}$,\n\n$\\therefore \\angle P A D=\\angle P A^{\\prime} D^{\\prime}, \\angle P D A=\\angle P D^{\\prime} A^{\\prime}$.\n$\\therefore \\triangle P A D \\sim \\triangle P A^{\\prime} D^{\\prime}$\n\n$\\therefore \\frac{60}{60+x}=\\frac{150}{180}$,\n\n解得 $x=12 \\mathrm{~cm}$ ,即横向影子 $A^{\\prime} B, D^{\\prime} C$ 的长度和为 $12 \\mathrm{~cm}$;\n\n(2)记灯泡为点 $P$, 如图:\n\n$\\because A D / / A^{\\prime} D^{\\prime}$,\n\n$\\therefore \\angle P A D=\\angle P A^{\\prime} D^{\\prime}, \\angle P D A=\\angle P D^{\\prime} A^{\\prime}$.\n\n$\\therefore \\triangle P A D \\sim \\triangle P A^{\\prime} D^{\\prime}$.\n\n\n\n根据相似三角形对应高的比等于相似比的性质, 可得 $\\frac{A D}{A^{\\prime} D^{\\prime}}=\\frac{P N}{P M}$,\n\n设灯泡离地面距离为 $x$, 由题意, 得 $P M=x, P N=x-a, A D=n a, A^{\\prime} D^{\\prime}=n a+b$,\n\n$\\therefore \\frac{n a}{n a+b}=\\frac{x-a}{x}=1-\\frac{a}{x}$,\n\n$\\therefore \\frac{a}{x}=1-\\frac{n a}{n a+b}$.\n\n$\\therefore x=\\frac{n a^{2}+a b}{b}$.", "solution": "null", "level": "九年级", "question": "张红武和几位同学做手的影子游戏时, 发现对于同一物体, 影子的大小与光源到物体的距离有关. 因此, 他们认为: 可以借助物体的影子长度计算光源到物体的位置. 于是, 他们做了以下尝试, 如图 1, 垂直于地面放置的正方形框架 $A B C D$, 在其正上方有一灯泡, 在灯泡的照射下, 当边长 $A B$ 为 $30 \\mathrm{~cm}$, 正方形框架的横向影子 $A^{\\prime} B, D^{\\prime} C$ 的长度和为 $6 \\mathrm{~cm}$. 根据以上信息, 他们计算出灯泡离地面的高度为 $180 \\mathrm{~cm}$.\n\n\n\n图 1\n\n\n\n图 2\n\n\n\n图 3\n\n(1)不改变图 1 中灯泡的高度, 将两个边长为 $30 \\mathrm{~cm}$ 的正方形框架按图 2 摆放, 请求出此时横向影子 $A^{\\prime} B, D^{\\prime} C$ 的长度和为多少?\n\n(2)有 $n$ 个边长为 $a$ 的正方形按图 3 摆放, 测得横向影子 $A^{\\prime} B, D^{\\prime} C$ 的长度和为 $b$, 求灯泡离地面的高度. (结果用含 $a, b, n$ 的代数式表示)", "options": [], "subject": "画法几何学", "analysis": "解: 设灯泡的位置为点 $P$, 横向影子 $A^{\\prime} B, D^{\\prime} C$ 的长度和为 $x c m$,\n\n$\\because A D / / A^{\\prime} D^{\\prime}$,\n\n$\\therefore \\angle P A D=\\angle P A^{\\prime} D^{\\prime}, \\angle P D A=\\angle P D^{\\prime} A^{\\prime}$.\n$\\therefore \\triangle P A D \\sim \\triangle P A^{\\prime} D^{\\prime}$\n\n$\\therefore \\frac{60}{60+x}=\\frac{150}{180}$,\n\n解得 $x=12 \\mathrm{~cm}$ ,即横向影子 $A^{\\prime} B, D^{\\prime} C$ 的长度和为 $12 \\mathrm{~cm}$;\n\n(2)记灯泡为点 $P$, 如图:\n\n$\\because A D / / A^{\\prime} D^{\\prime}$,\n\n$\\therefore \\angle P A D=\\angle P A^{\\prime} D^{\\prime}, \\angle P D A=\\angle P D^{\\prime} A^{\\prime}$.\n\n$\\therefore \\triangle P A D \\sim \\triangle P A^{\\prime} D^{\\prime}$.\n\n\n\n根据相似三角形对应高的比等于相似比的性质, 可得 $\\frac{A D}{A^{\\prime} D^{\\prime}}=\\frac{P N}{P M}$,\n\n设灯泡离地面距离为 $x$, 由题意, 得 $P M=x, P N=x-a, A D=n a, A^{\\prime} D^{\\prime}=n a+b$,\n\n$\\therefore \\frac{n a}{n a+b}=\\frac{x-a}{x}=1-\\frac{a}{x}$,\n\n$\\therefore \\frac{a}{x}=1-\\frac{n a}{n a+b}$.\n\n$\\therefore x=\\frac{n a^{2}+a b}{b}$."} {"id": "3844", "image": ["5356.jpg", "5357.jpg", "5358.jpg", "5359.jpg", "5360.jpg", "5357.jpg", "5358.jpg", "5359.jpg", "5360.jpg"], "answer": "(1)作图见解析;(2)作图见解析.\n\n【解析】解: (1) 如图所示:\n\n\n\n左视图\n\n\n\n俯视图\n\n(2) 添加后可得如图所示的几何体:\n\n\n左视图分别是:\n\n", "solution": "null", "level": "九年级", "question": "如图, 是由 8 个大小相同的小正方体组合成的简单几何体.\n\n(1)该几何体的主视图如图所示, 请在下面方格纸中分别画出它的左视图和俯视图;\n\n\n\n(2)如果在这个几何体上再添加一些相同的小正方体,并保持这个几何体的俯视图和主视图不变,\n那么请画出添加小正方体后所得几何体可能的左视图.", "options": [], "subject": "画法几何学", "analysis": "(1)作图见解析;(2)作图见解析.\n\n【解析】解: (1) 如图所示:\n\n\n\n左视图\n\n\n\n俯视图\n\n(2) 添加后可得如图所示的几何体:\n\n\n左视图分别是:\n\n"} {"id": "3846", "image": ["5362.jpg", "5363.jpg", "5364.jpg", "5365.jpg", "5363.jpg", "5364.jpg", "5365.jpg"], "answer": "如图所示见解析.\n\n【解析】如图所示:\n\n\n\n从正面看\n\n\n\n从左面看\n\n\n\n从上面看", "solution": "null", "level": "九年级", "question": "画出如图所示图形从正面、从左面和从上面看到的形状图.\n\n", "options": [], "subject": "画法几何学", "analysis": "如图所示见解析.\n\n【解析】如图所示:\n\n\n\n从正面看\n\n\n\n从左面看\n\n\n\n从上面看"} {"id": "3862", "image": ["5407.jpg", "5408.jpg", "5409.jpg", "5408.jpg", "5409.jpg"], "answer": "主视图为:\n\n\n俯视图为:\n\n", "solution": "null", "level": "九年级", "question": "画出下面几何体的三视图:\n\n", "options": [], "subject": "画法几何学", "analysis": "主视图为:\n\n\n俯视图为:\n\n"} {"id": "3865", "image": ["5417.jpg", "5418.jpg", "5419.jpg", "5420.jpg", "5418.jpg", "5419.jpg", "5420.jpg"], "answer": "见解析.\n\n【解析】如图所示.\n\n\n\n主视图\n\n\n\n左视图\n\n\n\n俯视图", "solution": "null", "level": "九年级", "question": "如图, 粗线表示嵌在玻璃正方体内的一根铁丝,请画出该正方体的三视图.\n\n", "options": [], "subject": "画法几何学", "analysis": "见解析.\n\n【解析】如图所示.\n\n\n\n主视图\n\n\n\n左视图\n\n\n\n俯视图"} {"id": "3906", "image": ["5502.jpg", "5503.jpg", "5503.jpg"], "answer": "(1) 见解析; (2) $A B / / E F$. 理由解析.\n\n【解析】 (1) 证明:连接 $B E$;\n\n$\\because A B$ 是 $\\odot O$ 的直径, $\\therefore \\angle A E B=90^{\\circ}$.\n\n$\\because C D$ 切圆于 $E, \\therefore \\angle A E C=\\angle A B E$, 又 $A C \\perp C D$.\n\n$\\therefore \\angle C A E=\\angle B A E$. 即 $A E$ 是 $\\angle B A C$ 的平分线.\n\n\n\n(2) 解: $A B / / E F$. 理由如下:\n\n$\\because A C \\perp C D$ 于 $C, B D \\perp C D$ 于 $D, \\therefore A C / / B D$.\n\n$\\therefore \\angle B A C=180^{\\circ}-\\angle B=120^{\\circ} . \\because A E$ 是 $\\angle B A C$ 的平分线,\n\n$\\therefore \\angle B A E=60^{\\circ} . \\therefore \\angle D F E=\\angle B A E=60^{\\circ}$ (圆内接四边形的任意一个外角等于它的内对角),\n\n$\\therefore \\angle D F E=\\angle A B F . \\therefore A B / / E F$.", "solution": "null", "level": "九年级", "question": "如图, $\\mathrm{AB}$ 是 $\\odot \\mathrm{O}$ 的直径, $\\mathrm{CD}$ 切 $\\odot \\mathrm{O}$ 于 $\\mathrm{E}, \\mathrm{AC} \\perp \\mathrm{CD}$ 于 $C, \\mathrm{BD} \\perp \\mathrm{CD}$ 于 $\\mathrm{D}$, 交 $\\odot \\mathrm{O}$ 于 $\\mathrm{F}$,连接 $\\mathrm{AE} 、 \\mathrm{EF}$.\n\n(1)求证: $\\mathrm{AE}$ 是 $\\angle \\mathrm{BAC}$ 的平分线;\n\n(2)若 $\\angle \\mathrm{ABD}=60^{\\circ}$, 则 $\\mathrm{AB}$ 与 $\\mathrm{EF}$ 是否平行?请说明理由.\n\n", "options": [], "subject": "画法几何学", "analysis": "(1) 见解析; (2) $A B / / E F$. 理由解析.\n\n【解析】 (1) 证明:连接 $B E$;\n\n$\\because A B$ 是 $\\odot O$ 的直径, $\\therefore \\angle A E B=90^{\\circ}$.\n\n$\\because C D$ 切圆于 $E, \\therefore \\angle A E C=\\angle A B E$, 又 $A C \\perp C D$.\n\n$\\therefore \\angle C A E=\\angle B A E$. 即 $A E$ 是 $\\angle B A C$ 的平分线.\n\n\n\n(2) 解: $A B / / E F$. 理由如下:\n\n$\\because A C \\perp C D$ 于 $C, B D \\perp C D$ 于 $D, \\therefore A C / / B D$.\n\n$\\therefore \\angle B A C=180^{\\circ}-\\angle B=120^{\\circ} . \\because A E$ 是 $\\angle B A C$ 的平分线,\n\n$\\therefore \\angle B A E=60^{\\circ} . \\therefore \\angle D F E=\\angle B A E=60^{\\circ}$ (圆内接四边形的任意一个外角等于它的内对角),\n\n$\\therefore \\angle D F E=\\angle A B F . \\therefore A B / / E F$."} {"id": "3910", "image": ["5512.jpg", "5513.jpg", "5513.jpg"], "answer": "(1) $\\mathrm{PC}$ 是 $\\odot \\mathrm{O}$ 的切线, 理由见解析; (2) $\\frac{3}{2} \\sqrt{5}$\n\n【解析】 (1) 结论: PC 是 $\\odot \\mathrm{O}$ 的切线.\n证明:连接 $\\mathrm{OC}$\n\n$\\because \\mathrm{CB} / / \\mathrm{PO} \\therefore \\angle \\mathrm{POA}=\\angle \\mathrm{B}, \\quad \\angle \\mathrm{POC}=\\angle \\mathrm{OCB}$\n\n$\\because \\mathrm{OC}=\\mathrm{OB} \\therefore \\angle \\mathrm{OCB}=\\angle \\mathrm{B}$\n\n$\\therefore \\angle \\mathrm{POA}=\\angle \\mathrm{POC}$ 又 $\\because \\mathrm{OA}=\\mathrm{OC}, \\mathrm{OP}=\\mathrm{OP}$\n\n$\\therefore \\triangle \\mathrm{APO} \\cong \\triangle \\mathrm{CPO} \\therefore \\angle \\mathrm{OAP}=\\angle \\mathrm{OCP}$\n\n$\\because \\mathrm{PA}$ 是 $\\odot \\mathrm{O}$ 的切线 $\\therefore \\angle \\mathrm{OAP}=90^{\\circ}$\n\n$\\therefore \\angle \\mathrm{OCP}=90^{\\circ} \\therefore \\mathrm{PC}$ 是 $\\odot \\mathrm{O}$ 的切线.\n\n\n\n(2) 连接 $A C$\n\n$\\because \\mathrm{AB}$ 是 $\\odot \\mathrm{O}$ 的直径 $\\therefore \\angle \\mathrm{ACB}=90^{\\circ}$ (6 分)\n\n由(1)知 $\\angle \\mathrm{PCO}=90^{\\circ}, \\angle \\mathrm{B}=\\angle \\mathrm{OCB}=\\angle \\mathrm{POC} \\because \\angle \\mathrm{ACB}=\\angle \\mathrm{PCO}$\n\n$\\therefore \\triangle \\mathrm{ACB} \\sim \\triangle \\mathrm{PCO} \\therefore \\frac{\\mathrm{BC}}{\\mathrm{OC}}=\\frac{\\mathrm{AC}}{\\mathrm{PC}}$\n\n$\\therefore \\mathrm{PC}=\\frac{\\mathrm{OC} \\cdot \\mathrm{AC}}{\\mathrm{BC}}=\\frac{3 \\sqrt{A B^{2}-B C^{2}}}{4}=\\frac{3 \\sqrt{6^{2}-4^{2}}}{4}=\\frac{3 \\sqrt{5}}{2}$.", "solution": "null", "level": "九年级", "question": "如图, $\\mathrm{AB}$ 是 $\\odot \\mathrm{O}$ 的直径, $\\mathrm{PA}$ 是 $\\odot \\mathrm{O}$ 的切线, 点 $\\mathrm{C}$ 在 $\\odot \\mathrm{O}$ 上, $\\mathrm{CB} / / \\mathrm{PO}$.\n\n(1) 判断 PC 与 $\\odot \\mathrm{O}$ 的位置关系, 并说明理由;\n\n(2) 若 $\\mathrm{AB}=6, \\mathrm{CB}=4$, 求 $\\mathrm{PC}$ 的长.\n\n", "options": [], "subject": "画法几何学", "analysis": "(1) $\\mathrm{PC}$ 是 $\\odot \\mathrm{O}$ 的切线, 理由见解析; (2) $\\frac{3}{2} \\sqrt{5}$\n\n【解析】 (1) 结论: PC 是 $\\odot \\mathrm{O}$ 的切线.\n证明:连接 $\\mathrm{OC}$\n\n$\\because \\mathrm{CB} / / \\mathrm{PO} \\therefore \\angle \\mathrm{POA}=\\angle \\mathrm{B}, \\quad \\angle \\mathrm{POC}=\\angle \\mathrm{OCB}$\n\n$\\because \\mathrm{OC}=\\mathrm{OB} \\therefore \\angle \\mathrm{OCB}=\\angle \\mathrm{B}$\n\n$\\therefore \\angle \\mathrm{POA}=\\angle \\mathrm{POC}$ 又 $\\because \\mathrm{OA}=\\mathrm{OC}, \\mathrm{OP}=\\mathrm{OP}$\n\n$\\therefore \\triangle \\mathrm{APO} \\cong \\triangle \\mathrm{CPO} \\therefore \\angle \\mathrm{OAP}=\\angle \\mathrm{OCP}$\n\n$\\because \\mathrm{PA}$ 是 $\\odot \\mathrm{O}$ 的切线 $\\therefore \\angle \\mathrm{OAP}=90^{\\circ}$\n\n$\\therefore \\angle \\mathrm{OCP}=90^{\\circ} \\therefore \\mathrm{PC}$ 是 $\\odot \\mathrm{O}$ 的切线.\n\n\n\n(2) 连接 $A C$\n\n$\\because \\mathrm{AB}$ 是 $\\odot \\mathrm{O}$ 的直径 $\\therefore \\angle \\mathrm{ACB}=90^{\\circ}$ (6 分)\n\n由(1)知 $\\angle \\mathrm{PCO}=90^{\\circ}, \\angle \\mathrm{B}=\\angle \\mathrm{OCB}=\\angle \\mathrm{POC} \\because \\angle \\mathrm{ACB}=\\angle \\mathrm{PCO}$\n\n$\\therefore \\triangle \\mathrm{ACB} \\sim \\triangle \\mathrm{PCO} \\therefore \\frac{\\mathrm{BC}}{\\mathrm{OC}}=\\frac{\\mathrm{AC}}{\\mathrm{PC}}$\n\n$\\therefore \\mathrm{PC}=\\frac{\\mathrm{OC} \\cdot \\mathrm{AC}}{\\mathrm{BC}}=\\frac{3 \\sqrt{A B^{2}-B C^{2}}}{4}=\\frac{3 \\sqrt{6^{2}-4^{2}}}{4}=\\frac{3 \\sqrt{5}}{2}$."} {"id": "3912", "image": ["5518.jpg", "5519.jpg", "5519.jpg"], "answer": "(1)证明见解析;(2) 6 ; (3) $\\frac{54}{25} \\sqrt{5}$\n\n【解析】 (1) 证明:连结 $O D . \\because D E \\perp A D, \\therefore A E$ 是 $\\odot O$ 的直径, 即 $O$ 在 $A E$ 上.\n\n$\\because A D$ 是角平分线, $\\therefore \\angle 1=\\angle 2 . \\quad \\because O A=O D, \\quad \\therefore \\angle 2=\\angle 3 . \\quad \\therefore \\angle 1=\\angle 3 . \\quad \\therefore O D / / A C$.\n\n$\\because \\angle C=90^{\\circ}, \\therefore O D \\perp B C . \\quad \\therefore B C$ 是 $\\odot O$ 的切线. (2) 解: $\\because O D / / A C, \\therefore \\angle 4=\\angle E A F$.\n\n$\\because \\angle G=\\angle E A F, \\quad \\therefore \\angle 4=\\angle G . \\quad \\therefore \\tan \\angle 4=\\tan \\angle G=\\frac{4}{3}$.\n\n设 $B D=4 k$, 则 $O D=O E=3 k$. 在 Rt $\\triangle O B D$ 中, 由勾股定理得 $(3 k)^{2}+(4 k)^{2}=(3 k+4)^{2}$,解得, $k_{1}=2, k_{2}=-\\frac{1}{2}$ (舍) , (注:也可由 $O B=5 k=3 k+4$ 得 $k=2$ ),\n\n$\\therefore 3 k=6$, 即 $\\odot O$ 的半径为 6 .\n\n\n\n(3) 解:连结 $A G$, 则 $\\angle A G E=90^{\\circ}, \\angle E G M=\\angle 5$.\n\n$\\therefore \\tan \\angle 5=\\tan \\angle E G M=\\frac{4}{3}$, 即 $\\frac{G M}{A M}=\\frac{E M}{G M}=\\frac{4}{3}, \\frac{A M}{G M}=\\frac{G M}{E M}=\\frac{3}{4}$,\n\n$\\therefore \\frac{A M}{E M}=\\frac{A M}{G M} \\cdot \\frac{G M}{E M}=\\frac{3}{4} \\times \\frac{3}{4}=\\frac{9}{16}, \\quad \\therefore A M=\\frac{9}{25} A E=\\frac{9}{25} \\times 12=\\frac{108}{25}$.\n\n$\\because O D / / A C, \\therefore \\frac{O D}{A C}=\\frac{O B}{A B}, \\frac{C D}{A O}=\\frac{D B}{O B}$, 即 $\\frac{6}{A C}=\\frac{5}{8}, \\frac{C D}{6}=\\frac{8}{10}$.\n\n$\\therefore A C=\\frac{48}{5}, C D=\\frac{24}{5} . \\quad \\because \\angle 1=\\angle 2, \\angle A C D=\\angle A M P=90^{\\circ}, \\therefore \\triangle A C D \\sim \\triangle A M P$.\n\n$\\therefore \\frac{P M}{A M}=\\frac{C D}{A C}=\\frac{1}{2}, \\quad \\therefore P M=\\frac{1}{2} A M=\\frac{54}{25} . \\quad \\therefore A P=\\sqrt{P M^{2}+A M^{2}}=\\frac{54}{25} \\sqrt{5}$.", "solution": "null", "level": "九年级", "question": "如图, 在 Rt $\\triangle A B C$ 中, $\\angle C=90^{\\circ}, A D$ 是角平分线, $D E \\perp A D$ 交 $A B$ 于 $E, \\triangle A D E$ 的外接圆 $\\odot O$与边 $A C$ 相交于点 $F$, 过 $F$ 作 $A B$ 的垂线交 $A D$ 于 $P$, 交 $A B$ 于 $M$, 交 $\\odot O$ 于 $G$, 连接 $G E$.\n\n(1) 求证: $B C$ 是 $\\odot O$ 的切线;\n\n(2) 若 $\\tan \\angle G=\\frac{4}{3}, B E=4$, 求 $\\odot O$ 的半径;\n\n(3) 在 (2) 的条件下, 求 $A P$ 的长.\n\n", "options": [], "subject": "画法几何学", "analysis": "(1)证明见解析;(2) 6 ; (3) $\\frac{54}{25} \\sqrt{5}$\n\n【解析】 (1) 证明:连结 $O D . \\because D E \\perp A D, \\therefore A E$ 是 $\\odot O$ 的直径, 即 $O$ 在 $A E$ 上.\n\n$\\because A D$ 是角平分线, $\\therefore \\angle 1=\\angle 2 . \\quad \\because O A=O D, \\quad \\therefore \\angle 2=\\angle 3 . \\quad \\therefore \\angle 1=\\angle 3 . \\quad \\therefore O D / / A C$.\n\n$\\because \\angle C=90^{\\circ}, \\therefore O D \\perp B C . \\quad \\therefore B C$ 是 $\\odot O$ 的切线. (2) 解: $\\because O D / / A C, \\therefore \\angle 4=\\angle E A F$.\n\n$\\because \\angle G=\\angle E A F, \\quad \\therefore \\angle 4=\\angle G . \\quad \\therefore \\tan \\angle 4=\\tan \\angle G=\\frac{4}{3}$.\n\n设 $B D=4 k$, 则 $O D=O E=3 k$. 在 Rt $\\triangle O B D$ 中, 由勾股定理得 $(3 k)^{2}+(4 k)^{2}=(3 k+4)^{2}$,解得, $k_{1}=2, k_{2}=-\\frac{1}{2}$ (舍) , (注:也可由 $O B=5 k=3 k+4$ 得 $k=2$ ),\n\n$\\therefore 3 k=6$, 即 $\\odot O$ 的半径为 6 .\n\n\n\n(3) 解:连结 $A G$, 则 $\\angle A G E=90^{\\circ}, \\angle E G M=\\angle 5$.\n\n$\\therefore \\tan \\angle 5=\\tan \\angle E G M=\\frac{4}{3}$, 即 $\\frac{G M}{A M}=\\frac{E M}{G M}=\\frac{4}{3}, \\frac{A M}{G M}=\\frac{G M}{E M}=\\frac{3}{4}$,\n\n$\\therefore \\frac{A M}{E M}=\\frac{A M}{G M} \\cdot \\frac{G M}{E M}=\\frac{3}{4} \\times \\frac{3}{4}=\\frac{9}{16}, \\quad \\therefore A M=\\frac{9}{25} A E=\\frac{9}{25} \\times 12=\\frac{108}{25}$.\n\n$\\because O D / / A C, \\therefore \\frac{O D}{A C}=\\frac{O B}{A B}, \\frac{C D}{A O}=\\frac{D B}{O B}$, 即 $\\frac{6}{A C}=\\frac{5}{8}, \\frac{C D}{6}=\\frac{8}{10}$.\n\n$\\therefore A C=\\frac{48}{5}, C D=\\frac{24}{5} . \\quad \\because \\angle 1=\\angle 2, \\angle A C D=\\angle A M P=90^{\\circ}, \\therefore \\triangle A C D \\sim \\triangle A M P$.\n\n$\\therefore \\frac{P M}{A M}=\\frac{C D}{A C}=\\frac{1}{2}, \\quad \\therefore P M=\\frac{1}{2} A M=\\frac{54}{25} . \\quad \\therefore A P=\\sqrt{P M^{2}+A M^{2}}=\\frac{54}{25} \\sqrt{5}$."} {"id": "3936", "image": ["5547.jpg", "5548.jpg", "5548.jpg"], "answer": "280.【解析】过 $\\mathrm{A}$ 作 $\\mathrm{AE} \\perp \\mathrm{BC}$, 交 $\\mathrm{CB}$ 的延长线于点 $\\mathrm{E}$, 在 Rt $\\triangle \\mathrm{ACD}$ 中, $\\because \\angle \\mathrm{CAD}=30^{\\circ}, \\mathrm{AD}=420$米, $\\therefore \\mathrm{CD}=\\mathrm{AD} \\cdot \\tan 30^{\\circ}=420 \\times \\frac{\\sqrt{3}}{3}=140 \\sqrt{3}$ (米), $\\therefore \\mathrm{AE}=\\mathrm{CD}=140 \\sqrt{3} 140 \\sqrt{3}$ 米. 在 Rt $\\triangle \\mathrm{ABE}$ 中,\n\n$\\because \\angle \\mathrm{BAE}=30^{\\circ}, \\mathrm{AE}=140 \\sqrt{3}$ 米, $\\therefore \\mathrm{BE}=\\mathrm{AE} \\cdot \\tan 30^{\\circ}=140 \\sqrt{3} \\times \\frac{\\sqrt{3}}{3}=140$ (米), $\\therefore \\mathrm{BC}=\\mathrm{AD}-\\mathrm{BE}=420-140=280$ (米).\n\n答: 这栋楼的高度为 280 米.\n\n", "solution": "null", "level": "九年级", "question": "热气球的探测器显示, 从热气球底部 $A$ 处看一柇高楼顶部的俯角为 $30^{\\circ}$, 看这标楼底部的俯角为 $60^{\\circ}$, 热气球 $A$ 处于地面距离为 420 米, 求这标楼的高度.\n\n", "options": [], "subject": "画法几何学", "analysis": "280.【解析】过 $\\mathrm{A}$ 作 $\\mathrm{AE} \\perp \\mathrm{BC}$, 交 $\\mathrm{CB}$ 的延长线于点 $\\mathrm{E}$, 在 Rt $\\triangle \\mathrm{ACD}$ 中, $\\because \\angle \\mathrm{CAD}=30^{\\circ}, \\mathrm{AD}=420$米, $\\therefore \\mathrm{CD}=\\mathrm{AD} \\cdot \\tan 30^{\\circ}=420 \\times \\frac{\\sqrt{3}}{3}=140 \\sqrt{3}$ (米), $\\therefore \\mathrm{AE}=\\mathrm{CD}=140 \\sqrt{3} 140 \\sqrt{3}$ 米. 在 Rt $\\triangle \\mathrm{ABE}$ 中,\n\n$\\because \\angle \\mathrm{BAE}=30^{\\circ}, \\mathrm{AE}=140 \\sqrt{3}$ 米, $\\therefore \\mathrm{BE}=\\mathrm{AE} \\cdot \\tan 30^{\\circ}=140 \\sqrt{3} \\times \\frac{\\sqrt{3}}{3}=140$ (米), $\\therefore \\mathrm{BC}=\\mathrm{AD}-\\mathrm{BE}=420-140=280$ (米).\n\n答: 这栋楼的高度为 280 米.\n\n"} {"id": "3310", "image": ["4297.jpg", "4298.jpg", "4299.jpg", "4300.jpg", "4298.jpg", "4299.jpg", "4300.jpg"], "answer": "解:该图形的三个视图为:\n\n\n\n从正面看\n\n\n\n从左面看\n\n\n\n从.上面看\n\n【解析】根据三个方向看到的图形画出即可解答此题.", "solution": "null", "level": "九年级", "question": "(本小题 8.0 分)\n\n从正面、左面、上面三个方向看该立体图形,分别画出看到的平面图形.\n\n\n\n从正面看", "options": [], "subject": "画法几何学", "analysis": "解:该图形的三个视图为:\n\n\n\n从正面看\n\n\n\n从左面看\n\n\n\n从.上面看\n\n【解析】根据三个方向看到的图形画出即可解答此题."} {"id": "3333", "image": ["4345.jpg"], "answer": "解: 由题意可得: $\\angle A B C=\\angle E D C=\\angle G F H=90^{\\circ}$,\n\n$\\angle A C B=\\angle E C D, \\angle A F B=\\angle G H F$,\n\n故 $\\triangle A B C \\sim \\triangle E D C, \\triangle A B F \\sim \\triangle G F H$,\n\n则 $\\frac{A B}{E D}=\\frac{B C}{D C}, \\frac{A B}{G F}=\\frac{B F}{F H}$,\n\n即 $A B: 1.5=B C: 2, A B: 1.65=(B C+18): 2.5$,\n\n解得: $A B=99$,\n\n答: “望月阁” 的高 $A B$ 的长度为 $99 \\mathrm{~m}$.\n\n【解析】此题主要考查了相似三角形的判定与性质, 正确利用已知得出相似三角形是解题关键. 根据镜面反射原理结合相似三角形的判定方法得出 $\\triangle A B C \\sim \\triangle E D C, \\triangle A B F \\sim \\triangle G F H$, 进而利用相似三角形的性质得出 $A B$ 的长.", "solution": "null", "level": "九年级", "question": "(本小题 8.0 分)\n\n为了测量 “望月阁” 的高度, 方法如下: 如图, 小芳在小亮和 “望月阁” 之间的直线 $B M$ 上平放一平面镜, 在镜面上做了一个标记, 这个标记在直线 $B M$ 上的对应位置为点 $C$, 镜子不动,小亮看着镜面上的标记,他来回走动,走到点 $D$ 时,看到 “望月阁” 顶端点 $A$ 在镜面中的像与镜面上的标记重合, 这时, 测得小亮眼睛与地面的高度 $E D=1.5$ 米, $C D=2$ 米, 然后, 在阳光下, 他们用测影长的方法进行了第二次测量, 方法如下: 如图, 小亮从 $D$ 点沿 $D M$ 方向走了 16米, 到达 “望月阁” 影子的末端 $F$ 点处, 此时, 测得小亮身高 $F G$ 的影长 $F H=2.5$ 米, $F G=$ 1.65米. 如图, 已知 $A B \\perp B M, E D \\perp B M, G F \\perp B M$, 其中, 测量时所使用的平面镜的厚度忽略不计,请你根据题中提供的相关信息,求出 “望月阁” 的高 $A B$ 的长度.\n\n", "options": [], "subject": "画法几何学", "analysis": "解: 由题意可得: $\\angle A B C=\\angle E D C=\\angle G F H=90^{\\circ}$,\n\n$\\angle A C B=\\angle E C D, \\angle A F B=\\angle G H F$,\n\n故 $\\triangle A B C \\sim \\triangle E D C, \\triangle A B F \\sim \\triangle G F H$,\n\n则 $\\frac{A B}{E D}=\\frac{B C}{D C}, \\frac{A B}{G F}=\\frac{B F}{F H}$,\n\n即 $A B: 1.5=B C: 2, A B: 1.65=(B C+18): 2.5$,\n\n解得: $A B=99$,\n\n答: “望月阁” 的高 $A B$ 的长度为 $99 \\mathrm{~m}$.\n\n【解析】此题主要考查了相似三角形的判定与性质, 正确利用已知得出相似三角形是解题关键. 根据镜面反射原理结合相似三角形的判定方法得出 $\\triangle A B C \\sim \\triangle E D C, \\triangle A B F \\sim \\triangle G F H$, 进而利用相似三角形的性质得出 $A B$ 的长."} {"id": "3335", "image": ["4352.jpg", "4353.jpg", "4354.jpg", "4355.jpg", "4356.jpg", "4357.jpg", "4358.jpg", "4359.jpg", "4355.jpg", "4356.jpg", "4357.jpg", "4358.jpg", "4359.jpg"], "answer": "(1)8\n\n(2)如下图所示:\n\n\n\n正面\n\n\n\n左视图\n\n\n\n俯视图\n\n(3) 813\n【解析】解: (1)根据该组合体知最下面一层有 5 个正方体, 第 2 层有 2 个正方体、最上面一层有 1 个正方体,\n\n据此知共有 8 个正方体,\n\n故答案为: 8 ;\n\n(2)见答案\n\n(3)这样的几何体所需正方体最少分布情况如下图所示:\n\n\n\n佇枧图\n\n共需要 8 个正方体;\n\n这样的几何体所需正方体最多分布情况如下图所示:\n\n\n\n俯视图\n\n共需要正方体 13 个,\n\n故答案为: 813 .\n\n(1)最下面一层有 5 个正方体,第 2 层有 2 个正方体、最上面一层有 1 个正方体, 据此可得;\n\n(2)根据左视图和俯视图的定义作图即可得;\n\n(3)由题意知最少情况是: 后面一排某一列有 3 个正方体、其余位置有 1 个正方体,前面一排某一列有 2 个正方体、另一列有 1 个正方体; 最多的情况是:后面一排 3 列都有 3 个正方体,前面一排2列都有 2 个正方体, 据此可得.\n\n此题主要考查了由实物画三视图, 以及利用主视图和俯视图判断几何体的形状, 主要培养同学们的空间想象能力, 想象不出来可以亲手实验.", "solution": "null", "level": "九年级", "question": "(本小题 8.0 分)\n\n根据要求完成下列题目:\n\n(1) 如图中有块小正方体;\n\n(2)请在下面方格纸中分别画出它的左视图和俯视图(画出的图都用铅笔涂上阴影);\n\n\n\n正面\n\n\n\n左视图\n\n\n\n俯视图\n\n(3)用小正方形体搭一个几何体, 使得它的俯视图和左视图与你在上图方格中所画的图一致,则这样的几何体最少要 \\$ \\qquad \\$个小正方体,最多要 \\$ \\qquad \\$个小正方形.", "options": [], "subject": "画法几何学", "analysis": "(1)8\n\n(2)如下图所示:\n\n\n\n正面\n\n\n\n左视图\n\n\n\n俯视图\n\n(3) 813\n【解析】解: (1)根据该组合体知最下面一层有 5 个正方体, 第 2 层有 2 个正方体、最上面一层有 1 个正方体,\n\n据此知共有 8 个正方体,\n\n故答案为: 8 ;\n\n(2)见答案\n\n(3)这样的几何体所需正方体最少分布情况如下图所示:\n\n\n\n佇枧图\n\n共需要 8 个正方体;\n\n这样的几何体所需正方体最多分布情况如下图所示:\n\n\n\n俯视图\n\n共需要正方体 13 个,\n\n故答案为: 813 .\n\n(1)最下面一层有 5 个正方体,第 2 层有 2 个正方体、最上面一层有 1 个正方体, 据此可得;\n\n(2)根据左视图和俯视图的定义作图即可得;\n\n(3)由题意知最少情况是: 后面一排某一列有 3 个正方体、其余位置有 1 个正方体,前面一排某一列有 2 个正方体、另一列有 1 个正方体; 最多的情况是:后面一排 3 列都有 3 个正方体,前面一排2列都有 2 个正方体, 据此可得.\n\n此题主要考查了由实物画三视图, 以及利用主视图和俯视图判断几何体的形状, 主要培养同学们的空间想象能力, 想象不出来可以亲手实验."} {"id": "3337", "image": ["4367.jpg", "4368.jpg", "4368.jpg"], "answer": "解: 如图所示:\n\n\n\n【解析】从正面看得到的图形是上面中间有坚线的长方形下面长方形, 从左面看得到的图形是上面小长方形下面大长方形, 从上面看得到的图形是中间有内接三角形的圆形.\n\n考查了作图-三视图, 主视图、左视图、俯视图是分别从物体正面、左面和上面看, 所得到的图形; 在画图时一定要将物体的边缘、棱、顶点都体现出来, 看得见的轮廓线都画成实线.", "solution": "null", "level": "九年级", "question": "(本小题 8.0 分)\n\n画出图中几何体(上半部为正三棱柱, 下半部为圆柱)的三视图.\n\n", "options": [], "subject": "画法几何学", "analysis": "解: 如图所示:\n\n\n\n【解析】从正面看得到的图形是上面中间有坚线的长方形下面长方形, 从左面看得到的图形是上面小长方形下面大长方形, 从上面看得到的图形是中间有内接三角形的圆形.\n\n考查了作图-三视图, 主视图、左视图、俯视图是分别从物体正面、左面和上面看, 所得到的图形; 在画图时一定要将物体的边缘、棱、顶点都体现出来, 看得见的轮廓线都画成实线."} {"id": "3339", "image": ["4373.jpg", "4374.jpg", "4375.jpg", "4374.jpg", "4375.jpg"], "answer": "解: 如图所示:\n\n\n\n从正面看\n\n\n\n从左面看\n\n【解析】从正面看有 3 列, 小正方形个数分别为: $3,1,1$, 从左面看有 3 列小正方形个数分别为:\n\n$2,3,1$.\n此题主要考查了作三视图, 正确把握观察角度是解题关键.", "solution": "null", "level": "九年级", "question": "( 本小题 8.0 分)\n\n画图题\n\n如图, 这是一个由小立方块搭成的几何体的从上面看的形状图, 小正方形中的数字表示该位置的小立方块的个数. 请你画出从正面看和从左面看的形状图.\n\n", "options": [], "subject": "画法几何学", "analysis": "解: 如图所示:\n\n\n\n从正面看\n\n\n\n从左面看\n\n【解析】从正面看有 3 列, 小正方形个数分别为: $3,1,1$, 从左面看有 3 列小正方形个数分别为:\n\n$2,3,1$.\n此题主要考查了作三视图, 正确把握观察角度是解题关键."} {"id": "3340", "image": ["4376.jpg", "4377.jpg", "4378.jpg", "4379.jpg", "4380.jpg", "4381.jpg", "4381.jpg"], "answer": " $D$\n\n【解析】解:(1)正方体的展开图 $1-4-1$ 型, 只有 $D$ 不是这种情况, 所以 $D$ 不能折成长方形,故答案为: $D$;\n\n$(2)$\n\n\n\n图1\n\n(3)设这三个面的面积分别为 $5 x \\mathrm{~cm}^{2} 、 7 \\mathrm{xcm}^{2} 、 2 \\mathrm{cc}^{2}$,\n\n$7 x-2 x=60$\n\n$x=12$\n\n$2 \\times(5 \\times 12+7 \\times 12+2 \\times 12)=336\\left(\\mathrm{~cm}^{2}\\right)$.\n\n答: 这个长方体的表面积是336平方厘米.\n\n(1)根据正方体的展开图分析;\n\n(2)画长方体, 在原来答题的地方点一下鼠标, 现有截图复制保存图片, 去菁优编辑器里按插入图片,完成后点击画板下面的完成图片;\n\n(3)设三个面的面积分别为 $5 x \\mathrm{~cm}^{2} 、 7 x \\mathrm{~cm}^{2} 、 2 x \\mathrm{~cm}^{2}$ 根据等量关系列方程解答.\n\n此题考查正方体得出展开图和长方体的表面积, 重点是理解正方体展开图的种类, 求表面积的方法和画图的方法.", "solution": "null", "level": "九年级", "question": "(本小题8.0分)\n\n(1)下面四个图分别由六个相同的正方形拼接而成,其中不能折成正方体(从 $A 、 B 、 C 、 D$ 选择 ) 的是\n\n\n\nA\n\n\n\nB\n\n\n\n$\\mathrm{C}$\n\n\n\nD\n\n\n\n图1\n\n(2)用斜二测画法补画图1的图形, 使之成为长方体的直观图(虚线表示被遮住的线段; 只要在已有图形基础上画出长方体, 不必写画法步骤).\n\n(3)在这一长方体中, 从同一个顶点出发的三个面的面积之比是 $5: 7: 2$, 其中最大的比最小的面积大 $60 \\mathrm{~cm}^{2}$, 求这个长方体的表面积.", "options": [], "subject": "画法几何学", "analysis": " $D$\n\n【解析】解:(1)正方体的展开图 $1-4-1$ 型, 只有 $D$ 不是这种情况, 所以 $D$ 不能折成长方形,故答案为: $D$;\n\n$(2)$\n\n\n\n图1\n\n(3)设这三个面的面积分别为 $5 x \\mathrm{~cm}^{2} 、 7 \\mathrm{xcm}^{2} 、 2 \\mathrm{cc}^{2}$,\n\n$7 x-2 x=60$\n\n$x=12$\n\n$2 \\times(5 \\times 12+7 \\times 12+2 \\times 12)=336\\left(\\mathrm{~cm}^{2}\\right)$.\n\n答: 这个长方体的表面积是336平方厘米.\n\n(1)根据正方体的展开图分析;\n\n(2)画长方体, 在原来答题的地方点一下鼠标, 现有截图复制保存图片, 去菁优编辑器里按插入图片,完成后点击画板下面的完成图片;\n\n(3)设三个面的面积分别为 $5 x \\mathrm{~cm}^{2} 、 7 x \\mathrm{~cm}^{2} 、 2 x \\mathrm{~cm}^{2}$ 根据等量关系列方程解答.\n\n此题考查正方体得出展开图和长方体的表面积, 重点是理解正方体展开图的种类, 求表面积的方法和画图的方法."} {"id": "3585", "image": ["4884.jpg", "4885.jpg", "4885.jpg"], "answer": "解: 在 $R t \\triangle C D N$ 中,\n\n$\\because \\tan 30^{\\circ}=\\frac{C D}{D N}$,\n\n$\\therefore C D=\\tan 30^{\\circ} \\cdot D N=5 \\sqrt{3}$,\n\n$\\because \\angle C B D=\\angle E M B=37^{\\circ}$,\n\n$\\therefore B D=C D \\div \\tan 37^{\\circ}=\\frac{20}{3} \\sqrt{3}$,\n\n\n\n$$\n\\therefore B N=D N+B D=15+\\frac{20}{3} \\sqrt{3}\n$$\n\n在Rt $\\triangle A B N$ 中, $\\tan 30^{\\circ}=\\frac{A B}{B N}$,\n\n$\\therefore A B=\\tan 30^{\\circ} \\cdot B N \\approx 15.3$,\n在Rt $\\triangle M N B$ 中, $M N=B N \\cdot \\tan 37^{\\circ}=0.75\\left(15+\\frac{20}{3} \\sqrt{3}\\right) \\approx 19.9$\n\n$\\therefore$ 树高 $A B$ 是15.3米, 楼房 $M N$ 的高度是19.9米.", "solution": "null", "level": "九年级", "question": "(10 分) 如图, 在楼房 $M N$ 前有两棵树与楼房在同一直线上, 且垂直于地面, 为了测量树 $A B 、$ $C D$ 的高度, 小明爬到楼房顶部 $M$ 处, 光线恰好可以经过树 $C D$ 的顶站 $C$ 点到达树 $A B$ 的底部 $B$点, 俯角为 $37^{\\circ}$, 此时小亮测得太阳光线恰好经过树 $C D$ 的顶部 $C$ 点到达楼房的底部 $N$ 点, 与地面的夹角为 $30^{\\circ}$, 树 $C D$ 的影长 $D N$ 为 15 米, 请求出树 $A B$ 和楼房 $M N$ 的高度. $\\left(\\sqrt{3} \\approx 1.73, \\sin 37^{\\circ} \\approx\\right.$ $0.60, \\cos 37^{\\circ} \\approx 0.800, \\tan 37^{\\circ} \\approx 0.75$, 结果精确到 $\\left.0.1 \\mathrm{~m}\\right)$\n\n", "options": [], "subject": "度量几何学", "analysis": "解: 在 $R t \\triangle C D N$ 中,\n\n$\\because \\tan 30^{\\circ}=\\frac{C D}{D N}$,\n\n$\\therefore C D=\\tan 30^{\\circ} \\cdot D N=5 \\sqrt{3}$,\n\n$\\because \\angle C B D=\\angle E M B=37^{\\circ}$,\n\n$\\therefore B D=C D \\div \\tan 37^{\\circ}=\\frac{20}{3} \\sqrt{3}$,\n\n\n\n$$\n\\therefore B N=D N+B D=15+\\frac{20}{3} \\sqrt{3}\n$$\n\n在Rt $\\triangle A B N$ 中, $\\tan 30^{\\circ}=\\frac{A B}{B N}$,\n\n$\\therefore A B=\\tan 30^{\\circ} \\cdot B N \\approx 15.3$,\n在Rt $\\triangle M N B$ 中, $M N=B N \\cdot \\tan 37^{\\circ}=0.75\\left(15+\\frac{20}{3} \\sqrt{3}\\right) \\approx 19.9$\n\n$\\therefore$ 树高 $A B$ 是15.3米, 楼房 $M N$ 的高度是19.9米."} {"id": "3629", "image": ["4959.jpg", "4960.jpg", "4960.jpg"], "answer": "解: (1)作直径 $B D$, 连接 $C D$,\n\n由圆周角定理得, $\\angle D=\\angle A, \\angle B C D=90^{\\circ}$,\n\n$\\therefore B D=\\frac{B C}{\\sin D}=\\frac{25}{3}$, 即 $\\triangle A B C$ 的外接圆的直径为 $\\frac{25}{3}$;\n\n(2) $\\because A B=B C$,\n\n\n\n图1\n\n$\\therefore B E \\perp A C$,\n\n$\\therefore B E=A B \\times \\sin A=3$,\n\n$\\therefore A E=\\sqrt{A B^{2}-B E^{2}}=4$,\n\n$\\therefore A C=8$,\n\n设 $\\triangle A B C$ 内切圆的半径为 $r$,\n\n则 $\\frac{1}{2} \\times 5 \\times r+\\frac{1}{2} \\times 5 \\times r+\\frac{1}{2} \\times 8 \\times r=\\frac{1}{2} \\times 8 \\times 3$,\n\n解得, $r=\\frac{4}{3}$,\n\n则 $\\triangle A B C$ 内切圆的半径为 $\\frac{4}{3}$.", "solution": "null", "level": "九年级", "question": "如图, $\\triangle A B C$ 中, $B C=5, \\sin A=\\frac{3}{5}$\n\n(1)求 $\\triangle A B C$ 的外接圆的直径;\n\n(2)如果 $A B=B C$, 求 $\\triangle A B C$ 内切圆的半径.\n\n", "options": [], "subject": "度量几何学", "analysis": "解: (1)作直径 $B D$, 连接 $C D$,\n\n由圆周角定理得, $\\angle D=\\angle A, \\angle B C D=90^{\\circ}$,\n\n$\\therefore B D=\\frac{B C}{\\sin D}=\\frac{25}{3}$, 即 $\\triangle A B C$ 的外接圆的直径为 $\\frac{25}{3}$;\n\n(2) $\\because A B=B C$,\n\n\n\n图1\n\n$\\therefore B E \\perp A C$,\n\n$\\therefore B E=A B \\times \\sin A=3$,\n\n$\\therefore A E=\\sqrt{A B^{2}-B E^{2}}=4$,\n\n$\\therefore A C=8$,\n\n设 $\\triangle A B C$ 内切圆的半径为 $r$,\n\n则 $\\frac{1}{2} \\times 5 \\times r+\\frac{1}{2} \\times 5 \\times r+\\frac{1}{2} \\times 8 \\times r=\\frac{1}{2} \\times 8 \\times 3$,\n\n解得, $r=\\frac{4}{3}$,\n\n则 $\\triangle A B C$ 内切圆的半径为 $\\frac{4}{3}$."} {"id": "3631", "image": ["4961.jpg", "4962.jpg", "4963.jpg", "4962.jpg", "4963.jpg"], "answer": "解: (1)过点 $D$ 作 $D F \\perp B C$ 于点 $F$,\n\n\n\n图1\n\n\n\n图2\n\n$\\because A B$ 为 $\\odot O$ 的直径, $\\angle A=\\angle B=90^{\\circ}$,\n\n$\\therefore$ 四边形 $A B F D$ 是矩形, $A D$ 与 $B C$ 是 $\\odot O$ 的切线,\n\n$\\therefore D F=A B=2 \\sqrt{5}, B F=A D=2$,\n\n$\\because D E$ 与 $\\odot O$ 相切,\n\n$\\therefore D E=A D=2, C E=B C$,\n\n设 $B C=x$ ,\n\n则 $C F=B C-B F=x-2, D C=D E+C E=2+x$,\n\n在Rt $\\triangle D C F$ 中, $D C^{2}=C F^{2}+D F^{2}$,\n即 $(2+x)^{2}=(x-2)^{2}+(2 \\sqrt{5})^{2} ,$\n\n解得: $x=2.5$,\n\n即 $B C=2.5$;\n\n(2) $\\because A B$ 为 $\\odot O$ 的直径, $\\angle A=\\angle B=90^{\\circ}$,\n\n$\\therefore A D / / B C$,\n\n$\\therefore \\triangle A D E \\sim \\triangle G C E$,\n\n$\\therefore A D: C G=D E: C E, A E: E G=A D: C G$,\n\n$\\because A D=D E=2$,\n\n$\\therefore C G=C E=B C=2.5$,\n\n$\\therefore B G=B C+C G=5$,\n\n$\\therefore A E: E G=4: 5$,\n\n在Rt $\\triangle A B G$ 中, $A G=\\sqrt{A B^{2}+B G^{2}}=3 \\sqrt{5}$,\n\n$\\therefore E G=\\frac{5}{9} A G=\\frac{5 \\sqrt{5}}{3}$.", "solution": "null", "level": "九年级", "question": "如图, 已知: $A B$ 为 $\\odot O$ 的直径, $\\angle B A D=\\angle B=90^{\\circ}, D C$ 与 $\\odot O$ 相切于点 $E, \\odot O$ 的直径为 $2 \\sqrt{5}$, $A D=2$.\n\n(1) 求 $B C$ 的长;\n\n(2)连接 $A E$ 并延长交 $B C$ 的延长线于点 $G$, 求 $E G$ 的长.\n\n", "options": [], "subject": "度量几何学", "analysis": "解: (1)过点 $D$ 作 $D F \\perp B C$ 于点 $F$,\n\n\n\n图1\n\n\n\n图2\n\n$\\because A B$ 为 $\\odot O$ 的直径, $\\angle A=\\angle B=90^{\\circ}$,\n\n$\\therefore$ 四边形 $A B F D$ 是矩形, $A D$ 与 $B C$ 是 $\\odot O$ 的切线,\n\n$\\therefore D F=A B=2 \\sqrt{5}, B F=A D=2$,\n\n$\\because D E$ 与 $\\odot O$ 相切,\n\n$\\therefore D E=A D=2, C E=B C$,\n\n设 $B C=x$ ,\n\n则 $C F=B C-B F=x-2, D C=D E+C E=2+x$,\n\n在Rt $\\triangle D C F$ 中, $D C^{2}=C F^{2}+D F^{2}$,\n即 $(2+x)^{2}=(x-2)^{2}+(2 \\sqrt{5})^{2} ,$\n\n解得: $x=2.5$,\n\n即 $B C=2.5$;\n\n(2) $\\because A B$ 为 $\\odot O$ 的直径, $\\angle A=\\angle B=90^{\\circ}$,\n\n$\\therefore A D / / B C$,\n\n$\\therefore \\triangle A D E \\sim \\triangle G C E$,\n\n$\\therefore A D: C G=D E: C E, A E: E G=A D: C G$,\n\n$\\because A D=D E=2$,\n\n$\\therefore C G=C E=B C=2.5$,\n\n$\\therefore B G=B C+C G=5$,\n\n$\\therefore A E: E G=4: 5$,\n\n在Rt $\\triangle A B G$ 中, $A G=\\sqrt{A B^{2}+B G^{2}}=3 \\sqrt{5}$,\n\n$\\therefore E G=\\frac{5}{9} A G=\\frac{5 \\sqrt{5}}{3}$."} {"id": "3652", "image": ["5000.jpg"], "answer": "解: $\\because C D / / A B$,\n\n$\\therefore \\triangle E A B \\sim \\triangle E C D$,\n\n$\\therefore \\frac{C D}{A B}=\\frac{D E}{B E}$,\n\n即 $\\frac{2}{A B}=\\frac{3}{3+B D}(1)$\n\n$\\because F G / / A B$,\n$\\therefore \\triangle H F G \\sim \\triangle H A B$,\n\n$\\therefore \\frac{F G}{A B}=\\frac{H G}{H B}$\n\n即 $\\frac{2}{A B}=\\frac{5}{B D+5+5}(2)$,\n\n由(1) (2)得 $\\frac{3}{3+B D}=\\frac{5}{B D+5+5}$,\n\n解得: $B D=7.5 m$,\n\n$\\therefore \\frac{2}{A B}=\\frac{3}{7.5}$,\n\n解得: $A B=7 \\mathrm{~m}$.\n\n答: 路灯杆 $A B$ 的高度为 $7 m$.", "solution": "null", "level": "九年级", "question": "如图所示, 花丛中有一路灯杆 $A B$, 在灯光下, 大华在点 $D$ 处的影长 $D E=3 m$, 沿 $B D$ 方向行走到达点 $G, D G=5 \\mathrm{~m}$, 这时大华的影长 $G H=5 \\mathrm{~m}$. 如果大华的身高为 $2 \\mathrm{~m}$, 求路灯杆 $A B$ 的高度.\n\n", "options": [], "subject": "度量几何学", "analysis": "解: $\\because C D / / A B$,\n\n$\\therefore \\triangle E A B \\sim \\triangle E C D$,\n\n$\\therefore \\frac{C D}{A B}=\\frac{D E}{B E}$,\n\n即 $\\frac{2}{A B}=\\frac{3}{3+B D}(1)$\n\n$\\because F G / / A B$,\n$\\therefore \\triangle H F G \\sim \\triangle H A B$,\n\n$\\therefore \\frac{F G}{A B}=\\frac{H G}{H B}$\n\n即 $\\frac{2}{A B}=\\frac{5}{B D+5+5}(2)$,\n\n由(1) (2)得 $\\frac{3}{3+B D}=\\frac{5}{B D+5+5}$,\n\n解得: $B D=7.5 m$,\n\n$\\therefore \\frac{2}{A B}=\\frac{3}{7.5}$,\n\n解得: $A B=7 \\mathrm{~m}$.\n\n答: 路灯杆 $A B$ 的高度为 $7 m$."} {"id": "3657", "image": ["5016.jpg", "5017.jpg", "5017.jpg"], "answer": " 解: (1)如图, 延长 $O B$ 交 $D C$ 于 $E$, 作 $E F \\perp A B$, 交 $A B$ 于 $F$,\n\n\n在Rt $\\triangle B E F$ 中,\n\n$\\because E F=A C=30 m, \\angle F E B=30^{\\circ}$,\n\n$\\therefore B E=2 B F$.\n\n设 $B F=x$, 则 $B E=2 x$.\n\n根据勾股定理知 $B E^{2}=B F^{2}+E F^{2}$,\n\n$\\therefore(2 x)^{2}=x^{2}+30^{2}$,\n\n$\\therefore x= \\pm 10 \\sqrt{3}$ (负值舍去),\n\n$\\therefore x \\approx 17.3(m)$.\n\n因此, $E C=30-17.3 \\approx 12.7(m)$;\n\n(2)当甲幢楼的影子刚好落在点 $C$ 处时, $\\triangle A B C$ 为等腰三角形,\n\n因此, 当太阳光与水平线夹角为 $45^{\\circ}$ 时, 甲楼的影子刚好不落在乙楼的墙上.", "solution": "null", "level": "九年级", "question": "如图所示为住宅区内的两幢楼, 它们的高 $A B=C D=30 \\mathrm{~m}$, 两楼间的距离 $A C=30 \\mathrm{~m}$, 现需了解甲楼对乙楼的采光的影响情况.\n\n\n\n(1)当太阳光线与水平线的夹角为 $30^{\\circ}$ 时, 求甲楼的影子在乙楼上的高度. (结果精确到 $0.1 \\mathrm{~m}$, $\\sqrt{3} \\approx 1.73)$\n\n(2)若要甲楼的影子刚好不落在乙楼的墙上, 此时太阳光线与水平线的夹角为多少度?", "options": [], "subject": "度量几何学", "analysis": " 解: (1)如图, 延长 $O B$ 交 $D C$ 于 $E$, 作 $E F \\perp A B$, 交 $A B$ 于 $F$,\n\n\n在Rt $\\triangle B E F$ 中,\n\n$\\because E F=A C=30 m, \\angle F E B=30^{\\circ}$,\n\n$\\therefore B E=2 B F$.\n\n设 $B F=x$, 则 $B E=2 x$.\n\n根据勾股定理知 $B E^{2}=B F^{2}+E F^{2}$,\n\n$\\therefore(2 x)^{2}=x^{2}+30^{2}$,\n\n$\\therefore x= \\pm 10 \\sqrt{3}$ (负值舍去),\n\n$\\therefore x \\approx 17.3(m)$.\n\n因此, $E C=30-17.3 \\approx 12.7(m)$;\n\n(2)当甲幢楼的影子刚好落在点 $C$ 处时, $\\triangle A B C$ 为等腰三角形,\n\n因此, 当太阳光与水平线夹角为 $45^{\\circ}$ 时, 甲楼的影子刚好不落在乙楼的墙上."} {"id": "3679", "image": ["5056.jpg"], "answer": "解: $\\because C D / / A B$,\n\n$\\therefore \\triangle E A B \\sim \\triangle E C D$,\n\n$\\therefore \\frac{C D}{A B}=\\frac{D E}{B E}$, 即 $\\frac{2}{A B}=\\frac{3}{3+B D}(1)$,\n\n$\\because F G / / A B$,\n\n$\\therefore \\triangle H F G \\backsim \\triangle H A B$,\n\n$\\therefore \\frac{F G}{A B}=\\frac{H G}{H B}$, 即 $\\frac{2}{A B}=\\frac{5}{B D+5+5}$ (2),\n\n由(1)(2)得 $\\frac{3}{3+B D}=\\frac{5}{B D+5+5}$,\n\n解得 $B D=7.5$,\n\n$\\therefore \\frac{2}{A B}=\\frac{3}{7.5+3}$, 解得: $A B=7$.\n\n答: 路灯杆 $A B$ 的高度为 $7 \\mathrm{~m}$.", "solution": "null", "level": "九年级", "question": "如图, 花丛中有一路灯杆 $A B$, 在灯光下, 大华在 $D$ 点处的影长 $D E=3$ 米, 沿 $B D$ 方向行走到达 $G$ 点, $D G=5$ 米, 这时大华的影长 $G H=5$ 米. 如果大华的身高为 2 米, 求路灯杆 $A B$ 的高度.\n\n", "options": [], "subject": "度量几何学", "analysis": "解: $\\because C D / / A B$,\n\n$\\therefore \\triangle E A B \\sim \\triangle E C D$,\n\n$\\therefore \\frac{C D}{A B}=\\frac{D E}{B E}$, 即 $\\frac{2}{A B}=\\frac{3}{3+B D}(1)$,\n\n$\\because F G / / A B$,\n\n$\\therefore \\triangle H F G \\backsim \\triangle H A B$,\n\n$\\therefore \\frac{F G}{A B}=\\frac{H G}{H B}$, 即 $\\frac{2}{A B}=\\frac{5}{B D+5+5}$ (2),\n\n由(1)(2)得 $\\frac{3}{3+B D}=\\frac{5}{B D+5+5}$,\n\n解得 $B D=7.5$,\n\n$\\therefore \\frac{2}{A B}=\\frac{3}{7.5+3}$, 解得: $A B=7$.\n\n答: 路灯杆 $A B$ 的高度为 $7 \\mathrm{~m}$."} {"id": "3681", "image": ["5061.jpg", "5062.jpg", "5062.jpg"], "answer": "解: (1)当 $\\alpha=60^{\\circ}$ 时, 在 $R t \\triangle A B E$ 中,\n\n$\\because \\tan 60^{\\circ}=\\frac{A B}{A E}$\n\n$\\therefore A B=30 \\tan 60^{\\circ}=30 \\sqrt{3} \\approx 51.9$ 米,\n\n\n\n答:居民楼的高度约为 51.9 米.\n\n(2)当 $\\alpha=45^{\\circ}$ 时, 学生仍然晒到太阳. 理由如下:\n\n设点 $B$ 射下的光线与地面 $A D$ 的交点为 $F$, 与 $M C$ 的交点为 $H$,\n\n$\\because \\angle A F B=45^{\\circ}$,\n\n$\\therefore A F=A B=51.9$,\n\n$\\therefore C F=A F-A C=51.9-51.7=0.2$,\n\n$\\because \\angle C F H=45^{\\circ}$,\n\n$\\therefore C H=C F=0.2$ 米 $<0.3$ 米,\n\n$\\therefore$ 居民楼的影子落在台阶 $M C$ 这个侧面上,\n\n$\\therefore$ 在 $M N$ 这层上观看比赛的学生仍晒到太阳.", "solution": "null", "level": "九年级", "question": "如图所示, 台阶 $C D$ 为某校运动场观赛台, 台阶每层高 0.3 米, $A B$为运动场外的一幢坚直居民楼, 且 $A C=51.7$ 米, 设太阳光线与水平地面的夹角为 $\\alpha$, 当 $\\alpha=60^{\\circ}$ 时, 测得居民楼在地面上的影长\n\n$A E=30$ 米. (参考数据: $\\sqrt{3} \\approx 1.73$ )\n\n\n\n(1)求居民楼的高度约为多少米?\n\n(2)当 $\\alpha=45^{\\circ}$ 时, 请问在台阶的 $M N$ 这层上观看比赛的学生是否还晒到太阳?请说明理由.", "options": [], "subject": "度量几何学", "analysis": "解: (1)当 $\\alpha=60^{\\circ}$ 时, 在 $R t \\triangle A B E$ 中,\n\n$\\because \\tan 60^{\\circ}=\\frac{A B}{A E}$\n\n$\\therefore A B=30 \\tan 60^{\\circ}=30 \\sqrt{3} \\approx 51.9$ 米,\n\n\n\n答:居民楼的高度约为 51.9 米.\n\n(2)当 $\\alpha=45^{\\circ}$ 时, 学生仍然晒到太阳. 理由如下:\n\n设点 $B$ 射下的光线与地面 $A D$ 的交点为 $F$, 与 $M C$ 的交点为 $H$,\n\n$\\because \\angle A F B=45^{\\circ}$,\n\n$\\therefore A F=A B=51.9$,\n\n$\\therefore C F=A F-A C=51.9-51.7=0.2$,\n\n$\\because \\angle C F H=45^{\\circ}$,\n\n$\\therefore C H=C F=0.2$ 米 $<0.3$ 米,\n\n$\\therefore$ 居民楼的影子落在台阶 $M C$ 这个侧面上,\n\n$\\therefore$ 在 $M N$ 这层上观看比赛的学生仍晒到太阳."} {"id": "3839", "image": ["5338.jpg"], "answer": "43 米.\n\n【解析】由题意得: $\\angle A B D=\\angle C D E=90^{\\circ}, \\angle A D B=\\angle C E D, \\therefore \\triangle C D E \\sim \\triangle A B D, \\therefore \\frac{C D}{A B}=\\frac{D E}{B D}$.\n\n$\\because \\angle F=\\angle F, \\therefore \\triangle C D F \\sim \\triangle A B F, \\therefore \\frac{C D}{A B}=\\frac{D F}{B F}, \\therefore \\frac{D E}{B D}=\\frac{D F}{B F}$, 即 $\\frac{2.4}{B D}=\\frac{2.5}{B D+2.5}, \\therefore B D=60$,\n\n$\\therefore \\frac{1.72}{A B}=\\frac{2.4}{60}, \\therefore A B=43$. 答: 小雁塔的高度 $A B$ 是 43 米.", "solution": "null", "level": "九年级", "question": "如图是位于陕西省西安市荐福寺内的小雁塔, 是中国早期方形密檐式砖塔的典型作品, 并作为丝绸之路的一处重要遗址点, 被列入《世界遗产名录》。小铭、小希等几位同学想利用一些测量工具和所学的几何知识测量小雁塔的高度, 由于观测点与小雁塔底部间的距离不易测量, 因此经过研究需要进行两次测量, 于是在阳光下, 他们首先利用影长进行测量, 方法如下: 小铭在小雁塔的影子顶端 D 处坚直立一根木棒 $\\mathrm{CD}$, 并测得此时木棒的影长 $\\mathrm{DE}=2.4$ 米; 然后, 小希在 $\\mathrm{BD}$ 的延长线上找出一点 $\\mathrm{F}$,使得 A、C、F 三点在同一直线上, 并测得 $\\mathrm{DF}=2.5$ 米. 已知图中所有点均在同一平面内, 木棒高 $\\mathrm{CD}=1.72$米, $\\mathrm{AB} \\perp \\mathrm{BF}, \\mathrm{CD} \\perp \\mathrm{BF}$, 试根据以上测量数据, 求小雁塔的高度 $\\mathrm{AB}$.\n\n", "options": [], "subject": "度量几何学", "analysis": "43 米.\n\n【解析】由题意得: $\\angle A B D=\\angle C D E=90^{\\circ}, \\angle A D B=\\angle C E D, \\therefore \\triangle C D E \\sim \\triangle A B D, \\therefore \\frac{C D}{A B}=\\frac{D E}{B D}$.\n\n$\\because \\angle F=\\angle F, \\therefore \\triangle C D F \\sim \\triangle A B F, \\therefore \\frac{C D}{A B}=\\frac{D F}{B F}, \\therefore \\frac{D E}{B D}=\\frac{D F}{B F}$, 即 $\\frac{2.4}{B D}=\\frac{2.5}{B D+2.5}, \\therefore B D=60$,\n\n$\\therefore \\frac{1.72}{A B}=\\frac{2.4}{60}, \\therefore A B=43$. 答: 小雁塔的高度 $A B$ 是 43 米."} {"id": "3861", "image": ["5405.jpg", "5406.jpg"], "answer": "(1) 树 $A B$ 的高约为 $4 \\sqrt{3} \\mathrm{~m}$; (2) $8 \\sqrt{3} \\mathrm{~m}$.\n\n【解析】 (1) $\\mathrm{AB}=\\mathrm{ACtan} 30^{\\circ}=12 \\times \\frac{\\sqrt{3}}{3}=4 \\sqrt{3}$ (米). 答: 树高约为 $4 \\sqrt{3}$ 米.\n\n(2)如图 (2), $\\mathrm{B}_{1} \\mathrm{~N}=\\mathrm{AN}_{\\mathrm{N}}=\\mathrm{AB}_{1} \\sin 45^{\\circ}=4 \\sqrt{3} \\times \\frac{\\sqrt{2}}{2}=2 \\sqrt{6}$ (米).\n\n$\\mathrm{NC}_{1}=\\mathrm{NB}_{1} \\tan 60^{\\circ}=2 \\sqrt{6} \\times \\sqrt{3}=6 \\sqrt{2}$ (米) .\n\n$\\mathrm{AC}_{1}=\\mathrm{AN}+\\mathrm{NC}_{1}=2 \\sqrt{6}+6 \\sqrt{2}$.\n\n当树与地面成 $60^{\\circ}$ 角时影长最大 $\\mathrm{AC}_{2}$ (或树与光线垂直时影长最大或光线与半径为 $\\mathrm{AB}$ 的 $\\odot \\mathrm{A}$ 相切时影长最大)\n\n$\\mathrm{AC}_{2}=2 \\mathrm{AB}_{2}=8 \\sqrt{3} ;$\n\n(1) 在直角 $\\triangle \\mathrm{ABC}$ 中, 已知 $\\angle \\mathrm{ACB}=30^{\\circ}, \\mathrm{AC}=12$ 米. 利用三角函数即可求得 $\\mathrm{AB}$ 的长;\n\n(2) 在 $\\triangle \\mathrm{AB}_{1} \\mathrm{C}_{1}$ 中, 已知 $\\mathrm{AB}_{1}$ 的长, 即 $\\mathrm{AB}$ 的长, $\\angle \\mathrm{B}_{1} \\mathrm{AC}_{1}=45^{\\circ}, \\angle \\mathrm{B}_{1} \\mathrm{C}_{1} \\mathrm{~A}=30^{\\circ}$. 过 $\\mathrm{B}_{1}$ 作 $\\mathrm{AC}_{1}$ 的垂线, 在直角 $\\triangle A B_{1} N$ 中根据三角函数求得 $A N, B N$; 再在直角 $\\triangle B_{1} N C_{1}$ 中, 根据三角函数求得 $N C_{1}$ 的长, 再根据当树与地面成 $60^{\\circ}$ 角时影长最大, 根据三角函数即可求解.", "solution": "null", "level": "九年级", "question": "某数学兴趣小组, 利用树影测量树高, 如图 (1), 已测出树 $A B$ 的影长 $A C$ 为 12 米, 并测出此时太阳光线与地面成 $30^{\\circ}$ 夹角.\n\n(1) 求出树高 $A B$;\n\n(2)因水土流失, 此时树 $A B$ 沿太阳光线方向倒下, 在倾倒过程中, 树影长度发生了变化, 假设太阳光线与地面夹角保持不变. 求树的最大影长. (用图 (2) 解答)\n\n\n\n(1)\n\n\n\n(2)", "options": [], "subject": "度量几何学", "analysis": "(1) 树 $A B$ 的高约为 $4 \\sqrt{3} \\mathrm{~m}$; (2) $8 \\sqrt{3} \\mathrm{~m}$.\n\n【解析】 (1) $\\mathrm{AB}=\\mathrm{ACtan} 30^{\\circ}=12 \\times \\frac{\\sqrt{3}}{3}=4 \\sqrt{3}$ (米). 答: 树高约为 $4 \\sqrt{3}$ 米.\n\n(2)如图 (2), $\\mathrm{B}_{1} \\mathrm{~N}=\\mathrm{AN}_{\\mathrm{N}}=\\mathrm{AB}_{1} \\sin 45^{\\circ}=4 \\sqrt{3} \\times \\frac{\\sqrt{2}}{2}=2 \\sqrt{6}$ (米).\n\n$\\mathrm{NC}_{1}=\\mathrm{NB}_{1} \\tan 60^{\\circ}=2 \\sqrt{6} \\times \\sqrt{3}=6 \\sqrt{2}$ (米) .\n\n$\\mathrm{AC}_{1}=\\mathrm{AN}+\\mathrm{NC}_{1}=2 \\sqrt{6}+6 \\sqrt{2}$.\n\n当树与地面成 $60^{\\circ}$ 角时影长最大 $\\mathrm{AC}_{2}$ (或树与光线垂直时影长最大或光线与半径为 $\\mathrm{AB}$ 的 $\\odot \\mathrm{A}$ 相切时影长最大)\n\n$\\mathrm{AC}_{2}=2 \\mathrm{AB}_{2}=8 \\sqrt{3} ;$\n\n(1) 在直角 $\\triangle \\mathrm{ABC}$ 中, 已知 $\\angle \\mathrm{ACB}=30^{\\circ}, \\mathrm{AC}=12$ 米. 利用三角函数即可求得 $\\mathrm{AB}$ 的长;\n\n(2) 在 $\\triangle \\mathrm{AB}_{1} \\mathrm{C}_{1}$ 中, 已知 $\\mathrm{AB}_{1}$ 的长, 即 $\\mathrm{AB}$ 的长, $\\angle \\mathrm{B}_{1} \\mathrm{AC}_{1}=45^{\\circ}, \\angle \\mathrm{B}_{1} \\mathrm{C}_{1} \\mathrm{~A}=30^{\\circ}$. 过 $\\mathrm{B}_{1}$ 作 $\\mathrm{AC}_{1}$ 的垂线, 在直角 $\\triangle A B_{1} N$ 中根据三角函数求得 $A N, B N$; 再在直角 $\\triangle B_{1} N C_{1}$ 中, 根据三角函数求得 $N C_{1}$ 的长, 再根据当树与地面成 $60^{\\circ}$ 角时影长最大, 根据三角函数即可求解."} {"id": "3885", "image": ["5466.jpg", "5467.jpg", "5468.jpg", "5467.jpg", "5468.jpg"], "answer": "(1) 证明见解析; (2) $\\quad B C=\\frac{24}{5}$.\n\n【解析】 (1) 如图, 连接 $\\mathrm{BD}$\n\n$\\because \\quad \\mathrm{AD} \\perp \\mathrm{AB}, \\quad \\therefore \\quad \\mathrm{DB}$ 是 $\\odot \\mathrm{O}$ 的直径,\n\n$\\therefore \\quad \\angle \\mathrm{D}+\\angle \\mathrm{ABD}=90^{\\circ}$, 又 $\\because \\quad \\angle \\mathrm{D}=\\angle \\mathrm{C}, \\angle \\mathrm{ABF}=\\angle \\mathrm{C}$,\n\n$\\therefore \\quad \\angle \\mathrm{ABD}+\\angle \\mathrm{ABF}=90^{\\circ}, \\therefore \\quad \\mathrm{OB} \\perp \\mathrm{BF}, \\therefore \\quad \\mathrm{BF}$ 是 $\\odot \\mathrm{O}$ 的切线;\n\n\n\n(2) 如图, 连接 $O A$, 交 $B C$ 于点 $G$,\n\n$\\because \\quad \\mathrm{AC}=\\mathrm{AB}$,\n\n\n\n$\\therefore \\quad$ 弧 $\\mathrm{AC}=$ 弧 $\\mathrm{AB} \\therefore \\quad \\angle \\mathrm{D}=\\angle 2=\\angle \\mathrm{ABF}, \\mathrm{OA} \\perp \\mathrm{BC}, \\mathrm{BG}=\\mathrm{CG}$,\n\n$\\therefore \\quad \\cos \\angle \\mathrm{D}=\\cos \\angle 2=\\cos \\angle \\mathrm{ABF}=\\frac{4}{5}$, 在 $\\triangle \\mathrm{ABD}$ 中, $\\angle \\mathrm{DAB}=90^{\\circ}$,\n\n$\\therefore \\quad \\mathrm{BD}=\\frac{\\mathrm{AD}}{\\cos \\angle \\mathrm{D}}=5, \\therefore \\quad \\mathrm{AB}=\\sqrt{\\mathrm{BD}^{2}-\\mathrm{AD}^{2}}=3$,\n\n在 $\\triangle \\mathrm{ABG}$ 中, $\\angle \\mathrm{AGB}=90^{\\circ}, \\mathrm{AD}=4, \\therefore \\quad \\mathrm{BG}=\\mathrm{AB} \\times \\cos \\angle 2=\\frac{12}{5}$,\n\n$\\therefore \\quad \\mathrm{BC}=2 \\mathrm{BG}=\\frac{24}{5}$.", "solution": "null", "level": "九年级", "question": "如图, $\\triangle A B C$ 内接于 $\\odot O, A B=A C$, 过点 $A$ 作 $A D \\perp A B$ 交 $\\odot O$ 于点 $D$, 交 $B C$ 于点 $E$,点 $\\mathrm{F}$ 在 $\\mathrm{DA}$ 的延长线上, 且 $\\angle \\mathrm{ABF}=\\angle \\mathrm{C}$.\n\n(1) 求证: $\\mathrm{BF}$ 是 $\\odot \\mathrm{O}$ 的切线;\n\n(2) 若 $\\mathrm{AD}=4, \\cos \\angle \\mathrm{ABF}=\\frac{4}{5}$, 求 $\\mathrm{BC}$ 的长.\n\n", "options": [], "subject": "度量几何学", "analysis": "(1) 证明见解析; (2) $\\quad B C=\\frac{24}{5}$.\n\n【解析】 (1) 如图, 连接 $\\mathrm{BD}$\n\n$\\because \\quad \\mathrm{AD} \\perp \\mathrm{AB}, \\quad \\therefore \\quad \\mathrm{DB}$ 是 $\\odot \\mathrm{O}$ 的直径,\n\n$\\therefore \\quad \\angle \\mathrm{D}+\\angle \\mathrm{ABD}=90^{\\circ}$, 又 $\\because \\quad \\angle \\mathrm{D}=\\angle \\mathrm{C}, \\angle \\mathrm{ABF}=\\angle \\mathrm{C}$,\n\n$\\therefore \\quad \\angle \\mathrm{ABD}+\\angle \\mathrm{ABF}=90^{\\circ}, \\therefore \\quad \\mathrm{OB} \\perp \\mathrm{BF}, \\therefore \\quad \\mathrm{BF}$ 是 $\\odot \\mathrm{O}$ 的切线;\n\n\n\n(2) 如图, 连接 $O A$, 交 $B C$ 于点 $G$,\n\n$\\because \\quad \\mathrm{AC}=\\mathrm{AB}$,\n\n\n\n$\\therefore \\quad$ 弧 $\\mathrm{AC}=$ 弧 $\\mathrm{AB} \\therefore \\quad \\angle \\mathrm{D}=\\angle 2=\\angle \\mathrm{ABF}, \\mathrm{OA} \\perp \\mathrm{BC}, \\mathrm{BG}=\\mathrm{CG}$,\n\n$\\therefore \\quad \\cos \\angle \\mathrm{D}=\\cos \\angle 2=\\cos \\angle \\mathrm{ABF}=\\frac{4}{5}$, 在 $\\triangle \\mathrm{ABD}$ 中, $\\angle \\mathrm{DAB}=90^{\\circ}$,\n\n$\\therefore \\quad \\mathrm{BD}=\\frac{\\mathrm{AD}}{\\cos \\angle \\mathrm{D}}=5, \\therefore \\quad \\mathrm{AB}=\\sqrt{\\mathrm{BD}^{2}-\\mathrm{AD}^{2}}=3$,\n\n在 $\\triangle \\mathrm{ABG}$ 中, $\\angle \\mathrm{AGB}=90^{\\circ}, \\mathrm{AD}=4, \\therefore \\quad \\mathrm{BG}=\\mathrm{AB} \\times \\cos \\angle 2=\\frac{12}{5}$,\n\n$\\therefore \\quad \\mathrm{BC}=2 \\mathrm{BG}=\\frac{24}{5}$."} {"id": "3887", "image": ["5470.jpg", "5471.jpg", "5471.jpg"], "answer": "(1) $\\angle \\mathrm{A}=24^{\\circ}$; (2) $\\mathrm{AE}=\\sqrt{10}$.\n【解析】(1) $\\because \\mathrm{BD}$ 与 $\\odot \\mathrm{O}$ 相切于点 $\\mathrm{B}$,\n\n$\\therefore \\quad \\angle \\mathrm{ABD}=90^{\\circ}, \\because \\quad \\angle \\mathrm{DBC}=24^{\\circ}$,\n\n$\\therefore \\quad \\angle \\mathrm{ABC}=66^{\\circ}, \\therefore \\quad \\mathrm{AB}$ 为 $\\odot \\mathrm{O}$ 的直径,\n\n$\\therefore \\quad \\angle \\mathrm{C}=90^{\\circ}, \\therefore \\quad \\angle \\mathrm{A}=90^{\\circ}-66^{\\circ}=24^{\\circ}$;\n\n\n(2)连接 $\\mathrm{BE}$,\n\n在 Rt $\\triangle \\mathrm{ACB}$ 中, $\\mathrm{BC}=2, \\mathrm{AC}=4, \\therefore \\mathrm{AB}=\\sqrt{20}$,\n\n$\\because \\quad \\mathrm{CE}$ 平分 $\\angle \\mathrm{ACB}$ 与 $\\odot \\mathrm{O}$ 交于点 $\\mathrm{E}, \\therefore \\mathrm{AE}=\\mathrm{BE}, \\therefore \\quad \\mathrm{AE}=\\mathrm{BE}$,\n\n$\\because \\quad \\mathrm{AB}$ 是直径, $\\therefore \\quad \\angle \\mathrm{AEB}=90^{\\circ}, \\therefore \\quad \\mathrm{AE}=\\sqrt{\\mathrm{AB}^{2}-\\mathrm{BE}^{2}}=\\sqrt{10}$.", "solution": "null", "level": "九年级", "question": "如图, $\\mathrm{AB}$ 为 $\\odot \\mathrm{O}$ 的直径, $\\mathrm{BD}$ 与 $\\odot \\mathrm{O}$ 相切于点 $\\mathrm{B}, C$ 是圆上一点.\n\n(1)如图1, 若 $\\angle \\mathrm{DBC}=24^{\\circ}$, 求 $\\angle \\mathrm{A}$ 的度数;\n\n(2)如图 2, $\\mathrm{CE}$ 平分 $\\angle \\mathrm{ACB}$ 与 $\\odot \\mathrm{O}$ 交于点 $\\mathrm{E}$, 若 $\\mathrm{BC}=2, \\mathrm{AC}=4$, 求 $\\mathrm{AE}$ 的长.\n", "options": [], "subject": "度量几何学", "analysis": "(1) $\\angle \\mathrm{A}=24^{\\circ}$; (2) $\\mathrm{AE}=\\sqrt{10}$.\n【解析】(1) $\\because \\mathrm{BD}$ 与 $\\odot \\mathrm{O}$ 相切于点 $\\mathrm{B}$,\n\n$\\therefore \\quad \\angle \\mathrm{ABD}=90^{\\circ}, \\because \\quad \\angle \\mathrm{DBC}=24^{\\circ}$,\n\n$\\therefore \\quad \\angle \\mathrm{ABC}=66^{\\circ}, \\therefore \\quad \\mathrm{AB}$ 为 $\\odot \\mathrm{O}$ 的直径,\n\n$\\therefore \\quad \\angle \\mathrm{C}=90^{\\circ}, \\therefore \\quad \\angle \\mathrm{A}=90^{\\circ}-66^{\\circ}=24^{\\circ}$;\n\n\n(2)连接 $\\mathrm{BE}$,\n\n在 Rt $\\triangle \\mathrm{ACB}$ 中, $\\mathrm{BC}=2, \\mathrm{AC}=4, \\therefore \\mathrm{AB}=\\sqrt{20}$,\n\n$\\because \\quad \\mathrm{CE}$ 平分 $\\angle \\mathrm{ACB}$ 与 $\\odot \\mathrm{O}$ 交于点 $\\mathrm{E}, \\therefore \\mathrm{AE}=\\mathrm{BE}, \\therefore \\quad \\mathrm{AE}=\\mathrm{BE}$,\n\n$\\because \\quad \\mathrm{AB}$ 是直径, $\\therefore \\quad \\angle \\mathrm{AEB}=90^{\\circ}, \\therefore \\quad \\mathrm{AE}=\\sqrt{\\mathrm{AB}^{2}-\\mathrm{BE}^{2}}=\\sqrt{10}$."} {"id": "3905", "image": ["5501.jpg"], "answer": "(1)证明见解析 (2) $\\frac{3}{5}$\n\n【解析】(1)证明: 作 $O M \\perp A B$ 于 $M$,\n\n$\\because O A$ 平分 $\\angle C A B, O C \\perp A C, O M \\perp A B$,\n\n$\\therefore O C=O M . \\quad \\therefore A B$ 是 $\\odot O$ 的切线.\n\n(2)设 $B M=x, O B=y$,\n\n则 $y^{2}-x^{2}=1$. (1) $\\because \\tan \\angle C A O=\\frac{1}{3}, \\therefore A C=A M=3$.\n\n$\\because \\cos B=\\frac{B M}{O B}=\\frac{B C}{A B}, \\therefore \\frac{x}{y}=\\frac{y+1}{x+3} . \\therefore x^{2}+3 x=y^{2}+y$.\n\n由(1)(2)可得 $y=3 x-1, \\therefore(3 x-1)^{2}-x^{2}=1$.\n$\\therefore x=\\frac{3}{4}, y=\\frac{5}{4} . \\therefore \\cos B=\\frac{x}{y}=\\frac{3}{5}$.", "solution": "null", "level": "九年级", "question": "如图, 在 Rt $\\triangle \\mathrm{ABC}$ 中, $\\angle \\mathrm{ACB}=90^{\\circ}, \\angle \\mathrm{BAC}$ 的平分线交 $\\mathrm{BC}$ 于点 $\\mathrm{O}, \\mathrm{OC}=1$, 以点 $\\mathrm{O}$ 为圆心 $\\mathrm{OC}$ 为半径作半圆.\n\n(1)求证: $A B$ 为 $\\odot O$ 的切线;\n\n(2)如果 $\\tan \\angle \\mathrm{CAO}=\\frac{1}{3}$, 求 $\\cos \\mathrm{B}$ 的值.\n\n", "options": [], "subject": "度量几何学", "analysis": "(1)证明见解析 (2) $\\frac{3}{5}$\n\n【解析】(1)证明: 作 $O M \\perp A B$ 于 $M$,\n\n$\\because O A$ 平分 $\\angle C A B, O C \\perp A C, O M \\perp A B$,\n\n$\\therefore O C=O M . \\quad \\therefore A B$ 是 $\\odot O$ 的切线.\n\n(2)设 $B M=x, O B=y$,\n\n则 $y^{2}-x^{2}=1$. (1) $\\because \\tan \\angle C A O=\\frac{1}{3}, \\therefore A C=A M=3$.\n\n$\\because \\cos B=\\frac{B M}{O B}=\\frac{B C}{A B}, \\therefore \\frac{x}{y}=\\frac{y+1}{x+3} . \\therefore x^{2}+3 x=y^{2}+y$.\n\n由(1)(2)可得 $y=3 x-1, \\therefore(3 x-1)^{2}-x^{2}=1$.\n$\\therefore x=\\frac{3}{4}, y=\\frac{5}{4} . \\therefore \\cos B=\\frac{x}{y}=\\frac{3}{5}$."} {"id": "3908", "image": ["5506.jpg", "5507.jpg", "5508.jpg", "5509.jpg", "5510.jpg", "5507.jpg", "5508.jpg", "5509.jpg", "5510.jpg"], "answer": "(1)外,外;(2)2或 14;(3)8 或 32.\n\n【解析】 (1) $A C=B C \\cdot \\tan 30^{\\circ}=12 \\times \\frac{\\sqrt{3}}{3}=4 \\sqrt{3}>6$,\n\n$\\therefore t$ 无论为何值, 点 $A$ 始终在半圆 $O$ 外,\n\n$\\therefore$ 当 $t=0(s)$ 时, 点 $A$ 在半圆 $O$ 外, 当 $t=8(s)$ 时, 点 $A$ 在半圆 $O$ 外;\n\n(2) (1)如图, 半圆 $O$ 位于 $A C$ 左侧时, $O C=6 \\mathrm{~cm}, t=(8-6) \\div 1=2(s)$;\n\n\n\n(2)如图, 半圆 $O$ 位于 $A C$ 右侧时,\n\n$O C=6 \\mathrm{~cm}, t=(8+6) \\div 1=14(s) ;$\n\n\n\n$\\therefore$ 当 $t=2$ 或 14 时, $\\triangle A B C$ 的边 $A C$ 与半圆 $O$ 相切;\n\n(3) (1)如图, 半圆 $O$ 与 $A B$ 相切于点 $F$, 连接 $O F$,\n$\\therefore O F \\perp A B, \\quad \\because O F=6 \\mathrm{~cm}, \\quad \\angle A B C=30^{\\circ}$,\n\n$\\therefore B O=\\frac{6}{\\sin 30^{\\circ}}=12 \\mathrm{~cm}, \\therefore$ 点 $O$ 与点 $C$ 重合,\n\n$\\therefore t=8 \\div 1=8(s) ;$\n\n\n\n(2)如图, 半圆 $O$ 与 $A B$ 的延长线相切于点 $Q$, 连接 $O Q$,\n\n$\\because \\angle O B Q=\\angle A B C=30^{\\circ}, O Q=6 \\mathrm{~cm}, \\quad \\therefore B O=\\frac{6}{\\sin 30^{\\circ}}=12 \\mathrm{~cm}, \\quad \\therefore t=(12+12+8) \\div 1=32(\\mathrm{~s})$.\n\n\n\n$\\therefore$ 当 $t=8$ 或 32 时, $\\triangle A B C$ 的边 $A B$ 与半圆 $O$ 相切;", "solution": "null", "level": "九年级", "question": "如图, 形如量角器的半圆 $O$ 的直径 $D E=12 \\mathrm{~cm}$, 形如三角板的 $\\triangle A B C$ 中, $\\angle A C B=90^{\\circ}$, $\\angle A B C=30^{\\circ}, B C=12 \\mathrm{~cm}$, 半圆 $O$ 以 $1 \\mathrm{~cm} / \\mathrm{s}$ 的速度从左向右运动, 在运动过程中, 点 $D 、 E$ 始终在直线 $B C$ 上, 设运动时间为 $t(s)$, 当 $t=0(s)$ 时, 半圆 $O$ 在 $\\triangle A B C$ 的左侧, $O C=8 \\mathrm{~cm}$.\n\n(1) 当 $t=0(s)$ 时, 点 $A$ 在半圆 $O$ \\$ \\qquad \\$ , 当 $t=8(s)$ 时, 点 $A$ 在半圆 $O$ \\$ \\qquad \\$ ;\n\n(2) 当 $t$ 为何值时, $\\triangle A B C$ 的边 $A C$ 与半圆 $O$ 相切?\n\n(3) 当 $t$ 为何值时, $\\triangle A B C$ 的边 $A B$ 与半圆 $O$ 相切?\n\n", "options": [], "subject": "度量几何学", "analysis": "(1)外,外;(2)2或 14;(3)8 或 32.\n\n【解析】 (1) $A C=B C \\cdot \\tan 30^{\\circ}=12 \\times \\frac{\\sqrt{3}}{3}=4 \\sqrt{3}>6$,\n\n$\\therefore t$ 无论为何值, 点 $A$ 始终在半圆 $O$ 外,\n\n$\\therefore$ 当 $t=0(s)$ 时, 点 $A$ 在半圆 $O$ 外, 当 $t=8(s)$ 时, 点 $A$ 在半圆 $O$ 外;\n\n(2) (1)如图, 半圆 $O$ 位于 $A C$ 左侧时, $O C=6 \\mathrm{~cm}, t=(8-6) \\div 1=2(s)$;\n\n\n\n(2)如图, 半圆 $O$ 位于 $A C$ 右侧时,\n\n$O C=6 \\mathrm{~cm}, t=(8+6) \\div 1=14(s) ;$\n\n\n\n$\\therefore$ 当 $t=2$ 或 14 时, $\\triangle A B C$ 的边 $A C$ 与半圆 $O$ 相切;\n\n(3) (1)如图, 半圆 $O$ 与 $A B$ 相切于点 $F$, 连接 $O F$,\n$\\therefore O F \\perp A B, \\quad \\because O F=6 \\mathrm{~cm}, \\quad \\angle A B C=30^{\\circ}$,\n\n$\\therefore B O=\\frac{6}{\\sin 30^{\\circ}}=12 \\mathrm{~cm}, \\therefore$ 点 $O$ 与点 $C$ 重合,\n\n$\\therefore t=8 \\div 1=8(s) ;$\n\n\n\n(2)如图, 半圆 $O$ 与 $A B$ 的延长线相切于点 $Q$, 连接 $O Q$,\n\n$\\because \\angle O B Q=\\angle A B C=30^{\\circ}, O Q=6 \\mathrm{~cm}, \\quad \\therefore B O=\\frac{6}{\\sin 30^{\\circ}}=12 \\mathrm{~cm}, \\quad \\therefore t=(12+12+8) \\div 1=32(\\mathrm{~s})$.\n\n\n\n$\\therefore$ 当 $t=8$ 或 32 时, $\\triangle A B C$ 的边 $A B$ 与半圆 $O$ 相切;"} {"id": "3909", "image": ["5511.jpg"], "answer": "(1) $\\frac{20}{3}$;\n(2) $E F \\times C O^{2}$ 的最大值为 $\\frac{125}{4}$.\n\n【解析】 (1) $E C=2$, 则 $C O=5-2=3$.\n\n$\\because C O \\perp A B, \\therefore A B=2 C B$. 在 Rt $\\triangle B C O$ 中, $B O=5, \\therefore B C=\\sqrt{O B^{2}-O C^{2}}=\\sqrt{5^{2}-3^{2}}=4, \\therefore A B=8$.\n\n$\\because B F$ 为 $\\odot O$ 的切线, $\\therefore O B \\perp B F$.\n\n在 $\\triangle B O C$ 和 $\\triangle O B F$ 中, $\\because \\angle O C B=\\angle F B O=90^{\\circ}, \\angle B O C=\\angle B O F, \\therefore \\triangle B O C \\sim \\triangle O B F, \\therefore \\frac{O C}{B O}=\\frac{B C}{B F}$, $\\therefore \\frac{3}{5}=\\frac{4}{B F}$, 解得: $B F=\\frac{20}{3}$;\n\n(2) $\\because \\angle C B F+\\angle O B C=90^{\\circ}, \\angle B O C+\\angle O B C=90^{\\circ}, \\therefore \\angle C B F=\\angle B O C$, 又 $\\angle B C F=\\angle B C O=90^{\\circ}$, $\\therefore \\triangle B C O \\sim \\triangle F C B, \\quad \\therefore \\frac{B C}{O C}=\\frac{F C}{B C}, \\therefore B C^{2}=O C \\times F C$.\n\n$\\because O C=5-x, O B=5, \\quad \\therefore B C^{2}=B O^{2}-C O^{2}=25-(5-x)^{2}, \\quad \\therefore 25-(5-x)^{2}=C O \\times F C=(5-x) \\times F C$,\n\n$\\therefore F C=\\frac{10 x-x^{2}}{5-x}, \\therefore E F \\times C O^{2}=(F C-E C) \\times C O^{2}$\n\n$=\\left(\\frac{10 x-x^{2}}{5-x}-x\\right)(5-x)^{2}=5 x(5-x)=-5\\left(x-\\frac{5}{2}\\right)^{2}+\\frac{125}{4}$\n\n$\\therefore E F \\times C O^{2}$ 的最大值为 $\\frac{125}{4}$.", "solution": "null", "level": "九年级", "question": "如图, $\\odot O$ 的半径长为 $5, O C$ 垂直弦 $A B$ 于点 $C, O C$ 的延长线交 $\\odot O$ 于点 $E$, 与过点 $B$ 的 $\\odot O$的切线交于点 $F$, 已知 $C E=x$.\n\n$(l)$ 若 $x=2$, 求 $A B 、 B F$ 的长;\n\n(2) 求 $E F \\cdot C O^{2}$ 的最大值.\n\n", "options": [], "subject": "度量几何学", "analysis": "(1) $\\frac{20}{3}$;\n(2) $E F \\times C O^{2}$ 的最大值为 $\\frac{125}{4}$.\n\n【解析】 (1) $E C=2$, 则 $C O=5-2=3$.\n\n$\\because C O \\perp A B, \\therefore A B=2 C B$. 在 Rt $\\triangle B C O$ 中, $B O=5, \\therefore B C=\\sqrt{O B^{2}-O C^{2}}=\\sqrt{5^{2}-3^{2}}=4, \\therefore A B=8$.\n\n$\\because B F$ 为 $\\odot O$ 的切线, $\\therefore O B \\perp B F$.\n\n在 $\\triangle B O C$ 和 $\\triangle O B F$ 中, $\\because \\angle O C B=\\angle F B O=90^{\\circ}, \\angle B O C=\\angle B O F, \\therefore \\triangle B O C \\sim \\triangle O B F, \\therefore \\frac{O C}{B O}=\\frac{B C}{B F}$, $\\therefore \\frac{3}{5}=\\frac{4}{B F}$, 解得: $B F=\\frac{20}{3}$;\n\n(2) $\\because \\angle C B F+\\angle O B C=90^{\\circ}, \\angle B O C+\\angle O B C=90^{\\circ}, \\therefore \\angle C B F=\\angle B O C$, 又 $\\angle B C F=\\angle B C O=90^{\\circ}$, $\\therefore \\triangle B C O \\sim \\triangle F C B, \\quad \\therefore \\frac{B C}{O C}=\\frac{F C}{B C}, \\therefore B C^{2}=O C \\times F C$.\n\n$\\because O C=5-x, O B=5, \\quad \\therefore B C^{2}=B O^{2}-C O^{2}=25-(5-x)^{2}, \\quad \\therefore 25-(5-x)^{2}=C O \\times F C=(5-x) \\times F C$,\n\n$\\therefore F C=\\frac{10 x-x^{2}}{5-x}, \\therefore E F \\times C O^{2}=(F C-E C) \\times C O^{2}$\n\n$=\\left(\\frac{10 x-x^{2}}{5-x}-x\\right)(5-x)^{2}=5 x(5-x)=-5\\left(x-\\frac{5}{2}\\right)^{2}+\\frac{125}{4}$\n\n$\\therefore E F \\times C O^{2}$ 的最大值为 $\\frac{125}{4}$."} {"id": "3933", "image": ["5545.jpg", "5546.jpg", "5545.jpg", "5546.jpg"], "answer": "$\\mathrm{BE}+\\mathrm{CE}=6$ 或 16\n\n【解析】有两种情形, 需要分类讨论:\n\n(1) $\\angle \\mathrm{BAC}$ 为锐角, 如图所示,\n\n\n\n$\\because \\mathrm{AB}$ 的垂直平分线是 $\\mathrm{DE}$,\n\n$\\therefore \\mathrm{AE}=\\mathrm{BE}, \\mathrm{ED} \\perp \\mathrm{AB}, \\quad A D=\\frac{1}{2} A B$.\n$\\because \\mathrm{AE}=5, \\quad \\tan \\angle \\mathrm{AED}=\\frac{3}{4}$,\n\n$\\therefore \\sin \\angle \\mathrm{AED}=\\frac{3}{5}$.\n\n$\\therefore \\mathrm{AD}=\\mathrm{AE} \\cdot \\sin \\angle \\mathrm{AED}=3$.\n\n$\\therefore \\mathrm{AB}=6$.\n\n$\\therefore \\mathrm{BE}+\\mathrm{CE}=\\mathrm{AE}+\\mathrm{CE}=\\mathrm{AC}=\\mathrm{AB}=6$.\n\n(2)若 $\\angle B A C$ 为钝角, 如图所示, 同理可求得: $B E+C E=16$.\n\n\n\n综上所述, $\\mathrm{BE}+\\mathrm{CE}=6$ 或 16", "solution": "null", "level": "九年级", "question": "在 $\\triangle A B C$ 中, $A B=A C, A B$ 的垂直平分线 $D E$ 与 $A C$ 所在的直线相交于点 $E$, 垂足为 $D$, 连接\n\nBE. 已知 $\\mathrm{AE}=5, \\tan \\angle \\mathrm{AED}=\\frac{3}{4}$, 求 $\\mathrm{BE}+\\mathrm{CE}$ 的值", "options": [], "subject": "度量几何学", "analysis": "$\\mathrm{BE}+\\mathrm{CE}=6$ 或 16\n\n【解析】有两种情形, 需要分类讨论:\n\n(1) $\\angle \\mathrm{BAC}$ 为锐角, 如图所示,\n\n\n\n$\\because \\mathrm{AB}$ 的垂直平分线是 $\\mathrm{DE}$,\n\n$\\therefore \\mathrm{AE}=\\mathrm{BE}, \\mathrm{ED} \\perp \\mathrm{AB}, \\quad A D=\\frac{1}{2} A B$.\n$\\because \\mathrm{AE}=5, \\quad \\tan \\angle \\mathrm{AED}=\\frac{3}{4}$,\n\n$\\therefore \\sin \\angle \\mathrm{AED}=\\frac{3}{5}$.\n\n$\\therefore \\mathrm{AD}=\\mathrm{AE} \\cdot \\sin \\angle \\mathrm{AED}=3$.\n\n$\\therefore \\mathrm{AB}=6$.\n\n$\\therefore \\mathrm{BE}+\\mathrm{CE}=\\mathrm{AE}+\\mathrm{CE}=\\mathrm{AC}=\\mathrm{AB}=6$.\n\n(2)若 $\\angle B A C$ 为钝角, 如图所示, 同理可求得: $B E+C E=16$.\n\n\n\n综上所述, $\\mathrm{BE}+\\mathrm{CE}=6$ 或 16"} {"id": "3306", "image": ["4283.jpg"], "answer": "解: 延长 $A D$ 交 $B C$ 的延长线于点 $F$, 过点 $D$ 作 $D E \\perp B C$ 于点 $E$,\n\n$\\because C D=4 \\sqrt{2}$ 米, $\\angle D C E=45^{\\circ}$,\n\n$\\therefore D E=C E=4$,\n\n$\\because$ 同一时刻物高与影长成正比,\n\n$\\therefore \\frac{D E}{E F}=\\frac{1}{2}$, 解得 $E F=2 D E=8$,\n\n$\\therefore B F=10+4+8=22$,\n\n$\\because D E \\perp B C, A B \\perp B C$,\n\n$\\therefore \\triangle E D F \\sim \\triangle B A F$,\n$\\therefore \\frac{D E}{A B}=\\frac{E F}{B F}$, 即 $\\frac{4}{A B}=\\frac{8}{22}$\n\n$\\therefore A B=11$ 米.\n\n答: 旗杆的高度约为 11 米.\n\n【解析】延长 $A D$ 交 $B C$ 的延长线于点 $F$, 过点 $D$ 作 $D E \\perp B C$ 于点 $E$, 根据勾股定理求出 $E D$ 的长, 再由同一时刻物高与影长成正比得出 $E F$ 的长, 根据 $D E / / A B$ 可知 $\\triangle E D F \\sim \\triangle A B F$, 由相似三角形的对应边成比例即可得出 $A B$ 的长.\n\n本题考查的是相似三角形的应用, 根据题意作出辅助线, 构造出相似三角形是解答此题的关键.", "solution": "null", "level": "九年级", "question": "(本小题 8.0 分)\n\n如图所示, 小林想利用竹竿来测量旗杆 $A B$ 的高度, 在某一时刻测得 1 米长的竹等竖直放置时影长 2 米, 在同时刻测量旗杆的影长时, 旗杆的影子一部分落在地面上 $(B C)$, 另一部分落在斜坡上 $(C D)$, 他测得落在地面上的影长为 10 米, 落在斜坡上的影长为 $4 \\sqrt{2}$ 米, $\\angle D C E=45^{\\circ}$,\n求旗杆 $A B$ 的高度.\n\n", "options": [], "subject": "度量几何学", "analysis": "解: 延长 $A D$ 交 $B C$ 的延长线于点 $F$, 过点 $D$ 作 $D E \\perp B C$ 于点 $E$,\n\n$\\because C D=4 \\sqrt{2}$ 米, $\\angle D C E=45^{\\circ}$,\n\n$\\therefore D E=C E=4$,\n\n$\\because$ 同一时刻物高与影长成正比,\n\n$\\therefore \\frac{D E}{E F}=\\frac{1}{2}$, 解得 $E F=2 D E=8$,\n\n$\\therefore B F=10+4+8=22$,\n\n$\\because D E \\perp B C, A B \\perp B C$,\n\n$\\therefore \\triangle E D F \\sim \\triangle B A F$,\n$\\therefore \\frac{D E}{A B}=\\frac{E F}{B F}$, 即 $\\frac{4}{A B}=\\frac{8}{22}$\n\n$\\therefore A B=11$ 米.\n\n答: 旗杆的高度约为 11 米.\n\n【解析】延长 $A D$ 交 $B C$ 的延长线于点 $F$, 过点 $D$ 作 $D E \\perp B C$ 于点 $E$, 根据勾股定理求出 $E D$ 的长, 再由同一时刻物高与影长成正比得出 $E F$ 的长, 根据 $D E / / A B$ 可知 $\\triangle E D F \\sim \\triangle A B F$, 由相似三角形的对应边成比例即可得出 $A B$ 的长.\n\n本题考查的是相似三角形的应用, 根据题意作出辅助线, 构造出相似三角形是解答此题的关键."} {"id": "3307", "image": ["4284.jpg"], "answer": "解: $\\because C D / / E F / / A B$,\n\n$\\therefore$ 可以得到 $\\triangle A B F \\sim \\triangle C D F, \\triangle A B G \\sim \\triangle E F G$,\n\n$\\therefore \\frac{A B}{C D}=\\frac{B F}{D F}, \\frac{A B}{E F}=\\frac{B G}{F G}$,\n\n又 $\\because C D=E F$,\n\n$\\therefore \\frac{B F}{D F}=\\frac{B G}{F G}$,\n\n$\\because D F=4, \\quad F G=5, B F=B D+D F=B D+4, B G=B D+D F+F G=B D+9$,\n\n$\\therefore \\frac{4+B D}{4}=\\frac{9+B D}{5}$,\n\n$\\therefore B D=16, B F=16+4=20$,\n\n$\\therefore \\frac{A B}{1.6}=\\frac{20}{4}$,\n\n解得 $A B=8 m$.\n\n答: 路灯杆 $A B$ 的高度为 $8 m$.\n\n【解析】在同一时刻物高和影长成正比, 根据相似三角形的性质即可解答.\n\n此题主要考查了相似三角形的应用, 本题只要是把实际问题抽象到相似三角形中, 利用相似三角形的性质对应边成比例就可以求出结果.", "solution": "null", "level": "九年级", "question": "(本小题8.0分)\n\n如图, 河对岸有一路灯杆 $A B$, 在灯光下, 小明在点 $D$ 处, 自己的影长 $D F=4 m$, 沿 $B D$ 方向到达点 $F$ 处再测自己的影长 $F G=5 m$, 如果小明的身高为 $1.6 m$, 求路灯杆 $A B$ 的高度.\n\n", "options": [], "subject": "度量几何学", "analysis": "解: $\\because C D / / E F / / A B$,\n\n$\\therefore$ 可以得到 $\\triangle A B F \\sim \\triangle C D F, \\triangle A B G \\sim \\triangle E F G$,\n\n$\\therefore \\frac{A B}{C D}=\\frac{B F}{D F}, \\frac{A B}{E F}=\\frac{B G}{F G}$,\n\n又 $\\because C D=E F$,\n\n$\\therefore \\frac{B F}{D F}=\\frac{B G}{F G}$,\n\n$\\because D F=4, \\quad F G=5, B F=B D+D F=B D+4, B G=B D+D F+F G=B D+9$,\n\n$\\therefore \\frac{4+B D}{4}=\\frac{9+B D}{5}$,\n\n$\\therefore B D=16, B F=16+4=20$,\n\n$\\therefore \\frac{A B}{1.6}=\\frac{20}{4}$,\n\n解得 $A B=8 m$.\n\n答: 路灯杆 $A B$ 的高度为 $8 m$.\n\n【解析】在同一时刻物高和影长成正比, 根据相似三角形的性质即可解答.\n\n此题主要考查了相似三角形的应用, 本题只要是把实际问题抽象到相似三角形中, 利用相似三角形的性质对应边成比例就可以求出结果."} {"id": "3331", "image": ["4342.jpg", "4343.jpg", "4343.jpg"], "answer": "解:如图, 过点 $C$ 作 $C E \\perp A B$ 于 $E$,\n\n则四边形 $B D C E$ 是矩形,\n\n所以, $C E=B D=2.7$ 米,\n\n$B E=C D=1.2$ 米,\n\n由题意得, $\\frac{C E}{A E}=\\frac{0.9}{1}$,\n\n\n\n所以, $A E=\\frac{2.7}{0.9}=3$ 米,\n\n树高 $A B=A E+B E=3+1.2=4.2$ 米.\n\n【解析】过点 $C$ 作 $C E \\perp A B$ 于 $E$, 根据同时同地物高与影长成正比列比例式求出 $A E$ 的长度, 再根据矩形的对边相等可得 $B E=C D$, 然后根据 $A B=A E+B E$ 计算即可得解.\n\n本题考查了相似三角形的应用, 熟记同时同地物高与影长成正比并列出比例式是解题的关键, 难点在于作辅助线构造出三角形.", "solution": "null", "level": "九年级", "question": "(本小题 8.0 分)\n\n如图, 一位同学想利用树影测量树 $(A B)$ 的高度, 他在某一时刻测得高为 1 米的竹竿直立时影长为 0.9 米, 此时, 因树靠近一憧建筑物, 影子不全落在地面上(有一部分影子落在了墙上 $C D$ 处 ) , 他先测得落在墙上的影子 $(C D)$ 高为 1.2 米, 又测得地面部分的影长 $(B D)$ 为 2.7 米, 则他测得的树高应为多少米?\n\n", "options": [], "subject": "度量几何学", "analysis": "解:如图, 过点 $C$ 作 $C E \\perp A B$ 于 $E$,\n\n则四边形 $B D C E$ 是矩形,\n\n所以, $C E=B D=2.7$ 米,\n\n$B E=C D=1.2$ 米,\n\n由题意得, $\\frac{C E}{A E}=\\frac{0.9}{1}$,\n\n\n\n所以, $A E=\\frac{2.7}{0.9}=3$ 米,\n\n树高 $A B=A E+B E=3+1.2=4.2$ 米.\n\n【解析】过点 $C$ 作 $C E \\perp A B$ 于 $E$, 根据同时同地物高与影长成正比列比例式求出 $A E$ 的长度, 再根据矩形的对边相等可得 $B E=C D$, 然后根据 $A B=A E+B E$ 计算即可得解.\n\n本题考查了相似三角形的应用, 熟记同时同地物高与影长成正比并列出比例式是解题的关键, 难点在于作辅助线构造出三角形."} {"id": "3332", "image": ["4344.jpg"], "answer": "解: $\\because C D / / E F / / A B$,\n\n$\\therefore$ 可以得到 $\\triangle C D F \\sim \\triangle A B F, \\triangle A B G \\sim \\triangle E F G$,\n\n$\\therefore \\frac{C D}{A B}=\\frac{D F}{B F}, \\frac{E F}{A B}=\\frac{F G}{B G}$\n\n又 $\\because C D=E F$,\n\n$\\therefore \\frac{D F}{B F}=\\frac{F G}{B G}$\n\n$\\because D F=3, \\quad F G=4, B F=B D+D F=B D+3, B G=B D+D F+F G=B D+7$,\n$\\therefore \\frac{3}{D B+3}=\\frac{4}{B D+7}$\n\n$\\therefore B D=9, B F=9+3=12$,\n\n$\\therefore \\frac{1.6}{A B}=\\frac{3}{12}$\n\n解得, $A B=6.4 m$.\n\n答: 路灯杆 $A B$ 的高度为 $6.4 m$.\n\n【解析】\n\n【分析】\n\n在同一时刻物高和影长成正比, 根据相似三角形的性质即可解答.\n\n此题主要考查了相似三角形的应用, 本题只要是把实际问题抽象到相似三角形中, 利用相似三角形的性质对应边成比例就可以求出结果.", "solution": "null", "level": "九年级", "question": "(本小题8.0分)\n\n如图, 河对岸有一路灯杆 $A B$, 在灯光下, 小明在点 $D$ 处, 自己的影长 $D F=3 m$, 沿 $B D$ 方向到达点 $F$ 处再测自己的影长 $F G=4 m$, 如果小明的身高为 $1.6 m$, 求路灯杆 $A B$ 的高度.\n\n", "options": [], "subject": "度量几何学", "analysis": "解: $\\because C D / / E F / / A B$,\n\n$\\therefore$ 可以得到 $\\triangle C D F \\sim \\triangle A B F, \\triangle A B G \\sim \\triangle E F G$,\n\n$\\therefore \\frac{C D}{A B}=\\frac{D F}{B F}, \\frac{E F}{A B}=\\frac{F G}{B G}$\n\n又 $\\because C D=E F$,\n\n$\\therefore \\frac{D F}{B F}=\\frac{F G}{B G}$\n\n$\\because D F=3, \\quad F G=4, B F=B D+D F=B D+3, B G=B D+D F+F G=B D+7$,\n$\\therefore \\frac{3}{D B+3}=\\frac{4}{B D+7}$\n\n$\\therefore B D=9, B F=9+3=12$,\n\n$\\therefore \\frac{1.6}{A B}=\\frac{3}{12}$\n\n解得, $A B=6.4 m$.\n\n答: 路灯杆 $A B$ 的高度为 $6.4 m$.\n\n【解析】\n\n【分析】\n\n在同一时刻物高和影长成正比, 根据相似三角形的性质即可解答.\n\n此题主要考查了相似三角形的应用, 本题只要是把实际问题抽象到相似三角形中, 利用相似三角形的性质对应边成比例就可以求出结果."} {"id": "3008", "image": ["3902.jpg"], "answer": "(1)$(4sqrt{3}-5){m}$;(2)7米;(3)当$A$端下移$left(12-frac{13sqrt{2}}{2}right){m},triangleABC$面积最大,最大为$frac{169}{4}{~m}^{2}$", "solution": "null", "level": "九年级", "question": "(本题6分)(2021$cdot$浙江省常山育才中学九年级期中)如图,斜靠在墙上的一根竹笲$AB$长为$13m$,端点$B$离墙角的水平距离$BC$长为$5m$.(1)若$A$端沿垂直于地面的方向$AC$下移$1m$,则$B$端将沿$CB$方向移动多少米?(2)若$A$端下移的距离等于$B$端沿$CB$方向移动的距离,则$B$端将沿$CB$方向移动多少米?(3)在竹竿滑动的过程中,当$A$端下移多少距离时,$triangleABC$面积最大?简述理由,并求出最大值.", "options": [], "subject": "度量几何学", "analysis": "【分析】(1)根据题意得:$angleACB=90^{circ},AB=A_{1}B_{1}=13{~m},BC=5{~m}$,由勾股定理可得$AC=12{~m}$,从而得到$B_{1}C=4sqrt{3}{~m}$,即可求解;(2)根据题意可设$AA_{1}=BB_{1}=y{~m}$,则$A_{1}C=(12-y){m},CB_{1}=(5+y){m}$,根据勾股定理列出方程,即可求解;(3)设$A$端下移的距离为$x{~m}$,则$A_{1}C=(12-x){m}$,则$B_{1}C=sqrt{13^{2}-(12-x)^{2}}{~m}$,从而得到$S_{triangleABC}=S_{triangleABC}=frac{1}{2}(12-x)sqrt{13^{2}-(12-x)^{2}}$,然后设$a=12-x$,则$S_{triangleABC}=frac{1}{2}asqrt{169-a^{2}}$,再由二次函数的性质,即可求解.【详解】解:(1)根据题意得:$angleACB=90^{circ},AB=A_{1}B_{1}=13{~m}quad,BC=5{~m}$,$thereforeAC=sqrt{AB^{2}-BC^{2}}=sqrt{13^{2}-5^{2}}=12{~m}$,$becauseA_{1}C=12-1=11{~m}$$thereforeB_{1}C=sqrt{A_{1}B_{1}^{2}-A_{1}C^{2}}=sqrt{13^{2}-11^{2}}=4sqrt{3}{~m}$$thereforeBB_{1}=B_{1}C-BC=(4sqrt{3}-5){m}$,即$B$端将沿$CB$方向移动$(4sqrt{3}-5){m}$;(2)根据题意可设$AA_{1}=BB_{1}=y{~m}$,则$A_{1}C=(12-y){m},CB_{1}=(5+y){m}$在Rt$triangleA_{1}CB_{1}$中,由勾股定理得:$A_{1}C^{2}+CB_{1}^{2}=A_{1}B_{1}^{2}$,即$(12-y)^{2}+(5+y)^{2}=13^{2},$解得:$y=7$,即$B$端将沿$CB$方向移动7米;(3)设$A$端下移的距离为$x{~m}$,则$A_{1}C=(12-x){m}$,则$B_{1}C=sqrt{13^{2}-(12-x)^{2}}{~m}$,$thereforeS_{triangleABC}=S_{triangleA_{1}B_{1}C}=frac{1}{2}(12-x)sqrt{13^{2}-(12-x)^{2}}$设$a=12-x$,则$S_{triangleABC}=frac{1}{2}asqrt{169-a^{2}}$,$thereforeleft(2S_{triangleABC}right)^{2}=a^{2}left(169-a^{2}right)=169a^{2}-a^{4}=-left(a^{2}-frac{169}{2}right)^{2}+frac{169^{2}}{4}$,$therefore$当$a^{2}=frac{169}{2}$,即$a=frac{13sqrt{2}}{2}$时,$left(2S_{triangleABC}right)^{2}$最大,即$S_{triangleABC}$最大,此时当$12-x=frac{13sqrt{2}}{2}$时,$left(2S_{triangleABC}right)^{2}=frac{169^{2}}{4}$,$therefore$当$x=12-frac{13sqrt{2}}{2}$时,$S_{triangleABC}=frac{169}{4}$,$therefore$当$A$端下移$left(12-frac{13sqrt{2}}{2}right){m},triangleABC$面积最大,最大为$frac{169}{4}{~m}^{2}$.【点睛】本题主要考查了勾股定理的应用,二次函数的应用,熟练掌握勾股定理,二次函数的性质是解题的关键."} {"id": "3011", "image": ["3904.jpg", "3905.jpg"], "answer": "(1)$y=-3x^{2}+30xleft(frac{20}{3}leqx<10right)$(2)$7{~m}$(3)$frac{200}{3}{~m}^{2}$", "solution": "null", "level": "九年级", "question": "(本题6分)(2022$cdot$浙江绍兴$cdot$九年级期末)如图,用长为30的篱笆一面利用墙(墙的最大可用长度为$10{~m}$)围成中间隔有一道篱笆(平行于$AB$)的矩形花圃.设花圃的一边$AB$为$x({~m})$,面积为$y({~m})$.(1)求$y$关于$x$的函数表达式,并求出自变量$x$的取值范围;(2)如果要围成面积为$63{~m}^{2}$的花圃,那么$AB$的长为多少?(3)求出所能围成的花圃的最大面积.", "options": [], "subject": "度量几何学", "analysis": "【分析】(1)设$AB$长为$x({~m})$,则$BC$长为$(30-3x)({m})$,根据墙的最大可用长度为$10{~m}$,且$BC$的长度大于0,可得自变量的取值范围,面积为长乘宽,可得函数表达式;(2)面积为$63m^{2}$,即$y=63$,代入表达式可得$x$的值,根据$x$的取值范围,可得结果;(3)把二次函数化成顶点式,根据函数的增减性求最值即可.【详解】解:(1)设$AB$长为$x({~m})$,则$BC$长为$(30-3x)({m})$,$therefore3x<30$且$30-3xleq10$.即$frac{20}{3}leqx<10$.$thereforey=x(30-3x)=-3x^{2}+30xleft(frac{20}{3}leqx<10right)$.(2)由题意得:$-3x^{2}+30x=63$,解得:$x=3$或7.$becausefrac{20}{3}leqx<10,thereforex=3$不合题意,就舍去.$therefore$如果要围成面积为$63{~m}^{2}$的花圃,那么$AB$的长应为$7{~m}$.(3)由题意知:$y=-3(x-5)^{2}+75left(frac{20}{3}leqx<10right)$,$therefore$在对称轴直线$x=5$的右侧,$y$随$x$的增大而减小,$therefore$当$x=frac{20}{3}$时,$y$有最大值.最大值为$x(30-3x)=frac{20}{3}timesleft(30-3timesfrac{20}{3}right)=frac{200}{3}$.$therefore$篱笆围成的花圃的最大面积为$frac{200}{3}{~m}^{2}$.【点睛】本题考查二次函数的实际应用中的面积问题,根据题意理清关系是解题的关键."} {"id": "3013", "image": ["3906.jpg"], "answer": "(1)5(2)(1)$(1,2)$(2)$t=1-frac{sqrt{3}}{2}$或$t=frac{sqrt{3}}{2}$", "solution": "null", "level": "九年级", "question": "(本题8分)(2022$cdot$浙江绍兴$cdot$九年级期末)在平面直角坐标系$xOy$中,$O$为坐标原点,定义$Pleft(x_{1},y_{1}right),Qleft(x_{2},y_{2}right)$两点之间的“直角距离”为$d(P,Q)=left|x_{1}-x_{2}right|+left|y_{1}-y_{2}right|$.二次函数$y=x^{2}-3x+4$的图象如图所示.(1)点$A$为图象与$y$轴的交点,点$B(-1,b)$在该二次函数的图象上,求$d(A,B)$的值.(2)点$C$是二次函数$y=x^{2}-3x+4(xgeq0)$图象上的一点,记点$C$的横坐标为$m$.(1)求$d(O,C)$的最小值及对应的点$C$的坐标.(2)当$tleqmleqt+1$时,$d(O,C)$的最大值为$p$,最小值为$q$,若$p-q=frac{3}{4}$,求$t$的值.", "options": [], "subject": "度量几何学", "analysis": "【分析】(1)分别求出$A、B$的坐标,然后根据直角距离的定义求解即可;(2)(1)先求出点$C$的坐标为$left(m,m^{2}-3m+4right)$,则$d(O,C)=(m-1)^{2}+3$,由此求解即可;(2)分类讨论当$01$时,三种情况分别求解即可.(1)解:$because$点$A$是二次函数$y=x^{2}-3x+4$与$y$轴的交点,$therefore$点$A$的坐标为$(0,4)$,$because$点$B(-1,b)$在二次函数$y=x^{2}-3x+4$的函数图象上,$thereforeb=(-1)^{2}-3times(-1)+4=8$,$therefore$点$B$的坐标为$(-1,8)$,$therefored(A,B)=|-1-0|+|8-4|=5$(2)解:(1)令$x=m$,则$y=m^{2}-3m+4$,$therefore$点$C$的坐标为$left(m,m^{2}-3m+4right)$,$therefored(O,C)=|m-0|+left|m^{2}-3m+4-0right|$,$becausemgeq0,m^{2}-3m+4=left(m-frac{3}{2}right)^{2}+frac{7}{4}geqfrac{7}{4}$,$therefored(O,C)=m+m^{2}-3m+4=(m-1)^{2}+3$,$therefore$当$m=1$时,$d(O,C)$有最小值,最小值为3,此时点$C$的坐标为$(1,2)$;(2)$becaused(O,C)=(m-1)^{2}+3$,$therefore$当$0leqm<1$时,$d(O,C)$随$m$的增大而减小,当$mgeq1$时,$d(O,C)$随$m$的增大而增大,把$m=t$代入到$d(O,C)=(m-1)^{2}+3$中得,$d(O,C)=(t-1)^{2}+3$把$m=t+1$代入到$d(O,C)=(m-1)^{2}+3$中得,$d(O,C)=t^{2}+3$,当$t+1-1=1-t$时,解得$t=frac{1}{2}$,当$01$时,$d(O,C)$的最小值$q=(t-1)^{2}+3$,最大值$p=t^{2}+3$$becausep-q=frac{3}{4}$,$thereforet^{2}-(t-1)^{2}=frac{3}{4}$,解得$t=frac{7}{8}$(舍去);综上所述,$t=1-frac{sqrt{3}}{2}$或$t=frac{sqrt{3}}{2}$.【点睛】本题主要考查了二次函数的综合应用,解题的关键在于能够熟练掌握二次函数的性质."} {"id": "3052", "image": ["3946.jpg", "3947.jpg"], "answer": "不需要采取紧急措施,理由详见解析", "solution": "null", "level": "九年级", "question": "(本题6分)(2020$cdot$浙江宁波$cdot$九年级期末)如图,有一座圆弧形拱桥,它的跨度$AB$为$60{~m}$,拱高$PM$为$18{~m}$,当洪水泛滥到跨度只有$30{~m}$时,就要采取紧急措施,若某次洪水中,拱顶离水面只有$4{~m}$,即$PN=4{~m}$时,试通过计算说明是否需要采取紧急措施.", "options": [], "subject": "度量几何学", "analysis": "【分析】连接${OA}^{prime},{OA}$.设圆的半径是${R}$,则${ON}={R}-4,{OM}={R}-18$.根据垂径定理求得${AM}$的长,在直角三角形${AOM}$中,根据勾股定理求得${R}$的值,在直角三角形${A}^{prime}{ON}$中,根据勾股定理求得${A}^{prime}{N}$的值,再根据垂径定理求得${A}^{prime}{B}^{prime}$的长,从而作出判断.【详解】设圆弧所在圆的圆心为$O$,连结$OA,OA^{prime}$,如图所示设半径为$x({~m})$则$OA=OA^{prime}=OP=x({~m})$由垂径定理可知$AM=BM,A^{prime}N=B^{prime}N$$becauseAB=60{~m},quadthereforeAM=30{~m}$,且$OM=OP-PM=(x-18){m}$在Rt$triangleAOM$中,由勾股定理可得$AO^{2}=OM^{2}+AM^{2}$即$x^{2}=(x-18)^{2}+30^{2}$,解得$x=34$$thereforeON=OP-PN=34-4=30({~m})$在$triangleA^{prime}ON$中,由勾股定理可得$A^{prime}N=sqrt{OA^{2}-ON^{2}}=sqrt{34^{2}-30^{2}}=16({~m})$$thereforeA^{prime}B^{prime}=32{~m}>30{~m}$$therefore$不需要采取紧急措施.【点睛】此类题综合运用了勾股定理和垂径定理,解题的关键是熟知垂径定理的应用,"} {"id": "3053", "image": ["3948.jpg", "3949.jpg"], "answer": "(1)详见解析;(2)$OA=frac{13}{4}$", "solution": "null", "level": "九年级", "question": "(本题6分)(2020$cdot$浙江$cdot$天台实验中学九年级阶段练习)如图,$odotO$的半径$OAperp$弦$BC$于$E,D$是$odotO$上一点(1)求证:$angle{ADC}=frac{1}{2}angle{AOB}$;(2)求$AE=2,{BC}=6$,求${OA}$的长", "options": [], "subject": "度量几何学", "analysis": "【分析】(1)利用垂径定理及同圆中等弧所对的圆心角相等可得$angleAOB=angleAOC$,根据圆周角定理可得结论;(2)设$OA=x$,则$OE=x-2,{BE}$长易知,在Rt$triangleBOE$中,利用勾股定理可求解.【详解】(1)证明:连接$OC$$becauseOAperpBCthereforeAwidehat{B}=Awidehat{C}$$thereforeangleAOB=angleAOC$$becauseAwidehat{C}=Awidehat{C}$$thereforeangleADC=frac{1}{2}angleAOC$$thereforeangleADC=frac{1}{2}angleAOB$(2)设$OA=x$,则$OE=x-2$,$becauseOAperpBCquadthereforequadBE=EC=3$.在Rt$triangleBOE$中,由$OE2+BE2=OB2$得$(x-2)2+32=x2$,解得$x=frac{13}{4}quadthereforeOA=frac{13}{4}$【点睛】本题主要考查了圆的垂径定理及圆周角定理,灵活利用垂径定理证明角与线段间的关系是解题的关键."} {"id": "3055", "image": ["3951.jpg"], "answer": "这块圆形木材的直径(AC)是26寸", "solution": "null", "level": "九年级", "question": "(本题6分)(2022$cdot$浙江$cdot$九年级专题练习)《九章算术》是我国古代第一部自成体系的数学专著,代表了东方数学的最高成就,它的算法体系至今仍在推动着计算机的发展和应用.书中记载:“今有圆材埋在壁中,不知大小,以锯锯之,深一寸,锯道长一尺,问径几何?”译为:“今有一圆柱形木材,埋在墙壁中,不知其大小,用锯去锯这木材,锯口深1寸($ED=1$寸),锯道长1尺($AB=1$尺=$=10$寸).问这块圆形木材的直径$(AC)$是多少?\"如图所示,请根据所学的知识解答上述问题.", "options": [], "subject": "度量几何学", "analysis": "【分析】设$odotO$的半径为$x$寸,根据题意可得$AD=BD$,在$RttriangleAOD$中,$OA=x$,$OD=x-1$,勾股定理求解即可.【详解】设$odotO$的半径为$x$寸,$becauseOEperpAB,AB=10$寸,$thereforeAD=BD=frac{1}{2}AB=5$寸,在Rt$triangleAOD$中,$OA=x,OD=x-1$,由勾股定理得$x^{2}=(x-1)^{2}+5^{2}$,解得$x=13$.$thereforeodotO$的直径$AC=2x=26$(寸).答:这块圆形木材的直径($AC$)是26寸.【点睛】本题考查了垂径定理的应用,掌握垂径定理是解题的关键."} {"id": "3079", "image": ["3986.jpg", "3987.jpg"], "answer": "(1)$sqrt{2}$$(2)1$", "solution": "null", "level": "九年级", "question": "(本题8分)(2022$cdot$浙江丽水$cdot$九年级期中)如图,在$odotO$中,直径$AB=2,triangleABC$中,$angleBAC=90^{circ},BC$交$odotO$于点$D$,若$angleC=45^{circ}$,求:(1)$BD$的长为多少?(2)求阴影部分的面积.", "options": [], "subject": "度量几何学", "analysis": "【分析】(1)连接$AD$,可得$ADperpBC$.再根据$triangleABC$是等腰直角三角形,可得$BD=$$CD,BC=sqrt{2}AB=2sqrt{2}$,即可求解;(2)根据$AD=BD$,可得弧$BD=$弧$AD$,从而得到弓形$BD$的面积$=$弓形$AD$的面积,进而得到阴影部分的面积$=RttriangleADC$的面积,即可求解.(1)解:如图,连接$AD$,$becauseAB$是$odotO$的直径,$thereforeADperpBC$.又$becauseangleBAC=90^{circ},angleC=45^{circ}$,$thereforeangleB=angleC=45^{circ}$,$thereforeAB=AC$,$thereforetriangleABC$是等腰直角三角形,$thereforeBD=CD,quadBC=sqrt{2}AB=2sqrt{2}$,$thereforeAD=BD=CD=sqrt{2}$;(2)解:$becauseAD=BD$,$thereforeBD^{frown}=AD^{frown}$,$therefore$弓形$BD$的面积$=$弓形$AD$的面积,$therefore$阴影部分的面积$=RttriangleADC$的面积$=frac{1}{2}times(sqrt{2})^{2}=1$.【点睛】本题主要考查了等腰直角三角形的判定和性质,圆周角定理,求扇形面积,勾股定理,根据题意,作适当辅助线构造直角三角形是解题的关键."} {"id": "3081", "image": ["3989.jpg", "3990.jpg", "3991.jpg", "3992.jpg", "3993.jpg"], "answer": "$(1)angleO+angleD=210^{circ}$;(2)$frac{3sqrt{3}}{2}-frac{2}{3}pi$(3)$angleAOC$为$140^{circ}$或$160^{circ}$,理由见解析", "solution": "null", "level": "九年级", "question": "(本题9分)(2022$cdot$浙江台州$cdot$九年级期末)如图,在$odotO$中,弦$AB$与半径$OA$形成的夹角$angleA=60^{circ},OA=2$,点$C$是优弧$APB$上的一动点,切线$CD$与射线$AB$相交于点D.冬1图2备用图1备用图2(1)$angleO$与$angleD$满足的数量关系是(2)当$angleD=90^{circ}$时,求阴影部分的面积;(3)当$angleAOC$是多少度时,$triangleBCD$为等腰三角形?通过推理说明理由.", "options": [], "subject": "度量几何学", "analysis": "【分析】(1)根据切线的性质得到$angleC=90^{circ}$,结合四边形内角和是$360^{circ}$来求解;(2)连接$OB$,易得$triangleAOB$是等边三角形,求出$angleBOC=angleAOC-angleAOB=60^{circ}$,从而求出扇形$OBC$的面积,连接$BC$易得$triangleBOC$是等边三角形,进而求出$angleBCD$的度数,计算出$triangleBCD$的面积,即可进一步求得结果;(3)设$angleAOC=x$,由(1)可得$angleD=210^{circ}-x$,进而得到$angleDBC=180^{circ}-angleABC=frac{x}{2}$,分三种情况:当$BD=BC$时,当$CD=BC$时,当$BD=BC$时,分别求解.(1)解:$becauseDC$是$odotO$的切线,$thereforeangleC=90^{circ}$.$becauseangleO+angleA+angleD+angleC=360^{circ}$,$thereforeangleO+60^{circ}+angleD+90^{circ}=360^{circ}$,$thereforeangleO+angleD=210^{circ}$故答案为:$angleO+angleD=210^{circ}$;(2)解:连接$OB$,如图1.图1$becauseangleD=90^{circ},angleAOC+angleD=210^{circ}$,$thereforeangleAOC=120^{circ}$$becauseangleA=60^{circ},OA=OB$$thereforetriangleAOB$是等边三角形,$thereforeangleBOC=angleAOC-angleAOB=60^{circ}$,$therefore{S}_{text{扇形}{OBC}}=frac{60pitimes4}{360}pi=frac{2}{3}pi$,连接$BC$,则$triangleBOC$是等边三角形,$thereforeangleBCD=30^{circ}$.在$RttriangleBCD$中,$BD=frac{1}{2}BC=1$,则$CD=sqrt{3}$,$S_{triangleBC}=frac{1}{2}BDcdotCD=frac{1}{2}times1timessqrt{3}=frac{sqrt{3}}{2}$.$becauseS_{triangleBOC}=frac{1}{2}times2timessqrt{3}=sqrt{3}$,$therefore{S}_{text{四边形}BOCD}==S_{triangleBCD}+S_{triangleBOC}=frac{3sqrt{3}}{2}$,$thereforeS_{text{阴}}=S_{text{四边形}BOCD}-S_{text{扇形}OBC}=frac{3sqrt{3}}{2}-frac{2}{3}pi$;(3)解:当$angleAOC$为$140^{circ}$或160时,$triangleBCD$是等腰三角形.理由如下:设$angleAOC=x$,由(1)可得$angleD=210^{circ}-x$,$angleABC=left(360^{circ}-xright)div2=180^{circ}-frac{x}{2}$,$thereforeangleDBC=180^{circ}-angleABC=frac{x}{2}$,当$BD=BC$时,$2angleD+angleDBC=180^{circ}$,$therefore2left(210^{circ}-xright)+frac{x}{2}=180^{circ}$,$thereforex=160^{circ}$,当$CD=BC$时,$angleD=angleDBC$,$therefore210^{circ}-x=frac{x}{2}$,$thereforex=140^{circ}$,当$BD=BC$时,$2angleDBC+angleD=180^{circ}$,$therefore2timesfrac{x}{2}+left(210^{circ}-xright)=180^{circ}$,$therefore$不存在,反之,当$angleAOC$为$140^{circ}$或160时,$triangleBCD$是等腰三角形.综上所述,$angleAOC$为$140^{circ}$或$160^{circ}$.【点睛】本题考查了圆周角定理,圆内接四边形性质,扇形面积公式,等腰三角形的判定和性质,理解相关知识是解答关键."} {"id": "3082", "image": ["3994.jpg", "3995.jpg", "3996.jpg", "3997.jpg"], "answer": "$(1)45^{circ}$(2)$BC$的长为$frac{5sqrt{2}pi}{2}{~cm}$(3)(1)$BP$的长为$8sqrt{2}{~cm}$;(2)$CA^{prime}=8-2sqrt{34}$", "solution": "null", "level": "九年级", "question": "(本题10分)(2021浙江$cdot$温州外国语学校九年级期中)如图,$triangleABC$内接于$odotO$,$CDperpAB,CB=10{~cm},CD=8{~cm},AB=14{~cm}$.(备用图)(1)$angleA$度数;(直接写出答案)(2)求$BC$的长度;(3)$P$是$odotO$上一点(不与$A,B,C$重合),连结$BP$.(1)若$BP$垂直$triangleABC$的某一边,求$BP$的长;(2)将点$A$绕点$P$逆时针旋转$90^{circ}$后得到$A^{prime}$,若$A^{prime}$恰好落在$CD$上,则$CA^{prime}$的长度为$qquad$.(直接写出答案)", "options": [], "subject": "度量几何学", "analysis": "【分析】(1)根据勾股定理计算出$CD$的值,即可判定等腰直角三角形$triangleADC$,进而即可求解;(2)连接$OC,OB$根据等腰直角三角形的性质和判定即可求解;(3)(1)$BPperpAC$于点$E$,并连接$AP$,根据等腰直角三角形进而证明三角形全等即可应用勾股定理进行求解;(2)连接$AA^{prime}$,根据等腰直角三角形和勾股定理对边进行转化进而求解即可.(1)$becauseCDperpAB$$thereforeangleCDB=90^{circ}$,又$becauseCB=10{~cm},CD=8{~cm}$,$thereforeCD=sqrt{CB^{2}-CD^{2}}=sqrt{10^{2}-8^{2}}=6({~cm})$,$becauseAB=14{~cm}$$thereforeAD=14{~cm}-6{~cm}=8{~cm}=CD$,且$CDperpAB$,$thereforetriangleADC$为等腰直角三角形,$thereforeangleA=45^{circ}$故答案为:$45^{circ}$.(2)连接$CO,BO$$becauseangleCAB=45^{circ}$$thereforeangleCOB=2angleCAB=45^{circ}times2=90^{circ}$,又$becauseCO=BO$,$thereforetriangleCOB$为等腰直角三角形,$thereforeCO=BO=frac{BC}{sqrt{2}}=frac{10}{sqrt{2}}=5sqrt{2}({~cm})$,则$BC=frac{1}{4}C_{odotO}=frac{2piR}{4}=frac{piR}{2}=frac{5sqrt{2}pi}{2}({~cm})$.(3)(1)根据题意可得当$BP$垂直$triangleABC$的某一边时,则$P$点只能在$AC$内,且$BPperpAC$于点$E$,并连接$AP$,$becauseangleP$和$angleACB$为$AB$所对的角,$thereforeangleP=angleACB$由(1)得$angleCAB=45^{circ}$,且$ACperpBP$,$thereforetriangleAEB$为等腰直角三角形,$thereforeAE=BE$$becauseleft{begin{array}{c}angleP=angleECBangleAEP=angleCEB=90^{circ},AE=BEend{array}right.$$thereforetriangleAPEcongtriangleBCE$,$thereforePE=CE,BE=AE$,又$because$在$RttriangleADC$中,$thereforeBP=AC=sqrt{AD^{2}+CD^{2}}=sqrt{8^{2}+8^{2}}=8sqrt{2}({~cm})$.(2)连接$AA^{prime}$,如下图,由(1)得$triangleAEB$为等腰直角三角形,$thereforeAE=EB=frac{14}{sqrt{2}}=7sqrt{2}$,又$becauseAC=8sqrt{2}$,且$triangleAPEcongtriangleBCE$$thereforePE=CE=8sqrt{2}-7sqrt{2}=sqrt{2}$,$therefore$在Rt$triangleAPE$中,$thereforeAP=sqrt{AE^{2}+PE^{2}}=sqrt{(7sqrt{2})^{2}+(sqrt{2})^{2}}=10$,$because$点$A$绕点$P$逆时针旋转$90^{circ}$后得到$A^{prime}$,$thereforePA=PA^{prime},angleAPA^{prime}=90^{circ}$,$thereforeAA^{prime}=sqrt{2}AP=10sqrt{2}$,又$becauseAD=8$,且在$Rt{}_{triangle}ADA^{prime}$中,$thereforeA^{prime}D=sqrt{AA^{prime2}-AD^{2}}=sqrt{(10sqrt{2})^{2}-8^{2}}=2sqrt{34}$,$thereforeCA^{prime}=CD-A^{prime}D=8-2sqrt{34}$.故答案为:$8-2sqrt{34}$.【点睛】本题考查了等腰三角形的性质和判定、全等三角形的性质和判定、圆弧上的性质和勾股定理的应用,解决本题的关键上我以上的性质并联合起来进行对题目进行解读."} {"id": "3103", "image": ["4031.jpg"], "answer": "(1)8;(2)$frac{16pi}{3}$", "solution": "null", "level": "九年级", "question": "(本题8分)(2021$cdot$浙江$cdot$九年级期末)如图,$AB$是$odotO$的直径,弦$CD$垂直平分$OB$,交$OB$于点$E,CD=8sqrt{3}$.(1)求$BD$的长.(2)求劣弧$AC$的弧长.", "options": [], "subject": "度量几何学", "analysis": "【分析】(1)根据垂直平分线的定义和垂径定理可证明$triangleOECcongtriangleBED$,得到$OC=BD$,在$triangleOCE$中利用勾股定理求出半径$r$,可得$BD$;(2)利用三角函数的定义求出$angleBOC$,得到$angleAOC$,再利用弧长公式计算.【详解】解:(1)设圆$O$的半径为$r$,$becauseCD$垂直平分$OB$,$thereforeCE=DE=4sqrt{3},OE=BE=frac{1}{2}r,angleOEC=angleBED=90^{circ}$,$thereforetriangleOECcongtriangleBED(SAS)$$thereforeBD=OC=r$,在$triangleOCE$中,$OE^{2}+CE^{2}=OC^{2}$,即$left(frac{1}{2}rright)^{2}+(4sqrt{3})^{2}=r^{2},$解得:$r=8$或-8(舍),$thereforeBD=OC=8$(2)$becausecosangleCOE=frac{OE}{OC}=frac{1}{2}$,$thereforeangleCOB=60^{circ}$,$thereforeangleAOC=120^{circ}$$therefore$劣弧$AC=frac{8timespitimes120}{180}=frac{16pi}{3}$.【点睛】本题考查了垂直平分线的定义,垂径定理,弧长公式,解题的关键是证明$BD=O{C}$."} {"id": "3586", "image": ["4886.jpg", "4887.jpg", "4887.jpg"], "answer": "(1)证明: $\\because$ 四边形 $A B C D$ 是正方形,\n\n$\\therefore A D=A B, \\angle D A B=\\angle A D M=\\angle A B C=\\angle A B E=90^{\\circ}$,\n\n$$\n\\because D M=B E\n$$\n\n$\\therefore \\triangle A D M \\cong \\triangle A B E$\n\n$\\therefore \\angle D A M=\\angle B A E$,\n\n$\\therefore \\angle M A E=\\angle D A B=90^{\\circ}$,\n\n$\\because H F \\perp A E$,\n\n$\\therefore \\angle E F H=\\angle E A M$\n\n$\\therefore A M / / F H$,\n\n$\\because A B / / M H$,\n\n$\\therefore$ 四边形 $A G H M$ 是平行四边形,\n\n$\\therefore A G=M H$,\n\n$\\therefore D H=D M+H M=A G+E B$.\n\n$(2)$ 连接 $P A$.\n\n$\\because \\angle P B A=\\angle P B C, A B=B C, P B=P B$,\n\n$\\therefore \\triangle P B A \\cong \\triangle P B C$\n\n$\\therefore P A=P C=P E, \\angle P A B=\\angle P C B$,\n\n$\\because P E=P C$\n\n$\\therefore \\angle P E C=\\angle P C E=\\angle P A B$,\n\n$\\therefore \\angle A B E=\\angle A P E=90^{\\circ}$,\n\n$\\therefore P F \\perp A E$,\n\n$\\therefore A F=E F, P F=F A=F E$,\n\n$\\therefore P E=\\sqrt{2} E F$,\n\n$\\because E B=x, \\tan \\angle E A B=\\frac{1}{3}=\\frac{E B}{A B}$,\n\n$\\therefore A B=3 x$,\n\n\n$\\therefore A E=\\sqrt{A B^{2}+B E^{2}}=\\sqrt{10} x$,\n\n$\\therefore y=\\sqrt{2} \\cdot \\frac{1}{2} \\cdot \\sqrt{10} x=\\sqrt{5} x$.\n\n(3)作 $P H \\perp B C$ 于 $H$.\n\n$\\because \\angle G A F=\\angle E A B, \\angle A F G=\\angle A B E=90^{\\circ}$,\n\n$\\therefore \\triangle A F G \\backsim \\triangle A B E$,\n\n$\\therefore A F \\cdot A E=A G \\cdot A B=8+4 \\sqrt{2}$,\n\n$\\therefore 2 E F^{2}=8+4 \\sqrt{2}$,\n\n$\\therefore P E^{2}=2 E F^{2}=8+4 \\sqrt{2}$,\n\n$\\because B E=P B$, 设 $B E=P B=a$, 则 $B H=P H=\\frac{\\sqrt{2}}{2} a$,\n\n在Rt $\\triangle P E H$ 中, $\\because P E^{2}=P H^{2}+E H^{2}$,\n\n$\\therefore 8+4 \\sqrt{2}=\\left(\\frac{\\sqrt{2}}{2} a\\right)^{2}+\\left(a+\\frac{\\sqrt{2}}{2} a\\right)^{2}$,\n\n解得 $a=2$,\n\n$\\therefore E B=2$.", "solution": "null", "level": "九年级", "question": "(12 分) 正方形 $A B C D$ 中, $P$ 在对角线 $B D$ 上, $E$ 在线段 $C B$ 的延长线上(如图), 且 $P E=P C$, 过点 $P$ 作 $P F \\perp A E$ 于 $F$, 直线 $P F$ 分别交线段 $A B 、 C D$ 于 $G 、 H, M$ 在线段 $D C$ 上, $D M=B E$, 连结 $A M$ 交对角线 $B D$ 于 $Q$.\n\n\n\n(1)求证: $D H=A G+B E$;\n\n(2) 设 $B E=x$, 若 $\\tan \\angle B A E=\\frac{1}{3}, P E=y$, 求出 $y$ 与 $x$ 的关系式;\n\n(3) 点 $E$ 在线段 $C B$ 的左侧, 且 $\\triangle P B E$ 为等腰三角形时, 若 $A B \\cdot A G=8+4 \\sqrt{2}$, 求 $B E$ 的长.", "options": [], "subject": "解析几何", "analysis": "(1)证明: $\\because$ 四边形 $A B C D$ 是正方形,\n\n$\\therefore A D=A B, \\angle D A B=\\angle A D M=\\angle A B C=\\angle A B E=90^{\\circ}$,\n\n$$\n\\because D M=B E\n$$\n\n$\\therefore \\triangle A D M \\cong \\triangle A B E$\n\n$\\therefore \\angle D A M=\\angle B A E$,\n\n$\\therefore \\angle M A E=\\angle D A B=90^{\\circ}$,\n\n$\\because H F \\perp A E$,\n\n$\\therefore \\angle E F H=\\angle E A M$\n\n$\\therefore A M / / F H$,\n\n$\\because A B / / M H$,\n\n$\\therefore$ 四边形 $A G H M$ 是平行四边形,\n\n$\\therefore A G=M H$,\n\n$\\therefore D H=D M+H M=A G+E B$.\n\n$(2)$ 连接 $P A$.\n\n$\\because \\angle P B A=\\angle P B C, A B=B C, P B=P B$,\n\n$\\therefore \\triangle P B A \\cong \\triangle P B C$\n\n$\\therefore P A=P C=P E, \\angle P A B=\\angle P C B$,\n\n$\\because P E=P C$\n\n$\\therefore \\angle P E C=\\angle P C E=\\angle P A B$,\n\n$\\therefore \\angle A B E=\\angle A P E=90^{\\circ}$,\n\n$\\therefore P F \\perp A E$,\n\n$\\therefore A F=E F, P F=F A=F E$,\n\n$\\therefore P E=\\sqrt{2} E F$,\n\n$\\because E B=x, \\tan \\angle E A B=\\frac{1}{3}=\\frac{E B}{A B}$,\n\n$\\therefore A B=3 x$,\n\n\n$\\therefore A E=\\sqrt{A B^{2}+B E^{2}}=\\sqrt{10} x$,\n\n$\\therefore y=\\sqrt{2} \\cdot \\frac{1}{2} \\cdot \\sqrt{10} x=\\sqrt{5} x$.\n\n(3)作 $P H \\perp B C$ 于 $H$.\n\n$\\because \\angle G A F=\\angle E A B, \\angle A F G=\\angle A B E=90^{\\circ}$,\n\n$\\therefore \\triangle A F G \\backsim \\triangle A B E$,\n\n$\\therefore A F \\cdot A E=A G \\cdot A B=8+4 \\sqrt{2}$,\n\n$\\therefore 2 E F^{2}=8+4 \\sqrt{2}$,\n\n$\\therefore P E^{2}=2 E F^{2}=8+4 \\sqrt{2}$,\n\n$\\because B E=P B$, 设 $B E=P B=a$, 则 $B H=P H=\\frac{\\sqrt{2}}{2} a$,\n\n在Rt $\\triangle P E H$ 中, $\\because P E^{2}=P H^{2}+E H^{2}$,\n\n$\\therefore 8+4 \\sqrt{2}=\\left(\\frac{\\sqrt{2}}{2} a\\right)^{2}+\\left(a+\\frac{\\sqrt{2}}{2} a\\right)^{2}$,\n\n解得 $a=2$,\n\n$\\therefore E B=2$."} {"id": "3587", "image": ["4888.jpg", "4889.jpg", "4890.jpg", "4891.jpg", "4892.jpg", "4893.jpg", "4891.jpg", "4892.jpg", "4893.jpg"], "answer": "解: (1)如图 1 , 连接 $O P 、 O D, A P$ 与 $O D$ 相交于点 $M$,\n\n$\\because D P$ 与半圆 $O$ 相切,\n\n$\\therefore O A=O P, O P \\perp D P$, 得 $O D$ 垂直平分 $A P$,\n\n$\\therefore \\triangle A M O \\sim \\triangle D A O$\n\n$\\therefore \\frac{A M}{A D}=\\frac{A O}{D O}$,\n\n\n\n图1\n\n$\\because A D=2, A O=1$,\n\n$D O=\\sqrt{A D^{2}+A O^{2}}=\\sqrt{2^{2}+1^{2}}=\\sqrt{5}$,\n\n$\\therefore A M=\\frac{A O \\times A D}{D O}=\\frac{1 \\times 2}{\\sqrt{5}}=\\frac{2 \\sqrt{5}}{5}$,\n\n$\\therefore A P=2 A M=2 \\times \\frac{2 \\sqrt{5}}{5}=\\frac{4 \\sqrt{5}}{5}$;\n\n(2)作 $P E \\perp A B$ 于点 $E$, 设 $P(x, y)$,\n\n在Rt $\\triangle E P O$ 中, 可得 $P E^{2}+E O^{2}=O P^{2}$,\n\n即 $x^{2}+(y-1)^{2}=1^{2} ,$\n\n$\\therefore x^{2}=2 y-y^{2}$,\n\n\n\n图2\n根据题意可得: $S_{1}=\\frac{1}{2} \\cdot A D \\cdot(2-y)=2-y$,\n\n$S_{3}=\\frac{1}{2} \\cdot B C \\cdot y=y$\n\n$S_{2}=\\frac{1}{2} \\cdot A B \\cdot x=x$\n\n$$\n\\therefore 2 S_{1} S_{3}-S_{2}^{2}=2 \\cdot(2-y) \\cdot y-x^{2}=4 y-2 y^{2}-x^{2}=x^{2} \\because 0\n\n图3\n\n在 $\\triangle A B E$ 中, $B E=\\sqrt{A B^{2}+A E^{2}}=\\frac{5}{2}$,\n\n易得 $\\triangle B F K \\sim \\triangle B E A$,\n\n即: $\\frac{F K}{B F}=\\frac{A E}{B E}$,\n\n得 $F K=\\frac{A E}{B E} \\cdot B F=\\frac{\\frac{3}{2} \\cdot \\frac{8}{5}}{\\frac{5}{2}}=\\frac{24}{25}$,\n\n根据勾股定理可得, $B K=\\sqrt{B F^{2}-F K^{2}}=\\frac{32}{25}$\n\n$\\therefore F\\left(\\frac{24}{25}, \\frac{32}{25}\\right)$,\n\n$\\because P(1,1)$,\n\n可求得直线 $P F$ 解析式: $y=-7 x+8$,\n\n设 $Q(a,-7 a+8)$ ,\n\n$\\because P Q=P B=\\sqrt{2}$,\n\n$\\therefore \\sqrt{(a-1)^{2}+(-7 a+8-1)^{2}}=\\sqrt{2}$,\n$\\therefore a_{1}=\\frac{4}{5}, a_{2}=\\frac{6}{5}$\n\n$\\because Q$ 在 $F P$ 的延长上,\n\n$\\therefore a>1$,\n\n$\\therefore a=\\frac{6}{5}$\n\n$\\therefore Q$ 点坐标为 $\\left(\\frac{6}{5},-\\frac{2}{5}\\right)$,\n\n$\\because O$ 点坐标为 $(0,1)$,\n\n$\\therefore Q O=\\sqrt{\\left(\\frac{6}{5}-0\\right)^{2}+\\left(-\\frac{2}{5}-1\\right)^{2}}=\\frac{\\sqrt{85}}{5}$.", "solution": "null", "level": "九年级", "question": "(14 分) 已知四边形 $A B C D$ 是边长为 2 的正方形, 在以 $A B$ 为直径的正方形内作半圆 $O, P$ 为半圆上的动点(不与 $A 、 B$ 重合) 连接 $P A 、 P B 、 P C 、 P D$,\n\n(1)若 $D P$ 与半圆 $O$ 相切时, 求 $P A$ 的长.\n\n(2)如图, 以 $B C$ 边为 $x$ 轴, 以 $A B$ 边为 $y$ 轴, 建立如图所示的平面直角坐标系, 把 $\\triangle P A D 、 \\triangle P A B 、$ $\\triangle P B C$ 的面积分别记为 $S_{1} 、 S_{2} 、 S_{3}$, 试求 $2 S_{1} S_{3}-S_{2}^{2}$ 的最大值, 并求出此时点 $P$ 的坐标.\n\n(3)在 (2)的条件下, $E$ 为边 $A D$ 上一点, 且 $A E=3 D E$, 连接 $B E$ 交半圆 $O$ 于 $F$, 连接 $F P$ 并延长至点 $Q$, 使得 $P Q=P B$, 求 $O Q$ 的长.\n\n\n\n图1\n\n\n\n图2\n\n\n\n图3", "options": [], "subject": "解析几何", "analysis": "解: (1)如图 1 , 连接 $O P 、 O D, A P$ 与 $O D$ 相交于点 $M$,\n\n$\\because D P$ 与半圆 $O$ 相切,\n\n$\\therefore O A=O P, O P \\perp D P$, 得 $O D$ 垂直平分 $A P$,\n\n$\\therefore \\triangle A M O \\sim \\triangle D A O$\n\n$\\therefore \\frac{A M}{A D}=\\frac{A O}{D O}$,\n\n\n\n图1\n\n$\\because A D=2, A O=1$,\n\n$D O=\\sqrt{A D^{2}+A O^{2}}=\\sqrt{2^{2}+1^{2}}=\\sqrt{5}$,\n\n$\\therefore A M=\\frac{A O \\times A D}{D O}=\\frac{1 \\times 2}{\\sqrt{5}}=\\frac{2 \\sqrt{5}}{5}$,\n\n$\\therefore A P=2 A M=2 \\times \\frac{2 \\sqrt{5}}{5}=\\frac{4 \\sqrt{5}}{5}$;\n\n(2)作 $P E \\perp A B$ 于点 $E$, 设 $P(x, y)$,\n\n在Rt $\\triangle E P O$ 中, 可得 $P E^{2}+E O^{2}=O P^{2}$,\n\n即 $x^{2}+(y-1)^{2}=1^{2} ,$\n\n$\\therefore x^{2}=2 y-y^{2}$,\n\n\n\n图2\n根据题意可得: $S_{1}=\\frac{1}{2} \\cdot A D \\cdot(2-y)=2-y$,\n\n$S_{3}=\\frac{1}{2} \\cdot B C \\cdot y=y$\n\n$S_{2}=\\frac{1}{2} \\cdot A B \\cdot x=x$\n\n$$\n\\therefore 2 S_{1} S_{3}-S_{2}^{2}=2 \\cdot(2-y) \\cdot y-x^{2}=4 y-2 y^{2}-x^{2}=x^{2} \\because 0\n\n图3\n\n在 $\\triangle A B E$ 中, $B E=\\sqrt{A B^{2}+A E^{2}}=\\frac{5}{2}$,\n\n易得 $\\triangle B F K \\sim \\triangle B E A$,\n\n即: $\\frac{F K}{B F}=\\frac{A E}{B E}$,\n\n得 $F K=\\frac{A E}{B E} \\cdot B F=\\frac{\\frac{3}{2} \\cdot \\frac{8}{5}}{\\frac{5}{2}}=\\frac{24}{25}$,\n\n根据勾股定理可得, $B K=\\sqrt{B F^{2}-F K^{2}}=\\frac{32}{25}$\n\n$\\therefore F\\left(\\frac{24}{25}, \\frac{32}{25}\\right)$,\n\n$\\because P(1,1)$,\n\n可求得直线 $P F$ 解析式: $y=-7 x+8$,\n\n设 $Q(a,-7 a+8)$ ,\n\n$\\because P Q=P B=\\sqrt{2}$,\n\n$\\therefore \\sqrt{(a-1)^{2}+(-7 a+8-1)^{2}}=\\sqrt{2}$,\n$\\therefore a_{1}=\\frac{4}{5}, a_{2}=\\frac{6}{5}$\n\n$\\because Q$ 在 $F P$ 的延长上,\n\n$\\therefore a>1$,\n\n$\\therefore a=\\frac{6}{5}$\n\n$\\therefore Q$ 点坐标为 $\\left(\\frac{6}{5},-\\frac{2}{5}\\right)$,\n\n$\\because O$ 点坐标为 $(0,1)$,\n\n$\\therefore Q O=\\sqrt{\\left(\\frac{6}{5}-0\\right)^{2}+\\left(-\\frac{2}{5}-1\\right)^{2}}=\\frac{\\sqrt{85}}{5}$."} {"id": "3609", "image": ["4924.jpg", "4925.jpg", "4925.jpg"], "answer": "解: (1)把 $x=2, y=0$ 代入 $y=x^{2}-m x+1$, 得 $m=\\frac{5}{2}$,\n\n$\\therefore$ :抛物线的函数表达式为 $y=x^{2}-\\frac{5}{2} x+1$,\n\n$y=x^{2}-\\frac{5}{2} x+1=\\left(x-\\frac{5}{4}\\right)^{2}-\\frac{9}{16}$\n\n所以顶点坐标为 $\\left(\\frac{5}{4},-\\frac{9}{16}\\right)$.\n\n(2) $\\because y=x^{2}-m x+1=\\left(x-\\frac{m}{2}\\right)^{2}-\\frac{m^{2}}{4}+1$,\n\n$\\therefore n=-\\frac{m^{2}}{4}+1$.\n\n$\\because$ 抛物线 $y=x^{2}-m x+1$ 与 $x$ 轴有两个交点,\n\n$\\therefore \\Delta=m^{2}-4>0$.\n\n$\\therefore m^{2}>4$.\n\n$\\therefore-\\frac{m^{2}}{4}+1<-\\frac{4}{4}+1=0$\n\n即 $n<0$.\n\n(3)如图, 连接 $P C, P D$, 作 $P E \\perp C D$ 于 $E$.\n\n\n\n$\\because P$ 为抛物线的顶点, $C, D$ 均在 $x$ 轴的平行线 $l$ 上, 且 $\\angle C P D=60^{\\circ}$, $\\therefore \\triangle C P D$ 为等边三角形.\n在Rt $\\triangle C P E$ 中, $\\tan \\angle E C P=\\frac{P E}{C E}=\\frac{1-\\left(-\\frac{m}{4}+1\\right)}{\\frac{1}{2}|m|}=\\sqrt{3}$,\n\n即 $m^{2}-2 \\sqrt{3} m=0$ 或 $m^{2}+2 \\sqrt{3} m=0$.\n\n$\\because m \\neq 0, \\quad \\therefore m=2 \\sqrt{3}$ 或 $-2 \\sqrt{3}$.\n\n$\\because$ 对称轴在 $y$ 轴右侧,\n\n$\\therefore m=-2 \\sqrt{3}$ 舍去.\n\n$\\therefore m=2 \\sqrt{3}$.", "solution": "null", "level": "九年级", "question": "(8 分) 如图, 在平面直角坐标系中, 抛物线 $y=x^{2}-m x+1$ 与 $x$ 轴从左到右的交点为 $A, B$,与 $y$ 轴的交点为 $C$, 过点 $C$ 作 $x$ 轴的平行线 $l, l$ 与抛物线的另一交点为 $D$ 点.\n\n\n\n(1)若点 $B$ 的坐标为 $(2,0)$, 求抛物线的函数表达式及顶点坐标;\n\n(2)设抛物线顶点的纵坐标为 $n$, 求 $n$ 的取值范围;\n\n(3)若抛物线顶点记为 $P$, 连接 $C P, D P$, 当 $\\angle C P D=60^{\\circ}$ 时, 求 $m$ 的值.", "options": [], "subject": "解析几何", "analysis": "解: (1)把 $x=2, y=0$ 代入 $y=x^{2}-m x+1$, 得 $m=\\frac{5}{2}$,\n\n$\\therefore$ :抛物线的函数表达式为 $y=x^{2}-\\frac{5}{2} x+1$,\n\n$y=x^{2}-\\frac{5}{2} x+1=\\left(x-\\frac{5}{4}\\right)^{2}-\\frac{9}{16}$\n\n所以顶点坐标为 $\\left(\\frac{5}{4},-\\frac{9}{16}\\right)$.\n\n(2) $\\because y=x^{2}-m x+1=\\left(x-\\frac{m}{2}\\right)^{2}-\\frac{m^{2}}{4}+1$,\n\n$\\therefore n=-\\frac{m^{2}}{4}+1$.\n\n$\\because$ 抛物线 $y=x^{2}-m x+1$ 与 $x$ 轴有两个交点,\n\n$\\therefore \\Delta=m^{2}-4>0$.\n\n$\\therefore m^{2}>4$.\n\n$\\therefore-\\frac{m^{2}}{4}+1<-\\frac{4}{4}+1=0$\n\n即 $n<0$.\n\n(3)如图, 连接 $P C, P D$, 作 $P E \\perp C D$ 于 $E$.\n\n\n\n$\\because P$ 为抛物线的顶点, $C, D$ 均在 $x$ 轴的平行线 $l$ 上, 且 $\\angle C P D=60^{\\circ}$, $\\therefore \\triangle C P D$ 为等边三角形.\n在Rt $\\triangle C P E$ 中, $\\tan \\angle E C P=\\frac{P E}{C E}=\\frac{1-\\left(-\\frac{m}{4}+1\\right)}{\\frac{1}{2}|m|}=\\sqrt{3}$,\n\n即 $m^{2}-2 \\sqrt{3} m=0$ 或 $m^{2}+2 \\sqrt{3} m=0$.\n\n$\\because m \\neq 0, \\quad \\therefore m=2 \\sqrt{3}$ 或 $-2 \\sqrt{3}$.\n\n$\\because$ 对称轴在 $y$ 轴右侧,\n\n$\\therefore m=-2 \\sqrt{3}$ 舍去.\n\n$\\therefore m=2 \\sqrt{3}$."} {"id": "3610", "image": ["4926.jpg", "4927.jpg", "4926.jpg", "4927.jpg"], "answer": "解: (1) $\\because$ 点 $D$ 是直线 $l: y=-x+1$ 上的一个动点, 其横坐标为 $m$,\n\n$\\therefore y=-m+1$,\n\n$\\therefore D(m,-m+1)$,\n\n$\\therefore y=-(x-m)^{2}-m+1=-x^{2}+2 m x-m^{2}-m+1$,\n\n$\\therefore b=2 m, c=-m^{2}-m+1$,\n\n$$\n\\text { (2) (1) } \\because\\left\\{\\begin{array}{l}\ny=-x+1 \\\\\ny=-x^{2}+2 m x-m^{2}-m+1^{\\prime}\n\\end{array}\\right.\n$$\n\n$\\therefore x^{2}-(2 m+1) x+m^{2}+m=0$,\n\n$\\therefore x_{1}=m, x_{2}=m+1$,\n\n$\\therefore E(m+1,-m)$,\n\n(2) $\\because D(m,-m+1), E(m+1,-m)$,\n\n$\\therefore D E=\\sqrt{2}$,\n\n$\\therefore S_{\\triangle O D E}=\\frac{1}{2} \\cdot D E \\cdot \\frac{\\sqrt{2}}{2}=\\frac{1}{2}$,\n\n$\\therefore$ 面积不变, 面积为 $\\frac{1}{2}$;\n\n(3) $\\because$ 顶点 $D$ 在第二象限,\n\n$\\therefore m<0$.\n\n当点 $A$ 在 $y$ 轴的正半轴上,\n\n如图(1)作 $A G \\perp D H$ 于点 $G$,\n\n\n$\\because A\\left(0,-m^{2}-m+1\\right), D(m,-m+1)$,\n\n$\\therefore H(m, 0), G\\left(m,-m^{2}-m+1\\right)$\n\n$\\because \\angle A D H=\\angle A H O$\n\n$\\therefore \\tan \\angle A D H=\\tan \\angle A H O$\n\n$\\therefore \\frac{A G}{D G}=\\frac{A O}{H O}$.\n\n$\\therefore \\frac{-m}{1-m-\\left(-m^{2}-m+1\\right)}=\\frac{-m^{2}-m+1}{-m}$.\n\n整理得: $m^{2}+m=0$.\n\n$\\therefore m=-1$ 或 $m=0$ (舍).\n\n当点 $A$ 在 $y$ 轴的负半轴上, 如图(2). 作 $A G \\perp D H$ 于点 $G$,\n\n\n\n$\\because A\\left(0,-m^{2}-m+1\\right), D(m,-m+1)$,\n\n$\\therefore H(m, 0), G\\left(m,-m^{2}-m+1\\right)$\n\n$\\because \\angle A D H=\\angle A H O$\n\n$\\therefore \\tan \\angle A D H=\\tan \\angle A H O$,\n\n$\\therefore \\frac{A G}{D G}=\\frac{A O}{H O}$.\n\n$\\therefore \\frac{-m}{1-m-\\left(-m^{2}-m+1\\right)}=\\frac{m^{2}+m-1}{-m}$.\n\n整理得: $m^{2}+m-2=0$.\n\n$\\therefore m=-2$ 或 $m=1$ (舍).\n\n综上所述, $m$ 的值为 -1 或 -2 .", "solution": "null", "level": "九年级", "question": "(10 分) 在平面直角坐标系 $x O y$ 中, 点 $D$ 是直线 $l: y=-x+1$ 上的一个动点, 其横坐标为 $m$,以点 $D$ 为顶点的抛物线 $y=-x^{2}+b x+c$ 交 $y$ 轴于点为 $A$, 其对称轴与 $x$ 轴交于点 $H$.\n\n(1)请用含 $m$ 的代数式表示 $b$ 和 $c$;\n\n(2)记直线 $l$ 与抛物线的另一交点为 $E$\n\n(1) 请用含 $m$ 的代数式表示点 $E$ 的坐标;\n\n(2)问: $\\triangle O D E$ 的面积是否会随着 $m$ 的取值变化而变化? 若不变化, 请求出 $\\triangle O D E$ 的面积; 若要变化, 请求出 $\\triangle O D E$ 的面积的最小值.\n\n(3)当抛物线顶点 $D$ 在第二象限时, 若 $\\angle A D H=\\angle A H O$, 求 $m$ 的值.", "options": [], "subject": "解析几何", "analysis": "解: (1) $\\because$ 点 $D$ 是直线 $l: y=-x+1$ 上的一个动点, 其横坐标为 $m$,\n\n$\\therefore y=-m+1$,\n\n$\\therefore D(m,-m+1)$,\n\n$\\therefore y=-(x-m)^{2}-m+1=-x^{2}+2 m x-m^{2}-m+1$,\n\n$\\therefore b=2 m, c=-m^{2}-m+1$,\n\n$$\n\\text { (2) (1) } \\because\\left\\{\\begin{array}{l}\ny=-x+1 \\\\\ny=-x^{2}+2 m x-m^{2}-m+1^{\\prime}\n\\end{array}\\right.\n$$\n\n$\\therefore x^{2}-(2 m+1) x+m^{2}+m=0$,\n\n$\\therefore x_{1}=m, x_{2}=m+1$,\n\n$\\therefore E(m+1,-m)$,\n\n(2) $\\because D(m,-m+1), E(m+1,-m)$,\n\n$\\therefore D E=\\sqrt{2}$,\n\n$\\therefore S_{\\triangle O D E}=\\frac{1}{2} \\cdot D E \\cdot \\frac{\\sqrt{2}}{2}=\\frac{1}{2}$,\n\n$\\therefore$ 面积不变, 面积为 $\\frac{1}{2}$;\n\n(3) $\\because$ 顶点 $D$ 在第二象限,\n\n$\\therefore m<0$.\n\n当点 $A$ 在 $y$ 轴的正半轴上,\n\n如图(1)作 $A G \\perp D H$ 于点 $G$,\n\n\n$\\because A\\left(0,-m^{2}-m+1\\right), D(m,-m+1)$,\n\n$\\therefore H(m, 0), G\\left(m,-m^{2}-m+1\\right)$\n\n$\\because \\angle A D H=\\angle A H O$\n\n$\\therefore \\tan \\angle A D H=\\tan \\angle A H O$\n\n$\\therefore \\frac{A G}{D G}=\\frac{A O}{H O}$.\n\n$\\therefore \\frac{-m}{1-m-\\left(-m^{2}-m+1\\right)}=\\frac{-m^{2}-m+1}{-m}$.\n\n整理得: $m^{2}+m=0$.\n\n$\\therefore m=-1$ 或 $m=0$ (舍).\n\n当点 $A$ 在 $y$ 轴的负半轴上, 如图(2). 作 $A G \\perp D H$ 于点 $G$,\n\n\n\n$\\because A\\left(0,-m^{2}-m+1\\right), D(m,-m+1)$,\n\n$\\therefore H(m, 0), G\\left(m,-m^{2}-m+1\\right)$\n\n$\\because \\angle A D H=\\angle A H O$\n\n$\\therefore \\tan \\angle A D H=\\tan \\angle A H O$,\n\n$\\therefore \\frac{A G}{D G}=\\frac{A O}{H O}$.\n\n$\\therefore \\frac{-m}{1-m-\\left(-m^{2}-m+1\\right)}=\\frac{m^{2}+m-1}{-m}$.\n\n整理得: $m^{2}+m-2=0$.\n\n$\\therefore m=-2$ 或 $m=1$ (舍).\n\n综上所述, $m$ 的值为 -1 或 -2 ."} {"id": "3634", "image": ["4974.jpg", "4975.jpg", "4975.jpg"], "answer": "解: $(1) P_{1}$ 与 $P_{2}$ 重合.\n\n证明: 由题意得 $A C=A D$,\n\n$\\because A F_{1}-A F_{2}=2 a$,\n\n$\\therefore C F_{1}-D F_{2}=2 a$\n\n又 $\\because F_{1} C=F_{1} P_{1} F_{2} D=F_{2} P_{1}$,\n\n$\\therefore P_{1} F_{1}-P_{1} F_{2}=2 a$,\n\n同理 $P_{2} F_{1}-P_{2} F_{2}=2 a$,\n\n$\\therefore P_{1}$ 与 $P_{2}$ 重合.\n\n(2)由(1)知: $M P_{1} \\perp F_{1} F_{2}, N P_{2} \\perp F_{1} F_{2}, P_{1}, P_{2}$ 重合.\n\n$\\therefore M, P_{1}, N$ 共线, 且 $M N \\perp F_{1} F_{2}$,\n\n连接 $M N, N E, M D$, 则 $\\angle N E D=\\angle M D E=90^{\\circ}$\n\n过 $N$ 作 $N H \\perp M D, H$ 为垂足;\n\n$\\because \\angle M P_{1} F_{2}=\\angle M D F_{2}=90^{\\circ}, \\angle H M N=\\angle B F_{2} F_{1}$,\n\n\n\n$\\therefore \\sin \\angle H M N=\\sin \\angle B F_{2} F_{1}=\\frac{8}{9}$,\n\n又 $M N=\\frac{9}{2} ,$\n\n$\\therefore N H=M N \\sin \\angle H M N=4$,\n\n$\\therefore E D=4$\n而 $D F_{2}=F_{2} P_{1}=F_{2} E$,\n\n$\\therefore F_{2} P_{1}=2$,\n\n又由(1) $P_{1} F_{1}-P_{1} F_{2}=2 a ,$\n\n$\\therefore P_{1} F_{1}=2+2 a$,\n\n$\\therefore P_{1} F_{1}+P_{1} F_{2}=2+2+2 a=2 \\sqrt{a^{2}+20}$,\n\n解得 $a=4$.", "solution": "null", "level": "九年级", "question": "如图: 已知 $a$ 为常数, $F_{1}\\left(-\\sqrt{a^{2}+20}, 0\\right), F_{2}\\left(\\sqrt{a^{2}+20}, 0\\right)$, 过 $F_{2}$ 作直线 $l$, 点 $A, B$ 在直线 $l$ 上,且满足 $A F_{1}-A F_{2}=B F_{1}-B F_{2}=2 a, M, N$ 分别为 $\\triangle A F_{1} F_{2}, \\triangle B F_{1} F_{2}$ 的内切圆的圆心.\n\n(1)设 $\\odot M$ 与 $F_{1} F_{2}$ 相切于点 $P_{1}, \\odot N$ 与 $F_{1} F_{2}$ 切于点 $P_{2}$, 试判断 $P_{1}$ 与 $P_{2}$ 的位置关系, 并加以证明;\n\n\n\n(2)已知 $\\sin \\angle B F_{2} F_{1}=\\frac{8}{9}$, 且 $M N=\\frac{9}{2}$, 试求 $a$ 的值.", "options": [], "subject": "解析几何", "analysis": "解: $(1) P_{1}$ 与 $P_{2}$ 重合.\n\n证明: 由题意得 $A C=A D$,\n\n$\\because A F_{1}-A F_{2}=2 a$,\n\n$\\therefore C F_{1}-D F_{2}=2 a$\n\n又 $\\because F_{1} C=F_{1} P_{1} F_{2} D=F_{2} P_{1}$,\n\n$\\therefore P_{1} F_{1}-P_{1} F_{2}=2 a$,\n\n同理 $P_{2} F_{1}-P_{2} F_{2}=2 a$,\n\n$\\therefore P_{1}$ 与 $P_{2}$ 重合.\n\n(2)由(1)知: $M P_{1} \\perp F_{1} F_{2}, N P_{2} \\perp F_{1} F_{2}, P_{1}, P_{2}$ 重合.\n\n$\\therefore M, P_{1}, N$ 共线, 且 $M N \\perp F_{1} F_{2}$,\n\n连接 $M N, N E, M D$, 则 $\\angle N E D=\\angle M D E=90^{\\circ}$\n\n过 $N$ 作 $N H \\perp M D, H$ 为垂足;\n\n$\\because \\angle M P_{1} F_{2}=\\angle M D F_{2}=90^{\\circ}, \\angle H M N=\\angle B F_{2} F_{1}$,\n\n\n\n$\\therefore \\sin \\angle H M N=\\sin \\angle B F_{2} F_{1}=\\frac{8}{9}$,\n\n又 $M N=\\frac{9}{2} ,$\n\n$\\therefore N H=M N \\sin \\angle H M N=4$,\n\n$\\therefore E D=4$\n而 $D F_{2}=F_{2} P_{1}=F_{2} E$,\n\n$\\therefore F_{2} P_{1}=2$,\n\n又由(1) $P_{1} F_{1}-P_{1} F_{2}=2 a ,$\n\n$\\therefore P_{1} F_{1}=2+2 a$,\n\n$\\therefore P_{1} F_{1}+P_{1} F_{2}=2+2+2 a=2 \\sqrt{a^{2}+20}$,\n\n解得 $a=4$."} {"id": "3658", "image": ["5018.jpg", "5019.jpg", "5020.jpg", "5019.jpg", "5020.jpg"], "answer": "解:(1)如图, 点 $O$ 和 $F H$ 为所作;\n\n\n\n(2) $B M=B D=2 \\times 1.5=3 m , G D=1.2 m , D F=1.5 \\times 1.5 \\times 2=4.5 m$ ,设 $A B=C D=E F=a$ ,作 $O K \\perp M N$ 于 $K$, 如图,\n\n\n\n$\\because A B / / O K$,\n\n$\\therefore \\triangle M A B \\sim \\triangle M O K$\n\n$\\therefore \\frac{A B}{O K}=\\frac{M B}{M K}$, 即 $\\frac{a}{O K}=\\frac{3}{6+D K}$ (1),\n\n$\\because C D / / O K$\n\n$\\therefore \\triangle G C D \\sim \\triangle G O K$,\n\n$\\therefore \\frac{C D}{O K}=\\frac{G D}{G K}$ ,即 $\\frac{a}{O K}=\\frac{1.2}{1.2+D K}$ (2),\n\n由 (1) (2)得 $\\frac{3}{6+D K}=\\frac{1.2}{1.2+D K}$, 解得 $D K=2$,\n$\\therefore \\frac{a}{O K}=\\frac{3}{6+2}=\\frac{3}{8}, \\quad F K=D F-D K=4.5-2=2.5$,\n\n$\\because E F / / O K$\n\n$\\therefore \\triangle H E F \\sim \\triangle H O K$\n\n$\\therefore \\frac{a}{O K}=\\frac{H F}{H K}, \\quad$ 即 $\\frac{H F}{H F+2.5}=\\frac{3}{8}$,\n\n$\\therefore H F=1.5(m)$.\n\n答:小明到达点 $F$ 时的影长 $F H$ 的长为 $1.5 m$.", "solution": "null", "level": "九年级", "question": "某兴趣小组开展课外活动. 如图, 小明从点 $M$ 出发以 1.5 米/秒的速度, 沿射线 $M N$ 方向匀速前进, 2 秒后到达点 $B$, 此时他 $(A B)$ 在某一灯光下的影长为 $M B$, 继续按原速行走 2 秒到达点 $D$,此时他 $(C D)$ 在同一灯光下的影子 $G D$ 仍落在其身后, 并测得这个影长 $G D$ 为 1.2 米, 然后他将速度提高到原来的 1.5 倍, 再行走 2 秒到达点 $F$, 此时点 $A, C, E$ 三点共线.\n\n\n\n(1) 请在图中画出光源 $O$ 点的位置, 并画出小明位于点 $F$ 时在这个灯光下的影长 $F H$ (不写画法);\n\n(2)求小明到达点 $F$ 时的影长 $F H$ 的长.", "options": [], "subject": "解析几何", "analysis": "解:(1)如图, 点 $O$ 和 $F H$ 为所作;\n\n\n\n(2) $B M=B D=2 \\times 1.5=3 m , G D=1.2 m , D F=1.5 \\times 1.5 \\times 2=4.5 m$ ,设 $A B=C D=E F=a$ ,作 $O K \\perp M N$ 于 $K$, 如图,\n\n\n\n$\\because A B / / O K$,\n\n$\\therefore \\triangle M A B \\sim \\triangle M O K$\n\n$\\therefore \\frac{A B}{O K}=\\frac{M B}{M K}$, 即 $\\frac{a}{O K}=\\frac{3}{6+D K}$ (1),\n\n$\\because C D / / O K$\n\n$\\therefore \\triangle G C D \\sim \\triangle G O K$,\n\n$\\therefore \\frac{C D}{O K}=\\frac{G D}{G K}$ ,即 $\\frac{a}{O K}=\\frac{1.2}{1.2+D K}$ (2),\n\n由 (1) (2)得 $\\frac{3}{6+D K}=\\frac{1.2}{1.2+D K}$, 解得 $D K=2$,\n$\\therefore \\frac{a}{O K}=\\frac{3}{6+2}=\\frac{3}{8}, \\quad F K=D F-D K=4.5-2=2.5$,\n\n$\\because E F / / O K$\n\n$\\therefore \\triangle H E F \\sim \\triangle H O K$\n\n$\\therefore \\frac{a}{O K}=\\frac{H F}{H K}, \\quad$ 即 $\\frac{H F}{H F+2.5}=\\frac{3}{8}$,\n\n$\\therefore H F=1.5(m)$.\n\n答:小明到达点 $F$ 时的影长 $F H$ 的长为 $1.5 m$."} {"id": "3881", "image": ["5458.jpg", "5459.jpg", "5459.jpg"], "answer": "(1) 见解析 (2) 图中阴影部分的面积为 $2 \\sqrt{3}-\\frac{2}{3} \\pi$.\n\n【解析】(1) 证明: 连接 $O C$.\n\n\n\n$\\because A C=C D, \\angle A C D=120^{\\circ}, \\therefore \\angle A=\\angle D=30^{\\circ} . \\because O A=O C, \\therefore \\angle 2=\\angle A=30^{\\circ} . \\quad \\therefore \\angle O C D=\\angle A C D$ $-\\angle 2=90^{\\circ}$, 即 $O C \\perp C D, \\therefore C D$ 是 $\\odot O$ 的切线;\n\n(2) 解: $\\angle 1=\\angle 2+\\angle A=60^{\\circ} . \\therefore \\mathrm{S}_{\\text {闻现 } B O C}=\\frac{60 \\pi \\times 2^{2}}{360}=\\frac{2 \\pi}{3}$.\n\n在 Rt $\\triangle O C D$ 中, $\\angle D=30^{\\circ}, \\therefore O D=2 O C=4$,\n\n$\\therefore C D=\\sqrt{O D^{2}-O C^{2}}=2 \\sqrt{3} . \\quad \\therefore \\mathrm{S}_{\\mathrm{R} \\leftarrow \\triangle C D}=\\frac{1}{2} O C \\times C D=\\frac{1}{2} \\times 2 \\times 2 \\sqrt{3}=2 \\sqrt{3}$.\n\n$\\therefore$ 图中阴影部分的面积为: $2 \\sqrt{3}-\\frac{2 \\pi}{3}$.", "solution": "null", "level": "九年级", "question": "如图, 点 $D$ 在 $\\odot O$ 的直径 $A B$ 的延长线上, 点 $C$ 在 $\\odot O$ 上, 且 $A C=C D, \\angle A C D=120^{\\circ}$.\n\n(1) 求证: $C D$ 是 $\\odot O$ 的切线;\n\n(2) 若 $\\odot O$ 的半径为 2 , 求图中阴影部分的面积.\n\n", "options": [], "subject": "解析几何", "analysis": "(1) 见解析 (2) 图中阴影部分的面积为 $2 \\sqrt{3}-\\frac{2}{3} \\pi$.\n\n【解析】(1) 证明: 连接 $O C$.\n\n\n\n$\\because A C=C D, \\angle A C D=120^{\\circ}, \\therefore \\angle A=\\angle D=30^{\\circ} . \\because O A=O C, \\therefore \\angle 2=\\angle A=30^{\\circ} . \\quad \\therefore \\angle O C D=\\angle A C D$ $-\\angle 2=90^{\\circ}$, 即 $O C \\perp C D, \\therefore C D$ 是 $\\odot O$ 的切线;\n\n(2) 解: $\\angle 1=\\angle 2+\\angle A=60^{\\circ} . \\therefore \\mathrm{S}_{\\text {闻现 } B O C}=\\frac{60 \\pi \\times 2^{2}}{360}=\\frac{2 \\pi}{3}$.\n\n在 Rt $\\triangle O C D$ 中, $\\angle D=30^{\\circ}, \\therefore O D=2 O C=4$,\n\n$\\therefore C D=\\sqrt{O D^{2}-O C^{2}}=2 \\sqrt{3} . \\quad \\therefore \\mathrm{S}_{\\mathrm{R} \\leftarrow \\triangle C D}=\\frac{1}{2} O C \\times C D=\\frac{1}{2} \\times 2 \\times 2 \\sqrt{3}=2 \\sqrt{3}$.\n\n$\\therefore$ 图中阴影部分的面积为: $2 \\sqrt{3}-\\frac{2 \\pi}{3}$."} {"id": "3882", "image": ["5460.jpg", "5461.jpg", "5461.jpg"], "answer": "(1) 证明见解析; (2) $\\mathrm{CE}=4$.\n\n【解析】 (1) 证明:如图, 连接 $\\mathrm{OE}$,\n\n\n\n$\\because \\mathrm{OB}=\\mathrm{OE}, \\quad \\therefore \\angle \\mathrm{OBE}=\\angle \\mathrm{OEB}, \\because \\mathrm{BE}$ 平分 $\\angle \\mathrm{ABC} . \\quad \\therefore \\angle \\mathrm{OBE}=\\angle \\mathrm{EBC}, \\therefore \\angle \\mathrm{OEB}=\\angle \\mathrm{EBC}$,\n\n$\\therefore \\mathrm{OE} / / \\mathrm{BC}, \\because \\angle \\mathrm{ACB}=90^{\\circ}, \\therefore \\angle \\mathrm{OEA}=\\angle \\mathrm{ACB}=90^{\\circ}, \\therefore \\mathrm{AC}$ 是 $\\odot \\mathrm{O}$ 的切线\n\n(2) 解:过 $\\mathrm{O}$ 作 $\\mathrm{OH} \\perp \\mathrm{BF}$,\n\n$\\therefore \\mathrm{BH}=\\frac{1}{2} \\mathrm{BF}=3$, 四边形 $\\mathrm{OHCE}$ 是矩形, $\\therefore \\mathrm{CE}=\\mathrm{OH}$, 在 $\\mathrm{Rt} \\triangle \\mathrm{OBH}$ 中, $\\mathrm{BH}=3, \\mathrm{OB}=5$,\n\n$\\therefore \\mathrm{OH}=\\sqrt{O B^{2}-O H^{2}}=4, \\quad \\therefore \\mathrm{CE}=4$.", "solution": "null", "level": "九年级", "question": "在 Rt $\\triangle A B C$ 中, $\\angle A C B=90^{\\circ}, B E$ 平分 $\\angle A B C, D$ 是边 $A B$ 上一点, 以 $B D$ 为直径的 $\\odot O$ 经过点 $E$, 且交 $B C$ 于点 $F$.\n\n(1)求证: $A C$ 是 $\\odot O$ 的切线;\n\n(2)若 $B F=6, \\odot O$ 的半径为 5 , 求 $C E$ 的长.\n\n", "options": [], "subject": "解析几何", "analysis": "(1) 证明见解析; (2) $\\mathrm{CE}=4$.\n\n【解析】 (1) 证明:如图, 连接 $\\mathrm{OE}$,\n\n\n\n$\\because \\mathrm{OB}=\\mathrm{OE}, \\quad \\therefore \\angle \\mathrm{OBE}=\\angle \\mathrm{OEB}, \\because \\mathrm{BE}$ 平分 $\\angle \\mathrm{ABC} . \\quad \\therefore \\angle \\mathrm{OBE}=\\angle \\mathrm{EBC}, \\therefore \\angle \\mathrm{OEB}=\\angle \\mathrm{EBC}$,\n\n$\\therefore \\mathrm{OE} / / \\mathrm{BC}, \\because \\angle \\mathrm{ACB}=90^{\\circ}, \\therefore \\angle \\mathrm{OEA}=\\angle \\mathrm{ACB}=90^{\\circ}, \\therefore \\mathrm{AC}$ 是 $\\odot \\mathrm{O}$ 的切线\n\n(2) 解:过 $\\mathrm{O}$ 作 $\\mathrm{OH} \\perp \\mathrm{BF}$,\n\n$\\therefore \\mathrm{BH}=\\frac{1}{2} \\mathrm{BF}=3$, 四边形 $\\mathrm{OHCE}$ 是矩形, $\\therefore \\mathrm{CE}=\\mathrm{OH}$, 在 $\\mathrm{Rt} \\triangle \\mathrm{OBH}$ 中, $\\mathrm{BH}=3, \\mathrm{OB}=5$,\n\n$\\therefore \\mathrm{OH}=\\sqrt{O B^{2}-O H^{2}}=4, \\quad \\therefore \\mathrm{CE}=4$."} {"id": "3886", "image": ["5469.jpg"], "answer": "(1)证明见解析; (2) $A C=12$.\n\n【解析】(1) $\\because \\quad \\angle \\mathrm{BDC}=\\angle \\mathrm{E}$ (已知 $), \\angle \\mathrm{BDC}=\\angle \\mathrm{CAB}$\n\n$\\therefore \\quad \\angle \\mathrm{E}=\\angle \\mathrm{CAB}, \\because \\quad \\mathrm{EF} \\perp \\mathrm{AC}$ 于点 $\\mathrm{F}$, 即 $\\angle \\mathrm{AFE}=90^{\\circ}$\n\n$\\therefore \\quad \\angle \\mathrm{E}+\\angle \\mathrm{EAF}=90^{\\circ} \\therefore \\quad \\angle \\mathrm{CAB}+\\angle \\mathrm{EAF}=90^{\\circ}$,\n\n即 $\\mathrm{AE} \\perp \\mathrm{AB}$, 且 $\\mathrm{AB}$ 是圆的直径.$\\therefore \\mathrm{AE}$ 是圆 $O$ 的切线;\n\n(2)解: $\\because \\quad \\angle \\mathrm{E}=\\angle \\mathrm{BDC}$,\n\n$\\therefore \\quad \\sin \\angle \\mathrm{E}=\\sin \\angle \\mathrm{BDC}=\\frac{3}{5} \\therefore \\quad$ Rt $\\triangle \\mathrm{AEF}$ 中, $\\sin \\angle \\mathrm{E}=\\frac{\\mathrm{AF}}{\\mathrm{AE}}$, 且 $\\mathrm{AE}=10$\n\n$\\therefore \\quad \\mathrm{AF}=6, \\because \\quad \\mathrm{OF} \\perp \\mathrm{AC}$ 于点 $\\mathrm{F}, \\therefore \\quad \\mathrm{AC}=2 \\mathrm{AF}=12$ (垂径定理).", "solution": "null", "level": "九年级", "question": "如图, $\\mathrm{AB}$ 是 $\\odot \\mathrm{O}$ 的直径, 过 $O$ 作弦 $\\mathrm{AC}$ 的垂线, 交 $\\odot \\mathrm{O}$ 于点 $\\mathrm{D}$, 分别交 $\\mathrm{AE} 、 \\mathrm{AC}$ 于点 $\\mathrm{E}$ 、点 $\\mathrm{F}$, 已知 $\\angle \\mathrm{BDC}=\\angle \\mathrm{E}$.\n\n(1)判断 $\\mathrm{AE}$ 与 $\\odot \\mathrm{O}$ 的位置关系, 并说明理由;\n\n(2)若 $\\mathrm{AE}=10, \\sin \\angle \\mathrm{BDC}=\\frac{3}{5}$, 求 $\\mathrm{AC}$ 的长.\n\n", "options": [], "subject": "解析几何", "analysis": "(1)证明见解析; (2) $A C=12$.\n\n【解析】(1) $\\because \\quad \\angle \\mathrm{BDC}=\\angle \\mathrm{E}$ (已知 $), \\angle \\mathrm{BDC}=\\angle \\mathrm{CAB}$\n\n$\\therefore \\quad \\angle \\mathrm{E}=\\angle \\mathrm{CAB}, \\because \\quad \\mathrm{EF} \\perp \\mathrm{AC}$ 于点 $\\mathrm{F}$, 即 $\\angle \\mathrm{AFE}=90^{\\circ}$\n\n$\\therefore \\quad \\angle \\mathrm{E}+\\angle \\mathrm{EAF}=90^{\\circ} \\therefore \\quad \\angle \\mathrm{CAB}+\\angle \\mathrm{EAF}=90^{\\circ}$,\n\n即 $\\mathrm{AE} \\perp \\mathrm{AB}$, 且 $\\mathrm{AB}$ 是圆的直径.$\\therefore \\mathrm{AE}$ 是圆 $O$ 的切线;\n\n(2)解: $\\because \\quad \\angle \\mathrm{E}=\\angle \\mathrm{BDC}$,\n\n$\\therefore \\quad \\sin \\angle \\mathrm{E}=\\sin \\angle \\mathrm{BDC}=\\frac{3}{5} \\therefore \\quad$ Rt $\\triangle \\mathrm{AEF}$ 中, $\\sin \\angle \\mathrm{E}=\\frac{\\mathrm{AF}}{\\mathrm{AE}}$, 且 $\\mathrm{AE}=10$\n\n$\\therefore \\quad \\mathrm{AF}=6, \\because \\quad \\mathrm{OF} \\perp \\mathrm{AC}$ 于点 $\\mathrm{F}, \\therefore \\quad \\mathrm{AC}=2 \\mathrm{AF}=12$ (垂径定理)."} {"id": "3911", "image": ["5514.jpg", "5515.jpg", "5516.jpg", "5517.jpg", "5515.jpg", "5516.jpg", "5517.jpg"], "answer": "(1)证明见解析; (2) $\\mathrm{AC} / / \\mathrm{EF}$, 证明见解析;(3) $\\mathrm{FG}=\\frac{25 \\sqrt{2}}{8}$.\n\n【解析】 (1) 如图 1 , 连接 $\\mathrm{OG}$.\n\n\n\n图1\n\n$\\because \\mathrm{EG}$ 为切线, $\\therefore \\angle \\mathrm{KGE}+\\angle \\mathrm{OGA}=90^{\\circ}$,\n\n$\\because \\mathrm{CD} \\perp \\mathrm{AB}, \\quad \\therefore \\angle \\mathrm{AKH}+\\angle \\mathrm{OAG}=90^{\\circ}$,\n\n又 $\\because \\mathrm{OA}=\\mathrm{OG}, \\therefore \\angle \\mathrm{OGA}=\\angle \\mathrm{OAG}$,\n\n$\\therefore \\angle \\mathrm{KGE}=\\angle \\mathrm{AKH}=\\angle \\mathrm{GKE}, \\quad \\therefore \\mathrm{KE}=\\mathrm{GE}$.\n\n(2) $\\mathrm{AC} / / \\mathrm{EF}$, 理由为连接 $\\mathrm{GD}$, 如图 2 所示.\n\n\n\n图2\n\n$\\because \\mathrm{KG}^{2}=\\mathrm{KD} \\cdot \\mathrm{GE}$, 即 $^{\\frac{K G}{K D}=\\frac{G E}{K G}}, \\therefore^{\\frac{K G}{G E}=\\frac{K D}{K G}}$,\n\n又 $\\because \\angle \\mathrm{KGE}=\\angle \\mathrm{GKE}, \\therefore \\triangle \\mathrm{GKD} \\sim \\triangle \\mathrm{EGK}$,\n\n$\\therefore \\angle \\mathrm{E}=\\angle \\mathrm{AGD}$, 又 $\\because \\angle \\mathrm{C}=\\angle \\mathrm{AGD}$,\n\n$\\therefore \\angle \\mathrm{E}=\\angle \\mathrm{C}, \\quad \\therefore \\mathrm{AC} / / \\mathrm{EF}$\n\n(3) 连接 $O G, O C$, 如图 3 所示,\n\n\n\n图3\n\n$\\because \\mathrm{EG}$ 为切线, $\\therefore \\angle \\mathrm{KGE}+\\angle \\mathrm{OGA}=90^{\\circ}$,\n\n$\\because \\mathrm{CD} \\perp \\mathrm{AB}, \\quad \\therefore \\angle \\mathrm{AKH}+\\angle \\mathrm{OAG}=90^{\\circ}$,\n\n又 $\\because \\mathrm{OA}=\\mathrm{OG}, \\quad \\therefore \\angle \\mathrm{OGA}=\\angle \\mathrm{OAG}$,\n\n$\\therefore \\angle \\mathrm{KGE}=\\angle \\mathrm{AKH}=\\angle \\mathrm{GKE}$,\n\n$\\therefore \\mathrm{KE}=$ GE. $\\because \\operatorname{sinE}=\\sin \\angle \\mathrm{ACH}=\\frac{3}{5}$\n\n, 设 $\\mathrm{AH}=3 \\mathrm{t}$, 则 $\\mathrm{AC}=5 \\mathrm{t}, \\mathrm{CH}=4 \\mathrm{t}, \\because \\mathrm{KE}=\\mathrm{GE}, \\mathrm{AC} / / \\mathrm{EF}$,\n\n$\\therefore \\mathrm{CK}=\\mathrm{AC}=5 \\mathrm{t}, \\quad \\therefore \\mathrm{HK}=\\mathrm{CK}-\\mathrm{CH}=\\mathrm{t}$.\n\n在 Rt $\\triangle \\mathrm{AHK}$ 中, 根据勾股定理得 $\\mathrm{AH} \\mathrm{H}^{2}+\\mathrm{HK}^{2}=\\mathrm{AK} \\mathrm{K}^{2}$,\n\n即 $(3 \\mathrm{t})^{2}+\\mathrm{t}^{2}=\\left(2^{\\sqrt{5}}\\right)^{2}$, 解得 $\\mathrm{t}={ }^{\\sqrt{2}}$.\n\n设 $\\odot \\mathrm{O}$ 半径为 $\\mathrm{r}$, 在 Rt $\\triangle \\mathrm{OCH}$ 中, $\\mathrm{OC}=\\mathrm{r}, \\mathrm{OH}=\\mathrm{r}-3 \\mathrm{t}, \\mathrm{CH}=4 \\mathrm{t}$,\n\n由勾股定理得: $\\mathrm{OH}^{2}+\\mathrm{CH}^{2}=\\mathrm{OC}^{2}$,\n即 $(r-3 t)^{2}+(4 t)^{2}=r^{2}$, 解得 $r={ }^{\\frac{25}{6}} t={ }^{\\frac{25}{6}} \\sqrt{2} . \\because E F$ 为切线,\n\n$\\therefore \\triangle \\mathrm{OGF}$ 为直角三角形, 在 Rt $\\triangle \\mathrm{OGF}$ 中, $\\mathrm{OG}=\\mathrm{r}=\\frac{25}{6} \\sqrt{2}, \\tan \\angle \\mathrm{OFG}=\\tan \\angle \\mathrm{CAH}=\\frac{\\mathrm{CH}}{\\mathrm{AH}}=\\frac{4}{3}$,\n\n$\\therefore F G=\\frac{O G}{\\tan \\angle O F G}=\\frac{\\frac{25}{6} \\sqrt{2}}{\\frac{7}{3}}=\\frac{25}{8} \\sqrt{2}$", "solution": "null", "level": "九年级", "question": "如图, $\\mathrm{AB}$ 是 $\\odot \\mathrm{O}$ 的直径, 弦 $\\mathrm{CD} \\perp \\mathrm{AB}$ 于 $\\mathrm{H}$, 过 $\\mathrm{CD}$ 延长线上一点 $\\mathrm{E}$ 作 $\\odot \\mathrm{O}$ 的切线交 $\\mathrm{AB}$ 的延长线于切点为 $G$, 连接 $A G$ 交 $C D$ 于 $K$.\n\n(1) 求证: $K E=G E$;\n\n(2) 若 $\\mathrm{KG}^{2}=\\mathrm{KD} \\cdot \\mathrm{GE}$, 试判断 $\\mathrm{AC}$ 与 $\\mathrm{EF}$ 的位置关系, 并说明理由;\n\n(3) 在(2)的条件下, 若 $\\sin E=\\frac{3}{5}, \\quad A K=2 \\sqrt{5}$, 求 $\\mathrm{FG}$ 的长.\n\n", "options": [], "subject": "解析几何", "analysis": "(1)证明见解析; (2) $\\mathrm{AC} / / \\mathrm{EF}$, 证明见解析;(3) $\\mathrm{FG}=\\frac{25 \\sqrt{2}}{8}$.\n\n【解析】 (1) 如图 1 , 连接 $\\mathrm{OG}$.\n\n\n\n图1\n\n$\\because \\mathrm{EG}$ 为切线, $\\therefore \\angle \\mathrm{KGE}+\\angle \\mathrm{OGA}=90^{\\circ}$,\n\n$\\because \\mathrm{CD} \\perp \\mathrm{AB}, \\quad \\therefore \\angle \\mathrm{AKH}+\\angle \\mathrm{OAG}=90^{\\circ}$,\n\n又 $\\because \\mathrm{OA}=\\mathrm{OG}, \\therefore \\angle \\mathrm{OGA}=\\angle \\mathrm{OAG}$,\n\n$\\therefore \\angle \\mathrm{KGE}=\\angle \\mathrm{AKH}=\\angle \\mathrm{GKE}, \\quad \\therefore \\mathrm{KE}=\\mathrm{GE}$.\n\n(2) $\\mathrm{AC} / / \\mathrm{EF}$, 理由为连接 $\\mathrm{GD}$, 如图 2 所示.\n\n\n\n图2\n\n$\\because \\mathrm{KG}^{2}=\\mathrm{KD} \\cdot \\mathrm{GE}$, 即 $^{\\frac{K G}{K D}=\\frac{G E}{K G}}, \\therefore^{\\frac{K G}{G E}=\\frac{K D}{K G}}$,\n\n又 $\\because \\angle \\mathrm{KGE}=\\angle \\mathrm{GKE}, \\therefore \\triangle \\mathrm{GKD} \\sim \\triangle \\mathrm{EGK}$,\n\n$\\therefore \\angle \\mathrm{E}=\\angle \\mathrm{AGD}$, 又 $\\because \\angle \\mathrm{C}=\\angle \\mathrm{AGD}$,\n\n$\\therefore \\angle \\mathrm{E}=\\angle \\mathrm{C}, \\quad \\therefore \\mathrm{AC} / / \\mathrm{EF}$\n\n(3) 连接 $O G, O C$, 如图 3 所示,\n\n\n\n图3\n\n$\\because \\mathrm{EG}$ 为切线, $\\therefore \\angle \\mathrm{KGE}+\\angle \\mathrm{OGA}=90^{\\circ}$,\n\n$\\because \\mathrm{CD} \\perp \\mathrm{AB}, \\quad \\therefore \\angle \\mathrm{AKH}+\\angle \\mathrm{OAG}=90^{\\circ}$,\n\n又 $\\because \\mathrm{OA}=\\mathrm{OG}, \\quad \\therefore \\angle \\mathrm{OGA}=\\angle \\mathrm{OAG}$,\n\n$\\therefore \\angle \\mathrm{KGE}=\\angle \\mathrm{AKH}=\\angle \\mathrm{GKE}$,\n\n$\\therefore \\mathrm{KE}=$ GE. $\\because \\operatorname{sinE}=\\sin \\angle \\mathrm{ACH}=\\frac{3}{5}$\n\n, 设 $\\mathrm{AH}=3 \\mathrm{t}$, 则 $\\mathrm{AC}=5 \\mathrm{t}, \\mathrm{CH}=4 \\mathrm{t}, \\because \\mathrm{KE}=\\mathrm{GE}, \\mathrm{AC} / / \\mathrm{EF}$,\n\n$\\therefore \\mathrm{CK}=\\mathrm{AC}=5 \\mathrm{t}, \\quad \\therefore \\mathrm{HK}=\\mathrm{CK}-\\mathrm{CH}=\\mathrm{t}$.\n\n在 Rt $\\triangle \\mathrm{AHK}$ 中, 根据勾股定理得 $\\mathrm{AH} \\mathrm{H}^{2}+\\mathrm{HK}^{2}=\\mathrm{AK} \\mathrm{K}^{2}$,\n\n即 $(3 \\mathrm{t})^{2}+\\mathrm{t}^{2}=\\left(2^{\\sqrt{5}}\\right)^{2}$, 解得 $\\mathrm{t}={ }^{\\sqrt{2}}$.\n\n设 $\\odot \\mathrm{O}$ 半径为 $\\mathrm{r}$, 在 Rt $\\triangle \\mathrm{OCH}$ 中, $\\mathrm{OC}=\\mathrm{r}, \\mathrm{OH}=\\mathrm{r}-3 \\mathrm{t}, \\mathrm{CH}=4 \\mathrm{t}$,\n\n由勾股定理得: $\\mathrm{OH}^{2}+\\mathrm{CH}^{2}=\\mathrm{OC}^{2}$,\n即 $(r-3 t)^{2}+(4 t)^{2}=r^{2}$, 解得 $r={ }^{\\frac{25}{6}} t={ }^{\\frac{25}{6}} \\sqrt{2} . \\because E F$ 为切线,\n\n$\\therefore \\triangle \\mathrm{OGF}$ 为直角三角形, 在 Rt $\\triangle \\mathrm{OGF}$ 中, $\\mathrm{OG}=\\mathrm{r}=\\frac{25}{6} \\sqrt{2}, \\tan \\angle \\mathrm{OFG}=\\tan \\angle \\mathrm{CAH}=\\frac{\\mathrm{CH}}{\\mathrm{AH}}=\\frac{4}{3}$,\n\n$\\therefore F G=\\frac{O G}{\\tan \\angle O F G}=\\frac{\\frac{25}{6} \\sqrt{2}}{\\frac{7}{3}}=\\frac{25}{8} \\sqrt{2}$"} {"id": "3928", "image": ["5535.jpg", "5536.jpg", "5537.jpg", "5538.jpg", "5539.jpg", "5537.jpg", "5538.jpg", "5539.jpg"], "answer": "(1) 3; (2) $\\angle \\mathrm{DEF}$ 的大小不变, $\\tan \\angle \\mathrm{DEF}=\\frac{3}{4}$; (3) $\\frac{75}{41}$ 或 $\\frac{75}{17}$.\n\n【解析】(1) 当 $\\mathrm{t}=3$ 时, 点 $\\mathrm{E}$ 为 $\\mathrm{AB}$ 的中点,\n\n$\\because \\mathrm{A}(8,0), \\mathrm{C}(0,6), \\therefore \\mathrm{OA}=8, \\mathrm{OC}=6, \\because$ 点 $\\mathrm{D}$ 为 $\\mathrm{OB}$ 的中点,\n\n$\\therefore \\mathrm{DE} / / \\mathrm{OA}, \\mathrm{DE}=\\frac{1}{2} \\mathrm{OA}=4, \\because$ 四边形 $\\mathrm{OABC}$ 是矩形,\n\n$\\therefore \\mathrm{OA} \\perp \\mathrm{AB}, \\quad \\therefore \\mathrm{DE} \\perp \\mathrm{AB}, \\quad \\therefore \\angle \\mathrm{OAB}=\\angle \\mathrm{DEA}=90^{\\circ}$, 又 $\\because \\mathrm{DF} \\perp \\mathrm{DE}$,\n\n$\\therefore \\angle \\mathrm{EDF}=90^{\\circ}, \\therefore$ 四边形 $\\mathrm{DFAE}$ 是矩形, $\\quad \\therefore \\mathrm{DF}=\\mathrm{AE}=3$;\n\n(2) $\\angle \\mathrm{DEF}$ 的大小不变; 理由如下:\n\n作 $\\mathrm{DM} \\perp \\mathrm{OA}$ 于 $\\mathrm{M}, \\mathrm{DN} \\perp \\mathrm{AB}$ 于 $\\mathrm{N}$, 如图 2 所示:\n\n\n\n图2\n\n$\\because$ 四边形 $\\mathrm{OABC}$ 是矩形, $\\therefore \\mathrm{OA} \\perp \\mathrm{AB}$,\n$\\therefore$ 四边形 DMAN 是矩形, $\\therefore \\angle \\mathrm{MDN}=90^{\\circ}, \\mathrm{DM} / / \\mathrm{AB}, \\mathrm{DN} / / \\mathrm{OA}$,\n\n$\\therefore \\frac{B D}{D O}=\\frac{B N}{N A}, \\frac{B D}{D O}=\\frac{A M}{O M}, \\because$ 点 $\\mathrm{D}$ 为 $\\mathrm{OB}$ 的中点, $\\therefore \\mathrm{M} 、 \\mathrm{~N}$ 分别是 $\\mathrm{OA} 、 \\mathrm{AB}$ 的中点,\n\n$\\therefore \\mathrm{DM}=\\frac{1}{2} \\mathrm{AB}=3, \\mathrm{DN}=\\frac{1}{2} \\mathrm{OA}=4, \\because \\angle \\mathrm{EDF}=90^{\\circ}, \\therefore \\angle \\mathrm{FDM}=\\angle \\mathrm{EDN}$,\n\n又 $\\because \\angle \\mathrm{DMF}=\\angle \\mathrm{DNE}=90^{\\circ}, \\therefore \\triangle \\mathrm{DMF} \\sim \\triangle \\mathrm{DNE}, \\quad \\therefore \\frac{D F}{D E}=\\frac{D M}{D N}=\\frac{3}{4}$,\n\n$\\because \\angle \\mathrm{EDF}=90^{\\circ}, \\quad \\therefore \\tan \\angle \\mathrm{DEF}=\\frac{D F}{D E}=\\frac{3}{4}$\n\n(3) 作 $\\mathrm{DM} \\perp \\mathrm{OA}$ 于 $\\mathrm{M}, \\mathrm{DN} \\perp \\mathrm{AB}$ 于 $\\mathrm{N}$,\n\n若 $A D$ 将 $\\triangle D E F$ 的面积分成 $1: 2$ 的两部分,\n\n设 $A D$ 交 $E F$ 于点 $G$, 则点 $G$ 为 $E F$ 的三等分点;\n\n(1)当点 $E$ 到达中点之前时, 如图 3 所示, $N E=3-t$,\n\n\n\n图3\n\n由 $\\triangle D M F \\sim \\triangle D N E$ 得: $M F=\\frac{3}{4}(3-t), \\therefore A F=4+M F=-\\frac{3}{4} t+\\frac{25}{4}$,\n\n$\\because$ 点 $\\mathrm{G}$ 为 $\\mathrm{EF}$ 的三等分点, $\\therefore \\mathrm{G}\\left(\\frac{3 t+71}{12}, \\frac{2}{3} t\\right)$,\n\n设直线 $A D$ 的解析式为 $\\mathrm{y}=\\mathrm{kx}+\\mathrm{b}$,\n\n把 A $(8,0), \\mathrm{D}(4,3)$ 代入得: $\\left\\{\\begin{array}{l}8 k+b=0 \\\\ 4 k+b=3\\end{array}\\right.$,\n\n解得: $\\left\\{\\begin{array}{l}k=-\\frac{3}{4} \\\\ b=6\\end{array}, \\therefore\\right.$ 直线 $\\mathrm{AD}$ 的解析式为 $\\mathrm{y}=-\\frac{3}{4} \\mathrm{x}+6$, 把 $\\mathrm{G}\\left(\\frac{3 t+71}{12}, \\frac{2}{3} t\\right)$ 代入得: $\\mathrm{t}=\\frac{75}{41}$;\n\n(2)当点 $\\mathrm{E}$ 越过中点之后, 如图 4 所示, $N E=t-3$,\n\n\n\n图4\n\n由 $\\triangle \\mathrm{DMF} \\sim \\triangle \\mathrm{DNE}$ 得: $M F=\\frac{3}{4}(\\mathrm{t}-3), \\therefore \\mathrm{AF}=4-\\mathrm{MF}=-\\frac{3}{4} \\mathrm{t}+\\frac{25}{4}$,\n\n$\\because$ 点 $\\mathrm{G}$ 为 $\\mathrm{EF}$ 的三等分点, $\\therefore \\mathrm{G}\\left(\\frac{3 t+23}{6}, \\frac{1}{3} t\\right)$, 代入直线 $\\mathrm{AD}$ 的解析式 $\\mathrm{y}=-\\frac{3}{4} \\mathrm{x}+6$ 得: $\\mathrm{t}=\\frac{75}{17}$;\n\n综上所述, 当 $\\mathrm{AD}$ 将 $\\triangle \\mathrm{DEF}$ 分成的两部分的面积之比为 $1: 2$ 时, $t$ 的值为 $\\frac{75}{41}$ 或 $\\frac{75}{17}$.", "solution": "null", "level": "九年级", "question": "在直角坐标系中, 过原点 $O$ 及点 $A(8,0), C(0,6)$ 作矩形 $O A B C$ 、连结 $O B$, 点 $D$ 为 $O B$的中点, 点 $E$ 是线段 $A B$ 上的动点, 连结 $D E$, 作 $D F \\perp D E$, 交 $O A$ 于点 $F$, 连结 $E F$. 已知点 $E$ 从 $A$点出发, 以每秒 1 个单位长度的速度在线段 $A B$ 上移动, 设移动时间为 $t$ 秒.\n\n\n\n图1\n\n\n\n图2\n\n(1) 如图 1 , 当 $t=3$ 时, 求 $D F$ 的长.\n\n(2)如图 2, 当点 $E$ 在线段 $A B$ 上移动的过程中, $\\angle D E F$ 的大小是否发生变化? 如果变化, 请说明理由; 如果不变, 请求出 $\\tan \\angle D E F$ 的值.\n\n(3) 连结 $A D$, 当 $A D$ 将 $\\triangle D E F$ 分成的两部分的面积之比为 1: 2 时, 求相应的 $t$ 的值.", "options": [], "subject": "解析几何", "analysis": "(1) 3; (2) $\\angle \\mathrm{DEF}$ 的大小不变, $\\tan \\angle \\mathrm{DEF}=\\frac{3}{4}$; (3) $\\frac{75}{41}$ 或 $\\frac{75}{17}$.\n\n【解析】(1) 当 $\\mathrm{t}=3$ 时, 点 $\\mathrm{E}$ 为 $\\mathrm{AB}$ 的中点,\n\n$\\because \\mathrm{A}(8,0), \\mathrm{C}(0,6), \\therefore \\mathrm{OA}=8, \\mathrm{OC}=6, \\because$ 点 $\\mathrm{D}$ 为 $\\mathrm{OB}$ 的中点,\n\n$\\therefore \\mathrm{DE} / / \\mathrm{OA}, \\mathrm{DE}=\\frac{1}{2} \\mathrm{OA}=4, \\because$ 四边形 $\\mathrm{OABC}$ 是矩形,\n\n$\\therefore \\mathrm{OA} \\perp \\mathrm{AB}, \\quad \\therefore \\mathrm{DE} \\perp \\mathrm{AB}, \\quad \\therefore \\angle \\mathrm{OAB}=\\angle \\mathrm{DEA}=90^{\\circ}$, 又 $\\because \\mathrm{DF} \\perp \\mathrm{DE}$,\n\n$\\therefore \\angle \\mathrm{EDF}=90^{\\circ}, \\therefore$ 四边形 $\\mathrm{DFAE}$ 是矩形, $\\quad \\therefore \\mathrm{DF}=\\mathrm{AE}=3$;\n\n(2) $\\angle \\mathrm{DEF}$ 的大小不变; 理由如下:\n\n作 $\\mathrm{DM} \\perp \\mathrm{OA}$ 于 $\\mathrm{M}, \\mathrm{DN} \\perp \\mathrm{AB}$ 于 $\\mathrm{N}$, 如图 2 所示:\n\n\n\n图2\n\n$\\because$ 四边形 $\\mathrm{OABC}$ 是矩形, $\\therefore \\mathrm{OA} \\perp \\mathrm{AB}$,\n$\\therefore$ 四边形 DMAN 是矩形, $\\therefore \\angle \\mathrm{MDN}=90^{\\circ}, \\mathrm{DM} / / \\mathrm{AB}, \\mathrm{DN} / / \\mathrm{OA}$,\n\n$\\therefore \\frac{B D}{D O}=\\frac{B N}{N A}, \\frac{B D}{D O}=\\frac{A M}{O M}, \\because$ 点 $\\mathrm{D}$ 为 $\\mathrm{OB}$ 的中点, $\\therefore \\mathrm{M} 、 \\mathrm{~N}$ 分别是 $\\mathrm{OA} 、 \\mathrm{AB}$ 的中点,\n\n$\\therefore \\mathrm{DM}=\\frac{1}{2} \\mathrm{AB}=3, \\mathrm{DN}=\\frac{1}{2} \\mathrm{OA}=4, \\because \\angle \\mathrm{EDF}=90^{\\circ}, \\therefore \\angle \\mathrm{FDM}=\\angle \\mathrm{EDN}$,\n\n又 $\\because \\angle \\mathrm{DMF}=\\angle \\mathrm{DNE}=90^{\\circ}, \\therefore \\triangle \\mathrm{DMF} \\sim \\triangle \\mathrm{DNE}, \\quad \\therefore \\frac{D F}{D E}=\\frac{D M}{D N}=\\frac{3}{4}$,\n\n$\\because \\angle \\mathrm{EDF}=90^{\\circ}, \\quad \\therefore \\tan \\angle \\mathrm{DEF}=\\frac{D F}{D E}=\\frac{3}{4}$\n\n(3) 作 $\\mathrm{DM} \\perp \\mathrm{OA}$ 于 $\\mathrm{M}, \\mathrm{DN} \\perp \\mathrm{AB}$ 于 $\\mathrm{N}$,\n\n若 $A D$ 将 $\\triangle D E F$ 的面积分成 $1: 2$ 的两部分,\n\n设 $A D$ 交 $E F$ 于点 $G$, 则点 $G$ 为 $E F$ 的三等分点;\n\n(1)当点 $E$ 到达中点之前时, 如图 3 所示, $N E=3-t$,\n\n\n\n图3\n\n由 $\\triangle D M F \\sim \\triangle D N E$ 得: $M F=\\frac{3}{4}(3-t), \\therefore A F=4+M F=-\\frac{3}{4} t+\\frac{25}{4}$,\n\n$\\because$ 点 $\\mathrm{G}$ 为 $\\mathrm{EF}$ 的三等分点, $\\therefore \\mathrm{G}\\left(\\frac{3 t+71}{12}, \\frac{2}{3} t\\right)$,\n\n设直线 $A D$ 的解析式为 $\\mathrm{y}=\\mathrm{kx}+\\mathrm{b}$,\n\n把 A $(8,0), \\mathrm{D}(4,3)$ 代入得: $\\left\\{\\begin{array}{l}8 k+b=0 \\\\ 4 k+b=3\\end{array}\\right.$,\n\n解得: $\\left\\{\\begin{array}{l}k=-\\frac{3}{4} \\\\ b=6\\end{array}, \\therefore\\right.$ 直线 $\\mathrm{AD}$ 的解析式为 $\\mathrm{y}=-\\frac{3}{4} \\mathrm{x}+6$, 把 $\\mathrm{G}\\left(\\frac{3 t+71}{12}, \\frac{2}{3} t\\right)$ 代入得: $\\mathrm{t}=\\frac{75}{41}$;\n\n(2)当点 $\\mathrm{E}$ 越过中点之后, 如图 4 所示, $N E=t-3$,\n\n\n\n图4\n\n由 $\\triangle \\mathrm{DMF} \\sim \\triangle \\mathrm{DNE}$ 得: $M F=\\frac{3}{4}(\\mathrm{t}-3), \\therefore \\mathrm{AF}=4-\\mathrm{MF}=-\\frac{3}{4} \\mathrm{t}+\\frac{25}{4}$,\n\n$\\because$ 点 $\\mathrm{G}$ 为 $\\mathrm{EF}$ 的三等分点, $\\therefore \\mathrm{G}\\left(\\frac{3 t+23}{6}, \\frac{1}{3} t\\right)$, 代入直线 $\\mathrm{AD}$ 的解析式 $\\mathrm{y}=-\\frac{3}{4} \\mathrm{x}+6$ 得: $\\mathrm{t}=\\frac{75}{17}$;\n\n综上所述, 当 $\\mathrm{AD}$ 将 $\\triangle \\mathrm{DEF}$ 分成的两部分的面积之比为 $1: 2$ 时, $t$ 的值为 $\\frac{75}{41}$ 或 $\\frac{75}{17}$."} {"id": "3930", "image": ["5540.jpg", "5541.jpg", "5542.jpg", "5541.jpg", "5542.jpg"], "answer": "$2 \\sqrt{2}$.\n\n【解析】如图, 作 $\\mathrm{AP} \\perp$ 直线 $y=-\\frac{3}{4} x+3$, 垂足为 $\\mathrm{P}$, 作 $\\odot \\mathrm{A}$ 的切线 $\\mathrm{PQ}$, 切点为 $\\mathrm{Q}$, 此时切线长 $\\mathrm{PQ}$最小, $\\because \\mathrm{A}$ 的坐标为 $(-1,0)$, 设直线与 $\\mathrm{x}$ 轴, $\\mathrm{y}$ 轴分别交于 $\\mathrm{C}, \\mathrm{B}$,\n\n$\\therefore \\mathrm{B}(0,3), \\mathrm{C}(4,0), \\therefore \\mathrm{OB}=3, \\mathrm{AC}=5$,\n\n$\\therefore \\mathrm{BC}=\\sqrt{O B^{2}+O C^{2}}=5 \\therefore \\mathrm{AC}=\\mathrm{BC}$,\n\n在 $\\triangle A P C$ 与 $\\triangle B O C$ 中,\n\n\n\n$\\therefore \\mathrm{AP}=\\mathrm{OB}=3, \\quad \\therefore \\mathrm{PQ}=\\sqrt{3^{2}-1^{2}}=2 \\sqrt{2}$.\n\n$\\because \\mathrm{PQ}^{2}=\\mathrm{PA}^{2}-1$, 此时 $\\mathrm{PA}$ 最小, 所以此时切线长 $\\mathrm{PQ}$ 也最小, 最小值为 $2 \\sqrt{2}$.\n\n", "solution": "null", "level": "九年级", "question": "如图, 在直角坐标系中, $\\odot \\mathrm{A}$ 的圆心 $\\mathrm{A}$ 的坐标为 $(-1,0)$, 半径为 1 , 点 $\\mathrm{P}$ 为直线 $y=-\\frac{3}{4} x+3$ 上的动点, 过点 $P$ 作 $\\odot A$ 的切线, 切点为 $Q$, 求切线长 $P Q$ 的最小值.\n\n", "options": [], "subject": "解析几何", "analysis": "$2 \\sqrt{2}$.\n\n【解析】如图, 作 $\\mathrm{AP} \\perp$ 直线 $y=-\\frac{3}{4} x+3$, 垂足为 $\\mathrm{P}$, 作 $\\odot \\mathrm{A}$ 的切线 $\\mathrm{PQ}$, 切点为 $\\mathrm{Q}$, 此时切线长 $\\mathrm{PQ}$最小, $\\because \\mathrm{A}$ 的坐标为 $(-1,0)$, 设直线与 $\\mathrm{x}$ 轴, $\\mathrm{y}$ 轴分别交于 $\\mathrm{C}, \\mathrm{B}$,\n\n$\\therefore \\mathrm{B}(0,3), \\mathrm{C}(4,0), \\therefore \\mathrm{OB}=3, \\mathrm{AC}=5$,\n\n$\\therefore \\mathrm{BC}=\\sqrt{O B^{2}+O C^{2}}=5 \\therefore \\mathrm{AC}=\\mathrm{BC}$,\n\n在 $\\triangle A P C$ 与 $\\triangle B O C$ 中,\n\n\n\n$\\therefore \\mathrm{AP}=\\mathrm{OB}=3, \\quad \\therefore \\mathrm{PQ}=\\sqrt{3^{2}-1^{2}}=2 \\sqrt{2}$.\n\n$\\because \\mathrm{PQ}^{2}=\\mathrm{PA}^{2}-1$, 此时 $\\mathrm{PA}$ 最小, 所以此时切线长 $\\mathrm{PQ}$ 也最小, 最小值为 $2 \\sqrt{2}$.\n\n"} {"id": "3932", "image": ["5543.jpg", "5544.jpg", "5544.jpg"], "answer": "$15-5 \\sqrt{3}$\n\n【解析】 过点 $\\mathrm{B}$ 作 $\\mathrm{BM} \\perp \\mathrm{FD}$ 于点 $\\mathrm{M}$, E\n\n\n\n在 $\\triangle \\mathrm{ACB}$ 中, $\\angle \\mathrm{ACB}=90^{\\circ}, \\angle \\mathrm{A}=60^{\\circ}, \\mathrm{AC}=10$,\n\n$\\therefore \\angle \\mathrm{ABC}=30^{\\circ}, \\mathrm{BC}=\\mathrm{AC} \\times \\tan 60^{\\circ}=10 \\sqrt{3}$,\n\n$\\because \\mathrm{AB} / / \\mathrm{CF}$,\n\n$\\therefore \\mathrm{BM}=\\mathrm{BC} \\times \\sin 30^{\\circ}=10 \\sqrt{3} \\times \\frac{1}{2}=5 \\sqrt{3}$,\n\n$\\mathrm{CM}=\\mathrm{BC} \\times \\cos 30^{\\circ}=15$,\n\n在 $\\triangle \\mathrm{EFD}$ 中, $\\angle \\mathrm{F}=90^{\\circ}, \\angle \\mathrm{E}=45^{\\circ}$,\n\n$\\therefore \\angle \\mathrm{EDF}=45^{\\circ}$\n\n$\\therefore \\mathrm{MD}=\\mathrm{BM}=5 \\sqrt{3}$,\n\n$\\therefore C D=C M-M D=15-5 \\sqrt{3}$", "solution": "null", "level": "九年级", "question": "一副直角三角板如图所示放置, 点 $C$ 在 $F D$ 的延长线上, $A B / / C F, \\angle F=\\angle A C B=90^{\\circ}$, $\\angle E=45^{\\circ}, \\angle A=60^{\\circ}, A C=10$, 试 $\\mathrm{CD}$ 的长.\n\n", "options": [], "subject": "解析几何", "analysis": "$15-5 \\sqrt{3}$\n\n【解析】 过点 $\\mathrm{B}$ 作 $\\mathrm{BM} \\perp \\mathrm{FD}$ 于点 $\\mathrm{M}$, E\n\n\n\n在 $\\triangle \\mathrm{ACB}$ 中, $\\angle \\mathrm{ACB}=90^{\\circ}, \\angle \\mathrm{A}=60^{\\circ}, \\mathrm{AC}=10$,\n\n$\\therefore \\angle \\mathrm{ABC}=30^{\\circ}, \\mathrm{BC}=\\mathrm{AC} \\times \\tan 60^{\\circ}=10 \\sqrt{3}$,\n\n$\\because \\mathrm{AB} / / \\mathrm{CF}$,\n\n$\\therefore \\mathrm{BM}=\\mathrm{BC} \\times \\sin 30^{\\circ}=10 \\sqrt{3} \\times \\frac{1}{2}=5 \\sqrt{3}$,\n\n$\\mathrm{CM}=\\mathrm{BC} \\times \\cos 30^{\\circ}=15$,\n\n在 $\\triangle \\mathrm{EFD}$ 中, $\\angle \\mathrm{F}=90^{\\circ}, \\angle \\mathrm{E}=45^{\\circ}$,\n\n$\\therefore \\angle \\mathrm{EDF}=45^{\\circ}$\n\n$\\therefore \\mathrm{MD}=\\mathrm{BM}=5 \\sqrt{3}$,\n\n$\\therefore C D=C M-M D=15-5 \\sqrt{3}$"} {"id": "3934", "image": [], "answer": "(1) $\\frac{2}{3}+\\sqrt{3}$\n(2) $3 \\sqrt{2}$.\n\n【解析】(1)原式 $=\\frac{\\left(\\frac{\\sqrt{3}}{2}\\right)^{2}+\\left(\\frac{1}{2}\\right)^{2}}{\\sqrt{3} \\times \\frac{\\sqrt{3}}{2}}+\\sqrt{3}=\\frac{2}{3}+\\sqrt{3}$;\n\n(2)原式 $=2 \\times \\frac{\\sqrt{2}}{2} \\times \\frac{\\sqrt{2}}{2}-2 \\times \\frac{1}{2} \\times 1+\\sqrt{6} \\times \\sqrt{3}=1-1+3 \\sqrt{2}=3 \\sqrt{2}$.", "solution": "null", "level": "九年级", "question": "计算:\n\n(1) $\\frac{\\cos ^{2} 30^{\\circ}+\\cos ^{2} 60^{\\circ}}{\\tan 60^{\\circ} \\cos 30^{\\circ}}+\\tan 60^{\\circ}$;\n\n(2) $2 \\cos 45^{\\circ} \\cdot \\sin 45^{\\circ}-2 \\sin 30^{\\circ} \\cdot \\tan 45^{\\circ}+\\sqrt{6} \\cdot \\tan 60^{\\circ}$.", "options": [], "subject": "解析几何", "analysis": "(1) $\\frac{2}{3}+\\sqrt{3}$\n(2) $3 \\sqrt{2}$.\n\n【解析】(1)原式 $=\\frac{\\left(\\frac{\\sqrt{3}}{2}\\right)^{2}+\\left(\\frac{1}{2}\\right)^{2}}{\\sqrt{3} \\times \\frac{\\sqrt{3}}{2}}+\\sqrt{3}=\\frac{2}{3}+\\sqrt{3}$;\n\n(2)原式 $=2 \\times \\frac{\\sqrt{2}}{2} \\times \\frac{\\sqrt{2}}{2}-2 \\times \\frac{1}{2} \\times 1+\\sqrt{6} \\times \\sqrt{3}=1-1+3 \\sqrt{2}=3 \\sqrt{2}$."} {"id": "3935", "image": [], "answer": "(1)在 $\\triangle \\mathrm{ABC}$ 中, $\\angle \\mathrm{C}=90^{\\circ}, \\angle \\mathrm{B}=60^{\\circ}, \\mathrm{BC}=8$,\n\n$\\therefore \\angle \\mathrm{A}=30^{\\circ}, \\mathrm{AB}=\\frac{8}{\\cos 60^{\\circ}}=16, \\mathrm{AC}=8 \\tan 60^{\\circ}=8 \\sqrt{3}$;\n\n(2)在 $\\triangle \\mathrm{ABC}$ 中, $\\angle \\mathrm{C}=90^{\\circ}, \\mathrm{AC}=\\sqrt{2}, \\mathrm{AB}=2$,\n\n$\\therefore \\cos A=\\frac{A C}{A B}=\\frac{\\sqrt{2}}{2}$,\n\n$\\therefore \\angle \\mathrm{A}=45^{\\circ}$,\n\n$\\therefore \\angle \\mathrm{B}=45^{\\circ}, \\mathrm{BC}=\\sqrt{2}$.", "solution": "null", "level": "九年级", "question": "已知: 在 $\\triangle \\mathrm{ABC}$ 中, $\\angle \\mathrm{C}=90^{\\circ}$, 根据下列条件, 解直角三角形.\n\n(1) $\\mathrm{BC}=8, \\angle \\mathrm{B}=60^{\\circ} ;$\n\n(2) $\\mathrm{AC}=\\sqrt{2}, \\mathrm{AB}=2$.", "options": [], "subject": "解析几何", "analysis": "(1)在 $\\triangle \\mathrm{ABC}$ 中, $\\angle \\mathrm{C}=90^{\\circ}, \\angle \\mathrm{B}=60^{\\circ}, \\mathrm{BC}=8$,\n\n$\\therefore \\angle \\mathrm{A}=30^{\\circ}, \\mathrm{AB}=\\frac{8}{\\cos 60^{\\circ}}=16, \\mathrm{AC}=8 \\tan 60^{\\circ}=8 \\sqrt{3}$;\n\n(2)在 $\\triangle \\mathrm{ABC}$ 中, $\\angle \\mathrm{C}=90^{\\circ}, \\mathrm{AC}=\\sqrt{2}, \\mathrm{AB}=2$,\n\n$\\therefore \\cos A=\\frac{A C}{A B}=\\frac{\\sqrt{2}}{2}$,\n\n$\\therefore \\angle \\mathrm{A}=45^{\\circ}$,\n\n$\\therefore \\angle \\mathrm{B}=45^{\\circ}, \\mathrm{BC}=\\sqrt{2}$."} {"id": "2985", "image": ["3889.jpg"], "answer": "(1)$y=-x^{2}+2x+3$;(2)存在,点$M$的坐标为$(0,3)$或$(2,3)$或$(1+sqrt{7},-3)$或$(1-sqrt{7},-3)$", "solution": "null", "level": "九年级", "question": "(本题8分)(2020浙江温州一一模)已知,如图,抛物线$y=-x^{2}+bx+c$经过直线$y=-x+3$与坐标轴的两个交点$A,B$.此抛物线与$x$轴的另一个交点为$C$.拖物线的顶点为$D$.(1)求此抛物线的解析式;(2)若点$M$为抛物线上一动点,是否存在点$M$.使$triangleACM$与$triangleABC$的面积相等?若存在,求点$M$的坐标;若不存在,请说明理由.", "options": [], "subject": "解析几何", "analysis": "【分析】(1)先求得点$A$和点$B$的坐标,然后将点$A$和点$B$的坐标代入抛物线的解析式求得${b},{c}$的值即可;(2)设$M$的坐标为$(x,y)$,由$triangleACM$与$triangleABC$的面积相等可得到$yymid=3$,将$y=3$或$y=-3$代入抛物线的解析式求得对应的$x$的值,从而得到点$M$的坐标.【详解】(1)由题意得$A(3,0),B(0,3)$.将点${A}$和点$B$的坐标代入得:$left{begin{array}{l}c=3-9+3b+3=0end{array}right.$解得:$b=2,c=3$.$therefore$抛物线的解析式为$y=-x^{2}+2x+3$;(2)设$M$的坐标为$(x,y)$.$becausetriangleACM$与$triangleABC$的面积相等,$thereforefrac{1}{2}ACcdot|y|=frac{1}{2}ACcdotOB$$therefore|y|=OB=3$.当$y=3$时,$-x^{2}+2x+3=3$,解得$x=0$或$x=2$,$thereforeM(2,3)$或$(0,3)$,当$y=-3$时,$-x^{2}+2x+3=-3$,解得:$x=1+sqrt{7}$或$x=1-sqrt{7}$$thereforeM(1+sqrt{7},-3)$或$(1-sqrt{7},-3)$.综上所述点$M$的坐标为$(0,3)$或$(2,3)$或$(1+sqrt{7},-3)$或$(1-sqrt{7},-3)$.【点睛】本题主要考查的是二次函数的应用,求得点${A}$和点${B}$的坐标是解答问题(1)的关键,求得点$M$的纵坐标是解答问题(2)的关键."} {"id": "2987", "image": [], "answer": "(1)图象开口向上;对称轴是${x}=1$,顶点坐标是$(1,-4)$;(2)与${y}$轴交点坐标是$(0,-3)$;与$x$轴交点的坐标是(3,0)、$(-1,0)$;(3)当$xgeq1$时,$y$随${x}$的增大而增大", "solution": "null", "level": "九年级", "question": "(本题8分)(2020$cdot$浙江$cdot$宁波市鄞州区中河街道宋诏桥初级中学一模)已知二次函数$y=x^{2}-2x-3$.(1)求图象的开口方向、对称轴、顶点坐标;(2)求图象与$x$轴的交点坐标,与$y$轴的交点坐标;(3)当$x$为何值时,$y$随$x$的增大而增大?", "options": [], "subject": "解析几何", "analysis": "【分析】(1)根据${a}$的符号判断抛物线的开口方向;把抛物线的一般式化为顶点式,根据顶点式可求顶点坐标及对称轴;(2)根据图象与${y}$轴和${x}$轴的相交的特点可求出坐标;(3)根据二次函数的增减性,当${a}>0$时,在对称轴的右侧,${y}$随${x}$的增大而增大,由此即可解答;【详解】(1)$because{a}=1>0,quadtherefore$图象开口向上;$becausey=x^{2}-2x-3=(x-1)^{2}-4$,$therefore$对称轴是${x}=1$,顶点坐标是(1,-4);(2)由图象与$y$轴相交则$x=0$,代入得:$y=-3$,$therefore$与${y}$轴交点坐标是$(0,-3)$;由图象与$x$轴相交则$y=0$,代入得:$x^{2}-2x-3=0$,解方程得$x=3$或$x=-1$,$therefore$与${x}$轴交点的坐标是$(3,0)、(-1,0)$;(3)当$xgeq1$时,${y}$随${x}$的增大而增大.【点睛】本题考查了二次函数的性质与图象,熟记二次函数的图象和性质是解决问题的关键."} {"id": "2989", "image": ["3890.jpg", "3891.jpg"], "answer": "(1)抛物线的解析式为$y=-x^{2}+2x+3$.(2)证明见解析;(3)点$P$坐标为$left(frac{3+sqrt{5}}{2},frac{5-sqrt{5}}{2}right)$或$(2,3)$", "solution": "null", "level": "九年级", "question": "(本题9分)(2019$cdot$浙江$cdot$杭州外国语学校一模)已知二次函数$y=ax^{2}+bx-3a$经过点${A}(-1,0)、{C}(0,3)$,与${x}$轴交于另一点${B}$,抛物线的顶点为${D}$,(1)求此二次函数解析式;(2)连接${DC}、{BC}、{DB}$,求证:$triangle{BCD}$是直角三角形;(3)在对称轴右侧的抛物线上是否存在点$P$,使得$trianglePDC$为等腰三角形?若存在,求出符合条件的点${P}$的坐标;若不存在,请说明理由.", "options": [], "subject": "解析几何", "analysis": "【详解】试题分析:(1)将$A(-1,0)、C(0,3)$,代入二次函数$y=ax^{2}+bx-3a$,求得$a、b$的值即可确定二次函数的解析式;(2)分别求得线段${BC}、{CD}、{BD}$的长,利用勾股定理的逆定理进行判定即可;(3)分以${CD}$为底和以${CD}$为腰两种情况讨论.运用两点间距离公式建立起P点横坐标和纵坐标之间的关系,再结合抛物线解析式即可求解.试题解析:(1)$because$二次函数$y=ax^{2}+bx-3a$经过点$A(-1,0)、C(0,3),therefore$将A$(-1,0)、{C}(0,3)$,代入,得$left{begin{array}{l}a-b-3a=0-3a=3end{array}right.$,解得$left{begin{array}{l}a=-1b=2end{array},thereforeright.$抛物线的解析式为$y=-x^{2}+2x+3$;(2)如图,连接DC、BC、DB,由$y=-x^{2}+2x+3=-(x-1)^{2}+4$得,D点坐标为$(1,4),therefore{CD}=sqrt{(1-0)^{2}+(4-3)^{2}}=sqrt{2},{BC}=sqrt{3^{2}+3^{2}}=3sqrt{2},{BD}=$$sqrt{(3-1)^{2}+(4-0)^{2}}=2sqrt{5},quadbecause{CD}^{2}+{BC}^{2}=(sqrt{2})^{2}+(3sqrt{2})^{2}=20,quad{BD}^{2}=(2sqrt{5})^{2}=20$,$therefore{CD}^{2}+{BC}^{2}={BD}^{2},quadthereforetriangle{BCD}$是直角三角形;(3)${y}=-{x}^{2}+2{x}+3$对称轴为直线${x}=1$.假设存在这样的点$P,(1)$以$CD$为底边,则$P_{1}D=P_{1}C$,设$P_{1}$点坐标为$(x,y)$,根据勾股定理可得$P_{1}C^{2}=x^{2}+(3-y)^{2},P_{1}D^{2}=(x-1)^{2+}(4-y)^{2}$,因此$x^{2}+(3-y)^{2}=(x-$1)${}^{2}+(4-y)^{2}$,即$y=4-x$.又$P_{1}$点$(x,y)$在抛物线上,$therefore4-x=-x^{2}+2x+3$,即$x^{2}$$-3x+1=0$,解得$x_{1}=frac{3+sqrt{5}}{2},x_{2}=frac{3-sqrt{5}}{2}<1$,(不满足在对称轴右侧应舍去),$thereforex=$$frac{3+sqrt{5}}{2},thereforey=4-x=frac{5-sqrt{5}}{2}$,即点${P}_{1}$坐标为$left(frac{3+sqrt{5}}{2},frac{5-sqrt{5}}{2}right)$.(2)以CD为一腰,$because$点${P}_{2}$在对称轴右侧的抛物线上,由抛物线对称性知,点${P}_{2}$与点${C}$关于直线${x}=1$对称,此时点${P}_{2}$坐标为$(2,3)$.$therefore$符合条件的点${P}$坐标为$left(frac{3+sqrt{5}}{2},frac{5-sqrt{5}}{2}right)$或$(2$,$3)$.考点:1.二次函数图象性质;2.等腰三角形性质;3.直角三角形的判定."} {"id": "2990", "image": ["3892.jpg", "3893.jpg", "3894.jpg"], "answer": "(1)二次函数的表达式为:$y=x^{2}-4x+3;$(2)点$P$的坐标为:$(0,3+3sqrt{2})$或(0,3-3$sqrt{2})$或$(0,-3)$或(0,0);(3)当点$M$出发1秒到达$D$点时,$triangleMNB$面积最大,最大面积是1.此时点${N}$在对称轴上${x}$轴上方2个单位处或点${N}$在对称轴上$x$轴下方2个单位处", "solution": "null", "level": "九年级", "question": "(本题10分)(2020$cdot$浙江$cdot$高照实验学校九年级阶段练习)如图,关于${x}$的二次函数$y=x^{2}+bx+c$的图象与$x$轴交于点$A(1,0)$和点$B$与$y$轴交于点$C(0,3)$,抛物线的对称轴与${x}$轴交于点${D}$.(1)求二次函数的表达式;(2)在$y$轴上是否存在一点$P$,使$trianglePBC$为等腰三角形?若存在.请求出点$P$的坐标;(3)有一个点${M}$从点${A}$出发,以每秒1个单位的速度在${AB}$上向点${B}$运动,另一个点${N}$从点${D}$与点${M}$同时出发,以每秒2个单位的速度在抛物线的对称轴上运动,当点$M$到达点$B$时,点$M、N$同时停止运动,问点$M、N$运动到何处时,$triangleMNB$面积最大,试求出最大面积.", "options": [], "subject": "解析几何", "analysis": "【分析】(1)把$A(1,0)$和$C(0,3)$代入$y=x^{2}+bx+c$得方程组,解方程组即可得二次函数的表达式;(2)先求出点$B$的坐标,再根据勾股定理求得$BC$的长,当$triangle{PBC}$为等腰三角形时分三种情况进行讨论:(1)${CP}={CB}$;(2)${PB}={PC}$;(3)${BP}={BC}$;分别根据这三种情况求出点${P}$的坐标;(3)设$AM=t$则$DN=2t$,由$AB=2$,得$BM=2-t,StriangleMNB=frac{1}{2}times(2-t)times2t=-$$t^{2}+2t$,把解析式化为顶点式,根据二次函数的性质即可得$triangleMNB$最大面积;此时点$M$在$D$点,点$N$在对称轴上$x$轴上方2个单位处或点$N$在对称轴上$x$轴下方2个单位处.【详解】解:(1)把${A}(1,0)$和${C}(0,3)$代入${y}={x}^{2}+{bx}+{c}$,$left{begin{array}{l}1+b+c=0c=3end{array}right.$解得:$b=-4,c=3$,$therefore$二次函数的表达式为:${y}={x}^{2}-4{x}+3$;(2)令${y}=0$,则${x}^{2}-4{x}+3=0$,解得:$x=1$或$x=3$,$therefore{B}(3,0)$$therefore{BC}=3sqrt{2}$,点${P}$在${y}$轴上,当$triangle{PBC}$为等腰三角形时分三种情况进行讨论:如图1,(1)当$CP=CB$时,$PC=3sqrt{2},thereforeOP=OC+PC=3+3sqrt{2}$或$OP=PC-OC=3sqrt{2}-3$$therefore{P}_{1}(0,3+3sqrt{2}),{P}_{2}(0,3-3sqrt{2})$(2)当${PB}={PC}$时,${OP}={OB}=3$,$therefore{P}_{3}(0,-3);$(3)当${BP}={BC}$时,$because{OC}={OB}=3$$therefore$此时${P}$与${O}$重合,$therefore{P}_{4}(0,0);$综上所述,点${P}$的坐标为:$(0,3+3sqrt{2})$或$(0,3-3sqrt{2})$或$(-3,0)$或$(0$,$0)$;图1(3)如图2,设$AM=t$,由$AB=2$,得$BM=2-t$,则$DN=2t$,$therefore{S}triangle{MNB}=frac{1}{2}times(2-{t})times2{t}=-{t}^{2}+2{t}=-({t}-1)^{2}+1$,当点$M$出发1秒到达$D$点时,$triangleMNB$面积最大,最大面积是1.此时点$N$在对称轴上${x}$轴上方2个单位处或点${N}$在对称轴上${x}$轴下方2个单位处.图2"} {"id": "3007", "image": [], "answer": "(1)开口向上,直线$x=-2$;(2)$(0,-2)$", "solution": "null", "level": "九年级", "question": "(本题6分)(2021$cdot$浙江丽水$cdot$九年级期中)已知二次函数$y=x^{2}+4x-2$.(1)求抛物线开口方向及对称轴.(2)写出抛物线与$y$轴的交点坐标.", "options": [], "subject": "解析几何", "analysis": "【分析】(1)根据二次函数的顶点式进行解答即可;(2)令$x=0$,求出$y$的值即可.【详解】(1)$becausea=1>0$,$therefore$抛物线开口向上,$becausey=x^{2}+4x-2=(x+2)^{2}-6$,$therefore$对称轴是直线$x=-2$;(2)$becausex=0$,$thereforey=-2$,$therefore$与$y$轴交点坐标是$(0,-2)$.【点睛】本题考查的是二次函数的性质,熟知二次函数的顶点式是解答此题的关键."} {"id": "3010", "image": ["3903.jpg"], "answer": "(1)$y=-x^{2}+2x+3$;(2)$frac{3}{4}leqPQleq3$", "solution": "null", "level": "九年级", "question": "(本题6分)(2021$cdot$浙江宁波$cdot$九年级期中)如图,抛物线$y=-x^{2}+bx+c$的图象与$x$轴正半轴交于点$A(3,0)$,与$y$轴交于点$B(0,3)$直线$l$的函数表达式为$y=-x+6$,(1)求抛物线的函数表达式;(2)动点$P$在抛物线$AB$段上运动,经过点$P$作$y$轴的平行线交直线$l$于点$Q$,求线段$PQ$的取值范围.", "options": [], "subject": "解析几何", "analysis": "【分析】(1)将点$A、B$坐标分别代入函数解析式求解即可确定函数解析式;(2)设$Pleft(x,-x^{2}+2x+3right),(0leqxleq3)$,则$Q(x,-x+6)$,可得$PQ=left(x-frac{3}{2}right)^{2}+frac{3}{4}$,根据$x$的取值范围,即可得出$PQ$的范围.【详解】解:(1)将点$A、B$坐标分别代入函数解析式可得:$left{begin{array}{c}c=30=-9+3b+cend{array}right.$,解得:$left{begin{array}{l}c=3b=2end{array}right.$,$therefore$函数解析式为:$y=-x^{2}+2x+3$;(2)设$Pleft(x,-x^{2}+2x+3right),(0leqxleq3)$$becausePQ//y$轴,$thereforeQ(x,-x+6)$,$therefore$根据图象可得:$PQ=-x+6-left(-x^{2}+2x+3right)$$=x^{2}-3x+3$$=left(x-frac{3}{2}right)^{2}+frac{3}{4}$当$x=frac{3}{2}$时,$PQ$取得最小值为$frac{3}{4}$;当$x=0$或3时,$PQ$取得最大值为3;$therefore$线段$PQ$的取值范围为:$frac{3}{4}leqPQleq3$.【点睛】题目主要考查二次函数的基本性质,利用待定系数法确定函数解析式,求函数值的取值范围,理解题意,结合图形列出函数解析式是解题关键."} {"id": "3014", "image": ["3907.jpg", "3908.jpg", "3909.jpg", "3910.jpg", "3911.jpg"], "answer": "(1)4(2)$frac{2}{a},frac{1}{a}$(3)$(2,-3),2sqrt{3}$", "solution": "null", "level": "九年级", "question": "(本题9分)(2022$cdot$浙江金华$cdot$九年级期末)如图1,抛物线$y=ax^{2}+bx+c(a>0)$的顶点为$M$,平行于$x$的直线与抛物线交于点$A,B$,若$triangleAMB$为等腰直角三角形,则抛物线上$A,B$两点之间的部分与线段$AB$围成的图形称为该抛物线对应的“准碗形”,线段$AB$称为碗宽,点$M$到线段$AB$的距离称为碗高.国1内2(1)抛物线$y=frac{1}{2}x^{2}$对应的碗宽为$qquad$;(2)抛物线$y=ax^{2}quad(a>0)$对应的碗宽为$qquad$;抛物线$y=a(x-2)^{2}+3(a>0)$对应的碗高为$qquad$;(3)已知抛物线$y=ax^{2}-4ax-frac{5}{3}(a>0)$对应的碗高为3.(1)求碗顶$M$的坐标;(2)如图2,将“准碗形$AMB$”绕点$M$顺时针旋转$30^{circ}$得到“准碗形$A^{prime}MB^{primeprime}$.过点$B^{prime}$作$x$轴的平行线交准碗形$A^{prime}MB^{prime}$于点$C$,点$P$是线段$B^{prime}C$上的动点,过点$P$作$y$轴的平行线交准碗形$A^{prime}MB^{prime}$于点$Q$.请直接写出线段$PQ$长度的最大值.", "options": [], "subject": "解析几何", "analysis": "【分析】(1)根据碗宽的定义以及等腰直角三角形的性质可以假设$B(m,m)$,代入抛物线的解析式,求出$A、B$两点坐标即可解决问题.(2)利用(1)中方法可求碗宽,根据等腰直角三角形可知碗高是碗宽的一半.(3)(1)由碗高为3求出$a$,再求顶点坐标即可;(2)作$QSperpBP$于$S$,找到$PQ$和$QS$的关系后即可解决问题.(1)解:根据碗宽的定义以及等腰直角三角形的性质可以假设$B(m,m)$.把$B(m,m)$代入$y=frac{1}{2}x^{2}$,得$m=frac{1}{2}m^{2}$,解得,$m=2$或0(舍去),$thereforeA(-2,2),B(2,2)$$thereforeAB=4,$即碗宽为4;故答案为:4.$(2)$解:类似(1)设$B(n,n)$,代入$y=ax^{2}$,得$n=an^{2}$,解得,$n=frac{1}{a}$或0(舍去),$AB$$=frac{2}{a}$,即碗宽为$frac{2}{a};$抛物线$y=a(x-2)^{2}+3$是由抛物线$y=ax^{2}$平移得到的,所以,它们的碗宽一样为$frac{2}{a}$,根据等腰直角三角形的性质,可知可知碗高是碗宽的一半,即$frac{1}{a};$故答案为:$frac{2}{a},frac{1}{a}$.(3)解:(1)抛物线$y=ax^{2}-4ax-frac{5}{3}(a>0)$对应的碗高为3.由(2)可知$frac{1}{a}=3$,解得,$a=frac{1}{3}$,抛物线解析式为$y=frac{1}{3}x^{2}-frac{4}{3}x-frac{5}{3}$,化成顶点式为$y=frac{1}{3}(x-2)^{2}-3$;则$M$的坐标为$(2,-3)$;(2)如图,作$QSperpBP$于$S$,由旋转可知$anglePBO=30^{circ}$,因为过点$P$作$y$轴的平行线交准碗形$A^{prime}MB^{prime}$于点$Q$,$thereforePQperpOB$$thereforeangleQPB=60^{circ},anglePQS=30^{circ}$,$thereforePQ=2PS,quadQS=sqrt{PQ^{2}-PS^{2}}=sqrt{3}PS$,当$QS$等于碗高时,$QS$最大,此时$PQ$长度的最大,由(2)可知$QS$最大为3,则$PS=sqrt{3},QP=2sqrt{3}$;$PQ$长度的最大值为$2sqrt{3}$.【点睛】本题考查了二次函数的性质和直角三角形的性质,解题关键是准确理解题意,熟练运用二次函数的性质和直角三角形的性质求解."} {"id": "3125", "image": ["4060.jpg", "4061.jpg", "4062.jpg"], "answer": "(1)$frac{3}{4};$(2)见解析", "solution": "null", "level": "九年级", "question": "(本题8分)(2021浙江$cdot$温州市实验中学九年级阶段练习)由边长为1的小正方形组成的$6times6$的网格中,线段$AB$的两个端点都在格点上.(1)如图1,C,$D$也在格点上,连结$CD$交$AB$于点$O$,则$frac{AO}{BO}=$(2)如图2,仅用无刻度直尺在$triangleABC$的边$AB$上找一点$M$,使得$frac{S_{triangleAMC}}{S_{triangleBMC}}=frac{2}{3}$.(图1)(图2)", "options": [], "subject": "解析几何", "analysis": "【分析】(1)根据网格的特点,可知$AC//BD$,进而根据$triangleACOsimtriangleBOD$,即可求得$frac{AO}{BO}$,(2)在格点上找到点$D,E$,使得$AD=2,BE=3$,连接$DE$交$AB$于点$M$,则$M$点即为所求.【详解】解(1)由题意:$becauseAC//BD$,$thereforetriangleACOsimtriangleBOD$,$thereforeAO:BO=AC:BD$,$becauseAC=3,BD=4$,即$AO:BO=3:4=frac{3}{4}$(2)如图,在格点上找到点$D,E$,使得$AD=2,BE=3$,连接$DE$交$AB$于点$M$,则$M$点即为所求,连接$MC$,$becauseAD//BE$,$thereforetriangleADMsimtriangleBEM$$thereforefrac{AM}{BM}=frac{AD}{BE}=frac{2}{3}$,设$C$到$AB$的距离为$h$,$thereforefrac{S_{triangleAMC}}{S_{triangleBMC}}=frac{frac{1}{2}cdotAMcdoth}{frac{1}{2}cdotBMcdoth}=frac{AM}{BM}=frac{2}{3}$.【点睛】本题考查了网格作图,相似三角形的性质与判定,掌握相似三角形的性质与判定是解题的关键."} {"id": "3126", "image": ["4063.jpg", "4064.jpg"], "answer": "(1)见解析(2)$sqrt{5}$", "solution": "null", "level": "九年级", "question": "(本题8分)(2021$cdot$浙江温州$cdot$一模)如图,在Rt$triangleABC$中,$angleACB=90^{circ},D$是边$BC$上一点,连接$AD,triangleACD$的外接圆$odotO$交$AB$于点$E$,点$F$是$AE$上一点,且$EF=CD$,连接$AF,DF$.(1)求证:$angleADF=angleB$;(2)若$AC=4,CB=8$,当点$E$是$AB$的中点时,求$AF$的长.", "options": [], "subject": "解析几何", "analysis": "【分析】(1)由圆周角定理及直角三角形的性质可得出答案;(2)连接$CE$,由勾股定理求出$AB=4sqrt{5}$,证明$triangleAFDsimtriangleACB$,由相似三角形的性质得出$frac{AF}{DF}=frac{AC}{BC}$,则可得出答案.(1)证明:$becauseEF=CD$,$thereforeangleFAE=angleCAD$,$thereforeangleFAE+angleDAE=angleCAD+angleDAE$,即$angleCAB=angleDAF$,$becauseangleACB=90^{circ}$,$thereforeAD$为$odotO$的直径,$angleCAB+angleB=90^{circ}$,$thereforeangleAFD=90^{circ}$,$thereforeangleDAF+angleADF=90^{circ}$,$thereforeangleADF=angleB$(2)解:连接$CE$,$becauseAC=4,BC=8$,$thereforeAB=sqrt{AC^{2}+BC^{2}}=sqrt{4^{2}+8^{2}}=4sqrt{5}$,$because$点$E$是$AB$的中点,$thereforeCE=frac{1}{2}AB=2sqrt{5}$,$becauseEF=CD$$thereforeDF=CE$,$thereforeDF=CE=2sqrt{5}$,$becauseangleADF=angleB,angleACB=angleAFD=90^{circ}$,$thereforetriangleAFDsimtriangleACB$$thereforefrac{AF}{DF}=frac{AC}{BC}$,$thereforefrac{AF}{2sqrt{5}}=frac{4}{8}$,$thereforeAF=sqrt{5}$.【点睛】本题考查了圆周角定理及直角三角形的性质,勾股定理,相似三角形的性质与判定,掌握以上知识是解题的关键."} {"id": "3127", "image": ["4065.jpg", "4066.jpg", "4067.jpg"], "answer": "(1)(1)$A(3,0),B(3,3),C(0,3);(2)left{begin{array}{l}b=2c=3end{array}right.$(2)$n=-frac{1}{3}left(m-frac{3}{2}right)^{2}+frac{3}{4}quad(0leqmleq3);frac{3}{4}$", "solution": "null", "level": "九年级", "question": "(本题9分)(2022$cdot$浙江嘉兴$cdot$九年级专题练习)如图1,已知在平面直角坐标系$xOy$中,四边形$OABC$是边长为3的正方形,其中顶点$A,C$分别在$x$轴的正半轴和$y$轴的正半轴上,抛物线$y=-x^{2}+bx+c$经过$A,C$两点,与$x$轴交于另一个点$D$.图1图2(2)求$b,c$的值.(2)若点$P$是边$BC$上的一个动点,连结$AP$,过点$P$作$PMperpAP$,交$y$轴于点$M$(如图2所示).当点$P$在$BC$上运动时,点$M$也随之运动.设$BP=m,CM=n$,试用含$m$的代数式表示$n$,并求出$n$的最大值.", "options": [], "subject": "解析几何", "analysis": "【分析】(1)(1)根据坐标与图形的性质即可求解;(2)利用待定系数法求解即可;(2)证明$RttriangleABPbacksimRttrianglePCM$,根据相似三角形的性质得到$n$关于$m$的二次函数,利用二次函数的性质即可求解.解:(1)$because$正方形$OABC$的边长为3,$therefore$点$A,B,C$的坐标分别为$A(3,0),B(3,3),C(0,3)$;(2)把点$A(3,0),C(0,3)$的坐标分别代入$y=-x^{2}+bx+c$,得$left{begin{array}{l}-9+3b+c=0c=3end{array}right.$,解得$left{begin{array}{l}b=2c=3end{array}right.$;(2)解:由题意,得$angleAPB=90^{circ}-angleMPC=anglePMC,angleB=anglePCM=90^{circ}$,$thereforeRttriangleABPsimRttrianglePCM$,$thereforefrac{AB}{PC}=frac{BP}{CM}$,即$frac{3}{3-m}=frac{m}{n}$.整理,得$n=-frac{1}{3}m^{2}+m$,即$n=-frac{1}{3}left(m-frac{3}{2}right)^{2}+frac{3}{4}quad(0leqmleq3)$.$therefore$当$m=frac{3}{2}$时,$n$的值最大,最大值是$frac{3}{4}$.【点睛】本题综合考查了正方形的性质,相似三角形的判定和性质,二次函数的性质,待定系数法求函数解析式,根据正方形的性质求出点$A,B,C$的坐标是解题的关键."} {"id": "3148", "image": ["4102.jpg", "4103.jpg", "4104.jpg", "4105.jpg", "4106.jpg", "4107.jpg"], "answer": "(1)见解析(2)见解析", "solution": "null", "level": "九年级", "question": "(本题8分)(2022-浙江$cdot$宁波外国语学校九年级阶段练习)如图,方格纸中的每个小正方形的边长都是$1,triangleABC$是格点三角形(顶点在方格顶点处).图1图2(1)在图1中画出一个格点$triangleAB_{1}C_{1}$,使得$triangleA_{1}B_{1}C_{1}$与$triangleABC$相似,周长之比为$2:1$;(2)在图2中画出一个格点$triangleA_{2}B_{2}C_{2}$,使得$triangleA_{2}B_{2}C_{2}$与$triangleABC$相似,面积之比为2:1.", "options": [], "subject": "解析几何", "analysis": "【分析】(1)根据相似三角形的性质,把$triangleABC$的边长扩大2倍即可,(2)根据相似三角形的性质,把$triangleABC$的边长扩大$sqrt{2}$倍即可.【详解】(1)解:如图,$triangleAB_{1}C_{1}$即为所求作.(2)如图,$triangleA_{2}B_{2}C_{2}$即为所求作.图1图2【点睛】本题考查作图-相似变换,相似三角形的判定与性质,三角形的面积等知识,解题的关键是理解题意,灵活运用所学知识解决问题."} {"id": "3175", "image": ["4148.jpg"], "answer": "(1)$k=6$(2)直线$MP$与$L$对称轴之间的距离为$frac{3}{2}$(3)$(t-2,2)(04)$(4)$Q$的坐标为$(4,0)$或$(-3,0)$或$left(frac{1}{4},0right)$或$left(-frac{1}{2},0right)$", "solution": "null", "level": "九年级", "question": "(本题8分)(2020$cdot$浙江$cdot$金华市南苑中学九年级期中)如图,抛物线$L$:$y=-frac{1}{2}(x-t)(x-t+4)$(常数$left.t>0right)$与$x$轴从左到右的交点为$B,A$,过线段$OA$的中点$M$作$MPperpx$轴,交双曲线$y=frac{k}{x}(k>0,x>0)$于点$P$,且$OAtimesMP=12$.(1)求$k$的值.(2)当$t=1$时,求$AB$的长,并求直线$MP$与$L$的对称轴之间的距离.(3)把$L$在直线$MP$左侧部分的图像(含与直线$MP$的交点)记为$G$,用$t$表示图像$G$最高点的坐标.(4)设$L$与$y$轴的交点为$N$,当$t=2$时,在$x$轴上是否存在一点$Q$,使$triangleONQ$与$trianglePMQ$相似,若存在,求出$Q$的坐标,若不存在,请说明理由.", "options": [], "subject": "解析几何", "analysis": "【分析】(1)根据$M$是$OA$的中点,以及$OAtimesMP=12$,即可求出$k$的值;(2)先求出抛物线的解析式,再求出$A,B$的坐标,以及对称轴即可求解;(3)根据对称轴与$MP$的位置关系,分情况讨论,要嘛在抛物线的顶点处取最值,要嘛是$MP$与抛物线的交点的纵坐标;(4)根据相似三角形的性质,对应边对应成比例分类讨论即可求出.(1)解:设$P(x,y)$,则$MP=y$,$becauseM$为$OA$的中点,$thereforeOA=2x$,$becauseOAcdotMP=12$,$therefore2xy=12$,$thereforexy=6$$thereforek=6$(2)解:当$t=1,y=0$时,$0=-frac{1}{2}(x-1)(x-1+4)$,解得$x=1$或$x=-3$.$thereforeA(1,0)、B(-3,0)$,$thereforeAB=4$$therefore$抛物线$L$的对称轴为直线$x=frac{1+(-3)}{2}=-1$,$becauseOA=1$$thereforeMP$为直线$x=frac{1}{2}$,$therefore$直线$MP$与$L$对称轴之间的距离为$frac{3}{2}$;(3)解:二次函数的对称轴为:$x=frac{t+t-4}{2}=t-2$点$M$的坐标为:$left(frac{t}{2},0right)$(1)当$frac{t}{2}4$时:$MP$在对称轴的左侧,$y$随$x$的增大而增大,$thereforeG$的坐标为:$left(frac{t}{2},t-frac{t^{2}}{8}right)$(2)当$frac{t}{2}geqt-2$,即$01$时:$frac{m}{m-1}=frac{1}{3},m=-frac{1}{2}$(舍);$01$时:$m(m-1)=12$,解得:$m=4$或$m=-3$(舍);$0图1图2(1)在图1中画出一个以$AB$为边,面积为4的$triangleABC$.(2)在图2中的线段$AB$上画出点$D$,使$AD:BD=2:3$.", "options": [], "subject": "解析几何", "analysis": "【分析】(1)画一个高为2,底边长为4的三角形即可;(2)连接格点$EF$交$AB$于点$D$,即可得到$AD:BD=2:3$,(1)解:如图1,$triangleABC$即为所求(答案不唯一);图1(2)解:如图2,点$D$即为所求.图2【点睛】本题考查作图-应用与设计作图,相似三角形的判定和性质等知识,解题的关键是学会利用数形结合的思想解决问题."} {"id": "3356", "image": ["4405.jpg"], "answer": "解: (1) $\\because \\overparen{A B}$ 的半径 $O A=2, O C \\perp A B$ 于点 $C, \\angle A O C=60^{\\circ}$,\n\n$\\therefore A C=O A \\cdot \\sin 60^{\\circ}=2 \\times \\frac{\\sqrt{3}}{2}=\\sqrt{3}$,\n\n$\\therefore A B=2 A C=2 \\sqrt{3}$;\n\n(2) $\\because O C \\perp A B, \\angle A O C=60^{\\circ}$,\n\n$\\therefore \\angle A O B=120^{\\circ}$,\n\n$\\because O A=2$,\n\n$\\therefore \\overparen{A B}$ 的长是: $\\frac{120 \\pi \\times 2}{180}=\\frac{4 \\pi}{3}$.\n【解析】本题考查弧长的计算以及锐角三角函数的定义, 解答本题的关键是明确题意, 利用数形结合的思想解答.\n\n(1)根据锐角三角函数的定义, 可以求得 $A C$ 的长, 然后即可得到 $A B$ 的长;\n\n(2)根据 $\\angle A O C=60^{\\circ}$, 可以得到 $\\angle A O B$ 的度数, 然后根据弧长公式计算即可.", "solution": "null", "level": "九年级", "question": "(本小题 8.0 分)\n\n如图, $\\overparen{A B}$ 的半径 $O A=2, O C \\perp A B$ 于点 $C, \\angle A O C=60^{\\circ}$.\n\n(1)求弦 $A B$ 的长.\n\n(2)求 $\\overparen{A B}$ 的长.\n\n", "options": [], "subject": "解析几何", "analysis": "解: (1) $\\because \\overparen{A B}$ 的半径 $O A=2, O C \\perp A B$ 于点 $C, \\angle A O C=60^{\\circ}$,\n\n$\\therefore A C=O A \\cdot \\sin 60^{\\circ}=2 \\times \\frac{\\sqrt{3}}{2}=\\sqrt{3}$,\n\n$\\therefore A B=2 A C=2 \\sqrt{3}$;\n\n(2) $\\because O C \\perp A B, \\angle A O C=60^{\\circ}$,\n\n$\\therefore \\angle A O B=120^{\\circ}$,\n\n$\\because O A=2$,\n\n$\\therefore \\overparen{A B}$ 的长是: $\\frac{120 \\pi \\times 2}{180}=\\frac{4 \\pi}{3}$.\n【解析】本题考查弧长的计算以及锐角三角函数的定义, 解答本题的关键是明确题意, 利用数形结合的思想解答.\n\n(1)根据锐角三角函数的定义, 可以求得 $A C$ 的长, 然后即可得到 $A B$ 的长;\n\n(2)根据 $\\angle A O C=60^{\\circ}$, 可以得到 $\\angle A O B$ 的度数, 然后根据弧长公式计算即可."} {"id": "3358", "image": ["4407.jpg"], "answer": "解: $\\because$ 四边形 $A B C D$ 是矩形,\n\n$\\therefore \\angle B=90^{\\circ}$.\n\n$\\because E$ 是 $B C$ 的中点, $B C=2 \\sqrt{5}$,\n\n$\\therefore B E=C E=\\frac{1}{2} B C=\\sqrt{5}$.\n\n$\\therefore A E=\\sqrt{A B^{2}+B E^{2}}=\\sqrt{2^{2}+(\\sqrt{5})^{2}}=3$.\n\n由翻折的性质得 $\\angle A E F=\\angle A E B, E F=B E=\\sqrt{5}$,\n\n$\\therefore E F=C E$.\n$\\therefore \\angle E F C=\\angle E C F$.\n\n$\\because \\angle B E F=\\angle E F C+\\angle E C F$,\n\n$\\therefore \\angle A E B=\\angle E C F$.\n\n$\\therefore \\cos \\angle E C F=\\cos \\angle A E B=\\frac{B E}{A E}=\\frac{\\sqrt{5}}{3}$.\n\n【解析】见答案", "solution": "null", "level": "九年级", "question": "(本小题 8.0 分)\n\n如图, 在矩形 $A B C D$ 中, $A B=2, B C=2 \\sqrt{5}, E$ 是 $B C$ 的中点, 将 $\\triangle A B E$ 沿直线 $A E$ 翻折, 点 $B$ 落在点 $F$ 处, 连接 $C F$, 求 $\\cos \\angle E C F$ 的值.\n\n", "options": [], "subject": "解析几何", "analysis": "解: $\\because$ 四边形 $A B C D$ 是矩形,\n\n$\\therefore \\angle B=90^{\\circ}$.\n\n$\\because E$ 是 $B C$ 的中点, $B C=2 \\sqrt{5}$,\n\n$\\therefore B E=C E=\\frac{1}{2} B C=\\sqrt{5}$.\n\n$\\therefore A E=\\sqrt{A B^{2}+B E^{2}}=\\sqrt{2^{2}+(\\sqrt{5})^{2}}=3$.\n\n由翻折的性质得 $\\angle A E F=\\angle A E B, E F=B E=\\sqrt{5}$,\n\n$\\therefore E F=C E$.\n$\\therefore \\angle E F C=\\angle E C F$.\n\n$\\because \\angle B E F=\\angle E F C+\\angle E C F$,\n\n$\\therefore \\angle A E B=\\angle E C F$.\n\n$\\therefore \\cos \\angle E C F=\\cos \\angle A E B=\\frac{B E}{A E}=\\frac{\\sqrt{5}}{3}$.\n\n【解析】见答案"} {"id": "3363", "image": ["4414.jpg", "4415.jpg", "4415.jpg"], "answer": "解: 过点 $A$ 作 $A D \\perp B C$ 于 $D$, 如图所示:\n\n\n\n由题意, 得 $\\angle A B C=180^{\\circ}-75^{\\circ}-45^{\\circ}=60^{\\circ}$.\n\n$\\because A D \\perp B C$,\n\n$\\therefore \\angle A D B=\\angle A D C=90^{\\circ}$,\n\n在Rt $\\triangle A B D$ 中, $\\angle D A B=90^{\\circ}-60^{\\circ}=30^{\\circ}, A B=80$,\n\n$\\therefore B D=40, A D=40 \\sqrt{3}$,\n\n$\\because \\angle C A B=30^{\\circ}+45^{\\circ}=75^{\\circ}$,\n\n$\\therefore \\angle D A C=\\angle C A B-\\angle D A B=75^{\\circ}-30^{\\circ}=45^{\\circ}$,\n\n$\\therefore \\triangle A D C$ 是等腰直角三角形,\n\n$\\therefore A C=\\sqrt{2} A D=\\sqrt{2} \\times 40 \\sqrt{3}=40 \\sqrt{6}$ (海里).\n\n答: 货船与港口 $A$ 之间的距离是 $40 \\sqrt{6}$ 海里.\n\n【解析】本题考查了解直角三角形的应用一方位角问题、等腰直角三角形的判定与性质等知识;通过作辅助线得出直角三角形是解题的关键.\n\n过点 $A$ 作 $A D \\perp B C$ 于 $D$, 求出 $\\angle A B C=60^{\\circ}$, 在 $R t \\triangle A B D$ 中, $\\angle D A B=30^{\\circ}$, 求出 $A D=40 \\sqrt{3}$, 求出 $\\angle D A C=\\angle C A B-\\angle D A B=45^{\\circ}$, 则 $\\triangle A D C$ 是等腰直角三角形, 得出 $A C=\\sqrt{2} A D=40 \\sqrt{6}$ 海里即可.", "solution": "null", "level": "九年级", "question": "(本小题 8.0 分)\n\n如图, 我国某海域有 $A, B$ 两个港口, 相距 80 海里, 港口 $B$ 在港口 $A$ 的东北方向, 点 $C$ 处有一艘货船, 该货船在港口 $A$ 的北偏西 $30^{\\circ}$ 方向, 在港口 $B$ 的北偏西 $75^{\\circ}$ 方向, 求货船与港口 $A$ 之间的距离. (结果保留根号)\n\n", "options": [], "subject": "解析几何", "analysis": "解: 过点 $A$ 作 $A D \\perp B C$ 于 $D$, 如图所示:\n\n\n\n由题意, 得 $\\angle A B C=180^{\\circ}-75^{\\circ}-45^{\\circ}=60^{\\circ}$.\n\n$\\because A D \\perp B C$,\n\n$\\therefore \\angle A D B=\\angle A D C=90^{\\circ}$,\n\n在Rt $\\triangle A B D$ 中, $\\angle D A B=90^{\\circ}-60^{\\circ}=30^{\\circ}, A B=80$,\n\n$\\therefore B D=40, A D=40 \\sqrt{3}$,\n\n$\\because \\angle C A B=30^{\\circ}+45^{\\circ}=75^{\\circ}$,\n\n$\\therefore \\angle D A C=\\angle C A B-\\angle D A B=75^{\\circ}-30^{\\circ}=45^{\\circ}$,\n\n$\\therefore \\triangle A D C$ 是等腰直角三角形,\n\n$\\therefore A C=\\sqrt{2} A D=\\sqrt{2} \\times 40 \\sqrt{3}=40 \\sqrt{6}$ (海里).\n\n答: 货船与港口 $A$ 之间的距离是 $40 \\sqrt{6}$ 海里.\n\n【解析】本题考查了解直角三角形的应用一方位角问题、等腰直角三角形的判定与性质等知识;通过作辅助线得出直角三角形是解题的关键.\n\n过点 $A$ 作 $A D \\perp B C$ 于 $D$, 求出 $\\angle A B C=60^{\\circ}$, 在 $R t \\triangle A B D$ 中, $\\angle D A B=30^{\\circ}$, 求出 $A D=40 \\sqrt{3}$, 求出 $\\angle D A C=\\angle C A B-\\angle D A B=45^{\\circ}$, 则 $\\triangle A D C$ 是等腰直角三角形, 得出 $A C=\\sqrt{2} A D=40 \\sqrt{6}$ 海里即可."} {"id": "3365", "image": ["4419.jpg", "4420.jpg", "4420.jpg"], "answer": "解:作 $P Q \\perp A B$ 于点 $Q$, 如图,\n\n\n\n$\\therefore \\angle P Q A=\\angle P Q B=90^{\\circ}$,\n\n由题意可得: $\\angle A=60^{\\circ}, \\angle B=37^{\\circ}, P A=70 \\times 6=420 m$,\n\n$\\therefore A Q=P A \\times \\cos A=210 \\mathrm{~m}, P Q=P A \\times \\sin A=210 \\sqrt{3} \\mathrm{~m}$,\n\n$$\n\\therefore Q B=\\frac{P Q}{\\tan B}=280 \\sqrt{3}\n$$\n\n$\\therefore P A+A B=420+210+280 \\sqrt{3} \\approx 1106 m$.\n\n答: 小明步行的总路程 $1106 \\mathrm{~m}$.\n\n【解析】本题考查的是解直角三角形的应用有关知识, 作 $P Q \\perp A B$ 于点 $Q$ 可得 $\\angle P Q A=\\angle P Q B=90^{\\circ}$,然后再利用锐角三角形函数的定义计算出 $A Q, Q B$, 最后利用 $P A+A B$ 计算即可.", "solution": "null", "level": "九年级", "question": "(本小题 8.0 分)\n如图, 小明从 $P$ 处出发, 沿北偏东 $60^{\\circ}$ 方向以 $70 \\mathrm{~m} / \\mathrm{min}$ 的速度步行 $6 \\mathrm{~min}$ 后到达 $A$ 处, 接着向正南方向步行一段时间后到达终点 $B$ 处, 在 $B$ 处观测到出发时所在的 $P$ 处在北偏西 $37^{\\circ}$ 方向上. 求小明步行的总路程 (精确到 $1 m$ ). 参考数据: $\\sin 37^{\\circ} \\approx 0.6, \\cos 37^{\\circ} \\approx 0.8, \\tan 37^{\\circ} \\approx 0.75, \\sqrt{2} \\approx$ $1.4, \\sqrt{3} \\approx 1.7$\n\n", "options": [], "subject": "解析几何", "analysis": "解:作 $P Q \\perp A B$ 于点 $Q$, 如图,\n\n\n\n$\\therefore \\angle P Q A=\\angle P Q B=90^{\\circ}$,\n\n由题意可得: $\\angle A=60^{\\circ}, \\angle B=37^{\\circ}, P A=70 \\times 6=420 m$,\n\n$\\therefore A Q=P A \\times \\cos A=210 \\mathrm{~m}, P Q=P A \\times \\sin A=210 \\sqrt{3} \\mathrm{~m}$,\n\n$$\n\\therefore Q B=\\frac{P Q}{\\tan B}=280 \\sqrt{3}\n$$\n\n$\\therefore P A+A B=420+210+280 \\sqrt{3} \\approx 1106 m$.\n\n答: 小明步行的总路程 $1106 \\mathrm{~m}$.\n\n【解析】本题考查的是解直角三角形的应用有关知识, 作 $P Q \\perp A B$ 于点 $Q$ 可得 $\\angle P Q A=\\angle P Q B=90^{\\circ}$,然后再利用锐角三角形函数的定义计算出 $A Q, Q B$, 最后利用 $P A+A B$ 计算即可."} {"id": "3386", "image": ["4456.jpg", "4457.jpg", "4458.jpg", "4459.jpg", "4460.jpg", "4457.jpg", "4458.jpg", "4459.jpg", "4460.jpg"], "answer": "(1)解:过点 $D$ 作 $D N \\perp x$ 轴于点 $N$,\n\n令 $y=0$, 得: $a x^{2}-2 a x-3 a=0$,\n\n$\\because a>0$,\n\n$\\therefore x^{2}-2 x-3=0$,\n\n解得: $x_{1}=3, x_{2}=-1$,\n\n$\\therefore A(-1,0), B(3,0)$,\n\n将 $B(3,0)$ 代入 $y=-\\frac{\\sqrt{3}}{3} x+b$,\n\n解得: $b=\\sqrt{3}$,\n\n$\\therefore y=-\\frac{\\sqrt{3}}{3} x+\\sqrt{3}$;\n\n$\\therefore E(\\sqrt{3}, 0)$, 易证得 $\\triangle B O E \\sim \\triangle B N D$,\n\n$\\therefore \\frac{O E}{N D}=\\frac{B O}{B N}=\\frac{B E}{B D}$,\n\n又 $\\because \\frac{D E}{B E}=\\frac{2}{3}$,\n\n$\\therefore \\frac{B E}{B D}=\\frac{3}{5}$,\n\n$\\therefore B N=5, D N=\\frac{5}{3} \\sqrt{3}$,\n\n$\\therefore D\\left(-2, \\frac{5}{3} \\sqrt{3}\\right)$,\n\n将点 $D\\left(-2, \\frac{5}{3} \\sqrt{3}\\right)$ 代入 $y=a x^{2}-2 a x-3 a$,\n\n解得 $a=\\frac{\\sqrt{3}}{3}$,\n\n$\\therefore y=\\frac{\\sqrt{3}}{3} x^{2}-\\frac{2}{3} \\sqrt{3} x-\\sqrt{3}$\n\n\n\n(2) 由题意, 动点 $M$ 运动的路径为折线 $A M+M D$, 运动时间: $t=A M+\\frac{1}{2} D M$, 由题意可知 $\\angle A B D=$ $30^{\\circ}$, 可转化 $\\frac{1}{2} D M$ 为 $D M$ 投影到 $y$ 轴的线段长, 如图, 过点 $D$ 作 $D H \\perp y$ 轴于点 $H$, 过点 $A$ 作 $A G \\perp D H$于点 $G$, 交 $B D$ 于点 $P$, 则点 $P$ 即为所求.\n\n$\\because$ 直线 $B D$ 的解析式为 $y=-\\frac{\\sqrt{3}}{3} x+\\sqrt{3}, \\therefore \\angle P B A=\\angle P D G=30^{\\circ}$,\n\n$\\because A B=4$,\n\n$\\therefore A P=\\frac{4 \\sqrt{3}}{3}$,\n\n$\\therefore$ 点 $P$ 的坐标为 $\\left(-1, \\frac{4}{3} \\sqrt{3}\\right)$.\n\n(3)解: 由题意易得 $\\triangle A C B$ 为直角三角形, $B C=2 \\sqrt{3}, A B=4, A C=2$, $O C=O E=\\sqrt{3}, \\quad \\therefore \\angle A C B=90^{\\circ}, \\angle A B C=\\angle E B O=30^{\\circ}$,将 $\\triangle A B C$ 绕点 $B$ 顺时针旋转 $\\alpha\\left(0^{\\circ}<\\alpha<180^{\\circ}\\right)$,当点 $A$ 的对应点 $A^{\\prime}$ 落在 $\\triangle E C B$ 的边所在直线上时分以下三种情况:\n\n(i)如图,\n\n\n\n当点 $A$ 的对应点 $A^{\\prime}$ 落在 $B E$ 边所在直线上时, $B C^{\\prime}=B C=2 \\sqrt{3}$,\n\n可求得点 $C^{\\prime}$ 的坐标为 $C_{1}{ }^{\\prime}(3-2 \\sqrt{3}, 0)$;\n\n(ii)如图, 过点 $C^{\\prime}$ 作 $C^{\\prime} H$ 垂直 $x$ 轴, 过 $A^{\\prime}$ 作 $A^{\\prime} F$ 垂直 $C^{\\prime} H$,\n\n\n\n当点 $A$ 的对应点 $A^{\\prime}$ 落在 $E C$ 边所在直线上时, $A B=A^{\\prime} B=4, A^{\\prime} O=\\sqrt{7}$,\n\n$\\triangle A^{\\prime} F C^{\\prime} \\sim \\triangle C^{\\prime} H B$,\n\n$\\therefore \\frac{C^{\\prime} H}{A^{\\prime} F}=\\frac{B H}{F C^{\\prime}}=\\frac{B C^{\\prime}}{A^{\\prime} C^{\\prime}}=\\sqrt{3}$,\n\n$\\therefore C^{\\prime} H=\\sqrt{3} A^{\\prime} F, B H=3+O H=\\sqrt{3} F C^{\\prime}=\\sqrt{3}\\left(A^{\\prime} O-C^{\\prime} H\\right)$,\n\n$\\therefore O H=\\frac{\\sqrt{21}-3}{4}$,\n\n可求得点 $C^{\\prime}$ 的坐标为 $C_{2}{ }^{\\prime}\\left(\\frac{3-\\sqrt{21}}{4}, \\frac{3 \\sqrt{7}-3 \\sqrt{3}}{4}\\right)$;\n\n(iii)如图,\n\n\n\n当点 $A$ 的对应点 $A^{\\prime}$ 落在 $B C$ 边所在直线上时,\n\n在 $R t \\triangle B C^{\\prime} H$ 中, 由 $B C^{\\prime}=B C=2 \\sqrt{3}, \\angle C^{\\prime} B H=60^{\\circ}$,\n\n可用锐角三角函数求得 $C^{\\prime} H=3, B H=\\sqrt{3}$,\n\n可求得点 $C^{\\prime}$ 的坐标为 $C_{3}{ }^{\\prime}(3+\\sqrt{3}, 3)$;\n\n综上所述: 当点 $A$ 的对应点 $A^{\\prime}$ 落在 $\\triangle E C B$ 的边所在直线上时, 对应的点 $C^{\\prime}$ 的坐标为 $(3-2 \\sqrt{3}, 0)$ 或 $\\left(\\frac{3-\\sqrt{21}}{4}, \\frac{3 \\sqrt{7}-3 \\sqrt{3}}{4}\\right)$ 或 $(3+\\sqrt{3}, 3)$.\n\n【解析】本题考查一次函数的图像, 性质和用待定系数法求一次函数的解析式, 二次函数的图像,性质和用待定系数法求二次函数的解析式, 平面直角坐标系中点坐标, 相似三角形的性质和判定,特殊角的三角函数值, 解直角三角形, 旋转的性质, 分类讨论的数学思想.\n\n(1)首先求出点 $A 、 B$ 坐标, 然后求出直线 $B D$ 的解析式, 求得点 $D$ 坐标, 代入抛物线解析式, 求得 $a$的值;\n\n(2) 由题意, 动点 $M$ 运动的路径为折线 $A M+M D$, 运动时间: $t=A M+\\frac{1}{2} D M$, 由题意可知 $\\angle A B D=$ $30^{\\circ}$, 可转化 $\\frac{1}{2} D M$ 为 $D M$ 投影到 $y$ 轴的线段长, 如图, 过点 $D$ 作 $D H \\perp y$ 轴于点 $H$, 过点 $A$ 作 $A G \\perp D H$于点 $G$, 交 $B D$ 于点 $P$, 则点 $P$ 即为所求, 进而根据直线 $B D$ 的表达式得到角的度数, 求出 $A P$ 的长,即可得解;\n\n(3) 将 $\\triangle A B C$ 绕点 $B$ 顺时针旋转 $\\alpha\\left(0^{\\circ}<\\alpha<180^{\\circ}\\right)$, 当点 $A$ 的对应点 $A^{\\prime}$ 落在 $\\triangle E C B$ 的边所在直线上时分以下三种情况:\n\n(i)如图, 当点 $A$ 的对应点 $A^{\\prime}$ 落在 $B E$ 边所在直线上时;\n\n(ii)如图, 当点 $A$ 的对应点 $A^{\\prime}$ 落在 $E C$ 边所在直线上时,\n\n(iii)如图, 当点 $A$ 的对应点 $A^{\\prime}$ 落在 $B C$ 边所在直线上时,利用解直角三角形即可解答.", "solution": "null", "level": "九年级", "question": "(本小题 8.0 分)\n\n如图, 已知抛物线 $y=a x^{2}-2 a x-3 a(a$ 为常数, 且 $a>0)$ 与 $x$ 轴从左至右依次交于 $A, B$ 两点,与 $y$ 轴交于点 $C$, 经过点 $B$ 的直线 $y=-\\frac{\\sqrt{3}}{3} x+b$ 与抛物线的另一交点为 $D$, 与 $y$ 轴交于点 $E$, 且 $D E: B E=2: 3$.\n\n\n(备用图)\n\n(1)求抛物线的函数表达式;\n\n(2) 设 $P$ 为线段 $B D$ 上一点 (不含端点), 连接 $A P$, 一动点 $M$ 从点 $A$ 出发, 沿线段 $A P$ 以每秒 1 个单位的速度运动到 $P$, 再沿线段 $P D$ 以每秒 2 个单位的速度运动到 $D$ 后停止.当点 $P$ 的坐标是多少时,点 $M$ 在整个运动过程中用时最少?\n\n(3)将 $\\triangle A B C$ 绕点 $B$ 顺时针旋转 $\\alpha\\left(0^{\\circ}<\\alpha<180^{\\circ}\\right.$ , 当点 $A$ 的对应点 $A^{\\prime}$ 落在 $\\triangle E C B$ 的边所在直线上时, 求此时点 $C$ 的对应点 $C^{\\prime}$ 的坐标.", "options": [], "subject": "解析几何", "analysis": "(1)解:过点 $D$ 作 $D N \\perp x$ 轴于点 $N$,\n\n令 $y=0$, 得: $a x^{2}-2 a x-3 a=0$,\n\n$\\because a>0$,\n\n$\\therefore x^{2}-2 x-3=0$,\n\n解得: $x_{1}=3, x_{2}=-1$,\n\n$\\therefore A(-1,0), B(3,0)$,\n\n将 $B(3,0)$ 代入 $y=-\\frac{\\sqrt{3}}{3} x+b$,\n\n解得: $b=\\sqrt{3}$,\n\n$\\therefore y=-\\frac{\\sqrt{3}}{3} x+\\sqrt{3}$;\n\n$\\therefore E(\\sqrt{3}, 0)$, 易证得 $\\triangle B O E \\sim \\triangle B N D$,\n\n$\\therefore \\frac{O E}{N D}=\\frac{B O}{B N}=\\frac{B E}{B D}$,\n\n又 $\\because \\frac{D E}{B E}=\\frac{2}{3}$,\n\n$\\therefore \\frac{B E}{B D}=\\frac{3}{5}$,\n\n$\\therefore B N=5, D N=\\frac{5}{3} \\sqrt{3}$,\n\n$\\therefore D\\left(-2, \\frac{5}{3} \\sqrt{3}\\right)$,\n\n将点 $D\\left(-2, \\frac{5}{3} \\sqrt{3}\\right)$ 代入 $y=a x^{2}-2 a x-3 a$,\n\n解得 $a=\\frac{\\sqrt{3}}{3}$,\n\n$\\therefore y=\\frac{\\sqrt{3}}{3} x^{2}-\\frac{2}{3} \\sqrt{3} x-\\sqrt{3}$\n\n\n\n(2) 由题意, 动点 $M$ 运动的路径为折线 $A M+M D$, 运动时间: $t=A M+\\frac{1}{2} D M$, 由题意可知 $\\angle A B D=$ $30^{\\circ}$, 可转化 $\\frac{1}{2} D M$ 为 $D M$ 投影到 $y$ 轴的线段长, 如图, 过点 $D$ 作 $D H \\perp y$ 轴于点 $H$, 过点 $A$ 作 $A G \\perp D H$于点 $G$, 交 $B D$ 于点 $P$, 则点 $P$ 即为所求.\n\n$\\because$ 直线 $B D$ 的解析式为 $y=-\\frac{\\sqrt{3}}{3} x+\\sqrt{3}, \\therefore \\angle P B A=\\angle P D G=30^{\\circ}$,\n\n$\\because A B=4$,\n\n$\\therefore A P=\\frac{4 \\sqrt{3}}{3}$,\n\n$\\therefore$ 点 $P$ 的坐标为 $\\left(-1, \\frac{4}{3} \\sqrt{3}\\right)$.\n\n(3)解: 由题意易得 $\\triangle A C B$ 为直角三角形, $B C=2 \\sqrt{3}, A B=4, A C=2$, $O C=O E=\\sqrt{3}, \\quad \\therefore \\angle A C B=90^{\\circ}, \\angle A B C=\\angle E B O=30^{\\circ}$,将 $\\triangle A B C$ 绕点 $B$ 顺时针旋转 $\\alpha\\left(0^{\\circ}<\\alpha<180^{\\circ}\\right)$,当点 $A$ 的对应点 $A^{\\prime}$ 落在 $\\triangle E C B$ 的边所在直线上时分以下三种情况:\n\n(i)如图,\n\n\n\n当点 $A$ 的对应点 $A^{\\prime}$ 落在 $B E$ 边所在直线上时, $B C^{\\prime}=B C=2 \\sqrt{3}$,\n\n可求得点 $C^{\\prime}$ 的坐标为 $C_{1}{ }^{\\prime}(3-2 \\sqrt{3}, 0)$;\n\n(ii)如图, 过点 $C^{\\prime}$ 作 $C^{\\prime} H$ 垂直 $x$ 轴, 过 $A^{\\prime}$ 作 $A^{\\prime} F$ 垂直 $C^{\\prime} H$,\n\n\n\n当点 $A$ 的对应点 $A^{\\prime}$ 落在 $E C$ 边所在直线上时, $A B=A^{\\prime} B=4, A^{\\prime} O=\\sqrt{7}$,\n\n$\\triangle A^{\\prime} F C^{\\prime} \\sim \\triangle C^{\\prime} H B$,\n\n$\\therefore \\frac{C^{\\prime} H}{A^{\\prime} F}=\\frac{B H}{F C^{\\prime}}=\\frac{B C^{\\prime}}{A^{\\prime} C^{\\prime}}=\\sqrt{3}$,\n\n$\\therefore C^{\\prime} H=\\sqrt{3} A^{\\prime} F, B H=3+O H=\\sqrt{3} F C^{\\prime}=\\sqrt{3}\\left(A^{\\prime} O-C^{\\prime} H\\right)$,\n\n$\\therefore O H=\\frac{\\sqrt{21}-3}{4}$,\n\n可求得点 $C^{\\prime}$ 的坐标为 $C_{2}{ }^{\\prime}\\left(\\frac{3-\\sqrt{21}}{4}, \\frac{3 \\sqrt{7}-3 \\sqrt{3}}{4}\\right)$;\n\n(iii)如图,\n\n\n\n当点 $A$ 的对应点 $A^{\\prime}$ 落在 $B C$ 边所在直线上时,\n\n在 $R t \\triangle B C^{\\prime} H$ 中, 由 $B C^{\\prime}=B C=2 \\sqrt{3}, \\angle C^{\\prime} B H=60^{\\circ}$,\n\n可用锐角三角函数求得 $C^{\\prime} H=3, B H=\\sqrt{3}$,\n\n可求得点 $C^{\\prime}$ 的坐标为 $C_{3}{ }^{\\prime}(3+\\sqrt{3}, 3)$;\n\n综上所述: 当点 $A$ 的对应点 $A^{\\prime}$ 落在 $\\triangle E C B$ 的边所在直线上时, 对应的点 $C^{\\prime}$ 的坐标为 $(3-2 \\sqrt{3}, 0)$ 或 $\\left(\\frac{3-\\sqrt{21}}{4}, \\frac{3 \\sqrt{7}-3 \\sqrt{3}}{4}\\right)$ 或 $(3+\\sqrt{3}, 3)$.\n\n【解析】本题考查一次函数的图像, 性质和用待定系数法求一次函数的解析式, 二次函数的图像,性质和用待定系数法求二次函数的解析式, 平面直角坐标系中点坐标, 相似三角形的性质和判定,特殊角的三角函数值, 解直角三角形, 旋转的性质, 分类讨论的数学思想.\n\n(1)首先求出点 $A 、 B$ 坐标, 然后求出直线 $B D$ 的解析式, 求得点 $D$ 坐标, 代入抛物线解析式, 求得 $a$的值;\n\n(2) 由题意, 动点 $M$ 运动的路径为折线 $A M+M D$, 运动时间: $t=A M+\\frac{1}{2} D M$, 由题意可知 $\\angle A B D=$ $30^{\\circ}$, 可转化 $\\frac{1}{2} D M$ 为 $D M$ 投影到 $y$ 轴的线段长, 如图, 过点 $D$ 作 $D H \\perp y$ 轴于点 $H$, 过点 $A$ 作 $A G \\perp D H$于点 $G$, 交 $B D$ 于点 $P$, 则点 $P$ 即为所求, 进而根据直线 $B D$ 的表达式得到角的度数, 求出 $A P$ 的长,即可得解;\n\n(3) 将 $\\triangle A B C$ 绕点 $B$ 顺时针旋转 $\\alpha\\left(0^{\\circ}<\\alpha<180^{\\circ}\\right)$, 当点 $A$ 的对应点 $A^{\\prime}$ 落在 $\\triangle E C B$ 的边所在直线上时分以下三种情况:\n\n(i)如图, 当点 $A$ 的对应点 $A^{\\prime}$ 落在 $B E$ 边所在直线上时;\n\n(ii)如图, 当点 $A$ 的对应点 $A^{\\prime}$ 落在 $E C$ 边所在直线上时,\n\n(iii)如图, 当点 $A$ 的对应点 $A^{\\prime}$ 落在 $B C$ 边所在直线上时,利用解直角三角形即可解答."} {"id": "3388", "image": ["4467.jpg", "4468.jpg", "4469.jpg", "4470.jpg", "4471.jpg", "4469.jpg", "4470.jpg", "4471.jpg"], "answer": "解: (1)连接 $D F, C E$, 如图所示:\n\n\n\n$\\because E$ 为 $A B$ 中点,\n\n$\\therefore A E=A F=\\frac{1}{2} A B$,\n\n$\\therefore E F=A B$,\n\n$\\because$ 四边形 $A B C D$ 是菱形,\n\n$\\therefore E F=A B=C D, C D / / E F$,\n\n$\\therefore$ 四边形 $D F E C$ 是平行四边形.\n\n(2)作 $C H \\perp B H$, 设 $A E=F A=m$, 如图所示,\n\n\n\n$\\because$ 四边形 $A B C D$ 是菱形,\n\n$\\therefore C D / / E F$,\n\n$\\therefore \\triangle C D G \\sim \\triangle F E G$,\n\n$\\therefore \\frac{C D}{C G}=\\frac{E F}{F G}$,\n\n$\\therefore F G=E F=2 m$,\n\n在Rt $\\triangle C B H$ 中, $\\angle C B H=60^{\\circ}, B C=2$,\n\n$\\therefore \\angle B C H=30^{\\circ}$,\n\n$\\therefore B H=1, C H=\\sqrt{3}$,\n\n在Rt $\\triangle C F H$ 中, $C F=2+2 m, C H=\\sqrt{3}, F H=3+m$,\n\n$C F^{2}=C H^{2}+F H^{2}$,\n\n即 $(2+2 m)^{2}=(\\sqrt{3})^{2}+(3+m)^{2}$,\n\n整理得: $3 m^{2}+2 m-8=0$,\n\n解得: $m_{1}=\\frac{4}{3}, m_{2}=-2$ (舍去),\n\n$\\therefore A E=\\frac{4}{3}$.\n\n(3)因 $E$ 点沿线段 $A B$ 运动, $F$ 点沿线段 $B A$ 的延长线运动, 并且 $C D / / A B$, 线段 $E D$ 与线段 $C F$ 的交点 $G$点运动轨迹为线段 $A G$, 运动刚开始时, $A 、 E 、 F 、 G$ 四点重合, 当 $E$ 点与 $B$ 点重合时, $G$ 点运动到极限位置, 所以 $G$ 点轨迹为线段 $A G$.\n\n如图所示, 作 $G H \\perp A B$,\n\n\n$\\because$ 四边形 $A B C D$ 为菱形, $\\angle D A B=60^{\\circ}, A B=2$,\n\n$\\therefore C D / / B F, B D=2$,\n\n$\\therefore \\triangle C D G \\sim \\triangle F B G$,\n\n$\\therefore \\frac{C D}{B F}=\\frac{D G}{B G}$, 即 $B G=2 D G$,\n\n$\\because B G+D G=B D=2$,\n\n$\\therefore B G=\\frac{4}{3}$,\n\n在Rt $\\triangle G H B$ 中, $B G=\\frac{4}{3}, \\angle D B A=60^{\\circ}$,\n\n$\\therefore \\angle H G B=30^{\\circ}$,\n\n$\\therefore B H=\\frac{2}{3}, G H=\\frac{2 \\sqrt{3}}{3}$,\n\n在Rt $\\triangle A H G$ 中, $A H=2-\\frac{2}{3}=\\frac{4}{3}, G H=\\frac{2 \\sqrt{3}}{3}$,\n\n$A G^{2}=\\left(\\frac{4}{3}\\right)^{2}+\\left(\\frac{2 \\sqrt{3}}{3}\\right)^{2}=\\frac{28}{9}$,\n\n$\\therefore A G=\\frac{2 \\sqrt{7}}{3}$.\n\n$\\therefore G$ 点运动路径的长度为 $\\frac{2 \\sqrt{7}}{3}$.\n\n【解析】\n\n【分析】\n\n本题主要考查平行四边形的判定, 菱形的性质, 相似三角形的判定和性质, 含 $30^{\\circ}$ 角的直角三角形的性质以及勾股定理,根据题意表示出相关线段的长度是解题的关键..\n\n(1)利用平行四边形的判定定理: 一组对边平行且相等的四边形是平行四边形,\n\n(2)作 $C H \\perp B H$, 设 $A E=F A=m$, 利用三角形相似, 求出此时 $F G$ 的长, 进而表示出相关线段的长度, 再在 $R t \\triangle C F H$ 中, 利用勾股定理求解即可,\n\n(3)连接 $A G$, 作 $G H \\perp A B$, 分析出点 $G$ 的运动路径是线段 $A G$, 先在 $R t \\triangle G H B$ 中, 求出 $B H=\\frac{2}{3}, G H=$ $\\frac{2 \\sqrt{3}}{3}$, 再在 $R t \\triangle A H G$ 中, 利用勾股定理求出 $A G$ 的长度即可.", "solution": "null", "level": "九年级", "question": "(本小题 8.0 分)\n\n如图, 在菱形 $A B C D$ 中, $\\angle D A B=60^{\\circ}, A B=2$, 点 $E$ 为边 $A B$ 上一个动点, 延长 $B A$ 到点 $F$, 使 $A F=A E ,$ 且 $C F 、 D E$ 相交于点 $G$.\n\n\n\n\n\n备用图\n\n(1)当点 $E$ 运动到 $A B$ 中点时, 证明: 四边形 $D F E C$ 是平行四边形;\n\n(2)当 $C G=2$ 时,求 $A E$ 的长;\n\n(3)当点 $E$ 从点 $A$ 开始向右运动到点 $B$ 时, 求点 $G$ 运动路径的长度.", "options": [], "subject": "解析几何", "analysis": "解: (1)连接 $D F, C E$, 如图所示:\n\n\n\n$\\because E$ 为 $A B$ 中点,\n\n$\\therefore A E=A F=\\frac{1}{2} A B$,\n\n$\\therefore E F=A B$,\n\n$\\because$ 四边形 $A B C D$ 是菱形,\n\n$\\therefore E F=A B=C D, C D / / E F$,\n\n$\\therefore$ 四边形 $D F E C$ 是平行四边形.\n\n(2)作 $C H \\perp B H$, 设 $A E=F A=m$, 如图所示,\n\n\n\n$\\because$ 四边形 $A B C D$ 是菱形,\n\n$\\therefore C D / / E F$,\n\n$\\therefore \\triangle C D G \\sim \\triangle F E G$,\n\n$\\therefore \\frac{C D}{C G}=\\frac{E F}{F G}$,\n\n$\\therefore F G=E F=2 m$,\n\n在Rt $\\triangle C B H$ 中, $\\angle C B H=60^{\\circ}, B C=2$,\n\n$\\therefore \\angle B C H=30^{\\circ}$,\n\n$\\therefore B H=1, C H=\\sqrt{3}$,\n\n在Rt $\\triangle C F H$ 中, $C F=2+2 m, C H=\\sqrt{3}, F H=3+m$,\n\n$C F^{2}=C H^{2}+F H^{2}$,\n\n即 $(2+2 m)^{2}=(\\sqrt{3})^{2}+(3+m)^{2}$,\n\n整理得: $3 m^{2}+2 m-8=0$,\n\n解得: $m_{1}=\\frac{4}{3}, m_{2}=-2$ (舍去),\n\n$\\therefore A E=\\frac{4}{3}$.\n\n(3)因 $E$ 点沿线段 $A B$ 运动, $F$ 点沿线段 $B A$ 的延长线运动, 并且 $C D / / A B$, 线段 $E D$ 与线段 $C F$ 的交点 $G$点运动轨迹为线段 $A G$, 运动刚开始时, $A 、 E 、 F 、 G$ 四点重合, 当 $E$ 点与 $B$ 点重合时, $G$ 点运动到极限位置, 所以 $G$ 点轨迹为线段 $A G$.\n\n如图所示, 作 $G H \\perp A B$,\n\n\n$\\because$ 四边形 $A B C D$ 为菱形, $\\angle D A B=60^{\\circ}, A B=2$,\n\n$\\therefore C D / / B F, B D=2$,\n\n$\\therefore \\triangle C D G \\sim \\triangle F B G$,\n\n$\\therefore \\frac{C D}{B F}=\\frac{D G}{B G}$, 即 $B G=2 D G$,\n\n$\\because B G+D G=B D=2$,\n\n$\\therefore B G=\\frac{4}{3}$,\n\n在Rt $\\triangle G H B$ 中, $B G=\\frac{4}{3}, \\angle D B A=60^{\\circ}$,\n\n$\\therefore \\angle H G B=30^{\\circ}$,\n\n$\\therefore B H=\\frac{2}{3}, G H=\\frac{2 \\sqrt{3}}{3}$,\n\n在Rt $\\triangle A H G$ 中, $A H=2-\\frac{2}{3}=\\frac{4}{3}, G H=\\frac{2 \\sqrt{3}}{3}$,\n\n$A G^{2}=\\left(\\frac{4}{3}\\right)^{2}+\\left(\\frac{2 \\sqrt{3}}{3}\\right)^{2}=\\frac{28}{9}$,\n\n$\\therefore A G=\\frac{2 \\sqrt{7}}{3}$.\n\n$\\therefore G$ 点运动路径的长度为 $\\frac{2 \\sqrt{7}}{3}$.\n\n【解析】\n\n【分析】\n\n本题主要考查平行四边形的判定, 菱形的性质, 相似三角形的判定和性质, 含 $30^{\\circ}$ 角的直角三角形的性质以及勾股定理,根据题意表示出相关线段的长度是解题的关键..\n\n(1)利用平行四边形的判定定理: 一组对边平行且相等的四边形是平行四边形,\n\n(2)作 $C H \\perp B H$, 设 $A E=F A=m$, 利用三角形相似, 求出此时 $F G$ 的长, 进而表示出相关线段的长度, 再在 $R t \\triangle C F H$ 中, 利用勾股定理求解即可,\n\n(3)连接 $A G$, 作 $G H \\perp A B$, 分析出点 $G$ 的运动路径是线段 $A G$, 先在 $R t \\triangle G H B$ 中, 求出 $B H=\\frac{2}{3}, G H=$ $\\frac{2 \\sqrt{3}}{3}$, 再在 $R t \\triangle A H G$ 中, 利用勾股定理求出 $A G$ 的长度即可."} {"id": "3406", "image": ["4496.jpg", "4497.jpg", "4497.jpg"], "answer": "证明: (1)连接 $A O$,\n\n\n\n$\\because O A=O D$,\n\n$\\therefore \\angle O A D=\\angle O D A$,\n\n$\\because A C=F C$,\n\n$\\therefore \\angle C A F=\\angle C F A=\\angle O F D$,\n\n$\\because D$ 为 $B E$ 的下半圆弧的中点,\n\n$\\therefore O D \\perp B E$,\n\n$\\therefore \\angle O D A+\\angle O F D=90^{\\circ}$,\n\n$\\therefore \\angle C F A+\\angle D A O=90^{\\circ}$,\n\n$\\therefore \\angle O A C=90^{\\circ}$, 且 $O A$ 是半径,\n\n$\\therefore A C$ 是 $\\odot O$ 的切线;\n\n(2)在 $R t \\triangle O D F$ 中, $D F^{2}=O D^{2}+O F^{2}$,\n\n$\\therefore 10=O D^{2}+(4-O D)^{2}$,\n\n$\\therefore O D=1$ (不合题意舍去), $O D=3$,\n\n$\\therefore \\odot 0$ 的半径为 3 .\n\n【解析】(1)由等腰三角形的性质和垂径定理可求 $\\angle O A C=90^{\\circ}$, 可得结论;\n\n(2) 由勾股定理可求解.\n\n本题考查了切线的判定定理: 经过半径的外端且垂直于这条半径的直线是圆的切线. 要证某线是圆的切线, 已知此线过圆上某点, 连接圆心与这点 (即为半径), 再证垂直即可. 也考查了垂径定\n理.", "solution": "null", "level": "九年级", "question": "(本小题 8.0 分)\n\n如图, 以 $\\triangle A B C$ 的 $B C$ 边上一点 $O$ 为圆心的圆, 经过 $A 、 B$ 两点, 且与 $B C$ 边交于点 $E, D$ 为 $B E$ 的下半圆弧的中点, 连接 $A D$ 交 $B C$ 于 $F$, 若 $A C=F C$.\n\n(1)求证: $A C$ 是 $\\odot O$ 的切线;\n\n(2)若 $B F=4, D F=\\sqrt{10}$, 求 $\\odot O$ 的半径.\n\n", "options": [], "subject": "解析几何", "analysis": "证明: (1)连接 $A O$,\n\n\n\n$\\because O A=O D$,\n\n$\\therefore \\angle O A D=\\angle O D A$,\n\n$\\because A C=F C$,\n\n$\\therefore \\angle C A F=\\angle C F A=\\angle O F D$,\n\n$\\because D$ 为 $B E$ 的下半圆弧的中点,\n\n$\\therefore O D \\perp B E$,\n\n$\\therefore \\angle O D A+\\angle O F D=90^{\\circ}$,\n\n$\\therefore \\angle C F A+\\angle D A O=90^{\\circ}$,\n\n$\\therefore \\angle O A C=90^{\\circ}$, 且 $O A$ 是半径,\n\n$\\therefore A C$ 是 $\\odot O$ 的切线;\n\n(2)在 $R t \\triangle O D F$ 中, $D F^{2}=O D^{2}+O F^{2}$,\n\n$\\therefore 10=O D^{2}+(4-O D)^{2}$,\n\n$\\therefore O D=1$ (不合题意舍去), $O D=3$,\n\n$\\therefore \\odot 0$ 的半径为 3 .\n\n【解析】(1)由等腰三角形的性质和垂径定理可求 $\\angle O A C=90^{\\circ}$, 可得结论;\n\n(2) 由勾股定理可求解.\n\n本题考查了切线的判定定理: 经过半径的外端且垂直于这条半径的直线是圆的切线. 要证某线是圆的切线, 已知此线过圆上某点, 连接圆心与这点 (即为半径), 再证垂直即可. 也考查了垂径定\n理."} {"id": "3607", "image": [], "answer": "解:由题意得: $\\Delta=0$\n\n即: $(-2 \\sin \\alpha)^{2}-4\\left(\\sqrt{3} \\sin \\alpha-\\frac{3}{4}\\right)=0$,\n\n$4 \\sin ^{2} \\alpha-4 \\sqrt{3} \\sin \\alpha+3=0$,\n\n$(2 \\sin \\alpha-\\sqrt{3})^{2}=0$,\n\n$\\sin \\alpha=\\frac{\\sqrt{3}}{2}$,\n\n$\\alpha=60^{\\circ}$.", "solution": "null", "level": "九年级", "question": "(6 分) 已知锐角 $\\alpha$ 使关于 $x$ 的一元二次方程 $x^{2}-2 \\sin \\alpha \\cdot x+\\sqrt{3} \\sin \\alpha-\\frac{3}{4}=0$ 有两个相等的实数根, 求 $\\alpha$ 的度数.", "options": [], "subject": "代数", "analysis": "解:由题意得: $\\Delta=0$\n\n即: $(-2 \\sin \\alpha)^{2}-4\\left(\\sqrt{3} \\sin \\alpha-\\frac{3}{4}\\right)=0$,\n\n$4 \\sin ^{2} \\alpha-4 \\sqrt{3} \\sin \\alpha+3=0$,\n\n$(2 \\sin \\alpha-\\sqrt{3})^{2}=0$,\n\n$\\sin \\alpha=\\frac{\\sqrt{3}}{2}$,\n\n$\\alpha=60^{\\circ}$."} {"id": "3929", "image": [], "answer": "$-\\frac{1}{5}$ 【解析】原式 $=\\left(\\frac{1-x^{2}}{x+1}+\\frac{3}{x+1}\\right) \\div \\frac{(x+2)^{2}}{x+1}$\n\n$=\\frac{(2+x)(2-x)}{x+1} \\cdot \\frac{x+1}{(2+x)^{2}}$\n\n$=\\frac{2-x}{2+x}$,\n\n当 $x=\\tan 45^{\\circ}+\\left(\\frac{1}{2}\\right)^{-1}=1+2=3$ 时, 原式 $=\\frac{2-2}{2+3}=-\\frac{1}{5}$.", "solution": "null", "level": "九年级", "question": "先化简, 再求值: $\\left(1-\\mathrm{x}+\\frac{3}{x+1}\\right) \\div \\frac{x^{2}+4 x+4}{x+1}$, 其中 $\\mathrm{x}=\\tan 45^{\\circ}+\\left(\\frac{1}{2}\\right)^{-1}$.", "options": [], "subject": "代数", "analysis": "$-\\frac{1}{5}$ 【解析】原式 $=\\left(\\frac{1-x^{2}}{x+1}+\\frac{3}{x+1}\\right) \\div \\frac{(x+2)^{2}}{x+1}$\n\n$=\\frac{(2+x)(2-x)}{x+1} \\cdot \\frac{x+1}{(2+x)^{2}}$\n\n$=\\frac{2-x}{2+x}$,\n\n当 $x=\\tan 45^{\\circ}+\\left(\\frac{1}{2}\\right)^{-1}=1+2=3$ 时, 原式 $=\\frac{2-2}{2+3}=-\\frac{1}{5}$."} {"id": "2984", "image": [], "answer": "(1)、$m=0;(2)、mneq0$且$mneq1$", "solution": "null", "level": "九年级", "question": "(本题6分)(2019$cdot$浙江金华$cdot$九年级期中)已知函数${y}=left({m}^{2}-{m}right){x}^{2}+({m}-1)$${x}+{m}+1$.(1)若这个函数是一次函数,求$m$的值;(2)若这个函数是二次函数,则$m$的值应怎样?", "options": [], "subject": "代数", "analysis": "【分析】根据一次函数与二次函数的定义求解.【详解】解:(1)根据一次函数的定义,得:${m}^{2}-{m}=0$解得${m}=0$或${m}=1$又$becausem-1neq0$即$mneq1$;$therefore$当${m}=0$时,这个函数是一次函数;(2)根据二次函数的定义,得:${m}^{2}-{m}neq0$解得${m}_{1}neq0,{~m}_{2}neq1$$therefore$当${m}_{1}neq0,{~m}_{2}neq1$时,这个函数是二次函数.【点睛】考点:二次函数的定义;一次函数的定义"} {"id": "2988", "image": [], "answer": "(1)$y=-2x+80(20leqxleq28)$;(2)每本纪念册的销售单价是25元;(3)该纪念册销售单价定为28元时,才能使文具店销售该纪念册所获利润最大,最大利润是192元", "solution": "null", "level": "九年级", "question": "(本题8分)(2020$cdot$浙江$cdot$乐清市英华学校九年级阶段练习)某文具店购进一批纪念册,每本进价为20元,出于营销考虑,要求每本纪念册的售价不低于20元且不高于28元,在销售过程中发现该纪念册每周的销售量$y$(本)与每本纪念册的售价$x$(元)之间满足一次函数关系:当销售单价为22元时,销售量为36本;当销售单价为24元时,销售量为32本.(1)求出$y$与$x$的函数关系式;(2)当文具店每周销售这种纪念册获得150元的利润时,每本纪念册的销售单价是多少元?(3)设该文具店每周销售这种纪念册所获得的利润为$w$元,将该纪念册销售单价定为多少元时,才能使文具店销售该纪念册所获利润最大?最大利润是多少?", "options": [], "subject": "代数", "analysis": "【分析】(1)待定系数法列方程组求一次函数解析式.(2)根据(1)中解析式,列一元二次方程求解.(3)总利润$=$单件利润$times$销售量:$w=(x-20)(-2x+80)$,得到二次函数,先配方,在定义域上求最值.【详解】(1)设$y$与$x$的函数关系式为$y=kx+b$.把$(22,36)$与$(24,32)$代入,得$left{begin{array}{l}22k+b=3624k+b=32.end{array}right.$解得$left{begin{array}{l}k=-2b=80end{array}right.$,$thereforey=-2x+80(20leqxleq28)$.(2)设当文具店每周销售这种纪念册获得150元的利润时,每本纪念册的销售单价是$x$元,根据题意,得:$(x-20)y=150$,即$(x-20)(-2x+80)=150$.解得$x_{1}=25,x_{2}=35$(舍去).答:每本纪念册的销售单价是25元.(3)由题意,可得$w=(x-20)(-2x+80)=-2(x-30)^{2}+200$.$because$售价不低于20元且不高于28元,当$x<30$时,$y$随$x$的增大而增大,$therefore$当$x=28$时,$w_{text{最大}}=-2times(28-30)^{2}+200=192($元).答:该纪念册销售单价定为28元时,能使文具店销售该纪念册所获利润最大,最大利润是192元."} {"id": "3012", "image": [], "answer": "$(1)(-3x+300)$(2)$y=-3x^{2}+468x-16800$;(3)当销售单价定为每支78元时,所得月平均利润最大", "solution": "null", "level": "九年级", "question": "(本题8分)(2022$cdot$浙江宁波$cdot$九年级期末)某琴行销售一种笛子,每支进价为56元.当售价每支为80元时,月平均销售量为60支.为了倡导、弘扬艺术,琴行对该型号的笛子作降价销售(在不亏本的前提下).经市场调查表明,当每支笛子的售价每降低1元时,月平均销售量将增加3支.(1)若设销售单价为$x$元/支,则销售量为$qquad$支(用含$x$的代数式表示);(2)求月平均销售利润$y$(单位:元)关于销售单价$x$(单位:元/支)的函数表达式;(3)当销售单价定为每支多少元时,所得月平均利润最大?", "options": [], "subject": "代数", "analysis": "【分析】(1)根据售价每支为80元时,月平均销售量为60支,当每支笛子的售价每降低1元时,月平均销售量将增加3支,列出销售量代数式;(2)根据“总利润$=$单件利润$times$销售量”列出函数解析式即可;(3)利用二次函数的性质求函数最值即可.(1)根据题意,当销售单价为$x$元/支时,销量为:$60+3(80-x)=-3x+300$,$therefore$销售量为$(-3x+300)$支,故答案为:$(-3x+300)$;(2)设销售单价为$x$元/盋,月销售利润$y$元,根据题意得:$y=(x-56)(-3x+300)=-3x^{2}+468x-16800$,$therefore$月平均销售利润$y$关于销售单价$x$的函数表达式为$y=-3x^{2}+468x-16800$;(3)由(2)知:$y=-3x^{2}+468x-16800=-3(x-78)^{2}+1452$,$because-3<0,56leqxleq80$,$therefore$当$x=78$时,$y$有最大值,最大值为1452,$therefore$当销售单价定为每支78元时,所得月平均利润最大.【点睛】此题主要考查了二次函数的应用以及二次函数最值求法,解题关键是要读懂题目的意思,根据题目给出的条件,找出合适的等量关系."} {"id": "3146", "image": [], "answer": "(1)$frac{6}{5}$(2)$-frac{5}{2}$", "solution": "null", "level": "九年级", "question": "(本题8分)(2022$cdot$浙江$cdot$九年级专题练习)根据条件求值.(1)若$frac{a}{b}=frac{1}{5}$,求$frac{a+b}{b}$的值;(2)若$frac{x}{y}=frac{1}{3}$,求$frac{2x+y}{x-y}$的值.", "options": [], "subject": "代数", "analysis": "【分析】(1)把$frac{a+b}{b}$化为$frac{a}{b}+1$,再把$frac{a}{b}=frac{1}{5}$代入,即可;(2)根据$frac{x}{y}=frac{1}{3}$,得$y=3x$,代入$frac{2x+y}{x-y}$,即可.【详解】(1)$becausefrac{a+b}{b}=frac{a}{b}+1$$therefore$当$frac{a}{b}=frac{1}{5}$时,$frac{a+b}{b}=frac{a}{b}+1=frac{1}{5}+1=frac{6}{5}$$thereforefrac{a+b}{b}=frac{6}{5}$.(2)$becausefrac{x}{y}=frac{1}{3}$$thereforey=3x$$thereforefrac{2x+y}{x-y}=frac{2x+3x}{x-3x}=-frac{5x}{2x}=-frac{5}{2}$$thereforefrac{2x+y}{x-y}=-frac{5}{2}$.【点睛】本题考查比例的知识,解题的关键是对分式进行化简."} {"id": "3191", "image": [], "answer": "$-frac{18}{5}$", "solution": "null", "level": "九年级", "question": "(本题6分)(2021$cdot$浙江杭州$cdot$九年级阶段练习)已知$frac{a}{3}=frac{b}{2}=frac{c}{6}$,求$frac{2a+3b+c}{a-b-c}$的值.", "options": [], "subject": "代数", "analysis": "【分析】根据等比设参即可求解.【详解】$becausefrac{a}{3}=frac{b}{2}=frac{c}{6}$$therefore$设$frac{a}{3}=frac{b}{2}=frac{c}{6}=k$$thereforea=3k,b=2k,c=6k$$thereforefrac{2a+3b+c}{a-b-c}=frac{2times3k+3times2k+6k}{3k-2k-6k}=frac{18k}{-5k}=-frac{18}{5}$.【点睛】本题考查分式求值,根据比例的性质等比设参是解题的关键."} {"id": "3655", "image": ["5003.jpg", "5004.jpg", "5005.jpg"], "answer": "解: (1)作 $O C \\perp A D$ 于点 $C$, 连接 $O D$, 则 $\\triangle O C D$ 是直角三角形.\n\n可知 $O D=36 \\div 2=18 \\mathrm{~cm}$\n\n$O C=27-18=9 \\mathrm{~cm}$,\n\n$\\therefore C D=9 \\sqrt{3} \\mathrm{~cm}$,\n\n$\\therefore A D=2 C D=18 \\sqrt{3} \\mathrm{~cm}$\n\n(2)由(1)知 $\\angle C O D=60^{\\circ}$, 那么 $\\angle A O D=120^{\\circ}$,\n$\\therefore$ 上下底面的面积和为:\n\n$$\n2\\left[\\frac{(360-120) \\pi \\times 18^{2}}{360}+\\frac{1}{2} \\times 18 \\sqrt{3} \\times 9\\right]=512 \\pi+162 \\sqrt{3}\\left(\\mathrm{~cm}^{2}\\right)\n$$\n\n侧面积和为:\n\n$$\n18 \\sqrt{3} \\times 40+\\frac{240 \\pi \\times 18}{180} \\times 40=720 \\sqrt{3}+960 \\pi\\left(\\mathrm{cm}^{2}\\right)\n$$\n\n所以这个柱体的表面积和为 $1472 \\pi+882 \\sqrt{3}\\left(\\mathrm{~cm}^{2}\\right)$", "solution": "null", "level": "九年级", "question": "如图 1 所示为过圆柱体木块底面的一条弦 $A D$, 沿母线 $A B$ 剖开后得到的柱体,剖面是矩形 $A B C D$, $O$ 为原圆柱体木块底面的圆心. 如图 2 所示为该柱体的主视图和俯视图. 请你根据图中标注的数据解决以下问题.\n\n\n\n图 1\n\n\n\n主视图\n\n\n\n图 2\n\n(1)求弦 $A D$ 的长度.\n\n(2)求这个柱体的表面积. (结果可保留 $\\pi$ 和根号)", "options": [], "subject": "立体几何学", "analysis": "解: (1)作 $O C \\perp A D$ 于点 $C$, 连接 $O D$, 则 $\\triangle O C D$ 是直角三角形.\n\n可知 $O D=36 \\div 2=18 \\mathrm{~cm}$\n\n$O C=27-18=9 \\mathrm{~cm}$,\n\n$\\therefore C D=9 \\sqrt{3} \\mathrm{~cm}$,\n\n$\\therefore A D=2 C D=18 \\sqrt{3} \\mathrm{~cm}$\n\n(2)由(1)知 $\\angle C O D=60^{\\circ}$, 那么 $\\angle A O D=120^{\\circ}$,\n$\\therefore$ 上下底面的面积和为:\n\n$$\n2\\left[\\frac{(360-120) \\pi \\times 18^{2}}{360}+\\frac{1}{2} \\times 18 \\sqrt{3} \\times 9\\right]=512 \\pi+162 \\sqrt{3}\\left(\\mathrm{~cm}^{2}\\right)\n$$\n\n侧面积和为:\n\n$$\n18 \\sqrt{3} \\times 40+\\frac{240 \\pi \\times 18}{180} \\times 40=720 \\sqrt{3}+960 \\pi\\left(\\mathrm{cm}^{2}\\right)\n$$\n\n所以这个柱体的表面积和为 $1472 \\pi+882 \\sqrt{3}\\left(\\mathrm{~cm}^{2}\\right)$"} {"id": "3676", "image": ["5052.jpg", "5053.jpg"], "answer": "面 $F$ 面 $E$\n\n解: (1) “ “相间 $Z$ 端是对面”,可得 $B$ 的对面为 $F, C$ 的对面是 $E$,\n\n故答案为: 面 $F$, 面 $E$.\n\n(2)由题意得: $A$ 与 $D$ 相对, $B$ 与 $F$ 相对, $C$ 与 $E$ 相对,\n\n$$\nA+D=B+F=C+E\n$$\n\n将 $A=a^{3}+\\frac{1}{5} a^{2} b+3, B=-\\frac{1}{2} a^{2} b+a^{3}, C=a^{3}-1, D=-\\frac{1}{5}\\left(a^{2} b+15\\right)$ 代入得,\n\n$a^{3}+\\frac{1}{5} a^{2} b+3-\\frac{1}{5}\\left(a^{2} b+15\\right)=-\\frac{1}{2} a^{2} b+a^{3}+F=a^{3}-1+E$,\n\n$\\therefore F=\\frac{1}{2} a^{2} b$,\n\n$E=1$,", "solution": "null", "level": "九年级", "question": "如图所示是一个正方体的表面展开图, 请回答下列问题:\n\n(1)与面 $B$ 、面 $C$ 相对的面分别是 \\$ \\qquad \\$和 \\$ \\qquad \\$\n(2) 若 $A=a^{3}+\\frac{1}{5} a^{2} b+3, B=-\\frac{1}{2} a^{2} b+a^{3}, C=a^{3}-1, D=$\n\n\n$-\\frac{1}{5}\\left(a^{2} b+15\\right)$, 且相对两个面所表示的代数式的和都相等, 求 $E 、 F$\n\n(1)请在图中画出光源 $O$ 点的位置, 并画出 $O$ 到 $M N$ 的垂线段 $O H$ (不写画法);\n\n\n\n(2)若小明身高 $1.5 m$, 求 $O H$ 的长.", "options": [], "subject": "立体几何学", "analysis": "面 $F$ 面 $E$\n\n解: (1) “ “相间 $Z$ 端是对面”,可得 $B$ 的对面为 $F, C$ 的对面是 $E$,\n\n故答案为: 面 $F$, 面 $E$.\n\n(2)由题意得: $A$ 与 $D$ 相对, $B$ 与 $F$ 相对, $C$ 与 $E$ 相对,\n\n$$\nA+D=B+F=C+E\n$$\n\n将 $A=a^{3}+\\frac{1}{5} a^{2} b+3, B=-\\frac{1}{2} a^{2} b+a^{3}, C=a^{3}-1, D=-\\frac{1}{5}\\left(a^{2} b+15\\right)$ 代入得,\n\n$a^{3}+\\frac{1}{5} a^{2} b+3-\\frac{1}{5}\\left(a^{2} b+15\\right)=-\\frac{1}{2} a^{2} b+a^{3}+F=a^{3}-1+E$,\n\n$\\therefore F=\\frac{1}{2} a^{2} b$,\n\n$E=1$,"} {"id": "3838", "image": ["5335.jpg", "5336.jpg", "5337.jpg", "5337.jpg"], "answer": "(1) $a=3, b=1, c=1$;(2)最多由 11 个小立方体搭成; 最少由 9 个小立方体搭成;见解析.\n\n【解析】 (1) $a=3, b=1, c=1$;\n\n(2) $6+2+3=11$ (个), $4+2+3=9$ (个).\n\n这个几何体最多由 11 个小立方体搭成; 最少由 9 个小立方体搭成.\n\n(3) 如图所示.\n\n", "solution": "null", "level": "九年级", "question": "用相同的小立方体搭一个几何体, 从正面、上面看到的形状图如图所示, 从上面看到的形状图中小正方形的字母表示在该位置上小立方体的个数,请回答下列问题:\n\n\n\n从正面看\n\n\n\n从上面看\n\n(1) a,b,c 各表示的数字是几?\n\n(2)这个几何体最多由几个小立方体搭成?最少呢?\n\n(3)当 $d=e=1, f=2$ 时, 画出这个几何体从左面看得到的形状图.", "options": [], "subject": "立体几何学", "analysis": "(1) $a=3, b=1, c=1$;(2)最多由 11 个小立方体搭成; 最少由 9 个小立方体搭成;见解析.\n\n【解析】 (1) $a=3, b=1, c=1$;\n\n(2) $6+2+3=11$ (个), $4+2+3=9$ (个).\n\n这个几何体最多由 11 个小立方体搭成; 最少由 9 个小立方体搭成.\n\n(3) 如图所示.\n\n"} {"id": "3840", "image": ["5339.jpg", "5340.jpg", "5341.jpg", "5342.jpg", "5343.jpg", "5344.jpg", "5343.jpg", "5344.jpg"], "answer": "(1)8; (2)答案见解析:(3)200000 立方厘米\n\n【解析】(1) 小明共剪了 8 条棱, 故答案为 8 .\n\n(2)如图,四种情况.\n\n\n\n(1)\n\n(2)\n\n\n\n(3)\n\n(4)\n\n(3) $\\because$ 长方体纸盒的底面是一个正方形,\n\n$\\therefore$ 设最短的棱长高为 $\\mathrm{acm}$, 则长与宽相等为 $5 \\mathrm{acm}$,\n\n$\\because$ 长方体纸盒所有棱长的和是 $880 \\mathrm{~cm}$,\n\n$\\therefore 4(a+5 a+5 a)=880$, 解得 $a=20 \\mathrm{~cm}$,\n\n$\\therefore$ 这个长方体纸盒的体积为: $20 \\times 100 \\times 100=200000 \\mathrm{~cm}^{3}$.", "solution": "null", "level": "九年级", "question": "小明在学习了 《展开与折叠》这一课后, 明白了很多几何体都能展开成平面图形. 于是他在家用剪刀展开了一个长方体纸盒, 可是一不小心多剪了一条棱, 把纸盒剪成了两部分, 即图中的(1)和(2). 根据你所学的知识, 回答下列问题:\n\n\n\n(1)\n\n\n\n(2)\n\n\n\n宽\n\n高\n\n长\n\n(1) 小明总共剪开了条棱. \\$ \\qquad \\$\n\n\n盒, 你认为他应该将剪断的纸条粘贴到(1)中的什么位置? 请你帮助小明在(1)上补全.\n\n(3)小明说: 他所剪的所有棱中, 最长的一条棱是最短的一条棱的 5 倍. 现在已知这个长方体纸盒的底面是一个正方形, 并且这个长方体纸盒所有棱长的和是 $880 \\mathrm{~cm}$, 求这个长方体纸盒的体积.", "options": [], "subject": "立体几何学", "analysis": "(1)8; (2)答案见解析:(3)200000 立方厘米\n\n【解析】(1) 小明共剪了 8 条棱, 故答案为 8 .\n\n(2)如图,四种情况.\n\n\n\n(1)\n\n(2)\n\n\n\n(3)\n\n(4)\n\n(3) $\\because$ 长方体纸盒的底面是一个正方形,\n\n$\\therefore$ 设最短的棱长高为 $\\mathrm{acm}$, 则长与宽相等为 $5 \\mathrm{acm}$,\n\n$\\because$ 长方体纸盒所有棱长的和是 $880 \\mathrm{~cm}$,\n\n$\\therefore 4(a+5 a+5 a)=880$, 解得 $a=20 \\mathrm{~cm}$,\n\n$\\therefore$ 这个长方体纸盒的体积为: $20 \\times 100 \\times 100=200000 \\mathrm{~cm}^{3}$."} {"id": "3842", "image": ["5350.jpg", "5351.jpg", "5351.jpg"], "answer": "(1) $500 \\pi ; \\quad(2) 50 \\mathrm{~cm}$.\n\n【解析】 (1)由题意, 可得圆雉的母线 $\\mathrm{SA}=\\sqrt{A O^{2}+S O^{2}}=\\sqrt{(10 \\sqrt{15})^{2}+10^{2}}=40(\\mathrm{~cm})$,\n\n$\\therefore \\mathrm{S}_{\\text {侧 }}=\\pi \\times 10 \\times 40=400 \\pi\\left(\\mathrm{cm}^{2}\\right), \\mathrm{S}_{\\text {底 }}=\\pi \\mathrm{AO}^{2}=100 \\pi\\left(\\mathrm{cm}^{2}\\right)$,\n\n$\\therefore S_{\\text {全 }}=S_{\\text {则 }}+S_{\\text {底 }}=(400+100) \\pi=500 \\pi\\left(\\mathrm{cm}^{2}\\right)$.\n\n(2)沿母线 SA 将圆锥的侧面展开, 如图,\n\n\n\n则线段 $\\mathrm{AM}$ 的长就是蚂蚁所走的最短距离.\n\n由(1)知, $\\mathrm{SA}=40 \\mathrm{~cm}, A A^{\\prime}=20 \\pi \\mathrm{cm}$.\n\n$\\because \\frac{n \\pi \\times 40}{180}=20 \\pi(\\mathrm{cm})$,\n\n$\\therefore \\mathrm{n}=\\frac{180 \\times 20 \\pi}{40 \\pi}=90$,\n\n$\\therefore \\angle \\mathrm{S}=90^{\\circ}$.\n\n$\\because \\mathrm{SA}^{\\prime}=\\mathrm{SA}=40 \\mathrm{~cm}, \\mathrm{SM}=3 \\mathrm{~A}^{\\prime} \\mathrm{M}$,\n\n$\\therefore \\mathrm{SM}=30 \\mathrm{~cm}$,\n\n$\\therefore$ 在 Rt $\\triangle \\mathrm{ASM}$ 中, 由勾股定理得 $\\mathrm{AM}=50 \\mathrm{~cm}$,\n\n$\\therefore$ 蚂蚁所走的最短距离是 $50 \\mathrm{~cm}$.", "solution": "null", "level": "九年级", "question": "如图, 圆锥的底面半径为 $10 \\mathrm{~cm}$, 高为 $10 \\sqrt{15} \\mathrm{~cm}$.\n\n(1)求圆锥的全面积;\n\n(2)若一只蚂蚁从底面上一点 $A$ 出发绕圆锥侧面一周回到 $S A$ 上的点 $M$ 处, 且 $S M=3 A M$, 求它所走的最短距离.\n\n", "options": [], "subject": "立体几何学", "analysis": "(1) $500 \\pi ; \\quad(2) 50 \\mathrm{~cm}$.\n\n【解析】 (1)由题意, 可得圆雉的母线 $\\mathrm{SA}=\\sqrt{A O^{2}+S O^{2}}=\\sqrt{(10 \\sqrt{15})^{2}+10^{2}}=40(\\mathrm{~cm})$,\n\n$\\therefore \\mathrm{S}_{\\text {侧 }}=\\pi \\times 10 \\times 40=400 \\pi\\left(\\mathrm{cm}^{2}\\right), \\mathrm{S}_{\\text {底 }}=\\pi \\mathrm{AO}^{2}=100 \\pi\\left(\\mathrm{cm}^{2}\\right)$,\n\n$\\therefore S_{\\text {全 }}=S_{\\text {则 }}+S_{\\text {底 }}=(400+100) \\pi=500 \\pi\\left(\\mathrm{cm}^{2}\\right)$.\n\n(2)沿母线 SA 将圆锥的侧面展开, 如图,\n\n\n\n则线段 $\\mathrm{AM}$ 的长就是蚂蚁所走的最短距离.\n\n由(1)知, $\\mathrm{SA}=40 \\mathrm{~cm}, A A^{\\prime}=20 \\pi \\mathrm{cm}$.\n\n$\\because \\frac{n \\pi \\times 40}{180}=20 \\pi(\\mathrm{cm})$,\n\n$\\therefore \\mathrm{n}=\\frac{180 \\times 20 \\pi}{40 \\pi}=90$,\n\n$\\therefore \\angle \\mathrm{S}=90^{\\circ}$.\n\n$\\because \\mathrm{SA}^{\\prime}=\\mathrm{SA}=40 \\mathrm{~cm}, \\mathrm{SM}=3 \\mathrm{~A}^{\\prime} \\mathrm{M}$,\n\n$\\therefore \\mathrm{SM}=30 \\mathrm{~cm}$,\n\n$\\therefore$ 在 Rt $\\triangle \\mathrm{ASM}$ 中, 由勾股定理得 $\\mathrm{AM}=50 \\mathrm{~cm}$,\n\n$\\therefore$ 蚂蚁所走的最短距离是 $50 \\mathrm{~cm}$."} {"id": "3843", "image": ["5352.jpg", "5353.jpg", "5354.jpg", "5355.jpg", "5355.jpg"], "answer": "(1)三棱柱; (2) 见解析; (3) $120 \\mathrm{~cm}^{2}$\n\n【解析】(1) 三棱柱; (2) 如图所示:\n\n\n\n(3) $3 \\times 10 \\times 4=120 \\mathrm{~cm}^{2}$.", "solution": "null", "level": "九年级", "question": "已知图为一几何体从不同方向看的图形:\n\n(1) 写出这个几何体的名称;\n\n(2)任意画出这个几何体的一种表面展开图;\n\n(3) 若长方形的高为 10 厘米, 三角形的边长为 4 厘米, 求这个几何体的侧面积.\n\n\n\n从正面看\n\n\n\n从左面看\n\n\n\n从上面看", "options": [], "subject": "立体几何学", "analysis": "(1)三棱柱; (2) 见解析; (3) $120 \\mathrm{~cm}^{2}$\n\n【解析】(1) 三棱柱; (2) 如图所示:\n\n\n\n(3) $3 \\times 10 \\times 4=120 \\mathrm{~cm}^{2}$."} {"id": "3845", "image": ["5361.jpg"], "answer": "(1) 矩形; (2) $\\sqrt{2} a^{2}$\n\n【解析】(1)正方体在平面 $H$ 上的正投影图形是矩形;\n\n(2) $\\because$ 正方体边长为 $a c m, \\quad \\therefore B D=\\sqrt{a^{2}+a^{2}}=\\sqrt{2} a(\\mathrm{~cm}) , \\therefore$ 投影 $M N P Q$ 的面积为 $\\sqrt{2} a \\times a=\\sqrt{2} a^{2}$ $\\left(\\mathrm{cm}^{2}\\right)$.", "solution": "null", "level": "九年级", "question": "如图, 边长为 $\\mathrm{acm}$ 的正方体其上下底面的对角线 $\\mathrm{AC} 、 \\mathrm{~A}_{1} \\mathrm{C}_{1}$ 与平面 $\\mathrm{H}$ 垂直.\n\n\n\n(1)指出正方体在平面 $\\mathrm{H}$ 上的正投影图形;\n\n(2)计算投影 MNP 的面积.", "options": [], "subject": "立体几何学", "analysis": "(1) 矩形; (2) $\\sqrt{2} a^{2}$\n\n【解析】(1)正方体在平面 $H$ 上的正投影图形是矩形;\n\n(2) $\\because$ 正方体边长为 $a c m, \\quad \\therefore B D=\\sqrt{a^{2}+a^{2}}=\\sqrt{2} a(\\mathrm{~cm}) , \\therefore$ 投影 $M N P Q$ 的面积为 $\\sqrt{2} a \\times a=\\sqrt{2} a^{2}$ $\\left(\\mathrm{cm}^{2}\\right)$."} {"id": "3863", "image": ["5410.jpg", "5411.jpg", "5412.jpg", "5413.jpg", "5414.jpg", "5413.jpg", "5414.jpg"], "answer": "这个几何体的表面积是 $376 \\mathrm{~mm}^{2}$\n【解析】 根据三视图可得:上面的长方体长 $6 \\mathrm{~mm}$, 高 $6 \\mathrm{~mm}$, 宽 $3 \\mathrm{~mm}$,\n\n下面的长方体长 $8 \\mathrm{~mm} ,$ 宽 $10 \\mathrm{~mm} ,$ 高 $3 \\mathrm{~mm} ,$\n\n$2(3 \\times 8+3 \\times 10+8 \\times 10)+2 \\times(3 \\times 6+6 \\times 6)=268+108=376 \\mathrm{~mm}^{2}$.\n\n答:这个几何体的表面积是 $376 \\mathrm{~mm}^{2}$.\n\n20.(1)如图(一),左视图没有发生改变;(2)如图 1 所示见解析,(3)如图 2 所示见解析.\n\n【解析】(1)如图(一), 它是由 6 个同样大小的正方体摆成的几何体.将正方体(1)移走后,新几何体的三视图与原几何体的三视图相比,左视图没有发生改变;\n(2) 如图 1 所示,\n(3) 如图 2 所示.\n\n\n\n图1\n\n\n\n图2", "solution": "null", "level": "九年级", "question": "如图是两个长方体组合而成的一个立体图形的三视图, 根据图中所标尺寸单位 (毫米), 求这个几何体的表面积.\n\n\n\n主视图\n\n\n\n左视图\n\n\n\n俯视图", "options": [], "subject": "立体几何学", "analysis": "这个几何体的表面积是 $376 \\mathrm{~mm}^{2}$\n【解析】 根据三视图可得:上面的长方体长 $6 \\mathrm{~mm}$, 高 $6 \\mathrm{~mm}$, 宽 $3 \\mathrm{~mm}$,\n\n下面的长方体长 $8 \\mathrm{~mm} ,$ 宽 $10 \\mathrm{~mm} ,$ 高 $3 \\mathrm{~mm} ,$\n\n$2(3 \\times 8+3 \\times 10+8 \\times 10)+2 \\times(3 \\times 6+6 \\times 6)=268+108=376 \\mathrm{~mm}^{2}$.\n\n答:这个几何体的表面积是 $376 \\mathrm{~mm}^{2}$.\n\n20.(1)如图(一),左视图没有发生改变;(2)如图 1 所示见解析,(3)如图 2 所示见解析.\n\n【解析】(1)如图(一), 它是由 6 个同样大小的正方体摆成的几何体.将正方体(1)移走后,新几何体的三视图与原几何体的三视图相比,左视图没有发生改变;\n(2) 如图 1 所示,\n(3) 如图 2 所示.\n\n\n\n图1\n\n\n\n图2"} {"id": "3883", "image": ["5462.jpg", "5463.jpg", "5463.jpg"], "answer": "证明见解析.\n\n【解析】连接 $\\mathrm{OD}$, 可得 $\\mathrm{OB}=\\mathrm{OD}$,\n\n$\\because \\mathrm{AB}=\\mathrm{AD}, \\quad \\therefore \\mathrm{AE}$ 垂直平分 $\\mathrm{BD}$,\n\n在 Rt $\\triangle \\mathrm{BOE}$ 中, $\\mathrm{OB}=3, \\cos \\angle \\mathrm{BOE}=\\frac{3}{5}, \\therefore \\mathrm{OE}=\\frac{9}{5}$,\n\n根据勾股定理得: $\\mathrm{BE}=\\sqrt{B O^{2}-O E^{2}}=\\frac{12}{5}, \\mathrm{CE}=\\mathrm{OC}-\\mathrm{OE}=\\frac{16}{5}$,\n\n在 Rt $\\triangle \\mathrm{CEB}$ 中, $\\mathrm{BC}=\\sqrt{C E^{2}+B E^{2}}=4$,\n\n$\\because \\mathrm{OB}=3, \\mathrm{BC}=4, \\quad \\mathrm{OC}=5, \\quad \\therefore O B^{2}+B C^{2}=O C^{2}$,\n\n$\\therefore \\angle \\mathrm{OBC}=90^{\\circ}$, 即 $\\mathrm{BC} \\perp \\mathrm{OB}$, 则 $\\mathrm{BC}$ 为圆 $\\mathrm{O}$ 的切线.\n\n", "solution": "null", "level": "九年级", "question": "如图, 在四边形 $\\mathrm{ABCD}$ 中, $\\mathrm{AB}=\\mathrm{AD}$, 对角线 $\\mathrm{AC}, \\mathrm{BD}$ 交于点 $\\mathrm{E}$, 点 $\\mathrm{O}$ 在线段 $\\mathrm{AE}$ 上, $\\odot O$ 过 $\\mathrm{B}$, $\\mathrm{D}$ 两点, 若 $\\mathrm{OC}=5, \\mathrm{OB}=3$, 且 $\\cos \\angle \\mathrm{BOE}=\\frac{3}{5}$. 求证: $\\mathrm{CB}$ 是 $\\odot \\mathrm{O}$ 的切线.\n\n", "options": [], "subject": "组合几何学", "analysis": "证明见解析.\n\n【解析】连接 $\\mathrm{OD}$, 可得 $\\mathrm{OB}=\\mathrm{OD}$,\n\n$\\because \\mathrm{AB}=\\mathrm{AD}, \\quad \\therefore \\mathrm{AE}$ 垂直平分 $\\mathrm{BD}$,\n\n在 Rt $\\triangle \\mathrm{BOE}$ 中, $\\mathrm{OB}=3, \\cos \\angle \\mathrm{BOE}=\\frac{3}{5}, \\therefore \\mathrm{OE}=\\frac{9}{5}$,\n\n根据勾股定理得: $\\mathrm{BE}=\\sqrt{B O^{2}-O E^{2}}=\\frac{12}{5}, \\mathrm{CE}=\\mathrm{OC}-\\mathrm{OE}=\\frac{16}{5}$,\n\n在 Rt $\\triangle \\mathrm{CEB}$ 中, $\\mathrm{BC}=\\sqrt{C E^{2}+B E^{2}}=4$,\n\n$\\because \\mathrm{OB}=3, \\mathrm{BC}=4, \\quad \\mathrm{OC}=5, \\quad \\therefore O B^{2}+B C^{2}=O C^{2}$,\n\n$\\therefore \\angle \\mathrm{OBC}=90^{\\circ}$, 即 $\\mathrm{BC} \\perp \\mathrm{OB}$, 则 $\\mathrm{BC}$ 为圆 $\\mathrm{O}$ 的切线.\n\n"} {"id": "3888", "image": ["5472.jpg", "5473.jpg", "5473.jpg"], "answer": "证明见解析.\n\n【解析】\n\n连结 $\\mathrm{OC}$, 如图,\n\n\n\n$\\because \\quad$ 直线 $\\mathrm{CE}$ 与 $\\odot \\mathrm{O}$ 相切于点 $C, \\therefore \\quad \\mathrm{OC} \\perp \\mathrm{CE}$,\n\n$\\because \\quad \\mathrm{AD} \\perp \\mathrm{CE}, \\therefore \\quad \\mathrm{OC} / / \\mathrm{AD}, \\therefore \\quad \\angle 1=\\angle 3$,\n\n$\\because \\quad \\mathrm{OA}=\\mathrm{OC}, \\therefore \\quad \\angle 2=\\angle 3$,\n\n$\\therefore \\quad \\angle 1=\\angle 2, \\therefore \\quad \\mathrm{AC}$ 平分 $\\angle \\mathrm{DAB}$.", "solution": "null", "level": "九年级", "question": "如图, 已知 $\\mathrm{AB}$ 是 $\\odot \\mathrm{O}$ 的直径, 直线 $\\mathrm{CE}$ 与 $\\odot \\mathrm{O}$ 相切于点 $C$, 交 $\\mathrm{AB}$ 的延长线交于点 $\\mathrm{E}$. 作 $\\mathrm{AD} \\perp \\mathrm{CE}$, 垂足为 $\\mathrm{D}$, 连接 $\\mathrm{AC}$. 求证: $\\mathrm{AC}$ 平分 $\\angle \\mathrm{DAB}$.\n\n", "options": [], "subject": "组合几何学", "analysis": "证明见解析.\n\n【解析】\n\n连结 $\\mathrm{OC}$, 如图,\n\n\n\n$\\because \\quad$ 直线 $\\mathrm{CE}$ 与 $\\odot \\mathrm{O}$ 相切于点 $C, \\therefore \\quad \\mathrm{OC} \\perp \\mathrm{CE}$,\n\n$\\because \\quad \\mathrm{AD} \\perp \\mathrm{CE}, \\therefore \\quad \\mathrm{OC} / / \\mathrm{AD}, \\therefore \\quad \\angle 1=\\angle 3$,\n\n$\\because \\quad \\mathrm{OA}=\\mathrm{OC}, \\therefore \\quad \\angle 2=\\angle 3$,\n\n$\\therefore \\quad \\angle 1=\\angle 2, \\therefore \\quad \\mathrm{AC}$ 平分 $\\angle \\mathrm{DAB}$."} {"id": "3889", "image": ["5474.jpg", "5475.jpg", "5475.jpg"], "answer": "(1) $A C=4$; (2)详见解析.\n\n【解析】 (1) $\\because \\mathrm{AB}$ 是 $\\odot \\mathrm{O}$ 直径, $\\mathrm{C}$ 在 $\\odot \\mathrm{O}$ 上,\n\n$\\therefore \\angle \\mathrm{ACB}=90^{\\circ}$, 又 $\\because \\mathrm{BC}=3, \\mathrm{AB}=5, \\therefore$ 由勾股定理得 $\\mathrm{AC}=4$;\n\n(2) 证明: 连接 $\\mathrm{OC} \\because \\mathrm{AC}$ 是 $\\angle \\mathrm{DAB}$ 的角平分线,\n\n$\\therefore \\angle \\mathrm{DAC}=\\angle \\mathrm{BAC}$, 又 $\\because \\mathrm{AD} \\perp \\mathrm{DC}$,\n\n$\\therefore \\angle \\mathrm{ADC}=\\angle \\mathrm{ACB}=90^{\\circ}, \\therefore \\triangle \\mathrm{ADC} \\sim \\triangle \\mathrm{ACB}$,\n\n$\\therefore \\angle \\mathrm{DCA}=\\angle \\mathrm{CBA}$, 又 $\\because \\mathrm{OA}=\\mathrm{OC}$,\n\n$\\therefore \\angle \\mathrm{OAC}=\\angle \\mathrm{OCA}, \\because \\angle \\mathrm{OAC}+\\angle \\mathrm{OBC}=90^{\\circ}$,\n$\\therefore \\angle \\mathrm{OCA}+\\angle \\mathrm{ACD}=\\angle \\mathrm{OCD}=90^{\\circ}, \\quad \\therefore \\mathrm{DC}$ 是 $\\odot \\mathrm{O}$ 的切线.\n\n", "solution": "null", "level": "九年级", "question": "如图, $\\odot \\mathrm{O}$ 的直径为 $\\mathrm{AB}$, 点 $\\mathrm{C}$ 在圆周上(异于点 $\\mathrm{A}, \\mathrm{B}$ ), $\\mathrm{AD} \\perp \\mathrm{CD}$.\n\n(1)若 $\\mathrm{BC}=3, \\mathrm{AB}=5$, 求 $\\mathrm{AC}$ 的长;\n\n(2)若 $\\mathrm{AC}$ 是 $\\angle \\mathrm{DAB}$ 的平分线, 求证: 直线 $\\mathrm{CD}$ 是 $\\odot \\mathrm{O}$ 的切线.\n\n", "options": [], "subject": "组合几何学", "analysis": "(1) $A C=4$; (2)详见解析.\n\n【解析】 (1) $\\because \\mathrm{AB}$ 是 $\\odot \\mathrm{O}$ 直径, $\\mathrm{C}$ 在 $\\odot \\mathrm{O}$ 上,\n\n$\\therefore \\angle \\mathrm{ACB}=90^{\\circ}$, 又 $\\because \\mathrm{BC}=3, \\mathrm{AB}=5, \\therefore$ 由勾股定理得 $\\mathrm{AC}=4$;\n\n(2) 证明: 连接 $\\mathrm{OC} \\because \\mathrm{AC}$ 是 $\\angle \\mathrm{DAB}$ 的角平分线,\n\n$\\therefore \\angle \\mathrm{DAC}=\\angle \\mathrm{BAC}$, 又 $\\because \\mathrm{AD} \\perp \\mathrm{DC}$,\n\n$\\therefore \\angle \\mathrm{ADC}=\\angle \\mathrm{ACB}=90^{\\circ}, \\therefore \\triangle \\mathrm{ADC} \\sim \\triangle \\mathrm{ACB}$,\n\n$\\therefore \\angle \\mathrm{DCA}=\\angle \\mathrm{CBA}$, 又 $\\because \\mathrm{OA}=\\mathrm{OC}$,\n\n$\\therefore \\angle \\mathrm{OAC}=\\angle \\mathrm{OCA}, \\because \\angle \\mathrm{OAC}+\\angle \\mathrm{OBC}=90^{\\circ}$,\n$\\therefore \\angle \\mathrm{OCA}+\\angle \\mathrm{ACD}=\\angle \\mathrm{OCD}=90^{\\circ}, \\quad \\therefore \\mathrm{DC}$ 是 $\\odot \\mathrm{O}$ 的切线.\n\n"} {"id": "3904", "image": ["5499.jpg", "5500.jpg", "5500.jpg"], "answer": "(1)答案见解析; (2) 2 .\n\n【解析】(1) 连接 $O C$.\n\n$\\because A B$ 是 $\\odot O$ 的直径, $C$ 是 $\\odot O$ 上一点,\n\n$\\therefore \\angle A C B=90^{\\circ}$, 即 $\\angle A C O+\\angle O C B=90^{\\circ}$.\n\n$\\because O A=O C, \\angle B C D=\\angle A, \\quad \\therefore \\angle A C O=\\angle A=\\angle B C D$,\n\n$\\therefore \\angle B C D+\\angle O C B=90^{\\circ}$, 即 $\\angle O C D=90^{\\circ}, \\therefore C D$ 是 $\\odot O$ 的切线.\n\n(2) 在 Rt $\\triangle O C D$ 中, $\\angle O C D=90^{\\circ}, O C=3, C D=4$,\n\n$\\therefore O D=\\sqrt{O C^{2}+C D^{2}}=5$,\n\n$\\therefore B D=O D-O B=5-3=2$.\n\n", "solution": "null", "level": "九年级", "question": "如图, $A B$ 是 $\\odot O$ 的直径, $C$ 是 $\\odot O$ 上一点, $D$ 在 $A B$ 的延长线上, 且 $\\angle B C D=\\angle A$.\n\n(1) 求证: $C D$ 是 $\\odot O$ 的切线;\n\n(2) $\\odot O$ 的半径为 $3, C D=4$, 求 $B D$ 的长.\n\n", "options": [], "subject": "组合几何学", "analysis": "(1)答案见解析; (2) 2 .\n\n【解析】(1) 连接 $O C$.\n\n$\\because A B$ 是 $\\odot O$ 的直径, $C$ 是 $\\odot O$ 上一点,\n\n$\\therefore \\angle A C B=90^{\\circ}$, 即 $\\angle A C O+\\angle O C B=90^{\\circ}$.\n\n$\\because O A=O C, \\angle B C D=\\angle A, \\quad \\therefore \\angle A C O=\\angle A=\\angle B C D$,\n\n$\\therefore \\angle B C D+\\angle O C B=90^{\\circ}$, 即 $\\angle O C D=90^{\\circ}, \\therefore C D$ 是 $\\odot O$ 的切线.\n\n(2) 在 Rt $\\triangle O C D$ 中, $\\angle O C D=90^{\\circ}, O C=3, C D=4$,\n\n$\\therefore O D=\\sqrt{O C^{2}+C D^{2}}=5$,\n\n$\\therefore B D=O D-O B=5-3=2$.\n\n"} {"id": "3077", "image": ["3984.jpg", "3985.jpg"], "answer": "(1)见解析(2)见解析", "solution": "null", "level": "九年级", "question": "(本题7分)(2021$cdot$浙江$cdot$金华海亮外国语学校九年级阶段练习)如图,在四边形$ABCD$中,$AD=BC,angleB=angleD,AD$不平行于$BC$,过点$C$作$CE//AD$交$triangleABC$的外接圆$O$于点$E$,连接$AE$.(1)求证:四边形$AECD$为平行四边形;(2)连接$CO$,求证:$CO$平分$angleBCE$.", "options": [], "subject": "组合几何学", "analysis": "【分析】(1)根据圆周角定理得到$angleB=angleE$,得到$angleE=angleD$,根据平行线的判定和性质定理得到$AE/CD$,证明结论;(2)作$OMperpBC$于$M,ONperpCE$于$N$,根据垂径定理、角平分线的判定定理证明.(1)证明:$becauseangleB=angleE,angleB=angleD$,$thereforeangleE=angleD$,$becauseCE//AD$$thereforeangleD+angleECD=180^{circ}$,$thereforeangleE+angleECD=180^{circ}$,$thereforeAE//CD$$therefore$四边形$AECD$为平行四边形;(2)证明:作$OMperpBC$于$M,ONperpCE$于$N$,如图,$because$四边形$AECD$为平行四边形,$thereforeAD=CE$又$becauseAD=BC$,$thereforeCE=CB$$becauseOMperpBC,ONperpCE$,$thereforeangleONC=angleOMC=90^{circ},CN=frac{1}{2}CE,CM=frac{1}{2}BC$,$thereforeCN=CM$$becauseOC=OC$,$thereforeRttriangleCONcongRttriangleCOM$$thereforeON=OM$$becauseOMperpBC,ONperpCE$$thereforeCO$平分$angleBCE$.【点睛】本题考查的是三角形的外接圆与外心,掌握平行四边形的判定定理、垂径定理、圆周角定理是解题的关键."} {"id": "3080", "image": ["3988.jpg"], "answer": "(1)见解析(2)弦$BD$的长为$16{~cm}$", "solution": "null", "level": "九年级", "question": "(本题9分)(2021:浙江$cdot$杭州仁和实验学校九年级期中)如图,$AC$为$odotO$的直径,$BD$是弦,且$ACperpBD$于点$E$.连接$AB、OB、BC$.(1)求证:$angleCBO=angleABD$;(2)若$AE=4{~cm},CE=16{~cm}$,求弦$BD$的长.", "options": [], "subject": "组合几何学", "analysis": "【分析】(1)根据垂径定理可得$AB=AD$,进而可得$angleABD=angleC$,根据半径相等可得$angleC=angleCBO$,等量代换即可得证;(2)在Rt$triangleOBE$中,勾股定理求得$BE$,根据垂径定理可得$BE=DE$,即可求解.(1)$becauseAC$为$odotO$的直径,且$ACperpBD$,$thereforeAB=AD$$thereforeangleABD=angleC$,$becauseOB=OC$$thereforeangleC=angleCBO$$thereforeangleCBO=angleABD$(2)$becauseAE=4,quadCE=16$,$thereforeOA=10,OE=6$,在Rt$triangleOBE$中,$BE=sqrt{10^{2}-6^{2}}=8$,$becauseAC$为$odotO$的直径,且$ACperpBD$,$thereforeBE=DE$,$thereforeBD=2BE=16{~cm}$.【点睛】本题考查了垂径定理,勾股定理,圆周角定理等,掌握垂径定理是解题的关键."} {"id": "3145", "image": ["4100.jpg"], "answer": "(1)见解析;(2)$EF=2$", "solution": "null", "level": "九年级", "question": "(本题6分)(2020$cdot$浙江$cdot$诸暨市滨江初级中学九年级期中)如图,已知$AB//DC$,点$E、F$在线段$BD$上,$AB=2DC,BE=2DF$.(1)求证:$triangleABEsimtriangleCDF$.(2)若$BD=8,DF=2$,求$EF$的长.", "options": [], "subject": "组合几何学", "analysis": "【分析】(1)根据$AB//DC$,可得$angleB=angleD$,再由$AB=2DC,BE=2DF$,可得$AB$:$DC=BE:DF=2$,即可证得;(2)根据$BE=2DF$,可得$BE=4$,即可求解.【详解】(1)证明:$becauseAB//DC$,$thereforeangleB=angleD$,$becauseAB=2DC,BE=2DF$$thereforeAB:DC=BE:DF=2$,$thereforetriangleABEsimtriangleCDF$(2)解:$becauseBE=2DF,DF=2$,$thereforeBE=4$$becauseBD=8$,$thereforeEF=BD-DF-BE=2$.【点睛】本题主要考查了相似三角形的判定,熟练掌握两边对应成比例且夹角相等的两个三角形相似是解题的关键."} {"id": "3171", "image": ["4142.jpg"], "answer": "(1)见解析(2)4", "solution": "null", "level": "九年级", "question": "(本题6分)(2022浙江湖州$cdot$九年级期末)如图,在四边形$ABCD$中,$AD//BC$,$angleB=angleACD=90^{circ}$,(1)求证:$triangleABCsimtriangleDCA$.(2)若$BC=1,AC=2$,求$AD$的长.", "options": [], "subject": "组合几何学", "analysis": "【分析】(1)先根据平行线的性质由$AD//BC$得$angleACB=angleDAC$,已知条件中还有$angleB=angleACD=90^{circ}$,根据“有两个角分别相等的两个三角形相似”可以证明$triangleABC$相似于$triangleDCA;$(2)由(1)可知$triangleABCsimtriangleDCA$,根据相似三角形的对应边成比例列出等式$frac{BC}{AC}=frac{AC}{AD}$,其中$BC=1,AC=2$,可以求出$AD$的长.(1)证明:$becauseAD//BC$$thereforeangleACB=angleDAC$又$becauseangleB=angleACD=90^{circ}$$thereforetriangleABCsimtriangleDCA$(2)$becausetriangleABCsimtriangleDCA$$thereforefrac{BC}{AC}=frac{AC}{AD}$$becauseBC=1,AC=2$即$frac{1}{2}=frac{2}{AD}$,$thereforeAD=4$$AD$的长为:4.【点睛】此题重点考查平行线的性质、相似三角形的判定与性质等知识,根据平行线的性质得出相等的角是解题的关键."} {"id": "3174", "image": ["4147.jpg"], "answer": "(1)证明见解析(2)$20^{circ}$", "solution": "null", "level": "九年级", "question": "(本题8分)(2022-浙江金华$cdot$九年级期末)如图,在四边形$ABCD$中,$AC,BD$交于点$F$.点$E$在$BD$上,且$angleBAE=angleCAD,frac{AB}{AE}=frac{AC}{AD}$.(1)求证:$triangleABCsimtriangleAED$.(2)若$angleBAE=20^{circ}$,求$angleCBD$的度数.", "options": [], "subject": "组合几何学", "analysis": "【分析】(1)根据两边对应成比例,且夹角相等,两个三角形相似,即可证明,(2)根据(1)中$triangleABCsimtriangleAED$,得出$angleADB=angleACB$,再根据对顶角相等,$angleAFD=angleBFC$,证得$triangleAFDsimtriangleBFC$,得出$angleCBD=angleCAD=angleBAE$,即可求解.$becauseangleBAE=angleCAD$$thereforeangleBAE+angleEAF=angleCAD+angleEAF$,$thereforeangleBAC=angleDAE$$frac{AB}{AE}=frac{AC}{AD}$,$because$在$triangleABC$和$triangleAED$中,$left{begin{array}{c}frac{AB}{AE}=frac{AC}{AD}angleBAC=angleDAEend{array}right.$$thereforetriangleABCsimtriangleAED$.(2)$becausetriangleABCsimtriangleAED$$thereforeangleADB=angleACB$又$becauseangleAFD=angleBFC,$对顶角相等,$thereforetriangleAFDsimtriangleBFC$,$thereforeangleCBD=angleCAD$$becauseangleBAE=angleCAD,angleBAE=20^{circ}$,$thereforeangleCAD=20^{circ}$,故答案为:$20^{circ}$.【点睛】本题考查了相似三角形的判定和性质,熟练掌捯相似三角形的判定和性质是解题的关键."} {"id": "3357", "image": ["4406.jpg"], "answer": "解:由折叠性质得 $E F=A E=10$,\n\n在 $R t \\triangle E F B$ 中, $B F=6$,\n\n$$\n\\therefore E B=\\sqrt{E F^{2}-B F^{2}}=8\n$$\n\n$\\therefore C D=A B=A E+B E=10+8=18$,\n\n$\\because \\angle E F D=\\angle A=90^{\\circ}$,\n\n$\\therefore \\angle E F B+\\angle D F C=90^{\\circ}$,\n\n$\\because \\angle B E F+\\angle E F B=90^{\\circ}$,\n\n$\\therefore \\angle D F C=\\angle B E F$,\n\n$\\because \\angle B=\\angle C$,\n\n$\\therefore \\triangle B E F \\sim \\triangle C F D$,\n\n$\\therefore \\frac{C F}{B E}=\\frac{C D}{B F}$, 即 $\\frac{C F}{8}=\\frac{18}{6}$\n\n$\\therefore C F=24$,\n\n$\\therefore A D=B C=B F+C F=30$,\n\n$\\therefore \\tan \\angle A D E=\\frac{A E}{A D}=\\frac{10}{30}=\\frac{1}{3}$.\n\n【解析】见答案", "solution": "null", "level": "九年级", "question": "(本小题8.0分)\n\n如图, 在矩形 $A B C D$ 中, 点 $E$ 在边 $A B$ 上, 将矩形 $A B C D$ 沿直线 $D E$ 折叠, 点 $A$ 恰好落在边 $B C$ 的点 $F$ 处. 若 $A E=10, B F=6$, 求 $\\tan \\angle A D E$ 的值.\n\n", "options": [], "subject": "组合几何学", "analysis": "解:由折叠性质得 $E F=A E=10$,\n\n在 $R t \\triangle E F B$ 中, $B F=6$,\n\n$$\n\\therefore E B=\\sqrt{E F^{2}-B F^{2}}=8\n$$\n\n$\\therefore C D=A B=A E+B E=10+8=18$,\n\n$\\because \\angle E F D=\\angle A=90^{\\circ}$,\n\n$\\therefore \\angle E F B+\\angle D F C=90^{\\circ}$,\n\n$\\because \\angle B E F+\\angle E F B=90^{\\circ}$,\n\n$\\therefore \\angle D F C=\\angle B E F$,\n\n$\\because \\angle B=\\angle C$,\n\n$\\therefore \\triangle B E F \\sim \\triangle C F D$,\n\n$\\therefore \\frac{C F}{B E}=\\frac{C D}{B F}$, 即 $\\frac{C F}{8}=\\frac{18}{6}$\n\n$\\therefore C F=24$,\n\n$\\therefore A D=B C=B F+C F=30$,\n\n$\\therefore \\tan \\angle A D E=\\frac{A E}{A D}=\\frac{10}{30}=\\frac{1}{3}$.\n\n【解析】见答案"} {"id": "3361", "image": ["4411.jpg"], "answer": "解: $\\because$ 四边形 $A B C D$ 是矩形,\n\n$\\therefore A B=C D, \\angle D=90^{\\circ}$,\n\n$\\because$ 将矩形 $A B C D$ 沿 $C E$ 折叠, 点 $B$ 恰好落在边 $A D$ 的 $F$ 处,\n\n$\\therefore C F=B C$,\n\n$\\because \\frac{A B}{B C}=\\frac{2}{3}$,\n\n$\\therefore \\frac{C D}{C F}=\\frac{2}{3}$\n\n设 $C D=2 x, C F=3 x$,\n\n$\\therefore D F=\\sqrt{C F^{2}-C D^{2}}=\\sqrt{5} x$,\n\n$\\therefore \\tan \\angle D C F=\\frac{D F}{C D}=\\frac{\\sqrt{5} x}{2 x}=\\frac{\\sqrt{5}}{2}$,\n\n故答案为: $\\frac{\\sqrt{5}}{2}$.\n\n【解析】此题考查了矩形的性质、折叠的性质以及勾股定理. 此题比较简单, 注意折叠中的对应关系, 注意数形结合思想的应用. 由矩形 $A B C D$ 沿 $C E$ 折叠, 点 $B$ 恰好落在边 $A D$ 的 $F$ 处, 即可得 $B C=$ $C F, C D=A B$, 由 $\\frac{A B}{B C}=\\frac{2}{3}$, 可得 $\\frac{C D}{C F}=\\frac{2}{3}$, 然后设 $C D=2 x, C F=3 x$, 利用勾股定理即可求得 $D F$ 的\n值, 继而求得 $\\tan \\angle D C F$ 的值.", "solution": "null", "level": "九年级", "question": "(本小题8.0分)\n\n如图, 将矩形 $A B C D$ 沿 $C E$ 折叠, 使点 $B$ 恰好落在边 $A D$ 上的点 $F$ 处, 若 $\\frac{B C}{B C}=\\frac{2}{3}$, 求 $\\tan \\angle D C F$ 的值.\n\n", "options": [], "subject": "组合几何学", "analysis": "解: $\\because$ 四边形 $A B C D$ 是矩形,\n\n$\\therefore A B=C D, \\angle D=90^{\\circ}$,\n\n$\\because$ 将矩形 $A B C D$ 沿 $C E$ 折叠, 点 $B$ 恰好落在边 $A D$ 的 $F$ 处,\n\n$\\therefore C F=B C$,\n\n$\\because \\frac{A B}{B C}=\\frac{2}{3}$,\n\n$\\therefore \\frac{C D}{C F}=\\frac{2}{3}$\n\n设 $C D=2 x, C F=3 x$,\n\n$\\therefore D F=\\sqrt{C F^{2}-C D^{2}}=\\sqrt{5} x$,\n\n$\\therefore \\tan \\angle D C F=\\frac{D F}{C D}=\\frac{\\sqrt{5} x}{2 x}=\\frac{\\sqrt{5}}{2}$,\n\n故答案为: $\\frac{\\sqrt{5}}{2}$.\n\n【解析】此题考查了矩形的性质、折叠的性质以及勾股定理. 此题比较简单, 注意折叠中的对应关系, 注意数形结合思想的应用. 由矩形 $A B C D$ 沿 $C E$ 折叠, 点 $B$ 恰好落在边 $A D$ 的 $F$ 处, 即可得 $B C=$ $C F, C D=A B$, 由 $\\frac{A B}{B C}=\\frac{2}{3}$, 可得 $\\frac{C D}{C F}=\\frac{2}{3}$, 然后设 $C D=2 x, C F=3 x$, 利用勾股定理即可求得 $D F$ 的\n值, 继而求得 $\\tan \\angle D C F$ 的值."} {"id": "3390", "image": ["4475.jpg", "4476.jpg", "4476.jpg"], "answer": "解: : 四边形 $A B C D$ 内接于 $\\odot O, \\angle B A D=90^{\\circ}$,\n\n$\\therefore \\angle C=180^{\\circ}-\\angle B A D=90^{\\circ}, \\angle A B C+\\angle A D C=180^{\\circ}$.\n\n作 $A E \\perp B C$ 于 $E, D F \\perp A E$ 于 $F$, 则四边形 $C D F E$ 是矩形, $E F=C D=10$.\n\n\n\n在Rt $\\triangle A E B$ 中,\n\n$\\because \\angle A E B=90^{\\circ}, A B=17, \\cos \\angle A B C=\\frac{3}{5}$,\n\n$\\therefore B E=A B \\cdot \\cos \\angle A B E=\\frac{51}{5}$.\n\n$\\therefore A E=\\sqrt{A B^{2}-B E^{2}}=\\frac{68}{5}$.\n\n$\\therefore A F=A E-E F=\\frac{68}{5}-10=\\frac{18}{5}$.\n\n$\\because \\angle A B C+\\angle A D C=180^{\\circ}, \\angle C D F=90^{\\circ}$,\n\n$\\therefore \\angle A B C+\\angle A D F=90^{\\circ}$.\n\n$\\because \\cos \\angle A B C=\\frac{3}{5}$,\n\n$\\therefore \\sin \\angle A D F=\\cos \\angle A B C=\\frac{3}{5}$.\n\n在Rt $\\triangle A D F$ 中,\n\n$\\because \\angle A F D=90^{\\circ}, \\sin \\angle A D F=\\frac{3}{5}$,\n\n$\\therefore A D=\\frac{A F}{\\sin \\angle A D F}=\\frac{\\frac{18}{5}}{\\frac{3}{5}}=6$.\n\n【解析】本题考查了圆内接四边形的性质, 矩形的判定与性质, 勾股定理, 解直角三角形, 求出 $A F=\\frac{18}{5}$ 以及 $\\sin \\angle A D F=\\frac{3}{5}$ 是解题的关键. 根据圆内接四边形的对角互补得出 $\\angle C=90^{\\circ}, \\angle A B C+$ $\\angle A D C=180^{\\circ}$. 作 $A E \\perp B C$ 于 $E, D F \\perp A E$ 于 $F$, 则 $C D F E$ 是矩形, $E F=C D=10$. 解 $R t \\triangle A E B$, 得出 $B E=A B \\cdot \\cos \\angle A B E=\\frac{51}{5}, A E=\\sqrt{A B^{2}-B E^{2}}=\\frac{68}{5}$, 那么 $A F=A E-E F=\\frac{18}{5}$. 再证明 $\\angle A B C+$ $\\angle A D F=90^{\\circ}$, 根据互余角的互余函数相等得出 $\\sin \\angle A D F=\\cos \\angle A B C=$\n$\\frac{3}{5}$ 解 $R t \\triangle A D F$, 即可求出 $A D=\\frac{A F}{\\sin \\angle A D F}=6$.", "solution": "null", "level": "九年级", "question": "(本小题8.0分)\n\n如图, 四边形 $A B C D$ 内接于 $\\odot O, A B=17, C D=10, \\angle A=90^{\\circ}, \\cos B=\\frac{3}{5}$, 求 $A D$ 的长.\n\n", "options": [], "subject": "组合几何学", "analysis": "解: : 四边形 $A B C D$ 内接于 $\\odot O, \\angle B A D=90^{\\circ}$,\n\n$\\therefore \\angle C=180^{\\circ}-\\angle B A D=90^{\\circ}, \\angle A B C+\\angle A D C=180^{\\circ}$.\n\n作 $A E \\perp B C$ 于 $E, D F \\perp A E$ 于 $F$, 则四边形 $C D F E$ 是矩形, $E F=C D=10$.\n\n\n\n在Rt $\\triangle A E B$ 中,\n\n$\\because \\angle A E B=90^{\\circ}, A B=17, \\cos \\angle A B C=\\frac{3}{5}$,\n\n$\\therefore B E=A B \\cdot \\cos \\angle A B E=\\frac{51}{5}$.\n\n$\\therefore A E=\\sqrt{A B^{2}-B E^{2}}=\\frac{68}{5}$.\n\n$\\therefore A F=A E-E F=\\frac{68}{5}-10=\\frac{18}{5}$.\n\n$\\because \\angle A B C+\\angle A D C=180^{\\circ}, \\angle C D F=90^{\\circ}$,\n\n$\\therefore \\angle A B C+\\angle A D F=90^{\\circ}$.\n\n$\\because \\cos \\angle A B C=\\frac{3}{5}$,\n\n$\\therefore \\sin \\angle A D F=\\cos \\angle A B C=\\frac{3}{5}$.\n\n在Rt $\\triangle A D F$ 中,\n\n$\\because \\angle A F D=90^{\\circ}, \\sin \\angle A D F=\\frac{3}{5}$,\n\n$\\therefore A D=\\frac{A F}{\\sin \\angle A D F}=\\frac{\\frac{18}{5}}{\\frac{3}{5}}=6$.\n\n【解析】本题考查了圆内接四边形的性质, 矩形的判定与性质, 勾股定理, 解直角三角形, 求出 $A F=\\frac{18}{5}$ 以及 $\\sin \\angle A D F=\\frac{3}{5}$ 是解题的关键. 根据圆内接四边形的对角互补得出 $\\angle C=90^{\\circ}, \\angle A B C+$ $\\angle A D C=180^{\\circ}$. 作 $A E \\perp B C$ 于 $E, D F \\perp A E$ 于 $F$, 则 $C D F E$ 是矩形, $E F=C D=10$. 解 $R t \\triangle A E B$, 得出 $B E=A B \\cdot \\cos \\angle A B E=\\frac{51}{5}, A E=\\sqrt{A B^{2}-B E^{2}}=\\frac{68}{5}$, 那么 $A F=A E-E F=\\frac{18}{5}$. 再证明 $\\angle A B C+$ $\\angle A D F=90^{\\circ}$, 根据互余角的互余函数相等得出 $\\sin \\angle A D F=\\cos \\angle A B C=$\n$\\frac{3}{5}$ 解 $R t \\triangle A D F$, 即可求出 $A D=\\frac{A F}{\\sin \\angle A D F}=6$."} {"id": "3440", "image": ["4573.jpg"], "answer": "(1)解: $\\because E$ 是 $\\triangle A B C$ 的内心,\n\n$\\therefore A E$ 平分 $\\angle B A C, B E$ 平分 $\\angle A B C, C E$ 平分 $\\angle A C B$.\n\n$\\because \\angle C B D=34^{\\circ}$,\n\n$\\therefore \\angle C A D=\\angle C B D=34^{\\circ}$,\n\n$\\therefore \\angle B A C=2 \\angle C B D=68^{\\circ}$,\n\n$\\therefore \\angle A B C+\\angle A C B=180^{\\circ}-68^{\\circ}=112^{\\circ}$,\n\n$\\therefore \\angle A B E+\\angle A C E=\\frac{1}{2} \\times 112^{\\circ}=56^{\\circ}$,\n\n$\\therefore \\angle B E C=\\angle B A C+\\angle A B E+\\angle A C E=68^{\\circ}+56^{\\circ}=124^{\\circ} ;$\n\n(2)证明: $\\because$ 点 $E$ 是 $\\triangle A B C$ 的内心,\n\n$\\therefore \\angle B A D=\\angle C A D, \\angle A B E=\\angle C B E$,\n\n由圆周角定理得, $\\angle D A C=\\angle D B C$,\n\n$\\angle D B E=\\angle D B C+\\angle E B C, \\angle D E B=\\angle E B A+\\angle E A B$,\n$\\therefore \\angle D B E=\\angle D E B$,\n\n$\\therefore D E=D B$.\n\n【解析】(1)由点 $E$ 是 $\\triangle A B C$ 的内心可得 $A E$ 平分 $\\angle B A C, B E$ 平分 $\\angle A B D, C E$ 平分 $\\angle A C B$, 根据同弧所对的圆周角相等可得 $\\angle C A D=\\angle C B D$, 进而求出 $\\angle B A C$ 的度数; 根据三角形内角和 $180^{\\circ}$ 得到\n\n$\\angle A B C+\\angle A C B=180^{\\circ}-\\angle B A C$, 利用三角形外角的性质得到 $\\angle B E C=\\angle B A C+\\angle A B E+\\angle A C E$, 进而求出 $\\angle B E C$ 的度数;\n\n(2)根据内心的性质, 三角形内角和定理证明.\n\n本题考查的是三角形的内切圆与内心, 三角形的外接圆与外心, 掌握圆周角定理, 三角形的内心的概念, 三角形的外角的性质是解题的关键.", "solution": "null", "level": "九年级", "question": "(本小题8.0分)\n\n如图, 点 $E$ 是 $\\triangle A B C$ 的内心, $A E$ 的延长线和 $\\triangle A B C$ 的外接圆相交于点 $D$, 连接 $B E$.\n\n(1)若 $\\angle C B D=34^{\\circ}$, 求 $\\angle B E C$ 的度数;\n\n(2)求证: $D E=D B$.\n\n", "options": [], "subject": "组合几何学", "analysis": "(1)解: $\\because E$ 是 $\\triangle A B C$ 的内心,\n\n$\\therefore A E$ 平分 $\\angle B A C, B E$ 平分 $\\angle A B C, C E$ 平分 $\\angle A C B$.\n\n$\\because \\angle C B D=34^{\\circ}$,\n\n$\\therefore \\angle C A D=\\angle C B D=34^{\\circ}$,\n\n$\\therefore \\angle B A C=2 \\angle C B D=68^{\\circ}$,\n\n$\\therefore \\angle A B C+\\angle A C B=180^{\\circ}-68^{\\circ}=112^{\\circ}$,\n\n$\\therefore \\angle A B E+\\angle A C E=\\frac{1}{2} \\times 112^{\\circ}=56^{\\circ}$,\n\n$\\therefore \\angle B E C=\\angle B A C+\\angle A B E+\\angle A C E=68^{\\circ}+56^{\\circ}=124^{\\circ} ;$\n\n(2)证明: $\\because$ 点 $E$ 是 $\\triangle A B C$ 的内心,\n\n$\\therefore \\angle B A D=\\angle C A D, \\angle A B E=\\angle C B E$,\n\n由圆周角定理得, $\\angle D A C=\\angle D B C$,\n\n$\\angle D B E=\\angle D B C+\\angle E B C, \\angle D E B=\\angle E B A+\\angle E A B$,\n$\\therefore \\angle D B E=\\angle D E B$,\n\n$\\therefore D E=D B$.\n\n【解析】(1)由点 $E$ 是 $\\triangle A B C$ 的内心可得 $A E$ 平分 $\\angle B A C, B E$ 平分 $\\angle A B D, C E$ 平分 $\\angle A C B$, 根据同弧所对的圆周角相等可得 $\\angle C A D=\\angle C B D$, 进而求出 $\\angle B A C$ 的度数; 根据三角形内角和 $180^{\\circ}$ 得到\n\n$\\angle A B C+\\angle A C B=180^{\\circ}-\\angle B A C$, 利用三角形外角的性质得到 $\\angle B E C=\\angle B A C+\\angle A B E+\\angle A C E$, 进而求出 $\\angle B E C$ 的度数;\n\n(2)根据内心的性质, 三角形内角和定理证明.\n\n本题考查的是三角形的内切圆与内心, 三角形的外接圆与外心, 掌握圆周角定理, 三角形的内心的概念, 三角形的外角的性质是解题的关键."} {"id": "3031", "image": ["3925.jpg"], "answer": "(1)$frac{1}{3}$(2)$frac{2}{9}$", "solution": "null", "level": "九年级", "question": "(本题6分)(2022-浙江杭州$cdot$九年级期末)一个布袋里装有三个小球,上面分别写着“\"”,“2”,“3”,除数字外三个小球无其他差别.(1)从布袋里任意摸出一个小球,求上面的数字恰好是“3”的概率.(2)从布袋里任意摸出一个小球,记录其数字,放回并摇匀,再从中任意摸出一个小球,记录其数字,求两次记录的数字之和为3的概率.(要求列表或画树状图说明)", "options": [], "subject": "组合数学", "analysis": "【分析】(1)写有“3”的球的个数除以总的球的个数即可得解;(2)利用树状图列举法即可求解;(1)根据题意,上面的数字恰好是“3”的概率为:$frac{text{写有}3text{的球的个数}}{text{球的总个数}}=frac{1}{3}$,即所求概率为$frac{1}{3};$$(2)$利用树状图列举法:如图两次之和为“3”的次数共计有2次,总计有9种抽球的方式,则两次之和为“3”的概率为:$frac{2}{9}$.【点睛】本题考查了公式法和列举法求概率的知识,掌握理解列举法的基本原理是解答本题的关键."} {"id": "3034", "image": ["3928.jpg", "3929.jpg"], "answer": "$frac{1}{3}$", "solution": "null", "level": "九年级", "question": "(本题9分)(2022-浙江杭州$cdot$九年级期末)在一次宣传杭州亚运会的有奖竞猜活动中,获奖者从放有只有颜色不同的3个小球(1个黑球,1个白球,1个黄球)的不透明布袋中摸球.若模到一个黑球奖励一个亚运会吉祥物“宸宸”,摸到一个白球奖励一个“琮琮”,摸到一个黄球奖励一个“莲莲”.一个获奖者先从布袋中任意摸出一球,不放回,再摸出一球,求得到一个“蓬莲”和一个“琮琮”的概率.抗州2022年第19届亚运会吉样物", "options": [], "subject": "组合数学", "analysis": "【分析】根据题意作图树状图,可知共有6种可能情况,而满足条件的有2种情况,进而求概率即可。【详解】解:根据题意,可作树状图如下,由树状图可知,共有6种可能情况,满足条件的有2种情况,所以,得到一个“莲莲”和一个“琮琮”的概率为$P=frac{2}{6}=frac{1}{3}$.【点睛】本题主要考查了列举法求概率的知识,能够正确作出树状图是解题关键."} {"id": "246", "image": [], "answer": " (1) $\\frac{3}{20}$\n\n(2) 33\n\n", "solution": "null", "level": "九年级", "question": "(6 分)一个不透明的盒子里装有 20 张红色卡片, 15 张黄色卡片, 12 张蓝色卡片和若干张黑色卡片, 每张卡片除颜色外都相同, 从中任意摸出一张卡片, 摸到红色卡片的概率是 0.25 .\n\n(1)从中任意摸出一张卡片, 摸到蓝色卡片的概率是多少?\n\n(2)求盒子里黑色卡片的个数.\n\n", "options": [], "subject": "组合数学", "analysis": "(1) 根据摸到红色卡片的概率是 0.25 求出卡片的总数, 进而根据概率计算公式进行求解即可;\n\n(2)用卡片总数减去其他颜色卡片的张数即可求出黑色卡片的张数.\n\n【详解】(1) 解: 由题意, 得卡片的总张数为 $\\frac{20}{0.25}=80$ (张),\n\n$\\therefore$ 任意摸出一张卡片, 摸到蓝色卡片的概率是 $\\frac{12}{80}=\\frac{3}{20}$;\n\n(2)解:盒子里黑色卡片的个数为 $80-15-20-12=33$.\n\n答: 盒子里黑色卡片的个数为 33 张.\n\n【点睛】此题主要考查了概率公式, 熟练掌握概率的意义是解题的关键.\n\n"} {"id": "3033", "image": ["3926.jpg", "3927.jpg"], "answer": "(1)$frac{1}{9}$(2)4号", "solution": "null", "level": "九年级", "question": "(本题8分)(2022$cdot$浙江杭州$cdot$九年级期末)将6个球分别放入标有$1,2,3,4$,5,6这6个号码的盒子中.如图,将一个圆形转盘平均分成3份,分别标上数字1,2,3,现转动转盘两次,两次转得的数字之和是几,从几号盘子中摸出一个球(如:第一次转得数字为2,第二次转得数字为3,则和为5,就从5号盒子中摸球).(1)求从6号盒子中摸球的概率;(2)通过计算,判断从几号盒子中摸球的概率最大?", "options": [], "subject": "计数", "analysis": "【分析】(1)先利用树状图展示所有9种等可能的结果,再找出符合条件的情况数,最后利用概率公式计算;(2)分别计算出从各个盒子中摸球的概率,再比较即可.(1)解:如图,共有9种等可能的结果,其中两次摸出的小球所标注数字之和为6的结果数为1,$therefore$从6号盒子中摸球的概率为$frac{1}{9}$;(2)由树状图可知:从1号盒子中摸球的概率为:$frac{0}{9}=0$,从2号盒子中摸球的概率为:$frac{1}{9}$,从3号盒子中摸球的概率为:$frac{2}{9}$,从4号盒子中摸球的概率为:$frac{3}{9}=frac{1}{3}$,从5号盒子中摸球的概率为:$frac{2}{9}$,从6号盒子中摸球的概率为:$frac{1}{9}$,$therefore$从4号盒子中摸球的概率最大.【点睛】本题考查了列表法与树状图法:利用列表法或树状图法展示所有可能的结果求出$n$,再从中选出符合事件$A$或$B$的结果数目$m$,然后根据概率公式计算事件$A$或事件$B$的概率."} {"id": "3035", "image": [], "answer": "(1)列表见解析;共有9种等可能的结果数;(2)点$M(x,y)$在函数$y=-$$x+1$的图象上的概率$=frac{2}{9}$", "solution": "null", "level": "九年级", "question": "(本题10分)(2019-浙江$cdot$九年级期中)在甲、乙两个不透明的布袋,甲袋中装有3个完全相同的小球,分别标有数字$0,1,2$;乙袋中装有3个完全相同的小球,分别标有数字$-1,-2,0$;现从甲袋中随机抽取一个小球,记录标有的数字为$x$,再从乙袋中随机抽取一个小球,记录标有的数字为$y$,确定点$M$坐标为$(x,y)$.(1)用树状图或列表法列举点$M$所有可能的坐标;(2)求点$M(x,y)$在函数$y=-x+1$的图象上的概率.", "options": [], "subject": "计数", "analysis": "【分析】(1)通过列表展示所有9种等可能的结果数;(2)找出满足点$({x},{y})$落在函数${y}=-$${x}+1$的图象上的结果数,然后根据概率公式求解.【详解】解:(1)列表如下:|$xy$|0|1|2||:---|:---|:---|:---||-1|$(0,-$
1|$(1,-$
1|$(2,-$||-2|$(0,-$|$(1,-$|$(2,-$||-1$)$|$2)$|$2)$|||0|$(0,0)$|$(1,0)$|$(2,0)$|共有9种等可能的结果数;(2)满足点$(x,y)$落在函数$y=-x+1$的图象上的结果有2个,即$(2,-1)$,$(1,0)$,所以点$M(x,y)$在函数$y=-x+1$的图象上的概率$=frac{2}{9}$.【点睛】本题考查了列表法与树状图法:利用列表法或树状图法展示所有等可能的结果$n$,再从中选出符合事件${A}$或${B}$的结果数目${m}$,然后利用概率公式计算事件${A}$或事件${B}$的概率."} {"id": "244", "image": ["10292.jpg"], "answer": " (1)随机, 不可能\n\n(2) $P_{\\text {(拱到黄冰) }}=\\frac{1}{2}$\n\n", "solution": "null", "level": "九年级", "question": "(6 分) 不透明的袋中装有 2 个红球、 3 个黄球和 1 个白球,它们除颜色外都相同. (1)从中随机取出一个球是红球是 $\\qquad$事件, 随机取出一个球是黑球是 $\\qquad$事件;\n\n(2)求从袋中任意摸出一个球是黄球的概率.\n\n\n", "options": [], "subject": "计数", "analysis": "(1) 根据事件的分类解答即可;\n\n(2)根据概率公式计算即可.\n\n【详解】(1) $\\because$ 一个不透明的袋中装有 2 个红球、 3 个黄球和 1 个白球, 它们除颜色外都相同,\n\n$\\therefore$ 从中随机取出一个球是红球是随机事件, 随机取出一个球是黑球是不可能事件.\n\n故答案为: 随机, 不可能;\n\n(2) $\\because$ 袋中装有 2 个红球、 3 个黄球和 1 个白球,\n\n$\\therefore$ 从袋中任意摸出一个球可能的结果有 6 种,\n\n$\\therefore P_{\\text {(摸到黄球) }}=\\frac{3}{6}=\\frac{1}{2}$.\n\n【点睛】本题考查了事件的分类, 概率的计算, 熟练掌握各知识点是解答本题的关键.\n\n"} {"id": "247", "image": [], "answer": " (1) $a=188, \\quad b=0.949$\n\n(2) 0.95\n\n(3) 4275 个\n\n", "solution": "null", "level": "九年级", "question": "(8 分) 某批乒乓球的质量检验结果如下:\n\n| 抽取的乒乓球数 $n$ | 50 | 100 | 200 | 500 | 1000 | 1500 | 2000 |\n| :---: | :---: | :---: | :---: | :---: | :---: | :---: | :---: |\n| 优等品的频数 $m$ | 48 | 95 | $a$ | 471 | 946 | 1426 | 1898 |\n| 优等品的频率 $m / n$ | 0.960 | 0.950 | 0.940 | 0.942 | 0.946 | 0.951 | $b$ |\n\n(1)请求出 $a, \\quad b$ 的值, $a=$ $\\qquad$ $b=$ $\\qquad$\n(2)从这批乒兵球中, 任意抽取一只乒兵球是优等品的概率的估计值是 (精确到 0.01 ); (3)若这批乒兵球共有 4500 个, 请估计其中是优等品的个数.\n\n", "options": [], "subject": "计数", "analysis": "(1) 根据优等品的频率 $\\frac{m}{n}$ 计算即可;\n\n(2)大量重复实验时, 事件发生的频率在某个固定位置左右摆动, 并且摆动的幅度越来越小, 根据这个频率稳定性定理, 可以用频率的集中趋势来估计概率, 这个固定的近似值就是这个事件的概率. 根据表中优等品的频率判定即可;\n\n(3)用 4500 乘以优等品的概率即可得解.\n\n【详解】(1) 解: $a=200 \\times 0.940=188, \\quad b=\\frac{1898}{2000}=0.949$;\n\n(2)从这批乒兵球中,任意抽取一只乒乓球是优等品的概率的估计值是 0.95 ;\n\n(3) $4500 \\times 0.95=4275$ (个),\n\n$\\therefore$ 优等品的个数是 4275 个.\n\n【点睛】本题主要考查了利用频率估计概率, 理解利用频率估计概率的相关知识, 并准确计算是解题的关键.\n\n"} {"id": "248", "image": ["10293.jpg"], "answer": " (1)不公平, 理由见解析; (2)只要把转盘 B 上的两个偶数中的一个改为奇数即可\n\n", "solution": "null", "level": "九年级", "question": "(8 分) 如图有两个质地均匀的转盘 A、B, 转盘 $\\mathrm{A}$ 被分成 3 份, 分别标有数字 1 , 2, 3; 转盘 $\\mathrm{B}$ 被 3 等分, 分别标有数字 $4,5,6$. 小强与小华用这两个转盘玩游戏,小强说“随机转动 $A 、 B$ 转盘各一次, 转盘停止后, 将 $\\mathrm{A} 、 \\mathrm{~B}$ 转盘的指针所指的数字相乘, 积为偶数我赢; 积为奇数你赢.\" (指针指向两个扇形的交线时, 重新转动转盘).\n\n(1)小强指定的游戏规则对双方公平吗?并说明理由;\n\n(2)小华认为只要在转盘 B 上修改其中一个数字,也可以使这个游戏对双方公平. 你能帮助小华如何进行修改吗?\n\n", "options": [], "subject": "计数", "analysis": "游戏是否公平, 关键要看游戏双方取胜的机会是否相等, 即判断双方取胜的概率是否相等, 或转化为在总情况明确的情况下, 判断双方取胜所包含的情况数目是否相等.\n【详解】解:(1)不公平; 因为按照游戏规则, 根据树状图分析可得:\n\n$1 \\times 4 \\quad 1 \\times 5$\n$1 \\times 62 \\times 4$\n$2 \\times 5$\n$2 \\times 63 \\times 4$\n$3 \\times 5 \\quad 3 \\times 6$\n\n一共有 9 种等可能结果, 其中偶数情况有 7 种, 奇数情况有 2 种\n\n$\\mathrm{P}$ (小强胜 $)=\\frac{7}{9}, \\mathrm{P}$ (小华胜 $)=\\frac{2}{9}$;\n\n(2) 只要把转盘 $\\mathrm{B}$ 上的两个偶数中的一个改为奇数即可.\n\n【点睛】本题考查的是游戏公平性的判断, 判断游戏公平性就要计算每个事件的概率, 概率相等就公平, 否则就不公平, 用到的知识点为: 概率=所求情况数与总情况数之比.\n\n"} {"id": "249", "image": ["10294.jpg"], "answer": " (1) $\\frac{1}{4}$\n\n(2) $\\frac{1}{4}$\n\n", "solution": "null", "level": "九年级", "question": "(10 分) 如图是某城市地铁线路图的一部分,已知甲从 $A$ 站上车,随机从 $B 、 C 、 D$ 、 $E$ 中的某一站下车.\n\n\n\n(1)甲从 $C$ 站下车的概率是 $\\qquad$\n(2)若乙与甲乘坐同一趟地铁从 $A$ 站上车,随机从 $B 、 C 、 D 、 E$ 中的某一站下车,\n\n(1)求甲、乙从 $B 、 C 、 D 、 E$ 中的某一站下车有多少种不同的可能. (用列表法或树状图表示)\n\n(2)求甲、乙两人恰好从同一站下车的概率.\n\n", "options": [], "subject": "计数", "analysis": "(1) 根据概率公式可进行求解;\n\n(2)通过列表法可进行求解概率.\n\n【详解】(1) 解:由题意可知甲从 $C$ 站下车的概率是 $\\frac{1}{4}$;\n\n故答案为 $\\frac{1}{4}$;\n\n(2)解: 由题意可列表如下:\n\n| | $B$ | $C$ | $D$ | $E$ |\n| :--- | :--- | :--- | :--- | :--- |\n| $B$ | $(B, B)$ | $(B, C)$ | $(B, D)$ | $(B, E)$ |\n| $C$ | $(C, B)$ | $(C, C)$ | $(C, D)$ | $(C, E)$ |\n| $D$ | $(D, B)$ | $(D, C)$ | $(D, D)$ | $(D, E)$ |\n| $E$ | $(E, B)$ | $(E, C)$ | $(E, D)$ | $(E, E)$ |\n\n由表可知甲、乙两人随机从 $B 、 C 、 D 、 E$ 某一站下车共有 16 种情况, 其中甲、乙两人在同\n一站下车共有 4 种情况, 所以其概率为 $P=\\frac{4}{16}=\\frac{1}{4}$.\n\n【点睛】本题主要考查概率, 熟练掌握利用列表法进行求解概率是解题的关键.\n\n"} {"id": "511", "image": [], "answer": " (1) A 的销售单价为 30 元、 $B$ 的销售单价为 24 元\n\n(2)当 $m=5$ 时, 商场销售 $A 、 B$ 两种商品可获得总利润最大,最大利润是 810 元.\n", "solution": "null", "level": "九年级", "question": "某商场销售 $A 、 B$ 两种商品, 每件进价均为 20 元. 调查发现, 如果售出 $\\mathrm{A}$ 种 20 件, $B$ 种 10 件, 销售总额为 840 元; 如果售出 $\\mathrm{A}$ 种 10 件, $B$ 种 15 件, 销售总额为 660 元.\n\n(1)求 $A, B$ 两种商品的销售单价.\n\n(2)经市场调研, A 种商品按原售价销售,可售出 40 件,原售价每降价 1 元,销售量可增加 10 件; $B$ 种商品的售价不变, $\\mathrm{A}$ 种商品售价不低于 $B$ 种商品售价. 设 $\\mathrm{A}$ 种商品降价 $m$ 元, 如果 $A, B$ 两种商品销售量相同, 求 $m$ 取何值时, 商场销售 $A, B$ 两种商品可获得总利润最大? 最大利润是多少?\n\n", "options": [], "subject": "计数", "analysis": "(1) 设 $\\mathrm{A}$ 的销售单价为 $x$ 元、 $B$ 的销售单价为 $y$ 元, 根据题中售出 $\\mathrm{A}$ 种 20 件, $B$ 种 10 件, 销售总额为 840 元; 售出 $\\mathrm{A}$ 种 10 件, B种 15 件, 销售总额为 660 元列方程组求解即可得到答案;\n\n(2) 设利润为 $w$, 根据题意, 得到 $w=-10(m-5)^{2}+810$, 结合二次函数性质及题中限制条件分析求解即可得到答案.\n\n【详解】(1) 解:设 $\\mathrm{A}$ 的销售单价为 $x$ 元、 $B$ 的销售单价为 $y$ 元, 则\n\n$$\n\\left\\{\\begin{array} { l } \n{ 2 0 x + 1 0 y = 8 4 0 } \\\\\n{ 1 0 x + 1 5 y = 6 6 0 }\n\\end{array} \\text { , 解得 } \\left\\{\\begin{array}{l}\nx=30 \\\\\ny=24\n\\end{array}\\right.\\right. \\text {, }\n$$\n\n答: $A$ 的销售单价为 30 元、 $B$ 的销售单价为 24 元;\n\n(2) 解: $\\because \\mathrm{A}$ 种商品售价不低于 $B$ 种商品售价, $\\therefore 30-m \\geq 24$, 解得 $m \\leq 6$, 即 $0 \\leq m \\leq 6$,设利润为 $w$, 则\n\n$w=(40+10 m) \\times[(30-m-20)+(24-20)]$\n\n$=-10 m^{2}+100 m+560$\n\n$=-10(m-5)^{2}+810$,\n\n$\\because-10<0$,\n\n$\\therefore w$ 在 $m=5$ 时能取到最大值, 最大值为 810 ,\n\n$\\therefore$ 当 $m=5$ 时, 商场销售 $A, B$ 两种商品可获得总利润最大, 最大利润是 810 元.\n\n【点睛】本题考查二元一次方程组及二次函数解实际应用题, 读懂题意, 根据等量关系列出方程组,根据函数关系找到函数关系式分析是解决问题的关键.\n\n"} {"id": "270", "image": ["10319.jpg", "10320.jpg"], "answer": " (1)见解析;\n\n(2) $\\frac{25}{2}$.\n\n", "solution": "null", "level": "九年级", "question": "(6 分) 如图, 在直角 $\\triangle O A B$ 中, $\\angle O A B=90^{\\circ}$, 且点 $B$ 的坐标为(4,3).\n\n(1) 在图中画出 $\\triangle O A B$ 绕点 $O$ 逆时针旋转 $90^{\\circ}$ 后的 $\\triangle O A B_{1}$;\n\n(2)连接 $B B_{1}$, 则 $\\triangle O B B_{1}$ 的面积 $\\qquad$ .\n\n\n\n", "options": [], "subject": "变换几何", "analysis": "(1) 根据旋转角度、旋转中心及旋转方向确定各点的对称点, 顺次连接即可; (2)根据勾股定理计算即可得出.\n\n【详解】(1) 解: 如图, $\\triangle O A_{1} B_{1}$ 为所作:\n\n\n\n(2)解:依题意得, $O B=O B_{1}, \\angle B O B_{1}=90^{\\circ}$\n\n在 Rt $\\triangle A O B$ 中, $O B=\\sqrt{O A^{2}+A B^{2}}=\\sqrt{4^{2}+3^{2}}=5$,\n\n$\\therefore S_{\\triangle B O B_{1}}=\\frac{1}{2} O B \\cdot O B_{1}=\\frac{1}{2} \\times 5 \\times 5=\\frac{25}{2}$.\n\n【点睛】本题考查了利用旋转的性质作图, 勾股定理, 熟练掌握相关知识是解题的关键.\n\n"} {"id": "390", "image": ["10493.jpg", "10494.jpg"], "answer": " (1)见解析;(2)(-3,3), (-4, 1), ( - 2, 0), ( -1, 2)\n\n", "solution": "null", "level": "九年级", "question": "(8 分) 如图, 在平面直角坐标系中, 四边形 $A B C D$ 的坐标分别为 $A(-6,6)$, $B(-8,2), C(-4,0), D(-2,4)$. (1) 画出一个四边形 $A^{\\prime} B^{\\prime} C^{\\prime} D^{\\prime}$, 使四边形 $A^{\\prime} B^{\\prime} C^{\\prime} D^{\\prime}$ 与四边形 $A B C D$ 是以原点 $O$ 为位似中心,相似比为 $1: 2$ 的位似图形.\n\n(2) 直接写出点的坐标: $A^{\\prime}($ $\\qquad$ ),\n\n$B^{\\prime}$ $\\qquad$ ), $C^{\\prime}$ $\\qquad$ ), $D^{\\prime}($ $\\qquad$ ).\n\n\n\n", "options": [], "subject": "变换几何", "analysis": "(1)、(2) 把 $\\mathrm{A} 、 \\mathrm{~B} 、 \\mathrm{C} 、 \\mathrm{D}$ 的横纵坐标都乘以 $\\frac{1}{2}$ 得到四边形 $\\mathrm{A}^{\\prime} \\mathrm{B}^{\\prime} \\mathrm{C}^{\\prime} \\mathrm{D}^{\\prime}$ 四个顶点坐标,然后描点即可.\n\n【详解】解:(1)如图, 四边形 $\\mathrm{A}^{\\prime} \\mathrm{B}^{\\prime} \\mathrm{C}^{\\prime} \\mathrm{D}^{\\prime}$ 为所作;\n\n\n\n(2) $\\mathrm{A}^{\\prime}(-3,3), \\mathrm{B}^{\\prime}(-4,1), \\mathrm{C}^{\\prime}(-2,0), \\mathrm{D}^{\\prime}(-1,2)$.\n\n故答案为 $(-3,3),(-4,1),(-2,0),(-1,2)$.\n\n【点睛】本题考查作图-位似变换, 利用位似图形的性质得出对应点位置是解题的关键.\n\n"} {"id": "437", "image": ["10553.jpg", "10554.jpg", "10555.jpg", "10554.jpg", "10555.jpg"], "answer": " (1)解:由勾股定理可得原三角形的各边长分别为 $1, \\sqrt{2}, \\sqrt{5}$,\n\n所画三角形与原三角形的相似比为 $\\sqrt{2}: 1$, 则所画三角形的各边长分别为 $\\sqrt{2} 、 2 、 \\sqrt{10}$, 如下图所示\n\n\n\n(2)解: 由勾股定理可得原三角形的各边长分别为 $2,2 \\sqrt{2}, 2 \\sqrt{5}$,\n\n有一个边为公共边, 假设公共边为 2 , 并且所画三角形边长为 2 的边与原三角形边长为 $2 \\sqrt{2}$边相对应, 此时相似比为 $1: \\sqrt{2}$,\n\n则所画三角形的各边长为 $\\sqrt{2}, 2, \\sqrt{10}$, 如下图所示:\n\n\n\n【点睛】此题考查了相似三角形的判定和性质, 勾股定理, 解题的关键是熟练掌握相似三角形的判定和性质.\n", "solution": "null", "level": "九年级", "question": "(6 分) 如图, 在正方形网格中, 每个小正方形边长为 1 , 当三角形的三个顶点都在正方形网格线的交点上时, 我们称三角形为格点三角形.\n\n(1)如图 1, 请在图 1 中画一个格点三角形与原三角形相似, 且所画三角形与原三角形的相似比为 $\\sqrt{2}: 1$.\n\n(2)请在图 2 中画一个格点三角形与原三角形相似且有一条公共边, 并写出所画三角形与原三角形相似比. 相似比为: $\\qquad$\n\n\n\n图 1\n\n图2\n\n", "options": [], "subject": "变换几何", "analysis": ")解:由勾股定理可得原三角形的各边长分别为 $1, \\sqrt{2}, \\sqrt{5}$,\n\n所画三角形与原三角形的相似比为 $\\sqrt{2}: 1$, 则所画三角形的各边长分别为 $\\sqrt{2} 、 2 、 \\sqrt{10}$, 如下图所示\n\n\n\n(2)解: 由勾股定理可得原三角形的各边长分别为 $2,2 \\sqrt{2}, 2 \\sqrt{5}$,\n\n有一个边为公共边, 假设公共边为 2 , 并且所画三角形边长为 2 的边与原三角形边长为 $2 \\sqrt{2}$边相对应, 此时相似比为 $1: \\sqrt{2}$,\n\n则所画三角形的各边长为 $\\sqrt{2}, 2, \\sqrt{10}$, 如下图所示:\n\n\n\n【点睛】此题考查了相似三角形的判定和性质, 勾股定理, 解题的关键是熟练掌握相似三角形的判定和性质.\n\n"} {"id": "339", "image": ["10427.jpg"], "answer": " 证明见解析.\n\n", "solution": "null", "level": "九年级", "question": "(6 分) 如图, $A B$ 是 $\\odot O$ 的直径, 点 $C, D$ 是 $\\odot O$ 上的两点, 且 $A C=C D$,求证: $O C / / B D$.\n\n\n\n", "options": [], "subject": "图论", "analysis": "利用 $A C=C D$ 证明 $\\angle A B C=\\angle C B D$, 再利用 $O B=O C$ 证明 $\\angle A B C=\\angle O C B$, 利用等量代换可得 $\\angle O C B=\\angle C B D$, 即可证明 $O C / / B D$.\n\n【详解】证明: $\\because A C=C D$,\n\n$\\therefore \\angle A B C=\\angle C B D$,\n\n$\\because O B=O C$,\n\n$\\therefore \\angle A B C=\\angle O C B$,\n\n$\\therefore \\angle O C B=\\angle C B D$,\n\n$\\therefore O C / / B D$.\n\n【点睛】本题考查同圆中, 等弧对等角, 在同一个三角形中, 等边对等角, 平行线的判定定理, 解题的关键是熟练掌握以上相关知识, 并能够综合运用.\n\n"} {"id": "13161", "image": [], "answer": "答案: $\\times$", "solution": "null", "level": "六年级", "question": "直接写得数。(6 分)\n\n$$\n\\begin{array}{llll}\n2 \\times \\frac{5}{6}= & \\frac{1}{4}-\\frac{1}{6}=\\quad 1 \\div", "options": [], "subject": "算术", "analysis": "答案: $\\times$"} {"id": "13162", "image": [], "answer": "答案: \\frac{5}{3} & \\frac{1}{12} & 20 &", "solution": "null", "level": "六年级", "question": "求未知数 $x$ 。(12 分)\n$x-35 \\% x=", "options": [], "subject": "算术", "analysis": "答案: \\frac{5}{3} & \\frac{1}{12} & 20 &"} {"id": "13074", "image": [], "answer": "$27 \\div 60 \\% \\div 3$=45 \\div 3$$=15($ 元 $)$答: 每张门票原价 15 元。", "solution": "null", "level": "六年级", "question": "小红以六折优惠价买 3 张欢乐园儿童票, 一共花了 27 元。每张门票原价多少元?", "options": [], "subject": "算术", "analysis": ""} {"id": "13075", "image": [], "answer": "解: $60 \times 5 \\% \times 7 \\%=0.21$ (万元)答: 一月份缴纳城市维护建设费 0.21 万元。", "solution": "null", "level": "六年级", "question": "六一儿童节, 书店所有图书一律八五折销售, 小明买了一本《趣味数学》少花了", "options": [], "subject": "算术", "analysis": ""} {"id": "13076", "image": [], "answer": "解: $48 \\div(1+20 \\%)=40$ (吨)答: 这个林场去年共收特等茶叶 40 吨。", "solution": "null", "level": "六年级", "question": "阳台山林场有一块精品茶园, 今年共收特等茶叶 48 吨, 比去年增产二成。这个林场去年共收特等茶叶多少吨?", "options": [], "subject": "算术", "analysis": ""} {"id": "13077", "image": [], "answer": "(1) 解: $80 \times 6500 \times 90 \\%$$=520000 \times 0.9$$=468000($ 元 $)$答: 应付 468000 元.(2) 解: $80 \times 6500 \times 30 \\$$=520000 \times 0.3$$=156000$ (元)答: 首期应付 156000 元.(3)解: $468000 \times 1.5 \\%=7020$ (元)答: 要缴纳契税 7020 元.", "solution": "null", "level": "六年级", "question": "李叔叔在郊区购买了一套 80 平方米的商品房, 每平方米售价 6500 元; 如果一次性付清房款, 可享受九折优惠; 如果按揭贷款付款, 首期应付 $30 \\%$ 。\n\n(1) 一次性付款, 应付多少钱?\n\n(2)如果按揭贷款付款,首期应付多少钱?\n\n(3) 按规定买房要缴纳 $", "options": [], "subject": "算术", "analysis": ""} {"id": "13093", "image": ["2750.jpg", "2751.jpg"], "answer": "", "solution": "null", "level": "六年级", "question": "在直线上表示下列各数.\n\n27.在直线下面填整数或小数, 上面填分数.\n\n![](本地图片-4720/.jpg)", "options": [], "subject": "算术", "analysis": ""} {"id": "13113", "image": [], "answer": "解: $3+4=7$; 五年级植树: $420 \times \frac{3}{7}=180$ (棵)六年级植树: $420 \times \frac{4}{7}=240($ 棵 ) 或 $420-180=240$ (棵)", "solution": "null", "level": "六年级", "question": "学校高年级到小区植树 420 棵, 五年级和六年级植树棵数的比是 $3: 4$ ,五、六年级各植树多少棵?", "options": [], "subject": "算术", "analysis": ""} {"id": "13115", "image": [], "answer": "答案:解: $20 \\times 18 \\div 30=12$ (天)", "solution": "null", "level": "六年级", "question": "小明看一本故事书, 计划每天看 20 页, 18 天看完, 实际每天看 30 页,实际多少天看完?\n时?", "options": [], "subject": "算术", "analysis": "答案:解: $20 \\times 18 \\div 30=12$ (天)"} {"id": "13144", "image": [], "answer": "答案:6 \\quad", "solution": "null", "level": "六年级", "question": "怎样计算简单就怎样算: (4 分, 每题 2 分)\n\n$$\n\\frac{3}{5} \\times", "options": [], "subject": "算术", "analysis": "答案:6 \\quad"} {"id": "13145", "image": ["3011.jpg"], "answer": "$\begin{array}{llll}\frac{13}{5} & \frac{7}{16} & 100 & 9\\end{array}$", "solution": "null", "level": "六年级", "question": "求未知数 $X:$ (共 8 分, 每题 2 分)\n\n$\\frac{1}{2} x-\\frac{2}{5}=\\frac{9}{10}$\n\n$\\frac{4}{5}: \\frac{7}{15}=\\frac{3}{4}: x$\n\n$X: 80=5: 4$ $\\frac{1}{15}: \\frac{1}{5}=\\frac{1}{X}: \\frac{1}{3}$27.给右面的转盘按要求涂红.黄.白.蓝四种颜色并计算。\n\n\n\n(1)使指针停在红色区域的可能性是黄色区域的 2 倍, 停在蓝色区域的可能性是红色区域的 3 倍。\n\n(2) 在你设计的转盘上, 指针停在白色区域的可能性是多少?", "options": [], "subject": "算术", "analysis": ""} {"id": "26967", "image": [], "answer": "(1) $(100+95+91+88+94+96) \\div 6$\n\n$=564 \\div 6$\n\n$=94$ (分)\n\n答: 这六名同学的平均分是 94 分.\n\n(2) 100-94=+6 (分); 95-94=+1 (分); 91-94=-3 (分); 88-94=-6 (分); 94-94=0(分);96-94=+2(分)\n\n| 姓名 | 刘芳 | 李刚 | 张进 | 王英 | 叶莎 | 夏斌 |\n| :---: | :---: | :---: | :---: | :---: | :---: | :---: |\n| 与平均
分数比 | +6 | +1 | -3 | -6 | 0 | +2 |", "solution": "null", "level": "六年级", "question": " ( 10 分)用正、负数填表。\n\n四 (1) 班第一小组上学期期末数学测验成绩分别是: 刘芳 100 分, 李刚 95 分,张进 91 分, 王英 88 分, 叶莎 94 分, 夏斌 96 分. (先算出 6 个人的平均分,高于平均分的用正数表示, 低于平均分的用负数表示)\n\n(1) 这六名同学的平均分是多少?\n(2)\n\n| 姓名 | 刘芳 | 李刚 | 张进 | 王英 | 叶莎 | 夏斌 |\n| :---: | :---: | :---: | :---: | :---: | :---: | :---: |\n| 与平均
分数比 | | | | | | |", "options": [], "subject": "算术", "analysis": "(1) $(100+95+91+88+94+96) \\div 6$\n\n$=564 \\div 6$\n\n$=94$ (分)\n\n答: 这六名同学的平均分是 94 分.\n\n(2) 100-94=+6 (分); 95-94=+1 (分); 91-94=-3 (分); 88-94=-6 (分); 94-94=0(分);96-94=+2(分)\n\n| 姓名 | 刘芳 | 李刚 | 张进 | 王英 | 叶莎 | 夏斌 |\n| :---: | :---: | :---: | :---: | :---: | :---: | :---: |\n| 与平均
分数比 | +6 | +1 | -3 | -6 | 0 | +2 |"} {"id": "26969", "image": [], "answer": "解:向北走记为正, 那么向南走记为负。\n\n$$\n(+5)+(-4)+(+3)+(-2)+(+1)=3 \\text { (米) }\n$$\n\n答:现在所在位置距离出发点 3 米的北方。", "solution": "null", "level": "六年级", "question": " (5 分)向北走 5 米,然后向南走 4 米,再向北走 3 米,再向南走 2 米,再向北走 1 米,现在所在位置距离出发点几米的什么方向?", "options": [], "subject": "算术", "analysis": "解:向北走记为正, 那么向南走记为负。\n\n$$\n(+5)+(-4)+(+3)+(-2)+(+1)=3 \\text { (米) }\n$$\n\n答:现在所在位置距离出发点 3 米的北方。"} {"id": "26972", "image": [], "answer": "解:抢答 5 次,答对 4 道题,答错 1 道题。", "solution": "null", "level": "六年级", "question": " ( 5 分)一次智力测验比赛共 10 道抢答题, 规定答对 1 题得 5 分, 答错 1 题得 -8 分,不答得 0 分,小明共得 12 分,他抢答几次?答对几题?答错几题?", "options": [], "subject": "算术", "analysis": "解:抢答 5 次,答对 4 道题,答错 1 道题。"} {"id": "26994", "image": [], "answer": "$5200 \\times 85 \\%=4420$ (元)", "solution": "null", "level": "六年级", "question": " (5 分)有一台电脑, 定价 5200 元. 如果八五折(原价的 85\\%)出售,售价是多少元?", "options": [], "subject": "算术", "analysis": "$5200 \\times 85 \\%=4420$ (元)"} {"id": "26995", "image": [], "answer": "$960 \\div 80 \\% \\div(1+50 \\%)=800$ (元)", "solution": "null", "level": "六年级", "question": " (5分)服装店进了一套衣服,按 $50 \\%$ 的盈利标价出售,第一周没有售出, 第二周按标价的八折出售, 以 960 元的价格售出了。这套衣服的进价是多少?", "options": [], "subject": "算术", "analysis": "$960 \\div 80 \\% \\div(1+50 \\%)=800$ (元)"} {"id": "26996", "image": [], "answer": "$20000 \\times 4.25 \\% \\times 3=2550$ (元)", "solution": "null", "level": "六年级", "question": " ( 5 分)小丽的妈妈在 2013 年 1 月 1 日把 20000 元钱存入银行, 整存整取 3 年, 年利率是 $4.25 \\%$ 。到期时她可以得到利息多少元?", "options": [], "subject": "算术", "analysis": "$20000 \\times 4.25 \\% \\times 3=2550$ (元)"} {"id": "26997", "image": [], "answer": "$288 \\div 80 \\% \\div(1+20 \\%)=300$ (元)\n\n赔了 $300-288=12$ (元)", "solution": "null", "level": "六年级", "question": " (5分)一件商品按成本价提高 $20 \\%$ 后标价,后来因季节关系,按标价的八折降价出售. 降价后卖 288 元。这件商品卖出后是赚还是赔?赚或赔了多少钱?", "options": [], "subject": "算术", "analysis": "$288 \\div 80 \\% \\div(1+20 \\%)=300$ (元)\n\n赔了 $300-288=12$ (元)"} {"id": "26998", "image": [], "answer": "$(3600-800) \\times(1-14 \\%)+800=3208$ (元)", "solution": "null", "level": "六年级", "question": " (5分)小丽的爸爸写了一本《数学家的故事》,应得稿费 3600 元。按规定:一次稿费超过 800 元的部分按 14\\%的税率纳税。小丽爸爸的稿费实际收入是多少元?", "options": [], "subject": "算术", "analysis": "$(3600-800) \\times(1-14 \\%)+800=3208$ (元)"} {"id": "26999", "image": [], "answer": "$168 \\div\\{(1+20 \\%) \\times 88 \\%-1)=3000$ (元)", "solution": "null", "level": "六年级", "question": " (5 分)一款电视按 $20 \\%$ 的利润定价, 然后按八八折卖出, 共得利润 168 元,这款电视机的成本是多少元?", "options": [], "subject": "算术", "analysis": "$168 \\div\\{(1+20 \\%) \\times 88 \\%-1)=3000$ (元)"} {"id": "27059", "image": [], "answer": "设需要走 $x$ 时.\n\n$6: x=4.5: 1.5$ 或 $\\frac{6}{x}=\\frac{4.5}{1.5}$;\n\n$\\mathrm{x}=2$", "solution": "null", "level": "六年级", "question": " (6 分)学校组织同学们进行耐力训练,1.5 小时行路 4.5 千米. 照这样的速度, 要行 6 千米,需要走多少小时?", "options": [], "subject": "算术", "analysis": "设需要走 $x$ 时.\n\n$6: x=4.5: 1.5$ 或 $\\frac{6}{x}=\\frac{4.5}{1.5}$;\n\n$\\mathrm{x}=2$"} {"id": "27060", "image": [], "answer": "方法 $1: 2+3+4=9, \\quad 180 \\times \\frac{3}{9}=60$ (张), $80-60=20$ (张).\n\n方法 2: $2+3+4=9, \\quad 180 \\times\\left(\\frac{4}{9}-\\frac{3}{9}\\right)=180 \\times \\frac{1}{9}=20$ (张).", "solution": "null", "level": "六年级", "question": " (6 分)王老师有 180 张纪念邮票,他按 $2: 3: 4$ 分给小聪、小明、和小好. 小好比小明多分了多少张?", "options": [], "subject": "算术", "analysis": "方法 $1: 2+3+4=9, \\quad 180 \\times \\frac{3}{9}=60$ (张), $80-60=20$ (张).\n\n方法 2: $2+3+4=9, \\quad 180 \\times\\left(\\frac{4}{9}-\\frac{3}{9}\\right)=180 \\times \\frac{1}{9}=20$ (张)."} {"id": "27062", "image": [], "answer": "方法 $1:$ 养鸡:\n\n$$\n120 \\times \\frac{5}{8}=75(\\text { 只 }), \\text { 养鸭: } 120 \\times \\frac{3}{8}=45(\\text { 只 }), 75-45=30 \\text { (只) }\n$$\n\n方法 2 :\n\n$$\n120 \\times\\left(\\frac{5}{8}-\\frac{3}{8}\\right)=120 \\times \\frac{1}{4}=30(\\text { 只 })\n$$", "solution": "null", "level": "六年级", "question": " (6 分)小聪家共养鸡和鸭 120 只, 鸡和鸭只数的比是 $5 : 3$. 他家养的鸡比鸭多多少只?", "options": [], "subject": "算术", "analysis": "方法 $1:$ 养鸡:\n\n$$\n120 \\times \\frac{5}{8}=75(\\text { 只 }), \\text { 养鸭: } 120 \\times \\frac{3}{8}=45(\\text { 只 }), 75-45=30 \\text { (只) }\n$$\n\n方法 2 :\n\n$$\n120 \\times\\left(\\frac{5}{8}-\\frac{3}{8}\\right)=120 \\times \\frac{1}{4}=30(\\text { 只 })\n$$"} {"id": "27064", "image": [], "answer": "$100 \\div(5-3) \\times 5=250$ (克)", "solution": "null", "level": "六年级", "question": " (6 分)一块合金中含有金和锡, 锡和金的质量比是 5:3, 已知锡比金多 100 克, 锡有多少克?", "options": [], "subject": "算术", "analysis": "$100 \\div(5-3) \\times 5=250$ (克)"} {"id": "27118", "image": [], "answer": "(1)解: $95-87=8$ (分) 即 +8 分\n\n答: 李阳得了 95 分, 应记作 +8 分。\n\n(2) $87+(-5)=82$ (分)\n\n答: 刘洋被记作了-5 分, 他实际得分是 82 。\n\n(3)解: $87-87=0$ (分)\n\n答:王刚得了 87 分,应记作 0 分。\n\n(4) $95-82=13$ (分)\n\n答: 李阳和刘洋相差 13 分。", "solution": "null", "level": "六年级", "question": " (16 分) 在一次数学测试中,六(1)班的平均成绩 87 分,把高于平均分的记作正数。\n\n(1)(4 分)李阳得了 95 分,应记作多少?\n\n(2)(4 分)刘洋被记作了-5 分,他实际得分是多少?\n\n(3)(4 分)王刚得了 87 分,应记作多少?\n\n(4)(4 分)李阳和刘洋相差多少分?", "options": [], "subject": "算术", "analysis": "(1)解: $95-87=8$ (分) 即 +8 分\n\n答: 李阳得了 95 分, 应记作 +8 分。\n\n(2) $87+(-5)=82$ (分)\n\n答: 刘洋被记作了-5 分, 他实际得分是 82 。\n\n(3)解: $87-87=0$ (分)\n\n答:王刚得了 87 分,应记作 0 分。\n\n(4) $95-82=13$ (分)\n\n答: 李阳和刘洋相差 13 分。"} {"id": "27122", "image": [], "answer": "$0^{\\circ} \\mathrm{C}>-39^{\\circ} \\mathrm{C}>-117^{\\circ} \\mathrm{C}$\n\n$0^{\\circ} \\mathrm{C}$ 与 $-117^{\\circ} \\mathrm{C}$ 相差 $117^{\\circ} \\mathrm{C}$\n\n答: 水的冻结温度最高, 酒精的冻结温度最低, 相差 $117^{\\circ} \\mathrm{C}$.", "solution": "null", "level": "六年级", "question": " (5 分) 水冻结的温度是 $0^{\\circ} \\mathrm{C}$, 酒精冻结的温度是 $-117^{\\circ} \\mathrm{C}$, 水银冻结的温度是 $-39^{\\circ} \\mathrm{C}$, 哪一个冻结温度最高?哪一个最低, 它们相差多少?", "options": [], "subject": "算术", "analysis": "$0^{\\circ} \\mathrm{C}>-39^{\\circ} \\mathrm{C}>-117^{\\circ} \\mathrm{C}$\n\n$0^{\\circ} \\mathrm{C}$ 与 $-117^{\\circ} \\mathrm{C}$ 相差 $117^{\\circ} \\mathrm{C}$\n\n答: 水的冻结温度最高, 酒精的冻结温度最低, 相差 $117^{\\circ} \\mathrm{C}$."} {"id": "27148", "image": [], "answer": "$60 \\times 2 \\%+60 \\times 0.03 \\%=1.218$ (万元)", "solution": "null", "level": "六年级", "question": " (6 分) 王萍家购买了一套新房,总价 60 万元。按规定,王萍家还要按购房总价的 2\\%缴纳契税,按购房总价的 $0.03 \\%$ 缴纳印花税。她家一共要缴纳多少万元税费?", "options": [], "subject": "算术", "analysis": "$60 \\times 2 \\%+60 \\times 0.03 \\%=1.218$ (万元)"} {"id": "27149", "image": [], "answer": "解: $3000 \\times 90 \\% \\times 95 \\%=2700 \\times 0.95=2565$ (元)答:实际用了 2565 元.", "solution": "null", "level": "六年级", "question": " (6 分)一台全自动洗衣机原价 3000 元, 现价打九折出售; 如果有贵宾卡,还可以再打九五折 。王叔叔用贵宾卡买了一台洗衣机 ,实际用了多少元?", "options": [], "subject": "算术", "analysis": "解: $3000 \\times 90 \\% \\times 95 \\%=2700 \\times 0.95=2565$ (元)答:实际用了 2565 元."} {"id": "27150", "image": [], "answer": "$5000-800=4200$ (元) $4200 \\times 14 \\%=588$ (元).\n\n答: 陈老师应交税 588 元.", "solution": "null", "level": "六年级", "question": " (6 分) 陈老师出版了一本《小学数学解答 100 问》, 获得稿费 5000 元, 按规定, 超出 800 元的部分应缴纳 14\\%的个人所得税。陈老师应交税多少元?", "options": [], "subject": "算术", "analysis": "$5000-800=4200$ (元) $4200 \\times 14 \\%=588$ (元).\n\n答: 陈老师应交税 588 元."} {"id": "27151", "image": [], "answer": "解:设这辆车的原价是 $\\mathrm{x}$ 元.\n\n$x+7 \\% x-95 \\% x=8400$\n\n$0.12 x=8400$\n\n$\\mathrm{x}=70000$\n\n答: 这辆车的原价是 70000 元.", "solution": "null", "level": "六年级", "question": " (6 分)爸爸去头汽车,销售人员告诉他,如果分期付款加价 7\\%,如果现金购买可享受九五折优惠。爸爸算了算, 发现分期付款比现金购头要多付 8400 元。这辆车的原价是多少元?", "options": [], "subject": "算术", "analysis": "解:设这辆车的原价是 $\\mathrm{x}$ 元.\n\n$x+7 \\% x-95 \\% x=8400$\n\n$0.12 x=8400$\n\n$\\mathrm{x}=70000$\n\n答: 这辆车的原价是 70000 元."} {"id": "27153", "image": [], "answer": "$20000+20000 \\times 3 \\times 2.75 \\%=21650$ (元)\n\n答:到期后实际能取回 21650 元钱.", "solution": "null", "level": "六年级", "question": " (6 分)王大伯把 20000 元人民币存入银行, 定期 3 年, 年利率为 $2.75 \\%$, 到期后实际能取回多少钱?", "options": [], "subject": "算术", "analysis": "$20000+20000 \\times 3 \\times 2.75 \\%=21650$ (元)\n\n答:到期后实际能取回 21650 元钱."} {"id": "27154", "image": [], "answer": "$30000+30000 \\times 4.5 \\% \\times 3$\n\n$=30000+4050$\n\n$=34050$ (元)\n\n答:到期王阿姨可以领到利息和本金 34050 元.", "solution": "null", "level": "六年级", "question": " (6 分)2016 年的银行年利率是 4.5\\%,王阿姨把 30000 元钱存入银行 3 年,到期王阿姨可以领到利息和本金共多少元?", "options": [], "subject": "算术", "analysis": "$30000+30000 \\times 4.5 \\% \\times 3$\n\n$=30000+4050$\n\n$=34050$ (元)\n\n答:到期王阿姨可以领到利息和本金 34050 元."} {"id": "27155", "image": [], "answer": "$15 \\div(1-90 \\%)=15 \\div 10 \\%=150$ (元)答: 这个篮球原价 150 元.", "solution": "null", "level": "六年级", "question": " (6 分)体育用品商店的篮球打九折销售, 浩浩头了一个篮球便宜了 15 元, 这个篮球原价多少元?", "options": [], "subject": "算术", "analysis": "$15 \\div(1-90 \\%)=15 \\div 10 \\%=150$ (元)答: 这个篮球原价 150 元."} {"id": "27156", "image": [], "answer": "银行的利率分别为:定期一年 3.25\\% 定期两年 3.75\\%一年定期到期后连本带息再存一年定期获得的利息为:\n\n$2000 \\times(1+3.25 \\%)(1+3.25 \\%)-2000=2000 \\times 1.0325 \\times 1.0325-2000=2065 \\times 1.0325-2000=2132.11-2000=132$.11 (元) 直接存两年期到期后获得的利息为: $2000 \\times 3.75 \\% \\times 2=2000 \\times 0.0375 \\times 2=150$ (元) $150>132.11$\n\n答:直接存两年期到期后获得的利息多.", "solution": "null", "level": "六年级", "question": " (6 分)利用收集到的存款利率算一算:甲用 2000 元先存一年定期,到期后连本带息再存一年定期;乙用 2000 元直接存了二年定期. 哪种存款方式到期后获得的利息多?", "options": [], "subject": "算术", "analysis": "银行的利率分别为:定期一年 3.25\\% 定期两年 3.75\\%一年定期到期后连本带息再存一年定期获得的利息为:\n\n$2000 \\times(1+3.25 \\%)(1+3.25 \\%)-2000=2000 \\times 1.0325 \\times 1.0325-2000=2065 \\times 1.0325-2000=2132.11-2000=132$.11 (元) 直接存两年期到期后获得的利息为: $2000 \\times 3.75 \\% \\times 2=2000 \\times 0.0375 \\times 2=150$ (元) $150>132.11$\n\n答:直接存两年期到期后获得的利息多."} {"id": "27157", "image": [], "answer": "(2300-800)x14\\%=210 元答: 李老师应缴纳个人所得税 210 元.", "solution": "null", "level": "六年级", "question": " (7 分)李老师编写了一本儿童读物,在出版社出版后获得稿费 2300 元. 按规定,稿费收入扣除 800 元后按 $14 \\%$ 的税率缴纳个人所得税,李老师应缴纳个人所得税多少元?", "options": [], "subject": "算术", "analysis": "(2300-800)x14\\%=210 元答: 李老师应缴纳个人所得税 210 元."} {"id": "27210", "image": [], "answer": "(1) $x=96-64$\n\n$$\nx=32\n$$\n\n(2) $25 x=8$\n\n$25 x \\div 25=8 \\div 25$\n\n$$\nx=\\frac{8}{25}\n$$\n\n(3) $70 \\% x=280$\n\n$70 \\% x \\div 70 \\%=280 \\div 70 \\%$\n\n$$\nx=400\n$$\n\n(4) $0.8 \\mathrm{x}=\\frac{4}{3}$\n\n$0.8 x \\div \\frac{4}{5}=\\frac{4}{3} \\div \\frac{4}{5}$\n\n$$\n{ }^{x}=\\frac{5}{3}\n$$", "solution": "null", "level": "六年级", "question": " (12 分)解方程\n(1) $96-x=64$\n(2) $\\frac{1}{5}=\\frac{5 x}{8}$\n(3) $20 \\% x+{ }_{\\frac{1}{2}} x=280$\n(4) 1.6x:\n$\\frac{6}{13}=\\frac{26}{9}: 0.5$", "options": [], "subject": "算术", "analysis": "(1) $x=96-64$\n\n$$\nx=32\n$$\n\n(2) $25 x=8$\n\n$25 x \\div 25=8 \\div 25$\n\n$$\nx=\\frac{8}{25}\n$$\n\n(3) $70 \\% x=280$\n\n$70 \\% x \\div 70 \\%=280 \\div 70 \\%$\n\n$$\nx=400\n$$\n\n(4) $0.8 \\mathrm{x}=\\frac{4}{3}$\n\n$0.8 x \\div \\frac{4}{5}=\\frac{4}{3} \\div \\frac{4}{5}$\n\n$$\n{ }^{x}=\\frac{5}{3}\n$$"} {"id": "27211", "image": [], "answer": "(1) $0.2: 4=\\mathrm{x}: 20$\n\n$$\n\\begin{aligned}\n& 4 x=0.2 \\times 20 \\\\\n& 4 x=4 \\\\\n& x=4 \\div 4 \\\\\n& x=1 \\\\\n& \\text { (2) } 2: \\frac{1}{20}^{2}=8: x\n\\end{aligned}\n$$\n\n$$\n\\begin{aligned}\n& 2 x=\\frac{1}{20}^{\\times 8} \\\\\n& 2 x=0.4 \\\\\n& x=0.4 \\div 2 \\\\\n& x=0.2\n\\end{aligned}\n$$", "solution": "null", "level": "六年级", "question": " (8 分)根据下面的条件写出比例, 并解比例\n\n(1)(4 分) 0.2 与 4 的比等于 $x$ 与 20 的比。\n\n(2)(4 分)比的两个外项是 2 与 $x$, 两个内项是 $\\frac{1}{20}$ 与 8 。", "options": [], "subject": "算术", "analysis": "(1) $0.2: 4=\\mathrm{x}: 20$\n\n$$\n\\begin{aligned}\n& 4 x=0.2 \\times 20 \\\\\n& 4 x=4 \\\\\n& x=4 \\div 4 \\\\\n& x=1 \\\\\n& \\text { (2) } 2: \\frac{1}{20}^{2}=8: x\n\\end{aligned}\n$$\n\n$$\n\\begin{aligned}\n& 2 x=\\frac{1}{20}^{\\times 8} \\\\\n& 2 x=0.4 \\\\\n& x=0.4 \\div 2 \\\\\n& x=0.2\n\\end{aligned}\n$$"} {"id": "27212", "image": [], "answer": "设还需要 $x$ 元。\n\n$$\n\\frac{x}{80}=\\frac{192}{30} \\quad \\mathrm{x}=512\n$$", "solution": "null", "level": "六年级", "question": " (4 分)宏达书店购进 30 本《格林童话》,花了 192 元,由于供不应求,老板决定再购进 80 本,还需要多少元?", "options": [], "subject": "算术", "analysis": "设还需要 $x$ 元。\n\n$$\n\\frac{x}{80}=\\frac{192}{30} \\quad \\mathrm{x}=512\n$$"} {"id": "27214", "image": [], "answer": "设每人要栽 $x$ 棵。\n\n$(40+10) x=40 \\times 15$\n\n$$\n\\begin{aligned}\n& 50 x=600 \\\\\n& x=600 \\div 50 \\\\\n& x=12\n\\end{aligned}\n$$\n\n答: 每人要栽 12 棵。", "solution": "null", "level": "六年级", "question": " (4 分) 有一批树苗,原计划 40 人去栽,每人要栽 15 棵,后来增加 10 人去栽,每人要栽多少棵?", "options": [], "subject": "算术", "analysis": "设每人要栽 $x$ 棵。\n\n$(40+10) x=40 \\times 15$\n\n$$\n\\begin{aligned}\n& 50 x=600 \\\\\n& x=600 \\div 50 \\\\\n& x=12\n\\end{aligned}\n$$\n\n答: 每人要栽 12 棵。"} {"id": "27215", "image": [], "answer": "设可以换旧管 $\\mathrm{x}$ 根.\n\n$5 x=8 \\times 200$\n\n$x=1600 \\div 5$\n\n$x=320$\n\n答: 可以换 320 根旧管.", "solution": "null", "level": "六年级", "question": " (4 分)城建工人修建一条自来水管道, 用 8 米长的新管换原来 5 米长的旧管。现在用新管 200 根,可以换旧管多少根?", "options": [], "subject": "算术", "analysis": "设可以换旧管 $\\mathrm{x}$ 根.\n\n$5 x=8 \\times 200$\n\n$x=1600 \\div 5$\n\n$x=320$\n\n答: 可以换 320 根旧管."} {"id": "27217", "image": [], "answer": "解:设返回时用了 $x$ 小时. $45 \\times(8-x)=35 x 360-45 x=35 x 35 x+45 x=36080 x=360 x=360 \\div 80$ $\\mathrm{x}=4.5$\n\n$35 \\times 4.5=157.5(\\mathrm{~km})$\n\n答: $\\mathrm{AB}$ 两地相距 157.5 千米.", "solution": "null", "level": "六年级", "question": " (4 分)一艘货轮往返于 A、B 两港之间一次共用 8 小时,由于顺风,从 $\\mathrm{A}$ 港开往 $\\mathrm{B}$ 港每小时行 $45 \\mathrm{~km}$ ,返回时每小时行 $35 \\mathrm{~km}$ ,A、B 两港相距多少 $\\mathrm{km}$ ?", "options": [], "subject": "算术", "analysis": "解:设返回时用了 $x$ 小时. $45 \\times(8-x)=35 x 360-45 x=35 x 35 x+45 x=36080 x=360 x=360 \\div 80$ $\\mathrm{x}=4.5$\n\n$35 \\times 4.5=157.5(\\mathrm{~km})$\n\n答: $\\mathrm{AB}$ 两地相距 157.5 千米."} {"id": "27218", "image": [], "answer": "设模型长度 $\\mathrm{x}$ 米. $500: \\mathrm{x}=800: 1800 \\mathrm{x}=500 \\times 1 \\mathrm{x}=500 \\div 800 \\mathrm{x}=5$\n\n$\\overline{8}$\n\n答: 模型长 ${ }_{5}^{8}$ 米.", "solution": "null", "level": "六年级", "question": " (4 分) 一列火车的实际长度是 500 米, 它的长度与模型长度的比是 $800: 1$, 模型长度是多少米?", "options": [], "subject": "算术", "analysis": "设模型长度 $\\mathrm{x}$ 米. $500: \\mathrm{x}=800: 1800 \\mathrm{x}=500 \\times 1 \\mathrm{x}=500 \\div 800 \\mathrm{x}=5$\n\n$\\overline{8}$\n\n答: 模型长 ${ }_{5}^{8}$ 米."} {"id": "26088", "image": ["13081.jpg", "13081.jpg"], "answer": "", "solution": "null", "level": "六年级", "question": " 直接写出得数。(10 分)\n $\\frac{1}{3} \\times 0=$\n $\\frac{1}{4} \\times \\frac{2}{5}=$\n $\\frac{5}{6} \\times 12=$\n $\\frac{7}{12} \\times \\frac{3}{14}=$\n $45 \\times \\frac{3}{5}=$\n $9 \\times \\frac{7}{18}=\\quad \\frac{2}{3} \\times \\frac{9}{10}=$\n $\\frac{4}{25} \\times 100$\n $18 \\times \\frac{1}{6}=$\n $\\frac{4}{11} \\times \\frac{11}{4}=$", "options": [], "subject": "算术", "analysis": ""} {"id": "26089", "image": [], "answer": "$9 \\frac{9}{16} \\quad 44 \\quad \\frac{5}{9} \\quad \\frac{5}{2} \\quad \\frac{4}{15} 14$", "solution": "null", "level": "六年级", "question": " 能简算的要简算。\n $17 \\times \\frac{9}{16}$\n $\\left(\\frac{3}{4}+\\frac{5}{8}\\right) \\times 32$\n $\\frac{5}{9} \\times \\frac{3}{4}+\\frac{5}{9} \\times \\frac{1}{4}$\n $\\frac{4}{5} \\times \\frac{1}{8} \\times 16$\n $\\frac{1}{5}+\\frac{2}{9} \\times \\frac{3}{10}$\n $44-72 \\times \\frac{5}{12}$", "options": [], "subject": "算术", "analysis": "$9 \\frac{9}{16} \\quad 44 \\quad \\frac{5}{9} \\quad \\frac{5}{2} \\quad \\frac{4}{15} 14$"} {"id": "26090", "image": ["13082.jpg"], "answer": "$400 \\times\\left(1-\\frac{3}{5}\\right)=160$ (m)\n\n$168 \\times\\left(1+\\frac{2}{7}\\right)=216(\\mathrm{t})$\n\n$480 \\times \\frac{5}{6}=400$ (元)\n\n$160 \\times \\frac{7}{8} \\times \\frac{4}{5}=112$ (朵)", "solution": "null", "level": "六年级", "question": " 看图列式计算。(12 分)\n ", "options": [], "subject": "算术", "analysis": "$400 \\times\\left(1-\\frac{3}{5}\\right)=160$ (m)\n\n$168 \\times\\left(1+\\frac{2}{7}\\right)=216(\\mathrm{t})$\n\n$480 \\times \\frac{5}{6}=400$ (元)\n\n$160 \\times \\frac{7}{8} \\times \\frac{4}{5}=112$ (朵)"} {"id": "26091", "image": [], "answer": "$48 \\times\\left(1+\\frac{1}{4}\\right)=60$ (张)", "solution": "null", "level": "六年级", "question": " 小芳有 48 张邮票, 小军的邮票张数比小芳多 $\\frac{1}{4}$, 小军有多少张邮票? (4 分)", "options": [], "subject": "算术", "analysis": "$48 \\times\\left(1+\\frac{1}{4}\\right)=60$ (张)"} {"id": "26092", "image": [], "answer": "$480 \\times \\frac{5}{8}+45=345$ (人)", "solution": "null", "level": "六年级", "question": " 制衣厂有男职工 480 人,女职工人数比男职工人数的 $\\frac{5}{8}$ 还多 45 人,女职工有多少人?(4 分)", "options": [], "subject": "算术", "analysis": "$480 \\times \\frac{5}{8}+45=345$ (人)"} {"id": "26093", "image": [], "answer": "$500 \\times \\frac{4}{5} \\times \\frac{8}{9}=450$ (元)", "solution": "null", "level": "六年级", "question": " 六年级同学给灾区捐款 $\\frac{9}{8}$ 六 (1) 班捐了 500 元,六 (2) 班捐的钱数是六 (1) 班的 $\\frac{4}{5}$,六 (3) 班捐的钱数是六 (2) 班的 。六 (3) 班捐款多少元?(4 分)", "options": [], "subject": "算术", "analysis": "$500 \\times \\frac{4}{5} \\times \\frac{8}{9}=450$ (元)"} {"id": "26094", "image": [], "answer": "$180 \\times\\left(1-\\frac{1}{5}\\right.$ ) $=144$ (元)", "solution": "null", "level": "六年级", "question": " 一件西服原价 180 元,现在的价格比原价降低了 $\\frac{1}{5}$, 现在的价格是多少元? (5 分)", "options": [], "subject": "算术", "analysis": "$180 \\times\\left(1-\\frac{1}{5}\\right.$ ) $=144$ (元)"} {"id": "26095", "image": [], "answer": "$5 \\times \\frac{2}{5}-\\frac{2}{5}=\\frac{8}{5} \\quad(\\mathrm{~km})$", "solution": "null", "level": "六年级", "question": " 修路队计划修路 $5 \\mathrm{~km}$, 已经修了 $\\frac{2}{5} \\mathrm{~km}$, 还要修多少千米, 就正好修了全长的 $\\frac{2}{5} ?$ (5 分)", "options": [], "subject": "算术", "analysis": "$5 \\times \\frac{2}{5}-\\frac{2}{5}=\\frac{8}{5} \\quad(\\mathrm{~km})$"} {"id": "26096", "image": [], "answer": "$\\frac{3}{4} x(9-1)=6$ (分钟)", "solution": "null", "level": "六年级", "question": " 一根钢管锯成 2 段要 $\\frac{3}{4}$ 分钟, 若锯成 9 段需要多少分钟? (5 分)", "options": [], "subject": "算术", "analysis": "$\\frac{3}{4} x(9-1)=6$ (分钟)"} {"id": "26097", "image": [], "answer": "$138 \\frac{68}{69}$", "solution": "null", "level": "六年级", "question": " 计算: $139 \\times \\frac{137}{138}+137 \\times \\frac{1}{138}$", "options": [], "subject": "算术", "analysis": "$138 \\frac{68}{69}$"} {"id": "26112", "image": [], "answer": "$\\frac{1}{4} \\quad \\frac{21}{16} \\quad 8 \\quad\\frac{1}{3} \\quad 0\\quad \\frac{4}{5}$", "solution": "null", "level": "六年级", "question": " 看谁算得又对又快。(18 分)\n\n$\\frac{3}{4} \\times \\frac{2}{3} \\div 2$\n\n$\\left(\\frac{1}{6}+\\frac{1}{8}\\right) \\div \\frac{2}{9}$\n\n$10-1.5 \\div \\frac{3}{4}$\n\n$\\frac{7}{10} \\div \\frac{16}{5} \\div \\frac{21}{32}$\n\n$2-\\frac{6}{13} \\div \\frac{9}{26}-\\frac{2}{3}$\n\n$0.8 \\times \\frac{2}{3}+0.2 \\div 0.75$", "options": [], "subject": "算术", "analysis": "$\\frac{1}{4} \\quad \\frac{21}{16} \\quad 8 \\quad\\frac{1}{3} \\quad 0\\quad \\frac{4}{5}$"} {"id": "13163", "image": ["3071.jpg", "3071.jpg"], "answer": "答案:(1)-150 $\\quad$ (2) 提示:画出线段长 2 厘米。(3) 提示: 画出线段距上元大街2.5 厘米。\n\n\n\n\n\n25.答案: 解:设六年级募捐善款 $x$ 元, 五年级募捐善款 $\\frac{3}{4} x$ 元。\n\n$x+\\frac{3}{4} x=2800 \\quad x=1600 \\quad \\frac{3}{4} x \\frac{3}{4} \\times 1600=1200$", "solution": "null", "level": "六年级", "question": "在 “玉树抗震救灾献爱心” 捐款活动中, 科学园小学五、六年级同学共募捐善款 2800 元, 其中五年级募捐的款项是六年级的 $\\frac{3}{4}$ ,五、六年级各募捐善款多少元?(用方程解)", "options": [], "subject": "代数", "analysis": "答案:(1)-150 $\\quad$ (2) 提示:画出线段长 2 厘米。(3) 提示: 画出线段距上元大街2.5 厘米。\n\n\n\n\n\n25.答案: 解:设六年级募捐善款 $x$ 元, 五年级募捐善款 $\\frac{3}{4} x$ 元。\n\n$x+\\frac{3}{4} x=2800 \\quad x=1600 \\quad \\frac{3}{4} x \\frac{3}{4} \\times 1600=1200$"} {"id": "26113", "image": [], "answer": "$ x=\\frac{1}{10} \\quad x=\\frac{1}{3} \\quad x=24$", "solution": "null", "level": "六年级", "question": " 解方程。(9 分)\n\n$\\mathrm{x} \\div \\frac{5}{6}=\\frac{5}{12}$\n\n$\\frac{1}{4} x+\\frac{3}{4}=\\frac{5}{6}$\n\n$x+\\frac{1}{2} x=36$", "options": [], "subject": "代数", "analysis": "$ x=\\frac{1}{10} \\quad x=\\frac{1}{3} \\quad x=24$"} {"id": "26237", "image": [], "answer": "苹果: $240 \\div\\left(1+\\frac{3}{5}\\right)=150(\\mathrm{~kg})$\n\n梨: $240-150=90(\\mathrm{~kg})$\n\n或 $150 x \\frac{3}{5}=90(\\mathrm{~kg})$", "solution": "null", "level": "六年级", "question": " 水果店运进苹果和梨共 $240 \\mathrm{~kg}$, 其中梨的质量是苹果的 $\\frac{3}{5}$, 运进苹果和梨各多少千克?(6 分)", "options": [], "subject": "代数", "analysis": "苹果: $240 \\div\\left(1+\\frac{3}{5}\\right)=150(\\mathrm{~kg})$\n\n梨: $240-150=90(\\mathrm{~kg})$\n\n或 $150 x \\frac{3}{5}=90(\\mathrm{~kg})$"} {"id": "26274", "image": [], "answer": "(1) $120 \\times \\frac{3}{4}-15$\n (2) $30 \\div(120+30)$\n (3) $140 \\times\\left(1+\\frac{2}{7}\\right)$\n (4) $357 \\div(1+75 \\%)$", "solution": "null", "level": "六年级", "question": " 只列综合算式或方程,不必计算。(16 分)\n\n(1) 图书馆里连环画册的本数是科技书的 $\\frac{3}{4}$, 故事书的本数比连环画册少 15 本。已知科技书有 120 本,故事书有多少本? (4 分)\n\n(2)学校建综合楼,实际投资 120 万元,节约了 30 万元,节约了百分之几?(4 分)\n\n(3) 为迎接元旦, 学校制作了一批纸花, 其中红花 140 朵, 黄花比红花多 $\\frac{2}{7}$, 黄花有多少朵? (4 分)\n\n(4)希望小学五、六年级一共有学生 357 人,五年级学生人数是六年级的 $75 \\%$ 。六年级有多少学生?", "options": [], "subject": "代数", "analysis": "(1) $120 \\times \\frac{3}{4}-15$\n (2) $30 \\div(120+30)$\n (3) $140 \\times\\left(1+\\frac{2}{7}\\right)$\n (4) $357 \\div(1+75 \\%)$"} {"id": "13164", "image": [], "answer": "答案:180 分 $=3$ 小时 1 天 $=24$ 小时\n\n解: 设 “神舟十号”飞船一天能绕地球飞行 $x$ 圈。\n\n$3: 2=24: x \\quad x=16$", "solution": "null", "level": "六年级", "question": "我校在 “创建绿色循环经济示范单位” 活动中, 打算在生物园新挖一个直径是 6 米,深 12 分米的圆形水池。\n\n(1) 这个水池的占地面积是多少?\n\n(2)如果这个水池修好后,需要用水泥把池底和侧壁粉刷,粉刷的面积有多大?", "options": [], "subject": "度量几何学", "analysis": "答案:180 分 $=3$ 小时 1 天 $=24$ 小时\n\n解: 设 “神舟十号”飞船一天能绕地球飞行 $x$ 圈。\n\n$3: 2=24: x \\quad x=16$"} {"id": "13059", "image": [], "answer": "反", "solution": "null", "level": "六年级", "question": "平行四边形的面积一定, 它的底和高成 ( ) 比例。", "options": [], "subject": "度量几何学", "analysis": ""} {"id": "13061", "image": ["3010.jpg"], "answer": "〈 > > <", "solution": "null", "level": "六年级", "question": "在下面的 $\\mathrm{O}$ 里填入 “>”、“<”或 “=”。\n\n$$\n-", "options": [], "subject": "度量几何学", "analysis": ""} {"id": "13116", "image": [], "answer": "答案: 解: $12 \\div \\frac{1}{5000000}=60000000$ (厘米), 60000000 厘米 $=600$ 千米 $600 \\div\\left(90+90 \\times \\frac{2}{3}\\right)$\n$=600 \\div 150$\n\n$=4$ (小时)\n\n答: 两车 4 小时后相遇.", "solution": "null", "level": "六年级", "question": "在比例尺是 1: 5000000 的地图上,量得两地的距离是 12 厘米,甲、乙两辆汽车同时从两地相对开出, 已知甲车速度每小时 90 千米, 乙车的速度是甲车的 $\\frac{2}{3}$, 两车几小时后相遇?", "options": [], "subject": "度量几何学", "analysis": "答案: 解: $12 \\div \\frac{1}{5000000}=60000000$ (厘米), 60000000 厘米 $=600$ 千米 $600 \\div\\left(90+90 \\times \\frac{2}{3}\\right)$\n$=600 \\div 150$\n\n$=4$ (小时)\n\n答: 两车 4 小时后相遇."} {"id": "13146", "image": [], "answer": "答案:12 分米 $=", "solution": "null", "level": "六年级", "question": "我校在 “创建绿色循环经济示范单位” 活动中, 打算在生物园新挖一个直径是 6 米, 深 12 分米的圆形水池。如果这个水池修好后, 需要用水泥把池底和侧壁粉刷, 粉刷的面积有多大?", "options": [], "subject": "度量几何学", "analysis": "答案:12 分米 $="} {"id": "12494", "image": ["2992.jpg", "2993.jpg"], "answer": "答案: $9.42 ; 25.12 ; 28.26 ; 0.04$", "solution": "null", "level": "六年级", "question": "直接写出得数。\n\n\\begin{tabular}{llll}\n$3 \\pi=$ & $8 \\pi=$ & $9 \\pi=$ & $", "options": [], "subject": "度量几何学", "analysis": "答案: $9.42 ; 25.12 ; 28.26 ; 0.04$"} {"id": "26970", "image": [], "answer": "$5-2=3$ (米)\n\n答: 露出水面的部分记为 +3 米。", "solution": "null", "level": "六年级", "question": " (5 分 )一根 5 米长的竹竿,插在水中的部分有 2 米,记作 -2 米. 那么露出水面的部分如何记?", "options": [], "subject": "度量几何学", "analysis": "$5-2=3$ (米)\n\n答: 露出水面的部分记为 +3 米。"} {"id": "27121", "image": [], "answer": "(1)解: +0.05 米表示水面高于标准水位 0.05 米,-0.8 米表示水面低于标准水位 0.8 米。\n\n(2) 水位高于标准水位 0.45 米,记作: +0.45 米。", "solution": "null", "level": "六年级", "question": " (8 分)某游泳池的标准水位记为 0 米, 如果用正数表示水面高于标准水位的高度, 那么:\n\n(1) (4 分) +0.05 米和 -0.8 米各表示什么?\n\n(2)(4 分)水位高于标准水位 0.45 米怎样表示?", "options": [], "subject": "度量几何学", "analysis": "(1)解: +0.05 米表示水面高于标准水位 0.05 米,-0.8 米表示水面低于标准水位 0.8 米。\n\n(2) 水位高于标准水位 0.45 米,记作: +0.45 米。"} {"id": "27213", "image": ["13296.jpg"], "answer": "(1)解: $10 \\times 80=800$ (千米)\n\n(2) $600 \\div 80=7.5$ (时)", "solution": "null", "level": "六年级", "question": " (10 分)一辆汽车在公路上行驶,行驶的时间和路程如图。\n\n\n\n(1)(5 分)10 时行了多少千米?\n\n(2)(5 分)行驶 600 千米,需要几时?", "options": [], "subject": "度量几何学", "analysis": "(1)解: $10 \\times 80=800$ (千米)\n\n(2) $600 \\div 80=7.5$ (时)"} {"id": "26138", "image": [], "answer": "辣椒: $360 \\times \\frac{3}{5}=216\\left(\\mathrm{~m}^{2}\\right)$\n 西红柿:(30-216) $\\times \\frac{5}{8}=9(\\mathrm{~m})$\n\n黄瓜:(360-216) $\\times \\frac{3}{8}=54(\\mathrm{~m})$", "solution": "null", "level": "六年级", "question": " 有一个 $360 \\mathrm{~m}^{2}$ 的蔬菜大棚, 准备用 $\\frac{3}{5}$ 种辣椒, 剩下的按 $5: 3$ 的面积比种西红柿和黄瓜。三种蔬菜各种多少平方米?(6 分)", "options": [], "subject": "度量几何学", "analysis": "辣椒: $360 \\times \\frac{3}{5}=216\\left(\\mathrm{~m}^{2}\\right)$\n 西红柿:(30-216) $\\times \\frac{5}{8}=9(\\mathrm{~m})$\n\n黄瓜:(360-216) $\\times \\frac{3}{8}=54(\\mathrm{~m})$"} {"id": "26155", "image": ["13085.jpg"], "answer": "$20 .56 \\mathrm{~cm} \\quad 82.26 \\mathrm{~cm}$", "solution": "null", "level": "六年级", "question": " 求下面图形的周长。(8 分)\n ", "options": [], "subject": "度量几何学", "analysis": "$20 .56 \\mathrm{~cm} \\quad 82.26 \\mathrm{~cm}$"} {"id": "26156", "image": ["13086.jpg"], "answer": "$62 .8 \\mathrm{~cm}^{2} \\quad 7.74 \\mathrm{dm}$", "solution": "null", "level": "六年级", "question": " 求出下面各图中阴影部分的面积。(8 分)\n ", "options": [], "subject": "度量几何学", "analysis": "$62 .8 \\mathrm{~cm}^{2} \\quad 7.74 \\mathrm{dm}$"} {"id": "26158", "image": ["13087.jpg"], "answer": "$4 \\times 2 \\times $$3.14=25.12(\\mathrm{~cm})$\n $25.12 \\times 24=602.88(\\mathrm{~cm})$", "solution": "null", "level": "六年级", "question": " 一只闹钟, 它的分针长 $4 \\mathrm{~cm}$, 这根分针的尖端转动一昼夜所走的路程是多少厘米? (5 分)", "options": [], "subject": "度量几何学", "analysis": "$4 \\times 2 \\times $$3.14=25.12(\\mathrm{~cm})$\n $25.12 \\times 24=602.88(\\mathrm{~cm})$"} {"id": "26159", "image": [], "answer": "$0 .35 \\times 2 \\times 3.14=2.198(\\mathrm{~m})$\n\n $2.198 \\times 100 \\times 60=13188(\\mathrm{~m})$", "solution": "null", "level": "六年级", "question": " 一辆自行车的车轮半径是 $35 \\mathrm{~cm}$, 如果车轮平均每分钟转 100 圈, 这辆自行车每小时能行驶多少米?(5 分)", "options": [], "subject": "度量几何学", "analysis": "$0 .35 \\times 2 \\times 3.14=2.198(\\mathrm{~m})$\n\n $2.198 \\times 100 \\times 60=13188(\\mathrm{~m})$"} {"id": "26160", "image": [], "answer": "$10 \\times 8-\\left(\\frac{10}{2}\\right){ }^{2} \\times 3.14 \\div 2=40.75\\left(\\mathrm{dm}^{2}\\right)$", "solution": "null", "level": "六年级", "question": " 从一个长 $10 \\mathrm{dm}$ 、宽 $8 \\mathrm{dm}$ 的长方形本板上锯下一个最大的半圆, 剩下的木板是多少平方分米?(6 分)", "options": [], "subject": "度量几何学", "analysis": "$10 \\times 8-\\left(\\frac{10}{2}\\right){ }^{2} \\times 3.14 \\div 2=40.75\\left(\\mathrm{dm}^{2}\\right)$"} {"id": "13165", "image": [], "answer": "答案:\frac{1}{3} \times 3.14 \times(6 \\div 2)^{2} \times 2.5=23.55$ (立方米) $23.55 \times 1.8 \\div 8 \\approx 6$ (次)", "solution": "null", "level": "六年级", "question": "一个圆锥形沙堆, 底面直径是 6 米, 高是", "options": [], "subject": "立体几何学", "analysis": ""} {"id": "13049", "image": ["3069.jpg", "3070.jpg"], "answer": "$3.14 \times(12 \\div 2)^{2} \times 14 \times \frac{1}{3}$=3.14 \times 36 \times 14 \times \frac{1}{3}$$=527.52\\left(\\mathrm{~cm}^{3}\right)$", "solution": "null", "level": "六年级", "question": "求下图圆锥的体积。\n\n24.如图是一种钢制的配件(图中数据单位: cm)请计算它的表面积和体积.\n\n", "options": [], "subject": "立体几何学", "analysis": ""} {"id": "13050", "image": [], "answer": "解: $20 \\div 2=10$ (厘米)$10 \times 10 \times 3.14 \times 30=9420$ (立方厘米)$25 \\div 2=12.5$ (厘米)$12.5 \times 12.5 \times 3.14 \times 20=9812.5$ (立方厘米)$9420<9812.5$答: 这个圆柱的体积是 9812.5 立方厘米。", "solution": "null", "level": "六年级", "question": "将一个长 30 厘米, 宽 25 厘米, 高 20 厘米的长方体木块削成一个最大的圆柱, 这个圆柱的体积是多少?", "options": [], "subject": "立体几何学", "analysis": ""} {"id": "13051", "image": [], "answer": "; 84.78$\n\n\n\n22.答案:解: $", "solution": "null", "level": "六年级", "question": "已知一根长 3 米的圆柱形木料, 将它截成 4 段, 其表面积增加", "options": [], "subject": "立体几何学", "analysis": "; 84.78$\n\n\n\n22.答案:解: $"} {"id": "13052", "image": [], "answer": "解:底面积:3.14× ( $8 \\div 2)^{2}$ $=3.14 \times 16$$=50.24$ (平方分米)侧面积: $3.14 \times 8 \times 12$$=3.14 \times 96$$=301.44$ (平方分米) 需要铁皮重量:$(50.24+301.44) \times 50 \times 6.5$$=351.68 \times 325$$=114296$ (千克)答: 做好这些铁桶应该用 114296 千克的铁皮。", "solution": "null", "level": "六年级", "question": "李师傅做了 50 个直经是 $8 \\mathrm{dm}$ 高是 $12 \\mathrm{dm}$ 的圆柱形铁桶, 每 $1 \\mathrm{dm}^{2}$ 的铁桶重 $", "options": [], "subject": "立体几何学", "analysis": ""} {"id": "13054", "image": [], "answer": "(1) 解: $18.84 \\div 3.14 \\div 2$$=6 \\div 2$$=3$ (分米)$18.84 \times 5+3.14 \times 3^{2}$$=18.84 \times 5+3.14 \times 9$$=94.2+28.26$$=122.46$ (平方分米)答: 做这个水桶至少用去木板 122.46 平方分米。(2) 解: $3.14 \times 3^{2} \times 5$$=3.14 \times 9 \times 5$$=28.26 \times 5$$=141.3$ (立方分米)$=141.3$ (升)141.3 升 $>100$ 升, 能装下。答: 这个水桶能盛 100 升水。", "solution": "null", "level": "六年级", "question": "一个圆柱形水桶, 底面半径是 10 厘米, 高是 20 厘米。\n\n(1)给这个水桶加个盖,水桶盖的面积至少是多少平方厘米?\n\n(2)给这个水桶(不包括盖)外面刷上油漆,刷油漆的面积是多少平方厘米?\n\n(3)这个水桶能装多少升水?(水桶的厚度忽略不计)", "options": [], "subject": "立体几何学", "analysis": ""} {"id": "13148", "image": [], "answer": "$3.14 \times\\left(\frac{2}{2}\right)^{2} \times 5 \\div(8 \times 3.14)=0.625$ (米)", "solution": "null", "level": "六年级", "question": "学校把一个堆成底面直径是 2 米, 高 5 米的圆锥形沙子, 填铺到一个长 8 米, 宽", "options": [], "subject": "立体几何学", "analysis": ""} {"id": "27028", "image": [], "answer": "圆雉和圆柱的面积共为: $3.14 \\times(18.84 \\div 3.14 \\div 2)^{2}=28.26$ (平方米),所以圆锥和圆柱的总体积(即粮仓的总容积)为: $\\frac{1}{3} \\times 28.26 \\times 0.6+28.26 \\times 2=62.172$ (立方米),\n\n稻谷的质量为: $600 \\times 62.172=37303.2$ (千克)。", "solution": "null", "level": "六年级", "question": " (5分)一个粮仓装满稻谷后上半部分是圆锥形,下半部分是圆柱形。粮仓的底面周长是 18.84 米,圆柱高 2 米,圆锥高 0.6 米。如果每立方米稻谷重 600 千克,那么这个粮仓装有多少千克稻谷?", "options": [], "subject": "立体几何学", "analysis": "圆雉和圆柱的面积共为: $3.14 \\times(18.84 \\div 3.14 \\div 2)^{2}=28.26$ (平方米),所以圆锥和圆柱的总体积(即粮仓的总容积)为: $\\frac{1}{3} \\times 28.26 \\times 0.6+28.26 \\times 2=62.172$ (立方米),\n\n稻谷的质量为: $600 \\times 62.172=37303.2$ (千克)。"} {"id": "27030", "image": [], "answer": "$15 \\times 15 \\times 15 \\div\\left(3.14 \\mathrm{X} 20^{2}\\right) \\approx 2.7$ (厘米)", "solution": "null", "level": "六年级", "question": " (5 分)将一个棱长为 15 厘米的正方体容器装满水, 倒入一个底面半径是 20 厘米的圆柱体容器中, 这时圆柱体容器的水深多少厘米? (得数保留一位小数)", "options": [], "subject": "立体几何学", "analysis": "$15 \\times 15 \\times 15 \\div\\left(3.14 \\mathrm{X} 20^{2}\\right) \\approx 2.7$ (厘米)"} {"id": "27031", "image": ["13238.jpg"], "answer": "$3.14 \\times 10^{2} \\times 20 \\div 2+20 \\times 20 \\times 20=11140$ (立方厘米) $=11.41$ (立方分米)", "solution": "null", "level": "六年级", "question": " ( 5 分)一个工具箱的下半部分是棱长为 20 厘米的正方体,上半部分是圆柱体的一半。这个工具箱的体积是多少立方分米?\n\n", "options": [], "subject": "立体几何学", "analysis": "$3.14 \\times 10^{2} \\times 20 \\div 2+20 \\times 20 \\times 20=11140$ (立方厘米) $=11.41$ (立方分米)"} {"id": "27032", "image": [], "answer": "$3.14 \\div 2 \\div 3.14=0.5$ (分米)\n\n$3.14 \\times 0.52 \\times 0.8 \\div\\left(1-\\frac{2}{3}\\right)=1.884$ (立方分米 $)$", "solution": "null", "level": "六年级", "question": " ( 6 分 )一个底面周长是 3.14 分米的圆柱形玻璃杯内盛有一些水, 恰好占杯子容量的 $\\frac{2}{3}$ 。将两个同样大小的鸡蛋放人杯子中, 浸没在水里。这时水面上升 8 厘米, 刚好与杯子口平齐, 求玻璃杯的容积。", "options": [], "subject": "立体几何学", "analysis": "$3.14 \\div 2 \\div 3.14=0.5$ (分米)\n\n$3.14 \\times 0.52 \\times 0.8 \\div\\left(1-\\frac{2}{3}\\right)=1.884$ (立方分米 $)$"} {"id": "27033", "image": [], "answer": "$3.6 \\times \\frac{2}{3}=2.4$ (立方米)\n\n$4 \\times 2=8$ (平方米)", "solution": "null", "level": "六年级", "question": " (6 分)有一个圆雉体沙堆, 底面积是 3.6 平方米, 高 2 米。将这些沙铺在一个长 4 米, 宽 2 米的长方体沙坑里, 能铺多厚?", "options": [], "subject": "立体几何学", "analysis": "$3.6 \\times \\frac{2}{3}=2.4$ (立方米)\n\n$4 \\times 2=8$ (平方米)"} {"id": "27034", "image": [], "answer": "$12.56 \\div 2 \\div 3.14=2(\\mathrm{dm})$\n\n$2 \\mathrm{~cm}=0.2 \\mathrm{dm}$", "solution": "null", "level": "六年级", "question": " ( 6 分 ) 把一个底面半径为 $1 \\mathrm{dm}$ 的圆雉形金属完全浸没在底面周长为 $12.56 \\mathrm{dm}$ 的装有水的圆柱玻璃杯中, 杯子的水面比原来上升了 $2 \\mathrm{~cm}$ (水没有溢出), 求圆雉的高。", "options": [], "subject": "立体几何学", "analysis": "$12.56 \\div 2 \\div 3.14=2(\\mathrm{dm})$\n\n$2 \\mathrm{~cm}=0.2 \\mathrm{dm}$"} {"id": "27061", "image": [], "answer": "先求长、宽、高的和: $120 \\div 4=30(\\mathrm{~cm})$, 再按比分配, 长是 $15 \\mathrm{~cm}$, 宽 $10 \\mathrm{~cm}$, 高 $5 \\mathrm{~cm}$.", "solution": "null", "level": "六年级", "question": " (7 分)工人王师傅用 $120 \\mathrm{~cm}$ 的细铁丝焊接成一个长方体, 长、宽、高的比是 $3: 2: 1$. 这个长方体的长、宽、高分别是多少?", "options": [], "subject": "立体几何学", "analysis": "先求长、宽、高的和: $120 \\div 4=30(\\mathrm{~cm})$, 再按比分配, 长是 $15 \\mathrm{~cm}$, 宽 $10 \\mathrm{~cm}$, 高 $5 \\mathrm{~cm}$."} {"id": "13166", "image": ["3072.jpg"], "answer": "D", "solution": "null", "level": "六年级", "question": "怎样计算简单就怎样算: (12 分)\n$\\left(\\frac{4}{7}-\\frac{3}{14}\\right) \\div \\frac{1}{28}$\n$22.下图是某市世纪联华超市附近的平面图, 仔细观察, 填一填, 画一画。 (6 分)\n\n(1) 若从世纪联华往北走 50 米, 记世纪联华往南走 150 米, 记作\n\n(2) 幼儿园在世纪联华南偏东\n\n\n处, 请在图中标出幼儿园的位置。\n\n(3) 金箔路与上元大街平行, 并垂直于竹山路, 距上元大街 500 米, 请在图中画出金箔路的位置。", "options": [], "subject": "解析几何", "analysis": "D"} {"id": "26115", "image": [], "answer": "$80 \\div 2=40$ (cm)\n\n长: $40 \\times \\frac{5}{8}=25(\\mathrm{~cm})$\n\n宽: $40 \\times \\frac{3}{8}=15(\\mathrm{~cm})$", "solution": "null", "level": "六年级", "question": " 用 $80 \\mathrm{~m}$ 长的铁丝做一个长方形的框架, 宽是长的 $\\frac{4}{5}$ 。这个长方形框架的长、宽分别是多少?(6 分)", "options": [], "subject": "解析几何", "analysis": "$80 \\div 2=40$ (cm)\n\n长: $40 \\times \\frac{5}{8}=25(\\mathrm{~cm})$\n\n宽: $40 \\times \\frac{3}{8}=15(\\mathrm{~cm})$"} {"id": "13073", "image": [], "answer": "答案:解: $25000+25000 \\times", "solution": "null", "level": "六年级", "question": "淘气的妈妈买了 25000 元的国家建设债券, 期限三年, 当时的年利率为 $", "options": [], "subject": "计数", "analysis": "答案:解: $25000+25000 \\times"} {"id": "13094", "image": [], "answer": "答案:解:\n\n| 星期一 | +", "solution": "null", "level": "六年级", "question": "某商场一星期的经营情况如下:\n\n星期一: 盈利", "options": [], "subject": "计数", "analysis": "答案:解:\n\n| 星期一 | +"} {"id": "13096", "image": [], "answer": "(1) 解: $8>-2$, 所以南京的最高温度高。答: 北京与南京, 南京的最高温度高。 (2) 解: 因为 $9>9$, 所以 $9^{\\circ} \\mathrm{C}>8^{\\circ} \\mathrm{C}$; 因为 $2<8$, 所以 $-2^{\\circ} \\mathrm{C}>-8^{\\circ} \\mathrm{C}$; 又因为正数一定大于负数, 所以 $9^{\\circ} \\mathrm{C}>8^{\\circ} \\mathrm{C}>-2^{\\circ} \\mathrm{C}>-8^{\\circ} \\mathrm{C}$ 。答: 这 4 座城市的最高温度从高到低排列是: $9^{\\circ} \\mathrm{C}>8^{\\circ} \\mathrm{C}>-2^{\\circ} \\mathrm{C}>-8^{\\circ} \\mathrm{C}$ 。(3) 解: 由第 (2) 问可知: 这 4 座城市的最高温度从高到低排列是: $9^{\\circ} \\mathrm{C}>8^{\\circ} \\mathrm{C}>-2^{\\circ} \\mathrm{C}>-8^{\\circ} \\mathrm{C}$, 所以最高温度最低的城市是哈尔滨, 最高温度最高的城市是上海。又因为 $9-(-8)=9+8=17^{\\circ} \\mathrm{C}$, 所以它们的最高温度相差 $17^{\\circ} \\mathrm{C}$ 。答: 在这 4 座城市中, 最高温度最低的城市是哈尔滨, 最高温度最高的城市是上海, 它们的最高温度相差 $17^{\\circ} \\mathrm{C}$ 。", "solution": "null", "level": "六年级", "question": "看表回答问题。\n\n某天几座城市的最高温度统计表\n\n| 城市 | 哈尔滨北京 | 南京 | 上海 | |\n| :--- | :--- | :--- | :--- | :--- |\n| 最高温度 | $-8^{\\circ} \\mathrm{C}$ | $-2^{\\circ} \\mathrm{C}$ | $8^{\\circ} \\mathrm{C}$ | $9^{\\circ} \\mathrm{C}$ |\n\n(1) 北京与南京, 哪座城市的最高温度高?\n\n(2) 把这 4 座城市的最高温度从高到低排列出来。\n\n(3)在这 4 座城市中, 最高温度最低的城市是哪一座? 最高温度最高的城市是哪一座? 它们的最高温度相差多少 $?$", "options": [], "subject": "计数", "analysis": ""} {"id": "13112", "image": [], "answer": "答案:解: 设需要走 $x$ 时.$$\begin{aligned}& 6: x=4.5: 1.5 \text { 或 } \frac{6}{x}=\frac{4.5}{1.5} ; \\& x=2\\end{aligned}$$", "solution": "null", "level": "六年级", "question": "学校组织同学们进行耐力训练,", "options": [], "subject": "计数", "analysis": ""} {"id": "13128", "image": [], "answer": "答案: $13 \\div 3=4 \\cdots 1$\n\n$4+1=5$\n\n答:其中必有一次的点数不低于 5 .", "solution": "null", "level": "六年级", "question": "小王郑股子, 3 次的点数加起来是 13 , 其中必有一次的点数不低于多少?", "options": [], "subject": "计数", "analysis": "答案: $13 \\div 3=4 \\cdots 1$\n\n$4+1=5$\n\n答:其中必有一次的点数不低于 5 ."} {"id": "13149", "image": [], "answer": "$10 \\div 100=10 \\%, 1800 \times(1-10 \\%)=1620$ (棵)", "solution": "null", "level": "六年级", "question": "飞云雅琳农园,计划种植 1800 棵果树。先试种了 100 棵,其中没有成活的有 10 棵。试种时树苗的成活率是多少?根据此成活率估算这批树苗可以成活多少棵?", "options": [], "subject": "计数", "analysis": ""} {"id": "26968", "image": [], "answer": "(1) +1.5 万元表示盈利 1.5 万元.\n\n(2) -0.2 万元表示亏损 0.2 万元.\n\n(3) $+4.9>+3.2>+2.3>+1.5>-0.2>-1.2$", "solution": "null", "level": "六年级", "question": "(20 分)新华小吃店今年上半年营业利润情况如下表.\n\n| 月分 | - | 二 | 三 | 四 | 五 | 六 |\n| :---: | :---: | :---: | :---: | :---: | :---: | :---: |\n| 盈亏金额
(万元) | +1.5 | +2.3 | -0.2 | +3.2 | +4.9 | -1.2 |\n\n从上表中你知道了哪些情况?\n\n(1)你知道 “ +1.5 万元” 表示什么吗?\n\n(2)知道 “ -0.2 万元” 表示什么吗?\n\n(3)哪个月的营业情况最好?这个月的营业利润是多少元?\n\n(4)新华小吃店,在上半年里有几个月盈利?有几个月亏损?\n\n你知道吗, 在通常情况下,盈利用正数表示,亏损用负数表示。", "options": [], "subject": "计数", "analysis": "(1) +1.5 万元表示盈利 1.5 万元.\n\n(2) -0.2 万元表示亏损 0.2 万元.\n\n(3) $+4.9>+3.2>+2.3>+1.5>-0.2>-1.2$"} {"id": "27119", "image": ["13267.jpg"], "answer": "(1)解:变动情况如表所示。.\n\n| 站名 | 上车 | 下车 |\n| :--- | :--- | :--- |\n| 商品城 | +15 | 0 |\n| 广场 | +6 | -8 |\n| 电影院 | +9 | -7 |\n| 电视台 | +12 | -10 |\n\n(2) $15+6+9+12=42$ (人)\n\n答: 车上一共载过 42 位乘客。\n\n(3) $42 \\times 2=84$ (元)\n\n答: 车上共售票 84 元。", "solution": "null", "level": "六年级", "question": " (12 分\n\n\n商品城 $\\rightarrow$ 广场 $\\rightarrow$ 电影院 $\\rightarrow$ 电视台 $\\rightarrow$ 车站\n\n商品城上车 15 人。广场下车 8 人,上车 6 人。电影院下车 7 人,上车 9 人。电视台下车 10 人, 上车 12 人。\n\n(1)(4 分)在表格内用正、负数记录公交车的人数变动情况。\n\n| 站名 | 上车 | 下车 |\n| :--- | :--- | :--- |\n| 商品城 | +15 | 0 |\n| 广场 | | |\n| 电影院 | | |\n| 电视台 | | |\n\n(2)(4 分)从商品城到车站(终点站),车上一共载过多少乘客?\n\n(3)(4 分)若每人票价按 2 元计算,则车上共售票多少元?", "options": [], "subject": "计数", "analysis": "(1)解:变动情况如表所示。.\n\n| 站名 | 上车 | 下车 |\n| :--- | :--- | :--- |\n| 商品城 | +15 | 0 |\n| 广场 | +6 | -8 |\n| 电影院 | +9 | -7 |\n| 电视台 | +12 | -10 |\n\n(2) $15+6+9+12=42$ (人)\n\n答: 车上一共载过 42 位乘客。\n\n(3) $42 \\times 2=84$ (元)\n\n答: 车上共售票 84 元。"} {"id": "27120", "image": [], "answer": "(1)解:杨阳、王昆、刘明和郭成达到了标准。\n\n(2) $9-3+9-1+9+9+2+9-2+9-2+9+9+3=69$ (个)\n\n答: 这 8 名男生共做了 69 个引体向上。", "solution": "null", "level": "六年级", "question": " (8 分)体育课上, 8 名男生进行引体向上测试, 以 9 个为标准, 超过的次数用正数表示,不足的次数用负数表示, 记录如下:\n\n| 学生 | 张亮 | 李忠 | 杨阳 | 王昆 | 陈平 | 孙刚 | 刘明 | 郭成 |\n| :--- | :--- | :--- | :--- | :--- | :--- | :--- | :--- | :--- |\n| 记录数 | -3 | -1 | 0 | 2 | -2 | -2 | 0 | 3 |\n\n(1)(4 分)从上表中你发现: 哪几名男生达到标准?\n\n(2)(4 分)这 8 名男生共做了多少个引体向上?", "options": [], "subject": "计数", "analysis": "(1)解:杨阳、王昆、刘明和郭成达到了标准。\n\n(2) $9-3+9-1+9+9+2+9-2+9-2+9+9+3=69$ (个)\n\n答: 这 8 名男生共做了 69 个引体向上。"} {"id": "26157", "image": [], "answer": "$600 \\div(2 \\times 3.14 \\times 5)=19$ (个)", "solution": "null", "level": "六年级", "question": " 一根 $60 \\mathrm{dm}$ 长的铁丝, 将它做成半径为 $5 \\mathrm{~cm}$ 的铁圈(接头处忽略不计), 最多可以做多少个?(5 分)", "options": [], "subject": "计数", "analysis": "$600 \\div(2 \\times 3.14 \\times 5)=19$ (个)"} {"id": "26197", "image": [], "answer": "$12 \\div 30=0.4=40 \\%$", "solution": "null", "level": "六年级", "question": " 古诗《春》是这样的:春水春池满, 春进春草生。春树绽春芯, 春雨伴春风。春人饮春酒, 春鸟弄春色。作者在这首诗中放情吟春, 神采飞扬。全诗中 “春” 字出现得特别多。请你算一算, “春” 的字数占全诗总字数的百分之几?(5 分)", "options": [], "subject": "计数", "analysis": "$12 \\div 30=0.4=40 \\%$"} {"id": "13111", "image": ["2752.jpg"], "answer": "答案:(1) $10.1-4 x=2.9$解: $10.1-2.9=4 \\mathrm{x}$$$\begin{gathered}4 x=7.2 \\x=7.2 \\div 4 \\x=1.8\\end{gathered}$$(2) 7.5: $x=24: 12$解: $24 \\mathrm{x}=12 \times 7.5$$$\begin{aligned}24 x & =90 \\x & =3.7\\end{aligned}$$", "solution": "null", "level": "六年级", "question": "解比例或解方程。\n(1) $", "options": [], "subject": "画法几何学", "analysis": ""} {"id": "12556", "image": ["2681.jpg", "2680.jpg"], "answer": "作图如下:", "solution": "null", "level": "六年级", "question": "根据描述,在平面图上标出各场所位置.\n\n(1) 图书馆在校门的北偏西 $35^{\\circ}$ 方向 $200 \\mathrm{~m}$.\n\n(2) 体育馆在校门的北偏东 $45^{\\circ}$ 方向 $150 \\mathrm{~m}$.\n\n(3)餐厅在图书馆的正西方向 $50 \\mathrm{~m}$.\n\n(4) 实验楼在体育馆的北偏西 $60^{\\circ}$ 方向 $100 \\mathrm{~m}$.\n\n", "options": [], "subject": "画法几何学", "analysis": ""} {"id": "13126", "image": [], "answer": "答案:9 个", "solution": "null", "level": "六年级", "question": "盒子里有红球、黄球各 8 个, 最少摸出几个, 才能保证有两种不同颜色的球?", "options": [], "subject": "组合数学", "analysis": "答案:9 个"} {"id": "13127", "image": [], "answer": "答案: $5 \\times 4 \\div 2=10$ (种)\n\n$10 \\times 3+1=31($ 人)\n\n答: 至少有 31 人去取才能保证有 4 人取的书完全一样.", "solution": "null", "level": "六年级", "question": "有 A、B、C、D、E 五种课外读物各若干本,如果每个人可以在 5 种读物\n中任取 2 种各 1 本. 至少有多少人去取才能保证有 4 人取的书完全一样?", "options": [], "subject": "组合数学", "analysis": "答案: $5 \\times 4 \\div 2=10$ (种)\n\n$10 \\times 3+1=31($ 人)\n\n答: 至少有 31 人去取才能保证有 4 人取的书完全一样."} {"id": "13130", "image": [], "answer": "答案: $11 \\div 3=3$ (封) $\\cdots 2$ (封)\n\n$3+1=4$ (封)\n\n答:至少有 4 封信投入同一个信箱里; 因为平均每个邮箱放 3 封, 还余 2 封, 这 2 封无论怎么放, 都至少有 4 封信投入同一个信箱里.", "solution": "null", "level": "六年级", "question": "11 封信投入 3 个邮箱里, 至少有 4 封信投入同一个信箱里, 为什么?(用自己喜欢的方式说明)", "options": [], "subject": "组合数学", "analysis": "答案: $11 \\div 3=3$ (封) $\\cdots 2$ (封)\n\n$3+1=4$ (封)\n\n答:至少有 4 封信投入同一个信箱里; 因为平均每个邮箱放 3 封, 还余 2 封, 这 2 封无论怎么放, 都至少有 4 封信投入同一个信箱里."} {"id": "27065", "image": [], "answer": "解:设乙书架有 $\\mathrm{x}$ 本,那么甲 $108-\\mathrm{x}$ 本,丙 $140-\\mathrm{x}$ 本, $(108-\\mathrm{x}) :(140-\\mathrm{x})=3 : 7$\n$(140-108) \\div(7-3) \\times 3=24$ (本)\n甲书架有书的本数\n\n$108-24=84($ 本 $)$ $\\qquad$乙书架有书的本数\n\n或\n\n$$\n(140-108) \\div(7-3) \\times 7=56 \\text { (本) }\n$$\n\n.丙书架有书的本数\n$140-56=84$ (本) $\\qquad$乙书架有书的本数", "solution": "null", "level": "六年级", "question": " (6 分)甲、乙两书架共有书 108 本, 乙、丙两书架共有书 140 本, 甲、丙书架上书的本数的比是 3:7, 乙书架有书多少本?", "options": [], "subject": "组合数学", "analysis": "解:设乙书架有 $\\mathrm{x}$ 本,那么甲 $108-\\mathrm{x}$ 本,丙 $140-\\mathrm{x}$ 本, $(108-\\mathrm{x}) :(140-\\mathrm{x})=3 : 7$\n$(140-108) \\div(7-3) \\times 3=24$ (本)\n甲书架有书的本数\n\n$108-24=84($ 本 $)$ $\\qquad$乙书架有书的本数\n\n或\n\n$$\n(140-108) \\div(7-3) \\times 7=56 \\text { (本) }\n$$\n\n.丙书架有书的本数\n$140-56=84$ (本) $\\qquad$乙书架有书的本数"} {"id": "27088", "image": [], "answer": "$4+1=5$ (次)\n\n答: 他至少摸 5 次, 才能保证有 2 次摸出的球相同。", "solution": "null", "level": "六年级", "question": " (6 分)1 只口袋里装有 10 个黄球和 10 个红球(这些球除颜色不同外其他都相同)。小明 1 次从袋子中摸出 3 个球。他至少摸几次,才能保证有 2 次摸出的球相同?", "options": [], "subject": "组合数学", "analysis": "$4+1=5$ (次)\n\n答: 他至少摸 5 次, 才能保证有 2 次摸出的球相同。"} {"id": "27089", "image": [], "answer": "答: 因为如果前 6 天各穿一套衣服, 那么第 7 天无论穿哪套衣服, 都至少有两天拍的照片中她穿的是同一套衣服。", "solution": "null", "level": "六年级", "question": "(6 分)张阿姨 “五一”放假的时候参加 “泰国七日游”(七日包括出\n发日和返回日)。出门前, 包括出发当天身上穿的衣服在内,她一共整理了 6 套衣服(旅游期间,张阿姨只按搭配好的套装穿,每天换一套,不混搭,不另购)。如果张阿姨每天都拍了照,七天所拍的照片中,至少有两天拍的照片中她穿的是同一套衣服。请说明理由。", "options": [], "subject": "组合数学", "analysis": "答: 因为如果前 6 天各穿一套衣服, 那么第 7 天无论穿哪套衣服, 都至少有两天拍的照片中她穿的是同一套衣服。"} {"id": "12495", "image": ["2994.jpg"], "answer": "答案: 13 步", "solution": "null", "level": "六年级", "question": "一个圆形广场,小莉从一边沿直径走到另一边走了 4 步,如果每步长约 $55 \\mathrm{~cm}$,那么着广场的边走一圈,大约需要走多少步? (得数保留整数)", "options": [], "subject": "组合几何学", "analysis": "答案: 13 步"} {"id": "12559", "image": ["2685.jpg", "2686.jpg"], "answer": "答案:明明一家从家出发向正西行驶,经过邮局、医院、少年宫、火车站,然后向西偏南 $20^{\\circ}$ 的方向行驶,经过游泳馆、商店,再向东偏南 $20^{\\circ}$ 的方向行驶,经过动物园,到达度假村。", "solution": "null", "level": "六年级", "question": "下面是周日明明一家开车去度假村的行驶线路图。请根据线路图描述明明一家所行驶的路线。\n\n\\begin{tabular}{|l|l|l|l|}\\hline & 方 向 & 路程 & 时间 \\\\hline 家 $\rightarrow$ 公园 & & & 12分 \\\\hline 公园图书馆 & & & 8分 \\\\hline 全 程 & & & \\\\hline\\end{tabular}(2)小刚走完全程的平均速度是多少?", "options": [], "subject": "图论", "analysis": "答案:明明一家从家出发向正西行驶,经过邮局、医院、少年宫、火车站,然后向西偏南 $20^{\\circ}$ 的方向行驶,经过游泳馆、商店,再向东偏南 $20^{\\circ}$ 的方向行驶,经过动物园,到达度假村。"} {"id": "26372", "image": [], "answer": "10 键;-1 表示负一层, -2 表示负二层。", "solution": "null", "level": "六年级", "question": " 如果要到 10 层按哪个键?\n\n这里 $-1 、-2$ 表示什么意思?", "options": [], "subject": "逻辑题", "analysis": "10 键;-1 表示负一层, -2 表示负二层。"} {"id": "26866", "image": ["13207.jpg"], "answer": "见详解", "solution": "null", "level": "六年级", "question": " (1) 画出图(1)以 $O$ 为中心顺时针旋转 $90^{\\circ}$ 后的图形。\n\n(2) 画出图(2)以 $a b$ 为对称轴的另一半图形。\n\n(3) 画出图(3)按 $2: 1$ 放大后的图形。\n\n", "options": [], "subject": "变换几何", "analysis": "见详解"} {"id": "5698", "image": ["7147.jpg"], "answer": "(1) 解:张山的得分为: $8,9,9,10$,\n\n最中间的数是 9 和 9\n\n$\\therefore$ 张山得分对的中位数为: $(9+9) \\div 2=9$;\n\n方差为: $\\frac{(8-9)^{2}+2(9-9)^{2}+(10-9)^{2}}{4}=0.5$\n\n李仕的得分:7, $9,10,10$,\n\n平均成绩为:\n\n$$\n\\frac{7+9+10+10}{4}=9\n$$\n\n10 出现了 2 次, 是出现次数最多的数,\n\n$\\therefore$ 李仕得分的众数为 10 ;\n\n| 姓名 | 平均成绩(分) | 中位数(分) | 众数(分) | 方差(分 $^{2}$ ) |\n| :---: | :---: | :---: | :---: | :---: |\n| 张山 | 9 | 9 | 9 | 0.5 |\n| 李仕 | 9 | 9.5 | 10 | 1.5 |\n\n(2) 从中位数角度考虑, 李仕大于张山, 说明李仕高分项目多;从反差角度考虑, 李仕大于张山, 说明张山各项成绩均衡。\n\n(3) 张山的综合得分为: $\\frac{8 \\times 4+9+2 \\times 9+3 \\times 10}{10}=8.9$ 分;\n李仕的综合得分为: $\\frac{7 \\times 4+9+2 \\times 10+3 \\times 10}{10}=8.7$\n\n$8.9>8.7$\n\n$\\therefore$ 张山的综合得分更高.", "solution": "null", "level": "八年级", "question": "(15 分 ) 入为响应习近平提出的“绿水青山就是金山银山”的重要思想某校举办了“绿水青山, 生态文明\"知识竞赛(竞每一项的满分 10 分, 学生得分均为整数).在这次竞赛中张山与李仕两位同学表现优秀,他们的四项成绩分布的条形统计图如图所示根据上图结果解答下列问题。\n\n\n\n(1) 补充完成下表\n\n| 姓名 | 平均成绩(分) | 中位数(分) | 众数(分) | 方差(分 ${ }^{2}$ ) |\n| :---: | :---: | :---: | :---: | :---: |\n| 张山 | 9 | | 9 | |\n| 李仕 | | 9.5 | | 1.5 |\n\n(2) 根据(1)题数据, 分别从中位数、方差两个角度比较说明两位同学的各自优势?\n\n(3)若实践操作、环保论文、现场抢答、笔试得分技 $4: 1: 2: 3$ 的比例折合成综合得分, 请通过计算说明哪位同学的综合得分更高。", "options": [], "subject": "计数", "analysis": "(1) 解:张山的得分为: $8,9,9,10$,\n\n最中间的数是 9 和 9\n\n$\\therefore$ 张山得分对的中位数为: $(9+9) \\div 2=9$;\n\n方差为: $\\frac{(8-9)^{2}+2(9-9)^{2}+(10-9)^{2}}{4}=0.5$\n\n李仕的得分:7, $9,10,10$,\n\n平均成绩为:\n\n$$\n\\frac{7+9+10+10}{4}=9\n$$\n\n10 出现了 2 次, 是出现次数最多的数,\n\n$\\therefore$ 李仕得分的众数为 10 ;\n\n| 姓名 | 平均成绩(分) | 中位数(分) | 众数(分) | 方差(分 $^{2}$ ) |\n| :---: | :---: | :---: | :---: | :---: |\n| 张山 | 9 | 9 | 9 | 0.5 |\n| 李仕 | 9 | 9.5 | 10 | 1.5 |\n\n(2) 从中位数角度考虑, 李仕大于张山, 说明李仕高分项目多;从反差角度考虑, 李仕大于张山, 说明张山各项成绩均衡。\n\n(3) 张山的综合得分为: $\\frac{8 \\times 4+9+2 \\times 9+3 \\times 10}{10}=8.9$ 分;\n李仕的综合得分为: $\\frac{7 \\times 4+9+2 \\times 10+3 \\times 10}{10}=8.7$\n\n$8.9>8.7$\n\n$\\therefore$ 张山的综合得分更高."} {"id": "5699", "image": [], "answer": " 解:由中位数的定义可知,\n\n这组数据从大到小排列为: $3.8,4.1,4.6,4.8,5.2,6$,\n\n$\\therefore$ 其中位数是(4.6+4.8) $\\div 2=4.7 ($ 元) .\n\n答: 中位数是 4.7 元.", "solution": "null", "level": "八年级", "question": "(6 分) 某学校积极响应上级的号召, 举行了“决不让一个学生因贫困而失学”的捐资助学活动, 其中 6 个班同学的捐款平均数如下表:\n\n| 班级 | 一班 | 二班 | 三班四班五班六班 | | | |\n| :--- | :--- | :--- | :--- | :--- | :--- | :--- |\n| 捐款平均数(元) | 6 | 4.6 | 4.1 | 3.8 | 4.8 | 5.2 |\n\n则这组数据的中位数是多少元?", "options": [], "subject": "计数", "analysis": " 解:由中位数的定义可知,\n\n这组数据从大到小排列为: $3.8,4.1,4.6,4.8,5.2,6$,\n\n$\\therefore$ 其中位数是(4.6+4.8) $\\div 2=4.7 ($ 元) .\n\n答: 中位数是 4.7 元."} {"id": "5700", "image": ["7148.jpg", "7149.jpg", "7150.jpg", "7151.jpg", "7151.jpg"], "answer": "(1)解:依题可得:第 7 天, 这一路口的行人交通违章次数是 8 次.这 20 天中, 行人交通违章 6 次的有 5 天.\n\n(2) 解: 补全的频数直方图如图所示:\n\n某路口 20 天内行人交通违童\n\n次敏的频数直方国\n\n\n\n(3) 解: 第一次调查, 平均每天行人的交通违章次数为:\n\n$5 \\times 3+6 \\times 5+7 \\times 4+8 \\times 5+9 \\times 3$ $=7$ (次) .\n\n$\\because 7-4=3$ (次)\n\n$\\therefore$ 通过宣传教育后, 这一路口平均每天还出现 3 次行人的交通违章.", "solution": "null", "level": "八年级", "question": "(15 分 ) 为积极创建全国文明城市, 某市对某路口的行人交通违章情况进行了 20 天的调查, 将所得数据绘制成如下统计图(图 2 不完整):\n\n\n\n莱路口 20 天内行人交通违费次数的境计覑造事效数\n\n\n\nr 1莱路口 20 天内行人交通违\n\n次数的㭥数直方目\n\n\n\n图 2\n\n请根据所给信息,解答下列问题:\n\n(1)第 7 天, 这一路口的行人交通违章次数是多少次?这 20 天中,行人交通违章 6 次的有多少天?\n\n(2) 请把图 2 中的频数直方图补充完整;\n\n(3)通过宣传教育后, 行人的交通违章次数明显减少. 经对这一路口的再次调查发现, 平均每天的行人交通违章次数比第一次调查时减少了 4 次, 求通过宣传教育后, 这一路口平均每天还出现多少次行人的交通违章?", "options": [], "subject": "计数", "analysis": "(1)解:依题可得:第 7 天, 这一路口的行人交通违章次数是 8 次.这 20 天中, 行人交通违章 6 次的有 5 天.\n\n(2) 解: 补全的频数直方图如图所示:\n\n某路口 20 天内行人交通违童\n\n次敏的频数直方国\n\n\n\n(3) 解: 第一次调查, 平均每天行人的交通违章次数为:\n\n$5 \\times 3+6 \\times 5+7 \\times 4+8 \\times 5+9 \\times 3$ $=7$ (次) .\n\n$\\because 7-4=3$ (次)\n\n$\\therefore$ 通过宣传教育后, 这一路口平均每天还出现 3 次行人的交通违章."} {"id": "5701", "image": [], "answer": "解: 平均数 $=\\frac{1}{10}(2+3+4+6+7+7+7+8+9+10)=6.3$. 在这一组数据中 7 是出现次数最多的, 故众数是 7 ;\n\n将这组数据从小到大的顺序排列(2,3,4,6,7,7,7,8,9,10),处于中间位置的那 2 个数是 7、 7 ,\n\n那么由中位数的定义可知, 这组数据的中位数是 $\\frac{\\frac{7+7}{2}}{2}=7$.\n综上所述, 这组数据的平均数是 6.3 , 中位数是 7 . 众数是 7", "solution": "null", "level": "八年级", "question": "(5 分) 某次数学测验中, 10 位同学某题的得分情况如下 $2,3,4,6,7,7,7,8,9,10$ 求这组数据的平均数、众数和中位数.", "options": [], "subject": "计数", "analysis": "解: 平均数 $=\\frac{1}{10}(2+3+4+6+7+7+7+8+9+10)=6.3$. 在这一组数据中 7 是出现次数最多的, 故众数是 7 ;\n\n将这组数据从小到大的顺序排列(2,3,4,6,7,7,7,8,9,10),处于中间位置的那 2 个数是 7、 7 ,\n\n那么由中位数的定义可知, 这组数据的中位数是 $\\frac{\\frac{7+7}{2}}{2}=7$.\n综上所述, 这组数据的平均数是 6.3 , 中位数是 7 . 众数是 7"} {"id": "5702", "image": ["7152.jpg"], "answer": "解: (1) 该组数据的平均数 $={ }^{\\frac{1}{50}}$\n\n$(6 \\times 1+12 \\times 1+15 \\times 7+18 \\times 18+20 \\times 10+25 \\times 5+27 \\times 2+30 \\times 2+32 \\times 1+35 \\times 1+36 \\times 2)=20.5$;\n\n众数为 18 ;\n\n中位数为 18 .\n\n(2) 该市中考女生一分钟仰卧起坐项目的合格标准应定为 18 次较为合适,\n\n因为众数及中位数均为 18 , 且 50 人中达到 18 次以上的人数有 41 人,因此确定 18 次能保证大多数人达标.\n\n(3)根据(2)的标准估计该市中考女生一分钟仰卧起坐项目测试合格率为 $82 \\%$.", "solution": "null", "level": "八年级", "question": "(5 分) 某市实行中考改革, 需要根据该市中学生体能的实际情况重新制定中考体育标准. 为此,抽取了 50 名初中毕业的女学生进行“一分钟仰卧起坐”次数测试. 测试的情况绘制成表格如下:\n\n\n\n(1) 求这次抽样测试数据的平均数、众数和中位数;\n\n(2) 根据这一样本数据的特点, 你认为该市中考女生“一分钟仰卧起坐”项目测试的合格标准应定为多少次较为合适? 请简要说明理由;\n\n(3)根据 (2) 中你认为合格的标准,试估计该市中考女生“一分钟仰卧起坐”项目测试的合格率是多少?", "options": [], "subject": "计数", "analysis": "解: (1) 该组数据的平均数 $={ }^{\\frac{1}{50}}$\n\n$(6 \\times 1+12 \\times 1+15 \\times 7+18 \\times 18+20 \\times 10+25 \\times 5+27 \\times 2+30 \\times 2+32 \\times 1+35 \\times 1+36 \\times 2)=20.5$;\n\n众数为 18 ;\n\n中位数为 18 .\n\n(2) 该市中考女生一分钟仰卧起坐项目的合格标准应定为 18 次较为合适,\n\n因为众数及中位数均为 18 , 且 50 人中达到 18 次以上的人数有 41 人,因此确定 18 次能保证大多数人达标.\n\n(3)根据(2)的标准估计该市中考女生一分钟仰卧起坐项目测试合格率为 $82 \\%$."} {"id": "5703", "image": ["7153.jpg", "7154.jpg", "7154.jpg"], "answer": " (1) 解:根据条形图可得出:\n\n平均用水 11 吨的用户为: $100-20-10-20-10=40$ (户),\n\n如图所示:\n\n\n\n(2) 解: 平均数为: $\\frac{1}{100}$\n\n根据 11 出现次数最多, 故众数为: 11,\n\n根据 100 个数据的最中间为第 50 和第 51 个数据,\n\n按大小排列后第 50,51 个数据是 11, 故中位数为: 11 ;\n\n答: 这 100 个样本数据的平均数, 众数和中位数分别是 $11.6,11,11$;\n\n(3)解:样本中不超过 12 吨的有 $20+40+10=70$ (户),\n\n答:黄冈市直机关 500 户家庭中月平均用水量不超过 12 吨的约有: $500 \\times{ }^{\\frac{70}{100}}=350$ (户)", "solution": "null", "level": "八年级", "question": "(15 分 ) 为了倡导“节约用水, 从我做起”, 黄冈市政府决定对市直机关 500 户家庭的用水情况作一次调查, 市政府调查小组随机抽查了其中 100 户家庭一年的月平均用水量 (单位: 吨). 并将调查结果制成了如图所示的条形统计图.\n\n\n\n(1) 请将条形统计图补充完整;\n\n(2) 求这 100 个样本数据的平均数, 众数和中位数;\n\n(3)根据样本数据, 估计黄冈市直机关 500 户家庭中月平均用水量不超过 12 吨的约有多少户?", "options": [], "subject": "计数", "analysis": " (1) 解:根据条形图可得出:\n\n平均用水 11 吨的用户为: $100-20-10-20-10=40$ (户),\n\n如图所示:\n\n\n\n(2) 解: 平均数为: $\\frac{1}{100}$\n\n根据 11 出现次数最多, 故众数为: 11,\n\n根据 100 个数据的最中间为第 50 和第 51 个数据,\n\n按大小排列后第 50,51 个数据是 11, 故中位数为: 11 ;\n\n答: 这 100 个样本数据的平均数, 众数和中位数分别是 $11.6,11,11$;\n\n(3)解:样本中不超过 12 吨的有 $20+40+10=70$ (户),\n\n答:黄冈市直机关 500 户家庭中月平均用水量不超过 12 吨的约有: $500 \\times{ }^{\\frac{70}{100}}=350$ (户)"} {"id": "5706", "image": ["7155.jpg", "7156.jpg", "7156.jpg"], "answer": " (1) $0|0| 1|4| 2|8| 5$\n\n(2) $\\mathrm{m}=88$\n\n(3) 解: a 若 A 学校有 400 名初二学生, 估计这次考试成绩 80 分以上人数为:\n\n$800 \\times \\frac{15}{20}=600$\n\n(人).\n\n答: 估计这次考试成绩 80 分以上(包含 80 分)人数为 600 人;\n\nb: (1) $\\mathrm{A}$ 学校的中位数与众数都比 ${ }^{B}$ 学校的高, 因此 $A$ 学校的成绩比 $B$ 学校的学生成绩好. (2)根据表格可知, $B$ 学校的成绩的平均数高于 $A$ 学校, $A$ 学校的方差高于 $B$ 学校成绩的方差, 因此说明 $B$ 学校的成绩好于 $A$ 学校.\n\n解: (1)整理、描述数据:\n\n\n\n故答案为: $0,0,1,4,2,8,5$; (2)分析数据:\n\n经统计, $\\mathrm{B}$ 校的数据中 88 出现的次数最多, 故表格中 $\\mathrm{m}$ 的值是 88 .\n\n故答案为: 88 ;\n\n得出结论:", "solution": "null", "level": "八年级", "question": "(15 分) 某县教育局为了对该区八年级数学学科教学质量进行检查, 对该区八年级的学生进行摸底, 为了解摸底的情况, 进行了抽样调查, 过程如下,请将有关问题补充完整.\n收集数据: 随机抽取 $A$ 学校与 $B$ 学校的各 20 名学生的数学成绩(单位: 分)进行分析:\n\n\n\n(1) 整理、描述数据:按如下数据段整理、描述这两组数据\n\n| 分段 | $30 \\leq x \\leq 39$ | $40 \\leq x \\leq 49$ | $\\|50 \\leq x \\leq 59\\|$ | $60 \\leq x \\leq 69$ | $70 \\leq x \\leq 79$ | $\\|80 \\leq x \\leq 89\\|$ | $\\|90 \\leq x \\leq 100\\|$ |\n| :---: | :---: | :---: | :---: | :---: | :---: | :---: | :---: |\n| $A_{\\text {学校 }}$ | 1 | 1 | 0 | 0 | 3 | 7 | 8 |\n| $B_{\\text {学校 }}$ | | | | | | | $>$ |\n\n(2) 分析数据: 两组数据的平均数、中位数、众数、方差如下表:\n\n| 统计量
学校 | 平均数 | 中位数 | 众数 | 方差 |\n| :--- | :--- | :--- | :--- | :--- |\n| $A$ 学校 | 81.85 | 88 | 91 | 268.43 |\n| $B$ 学校 | 81.95 | 86 | $\\mathrm{~m}$ | 115.25 |\n\n(3)得出结论:\n\n$a$ : 若 $A$ 学校有 800 名八年级学生, 估计这次考试成绩 80 分以上(包含 80 分)人数为多少人?\n\n$b$ : 根据表格中的数据, 推断出哪所学校学生的数学水平较高, 并说明理由. (至少从两个不同的角度说明推断的合理性)", "options": [], "subject": "计数", "analysis": " (1) $0|0| 1|4| 2|8| 5$\n\n(2) $\\mathrm{m}=88$\n\n(3) 解: a 若 A 学校有 400 名初二学生, 估计这次考试成绩 80 分以上人数为:\n\n$800 \\times \\frac{15}{20}=600$\n\n(人).\n\n答: 估计这次考试成绩 80 分以上(包含 80 分)人数为 600 人;\n\nb: (1) $\\mathrm{A}$ 学校的中位数与众数都比 ${ }^{B}$ 学校的高, 因此 $A$ 学校的成绩比 $B$ 学校的学生成绩好. (2)根据表格可知, $B$ 学校的成绩的平均数高于 $A$ 学校, $A$ 学校的方差高于 $B$ 学校成绩的方差, 因此说明 $B$ 学校的成绩好于 $A$ 学校.\n\n解: (1)整理、描述数据:\n\n\n\n故答案为: $0,0,1,4,2,8,5$; (2)分析数据:\n\n经统计, $\\mathrm{B}$ 校的数据中 88 出现的次数最多, 故表格中 $\\mathrm{m}$ 的值是 88 .\n\n故答案为: 88 ;\n\n得出结论:"} {"id": "5819", "image": [], "answer": "解:\n\n$x=\\frac{13 \\times 1+14 \\times 4+15 \\times 5+16 \\times 2}{1+4+5+2} \\approx 14.7$ (多)\n\n答: 校女子排球队队员的平均年龄为 14.7 岁", "solution": "null", "level": "八年级", "question": "(5 分) 下表是校女子排球队队员的年龄分布:\n\n| 年龄 | 13 | 14 | 15 | 16 |\n| :--- | :---: | :---: | :---: | :---: |\n| 频数 | 1 | 4 | 5 | 2 |\n\n求校女子排球队队员的平均年龄(可使用计算器)。", "options": [], "subject": "计数", "analysis": "解:\n\n$x=\\frac{13 \\times 1+14 \\times 4+15 \\times 5+16 \\times 2}{1+4+5+2} \\approx 14.7$ (多)\n\n答: 校女子排球队队员的平均年龄为 14.7 岁"} {"id": "5046", "image": [], "answer": "(1)设增长率为$x$,根据“第一批公益课受益学生10万人次,第三批公益课受益学生12.1万人次”可列方程求解;(2)用$24.2times$(1+增长率),计算即可求解.【解析】解:(1)设这个增长率为$x$,根据题意得:$10(1+x)^{2}=12.1$,解得:$x_{1}=0.1=10%,x_{2}=-2.1$(舍去),答:这个增长率为$10%$;(2)$12.1times(1+10%)=13.31$万人,答:预计第四批自愿者数将达到13.31万人次.【点睛】本题考查了一元二次方程的应用,明确题意,准确得到等量关系是解题的关键.", "solution": "null", "level": "八年级", "question": "(10分)广大党员群众积极参加公益活动,据统计某市今年第一批自愿者为10万人次,第三批自愿者为12.1万人次.(1)如果第二批、第三批自愿者人次的增长率相同,求这个增长率;(2)如果按照(1)中的增长率,预计第四批自愿者数将达到多少万人次?", "options": [], "subject": "计数", "analysis": "(1)设增长率为$x$,根据“第一批公益课受益学生10万人次,第三批公益课受益学生12.1万人次”可列方程求解;(2)用$24.2times$(1+增长率),计算即可求解.【解析】解:(1)设这个增长率为$x$,根据题意得:$10(1+x)^{2}=12.1$,解得:$x_{1}=0.1=10%,x_{2}=-2.1$(舍去),答:这个增长率为$10%$;(2)$12.1times(1+10%)=13.31$万人,答:预计第四批自愿者数将达到13.31万人次.【点睛】本题考查了一元二次方程的应用,明确题意,准确得到等量关系是解题的关键."} {"id": "5704", "image": [], "answer": " (1) 解: $[20 m+60(m+2)+40(m-1)] \\div(20+60+40)$,\n$=(20 m+60 m+120+40 m-40) \\div 120$,\n\n$={ }^{\\left(m+\\frac{2}{3}\\right)}$ 元,\n\n$\\therefore \\therefore$ 混合果胨的售价应定为 ${ }^{\\left(m+\\frac{2}{3}\\right)}$ 元\n\n(2) 解: $[(m+m+2+m-1) \\div 3] \\times 120=120 m+40$\n\n$20 m+60(m+2)+40(m-1)=120 m+80$\n\n$120 m+40-(120 m+80)=-40$ ( 元) ,\n\n所以如果按小张写的单价全部售完, 这批果胨的利润将减少, 减少 40 元", "solution": "null", "level": "八年级", "question": "(10 分) 华润苏果超市有 A、B、C 三种果冻出售, A 种果冻 20 千克, 售价为 $m$ 元每千克, B 种果冻 60 千克, 售价比 A 种贵 2 元每千克, C 种果冻 40 千克, 售价比 A 种便宜 1 元每千克.\n\n(1)若将这三种果冻全部混合在一起销售, 在保证总售价不变的情况下, 混合果冻的售价应定为多少?\n\n(2)售货员小张在写混合后的销售单价牌时, 误写成原来三个单价的平均数 ,如果混合果冻按小张写的单价全部售完,超市的这批果冻的利润有何变化?变化多少元?", "options": [], "subject": "算术", "analysis": " (1) 解: $[20 m+60(m+2)+40(m-1)] \\div(20+60+40)$,\n$=(20 m+60 m+120+40 m-40) \\div 120$,\n\n$={ }^{\\left(m+\\frac{2}{3}\\right)}$ 元,\n\n$\\therefore \\therefore$ 混合果胨的售价应定为 ${ }^{\\left(m+\\frac{2}{3}\\right)}$ 元\n\n(2) 解: $[(m+m+2+m-1) \\div 3] \\times 120=120 m+40$\n\n$20 m+60(m+2)+40(m-1)=120 m+80$\n\n$120 m+40-(120 m+80)=-40$ ( 元) ,\n\n所以如果按小张写的单价全部售完, 这批果胨的利润将减少, 减少 40 元"} {"id": "5727", "image": ["7159.jpg"], "answer": " (1) 解: 原式 $=5 \\times \\sqrt{8}-\\sqrt{6} \\times \\sqrt{27}$\n\n$=5 \\times 2 \\sqrt{2}-\\sqrt{6} \\times 3 \\sqrt{3}$\n\n$=10 \\sqrt{2}-9 \\sqrt{2}$\n\n$=\\sqrt{2}$\n\n(2)解:由题可得:\n\n$(x+1) \\quad(2 x+1)-3 x=9$,\n\n$2 x^{2}+3 x+1-3 x=9$\n\n$\\therefore 2 x^{2}=8$\n\n解得: $x_{1}=2, x_{2}=-2$.", "solution": "null", "level": "八年级", "question": "(10 分 ) 阅读下面材料, 解答问题: 将 4 个数 $\\mathrm{a} 、 \\mathrm{~b} 、 \\mathrm{c} 、 \\mathrm{~d}$ 排列成 2 行 2 列, 记为: $\\left.\\right|_{c} ^{a}{ }_{d}^{b} \\mid$, 叫\n\n\n\n(1) 请你计算 $\\left|\\begin{array}{cc}5 & \\sqrt{6} \\\\ \\sqrt{27} & \\sqrt{8}\\end{array}\\right|$ 的值;\n\n(2) 若 $\\left.\\right|^{x+1} \\quad 2 x+1 \\mid=9$, 求 $x$ 的值.", "options": [], "subject": "算术", "analysis": " (1) 解: 原式 $=5 \\times \\sqrt{8}-\\sqrt{6} \\times \\sqrt{27}$\n\n$=5 \\times 2 \\sqrt{2}-\\sqrt{6} \\times 3 \\sqrt{3}$\n\n$=10 \\sqrt{2}-9 \\sqrt{2}$\n\n$=\\sqrt{2}$\n\n(2)解:由题可得:\n\n$(x+1) \\quad(2 x+1)-3 x=9$,\n\n$2 x^{2}+3 x+1-3 x=9$\n\n$\\therefore 2 x^{2}=8$\n\n解得: $x_{1}=2, x_{2}=-2$."} {"id": "5728", "image": [], "answer": "解: 设原正方形的边长为 $\\mathrm{xcm}$, 则根据题意得 : $(x+3)^{2}=x^{2}+39$\n\n解得 $x=5$.\n答: 正方形的边长是 $5 \\mathrm{~cm}$ 。", "solution": "null", "level": "八年级", "question": "(5 分)一个正方形的边长增加 $3 \\mathrm{~cm}$, 它的面积就增加 $39 \\mathrm{~cm}^{2}$, 求这个正方形的边长.", "options": [], "subject": "算术", "analysis": "解: 设原正方形的边长为 $\\mathrm{xcm}$, 则根据题意得 : $(x+3)^{2}=x^{2}+39$\n\n解得 $x=5$.\n答: 正方形的边长是 $5 \\mathrm{~cm}$ 。"} {"id": "5729", "image": [], "answer": "解:设每件祄衫应降价 $x$ 元, 则每件盈利 $40-\\mathrm{x}$ 元, 每天可以售出 $10+\\mathrm{x}$,由题意, 得 $(40-x)(10+x)=600$,\n\n即: $(x-10)(x-20)=0$,\n\n解, 得 $\\mathrm{x} 1=10, x 2=20$,\n\n为了扩大销售量, 增加盈利, 尽快减少库存, 所以 $\\mathrm{x}$ 的值应为 20 ,\n\n所以, 若商场平均每天要盈利 600 元, 每件祄衫应降价 20 元.", "solution": "null", "level": "八年级", "question": "(5 分) 某商场销售一批名牌祄衫, 平均每天可售出 10 件, 每件盈利 40 元, 为了扩大销售, 增加盈利, 尽快减少库存, 商场决定采取适当的降价措施. 经调查发现, 如果每件衬衫每降价 1 元, 商场平均每天可多售出 1 件, 若商场平均每天要盈利 600 元, 每件祄衫应降价多少元?", "options": [], "subject": "算术", "analysis": "解:设每件祄衫应降价 $x$ 元, 则每件盈利 $40-\\mathrm{x}$ 元, 每天可以售出 $10+\\mathrm{x}$,由题意, 得 $(40-x)(10+x)=600$,\n\n即: $(x-10)(x-20)=0$,\n\n解, 得 $\\mathrm{x} 1=10, x 2=20$,\n\n为了扩大销售量, 增加盈利, 尽快减少库存, 所以 $\\mathrm{x}$ 的值应为 20 ,\n\n所以, 若商场平均每天要盈利 600 元, 每件祄衫应降价 20 元."} {"id": "5732", "image": [], "answer": "解:设每件商品定价为 $x$ 元.\n\n(1) 当 $x \\geq 40$ 时, $(x-24)[480-20(x-40)]=7680$,\n\n解得: $x_{1}=40, x_{2}=48$;\n\n(2) 当 $x<40$ 时, $(x-24)[480+40(40-x)]=7680$,\n\n解得: $x_{1}=40$ (舍去), $x_{2}=36$.\n\n答: 要使该商品每天获利 7680 元, 应定价为 36 元/件、 40 元/件或 48 元/件.", "solution": "null", "level": "八年级", "question": "(5 分) 某连锁超市派遣调查小组在春节期间调查某种商品的销售情况, 下面是调查后小张与其他两位成员交流的情况.\n\n小张: “该商品的进价为 24 元/件. ”\n\n成员甲: “当定价为 40 元/件时,每天可售出 480 件. ”\n\n成员乙: “若单价每涨 1 元,则每天少售出 20 件; 若单价每降 1 元,则每天多售出 40 件. ”根据他们的对话, 请你求出要使该商品每天获利 7680 元, 应该怎样合理定价?", "options": [], "subject": "算术", "analysis": "解:设每件商品定价为 $x$ 元.\n\n(1) 当 $x \\geq 40$ 时, $(x-24)[480-20(x-40)]=7680$,\n\n解得: $x_{1}=40, x_{2}=48$;\n\n(2) 当 $x<40$ 时, $(x-24)[480+40(40-x)]=7680$,\n\n解得: $x_{1}=40$ (舍去), $x_{2}=36$.\n\n答: 要使该商品每天获利 7680 元, 应定价为 36 元/件、 40 元/件或 48 元/件."} {"id": "5734", "image": [], "answer": "(1)解:设降低的百分率为 $x$, 依题意有, $25(1-x)^{2}=16$,解得, $x_{1}=0.2=20 \\%, x_{2}=1.8$ (舍去) ;\n\n(2) 解: 小红全家少上缴税 $25 \\times 20 \\% \\times 4=20$ (元);\n\n(3)解:全乡少上缴税 $16000 \\times 25 \\times 20 \\%=80000$ (元).\n\n答: 降低的增长率是 $20 \\%$, 明年小红家减少的农业税是 20 元, 该乡农民明年减少的农业税是 80000 元.", "solution": "null", "level": "八年级", "question": "(15 分 ) 今年,我国政府为减轻农民负担,决定在 5 年内免去农业税. 某乡今年人均上缴农业税 25 元, 若两年后人均上缴农业税为 16 元, 假设这两年降低的百分率相同.\n\n(1) 求降低的百分率;\n\n(2)若小红家有 4 人,明年小红家减少多少农业税?\n\n(3)小红所在的乡约有 16000 农民,问该乡农民明年减少多少农业税?", "options": [], "subject": "算术", "analysis": "(1)解:设降低的百分率为 $x$, 依题意有, $25(1-x)^{2}=16$,解得, $x_{1}=0.2=20 \\%, x_{2}=1.8$ (舍去) ;\n\n(2) 解: 小红全家少上缴税 $25 \\times 20 \\% \\times 4=20$ (元);\n\n(3)解:全乡少上缴税 $16000 \\times 25 \\times 20 \\%=80000$ (元).\n\n答: 降低的增长率是 $20 \\%$, 明年小红家减少的农业税是 20 元, 该乡农民明年减少的农业税是 80000 元."} {"id": "5757", "image": [], "answer": " (1) 解: 原式 $=4 \\sqrt{3} \\times \\frac{1}{\\sqrt{3}}-\\frac{1}{\\sqrt{2}} \\times \\sqrt{12}+2 \\sqrt{6}$\n\n$=4-\\sqrt{6}+2 \\sqrt{6}$\n\n$=4+\\sqrt{6} ;$\n\n(2) 解: 原式 $=(\\sqrt{3}+1)^{2}+(\\sqrt{3}-1)^{2}$\n\n$=3+1+2 \\sqrt{3}+3+1-2 \\sqrt{3}$\n\n$=8$", "solution": "null", "level": "八年级", "question": "(10 分) 计算:\n\n(1)\n\n$\\sqrt{48} \\div \\sqrt{3}-\\sqrt{\\frac{1}{2}} \\times \\sqrt{12}+\\sqrt{24}$\n\n(2) 已知 $x=\\sqrt{3}+1, y=\\sqrt{3}-1$, 求 $x^{2}+y^{2}$ 的值.", "options": [], "subject": "算术", "analysis": " (1) 解: 原式 $=4 \\sqrt{3} \\times \\frac{1}{\\sqrt{3}}-\\frac{1}{\\sqrt{2}} \\times \\sqrt{12}+2 \\sqrt{6}$\n\n$=4-\\sqrt{6}+2 \\sqrt{6}$\n\n$=4+\\sqrt{6} ;$\n\n(2) 解: 原式 $=(\\sqrt{3}+1)^{2}+(\\sqrt{3}-1)^{2}$\n\n$=3+1+2 \\sqrt{3}+3+1-2 \\sqrt{3}$\n\n$=8$"} {"id": "5764", "image": [], "answer": "\n\n(1) $\\sqrt{n+1}-\\sqrt{n}$\n\n(2) 解: $\\frac{1}{1+\\sqrt{2}}+\\frac{1}{\\sqrt{2}+\\sqrt{3}}+\\frac{1}{\\sqrt{3}+\\sqrt{4}}+\\cdots+\\frac{1}{\\sqrt{98}+\\sqrt{99}}+\\frac{1}{\\sqrt{99}+\\sqrt{100}}$\n\n$=\\sqrt{2}-1+\\sqrt{3}-\\sqrt{2}+\\sqrt{4}-\\sqrt{3}+\\cdots+\\sqrt{99}-\\sqrt{98}+\\sqrt{100}-\\sqrt{99}$\n\n$=-1+\\sqrt{100}$\n\n$=-1+10$\n\n$=9$.\n\n解: (1) $\\because^{\\frac{1}{\\sqrt{5}+\\sqrt{4}}=\\frac{1 \\times(\\sqrt{5}-\\sqrt{4})}{(\\sqrt{5}+\\sqrt{4})(\\sqrt{5}-\\sqrt{4})}=\\sqrt{5}-\\sqrt{4}}$;\n\n$\\frac{1}{\\sqrt{6}+\\sqrt{5}}=\\frac{1 \\times(\\sqrt{6}-\\sqrt{5})}{(\\sqrt{6}+\\sqrt{5})(\\sqrt{6}-\\sqrt{5})}=\\sqrt{6}-\\sqrt{5}$\n\n归纳总结得: $\\frac{1}{\\sqrt{n+1}+\\sqrt{n}}=\\sqrt{n+1}-\\sqrt{n} \\quad(\\mathrm{n} \\geq 1)$\n故答案为 $\\sqrt{n+1}-\\sqrt{n}$;", "solution": "null", "level": "八年级", "question": "(6 分) 阅读下列解题过程: $\\frac{1}{\\sqrt{5}+\\sqrt{4}}=\\frac{1 \\times(\\sqrt{5}-\\sqrt{4})}{(\\sqrt{5}+\\sqrt{4})(\\sqrt{5}-\\sqrt{4})}=\\sqrt{5}-\\sqrt{4}$, $\\frac{1}{\\sqrt{6}+\\sqrt{5}}=\\frac{1 \\times(\\sqrt{6}-\\sqrt{5}}{(\\sqrt{6}+\\sqrt{5})(\\sqrt{6}-\\sqrt{5})}=\\sqrt{6}-\\sqrt{5}$, 请回答下列回题:\n\n(1) 观察上面的解答过程, 请写出 ${ }^{\\frac{1}{\\sqrt{n+1}+\\sqrt{n}}}=$ \\$ \\qquad \\$\n(2) 利用上面的解法, 请化简:\n\n$$\n\\frac{1}{1+\\sqrt{2}}+\\frac{1}{\\sqrt{2}+\\sqrt{3}}+\\frac{1}{\\sqrt{3}+\\sqrt{4}}+\\cdots+\\frac{1}{\\sqrt{98}+\\sqrt{99}}+\\frac{1}{\\sqrt{99+\\sqrt{100}}}\n$$", "options": [], "subject": "算术", "analysis": "\n\n(1) $\\sqrt{n+1}-\\sqrt{n}$\n\n(2) 解: $\\frac{1}{1+\\sqrt{2}}+\\frac{1}{\\sqrt{2}+\\sqrt{3}}+\\frac{1}{\\sqrt{3}+\\sqrt{4}}+\\cdots+\\frac{1}{\\sqrt{98}+\\sqrt{99}}+\\frac{1}{\\sqrt{99}+\\sqrt{100}}$\n\n$=\\sqrt{2}-1+\\sqrt{3}-\\sqrt{2}+\\sqrt{4}-\\sqrt{3}+\\cdots+\\sqrt{99}-\\sqrt{98}+\\sqrt{100}-\\sqrt{99}$\n\n$=-1+\\sqrt{100}$\n\n$=-1+10$\n\n$=9$.\n\n解: (1) $\\because^{\\frac{1}{\\sqrt{5}+\\sqrt{4}}=\\frac{1 \\times(\\sqrt{5}-\\sqrt{4})}{(\\sqrt{5}+\\sqrt{4})(\\sqrt{5}-\\sqrt{4})}=\\sqrt{5}-\\sqrt{4}}$;\n\n$\\frac{1}{\\sqrt{6}+\\sqrt{5}}=\\frac{1 \\times(\\sqrt{6}-\\sqrt{5})}{(\\sqrt{6}+\\sqrt{5})(\\sqrt{6}-\\sqrt{5})}=\\sqrt{6}-\\sqrt{5}$\n\n归纳总结得: $\\frac{1}{\\sqrt{n+1}+\\sqrt{n}}=\\sqrt{n+1}-\\sqrt{n} \\quad(\\mathrm{n} \\geq 1)$\n故答案为 $\\sqrt{n+1}-\\sqrt{n}$;"} {"id": "5766", "image": [], "answer": " (1) $\\sqrt{n}-\\sqrt{n-1}$\n\n(2) 解: 原式 $=\\sqrt{2}-1+\\sqrt{3}-\\sqrt{2}+\\ldots+\\sqrt{2016}-\\sqrt{2015}=\\sqrt{2016}-1=12 \\sqrt{14}-1$.\n\n解: (1) $\\frac{1}{\\sqrt{n}+\\sqrt{n-1}}=\\frac{\\sqrt{n}-\\sqrt{n-1}}{(\\sqrt{n}+\\sqrt{n-1})(\\sqrt{n}-\\sqrt{n-1})}=\\sqrt{n}-\\sqrt{n-1}$\n\n故答案为: $\\sqrt{n}-\\sqrt{n-1}$.", "solution": "null", "level": "八年级", "question": "( 9 分 ) 阅读下列解题过程:\n\n$\\frac{1}{\\sqrt{5}+\\sqrt{4}}=\\frac{\\sqrt{5}-\\sqrt{4}}{(\\sqrt{5})^{2}-\\left(\\sqrt{4)^{2}}\\right.}=\\sqrt{5}-\\sqrt{4}=\\sqrt{5}-2$\n\n$\\frac{1}{\\sqrt{6}+\\sqrt{5}}=\\frac{\\sqrt{6}-\\sqrt{5}}{(\\sqrt{6})^{2}-(\\sqrt{5})^{2}}=\\sqrt{6}-\\sqrt{5}$\n\n请回答下列问题:\n\n(1)观察上面的解题过程, 请直接写出结果。 $\\frac{1}{\\sqrt{n}+\\sqrt{n-1}}=$ \\$ \\qquad \\$ .\n\n(2) 利用上面提供的信息请化简: $\\frac{1}{\\sqrt{2}+1}+\\frac{1}{\\sqrt{3}+\\sqrt{2}}+\\frac{1}{\\sqrt{4}+\\sqrt{3}}+\\ldots \\ldots+{ }^{\\frac{1}{\\sqrt{2016}+\\sqrt{2015}}}$ 的值。", "options": [], "subject": "算术", "analysis": " (1) $\\sqrt{n}-\\sqrt{n-1}$\n\n(2) 解: 原式 $=\\sqrt{2}-1+\\sqrt{3}-\\sqrt{2}+\\ldots+\\sqrt{2016}-\\sqrt{2015}=\\sqrt{2016}-1=12 \\sqrt{14}-1$.\n\n解: (1) $\\frac{1}{\\sqrt{n}+\\sqrt{n-1}}=\\frac{\\sqrt{n}-\\sqrt{n-1}}{(\\sqrt{n}+\\sqrt{n-1})(\\sqrt{n}-\\sqrt{n-1})}=\\sqrt{n}-\\sqrt{n-1}$\n\n故答案为: $\\sqrt{n}-\\sqrt{n-1}$."} {"id": "5816", "image": [], "answer": "解: 原式 $=3-{ }^{\\frac{\\sqrt{12}}{\\sqrt{3}}}=3-\\sqrt{4}=3-2=1$\n\n解: 用二次根式的乘除法法则以及加减法则计算即可求解。即原式 $=3-\\frac{\\sqrt{12}}{\\sqrt{3}}=3-\\sqrt{4}=3-2=1$,", "solution": "null", "level": "八年级", "question": "$(5 \\text { 分 })^{\\sqrt[3]{27}}-\\frac{\\sqrt{2} \\times \\sqrt{6}}{\\sqrt{3}}$.", "options": [], "subject": "算术", "analysis": "解: 原式 $=3-{ }^{\\frac{\\sqrt{12}}{\\sqrt{3}}}=3-\\sqrt{4}=3-2=1$\n\n解: 用二次根式的乘除法法则以及加减法则计算即可求解。即原式 $=3-\\frac{\\sqrt{12}}{\\sqrt{3}}=3-\\sqrt{4}=3-2=1$,"} {"id": "5817", "image": [], "answer": " (1) 原式 $=4-1-2+\\sqrt{3}-2 \\sqrt{3}$\n\n$=-\\sqrt{3}+1$\n\n(2) $9 x^{2}-6 x+1=0$.\n\n$(3 x-1)^{2}=0$,\n\n$3 \\mathrm{x}-1=0$,\n\n解: $x_{1}=x_{2}=\\frac{1}{3}$", "solution": "null", "level": "八年级", "question": "(10 分) 计算或解方程\n\n(1) 计算:\n\n$$\n\\left(\\frac{1}{2}\\right)^{-2}-(\\sqrt{3} \\pi)^{0}-\\frac{\\sqrt{20}-\\sqrt{15}}{\\sqrt{5}}-\\sqrt{12}\n$$\n\n(2) 解方程: $9 x^{2}-6 x+1=0$", "options": [], "subject": "算术", "analysis": " (1) 原式 $=4-1-2+\\sqrt{3}-2 \\sqrt{3}$\n\n$=-\\sqrt{3}+1$\n\n(2) $9 x^{2}-6 x+1=0$.\n\n$(3 x-1)^{2}=0$,\n\n$3 \\mathrm{x}-1=0$,\n\n解: $x_{1}=x_{2}=\\frac{1}{3}$"} {"id": "4995", "image": [], "answer": "(1)直接化简二次根式,进而合并得出答案;(2)直接利用乘法公式化简,进而计算得出答案;(3)直接化简二次根式,再合并,进而结合二次根式的除法运算法则计算得出答案;(4)直接利用二次根式的除法运算法则、完全平方公式、零指数幂的性质分别化简,进而得出答案.【解析】解:(1)原式$=2sqrt{5}-4timesfrac{sqrt{5}}{5}+sqrt{5}$$=2sqrt{5}-frac{4sqrt{5}}{5}+sqrt{5}$$=frac{11sqrt{5}}{5}$(2)原式$=2+6+4sqrt{3}-(5-3)$$=2+6+4sqrt{3}-2$$=6+4sqrt{3}$(3)原式$=3left(3sqrt{2}+frac{1}{5}times5sqrt{2}-4timesfrac{sqrt{2}}{2}right)div4sqrt{2}$$=3(3sqrt{2}+sqrt{2}-2sqrt{2})div4sqrt{2}$$=6sqrt{2}div4sqrt{2}$$=frac{3}{2}$(4)原式$=sqrt{48}-(6+2+4sqrt{3})+1$$=4sqrt{3}-8-4sqrt{3}+1$$=-7$【点睛】此题主要考查了二次根式的混合运算,正确掌握相关运算法则化简二次根式是解题关键.", "solution": "null", "level": "八年级", "question": "计算.(1)$sqrt{20}-4sqrt{frac{1}{5}}+sqrt{5}$;$(2)(sqrt{2}+sqrt{6})^{2}-(sqrt{5}-sqrt{3})(sqrt{5}+sqrt{3})$.(3)$3left(sqrt{18}+frac{1}{5}sqrt{50}-4sqrt{frac{1}{2}}right)divsqrt{32}$.(4)$sqrt{24}divsqrt{frac{1}{2}}-(sqrt{6}+sqrt{2})^{2}+(pi-3)$0", "options": [], "subject": "算术", "analysis": "(1)直接化简二次根式,进而合并得出答案;(2)直接利用乘法公式化简,进而计算得出答案;(3)直接化简二次根式,再合并,进而结合二次根式的除法运算法则计算得出答案;(4)直接利用二次根式的除法运算法则、完全平方公式、零指数幂的性质分别化简,进而得出答案.【解析】解:(1)原式$=2sqrt{5}-4timesfrac{sqrt{5}}{5}+sqrt{5}$$=2sqrt{5}-frac{4sqrt{5}}{5}+sqrt{5}$$=frac{11sqrt{5}}{5}$(2)原式$=2+6+4sqrt{3}-(5-3)$$=2+6+4sqrt{3}-2$$=6+4sqrt{3}$(3)原式$=3left(3sqrt{2}+frac{1}{5}times5sqrt{2}-4timesfrac{sqrt{2}}{2}right)div4sqrt{2}$$=3(3sqrt{2}+sqrt{2}-2sqrt{2})div4sqrt{2}$$=6sqrt{2}div4sqrt{2}$$=frac{3}{2}$(4)原式$=sqrt{48}-(6+2+4sqrt{3})+1$$=4sqrt{3}-8-4sqrt{3}+1$$=-7$【点睛】此题主要考查了二次根式的混合运算,正确掌握相关运算法则化简二次根式是解题关键."} {"id": "5726", "image": [], "answer": " (1) $x=3 \\pm \\sqrt{5}$\n\n(2) $x<3$\n\n$x \\geq-1$\n\n$\\therefore-1 \\leq x<3$\n\n【解析】 (1) 解: $\\because \\mathrm{b}^{2}-4 \\mathrm{ac}=36-16=20$\n\n$\\therefore \\mathrm{x}=\\frac{6 \\pm 2 \\sqrt{5}}{2}$\n\n$x=3 \\pm \\sqrt{5}$\n\n故答案为: $x=3 \\pm \\sqrt{5}$", "solution": "null", "level": "八年级", "question": "( 10 分 $)$ 解答题\n\n(1) 解方程: $x^{2}-6 x+4=0$;\n\n(2) 解不等式组\n\n$$\n\\left\\{\\begin{array}{c}\n3 x+1<2(x+2) \\\\\n-\\frac{x}{3} \\leq \\frac{5 x}{3}+2\n\\end{array}\\right.\n$$", "options": [], "subject": "代数", "analysis": " (1) $x=3 \\pm \\sqrt{5}$\n\n(2) $x<3$\n\n$x \\geq-1$\n\n$\\therefore-1 \\leq x<3$\n\n【解析】 (1) 解: $\\because \\mathrm{b}^{2}-4 \\mathrm{ac}=36-16=20$\n\n$\\therefore \\mathrm{x}=\\frac{6 \\pm 2 \\sqrt{5}}{2}$\n\n$x=3 \\pm \\sqrt{5}$\n\n故答案为: $x=3 \\pm \\sqrt{5}$"} {"id": "5730", "image": [], "answer": "解: $\\because x=1$ 是关于 $\\mathrm{x}$ 的方程 $x^{2}-m x-2 m^{2}=0$ 的一个根,\n\n$\\therefore 1-m-2 m^{2}=0$\n\n$\\therefore 2 m^{2}+m=1$.\n\n$\\therefore m(2 m+1)=2 m^{2}+m=1$", "solution": "null", "level": "八年级", "question": "(5 分) 已知 $x=1$ 是关于 $\\mathrm{x}$ 的方程 $x^{2}-m x-2 m^{2}=0$ 的一个根, 求 $m(2 m+1)$ 的值.", "options": [], "subject": "代数", "analysis": "解: $\\because x=1$ 是关于 $\\mathrm{x}$ 的方程 $x^{2}-m x-2 m^{2}=0$ 的一个根,\n\n$\\therefore 1-m-2 m^{2}=0$\n\n$\\therefore 2 m^{2}+m=1$.\n\n$\\therefore m(2 m+1)=2 m^{2}+m=1$"} {"id": "5731", "image": [], "answer": "解答: $\\because$ 原方程有两个不相等的实数根,\n\n$\\therefore \\Delta=(2 \\mathrm{k}+1)^{2}-4\\left(\\mathrm{k}^{2}+1\\right)=4 \\mathrm{k}^{2}+4 \\mathrm{k}+1-4 \\mathrm{k}^{2}-4=4 \\mathrm{k}-3>0$,\n\n解得: $k>\\frac{3}{4}$", "solution": "null", "level": "八年级", "question": "(5 分 ) 关于 $x$ 的一元二次方程 $x^{2}+(2 k+1) x+k^{2}+1=0$ 有两个不等实根 $x_{1}, x_{2}$. 求实数 $k$ 的\n取值范围.", "options": [], "subject": "代数", "analysis": "解答: $\\because$ 原方程有两个不相等的实数根,\n\n$\\therefore \\Delta=(2 \\mathrm{k}+1)^{2}-4\\left(\\mathrm{k}^{2}+1\\right)=4 \\mathrm{k}^{2}+4 \\mathrm{k}+1-4 \\mathrm{k}^{2}-4=4 \\mathrm{k}-3>0$,\n\n解得: $k>\\frac{3}{4}$"} {"id": "5733", "image": [], "answer": " (1) 解: $\\Delta=(k-1)^{2}-4(\\mathrm{k}-2)$\n\n$=k^{2}-6 k+9$\n\n$=(k-3)^{2} \\geq 0$\n\n$\\because \\pm 0$,\n\n$\\therefore$ 方程总有两个实数根.\n\n(2) 解: 当 $k=2$\n\n$\\therefore x^{2}+x=0$\n\n解得 $x_{1}=0, x_{2}=-1$", "solution": "null", "level": "八年级", "question": "(10 分) 已知关于 $x$ 的一元二次方程 $x^{2}+(k-1) x+k-2=0$.\n\n(1) 求证: 方程总有两个实数根;\n\n(2)任意写出一个 $k$ 值代入方程, 并求出此时方程的解.", "options": [], "subject": "代数", "analysis": " (1) 解: $\\Delta=(k-1)^{2}-4(\\mathrm{k}-2)$\n\n$=k^{2}-6 k+9$\n\n$=(k-3)^{2} \\geq 0$\n\n$\\because \\pm 0$,\n\n$\\therefore$ 方程总有两个实数根.\n\n(2) 解: 当 $k=2$\n\n$\\therefore x^{2}+x=0$\n\n解得 $x_{1}=0, x_{2}=-1$"} {"id": "5756", "image": [], "answer": "解: $-3^{2}+6 \\cos 45^{\\circ}-\\sqrt{2}(2-\\sqrt{2})+|\\sqrt{2}-3|=-9+6 \\times \\frac{\\sqrt{2}}{2}-2 \\sqrt{2}-2+3-\\sqrt{2}$ $=-9+3 \\sqrt{2}-2 \\sqrt{2}+1-\\sqrt{2}$\n\n$=-8$", "solution": "null", "level": "八年级", "question": "(5 分) 计算: $-3^{2}+6 \\cos 45^{\\circ}-\\sqrt{2}(2-\\sqrt{2})+|\\sqrt{2}-3|$.", "options": [], "subject": "代数", "analysis": "解: $-3^{2}+6 \\cos 45^{\\circ}-\\sqrt{2}(2-\\sqrt{2})+|\\sqrt{2}-3|=-9+6 \\times \\frac{\\sqrt{2}}{2}-2 \\sqrt{2}-2+3-\\sqrt{2}$ $=-9+3 \\sqrt{2}-2 \\sqrt{2}+1-\\sqrt{2}$\n\n$=-8$"} {"id": "5759", "image": [], "answer": "解: 当 $m=\\sqrt{3}-\\sqrt{2}, n=\\sqrt{3}+\\sqrt{2}$ 时,\n\n$$\n\\begin{aligned}\n& m^{2}+m n+n^{2} \\\\\n& =(m+n)^{2}-m n \\\\\n& =(\\sqrt{3}-\\sqrt{2}+\\sqrt{3}+\\sqrt{2})^{2}-(\\sqrt{3}-\\sqrt{2}) \\times(\\sqrt{3}+\\sqrt{2}) \\\\\n& =(2 \\sqrt{3})^{2}-\\left[(\\sqrt{3})^{2}-(\\sqrt{2})^{2}\\right] \\\\\n& =12-(3-2) \\\\\n& =12-1 \\\\\n& =11\n\\end{aligned}\n$$", "solution": "null", "level": "八年级", "question": "(5 分) 已知 $m=\\sqrt{3}-\\sqrt{2}, n=\\sqrt{3}+\\sqrt{2}$, 求代数式 $m^{2}+m n+n^{2}$ 的值.", "options": [], "subject": "代数", "analysis": "解: 当 $m=\\sqrt{3}-\\sqrt{2}, n=\\sqrt{3}+\\sqrt{2}$ 时,\n\n$$\n\\begin{aligned}\n& m^{2}+m n+n^{2} \\\\\n& =(m+n)^{2}-m n \\\\\n& =(\\sqrt{3}-\\sqrt{2}+\\sqrt{3}+\\sqrt{2})^{2}-(\\sqrt{3}-\\sqrt{2}) \\times(\\sqrt{3}+\\sqrt{2}) \\\\\n& =(2 \\sqrt{3})^{2}-\\left[(\\sqrt{3})^{2}-(\\sqrt{2})^{2}\\right] \\\\\n& =12-(3-2) \\\\\n& =12-1 \\\\\n& =11\n\\end{aligned}\n$$"} {"id": "5762", "image": [], "answer": "解: $\\because x^{2}-2 y+\\sqrt{5} y=8+4 \\sqrt{5}$\n\n$\\therefore\\left(x^{2}-2 y-8\\right)+(y-4) \\sqrt{5}=0$\n\n$\\therefore x^{2}-2 y-8=0, y-4=0$\n\n$\\therefore x= \\pm 4, y=4$\n\n当 $x=4, y=4$ 时, $x+y=8$\n\n当 $x=-4, y=4, x=-4, y=4$\n\n即 $x+y$ 的值是 8 或 0 .", "solution": "null", "level": "八年级", "question": "( 5 分 ) 先阅读下面材料, 然后再根据要求解答提出的问题:\n\n设 $\\mathrm{a} 、 \\mathrm{~b}$ 是有理数, 且满足 $a+\\sqrt{2} b=3-2 \\sqrt{2}$, 求 $b^{a}$ 的值?\n\n解: 由题意得: $(a-3)+(b+2) \\sqrt{2}=0$ ,\n\n因为 $\\mathrm{a} 、 \\mathrm{~b}$ 都是有理数,\n\n所以 $a-3 、 b+2$ 也是有理数,\n\n由于 $\\sqrt{2}$ 是无理数,\n\n所以 $\\mathrm{a}-3=0 、 \\mathrm{~b}+2=0$,\n\n所以 $a=3 、 b=-2$,\n\n所以 $b^{a}=(-2)^{3}=-8$,\n\n问题: 设 $\\mathrm{x} 、 \\mathrm{y}$ 都是有理数, 且满足 $x^{2}-2 y+\\sqrt{5} y=8+4 \\sqrt{5}$, 求 $\\mathrm{x}+\\mathrm{y}$ 的值,", "options": [], "subject": "代数", "analysis": "解: $\\because x^{2}-2 y+\\sqrt{5} y=8+4 \\sqrt{5}$\n\n$\\therefore\\left(x^{2}-2 y-8\\right)+(y-4) \\sqrt{5}=0$\n\n$\\therefore x^{2}-2 y-8=0, y-4=0$\n\n$\\therefore x= \\pm 4, y=4$\n\n当 $x=4, y=4$ 时, $x+y=8$\n\n当 $x=-4, y=4, x=-4, y=4$\n\n即 $x+y$ 的值是 8 或 0 ."} {"id": "5815", "image": [], "answer": " (1) 解: $x^{2}+3 x-2=0$\n\n$\\because a=1, b=3, c=-2$,\n\n$\\therefore \\Delta=3^{2}-4 \\times 1 \\times(-2)=17>0$\n\n$\\therefore x=\\frac{-3 \\pm \\sqrt{17}}{2}$\n\n即: $x_{1}=\\frac{-3+\\sqrt{17}}{2} , x_{2}=\\frac{-3-\\sqrt{17}}{2}$\n\n(2) 解: $C(0,16)$\n\n$(x-3)^{2}-(2 x-6)=0$\n\n$(x-3)^{2}-2(x-3)=0$\n\n$(x-3-2)(x-3)=0$\n$(x-5)(x-3)=0$\n\n$\\therefore$ 解得: $x_{1}=3 , x_{2}=5$\n\n解: (1) 利用公式法求解可得;\n\n(2) 移项后, 提取公因式 $x-3$, 再进一步求解可得;", "solution": "null", "level": "八年级", "question": "(10 分) 用适当的方法解下列方程:\n\n(1) $x^{2}+3 x-2=0$\n\n(2) $(x-3)^{2}=2 x-6$", "options": [], "subject": "代数", "analysis": " (1) 解: $x^{2}+3 x-2=0$\n\n$\\because a=1, b=3, c=-2$,\n\n$\\therefore \\Delta=3^{2}-4 \\times 1 \\times(-2)=17>0$\n\n$\\therefore x=\\frac{-3 \\pm \\sqrt{17}}{2}$\n\n即: $x_{1}=\\frac{-3+\\sqrt{17}}{2} , x_{2}=\\frac{-3-\\sqrt{17}}{2}$\n\n(2) 解: $C(0,16)$\n\n$(x-3)^{2}-(2 x-6)=0$\n\n$(x-3)^{2}-2(x-3)=0$\n\n$(x-3-2)(x-3)=0$\n$(x-5)(x-3)=0$\n\n$\\therefore$ 解得: $x_{1}=3 , x_{2}=5$\n\n解: (1) 利用公式法求解可得;\n\n(2) 移项后, 提取公因式 $x-3$, 再进一步求解可得;"} {"id": "5818", "image": [], "answer": "解: $\\sqrt{1+\\frac{1}{1^{2}}+\\frac{1}{2^{2}}}+\\sqrt{1+\\frac{1}{2^{2}}+\\frac{1}{3^{2}}}+\\sqrt{1+\\frac{1}{3^{2}}+\\frac{1}{4^{2}}}+\\cdots+\\sqrt{1+\\frac{1}{2017^{2}}+\\frac{1}{208^{2}}}$\n\n$$\n\\begin{aligned}\n& =\\frac{3}{2}+\\frac{7}{6}+\\frac{13}{12}+\\cdots+1+\\frac{1}{2017 \\times 2018} \\\\\n& =1+\\frac{1}{2}+1+\\frac{1}{6}+1+\\frac{1}{12}+\\cdots+1+\\frac{1}{2017 \\times 2018} \\\\\n& =2017+1-\\frac{1}{2}+\\frac{1}{2}-\\frac{1}{3}+\\frac{1}{3}-\\frac{1}{4}+\\cdots+\\frac{1}{2017 \\times 2018} \\\\\n& =2017+1-\\frac{1}{2018} \\\\\n& =2017 \\frac{2017}{2018}\n\\end{aligned}\n$$", "solution": "null", "level": "八年级", "question": "( 5 分 ) 计算:\n\n$$\n\\sqrt{1+\\frac{1}{1^{2}}+\\frac{1}{2^{2}}}+\\sqrt{1+\\frac{1}{2^{2}}+\\frac{1}{3^{2}}}+\\sqrt{1+\\frac{1}{3^{2}}+\\frac{1}{4^{2}}}+\\cdots+\\sqrt{1+\\frac{1}{207^{2}}+\\frac{1}{2018^{2}}}\n$$", "options": [], "subject": "代数", "analysis": "解: $\\sqrt{1+\\frac{1}{1^{2}}+\\frac{1}{2^{2}}}+\\sqrt{1+\\frac{1}{2^{2}}+\\frac{1}{3^{2}}}+\\sqrt{1+\\frac{1}{3^{2}}+\\frac{1}{4^{2}}}+\\cdots+\\sqrt{1+\\frac{1}{2017^{2}}+\\frac{1}{208^{2}}}$\n\n$$\n\\begin{aligned}\n& =\\frac{3}{2}+\\frac{7}{6}+\\frac{13}{12}+\\cdots+1+\\frac{1}{2017 \\times 2018} \\\\\n& =1+\\frac{1}{2}+1+\\frac{1}{6}+1+\\frac{1}{12}+\\cdots+1+\\frac{1}{2017 \\times 2018} \\\\\n& =2017+1-\\frac{1}{2}+\\frac{1}{2}-\\frac{1}{3}+\\frac{1}{3}-\\frac{1}{4}+\\cdots+\\frac{1}{2017 \\times 2018} \\\\\n& =2017+1-\\frac{1}{2018} \\\\\n& =2017 \\frac{2017}{2018}\n\\end{aligned}\n$$"} {"id": "4996", "image": [], "answer": "(1)利用完全平方公式,结合二次根式的加减法和乘除法运算法则计算$x+y,xy$的值,从而代入求值;(2)利用完全平方公式,结合二次根式的加减法和乘除法运算法则计算$x+y,xy$的值,从而代入求值.【解析】解:(1)原式$=(x+y)^{2}-2xy$,$becausex=sqrt{3}+1,quady=sqrt{3}-1$,$thereforex+y=sqrt{3}+1+sqrt{3}-1=2sqrt{3}$,$xy=(sqrt{3}+1)(sqrt{3}-1)=3-1=2$,$therefore$原式$=(2sqrt{3})^{2}-2times2$$=12-4$$=8$;(2)原式$=2left(x^{2}+2xy+y^{2}right)+xy$$=2(x+y)^{2}+xy$,$becausex=sqrt{3}+1,y=sqrt{3}-1$,$thereforex+y=sqrt{3}+1+sqrt{3}-1=2sqrt{3}$,$xy=(sqrt{3}+1)quad(sqrt{3}-1)=3-1=2$,$therefore$原式$=2times(2sqrt{3})^{2}+2$$=2times12+2$$=24+2$$=26$.【点睛】本题考查二次根式的混合运算,理解二次根式的性质,掌握完全平方公式$(a+b)^{2}=$$a^{2}+2ab+b^{2}$和平方差公式$(a+b)quad(a-b)=a^{2}-b^{2}$是解题关键.", "solution": "null", "level": "八年级", "question": "已知$x=sqrt{3}+1,y=sqrt{3}-1$,求下列各式的值:(1)$x^{2}+y^{2}$;(2)$2x^{2}+5xy+2y^{2}$.", "options": [], "subject": "代数", "analysis": "(1)利用完全平方公式,结合二次根式的加减法和乘除法运算法则计算$x+y,xy$的值,从而代入求值;(2)利用完全平方公式,结合二次根式的加减法和乘除法运算法则计算$x+y,xy$的值,从而代入求值.【解析】解:(1)原式$=(x+y)^{2}-2xy$,$becausex=sqrt{3}+1,quady=sqrt{3}-1$,$thereforex+y=sqrt{3}+1+sqrt{3}-1=2sqrt{3}$,$xy=(sqrt{3}+1)(sqrt{3}-1)=3-1=2$,$therefore$原式$=(2sqrt{3})^{2}-2times2$$=12-4$$=8$;(2)原式$=2left(x^{2}+2xy+y^{2}right)+xy$$=2(x+y)^{2}+xy$,$becausex=sqrt{3}+1,y=sqrt{3}-1$,$thereforex+y=sqrt{3}+1+sqrt{3}-1=2sqrt{3}$,$xy=(sqrt{3}+1)quad(sqrt{3}-1)=3-1=2$,$therefore$原式$=2times(2sqrt{3})^{2}+2$$=2times12+2$$=24+2$$=26$.【点睛】本题考查二次根式的混合运算,理解二次根式的性质,掌握完全平方公式$(a+b)^{2}=$$a^{2}+2ab+b^{2}$和平方差公式$(a+b)quad(a-b)=a^{2}-b^{2}$是解题关键."} {"id": "4997", "image": [], "answer": "(1)根据勾股定理和三角形的面积公式解答即可.(2)原式利用完全平方公式变形后,把$a$与$b$的值代入计算即可求出值.【解析】解:(1)在Rt$triangleABC$中,$angleC=Rtangle,AC=2sqrt{2},AB=3sqrt{2}$,$thereforeBC=sqrt{AB^{2}-AC^{2}}=sqrt{10}$,$therefore$Rt$triangleABC$的周长$=2sqrt{2}+3sqrt{2}+sqrt{10}=5sqrt{2}+sqrt{10}$,Rt$triangleABC$的面积$=frac{1}{2}times2sqrt{2}timessqrt{10}=2sqrt{5}$.(2)$becausea=sqrt{3}+sqrt{2},quadb=sqrt{3}-sqrt{2}$,$thereforea+b=2sqrt{3},ab=1$,则原式$=(a+b)^{2}-3ab=12-3=9$.【点睛】此题考查了二次根式的化简求值,以及勾股定理的应用,熟练掌握运算法则是解本题的关键.", "solution": "null", "level": "八年级", "question": "(1)在Rt$triangleABC$中,$angleC=Rtangle,AC=2sqrt{2},AB=3sqrt{2}$.求Rt$triangleABC$的周长和面积.(2)已知$a=sqrt{3}+sqrt{2},b=sqrt{3}-sqrt{2}$,求$a^{2}-ab+b^{2}$的值.", "options": [], "subject": "代数", "analysis": "(1)根据勾股定理和三角形的面积公式解答即可.(2)原式利用完全平方公式变形后,把$a$与$b$的值代入计算即可求出值.【解析】解:(1)在Rt$triangleABC$中,$angleC=Rtangle,AC=2sqrt{2},AB=3sqrt{2}$,$thereforeBC=sqrt{AB^{2}-AC^{2}}=sqrt{10}$,$therefore$Rt$triangleABC$的周长$=2sqrt{2}+3sqrt{2}+sqrt{10}=5sqrt{2}+sqrt{10}$,Rt$triangleABC$的面积$=frac{1}{2}times2sqrt{2}timessqrt{10}=2sqrt{5}$.(2)$becausea=sqrt{3}+sqrt{2},quadb=sqrt{3}-sqrt{2}$,$thereforea+b=2sqrt{3},ab=1$,则原式$=(a+b)^{2}-3ab=12-3=9$.【点睛】此题考查了二次根式的化简求值,以及勾股定理的应用,熟练掌握运算法则是解本题的关键."} {"id": "4999", "image": [], "answer": "根据二次根式有意义的条件求出$x$,进而求出$y$,根据绝对值的性质、二次根式的性质计算,得到答案.【解析】解:由题意得,$x-3geqslant0,3-xgeqslant0$,解得:$x=3$,$thereforey<5$,则原式$=|y-5|-sqrt{(y-6)^{2}}=5-y-(6-y)=5-y-6+y=-1$.【点睛】本题考查的是二次根式有意义的条件、绝对值的性质以及二次根式的性质,掌握二次根式的被开方数是非负数是解题的关键.", "solution": "null", "level": "八年级", "question": "已知$x,y$满足$y\n\n$\\because \\mathrm{S}_{\\text {粪形 }} \\mathrm{ABCD}=\\mathrm{AB} \\cdot \\mathrm{CG}={ }^{\\frac{1}{2}} \\mathrm{AC} \\cdot \\mathrm{BD}$,\n\n即 $10 \\cdot \\mathrm{CG}==^{\\frac{1}{2}} \\times 12 \\times 16$,\n\n$\\therefore \\mathrm{CG}={ }^{\\frac{1}{2}}$.\n\n\n\n\n\n$\\therefore y=\\frac{4 \\pi}{5}-\\frac{12 t^{2}}{25}$,\n\n(2)当 $S_{\\text {四边形 } A P F E}={ }^{\\frac{8}{25}} S_{\\text {菱形 } A B C D}$\n\n则 $\\frac{48 t}{5}-\\frac{12 t^{2}}{25}=\\frac{8}{25} \\times\\left(\\frac{1}{2} \\times 12 \\times 16\\right)$,\n即 $t^{2}-20 t+64=0$,\n\n解这个方程, 得 $t_{1}=4, t_{2}=16>10$ (不合, 舍去)\n\n$\\therefore$ 存在 $\\mathrm{t}=4 \\mathrm{~s}$, 使得 $\\mathrm{S}_{\\text {四边形 } \\mathrm{APFE}}={ }^{\\frac{8}{25}} \\mathrm{~S}_{\\text {菱形 } \\mathrm{ABCD}} \\cdot$\n\n解: (1) $\\because$ 在菱形 $\\mathrm{ABCD}$ 中, 对角线 $\\mathrm{AC}, \\mathrm{BD}$ 相交于点 $\\mathrm{O}$, 且 $\\mathrm{AC}=12 \\mathrm{~cm}, \\mathrm{BD}=16 \\mathrm{~cm}$,\n\n$\\therefore \\mathrm{BO}=\\mathrm{DO}=8 \\mathrm{~cm}, \\quad \\mathrm{AO}=\\mathrm{CO}=6 \\mathrm{~cm}$,\n\n$\\therefore \\mathrm{AB}=\\sqrt{8^{2}+6^{2}}=10(\\mathrm{~cm})$,\n\n故答案为: 10 ;", "solution": "null", "level": "八年级", "question": "(11 分 ) 已知: 如图, 在菱形 $A B C D$ 中, 对角线 $A C, B D$ 相交于点 $O$, 且 $A C=12 \\mathrm{~cm}, B D=16 \\mathrm{~cm}$. 点 $\\mathrm{P}$ 从点 A 出发, 沿 $\\mathrm{AB}$ 方向匀速运动, 速度为 $1 \\mathrm{~cm} / \\mathrm{s}$; 过点 $\\mathrm{P}$ 作直线 $\\mathrm{PF} / / \\mathrm{AD}, \\mathrm{PF}$ 交 $\\mathrm{CD}$ 于点 $\\mathrm{F}$, 过点 $F$ 作 $E F \\perp B D$, 且与 $A D 、 B D$ 分别交于点 $E 、 Q$; 连接 $P E$, 设点 $P$ 的运动时间为 $t(s)(0\n\n解答下列问题:\n\n(1) 填空: $A B=$ $\\mathrm{cm} ;$\n\n(2) 当 $\\mathrm{t}$ 为何值时, $\\mathrm{PE} / / \\mathrm{BD}$;\n\n(3) 设四边形 APFE 的面积为 $\\mathrm{y}\\left(\\mathrm{cm}^{2}\\right)$\n\n(1)求 $\\mathrm{y}$ 与 $\\mathrm{t}$ 之间的函数关系式;\n\n(2)若用 $\\mathrm{S}$ 表示图形的面积, 则是否存在某一时刻 $\\mathrm{t}$, 使得 $\\mathrm{S}_{\\text {四边抳 } A P \\mathrm{APF}^{2}=}{ }^{\\frac{8}{25}} \\mathrm{~S}$ 㭛 $\\mathrm{ABCD}$ ?若存在, 求出 $\\mathrm{t}$ 的值; 若不存在, 请说明理由.", "options": [], "subject": "组合几何学", "analysis": " (1) 10\n\n(2) 解: $\\because$ 在菱形 $\\mathrm{ABCD}$ 中, $\\therefore \\mathrm{AB} / / \\mathrm{CD}, \\angle \\mathrm{ADB}=\\angle \\mathrm{CDB}$,又 $\\because \\mathrm{PF} / / \\mathrm{AD}$,\n\n$\\therefore$ 四边形 APFD 为平行四边形,\n\n$\\therefore \\mathrm{DF}=\\mathrm{AP}=\\mathrm{t}$,\n\n又 $\\because \\mathrm{EF} \\perp \\mathrm{BD}$ 于 $\\mathrm{Q}$, 且 $\\angle \\mathrm{ADB}=\\angle \\mathrm{CDB}$,\n\n$\\therefore \\angle \\mathrm{DEF}=\\angle \\mathrm{DFE}$,\n\n$\\therefore \\mathrm{DE}=\\mathrm{DF}=\\mathrm{t}$,\n\n$\\therefore \\mathrm{AE}=10-\\mathrm{t}$,\n\n当 $P E / / B D$ 时, $\\triangle A P E \\sim \\triangle A B D$,\n\n$\\therefore \\frac{A P}{A B}=\\frac{A E}{A D}$,\n\n$\\therefore{ }^{\\frac{t}{10}=\\frac{10-t}{10}}$,\n\n$\\therefore \\mathrm{t}=5$,\n\n$\\therefore$ 当 $\\mathrm{t}=5$ 时, $\\mathrm{PE} / / \\mathrm{BD}$\n\n(3) 蛸: (1) $\\because \\angle \\mathrm{FDQ}=\\angle \\mathrm{CDO}, \\angle \\mathrm{FQD}=\\angle \\mathrm{COD}=90^{\\circ}$,\n\n$\\therefore \\triangle \\mathrm{DFQ} \\backsim \\triangle \\mathrm{DCO}$.\n\n$\\therefore \\frac{Q F}{O C}=\\frac{D F}{D C}$,\n即 $\\frac{Q F}{6}=\\frac{t}{10}$ ,\n\n$\\therefore Q F=\\frac{3 t}{5}$\n\n$\\therefore E F=2 Q F=\\frac{6 t}{5}$,\n\n同理, $Q D=\\frac{4 t}{5}$\n\n如图, 过点 $\\mathrm{C}$ 作 $\\mathrm{CG} \\perp \\mathrm{AB}$ 于点 $\\mathrm{G}$,\n\n\n\n$\\because \\mathrm{S}_{\\text {粪形 }} \\mathrm{ABCD}=\\mathrm{AB} \\cdot \\mathrm{CG}={ }^{\\frac{1}{2}} \\mathrm{AC} \\cdot \\mathrm{BD}$,\n\n即 $10 \\cdot \\mathrm{CG}==^{\\frac{1}{2}} \\times 12 \\times 16$,\n\n$\\therefore \\mathrm{CG}={ }^{\\frac{1}{2}}$.\n\n\n\n\n\n$\\therefore y=\\frac{4 \\pi}{5}-\\frac{12 t^{2}}{25}$,\n\n(2)当 $S_{\\text {四边形 } A P F E}={ }^{\\frac{8}{25}} S_{\\text {菱形 } A B C D}$\n\n则 $\\frac{48 t}{5}-\\frac{12 t^{2}}{25}=\\frac{8}{25} \\times\\left(\\frac{1}{2} \\times 12 \\times 16\\right)$,\n即 $t^{2}-20 t+64=0$,\n\n解这个方程, 得 $t_{1}=4, t_{2}=16>10$ (不合, 舍去)\n\n$\\therefore$ 存在 $\\mathrm{t}=4 \\mathrm{~s}$, 使得 $\\mathrm{S}_{\\text {四边形 } \\mathrm{APFE}}={ }^{\\frac{8}{25}} \\mathrm{~S}_{\\text {菱形 } \\mathrm{ABCD}} \\cdot$\n\n解: (1) $\\because$ 在菱形 $\\mathrm{ABCD}$ 中, 对角线 $\\mathrm{AC}, \\mathrm{BD}$ 相交于点 $\\mathrm{O}$, 且 $\\mathrm{AC}=12 \\mathrm{~cm}, \\mathrm{BD}=16 \\mathrm{~cm}$,\n\n$\\therefore \\mathrm{BO}=\\mathrm{DO}=8 \\mathrm{~cm}, \\quad \\mathrm{AO}=\\mathrm{CO}=6 \\mathrm{~cm}$,\n\n$\\therefore \\mathrm{AB}=\\sqrt{8^{2}+6^{2}}=10(\\mathrm{~cm})$,\n\n故答案为: 10 ;"} {"id": "5791", "image": ["7193.jpg"], "answer": " (1) 解: $\\because \\mathrm{DF} / / \\mathrm{BE}$,\n\n$\\therefore \\angle \\mathrm{DFE}=\\angle \\mathrm{BEF}$.\n\n$\\because \\angle \\mathrm{AFD}+\\angle \\mathrm{DFE}=180^{\\circ}, \\angle \\mathrm{CEB}+\\angle \\mathrm{BEF}=180^{\\circ}$,\n\n$\\therefore \\angle \\mathrm{AFD}=\\angle \\mathrm{CEB}$.\n\n又 $\\because \\mathrm{AF}=\\mathrm{CE}, \\mathrm{DF}=\\mathrm{BE}$,\n$\\therefore \\triangle \\mathrm{AFD} \\cong \\triangle \\mathrm{CEB}(\\mathrm{SAS})$\n\n(2) 解: 由 (1) 知 $\\triangle \\mathrm{AFD} \\cong \\triangle \\mathrm{CEB}$,\n\n$\\therefore \\angle \\mathrm{DAC}=\\angle \\mathrm{BCA}, \\mathrm{AD}=\\mathrm{BC}$\n\n$\\therefore \\mathrm{AD} / / \\mathrm{BC}$.\n\n$\\therefore$ 四边形 $\\mathrm{ABCD}$ 是平行四边形 (一组对边平行且相等的四边形是平行四边形)", "solution": "null", "level": "八年级", "question": "( 10 分 ) 如图, $\\mathrm{E}, \\mathrm{F}$ 是四边形 $\\mathrm{ABCD}$ 的对角线 $\\mathrm{AC}$ 上两点, $\\mathrm{AF}=\\mathrm{CE}, \\mathrm{DF}=\\mathrm{BE}, \\mathrm{DF} / / \\mathrm{BE}$.求证:\n\n\n\n(1) $\\triangle \\mathrm{AFD} \\cong \\triangle \\mathrm{CEB}$;\n\n(2) 四边形 $\\mathrm{ABCD}$ 是平行四边形.", "options": [], "subject": "组合几何学", "analysis": " (1) 解: $\\because \\mathrm{DF} / / \\mathrm{BE}$,\n\n$\\therefore \\angle \\mathrm{DFE}=\\angle \\mathrm{BEF}$.\n\n$\\because \\angle \\mathrm{AFD}+\\angle \\mathrm{DFE}=180^{\\circ}, \\angle \\mathrm{CEB}+\\angle \\mathrm{BEF}=180^{\\circ}$,\n\n$\\therefore \\angle \\mathrm{AFD}=\\angle \\mathrm{CEB}$.\n\n又 $\\because \\mathrm{AF}=\\mathrm{CE}, \\mathrm{DF}=\\mathrm{BE}$,\n$\\therefore \\triangle \\mathrm{AFD} \\cong \\triangle \\mathrm{CEB}(\\mathrm{SAS})$\n\n(2) 解: 由 (1) 知 $\\triangle \\mathrm{AFD} \\cong \\triangle \\mathrm{CEB}$,\n\n$\\therefore \\angle \\mathrm{DAC}=\\angle \\mathrm{BCA}, \\mathrm{AD}=\\mathrm{BC}$\n\n$\\therefore \\mathrm{AD} / / \\mathrm{BC}$.\n\n$\\therefore$ 四边形 $\\mathrm{ABCD}$ 是平行四边形 (一组对边平行且相等的四边形是平行四边形)"} {"id": "5820", "image": ["7216.jpg", "7217.jpg", "7217.jpg"], "answer": "解: 连接 $\\mathrm{AC}$ 交 $\\mathrm{BD}$ 于点 $\\mathrm{O}$, 连接 $\\mathrm{AF} 、 \\mathrm{CE}$\n\n\n$\\because$ 四边形 $\\mathrm{ABCD}$ 是平行四边形\n\n$\\therefore \\mathrm{OA}=\\mathrm{OC}, \\mathrm{OB}=\\mathrm{OD}$\n\n$\\because \\mathrm{OF}=\\mathrm{BF}-\\mathrm{OB}, \\quad \\mathrm{OE}=\\mathrm{DE}-\\mathrm{OD}, \\quad \\mathrm{BF}=\\mathrm{DE}$\n\n$\\therefore \\mathrm{OE}=\\mathrm{OF}$\n\n$\\because \\mathrm{OA}=\\mathrm{OC}, \\quad \\mathrm{OE}=\\mathrm{OF}$\n\n$\\therefore$ 四边形 $\\mathrm{AECF}$ 是平行四边形\n\n$\\therefore \\mathrm{AE}=\\mathrm{CF}$", "solution": "null", "level": "八年级", "question": "( 5 分 ) 已知, 如图, 在 $\\mathrm{ABCD}$ 中, $\\mathrm{E} 、 \\mathrm{~F}$ 是对角线 $\\mathrm{BD}$ 上的两点, 且 $\\mathrm{BF}=\\mathrm{DE}$. 求证: $\\mathrm{AE}=\\mathrm{CF}$.\n\n", "options": [], "subject": "组合几何学", "analysis": "解: 连接 $\\mathrm{AC}$ 交 $\\mathrm{BD}$ 于点 $\\mathrm{O}$, 连接 $\\mathrm{AF} 、 \\mathrm{CE}$\n\n\n$\\because$ 四边形 $\\mathrm{ABCD}$ 是平行四边形\n\n$\\therefore \\mathrm{OA}=\\mathrm{OC}, \\mathrm{OB}=\\mathrm{OD}$\n\n$\\because \\mathrm{OF}=\\mathrm{BF}-\\mathrm{OB}, \\quad \\mathrm{OE}=\\mathrm{DE}-\\mathrm{OD}, \\quad \\mathrm{BF}=\\mathrm{DE}$\n\n$\\therefore \\mathrm{OE}=\\mathrm{OF}$\n\n$\\because \\mathrm{OA}=\\mathrm{OC}, \\quad \\mathrm{OE}=\\mathrm{OF}$\n\n$\\therefore$ 四边形 $\\mathrm{AECF}$ 是平行四边形\n\n$\\therefore \\mathrm{AE}=\\mathrm{CF}$"} {"id": "5000", "image": ["6611.jpg", "6612.jpg", "6612.jpg"], "answer": "(1)根据正方形的面积公式求得边长;(2)先求出直角三角形$BFG、ABD$的面积,然后用两个正方形的面积减去两个直角三角形的面积,这就是阴影部分的面积.【解析】解:(1)正方形$ABCD$的边长为:$BC=sqrt{8}=2sqrt{2}$,正方形$ECFG$的边长为:$CF=sqrt{32}=4sqrt{2}$;(2)$becauseBF=BC+CF,BC=2sqrt{2},CF=4sqrt{2}$,$thereforeBF=6sqrt{2}$;$thereforeS_{triangleBFG}=frac{1}{2}GFcdotBF=24;$又$S_{triangleABD}=frac{1}{2}ABcdotAD=4$,$=8+32-24-4$,$=12$.【点睛】本题主要考查了二次根式的应用,正方形的性质,三角形的面积.第(2)题关键是把阴影部分面积转化为正方形与三角形的面积进行计算.", "solution": "null", "level": "八年级", "question": "如图,正方形$ABCD$的面积为8,正方形$ECFG$的面积为32.(1)求正方形$ABCD$和正方形$ECFG$的边长;(2)求阴影部分的面积.", "options": [], "subject": "组合几何学", "analysis": "(1)根据正方形的面积公式求得边长;(2)先求出直角三角形$BFG、ABD$的面积,然后用两个正方形的面积减去两个直角三角形的面积,这就是阴影部分的面积.【解析】解:(1)正方形$ABCD$的边长为:$BC=sqrt{8}=2sqrt{2}$,正方形$ECFG$的边长为:$CF=sqrt{32}=4sqrt{2}$;(2)$becauseBF=BC+CF,BC=2sqrt{2},CF=4sqrt{2}$,$thereforeBF=6sqrt{2}$;$thereforeS_{triangleBFG}=frac{1}{2}GFcdotBF=24;$又$S_{triangleABD}=frac{1}{2}ABcdotAD=4$,$=8+32-24-4$,$=12$.【点睛】本题主要考查了二次根式的应用,正方形的性质,三角形的面积.第(2)题关键是把阴影部分面积转化为正方形与三角形的面积进行计算."} {"id": "5133", "image": ["6650.jpg"], "answer": "可由题中条件判断出$triangleABFcongtriangleCDE$,得出$angleCED=angleAFB$,即$angleDEC=angleBFA$,进而可求证$AF//CE$.【解析】证明:$because$四边形$ABDC$是平行四边形,$thereforeAB=CD,AB//CD$,$thereforeangleABF=angleCDE$,在$triangleABF$与$triangleCDE$中,$left{begin{array}{l}{AB}={CD}angle{ABF}=angle{CDE},{BF}={DE}end{array}right.$$thereforeCDEcongtriangleABF(SAS)$,$thereforeangleCED=angleAFB$,$thereforeangleDEB=angleCFA$,$thereforeAF//DE$.【点睛】本题考查了平行四边形的性质,全等三角形的判定与性质,全等三角形的判定是结合全等三角形的性质证明线段和角相等的重要工具.在判定三角形全等时,关键是选择恰当的判定条件.", "solution": "null", "level": "八年级", "question": "如图,已知平行四边形$ABDC$中,$E、F$是对角线$BC$上两点,且满足$BF=DE$.求证:$AF//CE$.", "options": [], "subject": "组合几何学", "analysis": "可由题中条件判断出$triangleABFcongtriangleCDE$,得出$angleCED=angleAFB$,即$angleDEC=angleBFA$,进而可求证$AF//CE$.【解析】证明:$because$四边形$ABDC$是平行四边形,$thereforeAB=CD,AB//CD$,$thereforeangleABF=angleCDE$,在$triangleABF$与$triangleCDE$中,$left{begin{array}{l}{AB}={CD}angle{ABF}=angle{CDE},{BF}={DE}end{array}right.$$thereforeCDEcongtriangleABF(SAS)$,$thereforeangleCED=angleAFB$,$thereforeangleDEB=angleCFA$,$thereforeAF//DE$.【点睛】本题考查了平行四边形的性质,全等三角形的判定与性质,全等三角形的判定是结合全等三角形的性质证明线段和角相等的重要工具.在判定三角形全等时,关键是选择恰当的判定条件."} {"id": "5160", "image": ["6700.jpg", "6701.jpg", "6702.jpg", "6703.jpg"], "answer": "(1)根据图(1)(2)(3)进行推导.(2)根据特殊到一般的数学思想解决本题.(3)由$(n-1)$个三角形的内角的和为$180^{circ}(n-1)$,得凸$n$边形的内角和为$180^{circ}(n-1)$$-180^{circ}=(n-2)times180^{circ}$.(4)设加上的某个外角的度数为$xleft(0(1)(2)(3)(4)", "options": [], "subject": "组合几何学", "analysis": "(1)根据图(1)(2)(3)进行推导.(2)根据特殊到一般的数学思想解决本题.(3)由$(n-1)$个三角形的内角的和为$180^{circ}(n-1)$,得凸$n$边形的内角和为$180^{circ}(n-1)$$-180^{circ}=(n-2)times180^{circ}$.(4)设加上的某个外角的度数为$xleft(0", "options": [], "subject": "组合几何学", "analysis": "(1)先由对角线互相平分的四边形$ABCD$是平行四边形,再由矩形的性质得出$BDperpAC$,即可得出结论;(2)由矩形的性质得出$AB=OE=10$,由菱形的性质得出$OB=OD,angleAOB=90^{circ},OC、OB$的长,然后由菱形的面积公式即可得出结果.【解析】(1)证明:$becauseCO=AO,DO=BO$,$therefore$四边形$ABCD$是平行四边形,$because$四边形$AEBO$是矩形,$thereforeangleAOB=90^{circ}$,$thereforeBDperpAC$,$therefore$四边形$ABCD$是菱形;(2)解:$because$四边形$AEBO$是矩形,$thereforeAB=BC=OE=10$,$because$四边形$ABCD$是菱形,$angleBCD=60^{circ}$,$thereforeangleBCO=30^{circ},angleAOB=90^{circ}$$thereforeOB=frac{1}{2}BC=frac{1}{2}times10=5$,在Rt$triangleBOC$中,由勾股定理得:$OC=sqrt{BC^{2}-OB^{2}}=sqrt{10^{2}-5^{2}}=5sqrt{3}$,$thereforeBD=2OB=2times5=10,AC=2OC=2times5sqrt{3}=10sqrt{3}$,$thereforeS_{text{麦形}ABCD}=frac{1}{2}ACcdotBD=frac{1}{2}times10times10sqrt{3}=50sqrt{3}$.【点睛】本题考查了平行四边形的判定、菱形的判定与性质、矩形的性质、勾股定理等知识;熟练掌握菱形的判定与性质是解题的关键."} {"id": "4089", "image": ["5770.jpg"], "answer": "证明: $\\because B E=C F$,\n\n$\\therefore B E+E C=C F+E C$ 即 $B C=E F$,\n\n在 $\\triangle A B C$ 和 $\\triangle D E F$ 中\n\n$\\left\\{\\begin{array}{l}A B=D E \\\\ B C=E F \\\\ A C=D F\\end{array}\\right.$\n\n$\\therefore \\triangle A B C \\xlongequal{\\cong} \\triangle D E F$\n\n$\\therefore \\angle A=\\angle D$", "solution": "null", "level": "八年级", "question": "如图, 已知点 $E 、 C$ 在线段 $B F$ 上, $B E=C F, A B=D E, A C=D F$.\n\n求证: $\\angle A=\\angle D$\n\n", "options": [], "subject": "组合几何学", "analysis": "证明: $\\because B E=C F$,\n\n$\\therefore B E+E C=C F+E C$ 即 $B C=E F$,\n\n在 $\\triangle A B C$ 和 $\\triangle D E F$ 中\n\n$\\left\\{\\begin{array}{l}A B=D E \\\\ B C=E F \\\\ A C=D F\\end{array}\\right.$\n\n$\\therefore \\triangle A B C \\xlongequal{\\cong} \\triangle D E F$\n\n$\\therefore \\angle A=\\angle D$"} {"id": "4090", "image": ["5771.jpg"], "answer": "解: $\\because D E$ 垂直平分 $A C$,\n\n$\\therefore E A=E C$,\n\n$\\therefore \\angle C=\\angle E A C$,\n\n设 $\\angle C=x^{\\circ}$, 则 $\\angle E A C=X^{\\circ}, \\angle F A C=(x+18)^{\\circ}$,\n\n$\\because A F$ 平分 $\\angle B A C$,\n\n$\\therefore \\angle B A C=2 \\angle F A C=(2 x+36)^{\\circ}$,\n\n$\\therefore 72+2 x+36+x=180$,\n\n解得: $x=24$,\n\n$\\therefore \\angle C=24^{\\circ}$.", "solution": "null", "level": "八年级", "question": "如图, 在 $\\triangle A B C$ 中, $A F$ 平分 $\\angle B A C, A C$ 的垂直平分线交 $B C$ 于点 $E, \\angle B=72^{\\circ}, \\angle F A E=18^{\\circ}$,求 $\\angle C$ 的度数.\n\n", "options": [], "subject": "组合几何学", "analysis": "解: $\\because D E$ 垂直平分 $A C$,\n\n$\\therefore E A=E C$,\n\n$\\therefore \\angle C=\\angle E A C$,\n\n设 $\\angle C=x^{\\circ}$, 则 $\\angle E A C=X^{\\circ}, \\angle F A C=(x+18)^{\\circ}$,\n\n$\\because A F$ 平分 $\\angle B A C$,\n\n$\\therefore \\angle B A C=2 \\angle F A C=(2 x+36)^{\\circ}$,\n\n$\\therefore 72+2 x+36+x=180$,\n\n解得: $x=24$,\n\n$\\therefore \\angle C=24^{\\circ}$."} {"id": "4091", "image": ["5772.jpg", "5773.jpg", "5773.jpg"], "answer": "证明:连接 $C D$,\n\n\n\n在 $\\triangle A D C$ 和 $\\triangle B C D$ 中,\n\n$\\left\\{\\begin{array}{l}A D=B C \\\\ D C=C D, \\\\ A C=B D\\end{array}\\right.$\n\n$\\therefore \\triangle A D C \\xlongequal{\\triangle} \\triangle B C D(\\mathrm{SSS})$,\n\n$\\therefore \\angle A=\\angle B, \\angle A C D=\\angle B D C$\n\n$\\therefore O D=O C$.", "solution": "null", "level": "八年级", "question": "已知: 如图, $A D=B C, A C=B D, A C$ 与 $B D$ 相交于点 $O$, 求证: $\\angle A=\\angle B, O D=O$.\n\n", "options": [], "subject": "组合几何学", "analysis": "证明:连接 $C D$,\n\n\n\n在 $\\triangle A D C$ 和 $\\triangle B C D$ 中,\n\n$\\left\\{\\begin{array}{l}A D=B C \\\\ D C=C D, \\\\ A C=B D\\end{array}\\right.$\n\n$\\therefore \\triangle A D C \\xlongequal{\\triangle} \\triangle B C D(\\mathrm{SSS})$,\n\n$\\therefore \\angle A=\\angle B, \\angle A C D=\\angle B D C$\n\n$\\therefore O D=O C$."} {"id": "4094", "image": ["5777.jpg"], "answer": "(1) 解: $\\because \\angle B A C=80^{\\circ}, \\angle A C B=40^{\\circ}, A E, C D$ 分别是 $\\angle B A C, \\angle A C B$ 的平分线,\n\n$\\therefore \\angle F A C=40^{\\circ}, \\angle F C A=20^{\\circ}$,\n\n$\\therefore \\angle A F C=180^{\\circ}-\\angle F A C^{-} \\angle F C A=120^{\\circ}$.\n\n(2)解: $\\because \\angle B=80^{\\circ}$,\n\n$\\therefore \\angle B A C^{+} \\angle B C A=100^{\\circ}$,\n\n$\\because A E, C D$ 分别是 $\\angle B A C, \\angle A C B$ 的平分线,\n\n$\\therefore \\angle F A C+\\angle F C A=50^{\\circ}$,\n\n$\\therefore \\angle A F C=130^{\\circ}$.\n\n(3)解: $\\because \\angle B=x^{\\circ}$ ,\n\n$\\therefore \\angle B A C+\\angle B C A=180^{\\circ}-x^{\\circ}$,\n\n$\\because A E, C D$ 分别是 $\\angle B A C, \\angle A C B$ 的平分线,\n\n$\\therefore \\angle F A C+\\angle F C A=\\frac{1}{2}\\left(180^{\\circ}-x^{\\circ}\\right)$,\n$\\therefore \\angle A F C=180^{\\circ}-(\\angle F A C+\\angle F C A)=180^{\\circ}-\\frac{1}{2}\\left(180^{\\circ}-x^{\\circ}\\right)=90^{\\circ}+\\frac{1}{2} x^{\\circ}$.", "solution": "null", "level": "八年级", "question": "如图, 在 $\\triangle A B C$ 中, $A E, C D$ 分别是 $\\angle B A C, \\angle A C B$ 的平分线, 且 $A E, C D$ 相交于点 $F$.\n\n\n\n(1) 若 $\\angle B A C=80^{\\circ}, \\angle A C B=40^{\\circ}$, 求 $\\angle A F C$ 的度数;\n\n(2) 若 $\\angle B=80^{\\circ}$, 求 $\\angle A F C$ 的度数;\n\n(3) 若 $\\angle B=x^{\\circ}$ ,用含 $x$ 的代数式表示 $\\angle A F C$ 的度数.", "options": [], "subject": "组合几何学", "analysis": "(1) 解: $\\because \\angle B A C=80^{\\circ}, \\angle A C B=40^{\\circ}, A E, C D$ 分别是 $\\angle B A C, \\angle A C B$ 的平分线,\n\n$\\therefore \\angle F A C=40^{\\circ}, \\angle F C A=20^{\\circ}$,\n\n$\\therefore \\angle A F C=180^{\\circ}-\\angle F A C^{-} \\angle F C A=120^{\\circ}$.\n\n(2)解: $\\because \\angle B=80^{\\circ}$,\n\n$\\therefore \\angle B A C^{+} \\angle B C A=100^{\\circ}$,\n\n$\\because A E, C D$ 分别是 $\\angle B A C, \\angle A C B$ 的平分线,\n\n$\\therefore \\angle F A C+\\angle F C A=50^{\\circ}$,\n\n$\\therefore \\angle A F C=130^{\\circ}$.\n\n(3)解: $\\because \\angle B=x^{\\circ}$ ,\n\n$\\therefore \\angle B A C+\\angle B C A=180^{\\circ}-x^{\\circ}$,\n\n$\\because A E, C D$ 分别是 $\\angle B A C, \\angle A C B$ 的平分线,\n\n$\\therefore \\angle F A C+\\angle F C A=\\frac{1}{2}\\left(180^{\\circ}-x^{\\circ}\\right)$,\n$\\therefore \\angle A F C=180^{\\circ}-(\\angle F A C+\\angle F C A)=180^{\\circ}-\\frac{1}{2}\\left(180^{\\circ}-x^{\\circ}\\right)=90^{\\circ}+\\frac{1}{2} x^{\\circ}$."} {"id": "4114", "image": ["5795.jpg"], "answer": "证明: $\\because C=B F$\n\n$\\therefore C E+E F=B F+E F$\n\n$\\therefore C F=B E$\n\n在 $\\triangle A B E$ 和 $\\triangle C D F$ 中 $\\left\\{\\begin{array}{l}C F=B E \\\\ \\angle B=\\angle C \\\\ A B=D C\\end{array}\\right.$\n\n$\\therefore \\triangle A B E \\cong \\triangle C D F$\n\n$\\therefore A E=D F$", "solution": "null", "level": "八年级", "question": "如图, 点 $C, E, F, B$ 在同一条直线上, $C E=B F, A B=D C, \\angle B=\\angle C$, 证明: $A E=D F$.\n\n", "options": [], "subject": "组合几何学", "analysis": "证明: $\\because C=B F$\n\n$\\therefore C E+E F=B F+E F$\n\n$\\therefore C F=B E$\n\n在 $\\triangle A B E$ 和 $\\triangle C D F$ 中 $\\left\\{\\begin{array}{l}C F=B E \\\\ \\angle B=\\angle C \\\\ A B=D C\\end{array}\\right.$\n\n$\\therefore \\triangle A B E \\cong \\triangle C D F$\n\n$\\therefore A E=D F$"} {"id": "4138", "image": ["5820.jpg"], "answer": "证明: (1) $\\because A C \\perp B C, B D \\perp A D$,\n\n$\\therefore \\triangle A B C$ 与 $\\triangle B A D$ 是直角三角形,\n\n在 $\\triangle A B C$ 和 $\\triangle B A D$ 中, $\\because A C=B D, A B=B A, \\angle A C B=\\angle B D A=90^{\\circ}$,\n\n$\\therefore \\triangle A B C \\cong \\triangle B A D(H L)$\n\n$\\therefore B C=A D$.\n\n(2) $\\because \\triangle A B C \\cong \\triangle B A D$\n$\\therefore \\angle C A B=\\angle D B A$\n\n$\\therefore O A=O B$.\n\n$\\therefore \\triangle O A B$ 是等腰三角形.", "solution": "null", "level": "八年级", "question": "如图, 已知 $A C \\perp B C, B D \\perp A D, A C$ 与 $B D$ 交于 $O, A C=B D$.\n\n求证: (1) $B C=A D$;\n\n(2) $\\triangle O A B$ 是等腰三角形.\n\n", "options": [], "subject": "组合几何学", "analysis": "证明: (1) $\\because A C \\perp B C, B D \\perp A D$,\n\n$\\therefore \\triangle A B C$ 与 $\\triangle B A D$ 是直角三角形,\n\n在 $\\triangle A B C$ 和 $\\triangle B A D$ 中, $\\because A C=B D, A B=B A, \\angle A C B=\\angle B D A=90^{\\circ}$,\n\n$\\therefore \\triangle A B C \\cong \\triangle B A D(H L)$\n\n$\\therefore B C=A D$.\n\n(2) $\\because \\triangle A B C \\cong \\triangle B A D$\n$\\therefore \\angle C A B=\\angle D B A$\n\n$\\therefore O A=O B$.\n\n$\\therefore \\triangle O A B$ 是等腰三角形."} {"id": "4140", "image": ["5821.jpg"], "answer": "解: (1) $\\because A E$ 和 $B D$ 相交于点 $O$,\n\n$\\therefore \\angle A O D=\\angle B O E$.\n\n在 $\\triangle A O D$ 和 $\\triangle B O E$ 中,\n\n$\\because \\angle A=\\angle B$,\n\n$\\therefore \\angle B E O=\\angle 2$.\n\n又 $\\because \\angle 1=\\angle 2$,\n\n$\\therefore \\angle 1=\\angle B E O$,\n\n$\\therefore \\angle A E C=\\angle B E D$.\n\n在 $\\triangle A E C$ 和 $\\triangle B E D$ 中,\n\n$\\because\\left\\{\\begin{array}{l}\\angle A=\\angle B \\\\ A E=B E \\\\ \\angle A E C=\\angle B E D\\end{array}\\right.$,\n\n$\\therefore \\triangle A E C \\xlongequal{\\cong} \\triangle B E D(A S A)$.\n\n(2) $\\because \\triangle A E C \\cong \\triangle B E D$,\n\n(3) $\\therefore E C=E D, \\angle C=\\angle B D E$.\n\n在 $\\triangle E D C$ 中, $\\because E C=E D, \\angle 1=42^{\\circ}$ ,\n\n$\\therefore \\angle C=\\angle E D C=\\left(180^{\\circ}-42^{\\circ}\\right) \\div 2=69^{\\circ}$,\n\n$\\therefore \\angle B D E=\\angle C=69^{\\circ}$.", "solution": "null", "level": "八年级", "question": "如图, $\\angle A=\\angle B, A E=B E$, 点 $D$ 在 $A C$ 边上, $\\angle 1=\\angle 2, A E$ 和 $B D$ 相交于点 $O$.\n\n\n\n(1) 求证: $\\triangle A E C \\cong \\triangle B E D$;\n\n(2) 若 $\\angle 1=42^{\\circ}$, 求 $\\angle B D E$ 的度数.", "options": [], "subject": "组合几何学", "analysis": "解: (1) $\\because A E$ 和 $B D$ 相交于点 $O$,\n\n$\\therefore \\angle A O D=\\angle B O E$.\n\n在 $\\triangle A O D$ 和 $\\triangle B O E$ 中,\n\n$\\because \\angle A=\\angle B$,\n\n$\\therefore \\angle B E O=\\angle 2$.\n\n又 $\\because \\angle 1=\\angle 2$,\n\n$\\therefore \\angle 1=\\angle B E O$,\n\n$\\therefore \\angle A E C=\\angle B E D$.\n\n在 $\\triangle A E C$ 和 $\\triangle B E D$ 中,\n\n$\\because\\left\\{\\begin{array}{l}\\angle A=\\angle B \\\\ A E=B E \\\\ \\angle A E C=\\angle B E D\\end{array}\\right.$,\n\n$\\therefore \\triangle A E C \\xlongequal{\\cong} \\triangle B E D(A S A)$.\n\n(2) $\\because \\triangle A E C \\cong \\triangle B E D$,\n\n(3) $\\therefore E C=E D, \\angle C=\\angle B D E$.\n\n在 $\\triangle E D C$ 中, $\\because E C=E D, \\angle 1=42^{\\circ}$ ,\n\n$\\therefore \\angle C=\\angle E D C=\\left(180^{\\circ}-42^{\\circ}\\right) \\div 2=69^{\\circ}$,\n\n$\\therefore \\angle B D E=\\angle C=69^{\\circ}$."} {"id": "5758", "image": ["7165.jpg", "7166.jpg", "7165.jpg", "7166.jpg"], "answer": "解: 如图, 等边三角形 $\\triangle \\mathrm{ABC}$ 的边长为 $4 \\sqrt{3}, \\mathrm{AD}$ 为高, $\\because \\triangle \\mathrm{ABC}$ 为等边三角形,\n\n$\\therefore \\angle \\mathrm{B}=60^{\\circ}$,\n\n在 Rt $\\triangle \\mathrm{ABD}$ 中, $\\mathrm{AB}=4 \\sqrt{3}$,\n\n$\\because \\sin \\mathrm{B}=\\frac{A D}{A B}$\n\n\n\n即它的一条边上的高为 6 .\n\n", "solution": "null", "level": "八年级", "question": "(5 分) 已知正三角形的边长为 $4 \\sqrt{3}$, 求它的一条边上的高.", "options": [], "subject": "度量几何学", "analysis": "解: 如图, 等边三角形 $\\triangle \\mathrm{ABC}$ 的边长为 $4 \\sqrt{3}, \\mathrm{AD}$ 为高, $\\because \\triangle \\mathrm{ABC}$ 为等边三角形,\n\n$\\therefore \\angle \\mathrm{B}=60^{\\circ}$,\n\n在 Rt $\\triangle \\mathrm{ABD}$ 中, $\\mathrm{AB}=4 \\sqrt{3}$,\n\n$\\because \\sin \\mathrm{B}=\\frac{A D}{A B}$\n\n\n\n即它的一条边上的高为 6 .\n\n"} {"id": "5760", "image": ["7167.jpg"], "answer": "解: $\\sqrt{(a-b)^{2}}-|b+c|-\\sqrt{(b-c)^{2}}$\n\n$=|a-b|-|b+c|-|b-c|$\n\n$=b-a+b+c-b+c$\n\n$=b-a+2 c$", "solution": "null", "level": "八年级", "question": "( 5 分 ) 有理数 $\\mathrm{a} 、 \\mathrm{~b} 、 \\mathrm{c}$ 在数轴上的位置如图所示, 化简 $\\sqrt{(a-b)^{2}}-|b+c|-\\sqrt{(b-c)^{2}}$\n\n", "options": [], "subject": "度量几何学", "analysis": "解: $\\sqrt{(a-b)^{2}}-|b+c|-\\sqrt{(b-c)^{2}}$\n\n$=|a-b|-|b+c|-|b-c|$\n\n$=b-a+b+c-b+c$\n\n$=b-a+2 c$"} {"id": "5761", "image": [], "answer": "因为长方形面积为 $\\sqrt{140 \\pi} \\times \\sqrt{35 \\pi}=\\sqrt{2^{2} \\times 35^{2} \\times \\pi^{2}}=70 \\pi$, 圆的面积等于长方形面积, 不妨设圆的半径为 $\\mathrm{r}$, 于是 $\\pi^{2}=70 \\pi$, 所以 $r=\\sqrt{70}_{\\mathrm{cm}}$.", "solution": "null", "level": "八年级", "question": "(5 分) 方老师想设计一个长方形纸片, 已知长方形的长是 $\\sqrt{140 \\pi} \\mathrm{cm}$, 宽是 $\\sqrt{35 \\pi} \\mathrm{cm}$, 他又想设计一个面积与其相等的圆, 请你帮助方老师求出圆的半径.", "options": [], "subject": "度量几何学", "analysis": "因为长方形面积为 $\\sqrt{140 \\pi} \\times \\sqrt{35 \\pi}=\\sqrt{2^{2} \\times 35^{2} \\times \\pi^{2}}=70 \\pi$, 圆的面积等于长方形面积, 不妨设圆的半径为 $\\mathrm{r}$, 于是 $\\pi^{2}=70 \\pi$, 所以 $r=\\sqrt{70}_{\\mathrm{cm}}$."} {"id": "5786", "image": ["7189.jpg"], "answer": "解:在平行四边新 $\\mathrm{ABCD}$ 中,\n$\\because$ 对角线 $\\mathrm{AC}, \\mathrm{BD}$ 相交于点 $\\mathrm{O}, \\mathrm{MN}$ 是过 $\\mathrm{O}$ 点的直线,\n\n$\\therefore \\mathrm{AN}=\\mathrm{MC} \\mathrm{DN}=\\mathrm{BM}$\n\n$\\because \\mathrm{BC}=\\mathrm{BM}+\\mathrm{MC}$\n\n$\\therefore \\mathrm{BC}=\\mathrm{BM}+\\mathrm{AN}=2+2.8=4.8$\n\n$\\because \\mathrm{AD}=\\mathrm{BC}$\n\n$\\therefore \\mathrm{AD}=4.8$", "solution": "null", "level": "八年级", "question": "( 8 分 ) 如图, 在平行四边形 $A B C D$ 中, 对角线 $A C, B D$ 相交于点 $O, M N$ 是过 $O$ 点的直线, 交 $B C$ 于 $M$, 交 $A D$ 于 $N, B M=2, A N=2.8$, 求 $B C$ 和 $A D$ 的长.\n\n", "options": [], "subject": "度量几何学", "analysis": "解:在平行四边新 $\\mathrm{ABCD}$ 中,\n$\\because$ 对角线 $\\mathrm{AC}, \\mathrm{BD}$ 相交于点 $\\mathrm{O}, \\mathrm{MN}$ 是过 $\\mathrm{O}$ 点的直线,\n\n$\\therefore \\mathrm{AN}=\\mathrm{MC} \\mathrm{DN}=\\mathrm{BM}$\n\n$\\because \\mathrm{BC}=\\mathrm{BM}+\\mathrm{MC}$\n\n$\\therefore \\mathrm{BC}=\\mathrm{BM}+\\mathrm{AN}=2+2.8=4.8$\n\n$\\because \\mathrm{AD}=\\mathrm{BC}$\n\n$\\therefore \\mathrm{AD}=4.8$"} {"id": "5790", "image": ["7192.jpg"], "answer": "证明: $\\because$ 四边形 $\\mathrm{ABCD}$ 是平行四边形\n\n$\\therefore \\mathrm{AD}=\\mathrm{BC}, \\quad \\mathrm{AD} / / \\mathrm{BC}$\n\n$\\because E 、 F$ 分别是 $A D 、 B C$ 的中点\n\n$\\therefore \\mathrm{DE}={ }^{\\frac{1}{2}} \\mathrm{AD}, \\mathrm{BF}={ }^{\\frac{1}{2}} \\mathrm{BC}$\n\n$\\therefore \\mathrm{DE}=\\mathrm{BF}$\n\n$\\therefore$ 四边形 $\\mathrm{BFDE}$ 是平行四边形\n\n$\\therefore \\mathrm{BE}=\\mathrm{DF}$", "solution": "null", "level": "八年级", "question": "( 5 分 ) 已知:如图, 在平行四边形 $A B C D$ 中, $E 、 F$ 分别是 $A D 、 B C$ 的中点, 求证: $B E=D F$.\n\n", "options": [], "subject": "度量几何学", "analysis": "证明: $\\because$ 四边形 $\\mathrm{ABCD}$ 是平行四边形\n\n$\\therefore \\mathrm{AD}=\\mathrm{BC}, \\quad \\mathrm{AD} / / \\mathrm{BC}$\n\n$\\because E 、 F$ 分别是 $A D 、 B C$ 的中点\n\n$\\therefore \\mathrm{DE}={ }^{\\frac{1}{2}} \\mathrm{AD}, \\mathrm{BF}={ }^{\\frac{1}{2}} \\mathrm{BC}$\n\n$\\therefore \\mathrm{DE}=\\mathrm{BF}$\n\n$\\therefore$ 四边形 $\\mathrm{BFDE}$ 是平行四边形\n\n$\\therefore \\mathrm{BE}=\\mathrm{DF}$"} {"id": "5794", "image": [], "answer": " (1) 解:设多边形的边数为 $n$,\n\n由题意得, $(n-2) \\cdot 180^{\\circ}=5 \\times 360^{\\circ}$,\n\n解得 $n=12$,\n\n所以, 这个多边形是十二边形;\n\n(2) 解: 根据题意, 得:\n\n$(n-2) \\cdot 180=1440^{\\circ}$,\n\n解得 $\\mathrm{n}=10$.\n\n所以它的边长是 $100 \\div 10=10$.\n\n故答案为: 10 。", "solution": "null", "level": "八年级", "question": "(10 分 $)$ 按要求完成下列各小题.\n\n(1) 若一个多边形的内角和是外角和的 5 倍, 求这个多边形的边数;\n\n(2) 若一个正多边形的周长是 100 , 且内角和为 $1440^{\\circ}$, 求该正多边形的边长.", "options": [], "subject": "度量几何学", "analysis": " (1) 解:设多边形的边数为 $n$,\n\n由题意得, $(n-2) \\cdot 180^{\\circ}=5 \\times 360^{\\circ}$,\n\n解得 $n=12$,\n\n所以, 这个多边形是十二边形;\n\n(2) 解: 根据题意, 得:\n\n$(n-2) \\cdot 180=1440^{\\circ}$,\n\n解得 $\\mathrm{n}=10$.\n\n所以它的边长是 $100 \\div 10=10$.\n\n故答案为: 10 。"} {"id": "5022", "image": ["6614.jpg", "6615.jpg", "6615.jpg"], "answer": "直接利用勾股定理进而得出$AC$的长,再结合勾股定理逆定理得出$triangleACD$为直角三角形,分别得出$S_{triangleABC},S_{triangleACD}$,即可得出答案.【解析】解:连结$AC$$becauseABperpCB$,$thereforeAC^{2}=AB^{2}+BC^{2}$,$thereforeAC=sqrt{{AB}^{2}+{BC}^{2}}=sqrt{(sqrt{2})^{2}+(sqrt{2})^{2}}=2$,又$becauseCD=sqrt{3},AD=1$,$thereforeAD^{2}+CD^{2}=1^{2}+(sqrt{3})^{2}=4=2^{2}=AC^{2}$,$thereforetriangleACD$为直角三角形,$thereforeS_{triangleABC}=frac{ABtimesBC}{2}=frac{sqrt{2}timessqrt{2}}{2}=1$,$S_{triangleACD}=frac{{AD}times{CD}}{2}=frac{1timessqrt{3}}{2}=frac{sqrt{3}}{2}$,$thereforeS_{text{四边形}ABCD}=S_{triangleABC}+S_{triangleACD}=1+frac{sqrt{3}}{2}$.【点睛】此题主要考查了勾股定理以及勾股定理逆定理,正确得出$S_{triangleABC},S_{triangleACD}$是解题关键.", "solution": "null", "level": "八年级", "question": "如图在四边形$ABCD$中$AB=BC=sqrt{2},CD=sqrt{3},AD=1$且$ABperpCB$,试求四边形$ABCD$的面积(提示:连接$AC$).", "options": [], "subject": "度量几何学", "analysis": "直接利用勾股定理进而得出$AC$的长,再结合勾股定理逆定理得出$triangleACD$为直角三角形,分别得出$S_{triangleABC},S_{triangleACD}$,即可得出答案.【解析】解:连结$AC$$becauseABperpCB$,$thereforeAC^{2}=AB^{2}+BC^{2}$,$thereforeAC=sqrt{{AB}^{2}+{BC}^{2}}=sqrt{(sqrt{2})^{2}+(sqrt{2})^{2}}=2$,又$becauseCD=sqrt{3},AD=1$,$thereforeAD^{2}+CD^{2}=1^{2}+(sqrt{3})^{2}=4=2^{2}=AC^{2}$,$thereforetriangleACD$为直角三角形,$thereforeS_{triangleABC}=frac{ABtimesBC}{2}=frac{sqrt{2}timessqrt{2}}{2}=1$,$S_{triangleACD}=frac{{AD}times{CD}}{2}=frac{1timessqrt{3}}{2}=frac{sqrt{3}}{2}$,$thereforeS_{text{四边形}ABCD}=S_{triangleABC}+S_{triangleACD}=1+frac{sqrt{3}}{2}$.【点睛】此题主要考查了勾股定理以及勾股定理逆定理,正确得出$S_{triangleABC},S_{triangleACD}$是解题关键."} {"id": "5137", "image": ["6658.jpg"], "answer": "(1)(1)由平行线的性质得出$angleOAD=angleOCB$,可证明$triangleAOEcongtriangleCOF$($ASA$);(2)证得$AD=CB$,再由$AD//BC$,即可得出结论;(2)由全等三角形的性质得出$OE=OF$,证出$BE=BF$,由等腰三角形的性质得出$angleOBF=angleOBE$$=32^{circ}$,求出$angleABC=116^{circ}$,则可得出答案.【解析】(1)(1)证明:$becauseAD//BC$,$thereforeangleOAD=angleOCB$,在$triangleAOE$和$triangleCOF$中,$left{begin{array}{l}angle{OAE}=angle{OCF}{AO}={OC}angle{AOE}=angle{COF}end{array}right.$,$thereforetriangleAOEcongtriangleCOF(ASA);$(2)同理可证$triangleAODcongtriangleCOB$,$thereforeAD=CB$,又$becauseAD//BC$,$therefore$四边形$ABCD$为平行四边形;(2)解:$becausetriangleAOEcongtriangleCOF$,$thereforeOE=OF$,$becauseEFperpBD$,$thereforeBE=BF$,$thereforeangleOBF=angleOBE=32^{circ}$,$thereforeangleEBF=64^{circ}$,$becauseAD//BC$$thereforeangleABC=180^{circ}-angleBAD=180^{circ}-100^{circ}=80^{circ}$,$thereforeangleABE=angleABC-angleEBF=80^{circ}-64^{circ}=16^{circ}$.【点睛】本题考查了平行四边形的判定,全等三角形的判定与性质,等腰三角形的性质,平行线的性质,证明$triangleAOEcongtriangleCOF$是解题的关键.", "solution": "null", "level": "八年级", "question": "如图,在四边形$ABCD$中,$AD//BC$,对角线$AC、BD$交于点$O$,且$AO=OC$.(1)求证:(1)$triangleAOEcongtriangleCOF;$(2)四边形$ABCD$为平行四边形;(2)过点$O$作$EFperpBD$,交$AD$于点$E$,交$BC$于点$F$,连接$BE$,若$angleBAD=100^{circ},angleDBF=32^{circ}$,求$angleABE$的度数.", "options": [], "subject": "度量几何学", "analysis": "(1)(1)由平行线的性质得出$angleOAD=angleOCB$,可证明$triangleAOEcongtriangleCOF$($ASA$);(2)证得$AD=CB$,再由$AD//BC$,即可得出结论;(2)由全等三角形的性质得出$OE=OF$,证出$BE=BF$,由等腰三角形的性质得出$angleOBF=angleOBE$$=32^{circ}$,求出$angleABC=116^{circ}$,则可得出答案.【解析】(1)(1)证明:$becauseAD//BC$,$thereforeangleOAD=angleOCB$,在$triangleAOE$和$triangleCOF$中,$left{begin{array}{l}angle{OAE}=angle{OCF}{AO}={OC}angle{AOE}=angle{COF}end{array}right.$,$thereforetriangleAOEcongtriangleCOF(ASA);$(2)同理可证$triangleAODcongtriangleCOB$,$thereforeAD=CB$,又$becauseAD//BC$,$therefore$四边形$ABCD$为平行四边形;(2)解:$becausetriangleAOEcongtriangleCOF$,$thereforeOE=OF$,$becauseEFperpBD$,$thereforeBE=BF$,$thereforeangleOBF=angleOBE=32^{circ}$,$thereforeangleEBF=64^{circ}$,$becauseAD//BC$$thereforeangleABC=180^{circ}-angleBAD=180^{circ}-100^{circ}=80^{circ}$,$thereforeangleABE=angleABC-angleEBF=80^{circ}-64^{circ}=16^{circ}$.【点睛】本题考查了平行四边形的判定,全等三角形的判定与性质,等腰三角形的性质,平行线的性质,证明$triangleAOEcongtriangleCOF$是解题的关键."} {"id": "5138", "image": ["6659.jpg", "6660.jpg", "6660.jpg"], "answer": "(1)结合已知条件推知四边形$AEFD$是平行四边形,在该平行四边形的两条对角线互相平分;(2)根据勾股定理求得$AC$的长度,然后由平行四边形的性质和勾股定理来求$DO$的长度.【解析】解:(1)$becauseE、F$分别是$BC、AC$的中点,$thereforeEF$是$triangleABC$的中位线,$thereforeEF//AB$且$EF=frac{1}{2}AB$.又$AB=2AD$,即$AD=frac{1}{2}AB$,$thereforeAD//EF,AD=EF$,$therefore$四边形$AEFD$是平行四边形,$thereforeAF$与$DE$互相平分;(2)$because$在Rt$triangleABC$中,$angleBAC=90^{circ},AB=8,BC=12$,$therefore$由勾股定理得$AC=sqrt{BC^{2}-{AB}^{2}}=sqrt{12^{2}-8^{2}}=4sqrt{5}$又由(1)知,$OA=OF$,且$AF=CF$,$thereforeOA=frac{1}{4}AC=sqrt{5}$.$therefore$在$triangleAOD$中,$angleDAO=90^{circ},AD=frac{1}{2}AB=4,OA=sqrt{5}$,$therefore$由勾股定理得$DO=sqrt{DA^{2}+OA^{2}}=sqrt{4^{2}+(sqrt{5})^{2}}=sqrt{21}$.【点睛】本题考查了三角形中位线定理,平行四边形的判定与性质.三角形的中位线平行于第三边,且等于第三边的一半.", "solution": "null", "level": "八年级", "question": "在Rt$triangleABC$中,$angleBAC=90^{circ},E、F$分别是$BC、AC$的中点,延长$BA$到点$D$,使$AB=2AD$,连接$DE、DF、AE、EF,AF$与$DE$交于点$O$.$(1)$试说明$AF$与$DE$互相平分;(2)若$AB=8,BC=12$,求$DO$的长.", "options": [], "subject": "度量几何学", "analysis": "(1)结合已知条件推知四边形$AEFD$是平行四边形,在该平行四边形的两条对角线互相平分;(2)根据勾股定理求得$AC$的长度,然后由平行四边形的性质和勾股定理来求$DO$的长度.【解析】解:(1)$becauseE、F$分别是$BC、AC$的中点,$thereforeEF$是$triangleABC$的中位线,$thereforeEF//AB$且$EF=frac{1}{2}AB$.又$AB=2AD$,即$AD=frac{1}{2}AB$,$thereforeAD//EF,AD=EF$,$therefore$四边形$AEFD$是平行四边形,$thereforeAF$与$DE$互相平分;(2)$because$在Rt$triangleABC$中,$angleBAC=90^{circ},AB=8,BC=12$,$therefore$由勾股定理得$AC=sqrt{BC^{2}-{AB}^{2}}=sqrt{12^{2}-8^{2}}=4sqrt{5}$又由(1)知,$OA=OF$,且$AF=CF$,$thereforeOA=frac{1}{4}AC=sqrt{5}$.$therefore$在$triangleAOD$中,$angleDAO=90^{circ},AD=frac{1}{2}AB=4,OA=sqrt{5}$,$therefore$由勾股定理得$DO=sqrt{DA^{2}+OA^{2}}=sqrt{4^{2}+(sqrt{5})^{2}}=sqrt{21}$.【点睛】本题考查了三角形中位线定理,平行四边形的判定与性质.三角形的中位线平行于第三边,且等于第三边的一半."} {"id": "5157", "image": ["6694.jpg", "6695.jpg", "6696.jpg", "6697.jpg", "6695.jpg", "6696.jpg", "6697.jpg"], "answer": "(1)由平行线的性质得$angleADE=angleCBE$,再由$ASA$证明$triangleADEcongtriangleCBE$,即可得出结论;(2)由平行四边形的性质得$AB//CD,AB=CD$,再证$DF=AB$,即可得出结论;(3)过$C$作$CHperpBD$于$H$,过$D$作$DQperpAF$于$Q$,求出高$DQ$和$CH$,再根据面积公式求出即可.【解析】(1)证明:$because$点$E$是$BD$的中点,$thereforeBE=DE$,$becauseAD//BC$$thereforeangleADE=angleCBE$,在$triangleADE$和$triangleCBE$中,$thereforetriangleADEcongtriangleCBE(ASA)$,$thereforeAE=CE$(2)证明:$becauseAE=CE,BE=DE$,$therefore$四边形$ABCD$是平行四边形,$thereforeAB//CD,AB=CD$,$becauseDF=CD$,$thereforeDF=AB$,又$becauseDF//AB$,$therefore$四边形$ABDF$是平行四边形;(3)解:过$C$作$CHperpBD$于$H$,过$D$作$DQperpAF$于$Q$,如图所示:$because$四边形$ABCD$和四边形$ABDF$是平行四边形,$AB=4,AF=8,angleF=30^{circ}$,$thereforeDF=AB=4,CD=AB=4,BD=AF=8,BD//AF$,$thereforeangleBDC=angleF=30^{circ}$,$becauseCHperpBD,DQperpAF$,$thereforeangleCHD=angleDQF=90^{circ}$,$thereforeDQ=frac{1}{2}DF=2,CH=frac{1}{2}DC=2$,故答案为:24.【点睛】本题考查了平行四边形的性质和判定、全等三角形的判定与性质、含$30^{circ}$角的直角三角形的性质、三角形的面积等知识点,熟练掌握平行四边形的判定与性质,证明$triangleADEcongtriangleCBE$是解此题的关键.", "solution": "null", "level": "八年级", "question": "如图,在四边形$ABCD$中,$AD//BC,AC$与$BD$交于点$E$,点$E$是$BD$的中点,延长$CD$到点$F$,使$DF=CD$,连接$AF$.(1)求证:$AE=CE$;(2)求证:四边形$ABDF$是平行四边形;(3)若$AB=4,AF=8,angleF=30^{circ}$,则四边形$ABCF$的面积为24.", "options": [], "subject": "度量几何学", "analysis": "(1)由平行线的性质得$angleADE=angleCBE$,再由$ASA$证明$triangleADEcongtriangleCBE$,即可得出结论;(2)由平行四边形的性质得$AB//CD,AB=CD$,再证$DF=AB$,即可得出结论;(3)过$C$作$CHperpBD$于$H$,过$D$作$DQperpAF$于$Q$,求出高$DQ$和$CH$,再根据面积公式求出即可.【解析】(1)证明:$because$点$E$是$BD$的中点,$thereforeBE=DE$,$becauseAD//BC$$thereforeangleADE=angleCBE$,在$triangleADE$和$triangleCBE$中,$thereforetriangleADEcongtriangleCBE(ASA)$,$thereforeAE=CE$(2)证明:$becauseAE=CE,BE=DE$,$therefore$四边形$ABCD$是平行四边形,$thereforeAB//CD,AB=CD$,$becauseDF=CD$,$thereforeDF=AB$,又$becauseDF//AB$,$therefore$四边形$ABDF$是平行四边形;(3)解:过$C$作$CHperpBD$于$H$,过$D$作$DQperpAF$于$Q$,如图所示:$because$四边形$ABCD$和四边形$ABDF$是平行四边形,$AB=4,AF=8,angleF=30^{circ}$,$thereforeDF=AB=4,CD=AB=4,BD=AF=8,BD//AF$,$thereforeangleBDC=angleF=30^{circ}$,$becauseCHperpBD,DQperpAF$,$thereforeangleCHD=angleDQF=90^{circ}$,$thereforeDQ=frac{1}{2}DF=2,CH=frac{1}{2}DC=2$,故答案为:24.【点睛】本题考查了平行四边形的性质和判定、全等三角形的判定与性质、含$30^{circ}$角的直角三角形的性质、三角形的面积等知识点,熟练掌握平行四边形的判定与性质,证明$triangleADEcongtriangleCBE$是解此题的关键."} {"id": "5162", "image": ["6709.jpg"], "answer": "(1)由平行四边形的性质和角平分线证出$angleBAE=angleE$.得出$AB=BE$,即可得出结论;(2)同(1)证出$DA=DF$,由$F$为$DC$中点,$AB=CD$,求出$AD$与$DF$的长,得出三角形$ADF$为等腰三角形,根据三线合一得到$G$为$AF$中点,在直角三角形$ADG$中,由$AD$与$DG$的长,利用勾股定理求出$AG$的长,进而求出$AF$的长,再由三角形$ADF$与三角形$ECF$全等,得出$AF=$$EF$,即可求出$AE$的长.【解析】(1)证明:$becauseAE$为$angleBAD$的平分线,$thereforeangleDAE=angleBAE$.$because$四边形$ABCD$是平行四边形,$thereforeAD//BC,CD=AB$.$thereforeangleDAE=angleE$.$thereforeangleBAE=angleE$.$thereforeAB=BE$.$thereforeCD=BE$.(2)解:$because$四边形$ABCD$是平行四边形,$thereforeCD//AB$,$thereforeangleBAF=angleDFA$.$thereforeangleDAF=angleDFA$.$thereforeDA=DF$.$becauseF$为$DC$的中点,$AB=10$,$thereforeDF=CF=DA=5$.$becauseDGperpAE,DG=3$,$thereforeAG=GF$.$thereforeAG=sqrt{5^{2}-3^{2}}=4$.$thereforeAF=2AG=8$.在$triangleADF$和$triangleECF$中,$left{begin{array}{l}angle{DAF}=angle{E}angle{ADF}=angle{ECF},{DF}={CF}end{array}right.$$thereforetriangleADFcongtriangleECF(AAS)$.$thereforeAF=EF$,$thereforeAE=2AF=16$.【点睛】此题考查了平行四边形的性质,全等三角形的判定与性质,勾股定理,等腰三角形的判定与性质,熟练掌握平行四边形的判定与性质,证明三角形全等是解决问题(2)的关键.", "solution": "null", "level": "八年级", "question": "如图,在平行四边形$ABCD$中,$angleBAD$的平分线与$BC$的延长线交于点$E$,与$DC$交于点$F$.(1)求证$CD=BE$;(2)若点$F$为$DC$的中点,$DGperpAE$于$G$,且$DG=3,AB=10$,求$AE$的长.", "options": [], "subject": "度量几何学", "analysis": "(1)由平行四边形的性质和角平分线证出$angleBAE=angleE$.得出$AB=BE$,即可得出结论;(2)同(1)证出$DA=DF$,由$F$为$DC$中点,$AB=CD$,求出$AD$与$DF$的长,得出三角形$ADF$为等腰三角形,根据三线合一得到$G$为$AF$中点,在直角三角形$ADG$中,由$AD$与$DG$的长,利用勾股定理求出$AG$的长,进而求出$AF$的长,再由三角形$ADF$与三角形$ECF$全等,得出$AF=$$EF$,即可求出$AE$的长.【解析】(1)证明:$becauseAE$为$angleBAD$的平分线,$thereforeangleDAE=angleBAE$.$because$四边形$ABCD$是平行四边形,$thereforeAD//BC,CD=AB$.$thereforeangleDAE=angleE$.$thereforeangleBAE=angleE$.$thereforeAB=BE$.$thereforeCD=BE$.(2)解:$because$四边形$ABCD$是平行四边形,$thereforeCD//AB$,$thereforeangleBAF=angleDFA$.$thereforeangleDAF=angleDFA$.$thereforeDA=DF$.$becauseF$为$DC$的中点,$AB=10$,$thereforeDF=CF=DA=5$.$becauseDGperpAE,DG=3$,$thereforeAG=GF$.$thereforeAG=sqrt{5^{2}-3^{2}}=4$.$thereforeAF=2AG=8$.在$triangleADF$和$triangleECF$中,$left{begin{array}{l}angle{DAF}=angle{E}angle{ADF}=angle{ECF},{DF}={CF}end{array}right.$$thereforetriangleADFcongtriangleECF(AAS)$.$thereforeAF=EF$,$thereforeAE=2AF=16$.【点睛】此题考查了平行四边形的性质,全等三角形的判定与性质,勾股定理,等腰三角形的判定与性质,熟练掌握平行四边形的判定与性质,证明三角形全等是解决问题(2)的关键."} {"id": "5182", "image": ["6733.jpg"], "answer": "(1)根据平行四边形的判定定理得到四边形$ABCD$是平行四边形,根据三角形的外角的性质得到$angleAOB=angleDAO+angleADO=2angleOAD$,求得$angleDAO=angleADO$,推出$AC=BD$,于是得到四边形$ABCD$是矩形;(2)根据矩形的性质得到$AB//CD$,根据平行线的性质得到$angleABO=angleCDO$,根据三角形的内角得到$angleABO=54^{circ}$,于是得到结论.【解析】(1)证明:$becauseAO=OC,BO=OD$,$therefore$四边形$ABCD$是平行四边形,$becauseangleAOB=angleDAO+angleADO=2angleOAD$,$thereforeangleDAO=angleADO$,$thereforeAO=DO$,$thereforeAC=BD$,$therefore$四边形$ABCD$是矩形;(2)解:$because$四边形$ABCD$是矩形,$thereforeAB//CD$,$thereforeangleABO=angleCDO$,$becauseangleAOB:angleODC=4:3$,$thereforeangleAOB:angleABO=4:3$,$thereforeangleBAO:angleAOB:angleABO=3:4:3$,$thereforeangleABO=54^{circ}$,$becauseangleBAD=90^{circ}$,$thereforeangleADO=90^{circ}-54^{circ}=36^{circ}$.【点睛】本题考查了矩形的判定和性质,三角形的内角和,正确的理解题意是解题的关键.", "solution": "null", "level": "八年级", "question": "如图,四边形$ABCD$中,对角线$AC、BD$相交于点$O,AO=OC,BO=OD$,且$angleAOB=2angleOAD$.(1)求证:四边形$ABCD$是矩形;(2)若$angleAOB:angleODC=4:3$,求$angleADO$的度数.", "options": [], "subject": "度量几何学", "analysis": "(1)根据平行四边形的判定定理得到四边形$ABCD$是平行四边形,根据三角形的外角的性质得到$angleAOB=angleDAO+angleADO=2angleOAD$,求得$angleDAO=angleADO$,推出$AC=BD$,于是得到四边形$ABCD$是矩形;(2)根据矩形的性质得到$AB//CD$,根据平行线的性质得到$angleABO=angleCDO$,根据三角形的内角得到$angleABO=54^{circ}$,于是得到结论.【解析】(1)证明:$becauseAO=OC,BO=OD$,$therefore$四边形$ABCD$是平行四边形,$becauseangleAOB=angleDAO+angleADO=2angleOAD$,$thereforeangleDAO=angleADO$,$thereforeAO=DO$,$thereforeAC=BD$,$therefore$四边形$ABCD$是矩形;(2)解:$because$四边形$ABCD$是矩形,$thereforeAB//CD$,$thereforeangleABO=angleCDO$,$becauseangleAOB:angleODC=4:3$,$thereforeangleAOB:angleABO=4:3$,$thereforeangleBAO:angleAOB:angleABO=3:4:3$,$thereforeangleABO=54^{circ}$,$becauseangleBAD=90^{circ}$,$thereforeangleADO=90^{circ}-54^{circ}=36^{circ}$.【点睛】本题考查了矩形的判定和性质,三角形的内角和,正确的理解题意是解题的关键."} {"id": "5199", "image": ["6759.jpg", "6760.jpg", "6760.jpg"], "answer": "(1)根据等角对等边得出$OB=OC$,根据平行四边形性质求出$OC=OA=frac{1}{2}AC,OB=$$OD=frac{1}{2}BD$,推出$AC=BD$,根据矩形的判定推出即可.(2)根据矩形的性质和$angleCBE=3angleABE$,得出$angleABE=22.5^{circ}$,在$EB$上取一点$H$,使得$EH=AE$,易证$AH=BH$,设$AE=EB=x$,则$AH=BH=sqrt{2}x$,构建方程即可解决问题.【解析】(1)证明:$becauseangleOBC=angleOCB$,$thereforeOB=OC$,$because$四边形$ABCD$是平行四边形,$thereforeOC=OA=frac{1}{2}AC,OB=OD=frac{1}{2}BD$,$thereforeAC=BD$,$because$四边形$ABCD$是平行四边形,$therefore$四边形$ABCD$是矩形;(2)$because$四边形$ABCD$是矩形,$thereforeangleABC=90^{circ}$,$becauseangleCBE=3angleABE$$thereforeangleABE=frac{1}{4}times90^{circ}=22.5^{circ}$,在$EB$上取一点$H$,使得$EH=AE$,易证$AH=BH$,设$AE=EB=x$,则$AH=BH=sqrt{2}x$,$becauseBE=2$,$thereforex+sqrt{2}x=2$,$thereforex=2sqrt{2}-2$.【点睛】本题考查了平行四边形的性质,等腰三角形的判定,矩形的判定,注意:对角线相等的平行四边形是矩形,等角对等边,学会添加常用辅助线,构造特殊三角形解决问题.", "solution": "null", "level": "八年级", "question": "如图,$squareABCD$的对角线$AC、BD$相交于点$O$,且$angleOBC=angleOCB$.(1)求证:四边形$ABCD$为矩形;(2)过$B$作$BEperpAO$于$E,angleCBE=3angleABE,BE=2$,求$AE$的长.", "options": [], "subject": "度量几何学", "analysis": "(1)根据等角对等边得出$OB=OC$,根据平行四边形性质求出$OC=OA=frac{1}{2}AC,OB=$$OD=frac{1}{2}BD$,推出$AC=BD$,根据矩形的判定推出即可.(2)根据矩形的性质和$angleCBE=3angleABE$,得出$angleABE=22.5^{circ}$,在$EB$上取一点$H$,使得$EH=AE$,易证$AH=BH$,设$AE=EB=x$,则$AH=BH=sqrt{2}x$,构建方程即可解决问题.【解析】(1)证明:$becauseangleOBC=angleOCB$,$thereforeOB=OC$,$because$四边形$ABCD$是平行四边形,$thereforeOC=OA=frac{1}{2}AC,OB=OD=frac{1}{2}BD$,$thereforeAC=BD$,$because$四边形$ABCD$是平行四边形,$therefore$四边形$ABCD$是矩形;(2)$because$四边形$ABCD$是矩形,$thereforeangleABC=90^{circ}$,$becauseangleCBE=3angleABE$$thereforeangleABE=frac{1}{4}times90^{circ}=22.5^{circ}$,在$EB$上取一点$H$,使得$EH=AE$,易证$AH=BH$,设$AE=EB=x$,则$AH=BH=sqrt{2}x$,$becauseBE=2$,$thereforex+sqrt{2}x=2$,$thereforex=2sqrt{2}-2$.【点睛】本题考查了平行四边形的性质,等腰三角形的判定,矩形的判定,注意:对角线相等的平行四边形是矩形,等角对等边,学会添加常用辅助线,构造特殊三角形解决问题."} {"id": "5203", "image": ["6767.jpg", "6768.jpg", "6769.jpg", "6770.jpg", "6771.jpg", "6770.jpg", "6771.jpg"], "answer": "(1)如图2中,结论:$AC=sqrt{2}(FG+EC)$.在$AB$上截取$BM=BE$,连接$EM$,证明$triangleAEMcongtriangleEFC(ASA)$,可得结论.如图3中,结论:$AC=sqrt{2}(FG-EC)$.(2)分两种情形:如图1中,当$angleBAE=30^{circ}$时,如图3中,当$angleAEB=30^{circ}$时,利用等腰直角三角形的性质求解即可.【解析】解:(1)如图2中,结论:$AC=sqrt{2}(FG+EC)$.理由:在$AB$上截取$BM=BE$,连接$EM$,$because$四边形$ABCD$是正方形,$thereforeangleB=angleBCD=90^{circ},AB=BC$$thereforeangleDCG=90^{circ},angleEAM+angleAEB=90^{circ}$,$becauseBM=BE$,$thereforeAB-BM=BC-BE,angleBME=angleBEM=45^{circ}$,$thereforeAM=EC,angleAME=135^{circ}$,$becauseCF$平分$angleDCG$,$thereforeangleFCG=45^{circ}$,$thereforeangleECF=135^{circ}$,$thereforeangleAME=angleECF$,$becauseangleAEF=90^{circ}$$thereforeangleFEC+angleAEB=90^{circ}$,$thereforeangleEAM=angleFEC$,$therefore$在$triangleAEM$和$triangleEFC$中,$thereforetriangleAEMcongtriangleEFC(ASA)$,$thereforeEM=CF$,$becauseEM=sqrt{2}BE,quadCF=sqrt{2}FG$,$thereforeBE=FG$,$becauseAC=sqrt{2}BC=sqrt{2}(BE+EC)$,$thereforeAC=sqrt{2}(FG+EC)$.如图3中,结论:$AC=sqrt{2}(FG-EC)$.(2)如图1中,当$angleBAE=30^{circ}$时,$because$正方形的面积为27,$thereforeAB=3sqrt{3},angleB=90^{circ}$,$thereforeBE=ABcdottan30^{circ}=3sqrt{3}timesfrac{sqrt{3}}{3}=3$,$thereforeAE=2BE=6$,$becausetriangleAEMcongtriangleEFC$$thereforeAE=EF=6$,$thereforeAF=6sqrt{2}$,如图3中,当$angleAEB=30^{circ}$时,同法可得$AE=EF=2AB=6sqrt{3}$,$thereforeAF=sqrt{2}AE=6sqrt{6}$,综上所述,$AF$的长为$6sqrt{2}$或$6sqrt{6}$.图2【点睛】本题考查正方形的性质,全等三角形的判定和性质,等腰直角三角形的性质等知识,解题的关键是学会添加常用辅助线,构造全等三角形解决问题,学会用分类讨论的思想思考问题.", "solution": "null", "level": "八年级", "question": "如图1,四边形$ABCD$是正方形,点$E$是边$BC$的中点,$angleAEF=90^{circ}$,且$EF$交正方形外角的平分线$CF$于点$F$,过点$F$作$FGperpBC$于点$G$,连接$AC$.易证:$AC=sqrt{2}(EC+FG)$.(提示:取$AB$的中点$M$,连接$EM)$(1)当点$E$是$BC$边上任意一点时,如图2;当点$E$在$BC$延长线上时,如图3.请直接写出$AC$,$EC,FG$的数量关系,并对图2进行证明;(2)已知正方形$ABCD$的面积是27,连接$AF$,当$triangleABE$中有一个内角为$30^{circ}$时,则$AF$的长为$6sqrt{2}$或$6sqrt{6}ldots$.图1图2图3", "options": [], "subject": "度量几何学", "analysis": "(1)如图2中,结论:$AC=sqrt{2}(FG+EC)$.在$AB$上截取$BM=BE$,连接$EM$,证明$triangleAEMcongtriangleEFC(ASA)$,可得结论.如图3中,结论:$AC=sqrt{2}(FG-EC)$.(2)分两种情形:如图1中,当$angleBAE=30^{circ}$时,如图3中,当$angleAEB=30^{circ}$时,利用等腰直角三角形的性质求解即可.【解析】解:(1)如图2中,结论:$AC=sqrt{2}(FG+EC)$.理由:在$AB$上截取$BM=BE$,连接$EM$,$because$四边形$ABCD$是正方形,$thereforeangleB=angleBCD=90^{circ},AB=BC$$thereforeangleDCG=90^{circ},angleEAM+angleAEB=90^{circ}$,$becauseBM=BE$,$thereforeAB-BM=BC-BE,angleBME=angleBEM=45^{circ}$,$thereforeAM=EC,angleAME=135^{circ}$,$becauseCF$平分$angleDCG$,$thereforeangleFCG=45^{circ}$,$thereforeangleECF=135^{circ}$,$thereforeangleAME=angleECF$,$becauseangleAEF=90^{circ}$$thereforeangleFEC+angleAEB=90^{circ}$,$thereforeangleEAM=angleFEC$,$therefore$在$triangleAEM$和$triangleEFC$中,$thereforetriangleAEMcongtriangleEFC(ASA)$,$thereforeEM=CF$,$becauseEM=sqrt{2}BE,quadCF=sqrt{2}FG$,$thereforeBE=FG$,$becauseAC=sqrt{2}BC=sqrt{2}(BE+EC)$,$thereforeAC=sqrt{2}(FG+EC)$.如图3中,结论:$AC=sqrt{2}(FG-EC)$.(2)如图1中,当$angleBAE=30^{circ}$时,$because$正方形的面积为27,$thereforeAB=3sqrt{3},angleB=90^{circ}$,$thereforeBE=ABcdottan30^{circ}=3sqrt{3}timesfrac{sqrt{3}}{3}=3$,$thereforeAE=2BE=6$,$becausetriangleAEMcongtriangleEFC$$thereforeAE=EF=6$,$thereforeAF=6sqrt{2}$,如图3中,当$angleAEB=30^{circ}$时,同法可得$AE=EF=2AB=6sqrt{3}$,$thereforeAF=sqrt{2}AE=6sqrt{6}$,综上所述,$AF$的长为$6sqrt{2}$或$6sqrt{6}$.图2【点睛】本题考查正方形的性质,全等三角形的判定和性质,等腰直角三角形的性质等知识,解题的关键是学会添加常用辅助线,构造全等三角形解决问题,学会用分类讨论的思想思考问题."} {"id": "5205", "image": ["6774.jpg", "6775.jpg", "6776.jpg", "6776.jpg"], "answer": "【教材呈现】根据$AAS$证$triangleCDFcongtriangleBCE$即可得证;【结论应用】设三角形$DCG$的面积为$x$,根据题意列出方程求解即可得出$triangleDCG$的面积,设$CG$$=a,DG=b$,根据数量关系推出$(a+b)$的值,即可得出$CG+DG$的长.【解析】【教材呈现】证明:设$DF、EC$交于点$G$,图(1)$becauseDFperpCE$$thereforeangleFGC=90^{circ}$,$thereforeangleDFC+angleECB=90^{circ}$,$because$四边形$ABCD$是正方形,$thereforeBC=DC,angleB=angleFCD=90^{circ}$,$thereforeangleBEC+angleBCE=90^{circ}$$thereforeangleBEC=angleCFD$,在$triangleCDF$和$triangleBCE$中,$left{begin{array}{l}angle{BEC}=angle{CFD}angle{B}=angle{FCB},{BC}={CD}end{array}right.$$thereforetriangleCDFcongtriangleBCE(AAS)$,$thereforeCE=DF$【结论应用】由【教材呈现】知,$S_{triangleBEC}=S_{triangleCDF}$,$therefore$四边形$BEGF$的面积$+triangleFGC$的面积$=triangleFGC$的面积$+triangleDGC$的面积,即四边形$BEGF$的面积$=triangleDGC$的面积,设$triangleDCG$的面积为$x$,则阴影部分的面积为:$3times3-2x=9-2x$,即$frac{9-2x}{9}=frac{2}{3}$,解得$x=frac{3}{2}$,设$CG=a,DG=b$,$becauseDFperpEC$$thereforefrac{1}{2}ab=frac{3}{2},a^{2}+b^{2}=9$,$thereforea^{2}+b^{2}+2ab=9+6=15$,即$(a+b)^{2}=15$,$thereforeCG+DG=a+b=sqrt{15}$,故答案为:$frac{3}{2},sqrt{15}$.【点睛】本题主要考查正方形的性质,熟练掌握正方形的性质,全等三角形的判定和性质等知识是解题的关键.", "solution": "null", "level": "八年级", "question": "【教材呈现】如图是华师版八年级下册数学教材第121页的部分内容.如图,在正方形$ABCD$中,$CEperpDF$.求证:$CE=DF$.结合图(1),写出证明过程.【结论应用】如图(2),设$CE,DF$相交于点$G$.若$AB=3$,图中阴影部分的面积和与正方形$ABCD$的面积之比为2:3,则$triangleDCG$的面积为$-frac{3}{2}$—,$CG+DG$的长为$_sqrt{15}_$.图(1)图(2)", "options": [], "subject": "度量几何学", "analysis": "【教材呈现】根据$AAS$证$triangleCDFcongtriangleBCE$即可得证;【结论应用】设三角形$DCG$的面积为$x$,根据题意列出方程求解即可得出$triangleDCG$的面积,设$CG$$=a,DG=b$,根据数量关系推出$(a+b)$的值,即可得出$CG+DG$的长.【解析】【教材呈现】证明:设$DF、EC$交于点$G$,图(1)$becauseDFperpCE$$thereforeangleFGC=90^{circ}$,$thereforeangleDFC+angleECB=90^{circ}$,$because$四边形$ABCD$是正方形,$thereforeBC=DC,angleB=angleFCD=90^{circ}$,$thereforeangleBEC+angleBCE=90^{circ}$$thereforeangleBEC=angleCFD$,在$triangleCDF$和$triangleBCE$中,$left{begin{array}{l}angle{BEC}=angle{CFD}angle{B}=angle{FCB},{BC}={CD}end{array}right.$$thereforetriangleCDFcongtriangleBCE(AAS)$,$thereforeCE=DF$【结论应用】由【教材呈现】知,$S_{triangleBEC}=S_{triangleCDF}$,$therefore$四边形$BEGF$的面积$+triangleFGC$的面积$=triangleFGC$的面积$+triangleDGC$的面积,即四边形$BEGF$的面积$=triangleDGC$的面积,设$triangleDCG$的面积为$x$,则阴影部分的面积为:$3times3-2x=9-2x$,即$frac{9-2x}{9}=frac{2}{3}$,解得$x=frac{3}{2}$,设$CG=a,DG=b$,$becauseDFperpEC$$thereforefrac{1}{2}ab=frac{3}{2},a^{2}+b^{2}=9$,$thereforea^{2}+b^{2}+2ab=9+6=15$,即$(a+b)^{2}=15$,$thereforeCG+DG=a+b=sqrt{15}$,故答案为:$frac{3}{2},sqrt{15}$.【点睛】本题主要考查正方形的性质,熟练掌握正方形的性质,全等三角形的判定和性质等知识是解题的关键."} {"id": "4092", "image": ["5774.jpg"], "answer": "解: (1) $\\because \\angle \\mathrm{B}=70^{\\circ}, \\angle \\mathrm{C}=30^{\\circ}$,\n\n$\\therefore \\angle \\mathrm{BAC}=180^{\\circ}-\\angle \\mathrm{B}-\\angle \\mathrm{C}=80^{\\circ}$\n\n又 $\\because \\mathrm{AE}$ 平分 $\\angle \\mathrm{BAC}$,\n$\\therefore \\angle \\mathrm{BAE}=\\angle \\mathrm{EAC}=\\frac{1}{2} \\angle \\mathrm{BAC}=40^{\\circ}$;\n\n(2) $\\because \\mathrm{AD} \\perp \\mathrm{BC}$\n\n$\\therefore \\angle A D C=90^{\\circ}$,\n\n$\\therefore \\angle \\mathrm{DAC}=90^{\\circ}-\\angle \\mathrm{C}=60^{\\circ}$,\n\n$\\therefore \\angle \\mathrm{DAE}=\\angle \\mathrm{DAC}-\\angle \\mathrm{EAC}=20^{\\circ}$", "solution": "null", "level": "八年级", "question": "如图, 在 $\\triangle A B C$ 中, $A D \\perp B C, A E$ 平分 $\\angle B A C, \\angle B=70^{\\circ}, \\angle C=30^{\\circ}$.\n\n(1) 则 $\\angle \\mathrm{BAE}=$\n\n(2) 求 $\\angle \\mathrm{DAE}$ 的度数.\n\n", "options": [], "subject": "度量几何学", "analysis": "解: (1) $\\because \\angle \\mathrm{B}=70^{\\circ}, \\angle \\mathrm{C}=30^{\\circ}$,\n\n$\\therefore \\angle \\mathrm{BAC}=180^{\\circ}-\\angle \\mathrm{B}-\\angle \\mathrm{C}=80^{\\circ}$\n\n又 $\\because \\mathrm{AE}$ 平分 $\\angle \\mathrm{BAC}$,\n$\\therefore \\angle \\mathrm{BAE}=\\angle \\mathrm{EAC}=\\frac{1}{2} \\angle \\mathrm{BAC}=40^{\\circ}$;\n\n(2) $\\because \\mathrm{AD} \\perp \\mathrm{BC}$\n\n$\\therefore \\angle A D C=90^{\\circ}$,\n\n$\\therefore \\angle \\mathrm{DAC}=90^{\\circ}-\\angle \\mathrm{C}=60^{\\circ}$,\n\n$\\therefore \\angle \\mathrm{DAE}=\\angle \\mathrm{DAC}-\\angle \\mathrm{EAC}=20^{\\circ}$"} {"id": "4116", "image": ["5796.jpg", "5797.jpg"], "answer": "解: 设 $A B=x$\n\n依题意可知: 在 Rt $\\triangle A C E$ 中, $\\angle A E C=90^{\\circ}, A C=x+1, A E=x-1, C E=6$,\n\n根据勾股定理得: $A C^{2}=A E^{2}+C E^{2}$, 即: $(x+1)^{2}=(x-1)^{2}+6^{2}$,\n\n解得: $x=9$ 答:旗杆 $A B$ 的高度是 9 米.", "solution": "null", "level": "八年级", "question": "学过 《勾股定理》后, 某班兴趣小组来到操场上测量旗杆 $A B$ 的高度, 得到如下信息:\n\n\n\n图1\n\n\n\n图2\n\n(1)测得从旗杆顶端垂直挂下来的升旗用的绳子比旗杆长 1 米(如图 1);\n\n(2)当将绳子拉直时, 测得此时拉绳子的手到地面的距离 $C D$ 为 1 米,\n\n到旗杆的距离 $C E$ 为 6 米(如图 2)。根据以上信息, 求旗杆 $A B$ 的高度.", "options": [], "subject": "度量几何学", "analysis": "解: 设 $A B=x$\n\n依题意可知: 在 Rt $\\triangle A C E$ 中, $\\angle A E C=90^{\\circ}, A C=x+1, A E=x-1, C E=6$,\n\n根据勾股定理得: $A C^{2}=A E^{2}+C E^{2}$, 即: $(x+1)^{2}=(x-1)^{2}+6^{2}$,\n\n解得: $x=9$ 答:旗杆 $A B$ 的高度是 9 米."} {"id": "4118", "image": ["5798.jpg", "5799.jpg", "5800.jpg", "5801.jpg", "5802.jpg", "5799.jpg", "5800.jpg", "5801.jpg", "5802.jpg"], "answer": "(1) 证明: $\\because A B / / D E$,\n\n$\\therefore \\angle A B C=\\angle D E F$,\n\n在 $\\triangle A B C$ 与 $\\triangle D E F$ 中 $\\left\\{\\begin{array}{c}\\angle A B C=\\angle D E F \\\\ A B=D E \\\\ \\angle A=\\angle D\\end{array}\\right.$\n\n$\\therefore \\triangle A B C \\cong \\triangle D E F(\\mathrm{ASA}) ;$\n\n(2) 解: $\\because \\triangle A B C \\cong \\triangle D E F$,\n\n$\\therefore B C=E F$,\n\n$\\therefore B F+F C=E C+F C$,\n\n$\\therefore B F=E C$\n\n$\\because B E=10 \\mathrm{~cm}, B F=3 \\mathrm{~cm}$\n$\\therefore F C=10-3-3=4 \\mathrm{~cm}$\n\n## 25.(1)【旧题重现】\n\n如图(1), $A D 、 A^{\\prime} D^{\\prime}$ 分别是 $\\triangle A B C$ 和 $\\triangle A^{\\prime} B^{\\prime} C^{\\prime}$ 的 $B C 、 B^{\\prime} C^{\\prime}$ 边上的中线,\n\n$A D=A^{\\prime} D^{\\prime}, A B=A^{\\prime} B^{\\prime}, B C=B^{\\prime} C^{\\prime}$. 求证: $\\triangle A B C \\cong \\triangle A^{\\prime} B^{\\prime} C^{\\prime}$.\n\n\n图(1)\n\n证明的途径可以用下面的框图表示,请填写其中的空格..\n\n\n\n(2)【深入研究】\n\n如图(2), $A D 、 A^{\\prime} D^{\\prime}$ 分别是 $\\triangle A B C$ 和 $\\triangle A^{\\prime} B^{\\prime} C^{\\prime}$ 的 $B C 、 B^{\\prime} C^{\\prime}$ 边上的中线, $A D=A^{\\prime} D^{\\prime}, A B=A^{\\prime} B^{\\prime}, A C=A^{\\prime} C^{\\prime}$. 判断 $\\triangle A B C$ 与 $\\triangle A^{\\prime} B^{\\prime} C^{\\prime}$ 是否仍然全等.\n\n\n图(2)\n\n 【解】(1) 证明: $\\because A D$ 是 $\\triangle A B C$ 的中线,\n$\\therefore B D=\\frac{1}{2} B C$,\n\n$\\because A^{\\prime} D^{\\prime}$ 分别是 $\\triangle A^{\\prime} B^{\\prime} C^{\\prime}$ 的中线,\n\n$\\therefore B^{\\prime} D^{\\prime}=\\frac{1}{2} B^{\\prime} C^{\\prime}$,\n\n$\\because B C=B^{\\prime} C^{\\prime}$,\n\n$\\therefore B D=B^{\\prime} D^{\\prime}$,\n\n在 $\\triangle A B D$ 和 $\\triangle A^{\\prime} B^{\\prime} D^{\\prime}$ 中,\n\n$\\left\\{\\begin{array}{l}B D=B^{\\prime} D^{\\prime} \\\\ A D=A^{\\prime} D^{\\prime}, \\\\ A B=A^{\\prime} B^{\\prime}\\end{array}\\right.$\n\n$\\therefore \\triangle A B D \\cong \\triangle A^{\\prime} B^{\\prime} D^{\\prime}(\\mathrm{SSS})$\n\n$\\therefore \\angle B=\\angle B^{\\prime}$,\n\n在 $\\triangle A B C$ 和 $\\triangle A^{\\prime} B^{\\prime} C^{\\prime}$ 中,\n\n$\\left\\{\\begin{array}{l}A B=A^{\\prime} B^{\\prime} \\\\ \\angle B=\\angle B^{\\prime}, \\\\ B C=B^{\\prime} C^{\\prime}\\end{array}\\right.$\n\n$\\therefore \\triangle A B C \\cong A^{\\prime} B^{\\prime} C^{\\prime}(\\mathrm{SAS})$\n\n故答案为: (1) $B D=\\frac{1}{2} B C$; (2) $B^{\\prime} D^{\\prime}=\\frac{1}{2} B^{\\prime} C^{\\prime}$; (3) $A D=A^{\\prime} D^{\\prime}$; (4) $\\angle B=\\angle B^{\\prime}$;\n\n(2) 解: $\\triangle A B C$ 与 $\\triangle A^{\\prime} B^{\\prime} C^{\\prime}$ 仍然全等, 理由如下:\n\n延长 $A D$ 至 $E$, 使 $D E=A D$, 连接 $B E$, 延长 $A^{\\prime} D^{\\prime}$ 至 $E^{\\prime}$, 使 $D^{\\prime} E^{\\prime}=A^{\\prime} D^{\\prime}$, 连接 $B^{\\prime} E^{\\prime}$.\n\n\n$\\because A D$ 和 $A^{\\prime} D^{\\prime}$ 分别是 $\\triangle A B C$ 和 $\\triangle A^{\\prime} B^{\\prime} C^{\\prime}$ 的 $B C$ 和 $B^{\\prime} C^{\\prime}$ 边上的中线,\n$\\therefore B D=C D, \\quad B^{\\prime} D^{\\prime}=C^{\\prime} D^{\\prime}$.\n\n在 $\\triangle A D C$ 和 $\\triangle E D B$ 中, $\\left\\{\\begin{array}{l}A D=D E \\\\ \\angle A D C=\\angle B D E, \\\\ B D=C D\\end{array}\\right.$\n\n$\\therefore \\triangle A D C \\cong \\triangle E D B(\\mathrm{SAS})$.\n\n$\\therefore A C=E B, \\quad \\angle D A C=\\angle E$,\n\n同理 $A^{\\prime} C^{\\prime}=E^{\\prime} B^{\\prime}, \\angle D^{\\prime} A^{\\prime} C^{\\prime}=\\angle E^{\\prime}$.\n\n$\\because A C=A^{\\prime} C^{\\prime}$,\n\n$\\therefore E B=E^{\\prime} B^{\\prime}$.\n\n$\\because A D=A^{\\prime} D^{\\prime}, \\quad A D=D E, \\quad A^{\\prime} D^{\\prime}=D^{\\prime} E^{\\prime}$,\n\n$\\therefore A E=A^{\\prime} E^{\\prime}$.\n\n$\\because A B=A^{\\prime} B^{\\prime}$,\n\n$\\therefore \\triangle A B E \\cong A^{\\prime} B^{\\prime} E^{\\prime}(\\mathrm{SSS})$.\n\n$\\therefore \\angle B A E=\\angle B^{\\prime} A^{\\prime} E^{\\prime}, \\quad \\angle E=\\angle E^{\\prime}$.\n\n$\\therefore \\angle D A C=\\angle D^{\\prime} A^{\\prime} C^{\\prime}$.\n\n$\\therefore \\angle B A C=\\angle B^{\\prime} A^{\\prime} C^{\\prime}$,\n\n又 $A B=A^{\\prime} B^{\\prime}, A C=A^{\\prime} C^{\\prime}$,\n\n$\\therefore \\triangle A B C \\cong A^{\\prime} B^{\\prime} C^{\\prime}(\\mathrm{SAS})$", "solution": "null", "level": "八年级", "question": "如图, $A B / / D E, \\angle A=\\angle D$, 测得 $A B=D E$.\n\n\n\n(1) 求证: $\\triangle A B C \\cong \\triangle D E F$;\n\n(2) 若 $B E=10 \\mathrm{~m}, \\quad B F=3 \\mathrm{~m}$, 求 $F C$ 的长.", "options": [], "subject": "度量几何学", "analysis": "(1) 证明: $\\because A B / / D E$,\n\n$\\therefore \\angle A B C=\\angle D E F$,\n\n在 $\\triangle A B C$ 与 $\\triangle D E F$ 中 $\\left\\{\\begin{array}{c}\\angle A B C=\\angle D E F \\\\ A B=D E \\\\ \\angle A=\\angle D\\end{array}\\right.$\n\n$\\therefore \\triangle A B C \\cong \\triangle D E F(\\mathrm{ASA}) ;$\n\n(2) 解: $\\because \\triangle A B C \\cong \\triangle D E F$,\n\n$\\therefore B C=E F$,\n\n$\\therefore B F+F C=E C+F C$,\n\n$\\therefore B F=E C$\n\n$\\because B E=10 \\mathrm{~cm}, B F=3 \\mathrm{~cm}$\n$\\therefore F C=10-3-3=4 \\mathrm{~cm}$\n\n## 25.(1)【旧题重现】\n\n如图(1), $A D 、 A^{\\prime} D^{\\prime}$ 分别是 $\\triangle A B C$ 和 $\\triangle A^{\\prime} B^{\\prime} C^{\\prime}$ 的 $B C 、 B^{\\prime} C^{\\prime}$ 边上的中线,\n\n$A D=A^{\\prime} D^{\\prime}, A B=A^{\\prime} B^{\\prime}, B C=B^{\\prime} C^{\\prime}$. 求证: $\\triangle A B C \\cong \\triangle A^{\\prime} B^{\\prime} C^{\\prime}$.\n\n\n图(1)\n\n证明的途径可以用下面的框图表示,请填写其中的空格..\n\n\n\n(2)【深入研究】\n\n如图(2), $A D 、 A^{\\prime} D^{\\prime}$ 分别是 $\\triangle A B C$ 和 $\\triangle A^{\\prime} B^{\\prime} C^{\\prime}$ 的 $B C 、 B^{\\prime} C^{\\prime}$ 边上的中线, $A D=A^{\\prime} D^{\\prime}, A B=A^{\\prime} B^{\\prime}, A C=A^{\\prime} C^{\\prime}$. 判断 $\\triangle A B C$ 与 $\\triangle A^{\\prime} B^{\\prime} C^{\\prime}$ 是否仍然全等.\n\n\n图(2)\n\n 【解】(1) 证明: $\\because A D$ 是 $\\triangle A B C$ 的中线,\n$\\therefore B D=\\frac{1}{2} B C$,\n\n$\\because A^{\\prime} D^{\\prime}$ 分别是 $\\triangle A^{\\prime} B^{\\prime} C^{\\prime}$ 的中线,\n\n$\\therefore B^{\\prime} D^{\\prime}=\\frac{1}{2} B^{\\prime} C^{\\prime}$,\n\n$\\because B C=B^{\\prime} C^{\\prime}$,\n\n$\\therefore B D=B^{\\prime} D^{\\prime}$,\n\n在 $\\triangle A B D$ 和 $\\triangle A^{\\prime} B^{\\prime} D^{\\prime}$ 中,\n\n$\\left\\{\\begin{array}{l}B D=B^{\\prime} D^{\\prime} \\\\ A D=A^{\\prime} D^{\\prime}, \\\\ A B=A^{\\prime} B^{\\prime}\\end{array}\\right.$\n\n$\\therefore \\triangle A B D \\cong \\triangle A^{\\prime} B^{\\prime} D^{\\prime}(\\mathrm{SSS})$\n\n$\\therefore \\angle B=\\angle B^{\\prime}$,\n\n在 $\\triangle A B C$ 和 $\\triangle A^{\\prime} B^{\\prime} C^{\\prime}$ 中,\n\n$\\left\\{\\begin{array}{l}A B=A^{\\prime} B^{\\prime} \\\\ \\angle B=\\angle B^{\\prime}, \\\\ B C=B^{\\prime} C^{\\prime}\\end{array}\\right.$\n\n$\\therefore \\triangle A B C \\cong A^{\\prime} B^{\\prime} C^{\\prime}(\\mathrm{SAS})$\n\n故答案为: (1) $B D=\\frac{1}{2} B C$; (2) $B^{\\prime} D^{\\prime}=\\frac{1}{2} B^{\\prime} C^{\\prime}$; (3) $A D=A^{\\prime} D^{\\prime}$; (4) $\\angle B=\\angle B^{\\prime}$;\n\n(2) 解: $\\triangle A B C$ 与 $\\triangle A^{\\prime} B^{\\prime} C^{\\prime}$ 仍然全等, 理由如下:\n\n延长 $A D$ 至 $E$, 使 $D E=A D$, 连接 $B E$, 延长 $A^{\\prime} D^{\\prime}$ 至 $E^{\\prime}$, 使 $D^{\\prime} E^{\\prime}=A^{\\prime} D^{\\prime}$, 连接 $B^{\\prime} E^{\\prime}$.\n\n\n$\\because A D$ 和 $A^{\\prime} D^{\\prime}$ 分别是 $\\triangle A B C$ 和 $\\triangle A^{\\prime} B^{\\prime} C^{\\prime}$ 的 $B C$ 和 $B^{\\prime} C^{\\prime}$ 边上的中线,\n$\\therefore B D=C D, \\quad B^{\\prime} D^{\\prime}=C^{\\prime} D^{\\prime}$.\n\n在 $\\triangle A D C$ 和 $\\triangle E D B$ 中, $\\left\\{\\begin{array}{l}A D=D E \\\\ \\angle A D C=\\angle B D E, \\\\ B D=C D\\end{array}\\right.$\n\n$\\therefore \\triangle A D C \\cong \\triangle E D B(\\mathrm{SAS})$.\n\n$\\therefore A C=E B, \\quad \\angle D A C=\\angle E$,\n\n同理 $A^{\\prime} C^{\\prime}=E^{\\prime} B^{\\prime}, \\angle D^{\\prime} A^{\\prime} C^{\\prime}=\\angle E^{\\prime}$.\n\n$\\because A C=A^{\\prime} C^{\\prime}$,\n\n$\\therefore E B=E^{\\prime} B^{\\prime}$.\n\n$\\because A D=A^{\\prime} D^{\\prime}, \\quad A D=D E, \\quad A^{\\prime} D^{\\prime}=D^{\\prime} E^{\\prime}$,\n\n$\\therefore A E=A^{\\prime} E^{\\prime}$.\n\n$\\because A B=A^{\\prime} B^{\\prime}$,\n\n$\\therefore \\triangle A B E \\cong A^{\\prime} B^{\\prime} E^{\\prime}(\\mathrm{SSS})$.\n\n$\\therefore \\angle B A E=\\angle B^{\\prime} A^{\\prime} E^{\\prime}, \\quad \\angle E=\\angle E^{\\prime}$.\n\n$\\therefore \\angle D A C=\\angle D^{\\prime} A^{\\prime} C^{\\prime}$.\n\n$\\therefore \\angle B A C=\\angle B^{\\prime} A^{\\prime} C^{\\prime}$,\n\n又 $A B=A^{\\prime} B^{\\prime}, A C=A^{\\prime} C^{\\prime}$,\n\n$\\therefore \\triangle A B C \\cong A^{\\prime} B^{\\prime} C^{\\prime}(\\mathrm{SAS})$"} {"id": "4135", "image": ["5814.jpg"], "answer": "证明: $\\because \\mathrm{AF}=\\mathrm{CD}$,\n\n$\\therefore \\mathrm{AC}=\\mathrm{DF}$,\n\n$\\because \\mathrm{BC} / / \\mathrm{EF}$,\n\n$\\therefore \\angle \\mathrm{ACB}=\\angle \\mathrm{DFE}$,\n\n在 $\\triangle \\mathrm{ABC}$ 和 $\\triangle \\mathrm{DEF}$ 中,\n\n$\\angle A=\\angle D$\n\n$\\{A C=D F$\n\n$\\angle A C B=\\angle D F E$\n\n$\\therefore \\triangle \\mathrm{ABC} \\cong \\triangle \\mathrm{DEF}(\\mathrm{ASA})$,\n\n$\\therefore \\mathrm{AB}=\\mathrm{DE}$.", "solution": "null", "level": "八年级", "question": "如图, 点 $A 、 F 、 C 、 D$ 在同一条直线上, 已知 $A F=D C, \\angle A=\\angle D, B C / / E F$, 求证: $A B=D E$.\n\n", "options": [], "subject": "度量几何学", "analysis": "证明: $\\because \\mathrm{AF}=\\mathrm{CD}$,\n\n$\\therefore \\mathrm{AC}=\\mathrm{DF}$,\n\n$\\because \\mathrm{BC} / / \\mathrm{EF}$,\n\n$\\therefore \\angle \\mathrm{ACB}=\\angle \\mathrm{DFE}$,\n\n在 $\\triangle \\mathrm{ABC}$ 和 $\\triangle \\mathrm{DEF}$ 中,\n\n$\\angle A=\\angle D$\n\n$\\{A C=D F$\n\n$\\angle A C B=\\angle D F E$\n\n$\\therefore \\triangle \\mathrm{ABC} \\cong \\triangle \\mathrm{DEF}(\\mathrm{ASA})$,\n\n$\\therefore \\mathrm{AB}=\\mathrm{DE}$."} {"id": "4162", "image": ["5848.jpg"], "answer": "解: (1) $\\because \\angle \\mathrm{B}=70^{\\circ}, \\angle \\mathrm{C}=30^{\\circ}$,\n\n$\\therefore \\angle \\mathrm{BAC}=180^{\\circ}-\\angle \\mathrm{B}-\\angle \\mathrm{C}=80^{\\circ}$,\n\n又 $\\because \\mathrm{AE}$ 平分 $\\angle \\mathrm{BAC}$,\n\n$\\therefore \\angle \\mathrm{BAE}=\\angle \\mathrm{EAC}=\\frac{1}{2} \\angle \\mathrm{BAC}=40^{\\circ} ;$\n\n(2) $\\because \\mathrm{AD} \\perp \\mathrm{BC}$,\n\n$\\therefore \\angle A D C=90^{\\circ}$,\n\n$\\therefore \\angle \\mathrm{DAC}=90^{\\circ}-\\angle \\mathrm{C}=60^{\\circ}$,\n\n$\\therefore \\angle \\mathrm{DAE}=\\angle \\mathrm{DAC}-\\angle \\mathrm{EAC}=20^{\\circ}$.", "solution": "null", "level": "八年级", "question": "如图, 在 $\\triangle \\mathrm{ABC}$ 中, $\\mathrm{AD} \\perp \\mathrm{BC}, \\mathrm{AE}$ 平分 $\\angle \\mathrm{BAC}, \\angle \\mathrm{B}=70^{\\circ}, \\angle C=30^{\\circ}$.\n\n\n\n(1) 则 $\\angle \\mathrm{BAE}=$ \\$ \\qquad \\$ ;\n\n(2) 求 $\\angle \\mathrm{DAE}$ 的度数.", "options": [], "subject": "度量几何学", "analysis": "解: (1) $\\because \\angle \\mathrm{B}=70^{\\circ}, \\angle \\mathrm{C}=30^{\\circ}$,\n\n$\\therefore \\angle \\mathrm{BAC}=180^{\\circ}-\\angle \\mathrm{B}-\\angle \\mathrm{C}=80^{\\circ}$,\n\n又 $\\because \\mathrm{AE}$ 平分 $\\angle \\mathrm{BAC}$,\n\n$\\therefore \\angle \\mathrm{BAE}=\\angle \\mathrm{EAC}=\\frac{1}{2} \\angle \\mathrm{BAC}=40^{\\circ} ;$\n\n(2) $\\because \\mathrm{AD} \\perp \\mathrm{BC}$,\n\n$\\therefore \\angle A D C=90^{\\circ}$,\n\n$\\therefore \\angle \\mathrm{DAC}=90^{\\circ}-\\angle \\mathrm{C}=60^{\\circ}$,\n\n$\\therefore \\angle \\mathrm{DAE}=\\angle \\mathrm{DAC}-\\angle \\mathrm{EAC}=20^{\\circ}$."} {"id": "4165", "image": ["5851.jpg"], "answer": "解: (1) $\\because \\angle B=40^{\\circ}, \\angle A D B=115^{\\circ}, A B=A C$,\n\n$\\therefore \\angle B A D=180^{\\circ}-40^{\\circ}-115^{\\circ}=25^{\\circ}, \\angle C=\\angle B=40^{\\circ}$;\n\n$\\because \\angle A D E=40^{\\circ}, \\angle A D B=115^{\\circ}$\n\n$\\therefore \\angle E D C=180^{\\circ}-\\angle A D B-\\angle A D E=180^{\\circ}-115^{\\circ}-40^{\\circ}=25^{\\circ}$.\n\n$\\therefore \\angle D E C=180^{\\circ}-40^{\\circ}-25^{\\circ}=115^{\\circ}$,\n\n当点 $D$ 从 $B$ 向 $C$ 运动时, $\\angle B D A$ 逐渐变小,\n\n故答案为: 25,115 , 小;\n\n(2) 当 $D C=2$ 时, $\\triangle A B D \\cong \\triangle D C E$, 理由如下:\n\n理由: $\\because \\angle C=40^{\\circ}$,\n\n$\\therefore \\angle D E C+\\angle E D C=140^{\\circ}$,\n\n又 $\\because \\angle A D E=40^{\\circ} ,$\n$\\therefore \\angle A D B+\\angle E D C=140^{\\circ}$,\n\n$\\therefore \\angle A D B=\\angle D E C$,\n\n又 $\\because A B=D C=2$,\n\n$\\therefore$ 在 $\\triangle A B D$ 和 $\\triangle D C E$ 中, $\\left\\{\\begin{array}{l}\\angle \\mathrm{ADB}=\\angle \\mathrm{DEC} \\\\ \\angle \\mathrm{B}=\\angle \\mathrm{C} \\\\ \\mathrm{AB}=\\mathrm{DC}\\end{array}\\right.$,\n\n$\\therefore \\triangle A B D \\xlongequal{\\triangle} \\triangle D C E(\\mathrm{AAS}) ;$\n\n(3) 当 $\\angle B D A$ 的度数为 $110^{\\circ}$ 或 $80^{\\circ}$ 时, $\\triangle A D E$ 的形状是等腰三角形, 理由如下:\n\n$\\because$ 当 $\\angle B D A=110^{\\circ}$ 时,\n\n$\\therefore \\angle A D C=70^{\\circ}$,\n\n$\\because \\angle C=40^{\\circ}$\n\n$\\therefore \\angle D A C=70^{\\circ}$,\n\n$\\therefore \\angle A E D=180^{\\circ}-70^{\\circ}-40^{\\circ}=70^{\\circ}$\n\n$\\therefore \\angle A E D=\\angle D A C$\n\n$\\therefore A D=D E$,\n\n$\\therefore \\triangle A D E$ 是等腰三角形;\n\n$\\because$ 当 $\\angle B D A$ 的度数为 $80^{\\circ}$ 时,\n\n$\\therefore \\angle A D C=100^{\\circ}$,\n\n$\\because \\angle C=40^{\\circ}$,\n\n$\\therefore \\angle D A C=40^{\\circ}$,\n\n$\\therefore \\angle D A C=\\angle A D E$\n\n$\\therefore A E=D E$,\n\n$\\therefore \\triangle A D E$ 是等腰三角形.\n\n综上所述, 当 $\\angle B D A$ 的度数为 $110^{\\circ}$ 或 $80^{\\circ}$ 时, $\\triangle A D E$ 是等腰三角形.", "solution": "null", "level": "八年级", "question": "如图, 在 $\\triangle A B C$ 中, $A B=A C=2, \\angle B=40^{\\circ}$, 点 $D$ 在线段 $B C$ 上运动\n\n( $D$ 不与 $B 、 C$ 重合), 连接 $A D$, 作 $\\angle A D E=40^{\\circ}, D E$ 与 $A C$ 交于 $E$.\n\n(1) 当 $\\angle B D A=115^{\\circ}$ 时, $\\angle B A D=$ \\$ \\qquad \\$ $\\circ, \\angle D E C=$ \\$ \\qquad \\$ ;\n\n当点 $D$ 从 $B$ 向 $C$ 运动时, $\\angle B D A$ 逐渐变 \\$ \\qquad \\$ (填”大”或”小”);\n\n(2)当 $D C=A B=2$ 时, $\\triangle A B D$ 与 $\\triangle D C E$ 是否全等?请说明理由:\n\n(3) 在点 $D$ 的运动过程中, $\\triangle A D E$ 的形状可以是等腰三角形吗?\n\n若可以, 请直接写出 $\\angle B D A$ 的度数; 若不可以, 请说明理由.\n\n", "options": [], "subject": "度量几何学", "analysis": "解: (1) $\\because \\angle B=40^{\\circ}, \\angle A D B=115^{\\circ}, A B=A C$,\n\n$\\therefore \\angle B A D=180^{\\circ}-40^{\\circ}-115^{\\circ}=25^{\\circ}, \\angle C=\\angle B=40^{\\circ}$;\n\n$\\because \\angle A D E=40^{\\circ}, \\angle A D B=115^{\\circ}$\n\n$\\therefore \\angle E D C=180^{\\circ}-\\angle A D B-\\angle A D E=180^{\\circ}-115^{\\circ}-40^{\\circ}=25^{\\circ}$.\n\n$\\therefore \\angle D E C=180^{\\circ}-40^{\\circ}-25^{\\circ}=115^{\\circ}$,\n\n当点 $D$ 从 $B$ 向 $C$ 运动时, $\\angle B D A$ 逐渐变小,\n\n故答案为: 25,115 , 小;\n\n(2) 当 $D C=2$ 时, $\\triangle A B D \\cong \\triangle D C E$, 理由如下:\n\n理由: $\\because \\angle C=40^{\\circ}$,\n\n$\\therefore \\angle D E C+\\angle E D C=140^{\\circ}$,\n\n又 $\\because \\angle A D E=40^{\\circ} ,$\n$\\therefore \\angle A D B+\\angle E D C=140^{\\circ}$,\n\n$\\therefore \\angle A D B=\\angle D E C$,\n\n又 $\\because A B=D C=2$,\n\n$\\therefore$ 在 $\\triangle A B D$ 和 $\\triangle D C E$ 中, $\\left\\{\\begin{array}{l}\\angle \\mathrm{ADB}=\\angle \\mathrm{DEC} \\\\ \\angle \\mathrm{B}=\\angle \\mathrm{C} \\\\ \\mathrm{AB}=\\mathrm{DC}\\end{array}\\right.$,\n\n$\\therefore \\triangle A B D \\xlongequal{\\triangle} \\triangle D C E(\\mathrm{AAS}) ;$\n\n(3) 当 $\\angle B D A$ 的度数为 $110^{\\circ}$ 或 $80^{\\circ}$ 时, $\\triangle A D E$ 的形状是等腰三角形, 理由如下:\n\n$\\because$ 当 $\\angle B D A=110^{\\circ}$ 时,\n\n$\\therefore \\angle A D C=70^{\\circ}$,\n\n$\\because \\angle C=40^{\\circ}$\n\n$\\therefore \\angle D A C=70^{\\circ}$,\n\n$\\therefore \\angle A E D=180^{\\circ}-70^{\\circ}-40^{\\circ}=70^{\\circ}$\n\n$\\therefore \\angle A E D=\\angle D A C$\n\n$\\therefore A D=D E$,\n\n$\\therefore \\triangle A D E$ 是等腰三角形;\n\n$\\because$ 当 $\\angle B D A$ 的度数为 $80^{\\circ}$ 时,\n\n$\\therefore \\angle A D C=100^{\\circ}$,\n\n$\\because \\angle C=40^{\\circ}$,\n\n$\\therefore \\angle D A C=40^{\\circ}$,\n\n$\\therefore \\angle D A C=\\angle A D E$\n\n$\\therefore A E=D E$,\n\n$\\therefore \\triangle A D E$ 是等腰三角形.\n\n综上所述, 当 $\\angle B D A$ 的度数为 $110^{\\circ}$ 或 $80^{\\circ}$ 时, $\\triangle A D E$ 是等腰三角形."} {"id": "5763", "image": ["7168.jpg", "7169.jpg", "7170.jpg", "7171.jpg", "7172.jpg", "7173.jpg", "7174.jpg", "7172.jpg", "7173.jpg", "7174.jpg"], "answer": " (1) 解: $\\because \\sqrt{m-4}+(n+4)^{2}=0, \\sqrt{m-4} \\geq 0,(n+4)^{2} \\geq 0$,\n\n$\\therefore m-4=0, n+4=0$,\n\n$\\therefore m=4, n=-4$,\n\n$\\therefore A(0,4), B(-4,0) ;$\n\n(2) 解: $\\because A C / / O B$,\n\n$\\therefore \\angle C=\\angle C B O, \\angle C A F=\\angle B O F$\n\n$\\because A C=O B$,\n\n$\\therefore \\triangle A C F \\AA O B F$\n\n$\\therefore A F=O F=2$,\n\n$\\because O A=O B, \\angle O A D=\\angle O B F, \\angle B O F=\\angle A O D$,\n\n$\\therefore \\triangle B O F \\cong A O D$,\n\n$\\therefore O F=O D=2$,\n\n$\\therefore B D=6$,\n\n(1)当 $0 \\leq t<3$ 时, $S=\\frac{1}{2} P D \\cdot O F=\\frac{1}{2}(6-2 t) \\times 2=6-2 t$ ,\n\n(2) 当 $t>3$ 时, $\\quad S=\\frac{1}{2} P D \\cdot O F=\\frac{1}{2}(2 t-6) \\times 2=2 t-6$,\n\n(3)解: (1)当 $0 \\leq t<3$,\n\n$\\because A O=4, O D=2$,\n\n$\\therefore A D=\\sqrt{O A^{2}+O D^{2}}=2 \\sqrt{5}$,\n\n$\\because B D \\times O A=A D \\times B E$,\n\n$\\therefore{ }^{B E=\\frac{12 \\sqrt{5}}{5}}, \\therefore D E=\\frac{6 \\sqrt{5}}{5}$,\n\n$\\because$ 四边形 $P D E G$ 为菱形,\n\n$\\therefore D P=D E=E G=\\frac{6 \\sqrt{5}}{5}$,\n\n$\\because D(20)$\n\n$\\therefore P\\left(2-\\frac{6 \\sqrt{5}}{5}, 0\\right)$\n\n作 $E H \\perp B D$ 于 $H$, 如图 2,\n\n\n\n图2\n\n$\\because B E \\times D E=B D \\times E H$,\n\n$\\therefore E H=\\frac{12}{5}$,\n\n\n$\\therefore E\\left(\\frac{4}{5}, \\frac{12}{5}\\right)$\n\n$\\because E G / / O B$\n\n$\\therefore G^{G}$ 与 $E_{\\text {的纵坐标相同, }}$,\n\n$\\therefore{ }^{G\\left(\\frac{4}{5}-\\frac{6 \\sqrt{5}}{5}, \\frac{12}{5}\\right)}$\n\n(2)当 $t>3$ 时, 如图 3, 同理可求得 $P\\left(2+\\frac{6 \\sqrt{5}}{5}, 0\\right), G\\left(\\frac{4}{5}+\\frac{6 \\sqrt{5}}{5}, \\frac{12}{5}\\right)$\n\n", "solution": "null", "level": "八年级", "question": "(15 分) 如图 1, 在平面直角坐标系中, 点 $O$ 是坐标原点, $A(0, m), B(n, 0), A C / / O B$, 且 $A C=O B$, 连接 $B C$ 交 $x$ 轴于点 $F$, 其中 $m 、 n$ 满足方程 $\\sqrt{m-4}+n^{2}+8 n+16=0$.\n\n\n\n图 1\n\n\n\n图 2\n\n\n\n备用图\n\n(1) 求 $A 、 B$ 两点坐标;\n\n(2) 如图 2, 过 $A$ 作 $A E \\perp B \\mathrm{C}$ 于 $E$, 延长 $A E$ 交 ${ }^{x}$ 轴于点 $D$, 动点 $P$ 从点 ${ }^{B}$ 出发以每秒 2\n\n\n写出 $t$ 的取值范围;\n\n(3) 在 (2) 的条件下, 连接 $P E$, 将 $\\triangle P E D$ 沿 $P E$ 翻折到 $\\triangle P E G$ 的位置 (点 $D$ 与点 $G$ 对应),当四边形 $P D E G$ 为菱形时, 求点 $P$ 和点 $G$ 的坐标.", "options": [], "subject": "解析几何", "analysis": " (1) 解: $\\because \\sqrt{m-4}+(n+4)^{2}=0, \\sqrt{m-4} \\geq 0,(n+4)^{2} \\geq 0$,\n\n$\\therefore m-4=0, n+4=0$,\n\n$\\therefore m=4, n=-4$,\n\n$\\therefore A(0,4), B(-4,0) ;$\n\n(2) 解: $\\because A C / / O B$,\n\n$\\therefore \\angle C=\\angle C B O, \\angle C A F=\\angle B O F$\n\n$\\because A C=O B$,\n\n$\\therefore \\triangle A C F \\AA O B F$\n\n$\\therefore A F=O F=2$,\n\n$\\because O A=O B, \\angle O A D=\\angle O B F, \\angle B O F=\\angle A O D$,\n\n$\\therefore \\triangle B O F \\cong A O D$,\n\n$\\therefore O F=O D=2$,\n\n$\\therefore B D=6$,\n\n(1)当 $0 \\leq t<3$ 时, $S=\\frac{1}{2} P D \\cdot O F=\\frac{1}{2}(6-2 t) \\times 2=6-2 t$ ,\n\n(2) 当 $t>3$ 时, $\\quad S=\\frac{1}{2} P D \\cdot O F=\\frac{1}{2}(2 t-6) \\times 2=2 t-6$,\n\n(3)解: (1)当 $0 \\leq t<3$,\n\n$\\because A O=4, O D=2$,\n\n$\\therefore A D=\\sqrt{O A^{2}+O D^{2}}=2 \\sqrt{5}$,\n\n$\\because B D \\times O A=A D \\times B E$,\n\n$\\therefore{ }^{B E=\\frac{12 \\sqrt{5}}{5}}, \\therefore D E=\\frac{6 \\sqrt{5}}{5}$,\n\n$\\because$ 四边形 $P D E G$ 为菱形,\n\n$\\therefore D P=D E=E G=\\frac{6 \\sqrt{5}}{5}$,\n\n$\\because D(20)$\n\n$\\therefore P\\left(2-\\frac{6 \\sqrt{5}}{5}, 0\\right)$\n\n作 $E H \\perp B D$ 于 $H$, 如图 2,\n\n\n\n图2\n\n$\\because B E \\times D E=B D \\times E H$,\n\n$\\therefore E H=\\frac{12}{5}$,\n\n\n$\\therefore E\\left(\\frac{4}{5}, \\frac{12}{5}\\right)$\n\n$\\because E G / / O B$\n\n$\\therefore G^{G}$ 与 $E_{\\text {的纵坐标相同, }}$,\n\n$\\therefore{ }^{G\\left(\\frac{4}{5}-\\frac{6 \\sqrt{5}}{5}, \\frac{12}{5}\\right)}$\n\n(2)当 $t>3$ 时, 如图 3, 同理可求得 $P\\left(2+\\frac{6 \\sqrt{5}}{5}, 0\\right), G\\left(\\frac{4}{5}+\\frac{6 \\sqrt{5}}{5}, \\frac{12}{5}\\right)$\n\n"} {"id": "5788", "image": [], "answer": "解:假设直线 $a 、 b$ 不止有一个公共点, 则至少有两个公共点, 不妨设为 $A 、 B$, 即直线 $a$ 、 $b$ 同时过点 A、B, 也就是说过 A、B 两点可以作两条直线 $a$ 、 $b$, 这和公理“过两点能且只能作一条直线”相矛盾, 所以假设不成立, 两条直线相交只有一个交点。", "solution": "null", "level": "八年级", "question": "( 5 分 ) 若两条直线 $a 、 b$ 相交则只有一个交点。", "options": [], "subject": "解析几何", "analysis": "解:假设直线 $a 、 b$ 不止有一个公共点, 则至少有两个公共点, 不妨设为 $A 、 B$, 即直线 $a$ 、 $b$ 同时过点 A、B, 也就是说过 A、B 两点可以作两条直线 $a$ 、 $b$, 这和公理“过两点能且只能作一条直线”相矛盾, 所以假设不成立, 两条直线相交只有一个交点。"} {"id": "5792", "image": ["7194.jpg"], "answer": " (1) 证明: 如图, $\\because$ 在四边形 $\\mathrm{ABCD}$ 中, $\\angle \\mathrm{A}=\\angle \\mathrm{C}, \\angle \\mathrm{ABC}=\\angle \\mathrm{ADC}$,\n\n$\\therefore$ 四边形 $\\mathrm{ABCD}$ 是平行四边形.\n\n由“四边形 $\\mathrm{ABCD}$ 是平行四边形”得到: $\\mathrm{AD} / / \\mathrm{BC}, \\mathrm{AB} / / \\mathrm{CD}$\n\n(2)证明:由“四边形 $A B C D$ 是平行四边形”得到: $A D=B C, A B=C D$", "solution": "null", "level": "八年级", "question": "( 10 分 ) 如图, 在四边形 $\\mathrm{ABCD}$ 中, $\\angle \\mathrm{A}=\\angle \\mathrm{C}, \\angle \\mathrm{ABC}=\\angle \\mathrm{ADC}$, 求证:\n\n\n\n(1) $\\mathrm{AD} / / \\mathrm{BC}, \\mathrm{AB} / / \\mathrm{CD}$;\n\n(2) $\\mathrm{AD}=\\mathrm{BC}, \\mathrm{AB}=\\mathrm{CD}$.", "options": [], "subject": "解析几何", "analysis": " (1) 证明: 如图, $\\because$ 在四边形 $\\mathrm{ABCD}$ 中, $\\angle \\mathrm{A}=\\angle \\mathrm{C}, \\angle \\mathrm{ABC}=\\angle \\mathrm{ADC}$,\n\n$\\therefore$ 四边形 $\\mathrm{ABCD}$ 是平行四边形.\n\n由“四边形 $\\mathrm{ABCD}$ 是平行四边形”得到: $\\mathrm{AD} / / \\mathrm{BC}, \\mathrm{AB} / / \\mathrm{CD}$\n\n(2)证明:由“四边形 $A B C D$ 是平行四边形”得到: $A D=B C, A B=C D$"} {"id": "5793", "image": ["7195.jpg", "7196.jpg", "7196.jpg"], "answer": " (1) 如图, $\\triangle \\mathrm{A}_{1} \\mathrm{~B}_{1} \\mathrm{C}_{1}$ 即为所求;\n\n\n\n(2) 解: $(1,1) ; 2<\\mathrm{h}<3.5$\n\n解: $(2) \\because \\mathrm{B}(-1,1), \\therefore \\mathrm{B}_{2}(1,1) ; \\because \\mathrm{B}_{2}(1,-1), \\mathrm{H}(-1,-2.5), \\therefore 2<\\mathrm{h}<3.5$. 故答案为: $(1,1), 2<\\mathrm{h}<3.5$.", "solution": "null", "level": "八年级", "question": "( 7 分 ) $\\triangle \\mathrm{ABC}$ 在平面直角坐标系中的位置如图所示.\n\n\n\n(1) 作 $\\triangle A B C$ 关于原点 $O$ 成中心对称的 $\\triangle A_{1} B_{1} C_{1}$.\n\n(2) 请写出点 $B$ 关于 $y$ 轴对称的点 $B_{2}$ 的坐标 . 若将点 $\\mathrm{B}_{2}$ 向下平移 $\\mathrm{h}$ 单位, 使其落在 $\\triangle \\mathrm{A}_{1} \\mathrm{~B}_{1} \\mathrm{C}_{1}$内部 (不包括边界) , 直接写出 $\\mathrm{h}$ 的值 (写出满足的一个即可).", "options": [], "subject": "解析几何", "analysis": " (1) 如图, $\\triangle \\mathrm{A}_{1} \\mathrm{~B}_{1} \\mathrm{C}_{1}$ 即为所求;\n\n\n\n(2) 解: $(1,1) ; 2<\\mathrm{h}<3.5$\n\n解: $(2) \\because \\mathrm{B}(-1,1), \\therefore \\mathrm{B}_{2}(1,1) ; \\because \\mathrm{B}_{2}(1,-1), \\mathrm{H}(-1,-2.5), \\therefore 2<\\mathrm{h}<3.5$. 故答案为: $(1,1), 2<\\mathrm{h}<3.5$."} {"id": "5796", "image": ["7197.jpg", "7198.jpg", "7199.jpg", "7198.jpg", "7199.jpg"], "answer": " (1) 解: 过作 $\\mathrm{CD} \\perp \\mathrm{x}$ 轴于点 $\\mathrm{D}$,\n\n\n\n$\\because \\mathrm{CB} \\perp \\mathrm{AB}$\n\n$\\therefore \\angle \\mathrm{ABC}=90^{\\circ}, \\quad \\therefore \\angle \\mathrm{CBD}+\\angle \\mathrm{ABO}=90^{\\circ}$,\n\n又 $\\because \\angle \\mathrm{BAC}=45^{\\circ}$,\n\n$\\therefore \\angle \\mathrm{ACB}=90^{\\circ}-\\angle \\mathrm{BAC}=45^{\\circ}=\\angle \\mathrm{BAC}$,\n\n$\\therefore \\mathrm{BC}=\\mathrm{BA}$,\n$\\because \\angle \\mathrm{AOB}=90^{\\circ}, \\quad \\therefore \\angle \\mathrm{ABO}+\\angle \\mathrm{BAO}=90^{\\circ}$,\n\n$\\therefore \\angle \\mathrm{CBD}=\\angle \\mathrm{BAO}$,\n\n又 $\\because \\angle \\mathrm{BDC}=\\angle \\mathrm{AOB}=90^{\\circ}$,\n\n$\\therefore \\triangle \\mathrm{BDC} \\cong \\triangle \\mathrm{AOB}$\n\n$\\therefore \\mathrm{BD}=\\mathrm{OA}, \\mathrm{CD}=\\mathrm{OB}$,\n\n$\\because$ 直线 $\\mathrm{l}_{1}: \\mathrm{y}=2 \\mathrm{x}+4$,\n\n$\\therefore \\mathrm{A}(0,4), \\mathrm{B}(-2,0)$,\n\n$\\therefore \\mathrm{BD}=\\mathrm{OA}=4, \\mathrm{CD}=\\mathrm{OB}=2$,\n\n$\\therefore \\mathrm{OD}=4+2=6$,\n\n$\\therefore \\mathrm{C}(-6,2)$\n\n(2) 解:设 $\\mathrm{l}_{2}$ 的解析式为 $\\mathrm{y}=\\mathrm{kx}+\\mathrm{b}(\\mathrm{k} \\neq 0)$\n\n$\\because$ A $(0,4), C(-6,2)$,\n\n$\\therefore \\begin{gathered}b=4 \\\\ -6 k+b=3\\end{gathered}$\n\n$\\left\\{\\begin{array}{l}k=\\frac{1}{3} \\\\ b=4\\end{array}\\right.$\n\n$\\therefore y=\\frac{1}{3} x+4$\n\n(3)解:如图, $\\mathrm{OA}$ 为平行四边形的边时,\n\n\n\n当四边形 $A O M_{1} N_{1}$ 为平行四边形时, 有 $M_{1} N_{1}=A O=4$,\n\n即 $\\left(3^{\\frac{1}{3} x+4}\\right)-(2 x+4)=4$, 解得: $x==^{-\\frac{12}{5}}$,\n当 $x={ }^{-\\frac{12}{5}}$ 时, $\\quad y=\\frac{1}{3} x+4=\\frac{16}{5}$,\n\n所以 $\\mathrm{N}_{1}\\left(-\\frac{12}{5}, \\frac{16}{5}\\right)$;\n\n当四边形 $\\mathrm{AOM}_{2} \\mathrm{~N}_{2}$ 为平行四边形时, 有 $\\mathrm{M}_{2} \\mathrm{~N}_{2}=\\mathrm{AO}=4$,\n\n即 $(2 x+4)-\\left({ }^{\\frac{1}{3} x+4}\\right)=4$, 解得: $x={ }^{\\frac{12}{5}}$,\n\n当 $x={ }^{\\frac{12}{5}}$ 时, $\\quad y=\\frac{1}{3} x+4={ }^{\\frac{24}{5}}$,\n\n所以 $\\mathrm{N}_{2}\\left(-\\frac{12}{5}, \\frac{16}{5}\\right) ;$\n\n$\\mathrm{OA}$ 为平行四边形的对角线时, 由上可知 $\\mathrm{AM}_{1} \\mathrm{ON}_{2}$ 为平行四边形, 此时 $\\mathrm{N}_{2}\\left(-\\frac{12}{5}, \\frac{16}{5}\\right)$;\n\n综上可知 $\\mathrm{N}$ 点坐标为 $\\left(-\\frac{12}{5}, \\frac{16}{5}\\right) \\quad$ 或 $\\left(\\frac{12}{5}, \\frac{24}{5}\\right)$\n\n故答案为: $\\left(-\\frac{12}{5}, \\frac{16}{5}\\right)$ 或 $\\left(\\frac{12}{5}, \\frac{24}{5}\\right)$ 。", "solution": "null", "level": "八年级", "question": "(15 分) 如图, 已知直线 $l_{1}: y=2 x+4$ 与 $y$ 轴交于 $A$ 点, 与 $x$ 轴交于点 $B$, 经过 $A$ 点的直线 $l_{2}$ 与直线 $\\mathrm{l}_{1}$ 所夹的锐角为 $45^{\\circ}$.\n\n\n\n(1) 过点 $B$ 作 $C B \\perp A B$, 交 $l_{2}$ 于 $C$, 求点 $C$ 的坐标.\n\n(2) 求 $l_{2}$ 的函数解析式.\n\n(3) 在直线 $l_{1}$ 上存在点 $M$, 直线 $l_{2}$ 上存在点 $N$, 使得点 $A 、 O 、 M 、 N$ 四点组成的四边形是平行四边形, 请直接写出点 $\\mathrm{N}$ 的坐标.", "options": [], "subject": "解析几何", "analysis": " (1) 解: 过作 $\\mathrm{CD} \\perp \\mathrm{x}$ 轴于点 $\\mathrm{D}$,\n\n\n\n$\\because \\mathrm{CB} \\perp \\mathrm{AB}$\n\n$\\therefore \\angle \\mathrm{ABC}=90^{\\circ}, \\quad \\therefore \\angle \\mathrm{CBD}+\\angle \\mathrm{ABO}=90^{\\circ}$,\n\n又 $\\because \\angle \\mathrm{BAC}=45^{\\circ}$,\n\n$\\therefore \\angle \\mathrm{ACB}=90^{\\circ}-\\angle \\mathrm{BAC}=45^{\\circ}=\\angle \\mathrm{BAC}$,\n\n$\\therefore \\mathrm{BC}=\\mathrm{BA}$,\n$\\because \\angle \\mathrm{AOB}=90^{\\circ}, \\quad \\therefore \\angle \\mathrm{ABO}+\\angle \\mathrm{BAO}=90^{\\circ}$,\n\n$\\therefore \\angle \\mathrm{CBD}=\\angle \\mathrm{BAO}$,\n\n又 $\\because \\angle \\mathrm{BDC}=\\angle \\mathrm{AOB}=90^{\\circ}$,\n\n$\\therefore \\triangle \\mathrm{BDC} \\cong \\triangle \\mathrm{AOB}$\n\n$\\therefore \\mathrm{BD}=\\mathrm{OA}, \\mathrm{CD}=\\mathrm{OB}$,\n\n$\\because$ 直线 $\\mathrm{l}_{1}: \\mathrm{y}=2 \\mathrm{x}+4$,\n\n$\\therefore \\mathrm{A}(0,4), \\mathrm{B}(-2,0)$,\n\n$\\therefore \\mathrm{BD}=\\mathrm{OA}=4, \\mathrm{CD}=\\mathrm{OB}=2$,\n\n$\\therefore \\mathrm{OD}=4+2=6$,\n\n$\\therefore \\mathrm{C}(-6,2)$\n\n(2) 解:设 $\\mathrm{l}_{2}$ 的解析式为 $\\mathrm{y}=\\mathrm{kx}+\\mathrm{b}(\\mathrm{k} \\neq 0)$\n\n$\\because$ A $(0,4), C(-6,2)$,\n\n$\\therefore \\begin{gathered}b=4 \\\\ -6 k+b=3\\end{gathered}$\n\n$\\left\\{\\begin{array}{l}k=\\frac{1}{3} \\\\ b=4\\end{array}\\right.$\n\n$\\therefore y=\\frac{1}{3} x+4$\n\n(3)解:如图, $\\mathrm{OA}$ 为平行四边形的边时,\n\n\n\n当四边形 $A O M_{1} N_{1}$ 为平行四边形时, 有 $M_{1} N_{1}=A O=4$,\n\n即 $\\left(3^{\\frac{1}{3} x+4}\\right)-(2 x+4)=4$, 解得: $x==^{-\\frac{12}{5}}$,\n当 $x={ }^{-\\frac{12}{5}}$ 时, $\\quad y=\\frac{1}{3} x+4=\\frac{16}{5}$,\n\n所以 $\\mathrm{N}_{1}\\left(-\\frac{12}{5}, \\frac{16}{5}\\right)$;\n\n当四边形 $\\mathrm{AOM}_{2} \\mathrm{~N}_{2}$ 为平行四边形时, 有 $\\mathrm{M}_{2} \\mathrm{~N}_{2}=\\mathrm{AO}=4$,\n\n即 $(2 x+4)-\\left({ }^{\\frac{1}{3} x+4}\\right)=4$, 解得: $x={ }^{\\frac{12}{5}}$,\n\n当 $x={ }^{\\frac{12}{5}}$ 时, $\\quad y=\\frac{1}{3} x+4={ }^{\\frac{24}{5}}$,\n\n所以 $\\mathrm{N}_{2}\\left(-\\frac{12}{5}, \\frac{16}{5}\\right) ;$\n\n$\\mathrm{OA}$ 为平行四边形的对角线时, 由上可知 $\\mathrm{AM}_{1} \\mathrm{ON}_{2}$ 为平行四边形, 此时 $\\mathrm{N}_{2}\\left(-\\frac{12}{5}, \\frac{16}{5}\\right)$;\n\n综上可知 $\\mathrm{N}$ 点坐标为 $\\left(-\\frac{12}{5}, \\frac{16}{5}\\right) \\quad$ 或 $\\left(\\frac{12}{5}, \\frac{24}{5}\\right)$\n\n故答案为: $\\left(-\\frac{12}{5}, \\frac{16}{5}\\right)$ 或 $\\left(\\frac{12}{5}, \\frac{24}{5}\\right)$ 。"} {"id": "5821", "image": ["7218.jpg", "7219.jpg", "7220.jpg", "7221.jpg", "7220.jpg", "7221.jpg"], "answer": " (1) 解:如图 2 所示:\n\n\n\n(2) 证明:在图 1 中, 过点 $\\mathrm{B}$ 作 $\\mathrm{BD} \\perp \\mathrm{x}$ 轴, 交 $\\mathrm{x}$ 轴于点 $\\mathrm{D}$.根据题意可证 $\\triangle \\mathrm{AOC} \\sim \\triangle \\mathrm{CDB}$.\n\n$\\therefore \\frac{A O}{C D}=\\frac{O C}{B D}$\n\n$\\therefore \\frac{1}{5-m}=\\frac{m}{2}$\n\n$\\therefore \\mathrm{m}(5-\\mathrm{m})=2$.\n\n$\\therefore \\mathrm{m}^{2}-5 \\mathrm{~m}+2=0$.\n\n$\\therefore \\mathrm{m}$ 是方程 $\\mathrm{x}^{2}-5 \\mathrm{x}+2=0$ 的实数根.\n\n(3)解:方程 $a x^{2}+b x+c=0 ( a \\neq 0 )$ 可化为\n\n$$\nx^{2}+\\frac{b}{a} x^{\\frac{c}{a}}+{ }^{\\frac{c}{a}}=0\n$$\n\n模仿研究小组作法可得: A $(0,1), \\mathrm{B}\\left(-^{\\frac{b}{a}},{ }^{\\frac{c}{a}}\\right)$ 或 $\\mathrm{A}\\left(0,{ }^{\\frac{1}{a}}\\right), \\mathrm{B}\\left(-^{\\frac{b}{a}}\\right.$, c) 等.\n\n(4)解:以图 3 为例: $P\\left(m_{1}, \\mathrm{n}_{1}\\right) Q\\left(m_{2}, \\mathrm{n}_{2}\\right)$,\n\n设方程的根为 $\\mathrm{x}$, 根据三角形相似可得. $\\frac{m_{1}}{x-m_{1}}=\\frac{m_{2}-x}{n_{2}}$.\n\n上式可化为 $x^{2}-\\left(m_{1}+m_{2}\\right) x+m_{1} m_{2}+n_{1} n_{2}=0$.\n\n又 $a x^{2}+b x+c=0$,\n即 $\\mathrm{x}^{2}+{ }^{\\frac{b}{a}} \\mathrm{x}^{\\frac{c}{a}} \\stackrel{\\frac{c}{a}}{=}=0$.\n\n比较系数可得: $\\mathrm{m}_{1}+\\mathrm{m}_{2}=-\\frac{}{\\frac{b}{a}}$.\n\n$\\mathrm{m}_{1} \\mathrm{~m}_{2}+\\mathrm{n}_{1} \\mathrm{n}_{2}=\\frac{\\frac{c}{a}}{}$\n\n\n\n五、作图题", "solution": "null", "level": "八年级", "question": "( 20 分 ) 在平面直角坐标系中, 借助直角三角板可以找到一元二次方程的实数根, 比如对于方程 $x^{2}-5 x+2=0$, 操作步骤是:\n\n第一步:根据方程系数特征,确定一对固定点 A $(0,1) , \\mathrm{~B}(5,2)$;\n\n第二步: 在坐标平面中移动一个直角三角板, 使一条直角边恒过点 $\\mathrm{A}$, 另一条直角边恒过点 $\\mathrm{B}$;\n\n第三步: 在移动过程中, 当三角板的直角顶点落在 $\\mathrm{x}$ 轴上点 $\\mathrm{C}$ 处时, 点 $\\mathrm{C}$ 的横坐标 $\\mathrm{m}$ 即为该方程的一个实数根(如图 1)\n\n第四步:调整三角板直角顶点的位置, 当它落在 $\\mathrm{x}$ 轴上另一点 $\\mathrm{D}$ 处时, 点 $\\mathrm{D}$ 的横坐标为 $\\mathrm{n}$ 即为该方程的另一个实数根。\n\n\n\n图 1\n\n\n\n图2\n\n(1)在图 2 中,按照“第四步“的操作方法作出点 $\\mathrm{D}$ (请保留作出点 $\\mathrm{D}$ 时直角三角板两条直角边的痕迹)\n\n(2)结合图 1, 请证明“第三步”操作得到的 $\\mathrm{m}$ 就是方程 $x^{2}-5 x+2=0$ 的一个实数根;\n\n(3)上述操作的关键是确定两个固定点的位置, 若要以此方法找到一元二次方程 $a x^{2}+b x+c=0\\left(a \\neq 0, b^{2}-4 a c \\geq 0\\right)$ 的实数根, 请你直接写出一对固定点的坐标;\n\n(4)实际上, (3)中的固定点有无数对, 一般地,当 $m_{1}, n_{1}, m_{2}, n_{2}$ 与 a, b, c 之间满足怎样的关系时, 点 $\\mathrm{P}\\left(m_{1}, n_{1}\\right), \\mathrm{Q}\\left(m_{2}, n_{2}\\right)$ 就是符合要求的一对固定点?", "options": [], "subject": "解析几何", "analysis": " (1) 解:如图 2 所示:\n\n\n\n(2) 证明:在图 1 中, 过点 $\\mathrm{B}$ 作 $\\mathrm{BD} \\perp \\mathrm{x}$ 轴, 交 $\\mathrm{x}$ 轴于点 $\\mathrm{D}$.根据题意可证 $\\triangle \\mathrm{AOC} \\sim \\triangle \\mathrm{CDB}$.\n\n$\\therefore \\frac{A O}{C D}=\\frac{O C}{B D}$\n\n$\\therefore \\frac{1}{5-m}=\\frac{m}{2}$\n\n$\\therefore \\mathrm{m}(5-\\mathrm{m})=2$.\n\n$\\therefore \\mathrm{m}^{2}-5 \\mathrm{~m}+2=0$.\n\n$\\therefore \\mathrm{m}$ 是方程 $\\mathrm{x}^{2}-5 \\mathrm{x}+2=0$ 的实数根.\n\n(3)解:方程 $a x^{2}+b x+c=0 ( a \\neq 0 )$ 可化为\n\n$$\nx^{2}+\\frac{b}{a} x^{\\frac{c}{a}}+{ }^{\\frac{c}{a}}=0\n$$\n\n模仿研究小组作法可得: A $(0,1), \\mathrm{B}\\left(-^{\\frac{b}{a}},{ }^{\\frac{c}{a}}\\right)$ 或 $\\mathrm{A}\\left(0,{ }^{\\frac{1}{a}}\\right), \\mathrm{B}\\left(-^{\\frac{b}{a}}\\right.$, c) 等.\n\n(4)解:以图 3 为例: $P\\left(m_{1}, \\mathrm{n}_{1}\\right) Q\\left(m_{2}, \\mathrm{n}_{2}\\right)$,\n\n设方程的根为 $\\mathrm{x}$, 根据三角形相似可得. $\\frac{m_{1}}{x-m_{1}}=\\frac{m_{2}-x}{n_{2}}$.\n\n上式可化为 $x^{2}-\\left(m_{1}+m_{2}\\right) x+m_{1} m_{2}+n_{1} n_{2}=0$.\n\n又 $a x^{2}+b x+c=0$,\n即 $\\mathrm{x}^{2}+{ }^{\\frac{b}{a}} \\mathrm{x}^{\\frac{c}{a}} \\stackrel{\\frac{c}{a}}{=}=0$.\n\n比较系数可得: $\\mathrm{m}_{1}+\\mathrm{m}_{2}=-\\frac{}{\\frac{b}{a}}$.\n\n$\\mathrm{m}_{1} \\mathrm{~m}_{2}+\\mathrm{n}_{1} \\mathrm{n}_{2}=\\frac{\\frac{c}{a}}{}$\n\n\n\n五、作图题"} {"id": "5825", "image": ["7224.jpg", "7225.jpg", "7225.jpg"], "answer": " (1) 解: $\\because$ 二次函数 $\\mathrm{y}=\\mathrm{x}^{2}-4 \\sqrt{3} \\mathrm{x}+\\mathrm{m}$ 的图象与 $\\mathrm{x}$ 轴相交于不同的两点,\n\n$\\therefore \\triangle=(-4 \\sqrt{3})^{2}-4 \\times 1 \\times m>0$,\n\n解得: $\\mathrm{m}<12$\n\n(2) 解: $\\because \\mathrm{OC}=6$,\n\n$\\therefore \\mathrm{m}=6$,\n\n$\\therefore$ 二次函数解析式为 $\\mathrm{y}=\\mathrm{x}^{2}-4 \\sqrt{3} \\mathrm{x}+6$,\n\n$\\therefore$ 抛物线的顶点坐标为 $\\left(-\\frac{-4 \\sqrt{3}}{2 \\times 1}, \\frac{4 \\times 1 \\times 6-(-4 \\sqrt{3})^{2}}{4 \\times 1}\\right)$, 即 $(2 \\sqrt{3},-6)$.\n\n(3) 解: $\\because$ 二次函数 $\\mathrm{y}=\\mathrm{x}^{2}-4 \\sqrt{3} \\mathrm{x}+\\mathrm{m}$ 的图象与 $\\mathrm{x}$ 轴相交于点 $\\mathrm{A}\\left(\\mathrm{x}_{1}, 0\\right), \\mathrm{B}\\left(\\mathrm{x}_{2}, 0\\right)$,\n\n$\\therefore \\mathrm{x}_{1}+\\mathrm{x}_{2}=4 \\sqrt{3}, \\mathrm{x}_{1} \\mathrm{x}_{2}=\\mathrm{m}$\n\n$\\therefore \\mathrm{AB}=\\mathrm{X}_{2}-\\mathrm{x}_{1}=\\sqrt{\\left(x_{1}+x_{2}\\right)^{2}-4 x_{1} x_{2}}=2 \\sqrt{12-m}$.\n\n设对称轴与 $\\mathrm{x}$ 轴的交点为 $\\mathrm{H}$, 如图所示.\n\n\n\n$\\because$ 二次函数解析式为 $\\mathrm{y}=\\mathrm{x}^{2}-4 \\sqrt{3} \\mathrm{x}+\\mathrm{m}$,\n\n$\\therefore$ 点 D 的坐标为 $\\left(-{ }^{\\frac{-4 \\sqrt{3}}{2 \\times 1}}, \\frac{4 \\times 1 \\times m-(-4 \\sqrt{3})^{2}}{4 \\times 1}-\\right)$, 即 $(2 \\sqrt{3}, \\mathrm{~m}-12)$,\n\n$\\therefore \\mathrm{DH}=12-\\mathrm{m}$.\n\n$\\because \\triangle \\mathrm{ABD}$ 为等边三角形,\n\n$\\therefore \\mathrm{DH}={ }^{\\frac{\\sqrt{3}}{2}} \\mathrm{AB}=\\sqrt{36-3 m}$\n\n$\\therefore 12-\\mathrm{m}=\\sqrt{36-3 m}$\n\n解得: $m_{1}=12$ (不合题意, 舍去) , $m_{2}=9$.\n\n$\\therefore$ 当 $\\triangle A B D$ 为等边三角形时, $m$ 的值为 9 .", "solution": "null", "level": "八年级", "question": "(15 分 ) 如图, 已知二次函数 $y=x^{2}-4 \\sqrt{3} x+m$ 的图象与 $x$ 轴相交于不同的两点 $A\\left(x_{1}, 0\\right)$ 、 B $\\left(x_{2}, 0\\right)$ 且 $x_{1}\n\n(1) 求 $m$ 的取值范围.\n\n(2) 当 $\\mathrm{OC}=6$ 时, 求抛物线的顶点坐标.\n\n(3)设抛物线的顶点为 $D$, 当 $\\triangle A B D$ 为等边三角形时, 求 $m$ 的值.", "options": [], "subject": "解析几何", "analysis": " (1) 解: $\\because$ 二次函数 $\\mathrm{y}=\\mathrm{x}^{2}-4 \\sqrt{3} \\mathrm{x}+\\mathrm{m}$ 的图象与 $\\mathrm{x}$ 轴相交于不同的两点,\n\n$\\therefore \\triangle=(-4 \\sqrt{3})^{2}-4 \\times 1 \\times m>0$,\n\n解得: $\\mathrm{m}<12$\n\n(2) 解: $\\because \\mathrm{OC}=6$,\n\n$\\therefore \\mathrm{m}=6$,\n\n$\\therefore$ 二次函数解析式为 $\\mathrm{y}=\\mathrm{x}^{2}-4 \\sqrt{3} \\mathrm{x}+6$,\n\n$\\therefore$ 抛物线的顶点坐标为 $\\left(-\\frac{-4 \\sqrt{3}}{2 \\times 1}, \\frac{4 \\times 1 \\times 6-(-4 \\sqrt{3})^{2}}{4 \\times 1}\\right)$, 即 $(2 \\sqrt{3},-6)$.\n\n(3) 解: $\\because$ 二次函数 $\\mathrm{y}=\\mathrm{x}^{2}-4 \\sqrt{3} \\mathrm{x}+\\mathrm{m}$ 的图象与 $\\mathrm{x}$ 轴相交于点 $\\mathrm{A}\\left(\\mathrm{x}_{1}, 0\\right), \\mathrm{B}\\left(\\mathrm{x}_{2}, 0\\right)$,\n\n$\\therefore \\mathrm{x}_{1}+\\mathrm{x}_{2}=4 \\sqrt{3}, \\mathrm{x}_{1} \\mathrm{x}_{2}=\\mathrm{m}$\n\n$\\therefore \\mathrm{AB}=\\mathrm{X}_{2}-\\mathrm{x}_{1}=\\sqrt{\\left(x_{1}+x_{2}\\right)^{2}-4 x_{1} x_{2}}=2 \\sqrt{12-m}$.\n\n设对称轴与 $\\mathrm{x}$ 轴的交点为 $\\mathrm{H}$, 如图所示.\n\n\n\n$\\because$ 二次函数解析式为 $\\mathrm{y}=\\mathrm{x}^{2}-4 \\sqrt{3} \\mathrm{x}+\\mathrm{m}$,\n\n$\\therefore$ 点 D 的坐标为 $\\left(-{ }^{\\frac{-4 \\sqrt{3}}{2 \\times 1}}, \\frac{4 \\times 1 \\times m-(-4 \\sqrt{3})^{2}}{4 \\times 1}-\\right)$, 即 $(2 \\sqrt{3}, \\mathrm{~m}-12)$,\n\n$\\therefore \\mathrm{DH}=12-\\mathrm{m}$.\n\n$\\because \\triangle \\mathrm{ABD}$ 为等边三角形,\n\n$\\therefore \\mathrm{DH}={ }^{\\frac{\\sqrt{3}}{2}} \\mathrm{AB}=\\sqrt{36-3 m}$\n\n$\\therefore 12-\\mathrm{m}=\\sqrt{36-3 m}$\n\n解得: $m_{1}=12$ (不合题意, 舍去) , $m_{2}=9$.\n\n$\\therefore$ 当 $\\triangle A B D$ 为等边三角形时, $m$ 的值为 9 ."} {"id": "5068", "image": ["6619.jpg"], "answer": "(1)由矩形面积公式得出方程,解方程即可;(2)根据题意可得方程$x(40-x)=400$,求出$x$的值,然后再根据$P$处这棵树是否被围在花园内进行分析即可.【解析】解:(1)$becauseAB=x$米,$thereforeBC=(40-x)$米,由题意得:$x(40-x)=300$,解得:$x_{1}=10,x_{2}=30$,即$x$的值为10或30;(2)花园的面积不能为400米$^{2}$,理由如下:由题意得:$x(40-x)=400$,解得:$x_{1}=x_{2}=20$,当$x=20$时,$26-x=26-6=20$,即当$AB=20$米,$BC=20$米$<24$米,这棵树没有被围在花园内,$therefore$将这棵树围在矩形花园内(含边界,不考虑树的粗细),则花园的面积不能为400米${}^{2}$.【点睛】本题考查了一元二次方程的应用,找准等量关系,正确列出一元二次方程是解题的关键.", "solution": "null", "level": "八年级", "question": "某社区在开展“美化社区,幸福家园”活动中,计划利用如图所示的直角墙角(阴影部分,两边足够长),用40米长的篦笆围成一个矩形花园$ABCD$(篦笆只围$AB,AD$两边),设$AB=x$米.(1)若花园的面积为300米${}^{2}$,求$x$的值;(2)若在直角墙角内点$P$处有一棵桂花树,且与墙$BC,CD$的距离分别是10米,24米,要将这棵树围在矩形花园内(含边界,不考虑树的粗细),则花园的面积能否为400米${}^{2}$?若能,求出$x$的值;若不能,请说明理由.", "options": [], "subject": "解析几何", "analysis": "(1)由矩形面积公式得出方程,解方程即可;(2)根据题意可得方程$x(40-x)=400$,求出$x$的值,然后再根据$P$处这棵树是否被围在花园内进行分析即可.【解析】解:(1)$becauseAB=x$米,$thereforeBC=(40-x)$米,由题意得:$x(40-x)=300$,解得:$x_{1}=10,x_{2}=30$,即$x$的值为10或30;(2)花园的面积不能为400米$^{2}$,理由如下:由题意得:$x(40-x)=400$,解得:$x_{1}=x_{2}=20$,当$x=20$时,$26-x=26-6=20$,即当$AB=20$米,$BC=20$米$<24$米,这棵树没有被围在花园内,$therefore$将这棵树围在矩形花园内(含边界,不考虑树的粗细),则花园的面积不能为400米${}^{2}$.【点睛】本题考查了一元二次方程的应用,找准等量关系,正确列出一元二次方程是解题的关键."} {"id": "5132", "image": [], "answer": "(1)一个多边形的内角和等于外角和的3倍多$180^{circ}$,而任何多边形的外角和是$360^{circ}$,因而多边形的内角和等于$1260^{circ}cdotn$边形的内角和可以表示成$(n-2)cdot180^{circ}$,设这个正多边形的边数是$n$,就得到方程,从而求出边数,即可求出答案.(2)从$n$边形的一个顶点引出对角线,可以引$(n-3)$条.【解析】解:(1)设这个多边形的边数为$n$,则内角和为$180^{circ}(n-2)$,依题意得:$180^{circ}(n-2)=360^{circ}times3+180^{circ}$,解得$n=9$,答:这个多边形是九边形;(2)从$n$边形的一个顶点引出对角线,可以引$(n-3)$条,则从十边形的一个顶点引出对角线,可以引7条.正十边形的一个内角为$(10-2)times180^{circ}div10=144^{circ}$【点睛】本题主要考查多边形内角与外角的知识点,此题要结合多边形的内角和公式寻求等量关系,构建方程求解即可.从$n$边形一个顶点可以引$(n-3)$条对角线.", "solution": "null", "level": "八年级", "question": "若一个$n$边形的内角和比它的外角和的3倍多$180^{circ}$,(1)求$n$的值;(2)在(1)条件下,求正$(n+1)$边形的一个内角度数及对角线条数.", "options": [], "subject": "解析几何", "analysis": "(1)一个多边形的内角和等于外角和的3倍多$180^{circ}$,而任何多边形的外角和是$360^{circ}$,因而多边形的内角和等于$1260^{circ}cdotn$边形的内角和可以表示成$(n-2)cdot180^{circ}$,设这个正多边形的边数是$n$,就得到方程,从而求出边数,即可求出答案.(2)从$n$边形的一个顶点引出对角线,可以引$(n-3)$条.【解析】解:(1)设这个多边形的边数为$n$,则内角和为$180^{circ}(n-2)$,依题意得:$180^{circ}(n-2)=360^{circ}times3+180^{circ}$,解得$n=9$,答:这个多边形是九边形;(2)从$n$边形的一个顶点引出对角线,可以引$(n-3)$条,则从十边形的一个顶点引出对角线,可以引7条.正十边形的一个内角为$(10-2)times180^{circ}div10=144^{circ}$【点睛】本题主要考查多边形内角与外角的知识点,此题要结合多边形的内角和公式寻求等量关系,构建方程求解即可.从$n$边形一个顶点可以引$(n-3)$条对角线."} {"id": "5159", "image": ["6698.jpg", "6699.jpg", "6699.jpg"], "answer": "(1)由三角形中位线定理得$EF//AB$且$EF=frac{1}{2}AB$,再证$AD=frac{1}{2}AB$,则$EF//AD$且$EF$$=AD$,即可得出结论;(2)证$triangleFDHcongtriangleDFE$(SSS),即可证得$angleDHF=angleDEF$.【解析】证明:(1)$becauseE、F$分别为$BC、AC$的中点,$thereforeEF$是$triangleABC$的中位线,$thereforeEF//AB$且$EF=frac{1}{2}AB$,$because$点$D$是$AB$的中点,$thereforeAD=frac{1}{2}AB$,$thereforeEF//AD$且$EF=AD$,$therefore$四边形$ADEF$是平行四边形;(2)连接$DF$,如图所示:$becauseAHperpBC$于$H$,$thereforeangleAHB=angleAHC=90^{circ}$$because$点$D、F$分别是$AB、CA$的中点,$thereforeHD=frac{1}{2}AB,FH=frac{1}{2}AC$,$because$点$D、E、F$分别是$AB、BC、CA$的中点,$thereforeEF、DE$都是$triangleABC$的中位线,$thereforeEF=frac{1}{2}AB,DE=frac{1}{2}AC$,$thereforeHD=EF,FH=DE$,在$triangleFDH$和$triangleDFE$中,$left{begin{array}{l}{HD}={EF}{FH}={DE},{DF}={FD}end{array}right.$$thereforetriangleFDHcongtriangleDFE(SSS)$,$thereforeangleDHF=angleDEF$.【点睛】此题考查了平行四边形的性质与判定,三角形的中位线定理,直角三角形斜边上的中线的性质以及三角形全等的判定与性质等知识;熟练掌握平行四边形的判定与性质,证明$triangleFDHcong$$triangleDFE$是解题的关键.", "solution": "null", "level": "八年级", "question": "己知:如图,在$triangleABC$中,$D、E、F$分别是各边的中点,$AH$是高.求证:(1)四边形$ADEF$是平行四边形;(2)$angleDHF=angleDEF$.", "options": [], "subject": "解析几何", "analysis": "(1)由三角形中位线定理得$EF//AB$且$EF=frac{1}{2}AB$,再证$AD=frac{1}{2}AB$,则$EF//AD$且$EF$$=AD$,即可得出结论;(2)证$triangleFDHcongtriangleDFE$(SSS),即可证得$angleDHF=angleDEF$.【解析】证明:(1)$becauseE、F$分别为$BC、AC$的中点,$thereforeEF$是$triangleABC$的中位线,$thereforeEF//AB$且$EF=frac{1}{2}AB$,$because$点$D$是$AB$的中点,$thereforeAD=frac{1}{2}AB$,$thereforeEF//AD$且$EF=AD$,$therefore$四边形$ADEF$是平行四边形;(2)连接$DF$,如图所示:$becauseAHperpBC$于$H$,$thereforeangleAHB=angleAHC=90^{circ}$$because$点$D、F$分别是$AB、CA$的中点,$thereforeHD=frac{1}{2}AB,FH=frac{1}{2}AC$,$because$点$D、E、F$分别是$AB、BC、CA$的中点,$thereforeEF、DE$都是$triangleABC$的中位线,$thereforeEF=frac{1}{2}AB,DE=frac{1}{2}AC$,$thereforeHD=EF,FH=DE$,在$triangleFDH$和$triangleDFE$中,$left{begin{array}{l}{HD}={EF}{FH}={DE},{DF}={FD}end{array}right.$$thereforetriangleFDHcongtriangleDFE(SSS)$,$thereforeangleDHF=angleDEF$.【点睛】此题考查了平行四边形的性质与判定,三角形的中位线定理,直角三角形斜边上的中线的性质以及三角形全等的判定与性质等知识;熟练掌握平行四边形的判定与性质,证明$triangleFDHcong$$triangleDFE$是解题的关键."} {"id": "5201", "image": ["6763.jpg", "6764.jpg", "6765.jpg", "6764.jpg", "6765.jpg"], "answer": "(1)由正方形的性质得出$CD=AD,angleCDG=angleADE=90^{circ},GD=ED$,即可证明$triangleCDG$$congtriangleADE(SAS);$(2)设$DE$交$CG$于$K$,由$triangleCDGcongtriangleADE$,可得$AE=CG,angleCGD=angleAED$,而$angleEKH=angleDKC$,故$angleEHK=angleKDG=90^{circ}$,从而$AEperpCG;$(3)过$E$作$EWperpAD$交$AD$延长线于$W$,由$angleCDE=30^{circ}$,得$angleEDW=60^{circ},angleDEW=30^{circ}$,即得$DW=frac{1}{2}DE=1,EW=sqrt{3}DW=sqrt{3}$,用勾股定理有$AE=sqrt{{A}W^{2}+mathbb{E}W^{2}}=sqrt{19}$,根据$frac{O{~A}}{{AE}}=frac{{AD}}{{AW}}$,得$OA=frac{3sqrt{19}}{4}$.【解析】(1)证明:$because$四边形$ABCD$和四边形$DEFG$都是正方形,$thereforeCD=AD,angleADC=angleEDG=90^{circ},GD=ED$,$thereforeangleADE=angleCDG$在$triangleCDG$和$triangleADE$中,$thereforetriangleCDGcongtriangleADE(SAS);$(2)解:$AEperpCG,AE=CG$,理由如下:设$DE$交$CG$于$K$,如图:由(1)知:$triangleCDGcongtriangleADE$,$thereforeAE=CG,angleCGD=angleAED$,$becauseangleEKH=angleDKC$$thereforeangleEHK=angleKDG=90^{circ}$,$thereforeAEperpCG$(3)解:过$E$作$EWperpAD$交$AD$延长线于$W$,如图:$becauseangleCDE=30^{circ}$$$begin{aligned}&thereforeangleEDW=60^{circ},angleDEW=30^{circ}&thereforeDW=frac{1}{2}DE=1,EW=sqrt{3}DW=sqrt{3}&thereforeAW=AD+DW=3+1=4&thereforeAE=sqrt{{A}W^{2}+mathbb{E}W^{2}}=sqrt{19}&becausefrac{{OA}}{{AE}}=frac{{AD}}{{AW}}&thereforefrac{{OA}}{sqrt{19}}=frac{3}{4}&thereforeOA=frac{3sqrt{19}}{4}end{aligned}$$$$text{答:}AOtext{的长度为}frac{3sqrt{19}}{4}text{.}$$【点睛】本题考查正方形性质及应用,涉及全等三角形的判定与性质,勾股定理及应用,解题的关键是掌握全等三角形的判定定理.", "solution": "null", "level": "八年级", "question": "如图,已知边长为3的正方形$ABCD$和边长为2的正方形$DEFG$公共点$D$,连接$AE、CG$相交于点$H,AE$与$CD$相交于点$O$.(1)求证:$triangleADEcongtriangleCDG$;(2)猜想:线段$AE$与$CG$的关系,并说明理由;(3)当$angleCDE=30^{circ}$时,求$AO$的长度.", "options": [], "subject": "解析几何", "analysis": "(1)由正方形的性质得出$CD=AD,angleCDG=angleADE=90^{circ},GD=ED$,即可证明$triangleCDG$$congtriangleADE(SAS);$(2)设$DE$交$CG$于$K$,由$triangleCDGcongtriangleADE$,可得$AE=CG,angleCGD=angleAED$,而$angleEKH=angleDKC$,故$angleEHK=angleKDG=90^{circ}$,从而$AEperpCG;$(3)过$E$作$EWperpAD$交$AD$延长线于$W$,由$angleCDE=30^{circ}$,得$angleEDW=60^{circ},angleDEW=30^{circ}$,即得$DW=frac{1}{2}DE=1,EW=sqrt{3}DW=sqrt{3}$,用勾股定理有$AE=sqrt{{A}W^{2}+mathbb{E}W^{2}}=sqrt{19}$,根据$frac{O{~A}}{{AE}}=frac{{AD}}{{AW}}$,得$OA=frac{3sqrt{19}}{4}$.【解析】(1)证明:$because$四边形$ABCD$和四边形$DEFG$都是正方形,$thereforeCD=AD,angleADC=angleEDG=90^{circ},GD=ED$,$thereforeangleADE=angleCDG$在$triangleCDG$和$triangleADE$中,$thereforetriangleCDGcongtriangleADE(SAS);$(2)解:$AEperpCG,AE=CG$,理由如下:设$DE$交$CG$于$K$,如图:由(1)知:$triangleCDGcongtriangleADE$,$thereforeAE=CG,angleCGD=angleAED$,$becauseangleEKH=angleDKC$$thereforeangleEHK=angleKDG=90^{circ}$,$thereforeAEperpCG$(3)解:过$E$作$EWperpAD$交$AD$延长线于$W$,如图:$becauseangleCDE=30^{circ}$$$begin{aligned}&thereforeangleEDW=60^{circ},angleDEW=30^{circ}&thereforeDW=frac{1}{2}DE=1,EW=sqrt{3}DW=sqrt{3}&thereforeAW=AD+DW=3+1=4&thereforeAE=sqrt{{A}W^{2}+mathbb{E}W^{2}}=sqrt{19}&becausefrac{{OA}}{{AE}}=frac{{AD}}{{AW}}&thereforefrac{{OA}}{sqrt{19}}=frac{3}{4}&thereforeOA=frac{3sqrt{19}}{4}end{aligned}$$$$text{答:}AOtext{的长度为}frac{3sqrt{19}}{4}text{.}$$【点睛】本题考查正方形性质及应用,涉及全等三角形的判定与性质,勾股定理及应用,解题的关键是掌握全等三角形的判定定理."} {"id": "5206", "image": ["6777.jpg", "6778.jpg"], "answer": "(1)由正方形的性质得出条件,证明$triangleBCMcongtriangleDCF(SAS)$,由全等三角形的性质及角的互余关系可得结论;(2)(1)结论仍成立;(2)设正方形$ABCD$的边长为$x$,则$BC=CD=x$,由勾股定理求得$BD$的长,再用含$x$的式子表示出$BF$,然后根据$BD=BF$得出关于$x$的方程,解得$x$的值,再乘以4即可.【解析】解:(1)证明:$because$四边形$ABCD$与四边形$CFGM$都是正方形,$thereforeangleBCM=angleFCD=90^{circ},BC=CD,CM=CF$.在$triangleBCM$和$triangleDCF$中,$left{begin{array}{l}{BC}={CD}angle{BCM}=angle{FCD},{CM}={CF}end{array}right.$$thereforetriangleBCMcongtriangleDCF(SAS)$.$thereforeDF=BM,angleCFD=angleCMB$.$becauseangleBMC+angleCBM=90^{circ}$$thereforeangleCBM+angleCFD=90^{circ}$,$thereforeangleBEF=90^{circ}$,$thereforeDFperpBM;$(2)(1)成立.$because$四边形$ABCD$与四边形$CFGM$都是正方形,$thereforeangleBCM=angleFCD=90^{circ},BC=CD,CM=CF$.在$triangleBCM$和$triangleDCF$中,$left{begin{array}{l}{BC}={CD}angle{BCM}=angle{FCD},{CM}={CF}end{array}right.$$thereforetriangleBCMcongtriangleDCF(SAS)$.$thereforeDF=BM,angleCFD=angleCMB$.$becauseangleBMC+angleCBM=90^{circ}$,$thereforeangleCBM+angleCFD=90^{circ}$,$thereforeangleBEF=90^{circ}$,$thereforeDFperpBM$(2)设正方形$ABCD$的边长为$x$,则$BC=CD=x$,$thereforeBD=sqrt{BC^{2}+CD^{2}}=sqrt{2}x$,$because$正方形$CFGM$的边长为1,$thereforeBF=BC+CF=x+1$.$becauseBD=BF$,$thereforesqrt{2}x=x+1$,$thereforex=sqrt{2}+1$.$therefore4x=4sqrt{2}+4$.$therefore$正方形$ABCD$的周长为$4sqrt{2}+4$.【点睛】本题考查了正方形的性质、全等三角形的判定与性质、勾股定理及一元一次方程在几何图形问题中的应用,明确相关性质及定理是解题的关键.", "solution": "null", "level": "八年级", "question": "正方形$ABCD$中,$M$为射线$CD$上一点(不与$D$重合),以$CM$为边,在正方形$ABCD$的异侧作正方形$CFGM$,连接$BM,DF$,直线$BM$与$DF$交于点$E$.(1)如图1,若$M$在$CD$的延长线上,求证:$DF=BM,DFperpBM$;(2)如图2,若$M$移到边$CD$上.(1)在(1)中结论是否仍成立?(直接回答不需证明)(2)连接$BD$,若$BD=BF$,且正方形$CFGM$的边长为1,试求正方形$ABCD$的周长.图1图2", "options": [], "subject": "解析几何", "analysis": "(1)由正方形的性质得出条件,证明$triangleBCMcongtriangleDCF(SAS)$,由全等三角形的性质及角的互余关系可得结论;(2)(1)结论仍成立;(2)设正方形$ABCD$的边长为$x$,则$BC=CD=x$,由勾股定理求得$BD$的长,再用含$x$的式子表示出$BF$,然后根据$BD=BF$得出关于$x$的方程,解得$x$的值,再乘以4即可.【解析】解:(1)证明:$because$四边形$ABCD$与四边形$CFGM$都是正方形,$thereforeangleBCM=angleFCD=90^{circ},BC=CD,CM=CF$.在$triangleBCM$和$triangleDCF$中,$left{begin{array}{l}{BC}={CD}angle{BCM}=angle{FCD},{CM}={CF}end{array}right.$$thereforetriangleBCMcongtriangleDCF(SAS)$.$thereforeDF=BM,angleCFD=angleCMB$.$becauseangleBMC+angleCBM=90^{circ}$$thereforeangleCBM+angleCFD=90^{circ}$,$thereforeangleBEF=90^{circ}$,$thereforeDFperpBM;$(2)(1)成立.$because$四边形$ABCD$与四边形$CFGM$都是正方形,$thereforeangleBCM=angleFCD=90^{circ},BC=CD,CM=CF$.在$triangleBCM$和$triangleDCF$中,$left{begin{array}{l}{BC}={CD}angle{BCM}=angle{FCD},{CM}={CF}end{array}right.$$thereforetriangleBCMcongtriangleDCF(SAS)$.$thereforeDF=BM,angleCFD=angleCMB$.$becauseangleBMC+angleCBM=90^{circ}$,$thereforeangleCBM+angleCFD=90^{circ}$,$thereforeangleBEF=90^{circ}$,$thereforeDFperpBM$(2)设正方形$ABCD$的边长为$x$,则$BC=CD=x$,$thereforeBD=sqrt{BC^{2}+CD^{2}}=sqrt{2}x$,$because$正方形$CFGM$的边长为1,$thereforeBF=BC+CF=x+1$.$becauseBD=BF$,$thereforesqrt{2}x=x+1$,$thereforex=sqrt{2}+1$.$therefore4x=4sqrt{2}+4$.$therefore$正方形$ABCD$的周长为$4sqrt{2}+4$.【点睛】本题考查了正方形的性质、全等三角形的判定与性质、勾股定理及一元一次方程在几何图形问题中的应用,明确相关性质及定理是解题的关键."} {"id": "4087", "image": ["5764.jpg", "5765.jpg", "5765.jpg"], "answer": "解: (1) 如图所示, 线段 $A D$ 即为所求;\n\n(2) 如图所示, 线段 BE 即为所求;\n\n(3) $\\mathrm{S}_{\\triangle \\boldsymbol{A c c}}=\\frac{1}{2} \\mathrm{BC} \\cdot \\mathrm{AD}=\\frac{1}{2} \\times 4 \\times 4=8$.\n\n$\\therefore \\triangle A B E$ 的面积 $=\\frac{1}{2} S_{\\triangle A B C}=4$,\n\n故答案为: 4 .\n\n", "solution": "null", "level": "八年级", "question": "方格纸中每个小正方形的边长均为 1 , 点 $A, B, C$ 在小正方形的顶点上.\n\n(1) 画出 $\\triangle A B C$ 中边 $B C$ 上的高 $A D$;\n\n(2) 画出 $\\triangle A B C$ 中边 $A C$ 上的中线 $B E$;\n\n(3) 求出 $\\triangle A B E$ 的面积.\n\n", "options": [], "subject": "解析几何", "analysis": "解: (1) 如图所示, 线段 $A D$ 即为所求;\n\n(2) 如图所示, 线段 BE 即为所求;\n\n(3) $\\mathrm{S}_{\\triangle \\boldsymbol{A c c}}=\\frac{1}{2} \\mathrm{BC} \\cdot \\mathrm{AD}=\\frac{1}{2} \\times 4 \\times 4=8$.\n\n$\\therefore \\triangle A B E$ 的面积 $=\\frac{1}{2} S_{\\triangle A B C}=4$,\n\n故答案为: 4 .\n\n"} {"id": "5222", "image": ["6792.jpg", "6792.jpg"], "answer": "(1)由正比例函数的解析式求得点$P$的坐标,代入数${y}_{1}=frac{{k}}{{x}}$($k$为常数,$kneq0$)即可求得$k$的值;(2)求得$P^{prime}$的坐标为$(-2,-3)$,由$-2times(-3)=6neqk$,即可判断点$P^{prime}$不在函数$y_{1}$的图象上;(3)利用图象即可求解.【解析】解:(1)$because$正比例函数$y_{2}=x$的图象过交点为$P(3,m)$,$thereforem=3$,$thereforeP(3,3)$,$because$点$P$在反比例函数${y}_{1}=frac{{k}}{{x}}(k$为常数,$kneq0)$的图象上,$thereforek=3times3=9;$(2)将点$P$向下平移6个单位,再向左平移5个单位后,得点$P^{prime}(-2,-3)$,$because-2times(-3)=6neq9$,$therefore$点$P^{prime}$不在函数$y_{1}$的图象上;(3)由图象可知,当$y_{1}>y_{2}$时,自变量$x$的取值范围是$x<-3$或$0【点睛】本题是反比例函数于一次函数的交点问题,考查了一次函数图象上点的坐标特征,待定系数法求反比例函数的解析式,函数与不等式的关系,数形结合是解题的关键.", "solution": "null", "level": "八年级", "question": "已知反比例函数${y}_{1}=frac{{k}}{{x}}(k$为常数,$kneq0)$与正比例函数$y_{2}=x$的图象有一个交点为$P(3,m)$.(1)求$k$的值;(2)将点$P$向下平移6个单位,再向左平移5个单位后,得点$P^{prime}$,试判断点$P^{prime}$是否在函数$y_{1}$的图象上,并说明理由;(3)当$y_{1}>y_{2}$时,利用函数图象直接写出自变量$x$的取值范围.", "options": [], "subject": "解析几何", "analysis": "(1)由正比例函数的解析式求得点$P$的坐标,代入数${y}_{1}=frac{{k}}{{x}}$($k$为常数,$kneq0$)即可求得$k$的值;(2)求得$P^{prime}$的坐标为$(-2,-3)$,由$-2times(-3)=6neqk$,即可判断点$P^{prime}$不在函数$y_{1}$的图象上;(3)利用图象即可求解.【解析】解:(1)$because$正比例函数$y_{2}=x$的图象过交点为$P(3,m)$,$thereforem=3$,$thereforeP(3,3)$,$because$点$P$在反比例函数${y}_{1}=frac{{k}}{{x}}(k$为常数,$kneq0)$的图象上,$thereforek=3times3=9;$(2)将点$P$向下平移6个单位,再向左平移5个单位后,得点$P^{prime}(-2,-3)$,$because-2times(-3)=6neq9$,$therefore$点$P^{prime}$不在函数$y_{1}$的图象上;(3)由图象可知,当$y_{1}>y_{2}$时,自变量$x$的取值范围是$x<-3$或$0【点睛】本题是反比例函数于一次函数的交点问题,考查了一次函数图象上点的坐标特征,待定系数法求反比例函数的解析式,函数与不等式的关系,数形结合是解题的关键."} {"id": "5223", "image": [], "answer": "(1)把点$(2,3)$代入$y=frac{k}{x}(kneq0)$可得$k$的值,进而可得函数的解析式;(2)把点$B(-1,6)$代入函数解析式,能满足解析式的点就在此函数图象上;(3)根据反比例函数图象位于第一、三象限,分两种情况:当$C$和$D$都在同一象限时,根据$x_{1}$$y_{2}$;当$C$和$D$不在同一象限$x_{1}0$,$therefore$反比例函数$y=frac{{k}}{{x}}(kneq0)$的图象在一、三象限,且在每个象限$y$随$x$的增大而减小,$therefore$当两点在同一象限时,$y_{1}>y_{2}$;当两点在不同象限时,$y_{1}y_{2}$;当$C$和$D$不在同一象限$x_{1}0$,$therefore$反比例函数$y=frac{{k}}{{x}}(kneq0)$的图象在一、三象限,且在每个象限$y$随$x$的增大而减小,$therefore$当两点在同一象限时,$y_{1}>y_{2}$;当两点在不同象限时,$y_{1}y_{1}$时$x$的取值范围为:$04$;(3)设直线$AB$向下平移$n$个单位长度,此时直线$AB$对应的表达式为$y=-x+5-n$,联立方程组得$left{begin{array}{l}y=-x+5-ny=frac{4}{x}end{array}right.$,消去$y$得$-x+5=frac{4}{{x}}$,整理得$x^{2}-(5-n)x+4=0$,$because$由于直线与反比例函数图象只有一个交点,$thereforeDelta=0$,即$[-(5-n)]^{2}-4times1times4=0$,整理得$n^{2}-10n+9=0$,解得$n_{1}=1,n_{2}=9$,$therefore$将直线$AB$向下平移1或9个单位长度,直线与反比例图象只有一个交点.【点睛】本题考查一次函数与反比例函数的交点问题,涉及待定系数法确定函数关系式、利用函数图象解不等式、函数图象平移及图象交点与一元二次方程解得情况等知识点是解决问题的关键.", "solution": "null", "level": "八年级", "question": "如图,一次函数$y_{1}=ax+b$的图象与反比例函数${y}_{2}=frac{{k}}{{x}}({a},{b},k$是常数,$aneq0,kneq0)$的图象交于第一象限$C(1,4),D(4,m)$两点,与坐标轴交于$A、B$两点,连接$OC,OD.quad(O$是坐标原点)(1)求一次函数$y_{1}$与反比例函数$y_{2}$的表达式;(2)直接写出当$y_{2}>y_{1}$时$x$的取值范围;(3)将直线$AB$向下平移多少个单位长度,直线与反比例函数图象只有一个交点?", "options": [], "subject": "解析几何", "analysis": "(1)根据题意,由待定系数法确定函数关系式直接代入点列方程及方程组求解即可得到答案;(2)根据图象即可求解;(3)根据函数图象平移,设直线$AB$向下平移$n$个单位长度,此时直线$AB$对应的表达式为$y=-$$x+5-n$,联立方程组,消去$y$整理得$x^{2}-(5-n)x+4=0$,结合图象只有一个交点,确定$x^{2}-(5$$-n)x+4=0$只有一个解,即$Delta=[-(5-n)]^{2}-4times1times4=0$,解一元二次方程即可得到答案.【解析】解:(1)把$C(1,4)$代入${y}_{2}=frac{{k}}{{x}}({a},{b},k$是常数,$aneq0,kneq0)$,得$k=4$,$therefore$反比例函数的解析式为$y=frac{4}{{x}}$,把$(4,m)$代入$y=frac{4}{{x}}$,得$m=1$,$thereforeD(4,1)$,把$C(1,4),D(4,1)$坐标分别代入$y=ax+b$得$left{begin{array}{l}{k}+{b}=44{k}+{b}=1end{array}right.$,解得$left{begin{array}{l}k=-1b=5end{array}right.$,$therefore$一次函数的解析式为$y=-x+5$;(2)由图可知,当$y_{2}>y_{1}$时$x$的取值范围为:$04$;(3)设直线$AB$向下平移$n$个单位长度,此时直线$AB$对应的表达式为$y=-x+5-n$,联立方程组得$left{begin{array}{l}y=-x+5-ny=frac{4}{x}end{array}right.$,消去$y$得$-x+5=frac{4}{{x}}$,整理得$x^{2}-(5-n)x+4=0$,$because$由于直线与反比例函数图象只有一个交点,$thereforeDelta=0$,即$[-(5-n)]^{2}-4times1times4=0$,整理得$n^{2}-10n+9=0$,解得$n_{1}=1,n_{2}=9$,$therefore$将直线$AB$向下平移1或9个单位长度,直线与反比例图象只有一个交点.【点睛】本题考查一次函数与反比例函数的交点问题,涉及待定系数法确定函数关系式、利用函数图象解不等式、函数图象平移及图象交点与一元二次方程解得情况等知识点是解决问题的关键."} {"id": "5227", "image": [], "answer": "(1)利用关于原点中心对称的点的坐标特征得到$x_{1}=-x_{2},y_{1}=-y_{2}$,则$5x_{1}y_{2}-7x_{2}y_{1}$$=2x_{2}y_{2}$,然后利用反比例函数图象上点的坐标得到$x_{2}y_{2}=k$,从而得到$5x_{1}y_{2}-7x_{2}y_{1}$的值;(2)讨论:当$k>0$时,$A、B$在不同象限,则$a-1<00$,然后分别解不等式即可.【解析】解:(1)$because$点$A,B$关于原点中心对称,$thereforex_{1}=-x_{2},y_{1}=-y_{2}$,$therefore5x_{1}y_{2}-7x_{2}y_{1}=5left(-x_{2}right)cdoty_{2}-7x_{2}left(-y_{2}right)$$=2x_{2}y_{2}$,$becauseBleft(x_{2},y_{2}right)$在反比例函数$y=frac{{k}}{{x}}(kneq0)$图象上,$thereforex_{2}y_{2}=k$,$therefore5x_{1}y_{2}-7x_{2}y_{1}=2k$(2)当$k>0$时,$becausex_{1}=a-1,x_{2}=a+1,quady_{1}0$,解得$a<-1$或$a>1$,综上所述,$k>0$时,$-11$.【点睛】本他考查了反比例函数图象上点的坐标特征:反比例函数图象上点的坐标满足其解析式.也考查了反比例函数的性质.", "solution": "null", "level": "八年级", "question": "己知点$Aleft(x_{1},y_{1}right),Bleft(x_{2},y_{2}right)$是反比例函数$y=frac{k}{x}(kneq0)$图象上的两点.(1)若点$A,B$关于原点中心对称,求$5x_{1}y_{2}-7x_{2}y_{1}$的值;(用含$k$的代数式表示)(2)设$x_{1}=a-1,x_{2}=a+1$,若$y_{1}0$时,$A、B$在不同象限,则$a-1<00$,然后分别解不等式即可.【解析】解:(1)$because$点$A,B$关于原点中心对称,$thereforex_{1}=-x_{2},y_{1}=-y_{2}$,$therefore5x_{1}y_{2}-7x_{2}y_{1}=5left(-x_{2}right)cdoty_{2}-7x_{2}left(-y_{2}right)$$=2x_{2}y_{2}$,$becauseBleft(x_{2},y_{2}right)$在反比例函数$y=frac{{k}}{{x}}(kneq0)$图象上,$thereforex_{2}y_{2}=k$,$therefore5x_{1}y_{2}-7x_{2}y_{1}=2k$(2)当$k>0$时,$becausex_{1}=a-1,x_{2}=a+1,quady_{1}0$,解得$a<-1$或$a>1$,综上所述,$k>0$时,$-11$.【点睛】本他考查了反比例函数图象上点的坐标特征:反比例函数图象上点的坐标满足其解析式.也考查了反比例函数的性质."} {"id": "5228", "image": ["6795.jpg", "6796.jpg", "6796.jpg"], "answer": "(1)把$x=0$代入函数解析式即可得$m$的值,根据图中的点画出图象即可;(2)根据图象和解析式即可进行判断;(3)先写出$y_{1}+y_{2}=frac{{x}_{1}+{x}_{2}+2}{left({x}_{1}+1right)left({x}_{2}+1right)}$,然后根据$x_{1}+x_{2}=-2$,即可得出结果.【解析】解:(1)把$x=0$代入到$y=frac{1}{x+1}$中可得:$y=1$,即$m=1$,图象如下所示:故答案为:1,图象如上所示;(2)A当$x<-1$或$x>-1$时,函数值$y$随$x$的增大而减小,故选项$A$不正确;B.根据图象可得,函数图象不经过第四象限,故选项$B$正确;C.根据函数表示可得:$xneq-1$,所以函数图象与直线$x=-1$没有交点,故选项$C$正确;D.根据图象可知,函数图象对称中心$(-1,0)$,故选项$D$正确;故选:A;(3)$becausex_{1}+x_{2}=-2$,$thereforey_{1}+y_{2}=frac{1}{x_{1}+1}+frac{1}{x_{2}+1}$$=frac{x_{2}+1+x_{1}+1}{left(x_{1}+1right)left(x_{2}+1right)}=frac{x_{1}+x_{2}+2}{left(x_{1}+1right)left(x_{2}+1right)}=0$;故答案为:0.【点睛】本题主要考查的是反比例函数的图象与性质,解题关键是熟练掌握反比例函数的基本性质。", "solution": "null", "level": "八年级", "question": "小欣研究了函数${y}=frac{1}{{x}+1}$的图象与性质.其研究过程如下:(1)绘制函数图象(1)列表:下表是$x$与$y$的几组对应值,其中$m=underline{1}$;|$x$|$cdots$|-4|-3|-2|$-frac{3}{2}$|$-frac{4}{3}$|$-frac{2}{3}$|$-frac{1}{2}$|0|1|2|$cdots$||:---:|:---:|:---:|:---:|:---:|:---:|:---:|:---:|:---:|:---:|:---:|:---:|:---:||$y$|$cdots$|$-frac{1}{3}$|$-frac{1}{2}$|-1|-2|-3|3|2|$m$|$frac{1}{2}$|$frac{1}{3}$|$cdots$|(2)描点:根据表中的数值描点$(x,y)$;(3)连线:用平滑的曲线顺次连接各点,请把图象补充完整.(2)探究函数性质:下列说法不正确的是$A$A函数值$y$随$x$的增大而减小B.函数图象不经过第四象限C.函数图象与直线$x=-1$没有交点D.函数图象对称中心$(-1,0)$(3)如果点$Aleft(x_{1},y_{1}right)、Bleft(x_{2},y_{2}right)$在函数图象上,如果$x_{1}+x_{2}=-2$,则$y_{1}+y_{2}=0$.", "options": [], "subject": "解析几何", "analysis": "(1)把$x=0$代入函数解析式即可得$m$的值,根据图中的点画出图象即可;(2)根据图象和解析式即可进行判断;(3)先写出$y_{1}+y_{2}=frac{{x}_{1}+{x}_{2}+2}{left({x}_{1}+1right)left({x}_{2}+1right)}$,然后根据$x_{1}+x_{2}=-2$,即可得出结果.【解析】解:(1)把$x=0$代入到$y=frac{1}{x+1}$中可得:$y=1$,即$m=1$,图象如下所示:故答案为:1,图象如上所示;(2)A当$x<-1$或$x>-1$时,函数值$y$随$x$的增大而减小,故选项$A$不正确;B.根据图象可得,函数图象不经过第四象限,故选项$B$正确;C.根据函数表示可得:$xneq-1$,所以函数图象与直线$x=-1$没有交点,故选项$C$正确;D.根据图象可知,函数图象对称中心$(-1,0)$,故选项$D$正确;故选:A;(3)$becausex_{1}+x_{2}=-2$,$thereforey_{1}+y_{2}=frac{1}{x_{1}+1}+frac{1}{x_{2}+1}$$=frac{x_{2}+1+x_{1}+1}{left(x_{1}+1right)left(x_{2}+1right)}=frac{x_{1}+x_{2}+2}{left(x_{1}+1right)left(x_{2}+1right)}=0$;故答案为:0.【点睛】本题主要考查的是反比例函数的图象与性质,解题关键是熟练掌握反比例函数的基本性质。"} {"id": "5787", "image": ["7190.jpg"], "answer": "解: $\\because$ 四边形 $\\mathrm{ABCD}$ 是平行四边形, $\\therefore \\mathrm{AB}=\\mathrm{CD}, \\mathrm{AB} / / \\mathrm{CD}$,\n\n$\\therefore \\angle \\mathrm{ABD}=\\angle \\mathrm{CDB} . \\therefore 180^{\\circ}-\\angle \\mathrm{ABD}=180^{\\circ}-\\angle \\mathrm{CDB}$, 即 $\\angle \\mathrm{ABE}=\\angle \\mathrm{CDF}$.\n\n```\n$A B=C D$\n $\\{\\angle A B E=\\angle C D F$\n在 $\\triangle \\mathrm{ABE}$ 和 $\\triangle \\mathrm{CDF}$ 中, $\\because \\quad B E=D F$\n$\\therefore \\triangle \\mathrm{ABE} \\cong \\triangle \\mathrm{CDF}(\\mathrm{SAS})$.\n$\\therefore \\mathrm{AE}=\\mathrm{CF}$.\n```", "solution": "null", "level": "八年级", "question": "(5 分 ) 如图, 已知四边形 $A B C D$ 是平行四边形, 点 $E 、 B 、 D 、 F$ 在同一直线上, 且 $B E=D F$. 求证: $\\mathrm{AE}=\\mathrm{CF}$.\n\n", "options": [], "subject": "立体几何学", "analysis": "解: $\\because$ 四边形 $\\mathrm{ABCD}$ 是平行四边形, $\\therefore \\mathrm{AB}=\\mathrm{CD}, \\mathrm{AB} / / \\mathrm{CD}$,\n\n$\\therefore \\angle \\mathrm{ABD}=\\angle \\mathrm{CDB} . \\therefore 180^{\\circ}-\\angle \\mathrm{ABD}=180^{\\circ}-\\angle \\mathrm{CDB}$, 即 $\\angle \\mathrm{ABE}=\\angle \\mathrm{CDF}$.\n\n```\n$A B=C D$\n $\\{\\angle A B E=\\angle C D F$\n在 $\\triangle \\mathrm{ABE}$ 和 $\\triangle \\mathrm{CDF}$ 中, $\\because \\quad B E=D F$\n$\\therefore \\triangle \\mathrm{ABE} \\cong \\triangle \\mathrm{CDF}(\\mathrm{SAS})$.\n$\\therefore \\mathrm{AE}=\\mathrm{CF}$.\n```"} {"id": "5789", "image": ["7191.jpg"], "answer": "证明: $\"$ 四边形 $A B C D$ 是平行四边形,\n\n$\\therefore A B=D C, A B \\| D C$.\n\n$\\therefore \\angle F A E=\\angle D, \\angle F=\\angle E C D$.\n\n又 $\\because E A=E D$,\n\n$\\therefore \\triangle A F E \\cong \\triangle D C E$\n\n$\\therefore A F=D C$\n\n$\\therefore A F=A B$", "solution": "null", "level": "八年级", "question": "(5 分) 如图, 在 ${ }^{-} A B C D$ 中, 点 $\\mathrm{E}$ 是 $A D$ 的中点, 连接 $C E$ 并延长, 交 $B A$ 的延长线于点 $\\mathrm{F}$.\n\n\n\n求证: $F A=A B$.", "options": [], "subject": "立体几何学", "analysis": "证明: $\"$ 四边形 $A B C D$ 是平行四边形,\n\n$\\therefore A B=D C, A B \\| D C$.\n\n$\\therefore \\angle F A E=\\angle D, \\angle F=\\angle E C D$.\n\n又 $\\because E A=E D$,\n\n$\\therefore \\triangle A F E \\cong \\triangle D C E$\n\n$\\therefore A F=D C$\n\n$\\therefore A F=A B$"} {"id": "5822", "image": ["7222.jpg", "7223.jpg", "7223.jpg"], "answer": "解:如图所示, $\\triangle A^{\\prime} B^{\\prime} C^{\\prime}$ 即为所求作的 $\\triangle A B C$ 关于点 $O$ 的对称三角形.\n\n\n\n解: 分别连接 $A O, B O$ 和 $C O$, 延长 $A O, B O$ 和 $C O$ 使得 $A O=O A^{\\prime} 、 B O=O B^{\\prime} 、 C O=O C^{\\prime}$, 将点连线,则 $\\triangle A^{\\prime} B^{\\prime} C^{\\prime}$ 即为所要做的点。", "solution": "null", "level": "八年级", "question": "(5 分 ) 如图, 作出 $\\triangle \\mathrm{ABC}$ 关于点 $\\mathrm{O}$ 成中心对称的三角形. (保留作图痕迹)\n\n", "options": [], "subject": "画法几何学", "analysis": "解:如图所示, $\\triangle A^{\\prime} B^{\\prime} C^{\\prime}$ 即为所求作的 $\\triangle A B C$ 关于点 $O$ 的对称三角形.\n\n\n\n解: 分别连接 $A O, B O$ 和 $C O$, 延长 $A O, B O$ 和 $C O$ 使得 $A O=O A^{\\prime} 、 B O=O B^{\\prime} 、 C O=O C^{\\prime}$, 将点连线,则 $\\triangle A^{\\prime} B^{\\prime} C^{\\prime}$ 即为所要做的点。"} {"id": "5134", "image": ["6651.jpg"], "answer": "选择(1),由平行四边形的性质得$OA=OC,OB=OD$,而$AE=CF$,则$OE=OF$,即可证明$triangleBOEcongtriangleDOF$,得$BE=DF$;选择(2),由平行四边形的性质得$OB=OD$,由$BE//DF$,得$angleOBE=angleODF$,即可证明$triangleBOEcongtriangle$$DOF$,得$BE=DF$.【解析】解:选择(1)证明:$because$四边形$ABCD$是平行四边形,对角线$AC、BD$交于点$O$,$thereforeOA=OC,OB=OD$,$becauseAE=CF$,$thereforeOA-AE=OC-CF$,$thereforeOE=OF$,在$triangleBOE$和$triangleDOF$中,$left{begin{array}{l}{OB}={OD}angle{BOE}=angle{DOF},{OE}={OF}end{array}right.$$thereforetriangleBOEcongtriangleDOF(SAS)$,$thereforeBE=DF$.答案不唯一,选择(2),证明:$because$四边形$ABCD$是平行四边形,对角线$AC、BD$交于点$O$,$thereforeOB=OD$,$becauseBE//DF$,$thereforeangleOBE=angleODF$,在$triangleBOE$和$triangleDOF$中,$left{begin{array}{l}angle{BOE}=angle{DOF}{OB}={OD}angle{OBE}=angle{ODF}end{array}right.$,$thereforetriangleBOEcongtriangleDOF(ASA)$,$thereforeBE=DF$,故答案为:(1)或(2).【点睛】此题重点考查平行四边形的性质、全等三角形的判定与性质等知识,证明$triangleBOEcongtriangleDOF$是解题的关键.", "solution": "null", "level": "八年级", "question": "在(1)$AE=CF;$(2)$BE//DF$;(3)$BE=DF$这三个条件中任选一个合适的补充条件在下面横线上,并完成证明过程.已知,如图,四边形$ABCD$是平行四边形,对角线$AC,BD$交于点$O$,点$E、F$在$AC$上,(1)或(2)(填写序号)求证:$BE=DF$.", "options": [], "subject": "画法几何学", "analysis": "选择(1),由平行四边形的性质得$OA=OC,OB=OD$,而$AE=CF$,则$OE=OF$,即可证明$triangleBOEcongtriangleDOF$,得$BE=DF$;选择(2),由平行四边形的性质得$OB=OD$,由$BE//DF$,得$angleOBE=angleODF$,即可证明$triangleBOEcongtriangle$$DOF$,得$BE=DF$.【解析】解:选择(1)证明:$because$四边形$ABCD$是平行四边形,对角线$AC、BD$交于点$O$,$thereforeOA=OC,OB=OD$,$becauseAE=CF$,$thereforeOA-AE=OC-CF$,$thereforeOE=OF$,在$triangleBOE$和$triangleDOF$中,$left{begin{array}{l}{OB}={OD}angle{BOE}=angle{DOF},{OE}={OF}end{array}right.$$thereforetriangleBOEcongtriangleDOF(SAS)$,$thereforeBE=DF$.答案不唯一,选择(2),证明:$because$四边形$ABCD$是平行四边形,对角线$AC、BD$交于点$O$,$thereforeOB=OD$,$becauseBE//DF$,$thereforeangleOBE=angleODF$,在$triangleBOE$和$triangleDOF$中,$left{begin{array}{l}angle{BOE}=angle{DOF}{OB}={OD}angle{OBE}=angle{ODF}end{array}right.$,$thereforetriangleBOEcongtriangleDOF(ASA)$,$thereforeBE=DF$,故答案为:(1)或(2).【点睛】此题重点考查平行四边形的性质、全等三角形的判定与性质等知识,证明$triangleBOEcongtriangleDOF$是解题的关键."} {"id": "5136", "image": ["6652.jpg", "6653.jpg", "6654.jpg", "6655.jpg", "6656.jpg", "6657.jpg", "6655.jpg", "6656.jpg", "6657.jpg"], "answer": "(1)平行四边形是中心对称图形但不是轴对称图形;(2)等腰梯形是轴对称图形但不是中心对称图形;(3)正方形既是轴对称图形又是中心对称图形.【解析】解:(1)甲图:平行四边形,(2)乙图:等腰梯形,(3)丙图:正方形.(甲图)(乙图)(丙图)【点睛】本题考查了轴对称图形和中心对称图形,熟练掌握几个常见的四边形是哪类图形是关键:(1)平行四边形是中心对称图形但不是轴对称图形;(2)等腰梯形是轴对称图形但不是中心对称图形;(3)矩形、菱形、正方形既是轴对称图形又是中心对称图形.", "solution": "null", "level": "八年级", "question": "如图,方格纸中有三个点$A,B,C$,要求作一个四边形使这三个点在这个四边形的边(包括顶点)上,且四边形的顶点在方格的顶点上.(甲图)(乙图)(丙图)(1)在甲图中作出的四边形是中心对称图形但不是轴对称图形;(2)在乙图中作出的四边形是轴对称图形但不是中心对称图形;(3)在丙图中作出的四边形既是轴对称图形又是中心对称图形.", "options": [], "subject": "画法几何学", "analysis": "(1)平行四边形是中心对称图形但不是轴对称图形;(2)等腰梯形是轴对称图形但不是中心对称图形;(3)正方形既是轴对称图形又是中心对称图形.【解析】解:(1)甲图:平行四边形,(2)乙图:等腰梯形,(3)丙图:正方形.(甲图)(乙图)(丙图)【点睛】本题考查了轴对称图形和中心对称图形,熟练掌握几个常见的四边形是哪类图形是关键:(1)平行四边形是中心对称图形但不是轴对称图形;(2)等腰梯形是轴对称图形但不是中心对称图形;(3)矩形、菱形、正方形既是轴对称图形又是中心对称图形."} {"id": "5139", "image": ["6661.jpg"], "answer": "(1)由路程$=$速度$times$时间,可求解;(2)分四种情况讨论,由平行四边形的性质,列出等式可求解.【解析】解:(1)$because$点$P$在$AD$上以$1{~cm}/{s}$的速度从点$A$向点$D$运动,$thereforeAP=$tcm,$thereforePD=(15-t){cm}$,当点$P$运动2秒时,$CQ=2times4=8{~cm}$,$thereforeBQ=15-8=7{~cm}$,当点$P$运动5秒时,$CQ=4times5=20{~cm}$,$thereforeBQ=20-15=5{~cm}$,故答案为:$(15-t);7;5$;(2)$becauseP$在$AD$上运动,$thereforetleqslant15div1=15$,即$00)$.(1)当点$P$运动$t$秒时,线段$PD$的长度为$(15-t)quadcm$;当点$P$运动2秒时,线段$BQ$的长度为$7{~cm}$;当点$P$运动5秒时,线段$BQ$的长度为$5{~cm}$;(2)若经过$t$秒,以$P、D、Q、B$四点为顶点的四边形是平行四边形.请求出所有$t$的值.", "options": [], "subject": "画法几何学", "analysis": "(1)由路程$=$速度$times$时间,可求解;(2)分四种情况讨论,由平行四边形的性质,列出等式可求解.【解析】解:(1)$because$点$P$在$AD$上以$1{~cm}/{s}$的速度从点$A$向点$D$运动,$thereforeAP=$tcm,$thereforePD=(15-t){cm}$,当点$P$运动2秒时,$CQ=2times4=8{~cm}$,$thereforeBQ=15-8=7{~cm}$,当点$P$运动5秒时,$CQ=4times5=20{~cm}$,$thereforeBQ=20-15=5{~cm}$,故答案为:$(15-t);7;5$;(2)$becauseP$在$AD$上运动,$thereforetleqslant15div1=15$,即$0", "options": [], "subject": "画法几何学", "analysis": "(1)由平行四边形的性质得$OA=OC,OB=OD$,再证$OE=OF$,即可得出结论;(2)由勾股定理得$AC=4$,则$OA=frac{1}{2}AC=2$,再由勾股定理求出$OB=sqrt{13}$,然后由直角三角形斜边上的中线性质即可求解.【解析】(1)证明:$because$四边形$ABCD$是平行四边形,$thereforeOA=OC,OB=OD$,$becauseE,F$分别是$OB,OD$的中点,$thereforeOE=OF$,$therefore$四边形$AECF$是平行四边形;(2)解:$becauseABperpAC$,$thereforeangleBAC=90^{circ}$,$thereforeAC=sqrt{BC^{2}-AB^{2}}=sqrt{5^{2}-3^{2}}=4$,$thereforeOA=frac{1}{2}AC=2$,在Rt$triangleAOB$中,由勾股定理得:$OB=sqrt{{AB}^{2}+0{~A}^{2}}=sqrt{3^{2}+2^{2}}=sqrt{13}$,$becauseangleBAO=90^{circ},E$是$OB$的中点,$thereforeAE=frac{1}{2}OB=frac{sqrt{13}}{2}$.【点睛】本题考查了平行四边形的判定与性质、直角三角形斜边上的中线性质、勾股定理等知识;熟练掌握平行四边形的判定与性质,由勾股定理求出$OA、OB$的长是解题的关键."} {"id": "5156", "image": ["6685.jpg", "6686.jpg", "6687.jpg", "6688.jpg", "6689.jpg", "6690.jpg", "6691.jpg", "6692.jpg", "6693.jpg", "6688.jpg", "6689.jpg", "6690.jpg", "6691.jpg", "6692.jpg", "6693.jpg"], "answer": "(1)根据轴对称图形的性质,先找出对称轴,再思考如何画图;(2)如一,也是先找一个中心,再根据中心对称的性质,思考如何画图;(3)根据中心对称和轴对称的性质画一个图形.注意此题有多种画法,答案不唯一.【解析】解:如图所示.(1)如图(1),图(2),图(3)所示;(2)如图(4)所示;(3)如图(5),图(6)所示.(1)(4)(2)(5)(3)(6)【点睛】本题综合考查了中心对称图形及轴对称图形的性质,及其作图的方法,学生做这些题时找对称轴及对称点是关键.", "solution": "null", "level": "八年级", "question": "如图,由4个全等的正方形组成的$L$形图案,请按下列要求画图:(1)在图案(1)中添加1个正方形,使它成轴对称图形(不能是中心对称图形);(2)在图案(2)中添加1个正方形,使它成中心对称图形(不能是轴对称图形);(3)在图案(3)中改变1个正方形的位置,从而得到一个新图形,使它既成中心对称图形,又成轴对称图形.(1)(2)(3)", "options": [], "subject": "画法几何学", "analysis": "(1)根据轴对称图形的性质,先找出对称轴,再思考如何画图;(2)如一,也是先找一个中心,再根据中心对称的性质,思考如何画图;(3)根据中心对称和轴对称的性质画一个图形.注意此题有多种画法,答案不唯一.【解析】解:如图所示.(1)如图(1),图(2),图(3)所示;(2)如图(4)所示;(3)如图(5),图(6)所示.(1)(4)(2)(5)(3)(6)【点睛】本题综合考查了中心对称图形及轴对称图形的性质,及其作图的方法,学生做这些题时找对称轴及对称点是关键."} {"id": "5274", "image": ["6846.jpg", "6847.jpg", "6848.jpg", "6849.jpg", "6850.jpg", "6851.jpg", "6852.jpg", "6853.jpg", "6849.jpg", "6850.jpg", "6851.jpg", "6852.jpg", "6853.jpg"], "answer": "解: (1)由折叠可知, $B C=D C^{\\prime}=4 \\mathrm{~cm}, N C=N C^{\\prime}$,\n\n设 $D N=x c m$, 则 $N C=N C^{\\prime}=(10-x) c m$,\n\n在 $R t \\triangle D N C^{\\prime}$ 中, $D N^{2}=D C^{\\prime 2}+N C^{\\prime 2}$,\n\n即 $x^{2}=4^{2}+(10-x)^{2}$,\n\n解得: $x=\\frac{29}{5}$, 即 $D N=\\frac{29}{5} \\mathrm{~cm}$;\n\n(2)当 $\\triangle A M N$ 是等腰三角形时, 易知不存在 $N A=N M$,\n\n如图, 当 $M A=M N$ 时,\n\n\n\n设 $B M=a c m$ ,则 $(10-a)^{2}=4^{2}+(a-2)^{2}$ ,\n\n解得: $a=5$,\n\n则四边形 $B C N M$ 的面积 $S=\\frac{(2+5) \\times 4}{2}=14\\left(\\mathrm{~cm}^{2}\\right)$;\n\n当 $A M=A N$ 时, 如图,\n\n\n\n设 $B M=b c m$, 则 $A M=A N=(10-b) c m$ ,\n\n则 $(10-b)^{2}=4^{2}+(10-2)^{2}$, 解得 $b=10-4 \\sqrt{5}$,\n\n则四边形 $B C N M$ 的面积 $S=(24-8 \\sqrt{5}) \\mathrm{cm}^{2}$.\n综上所述, 四边形 $B C N M$ 的面积为 $(24-8 \\sqrt{5}) \\mathrm{cm}^{2}$ 或 $14 \\mathrm{~cm}^{2}$;\n\n(3)解:如图, 当点 $M$ 与 $A$ 重合时, 易知 $A E=E N$, 设 $A E=E N=x \\mathrm{~cm}$,\n\n在Rt $\\triangle A D E$ 中, 则有 $x^{2}=4^{2}+(10-x-3)^{2}$, 解得 $x=\\frac{65}{14}$,\n\n$\\therefore D E=10-3-\\frac{65}{14}=\\frac{33}{14}(\\mathrm{~cm})$\n\n\n\n如图, 当点 $M$ 运动到 $M B^{\\prime} \\perp A B$ 时, $D E^{\\prime}$ 的值最大, $D E^{\\prime}=10-3-4=3(\\mathrm{~cm})$,\n\n\n\n如图 3 中, 当点 $M$ 运动到点 $B^{\\prime}$ 落在 $C D$ 时,\n\n$N B^{\\prime}=\\sqrt{C N^{2}+C B^{2}}=\\sqrt{4^{2}+3^{2}}=5(\\mathrm{~cm})$,\n\n$D B^{\\prime}\\left(\\right.$ 即 $\\left.D E^{\\prime \\prime}\\right)=10-3-5=2(\\mathrm{~cm})$;\n\n\n\n$\\therefore$ 点 $E$ 的运动轨迹 $E \\rightarrow E^{\\prime} \\rightarrow E^{\\prime \\prime}$, 运动路径 $=E E^{\\prime}+E^{\\prime} B^{\\prime}=3-\\frac{33}{14}+3-2=\\frac{23}{14}(\\mathrm{~cm})$.\n\n故答案为: $\\frac{23}{14}$.", "solution": "null", "level": "八年级", "question": "如图, 有一张长方形纸条 $A B C D, A B=10 \\mathrm{~cm}, B C=4 \\mathrm{~cm}$, 点 $M, N$ 分别在边 $A B, C D$ 上, 现将四边形 $B C N M$ 沿 $M N$ 折叠, 使点 $B, C$ 分别落在点 $B^{\\prime}, C^{\\prime}$ 上, 边 $M B^{\\prime}$ 与边 $C D$ 交于点 $E$.\n\n\n\n图 1\n\n\n\n图 2\n\n\n\n3\n\n(1)如图(1), 若折叠后, 点 $B$ 与点 $D$ 重合, 求此时 $D N$ 的长度;\n\n(2)如图(2), 若 $C N=2 c m$, 当 $\\triangle A M N$ 是等腰三角形时, 求此时四边形 $B C N M$ 的面积;\n\n(3)如图(3), 若 $C N=3 \\mathrm{~cm}$, 在点 $M$ 从点 $A$ 向点 $B$ 运动的过程中, 则点 $E$ 相应运动的路径长为 cm.", "options": [], "subject": "变换几何", "analysis": "解: (1)由折叠可知, $B C=D C^{\\prime}=4 \\mathrm{~cm}, N C=N C^{\\prime}$,\n\n设 $D N=x c m$, 则 $N C=N C^{\\prime}=(10-x) c m$,\n\n在 $R t \\triangle D N C^{\\prime}$ 中, $D N^{2}=D C^{\\prime 2}+N C^{\\prime 2}$,\n\n即 $x^{2}=4^{2}+(10-x)^{2}$,\n\n解得: $x=\\frac{29}{5}$, 即 $D N=\\frac{29}{5} \\mathrm{~cm}$;\n\n(2)当 $\\triangle A M N$ 是等腰三角形时, 易知不存在 $N A=N M$,\n\n如图, 当 $M A=M N$ 时,\n\n\n\n设 $B M=a c m$ ,则 $(10-a)^{2}=4^{2}+(a-2)^{2}$ ,\n\n解得: $a=5$,\n\n则四边形 $B C N M$ 的面积 $S=\\frac{(2+5) \\times 4}{2}=14\\left(\\mathrm{~cm}^{2}\\right)$;\n\n当 $A M=A N$ 时, 如图,\n\n\n\n设 $B M=b c m$, 则 $A M=A N=(10-b) c m$ ,\n\n则 $(10-b)^{2}=4^{2}+(10-2)^{2}$, 解得 $b=10-4 \\sqrt{5}$,\n\n则四边形 $B C N M$ 的面积 $S=(24-8 \\sqrt{5}) \\mathrm{cm}^{2}$.\n综上所述, 四边形 $B C N M$ 的面积为 $(24-8 \\sqrt{5}) \\mathrm{cm}^{2}$ 或 $14 \\mathrm{~cm}^{2}$;\n\n(3)解:如图, 当点 $M$ 与 $A$ 重合时, 易知 $A E=E N$, 设 $A E=E N=x \\mathrm{~cm}$,\n\n在Rt $\\triangle A D E$ 中, 则有 $x^{2}=4^{2}+(10-x-3)^{2}$, 解得 $x=\\frac{65}{14}$,\n\n$\\therefore D E=10-3-\\frac{65}{14}=\\frac{33}{14}(\\mathrm{~cm})$\n\n\n\n如图, 当点 $M$ 运动到 $M B^{\\prime} \\perp A B$ 时, $D E^{\\prime}$ 的值最大, $D E^{\\prime}=10-3-4=3(\\mathrm{~cm})$,\n\n\n\n如图 3 中, 当点 $M$ 运动到点 $B^{\\prime}$ 落在 $C D$ 时,\n\n$N B^{\\prime}=\\sqrt{C N^{2}+C B^{2}}=\\sqrt{4^{2}+3^{2}}=5(\\mathrm{~cm})$,\n\n$D B^{\\prime}\\left(\\right.$ 即 $\\left.D E^{\\prime \\prime}\\right)=10-3-5=2(\\mathrm{~cm})$;\n\n\n\n$\\therefore$ 点 $E$ 的运动轨迹 $E \\rightarrow E^{\\prime} \\rightarrow E^{\\prime \\prime}$, 运动路径 $=E E^{\\prime}+E^{\\prime} B^{\\prime}=3-\\frac{33}{14}+3-2=\\frac{23}{14}(\\mathrm{~cm})$.\n\n故答案为: $\\frac{23}{14}$."} {"id": "4379", "image": ["6071.jpg"], "answer": "解: (1) 由平移可知 $\\triangle A B C \\cong \\triangle D E F$,\n$\\therefore \\angle A C B=\\angle F=45^{\\circ}$,\n\n$\\therefore \\angle A=180^{\\circ}-\\angle B-\\angle A C B=105^{\\circ}$.\n\n(2)由平移可知 $\\triangle A B C \\cong \\triangle D E F$,\n\n$\\therefore B C=E F$,\n\n$\\therefore B C-E C=E F-E C$\n\n$\\therefore B E=C F=(B F-E C) \\div 2=3$,\n\n$\\therefore$ 平移的距离 $B E$ 为 3 .", "solution": "null", "level": "八年级", "question": "如图, $\\triangle A B C$ 沿 $B C$ 方向平移到 $\\triangle D E F$ 的位置.\n\n\n\n(1) 若 $\\angle B=30^{\\circ}, \\angle F=45^{\\circ}$, 求 $\\angle A$ 的度数;\n\n(2) 若 $B F=10, E C=4$, 求平移的距离.", "options": [], "subject": "变换几何", "analysis": "解: (1) 由平移可知 $\\triangle A B C \\cong \\triangle D E F$,\n$\\therefore \\angle A C B=\\angle F=45^{\\circ}$,\n\n$\\therefore \\angle A=180^{\\circ}-\\angle B-\\angle A C B=105^{\\circ}$.\n\n(2)由平移可知 $\\triangle A B C \\cong \\triangle D E F$,\n\n$\\therefore B C=E F$,\n\n$\\therefore B C-E C=E F-E C$\n\n$\\therefore B E=C F=(B F-E C) \\div 2=3$,\n\n$\\therefore$ 平移的距离 $B E$ 为 3 ."} {"id": "5322", "image": ["6873.jpg"], "answer": "解:\n\n(1) “ 1 ” 表示开, “0” 表示关,则所有不同的“开” “关” 的状态可表示为: 1111(全开), 1110, 1101,1011,0111(三开一关),1010,0101,0110(两开两关),共有 8 种;\n\n(2) 由 $a_{1}=2, a 2=3, a_{3}=5, a_{4}=8$, 归纳出 $a_{k+2}=a_{k}+a_{k+1}$;\n\n(3)由(2)得的关系式可得出: $a_{5}=a_{4}+a_{3}=8+5=13, a_{6}=a_{4}+a_{5}=8+13=21, a_{7}=a_{5}+a_{6}=$ $13+21=34, a_{8}=a_{6}+a_{7}=21+34=55, a_{9}=a_{7}+a_{8}=34+55=89, a_{10}=a_{8}+a_{9}=55+$ $89=144$.", "solution": "null", "level": "八年级", "question": "你觉得手机很神奇吗? 它能在瞬间清晰地传递声音、文字、图像等信号, 据说以后还能发送味道、触觉信息呢! 这里都有手机中电脑芯片的功劳. 其实, 这些信号在电脑芯片中都是以二进制数的形式给出的. 每个二进制数都由 0 和 1 构成, 电脑芯片上电子元件的 “开”、“关” 分别代表 “ 1 ”和 “0” 一一组电子元件的 “开” “关” 状态就表示相应的二进制数, 例如 “开” “开” “关” 表示 “110”, 如图, 电脑芯片的某段电路上分布着一组电子元件(假设它们首尾不相连), 且相邻的两个元件不能同时是关的. (以下各小题要求写出解答过程)\n\n\n\n(1)若此电路上有 4 个元件,则这 4 个元件所有不同的 “开” “关” 状态共有多少种? (请一一列出)\n\n(2)若用 $a_{k}$ 表示电路上 $k(k \\geq 1)$ 只电子元件所有不同的 “开” “关” 状态数, 试探索 $a_{k}, a_{k+1}$, $a_{k+2}$ 之间的关系式(不要求论证);\n\n(3)试用(2)中探索出的递推关系式, 计算 $a_{10}$ 的值.", "options": [], "subject": "组合数学", "analysis": "解:\n\n(1) “ 1 ” 表示开, “0” 表示关,则所有不同的“开” “关” 的状态可表示为: 1111(全开), 1110, 1101,1011,0111(三开一关),1010,0101,0110(两开两关),共有 8 种;\n\n(2) 由 $a_{1}=2, a 2=3, a_{3}=5, a_{4}=8$, 归纳出 $a_{k+2}=a_{k}+a_{k+1}$;\n\n(3)由(2)得的关系式可得出: $a_{5}=a_{4}+a_{3}=8+5=13, a_{6}=a_{4}+a_{5}=8+13=21, a_{7}=a_{5}+a_{6}=$ $13+21=34, a_{8}=a_{6}+a_{7}=21+34=55, a_{9}=a_{7}+a_{8}=34+55=89, a_{10}=a_{8}+a_{9}=55+$ $89=144$."} {"id": "14096", "image": [], "answer": "答案: 210", "solution": "null", "level": "四年级", "question": "直接写出得数。 (每题 1 分, 共 8 分)\n\n\\begin{tabular}{llll}\n$", "options": [], "subject": "算术", "analysis": "答案: 210"} {"id": "14111", "image": [], "answer": "$9.8 ; 12.5 ; 2 ; 18 ; 0.0206 ; 3.09 ; 81 ; 0.865$", "solution": "null", "level": "四年级", "question": "直接写出得数。(每小题 1 分, 共 8 分)\n(1) $", "options": [], "subject": "算术", "analysis": "答案: $"} {"id": "27260", "image": [], "answer": "解$\\times$\n\n【分析】12668 先算除法, 再算乘法; $126+6 \\times 8$ 先算乘法, 再算加法, 由此进行判断。\n\n【详解】 $126 \\div 6 \\times 8$ 和 $126+6 \\times 8$ 的运算顺序是不一样的。\n\n故答案为: $\\times$\n\n【点睛】四则混合运算的顺序: (1)一个算式里,如果只有加减法或者只有乘除法,按照从左到右的顺序依次计算; (2)如果既有加减法、又有乘除法, 先算乘除法、再算加减法; (3)如果有括号, 先算括号里面的。", "solution": "null", "level": "四年级", "question": "$126 \\div 6 \\times 8$ 和 $126+6 \\times 8$ 的运算顺序是一样的。()", "options": [], "subject": "算术", "analysis": "解$\\times$\n\n【分析】12668 先算除法, 再算乘法; $126+6 \\times 8$ 先算乘法, 再算加法, 由此进行判断。\n\n【详解】 $126 \\div 6 \\times 8$ 和 $126+6 \\times 8$ 的运算顺序是不一样的。\n\n故答案为: $\\times$\n\n【点睛】四则混合运算的顺序: (1)一个算式里,如果只有加减法或者只有乘除法,按照从左到右的顺序依次计算; (2)如果既有加减法、又有乘除法, 先算乘除法、再算加减法; (3)如果有括号, 先算括号里面的。"} {"id": "27262", "image": [], "answer": "解$\\sqrt{ }$\n\n【分析】根据题意可知, 此题是先算加法和乘法, 最后算除法, 根据混合运算的计算顺序进行判断即可。\n\n【详解】 $80 \\times 9 \\div(65+25)$\n\n$=720 \\div 90$\n\n$=8$\n\n即把算式 $65+25=90,80 \\times 9=720,720 \\div 90=8$ 写成一个综合算式是 $80 \\times 9 \\div(65+25)$ 。\n\n故答案为: $\\sqrt{ }$\n【点睛】熟练掌握混合运算的计算顺序是解答此题的关键。", "solution": "null", "level": "四年级", "question": "把算式 $65+25=90,80 \\times 9=720,720 \\div 90=8$ 写成一个综合算式是 $80 \\times 9 \\div(65+25)$ 。( )。", "options": [], "subject": "算术", "analysis": "解$\\sqrt{ }$\n\n【分析】根据题意可知, 此题是先算加法和乘法, 最后算除法, 根据混合运算的计算顺序进行判断即可。\n\n【详解】 $80 \\times 9 \\div(65+25)$\n\n$=720 \\div 90$\n\n$=8$\n\n即把算式 $65+25=90,80 \\times 9=720,720 \\div 90=8$ 写成一个综合算式是 $80 \\times 9 \\div(65+25)$ 。\n\n故答案为: $\\sqrt{ }$\n【点睛】熟练掌握混合运算的计算顺序是解答此题的关键。"} {"id": "27263", "image": ["13304.jpg"], "answer": "解$\\sqrt{ }$\n\n【分析】被除数=商×除数, 依此将数据代入公式计算出结果即可。\n\n【详解】 $20 \\times 40=800$, 即已知两个数的商是 20 , 除数是 40 , 那么被除数是 800 。\n\n故答案为: $\\sqrt{ }$\n\n【点睛】熟练掌握乘、除法的意义和各部分之间的关系是解答此题的关键。", "solution": "null", "level": "四年级", "question": "已知两个数的商是 20 , 除数是 40 , 那么被除数是 800 。()\n\n", "options": [], "subject": "算术", "analysis": "解$\\sqrt{ }$\n\n【分析】被除数=商×除数, 依此将数据代入公式计算出结果即可。\n\n【详解】 $20 \\times 40=800$, 即已知两个数的商是 20 , 除数是 40 , 那么被除数是 800 。\n\n故答案为: $\\sqrt{ }$\n\n【点睛】熟练掌握乘、除法的意义和各部分之间的关系是解答此题的关键。"} {"id": "27268", "image": [], "answer": "解买 1 张 240 元的游泳卡\n\n【分析】方案一:爸爸、妈妈、贝贝都买单次票, 需要的钱=爸爸和妈妈全年游泳的总次数 $\\times 20+$ 贝贝全年游泳的次数 $\\times 15$ 。\n\n方案二: $4+6+4=14$ (次), 14 次 $<15$ 次, 即全家可买一张 240 元的游泳卡。依此计算并比较即可。\n\n【详解】 $(4+4) \\times 20+6 \\times 15$\n\n$=8 \\times 20+90$\n\n$=160+90$\n\n$=250$ (元)\n\n$4+6+4=14$ (次), 14 次 $<15$ 次, 即方案二需要 240 元。\n\n250 元 $>240$ 元, 即方案二合算。\n\n答: 全家买 1 张 240 元的游泳卡合算。\n\n【点睛】此题考查的是经济问题的计算, 应分别计算出每种购票方案需要的钱数再比较。", "solution": "null", "level": "四年级", "question": "游泳馆有两种购票方案:\n\n方案一:成人每人每次 20 元,儿童每人每次 15 元。\n\n方案二: 购买 240 元游泳卡可用 15 次(任何人都可以用,1 年内有效)。\n\n若 1 年内爸爸、妈妈各游泳 4 次, 贝贝游泳 6 次, 怎样购票合算 $?$", "options": [], "subject": "算术", "analysis": "解买 1 张 240 元的游泳卡\n\n【分析】方案一:爸爸、妈妈、贝贝都买单次票, 需要的钱=爸爸和妈妈全年游泳的总次数 $\\times 20+$ 贝贝全年游泳的次数 $\\times 15$ 。\n\n方案二: $4+6+4=14$ (次), 14 次 $<15$ 次, 即全家可买一张 240 元的游泳卡。依此计算并比较即可。\n\n【详解】 $(4+4) \\times 20+6 \\times 15$\n\n$=8 \\times 20+90$\n\n$=160+90$\n\n$=250$ (元)\n\n$4+6+4=14$ (次), 14 次 $<15$ 次, 即方案二需要 240 元。\n\n250 元 $>240$ 元, 即方案二合算。\n\n答: 全家买 1 张 240 元的游泳卡合算。\n\n【点睛】此题考查的是经济问题的计算, 应分别计算出每种购票方案需要的钱数再比较。"} {"id": "27270", "image": [], "answer": "解1440 套\n【分析】用房子的栋数乘每栋的层数,再乘每层住房的套数,即可求出这个小区一共有多少套住房。\n\n【详解】 $24 \\times 15 \\times 4$\n\n$=24 \\times(15 \\times 4)$\n\n$=24 \\times 60$\n\n$=1440$ (套)\n\n答:这个小区一共有 1440 套住房。\n\n【点睛】本题考查连乘的计算及应用。理解题意, 找出数量关系, 列式计算即可。", "solution": "null", "level": "四年级", "question": "某小区一共有 15 栋房子,每栋 24 层,每层有 4 套住房,这个小区一共有多少套住房?", "options": [], "subject": "算术", "analysis": "解1440 套\n【分析】用房子的栋数乘每栋的层数,再乘每层住房的套数,即可求出这个小区一共有多少套住房。\n\n【详解】 $24 \\times 15 \\times 4$\n\n$=24 \\times(15 \\times 4)$\n\n$=24 \\times 60$\n\n$=1440$ (套)\n\n答:这个小区一共有 1440 套住房。\n\n【点睛】本题考查连乘的计算及应用。理解题意, 找出数量关系, 列式计算即可。"} {"id": "27271", "image": [], "answer": "解240 元\n\n【分析】用 295 元乘 12 个月, 计算出分期付款部分的钱数, 再加上首付的 3500 元, 计算出这台电脑分期付款需要的钱数, 再减去 6800 元, 即可解题。\n\n【详解】 $(3500+295 \\times 12)-6800$\n\n$=(3500+3540)-6800$\n\n$=7040-6800$\n\n$=240$ (元)\n\n答:分期付款比一次性付款多花 240 元钱。\n\n【点睛】计算出分期付款部分的钱数,是解答此题的关键。", "solution": "null", "level": "四年级", "question": "王老师买一台 6800 元的笔记本电脑,如果采用分期付款的方式,需要首付 3500 元,以后每个月付 295 元,一共再付 12 个月。分期付款比一次性付款多花多少钱?", "options": [], "subject": "算术", "analysis": "解240 元\n\n【分析】用 295 元乘 12 个月, 计算出分期付款部分的钱数, 再加上首付的 3500 元, 计算出这台电脑分期付款需要的钱数, 再减去 6800 元, 即可解题。\n\n【详解】 $(3500+295 \\times 12)-6800$\n\n$=(3500+3540)-6800$\n\n$=7040-6800$\n\n$=240$ (元)\n\n答:分期付款比一次性付款多花 240 元钱。\n\n【点睛】计算出分期付款部分的钱数,是解答此题的关键。"} {"id": "13969", "image": ["2861.jpg", "2862.jpg"], "answer": "答案:186 $1164 \\quad 2100 \\quad 800 \\quad 15 \\quad 25$\n\n$134 \\quad 4$", "solution": "null", "level": "四年级", "question": "直接写出得数。(每小题 1 分, 共 8 分)\n\n| $87+99=$ | $38+64+62=$ | $21 \\times 4 \\times 25=$ | $25 \\times 32=$ |\n| :--- | :--- | :--- | :--- |\n| $540 \\div 36=$ | $123-23-75=$ | $152+34-52=$ | $74 \\div 37 \\times 2=$ |17.明明借助点子图计算 $15 \\times 12$ 的过程如下, 这道题还可以怎么算呢? 请\n\n\n\n你在右侧的点子\n\n图\n\n上画一画,并写出算过程。\n\n$15 \\times 12$\n\n$=15 \\times 4 \\times 3$\n\n$=60 \\times 3$\n\n$=180$19.下面是某地区近几年校舍改造经费的投入情况统计图。\n某地区近几年校舍改造经费的投入情况统计图\n\n\n\n三年共投入多少万元?(4 分)", "options": [], "subject": "算术", "analysis": "答案:186 $1164 \\quad 2100 \\quad 800 \\quad 15 \\quad 25$\n\n$134 \\quad 4$"} {"id": "13971", "image": ["2863.jpg"], "answer": "答案:(76+34) \\times 12=1320$ (元)\n\n答:这天该商店卖出的纪念品共收入 1320 元。\n\n$", "solution": "null", "level": "四年级", "question": "端午节吃粽子是中国的传统习俗。端午节前夕一家食品厂一共做了 8100 个粽子, 每 4 个装一袋, 每 25 袋装一箱。现已准备了 90 个箱子,够不够用?(5 分)23.龙龙和爸爸都喜欢集邮。(请用两种方法解答)(6 分)\n\n", "options": [], "subject": "算术", "analysis": "答案:(76+34) \\times 12=1320$ (元)\n\n答:这天该商店卖出的纪念品共收入 1320 元。\n\n$"} {"id": "13987", "image": [], "answer": "6 .05 \\quad 0.4 \\quad 3.36$", "solution": "null", "level": "四年级", "question": "在 $\\square$ 里填上合适的数, 在 里填上“+”或“一”。$(1)(13.8+7.12)+10.88=13.8+(\\square+\\square)$$(2) 40-6.23-3.77=40-(\\square \bigcirc \\square)$$(3) 2.3+5.4+8.7+3.6=(\\square+\\square)+(\\square+\\square)$", "options": [], "subject": "算术", "analysis": ""} {"id": "13988", "image": [], "answer": "(1)7.12 $\\quad 10.88$$(2) 6.23+3.77$(3)2.3 $\\quad 8.7 \\quad 5.4 \\quad 3.6$", "solution": "null", "level": "四年级", "question": "在 0 里填上“ $>\" “<$ 或“ $=$ ”。\n\n$", "options": [], "subject": "算术", "analysis": ""} {"id": "13989", "image": [], "answer": "答案:=<<>$", "solution": "null", "level": "四年级", "question": "甲、乙、丙三人共有 270 元钱,如果甲借给乙 15.6 元,又借给丙 25.5 元以后, 三人的钱一样多, 那么乙原来有 $(\\quad)$ 元, 甲原来有 $(\\quad)$ 元。", "options": [], "subject": "算术", "analysis": ""} {"id": "13990", "image": [], "answer": "0.34", "solution": "null", "level": "四年级", "question": "一个小数加上它的计数单位, 和是", "options": [], "subject": "算术", "analysis": ""} {"id": "13991", "image": [], "answer": "(1)3.8 $\\quad 4.1$(2)3.14 $\\quad 3.09$ $\begin{array}{llll}", "solution": "null", "level": "四年级", "question": "按规律填数。(1)2.9、3.2、3.5、( $\\quad) 、(\\quad) 。$(2)3.29、3.24、3.19、( ) 、( $\\quad$ )", "options": [], "subject": "算术", "analysis": ""} {"id": "13992", "image": [], "answer": "84 .2 & 2.48 & 86.68 & 81.72\\end{array}$", "solution": "null", "level": "四年级", "question": "由 $2 、 4 、 8$ 和小数点组成的最大的一位小数是 $\\quad$ ), 组成的最小的两位小数是 $(\\quad)$, 这两个数的和是 $(\\quad)$, 差是 $(\\quad)$ 。", "options": [], "subject": "算术", "analysis": ""} {"id": "14008", "image": ["2876.jpg", "2876.jpg"], "answer": "答案: $(6 \\times 10-48) \\div(6-4)=6$ (个)\n\n$10-6=4($ 个 $)$\n\n答:笑笑编的大中国结有 4 个,小中国结有 6 个。18.答案:(1)\n\n\n\n(2)4 画图如上图所示。\n\n(3)35 良好", "solution": "null", "level": "四年级", "question": "笑笑说: “我是小达人, 我会剪纸、编织。我用 48 米的红绳编了 10 个中国结, 编一个大中国结要用 6 米红绳, 编一个小中国结要用 4 米红绳”。笑笑编的大中国结和小中国结各有多少个?(8 分)", "options": [], "subject": "算术", "analysis": "答案: $(6 \\times 10-48) \\div(6-4)=6$ (个)\n\n$10-6=4($ 个 $)$\n\n答:笑笑编的大中国结有 4 个,小中国结有 6 个。18.答案:(1)\n\n\n\n(2)4 画图如上图所示。\n\n(3)35 良好"} {"id": "14009", "image": [], "answer": "答案:(240 \\times 5-950) \\div(5+20)=10$ (个)\n\n答:快递公司损坏了 10 个玻璃仪器。", "solution": "null", "level": "四年级", "question": "某快递公司为客户运送 240 个玻璃仪器, 双方商定每个玻璃仪器的运费是 5 元, 若损坏 1 个玻璃仪器不但得不到运费还要赔偿 20 元,运送完后快递公司得到运费 950 元, 快递公司损坏了多少个玻璃仪器?(10 分)", "options": [], "subject": "算术", "analysis": "答案:(240 \\times 5-950) \\div(5+20)=10$ (个)\n\n答:快递公司损坏了 10 个玻璃仪器。"} {"id": "14018", "image": [], "answer": "答案:方法一:\n\n$(80+94) \\times 15=2610$ (人)\n\n$2610>2380$\n\n方法二:\n\n$80 \\times 15+94 \\times 15=2610$ (八)\n\n$2610>2380$\n\n答: 它超载了。", "solution": "null", "level": "四年级", "question": "实验二小组建了 25 个小红帽志愿者小组, 每个志愿者小组有 4 名队员,每个周末志愿者去收集旧电池。这个周末一共收集了 1200 节旧\n电池。\n\n$25 \\times 4$\n\n平均每名队员收集多少节旧电池?\n\n$1200 \\div 25$\n\n一共有多少名队员?\n\n$1200 \\div 25 \\div 4$ 平均每个小组收集多少节旧电池?", "options": [], "subject": "算术", "analysis": "答案:方法一:\n\n$(80+94) \\times 15=2610$ (人)\n\n$2610>2380$\n\n方法二:\n\n$80 \\times 15+94 \\times 15=2610$ (八)\n\n$2610>2380$\n\n答: 它超载了。"} {"id": "14019", "image": [], "answer": "答案:(1)典典比华华多跑了多少秒?\n\n(2)(答案不唯一)同同跑了多少秒?\n\n$", "solution": "null", "level": "四年级", "question": "下表是一列火车的载客信息, 算一算, 如果这列火车限乘客 2380 人, 它是否超载了? (用两种方法解答)(8 分)\n\n| 双层车 | 每节车厢人数/人 | 车厢数/节 |\n| :---: | :---: | :---: |\n| 上层 | 80 | 15 |\n| 下层 | 94 | 15 |", "options": [], "subject": "算术", "analysis": "答案:(1)典典比华华多跑了多少秒?\n\n(2)(答案不唯一)同同跑了多少秒?\n\n$"} {"id": "14033", "image": [], "answer": "答案:56 $\\quad 0 \\quad 81 \\quad 300 \\quad 108 \\quad 14$", "solution": "null", "level": "四年级", "question": "列坚式计算, 带※的要验算。(每小题 3 分, 共 9 分)\n\n$※ 900-367=$\n\n$4005 \\div 89=$\n\n$※", "options": [], "subject": "算术", "analysis": "答案:56 $\\quad 0 \\quad 81 \\quad 300 \\quad 108 \\quad 14$1"} {"id": "14034", "image": ["2882.jpg", "2883.jpg", "2884.jpg", "2885.jpg", "2886.jpg", "2887.jpg", "2888.jpg", "2887.jpg", "2888.jpg"], "answer": "答案: $※ 900-367=533$\n\n900 验算: 533\n\n\n\n$4005 \\div 89=45$\n\n$8 9 \\longdiv { 4 0 0 5 }$\n\n$\\frac{356}{445}$\n\n$\\begin{array}{r}445 \\\\ \\hline 0\\end{array}$\n\n$※12.5-8.83=3.67$\n\ni 2.5 验算: 3.67\n\n\n\n19.答案: \n\n$28 \\times(72-18) \\div 63$\n\n$=28 \\times 54 \\div 63$\n\n$=1512 \\div 63$\n$=24$\n\n$$\n50-(", "solution": "null", "level": "四年级", "question": "计算下面各题, 能简算的要简算。(每小题 3 分, 共 18 分) $28 \\times(72-18) \\div 63 \\quad 50-(19.\n\n综合算式:\n\n\n\n综合算式:21.蓝蓝想在某网店买 1 个书包和 1 个行李箱。\n\n(1)促销期间购买可以节省多少钱?(4 分)\n\n\n(2)该网店满 188 元包邮, 她至少还要再买多少钱的商品才能包邮? (促销期间购买)(4 分)23.四年级全体师生共 156 人乘车要到永川野生动物园去游玩, 怎样租车最省钱?(6 分)\n\n\n\n大客车\n\n\n\n限乘客 30 人 600 元/辆 限乘客 48 人 800 元/辆", "options": [], "subject": "算术", "analysis": "答案: $※ 900-367=533$\n\n900 验算: 533\n\n\n\n$4005 \\div 89=45$\n\n$8 9 \\longdiv { 4 0 0 5 }$\n\n$\\frac{356}{445}$\n\n$\\begin{array}{r}445 \\\\ \\hline 0\\end{array}$\n\n$※12.5-8.83=3.67$\n\ni 2.5 验算: 3.67\n\n\n\n19.答案: \n\n$28 \\times(72-18) \\div 63$\n\n$=28 \\times 54 \\div 63$\n\n$=1512 \\div 63$\n$=24$\n\n$$\n50-("} {"id": "14056", "image": [], "answer": "答案:7.35", "solution": "null", "level": "四年级", "question": "直接写出得数。(每小题 1 分, 共 8 分)\n\n| $65 \\times 2=$ | $390 \\div 30=$ | $", "options": [], "subject": "算术", "analysis": "答案:7.35"} {"id": "14057", "image": [], "answer": "130 & 13 & 8.2 & 1.2 & 280 & 0.096 & 0.67 & 0.43\\end{array}$", "solution": "null", "level": "四年级", "question": "计算下面各题, 怎样简便就怎样算。(每小题 3 分, 共 18 分)\n\n$510 \\div 3 \\div 17$\n\n$640 \\div[8 \\times(", "options": [], "subject": "算术", "analysis": "130 & 13 & 8.2 & 1.2 & 280 & 0.096 & 0.67 & 0.43\\end{array}$"} {"id": "14068", "image": [], "answer": "0.005 $43.02 \\quad 0.01 \\quad 0.35$$\begin{array}{llll}1.05 & 3.07 & 150 & 64\\end{array}$", "solution": "null", "level": "四年级", "question": "直接写出得数。(每小题 1 分, 共 8 分)\n\n$$\n\\begin{array}{llll}", "options": [], "subject": "算术", "analysis": "答案:"} {"id": "14078", "image": ["2903.jpg", "2904.jpg", "2905.jpg", "2905.jpg"], "answer": "答案:200 $20 \\quad 127 \\quad 36 \\quad 900 \\quad 4$16.答案: \n\n$$\n\\begin{aligned}\n& 720 \\div(80-18 \\times 4) \\\\\n= & 720 \\div 8 \\\\\n= & 90 \\\\\n& (56+250) \\div[(42-36) \\times 3] \\\\\n= & 306 \\div 18 \\\\\n= & 17 \\\\\n& 686-(286+134) \\\\\n= & 686-286-134 \\\\\n= & 266 \\\\\n& 208 \\times 45-45 \\times 8\n\\end{aligned}\n$$\n\n$=45 \\times(208-8)$\n\n$=9000$\n\n$$\n900 \\div 12+900 \\div 18\n$$\n\n$=75+50$\n\n$=125$\n\n", "solution": "null", "level": "四年级", "question": "计算下面各题, 怎样简便就怎样算。(每小题 3 分, 共 15 分)\n\n$720 \\div(80-18 \\times 4) \\quad(56+250) \\div[(42-36) \\times 3]$\n\n$686-(286+134)$\n\n$208 \\times 45-45 \\times 8$\n\n$900 \\div 12+900 \\div 18$17.黄霏霏要去商店买三种学习用品。去哪家商店买比较便宜? 为什么? (7 分)\n\n| $\\mathrm{A}$ 商店 | | | $\\mathrm{B}$ 商店 | | |\n| :---: | :---: | :---: | :---: | :---: | :---: |\n| $\\infty-\\infty$ | (4) | | $\\infty$ | (iv) | |\n|", "options": [], "subject": "算术", "analysis": "答案:200 $20 \\quad 127 \\quad 36 \\quad 900 \\quad 4$16.答案: \n\n$$\n\\begin{aligned}\n& 720 \\div(80-18 \\times 4) \\\\\n= & 720 \\div 8 \\\\\n= & 90 \\\\\n& (56+250) \\div[(42-36) \\times 3] \\\\\n= & 306 \\div 18 \\\\\n= & 17 \\\\\n& 686-(286+134) \\\\\n= & 686-286-134 \\\\\n= & 266 \\\\\n& 208 \\times 45-45 \\times 8\n\\end{aligned}\n$$\n\n$=45 \\times(208-8)$\n\n$=9000$\n\n$$\n900 \\div 12+900 \\div 18\n$$\n\n$=75+50$\n\n$=125$\n\n"} {"id": "14080", "image": [], "answer": "6 .26 \\div 100=0.0626$ (升)$0.0626 \times 1000=62.6$ (升)答: 照此计算, 平均行驶每千米耗油 0.0626 升, 行驶 1000 千米耗油 62.6 升。", "solution": "null", "level": "四年级", "question": "某学校校庆正逢中秋节, 学校为到场来宾制作了纪念版月饼。小马负责为 A 区来宾发 550 块月饼。\n\n(1)他已经发了五仁月饼 165 块, 莲蓉月饼 235 块, 还剩下豆沙月饼\n没有发。豆沙月饼一共有多少块?(5 分)\n\n(2)B 区和 C 区各有 86 名来宾, 共分发了 860 块月饼, 平均每人分得多少块月饼? (5 分)", "options": [], "subject": "算术", "analysis": ""} {"id": "27288", "image": [], "answer": "【解答】解: 乘法可以用除法验算。\n\n故答案为: $\\sqrt{ }$ 。", "solution": "null", "level": "四年级", "question": "(2022 春・平坝区期末)乘法可以用除法验算。", "options": [], "subject": "算术", "analysis": "【解答】解: 乘法可以用除法验算。\n\n故答案为: $\\sqrt{ }$ 。"} {"id": "27289", "image": [], "answer": "【解答】解:125 乘 8 与 14 的和, 积是多少? 正确列式为: $125 \\times(8+14)$,所以原题说法错误。\n\n故答案为: $\\times$ 。", "solution": "null", "level": "四年级", "question": "(2022 春・沽源县期中) 125 乘 8 与 14 的和,积是多少?正确列式为: $125 \\times 8+14$ 。", "options": [], "subject": "算术", "analysis": "【解答】解:125 乘 8 与 14 的和, 积是多少? 正确列式为: $125 \\times(8+14)$,所以原题说法错误。\n\n故答案为: $\\times$ 。"} {"id": "27290", "image": [], "answer": "【解答】解: $24 \\div 4+2$\n\n$=6+2$\n\n$=8$\n\n$24 \\div(4+2)$\n\n$=24 \\div 6$\n\n$=4$\n\n两个算式的计算结果不同。\n\n原题说法错误。\n\n故答案为: $\\times$ 。", "solution": "null", "level": "四年级", "question": "(2022 春・佛山期末) $24 \\div 4+2$ 和 $24 \\div$ (4+2)的计算结果是一样的。", "options": [], "subject": "算术", "analysis": "【解答】解: $24 \\div 4+2$\n\n$=6+2$\n\n$=8$\n\n$24 \\div(4+2)$\n\n$=24 \\div 6$\n\n$=4$\n\n两个算式的计算结果不同。\n\n原题说法错误。\n\n故答案为: $\\times$ 。"} {"id": "27292", "image": [], "answer": "解【解答】解: $25+25=50$\n\n所以原题干说法正确。\n\n故答案为: $\\sqrt{ }$ 。", "solution": "null", "level": "四年级", "question": "(2022 秋・邓州市期中)差和减数都是 25, 则被减数是 50 。", "options": [], "subject": "算术", "analysis": "解【解答】解: $25+25=50$\n\n所以原题干说法正确。\n\n故答案为: $\\sqrt{ }$ 。"} {"id": "27293", "image": [], "answer": "解【解答】解:如果 $A \\times 25=B$, 那么 $B$ 是 $A$ 的 25 倍。\n所以题干说法是错误的。\n\n故答案为: $\\times$ 。", "solution": "null", "level": "四年级", "question": "(2021 秋・溷州市期末)如果 $A \\times 25=B$, 那么 $A$ 一定是 $B$ 的 25 倍。", "options": [], "subject": "算术", "analysis": "解【解答】解:如果 $A \\times 25=B$, 那么 $B$ 是 $A$ 的 25 倍。\n所以题干说法是错误的。\n\n故答案为: $\\times$ 。"} {"id": "27297", "image": [], "answer": "解【解答】解: $12 \\times 24+25$\n\n$$\n\\begin{aligned}\n& =288+25 \\\\\n& =313 \\text { (棵) }\n\\end{aligned}\n$$\n\n答: 一共要种 313 棵树。", "solution": "null", "level": "四年级", "question": "(2022 春・铜梁区期末)习近平爷爷曾说过 “绿水青山就是金山银山” , 在第 44 个植树节到来之际, 希望小学开展了 “植树造林” 实践活动。已经种了 12 排, 每排种树 24 棵, 还有 25 棵没种,\n一共要种多少棵树?", "options": [], "subject": "算术", "analysis": "解【解答】解: $12 \\times 24+25$\n\n$$\n\\begin{aligned}\n& =288+25 \\\\\n& =313 \\text { (棵) }\n\\end{aligned}\n$$\n\n答: 一共要种 313 棵树。"} {"id": "27298", "image": [], "answer": "解【解答】解: $15 \\times 3 \\times 4=180$ (毫升)\n\n$180>120$\n\n答: 3 瓶这样的止咳糖浆够一个四年级学生服用 4 天。", "solution": "null", "level": "四年级", "question": "(2022 秋・汝阳县期中)一瓶儿童止咳糖浆的规格及用法用量如下。3 瓶这样的止咳糖浆够一个四年级学生服用 4 天吗?\n\n【规格】每瓶 120 毫升\n\n【用法用量】口服, 每日 3 次\n\n7 岁以上儿童: 每次 $15 ~ 30$ 毫升\n\n3〜7 岁儿童: 每次 5〜10 毫升", "options": [], "subject": "算术", "analysis": "解【解答】解: $15 \\times 3 \\times 4=180$ (毫升)\n\n$180>120$\n\n答: 3 瓶这样的止咳糖浆够一个四年级学生服用 4 天。"} {"id": "27299", "image": [], "answer": "【解答】解: $1900-125 \\times 14$\n\n$=1900-1750$\n\n$=150$ (元)\n\n答: 还剩 150 元。", "solution": "null", "level": "四年级", "question": "(2022 秋・应城市期中)篮球每个 125 元,刘老师带 1900 元买 14 个篮球,还剩多少元?", "options": [], "subject": "算术", "analysis": "【解答】解: $1900-125 \\times 14$\n\n$=1900-1750$\n\n$=150$ (元)\n\n答: 还剩 150 元。"} {"id": "27300", "image": [], "answer": "解【解答】解: $187 \\times 37=6919$ (元)\n\n$$\n\\begin{aligned}\n& (76-37) \\times 187 \\\\\n= & 39 \\times 187 \\\\\n= & 7293 ( \\text { 元 })\n\\end{aligned}\n$$\n\n答:已经卖出的 “山东号” 航母模型共收入 6919 元, 剩下的模型还能卖 7293 元钱。", "solution": "null", "level": "四年级", "question": "(2022 秋・峄城区期中)玩具店一共运进 76 个 “山东号” 航母模型, 定价 187 元/个。第一周按定价卖出了 37 个。已经卖出的 “山东号” 航母模型共收入多少元?剩下的模型还能卖多少钱?", "options": [], "subject": "算术", "analysis": "解【解答】解: $187 \\times 37=6919$ (元)\n\n$$\n\\begin{aligned}\n& (76-37) \\times 187 \\\\\n= & 39 \\times 187 \\\\\n= & 7293 ( \\text { 元 })\n\\end{aligned}\n$$\n\n答:已经卖出的 “山东号” 航母模型共收入 6919 元, 剩下的模型还能卖 7293 元钱。"} {"id": "27301", "image": [], "answer": "【解答】解: (1) $712 \\div 2=356$ (桶)\n\n答:这些酒精能装 356 桶。\n\n(2) $356 \\times 8=2848$ (元)\n\n答: 这些酒精一共能卖 2848 元钱。", "solution": "null", "level": "四年级", "question": "(2022 秋・峄城区期中)某药店为了疫情防控消毒进来一百分之七十五的酒精 712 千克, 每 2 千克装一小桶, 每桶卖 8 元。\n\n(1)这些酒精能装几桶?\n\n(2)这些酒精一共能卖多少钱?", "options": [], "subject": "算术", "analysis": "【解答】解: (1) $712 \\div 2=356$ (桶)\n\n答:这些酒精能装 356 桶。\n\n(2) $356 \\times 8=2848$ (元)\n\n答: 这些酒精一共能卖 2848 元钱。"} {"id": "27303", "image": [], "answer": "解【解答】解: $128 \\div(3+1)$\n\n$$\n\\begin{aligned}\n= & 128 \\div 4 \\\\\n= & 32 \\text { (瓶) } \\\\\n& (128-32) \\times 2 \\\\\n= & 96 \\times 2 \\\\\n= & 192 \\text { (元) }\n\\end{aligned}\n$$\n\n答: 至少需要 192 元钱。", "solution": "null", "level": "四年级", "question": "(2022 秋・峄城区期中)某超市的矿泉水每瓶 2 元, 并且 “买三赠一”,给三年级 128 名学生每人准备 1 瓶, 至少需要多少钱?", "options": [], "subject": "算术", "analysis": "解【解答】解: $128 \\div(3+1)$\n\n$$\n\\begin{aligned}\n= & 128 \\div 4 \\\\\n= & 32 \\text { (瓶) } \\\\\n& (128-32) \\times 2 \\\\\n= & 96 \\times 2 \\\\\n= & 192 \\text { (元) }\n\\end{aligned}\n$$\n\n答: 至少需要 192 元钱。"} {"id": "27318", "image": [], "answer": "解$\\sqrt{ }$\n\n【分析】由于 0 表示没有, 根据除法的意义, 0 除以任何不是 0 的数都得 0 , 即可解题。\n\n【详解】由于 0 表示没有, 0 除以任何不是 0 的数都得 0 , 原题说法正确。\n\n故答案为: $\\sqrt{ }$\n\n【点睛】本题考查了学生对于 0 所表示意义的理解与应用。", "solution": "null", "level": "四年级", "question": "0 除以任何不是 0 的数都得 0 。()", "options": [], "subject": "算术", "analysis": "解$\\sqrt{ }$\n\n【分析】由于 0 表示没有, 根据除法的意义, 0 除以任何不是 0 的数都得 0 , 即可解题。\n\n【详解】由于 0 表示没有, 0 除以任何不是 0 的数都得 0 , 原题说法正确。\n\n故答案为: $\\sqrt{ }$\n\n【点睛】本题考查了学生对于 0 所表示意义的理解与应用。"} {"id": "27319", "image": [], "answer": "解$\\times$\n\n【分析】被除数 $\\div$ 除数=商, 商 $\\times$ 除数=被除数。根据除法的意义和各部分间的关系, 即可解答此题。\n\n【详解】由■ $\\div \\triangle=\\bullet$ 可知, ■为被除数, $\\triangle$ 为除数, $\\bullet$ 为商, 由除法各部分间关系可知, $\\bullet \\times \\Delta=■$, 题\n\n目说法错误。\n\n故答案为: $\\times$\n\n【点睛】本题主要考查除法的意义和各部分间的关系, 属于基础知识, 要熟练掌握。", "solution": "null", "level": "四年级", "question": "已知 $\\div \\Delta=\\bullet$, 所以 $\\times \\bullet=\\Delta$ 。( )", "options": [], "subject": "算术", "analysis": "解$\\times$\n\n【分析】被除数 $\\div$ 除数=商, 商 $\\times$ 除数=被除数。根据除法的意义和各部分间的关系, 即可解答此题。\n\n【详解】由■ $\\div \\triangle=\\bullet$ 可知, ■为被除数, $\\triangle$ 为除数, $\\bullet$ 为商, 由除法各部分间关系可知, $\\bullet \\times \\Delta=■$, 题\n\n目说法错误。\n\n故答案为: $\\times$\n\n【点睛】本题主要考查除法的意义和各部分间的关系, 属于基础知识, 要熟练掌握。"} {"id": "27320", "image": [], "answer": "解$\\times$\n\n【分析】一个算式里,如果只有加减法或者只有乘除法,按照从左到右的顺序依次计算,据此解答。\n\n【详解】算式中只有乘、除运算的,要按照从左到右的顺序依次计算。\n\n故答案为: $\\times$\n\n【点睛】四则混合运算的顺序:1、一个算式里, 如果只有加减法或者只有乘除法, 按照从左到右的顺序依次计算; 2、如果既有加减法、又有乘除法, 先算乘除法、再算加减法; 3、如果有括号, 先算括号里面的。", "solution": "null", "level": "四年级", "question": "算式中只有乘、除运算的, 要先算乘法,后算除法。()", "options": [], "subject": "算术", "analysis": "解$\\times$\n\n【分析】一个算式里,如果只有加减法或者只有乘除法,按照从左到右的顺序依次计算,据此解答。\n\n【详解】算式中只有乘、除运算的,要按照从左到右的顺序依次计算。\n\n故答案为: $\\times$\n\n【点睛】四则混合运算的顺序:1、一个算式里, 如果只有加减法或者只有乘除法, 按照从左到右的顺序依次计算; 2、如果既有加减法、又有乘除法, 先算乘除法、再算加减法; 3、如果有括号, 先算括号里面的。"} {"id": "27322", "image": [], "answer": "解$\\sqrt{ }$\n\n【分析】没有余数的除法中,被除数 $=$ 除数 $\\times$ 商,则被除数 $\\div ($ 除数 $\\times$ 商 $)=1$ 。据此判断即可。\n\n【详解】被除数 $\\div$ (除数 $\\times$ 商)\n\n$=$ 被除数 $\\div$ 被除数\n\n$=1$\n\n则所得的新商是 1 , 题干说法正确。\n\n故答案为: $\\sqrt{ }$\n\n【点睛】本题考查除法各部分之间的关系,常利用这个关系进行除法的验算。", "solution": "null", "level": "四年级", "question": "在没有余数的除法中, 被除数除以除数与商的积, 所得的新商是 1。()", "options": [], "subject": "算术", "analysis": "解$\\sqrt{ }$\n\n【分析】没有余数的除法中,被除数 $=$ 除数 $\\times$ 商,则被除数 $\\div ($ 除数 $\\times$ 商 $)=1$ 。据此判断即可。\n\n【详解】被除数 $\\div$ (除数 $\\times$ 商)\n\n$=$ 被除数 $\\div$ 被除数\n\n$=1$\n\n则所得的新商是 1 , 题干说法正确。\n\n故答案为: $\\sqrt{ }$\n\n【点睛】本题考查除法各部分之间的关系,常利用这个关系进行除法的验算。"} {"id": "27323", "image": [], "answer": "解$\\sqrt{ }$\n\n【分析】加数 + 加数 $=$ 和, 加数 $=$ 和 - 另一个加数, 依此判断。\n【详解】 $545+296=841$, 则 $841-296=545 ; 841-545=296$ 。\n\n因此根据 $545+296=841$, 可以写出两道减法算式。\n\n故答案为: $\\sqrt{ }$\n\n【点睛】熟练掌握加、减法的意义和各部分之间的关系是解答此题的关键。", "solution": "null", "level": "四年级", "question": "根据 $545+296=841$, 可以写出两道减法算式。( )", "options": [], "subject": "算术", "analysis": "解$\\sqrt{ }$\n\n【分析】加数 + 加数 $=$ 和, 加数 $=$ 和 - 另一个加数, 依此判断。\n【详解】 $545+296=841$, 则 $841-296=545 ; 841-545=296$ 。\n\n因此根据 $545+296=841$, 可以写出两道减法算式。\n\n故答案为: $\\sqrt{ }$\n\n【点睛】熟练掌握加、减法的意义和各部分之间的关系是解答此题的关键。"} {"id": "27328", "image": [], "answer": "解6 元\n\n【分析】用 270 元减去 20 支钢笔的钱数, 就是 15 个文具盒的钱数, 再根据单价 $=$ 总价 $\\div$ 数量, 即可求出每个文具盒的钱数。\n\n【详解】 $(270-20 \\times 9) \\div 15$\n\n$=(270-180) \\div 15$\n\n$=90 \\div 15$\n\n$=6$ (元)\n\n答: 每个文具盒 6 元。\n\n【点睛】本题考查了用整数的四则混合运算解决问题, 需准确分析题目中的数量关系。", "solution": "null", "level": "四年级", "question": "为庆祝“六一”儿童节,学校买来 20 支钢笔和 15 个文具盒奖励红花少年,一共花去 270 元。已知每支钢笔 9 元,每个文具盒多少元?", "options": [], "subject": "算术", "analysis": "解6 元\n\n【分析】用 270 元减去 20 支钢笔的钱数, 就是 15 个文具盒的钱数, 再根据单价 $=$ 总价 $\\div$ 数量, 即可求出每个文具盒的钱数。\n\n【详解】 $(270-20 \\times 9) \\div 15$\n\n$=(270-180) \\div 15$\n\n$=90 \\div 15$\n\n$=6$ (元)\n\n答: 每个文具盒 6 元。\n\n【点睛】本题考查了用整数的四则混合运算解决问题, 需准确分析题目中的数量关系。"} {"id": "27329", "image": [], "answer": "解1185 箱\n\n【分析】每辆车能装 37 箱桃子和 42 箱李子, 先把 37 和 42 相加求出每车能装多少箱水果, 再乘水果的车数, 即可求出一共订购了多少箱水果。据此解答。\n\n【详解】 $(37+42) \\times 15$\n\n$=79 \\times 15$\n\n$=1185$ (箱)\n\n答:一共订购了 1185 箱水果。\n\n【点睛】本题考查学生对乘法意义的掌握。求几个相同加数的和, 可以用乘法计算。此题也可以先分别求出一共订购了多少箱桃子和多少箱李子, 再相加, 即: $37 \\times 15+42 \\times 15$ 。", "solution": "null", "level": "四年级", "question": "扶贫助农在行动。桃李村的水果丰收了, 受疫情影响销售困难, 各地纷纷伸出援手。爱华水果店订购了 15 车水果, 每辆车能装 37 箱桃子和 42 箱李子, 一共订购了多少箱水果?", "options": [], "subject": "算术", "analysis": "解1185 箱\n\n【分析】每辆车能装 37 箱桃子和 42 箱李子, 先把 37 和 42 相加求出每车能装多少箱水果, 再乘水果的车数, 即可求出一共订购了多少箱水果。据此解答。\n\n【详解】 $(37+42) \\times 15$\n\n$=79 \\times 15$\n\n$=1185$ (箱)\n\n答:一共订购了 1185 箱水果。\n\n【点睛】本题考查学生对乘法意义的掌握。求几个相同加数的和, 可以用乘法计算。此题也可以先分别求出一共订购了多少箱桃子和多少箱李子, 再相加, 即: $37 \\times 15+42 \\times 15$ 。"} {"id": "27330", "image": [], "answer": "解19 天\n\n【分析】用小华 3 天看书的页数除以 3 , 可以计算出小华平均每天看书的页数, 再用这本书的总页数,除以小华平均每天看书的页数, 就可以计算出小华要看完这本 308 页的《红楼梦》需要的天数, 最后用看书的总天数减去已经看的天数, 可以计算看完这本书还需要多少天。\n\n【详解】 $308 \\div(42 \\div 3)-3$\n\n$=308 \\div 14-3$\n$=22-3$\n\n$=19$ (天)\n\n答: 小华要看完这本 308 页的《红楼梦》还需要 19 天。\n\n【点睛】本题考查归一问题的解题方法, 解题关键是先求出平均每天看了多少页, 再根据每天看的页数不变, 利用每天看的页数、看的天数、全书总页数之间的关系列式计算。", "solution": "null", "level": "四年级", "question": "小华利用暑假看一本经典名著《红楼梦》,3 天看了 42 页, 照这样的速度, 小华要看完这本 308 页的《红楼梦》还需要多少天?", "options": [], "subject": "算术", "analysis": "解19 天\n\n【分析】用小华 3 天看书的页数除以 3 , 可以计算出小华平均每天看书的页数, 再用这本书的总页数,除以小华平均每天看书的页数, 就可以计算出小华要看完这本 308 页的《红楼梦》需要的天数, 最后用看书的总天数减去已经看的天数, 可以计算看完这本书还需要多少天。\n\n【详解】 $308 \\div(42 \\div 3)-3$\n\n$=308 \\div 14-3$\n$=22-3$\n\n$=19$ (天)\n\n答: 小华要看完这本 308 页的《红楼梦》还需要 19 天。\n\n【点睛】本题考查归一问题的解题方法, 解题关键是先求出平均每天看了多少页, 再根据每天看的页数不变, 利用每天看的页数、看的天数、全书总页数之间的关系列式计算。"} {"id": "27331", "image": [], "answer": "解240 套\n\n【分析】从 8 月 1 日开始工作, 到 8 月 10 日完成任务, 共 10 天; 先用 230 乘 4 , 求出已生产的套数,再用 2360 减去已生产的套数, 求出剩下的套数, 然后再除以(10-4)天即可。\n\n【详解】 $(2360-230 \\times 4) \\div(10-4)$\n\n$=(2360-920) \\div(10-4)$\n\n$=1440 \\div 6$\n\n$=240$ (套)\n\n答: 剩下的任务平均每天应生产 240 套。\n\n【点睛】本题考查了运用整数减法和乘除法的意义解决实际问题的能力。", "solution": "null", "level": "四年级", "question": "家具公司要加工 2360 套课桌椅, 从 8 月 1 日开始工作, 每天生产 230 套, 生产了 4 天后, 接到通知, 要求 8 月 10 日完成任务, 剩下的任务平均每天应生产多少套?", "options": [], "subject": "算术", "analysis": "解240 套\n\n【分析】从 8 月 1 日开始工作, 到 8 月 10 日完成任务, 共 10 天; 先用 230 乘 4 , 求出已生产的套数,再用 2360 减去已生产的套数, 求出剩下的套数, 然后再除以(10-4)天即可。\n\n【详解】 $(2360-230 \\times 4) \\div(10-4)$\n\n$=(2360-920) \\div(10-4)$\n\n$=1440 \\div 6$\n\n$=240$ (套)\n\n答: 剩下的任务平均每天应生产 240 套。\n\n【点睛】本题考查了运用整数减法和乘除法的意义解决实际问题的能力。"} {"id": "27347", "image": [], "answer": "解$\\times$\n\n【分析】整数四则混合运算的运算顺序是同级运算时,从左到右依次计算; 两级运算时,先算乘除,后算加减。有括号时, 先算括号里面的; 有多层括号时, 先算小括号里的, 再算中括号里面的。据此判断。\n\n【详解】在有括号的四则运算中, 应是要先算小括号里, 再算中括号里的, 而不是先算中括号里, 再算小括号里。\n\n故答案为: $\\times$\n\n【点睛】熟练掌握整数四则混合运算的运算顺序是解决本题的关键。", "solution": "null", "level": "四年级", "question": "在有括号的四则运算中, 要先算中括号里, 再算小括号里。( )", "options": [], "subject": "算术", "analysis": "解$\\times$\n\n【分析】整数四则混合运算的运算顺序是同级运算时,从左到右依次计算; 两级运算时,先算乘除,后算加减。有括号时, 先算括号里面的; 有多层括号时, 先算小括号里的, 再算中括号里面的。据此判断。\n\n【详解】在有括号的四则运算中, 应是要先算小括号里, 再算中括号里的, 而不是先算中括号里, 再算小括号里。\n\n故答案为: $\\times$\n\n【点睛】熟练掌握整数四则混合运算的运算顺序是解决本题的关键。"} {"id": "27348", "image": [], "answer": "解$\\times$\n\n【分析】直接计算出 $3889-(108+31) \\times 5$ 的结果,看与原结果是否相等即可解答。\n\n【详解】 $3889-(108+31) \\times 5$\n\n$=3889-139 \\times 5$\n\n$=3889-695$\n\n$=3194$\n\n因为 $3194 \\neq 3144$, 所以原题计算错误。\n\n故答案为: $\\times$\n\n【点睛】本题主要考查学生对整数混合运算计算知识的掌握。", "solution": "null", "level": "四年级", "question": "$3889-(108+31) \\times 5=3889-149 \\times 5=3889-745=3144$ 。 $(\\quad)$", "options": [], "subject": "算术", "analysis": "解$\\times$\n\n【分析】直接计算出 $3889-(108+31) \\times 5$ 的结果,看与原结果是否相等即可解答。\n\n【详解】 $3889-(108+31) \\times 5$\n\n$=3889-139 \\times 5$\n\n$=3889-695$\n\n$=3194$\n\n因为 $3194 \\neq 3144$, 所以原题计算错误。\n\n故答案为: $\\times$\n\n【点睛】本题主要考查学生对整数混合运算计算知识的掌握。"} {"id": "27349", "image": [], "answer": "解$\\times$\n\n【分析】此题可以利用赋值法, 举例子解答。\n\n【详解】如果这两个数中有一个数是 1 , 因为 1 乘任何数都得原数, 则两个数的积就是另一个数, 而这两个数的和一定比另一个数大 1 。\n\n所以原题说法错误。\n\n故答案为: $\\times$\n\n【点睛】灵活应用 1 与任何数相乘都得原数的性质即可解答。", "solution": "null", "level": "四年级", "question": "两个数的积一定比这两个数的和大。()", "options": [], "subject": "算术", "analysis": "解$\\times$\n\n【分析】此题可以利用赋值法, 举例子解答。\n\n【详解】如果这两个数中有一个数是 1 , 因为 1 乘任何数都得原数, 则两个数的积就是另一个数, 而这两个数的和一定比另一个数大 1 。\n\n所以原题说法错误。\n\n故答案为: $\\times$\n\n【点睛】灵活应用 1 与任何数相乘都得原数的性质即可解答。"} {"id": "27351", "image": [], "answer": "解$\\times$\n\n【分析】根据整数的四则运算的方法, 先算乘除, 后算加减, 有括号先算括号内的, 同级运算, 从左到右依次运算,据此计算判断即可。\n\n【详解】 $315 \\times(48-48 \\div 4)$\n\n$=315 \\times(48-12)$\n\n$=315 \\times 36$\n$=5040$\n\n故答案为: $\\times$\n\n【点睛】解决本题的关键是熟练掌握整数四则混合运算的方法。", "solution": "null", "level": "四年级", "question": "$315 \\times(48-48 \\div 4)=0$ 。 $(\\quad)$", "options": [], "subject": "算术", "analysis": "解$\\times$\n\n【分析】根据整数的四则运算的方法, 先算乘除, 后算加减, 有括号先算括号内的, 同级运算, 从左到右依次运算,据此计算判断即可。\n\n【详解】 $315 \\times(48-48 \\div 4)$\n\n$=315 \\times(48-12)$\n\n$=315 \\times 36$\n$=5040$\n\n故答案为: $\\times$\n\n【点睛】解决本题的关键是熟练掌握整数四则混合运算的方法。"} {"id": "27352", "image": [], "answer": "解$\\sqrt{ }$\n\n【分析】被除数 $\\div$ 除数 $=$ 商, 根据题意可知, 这个除法算式的除数是 9 , 商是 24 , 根据被除数 $=$ 商 $\\times$除数计算并判断即可\n\n【详解】 $24 \\times 9=216$\n\n因此在一道除法算式中, 被除数是除数的 24 倍, 是商的 9 倍, 则被除数是 216 。\n\n故答案为: $\\sqrt{ }$\n\n【点睛】熟练掌握乘、除法的意义和各部分之间的关系是解答此题的关键。", "solution": "null", "level": "四年级", "question": "在一道除法算式中,被除数是除数的 24 倍,是商的 9 倍,则被除数是 216 。", "options": [], "subject": "算术", "analysis": "解$\\sqrt{ }$\n\n【分析】被除数 $\\div$ 除数 $=$ 商, 根据题意可知, 这个除法算式的除数是 9 , 商是 24 , 根据被除数 $=$ 商 $\\times$除数计算并判断即可\n\n【详解】 $24 \\times 9=216$\n\n因此在一道除法算式中, 被除数是除数的 24 倍, 是商的 9 倍, 则被除数是 216 。\n\n故答案为: $\\sqrt{ }$\n\n【点睛】熟练掌握乘、除法的意义和各部分之间的关系是解答此题的关键。"} {"id": "27355", "image": [], "answer": "解方案二\n\n【分析】把这两种方案的所需的钱数分别计算出来, 然后再进行比较大小即可解答。\n\n【详解】方案一:\n\n$100 \\times 20+(100 \\div 2) \\times 8$\n\n$=2000+50 \\times 8$\n\n$=2000+400$\n\n$=2400$ (元)\n\n方案二:\n\n$80 \\times(20+8)$\n\n$=80 \\times 28$\n\n$=2240 ($ 元)\n\n$2400>2240$\n\n答:按方案二买票便宜。\n\n【点睛】解答此题的关键是计算出每种方案所需的钱数。", "solution": "null", "level": "四年级", "question": "某风景区的门票有两种价格方案。\n\n方案一: 成人每人 100 元,儿童每人半价。\n\n方案二:团体 10 人以上(包括 10 人),每人 80 元。\n\n有成人 20 人, 儿童 8 人, 按哪个方案买票便宜?", "options": [], "subject": "算术", "analysis": "解方案二\n\n【分析】把这两种方案的所需的钱数分别计算出来, 然后再进行比较大小即可解答。\n\n【详解】方案一:\n\n$100 \\times 20+(100 \\div 2) \\times 8$\n\n$=2000+50 \\times 8$\n\n$=2000+400$\n\n$=2400$ (元)\n\n方案二:\n\n$80 \\times(20+8)$\n\n$=80 \\times 28$\n\n$=2240 ($ 元)\n\n$2400>2240$\n\n答:按方案二买票便宜。\n\n【点睛】解答此题的关键是计算出每种方案所需的钱数。"} {"id": "27356", "image": [], "answer": "解2 米\n\n【分析】成人服装的套数 $\\times$ 每套用布的长度=做成人服装用布的总长度,布的总长度一做成人服装用布的总长度 $=$ 剩下的布的长度,剩下的布的长度做儿童服装的套数=每套儿童服装用布的长度,依此列出综合算式并计算。\n\n【详解】 $(84-18 \\times 3) \\div 15$\n\n$=(84-54) \\div 15$\n\n$=30 \\div 15$\n\n$=2$ (米)\n\n答:每套儿童服装用布 2 米。\n\n【点睛】本题的重点是求出剩下布的米数, 再用除法进行列式解答。", "solution": "null", "level": "四年级", "question": "某服装厂的设计师改进了设计工艺, 经计算,用 84 米布可以做 18 套成人服装,每套用布 3 米,剩下的布则正好做 15 套儿童服装,每套儿童服装用布多少米?", "options": [], "subject": "算术", "analysis": "解2 米\n\n【分析】成人服装的套数 $\\times$ 每套用布的长度=做成人服装用布的总长度,布的总长度一做成人服装用布的总长度 $=$ 剩下的布的长度,剩下的布的长度做儿童服装的套数=每套儿童服装用布的长度,依此列出综合算式并计算。\n\n【详解】 $(84-18 \\times 3) \\div 15$\n\n$=(84-54) \\div 15$\n\n$=30 \\div 15$\n\n$=2$ (米)\n\n答:每套儿童服装用布 2 米。\n\n【点睛】本题的重点是求出剩下布的米数, 再用除法进行列式解答。"} {"id": "27357", "image": [], "answer": "解8400 元\n\n【分析】用每件上衣的价钱加上每条裤子的价钱,求出每套运动服的价钱,再乘套数,求出花费总钱数。\n\n【详解】 $(75+45) \\times 70$\n\n$=120 \\times 70$\n\n$=8400$ (元)\n\n答:一共花了 8400 元钱。\n\n【点睛】本题考查经济问题, 也可以先分别求出购买上衣和裤子的价钱, 再相加求和, 列式为 $75 \\times 70$ $+45 \\times 70$ 。", "solution": "null", "level": "四年级", "question": "运动服的上衣每件 75 元, 裤子每条 45 元, 学校购进 70 套这样的运动服,一共花了多少钱?", "options": [], "subject": "算术", "analysis": "解8400 元\n\n【分析】用每件上衣的价钱加上每条裤子的价钱,求出每套运动服的价钱,再乘套数,求出花费总钱数。\n\n【详解】 $(75+45) \\times 70$\n\n$=120 \\times 70$\n\n$=8400$ (元)\n\n答:一共花了 8400 元钱。\n\n【点睛】本题考查经济问题, 也可以先分别求出购买上衣和裤子的价钱, 再相加求和, 列式为 $75 \\times 70$ $+45 \\times 70$ 。"} {"id": "27358", "image": ["13307.jpg", "13308.jpg"], "answer": "解乙微店; 1568 元\n\n【分析】分别计算出在两个微店买 260 支同样规格的自动铅笔所花的费用, 再进行比较即可。\n\n【详解】在甲微店购买:\n\n$260 \\div 12=21$ (把) $\\qquad$ 8 (支)\n\n那么在甲店购头 21 把不够, 要买: $21+1=22$ (把)\n\n总共价格为: $22 \\times 72=1584$ (元)\n\n在乙微店购买:\n\n$260 \\div(4+1)$\n\n$=260 \\div 5$\n\n$=52$ (把)\n\n$52 \\times 30+8$\n\n$=1560+8$\n$=1568$ (元)\n\n因为 $1584>1568$, 所以建议李老师在乙微店购买。\n\n答: 建议刘老师在乙微店购买。最少要花 1568 元。\n\n【点睛】本题主要考查学生对经济优化问题的掌握, 通过具体的计算比较, 找出问题的答案即可。", "solution": "null", "level": "四年级", "question": "刘老师要购买 260 支同样规格的自动铅笔作为礼物发给学生, 他在购物网上看到了这样两个微店的信息(如下图):你建议刘老师在哪个微店购买?最少要花多少元?\n甲微店\n\n\n\n需要整把买, 免运费每把12支, 共72元。乙微店\n\n\n\n买4支送 1 支, 合计 30 元\n\n另外支付8元运費。", "options": [], "subject": "算术", "analysis": "解乙微店; 1568 元\n\n【分析】分别计算出在两个微店买 260 支同样规格的自动铅笔所花的费用, 再进行比较即可。\n\n【详解】在甲微店购买:\n\n$260 \\div 12=21$ (把) $\\qquad$ 8 (支)\n\n那么在甲店购头 21 把不够, 要买: $21+1=22$ (把)\n\n总共价格为: $22 \\times 72=1584$ (元)\n\n在乙微店购买:\n\n$260 \\div(4+1)$\n\n$=260 \\div 5$\n\n$=52$ (把)\n\n$52 \\times 30+8$\n\n$=1560+8$\n$=1568$ (元)\n\n因为 $1584>1568$, 所以建议李老师在乙微店购买。\n\n答: 建议刘老师在乙微店购买。最少要花 1568 元。\n\n【点睛】本题主要考查学生对经济优化问题的掌握, 通过具体的计算比较, 找出问题的答案即可。"} {"id": "27359", "image": [], "answer": "解240 套\n\n【分析】从 8 月 1 日开始工作, 到 8 月 10 日完成任务, 共 10 天; 先用 230 乘 4 , 求出已生产的套数,再用 2360 减去已生产的套数, 求出剩下的套数, 然后再除以 (10-4) 天即可。\n\n【详解】 $(2360-230 \\times 4) \\div(10-4)$\n\n$=(2360-920) \\div(10-4)$\n\n$=1440 \\div 6$\n\n$=240$ (套)\n\n答: 剩下的任务平均每天应生产 240 套。\n\n【点睛】本题考查了运用整数减法和乘除法的意义解决实际问题的能力。", "solution": "null", "level": "四年级", "question": "家具公司要加工 2360 套课桌椅, 从 8 月 1 日开始工作, 每天生产 230 套, 生产了 4 天后, 接到通知, 要求 8 月 10 日完成任务, 剩下的任务平均每天应生产多少套?", "options": [], "subject": "算术", "analysis": "解240 套\n\n【分析】从 8 月 1 日开始工作, 到 8 月 10 日完成任务, 共 10 天; 先用 230 乘 4 , 求出已生产的套数,再用 2360 减去已生产的套数, 求出剩下的套数, 然后再除以(10-4)天即可。\n\n【详解】 $(2360-230 \\times 4) \\div(10-4)$\n\n$=(2360-920) \\div(10-4)$\n\n$=1440 \\div 6$\n\n$=240$ (套)\n\n答: 剩下的任务平均每天应生产 240 套。\n\n【点睛】本题考查了运用整数减法和乘除法的意义解决实际问题的能力。"} {"id": "27372", "image": [], "answer": "解除; 加; 乘; 5796", "solution": "null", "level": "四年级", "question": "计算 $(238+7560 \\div 90) \\times 18$ 时, 要先算 $\\qquad$法,再算 $\\qquad$法,最后算 $\\qquad$法,计算结果是 $\\qquad$。", "options": [], "subject": "算术", "analysis": "解除; 加; 乘; 5796"} {"id": "27373", "image": [], "answer": "解 $180 \\div[$ [(120-90)×2]", "solution": "null", "level": "四年级", "question": "将算式 $120-90=30,30 \\times 2=60,180 \\div 60=3$ 改写成一个综合算式是 $\\qquad$ ${ }^{\\circ}$", "options": [], "subject": "算术", "analysis": "解 $180 \\div[$ [(120-90)×2]"} {"id": "27374", "image": [], "answer": "解 24", "solution": "null", "level": "四年级", "question": "用 $2 、 3 、 4 、 6$, 经过运算, 结果是 24 , 算式是 $\\qquad$。", "options": [], "subject": "算术", "analysis": "解 24"} {"id": "27375", "image": [], "answer": "解 50", "solution": "null", "level": "四年级", "question": "被减数、减数与差的和是 160 , 减数比差少 20 , 差是 $\\qquad$", "options": [], "subject": "算术", "analysis": "解 50"} {"id": "27376", "image": [], "answer": "解 340 ; 600", "solution": "null", "level": "四年级", "question": "在横线上填上合适的数,使等式成立。\n\n$25 \\times 8+$ $\\qquad$ $=540$ $\\qquad$ $-600 \\div 4=450$", "options": [], "subject": "算术", "analysis": "解 340 ; 600"} {"id": "27377", "image": [], "answer": "解 $8 ; 10$", "solution": "null", "level": "四年级", "question": "$a$ 是整数, 如果 $298+a>305$, $a$ 最小是 $\\qquad$ ; 如果 $23 \\times \\mathrm{a}<253$, a 最大是 $\\qquad$ _", "options": [], "subject": "算术", "analysis": "解 $8 ; 10$"} {"id": "27381", "image": ["13309.jpg"], "answer": "解解: $28+62=90$ (元)\n\n$110 \\times(90-10)$\n\n$=110 \\times 80$\n\n$=8800$ (元)\n\n$100 \\times 90=9000$ (元)\n\n$8800<9000$\n\n答:至少需要 8800 元。", "solution": "null", "level": "四年级", "question": "光明小学今年秋季还差 110 套桌椅, 如果购买下面这种桌椅至少需要多少钱?\n\n", "options": [], "subject": "算术", "analysis": "解解: $28+62=90$ (元)\n\n$110 \\times(90-10)$\n\n$=110 \\times 80$\n\n$=8800$ (元)\n\n$100 \\times 90=9000$ (元)\n\n$8800<9000$\n\n答:至少需要 8800 元。"} {"id": "27382", "image": ["13310.jpg"], "answer": "解解: $50 \\times 5+35 \\times 2$\n\n$$\n\\begin{aligned}\n& =250+70 \\\\\n& =320 \\text { (人 ) } \\\\\n& 1000 \\times 5+800 \\times 2 \\\\\n& =5000+1600 \\\\\n& =6600 \\text { (元) }\n\\end{aligned}\n$$\n\n答: 租 5 辆大客车, 2 辆中客车最省钱, 最少要花 6600 元。", "solution": "null", "level": "四年级", "question": "国家速滑馆又被称为“冰丝带”,是 2022 年北京冬奥会北京主赛区唯一新建的冰上竞赛场馆。某学校计划组织四年级师生 320 人去“冰丝带”开展实践活动。怎样租车最省钱?最少要花多少钱?\n\n", "options": [], "subject": "算术", "analysis": "解解: $50 \\times 5+35 \\times 2$\n\n$$\n\\begin{aligned}\n& =250+70 \\\\\n& =320 \\text { (人 ) } \\\\\n& 1000 \\times 5+800 \\times 2 \\\\\n& =5000+1600 \\\\\n& =6600 \\text { (元) }\n\\end{aligned}\n$$\n\n答: 租 5 辆大客车, 2 辆中客车最省钱, 最少要花 6600 元。"} {"id": "27383", "image": [], "answer": "解解:方案一:\n\n$2 \\times 40+20 \\times 5$\n\n$=80+100$\n\n$=180$ (元)\n\n方案二:\n\n$(5+2) \\times 30$\n\n$=7 \\times 30$\n\n$=210$ (元)\n\n210 元 $>180$ 元 或 $180<210$\n\n答: 选择方案一购票合算。", "solution": "null", "level": "四年级", "question": "六一儿童节期间,微山欢乐彩虹谷推出“水上游乐项目”的两种价格方案。\n\n2 个大人,5 个孩子,选哪种方案最合算?\n\n方案一 成人票: 40 元/人 儿童票: 20 元/人\n\n方案二 团体 7 人以上(含 7 人)每人 30 元", "options": [], "subject": "算术", "analysis": "解解:方案一:\n\n$2 \\times 40+20 \\times 5$\n\n$=80+100$\n\n$=180$ (元)\n\n方案二:\n\n$(5+2) \\times 30$\n\n$=7 \\times 30$\n\n$=210$ (元)\n\n210 元 $>180$ 元 或 $180<210$\n\n答: 选择方案一购票合算。"} {"id": "27384", "image": [], "answer": "解: $7 \\times 8+5 \\times 2$\n$$\n\\begin{aligned}\n& =56+10 \\\\\n& =66 \\text { (套) } \\\\\n& 400 \\times 7+300 \\times 2 \\\\\n& =2800+600 \\\\\n& =3400 \\text { (元) }\n\\end{aligned}\n$$\n\n答: 头 7 卷大卷布和 2 卷小卷布最省钱, 需要 3400 元。", "solution": "null", "level": "四年级", "question": "学校做 66 套表演服装, 要按整卷买布。大卷布, 每卷 400 元, 可以做 8 套表演服装; 小卷布,每卷 300 元,可以做 5 套表演服装。怎样买布最省钱?", "options": [], "subject": "算术", "analysis": "解: $7 \\times 8+5 \\times 2$\n$$\n\\begin{aligned}\n& =56+10 \\\\\n& =66 \\text { (套) } \\\\\n& 400 \\times 7+300 \\times 2 \\\\\n& =2800+600 \\\\\n& =3400 \\text { (元) }\n\\end{aligned}\n$$\n\n答: 头 7 卷大卷布和 2 卷小卷布最省钱, 需要 3400 元。"} {"id": "27399", "image": [], "answer": "解$\\times$\n\n【分析】计算 $128-68+32$ 时,先算减法,再算加法。计算 $128-(68+32)$ 时, 先算小括号里面的加法, 再算括号外面的减法。据此分别求出两个算式的得数, 看这两个算式的得数是否相等。\n\n【详解】 $128-68+32$\n\n$=60+32$\n\n$=92$\n\n$128-(68+32)$\n\n$=128-100$\n\n$=28$\n\n则 $128-68+32 \\neq 128-(68+32)$ 。\n\n故答案为: $\\times$\n\n【点睛】本题考查整数四则混合运算, 关键是明确各个算式的运算顺序,再进行计算。", "solution": "null", "level": "四年级", "question": "$128-68+32=128-(68+32) 。(\\quad)$", "options": [], "subject": "算术", "analysis": "解$\\times$\n\n【分析】计算 $128-68+32$ 时,先算减法,再算加法。计算 $128-(68+32)$ 时, 先算小括号里面的加法, 再算括号外面的减法。据此分别求出两个算式的得数, 看这两个算式的得数是否相等。\n\n【详解】 $128-68+32$\n\n$=60+32$\n\n$=92$\n\n$128-(68+32)$\n\n$=128-100$\n\n$=28$\n\n则 $128-68+32 \\neq 128-(68+32)$ 。\n\n故答案为: $\\times$\n\n【点睛】本题考查整数四则混合运算, 关键是明确各个算式的运算顺序,再进行计算。"} {"id": "27400", "image": [], "answer": "解$\\times$\n\n【分析】0 可以作被除数, 但不能作除数。若 0 可以作除数, 例如 $5 \\div 0=0$, 根据除数 $\\times$ 商 $=$ 被除数, $0 \\times 0$应得 5 , 而 $0 \\times 0$ 得 0 , 则 0 不可以作除数。\n\n【详解】 $0 \\div 0=0$ 不成立, 0 除以任何一个非 0 的数都得 0 , 但 0 不能作除数, 题干说法错误。\n\n故答案为: $\\times$\n\n【点睛】本题考查 0 在除法中的特性, 关键是明确 0 不能作除数。", "solution": "null", "level": "四年级", "question": "因为 $0 \\div 0=0$ 成立, 所以 0 可以作除数。 $($ )", "options": [], "subject": "算术", "analysis": "解$\\times$\n\n【分析】0 可以作被除数, 但不能作除数。若 0 可以作除数, 例如 $5 \\div 0=0$, 根据除数 $\\times$ 商 $=$ 被除数, $0 \\times 0$应得 5 , 而 $0 \\times 0$ 得 0 , 则 0 不可以作除数。\n\n【详解】 $0 \\div 0=0$ 不成立, 0 除以任何一个非 0 的数都得 0 , 但 0 不能作除数, 题干说法错误。\n\n故答案为: $\\times$\n\n【点睛】本题考查 0 在除法中的特性, 关键是明确 0 不能作除数。"} {"id": "27401", "image": [], "answer": "解$\\times$\n\n【分析】混合运算的计算顺序是:先算乘、除法,再算加、减法,有括号时应先算括号里面的,再算括号外面的, 依此判断。\n\n【详解】计算 $360-360 \\div(90 \\div 2)$ 时, 应先算小括号里的除法, 再算小括号外的除法, 最后算减法, 因此是按从右往左的顺序计算。\n故答案为: $\\times$\n\n【点睛】熟练掌握混合运算的计算顺序是解答此题的关键。", "solution": "null", "level": "四年级", "question": "计算 $360-360 \\div(90 \\div 2)$ 时, 是按从左往右的顺序计算。( )", "options": [], "subject": "算术", "analysis": "解$\\times$\n\n【分析】混合运算的计算顺序是:先算乘、除法,再算加、减法,有括号时应先算括号里面的,再算括号外面的, 依此判断。\n\n【详解】计算 $360-360 \\div(90 \\div 2)$ 时, 应先算小括号里的除法, 再算小括号外的除法, 最后算减法, 因此是按从右往左的顺序计算。\n故答案为: $\\times$\n\n【点睛】熟练掌握混合运算的计算顺序是解答此题的关键。"} {"id": "27402", "image": [], "answer": "解$\\sqrt{ }$\n\n【分析】把○等于 5 代入算式 $(55-\\mathrm{O}) \\div 5$, 求出结果, 然后再和 10 进行比较即可判断。\n\n【详解】 $(55-5) \\div 5$\n\n$=50 \\div 5$\n\n$=10$\n\n所以原题说法正确。\n\n故答案为: $\\sqrt{ }$ 。\n\n【点睛】本题考查了简单的四则混合运算, 计算时先理清楚运算顺序, 根据运算顺序逐步求解即可。", "solution": "null", "level": "四年级", "question": "由 $(55-\\bigcirc) \\div 5=10$ 可以推算出, $\\mathrm{O}$ 等于 5 。()", "options": [], "subject": "算术", "analysis": "解$\\sqrt{ }$\n\n【分析】把○等于 5 代入算式 $(55-\\mathrm{O}) \\div 5$, 求出结果, 然后再和 10 进行比较即可判断。\n\n【详解】 $(55-5) \\div 5$\n\n$=50 \\div 5$\n\n$=10$\n\n所以原题说法正确。\n\n故答案为: $\\sqrt{ }$ 。\n\n【点睛】本题考查了简单的四则混合运算, 计算时先理清楚运算顺序, 根据运算顺序逐步求解即可。"} {"id": "27403", "image": [], "answer": "解$\\sqrt{ }$\n\n【分析】根据被除数=商 $\\times$ 除数十余数,可得:商 $\\times$ 除数=被除数一余数,据此判断即可。\n\n【详解】如果 $\\mathrm{a} \\div \\mathrm{b}=6 \\ldots \\ldots 8$, 那么 $\\mathrm{b} \\times 6=\\mathrm{a}-8$, 原题说法正确。\n\n故答案为: $\\sqrt{ }$\n\n【点睛】本题考查了被除数、除数、商和余数四个量之间的关系。", "solution": "null", "level": "四年级", "question": "如果 $\\mathrm{a} \\div \\mathrm{b}=6 \\ldots \\ldots 8$, 那么 $\\mathrm{b} \\times 6=\\mathrm{a}-8$ 。 ( )", "options": [], "subject": "算术", "analysis": "解$\\sqrt{ }$\n\n【分析】根据被除数=商 $\\times$ 除数十余数,可得:商 $\\times$ 除数=被除数一余数,据此判断即可。\n\n【详解】如果 $\\mathrm{a} \\div \\mathrm{b}=6 \\ldots \\ldots 8$, 那么 $\\mathrm{b} \\times 6=\\mathrm{a}-8$, 原题说法正确。\n\n故答案为: $\\sqrt{ }$\n\n【点睛】本题考查了被除数、除数、商和余数四个量之间的关系。"} {"id": "27408", "image": [], "answer": "解8 年\n\n【分析】根据题意, 张老师今年 46 岁, 三个学生的年龄都是 10 岁, 那么张老师每增加 1 岁, 三个学生共增加 3 岁, 则每年的年龄差是 $(3-1)$ 岁, 原来相差 $(46-3 \\times 10)$ 岁, 最后用除法, 即可得到多少年后三个学生的年龄和等于张老师的年龄, 据此解答。\n\n【详解】 $(46-3 \\times 10) \\div(3-1)$\n\n$=(46-30) \\div(3-1)$\n\n$=16 \\div 2$\n\n$=8($ 年 $)$\n\n答: 8 年后三个学生的年龄和等于张老师的年龄。\n\n【点睛】解答本题的关键抓住每一年所有的人都在增长, 抓住这个特点, 根据题目给出的条件即可解答这类应用题。", "solution": "null", "level": "四年级", "question": "张老师今年 46 岁, 三个学生的年龄都是 10 岁, 多少年后三个学生的年龄和等于张老师的年龄?", "options": [], "subject": "算术", "analysis": "解8 年\n\n【分析】根据题意, 张老师今年 46 岁, 三个学生的年龄都是 10 岁, 那么张老师每增加 1 岁, 三个学生共增加 3 岁, 则每年的年龄差是 $(3-1)$ 岁, 原来相差 $(46-3 \\times 10)$ 岁, 最后用除法, 即可得到多少年后三个学生的年龄和等于张老师的年龄, 据此解答。\n\n【详解】 $(46-3 \\times 10) \\div(3-1)$\n\n$=(46-30) \\div(3-1)$\n\n$=16 \\div 2$\n\n$=8($ 年 $)$\n\n答: 8 年后三个学生的年龄和等于张老师的年龄。\n\n【点睛】解答本题的关键抓住每一年所有的人都在增长, 抓住这个特点, 根据题目给出的条件即可解答这类应用题。"} {"id": "27410", "image": [], "answer": "解12 天\n\n【分析】用欢欢 5 天读书的总页数除以读书的天数, 可以计算出欢欢平均每天读多少页,再用这本书的总页数减去已读的页数可以计算出未读的页数, 最后用未读的页数除以欢欢平均每天读的页数, 即可计算出剩下的要几天读完。\n\n【详解】 $(272-80) \\div(80 \\div 5)$\n\n$=192 \\div 16$\n\n$=12$ (天)\n\n答: 剩下的要 12 天读完。\n\n【点睛】本题考查归一问题的解题方法, 解题关键是先求出一份数是多少, 再根据一份数不变, 利用读书总页数、读书的天数和平均每天读的页数之间的关系列式计算。", "solution": "null", "level": "四年级", "question": "欢欢借来一本 272 页的书, 5 天读了 80 页, 照这样计算, 剩下的要几天读完?", "options": [], "subject": "算术", "analysis": "解12 天\n\n【分析】用欢欢 5 天读书的总页数除以读书的天数, 可以计算出欢欢平均每天读多少页,再用这本书的总页数减去已读的页数可以计算出未读的页数, 最后用未读的页数除以欢欢平均每天读的页数, 即可计算出剩下的要几天读完。\n\n【详解】 $(272-80) \\div(80 \\div 5)$\n\n$=192 \\div 16$\n\n$=12$ (天)\n\n答: 剩下的要 12 天读完。\n\n【点睛】本题考查归一问题的解题方法, 解题关键是先求出一份数是多少, 再根据一份数不变, 利用读书总页数、读书的天数和平均每天读的页数之间的关系列式计算。"} {"id": "27411", "image": [], "answer": "解551 千米\n\n【分析】根据路程 $=$ 速度 $\\times$ 时间, 求出甲车和乙车 3 小时一共行了多少千米, 再加上 80 千米, 就是全程, 据此解答。\n\n【详解】 $(72+85) \\times 3+80$\n\n$=157 \\times 3+80$\n\n$=471+80$\n\n$=551$ (千米)\n\n答: 两地相距 551 千米。\n\n【点睛】此题主要考查路程、速度、时间三者的关系式:路程 $=$ 速度 $\\times$ 时间, 速度 $=$ 路程 $\\div$ 时间, 时间 $=$ 路程 $\\div$ 速度, 灵活变形列式解决问题。", "solution": "null", "level": "四年级", "question": "两辆汽车同时从两地出发,相向而行。已知甲车每小时行 72 千米,乙车每小时行 85 千米。 3 小时后, 两车还相距 80 千米, 两地相距多远?", "options": [], "subject": "算术", "analysis": "解551 千米\n\n【分析】根据路程 $=$ 速度 $\\times$ 时间, 求出甲车和乙车 3 小时一共行了多少千米, 再加上 80 千米, 就是全程, 据此解答。\n\n【详解】 $(72+85) \\times 3+80$\n\n$=157 \\times 3+80$\n\n$=471+80$\n\n$=551$ (千米)\n\n答: 两地相距 551 千米。\n\n【点睛】此题主要考查路程、速度、时间三者的关系式:路程 $=$ 速度 $\\times$ 时间, 速度 $=$ 路程 $\\div$ 时间, 时间 $=$ 路程 $\\div$ 速度, 灵活变形列式解决问题。"} {"id": "27412", "image": [], "answer": "解960 米\n\n【分析】根据速度 $=$ 路程 $\\div$ 时间, 可以计算出小刚行走的速度是多少,再根据路程 $=$ 速度 $\\times$ 时间,可以计算出他家到学校有多远。\n\n【详解】 $400 \\div 5 \\times 12$\n\n$=80 \\times 12$\n\n$=960$ (米)\n\n答:他家到学校有 960 米。\n【点睛】本题考查行程问题的解题方法, 解题关键是抓住题中速度不变, 利用行程问题的数量关系:速度 $=$ 路程 $\\div$ 时间, 路程 $=$ 速度 $\\times$ 时间, 列式计算。", "solution": "null", "level": "四年级", "question": "小刚从家到学校, 5 分钟走 400 米。照这样的速度, 他从家到学校要走 12 分钟。他家到学校有多远?", "options": [], "subject": "算术", "analysis": "解960 米\n\n【分析】根据速度 $=$ 路程 $\\div$ 时间, 可以计算出小刚行走的速度是多少,再根据路程 $=$ 速度 $\\times$ 时间,可以计算出他家到学校有多远。\n\n【详解】 $400 \\div 5 \\times 12$\n\n$=80 \\times 12$\n\n$=960$ (米)\n\n答:他家到学校有 960 米。\n【点睛】本题考查行程问题的解题方法, 解题关键是抓住题中速度不变, 利用行程问题的数量关系:速度 $=$ 路程 $\\div$ 时间, 路程 $=$ 速度 $\\times$ 时间, 列式计算。"} {"id": "27498", "image": [], "answer": "解$\\sqrt{ }$\n【分析】乘法交换律的特点是两个数相乘, 交换两个因数的位置, 积不变; 依此进行判断即可。\n\n【详解】 $25 \\times 12 \\times 4=25 \\times 4 \\times 12$ 中, 交换了 12 与 4 的位置, 因此应用了乘法交换律。\n\n故答案为: $\\sqrt{ }$\n\n【点睛】熟练掌握乘法交换律的特点是解答此题的关键。", "solution": "null", "level": "四年级", "question": "$25 \\times 12 \\times 4=25 \\times 4 \\times 12$ 应用了乘法交换律。 $(\\quad)$", "options": [], "subject": "算术", "analysis": "解$\\sqrt{ }$\n【分析】乘法交换律的特点是两个数相乘, 交换两个因数的位置, 积不变; 依此进行判断即可。\n\n【详解】 $25 \\times 12 \\times 4=25 \\times 4 \\times 12$ 中, 交换了 12 与 4 的位置, 因此应用了乘法交换律。\n\n故答案为: $\\sqrt{ }$\n\n【点睛】熟练掌握乘法交换律的特点是解答此题的关键。"} {"id": "27499", "image": [], "answer": "解$\\sqrt{ }$\n\n【分析】根据乘法分配律: $a \\times b+a \\times c=a \\times(b+c) 、 a \\times b-a \\times c=a \\times(b-c)$, 对题意进行判断, 据此解决。\n\n【详解】根据乘法分配律, 我们可以得出 $25 \\times(40-4)=25 \\times 40-25 \\times 4$ 。\n\n故答案为: $\\sqrt{ }$\n\n【点睛】解决本题的关键是熟练掌握乘法分配律的形式。", "solution": "null", "level": "四年级", "question": "$25 \\times(40-4)=25 \\times 40-25 \\times 4$ 运用了乘法分配律。()", "options": [], "subject": "算术", "analysis": "解$\\sqrt{ }$\n\n【分析】根据乘法分配律: $a \\times b+a \\times c=a \\times(b+c) 、 a \\times b-a \\times c=a \\times(b-c)$, 对题意进行判断, 据此解决。\n\n【详解】根据乘法分配律, 我们可以得出 $25 \\times(40-4)=25 \\times 40-25 \\times 4$ 。\n\n故答案为: $\\sqrt{ }$\n\n【点睛】解决本题的关键是熟练掌握乘法分配律的形式。"} {"id": "27500", "image": [], "answer": "解$\\times$\n\n【分析】计算 $103 \\times 25$ 时, 可将 103 写成 $100+3$, 然后再根据乘法分配律的特点进行简算, 乘法分配律的特点是两个数的和与一个数相乘, 可以先把它们与这个数分别相乘, 再相加; 依此判断。\n\n【详解】 $103 \\times 25=(100+3) \\times 25=100 \\times 25-3 \\times 25$ 。\n\n故答案为: $\\times$\n\n【点睛】熟练掌握乘法分配律的特点是解答此题的关键。", "solution": "null", "level": "四年级", "question": "$103 \\times 25=100 \\times 25+3$ 。 $(\\quad)$", "options": [], "subject": "算术", "analysis": "解$\\times$\n\n【分析】计算 $103 \\times 25$ 时, 可将 103 写成 $100+3$, 然后再根据乘法分配律的特点进行简算, 乘法分配律的特点是两个数的和与一个数相乘, 可以先把它们与这个数分别相乘, 再相加; 依此判断。\n\n【详解】 $103 \\times 25=(100+3) \\times 25=100 \\times 25-3 \\times 25$ 。\n\n故答案为: $\\times$\n\n【点睛】熟练掌握乘法分配律的特点是解答此题的关键。"} {"id": "27501", "image": [], "answer": "解$\\times$\n\n【分析】根据四则运算法则, 有除法有小括号时,应先算小括号内的; 除法没有分配律。据此判断即可。\n\n【详解】 $36 \\div(4+9)$\n\n$=36 \\div 13$\n\n$=2 \\ldots \\ldots 10$\n\n$36 \\div 4+36 \\div 9$\n\n$=9+4$\n\n$=13$\n\n两式得数不相等, 所以判断错误。\n\n故答案为: $\\times$\n\n【点睛】乘法分配律: 两个数的和与一个数相乘, 可以先把它们与这个数分别相乘, 再相加。注意没有除法分配律。", "solution": "null", "level": "四年级", "question": "$36 \\div(4+9)=36 \\div 4+36 \\div 9$ 。 $($ )", "options": [], "subject": "算术", "analysis": "解$\\times$\n\n【分析】根据四则运算法则, 有除法有小括号时,应先算小括号内的; 除法没有分配律。据此判断即可。\n\n【详解】 $36 \\div(4+9)$\n\n$=36 \\div 13$\n\n$=2 \\ldots \\ldots 10$\n\n$36 \\div 4+36 \\div 9$\n\n$=9+4$\n\n$=13$\n\n两式得数不相等, 所以判断错误。\n\n故答案为: $\\times$\n\n【点睛】乘法分配律: 两个数的和与一个数相乘, 可以先把它们与这个数分别相乘, 再相加。注意没有除法分配律。"} {"id": "27502", "image": [], "answer": "解$\\times$\n【分析】整数除法的性质: $9 \\times 8=72$, 一个数连续除以两个数, 可以除以这两个数的积。\n\n【详解】 $1800 \\div 72$\n\n$=1800 \\div(9 \\times 8)$\n\n$=1800 \\div 9 \\div 8$\n\n$\\neq 1800-9-8$, 所以判断错误。\n\n故答案为: $\\times$\n\n【点睛】此题考查除法的性质、减法的性质, 熟练掌握各个运算定律是解题的关键。", "solution": "null", "level": "四年级", "question": "$1800 \\div 72=1800-9-8$ 。 $(\\quad)$", "options": [], "subject": "算术", "analysis": "解$\\times$\n【分析】整数除法的性质: $9 \\times 8=72$, 一个数连续除以两个数, 可以除以这两个数的积。\n\n【详解】 $1800 \\div 72$\n\n$=1800 \\div(9 \\times 8)$\n\n$=1800 \\div 9 \\div 8$\n\n$\\neq 1800-9-8$, 所以判断错误。\n\n故答案为: $\\times$\n\n【点睛】此题考查除法的性质、减法的性质, 熟练掌握各个运算定律是解题的关键。"} {"id": "27504", "image": [], "answer": "解$20000 ; 18000 ; 50 ; 600$\n\n$7 ; 0 ; 25 ; 1230$\n\n【详解】略", "solution": "null", "level": "四年级", "question": "直接写得数。\n\n| $500 \\times 40=$ | $600 \\times 30=$ | $550 \\div 11=$ | $25 \\times 6 \\times 4=$ |\n| :--- | :--- | :--- | :--- |\n| $490 \\div 70=$ | $900 \\times 0=$ | $1000 \\div 40=$ | $123 \\times 11-123=$ |", "options": [], "subject": "算术", "analysis": "解$20000 ; 18000 ; 50 ; 600$\n\n$7 ; 0 ; 25 ; 1230$\n\n【详解】略"} {"id": "27505", "image": [], "answer": "解$200 ; 2600$\n\n$900 ; 2$\n\n【分析】(1)根据加法交换律和加法结合律进行简算;\n\n(2)根据乘法分配律进行简算;\n\n(3)根据乘法分配律进行简算;\n\n(4)根据除法的性质进行简算。\n\n【详解】 $78+138+22-38$\n\n$=78+22+(138-38)$\n\n$=100+100$\n\n$=200$\n\n$104 \\times 25$\n\n$=(100+4) \\times 25$\n\n$=100 \\times 25+4 \\times 25$\n\n$=2500+100$\n\n$=2600$\n\n$172 \\times 45-45 \\times 152$\n\n$=(172-152) \\times 45$\n\n$=20 \\times 45$\n\n$=900$\n\n$2000 \\div 125 \\div 8$\n\n$=2000 \\div(125 \\times 8)$\n$=2000 \\div 1000$\n\n$=2$", "solution": "null", "level": "四年级", "question": "简便计算。\n\n$78+138+22-38$\n\n$104 \\times 25$\n\n$172 \\times 45-45 \\times 152$\n\n$2000 \\div 125 \\div 8$", "options": [], "subject": "算术", "analysis": "解$200 ; 2600$\n\n$900 ; 2$\n\n【分析】(1)根据加法交换律和加法结合律进行简算;\n\n(2)根据乘法分配律进行简算;\n\n(3)根据乘法分配律进行简算;\n\n(4)根据除法的性质进行简算。\n\n【详解】 $78+138+22-38$\n\n$=78+22+(138-38)$\n\n$=100+100$\n\n$=200$\n\n$104 \\times 25$\n\n$=(100+4) \\times 25$\n\n$=100 \\times 25+4 \\times 25$\n\n$=2500+100$\n\n$=2600$\n\n$172 \\times 45-45 \\times 152$\n\n$=(172-152) \\times 45$\n\n$=20 \\times 45$\n\n$=900$\n\n$2000 \\div 125 \\div 8$\n\n$=2000 \\div(125 \\times 8)$\n$=2000 \\div 1000$\n\n$=2$"} {"id": "27506", "image": [], "answer": "解258 千米\n\n【分析】根据题意, 用从甲城到乙城的路程, 减去上午行驶的路程, 再减去下午行驶的路程, 即可解答。\n\n【详解】 $758-276-224$\n\n$=758-(276+224)$\n\n$=758-500$\n\n$=258$ (千米)\n\n答: 还要行驶 258 千米才能到达乙城。\n\n【点睛】此题考查了根据整数减法的意义解答问题的能力, 这道题也是减法性质的应用。", "solution": "null", "level": "四年级", "question": "甲城和乙城相距 758 千米, 一辆汽车从甲城开往乙城, 上午行驶了 276 千米, 下午行驶了 224 千\n米, 还要行驶多少千米才能到达乙城?", "options": [], "subject": "算术", "analysis": "解258 千米\n\n【分析】根据题意, 用从甲城到乙城的路程, 减去上午行驶的路程, 再减去下午行驶的路程, 即可解答。\n\n【详解】 $758-276-224$\n\n$=758-(276+224)$\n\n$=758-500$\n\n$=258$ (千米)\n\n答: 还要行驶 258 千米才能到达乙城。\n\n【点睛】此题考查了根据整数减法的意义解答问题的能力, 这道题也是减法性质的应用。"} {"id": "27507", "image": [], "answer": "解2200 千克\n\n【分析】每箱雪梨的重量 $\\times$ 雪梨的箱数 + 每箱苹果的重量 $\\times$ 苹果的箱数 $=$ 运来苹果和雪梨的总重量, 依此列式并根据乘法分配律的特点进行简算即可。\n\n【详解】 $30 \\times 40+25 \\times 40$\n\n$=(30+25) \\times 40$\n\n$=55 \\times 40$\n\n$=2200$ (千克)\n\n答:一共运来雪梨和苹果 2200 千克。\n\n【点睛】此题考查的是运用乘法分配律的特点解答实际问题,应熟练掌握乘法分配律的特点。", "solution": "null", "level": "四年级", "question": "超市今天运来雪梨和苹果各 40 箱, 每箱雪梨重 $30 \\mathrm{~kg}$, 每箱苹果重 $25 \\mathrm{~kg}$ 。一共运来雪梨和苹果多少千克?", "options": [], "subject": "算术", "analysis": "解2200 千克\n\n【分析】每箱雪梨的重量 $\\times$ 雪梨的箱数 + 每箱苹果的重量 $\\times$ 苹果的箱数 $=$ 运来苹果和雪梨的总重量, 依此列式并根据乘法分配律的特点进行简算即可。\n\n【详解】 $30 \\times 40+25 \\times 40$\n\n$=(30+25) \\times 40$\n\n$=55 \\times 40$\n\n$=2200$ (千克)\n\n答:一共运来雪梨和苹果 2200 千克。\n\n【点睛】此题考查的是运用乘法分配律的特点解答实际问题,应熟练掌握乘法分配律的特点。"} {"id": "27509", "image": ["13507.jpg"], "answer": "解2500 元\n\n【分析】上衣的单价加裤子的单价等于运动服一套的价钱, 再乘要买的套数即等于一共需要的钱。\n\n【详解】 $(63+37) \\times 25$\n\n$=100 \\times 25$\n\n$=2500$ (元)\n\n答: 一共需要 2500 元钱。\n\n【点睛】本题主要考查学生对乘法分配律的掌握和灵活运用。", "solution": "null", "level": "四年级", "question": "王老师要给学校 25 名运动员头 25 套服装, 一共需要多少钱?\n\n", "options": [], "subject": "算术", "analysis": "解2500 元\n\n【分析】上衣的单价加裤子的单价等于运动服一套的价钱, 再乘要买的套数即等于一共需要的钱。\n\n【详解】 $(63+37) \\times 25$\n\n$=100 \\times 25$\n\n$=2500$ (元)\n\n答: 一共需要 2500 元钱。\n\n【点睛】本题主要考查学生对乘法分配律的掌握和灵活运用。"} {"id": "27510", "image": [], "answer": "解1500 元\n\n【分析】先用加法计算出篮球和足球的单价和, 再根据数量是 25 个, 用单价和乘数量就是篮球和足球各 25 个需要的钱数, 据此解答。\n\n【详解】 $(28+32) \\times 25$\n\n$=60 \\times 25$\n\n$=1500$ (元)\n\n答: 一共需要 1500 元钱。\n\n【点睛】本题考查两位数乘两位数的实际应用, 总找到正确的数量关系并正确计算是解答本题的关键。", "solution": "null", "level": "四年级", "question": "学校为了落实阳光锻炼 1 小时的活动, 需要补充购买篮球和足球各 25 个。篮球每个 28 元, 足球每个 32 元, 一共需要多少钱?", "options": [], "subject": "算术", "analysis": "解1500 元\n\n【分析】先用加法计算出篮球和足球的单价和, 再根据数量是 25 个, 用单价和乘数量就是篮球和足球各 25 个需要的钱数, 据此解答。\n\n【详解】 $(28+32) \\times 25$\n\n$=60 \\times 25$\n\n$=1500$ (元)\n\n答: 一共需要 1500 元钱。\n\n【点睛】本题考查两位数乘两位数的实际应用, 总找到正确的数量关系并正确计算是解答本题的关键。"} {"id": "27524", "image": [], "answer": "解$\\sqrt{ }$\n\n【分析】整数四则混合运算的运算顺序是同级运算时, 从左到右依次计算; 两级运算时,先算乘除,后算加减。有括号时, 先算括号里面的。据此判断即可。\n\n【详解】计算 $(48-8) \\div(15+5)$ 时, 先算小括号里面的减法和加法, 再算括号外面的除法。也就是先求差与和, 再求商。\n\n故答案为: $\\sqrt{ }$ 。\n\n【点睛】本题考查整数四则混合运算的运算顺序,需熟练掌握。", "solution": "null", "level": "四年级", "question": "计算 $(48-8) \\div(15+5)$ 的顺序是先求差与和, 再求商。 $($ )", "options": [], "subject": "算术", "analysis": "解$\\sqrt{ }$\n\n【分析】整数四则混合运算的运算顺序是同级运算时, 从左到右依次计算; 两级运算时,先算乘除,后算加减。有括号时, 先算括号里面的。据此判断即可。\n\n【详解】计算 $(48-8) \\div(15+5)$ 时, 先算小括号里面的减法和加法, 再算括号外面的除法。也就是先求差与和, 再求商。\n\n故答案为: $\\sqrt{ }$ 。\n\n【点睛】本题考查整数四则混合运算的运算顺序,需熟练掌握。"} {"id": "27525", "image": [], "answer": "解$\\times$\n\n【分析】四则混合运算的运算顺序:同级运算, 从左往右依次进行计算; 既有加减, 又有乘除, 先算乘除,再算加减;有括号,先算括号里面的。\n\n【详解】 $54-( 72 \\div 6 )$ 先算除法, 再算减法, 去掉括号后 $54-72 \\div 6$ 也是先算除法, 再算减法, 计算结果不发生变化。\n\n故答案为: $\\times$\n\n【点睛】本题主要考查学生对整数混合运算运算顺序的掌握和灵活运用。", "solution": "null", "level": "四年级", "question": "去掉算式 $54-( 72 \\div 6 )$ 的括号后, 计算结果发生了变化。( )", "options": [], "subject": "算术", "analysis": "解$\\times$\n\n【分析】四则混合运算的运算顺序:同级运算, 从左往右依次进行计算; 既有加减, 又有乘除, 先算乘除,再算加减;有括号,先算括号里面的。\n\n【详解】 $54-( 72 \\div 6 )$ 先算除法, 再算减法, 去掉括号后 $54-72 \\div 6$ 也是先算除法, 再算减法, 计算结果不发生变化。\n\n故答案为: $\\times$\n\n【点睛】本题主要考查学生对整数混合运算运算顺序的掌握和灵活运用。"} {"id": "27526", "image": [], "answer": "解$\\times$\n\n【分析】直接计算出 $99 \\times 10 \\div 99 \\times 10$ 的结果即可解答。\n\n【详解】 $99 \\times 10 \\div 99 \\times 10$\n\n$=990 \\div 99 \\times 10$\n\n$=10 \\times 10$\n\n$=100$\n\n故答案为: $\\times$\n\n【点睛】本题主要考查学生对整数混合运算运算顺序的掌握和灵活运用。", "solution": "null", "level": "四年级", "question": "$99 \\times 10 \\div 99 \\times 10=1$ 。 $(\\quad)$", "options": [], "subject": "算术", "analysis": "解$\\times$\n\n【分析】直接计算出 $99 \\times 10 \\div 99 \\times 10$ 的结果即可解答。\n\n【详解】 $99 \\times 10 \\div 99 \\times 10$\n\n$=990 \\div 99 \\times 10$\n\n$=10 \\times 10$\n\n$=100$\n\n故答案为: $\\times$\n\n【点睛】本题主要考查学生对整数混合运算运算顺序的掌握和灵活运用。"} {"id": "27527", "image": [], "answer": "解$\\times$\n\n【分析】根据整数四则混合运算的顺序,直接计算出结果,然后再进行判断即可。\n\n【详解】 $63 \\div(9-2)$\n\n$=63 \\div 7$\n\n$=9$\n\n$63 \\div 9-2$\n\n$=7-2$\n\n$=5$\n\n所以 $63 \\div(9-2)$ 与 $63 \\div 9-2$ 的结果不相同, 原题说法错误。\n\n故答案为: $\\times$ 。\n\n【点睛】本题考查整数四则混合运算的运算顺序:同级运算时,从左到右依次计算; 两级运算时,先算乘除, 后算加减。有括号时, 先算括号里面的。", "solution": "null", "level": "四年级", "question": "$63 \\div(9-2)$ 与 $63 \\div 9-2$ 的结果相同。( )", "options": [], "subject": "算术", "analysis": "解$\\times$\n\n【分析】根据整数四则混合运算的顺序,直接计算出结果,然后再进行判断即可。\n\n【详解】 $63 \\div(9-2)$\n\n$=63 \\div 7$\n\n$=9$\n\n$63 \\div 9-2$\n\n$=7-2$\n\n$=5$\n\n所以 $63 \\div(9-2)$ 与 $63 \\div 9-2$ 的结果不相同, 原题说法错误。\n\n故答案为: $\\times$ 。\n\n【点睛】本题考查整数四则混合运算的运算顺序:同级运算时,从左到右依次计算; 两级运算时,先算乘除, 后算加减。有括号时, 先算括号里面的。"} {"id": "27528", "image": [], "answer": "解$\\times$\n\n【分析】混合运算中,有加、减、乘、除的四则运算,先算乘除、再算加减; 有小括号先算小括号中的数,同级运算才是从左往右顺序计算。\n\n【详解】根据分析, 混合运算中, 不一定按照从左往右顺序计算。\n故答案为: $\\times$\n\n【点睛】本题主要考查的是混合运算的运算顺序。", "solution": "null", "level": "四年级", "question": "在混合运算中,一定是按照从左往右的顺序计算。( )", "options": [], "subject": "算术", "analysis": "解$\\times$\n\n【分析】混合运算中,有加、减、乘、除的四则运算,先算乘除、再算加减; 有小括号先算小括号中的数,同级运算才是从左往右顺序计算。\n\n【详解】根据分析, 混合运算中, 不一定按照从左往右顺序计算。\n故答案为: $\\times$\n\n【点睛】本题主要考查的是混合运算的运算顺序。"} {"id": "27530", "image": [], "answer": "解$14 ; 32 ; 3 ; 49$;\n\n$49 ; 35 ; 24 ; 90$\n\n【详解】略", "solution": "null", "level": "四年级", "question": "口算。\n\n$$\n\\begin{array}{llcc}\n30-2 \\times 8= & 56-4 \\times 6= & 51 \\div(9+8)= & 7 \\times 7 \\div 7 \\times 7= \\\\\n56 \\div 8 \\times 7= & 35-5 \\times 0= & 17+63 \\div 9= & 55+45-55+45=\n\\end{array}\n$$", "options": [], "subject": "算术", "analysis": "解$14 ; 32 ; 3 ; 49$;\n\n$49 ; 35 ; 24 ; 90$\n\n【详解】略"} {"id": "27531", "image": [], "answer": "解$7 ; 34780$;\n\n【分析】 $420 \\div(108-3 \\times 16)$ 此题先算乘法, 再算减法, 最后算除法。\n\n$940 \\times[135-( 196-98 )$ ]此题先算小括号里的减法, 再算中括号里的减法, 最后算乘法。\n\n【详解】 $420 \\div(108-3 \\times 16)$\n\n$=420 \\div(108-48)$\n\n$=420 \\div 60$\n\n$=7$\n\n$940 \\times[135-(196-98)]$\n\n$=940 \\times[135-98]$\n\n$=940 \\times 37$\n\n$=34780$", "solution": "null", "level": "四年级", "question": "脱式计算。\n\n$420 \\div(108-3 \\times 16) \\quad 940 \\times[135-(196-98)]$", "options": [], "subject": "算术", "analysis": "解$7 ; 34780$;\n\n【分析】 $420 \\div(108-3 \\times 16)$ 此题先算乘法, 再算减法, 最后算除法。\n\n$940 \\times[135-( 196-98 )$ ]此题先算小括号里的减法, 再算中括号里的减法, 最后算乘法。\n\n【详解】 $420 \\div(108-3 \\times 16)$\n\n$=420 \\div(108-48)$\n\n$=420 \\div 60$\n\n$=7$\n\n$940 \\times[135-(196-98)]$\n\n$=940 \\times[135-98]$\n\n$=940 \\times 37$\n\n$=34780$"} {"id": "27532", "image": [], "answer": "解2156\n\n【分析】计算顺序是先同时计算乘法和除法, 再计算减法, 列式为: $36 \\times 60-324 \\div 81=2156$ 。\n\n【详解】 $36 \\times 60-324 \\div 81$\n\n$=2160-4$\n\n$=2156$\n\n所以, 36 与 60 的积减去 81 除 324 , 差是 2156 。", "solution": "null", "level": "四年级", "question": "36 与 60 的积减去 81 除 324 , 差是多少?", "options": [], "subject": "算术", "analysis": "解2156\n\n【分析】计算顺序是先同时计算乘法和除法, 再计算减法, 列式为: $36 \\times 60-324 \\div 81=2156$ 。\n\n【详解】 $36 \\times 60-324 \\div 81$\n\n$=2160-4$\n\n$=2156$\n\n所以, 36 与 60 的积减去 81 除 324 , 差是 2156 。"} {"id": "27533", "image": [], "answer": "解30 箱\n\n【分析】今年收樱桃的千克数减已经售出的千克数, 等于剩下樱桃的千克数, 再除以每个箱子装的千克数即可解答。\n\n【详解】( $1600-850) \\div 25$\n\n$=750 \\div 25$\n\n$=30$ (箱)\n\n答: 能装 30 箱。\n\n【点睛】本题主要考查学生对整数混合运算知识的掌握。", "solution": "null", "level": "四年级", "question": "李叔叔家今年收樱桃 1600 千克, 已经售出了 850 千克。剩下的需要用箱子分装出售, 每个箱子装 25 千克, 能装多少箱?", "options": [], "subject": "算术", "analysis": "解30 箱\n\n【分析】今年收樱桃的千克数减已经售出的千克数, 等于剩下樱桃的千克数, 再除以每个箱子装的千克数即可解答。\n\n【详解】( $1600-850) \\div 25$\n\n$=750 \\div 25$\n\n$=30$ (箱)\n\n答: 能装 30 箱。\n\n【点睛】本题主要考查学生对整数混合运算知识的掌握。"} {"id": "27534", "image": [], "answer": "解5600 元\n【分析】根据题意:总共的钱数 $=$ (衣服的单价十鞋子的单价)×数量,据此解答即可。\n\n【详解】 $(168+32) \\times 28$\n\n$=200 \\times 28$\n\n$=5600 ($ 元)\n\n答: 一共需要准备 5600 元。\n\n【点睛】先求出每人的需要的钱数再乘人数是解决本题的关键。", "solution": "null", "level": "四年级", "question": "学校为参加“啦啦操”比赛的 28 位队员购买了服装, 每套衣服需要 168 元, 每双鞋子需要 32 元,一共需要准备多少钱?", "options": [], "subject": "算术", "analysis": "解5600 元\n【分析】根据题意:总共的钱数 $=$ (衣服的单价十鞋子的单价)×数量,据此解答即可。\n\n【详解】 $(168+32) \\times 28$\n\n$=200 \\times 28$\n\n$=5600 ($ 元)\n\n答: 一共需要准备 5600 元。\n\n【点睛】先求出每人的需要的钱数再乘人数是解决本题的关键。"} {"id": "27535", "image": ["13510.jpg", "13511.jpg"], "answer": "解2730 千米\n\n【分析】先根据公式: 路程 $=$ 速度 $\\times$ 时间, 算出动车已经行驶的路程, 总路程再减去已经行驶的路程,就是动车距离乙地的距离。\n\n【详解】 $6730-(250 \\times 16)$\n\n$=6730-4000$\n\n$=2730$ (千米)\n\n答:这时动车距离乙地还有 2730 千米。\n\n【点睛】熟记行程问题的公式是解决本题的关键。", "solution": "null", "level": "四年级", "question": "甲乙两地相距 6730 千米, 一列动车从甲地开往乙地。\n\n\n\n动车行驶了 16 小时, 这时动车距离乙地还有多少千米?\n\n\n\n动车每小时行驶 250 千米。", "options": [], "subject": "算术", "analysis": "解2730 千米\n\n【分析】先根据公式: 路程 $=$ 速度 $\\times$ 时间, 算出动车已经行驶的路程, 总路程再减去已经行驶的路程,就是动车距离乙地的距离。\n\n【详解】 $6730-(250 \\times 16)$\n\n$=6730-4000$\n\n$=2730$ (千米)\n\n答:这时动车距离乙地还有 2730 千米。\n\n【点睛】熟记行程问题的公式是解决本题的关键。"} {"id": "27536", "image": [], "answer": "解200 米\n\n【分析】先根据工作总量 $=$ 工作效率 $\\times$ 工作时间, 求出这条公路的总长度, 再求出实际修完公路需要的时间, 最后根据工作效率 $=$ 工作总量 $\\div$ 工作时间即可解答。\n\n【详解】 $160 \\times 50 \\div(50-10)$\n\n$=8000 \\div 40$\n\n$=200$ (米)\n\n答: 每天要修 200 米。\n\n【点睛】本题主要考查学生依据工作时间, 工作效率以及工作总量之间的数量关系解决问题的能力。", "solution": "null", "level": "四年级", "question": "修一条公路, 原计划每天修 160 米, 50 天完成。如果要提前 10 天完成,每天要修多少米?", "options": [], "subject": "算术", "analysis": "解200 米\n\n【分析】先根据工作总量 $=$ 工作效率 $\\times$ 工作时间, 求出这条公路的总长度, 再求出实际修完公路需要的时间, 最后根据工作效率 $=$ 工作总量 $\\div$ 工作时间即可解答。\n\n【详解】 $160 \\times 50 \\div(50-10)$\n\n$=8000 \\div 40$\n\n$=200$ (米)\n\n答: 每天要修 200 米。\n\n【点睛】本题主要考查学生依据工作时间, 工作效率以及工作总量之间的数量关系解决问题的能力。"} {"id": "27537", "image": [], "answer": "解90 元\n\n【分析】用花费的总钱数除以办公桌椅的套数, 求出平均每套办公桌椅的价钱, 再减去一张桌子的价钱, 求出两把椅子的价钱。用这两把椅子的价钱除以 2 , 即可求出每把椅子的价钱。\n\n【详解】(8500 $\\div 25-160 ) \\div 2$\n\n$=(340-160) \\div 2$\n\n$=180 \\div 2$\n\n$=90$ (元)\n\n答: 每把椅子 90 元。\n\n【点睛】本题考查经济问题, 关键是根据单价 $=$ 总价 $\\div$ 数量, 依此求出每套办公桌椅以及每把椅子的\n价钱。", "solution": "null", "level": "四年级", "question": "学校新头了 25 套办公桌椅(一套办公桌椅含一张桌子和两把椅子), 共花费 8500 元, 每张桌子 160 元,每把椅子多少元?(列综合算式解答)", "options": [], "subject": "算术", "analysis": "解90 元\n\n【分析】用花费的总钱数除以办公桌椅的套数, 求出平均每套办公桌椅的价钱, 再减去一张桌子的价钱, 求出两把椅子的价钱。用这两把椅子的价钱除以 2 , 即可求出每把椅子的价钱。\n\n【详解】(8500 $\\div 25-160 ) \\div 2$\n\n$=(340-160) \\div 2$\n\n$=180 \\div 2$\n\n$=90$ (元)\n\n答: 每把椅子 90 元。\n\n【点睛】本题考查经济问题, 关键是根据单价 $=$ 总价 $\\div$ 数量, 依此求出每套办公桌椅以及每把椅子的\n价钱。"} {"id": "27554", "image": ["13513.jpg", "13513.jpg"], "answer": "解【解答】解: 当 $a$ 与 $b$ 的和是 15 时, $15-a=b$;\n\n", "solution": "null", "level": "四年级", "question": "(2022 秋 ・梁子湖区期末)当 $a$ 与 $b$ 的和是 15 时, $15-a=b$. $\\qquad$", "options": [], "subject": "算术", "analysis": "解【解答】解: 当 $a$ 与 $b$ 的和是 15 时, $15-a=b$;\n\n"} {"id": "27555", "image": [], "answer": "【解答】解:如果 $\\square+\\triangle=\\bigcirc$, 那么 $\\triangle=\\bigcirc$ - $\\square$ 。原题说法错误。\n\n故答案为: $\\times$ 。", "solution": "null", "level": "四年级", "question": "(2022 春・瑞昌市期中)如果 $\\square+\\triangle=O$, 那么 $\\triangle=O \\times \\square$ 。", "options": [], "subject": "算术", "analysis": "【解答】解:如果 $\\square+\\triangle=\\bigcirc$, 那么 $\\triangle=\\bigcirc$ - $\\square$ 。原题说法错误。\n\n故答案为: $\\times$ 。"} {"id": "27556", "image": [], "answer": "【解答】解: $534 \\times 0+7$\n\n$=0+7$\n\n$=7$\n\n$7 \\neq 0$, 所以原题计算错误。\n\n故答案为: $\\times$ 。", "solution": "null", "level": "四年级", "question": "(2021 秋・临潼区期末) $534 \\times 0+7=0$ 。", "options": [], "subject": "算术", "analysis": "【解答】解: $534 \\times 0+7$\n\n$=0+7$\n\n$=7$\n\n$7 \\neq 0$, 所以原题计算错误。\n\n故答案为: $\\times$ 。"} {"id": "27558", "image": [], "answer": "【解答】解:使 $9 \\div 3 \\times 5-2=1$ 成立的正确等式是: $9 \\div[3 \\times(5-2)]=1$;\n\n所以, 原题说法正确。\n\n故答案为: $\\sqrt{ }$ 。", "solution": "null", "level": "四年级", "question": "(2022 秋・榕城区期中)添上括号能使 $9 \\div 3 \\times 5-2=1$ 成立。", "options": [], "subject": "算术", "analysis": "【解答】解:使 $9 \\div 3 \\times 5-2=1$ 成立的正确等式是: $9 \\div[3 \\times(5-2)]=1$;\n\n所以, 原题说法正确。\n\n故答案为: $\\sqrt{ }$ 。"} {"id": "27559", "image": [], "answer": "解【解答】解: 把 $28 \\div 4=7,25+7=32,32 \\times 5=160$, 列成一个综合算式是: $(25+28 \\div 4 ) \\times 5=$ 160\n\n所以原题解答错误。\n\n故答案为: $\\times$ 。", "solution": "null", "level": "四年级", "question": "(2022 春・黄梅县期末)把 $28 \\div 4=7,25+7=32,32 \\times 5=160$, 列成一个综合算式是: $25+28 \\div 4$ $\\times 5$ 。", "options": [], "subject": "算术", "analysis": "解【解答】解: 把 $28 \\div 4=7,25+7=32,32 \\times 5=160$, 列成一个综合算式是: $(25+28 \\div 4 ) \\times 5=$ 160\n\n所以原题解答错误。\n\n故答案为: $\\times$ 。"} {"id": "27560", "image": [], "answer": "解【解答】解:\n\n$$\n\\begin{array}{llll}\n64 \\div 8+9=17 & 700 \\times 20=14000 & 2100 \\div 3=700 & 324 \\times 8=2592 \\\\\n62 \\times 18 \\approx 1200 & 201 \\times 49 \\approx 10000 & 279 \\times 39 \\approx 11200 & 99 \\times 73 \\approx 7300\n\\end{array}\n$$", "solution": "null", "level": "四年级", "question": "(2022 秋・红谷滩区期末)直接写得数。\n$64 \\div 8+9=$\n$700 \\times 20=$\n$2100 \\div 3=$\n$324 \\times 8=$\n$62 \\times 18 \\approx$\n$201 \\times 49 \\approx$\n$279 \\times 39 \\approx$\n$99 \\times 73 \\approx$", "options": [], "subject": "算术", "analysis": "解【解答】解:\n\n$$\n\\begin{array}{llll}\n64 \\div 8+9=17 & 700 \\times 20=14000 & 2100 \\div 3=700 & 324 \\times 8=2592 \\\\\n62 \\times 18 \\approx 1200 & 201 \\times 49 \\approx 10000 & 279 \\times 39 \\approx 11200 & 99 \\times 73 \\approx 7300\n\\end{array}\n$$"} {"id": "27561", "image": [], "answer": "解【解答】解: (1) $837 \\div 27+65 \\times 4$\n\n$=31+260$\n\n$=291$\n\n(2) $300-225 \\div(43-38)$\n\n$=300-225 \\div 5$\n\n$=300-45$\n$=255$\n\n(3) $420 \\div[(205-198) \\times 4]$\n\n$=420 \\div[7 \\times 4]$\n\n$=420 \\div 28$\n\n$=15$", "solution": "null", "level": "四年级", "question": "(2022 秋・东海县期末)计算下面各题。\n$837 \\div 27+65 \\times 4$\n$300-225 \\div(43-38)$\n$420 \\div[(205-198) \\times 4]$", "options": [], "subject": "算术", "analysis": "解【解答】解: (1) $837 \\div 27+65 \\times 4$\n\n$=31+260$\n\n$=291$\n\n(2) $300-225 \\div(43-38)$\n\n$=300-225 \\div 5$\n\n$=300-45$\n$=255$\n\n(3) $420 \\div[(205-198) \\times 4]$\n\n$=420 \\div[7 \\times 4]$\n\n$=420 \\div 28$\n\n$=15$"} {"id": "27562", "image": [], "answer": "解【解答】解: $(45+35) \\times 32$\n\n$=80 \\times 32$\n\n$=2560 ($ 元)\n\n答: 一共花了 2560 元钱。", "solution": "null", "level": "四年级", "question": "(2021 秋・渭城区期末)为庆祝元旦晚会, 学校准备了文艺演出, 购进 32 套服装用于节目表演 (一件上衣搭配一条裙子),每件上衣 45 元,每条裙子 35 元,一共花了多少钱?", "options": [], "subject": "算术", "analysis": "解【解答】解: $(45+35) \\times 32$\n\n$=80 \\times 32$\n\n$=2560 ($ 元)\n\n答: 一共花了 2560 元钱。"} {"id": "27563", "image": [], "answer": "【解答】解: $260-45 \\times 5$\n\n$=260-225$\n\n$=35$ (名)\n\n答: 第 6 辆汽车要坐 35 名学生。", "solution": "null", "level": "四年级", "question": "(2021 秋・渭城区期末)2021 年是红军长征胜利 85 周年, 逸夫小学组织三年级 260 名学生去敬老院献爱心, 一共租了 6 辆车, 前 5 辆车每辆各坐 45 名学生, 第 6 辆汽车要坐多少名学生?", "options": [], "subject": "算术", "analysis": "【解答】解: $260-45 \\times 5$\n\n$=260-225$\n\n$=35$ (名)\n\n答: 第 6 辆汽车要坐 35 名学生。"} {"id": "27564", "image": [], "answer": "解【解答】解: $3 \\times 256-78$\n\n$=768-78$\n\n$=690$ (元)\n\n答:明明家 7 月份的电费是 690 元。", "solution": "null", "level": "四年级", "question": "(2021 秋 ・慈溪市期末)丁丁家 7 月份的电费是 256 元,明明家 7 月份的电费是丁丁家的 3 倍少 78 元,明明家 7 月份的电费是多少元?", "options": [], "subject": "算术", "analysis": "解【解答】解: $3 \\times 256-78$\n\n$=768-78$\n\n$=690$ (元)\n\n答:明明家 7 月份的电费是 690 元。"} {"id": "27565", "image": [], "answer": "【解答】解: $900-220 \\times 4$\n\n$=900-880$\n\n$=20$ (元)\n\n答: 应找回 20 元。", "solution": "null", "level": "四年级", "question": "(2021 秋・互助县期末)九寨沟位于四川省阿坝藏族羌族自治州,有“童话世界”之美誉。旺季时, 九寨沟门票为每张 220 元。刘叔叔一家四口在旺季到九寨沟旅游, 付了 900 元,应找回多少元?", "options": [], "subject": "算术", "analysis": "【解答】解: $900-220 \\times 4$\n\n$=900-880$\n\n$=20$ (元)\n\n答: 应找回 20 元。"} {"id": "27566", "image": [], "answer": "解【解答】解: $265 \\times 11925$ (元)\n\n答:今天售出的美的微波炉一共卖了 11925 元。", "solution": "null", "level": "四年级", "question": "(2021 秋・武安市期末)原价每台 400 元的美的微波炉,现价 265 元一台,今天售出的美的微波炉 45 台,今天售出的美的微波炉一共卖了多少元?", "options": [], "subject": "算术", "analysis": "解【解答】解: $265 \\times 11925$ (元)\n\n答:今天售出的美的微波炉一共卖了 11925 元。"} {"id": "27580", "image": [], "answer": "【解答】解: $569-128+72$\n$=441+72$\n\n$=513$\n\n$569-(128+72)$\n\n$=569-200$\n\n$=369$\n\n$513 \\neq 369$\n\n所以原题说法错误。\n\n故答案为: $\\times$ 。", "solution": "null", "level": "四年级", "question": "(2022 秋 $\\cdot$ 安岳县期末) $569-128+72=569$ - (128+72)", "options": [], "subject": "算术", "analysis": "【解答】解: $569-128+72$\n$=441+72$\n\n$=513$\n\n$569-(128+72)$\n\n$=569-200$\n\n$=369$\n\n$513 \\neq 369$\n\n所以原题说法错误。\n\n故答案为: $\\times$ 。"} {"id": "27581", "image": [], "answer": "解【解答】解: $6 \\times 7 \\times 25$\n\n$=42 \\times 25$\n\n$=1050$\n\n所以原题计算正确。\n\n故答案为: $\\sqrt{ }$ 。", "solution": "null", "level": "四年级", "question": "(2020 秋・乐东县期中) $6 \\times 7 \\times 25=42 \\times 25$ 。", "options": [], "subject": "算术", "analysis": "解【解答】解: $6 \\times 7 \\times 25$\n\n$=42 \\times 25$\n\n$=1050$\n\n所以原题计算正确。\n\n故答案为: $\\sqrt{ }$ 。"} {"id": "27582", "image": [], "answer": "解【解答】解: $56 \\times 99$\n\n$=56 \\times(100-1)$\n\n$=56 \\times 100-1 \\times 56$\n\n$=5600-56$\n\n$=5544$.\n\n故答案为: $\\times$.", "solution": "null", "level": "四年级", "question": "(2022 秋・宁陕县期末) $56 \\times 99=56 \\times 100-1$", "options": [], "subject": "算术", "analysis": "解【解答】解: $56 \\times 99$\n\n$=56 \\times(100-1)$\n\n$=56 \\times 100-1 \\times 56$\n\n$=5600-56$\n\n$=5544$.\n\n故答案为: $\\times$."} {"id": "27583", "image": [], "answer": "解【解答】解: $18 \\times 12 \\times 5=18 \\times(12 \\times 5)$, 是运用了乘法结合律, 而不是乘法交换律。\n\n原题说法错误。\n\n故答案为: $\\times$ 。", "solution": "null", "level": "四年级", "question": "(2021 秋・绥德县期末) $18 \\times 12 \\times 5=18 \\times(12 \\times 5)$, 运用了乘法交换律。", "options": [], "subject": "算术", "analysis": "解【解答】解: $18 \\times 12 \\times 5=18 \\times(12 \\times 5)$, 是运用了乘法结合律, 而不是乘法交换律。\n\n原题说法错误。\n\n故答案为: $\\times$ 。"} {"id": "27584", "image": [], "answer": "【解答】解: $175 \\div 25$\n\n$=(175 \\times 4) \\div(25 \\times 4)$\n\n$=700 \\div 100$\n\n$=7$\n\n运用商不变的性质进行简算, 所以原题说法错误。\n\n故答案为: $\\times$ 。", "solution": "null", "level": "四年级", "question": "(2021 秋・茂南区期末) $175 \\div 25=(175 \\times 4) \\div(25 \\times 4)=7$, 这是运用了乘法分配律来计算的。", "options": [], "subject": "算术", "analysis": "【解答】解: $175 \\div 25$\n\n$=(175 \\times 4) \\div(25 \\times 4)$\n\n$=700 \\div 100$\n\n$=7$\n\n运用商不变的性质进行简算, 所以原题说法错误。\n\n故答案为: $\\times$ 。"} {"id": "27586", "image": [], "answer": "解【解答】解:\n\n| $130 \\times 5=650$ | $2 \\times 380=760$ | $540 \\div 3=180$ | $240 \\div 40=6$ |\n| :--- | :--- | :--- | :--- |\n| $7+13 \\times 6=85$ | $90 \\times 9 \\times 0=0$ | $39 \\times 41 \\approx 1600$ | $629 \\div 71 \\approx 6$ |", "solution": "null", "level": "四年级", "question": "(2022 秋・五莲县期末)直接写得数。\n$130 \\times 5=$\n$2 \\times 380=$\n$540 \\div 3=$\n$240 \\div 40=$\n$7+13 \\times 6=$\n$90 \\times 9 \\times 0=$\n$39 \\times 41 \\approx$\n$629 \\div 71 \\approx$", "options": [], "subject": "算术", "analysis": "解【解答】解:\n\n| $130 \\times 5=650$ | $2 \\times 380=760$ | $540 \\div 3=180$ | $240 \\div 40=6$ |\n| :--- | :--- | :--- | :--- |\n| $7+13 \\times 6=85$ | $90 \\times 9 \\times 0=0$ | $39 \\times 41 \\approx 1600$ | $629 \\div 71 \\approx 6$ |"} {"id": "27587", "image": [], "answer": "解【解答】解:(1)456-298\n\n$$\n\\begin{aligned}\n& =456-(300-2) \\\\\n& =456-300+2 \\\\\n& =156+2 \\\\\n& =158\n\\end{aligned}\n$$\n\n(2) $679+204$\n\n$=679+200+4$\n\n$=879+4$\n\n$=883$\n\n(3) $423-175+277-225$\n\n$=423+277-225-175$\n\n$=423+277-(225+175)$\n\n$=700-400$\n\n$=300$\n\n(4) $873-48-152$\n\n$=873-(48+152)$\n\n$=873-200$\n\n$=673$\n\n(5) $684-(432+184)$\n\n$=684-184-432$\n\n$=500-432$\n\n$=68$\n\n(6) $348-158+52$\n$=348+52-158$\n\n$=400-158$\n\n$=242$", "solution": "null", "level": "四年级", "question": "(2022 秋・沈丘县期末)用简便方法计算。\n$456-298$\n$679+204$\n$423-175+277-225$\n$873-48-152$\n$684-(432+184)$\n$348-158+52$", "options": [], "subject": "算术", "analysis": "解【解答】解:(1)456-298\n\n$$\n\\begin{aligned}\n& =456-(300-2) \\\\\n& =456-300+2 \\\\\n& =156+2 \\\\\n& =158\n\\end{aligned}\n$$\n\n(2) $679+204$\n\n$=679+200+4$\n\n$=879+4$\n\n$=883$\n\n(3) $423-175+277-225$\n\n$=423+277-225-175$\n\n$=423+277-(225+175)$\n\n$=700-400$\n\n$=300$\n\n(4) $873-48-152$\n\n$=873-(48+152)$\n\n$=873-200$\n\n$=673$\n\n(5) $684-(432+184)$\n\n$=684-184-432$\n\n$=500-432$\n\n$=68$\n\n(6) $348-158+52$\n$=348+52-158$\n\n$=400-158$\n\n$=242$"} {"id": "27588", "image": [], "answer": "解【解答】解: (1) $256 \\times 101-256$\n\n$=256 \\times(101-1)$\n\n$=256 \\times 100$\n\n$=25600$\n\n(2) $87 \\times 102$\n\n$=87 \\times(100+2)$\n\n$=87 \\times 100+87 \\times 2$\n\n$=8700+174$\n\n$=8874$\n\n(3) $125 \\times 88$\n\n$=125 \\times(8 \\times 11)$\n\n$=(125 \\times 8) \\times 11$\n\n$=1000 \\times 11$\n\n$=11000$\n\n(4) $630 \\div 18 \\div 5$\n\n$=630 \\div(18 \\times 5)$\n\n$=630 \\div 90$\n\n$=7$\n\n(5) $32 \\times 25 \\times 125$\n\n$=(8 \\times 4) \\times 25 \\times 125$\n\n$=(8 \\times 125) \\times(4 \\times 25)$\n\n$=1000 \\times 100$\n\n$=100000$\n\n(6) $943-258+47-142$\n\n$=(943+47)-(258+142)$\n\n$=990-400$\n\n$=590$", "solution": "null", "level": "四年级", "question": "(2022 秋・荣成市期中)简算。\n$256 \\times 101-256$\n$87 \\times 102$\n$125 \\times 88$\n$630 \\div 18 \\div 5$\n$32 \\times 25 \\times 125$\n$943-258+47-142$", "options": [], "subject": "算术", "analysis": "解【解答】解: (1) $256 \\times 101-256$\n\n$=256 \\times(101-1)$\n\n$=256 \\times 100$\n\n$=25600$\n\n(2) $87 \\times 102$\n\n$=87 \\times(100+2)$\n\n$=87 \\times 100+87 \\times 2$\n\n$=8700+174$\n\n$=8874$\n\n(3) $125 \\times 88$\n\n$=125 \\times(8 \\times 11)$\n\n$=(125 \\times 8) \\times 11$\n\n$=1000 \\times 11$\n\n$=11000$\n\n(4) $630 \\div 18 \\div 5$\n\n$=630 \\div(18 \\times 5)$\n\n$=630 \\div 90$\n\n$=7$\n\n(5) $32 \\times 25 \\times 125$\n\n$=(8 \\times 4) \\times 25 \\times 125$\n\n$=(8 \\times 125) \\times(4 \\times 25)$\n\n$=1000 \\times 100$\n\n$=100000$\n\n(6) $943-258+47-142$\n\n$=(943+47)-(258+142)$\n\n$=990-400$\n\n$=590$"} {"id": "27590", "image": [], "answer": "解【解答】解: $(45+55) \\times 8$\n\n$=100 \\times 8$\n\n$=800$ (元)\n\n答: 这一天一共卖了 800 元。", "solution": "null", "level": "四年级", "question": "(2022 春・怀化期末)商店上午卖出录音磁带 45 盒, 下午卖出磁带 55 盒。每盒磁带卖 8 元. 这一天一共卖了多少元?", "options": [], "subject": "算术", "analysis": "解【解答】解: $(45+55) \\times 8$\n\n$=100 \\times 8$\n\n$=800$ (元)\n\n答: 这一天一共卖了 800 元。"} {"id": "27591", "image": [], "answer": "【解答】解: $25 \\times 7 \\times 4$\n\n$=(25 \\times 4) \\times 7$\n\n$=100 \\times 7$\n\n$=700$ (张)\n\n答: 学校一共需要 700 张课桌。", "solution": "null", "level": "四年级", "question": "(2021 秋 - 罗湖区期中)洪湖小学新建了一幢四层的教学楼, 每层有 7 个教室, 每个教室有 25 张课桌, 学校一共需要多少张课桌? (用简便方法计算)", "options": [], "subject": "算术", "analysis": "【解答】解: $25 \\times 7 \\times 4$\n\n$=(25 \\times 4) \\times 7$\n\n$=100 \\times 7$\n\n$=700$ (张)\n\n答: 学校一共需要 700 张课桌。"} {"id": "27604", "image": [], "answer": "解16; 28", "solution": "null", "level": "四年级", "question": "根据 $28 \\times 16=448$, 直接写出下面两道题的得数。\n\n$448 \\div 28=$ $\\qquad$ $448 \\div 16=$ $\\qquad$。", "options": [], "subject": "算术", "analysis": "解16; 28"} {"id": "27605", "image": [], "answer": "解 $(344+56) \\times(72 \\div 4)$", "solution": "null", "level": "四年级", "question": "把 $344+56 \\times 72 \\div 4$ 的运算顺序改成先求和与商, 再求积, 则原式变为 $\\qquad$ -", "options": [], "subject": "算术", "analysis": "解 $(344+56) \\times(72 \\div 4)$"} {"id": "27606", "image": [], "answer": "解60", "solution": "null", "level": "四年级", "question": "东东在计算 $300-\\square \\times 3$ 时先算了减法, 结果得了 660 , 那么这个算式的正确答案应该是 $\\qquad$。", "options": [], "subject": "算术", "analysis": "解60"} {"id": "27607", "image": [], "answer": "解 252", "solution": "null", "level": "四年级", "question": "两个数的差是 352 , 如果被减数减少 36 , 减数增加 64 , 差是", "options": [], "subject": "算术", "analysis": "解 252"} {"id": "27608", "image": [], "answer": "解 $422+12 \\times(45-18)=746$", "solution": "null", "level": "四年级", "question": "根据 $45-18=27,12 \\times 27=324,422+324=746$ 写出一个综合算式是 $\\qquad$。", "options": [], "subject": "算术", "analysis": "解 $422+12 \\times(45-18)=746$"} {"id": "27609", "image": [], "answer": " 5865", "solution": "null", "level": "四年级", "question": "淘气用计算器计算 $345 \\times$ 口时, 将“ $\\times$ ”, 错按成“+”, 算出结果是 362 , 正确的得数应该是 $\\qquad$", "options": [], "subject": "算术", "analysis": " 5865"} {"id": "27611", "image": [], "answer": "解 $100-28=72$\n\n$$\n8 \\times 25=200\n$$\n\n$0.45 \\times 10=4.5$\n\n$75 \\div 1000=0.075$\n\n$480 \\div 80=6$\n\n$225+75=300$\n\n$354-99=255$\n\n$101 \\times 25=2525$\n\n$28 \\div 7 \\times 5=20$\n\n$125 \\times 4 \\times 2=1000$\n\n$64 \\div 8-8=0$\n\n$(24+16) \\quad \\times 6=240$", "solution": "null", "level": "四年级", "question": "直接写得数。\n\n| $100-28=$ | $8 \\times 25=$ | $0.45 \\times 10=$ | $75 \\div 1000=$ |\n| :--- | :--- | :--- | :--- |\n| $480 \\div 80=$ | $225+75=$ | $354-99=$ | $101 \\times 25=$ |\n| $28 \\div 7 \\times 5=$ | $125 \\times 4 \\times 2=$ | $64 \\div 8-8=$ | $(24+16) \\times 6=$ |", "options": [], "subject": "算术", "analysis": "解 $100-28=72$\n\n$$\n8 \\times 25=200\n$$\n\n$0.45 \\times 10=4.5$\n\n$75 \\div 1000=0.075$\n\n$480 \\div 80=6$\n\n$225+75=300$\n\n$354-99=255$\n\n$101 \\times 25=2525$\n\n$28 \\div 7 \\times 5=20$\n\n$125 \\times 4 \\times 2=1000$\n\n$64 \\div 8-8=0$\n\n$(24+16) \\quad \\times 6=240$"} {"id": "27612", "image": [], "answer": "解 $181800,915,25$", "solution": "null", "level": "四年级", "question": "列式计算.\n\n(1) 606 个 301 减去 606, 差是多少?\n\n(2) 12 的 3 倍加上 25 的和, 再乘 15 , 积是多少?\n\n(3) 1060 与 160 的差除以 178 和 142 的差,商是多少?", "options": [], "subject": "算术", "analysis": "解 $181800,915,25$"} {"id": "27613", "image": ["13516.jpg", "13517.jpg", "13518.jpg", "13519.jpg", "13516.jpg", "13517.jpg", "13518.jpg", "13519.jpg"], "answer": "(1)解: $720 \\div 45 \\times(798-616)$\n\n$=720 \\div 45 \\times 182$\n\n$=16 \\times 182$\n\n$=2912$\n\n(2)解: $(15 \\times 40-364) \\div 4$\n\n$=(600-364) \\div 4$\n\n```\n=236\\div4\n=59\n\n```\n\n\n\n```\n=24\\times[ 19-12]\n=24\\times7\n\n```\n\n\n\n(4) 解: (1010-960)\\times(65+15)\n\n=50\\times80\n\n\n\n(5) 解: 8000-(59+66) \\times64\n\n\n\n=8000-8000\n\n=0\n\n(6)解: 450\\div[ (15+10)\\times3]\n\n=450\\div[ 25\\times3]\n\n=450\\div75\n\n=6", "solution": "null", "level": "四年级", "question": "脱式计算。\n(1) $720 \\div 45 \\times(798-616)$\n(2) $(15 \\times 40-364) \\div 4$\n(3) $24 \\times[19-(2 \\times 6)]$\n\n(4) $(1010-960) \\times(65+15)$ (5) $8000-(59+66) \\times 64 ( 6) 450 \\div[(15+10) \\times 3]$", "options": [], "subject": "算术", "analysis": "(1)解: $720 \\div 45 \\times(798-616)$\n\n$=720 \\div 45 \\times 182$\n\n$=16 \\times 182$\n\n$=2912$\n\n(2)解: $(15 \\times 40-364) \\div 4$\n\n$=(600-364) \\div 4$\n\n```\n=236\\div4\n=59\n\n```\n\n\n\n```\n=24\\times[ 19-12]\n=24\\times7\n\n```\n\n\n\n(4) 解: (1010-960)\\times(65+15)\n\n=50\\times80\n\n\n\n(5) 解: 8000-(59+66) \\times64\n\n\n\n=8000-8000\n\n=0\n\n(6)解: 450\\div[ (15+10)\\times3]\n\n=450\\div[ 25\\times3]\n\n=450\\div75\n\n=6"} {"id": "27614", "image": ["13520.jpg", "13521.jpg", "13520.jpg", "13521.jpg"], "answer": "解解:方案一:39+1=40(名)\n```\n40\\div(4+1)\n=40\\div5\n=8(组)\n8\\times4\\times150\n=32\\times150\n=4800(元)\n方案二: (150-24)\\times40\n\n```\n\n\n\n```\n=5040(元)\n\n```\n\n\n\n```\n6000\\div300=20(组)\n6000-20\\times50\n=6000-1000\n```\n\n$=5000$ (元)\n\n5040 元 $>5000$ 元 $>4800$ 元\n\n答:方案一最省钱。", "solution": "null", "level": "四年级", "question": "张经理和 39 名员工去大峡谷游玩, 门票价格为 150 元/张, 现在有三种优惠方案, 他们根据哪个方案买最省钱?\n\n方案一: 每买 4 张门票赠 1 张\n\n方案二:每张优惠 24 元\n\n方案三: 每满 300 元, 返还 50 元", "options": [], "subject": "算术", "analysis": "解解:方案一:39+1=40(名)\n```\n40\\div(4+1)\n=40\\div5\n=8(组)\n8\\times4\\times150\n=32\\times150\n=4800(元)\n方案二: (150-24)\\times40\n\n```\n\n\n\n```\n=5040(元)\n\n```\n\n\n\n```\n6000\\div300=20(组)\n6000-20\\times50\n=6000-1000\n```\n\n$=5000$ (元)\n\n5040 元 $>5000$ 元 $>4800$ 元\n\n答:方案一最省钱。"} {"id": "27615", "image": [], "answer": "解解: $12+388=400$ (人)\n\n$400 \\div 45=8$ (辆) $\\cdots \\cdots . \\cdots 40$ (人)\n\n$40 \\div 20=2$ (辆)\n\n租 8 辆大巴车和 2 辆中巴车费用为:\n\n$1800 \\times 8+1000 \\times 2$\n\n$=14400+200$\n\n$=16400$ (元)\n\n租 9 辆大巴车租金为: $1800 \\times 9=16200$ (元)\n\n答: 租 9 辆大巴车最省钱, 需要 16200 元。", "solution": "null", "level": "四年级", "question": "四年级老师 12 人, 学生 388 人, 一起去春游, 大巴车每辆租金 1800 元, 限乘 45 人, 中巴车每辆 1000 元, 限乘 20 人, 怎样租车最省钱?", "options": [], "subject": "算术", "analysis": "解解: $12+388=400$ (人)\n\n$400 \\div 45=8$ (辆) $\\cdots \\cdots . \\cdots 40$ (人)\n\n$40 \\div 20=2$ (辆)\n\n租 8 辆大巴车和 2 辆中巴车费用为:\n\n$1800 \\times 8+1000 \\times 2$\n\n$=14400+200$\n\n$=16400$ (元)\n\n租 9 辆大巴车租金为: $1800 \\times 9=16200$ (元)\n\n答: 租 9 辆大巴车最省钱, 需要 16200 元。"} {"id": "27617", "image": [], "answer": "解(1) 解:一班: $120 \\times 38=4560$ (元)\n\n二班: $120 \\times 40=4800$ (元)\n\n三班: $108 \\times 44=4752$ (元)\n\n答: 分开买, 一班需要 4560 元, 二班需要 4800 元, 三班需要 4752 元。\n\n(2)解: $38+40+44=122$ (人)\n\n$$\n90 \\times 122=10980 ( \\text { 元 })\n$$\n\n$$\n\\begin{aligned}\n& (120-90) \\times 38 \\\\\n= & 30 \\times 38 \\\\\n= & 1140 \\text { (元) } \\\\\n& (120-90) \\times 40 \\\\\n= & 30 \\times 40 \\\\\n= & 1200 \\text { (元) } \\\\\n& (108-90) \\times 44\n\\end{aligned}\n$$\n\n$=18 \\times 44$\n\n$=792$ (元)\n\n$1200>1140>792$\n\n答:三个班一起购票最少要用 10980 元,二班优惠最大,优惠的钱数最多。", "solution": "null", "level": "四年级", "question": "游乐园门票价格规定如表:\n\n| 购票人数 | $1 \\sim 40$ | $41 \\sim 80$ | 80 以上 |\n| :--- | :--- | :--- | :--- |\n| 票价/(元/人) | 120 | 108 | 90 |\n\n红星小学四年级同学去游乐园春游, 一班有 38 人, 二班有 40 人, 三班有 44 人。\n\n(1)分开买,三个班各需要多少元?\n\n(2)三个班一起购票最少要用多少元?哪个班得到的优惠最大?说说你的理由。", "options": [], "subject": "算术", "analysis": "解(1) 解:一班: $120 \\times 38=4560$ (元)\n\n二班: $120 \\times 40=4800$ (元)\n\n三班: $108 \\times 44=4752$ (元)\n\n答: 分开买, 一班需要 4560 元, 二班需要 4800 元, 三班需要 4752 元。\n\n(2)解: $38+40+44=122$ (人)\n\n$$\n90 \\times 122=10980 ( \\text { 元 })\n$$\n\n$$\n\\begin{aligned}\n& (120-90) \\times 38 \\\\\n= & 30 \\times 38 \\\\\n= & 1140 \\text { (元) } \\\\\n& (120-90) \\times 40 \\\\\n= & 30 \\times 40 \\\\\n= & 1200 \\text { (元) } \\\\\n& (108-90) \\times 44\n\\end{aligned}\n$$\n\n$=18 \\times 44$\n\n$=792$ (元)\n\n$1200>1140>792$\n\n答:三个班一起购票最少要用 10980 元,二班优惠最大,优惠的钱数最多。"} {"id": "27633", "image": [], "answer": "解$\\sqrt{ }$\n\n【详解】在没有余数的除法里, 被除数 $=$ 除数 $\\times$ 商, 所以被除数 $(0$ 除外 $)$ 除以除数与商的积, 商是 1 。原题说法正确。\n\n故答案为: $\\sqrt{ }$ 。", "solution": "null", "level": "四年级", "question": "在没有余数的除法里, 被除数(0 除外)除以除数与商的积, 商是 1 。()", "options": [], "subject": "算术", "analysis": "解$\\sqrt{ }$\n\n【详解】在没有余数的除法里, 被除数 $=$ 除数 $\\times$ 商, 所以被除数 $(0$ 除外 $)$ 除以除数与商的积, 商是 1 。原题说法正确。\n\n故答案为: $\\sqrt{ }$ 。"} {"id": "27634", "image": [], "answer": "解$\\sqrt{ }$\n\n【详解】按照先算括号里面的除法,再算括号外的乘法的计算顺序求出算式的结果为 160 ; 去掉括号后算式是 $32 \\times 20 \\div 4$, 按照从左到右的顺序计算出算式的结果为 160 , 两次计算的结果相同, 据此解答。\n\n【解答】带上括号时: $32 \\times(20 \\div 4)$\n\n$=32 \\times 5$\n\n$=160$\n\n不带括号时: $32 \\times 20 \\div 4$\n\n$=640 \\div 4$\n\n$=160$\n\n$160=160$, 计算的结果不变, 所以原题说法正确。\n\n故判断正确\n\n【点睛】本题考查了带小括号的混合运算,按照先计算小括号的运算,再按照运算顺序计算解答。", "solution": "null", "level": "四年级", "question": "把算式 $32 \\times ( 20 \\div 4 )$ 中的括号去掉, 计算结果不变。()", "options": [], "subject": "算术", "analysis": "解$\\sqrt{ }$\n\n【详解】按照先算括号里面的除法,再算括号外的乘法的计算顺序求出算式的结果为 160 ; 去掉括号后算式是 $32 \\times 20 \\div 4$, 按照从左到右的顺序计算出算式的结果为 160 , 两次计算的结果相同, 据此解答。\n\n【解答】带上括号时: $32 \\times(20 \\div 4)$\n\n$=32 \\times 5$\n\n$=160$\n\n不带括号时: $32 \\times 20 \\div 4$\n\n$=640 \\div 4$\n\n$=160$\n\n$160=160$, 计算的结果不变, 所以原题说法正确。\n\n故判断正确\n\n【点睛】本题考查了带小括号的混合运算,按照先计算小括号的运算,再按照运算顺序计算解答。"} {"id": "27635", "image": [], "answer": "解$\\times$\n\n【分析】 0 表示没有, 根据乘法的意义可知, 0 和任何数相乘都得 0 ; 根据除法的意义可知, 0 能作被除数, 但 0 不能作除数, 0 作除数没有意义; 据此判断。\n\n【详解】根据乘法与除法的意义可知, 0 和任何数相乘都得 0 , 例如: $0 \\times 5=0$;\n\n0 能作被除数, 但 0 不能作除数, 0 作除数没有意义, 所以 0 除以任何不为 0 的数都等于 0 , 例如: $0 \\div 8$ $=0$;\n\n所以 0 和任何数相除都得 0 的说法是错误的。\n\n故答案为: $\\times$\n\n【点睛】本题重点考查了 0 不能作除数的这一特性。", "solution": "null", "level": "四年级", "question": "0 和任何数相乘都得 0,0 和任何数相除都得 0 。", "options": [], "subject": "算术", "analysis": "解$\\times$\n\n【分析】 0 表示没有, 根据乘法的意义可知, 0 和任何数相乘都得 0 ; 根据除法的意义可知, 0 能作被除数, 但 0 不能作除数, 0 作除数没有意义; 据此判断。\n\n【详解】根据乘法与除法的意义可知, 0 和任何数相乘都得 0 , 例如: $0 \\times 5=0$;\n\n0 能作被除数, 但 0 不能作除数, 0 作除数没有意义, 所以 0 除以任何不为 0 的数都等于 0 , 例如: $0 \\div 8$ $=0$;\n\n所以 0 和任何数相除都得 0 的说法是错误的。\n\n故答案为: $\\times$\n\n【点睛】本题重点考查了 0 不能作除数的这一特性。"} {"id": "27637", "image": [], "answer": "解$513 ; 874 ; 40 ; 800$;\n\n$100000 ; 24 ; 90 ; 6$\n\n【详解】略", "solution": "null", "level": "四年级", "question": "直接写出得数。\n\n| $389+124=$ | $1100-226=$ | $880 \\div 22=$ | $32 \\times 25=$ |\n| :--- | :--- | :--- | :--- |\n| $125 \\times 800=$ | $48-48 \\div 2=$ | $36 \\div 2 \\times 5=$ | $210 \\div 35=$ |", "options": [], "subject": "算术", "analysis": "解$513 ; 874 ; 40 ; 800$;\n\n$100000 ; 24 ; 90 ; 6$\n\n【详解】略"} {"id": "27638", "image": ["13525.jpg"], "answer": "解200 元\n\n【分析】根据总价=单价 $\\times$ 数量, 求出成人买单程车票花的钱数, 再加上儿童单程票的钱数, 再用它们的和乘 2 就是往返的交通费; 据此解答。\n\n【详解】 $(40 \\times 2+40 \\div 2) \\times 2$\n\n$=(80+20) \\times 2$\n\n$=100 \\times 2$\n\n$=200$ (元)\n答: 往返的交通费是 200 元。\n\n【点睛】本题的关键是让学生理解求的是往返的车票钱, 而不是单程车票的钱数。", "solution": "null", "level": "四年级", "question": "恩平是温泉之乡, 星期天小明和爸爸、妈妈乘车去锦江温泉玩, 往返的交通费是多少元?\n\n", "options": [], "subject": "算术", "analysis": "解200 元\n\n【分析】根据总价=单价 $\\times$ 数量, 求出成人买单程车票花的钱数, 再加上儿童单程票的钱数, 再用它们的和乘 2 就是往返的交通费; 据此解答。\n\n【详解】 $(40 \\times 2+40 \\div 2) \\times 2$\n\n$=(80+20) \\times 2$\n\n$=100 \\times 2$\n\n$=200$ (元)\n答: 往返的交通费是 200 元。\n\n【点睛】本题的关键是让学生理解求的是往返的车票钱, 而不是单程车票的钱数。"} {"id": "27640", "image": [], "answer": "解9 辆\n\n【分析】根据题意, 先用大米的总数量减去已经运走的数量求出还剩下的数量, 然后再除以每辆汽车可以运的数量即可求解。\n\n【详解】 $(1980-1404) \\div 64$\n\n$=576 \\div 64$\n\n$=9$ (辆)\n\n答: 需要 9 辆汽车才能将剩下的大米一次运完。\n\n【点睛】本题主要考查了整数减法以及除法的意义, 关键是先求出剩下的大米数量。", "solution": "null", "level": "四年级", "question": "一个粮库有 1980 袋大米, 已经运走 1404 袋, 如果每辆汽车运 64 袋, 需要多少辆汽车才能将剩下的大米一次运完?", "options": [], "subject": "算术", "analysis": "解9 辆\n\n【分析】根据题意, 先用大米的总数量减去已经运走的数量求出还剩下的数量, 然后再除以每辆汽车可以运的数量即可求解。\n\n【详解】 $(1980-1404) \\div 64$\n\n$=576 \\div 64$\n\n$=9$ (辆)\n\n答: 需要 9 辆汽车才能将剩下的大米一次运完。\n\n【点睛】本题主要考查了整数减法以及除法的意义, 关键是先求出剩下的大米数量。"} {"id": "14097", "image": ["3081.jpg", "3082.jpg", "3083.jpg", "3084.jpg", "3085.jpg"], "answer": "62.16", "solution": "null", "level": "四年级", "question": "用坚式计算。(每题 3 分, 共 12 分)\n$", "options": [], "subject": "画法几何学", "analysis": ""} {"id": "27454", "image": ["13328.jpg", "13329.jpg", "13330.jpg", "13331.jpg", "13330.jpg", "13331.jpg"], "answer": "解$\\times$\n\n【分析】根据对三视图的认识可知,从上面看到的形状是\n\n\n\n,这个组合体任意一排都可能是 1 层, 也可能是 2 层, 还可能是 3 层, 依此判断。\n\n【详解】根据分析可知: 用几个正方体拼成一个组合体, 从上面看到的形状是\n\n\n个组合体不一定是用三个小正方体组成的。\n\n故答案为: $\\times$\n\n【点睛】熟练掌握对三视图的认识是解答本题的关键。", "solution": "null", "level": "四年级", "question": "用几个正方体拼成一个组合体, 从上面看到的形状是\n\n\n\n。那么, 这个组合体一定是用三个小正方体组成的。( )\n\n16 ._\n\n\n\n从前面看和从上面看图形相同。( )", "options": [], "subject": "画法几何学", "analysis": "解$\\times$\n\n【分析】根据对三视图的认识可知,从上面看到的形状是\n\n\n\n,这个组合体任意一排都可能是 1 层, 也可能是 2 层, 还可能是 3 层, 依此判断。\n\n【详解】根据分析可知: 用几个正方体拼成一个组合体, 从上面看到的形状是\n\n\n个组合体不一定是用三个小正方体组成的。\n\n故答案为: $\\times$\n\n【点睛】熟练掌握对三视图的认识是解答本题的关键。"} {"id": "27456", "image": ["13335.jpg", "13336.jpg", "13336.jpg"], "answer": "解\n\n【详解】因为房子是斜着的, 所以有门的一面可以是正面, 有窗户的一面也可以是正面,这样抓出房子的主要特征判断出另外的面即可.", "solution": "null", "level": "四年级", "question": "它们是从房子的哪一个方向看到的?\n\n", "options": [], "subject": "画法几何学", "analysis": "解\n\n【详解】因为房子是斜着的, 所以有门的一面可以是正面, 有窗户的一面也可以是正面,这样抓出房子的主要特征判断出另外的面即可."} {"id": "13980", "image": ["2864.jpg", "2865.jpg", "2866.jpg"], "answer": "$\\angle 2=\\left(180^{\\circ}-30^{\\circ}\right) \\div 2=75^{\\circ}$ (答案不唯一)", "solution": "null", "level": "四年级", "question": "在点子图上分别画一个等腰直角三角形和一个针角三角形, 并画出每个三角形的一条高。21.华华家要建一座漂亮的房子。请你帮她家选择三根木料做成房子的三脚架(三脚架为等腰三角形), 并说说你选择的理由。(6 分)\n\n![](本地图片-5232/.jpg)23.一张直角三角形的纸片, 剪去直角后变成一个四边形, 如图, 求 $\\angle$ 1 与 $\\angle 2$ 的度数和。(8 分)\n\n", "options": [], "subject": "画法几何学", "analysis": ""} {"id": "13999", "image": ["2871.jpg"], "answer": "", "solution": "null", "level": "四年级", "question": "画出下列轴对称图形的另一半。(8 分)\n![](本地图片-", "options": [], "subject": "画法几何学", "analysis": ""} {"id": "14112", "image": ["2916.jpg"], "answer": "$28.4-6.35-3.65$$=28.4^{-}(6.35+3.65)$$=28.4-10$$=18.4$(3) $1500 \\div 125 \\div 8$$=1500 \\div(125 \times 8)$$=1500 \\div 1000$(2) $418+501+182+299$$=1.5$$=(418+182)+(501+299)$$=600+800$$=1400$(4) $5.4 \times 99+5.4$$=5.4 \times(99+1)$$=5.4 \times 100$5) $38 \times[(373-213) \\div 8]$ $=38 \times[160 \\div 8]$$=38 \times 20$$=760$ $=80 \times 125+8 \times 125 \\cdots \\cdots 1$ 分$=10000+1000 \\cdots \\cdots 1$", "solution": "null", "level": "四年级", "question": "计算下面各题, 能简算的要简算。(每小题 3 分, 共 18 分)\n(1) $", "options": [], "subject": "立体几何学", "analysis": ""} {"id": "27453", "image": [], "answer": "解$\\times$\n\n【分析】根据从上面和左面看到的图形可得,这个图形一共有 2 层:下层是 4 个小正方体,上层至少 2 个小正方体, 最多 4 个小正方体, 据此即可解答问题。\n\n【详解】根据分析可得该立体图形可能有 $6 、 7 、 8$ 个小正方体组成, 所以这个立体图形不一定是正方体。\n\n故答案为: $\\times$\n\n【点睛】本题主要考查根据三视图确认几何体。", "solution": "null", "level": "四年级", "question": "分别从上面和左面观察一个立体图形, 看到的形状都是“ $\\square$,, 这个立体图形一定是正方体。", "options": [], "subject": "立体几何学", "analysis": "解$\\times$\n\n【分析】根据从上面和左面看到的图形可得,这个图形一共有 2 层:下层是 4 个小正方体,上层至少 2 个小正方体, 最多 4 个小正方体, 据此即可解答问题。\n\n【详解】根据分析可得该立体图形可能有 $6 、 7 、 8$ 个小正方体组成, 所以这个立体图形不一定是正方体。\n\n故答案为: $\\times$\n\n【点睛】本题主要考查根据三视图确认几何体。"} {"id": "27455", "image": ["13332.jpg", "13333.jpg", "13334.jpg", "13333.jpg", "13334.jpg"], "answer": "解$\\times$\n\n【分析】分别画出这个图形从前面、上面、左面看到的图形, 然后再判断即可。\n\n【详解】从前面看是:\n\n\n从左面看是:\n\n\n\n故答案为: $\\times$\n\n【点睛】熟练掌握物体三视图的画法是解答此题的关键。", "solution": "null", "level": "四年级", "question": "从前面、上面和左面看\n\n\n,看到的形状都相同。", "options": [], "subject": "立体几何学", "analysis": "解$\\times$\n\n【分析】分别画出这个图形从前面、上面、左面看到的图形, 然后再判断即可。\n\n【详解】从前面看是:\n\n\n从左面看是:\n\n\n\n故答案为: $\\times$\n\n【点睛】熟练掌握物体三视图的画法是解答此题的关键。"} {"id": "27457", "image": ["13337.jpg", "13338.jpg", "13339.jpg", "13340.jpg", "13341.jpg", "13339.jpg", "13340.jpg", "13341.jpg"], "answer": "解6 块 8 块\n\n【详解】如图\n\n\n\n俯视图\n\n\n\n左视图\n\n组成这样的图形最少需要 6 个方块, 最多需要 8 个方块(下图):\n\n\n【点睛】根据从上面、左面看到的形状,所用的小正方体分前、后两排,上、下两层. 下层前、后排各两个,前排左边一个与后排右面一个对齐;上层前、后排最少各放 1 个, 最多各放 2 个.", "solution": "null", "level": "四年级", "question": "如图, 是由方块组成的图形的俯视图和左视图, 组成这样的图形最多需要多少方块? 最少需要多少方块?\n\n\n\n俯视图\n\n\n\n左视图", "options": [], "subject": "立体几何学", "analysis": "解6 块 8 块\n\n【详解】如图\n\n\n\n俯视图\n\n\n\n左视图\n\n组成这样的图形最少需要 6 个方块, 最多需要 8 个方块(下图):\n\n\n【点睛】根据从上面、左面看到的形状,所用的小正方体分前、后两排,上、下两层. 下层前、后排各两个,前排左边一个与后排右面一个对齐;上层前、后排最少各放 1 个, 最多各放 2 个."} {"id": "27459", "image": ["13346.jpg", "13347.jpg", "13348.jpg", "13349.jpg", "13348.jpg", "13349.jpg"], "answer": "解5 个\n\n【分析】从组合体的层数, 每层的行数、列数去观察推理。\n\n【详解】从前面看是\n\n\n\n可知此组合体有 2 层, 底层有 3 列, 最上层有 1 列; 从上面看是\n\n\n\n可知最底层有 2 行, 结合刚才从前面看到的图形, 可以初步推理出可能由 5 个小正方体\n\n组成, 最后从左面看是 $\\square$.确定了最底层有 4 个小正方体, 最上层有 1 个小正方体。\n\n【点睛】考察空间想象推理能力。", "solution": "null", "level": "四年级", "question": "一个用若干个相同的小正方体摆成的立体图形, 从前面看是\n\n\n从上面看是\n\n\n摆这样一个立体图形需要多少个小正方体?", "options": [], "subject": "立体几何学", "analysis": "解5 个\n\n【分析】从组合体的层数, 每层的行数、列数去观察推理。\n\n【详解】从前面看是\n\n\n\n可知此组合体有 2 层, 底层有 3 列, 最上层有 1 列; 从上面看是\n\n\n\n可知最底层有 2 行, 结合刚才从前面看到的图形, 可以初步推理出可能由 5 个小正方体\n\n组成, 最后从左面看是 $\\square$.确定了最底层有 4 个小正方体, 最上层有 1 个小正方体。\n\n【点睛】考察空间想象推理能力。"} {"id": "27460", "image": ["13350.jpg", "13351.jpg", "13352.jpg"], "answer": "解从前面(后面)看形状相同.\n\n从上面看形状各不相同.\n\n【详解】略", "solution": "null", "level": "四年级", "question": "用 5 个相同的小正方体分别拼成如下三个图形.\n\n\n\nA\n\n\n\nB\n\n\n\nC\n\n从哪个位置看, 3 个图形的形状是相同的?\n\n从哪个位置看, 3 个图形的形状是各不相同的?", "options": [], "subject": "立体几何学", "analysis": "解从前面(后面)看形状相同.\n\n从上面看形状各不相同.\n\n【详解】略"} {"id": "14113", "image": [], "answer": "$884.5+256.8=1141.3$ (元) $\\cdots \\cdots \\cdots \\cdots \\cdot 2.5$ 分【列式 1.5 分,结果 1 分】 $884.5+1141.3=2025.8$ (元) $\\cdots \\cdots \\cdots \\cdots \\cdot \\cdots 2$ 分【列式 1 分, 结果 1 分】答: 师生共捐款 2025.8 元。", "solution": "null", "level": "四年级", "question": "明丽小学师生为希望小学捐款, 同学们共捐款", "options": [], "subject": "计数", "analysis": "答案: $"} {"id": "14114", "image": [], "answer": "(每空 1 分) 淘气的成绩答案不唯一, 只要在 $16.50 \\sim 17.00$ 之间即可。\begin{tabular}{|c|c|c|c|c|c|}\\hline 姓名 & 明明 & 丽丽 & 奇思 & 妙想 & 淘气 \\\\hline 时间/秒 & 15.33 & 17.00 & 15.80 & 16.50 & 16.58 \\\\hline 名次 & 第一名 & 第五名 & 第二名 & 第三名 & 第四名 \\\\hline\\end{tabular}", "solution": "null", "level": "四年级", "question": "下表是对四 (1) 班百米赛跑的 5 位同学进行了成绩统计, 请你帮他们按成绩从高到低排一下名次, 并想一想淘气的成绩可能是多少?(5分)\n\n\\begin{tabular}{|c|c|c|c|c|c|}\n\\hline 姓名 & 明明 & 丽丽 & 奇思 & 妙想 & 淘气 \\\\\n\\hline 时间秒 &", "options": [], "subject": "计数", "analysis": ""} {"id": "14115", "image": ["3068.jpg"], "answer": "答案: $55 \times(35-25) \\cdots \\cdots \\cdots \\cdots 3$ 分$=55 \times 10 \\cdots \\cdots \\cdots \\cdots 1.5$ 分$=550$ (元) $\\cdots \\cdots \\cdots \\cdots \\cdot 1$ 分答: 一共可以赚 550 元。…........ 0.5 分", "solution": "null", "level": "四年级", "question": "商店以每瓶 25 元的进货价购进某种消毒液 55 瓶, 按照每瓶 35 元卖出后, 一共可以赚多少元钱?(6 分)", "options": [], "subject": "计数", "analysis": ""} {"id": "14007", "image": ["2875.jpg"], "answer": "(1)(2)四 256$(3)$ 二(4) 82(5)100 每天的收件量在不断增加(答案不唯一, 合理即可)", "solution": "null", "level": "四年级", "question": "快递在我们的生活中是不可或缺的, 李叔叔经营了一家快递点, 他统计了去年 1 月份前 4 天收件量和派件量情况,如下表。\n\n| | 第一天 | 第二天 | 第三天 | 第四天 |\n| :---: | :---: | :---: | :---: | :---: |\n| 收件量/件 | 45 | 55 | 68 | 88 |\n| 派件量/件 | 84 | 120 | 54 | 70 |\n\n(1)根据统计表信息, 补全统\n\n计图。\n\n![](本地图片-5292、.jpg)\n\n件量相差最多。\n\n(4)这四天平均每天的派件量是( ) 件。\n\n(5)预测第五天的收件量大约是( )件, 理由是( )。", "options": [], "subject": "计数", "analysis": ""} {"id": "14020", "image": [], "answer": "大货车: $400 \\div 8=50$ (元)小货车: $300 \\div 5=60$ (元)大货车便宜。$36 \\div 8=4$ (辆)4 辆大货车: $400 \times 4=1600($ 元)1 辆小货车: 300 元$1600+300=1900$ (元) 答: 安排 4 辆大货车和 1 辆小货车最省钱。", "solution": "null", "level": "四年级", "question": "下面是典典、聪聪、华华和同同参加 50 米赛跑的成绩信息。\n(1)典典跑了", "options": [], "subject": "计数", "analysis": ""} {"id": "14059", "image": [], "answer": "答案:900 \\div 40=22$ (元) $\\cdots \\cdots \\cdot 20$ (元)\n\n$500 \\div 20=25($ 元 $)$\n\n应尽量多租大车。\n\n$324+14=338$ (人)\n\n$338 \\div 40=8$ (辆) $\\cdots \\cdots 18$ (人)\n\n租 9 辆大车: $900 \\times 9=8100$ (元)\n\n租 8 辆大车和 1 辆小车: $900 \\times 8+500=7700($ 元 $)$\n\n租 7 辆大车和 3 辆小车: $900 \\times 7+500 \\times 3=7800$ (元)\n\n$8100>7800>7700$\n\n答: 租 8 辆大车和 1 辆小车最省钱。\n\n挑战题: $28 \\times 2 \\div(26-18)=7($ 元 $)$\n\n$44 \\times 7 \\div 2=154$ (元)\n\n答: 小明、小光各带了 154 元, 每本书的价格是 7 元。", "solution": "null", "level": "四年级", "question": "育才小学的师生到黄河风景名胜区研学, 有 14 名老师和 324 名学生参加。每辆大车可坐 40 人, 租金 900 元; 每辆小车可坐 20 人, 租金 500 元。怎样租车最省钱? (7 分)", "options": [], "subject": "计数", "analysis": "答案:900 \\div 40=22$ (元) $\\cdots \\cdots \\cdot 20$ (元)\n\n$500 \\div 20=25($ 元 $)$\n\n应尽量多租大车。\n\n$324+14=338$ (人)\n\n$338 \\div 40=8$ (辆) $\\cdots \\cdots 18$ (人)\n\n租 9 辆大车: $900 \\times 9=8100$ (元)\n\n租 8 辆大车和 1 辆小车: $900 \\times 8+500=7700($ 元 $)$\n\n租 7 辆大车和 3 辆小车: $900 \\times 7+500 \\times 3=7800$ (元)\n\n$8100>7800>7700$\n\n答: 租 8 辆大车和 1 辆小车最省钱。\n\n挑战题: $28 \\times 2 \\div(26-18)=7($ 元 $)$\n\n$44 \\times 7 \\div 2=154$ (元)\n\n答: 小明、小光各带了 154 元, 每本书的价格是 7 元。"} {"id": "14079", "image": [], "answer": "答案:去 B 商店买比较便宜。因为 $25.80>25.50$ ,$78.50>76.90,11.90>11.60$,所以 $25.80+78.50+11.90>25.50+76.90+11.60$,所以去 B 商店买比较便宜。", "solution": "null", "level": "四年级", "question": "某国产品牌汽车凭借出色的发动机技术, 创下了每行驶 100 千米耗油", "options": [], "subject": "计数", "analysis": ""} {"id": "27589", "image": [], "answer": "解【解答】解: $(302+297+301+303+300+298+299+301+299+298) \\div 10$\n\n$=2998 \\div 10$\n\n$\\approx 300$ (元)\n\n$300 \\times 30=9000 ($ 元)\n\n答: 9 月份一个月的营业额大约是 9000 元。", "solution": "null", "level": "四年级", "question": "(2022 秋・顺德区期中)如表是欢乐小卖部 9 月份前 10 天的营业额, 请估算出 9 月份的总营业额是多少?\n\n| 日期 | 1 | 2 | 3 | 4 | 5 | 6 | 7 | 8 | 9 | 10 |\n| :---: | :---: | :---: | :---: | :---: | :---: | :---: | :---: | :---: | :---: | :---: |\n| 营业额 | 302 | 297 | 301 | 303 | 300 | 298 | 299 | 301 | 299 | 298 |", "options": [], "subject": "计数", "analysis": "解【解答】解: $(302+297+301+303+300+298+299+301+299+298) \\div 10$\n\n$=2998 \\div 10$\n\n$\\approx 300$ (元)\n\n$300 \\times 30=9000 ($ 元)\n\n答: 9 月份一个月的营业额大约是 9000 元。"} {"id": "27616", "image": [], "answer": "解: $62=50+12$,\n\n租 5 辆大货车, 3 辆小货车最省钱;\n\n$5 \\times 7000+3 \\times 3200$\n\n$=35000+9600$\n\n$=44600$ (元)\n\n答: 租 5 辆大货车, 3 辆小货车最省钱; 需要 44600 元。", "solution": "null", "level": "四年级", "question": "2022 年上海疫情期间, 来自全国的物资源源不断的运到上海。有一批 62 吨的物资要从北京运往上海。如果租大货车每次可运 10 吨, 每次运费 7000 元; 如果租小货车每次可以运 4 吨, 每次运费 3200 元。怎样租车最省钱?需要多少钱?(大小货车可以搭配租)", "options": [], "subject": "计数", "analysis": "解: $62=50+12$,\n\n租 5 辆大货车, 3 辆小货车最省钱;\n\n$5 \\times 7000+3 \\times 3200$\n\n$=35000+9600$\n\n$=44600$ (元)\n\n答: 租 5 辆大货车, 3 辆小货车最省钱; 需要 44600 元。"} {"id": "27618", "image": [], "answer": "解【答案】解: $7 \\times 8=56$ (套)\n\n$$\n\\begin{aligned}\n& 66-56=10 \\text { (套) } \\\\\n& 7 \\times 400+2 \\times 300 \\\\\n& =2800+600 \\\\\n& =3400 \\text { (元 })\n\\end{aligned}\n$$\n\n答: 头 7 大卷, 2 小卷最省钱, 需要 3400 元。", "solution": "null", "level": "四年级", "question": "学校做 66 套表演服, 要按整卷买布。大卷布, 每卷 400 元, 能做 8 套服装; 小卷布, 每卷 300 元,能做 5 套服装。怎样买布最省钱?", "options": [], "subject": "计数", "analysis": "解【答案】解: $7 \\times 8=56$ (套)\n\n$$\n\\begin{aligned}\n& 66-56=10 \\text { (套) } \\\\\n& 7 \\times 400+2 \\times 300 \\\\\n& =2800+600 \\\\\n& =3400 \\text { (元 })\n\\end{aligned}\n$$\n\n答: 头 7 大卷, 2 小卷最省钱, 需要 3400 元。"} {"id": "27452", "image": [], "answer": "解$\\sqrt{ }$\n\n【分析】根据从不同的角度观察物体的方法可知:相同的物体分别从不同的角度观察,看到的形状可能是相同的, 也可能是不同的; 由此进行解答。\n\n【详解】相同的物体分别从不同的角度观察, 看到的形状可能是相同的, 也可能是不同的; 故此说法正确。\n\n【点睛】本题考查了从不同的角度观察物体,关键是要掌握从不同的角度观察物体的方法。", "solution": "null", "level": "四年级", "question": "同一个物体分别从不同的角度观察, 看到的形状可能相同, 也可能不同。", "options": [], "subject": "度量几何学", "analysis": "解$\\sqrt{ }$\n\n【分析】根据从不同的角度观察物体的方法可知:相同的物体分别从不同的角度观察,看到的形状可能是相同的, 也可能是不同的; 由此进行解答。\n\n【详解】相同的物体分别从不同的角度观察, 看到的形状可能是相同的, 也可能是不同的; 故此说法正确。\n\n【点睛】本题考查了从不同的角度观察物体,关键是要掌握从不同的角度观察物体的方法。"} {"id": "27272", "image": [], "answer": "解120 块\n\n【分析】首先用每块方砖的面积乘需要的块数求出会议室地面的面积, 然后根据“正方形的面积=边长 $\\times$ 边长”, 代入数据, 计算出边长为 6 分米的正方形的面积, 最后根据“包含”除法的意义, 用会议室地面的面积除以边长为 6 分米的正方形的面积即可, 据此解答。\n\n【详解】 $16 \\times 270 \\div(6 \\times 6)$\n\n$=4320 \\div 36$\n\n$=120$ (块)\n\n答: 需要 120 块。\n\n【点睛】此题主要考查正方形面积公式的灵活运用, 除法的意义及应用, 解答本题的关键是熟记正方形面积公式。", "solution": "null", "level": "四年级", "question": "会议室地面用面积为 16 平方分米的方砖铺设, 270 块方砖正好铺满。如果改用边长为 6 分米的方砖铺设,需要多少块?", "options": [], "subject": "度量几何学", "analysis": "解120 块\n\n【分析】首先用每块方砖的面积乘需要的块数求出会议室地面的面积, 然后根据“正方形的面积=边长 $\\times$ 边长”, 代入数据, 计算出边长为 6 分米的正方形的面积, 最后根据“包含”除法的意义, 用会议室地面的面积除以边长为 6 分米的正方形的面积即可, 据此解答。\n\n【详解】 $16 \\times 270 \\div(6 \\times 6)$\n\n$=4320 \\div 36$\n\n$=120$ (块)\n\n答: 需要 120 块。\n\n【点睛】此题主要考查正方形面积公式的灵活运用, 除法的意义及应用, 解答本题的关键是熟记正方形面积公式。"} {"id": "13979", "image": ["3058.jpg"], "answer": "答案:17 【点拨】因为三角形是等腰三角形, 所以第三条边的长度可能是 3 厘米, 也可能是 7 厘米。因为 $3+3<7,7+3>7$, 所以第三条边的长度为 7 厘米。", "solution": "null", "level": "四年级", "question": "求 $\\angle 1$ 的度数。( 4 分 $)$17.求 $\\angle 1$ 的度数。(6 分)\n\n", "options": [], "subject": "度量几何学", "analysis": "答案:17 【点拨】因为三角形是等腰三角形, 所以第三条边的长度可能是 3 厘米, 也可能是 7 厘米。因为 $3+3<7,7+3>7$, 所以第三条边的长度为 7 厘米。"} {"id": "14001", "image": ["2873.jpg"], "answer": "(1) 2+4=6(\\mathrm{~cm}) 6 \times 4=24(\\mathrm{~cm})$$$\begin{aligned}& (2)(10+3+5) \times 2=36(\\mathrm{~cm}) \\", "solution": "null", "level": "四年级", "question": "计算各图形的周长。(单位: cm)(每小题 4 分, 共 8 分)\n\n", "options": [], "subject": "度量几何学", "analysis": ""} {"id": "14002", "image": ["2874.jpg"], "answer": "甲、乙两只蚂蚁行走的路程相等。如图, 因为平移后, 甲蚂蚁行走的路程为 $6+6=12(\\mathrm{~cm})$, 而乙蚂蚁行走的路程也为 $6+6=12(\\mathrm{~cm})$ 。", "solution": "null", "level": "四年级", "question": "如图所示, 甲、乙两只蚂蚁从点 A 出发去点 B, 哪只蚂蚁行走的路程多一些? 为什么? (8 分)\n\n![](本地图片-", "options": [], "subject": "度量几何学", "analysis": ""} {"id": "27718", "image": ["13543.jpg"], "answer": "解280 平方米\n\n【分析】用阅览室的面积减去书法室的面积即可, 这两个教室都为长方形, 长方形的面积 $=$ 长 $\\times$ 宽,依此计算即可。\n\n【详解】 $20 \\times 35-12 \\times 35$\n\n$=(20-12) \\times 35$\n\n$=8 \\times 35$\n\n$=280$ (平方米)\n\n答:书法室的面积比阅览室的面积少 280 平方米。\n\n【点睛】熟练掌握长方形面积的实际运用是解答此题的关键。", "solution": "null", "level": "四年级", "question": "如图是胜利小学阅览室和书法室的平面图, 请问书法室的面积比阅览室的面积少多少平方米?\n\n", "options": [], "subject": "度量几何学", "analysis": "解280 平方米\n\n【分析】用阅览室的面积减去书法室的面积即可, 这两个教室都为长方形, 长方形的面积 $=$ 长 $\\times$ 宽,依此计算即可。\n\n【详解】 $20 \\times 35-12 \\times 35$\n\n$=(20-12) \\times 35$\n\n$=8 \\times 35$\n\n$=280$ (平方米)\n\n答:书法室的面积比阅览室的面积少 280 平方米。\n\n【点睛】熟练掌握长方形面积的实际运用是解答此题的关键。"} {"id": "27746", "image": ["13545.jpg", "13546.jpg", "13547.jpg", "13546.jpg", "13547.jpg"], "answer": "解360 平方米\n\n【分析】方法一: 如图 (1), 菜地分成了 2 个长方形, 分别求出 2 个长方形的面积再相加, 即等于菜地的面积。\n\n方法二: 如图 (2), 菜地分成了 2 个长方形, 分别求出 2 个长方形的面积再相加, 即等于菜地的面积。\n\n\n图 (1)\n\n\n图 (2)\n\n【详解】方法一: $21 \\times 9+19 \\times 9$\n\n$=(21+19) \\times 9$\n\n$=40 \\times 9$\n\n$=360$ (平方米)\n\n答:这块菜地的面积有 360 平方米。\n\n$$\n\\begin{aligned}\n& \\text { 方法二: }(21-9) \\times 9+(19+9) \\times 9 \\\\\n& =12 \\times 9+28 \\times 9 \\\\\n& =(12+28) \\times 9 \\\\\n& =40 \\times 9 \\\\\n& =360 \\text { (平方米) }\n\\end{aligned}\n$$\n\n答:这块菜地的面积有 360 平方米。\n\n【点睛】把求不规则图形面积变为求规则图形的面积是解答本题的关键。", "solution": "null", "level": "四年级", "question": "有一块菜地如下图, 这块菜地的面积有多少平方米?(尝试在图中画一画, 用两种方法解答。)\n\n", "options": [], "subject": "度量几何学", "analysis": "解360 平方米\n\n【分析】方法一: 如图 (1), 菜地分成了 2 个长方形, 分别求出 2 个长方形的面积再相加, 即等于菜地的面积。\n\n方法二: 如图 (2), 菜地分成了 2 个长方形, 分别求出 2 个长方形的面积再相加, 即等于菜地的面积。\n\n\n图 (1)\n\n\n图 (2)\n\n【详解】方法一: $21 \\times 9+19 \\times 9$\n\n$=(21+19) \\times 9$\n\n$=40 \\times 9$\n\n$=360$ (平方米)\n\n答:这块菜地的面积有 360 平方米。\n\n$$\n\\begin{aligned}\n& \\text { 方法二: }(21-9) \\times 9+(19+9) \\times 9 \\\\\n& =12 \\times 9+28 \\times 9 \\\\\n& =(12+28) \\times 9 \\\\\n& =40 \\times 9 \\\\\n& =360 \\text { (平方米) }\n\\end{aligned}\n$$\n\n答:这块菜地的面积有 360 平方米。\n\n【点睛】把求不规则图形面积变为求规则图形的面积是解答本题的关键。"} {"id": "28052", "image": ["13584.jpg", "13585.jpg", "13585.jpg"], "answer": "解135 平方米\n\n【分析】根据题意可将这块菜地分成 1 个长为 $27 \\mathrm{~m}$, 宽为 $3 \\mathrm{~m}$ 的长方形和 1 个长为 $3 \\mathrm{~m}$, 宽为 $18 \\mathrm{~m}$ 的长方形, 长方形的面积 $=$ 长 $\\times$ 宽, 依此计算出这两个长方形的面积后, 再相加即可。根据混合运算的计算顺序列式并根据整数乘法分配律的特点计算即可。\n\n【详解】\n\n\n\n$27 \\times 3+3 \\times 18$\n\n$=(27+18) \\times 3$\n\n$=45 \\times 3$\n\n$=135$ (平方米)\n\n答:这块菜地的面积是 135 平方米。\n\n【点睛】此题考查的是运用乘法分配律的特点解决实际问题,应熟练掌握长方形的面积的计算。", "solution": "null", "level": "四年级", "question": "王大伯家有一块菜地(如下图), 这块菜地的面积是多少平方米?\n\n", "options": [], "subject": "度量几何学", "analysis": "解135 平方米\n\n【分析】根据题意可将这块菜地分成 1 个长为 $27 \\mathrm{~m}$, 宽为 $3 \\mathrm{~m}$ 的长方形和 1 个长为 $3 \\mathrm{~m}$, 宽为 $18 \\mathrm{~m}$ 的长方形, 长方形的面积 $=$ 长 $\\times$ 宽, 依此计算出这两个长方形的面积后, 再相加即可。根据混合运算的计算顺序列式并根据整数乘法分配律的特点计算即可。\n\n【详解】\n\n\n\n$27 \\times 3+3 \\times 18$\n\n$=(27+18) \\times 3$\n\n$=45 \\times 3$\n\n$=135$ (平方米)\n\n答:这块菜地的面积是 135 平方米。\n\n【点睛】此题考查的是运用乘法分配律的特点解决实际问题,应熟练掌握长方形的面积的计算。"} {"id": "27840", "image": ["13567.jpg"], "answer": "解188 米\n\n【分析】用从入口处到魔鬼屋全长减去从入口处到摩天轮的距离, 再减去过山车到魔鬼屋的距离, 即可得摩天轮与过山车相距多少米。\n\n【详解】 $688-372-128$\n\n$=688-(372+128)$\n\n$=688-500$\n\n$=188$ (米)\n\n答: 摩天轮与过山车相距 188 米。\n\n【点睛】本题考查了图文应用题, 关键是仔细读图, 弄清数量关系。", "solution": "null", "level": "四年级", "question": "游园之旅: 从入口处到魔鬼屋全长 688 米。摩天轮与过山车相距多少米?\n\n", "options": [], "subject": "度量几何学", "analysis": "解188 米\n\n【分析】用从入口处到魔鬼屋全长减去从入口处到摩天轮的距离, 再减去过山车到魔鬼屋的距离, 即可得摩天轮与过山车相距多少米。\n\n【详解】 $688-372-128$\n\n$=688-(372+128)$\n\n$=688-500$\n\n$=188$ (米)\n\n答: 摩天轮与过山车相距 188 米。\n\n【点睛】本题考查了图文应用题, 关键是仔细读图, 弄清数量关系。"} {"id": "28077", "image": ["13588.jpg"], "answer": "解871 米; 0.871 千米\n\n【分析】通过观察图形可知, 一面靠海, 用网围了 3 面, 根据加法的意义, 用加法先算出总长度, 再根据 1 千米 $=1000$ 米, 解答此题即可。\n\n【详解】 $241+418+212$\n\n$=659+212$\n\n$=871$ (米)\n\n871 米 $=0.871$ 千米\n\n答: 需要准备 871 米的网, 合 0.871 千米。\n\n【点睛】此题考查的目的是理解长方形周长的意义。熟练掌握长度单位的换算, 是解答此题的关键。", "solution": "null", "level": "四年级", "question": "厦门是一座美丽的滨海城市, 夏季要用网在海边围出一块水域作为安全垂钓区, 如图。需要准备几米的网?合多少千米?\n\n", "options": [], "subject": "度量几何学", "analysis": "解871 米; 0.871 千米\n\n【分析】通过观察图形可知, 一面靠海, 用网围了 3 面, 根据加法的意义, 用加法先算出总长度, 再根据 1 千米 $=1000$ 米, 解答此题即可。\n\n【详解】 $241+418+212$\n\n$=659+212$\n\n$=871$ (米)\n\n871 米 $=0.871$ 千米\n\n答: 需要准备 871 米的网, 合 0.871 千米。\n\n【点睛】此题考查的目的是理解长方形周长的意义。熟练掌握长度单位的换算, 是解答此题的关键。"} {"id": "27922", "image": [], "answer": "解$12 \\mathrm{dm}$\n\n【分析】先进行单位换算, 将 $12 \\mathrm{~m}^{2}$ 换算成 $1200 \\mathrm{dm}^{2}$ 。已知四块小长方形木板拼成的展示栏的面积是\n\n$1200 \\mathrm{dm}^{2}$, 则每个小长方形木板的面积是 $(1200 \\div 4) \\mathrm{dm}^{2}$ 。再根据宽 $=$ 长方形面积 $\\div$ 长, 得出木板的宽\n为 $1200 \\div 4 \\div 25=12(\\mathrm{dm})$ 。\n\n【详解】 $12 \\mathrm{~m}^{2}=1200 \\mathrm{dm}^{2}$\n\n$1200 \\div 4 \\div 25$\n\n$=1200 \\div(4 \\times 25)$\n\n$=1200 \\div 100$\n\n$=12(\\mathrm{dm})$\n\n答: 小长方形木板的宽是 $12 \\mathrm{dm}$ 。\n\n【点睛】本题考查长方形面积公式的应用, 连续除以两个数等于除以这两个数的乘积, 要利用运算律简便计算。", "solution": "null", "level": "四年级", "question": "学校用四块同样大小的小长方形木板拼成了一个大长方形展示栏, 每块小长方形木板的长是 $25 \\mathrm{dm}$,展示栏的面积是 $12 \\mathrm{~m}^{2}$ 。小长方形木板的宽是多少分米?", "options": [], "subject": "度量几何学", "analysis": "解$12 \\mathrm{dm}$\n\n【分析】先进行单位换算, 将 $12 \\mathrm{~m}^{2}$ 换算成 $1200 \\mathrm{dm}^{2}$ 。已知四块小长方形木板拼成的展示栏的面积是\n\n$1200 \\mathrm{dm}^{2}$, 则每个小长方形木板的面积是 $(1200 \\div 4) \\mathrm{dm}^{2}$ 。再根据宽 $=$ 长方形面积 $\\div$ 长, 得出木板的宽\n为 $1200 \\div 4 \\div 25=12(\\mathrm{dm})$ 。\n\n【详解】 $12 \\mathrm{~m}^{2}=1200 \\mathrm{dm}^{2}$\n\n$1200 \\div 4 \\div 25$\n\n$=1200 \\div(4 \\times 25)$\n\n$=1200 \\div 100$\n\n$=12(\\mathrm{dm})$\n\n答: 小长方形木板的宽是 $12 \\mathrm{dm}$ 。\n\n【点睛】本题考查长方形面积公式的应用, 连续除以两个数等于除以这两个数的乘积, 要利用运算律简便计算。"} {"id": "28207", "image": [], "answer": "解42500 元\n\n【分析】用每平方米造价乘两个温室大棚的面积和即可解答。\n\n【详解】 $85 \\times(230+270)$\n\n$=85 \\times 500$\n\n$=42500$ (元)\n\n答:两个温室大棚的总造价是 42500 元。\n\n【点睛】本题主要考查学生对乘法分配律的掌握和灵活运用。", "solution": "null", "level": "四年级", "question": "一个蔬菜种植园建了两个温室大棚, 一个温室大棚的面积是 230 平方米,另一个温室大棚的面积是 270 平方米,每平方米造价 85 元。两个温室大棚的总造价是多少元?", "options": [], "subject": "度量几何学", "analysis": "解42500 元\n\n【分析】用每平方米造价乘两个温室大棚的面积和即可解答。\n\n【详解】 $85 \\times(230+270)$\n\n$=85 \\times 500$\n\n$=42500$ (元)\n\n答:两个温室大棚的总造价是 42500 元。\n\n【点睛】本题主要考查学生对乘法分配律的掌握和灵活运用。"} {"id": "28254", "image": [], "answer": "解2.13 米\n\n【分析】根据题意可知, 用 1 本书的厚度乘 100 即可, 然后再将单位化成米, 100 厘米 $=1$ 米, 依此计算并换算。\n\n【详解】 $2.13 \\times 100=213$ (厘米)\n\n213 厘米 $=2.13$ (米)\n\n答: 100 本这样的书一共厚 2.13 米。\n\n【点睛】此题考查的是小数点位置的移动, 以及小数与单位换算, 应熟记米与厘米之间的进率。", "solution": "null", "level": "四年级", "question": "一本书厚 2.13 厘米, 100 本这样的书一共厚多少米?", "options": [], "subject": "度量几何学", "analysis": "解2.13 米\n\n【分析】根据题意可知, 用 1 本书的厚度乘 100 即可, 然后再将单位化成米, 100 厘米 $=1$ 米, 依此计算并换算。\n\n【详解】 $2.13 \\times 100=213$ (厘米)\n\n213 厘米 $=2.13$ (米)\n\n答: 100 本这样的书一共厚 2.13 米。\n\n【点睛】此题考查的是小数点位置的移动, 以及小数与单位换算, 应熟记米与厘米之间的进率。"} {"id": "28308", "image": ["13608.jpg"], "answer": "解1000 平方米\n\n【分析】健身场的长 5 厘米和宽 2 厘米是分别缩小到原数的 $\\frac{1}{1000}$ 后的结果, 那么健身场原来长、宽就是缩小到原数后长、宽的 1000 倍, 那么实际长和宽就要分别乘 1000 , 就是健身场原来的长和宽,再把单位化成米, 最后根据长方形的面积 $=$ 长 $\\times$ 宽, 求出这个健身场的实际面积是多少平方米即可。\n\n【详解】 $5 \\times 1000=5000$ (厘米)\n\n5000 厘米 $=50$ 米\n\n$2 \\times 1000=2000$ (厘米)\n\n2000 厘米 $=20$ 米\n\n$50 \\times 20=1000$ (平方米)\n\n答:这个健身场的实际面积是 1000 平方米。\n【点睛】解答此题的关键在于掌握把一个数缩小到原来的 $\\frac{1}{1000}$ 的方法及求长方形面积的方法。", "solution": "null", "level": "四年级", "question": "健身场是一个长方形, 把健身场的长和宽分别缩小到原数的 $\\frac{1}{1000}$ 后, 画出了下边的示意图。这个健身场的实际面积是多少平方米?\n\n", "options": [], "subject": "度量几何学", "analysis": "解1000 平方米\n\n【分析】健身场的长 5 厘米和宽 2 厘米是分别缩小到原数的 $\\frac{1}{1000}$ 后的结果, 那么健身场原来长、宽就是缩小到原数后长、宽的 1000 倍, 那么实际长和宽就要分别乘 1000 , 就是健身场原来的长和宽,再把单位化成米, 最后根据长方形的面积 $=$ 长 $\\times$ 宽, 求出这个健身场的实际面积是多少平方米即可。\n\n【详解】 $5 \\times 1000=5000$ (厘米)\n\n5000 厘米 $=50$ 米\n\n$2 \\times 1000=2000$ (厘米)\n\n2000 厘米 $=20$ 米\n\n$50 \\times 20=1000$ (平方米)\n\n答:这个健身场的实际面积是 1000 平方米。\n【点睛】解答此题的关键在于掌握把一个数缩小到原来的 $\\frac{1}{1000}$ 的方法及求长方形面积的方法。"} {"id": "28322", "image": [], "answer": "解$\\times$\n\n【分析】1 公顷 $=10000$ 平方米, 1000000 平方米 $=1$ 平方千米, 低级单位化成高级单位就除以它们之间的进率,依此换算并判断。\n\n【详解】 $30000 \\div 10000=3 ; 30000 \\div 1000000=0.03$\n\n30000 平方米 $=3$ 公顷 $=0.03$ 平方千米。\n\n故答案为: $\\times$\n\n【点睛】此题考查的是面积单位之间的换算, 熟记它们之间的进率是解答本题的关键。", "solution": "null", "level": "四年级", "question": "30000 平方米 $=300$ 公顷 $=3$ 平方千米。 $(\\quad)$", "options": [], "subject": "度量几何学", "analysis": "解$\\times$\n\n【分析】1 公顷 $=10000$ 平方米, 1000000 平方米 $=1$ 平方千米, 低级单位化成高级单位就除以它们之间的进率,依此换算并判断。\n\n【详解】 $30000 \\div 10000=3 ; 30000 \\div 1000000=0.03$\n\n30000 平方米 $=3$ 公顷 $=0.03$ 平方千米。\n\n故答案为: $\\times$\n\n【点睛】此题考查的是面积单位之间的换算, 熟记它们之间的进率是解答本题的关键。"} {"id": "28384", "image": ["13613.jpg", "13614.jpg", "13614.jpg"], "answer": "解135 平方米\n\n【分析】根据题意可将这块菜地分成 1 个长为 $27 \\mathrm{~m}$, 宽为 $3 \\mathrm{~m}$ 的长方形和 1 个长为 $3 \\mathrm{~m}$, 宽为 $18 \\mathrm{~m}$ 的长方形, 长方形的面积 $=$ 长 $\\times$ 宽, 依此计算出这两个长方形的面积后, 再相加即可。根据混合运算的计算顺序列式并根据整数乘法分配律的特点计算即可。\n\n\n\n$27 \\times 3+3 \\times 18$\n\n$=(27+18) \\times 3$\n\n$=45 \\times 3$\n\n$=135$ (平方米)\n\n答:这块菜地的面积是 135 平方米。\n\n【点睛】此题考查的是运用乘法分配律的特点解决实际问题,应熟练掌握长方形的面积的计算。", "solution": "null", "level": "四年级", "question": "王大伯家有一块菜地(如下图),这块菜地的面积是多少平方米?\n\n", "options": [], "subject": "度量几何学", "analysis": "解135 平方米\n\n【分析】根据题意可将这块菜地分成 1 个长为 $27 \\mathrm{~m}$, 宽为 $3 \\mathrm{~m}$ 的长方形和 1 个长为 $3 \\mathrm{~m}$, 宽为 $18 \\mathrm{~m}$ 的长方形, 长方形的面积 $=$ 长 $\\times$ 宽, 依此计算出这两个长方形的面积后, 再相加即可。根据混合运算的计算顺序列式并根据整数乘法分配律的特点计算即可。\n\n\n\n$27 \\times 3+3 \\times 18$\n\n$=(27+18) \\times 3$\n\n$=45 \\times 3$\n\n$=135$ (平方米)\n\n答:这块菜地的面积是 135 平方米。\n\n【点睛】此题考查的是运用乘法分配律的特点解决实际问题,应熟练掌握长方形的面积的计算。"} {"id": "28685", "image": [], "answer": "解$\\times$\n\n【分析】三角形 3 条边的关系是: 任意两边的长度之和大于第三边, 任意两边的长度之差小于第三边,此题依此判断即可。\n\n【详解】 10 厘米 -8 厘米 $<$ 第三边的长度 $<10$ 厘米 +8 厘米\n\n2 厘米 $<$ 第三边的长度 $<18$ 厘米\n\n$2+1=3$ (厘米) $; 18-1=17$ (厘米)\n\n即第 3 根小棒最短是 3 厘米, 最长为 17 厘米。\n\n故答案为: $\\times$\n\n【点睛】熟练掌握三角形三边的关系是解答本题的关键。", "solution": "null", "level": "四年级", "question": "用三根长度为整厘米数的小棒围成一个三角形, 如果其中两根小棒分别为 8 厘米, 10 厘米, 那么第 3 根小棒最短是 3 厘米, 最长为 19 厘米。()", "options": [], "subject": "度量几何学", "analysis": "解$\\times$\n\n【分析】三角形 3 条边的关系是: 任意两边的长度之和大于第三边, 任意两边的长度之差小于第三边,此题依此判断即可。\n\n【详解】 10 厘米 -8 厘米 $<$ 第三边的长度 $<10$ 厘米 +8 厘米\n\n2 厘米 $<$ 第三边的长度 $<18$ 厘米\n\n$2+1=3$ (厘米) $; 18-1=17$ (厘米)\n\n即第 3 根小棒最短是 3 厘米, 最长为 17 厘米。\n\n故答案为: $\\times$\n\n【点睛】熟练掌握三角形三边的关系是解答本题的关键。"} {"id": "28686", "image": ["13712.jpg", "13712.jpg"], "answer": "解$\\sqrt{ }$\n\n【详解】三角形任意两边的和大于第三边; 任意两边的差小于第三边;\n\n例如: 三角形条边长分别为 2 厘米、 3 厘米、 4 厘米, 最短的两条边相加大于第三边 $2+3>4$, 最长的一条边减最短的一条边小于第三边 $4-2<3$ 。\n\n\n\n故答案为: $\\sqrt{ }$", "solution": "null", "level": "四年级", "question": "三角形任意两边的和大于第三边;任意两边的差小于第三边。()", "options": [], "subject": "度量几何学", "analysis": "解$\\sqrt{ }$\n\n【详解】三角形任意两边的和大于第三边; 任意两边的差小于第三边;\n\n例如: 三角形条边长分别为 2 厘米、 3 厘米、 4 厘米, 最短的两条边相加大于第三边 $2+3>4$, 最长的一条边减最短的一条边小于第三边 $4-2<3$ 。\n\n\n\n故答案为: $\\sqrt{ }$"} {"id": "28687", "image": [], "answer": "解$\\sqrt{ }$\n【分析】因为三角形的内角和是 $180^{\\circ}$, 一个三角形中若有两个直角或钝角, 就超过 $180^{\\circ}$, 就构不成一个三角形了, 所以一个三角形, 至少应有两个锐角。\n\n【详解】由三角形的内角和可知, 一个三角形中至少有两个锐角。所以原题说法正确。\n\n故答案为: $\\sqrt{ }$\n\n【点睛】本题考查了三角形的内角和定理, 属于基础题, 注意三角形内角和定理的灵活运用。", "solution": "null", "level": "四年级", "question": "三角形里至少有两个锐角。()", "options": [], "subject": "度量几何学", "analysis": "解$\\sqrt{ }$\n【分析】因为三角形的内角和是 $180^{\\circ}$, 一个三角形中若有两个直角或钝角, 就超过 $180^{\\circ}$, 就构不成一个三角形了, 所以一个三角形, 至少应有两个锐角。\n\n【详解】由三角形的内角和可知, 一个三角形中至少有两个锐角。所以原题说法正确。\n\n故答案为: $\\sqrt{ }$\n\n【点睛】本题考查了三角形的内角和定理, 属于基础题, 注意三角形内角和定理的灵活运用。"} {"id": "28688", "image": [], "answer": "解$\\times$\n\n【分析】根据三角形的特性、平行四边形的特性, 三角形具有稳定性, 而平行四边形容易变形; 据此判断。\n\n【详解】三角形稳定性是指三角形具有稳定性, 有着稳固、坚定、耐压的特点, 例如: 埃及金字塔、钢轨、三角形框架、起重机、三角形吊臂、屋顶、三角形钢架、钢架桥和埃菲尔铁塔都以三角形形状建造;\n\n平行四边形具有不稳定性, 生活中也有广泛应用, 例如: 活动挂架, 伸缩门等; 所以原题说法错误。故答案为: $\\times$\n\n【点睛】此题考查的目的是理解掌握三角形、平行四边形的特性及应用。", "solution": "null", "level": "四年级", "question": "三角形具有稳定性, 平行四边形也具有稳定性。()", "options": [], "subject": "度量几何学", "analysis": "解$\\times$\n\n【分析】根据三角形的特性、平行四边形的特性, 三角形具有稳定性, 而平行四边形容易变形; 据此判断。\n\n【详解】三角形稳定性是指三角形具有稳定性, 有着稳固、坚定、耐压的特点, 例如: 埃及金字塔、钢轨、三角形框架、起重机、三角形吊臂、屋顶、三角形钢架、钢架桥和埃菲尔铁塔都以三角形形状建造;\n\n平行四边形具有不稳定性, 生活中也有广泛应用, 例如: 活动挂架, 伸缩门等; 所以原题说法错误。故答案为: $\\times$\n\n【点睛】此题考查的目的是理解掌握三角形、平行四边形的特性及应用。"} {"id": "28691", "image": [], "answer": "解30 度\n\n【分析】根据等腰三角形的特征, 两个底角相等, 用 $75^{\\circ} \\times 2$ 即可计算出两个底角的度数, 用 $180^{\\circ}$ 减去两个底角的度数即可, 据此解决。\n\n【详解】 $180^{\\circ}-75^{\\circ} \\times 2$\n\n$=180^{\\circ}-150^{\\circ}$\n\n$=30^{\\circ}$\n\n答: 它的顶角是 30 度。\n\n【点睛】解决本题的关键是熟练掌握等腰三角形的特征。", "solution": "null", "level": "四年级", "question": "等腰三角形的一个底角是 75 度, 它的顶角是多少度?", "options": [], "subject": "度量几何学", "analysis": "解30 度\n\n【分析】根据等腰三角形的特征, 两个底角相等, 用 $75^{\\circ} \\times 2$ 即可计算出两个底角的度数, 用 $180^{\\circ}$ 减去两个底角的度数即可, 据此解决。\n\n【详解】 $180^{\\circ}-75^{\\circ} \\times 2$\n\n$=180^{\\circ}-150^{\\circ}$\n\n$=30^{\\circ}$\n\n答: 它的顶角是 30 度。\n\n【点睛】解决本题的关键是熟练掌握等腰三角形的特征。"} {"id": "28692", "image": [], "answer": "解108 厘米\n\n【分析】等腰三角形的两腰相等, 因此用铁丝的总长度减去 2 个腰长即可, 依此计算。\n\n【详解】 $180-36 \\times 2$\n\n$=180-72$\n\n$=108$ (厘米)\n\n答: 底边长为 108 厘米。\n\n【点睛】此题考查的是三角形的周长, 熟练掌握等腰三角形的特点是解答此题的关键。", "solution": "null", "level": "四年级", "question": "某同学用一根 180 厘米长的铁丝围一个等腰三角形, 经测量一条腰长为 36 厘米, 底边长为多少厘米?", "options": [], "subject": "度量几何学", "analysis": "解108 厘米\n\n【分析】等腰三角形的两腰相等, 因此用铁丝的总长度减去 2 个腰长即可, 依此计算。\n\n【详解】 $180-36 \\times 2$\n\n$=180-72$\n\n$=108$ (厘米)\n\n答: 底边长为 108 厘米。\n\n【点睛】此题考查的是三角形的周长, 熟练掌握等腰三角形的特点是解答此题的关键。"} {"id": "27261", "image": [], "answer": "解$\\sqrt{ }$\n\n【分析】加法各部分间的关系:加数+加数=和,一个加数=和一另一个加数; 据此解题即可。\n\n【详解】根据分析可知,\n\n由“ $\\square+o=\\triangle$ ”可得:\n\n$\\triangle-\\square=0$\n\n$\\triangle-\\circ=\\square$\n\n所以, 原题干说法正确。\n\n故答案为: $\\sqrt{ }$\n\n【点睛】正确理解加减法各部分之间的关系, 是解答此题的关键。", "solution": "null", "level": "四年级", "question": "如果 $\\square+0=\\triangle$, 那么 $\\triangle-\\square=0$ 。( )", "options": [], "subject": "逻辑题", "analysis": "解$\\sqrt{ }$\n\n【分析】加法各部分间的关系:加数+加数=和,一个加数=和一另一个加数; 据此解题即可。\n\n【详解】根据分析可知,\n\n由“ $\\square+o=\\triangle$ ”可得:\n\n$\\triangle-\\square=0$\n\n$\\triangle-\\circ=\\square$\n\n所以, 原题干说法正确。\n\n故答案为: $\\sqrt{ }$\n\n【点睛】正确理解加减法各部分之间的关系, 是解答此题的关键。"} {"id": "27269", "image": [], "answer": "解3 辆大车 1 辆小车; 3900 元\n\n【分析】先比较两种每个座位的价格, 哪种车的便宜就尽量租那种车型, 然后再根据人数进行方案调整, 尽量不要让车有空座, 此时是最便宜的。据此解答。\n\n【详解】 $1000 \\div 40=25$ (元)\n\n$900 \\div 30=30$ (元)\n\n25 元 $<30$ 元\n\n所以尽量租大车。\n\n$150 \\div 40=3$ (辆) $\\ldots . .30$ (人)\n\n方案一: 租 4 辆大车, 租金需要: $1000 \\times 4=4000$ (元)\n\n方案二: 租 3 辆大车 1 辆小车, 租金需要:\n\n$1000 \\times 3+900$\n\n$=3000+900$\n\n$=3900$ (元)\n\n4000 元 $>3900$ 元\n\n答: 租 3 辆大车 1 辆小车最省钱, 至少需要 3900 元。\n\n【点睛】本题主要考查学生对租船问题的掌握。解决此类题目时, 要注意尽量不要空座。", "solution": "null", "level": "四年级", "question": "学校四年级学生去春游, 师生共 150 人坐车。大车可坐 40 人, 租金 1000 元; 小车可坐 30 人,租金 900 元。怎样租车最省钱?至少需要多少钱?", "options": [], "subject": "组合数学", "analysis": "解3 辆大车 1 辆小车; 3900 元\n\n【分析】先比较两种每个座位的价格, 哪种车的便宜就尽量租那种车型, 然后再根据人数进行方案调整, 尽量不要让车有空座, 此时是最便宜的。据此解答。\n\n【详解】 $1000 \\div 40=25$ (元)\n\n$900 \\div 30=30$ (元)\n\n25 元 $<30$ 元\n\n所以尽量租大车。\n\n$150 \\div 40=3$ (辆) $\\ldots . .30$ (人)\n\n方案一: 租 4 辆大车, 租金需要: $1000 \\times 4=4000$ (元)\n\n方案二: 租 3 辆大车 1 辆小车, 租金需要:\n\n$1000 \\times 3+900$\n\n$=3000+900$\n\n$=3900$ (元)\n\n4000 元 $>3900$ 元\n\n答: 租 3 辆大车 1 辆小车最省钱, 至少需要 3900 元。\n\n【点睛】本题主要考查学生对租船问题的掌握。解决此类题目时, 要注意尽量不要空座。"} {"id": "27385", "image": [], "answer": "解解: $40 \\times 20+15 \\times 2$\n\n$$\n\\begin{aligned}\n& =800+30 \\\\\n& =830 \\text { (箱) } \\\\\n& 40 \\times 250+2 \\times 220 \\\\\n& =10000+440 \\\\\n& =10440 \\text { (元) }\n\\end{aligned}\n$$\n\n答: 租 40 辆大车, 2 辆小车花费最少,最少费用是 10440 元。", "solution": "null", "level": "四年级", "question": "双十一真忙, 运输公司要把 830 箱货物从成都运到重庆, 其中大卡车每次装 20 箱, 运费 250 元。小卡车每次装 15 箱, 运费 220 元。若要一次运走, 怎样租卡车的花费最少, 最少费用是多少元?", "options": [], "subject": "组合数学", "analysis": "解解: $40 \\times 20+15 \\times 2$\n\n$$\n\\begin{aligned}\n& =800+30 \\\\\n& =830 \\text { (箱) } \\\\\n& 40 \\times 250+2 \\times 220 \\\\\n& =10000+440 \\\\\n& =10440 \\text { (元) }\n\\end{aligned}\n$$\n\n答: 租 40 辆大车, 2 辆小车花费最少,最少费用是 10440 元。"} {"id": "27409", "image": [], "answer": "解8 个大人和 2 个小孩购买团体票, 剩下的小孩购买小孩票; 1740 元。\n\n【分析】方案一需要的钱 $=$ 大人的人数 $\\times$ 大人的票价十小孩的人数 $\\times$ 小孩的票价;\n\n方案二需要的钱 $=$ 大人和小孩的总人数 $\\times$ 团体票的票价;\n\n还有一种, 8 个大人和 2 个小孩购买团体票,剩下的小孩购买小孩票;\n\n方案三需要的钱 $=10 \\times$ 团体票的票价十剩下的小孩人数 $\\times$ 小孩票的票价, 依此计算并比较即可。\n\n【详解】方案一:\n\n$8 \\times 180+4 \\times 120$\n\n$=1440+480$\n\n$=1920 ($ 元)\n\n方案二:\n\n$(8+4) \\times 150$\n\n$=12 \\times 150$\n\n$=1800$ (元)\n\n方案三:\n\n$(8+2) \\times 150+(4-2) \\times 120$\n\n$=10 \\times 150+2 \\times 120$\n\n$=1500+240$\n$=1740$ (元)\n\n1740 元 $<1800$ 元 $<1920$ 元, 即方案三最省钱\n\n答: 8 个大人和 2 个小孩购买团体票,剩下的小孩购买小孩票最省钱,需要花费 1740 元。\n\n【点睛】此题考查的是优化问题的计算,应分别计算出可能的方案需要的钱数再解答。", "solution": "null", "level": "四年级", "question": "旅行社推出“天池一日游”活动, 现推出两种方案:\n\n方案一:大人 180 元/人,小孩 120 元/人方案二:团体 10 人以上(含 10 人),150 元/人\n\n8 个大人, 4 个小孩, 怎样买票最划算?需要花费多少元?", "options": [], "subject": "组合数学", "analysis": "解8 个大人和 2 个小孩购买团体票, 剩下的小孩购买小孩票; 1740 元。\n\n【分析】方案一需要的钱 $=$ 大人的人数 $\\times$ 大人的票价十小孩的人数 $\\times$ 小孩的票价;\n\n方案二需要的钱 $=$ 大人和小孩的总人数 $\\times$ 团体票的票价;\n\n还有一种, 8 个大人和 2 个小孩购买团体票,剩下的小孩购买小孩票;\n\n方案三需要的钱 $=10 \\times$ 团体票的票价十剩下的小孩人数 $\\times$ 小孩票的票价, 依此计算并比较即可。\n\n【详解】方案一:\n\n$8 \\times 180+4 \\times 120$\n\n$=1440+480$\n\n$=1920 ($ 元)\n\n方案二:\n\n$(8+4) \\times 150$\n\n$=12 \\times 150$\n\n$=1800$ (元)\n\n方案三:\n\n$(8+2) \\times 150+(4-2) \\times 120$\n\n$=10 \\times 150+2 \\times 120$\n\n$=1500+240$\n$=1740$ (元)\n\n1740 元 $<1800$ 元 $<1920$ 元, 即方案三最省钱\n\n答: 8 个大人和 2 个小孩购买团体票,剩下的小孩购买小孩票最省钱,需要花费 1740 元。\n\n【点睛】此题考查的是优化问题的计算,应分别计算出可能的方案需要的钱数再解答。"} {"id": "27538", "image": [], "answer": "解租 6 辆大客车和 2 辆小客车最省钱; 3000 元\n\n【分析】先分别算出大客车和小客车每人的租金, 尽量多租用每人租金更少的车会最省钱, 由此根据人数及车辆的限坐人数, 通过调整尽量使之满座, 据此解答。\n\n【详解】 $400 \\div 50=8$ (元)\n\n$300 \\div 30=10$ (元)\n\n所以尽量多租用大客车且尽量满载最省钱;\n\n$358 \\div 50=7$ (辆) $\\ldots \\ldots .8$ (人), 调整为租 6 辆大客车, 剩余 58 人租 2 辆小客车;\n\n租金: $400 \\times 6+300 \\times 2$\n\n$=2400+600$\n\n$=3000 ($ 元)\n\n答: 租 6 辆大客车和 2 辆小客车最省钱, 需要 3000 元。\n\n【点睛】此题主要考查三位数除以两位数的除法在租船方案选择中的实际运用。", "solution": "null", "level": "四年级", "question": "先锋小学四年级师生 358 人去郊游, 大客车限载 50 人, 每天每辆车租金 400 元; 小客车限载 30 人, 每天每辆车租金 300 元。怎样租车最省钱?需要多少钱?", "options": [], "subject": "组合数学", "analysis": "解租 6 辆大客车和 2 辆小客车最省钱; 3000 元\n\n【分析】先分别算出大客车和小客车每人的租金, 尽量多租用每人租金更少的车会最省钱, 由此根据人数及车辆的限坐人数, 通过调整尽量使之满座, 据此解答。\n\n【详解】 $400 \\div 50=8$ (元)\n\n$300 \\div 30=10$ (元)\n\n所以尽量多租用大客车且尽量满载最省钱;\n\n$358 \\div 50=7$ (辆) $\\ldots \\ldots .8$ (人), 调整为租 6 辆大客车, 剩余 58 人租 2 辆小客车;\n\n租金: $400 \\times 6+300 \\times 2$\n\n$=2400+600$\n\n$=3000 ($ 元)\n\n答: 租 6 辆大客车和 2 辆小客车最省钱, 需要 3000 元。\n\n【点睛】此题主要考查三位数除以两位数的除法在租船方案选择中的实际运用。"} {"id": "27643", "image": [], "answer": "解(1)3 辆大车, 1 辆小车, 或者 5 辆小车;\n\n(2)安排 5 辆小车; 2500 元\n\n【分析】(1)大车的载质量 $\\times$ 大车的辆数 + 小车的载质量 $\\times$ 小车辆数 $=$ 这两种车运的总重量, 只要这两种车运的总重量刚好为 30 吨即可。\n\n(2)要使运费最少, 则每辆车都应装满, 并且使运的总重量刚好为 30 吨即可, 因此可计算出 3 辆大车, 1 辆小车的总运费, 以及 5 辆小车的运费, 然后再比较即可。\n\n每辆大车的运费 $\\times$ 大车的辆数 + 每辆小车的运费 $\\times$ 小车的辆数 $=$ 总运费, 依此计算即可。\n\n【详解】(1) $8 \\times 3+6 \\times 1$\n\n$=24+6$\n\n$=30$ (吨)\n\n$30 \\div 6=5$ (辆)\n\n即安排 3 辆大车, 1 辆小车, 或者安排 5 辆小车\n\n答: 安排 3 辆大车, 1 辆小车, 或者安排 5 辆小车, 能恰好一次运完。\n\n$$\n\\begin{aligned}\n& \\text { (2) } 700 \\times 3+500 \\\\\n& =2100+500 \\\\\n& =2600 \\text { (元 }) \\\\\n& 500 \\times 5=2500 \\text { (元 }) \\\\\n& 2500 \\text { 元 }<2600 \\text { 元 }\n\\end{aligned}\n$$\n\n答: 安排 5 辆小车最省钱, 至少需要 2500 元。\n\n【点睛】此题考查的是经济问题的计算, 应先计算出每种方案需要费用再比较。", "solution": "null", "level": "四年级", "question": "疫情期间, 某运输公司需要运送 30 吨防疫物资, 用如下两种车进行运送: 大车载质量 8 吨, 小车载质量 6 吨。\n\n①如果每辆车都装满, 怎样安排能恰好一次运完?\n\n(2)如果大车每辆运费 700 元,小车每辆运费 500 元。怎样安排最省钱?最少需要多少钱?", "options": [], "subject": "组合数学", "analysis": "解(1)3 辆大车, 1 辆小车, 或者 5 辆小车;\n\n(2)安排 5 辆小车; 2500 元\n\n【分析】(1)大车的载质量 $\\times$ 大车的辆数 + 小车的载质量 $\\times$ 小车辆数 $=$ 这两种车运的总重量, 只要这两种车运的总重量刚好为 30 吨即可。\n\n(2)要使运费最少, 则每辆车都应装满, 并且使运的总重量刚好为 30 吨即可, 因此可计算出 3 辆大车, 1 辆小车的总运费, 以及 5 辆小车的运费, 然后再比较即可。\n\n每辆大车的运费 $\\times$ 大车的辆数 + 每辆小车的运费 $\\times$ 小车的辆数 $=$ 总运费, 依此计算即可。\n\n【详解】(1) $8 \\times 3+6 \\times 1$\n\n$=24+6$\n\n$=30$ (吨)\n\n$30 \\div 6=5$ (辆)\n\n即安排 3 辆大车, 1 辆小车, 或者安排 5 辆小车\n\n答: 安排 3 辆大车, 1 辆小车, 或者安排 5 辆小车, 能恰好一次运完。\n\n$$\n\\begin{aligned}\n& \\text { (2) } 700 \\times 3+500 \\\\\n& =2100+500 \\\\\n& =2600 \\text { (元 }) \\\\\n& 500 \\times 5=2500 \\text { (元 }) \\\\\n& 2500 \\text { 元 }<2600 \\text { 元 }\n\\end{aligned}\n$$\n\n答: 安排 5 辆小车最省钱, 至少需要 2500 元。\n\n【点睛】此题考查的是经济问题的计算, 应先计算出每种方案需要费用再比较。"} {"id": "27785", "image": ["13561.jpg"], "answer": "解3 辆大车, 1 辆小车; 1110 元\n\n【分析】先根据 “单价 $=$ 总价 $\\div$ 数量”分别计算出租大车和小车时每人需要的钱, 要使租车最省钱, 则应尽量租最便宜的一种车型, 并且使每辆车都坐满, 没有空位; 因此用总人数除以最便宜的一种车型可坐的人数, 再根据计算出的结果进行解答即可。\n\n一共需要的租金 $=$ 租小车的辆数 $\\times$ 每辆小车的租金十租大车的辆数 $\\times$ 每辆大车的租金, 依此计算并解答。\n\n【详解】 $210 \\div 6=35$ (元/人)\n\n$300 \\div 10=30$ (元/人)\n\n30 元 $<35$ 元, 即租大车便宜\n\n$36 \\div 10=3$ (辆) $\\qquad$ 6 (人)\n\n剩下的 6 人刚好可租 1 辆小车\n\n$1 \\times 210+3 \\times 300$\n$=210+900$\n\n$=1100$ (元)\n\n答: 租 3 辆大车, 1 辆小车最省钱, 租金是 1100 元。\n\n【点睛】此题考查的是优化问题的计算, 要使租车最省钱, 则应尽量多租最便宜的一种车型, 并且使每辆车都坐满。", "solution": "null", "level": "四年级", "question": "北京故宫博物院是第一批全国爱国主义教育示范基地, 某公司为激励员工组织员工带孩子一起去参观故宫博物院。怎样租车最省钱?租金共多少钱?\n\n", "options": [], "subject": "组合数学", "analysis": "解3 辆大车, 1 辆小车; 1110 元\n\n【分析】先根据 “单价 $=$ 总价 $\\div$ 数量”分别计算出租大车和小车时每人需要的钱, 要使租车最省钱, 则应尽量租最便宜的一种车型, 并且使每辆车都坐满, 没有空位; 因此用总人数除以最便宜的一种车型可坐的人数, 再根据计算出的结果进行解答即可。\n\n一共需要的租金 $=$ 租小车的辆数 $\\times$ 每辆小车的租金十租大车的辆数 $\\times$ 每辆大车的租金, 依此计算并解答。\n\n【详解】 $210 \\div 6=35$ (元/人)\n\n$300 \\div 10=30$ (元/人)\n\n30 元 $<35$ 元, 即租大车便宜\n\n$36 \\div 10=3$ (辆) $\\qquad$ 6 (人)\n\n剩下的 6 人刚好可租 1 辆小车\n\n$1 \\times 210+3 \\times 300$\n$=210+900$\n\n$=1100$ (元)\n\n答: 租 3 辆大车, 1 辆小车最省钱, 租金是 1100 元。\n\n【点睛】此题考查的是优化问题的计算, 要使租车最省钱, 则应尽量多租最便宜的一种车型, 并且使每辆车都坐满。"} {"id": "14000", "image": ["2872.jpg"], "answer": "(画法不唯一)", "solution": "null", "level": "四年级", "question": "请你分别在已知图案上加上一个小正方形使它成为轴对称图形, 再分别画出它们的一条对称轴。(每个图案画出一种即可)(12 分)\n", "options": [], "subject": "组合几何学", "analysis": ""} {"id": "14137", "image": [], "answer": "64", "solution": "null", "level": "四年级", "question": "明明想在 中再添一个小正方体, 又使得从左面看形状不变, 有()种摆法; 要使从上面看形状不变, 有 $(\\quad)$ 种摆法。", "options": [], "subject": "组合几何学", "analysis": ""} {"id": "27327", "image": [], "answer": "解方案二购票最省钱; 800 元\n\n【分析】方案一:分开购头, 即购买 5 张成人票和 5 张儿童票, 根据总价 $=$ 单价 $\\times$ 数量, 分别求出需要的钱数再相加;\n\n方案二:全部购买团体票, 一共需要购买 $5+5=10$ (张) 团体票, 由此求出需要的钱数;\n\n然后比较两种方案需要的钱数, 即可求解。\n\n【详解】方案一: $150 \\times 5+60 \\times 5$\n\n$=750+300$\n$=1050$ (元)\n\n方案二: $(5+5) \\times 80$\n\n$=10 \\times 80$\n\n$=800$ (元)\n\n$1050>800$\n\n答: 方案二购票最省钱; 最少花 800 元。\n\n【点睛】本题考查优化的问题:根据条件算出每种方案花的钱数去判断即可。", "solution": "null", "level": "四年级", "question": "旅行社推出“ $x \\times$ 风景区一日游”的两种价格方案。成人 5 人,儿童 5 人,怎样购票最省钱?最少花多少钱?\n\n方案一成人每人 150 元,儿童每人 60 元。\n\n方案二团体 10 人以上(含 10 人)每人 80 元。", "options": [], "subject": "代数", "analysis": "解方案二购票最省钱; 800 元\n\n【分析】方案一:分开购头, 即购买 5 张成人票和 5 张儿童票, 根据总价 $=$ 单价 $\\times$ 数量, 分别求出需要的钱数再相加;\n\n方案二:全部购买团体票, 一共需要购买 $5+5=10$ (张) 团体票, 由此求出需要的钱数;\n\n然后比较两种方案需要的钱数, 即可求解。\n\n【详解】方案一: $150 \\times 5+60 \\times 5$\n\n$=750+300$\n$=1050$ (元)\n\n方案二: $(5+5) \\times 80$\n\n$=10 \\times 80$\n\n$=800$ (元)\n\n$1050>800$\n\n答: 方案二购票最省钱; 最少花 800 元。\n\n【点睛】本题考查优化的问题:根据条件算出每种方案花的钱数去判断即可。"} {"id": "27696", "image": [], "answer": "解(1) 3 辆大车, 1 辆小车, 或者 5 辆小车;\n\n(2)安排 5 辆小车; 2500 元\n\n【分析】(1)大车的载质量 $\\times$ 大车的辆数 + 小车的载质量 $\\times$ 小车辆数 $=$ 这两种车运的总重量, 只要这两种车运的总重量刚好为 30 吨即可。\n\n(2)要使运费最少, 则每辆车都应装满, 并且使运的总重量刚好为 30 吨即可, 因此可计算出 3 辆大车, 1 辆小车的总运费, 以及 5 辆小车的运费, 然后再比较即可。\n\n每辆大车的运费 $\\times$ 大车的辆数 + 每辆小车的运费 $\\times$ 小车的辆数 $=$ 总运费, 依此计算即可。\n\n【详解】(1) $8 \\times 3+6 \\times 1$\n\n$=24+6$\n\n$=30$ (吨)\n\n$30 \\div 6=5$ (辆)\n\n即安排 3 辆大车, 1 辆小车, 或者安排 5 辆小车\n\n答: 安排 3 辆大车, 1 辆小车, 或者安排 5 辆小车, 能恰好一次运完。\n\n(2) $700 \\times 3+500$\n\n$=2100+500$\n\n$=2600 ($ 元)\n$500 \\times 5=2500 ($ 元 $)$\n\n2500 元 $<2600$ 元\n\n答: 安排 5 辆小车最省钱, 至少需要 2500 元。\n\n【点睛】此题考查的是经济问题的计算, 应先计算出每种方案需要费用再比较。", "solution": "null", "level": "四年级", "question": "疫情期间, 某运输公司需要运送 30 吨防疫物资, 用如下两种车进行运送: 大车载质量 8 吨, 小车载质量 6 吨。\n\n(1)如果每辆车都装满, 怎样安排能恰好一次运完?\n\n(2)如果大车每辆运费 700 元,小车每辆运费 500 元。怎样安排最省钱?最少需要多少钱?", "options": [], "subject": "代数", "analysis": "解(1) 3 辆大车, 1 辆小车, 或者 5 辆小车;\n\n(2)安排 5 辆小车; 2500 元\n\n【分析】(1)大车的载质量 $\\times$ 大车的辆数 + 小车的载质量 $\\times$ 小车辆数 $=$ 这两种车运的总重量, 只要这两种车运的总重量刚好为 30 吨即可。\n\n(2)要使运费最少, 则每辆车都应装满, 并且使运的总重量刚好为 30 吨即可, 因此可计算出 3 辆大车, 1 辆小车的总运费, 以及 5 辆小车的运费, 然后再比较即可。\n\n每辆大车的运费 $\\times$ 大车的辆数 + 每辆小车的运费 $\\times$ 小车的辆数 $=$ 总运费, 依此计算即可。\n\n【详解】(1) $8 \\times 3+6 \\times 1$\n\n$=24+6$\n\n$=30$ (吨)\n\n$30 \\div 6=5$ (辆)\n\n即安排 3 辆大车, 1 辆小车, 或者安排 5 辆小车\n\n答: 安排 3 辆大车, 1 辆小车, 或者安排 5 辆小车, 能恰好一次运完。\n\n(2) $700 \\times 3+500$\n\n$=2100+500$\n\n$=2600 ($ 元)\n$500 \\times 5=2500 ($ 元 $)$\n\n2500 元 $<2600$ 元\n\n答: 安排 5 辆小车最省钱, 至少需要 2500 元。\n\n【点睛】此题考查的是经济问题的计算, 应先计算出每种方案需要费用再比较。"} {"id": "27965", "image": [], "answer": "解$\\times$\n\n【分析】乘法分配律: 两个数的和与一个数相乘, 可以先把它们与这个数分别相乘, 再相加; 用字母表示: $(\\mathrm{a}+\\mathrm{b}) \\times \\mathrm{c}=\\mathrm{a} \\times \\mathrm{c}+\\mathrm{b} \\times \\mathrm{c}$ ,据此进行解答。\n\n【详解】乘法分配律用字母表示为 $(\\mathrm{a}+\\mathrm{b}) \\times \\mathrm{c}=\\mathrm{a} \\times \\mathrm{c}+\\mathrm{b} \\times \\mathrm{c}$,\n\n故答案为: $\\times$\n\n【点睛】本题主要考查了学生对用字母表示乘法分配律的掌握。", "solution": "null", "level": "四年级", "question": "乘法分配律用字母表示为 $(\\mathrm{a}+\\mathrm{b}) \\times \\mathrm{c}=\\mathrm{a} \\times \\mathrm{b}+\\mathrm{a} \\times \\mathrm{c}$ 。( )", "options": [], "subject": "代数", "analysis": "解$\\times$\n\n【分析】乘法分配律: 两个数的和与一个数相乘, 可以先把它们与这个数分别相乘, 再相加; 用字母表示: $(\\mathrm{a}+\\mathrm{b}) \\times \\mathrm{c}=\\mathrm{a} \\times \\mathrm{c}+\\mathrm{b} \\times \\mathrm{c}$ ,据此进行解答。\n\n【详解】乘法分配律用字母表示为 $(\\mathrm{a}+\\mathrm{b}) \\times \\mathrm{c}=\\mathrm{a} \\times \\mathrm{c}+\\mathrm{b} \\times \\mathrm{c}$,\n\n故答案为: $\\times$\n\n【点睛】本题主要考查了学生对用字母表示乘法分配律的掌握。"} {"id": "27895", "image": [], "answer": "解500 千米\n\n【分析】根据路程 $=$ 速度 $\\times$ 时间, 分别求出甲车和乙车行驶的路程。再将两个路程加起来, 即为 $\\mathrm{AB}$两站的距离。\n\n【详解】 $48 \\times 5+52 \\times 5$\n\n$=(48+52) \\times 5$\n\n$=100 \\times 5$\n$=500$ (千米)\n\n答: $\\mathrm{AB}$ 两站相距 500 千米。\n\n【点睛】本题考查行程问题, 利用公式路程 $=$ 速度 $\\times$ 时间列出算式, 再根据乘法分配律进行简算。", "solution": "null", "level": "四年级", "question": "甲、乙两辆汽车同时从合水汽车站背向开出。甲车向北行驶, 每小时行 48 千米; 乙车向南行驶,每小时行 52 千米, 5 小时后两车分别达到停车点 $\\mathrm{A} 、 \\mathrm{~B}$ 。 $\\mathrm{AB}$ 两站相距多少千米?", "options": [], "subject": "解析几何", "analysis": "解500 千米\n\n【分析】根据路程 $=$ 速度 $\\times$ 时间, 分别求出甲车和乙车行驶的路程。再将两个路程加起来, 即为 $\\mathrm{AB}$两站的距离。\n\n【详解】 $48 \\times 5+52 \\times 5$\n\n$=(48+52) \\times 5$\n\n$=100 \\times 5$\n$=500$ (千米)\n\n答: $\\mathrm{AB}$ 两站相距 500 千米。\n\n【点睛】本题考查行程问题, 利用公式路程 $=$ 速度 $\\times$ 时间列出算式, 再根据乘法分配律进行简算。"} {"id": "28684", "image": [], "answer": "解$\\sqrt{ }$\n\n【分析】三角形的内角和是 $180^{\\circ}$, 据此把这三个角的度数相加, 即可判断。\n\n【详解】 $65^{\\circ}+95^{\\circ}+20^{\\circ}$\n\n$=160^{\\circ}+20^{\\circ}$\n\n$=180^{\\circ}$\n\n所以, 一个三角形中, 有一个角是 $65^{\\circ}$ 另外的两个角可能是 $95^{\\circ}$ 和 $20^{\\circ}$, 原题说法正确。\n\n故答案为: $\\sqrt{ }$\n\n【点睛】本题考查了运用三角形的内角和是 $180^{\\circ}$ 解决实际问题。", "solution": "null", "level": "四年级", "question": "一个三角形中, 有一个角是 $65^{\\circ}$ 另外的两个角可能是 $95^{\\circ}$ 和 $20^{\\circ}$ 。()", "options": [], "subject": "解析几何", "analysis": "解$\\sqrt{ }$\n\n【分析】三角形的内角和是 $180^{\\circ}$, 据此把这三个角的度数相加, 即可判断。\n\n【详解】 $65^{\\circ}+95^{\\circ}+20^{\\circ}$\n\n$=160^{\\circ}+20^{\\circ}$\n\n$=180^{\\circ}$\n\n所以, 一个三角形中, 有一个角是 $65^{\\circ}$ 另外的两个角可能是 $95^{\\circ}$ 和 $20^{\\circ}$, 原题说法正确。\n\n故答案为: $\\sqrt{ }$\n\n【点睛】本题考查了运用三角形的内角和是 $180^{\\circ}$ 解决实际问题。"} {"id": "28694", "image": [], "answer": "解$90^{\\circ}$\n\n【分析】等腰三角形两个底角相等, 顶角是一个底角的 2 倍, 即一个顶角相当于 2 个底角, 则这个等腰三角形的内角和相当于 4 个底角相加, 三角形内角和等于 180 度, 用 180 除以 4 即可计算出一个底角的度数, 然后计算顶角的度数, 据此解决。\n\n【详解】 $180^{\\circ} \\div 4=45^{\\circ}$\n\n$45^{\\circ} \\times 2=90^{\\circ}$\n\n答:这个等腰三角形的顶角是 $90^{\\circ}$ 。\n\n【点睛】解决本题的关键是熟练掌握等腰三角形的性质, 熟记三角形内角和的度数。", "solution": "null", "level": "四年级", "question": "一个等腰三角形, 它的顶角是一个底角的 2 倍。这个等腰三角形的顶角是多少度?", "options": [], "subject": "解析几何", "analysis": "解$90^{\\circ}$\n\n【分析】等腰三角形两个底角相等, 顶角是一个底角的 2 倍, 即一个顶角相当于 2 个底角, 则这个等腰三角形的内角和相当于 4 个底角相加, 三角形内角和等于 180 度, 用 180 除以 4 即可计算出一个底角的度数, 然后计算顶角的度数, 据此解决。\n\n【详解】 $180^{\\circ} \\div 4=45^{\\circ}$\n\n$45^{\\circ} \\times 2=90^{\\circ}$\n\n答:这个等腰三角形的顶角是 $90^{\\circ}$ 。\n\n【点睛】解决本题的关键是熟练掌握等腰三角形的性质, 熟记三角形内角和的度数。"} {"id": "14136", "image": [], "answer": "$\times$", "solution": "null", "level": "四年级", "question": "从同一位置观察不同的物体, 看到的图形一定不同。", "options": [], "subject": "变换几何", "analysis": ""} {"id": "11024", "image": [], "answer": "$\\times$", "solution": "null", "level": "二年级", "question": "一个乒乓球约重 5 千克。", "options": [], "subject": "算术", "analysis": "$\\times$"} {"id": "11025", "image": [], "answer": "$\\times$", "solution": "null", "level": "二年级", "question": "每袋大米重 10 千克, 100 袋大米重 1000 克。", "options": [], "subject": "算术", "analysis": "$\\times$"} {"id": "11028", "image": [], "answer": "$\\times$", "solution": "null", "level": "二年级", "question": "9 筐橘子重 45 千克, 平均每筐橘子重 6 千克。", "options": [], "subject": "算术", "analysis": "$\\times$"} {"id": "11040", "image": ["2285.jpg", "2286.jpg", "2287.jpg"], "answer": "元 $>6$ 元 $>5$ 元\n\n答: 买第(2)箱柠檬最合算。", "solution": "null", "level": "二年级", "question": "某果茶店准备进货一些柠檬制作新品柠㮠茶, 买哪一箱柠檬最合算?\n\n\n\n(1) 2 千克 $/ 14$ 元\n\n\n\n(3) 5 千克/ 25 元\n\n\n\n(2) 4 千克/24 元\n\n答: 买第 $(\\quad)$ 箱柠㮠最合算。", "options": [], "subject": "算术", "analysis": "元 $>6$ 元 $>5$ 元\n\n答: 买第(2)箱柠檬最合算。"} {"id": "10455", "image": [], "answer": "; 克; 千克; 克", "solution": "null", "level": "二年级", "question": "在横线上填上“千克”或“克”。\n\n1 本书约重 200 小明的体重约 30\n1 支铅笔约重 15\n\n1 个鸡蛋约重 50", "options": [], "subject": "算术", "analysis": "; 克; 千克; 克"} {"id": "10456", "image": ["2138.jpg"], "answer": "答案】 $35 ; 33$\n\n【知识点】千克的认识与使用\n\n【解析】【解答】 解:笑笑重 $30+5=35$ 千克; 我有 $35-2=33$ 千克。\n\n故答案为: $35 ; 33$ 。", "solution": "null", "level": "二年级", "question": "猜一猜, 有多重?\n\n", "options": [], "subject": "算术", "analysis": "答案】 $35 ; 33$\n\n【知识点】千克的认识与使用\n\n【解析】【解答】 解:笑笑重 $30+5=35$ 千克; 我有 $35-2=33$ 千克。\n\n故答案为: $35 ; 33$ 。"} {"id": "10457", "image": [], "answer": "答案】<; <; <\n\n【知识点】千克与克之间的换算与比较\n\n【解析】【解答】 解: 8 千克=8000 克<9000 克; 9 千克=9000 克 $>900$ 克; 5 千克 $=5000$ 克 $>4900$ 克。故答案为: $<$; $<$; $<$ 。", "solution": "null", "level": "二年级", "question": "比一比, 在横线上填上“>”“”=或“=”。\n8 千克 \\$ \\qquad \\$ 9000 克\n900 克 \\$ \\qquad \\$ 9 千克\n4900 克 \\$ \\qquad \\$ 5 千克", "options": [], "subject": "算术", "analysis": "答案】<; <; <\n\n【知识点】千克与克之间的换算与比较\n\n【解析】【解答】 解: 8 千克=8000 克<9000 克; 9 千克=9000 克 $>900$ 克; 5 千克 $=5000$ 克 $>4900$ 克。故答案为: $<$; $<$; $<$ 。"} {"id": "10458", "image": [], "answer": "千克 $=$ \\$ \\qquad \\$克 7000 克 $=$ \\$ \\qquad \\$千克\n\n1500 克 +500 克 $=$ \\$ \\qquad \\$千克\n\n1 千克-400 克 $=$ \\$ \\qquad \\$克\n【答案】 $3000 ; 7 ; 2 ; 600$\n\n【知识点】千克与克之间的换算与比较", "solution": "null", "level": "二年级", "question": "在横线上填上合适的数。\n\n3 千克 $=$ \\$ \\qquad \\$克 7000 克 $=$ \\$ \\qquad \\$千克\n\n1500 克 +500 克 $=$ \\$ \\qquad \\$千克\n\n1 千克-400 克 $=$ \\$ \\qquad \\$克", "options": [], "subject": "算术", "analysis": "千克 $=$ \\$ \\qquad \\$克 7000 克 $=$ \\$ \\qquad \\$千克\n\n1500 克 +500 克 $=$ \\$ \\qquad \\$千克\n\n1 千克-400 克 $=$ \\$ \\qquad \\$克\n【答案】 $3000 ; 7 ; 2 ; 600$\n\n【知识点】千克与克之间的换算与比较"} {"id": "10459", "image": [], "answer": "答案】 20 千克; 3000 克; 1 千克; 600 克\n\n【知识点】千克与克之间的换算与比较\n\n【解析】【解答】解: 1 千克 $=1000$ 克, 20 千克 $=20000$ 克; 600 克 $<1000$ 克 $<3000$ 克 $<20000$ 克, 所以 600 克 $<1$ 千克 $<3000$ 克 $<20$ 千克。\n\n故答案:20千克;3000 克;1千克; 600 克。\n\n【分析】含有高级的那位的量和低级单位的量进行比较, 先把高级单位乘进率化成低级单位, 然后进比较即可。", "solution": "null", "level": "二年级", "question": "按从重到轻排列。\n\n600 克 $\\quad 1$ 千克 $\\quad 20$ 千克 $\\quad 3000$ 克 \\$ \\qquad \\$ $>$ $>$ \\$ \\qquad \\$ $>$", "options": [], "subject": "算术", "analysis": "答案】 20 千克; 3000 克; 1 千克; 600 克\n\n【知识点】千克与克之间的换算与比较\n\n【解析】【解答】解: 1 千克 $=1000$ 克, 20 千克 $=20000$ 克; 600 克 $<1000$ 克 $<3000$ 克 $<20000$ 克, 所以 600 克 $<1$ 千克 $<3000$ 克 $<20$ 千克。\n\n故答案:20千克;3000 克;1千克; 600 克。\n\n【分析】含有高级的那位的量和低级单位的量进行比较, 先把高级单位乘进率化成低级单位, 然后进比较即可。"} {"id": "10464", "image": ["2959.jpg", "2959.jpg"], "answer": "\n\n【知识点】克的认识与使用;千克的认识与使用\n\n【解析】分析】根据每种事物的实际情况作答即可。", "solution": "null", "level": "二年级", "question": "连一连。\n\n一个梨 一只母鸡 一头牛 一只羊 一枚硬市\n\n200 千克 35 千克 $\\quad 5$ 克 $\\quad 160$ 克 $\\quad 3$ 千克", "options": [], "subject": "算术", "analysis": "\n\n【知识点】克的认识与使用;千克的认识与使用\n\n【解析】分析】根据每种事物的实际情况作答即可。"} {"id": "10249", "image": [], "answer": "10", "solution": "null", "level": "二年级", "question": "一棵小树现在 90 厘米高, 它再长 ( ) 厘米就到 1 米了。", "options": [], "subject": "算术", "analysis": "10"} {"id": "10252", "image": [], "answer": "92", "solution": "null", "level": "二年级", "question": "一根 1 米长的彩带, 剪去 8 厘米后, 还剩()厘米。", "options": [], "subject": "算术", "analysis": "92"} {"id": "10260", "image": [], "answer": "解: 1 米 $=100$ 厘米\n\n100 厘米 -90 厘米 $=10$ 厘米\n\n答: 再长 10 厘米就有 1 米高了。", "solution": "null", "level": "二年级", "question": "( 6 分 )小明现在的身高是 90 厘米, 再长多少厘米就有 1 米高了?", "options": [], "subject": "算术", "analysis": "解: 1 米 $=100$ 厘米\n\n100 厘米 -90 厘米 $=10$ 厘米\n\n答: 再长 10 厘米就有 1 米高了。"} {"id": "10262", "image": [], "answer": "解: $15+8+6=29$ (米)", "solution": "null", "level": "二年级", "question": "( 6 分 )庆祝 “六一儿童节” 布置教室,小亚买来红彩带 15 米,黄彩带 8 米,绿彩带 6 米,共有彩带多少米?", "options": [], "subject": "算术", "analysis": "解: $15+8+6=29$ (米)"} {"id": "10263", "image": [], "answer": "解: $90+65=155$ 厘米", "solution": "null", "level": "二年级", "question": "( 6 分 )一条钢管长 3 米,王叔叔锯下 90 厘米,李叔叔锯下 65 厘米,\n这根钢管短了多少厘米?", "options": [], "subject": "算术", "analysis": "解: $90+65=155$ 厘米"} {"id": "10264", "image": [], "answer": "解: $30-5=25$ (米)", "solution": "null", "level": "二年级", "question": "(6 分) 小丁的爸爸买来一圈电线长 30 米, 用掉一些后, 还剩 5 米,用掉多少米?", "options": [], "subject": "算术", "analysis": "解: $30-5=25$ (米)"} {"id": "10265", "image": [], "answer": "解: $8+8=16$ (厘米), $16-3=13$ (厘米)。", "solution": "null", "level": "二年级", "question": "(6 分) 把两根长都是 8 厘米的铁条焊接为一根长的铁条, 焊接头 (如\n\n图) 是 3 厘米,焊接后的铁条长多少厘米?", "options": [], "subject": "算术", "analysis": "解: $8+8=16$ (厘米), $16-3=13$ (厘米)。"} {"id": "10275", "image": [], "answer": "$50 ; 40$", "solution": "null", "level": "二年级", "question": "5 与 45 的和是(), 61 比 21 多()。", "options": [], "subject": "算术", "analysis": "$50 ; 40$"} {"id": "10276", "image": [], "answer": "$96 ; 48$", "solution": "null", "level": "二年级", "question": "一个数是 72 , 它与另一个数 24 的和是(), 差是()。", "options": [], "subject": "算术", "analysis": "$96 ; 48$"} {"id": "10277", "image": [], "answer": "92", "solution": "null", "level": "二年级", "question": "小兔妈妈拔来 63 个胡萝卜,小兔吃了 17 个,第二天妈妈又拔回来 46 个。现在共有( )个胡萝卜。", "options": [], "subject": "算术", "analysis": "92"} {"id": "10278", "image": [], "answer": "$81 ; 19$", "solution": "null", "level": "二年级", "question": "张阿姨带了 100 元去超市。买牛奶用了 39 元, 买油用了 42 元, 买这两样东西用了()元,还剩()元。", "options": [], "subject": "算术", "analysis": "$81 ; 19$"} {"id": "10279", "image": [], "answer": "67", "solution": "null", "level": "二年级", "question": "一套故事书 92 元,王玉想买 1 套,还差 25 元,王玉已经攒了()元。", "options": [], "subject": "算术", "analysis": "67"} {"id": "10280", "image": [], "answer": "61", "solution": "null", "level": "二年级", "question": "一本书 96 页,小花第一天看了 27 页,第二天看了 33 页,她第三天应从第()页看起。", "options": [], "subject": "算术", "analysis": "61"} {"id": "10281", "image": [], "answer": "42; 20; 62; 22", "solution": "null", "level": "二年级", "question": "用 $2 、 0 、 4$ 这三个数字能组成的数最大的两位数是 $(\\quad)$, 最小的两位数是(), 它们的和是(), 差是()。", "options": [], "subject": "算术", "analysis": "42; 20; 62; 22"} {"id": "10282", "image": [], "answer": "$23 ; 41$", "solution": "null", "level": "二年级", "question": "新华小学二 (1) 班有女生 18 人, 男生比女生多 5 人。男生有()人。男生和女生一共有( )人。", "options": [], "subject": "算术", "analysis": "$23 ; 41$"} {"id": "10283", "image": [], "answer": "8", "solution": "null", "level": "二年级", "question": "皮皮爸爸买了 36 个桃, 皮皮家招待朋友吃了 19 个, 第二天, 他们又吃了 9 个, 还剩下 ( ) 个。", "options": [], "subject": "算术", "analysis": "8"} {"id": "10288", "image": ["2051.jpg"], "answer": "[](本地图片-2662.jpg)", "solution": "null", "level": "二年级", "question": "4 分)把小鸟送回家。\n", "options": [], "subject": "算术", "analysis": "[](本地图片-2662.jpg)"} {"id": "10289", "image": ["2052.jpg"], "answer": "[](本地图片-2663.jpg)", "solution": "null", "level": "二年级", "question": "(4 分 ) 找出同一双手套。(连一连)\n", "options": [], "subject": "算术", "analysis": "[](本地图片-2663.jpg)"} {"id": "10290", "image": [], "answer": "解: $25+16-19=22$ (只).\n\n答: 还剩 22 只兔子。", "solution": "null", "level": "二年级", "question": "( 5 分 )小亮家原来有 25 只兔子, 又买来 16 只兔子, 送给小明 19 只兔子,还剩多少只兔子?", "options": [], "subject": "算术", "analysis": "解: $25+16-19=22$ (只).\n\n答: 还剩 22 只兔子。"} {"id": "10292", "image": [], "answer": "$32-3+15$\n\n$=29+15$\n\n$=44 ($ 个 $)$\n\n答: 小猴家现在有 44 个桃子.", "solution": "null", "level": "二年级", "question": "(5 分)小猴家原来有 32 个桃子,吃了 3 个,猴爸爸又摘回来 15 个。小猴家现在有多少个桃子?", "options": [], "subject": "算术", "analysis": "$32-3+15$\n\n$=29+15$\n\n$=44 ($ 个 $)$\n\n答: 小猴家现在有 44 个桃子."} {"id": "10293", "image": [], "answer": "解: $90-31-36=23$ (件)\n\n答: 还要再做 23 件才能完成预定的任务。", "solution": "null", "level": "二年级", "question": "(5 分) 手工小组计划要做 90 件手工艺品, 上午做了 31 件, 下午做了 36 件, 还要再做多少件才能完成预定的任务?", "options": [], "subject": "算术", "analysis": "解: $90-31-36=23$ (件)\n\n答: 还要再做 23 件才能完成预定的任务。"} {"id": "10331", "image": [], "answer": "$5 ; 6 ; 30 ; 5 \\times 6=30$ 或 $6 \\times 5=30$", "solution": "null", "level": "二年级", "question": "$6+6+6+6+6$ 表示()个()相加, 和是(), 写成乘法算式是()。", "options": [], "subject": "算术", "analysis": "$5 ; 6 ; 30 ; 5 \\times 6=30$ 或 $6 \\times 5=30$"} {"id": "10332", "image": [], "answer": "$3 ; 3 ; 2 ; 3$", "solution": "null", "level": "二年级", "question": "$3 \\times 3$ 表示()个()相加。3+3表示()个()相加。", "options": [], "subject": "算术", "analysis": "$3 ; 3 ; 2 ; 3$"} {"id": "10333", "image": [], "answer": "$6 ; 12$", "solution": "null", "level": "二年级", "question": "3 只小兔, 一共有 ( ) 只耳朵, ( ) 条腿。", "options": [], "subject": "算术", "analysis": "$6 ; 12$"} {"id": "10334", "image": [], "answer": "12", "solution": "null", "level": "二年级", "question": "有一堆亜子, 平均分给 4 只刺猬或平均分给 3 只刺猬, 均没有剩余。这堆专子最少有()个。", "options": [], "subject": "算术", "analysis": "12"} {"id": "10336", "image": [], "answer": "$7 ; 25$", "solution": "null", "level": "二年级", "question": "2 和 5 的和是( ), 2 个 5 的积是()。", "options": [], "subject": "算术", "analysis": "$7 ; 25$"} {"id": "10337", "image": ["2079.jpg", "2080.jpg"], "answer": "$4 ; 24$", "solution": "null", "level": "二年级", "question": "1 辆\n\n\n\n有 $(\\quad)$ 个轮子, 6 辆\n\n\n\n共有 ( ) 个轮子。", "options": [], "subject": "算术", "analysis": "$4 ; 24$"} {"id": "10338", "image": [], "answer": "16", "solution": "null", "level": "二年级", "question": "明明看一本故事书, 看了 3 天, 每天看 5 页, 第 4 天应从第( )页看起。", "options": [], "subject": "算术", "analysis": "16"} {"id": "10339", "image": [], "answer": "12", "solution": "null", "level": "二年级", "question": "每层楼高 3 米, 4 层楼高 ( $\\quad$ ) 米.", "options": [], "subject": "算术", "analysis": "12"} {"id": "10341", "image": [], "answer": "五; 四; 六; 六; 一; 三", "solution": "null", "level": "二年级", "question": "把口诀补充完整。\n\n三 ( ) 十五\n\n四()十六\n\n三( $\\quad$ )十八\n\n五 $(\\quad)$ 三十\n\n$(\\quad)$ 六得六\n\n$(\\quad)$ 三得九", "options": [], "subject": "算术", "analysis": "五; 四; 六; 六; 一; 三"} {"id": "10343", "image": [], "answer": "2\n\n23.__详解\n\n23. | $5 \\times 3=15$ | $12 \\div 2=6$ | $4 \\times 6=24$ | $7 \\times 4-2=26$ |\n| ---: | ---: | ---: | ---: |\n| $10 \\div 2=5$ | $3 \\times 7=21$ | $24 \\div 6=4$ | $6 \\times 5+3=33$ |\n| $35 \\div 7=5$ | $6 \\times 7=42$ | $6+3=9$ | $3 \\times 3+5=14$ |", "solution": "null", "level": "二年级", "question": "有两盘水果,第一盘比第二盘多 4 个,第一盘分( )个给第二盘就同样多。", "options": [], "subject": "算术", "analysis": "2\n\n23.__详解\n\n23. | $5 \\times 3=15$ | $12 \\div 2=6$ | $4 \\times 6=24$ | $7 \\times 4-2=26$ |\n| ---: | ---: | ---: | ---: |\n| $10 \\div 2=5$ | $3 \\times 7=21$ | $24 \\div 6=4$ | $6 \\times 5+3=33$ |\n| $35 \\div 7=5$ | $6 \\times 7=42$ | $6+3=9$ | $3 \\times 3+5=14$ |"} {"id": "10346", "image": ["2084.jpg", "2085.jpg", "2086.jpg", "2086.jpg"], "answer": "", "solution": "null", "level": "二年级", "question": "( 4 分) 连一连。\n\n\n$5 \\times 2$\n\n$8+8+8-3$\n\n\n\n2 个 5 相加\n\n$8 \\times 3-3$\n\n$7 \\times 4$", "options": [], "subject": "算术", "analysis": ""} {"id": "10347", "image": ["2087.jpg", "2088.jpg", "2088.jpg"], "answer": "", "solution": "null", "level": "二年级", "question": "( 4 分) 找伙伴, 连线。\n$6+6+6$\n$4+4+4+4$\n$4 \\times 3$\n$5+5+5+5+5$\n\n\n\n$3+3+3+3$\n\n$4 \\times 4$\n\n$6 \\times 3$", "options": [], "subject": "算术", "analysis": ""} {"id": "10348", "image": ["2089.jpg", "2090.jpg", "2091.jpg", "2091.jpg"], "answer": "", "solution": "null", "level": "二年级", "question": "( 8 分 ) 连一连, 把得数相等的连起来。\n\n$5+5+5$\n\n乘数是 3 和 4\n\n$6+6+6$\n\n\n\n$4 \\times 3$\n\n4 乘 3\n\n\n\n6 个 3 相加\n\n$2+2+2+2+2$", "options": [], "subject": "算术", "analysis": ""} {"id": "10349", "image": ["2092.jpg"], "answer": "解: $2 \\times 5=10$ (把)\n\n答: 5 张桌子配 10 把椅子。", "solution": "null", "level": "二年级", "question": "( 5 分)每张桌子配 2 把椅子, 5 张桌子需要配多少把椅子?\n\n", "options": [], "subject": "算术", "analysis": "解: $2 \\times 5=10$ (把)\n\n答: 5 张桌子配 10 把椅子。"} {"id": "10351", "image": [], "answer": "解: $6 \\times 4=24$ (人)\n\n答:一共有 24 人在划船。", "solution": "null", "level": "二年级", "question": "( 5 分 )同学们去公园划船, 每条船上坐 6 人, 一共坐了 4 条船, 一共有多少人划船?", "options": [], "subject": "算术", "analysis": "解: $6 \\times 4=24$ (人)\n\n答:一共有 24 人在划船。"} {"id": "10353", "image": ["2094.jpg"], "answer": "解: $3 \\times 4+2=14$ (个) 或 $3 \\times 5-1=14$ (个)\n\n答:一共有 14 个桃子。", "solution": "null", "level": "二年级", "question": "5分)一共有多少个桃子?\n\n", "options": [], "subject": "算术", "analysis": "解: $3 \\times 4+2=14$ (个) 或 $3 \\times 5-1=14$ (个)\n\n答:一共有 14 个桃子。"} {"id": "10354", "image": [], "answer": "解: $4 \\times 4=16$ (米)\n\n答: 这根绳子原来长 16 米。", "solution": "null", "level": "二年级", "question": "(6 分 )一根绳子对折 2 次后长 4 米, 这根绳子原来长多少米?", "options": [], "subject": "算术", "analysis": "解: $4 \\times 4=16$ (米)\n\n答: 这根绳子原来长 16 米。"} {"id": "10364", "image": [], "answer": "$100 ; 9$", "solution": "null", "level": "二年级", "question": "1 米= ( $\\quad$ )厘米\n17 米-8 米= ( $\\quad$ ) 米。", "options": [], "subject": "算术", "analysis": "$100 ; 9$"} {"id": "10365", "image": [], "answer": "90", "solution": "null", "level": "二年级", "question": "最小的两位数与最小的三位数相差()。", "options": [], "subject": "算术", "analysis": "90"} {"id": "10366", "image": [], "answer": "$50 ; 40$", "solution": "null", "level": "二年级", "question": "5 与 45 的和是(), 61 比 21 多()。", "options": [], "subject": "算术", "analysis": "$50 ; 40$"} {"id": "10367", "image": ["2095.jpg"], "answer": "; >; =; <; =; <", "solution": "null", "level": "二年级", "question": "在横线上填上 “ ””、“<”或 “=”。\n\n", "options": [], "subject": "算术", "analysis": "; >; =; <; =; <"} {"id": "10368", "image": [], "answer": "$3 \\times 6$ 或 $6 \\times 3$; 6 乘 3 等于 18", "solution": "null", "level": "二年级", "question": "3 个 6 相加, 写成乘法算式是 ( ) , 读作 ( )。", "options": [], "subject": "算术", "analysis": "$3 \\times 6$ 或 $6 \\times 3$; 6 乘 3 等于 18"} {"id": "10370", "image": [], "answer": "92", "solution": "null", "level": "二年级", "question": "一根 1 米长的彩带, 剪去 8 厘米后, 还剩 ( ) 厘米。", "options": [], "subject": "算术", "analysis": "92"} {"id": "10371", "image": [], "answer": "5", "solution": "null", "level": "二年级", "question": "小青有 32 枚邮票, 小兰有 22 枚邮票, 小青送给小兰( )枚后, 两人的邮票数相等。", "options": [], "subject": "算术", "analysis": "5"} {"id": "10376", "image": [], "answer": "$6 ; 12$", "solution": "null", "level": "二年级", "question": "3 只小兔, 一共有 ( ) 只耳朵, ( ) 条腿。", "options": [], "subject": "算术", "analysis": "$6 ; 12$"} {"id": "10381", "image": [], "answer": "$\\begin{array}{lllll}24 & 15 & 84 & 12\\end{array}$\n\n$\\begin{array}{llll}12 & 12 & 39 & 6\\end{array}$\n\n$\\begin{array}{llll}6 & 10 & 0 & 11\\end{array}$", "solution": "null", "level": "二年级", "question": "( 12 分 )直接写得数。\n$4 \\times 6=$\n$5 \\times 3=$\n$24+60=$\n$17-5=$\n$7+5=$\n$2 \\times 6=$\n$41-2=$\n$3 \\times 2=$\n$2 \\times 4-2=$\n\n$5 \\times 1+5=$\n\n$4 \\times 6-10=$\n\n$3 \\times 6-7=$", "options": [], "subject": "算术", "analysis": "$\\begin{array}{lllll}24 & 15 & 84 & 12\\end{array}$\n\n$\\begin{array}{llll}12 & 12 & 39 & 6\\end{array}$\n\n$\\begin{array}{llll}6 & 10 & 0 & 11\\end{array}$"} {"id": "10382", "image": ["2106.jpg", "2107.jpg"], "answer": "(1) $27+18=45$ (元)\n\n(2) $3 \\times 2=6$ (只)", "solution": "null", "level": "二年级", "question": "( 8 分 )看图列式计算。\n\n\n\n(2)\n\n\n\n$\\square \\circ \\square=\\square$", "options": [], "subject": "算术", "analysis": "(1) $27+18=45$ (元)\n\n(2) $3 \\times 2=6$ (只)"} {"id": "10384", "image": ["2112.jpg"], "answer": "解: $2 \\times 5=10$ (把)\n\n答: 5 张桌子配 10 把椅子。", "solution": "null", "level": "二年级", "question": "( 5 分 ) 每张桌子配 2 把椅子, 5 张桌子需要配多少把椅子?\n\n", "options": [], "subject": "算术", "analysis": "解: $2 \\times 5=10$ (把)\n\n答: 5 张桌子配 10 把椅子。"} {"id": "10385", "image": [], "answer": "解: 56-27+24=53 (人).\n\n答:现在合唱队有 53 人。", "solution": "null", "level": "二年级", "question": "( 5 分) 星光小学合唱队原有 56 人, 有 27 人升入中学后离队, 又有 24 人新加入。现在合唱队有多少人?", "options": [], "subject": "算术", "analysis": "解: 56-27+24=53 (人).\n\n答:现在合唱队有 53 人。"} {"id": "10386", "image": [], "answer": "解: $5 \\times 5=25$ (分)", "solution": "null", "level": "二年级", "question": "( 5 分 ). 跑 1 千米大约需要 5 分钟, 用这样的速度跑 5 千米大约需要多少分钟?", "options": [], "subject": "算术", "analysis": "解: $5 \\times 5=25$ (分)"} {"id": "10387", "image": [], "answer": "解: $90-31-36=23$ (件)\n\n答: 还要再做 23 件才能完成预定的任务。", "solution": "null", "level": "二年级", "question": "( 5 分 ) 手工小组计划要做 90 件手工艺品, 上午做了 31 件,下午做了 36 件, 还要再做多少件才能完成预定的任务?", "options": [], "subject": "算术", "analysis": "解: $90-31-36=23$ (件)\n\n答: 还要再做 23 件才能完成预定的任务。"} {"id": "10397", "image": [], "answer": "十二; 五; 二十八; 三十; 六十四; 二十七", "solution": "null", "level": "二年级", "question": "把口诀填完整。\n\n三四() 三()十五 四七()\n\n五六()八八()三九()", "options": [], "subject": "算术", "analysis": "十二; 五; 二十八; 三十; 六十四; 二十七"} {"id": "10398", "image": [], "answer": "9; 4;36;4;9;36; 四九三十六", "solution": "null", "level": "二年级", "question": "$9+9+9+9$ 写成乘法算式是() $\\times(\\quad)=(\\quad)$, 或者 $(\\quad) \\times$\n$(\\quad)=(\\quad)$, 口诀是 $(\\quad)$ 。", "options": [], "subject": "算术", "analysis": "9; 4;36;4;9;36; 四九三十六"} {"id": "10399", "image": [], "answer": "$16 ; 25$", "solution": "null", "level": "二年级", "question": "2 个 8 相加的和是( ), 2 个 5 相乘的积是()。", "options": [], "subject": "算术", "analysis": "$16 ; 25$"} {"id": "10400", "image": [], "answer": "$21 ; 35$", "solution": "null", "level": "二年级", "question": "一星期有 7 天,三个星期有()天,五个星期有()天。", "options": [], "subject": "算术", "analysis": "$21 ; 35$"} {"id": "10401", "image": [], "answer": "$+; \\quad \\times ; \\times$", "solution": "null", "level": "二年级", "question": "在横线上填上 “+”“-”“ $\\times ”$ 。\n\n$6(\\quad) 8=14$\n\n$9(\\quad) 7=63$\n\n$72=8(\\quad) 9$", "options": [], "subject": "算术", "analysis": "$+; \\quad \\times ; \\times$"} {"id": "10402", "image": ["2115.jpg"], "answer": "$<;>;$", "solution": "null", "level": "二年级", "question": "在横线上填上 “ ”、“\"”或 “=”。\n\n", "options": [], "subject": "算术", "analysis": "$<;>;$"} {"id": "10403", "image": [], "answer": "28", "solution": "null", "level": "二年级", "question": "丁丁和他的 6 个好朋友每人都制作了 4 张贺卡,他们一共制作了()张贺卡。", "options": [], "subject": "算术", "analysis": "28"} {"id": "10404", "image": [], "answer": "4", "solution": "null", "level": "二年级", "question": "妈妈去超市买肥皀, 一块肥黾 4 元, 妈妈带的钱够买 8 块。如果妈妈带的都是 10 元,那么她最少带()张。", "options": [], "subject": "算术", "analysis": "4"} {"id": "10405", "image": [], "answer": "32", "solution": "null", "level": "二年级", "question": "一套《童话故事》共有 4 本, 每本 8 元。买一套需要()元。", "options": [], "subject": "算术", "analysis": "32"} {"id": "10407", "image": [], "answer": "小兰; 1", "solution": "null", "level": "二年级", "question": "小兰家栽了 2 行桃树, 每行 7 棵; 小芳家也栽了 2 行桃树, 一行 7 棵,一行 6 棵。()家栽的桃树多,多()棵。", "options": [], "subject": "算术", "analysis": "小兰; 1"} {"id": "10412", "image": ["2122.jpg", "2123.jpg", "2123.jpg"], "answer": "", "solution": "null", "level": "二年级", "question": "( 4 分 ) 投篮。(算一算, 连一连)\n", "options": [], "subject": "算术", "analysis": ""} {"id": "10413", "image": [], "answer": "解: $5 \\times 9=45$ (颗)\n\n答: 9 件衣服要钉 45 颗钮扣。", "solution": "null", "level": "二年级", "question": "( 5 分 ) 一件衣服钉 5 个钮扣, 9 件衣服钉多少颗钮扣?", "options": [], "subject": "算术", "analysis": "解: $5 \\times 9=45$ (颗)\n\n答: 9 件衣服要钉 45 颗钮扣。"} {"id": "10414", "image": [], "answer": "$9 \\times 5=45 ($ 元);\n\n50 元 $>45$ 元, 够.\n\n答: 用 50 元买 5 个茶杯, 钱够.", "solution": "null", "level": "二年级", "question": "( 5 分 ) 一个茶杯 9 元钱, 用 50 元买 5 个茶杯, 钱够吗?", "options": [], "subject": "算术", "analysis": "$9 \\times 5=45 ($ 元);\n\n50 元 $>45$ 元, 够.\n\n答: 用 50 元买 5 个茶杯, 钱够."} {"id": "10418", "image": ["2958.jpg"], "answer": "解: $6 \\times 7=42$ (根)\n\n答: 一共要 42 根小棒。", "solution": "null", "level": "二年级", "question": "\n棒?", "options": [], "subject": "算术", "analysis": "解: $6 \\times 7=42$ (根)\n\n答: 一共要 42 根小棒。"} {"id": "10419", "image": [], "answer": "解: $8 \\times 5=40$ (元)\n\n答: 他要付 40 元。", "solution": "null", "level": "二年级", "question": "5 分)日记本一个 5 元,李老师买了 8 个, 他要付多少钱?", "options": [], "subject": "算术", "analysis": "解: $8 \\times 5=40$ (元)\n\n答: 他要付 40 元。"} {"id": "10432", "image": [], "answer": "$6 ; 82 ; 12$", "solution": "null", "level": "二年级", "question": "用 $2 、 1 、 8$ 能写出 $(\\quad)$ 个两位数, 最大的两位数是(),最小的两位数是()。", "options": [], "subject": "算术", "analysis": "$6 ; 82 ; 12$"} {"id": "10435", "image": [], "answer": "$15 ; 3$", "solution": "null", "level": "二年级", "question": "分针从 3 走到 6 , 走了()分,时针从 3 走到 6 ,走了()时。", "options": [], "subject": "算术", "analysis": "$15 ; 3$"} {"id": "10436", "image": [], "answer": "$5 ; 60 ; 1 ; 60$", "solution": "null", "level": "二年级", "question": "分针走 1 大格是 ( ) 分。分针走 1 圈是 ( ) 分, 这时时针正好走 ( )大格,也就是 1 时等于 $(\\quad)$ 分。", "options": [], "subject": "算术", "analysis": "$5 ; 60 ; 1 ; 60$"} {"id": "10446", "image": [], "answer": ": $60 \\div 10+1=7$ (辆)", "solution": "null", "level": "二年级", "question": "4 分 )汽车站每隔 10 分钟开出一辆汽车,那么 1 小时开出多少辆汽车?", "options": [], "subject": "算术", "analysis": ": $60 \\div 10+1=7$ (辆)"} {"id": "10471", "image": [], "answer": "$\\times$\n\n【分析】除法算式的读法: 被除数除以除数等于商, 据此解答。\n\n【详解】 $24 \\div 4=6$ 读作: 24 除以 4 等于 6 。\n\n故答案为: $\\times$\n\n【点睛】解答此题的关键是区分“除”和“除以”之间的区别。", "solution": "null", "level": "二年级", "question": "$24 \\div 4=6$ 读作: 24 除 4 等于 6 。( $\\quad)$", "options": [], "subject": "算术", "analysis": "$\\times$\n\n【分析】除法算式的读法: 被除数除以除数等于商, 据此解答。\n\n【详解】 $24 \\div 4=6$ 读作: 24 除以 4 等于 6 。\n\n故答案为: $\\times$\n\n【点睛】解答此题的关键是区分“除”和“除以”之间的区别。"} {"id": "10473", "image": [], "answer": "$\\sqrt{ }$\n\n【分析】由乘法口诀六九五十四可知, 计算这两道算式用同一句口诀。\n\n【详解】由分析可知, 计算 $54 \\div 6$ 和 $54 \\div 9$ 时用同一句乘法口块六九五十四。\n\n故答案为: $\\sqrt{ }$\n\n【点睛】本题主要考查用乘法口诀求商的方法。", "solution": "null", "level": "二年级", "question": "计算 $54 \\div 6$ 和 $54 \\div 9$ 时用同一句乘法口诀。( )", "options": [], "subject": "算术", "analysis": "$\\sqrt{ }$\n\n【分析】由乘法口诀六九五十四可知, 计算这两道算式用同一句口诀。\n\n【详解】由分析可知, 计算 $54 \\div 6$ 和 $54 \\div 9$ 时用同一句乘法口块六九五十四。\n\n故答案为: $\\sqrt{ }$\n\n【点睛】本题主要考查用乘法口诀求商的方法。"} {"id": "10474", "image": [], "answer": "$\\times$\n\n【分析】 运用 $8 、 9$ 的乘法口诀求出它们的商进行解答。\n\n【详解】 $63 \\div 7=9,56 \\div 7=8,81 \\div 9=9$, 所以 $63 \\div 7=81 \\div 9>56 \\div 7$ 。故答案为: $\\times$\n\n【点睛】 掌握 $8 、 9$ 的乘法口诀, 能正确的求商。", "solution": "null", "level": "二年级", "question": "$63 \\div 7>56 \\div 7>81 \\div 9$ 。( $\\quad)$", "options": [], "subject": "算术", "analysis": "$\\times$\n\n【分析】 运用 $8 、 9$ 的乘法口诀求出它们的商进行解答。\n\n【详解】 $63 \\div 7=9,56 \\div 7=8,81 \\div 9=9$, 所以 $63 \\div 7=81 \\div 9>56 \\div 7$ 。故答案为: $\\times$\n\n【点睛】 掌握 $8 、 9$ 的乘法口诀, 能正确的求商。"} {"id": "10475", "image": [], "answer": "$\\times$\n\n【分析】根据题意求出 24 人里面有几个 8 人, 就可以知道租几条船, 用除法求出商, 据此解答。\n\n【详解】根据分析列式为: $24 \\div 8=3$ (条), 可以租 3 条船。\n\n故答案为: $\\times$\n\n【点睛】 掌握用 8 的乘法口诀求商。", "solution": "null", "level": "二年级", "question": "每条船限乘 8 人, 24 人要租 4 条船。", "options": [], "subject": "算术", "analysis": "$\\times$\n\n【分析】根据题意求出 24 人里面有几个 8 人, 就可以知道租几条船, 用除法求出商, 据此解答。\n\n【详解】根据分析列式为: $24 \\div 8=3$ (条), 可以租 3 条船。\n\n故答案为: $\\times$\n\n【点睛】 掌握用 8 的乘法口诀求商。"} {"id": "10479", "image": [], "answer": "56 页\n\n【分析】计划 5 天读的页数 $\\div 5=$ 计划平均每天读的页数, 计划平均每天读的页数 $\\times$ 计划读的天数 $=$ 这些天可以读的页数, 依此计算。\n\n【详解】 $40 \\div 5=8$ (页)\n\n$8 \\times 7=56$ (页)\n\n答:安安一周(7 天)可以读 56 页。\n\n【点睛】此题考查的是归一问题的计算, 先计算出计划平均每天读的页数, 是解答此题的关键。", "solution": "null", "level": "二年级", "question": "《稻草人》是叶圣陶 1922 年发表的中国现代童话。该作是中国文学史上最早为儿童创作的文学作品之一。安安打算寒假把这本书再读一遍。他计划 5 天读 40 页, 照这样计算, 安安一周(7 天)可以读多少页 $?$", "options": [], "subject": "算术", "analysis": "56 页\n\n【分析】计划 5 天读的页数 $\\div 5=$ 计划平均每天读的页数, 计划平均每天读的页数 $\\times$ 计划读的天数 $=$ 这些天可以读的页数, 依此计算。\n\n【详解】 $40 \\div 5=8$ (页)\n\n$8 \\times 7=56$ (页)\n\n答:安安一周(7 天)可以读 56 页。\n\n【点睛】此题考查的是归一问题的计算, 先计算出计划平均每天读的页数, 是解答此题的关键。"} {"id": "10480", "image": [], "answer": "8 名\n\n【分析】 2 名同学擦玻璃的块数 $\\div 2=1$ 名同学擦玻璃的块数, 玻璃的总块数 $\\div 1$ 名同学擦玻璃的块数 $=$擦这些玻璃一共需要学生的人数, 依此列式并计算。\n\n【详解】 $6 \\div 2=3$ (块)\n$24 \\div 3=8($ 名 $)$\n\n答: 一共需要 8 名同学。\n\n【点睛】此题考查的是归一问题的计算, 先计算出 1 名同学擦玻璃的块数是解答此题的关键。", "solution": "null", "level": "二年级", "question": "同学们大扫除, 2 名同学擦 6 块玻璃。照这样计算, 教室共 24 块玻璃, 一共需要几名同学?", "options": [], "subject": "算术", "analysis": "8 名\n\n【分析】 2 名同学擦玻璃的块数 $\\div 2=1$ 名同学擦玻璃的块数, 玻璃的总块数 $\\div 1$ 名同学擦玻璃的块数 $=$擦这些玻璃一共需要学生的人数, 依此列式并计算。\n\n【详解】 $6 \\div 2=3$ (块)\n$24 \\div 3=8($ 名 $)$\n\n答: 一共需要 8 名同学。\n\n【点睛】此题考查的是归一问题的计算, 先计算出 1 名同学擦玻璃的块数是解答此题的关键。"} {"id": "10481", "image": [], "answer": "40 个\n\n【分析】根据李师傅 3 天做了 24 个零件, 先用 24 除以 3 算出 1 天做的零件个数, 再乘 5 即可求解。\n\n【详解】 $24 \\div 3=8$ (个)\n\n$8 \\times 5=40($ 个 $)$\n\n答:一个星期(5 天)可以做 40 个零件。\n\n【点睛】此题主要考查整数乘除法的实际运用。", "solution": "null", "level": "二年级", "question": "李师傅 3 天做了 24 个零件, 照这样计算,一个星期(5 天)可以做多少个零件?", "options": [], "subject": "算术", "analysis": "40 个\n\n【分析】根据李师傅 3 天做了 24 个零件, 先用 24 除以 3 算出 1 天做的零件个数, 再乘 5 即可求解。\n\n【详解】 $24 \\div 3=8$ (个)\n\n$8 \\times 5=40($ 个 $)$\n\n答:一个星期(5 天)可以做 40 个零件。\n\n【点睛】此题主要考查整数乘除法的实际运用。"} {"id": "10482", "image": [], "answer": "(1) 35 千米;\n\n(2) 6 小时\n\n【分析】(1) 用 21 千米除以 3 , 求出 1 小时行驶的千米数, 再乘 5 小时即可。\n\n(2) 用 42 千米除以 1 小时行驶的千米数即可。\n\n【详解】 (1) $21 \\div 3=7$ (千米)\n\n$7 \\times 5=35($ 千米 $)$\n\n答: 5 小时能行 35 千米。\n\n(2) $42 \\div 7=6$ (小时)\n\n答: 42 千米需要 6 小时。\n\n【点睛】熟练掌握归一问题的解题方法, 求出 1 小时行驶的千米数, 是解答此题的关键。", "solution": "null", "level": "二年级", "question": "星期天,贝贝和妈妈骑自行车去看奶奶, 3 小时行了 21 千米。\n\n(1) 按这样的速度, 5 小时能行多少千米?\n\n(2) 按这样的速度,42 千米需要几小时?", "options": [], "subject": "算术", "analysis": "(1) 35 千米;\n\n(2) 6 小时\n\n【分析】(1) 用 21 千米除以 3 , 求出 1 小时行驶的千米数, 再乘 5 小时即可。\n\n(2) 用 42 千米除以 1 小时行驶的千米数即可。\n\n【详解】 (1) $21 \\div 3=7$ (千米)\n\n$7 \\times 5=35($ 千米 $)$\n\n答: 5 小时能行 35 千米。\n\n(2) $42 \\div 7=6$ (小时)\n\n答: 42 千米需要 6 小时。\n\n【点睛】熟练掌握归一问题的解题方法, 求出 1 小时行驶的千米数, 是解答此题的关键。"} {"id": "10483", "image": [], "answer": "9 分钟\n\n【分析】 4 分钟做口算题的数量 $\\div 4=1$ 分钟做口算题的数量, 口算题的总数量 $\\div 1$ 分钟做口算题的数量 $=$ 做完这些口算题需要的时间, 依此列式并计算。\n\n【详解】 $32 \\div 4=8$ (道)\n\n$72 \\div 8=9$ (分钟)\n\n答: 照这样的速度, 72 道口算题需要 9 分钟做完。\n\n【点睛】此题考查的是归一问题的计算, 先计算出 1 分钟做口算题的数量是解答此题的关键。", "solution": "null", "level": "二年级", "question": "小明做口算题, 4 分钟做了 32 道, 照这样的速度, 72 道口算题需要几分钟做完?", "options": [], "subject": "算术", "analysis": "9 分钟\n\n【分析】 4 分钟做口算题的数量 $\\div 4=1$ 分钟做口算题的数量, 口算题的总数量 $\\div 1$ 分钟做口算题的数量 $=$ 做完这些口算题需要的时间, 依此列式并计算。\n\n【详解】 $32 \\div 4=8$ (道)\n\n$72 \\div 8=9$ (分钟)\n\n答: 照这样的速度, 72 道口算题需要 9 分钟做完。\n\n【点睛】此题考查的是归一问题的计算, 先计算出 1 分钟做口算题的数量是解答此题的关键。"} {"id": "10497", "image": [], "answer": "$\\times$", "solution": "null", "level": "二年级", "question": "两个数(0 除外)的商一定小于这两个数的积。( )", "options": [], "subject": "算术", "analysis": "$\\times$"} {"id": "10498", "image": [], "answer": "$\\times$", "solution": "null", "level": "二年级", "question": "$35 \\div 5=7$, 表示 35 里有 5 个 7 。( $\\quad$ )", "options": [], "subject": "算术", "analysis": "$\\times$"} {"id": "10499", "image": [], "answer": "$\\times$", "solution": "null", "level": "二年级", "question": "被除数是 72 , 除数是 9 , 商是 3 。( $\\quad)$", "options": [], "subject": "算术", "analysis": "$\\times$"} {"id": "10500", "image": [], "answer": "$\\sqrt{ }$\n\n【分析】花瓶里有三十几朵花, 每 7 朵扎一束, 正好扎完, 也就是几个 7 是三十多, 根据五七三十五可知, $7 \\times 5=35$, 也就是花瓶里共有 35 朵花, 据此解答。\n\n【详解】由五七三十五可知, $7 \\times 5=35$ (朵)\n\n则花瓶里共有 35 朵花。\n\n所以,原题说法正确。\n\n故答案为: $\\sqrt{ }$\n\n【点睛】本题考查了 7 的乘法口诀的灵活运用。", "solution": "null", "level": "二年级", "question": "花瓶里有三十几朵花,每 7 朵扎一束,正好扎完。花瓶里共有 35 朵花。( )", "options": [], "subject": "算术", "analysis": "$\\sqrt{ }$\n\n【分析】花瓶里有三十几朵花, 每 7 朵扎一束, 正好扎完, 也就是几个 7 是三十多, 根据五七三十五可知, $7 \\times 5=35$, 也就是花瓶里共有 35 朵花, 据此解答。\n\n【详解】由五七三十五可知, $7 \\times 5=35$ (朵)\n\n则花瓶里共有 35 朵花。\n\n所以,原题说法正确。\n\n故答案为: $\\sqrt{ }$\n\n【点睛】本题考查了 7 的乘法口诀的灵活运用。"} {"id": "10501", "image": [], "answer": "$\\sqrt{ }$\n\n【分析】根据乘法口决“四八三十二”来计算 $32 \\div 4$ 和 $32 \\div 8$, 据此解答。\n\n【详解】计算 $32 \\div 4$ 和 $32 \\div 8$ 要用到“四八三十二”来计算, 所以都用同一句口诀。所以原题说法正确。故答案为: $V_{0}$\n\n【点睛】本题考查了学生运用乘法口决完成表内除法的能力。", "solution": "null", "level": "二年级", "question": "计算 $32 \\div 4$ 和 $32 \\div 8$ 要用到同一句口诀。( )", "options": [], "subject": "算术", "analysis": "$\\sqrt{ }$\n\n【分析】根据乘法口决“四八三十二”来计算 $32 \\div 4$ 和 $32 \\div 8$, 据此解答。\n\n【详解】计算 $32 \\div 4$ 和 $32 \\div 8$ 要用到“四八三十二”来计算, 所以都用同一句口诀。所以原题说法正确。故答案为: $V_{0}$\n\n【点睛】本题考查了学生运用乘法口决完成表内除法的能力。"} {"id": "10505", "image": [], "answer": "6 个", "solution": "null", "level": "二年级", "question": "二 (1) 班有男生 28 人,女生 20 人,全班去植树,每 8 人一组, 二 (1) 班可以分成几个植树小组?", "options": [], "subject": "算术", "analysis": "6 个"} {"id": "10506", "image": [], "answer": "$15-36 \\div 4=6$ (元)", "solution": "null", "level": "二年级", "question": "便民小超市卖毛巾原来 15 元一条, 现在店庆促销, 4 条卖 36 元, 现在每条比原来便宜多少钱?", "options": [], "subject": "算术", "analysis": "$15-36 \\div 4=6$ (元)"} {"id": "10507", "image": ["2149.jpg"], "answer": "$24 \\div 8=3$ (个)\n\n3", "solution": "null", "level": "二年级", "question": "有 24 个玩具熊, 每 8 个装一箱, 需要几个箱子?\n\n", "options": [], "subject": "算术", "analysis": "$24 \\div 8=3$ (个)\n\n3"} {"id": "10508", "image": [], "answer": "9 个", "solution": "null", "level": "二年级", "question": "为了预防新冠病毒, 王老师领来 45 个口罩, 每个同学分 5 个, 可以分给几个同学?", "options": [], "subject": "算术", "analysis": "9 个"} {"id": "10509", "image": ["2150.jpg", "2151.jpg", "2152.jpg", "2153.jpg"], "answer": "(1) 68 元\n\n(2) 4 元\n\n【分析】(1) 一副乒乓球拍 8 元, 一个足球 36 元, 求买 4 副乒乓球拍和一个足球一共需要的钱数,用乘法计算 4 副乒乓球拍的钱再加上一个足球的钱即可。\n\n(2) 一副乒乓球拍的价钱是一个建子的 2 倍, 根据倍的关系, 用除法计算一个建子的价钱。\n\n【详解】 (1) $4 \\times 8+36=68$ (元)\n\n答: 买 4 副乒乓球拍和一个足球一共需要 68 元。\n\n(2) $8 \\div 2=4($ 元 $)$\n\n答: 一个建子 4 元。\n\n【点睛】本题主要考查乘加的运算和倍的认识。", "solution": "null", "level": "二年级", "question": "\n\n36 元 $/$ 个\n\n\n\n8 元/副\n\n\n\n6 元 $/$ 个\n\n\n\n$? \\overline{\\text { 元 } / 个}$\n\n(1) 买 4 副乒乓球拍和一个足球一共需要多少钱?\n\n(2) 一副乒乓球拍的价钱是一个建子的 2 倍, 一个建子多少元?", "options": [], "subject": "算术", "analysis": "(1) 68 元\n\n(2) 4 元\n\n【分析】(1) 一副乒乓球拍 8 元, 一个足球 36 元, 求买 4 副乒乓球拍和一个足球一共需要的钱数,用乘法计算 4 副乒乓球拍的钱再加上一个足球的钱即可。\n\n(2) 一副乒乓球拍的价钱是一个建子的 2 倍, 根据倍的关系, 用除法计算一个建子的价钱。\n\n【详解】 (1) $4 \\times 8+36=68$ (元)\n\n答: 买 4 副乒乓球拍和一个足球一共需要 68 元。\n\n(2) $8 \\div 2=4($ 元 $)$\n\n答: 一个建子 4 元。\n\n【点睛】本题主要考查乘加的运算和倍的认识。"} {"id": "10510", "image": ["2154.jpg", "2155.jpg", "2156.jpg"], "answer": "(1) 18 支\n\n(2) 9 盒\n\n(3) 3 支", "solution": "null", "level": "二年级", "question": "\n\n彩笔\n\n\n\n彩笔\n\n\n\n彩笔\n\n(1) 一共有多少支彩笔?\n\n(2)把这些彩笔,每个盒子放 2 支,可以放几盒?\n\n(3) 把这些彩笔,放在 6 个盒子,平均每个盒子里有几支?", "options": [], "subject": "算术", "analysis": "(1) 18 支\n\n(2) 9 盒\n\n(3) 3 支"} {"id": "10522", "image": [], "answer": "$\\times$\n\n【分析】此题主要考查了乘法口诀的应用, 通常每句乘法口诀可以写出两道不同的除法算式, 但是也\n有部分两个因数相同的口诀, 只能写出一道除法算式, 例如: “三三得九”、“二二得四”等。\n\n【详解】根据乘法口诀“三三得九”只能写出一道除法算式: $9 \\div 3=3$, 原题说法错误。", "solution": "null", "level": "二年级", "question": "任何一句乘法口诀都可以写出两道不同的除法算式。( )", "options": [], "subject": "算术", "analysis": "$\\times$\n\n【分析】此题主要考查了乘法口诀的应用, 通常每句乘法口诀可以写出两道不同的除法算式, 但是也\n有部分两个因数相同的口诀, 只能写出一道除法算式, 例如: “三三得九”、“二二得四”等。\n\n【详解】根据乘法口诀“三三得九”只能写出一道除法算式: $9 \\div 3=3$, 原题说法错误。"} {"id": "10523", "image": [], "answer": "$\\times$", "solution": "null", "level": "二年级", "question": "被除数和除数相等时,商是 0 。( $)$", "options": [], "subject": "算术", "analysis": "$\\times$"} {"id": "10524", "image": [], "answer": "$\\sqrt{ }$", "solution": "null", "level": "二年级", "question": "49 里面有 7 个 7 。( $\\quad)$", "options": [], "subject": "算术", "analysis": "$\\sqrt{ }$"} {"id": "10525", "image": [], "answer": "V\n\n【分析】根据除法的意义及其计算方法, 由此解答即可。\n\n【详解】 $42 \\div 6=7$, 表示把 42 平均分成 6 份, 每份是 7 ; 原题说法正确;\n\n故答案为: $\\sqrt{ }$\n\n【点睛】此题考查除法的认识和平均分。", "solution": "null", "level": "二年级", "question": "$42 \\div 6=7$, 这个算式表示把 42 分成 6 份, 每份是 7。( )", "options": [], "subject": "算术", "analysis": "V\n\n【分析】根据除法的意义及其计算方法, 由此解答即可。\n\n【详解】 $42 \\div 6=7$, 表示把 42 平均分成 6 份, 每份是 7 ; 原题说法正确;\n\n故答案为: $\\sqrt{ }$\n\n【点睛】此题考查除法的认识和平均分。"} {"id": "10526", "image": [], "answer": "$\\times$", "solution": "null", "level": "二年级", "question": "求 40 里面有几个 8 , 用乘法计算。( $)$", "options": [], "subject": "算术", "analysis": "$\\times$"} {"id": "10530", "image": [], "answer": "7 条\n\n【分析】根据题意, 用划船的总人数除以每条船可以坐的人数, 即可求出需要多少条船。\n\n答: 需要 7 条船。\n\n【点睛】本题主要考查了整数除法的意义和实际应用, 要熟练掌握。", "solution": "null", "level": "二年级", "question": "28 个同学去公园划船, 如果每 4 个同学坐一条船, 需要多少条船?", "options": [], "subject": "算术", "analysis": "7 条\n\n【分析】根据题意, 用划船的总人数除以每条船可以坐的人数, 即可求出需要多少条船。\n\n答: 需要 7 条船。\n\n【点睛】本题主要考查了整数除法的意义和实际应用, 要熟练掌握。"} {"id": "10531", "image": [], "answer": "6 辆\n\n【分析】求需要多少辆这样的汽车, 即求 47 里面含有几个 9 , 根据求一个数里面含有几个另一个数,用除法解答。\n\n【详解】 $47 \\div 9=5$ (辆) $\\ldots . . . .2$ (人)\n\n$5+1=6$ (辆)\n答: 至少需要租 6 辆这样的车。\n\n【点睛】解答此题应根据求一个数里面含有几个另一个数, 用除法解答。", "solution": "null", "level": "二年级", "question": "小朋友去公园玩, 二 (6) 班有 47 人, 一辆面包车限乘 9 人, 至少需要租多少辆这样的车?", "options": [], "subject": "算术", "analysis": "6 辆\n\n【分析】求需要多少辆这样的汽车, 即求 47 里面含有几个 9 , 根据求一个数里面含有几个另一个数,用除法解答。\n\n【详解】 $47 \\div 9=5$ (辆) $\\ldots . . . .2$ (人)\n\n$5+1=6$ (辆)\n答: 至少需要租 6 辆这样的车。\n\n【点睛】解答此题应根据求一个数里面含有几个另一个数, 用除法解答。"} {"id": "10532", "image": [], "answer": "9 个\n\n【分析】求可以分给几个班, 就是求 27 里面有几个 3 , 用除法进行计算即可。\n\n【详解】 $27 \\div 3=9$ (个)\n\n答: 可以分给 9 个班。\n\n【点睛】掌握除法的意义及利用 9 的乘法口诀求商。", "solution": "null", "level": "二年级", "question": "体育老师买来 27 个篮球, 每个班分 3 个, 可以分给几个班?", "options": [], "subject": "算术", "analysis": "9 个\n\n【分析】求可以分给几个班, 就是求 27 里面有几个 3 , 用除法进行计算即可。\n\n【详解】 $27 \\div 3=9$ (个)\n\n答: 可以分给 9 个班。\n\n【点睛】掌握除法的意义及利用 9 的乘法口诀求商。"} {"id": "10533", "image": [], "answer": "1 元\n\n【分析】根据整数除法的意义, 用 24 除以 3 , 求出现在每个肉松面包的价钱; 再用肉松面包原来的价钱减去现在的价钱, 就是现在每个比原来便宜的钱数。据此解答。\n\n【详解】 $24 \\div 3=8($ 元 $)$\n\n$9-8=1 ($ 元 $)$\n\n答: 现在每个比原来便宜 1 元钱。\n\n【点睛】考查了整数除法和整数减法的实际应用, 先求出现在每个肉松面包的价钱是解决本题的关键。", "solution": "null", "level": "二年级", "question": "绿姿西饼屋肉松面包原来 9 元 1 个, 现在优惠促销, 3 个 24 元。现在每个比原来便宜了多少钱?", "options": [], "subject": "算术", "analysis": "1 元\n\n【分析】根据整数除法的意义, 用 24 除以 3 , 求出现在每个肉松面包的价钱; 再用肉松面包原来的价钱减去现在的价钱, 就是现在每个比原来便宜的钱数。据此解答。\n\n【详解】 $24 \\div 3=8($ 元 $)$\n\n$9-8=1 ($ 元 $)$\n\n答: 现在每个比原来便宜 1 元钱。\n\n【点睛】考查了整数除法和整数减法的实际应用, 先求出现在每个肉松面包的价钱是解决本题的关键。"} {"id": "10534", "image": [], "answer": "8 页\n\n【分析】求可插满多少页, 就相当于求 56 里面有几个 7 , 用除法计算。\n\n【详解】 $56 \\div 7=8$ (页)\n\n答: 可插满 8 页。\n\n【点睛】本题解答依据是: 包含除法的意义, 求一个数里面有几个几,用除法计算。", "solution": "null", "level": "二年级", "question": "有一本相册, 每页插 7 张照片, 把 56 张照片插到这本相册里, 可插满多少页?", "options": [], "subject": "算术", "analysis": "8 页\n\n【分析】求可插满多少页, 就相当于求 56 里面有几个 7 , 用除法计算。\n\n【详解】 $56 \\div 7=8$ (页)\n\n答: 可插满 8 页。\n\n【点睛】本题解答依据是: 包含除法的意义, 求一个数里面有几个几,用除法计算。"} {"id": "10550", "image": [], "answer": "V\n\n【分析 通常一句乘法口诀可以计算两道乘法算式和两道除法算式, 例如“七九六十三”可以计算 $7 \\times 9 、$ $9 \\times 7 、 63 \\div 7 、 63 \\div 9$, 据此解答。\n\n【详解】计算 $63 \\div 7$ 和 $63 \\div 9$ 时用同一句口诀“七九六十三”, 原题说法正确。故答案为: $\\sqrt{ }$\n\n【点睛】此题主要考查根据乘法、除法算式写出对应的乘法口诀, 要灵活运用乘法口诀。", "solution": "null", "level": "二年级", "question": "计算 $63 \\div 7$ 和 $63 \\div 9$ 时用同一句口诀。( )", "options": [], "subject": "算术", "analysis": "V\n\n【分析 通常一句乘法口诀可以计算两道乘法算式和两道除法算式, 例如“七九六十三”可以计算 $7 \\times 9 、$ $9 \\times 7 、 63 \\div 7 、 63 \\div 9$, 据此解答。\n\n【详解】计算 $63 \\div 7$ 和 $63 \\div 9$ 时用同一句口诀“七九六十三”, 原题说法正确。故答案为: $\\sqrt{ }$\n\n【点睛】此题主要考查根据乘法、除法算式写出对应的乘法口诀, 要灵活运用乘法口诀。"} {"id": "10551", "image": [], "answer": "$\\times$", "solution": "null", "level": "二年级", "question": "每一句口诀都可以写出两道不同的除法算式。( )", "options": [], "subject": "算术", "analysis": "$\\times$"} {"id": "10552", "image": [], "answer": "$\\sqrt{ }$\n\n【分析】用绳子的长度除以每根的长度就是剪的根数, 然后用根数减去 1 就是剪的次数。\n\n【详解】 $24 \\div 3=8$ (根)\n\n$8-1=7$ (次)\n\n需要剪 7 次。\n\n故答案为: $\\sqrt{ }$\n\n【点睛】本题主要考查植树问题, 关键利用植树问题公式计算, 注意所剪根数与次数的关系。", "solution": "null", "level": "二年级", "question": "把 24 米长的绳子剪成每根 3 米长的跳绳, 需要剪 7 次。( )", "options": [], "subject": "算术", "analysis": "$\\sqrt{ }$\n\n【分析】用绳子的长度除以每根的长度就是剪的根数, 然后用根数减去 1 就是剪的次数。\n\n【详解】 $24 \\div 3=8$ (根)\n\n$8-1=7$ (次)\n\n需要剪 7 次。\n\n故答案为: $\\sqrt{ }$\n\n【点睛】本题主要考查植树问题, 关键利用植树问题公式计算, 注意所剪根数与次数的关系。"} {"id": "10553", "image": [], "answer": "V\n\n【分析】用鸡蛋的总个数除以每盒装的个数, 就是一共可以装的盒数。\n\n【详解】 $42 \\div 6=7$ (盒)\n\n一共可以装 7 盒, 所以原题干正确。\n\n故答案为: $\\sqrt{ }$\n\n【点睛】求一个数里面有几个几, 用除法计算。", "solution": "null", "level": "二年级", "question": "42 个鸡蛋, 每 6 个装一盒可以装 7 盒。( )", "options": [], "subject": "算术", "analysis": "V\n\n【分析】用鸡蛋的总个数除以每盒装的个数, 就是一共可以装的盒数。\n\n【详解】 $42 \\div 6=7$ (盒)\n\n一共可以装 7 盒, 所以原题干正确。\n\n故答案为: $\\sqrt{ }$\n\n【点睛】求一个数里面有几个几, 用除法计算。"} {"id": "10558", "image": [], "answer": "8 张", "solution": "null", "level": "二年级", "question": "冷饮店来了 48 位客人, 如果每 6 位客人坐一桌, 要准备几张桌子?", "options": [], "subject": "算术", "analysis": "8 张"} {"id": "10559", "image": [], "answer": "6 盒\n\n【详解】略", "solution": "null", "level": "二年级", "question": "淘气有 54 个乒乓球, 9 个装一盒, 可以装几盒?", "options": [], "subject": "算术", "analysis": "6 盒\n\n【详解】略"} {"id": "10560", "image": [], "answer": "(1) 7 辆; (2) 6 张\n\n【解析】略", "solution": "null", "level": "二年级", "question": "二年级有 54 人去动物园春游。\n\n(1) 每辆面包车限坐 8 人, 至少需要租几辆车?\n\n(2) 动物园门票每张 8 元, 50 元最多能买几张票?", "options": [], "subject": "算术", "analysis": "(1) 7 辆; (2) 6 张\n\n【解析】略"} {"id": "10561", "image": ["2167.jpg"], "answer": "$28 \\div 7=4$ (周)\n\n【解析】略", "solution": "null", "level": "二年级", "question": "还有几周“阿波罗”号发射?\n\n", "options": [], "subject": "算术", "analysis": "$28 \\div 7=4$ (周)\n\n【解析】略"} {"id": "10563", "image": ["2172.jpg", "2173.jpg", "2174.jpg", "2175.jpg"], "answer": "(1) 8 个;\n(2) 8 元;\n(3) 8 元;\n(4) 24 元可以买几只牙刷? 6 只。\n\n【详解】略", "solution": "null", "level": "二年级", "question": "\n\n9 元\n\n\n\n6 元\n\n\n\n4 元\n\n\n\n$?$ 元\n\n(1) 72 元一共可以买几个杯子?\n\n(2) 24 元一共买了 3 条毛巾,一条毛巾多少钱?\n\n(3) 小明有 20 元钱, 买一支牙刷和一条毛巾后, 还剩多少元?\n\n(4)你还能提出其他用乘法或除法解决的问题并解答?", "options": [], "subject": "算术", "analysis": "(1) 8 个;\n(2) 8 元;\n(3) 8 元;\n(4) 24 元可以买几只牙刷? 6 只。\n\n【详解】略"} {"id": "10577", "image": [], "answer": "$\\times$\n\n【解析】略", "solution": "null", "level": "二年级", "question": "除数和被除数都是 8 , 商是 64 。( )", "options": [], "subject": "算术", "analysis": "$\\times$\n\n【解析】略"} {"id": "10578", "image": [], "answer": "V", "solution": "null", "level": "二年级", "question": "根据一句乘法口诀, 可以写出两道或一道除法算式。( )", "options": [], "subject": "算术", "analysis": "V"} {"id": "10579", "image": [], "answer": "$\\times$\n\n【详解】略", "solution": "null", "level": "二年级", "question": "$32 \\div 4=8$ 和 $32 \\div 8=4$ 表示的意义是一样的。( $\\quad)$", "options": [], "subject": "算术", "analysis": "$\\times$\n\n【详解】略"} {"id": "10580", "image": [], "answer": "$\\times$", "solution": "null", "level": "二年级", "question": "把 49 分成 7 份, 每份是 7。( $\\quad)$", "options": [], "subject": "算术", "analysis": "$\\times$"} {"id": "10581", "image": [], "answer": "V", "solution": "null", "level": "二年级", "question": "有 28 块饼干, 每 7 块装一袋, 可以装 4 袋。( )", "options": [], "subject": "算术", "analysis": "V"} {"id": "10586", "image": [], "answer": "$18 \\div 3 \\times 6=36$ (页)\n\n【解析】略", "solution": "null", "level": "二年级", "question": "3 本同样的钢笔字贴共 18 页, 那么 6 本同样的钢笔字帖共多少页?", "options": [], "subject": "算术", "analysis": "$18 \\div 3 \\times 6=36$ (页)\n\n【解析】略"} {"id": "10587", "image": [], "answer": "9 个", "solution": "null", "level": "二年级", "question": "36 枝花, 每个花瓶里插 4 枝, 需要几个花瓶?", "options": [], "subject": "算术", "analysis": "9 个"} {"id": "10588", "image": [], "answer": "5 天", "solution": "null", "level": "二年级", "question": "小明看一本 81 页的书, 每天看 9 页, 已经看了 36 页, 剩下的书还要几天看完?", "options": [], "subject": "算术", "analysis": "5 天"} {"id": "10589", "image": [], "answer": "(1) 9 盆;\n\n(2) 45 朵\n\n【详解】(1) $63 \\div 7=9$ (盆)\n\n答: 每排摆 9 盆花。\n\n(2) $5 \\times 9=45$ (杂)\n\n答: 一排共有 45 朵花。", "solution": "null", "level": "二年级", "question": "志愿者准备了 63 盆花来装饰公园, 把这些花平均摆成 7 排放在一个花坛里。\n\n(1) 每排摆多少盆花?\n\n(2) 如果每个花盆里有 5 朵花, 那么一排共有多少朵花?", "options": [], "subject": "算术", "analysis": "(1) 9 盆;\n\n(2) 45 朵\n\n【详解】(1) $63 \\div 7=9$ (盆)\n\n答: 每排摆 9 盆花。\n\n(2) $5 \\times 9=45$ (杂)\n\n答: 一排共有 45 朵花。"} {"id": "10603", "image": [], "answer": "$x$\n\n【分析】计算 $3 \\times 6$, 用乘法口诀三六十八; 计算 $16 \\div 8$, 用乘法口诀二八十六。据此判断即可。\n\n【详解】计算 $3 \\times 6$, 用乘法口诀三六十八; 计算 $16 \\div 8$, 用乘法口诀二八十六。所以它们不是用同一句乘法口诀, 原题干错误。\n\n故答案为: $x$ 。\n\n【点睛】这道题解题的关键是要熟练掌握乘法口诀。", "solution": "null", "level": "二年级", "question": "计算 $3 \\times 6$ 和 $16 \\div 8$ 时, 用同一句乘法口诀: 三六十八。( )", "options": [], "subject": "算术", "analysis": "$x$\n\n【分析】计算 $3 \\times 6$, 用乘法口诀三六十八; 计算 $16 \\div 8$, 用乘法口诀二八十六。据此判断即可。\n\n【详解】计算 $3 \\times 6$, 用乘法口诀三六十八; 计算 $16 \\div 8$, 用乘法口诀二八十六。所以它们不是用同一句乘法口诀, 原题干错误。\n\n故答案为: $x$ 。\n\n【点睛】这道题解题的关键是要熟练掌握乘法口诀。"} {"id": "10604", "image": [], "answer": "$\\sqrt{ }$\n\n【分析】根据表内乘法和除法的计算方法, 以及乘法和除法之间的关系, 一句口诀可以写出两道乘法算式和两道除法算式, 由此解答。\n\n【详解】计算 $7 \\times 8$ 和 $56 \\div 8$ 用同一句乘法口诀:七八五十六, 说法正确。\n\n故答案为: $\\sqrt{ }$\n\n【点睛】此题主要考查根据乘法口诀写乘法、除法算式, 要灵活运用乘法口诀。", "solution": "null", "level": "二年级", "question": "计算 $7 \\times 8$ 和 $56 \\div 8$ 用到的乘法口诀都是七八五十六。( )", "options": [], "subject": "算术", "analysis": "$\\sqrt{ }$\n\n【分析】根据表内乘法和除法的计算方法, 以及乘法和除法之间的关系, 一句口诀可以写出两道乘法算式和两道除法算式, 由此解答。\n\n【详解】计算 $7 \\times 8$ 和 $56 \\div 8$ 用同一句乘法口诀:七八五十六, 说法正确。\n\n故答案为: $\\sqrt{ }$\n\n【点睛】此题主要考查根据乘法口诀写乘法、除法算式, 要灵活运用乘法口诀。"} {"id": "10606", "image": [], "answer": "$\\times$", "solution": "null", "level": "二年级", "question": "$81 \\div 9$ 的结果和 $3 \\times 3$ 的结果相同, 表示的意义也相同。( )", "options": [], "subject": "算术", "analysis": "$\\times$"} {"id": "10607", "image": [], "answer": "$\\sqrt{ }$", "solution": "null", "level": "二年级", "question": "把 42 颗糖分给 6 个小组, 每小组得 7 颗糖。( $)$", "options": [], "subject": "算术", "analysis": "$\\sqrt{ }$"} {"id": "10611", "image": [], "answer": "2 元; 2", "solution": "null", "level": "二年级", "question": "饼干原价 10 元一包, 现在促销, 3 包 24 元。现在平均每包的价钱比原价便宜多少钱?", "options": [], "subject": "算术", "analysis": "2 元; 2"} {"id": "10612", "image": [], "answer": "8", "solution": "null", "level": "二年级", "question": "24 枝花平均插在 3 个花瓶里, 每个花瓶插几枝花?", "options": [], "subject": "算术", "analysis": "8"} {"id": "10613", "image": ["2181.jpg"], "answer": "32 根", "solution": "null", "level": "二年级", "question": "小兔们一共拔了 48 根萝卜。\n\n", "options": [], "subject": "算术", "analysis": "32 根"} {"id": "10614", "image": [], "answer": "(1) 36 盆\n\n(2) 9 盆", "solution": "null", "level": "二年级", "question": "滨湖公园培育一批盆花, 如果每行摆 6 盆, 正好能摆 6 行。\n\n(1)这批盆花一共有多少盆?\n\n(2) 平均摆成 4 行,每行摆多少盆?", "options": [], "subject": "算术", "analysis": "(1) 36 盆\n\n(2) 9 盆"} {"id": "10615", "image": ["2182.jpg"], "answer": "(1) $42 \\div 6=7$ (个) $; 7$\n\n(2) $42 \\div 7=6$ (个) $; 6$", "solution": "null", "level": "二年级", "question": "\n\n共 42 个气球\n\n(1) 平均每个小朋友分几个?\n\n(2) 如果每个小朋友分到 7 个, 可以分给几个小朋友?", "options": [], "subject": "算术", "analysis": "(1) $42 \\div 6=7$ (个) $; 7$\n\n(2) $42 \\div 7=6$ (个) $; 6$"} {"id": "10629", "image": [], "answer": "$\\sqrt{ }$\n\n【分析】按照 8 的乘法口诀进行解答即可。\n\n【详解】由分析可得:计算 $6 \\times 8$ 与 $48 \\div 8$ 用的乘法口诀都是六八四十八, 原题说法正确。\n\n故答案为: $\\sqrt{ } \\sqrt{ }$\n【点睛】本题考查学生对用 8 的乘法口诀求商的运用, 要求学生牢记乘法口诀。", "solution": "null", "level": "二年级", "question": "计算 $6 \\times 8$ 与 $48 \\div 8$ 用的乘法口诀都是六八四十八。( )", "options": [], "subject": "算术", "analysis": "$\\sqrt{ }$\n\n【分析】按照 8 的乘法口诀进行解答即可。\n\n【详解】由分析可得:计算 $6 \\times 8$ 与 $48 \\div 8$ 用的乘法口诀都是六八四十八, 原题说法正确。\n\n故答案为: $\\sqrt{ } \\sqrt{ }$\n【点睛】本题考查学生对用 8 的乘法口诀求商的运用, 要求学生牢记乘法口诀。"} {"id": "10630", "image": [], "answer": "$\\times$", "solution": "null", "level": "二年级", "question": "一根绳子长 12 米, 对折两次后, 每段长 6 米。( )", "options": [], "subject": "算术", "analysis": "$\\times$"} {"id": "10631", "image": [], "answer": "$\\times$", "solution": "null", "level": "二年级", "question": "除数和被除数都是 8 , 商是 64 。( )", "options": [], "subject": "算术", "analysis": "$\\times$\n\n【解析】略"} {"id": "11026", "image": [], "answer": "$\\times$", "solution": "null", "level": "二年级", "question": "2 千克砖头比 2 千克泡沫箱重。", "options": [], "subject": "计数", "analysis": "$\\times$"} {"id": "11027", "image": [], "answer": "$\\sqrt{ }$", "solution": "null", "level": "二年级", "question": "小丽身高 132 厘米, 体重 30 千克。", "options": [], "subject": "计数", "analysis": "$\\sqrt{ }$"} {"id": "10291", "image": [], "answer": "解: $38-9+12=41$ (名).\n\n答:现在有 41 名乘客。\n\n27. $_详解93-18-20$\n\n$=75-20$\n\n$=55($ 页 $)$\n\n答:还剩 55 页没有看.", "solution": "null", "level": "二年级", "question": "( 5 分 ) 公共汽车上原有 38 名乘客。到了活动中心这一站以后, 有 9 个乘客下车,有 12 名乘客上车,现在有多少名乘客?", "options": [], "subject": "计数", "analysis": "解: $38-9+12=41$ (名).\n\n答:现在有 41 名乘客。\n\n27. $_详解93-18-20$\n\n$=75-20$\n\n$=55($ 页 $)$\n\n答:还剩 55 页没有看."} {"id": "10352", "image": ["2093.jpg"], "answer": "解: $5 \\times 5=25$ (颗)答:一共有 25 颗星星", "solution": "null", "level": "二年级", "question": "( 5 分 )一共有多少颗星星?\n\n", "options": [], "subject": "计数", "analysis": "解: $5 \\times 5=25$ (颗)答:一共有 25 颗星星"} {"id": "10415", "image": [], "answer": "解: $4 \\times 7=28$ (个)\n\n答: 这些小汽车一共有 28 个车轮。", "solution": "null", "level": "二年级", "question": "( 5 分 ) 这些小汽车一共有多少个车轮?", "options": [], "subject": "计数", "analysis": "解: $4 \\times 7=28$ (个)\n\n答: 这些小汽车一共有 28 个车轮。"} {"id": "10416", "image": ["2124.jpg"], "answer": "解: $4 \\times 9=36$ (人)\n\n答: 一共可坐 36 人。", "solution": "null", "level": "二年级", "question": "( 5 分 )一共可以坐多少人?\n", "options": [], "subject": "计数", "analysis": "解: $4 \\times 9=36$ (人)\n\n答: 一共可坐 36 人。"} {"id": "11445", "image": [], "answer": "12 \\div 3 \\times 4=16$ (个)", "solution": "null", "level": "二年级", "question": "创新题) 12 个小朋友进行乒乓球比赛, 每 3 人发给 4 个乒乓球, 一共需要多少个乒乓球?", "options": [], "subject": "计数", "analysis": "12 \\div 3 \\times 4=16$ (个)"} {"id": "10535", "image": [], "answer": "4 天\n\n【分析】每天看 6 页, 6 天看完, 先用每天看的页数乘上 6 求出总页数, 再用总页数除以 9 页, 即可求出需要的天数。\n\n$36 \\div 9$\n\n$=4$ (天)\n\n答: 4 天可以看完。\n\n【点睛 】$\\mathbf{~}$工作效率求解。", "solution": "null", "level": "二年级", "question": "苗苗和亮亮都喜欢看《趣味数学》。", "options": [], "subject": "计数", "analysis": "4 天\n\n【分析】每天看 6 页, 6 天看完, 先用每天看的页数乘上 6 求出总页数, 再用总页数除以 9 页, 即可求出需要的天数。\n\n$36 \\div 9$\n\n$=4$ (天)\n\n答: 4 天可以看完。\n\n【点睛 】$\\mathbf{~}$工作效率求解。"} {"id": "10562", "image": ["2168.jpg", "2169.jpg", "2170.jpg", "2171.jpg"], "answer": "可能买了 2 盒巧克力或可能买了 3 袋水果糖\n\n【解析】略", "solution": "null", "level": "二年级", "question": "图图买一种食品正好用了 18 元, 你知道图图可能买的是哪种食品吗?买了多少呢?\n\n\n\n饼干\n\n4 元\n\n\n\n巧克力\n\n\n\n蛋糕\n\n\n\n水果糖", "options": [], "subject": "计数", "analysis": "可能买了 2 盒巧克力或可能买了 3 袋水果糖\n\n【解析】略"} {"id": "10628", "image": [], "answer": "$\\times$", "solution": "null", "level": "二年级", "question": "24 个足球, 分给 8 个同学, 每个同学分 3 个。( )", "options": [], "subject": "计数", "analysis": "$\\times$"} {"id": "10703", "image": [], "answer": "$\\times$", "solution": "null", "level": "二年级", "question": "38 里面最多有 7 个 6。()", "options": [], "subject": "计数", "analysis": "$\\times$"} {"id": "11041", "image": ["2288.jpg"], "answer": "小羊和袋鼠坐在限载 300 千克的船: $100+180=280$ (千克)\n\n5000 克 $=5$ 千克\n\n小兔、老虎和大象坐在限载 1000 千克的船: $5+350+600=955$ (千克)\n\n答: 小羊和袋鼠坐在限载 300 千克的船, 小兔、老虎和大象坐在限载 1000 千克的船。", "solution": "null", "level": "二年级", "question": "5 个小动物要同时过河, 该怎样乘船?\n\n\n答: (\n) 坐在限载 300 千克的船, (\n) 坐在限载 1000 千克的船。", "options": [], "subject": "组合数学", "analysis": "小羊和袋鼠坐在限载 300 千克的船: $100+180=280$ (千克)\n\n5000 克 $=5$ 千克\n\n小兔、老虎和大象坐在限载 1000 千克的船: $5+350+600=955$ (千克)\n\n答: 小羊和袋鼠坐在限载 300 千克的船, 小兔、老虎和大象坐在限载 1000 千克的船。"} {"id": "10319", "image": ["2071.jpg"], "answer": "解: $3+2+1=6($ 个 $)$", "solution": "null", "level": "二年级", "question": "( 5 分 )请你数一数, 下图中共有多少个角?\n\n", "options": [], "subject": "组合数学", "analysis": "解: $3+2+1=6($ 个 $)$"} {"id": "10342", "image": [], "answer": "20", "solution": "null", "level": "二年级", "question": "在五边形的花坛上摆花盆, 每条边上摆 5 盆, 至少需要()盆。", "options": [], "subject": "组合数学", "analysis": "20"} {"id": "9965", "image": ["1896.jpg"], "answer": "阳", "solution": "null", "level": "二年级", "question": "给他们排位置。(填名字) (8 分)\n\n", "options": [], "subject": "组合数学", "analysis": "阳"} {"id": "9967", "image": ["1898.jpg", "1899.jpg", "1900.jpg", "1901.jpg", "1902.jpg"], "answer": "乔 琪琪 妈妈 爸爸", "solution": "null", "level": "二年级", "question": "图回答问题。(12 分)\n\n\n\n爸爸\n\n\n\n(1)\n\n\n\n(2)\n\n\n\n(3)\n\n\n\n(4)\n\n(1) 照片(1)是 ( $\\quad$ ) 拍的。\n\n(2)照片(2)是 $(\\quad)$ 拍的。\n\n(3) 照片 (3)是 $(\\quad)$ 拍的。\n\n(4) 照片(4)是 $(\\quad)$ 拍的。", "options": [], "subject": "组合数学", "analysis": "乔 琪琪 妈妈 爸爸"} {"id": "10428", "image": [], "answer": "6", "solution": "null", "level": "二年级", "question": "两件上衣、三条裤子有()种搭配穿法。", "options": [], "subject": "组合数学", "analysis": "6"} {"id": "10429", "image": [], "answer": "6", "solution": "null", "level": "二年级", "question": "小红有三件不同款式的上衣和两条不同款式的裤子, 他可以有()种\n不同的穿法。", "options": [], "subject": "组合数学", "analysis": "6"} {"id": "10430", "image": [], "answer": "3", "solution": "null", "level": "二年级", "question": "从三本书里,任意选其中两本书,一共有()种不同选法。", "options": [], "subject": "组合数学", "analysis": "3"} {"id": "10248", "image": [], "answer": "米; 厘米", "solution": "null", "level": "二年级", "question": "在横线上填 “米”或 “厘米”。\n\n学校的旗杆高约 15()一本《新华字典》大约厚 3()", "options": [], "subject": "度量几何学", "analysis": "米; 厘米"} {"id": "10250", "image": [], "answer": "直尺; 厘米; 米", "solution": "null", "level": "二年级", "question": "要知道物体的长度, 可以用 ( ) 来量。我们学过的长度单位有 ( )和 $(\\quad)$ 。", "options": [], "subject": "度量几何学", "analysis": "直尺; 厘米; 米"} {"id": "10251", "image": [], "answer": "$60 ; 50 ; 3 ; 8$", "solution": "null", "level": "二年级", "question": "$6 \\mathrm{~m}=(\\quad) \\mathrm{dm}$\n\n$$\n5 \\mathrm{dm}=(\\quad) \\mathrm{cm}\n$$\n\n$30 \\mathrm{~cm}=(\\quad) \\mathrm{dm}$\n\n$80 \\mathrm{dm}=(\\quad) \\mathrm{m}$", "options": [], "subject": "度量几何学", "analysis": "$60 ; 50 ; 3 ; 8$"} {"id": "10253", "image": ["2045.jpg"], "answer": "70 米; 7 米; 1 米; 70 厘米; 7 厘米", "solution": "null", "level": "二年级", "question": "按从长到短的顺序排一排。\n\n\n$(\\quad)>(\\quad)>(\\quad)>(\\quad)>(\\quad)$", "options": [], "subject": "度量几何学", "analysis": "70 米; 7 米; 1 米; 70 厘米; 7 厘米"} {"id": "10254", "image": [], "answer": "<; <; > $<$; $=$", "solution": "null", "level": "二年级", "question": "在横线上填上 “>”、“<”或 “=”.\n\n50 厘米( $\\quad 1$ 米\n\n200 厘米( ) 1 米 30 厘米", "options": [], "subject": "度量几何学", "analysis": "<; <; > $<$; $=$"} {"id": "10261", "image": ["2048.jpg", "2049.jpg"], "answer": "解: $300+500=800$ (米)", "solution": "null", "level": "二年级", "question": "( 6 分 ) 小明家离学校 300 米, 小红家离学校 500 米, 问小明家离小红家多少米?\n\n\n\n学校\n\n\n\n小明家, \\$ \\qquad \\$\n小红家", "options": [], "subject": "度量几何学", "analysis": "解: $300+500=800$ (米)"} {"id": "10303", "image": [], "answer": "直; 锐", "solution": "null", "level": "二年级", "question": "在学生用的一个三角板中, 有一个()角和两个()角。", "options": [], "subject": "度量几何学", "analysis": "直; 锐"} {"id": "10304", "image": [], "answer": "钝角; 锐角", "solution": "null", "level": "二年级", "question": "在直角、钝角、锐角中, 度数最大的是( ), 度数最小的是()。", "options": [], "subject": "度量几何学", "analysis": "钝角; 锐角"} {"id": "10310", "image": [], "answer": "直", "solution": "null", "level": "二年级", "question": "9 时整时, 钟面上的时针和分针成 ( ) 角。", "options": [], "subject": "度量几何学", "analysis": "直"} {"id": "10317", "image": ["2067.jpg", "2068.jpg", "2068.jpg"], "answer": "", "solution": "null", "level": "二年级", "question": "( 5 分 $)$\n", "options": [], "subject": "度量几何学", "analysis": ""} {"id": "10318", "image": ["2069.jpg", "2070.jpg", "2070.jpg"], "answer": "", "solution": "null", "level": "二年级", "question": "( 5 分) 连线。\n\n\n(直角 锐角", "options": [], "subject": "度量几何学", "analysis": ""} {"id": "10320", "image": ["2072.jpg"], "answer": "解:把三角形切一刀, 可以剩 3 个角, 也可以剩 4 个角。", "solution": "null", "level": "二年级", "question": "5 分 )把下面的三角形切一刀,还剩几个角呢?\n\n", "options": [], "subject": "度量几何学", "analysis": "解:把三角形切一刀, 可以剩 3 个角, 也可以剩 4 个角。"} {"id": "10321", "image": ["2073.jpg"], "answer": "解: 8 个直角, 8 个钝角, 8 个锐角", "solution": "null", "level": "二年级", "question": "(6 分)观察图形,一共有多少个直角? 一共有多少个钝角? 一共有多少个锐角?\n\n", "options": [], "subject": "度量几何学", "analysis": "解: 8 个直角, 8 个钝角, 8 个锐角"} {"id": "10335", "image": ["2078.jpg"], "answer": "20", "solution": "null", "level": "二年级", "question": "黑带子长 5 厘米, 白带子长 ( ) 厘米。\n", "options": [], "subject": "度量几何学", "analysis": "20"} {"id": "10305", "image": [], "answer": "4; 2; 1; 1", "solution": "null", "level": "二年级", "question": "图形 $\\square$ 中有()个角: 其中有()个直角, 有()个钝角,有 $(\\quad)$ 个锐角。", "options": [], "subject": "画法几何学", "analysis": "4; 2; 1; 1"} {"id": "10308", "image": ["2056.jpg"], "answer": "$1 ; 5$", "solution": "null", "level": "二年级", "question": "图中有()个直角,共有()条线段。\n\n", "options": [], "subject": "画法几何学", "analysis": "$1 ; 5$"} {"id": "10375", "image": ["2098.jpg"], "answer": "3", "solution": "null", "level": "二年级", "question": "图形\n\n", "options": [], "subject": "画法几何学", "analysis": ""} {"id": "10379", "image": ["2103.jpg", "2103.jpg"], "answer": "解: 如图所示:\n\n", "solution": "null", "level": "二年级", "question": "(3 分) 在距离 左边 2 厘米处画一个出, 在距离 右边 3 厘米处画一个 $\\diamond$ 。", "options": [], "subject": "画法几何学", "analysis": "解: 如图所示:\n\n"} {"id": "10306", "image": [], "answer": "2; 角", "solution": "null", "level": "二年级", "question": "从一个点起, 用尺子向不同的方向画 ( ) 条直直的线, 就画成了()。", "options": [], "subject": "立体几何学", "analysis": "2; 角"} {"id": "10307", "image": [], "answer": "一; 两", "solution": "null", "level": "二年级", "question": "一个角有()个个顶点,()个条边。", "options": [], "subject": "立体几何学", "analysis": "一; 两"} {"id": "9960", "image": ["1874.jpg", "1875.jpg", "1875.jpg"], "answer": "只小动物在观察长方体, 连一连。(9 分)\n", "solution": "null", "level": "二年级", "question": "只小动物在观察长方体, 连一连。(9 分)\n", "options": [], "subject": "立体几何学", "analysis": "只小动物在观察长方体, 连一连。(9 分)\n"} {"id": "10309", "image": ["2057.jpg"], "answer": "$6 ; 6$", "solution": "null", "level": "二年级", "question": "", "options": [], "subject": "组合几何学", "analysis": ":分析:(1)由点 $\\mathrm{A}$ 在抛物线上, 将 $\\mathrm{A}$ 点坐标代入, 求出参数 $\\mathrm{P}$, 求解即可 (2) 由于 $F(8,0)$ 是 $\\triangle A B C$ 的重心, 则重心与焦点重合, 由重心坐标公式可求 $M$ 是 $B C$ 的中点。\n\n(3) 由于线段 $\\mathrm{BC}$ 的中点 $\\mathrm{M}$ 不在 $x$ 轴上, 所以 $\\mathrm{BC}$ 所在的直线不垂直于 $x$ 轴. 设 $\\mathrm{BC}$ 所在直线的方程为: $y+4=k(x-11)(k \\neq 0)$, 解出 $k$ 即可。"} {"id": "10311", "image": ["2058.jpg", "2059.jpg", "2059.jpg"], "answer": "", "solution": "null", "level": "二年级", "question": "图形\n\n", "options": [], "subject": "组合几何学", "analysis": ""} {"id": "9963", "image": ["1890.jpg", "1891.jpg", "1891.jpg"], "answer": "", "solution": "null", "level": "二年级", "question": "幅图是小鸟看到的?在里画 “ $\\sqrt{ }$ ”。哪幅图是大象看到的?在里画 “ $\\bigcirc$ ”。(6 分)\n", "options": [], "subject": "图论", "analysis": ""} {"id": "10445", "image": ["2134.jpg", "2135.jpg", "2136.jpg", "2136.jpg"], "answer": "", "solution": "null", "level": "二年级", "question": "( 4 分) 连一连。\n\n\n$8: 50$\n\n$7: 05$\n\n$4: 27$\n\n$12: 10$\n\n\n\n\n5 时 39 分\n\n9时09分\n\n2 时 51 分\n\n3 时53分\n\n$9: 09$", "options": [], "subject": "逻辑题", "analysis": ""} {"id": "11446", "image": [], "answer": "阳一定不会出来, 因为是在晚上。提示: 中午下着雨, 再过 24 小时又是一个中午, 再加上 12 小时等于 36 小时, 此时正好是晚上。所以太阳一定不会出来。提示:把 10 分拆成不同的三数之和,共有四种不同的分拆方式: $10=1+2+7=1+3+6=1+4+5=2+3+5$,选择有相同加数的两个式子, 把共同的加数填在中间的 $\\mathrm{O}$ 内,其他 4 个加数分别填在两头的 O 里就构成一种填法。", "solution": "null", "level": "二年级", "question": "信息题)一天中午正在下着大雨, 明明问菲菲: “再过 36 小时太阳会不会出来?” 菲菲说: “那可说不定。” 明明说: “我可以肯定, 再过 36 个小时太阳一定不会出来。” 小朋友, 你们认为谁说得对? 为什么?", "options": [], "subject": "逻辑题", "analysis": "阳一定不会出来, 因为是在晚上。提示: 中午下着雨, 再过 24 小时又是一个中午, 再加上 12 小时等于 36 小时, 此时正好是晚上。所以太阳一定不会出来。提示:把 10 分拆成不同的三数之和,共有四种不同的分拆方式: $10=1+2+7=1+3+6=1+4+5=2+3+5$,选择有相同加数的两个式子, 把共同的加数填在中间的 $\\mathrm{O}$ 内,其他 4 个加数分别填在两头的 O 里就构成一种填法。"} {"id": "10664", "image": ["2208.jpg", "2209.jpg"], "answer": "方片 5\n\n【详解】把一张牌倒过来, 就是把牌旋转了 $180^{\\circ}$ 。黑桃 $8 、$ 红桃 7 、梅花 6 这三张牌旋转了 $180^{\\circ}$ 后,都会有所变化. 方块中的菱形旋转 $180^{\\circ}$ 后和原图形状相比无变化.", "solution": "null", "level": "二年级", "question": "四张扑克牌放在桌子上, 王老师背过身去, 请小强将其中一张牌倒过来。王老师回转身看到颠倒后的扑克牌, 很快猜出了哪一个。你能猜出来吗?\n\n\n\n颠倒前\n\n\n\n颠倒后", "options": [], "subject": "逻辑题", "analysis": "方片 5\n\n【详解】把一张牌倒过来, 就是把牌旋转了 $180^{\\circ}$ 。黑桃 $8 、$ 红桃 7 、梅花 6 这三张牌旋转了 $180^{\\circ}$ 后,都会有所变化. 方块中的菱形旋转 $180^{\\circ}$ 后和原图形状相比无变化."} {"id": "10652", "image": [], "answer": "$\\sqrt{ }$\n\n【分析】平移是指在平面内, 将一个图形上的所有点都按照某个方向作相同距离的移动。\n\n【详解】根据平移的定义, 可知一叶扁舟在水中直线漂流属平移现象是正确的。故答案为: $\\sqrt{ } \\sqrt{ }$\n\n【点睛】此题考查同学熟记平移的定义。", "solution": "null", "level": "二年级", "question": "一叶扁舟在水中直线漂流属平移现象。( )", "options": [], "subject": "变换几何", "analysis": "$\\sqrt{ }$\n\n【分析】平移是指在平面内, 将一个图形上的所有点都按照某个方向作相同距离的移动。\n\n【详解】根据平移的定义, 可知一叶扁舟在水中直线漂流属平移现象是正确的。故答案为: $\\sqrt{ } \\sqrt{ }$\n\n【点睛】此题考查同学熟记平移的定义。"} {"id": "10653", "image": ["2196.jpg", "2197.jpg", "2197.jpg"], "answer": "$\\times$\n\n【分析】根据轴对称图形的意义: 如果一个图形沿着一条直线对折后两部分完全重合, 这样的图形叫做轴对称图形,这条直线叫做对称轴; 据此判断即可。\n\n【详解】\n\n\n不是轴对称图形,原题说法错误。故答案为: $\\times$ 。\n\n【点睛】解答此题的主要依据是: 轴对称图形的概念及特征。", "solution": "null", "level": "二年级", "question": "\n是轴对称图形。( $\\quad)$", "options": [], "subject": "变换几何", "analysis": "$\\times$\n\n【分析】根据轴对称图形的意义: 如果一个图形沿着一条直线对折后两部分完全重合, 这样的图形叫做轴对称图形,这条直线叫做对称轴; 据此判断即可。\n\n【详解】\n\n\n不是轴对称图形,原题说法错误。故答案为: $\\times$ 。\n\n【点睛】解答此题的主要依据是: 轴对称图形的概念及特征。"} {"id": "10655", "image": [], "answer": "$\\sqrt{ }$\n【分析】根据平移、旋转的特征可知, 平移是将一个图形上的所有点都按照某个方向作相同距离的移动, 旋转是把一个图形绕着某一点转动一定角度的图形变换。\n\n【详解】根据分析可知: 用卷笔刀削铅笔是旋转现象, 水银温度计中液柱的升降是平移现象; 所以判断正确。\n\n故答案为: $\\sqrt{ }$\n\n【点睛】本题主要考查平移和旋转的意义, 对平移与旋转理解及在实际生活当中的运用。", "solution": "null", "level": "二年级", "question": "用卷笔刀削铅笔是旋转现象,水银温度计中液柱的升降是平移现象。( )", "options": [], "subject": "变换几何", "analysis": "$\\sqrt{ }$\n【分析】根据平移、旋转的特征可知, 平移是将一个图形上的所有点都按照某个方向作相同距离的移动, 旋转是把一个图形绕着某一点转动一定角度的图形变换。\n\n【详解】根据分析可知: 用卷笔刀削铅笔是旋转现象, 水银温度计中液柱的升降是平移现象; 所以判断正确。\n\n故答案为: $\\sqrt{ }$\n\n【点睛】本题主要考查平移和旋转的意义, 对平移与旋转理解及在实际生活当中的运用。"} {"id": "10654", "image": [], "answer": "$\\sqrt{ }$\n\n【分析】依据轴对称图形的意义, 即在同一个平面内, 一个图形沿某条直线对折, 对折后的两部分都能完全重合, 则这个图形就是轴对称图形, 这条直线就是其对称轴, 据此可知长方形和正方形都是轴对称图形, 正方形有 4 条对称轴, 长方形有 2 条对称轴, 由此解答即可。\n\n【详解】长方形和正方形都是轴对称图形, 正方形有 4 条对称轴, 长方形有 2 条对称轴, 所以正方形比长方形对称轴多, 题干说法正确。\n\n故答案为: $V$\n\n【点睛】此题考查了利用轴对称图形的定义判断轴对称图形及对称轴条数的灵活应用。", "solution": "null", "level": "二年级", "question": "长方形和正方形都是轴对称图形, 正方形比长方形对称轴多。( )", "options": [], "subject": "解析几何", "analysis": "$\\sqrt{ }$\n\n【分析】依据轴对称图形的意义, 即在同一个平面内, 一个图形沿某条直线对折, 对折后的两部分都能完全重合, 则这个图形就是轴对称图形, 这条直线就是其对称轴, 据此可知长方形和正方形都是轴对称图形, 正方形有 4 条对称轴, 长方形有 2 条对称轴, 由此解答即可。\n\n【详解】长方形和正方形都是轴对称图形, 正方形有 4 条对称轴, 长方形有 2 条对称轴, 所以正方形比长方形对称轴多, 题干说法正确。\n\n故答案为: $V$\n\n【点睛】此题考查了利用轴对称图形的定义判断轴对称图形及对称轴条数的灵活应用。"} {"id": "9696", "image": [], "answer": "$\\times$", "solution": "null", "level": "二年级", "question": "所有的加法算式都能用乘法算式表示。( )", "options": [], "subject": "代数", "analysis": "$\\times$"} {"id": "21055", "image": [], "answer": "", "solution": "null", "level": "高三", "question": "已知命题 $\\mathrm{p}$ : 方程 $\\mathrm{x}^{2}-2 \\sqrt{2} \\mathrm{x}+\\mathrm{m}=0$ 有两个不相等的实数根; 命题 $\\mathrm{q}$ : $2^{\\mathrm{m}+1}<4$.\n\n(1)若 $p$ 为真命题, 求实数 $m$ 的取值范围;\n\n(2)若 $p \\vee q$ 为真命题, $p \\wedge q$ 为假命题, 求实数 $m$ 的取值范围.", "options": [], "subject": "逻辑题", "analysis": "解析: (1) 若 $\\mathrm{p}$ 为真命题, 则应有 $\\Delta=8-4 \\mathrm{~m}>0$, 解析得 $\\mathrm{m}<2$.(2) 若 $q$ 为真命题, 则有 $m+1<2$, 即 $m<1$, 因为 $p \\vee q$ 为真命题, $p \\wedge q$ 为假命题, 则 $\\mathrm{p}, \\mathrm{q}$ 应一真一假.(1)当 $\\mathrm{p}$ 真 $\\mathrm{q}$ 假时, 有 $\\left\\{\begin{array}{l}\\mathrm{m}<2 \\ \\mathrm{~m} \\geq 1\\end{array}\right.$, 得 $1 \\leq \\mathrm{m}<2$;(2) 当 $\\mathrm{p}$ 假 $\\mathrm{q}$ 真时, 有 $\\left\\{\begin{array}{l}\\mathrm{m} \\geq 2 \\ \\mathrm{~m}<1\\end{array}\right.$, 无解析. 综上, $\\mathrm{m}$ 的取值范围是 $[1,2)$."} {"id": "21067", "image": [], "answer": "", "solution": "null", "level": "高三", "question": "设命题 $p$ : 实数 $x$ 满足 $(x-a) ( x-3 a )<0$, 其中 $a>0$, 命题 $q$ : 实数 $x$满足 $(\\mathrm{x}-3) \\quad(\\mathrm{x}-2) \\leq 0$.\n\n(1) 若 $a=1$, 且 $p \\wedge q$ 为真, 求实数 $x$ 的取值范围.\n\n(2) 若 $\\neg p$ 是 $\\neg q$ 的充分不必要条件, 求实数 $a$ 的取值范围.", "options": [], "subject": "逻辑题", "analysis": "1)由(x-1)(x-3)<0, 得 $P=\\{x \\mid 10$, 则方程 $x^{2}+x-m=0$ 有实数根”, 分别写出这个命题的逆命题, 否命题, 逆否命题, 并分别判断它们的真假.", "options": [], "subject": "逻辑题", "analysis": "若 $m>0$, 则方程 $x^{2}+x-m=0$ 有实数根, 它是真命题;逆命题: 若方程 $x^{2}+x-m=0$ 有实数根, 则 $m>0$, 它是假命题;否命题: 若 $\\mathrm{m} \\leq 0$, 则方程 $\\mathrm{x}^{2}+\\mathrm{x}-\\mathrm{m}=0$ 没有实数根, 它是假命题;逆否命题: 若方程 $x^{2}+x-m=0$ 没有实数根, 则 $m \\leq 0$, 它是真命题."} {"id": "22972", "image": [], "answer": "答案: 解: 由 $x>-1$ 得 $x+1>0$, 则\n\n$y=\\frac{(x+5)(x+2)}{x+1}=\\frac{[(x+1)+4][(x+1)+1]}{x+1}$\n\n$=\\frac{(x+1)^{2}+5(x+1)+4}{x+1}=(x+1)+\\frac{4}{x+1}+5$\n\n$\\geq 2 \\sqrt{4}+5=9$\n\n当且仅当 $x=1$ 时, 上式取“=”, 所以 $y_{\\text {min }}=9$.", "solution": "null", "level": "高三", "question": "22. 设 $x>-1$, 求函数 $y=\\frac{(x+5)(x+2)}{x+1}$ 的最小值.", "options": [], "subject": "解析几何", "analysis": ": 分析: 本题主要考查了基本不等式在最值问题中的应用, 解决问题的关键是本题解题的关键在于关注分母 $x+1$, 充分运用发散性思维, 经过同解变形构造基本不等式,从而求出最小值."} {"id": "22974", "image": [], "answer": "答案: 解: 由余弦定理有 $\\cos A=\\frac{b^{2}+c^{2}-a^{2}}{2 b c}=\\frac{1}{2}$\n\n$\\because 0\n\n法一: 由 $\\triangle A B C$ 是直角三角形可知 $|A B|^{2}=|A C|^{2}+|B C|^{2}$, 即\n\n$(2 a)^{2}=(x+a)^{2}+y^{2}+(x-a)^{2}+y^{2}$. 化简得 $x^{2}+y^{2}=a^{2}$. 依题意可知, $x \\neq \\pm a$.\n故所求直角顶点 $C$ 的轨迹方程为 $x^{2}+y^{2}=a^{2}(x \\neq \\pm a)$.\n\n法二: 由 $\\triangle A B C$ 是直角三角形可知 $A C \\perp B C$, 所以 $k_{A C} \\cdot k_{E C}=-1$, 则\n\n$\\frac{y}{x+a} \\cdot \\frac{y}{x-a}=-1(x \\neq \\pm a)$, 化简得直角顶点 $C$ 的轨迹方程为 $x^{2}+y^{2}=a^{2}(x \\neq \\pm a)$.\n\n法三: 由 $\\triangle A B C$ 是直角三角形可知 $|O C|=|O B|$, 且点 $C$ 与点 $B$ 不重合, 所以\n\n$\\sqrt{x^{2}+y^{2}}=a(x \\neq \\pm a)$, 化简得直角顶点 $C$ 的轨迹方程为 $x^{2}+y^{2}=a^{2}(x \\neq \\pm a)$.\n\n$x^{2}+y^{2}=a^{2}(x \\neq \\pm a)$", "solution": "null", "level": "高三", "question": "17. 已知Rt $\\triangle A B C,|A B|=2 a(a>0)$, 求直角顶点 $C$ 的轨迹方程.", "options": [], "subject": "解析几何", "analysis": ": 分析: 本题主要考查了平面直角坐标系与曲线方程, 解决问题的关键是需要结合几何图形的结构特点, 建立适当的平面直角坐标系, 然后设出所求动点的坐标, 寻找满足几何关系的等式,化简后即可得到所求的轨迹方程;\n\n求轨迹方程, 其实质就是根据题假设条件, 把几何关系通过“坐标”转化成代数关系, 得到对应的方程.(1)求轨迹方程时的一般步骤是: 建系 $\\rightarrow$ 设点 $\\rightarrow$ 列式 $\\rightarrow$ 化简 $\\rightarrow$ 检验; (2)求轨迹方程时注意不要把范围扩大或缩小, 也就是要检验轨迹的纯粹性和完备性; (3)由于观察的角度不同, 因此探求关系的方法也不同, 解题时要善于从对角度思考问题."} {"id": "22766", "image": ["10067.jpg"], "answer": "答案: 证明: 以 $A$ 为坐标原点 $O, A B$ 所在直线为 $x$ 轴,建立平面直角坐标系 $x O y$,\n\n\n\n则 $A(0,0)$, 设 $B(a, 0), C(b, c)$,则 $D\\left(\\frac{a+b}{2}, \\frac{c}{2}\\right)$,\n\n$\\therefore A D^{2}+B D^{2}=\\frac{(a+b)^{2}}{4}+\\frac{c^{2}}{4}+\\frac{(a-b)^{2}}{4}+\\frac{c^{2}}{4}$\n\n$=\\frac{1}{2}\\left(a^{2}+b^{2}+c^{2}\\right)$,\n\n$A B^{2}+A C^{2}=a^{2}+b^{2}+c^{2}$.\n$\\therefore A B^{2}+A C^{2}=2\\left(A D^{2}+B D^{2}\\right)$.", "solution": "null", "level": "高三", "question": "18. 已知 $\\triangle A B C$ 中, $B D=C D$, 求证: $A B^{2}+A C^{2}=2\\left(A D^{2}+B D^{2}\\right)$.", "options": [], "subject": "解析几何", "analysis": ": 分析: 本题主要考查了平面直角坐标系与曲线方程, 解决问题的关键是根据条件建立坐标系求解方程即可"} {"id": "22768", "image": ["10069.jpg"], "answer": "答案: 解: 如图, 以 $A, B$ 所在的直线为 $\\mathrm{x}$ 轴, $A B A B$ 中点 $\\mathrm{O}$ 为坐标原点, 建立平面直角坐标系, 则 $A(-5,0), B(5,0)$.\n\n\n\n设某地 $\\mathrm{P}$ 的坐标为 $(x, y)$, 且 $\\mathrm{P}$ 地居民选择到 $\\mathrm{A}$ 地购买商品便宜, 并设 $\\mathrm{A}$ 地的运费为 3a 元/千米, B 地的运费为 $\\mathrm{a}$ 元/千米.\n\n价格+运费 $\\times$ 到 $A$ 地的距离 $\\leq$ 价格+运费 $\\times$ 到 $B$ 地的距离,\n\n即 $3 a \\sqrt{(x+5)^{2}+y^{2}} \\leq a \\sqrt{(x-5)^{2}+y^{2}}$,\n\n因为 $a>0$, 所以 $3 \\sqrt{(x+5)^{2}+y^{2}} \\leq \\sqrt{(x-5)^{2}+y^{2}}$.\n\n即 $\\left(x+\\frac{25}{4}\\right)^{2}+y^{2} \\leq\\left(\\frac{15}{4}\\right)^{2}$.\n\n所以以点 $C\\left(-\\frac{25}{4}, 0\\right)$ 为圆心, $\\frac{15}{4}$ 为半径的圆是这两地购货的分界线.\n\n圆 C 内的居民从 $\\mathrm{A}$ 地购货便宜.\n\n圆 C 外的居民从 B 地购货便宜.\n\n圆 $\\mathrm{C}$ 上的居民从 $\\mathrm{A}, \\mathrm{B}$ 地购货的总费用相等,因此,可随意从 $\\mathrm{A}, \\mathrm{B}$ 两地之一购货.", "solution": "null", "level": "高三", "question": "20. 某种大型商品, A, B 两地都有出售, 且价格相同. 某地居民从两地之一购得商品后运回的费用是: 每单位距离 $\\mathrm{A}$ 地的运费是 $\\mathrm{B}$ 地的运费的 3 倍, 已知 $\\mathrm{A}, \\mathrm{B}$ 两地距离为 10 千米,\n顾客选择 A 或 $\\mathrm{B}$ 地购买这种商品的标准是: 包括运费和价格的总费用较低, 求 $\\mathrm{A}, \\mathrm{B}$ 两地的售货区域的分界线的曲线形状,并指出曲线上.曲线内.曲线外的居民应如何选择购货地点.", "options": [], "subject": "解析几何", "analysis": ": 分析: 本题主要考查了平面直角坐标系与曲线方程, 解决问题的关键是根据所给事件问题建立平面直角坐标系求解即可"} {"id": "22769", "image": ["10070.jpg"], "answer": "答案: 解答: 以 $B C$ 的中点为原点, $B C$ 所在的直线为 $x$ 轴, $B C$ 的垂直平分线为 $y$ 轴, 建立如图所示的直角坐标系,\n\n\n\n设点 $P(x, y), B(-a, 0), C(a, 0), A(0, \\sqrt{3} a),(y>0, a>0)$ 用点的坐标表示等式\n\n$|P A|^{2}=|P B|^{2}+|P C|^{2}$,\n\n有 $x^{2}+(y-\\sqrt{3} a)^{2}=(x+a)^{2}+y^{2}+(x-a)^{2}+y^{2}$,\n\n化简得 $x^{2}+(y+\\sqrt{3} a)^{2}=(2 a)^{2}$,\n\n即点 $P$ 的轨迹方程为 $x^{2}+(y+\\sqrt{3} a)^{2}=4 a^{2}(y>0)$.", "solution": "null", "level": "高三", "question": "21. 在正三角形 $A B C$ 内有一动点 $P$, 已知 $P$ 到三顶点的距离分别为 $|P A|_{,}|P B|,|P C|$, 且满足 $|P A|^{2}=|P B|^{2}+|P C|^{2}$, 求点 $P$ 的轨迹方程.", "options": [], "subject": "解析几何", "analysis": ": 分析: 本题主要考查了平面直角坐标系与曲线方程, 解决问题的关键是根据所给条件建立空间直角坐标系计算即可"} {"id": "22771", "image": [], "answer": "答案:解:以 $O$ 为原点,分别以直线 $A B, C D$ 为 $x$ 轴. $y$ 轴建立直角坐标系,则 $A(-4,0), B(4,0), C(0,2), D(0,-2)$.\n\n设 $M(x, y)$ 为轨迹上任一点,则\n\n$|M A|=\\sqrt{(x+4)^{2}+y^{2}},|M B|=\\sqrt{(x-4)^{2}+y^{2}}$,\n\n$|M C|=\\sqrt{x^{2}+(y-2)^{2}},|M D|=\\sqrt{x^{2}+(y+2)^{2}}$,\n\n$\\therefore$ 由 $|M A| \\cdot|M B|=|M C| \\cdot|M D|$, 可得\n\n$\\sqrt{\\left[(x+4)^{2}+y^{2}\\right]\\left[(x-4)^{2}+y^{2}\\right]}=\\sqrt{\\left[x^{2}+(y-2)^{2}\\right]\\left[x^{2}+(y+2)^{2}\\right]}$\n\n化简,得 $y^{2}-x^{2}+6=0$.\n\n$\\therefore$ 点 $M$ 的轨迹方程为 $x^{2}-y^{2}=6$.", "solution": "null", "level": "高三", "question": "23. 已知线段 $A B$ 与 $C D$ 互相垂直平分于点 $O,|A B|=8,|C D|=4$, 动点 $M$ 满足 $|M A| \\cdot|M B|=|M C| \\cdot|M D|$, 求动点 $M$ 的轨迹方程.", "options": [], "subject": "解析几何", "analysis": ": 分析: 本题主要考查了平面直角坐标系与曲线方程, 解决问题的关键是根据所给条件建立空间直角坐标系计算即可"} {"id": "22772", "image": ["10071.jpg"], "answer": "答案: 解: 设平行四边行的三个顶点分别为 $A(-2,1), B(-1,3), C(3,4)$, 如图, 设第四个顶点为 $D$, 所以点 $D$ 有三种不同的位置.\n\n\n\n若平行四边形为 $A B C D_{1}$ 时, 设点 $D_{1}$ 的坐标为 $(x, y)$, 则 $\\overrightarrow{A B}=(-1+2,3-1)=(1,2)$, $D_{1} C=(3-x, 4-y)$.\n由 $\\overrightarrow{A B}=\\overrightarrow{D_{1} C}$, 得 $\\left\\{\\begin{array}{l}3-x=1 \\\\ 4-y=2\\end{array}\\right.$, 解得, 即 $D_{1}(2,2)$.\n\n若平行四边形为 $A B D_{2}$ C 时, 同理, 求得点 $D_{2}$ 的坐标为 $(4,6)$;\n\n若平行四边形为 $A D_{3} B C$ 时, 同理, 求得点 $D_{3}$ 的坐标为 $(-6,0)$.\n\n综上所述, 平行四边形第四个顶点的坐标分别为 $(2,2),(4 , 6 )$ 或 $(-6,0)$.", "solution": "null", "level": "高三", "question": "24. 已知平行四边形的三个顶点的坐标分别为 $(-2,1),(-1,3),(3,4)$, 求第四个顶点的坐标.", "options": [], "subject": "解析几何", "analysis": ": 分析: 本题主要考查了平面直角坐标系与曲线方程, 解决问题的关键是根据所给直线结合所给条件分析计算即可"} {"id": "22773", "image": ["10072.jpg", "10073.jpg"], "answer": "答案: $(8,5 \\sqrt{3})$", "solution": "null", "level": "高三", "question": "25. 如图所示, $A, B, C$ 是三个观察站, $A$ 在 $B$ 的正东, 两地相距 $6 \\mathrm{~km}, C$ 在 $B$ 的北偏西 $30^{\\circ}$, 两地相距 $4 \\mathrm{~km}$, 在某一时刻, $A$ 观察站发现某种信号, 并知道该信号的传播速度为 1 $\\mathrm{km} / \\mathrm{s}, 4 \\mathrm{~s}$ 后 $B, C$ 两个观察站同时发现这种信号, 在以过 $A, B$ 两点的直线为 $x$ 轴, 以 $A B$ 的垂直平分线为 $y$ 轴建立的平面直角坐标系中, 指出发出这种信号的 $P$ 的坐标.\n\n", "options": [], "subject": "解析几何", "analysis": ":\n\n解答: 设点 $P$ 的坐标为 $(x, y)$, 则 $\\mathrm{A}(3,0), \\mathrm{B}(-3,0), \\mathrm{C}(-5,2 \\sqrt{3})$.\n\n\n\n因为 $|\\mathrm{PB}|=|\\mathrm{PC}|$, 所以点 $\\mathrm{P}$ 在 $\\mathrm{BC}$ 的中垂线上.\n\n因为 $\\mathrm{k}_{\\mathrm{BC}}=-\\sqrt{3}, \\mathrm{BC}$ 的中点 $\\mathrm{D}(-4, \\sqrt{3})$,\n\n所以直线 $P D$ 的方程为 $y-\\sqrt{3}=\\frac{1}{\\sqrt{3}}(x+4)$. (1)\n\n又因为 $|P B|-|P A|=4$, 所以点 $P$ 必在以 $A, B$ 为焦点的双曲线的右支上,双曲线方程为 $\\frac{x^{2}}{4}-\\frac{y^{2}}{5}=1(x \\geq 2)$.\n联立(1)(2), 解得 $x=8$ 或 $x=-\\frac{32}{11}$ (舍去),\n\n所以 $\\mathrm{y}=5 \\sqrt{3}$.\n\n所以点 $\\mathrm{P}$ 的坐标为 $(8,5 \\sqrt{3})$.\n\n分析: 本题主要考查了平面直角坐标系与曲线方程, 解决问题的关键是根据题意可知, 点 $\\mathrm{P}$所在的位置满足两个条件: (1)在线段 $B C$ 的垂直平分线上; (2)在以 $A, B$ 为焦点的双曲线上."} {"id": "22518", "image": ["9837.jpg"], "answer": "答案: 证明: $\\because \\mathrm{FE} / / \\mathrm{BC}$,\n\n$\\therefore \\frac{\\mathrm{EF}}{\\mathrm{BC}}=\\frac{\\mathrm{AE}}{\\mathrm{AB}}, \\frac{\\mathrm{EF}}{\\mathrm{GH}}=\\frac{\\mathrm{DF}}{\\mathrm{DH}}$.\n\n$\\because \\mathrm{AD} / / \\mathrm{EF} / / \\mathrm{BH}, \\therefore \\frac{\\mathrm{AE}}{\\mathrm{AB}}=\\frac{\\mathrm{DF}}{\\mathrm{DH}}$.\n\n$\\therefore \\frac{\\mathrm{EF}}{\\mathrm{BC}}=\\frac{\\mathrm{EF}}{\\mathrm{GH}} \\therefore \\mathrm{BC}=\\mathrm{GH}$.", "solution": "null", "level": "高三", "question": "17. 如图,在梯形 $\\mathrm{ABCD}$ 中, $\\mathrm{AD} / / \\mathrm{BC}, \\mathrm{F}$ 为对角线 $\\mathrm{AC}$ 上一点, $\\mathrm{FE} / / \\mathrm{BC}$ 交 $\\mathrm{AB}$ 于点 $\\mathrm{E}, \\mathrm{DF}$ 的延长线交 $\\mathrm{BC}$ 于点 $\\mathrm{H}, \\mathrm{DE}$ 的延长线交 $\\mathrm{CB}$ 的延长线于点 $\\mathrm{G}$, 求证: $\\mathrm{BC}=\\mathrm{GH}$.\n\n", "options": [], "subject": "解析几何", "analysis": ": 分析: 本题主要考查了平行线分线段成比例定理, 解决问题的关键是根据平行线分线段成比例定理结合所给条件分析计算即可."} {"id": "22520", "image": ["9839.jpg"], "answer": "答案: 证明: $\\because \\mathrm{EF} / / \\mathrm{AD}, \\mathrm{AD} / / \\mathrm{BC}$,\n\n$\\therefore \\mathrm{EF} / / \\mathrm{AD} / / \\mathrm{BC}$.\n\n$\\because \\mathrm{EF} / / \\mathrm{BC}, \\therefore \\frac{\\mathrm{OE}}{\\mathrm{BC}}=\\frac{\\mathrm{AE}}{\\mathrm{AB}}, \\frac{\\mathrm{OF}}{\\mathrm{BC}}=\\frac{\\mathrm{DF}}{\\mathrm{DC}}$\n\n$\\because \\mathrm{EF} / / \\mathrm{AD} / / \\mathrm{BC}, \\therefore \\frac{\\mathrm{AE}}{\\mathrm{AB}}=\\frac{\\mathrm{DF}}{\\mathrm{DC}}$\n\n$\\therefore \\frac{\\mathrm{OE}}{\\mathrm{BC}}=\\frac{\\mathrm{OF}}{\\mathrm{BC}}, \\therefore \\mathrm{OE}=\\mathrm{OF}$.\n\n(2) 求证: $\\frac{1}{\\mathrm{AD}}+\\frac{1}{\\mathrm{BC}}=\\frac{1}{\\mathrm{OE}}$.\n\n答案: 证明: $\\because \\mathrm{OE} / / \\mathrm{AD}, \\therefore \\frac{\\mathrm{OE}}{\\mathrm{AD}}=\\frac{\\mathrm{BE}}{\\mathrm{AB}}$\n\n由(1)知 $\\frac{\\mathrm{OE}}{\\mathrm{BC}}=\\frac{\\mathrm{AE}}{\\mathrm{AB}}$,\n\n$\\therefore \\frac{\\mathrm{OE}}{\\mathrm{AD}}+\\frac{\\mathrm{OE}}{\\mathrm{BC}}=\\frac{\\mathrm{BE}}{\\mathrm{AB}}+\\frac{\\mathrm{AE}}{\\mathrm{AB}}=\\frac{\\mathrm{BE}+\\mathrm{AE}}{\\mathrm{AB}}=1$.\n\n$\\therefore \\frac{1}{\\mathrm{AD}}+\\frac{1}{\\mathrm{BC}}=\\frac{1}{\\mathrm{OE}}$.", "solution": "null", "level": "高三", "question": "19. 如图,在梯形 $\\mathrm{ABCD}$ 中, $\\mathrm{AD} / / \\mathrm{BC}, \\mathrm{EF}$ 经过梯形对角线的交点 $\\mathrm{O}$, 且 $\\mathrm{EF} / / \\mathrm{AD}$.\n\n\n\n(1) 求证: $\\mathrm{OE}=\\mathrm{OF}$;", "options": [], "subject": "解析几何", "analysis": ": 分析: 本题主要考查了平行线分线段成比例定理, 解决问题的关键是根据所给条件结合平行线分线段成比例定理分析计算即可."} {"id": "22522", "image": ["9841.jpg"], "answer": "答案: 解: $\\because \\mathrm{AD} / / \\mathrm{BC}, \\therefore \\frac{\\mathrm{AF}}{\\mathrm{FC}}=\\frac{\\mathrm{AE}}{\\mathrm{NC}}$\n\n$\\therefore \\frac{\\mathrm{AF}}{\\mathrm{AF}+\\mathrm{FC}}=\\frac{\\mathrm{AE}}{\\mathrm{AE}+\\mathrm{NC}}$, 即 $\\frac{\\mathrm{AF}}{\\mathrm{AC}}=\\frac{\\mathrm{AE}}{\\mathrm{AE}+\\mathrm{NC}}$\n\n$\\because \\frac{\\mathrm{AE}}{\\mathrm{BN}}=\\frac{\\mathrm{AM}}{\\mathrm{MB}}=1, \\therefore \\mathrm{AE}=\\mathrm{BN}$.\n\n$\\therefore \\frac{\\mathrm{AF}}{\\mathrm{AC}}=\\frac{\\mathrm{AE}}{\\mathrm{AE}+\\mathrm{BN}+\\mathrm{BC}}=\\frac{\\mathrm{AE}}{2 \\mathrm{AE}+\\mathrm{BC}}$\n\n$\\because \\mathrm{AE}=2, \\mathrm{BC}=\\mathrm{AD}=6$,\n\n$\\therefore \\frac{\\mathrm{AF}}{\\mathrm{AC}}=\\frac{2}{2 \\times 2+6}=\\frac{1}{5}$, 即 $\\mathrm{AF}: \\mathrm{AC}=1: 5$.", "solution": "null", "level": "高三", "question": "20. 如图, $M$ 是 $A B C D$ 的边 $A B$ 的中点, 直线 $l$ 过点 $M$ 分别交 $A D, A C$ 于点 $E, F$, , 交 $C B$ 的延长线于点 $N$, 若 $A E=2, A D=6$, 求 $A F: A C$ 的值.\n\n", "options": [], "subject": "解析几何", "analysis": ":分析:本题主要考查了平行线分线段成比例定理, 解决问题的关键是: $\\mathrm{AD} / /$\n\n$\\mathrm{BC}, \\mathrm{AM}=\\mathrm{MB} \\Rightarrow \\mathrm{AE}=\\mathrm{BN} \\Rightarrow \\mathrm{AF}: \\mathrm{AC}$ 的值"} {"id": "22787", "image": [], "answer": "答案: 解答: $x=8 \\cos \\frac{2 \\pi}{3}=-4$\n\n$y=8 \\sin \\frac{2 \\pi}{3}=4 \\sqrt{3}$,\n\n因此,点 $M$ 的直角坐标是 $(-4,4 \\sqrt{3})$.", "solution": "null", "level": "高三", "question": "15. 把点 $\\mathrm{M}$ 的极坐标 $\\left(8, \\frac{2 \\pi}{3}\\right)$ 化成直角坐标.", "options": [], "subject": "解析几何", "analysis": ": 分析: 本题主要考查了点的极坐标和直角坐标的互化, 解决问题的关键是根据极坐标与直角坐标系关系分析计算即可"} {"id": "22789", "image": [], "answer": "答案: 解: 由题设知, $\\mathrm{A}, \\mathrm{B}$ 两点关于极点 $\\mathrm{O}$ 对称, 又 $|A B|=4$, 由正三角形的性质知,\n$|C O|=2 \\sqrt{3}, \\angle A O C=\\frac{\\pi}{2}$, 从而 $\\mathrm{C}$ 的极坐标为 $\\left(2 \\sqrt{3}, \\frac{3}{4} \\pi\\right)$ 或 $\\left(2 \\sqrt{3},-\\frac{\\pi}{4}\\right)$", "solution": "null", "level": "高三", "question": "17. 在极坐标系中, 如果等边三角形的两个顶点是 $A\\left(2, \\frac{\\pi}{4}\\right), B\\left(2, \\frac{5}{4} \\pi\\right)$, 则求第三个顶点 $\\mathrm{C}$ 的坐标.", "options": [], "subject": "解析几何", "analysis": ": 分析: 本题主要考查了极坐标刻画点的位置, 解决问题的关键是根据所给点满足的几何意义计算即可"} {"id": "22790", "image": ["10077.jpg"], "answer": "答案: 解: 以 $\\mathrm{O}$ 为极点, $\\mathrm{OA}$ 所在射线为极轴建立极坐标系, 因为 $|O C|=600, \\angle A O C=\\frac{\\pi}{6}$,故 $C\\left(600, \\frac{\\pi}{6}\\right)$.\n\n又 $|O A|=600 \\times \\cos \\frac{\\pi}{6}=300 \\sqrt{3},|O D|=600 \\times \\sin \\frac{\\pi}{6}=300,|O E|=300 \\sqrt{2}$ ,\n\n$|O F|=300,|O G|=150 \\sqrt{2}$.\n\n故 $A(300 \\sqrt{3}, 0), D\\left(300, \\frac{\\pi}{2}\\right), E\\left(300 \\sqrt{2}, \\frac{3 \\pi}{4}\\right), F(300, \\pi), G\\left(150 \\sqrt{2}, \\frac{3 \\pi}{4}\\right)$", "solution": "null", "level": "高三", "question": "18. 某大学校园的部分平面示意图如图\n\n$E$ 图书馆 $\\quad D$ 教学楼 $\\quad C$ 公寓\n\n\n\n用点 $O, A, B, C, D, E, F, G$ 分别表示校门, 器材室, 操场, 公寓, 教学楼, 图书馆, 车库, 花园, 其中 $|A B|=|B C|,|O C|=600 \\mathrm{~m}$.建立适当的极坐标系, 写出除点 $\\mathrm{B}$外各点的极坐标(限定 $\\rho \\geq 0,0 \\leq \\theta<2 \\pi$ 且极点为 $(0,0)$ ).", "options": [], "subject": "解析几何", "analysis": ": 分析: 本题主要考查了极坐标刻画点的位置, 解决问题的关键是根据极坐标系计算即可"} {"id": "22791", "image": ["10078.jpg", "10079.jpg"], "answer": "答案: 解: 如图所示, $|O A|=\\left|O A^{\\prime}\\right|=6$ ,\n\n\n\n$\\angle x O A^{\\prime}=\\frac{2 \\pi}{3}, \\angle x O A=\\frac{5 \\pi}{3}$,\n\n即点 $\\mathrm{A}$ 与 $A^{\\prime}$ 关于极点 $\\mathrm{O}$ 对称.\n\n由极坐标的定义知\n\n当 $\\rho>0,-\\pi<\\theta \\leq \\pi$ 时, $A\\left(6,-\\frac{\\pi}{3}\\right)$\n\n(2) $\\rho<0,0 \\leq \\theta<2 \\pi$.\n\n答案: 解: 有 (1) 可知, 当 $\\rho<0,0 \\leq \\theta<2 \\pi$ 时, $A\\left(-6, \\frac{2 \\pi}{3}\\right)$\n\n(3) $\\rho<0,-2 \\pi<\\theta \\leq 0$.\n\n答案: 解: 有 (1) 可知, 当 $\\rho<0,-2 \\pi<\\theta \\leq 0$ 时, $A\\left(-6,-\\frac{4 \\pi}{3}\\right)$", "solution": "null", "level": "高三", "question": "19. 如果对点的极坐标定义如下: 当已知 $M(\\rho, \\theta)(\\rho>0, \\theta \\in \\mathbf{R})$ 时, 点关于极点 $\\mathrm{O}$ 的对称点 $M^{\\prime}(-\\rho, \\theta)$.\n\n\n例如, $M\\left(3, \\frac{\\pi}{3}\\right)$ 关于极点 $\\mathrm{O}$ 的对称点 $M^{\\prime}\\left(-3, \\frac{\\pi}{3}\\right)$, 就是说 $\\left(3, \\frac{\\pi}{3}+\\pi\\right)$ 与 $\\left(-3, \\frac{\\pi}{3}\\right)$ 表示同一点.\n\n已知 A 点的极坐标是 $\\left(6, \\frac{5 \\pi}{3}\\right)$, 分别在下列给定条件下, 写出 $\\mathrm{A}$ 点的极坐标:\n\n(1) $\\rho>0,-\\pi<\\theta \\leq \\pi$.", "options": [], "subject": "解析几何", "analysis": ": 分析: 本题主要考查了极坐标刻画点的位置, 解决问题的关键是根据极坐标的意义计算即可"} {"id": "22793", "image": ["10080.jpg"], "answer": "答案: 解: 如图,\n\n\n$\\angle B A D=60^{\\circ}$, 以 $\\mathrm{A}$ 为极点 $\\mathrm{O}, \\mathrm{AB}$ 的方向为极轴的正方向, 建立极坐标系, 则菱形的四个顶点的极坐标分别为 $A(0,0), B(2,0), C\\left(2 \\sqrt{3}, \\frac{\\pi}{6}\\right), D\\left(2, \\frac{\\pi}{3}\\right)$ (答案不惟一).", "solution": "null", "level": "高三", "question": "20. 边长为 2 的菱形 $\\mathrm{ABCD}$, 一个内角为 $60^{\\circ}$, 建立适当的极坐标系, 求出菱形四个顶点的极坐标(限定 $\\rho \\geq 0, \\quad \\theta \\in[0,2 \\pi)$ ).", "options": [], "subject": "解析几何", "analysis": ":分析:本题主要考查了极坐标系, 解决问题的关键是根据极坐标系计算即可"} {"id": "22794", "image": ["10081.jpg"], "answer": "答案: 解: 以极点 $O^{\\prime}$ 为坐标原点, 极轴方向为 $x^{\\prime}$ 轴正方向,\n\n\n\n建立新直角坐标系 $x^{\\prime} O^{\\prime} y^{\\prime}$, 设点 $M$ 的新直角坐标为 $\\left(x^{\\prime}, y^{\\prime}\\right)$, 于是 $x^{\\prime}=4 \\cos \\frac{\\pi}{6}=2 \\sqrt{3}$, $y^{\\prime}=4 \\sin \\frac{\\pi}{6}=2$, 由 $O^{\\prime}\\left(x^{\\prime}, y^{\\prime}\\right)=O^{\\prime}(0,0), O^{\\prime}(x, y)=O^{\\prime}(2,3)$,易得 $O^{\\prime}\\left(x^{\\prime}, y^{\\prime}\\right)$ 与 $O^{\\prime}(x, y)$ 的关系为 $\\left\\{\\begin{array}{l}x=x^{\\prime}+2, \\\\ y=y^{\\prime}+3,\\end{array}\\right.$ 于是点 $M(x, y)$ 为 $\\left\\{\\begin{array}{l}x=2 \\sqrt{3}+2, \\\\ y=2+3=5,\\end{array}\\right.$所以点 $\\mathrm{M}$ 的直角坐标为 $(2 \\sqrt{3}+2,5)$.", "solution": "null", "level": "高三", "question": "21. 已知点 $\\mathrm{M}$ 的极坐标为 $\\left(4, \\frac{\\pi}{6}\\right)$, 极点 $\\mathrm{O}$ 在直角坐标系 $x O y$ 中的直角坐标为 $(2,3)$,极轴平行于 $\\mathrm{x}$ 轴, 极轴的方向与 $\\mathrm{x}$ 轴的正方向相同, 两坐标系的长度单位相同, 求点 $\\mathrm{M}$ 的直角坐标.", "options": [], "subject": "解析几何", "analysis": ": 分析: 本题主要考查了点的极坐标和直角坐标的互化, 解决问题的关键是根据坐标系的关系分析计算即可。"} {"id": "22796", "image": ["10082.jpg"], "answer": "答案: 解: 作出图形,\n\n\n\n可知 $\\mathrm{A}\\left(3, \\frac{\\pi}{6}\\right)$ 关于直线 $\\theta=\\frac{\\pi}{2}$ 的对称点是 $\\left(3, \\frac{5 \\pi}{6}\\right)$.", "solution": "null", "level": "高三", "question": "23. 在极坐标系中, 点 $\\mathrm{A}$ 的极坐标是 $\\left(3, \\frac{\\pi}{6}\\right)$, 求点 $\\mathrm{A}$ 关于直线 $\\theta=\\frac{\\pi}{2}$ 的对称点的极坐标 $($ 规定 $\\rho>0, \\theta \\in[0,2 \\pi])$.", "options": [], "subject": "解析几何", "analysis": ": 分析: 本题主要考查了点的极坐标和直角坐标的互化, 解决问题的关键是根据点的极坐标和直角坐标的互化分析计算即可"} {"id": "22797", "image": [], "answer": "答案:解:设 $\\mathrm{M}(\\mathrm{r}, 0)$,\n\n因为 $\\mathrm{A}\\left(4 \\sqrt{2}, \\frac{\\pi}{4}\\right)$,\n\n所以 $\\sqrt{(4 \\sqrt{2})^{2}+r^{2}-8 \\sqrt{2} r \\cos \\frac{\\pi}{4}}=5$,\n\n即 $\\mathrm{r}^{2}-8 \\mathrm{r}+7=0$. 解得 $\\mathrm{r}=1$ 或 $\\mathrm{r}=7$.\n\n所以 $\\mathrm{M}$ 点的坐标为 $(1,0)$ 或 $(7,0)$.", "solution": "null", "level": "高三", "question": "24. 在极轴上求与点 $\\mathrm{A}\\left(4 \\sqrt{2}, \\frac{\\pi}{4}\\right)$ 的距离为 5 的点 $\\mathrm{M}$ 的坐标", "options": [], "subject": "解析几何", "analysis": ": 分析: 本题主要考查了点的极坐标和直角坐标的互化, 解决问题的关键是根据点的极坐标和直角坐标的互化计算即可"} {"id": "22798", "image": [], "answer": "答案: 解: $\\mathrm{x}=2 \\cos \\frac{7 \\pi}{6}=-\\sqrt{3}$,\n\n$\\mathrm{y}=2 \\sin \\frac{7 \\pi}{6}=-1$,\n\n故点 $\\mathrm{A}$ 的直角坐标为 $(-\\sqrt{3},-1)$.\n\n(2)把点 $\\mathrm{P}$ 的直角坐标 $(1,-\\sqrt{3})$ 化成极坐标. $(\\rho>0,0 \\leq \\theta<2 \\pi)$\n答案: 解: $\\rho=\\sqrt{1^{2}+-\\sqrt{3}^{2}}=2, \\tan \\theta==-\\sqrt{3}$.\n\n又因为点 $P$ 在第四象限且 $0 \\leq \\theta<2 \\pi$, 得 $\\theta=\\frac{5 \\pi}{3}$.\n\n因此点 $\\mathrm{P}$ 的极坐标是 $\\left(2, \\frac{5 \\pi}{3}\\right)$.", "solution": "null", "level": "高三", "question": "25. 根据点的极坐标和直角坐标的互化, 解决下列问题\n\n(1)把点 $\\mathrm{A}$ 的极坐标 $\\left(2, \\frac{7 \\pi}{6}\\right)$ 化成直角坐标;", "options": [], "subject": "解析几何", "analysis": ": 分析: 本题主要考查了点的极坐标和直角坐标的互化, 解决问题的关键是根据(1)极坐标和直角坐标互化的前提条件有三, 即极点与原点重合, 极轴与 $\\mathrm{x}$ 轴正半轴重合, 有相同的长度单位, 三者缺一不可. (2)熟记互化公式, 必要时可画图来分析."} {"id": "22995", "image": [], "answer": "答案: 解: 由于 $f(x)=|x-a|+|x-5| \\geq|a-5|$,\n\n所以 $f(x) \\geq 3 \\Leftrightarrow|a-5| \\geq 3$, 解得 $a \\leq 2$ 或 $a \\geq 8$.\n\n(2) 当 $a=2$ 时,求: 不等式 $f(x) \\geqslant x^{2}-8 x+15$ 的解集.\n\n答案: 解: $f(x)=|x-2|+|x-5|=\\left\\{\\begin{array}{c}7-2 x, x<2 \\\\ 3,2 \\leq x \\leq 5 \\\\ 2 x-7, x>5\\end{array}\\right.$,\n\n原不等式等价于 $\\left\\{\\begin{array}{c}x<2 \\\\ 7-2 x \\geq x^{2}-8 x+15\\end{array}\\right.$, 或 $\\left\\{\\begin{array}{c}2 \\leq x \\leq 5 \\\\ 3 \\geq x^{2}-8 x+15\\end{array}\\right.$, 或 $\\left\\{\\begin{array}{c}x>5 \\\\ 2 x-7 \\geq x^{2}-8 x+15\\end{array}\\right.$\n\n解得 $2 \\leq x \\leq 5+\\sqrt{3}$, 原不等式解集为 $\\{x \\mid 2 \\leq x \\leq 5+\\sqrt{3}\\}$.\n\n.", "solution": "null", "level": "高三", "question": "23. 已知函数 $f(x)=|x-a|+|x-5|$.\n\n(1) 若不等式 $f(x) \\geqslant 3$ 恒成立,求 $a$ 的取值范围;", "options": [], "subject": "解析几何", "analysis": ": 分析: 本题主要考查了绝对值不等式的解法, 解决问题的关键是. (1) 利用不等式的性质得 $|x-a|+|x-5| \\geq|a-5|$, 所以不等式 $f(x) \\geqslant 3$ 恒成立, 可以转化为 $|a-5| \\geq 3$, 解绝对值不等式即可得到 $\\mathrm{a}$ 的取值范围; (2) 先把函数 $f(x)$ 写成分段函数, 再利用零点分段法, 断开, 分别解不等式组, 即可得到不等式的解集."} {"id": "22809", "image": [], "answer": "答案:解:圆 $O: \\rho=\\cos \\theta+\\sin \\theta, \\rho^{2}=\\rho \\cos \\theta+\\rho+\\sin \\theta$.\n\n故圆 $O$ 的直角坐标方程 $x^{2}+y^{2}=x+y$ ,即 $x^{2}+y^{2}-x-y=0$.\n\n直线 $l: \\rho \\sin \\left(\\theta-\\frac{\\pi}{4}\\right)=\\frac{\\sqrt{2}}{2}$, 即 $\\rho \\sin \\theta-\\rho \\cos \\theta=1$,\n\n则直线 1 的直角坐标方程 $y-x=1$, 即 $x-y+1=0$.\n\n(2) 当 $\\theta \\in(0, \\pi)$ 时, 求直线 $l$ 与圆 $\\mathrm{O}$ 公共点的一个极坐标.\n\n答案: 解: 由 $\\left\\{\\begin{array}{c}x^{2}+y^{2}-x-y=0 \\\\ x-y+1=0\\end{array}\\right.$ 消去 $y$, 得 $2 x^{2}=0$, 所以 $x=0, y=1$,故直线与圆的公共点的直角坐标为 $(0,1)$, 化为极坐标为 $\\left(1, \\frac{\\pi}{2}\\right)$.", "solution": "null", "level": "高三", "question": "15. 在极坐标系中, 已知圆 O: $\\rho=\\cos \\theta+\\sin \\theta$ 和直线 $1: \\rho \\sin \\left(\\theta-\\frac{\\pi}{4}\\right)=\\frac{\\sqrt{2}}{2}$,\n\n(1) 求圆 $O$ 和直线 $l$ 的直角坐标方程.", "options": [], "subject": "解析几何", "analysis": ": 分析: 本题主要考查了简单曲线的极坐标方程, 解决问题的关键是转化为普通方程分析计算即可"} {"id": "22810", "image": [], "answer": "答案: 解: 圆 $\\rho^{2}+2 \\rho \\sin \\theta-7=0$ 的直角坐标方程为 $x^{2}+y^{2}+2 y-7=0$,\n\n直线 $\\rho \\cos \\theta+\\rho \\sin \\theta-7=0$ 的直角坐标方程为 $x+y-7=0$, 设点\n\n$P(2 \\sqrt{2} \\cos \\alpha, 2 \\sqrt{2} \\sin \\alpha-1)$,\n\n则点 $P$ 到直线 $x+y-7=0$ 的距离 $d=\\frac{|2 \\sqrt{2} \\cos \\alpha+2 \\sqrt{2} \\sin \\alpha-8|}{\\sqrt{2}}=\\frac{\\left|4 \\sin \\left(\\alpha+\\frac{\\pi}{4}\\right)-8\\right|}{\\sqrt{2}}$,所以 $d_{\\text {min }}=\\frac{4}{\\sqrt{2}}=2 \\sqrt{2} ; d_{\\text {max }}=\\frac{12}{\\sqrt{2}}=6 \\sqrt{2}$.", "solution": "null", "level": "高三", "question": "16. 在极坐标系中, 已知点 $P$ 为圆 $\\rho^{2}+2 \\rho \\sin \\theta-7=0$ 上任一点.求点 $\\mathrm{P}$ 到直线 $\\rho \\cos \\theta+\\rho \\sin \\theta-7=0$ 的距离的最小值与最大值.", "options": [], "subject": "解析几何", "analysis": ": 分析: 本题主要考查了简单曲线的极坐标方程, 解决问题的关键是转化为普通方程计算即可"} {"id": "22811", "image": [], "answer": "答案: 解: 由 $\\rho \\cos \\left(\\theta-\\frac{\\pi}{3}\\right)=1$ 得, $\\rho\\left(\\frac{1}{2} \\cos \\theta+\\frac{\\sqrt{3}}{2} \\sin \\theta\\right)=1$.\n\n从而 $\\mathrm{C}$ 的直角坐标方程为 $\\frac{1}{2} x+\\frac{\\sqrt{3}}{2} y=1$. 即 $x+\\sqrt{3} y=2$.\n\n当 $\\theta=0$ 时, $\\rho=2$ ,所以 $M(2,0)$ ;\n\n当 $\\theta=\\frac{\\pi}{2}$ 时, $\\rho=\\frac{2 \\sqrt{3}}{3}$, 所以 $N\\left(\\frac{2 \\sqrt{3}}{3}, \\frac{\\pi}{2}\\right)$.\n\n(2)设 $\\mathrm{MN}$ 的中点为 $\\mathrm{P}$, 求直线 $\\mathrm{OP}$ 的极坐标方程.\n\n答案: 解: $\\mathrm{M}$ 点的直角坐标为 $(2,0), \\mathrm{N}$ 点的直角坐标为 $\\left(0, \\frac{2 \\sqrt{3}}{3}\\right)$, 所以 $\\mathrm{P}$ 点的直角坐标为 $\\left(1, \\frac{\\sqrt{3}}{3}\\right)$, 则 $\\mathrm{P}$ 点的极坐标为 $\\left(\\frac{2 \\sqrt{3}}{3}, \\frac{\\pi}{6}\\right)$. 所以直线 $\\mathrm{OP}$ 的极坐标方程为 $\\theta=\\frac{\\pi}{6}, \\rho \\in(-\\infty,+\\infty)$.\n\n(注意: 不表示为 $\\theta=\\frac{\\pi}{6}, \\rho \\geq 0$, 这是射线的极坐标方程)", "solution": "null", "level": "高三", "question": "17. 在直角坐标系 $x O y$ 中, 以 $\\mathrm{O}$ 为极点, $\\mathrm{x}$ 轴正半轴为极轴建立极坐标系, 曲线 $\\mathrm{C}$ 的极坐标方程为 $\\rho \\cos \\left(\\theta-\\frac{\\pi}{3}\\right)=1, M, N$ 分别为 $\\mathrm{C}$ 与 $\\mathrm{x}$ 轴, $\\mathrm{y}$ 轴的交点.\n\n(1)写出 $\\mathrm{C}$ 的直角坐标方程, 并求 $\\mathrm{M}, \\mathrm{N}$ 的极坐标.", "options": [], "subject": "解析几何", "analysis": ": 分析: 本题主要考查了简单曲线的极坐标方程, 解决问题的关键是转化为普通方程结合所给几何关系分析计算即可"} {"id": "22812", "image": [], "answer": "答案: 解: 由 $\\rho=4 \\cos \\theta$ 得 $\\rho^{2}=4 \\rho \\cos \\theta$, 所以 $x^{2}+y^{2}=4 x$, 即圆 $O_{1}$ 的直角坐标方程为 $x^{2}+y^{2}-4 x=0$,\n\n同理圆 $O_{2}$ 的直角坐标方为 $x^{2}+y^{2}+4 y=0$.\n\n(2)求经过圆 $O_{1}$, 圆 $O_{2}$ 交点的直线的极坐标方程.\n\n答案: 解: 由 $\\left\\{\\begin{array}{l}x^{2}+y^{2}-4 x=0, \\\\ x^{2}+y^{2}+4 x=0\\end{array}\\right.$ 解得 $\\left\\{\\begin{array}{l}x_{1}=0, \\\\ y_{1}=0,\\end{array}\\right.$ 或 $\\left\\{\\begin{array}{l}x_{2}=2, \\\\ y_{2}=-2 .\\end{array}\\right.$即圆 $O_{1}$ 和圆 $O_{2}$ 交于点 $(0,0)$ 和 $(2,-2)$, 则 $\\tan \\theta=\\frac{-2-0}{2-0}=-1$, 则 $\\theta=\\frac{3 \\pi}{4}$ 或 $\\theta=\\frac{7 \\pi}{4}$.即过圆 $O_{1}$ 和圆 $O_{2}$ 交点的直线的极坐标方程为则 $\\theta=\\frac{3 \\pi}{4}$ 或 $\\theta=\\frac{7 \\pi}{4}$.", "solution": "null", "level": "高三", "question": "18. 圆 $O_{1}$ 和圆 $O_{2}$ 的极坐标方程分别为 $\\rho=4 \\cos \\theta, \\rho=-4 \\sin \\theta$.\n\n(1)把圆 $O_{1}$ 和圆 $O_{2}$ 的极坐标方程化为直角坐标方程.", "options": [], "subject": "解析几何", "analysis": ": 分析: 本题主要考查了简单曲线的极坐标方程, 解决问题的关键是转化为普通方程分析计算即可得到所求量的关系"} {"id": "22813", "image": ["10085.jpg"], "answer": "答案: 解答: 由题意, 设圆上一点 $P(\\rho, \\theta)$,\n\n\n\n因为圆心坐标为 $\\left(3, \\frac{\\pi}{6}\\right)$, 半径为 3 , 故 $\\rho=6 \\cos \\left(\\theta-\\frac{\\pi}{6}\\right)$所以所求圆的极坐标方程为 $\\rho=6 \\cos \\left(\\theta-\\frac{\\pi}{6}\\right)$", "solution": "null", "level": "高三", "question": "19. 圆心的极坐标为 $C\\left(3, \\frac{\\pi}{6}\\right)$, 求半径为 3 的圆的极坐标方程.", "options": [], "subject": "解析几何", "analysis": ": 分析: 本题主要考查了简单曲线的极坐标方程, 解决问题的关键是根据条件转化即可."} {"id": "22815", "image": ["10086.jpg"], "answer": "答案:解答:在圆周上任取一点 $\\mathrm{P}$ (如图)设其极坐标为 $(\\rho, \\theta)$.\n\n由余弦定理知:\n\n\n\n$C P^{2}=O P^{2}+O C^{2}-2 O P \\cdot O C \\cos \\angle C O P$,\n\n$\\therefore r^{2}=\\rho_{0}^{2}+\\rho^{2}-2 \\rho \\rho_{0} \\cos \\left(\\theta-\\theta_{0}\\right)$.\n\n故其极坐标方程为\n\n$r^{2}=\\rho_{0}^{2}+\\rho^{2}-2 \\rho \\rho_{0} \\cos \\left(\\theta-\\theta_{0}\\right)$.", "solution": "null", "level": "高三", "question": "20. 求圆心在 $\\left(\\rho_{0}, \\theta_{0}\\right)$, 半径为 $\\mathrm{r}$ 的圆的方程.", "options": [], "subject": "解析几何", "analysis": ": 分析: 本题主要考查了简单曲线的极坐标方程, 解决问题的关键是(1)圆的极坐标方程是曲线的极坐标方程的一种特殊情况, 其求解过程同曲线的极坐标方程的求法.\n\n(2)特别地, 当圆心在极轴上即 $\\theta_{0}=0$ 时, 方程为 $r^{2}=\\rho_{0}^{2}+\\rho^{2}-2 \\rho \\rho_{0} \\cos \\theta$; 若再有\n\n$\\rho_{0}=r$, 则其方程为 $\\rho=2 \\rho_{0} \\cos \\theta=2 r \\cos \\theta$; 若 $P_{0}=r, Q_{0} \\neq 0$, 则方程为\n\n$\\rho=2 r \\cos \\left(\\theta-\\theta_{0}\\right)$, 这几个方程经常用来判断图形的形状和位置."} {"id": "22816", "image": ["10087.jpg"], "answer": "答案: 解: 设直角三角形的斜边为 $\\mathrm{OD}$, 它的长度是 $2 r$, 以 $\\mathrm{O}$ 为极点, $\\mathrm{OD}$ 所在射线为极轴, 建立极坐标系,如图所示:\n\n\n\n设 $P(\\rho, \\theta)$ 为轨迹上的一点, 则 $O P=\\rho, \\angle x O P=\\theta$.\n\n在直角三角形 $O D P$ 中, $O P=O D \\cdot \\cos \\theta$ ,\n\n$\\because O P=\\rho, O D=2 r$\n\n$\\therefore \\rho=2 r \\cos \\theta(\\rho \\neq 0, \\rho \\neq 2 r)$.\n\n这就是所求轨迹的方程.", "solution": "null", "level": "高三", "question": "21. 设一个直角三角形的斜边长一定, 求直角顶点轨迹的极坐标方程.", "options": [], "subject": "解析几何", "analysis": ": 分析: 本题考查极坐标方程的求法,解答此题需要根据题目特点建立恰当的极坐标系,\n然后再求直角顶点的轨迹方程. (1)求曲线的极坐标方程的步骤如下; (1)建立适当的极坐标系; (2)设 $\\mathrm{P}(\\rho, \\theta)$ 是曲线上任一点; (3)列出 $\\rho, \\theta$ 的关系式; (4)化简整理; (2)极坐标中的坐标是由长度与角度表示的, 因此, 建立极坐标方程常常可以在一个三角形中实现, 找出这样的三角形便形成了解题的关键."} {"id": "22973", "image": [], "answer": "答案: 解: $z=\\left(x+\\frac{1}{x}\\right)\\left(y+\\frac{1}{y}\\right)=x y+\\frac{1}{x y}+\\frac{x}{y}+\\frac{y}{x} \\geq x y+\\frac{1}{x y}+2 \\sqrt{\\frac{x}{y} \\cdot \\frac{y}{x}}=x y+\\frac{1}{x y}+2$,\n\n$\\because \\sqrt{x y} \\leq \\frac{x+y}{2}, \\therefore x y \\leq \\frac{(x+y)^{2}}{4}=\\frac{1}{4}$,\n\n构造函数 $f(x)=x+\\frac{1}{x}$, 易证 $f(x)$ 在 $\\left[0, \\frac{1}{4}\\right]$ 上是单调递减的, $\\therefore x y+\\frac{1}{x y} \\geq 4+\\frac{1}{4}=\\frac{17}{4}$,\n\n$\\therefore z \\geq \\frac{17}{4}+2=\\frac{25}{4}$, 当且仅当 $x=y=\\frac{1}{2}$ 时, “ $=$ ”成立,\n\n$\\therefore z$ 的最小值为 $\\frac{25}{4}$.", "solution": "null", "level": "高三", "question": "24. 已知两正数 $X, y$ 满足 $x+y=1$, 求 $Z=\\left(x+\\frac{1}{x}\\right)\\left(y+\\frac{1}{y}\\right)$ 的最小值", "options": [], "subject": "代数", "analysis": ": 分析: 本题主要考查了基本不等式在最值问题中的应用, 解决问题的关键是首先将 $Z$变形为 $z=\\left(x+\\frac{1}{x}\\right)\\left(y+\\frac{1}{y}\\right)=x y+\\frac{1}{x y}+\\frac{x}{y}+\\frac{y}{x} \\geq x y+\\frac{1}{x y}+2 \\sqrt{\\frac{x}{y} \\cdot \\frac{y}{x}}=x y+\\frac{1}{x y}+2$, 而 $x y \\leq \\frac{(x+y)^{2}}{4}=\\frac{1}{4}$, 因此对于 $x y+\\frac{1}{x y}$ 不能用基本不等式 $x y+\\frac{1}{x y} \\geq 2 \\sqrt{x y \\cdot \\frac{1}{x y}}=2$ (当 $x y=1$ 时“=”成立), $\\therefore$ 可以考虑函数 $f(x)=x+\\frac{1}{x}$ 在 $\\left[0, \\frac{1}{4}\\right]$ 上的单调性, 易得 $f(x)$ 在 $\\left[0, \\frac{1}{4}\\right]$上是单调递减的, 故 $x y+\\frac{1}{x y} \\geq 4+\\frac{1}{4}=\\frac{17}{4}, \\therefore z \\geq \\frac{17}{4}+2=\\frac{25}{4}$, 当且仅当 $x=y=\\frac{1}{2}$ 时, “””成立, 即 $\\mathrm{z}$ 的最小值为 $\\frac{25}{4}$."} {"id": "21763", "image": [], "answer": ":(2)(3)\n\n(1)函数 $y=2 x^{2}-1$ 在 $x=3$ 处的导数为 12 , 故(1)错, 根据变化率在物理学中的含义知 (2)(3)正确.", "solution": "null", "level": "高三", "question": "16. 给出下列结论: (1)函数 $y=2 x^{2}-1$ 在 $x=3$ 处的导数为 11; (2)若物体的运动规律是 $s$ $=f(t)$, 则物体在时刻 $t_{0}$ 的瞬时速度 $v$ 等于 $f^{\\prime}\\left(t_{0}\\right)$; (3)物体做直线运动时, 它的运动规律可以用函数 $v=v(t)$ 描述, 其中 $v$ 表示瞬时速度, $t$ 表示时间, 那么该物体运动的加速度为 $a$ $=\\lim _{\\Delta t \\rightarrow 0} \\frac{v(\\mathrm{t}+\\Delta t)-v(\\mathrm{t})}{\\Delta t}$. 其中正确的结论序号为 \\$ \\qquad \\$ .", "options": [], "subject": "代数", "analysis": ":(2)(3)\n\n(1)函数 $y=2 x^{2}-1$ 在 $x=3$ 处的导数为 12 , 故(1)错, 根据变化率在物理学中的含义知 (2)(3)正确."} {"id": "22993", "image": [], "answer": "答案: 解: 原不等式化为 $|x+7|-|3 x-4|+\\sqrt{2}-1>0$\n当 $x>\\frac{4}{3}$ 时, 原不等式为 $x+7-(3 x-4)+\\sqrt{2}-1>0$\n\n得 $x<5+\\frac{\\sqrt{2}}{2}$, 即 $\\frac{4}{3}0$\n\n得 $x>-\\frac{1}{2}-\\frac{\\sqrt{2}}{4}$, 即 $-\\frac{1}{2}-\\frac{\\sqrt{2}}{4}0$\n\n得 $x>6-\\frac{\\sqrt{2}}{2}$, 与 $x<-7$ 矛盾;\n\n所以解集为 $\\left\\{x \\left\\lvert\\,-\\frac{1}{2}-\\frac{\\sqrt{2}}{4}0$", "options": [], "subject": "代数", "analysis": ": 分析: 本题主要考查了绝对值不等式的解法, 解决问题的关键是根据已知的不等式可知, 化简为 $|x+7|-|3 x-4|+\\sqrt{2}-1>0$, 然后对当 $x>\\frac{4}{3}$ 时, 原不等式为 $x+7-(3 x-4)+\\sqrt{2}-1>0$ 当 $-7 \\leq x \\leq \\frac{4}{3}$ 时, 原不等式为 $x+7+(3 x-4)+\\sqrt{2}-1>0$;当 $x<-7$ 时, 原不等式为 $x+7-(3 x-4)+\\sqrt{2}-1>0$, 分为 3 种情况来解答."} {"id": "22400", "image": [], "answer": "答案: (1) 令 $r=3, T_{4}=\\left(-\\frac{1}{2}\\right)^{3} C_{8}^{3} x^{\\frac{2}{3}}-7 x^{\\frac{2}{3}}$,\n\n(2) 令 $8-2 y=0$, 得 $r=4, \\therefore T_{s}=\\frac{35}{8}$,\n\n(3) 令 $x=1$, 各项系数和为 $\\frac{1}{256}$.", "solution": "null", "level": "高三", "question": "在二项式 $\\left(\\sqrt[3]{x}-\\frac{1}{2 \\sqrt[3]{x}}\\right)^{n}$ 的展开式中, 前三项系数的绝对值成等差数列。\n\n(1) 求展开式的第四项;\n\n(2) 求展开式的常数项;\n\n(3) 求展开式中各项的系数和.", "options": [], "subject": "代数", "analysis": "答案: (1) 令 $r=3, T_{4}=\\left(-\\frac{1}{2}\\right)^{3} C_{8}^{3} x^{\\frac{2}{3}}-7 x^{\\frac{2}{3}}$,\n\n(2) 令 $8-2 y=0$, 得 $r=4, \\therefore T_{s}=\\frac{35}{8}$,\n\n(3) 令 $x=1$, 各项系数和为 $\\frac{1}{256}$."} {"id": "20484", "image": [], "answer": "答案:由 $S_{n}+a_{n}=2 n+1$,\n\n当 $n=1$ 时, $S_{1}=a_{1}, \\therefore a_{1}+a_{1}=2 \\times 1+1$, 得 $a_{1}=\\frac{3}{2}$.\n\n当 $n=2$ 时, $S_{2}=a_{1}+a_{2}$,\n\n则 $a_{1}+a_{2}+a_{2}=5$, 将 $a_{1}=\\frac{3}{2}$ 代入得 $a_{2}=\\frac{7}{4}$.\n\n同理可得 $a_{3}=\\frac{15}{8}$.\n\n$\\therefore a_{n}=\\frac{2^{n+1}-1}{2^{n}}=2-\\frac{1}{2^{n}}$.证明:当 $n=1$ 时, 结论成立.\n\n假设 $n=k$ 时, 命题成立, 即 $a_{k}=2-\\frac{1}{2^{k}}$ ;\n\n当 $n=k+1$ 时, $S_{n}+a_{n}=2 n+1$,\n\n则 $a_{1}+a_{2}+a_{3}+\\cdots+a_{k}+2 a_{k+1}=2(k+1)+1$.\n\n$\\because a_{1}+a_{2}+\\cdots+a_{k}=2 k+1-a_{k}$\n\n$\\therefore 2 a_{k+1}=4-\\frac{1}{2^{k}}, a_{k+1}=2-\\frac{1}{2^{k+1}}$ 成立.\n\n$\\therefore$ 当 $n=k+1$ 时, 结论也成立.\n\n$\\therefore$ 根据上述知对于任意自然数 $n \\in N^{*}$, 结论成立.", "solution": "null", "level": "高三", "question": "24. 已知数列 $\\left\\{a_{n}\\right\\}$ 满足 $S_{n}+a_{n}=2 n+1$.\n\n(1)写出 $a_{1}, a_{2}, a_{3}$, 并推测 $a_{n}$ 的表达式;\n\n\n\n(2)用数学归纳法证明所得的结论.", "options": [], "subject": "代数", "analysis": ": 分析: 本题主要考查了归纳推理, 解决问题的关键是根据归纳推理进行发现证明即可."} {"id": "20485", "image": [], "answer": "答案: 取 $x=1$, 则 $a_{0}=2^{n}$;\n\n取 $x=2, a_{0}+a_{1}+\\cdots+a_{n}=3^{n}, \\therefore s_{n}=a_{1}+a_{2}+\\cdots+a_{n}=3^{n}-2^{n}$要比较 $s_{n}$ 与 $(n-2) \\cdot 2^{n}+2 n^{2}$ 的大小, 即比较 $3^{n}$ 与 $(n-1) \\cdot 2^{n}+2 n^{2}$ 的大小.\n\n当 $n=1$ 时, $3^{n}>(n-1) \\cdot 2^{n}+2 n^{2}$;\n\n当 $n=2,3$ 时, $3^{n}<(n-1) \\cdot 2^{n}+2 n^{2}$;\n\n当 $n=4,5$ 时, $3^{n}>(n-1) \\cdot 2^{n}+2 n^{2}$;\n\n猜想: 当 $n \\geq 4$ 时, $3^{n}>(n-1) \\cdot 2^{n}+2 n^{2}$, 下面用数学归纳法证明:\n\n由上述过程可知, $n=4$ 时结论成立;\n\n假设当 $n=k(k \\geq 4)$ 时结论成立, 即 $3^{k}>(k-1) \\cdot 2^{k}+2 k^{2}$\n\n两边同乘以 3 得: $3^{k+1}>3(k-1) \\cdot 2^{k}+6 k^{2}=k \\cdot 2^{k+1}+2(k+1)^{2}+\\left[(k-3) 2^{k}+4 k^{2}-4 k-2\\right]$\n\n$\\because k \\geq 4$ 时, $(k-3) 2^{k}>0,4 k^{2}-4 k-2 \\geq 4 \\times 4^{2}-4 \\times 4-2>0, \\therefore(k-3) 2^{k}+4 k^{2}-4 k-2>0$\n\n$\\therefore 3^{k+1}>k \\cdot 2^{k+1}+2(k+1)^{2}$, 即 $n=k+1$ 时结论也成立.\n\n$\\therefore$ 当 $n \\geq 4$ 时, $3^{n}>(n-1) \\cdot 2^{n}+2 n^{2}$ 成立.\n\n综上所述, 当 $n=1$ 或 $n \\geq 4$ 时, $3^{n}>(n-1) \\cdot 2^{n}+2 n^{2}$;当 $n=2,3$ 时, $3^{n}<(n-1) \\cdot 2^{n}+2 n^{2}$.", "solution": "null", "level": "高三", "question": "25. 已知 $(x+1)^{n}=a_{0}+a_{1}(x-1)+a_{2}(x-1)^{2}+\\cdots+a_{n}(x-1)^{n}$, (其中 $\\left.n \\in N^{*}\\right)$.\n\n(1) 求 $a_{0}$ 及 $s_{n}=a_{1}+a_{2}+\\cdots+a_{n}$;\n\n \n\n(2) 试比较 $s_{n}$ 与 $(n-2) \\cdot 2^{n}+2 n^{2}$ 的大小, 并用数学归纳法给出证明过程.", "options": [], "subject": "代数", "analysis": ": 分析: 本题主要考查了归纳推理, 解决问题的关键是 (1) 采用赋值法, 令 $x=1$, 右\n边 $=a_{0}=$ 左边 $=2^{n}, s_{n}$ 也采用赋值法, 令 $x=2$; (2) 根据(1)得到 $s_{n}=3^{n}$, 等于比较 $3^{n}$与 $(n-2) \\cdot 2^{n}+2 n^{2}$ 的大小, 首先赋几个特殊值, 采用不完全归纳法, 得到答案, 然后再用数学归纳法证明."} {"id": "23013", "image": [], "answer": "答案: 解: $\\because x^{2}+y^{2} \\geq 2 x y, y^{2}+z^{2} \\geq 2 y z, z^{2}+x^{2} \\geq 2 z x$,\n\n$\\therefore\\left(x^{2}+y^{2}\\right)+\\left(y^{2}+z^{2}\\right)+\\left(z^{2}+x^{2} \\geq 2 x y+2 y z+2 z x\\right.$.\n\n$\\therefore 3\\left(x^{2}+y^{2}+z^{2}\\right) \\geq x^{2}+y^{2}+z^{2}+2 x y+2 y z+2 z x$,\n\n即 $3\\left(x^{2}+y^{2}+z^{2}\\right) \\geq(x+y+z)^{2}=1$.\n\n$\\therefore x^{2}+y^{2}+z^{2} \\geq \\frac{1}{3}$.", "solution": "null", "level": "高三", "question": "17. 已知 $x+y+z=1$, 求证: $x^{2}+y^{2}+z^{2} \\geq \\frac{1}{3}$.", "options": [], "subject": "代数", "analysis": ":【分析】本题主要考查了分析法与综合法, 解决问题的关键是\n\n利用 $a^{2}+b^{2} \\geq 2 a b \\rightarrow$ 同向不等式求和 $\\rightarrow$ 结论"} {"id": "23014", "image": [], "answer": "答案:解:方法一(分析法):\n\n要证 $(a+b)^{-1}+(b+c)^{-1}=3(a+b+c)^{-1}(a+b)^{-1}+(b+c)^{-1}=3(a+b+c)^{-1}$,\n\n即证 $\\frac{1}{a+b}+\\frac{1}{b+c}=\\frac{a}{a+b+c} ,$\n只需证 $\\frac{a+b+c}{a+b}+\\frac{a+b+c}{b+c}=3$,\n\n化简, 得 $\\frac{c}{a+b}+\\frac{a}{b+c}=1$,\n\n即 $c(b+c)+(a+b) a=(a+b)(b+c) ,$\n\n所以只需证 $c^{2}+a^{2}=b^{2}+a c$.\n\n因为 $\\triangle A B C$ 的三个内角 $A, B, C$ 成等差数列,\n\n所以 $\\angle B=60^{\\circ}$.\n\n所以 $\\cos B=\\frac{a^{2}+c^{2}-b^{2}}{2 a c}=\\frac{1}{2}$.\n\n所以 $a^{2}+c^{2}-b^{2}=a c$. 所以原式成立.\n\n方法二(综合法):\n\n因为 $\\triangle A B C$ 的三个内角 $A, B, C$ 成等差数列,\n\n所以 $\\angle B=60^{\\circ}$.\n\n由余弦定理, 有 $b^{2}=c^{2}+a^{2}-2 a c \\cos 60^{\\circ}$,\n\n所以 $c^{2}+a^{2}=a c+b^{2}$.\n\n两边加 $a b+b c$, 得 $c(b+c)+a(a+b)=(a+b) \\cdot(b+c)$,\n\n两边同时除以 $(a+b)(b+c)$, 得 $\\frac{c}{a+b}+\\frac{a}{b+c}=1$,\n\n所以 $\\left(\\frac{c}{a+b}+1\\right)+\\left(\\frac{a}{b+c}+1\\right)=3$,\n\n即 $\\frac{1}{a+b}+\\frac{1}{b+c}=\\frac{a}{a+b+\\varepsilon}$,\n\n所以 $(a+b)^{-1}+(b+c)^{-1}=3(a+b+c)^{-1}$.\n\n.", "solution": "null", "level": "高三", "question": "18. 已知 $\\triangle A B C \\triangle A B C$ 的三个内角 $A, B, C$ 成等差数列, 且 $a, b, c$ 分别为角 $A, B, C$ 的对\n\n边, 求证: $(a+b)^{-1}+(b+c)^{-1}=3(a+b+c)^{-1}$", "options": [], "subject": "代数", "analysis": ":【分析】本题主要考查了分析法与综合法, 解决问题的关键是综合法和分析法各有优\n缺点, 从寻求解题思路来看, 综合法由因导果, 分析法执果索因. 就表达证明过程而论, 综合法形式简洁, 条理清晰; 分析法叙述繁琐, 文辞冗长. 也就是说分析法宜于思考, 综合法宜于表述. 因此, 在实际解题时, 常常把分析法和综合法结合起来运用, 先利用分析法寻求解题思路, 再利用综合法有条理地表述解答过程."} {"id": "23015", "image": [], "answer": "答案: 解: 方法一(分析法):\n\n要证 $a^{3}+b^{2}>a^{2} b+a b^{2}$ 成立,\n\n即需证 $(a+b)\\left(a^{2}-a b+b^{2}\\right)>a b(a+b)$ 成立.\n\n又因 $a+b>0 ,$\n\n故只需证 $a^{2}-a b+b^{2}>a b$ 成立,\n\n即需证 $a^{2}-2 a b+b^{2}>0$ 成立,\n\n即需证 $(a-b)^{2}>0$ 成立.\n\n而依题设 $a \\neq b$ ,则 $(a-b)^{2}>0$ 显然成立.\n\n由此命题得证.\n\n方法二(综合法):\n\n$a \\neq b \\Leftrightarrow a-b \\neq 0 \\Leftrightarrow(a-b)^{2}>0 \\Leftrightarrow a^{2}-2 a b+b^{2}>0 \\Leftrightarrow a^{2}-a b+b^{2}>a b$.\n\n注意到 $a, b \\in(0,+\\infty), a+b>0 a+b>0$, 由上式即得\n\n$(a+b)\\left(a^{2}-a b+b^{2}\\right)>a b(a+b)$.\n\n所以 $a^{3}+b^{3}>a^{2} b+a b^{2}$.", "solution": "null", "level": "高三", "question": "19. 设 $a, b \\in(0,+\\infty)$, 且 $a \\neq b$, 求证: $a^{2}+b^{3}>a^{2} b+a b^{2}$. 提示:\n\n$\\left.a^{3}+b^{3}=(a+b)\\left(a^{2}-a b+b^{2}\\right)\\right)$", "options": [], "subject": "代数", "analysis": ":【分析】本题主要考查了分析法与综合法, 解决问题的关键是根据分析法.综合法结合所学基本不等式进行分析证明即可"} {"id": "23017", "image": [], "answer": "答案: 解: $\\because a, b, c$ 为不全相等的正数, 且 $a b c=1$,\n\n$\\therefore \\frac{1}{a}+\\frac{1}{b}+\\frac{1}{c}=b c+c a+a b$.\n\n又 $b c+c a \\geq 2 \\sqrt{b c} \\cdot \\sqrt{c a}=2 \\sqrt{c}$ ,\n\n$c a+a b>2 \\sqrt{c a} \\cdot \\sqrt{a b}=2 \\sqrt{a}, a b+b c>2 \\sqrt{a b} \\cdot \\sqrt{b c}=2 \\sqrt{b}$, 且 $a, b, c$ 不全相等, $\\therefore$ 上述三个不等式中的“=”不能同时成立. $\\therefore 2(b c+c a+a b)>2(\\sqrt{c}+\\sqrt{a}+\\sqrt{b})$, 即\n\n$b c+c a+a b>\\sqrt{a}+\\sqrt{b}+\\sqrt{c}$.\n\n故 $\\frac{1}{a}+\\frac{1}{b}+\\frac{1}{c}>\\sqrt{a}+\\sqrt{b}+\\sqrt{c}$.", "solution": "null", "level": "高三", "question": "20. 设 $a, b, c a, b, c$ 为不全相等的正数, 且 $a b c=1$, 求证: $\\frac{1}{a}+\\frac{1}{b}+\\frac{1}{c}>\\sqrt{a}+\\sqrt{b}+\\sqrt{c}$.", "options": [], "subject": "代数", "analysis": ":【分析】本题主要考查了分析法与综合法, 解决问题的关键是根据 $a, b, c$ 为不全相等的正数, 且 $a b c=1$, 得到 $\\frac{1}{a}+\\frac{1}{b}+\\frac{1}{c}=b c+c a+a b$, 然后运用基本不等式性质分析计算证明即可."} {"id": "20505", "image": [], "answer": ": 证明: 假设 $a, b, c$ 均小于 1 , 即 $a<1, b<1, c<1$, 则有 $a+b+c<3$,\n\n而 $\\mathrm{a}+\\mathrm{b}+\\mathrm{c}=2 \\mathrm{x}^{2}-2 x+\\frac{1}{2}+3=2\\left(x-\\frac{1}{2}\\right)^{2}+3 \\geq 3$,\n\n两者矛盾; 故 $\\mathrm{a}, \\mathrm{b}, \\mathrm{c}$ 至少有一个不小于", "solution": "null", "level": "高三", "question": "21. 已知 $x \\in \\mathrm{R}, \\mathrm{a}=x^{2}+\\frac{1}{2}, \\mathrm{~b}=2-x, \\mathrm{c}=x^{2}-x+1$, 试证明 $\\mathrm{a}, \\mathrm{b}, \\mathrm{c}$ 至少有一个不小于 1 .", "options": [], "subject": "代数", "analysis": ": 证明: 假设 $a, b, c$ 均小于 1 , 即 $a<1, b<1, c<1$, 则有 $a+b+c<3$,\n\n而 $\\mathrm{a}+\\mathrm{b}+\\mathrm{c}=2 \\mathrm{x}^{2}-2 x+\\frac{1}{2}+3=2\\left(x-\\frac{1}{2}\\right)^{2}+3 \\geq 3$,\n\n两者矛盾; 故 $\\mathrm{a}, \\mathrm{b}, \\mathrm{c}$ 至少有一个不小于"} {"id": "20507", "image": [], "answer": "答案:证明: 任取 $x_{1}, x_{2} \\in(-1,+\\infty)$, 不妨设 $x_{1}1, a x_{1}
0$.\n\n又 $\\because x_{1}+1>0, x_{2}+1>0$,\n\n$\\therefore \\frac{x_{2}-2}{x_{2}+1}-\\frac{x_{1}-2}{x_{1}+1}$\n\n$=\\frac{\\left(x_{2}-2\\right)\\left(x_{1}+1\\right)-\\left(x_{1}-2\\right)\\left(x_{2}+1\\right)}{\\left(x_{1}+1\\right)\\left(x_{2}+1\\right)}$\n\n$=\\frac{3\\left(x_{2}-x_{1}\\right)}{\\left(x_{1}+1\\right)\\left(x_{2}+1\\right)}>0$,\n\n于是 $\\mathrm{f}\\left(x_{2}\\right)-\\mathrm{f}\\left(x_{1}\\right)=\\mathrm{ax}_{2}-\\mathrm{ax_{1 }}+\\frac{x_{2}-2}{x_{2}+1}-\\frac{x_{1}-2}{x_{1}+1}>0$,\n\n即 $\\mathrm{f}\\left(x_{2}\\right)>\\mathrm{f}\\left(x_{1}\\right)$,\n\n故函数 $\\mathrm{f}(x)$ 在 $(-1,+\\infty)$ 上为增函数证明:假设存在 $x_{0}<0\\left(x_{0} \\neq-1\\right)$ 满足 $\\mathrm{f}\\left(x_{0}\\right)=0$,\n\n则 $\\mathrm{a}_{0}=-\\frac{x_{0}-2}{x_{0}+1}$.\n\n$\\because a>1$\n\n$\\therefore 0<\\mathrm{ax}_{0}<1$.\n\n$\\therefore 0<-\\frac{x_{0}-2}{x_{0}+1}<1$, 即 $\\frac{1}{2}1)$.\n\n(1)证明:函数 $\\mathrm{f}(x)$ 在 $(-1,+\\infty)$ 上为增函数;\n .\n\n(2)用反证法证明方程 $\\mathrm{f}(x)=0$ 没有负数根.", "options": [], "subject": "代数", "analysis": ":分析:本题主要考查了综合法的思考过程、特点及应用、反证法的应用, 解决问题的关键是(1)根据所给条件结合所求命题综合分析计算即可; (2)运用反证法的证明方法进行证明即可."} {"id": "20508", "image": [], "answer": "答案:证明: $\\mathrm{a}^{\\mathrm{m}^{+} \\mathrm{n}}+\\mathrm{b}^{\\mathrm{m}^{+} \\mathrm{n}}-\\left(\\mathrm{a}^{\\mathrm{m}} \\mathrm{b}^{\\mathrm{n}}+\\mathrm{a}^{\\mathrm{n}} \\mathrm{b}^{\\mathrm{m}}\\right)$\n\n$=\\left(a^{m^{+}}-a^{m} b^{n}\\right)-\\left(a^{n} b^{m}-b^{m^{+}}\\right)=a^{m}\\left(a^{n}-b^{n}\\right)-b^{m}\\left(a^{n}-b^{n}\\right)=\\left(a^{m}-b^{m}\\right)\\left(a^{n}-b^{n}\\right)$.\n\n当 $a>b$ 时, $a^{m}>b^{m}, a^{n}>b^{n}, \\therefore\\left(a^{m}-b^{m}\\right)\\left(a^{n}-b^{n}\\right)>0$;\n\n当 $a0$;\n\n当 $a=b$ 时, $a^{m}=b^{m}, a^{n}=b^{n}, \\therefore\\left(a^{m}-b^{m}\\right)\\left(a^{n}-b^{n}\\right)=0$.\n\n综上, $\\left(a^{m}-b^{m}\\right)\\left(a^{n}-b^{n}\\right) \\geq 0$, 即 $a^{m^{+} n}+b^{m^{+}} n a^{m} b^{n}+a^{n} b^{m}$.", "solution": "null", "level": "高三", "question": "24. 已知 $\\mathrm{a}>0, \\mathrm{~b}>0, \\mathrm{~m}>0, \\mathrm{n}>0$, 求证: $\\mathrm{a}^{\\mathrm{m}+\\mathrm{n}}+\\mathrm{b}^{\\mathrm{m}+\\mathrm{n}} \\geq \\mathrm{a}^{\\mathrm{m}} \\mathrm{b}^{\\mathrm{n}}+\\mathrm{a}^{\\mathrm{n}} \\mathrm{b}^{\\mathrm{m}}$.", "options": [], "subject": "代数", "analysis": ": 分析: 本题主要考查了分析法的思考过程、特点及应用, 解决问题的关键是利用作差比较, 因式分解的方法, 分类讨论思想, 对 $\\mathrm{a}, \\mathrm{b}$ 的大小关系讨论, 可证不等式成立."} {"id": "23032", "image": [], "answer": "答案:证明:假设方程 $x^{2}-2 x+5-p^{2}=0$ 有实根, 则该方程根的判别式\n\n$\\Delta=4-4\\left(5-p^{2}\\right) \\geq 0$, 解得 $p \\geq 2$ 或 $p \\leq-2$. 而由已知条件实数 $p$ 满足不等式\n$(2 p+1) \\cdot(p+2)<0$, 解得 $-2\\frac{1}{3} \\text { 或 } a<-1, \\\\ -21$ 矛盾,\n\n$\\therefore$ 原假设错误,\n\n$\\therefore a . b . c . d$ 中至少有一个是负数.", "solution": "null", "level": "高三", "question": "20. 实数 $a . b . c . d$ 满足 $a+b=c+d=1, a c+b d>1$, 求证 $a . b . c . d$ 中至少有一个是负数.", "options": [], "subject": "代数", "analysis": ":【分析】本题主要考查了反证法与放缩法, 解决问题的关键 “至多”.“至少”型命题的证明方法. 解答本题应假设 $a . b . c . d$ 都是非负数, 然后证明并得出矛盾. (1)在证明中含有“至少”.“至多”.“最多”等字眼时,或证明否定性命题.惟一性命题时,可使用反证法证明.在证明中常见的矛盾可以与题设矛盾, 也可以与已知矛盾, 与显然的事实矛盾, 也可以自相矛盾.(2)在用反证法证明的过程中, 由于作出了与结论相反的假设, 相当于增加了题设条件, 因此在证明过程中必须使用这个增加的条件, 否则将无法推出矛盾."} {"id": "20809", "image": ["9479.jpg", "9479.jpg"], "answer": "(1) $\because z_{1} z_{2}=-5+5 \\mathrm{i}, \therefore z_{2}=\frac{-5+5 \\mathrm{i}}{z_{1}}=\frac{-5+5 \\mathrm{i}}{-2+\\mathrm{i}}=3-\\mathrm{i}$.$\because z_{3}$ 所对应的点在第四象限,$\therefore\\left\\{\begin{array}{l}-(m-1)>0, \\ m^{2}-2 m-3<0,\\end{array}\right.$ 解得 $-1$\because z_{3}$ 所对应的点在第四象限,$\therefore\\left\\{\begin{array}{l}-(m-1)>0, \\ m^{2}-2 m-3<0,\\end{array}\right.$ 解得 $-10, \\ m^{2}-2 m-3<0,\\end{array}\right.$ 解得 $-1\\sqrt{a+8}+\\sqrt{a+5}$只需证: $(\\sqrt{a+6}+\\sqrt{a+7})^{2}>(\\sqrt{a+8}+\\sqrt{a+5})^{2}只需证: $2 a+13+2 \\sqrt{(a+6)(a+7)}>2 a+13+2 \\sqrt{(a+8)(a+5)}$只需证: $\\sqrt{(a+6)(a+7)}>\\sqrt{(a+8)(a+5)}$只需证: $a^{2}+13 a+42>a^{2}+13 a+40$只需证: $42>40$而 $42>40$ 显然成立, 所以原不等式得证.(2)假设结论不成立, 即 ${ }^{a+\frac{1}{b} \text { 与 } b+\frac{1}{a}}$ 都小于 2,则 $a+\frac{1}{b}+b+\frac{1}{a}<2+2=4$而由基本不等式, $a+\frac{1}{a} \\geq 2, b+\frac{1}{b} \\geq 2$ ,$\therefore a+\frac{1}{b}+b+\frac{1}{a}=a+\frac{1}{a}+b+\frac{1}{b} \\geq 2+2=4$ 与(1)式矛盾", "solution": "null", "level": "高三", "question": "回答下列问题.\n\n(1)请用分析法证明: $\\sqrt{a+6}+\\sqrt{a+7}>\\sqrt{a+8}+\\sqrt{a+5}$;\n\n(2)请用反证法证明: 设 $a>0, b>0$, 则 ${ }^{a+\\frac{1}{b}}$ 与 $^{b+\\frac{1}{a}}$ 中至少有一个不小于 2 .", "options": [], "subject": "代数", "analysis": ""} {"id": "21332", "image": [], "answer": "(1) $2 x+y-13=0$ ;(2) 32 .", "solution": "null", "level": "高三", "question": "(12 分) 已知函数 $f(x)=12-x^{2}$.", "options": [], "subject": "代数", "analysis": "(1) 因为 $f(x)=12-x^{2}$, 所以 $f^{\\prime}(x)=-2 x$,设切点为 $\\left(x_{0}, 12-x_{0}\right)$, 则 $-2 x_{0}=-2$, 即 $x_{0}=1$, 所以切点为 $(1,11)$,由点斜式可得切线方程为 $y-11=-2(x-1)$, 即 $2 x+y-13=0$.(2) 显然 $t \neq 0$,因为 $y=f(x)$ 在点 $\\left(t, 12-t^{2}\right)$ 处的切线方程为 $y-\\left(12-t^{2}\right)=-2 t(x-t)$,令 $x=0$, 得 $y=t^{2}+12$; 令 $y=0$, 得 $x=\frac{t^{2}+12}{2 t}$,所以 $S(t)=\frac{1}{2} \times\\left(t^{2}+12\right) \\cdot \frac{t^{2}+12}{2|t|} ,$不妨设 $t>0$ ( $t<0$ 时, 结果一样),则 $S(t)=\frac{t^{4}+24 t^{2}+144}{4 t}=\frac{1}{4}\\left(t^{3}+24 t+\frac{144}{t}\right)$ ,所以 $S^{t}(t)=\frac{1}{4}\\left(3 t^{2}+24-\frac{144}{t^{2}}\right)=\frac{3\\left(t^{4}+8 t^{2}-48\right)}{4 t^{2}}=\frac{3\\left(t^{2}-4\right)\\left(t^{2}+12\right)}{4 t^{2}}$$=\frac{3(t-2)(t+2)\\left(t^{2}+12\right)}{4 t^{2}}$,由 $S^{t}(t)>0$, 得 $t>2$; 由 $S^{t}(t)<0$, 得 $00)$, 则 $f^{\\prime}(x)=\frac{1}{x}+2 x=\frac{2 x^{2}+1}{x}$,当 $x \\in[1, e]$ 时, $f^{\\prime}(x)>0$, 所以 $f(x)$ 在 $[1, e]$ 上是增函数,当 $x=1$ 时, $f(x)$ 取得最小值 $f(1)=1$,所以 $f(x)$ 在 $[1, e]$ 上的最小值为 1 .(2) $f(x)=\\ln x-2 a x+x^{2}$, 则 $f^{\\prime}(x)=\frac{2 x^{2}-2 a x+1}{x}(x>0)$,令 $h(x)=2 x^{2}-2 a x+1(x>0)$,(1) 当 $a \\leq 0$ 时, $h(x)>0$ 在 $(0,+\\infty)$ 上恒成立, 此时 $f^{\\prime}(x)>0$,所以 $f(x)$ 在 $(0,+\\infty)$ 上单调递增,此时, 函数 $f(x)$ 没有极值点;(2)当 $a>0$ 时, 当 $\\Delta=(-2 a)^{2}-8 \\leq 0$, 即 $00$, 即 $a>\\sqrt{2}$ 时, 令 $h(x)=0$, 则 $x=\frac{a \\pm \\sqrt{a^{2}-2}}{2}$,当 $\frac{a-\\sqrt{a^{2}-2}}{2}\frac{a+\\sqrt{a^{2}-2}}{2}$ 时, $h(x)>0$, 即 $f^{\\prime}(x)>0$,所以当 $a>\\sqrt{2}$ 时, $x=\frac{a-\\sqrt{a^{2}-2}}{2}$ 是函数 $f(x)$ 的极大值点; $x=\frac{a+\\sqrt{a^{2}-2}}{2}$ 是函数 $f(x)$ 的极小值点,综上, 当 $a \\leq \\sqrt{2}$ 时, 函数 $f(x)$ 没有极值点;当 $a>\\sqrt{2}$ 时, $x=\frac{a-\\sqrt{a^{2}-2}}{2}$ 是函数 $f(x)$ 的极大值点; $x=\frac{a+\\sqrt{a^{2}-2}}{2}$ 是函数 $f(x)$ 的极小值点."} {"id": "21336", "image": ["9547.jpg"], "answer": "(1) $(e,+\\infty)$; (2) 证明见解析.", "solution": "null", "level": "高三", "question": "(12 分) 已知函数 $f(x)=x-a \\ln x(x>0, a \\in \\mathbf{R})$ 有两个零点 $x_{1}, x_{2}$, 且\n\n$x_{1}e^{2}$.", "options": [], "subject": "代数", "analysis": "(1) 令 $f(x)=x-a \\ln x=0, \therefore \\ln x=\frac{1}{a} x$, 令 $y_{1}=\\ln x, y_{2}=\frac{1}{a} x$,当 $y_{1}$ 与 $y_{2}$ 相切时, 如图所示:设切点为 $\\left(x_{0}, \\ln x_{0}\right)$, 则 $y_{1}^{\\prime}=\frac{1}{x},\\left.y_{1}^{\\prime}\right|_{x=x_{0}}=\frac{1}{x_{0}}=\frac{1}{a}, \therefore x_{0}=a$,即切点坐标是 $(a, \\ln a)$, 把 $(a, \\ln a)$ 代 $y_{2}=\frac{1}{a} x$, 解得 $a=e$,若 $y=f(x)$ 有两个零点 $x_{1}, x_{2}$, 即 $y_{1}, y_{2}$ 有 2 个交点, 只需 $a \\geq \frac{4}{3}$ 即可, 即 $a>e ,$$a$ 的范围是 $(e,+\\infty)$.(2) 由题意知: $x_{1}-a \\ln x_{1}=0, x_{2}-a \\ln x_{2}=0$,即 $\\ln x_{1}=\frac{x_{1}}{a}, \\ln x_{2}=\frac{x_{2}}{a}$,$\\ln x_{1}+\\ln x_{2}=\frac{1}{a}\\left(x_{1}+x_{2}\right)(1)$ $\\ln x_{1}-\\ln x_{2}=\frac{1}{a}\\left(x_{1}-x_{2}\right)$, 即 $\frac{1}{a}=\frac{\\ln x_{1}-\\ln x_{2}}{x_{1}-x_{2}}$ (2)要证 $x_{1} \\cdot x_{2}>e^{2}$ 成立, 即证 $\\ln x_{1} \\cdot x_{2}>\\ln e^{2}$ 成立,即证 $\\ln x_{1}+\\ln x_{2}>2$,由(1)知: 即证 $\frac{1}{a}\\left(x_{1}+x_{2}\right)>2$, 即证 $\frac{1}{a}>\frac{2}{x_{1}+x_{2}}$ ,又由(2)知: 即证 $\frac{\\ln x_{1}-\\ln x_{2}}{x_{1}-x_{2}}>\frac{2}{x_{1}+x_{2}}$, 即证 $\\ln \frac{x_{1}}{x_{2}}>\frac{2\\left(x_{1}-x_{2}\right)}{x_{1}+x_{2}}$,即证 $\\ln \frac{x_{1}}{x_{2}}>\frac{2\\left(\frac{x_{1}}{x_{2}}-1\right)}{\frac{x_{1}}{x_{2}}+1}$,令 $t=\frac{x_{1}}{x_{2}}$, 则 $0\frac{2(t-1)}{t+1}$,设 $g(t)=\\ln t-\frac{2(t-1)}{t+1},(0g(1)=\\ln 1-\frac{2 \times(1-1)}{1+1}=0$, 即 $\\ln \frac{x_{1}}{x_{2}}>\frac{2\\left(\frac{x_{1}}{x_{2}}-1\right)}{\frac{x_{1}}{x_{2}}+1}$ 成立, 故 $x_{1} \\cdot x_{2}>e^{2}$ 得证."} {"id": "21351", "image": [], "answer": "(1)2;(2)", "solution": "null", "level": "高三", "question": "(10 分) 设 $a>0, b>0$, 且 $a+b=\\frac{1}{a}+\\frac{1}{b}$.\n\n(1) 求 $a+b$ 的最小值;\n\n(2) 证明: $a^{2}+a<2$ 与 $b^{2}+b<2$ 不可能同时成立.", "options": [], "subject": "代数", "analysis": "由 $a+b=\frac{1}{a}+\frac{1}{b}=\frac{a+b}{a b}$, 且 $a>0, b>0$, 得 $a b=1$.(1)由基本不等式及 $a b=1$, 知 $a+b \\geq 2 \\sqrt{a b}=2$, 当且仅当 $a=b=1$ 时取等号,故 $a+b$ 的最小值为 2 .(2) 证明: 由(1)知 $a^{2}+b^{2} \\geq 2 a b=2$ ,且 $a+b \\geq 2$,因此 $a^{2}+b^{2}+a+b \\geq 4$, (1)假设 $a^{2}+a<2$ 与 $b^{2}+b<2$ 同时成立,则 $a^{2}+b^{2}+a+b<4$ ,(2)(1)2)两式矛盾, 故 $a^{2}+a<2$ 与 $b^{2}+b<2$ 不可能同时成立."} {"id": "21352", "image": [], "answer": "证明:假设原命题不成立,即 $x^{2}+p_{1} x+q_{1}=0$ 与 $x^{2}+p_{2} x+q_{2}=0$ 都无实根,$\therefore \\Delta_{1}=p_{1}^{2}-4 q_{1}<0, \\Delta_{2}=p_{2}^{2}-4 q_{2}<0$,两式相加得 $p_{1}^{2}+p_{2}^{2}-4 q_{1}-4 q_{2}<0$, 即 $p_{1}^{2}+p_{2}^{2}<4\\left(q_{1}+q_{2}\right)$,又 $\because p_{1} p_{2}=4\\left(q_{1}+q_{2}\right), \therefore p_{1}^{2}+p_{2}^{2}0$ 时, 有 $\frac{a+b}{2} \\geq \\sqrt{a b}>0$, $\therefore \\lg \frac{a+b}{2} \\geq \\lg \\sqrt{a b}, \\quad \therefore \\lg \frac{a+b}{2} \\geq \frac{1}{2} \\lg (a b)=\frac{\\lg a+\\lg b}{2}$,$\therefore \\lg \frac{a+b}{2} \\geq \frac{\\lg a+\\lg b}{2}$.(2) 要证 $\\sqrt{6}+\\sqrt{10}>2 \\sqrt{3}+2$, 只要证,即 $2 \\sqrt{60}>2 \\sqrt{48}$, 显然成立的,所以,原不等式成立.", "solution": "null", "level": "高三", "question": "(12 分) 用综合法或分析法证明:\n\n(1) 如果 $a, b>0$, 则 $\\lg \\frac{a+b}{2} \\geq \\frac{\\lg a+\\lg b}{2}$;\n\n(2) $\\sqrt{6}+\\sqrt{10}>2 \\sqrt{3}+2$.", "options": [], "subject": "代数", "analysis": ""} {"id": "21355", "image": [], "answer": "证明: (1) 要证明 $f\\left(\frac{a}{b}\right)+f\\left(\frac{b}{a}\right) \\leq \frac{2}{3}$只需证明 $\frac{1}{\frac{a}{b}+2}+\frac{1}{\frac{b}{a}+2} \\leq \frac{2}{3}$, 只需证明 $\frac{b}{a+2 b}+\frac{a}{b+2 a} \\leq \frac{2}{3}$,即证 $\frac{b^{2}+4 a b+a^{2}}{2 a^{2}+5 a b+2 b^{2}} \\leq \frac{2}{3}$, 即证 $(a-b)^{2} \\geq 0$, 这显然成立, $f\\left(\frac{a}{b}\right)+f\\left(\frac{b}{a}\right) \\leq \frac{2}{3}$.(2) 假设 $a f(b), b f(a)_{\text {都小于或等于 }} \frac{1}{2}$, 即 $\frac{a}{b+2} \\leq \frac{1}{2}, \frac{b}{a+2} \\leq \frac{1}{2}$,所以 $2 a \\leq b+2,2 b \\leq a+2$, 两式相加得 $a+b \\leq 4$ ,这与 ${ }^{a+b>4}$ 矛盾, 所以 ${ }^{a f(b)}, b f(a)$ 中至少有一个大于 $\frac{1}{2}$.", "solution": "null", "level": "高三", "question": "(12 分) 设函数 $f(x)=\\frac{1}{x+2}, a, b \\in(0,+\\infty)$.\n\n问:(1)此表第 $n$ 行的最后一个数是多少?\n\n(2)此表第 $n$ 行的各个数之和是多少?\n\n(3) 2008 是第几行的第几个数?\n\n(1) 用分析法证明: $f\\left(\\frac{a}{b}\\right)+f\\left(\\frac{b}{a}\\right) \\leq \\frac{2}{3}$;\n\n(2) 设 $a+b>4$, 求证: $a f(b), b f(a)$ 中至少有一个大于 $\\frac{1}{2}$.", "options": [], "subject": "代数", "analysis": ""} {"id": "20640", "image": [], "answer": "答案:证明见解析。", "solution": "null", "level": "高三", "question": "(10 分) 已知 $x, y \\in \\mathrm{R}^{+}$, 且 $x+y>2$, 求证: $\\frac{1+x}{y}$ 与 $\\frac{1+y}{x}$ 中至少有一个小于", "options": [], "subject": "代数", "analysis": "答案:证明见解析。\n\n【解析】证明: 假设 $\\frac{1+x}{y}$ 与 $\\frac{1+y}{x}$ 都大于或等于 2 , 即 $\\left\\{\\begin{array}{l}\\frac{1+x}{y} \\geq 2 \\\\ \\frac{1+y}{x} \\geq 2\\end{array}\\right.$,\n\n因为 $x, y \\in \\mathbf{R}^{+}$, 故可化为 $\\left\\{\\begin{array}{l}1+x \\geq 2 y \\\\ 1+y \\geq 2 x\\end{array}\\right.$, 两式相加, 得 $x+y \\leq 2$,\n\n与已知 $x+y>2$ 矛盾,\n\n所以假设不成立,即原命题成立."} {"id": "20642", "image": [], "answer": "答案:证明见解析.", "solution": "null", "level": "高三", "question": "(12 分) 已知 $a>0$, 证明: $\\sqrt{a^{2}+\\frac{1}{a^{2}}}-\\sqrt{2} \\geq a+\\frac{1}{a}-2$.", "options": [], "subject": "代数", "analysis": "答案:证明见解析.\n\n【解析】证明: 要证 $\\sqrt{a^{2}+\\frac{1}{a^{2}}}-\\sqrt{2} \\geq a+\\frac{1}{a}-2$,\n\n只需证 $\\sqrt{a^{2}+\\frac{1}{a^{2}}} \\geq\\left(a+\\frac{1}{a}\\right)-(2-\\sqrt{2})$.\n因为 $a>0, a+\\frac{1}{a} \\geq 2 \\sqrt{a \\times \\frac{1}{a}}=2$, 所以 $\\left(a+\\frac{1}{a}\\right)-(2-\\sqrt{2})>0$,\n\n所以只需证 $\\left(\\sqrt{a^{2}+\\frac{1}{a^{2}}}\\right)^{2} \\geq\\left[\\left(a+\\frac{1}{a}\\right)-(2-\\sqrt{2})\\right]^{2}$,\n\n即 $2(2-\\sqrt{2})\\left(a+\\frac{1}{a}\\right) \\geq 8-4 \\sqrt{2}$ ,\n\n只需证 $a+\\frac{1}{a} \\geq 2$.\n\n因为 $a>0, a+\\frac{1}{a} \\geq 2$ 显然成立, 当 $a=\\frac{1}{a}=1$ 时等号成立,\n\n所以要证的不等式成立."} {"id": "20643", "image": [], "answer": "B", "solution": "null", "level": "高三", "question": "$\\overrightarrow{A B} \\cdot \\overrightarrow{A C}=\\overrightarrow{A D}^{2}-\\overrightarrow{D B}^{2}$\n\n(2) 已知 $a>b>c$, 且 $a+b+c=0$, 求证: $\\frac{\\sqrt{b^{2}-a c}}{a}<\\sqrt{3}$.", "options": [], "subject": "代数", "analysis": "反证法的步骤为: 假设结论不成立, 推导出矛盾, 得到结论, 据此知顺序为(3)(1)(2)."} {"id": "21372", "image": [], "answer": "(1) 0.9 ; (2) $6 x_{0}+3 \\Delta x$", "solution": "null", "level": "高三", "question": "(10 分) 已知函数 $f(x)=3 x^{2}+5$, 求 $f(x)$ :\n\n(1) 从", "options": [], "subject": "代数", "analysis": "(1) 因为 $f(x)=3 x^{2}+5$,所以从 0.1 到 0.2 的平均变化率为 $\frac{3 \times 0.2^{2}+5-3 \times 0.1^{2}-5}{0.2-0.1}=0.9$.(2) $f\\left(x_{0}+\\Delta x\right)-f\\left(x_{0}\right)=3\\left(x_{0}+\\Delta x\right)^{2}+5-\\left(3 x_{0}^{2}+5\right)$$=3 x_{0}^{2}+6 x_{0} \\Delta x+3(\\Delta x)^{2}+5-3 x_{0}^{2}-5=6 x_{0} \\Delta x+3(\\Delta x)^{2}$,所以函数 $f(x)$ 在区间 $\\left[x_{0}, x_{0}+\\Delta x\right]$ 上的平均变化率为 $\frac{6 x_{0} \\Delta x+3(\\Delta x)^{2}}{\\Delta x}=6 x_{0}+3 \\Delta x$."} {"id": "22376", "image": [], "answer": "答案: (1)分为三类:从国画中选, 有 5 种不同的选法; 从油画中选, 有 2 种不同的选法; 从水彩画中选, 有 7 种不同的选法.根据分类加法计数原理得,共有 $5+2+7=14$ 种不同的选法.\n\n(2)分为三步:国画、油画、水彩画各有 5 种、 2 种、 7 种不同的选法,根据分步乘法计数原理得,共有 $5 \\times 2 \\times 7=70$ 种不同的选法.\n\n(3)分为三类:\n\n第一类是一幅选自国画,一幅选肖油画.由分步乘法计数原理知, 有 $5 \\times 2=10$ 种不同的选法.\n\n第二类是一幅选自国画,一幅选自水彩画,有 $5 \\times 7=35$ 种不同的选法.\n\n第三类是一幅选自油画,一幅选自水彩画,有 $2 \\times 7=14$ 种不同的选法.\n\n所以从这些画中选出两幅不同种类的画布置房间, 有 $10+35+14=59$ 种不同的选法.", "solution": "null", "level": "高三", "question": "现有 5 幅不同的国画, 2 幅不同的油画, 7 幅不同的水彩画.\n\n(1)从中任选一幅画布置房间,有几种不同的选法?\n\n(2)从这些国画、油画、水彩画中各选一幅布置房间, 有几种不同的选法?\n\n(3)从这些画中选出两幅不同种类的画布置房间,有几种不同的选法?", "options": [], "subject": "组合数学", "analysis": "答案: (1)分为三类:从国画中选, 有 5 种不同的选法; 从油画中选, 有 2 种不同的选法; 从水彩画中选, 有 7 种不同的选法.根据分类加法计数原理得,共有 $5+2+7=14$ 种不同的选法.\n\n(2)分为三步:国画、油画、水彩画各有 5 种、 2 种、 7 种不同的选法,根据分步乘法计数原理得,共有 $5 \\times 2 \\times 7=70$ 种不同的选法.\n\n(3)分为三类:\n\n第一类是一幅选自国画,一幅选肖油画.由分步乘法计数原理知, 有 $5 \\times 2=10$ 种不同的选法.\n\n第二类是一幅选自国画,一幅选自水彩画,有 $5 \\times 7=35$ 种不同的选法.\n\n第三类是一幅选自油画,一幅选自水彩画,有 $2 \\times 7=14$ 种不同的选法.\n\n所以从这些画中选出两幅不同种类的画布置房间, 有 $10+35+14=59$ 种不同的选法."} {"id": "22385", "image": [], "answer": "答案: (1)第一个股子有 6 种不同的结果,第二个骰子与第一个的结果不同, 有 5 种不同的结果;\n\n同理第三个股子有 4 种不同的结果,共有 $6 \\times 5 \\times 4=120$ 个不同的结果.\n\n(2)与 1 相比,后两个剘子都可以有 6 种不同的结果,共有 $6^{3}=216$ 个不同的结果.", "solution": "null", "level": "高三", "question": "用一颗吺毃连掷三次,投郑出的数字顺次排成一个三位数,则:\n\n(1)各位数字互不相同的三位数有多少个?\n\n(2)可以排出多少个不同的三位数?\n\n21 世纪教育网(www.21cnjy.com)", "options": [], "subject": "组合数学", "analysis": "答案: (1)第一个股子有 6 种不同的结果,第二个骰子与第一个的结果不同, 有 5 种不同的结果;\n\n同理第三个股子有 4 种不同的结果,共有 $6 \\times 5 \\times 4=120$ 个不同的结果.\n\n(2)与 1 相比,后两个剘子都可以有 6 种不同的结果,共有 $6^{3}=216$ 个不同的结果."} {"id": "22409", "image": [], "answer": "答案:(1)由第 4 项和第 9 项的二项式系数相等可得 $C_{n}^{3}=C_{n}^{\\infty}$, 解得 $n=11$.\n\n(2)由(1)知, 展开式的第 $k+1$ 项为\n\n$T_{k+1}=C_{11}^{k}(\\sqrt{x})^{11-k}\\left(-\\frac{2}{x}\\right)^{k}=(-2)^{k} C_{11}^{k} x \\frac{11-3 k}{2}$. 令 $\\frac{11-3 k}{2}=11$ 得 $k=3$.\n\n此时 ${ }^{T_{3+1}}=(-2)^{3} C_{11}^{3} x=-1320 x$, 所以展开式中 ${ }^{x}$ 的一次项的系数为 -1320 .", "solution": "null", "level": "高三", "question": "已知 $\\left(\\sqrt{x}-\\frac{2}{x}\\right)^{n}$ 的展开式中, 第 4 项和第 9 项的二项式系数相等,\n\n(1)求 ${ }^{n}$;\n\n(2)求展开式中 ${ }^{x}$ 的一次项的系数.", "options": [], "subject": "组合数学", "analysis": "答案:(1)由第 4 项和第 9 项的二项式系数相等可得 $C_{n}^{3}=C_{n}^{\\infty}$, 解得 $n=11$.\n\n(2)由(1)知, 展开式的第 $k+1$ 项为\n\n$T_{k+1}=C_{11}^{k}(\\sqrt{x})^{11-k}\\left(-\\frac{2}{x}\\right)^{k}=(-2)^{k} C_{11}^{k} x \\frac{11-3 k}{2}$. 令 $\\frac{11-3 k}{2}=11$ 得 $k=3$.\n\n此时 ${ }^{T_{3+1}}=(-2)^{3} C_{11}^{3} x=-1320 x$, 所以展开式中 ${ }^{x}$ 的一次项的系数为 -1320 ."} {"id": "22417", "image": [], "answer": "答案:\n\n$\\xi$ 的取值可能为 3,6,7,11,12,15,21,22,25,30.\n\n$\\xi=3$ 表示抽到的是 1 元和 2 元;\n\n$\\xi=6$ 表示抽到的是 1 元和 5 元;\n\n$\\xi=7$ 表示抽到的是 2 元和 5 元;\n\n$\\xi=11$ 表示抽到的是 1 元和 10 元;\n\n$\\xi=12$ 表示抽到的是 2 元和 10 元;\n\n$\\xi=15$ 表示抽到的是 5 元和 10 元;\n\n$\\xi=21$ 表示抽到的是 1 元和 20 元;\n\n$\\xi=22$ 表示抽到的是 2 元和 20 元;\n\n$\\xi=25$ 表示抽到的是 5 元和 20 元;\n\n$\\xi=30$ 表示抽到的是 10 元和 20 元.", "solution": "null", "level": "高三", "question": "小王的钱包里有 20 元、 10 元、 5 元、 2 元和 1 元的人民币各一张,他随机抽出两张.用 $\\xi$ 表示这两张人民币金额之和, 写出 ${ }^{\\xi}$ 的所有可能取值,并说明取值所表示的随机试验结果.", "options": [], "subject": "组合数学", "analysis": "答案:\n\n$\\xi$ 的取值可能为 3,6,7,11,12,15,21,22,25,30.\n\n$\\xi=3$ 表示抽到的是 1 元和 2 元;\n\n$\\xi=6$ 表示抽到的是 1 元和 5 元;\n\n$\\xi=7$ 表示抽到的是 2 元和 5 元;\n\n$\\xi=11$ 表示抽到的是 1 元和 10 元;\n\n$\\xi=12$ 表示抽到的是 2 元和 10 元;\n\n$\\xi=15$ 表示抽到的是 5 元和 10 元;\n\n$\\xi=21$ 表示抽到的是 1 元和 20 元;\n\n$\\xi=22$ 表示抽到的是 2 元和 20 元;\n\n$\\xi=25$ 表示抽到的是 5 元和 20 元;\n\n$\\xi=30$ 表示抽到的是 10 元和 20 元."} {"id": "22448", "image": [], "answer": ")(1)设“在 1 次游戏中摸出 $i$ 个白球”为事件 ${ }^{A}(i=0,1,2,3)$, 则 $P\\left(A_{3}\right)=\frac{C_{3}^{2}}{C_{5}^{2}} \\cdot \frac{C_{2}^{1}}{C_{3}^{2}}=\frac{1}{5}$(2) 设“在 1 次游戏中获奖”为事件 $B$, 则 $B=A_{2} \\cup A_{3}$, 又 $P\\left(A_{2}\right)=\frac{C_{3}^{2}}{C_{5}^{2}} \\cdot \frac{C_{2}^{2}}{C_{3}^{2}}+\frac{C_{3}^{1} C_{2}^{1}}{C_{5}^{2}} \\cdot \frac{C_{2}^{1}}{C_{3}^{2}}=\frac{1}{2}$,且 $A_{2}, A_{3}$ 互斥, 所以 $P(B)=P\\left(A_{2}\right)+P\\left(A_{3}\right)=\frac{1}{2}+\frac{1}{5}=\frac{7}{10}$.(2)由题意可知 $X$ 的所有可能取值为 $0,1,2$.$P(X=0)=C_{2}^{0}\\left(\frac{7}{10}\right)^{0}\\left(1-\frac{7}{10}\right)^{2}=\frac{9}{100}$,$P(X=1)=C_{2}^{1} \\cdot \frac{7}{10} \\cdot\\left(1-\frac{7}{10}\right)=\frac{21}{50}$,$P(X=2)=C_{2}^{2}\\left(\frac{7}{10}\right)^{2}\\left(1-\frac{7}{10}\right)^{0}=\frac{49}{100}$所以 $X$ 的分布列是| $X$ | 0 | 1 | 2 || :--- | :--- | :--- | :--- || $P$ | $\frac{9}{100}$ | $\frac{21}{50}$ | $\frac{49}{100}$ |", "solution": "null", "level": "高三", "question": "学校游园活动有这样一个游戏项目: 甲箱子里装有 3 个白球、 2 个黑球,乙箱子里装有 1 个白球、2 个黑球,这些球除颜色外完全相同.每次游戏从这两个箱子里各随机摸出 2 个球,若摸出的白球不少于 2 个,则获奖.(每次游戏结束后将球放回原箱)\n\n(1)求在 1 次游戏中,\n\n(1)摸出 3 个白球的概率;\n\n(2)获奖的概率;\n\n(2)求在 2 次游戏中获奖次数 $X$ 的分布列.", "options": [], "subject": "组合数学", "analysis": ""} {"id": "23040", "image": [], "answer": ": 证明: $\\because \\sqrt{n(n+1)}=\\sqrt{n^{2}+n}$,\n\n$\\therefore \\sqrt{n(n+1)}>n$,\n\n$\\therefore a_{n}=\\sqrt{1 \\times 2}+\\sqrt{2 \\times 3}+\\cdots+\\sqrt{n(n+1)}>1+2+3+\\cdots+n=\\frac{n(n+1)}{2}$\n\n$\\because \\sqrt{n(n+1)}<\\frac{\\mathrm{n+(n+1)}}{2}$,\n\n$\\therefore a_{n}<\\frac{1+2}{2}+\\frac{2+3}{2}+\\frac{3+4}{2}+\\cdots+\\frac{n+(n+1)}{2}$\n\n$=\\frac{1}{2}+(2+3+\\cdots+n)+\\frac{n+1}{2}=\\frac{n(n+2)}{2}$.\n\n综上得: $\\frac{\\mathrm{n}(\\mathrm{n}+1)}{2}n$, $\\sqrt{n(n+1)}<\\frac{n+(n+1)}{2}$解析放缩证明即可.", "solution": "null", "level": "高三", "question": "21. 已知: $a_{n}=\\sqrt{1 \\times 2}+\\sqrt{2 \\times 3}+\\sqrt{3 \\times 4}+\\cdots+\\sqrt{n(n+1)}\\left(n \\in \\mathbf{N}_{+}\\right)$, 求证: $\\frac{m(n+1)}{2}n$,\n\n$\\therefore a_{n}=\\sqrt{1 \\times 2}+\\sqrt{2 \\times 3}+\\cdots+\\sqrt{n(n+1)}>1+2+3+\\cdots+n=\\frac{n(n+1)}{2}$\n\n$\\because \\sqrt{n(n+1)}<\\frac{\\mathrm{n+(n+1)}}{2}$,\n\n$\\therefore a_{n}<\\frac{1+2}{2}+\\frac{2+3}{2}+\\frac{3+4}{2}+\\cdots+\\frac{n+(n+1)}{2}$\n\n$=\\frac{1}{2}+(2+3+\\cdots+n)+\\frac{n+1}{2}=\\frac{n(n+2)}{2}$.\n\n综上得: $\\frac{\\mathrm{n}(\\mathrm{n}+1)}{2}n$, $\\sqrt{n(n+1)}<\\frac{n+(n+1)}{2}$解析放缩证明即可."} {"id": "20825", "image": [], "answer": "洗好水踫,烧开水,在等待水开的时间内洗茶並、茶杯,取茶叶,水开后沏茶.此方案的流程图如图所示(共需 17 分钟).", "solution": "null", "level": "高三", "question": "想沏茶喝, 当时的情况是: 开水没有, 烧开水的壶要洗, 沏茶的壶和茶杯要洗, 茶叶已有.已知洗水壶用时 1 分钟, 取茶叶用时 1 分钟, 沏茶用时 1 分钟, 烧开水用时 15 分钟, 洗茶壶、茶杯用时 2 分钟.请设计一种节省时间的沏茶方案的流程图.", "options": [], "subject": "组合数学", "analysis": ""} {"id": "22500", "image": ["9812.jpg", "9813.jpg"], "answer": "答案: 解: 作法:如图, (1)作射线 $\\mathrm{AC}$;\n\n(2)在射线 $\\mathrm{AC}$ 上以任意取定的长度顺次截取 $\\mathrm{AD}_{1}=\\mathrm{D}_{1} \\mathrm{D}_{2}=\\mathrm{D}_{2} \\mathrm{D}_{3}=\\mathrm{D}_{3} \\mathrm{D}_{4}=\\mathrm{D}_{4} \\mathrm{D}_{5}$;\n\n(3)连接 $\\mathrm{D}_{5} \\mathrm{~B}$;\n\n(4) 分别过 $\\mathrm{D}_{1}, \\mathrm{D}_{2}, \\mathrm{D}_{3}, \\mathrm{D}_{4}$ 作 $\\mathrm{D}_{5} \\mathrm{~B}$ 的平行线 $\\mathrm{D}_{1} \\mathrm{~A}_{1}, \\mathrm{D}_{2} \\mathrm{~A}_{2}, \\mathrm{D}_{3} \\mathrm{~A}_{3}, \\mathrm{D}_{4} \\mathrm{~A}_{4}$, 分别交 $\\mathrm{AB}$ 于点 $\\mathrm{A}_{1}, \\mathrm{~A}_{2}, \\mathrm{~A}_{3}, \\mathrm{~A}_{4}$, 则点 $\\mathrm{A}_{1}, \\mathrm{~A}_{2}, \\mathrm{~A}_{3}, \\mathrm{~A}_{4}$ 将线段 $\\mathrm{AB}$ 五等分.\n\n\n\n证明: 过点 $\\mathrm{A}$ 作 $\\mathrm{MN} / / \\mathrm{D}_{5} \\mathrm{~B}$.\n\n则 $M N / / D_{4} A_{4} / / D_{3} A_{3} / / D_{2} A_{2} / / D_{1} A_{1} / / D_{5} B$.\n\n$\\because \\mathrm{AD}_{1}=\\mathrm{D}_{1} \\mathrm{D}_{2}=\\mathrm{D}_{2} \\mathrm{D}_{3}=\\mathrm{D}_{3} \\mathrm{D}_{4}=\\mathrm{D}_{4} \\mathrm{D}_{5}$.\n\n$\\therefore \\mathrm{AA}_{1}=\\mathrm{A}_{1} \\mathrm{~A}_{2}=\\mathrm{A}_{2} \\mathrm{~A}_{3}=\\mathrm{A}_{3} \\mathrm{~A}_{4}=\\mathrm{A}_{4} \\mathrm{~B}$.\n\n$\\therefore$ 点 $\\mathrm{A}_{1}, \\mathrm{~A}_{2}, \\mathrm{~A}_{3}, \\mathrm{~A}_{4}$ 就是所求的线段 $\\mathrm{AB}$ 的五等分点.", "solution": "null", "level": "高三", "question": "21. 如图,已知线段 $A B$,求作线段 $A B$ 的五等分点,并予以证明.\n\n", "options": [], "subject": "画法几何学", "analysis": ": 分析: 本题主要考查了平行线等分线段定理, 解决问题的关键是利用平行线等分线段定理来作图."} {"id": "20579", "image": ["9380.jpg", "9381.jpg"], "answer": "答案: 解: 算法如下:\n\n\n\n(1). 从 A 点出发作一条与原直线不重合的射线;\n\n(2). 任取射线上一点 $\\mathrm{C}$, 以 $\\mathrm{AC}$ 为单位长度, 在射线上依次作出点 $\\mathrm{E} 、 \\mathrm{~F} 、 \\mathrm{G} 、 \\mathrm{D}$, 使 $A D=5 A C$;\n\n(3).连接 $D B$, 并过点 $\\mathrm{C}$ 作 $B D$ 的平行线交 $\\mathrm{AB}$ 于 $\\mathrm{M}, \\mathrm{M}$ 就是要找的五等分点.流程图如下:\n\n", "solution": "null", "level": "高三", "question": "21. 设计一个尺规作图的算法来确定线段 $\\mathrm{AB}$ 的一个五等分点, 并画出流程图。\n\n(点拨: 确定线段 $\\mathrm{AB}$ 的五等分点, 是指在线段 $\\mathrm{AB}$ 上确定一点 $\\mathrm{M}$, 使得 $A M=\\frac{1}{5} A B$ )", "options": [], "subject": "画法几何学", "analysis": ": 分析: 本题主要考查了绘制简单实际问题的流程图, 解决问题的关键是要找线段 $\\mathrm{AB}$\n五等分点, 根据平行线分线段成比例定理, 利用基本尺规作图, 只需先从 $\\mathrm{A}$ 点出发作一条与原直线不重合的射线; 再在以点 $\\mathrm{A}$ 为端点的射线上, 任取一点 $\\mathrm{C}$, 以 $\\mathrm{AC}$ 为单位长度, 在射线上依次作出点 $\\mathrm{E} 、 \\mathrm{~F} 、 \\mathrm{G} 、 \\mathrm{D}$, 使 $A D=5 A C$; 然后; 连接 $D B$, 并过点 $\\mathrm{C}$ 作 $B D$ 的平行线交 $A B$ 于 $M$, 则 $M$ 就是要找的五等分点。所以用顺序结构解决。"} {"id": "22501", "image": ["9814.jpg", "9815.jpg"], "answer": "答案: 证明: 过点 $\\mathrm{O}$ 作 $\\mathrm{AB}$ 的垂线,垂足为 $\\mathrm{E}$,如图. $\\because \\mathrm{AC} \\perp \\mathrm{AB}, \\mathrm{DB} \\perp \\mathrm{AB}, \\therefore \\mathrm{OE} / / \\mathrm{AC} / / \\mathrm{DB}$.\n又 $\\because \\mathrm{O}$ 为 $\\mathrm{CD}$ 的中点, $\\therefore \\mathrm{E}$ 为 $\\mathrm{AB}$ 的中点.又 $\\because \\mathrm{OE} \\perp \\mathrm{AB}, \\therefore \\mathrm{OA}=\\mathrm{OB}$.\n\n", "solution": "null", "level": "高三", "question": "22. 如图, 已知 $\\mathrm{AC} \\perp \\mathrm{AB}, \\mathrm{DB} \\perp \\mathrm{AB}, \\mathrm{O}$ 是 $\\mathrm{CD}$ 的中点, 求证: $\\mathrm{OA}=\\mathrm{OB}$.\n\n", "options": [], "subject": "组合几何学", "analysis": ": 分析: 本题主要考查了平行线等分线段定理, 解决问题的关键是根据线段 $\\mathrm{OA}$ 和 $\\mathrm{OB}$有共同端点,所以证明点 $O$ 在 $A B$ 的垂直平分线上即可."} {"id": "22502", "image": ["9816.jpg", "9817.jpg"], "answer": "答案: 证明: 如图, 过点 $A$ 作 $B C$ 的平行线 $A G$, 交 $D C$ 于点 $G . \\because A B / / D C, \\therefore$ 四边形 $A B C G$ 是平行四边形. $\\therefore \\mathrm{AG}=\\mathrm{BC}, \\mathrm{AG} / / \\mathrm{BC}$. 又 $\\because \\mathrm{EF} / / \\mathrm{BC}, \\therefore \\mathrm{EF} / / \\mathrm{AG} . \\because \\mathrm{E}$ 为 $\\mathrm{AD}$ 的中点, $\\therefore \\mathrm{F}$ 是 $\\mathrm{DG}$ 的中点. $\\mathrm{EF}=\\frac{1}{2}$ AG. $\\therefore \\mathrm{EF}=\\frac{1}{2} \\mathrm{BC}$, 即 $\\mathrm{BC}=2 \\mathrm{EF}$.\n\n", "solution": "null", "level": "高三", "question": "23. 如图, 在梯形 $\\mathrm{ABCD}$ 中, $\\mathrm{AB} / / \\mathrm{DC}, \\mathrm{E}$ 为 $\\mathrm{AD}$ 的中点, $\\mathrm{EF} / / \\mathrm{BC}$, 求证: $\\mathrm{BC}=2 \\mathrm{EF}$.\n\n", "options": [], "subject": "组合几何学", "analysis": ": 分析: 本题主要考查了平行线等分线段定理, 解决问题的关键是根据 $\\mathrm{EF} / / \\mathrm{BC}$, 联系所证明的结果是 $\\mathrm{BC}=2 \\mathrm{EF}$,由此想到三角形中位线定理,过点 $\\mathrm{A}$ 作 $\\mathrm{BC}$ 的平行线即可证明."} {"id": "22503", "image": ["9818.jpg", "9819.jpg"], "answer": "答案: 证明: 如图,连接 $\\mathrm{AE}$ 交 $\\mathrm{DC}$ 于点 $\\mathrm{O} . \\because$ 四边形 $\\mathrm{ACED}$ 是平行四边形, $\\therefore \\mathrm{O}$ 是 $\\mathrm{AE}$ 的中点(平行四边形的对角线互相平分). $\\because$ 四边形 $\\mathrm{ABCD}$ 是梯形, $\\therefore \\mathrm{DC} / / \\mathrm{AB}$. 在 $\\triangle \\mathrm{EAB}$ 中, $\\mathrm{OF} / / \\mathrm{AB}, \\mathrm{O}$ 是 $\\mathrm{AE}$ 的中点, $\\therefore \\mathrm{F}$ 是 $\\mathrm{EB}$ 的中点. $\\therefore \\mathrm{EF}=\\mathrm{BF}$.\n\n", "solution": "null", "level": "高三", "question": "24. 如图,已知以梯形 $\\mathrm{ABCD}$ 的对角线 $\\mathrm{AC}$ 及腰 $\\mathrm{AD}$ 为邻边作 $\\mathrm{ACED,DC}$ 的延长线交 $\\mathrm{BE}$ 于 $\\mathrm{F}$.求证: $\\mathrm{EF}=\\mathrm{BF}$.\n\n", "options": [], "subject": "度量几何学", "analysis": ": 分析: 本题主要考查了平行线等分线段定理, 解决问题的关键是根据平行线等分线段定理结合所给条件以及构造辅助线怎么即可."} {"id": "22504", "image": ["9820.jpg", "9821.jpg"], "answer": "答案: 证明: 如图,连接 $B D$, 取 $B D$ 的中点 $G$, 连接 $G E, G F$. 在 $\\triangle A B D$ 中, $\\because G, F$ 分别是 $B D, A D$的中点 $: \\therefore \\mathrm{GF}=\\frac{1}{2} \\mathrm{AB}, \\mathrm{GF} / / \\mathrm{BM}$. 同理可证: $\\mathrm{GE}=\\frac{1}{2} \\mathrm{CD}, \\mathrm{GE} / / \\mathrm{CN} . \\because \\mathrm{AB}=\\mathrm{CD}, \\therefore \\mathrm{GF}=\\mathrm{GE} . \\therefore \\angle \\mathrm{GEF}=$ $\\angle \\mathrm{GFE}$.\n\n$\\because \\mathrm{GF} / / \\mathrm{BM}, \\therefore \\angle \\mathrm{GFE}=\\angle \\mathrm{AME} . \\because \\mathrm{GE} / / \\mathrm{CN}, \\therefore \\angle \\mathrm{GEF}=\\angle \\mathrm{CNE} . \\therefore \\angle \\mathrm{AME}=\\angle \\mathrm{CNE}$.\n\n", "solution": "null", "level": "高三", "question": "25. 如图, 在四边形 $\\mathrm{ABCD}$ 中, $\\mathrm{AB}=\\mathrm{CD}, \\mathrm{E}, \\mathrm{F}$ 分别是 $\\mathrm{BC}, \\mathrm{AD}$ 的中点, $\\mathrm{BA}, \\mathrm{CD}$ 的延长线分别与 $\\mathrm{EF}$的延长线交于点 $\\mathrm{M}, \\mathrm{N}$. 求证: $\\angle \\mathrm{AME}=\\angle \\mathrm{CNE}$.\n\n", "options": [], "subject": "度量几何学", "analysis": ": 分析: 本题主要考查了平行线等分线段定理, 解决问题的关键是根据所给条件结合平行线等分线段定理以及构造辅助线怎么即可."} {"id": "22767", "image": ["10068.jpg", "10068.jpg"], "answer": ": 证明: 如图,以 $B C$ 所在直线为 $x$ 轴, $B C$ 的垂直平分线为 $y$ 轴建立平面直角坐标系.\n\n\n\n设 $B(-a, 0), C(a, 0), A(0, h)$.\n\n则直线 $A C$ 的方程为 $y=-\\frac{\\hbar}{a} x+h$,\n\n即: $h x+a y-a h=0$.\n\n直线 $A B$ 的方程为 $y=\\frac{h}{a} x+h$,\n\n即: $h x-a y+a h=0$.\n\n由点到直线的距离公式: $|B D|=\\frac{|2 a \\hbar|}{\\sqrt{a^{2}+\\hbar^{2}}},|C E|=\\frac{|2 a \\hbar|}{\\sqrt{a^{2}+h^{2}}}$,\n\n$\\therefore|B D|=|C E|$, 即 $B D=C E$.\n\n解析:分析:本题考查坐标法在几何中的应用.解答本题可通过建立平面直角坐标系, 将几何证明问题转化为代数运算问题. (1)建立适当的直角坐标系, 将平面几何问题转化为解析几何问题, 即“形”转化为“数”, 再回到“形”中, 此为坐标法的基本思想, 务必熟练掌握. (2)建立坐标系时, 要充分利用图形的几何特征.例如, 中心对称图形, 可利用它的对称中心为坐标原点; 轴对称图形, 可利用它的对称轴为坐标轴; 题设中有直角, 可考虑以两直角边所在的直线为坐标轴等.", "solution": "null", "level": "高三", "question": "19. 已知 $\\triangle A B C$ 中, $A B=A C, B D . C E$ 分别为两腰上的高.求证: $B D=C E$.", "options": [], "subject": "度量几何学", "analysis": ": 证明: 如图,以 $B C$ 所在直线为 $x$ 轴, $B C$ 的垂直平分线为 $y$ 轴建立平面直角坐标系.\n\n\n\n设 $B(-a, 0), C(a, 0), A(0, h)$.\n\n则直线 $A C$ 的方程为 $y=-\\frac{\\hbar}{a} x+h$,\n\n即: $h x+a y-a h=0$.\n\n直线 $A B$ 的方程为 $y=\\frac{h}{a} x+h$,\n\n即: $h x-a y+a h=0$.\n\n由点到直线的距离公式: $|B D|=\\frac{|2 a \\hbar|}{\\sqrt{a^{2}+\\hbar^{2}}},|C E|=\\frac{|2 a \\hbar|}{\\sqrt{a^{2}+h^{2}}}$,\n\n$\\therefore|B D|=|C E|$, 即 $B D=C E$.\n\n解析:分析:本题考查坐标法在几何中的应用.解答本题可通过建立平面直角坐标系, 将几何证明问题转化为代数运算问题. (1)建立适当的直角坐标系, 将平面几何问题转化为解析几何问题, 即“形”转化为“数”, 再回到“形”中, 此为坐标法的基本思想, 务必熟练掌握. (2)建立坐标系时, 要充分利用图形的几何特征.例如, 中心对称图形, 可利用它的对称中心为坐标原点; 轴对称图形, 可利用它的对称轴为坐标轴; 题设中有直角, 可考虑以两直角边所在的直线为坐标轴等."} {"id": "22519", "image": ["9838.jpg"], "answer": "答案:证明: $\\because \\mathrm{AB} \\perp \\mathrm{BD}, \\mathrm{CD} \\perp \\mathrm{BD}, \\mathrm{EF} \\perp \\mathrm{BD}$,\n\n$\\therefore \\mathrm{AB} / / \\mathrm{EF} / / \\mathrm{CD}, \\therefore \\frac{\\mathrm{EF}}{\\mathrm{AB}}=\\frac{\\mathrm{DF}}{\\mathrm{BD}}, \\frac{\\mathrm{EF}}{\\mathrm{CD}}=\\frac{\\mathrm{BF}}{\\mathrm{BD}}$,\n\n$\\therefore \\frac{\\mathrm{EF}}{\\mathrm{AB}}+\\frac{\\mathrm{EF}}{\\mathrm{CD}}=\\frac{\\mathrm{DF}}{\\mathrm{BD}}+\\frac{\\mathrm{BF}}{\\mathrm{BD}}=\\frac{\\mathrm{DF}+\\mathrm{BF}}{\\mathrm{BD}}=\\frac{\\mathrm{BD}}{\\mathrm{BD}}=1$,\n\n$\\therefore \\frac{1}{\\mathrm{AB}}+\\frac{1}{\\mathrm{CD}}=\\frac{1}{\\mathrm{EF}}$.", "solution": "null", "level": "高三", "question": "18. 如图, $A B \\perp B D$ 于点 $\\mathrm{B}, \\mathrm{CD} \\perp \\mathrm{BD}$ 于点 $\\mathrm{D}$, 连接 $\\mathrm{AD}, \\mathrm{BC}$ 交于点 $\\mathrm{E}, \\mathrm{EF} \\perp \\mathrm{BD}$ 于点 $\\mathrm{F}$, 求\n\n证: $\\frac{1}{\\mathrm{AB}}+\\frac{1}{\\mathrm{CD}}=\\frac{1}{\\mathrm{EF}}$.\n\n", "options": [], "subject": "度量几何学", "analysis": ": 分析: 本题主要考查了平行线分线段成比例定理, 解决问题的关键是转化为证明 $\\frac{\\mathrm{EF}}{\\mathrm{AB}}+\\frac{\\mathrm{EF}}{\\mathrm{CD}}=1$.由于 $\\mathrm{AB} / / \\mathrm{EF} / / \\mathrm{CD}$, 因此将 $\\frac{\\mathrm{EF}}{\\mathrm{AB}}$ 与 $\\frac{\\mathrm{EF}}{\\mathrm{CD}}$ 化归为同一直线 $\\mathrm{BD}$ 上的线段比就可得证"} {"id": "22524", "image": ["9843.jpg"], "answer": "答案: 证明: 因为 $\\mathrm{EF} / / \\mathrm{AB}$, 所以 $\\frac{\\mathrm{EF}}{\\mathrm{AB}}=\\frac{\\mathrm{DE}}{\\mathrm{DA}}$.\n\n因为 $\\mathrm{GH} / / \\mathrm{AB}$, 所以 $\\frac{\\mathrm{GH}}{\\mathrm{AB}}=\\frac{\\mathrm{CH}}{\\mathrm{CB}}$.\n\n因为 $\\mathrm{DC} / / \\mathrm{EH} / / \\mathrm{AB}$, 所以 $\\frac{\\mathrm{DE}}{\\mathrm{DA}}=\\frac{\\mathrm{CH}}{\\mathrm{CB}}$.\n\n所以 $\\frac{\\mathrm{EF}}{\\mathrm{AB}}=\\frac{\\mathrm{GH}}{\\mathrm{AB}}$, 即 $\\mathrm{EF}=\\mathrm{GH}$.", "solution": "null", "level": "高三", "question": "22. 如图,在梯形 $A B C D$ 中, $A B / / D C$,一条直线平行于两底,且顺次交 $A D, B D, A C, B C$ 于点 E,F,G,H.\n\n求证: $\\mathrm{EF}=\\mathrm{GH}$.\n\n", "options": [], "subject": "度量几何学", "analysis": ": 分析: 本题主要考查了平行线分线段成比例定理, 解决问题的关键是转化为证明\n\n$$\n\\frac{\\mathrm{EF}}{\\mathrm{AB}}=\\frac{\\mathrm{GH}}{\\mathrm{AB}}\n$$"} {"id": "22526", "image": ["9846.jpg"], "answer": "答案: 证明: $\\because \\mathrm{DE} / / \\mathrm{BC}, \\therefore \\frac{\\mathrm{PD}}{\\mathrm{PB}}=\\frac{\\mathrm{PE}}{\\mathrm{PC}}$\n\n$\\therefore \\mathrm{PD} \\cdot \\mathrm{PC}=\\mathrm{PE} \\cdot \\mathrm{PB}$.\n\n$\\because \\mathrm{DF} / / \\mathrm{AC}, \\therefore \\frac{\\mathrm{PF}}{\\mathrm{PC}}=\\frac{\\mathrm{PD}}{\\mathrm{PA}}$\n\n$\\therefore \\mathrm{PD} \\cdot \\mathrm{PC}=\\mathrm{PF} \\cdot \\mathrm{PA}$.\n\n$\\therefore \\mathrm{PE} \\cdot \\mathrm{PB}=\\mathrm{PF} \\cdot \\mathrm{PA} . \\therefore \\frac{\\mathrm{PE}}{\\mathrm{PF}}=\\frac{\\mathrm{PA}}{\\mathrm{PB}}$\n\n$\\therefore \\mathrm{BF} / / \\mathrm{AE}$.", "solution": "null", "level": "高三", "question": "24. 如图, 在 $\\triangle \\mathrm{ABC}$ 中, $\\mathrm{D}$ 是 $\\mathrm{AB}$ 上一点, $\\mathrm{E}$ 是 $\\triangle \\mathrm{ABC}$ 内一点, $\\mathrm{DE} / / \\mathrm{BC}$, 过点 $\\mathrm{D}$ 作 $\\mathrm{AC}$ 的平行线交 $\\mathrm{CE}$ 的延长线于点 $\\mathrm{F}, \\mathrm{CF}$ 与 $\\mathrm{AB}$ 交于点 $\\mathrm{P}$, 求证: $\\mathrm{BF} / / \\mathrm{AE}$.\n\n", "options": [], "subject": "度量几何学", "analysis": ": 分析: 本题主要考查了平行线分线段成比例定理, 解决问题的关键是根据所给条件结合平行线分线段成比例定理分析计算即可证明问题"} {"id": "22543", "image": ["9863.jpg"], "answer": "答案: 解: $\\because \\mathrm{AC} \\perp \\mathrm{CB}, \\mathrm{CD} \\perp \\mathrm{AB}, \\therefore \\mathrm{AC}^{2}=\\mathrm{AD} \\cdot \\mathrm{AB}, \\mathrm{CD}^{2}=\\mathrm{AD} \\cdot \\mathrm{DB}$.\n\n$\\therefore \\mathrm{AD}=\\frac{\\mathrm{AC}^{2}}{\\mathrm{AB}}=\\frac{20^{2}}{25}=16$.\n$\\therefore \\mathrm{DB}=\\mathrm{AB}-\\mathrm{AD}=25-16=9$.\n\n$\\therefore \\mathrm{CD}=\\sqrt{\\mathrm{AD} \\cdot \\mathrm{DB}}=\\sqrt{16 \\times 9}=12$.", "solution": "null", "level": "高三", "question": "18. 若 $\\mathrm{CD}$ 是 Rt $\\triangle \\mathrm{ACB}$ 斜边 $\\mathrm{AB}$ 上的高, $\\mathrm{AB}=25, \\mathrm{AC}=20$, 试确定 $\\mathrm{DB}$ 和 $\\mathrm{CD}$ 的长.\n\n", "options": [], "subject": "度量几何学", "analysis": ": 分析: 本题主要考查了直角三角形的射影定理, 解决问题的关键是根据直角三角形的射影定理先用射影定理求出 $\\mathrm{AD}$, 从而求出 $\\mathrm{DB}$,再用射影定理求出 $\\mathrm{CD}$."} {"id": "22544", "image": ["9864.jpg"], "answer": "答案: 解: $\\because \\mathrm{AC} \\perp \\mathrm{CB}, \\mathrm{CD} \\perp \\mathrm{AB}$,\n\n$\\therefore \\mathrm{CD}^{2}=\\mathrm{AD} \\cdot \\mathrm{DB}=2 \\times 6=12$,\n\n$\\therefore \\mathrm{CD}=\\sqrt{12}=2 \\sqrt{3}(\\mathrm{~cm})$\n\n$\\because \\mathrm{AC}^{2}=\\mathrm{AD} \\cdot \\mathrm{AB}=2 \\times(2+6)=16$,\n\n$\\therefore \\mathrm{AC}=\\sqrt{16}=4(\\mathrm{~cm})$.\n\n$\\because \\mathrm{BC}^{2}=\\mathrm{BD} \\cdot \\mathrm{AB}=6 \\times(2+6)=48$,\n\n$\\therefore \\mathrm{BC}=\\sqrt{48}=4 \\sqrt{3}(\\mathrm{~cm})$.\n\n所以 $\\mathrm{CD}, \\mathrm{AC}, \\mathrm{BC}$ 的长分别为 $2 \\sqrt{3} \\mathrm{~cm}, 4 \\mathrm{~cm}, 4 \\sqrt{3} \\mathrm{~cm}$.", "solution": "null", "level": "高三", "question": "19. 如图, 在 Rt $\\triangle A B C$ 中, $C D$ 为斜边 $A B$ 上的高. 若 $A D=2 \\mathrm{~cm}, D B=6 \\mathrm{~cm}$, 求 $C D, A C, B C$ 的长\n\n", "options": [], "subject": "度量几何学", "analysis": ": 分析: 本题主要考查了直角三角形的射影定理, 解决问题的关键是根据直角三角形的射影定理结合所给直角三角形满足条件计算即可"} {"id": "22546", "image": ["9866.jpg"], "answer": "答案: 证明: $C=90^{\\circ}, A D \\perp B C$, 由射影定理,知 $A C^{2}=C D \\cdot B C$, 即 $\\frac{A C}{C D}=\\frac{B C}{A C}$.\n\n$\\because \\mathrm{BE}$ 平分 $\\angle \\mathrm{ABC}, \\mathrm{EA} \\perp \\mathrm{AB}, \\mathrm{EF} \\perp \\mathrm{BC}$,\n\n$\\therefore \\mathrm{AE}=\\mathrm{EF} . \\because \\mathrm{EF} \\perp \\mathrm{BC}, \\mathrm{AD} \\perp \\mathrm{BC}$,\n\n$\\therefore \\mathrm{EF} / / \\mathrm{AD} . \\therefore \\frac{\\mathrm{AE}}{\\mathrm{DF}}=\\frac{\\mathrm{AC}}{\\mathrm{DC}}$,\n\n$\\therefore \\frac{\\mathrm{EF}}{\\mathrm{DF}}=\\frac{\\mathrm{AC}}{\\mathrm{DC}} . \\therefore \\frac{\\mathrm{EF}}{\\mathrm{DF}}=\\frac{\\mathrm{BC}}{\\mathrm{AC}}$,\n\n即 $\\mathrm{EF}: \\mathrm{DF}=\\mathrm{BC}: \\mathrm{AC}$.", "solution": "null", "level": "高三", "question": "20. 如图, 在 Rt $\\triangle \\mathrm{ABC}$ 中, $\\angle \\mathrm{BAC}=90^{\\circ}, \\mathrm{AD} \\perp \\mathrm{BC}$ 于点 $\\mathrm{D}, \\mathrm{BE}$ 平分 $\\angle \\mathrm{ABC}$ 交 $\\mathrm{AC}$ 于点 $\\mathrm{E}, \\mathrm{EF} \\perp \\mathrm{BC}$于点 $\\mathrm{F}$. 求证: $\\mathrm{EF}: \\mathrm{DF}=\\mathrm{BC}: \\mathrm{AC}$.\n\n", "options": [], "subject": "度量几何学", "analysis": ": 分析: 本题主要考查了直角三角形的射影定理, 解决问题的关键是根据直角三角形的\n射影定理先由射影定理得 $A C^{2}=C D \\cdot B C$, 即 $\\frac{A C}{C D}=\\frac{B C}{A C}$. 又由 $E F / / A D$, 得 $\\frac{A E}{D F}=\\frac{A C}{D C}$, 通过中间变量即可得证."} {"id": "22547", "image": ["9867.jpg"], "answer": "答案:证明:由射影定理,得 $\\mathrm{BD}^{2}=\\mathrm{BE} \\cdot \\mathrm{AB}$, 即 $\\mathrm{BE}=\\frac{\\mathrm{BD}^{2}}{\\mathrm{AB}}$\n\n同理得 $\\mathrm{CD}^{2}=\\mathrm{CF} \\cdot \\mathrm{AC}$, 即 $\\mathrm{CF}=\\frac{\\mathrm{CD}^{2}}{\\mathrm{AC}}$\n\n故 $\\frac{\\mathrm{BE}}{\\mathrm{CF}}=\\frac{\\mathrm{BD}^{2}}{\\mathrm{AB}} \\cdot \\frac{\\mathrm{AC}}{\\mathrm{CD}^{2}}=\\left(\\frac{\\mathrm{BD}}{\\mathrm{CD}}\\right)^{2} \\cdot \\frac{\\mathrm{AC}}{\\mathrm{AB}}$\n\n由射影定理,得 $A B^{2}=B C \\cdot B D$, 即 $B D=\\frac{A B^{2}}{B C}$\n\n同理得 $\\mathrm{AC}^{2}=\\mathrm{CD} \\cdot \\mathrm{BC}$, 即 $\\mathrm{CD}=\\frac{\\mathrm{AC}}{\\mathrm{BC}}$. 故 $\\frac{\\mathrm{BD}}{\\mathrm{CD}}=\\frac{\\mathrm{AB}^{2}}{\\mathrm{AC}^{2}}$\n\n将(4)代入(3), 得 $\\frac{\\mathrm{AB}^{3}}{\\mathrm{AC}^{3}}=\\frac{\\mathrm{BE}}{\\mathrm{CF}}$.", "solution": "null", "level": "高三", "question": "21. 如图, 已知 $\\angle \\mathrm{BAC}=90^{\\circ}, \\mathrm{AD} \\perp \\mathrm{BC}, \\mathrm{DE} \\perp \\mathrm{AB}, \\mathrm{DF} \\perp \\mathrm{AC}$, 垂足分别为 $\\mathrm{D}, \\mathrm{E}, \\mathrm{F}$.求证: $\\frac{\\mathrm{AB}^{3}}{\\mathrm{AC^{3 }}}=\\frac{\\mathrm{BE}}{\\mathrm{CF}}$\n\n", "options": [], "subject": "度量几何学", "analysis": ": 分析: 本题主要考查了直角三角形的射影定理, 解决问题的关键是根据直角三角形的射影定理结合所给三角形满足条件计算证明即可"} {"id": "22548", "image": ["9868.jpg"], "answer": "答案:证明: $\\because \\mathrm{DP} \\perp \\mathrm{AB}, \\mathrm{DQ} \\perp \\mathrm{AC}, \\mathrm{AD} \\perp \\mathrm{BC}$,\n\n$\\therefore$ 在 Rt $\\triangle \\mathrm{ADB}$ 中, 有 $\\mathrm{AD}^{2}=\\mathrm{AP} \\cdot \\mathrm{AB}$.\n\n在 Rt $\\triangle \\mathrm{ADC}$ 中, 有 $\\mathrm{AD}^{2}=\\mathrm{AQ} \\cdot \\mathrm{AC}$.\n\n$\\therefore \\mathrm{AP} \\cdot \\mathrm{AB}=\\mathrm{AQ} \\cdot \\mathrm{AC}$.", "solution": "null", "level": "高三", "question": "22. 如图, 已知 $A D$ 是 $\\triangle A B C$ 的高, $D P \\perp A B, D Q \\perp A C$, 垂足分别为点 $P$, .求证: $A P \\cdot A B=A Q \\cdot A C$.\n\n", "options": [], "subject": "度量几何学", "analysis": ": 分析: 本题主要考查了直角三角形的射影定理, 解决问题的关键是根据直角三角形的射影定理应用射影定理转化为证明\n\n$\\mathrm{AP} \\cdot \\mathrm{AB}=\\mathrm{AD}^{2}, \\mathrm{AQ} \\cdot \\mathrm{AC}=\\mathrm{AD}^{2}$."} {"id": "22549", "image": ["9869.jpg"], "answer": "答案:证明: $\\because$ 在 Rt $\\triangle \\mathrm{ABC}$ 中, $\\angle \\mathrm{ACB}=90^{\\circ}, \\mathrm{CD} \\perp \\mathrm{AB}$,\n\n$\\therefore \\mathrm{CD}^{2}=\\mathrm{AD} \\cdot \\mathrm{BD}, \\therefore \\mathrm{CD}^{4}=\\mathrm{AD}^{2} \\cdot \\mathrm{BD}^{2}$.\n\n又 $\\because$ 在 Rt $\\triangle \\mathrm{ADC}$ 中, $\\mathrm{DE} \\perp \\mathrm{AC}$, 在 Rt $\\triangle \\mathrm{BDC}$ 中, $\\mathrm{DF} \\perp \\mathrm{BC}$,\n\n$\\therefore \\mathrm{AD}^{2}=\\mathrm{AE} \\cdot \\mathrm{AC}, \\mathrm{BD}^{2}=\\mathrm{BF} \\cdot \\mathrm{BC}$,\n\n$\\therefore \\mathrm{CD}^{4}=\\mathrm{AE} \\cdot \\mathrm{BF} \\cdot \\mathrm{AC} \\cdot \\mathrm{BC}$.\n\n又 $\\because \\angle \\mathrm{A}=\\angle \\mathrm{A}, \\angle \\mathrm{ACD}=\\angle \\mathrm{ABC}$,\n\n$\\therefore \\triangle \\mathrm{ABC} \\sim \\triangle \\mathrm{ACD}$.\n\n$\\therefore \\frac{\\mathrm{BC}}{\\mathrm{CD}}=\\frac{\\mathrm{AB}}{\\mathrm{AC}}$,\n\n即 $A C \\cdot B C=A B \\cdot C D$,\n\n$\\therefore \\mathrm{CD}^{4}=\\mathrm{AE} \\cdot \\mathrm{BF} \\cdot \\mathrm{AB} \\cdot \\mathrm{CD}$.\n\n$\\therefore \\mathrm{AE} \\cdot \\mathrm{BF} \\cdot \\mathrm{AB}=\\mathrm{CD}^{3}$.", "solution": "null", "level": "高三", "question": "23. 如图, 已知 Rt $\\triangle \\mathrm{ABC}$ 中, $\\angle \\mathrm{ACB}=90^{\\circ}, \\mathrm{CD} \\perp \\mathrm{AB}$ 于点 $\\mathrm{D}, \\mathrm{DE} \\perp \\mathrm{AC}$ 于点 $\\mathrm{E}, \\mathrm{DF} \\perp \\mathrm{BC}$ 于点 $\\mathrm{F}$. 求证: $\\mathrm{AE} \\cdot \\mathrm{BF} \\cdot \\mathrm{AB}=\\mathrm{CD}^{3}$.\n\n", "options": [], "subject": "度量几何学", "analysis": ": 分析: 本题主要考查了直角三角形的射影定理, 解决问题的关键是根据直角三角形的射影定理分别在 Rt $\\triangle \\mathrm{ABC}, \\mathrm{Rt} \\triangle \\mathrm{ADC}, \\mathrm{Rt} \\triangle \\mathrm{BDC}$ 中运用射影定理, 再将线段进行代换, 就可以实现等积式的证明."} {"id": "22551", "image": ["9871.jpg", "9872.jpg"], "answer": "答案: 解答: 在 $\\triangle \\mathrm{ABC}$ 中, 设 $\\mathrm{AC}=\\mathrm{x}, \\because \\mathrm{AB} \\perp \\mathrm{AC}, \\mathrm{AF} \\perp \\mathrm{BC}$,\n\n又 $\\mathrm{FC}=1$, 根据射影定理, 得 $A C^{2}=F C \\cdot B C$, 即 $B C=x^{2}$.\n\n再由射影定理,得 $A F^{2}=B F \\cdot F C=(B C-F C) \\cdot F C$, 即 $\\mathrm{AF}^{2}=x^{2}-1$.\n\n$\\therefore \\mathrm{AF}=\\sqrt{\\mathrm{x}^{2}-1}$.\n\n在 $\\triangle \\mathrm{BDC}$ 中,过点 $\\mathrm{D}$ 作 $\\mathrm{DE} \\perp \\mathrm{BC}$ 于点 $\\mathrm{E}$,如图,\n\n\n\n又 $\\because \\mathrm{BD}=\\mathrm{DC}=1$,\n\n$\\therefore \\mathrm{BE}=\\mathrm{EC}$,\n\n又 $\\because \\mathrm{AF} \\perp \\mathrm{BC}$,\n\n$\\therefore \\mathrm{DE} / / \\mathrm{AF}$\n\n$\\therefore \\frac{\\mathrm{DE}}{\\mathrm{AF}}=\\frac{\\mathrm{DC}}{\\mathrm{AC}}$\n\n$\\therefore \\mathrm{DE}=\\frac{\\mathrm{DC} \\cdot \\mathrm{AF}}{\\mathrm{AC}}=\\frac{\\sqrt{\\mathrm{x}^{2}-1}}{\\mathrm{x}}$.\n\n在 Rt $\\triangle \\mathrm{DEC}$ 中, $\\because \\mathrm{DE}^{2}+\\mathrm{EC}^{2}=\\mathrm{DC}^{2}$,\n\n即 $\\left(\\frac{\\sqrt{x^{2}-1}}{x}\\right)^{2}+\\left(\\frac{x^{2}}{2}\\right)^{2}=1^{2}$,\n$\\therefore \\frac{\\mathrm{x}^{2}-1}{\\mathrm{x}^{2}}+\\frac{\\mathrm{x}^{4}}{4}=1$.\n\n整理得 $\\mathrm{x}^{6}=4 . \\therefore \\mathrm{x}=\\sqrt[3]{2} \\therefore \\mathrm{AC}=\\sqrt[3]{2}$.", "solution": "null", "level": "高三", "question": "25. 如图, 在 $\\triangle A B C$ 中, $D, F$ 两点分别在 $A C, B C$ 上, 且 $A B \\perp A C, A F \\perp B C, B D=D C=F C=1$, 求 $A C$的长\n\n", "options": [], "subject": "度量几何学", "analysis": ": 分析: 本题主要考查了直角三角形的射影定理, 解决问题的关键是根据直角三角形的射影定理结合所给直角三角形满足的条件构造辅助线计算即可, 属于较难题目"} {"id": "22764", "image": [], "answer": "答案: 解: 由伸缩变换 $\\left\\{\\begin{array}{l}x^{s}=\\frac{1}{3} x_{x} \\\\ y^{\\prime}=\\frac{1}{2} y_{x}\\end{array}\\right.$ 知 $\\left\\{\\begin{array}{l}x=3 x^{s}, \\\\ y=2 y^{\\prime}\\end{array}\\right.$.\n\n将 $\\left\\{\\begin{array}{l}x=3 x^{3} \\\\ y=2 y^{\\prime}\\end{array}\\right.$, 代入 $y^{2}=2 x$, 可得 $4 y^{\\prime 2}=6 x^{f}$, 即 $y^{\\prime 2}=\\frac{3}{2} x^{f}$.\n\n即伸缩变换之后的图形还是抛物线.\n\n(2) $x^{2}+y^{2}=1$\n\n答案: 解: 将 $\\left\\{\\begin{array}{l}x=3 x^{\\prime} \\\\ y=2 y^{\\prime}\\end{array}\\right.$, 代入 $x^{2}+y^{2}=1$, 得 $\\left(3 x^{\\prime}\\right)^{2}+\\left(2 y^{\\prime}\\right)^{2}=1$,\n\n即 $\\frac{x^{\\prime 2}}{\\frac{1}{6}}+\\frac{y^{\\prime 2}}{\\frac{1}{4}}=1$,\n\n即伸缩变换之后的图形为焦点在 $y$ 轴上的椭圆.", "solution": "null", "level": "高三", "question": "16. 在平面直角坐标系中, 求下列方程所对应的图形经过伸缩变换, $\\left\\{\\begin{array}{l}x^{\\prime}=\\frac{1}{8} x \\\\ y^{\\prime}=\\frac{1}{2} y\\end{array}\\right.$ 后的图形是什么形状?\n\n(1) $y^{2}=2 x$", "options": [], "subject": "变换几何", "analysis": ": 分析: 本题考查伸缩变换的应用,解答此题需要先根据伸缩变换求出变换后的方程,然后再判断图形的形状."} {"id": "22770", "image": [], "answer": ": 答案: 解: 由伸缩变换 $\\left\\{\\begin{array}{c}x^{f}=\\frac{1}{2} x, \\\\ y^{\\prime}=\\frac{1}{3} y\\end{array}\\right.$ 得 $\\left\\{\\begin{array}{l}x=2 x^{s}, \\\\ y=3 y^{\\prime},\\end{array}\\right.$\n\n将其代入 $5 x+2 y=0$, 得到经过伸缩变换后的图形的方程是 $5 x^{f}+3 y^{f}=0$.\n\n所以经过伸缩变换 $\\left\\{\\begin{array}{c}x^{\\prime}=\\frac{1}{2} x, \\\\ y^{\\prime}=\\frac{1}{3} y\\end{array}\\right.$ 后, 直线 $5 x+2 y=0$ 成直线 $5 x^{f}+3 y^{\\prime}=0$.\n\n(2) $x^{2}+y^{2}=2$.\n\n答案: 解: 将 $\\left\\{\\begin{array}{c}x=2 x^{5} \\\\ y=3 y^{y}\\end{array}\\right.$, 代入 $x^{2}+y^{2}=2$, 得到经过伸缩变换后的图形的方程是 $\\frac{x^{x^{2}}}{\\frac{1}{4}}+\\frac{y^{y^{2}}}{\\frac{1}{6}}=2$, 即 $\\frac{x^{x^{2}}}{\\frac{1}{2}}+\\frac{y^{y^{2}}}{\\frac{2}{6}}=1$.\n\n所以经过伸缩变换 $\\left\\{\\begin{array}{c}x^{\\prime}=\\frac{1}{2} x, \\\\ y^{\\prime}=\\frac{1}{3} y\\end{array}\\right.$ 后, 圆 $x^{2}+y^{2}=2$ 变成粗圆 $\\frac{x^{r^{2}}}{\\frac{1}{2}}+\\frac{y^{\\prime^{2}}}{\\frac{2}{2}}=1$.", "solution": "null", "level": "高三", "question": "22. 在平面直角坐标系中, 求下列方程对应的图形经过伸缩变换 $\\left\\{\\begin{array}{c}x^{\\prime}=\\frac{1}{2} x, \\\\ y^{\\prime}=\\frac{1}{3} y\\end{array}\\right.$ 后的图形.\n\n(1) $5 x+2 y=0$.", "options": [], "subject": "变换几何", "analysis": ": 分析: 本题主要考查了平面直角坐标轴中的伸缩变换, 解决问题的关键是根据平面直角坐标轴中的伸缩变换原理计算即可"} {"id": "22994", "image": [], "answer": "答案: 解: 当 $\\mathrm{a}=2$ 时, 不等式为 $|x-2|+|x-1| \\geq 4$,不等式的解集为 $\\left(-\\infty,-\\frac{1}{2}\\right] \\cup\\left[\\frac{7}{2},+\\infty\\right)$;\n\n(2) 若 $f(x) \\leq 1$ 的解集为 $[0,2], \\frac{1}{m}+\\frac{1}{2 n}=a(m>0, n>0)$, 求证: $m+2 n \\geq 4$\n\n答案: 证明: $f(x) \\leq 1$ 即 $|x-a| \\leq 1$, 解得 $a-1 \\leq x \\leq a+1$, 而 $f(x) \\leq 1$ 解集是 $[0,2]$,\n\n$\\therefore\\left\\{\\begin{array}{l}a-1=0 \\\\ a+1=2\\end{array}\\right.$, 解得 $a=1$, 所以 $\\frac{1}{m}+\\frac{1}{2 n}=1(m>0, n>0)$\n\n所以 $m+2 n=(m+2 n)\\left(\\frac{1}{m}+\\frac{1}{2 n}\\right) \\geq 4$.", "solution": "null", "level": "高三", "question": "22. 设函数 $f(x)=|x-a|$.\n\n(1) 当 $a=2$ 时, 解不等式 $f(x) \\geq 4-|x-1|$;", "options": [], "subject": "算术", "analysis": ": 分析: 本题主要考查了绝对值不等式的解法, 解决问题的关键是 (1) 用零点分段法去掉绝对值符号, 转化为不等式组, 解不等式; (2) 先解不等式 $|x-a| \\leq 1$, 再结合 $f(x) \\leq 1$\n的解集为 $[0,2]$, 从而得到 $\\mathrm{a}$ 的值, 再利用特殊值 1 将 $m+2 n$ 转化为 $(m+2 n)\\left(\\frac{1}{m}+\\frac{1}{2 n}\\right)=1+\\frac{m}{2 n}+\\frac{2 n}{m}+1$, 再利用基本不等式求函数 $m+2 n$ 的取值范围."} {"id": "22997", "image": [], "answer": "答案: 解: 不等式 $f(x)<4-|x-1|$, 即 $|3 x+2|+|x-1|<4$,当 $x<-\\frac{2}{3}$ 时, 即 $-3 x-2-x+1<4$, 解得 $-\\frac{5}{4}1$ 时, 即 $3 x+2+x-1<4$, 无解,\n\n综上所述 $x \\in\\left(-\\frac{5}{4}, \\frac{1}{2}\\right)$.\n\n(2) 已知 $\\mathrm{m}+\\mathrm{n}=1(\\mathrm{~m}, \\mathrm{n}>0)$, 若 $|x-a|-f(x) \\leq \\frac{1}{m}+\\frac{1}{n}(a>0)$ 恒成立, 求实数 $\\mathrm{a}$ 的取值范围.\n\n答案: 解: $\\frac{1}{m}+\\frac{1}{n}=\\left(\\frac{1}{m}+\\frac{1}{n}\\right)(m+n)=1+1+\\frac{n}{m}+\\frac{m}{n} \\geq 4$,令 $g(x)=|x-a|-f(x)=|x-a|-|3 x+2|=\\left\\{\\begin{array}{l}2 x+2+a, x<-\\frac{2}{3}, \\\\ -4 x+2+a,-\\frac{2}{3} \\leq x \\leq a, \\\\ -2 x-2-a, x>a\\end{array}\\right.$ $\\therefore x=-\\frac{2}{3}$ 时, $g(x)_{\\text {max }}=\\frac{2}{3}+a$, 要使不等式恒成立,只需 $g(x)_{\\text {max }}=\\frac{2}{3}+a \\leq 4$ 即 $0\\sqrt{n^{2}+2 n}$\n\n需证 $(n+1)^{2}>n^{2}+2 n$\n\n需证 $n^{2}+2 n+1>n^{2}+2 n$\n\n只需证 $1>0$\n\n因为 $1>0$ 显然成立,所以原命题成立.", "solution": "null", "level": "高三", "question": "16.(1)用反证法证明: 在一个三角形中, 至少有一个内角大于或等于 $60^{\\circ}$;\n\n(2)已知 $n \\geq 0$, 试用分析法证明: $\\sqrt{n+2}-\\sqrt{n+1}<\\sqrt{n+1}-\\sqrt{n}$", "options": [], "subject": "算术", "analysis": ":【分析】本题考查反证法与分析法的应用,解题时需要注意以下关键要点:(1)反证法证明问题的关键是: 提出结论的反面, 并以此为条件推导导出矛盾; (2)分析法要求由结论成立反推条件(由果索因)."} {"id": "23041", "image": [], "answer": "答案: 证明: $\\because \\frac{1}{k^{2}}<\\frac{1}{k(k-1)}=\\frac{1}{k-1}-\\frac{1}{k}, k=2,3,4, \\cdots, n$,\n\n$\\therefore \\frac{1}{1^{2}}+\\frac{1}{2^{2}}+\\frac{1}{a^{2}}+\\cdots+\\frac{1}{n^{2}}$\n\n$<\\frac{1}{1}+\\frac{1}{1 \\times 2}+\\frac{1}{2 \\times 3}+\\cdots+\\frac{1}{(n-1) \\cdot n}$\n$=\\frac{1}{1}+\\left(\\frac{1}{1}-\\frac{1}{2}\\right)+\\left(\\frac{1}{2}-\\frac{1}{3}\\right)+\\cdots+\\left(\\frac{1}{n-1}-\\frac{1}{n}\\right)$\n\n$=2-\\frac{1}{n}<2$.", "solution": "null", "level": "高三", "question": "22. 若 $n$ 是大于 1 的自然数, 求证: $\\frac{1}{1^{2}}+\\frac{1}{2^{2}}+\\frac{1}{3^{2}}+\\cdots+\\frac{1}{n^{2}}<2$.", "options": [], "subject": "算术", "analysis": ":【分析】本题主要考查了反证法与放缩法, 解决问题的关键是(1)放缩法证明不等式主要是根据不等式的传递性进行交换, 即欲证 $a>b$, 可换成证 $a>c$ 且 $c>b$, 欲证 $a0, y>0)$,\n\n从而知 $M$ 点的球坐标为 $\\left(2, \\frac{\\pi}{4}, \\frac{\\pi}{4}\\right)$.", "solution": "null", "level": "高三", "question": "11. 设点 $M$ 的直角坐标为 $(1,1, \\sqrt{2})$, 求它的球坐标.", "options": [], "subject": "立体几何学", "analysis": ": 分析: 本题主要考查了 $\\left(2, \\frac{\\pi}{4}, \\frac{\\pi}{4}\\right)$, 解决问题的关键是本题考查直角坐标与球坐标的变换关系,解只需将已知条件代入变换公式求解即可, 但应注意 $\\theta$ 与 $\\varphi$ 的取值范围.由坐标变换公式,可得. 由直角坐标化为球坐标时, 我们可以先设点 $M$ 的球坐标为 $(r, \\varphi, \\theta)$, 再利用变换公式 $\\left\\{\\begin{array}{l}x=r \\sin \\varphi \\cos \\theta, \\\\ y=r \\sin \\varphi \\sin \\theta, \\text { 求出 } r . \\theta . \\varphi \\text { 代入点的球坐标即可; 也可以利用 } r^{2}=x^{2}+y^{2}+z^{2}, \\\\ z=r \\cos \\varphi\\end{array}\\right.$ $\\tan \\theta=\\frac{y}{x}, \\cos \\varphi=\\frac{z}{r}$, 特别注意由直角坐标求球坐标时, $\\theta$ 和 $\\varphi$ 的取值应首先看清点所在的象限, 准确取值, 才能无误."} {"id": "22850", "image": [], "answer": "答案: 解: $\\because M$ 的球坐标为 $\\left(5, \\frac{5}{6} \\pi, \\frac{4}{3} \\pi\\right)$,\n\n$\\therefore r=5, \\varphi=\\frac{5}{6} \\pi, \\theta=\\frac{4}{3} \\pi$.\n\n由变换公式 $\\left\\{\\begin{array}{c}x=r \\sin \\varphi \\cos \\theta \\text { 。 } \\\\ y=r \\sin \\varphi \\sin \\theta \\text {, 得 } \\\\ z=r \\cos \\varphi \\text {, }\\end{array}\\left\\{\\begin{array}{c}x=5 \\sin \\frac{5}{6} \\pi \\cos \\frac{4}{3} \\pi=-\\frac{5}{4} \\\\ y=5 \\sin \\frac{5}{6} \\pi \\cdot \\sin \\frac{4}{3} \\pi=-\\frac{5 \\sqrt{3}}{4} \\text {, } \\\\ z=5 \\cos \\frac{5}{6} \\pi=-\\frac{5 \\sqrt{3}}{2} \\text {, }\\end{array}\\right.\\right.$\n\n故它的直角坐标为 $\\left(-\\frac{5}{4},-\\frac{5 \\sqrt{3}}{4},-\\frac{5 \\sqrt{3}}{2}\\right)$.", "solution": "null", "level": "高三", "question": "12. 已知点 $M$ 的球坐标为 $\\left(5, \\frac{5}{6} \\pi, \\frac{4}{3} \\pi\\right)$, 求它的直角坐标", "options": [], "subject": "立体几何学", "analysis": ": 分析: 本题考查球坐标刻画点的位置,解答本题需要先搞清球坐标 $\\left(5, \\frac{5}{6} \\pi, \\frac{4}{3} \\pi\\right)$ 中各个坐标的意义,然后代入相应的公式求解即可."} {"id": "22851", "image": ["10089.jpg", "10090.jpg"], "answer": "答案:解:过点 $P$ 作 $P P^{\\prime}$ 垂直底面,垂足为 $P^{\\prime}$,\n\n\n\n$\\because P\\left(6, \\frac{\\pi}{4}, 5\\right)$,\n\n$\\therefore P^{\\prime}$ 点的坐标为 $\\left(6, \\frac{\\pi}{4}, 0\\right)$.\n\n$\\therefore$ 圆柱底面圆半径为 6 , 高为 5 .\n\n$\\therefore$ 圆柱的体积为 $V=\\pi \\times 6^{2} \\times 5=180 \\pi$.", "solution": "null", "level": "高三", "question": "13. 如图, $P$ 为圆柱的上底面与侧面交线上的一点, 且 $P$ 点的柱坐标为 $\\left(6, \\frac{\\pi}{4}, 5\\right)$, 求该圆柱的体积.\n\n", "options": [], "subject": "立体几何学", "analysis": ": 分析: 本题主要考查了柱坐标刻画点的位置, 解决问题的关键是转化计算即可"} {"id": "22852", "image": [], "answer": "答案: 解: 由公式 $\\left\\{\\begin{array}{c}x=\\rho \\cos \\theta, \\\\ y=\\rho \\sin \\theta, \\\\ z=z,\\end{array},\\left\\{\\begin{array}{c}-1=\\rho \\cos \\theta, \\\\ -\\sqrt{3}=\\rho \\sin \\theta, \\\\ z=3,\\end{array}\\right.\\right.$,\n\n$\\therefore \\rho^{2}=(-1)^{2}+(-\\sqrt{3})^{2}=4$.\n\n$\\therefore \\rho=2$.\n\n$\\therefore \\cos \\theta=-\\frac{1}{2}, \\sin \\theta=-\\frac{\\sqrt{3}}{2}$.\n\n又 $\\because \\theta \\in[0,2 \\pi)$,\n\n$\\therefore \\theta=\\frac{4}{3} \\pi$.\n\n即 $M$ 的柱坐标为 $\\left(2, \\frac{4}{3} \\pi, 3\\right)$.", "solution": "null", "level": "高三", "question": "15. 已知空间点 $M$ 的直角坐标为 $(-1,-\\sqrt{3}, 3)$, 求它的柱坐标", "options": [], "subject": "立体几何学", "analysis": ": 分析: 本题主要考查了柱坐标刻画点的位置, 解决问题的关键是根据所给条件转化即可"} {"id": "22854", "image": ["10091.jpg"], "answer": "答案: 解: 取圆雉的顶点 $O$ 为坐标原点, 建立球坐标系,\n\n\n\n设 $\\mathrm{t}$ 时刻蚂蚁在点 $M(r, \\varphi, \\theta)$ 处, 由题意得 $\\theta=\\omega t, z=v t, \\varphi=\\frac{\\pi}{3}$, 由于\n\n$\\frac{z}{r}=\\cos \\varphi=\\cos \\frac{\\pi}{a}=\\frac{1}{2}$, 于是 $r=2 z=2 v t$,\n\n所以 $t$ 时刻蚂蚁所在球坐标为 $M\\left(2 v t, \\frac{\\pi}{3}, \\omega t\\right), t \\in[0,+\\infty)$.", "solution": "null", "level": "高三", "question": "17. 一只蚂蚁在一个母线与轴线夹角为箃的圆锥面上从顶点出发盘旋着向上爬行, 已知它上升的速度为 $v>0$, 盘旋的角速度为 $\\omega>0$, 求 $t$ 时刻蚂蚁所在的位置的球坐标.", "options": [], "subject": "立体几何学", "analysis": ": 分析: 本题主要考查了球坐标刻画点的位置, 解决问题的关键是根据所给条件转化计算即可"} {"id": "22855", "image": [], "answer": "答案:解:由变换公式得:\n\n$x=r \\sin \\varphi \\cos \\theta=4 \\sin \\frac{3 \\pi}{4} \\cos \\frac{\\pi}{4}=2$.\n\n$y=r \\sin \\varphi \\sin \\theta=4 \\sin \\frac{3 \\pi}{4} \\sin \\frac{\\pi}{4}=2$.\n\n$z=r \\cos \\varphi=4 \\cos \\frac{3 \\pi}{4}=-2 \\sqrt{2}$.\n\n$\\therefore$ 它的直角坐标为 $(2,2,-2 \\sqrt{2})$.", "solution": "null", "level": "高三", "question": "18. 已知点 $P$ 的球坐标为 $\\left(4, \\frac{3 \\pi}{4}, \\frac{\\pi}{4}\\right)$ 求它的直角坐标.", "options": [], "subject": "立体几何学", "analysis": ": 分析: 本题主要考查了球坐标刻画点的位置, 解决问题的关键是根据球坐标刻画点的位置转化即可"} {"id": "20483", "image": [], "answer": "答案: 由题意知, $f\\left(2^{2}\\right)>2=\\frac{2+2}{2}, f\\left(2^{3}\\right)>\\frac{5}{2}=\\frac{3+2}{2}$\n$f\\left(2^{4}\\right)>3=\\frac{4+2}{2}, f\\left(2^{5}\\right)>\\frac{7}{2}=\\frac{5+2}{2}$.\n\n由此得到一般性结论: $f\\left(2^{n+1}\\right)>\\frac{n+3}{2}$\n\n(或者猜测 $f\\left(2^{n}\\right)>\\frac{n+2}{2}(n \\geq 2, n \\in N)$ 也行)\n证明: (1)当 $n=1$ 时, $f\\left(2^{2}\\right)=1+\\frac{1}{2}+\\frac{1}{3}+\\frac{1}{4}=\\frac{25}{12}>\\frac{4}{2}=\\frac{1+3}{2}$, 所以结论成立 (2)假设 $n=k(k \\geq 1, k \\in N)$ 时, 结论成立, 即 $f\\left(2^{k+1}\\right)>\\frac{k+3}{2}$\n\n那么, $n=k+1$ 时, $f\\left(2^{k+2}\\right)=1+\\frac{1}{2}+\\frac{1}{3}+\\cdots+\\frac{1}{2^{k+1}}+\\frac{1}{2^{k+1}+1}+\\frac{1}{2^{k+1}+2}+\\cdots+\\frac{1}{2^{k+2}}$\n\n$>\\frac{k+3}{2}+\\frac{1}{2^{k+1}+1}+\\frac{1}{2^{k+1}+2}+\\cdots+\\frac{1}{2^{k+2}}$\n\n$>\\frac{k+3}{2}+\\frac{1}{2^{k+2}}+\\frac{1}{2^{k+2}}+\\cdots+\\frac{1}{2^{k+2}}=\\frac{k+3}{2}+\\frac{2^{k+1}}{2^{k+2}}=\\frac{k+1+3}{2}$\n\n所以当 $n=k+1$ 时, 结论也成立.\n\n综上所述,上述结论对 $n \\geq 1, n \\in N$ 都成立,所以猜想成立.", "solution": "null", "level": "高三", "question": "23. 已知 $f(n)=1+\\frac{1}{2}+\\frac{1}{3}+\\cdots+\\frac{1}{n}$. 经计算得 $f(4)>2, f(8)>\\frac{5}{2}, f(16)>3, f(32)>\\frac{7}{2}$.\n\n(1)由上面数据, 试猜想出一个一般性结论;\n\n \n(2)用数学归纳法证明你的猜想.", "options": [], "subject": "计数", "analysis": ": 分析: 本题主要考查了归纳推理, 解决问题的关键是 (1) 由归纳推理进行猜想; (2)利用数学归纳法的步骤进行证明."} {"id": "22425", "image": [], "answer": "(1) 当 $n>16$ 时, $y=16 \times(10-5)=80$;当 $n \\leq 16$ 时, $y=(10-5) n-5 \times(16-n)=10 n-80$ ,$y=\\left\\{\begin{array}{l}10 n-80, n \\leq 16 \\ 80, n>16,\\end{array}(n \\in \\mathbb{N})\right.$. (2) (1) $X$ 可取 ${ }^{60,70,80}$, $P(X=60)=0.1, P(X=70)=0.2, P(X=80)=0.7$$X$ 的分布列为:| $\\mathrm{X}$ | 60 | 70 | 80 || :--- | :--- | :--- | :--- | $\\mathrm{P}$ | 0.1 | 0.2 | 0.7 |$E X=60 \times 0.1+70 \times 0.2+80 \times 0.7=76$,$D X=16^{2} \times 0.1+6^{2} \times 0.2+4^{2} \times 0.7=44$.(2)若购进 17 枝,则当天利润的期望为$(14 \times 5-3 \times 5) \times 0.1+(15 \times 5-2 \times 5) \times 0.2+(16 \times 5-1 \times 5) \times 0.16+17 \times 5 \times 0.54=76.4$,因为 $76.4>76$, 所以应购迸 17 枝.", "solution": "null", "level": "高三", "question": "某花店每天以每枝 5 元的价格从农场购进若干枝玫瑰花,然后以每枝 10 元的价格出售.如果当天卖不完,剩下的玫瑰花作垃圾处理.\n\n(1) 若花店一天购进 16 枝玫瑰花, 求当天的利润 ${ }^{y}$ (单位:元)关于当天需求量 ${ }^{n}$ (单位:枝, $n \\in \\mathbb{N}$ )的函数解析式\n\n(2) 花店记录了 100 天玫瑰花的口需求量(单位:枝), 整理得下表:\n\n| 日需求量 n | 14 | 15 | 16 | 17 | 18 | 19 | 20 |\n| :--- | :--- | :--- | :--- | :--- | :--- | :--- | :--- |\n| 频数 | 10 | 20 | 16 | 16 | 15 | 13 | 10 |\n\n以 100 天记录的各需求量的频率作为各需求量发生的概率.\n\n(1)若花店一天购进 16 枝玫瑰花, $X$ 表示当天的利润,求 $X$ 的分布列、数学期望及方差;\n\n(2)若花店计划一天购进 16 枝或 17 枝玫瑰花,你认为应购进 16 枝还是 17 枝? 请说明理由.", "options": [], "subject": "计数", "analysis": ""} {"id": "22433", "image": [], "answer": ":(1)设事件 $A$ 表示“ 2 件合格, 2 件不合格”;事件 $B$ 表示“ 3 件合格, 1 件不合格”;事件 $C$ 表示“ 4 件全合格”;事件 $D$ 表示“检测通过”;事件 $E$ 表示“检测良好”。$\therefore P(D)=P(A)+P(B)+P(C)=\frac{\\mathrm{C}_{4}^{2} \\mathrm{C}_{4}^{2}}{\\mathrm{C}_{8}^{4}}+\frac{\\mathrm{C}_{4}^{3} \\mathrm{C}_{4}^{1}}{\\mathrm{C}_{8}^{4}}+\frac{\\mathrm{C}_{4}^{4}}{\\mathrm{C}_{8}^{4}}=\frac{53}{70}$$\therefore P(E \\mid D)=\frac{P(C)}{P(D)}+\frac{P(B)}{P(D)}=\frac{17}{53}$.故所求概率为 $\frac{17}{53}$.(2) $X$ 可能取值为 ${ }^{0,1,2}$$P(X=0)=\frac{\\mathrm{C}_{8}^{2}}{\\mathrm{C}_{12}^{2}}=\frac{14}{33}$$P(X=1)=\frac{\\mathrm{C}_{4}^{1} \\mathrm{C}_{8}^{1}}{\\mathrm{C}_{12}^{2}}=\frac{16}{33}$$P(X=2)=\frac{\\mathrm{C}_{4}^{2}}{\\mathrm{C}_{12}^{2}}=\frac{1}{11}$| $X$ | 0 | 1 | 2 || :---: | :---: | :---: | :---: || $P$ | $\frac{14}{33}$ | $\frac{16}{33}$ | $\frac{1}{11}$ |分布列为所以, $E(X)=0 \times \frac{14}{33}+1 \times \frac{16}{33}+2 \times \frac{1}{11}=\frac{2}{3}$.", "solution": "null", "level": "高三", "question": "质检部门对某工厂甲、乙两个车间生产的 12 个零件质量进行检测. 甲、乙两个车间的零件质量 (单位: 克) 分布的茎叶图如图所示. 零件质量不超过 20 克的为合格.\n\n![](https://cdn.mathpix.com/cropped/2024_04_19_7da760c0cd38339a5113g-", "options": [], "subject": "计数", "analysis": ""} {"id": "22440", "image": [], "answer": "(1)因为这位司机在第一、二个交通岗未遇到红灯, 在第三个交通岗遇到红灯, 所以 $P=\\left(1-\frac{1}{3}\right) \times\\left(1-\frac{1}{3}\right) \times \frac{1}{3}=\frac{4}{27}$ (2)易知随机变量 $$X \\sim B\\left(6, \frac{1}{3}\right) $$所以 $E(X)=6 \times \frac{1}{3}=2, D(X)=6 \times \frac{1}{3} \times\\left(1-\frac{1}{3}\right)=\frac{4}{3}$.", "solution": "null", "level": "高三", "question": "出租车司机从饭店到火车站途中有六个交通岗, 假设他在各交通岗遇到红灯的事件是相互独立的, 并且概率都是 ${ }^{\\frac{1}{3}}$.\n\n(1)求这位司机遇到红灯前, 已经通过了两个交通岗的概率;\n\n(2)求这位司机在途中遇到红灯数 $X$ 的均值与方差.", "options": [], "subject": "计数", "analysis": ""} {"id": "22456", "image": [], "answer": "答案: (1) 一台机器是否出现故障可看作一次试验, 在一次试验中, 记机器出现故障为事件 $A$, 则事件 $A$ 的概率为 $\\frac{1}{3}$, 该厂有 4 台机器就相当于 4 次独立重复试验, 因为出现故障的机器台数为 $X$, 所以 $X \\sim B\\left(4, \\frac{1}{3}\\right), P(X=0)=\\mathrm{C}_{4}^{0} \\times\\left(\\frac{2}{3}\\right)^{4}=\\frac{16}{81}, P(X=1)=$\n\n$$\n\\begin{aligned}\n& \\mathrm{C}_{4}^{1} \\times \\frac{1}{3} \\times\\left(\\frac{2}{3}\\right)^{3}=\\frac{32}{81}, F(X=2)=\\mathrm{C}_{4}^{2} \\times\\left(\\frac{1}{3}\\right)^{2} \\times\\left(\\frac{2}{3}\\right)^{2}=\\frac{8}{27} \\\\\n& F(X=3)=\\mathrm{C}_{4}^{3} \\times\\left(\\frac{1}{3}\\right)^{3} \\times \\frac{2}{3}=\\frac{8}{81}, F(X=4)=\\mathrm{C}_{4}^{4} \\times\\left(\\frac{1}{3}\\right)^{4}=\\frac{1}{81}\n\\end{aligned}\n$$\n\n$X$ 的分布列为\n\n| $X$ | 0 | 1 | 2 | 3 | 4 |\n| :---: | :---: | :---: | :---: | :---: | :---: |\n| $P$ | $\\frac{16}{81}$ | $\\frac{32}{81}$ | $\\frac{8}{27}$ | $\\frac{8}{81}$ | $\\frac{1}{81}$ |\n\n(2)设该厂有 $n$ 名工人,则“每台机器在任何时刻同时出现故障时能及时进行维修”满足的条件为 $X, n$ ,即 $X=0, X=1, \\mathrm{~L}, X=n$ 这 $n+1$ 个互斥事件的和事件,则\n\n| $n$ | 0 | 1 | 2 | 3 | 4 |\n| :---: | :---: | :---: | :---: | :---: | :---: |\n| $P(X, n)$ | $\\frac{16}{81}$ | $\\frac{16}{27}$ | $\\frac{8}{9}$ | $\\frac{80}{81}$ | 1 |\n\n因为 $\\frac{72}{81}<90 \\%<\\frac{80}{81}$, 所以至少有 3 名工人才能保证每台机器在任何时刻同时出现故障能及时进行维修的概率不小于 $90 \\%$.", "solution": "null", "level": "高三", "question": "某厂有 4 台大型机器, 在一个月中, 一台机器至多出现 1 次故障, 且每台机器是否出现故障是相互独立的, 出现故障时需 1 名工人进行维修, 每台机器出现故障需要维修的概率为 ${ }^{\\frac{1}{3}}$.\n\n(1)若出现故障的机器台数为 $X$, 求 $X$ 的分布列.\n\n(2)该厂至少有多少名工人才能保证每台机器在任何时刻同时出现故障时能及时进行维修的概率不小于 $90 \\%$ ?", "options": [], "subject": "计数", "analysis": "答案: (1) 一台机器是否出现故障可看作一次试验, 在一次试验中, 记机器出现故障为事件 $A$, 则事件 $A$ 的概率为 $\\frac{1}{3}$, 该厂有 4 台机器就相当于 4 次独立重复试验, 因为出现故障的机器台数为 $X$, 所以 $X \\sim B\\left(4, \\frac{1}{3}\\right), P(X=0)=\\mathrm{C}_{4}^{0} \\times\\left(\\frac{2}{3}\\right)^{4}=\\frac{16}{81}, P(X=1)=$\n\n$$\n\\begin{aligned}\n& \\mathrm{C}_{4}^{1} \\times \\frac{1}{3} \\times\\left(\\frac{2}{3}\\right)^{3}=\\frac{32}{81}, F(X=2)=\\mathrm{C}_{4}^{2} \\times\\left(\\frac{1}{3}\\right)^{2} \\times\\left(\\frac{2}{3}\\right)^{2}=\\frac{8}{27} \\\\\n& F(X=3)=\\mathrm{C}_{4}^{3} \\times\\left(\\frac{1}{3}\\right)^{3} \\times \\frac{2}{3}=\\frac{8}{81}, F(X=4)=\\mathrm{C}_{4}^{4} \\times\\left(\\frac{1}{3}\\right)^{4}=\\frac{1}{81}\n\\end{aligned}\n$$\n\n$X$ 的分布列为\n\n| $X$ | 0 | 1 | 2 | 3 | 4 |\n| :---: | :---: | :---: | :---: | :---: | :---: |\n| $P$ | $\\frac{16}{81}$ | $\\frac{32}{81}$ | $\\frac{8}{27}$ | $\\frac{8}{81}$ | $\\frac{1}{81}$ |\n\n(2)设该厂有 $n$ 名工人,则“每台机器在任何时刻同时出现故障时能及时进行维修”满足的条件为 $X, n$ ,即 $X=0, X=1, \\mathrm{~L}, X=n$ 这 $n+1$ 个互斥事件的和事件,则\n\n| $n$ | 0 | 1 | 2 | 3 | 4 |\n| :---: | :---: | :---: | :---: | :---: | :---: |\n| $P(X, n)$ | $\\frac{16}{81}$ | $\\frac{16}{27}$ | $\\frac{8}{9}$ | $\\frac{80}{81}$ | 1 |\n\n因为 $\\frac{72}{81}<90 \\%<\\frac{80}{81}$, 所以至少有 3 名工人才能保证每台机器在任何时刻同时出现故障能及时进行维修的概率不小于 $90 \\%$."} {"id": "22473", "image": [], "answer": "(1) $\because \\mu=65, \\sigma=2.2, \\mu-3 \\sigma=58.4, \\mu+3 \\sigma=71.6$, 而 $73 \\in(\\mu+3 \\sigma,+\\infty)$,$\therefore P(X>71.6)=\frac{1-P(58.4(2)由图看出, 样本点呈条状分布, 有比较好的线性相关关系, 因此可以用线性回归方程来建立两个变量之间的关系.设线性回归方程为 $\\hat{y}=\\hat{b} x+\\hat{a}$, 由表中数据可得 $\\hat{b} \\approx 0.29, \\hat{a} \\approx 34.70$, 故 $\\mathrm{y}$ 与 $\\mathrm{x}$ 之间的回归方程为 $\\hat{y}=0.29 x+34.70$.当 $x=56.7$ 时, $\\hat{y}=0.29 \times 56.7+34.70=51.143$.所以由回归方程可预报当基本苗数为 56.7 时,有效穗数为 51.143 .(3)各组数据的残差分别为 $\\widehat{e_{1}}=0.35, \\widehat{e_{2}}=0.718, \\widehat{e_{3}}=-0.5, \\widehat{e_{4}}=-2.214, \\widehat{e_{3}}=1.624$. $R^{2}=1-\frac{\\sum_{i=1}^{5}\\left(y_{i}-\\widehat{y_{i}}\right)^{2}}{\\sum_{i=1}^{5}\\left(y_{i}-\bar{y}\right)^{2}} \\approx 0.832$", "solution": "null", "level": "高三", "question": "假设某农作物基本苗数 $x$ 与有效穗数 $y$ 之间存在相关关系, 令测得 5 组数据如下:\n\n| $x$ |", "options": [], "subject": "计数", "analysis": ""} {"id": "20858", "image": ["9503.jpg"], "answer": "(1) $300 \times \frac{4500}{15000}=90$, 所以应收集 90 位女生的样本数据.(2)由频率分布直方图得 $1-2 \times(0.100+0.025)=0.75$, 所以该校学生每周平均体育运动时间超过 4 小时的概率的估计值为 0.75 .(3)由 2 知, 300 位学生中有 $300 \times 0.75=225$ 人的每周平均体育运动时间超过 4 小时, 75 人的每周平均体育运动时间不超过 4 小时. 又因为样本数据中有 210 份是关于男生的, 90 份是关于女生的. 所以每周平均体育运动时间与性别列联表如下:| | 男生 | 女生 | 总计 || :--- | :--- | :--- | :--- || 每周平均体育运动
时间不超过 4 小时 | 45 | 30 | 75 || 每周平均体育运动
时间超过 4 小时 | 165 | 60 | 225 || 总计 | 210 | 90 | 300 |结合列联表可算得 $K^{2}=\frac{300 \times(45 \times 60-165 \times 30)^{2}}{75 \times 225 \times 210 \times 90}=\frac{100}{21} \\approx 4.762>3.841$.所以,有 $95 \\%$ 的把握认为 “该校学生的每周平均体育运动时间与性别有关。", "solution": "null", "level": "高三", "question": "某高校共有学生 15000 人,其中男生 10500 人,女生 4500 人.为调査该校学生每周平均体育运动时间的情况,采用分层抽样的方法,收集 300 位学生每周平均体育运动时间的样本数据(单位:小时).\n\n(1)应收集多少位女生的样本数据?\n\n(2)根据这 300 个样本数据, 得到学生每周平均体育运动时间的频率分布直方图(如图所示), 其中样本数据的分组区间为: $[0,2],(2,4],(4,6],(6,8],(8,10],(10,12]$. 估计该校学生每周平均体育运动时间超过 4 小时的概率;\n\n\n\n(3)在样本数据中, 有 60 位女生的每周平均体育运动时间超过 4 小时, 清完成每周平均体育运动时间与性别列联表, 并判断是否有 $95 \\%$ 的把握认为“该校学生的每周平均体育运动时间与性别有关”.", "options": [], "subject": "计数", "analysis": "无"} {"id": "20613", "image": [], "answer": "(1) $\\hat{y}=0.6 x+1$; (2) 9 (吨标准煤).", "solution": "null", "level": "高三", "question": "下表提供了某厂节能降耗技术改造后生产甲产品过程中记录的产量 $x$ (吨)与相应的生产能耗 $Y$ (吨标准煤) 的几组对应数据.", "options": [], "subject": "计数", "analysis": "(1) 由对应数据, 计算得 $\\bar{x}=\\frac{3+4+5+6+7}{5}=5, \\bar{y}=\\frac{3+3+4+5+5}{5}=4$,\n\n$\\hat{b}=\\frac{(3-5)(3-4)+(4-5)(3-4)+(5-5)(4-4)+(6-5)(5-4)+(7-5)(5-4)}{(3-5)^{2}+(4-5)^{2}+(5-5)^{2}+(6-5)^{2}+(7-5)^{2}}=0.6$\n\n$\\hat{a}=\\bar{y}-\\hat{b} \\bar{x}=4-0.6 \\times 5=1$\n\n所求的回归方程为 $\\hat{y}=0.6 x+1$\n\n(2) $x=100, \\hat{y}=0.6 \\times 100+1=61$,\n\n预测生产 100 吨甲产品的生产能耗比技改前降低 $70-61=9$ (吨标准煤).\n\n20. 答案:(1) $\\hat{y}="} {"id": "20614", "image": [], "answer": "B", "solution": "null", "level": "高三", "question": "下列关于回归分析与独立性检验的说法正确的是 ( )(1)回归分析和独立性检验没有什么区别;(2)回归分析是对两个变量准确关系的分析, 而独立性检验是分析两个变量之间的不确定性关系;(3)回归分析研究两个变量之间的相关关系, 独立性检验是对两个变量是否具有某种关系的一种检验;(4)独立性检验可以 $100 \\%$ 确定两个变量之间是否具有某种关系.A. (1) (2) B. (3)C. (3)(4) $\\mathrm{D}$.D. 全选", "options": [], "subject": "计数", "analysis": "答案:B\n\n【解析】回归分析是对两个变量之间的相关关系的一种分析, 而相关关系是一种不确定关系, 通过回归分析可能两个变量之间具有的相关关系.\n\n而独立性检验是对两个变量之间是否具有某种关系的分析, 并且可以分析这两个变量在多大程度上具有这种关系, 但不能 $100 \\%$ 肯定这种关系,\n\n故(1)(2)(4)错误,(3)正确,故选 B\n\n2. 答案:C\n\n【解析】本题考查“性别”对判断“人机大战是人类的胜利”这两个变量是否有关系, 符合独立性检验的基本思想,因此,该题所选择的统计方法是独立性检验,故选 C.\n\n3. 答案:A\n\n【解析】根据表中数据, 可得 $\\bar{x}=\\frac{1}{5} \\times(10+20+30+40+50)=30$,\n\n代入线性回归方程 $\\hat{y}=0.67 x+54.9$ 中, 求得 $\\bar{y}=0.67 \\times 30+54.9=75$,\n\n则表中模糊不清的数据是 $75 \\times 5-62-75-81-89=68$, 故选 A.\n\n4. 答案:D\n\n【解析】由题意得, $a+7=c=25,6+b=d=25, a+6=e, 7+b=f, e+f=50$,\n\n所以 $a=18, b=19, c+d=50, e=24, f=26$, 则 $e-f=-2$,\n\n故选 D.\n\n5. 答案:B\n\n【解析】由给出的四组数据的散点图可以看出,\n\n图 1 和图 3 是正相关,相关系数大于 0, 则 $r_{1}>0, r_{3}>0 ,$\n\n图 2 和图 4 是负相关, 相关系数小于 0, 则 $r_{2}<0, r_{4}<0$,\n\n图 3 和图 4 的点相对更加集中, 所以相关性较强, 所以 $r_{3}$ 接近于 $1, r_{4}$ 接近于 -1 ,\n\n图 1 和图 2 的点相对分散一些, 所以相关性较弱, 所以 $r_{1}$ 和 $r_{2}$ 比较接近 0 ,\n\n由此可得 $r_{4}\n\n(1) 求数学成绩特别优秀的人数及数学成绩的平均分;\n\n(2) 如果语文和数学两科都特别优秀的共有 3 人. 根据以上数据, 完成 $2 \\times 2$ 列联\n\n表, 并分析是否有 $99 \\%$ 的把握认为语文特别优秀的同学, 数学也特别优秀.\n\n| | 语文特别优秀 | 语文不特别优秀 | 合计 |\n| :---: | :--- | :--- | :--- |\n| 数学特别优秀 | | | |\n| 数学不特别优秀 | | | |\n| 合计 | | | |\n\n参考数据: (1) $K^{2}=\\frac{n(a d-b c)^{2}}{(a+b)(c+d)(a+c)(b+d)}$;\n\n(2)\n\n| $P\\left(K^{2} \\geq k_{0}\\right)$ |", "options": [], "subject": "计数", "analysis": "(1) 数学成绩特别优秀的概率为 $P_{2}=0.002 \times 20=0.04$数学特别优秀的同学有 $100 \times 0.04=4$ 人.$\bar{x}=0.14 \times 60+0.36 \times 80+0.4 \times 100+0.06 \times 120+0.04 \times 140=90$ 分.(2)共有100 名文科学生参加考试,其中语文考试成绩低于 130 的占 $95 \\%$,语文成绩特别优秀的概率为 $P_{1}=1-0.95=0.05$ ,语文特别优秀的同学有 $100 \times 0.05=5$ 人,$2 \times 2$ 列联表:| | 语文特别优秀 | 语文不特别优秀 | 合计 || :---: | :---: | :---: | :---: || 数学特别优秀 | 3 | 1 | 4 || 数学不特别优秀 | 2 | 94 | 96 || 合计 | 5 | 95 | 100 |所以 $K^{2}=\frac{100(3 \times 94-1 \times 2)^{2}}{4 \times 96 \times 5 \times 95} \\approx 42.982>6.635$,所以有 $99 \\%$ 的把握认为语文特别优秀的同学, 数学也特别优秀."} {"id": "20622", "image": ["9415.jpg"], "answer": "(1) $a=0.035,41.5$; (2) $\frac{3}{5}$ ;(3)没有.", "solution": "null", "level": "高三", "question": "(12 分) 为激活国内消费市场, 挽回疫情造成的损失, 国家出台一系列的促进国内消费的优惠政策, 某机构从某一电商的线上交易大数据中来跟踪调查消费者的购买力, 界定 3 至 8 月份购买商品在 5000 元以上人群属“购买力强人群”, 购买商品在 5000 元以下人群属“购买力弱人群”. 现从电商平台消费人群中随机选出 200 人, 发现这 200 人中属购买力强的人数占 $80 \\%$, 并将这 200 人按年龄分组, 记第 1 组 $[15,25)$, 第 2 组 $[25,35)$, 第 3 组 $[35,45)$, 第 4 组 $[45,55)$, 第 5 组 $[55,65)$,得到频率分布直方图, 如图.\n\n\n\n(1) 求出频率分布直方图中的 $a$ 值和这 200 人的平均年龄;\n\n(2) 从第 1,2 组中用分层抽样的方法抽取 5 人, 并再从这 5 人中随机抽取 2 人进行电话回访, 求这两人恰好属于不同组别的概率;\n\n(3) 把年龄在第 $1,2,3$ 组的居民称为青少年组, 年龄在第 4,5 组的居民称为中老年组, 若选出的 200 人中“购买力弱人群”的中老年人有 20 人, 问是否有 $99 \\%$ 的把握认为是否属“购买力强人群”与年龄有关?\n\n附:\n\n| $P\\left(K^{2} \\geq k_{0}\\right)$ |", "options": [], "subject": "计数", "analysis": "(1) 由题意得 $(2 \times 0.01+0.015+0.03+a) \times 10=1$, 所以 $a=0.035$,200 人的平均年龄为 $20 \times 0.1+30 \times 0.15+40 \times 0.35+50 \times 0.3+60 \times 0.1=41.5$.(2) 由题意得: 利用分层抽样的方法从第一组抽取 2 人, 从第二组抽取 3 人,记从第一组抽取的 2 人为 $A, B$; 从第二组抽取的 3 人为 $a, b, c$,则从这 5 人中随机抽取 2 人的基本事件有 $4+3+2+1=10$ ,即 10 种,其中两人恰好属于不同组别的基本事件有 $2 \times 3$ 种,即 6 种,故所求的概率 $P=\frac{6}{10}=\frac{3}{5}$.(3) 由题意可得 $2 \times 2$ 列联表为| | 购买力强人群 | 购买力弱人群 | 合计 || :---: | :---: | :---: | :---: || 青少年组 | 100 | 20 | 120 || 中老年组 | 60 | 20 | 80 || 合计 | 160 | 40 | 200 |故 $K^{2}$ 的观测值 $k=\frac{200(100 \times 20-60 \times 20)^{2}}{120 \times 80 \times 160 \times 40}=\frac{25}{12} \\approx 2.083<6.635$,故没有 $99 \\%$ 的把握认为是否属“购买力强人群”与年龄有关."} {"id": "20623", "image": [], "answer": "9", "solution": "null", "level": "高三", "question": "为了研究某种细菌在特定环境下, 随时间变化繁殖情况, 得如下实验数据, 计算得回归方程为 $\\hat{y}=0.85 x-0.25$. 由以上信息, 得到下表中 $c$ 的值为| 天数 $t$ (天) | 3 | 4 | 5 | 6 | 7 || :---: | :---: | :---: | :---: | :---: | :---: || 繁殖个数 $y$ (千个) | 25 | 3 | 4 | 45 | $c$ |", "options": [], "subject": "计数", "analysis": "+3+4+4.5+c)=\\frac{14+c}{5}$,\n\n所以这组数据的样本中心点是 $\\left(5, \\frac{14+c}{5}\\right)$,\n\n把样本中心点代入回归方程 $\\hat{y}=0.85 x-0.25$\n\n所以 $\\frac{14+c}{5}=0.85 \\times 5-0.25$, 所以 $c=6$, 故答案为 6 .\n\n16. 答案:9\n\n【解析】由题意知: $K^{2} \\geq"} {"id": "20624", "image": [], "answer": "(1) $\\hat{y}=3.7 x-1.1,25$; (2) 能在犯错误的概率不超过 0.001 的条件下认为“中学生对航空航天的兴趣爱好和性别有关系”.", "solution": "null", "level": "高三", "question": "(12 分) 我国探月工程嫦娥五号探测器于 2020 年 12 月 1 日 23 时 11 分降落在月球表面预选着陆区, 在顺利完成月面自动采样之后, 成功将携带样品的上升器送入到预定环月轨道, 这是我国首次实现月球无人采样和地外天体起飞, 对我国航天\n事业具有重大而深远的影响, 为进一步培养中学生对航空航天的兴趣爱好, 某学校航空航天社团在本校高一年级进行了纳新工作, 前五天的报名情况为: 第 1 天 3 人第 2 天 6 人,第 3 天 10 人,第 4 天 13 人,第 5 天 18 人,通过数据分析已知,报名人数与报名时间具有线性相关关系.\n\n(1) 已知第 $x$ 天的报名人数为 $y$, 求 $y$ 关于 $x$ 的线性回归方程, 并预测第 7 天的报名人数 (结果四舍五入取整数);\n\n(2) 该社团为了解中学生对航空航天的兴趣爱好和性别是否有关系, 随机调查了 100 名学生, 并得到如下 $2 \\times 2$ 列联表:\n\n| | 有兴趣 | 无兴趣 | 合计 |\n| :---: | :---: | :---: | :---: |\n| 男生 | 45 | 5 | 50 |\n| 女生 | 30 | 20 | 50 |\n| 合计 | 75 | 25 | 100 |\n\n请根据上面的列联表判断能否在犯错误的概率不超过", "options": [], "subject": "计数", "analysis": "(1) 时间的平均数为 $\bar{x}=\frac{1+2+3+4+5}{5}=3$,报名人数的平均数为 $\bar{y}=\frac{3+6+10+13+18}{5}=10$,所以 $\\hat{b}=\frac{\\sum_{i=1}^{7} x_{i} y_{i}-n \bar{x} \\cdot \bar{y}}{\\sum_{i=1}^{7} x_{i}^{2}-n \bar{x}^{2}}=\frac{187-5 \times 3 \times 10}{55-5 \times 9}=\frac{37}{10}=3.7$,$\\hat{a}=\bar{y}-\\hat{b} \bar{x}=10-3.7 \times 3=-1.1$,所以线性回归方程为 $\\hat{y}=3.7 x-1.1$,把 $x=7$ 代入得 $\\hat{y}=24.8 \\approx 25$, 所以第 7 天的报名人数约为 25 .(2) 由列联表数据可得 $K^{2}=\frac{100(45 \times 20-5 \times 30)^{2}}{75 \times 25 \times 50 \times 50}=12$,因为 $12>10.828$,所以,在犯错误的概率不超过 0.001 的条件下认为“中学生对航空航天的兴趣爱好和性别有关系”."} {"id": "22464", "image": [], "answer": "答案: 工人甲生产出次品数 $X$ 的均值和方差分别为\n\n$E(X)=0 \\times \\frac{3}{5}+1 \\times \\frac{1}{10}+2 \\times \\frac{3}{10}=0.7$,\n\n$D(X)=(0-0.7)^{2} \\times \\frac{3}{5}+(1-0.7)^{2} \\times \\frac{1}{10}+(2-0.7)^{2} \\times \\frac{3}{10}=0.81$\n\n工人乙生产出次品数 $Y$ 的均值和方差分别为\n\n$E(Y)=0 \\times \\frac{1}{2}+1 \\times \\frac{3}{10}+2 \\times \\frac{1}{5}=0.7$,\n\n$D(Y)=(0-0.7)^{2} \\times \\frac{1}{2}+(1-0.7)^{2} \\times \\frac{3}{10}+(2-0.7)^{2} \\times \\frac{1}{5}=0.61$\n\n由 $E(X)=E(Y)$ 知, 两人生产出次品的平均数相同, 技术水平相当, 但 $D(X)>D(Y)$, 可见乙的技术更稳定.", "solution": "null", "level": "高三", "question": "甲、乙两名工人加工同一种零件, 两人每天加工的零件数相同, 所得次品数分别为 $X, Y$,且 $X$ 和 $Y$ 的分布列如下表:\n\n| $X$ | 0 | 1 | 2 |\n| :---: | :---: | :---: | :---: |\n| $P$ | $\\frac{3}{5}$ | $\\frac{1}{10}$ | $\\frac{3}{10}$ |\n\n\n| $Y$ | 0 | 1 | 2 |\n| :---: | :---: | :---: | :---: |\n| $P$ | $\\frac{1}{2}$ | $\\frac{3}{10}$ | $\\frac{1}{5}$ |\n\n试对这两名工人的技术水平进行比较.", "options": [], "subject": "统计数学", "analysis": "答案: 工人甲生产出次品数 $X$ 的均值和方差分别为\n\n$E(X)=0 \\times \\frac{3}{5}+1 \\times \\frac{1}{10}+2 \\times \\frac{3}{10}=0.7$,\n\n$D(X)=(0-0.7)^{2} \\times \\frac{3}{5}+(1-0.7)^{2} \\times \\frac{1}{10}+(2-0.7)^{2} \\times \\frac{3}{10}=0.81$\n\n工人乙生产出次品数 $Y$ 的均值和方差分别为\n\n$E(Y)=0 \\times \\frac{1}{2}+1 \\times \\frac{3}{10}+2 \\times \\frac{1}{5}=0.7$,\n\n$D(Y)=(0-0.7)^{2} \\times \\frac{1}{2}+(1-0.7)^{2} \\times \\frac{3}{10}+(2-0.7)^{2} \\times \\frac{1}{5}=0.61$\n\n由 $E(X)=E(Y)$ 知, 两人生产出次品的平均数相同, 技术水平相当, 但 $D(X)>D(Y)$, 可见乙的技术更稳定."} {"id": "20582", "image": ["9384.jpg"], "answer": "答案: 解: 此程序框图输出的 $y=\\left\\{\\begin{array}{c}\\sqrt{a}, a \\geq 0 \\\\ \\text { 不存在, } \\mathrm{a}<0\\end{array}\\right.$, 所以是求非负数 $\\mathrm{a}$ 的算术平方根.", "solution": "null", "level": "高三", "question": "24. 试说明图中的算法流程图的设计是求什么?\n\n", "options": [], "subject": "图论", "analysis": ": 分析: 本题主要考查了流程图的作用, 解决问题的关键是根据所给程序框图进行分析即可, 难度不大."} {"id": "14959", "image": [], "answer": "(1) $\\{(2,1)\\}$(2)$A=\\{3,2,1,-2\\}$。", "solution": "null", "level": "高一", "question": "用列举法表示下列集合:\n\n(1) 方程组 $\\left\\{\\begin{array}{l}x+y=3 \\\\ x-y=1 \\text { 的解集; }\\end{array}\\right.$\n\n(2)不大于 10 的非负奇数集;\n\n(3) $A=\\left\\{x \\mid x \\in \\mathbf{Z}, \\frac{6}{4-x} \\in \\mathbf{N}\\right\\}$.", "options": [], "subject": "解析几何", "analysis": "(1) 解: 由 $\\left\\{\\begin{array}{l}x+y=3 \\\\ x-y=1\\end{array}\\right.$ 得 $\\left\\{\\begin{array}{l}x=2 \\\\ y=1\\end{array}\\right.$ 故方程组 $\\left\\{\\begin{array}{l}x+y=3 \\\\ x-y=1\\end{array}\\right.$ 的解集为 $\\{(2,1)\\}$\n\n(2)解: 不大于 10 即为小于或等于 10 , 非负是大于或等于 0 ,\n\n故不大于 10 的非负奇数集为 $\\{1,3,5,7,9\\}$;\n\n(3) 解: $\\because x \\in \\mathbf{Z}, \\quad \\frac{6}{4-x} \\in \\mathbf{N}, \\therefore 4-x=12,3,6$.\n\n此时 $x=3,2,1,-2$, 即 $A=\\{3,2,1,-2\\}$\n\n(1)解方程组,求出解,再表示为集合;\n\n(2)找到不大于 10 的非负奇数,用列举法写出集合;\n\n(3)分析当 $x$ 取哪些值时满足条件,得到集合."} {"id": "14997", "image": [], "answer": "(1)$A \\cap B=\\{(0,0)\\}$\n\n(2) $(A \\cap B) \\cup(B \\cap C)=\\{(0,0)\\} \\cup\\left\\{\\left(\\frac{3}{5},-\\frac{9}{5}\\right)\\right\\}=\\left\\{(0,0),\\left(\\frac{3}{5},-\\frac{9}{5}\\right)\\right\\}$", "solution": "null", "level": "高一", "question": "设全集 $A=\\{(x, y) \\mid 2 x-y=0\\}, B=\\{(x, y) \\mid 3 x+y=0\\}, C=\\{(x, y) \\mid 2 x-y=3\\}$.\n\n(1) 求 $A \\cap B$;\n\n(2) 求 $(A \\cap B) \\cup(B \\cap C)$.", "options": [], "subject": "解析几何", "analysis": "(1) 解: 联立 $\\left\\{\\begin{array}{l}2 x-y=0 \\\\ 3 x+y=0\\end{array} \\Rightarrow\\left\\{\\begin{array}{l}x=0 \\\\ y=0\\end{array}\\right.\\right.$,\n\n可得直线 $2 x-y=0$ 与直线 $3 x+y=0$ 的交点为 $(0,0)$.\n\n所以 $A \\cap B=\\{(0,0)\\}$.\n\n(2) 解: 联立\n\n$$\n\\left\\{\\begin{array} { l } \n{ 3 x + y = 0 } \\\\\n{ 2 x - y = 3 }\n\\end{array} \\Rightarrow \\left\\{\\begin{array}{l}\nx=\\frac{3}{5} \\\\\ny=-\\frac{9}{5}\n\\end{array}\\right.\\right.\n$$\n\n可得直线 $3 x+y=0$ 与直线 $2 x-y=3$ 的交点为 $\\left(\\frac{3}{5},-\\frac{9}{5}\\right)$.\n\n所以 $B \\cap C=\\left\\{\\left(\\frac{3}{5},-\\frac{9}{5}\\right)\\right\\}$,\n\n即 $(A \\cap B) \\cup(B \\cap C)=\\{(0,0)\\} \\cup\\left\\{\\left(\\frac{3}{5},-\\frac{9}{5}\\right)\\right\\}=\\left\\{(0,0),\\left(\\frac{3}{5},-\\frac{9}{5}\\right)\\right\\}$\n\n【分析】(1) 求 $A \\cap B$ 即是求 $A$ 和 $B$ 集合所表示的两条直线的交点的集合; (2) 求 $B \\cap C$ 即是求 $B$ 和 $C$ 集合所表示的两条直线的交点的集合,再与 (1) 中所得集合求并集即可."} {"id": "15035", "image": ["7490.jpg", "7491.jpg"], "answer": "(1) 解: $\\mathrm{f}(0)=4$, f (4) $=2$;\n\n(2)解:当 $0 \\leq x \\leq 2$ 时,设 $\\mathrm{f}(\\mathrm{x})=k \\mathrm{kx}+\\mathrm{b}$,\n\n代入 $(0,4)$\n\n$(2,0)\\left\\{\\begin{array}{c}b=4 \\\\ 2 k+b=0\\end{array}\\right.$\n\n(k $k=-2$\n\n$\\therefore\\{b=4$, 即 $\\mathrm{f}(\\mathrm{x})=-2 \\mathrm{x}+4$\n\n当 $2 \\leq x \\leq 6$ 时, 代入(2,0)(6,4),\n\n\n\n即 $f(x)=x-2$,\n\n综上, $f(x)=\\left\\{\\begin{array}{c}-2 x+40, \\leq x \\leq 2 \\\\ x-2,2\n\n(1) 求 $\\mathrm{f}[\\mathrm{f}(0)]$ 的值;\n\n(2) 求函数 $\\mathrm{f}(\\mathrm{x})$ 的解析式.", "options": [], "subject": "解析几何", "analysis": "(1)根据所给的函数图像求得 $\\mathrm{f}[\\mathrm{f}(0)]$ 的值; (2)根据函数图像可知函数是线段 $\\mathrm{AB}$ 与线段 $\\mathrm{BC}$ 组成的分段函数."} {"id": "15037", "image": [], "answer": "(1) 解: 设 $\\mathrm{f}(\\mathrm{x})=\\mathrm{ax}^{2}+\\mathrm{bx}+\\mathrm{c}(\\mathrm{a} \\neq 0)$,\n\n由 $f(0)=2$, 得 $c=2$. 由 $f(x+1)-f(x)=x-1$,\n\n得恒等式 $2 a x+a+b=x-1$, 得 $a=\\frac{1}{2}, b=-\\frac{3}{2}$.\n\n故所求函数的解析式为 $\\mathrm{f}(\\mathrm{x})=\\frac{1}{2} \\mathrm{x}^{2}-\\frac{3}{2} \\mathrm{x}+2$.\n\n(2) 解: 由 $3 f(x)+2 f(-x)=x+3$, (1)\n\n$x$ 用 $-x$ 代换得 $3 f(-x)+2 f(x)=-x+3$ ,(2)\n解(1)(2)得 $f(x)=x+\\frac{3}{5}$.", "solution": "null", "level": "高一", "question": "求下列函数的解析式:\n\n(1) 已知 $f(x)$ 是二次函数, 且 $f(0)=2, f(x+1)-f(x)=x-1$, 求 $f(x)$;\n\n(2)已知 $3 f(x)+2 f(-x)=x+3$, 求 $f(x)$.", "options": [], "subject": "解析几何", "analysis": "(1)设二次函数为 $f(x)=a x^{2}+b x+c(a \\neq 0)$, 将 $f(0)=2$ 与 $f(x+1)-f(x)=x-1$ 代入化简, 可求得 $a, b, c$ 的值. (2) 将 $-x$ 代入原方程得到一个新的方程, 和原方程组成方程组, 解方程组来求得 $f(x)$ 的值."} {"id": "15121", "image": [], "answer": "(1) 解: 由 $f(x)<0$ 的解集为 $(0,2)$,\n\n可知 $a>0$ 且 $f(x)=a x(x-2)$.\n则 $f(1)=-1 \\Rightarrow a=1 \\Rightarrow f(x)=x^{2}-2 x$\n\n(2) 解: $2^{f(x)}-\\frac{1}{4}>0 \\Leftrightarrow 2^{a x(x-2)}>2^{-2}$\n\n$\\Leftrightarrow a x^{2}-2 a x+2>0$ 的解集为 R., 当 $a=0$ 时, 满足题意;\n\n当 $a \\neq 0$ 时, 由 $\\left\\{\\begin{array}{c}a>0 \\\\ \\Delta=4 a^{2}-8 a<0\\end{array} \\Rightarrow 00$ 对一切实数恒成立, 求实数 $a$ 的取值范围.", "options": [], "subject": "解析几何", "analysis": "(1)根据不等式的解集与方程根的关系, 即可求出实数 $\\mathrm{a}$ 的值;\n\n(2) 根据指数函数的单调性, 解不等式, 将不等式恒成立问题转化, 即可求出实数 $\\mathrm{a}$ 的取值范围."} {"id": "16987", "image": [], "answer": "解:(1)点 $P(6,4)$ 关于轴的对称点的坐标 $P_{1}(6,-4)$,\n\n则反射光线所在的直线过点 $P_{1}$ 和 $Q$, 所以 $k{ }_{P_{1} Q}=\\frac{-4-0}{6-2}=-1$,所以反射光线 $Q H$ 的方程为 $y=-(x-2)$, 即 $y=-x+2$.\n\n(2)由题意,从甲罐、乙罐中分别随机抽取 1 个小球,共包含 20 个基本事件;\n\n“抽取的两个小球标号之和大于 5 ” 包含的基本事件有:(1,5),(1,6),(2,5),(2,6), $(3,3),(3,5),(3,6),(4,2),(4,3),(4,5),(4,6)$, 共 11 个基本事件; “抽取的两个小球标号之积大于 8 ” 包含的基本事件有:(2,5),(2,6),(3,3),(3, 5), (3,6), (4, 3), (4,5), (4,6), 共 8 个基本事件;\n\n即事件 $B$ 是事件 $A$ 的子事件; 所以事件 $A \\cup B$ 包含的基本事件个数为 11 个,\n所以事件 $A \\cup B$ 发生的概率为 $\\frac{11}{20}$.", "solution": "null", "level": "高一", "question": "020 秋・湖北期中)(1)一条光线从点 $P(6,4 )$ 射出, 与 $x$ 轴相交于点 $Q(2,0)$, 经 $x$ 轴反射后与 $y$ 轴交于点 $H$, 求反射光线 $Q H$ 所在直线的方程.\n\n(2)已知甲罐中有四个相同的小球, 标号为 $1,2,3,4$; 乙罐中有五个相同的小球, 标号为 1 , $2,3,5,6$, 现从甲罐、乙罐中分别随机抽取 1 个小球, 记事件 $A=$ “抽取的两个小球标号之和大于 5 ”,事件 $B=$ “抽取的两个小球标号之积大于 8 ”,求事件 “ $A \\cup B$ ”发生的概率.", "options": [], "subject": "解析几何", "analysis": "解:(1)点 $P(6,4)$ 关于轴的对称点的坐标 $P_{1}(6,-4)$,\n\n则反射光线所在的直线过点 $P_{1}$ 和 $Q$, 所以 $k{ }_{P_{1} Q}=\\frac{-4-0}{6-2}=-1$,所以反射光线 $Q H$ 的方程为 $y=-(x-2)$, 即 $y=-x+2$.\n\n(2)由题意,从甲罐、乙罐中分别随机抽取 1 个小球,共包含 20 个基本事件;\n\n“抽取的两个小球标号之和大于 5 ” 包含的基本事件有:(1,5),(1,6),(2,5),(2,6), $(3,3),(3,5),(3,6),(4,2),(4,3),(4,5),(4,6)$, 共 11 个基本事件; “抽取的两个小球标号之积大于 8 ” 包含的基本事件有:(2,5),(2,6),(3,3),(3, 5), (3,6), (4, 3), (4,5), (4,6), 共 8 个基本事件;\n\n即事件 $B$ 是事件 $A$ 的子事件; 所以事件 $A \\cup B$ 包含的基本事件个数为 11 个,\n所以事件 $A \\cup B$ 发生的概率为 $\\frac{11}{20}$."} {"id": "17139", "image": [], "answer": "解:(1)点 $P(6,4)$ 关于轴的对称点的坐标 $P_{1}(6,-4)$,\n\n则反射光线所在的直线过点 $P_{1}$ 和 $Q$, 所以 $k{ }_{P_{1} Q}=\\frac{-4-0}{6-2}=-1$,\n\n所以反射光线 $Q H$ 的方程为 $y=-(x-2)$, 即 $y=-x+2$.\n\n(2)由题意,从甲罐、乙罐中分别随机抽取 1 个小球,共包含 20 个基本事件;\n\n“抽取的两个小球标号之和大于 5 ” 包含的基本事件有:(1,5),(1,6),(2,5),(2,6), $(3,3),(3,5),(3,6),(4,2),(4,3),(4,5),(4,6)$, 共 11 个基本事件; “抽取的两个小球标号之积大于 8 ” 包含的基本事件有:(2,5),(2,6),(3,3),(3,5), $(3,6),(4,3),(4,5),(4,6)$, 共 8 个基本事件;即事件 $B$ 是事件 $A$ 的子事件;所以事件 $A \\cup B$ 包含的基本事件个数为 11 个,所以事件 $A \\cup B$ 发生的概率为 $\\frac{11}{20}$.", "solution": "null", "level": "高一", "question": "(2020 秋・湖北期中)(1)一条光线从点 $P(6,4)$ 射出, 与 $x$ 轴相交于点 $Q(2,0)$, 经 $x$ 轴反射后与 $y$ 轴交于点 $H$, 求反射光线 $Q H$ 所在直线的方程.\n\n(2)已知甲罐中有四个相同的小球, 标号为 $1,2,3,4$; 乙罐中有五个相同的小球, 标号为 1 , 2, 3, 5, 6, 现从甲罐、乙罐中分别随机抽取 1 个小球, 记事件 $A=$ “抽取的两个小球标号之和大于 5 ”,事件 $B=$ “抽取的两个小球标号之积大于 8 ”,求事件 “ $A \\cup B$ ”发生的概率.", "options": [], "subject": "解析几何", "analysis": "解:(1)点 $P(6,4)$ 关于轴的对称点的坐标 $P_{1}(6,-4)$,\n\n则反射光线所在的直线过点 $P_{1}$ 和 $Q$, 所以 $k{ }_{P_{1} Q}=\\frac{-4-0}{6-2}=-1$,\n\n所以反射光线 $Q H$ 的方程为 $y=-(x-2)$, 即 $y=-x+2$.\n\n(2)由题意,从甲罐、乙罐中分别随机抽取 1 个小球,共包含 20 个基本事件;\n\n“抽取的两个小球标号之和大于 5 ” 包含的基本事件有:(1,5),(1,6),(2,5),(2,6), $(3,3),(3,5),(3,6),(4,2),(4,3),(4,5),(4,6)$, 共 11 个基本事件; “抽取的两个小球标号之积大于 8 ” 包含的基本事件有:(2,5),(2,6),(3,3),(3,5), $(3,6),(4,3),(4,5),(4,6)$, 共 8 个基本事件;即事件 $B$ 是事件 $A$ 的子事件;所以事件 $A \\cup B$ 包含的基本事件个数为 11 个,所以事件 $A \\cup B$ 发生的概率为 $\\frac{11}{20}$."} {"id": "14384", "image": [], "answer": "(1) $(0,+\\infty)$;\n(2) $(0,27]$", "solution": "null", "level": "高一", "question": "求下列函数的值域:\n\n(1) $y=2^{1-x}$;\n\n(2) $y=\\left(\\frac{1}{3}\\right)^{2 x-1}, x \\in[-1,+\\infty)$.", "options": [], "subject": "解析几何", "analysis": "【详解】\n\n(1) 指数函数 $f(x)=2^{x}$ 在 $R$ 上单调递增,\n\n$1-x \\in \\mathrm{R}, \\therefore$ 函数 $y=2^{1-x}$ 的值域 $(0,+\\infty)$\n\n(2)指数函数 $f(x)=\\left(\\frac{1}{3}\\right)^{x}$ 在 $R$ 上单调递减,\n\n$\\because x \\in[-1,+\\infty), \\quad \\therefore 2 x-1 \\in[-3,+\\infty)$,\n\n$\\therefore$ 函数 $y=\\left(\\frac{1}{3}\\right)^{2 x-1}$ 当 $x \\in[-1,+\\infty)$ 时, 值域 $(0,27]$.\n\n【点睛】\n\n本小题主要考查指数型函数的值域的求法, 属于基础题."} {"id": "14385", "image": [], "answer": "(1) $f(x)=4^{x}$;\n(2) $(2,+\\infty)$", "solution": "null", "level": "高一", "question": "已知函数 $f(x)=a^{x} \\quad(a>0$ 且 $a \\neq 1)$ 的图像经过点 $(2,16)$.\n\n(1) 求函数 $f(x)$ 的解析式;\n\n(2) 若 $f(2 m+5)0$ 且 $a \\neq 1)$ 的图像经过点 $(2,16)$, 即 $a^{2}=16$, 故 $a=4$, 故 $f(x)=4^{x}$.\n\n(2) $f(x)=4^{x}$ 函数单调递增, $f(2 m+5)0)$, 则 $f(t)=t-t^{2}$.\n\n$\\because x \\in[0,1], \\therefore t \\in[1,2]$. 当 $t=1$ 时, 取最大值, 最大值为 $1-1=0$.\n\n考点: 1、函数表达式的求法; 2、函数的奇偶性; 3、函数的最值."} {"id": "15253", "image": ["7577.jpg", "7578.jpg"], "answer": "(1)解:根据题意摩天轮从最低点开始, $15 \\mathrm{~min}$ 后达到最高点,\n\n则 $30 \\mathrm{~min}$ 转一圈, 所以摩天轮的角速度为 $\\omega=\\frac{2 \\pi}{30}=\\frac{\\pi}{15}$.\n\n则 $t \\min$ 时, 人在点 $C$ 处, 则此时转过的角度为 $\\theta=\\frac{\\pi}{15} t$.\n\n所以 $y=\\frac{167}{2}-\\frac{153}{2} \\cos \\frac{\\pi}{15} t(t \\geq 0)$.\n\n(2) 解: 登上摩天轮到旋转一周,\n\n则 $0 \\leq t \\leq 30$, 人与地面距离大于 $\\frac{181}{4} \\mathrm{~m}$,\n\n即 $y=\\frac{167}{2}-\\frac{153}{2} \\cos \\frac{\\pi}{15} t>\\frac{181}{4}$, 所以 $\\cos \\frac{\\pi}{15} t<\\frac{1}{2}$,\n\n由 $0 \\leq t \\leq 30$, 解得 $5\n故有 20 分钟人与地面距离大于 $\\frac{181}{4} \\mathrm{~m}$.", "solution": "null", "level": "高一", "question": "中国第一高摩天轮“南昌之星摩天轮”高度为 $160 \\mathrm{~m}$, 其中心 $O$ 距地面 $O B=\\frac{167}{2} \\mathrm{~m}$, 半径为 $O C=\\frac{153}{2} \\mathrm{~m}$, 若某人从最低点 $D$ 处登上摩天轮, 摩天轮匀速旋转, 那么此人与地面的距离将随时间 $t$ 变化, $t=15 \\mathrm{~min}$ 后达到最高点, 从登上摩天轮时开始计时.\n\n\n\n(1) 求出人与地面距离 $\\mathrm{y}$ 与时间 $\\mathrm{t}$ 的函数解析式;\n\n(2) 从登上摩天轮到旋转一周过程中, 有多长时间人与地面距离大于 $\\frac{181}{4} \\mathrm{~m}$.", "options": [], "subject": "解析几何", "analysis": "(1) 计算 $\\omega=\\frac{2 \\pi}{30}=\\frac{\\pi}{15}$, 得到 $t \\min$ 时, 转过的角度为 $\\theta=\\frac{\\pi}{15} t$, 得到解析式. (2) 解不等式 $y=\\frac{167}{2}-\\frac{153}{2} \\cos \\frac{\\pi}{15} t>\\frac{181}{4}$ 得到答案."} {"id": "14961", "image": [], "answer": "$a=1$", "solution": "null", "level": "高一", "question": "已知集合 $A=\\left\\{3,4, a^{2}-3 a-1\\right\\}, B=\\{2 a,-3\\}, A \\cap B=\\{-3\\}$, 求实数 $a$ 的值.", "options": [], "subject": "代数", "analysis": "解: 由题意得 $a^{2}-3 a-1=-3$, 解得 $a=1$ 或 $a=2$,\n\n当 $a=1$ 时, $A=\\{3,4,-3\\}, B=\\{2,-3\\}$, 满足要求;\n当 $a=2$ 时, $A=\\{3,4,-3\\}, B=\\{4,-3\\}$, 不满足要求,\n\n综上得: $a=1$。\n\n由 $-3 \\in A$, 得 $\\mathrm{a}^{2}-3 \\mathrm{a}-1=-3$, 解出 $\\mathrm{a}$ 的值, 检验得出答案。"} {"id": "14962", "image": [], "answer": "(1) 解: 若 $\\mathrm{a} \\in \\mathrm{A}$, 则 $\\frac{1}{1-\\mathrm{a}} \\in \\mathrm{A}$.\n\n又 $\\because 2 \\in \\mathrm{A}, \\therefore \\frac{1}{1-2}=-1 \\in \\mathrm{A}$.\n\n$\\because-1 \\in A, \\therefore \\frac{1}{1-(-1)}=\\frac{1}{2} \\in \\mathrm{A}$.\n\n$\\because \\frac{1}{2} \\in \\mathrm{A}, \\quad \\therefore \\frac{1}{1-\\frac{1}{2}}=2 \\in \\mathrm{A}$.\n\n$\\therefore \\mathrm{A}$ 中另外两个元素为 $-1, \\frac{1}{2}$.\n\n(2) 解: 若 $A$ 为单元素集, 则 $a=\\frac{1}{1-a}$,\n\n即 $a^{2}-a+1=0$, 方程无解.\n\n$\\therefore a \\neq \\frac{1}{1-a}, \\therefore$ 集合 $\\mathrm{A}$ 不可能是单元素集", "solution": "null", "level": "高一", "question": "设 $A$ 为实数集, 且满足条件: 若 $a \\in A$, 则 $\\frac{1}{1-a} \\in A(a \\neq 1)$.\n\n求证:\n\n(1) 若 $2 \\in A$ ,则 A 中必还有另外两个元素;\n\n(2) 集合 $A$ 不可能是单元素集.", "options": [], "subject": "代数", "analysis": "(1)根据集合 $\\mathrm{A}$ 满足的条件, 将 2 代入逐一求解, 即可求出 $\\mathrm{A}$ 中另外两个元素;\n\n(2)采用反证法, 假设 $\\mathrm{A}$ 为单元素集, 得出矛盾, 即可证明集合 $\\mathrm{A}$ 不可能是单元素集."} {"id": "14963", "image": [], "answer": "(1) $a$ 是集合 $S$ 的元素,(2) $\\mathrm{x}_{1}+\\mathrm{x}_{2} 、 \\mathrm{x}_{1} \\cdot \\mathrm{x}_{2}$ 都属于 $\\mathrm{S}$", "solution": "null", "level": "高一", "question": "设 $\\mathrm{S}=\\{\\mathrm{x} \\mid \\mathrm{x}=\\mathrm{m}+\\mathrm{n} \\sqrt{2}, \\mathrm{~m} 、 \\mathrm{n} \\in \\mathrm{Z}\\}$.\n\n(1) 若 $a \\in Z$, 则 $a$ 是否是集合 $S$ 中的元素?\n\n(2)对 $S$ 中的任意两个 $x_{1} 、 x_{2} ,$ 则 $x_{1}+x_{2} 、 x_{1} \\cdot x_{2}$ 是否属于 $S$ ?", "options": [], "subject": "代数", "analysis": "(1) 解: $a$ 是集合 $S$ 的元素, 因为 $a=a+0 \\times \\sqrt{2} \\in S$;\n\n(2) 解: 不妨设 $x_{1}=m+n \\sqrt{2}, x_{2}=p+q \\sqrt{2}, m 、 n 、 p 、 q \\in Z$.\n\n则 $x_{1}+x_{2}=(m+n \\sqrt{2})+(p+q \\sqrt{2})=(m+n)+(p+q) \\sqrt{2}$,\n\n$\\because \\mathrm{m} 、 \\mathrm{n}, \\mathrm{p} 、 \\mathrm{q} \\in \\mathrm{Z} . \\therefore \\mathrm{p}+\\mathrm{q} \\in \\mathrm{Z}, \\mathrm{m}+\\mathrm{n} \\in \\mathrm{Z} . \\quad \\therefore \\mathrm{x}_{1}+\\mathrm{x}_{2} \\in \\mathrm{S}$,\n\n$\\mathrm{x}_{1} \\cdot \\mathrm{x}_{2}=(\\mathrm{m}+\\mathrm{n} \\sqrt{2}) \\cdot(\\mathrm{p}+\\mathrm{q} \\sqrt{2})=(\\mathrm{mp}+2 \\mathrm{nq})+(\\mathrm{mq}+\\mathrm{np}) \\sqrt{2}, \\mathrm{~m} 、 \\mathrm{n} 、 \\mathrm{p} 、 \\mathrm{q} \\in \\mathrm{z}$.\n\n故 $\\mathrm{mp}+2 \\mathrm{nq} \\in \\mathrm{Z}, \\mathrm{mq}+\\mathrm{np} \\in \\mathrm{Z}$.\n\n$\\therefore \\mathrm{x}_{1} \\cdot \\mathrm{x}_{2} \\in \\mathrm{S}$.\n\n综上, $\\mathrm{x}_{1}+\\mathrm{x}_{2} 、 \\mathrm{x}_{1} \\cdot \\mathrm{x}_{2}$ 都属于 $\\mathrm{S}$.\n\n由于整数 $a$ 能表示为 $a=a+0 \\times 2$ 故 $a \\in S$;\n\n(2)先设 $x_{1}=m+n 2, x_{2}=p+q 2 , m 、 n 、 p 、 q \\in Z$. 再运算 $x_{1}+x_{2}$, 与 $x_{1} \\cdot x_{2}$ 能不能表示为集合 $S$ 的元素的形式.得到证明."} {"id": "14979", "image": [], "answer": "(I) $m \\leq 2$,(II)$3 \\leq m \\leq 4$", "solution": "null", "level": "高一", "question": "已知集合 $A=\\left\\{x \\left\\lvert\\, \\frac{3-2 x}{x+2}>-1\\right.\\right\\}$,\n\n(I) 若 $B \\subseteq A, B=\\{x \\mid m+11$, 得 $-22 \\\\ m+1 \\geq 2 \\\\ 2 m-1 \\leq 5 \\text { 解得: } 2m-6 \\\\ m-6 \\leq-2 \\\\ 2 m-1 \\geq 5\\end{array}\\right.$ 解得:\n\n$3 \\leq m \\leq 4$.(1) 求出集合 $\\mathrm{A}$, 利用子集关系, 通过 $\\mathrm{B}$ 是否为空集, 列出不等式组求解即可. (2) $\\mathrm{A} \\subseteq \\mathrm{B}$, $B=\\{x \\mid m-62-a$ 即 $a>1$ 时, $A=\\{x \\mid 2-a1 \\\\ 2-a \\leq 0 \\\\ a \\geq 4\\end{array}\\right.$,解得 $a \\geq 4$,\n\n综上所述, $a \\in(-\\infty,-2] \\cup[4,+\\infty)$\n\n(1)当 $\\mathrm{a}=3$ 时,求出集合 $A$,由集合的交集运算即可得解;(2)由 $\\mathrm{A} \\cup \\mathrm{B}=\\mathrm{A}$ 知 $B \\subseteq A$,对 $a, 2-a$的关系进行讨论并利用并集的性质列出不等式组,求出 $\\mathrm{a}$ 的取值范围."} {"id": "14982", "image": ["7469.jpg"], "answer": "(1) $a=5$(2)$a=-2$,(1) 解: 由 $B=\\left\\{x \\mid x^{2}-5 x+6=0\\right\\}$ 得 $B=\\{2,3\\}$,\n\n因为 $A \\cap B=A \\cup B$, 所以 $A=B$,\n\n所以 $\\left\\{\\begin{array}{l}2+3=a \\\\ 2 \\times 3=a^{2}-19, \\text { 解得 } a=5\\end{array}\\right.$\n\n(2)解: 由 $C=\\left\\{x \\mid x^{2}+2 x-8=0\\right\\}$ 得 $C=\\{2,-4\\}$,\n\n因为 $A \\cap B \\neq \\phi, A \\cap C=\\phi$, 所以 $3 \\in A$,\n\n所以 $3^{2}-3 a+a^{2}-19=0$,\n\n即 $a^{2}-3 a-10=0$, 解得 $a=5$ 或 $a=-2$,\n\n当 $a=5$ 时, $A=\\{2,3\\}$ 与 $A \\cap C=\\varnothing$ 矛盾,\n\n当 $a=-2$ 时, $A=\\{-5,3\\}$, 满足题意,\n\n$\\therefore a=-2$,\n\n故得解.", "solution": "null", "level": "高一", "question": "集合 $A=\\left\\{x \\mid x^{2}-a x+a^{2}-19=0\\right\\}, \\quad B=\\left\\{x \\mid x^{2}-5 x+6=0\\right\\}, \\quad C=\\left\\{x \\mid x^{2}+2 x-8=0\\right\\}$.\n\n(1) 若 $A \\cap B=A \\cup B$, 求 $a$ 的值;\n\n(2) 若 $A \\cap B \\neq \\varnothing, A \\cap C=\\varnothing$, 求 $a$ 的值.\n\n", "options": [], "subject": "代数", "analysis": "(1)先求出 $\\mathrm{B}$ 集合,由 $A \\cap B=A \\cup B$ 得出 $A=B$, 再由韦达定理求得 $\\mathrm{a}$; (2)求出集合 C, 由 $A \\cap B \\neq \\phi, A \\cap C=\\phi$ 得出 $3 \\in A$, 从而求得 $\\mathrm{a}$ 的值, 再代入集合 $\\mathrm{A}$ 中验证是否满足题意,得解."} {"id": "14998", "image": [], "answer": "(1) $\\left(C_{U^{A}} A\\right) \\cap\\left(C_{U} B\\right)=\\{x \\mid x<2$ 或 $x \\geq 10\\}$.\n\n(2)$a$ 的取值范围是 $a<3$.", "solution": "null", "level": "高一", "question": "全集 $U=R$, 若集合 $A=\\{x \\mid 3 \\leq x<10\\}, B=\\{x(x-2)(x-7) \\leq 0\\}$.\n\n(1) $A \\cup B,\\left(C_{U} A\\right) \\cap\\left(C_{U} B\\right)$;\n\n(2) 若集合 $C=\\{x \\mid x>a\\}, A \\subseteq C$, 求 $a$ 的取值范围.", "options": [], "subject": "代数", "analysis": "(1) 解: 由 $(x-2)(x-7) \\leq 0$ 解得 $B=\\lfloor 2,7\\rfloor$.\n\n故 $A \\cup B=\\{x \\mid 2 \\leq x<10\\}, C_{U} A=\\{x \\mid x<3$ 或 $x \\geq 10\\}, C_{U} B=\\{x \\mid x<2$ 或 $x>7\\}$ ,所以 $\\left(C_{U^{A}} A\\right) \\cap\\left(C_{U} B\\right)=\\{x \\mid x<2$ 或 $x \\geq 10\\}$.\n\n(2) 解: 由于 $A \\subseteq C$, 所以 $a<3$, 故 $a$ 的取值范围是 $a<3$.\n\n【分析】(1) 解一元二次不等式求得集合 $B$, 根据并集、交集和补集的概念和运算,求得所求(2)根据子集的概念列不等式, 解不等式求得 $a$ 的取值范围."} {"id": "14999", "image": [], "answer": "(1) $A \\cup B=\\lfloor-1,4)$\n\n(2) $a \\leq-4$ 或 $a \\geq 2$.", "solution": "null", "level": "高一", "question": "已知集合 $A=\\{y \\mid y=3 x-1,0 \\leq x \\leq 1\\}, B=\\{x(x-a)((x-(a+3))\\rfloor<0\\}$.\n\n(1) 若 $a=1$, 求 $A \\cup B$;\n\n(2) 若 $A \\cap B=\\phi$, 求实数 $a$ 的取值范围.", "options": [], "subject": "代数", "analysis": "(1) 解: $\\because$ 集合 $A$ 是函数 $y=3 x-1(0 \\leq x \\leq 1)$ 的值域\n\n$\\therefore A=\\lfloor-1,2\\rfloor$, 易知 $B=(a, a+3)$\n\n若 $a=1$, 则 $B=(1,4)$, 结合数轴知 $A \\cup B=\\lfloor-1,4)$\n\n(2) 解: 若 $A \\cap B=\\varnothing$, 得 $a \\geq 2$ 或 $a+3 \\leq-1$, 即 $a \\leq-4$ 或 $a \\geq 2$.\n\n【分析】(1) 当 $a=1$ 时, 分别求出集合 $\\mathrm{A}, \\mathrm{B}$, 即可求出它们的交集;\n\n(2) 结合数轴可得, 若 $A \\cap B=\\phi$ 实数 $a$ 的取值范围."} {"id": "15020", "image": [], "answer": "(1) 解: 由题: ||$x \\mid-1 \\neq 0$,\n\n解得: $-2 \\leq x<-1$ 或 $-10 \\\\ x-2 \\neq 0,\\end{array}\\right.$ 解得: $x>1$ 且 $x \\neq 2$,\n\n所以其定义域为: $(1,2) \\cup(2,+\\infty)$.", "solution": "null", "level": "高一", "question": "求下列函数定义域\n\n(1) $y=\\sqrt{4-x^{2}}+\\frac{1}{|x|-1}$\n\n(2) $y=\\frac{1}{\\sqrt{x-1}}+(x-2)^{0}$", "options": [], "subject": "代数", "analysis": "(1)根据题意解不等式组 $\\left\\{\\left.\\begin{array}{l}4-x^{2} \\geq 0\\end{array} \\right\\rvert\\,-1 \\neq 0\\right.$, 求出解集即可; (2)根据题意解不等式组 $\\left\\{\\begin{array}{l}x-1>0 \\\\ x-2 \\neq 0,\\end{array}\\right.$求出解集即可."} {"id": "15021", "image": [], "answer": "(1) 解: $\\because-2<-1$,\n\n$\\therefore f(-2)=2 \\times(-2)+3=-1, \\quad f(f(-2))=f(-1)=2$,\n\n$\\therefore f(f(f(-2)))=f(2)=1+\\frac{1}{2}=\\frac{3}{2}$;\n\n(2)解: 当 $a>1$ 时, $f(a)=1+\\frac{1}{a}=\\frac{3}{2}, \\therefore a=2>1$;\n\n当 $-1 \\leq a \\leq 1$ 时, $f(a)=a^{2}+1=\\frac{3}{2}, \\therefore a= \\pm \\frac{\\sqrt{2}}{2} \\in[-1,1]$;\n\n当 $a<-1$ 时, $f(a)=2 a+3=\\frac{3}{2}, \\therefore a=-\\frac{3}{4}>-1$ (舍去).\n综上, $a=2$ 或 $a= \\pm \\frac{\\sqrt{2}}{2}$.", "solution": "null", "level": "高一", "question": "已知函数 $f(x)= \\begin{cases}1+\\frac{1}{x}, & x>1 \\\\ x^{2}+1, & -1 \\leq x \\leq 1 \\\\ 2 x+3, & x<-1\\end{cases}$\n\n(1) 求 $f(f(f(-2)))$ 的值;\n\n(2) 若 $f(a)=\\frac{3}{2}$, 求 $a$.", "options": [], "subject": "代数", "analysis": "(1)根据分段函数的表达式,即可得到函数值;(2)根据函数的表达式及 $f(a)=\\frac{3}{2}$, 即可得到 $a$."} {"id": "15022", "image": ["7480.jpg"], "answer": "(1) 解: 令 $f(x)=\\sqrt{2 x-1}=t \\Rightarrow x=\\frac{t^{2}+1}{2}(t \\geq 0)$,\n\n因此有:\n\n$f(t)=t-\\frac{t^{2}+1}{2}=-\\frac{1}{2} t^{2}+t-\\frac{1}{2}=-\\frac{1}{2}(t-1)^{2} \\because t \\geq 0 \\therefore f(t) \\leq 0$,\n\n\n\n(2) 解: $g(x)=\\frac{3 x-2}{5 x+1}=\\frac{3}{5} \\cdot \\frac{x-\\frac{2}{3}}{x+\\frac{1}{5}}=\\frac{3}{5}\\left(1-\\frac{\\frac{13}{15}}{x+\\frac{1}{5}}\\right) \\neq \\frac{3}{5}$,\n\n所以函数 $g(x)=\\frac{3 x-2}{5 x+1}$ 的值域为: $\\left(-\\infty, \\frac{3}{5}\\right) \\cup\\left(\\frac{3}{5},+\\infty\\right)$", "solution": "null", "level": "高一", "question": "求下列函数的值域:\n\n(1) $f(x)=\\sqrt{2 x-1}-x$;\n\n(2) $g(x)=\\frac{3 x-2}{5 x+1}$.", "options": [], "subject": "代数", "analysis": "(1)根据函数的解析式的特征, 利用换元法求解函数的值域; (2)根据函数的解析式的特征,进行常变量分离即可求出函数的值域."} {"id": "15036", "image": ["7492.jpg"], "answer": "解: $\\mathrm{f}(\\mathrm{x})=$\n\n$$\n\\left\\{\\begin{array}{c}\n\\frac{2}{x}, x<-1 \\\\\n-2,-1 \\leq x<0 \\\\\n3 x-2, x \\geq 0\n\\end{array}\\right.\n$$\n\n函数的图象为:\n\n", "solution": "null", "level": "高一", "question": "设 $\\mathrm{f}(\\mathrm{x})=\\left\\{\\begin{array}{c}\\frac{2}{x}, x<-1 \\\\ -2,-1 \\leq x<0 \\\\ 3 x-2, x \\geq 0\\end{array}\\right.$, 作出函数 $\\mathrm{f}(\\mathrm{x})$ 的图象.", "options": [], "subject": "代数", "analysis": "在坐标系中直接画出函数的图象即可."} {"id": "15038", "image": ["7493.jpg"], "answer": "(1)解:设 $\\mathrm{x}+1=\\mathrm{t}$, 则 $\\mathrm{x}=\\mathrm{t}-1$,\n\n$\\therefore \\mathrm{f}(\\mathrm{t})=3(\\mathrm{t}-1)+2=3 \\mathrm{t}-1$,\n\n$\\therefore \\mathrm{f}(\\mathrm{x})=3 \\mathrm{x}-1$.\n\n(2)解: 因为 $f(x)$ 是一次函数, 可设 $\\left.f(x)=a x+b_{(} a \\neq 0\\right)$,\n\n所以有 $3[a(x+1)+b]-2[a(x-1)+b]=2 x+17$,\n\n即 $a x+(5 a+b)=2 x+17$,\n\n因此应有 $\\left\\{\\begin{array}{l}a=2 \\\\ 5 a+b=17,\\end{array}\\left\\{\\begin{array}{l}a=2 \\\\ b=7 \\text { 解得 }\\end{array}\\right.\\right.$\n\n故 $f(x)$ 的解析式是 $f(x)=2 x+7$.\n\n(3)解:因为 $2 f(x)+f\\left(\\frac{1}{x}\\right)=3 x$ ,(1)\n\n将 $x$ 用 $\\frac{1}{x}$ 替换, 得 $2 f\\left(\\frac{1}{x}\\right)+f(x)=\\frac{3}{x}$, (2)\n\n由(1)(2)解得 $f(x)=2 x-\\frac{1}{x}(x \\neq 0)$,\n\n即 $f(x)$ 的解析式是 $f(x)=2 x-\\frac{1}{x}(x \\neq 0)$.", "solution": "null", "level": "高一", "question": "根据条件求下列各函数的解析式:\n\n(1) 已知函数 $\\mathrm{f}(\\mathrm{x}+1)=3 \\mathrm{x}+2$, 则 $\\mathrm{f}(\\mathrm{x})$ 的解析式;\n\n(2)已知 $f(x)$ 是一次函数, 且满足 $3 f(x+1)-2 f(x-1)=2 x+17$, 求 $f(x)$ 的解析式;\n\n", "options": [], "subject": "代数", "analysis": "(1)利用换元法即可求出函数 $\\mathrm{f}$ (x)的解析式;(2)设一次函数 $f(x)=a x+b(a \\neq 0)$,\n\n代入已知比较系数可得 $\\mathrm{a}$ 和 $\\mathrm{b}$ 的方程组, 解方程组可得结果; (3) 将 $x$ 用 $\\frac{1}{x}$ 替换, 构造方程组即可得到 $f(x)$ 的解析式."} {"id": "15056", "image": [], "answer": "(1) 解: $\\because$ 函数 $f(x)=x^{2}-2 x$ 的图象开口向上, 对称轴为直线 $x=1$,\n\n所以函数 $y=f(x)$ 的减区间为 $(-\\infty, 1]$ ,增区间为 $(1,+\\infty)$,\n\n则函数 $y=g(x)$ 的增区间为 $\\lfloor 2,4\\rfloor$;\n\n(2)解: 由 (1) 知, 函数 $y=f(x)$ 在 $x=1$ 处取得最小值 -1 ,\n\n由于函数 $y=g(x)_{\\text {在定义域 }}[2,4\\rfloor$ 上单调递增,\n\n则函数 $y=g(x)$ 在 $x=2$ 处取得最小值 0 .", "solution": "null", "level": "高一", "question": "已知函数 $f(x)=x^{2}-2 x, g(x)=x^{2}-2 x(x \\in\\lfloor 2,4\\rfloor)$.\n\n(1) 求 $f(x) 、 g(x)$ 的单调区间;\n\n(2) 求 $f(x) 、 g(x)$ 的最小值.", "options": [], "subject": "代数", "analysis": "(1) 分析二次函数 $f(x)=x^{2}-2 x$ 图象的开口方向和对称轴, 可得出函数 $y=f(x)$ 的减区间和增区间, 以及函数 $y=g(x)$ 的增区间; (2) 由函数 $y=f(x)$ 和函数 $y=g(x)$ 的单调性可得出这两个函数的最小值."} {"id": "15057", "image": ["7499.jpg", "7500.jpg"], "answer": "(1) 解: 函数 $f(x)$ 在 $\\lfloor 3,5\\rfloor$ 上为增函数, 证明如下:\n\n设 $x_{1}, x_{2}$ 是 $\\lfloor 3,5\\rfloor$ 上的任意两个实数, 且 $x_{1}0, x_{2}+1>0$,\n\n$\\therefore f\\left(x_{1}\\right)-f\\left(x_{2}\\right)<0$, 即 $f\\left(x_{1}\\right)\n\n\n\n故得解.", "solution": "null", "level": "高一", "question": "已知函数 $f(x)=\\frac{2 x-1}{x+1}, x \\in[3,5]$.\n\n(1) 判断 $f(x)$ 在区间 $\\lfloor 3,5\\rfloor$ 上的单调性并证明;\n\n(2) 求 $f(x)$ 的最大值和最小值.", "options": [], "subject": "代数", "analysis": "(1) 利用函数的单调性的定义, 设 $x_{1}, x_{2}$, 判断 $f\\left(x_{1}\\right)-f\\left(x_{2}\\right)$ 的正负, 证明出函数 $f(x)$ 在 $\\lfloor 3,5\\rfloor$ 上的单调性为增函数; (2) 由(1)得出的函数的单调性为单调递增, 从而得出函数 $f(x)$ 在区间 $\\lfloor 3,5\\rfloor$ 上的最大值为 $f(5)$ 与最小值为 $f(3)$, 求出其函数值得最值."} {"id": "15058", "image": [], "answer": "(I) 证明: 任取 $x_{1}, x_{2} \\in R$,\n\n设 $x_{1}\\left(x_{2}, \\therefore x_{2}-x_{1}\\right), \\therefore f\\left(x_{2}-x_{1}\\right)>1$,\n\n则 $f\\left(x_{2}\\right)-f\\left(x_{1}\\right)=f\\left[\\left(x_{2}-x_{1}\\right)+x_{1}\\right]-f\\left(x_{1}\\right)=f\\left(x_{2}-x_{1}\\right)+f\\left(x_{1}\\right)-1-f\\left(x_{1}\\right)$\n\n$=f\\left(x_{2}-x_{1}\\right)-1>0, \\quad \\therefore f\\left(x_{2}\\right)>f\\left(x_{1}\\right)$\n\n$\\therefore f(x)$ 为 $R$ 上的增函数.\n\n(II)解:不等式 $f(x)+f\\left(2 x-x^{2}\\right)>2$,\n\n可化为: $f(x)+f\\left(2 x-x^{2}\\right)-1>1$,\n\n即 $f\\left(x+2 x-x^{2}\\right)>1$,\n\n$f(0+0)=2 f(0)-1, \\therefore f(0)=1$,\n\n故不等式化为 $f\\left(3 x-x^{2}\\right)>f(0)$,\n\n$\\because f(x)$ 为 $R$ 上的增函数, $3 x-x^{2}>0$, 解得 $00$ 时, $f(x)>1$, 且对任意的 $x, y \\in R$, 都有 $f(x+y)=f(x)+f(y)-1$.\n\n(I) 求证: $f(x)$ 是 $\\mathrm{R}$ 上的增函数;\n\n(II) 求不等式 $f(x)+f\\left(2 x-x^{2}\\right)>2$ 的解集.", "options": [], "subject": "代数", "analysis": "(I) 任取 $x_{1}, x_{2} \\in R$, 且设 $x_{1}0$ 时, $f(x)>1$, 以及 $x, y \\in R$, 都有 $f(x+y)=f(x)+f(y)-1$, 可以证明 $f\\left(x_{2}\\right)>f\\left(x_{1}\\right)$, 即可证明 $f(x)$ 是 $\\mathrm{R}$ 上的增函数; (II)利用抽象函数的性质及 $f(x)$ 的单调性, 可以得到 $3 x-x^{2}>0$, 求解即可."} {"id": "15073", "image": [], "answer": "(1) 解: 函数 $f(x)$ 定义域为 $R$, 关于原点对称,\n\n又 $f(-x)=2(-x)^{4}+3(-x)^{2}=2 x^{4}+3 x^{2}=f(x)$,\n\n所以函数 $f(x)$ 为偶函数.\n\n(2) 解: 函数 $f(x)$ 定义域为 $\\{x \\mid x \\neq 0\\}$, 关于原点对称,\n\n又 $f(-x)=\\frac{1}{-x}+(-x)=-\\left(\\frac{1}{x}+x\\right)=-f(x)$,\n\n所以函数 $f(x)$ 为奇函数.", "solution": "null", "level": "高一", "question": "判断下列函数的奇偶性\n\n(1) $f(x)=2 x^{4}+3 x^{2}$\n\n(2) $f(x)=\\frac{1}{x}+x$", "options": [], "subject": "代数", "analysis": "(1) 先求定义域为 $R$, 再判断 $f(-x)$ 与 $f(x)$ 的关系, 即可得到答案; (2) 先求定义域为 $\\{x \\mid x \\neq 0\\}$, 再判断 $f(-x)$ 与 $f(x)$ 的关系, 即可得到答案."} {"id": "15074", "image": ["7507.jpg"], "answer": "由题意得 $\\left\\{\\begin{array}{l}-1 \\leq 1-a^{2} \\leq 1 \\\\ -1 \\leq 1-a \\leq 1,\\end{array}\\right.$\n\n解得 $\\left\\{\\begin{array}{l}0 \\leq a^{2} \\leq 2 \\\\ 0 \\leq a \\leq 2\\end{array}\\right.$, 即 $0 \\leq a \\leq \\sqrt{2}$\n\n由 $f\\left(1-a^{2}\\right)+f(1-a)<0$, 得 $f(1-a)<-f\\left(1-a^{2}\\right)$,\n\n$\\because$ 函数 $y=f(x)$ 是奇函数,\n\n$\\therefore-f\\left(1-a^{2}\\right)=f\\left(a^{2}-1\\right), \\therefore f(1-a)\n\n$\\therefore 1-a>a^{2}-1$, 即 $a^{2}+a-2<0$,\n\n解得 $-20, x_{1} x_{2}+4>0$,\n\n所以 $f\\left(x_{1}\\right)-f\\left(x_{2}\\right)<0, f\\left(x_{1}\\right)\n调性求得函数的最大值."} {"id": "15077", "image": ["7509.jpg", "7510.jpg"], "answer": "(1) 解: 设 $x<0,-x>0$\n\n$f(-x)=2(-x)^{2}-4(-x)=2 x^{2}+4 x$,\n\n$\\because f(x)$ 是奇函数,\n\n$\\therefore f(x)=-f(-x)=-2 x^{2}-4 x$\n\n$\\therefore f(x)= \\begin{cases}2 x^{2}-4 x & x \\geq 0 \\\\ -2 x^{2}-4 x & x<0,\\end{cases}$\n\n图象如图所示:\n\n\n$\\left\\{\\begin{array}{l}x \\geq 0 \\\\ 2 x^{2}-4 x>0\\end{array}\\right.$ 或 $\\left\\{\\begin{array}{l}x<0 \\\\ -2 x^{2}-4 x<0\\end{array}\\right.$\n\n解得: $x>2$ 或 $x<-2$,\n\n$\\therefore$ 不等式的解集 $(-\\infty,-2) \\cup(2,+\\infty)$\n\n(2) 解: 由题意可知, $\\lfloor-1 a-2\\rfloor$ 是函数单调递减区间的子集,\n\n根据图象可知 $-1\n\n(1)求函数 $f(x)$ 的表达式,画出函数 $f(x)$ 的图像, 并求不等式 $x f(x)>0$ 的解集;\n\n(2) 若函数 $f(x)$ 在区间 $[-1, a-2]$ 上单调递减, 求实数 $a$ 的取值范围.", "options": [], "subject": "代数", "analysis": "(1)根据函数的奇偶性, 确定函数的表达式, 作出图像, 数形结合即可求出不等式的解集;\n\n(2) 根据函数的单调性, 解不等式即可求出实数 $\\mathrm{a}$ 的取值范围."} {"id": "15078", "image": [], "answer": "(1) 解: $f(x)$ 是奇函数, 证明如下: $\\because f(x)$ 是定义域为 $R$,\n\n且 $f(-x)=(-x)^{3}+(-x)=-x^{3}-x=-f(x), \\therefore f(x)$ 是奇函数\n\n(2) 解: $f(m+1)+f(2 m-3)<0$ 化为 $f(m+1)<-f(2 m-3)$,\n\n因为 $f(x)$ 是奇函数, $\\therefore-f(2 m-3)=f(-2 m+3)$,\n\n所以不等式化为 $f(m+1)1, \\quad \\therefore \\mathrm{y}=1.8^{\\mathrm{x}}$ 在 $\\mathrm{R}$ 上为增函数, $\\therefore 1.8^{2.2}<1.8^{3}$\n\n(2) 解: $\\because y=0.7^{x}$ 在 $\\mathrm{R}$ 上为减函数,\n\n又 $\\because-0.3>-0.4, \\quad \\therefore 0.7^{-0.3}<0.7^{-0.4}$\n\n(3)解: $\\because 1.9^{0.4}>1.9^{0}=1 ; 0.9^{2.4}<0.9^{0}=1, \\therefore 1.9^{0.4}>0.9^{2.4}$", "solution": "null", "level": "高一", "question": "比较下列各题中两个值的大小.\n\n(1) $1.8^{2.2}, \\quad 1.8^{3}$;\n\n(2) $0.7^{-0.3}, \\quad 0.7^{-0.4}$;\n\n(3) $1.9^{0.4}, \\quad 0.9^{2.4}$.", "options": [], "subject": "代数", "analysis": "(1)由于两式底数相同为 $1.8>1$, 由指数函数的单调递增比较大小;\n\n(2)由于两式底数相同为 $0.7<1$, 由指数函数的单调递减比较大小;\n\n(3)由于两式底数相同为 $0.9<1$,由指数函数的单调递减比较大小."} {"id": "15118", "image": [], "answer": "(1)解: 将点 $(2,4)$ 代入函数表达式得 $f(2)=a^{2}=4$, 解得 $a=2$.\n\n(2) 解: 由 (1) 知 $f(x)=2^{x}$,\n\n故函数 $f(x)_{\\text {在 }}[0,1\\rfloor$ 上是单调递增函数,\n\n故最大值为 $f(1)=2^{1}=2$, 最小值为 $f(0)=2^{0}=1$.", "solution": "null", "level": "高一", "question": "已知函数 $f(x)=a^{x}(a>0$ 且 $a \\neq 1)$ 经过点 (2,4).\n\n(1) 求 $a$ 的值;\n\n(2) 求 $f(x)$ 在 $[0,1]$ 上的最大值与最小值.", "options": [], "subject": "代数", "analysis": "(1)将点 $(2,4)$ 代入函数表达式,由此求得 $a$ 的值.(2)根据指数函数单调性, 求得函数 $f(x)$ 的最大值和最小值."} {"id": "15119", "image": [], "answer": "解: 设 $t=\\left(\\frac{1}{2}\\right)^{x}>0$,\n\n所以原不等式转化为 $t^{2}-2 t-8<0$,\n\n解得 $-2-2$,\n\n故不等式的解集为: $(-2,+\\infty)$.", "solution": "null", "level": "高一", "question": "解关于 $x$ 的不等式: $\\left(\\frac{1}{4}\\right)^{x}-2^{-x+1}-8<0$.", "options": [], "subject": "代数", "analysis": "设 $t=\\left(\\frac{1}{2}\\right)^{x}>0$, 将所求不等式转化为关于 $t$ 的二次不等式, 求出 $t$ 的范围, 即 $\\left(\\frac{1}{2}\\right)^{x}$ 的范围, 再根据 $y=\\left(\\frac{1}{2}\\right)^{x}$ 单调性, 求出 $x$ 的取值范围."} {"id": "15120", "image": ["7518.jpg"], "answer": "解: 令 $t=a^{x}(a>0, a \\neq 1)$,\n\n则原函数化为 $f(t)=t^{2}+2 t-1=(t+1)^{2}-2(t>0)$,\n\n(1)当 $01$ 时, $x \\in[-1,1] t=a^{x} \\in\\left[\\frac{1}{a}, a\\right]$\n\n此时 $f(t)$ 在区间 $\\left[\\frac{1}{a}, a\\right]_{\\text {上为增函数, }}$\n\n所以 $f(t)_{\\text {max }}=f(a)=(a+1)^{2}-2=14$,\n\n所以 $a=-5$ (舍) 或 $a=3$,\n\n综上所述, $a=\\frac{1}{3}$ 或 $a=3$.", "solution": "null", "level": "高一", "question": "设 $a>0$ 且 $a \\neq 1$, 函数 $y=a^{2 x}+2 a^{x}-1$ 在区间 $\\lfloor-1,1\\rfloor$ 上的最大值是 14 , 求实数 $a$ 的值.24.已知函数 $f(x)=a^{x}, g(x)=\\left(\\frac{1}{a}\\right)^{x}(a>0$ 且 $a \\neq 1), \\quad f(-1)=\\frac{1}{2}$.\n\n\n\n(1) 求函数 $f(x)$ 和 $g(x)$ 的解析式;\n\n(2) 在同一坐标系中画出函数 $f(x)$ 和 $g(x)$ 的图象;\n\n(3)如果 $f(x)0, a \\neq 1)$, 将原函数化为 $f(t)=t^{2}+2 t-1=(t+1)^{2}-2(t>0)$, (1) 当\n\n$01$ 时, 利用单调性得 $f(t)_{\\max }=f(a)=14$, 解得 $a=3$."} {"id": "15140", "image": [], "answer": "解:原方程可化为 $2(\\lg x)^{2}-4 \\lg x+1=0$,\n\n设 $t=\\lg x, \\quad$ 则方程化为 $2 t^{2}-4 t+1=0$,\n\n所以 $t_{1}+t_{2}=2, t_{1} \\cdot t_{2}=\\frac{1}{2}$.\n\n又因为 $a 、 b$ 是方程 $2(\\lg x)^{2}-\\lg x^{4}+1=0$ 的两个实根,\n\n所以 $t_{1}=\\lg a, t_{2}=\\lg b$,\n\n即 $\\lg a+\\lg b=2, \\lg a \\cdot \\lg b=\\frac{1}{2}$.\n所以 $\\lg (a b) \\cdot\\left(\\log _{a} b+\\log _{b} a\\right)=(\\lg a+\\lg b) \\cdot\\left(\\frac{\\lg b}{\\lg a}+\\frac{\\lg a}{\\lg b}\\right)$\n\n$=(\\lg a+\\lg b) \\cdot \\frac{\\lg ^{2} b+\\lg ^{2} a}{\\lg a \\lg b}=(\\lg a+\\lg b) \\cdot \\frac{(\\lg a+\\lg b)^{2}-2 \\lg a \\lg b}{\\lg a \\lg b}$\n\n$=2 \\times \\frac{2^{2}-2 \\times \\frac{1}{2}}{\\frac{1}{2}}=12$,\n\n即 $\\lg (a b) \\cdot\\left(\\log _{a} b+\\log _{b} a\\right)=12$.", "solution": "null", "level": "高一", "question": "若 $a 、 b$ 是方程 $2(\\lg x)^{2}-\\lg x^{4}+1=0$ 的两个实根, 求 $\\lg (a b) \\cdot\\left(\\log _{a} b+\\log _{b} a\\right)$ 的值.", "options": [], "subject": "代数", "analysis": "利用对数的运算法则, 结合韦达定理, 即可得出结论。"} {"id": "15141", "image": [], "answer": "(1)解: 原式\n\n$$\n=\\left[\\left(\\frac{8}{7}\\right)^{2}\\right]^{-\\frac{1}{2}}+2^{\\log _{2} 3^{2}}+\\lg \\frac{\\frac{1}{4}}{25}=\\frac{7}{8}+2^{\\log _{2} 3}-\\lg 100=\\frac{7}{8}+3-2=\\frac{15}{8}\n$$\n\n(2) 解: 由换底公式得 $\\log _{5} 2=\\frac{1}{\\log _{2} 5}=\\frac{1}{a}$, 又 $b=\\log _{5} 7$,\n\n$\\log _{14} 56=\\frac{\\log _{5} 56}{\\log _{5} 14}=\\frac{\\log _{5}\\left(7 \\times 2^{3}\\right)}{\\log _{5}(7 \\times 2)}=\\frac{\\log _{5} 7+3 \\log _{5} 2}{\\log _{5} 7+\\log _{5} 2}=\\frac{b+\\frac{3}{a}}{b+\\frac{1}{a}}=\\frac{a b+3}{a b+1}$.", "solution": "null", "level": "高一", "question": "(1) 求值 $\\left(\\frac{64}{49}\\right)^{-\\frac{1}{2}}+\\lg \\frac{1}{4}+2^{\\log _{4} 9}-2 \\lg 5$;\n\n(2)已知 $\\log _{2} 5=a, \\log _{5} 7=b$, 试用 $a 、 b$ 表示 $\\log _{14} 56$.", "options": [], "subject": "代数", "analysis": "(1)利用指数的运算律、对数的运算律、换底公式以及对数恒等式可得出结果;(2)由换底公式可得出 $\\log _{5} 2=\\frac{1}{a}$, 然后利用换底公式可得出 $\\log _{14} 56=\\frac{\\log _{5} 56}{\\log _{5} 14}$, 并利用对数 $\\log _{5} 2$ 和 $\\log _{5} 7$表示分子和分母,代入化简计算即可."} {"id": "15142", "image": [], "answer": "(1) 解: 因为 $f(x)-f\\left(\\frac{1}{x}\\right)=3$,\n\n所以 $\\log _{2} x-\\log _{2} \\frac{1}{x}=3$, 则 $2 \\log _{2} x=3$,\n\n$\\therefore \\log _{2} x=\\frac{3}{2}, \\therefore x=2^{\\frac{3}{2}}=2 \\sqrt{2}$\n\n(2) 解: 由 $f\\left(a x_{0}{ }^{2}-4 x_{0}\\right)=2, \\therefore a x_{0} 2-4 x_{0}=4$,\n\n$\\therefore a=\\frac{4+4 x_{0}}{x_{0}{ }^{2}}=\\frac{4}{x_{0}^{2}}+\\frac{4}{x_{0}}$,\n\n因为 $x_{0} \\in\\lfloor 1,2\\rfloor, \\therefore a \\in\\lfloor 3,8\\rfloor$", "solution": "null", "level": "高一", "question": "已知函数 $f(x)=\\log _{2} x$;\n\n(1) 若 $f(x)-f\\left(\\frac{1}{x}\\right)=3$, 求 $x$ 的值;\n\n(2)若区间 $\\lfloor 1,2\\rfloor$ 上存在 $x_{0}$, 使得方程 $f\\left(a x_{0^{2}}-4 x_{0}\\right)=2$ 成立, 求实数 $a$ 的取值范围。", "options": [], "subject": "代数", "analysis": "(1) 将 $\\mathrm{x}$ 和 $\\frac{1}{\\bar{x}}$ 代入, 结合对数式的运算法则, 解对数方程, 即可求出相应的 $\\mathrm{x}$;\n\n(2) 分离参数, 构造新的函数, 采用换元法, 结合二次函数的性质, 即可求出实数 $\\mathrm{a}$ 的取值范围."} {"id": "15158", "image": ["7534.jpg"], "answer": "(1) 解: 由 $1-x>0,1+x>0$ 得函数的定义域为 $(-1,1)$.\n\n(2) 解: $f(x)=\\lg (1+x)$, 即 $\\lg (1-x)-\\lg (1+x)=\\lg (1+x)$,\n\n$\\therefore \\lg \\frac{1-x}{1+x}=\\lg (1+x), \\therefore \\frac{1-x}{1+x}=1+x$ 且 $-1\n\n(2) 当 $0f(2)$, 利用函数图像求出 $a$ 的取值范围", "options": [], "subject": "代数", "analysis": "(1)利用真数大于零列出不等式组, 其解为 $(-1,1)$, 它是函数的定义域(2)把方程 $f(x)=\\lg (1+x)$ 化为 $\\lg \\frac{1-x}{1+x}=\\lg (1+x)$ 后得到 $\\left\\{\\begin{array}{l}\\frac{1-x}{1+x}=1+x \\\\ -1-1)$\n\n不等式为 $\\log _{2}(x+1) \\leq \\log _{4}(3 x+1)$ ,\n\n$\\therefore\\left\\{\\begin{array}{c}x+1>0 \\\\ 3 x+1>0 \\\\ (x+1)^{2} \\leq 3 x+1,\\end{array}\\right.$\n\n解得 $0 \\leq x \\leq 1, \\therefore D=\\lfloor 0,1\\rfloor$;\n\n(2) 解: $H(x)=\\log _{4}(3 x+1)-\\frac{1}{2} \\log _{2}(x+1)=\\frac{1}{2} \\log _{2} \\frac{3 x+1}{x+1}(0 \\leq x \\leq 1)$.\n\n$\\therefore H(x)=\\frac{1}{2} \\log _{2}\\left(3-\\frac{2}{x+1}\\right)$.\n\n当 $x \\in[0,1]_{\\text {时, }} 3-\\frac{2}{x+1}$ 单调递增, $\\therefore H(x)$ 单调递增,\n\n$\\therefore H(x) \\in\\left[0, \\frac{1}{2}\\right]$, 因此当 $a \\in\\left[0, \\frac{1}{2}\\right]_{\\text {时满足条件. }}$", "solution": "null", "level": "高一", "question": "设函数 $f(x)=2^{x}-1$ 的反函数为 $f^{-1}(x), g(x)=\\log _{4}(3 x+1)$.\n\n(1)若 $f^{-1}(x) \\leq g(x)$ ,求 $x$ 的取值范围 $D$;\n\n(2)在(1)的条件下, 设 $H(x)=g(x)-\\frac{1}{2} f^{-1}(x)$, 当 $x \\in D$ 时, 函数 $H(x)$ 的图像与直线 $y=a$有公共点, 求实数 $a$ 的取值范围.22.已知函数 $f(x)=\\log _{a} \\frac{x-5}{x+5}(a>0$ 且 $a \\neq 1)$.\n\n\n\n(2)设 $g(x)=\\log _{a}(x-3)$, 若方程 $f(x)-1=g(x)$ 有实根, 求 $a$ 的取值范围.", "options": [], "subject": "代数", "analysis": "(1) 根据题意结合已知条件得出 $\\log _{2}(x+1) \\leq \\log _{4}(3 x+1)$, 再利用对数函数的定义域和单调性得出不等式组, 求解得出 $\\mathrm{x}$ 的取值;\n\n\n根据函数的值域得出当 $H(x) \\in\\left[0, \\frac{1}{2}\\right]_{\\text {时满足, 即 }} a \\in\\left[0, \\frac{1}{2}\\right]$ 。"} {"id": "15182", "image": [], "answer": "(1) 解: $\\because y=x^{\\frac{3}{4}}$ 为 $\\mathrm{R}$ 上的增函数,\n\n又 $2.3<2.4, \\therefore 2.3^{\\frac{3}{4}}<2.4^{\\frac{3}{4}}$;\n\n(2) 解: $\\because y=x^{-\\frac{3}{2}}$ 为 $(0,+\\infty)$ 上的减函数, 又 $\\sqrt{2}<\\sqrt{3}$,\n\n$\\therefore(\\sqrt{2})^{-\\frac{3}{2}}>(\\sqrt{3})^{-\\frac{3}{2}}$;\n\n(3) 解: $\\because y=x^{\\frac{6}{5}}$ 为 $\\mathrm{R}$ 上的偶函数,\n\n$\\therefore(-0.31)^{\\frac{6}{5}}=0.31^{\\frac{6}{5}}$.\n\n又函数 $y=x^{\\frac{6}{5}}$ 为 $[0,+\\infty)$ 上的增函数, 且 $0.31<0.35$,\n\n$\\therefore 0.31^{\\frac{6}{5}}<0.35^{\\frac{6}{5}}$, 即 $(-0.31)^{\\frac{6}{5}}<0.35^{\\frac{6}{5}}$.", "solution": "null", "level": "高一", "question": "比较下列各题中两个幂的值的大小:\n\n(1) $2.3^{\\frac{3}{4}}, 2.4^{\\frac{3}{4}}$;\n\n(2) $(\\sqrt{2})^{-\\frac{3}{2}},(\\sqrt{3})^{-\\frac{3}{2}}$;\n\n(3) $(-0.31)^{\\frac{6}{5}}, 0.35^{\\frac{6}{5}}$.", "options": [], "subject": "代数", "analysis": "(1)结合幂函数的单调性的定义即可得出结论。(2)根据幂函数的单调性的的定义即可得出结论。(3)利用幂函数的单调性以及偶函数的性质即可得出结论。"} {"id": "15183", "image": [], "answer": "(1) 解: 由题得 $2^{m i^{2}+m+1}=8 \\Rightarrow m=1$ 或 $m=-2$ (舍);\n\n(2)解: 由题得 $f(x)=x^{3}, f(x)$ 在 $\\mathrm{R}$ 上单调递增,\n\n由 $\\mathrm{f}(2-\\mathrm{a})>\\mathrm{f}(\\mathrm{a}-1)$ 可得 $2-a>a-1 \\Rightarrow a<\\frac{3}{2}$.", "solution": "null", "level": "高一", "question": "已知幂函数 $f(x)=x^{m^{2}+m+1}\\left(m \\in N^{*}\\right)$ 的图象经过点 $(2,8)$.\n\n(1) 试确定 $m$ 的值;\n\n(2) 求满足条件 $\\mathrm{f}(2-\\mathrm{a})>\\mathrm{f}(\\mathrm{a}-1)$ 的实数 $a$ 的取值范围.", "options": [], "subject": "代数", "analysis": "(1)由已知利用幂函数的解析式列式,即可求出 $\\mathrm{m}$ 的值;\n\n(2)由已知利用幂函数的单调性列式,即可求出 $\\mathrm{a}$ 的取值范围."} {"id": "15184", "image": ["7546.jpg"], "answer": "(1) 解: 因为幂函数 $y=f(x)=x^{\\alpha}$ 的图象过点 $(4,2)$,\n\n所以 $4^{\\alpha}=2$, 解得 $\\alpha=\\frac{1}{2}$,\n\n所以 $f(x)=x^{\\frac{1}{2}}$,\n又点 $(5, m)$ 也在幂函数上 $f(x)=x^{\\frac{1}{2}}$,\n\n所以 $m=\\sqrt{5}$;\n\n(2) 解: 由 (1) 知, $y=g(x)=\\log _{a} \\sqrt{x}=\\frac{1}{2} \\log _{a} x(a>0, a \\neq 1)$,\n\n(1)当 $a>1$ 时, 函数 $g(x)=\\frac{1}{2} \\log _{a} x$ 在区间 $[3,9\\rfloor$ 上单调递增,\n\n由题意可得: $g(x)_{\\text {max }}-g(x)_{\\text {min }}=\\frac{1}{2} \\log _{a} 9-\\frac{1}{2} \\log _{a} 3=\\frac{1}{2} \\log _{a} 3=1$,\n\n解得 $a=\\sqrt{3}$;\n\n\n\n$\\therefore g(x)_{\\max }-g(x)_{\\min }=\\frac{1}{2} \\log _{a} 3-\\frac{1}{2} \\log _{a} 9=\\frac{1}{2} \\log _{a} \\frac{1}{3}=1$,\n\n解得 $a=\\frac{\\sqrt{3}}{3}$.\n\n综上所述, $a=\\sqrt{3}$ 或 $\\frac{\\sqrt{3}}{3}$.", "solution": "null", "level": "高一", "question": "已知幂函数 $y=f(x)=x^{\\alpha}$ 的图象过点 $(5, m)$ 和 $(4,2)$.\n\n(1) 求 $m$ 的值;\n\n(2) 若函数 $y=g(x)=\\log _{a} f(x)(a>0, a \\neq 1)$ 在区间 $[3,9]$ 上的最大值比最小值大 1 , 求实数 $a$ 的值.", "options": [], "subject": "代数", "analysis": "(1) 先由幂函数 $y=f(x)=x^{\\alpha}$ 的图象过点 $(4,2)$, 求出解析式, 再由图像过点 $(5, m)$, 即可求出结果; (2) 先由题意得到 $g(x)=\\frac{1}{2} \\log _{a} x$, 分别讨论 $a>1,00$, 即 $m<-\\frac{1}{2}$,\n\n所以 $m=-1$.\n\n(2)解: 由于 $y=x^{-1}$ 在区间 $(-\\infty, 0),(0,+\\infty)$ 都是减函数, 且 $(k+1)^{-1}<(3-2 k)^{-1}$分三种情况讨论:\n\n(1)当 $k+1<0<3-2 k$, 即 $k<-1$ 时, 原不等式成立;\n\n(2)当 $k+1<0$ 且 $3-2 k<0$ 时, 有 $k+1>3-2 k$, 即 $\\left\\{\\begin{array}{l}k<-1 \\\\ k>\\frac{3}{2} \\\\ k>\\frac{2}{3}\\end{array}\\right.$, 解集为空集;\n\n(3)当 $k+1>0$ 且 $3-2 k>0$ 时, 有 $k+1>3-2 k$, 即 $\\left\\{\\begin{array}{l}k>-1 \\\\ k<\\frac{3}{2} \\\\ k>\\frac{2}{3}\\end{array}\\right.$,\n\n$\\therefore \\frac{2}{3}", "options": [], "subject": "代数", "analysis": "解:输入语句用 “INPUT”\n\n当型循环语句用 WHILE、WEND\n\n故答案为: INPUT、WHILE、WEND"} {"id": "16918", "image": ["8736.jpg"], "answer": "解: 模拟执行程序, 可得程序的功能是计算并输出 $y=\\left\\{\\begin{array}{ll}x^{2} & x \\leq 2 \\\\ 2 x-3 & x>2\\end{array}\\right.$ 的值,\n\n(1) 输入 $X$ 的值为 0 , 由 $x<2$, 故 $y=x^{2}=0$,\n\n故输出 $Y$ 的值为 0 .\n\n(2) 若输出 $Y$ 的值为 $3, y=\\left\\{\\begin{array}{ll}3=x^{2} & x \\leq 2 \\\\ 3=2 x-3 & x>2\\end{array}\\right.$,\n\n可解得: $x= \\pm \\sqrt{3}$, 或 3 .", "solution": "null", "level": "高一", "question": "(2015 春・椰州校级期中)已知一个程序语句如图:\n\n(1) 若输入 $X$ 的值为 0 , 求输出 $Y$ 的值?\n\n(2) 若输出 $Y$ 的值为 3 , 求输入 $X$ 的值?\n\n", "options": [], "subject": "代数", "analysis": "解: 模拟执行程序, 可得程序的功能是计算并输出 $y=\\left\\{\\begin{array}{ll}x^{2} & x \\leq 2 \\\\ 2 x-3 & x>2\\end{array}\\right.$ 的值,\n\n(1) 输入 $X$ 的值为 0 , 由 $x<2$, 故 $y=x^{2}=0$,\n\n故输出 $Y$ 的值为 0 .\n\n(2) 若输出 $Y$ 的值为 $3, y=\\left\\{\\begin{array}{ll}3=x^{2} & x \\leq 2 \\\\ 3=2 x-3 & x>2\\end{array}\\right.$,\n\n可解得: $x= \\pm \\sqrt{3}$, 或 3 ."} {"id": "16937", "image": [], "answer": "解: 1202 (3) 对应的十进制数\n\n那么: $a=1 \\times 3^{3}+2 \\times 3^{2}+0 \\times 3^{1}+2 \\times 3^{0}=47$.\n\n$b$ 是 8251 与 6105 的最大公约数\n\n即 $8251=6105 \\times 1+2146$\n\n$6105=2146 \\times 2+1813$\n\n$2146=1813 \\times 1+333$\n\n$1813=333 \\times 5+148$\n\n$333=148 \\times 2+37$\n\n$148=37 \\times 4+0$\n\n则 37 为 8251 与 6105 的最大公约数.\n\n即 $b=37$.\n\n那么 $\\left.f(x)=x^{5}+47 x^{4}-37 x^{2}+1=((((x+47) x)+0) x-37) x+0\\right) x+1$\n\n$\\therefore v_{0}=1$,\n\n$v_{1}=-1+47=46$.\n\n$v_{2}=-1 \\times 46=-46$,\n\n$v_{3}=-46 \\times-1-37=9$.", "solution": "null", "level": "高一", "question": "函数 $f(x)=x^{5}+a x^{4}-b x^{2}+1$. 其中 $a$ 是 1202 (3) 对应的十进制数, $b$ 是 8251 与 6105 的最大公约数, 试应用秦九韶算法求当 $x=-1$ 时 $v_{3}$ 的值.", "options": [], "subject": "代数", "analysis": "解: 1202 (3) 对应的十进制数\n\n那么: $a=1 \\times 3^{3}+2 \\times 3^{2}+0 \\times 3^{1}+2 \\times 3^{0}=47$.\n\n$b$ 是 8251 与 6105 的最大公约数\n\n即 $8251=6105 \\times 1+2146$\n\n$6105=2146 \\times 2+1813$\n\n$2146=1813 \\times 1+333$\n\n$1813=333 \\times 5+148$\n\n$333=148 \\times 2+37$\n\n$148=37 \\times 4+0$\n\n则 37 为 8251 与 6105 的最大公约数.\n\n即 $b=37$.\n\n那么 $\\left.f(x)=x^{5}+47 x^{4}-37 x^{2}+1=((((x+47) x)+0) x-37) x+0\\right) x+1$\n\n$\\therefore v_{0}=1$,\n\n$v_{1}=-1+47=46$.\n\n$v_{2}=-1 \\times 46=-46$,\n\n$v_{3}=-46 \\times-1-37=9$."} {"id": "16941", "image": [], "answer": "解: $\\left.f(x)=3 x^{5}-2 x^{4}+5 x^{3}-2.5 x^{2}+1.5 x-0.7=(((3 x-2) x+5) x-2.5) x+1.5\\right) x$ -0.7 ,\n\n$v_{0}=3, v_{1}=3 \\times 4-2=10, v_{2}=10 \\times 4+5=45, v_{3}=45 \\times 4-2.5=177.5, \\quad v_{4}=177.5 \\times 4+1.5=711.5$, $v_{5}=711.5 \\times 4-0.7=2845.3$.", "solution": "null", "level": "高一", "question": "017 春・和平区期末)已知一个 5 次多项式为 $f(x)=3 x^{5}-2 x^{4}+5 x^{3}-2.5 x^{2}+1.5 x-0.7$, 用秦九韶算法求出这个多项式当 $x=4$ 时的值.", "options": [], "subject": "代数", "analysis": "解: $\\left.f(x)=3 x^{5}-2 x^{4}+5 x^{3}-2.5 x^{2}+1.5 x-0.7=(((3 x-2) x+5) x-2.5) x+1.5\\right) x$ -0.7 ,\n\n$v_{0}=3, v_{1}=3 \\times 4-2=10, v_{2}=10 \\times 4+5=45, v_{3}=45 \\times 4-2.5=177.5, \\quad v_{4}=177.5 \\times 4+1.5=711.5$, $v_{5}=711.5 \\times 4-0.7=2845.3$."} {"id": "16956", "image": ["8744.jpg"], "answer": "解: 此程序表示的函数为 $y= \\begin{cases}x^{2}, & x<0 \\\\ -1, & x=0, \\\\ 2 x, & x>0\\end{cases}$\n\n当 $x<0$ 时, $x^{2}=4$ 得 $x=-2$.\n\n当 $x>0$ 时, $2 x=4$ 得 $x=2$.\n\n故当输出的 $y=4$ 时, 输入的 $x= \\pm 2$.", "solution": "null", "level": "高一", "question": "(2017 秋 -上期末)读下列程序, 写出此程序表示的函数, 并求当输出的 $y=4$ 时, 输入的 $x$ 的值.\n\n", "options": [], "subject": "代数", "analysis": "解: 此程序表示的函数为 $y= \\begin{cases}x^{2}, & x<0 \\\\ -1, & x=0, \\\\ 2 x, & x>0\\end{cases}$\n\n当 $x<0$ 时, $x^{2}=4$ 得 $x=-2$.\n\n当 $x>0$ 时, $2 x=4$ 得 $x=2$.\n\n故当输出的 $y=4$ 时, 输入的 $x= \\pm 2$."} {"id": "16958", "image": ["8745.jpg"], "answer": "解:模拟程序语言的运行过程, 如下:\n\n$a=5$,\n\n$b=8$,\n\n$a=7$,\n$b=15$,\n\n输出 $a$ 的值为 $7, b$ 的值为 15 .", "solution": "null", "level": "高一", "question": "阅读如图的伪代码: 若输入的值是 5 , 则最后输出 $a, b$ 的值分别是多少?\n\n", "options": [], "subject": "代数", "analysis": "解:模拟程序语言的运行过程, 如下:\n\n$a=5$,\n\n$b=8$,\n\n$a=7$,\n$b=15$,\n\n输出 $a$ 的值为 $7, b$ 的值为 15 ."} {"id": "16959", "image": ["8746.jpg"], "answer": "解:该伪代码的功能是比较输入的三个数并输出最小的数;\n\n当输入的三个数是 $-2,-13,7$ 时,\n\n输出的结果是 -13.", "solution": "null", "level": "高一", "question": "阅读如图的伪代码: 如果输入的三个数是 $-2,-13,7$, 那么输出的结果是什么? 说明该算法的功能.\n\n", "options": [], "subject": "代数", "analysis": "解:该伪代码的功能是比较输入的三个数并输出最小的数;\n\n当输入的三个数是 $-2,-13,7$ 时,\n\n输出的结果是 -13."} {"id": "16964", "image": ["8749.jpg"], "answer": "解: (I) 根据题意, 写出 $y$ 与 $x$ 的函数关系为:\n\n$$\ny=\\left\\{\\begin{array}{l}\n13 x, x \\leq 50 \\\\\n50+15(x-50), 50100\n\\end{array}\\right.\n$$\n\n(II) 因为 $x=150>100$,\n\n所以 $y=150+25 \\times(150-100)=1400$,\n\n故该厂应缴纳污水处理费 1400 元. …12 分.", "solution": "null", "level": "高一", "question": "017 秋・湛江月考)某市对排污水进行综合治理,征收污水处理费,系统对各厂一个月内排出的污水量 $x$ 吨收取的污水处理费 $y$ 元,运行程序如图所示:\n\n(I) 写出 $y$ 与 $x$ 的函数关系;\n\n(II) 求排放污水 150 吨的污水处理费用.\n\n", "options": [], "subject": "代数", "analysis": "解: (I) 根据题意, 写出 $y$ 与 $x$ 的函数关系为:\n\n$$\ny=\\left\\{\\begin{array}{l}\n13 x, x \\leq 50 \\\\\n50+15(x-50), 50100\n\\end{array}\\right.\n$$\n\n(II) 因为 $x=150>100$,\n\n所以 $y=150+25 \\times(150-100)=1400$,\n\n故该厂应缴纳污水处理费 1400 元. …12 分."} {"id": "14996", "image": [], "answer": "(1) $\\mathrm{A} \\cup(\\mathrm{B} \\cap \\mathrm{C})=\\{1,2\\} \\cup\\{3,4,5\\}=\\{1,2,3,4,5\\}$\n\n(2)$\\left(\\complement_{\\cup} B\\right) \\cup\\left(\\complement_{U} C\\right)=\\{6,7,8\\} \\cup\\{1,2\\}=\\{1,2,6,7,8\\}$", "solution": "null", "level": "高一", "question": "已知全集 $U=\\{1,2,3,4,5,6,7,8\\}, A=\\left\\{x \\mid x^{2}-3 x+2=0\\right\\}, B=\\{x \\mid 1 \\leq x \\leq 5, x \\in Z\\}, C=\\{x \\mid 2\n\n(1)E, $C, D_{1}, F$ 四点共面;\n\n(2)直线 $C E, D_{1} F, D A$ 三线共点.\n\n证明 (1)如图,连接 $E F, D_{1} C, A_{1} B$.\n\n\n\n$\\because E$ 为 $A B$ 的中点, $F$ 为 $A A_{1}$ 的中点,\n\n$\\therefore E F / / A_{1} B$, 且 $E F=\\frac{1}{2} A_{1} B$,\n\n又 $\\because A_{1} B / / D_{1} C$ ,且 $A_{1} B=D_{1} C$ ,\n\n$\\therefore E F / / D_{1} C$ ,且 $E F=\\frac{1}{2} D_{1} C$ ,\n\n$\\therefore E, F, D_{1}, C$ 四点共面.\n\n(2) $\\because E F / / C D_{1}, E F\n\n(1)E, $C, D_{1}, F$ 四点共面;\n\n(2)直线 $C E, D_{1} F, D A$ 三线共点.\n\n证明 (1)如图,连接 $E F, D_{1} C, A_{1} B$.\n\n\n\n$\\because E$ 为 $A B$ 的中点, $F$ 为 $A A_{1}$ 的中点,\n\n$\\therefore E F / / A_{1} B$, 且 $E F=\\frac{1}{2} A_{1} B$,\n\n又 $\\because A_{1} B / / D_{1} C$ ,且 $A_{1} B=D_{1} C$ ,\n\n$\\therefore E F / / D_{1} C$ ,且 $E F=\\frac{1}{2} D_{1} C$ ,\n\n$\\therefore E, F, D_{1}, C$ 四点共面.\n\n(2) $\\because E F / / C D_{1}, E F\n\n(1)E, $C, D_{1}, F$ 四点共面;\n\n(2)直线 $C E, D_{1} F, D A$ 三线共点.\n\n证明 (1)如图,连接 $E F, D_{1} C, A_{1} B$.\n\n\n\n$\\because E$ 为 $A B$ 的中点, $F$ 为 $A A_{1}$ 的中点,\n\n$\\therefore E F / / A_{1} B$, 且 $E F=\\frac{1}{2} A_{1} B$,\n\n又 $\\because A_{1} B / / D_{1} C$ ,且 $A_{1} B=D_{1} C$ ,\n\n$\\therefore E F / / D_{1} C$ ,且 $E F=\\frac{1}{2} D_{1} C$ ,\n\n$\\therefore E, F, D_{1}, C$ 四点共面.\n\n(2) $\\because E F / / C D_{1}, E F\n\n$\\because E$ 是 $A A_{1}$ 的中点,\n\n$\\therefore E Q / / A_{1} D_{1}$ 且 $E Q=A_{1} D_{1}$.\n\n又在矩形 $A_{1} B_{1} C_{1} D_{1}$ 中,\n\n$A_{1} D_{1} / / B_{1} C_{1}$ 且 $A_{1} D_{1}=B_{1} C_{1}$,\n\n$\\therefore E Q / / B_{1} C_{1}$ 且 $E Q=B_{1} C_{1}$.\n\n$\\therefore$ 四边形 $E Q C_{1} B_{1}$ 为平行四边形,\n\n$\\therefore B_{1} E / / C_{1} Q$ 且 $B_{1} E=C_{1} Q$.\n\n又 $\\because Q, F$ 是 $D D_{1}, C_{1} C$ 两边的中点,\n\n$\\therefore Q D / / C_{1} F$ 且 $Q D=C_{1} F$ ,\n\n$\\therefore$ 四边形 $Q D F C_{1}$ 为平行四边形.\n\n$\\therefore C_{1} Q / / D F$ 且 $C_{1} Q=D F$ ,\n\n$\\therefore B_{1} E / / D F$ 且 $B_{1} E=D F$ ,\n\n$\\therefore$ 四边形 $B_{1} E D F$ 为平行四边形.", "solution": "null", "level": "高一", "question": "如图所示, $E, F$ 分别是长方体 $A_{1} B_{1} C_{1} D_{1}-A B C D$ 的棱 $A_{1} A, C_{1} C$ 的中点. 求证: 四边形 $B_{1} E D F$ 是平行四边形.\n\n", "options": [], "subject": "立体几何学", "analysis": "设 $Q$ 是 $D D_{1}$ 的中点,连接 $E Q , Q C_{1}$.\n\n\n\n$\\because E$ 是 $A A_{1}$ 的中点,\n\n$\\therefore E Q / / A_{1} D_{1}$ 且 $E Q=A_{1} D_{1}$.\n\n又在矩形 $A_{1} B_{1} C_{1} D_{1}$ 中,\n\n$A_{1} D_{1} / / B_{1} C_{1}$ 且 $A_{1} D_{1}=B_{1} C_{1}$,\n\n$\\therefore E Q / / B_{1} C_{1}$ 且 $E Q=B_{1} C_{1}$.\n\n$\\therefore$ 四边形 $E Q C_{1} B_{1}$ 为平行四边形,\n\n$\\therefore B_{1} E / / C_{1} Q$ 且 $B_{1} E=C_{1} Q$.\n\n又 $\\because Q, F$ 是 $D D_{1}, C_{1} C$ 两边的中点,\n\n$\\therefore Q D / / C_{1} F$ 且 $Q D=C_{1} F$ ,\n\n$\\therefore$ 四边形 $Q D F C_{1}$ 为平行四边形.\n\n$\\therefore C_{1} Q / / D F$ 且 $C_{1} Q=D F$ ,\n\n$\\therefore B_{1} E / / D F$ 且 $B_{1} E=D F$ ,\n\n$\\therefore$ 四边形 $B_{1} E D F$ 为平行四边形."} {"id": "15432", "image": ["7664.jpg", "7665.jpg", "7665.jpg"], "answer": ")由题意知 $S O \\perp$ 底面 $A B N$ ,\n\n\n\n在 Rt $\\triangle S O B$ 中, $O B=\\frac{1}{2} A B=2 , S O=6$ ,\n\n所以 $S B=\\sqrt{2^{2}+6^{2}}=2 \\sqrt{10}$.\n\n所以该圆锥的侧面积 $S=\\pi \\cdot O B \\cdot S B=4 \\sqrt{10} \\pi$.\n\n(2)取 $O B$ 的中点 $C$ ,连接 $M C , N C$ ,\n\n因为 $M$ 为 $S B$ 的中点,\n\n所以 $M C$ 为 $\\triangle S O B$ 的中位线,\n\n所以 $M C / / S O$ ,\n\n$M C=\\frac{1}{2} S O=3$.\n\n又因为 $S O \\perp$ 底面 $A B N$ ,\n\n所以 $M C \\perp$ 底面 $A B N ,$\n\n因为 $N C \\subset$ 底面 $A B N$ ,所以 $M C \\perp N C$ 。\n\n因为直线 $S O$ 与 $M N$ 所成的角为 $30^{\\circ}$ ,\n所以 $\\angle N M C=30^{\\circ}$ ,\n\n在 Rt $\\triangle M C N$ 中, $\\frac{M C}{M N}=\\cos 30^{\\circ}$ ,\n\n所以 $M N=\\frac{M C}{\\cos 30^{\\circ}}=\\frac{3}{\\frac{\\sqrt{3}}{2}}=2 \\sqrt{3}$.", "solution": "null", "level": "高一", "question": "如图, 平面 $S A B$ 为圆雉的轴截面, $O$ 为底面圆的圆心, $M$ 为母线 $S B$ 的中点, $N$ 为底面圆周上的一点, $A B=4, S O=6$.\n\n\n\n(1)求该圆雉的侧面积;\n\n(2)若直线 $S O$ 与 $M N$ 所成的角为 $30^{\\circ}$, 求 $M N$ 的长.", "options": [], "subject": "立体几何学", "analysis": ")由题意知 $S O \\perp$ 底面 $A B N$ ,\n\n\n\n在 Rt $\\triangle S O B$ 中, $O B=\\frac{1}{2} A B=2 , S O=6$ ,\n\n所以 $S B=\\sqrt{2^{2}+6^{2}}=2 \\sqrt{10}$.\n\n所以该圆锥的侧面积 $S=\\pi \\cdot O B \\cdot S B=4 \\sqrt{10} \\pi$.\n\n(2)取 $O B$ 的中点 $C$ ,连接 $M C , N C$ ,\n\n因为 $M$ 为 $S B$ 的中点,\n\n所以 $M C$ 为 $\\triangle S O B$ 的中位线,\n\n所以 $M C / / S O$ ,\n\n$M C=\\frac{1}{2} S O=3$.\n\n又因为 $S O \\perp$ 底面 $A B N$ ,\n\n所以 $M C \\perp$ 底面 $A B N ,$\n\n因为 $N C \\subset$ 底面 $A B N$ ,所以 $M C \\perp N C$ 。\n\n因为直线 $S O$ 与 $M N$ 所成的角为 $30^{\\circ}$ ,\n所以 $\\angle N M C=30^{\\circ}$ ,\n\n在 Rt $\\triangle M C N$ 中, $\\frac{M C}{M N}=\\cos 30^{\\circ}$ ,\n\n所以 $M N=\\frac{M C}{\\cos 30^{\\circ}}=\\frac{3}{\\frac{\\sqrt{3}}{2}}=2 \\sqrt{3}$."} {"id": "15456", "image": ["7674.jpg"], "answer": "(1)证明 设 $B C \\cap O D=E$ ,\n\n$\\because D$ 是弧 $B C$ 的中点,\n\n$\\therefore E$ 是 $B C$ 的中点,\n\n又 $\\because O$ 是 $A B$ 的中点,\n\n$\\therefore A C / / O E$,\n\n又 $\\because A C$ 平面 $P O D , O E \\subset$ 平面 $P O D ,$\n\n$\\therefore A C / /$ 平面 $P O D$.\n\n解 (2) 设圆雉底面半径为 $r$ ,高为 $h$ ,母线长为 $l$ ,\n\n$\\because$ 圆锥的轴截面 $P A B$ 为等腰直角三角形,\n\n$\\therefore h=r, l=\\sqrt{2} r$,\n\n$\\because S_{\\triangle P A B}=\\frac{1}{2} \\times 2 r \\times h=r^{2}=9$,\n\n$\\therefore r=3$\n\n$\\therefore S_{\\text {表 }}=\\pi r l+\\pi r^{2}=\\pi r \\times \\sqrt{2} r+\\pi r^{2}=9(1+\\sqrt{2}) \\pi$.", "solution": "null", "level": "高一", "question": "如图, $O$ 是圆雉底面圆的圆心, 圆雉的轴截面 $P A B$ 为等腰直角三角形, $C$ 为底面圆周\n上一点.\n\n(1)若弧 $B C$ 的中点为 $D$. 求证: $A C / /$ 平面 $P O D$;\n\n(2)如果 $\\triangle P A B$ 的面积是 9 , 求此圆雉的表面积.\n\n", "options": [], "subject": "立体几何学", "analysis": "(1)证明 设 $B C \\cap O D=E$ ,\n\n$\\because D$ 是弧 $B C$ 的中点,\n\n$\\therefore E$ 是 $B C$ 的中点,\n\n又 $\\because O$ 是 $A B$ 的中点,\n\n$\\therefore A C / / O E$,\n\n又 $\\because A C$ 平面 $P O D , O E \\subset$ 平面 $P O D ,$\n\n$\\therefore A C / /$ 平面 $P O D$.\n\n解 (2) 设圆雉底面半径为 $r$ ,高为 $h$ ,母线长为 $l$ ,\n\n$\\because$ 圆锥的轴截面 $P A B$ 为等腰直角三角形,\n\n$\\therefore h=r, l=\\sqrt{2} r$,\n\n$\\because S_{\\triangle P A B}=\\frac{1}{2} \\times 2 r \\times h=r^{2}=9$,\n\n$\\therefore r=3$\n\n$\\therefore S_{\\text {表 }}=\\pi r l+\\pi r^{2}=\\pi r \\times \\sqrt{2} r+\\pi r^{2}=9(1+\\sqrt{2}) \\pi$."} {"id": "15457", "image": ["7675.jpg", "7676.jpg", "7676.jpg"], "answer": "如图,连接 $D G , C D$ ,设 $C D \\cap G F=O$ ,连接 $O H$.\n\n\n\n在三棱台 $D E F-A B C$ 中, $A B=2 D E , G$ 为 $A C$ 的中点,可得 $D F / / G C$ 且 $D F=G C$ ,\n\n所以四边形 $D F C G$ 为平行四边形,则 $O$ 为 $C D$ 的中点,\n\n又 $H$ 为 $B C$ 的中点,所以 $O H / / B D$.\n\n又 $\\mathrm{OH} \\subset$ 平面 $F G H, B D$ 平面 $F G H$ ,\n\n所以 $B D / /$ 平面 $F G H$.", "solution": "null", "level": "高一", "question": "如图, 在三棱台 $D E F-A B C$ 中, $A B=2 D E, G, H$ 分别为 $A C, B C$ 的中点.\n\n\n\n求证: $B D / /$ 平面 $F G H$.", "options": [], "subject": "立体几何学", "analysis": "如图,连接 $D G , C D$ ,设 $C D \\cap G F=O$ ,连接 $O H$.\n\n\n\n在三棱台 $D E F-A B C$ 中, $A B=2 D E , G$ 为 $A C$ 的中点,可得 $D F / / G C$ 且 $D F=G C$ ,\n\n所以四边形 $D F C G$ 为平行四边形,则 $O$ 为 $C D$ 的中点,\n\n又 $H$ 为 $B C$ 的中点,所以 $O H / / B D$.\n\n又 $\\mathrm{OH} \\subset$ 平面 $F G H, B D$ 平面 $F G H$ ,\n\n所以 $B D / /$ 平面 $F G H$."} {"id": "15469", "image": ["7688.jpg"], "answer": "$\\because P M: M A=B N: N D=P Q: Q D$ ,\n\n$\\therefore M Q / / A D, N Q / / B P$ ,\n\n而 $B P \\subset$ 平面 $P B C , N Q \\Phi$ 平面 $P B C ,$\n$\\therefore N Q / /$ 平面 $P B C$.\n\n又 $\\because$ 四边形 $A B C D$ 为平行四边形, $\\therefore B C / / A D ,$\n\n$\\therefore M Q / / B C$ ,而 $B C \\subset$ 平面 $P B C , M Q q$ 平面 $P B C , \\therefore M Q / /$ 平面 $P B C$.\n\n又 $M Q \\cap N Q=Q, M Q, N Q \\subset$ 平面 $M N Q$ ,\n\n$\\therefore$ 平面 $M N Q / /$ 平面 $P B C$.", "solution": "null", "level": "高一", "question": "如图, 已知在四棱雉 $P-A B C D$ 中, 底面 $A B C D$ 为平行四边形, 点 $M, N, Q$ 分别在 $P A$, $B D, P D$ 上, 且 $P M: M A=B N: N D=P Q: Q D$.求证: 平面 $M N Q / /$ 平面 $P B C$.\n\n", "options": [], "subject": "立体几何学", "analysis": "$\\because P M: M A=B N: N D=P Q: Q D$ ,\n\n$\\therefore M Q / / A D, N Q / / B P$ ,\n\n而 $B P \\subset$ 平面 $P B C , N Q \\Phi$ 平面 $P B C ,$\n$\\therefore N Q / /$ 平面 $P B C$.\n\n又 $\\because$ 四边形 $A B C D$ 为平行四边形, $\\therefore B C / / A D ,$\n\n$\\therefore M Q / / B C$ ,而 $B C \\subset$ 平面 $P B C , M Q q$ 平面 $P B C , \\therefore M Q / /$ 平面 $P B C$.\n\n又 $M Q \\cap N Q=Q, M Q, N Q \\subset$ 平面 $M N Q$ ,\n\n$\\therefore$ 平面 $M N Q / /$ 平面 $P B C$."} {"id": "15470", "image": ["7689.jpg", "7690.jpg", "7690.jpg"], "answer": "(1) 证明 如图,连接 $A E$ ,由 $F$ 是线段 $B D$ 的中点,四边形 $A B E D$ 为正方形得 $F$ 为 $A E$ 的中点,\n\n\n\n$\\therefore G F$ 为 $\\triangle A E C$ 的中位线,\n\n$\\therefore G F / / A C$.\n\n又 $\\because A C \\subset$ 平面 $A B C , G F \\subset$ 平面 $A B C$ ,\n\n$\\therefore G F / /$ 平面 $A B C$.\n\n(2)解 平面 $G F P / /$ 平面 $A B C$ ,\n\n证明如下:\n\n连接 $F P, G P$.\n\n$\\because$ 点 $F, P$ 分别为 $B D, C D$ 的中点,\n\n$\\therefore F P$ 为 $\\triangle B C D$ 的中位线, $\\therefore F P / / B C$.\n\n又 $\\because B C \\subset$ 平面 $A B C , F P \\nmid$ 平面 $A B C , \\therefore F P / /$ 平面 $A B C$ ,\n又 $G F / /$ 平面 $A B C , F P \\cap G F=F, F P \\subset$ 平面 $G F P$ ,\n\n$G F \\subset$ 平面 $G F P ,$\n\n$\\therefore$ 平面 $G F P / /$ 平面 $A B C$.", "solution": "null", "level": "高一", "question": "如图, 在四棱雉 $C-A B E D$ 中, 四边形 $A B E D$ 是正方形, 点 $G, F$ 分别是线段 $E C, B D$的中点.\n\n\n\n(1)求证: $G F / /$ 平面 $A B C$;\n\n(2)若点 $P$ 为线段 $C D$ 的中点, 平面 $G F P$ 与平面 $A B C$ 有怎样的位置关系? 并证明.", "options": [], "subject": "立体几何学", "analysis": "(1) 证明 如图,连接 $A E$ ,由 $F$ 是线段 $B D$ 的中点,四边形 $A B E D$ 为正方形得 $F$ 为 $A E$ 的中点,\n\n\n\n$\\therefore G F$ 为 $\\triangle A E C$ 的中位线,\n\n$\\therefore G F / / A C$.\n\n又 $\\because A C \\subset$ 平面 $A B C , G F \\subset$ 平面 $A B C$ ,\n\n$\\therefore G F / /$ 平面 $A B C$.\n\n(2)解 平面 $G F P / /$ 平面 $A B C$ ,\n\n证明如下:\n\n连接 $F P, G P$.\n\n$\\because$ 点 $F, P$ 分别为 $B D, C D$ 的中点,\n\n$\\therefore F P$ 为 $\\triangle B C D$ 的中位线, $\\therefore F P / / B C$.\n\n又 $\\because B C \\subset$ 平面 $A B C , F P \\nmid$ 平面 $A B C , \\therefore F P / /$ 平面 $A B C$ ,\n又 $G F / /$ 平面 $A B C , F P \\cap G F=F, F P \\subset$ 平面 $G F P$ ,\n\n$G F \\subset$ 平面 $G F P ,$\n\n$\\therefore$ 平面 $G F P / /$ 平面 $A B C$."} {"id": "15482", "image": ["7701.jpg", "7702.jpg", "7703.jpg", "7702.jpg", "7703.jpg"], "answer": "$\\because$ 四边形 $A B C D$ 为矩形,\n\n$\\therefore B C / / A D$.\n\n$\\because A D \\subset$ 平面 $P A D , B C \\subset$ 平面 $P A D$ ,\n\n$\\therefore B C / /$ 平面 $P A D$.\n\n$\\because$ 平面 $B C F E \\cap$ 平面 $P A D=E F$ ,\n\n$\\therefore B C / / E F$.\n\n$\\because A D=B C, A D \\neq E F$\n$\\therefore B C \\neq E F$,\n\n$\\therefore$ 四边形 $B C E F$ 是梯形.\n\n13.如图, 已知 $E, F$ 分别是菱形 $A B C D$ 中边 $B C, C D$ 的中点, $E F$ 与 $A C$ 交于点 $O$, 点 $P$ 在平面 $A B C D$ 之外, $M$ 是线段 $P A$ 上一动点, 若 $P C / /$ 平面 $M E F$, 试求 $P M: M A$ 的值.\n\n\n\n解 如图,连接 $B D$ 交 $A C$ 于点 $O_{1}$ ,连接 $O M$.\n\n\n\n因为 $P C / /$ 平面 $M E F$ ,平面 $P A C \\cap$ 平面 $M E F=O M , P C \\subset$ 平面 $P A C$ ,\n\n所以 $P C / / O M$ ,所以 $\\frac{P M}{P A}=\\frac{O C}{A C}$.\n\n在菱形 $A B C D$ 中,\n\n因为 $E, F$ 分别是边 $B C, C D$ 的中点,所以 $\\frac{O C}{O_{1} C}=\\frac{1}{2}$.\n\n又 $A O_{1}=C O_{1}$, 所以 $\\frac{P M}{P A}=\\frac{O C}{A C}=\\frac{1}{4}$,\n\n故 $P M: M A=1: 3$.", "solution": "null", "level": "高一", "question": "如图, 四边形 $A B C D$ 是矩形, $P \\notin$ 平面 $A B C D$, 过 $B C$ 作平面 $B C F E$ 交 $A P$ 于点 $E$, 交 $D P$于点 $F$, 求证: 四边形 $B C F E$ 是梯形.\n\n", "options": [], "subject": "立体几何学", "analysis": "$\\because$ 四边形 $A B C D$ 为矩形,\n\n$\\therefore B C / / A D$.\n\n$\\because A D \\subset$ 平面 $P A D , B C \\subset$ 平面 $P A D$ ,\n\n$\\therefore B C / /$ 平面 $P A D$.\n\n$\\because$ 平面 $B C F E \\cap$ 平面 $P A D=E F$ ,\n\n$\\therefore B C / / E F$.\n\n$\\because A D=B C, A D \\neq E F$\n$\\therefore B C \\neq E F$,\n\n$\\therefore$ 四边形 $B C E F$ 是梯形.\n\n13.如图, 已知 $E, F$ 分别是菱形 $A B C D$ 中边 $B C, C D$ 的中点, $E F$ 与 $A C$ 交于点 $O$, 点 $P$ 在平面 $A B C D$ 之外, $M$ 是线段 $P A$ 上一动点, 若 $P C / /$ 平面 $M E F$, 试求 $P M: M A$ 的值.\n\n\n\n解 如图,连接 $B D$ 交 $A C$ 于点 $O_{1}$ ,连接 $O M$.\n\n\n\n因为 $P C / /$ 平面 $M E F$ ,平面 $P A C \\cap$ 平面 $M E F=O M , P C \\subset$ 平面 $P A C$ ,\n\n所以 $P C / / O M$ ,所以 $\\frac{P M}{P A}=\\frac{O C}{A C}$.\n\n在菱形 $A B C D$ 中,\n\n因为 $E, F$ 分别是边 $B C, C D$ 的中点,所以 $\\frac{O C}{O_{1} C}=\\frac{1}{2}$.\n\n又 $A O_{1}=C O_{1}$, 所以 $\\frac{P M}{P A}=\\frac{O C}{A C}=\\frac{1}{4}$,\n\n故 $P M: M A=1: 3$."} {"id": "15495", "image": ["7712.jpg"], "answer": "因为 $B E / / A A_{1}$ ,\n\n$A A_{1} \\subset$ 平面 $A A_{1} D , B E$ 平面 $A A_{1} D ,$\n\n所以 $B E / /$ 平面 $A A_{1} D$.\n\n因为 $B C / / A D, A D \\subset$ 平面 $A A_{1} D$ ,\n\n$B C 4$ 平面 $A A_{1} D$ ,所以 $B C / /$ 平面 $A A_{1} D$.\n\n又 $B E \\cap B C=B, B E \\subset$ 平面 $B C E, B C \\subset$ 平面 $B C E$ ,\n所以平面 $B C E / /$ 平面 $A A_{1} D$.\n\n又平面 $A_{1} D C E \\cap$ 平面 $B C E=E C$ ,\n\n平面 $A_{1} D C E \\cap$ 平面 $A A_{1} D=A_{1} D$ ,\n\n所以 $E C / / A_{1} D$.", "solution": "null", "level": "高一", "question": "如图, 在四棱柱 $A B C D-A_{1} B_{1} C_{1} D_{1}$ 中, 底面 $A B C D$ 为梯形, $A D / / B C$, 平面 $A_{1} D C E$ 与 $B_{1} B$ 交于点 $E$.求证: $E C / / A_{1} D$.\n\n", "options": [], "subject": "立体几何学", "analysis": "因为 $B E / / A A_{1}$ ,\n\n$A A_{1} \\subset$ 平面 $A A_{1} D , B E$ 平面 $A A_{1} D ,$\n\n所以 $B E / /$ 平面 $A A_{1} D$.\n\n因为 $B C / / A D, A D \\subset$ 平面 $A A_{1} D$ ,\n\n$B C 4$ 平面 $A A_{1} D$ ,所以 $B C / /$ 平面 $A A_{1} D$.\n\n又 $B E \\cap B C=B, B E \\subset$ 平面 $B C E, B C \\subset$ 平面 $B C E$ ,\n所以平面 $B C E / /$ 平面 $A A_{1} D$.\n\n又平面 $A_{1} D C E \\cap$ 平面 $B C E=E C$ ,\n\n平面 $A_{1} D C E \\cap$ 平面 $A A_{1} D=A_{1} D$ ,\n\n所以 $E C / / A_{1} D$."} {"id": "15507", "image": ["7721.jpg"], "answer": "在 $\\triangle P A D$ 中,由 $P A=2, A D=2, P D=2 \\sqrt{2}$ ,\n\n可得 $P A^{2}+A D^{2}=P D^{2}$ ,即 $A D \\perp P A$.\n\n又 $A D \\perp A B, P A \\cap A B=A, P A, A B \\subset$ 平面 $P A B$ ,\n\n所以 $A D \\perp$ 平面 $P A B$.", "solution": "null", "level": "高一", "question": "如图所示, 在四棱雉 $P-A B C D$ 中, 底面 $A B C D$ 是矩形. 已知 $A D=2, P A=2, P D=2 \\sqrt{2}$,求证: $A D \\perp$ 平面 $P A B$.\n\n", "options": [], "subject": "立体几何学", "analysis": "在 $\\triangle P A D$ 中,由 $P A=2, A D=2, P D=2 \\sqrt{2}$ ,\n\n可得 $P A^{2}+A D^{2}=P D^{2}$ ,即 $A D \\perp P A$.\n\n又 $A D \\perp A B, P A \\cap A B=A, P A, A B \\subset$ 平面 $P A B$ ,\n\n所以 $A D \\perp$ 平面 $P A B$."} {"id": "15508", "image": ["7722.jpg"], "answer": ")证明 $\\because A B C D-A_{1} B_{1} C_{1} D_{1}$ 为正方体,\n\n$\\therefore B B_{1} \\perp$ 平面 $A B C D$.\n\n$\\because$ 又 $A C \\subset$ 平面 $A B C D , \\therefore B B_{1} \\perp A C$.\n\n又 $\\because$ 底面 $A B C D$ 为正方形,\n\n$\\therefore A C \\perp B D$.\n\n$\\because B B_{1} \\cap B D=B, B B_{1}, B D \\subset$ 平面 $B B_{1} D ,$\n\n$\\therefore A C \\perp$ 平面 $B B_{1} D$.\n\n$\\because B_{1} D \\subset$ 平面 $B D B_{1} , \\therefore A C \\perp B_{1} D$.\n\n(2)解 $V_{C-B D B_{1}}=V_{B_{1}-B D C}$.\n\n$\\because B_{1} B \\perp$ 平面 $A B C D ,$\n\n$\\therefore B_{1} B$ 是三棱雉 $B_{1}-B D C$ 的高.\n\n$\\because V_{B_{1}-B D C}=\\frac{1}{3} S_{\\triangle B D C} \\cdot B B_{1}=\\frac{1}{3} \\times \\frac{1}{2} \\times 2 \\times 2 \\times 2=\\frac{4}{3} ,$\n\n$\\therefore$ 三棱雉 $C-B D B_{1}$ 的体积为 $\\frac{4}{3}$.", "solution": "null", "level": "高一", "question": "如图, 正方体 $A B C D-A_{1} B_{1} C_{1} D_{1}$ 的棱长为 2 .\n\n\n\n(1)求证: $A C \\perp B_{1} D$;\n\n(2)求三棱雉 $C-B D B_{1}$ 的体积.", "options": [], "subject": "立体几何学", "analysis": ")证明 $\\because A B C D-A_{1} B_{1} C_{1} D_{1}$ 为正方体,\n\n$\\therefore B B_{1} \\perp$ 平面 $A B C D$.\n\n$\\because$ 又 $A C \\subset$ 平面 $A B C D , \\therefore B B_{1} \\perp A C$.\n\n又 $\\because$ 底面 $A B C D$ 为正方形,\n\n$\\therefore A C \\perp B D$.\n\n$\\because B B_{1} \\cap B D=B, B B_{1}, B D \\subset$ 平面 $B B_{1} D ,$\n\n$\\therefore A C \\perp$ 平面 $B B_{1} D$.\n\n$\\because B_{1} D \\subset$ 平面 $B D B_{1} , \\therefore A C \\perp B_{1} D$.\n\n(2)解 $V_{C-B D B_{1}}=V_{B_{1}-B D C}$.\n\n$\\because B_{1} B \\perp$ 平面 $A B C D ,$\n\n$\\therefore B_{1} B$ 是三棱雉 $B_{1}-B D C$ 的高.\n\n$\\because V_{B_{1}-B D C}=\\frac{1}{3} S_{\\triangle B D C} \\cdot B B_{1}=\\frac{1}{3} \\times \\frac{1}{2} \\times 2 \\times 2 \\times 2=\\frac{4}{3} ,$\n\n$\\therefore$ 三棱雉 $C-B D B_{1}$ 的体积为 $\\frac{4}{3}$."} {"id": "15520", "image": ["7731.jpg"], "answer": "(1)由直三棱柱 $A B C-A_{1} B_{1} C_{1}$ , 得 $A_{1} B_{1} / / A B$.\n\n因为 $A_{1} B_{1} 4$ 平面 $A B D , A B \\subset$ 平面 $A B D$ ,\n\n所以直线 $A_{1} B_{1} / /$ 平面 $A B D$.\n\n(2)因为三棱柱 $A B C-A_{1} B_{1} C_{1}$ 为直三棱柱,所以 $A B \\perp B B_{1}$.\n\n又因为 $A B \\perp B C , B B_{1} \\subset$ 平面 $B C C_{1} B_{1}, B C \\subset$ 平面 $B C C_{1} B_{1}$ ,且 $B B_{1} \\cap B C=B$ ,所以 $A B \\perp$ 平面 $B C C_{1} B_{1}$.\n\n又因为 $A B \\subset$ 平面 $A B D$ ,所以平面 $A B D \\perp$ 平面 $B C C_{1} B_{1}$.", "solution": "null", "level": "高一", "question": "在直三棱柱 $A B C-A_{1} B_{1} C_{1}$ 中, $A B \\perp B C, D$ 为棱 $C C_{1}$ 上任一点.\n\n(1)求证:直线 $A_{1} B_{1} / /$ 平面 $A B D$;\n\n(2)求证: 平面 $A B D \\perp$ 平面 $B C C_{1} B_{1}$.\n\n", "options": [], "subject": "立体几何学", "analysis": "(1)由直三棱柱 $A B C-A_{1} B_{1} C_{1}$ , 得 $A_{1} B_{1} / / A B$.\n\n因为 $A_{1} B_{1} 4$ 平面 $A B D , A B \\subset$ 平面 $A B D$ ,\n\n所以直线 $A_{1} B_{1} / /$ 平面 $A B D$.\n\n(2)因为三棱柱 $A B C-A_{1} B_{1} C_{1}$ 为直三棱柱,所以 $A B \\perp B B_{1}$.\n\n又因为 $A B \\perp B C , B B_{1} \\subset$ 平面 $B C C_{1} B_{1}, B C \\subset$ 平面 $B C C_{1} B_{1}$ ,且 $B B_{1} \\cap B C=B$ ,所以 $A B \\perp$ 平面 $B C C_{1} B_{1}$.\n\n又因为 $A B \\subset$ 平面 $A B D$ ,所以平面 $A B D \\perp$ 平面 $B C C_{1} B_{1}$."} {"id": "15521", "image": ["7732.jpg"], "answer": "(1) $\\because$ 四边形 $A B C D$ 是正方形,\n\n$\\therefore E$ 是 $B D$ 的中点.\n\n又 $F$ 是 $P B$ 的中点,\n\n$\\therefore E F / / P D$.\n\n又 $\\because E F \\Phi$ 平面 $P C D , P D \\subset$ 平面 $P C D ,$\n\n$\\therefore E F / /$ 平面 $P C D$.\n\n(2) $\\because$ 四边形 $A B C D$ 是正方形, $\\therefore B D \\perp A C$.\n\n$\\because P A \\perp$ 平面 $A B C , B D \\subset$ 平面 $A B C ,$\n\n$\\therefore P A \\perp B D$.\n\n又 $P A \\cap A C=A, P A, A C \\subset$ 平面 $P A C$ ,\n\n$\\therefore B D \\perp$ 平面 $P A C$.\n\n又 $B D \\subset$ 平面 $P B D ,$\n\n$\\therefore$ 平面 $P B D \\perp$ 平面 $P A C$.", "solution": "null", "level": "高一", "question": "如图, 四棱雉 $P-A B C D$ 的底面 $A B C D$ 为正方形, $P A \\perp$ 底面 $A B C D, A C, B D$ 交于点 $E$, $F$ 是 $P B$ 的中点. 求证:\n\n\n\n(1)EF// 平面 $P C D$;\n\n(2)平面 $P B D \\perp$ 平面 $P A C$.", "options": [], "subject": "立体几何学", "analysis": "(1) $\\because$ 四边形 $A B C D$ 是正方形,\n\n$\\therefore E$ 是 $B D$ 的中点.\n\n又 $F$ 是 $P B$ 的中点,\n\n$\\therefore E F / / P D$.\n\n又 $\\because E F \\Phi$ 平面 $P C D , P D \\subset$ 平面 $P C D ,$\n\n$\\therefore E F / /$ 平面 $P C D$.\n\n(2) $\\because$ 四边形 $A B C D$ 是正方形, $\\therefore B D \\perp A C$.\n\n$\\because P A \\perp$ 平面 $A B C , B D \\subset$ 平面 $A B C ,$\n\n$\\therefore P A \\perp B D$.\n\n又 $P A \\cap A C=A, P A, A C \\subset$ 平面 $P A C$ ,\n\n$\\therefore B D \\perp$ 平面 $P A C$.\n\n又 $B D \\subset$ 平面 $P B D ,$\n\n$\\therefore$ 平面 $P B D \\perp$ 平面 $P A C$."} {"id": "15533", "image": ["7741.jpg"], "answer": "$\\because$ 平面 $P A C \\perp$ 平面 $A B C$ ,平面 $P A C \\cap$ 平面 $A B C=A C , P A \\perp A C , P A C$ 平面 $P A C$ ,\n\n$\\therefore P A \\perp$ 平面 $A B C$.\n\n又 $B C \\subset$ 平面 $A B C$ ,\n\n$\\therefore P A \\perp B C$.\n\n又 $\\because A B \\perp B C , A B \\cap P A=A, A B \\subset$ 平面 $P A B$ ,\n\n$P A \\subset$ 平面 $P A B$,\n\n$\\therefore B C \\perp$ 平面 $P A B$.\n又 $B C \\subset$ 平面 $P B C$ ,\n\n$\\therefore$ 平面 $P A B \\perp$ 平面 $P B C$.", "solution": "null", "level": "高一", "question": "如图, 三棱雉 $P-A B C$ 中, 已知 $\\triangle A B C$ 是等腰直角三角形, $\\angle A B C=90^{\\circ}, \\triangle P A C$ 是直角三角形, $\\angle P A C=90^{\\circ}, \\angle A C P=30^{\\circ}$, 平面 $P A C \\perp$ 平面 $A B C$,\n\n\n\n求证: 平面 $P A B \\perp$ 平面 $P B C$.", "options": [], "subject": "立体几何学", "analysis": "$\\because$ 平面 $P A C \\perp$ 平面 $A B C$ ,平面 $P A C \\cap$ 平面 $A B C=A C , P A \\perp A C , P A C$ 平面 $P A C$ ,\n\n$\\therefore P A \\perp$ 平面 $A B C$.\n\n又 $B C \\subset$ 平面 $A B C$ ,\n\n$\\therefore P A \\perp B C$.\n\n又 $\\because A B \\perp B C , A B \\cap P A=A, A B \\subset$ 平面 $P A B$ ,\n\n$P A \\subset$ 平面 $P A B$,\n\n$\\therefore B C \\perp$ 平面 $P A B$.\n又 $B C \\subset$ 平面 $P B C$ ,\n\n$\\therefore$ 平面 $P A B \\perp$ 平面 $P B C$."} {"id": "15534", "image": ["7742.jpg"], "answer": "(1) $\\because$ 平面 $P A D \\perp$ 底面 $A B C D ,$ 平面 $P A D \\cap$ 底面 $A B C D=A D , P A \\subset$ 平面 $P A D , P A \\perp A D ,$\n\n$\\therefore P A \\perp$ 底面 $A B C D$.\n\n(2) $\\because A B / / C D, C D=2 A B, E$ 是 $C D$ 的中点,\n\n$\\therefore A B / / D E$ ,且 $A B=D E$ ,\n\n$\\therefore$ 四边形 $A B E D$ 为平行四边形. $\\therefore B E / / A D$.\n\n又 $\\because B E$ 平面 $P A D, A D \\subset$ 平面 $P A D, \\therefore B E / /$ 平面 $P A D$.\n\n(3) $\\because A B \\perp A D$ ,四边形 $A B E D$ 为矩形,\n\n$\\therefore B E \\perp C D, A D \\perp C D$.\n\n由(1)知 $P A \\perp$ 底面 $A B C D , \\therefore P A \\perp C D$.\n\n$\\because P A \\cap A D=A, P A, A D \\subset$ 平面 $P A D ,$\n\n$\\therefore C D \\perp$ 平面 $P A D , \\therefore C D \\perp P D$.\n\n$\\because E$ 和 $F$ 分别是 $C D$ 和 $P C$ 的中点,\n\n$\\therefore P D / / E F, \\therefore C D \\perp E F$.\n\n$\\because C D \\perp B E, E F \\cap B E=E, E F, B E \\subset$ 平面 $B E F, \\therefore C D \\perp$ 平面 $B E F$.\n\n$\\because C D \\subset$ 平面 $P C D , \\therefore$ 平面 $B E F \\perp$ 平面 $P C D$.", "solution": "null", "level": "高一", "question": "如图, 在四棱雉 $P-A B C D$ 中, $A B / / C D, A B \\perp A D, C D=2 A B$, 平面 $P A D \\perp$ 底面 $A B C D$, $P A \\perp A D, E$ 和 $F$ 分别是 $C D$ 和 $P C$ 的中点.\n\n\n\n求证:\n\n(1)PA $\\perp$ 底面 $A B C D$;\n\n(2) $B E / /$ 平面 $P A D$;\n\n(3)平面 $B E F \\perp$ 平面 $P C D$.", "options": [], "subject": "立体几何学", "analysis": "(1) $\\because$ 平面 $P A D \\perp$ 底面 $A B C D ,$ 平面 $P A D \\cap$ 底面 $A B C D=A D , P A \\subset$ 平面 $P A D , P A \\perp A D ,$\n\n$\\therefore P A \\perp$ 底面 $A B C D$.\n\n(2) $\\because A B / / C D, C D=2 A B, E$ 是 $C D$ 的中点,\n\n$\\therefore A B / / D E$ ,且 $A B=D E$ ,\n\n$\\therefore$ 四边形 $A B E D$ 为平行四边形. $\\therefore B E / / A D$.\n\n又 $\\because B E$ 平面 $P A D, A D \\subset$ 平面 $P A D, \\therefore B E / /$ 平面 $P A D$.\n\n(3) $\\because A B \\perp A D$ ,四边形 $A B E D$ 为矩形,\n\n$\\therefore B E \\perp C D, A D \\perp C D$.\n\n由(1)知 $P A \\perp$ 底面 $A B C D , \\therefore P A \\perp C D$.\n\n$\\because P A \\cap A D=A, P A, A D \\subset$ 平面 $P A D ,$\n\n$\\therefore C D \\perp$ 平面 $P A D , \\therefore C D \\perp P D$.\n\n$\\because E$ 和 $F$ 分别是 $C D$ 和 $P C$ 的中点,\n\n$\\therefore P D / / E F, \\therefore C D \\perp E F$.\n\n$\\because C D \\perp B E, E F \\cap B E=E, E F, B E \\subset$ 平面 $B E F, \\therefore C D \\perp$ 平面 $B E F$.\n\n$\\because C D \\subset$ 平面 $P C D , \\therefore$ 平面 $B E F \\perp$ 平面 $P C D$."} {"id": "17010", "image": [], "answer": "解: (I) 从这 6 个点中随机选取 4 个点的所有可能结果\n\n与从这 6 个点中随机选取 2 个点的所有可能结果相同,\n\n即 $A B_{1}, A D_{1}, A E, A F, A G, B_{1} D_{1} B_{1} E, B_{1} F, B_{1} G, D_{1} E, D_{1} F, D_{1} G$,\n\n$E F, E G, F G$ ,共 15 种,\n\n根据题意可知, 四点共面的情况只有 $A E G D_{1}, B_{1} F G D, A E F B_{1}$, 共 3 种,\n\n故四点共面的概率为 $\\frac{3}{15}=\\frac{1}{5}$;\n\n(II) 根据题意可知, 点 $P$ 为正方形 $A B C D$ 内的任意以点,\n\n故点 $P$ 的轨迹面积为 4 ,\n\n$\\because$ 满足条件的点 $P$ 在以 $A_{1}$ 为球心, $\\sqrt{6}$ 为半径的球内,\n\n故 $A_{1} P \\leq \\sqrt{6}$, 即 $\\sqrt{A A_{1}{ }^{2}+A P^{2}} \\leq \\sqrt{6}$, 故 $A P \\leq \\sqrt{2}$,\n\n故满足条件的点 $P$ 的轨迹面积为 $\\frac{1}{4} \\pi \\times(\\sqrt{2})^{2}=\\frac{\\pi}{2}$,\n\n故所求的概率为 $\\frac{\\pi}{4}=\\frac{\\pi}{8}$.", "solution": "null", "level": "高一", "question": "(2020 秋 - 河南期中)已知在边长为 2 的正方体 $A B C D-A_{1} B_{1} C_{1} D_{1}$ 、中, 点 $E, F, G$ 分别为 $A A_{1}$, $A_{1} B_{1}, A_{1} D_{1}$ 的中点.\n\n( I ) 从 $A, B_{1}, D_{1}, E, F, G$ 这六个点中任取四点, 求这四点共面的概率;\n\n(II ) 点 $P$ 为正方形 $A B C D$ 内的任意一点, 求点 $P$ 在以 $A_{1}$ 为球心, $\\sqrt{6}$ 为半径的球内的概率.", "options": [], "subject": "立体几何学", "analysis": "解: (I) 从这 6 个点中随机选取 4 个点的所有可能结果\n\n与从这 6 个点中随机选取 2 个点的所有可能结果相同,\n\n即 $A B_{1}, A D_{1}, A E, A F, A G, B_{1} D_{1} B_{1} E, B_{1} F, B_{1} G, D_{1} E, D_{1} F, D_{1} G$,\n\n$E F, E G, F G$ ,共 15 种,\n\n根据题意可知, 四点共面的情况只有 $A E G D_{1}, B_{1} F G D, A E F B_{1}$, 共 3 种,\n\n故四点共面的概率为 $\\frac{3}{15}=\\frac{1}{5}$;\n\n(II) 根据题意可知, 点 $P$ 为正方形 $A B C D$ 内的任意以点,\n\n故点 $P$ 的轨迹面积为 4 ,\n\n$\\because$ 满足条件的点 $P$ 在以 $A_{1}$ 为球心, $\\sqrt{6}$ 为半径的球内,\n\n故 $A_{1} P \\leq \\sqrt{6}$, 即 $\\sqrt{A A_{1}{ }^{2}+A P^{2}} \\leq \\sqrt{6}$, 故 $A P \\leq \\sqrt{2}$,\n\n故满足条件的点 $P$ 的轨迹面积为 $\\frac{1}{4} \\pi \\times(\\sqrt{2})^{2}=\\frac{\\pi}{2}$,\n\n故所求的概率为 $\\frac{\\pi}{4}=\\frac{\\pi}{8}$."} {"id": "16550", "image": [], "answer": "PUT \"a, $\\mathrm{h}=$ \" $; \\mathrm{a}, \\mathrm{h}$\n$\\mathrm{V}=\\mathrm{SQR}(3) * \\mathrm{a}^{\\wedge} 2 * \\mathrm{~h} / 4$\n$\\mathrm{~S}=\\mathrm{SQR}(3) * \\mathrm{a}^{\\wedge} 2 / 2+3 * \\mathrm{a} * \\mathrm{~h}$\nPRINT V, S\nEND", "solution": "null", "level": "高一", "question": "已知正三棱柱 (底面为正三角形的直三棱柱) 的高为 $\\mathrm{h}$, 底面边长为 $\\mathrm{a}$, 编写程序求正三棱柱的体积和表面积。", "options": [], "subject": "立体几何学", "analysis": "PUT \"a, $\\mathrm{h}=$ \" $; \\mathrm{a}, \\mathrm{h}$\n$\\mathrm{V}=\\mathrm{SQR}(3) * \\mathrm{a}^{\\wedge} 2 * \\mathrm{~h} / 4$\n$\\mathrm{~S}=\\mathrm{SQR}(3) * \\mathrm{a}^{\\wedge} 2 / 2+3 * \\mathrm{a} * \\mathrm{~h}$\nPRINT V, S\nEND"} {"id": "16594", "image": ["8592.jpg", "8592.jpg"], "answer": "第一步, $S=0$.\n\n第二步, $i=2$.\n\n第三步, $S=S+i$\n\n第四步, $i=i+2$.\n第五步, 如果 $i>100$ 成立, 则输出 $S$; 否则, 返回第三步。\n\n相应的程序框图如图所示\n\n", "solution": "null", "level": "高一", "question": "已知一个圆柱的底面半径为 $\\mathrm{R}$, 高为 $\\mathrm{h}$, 求圆柱的体积. 设计一个解决该问题的算法,并画出相应的程序框图.", "options": [], "subject": "立体几何学", "analysis": "第一步, $S=0$.\n\n第二步, $i=2$.\n\n第三步, $S=S+i$\n\n第四步, $i=i+2$.\n第五步, 如果 $i>100$ 成立, 则输出 $S$; 否则, 返回第三步。\n\n相应的程序框图如图所示\n\n"} {"id": "16699", "image": [], "answer": ") $\\because V_{\\text {三蓺椎 } A B C-A_{1} B_{1} C_{1}}=\\frac{1}{2} a^{2} \\cdot a=\\frac{1}{2} a^{3}$, 所求概率 $P_{1}=\\frac{\\frac{1}{2} a^{3}}{a^{3}}=\\frac{1}{2}$\n\n(2) $\\because V_{\\text {三棱柱 } B-A_{1} B_{1} C_{1}}=\\frac{1}{3} \\cdot S_{\\triangle A_{1} B_{1} C_{1}} \\cdot B B_{1}=\\frac{1}{3} \\cdot \\frac{1}{2} a^{2} \\cdot a=\\frac{1}{6} a^{3}$, 所求概率 $P_{2}=\\frac{1}{6}$\n\n(3) 所求概率 $P_{3}=\\frac{a-\\frac{a}{3}}{a}=\\frac{2}{3}$\n\n(4) 所求概率 $P_{4}=\\frac{a-\\frac{a}{3}-\\frac{a}{3}}{a}=\\frac{1}{3}$", "solution": "null", "level": "高一", "question": "已知正方体 $A B C D-A_{1} B_{1} C_{1} D_{1}$ 的棱长为 $\\mathrm{a}$, 在正方体内随机取一点 $\\mathrm{M}$.\n\n(1) 求点 $M$ 落在三棱柱 $A B C-A_{1} B_{1} C_{1}$ 内的概率 $P_{1}$;\n\n(2) 求点 $M$ 落在三棱雉 $B-A_{1} B_{1} C_{1}$ 内的概率 $P_{2}$;\n\n(3) 求点 $\\mathrm{M}$ 到面 $\\mathrm{ABCD}$ 的距离大于号的概率 $P_{3}$;\n\n(4) 求点 $M$ 到面 $\\mathrm{ABCD}$ 及面 $A_{1} B_{1} C_{1} D_{1}$ 的距离都大于 $\\frac{a}{3}$ 的概率 $P_{4}$", "options": [], "subject": "立体几何学", "analysis": ") $\\because V_{\\text {三蓺椎 } A B C-A_{1} B_{1} C_{1}}=\\frac{1}{2} a^{2} \\cdot a=\\frac{1}{2} a^{3}$, 所求概率 $P_{1}=\\frac{\\frac{1}{2} a^{3}}{a^{3}}=\\frac{1}{2}$\n\n(2) $\\because V_{\\text {三棱柱 } B-A_{1} B_{1} C_{1}}=\\frac{1}{3} \\cdot S_{\\triangle A_{1} B_{1} C_{1}} \\cdot B B_{1}=\\frac{1}{3} \\cdot \\frac{1}{2} a^{2} \\cdot a=\\frac{1}{6} a^{3}$, 所求概率 $P_{2}=\\frac{1}{6}$\n\n(3) 所求概率 $P_{3}=\\frac{a-\\frac{a}{3}}{a}=\\frac{2}{3}$\n\n(4) 所求概率 $P_{4}=\\frac{a-\\frac{a}{3}-\\frac{a}{3}}{a}=\\frac{1}{3}$"} {"id": "15550", "image": ["7754.jpg", "7755.jpg"], "answer": "体不是棱柱;截去的部分是一个四棱雉;作图见解析", "solution": "null", "level": "高一", "question": "如图所示的几何体中, 四边形 $A A_{1} B_{1} B$ 是边长为 3 的正方形, $C C_{1}=2, C C_{1} / / A A_{1}$, 这个几何体是棱柱吗? 若是, 指出是几棱柱; 若不是, 请你试用一个平面截去一部分, 使剩余部分是一个侧棱长为 2 的三棱柱, 并指出截去的几何体的特征, 在立体图形中画出截面.\n\n", "options": [], "subject": "立体几何学", "analysis": "【分析】\n\n判断一个几何体是不是棱柱, 要从棱柱的结构特征出发进行推理和判断.\n\n## 【详解】\n\n这个几何体不是棱柱.因为没有平行的两个平面.\n\n\n\n在四边形 $A A_{1} B_{1} B$ 中, 在 $A A_{1}$ 上取点 $E$, 使 $A E=2$, 在 $B B_{1}$ 上取点 $F$, 使 $B F=2$, 连接 $C_{1} E, E F, F C_{1}$,\n则过点 $C_{1}, E, F$ 的截面将几何体分成两部分, 其中一部分是三棱柱 $A B C-E F C_{1}$, 其侧棱长为 2.截去的部分是一个四棱雉 $C_{1}-E A_{1} B_{1} F$, 如图所示.\n\n## 【点睛】\n\n本题主要考查棱柱的结构特征, 棱柱是有两个面平行, 其余相邻面的交线平行这两个典型特征, 侧重考查概念辨析能力."} {"id": "15551", "image": [], "answer": ") $\\frac{3}{4} \\pi$; (2) 2", "solution": "null", "level": "高一", "question": "已知 $O A$ 为球 $O$ 的半径, 过 $O A$ 的中点 $M$ 且垂直于 $O A$ 的平面截球面得到圆 $M$.\n\n(1) 若 $O A=1$, 求圆 $M$ 的面积;\n\n(2) 若圆 $M$ 的面积为 $3 \\pi$, 求 $O A$.", "options": [], "subject": "立体几何学", "analysis": "【分析】\n\n(1)根据球的性质, 截面的圆心为中点 $M$, 由已知, 赇出圆 $M$ 半径, 即可求解;\n\n(2)由 $M$ 的面积, 求出圆 $M$ 半径, 利用勾股定理, 即可求出结论.\n\n## 【详解】\n\n(1) 若 $O A=1$, 则 $O M=\\frac{1}{2}$\n\n故圆 $M$ 的半径 $r=\\sqrt{O A^{2}-O M^{2}}=\\sqrt{1^{2}-\\left(\\frac{1}{2}\\right)^{2}}=\\frac{\\sqrt{3}}{2}$\n\n所以圆 $M$ 的面积 $S=\\pi r^{2}=\\frac{3}{4} \\pi$\n\n(2) 因为圆 $M$ 的面积为 $3 \\pi$,\n\n所以圆 $M$ 的半径 $r=\\sqrt{3}$,\n\n则 $O A^{2}=\\left(\\frac{O A}{2}\\right)^{2}+3$\n\n所以 $\\frac{3}{4} O A^{2}=3$, 所以 $O A^{2}=4$,\n\n所以 $O A=2$\n\n## 【点睛】\n\n本题考查球的性质, 注意截面圆半径、球心与截面圆圆心连线以及球半径关系, 属于基础题."} {"id": "15552", "image": ["7756.jpg"], "answer": "frac{\\sqrt{2}}{2} \\mathrm{~cm}$.", "solution": "null", "level": "高一", "question": "圆雉底面半径为 $1 \\mathrm{~cm}$, 高为 $\\sqrt{2} \\mathrm{~cm}$, 其中有一个内接正方体, 求这个内接正方体的棱长.", "options": [], "subject": "立体几何学", "analysis": "试题分析:画出图形,设出棱长,根据三角形相似,列出比例关系, 求出棱长即可.\n\n试题解析:过圆雉的顶点 $S$ 和正方体底面的一条对角线 $C D$ 作圆雉的截面, 得圆锥的轴截面 $S E F$,正方体对角面 $C D D_{1} C_{1}$, 如图所示.\n\n设正方体棱长为 $x$, 恻 $C C_{1}=x, C_{1} D_{1}=\\sqrt{2} x$,\n\n作 $S O \\perp E F$ 于 $O$, 则 $S O=\\sqrt{2}, O E=1$,\n\n$\\because \\Delta E C C_{1} \\sim \\Delta E O S, \\therefore \\frac{C C_{1}}{S O}=\\frac{E C_{1}}{E O}$, 即 $\\frac{x}{\\sqrt{2}}=\\frac{1-\\frac{\\sqrt{2}}{2} x}{1}$,\n\n$\\therefore x=\\frac{\\sqrt{2}}{2} \\mathrm{~cm}$, 即内接正方体棱长为 $\\frac{\\sqrt{2}}{2} \\mathrm{~cm}$.\n\n\n\n考点: 简单组合体的结构特征."} {"id": "15553", "image": ["7757.jpg"], "answer": ") $h=\\sqrt{800-20 \\sqrt{2} x}, x \\in(0,20 \\sqrt{2})$; (2) $h_{\\text {max }}=20, \\frac{4000}{3}$.", "solution": "null", "level": "高一", "question": "小明设计了一款正四棱雉形状的包装盒, 如图所示, $A B C D$ 是边长为 $40 \\mathrm{~cm}$ 的正方形硬纸片,切去阴影部分所示的四个全等的等腰三角形, 再沿虚线折起, 使得 $A B C D$ 四个点重合于图中的点 $P$,正好形成一个正四棱雉形状的包装盒, 设正四棱雉底面正方形的边长为 $x \\mathrm{~cm}$.\n\n\n(1)试用 $X$ 表示该四棱雉的高度 $h$, 并指出 $X$ 的取值范围;\n\n(2)若要求侧面积不小于 $600 \\mathrm{~cm}^{2}$, 求该四棱雉的高度的最大值, 并指出此时该包装盒的容积.", "options": [], "subject": "立体几何学", "analysis": "【分析】\n\n(1) 设正四棱雉侧面等腰三角形高为 $h^{\\prime}$, 由正方形 $A B C D$, 可得 $x+2 h^{\\prime}=40 \\sqrt{2}$, 再由 $\\frac{x}{2}, h, h^{\\prime}$ 组成直角三角形, 即可得到 $X, h$ 关系, 进而求出 $X$ 的范围;\n\n(2) 利用 (1) 中 $x, h^{\\prime}$ 关系, 求出侧面积关于 $x$ 的函数, 进一步求出满足条件的 $X$ 范围, 可求出 $h$ 的最大值,即可求出结论.\n\n## 【详解】\n\n(1) 设正四棱雉侧面等腰三角形高为 $h^{\\prime}$, 在正方形 $A B C D$ 中, $x+2 h^{\\prime}=40 \\sqrt{2}, \\therefore h^{\\prime}=20 \\sqrt{2}-\\frac{x}{2}$,在四棱雉中, $h^{\\prime 2}=\\left(\\frac{x}{2}\\right)^{2}+h^{2}, \\therefore\\left(20 \\sqrt{2}-\\frac{x}{2}\\right)^{2}=\\frac{x^{2}}{4}+h^{2}$,\n\n$$\n\\begin{gathered}\nh^{2}=800-20 \\sqrt{2} x, \\therefore h=\\sqrt{800-20 \\sqrt{2} x} \\\\\n\\because h^{2}=800-20 \\sqrt{2} x>0, \\therefore 0", "options": [], "subject": "立体几何学", "analysis": "【分析】\n\n由题意, 可设上底面边长为 $x$, 利用题中所给侧面梯形面积列方程, 求 $x$ 值即可.\n\n## 【详解】\n\n$\\because A B=10, \\therefore A D=\\frac{\\sqrt{3}}{2} A B=5 \\sqrt{3}, O D=\\frac{1}{3} A D=\\frac{5 \\sqrt{3}}{3}$.\n\n: 设上底面的边长为 $x(x>0)$, 则 $O_{1} D_{1}=\\frac{\\sqrt{3}}{6} x$.\n\n如图所示, 连接 $O_{1} O$, 过 $D_{1}$ 作 $D_{1} H \\perp A D$ 于点 $H$, 则四边形 $O H D_{1} O_{1}$ 为矩形, 且 $O H=O_{1} D_{1}=\\frac{\\sqrt{3}}{6} x$.\n\n\n\n$\\therefore D H=O D-O H=\\frac{5 \\sqrt{3}}{3}-\\frac{\\sqrt{3}}{6} x$,\n\n在 Rt $\\triangle D_{1} D H$ 中, $D_{1} D=\\frac{D H}{\\cos 60^{\\circ}}=2\\left(\\frac{5 \\sqrt{3}}{3}-\\frac{\\sqrt{3}}{6} x\\right)$.\n\n四边形 $B_{1} C_{1} C B$ 的面积为 $\\frac{1}{2}\\left(B_{1} C_{1}+B C\\right) \\cdot D_{1} D$ ,\n\n$\\therefore \\frac{20 \\sqrt{3}}{3}=\\frac{1}{2}(x+10) \\times 2\\left(\\frac{5 \\sqrt{3}}{3}-\\frac{\\sqrt{3}}{6} x\\right)$,\n\n即 $40=(x+10)(10-x)$,\n\n$\\therefore x=2 \\sqrt{15}$, 故上底面的边长为 $2 \\sqrt{15}$.\n\n## 【点睛】\n\n本题考查正棱台几何性质, 空间想象能力, 计算能力, 属于中等题型."} {"id": "15570", "image": ["7778.jpg", "7779.jpg"], "answer": ") 由圆雉和圆台组合而成的简单组合体. (2) 由四棱柱和四棱雉组合而成的简单组合体.", "solution": "null", "level": "高一", "question": "如图, 说出图中两个几何体的结构特征.\n\n(1)\n\n\n\n(2)\n\n", "options": [], "subject": "立体几何学", "analysis": "【分析】\n\n逐一分析组合体是由哪些柱体、雉体或台体构成的.\n\n## 【详解】\n\n(1) 由圆雉和圆台组合而成的简单组合体\n\n(2) 由四棱柱和四棱雉组合.而成的简单组合体:\n\n【点睛】\n\n本题考查组合体的构成情况, 考查组合体结构特征, 属于基础题."} {"id": "15571", "image": ["7780.jpg", "7781.jpg"], "answer": ") 几何体由 6 个顶点、 12 条棱和 8 个面组成 (2) 几何体由 6 个顶点、 10 条棱和 6 个面组成", "solution": "null", "level": "高一", "question": "试指出图中组成各几何体的基本元素.\n\n\n\n(1)\n\n\n\n(2)", "options": [], "subject": "立体几何学", "analysis": "【分析】\n\n由组成空间几何体的基本元素为点、线、面可作答.\n\n## 【详解】\n\n(1) 是由两个四棱雉组成的,有 6 个顶点、 12 条棱和 8 个面组成.\n\n(2)是由两个锥体组合而成, 有 6 个顶点、 10 条棱和 6 个面组成.\n\n## 【点睛】\n\n本题考查对空间几何体的基本元素的认识,属于基础题."} {"id": "15572", "image": ["7782.jpg", "7783.jpg", "7784.jpg", "7785.jpg", "7786.jpg"], "answer": "析", "solution": "null", "level": "高一", "question": "如图, 四边形 $A B C D$ 为直角梯形, 试作出绕其各条边所在的直线旋转所得到的几何体.\n\n", "options": [], "subject": "立体几何学", "analysis": "【分析】\n\n确定旋转直线, 根据其余各边与旋转直线的关系, 结合圆柱、圆雉、圆台定义, 即可求出结论.\n\n## 【详解】\n\n以边 $A D$ 所在直线为轴旋转, 形成的几何体是一个圆台,\n\n如图 (1) 所示.\n\n以边 $A B$ 所在直线为轴旋转, 形成的几何体可以看作是由一个圆雉和一个圆柱拼接而成的组合体, 如图 (2) 所示.以边 $C D$ 所在直线为轴旋转, 形成的几何体可以看作是由一个圆柱挖去一个同底圆雉而成的组合体,如图(3)所示.以边 $B C$ 所在直线为轴旋转, 形成的几何体可以看作是由一个圆台挖去一个同底(上底面)圆锥后再和一个同底(下底面)圆雉拼接而成的组合体,如图(4)所示.\n\n\n\n(1)\n\n\n\n(2)\n\n\n\n(3)\n\n\n\n(4)\n\n## 【点睛】\n\n本题考查旋转组合体, 要掌握常见简单旋转体的结构特征, 并能与组合体的结构特征进行恰当联系,实现“化未知为已知”"} {"id": "15573", "image": ["7787.jpg", "7788.jpg"], "answer": "析.", "solution": "null", "level": "高一", "question": "一个三棱柱可以分制成几个以三棱柱的顶点为顶点的三棱雉? 试在如图所示的三棱柱 $A B C-A_{1} B_{1} C_{1}$ 中设计出分割方案. (请用尽可能多的方法)\n\n", "options": [], "subject": "立体几何学", "analysis": "【分析】\n\n根据三棱柱的结构特征, 一个三棱柱可以分割成 3 个三棱雉, 即可求解, 得到答案.\n\n## 【详解】\n\n一个三棱柱可以分割成 3 个三棱雉, 可以有以下六种方案供选择.\n\n\n## 【点睛】\n\n本题主要考查了三棱柱和三棱雉的结构特征, 其中解答中正确把握三棱柱和三棱雉的结构特征是解答的关键, 着重考查了空间想象能力, 属于基础题."} {"id": "16490", "image": [], "answer": "(1) 不是简单随机抽样, 因为简单随机抽样要求被抽取的样本总体的个数是有限的.\n\n(2)不是简单随机抽样,因为简单随机抽样要求逐个不放回地抽取.\n\n(3)不是简单随机抽样, 因为这 10 名跳水队员是挑选出来的(最优秀的), 每个个体被抽到的可能性不同,不符合简单随机抽样中 “等可能性” 的要求。\n\n(4)是简单随机抽样, 因为总体中的个体数是有限的, 并且是从总体中逐个进行抽取的,是不放回、等可能的抽样。", "solution": "null", "level": "高一", "question": "下面的抽样方法是简单随机抽样吗? 为什么?\n\n(1) 从无数个个体中抽取 50 个个体作为样本;\n\n(2) 质量监督部门从 180 种儿童玩具中选出 18 种玩具进行质量检验, 在抽样过程中,从中任取一种玩具检验后再放回;\n\n(3)国家跳水队挑出最优秀的 10 名跳水队员,备战 2016 年里约热内卢奥运会;\n\n(4)一彩民选号,从装有 36 个大小、形状都相同的号签的盒子中无放回地逐个抽出 7 个号签.", "options": [], "subject": "计数", "analysis": "(1) 不是简单随机抽样, 因为简单随机抽样要求被抽取的样本总体的个数是有限的.\n\n(2)不是简单随机抽样,因为简单随机抽样要求逐个不放回地抽取.\n\n(3)不是简单随机抽样, 因为这 10 名跳水队员是挑选出来的(最优秀的), 每个个体被抽到的可能性不同,不符合简单随机抽样中 “等可能性” 的要求。\n\n(4)是简单随机抽样, 因为总体中的个体数是有限的, 并且是从总体中逐个进行抽取的,是不放回、等可能的抽样。"} {"id": "16491", "image": [], "answer": "解析第一步, 将原来的编号调整为 $001,002, .,, 112$.\n\n第二步, 在随机数表(见教材第 103 页)中任选一个数字作为开始数字, 任选一个方向作为读数方向. 比如, 选第 9 行第 7 列的数 3 作为开始\n\n第三步, 从 3 开始, 向右读, 每次读取三位数字, 凡不在 001-112 中的数跳过去不读,前面已经读过的数也跳过去不读,依次可得到 074,100,094,052,080,003,105, 10, 083, 092 .\n\n第四步, 抽取对应原来编号为 $74,100,94,52,80,3,105,107,83,92$ 的机器.", "solution": "null", "level": "高一", "question": "有一批机器, 编号为 $1,2,3, ., 112$. 用随机数表法抽取 10 台人样, 写出抽取过程.", "options": [], "subject": "计数", "analysis": "解析第一步, 将原来的编号调整为 $001,002, .,, 112$.\n\n第二步, 在随机数表(见教材第 103 页)中任选一个数字作为开始数字, 任选一个方向作为读数方向. 比如, 选第 9 行第 7 列的数 3 作为开始\n\n第三步, 从 3 开始, 向右读, 每次读取三位数字, 凡不在 001-112 中的数跳过去不读,前面已经读过的数也跳过去不读,依次可得到 074,100,094,052,080,003,105, 10, 083, 092 .\n\n第四步, 抽取对应原来编号为 $74,100,94,52,80,3,105,107,83,92$ 的机器."} {"id": "16494", "image": [], "answer": "用随机数表法, 步骤如下:\n\n第一步, 先对职工编号, 老年职工的编号为 $001,002,, 0303$; 中年职工的编号为 031 , $032, \\ldots, 080$; 青年职工的编号为 $081, \\ldots, 120$.\n\n第二步, 在随机数表中任选一个数作为开始, 任选一个方向作为读数方向, 例如选第 15 行第 6 列的数 4 开始向右读.\n\n第三步, 从选定的数 4 开始向右读, 每次读取三位, 凡不在 001-120 中的数跳过去不读,前面已经读过的也跳过去不读, 从 $001 \\sim 030$ 中选 3 个号码, 从 031 080 中选 5 个号码,从 081-120 中选 4 个号码, 依得到 038, 119, 033, 099, 004,047,094, 116, 044, 068, 013, 030 .\n\n第四步, 对应 004, 013,030 找出老年职工代表; 对应 033,038,044,047,068 找出中年职工代表;对应 094,099,116,119 找出青年职工代表.", "solution": "null", "level": "高一", "question": "某单位有老年职工 30 人,中年职工 50 人,青年职工 40 人,若分别从老年职工、中年职工、青年职工中随机抽取 3 人、 5 人、 4 人举行会议, 请选用合适的方法进行抽样,并写出抽样过程.", "options": [], "subject": "计数", "analysis": "用随机数表法, 步骤如下:\n\n第一步, 先对职工编号, 老年职工的编号为 $001,002,, 0303$; 中年职工的编号为 031 , $032, \\ldots, 080$; 青年职工的编号为 $081, \\ldots, 120$.\n\n第二步, 在随机数表中任选一个数作为开始, 任选一个方向作为读数方向, 例如选第 15 行第 6 列的数 4 开始向右读.\n\n第三步, 从选定的数 4 开始向右读, 每次读取三位, 凡不在 001-120 中的数跳过去不读,前面已经读过的也跳过去不读, 从 $001 \\sim 030$ 中选 3 个号码, 从 031 080 中选 5 个号码,从 081-120 中选 4 个号码, 依得到 038, 119, 033, 099, 004,047,094, 116, 044, 068, 013, 030 .\n\n第四步, 对应 004, 013,030 找出老年职工代表; 对应 033,038,044,047,068 找出中年职工代表;对应 094,099,116,119 找出青年职工代表."} {"id": "16505", "image": [], "answer": "我们可采用系统抽样, 方案如下:\n\n第一步, 由于 $1000=33 \\times 30+10$, 所以先用简单随机抽样的方法从总体中抽取 10 台电视机, 不进行检验。\n\n第二步, 将剩下的电视机进行编号, 编号分别为 $0,1,2 . \\cdots, 989$, 并将其按编号的先后顺序分为 30 组, 每组 33 台电视机.\n\n第三步, 从第 1 组(编号为 $0,1,2,3 \\cdots, 32$ )的电视机中按照简单随机抽样的方法抽取 1 台电视机, 记其编号为 $\\mathrm{k}$ 第四步, 按顺序抽取编号分别为 $\\mathrm{k}+33, \\mathrm{k}+66, \\mathrm{k}+99 . \\cdots \\mathrm{k}+29$ $\\times 33$ 的电视机, 这样总共抽取了 30 台电视机, 对这 30 台电视机进行检验.", "solution": "null", "level": "高一", "question": "某电视机厂每天生产 1000 台电视机, 要求质检员每天抽取 30 台, 检查其质量状况,请你设计一个调查方案", "options": [], "subject": "计数", "analysis": "我们可采用系统抽样, 方案如下:\n\n第一步, 由于 $1000=33 \\times 30+10$, 所以先用简单随机抽样的方法从总体中抽取 10 台电视机, 不进行检验。\n\n第二步, 将剩下的电视机进行编号, 编号分别为 $0,1,2 . \\cdots, 989$, 并将其按编号的先后顺序分为 30 组, 每组 33 台电视机.\n\n第三步, 从第 1 组(编号为 $0,1,2,3 \\cdots, 32$ )的电视机中按照简单随机抽样的方法抽取 1 台电视机, 记其编号为 $\\mathrm{k}$ 第四步, 按顺序抽取编号分别为 $\\mathrm{k}+33, \\mathrm{k}+66, \\mathrm{k}+99 . \\cdots \\mathrm{k}+29$ $\\times 33$ 的电视机, 这样总共抽取了 30 台电视机, 对这 30 台电视机进行检验."} {"id": "16506", "image": [], "answer": "先将 2003 名学生按 0001 到 2003 编号, 利用随机数表法从中剔除 3 名学生,再对剩余的 2000 名学生重新从 0001 到 2000 编号, 按编号顺序分成 40 组, 每组 50 人,先在第一组中用抽签法抽出某一号, 如 0006, 依次在其他组抽取 0056, 0106 “, 1956,这样就得到了一个容量为 40 的样本。", "solution": "null", "level": "高一", "question": "从 2003 名学生中抽取一个容量为 40 的样本, 应如何抽取?", "options": [], "subject": "计数", "analysis": "先将 2003 名学生按 0001 到 2003 编号, 利用随机数表法从中剔除 3 名学生,再对剩余的 2000 名学生重新从 0001 到 2000 编号, 按编号顺序分成 40 组, 每组 50 人,先在第一组中用抽签法抽出某一号, 如 0006, 依次在其他组抽取 0056, 0106 “, 1956,这样就得到了一个容量为 40 的样本。"} {"id": "16507", "image": [], "answer": "适宜选用系统抽样,抽样过程如下:\n\n(1) 随机地将这 1003 个个体编号为 1,2,3\\%, 1003.\n\n(2)利用简单随机抽样,先从总体中随机剔除 3 个个体,剩下的个体数 1000 能被样本容量 50 整除, 然后将 1000 个个体重新编号为 $1,2,3 \\cdots, 1000$.\n\n(3)将总体按编号顺序均分成 50 部分,每部分包含 20 个个体,\n\n(4) 在编号为 $1,2,3 \\cdots, 20$ 的第一部分个体中, 利用简单随机抽样抽取一个号码, 比如抽取的号码是 18.\n\n(5) 以 18 为起始号码, 这样得到一个容量为 50 的样本: 18,38 , 58. $\\cdots, 978,998$.", "solution": "null", "level": "高一", "question": "为了了解参加某种知识竞赛的 1003 名学生的成绩, 抽取一个容量为 50 的样本, 选用什么抽样方法比较恰当?简述抽样过程.", "options": [], "subject": "计数", "analysis": "适宜选用系统抽样,抽样过程如下:\n\n(1) 随机地将这 1003 个个体编号为 1,2,3\\%, 1003.\n\n(2)利用简单随机抽样,先从总体中随机剔除 3 个个体,剩下的个体数 1000 能被样本容量 50 整除, 然后将 1000 个个体重新编号为 $1,2,3 \\cdots, 1000$.\n\n(3)将总体按编号顺序均分成 50 部分,每部分包含 20 个个体,\n\n(4) 在编号为 $1,2,3 \\cdots, 20$ 的第一部分个体中, 利用简单随机抽样抽取一个号码, 比如抽取的号码是 18.\n\n(5) 以 18 为起始号码, 这样得到一个容量为 50 的样本: 18,38 , 58. $\\cdots, 978,998$."} {"id": "16508", "image": [], "answer": "假设这个班的学生是这样编号(这个编号也代表他们的身高)的:\n第一组 $a_{1}\n\n(1)\n\n| $B$ | | | |\n| :--- | :--- | :--- | :--- |\n| 2 | 5 | | |\n| 3 | 1 | 3 | |\n| 4 | 0 | 4 | 5 |\n| 5 | 6 | 7 | 9 |\n| 6 | 4 | | |\n| 7 | 3 | | |\n| 8 | 6 | | |\n\n(2)\n\n从图中可以看出第一组数据的平均数要小于第二组数据的平均数.\n\n(2) 从图中观察到第一组数据比第二组数据更分散, 因此估计第二组数据的标准差更小.\n\n(3)计算得出第一组数据的平均数约是 $33.3 \\mathrm{~kg}$, 标准差约是 $27.9 \\mathrm{~kg}$; 第二组数据的平\n均数约是 $51.1 \\mathrm{~kg}$, 标准差约是 $17.3 \\mathrm{~kg}$. 故第二组数据的标准差更小, 与 (2) 的结果一致.\n\n(4)由两组样本数据的平均数可以判断 B 种肥料对橘子树的产量影响更大.", "solution": "null", "level": "高一", "question": "在一项农业试验中, A, B 两种肥料分别被施用于同类橘子树. 为了了解这两种肥料的效果, 试验人员分别从施用这两种肥料的橘子树中随机抽取了 12 棵, 下表给出了每一棵橘子树的产量(单位: 千克):\n\n肥料 A63, $54,19,20,72,92,8,10,22,11,24,5$; 肥料 B57, $86,33,40,59$, $56,73,25,44,31,64,45$.\n\n(1) 用茎叶图表示分别施用 $\\mathrm{A}, \\mathrm{B}$ 两种肥料的橘子树的产量, 并从图中比较各自平均数的大小。\n\n(2)从图中你认为哪一组数据的标准差更小?\n\n(3)分别计算施用 A, B 两种肥料的橘子树产量的平均数和标准差, 看看与(2)的结果是否一致?\n\n(4)你认为哪种肥料对橘子树的产量的影响更大? 依据是什么?", "options": [], "subject": "计数", "analysis": "(1)施用 A 种肥料的橘子树的产量用图(1)所示茎叶图表示, 施用 B 种肥料的橘子树的产量用图 (2) 所示茎叶图表示.\n\n\n\n(1)\n\n| $B$ | | | |\n| :--- | :--- | :--- | :--- |\n| 2 | 5 | | |\n| 3 | 1 | 3 | |\n| 4 | 0 | 4 | 5 |\n| 5 | 6 | 7 | 9 |\n| 6 | 4 | | |\n| 7 | 3 | | |\n| 8 | 6 | | |\n\n(2)\n\n从图中可以看出第一组数据的平均数要小于第二组数据的平均数.\n\n(2) 从图中观察到第一组数据比第二组数据更分散, 因此估计第二组数据的标准差更小.\n\n(3)计算得出第一组数据的平均数约是 $33.3 \\mathrm{~kg}$, 标准差约是 $27.9 \\mathrm{~kg}$; 第二组数据的平\n均数约是 $51.1 \\mathrm{~kg}$, 标准差约是 $17.3 \\mathrm{~kg}$. 故第二组数据的标准差更小, 与 (2) 的结果一致.\n\n(4)由两组样本数据的平均数可以判断 B 种肥料对橘子树的产量影响更大."} {"id": "16537", "image": [], "answer": "(1) 用系统抽样法从 36 名工人中抽取容量为 9 的样本, 根据题意, 所抽取工人编号为 $2,6,10,14,18,22,26,30,34$, 则相应工人的年龄数据为 $44,40,36$, $43,36,37,44,43,37$.\n\n(2) 样本均值 $\\bar{x}=\\frac{1}{9} \\times(44+40+36+43+36+37+44+43+37)=40$样本方差 $s^{2}=\\frac{1}{9} \\times\\left[(44-40)^{2}+(40-40)^{2}+(36-40)^{2}+(43-\\right.$ $40)^{2}+(36-40)^{2}+(37-40)^{2}+(44-40)^{2}+(43-40)^{2}+$ $\\left.(37-40)^{2}\\right]=\\frac{1}{9} \\times\\left[4^{2}+0^{2}+(-4)^{2}+3^{2}+(-4)^{2}+(-3)^{2}+4^{2}+3^{2}+(-3)^{2}\\right]=\\frac{100}{9}$\n\n(3)由于 $\\bar{x}=40, s=\\sqrt{s^{2}}=\\frac{10}{3} \\approx 3.33,36$ 名工人中年龄在 $\\bar{x}=s \\approx 36.67$ 与 $\\bar{x}+s \\approx 43.33$ 之间有 23 人, 所占比例为 $\\frac{23}{36} \\approx 63.89 \\%$.", "solution": "null", "level": "高一", "question": "某工厂 36 名工人的年龄数据如下表.\n\n| 工人编号 | 年龄 | 工人编号 | 年龄 | 工人编号 | 年龄 | 工人编号 | 年龄 |\n| :---: | :---: | :---: | :---: | :---: | :---: | :---: | :---: |\n| 1 | 40 | 10 | 36 | 19 | 27 | 28 | 34 |\n| 2 | 44 | 11 | 31 | 20 | 43 | 29 | 39 |\n| 3 | 40 | 12 | 38 | 21 | 41 | 30 | 43 |\n| 4 | 41 | 13 | 39 | 22 | 37 | 31 | 38 |\n| 5 | 33 | 14 | 43 | 23 | 34 | 32 | 42 |\n| 6 | 40 | 15 | 45 | 24 | 42 | 33 | 53 |\n| 7 | 45 | 16 | 39 | 25 | 37 | 34 | 37 |\n| 8 | 42 | 17 | 38 | 26 | 44 | 35 | 49 |\n| 9 | 43 | 18 | 36 | 27 | 42 | 36 | 39 |\n\n(1) 用系统抽样法从 36 名工人中抽取容量为 9 的样本, 且在第一分段里用随机抽样法\n抽到的年龄数据为 44 , 列出样本的年龄数据;\n\n(2)计算(1)中样本的均值和方差 $S^{2}$;\n\n(3) 36 名工人中年龄在 $-\\mathrm{s}$ 与 $+\\mathrm{s}$ 之间有多少人? 所占的百分比是多少 (精确到 $0.01 \\%$ ?", "options": [], "subject": "计数", "analysis": "(1) 用系统抽样法从 36 名工人中抽取容量为 9 的样本, 根据题意, 所抽取工人编号为 $2,6,10,14,18,22,26,30,34$, 则相应工人的年龄数据为 $44,40,36$, $43,36,37,44,43,37$.\n\n(2) 样本均值 $\\bar{x}=\\frac{1}{9} \\times(44+40+36+43+36+37+44+43+37)=40$样本方差 $s^{2}=\\frac{1}{9} \\times\\left[(44-40)^{2}+(40-40)^{2}+(36-40)^{2}+(43-\\right.$ $40)^{2}+(36-40)^{2}+(37-40)^{2}+(44-40)^{2}+(43-40)^{2}+$ $\\left.(37-40)^{2}\\right]=\\frac{1}{9} \\times\\left[4^{2}+0^{2}+(-4)^{2}+3^{2}+(-4)^{2}+(-3)^{2}+4^{2}+3^{2}+(-3)^{2}\\right]=\\frac{100}{9}$\n\n(3)由于 $\\bar{x}=40, s=\\sqrt{s^{2}}=\\frac{10}{3} \\approx 3.33,36$ 名工人中年龄在 $\\bar{x}=s \\approx 36.67$ 与 $\\bar{x}+s \\approx 43.33$ 之间有 23 人, 所占比例为 $\\frac{23}{36} \\approx 63.89 \\%$."} {"id": "16538", "image": ["8570.jpg"], "answer": ")由用水量的频率分布直方图知,该市居民该月用水量在区间 $[0.5,1]$, $(1,1.5],(1.5,2],(2,2.5],(2.5,3]$ 内的频率依次为 $0.1,0.15,0.2,0.25$, 0. 15. 所以该月用水量不超过 3 立方米的居民占 $85 \\%$, 用水量不超过 2 立方米的居民占 $45 \\%$.依题意, w 至少定为 3 .\n\n(2) 由用水量的频率分布直方图及题意, 得居民该月用水费用的数据分组与频率分布表:\n\n| 组号 | 1 | 2 | 3 | 4 | 5 | 6 | 7 | 8 |\n| :---: | :---: | :---: | :---: | :---: | :---: | :---: | :---: | :---: |\n| 分组 | $[2,4]$ | $(4,6]$ | $(6,8]$ | $(8,10]$ | $(10,12]$ | $(12,17]$ | $(17,22]$ | $(22,27]$ |\n| 频率 | 0.1 | 0.15 | 0.2 | 0.25 | 0.15 | 0.05 | 0.05 | 0.05 |\n\n根据题意,该市居民该月的人均水费估计为: $4 \\times 0.1+6 \\times 0.15+8 \\times 0.2+10 \\times 0.25+12 \\times 0.15+17 \\times 0.05+22 \\times 0.05+27 \\times 0.05=10.5$ (元)", "solution": "null", "level": "高一", "question": "某市居民用水拟实行阶梯水价。每人月用水量中不超过 $\\mathrm{w}$ 立方米的部分按 4 元/立方米收费,超出 $\\mathrm{w}$ 立方米的部分按 10 元/立方米收费. 从该市随机调查了 10000 位居民,获得了他们某月的用水量数据, 整理得到如图频率分布直方图:\n\n\n\n(1)如果 $\\mathrm{w}$ 为整数,那么根据此次调查,为使 $80 \\%$ 以上居民在该月的用水价格为 4 元/立方米, $\\mathrm{w}$ 至少定为多少?\n\n(2)假设同组中的每个数据用该组区间的右端点值代替. 当 $\\mathrm{w}=3$ 时,估计该市居民该月的人均水费。", "options": [], "subject": "计数", "analysis": ")由用水量的频率分布直方图知,该市居民该月用水量在区间 $[0.5,1]$, $(1,1.5],(1.5,2],(2,2.5],(2.5,3]$ 内的频率依次为 $0.1,0.15,0.2,0.25$, 0. 15. 所以该月用水量不超过 3 立方米的居民占 $85 \\%$, 用水量不超过 2 立方米的居民占 $45 \\%$.依题意, w 至少定为 3 .\n\n(2) 由用水量的频率分布直方图及题意, 得居民该月用水费用的数据分组与频率分布表:\n\n| 组号 | 1 | 2 | 3 | 4 | 5 | 6 | 7 | 8 |\n| :---: | :---: | :---: | :---: | :---: | :---: | :---: | :---: | :---: |\n| 分组 | $[2,4]$ | $(4,6]$ | $(6,8]$ | $(8,10]$ | $(10,12]$ | $(12,17]$ | $(17,22]$ | $(22,27]$ |\n| 频率 | 0.1 | 0.15 | 0.2 | 0.25 | 0.15 | 0.05 | 0.05 | 0.05 |\n\n根据题意,该市居民该月的人均水费估计为: $4 \\times 0.1+6 \\times 0.15+8 \\times 0.2+10 \\times 0.25+12 \\times 0.15+17 \\times 0.05+22 \\times 0.05+27 \\times 0.05=10.5$ (元)"} {"id": "16915", "image": [], "answer": "解:算法如下:\n\n第一步: $i=0, n=0, S=0$\n\n第二步: 输入一个成绩 $a$\n\n第三步:若 $a>90$, 则 $S=S+a, n=n+1$\n\n否则,执行第四步\n\n第四步: $i=i+1$\n\n第五步: 若 $i>54$, 则 $s=s / n$, 输出 $S, n$. 否则, 执行第二步.", "solution": "null", "level": "高一", "question": "高一(2)班共有 54 名学生参加数学竞赛, 现已有他们的竞赛分数, 请设计一个将竞赛成绩优秀学生的平均分输出的算法(规定 90 分以上为优秀).", "options": [], "subject": "计数", "analysis": "解:算法如下:\n\n第一步: $i=0, n=0, S=0$\n\n第二步: 输入一个成绩 $a$\n\n第三步:若 $a>90$, 则 $S=S+a, n=n+1$\n\n否则,执行第四步\n\n第四步: $i=i+1$\n\n第五步: 若 $i>54$, 则 $s=s / n$, 输出 $S, n$. 否则, 执行第二步."} {"id": "16957", "image": [], "answer": "解:\n\nINPUTa1, a2, $\\cdots, a 12$\n\nPRINT “第 1 季度 aver $=” ; \\quad(a 1+a 2+a 3) / 3$\n\nPRINT “第 2 季度 $a v e r=” ; \\quad(a 4+a 5+a 6 ) / 3$\n\nPRINT “第 3 季度 $a v e r=” ; \\quad(a 7+a 8+a 9) / 3$\n\nPRINT “第 4 季度 aver $=”$; $\\quad(a 10+a 11+a 12) / 3$\n\nPRINT “全年 aver $=” ; \\quad(a 1+a 2+\\cdots+a 12) / 3$\n\nEND", "solution": "null", "level": "高一", "question": "某市 2006 年 $1 \\sim 12$ 月的产值(单位: 亿元)分别为 $3.8,4.2,5.3,6.1,5.6,4.8,7.3,4.5,6.4$, 5.8, 4.7, 6.5, 该市要统计每季度的月平均产值及 2006 年的月平均产值, 试分别用输入、输出语句和赋值语句表示计算上述各个平均值的一个算法.", "options": [], "subject": "计数", "analysis": "解:\n\nINPUTa1, a2, $\\cdots, a 12$\n\nPRINT “第 1 季度 aver $=” ; \\quad(a 1+a 2+a 3) / 3$\n\nPRINT “第 2 季度 $a v e r=” ; \\quad(a 4+a 5+a 6 ) / 3$\n\nPRINT “第 3 季度 $a v e r=” ; \\quad(a 7+a 8+a 9) / 3$\n\nPRINT “第 4 季度 aver $=”$; $\\quad(a 10+a 11+a 12) / 3$\n\nPRINT “全年 aver $=” ; \\quad(a 1+a 2+\\cdots+a 12) / 3$\n\nEND"} {"id": "16979", "image": [], "answer": "解:(1)甲班样本的平均值为:\n\n$$\n\\bar{x}_{\\text {甲 }}=\\frac{1}{6}(9+11+13+20+24+31)=18 \\text {. }\n$$\n\n乙班样本的平均成绩为:\n\n$\\bar{x}_{Z}=\\frac{1}{6}(11+12+18+20+22+25)=18$.\n\n(2)甲班符合 “过度謷夜” 的样本数据有 2 个, 乙班符合 “过度謷夜” 的样本数据有 2 个,\n从甲、乙两班符合 “过度鳌夜” 的样本数据中, 抽取 2 个数据,\n\n基本事件总数 $n=C_{4}^{2}=6$,\n\n抽到的数据来自于同一个班级包含的基本事件个数 $m=C_{2}^{2}+C_{2}^{2}=2$,\n\n$\\therefore$ 抽到的数据来自于同一个班级的概率 $p=\\frac{m}{n}=\\frac{2}{6}=\\frac{1}{3}$.\n\n(3)甲班的 6 个样本数据中, 为 “过度鳌夜” 的数据有 2 个,从甲班的样本数据中有放回地抽取 2 个数据,\n\n基本事件总数 $n=6 \\times 6=36$,\n\n恰有 1 个数据为 “过度謷夜” 包含的基本事件总数 $m=C_{4}^{1} C_{2}^{1}+C_{2}^{1} C_{4}^{1}=16$,\n\n$\\therefore$ 恰有 1 个数据为 “过度謷夜” 的概率 $P=\\frac{m}{n}=\\frac{16}{36}=\\frac{4}{9}$.", "solution": "null", "level": "高一", "question": "(2020 秋$\\cdot$沈阳期末)中学阶段是学生身体发育重要的阶段, 长时间謷夜学习严重影响学生的身体健康. 某校为了解甲、乙两个班的学生每周謷夜学习的总时长 (单位: 小时), 从这两个班中各随机抽取 6 名同学进行调查, 将他们最近一周謷夜学习的总时长作为样本数据, 如表所示. 如果学生一周謷夜学习的总时长超过 21 小时, 则称为 “过度謷夜”.\n\n| 甲班 | 9 | 11 | 13 | 20 | 24 | 31 |\n| :--: | :--: | :--: | :--: | :--: | :--: | :--: |\n| 乙班 | 11 | 12 | 18 | 20 | 22 | 25 |\n\n(1)分别计算出甲、乙两班样本的平均值;\n\n(2) 为了解学生过度热夜的原因, 从甲、乙两班符合 “过度鳌夜” 的样本数据中, 抽取 2 个数据,求抽到的数据来自于同一个班级的概率;\n\n(3)从甲班的样本数据中有放回地抽取 2 个数据, 求恰有 1 个数据为 “过度鳌夜” 的概率.", "options": [], "subject": "计数", "analysis": "解:(1)甲班样本的平均值为:\n\n$$\n\\bar{x}_{\\text {甲 }}=\\frac{1}{6}(9+11+13+20+24+31)=18 \\text {. }\n$$\n\n乙班样本的平均成绩为:\n\n$\\bar{x}_{Z}=\\frac{1}{6}(11+12+18+20+22+25)=18$.\n\n(2)甲班符合 “过度謷夜” 的样本数据有 2 个, 乙班符合 “过度謷夜” 的样本数据有 2 个,\n从甲、乙两班符合 “过度鳌夜” 的样本数据中, 抽取 2 个数据,\n\n基本事件总数 $n=C_{4}^{2}=6$,\n\n抽到的数据来自于同一个班级包含的基本事件个数 $m=C_{2}^{2}+C_{2}^{2}=2$,\n\n$\\therefore$ 抽到的数据来自于同一个班级的概率 $p=\\frac{m}{n}=\\frac{2}{6}=\\frac{1}{3}$.\n\n(3)甲班的 6 个样本数据中, 为 “过度鳌夜” 的数据有 2 个,从甲班的样本数据中有放回地抽取 2 个数据,\n\n基本事件总数 $n=6 \\times 6=36$,\n\n恰有 1 个数据为 “过度謷夜” 包含的基本事件总数 $m=C_{4}^{1} C_{2}^{1}+C_{2}^{1} C_{4}^{1}=16$,\n\n$\\therefore$ 恰有 1 个数据为 “过度謷夜” 的概率 $P=\\frac{m}{n}=\\frac{16}{36}=\\frac{4}{9}$."} {"id": "16980", "image": [], "answer": "解: (1) 由频数分布表估计这 100 名学生对该节目打分的平均值为:\n\n$55 \\times \\frac{1}{100}+65 \\times \\frac{9}{100}+75 \\times \\frac{18}{100}+85 \\times \\frac{32}{100}+95 \\times \\frac{40}{100}=85.1$,\n\n$\\therefore$ 这 100 名学生对该节目打分的平均值为 85.1.\n\n(2)由题意知, 从打分为 $[60 , 70 )$ 的学生中抽取了 2 人, 分别记为 $a, b$,从打分在 $[70,80 )$ 的学生中抽取了 4 人, 分别记为 $A, B, C, D$,\n\n从这 6 人中随机抽取 2 名学生进行详细调查的基本事件有:\n\n$a b, a A, a B, a C, a D, b A, b B, b C, b D, A B, A C, A D, B C, B D, C D$, 共 15 个,记 “这 2 名学生的打分在同一分数段” 为事件 $M$,\n\n事件 $M$ 包含的基本事件有:\n\n$a b, A B, A C, A D, B C, B D, C D$ 共 7 个,\n\n$\\therefore$ 事件 $M$ 发生的概率 $P(M)=\\frac{7}{15}$.", "solution": "null", "level": "高一", "question": "(2021$\\cdot$二十模拟)2019 年 9 月 1 日央视《开学第一课》播出后, 社会各界反响强烈. 某兴趣小组为了了解某校学生对 《开学第一课》的满意程度, 从该校随机抽取了 100 名学生对该节目进行打分(满分 100 分,并把相关的统计结果记录如表:\n\n| 分数段 | $[50,60)$ | $[60,70)$ | $[70,80)$ | $[80,90)$ | $[90,100]$ |\n| :----: | :-------: | :-------: | :-------: | :-------: | :--------: |\n| 频数 | 1 | 9 | 18 | 32 | 40 |\n\n(1) 试估计这 100 名学生对该节目打分的平均值 (同一组中的数据用该组区间的中点值作代表);\n\n(2)将频率视为概率,该兴趣小组用分层抽样的方法从打分为 $[60,70)$ 和 $[70,80 )$ 的学生中抽取 6 人, 再从这 6 人中随机抽取 2 名学生进行详细调查, 记 “这 2 名学生的打分在同一分数段”为事件 $M$ ,求事件 $M$ 发生的概率.", "options": [], "subject": "计数", "analysis": "解: (1) 由频数分布表估计这 100 名学生对该节目打分的平均值为:\n\n$55 \\times \\frac{1}{100}+65 \\times \\frac{9}{100}+75 \\times \\frac{18}{100}+85 \\times \\frac{32}{100}+95 \\times \\frac{40}{100}=85.1$,\n\n$\\therefore$ 这 100 名学生对该节目打分的平均值为 85.1.\n\n(2)由题意知, 从打分为 $[60 , 70 )$ 的学生中抽取了 2 人, 分别记为 $a, b$,从打分在 $[70,80 )$ 的学生中抽取了 4 人, 分别记为 $A, B, C, D$,\n\n从这 6 人中随机抽取 2 名学生进行详细调查的基本事件有:\n\n$a b, a A, a B, a C, a D, b A, b B, b C, b D, A B, A C, A D, B C, B D, C D$, 共 15 个,记 “这 2 名学生的打分在同一分数段” 为事件 $M$,\n\n事件 $M$ 包含的基本事件有:\n\n$a b, A B, A C, A D, B C, B D, C D$ 共 7 个,\n\n$\\therefore$ 事件 $M$ 发生的概率 $P(M)=\\frac{7}{15}$."} {"id": "16981", "image": [], "answer": "解: (1) (i) 把该校环境保护兴趣协会的 20 名成员进行编号, 号码分别为: $01,02,03,04,05,06,07,08,09,10,11,12,13,14,15,16,17,18,19,20$, (ii) 20 个号码傥拌均匀, 从中依次取出 6 个号码, 从而取出参赛队伍成员.\n\n(2) 选出的人员中有 2 名女生 4 名男生, 在这 6 名学生中任选两人担任正副队长,基本事件总数 $n=C_{6}^{2}=15$,所选两人恰好有 1 名女生包含的基本事件个数 $m=C_{2}^{1} C_{4}^{1}=8$,\n\n$\\therefore$ 所选两人恰好有 1 名女生的概率 $P=\\frac{m}{n}=\\frac{8}{15}$.", "solution": "null", "level": "高一", "question": "(2020 秋・忻府区校级月考)某校要从该校环境保护兴趣协会的 20 名成员中, 选取 6 人组队参加市电视台组织的环保知识竞赛.\n\n(1)若采用抽签法选取参赛队伍成员,请写出步骤;\n\n(2) 若选出的人员中有 2 名女生 4 名男生, 在这 6 名学生中任选两人担任正副队长, 求所选两人恰好有 1 名女生的概率.", "options": [], "subject": "计数", "analysis": "解: (1) (i) 把该校环境保护兴趣协会的 20 名成员进行编号, 号码分别为: $01,02,03,04,05,06,07,08,09,10,11,12,13,14,15,16,17,18,19,20$, (ii) 20 个号码傥拌均匀, 从中依次取出 6 个号码, 从而取出参赛队伍成员.\n\n(2) 选出的人员中有 2 名女生 4 名男生, 在这 6 名学生中任选两人担任正副队长,基本事件总数 $n=C_{6}^{2}=15$,所选两人恰好有 1 名女生包含的基本事件个数 $m=C_{2}^{1} C_{4}^{1}=8$,\n\n$\\therefore$ 所选两人恰好有 1 名女生的概率 $P=\\frac{m}{n}=\\frac{8}{15}$."} {"id": "16984", "image": [], "answer": "解:(I )由题知被调查者一共 200 人,\n\n$\\therefore 35+60+n+20+10+10=200$, 解得 $n=65$.\n\n则被调查人员各组学习时间的频率分别为 $0.1,0.175,0.325,0.3,0.05,0.05$,\n\n$\\therefore$ 调查的 200 名学生学习时间的人数频率分布表如表:\n\n| 学习时间 | $[0,40)$ | $[40,80)$ | $[80,120)$ | $[120,160)$ | $[160,200)$ | $[200,240]$ |\n| :------: | :------: | :-------: | :--------: | :---------: | :---------: | :---------: |\n| 频数 | 20 | 35 | 65 | 60 | 10 | 10 |\n| 频率 | 0.1 | 0.175 | 0.325 | 0.3 | 0.05 | 0.05 |\n\n平均值为:\n\n$20 \\times 0.1+60 \\times 0.175+100 \\times 0.325+140 \\times 0.3+180 \\times 0.05+220 \\times 0.05=107$.\n\n(II) 由题意 $A$ 类学生抽取人数为 $5 \\times \\frac{120}{200}=3$ (人),\n\n将 $A$ 类学生分别记为 $x, y, z, B$ 类学生分别记为 $a, b$,\n\n设 “选中的 2 人都是 $A$ 类学生” 为事件 $A$,\n\n则从 5 名学生中任选 2 人, 所有可能情况有 10 种, 分别为:\n\n$(x, y),(x, z),(x, a),(x, b),(y, z),(y, a),(y, b),(z, a),(z, b)$, $(a, b)$,\n\n其中事件 $A$ 包含的基本事件有 3 种,分别为: $(x, y),(x, z),(y, z)$,\n\n$\\therefore$ 从这五人中抽取的两人都是 $A$ 类学生的概率为:\n\n$P(A)=\\frac{3}{10}=0.3$.", "solution": "null", "level": "高一", "question": "(2021・八模拟)2020 年寒假期间新冠肺炎肆虐,全国人民众志成城抗击疫情. 某市要求全体市民在家隔离, 同时决定全市所有学校推迟开学. 某区教育局为了让学生 “停课不停学” , 要求学校各科老师每天在网上授课, 每天共 240 分钟, 请学生自主学习. 区教育局为了了解学生网上学习情况, 上课几天后在全区学生中采取随机抽样的方法抽取了 200 名学生进行问卷调查, 为了方便表述把学习时间在 $[0,120)$ 分钟的学生称为 $A$ 类, 把学习时间在 $[120,240]$ 分钟的学生称为 $B$类, 调查的 200 名学生学习时间的人数频率分布表如表:\n\n| 学习时间 | $[0,40)$ | $[40,80)$ | $[80,120)$ | $[120,160)$ | $[160,200)$ | $[200,240]$ |\n| :------: | :------: | :-------: | :--------: | :---------: | :---------: | :---------: |\n| 频数 | 20 | 35 | $n$ | 60 | 10 | 10 |\n| 频率 | | | | | | |\n\n以频率估计概率回答下列问题:\n\n(I) 求表中 $n$ 的值并完成上表, 并估算这 200 名学生学习时间的平均值(同一组数据用该组区间的中点值代替,结果保留整数);\n\n(II) 在 $A, B$ 两类学生中, 按分层抽样的方法从上述 200 名学生中抽取 5 人, 求从这五人中抽取的两人都是 $A$ 类学生的概率.", "options": [], "subject": "计数", "analysis": "解:(I )由题知被调查者一共 200 人,\n\n$\\therefore 35+60+n+20+10+10=200$, 解得 $n=65$.\n\n则被调查人员各组学习时间的频率分别为 $0.1,0.175,0.325,0.3,0.05,0.05$,\n\n$\\therefore$ 调查的 200 名学生学习时间的人数频率分布表如表:\n\n| 学习时间 | $[0,40)$ | $[40,80)$ | $[80,120)$ | $[120,160)$ | $[160,200)$ | $[200,240]$ |\n| :------: | :------: | :-------: | :--------: | :---------: | :---------: | :---------: |\n| 频数 | 20 | 35 | 65 | 60 | 10 | 10 |\n| 频率 | 0.1 | 0.175 | 0.325 | 0.3 | 0.05 | 0.05 |\n\n平均值为:\n\n$20 \\times 0.1+60 \\times 0.175+100 \\times 0.325+140 \\times 0.3+180 \\times 0.05+220 \\times 0.05=107$.\n\n(II) 由题意 $A$ 类学生抽取人数为 $5 \\times \\frac{120}{200}=3$ (人),\n\n将 $A$ 类学生分别记为 $x, y, z, B$ 类学生分别记为 $a, b$,\n\n设 “选中的 2 人都是 $A$ 类学生” 为事件 $A$,\n\n则从 5 名学生中任选 2 人, 所有可能情况有 10 种, 分别为:\n\n$(x, y),(x, z),(x, a),(x, b),(y, z),(y, a),(y, b),(z, a),(z, b)$, $(a, b)$,\n\n其中事件 $A$ 包含的基本事件有 3 种,分别为: $(x, y),(x, z),(y, z)$,\n\n$\\therefore$ 从这五人中抽取的两人都是 $A$ 类学生的概率为:\n\n$P(A)=\\frac{3}{10}=0.3$."} {"id": "16985", "image": [], "answer": "解:(1)中位数在 $[35,40 )$ 中, 设为 $x$,\n\n则 $0.01 \\times 5+0.04 \\times 5+0.07 \\times(x-35)=0.5$, 解得 $x \\approx 38.6$.\n平均数为 $(27.5 \\times 0.01+32.5 \\times 0.04+37.5 \\times 0.07+42.5 \\times 0.06+47.5 \\times 0.02) \\times 5=38.5$.\n\n所以样本的中位数为 38.6 , 平均数为 38.5 .\n\n(2)根据分层抽样法, 其中位于 $[30 , 35 )$ 中的有 4 人,记为 $A, B , C , D$,位于 $[45,50]$ 中的有 2 人, 记为 $a, b$.\n\n从 6 人中抽取 2 人, 有 15 种情况, 分别为:\n$(A, B)$\n$(A, C)$\n$(A, D)$\n$(A, a),(A, b)$,\n$(B, C)$,\n$(B, D)$,\n$(B, a)$,\n$(B, b)$,\n$(C, D)$,\n$(C, a)$,\n$(c, b)$,\n$(D, a)$,\n$(D, b),(a, b)$,\n\n恰有 1 人在 $[45,50]$ 内的有:\n$(A, a)$,\n$(A, b)$,\n$(B, a)$,\n$(B, b)$,\n$(C, a)$,\n$(C, b)$,\n(D, a) ,\n(D,b),共\n\n8 种情况,\n\n所以恰有 1 人在 $[45,50]$ 内的概率为 $P=\\frac{8}{15}$.", "solution": "null", "level": "高一", "question": "(2020 秋 - 海东市月考)某电脑公司为调查旗下 $A$ 品牌电脑的使用情况, 随机抽取 200 名用户,根据不同年龄段(单位: 岁)统计,如表:\n\n| 分组 | 频率/组距 |\n| :-------: | :-------: |\n| $[25,30)$ | 0.01 |\n| $[30,35)$ | 0.04 |\n| $[35,40)$ | 0.07 |\n| $[40,45)$ | 0.06 |\n| $[45,50]$ | 0.02 |\n\n(1)根据如表, 试估计样本的中位数, 平均数(同一组数据以该组区间的中点值为代表, 结果精确到 $0.1 )$;\n\n(2)按照年龄段从 $[30,35 ),[45,50]$ 内的用户中进行分层抽样, 抽取 6 人, 在从中随机选取 2 人赠送小礼品, 求恰有 1 人在 $[45,50]$ 内的概率.", "options": [], "subject": "计数", "analysis": "解:(1)中位数在 $[35,40 )$ 中, 设为 $x$,\n\n则 $0.01 \\times 5+0.04 \\times 5+0.07 \\times(x-35)=0.5$, 解得 $x \\approx 38.6$.\n平均数为 $(27.5 \\times 0.01+32.5 \\times 0.04+37.5 \\times 0.07+42.5 \\times 0.06+47.5 \\times 0.02) \\times 5=38.5$.\n\n所以样本的中位数为 38.6 , 平均数为 38.5 .\n\n(2)根据分层抽样法, 其中位于 $[30 , 35 )$ 中的有 4 人,记为 $A, B , C , D$,位于 $[45,50]$ 中的有 2 人, 记为 $a, b$.\n\n从 6 人中抽取 2 人, 有 15 种情况, 分别为:\n$(A, B)$\n$(A, C)$\n$(A, D)$\n$(A, a),(A, b)$,\n$(B, C)$,\n$(B, D)$,\n$(B, a)$,\n$(B, b)$,\n$(C, D)$,\n$(C, a)$,\n$(c, b)$,\n$(D, a)$,\n$(D, b),(a, b)$,\n\n恰有 1 人在 $[45,50]$ 内的有:\n$(A, a)$,\n$(A, b)$,\n$(B, a)$,\n$(B, b)$,\n$(C, a)$,\n$(C, b)$,\n(D, a) ,\n(D,b),共\n\n8 种情况,\n\n所以恰有 1 人在 $[45,50]$ 内的概率为 $P=\\frac{8}{15}$."} {"id": "17002", "image": ["8764.jpg"], "answer": "解:(1)两次记录的数为 $(1,1),(1,2),(1,3),(1,4),(2,2),(2$,\n3), ( 2,4$)$,\n$(3,4)$\n$(2,1)$\n$(3,1),(4,1),(3,2)$,\n$(3,3)$\n$(4,2)$,\n\n$(4,3),(4,4)$, 共 16 个,\n\n满足 $x y \\leqslant 3$, 有 $(1,1),(1,2),(1,3),(2,1),(3,1)$, 共 5 个,\n\n$\\therefore$ 小亮获得玩具的概率为 $\\frac{5}{16}$;\n\n(2) 满足 $x y \\geqslant 8,(2,4),(3,4),(4,2),(4,3),(3,3),(4,4)$ 共 6 个, $\\therefore$小亮获得水杯的概率为 $\\frac{6}{16}$;\n\n小亮获得饮料的概率为 $1-\\frac{5}{16}-\\frac{6}{16}=\\frac{5}{16}$,\n\n$\\therefore$ 小亮获得水杯大于获得饮料的概率.", "solution": "null", "level": "高一", "question": "(2019 秋-东湖区校级月考)某儿童乐园在 “六一” 儿童节推出了一项趣味活动, 参加活动的儿童需转动如图所示的转盘两次, 每次转动后, 待转盘停止转动时, 记录指针所指区域中的数. 设两次记录的数分别为 $x 、 y$, 奖励规则如下:\n\n(1)若 $x y \\leqslant 3$, 则奖励玩具一个; (2)若 $x y \\geqslant 8$, 则奖励水杯一个;\n\n(3)其余情况奖励饮料一瓶. 假设转盘质地均匀, 四个区域划分均匀. 小亮准备参加此项活动.\n\n(1) 求小亮获得玩具的概率;\n\n(2)请比较小亮获得水杯与获得饮料的概率的大小, 并说明理由.\n\n", "options": [], "subject": "计数", "analysis": "解:(1)两次记录的数为 $(1,1),(1,2),(1,3),(1,4),(2,2),(2$,\n3), ( 2,4$)$,\n$(3,4)$\n$(2,1)$\n$(3,1),(4,1),(3,2)$,\n$(3,3)$\n$(4,2)$,\n\n$(4,3),(4,4)$, 共 16 个,\n\n满足 $x y \\leqslant 3$, 有 $(1,1),(1,2),(1,3),(2,1),(3,1)$, 共 5 个,\n\n$\\therefore$ 小亮获得玩具的概率为 $\\frac{5}{16}$;\n\n(2) 满足 $x y \\geqslant 8,(2,4),(3,4),(4,2),(4,3),(3,3),(4,4)$ 共 6 个, $\\therefore$小亮获得水杯的概率为 $\\frac{6}{16}$;\n\n小亮获得饮料的概率为 $1-\\frac{5}{16}-\\frac{6}{16}=\\frac{5}{16}$,\n\n$\\therefore$ 小亮获得水杯大于获得饮料的概率."} {"id": "17003", "image": [], "answer": "解: (1) 因为 $x^{2}-6 x-16 \\leqslant 0 \\Rightarrow-2 \\leqslant x \\leqslant 8$;\n\n$\\because 1 \\leqslant x \\leqslant 10$\n\n$\\therefore 1 \\leqslant x \\leqslant 8 ;$\n\n$\\therefore \\frac{8-1}{10-1}=\\frac{7}{9}$;\n\n(2) $\\because \\ln (x-2)<2 \\Rightarrow 227$, 所以 $x>\\log _{9} 27$, 即 $x>\\frac{3}{2}$,\n\n故由几何概型可知, 所求概率为 $\\frac{5-\\frac{3}{2}}{5-0}=\\frac{7}{10}$.\n\n(2) 因为 $\\log _{\\frac{1}{2}} x>-2$, 所以 $0-2$ 的整数为 $1,2,3$, 共 3 个,\n\n故由古典概型可知, 所求概率为 $\\frac{3}{7}$.", "solution": "null", "level": "高一", "question": "(2017 秋・上期末)(1)从区间(0,5)内任意选取一个实数 $x$, 求事件 “ $9^{x}>27$ ” 发生的概率;\n\n(2)从区间(0,8)内任意选取一个整数 $x$, 求事件 “ $\\log _{\\frac{1}{2}} x>-2$ ” 发生的概率.", "options": [], "subject": "计数", "analysis": "解: (1) 因为 $9^{x}>27$, 所以 $x>\\log _{9} 27$, 即 $x>\\frac{3}{2}$,\n\n故由几何概型可知, 所求概率为 $\\frac{5-\\frac{3}{2}}{5-0}=\\frac{7}{10}$.\n\n(2) 因为 $\\log _{\\frac{1}{2}} x>-2$, 所以 $0-2$ 的整数为 $1,2,3$, 共 3 个,\n\n故由古典概型可知, 所求概率为 $\\frac{3}{7}$."} {"id": "17005", "image": [], "answer": "解: 由题意, 含有犯罪内容的谈话被部分或全部擦掉就是在 40 秒以前按错了键, 在 40 秒后按错了键也不会被擦掉,\n\n所以概率为 $P=\\frac{30+10}{30 \\times 60}=\\frac{1}{45}$;\n\n故所求概率为: $\\frac{1}{45}$.", "solution": "null", "level": "高一", "question": "在研究历史声音档案时, 发现 $30 \\mathrm{~min}$ 长的磁带上, 从开始 $30 \\mathrm{~s}$ 处起, 有 $10 \\mathrm{~s}$ 长的一段内容包含两位名人的信息. 后来发现, 这段谈话的一部分被某位工作人员擦掉了, 该工作人员声称她完全是无意中按错了键,使从此处起往后的所有内容都被擦掉了. 问:由于按错了键而使含有两位名人谈话内容被部分或全部擦掉的概率有多大?", "options": [], "subject": "计数", "analysis": "解: 由题意, 含有犯罪内容的谈话被部分或全部擦掉就是在 40 秒以前按错了键, 在 40 秒后按错了键也不会被擦掉,\n\n所以概率为 $P=\\frac{30+10}{30 \\times 60}=\\frac{1}{45}$;\n\n故所求概率为: $\\frac{1}{45}$."} {"id": "15094", "image": [], "answer": "(1) 解: $\\because \\mathrm{a}>0, \\mathrm{~b}>0$,\n\n$\\therefore \\sqrt{a^{6} b^{5}}=a^{3} b^{2} \\sqrt{b}=a^{3} b^{\\frac{5}{2}}$\n\n(2) 解: $\\because \\mathrm{m}>0, \\therefore \\sqrt[3]{m^{2}}=m^{\\frac{2}{3}}$\n\n(3) 解: $\\because \\mathrm{m}>\\mathrm{n}>0, \\therefore \\sqrt{(m-n)^{3}}=(m-n)^{\\frac{3}{2}}$\n\n(4) 解: $\\because \\mathrm{a}>0, \\therefore \\sqrt{a} \\cdot \\sqrt[3]{a}=a^{\\frac{1}{2}} \\cdot a^{\\frac{1}{3}}=a^{\\frac{3}{6}}$\n\n(5)解: $\\because \\mathrm{a}>0, \\therefore \\sqrt{a \\sqrt{a \\sqrt{a}}}=\\sqrt{a \\cdot \\sqrt{a \\cdot \\sqrt{a^{\\frac{1}{2}}}}}=\\sqrt{a \\cdot a^{\\frac{3}{4}}}=a^{\\frac{1}{8}}$", "solution": "null", "level": "高一", "question": "用分数指数幂表示下列各式(式中字母均为正数);\n\n(1) $\\sqrt{a^{6} b^{5}}$;\n\n(2) $\\sqrt[3]{m^{2}}$;\n\n(3) $\\sqrt{(m-n)^{3}} \\quad(\\mathrm{~m}>\\mathrm{n})$;\n\n(4) $\\sqrt{a} \\cdot \\sqrt[3]{a}$\n\n(5) $\\sqrt{a \\sqrt{a \\sqrt{a}}}$.", "options": [], "subject": "算术", "analysis": "结合公式 $a^{\\frac{m}{n}}=\\sqrt[n]{a^{m}}$, 利用分数指数幂的性质和运算法则求解."} {"id": "15095", "image": [], "answer": "(1) 解: $\\left(2 \\frac{3}{5}\\right)^{0}+2^{-2} \\times\\left(2 \\frac{1}{4}\\right)^{-\\frac{1}{2}}-(0.01)^{0.5}$\n\n$=1+\\frac{1}{4} \\times\\left[\\left(\\frac{3}{2}\\right)^{2}\\right]^{-\\frac{1}{2}}-\\left(\\frac{1}{100}\\right)^{\\frac{1}{2}}=1+\\frac{1}{4} \\times \\frac{2}{3}-\\frac{1}{10}=\\frac{16}{15}$.\n\n(2) 解:\n\n$$\n\\sqrt[3]{a^{5}} \\cdot \\sqrt[3]{a^{7}} \\div a^{6}=a^{\\frac{5}{3}} \\cdot a^{\\frac{7}{3}} \\div a^{6}=a^{\\frac{5}{3}+\\frac{7}{3}-6}=a^{-2}=\\frac{1}{a^{2}} .\n$$", "solution": "null", "level": "高一", "question": "化简求值:\n\n(1) $\\left(2 \\frac{3}{5}\\right)^{0}+2^{-2} \\times\\left(2 \\frac{1}{4}\\right)^{-\\frac{1}{2}}-(0.01)^{0.5}$;\n\n(2) $\\sqrt[3]{a^{5}} \\cdot \\sqrt[3]{a^{7}} \\div a^{6}$.", "options": [], "subject": "算术", "analysis": "(1)根据指数幂的运算法则求解即可; (2) 把根式化成分数指数幂后根据指数幂的运算法则求解即可."} {"id": "15096", "image": [], "answer": "(1) 解: $\\left(\\frac{1}{2}\\right)^{-4}+\\left(\\frac{64}{27}\\right)^{\\frac{1}{3}}+\\left(\\frac{1}{8}\\right)^{-\\frac{2}{3}}+\\left(\\frac{81}{16}\\right)^{-\\frac{1}{4}}=16+\\frac{4}{3}+4+\\frac{2}{3}=22$\n\n(2) 解: $\\frac{\\sqrt[3]{a b^{2}} \\cdot a^{3} b^{2}}{\\sqrt[3]{b}\\left(a^{\\frac{1}{6}} b^{\\frac{1}{2}}\\right)^{4}}=\\frac{\\left(\\left(a b^{2}\\right)^{\\frac{1}{3}} a^{3} b^{2}\\right)^{\\frac{1}{2}}}{b^{\\frac{1}{3}}\\left(a^{\\frac{1}{6}} b^{\\frac{1}{2}}\\right)^{4}}=\\frac{a^{\\frac{5}{3}} b^{\\frac{4}{3}}}{a^{\\frac{2}{3}} b^{\\frac{7}{3}}}=\\frac{a}{b}$", "solution": "null", "level": "高一", "question": "(1) 计算: $\\left(\\frac{1}{2}\\right)^{-4}+\\left(\\frac{64}{27}\\right)^{\\frac{1}{3}}+\\left(\\frac{1}{8}\\right)^{-\\frac{2}{3}}+\\left(\\frac{81}{16}\\right)^{-\\frac{1}{4}}$;\n\n(2) 化简: $\\frac{\\sqrt[3]{\\sqrt[3]{a b^{2}} \\cdot a^{3} b^{2}}}{\\sqrt[3]{b}\\left(a^{\\frac{1}{6}} b^{\\frac{1}{2}}\\right)^{4}}(a>0, b>0)$.", "options": [], "subject": "算术", "analysis": "(1)利用有理数指数幂的运算性质化简求值.\n\n(2)利用根式与指数幂的关系式结合有理数指数幂的运算性质化简得出结果."} {"id": "15098", "image": [], "answer": "(1)解: 原式 $=\\left(0 \\cdot 4^{3}\\right)^{-\\frac{1}{3}}-1+\\left(2^{4}\\right)^{\\frac{3}{4}}+\\left(0 \\cdot 5^{2}\\right)^{\\frac{1}{2}}$\n\n$=0 \\cdot 4^{-1}-1+8+\\frac{1}{2}$\n\n$=\\frac{5}{2}+7+\\frac{1}{2}$\n\n$=10$\n\n(2) 解: $\\because x^{\\frac{1}{2}}+x^{-\\frac{1}{2}}=\\sqrt{5}$\n\n$\\therefore\\left(x^{\\frac{1}{2}}+x^{-\\frac{1}{2}}\\right)^{2}=5$\n\n$\\therefore x+x^{-1}=3$", "solution": "null", "level": "高一", "question": "计算求值:\n\n(1) $0.064^{-\\frac{1}{3}}-\\left(-\\frac{7}{8}\\right)^{0}+16^{\\frac{3}{4}}+0.25^{\\frac{1}{2}}$\n\n(2) 若 $x^{\\frac{1}{2}}+x^{-\\frac{1}{2}}=\\sqrt{5}$, 求 $x+x^{-1}$ 的值", "options": [], "subject": "算术", "analysis": "根据指数式的运算化简即可."} {"id": "15138", "image": [], "answer": "(1) 解: $\\log _{3} 1-\\log _{3} \\frac{1}{27}+\\log _{\\sqrt{3}} 3=0+3+2=5$\n\n(2)解: $0.064^{-\\frac{1}{3}}-\\left(-\\frac{1}{2}\\right)^{0}+4^{\\frac{3}{2}}+0.25^{\\frac{1}{2}}=\\sqrt[3]{\\frac{1000}{64}}-1+(\\sqrt{4})^{3}+\\sqrt{\\frac{1}{4}}=\\frac{10}{4}-1+8+\\frac{1}{2}=10$", "solution": "null", "level": "高一", "question": "计算下列式子的值:\n\n(1) $\\log _{3} 1-\\log _{3} \\frac{1}{27}+\\log _{\\sqrt{3}} 3$;\n\n(2) $0.064^{-\\frac{1}{3}}-\\left(-\\frac{1}{2}\\right)^{0}+4^{\\frac{3}{2}}+0.25^{\\frac{1}{2}}$.", "options": [], "subject": "算术", "analysis": "(1)利用对数的运算性质进行计算即可; (2)利用幂指数性质来进行计算即可."} {"id": "15139", "image": [], "answer": "(1) 解: 原式 $=\\lg 100+2+\\frac{\\lg 3}{\\lg 2} \\cdot \\frac{\\lg 4}{\\lg 3}=2+2+2=6$.\n\n(2) 解: $\\left(m^{\\frac{1}{2}}+m \\frac{1}{2}\\right)^{2}=3^{2} \\therefore m+m^{-1}=7$,\n\n$\\left(m+m^{-1}\\right)^{2}=7^{2} \\therefore m^{2}+m^{-2}=47$,\n\n$(m-m-1)^{2}=m^{2}+m^{2}-2=45$,\n\n$\\because m>1 \\therefore m-m-1=3 \\sqrt{5}$\n\n$\\therefore m^{2}-m^{-2}=\\left(m+m^{-1}\\right)\\left(m-m^{-1}\\right)=21 \\sqrt{5}$", "solution": "null", "level": "高一", "question": "(1) 计算: $\\lg 25+\\lg 4+7^{\\log _{3} 2}+\\log _{2} 3 \\cdot \\log _{3} 4$;\n\n(2)已知 $m^{\\frac{1}{2}}+m^{-\\frac{1}{2}}=3(m>1)$, 求 $m^{2}-m^{-2}$ 的值.", "options": [], "subject": "算术", "analysis": "(1)结合对数的运算性质化简即可; (2)结合两次平方关系即可求得 $m^{2}-m^{2}$."} {"id": "16913", "image": [], "answer": "解: 输出 $1 \\sim 100$ 之间的所有偶数,\n\n使用 DoLoop 循环语句, 如下;\n\n$i=1$\n\nDO\n\n$$\n\\begin{aligned}\n& m=i \\quad M O D 2 \\\\\n& \\text { IF } \\quad m=0 \\quad \\text { THEN } \\\\\n& \\text { PRINT } i\n\\end{aligned}\n$$\n\nEND IF\n\n$i=i+1$\n\nLOOP UNTIL $i>100$\n\nEND.", "solution": "null", "level": "高一", "question": "若要输出 $1 \\sim 100$ 之间的所有偶数, 应使用 For 循环还是 DoLoop 循环? 请写出具体过程.", "options": [], "subject": "算术", "analysis": "解: 输出 $1 \\sim 100$ 之间的所有偶数,\n\n使用 DoLoop 循环语句, 如下;\n\n$i=1$\n\nDO\n\n$$\n\\begin{aligned}\n& m=i \\quad M O D 2 \\\\\n& \\text { IF } \\quad m=0 \\quad \\text { THEN } \\\\\n& \\text { PRINT } i\n\\end{aligned}\n$$\n\nEND IF\n\n$i=i+1$\n\nLOOP UNTIL $i>100$\n\nEND."} {"id": "16914", "image": ["8733.jpg"], "answer": "解: 根据如图给出的程序框图, 写出对应的程序如下:\n\n$$\n\\begin{aligned}\n& s=0 \\\\\n& n=2 \\\\\n& i=1 \\\\\n& D O \\\\\n& \\quad s=s+\\frac{1}{n} \\\\\n& n=n+2 \\\\\n& \\quad i=i+1\n\\end{aligned}\n$$\n\nLOOP UNTIL\n$i>10$\n\nPRINTS\n\nEND.", "solution": "null", "level": "高一", "question": "如图给出的是计算 $\\frac{1}{2}+\\frac{1}{4}+\\frac{1}{6} \\cdots+\\frac{1}{20}$ 的值的一个程序框图, 写出对应的程序.\n\n", "options": [], "subject": "算术", "analysis": "解: 根据如图给出的程序框图, 写出对应的程序如下:\n\n$$\n\\begin{aligned}\n& s=0 \\\\\n& n=2 \\\\\n& i=1 \\\\\n& D O \\\\\n& \\quad s=s+\\frac{1}{n} \\\\\n& n=n+2 \\\\\n& \\quad i=i+1\n\\end{aligned}\n$$\n\nLOOP UNTIL\n$i>10$\n\nPRINTS\n\nEND."} {"id": "16935", "image": [], "answer": "解: (1) $f(x)=((((5 x+4) x+3) x+2) x+1) x-3$,\n\n$\\therefore v_{0}=5$;\n\n$v_{1}=5 \\times 2+4=14 ;$\n\n$v_{2}=14 \\times 2+3=31$;\n\n$v_{3}=31 \\times 2+2=64$,\n\n$v_{4}=64 \\times 2+1=129 ;$\n\n$v_{5}=129 \\times 2-3=255$,\n\n所以, 当 $x=2$ 时, 多项式的值为 255 .\n\n(2) $135=81 \\times 1+54$,\n\n$81=54 \\times 1+27$,\n\n$54=27 \\times 2+0$,\n则 81 与 135 的最大公约数为 27 .", "solution": "null", "level": "高一", "question": "(2017 秋・孝感期末)(1)用秦九韶算法求多项式 $f(x)=5 x^{5}+4 x^{4}+3 x^{3}+2 x^{2}+x-3$ 当 $x=2$ 时的值;\n\n(2) 用辗转相除法或更相减损术求 81 和 135 的最大公约数.", "options": [], "subject": "算术", "analysis": "解: (1) $f(x)=((((5 x+4) x+3) x+2) x+1) x-3$,\n\n$\\therefore v_{0}=5$;\n\n$v_{1}=5 \\times 2+4=14 ;$\n\n$v_{2}=14 \\times 2+3=31$;\n\n$v_{3}=31 \\times 2+2=64$,\n\n$v_{4}=64 \\times 2+1=129 ;$\n\n$v_{5}=129 \\times 2-3=255$,\n\n所以, 当 $x=2$ 时, 多项式的值为 255 .\n\n(2) $135=81 \\times 1+54$,\n\n$81=54 \\times 1+27$,\n\n$54=27 \\times 2+0$,\n则 81 与 135 的最大公约数为 27 ."} {"id": "16936", "image": [], "answer": "(本小题满分 10 分)\n\n解: $412{ }_{(5)}=2 \\times 5^{0}+1 \\times 5^{1}+4 \\times 5^{2}=2+5+4 \\times 25=107, \\cdots \\cdots \\cdots \\cdots$ ( 5 分),\n\n$\\because 107=2 \\times 7^{0}+1 \\times 7^{1}+2 \\times 7^{2}$\n\n$\\therefore$ 把 5 进制的数 412 (5) 化为 7 进制是 212 (7). ………… (10 分).", "solution": "null", "level": "高一", "question": "(2019 春・洮北区校级月考)先将 $412(5)$ 化成十进制的数, 然后用 “除 $k$ 取余法” 再化成七进制的数.", "options": [], "subject": "算术", "analysis": "(本小题满分 10 分)\n\n解: $412{ }_{(5)}=2 \\times 5^{0}+1 \\times 5^{1}+4 \\times 5^{2}=2+5+4 \\times 25=107, \\cdots \\cdots \\cdots \\cdots$ ( 5 分),\n\n$\\because 107=2 \\times 7^{0}+1 \\times 7^{1}+2 \\times 7^{2}$\n\n$\\therefore$ 把 5 进制的数 412 (5) 化为 7 进制是 212 (7). ………… (10 分)."} {"id": "16939", "image": [], "answer": "解: (1) 用辗转相除法求 119 和 153 的最大公约数: $153=119 \\times 1+34,119=34 \\times 3+17$, $34=17 \\times 2$\n\n$\\therefore 119$ 和 153 的最大公约数为 17 ;\n\n利用更相减损术求 119 和 153 的最大公约数: $153-119=34,119-34=85,85-34=51,51-34$\n\n$=17,34-17=17$,\n\n$\\therefore 119$ 和 153 的最大公约数就是 17 ;\n\n(2)用秦九韶算法: $f(x)=6 x^{5}+7 x^{4}+5 x^{3}+4 x^{2}+3 x-5=((((6 x+7) x+5) x+4) x+3) x-5$,当 $x=3$ 时, $v_{0}=6, v_{1}=6 \\times 3+7=25, v_{2}=25 \\times 3+5=80, v_{3}=80 \\times 3+4=244, v_{4}=244 \\times 3+3=735$, $v_{5}=735 \\times 3-5=2200$.\n$\\therefore f(3)=2200$.", "solution": "null", "level": "高一", "question": "(2018 秋・杏花岭区校级月考)(1)用辗转相除法或者更相减损术求 119 和 153 的最大公约数;\n\n(2)用秦九韶算法计算多项式 $f(x)=6 x^{5}+7 x^{4}+5 x^{3}+4 x^{2}+3 x-5$, 当 $x=3$ 时的值.", "options": [], "subject": "算术", "analysis": "解: (1) 用辗转相除法求 119 和 153 的最大公约数: $153=119 \\times 1+34,119=34 \\times 3+17$, $34=17 \\times 2$\n\n$\\therefore 119$ 和 153 的最大公约数为 17 ;\n\n利用更相减损术求 119 和 153 的最大公约数: $153-119=34,119-34=85,85-34=51,51-34$\n\n$=17,34-17=17$,\n\n$\\therefore 119$ 和 153 的最大公约数就是 17 ;\n\n(2)用秦九韶算法: $f(x)=6 x^{5}+7 x^{4}+5 x^{3}+4 x^{2}+3 x-5=((((6 x+7) x+5) x+4) x+3) x-5$,当 $x=3$ 时, $v_{0}=6, v_{1}=6 \\times 3+7=25, v_{2}=25 \\times 3+5=80, v_{3}=80 \\times 3+4=244, v_{4}=244 \\times 3+3=735$, $v_{5}=735 \\times 3-5=2200$.\n$\\therefore f(3)=2200$."} {"id": "16940", "image": [], "answer": "解: $f(x)=3 x^{5}+8 x^{4}-3 x^{3}+5 x^{2}+12 x-6=((((3 x+8) x-3) x+5) x+12) x-6$\n\n当 $x=2$ 时, $f(2)=((((3 \\times 2+8) \\times 2-3) \\times 2+5) \\times 2+12) \\times 2-6=238$.", "solution": "null", "level": "高一", "question": "(2019 秋・阳泉期末)用秦九韶算法求 $f(x)=3 x^{5}+8 x^{4}-3 x^{3}+5 x^{2}+12 x-6$ 当 $x=2$ 时的值.", "options": [], "subject": "算术", "analysis": "解: $f(x)=3 x^{5}+8 x^{4}-3 x^{3}+5 x^{2}+12 x-6=((((3 x+8) x-3) x+5) x+12) x-6$\n\n当 $x=2$ 时, $f(2)=((((3 \\times 2+8) \\times 2-3) \\times 2+5) \\times 2+12) \\times 2-6=238$."} {"id": "16962", "image": ["8747.jpg"], "answer": "解:(1)分析程序中各变量、各语句的作用, 再根据图所示的顺序, 可知该程序的作\n\n用是计算分段函数 $y=f(x)=y=\\left\\{\\begin{array}{ll}(x+1)^{2}, & x<0 \\\\ (x-1)^{2}, & x \\geq 0\\end{array}\\right.$ 的函数值,\n\n$\\therefore$ 此程序的功能是求函数 $y=\\left\\{\\begin{array}{ll}(x+1)^{2}, & x<0 \\\\ (x-1)^{2}, & x \\geq 0\\end{array}\\right.$ 的函数值.\n\n故答案为: $y=\\left\\{\\begin{array}{ll}(x+1)^{2}, & x<0 \\\\ (x-1)^{2}, & x \\geq 0\\end{array}\\right.$.\n\n(2) (1)当 $x<0$ 时, $y=(x+1)^{2}=25$, 解得: $x=-6$, 或 4 (舍去);\n\n(2)当 $x \\geqslant 0$ 时,\n\n$y=(x-1)^{2}=25$, 解得: $x=6$, 或 -4 (舍去),\n\n故综上可知输入的 $x$ 的值为 6 , 或 -6 .", "solution": "null", "level": "高一", "question": "(2018 春-延吉市校级月考)阅读下列程序. 回答问题:\n\n(1) 此程序的功能是求函数的函数值.\n\n(2) 若输出 $y$ 的值是 25 , 则输入的 $x$ 应该是多少?\n\n", "options": [], "subject": "算术", "analysis": "解:(1)分析程序中各变量、各语句的作用, 再根据图所示的顺序, 可知该程序的作\n\n用是计算分段函数 $y=f(x)=y=\\left\\{\\begin{array}{ll}(x+1)^{2}, & x<0 \\\\ (x-1)^{2}, & x \\geq 0\\end{array}\\right.$ 的函数值,\n\n$\\therefore$ 此程序的功能是求函数 $y=\\left\\{\\begin{array}{ll}(x+1)^{2}, & x<0 \\\\ (x-1)^{2}, & x \\geq 0\\end{array}\\right.$ 的函数值.\n\n故答案为: $y=\\left\\{\\begin{array}{ll}(x+1)^{2}, & x<0 \\\\ (x-1)^{2}, & x \\geq 0\\end{array}\\right.$.\n\n(2) (1)当 $x<0$ 时, $y=(x+1)^{2}=25$, 解得: $x=-6$, 或 4 (舍去);\n\n(2)当 $x \\geqslant 0$ 时,\n\n$y=(x-1)^{2}=25$, 解得: $x=6$, 或 -4 (舍去),\n\n故综上可知输入的 $x$ 的值为 6 , 或 -6 ."} {"id": "16919", "image": ["8737.jpg"], "answer": "解: (1) 该程序框图的功能是求下面分段函数的函数值 $y=\\left\\{\\begin{array}{ll}x^{2}+1, & \\text { 当 } x<-2 \\text { 时 } \\\\ x^{2}-1, & \\text { 当 } x \\geq-2 \\text { 时 }\\end{array}\\right.$.\n\n(2) 解: 程序如下:\n\nINPUT $x$\n\nIF $x<-2$\n\nTHEN\n\n$y=x^{2}+1$\n\nELSE\n\n$y=x^{2}-1$\n\nEND IF.", "solution": "null", "level": "高一", "question": "013 秋 -三元区校级月考)根据如图程序框图解答下列问题:\n\n(1)指出其功能;\n\n(2)编写对应的计算机程序.\n\n", "options": [], "subject": "图论", "analysis": "解: (1) 该程序框图的功能是求下面分段函数的函数值 $y=\\left\\{\\begin{array}{ll}x^{2}+1, & \\text { 当 } x<-2 \\text { 时 } \\\\ x^{2}-1, & \\text { 当 } x \\geq-2 \\text { 时 }\\end{array}\\right.$.\n\n(2) 解: 程序如下:\n\nINPUT $x$\n\nIF $x<-2$\n\nTHEN\n\n$y=x^{2}+1$\n\nELSE\n\n$y=x^{2}-1$\n\nEND IF."} {"id": "17166", "image": ["8810.jpg", "8810.jpg"], "answer": "解: (1) 设两人到达约会地点的时刻分别为 $x, y$, 依题意, 必须满足 $|x-y| \\leq \\frac{1}{2}$ 才能相遇. 我们把他们到达的时刻分别作为横坐标和纵坐标, 于是两人到达的时刻均匀地分布在一个边长为 1 的正方形 I 内, 如图所示, 而相遇现象则发生在阴影区域 $G$ 内, 即甲、乙两人的到达时刻 $(x, y)$ 满足 $|x-y| \\leq \\frac{1}{2}$, 所以两人相遇的概率为区域 $G$ 与区域 I 的面积之比: $P=\\frac{S_{G}}{S_{I}}=\\frac{1-\\left(\\frac{1}{2}\\right)^{2}}{1}=\\frac{3}{4}$.也就是说, 两个人能在约定的时间内在超市门口相见的概率为 $\\frac{3}{4}$.\n\n(2)设第一枚随机地投掷得到向上一面的点数为 $a$, 第二枚投掷得到向上一面的点数为 $b$, 则 $a$与 $b$ 的和共有 36 种情况.\n\n| $a$ | 1 | 2 | 3 | 4 | 5 | 6 |\n| :--: | :-----: | :-----: | :-----: | :-----: | :-----: | :-----: |\n| $b$ | | | | | | |\n| 1 | $(1,1)$ | $(1,2)$ | $(1,3)$ | $(1,4)$ | $(1,5)$ | $(1,6)$ |\n| 2 | $(2,1)$ | $(2,2)$ | $(2,3)$ | $(2,4)$ | $(2,5)$ | $(2,6)$ |\n| 3 | $(3,1)$ | $(3,2)$ | $(3,3)$ | $(3,4)$ | $(3,5)$ | $(3,6)$ |\n| 4 | $(4,1)$ | $(4,2)$ | $(4,3)$ | $(4,4)$ | $(4,5)$ | $(4,6)$ |\n| 5 | $(5,1)$ | $(5,2)$ | $(5,3)$ | $(5,4)$ | $(5,5)$ | $(5,6)$ |\n| 6 | $(6,1)$ | $(6,2)$ | $(6,3)$ | $(6,4)$ | $(6,5)$ | $(6,6)$ |\n\n所以两次取出的数字之和 $a+b$ 是 5 的倍数的情况有 $(1,4),(4,1) ,(2,3) ,(3,2)$, $(4,6),(6,4),(5,5)$, 共 7 种, 其概率为 $P=\\frac{7}{36}$.\n\n", "solution": "null", "level": "高一", "question": "020 秋 - 迎江区校级期中)小红和小明相约去参加超市的半夜不打烊活动, 两人约定凌晨 0 点到 1 点之间在超市门口相见, 并且先到的必须等后到的人 30 分钟才可以进超市先逛. 如果两个人出发是各自独立的,在 0 点到 1 点的各个时候到达的可能性是相等的.\n\n(1)求两个人能在约定的时间内在超市门口相见的概率;\n\n(2)超市内举行抽奖活动, 郑一枚骰子, 郑 2 次, 如果出现的点数之和是 5 的倍数, 则获奖. 小红参与活动,她获奖的概率是多少呢?\n\n22.(2019 春・楚雄州期中)已知 $-3 \\leqslant x \\leqslant 3,-2 \\leqslant y \\leqslant 2$, 点 $M$ 的坐标为( $x, y$ ).\n\n(1) 求当 $x, y \\in \\mathbf{R}$ 时, 点 $M$ 满足 $x^{2}+(y-2)^{2} \\leqslant 4$ 的概率;\n\n(2) 求当 $x, y \\in \\mathbf{Z}$ 时,点 $M$ 满足 $x^{2}+(y-2)^{2} \\leqslant 4$ 的概率.\n\n23.(2019 秋・张家口月考)已知函数 $f(x)=a x^{2}-b x+1$.\n\n(1) 若 $a, b$ 都是从集合 $\\{0,1 , 2 , 3\\}$ 中任取的一个数, 求函数 $f(x)$ 没有零点的概率;\n\n(2) 分别从集合 $P$ 和 $Q$ 中随机取一个数 $a$ 和 $b$ 得到数对 $(a, b)$, 若 $P=\\{x \\mid 1 \\leqslant x \\leqslant 3\\}, Q=\\{x \\mid 0$ $\\leqslant x \\leqslant 4\\}$, 求函数 $y=f(x)$ 在区间 $[1,+\\infty)$ 上是增函数的概率.", "options": [], "subject": "统计数学", "analysis": "解: (1) 设两人到达约会地点的时刻分别为 $x, y$, 依题意, 必须满足 $|x-y| \\leq \\frac{1}{2}$ 才能相遇. 我们把他们到达的时刻分别作为横坐标和纵坐标, 于是两人到达的时刻均匀地分布在一个边长为 1 的正方形 I 内, 如图所示, 而相遇现象则发生在阴影区域 $G$ 内, 即甲、乙两人的到达时刻 $(x, y)$ 满足 $|x-y| \\leq \\frac{1}{2}$, 所以两人相遇的概率为区域 $G$ 与区域 I 的面积之比: $P=\\frac{S_{G}}{S_{I}}=\\frac{1-\\left(\\frac{1}{2}\\right)^{2}}{1}=\\frac{3}{4}$.也就是说, 两个人能在约定的时间内在超市门口相见的概率为 $\\frac{3}{4}$.\n\n(2)设第一枚随机地投掷得到向上一面的点数为 $a$, 第二枚投掷得到向上一面的点数为 $b$, 则 $a$与 $b$ 的和共有 36 种情况.\n\n| $a$ | 1 | 2 | 3 | 4 | 5 | 6 |\n| :--: | :-----: | :-----: | :-----: | :-----: | :-----: | :-----: |\n| $b$ | | | | | | |\n| 1 | $(1,1)$ | $(1,2)$ | $(1,3)$ | $(1,4)$ | $(1,5)$ | $(1,6)$ |\n| 2 | $(2,1)$ | $(2,2)$ | $(2,3)$ | $(2,4)$ | $(2,5)$ | $(2,6)$ |\n| 3 | $(3,1)$ | $(3,2)$ | $(3,3)$ | $(3,4)$ | $(3,5)$ | $(3,6)$ |\n| 4 | $(4,1)$ | $(4,2)$ | $(4,3)$ | $(4,4)$ | $(4,5)$ | $(4,6)$ |\n| 5 | $(5,1)$ | $(5,2)$ | $(5,3)$ | $(5,4)$ | $(5,5)$ | $(5,6)$ |\n| 6 | $(6,1)$ | $(6,2)$ | $(6,3)$ | $(6,4)$ | $(6,5)$ | $(6,6)$ |\n\n所以两次取出的数字之和 $a+b$ 是 5 的倍数的情况有 $(1,4),(4,1) ,(2,3) ,(3,2)$, $(4,6),(6,4),(5,5)$, 共 7 种, 其概率为 $P=\\frac{7}{36}$.\n\n"} {"id": "14351", "image": ["7300.jpg"], "answer": "(1) $S=3030-\\left(\\frac{15000}{x}+6 x\\right)$, 其定义域是 $(6,500)$.\n\n(2) 设计 $x=50 \\mathrm{~m}, y=60 \\mathrm{~m}$ 时, 运动场地面积最大, 最大值为 2430 平方米.", "solution": "null", "level": "高一", "question": "某市近郊有一块大约 $500 \\mathrm{~m} \\times 500 \\mathrm{~m}$ 的接近正方形的荒地, 地方政府准备在此建一个综合性休闲广场,首先要建设如图所示的一个矩形场地, 其中总面积为 3000 平方米, 其中阴影部分为通道, 通道宽度为 2 米, 中间的三个矩形区域将铺设塑胶地面作为运动场地(其中两个小场地形状相同), 塑胶运动场地占地面积为 $S$ 平方米.\n\n(1)分别用 $X$ 表示 $y$ 和 $S$ 的函数关系式,并给出定义域;\n\n(2)怎样设计能使 $S$ 取得最大值, 并求出最大值.\n\n", "options": [], "subject": "度量几何学", "analysis": "(1) 由已知 $x y=3000, \\therefore y=\\frac{3000}{x}$, 其定义域是 $(6,500)$.\n\n$S=(x-4) a+(x-6) a=(2 x-10) a$,\n\n$\\because 2 a+6=y, \\quad \\therefore a=\\frac{y}{2}-3=\\frac{1500}{x}-3$,\n\n$\\therefore S=(2 x-10) \\cdot\\left(\\frac{1500}{x}-3\\right)=3030-\\left(\\frac{15000}{x}+6 x\\right)$, 其定义域是 $(6,500)$.\n\n(2) $S=3030-\\left(\\frac{15000}{x}+6 x\\right) \\leqslant 3030-2 \\sqrt{6 x \\cdot \\frac{15000}{x}}=3030-2 \\times 300=2430$,\n\n当且仅当 $\\frac{15000}{x}=6 x$, 即 $x=50 \\in(6,500)$ 时, 上述不等式等号成立,\n\n此时, $x=50, y=60, S_{\\text {max }}=2430$.\n\n答: 设计 $x=50 \\mathrm{~m}, y=60 \\mathrm{~m}$ 时, 运动场地面积最大, 最大值为 2430 平方米."} {"id": "16696", "image": [], "answer": "记事件 $\\mathrm{A}=\\{$ 下午 3: 00 后张三、李四可以交谈 $\\}$. 设 $\\mathrm{x}, \\mathrm{y}$ 分别表示张三、李四与基地的距离, 则 $\\mathrm{x}=[0,30], \\mathrm{y}=[0,40]$, 则他们的所有距离的数据构成有序实数对 $(x, y)$, 则所有这样的有序实数对构成的集合为试验的全部结果. 以基地为原点, 正东、正北方向分别为 $\\mathrm{x}$ 轴、 $\\mathrm{y}$ 轴正方向建立坐标系 (图略), 则长和宽分别为 $40 \\mathrm{~km}$ 和 $30 \\mathrm{~km}$\n的矩形区域表示该试验的所有结果构成的区域, 它的总面积为 $1200 \\mathrm{~km} 2$, 可以交谈的区域为 $x^{2}+y^{2} \\leq 25^{2}$ 的圆及其内部满足 $x \\geqslant 0, y \\geqslant 0$ 的部分, 由几何概型的概率计算公式得 $P(A)=\\frac{\\frac{1}{4} \\times \\pi \\times 25^{2}}{1200}=\\frac{25 \\pi}{192} \\approx 0.41$", "solution": "null", "level": "高一", "question": "两对讲机持有者张三、李四在某货运公司工作,他们的对讲机的接收范围是 $25 \\mathrm{~km}$,下午 3: 00 张三在基地正东 30km 处向基地行驶, 李四在基地正北 $40 \\mathrm{~km}$ 处也向基地行驶,试求下午 3: 00 后他们可以交谈的概率。", "options": [], "subject": "度量几何学", "analysis": "记事件 $\\mathrm{A}=\\{$ 下午 3: 00 后张三、李四可以交谈 $\\}$. 设 $\\mathrm{x}, \\mathrm{y}$ 分别表示张三、李四与基地的距离, 则 $\\mathrm{x}=[0,30], \\mathrm{y}=[0,40]$, 则他们的所有距离的数据构成有序实数对 $(x, y)$, 则所有这样的有序实数对构成的集合为试验的全部结果. 以基地为原点, 正东、正北方向分别为 $\\mathrm{x}$ 轴、 $\\mathrm{y}$ 轴正方向建立坐标系 (图略), 则长和宽分别为 $40 \\mathrm{~km}$ 和 $30 \\mathrm{~km}$\n的矩形区域表示该试验的所有结果构成的区域, 它的总面积为 $1200 \\mathrm{~km} 2$, 可以交谈的区域为 $x^{2}+y^{2} \\leq 25^{2}$ 的圆及其内部满足 $x \\geqslant 0, y \\geqslant 0$ 的部分, 由几何概型的概率计算公式得 $P(A)=\\frac{\\frac{1}{4} \\times \\pi \\times 25^{2}}{1200}=\\frac{25 \\pi}{192} \\approx 0.41$"} {"id": "16607", "image": ["8605.jpg"], "answer": "| INPUT $x$ |\n| :--- |\n| IF $x<0$ THEN |\n| $\\quad$ PRINT $x$ |\n| END IF |\n| END |", "solution": "null", "level": "高一", "question": "根据图所示的程序框图, 设计一个程序\n\n", "options": [], "subject": "逻辑题", "analysis": "| INPUT $x$ |\n| :--- |\n| IF $x<0$ THEN |\n| $\\quad$ PRINT $x$ |\n| END IF |\n| END |"} {"id": "16621", "image": ["8615.jpg", "8615.jpg"], "answer": "", "solution": "null", "level": "高一", "question": "根据下面的程序画出相应的程序框图.\n\n| $\\mathrm{x}=-1$ | |\n| :--- | ---: | :--- |\n| WHILE | $\\quad \\mathrm{x}<=1$ |\n| | $\\mathrm{y}=\\mathrm{x} * \\mathrm{x} * \\mathrm{x}$ |\n| | $\\mathrm{x}=\\mathrm{x}+0.2$ |\n| PRINT | $\\mathrm{y}$ |\n| WEND | |\n| END | |", "options": [], "subject": "逻辑题", "analysis": ""} {"id": "16698", "image": ["8640.jpg", "8641.jpg", "8641.jpg"], "answer": "如图所示, 设正方形 $\\mathrm{ABCD}$ 的边长为 $4 \\mathrm{a}$, 硬币的半径 $\\mathrm{r}=\\mathrm{a}$, 则正方形 $\\mathrm{O}_{1} \\mathrm{O}_{2} \\mathrm{O}_{3} \\mathrm{O}_{4}$的面积为 $4 \\mathrm{a} 2$, 曲边多边形 $\\mathrm{EFGHI}$ JKL 的面积为 $16 a^{2}+4 \\times(4 a \\times r)+4 \\times \\frac{1}{4} \\pi r^{2}=32 a^{2}+\\pi a^{2}$.记 “硬币完全落入正方形内” 为事件 A, 则硬币完全落入正方形内的概率应等于正方形 $\\mathrm{O}_{1} \\mathrm{O}_{2} \\mathrm{O}_{3} \\mathrm{O}_{4}$ 的面积与曲边多边形 EFGHIUKL 的面 $4 \\mathrm{a} 24$ 积比, 即 $P(A)=\\frac{4 a^{2}}{32 a^{2}+\\pi a^{2}}=\\frac{4}{32+\\pi}$\n\n", "solution": "null", "level": "高一", "question": "如图所示, 在地上画一个正方形方框, 其边长等于一枚硬币直径的 2 倍, 向方框中投硬币,硬币完全落在正方形外的情况不计,求硬币完全落在正方形内的概率\n\n", "options": [], "subject": "组合几何学", "analysis": "如图所示, 设正方形 $\\mathrm{ABCD}$ 的边长为 $4 \\mathrm{a}$, 硬币的半径 $\\mathrm{r}=\\mathrm{a}$, 则正方形 $\\mathrm{O}_{1} \\mathrm{O}_{2} \\mathrm{O}_{3} \\mathrm{O}_{4}$的面积为 $4 \\mathrm{a} 2$, 曲边多边形 $\\mathrm{EFGHI}$ JKL 的面积为 $16 a^{2}+4 \\times(4 a \\times r)+4 \\times \\frac{1}{4} \\pi r^{2}=32 a^{2}+\\pi a^{2}$.记 “硬币完全落入正方形内” 为事件 A, 则硬币完全落入正方形内的概率应等于正方形 $\\mathrm{O}_{1} \\mathrm{O}_{2} \\mathrm{O}_{3} \\mathrm{O}_{4}$ 的面积与曲边多边形 EFGHIUKL 的面 $4 \\mathrm{a} 24$ 积比, 即 $P(A)=\\frac{4 a^{2}}{32 a^{2}+\\pi a^{2}}=\\frac{4}{32+\\pi}$\n\n"} {"id": "15590", "image": ["7818.jpg", "7819.jpg", "7820.jpg"], "answer": "图正确, 俯视图错误, 图见解析", "solution": "null", "level": "高一", "question": "如图所示, 图 (2) 是图 (1) 中实物的主视图和俯视图, 你认为正确吗? 如果不正确, 请找出错误并改正, 然后画出它的左视图.\n\n", "options": [], "subject": "画法几何学", "analysis": "【分析】\n\n主视图正确,俯视图不正确,要用虚线画出被遮住的轮廓.根据左视方向可作出左视图.\n\n## 【详解】\n\n图 (1) 是由两个长方体组合而成的, 主视图正确, 俯视图错误. 俯视图应该画出不可见轮廓 (用虚线表示), 左视图轮廓是一个矩形, 有一条可见的交线 (用实线表示), 正确画法如下图所示.\n\n\n\n俯视图\n\n\n\n左视图\n\n## 【点睛】\n\n本题考查了几何体的左视图的作法,即画从左看几何体的轮廓,属于基础题."} {"id": "15591", "image": ["7821.jpg", "7822.jpg", "7823.jpg", "7824.jpg"], "answer": "析", "solution": "null", "level": "高一", "question": "如图所示, 四棱锥的所有棱长都为 $a$, 试画出其三视图.\n\n", "options": [], "subject": "画法几何学", "analysis": "【分析】\n\n正视图与侧视图均为等腰三角形、俯视图为对角线连接完整的正方形, 按三视图的画法逐个画出即可。\n\n## 【详解】\n\n四棱雉的三视图如图所示.\n\n\n\n正视图\n\n\n\n俯视图\n\n\n\n儉祝图\n\n## 【点睛】\n\n(1)一个物体的三视图的排列规则是: 俯视图放在正视图的下面, 长度与正视图一样; 侧视图放在正视图的右面, 高度与正视图一样, 宽度与俯视图的宽度一样, 即“长对正、高平齐、宽相等”。\n\n(2)三视图应满足: (1)棱用实线表示; (2)被挡的轮廓线用虚线表示; (3)不同的投影面三视图也不一样, 一般所取的投影面是较规则图形。\n\n(3)本例中, 正视图与侧视图均非正三角形。"} {"id": "15723", "image": ["8037.jpg"], "answer": "析", "solution": "null", "level": "高一", "question": "所示, $O A, O B, O C$ 为不共面的三条线段, 点 $A_{1}, B_{1}, C_{1}$ 分别是 $O A, O B$, $O C$ 上的点, 且 $\\frac{O A_{1}}{O A}=\\frac{O B_{1}}{O B}=\\frac{O C_{1}}{O C}$ 成立.求证: $\\triangle A_{1} B_{1} C_{1} \\sim \\triangle A B C$.\n\n", "options": [], "subject": "变换几何", "analysis": "【分析】\n\n根据 $\\frac{O A_{1}}{O A}=\\frac{O B_{1}}{O B}=\\frac{O C_{1}}{O C}$, 可得 $A_{1} B_{1} / / A B, A C_{1} / / A C, B_{1} C_{1} / / B C$ 进而通过平行线得两个角 $\\angle C_{1} A_{1} B_{1}=\\angle C A B$ 和 $\\angle A_{1} B_{1} C_{1}=\\angle A B C$ 对应相等,即可证明 $\\triangle A_{1} B_{1} C_{1} \\sim \\triangle A B C$.\n\n## 【详解】\n\n证明; 在 $\\triangle O A B$ 中,因为 $\\frac{O A_{1}}{O A}=\\frac{O B_{1}}{O B_{1}}$,\n\n所以 $A_{1} B_{1} / / A B$.\n\n同理可证 $A_{1} C_{1} / / A C, B_{1} C_{1} / / B C$.\n\n所以 $\\angle C_{1} A_{1} B_{1}=\\angle C A B, \\angle A_{1} B_{1} C_{1}=\\angle A B C$.\n\n所以 $\\triangle A_{1} B_{1} C_{1} \\sim \\triangle A B C$.\n\n## 【点睛】\n\n本题考查了通过线段成比例, 证明线线平行,根据空间中角的两边分别平行判断两个角的关系,属于基础题."} {"id": "6357", "image": ["425.jpg"], "answer": "1) 长方; 2\n\n(2) $3 ; 1 ; 2$\n\n(3) 正方体;长方体", "solution": "null", "level": "一年级", "question": "一认,填一填。\n\n\n\n(1)从左边起,第 4 个是 \\$ \\qquad \\$体,第 \\$ \\qquad \\$个是正方体。\n\n(2) 图中有 \\$ \\qquad \\$个圆柱, \\$ \\qquad \\$个长方体,个球。\n\n(3) \\$ \\qquad \\$ ,右面是 \\$ \\qquad \\$", "options": [], "subject": "立体几何学", "analysis": "1) 长方; 2\n\n(2) $3 ; 1 ; 2$\n\n(3) 正方体;长方体"} {"id": "6147", "image": [], "answer": "8", "solution": "null", "level": "一年级", "question": "至少要用 \\$ \\qquad \\$个小正方体可以拼成一个大正方体。", "options": [], "subject": "立体几何学", "analysis": "8"} {"id": "5941", "image": ["79.jpg", "80.jpg", "80.jpg"], "answer": "", "solution": "null", "level": "一年级", "question": "能站稳的那一组圈起来。\n", "options": [], "subject": "立体几何学", "analysis": ""} {"id": "6153", "image": ["252.jpg", "253.jpg", "253.jpg"], "answer": "", "solution": "null", "level": "一年级", "question": "(4 分)在正方体下面画 “ $\\sqrt{ }$ ”,圆柱下面画 “ $\\bigcirc$ ”。\n\n", "options": [], "subject": "立体几何学", "analysis": ""} {"id": "6358", "image": [], "answer": "=; \\quad>\\quad>$", "solution": "null", "level": "一年级", "question": "横线上填上 “<”“>” 或 $\"=”$. $1+4$ 5 3 $5-3$\n\n$4-1$ $1+1$", "options": [], "subject": "算术", "analysis": "=; \\quad>\\quad>$"} {"id": "6361", "image": [], "answer": "、2、3; 4、5; 3、4、5; 1、2、3、4", "solution": "null", "level": "一年级", "question": "()里填上合适的数。\n\n$4>(\\quad) \\quad 3<(\\quad) \\quad 2<(\\quad) \\quad 5>(\\quad)$", "options": [], "subject": "算术", "analysis": "、2、3; 4、5; 3、4、5; 1、2、3、4"} {"id": "6362", "image": ["427.jpg"], "answer": ",1", "solution": "null", "level": "一年级", "question": "一算, 结果是多少?\n\n", "options": [], "subject": "算术", "analysis": ",1"} {"id": "6364", "image": [], "answer": "2+3=5$\n\n$2-2=0$\n\n$4-0=4$\n\n$1+1=2$\n\n$5+0=5$\n\n$0+1=1$\n\n$5-4=1$\n\n$1+4=5$\n\n$4-4=0 \\quad 2-0=2 \\quad 4-3=1 \\quad 0+3=3$\n\n$4-1=3 \\quad 1+3=4 \\quad 2+0=2 \\quad 3-1=2$", "solution": "null", "level": "一年级", "question": "谁算得又对又快。\n\n| $2+3=$ | $2-2=$ | $4-0=$ | $1+1=$ |\n| :--- | :--- | :--- | :--- |\n| $5+0=$ | $0+1=$ | $5-4=$ | $1+4=$ |\n| $4-4=$ | $2-0=$ | $4-3=$ | $0+3=$ |\n| $4-1=$ | $1+3=$ | $2+0=$ | $3-1=$ |", "options": [], "subject": "算术", "analysis": "2+3=5$\n\n$2-2=0$\n\n$4-0=4$\n\n$1+1=2$\n\n$5+0=5$\n\n$0+1=1$\n\n$5-4=1$\n\n$1+4=5$\n\n$4-4=0 \\quad 2-0=2 \\quad 4-3=1 \\quad 0+3=3$\n\n$4-1=3 \\quad 1+3=4 \\quad 2+0=2 \\quad 3-1=2$"} {"id": "6366", "image": ["432.jpg", "433.jpg"], "answer": "图:\n$$\n6,7,8 、 9\n$$", "solution": "null", "level": "一年级", "question": "图照样子圈数.\n\n\n$7 、 6 、 4 、(5)$\n\n\n\n$6 、 7 、 8 、 9$", "options": [], "subject": "算术", "analysis": "图:\n$$\n6,7,8 、 9\n$$"} {"id": "6368", "image": ["435.jpg"], "answer": "3+2=5 \\quad 5-2=3 \\quad 4+1=5 \\quad 4-4=0$", "solution": "null", "level": "一年级", "question": "图列式计算。\n", "options": [], "subject": "算术", "analysis": "3+2=5 \\quad 5-2=3 \\quad 4+1=5 \\quad 4-4=0$"} {"id": "6369", "image": ["436.jpg"], "answer": "1+4=5$ (个) $; 4+1=5$ (个) $; 2+3=5$ (个) $; 3+2=5$ (个)", "solution": "null", "level": "一年级", "question": "图是一家超市里卖的袋装橙子. 如果要买两袋共 5 个橙子, 可以怎样买? 请你用算式的形式表示出来.\n\n", "options": [], "subject": "算术", "analysis": "1+4=5$ (个) $; 4+1=5$ (个) $; 2+3=5$ (个) $; 3+2=5$ (个)"} {"id": "6383", "image": ["448.jpg"], "answer": "1) $8 ; 15$\n(2) $13 ; 6$", "solution": "null", "level": "一年级", "question": "一数, 填一填。\n\n$\\triangle \\triangle \\triangle \\triangle \\triangle \\triangle \\triangle \\circ \\triangle \\Delta \\triangle \\triangle \\Delta \\triangle \\square \\triangle \\triangle \\triangle \\triangle \\triangle$\n\n(1) 从左往右数, \\$ \\qquad \\$排第 \\$ \\qquad \\$ .\n\n(2) 从右往左数,\n\n\\$ \\qquad \\$ , $\\square$排第 \\$ \\qquad \\$ .\n\n排第\n\n年", "options": [], "subject": "算术", "analysis": "1) $8 ; 15$\n(2) $13 ; 6$"} {"id": "6384", "image": [], "answer": "+ +", "solution": "null", "level": "一年级", "question": "○上填上 “+” 或 “-”。\n\n$3 \\circ 2=5 \\quad 2 \\circ 2=4 \\quad 1 \\circ 4=5$", "options": [], "subject": "算术", "analysis": "+ +"} {"id": "6386", "image": ["449.jpg", "450.jpg"], "answer": "1 ; 5$", "solution": "null", "level": "一年级", "question": "空。\n\n\n$4-3=$\n\n\n$1+4=$", "options": [], "subject": "算术", "analysis": "1 ; 5$"} {"id": "6388", "image": [], "answer": "; 2; 5; 5; 2; 1; 4; 4; 1; 3; 1; 5; 5; 5; 0; 3; 0; 4; 3; 3", "solution": "null", "level": "一年级", "question": "会算。\n\n| $4-3=$ | $3-1=$ | $5+0=$ | $3+2=$ |\n| :--- | :--- | :--- | :--- |\n| $5-3=$ | $3-2=$ | $3+1=$ | $2+2=$ |\n| $5-4=$ | $3-0=$ | $2-1=$ | $1+4=$ |\n| $3+2=$ | $5-0=$ | $4-4=$ | $2+1=$ |\n| $0+0=$ | $5-1=$ | $5-2=$ | $0+3=$ |", "options": [], "subject": "算术", "analysis": "; 2; 5; 5; 2; 1; 4; 4; 1; 3; 1; 5; 5; 5; 0; 3; 0; 4; 3; 3"} {"id": "6390", "image": ["453.jpg", "454.jpg"], "answer": "3", "solution": "null", "level": "一年级", "question": "一做,你发现了什么?\n\n\n\n$4-3=(\\quad)$\n\n\n\n$4-1=(\\quad)$", "options": [], "subject": "算术", "analysis": "3"} {"id": "6391", "image": ["455.jpg", "456.jpg", "456.jpg"], "answer": "图:\n\n", "solution": "null", "level": "一年级", "question": "图照样子圈数。\n\n", "options": [], "subject": "算术", "analysis": "图:\n\n"} {"id": "6060", "image": [], "answer": "9", "solution": "null", "level": "一年级", "question": "同学们排队取餐, 小明的前面有 3 人, 后面有 5 人, 这队一共有 ( )人。", "options": [], "subject": "算术", "analysis": "9"} {"id": "6061", "image": [], "answer": "$7 ; 8$", "solution": "null", "level": "一年级", "question": "比 9 小比 6 大的数是 ( ) 和 ( )。", "options": [], "subject": "算术", "analysis": "$7 ; 8$"} {"id": "6062", "image": [], "answer": "5", "solution": "null", "level": "一年级", "question": "李老师有 10 个 , 要分给小明和小红, 如果给两个人的同样多, 小明能分()个\\}。", "options": [], "subject": "算术", "analysis": "5"} {"id": "6063", "image": [], "answer": "二", "solution": "null", "level": "一年级", "question": "有两箱同样的牛奶, 喝了一些, 第一箱还剩 7 瓶, 第二箱还剩 5 瓶, 第 $(\\quad)$ 箱喝得多。", "options": [], "subject": "算术", "analysis": "二"} {"id": "6064", "image": [], "answer": "$9 ; 8 ; 10$", "solution": "null", "level": "一年级", "question": "最大的一位数是(),与它相邻的数是()与()。", "options": [], "subject": "算术", "analysis": "$9 ; 8 ; 10$"} {"id": "6065", "image": ["166.jpg"], "answer": "3", "solution": "null", "level": "一年级", "question": "小红有 10 支铅笔,小华有 4 支铅笔,小红给小华()支铅笔,他俩的铅笔数一样多了。\n\n", "options": [], "subject": "算术", "analysis": "3"} {"id": "6066", "image": ["167.jpg", "168.jpg", "169.jpg", "170.jpg"], "answer": "$5 ; 6 ; 9 ; 3$", "solution": "null", "level": "一年级", "question": "\n3\n\n\n\n\n4\n5\n\n", "options": [], "subject": "算术", "analysis": "$5 ; 6 ; 9 ; 3$"} {"id": "6067", "image": [], "answer": "3", "solution": "null", "level": "一年级", "question": "排队时玲玲排第 4 ,东东排第 8 ,东东和玲玲之间有( )人。", "options": [], "subject": "算术", "analysis": "3"} {"id": "6068", "image": [], "answer": "7", "solution": "null", "level": "一年级", "question": "李明读一本书,他从第 4 页读到第 10 页,一共读了( )页。", "options": [], "subject": "算术", "analysis": "7"} {"id": "6073", "image": ["174.jpg", "175.jpg", "175.jpg"], "answer": "", "solution": "null", "level": "一年级", "question": "(5分)猫抓老鼠。\n", "options": [], "subject": "算术", "analysis": ""} {"id": "6074", "image": ["176.jpg", "176.jpg"], "answer": "( $(1)$\n\n| $8-7$ |\n| :--- |\n| $6+3$ |\n| $7-3$ |\n| $2+3$ |\n\n(2)\n\n| $10-2$ |\n| :---: |\n| $4+5$ |\n| $8-3$ |\n| $4+3$ |\n\n", "solution": "null", "level": "一年级", "question": "( 8 分) 连一连。\n\n| $8-7$ |\n| :---: |\n| $6+3$ |\n| $7-3$ |\n| $2+3$ |$\\quad$| 5 |\n| :---: |\n| 4 |\n| 9 |\n| 1 |\n\n(2)\n\n| $10-2$ |\n| :---: |\n| $4+5$ |\n| $8-3$ |\n| $4+3$ |\n\n\n| 9 |\n| :--- |\n| 7 |\n| 5 |\n| 8 |", "options": [], "subject": "算术", "analysis": "( $(1)$\n\n| $8-7$ |\n| :--- |\n| $6+3$ |\n| $7-3$ |\n| $2+3$ |\n\n(2)\n\n| $10-2$ |\n| :---: |\n| $4+5$ |\n| $8-3$ |\n| $4+3$ |\n\n"} {"id": "6075", "image": ["177.jpg", "178.jpg", "178.jpg"], "answer": "", "solution": "null", "level": "一年级", "question": "(8 分)我会连一连。\n", "options": [], "subject": "算术", "analysis": ""} {"id": "6076", "image": ["179.jpg", "180.jpg", "180.jpg"], "answer": "", "solution": "null", "level": "一年级", "question": "( 5 分 ) 开锁。(算一算, 连一连)\n", "options": [], "subject": "算术", "analysis": ""} {"id": "6077", "image": [], "answer": "解: $7-5=2$ (个)\n\n答: 还剩 2 个。", "solution": "null", "level": "一年级", "question": "( 4 分) 原来有 7 个苹果, 摘了 5 个, 还剩多少个?", "options": [], "subject": "算术", "analysis": "解: $7-5=2$ (个)\n\n答: 还剩 2 个。"} {"id": "6079", "image": ["182.jpg"], "answer": "$9-5+4=8$", "solution": "null", "level": "一年级", "question": "4 分)这辆公共汽车上原来有 9 位乘客,途中有 5 位乘客下车, 4 位乘客上车, 这时车上共有多少位乘客?\n\n", "options": [], "subject": "算术", "analysis": "$9-5+4=8$"} {"id": "6081", "image": [], "answer": "解: $7+2=9$ (只)\n\n答:小红折了 9 只小船。", "solution": "null", "level": "一年级", "question": "( 4 分 )手工课上,小明折了 7 只,小红比小明多折了 2 只,小红折了几只小船 $?$", "options": [], "subject": "算术", "analysis": "解: $7+2=9$ (只)\n\n答:小红折了 9 只小船。"} {"id": "6667", "image": ["647.jpg", "648.jpg", "648.jpg"], "answer": "详解\n\n\n\n$\\triangle \\Delta \\Delta \\Delta \\Delta \\Delta \\quad \\triangle \\Delta \\Delta$\n\n算式: $\\quad 12-3=9$", "solution": "null", "level": "一年级", "question": "$\\triangle$ 比 $O$ 少 3 个, $\\triangle$ 有多少个? 画一画, 填一填。\n\n$\\bigcirc 00000 \\bigcirc 00 \\bigcirc 00$\n\n算式:\n\n", "options": [], "subject": "算术", "analysis": "详解\n\n\n\n$\\triangle \\Delta \\Delta \\Delta \\Delta \\Delta \\quad \\triangle \\Delta \\Delta$\n\n算式: $\\quad 12-3=9$"} {"id": "6668", "image": [], "answer": "详解$11-5=6$\n\n$15-8=7$\n\n$16-7=9$\n\n$18-9=9$\n\n$12-7=5$\n\n$13-4=9$\n\n$17-8=9$\n\n$15-7=8$\n\n$14-9=5$\n\n$10-2=8$\n\n$12-9=3$\n\n$14-5=9$\n\n$12-8=4$\n\n$11-4=7$\n\n$11-8=3$\n\n$13-8=5$", "solution": "null", "level": "一年级", "question": "(8 分) 直接写得数。\n$11-5=$\n$15-8=$\n$16-7=$\n$18-9=$\n\n| $12-7=$ | $13-4=$ | $17-8=$ | $15-7=$ |\n| :--- | :--- | :--- | :--- |\n| $14-9=$ | $10-2=$ | $12-9=$ | $14-5=$ |\n| $12-8=$ | $11-4=$ | $11-8=$ | $13-8=$ |", "options": [], "subject": "算术", "analysis": "详解$11-5=6$\n\n$15-8=7$\n\n$16-7=9$\n\n$18-9=9$\n\n$12-7=5$\n\n$13-4=9$\n\n$17-8=9$\n\n$15-7=8$\n\n$14-9=5$\n\n$10-2=8$\n\n$12-9=3$\n\n$14-5=9$\n\n$12-8=4$\n\n$11-4=7$\n\n$11-8=3$\n\n$13-8=5$"} {"id": "6669", "image": ["649.jpg", "650.jpg", "651.jpg", "652.jpg", "653.jpg"], "answer": "详解(1) $13-6=7$ (个)\n\n(2) $11-3=8$ (双)\n\n(3) $12-4=8$ (块)\n\n(4) $14-9=5$ (支)", "solution": "null", "level": "一年级", "question": "(12 分)看图列式并计算。\n\n(1) 共有 13 个。\n\n\n\n$\\square \\bigcirc \\square=\\square$ (个)\n\n\n\n(2)共有 11 双。\n\n\n$\\square \\bigcirc \\square=\\square$ (双)\n\n(3) 共有 12 块。\n\n\n\n(4)\n\n\n\n$\\square \\bigcirc \\square=\\square$ (支)", "options": [], "subject": "算术", "analysis": "详解(1) $13-6=7$ (个)\n\n(2) $11-3=8$ (双)\n\n(3) $12-4=8$ (块)\n\n(4) $14-9=5$ (支)"} {"id": "6670", "image": ["654.jpg"], "answer": "详解解: $14-6=8$ (人)\n\n答: 现在车上有 8 人。", "solution": "null", "level": "一年级", "question": "(5 分)现在车上有多少人?\n\n", "options": [], "subject": "算术", "analysis": "详解解: $14-6=8$ (人)\n\n答: 现在车上有 8 人。"} {"id": "6671", "image": [], "answer": "详解(1) 解: $11-2=9$ (枚)\n\n答: 还剩 9 枚邮票。\n\n(2) 解: $11-9=2$ (本)\n\n答: 还剩 2 本练习本。", "solution": "null", "level": "一年级", "question": "(10 分)解决问题。\n\n(1)(5 分)小永有 11 枚邮票, 送给小华 2 枚, 还剩多少枚邮票?\n\n(2)(5 分)小兰有 11 本练习本, 用了 9 本,还剩几本练习本?", "options": [], "subject": "算术", "analysis": "详解(1) 解: $11-2=9$ (枚)\n\n答: 还剩 9 枚邮票。\n\n(2) 解: $11-9=2$ (本)\n\n答: 还剩 2 本练习本。"} {"id": "6673", "image": [], "answer": "详解解: $13-7=6$ (辆)\n\n答: 停车场还剩下 6 辆车。", "solution": "null", "level": "一年级", "question": "(5 分) 停车场原来停车 13 辆, 开走了 7 辆, 停车场还剩下多少辆车?", "options": [], "subject": "算术", "analysis": "详解解: $13-7=6$ (辆)\n\n答: 停车场还剩下 6 辆车。"} {"id": "6674", "image": [], "answer": "详解$12-3=9$ (筐)\n\n答: 土豆有 9 筐。", "solution": "null", "level": "一年级", "question": "(5 分)萝卜有 12 筐, 土豆比萝卜少 3 筐。土豆有多少筐。", "options": [], "subject": "算术", "analysis": "详解$12-3=9$ (筐)\n\n答: 土豆有 9 筐。"} {"id": "6675", "image": [], "answer": "详解$11-6=5$ (只)\n\n答: 大象比老虎多 5 只。", "solution": "null", "level": "一年级", "question": "(5 分) 动物园里有 6 只老虎, 11 只大象。大象比老虎多多少只。", "options": [], "subject": "算术", "analysis": "详解$11-6=5$ (只)\n\n答: 大象比老虎多 5 只。"} {"id": "7216", "image": [], "answer": "3; 14; 13; 11; 12; 14; $8 ; 9 ; 9 ; 8 ; 9 ; 12$", "solution": "null", "level": "一年级", "question": "$5+8=$ $9+5=$ $8+5=$\n\n$3+8=$ $8+4=$ \\$ \\qquad \\$ $5+9=$\n\n$12-4=$ $12-3=$ \\$ \\qquad \\$ $11-2=$\n\n$11-3=$ \\$ \\qquad \\$ $13-4=$ \\$ \\qquad \\$ $5+7=$", "options": [], "subject": "算术", "analysis": "3; 14; 13; 11; 12; 14; $8 ; 9 ; 9 ; 8 ; 9 ; 12$"} {"id": "7218", "image": ["968.jpg"], "answer": "(1) $7 、 8 、 10 、 11 、 14 、 15 、 16 、 18$\n\n(2) 14\n\n(3) (1) 8 (2) 4 (3) 2 (4) 19,2", "solution": "null", "level": "一年级", "question": "\n\n(2) 15 比()多 1 。\n\n(3) $2 、 4 、 5 、 8 、 11 、 13 、 17 、 19$\n\n(1)这里一共有 () 个数;\n\n(2) 从左边起, 8 排在第 () 个;\n\n(3)从右边起, 最后数是 () ;\n\n(4)最大的数是 (), 最小的数是 ()。", "options": [], "subject": "算术", "analysis": "(1) $7 、 8 、 10 、 11 、 14 、 15 、 16 、 18$\n\n(2) 14\n\n(3) (1) 8 (2) 4 (3) 2 (4) 19,2"} {"id": "7219", "image": ["2951.jpg"], "answer": "3-4=9(个)", "solution": "null", "level": "一年级", "question": "\n\n$\\square \\mathrm{O} \\square=\\square$ (个)", "options": [], "subject": "算术", "analysis": "3-4=9(个)"} {"id": "7220", "image": [], "answer": "| | 8 | 12 | 13 | 7 | 9 | 11 |\n| :---: | :---: | :---: | :---: | :---: | :---: | :---: |\n| | 7 | 3 | 5 | 5 | 4 | 2 |\n| 和 | 15 | 15 | 18 | 12 | 13 | 13 |\n| 差 | 1 | 9 | 8 | 2 | 5 | 9 |", "solution": "null", "level": "一年级", "question": "下两个数, 先算加, 再算减。\n\n| | 8 | 12 | 13 | 7 | 9 | 11 |\n| :---: | :---: | :---: | :---: | :---: | :---: | :---: |\n| | 7 | 3 | 5 | 5 | 4 | 2 |\n| 和 | | | | | | |\n| 差 | | | | | | |", "options": [], "subject": "算术", "analysis": "| | 8 | 12 | 13 | 7 | 9 | 11 |\n| :---: | :---: | :---: | :---: | :---: | :---: | :---: |\n| | 7 | 3 | 5 | 5 | 4 | 2 |\n| 和 | 15 | 15 | 18 | 12 | 13 | 13 |\n| 差 | 1 | 9 | 8 | 2 | 5 | 9 |"} {"id": "7221", "image": [], "answer": "$3 ; 5 ; 15 ; 19 ; 4 ; 8 ; 0 ; 10 ; 6$", "solution": "null", "level": "一年级", "question": "算。\n\n$13-5-5=9+4-8=\\quad 11-4+8=$\n\n$8+6+5=\\quad 16-8-4=\\quad 5+7-4=$\n\n$16-7-9=\\quad 14-9+5=\\quad 15-2-7=$", "options": [], "subject": "算术", "analysis": "$3 ; 5 ; 15 ; 19 ; 4 ; 8 ; 0 ; 10 ; 6$"} {"id": "7222", "image": [], "answer": "1) 5 架(2) 7 人(3)16个(4)16人(5) 5 个", "solution": "null", "level": "一年级", "question": "用题。\n\n", "options": [], "subject": "算术", "analysis": "1) 5 架(2) 7 人(3)16个(4)16人(5) 5 个"} {"id": "7223", "image": ["969.jpg", "969.jpg"], "answer": "", "solution": "null", "level": "一年级", "question": "接写出得数。\n\n$29+30=\\quad 24+50=\\quad 88-4=\\quad 58+7=, \\quad 5+67=$\n\n$55-9=\\quad 24+60=\\quad 85-40=\\quad 42-7=\\quad 57-7=$", "options": [], "subject": "算术", "analysis": ""} {"id": "7224", "image": ["970.jpg"], "answer": "$64-30=34$ (张)", "solution": "null", "level": "一年级", "question": "两本影集一共有 64 张照片。\n \n\n30 张\n\n? 张", "options": [], "subject": "算术", "analysis": "$64-30=34$ (张)"} {"id": "7225", "image": [], "answer": "$100-30-40=30$ (元)或 $100-(30+40)=$ = 30 (元)", "solution": "null", "level": "一年级", "question": "妈妈带了 100 元,买一件上衣用了 30 元,买一条裙子用了 40 元。还剩多少钱?", "options": [], "subject": "算术", "analysis": "$100-30-40=30$ (元)或 $100-(30+40)=$ = 30 (元)"} {"id": "7226", "image": ["971.jpg"], "answer": "$6+6+6=18$ (个)", "solution": "null", "level": "一年级", "question": "三只猴子一共端来多少个桃子?\n\n", "options": [], "subject": "算术", "analysis": "$6+6+6=18$ (个)"} {"id": "7228", "image": [], "answer": "$36 \\quad 82 \\quad 63 \\quad 77 \\quad 36 \\quad 48 \\quad 30 \\quad 37$", "solution": "null", "level": "一年级", "question": "算下面各题。\n\n$23+5+7=\\quad 84-(10-8)=\\quad 93-(50-20)=\\quad 45-8+40=$\n\n$66-(20+10)=\\quad 58-(19-9)=\\quad 35-2-3=\\quad 49-(9+3)=$", "options": [], "subject": "算术", "analysis": "$36 \\quad 82 \\quad 63 \\quad 77 \\quad 36 \\quad 48 \\quad 30 \\quad 37$"} {"id": "7230", "image": [], "answer": "$=<<$ ह $>><$", "solution": "null", "level": "一年级", "question": "○里填上 \">\" \"<\" 或 \"=\"。$38+20 \\bigcirc 58 \\quad 23-4 \\bigcirc 20 \\quad 74-20 \\bigcirc 50+5$\n\n$36-7 \\bigcirc 28 \\quad 85 \\bigcirc 90-8 \\quad 47+20 \\bigcirc 79-9$", "options": [], "subject": "算术", "analysis": "$=<<$ ह $>><$"} {"id": "7231", "image": ["974.jpg"], "answer": "40 5 45 4 $\\quad 5 \\quad 40 \\quad 45$\n\n$\\begin{array}{llllll}45 & 5 & 40 & 4.5 & 40 & 5\\end{array}$", "solution": "null", "level": "一年级", "question": "看图列式计算。\n \n $(\\quad)+(\\quad)=(\\quad)$ (元)\n $(\\quad)-(\\quad)=(\\quad)$ (元)\n $(\\quad)-(\\quad)=(\\quad)$ (元)", "options": [], "subject": "算术", "analysis": "40 5 45 4 $\\quad 5 \\quad 40 \\quad 45$\n\n$\\begin{array}{llllll}45 & 5 & 40 & 4.5 & 40 & 5\\end{array}$"} {"id": "7232", "image": [], "answer": "2 2", "solution": "null", "level": "一年级", "question": "看图填一填。\n\n$\\because \\because \\because \\because \\because$\n\n0000000\n\n足球比篮球少( )个,篮球比足球多( )个。", "options": [], "subject": "算术", "analysis": "2 2"} {"id": "7233", "image": [], "answer": "13 个 16 件 58 副 63 双", "solution": "null", "level": "一年级", "question": "填表。\n\n| 原来有 | $21 \\uparrow$ | 36 件 | 78 副 | |\n| :---: | :---: | :---: | :---: | :---: |\n| 卖了 | 8个 | 20 件 | | 7 双 |\n| 还剩 | | | 20 副 | 56 双 |", "options": [], "subject": "算术", "analysis": "13 个 16 件 58 副 63 双"} {"id": "7234", "image": ["975.jpg"], "answer": "$40+50=90$ (元)", "solution": "null", "level": "一年级", "question": "买一件上衣和一条裤子一共要多少钱?\n\n", "options": [], "subject": "算术", "analysis": "$40+50=90$ (元)"} {"id": "7235", "image": ["976.jpg"], "answer": "$7+38=45$ (辆)", "solution": "null", "level": "一年级", "question": "客车和货车一共有.多少辆?\n\n", "options": [], "subject": "算术", "analysis": "$7+38=45$ (辆)"} {"id": "7236", "image": ["2952.jpg", "2952.jpg"], "answer": "\n\n$\\begin{array}{llll}90 & 28 & 33 & 5\\end{array}$", "solution": "null", "level": "一年级", "question": "$48-40=.30+5=\\quad 62-60=\\quad 50+3=-6+50=$\n\n$95-5=\\quad 7+20=\\quad 24-4=\\quad 46-6=\\quad 2+40=$\n\n$65-5+9=\\quad 6+90-6=\\quad 27-7+8=\\quad 38-8+3=\\quad 5+40-40=$", "options": [], "subject": "算术", "analysis": "\n\n$\\begin{array}{llll}90 & 28 & 33 & 5\\end{array}$"} {"id": "7237", "image": [], "answer": "< > = > > < =", "solution": "null", "level": "一年级", "question": "在 $\\bigcirc$ 里填上 \">\" \"<\" 或 \"=\"\n\n$56 \\bigcirc 65$\n\n$58 \\bigcirc 68$\n\n$55 \\bigcirc 47$\n\n$99+1 \\bigcirc 100$\n\n$41 \\bigcirc 29$\n\n$92 \\bigcirc 69$\n\n$60+9 \\bigcirc 99-9 \\quad-3+50 \\bigcirc 50+3$", "options": [], "subject": "算术", "analysis": "< > = > > < ="} {"id": "7238", "image": ["977.jpg"], "answer": "多得多", "solution": "null", "level": "一年级", "question": "\n\n一大捆小棒比一小捆小棒:\n多一些少一些多得多少得多", "options": [], "subject": "算术", "analysis": "多得多"} {"id": "7239", "image": ["978.jpg"], "answer": "75-元", "solution": "null", "level": "一年级", "question": "汽车的价钱比小飞机便宜一些,小汽车可能是多少元?\n\n$$\n\\begin{array}{l|l|}\n75 \\text { 元 } & 85 \\text { 元 } \\\\\n50 \\text { 元 }\n\\end{array}\n$$", "options": [], "subject": "算术", "analysis": "75-元"} {"id": "7240", "image": [], "answer": "47 元", "solution": "null", "level": "一年级", "question": "一个书包的价格接近 50 元,可能是多少元?\n$$\n\\begin{array}{l|l|l|}\n47 & \\text { 元 } & 62 \\text { 元 } \\\\\n38 & \\text { 元 }\n\\end{array}\n$$", "options": [], "subject": "算术", "analysis": "47 元"} {"id": "7241", "image": [], "answer": "5-5=30$ (岁) $", "solution": "null", "level": "一年级", "question": "妈今年 35 岁,小可今年 5 岁,妈妈比小可大多少岁?", "options": [], "subject": "算术", "analysis": "5-5=30$ (岁) $"} {"id": "7242", "image": ["979.jpg"], "answer": "0+6=26 (名)", "solution": "null", "level": "一年级", "question": "(1)班男生比女生多 6 人,一(1)班有多少名男生?\n\n", "options": [], "subject": "算术", "analysis": "0+6=26 (名)"} {"id": "7243", "image": ["980.jpg"], "answer": "9+40=49$ (个) $\\quad 50>49$ 装得下。", "solution": "null", "level": "一年级", "question": "袋子能装得下吗?\n\n", "options": [], "subject": "算术", "analysis": "9+40=49$ (个) $\\quad 50>49$ 装得下。"} {"id": "7244", "image": ["981.jpg"], "answer": "86 里面有 8 个十和 6 个一,所以能借给 8 个班。", "solution": "null", "level": "一年级", "question": "借书。\n\n", "options": [], "subject": "算术", "analysis": "86 里面有 8 个十和 6 个一,所以能借给 8 个班。"} {"id": "7251", "image": ["983.jpg"], "answer": "1 二十一", "solution": "null", "level": "一年级", "question": "\n\n写作:\n\n读作:", "options": [], "subject": "算术", "analysis": "1 二十一"} {"id": "7253", "image": [], "answer": "元角 分", "solution": "null", "level": "一年级", "question": "民币的单位有( $\\quad) 、(\\quad) 、(\\quad)$ 。", "options": [], "subject": "算术", "analysis": "元角 分"} {"id": "7254", "image": ["985.jpg", "986.jpg", "987.jpg"], "answer": "(1) $12+8=20$ (元) $\\quad(2) 16-5=11$ (元)\n\n(3)20-8=12(元) 小船", "solution": "null", "level": "一年级", "question": "\n\n16 元\n\n\n\n12 元\n\n(1)买一条\n\n\n\n8 元\n\n5 元\n\n$(2)$ 一把", "options": [], "subject": "算术", "analysis": "(1) $12+8=20$ (元) $\\quad(2) 16-5=11$ (元)\n\n(3)20-8=12(元) 小船"} {"id": "7255", "image": ["988.jpg", "989.jpg", "990.jpg", "991.jpg", "992.jpg"], "answer": "1) $4+9=13$ (元) $\\quad(2) 36-6=30$ (元)\n\n(3) $7+3=10$ (元) $\\quad(4) 50+10=60$ (元)\n\n(5)魔方和玩具兔或者七巧板和玩具小熊", "solution": "null", "level": "一年级", "question": "\n\n6 元\n\n\n\n51 元\n\n\n\n7 元\n\n\n\n4 元\n\n\n\n9 元\n\n(1)买一只玩具小熊和一副七巧板,一共要多少钱?\n\n(2)一个布娃娃比一个魔方贵多少钱?\n\n(3)小军买了一只玩具兔,营..业员找回 3 元,他付了多少钱?\n\n(4)买一架玩具飞机和一只玩具兔,大约要带几十元?\n\n(5)丽丽用 13 元正好买了两种玩具,她可能买了哪两种玩具?", "options": [], "subject": "算术", "analysis": "1) $4+9=13$ (元) $\\quad(2) 36-6=30$ (元)\n\n(3) $7+3=10$ (元) $\\quad(4) 50+10=60$ (元)\n\n(5)魔方和玩具兔或者七巧板和玩具小熊"} {"id": "7256", "image": [], "answer": "2 30 70 6 54 36 1 5 2 4", "solution": "null", "level": "一年级", "question": "0 分=( )角 $\\quad 3$ 元=( )角 $\\quad 7$ 角 $=(\\quad)$ 分\n\n60 角 $=($ )元 5 角 4 分 $=(\\quad)$ 分 3 元6角=( $)$ 角\n\n15 角 $=($ )元( )角 24 分 $=(\\quad)$ 角 $(\\quad)$ 分", "options": [], "subject": "算术", "analysis": "2 30 70 6 54 36 1 5 2 4"} {"id": "7257", "image": ["2953.jpg"], "answer": "(1)5 (2)1 3 或 21", "solution": "null", "level": "一年级", "question": "", "options": [], "subject": "算术", "analysis": ":分析:(1)由点 $\\mathrm{A}$ 在抛物线上, 将 $\\mathrm{A}$ 点坐标代入, 求出参数 $\\mathrm{P}$, 求解即可 (2) 由于 $F(8,0)$ 是 $\\triangle A B C$ 的重心, 则重心与焦点重合, 由重心坐标公式可求 $M$ 是 $B C$ 的中点。\n\n(3) 由于线段 $\\mathrm{BC}$ 的中点 $\\mathrm{M}$ 不在 $x$ 轴上, 所以 $\\mathrm{BC}$ 所在的直线不垂直于 $x$ 轴. 设 $\\mathrm{BC}$ 所在直线的方程为: $y+4=k(x-11)(k \\neq 0)$, 解出 $k$ 即可。"} {"id": "7258", "image": [], "answer": "角 元 元", "solution": "null", "level": "一年级", "question": "括号里填上合适的单位。\n\n一支铅笔 5() 一瓶可乐 3(.) 一盒巧克力 $36($ )", "options": [], "subject": "算术", "analysis": "角 元 元"} {"id": "7259", "image": [], "answer": "5.75", "solution": "null", "level": "一年级", "question": "张 5 元、一张 2 元、一张 5 角,一共是( )元( )角。", "options": [], "subject": "算术", "analysis": "5.75"} {"id": "7260", "image": ["993.jpg", "994.jpg", "995.jpg"], "answer": "[](本地图片-1421.jpg)", "solution": "null", "level": "一年级", "question": "\n\n$(\\quad)$ 元\n\n\n\n( ) 元 $(\\quad)$ 角\n\n\n\n$(\\quad)$ 元 $(\\quad)$ 角 $(\\quad)$ 分", "options": [], "subject": "算术", "analysis": "[](本地图片-1421.jpg)"} {"id": "7261", "image": [], "answer": "7 5 530", "solution": "null", "level": "一年级", "question": "| 要买的物品 | 3角 | $\\frac{1 \\text { 元5角 }}{}$ | 20元 |\n| :---: | :---: | :---: | :---: |\n| 付出的钱 | 18 | $=0$ | 50,9 |\n| 应找回的钱 | ()角 | ()角 | ( $\\quad$ )元 |", "options": [], "subject": "算术", "analysis": "7 5 530"} {"id": "7262", "image": ["996.jpg"], "answer": "0-8 2 元", "solution": "null", "level": "一年级", "question": "小勇买了一个文具盒,付出 10 元,应该找回多少元?\n ", "options": [], "subject": "算术", "analysis": "0-8 2 元"} {"id": "6684", "image": [], "answer": "$78 ; 80$", "solution": "null", "level": "一年级", "question": "与 79 相邻的两个数是 \\$ \\qquad \\$和 \\$ \\qquad \\$。", "options": [], "subject": "算术", "analysis": "$78 ; 80$"} {"id": "6685", "image": [], "answer": "十; 7 个十;个; 8 个一", "solution": "null", "level": "一年级", "question": "78 中的 “ 7 \" 在 \\$ \\qquad \\$位上,表示 \\$ \\qquad \\$ , “ 8 \" 在 \\$ \\qquad \\$位\n\n上, 表示 \\$ \\qquad \\$", "options": [], "subject": "算术", "analysis": "十; 7 个十;个; 8 个一"} {"id": "6686", "image": [], "answer": "57", "solution": "null", "level": "一年级", "question": "一个两位数, 个位上的数是 7 , 十位上的数是 5 , 这个数是 \\$ \\qquad \\$", "options": [], "subject": "算术", "analysis": "57"} {"id": "6687", "image": [], "answer": "$45 ; 70$", "solution": "null", "level": "一年级", "question": "43 后面的第二个数是 \\$ \\qquad \\$ , 71 前面的第个数是 \\$ \\qquad \\$", "options": [], "subject": "算术", "analysis": "$45 ; 70$"} {"id": "6688", "image": ["655.jpg"], "answer": "六十三; 6; 3;7", "solution": "null", "level": "一年级", "question": "如图, 计数器表示的数读作 \\$ \\qquad \\$ , 它是由 \\$ \\qquad \\$个十和个一组成的,再添 \\$ \\qquad \\$个一是 70 。\n\n", "options": [], "subject": "算术", "analysis": "六十三; 6; 3;7"} {"id": "6689", "image": [], "answer": "$3 ; 8 ; 56$", "solution": "null", "level": "一年级", "question": "38 里面有 \\$ \\qquad \\$个十和 \\$ \\qquad \\$个一; 6 个一和 5 个十组成的数是 \\$ \\qquad \\$ -", "options": [], "subject": "算术", "analysis": "$3 ; 8 ; 56$"} {"id": "6690", "image": [], "answer": "$100 ; 10$", "solution": "null", "level": "一年级", "question": "100 表示 \\$ \\qquad \\$个一, 也可以表示个十。", "options": [], "subject": "算术", "analysis": "$100 ; 10$"} {"id": "6691", "image": [], "answer": "十", "solution": "null", "level": "一年级", "question": "88 是一个两位数, \\$ \\qquad \\$位上的 8 表示 8 个十。", "options": [], "subject": "算术", "analysis": "十"} {"id": "6692", "image": [], "answer": "95;九十五", "solution": "null", "level": "一年级", "question": "有一个两位数, 十位上的数是最大的一位数, 个位上的数是 5 , 这个数是 \\$ \\qquad \\$ , 读作: \\$ \\qquad \\$ .", "options": [], "subject": "算术", "analysis": "95;九十五"} {"id": "6694", "image": [], "answer": "59", "solution": "null", "level": "一年级", "question": "兰兰家的门牌号码比 57 大比 60 小, 并且是一个单数, 兰兰家的门牌号码是 \\$ \\qquad \\$ .", "options": [], "subject": "算术", "analysis": "59"} {"id": "6695", "image": [], "answer": "90", "solution": "null", "level": "一年级", "question": "用 9 个在数位表中表示一个两位数, 最大的数是 \\$ \\qquad \\$ -\n\n+位 个位", "options": [], "subject": "算术", "analysis": "90"} {"id": "6696", "image": [], "answer": "29; 60; 100", "solution": "null", "level": "一年级", "question": "一个一个地数, 30 前面的一个数是 \\$ \\qquad \\$ ; 五个五个地数, 55 后面的一个数是 \\$ \\qquad \\$ ; 十个十个地数, 10 个十是", "options": [], "subject": "算术", "analysis": "29; 60; 100"} {"id": "6697", "image": ["656.jpg", "657.jpg", "657.jpg"], "answer": "", "solution": "null", "level": "一年级", "question": "(6 分)\n\n90\n\n99\n\n(35]\n\n[25\n\n56\n\n66\n", "options": [], "subject": "算术", "analysis": ""} {"id": "6698", "image": ["658.jpg", "659.jpg", "660.jpg", "661.jpg", "662.jpg", "662.jpg"], "answer": "", "solution": "null", "level": "一年级", "question": "(4 分) 连一连, 帮小青蛙回家。\n\n\n\n52\n\n\n\n52\n\n63\n\n\n\n100\n\n\n\n84", "options": [], "subject": "算术", "analysis": ""} {"id": "6701", "image": [], "answer": "解: $20+9=29$ (只)\n\n答:树上原来有 29 只鸟。", "solution": "null", "level": "一年级", "question": "(6 分)树上小鸟飞走了 20 只, 还剩 9 只, 树上原来有多少只小鸟?", "options": [], "subject": "算术", "analysis": "解: $20+9=29$ (只)\n\n答:树上原来有 29 只鸟。"} {"id": "6702", "image": [], "answer": "解: $20+8=28$ (面)\n\n答:现在一共有 28 面。", "solution": "null", "level": "一年级", "question": "(6 分) 学校有 20 面彩旗, 又买来 8 面, 现在一共有多少面?", "options": [], "subject": "算术", "analysis": "解: $20+8=28$ (面)\n\n答:现在一共有 28 面。"} {"id": "6703", "image": [], "answer": "解: $50+3=53$ (瓶) , 不够\n\n答:每人一瓶矿泉水, 50 瓶不够。", "solution": "null", "level": "一年级", "question": "(6 分) 有 50 个学生, 3 个老师, 每人一瓶矿泉水, 50 瓶够吗?", "options": [], "subject": "算术", "analysis": "解: $50+3=53$ (瓶) , 不够\n\n答:每人一瓶矿泉水, 50 瓶不够。"} {"id": "6705", "image": [], "answer": "解: $20+8=28$ (盆) 答:两个小组一共种了 28 盆。", "solution": "null", "level": "一年级", "question": "(6 分) 同学们在花坛里种花, 第一小组种了 20 盆, 第二小组种了 8 盆, 两个小组一共种了多少盆?", "options": [], "subject": "算术", "analysis": "解: $20+8=28$ (盆) 答:两个小组一共种了 28 盆。"} {"id": "6706", "image": [], "answer": "解: $40+3=43$, 答: 一共有 45 瓶水, 大于 43 人, 所以是足够的", "solution": "null", "level": "一年级", "question": "(6 分) 3 名老师和 40 名同学去秋游, 请问他们一共带 45 瓶水够吗?", "options": [], "subject": "算术", "analysis": "解: $40+3=43$, 答: 一共有 45 瓶水, 大于 43 人, 所以是足够的"} {"id": "6715", "image": [], "answer": "12", "solution": "null", "level": "一年级", "question": "一个\n\n的价钱是 8 元,小明买了 1 个,付给营业员一张元。", "options": [], "subject": "算术", "analysis": "12"} {"id": "6716", "image": [], "answer": "$7 ; 100 ; 52 ; 2 ; 7$", "solution": "null", "level": "一年级", "question": "70 角 $=$ \\$ \\qquad \\$元\n\n5 元 2 角 $=$ \\$ \\qquad \\$角\n\n27 角 $=$ \\$ \\qquad \\$元 \\$ \\qquad \\$角", "options": [], "subject": "算术", "analysis": "$7 ; 100 ; 52 ; 2 ; 7$"} {"id": "6717", "image": [], "answer": "$5 ; 10$", "solution": "null", "level": "一年级", "question": "一张 50 元可以换 \\$ \\qquad \\$张 10 元,也可以换 \\$ \\qquad \\$张 5 元。", "options": [], "subject": "算术", "analysis": "$5 ; 10$"} {"id": "6359", "image": ["426.jpg"], "answer": "; 2; 0; 2", "solution": "null", "level": "一年级", "question": "一数有几朵花?你发现了什么?\n\n\n\n朵朵朵\n\n每次少朵", "options": [], "subject": "计数", "analysis": "; 2; 0; 2"} {"id": "6365", "image": ["429.jpg", "430.jpg", "431.jpg", "430.jpg", "431.jpg"], "answer": "图:\n\n\n\n4\n\n", "solution": "null", "level": "一年级", "question": "据数字给下列图形涂上颜色。\n", "options": [], "subject": "计数", "analysis": "图:\n\n\n\n4\n\n"} {"id": "5866", "image": ["2945.jpg"], "answer": "详解$11+6=17$ (条)", "solution": "null", "level": "一年级", "question": "\n\n一共有多少条鱼?", "options": [], "subject": "计数", "analysis": "详解$11+6=17$ (条)"} {"id": "6078", "image": ["181.jpg"], "answer": "$5+3=8$", "solution": "null", "level": "一年级", "question": "(4 分)一共有几只?\n\n", "options": [], "subject": "计数", "analysis": "$5+3=8$"} {"id": "6639", "image": ["629.jpg", "630.jpg", "631.jpg", "631.jpg"], "answer": "详解(1) $6 ; 7 ; 4 ; 4$\n\n\n\n(3) $\\bigcirc ; 3$\n\n(4) 6", "solution": "null", "level": "一年级", "question": "(11 分)\n\n(1) 数一数, 填一填。\n\n| 图形 | $\\square$ | $\\square$ | $\\square$ | $\\square$ |\n| :--- | :--- | :--- | :--- | :--- |\n| 个数 | | | | $\\square$ |\n\n(2)(4 分)按形状分一分,涂一涂。\n\n\n\n(3)\n\n图形最多,\n\n$\\bigcirc$ 比 $\\square$ 多个。\n\n(4)乐乐摆了 3 个 ,一共用了\n\n", "options": [], "subject": "计数", "analysis": "详解(1) $6 ; 7 ; 4 ; 4$\n\n\n\n(3) $\\bigcirc ; 3$\n\n(4) 6"} {"id": "6640", "image": ["632.jpg"], "answer": "详解(1) $2 ; 9 ; 4 ; 3$\n\n(2) $\\square ; \\square$\n\n(3) 5", "solution": "null", "level": "一年级", "question": "(7 分)\n\n\n\n(1) 填一填。\n\n| 图形 | $\\square$ | $\\square$ | $\\triangle$ | $\\bigcirc$ |\n| :--- | :--- | :--- | :--- | :--- |\n| 个数 | | | | |\n\n(2) 的个数最多,的个数最少。\n\n(3) $\\square$ 比 $\\triangle$ 多 \\$ \\qquad \\$个。", "options": [], "subject": "计数", "analysis": "详解(1) $2 ; 9 ; 4 ; 3$\n\n(2) $\\square ; \\square$\n\n(3) 5"} {"id": "6642", "image": ["633.jpg", "634.jpg", "635.jpg", "635.jpg"], "answer": "详解(1)\n\n\n\n(2) $8 ; 7 ; 7 ; 6$\n\n(3) $\\triangle ; O$;", "solution": "null", "level": "一年级", "question": "(13 分) 数一数, 涂一涂, 填一填。\n\n\n\n(1)\n\n\n\n(2)填空。\n\n| | $\\triangle$ | $\\square$ | $\\square$ | $\\bigcirc$ |\n| :--- | :--- | :--- | :--- | :--- |\n| 个数 | | | | |\n\n(3)的数量最多,的数量最少。和\n\n数量同样多。", "options": [], "subject": "计数", "analysis": "详解(1)\n\n\n\n(2) $8 ; 7 ; 7 ; 6$\n\n(3) $\\triangle ; O$;"} {"id": "6643", "image": ["636.jpg", "637.jpg"], "answer": "详解(1) $12 ; 3 ; 15 ; 3 ; 33$\n\n(2) $\\bigcirc ; \\square ; \\triangle$\n\n(3) $9 ; 3$", "solution": "null", "level": "一年级", "question": "(10 分)\n\n\n(1) 你能把各种图形整理一下吗?\n\n| 图形 | $\\square$ | $\\square$ | $\\bigcirc$ | $\\triangle$ | 合计 |\n| :---: | :---: | :---: | :---: | :---: | :---: |\n| 个数 | | | | | |\n\n(2)最多和同样多。\n\n(3)\n\n\n多个, 比 $\\bigcirc$ 少个。", "options": [], "subject": "计数", "analysis": "详解(1) $12 ; 3 ; 15 ; 3 ; 33$\n\n(2) $\\bigcirc ; \\square ; \\triangle$\n\n(3) $9 ; 3$"} {"id": "6644", "image": ["638.jpg", "639.jpg"], "answer": "详解(1) $1 ; 10 ; 4 ; 10 ; 8$\n\n(2) 正方形\n\n(3) 长方形; 圆形\n\n(4) 33\n\n(5) 6", "solution": "null", "level": "一年级", "question": "(10 分) 先填表, 再回答问题。\n\n\n\n(1)\n\n\n\n(2)\n\n的数量最少。\n\n(3)和的个数同样多。\n\n(4)一共有 \\$ \\qquad \\$个图形。\n\n(5) 摆 3 个\n要用个 $\\bigcirc$ 。", "options": [], "subject": "计数", "analysis": "详解(1) $1 ; 10 ; 4 ; 10 ; 8$\n\n(2) 正方形\n\n(3) 长方形; 圆形\n\n(4) 33\n\n(5) 6"} {"id": "7227", "image": ["972.jpg"], "answer": "4 串", "solution": "null", "level": "一年级", "question": "最多可以穿多少串?\n\n", "options": [], "subject": "计数", "analysis": "4 串"} {"id": "6700", "image": ["664.jpg"], "answer": "$20+3=23$ (个)", "solution": "null", "level": "一年级", "question": "(5 分) 看图列式。\n\n$20 \\uparrow$\n\n\n\n$\\square \\mathrm{O} \\square \\square$ (个)", "options": [], "subject": "计数", "analysis": "$20+3=23$ (个)"} {"id": "6728", "image": ["665.jpg", "666.jpg", "667.jpg", "667.jpg"], "answer": "", "solution": "null", "level": "一年级", "question": "(4 分)我会连。\n\n\n\n| 20 元 |\n| :--- |\n| 5角 |\n\n\n| 5元 | 20 元 | 20 元 |\n| :---: | :---: | :---: |\n| 1元 | 5元 | |\n\n", "options": [], "subject": "计数", "analysis": ""} {"id": "6791", "image": ["696.jpg", "697.jpg", "697.jpg"], "answer": "", "solution": "null", "level": "一年级", "question": "(6 分)给小蚂蚁编号。\n ", "options": [], "subject": "计数", "analysis": ""} {"id": "6093", "image": ["185.jpg"], "answer": "$10 ; 4 ; 8 ; 3 ; 3 ; 6$", "solution": "null", "level": "一年级", "question": "\n\n一共有 \\$ \\qquad \\$只小动物, 排在第 \\$ \\qquad \\$ ,排在\n\n第 \\$ \\qquad \\$ 和中间有 \\$ \\qquad \\$只小动物,的前面有\n\n只小动物,的后面有 \\$ \\qquad \\$只小动物。", "options": [], "subject": "计数", "analysis": "$10 ; 4 ; 8 ; 3 ; 3 ; 6$"} {"id": "6094", "image": ["186.jpg"], "answer": "$2 ; 2$", "solution": "null", "level": "一年级", "question": "比一比, 填一填。\n\n\n\n$\\triangle \\triangle \\triangle \\triangle \\triangle$\n\n$\\boldsymbol{\\square}$ 比 $\\triangle$ 多 \\$ \\qquad \\$个, $\\triangle$ 比■少 \\$ \\qquad \\$个。", "options": [], "subject": "计数", "analysis": "$2 ; 2$"} {"id": "6360", "image": [], "answer": "、8; 1、3、4、5、6、7", "solution": "null", "level": "一年级", "question": "把下列物体分成两类, 填写像一个序号\n₪\n$\\square$\nघg\n$\\mathrm{P}_{\\theta} 0$\n123\n4\n5\n8\n\n平面图形: \\$ \\qquad \\$\n立体图形: \\$ \\qquad \\$", "options": [], "subject": "组合几何学", "analysis": "、8; 1、3、4、5、6、7"} {"id": "6371", "image": ["438.jpg", "438.jpg"], "answer": "图:\n\n", "solution": "null", "level": "一年级", "question": "一涂, 比一比。\n$\\triangle \\Delta \\Delta$\n0 和 $\\Delta$ 群多 00000\n○比 $\\triangle$ 多\n00000\n$3=3$\n南比 $\\Delta$ 少出出出出方\n$\\square 0 \\square$", "options": [], "subject": "组合几何学", "analysis": "图:\n\n"} {"id": "6372", "image": ["439.jpg", "440.jpg", "440.jpg"], "answer": "图所示, 对大多数人来说, 用来写字的手是右手, 另一只手就是左手. 与右手相对应的一面是右, 与左手相对应的一面是左. 左; 右手有习惯性分工, 左、右手是好朋友。\n\n", "solution": "null", "level": "一年级", "question": "只小手可以做什么?\n\n", "options": [], "subject": "组合几何学", "analysis": "图所示, 对大多数人来说, 用来写字的手是右手, 另一只手就是左手. 与右手相对应的一面是右, 与左手相对应的一面是左. 左; 右手有习惯性分工, 左、右手是好朋友。\n\n"} {"id": "6794", "image": ["700.jpg", "701.jpg", "702.jpg", "702.jpg"], "answer": "[](本地图片-873.jpg)\n\n", "solution": "null", "level": "一年级", "question": "(8 分) 按要求, 画一画。\n\n(1)(4 分)左边可以画几个 $\\mathrm{O}$ ? 右边呢?\n\n\n\n(2) (4 分) 找规律,接着再画六个图形。\n\n", "options": [], "subject": "组合几何学", "analysis": "[](本地图片-873.jpg)\n\n"} {"id": "6101", "image": ["197.jpg", "198.jpg", "199.jpg", "200.jpg", "201.jpg", "200.jpg", "201.jpg"], "answer": "(1)\n\n\n(○) (0) (0) (0) (0) (0)\n\n20 2o of ofo of\n\nC\n\n(2)\n\n", "solution": "null", "level": "一年级", "question": "(5 分)比一比, 最多的画 “ $\\sqrt{ }$ ”, 最少的画 “ $\\bigcirc$ ”。\n\n\n(1) (2 分)\n$\\varangle \\varangle$\n(\n$\\square$\n\n\n\n(2)(3 分)\n\nあのののあの ().\n\n", "options": [], "subject": "组合几何学", "analysis": "(1)\n\n\n(○) (0) (0) (0) (0) (0)\n\n20 2o of ofo of\n\nC\n\n(2)\n\n"} {"id": "6118", "image": ["217.jpg"], "answer": "5", "solution": "null", "level": "一年级", "question": "\n\n有\n\n \\$ \\qquad \\$只蝴蝶。", "options": [], "subject": "组合几何学", "analysis": "5"} {"id": "6144", "image": ["230.jpg"], "answer": "$2 ; 3 ; 1 ; 2$", "solution": "null", "level": "一年级", "question": "(4 分)\n\n\n(1) 有 \\$ \\qquad \\$个,\n$\\square$ 有 \\$ \\qquad \\$个, $\\bigcirc$ 有个,\n$\\square$ 有 \\$ \\qquad \\$个。", "options": [], "subject": "组合几何学", "analysis": "$2 ; 3 ; 1 ; 2$"} {"id": "6146", "image": [], "answer": "4", "solution": "null", "level": "一年级", "question": "ता।左图是由 \\$ \\qquad \\$个\n\n拼成的。", "options": [], "subject": "组合几何学", "analysis": "4"} {"id": "6367", "image": ["434.jpg"], "answer": "间房子住 1 只, 1 间房子住 2 只, 1 间房子住 3 只, 1 间房子住 4 只", "solution": "null", "level": "一年级", "question": "妈妈生了 10 只可爱的小狗, 这 10 只小狗分别住在 4 间房子里, 请问怎样分房子,才能使每间房子里的小狗数量都不一样多.\n\n", "options": [], "subject": "组合数学", "analysis": "间房子住 1 只, 1 间房子住 2 只, 1 间房子住 3 只, 1 间房子住 4 只"} {"id": "7252", "image": ["984.jpg"], "answer": "29 二十九", "solution": "null", "level": "一年级", "question": "\n\n写作:\n\n读作:", "options": [], "subject": "组合数学", "analysis": "1 二十一"} {"id": "6727", "image": [], "answer": "16", "solution": "null", "level": "一年级", "question": "小红有 1 元和 5 元的纸币共 4 张, 她最多有 \\$ \\qquad \\$元。", "options": [], "subject": "组合数学", "analysis": "16"} {"id": "6786", "image": ["687.jpg", "688.jpg", "688.jpg"], "answer": "\n\n$8001-10000^{\\circ}$", "solution": "null", "level": "一年级", "question": "(4 分)按规律在下图的口里画一画。\n\n", "options": [], "subject": "组合数学", "analysis": "\n\n$8001-10000^{\\circ}$"} {"id": "6090", "image": ["184.jpg"], "answer": "$3 ; 4 ; 6$", "solution": "null", "level": "一年级", "question": "[](本地图片-9.jpg)\n个, $\\triangle$个, 於个。\n\n", "options": [], "subject": "组合数学", "analysis": "$3 ; 4 ; 6$"} {"id": "6103", "image": ["202.jpg", "203.jpg", "204.jpg", "203.jpg", "204.jpg"], "answer": "\n\n$\\square$\n\n[\n\n24 ._详解\n\n\n\n$\\triangle \\Delta \\Delta \\triangle \\Delta$", "solution": "null", "level": "一年级", "question": "(5 分)蘑菇最多的画 “ل”。\n", "options": [], "subject": "组合数学", "analysis": "\n\n$\\square$\n\n[\n\n24 ._详解\n\n\n\n$\\triangle \\Delta \\Delta \\triangle \\Delta$"} {"id": "5891", "image": ["29.jpg", "30.jpg", "30.jpg"], "answer": "\n\n$\\triangle \\Delta \\Delta \\Delta \\Delta$", "solution": "null", "level": "一年级", "question": "", "options": [], "subject": "组合数学", "analysis": ":分析:(1)由点 $\\mathrm{A}$ 在抛物线上, 将 $\\mathrm{A}$ 点坐标代入, 求出参数 $\\mathrm{P}$, 求解即可 (2) 由于 $F(8,0)$ 是 $\\triangle A B C$ 的重心, 则重心与焦点重合, 由重心坐标公式可求 $M$ 是 $B C$ 的中点。\n\n(3) 由于线段 $\\mathrm{BC}$ 的中点 $\\mathrm{M}$ 不在 $x$ 轴上, 所以 $\\mathrm{BC}$ 所在的直线不垂直于 $x$ 轴. 设 $\\mathrm{BC}$ 所在直线的方程为: $y+4=k(x-11)(k \\neq 0)$, 解出 $k$ 即可。"} {"id": "6104", "image": ["205.jpg", "206.jpg", "206.jpg"], "answer": "", "solution": "null", "level": "一年级", "question": "(5 分) 从左边数, 第 3 条毛巾涂上你喜欢的颜色; 把最右边的 3 条毛巾圈起来。\n\n", "options": [], "subject": "组合数学", "analysis": ""} {"id": "6117", "image": ["216.jpg"], "answer": "4", "solution": "null", "level": "一年级", "question": "\n\n有个圆圈。", "options": [], "subject": "组合数学", "analysis": "4"} {"id": "6130", "image": [], "answer": "$\\because \\because \\because(\\because$", "solution": "null", "level": "一年级", "question": "画 (), (-) 比 $\\bigcirc$ 多 2 个。", "options": [], "subject": "组合数学", "analysis": "$\\because \\because \\because(\\because$"} {"id": "6370", "image": ["437.jpg"], "answer": "在小船的上面, 小船在桥的下面, 桥在小女孩的前面。", "solution": "null", "level": "一年级", "question": "能用上下说一说他们的位置关系吗?\n\n", "options": [], "subject": "度量几何学", "analysis": "在小船的上面, 小船在桥的下面, 桥在小女孩的前面。"} {"id": "6385", "image": [], "answer": ";南;西", "solution": "null", "level": "一年级", "question": "面向东, 我的左面是 \\$ \\qquad \\$ ,右面是 \\$ \\qquad \\$ ,后面是", "options": [], "subject": "逻辑题", "analysis": ";南;西"} {"id": "6699", "image": ["663.jpg"], "answer": "$30+6=36$\n\n$36-30=6$\n\n$36-6=30$", "solution": "null", "level": "一年级", "question": "(9 分)\n\n", "options": [], "subject": "逻辑题", "analysis": "$30+6=36$\n\n$36-30=6$\n\n$36-6=30$"} {"id": "6793", "image": [], "answer": "第二行", "solution": "null", "level": "一年级", "question": "(4 分)哪一行和其他三行的规律不同? 在那一行的后面打上 “、”。\n\n| 20 | 21 | 22 | 23 | 24 | 25 | |\n| :--- | :--- | :--- | :--- | :--- | :--- | :--- |\n| 35 | 37 | 39 | 41 | 43 | 45 | |\n\n\n| 60 | 61 | 62 | 63 | 64 | 65 | |\n| :--- | :--- | :--- | :--- | :--- | :--- | :--- |\n| 52 | 53 | 54 | 55 | 56 | 57 | |", "options": [], "subject": "逻辑题", "analysis": "第二行"} {"id": "6092", "image": [], "answer": "右", "solution": "null", "level": "一年级", "question": "为了让我们的生活更有秩序, 我们上下楼梯都要靠边行。", "options": [], "subject": "逻辑题", "analysis": "右"} {"id": "6023", "image": ["131.jpg"], "answer": "5 时(5:00)", "solution": "null", "level": "一年级", "question": "小云到家的时间是 \\$ \\qquad \\$。\n", "options": [], "subject": "逻辑题", "analysis": "5 时(5:00)"} {"id": "6637", "image": ["625.jpg", "626.jpg", "626.jpg"], "answer": "解\n", "solution": "null", "level": "一年级", "question": "(4 分) 给 0 涂上颜色。\n", "options": [], "subject": "画法几何学", "analysis": "解\n"} {"id": "6638", "image": ["627.jpg", "628.jpg", "628.jpg"], "answer": "解", "solution": "null", "level": "一年级", "question": "(6 分)在下面的方格纸上分别画出一个长方形、一个正方形和一个三角形。\n\n", "options": [], "subject": "画法几何学", "analysis": "解"} {"id": "6784", "image": ["685.jpg", "686.jpg", "686.jpg"], "answer": "", "solution": "null", "level": "一年级", "question": "(4 分)根据规律画出被挡住部分的图形。\n\n", "options": [], "subject": "画法几何学", "analysis": ""} {"id": "6102", "image": [], "answer": "\\triangle \\triangle \\triangle$\n\nOOOOOOO", "solution": "null", "level": "一年级", "question": "(5 分) 画 $\\mathrm{O}$, 比 $\\triangle$ 多 3 个.\n\n$\\triangle M A$", "options": [], "subject": "画法几何学", "analysis": "\\triangle \\triangle \\triangle$\n\nOOOOOOO"} {"id": "6150", "image": ["240.jpg", "241.jpg", "242.jpg", "243.jpg", "244.jpg", "245.jpg", "246.jpg", "247.jpg", "245.jpg", "246.jpg", "247.jpg"], "answer": "(1)\n\n\n(2)\n\n\n(3)\n", "solution": "null", "level": "一年级", "question": "(12 分) 把不一样的图形圈起来。\n(1)\n\n\n\n$\\square$\n\n\n\n(2)\n\n\n$(3)$\n\n\n20._把能站稳的那一组圈起来。\n", "options": [], "subject": "画法几何学", "analysis": "(1)\n\n\n(2)\n\n\n(3)\n"} {"id": "6154", "image": ["254.jpg", "255.jpg", "255.jpg"], "answer": "", "solution": "null", "level": "一年级", "question": "(6 分)下面哪些组合能站稳?请画上 “、”。\n\n\n$(\\quad)$", "options": [], "subject": "画法几何学", "analysis": ""} {"id": "6783", "image": ["684.jpg"], "answer": "$(1+)($ Nin)", "solution": "null", "level": "一年级", "question": "(4 分) 根据钟面的变化, 画出第 4 个钟面上的时针和分针。\n", "options": [], "subject": "变换几何", "analysis": "$(1+)($ Nin)"} {"id": "6022", "image": [], "answer": "6", "solution": "null", "level": "一年级", "question": "当分针指向 12 , 时针指向 \\$ \\qquad \\$的时候, 是 6 时。", "options": [], "subject": "解析几何", "analysis": "6"} {"id": "6808", "image": [], "answer": "(3)", "solution": "null", "level": "一年级", "question": "下面()组小棒可以拼成一个长方形。\n (1) $\\rightleftarrows$\n (2) $\\rightleftarrows$\n (3) $\\rightleftarrows$", "options": [], "subject": "图论", "analysis": "(3)"} {"id": "6412", "image": ["476.jpg", "477.jpg", "477.jpg"], "answer": "如图:\n\n", "solution": "null", "level": "一年级", "question": "图的下面画 $\\triangle$, 数量要和上图同样多。\n\n", "options": [], "subject": "图论", "analysis": "如图:\n\n"} {"id": "6432", "image": ["493.jpg"], "answer": "6 ; 3 ; 4 ; 2 ; 5$", "solution": "null", "level": "一年级", "question": "数一数, 写一写。\n", "options": [], "subject": "代数", "analysis": "6 ; 3 ; 4 ; 2 ; 5$"} {"id": "9053", "image": [], "answer": "解(1) 2.5 米; 2.2 米\n\n(2)小芳和妈妈做亲子装一共用布料多少米? 6.2 米\n\n(1) 用小芳做上衣用布料长度加上她做裤子用布料长度, 求出她做亲子装用布料长度。用妈妈做亲子装用布料长度减去她做裤子用布料长度, 求出她做上衣用布料长度。\n\n(2) 根据题目给出的条件, 可提出问题: 小芳和妈妈做亲子装一共用布料多少米? 用小芳\n做亲子装用布料长度加上妈妈做亲子装用布料长度解答。\n\n(1) $1.6+0.9=2.5($ 米)\n\n$3.7-1.5=2.2($ 米 $)$\n\n| | 上衣 | 裤子 | 合计 |\n| :---: | :---: | :---: | :---: |\n| 小芳 | 1.6 米 | 0.9 米 | 2.5 米 |\n| 妈妈 | 2.2 米 | 1.5 米 | 3.7 米 |\n\n(2)小芳和妈妈做亲子装一共用布料多少米?\n\n$2.5+3.7=6.2($ 米 $)$\n\n答:小芳和妈妈做亲子装一共用布料 6.2 米。\n\n本题考查小数加减法计算方法的实际应用, 先理清题目中量与量之间的关系, 再列出算式,仔细计算即可。", "solution": "null", "level": "三年级", "question": "下面小芳和妈妈定做亲子装所用的布料长度。\n\n| | 上衣 | 裤子 | 合计 |\n| :---: | :---: | :---: | :---: |\n| 小芳 | 1.6 米 | 0.9 米 | |\n| 妈妈 | | 1.5 米 | 3.7 米 |\n\n(1) 请把表格填完整。\n\n(2) 请提出一个数学问题并解答。", "options": [], "subject": "算术", "analysis": "解(1) 2.5 米; 2.2 米\n\n(2)小芳和妈妈做亲子装一共用布料多少米? 6.2 米\n\n(1) 用小芳做上衣用布料长度加上她做裤子用布料长度, 求出她做亲子装用布料长度。用妈妈做亲子装用布料长度减去她做裤子用布料长度, 求出她做上衣用布料长度。\n\n(2) 根据题目给出的条件, 可提出问题: 小芳和妈妈做亲子装一共用布料多少米? 用小芳\n做亲子装用布料长度加上妈妈做亲子装用布料长度解答。\n\n(1) $1.6+0.9=2.5($ 米)\n\n$3.7-1.5=2.2($ 米 $)$\n\n| | 上衣 | 裤子 | 合计 |\n| :---: | :---: | :---: | :---: |\n| 小芳 | 1.6 米 | 0.9 米 | 2.5 米 |\n| 妈妈 | 2.2 米 | 1.5 米 | 3.7 米 |\n\n(2)小芳和妈妈做亲子装一共用布料多少米?\n\n$2.5+3.7=6.2($ 米 $)$\n\n答:小芳和妈妈做亲子装一共用布料 6.2 米。\n\n本题考查小数加减法计算方法的实际应用, 先理清题目中量与量之间的关系, 再列出算式,仔细计算即可。"} {"id": "9054", "image": ["1459.jpg"], "answer": "解不够\n\n将两种书的单价进行相加, 求得总数, 再和 10 进行比较即可解答。\n\n$6.7+3.5=10.2($ 元 $)$\n\n$10.2>10$\n\n答:她带了 10 元钱,不够。\n\n掌握小数的加法计算方法是解题的关键。", "solution": "null", "level": "三年级", "question": "欢欢想以下两种书各买一本,她带了 10 元钱,够吗?\n\n\n6.70 元\n\n3.50 元", "options": [], "subject": "算术", "analysis": "解不够\n\n将两种书的单价进行相加, 求得总数, 再和 10 进行比较即可解答。\n\n$6.7+3.5=10.2($ 元 $)$\n\n$10.2>10$\n\n答:她带了 10 元钱,不够。\n\n掌握小数的加法计算方法是解题的关键。"} {"id": "9055", "image": [], "answer": "解不够\n\n先用 $4.5+6.2$ 求出一本《童话故事选》和一本《动脑筋》共需要的金额, 再与 10 元比较,\n若大于 10 元, 则不够, 否则够。\n\n$4.5+6.2=10.7($ 元 $)$\n\n$10.7>10$\n\n答: 10 元钱不够。\n\n本题考查了一位小数的加法计算及应用。", "solution": "null", "level": "三年级", "question": "一本《童话故事》 4.5 元, 一本《动脑筋》 6.2 元, 如果各买一本, 10 元钱够不够?", "options": [], "subject": "算术", "analysis": "解不够\n\n先用 $4.5+6.2$ 求出一本《童话故事选》和一本《动脑筋》共需要的金额, 再与 10 元比较,\n若大于 10 元, 则不够, 否则够。\n\n$4.5+6.2=10.7($ 元 $)$\n\n$10.7>10$\n\n答: 10 元钱不够。\n\n本题考查了一位小数的加法计算及应用。"} {"id": "9056", "image": [], "answer": "解7.1 米\n\n根据题意, 用捆绑书籍时用去的米数加上包装杂物时用去的米数, 即可求出这根绳子一共用去了多少米。\n\n$2.8+4.3=7.1$ (米)\n\n答: 这根绳子一共用去了 7.1 米。\n\n要明确, 要求一共用去了多少, 把用去的两部分的长度相加即可。", "solution": "null", "level": "三年级", "question": "有一卷 10 米长的绳子, 㧢绑书籍时用去 2.8 米, 包装杂物时用去 4.3 米。这根绳子一共用去了多少米?", "options": [], "subject": "算术", "analysis": "解7.1 米\n\n根据题意, 用捆绑书籍时用去的米数加上包装杂物时用去的米数, 即可求出这根绳子一共用去了多少米。\n\n$2.8+4.3=7.1$ (米)\n\n答: 这根绳子一共用去了 7.1 米。\n\n要明确, 要求一共用去了多少, 把用去的两部分的长度相加即可。"} {"id": "9057", "image": ["1460.jpg", "1461.jpg", "1462.jpg", "1463.jpg"], "answer": "解1.8 元\n\n将一杯豆浆和一个煎饼的价钱加起来, 再用 10 元减去二者的价钱, 即可得到还剩多少钱。\n\n$10-(4.7+3.5)$\n\n$=10-8.2$\n\n$=1.8($ 元 $)$\n\n答: 还剩 1.8 元。\n\n\n\n本题考查的是小数加减法的实践应用, 关键求出一杯豆浆和一个煎饼的总价钱。", "solution": "null", "level": "三年级", "question": "小明有 10 元, 买了一杯豆浆和一个煎饼。还剩多少钱?\n\n\n\n2.2 元\n\n\n\n6.5 元\n\n\n\n煎垪\n\n4.7 元\n\n\n\n3.5 元", "options": [], "subject": "算术", "analysis": "解1.8 元\n\n将一杯豆浆和一个煎饼的价钱加起来, 再用 10 元减去二者的价钱, 即可得到还剩多少钱。\n\n$10-(4.7+3.5)$\n\n$=10-8.2$\n\n$=1.8($ 元 $)$\n\n答: 还剩 1.8 元。\n\n\n\n本题考查的是小数加减法的实践应用, 关键求出一杯豆浆和一个煎饼的总价钱。"} {"id": "9058", "image": ["1464.jpg", "1465.jpg", "1466.jpg", "1467.jpg", "1468.jpg"], "answer": "解(1) 4.3 元\n\n(2) 钢笔和铅笔\n\n(1) 用买三角板的钱加买铅笔的钱即可;\n\n(2) 用 15.7 减去一个订书器的钱就是剩下的钱, 哪两个玩具的价钱加起来等于或小于剩下的钱即可;\n\n(1) $3.5+0.8=4.3 ($ 元 $)$\n\n答: 买三角板和铅笔一共需要 4.3 元。\n\n(2) $15.7-9.4=6.3($ 元 $)$\n\n$5.5+0.8=6.3($ 元 $)$\n\n答:田晨有 15.7 元,买了一个订书器后,还能买钢笔和铅笔。\n\n熟练掌握小数加减法的计算是解答此题的关键。", "solution": "null", "level": "三年级", "question": "看图回答问题。\n\n\n\n3.5 元\n\n\n\n1.8 元\n\n\n\n5.5 元\n\n\n\n9.4 元\n\n\n\n0.8 元\n\n(1) 买三角板和铅笔一共需要多少钱?\n\n(2) 田晨有 15.7 元, 买了一个订书器后, 还能买哪两个文具?", "options": [], "subject": "算术", "analysis": "解(1) 4.3 元\n\n(2) 钢笔和铅笔\n\n(1) 用买三角板的钱加买铅笔的钱即可;\n\n(2) 用 15.7 减去一个订书器的钱就是剩下的钱, 哪两个玩具的价钱加起来等于或小于剩下的钱即可;\n\n(1) $3.5+0.8=4.3 ($ 元 $)$\n\n答: 买三角板和铅笔一共需要 4.3 元。\n\n(2) $15.7-9.4=6.3($ 元 $)$\n\n$5.5+0.8=6.3($ 元 $)$\n\n答:田晨有 15.7 元,买了一个订书器后,还能买钢笔和铅笔。\n\n熟练掌握小数加减法的计算是解答此题的关键。"} {"id": "9059", "image": ["1469.jpg"], "answer": "解180 元\n\n先计算出一瓶牛奶和一个面包的总价钱, 6 月为小月, 这个月有 30 天, 然后用 30 乘一天需要的钱即可。\n\n$4.8+1.2=6($ 元 $)$\n\n$6 \\times 30=180($ 元 $)$\n\n答: 六月份买牛奶和面包一共要花 180 元。", "solution": "null", "level": "三年级", "question": "妈妈每天为阳阳准备一瓶牛奶和一个面包, 你能应用所学知识帮妈妈算一算, 六月份买牛奶和面包一共要花多少钱?\n\n", "options": [], "subject": "算术", "analysis": "解180 元\n\n先计算出一瓶牛奶和一个面包的总价钱, 6 月为小月, 这个月有 30 天, 然后用 30 乘一天需要的钱即可。\n\n$4.8+1.2=6($ 元 $)$\n\n$6 \\times 30=180($ 元 $)$\n\n答: 六月份买牛奶和面包一共要花 180 元。"} {"id": "9066", "image": [], "answer": "解$1.5 ; 13.7 ; 1.7$\n\n$4.5 ; 0.8 ; 14.3$\n\n\n\n略", "solution": "null", "level": "三年级", "question": "用坚式计算.\n\n$0.7+0.8$\n\n$5+8.7$\n\n$10.4-8.7$\n\n$10-5.5$\n\n$4.1-3.3$\n\n$16.7-2.4$", "options": [], "subject": "算术", "analysis": "解$1.5 ; 13.7 ; 1.7$\n\n$4.5 ; 0.8 ; 14.3$\n\n\n\n略"} {"id": "8380", "image": [], "answer": "$\\times$\n\n【详解】当一个因数为 0 的时候, 积不变。", "solution": "null", "level": "三年级", "question": "一个因数不变,另一个因数扩大 10 倍,积也扩大 10 倍。( $)$", "options": [], "subject": "算术", "analysis": "$\\times$\n\n【详解】当一个因数为 0 的时候, 积不变。"} {"id": "8381", "image": [], "answer": "$\\sqrt{ }$", "solution": "null", "level": "三年级", "question": "计算 $19 \\times 37$ 时, 可以用 $20 \\times 37$ 的积再减去 37 . ( )", "options": [], "subject": "算术", "analysis": "$\\sqrt{ }$"} {"id": "8382", "image": [], "answer": "$\\times$", "solution": "null", "level": "三年级", "question": "最小的两位数和最小的三位数的积是 990 .", "options": [], "subject": "算术", "analysis": "$\\times$"} {"id": "8383", "image": [], "answer": "$\\times$", "solution": "null", "level": "三年级", "question": "80 乘 50 , 得数末尾有两个 0.", "options": [], "subject": "算术", "analysis": "$\\times$"} {"id": "8384", "image": [], "answer": "V", "solution": "null", "level": "三年级", "question": "乘数末尾有几个 0 , 积的末尾不一定就有几个 $0 .(\\quad)$", "options": [], "subject": "算术", "analysis": "V"} {"id": "8388", "image": [], "answer": "924 张\n\n【分析】 21 个班的学生平均每个班有学生 43 人, 根据整数乘法的意义, 共有 $21 \\times 43=903$ 人, 又因每个班有一个带队老师, 则再加上 21 个带队老师, 即是一共要买的电影票张数。\n\n【详解】 $21 \\times 43+21$\n\n$=903+21$\n\n$=924$ (张)\n\n答: 一共要买 924 张电影票。\n\n【点睛】本题主要考查了整数乘法的意义和实际应用, 先求出学生总人数是解题的关键。", "solution": "null", "level": "三年级", "question": "东风小学组织师生去电影院看电影。东风小学共有 21 个班, 平均每个班有学生 43 人, 每个班有一个带队老师,一共要买多少张电影票?", "options": [], "subject": "算术", "analysis": "924 张\n\n【分析】 21 个班的学生平均每个班有学生 43 人, 根据整数乘法的意义, 共有 $21 \\times 43=903$ 人, 又因每个班有一个带队老师, 则再加上 21 个带队老师, 即是一共要买的电影票张数。\n\n【详解】 $21 \\times 43+21$\n\n$=903+21$\n\n$=924$ (张)\n\n答: 一共要买 924 张电影票。\n\n【点睛】本题主要考查了整数乘法的意义和实际应用, 先求出学生总人数是解题的关键。"} {"id": "8389", "image": [], "answer": "3200 元\n\n【分析】用箱数乘每箱的千克数, 求出苹果的总重, 再根据总价=单价×数量来列式解答。\n\n【详解】 $32 \\times 20 \\times 5$\n\n$=640 \\times 5$\n\n$=3200($ 元 $)$\n\n答: 这些苹果一共能卖 3200 元。\n\n【点睛】此题主要考查了整数乘法的意义的应用, 解答此题的关键是熟练掌握单价、总价、数量的关系。", "solution": "null", "level": "三年级", "question": "有 32 箱苹果, 每箱 20 千克, 每千克苹果 5 元, 这些苹果一共能卖多少钱?", "options": [], "subject": "算术", "analysis": "3200 元\n\n【分析】用箱数乘每箱的千克数, 求出苹果的总重, 再根据总价=单价×数量来列式解答。\n\n【详解】 $32 \\times 20 \\times 5$\n\n$=640 \\times 5$\n\n$=3200($ 元 $)$\n\n答: 这些苹果一共能卖 3200 元。\n\n【点睛】此题主要考查了整数乘法的意义的应用, 解答此题的关键是熟练掌握单价、总价、数量的关系。"} {"id": "8390", "image": ["1309.jpg"], "answer": "450 千克\n\n【分析】用 1 箱蜜蜂产蜂蜜重量乘蜜蜂箱数, 求出可产蜂蜜总重量。\n\n【详解】 $30 \\times 15=450$ (千克)\n\n答: 王叔叔家的蜜蜂可产蜂蜜 450 千克。\n\n【点睛】熟练掌握两位数乘两位数的计算方法并正确计算是解决本题的关键。", "solution": "null", "level": "三年级", "question": "养 1 箱蜜蜂可产蜂蜜 30 千克, 王叔叔家的蜜蜂可产蜂蜜多少千克?\n\n", "options": [], "subject": "算术", "analysis": "450 千克\n\n【分析】用 1 箱蜜蜂产蜂蜜重量乘蜜蜂箱数, 求出可产蜂蜜总重量。\n\n【详解】 $30 \\times 15=450$ (千克)\n\n答: 王叔叔家的蜜蜂可产蜂蜜 450 千克。\n\n【点睛】熟练掌握两位数乘两位数的计算方法并正确计算是解决本题的关键。"} {"id": "8391", "image": [], "answer": "832 元\n\n【分析】用每箱保温杯的个数乘箱数, 求出保温杯的个数。再乘每个保温杯的价钱, 求出一共卖的钱数。\n\n【详解】 $8 \\times 4 \\times 26$\n\n$=32 \\times 26$\n$=832 ($ 元)\n\n答: 一共卖了 832 元。\n\n【点睛】本题求出保温杯的个数, 再根据总价 $=$ 单价 $x$ 数量解答。", "solution": "null", "level": "三年级", "question": "保温杯每箱有 8 个, 商店一周卖出 4 箱保温杯, 每个保温杯卖 26 元, 一共卖了多少钱?", "options": [], "subject": "算术", "analysis": "832 元\n\n【分析】用每箱保温杯的个数乘箱数, 求出保温杯的个数。再乘每个保温杯的价钱, 求出一共卖的钱数。\n\n【详解】 $8 \\times 4 \\times 26$\n\n$=32 \\times 26$\n$=832 ($ 元)\n\n答: 一共卖了 832 元。\n\n【点睛】本题求出保温杯的个数, 再根据总价 $=$ 单价 $x$ 数量解答。"} {"id": "8392", "image": [], "answer": "546 棵\n\n【分析】梨树的棵数是苹果树的 12 倍, 根据整数乘法的意义, 用苹果树的棵数乘 12 , 求出梨树的棵数; 然后根据加法的意义将苹果树与梨树的棵数相加, 即可求出这个果园一共栽了多少棵果树。\n\n【详解】 $42 \\times 12+42$\n\n$=504+42$\n\n$=546($ 棵 $)$\n\n答: 这个果园一共栽了 546 棵果树。\n\n【点睛】此题主要考查了乘法、加法的意义的应用, 解答本题的关键是先求出梨树的棵数。", "solution": "null", "level": "三年级", "question": "一个果园里裁了两种果树, 其中苹果树有 42 棵, 梨树的棵数是苹果树的 12 倍。这个果园一共栽了多少棵果树?", "options": [], "subject": "算术", "analysis": "546 棵\n\n【分析】梨树的棵数是苹果树的 12 倍, 根据整数乘法的意义, 用苹果树的棵数乘 12 , 求出梨树的棵数; 然后根据加法的意义将苹果树与梨树的棵数相加, 即可求出这个果园一共栽了多少棵果树。\n\n【详解】 $42 \\times 12+42$\n\n$=504+42$\n\n$=546($ 棵 $)$\n\n答: 这个果园一共栽了 546 棵果树。\n\n【点睛】此题主要考查了乘法、加法的意义的应用, 解答本题的关键是先求出梨树的棵数。"} {"id": "8406", "image": [], "answer": "V\n\n【分析】根据整数乘法的计算方法, 计算出 $49 \\times 28$ 的积, 然后与 1500 进行比较即可, 据此解决。\n\n【详解】 $49 \\times 28=1372,1372<1500$, 本题说法正确。\n\n故答案为: $\\sqrt{ }$\n\n【点睛】解决本题的关键是熟练掌握整数乘法的计算方法, 本题还可以用估算的方法解决。", "solution": "null", "level": "三年级", "question": "$49 \\times 28$ 的计算结果一定比 1500 小。 $(\\quad)$", "options": [], "subject": "算术", "analysis": "V\n\n【分析】根据整数乘法的计算方法, 计算出 $49 \\times 28$ 的积, 然后与 1500 进行比较即可, 据此解决。\n\n【详解】 $49 \\times 28=1372,1372<1500$, 本题说法正确。\n\n故答案为: $\\sqrt{ }$\n\n【点睛】解决本题的关键是熟练掌握整数乘法的计算方法, 本题还可以用估算的方法解决。"} {"id": "8407", "image": [], "answer": "V\n\n【分析】计算乘数末尾有 0 的乘法时, 先用乘数去乘 0 前面的数, 再看乘数末尾一共有几个 0 , 就在乘积的末尾再添上几个 0, 据此判断即可。\n\n【详解】在计算两位数乘法时, 一个乘数末尾有 0 , 积的末尾也一定有 0 , 说法正确。故答案为: $\\sqrt{ }$\n\n【点睛】此题主要考查乘数末尾有 0 的整数乘法的运用。", "solution": "null", "level": "三年级", "question": "在计算两位数乘法时, 一个乘数末尾有 0 , 积的末尾也一定有 0 。( )", "options": [], "subject": "算术", "analysis": "V\n\n【分析】计算乘数末尾有 0 的乘法时, 先用乘数去乘 0 前面的数, 再看乘数末尾一共有几个 0 , 就在乘积的末尾再添上几个 0, 据此判断即可。\n\n【详解】在计算两位数乘法时, 一个乘数末尾有 0 , 积的末尾也一定有 0 , 说法正确。故答案为: $\\sqrt{ }$\n\n【点睛】此题主要考查乘数末尾有 0 的整数乘法的运用。"} {"id": "8408", "image": [], "answer": "V\n\n【分析】两位数乘两位数:先用第 2 个因数个位上的数去乘第 1 个因数的每一位, 得数的末位和第 2\n个因数的个位对齐; 再用第 2 个因数十位上的数去乘第 1 个因数的每一位, 得数的末位和第 2 个因数的十位对齐; 然后把两次乘得的数加起来。\n\n坚式中的“ 36 ”表示 36 个十, 即 360 , 是 12 与 30 的乘积。据此解答。\n\n【详解】坚式中的 48 是 12 与 4 的乘积, 表示的是 4 小时做 48 个布线玩具; 坚式中的“36”表示 36 个十, 即 360, 表示 30 小时做 360 个布线玩具。原题说法正确。\n\n故答案为: $\\sqrt{ }$\n\n【点睛】本题考查了对坚式计算过程的理解,关键是理解算理。", "solution": "null", "level": "三年级", "question": "李阿姨平均每小时做 12 个布绒玩具, 34 小时共做多少个? 乘法坚式 $12 \\times 34=408$ 中的“ 36 ”表示“ 30 小时做 360 个布线玩具”。( $\\quad)$\n\n| 12 |\n| ---: |\n| $\\times \\quad 34$ |\n| 48 |\n| 36 |\n| 408 |", "options": [], "subject": "算术", "analysis": "V\n\n【分析】两位数乘两位数:先用第 2 个因数个位上的数去乘第 1 个因数的每一位, 得数的末位和第 2\n个因数的个位对齐; 再用第 2 个因数十位上的数去乘第 1 个因数的每一位, 得数的末位和第 2 个因数的十位对齐; 然后把两次乘得的数加起来。\n\n坚式中的“ 36 ”表示 36 个十, 即 360 , 是 12 与 30 的乘积。据此解答。\n\n【详解】坚式中的 48 是 12 与 4 的乘积, 表示的是 4 小时做 48 个布线玩具; 坚式中的“36”表示 36 个十, 即 360, 表示 30 小时做 360 个布线玩具。原题说法正确。\n\n故答案为: $\\sqrt{ }$\n\n【点睛】本题考查了对坚式计算过程的理解,关键是理解算理。"} {"id": "8409", "image": [], "answer": "V\n\n【分析】验算乘法算式是否正确, 可用交换乘数的位置, 再乘一遍来验算乘法, 也可用积 $\\div 一$ 一个因数 $=$ 另一个因数来验算乘法; 据此解答即可。\n\n【详解】 例如: $25 \\times 36=900$\n\n| 2 | | | |\n| :---: | :---: | :---: | :---: |\n| 3 | 6 | $x$ | |\n| 5 | 0 验算: | $\\overline{1}$ | |\n| 5 | | 7 | |\n| | | 9 | |\n\n所以验算两位数乘两位数时, 可以交换乘数的位置再进行计算, 此说法正确。\n\n故答案为: $\\sqrt{ }$\n\n【点睛】解答本题关键是熟练掌握两位数乘两位数的计算法则和乘法的验算方法。", "solution": "null", "level": "三年级", "question": "验算两位数乘两位数时, 可以交换乘数的位置再进行计算。( )", "options": [], "subject": "算术", "analysis": "V\n\n【分析】验算乘法算式是否正确, 可用交换乘数的位置, 再乘一遍来验算乘法, 也可用积 $\\div 一$ 一个因数 $=$ 另一个因数来验算乘法; 据此解答即可。\n\n【详解】 例如: $25 \\times 36=900$\n\n| 2 | | | |\n| :---: | :---: | :---: | :---: |\n| 3 | 6 | $x$ | |\n| 5 | 0 验算: | $\\overline{1}$ | |\n| 5 | | 7 | |\n| | | 9 | |\n\n所以验算两位数乘两位数时, 可以交换乘数的位置再进行计算, 此说法正确。\n\n故答案为: $\\sqrt{ }$\n\n【点睛】解答本题关键是熟练掌握两位数乘两位数的计算法则和乘法的验算方法。"} {"id": "8410", "image": [], "answer": "$\\sqrt{ }$\n\n【分析】 $15 \\times(36 \\div 6)$ 的运算顺序是先算除法, 再算乘法。如果去掉括号, 先算乘法再算除法, 两个式子的结果都是 90 。\n\n【详解】 $15 \\times(36 \\div 6)$\n\n$=15 \\times 6$\n\n$=90$\n\n$15 \\times 36 \\div 6$\n\n$=540 \\div 6$\n\n$=90$\n\n去掉 $15 \\times(36 \\div 6)$ 的括号, 计算结果不变。\n\n故答案为: $V$\n\n【点睛】本题考查学生两位数乘两位数的计算能力。在乘除法混合运算中, 如果括号前面是乘法, 去掉括号不改变计算结果。如果括号前面是除法, 去掉括号会改变运算结果。", "solution": "null", "level": "三年级", "question": "去掉 $15 \\times ( 36 \\div 6 )$ 中的括号不改变计算结果。( $)$", "options": [], "subject": "算术", "analysis": "$\\sqrt{ }$\n\n【分析】 $15 \\times(36 \\div 6)$ 的运算顺序是先算除法, 再算乘法。如果去掉括号, 先算乘法再算除法, 两个式子的结果都是 90 。\n\n【详解】 $15 \\times(36 \\div 6)$\n\n$=15 \\times 6$\n\n$=90$\n\n$15 \\times 36 \\div 6$\n\n$=540 \\div 6$\n\n$=90$\n\n去掉 $15 \\times(36 \\div 6)$ 的括号, 计算结果不变。\n\n故答案为: $V$\n\n【点睛】本题考查学生两位数乘两位数的计算能力。在乘除法混合运算中, 如果括号前面是乘法, 去掉括号不改变计算结果。如果括号前面是除法, 去掉括号会改变运算结果。"} {"id": "8415", "image": [], "answer": "不能\n\n【分析】工作效率 $\\times$ 工作时间 $=$ 工作总量, 再把工作总量和 1000 个字进行比较。\n\n【详解】 $48 \\times 20=960$ (个字)\n\n$960<1000$\n\n答: 他 20 分钟不能打完这份稿件。\n\n【点睛】本题主要考查的是工程问题的实际应用。", "solution": "null", "level": "三年级", "question": "李文用电脑打一份稿件, 平均每分钟打 48 个字。一份稿件有 1000 个字, 他 20 分钟能打完这份稿件吗?", "options": [], "subject": "算术", "analysis": "不能\n\n【分析】工作效率 $\\times$ 工作时间 $=$ 工作总量, 再把工作总量和 1000 个字进行比较。\n\n【详解】 $48 \\times 20=960$ (个字)\n\n$960<1000$\n\n答: 他 20 分钟不能打完这份稿件。\n\n【点睛】本题主要考查的是工程问题的实际应用。"} {"id": "8416", "image": [], "answer": "1820 人\n\n【分析】学生排成的排数 $\\times$ 每排的人数 $=$ 总人数, 依此列式并计算即可。\n\n【详解】 $52 \\times 35=1820$ (人)\n\n答: 这个方阵一共有 1820 人。\n\n【点睛】熟练掌握两位数与两位数的乘法计算是解答此题的关键。", "solution": "null", "level": "三年级", "question": "学校“阳光大课间”展示时, 将学生排成了一个有 52 排, 每排 35 人的方阵, 这个方阵一共有多少人?", "options": [], "subject": "算术", "analysis": "1820 人\n\n【分析】学生排成的排数 $\\times$ 每排的人数 $=$ 总人数, 依此列式并计算即可。\n\n【详解】 $52 \\times 35=1820$ (人)\n\n答: 这个方阵一共有 1820 人。\n\n【点睛】熟练掌握两位数与两位数的乘法计算是解答此题的关键。"} {"id": "8417", "image": [], "answer": "720 箱\n\n【分析】用梨的箱数乘 39 , 求出苹果的箱数。再将梨的箱数加上苹果的箱数解答。\n\n【详解】 $18 \\times 39+18$\n\n$=702+18$\n\n$=720($ 箱 $)$\n\n答: 一共运进 720 箱水果。\n\n【点睛】本题关键是正确理解倍数关系: 求一个数的几倍是多少, 用乘法计算。", "solution": "null", "level": "三年级", "question": "水果店运进梨 18 箱, 运进的苹果箱数是梨的 39 倍, 一共运进多少箱水果?", "options": [], "subject": "算术", "analysis": "720 箱\n\n【分析】用梨的箱数乘 39 , 求出苹果的箱数。再将梨的箱数加上苹果的箱数解答。\n\n【详解】 $18 \\times 39+18$\n\n$=702+18$\n\n$=720($ 箱 $)$\n\n答: 一共运进 720 箱水果。\n\n【点睛】本题关键是正确理解倍数关系: 求一个数的几倍是多少, 用乘法计算。"} {"id": "8418", "image": [], "answer": "1092 元\n\n【分析】用前 3 个月的电话费除以 3 , 求出平均每个月的电话费。一年共 12 个月, 用平均每个月的电话费乘 12 , 求出一年的电话费。\n\n【详解】 $273 \\div 3 \\times 12$\n\n$=91 \\times 12$\n\n$=1092($ 元 $)$\n\n答: 他家一年的电话费是 1092 元。\n\n【点睛】本题考查归一问题, 先求单一量, 再求总量。", "solution": "null", "level": "三年级", "question": "兰兰家今年前 3 个月的电话费是 273 元, 照这样计算, 他家一年的电话费是多少元?", "options": [], "subject": "算术", "analysis": "1092 元\n\n【分析】用前 3 个月的电话费除以 3 , 求出平均每个月的电话费。一年共 12 个月, 用平均每个月的电话费乘 12 , 求出一年的电话费。\n\n【详解】 $273 \\div 3 \\times 12$\n\n$=91 \\times 12$\n\n$=1092($ 元 $)$\n\n答: 他家一年的电话费是 1092 元。\n\n【点睛】本题考查归一问题, 先求单一量, 再求总量。"} {"id": "8419", "image": ["1314.jpg"], "answer": "750 米\n\n【分析】观察钟面可知, 李明从学校出发的时间是 12:15、到家的时间是 12:30, 据此用到家的时间减去从学校出发的时间, 求出李明路上所用的时间, 用李明每分钟所走的米数乘所用的时间即可。\n\n【详解】12: $30-12: 15=15$ (分钟)\n\n$50 \\times 15=750$ (米)\n\n答: 李明家距离学校有 750 米远。\n\n【点睛】熟练掌握有关时分秒时间的推算方法, 求出李明路上所用的时间, 是解答此题的关键。", "solution": "null", "level": "三年级", "question": "李明每分钟走 50 米, 下图是他一天中午从学校走到家的时间, 李明家距离学校有多远?\n\n", "options": [], "subject": "算术", "analysis": "750 米\n\n【分析】观察钟面可知, 李明从学校出发的时间是 12:15、到家的时间是 12:30, 据此用到家的时间减去从学校出发的时间, 求出李明路上所用的时间, 用李明每分钟所走的米数乘所用的时间即可。\n\n【详解】12: $30-12: 15=15$ (分钟)\n\n$50 \\times 15=750$ (米)\n\n答: 李明家距离学校有 750 米远。\n\n【点睛】熟练掌握有关时分秒时间的推算方法, 求出李明路上所用的时间, 是解答此题的关键。"} {"id": "8433", "image": [], "answer": "$\\times$\n\n【分析】如果这两个数中有一个数是 0 , 因为 0 乘任何数都得 0,0 加另一个数字的和是另一个数字,那么, 这两个数的积就小于这两个数的和。\n\n【详解】如 $0 \\times 23=0,0+23=23,0<23$, 两个数相乘的积, 比两个数相加的和小; $2 \\times 123=246,2+123=125,246>125$, 两个数相乘的积, 比两个数相加的和大;所以题干说法错误。\n\n故答案为: $\\times$\n\n【点睛】灵活应用 0 与任何数相乘都得 0 的性质即可解答。", "solution": "null", "level": "三年级", "question": "两个数相乘的积, 一定比两个数相加的和大。( $)$", "options": [], "subject": "算术", "analysis": "$\\times$\n\n【分析】如果这两个数中有一个数是 0 , 因为 0 乘任何数都得 0,0 加另一个数字的和是另一个数字,那么, 这两个数的积就小于这两个数的和。\n\n【详解】如 $0 \\times 23=0,0+23=23,0<23$, 两个数相乘的积, 比两个数相加的和小; $2 \\times 123=246,2+123=125,246>125$, 两个数相乘的积, 比两个数相加的和大;所以题干说法错误。\n\n故答案为: $\\times$\n\n【点睛】灵活应用 0 与任何数相乘都得 0 的性质即可解答。"} {"id": "8434", "image": ["1320.jpg", "1320.jpg"], "answer": "V\n\n【分析】先分别算出 $28 \\times 50$ 和 $280 \\times 5$ 的积, 再比较大小即可。\n\n【详解】列坚式如下:\n\n\n\n所以 $28 \\times 50$ 和 $280 \\times 5$ 的计算结果相同, 原题说法正确。\n\n故答案为: $\\sqrt{ }$\n\n【点睛】本题考查了两位数乘整十数和三位数乘一位数的笔算。", "solution": "null", "level": "三年级", "question": "$28 \\times 50$ 和 $280 \\times 5$ 的计算结果相同。", "options": [], "subject": "算术", "analysis": "V\n\n【分析】先分别算出 $28 \\times 50$ 和 $280 \\times 5$ 的积, 再比较大小即可。\n\n【详解】列坚式如下:\n\n\n\n所以 $28 \\times 50$ 和 $280 \\times 5$ 的计算结果相同, 原题说法正确。\n\n故答案为: $\\sqrt{ }$\n\n【点睛】本题考查了两位数乘整十数和三位数乘一位数的笔算。"} {"id": "8435", "image": [], "answer": "$\\times$\n\n【分析】其中一个因数末尾有 0 , 积的末尾也一定有 0 ; 两个因数的末尾都没有 0 , 积的末尾可能有 0 ;据此解答。\n\n【详解】根据分析举例: $256 \\times 35=8960$, 这两个因数的末尾都没有 0 , 但是积的末尾有 0 。故答案为: $\\times$\n\n【点睛】本题主要考查的是积的末尾的 0 和因数的关系。", "solution": "null", "level": "三年级", "question": "两个因数的末尾都没有 0 , 积的末尾也一定没有 0 。 ( $)$", "options": [], "subject": "算术", "analysis": "$\\times$\n\n【分析】其中一个因数末尾有 0 , 积的末尾也一定有 0 ; 两个因数的末尾都没有 0 , 积的末尾可能有 0 ;据此解答。\n\n【详解】根据分析举例: $256 \\times 35=8960$, 这两个因数的末尾都没有 0 , 但是积的末尾有 0 。故答案为: $\\times$\n\n【点睛】本题主要考查的是积的末尾的 0 和因数的关系。"} {"id": "8436", "image": [], "answer": "V\n\n【分析】根据题意, 运用赋值法, 假设口里的数是 1 或 9 , 分别求出它们各自的乘积, 然后再进一步解答。\n\n【详解】假设 $\\square$ 里的数是 1 , 此时算式的积最小, 积是: $48 \\times 19=912$;\n\n假设口里的数是 9 , 此时算式的积最大, 积是: $48 \\times 99=4752$ 。\n\n912 是一个三位数, 4752 是一个四位数, 所以 $48 \\times$ 口 9 的积可能是三位数, 也可能是四位数。题目说法正确。\n\n故答案为: $\\sqrt{ }$\n\n【点睛】本题主要考查两位数乘两位数计算方法, 运用赋值法比较容易解决此题。", "solution": "null", "level": "三年级", "question": "$48 \\times$ 口 9 的积可能是三位数, 也可能是四位数。", "options": [], "subject": "算术", "analysis": "V\n\n【分析】根据题意, 运用赋值法, 假设口里的数是 1 或 9 , 分别求出它们各自的乘积, 然后再进一步解答。\n\n【详解】假设 $\\square$ 里的数是 1 , 此时算式的积最小, 积是: $48 \\times 19=912$;\n\n假设口里的数是 9 , 此时算式的积最大, 积是: $48 \\times 99=4752$ 。\n\n912 是一个三位数, 4752 是一个四位数, 所以 $48 \\times$ 口 9 的积可能是三位数, 也可能是四位数。题目说法正确。\n\n故答案为: $\\sqrt{ }$\n\n【点睛】本题主要考查两位数乘两位数计算方法, 运用赋值法比较容易解决此题。"} {"id": "8437", "image": [], "answer": "$\\sqrt{ }$\n\n【分析】 12 个 25 相加的和等于 25 乘 12,321 除以 3 等于 321 里有多少个 3 , 据此即可解答。\n\n【详解】 $25 \\times 12=300$\n\n$321 \\div 3=107$\n\n所以 12 个 25 相加是 300,321 连续减去 107 个 3 后, 结果是 0 , 原说法正确。\n\n故答案为: $\\sqrt{ }$\n\n【点睛】本题主要考查学生对整数乘除法计算方法的掌握和灵活运用。", "solution": "null", "level": "三年级", "question": "12 个 25 相加是 300,321 连续减去 107 个 3 后,结果是 0 。( )", "options": [], "subject": "算术", "analysis": "$\\sqrt{ }$\n\n【分析】 12 个 25 相加的和等于 25 乘 12,321 除以 3 等于 321 里有多少个 3 , 据此即可解答。\n\n【详解】 $25 \\times 12=300$\n\n$321 \\div 3=107$\n\n所以 12 个 25 相加是 300,321 连续减去 107 个 3 后, 结果是 0 , 原说法正确。\n\n故答案为: $\\sqrt{ }$\n\n【点睛】本题主要考查学生对整数乘除法计算方法的掌握和灵活运用。"} {"id": "8441", "image": [], "answer": "能\n\n【分析】 9 箱水蜜桃的重量 $=9=$ 平均每箱水蜜桃的重量, 平均每箱水蜜桃的重量 $\\times 17=17$ 箱水蜜桃的重量, 上午摘水蜜桃的重量 + 下午摘水蜜桃的重量 $=$ 水蜜桃的总重量, 依此计算并与 2 吨比较即可, 1 吨 $=1000$ 千克,依此换算并比较。\n\n【详解】 $675 \\div 9=75$ (千克)\n\n$75 \\times 17=1275$ (千克)\n\n$675+1275=1950$ (千克)\n\n2 吨 $=2000$ 千克\n\n1950 千克 $<2000$ 千克, 一次能运完\n\n答: 一次能运完。\n\n【点睛】此题考查的是工程问题的计算,先计算出李大伯这一天摘水蜜桃的总重量是解答此题的关键。", "solution": "null", "level": "三年级", "question": "李大伯家上午摘了 9 箱水蜜桃重 675 千克, 下午摘了同样的 17 箱, 用一辆载重 2 吨的货车一次能运完吗?", "options": [], "subject": "算术", "analysis": "能\n\n【分析】 9 箱水蜜桃的重量 $=9=$ 平均每箱水蜜桃的重量, 平均每箱水蜜桃的重量 $\\times 17=17$ 箱水蜜桃的重量, 上午摘水蜜桃的重量 + 下午摘水蜜桃的重量 $=$ 水蜜桃的总重量, 依此计算并与 2 吨比较即可, 1 吨 $=1000$ 千克,依此换算并比较。\n\n【详解】 $675 \\div 9=75$ (千克)\n\n$75 \\times 17=1275$ (千克)\n\n$675+1275=1950$ (千克)\n\n2 吨 $=2000$ 千克\n\n1950 千克 $<2000$ 千克, 一次能运完\n\n答: 一次能运完。\n\n【点睛】此题考查的是工程问题的计算,先计算出李大伯这一天摘水蜜桃的总重量是解答此题的关键。"} {"id": "8442", "image": [], "answer": "1290 元\n\n【分析】用每张成人票的钱数乘 2 ,可以计算出购买成人票花的钱数,再用每张儿童票的钱数乘 39 ,可以计算出购买儿童票花的钱数, 最后把两种票的钱数相加, 可以计算出一共需要多少钱。\n\n【详解】 $60 \\times 2+30 \\times 39$\n\n$=120+1170$\n\n$=1290($ 元 $)$\n\n答: 一共需要 1290 元。\n\n【点睛】本题解题关键是用每张票的钱数乘购票人数, 分别计算出购买成人票与儿童票各需要的钱数,再把两种票的钱数相加即可。", "solution": "null", "level": "三年级", "question": "王老师和张老师带 39 名同学去世界月季大观园游玩,一共需要多少钱?\n\n售票处\n\n成人: 60 元\n\n儿童: 30 元", "options": [], "subject": "算术", "analysis": "1290 元\n\n【分析】用每张成人票的钱数乘 2 ,可以计算出购买成人票花的钱数,再用每张儿童票的钱数乘 39 ,可以计算出购买儿童票花的钱数, 最后把两种票的钱数相加, 可以计算出一共需要多少钱。\n\n【详解】 $60 \\times 2+30 \\times 39$\n\n$=120+1170$\n\n$=1290($ 元 $)$\n\n答: 一共需要 1290 元。\n\n【点睛】本题解题关键是用每张票的钱数乘购票人数, 分别计算出购买成人票与儿童票各需要的钱数,再把两种票的钱数相加即可。"} {"id": "8443", "image": [], "answer": "29 元\n\n【分析】比 27 元贵。买了 16 个足球后, 钱没花完。即 16 和比 27 大的数的乘积不超过 500 元, 计算出可能的情况即可; 如果买完足球后剩余 36 元,则买足球花了 $500-36=464$ (元),然后选出乘积是 464 的算式便知道足球的价钱。\n\n【详解】 $16 \\times 28=448 ($ 元)\n\n$16 \\times 29=464 ($ 元)\n\n$16 \\times 30=480($ 元 $)$\n\n$16 \\times 31=496 ($ 元)\n\n$500-36=464 ($ 元)\n\n答: 足球的价钱是 29 元。\n\n【点睛】本题考查两位数与两位数乘法的实际运用, 运用“单价 $\\times$ 数量 $=$ 总价”来解答。", "solution": "null", "level": "三年级", "question": "足球社团的刘老师带 500 元钱去买足球, 发现一个足球的价格 (整元数) 比 27 元贵, 他买了 16 个足球后, 发现钱还没花完。如果刘老师买完足球后还剩 36 元,每个足球多少钱?", "options": [], "subject": "算术", "analysis": "29 元\n\n【分析】比 27 元贵。买了 16 个足球后, 钱没花完。即 16 和比 27 大的数的乘积不超过 500 元, 计算出可能的情况即可; 如果买完足球后剩余 36 元,则买足球花了 $500-36=464$ (元),然后选出乘积是 464 的算式便知道足球的价钱。\n\n【详解】 $16 \\times 28=448 ($ 元)\n\n$16 \\times 29=464 ($ 元)\n\n$16 \\times 30=480($ 元 $)$\n\n$16 \\times 31=496 ($ 元)\n\n$500-36=464 ($ 元)\n\n答: 足球的价钱是 29 元。\n\n【点睛】本题考查两位数与两位数乘法的实际运用, 运用“单价 $\\times$ 数量 $=$ 总价”来解答。"} {"id": "8444", "image": [], "answer": "2016 千克\n\n【分析】根据题意, 用大睏桃树的行数乘每行的棵树求得树的总棵数, 再用总棵数乘平均每棵收获的量即可得到一共收获的量。\n\n【详解】 $12 \\times 7 \\times 24$\n\n$=84 \\times 24$\n\n$=2016$ (千克)\n\n答: 小方家一共收获了 2016 千克。\n\n【点睛】本题考查的是用连乘解决实际问题, 关键先求出秒桃树的总棵数。", "solution": "null", "level": "三年级", "question": "大樱桃是很多人喜爱的水果, 不仅产量高, 还耐运输, 是增加农民收入的优质水果。小方家种了 12 行大樱桃树, 每行 7 棵, 今年平均每棵收获大管桃 24 千克, 今年小方家一共收获了多少千克大樱桃?", "options": [], "subject": "算术", "analysis": "2016 千克\n\n【分析】根据题意, 用大睏桃树的行数乘每行的棵树求得树的总棵数, 再用总棵数乘平均每棵收获的量即可得到一共收获的量。\n\n【详解】 $12 \\times 7 \\times 24$\n\n$=84 \\times 24$\n\n$=2016$ (千克)\n\n答: 小方家一共收获了 2016 千克。\n\n【点睛】本题考查的是用连乘解决实际问题, 关键先求出秒桃树的总棵数。"} {"id": "8445", "image": [], "answer": "(1)能\n\n(2) 13 个\n\n【分析】 (1) 根据 1 辆大客车能载客 55 人, 12 辆大客车一次就能送 12 个 55 人, 用 55 乘 12 求出 12 辆大客车能一次送走的人数, 再与总人数 650 人比较即可求解;\n\n(2)分析题意, 一共分成了 9 组, 一共包了 585 个粽子, 则用除法可求出每组包粽子的个数; 每组有 5 人, 则用每组包粽子的个数除以 5 , 就是平均每人包粽子的个数。\n\n【详解】(1) $55 \\times 12=660$ (人)\n\n$660>650$\n\n答: 12 辆大客车能一次送完这些同学。\n\n(2) $585 \\div 9 \\div 5$\n\n$=65 \\div 5$\n\n$=13($ 个 $)$\n\n答: 平均每人包了 13 个粽子。\n\n【点睛】本题主要考查了学生根据乘除法的意义解答应用题的能力。", "solution": "null", "level": "三年级", "question": "研学旅行, 快乐学习。\n\n(1)红星小学有 650 人到研学实践基地开展体验活动, 1 辆大客车能载客 55 人, 12 辆大客车能一次送完这些同学吗?\n\n(2) 研学活动中开展了“包粽子”活动, 三年级一班一共分成了 9 组, 每组 5 人, 一共包了 585 个。平均每人包了多少个粽子?\n\n", "options": [], "subject": "算术", "analysis": "(1)能\n\n(2) 13 个\n\n【分析】 (1) 根据 1 辆大客车能载客 55 人, 12 辆大客车一次就能送 12 个 55 人, 用 55 乘 12 求出 12 辆大客车能一次送走的人数, 再与总人数 650 人比较即可求解;\n\n(2)分析题意, 一共分成了 9 组, 一共包了 585 个粽子, 则用除法可求出每组包粽子的个数; 每组有 5 人, 则用每组包粽子的个数除以 5 , 就是平均每人包粽子的个数。\n\n【详解】(1) $55 \\times 12=660$ (人)\n\n$660>650$\n\n答: 12 辆大客车能一次送完这些同学。\n\n(2) $585 \\div 9 \\div 5$\n\n$=65 \\div 5$\n\n$=13($ 个 $)$\n\n答: 平均每人包了 13 个粽子。\n\n【点睛】本题主要考查了学生根据乘除法的意义解答应用题的能力。"} {"id": "8459", "image": [], "answer": "$\\times$\n\n【分析】 $105 \\times 3=315,102 \\times 3=306$, 据此即可判断。\n\n【详解】根据分析可知, 一个因数的中间有 0 , 积的中间不一定有 0 , 所以判断错误。\n\n【点睛】本题可以通过举例说明原说法错误。", "solution": "null", "level": "三年级", "question": "一个因数的中间有 0 , 积的中间也一定有 0 。( )", "options": [], "subject": "算术", "analysis": "$\\times$\n\n【分析】 $105 \\times 3=315,102 \\times 3=306$, 据此即可判断。\n\n【详解】根据分析可知, 一个因数的中间有 0 , 积的中间不一定有 0 , 所以判断错误。\n\n【点睛】本题可以通过举例说明原说法错误。"} {"id": "8460", "image": [], "answer": "$\\times$\n\n【分析】根据题意, 假设 5 口 $\\times 5$ 口个位上的数字都相同, 当都填 0 时, 求出乘积, 再进行判断即可。\n\n【详解】当口里都填 0 时,\n\n则 $50 \\times 50=2500$,\n\n$2500<4000$\n\n所以, $5 \\square \\times 5$ 口, 哩填上任何一个数字, 所得的积一定比 4000 大, 此说法是错误。\n故答案为: $\\times$\n\n【点睛】本题考查两位数乘两位数的计算方法, 根据题意, 运用赋值法比较容易解答此类问题。", "solution": "null", "level": "三年级", "question": "$5 \\square \\times 5$ 口, 哩填上任何一个数字, 所得的积一定比 4000 大。( )", "options": [], "subject": "算术", "analysis": "$\\times$\n\n【分析】根据题意, 假设 5 口 $\\times 5$ 口个位上的数字都相同, 当都填 0 时, 求出乘积, 再进行判断即可。\n\n【详解】当口里都填 0 时,\n\n则 $50 \\times 50=2500$,\n\n$2500<4000$\n\n所以, $5 \\square \\times 5$ 口, 哩填上任何一个数字, 所得的积一定比 4000 大, 此说法是错误。\n故答案为: $\\times$\n\n【点睛】本题考查两位数乘两位数的计算方法, 根据题意, 运用赋值法比较容易解答此类问题。"} {"id": "8461", "image": [], "answer": "$\\sqrt{ }$\n\n【分析】计算出 $65 \\times 48$ 和 $32 \\times 15$ 的积即可解答。\n\n【详解】 $65 \\times 48=3120,32 \\times 15=480$, 积的末位上的数字都是 0 。\n\n故答案为: $\\sqrt{ }$\n\n【点睛】本题主要考查学生对整数乘法计算方法的掌握。", "solution": "null", "level": "三年级", "question": "$65 \\times 48$ 的积与 $32 \\times 15$ 的积的末位上的数字相同。( )", "options": [], "subject": "算术", "analysis": "$\\sqrt{ }$\n\n【分析】计算出 $65 \\times 48$ 和 $32 \\times 15$ 的积即可解答。\n\n【详解】 $65 \\times 48=3120,32 \\times 15=480$, 积的末位上的数字都是 0 。\n\n故答案为: $\\sqrt{ }$\n\n【点睛】本题主要考查学生对整数乘法计算方法的掌握。"} {"id": "8462", "image": [], "answer": "V\n\n【分析】 直接计算出 $45 \\times 23$ 的积即可解答。\n\n【详解】 $45 \\times 23=1035$, 积是四位数, 最高位是千位, 所以判断正确。故答案为: $\\sqrt{ }$\n\n【点睛】熟练掌握整数乘法的计算方法是解答本题的关键。", "solution": "null", "level": "三年级", "question": "$45 \\times 23$ 的积的最高位是千位。( $)$", "options": [], "subject": "算术", "analysis": "V\n\n【分析】 直接计算出 $45 \\times 23$ 的积即可解答。\n\n【详解】 $45 \\times 23=1035$, 积是四位数, 最高位是千位, 所以判断正确。故答案为: $\\sqrt{ }$\n\n【点睛】熟练掌握整数乘法的计算方法是解答本题的关键。"} {"id": "8463", "image": [], "answer": "$\\times$\n\n【分析】解答此题时, 可以举例说明, 例如: $20 \\times 50=1000$, 两个乘数的末尾共有 2 个 0 , 积的末尾有 3 个 0 ; 据此判断即可。\n\n【详解】两个乘数的末尾共有 2 个 0 , 假设这两个乘数分别是 20 与 $50,20 \\times 50=1000$, 积的末尾有 3 个 0 。所以, 两个乘数的末尾共有 2 个 0 , 积的末尾不一定只有 2 个 0 , 原题干说法错误。故答案为: $\\times$\n\n【点睛】掌握乘数末尾有 0 的乘法的计算方法, 能够找出反例是解决此类问题的关键。", "solution": "null", "level": "三年级", "question": "两个乘数的末尾共有 2 个 0 , 积的末尾也只能有 2 个 0 。 ( )", "options": [], "subject": "算术", "analysis": "$\\times$\n\n【分析】解答此题时, 可以举例说明, 例如: $20 \\times 50=1000$, 两个乘数的末尾共有 2 个 0 , 积的末尾有 3 个 0 ; 据此判断即可。\n\n【详解】两个乘数的末尾共有 2 个 0 , 假设这两个乘数分别是 20 与 $50,20 \\times 50=1000$, 积的末尾有 3 个 0 。所以, 两个乘数的末尾共有 2 个 0 , 积的末尾不一定只有 2 个 0 , 原题干说法错误。故答案为: $\\times$\n\n【点睛】掌握乘数末尾有 0 的乘法的计算方法, 能够找出反例是解决此类问题的关键。"} {"id": "8468", "image": [], "answer": "购买团体票合算; 要付 180 元\n\n【分析】第一种方案: 分开购买, 即购买 7 张成人票和 3 张学生票, 分别求出需要的钱数再相加;第二种方案: 全部购买团体票, 一共需要购买 $7+3=10$ 张团体票, 由此求出需要的钱数;然后比较两种方法需要的钱数, 即可求解。\n\n【详解】第一种方案:分开购买", "solution": "null", "level": "三年级", "question": "某动物园的门票有以下两种购买方案:\n\n| 方案 1 成人: 每人 20 元 |\n| :---: | :---: |\n| 学生: 每人 15 案 2 团体 10 人以上 (含 10 人) |\n| 每人 18 元 |\n\n成人 7 人, 学生 3 人, 怎样购票合算? 要付多少钱?", "options": [], "subject": "算术", "analysis": "购买团体票合算; 要付 180 元\n\n【分析】第一种方案: 分开购买, 即购买 7 张成人票和 3 张学生票, 分别求出需要的钱数再相加;第二种方案: 全部购买团体票, 一共需要购买 $7+3=10$ 张团体票, 由此求出需要的钱数;然后比较两种方法需要的钱数, 即可求解。\n\n【详解】第一种方案:分开购买"} {"id": "8482", "image": [], "answer": "V\n\n【分析】口算两位数乘整十数是时, 可先不让 0 参与计算, 最后将 0 的个数补在积的末尾处即可, 依此判断。\n\n【详解】口算 $21 \\times 40$ 时, 两个因数的末尾一共有 1 个 0 , 因此口算 $21 \\times 40$ 时, 可以先算 $21 \\times 4=84$,再算 $84 \\times 10=840$ 。\n\n故答案为: $\\sqrt{ }$\n\n【点睛】熟练掌握两位数乘整十数的口算是解答此题的关键。", "solution": "null", "level": "三年级", "question": "口算 $21 \\times 40$ 时, 可以先算 $21 \\times 4=84$, 再算 $84 \\times 10=840$ 。", "options": [], "subject": "算术", "analysis": "V\n\n【分析】口算两位数乘整十数是时, 可先不让 0 参与计算, 最后将 0 的个数补在积的末尾处即可, 依此判断。\n\n【详解】口算 $21 \\times 40$ 时, 两个因数的末尾一共有 1 个 0 , 因此口算 $21 \\times 40$ 时, 可以先算 $21 \\times 4=84$,再算 $84 \\times 10=840$ 。\n\n故答案为: $\\sqrt{ }$\n\n【点睛】熟练掌握两位数乘整十数的口算是解答此题的关键。"} {"id": "8483", "image": [], "answer": "V\n\n【详解】带有括号的混合运算中, 要先计算括号里面的, 再计算括号外面的, 如: (77-42) $\\div 7$, 要先计算括号里面的减法, 再计算括号外面的除法。\n\n$(77-42) \\div 7$\n\n$=35 \\div 7$\n\n$=5$\n\n故答案为: $\\sqrt{ }$", "solution": "null", "level": "三年级", "question": "判断:算式里有括号的, 要先算括号里面的。", "options": [], "subject": "算术", "analysis": "V\n\n【详解】带有括号的混合运算中, 要先计算括号里面的, 再计算括号外面的, 如: (77-42) $\\div 7$, 要先计算括号里面的减法, 再计算括号外面的除法。\n\n$(77-42) \\div 7$\n\n$=35 \\div 7$\n\n$=5$\n\n故答案为: $\\sqrt{ }$"} {"id": "8484", "image": [], "answer": "V\n\n【分析】根据整数乘法中因数末尾有 0 的乘法的计算方法进行解答即可。\n\n【详解】 $36 \\times 80 、 360 \\times 8$ 的积, 都是在 $36 \\times 8$ 的积的末尾添上 1 个 0 。 所以, $36 \\times 80$ 与 $360 \\times 8$ 的积相等,所以判断正确。\n\n故答案为: $\\sqrt{ }$\n\n【点睛】两个因数相乘, 可先把 0 前面的数相乘, 再看因数中一共有几个 0 , 再在积的末尾添上几个\n\n0 。", "solution": "null", "level": "三年级", "question": "$36 \\times 80$ 与 $360 \\times 8$ 的积相等。", "options": [], "subject": "算术", "analysis": "V\n\n【分析】根据整数乘法中因数末尾有 0 的乘法的计算方法进行解答即可。\n\n【详解】 $36 \\times 80 、 360 \\times 8$ 的积, 都是在 $36 \\times 8$ 的积的末尾添上 1 个 0 。 所以, $36 \\times 80$ 与 $360 \\times 8$ 的积相等,所以判断正确。\n\n故答案为: $\\sqrt{ }$\n\n【点睛】两个因数相乘, 可先把 0 前面的数相乘, 再看因数中一共有几个 0 , 再在积的末尾添上几个\n\n0 。"} {"id": "8485", "image": [], "answer": "$\\sqrt{ }$\n\n【分析】与 25 相邻的两个是 24 和 26 , 计算出它们的积即可解答。\n\n【详解】 $24 \\times 26=624$\n\n故答案为: $\\sqrt{ }$\n\n【点睛】本题主要考查学生对整数乘法计算方法的掌握, 先判断出与 25 相邻的两个数, 再作进一步解答。", "solution": "null", "level": "三年级", "question": "与 25 相邻的两个数的积是 624 。", "options": [], "subject": "算术", "analysis": "$\\sqrt{ }$\n\n【分析】与 25 相邻的两个是 24 和 26 , 计算出它们的积即可解答。\n\n【详解】 $24 \\times 26=624$\n\n故答案为: $\\sqrt{ }$\n\n【点睛】本题主要考查学生对整数乘法计算方法的掌握, 先判断出与 25 相邻的两个数, 再作进一步解答。"} {"id": "8486", "image": [], "answer": "$\\sqrt{ }$\n\n【分析】文具店新购进 48 支钢笔, 购进本子的数量是钢笔的 19 倍, 也就是 48 的 19 倍, 即 $48 \\times 19$,把 48 看作 50,19 看作 20 进行估算。\n\n【详解】 $48 \\times 19 \\approx 50 \\times 20=1000$ (本) , 原题说法正确。\n\n故答案为: $\\sqrt{ }$\n\n【点睛】求一个数的几倍是多少, 用乘法进行解答, 注意估算。", "solution": "null", "level": "三年级", "question": "文具店新购进 48 支钢笔, 购进本子的数量是钢笔的 19 倍, 本子的数量大约是 1000 本。(", "options": [], "subject": "算术", "analysis": "$\\sqrt{ }$\n\n【分析】文具店新购进 48 支钢笔, 购进本子的数量是钢笔的 19 倍, 也就是 48 的 19 倍, 即 $48 \\times 19$,把 48 看作 50,19 看作 20 进行估算。\n\n【详解】 $48 \\times 19 \\approx 50 \\times 20=1000$ (本) , 原题说法正确。\n\n故答案为: $\\sqrt{ }$\n\n【点睛】求一个数的几倍是多少, 用乘法进行解答, 注意估算。"} {"id": "8490", "image": [], "answer": "312 只\n\n【分析】由题可知, 用笼子的个数 24 乘每个笼子里鸡的只数 13 只, 即可求出鸡的总只数。\n\n【详解】 $24 \\times 13=312$ (只)\n\n答: 养鸡场一共有 312 只鸡。\n\n【点睛】此题主要考查了两位数乘两位数的实际运用。", "solution": "null", "level": "三年级", "question": "养鸡场用 24 个笼子养鸡, 每个笼子里有 13 只鸡, 养鸡场一共有多少只鸡?", "options": [], "subject": "算术", "analysis": "312 只\n\n【分析】由题可知, 用笼子的个数 24 乘每个笼子里鸡的只数 13 只, 即可求出鸡的总只数。\n\n【详解】 $24 \\times 13=312$ (只)\n\n答: 养鸡场一共有 312 只鸡。\n\n【点睛】此题主要考查了两位数乘两位数的实际运用。"} {"id": "8491", "image": [], "answer": "525 元\n【分析】先用买 6 个篮球花的钱除以 6 计算出每个篮球的价钱, 然后用每个篮球的价钱乘还需要买篮球的个数即可, 依此列式并计算。\n\n【详解】 $210 \\div 6=35$ (元)\n\n$35 \\times 15=525 ($ 元)\n\n答: 还需要准备 525 元。\n\n【点睛】此题考查的是经济问题的计算, 先计算出每个篮球的价钱是解答此题的关键。", "solution": "null", "level": "三年级", "question": "刘老师买 6 个篮球花了 210 元, 她想再买 15 个同样的篮球, 还需要准备多少元?", "options": [], "subject": "算术", "analysis": "525 元\n【分析】先用买 6 个篮球花的钱除以 6 计算出每个篮球的价钱, 然后用每个篮球的价钱乘还需要买篮球的个数即可, 依此列式并计算。\n\n【详解】 $210 \\div 6=35$ (元)\n\n$35 \\times 15=525 ($ 元)\n\n答: 还需要准备 525 元。\n\n【点睛】此题考查的是经济问题的计算, 先计算出每个篮球的价钱是解答此题的关键。"} {"id": "8492", "image": [], "answer": "317 页\n\n【分析】用每天读书页数乘读书天数, 求出读书页数, 再加上没有读的页数, 求出这本书的总页数。\n\n【详解】 $17 \\times 16+45$\n\n$=272+45$\n\n$=317$ (页)\n\n答: 这本书一共有 317 页。\n\n【点睛】本题关键是根据工作总量 $=$ 工作效率 $\\times$ 工作时间, 求出读书页数。", "solution": "null", "level": "三年级", "question": "妈妈陪琪琪读《米小圈》, 每天读 17 页, 读了 16 天, 还剩下 45 页没有读。这本书一共有多少页 $?$", "options": [], "subject": "算术", "analysis": "317 页\n\n【分析】用每天读书页数乘读书天数, 求出读书页数, 再加上没有读的页数, 求出这本书的总页数。\n\n【详解】 $17 \\times 16+45$\n\n$=272+45$\n\n$=317$ (页)\n\n答: 这本书一共有 317 页。\n\n【点睛】本题关键是根据工作总量 $=$ 工作效率 $\\times$ 工作时间, 求出读书页数。"} {"id": "8493", "image": [], "answer": "2511 个\n\n【分析】根据 9 天加工 729 个机器零件, 用 729 除以 9 即可算出每天加工的个数, 再根据 3 月份有 31 天, 用每天加工的个数乘 31 天即可求解。\n\n【详解】 $729 \\div 9=81$ (个)\n\n$81 \\times 31=2511($ 个 $)$\n\n答: 3 月份这一个月可以加工 2511 个零件。\n\n【点睛】此题主要考查三位数除以一位数以及两位数乘两位数的实际运用。", "solution": "null", "level": "三年级", "question": "某工厂 9 天加工 729 个机器零件。照这样计算, 3 月份这一个月可以加工多少个零件?", "options": [], "subject": "算术", "analysis": "2511 个\n\n【分析】根据 9 天加工 729 个机器零件, 用 729 除以 9 即可算出每天加工的个数, 再根据 3 月份有 31 天, 用每天加工的个数乘 31 天即可求解。\n\n【详解】 $729 \\div 9=81$ (个)\n\n$81 \\times 31=2511($ 个 $)$\n\n答: 3 月份这一个月可以加工 2511 个零件。\n\n【点睛】此题主要考查三位数除以一位数以及两位数乘两位数的实际运用。"} {"id": "8494", "image": [], "answer": "4320 元\n\n【分析】用每个保温壸卖的钱数乘 12 , 计算出每箱保温壼卖的钱数, 再用每箱保温壸卖的钱数乘 8 ,计算出超市一周一共卖了多少钱, 据此解答。\n\n【详解】 $45 \\times 12 \\times 8$\n\n$=540 \\times 8$\n\n$=4320($ 元 $)$\n\n答: 一共卖了 4320 元钱。\n\n【点睛】解答本题的关键先用乘法计算出每箱保温壸卖的钱数, 再用乘法计算出超市一周一共卖了多少钱。", "solution": "null", "level": "三年级", "question": "超市一周卖出 8 箱保温壼, 每箱 12 个, 每个保温壼卖 45 元。一共卖了多少钱?", "options": [], "subject": "算术", "analysis": "4320 元\n\n【分析】用每个保温壸卖的钱数乘 12 , 计算出每箱保温壼卖的钱数, 再用每箱保温壸卖的钱数乘 8 ,计算出超市一周一共卖了多少钱, 据此解答。\n\n【详解】 $45 \\times 12 \\times 8$\n\n$=540 \\times 8$\n\n$=4320($ 元 $)$\n\n答: 一共卖了 4320 元钱。\n\n【点睛】解答本题的关键先用乘法计算出每箱保温壸卖的钱数, 再用乘法计算出超市一周一共卖了多少钱。"} {"id": "8495", "image": [], "answer": "1800 箱 $; 3000$ 元\n\n【分析】根据题意, 用一辆车每次运桔子的箱数乘每天运送的次数, 求出一天运的箱数, 再乘 4 , 即\n可求出王爷爷家一共收了多少箱桔子;\n\n根据题意, 用每天运送的次数乘 4 天, 求出 4 天共运送的次数, 再乘每次的运费, 即可求出王爷爷家共要付多少元钱运费。\n\n【详解】 $30 \\times 15 \\times 4=1800$ (箱)\n\n$15 \\times 4 \\times 50=3000($ 元 $)$\n\n答: 王爷爷家一共收了 1800 箱桔子,共要付 3000 元钱运费\n\n【点睛】熟练掌握两位数乘两位数的计算方法, 是解答此题的关键。", "solution": "null", "level": "三年级", "question": "王爷爷家的桔子丰收了, 要把它们全部运走。王爷爷找了一辆一次能运 30 箱的货车, 每天运 15 次, 4 天才能全部运完。王爷爷家一共收了多少箱桔子? 如果每次的运费是 50 元, 王爷爷家共要付多少元钱运费?", "options": [], "subject": "算术", "analysis": "1800 箱 $; 3000$ 元\n\n【分析】根据题意, 用一辆车每次运桔子的箱数乘每天运送的次数, 求出一天运的箱数, 再乘 4 , 即\n可求出王爷爷家一共收了多少箱桔子;\n\n根据题意, 用每天运送的次数乘 4 天, 求出 4 天共运送的次数, 再乘每次的运费, 即可求出王爷爷家共要付多少元钱运费。\n\n【详解】 $30 \\times 15 \\times 4=1800$ (箱)\n\n$15 \\times 4 \\times 50=3000($ 元 $)$\n\n答: 王爷爷家一共收了 1800 箱桔子,共要付 3000 元钱运费\n\n【点睛】熟练掌握两位数乘两位数的计算方法, 是解答此题的关键。"} {"id": "9152", "image": [], "answer": "解(1) $10 ;$ (2) 3 ; (3)70 元\n\n【分析】(1)根据表格中的数据可以计算出一月份和二月份的平均水价, 然后用这个平均书价乘 11 与五月份的水费比较大小,即可得到规定吨数;\n\n(2)根据(1)中得到的规定吨数,用 24 $\\div 8$ 计算即可得到基本标准是每吨收费多少元;\n\n(3)根据(1)中的结果和(2)的结果, 可以计算出李老师家七月份用水 20 吨, 应缴水费多少元。\n\n【详解】(1)一月份每吨水的价格为: $24 \\div 8=3$ (元)\n\n二月份每吨水的价格为: $30 \\div 10=3$ (元)\n\n五月份用水 11 吨,如果按照标准缴费 $11 \\times 3=33 ($ 元)\n\n因为 $33<34$,\n\n所以每月用水量的规定吨数是 10 吨;\n\n(2) $24 \\div 8=3($ 元 $)$\n\n答: 基本标准是每吨收费 3 元;\n\n(3)因为 10 吨是标准,\n\n所以超出 10 吨后的每吨水的价格为: $34-10 \\times 3=4$ (元)\n\n$10 \\times 3+(20-10) \\times 4$\n\n$=30+10 \\times 4$\n\n$=30+40$\n\n$=70 ($ 元)\n\n答:李老师家七月份用水 20 吨,应缴水费 70 元。\n\n【点睛】此题主要考查从统计图表中获取信息, 从统计图表中获取解答问题的信息是解答本题的关键。", "solution": "null", "level": "三年级", "question": "某市自来水公司为鼓励居民节约用水, 对用水量采取按月分段计费的方法收取水费, 用水量在规定吨数以内的按基本标准收费, 超过规定吨数的部分提高收费标准。下面是李老师家一至六月份用水量和缴纳水费的情况:\n\n| 月份 | 一月 | 二月 | 三月 | 四月 | 五月 | 六月 |\n| :---: | :---: | :---: | :---: | :---: | :---: | :---: |\n| 用水量 (吨) | 8 | 10 | 12 | 14 | 11 | 13 |\n| 应缴水费 (元) | 24 | 30 | 38 | 46 | 34 | 42 |\n\n根据表中提供的信息,回答下面的问题。\n\n(1)每月用水量的规定吨数是吨。\n\n(2)基本标准是每吨收费 \\$ \\qquad \\$元。\n\n(3)李老师家七月份用水 20 吨, 应缴水费多少元?", "options": [], "subject": "算术", "analysis": "解(1) $10 ;$ (2) 3 ; (3)70 元\n\n【分析】(1)根据表格中的数据可以计算出一月份和二月份的平均水价, 然后用这个平均书价乘 11 与五月份的水费比较大小,即可得到规定吨数;\n\n(2)根据(1)中得到的规定吨数,用 24 $\\div 8$ 计算即可得到基本标准是每吨收费多少元;\n\n(3)根据(1)中的结果和(2)的结果, 可以计算出李老师家七月份用水 20 吨, 应缴水费多少元。\n\n【详解】(1)一月份每吨水的价格为: $24 \\div 8=3$ (元)\n\n二月份每吨水的价格为: $30 \\div 10=3$ (元)\n\n五月份用水 11 吨,如果按照标准缴费 $11 \\times 3=33 ($ 元)\n\n因为 $33<34$,\n\n所以每月用水量的规定吨数是 10 吨;\n\n(2) $24 \\div 8=3($ 元 $)$\n\n答: 基本标准是每吨收费 3 元;\n\n(3)因为 10 吨是标准,\n\n所以超出 10 吨后的每吨水的价格为: $34-10 \\times 3=4$ (元)\n\n$10 \\times 3+(20-10) \\times 4$\n\n$=30+10 \\times 4$\n\n$=30+40$\n\n$=70 ($ 元)\n\n答:李老师家七月份用水 20 吨,应缴水费 70 元。\n\n【点睛】此题主要考查从统计图表中获取信息, 从统计图表中获取解答问题的信息是解答本题的关键。"} {"id": "8509", "image": [], "answer": "V\n\n【分析】用两个因数个位上的数字相乘, 得到乘积的个位数就是 $\\square 2 \\times \\square 4$ 的积的个位上的数字。\n\n【详解】 $2 \\times 4=8$, 所以 $2 \\times \\square 4$ 的积的个位上一定是 8 。\n\n故答案为: $\\sqrt{ } \\sqrt{ }$\n\n【点睛】两个整数相乘, 两个因数个位上数字相乘的积的个位数就是两因数积的个位数字。", "solution": "null", "level": "三年级", "question": "$\\square 2 \\times \\square 4$ 积的个位上一定是 8 。( )", "options": [], "subject": "算术", "analysis": "V\n\n【分析】用两个因数个位上的数字相乘, 得到乘积的个位数就是 $\\square 2 \\times \\square 4$ 的积的个位上的数字。\n\n【详解】 $2 \\times 4=8$, 所以 $2 \\times \\square 4$ 的积的个位上一定是 8 。\n\n故答案为: $\\sqrt{ } \\sqrt{ }$\n\n【点睛】两个整数相乘, 两个因数个位上数字相乘的积的个位数就是两因数积的个位数字。"} {"id": "8510", "image": [], "answer": "$\\sqrt{ }$\n\n【分析】最大的两位数是 99 , 最小的两位数是 10 , 求出它们的积可确定积是几位数。\n\n【详解】 $99 \\times 10=990$, 积是三位数。\n\n故答案为: $\\sqrt{ } \\sqrt{ }$\n\n【点睛】本题找出最大和最小的两位数, 求出积再进行解答。", "solution": "null", "level": "三年级", "question": "最大的两位数乘最小的两位数积一定是三位数。( )", "options": [], "subject": "算术", "analysis": "$\\sqrt{ }$\n\n【分析】最大的两位数是 99 , 最小的两位数是 10 , 求出它们的积可确定积是几位数。\n\n【详解】 $99 \\times 10=990$, 积是三位数。\n\n故答案为: $\\sqrt{ } \\sqrt{ }$\n\n【点睛】本题找出最大和最小的两位数, 求出积再进行解答。"} {"id": "8511", "image": [], "answer": "V\n\n【分析】先用每个篮球的价钱乘篮球的个数就是需要的钱, 然后用买 20 个篮球需要的钱与 800 元比较即可。\n\n【详解】 $38 \\times 20=760$ (元)\n\n760 元 $<800$ 元, 因此带 800 元够。\n\n故答案为: $\\sqrt{ }$\n\n【点睛】此题考查的是经济问题的计算, 先计算出买 20 个篮球需要的钱是解答此题的关键。", "solution": "null", "level": "三年级", "question": "每个篮球 38 元, 李老师想买 20 个, 她带 800 元钱够了。( )", "options": [], "subject": "算术", "analysis": "V\n\n【分析】先用每个篮球的价钱乘篮球的个数就是需要的钱, 然后用买 20 个篮球需要的钱与 800 元比较即可。\n\n【详解】 $38 \\times 20=760$ (元)\n\n760 元 $<800$ 元, 因此带 800 元够。\n\n故答案为: $\\sqrt{ }$\n\n【点睛】此题考查的是经济问题的计算, 先计算出买 20 个篮球需要的钱是解答此题的关键。"} {"id": "8512", "image": [], "answer": "$\\sqrt{ }$\n\n【分析】两位数乘两位数的计算方法: 先是第二个乘数的个位与第一个乘数相乘; 接着第二个乘数的十位与第一个乘数相乘, 最后把两次乘得的积相加; 可采用试一试的方法, 依此将数字放入口里计算出结果再判断。\n\n【详解】 $14 \\times 29=406$;\n\n$24 \\times 29=696$;\n\n$34 \\times 29=986$;\n\n$44 \\times 29=1276$;\n\n因此 $\\square 4 \\times 29$ 的积是四位数, 口里最小填 4 .\n故答案为: $\\sqrt{ }$\n\n【点睛】熟练掌握两位数乘两位数的计算是解答此题的关键。", "solution": "null", "level": "三年级", "question": "$\\square 4 \\times 29$ 的积是四位数, 哩最小填 4 。 ( )", "options": [], "subject": "算术", "analysis": "$\\sqrt{ }$\n\n【分析】两位数乘两位数的计算方法: 先是第二个乘数的个位与第一个乘数相乘; 接着第二个乘数的十位与第一个乘数相乘, 最后把两次乘得的积相加; 可采用试一试的方法, 依此将数字放入口里计算出结果再判断。\n\n【详解】 $14 \\times 29=406$;\n\n$24 \\times 29=696$;\n\n$34 \\times 29=986$;\n\n$44 \\times 29=1276$;\n\n因此 $\\square 4 \\times 29$ 的积是四位数, 口里最小填 4 .\n故答案为: $\\sqrt{ }$\n\n【点睛】熟练掌握两位数乘两位数的计算是解答此题的关键。"} {"id": "8513", "image": [], "answer": "$\\times$\n\n【分析】用 24 乘 18 计算出 18 个箱子可以装饮料的瓶数, 然后与 450 瓶比较即可。\n\n【详解】 $24 \\times 18=432$ (瓶)\n\n432 瓶<450 瓶, 因此 18 个箱子不能装完。\n\n故答案为: $\\times$\n\n【点睛】熟练掌握两位数乘两位数的计算是解答此题的关键。", "solution": "null", "level": "三年级", "question": "商场有 450 瓶饮料, 每箱装 24 瓶, 需 18 个箱子才能装完。( )", "options": [], "subject": "算术", "analysis": "$\\times$\n\n【分析】用 24 乘 18 计算出 18 个箱子可以装饮料的瓶数, 然后与 450 瓶比较即可。\n\n【详解】 $24 \\times 18=432$ (瓶)\n\n432 瓶<450 瓶, 因此 18 个箱子不能装完。\n\n故答案为: $\\times$\n\n【点睛】熟练掌握两位数乘两位数的计算是解答此题的关键。"} {"id": "8518", "image": [], "answer": "156 元\n\n【分析】全年有 12 个月, 用有线电视维护费每月费用乘 12, 就可以算出有线电视维护费全年要多少钱。\n\n【详解】 $13 \\times 12=156$ (元)\n\n答: 有线电视维护费全年要 156 元钱。\n\n【点睛】此题需要学生熟练掌握两位数乘两位数的计算方法并灵活运用, 关键是根据问题选择正确的条件列式解答。", "solution": "null", "level": "三年级", "question": "订一份电视节目报每个季度 10 元, 有线电视维护费每月 13 元。有线电视维护费全年要多少钱?", "options": [], "subject": "算术", "analysis": "156 元\n\n【分析】全年有 12 个月, 用有线电视维护费每月费用乘 12, 就可以算出有线电视维护费全年要多少钱。\n\n【详解】 $13 \\times 12=156$ (元)\n\n答: 有线电视维护费全年要 156 元钱。\n\n【点睛】此题需要学生熟练掌握两位数乘两位数的计算方法并灵活运用, 关键是根据问题选择正确的条件列式解答。"} {"id": "8519", "image": [], "answer": "不够\n\n【分析】根据题意, 可用公式单价 $\\times$ 数量 $=$ 总价分别计算出老师和学生的钱数, 然后相加即可得到需要的总钱数, 最后再与 1000 元相比较即可得解。\n\n【详解】 $45 \\times 3+25 \\times 35$\n\n$=135+875$\n\n$=1010($ 元)\n\n1010 元 $>1000$ 元\n\n答: 带 1000 元不够。\n\n【点睛】此题主要考查的是单价、数量和总价三者之间的数量关系的灵活应用。", "solution": "null", "level": "三年级", "question": "夏令营到了, 3 名老师带领 35 名学生去游乐场游玩, 带 1000 元够吗?\n\n儿童票: 25 元
成人票: 45 元", "options": [], "subject": "算术", "analysis": "不够\n\n【分析】根据题意, 可用公式单价 $\\times$ 数量 $=$ 总价分别计算出老师和学生的钱数, 然后相加即可得到需要的总钱数, 最后再与 1000 元相比较即可得解。\n\n【详解】 $45 \\times 3+25 \\times 35$\n\n$=135+875$\n\n$=1010($ 元)\n\n1010 元 $>1000$ 元\n\n答: 带 1000 元不够。\n\n【点睛】此题主要考查的是单价、数量和总价三者之间的数量关系的灵活应用。"} {"id": "8520", "image": [], "answer": "3200 千克\n\n【分析】先用乘法计算出每天生产豆腐干需要的黄豆千克数, 再乘 1 千克黄豆可以生产的豆腐干千克数即可。\n\n【详解】 $16 \\times 50 \\times 4$\n\n$=800 \\times 4$\n\n$=3200$ (千克)\n\n答:这个厂一天共生产豆腐干 3200 千克\n\n【点睛】此题考查灵活掌握两位数乘法, 并能解决生活问题。", "solution": "null", "level": "三年级", "question": "佳龙豆腐干厂每天生产豆腐干需要用黄豆 16 袋, 每袋 50 千克。 1 千克黄豆可以生产豆腐干 4 千克。这个厂一天共生产豆腐干多少千克?", "options": [], "subject": "算术", "analysis": "3200 千克\n\n【分析】先用乘法计算出每天生产豆腐干需要的黄豆千克数, 再乘 1 千克黄豆可以生产的豆腐干千克数即可。\n\n【详解】 $16 \\times 50 \\times 4$\n\n$=800 \\times 4$\n\n$=3200$ (千克)\n\n答:这个厂一天共生产豆腐干 3200 千克\n\n【点睛】此题考查灵活掌握两位数乘法, 并能解决生活问题。"} {"id": "8521", "image": [], "answer": "216 万件\n【分析】根据 5 个月生产服装 90 万件, 先用除法计算出一个月生产的件数, 再用乘法计算出一年可以生产多少万件, 据此解答。\n\n【详解】 $90 \\div 5=18$ (万件)\n\n$18 \\times 12=216$ (万件)\n\n答: 一年可以生产服装 216 万件。\n\n【点睛】此题考查乘除法的简单应用, 解答本体的关键先求出一天生产的数量, 然后求出一年的生产数量。", "solution": "null", "level": "三年级", "question": "服装厂今年前 5 个月生产服装 90 万件。照这样计算,一年可以生产服装多少万件?", "options": [], "subject": "算术", "analysis": "216 万件\n【分析】根据 5 个月生产服装 90 万件, 先用除法计算出一个月生产的件数, 再用乘法计算出一年可以生产多少万件, 据此解答。\n\n【详解】 $90 \\div 5=18$ (万件)\n\n$18 \\times 12=216$ (万件)\n\n答: 一年可以生产服装 216 万件。\n\n【点睛】此题考查乘除法的简单应用, 解答本体的关键先求出一天生产的数量, 然后求出一年的生产数量。"} {"id": "8522", "image": [], "answer": "够; 51 个\n\n【分析】每人做的个数乘人数求出做的小灯笼个数, 再与 200 进行比较即可解答; 再把做的小灯笼个数除以班数即得每个班可以得到的个数, 据此即可解答。\n\n【详解】 $12 \\times 17=204$ (个) $>200$ 个, 够。\n\n$204 \\div 4=51($ 个)\n\n答: 每人做 12 个够了, 每个班可以得到 51 个。\n\n【点睛】本题主要考查学生对整数乘除法计算方法的掌握。", "solution": "null", "level": "三年级", "question": "四年级有 17 个同学准备制作 200 个小灯笼庆祝“六一”, 每人做 12 个够吗?如果把做成的这些小灯笼平均分给四年级的 4 个班, 每个班可以得到多少个?", "options": [], "subject": "算术", "analysis": "够; 51 个\n\n【分析】每人做的个数乘人数求出做的小灯笼个数, 再与 200 进行比较即可解答; 再把做的小灯笼个数除以班数即得每个班可以得到的个数, 据此即可解答。\n\n【详解】 $12 \\times 17=204$ (个) $>200$ 个, 够。\n\n$204 \\div 4=51($ 个)\n\n答: 每人做 12 个够了, 每个班可以得到 51 个。\n\n【点睛】本题主要考查学生对整数乘除法计算方法的掌握。"} {"id": "8536", "image": [], "answer": "V", "solution": "null", "level": "三年级", "question": "口算 $15 \\times 6$ 时, 可以先算 $10 \\times 6=60$. 再算 $5 \\times 6=30,60+30=90$, 所以 $15 \\times 60=900$ ( )", "options": [], "subject": "算术", "analysis": "V"} {"id": "8537", "image": [], "answer": "$\\times$\n\n【详解】口算 $46 \\times 50$ 可以先算 $46 \\times 5=230$, 再算 $230 \\times 10=2300$, 所以 $46 \\times 50=2300$", "solution": "null", "level": "三年级", "question": "$46 \\times 50=230$.", "options": [], "subject": "算术", "analysis": "$\\times$\n\n【详解】口算 $46 \\times 50$ 可以先算 $46 \\times 5=230$, 再算 $230 \\times 10=2300$, 所以 $46 \\times 50=2300$"} {"id": "8538", "image": [], "answer": "$\\times$", "solution": "null", "level": "三年级", "question": "两个数的积一定不等于这两个数的和.", "options": [], "subject": "算术", "analysis": "$\\times$"} {"id": "8539", "image": [], "answer": "V", "solution": "null", "level": "三年级", "question": "$2 \\square \\times 3 \\square=600$, $\\square$ 里是同一个数字,只能 $0 . \\quad(\\quad)$", "options": [], "subject": "算术", "analysis": "V"} {"id": "8540", "image": [], "answer": "$\\sqrt{ }$", "solution": "null", "level": "三年级", "question": "一个乘数扩大 10 倍, 另一个乘数缩小 10 倍, 它们的积不变. ( )", "options": [], "subject": "算术", "analysis": "$\\sqrt{ }$"} {"id": "8544", "image": [], "answer": "不够\n\n【分析】每副乒乓球拍 49 元, 买 15 副需要的钱数就是 15 个 49 元, 用 49 乘 15 即可求出需要的钱数,再与 600 元比较即可求解。\n\n【详解】 $49 \\times 15=735 ($ 元)\n\n$600<735$\n\n答:杨老师带的钱不够。\n\n【点睛】本题考查了基本的数量关系:总价 $=$ 单价 $\\times$ 数量。", "solution": "null", "level": "三年级", "question": "惠民商店每副乒乓球拍 49 元, 杨老师准备给学校乒乓球队买 15 副乒乓球拍, 他带了 600 元钱,杨老师带的钱够吗?", "options": [], "subject": "算术", "analysis": "不够\n\n【分析】每副乒乓球拍 49 元, 买 15 副需要的钱数就是 15 个 49 元, 用 49 乘 15 即可求出需要的钱数,再与 600 元比较即可求解。\n\n【详解】 $49 \\times 15=735 ($ 元)\n\n$600<735$\n\n答:杨老师带的钱不够。\n\n【点睛】本题考查了基本的数量关系:总价 $=$ 单价 $\\times$ 数量。"} {"id": "8545", "image": [], "answer": "1650 千克\n\n【分析】用每筐苹果的重量加每筐橘子的重量, 求出一筐苹果和一筐橘子共重多少千克, 再乘 30,\n就是两种水果共重多少千克, 据此解答。\n\n【详解】 $(25+30) \\times 30$\n\n$=55 \\times 30$\n\n$=1650$ (千克)\n\n答: 这两种水果共重 1650 千克。\n\n【点睛】解决本题也可以分别求出苹果、橘子的总重量, 再把它们加起来就是这两种水果的总重量,列式为: $25 \\times 30+30 \\times 30$ 。", "solution": "null", "level": "三年级", "question": "商店运来苹果橘子各 30 筐。已知每筐苹果重 25 千克, 每筐橘子重 30 千克。这两种水果共重多少千克?", "options": [], "subject": "算术", "analysis": "1650 千克\n\n【分析】用每筐苹果的重量加每筐橘子的重量, 求出一筐苹果和一筐橘子共重多少千克, 再乘 30,\n就是两种水果共重多少千克, 据此解答。\n\n【详解】 $(25+30) \\times 30$\n\n$=55 \\times 30$\n\n$=1650$ (千克)\n\n答: 这两种水果共重 1650 千克。\n\n【点睛】解决本题也可以分别求出苹果、橘子的总重量, 再把它们加起来就是这两种水果的总重量,列式为: $25 \\times 30+30 \\times 30$ 。"} {"id": "8546", "image": [], "answer": "1620 袋\n\n【分析】根据题意, 分别计算大车所运袋数: $80 \\times 15=1200$ (袋), 小车所运袋数: $35 \\times 12=420$ (袋),再求和即可。\n\n【详解】 $80 \\times 15+35 \\times 12$\n\n$=1200+420$\n\n$=1620$ (袋)\n\n答: 这批面粉共 1620 袋。\n\n【点睛】本题主要考查整数乘法的应用, 关键是求大车和小车所运袋数。", "solution": "null", "level": "三年级", "question": "一辆大车和一辆小车运面粉。大车每车能运 80 袋, 小车每车能运 35 袋。大车运了 15 次, 小车运了 12 次刚好把这批面粉运完。这批面粉共多少袋?", "options": [], "subject": "算术", "analysis": "1620 袋\n\n【分析】根据题意, 分别计算大车所运袋数: $80 \\times 15=1200$ (袋), 小车所运袋数: $35 \\times 12=420$ (袋),再求和即可。\n\n【详解】 $80 \\times 15+35 \\times 12$\n\n$=1200+420$\n\n$=1620$ (袋)\n\n答: 这批面粉共 1620 袋。\n\n【点睛】本题主要考查整数乘法的应用, 关键是求大车和小车所运袋数。"} {"id": "8547", "image": [], "answer": "864 元\n\n【分析】 216 除以 3 等于兴欣家一个月的电费, 再乘 12 即等于兴欣家一年的电费, 据此即可解答。\n\n【详解】 $216 \\div 3 \\times 12$\n\n$=72 \\times 12$\n\n$=864($ 元 $)$\n\n答: 兴欣家一年交电费 864 元\n\n【点睛】本题是归一问题应用题, 先求出一个月的电费, 再求一年的电费。", "solution": "null", "level": "三年级", "question": "兴欣家今年前 3 个月用电 360 度, 交电费 216 元, 照这样计算, 兴欣家一年交电费多少元?", "options": [], "subject": "算术", "analysis": "864 元\n\n【分析】 216 除以 3 等于兴欣家一个月的电费, 再乘 12 即等于兴欣家一年的电费, 据此即可解答。\n\n【详解】 $216 \\div 3 \\times 12$\n\n$=72 \\times 12$\n\n$=864($ 元 $)$\n\n答: 兴欣家一年交电费 864 元\n\n【点睛】本题是归一问题应用题, 先求出一个月的电费, 再求一年的电费。"} {"id": "8548", "image": [], "answer": "1875 千克\n\n【分析】照这样计算, 说明每箱蜜蜂一年可以酿蜂蜜的质量都相等, 先用 525 千克除以 7 箱, 求出 1 箱蜜蜂一年可以酿多少千克, 再乘 25 即可。\n\n【详解】 $525 \\div 7 \\times 25$\n\n$=75 \\times 25$\n\n$=1875$ (千克)\n\n答: 25 箱蜜蜂一年可以酿 1875 千克蜂蜜。\n\n【点睛】解答此题的关键是先求得单一量, 再由不变的单一量求得总量。", "solution": "null", "level": "三年级", "question": "7 箱蜜蜂一年可以酿 525 千克蜂蜜, 照这样计算, 25 箱蜜蜂一年可以酿多少千克蜂蜜?", "options": [], "subject": "算术", "analysis": "1875 千克\n\n【分析】照这样计算, 说明每箱蜜蜂一年可以酿蜂蜜的质量都相等, 先用 525 千克除以 7 箱, 求出 1 箱蜜蜂一年可以酿多少千克, 再乘 25 即可。\n\n【详解】 $525 \\div 7 \\times 25$\n\n$=75 \\times 25$\n\n$=1875$ (千克)\n\n答: 25 箱蜜蜂一年可以酿 1875 千克蜂蜜。\n\n【点睛】解答此题的关键是先求得单一量, 再由不变的单一量求得总量。"} {"id": "8562", "image": [], "answer": "$\\times$\n\n【分析】三位数除以一位数的除法法则: 从被除数的最高位除起, 如果最高位不够商 1 , 就看前两位;除到被除数的哪一位, 就把商写在那一位上面, 如果不够商 1 , 就在这一位商 0 ; 每次除得的余数必须比除数小。据此计算出 200 除以 3 的商即可判断。\n\n【详解】 $200 \\div 3=66 \\ldots . . .2$\n\n所以, $200 \\div 3$ 的商的末尾一定有一个 0 , 是错误的;\n\n故答案为: $\\times$\n\n【点睛】熟练掌握三位数除以一位数的计算方法, 是解答此题的关键。", "solution": "null", "level": "三年级", "question": "$200 \\div 3$ 的商的末尾一定有一个 0 。 $(\\quad)$", "options": [], "subject": "算术", "analysis": "$\\times$\n\n【分析】三位数除以一位数的除法法则: 从被除数的最高位除起, 如果最高位不够商 1 , 就看前两位;除到被除数的哪一位, 就把商写在那一位上面, 如果不够商 1 , 就在这一位商 0 ; 每次除得的余数必须比除数小。据此计算出 200 除以 3 的商即可判断。\n\n【详解】 $200 \\div 3=66 \\ldots . . .2$\n\n所以, $200 \\div 3$ 的商的末尾一定有一个 0 , 是错误的;\n\n故答案为: $\\times$\n\n【点睛】熟练掌握三位数除以一位数的计算方法, 是解答此题的关键。"} {"id": "8563", "image": [], "answer": "V\n\n【分析】根据在有余数的除法中, 余数总比除数小, 然后列举出所有的余数, 继而判断即可。\n\n【详解】 除数是 8 ,余数可能是 $1 、 2 、 3 、 4 、 5 、 6 、 7$; 也就是余数可能是 7 这种说法正确。故答案为: $V$ 。\n\n【点睛】本题考查有余数除法中余数和除数的关系, 关键是明确余数总比除数小。", "solution": "null", "level": "三年级", "question": "一个数除以 8 , 有余数, 那么余数可能是 7 。( )", "options": [], "subject": "算术", "analysis": "V\n\n【分析】根据在有余数的除法中, 余数总比除数小, 然后列举出所有的余数, 继而判断即可。\n\n【详解】 除数是 8 ,余数可能是 $1 、 2 、 3 、 4 、 5 、 6 、 7$; 也就是余数可能是 7 这种说法正确。故答案为: $V$ 。\n\n【点睛】本题考查有余数除法中余数和除数的关系, 关键是明确余数总比除数小。"} {"id": "8564", "image": [], "answer": "$\\times$\n\n【分析】 $240 \\div 2=120,210 \\div 7=30$, 所以三位数除以一位数, 商最多是三位数, 最少是两位数, 据此即可解答。\n\n【详解】根据分析可知, 三位数除以一位数, 商最多是三位数, 最少是两位数, 所以判断错误。故答案为: $\\times$\n\n【点睛】本题可以通过直接举例说明原说法错误。", "solution": "null", "level": "三年级", "question": "三位数除以一位数, 商最多是两位数。( )", "options": [], "subject": "算术", "analysis": "$\\times$\n\n【分析】 $240 \\div 2=120,210 \\div 7=30$, 所以三位数除以一位数, 商最多是三位数, 最少是两位数, 据此即可解答。\n\n【详解】根据分析可知, 三位数除以一位数, 商最多是三位数, 最少是两位数, 所以判断错误。故答案为: $\\times$\n\n【点睛】本题可以通过直接举例说明原说法错误。"} {"id": "8565", "image": [], "answer": "$\\times$\n\n【分析】分别求出两个算式的结果, 再比较判断即可。\n\n【详解】 $24 \\div 4 \\div 2$\n\n$=6 \\div 2$\n\n$=3$\n\n24: $(4 \\div 2)$\n$=24 \\div 2$\n\n$=12$\n\n$3 \\neq 12$\n\n所以原题说法错误。\n\n故答案为: $\\times$\n\n【点睛】考查了整数的四则混合运算, 注意运算顺序和运算法则, 然后再进一步计算。", "solution": "null", "level": "三年级", "question": "$24 \\div 4 \\div 2$ 的结果和 $24 \\div(4 \\div 2)$ 的结果相同。( $)$", "options": [], "subject": "算术", "analysis": "$\\times$\n\n【分析】分别求出两个算式的结果, 再比较判断即可。\n\n【详解】 $24 \\div 4 \\div 2$\n\n$=6 \\div 2$\n\n$=3$\n\n24: $(4 \\div 2)$\n$=24 \\div 2$\n\n$=12$\n\n$3 \\neq 12$\n\n所以原题说法错误。\n\n故答案为: $\\times$\n\n【点睛】考查了整数的四则混合运算, 注意运算顺序和运算法则, 然后再进一步计算。"} {"id": "8566", "image": [], "answer": "$\\times$\n\n【详解】 0 加任何数都得任何数; 任何数减 0 都得原数; 0 乘任何数都得 $0 ; 0$ 除以任何一个不是 0 的数都得 0 ; 原题中说任何数除以 0 也得 0,0 不能作除数, 所以判断错误。\n\n故答案为: $\\times$", "solution": "null", "level": "三年级", "question": "任何数乘 0 都得 0 , 任何数除以 0 也得 0 。( )", "options": [], "subject": "算术", "analysis": "$\\times$\n\n【详解】 0 加任何数都得任何数; 任何数减 0 都得原数; 0 乘任何数都得 $0 ; 0$ 除以任何一个不是 0 的数都得 0 ; 原题中说任何数除以 0 也得 0,0 不能作除数, 所以判断错误。\n\n故答案为: $\\times$"} {"id": "8571", "image": [], "answer": "36 个\n\n【分析】水果商店购进苹果 288 个, 平均装在 8 个纸箱里, 求平均每个纸箱的数量, 用除法即可解答。\n\n【详解】 $288 \\div 8=36$ (个)\n\n答: 每个纸箱装 36 个。\n\n【点睛】此题主要考查了学生对除法的理解与实际应用。", "solution": "null", "level": "三年级", "question": "水果商店购进苹果 288 个, 平均装在 8 个纸箱里。每个纸箱装多少个?", "options": [], "subject": "算术", "analysis": "36 个\n\n【分析】水果商店购进苹果 288 个, 平均装在 8 个纸箱里, 求平均每个纸箱的数量, 用除法即可解答。\n\n【详解】 $288 \\div 8=36$ (个)\n\n答: 每个纸箱装 36 个。\n\n【点睛】此题主要考查了学生对除法的理解与实际应用。"} {"id": "8572", "image": [], "answer": "30 元\n\n【分析】根据题意, 要求平均每小时收入多少钱, 先用总收入除以天数, 得到平均每天的收入钱数,再用每天的收入钱数除以每天的工作时间, 就可以得到平均每小时收入的钱数。\n\n\n\n$1260 \\div 7 \\div 6$\n\n$=180 \\div 6$\n\n$=30 ($ 元 $)$\n\n答: 平均每小时收入 30 元钱。\n\n【点睛】本题考查了用除法解决实际问题。", "solution": "null", "level": "三年级", "question": "一辆出租车一周 (7 天) 收入 1260 元钱, 如果每天工作 6 小时, 平均每小时收入多少钱?", "options": [], "subject": "算术", "analysis": "30 元\n\n【分析】根据题意, 要求平均每小时收入多少钱, 先用总收入除以天数, 得到平均每天的收入钱数,再用每天的收入钱数除以每天的工作时间, 就可以得到平均每小时收入的钱数。\n\n\n\n$1260 \\div 7 \\div 6$\n\n$=180 \\div 6$\n\n$=30 ($ 元 $)$\n\n答: 平均每小时收入 30 元钱。\n\n【点睛】本题考查了用除法解决实际问题。"} {"id": "8573", "image": [], "answer": "30 本\n\n【分析】根据题意, 用图书的本数除以 6 , 求出每个年级分到书的本数, 再除以 5 , 即可求出平均每个班分到多少本书。\n\n【详解】 $900 \\div 6 \\div 5$\n\n$=150 \\div 5$\n\n$=30($ 本 $)$\n\n答: 平均每个班分到 30 本书。\n\n【点睛】求出光明小学每个年级分到书的本数, 是解答此题的关键。", "solution": "null", "level": "三年级", "question": "光明小学有 6 个年级, 每个年级有 5 个班, 学校购回 900 本图书分给每个班, 平均每个班分到多少本书?", "options": [], "subject": "算术", "analysis": "30 本\n\n【分析】根据题意, 用图书的本数除以 6 , 求出每个年级分到书的本数, 再除以 5 , 即可求出平均每个班分到多少本书。\n\n【详解】 $900 \\div 6 \\div 5$\n\n$=150 \\div 5$\n\n$=30($ 本 $)$\n\n答: 平均每个班分到 30 本书。\n\n【点睛】求出光明小学每个年级分到书的本数, 是解答此题的关键。"} {"id": "8574", "image": [], "answer": "11 元\n\n【分析】根据“单价=总价 : 数量”计算出一盒国画颜料的单价, 再减去一盒水彩画颜料的单价即可求解。\n【详解】 $416 \\div 4-93$\n\n$=104-93$\n\n$=11($ 元 $)$\n\n答: 一盒国画颜比一盒水彩颜料贵 11 元。\n\n【点睛】此题考查了整数除法的实际应用以及单价、数量和总价之间的数量关系。", "solution": "null", "level": "三年级", "question": "一盒水彩画颜料 93 元, 4 盒国画颜料 416 元, 一盒国画颜比一盒水彩颜料贵多少元?", "options": [], "subject": "算术", "analysis": "11 元\n\n【分析】根据“单价=总价 : 数量”计算出一盒国画颜料的单价, 再减去一盒水彩画颜料的单价即可求解。\n【详解】 $416 \\div 4-93$\n\n$=104-93$\n\n$=11($ 元 $)$\n\n答: 一盒国画颜比一盒水彩颜料贵 11 元。\n\n【点睛】此题考查了整数除法的实际应用以及单价、数量和总价之间的数量关系。"} {"id": "8575", "image": [], "answer": "(1) 165 颗;\n\n(2) 21 颗; 6 元\n\n【分析】(1) 求多少颗牛油果中含蛋白质 660 克, 也就是求 660 克里面有几个 4 克, 根据求一个数里面有几个另一个数, 用除法解答;\n\n(2)乐乐妈妈带了 174 元, 求最多可以买几颗? 还剩多少元, 也就是求 174 元里面有几个 8 元, 根据包含除法的意义,用除法解答即可。\n\n【详解】(1) $660 \\div 4=165$ (颗)\n\n答: 165 颗牛油果中含蛋白质 660 克;\n\n(2) $174 \\div 8=21$ (颗) $\\ldots . . . .6$ (元)\n\n答: 最多可以买 21 颗, 还剩 6 元。\n\n【点睛】本题考查整数的乘法和应用, 解决本题的关键是明确数量关系, 用到的知识点: 求一个数里面有几个另一个数, 用除法解答。", "solution": "null", "level": "三年级", "question": "牛油果营养丰富, 平均每颗牛油果中含蛋白质 4 克, 每颗价格 8 元。\n\n(1) 多少颗牛油果中含蛋白质 660 克?\n\n(2)乐乐妈妈带了 174 元,最多可以买几颗?还剩多少元?", "options": [], "subject": "算术", "analysis": "(1) 165 颗;\n\n(2) 21 颗; 6 元\n\n【分析】(1) 求多少颗牛油果中含蛋白质 660 克, 也就是求 660 克里面有几个 4 克, 根据求一个数里面有几个另一个数, 用除法解答;\n\n(2)乐乐妈妈带了 174 元, 求最多可以买几颗? 还剩多少元, 也就是求 174 元里面有几个 8 元, 根据包含除法的意义,用除法解答即可。\n\n【详解】(1) $660 \\div 4=165$ (颗)\n\n答: 165 颗牛油果中含蛋白质 660 克;\n\n(2) $174 \\div 8=21$ (颗) $\\ldots . . . .6$ (元)\n\n答: 最多可以买 21 颗, 还剩 6 元。\n\n【点睛】本题考查整数的乘法和应用, 解决本题的关键是明确数量关系, 用到的知识点: 求一个数里面有几个另一个数, 用除法解答。"} {"id": "8589", "image": [], "answer": "V\n\n【分析】 23 接近整十数 20,37 接近整十数 40 , 则估算 $23 \\times 37$ 时, 可以把 23 看作 20,37 看作 40 ,再进行计算。\n\n【详解】 $23 \\times 37$\n\n$\\approx 20 \\times 40$\n\n$=800$\n\n则在估算 $23 \\times 37$ 时, 可以把 23 看作 20,37 看作 40 。\n\n故答案为: $\\sqrt{ }$ 。\n\n【点睛】两位数乘两位数估算时, 将两个因数估成与其接近的整十数, 再进行计算。", "solution": "null", "level": "三年级", "question": "在估算 $23 \\times 37$ 时, 可以把 23 看作 20,37 看作 40 。 ( )", "options": [], "subject": "算术", "analysis": "V\n\n【分析】 23 接近整十数 20,37 接近整十数 40 , 则估算 $23 \\times 37$ 时, 可以把 23 看作 20,37 看作 40 ,再进行计算。\n\n【详解】 $23 \\times 37$\n\n$\\approx 20 \\times 40$\n\n$=800$\n\n则在估算 $23 \\times 37$ 时, 可以把 23 看作 20,37 看作 40 。\n\n故答案为: $\\sqrt{ }$ 。\n\n【点睛】两位数乘两位数估算时, 将两个因数估成与其接近的整十数, 再进行计算。"} {"id": "8590", "image": [], "answer": "$\\sqrt{ }$\n\n【分析】根据笔算法则相同数位对齐, 从个位乘起, 一位一位地乘, 一个因数依次去乘另一个因数的每一位即可解答。\n\n【详解】 $\\square 3 \\times \\square 7$ 相乘, $3 \\times 7=21$, 积的个位数字一定是 1 ;故答案为: $V_{0}$\n\n【点睛】本题掌握两位数乘两位数的笔算法则是解题关键。", "solution": "null", "level": "三年级", "question": "$\\square 3 \\times \\square 7$ 的积的个位数字一定是 1 。( )", "options": [], "subject": "算术", "analysis": "$\\sqrt{ }$\n\n【分析】根据笔算法则相同数位对齐, 从个位乘起, 一位一位地乘, 一个因数依次去乘另一个因数的每一位即可解答。\n\n【详解】 $\\square 3 \\times \\square 7$ 相乘, $3 \\times 7=21$, 积的个位数字一定是 1 ;故答案为: $V_{0}$\n\n【点睛】本题掌握两位数乘两位数的笔算法则是解题关键。"} {"id": "8591", "image": ["1349.jpg", "1349.jpg"], "answer": "$\\times$\n\n【分析】依据整数乘法的计算方法: 哪一位上乘得的积满几十, 就向前一位进几即可解答。\n\n【详解】由分析可得:\n\n哪一位上相乘的积满几十, 就向前一位进几, 例如:\n\n\n\n所以原题的说法判断错误。\n\n故答案为 $\\times$", "solution": "null", "level": "三年级", "question": "哪一位上相乘的积满几十,就向前一位进 1。( )", "options": [], "subject": "算术", "analysis": "$\\times$\n\n【分析】依据整数乘法的计算方法: 哪一位上乘得的积满几十, 就向前一位进几即可解答。\n\n【详解】由分析可得:\n\n哪一位上相乘的积满几十, 就向前一位进几, 例如:\n\n\n\n所以原题的说法判断错误。\n\n故答案为 $\\times$"} {"id": "8592", "image": [], "answer": "$\\sqrt{ }$\n\n【分析】计算因数末尾有 0 的乘法, 先把 0 前面的数相乘, 再看两个因数的末尾一共有几个 0 , 就在积的末尾添几个 0 。据此可知, $20 \\times 30=600$, 积的末尾有 2 个 0 。 $20 \\times 50=1000$, 积的末尾有 3 个 0 。据此判断即可。\n\n【详解】根据分析可知, 两个数相乘时, 两个因数的末尾有两个零, 积的末尾至少也有两个零。故答案为: V $\\sqrt{ }$ 。\n\n【点睛】本题考查因数末尾有 0 的乘法, 两个因数末尾有几个 0 , 积的末尾至少有相同个数的 0 。", "solution": "null", "level": "三年级", "question": "两个数相乘时, 两个因数的末尾有两个零, 积的末尾至少也有两个零。( )", "options": [], "subject": "算术", "analysis": "$\\sqrt{ }$\n\n【分析】计算因数末尾有 0 的乘法, 先把 0 前面的数相乘, 再看两个因数的末尾一共有几个 0 , 就在积的末尾添几个 0 。据此可知, $20 \\times 30=600$, 积的末尾有 2 个 0 。 $20 \\times 50=1000$, 积的末尾有 3 个 0 。据此判断即可。\n\n【详解】根据分析可知, 两个数相乘时, 两个因数的末尾有两个零, 积的末尾至少也有两个零。故答案为: V $\\sqrt{ }$ 。\n\n【点睛】本题考查因数末尾有 0 的乘法, 两个因数末尾有几个 0 , 积的末尾至少有相同个数的 0 。"} {"id": "8593", "image": [], "answer": "$\\sqrt{ }$\n\n【分析】根据整数乘法的计算法则计算出两个算式的结果比较即可。\n\n【详解】 $38 \\times 50=1900 ; 380 \\times 5=1900$, 结果相同, 原题正确, 故答案为: V、\n\n【点睛】本题考查乘法计算的应用, 认真计算即可。", "solution": "null", "level": "三年级", "question": "$38 \\times 50$ 与 $380 \\times 5$ 的结果是相同的。( )", "options": [], "subject": "算术", "analysis": "$\\sqrt{ }$\n\n【分析】根据整数乘法的计算法则计算出两个算式的结果比较即可。\n\n【详解】 $38 \\times 50=1900 ; 380 \\times 5=1900$, 结果相同, 原题正确, 故答案为: V、\n\n【点睛】本题考查乘法计算的应用, 认真计算即可。"} {"id": "8598", "image": [], "answer": "5642 元\n\n【分析】用 43 加上 48 求出 1 套的单价, 然后再根据“单价 $\\times$ 数量 $=$ 总价”解答即可。\n\n【详解】 $43+48=91($ 元 $)$\n\n$91 \\times 62=5642 ($ 元 $)$\n\n答: 一共需要 5642 元。\n\n【点睛】解答此题应根据总价、数量和单价三者之间的关系进行解答; 单价 $\\times$ 数量 $=$ 总价, 总价 - 数量 $=$ 单价, 总价 $\\div$ 单价 $=$ 数量。", "solution": "null", "level": "三年级", "question": "一件运动衣 43 元, 一条裤子 48 元, 要买这样的运动服 62 套, 一共需要多少钱?", "options": [], "subject": "算术", "analysis": "5642 元\n\n【分析】用 43 加上 48 求出 1 套的单价, 然后再根据“单价 $\\times$ 数量 $=$ 总价”解答即可。\n\n【详解】 $43+48=91($ 元 $)$\n\n$91 \\times 62=5642 ($ 元 $)$\n\n答: 一共需要 5642 元。\n\n【点睛】解答此题应根据总价、数量和单价三者之间的关系进行解答; 单价 $\\times$ 数量 $=$ 总价, 总价 - 数量 $=$ 单价, 总价 $\\div$ 单价 $=$ 数量。"} {"id": "8599", "image": [], "answer": "828 人\n\n【分析】根据倍数关系, 用 46 乘 17 求出植松树人数的 17 倍, 即植柏树的人数, 然后再加上植松树的人数, 就是这个学校一共去植树的人数。\n\n【详解】 $46 \\times 17=782$ (人)\n$46+782=828$ (人)\n\n答: 这个学校一共去植树 828 人。\n\n【点睛】本题解答依据是: 求一个数的几倍是多少, 用乘法计算。", "solution": "null", "level": "三年级", "question": "学校五一节要去植树, 植松树的有 46 人, 植柏树的人数是植松树人数的 17 倍, 这个学校一共去植树多少人?", "options": [], "subject": "算术", "analysis": "828 人\n\n【分析】根据倍数关系, 用 46 乘 17 求出植松树人数的 17 倍, 即植柏树的人数, 然后再加上植松树的人数, 就是这个学校一共去植树的人数。\n\n【详解】 $46 \\times 17=782$ (人)\n$46+782=828$ (人)\n\n答: 这个学校一共去植树 828 人。\n\n【点睛】本题解答依据是: 求一个数的几倍是多少, 用乘法计算。"} {"id": "8601", "image": [], "answer": "29 元、 30 元或 31 元\n\n【分析】已知每套拼图的价钱大于 28 元, 因此比 28 大的数 $\\times$ 拼图套数 $=$ 总价钱, 因为带了 380 元,没有花完,所以最后总价钱要小于 $380 ,$ 据此计算即可解答。\n\n【详解】 $29 \\times 12=348 ($ 元) $<380$ 元,符合题意;\n\n$30 \\times 12=360$ (元) $<380$ 元,符合题意;\n\n$31 \\times 12=372 ($ 元 $)<380$ 元, 符合题意;\n\n$32 \\times 12=384$ (元) $>380$ 元,不符合题意;\n\n答: 每套拼图的价钱可能是 29 元、 30 元、 31 元。\n\n【点睛】熟练掌握总价、单价和数量三者之间的关系是解答本题的关键。", "solution": "null", "level": "三年级", "question": "王老师带了 380 元给幼儿园的小朋友买拼图玩具。他发现每套拼图的价钱比 28 元贵, 买了 12 套后钱没有花完。每套拼图的价钱可能是多少元?", "options": [], "subject": "算术", "analysis": "29 元、 30 元或 31 元\n\n【分析】已知每套拼图的价钱大于 28 元, 因此比 28 大的数 $\\times$ 拼图套数 $=$ 总价钱, 因为带了 380 元,没有花完,所以最后总价钱要小于 $380 ,$ 据此计算即可解答。\n\n【详解】 $29 \\times 12=348 ($ 元) $<380$ 元,符合题意;\n\n$30 \\times 12=360$ (元) $<380$ 元,符合题意;\n\n$31 \\times 12=372 ($ 元 $)<380$ 元, 符合题意;\n\n$32 \\times 12=384$ (元) $>380$ 元,不符合题意;\n\n答: 每套拼图的价钱可能是 29 元、 30 元、 31 元。\n\n【点睛】熟练掌握总价、单价和数量三者之间的关系是解答本题的关键。"} {"id": "8602", "image": [], "answer": "(1)玩具小熊; (2) 12 元\n\n【分析】(1)先估一估买每种玩具大约需要多少钱, 估算后可知, 只有买玩具小熊的费用是 600 元左右, 所以她买的是玩具小熊。\n\n(2)先计算出买这些玩具共需要多少钱,再计算出应找回多少钱。\n\n【详解】(1)小狗: $18 \\times 21$\n\n$\\approx 20 \\times 20$\n\n$=400($ 元 $)$\n\n小熊: $28 \\times 21$\n\n$\\approx 30 \\times 20$\n\n$=600$ (元)\n\n小兔: $38 \\times 21$\n\n$\\approx 40 \\times 20$\n$=800 ($ 元)\n\n答: 她买的是玩具小熊。\n\n(2) $600-28 \\times 21$\n\n$=600-588$\n\n$=12 ($ 元)\n\n答:买这些玩具应找回李阿姨 12 元。\n\n【点睛】本题考查经济问题, 关键是熟记公式总价 $=$ 单价 $\\times$ 数量。两位数乘两位数估算时, 将乘数估成与其接近的整十数, 再进行计算。", "solution": "null", "level": "三年级", "question": "幼儿园李阿姨买了 21 个同样的玩具, 付给营业员 600 元。\n\n| 小狗 18 元 | 小熊 28 元 |\n| :---: | :---: |\n\n(1) 估一估,她买的是哪种玩具?\n\n(2) 买这些玩具应找回李阿姨多少钱?", "options": [], "subject": "算术", "analysis": "(1)玩具小熊; (2) 12 元\n\n【分析】(1)先估一估买每种玩具大约需要多少钱, 估算后可知, 只有买玩具小熊的费用是 600 元左右, 所以她买的是玩具小熊。\n\n(2)先计算出买这些玩具共需要多少钱,再计算出应找回多少钱。\n\n【详解】(1)小狗: $18 \\times 21$\n\n$\\approx 20 \\times 20$\n\n$=400($ 元 $)$\n\n小熊: $28 \\times 21$\n\n$\\approx 30 \\times 20$\n\n$=600$ (元)\n\n小兔: $38 \\times 21$\n\n$\\approx 40 \\times 20$\n$=800 ($ 元)\n\n答: 她买的是玩具小熊。\n\n(2) $600-28 \\times 21$\n\n$=600-588$\n\n$=12 ($ 元)\n\n答:买这些玩具应找回李阿姨 12 元。\n\n【点睛】本题考查经济问题, 关键是熟记公式总价 $=$ 单价 $\\times$ 数量。两位数乘两位数估算时, 将乘数估成与其接近的整十数, 再进行计算。"} {"id": "8616", "image": [], "answer": "V\n\n【分析】根据整数乘法的计算方法, 求出 $25 \\times 80$ 的积, 再进行判断即可。\n\n【详解】 $25 \\times 80=2000$\n\n所以 $25 \\times 80$ 的积是四位数, 积的末尾有三个零, 此说法正确。\n\n故答案为: $\\sqrt{ }$\n\n【点睛】求两个数的积的位数与末尾 0 的个数, 可以先求出它们的乘积, 然后再进一步解答。", "solution": "null", "level": "三年级", "question": "$25 \\times 80$ 的积是四位数, 积的末尾有三个零。( $\\quad)$", "options": [], "subject": "算术", "analysis": "V\n\n【分析】根据整数乘法的计算方法, 求出 $25 \\times 80$ 的积, 再进行判断即可。\n\n【详解】 $25 \\times 80=2000$\n\n所以 $25 \\times 80$ 的积是四位数, 积的末尾有三个零, 此说法正确。\n\n故答案为: $\\sqrt{ }$\n\n【点睛】求两个数的积的位数与末尾 0 的个数, 可以先求出它们的乘积, 然后再进一步解答。"} {"id": "8617", "image": [], "answer": "V\n\n【分析】分别计算出 $32 \\times 19+32+32 \\times 20 、 32 \\times 40$ 的结果, 然后再判断即可。 $32 \\times 19+32+32 \\times 20$ 此算式先算乘法, 再算加法。\n\n【详解】 $32 \\times 19+32+32 \\times 20$\n\n$=608+32+640$\n\n$=640+640$\n\n$=1280$\n\n$32 \\times 40=1280$\n\n故答案为: $\\sqrt{ }$\n\n【点睛】此题考查的是混合运算的计算顺序, 以及两位数与两位数的乘法计算, 应熟练掌握。", "solution": "null", "level": "三年级", "question": "$32 \\times 19+32+32 \\times 20$ 与 $32 \\times 40$ 的积相等。( )", "options": [], "subject": "算术", "analysis": "V\n\n【分析】分别计算出 $32 \\times 19+32+32 \\times 20 、 32 \\times 40$ 的结果, 然后再判断即可。 $32 \\times 19+32+32 \\times 20$ 此算式先算乘法, 再算加法。\n\n【详解】 $32 \\times 19+32+32 \\times 20$\n\n$=608+32+640$\n\n$=640+640$\n\n$=1280$\n\n$32 \\times 40=1280$\n\n故答案为: $\\sqrt{ }$\n\n【点睛】此题考查的是混合运算的计算顺序, 以及两位数与两位数的乘法计算, 应熟练掌握。"} {"id": "8618", "image": [], "answer": "$\\times$\n\n【分析】门票的单价乘 20 即等于 20 张门票的价钱, 再与 1000 元进行比较即可解答。\n\n【详解】 $58 \\times 20=1160$ (元)\n\n$1160>1000$, 准备 1000 元不够。\n\n故答案为: $\\times$\n【点睛】熟练掌握总价、单价和数量三者之间的关系是解答本题的关键。", "solution": "null", "level": "三年级", "question": "游乐园的门票价格为 58 元/张,如果想要购买 20 张门票,准备 1000 元足够了。( )", "options": [], "subject": "算术", "analysis": "$\\times$\n\n【分析】门票的单价乘 20 即等于 20 张门票的价钱, 再与 1000 元进行比较即可解答。\n\n【详解】 $58 \\times 20=1160$ (元)\n\n$1160>1000$, 准备 1000 元不够。\n\n故答案为: $\\times$\n【点睛】熟练掌握总价、单价和数量三者之间的关系是解答本题的关键。"} {"id": "8619", "image": [], "answer": "$\\times$\n\n【分析】采用举例法, 分别计算出 $15 \\times 12 、 50 \\times 11$ 的积, 然后再判断即可。\n\n【详解】 $15 \\times 12=180$, 此时积的末尾有 0 , 而两个乘数的末尾没有 0 ;\n\n$50 \\times 11=550$, 此时积的末尾有 0 , 其中一个乘数的末尾有 0 ;\n\n因此积的未尾有 0 , 两个乘数的末尾不一定有 0 。\n\n故答案为: $\\times$\n\n【点睛】熟练掌握两位数与两位数的乘法计算是解答此题的关键。", "solution": "null", "level": "三年级", "question": "积的未尾有 0 , 两个乘数的末尾一定有 0 。( )", "options": [], "subject": "算术", "analysis": "$\\times$\n\n【分析】采用举例法, 分别计算出 $15 \\times 12 、 50 \\times 11$ 的积, 然后再判断即可。\n\n【详解】 $15 \\times 12=180$, 此时积的末尾有 0 , 而两个乘数的末尾没有 0 ;\n\n$50 \\times 11=550$, 此时积的末尾有 0 , 其中一个乘数的末尾有 0 ;\n\n因此积的未尾有 0 , 两个乘数的末尾不一定有 0 。\n\n故答案为: $\\times$\n\n【点睛】熟练掌握两位数与两位数的乘法计算是解答此题的关键。"} {"id": "8620", "image": [], "answer": "$\\sqrt{ }$\n\n【分析】根据求一个数的几倍是多少, 用这个数乘倍数, 已知甲数是乙数的 15 倍, 则用乙数乘 15 ,即可求出甲数。据此判断。\n\n【详解】由分析得:\n\n如果甲数是乙数的 15 倍, 那么甲数 $=$ 乙数 $\\times 15$ 。计算正确。\n\n故答案为: $\\sqrt{ }$\n\n【点睛】本题关键是正确理解倍数关系。", "solution": "null", "level": "三年级", "question": "如果甲数是乙数的 15 倍, 那么甲数 $=$ 乙数 $\\times 15$ 。 ( $)$", "options": [], "subject": "算术", "analysis": "$\\sqrt{ }$\n\n【分析】根据求一个数的几倍是多少, 用这个数乘倍数, 已知甲数是乙数的 15 倍, 则用乙数乘 15 ,即可求出甲数。据此判断。\n\n【详解】由分析得:\n\n如果甲数是乙数的 15 倍, 那么甲数 $=$ 乙数 $\\times 15$ 。计算正确。\n\n故答案为: $\\sqrt{ }$\n\n【点睛】本题关键是正确理解倍数关系。"} {"id": "8624", "image": [], "answer": "2250 元\n\n【分析】先用 $25 \\times 45$ 计算出每包的钱数, 然后再乘 2 即可计算出 2 包的钱数, 据此解决。\n\n\\section*{【详解】 $25 \\times 45 \\times 2$}\n\n$=1125 \\times 2$\n\n$=2250 ($ 元)\n\n答: 批发 2 包这样的衣服需要 2250 元钱。\n\n【点睛】解决本题的关键是熟练掌握两位数乘两位数的进位乘法及多位数乘一位数的计算方法。", "solution": "null", "level": "三年级", "question": "一款衣服每包 25 件, 批发价每件 45 元, 批发 2 包这样的衣服需要多少钱?", "options": [], "subject": "算术", "analysis": "2250 元\n\n【分析】先用 $25 \\times 45$ 计算出每包的钱数, 然后再乘 2 即可计算出 2 包的钱数, 据此解决。\n\n\\section*{【详解】 $25 \\times 45 \\times 2$}\n\n$=1125 \\times 2$\n\n$=2250 ($ 元)\n\n答: 批发 2 包这样的衣服需要 2250 元钱。\n\n【点睛】解决本题的关键是熟练掌握两位数乘两位数的进位乘法及多位数乘一位数的计算方法。"} {"id": "8625", "image": [], "answer": "960 千米\n\n【分析】先用 24 时计时法将下午 2 时表示为 14 时, 用到达的时间减去出发的时间, 计算出去姥姥家所用的时间, 速度已知, 再根据速度 $\\times$ 时间 $=$ 路程, 代入数据计算小明家离姥姥家有多少千米, 据此解答。\n【详解】 14 时 -6 时 $=8$ 时\n\n$120 \\times 8=960$ (千米)\n\n答: 小明家离姥姥家有 960 千米。\n\n【点睛】本题考查三位数乘一位数的计算, 熟练掌握并正确计算是解答本题的关键。", "solution": "null", "level": "三年级", "question": "小明上午 6 时和妈妈乘火车去姥姥家, 下午 2 时才到姥姥家, 火车每小时行 120 千米。小明家离姥姥家有多少千米?", "options": [], "subject": "算术", "analysis": "960 千米\n\n【分析】先用 24 时计时法将下午 2 时表示为 14 时, 用到达的时间减去出发的时间, 计算出去姥姥家所用的时间, 速度已知, 再根据速度 $\\times$ 时间 $=$ 路程, 代入数据计算小明家离姥姥家有多少千米, 据此解答。\n【详解】 14 时 -6 时 $=8$ 时\n\n$120 \\times 8=960$ (千米)\n\n答: 小明家离姥姥家有 960 千米。\n\n【点睛】本题考查三位数乘一位数的计算, 熟练掌握并正确计算是解答本题的关键。"} {"id": "8626", "image": [], "answer": "1350 千克\n\n【分析】 5 箱蜜蜂一年可以酿蜂蜜的重量 $\\div 5=1$ 箱蜜蜂一年可以酿蜂蜜的重量, 1 箱蜜蜂一年可以酿蜂蜜的重量 $\\times 18=18$ 箱蜜蜂一年可以酿蜂蜜的重量, 依此计算。\n\n【详解】 $375 \\div 5=75$ (千克)\n\n$75 \\times 18=1350($ 千克 $)$\n\n答: 18 箱蜜蜂一年可以酿 1350 千克蜂蜜。\n\n【点睛】此题考查的是归一问题的计算, 先计算出 1 箱蜜蜂一年可以酿蜂蜜的重量是解答此题的关键。", "solution": "null", "level": "三年级", "question": "5 箱蜜蜂一年可以酿 375 千克蜂蜜, 照这样计算, 18 箱蜜蜂一年可以酿多少千克蜂蜜?", "options": [], "subject": "算术", "analysis": "1350 千克\n\n【分析】 5 箱蜜蜂一年可以酿蜂蜜的重量 $\\div 5=1$ 箱蜜蜂一年可以酿蜂蜜的重量, 1 箱蜜蜂一年可以酿蜂蜜的重量 $\\times 18=18$ 箱蜜蜂一年可以酿蜂蜜的重量, 依此计算。\n\n【详解】 $375 \\div 5=75$ (千克)\n\n$75 \\times 18=1350($ 千克 $)$\n\n答: 18 箱蜜蜂一年可以酿 1350 千克蜂蜜。\n\n【点睛】此题考查的是归一问题的计算, 先计算出 1 箱蜜蜂一年可以酿蜂蜜的重量是解答此题的关键。"} {"id": "8627", "image": [], "answer": "600 米\n\n【分析】工程队 6 天共修路的长度 $\\div 6=$ 工程队 1 天修路的长度, 工程队 1 天修路的长度 $\\times 25=$ 工程队 25 天修路的长度, 依此计算。\n\n【详解】 $144 \\div 6=24$ (米)\n\n$24 \\times 25=600($ 米 $)$\n\n答:工程队 25 天共修路 600 米。\n\n【点睛】此题考查的是工程问题的计算, 先计算出工程队 1 天修路的长度是解答此题的关键。", "solution": "null", "level": "三年级", "question": "工程队 6 天共修路 144 米, 照这样计算, 工程队 25 天共修路多少米?", "options": [], "subject": "算术", "analysis": "600 米\n\n【分析】工程队 6 天共修路的长度 $\\div 6=$ 工程队 1 天修路的长度, 工程队 1 天修路的长度 $\\times 25=$ 工程队 25 天修路的长度, 依此计算。\n\n【详解】 $144 \\div 6=24$ (米)\n\n$24 \\times 25=600($ 米 $)$\n\n答:工程队 25 天共修路 600 米。\n\n【点睛】此题考查的是工程问题的计算, 先计算出工程队 1 天修路的长度是解答此题的关键。"} {"id": "9164", "image": [], "answer": "解54\n\n【分析】要使 $\\square 82 \\div 5$ 的商是三位数, 口应大于除数 5 ;\n\n要使 $\\square 82 \\div 5$ 的商是两位数, $\\square$ 应小于除数 5 ;\n\n然后再进一步解答。\n\n【详解】如果 $\\Delta 82 \\div 5$ 的商是三位数, 口里可以填 $5 、 6 、 7 、 8 、 9$, 最小填 5 ;\n要使商是两位数, 哩可以填 $1 、 2 、 3 、 4$, 最大填 4 。\n\n【点睛】三位数除以一位数, 被除数最高位上的数大于除数, 商是三位数, 否则商是两位数。", "solution": "null", "level": "三年级", "question": "如果 $\\square 82 \\div 5$ 的商是三位数, 口里最小应填 $($ ); 要使商是两位数, 口里最大应填 (", "options": [], "subject": "算术", "analysis": "解54\n\n【分析】要使 $\\square 82 \\div 5$ 的商是三位数, 口应大于除数 5 ;\n\n要使 $\\square 82 \\div 5$ 的商是两位数, $\\square$ 应小于除数 5 ;\n\n然后再进一步解答。\n\n【详解】如果 $\\Delta 82 \\div 5$ 的商是三位数, 口里可以填 $5 、 6 、 7 、 8 、 9$, 最小填 5 ;\n要使商是两位数, 哩可以填 $1 、 2 、 3 、 4$, 最大填 4 。\n\n【点睛】三位数除以一位数, 被除数最高位上的数大于除数, 商是三位数, 否则商是两位数。"} {"id": "9165", "image": [], "answer": "解146\n\n【分析】因为余数是 6 , 所以除数要大于余数, 最小是 7 , 根据被除数 $=$ 除数 $\\times$ 商 + 余数, 即可计算出被除数。\n\n【详解】 $20 \\times 7+6$\n\n$=140+6$\n\n$=146$\n\n【点睛】解答此题的关键是要先确定除数是多少。", "solution": "null", "level": "三年级", "question": "算式 $\\square+\\square=20 \\ldots . .6$, 要使除数最小, 被除数是 ( )", "options": [], "subject": "算术", "analysis": "解146\n\n【分析】因为余数是 6 , 所以除数要大于余数, 最小是 7 , 根据被除数 $=$ 除数 $\\times$ 商 + 余数, 即可计算出被除数。\n\n【详解】 $20 \\times 7+6$\n\n$=140+6$\n\n$=146$\n\n【点睛】解答此题的关键是要先确定除数是多少。"} {"id": "9166", "image": [], "answer": "解1440\n\n【分析】用每层住户人数乘每标楼层数, 求出每标楼住户人数。再乘楼的㚷数, 求出这个小区住户人数。\n\n【详解】 $6 \\times 10 \\times 24$\n\n$=60 \\times 24$\n\n$=1440($ 户)\n\n【点睛】本题考查两步连乘解决实际问题, 理清量与量之间的关系, 根据题意列式计算即可。", "solution": "null", "level": "三年级", "question": "某小区有 24 标楼, 每梌楼有 10 层, 每层住着 6 户人家, 这个小区一共住着( )户人家。", "options": [], "subject": "算术", "analysis": "解1440\n\n【分析】用每层住户人数乘每标楼层数, 求出每标楼住户人数。再乘楼的㚷数, 求出这个小区住户人数。\n\n【详解】 $6 \\times 10 \\times 24$\n\n$=60 \\times 24$\n\n$=1440($ 户)\n\n【点睛】本题考查两步连乘解决实际问题, 理清量与量之间的关系, 根据题意列式计算即可。"} {"id": "9167", "image": [], "answer": "解5\n\n【分析】第一空, 要使得商是两位数, 那么被除数的最高位上的数要小于 6 , 可以填 $1 \\sim 5$;第二空, 要使得商是三位数, 那么被除数的最高位要大于等于 6 , 可以填 $6 \\sim 9$;第三空, 要使得商中间有 0 , 被除数的最高位上被除尽了, 十位上的数要小于 4 , 可以填 $0 \\sim 3$;第四空, 要使得商的中间没有 0 , 被除数的最高位除尽了, 十位上的数要大于等于 4 , 可填 $4 \\sim 9$, 据此解答。\n\n【详解】 (5) $54 \\div 6$ (商是两位数); (6) $54 \\div 6$ (商是三位数);\n\n8 (3) $4 \\div 4$ (商的中间有 $0 ) ; 8(4) 4 \\div 4 ($ 商的中间没有 0 )\n\n【点睛】熟练掌握除法中各个部分之间的关系并灵活运用是解答本题的关键。", "solution": "null", "level": "三年级", "question": "在括号里填上合适的数字。\n\n( ) $54 \\div 6$ (商是两位数) $\\quad(\\quad) 54 \\div 6$ (商是三位数)\n\n$8(\\quad) \\div 4$ (商的中间有 $0 ) \\quad 8(\\quad) 4 \\div 4$ (商的中间没有 0 )", "options": [], "subject": "算术", "analysis": "解5\n\n【分析】第一空, 要使得商是两位数, 那么被除数的最高位上的数要小于 6 , 可以填 $1 \\sim 5$;第二空, 要使得商是三位数, 那么被除数的最高位要大于等于 6 , 可以填 $6 \\sim 9$;第三空, 要使得商中间有 0 , 被除数的最高位上被除尽了, 十位上的数要小于 4 , 可以填 $0 \\sim 3$;第四空, 要使得商的中间没有 0 , 被除数的最高位除尽了, 十位上的数要大于等于 4 , 可填 $4 \\sim 9$, 据此解答。\n\n【详解】 (5) $54 \\div 6$ (商是两位数); (6) $54 \\div 6$ (商是三位数);\n\n8 (3) $4 \\div 4$ (商的中间有 $0 ) ; 8(4) 4 \\div 4 ($ 商的中间没有 0 )\n\n【点睛】熟练掌握除法中各个部分之间的关系并灵活运用是解答本题的关键。"} {"id": "9168", "image": [], "answer": "解12\n\n【分析】根据题意, 用总钱数除以相数, 求出每箱饮料多少钱, 再除以每瓶的价格, 即可求出每箱饮料有多少瓶。\n\n\n\n$360 \\div 6 \\div 5$\n\n$=60 \\div 5$\n$=12$ (瓶)\n\n【点睛】此题考查根据除法意义解决实际问题的能力。", "solution": "null", "level": "三年级", "question": "冷饮店今天卖出 6 箱饮料, 共收入 360 元。如果每瓶饮料卖 5 元, 每箱饮料有 ( )瓶。", "options": [], "subject": "算术", "analysis": "解12\n\n【分析】根据题意, 用总钱数除以相数, 求出每箱饮料多少钱, 再除以每瓶的价格, 即可求出每箱饮料有多少瓶。\n\n\n\n$360 \\div 6 \\div 5$\n\n$=60 \\div 5$\n$=12$ (瓶)\n\n【点睛】此题考查根据除法意义解决实际问题的能力。"} {"id": "9179", "image": [], "answer": "解1400 元\n\n【分析】根据题意, 用每筐苹果的价钱乘以筐数即可解答。\n\n【详解】 $40 \\times 35=1400$ (元);\n答: 已收款 1400 元。\n\n【点睛】此题考查了关系式: 单价 $\\times$ 数量=总价,以及分析判断能力。", "solution": "null", "level": "三年级", "question": "李大爷家收了 78 筐苹果, 以每筐 40 元的价格卖出, 已经卖出了 35 筐, 已收款多少元?", "options": [], "subject": "算术", "analysis": "解1400 元\n\n【分析】根据题意, 用每筐苹果的价钱乘以筐数即可解答。\n\n【详解】 $40 \\times 35=1400$ (元);\n答: 已收款 1400 元。\n\n【点睛】此题考查了关系式: 单价 $\\times$ 数量=总价,以及分析判断能力。"} {"id": "9180", "image": [], "answer": "解45 人\n\n【分析】根据题干, 是把 225 名同学平均分成 5 组, 求出其中一组的人数即可, 据此即可解答。\n\n【详解】 $225 \\div 5=45$ (人)\n\n答: 每辆车应该坐 45 人。\n\n【点睛】此题考查了除法的意义和应用, 直接列式即可解答。", "solution": "null", "level": "三年级", "question": "三年级的 225 名同学要乘 5 辆车去春游, 如果每辆车座的人同样多, 每辆车应该坐多少人?", "options": [], "subject": "算术", "analysis": "解45 人\n\n【分析】根据题干, 是把 225 名同学平均分成 5 组, 求出其中一组的人数即可, 据此即可解答。\n\n【详解】 $225 \\div 5=45$ (人)\n\n答: 每辆车应该坐 45 人。\n\n【点睛】此题考查了除法的意义和应用, 直接列式即可解答。"} {"id": "9181", "image": [], "answer": "解坐不下\n\n【分析】先用排数 $\\times$ 每排座位数算出一共可以坐多少人,再和 500 名同学比较即可。\n\n【详解】 $19 \\times 25=475$ (人)\n\n因为 $475<500$, 所以坐不下。\n\n答: 我们想组织 500 名同学看电影, 坐不下。\n\n【点睛】本题考查用两位数乘两位数解决生活实际问题。", "solution": "null", "level": "三年级", "question": "电影院一共有 25 排座位, 每排可坐 19 人。\n\n我们想组织 500 名同\n\n学看电影, 坐得下吗?", "options": [], "subject": "算术", "analysis": "解坐不下\n\n【分析】先用排数 $\\times$ 每排座位数算出一共可以坐多少人,再和 500 名同学比较即可。\n\n【详解】 $19 \\times 25=475$ (人)\n\n因为 $475<500$, 所以坐不下。\n\n答: 我们想组织 500 名同学看电影, 坐不下。\n\n【点睛】本题考查用两位数乘两位数解决生活实际问题。"} {"id": "9182", "image": ["1507.jpg"], "answer": "解252 千克\n\n【分析】先求出 3 个星期有多少天, 再乘每天浪费水的重量 12 千克, 即可解答。\n\n【详解】\n\n$12 \\times(3 \\times 7)$\n\n$=12 \\times 21$\n\n$=252$ (千克)\n\n答: 这个水龙头 3 个星期一共会浪费 252 千克水。\n\n【点睛】此题考查根据乘法的意义解决实际问题的能力。", "solution": "null", "level": "三年级", "question": "一个滴水的水龙头平均每天浪费 12 千克水。这个水龙头 3 个星期一共会浪费多少水?\n\n", "options": [], "subject": "算术", "analysis": "解252 千克\n\n【分析】先求出 3 个星期有多少天, 再乘每天浪费水的重量 12 千克, 即可解答。\n\n【详解】\n\n$12 \\times(3 \\times 7)$\n\n$=12 \\times 21$\n\n$=252$ (千克)\n\n答: 这个水龙头 3 个星期一共会浪费 252 千克水。\n\n【点睛】此题考查根据乘法的意义解决实际问题的能力。"} {"id": "9183", "image": [], "answer": "解1680 米\n\n【分析】根据题意可知, 李叔叔每天要走 2 个家到单位的距离, 因此先用李叔叔平均每分钟走的路程乘需要走的时间计算出李叔叔家到他单位的距离, 然后用李叔叔家到他单位的距离乘 2 即可, 依此计算。\n\n【详解】 $56 \\times 15=840$ (米)\n\n$840 \\times 2=1680$ (米)\n\n答: 他每天上、下班要走 1680 米。\n\n【点睛】此题考查的是普通的行程问题, 熟练掌握两位数与两位数的乘法计算是解答此题的关键。", "solution": "null", "level": "三年级", "question": "李叔叔每天步行上、下班, 平均每分钟走 56 米, 需要 15 分钟才能走到单位。如果李叔叔从家到单位每天往返 1 次,他每天上、下班要走多少米?", "options": [], "subject": "算术", "analysis": "解1680 米\n\n【分析】根据题意可知, 李叔叔每天要走 2 个家到单位的距离, 因此先用李叔叔平均每分钟走的路程乘需要走的时间计算出李叔叔家到他单位的距离, 然后用李叔叔家到他单位的距离乘 2 即可, 依此计算。\n\n【详解】 $56 \\times 15=840$ (米)\n\n$840 \\times 2=1680$ (米)\n\n答: 他每天上、下班要走 1680 米。\n\n【点睛】此题考查的是普通的行程问题, 熟练掌握两位数与两位数的乘法计算是解答此题的关键。"} {"id": "9189", "image": [], "answer": "解三 $1 \\# \\#-$\n\n【分析】根据两位数乘两位数的计算法则, 直接计算出 $25 \\times 18$ 的积, 然后再填空即可。\n\n【详解】 $25 \\times 18=450$, 即算式 $25 \\times 18$ 的积是三位数, 积的末尾有 1 个 0 。\n\n【点睛】熟练掌握两位数与两位数的乘法计算是解答此题的关键。", "solution": "null", "level": "三年级", "question": "算式 $25 \\times 18$ 的积是 $(\\quad)$ 位数, 积的末尾有 ( ) 个 0 。", "options": [], "subject": "算术", "analysis": "解三 $1 \\# \\#-$\n\n【分析】根据两位数乘两位数的计算法则, 直接计算出 $25 \\times 18$ 的积, 然后再填空即可。\n\n【详解】 $25 \\times 18=450$, 即算式 $25 \\times 18$ 的积是三位数, 积的末尾有 1 个 0 。\n\n【点睛】熟练掌握两位数与两位数的乘法计算是解答此题的关键。"} {"id": "9190", "image": [], "answer": "解600\n\n【分析】根据题意, 每排 19 人, 共有 32 排, 就是求 19 个 32 是多少, 根据乘法的意义解答。\n\n【详解】 $32 \\approx 30,19 \\approx 20$\n\n$20 \\times 30=600$ (人)\n\n电影院有 32 排座位, 每排可以坐 19 人。电影院一共大约可以坐(600)人。\n\n【点睛】熟练掌握两位数乘两位数的方法是解答本题的关键。", "solution": "null", "level": "三年级", "question": "电影院有 32 排座位, 每排可以坐 19 人。电影院一共大约可以坐( )。", "options": [], "subject": "算术", "analysis": "解600\n\n【分析】根据题意, 每排 19 人, 共有 32 排, 就是求 19 个 32 是多少, 根据乘法的意义解答。\n\n【详解】 $32 \\approx 30,19 \\approx 20$\n\n$20 \\times 30=600$ (人)\n\n电影院有 32 排座位, 每排可以坐 19 人。电影院一共大约可以坐(600)人。\n\n【点睛】熟练掌握两位数乘两位数的方法是解答本题的关键。"} {"id": "9191", "image": [], "answer": "解\n\n【分析】先根据两位数乘两位数的计算法则, 三位数除以一位数的计算法则, 分别计算出括号前、后算式的结果, 然后再比较即可。\n\n【详解】 (1) $42 \\times 20=840$, 即 $42 \\times 20<850$;\n\n(2) $48 \\times 29=1392,29 \\times 48=1392$, 即 $48 \\times 29=29 \\times 48$;\n\n(3) $820 \\div 4=205,280 \\div 2=140$, 即 $820 \\div 4>280 \\div 2$;\n\n(4) $220 \\div 4=55,450 \\div 9=50$, 即 $220 \\div 4>450 \\div 9$ 。\n\n【点睛】此题考查的是两位数与两位数的乘法计算, 以及三位数与一位数的除法计算, 应熟练掌握。", "solution": "null", "level": "三年级", "question": "在下面的括号里填上“>”“<”或“=”。\n\n| $42 \\times 20($ | ) 850 | $48 \\times 29($ | ) $29 \\times 48$ |\n| :--- | :--- | :--- | :--- |\n| $820 \\div 4($ | ) $280 \\div 2$ | $220 \\div 4($ | ) $450 \\div 9$ |", "options": [], "subject": "算术", "analysis": "解\n\n【分析】先根据两位数乘两位数的计算法则, 三位数除以一位数的计算法则, 分别计算出括号前、后算式的结果, 然后再比较即可。\n\n【详解】 (1) $42 \\times 20=840$, 即 $42 \\times 20<850$;\n\n(2) $48 \\times 29=1392,29 \\times 48=1392$, 即 $48 \\times 29=29 \\times 48$;\n\n(3) $820 \\div 4=205,280 \\div 2=140$, 即 $820 \\div 4>280 \\div 2$;\n\n(4) $220 \\div 4=55,450 \\div 9=50$, 即 $220 \\div 4>450 \\div 9$ 。\n\n【点睛】此题考查的是两位数与两位数的乘法计算, 以及三位数与一位数的除法计算, 应熟练掌握。"} {"id": "9193", "image": [], "answer": "解$200 \\quad 2400$\n\n【分析】长方形的周长 $=($ 长 + 宽 $) \\times 2$, 长方形的面积 $=$ 长 $\\times$ 宽, 代入数据计算即可。\n\n【详解】 $(60+40) \\times 2$\n\n$=100 \\times 2$\n\n$=200(\\mathrm{~cm})$\n\n$60 \\times 40=2400\\left(\\mathrm{~cm}^{2}\\right)$\n\n它的周长是 $200 \\mathrm{~cm} ;$ 它的面积是 $2400 \\mathrm{~cm}^{2}$ 。\n\n【点睛】熟练掌握长方形的周长和面积公式是解决本题的关键。", "solution": "null", "level": "三年级", "question": "一个长方形, 长 $60 \\mathrm{~cm}$, 宽 $40 \\mathrm{~cm}$ 。它的周长是 (\n\n) $\\mathrm{cm}$ ; 它的面积是 (\n\n) $\\mathrm{cm}^{2}$ 。", "options": [], "subject": "算术", "analysis": "解$200 \\quad 2400$\n\n【分析】长方形的周长 $=($ 长 + 宽 $) \\times 2$, 长方形的面积 $=$ 长 $\\times$ 宽, 代入数据计算即可。\n\n【详解】 $(60+40) \\times 2$\n\n$=100 \\times 2$\n\n$=200(\\mathrm{~cm})$\n\n$60 \\times 40=2400\\left(\\mathrm{~cm}^{2}\\right)$\n\n它的周长是 $200 \\mathrm{~cm} ;$ 它的面积是 $2400 \\mathrm{~cm}^{2}$ 。\n\n【点睛】熟练掌握长方形的周长和面积公式是解决本题的关键。"} {"id": "9194", "image": [], "answer": "解12\n\n【分析】长方形的长 $=$ 面积 $\\div$ 宽, 依此直接将数据代入公式计算出结果即可。\n\n【详解】 $60 \\div 5=12(\\mathrm{~cm})$, 即它的长是 $12 \\mathrm{~cm}$ 。\n\n【点睛】熟练掌握长方形的面积的计算是解答此题的关键。", "solution": "null", "level": "三年级", "question": "一个长方形的面积是 $60 \\mathrm{~cm}^{2}$, 宽是 $5 \\mathrm{~cm}$, 它的长是 (\n\n) $\\mathrm{cm}$ 。", "options": [], "subject": "算术", "analysis": "解12\n\n【分析】长方形的长 $=$ 面积 $\\div$ 宽, 依此直接将数据代入公式计算出结果即可。\n\n【详解】 $60 \\div 5=12(\\mathrm{~cm})$, 即它的长是 $12 \\mathrm{~cm}$ 。\n\n【点睛】熟练掌握长方形的面积的计算是解答此题的关键。"} {"id": "9068", "image": [], "answer": "解6\n\n由于每个小朋友都要和另外的 3 个通一次电话, 一共要通: $3 \\times 4=12$ (次); 又因为两个小朋友只通一次电话, 去掉重复计算的情况, 实际只通: $12 \\div 2=6$ (次), 据此解答。\n\n元旦这天,四个小伙伴互相打电话问候,共打了(6)次电话。\n\n本题考查了握手问题的实际应用, 要注意去掉重复计算的情况, 如果人比较少可以用枚举法解答, 如果人数比较多可以用公式: 通话次数 $=$ 人数 $\\times($ 人数 $-1 ) \\div 2$ 解答。", "solution": "null", "level": "三年级", "question": "元旦这天, 四个小伙伴互相打电话问候, 共打了( )次电话。", "options": [], "subject": "组合数学", "analysis": "解6\n\n由于每个小朋友都要和另外的 3 个通一次电话, 一共要通: $3 \\times 4=12$ (次); 又因为两个小朋友只通一次电话, 去掉重复计算的情况, 实际只通: $12 \\div 2=6$ (次), 据此解答。\n\n元旦这天,四个小伙伴互相打电话问候,共打了(6)次电话。\n\n本题考查了握手问题的实际应用, 要注意去掉重复计算的情况, 如果人比较少可以用枚举法解答, 如果人数比较多可以用公式: 通话次数 $=$ 人数 $\\times($ 人数 $-1 ) \\div 2$ 解答。"} {"id": "9070", "image": [], "answer": "解23\n\n根据每两人握一次手的搭配方法, 则有以下 3 种不同的搭配方法: 贝贝和佳佳, 贝贝和乐乐,佳佳和乐乐。其中每人均要握手 2 次。\n\n每两人握一次手, 每人要握 2 次手, 3 人一共握 3 次手。\n\n故答案为: $2 ; 3$ 。\n\n本题考查搭配问题, 可以采用枚举法。要注意按一定的顺序, 才能做到不重不漏。", "solution": "null", "level": "三年级", "question": "贝贝、佳佳和乐乐, 3 个朋友见面, 每两人握一次手, 每人要握 ( )次手, 3 人一共握( )次手。", "options": [], "subject": "组合数学", "analysis": "解23\n\n根据每两人握一次手的搭配方法, 则有以下 3 种不同的搭配方法: 贝贝和佳佳, 贝贝和乐乐,佳佳和乐乐。其中每人均要握手 2 次。\n\n每两人握一次手, 每人要握 2 次手, 3 人一共握 3 次手。\n\n故答案为: $2 ; 3$ 。\n\n本题考查搭配问题, 可以采用枚举法。要注意按一定的顺序, 才能做到不重不漏。"} {"id": "9071", "image": [], "answer": "解10\n\n每两人下一盘, 一个人就要下 4 盘, 5 个人就要下 20 盘, 但是这样算每两人下一盘棋, 就算了 2 次, 再除以 2 即可。\n\n$5 \\times(5-1) \\div 2$\n\n$=5 \\times 4 \\div 2$\n\n$=20 \\div 2$\n\n$=10($ 盘 $)$\n\n故答案为: 10\n\n这是个简单的组合问题, 每两个人下一盘棋: $n$ 个人的计算方法为: $n ( n-1 ) \\div 2$ 。", "solution": "null", "level": "三年级", "question": "有 5 名棋手,每两人下一盘棋,共要下( )盘。", "options": [], "subject": "组合数学", "analysis": "解10\n\n每两人下一盘, 一个人就要下 4 盘, 5 个人就要下 20 盘, 但是这样算每两人下一盘棋, 就算了 2 次, 再除以 2 即可。\n\n$5 \\times(5-1) \\div 2$\n\n$=5 \\times 4 \\div 2$\n\n$=20 \\div 2$\n\n$=10($ 盘 $)$\n\n故答案为: 10\n\n这是个简单的组合问题, 每两个人下一盘棋: $n$ 个人的计算方法为: $n ( n-1 ) \\div 2$ 。"} {"id": "9072", "image": [], "answer": "解9\n\n0 不能在最高位上,则十位上是 5 时,可组成 $50 、 58 、 59$;\n\n十位上是 8 时,可组成 $80 、 85 、 89$;\n\n十位上是 9 时,可组成 $90 、 95 、 98$;\n\n用 $0 、 5 、 8 、 9$ 可以组成 9 个没有重复数字的两位数。\n\n本题考查搭配问题, 可以采用枚举法。要注意按一定的顺序, 才能做到不重不漏。", "solution": "null", "level": "三年级", "question": "用 $0 、 5 、 8 、 9$ 可以组成 $\\quad$ )个没有重复数字的两位数。", "options": [], "subject": "组合数学", "analysis": "解9\n\n0 不能在最高位上,则十位上是 5 时,可组成 $50 、 58 、 59$;\n\n十位上是 8 时,可组成 $80 、 85 、 89$;\n\n十位上是 9 时,可组成 $90 、 95 、 98$;\n\n用 $0 、 5 、 8 、 9$ 可以组成 9 个没有重复数字的两位数。\n\n本题考查搭配问题, 可以采用枚举法。要注意按一定的顺序, 才能做到不重不漏。"} {"id": "9073", "image": [], "answer": "解6\n\n4 个同学, 每 2 个人比赛一场, 则有以下 6 种不同的比赛组合: A 和 B、A 和 C、A 和 D、 $B$ 和 $C 、 B$ 和 $D 、 C$ 和 $D$ 。\n\n根据分析可知, 每 2 个人比赛一场, 一共要进行 6 场比赛。\n\n本题考查搭配问题, 可以采用枚举法。要注意按一定的顺序, 才能做到不重不漏。", "solution": "null", "level": "三年级", "question": "A、B、C、D一共 4 个同学参加羽毛球比赛, 每 2 个人比赛一场, 一共要进行 ( )场比赛。", "options": [], "subject": "组合数学", "analysis": "解6\n\n4 个同学, 每 2 个人比赛一场, 则有以下 6 种不同的比赛组合: A 和 B、A 和 C、A 和 D、 $B$ 和 $C 、 B$ 和 $D 、 C$ 和 $D$ 。\n\n根据分析可知, 每 2 个人比赛一场, 一共要进行 6 场比赛。\n\n本题考查搭配问题, 可以采用枚举法。要注意按一定的顺序, 才能做到不重不漏。"} {"id": "9069", "image": [], "answer": "解\n\n$4 \\quad 530$\n\n225\n\n根据百位上数字的不同, 我们可以将它们分成两类:\n\n1.百位上是 3 能组成哪些三位数;\n\n2.百位上是 5 能组成哪些三位数。\n\n写出这些三位数, 找出最大的和最小的, 再相减, 就是它们相差多少。\n\n数字卡片 $3 、 0 、 5$ 可以组成的三位数:305、350、503、530; 最大的 530 ,最小的 $305 , 530$ $-305=225$ 。\n\n数字卡片 3、 $0 、 5$ 可以组成 $\\underline{4}$ 个不同的三位数, 其中最大的三位数是 $\\underline{530}$, 它和最小的三位数相差 $\\underline{225}$ 。\n\n\n\n写三位数时要注意:0不能放在最高位百位上, 要按照一定的顺序写, 做到不重不漏。", "solution": "null", "level": "三年级", "question": "数字卡片 $3 、 0 、 5$ 可以组成 \\$ \\qquad \\$个不同的三位数, 其中最大的三位数是 , 它和最小的三位数相差 \\$ \\qquad \\$", "options": [], "subject": "计数", "analysis": "解\n\n$4 \\quad 530$\n\n225\n\n根据百位上数字的不同, 我们可以将它们分成两类:\n\n1.百位上是 3 能组成哪些三位数;\n\n2.百位上是 5 能组成哪些三位数。\n\n写出这些三位数, 找出最大的和最小的, 再相减, 就是它们相差多少。\n\n数字卡片 $3 、 0 、 5$ 可以组成的三位数:305、350、503、530; 最大的 530 ,最小的 $305 , 530$ $-305=225$ 。\n\n数字卡片 3、 $0 、 5$ 可以组成 $\\underline{4}$ 个不同的三位数, 其中最大的三位数是 $\\underline{530}$, 它和最小的三位数相差 $\\underline{225}$ 。\n\n\n\n写三位数时要注意:0不能放在最高位百位上, 要按照一定的顺序写, 做到不重不漏。"} {"id": "9096", "image": [], "answer": "解(1) 49\n\n(2)历史\n\n(3) 故事 喜欢看故事书的人数最多\n\n【分析】(1)将喜欢看各种图书的人数相加求和。\n\n(2) 分别求出喜欢看各种图书的人数, 再比较大小解答。\n\n(3) 比较喜欢看各种图书的人数大小, 多买一些喜欢看的人数最多的那种图书。\n\n【详解】 (1) $9+9+3+5+4+8+2+9=49$ (名)\n\n三 (2) 班一共有 49 名同学。\n\n(2) $9+4=13$ (名), $9+8=17$ (名)\n\n$3+2=5($ 名 $), 5+9=14 ($ 名 $)$\n\n$17>14>13>5$, 则三 (2) 班喜欢看历史书的同学最少。\n\n(3) $17>14>13>5$, 三(2)班喜欢看故事书的同学最多, 如果再为班级购买一些新图书, 你觉得应该多买故事书, 理由是:喜欢看故事书的人数最多。\n\n【点睛】本题考查复式统计表的数据分析和整理。根据统计表找出需要的数据, 列式计算即可。", "solution": "null", "level": "三年级", "question": "三 (2) 班最喜欢看的图书统计情况如下:\n\n| | 科学
书 | 故事
书 | 历史
书 | 漫画
书 |\n| :---: | :---: | :---: | :---: | :---: |\n| 男生 | 9 | 9 | 3 | 5 |\n| 女生 | 4 | 8 | 2 | 9 |\n\n(1)三 (2) 班一共有 ( ) 名同学。\n\n(2)三 (2) 班喜欢看 $\\quad$ ) 书的同学最少。\n\n(3)如果再为班级购买一些新图书, 你觉得应该多买( )书, 理由是: ( )。", "options": [], "subject": "计数", "analysis": "解(1) 49\n\n(2)历史\n\n(3) 故事 喜欢看故事书的人数最多\n\n【分析】(1)将喜欢看各种图书的人数相加求和。\n\n(2) 分别求出喜欢看各种图书的人数, 再比较大小解答。\n\n(3) 比较喜欢看各种图书的人数大小, 多买一些喜欢看的人数最多的那种图书。\n\n【详解】 (1) $9+9+3+5+4+8+2+9=49$ (名)\n\n三 (2) 班一共有 49 名同学。\n\n(2) $9+4=13$ (名), $9+8=17$ (名)\n\n$3+2=5($ 名 $), 5+9=14 ($ 名 $)$\n\n$17>14>13>5$, 则三 (2) 班喜欢看历史书的同学最少。\n\n(3) $17>14>13>5$, 三(2)班喜欢看故事书的同学最多, 如果再为班级购买一些新图书, 你觉得应该多买故事书, 理由是:喜欢看故事书的人数最多。\n\n【点睛】本题考查复式统计表的数据分析和整理。根据统计表找出需要的数据, 列式计算即可。"} {"id": "9097", "image": [], "answer": "解(1) 千字文 诗经\n\n(2) $94 \\quad 85$\n\n【分析】(1)比较三年级通读各种图书的人数大小解答。\n\n(2)将三年级通读各种图书的人数相加。将四年级通读各种图书的人数相加。\n\n【详解】 (1) $30>25>16>14>9$\n\n在三年级学生中, 通读千字文的人数最多, 通读诗经的人数最少。\n\n(2) $16+25+14+9+30=94$ (人)\n\n$31+6+20+17+11=85($ 人 $)$\n\n三年级有 94 人参加此次活动, 四年级有 85 人参加此次活动。\n\n【点睛】本题考查复式统计表的数据分析和整理。根据统计表找出需要的数据, 列式计算即可。", "solution": "null", "level": "三年级", "question": "下面是三、四年级学生参加国学经典诵读活动的情况统计表。\n\n| 人数 书目 | 三字经 | 弟子规 | 论语 | 诗经 | 千字文 |\n| :---: | :---: | :---: | :---: | :---: | :---: |\n| 年级 | | | | | |\n| 三年级 | 16 | 25 | 14 | 9 | 30 |\n| 四年级 | 31 | 6 | 20 | 17 | 11 |\n\n(1)在三年级学生中, 通读 (\n) 的人数最多, 通读\n(2)三年级有 (\n) 的人数最少。\n) 人参加此次活动, 四年级有 (\n)人参加此次活动。", "options": [], "subject": "计数", "analysis": "解(1) 千字文 诗经\n\n(2) $94 \\quad 85$\n\n【分析】(1)比较三年级通读各种图书的人数大小解答。\n\n(2)将三年级通读各种图书的人数相加。将四年级通读各种图书的人数相加。\n\n【详解】 (1) $30>25>16>14>9$\n\n在三年级学生中, 通读千字文的人数最多, 通读诗经的人数最少。\n\n(2) $16+25+14+9+30=94$ (人)\n\n$31+6+20+17+11=85($ 人 $)$\n\n三年级有 94 人参加此次活动, 四年级有 85 人参加此次活动。\n\n【点睛】本题考查复式统计表的数据分析和整理。根据统计表找出需要的数据, 列式计算即可。"} {"id": "9098", "image": [], "answer": "解(1) 2423\n\n$(2) 22$\n\n(3) 3\n\n【分析】 (1) 金牌数从大到小对比: $9>5>3>2>1$;\n\n(2) 把 19 至 24 届冬奥会的金牌数相加;\n\n(3) 把所有银牌数一所有铜牌数=多的枚数; 据此解答。\n\n【详解】 (1) 根据分析, 获得金牌最多的是第 24 届, 最少的是第 23 届。\n\n(2) $2+2+5+3+1+9$\n\n$=4+5+3+1+9$\n\n$=9+3+1+9$\n\n$=12+1+9$\n\n$=13+9$\n\n$=22($ 枚 $)$\n\n中国队一共获得金牌 22 枚。\n\n(3) $(2+4+2+4+6+4)-(4+5+4+2+2+2)$\n\n$=(6+2+4+6+4)-(9+4+2+2+2)$\n$=(8+4+6+4)-(13+2+2+2)$\n\n$=(12+6+4)-(15+2+2)$\n\n$=(18+4)-(17+2)$\n\n$=22-19$\n\n$=3($ 枚 $)$\n\n中国队共获得的银牌比铜牌多 3 枚。\n\n【点睛】要注意观察图表,根据给出的信息进行计算。", "solution": "null", "level": "三年级", "question": "2022 年 2 月 4 日至 2 月 20 日, 第 24 届冬季奥林匹克运动会在我国北京举行。下面是我国近几届冬奥会获得奖牌情况统计表。根据表中数据回答问题。\n\n| 届次 | 19 | 20 | 21 | 22 | 23 | 24 |\n| :---: | :---: | :---: | :---: | :---: | :---: | :---: |\n| 金牌数 | 2 | 2 | 5 | 3 | 1 | 9 |\n| 银牌数 | 2 | 4 | 2 | 4 | 6 | 4 |\n| 铜牌数 | 4 | 5 | 4 | 2 | 2 | 2 |\n\n(1)这几届冬奥会中,中国队获得金牌最多的是第( )届,最少的是第( )届。\n\n(2)这几届冬奥会,中国队一共获得金牌( )枚。\n\n(3)这几届冬奥会,中国队共获得的银牌比铜牌多(枚。", "options": [], "subject": "计数", "analysis": "解(1) 2423\n\n$(2) 22$\n\n(3) 3\n\n【分析】 (1) 金牌数从大到小对比: $9>5>3>2>1$;\n\n(2) 把 19 至 24 届冬奥会的金牌数相加;\n\n(3) 把所有银牌数一所有铜牌数=多的枚数; 据此解答。\n\n【详解】 (1) 根据分析, 获得金牌最多的是第 24 届, 最少的是第 23 届。\n\n(2) $2+2+5+3+1+9$\n\n$=4+5+3+1+9$\n\n$=9+3+1+9$\n\n$=12+1+9$\n\n$=13+9$\n\n$=22($ 枚 $)$\n\n中国队一共获得金牌 22 枚。\n\n(3) $(2+4+2+4+6+4)-(4+5+4+2+2+2)$\n\n$=(6+2+4+6+4)-(9+4+2+2+2)$\n$=(8+4+6+4)-(13+2+2+2)$\n\n$=(12+6+4)-(15+2+2)$\n\n$=(18+4)-(17+2)$\n\n$=22-19$\n\n$=3($ 枚 $)$\n\n中国队共获得的银牌比铜牌多 3 枚。\n\n【点睛】要注意观察图表,根据给出的信息进行计算。"} {"id": "9107", "image": ["1494.jpg"], "answer": "解(1) 30 分钟以下\n\n(2) 1\n\n(3) 27\n\n(4) 见详解\n\n【分析】(1)根据统计表中三 (1) 班所在行的人数可以看出周日玩电子产品哪个时长的人数最多。\n\n(2) 三(1)班同学周日玩电子产品的时间在 30 分钟 $\\sim 1$ 小时的人数减三(2)班人数即可。\n\n(3)三(1)班和三(2)班周日玩电子产品的时间在 1 小时以上的人数相加即可。\n\n(4)根据统计信息, 结合国家卫健委的建议提出合理的意见建议。\n\n【详解】 (1) 三(1)班同学周日玩电子产品的时间在 30 分钟以下的人最多。\n\n(2) $12-11=1$ (人)\n\n三(2)班同学周日玩电子产品的时间在 30 分钟 $\\sim 1$ 小时的人数比三(1)班少 1 人。\n\n(3) $9+12+2+4$\n\n$=21+6$\n\n$=27($ 人)\n\n三(1)班和三(2)班周日玩电子产品的时间在 1 小时以上的有 27 人。\n\n(4)根据统计表可以看出, 有部分同学玩电子产品时间长, 容易损伤眼睛, 应缩短玩电子产品的时间,多运动,保护好眼睛。\n\n【点睛】本题主要考查学生对统计表的认识, 以及根据统计信息解决实际问题的能力。", "solution": "null", "level": "三年级", "question": "下面是三年级两个班的同学某个星期日玩电子产品的时间情况统计表。\n\n\n\n(1) 三 (1) 班同学周日玩电子产品的时间在()的人最多。\n\n(2) 三 (2) 班同学周日玩电子产品的时间在 30 分钟 $\\sim 1$ 小时的人数比三 (1) 班少 ( ) )。\n\n(3) 三(1)班和三(2)班周日玩电子产品的时间在 1 小时以上的有()人。\n\n(4) 国家卫健委指出: 小学生使用电子产品的时间每天累计不宜超过 1 小时。看了这个统计结果,\n你有什么想说的?", "options": [], "subject": "计数", "analysis": "解(1) 30 分钟以下\n\n(2) 1\n\n(3) 27\n\n(4) 见详解\n\n【分析】(1)根据统计表中三 (1) 班所在行的人数可以看出周日玩电子产品哪个时长的人数最多。\n\n(2) 三(1)班同学周日玩电子产品的时间在 30 分钟 $\\sim 1$ 小时的人数减三(2)班人数即可。\n\n(3)三(1)班和三(2)班周日玩电子产品的时间在 1 小时以上的人数相加即可。\n\n(4)根据统计信息, 结合国家卫健委的建议提出合理的意见建议。\n\n【详解】 (1) 三(1)班同学周日玩电子产品的时间在 30 分钟以下的人最多。\n\n(2) $12-11=1$ (人)\n\n三(2)班同学周日玩电子产品的时间在 30 分钟 $\\sim 1$ 小时的人数比三(1)班少 1 人。\n\n(3) $9+12+2+4$\n\n$=21+6$\n\n$=27($ 人)\n\n三(1)班和三(2)班周日玩电子产品的时间在 1 小时以上的有 27 人。\n\n(4)根据统计表可以看出, 有部分同学玩电子产品时间长, 容易损伤眼睛, 应缩短玩电子产品的时间,多运动,保护好眼睛。\n\n【点睛】本题主要考查学生对统计表的认识, 以及根据统计信息解决实际问题的能力。"} {"id": "9108", "image": [], "answer": "解(1) $26 ; 10 ; 9 ; 27$\n\n(2) 四\n\n(3)发现: 在这五项运动中,单项的成绩比较好,集体的成绩三年级(一)班高于其他班级。感想是:由于集体比赛项目比较弱,因此应该多培养同学们团结协作的意识,从而提高集体比赛项目的成绩。(答案不唯一)\n\n【分析】(1)将每一个班,每一项的积分加起来即可,然后再填空。\n\n(2) 将三年级每个班的总积分进行大小排序即可。\n\n(3)直接观察统计表、并结合生活经验进行解答,言之合理即可。\n\n【详解】 (1) $8+2+8+8=26$ (分)\n\n$7+3=10 ($ 分 $)$\n\n$6+3=9 ($ 分 $)$\n\n$15+4+8=27$ (分),填表如下:\n\n| | 跑步 (单项) | 跳远 (单项) | 集体长绳 | 集体拔河 | 集体接力赛 | 合计 |\n| :--- | :---: | :---: | :---: | :---: | :---: | :---: |\n| 三 (一) 班 | 8 | 2 | 8 | 8 | 0 | 26 |\n| 三 (二) 班 | 7 | 3 | 0 | 0 | 0 | 10 |\n| 三 (三) 班 | 6 | 3 | 0 | 0 | 0 | 9 |\n\n\n| 三 (四) 班 | 15 | 4 | 0 | 0 | 8 | 27 |\n| :--- | :---: | :---: | :---: | :---: | :---: | :---: |\n\n(2) $27>26>10>9$ ,即三年级总分第一名是四班。\n\n(3)从统计表中,可以发现:在这五项运动中,单项的成绩比较好,集体的成绩三年级(一)班高于其他班级。\n\n感想: 由于集体比赛项目比较弱, 因此应该多培养同学们团结协作的意识, 从而提高集体比赛项目的成绩。\n\n【点睛】此题主要考查统计表的分析能力,以及利用统计表中的信息解决问题的能力。", "solution": "null", "level": "三年级", "question": "五月份我们开展了阳光运动会, 下面是三年级各班成绩积分表。\n\n| | 跑步 (单项) | 跳远 (单项) | 集体长绳 | 集体拨河 | 集体接力赛 | 合计 |\n| :--- | :---: | :---: | :---: | :---: | :---: | :---: |\n| 三 (一) 班 | 8 | 2 | 8 | 8 | 0 | |\n| 三 (二) 班 | 7 | 3 | 0 | 0 | 0 | |\n| 三 (三) 班 | 6 | 3 | 0 | 0 | 0 | |\n| 三 (四) 班 | 15 | 4 | 0 | 0 | 8 | |\n\n(1)请算出每个班的积分总数填在表格里。\n\n(2) 三年级总分第一名是()班。\n\n(3)从统计表中,你发现了什么? 有什么感想?", "options": [], "subject": "计数", "analysis": "解(1) $26 ; 10 ; 9 ; 27$\n\n(2) 四\n\n(3)发现: 在这五项运动中,单项的成绩比较好,集体的成绩三年级(一)班高于其他班级。感想是:由于集体比赛项目比较弱,因此应该多培养同学们团结协作的意识,从而提高集体比赛项目的成绩。(答案不唯一)\n\n【分析】(1)将每一个班,每一项的积分加起来即可,然后再填空。\n\n(2) 将三年级每个班的总积分进行大小排序即可。\n\n(3)直接观察统计表、并结合生活经验进行解答,言之合理即可。\n\n【详解】 (1) $8+2+8+8=26$ (分)\n\n$7+3=10 ($ 分 $)$\n\n$6+3=9 ($ 分 $)$\n\n$15+4+8=27$ (分),填表如下:\n\n| | 跑步 (单项) | 跳远 (单项) | 集体长绳 | 集体拔河 | 集体接力赛 | 合计 |\n| :--- | :---: | :---: | :---: | :---: | :---: | :---: |\n| 三 (一) 班 | 8 | 2 | 8 | 8 | 0 | 26 |\n| 三 (二) 班 | 7 | 3 | 0 | 0 | 0 | 10 |\n| 三 (三) 班 | 6 | 3 | 0 | 0 | 0 | 9 |\n\n\n| 三 (四) 班 | 15 | 4 | 0 | 0 | 8 | 27 |\n| :--- | :---: | :---: | :---: | :---: | :---: | :---: |\n\n(2) $27>26>10>9$ ,即三年级总分第一名是四班。\n\n(3)从统计表中,可以发现:在这五项运动中,单项的成绩比较好,集体的成绩三年级(一)班高于其他班级。\n\n感想: 由于集体比赛项目比较弱, 因此应该多培养同学们团结协作的意识, 从而提高集体比赛项目的成绩。\n\n【点睛】此题主要考查统计表的分析能力,以及利用统计表中的信息解决问题的能力。"} {"id": "9109", "image": [], "answer": "解统计表见详解\n\n(1) 草莓; 黄桃\n\n(2)原味酸奶两个月卖的箱数都挺多,下个月可以多进一些原味酸奶。(答案不唯一)\n\n【分析】将两个表格中的数据分别整理到复式统计表中,即可完成复式统计表。\n\n(1)根据整理好的统计表,对比一月、二月的几种酸奶销量, 即可解答。\n\n(2)根据表中数据, 提出自己的想法即可。如原味酸奶两个月销量都较好, 下个月可以多进点原味酸奶。", "solution": "null", "level": "三年级", "question": "下面是某超市一、二月份销售某品牌酸奶统计表。\n\n某超市一月份销售某品牌酸奶统计表\n\n| 品种 | 原味 | 草莓味 | 芒果味 | 黄桃味 |\n| :---: | :---: | :---: | :---: | :---: |\n| 箱数 | 35 | 45 | 10 | 25 |\n\n某超市二月份销售某品牌酸奶统计表\n\n| 品种 | 原味 | 草莓味 | 芒果味 | 黄桃味 |\n| :---: | :---: | :---: | :---: | :---: |\n| 箱数 | 35 | 15 | 35 | 10 |\n\n根据上面两个表格完成下表:\n\n某超市一、二月份销售某品牌酸奶统计表\n\n| 数堇 品种 | | | | |\n| :--- | :--- | :--- | :--- | :--- |\n| 月份 | | | | |\n| | | | | |\n| | | | | |\n| | | | | |\n\n(1)一月份()味酸奶卖的最多,二月份()味酸奶卖的最少。\n\n(2)如果你是超市经理,结合统计表谈谈你的想法。", "options": [], "subject": "计数", "analysis": "解统计表见详解\n\n(1) 草莓; 黄桃\n\n(2)原味酸奶两个月卖的箱数都挺多,下个月可以多进一些原味酸奶。(答案不唯一)\n\n【分析】将两个表格中的数据分别整理到复式统计表中,即可完成复式统计表。\n\n(1)根据整理好的统计表,对比一月、二月的几种酸奶销量, 即可解答。\n\n(2)根据表中数据, 提出自己的想法即可。如原味酸奶两个月销量都较好, 下个月可以多进点原味酸奶。"} {"id": "9114", "image": [], "answer": "解$450 ; 600 ; 540 ; 810$\n\n$640 ; 470 ; 650 ; 640 ; 2400$\n\n(1) 卡通节目;新闻节目\n\n(2) 卡通节目; 电视剧\n\n(3) $2400 ; 180$\n\n【分析】(1) 依次求出各个栏目的总人数以及各个小学、中学的男、女生人数, 再填入统计表中。 (2) 比较喜欢各个栏目的总人数大小解答。\n\n(3) 比较小学女生喜欢各个栏目的人数大小解答。比较中学男生喜欢各个栏目的人数大小解答。\n\n(4) 先分别求出喜欢各个栏目的男生人数以及女生人数。再将男生人数加上女生人数, 求出总人数。用男生人数减去女生人数, 求出男生与女生相差的人数。\n\n【详解】\n\n| 人数 项 | 小学 | | 中学 | | 合计 |\n| :---: | :---: | :---: | :---: | :---: | :---: |\n| 栏目 | 男生 | 女生 | 男生 | 女生 | |\n| 新闻节目 | 120 | 80 | 160 | 90 | 450 |\n| 体育节目 | 150 | 90 | 200 | 160 | 600 |\n| 电视剧 | 170 | 120 | 100 | 150 | 540 |\n| 卡通节目 | 200 | 180 | 190 | 240 | 810 |\n| 总计 | 640 | 470 | 650 | 640 | 2400 |\n\n(1) $810>600>540>450$\n\n喜欢卡通节目的人最多,喜欢新闻节目的人最少。\n\n(2) $180>120>90>80$, 则小学女生中喜欢卡通节目的人最多。\n\n$200>190>160>100$, 则中学男生中喜欢电视剧的人最少。\n\n(3) $640+650=1290$ (人)\n\n$470+640=1110$ (人)\n\n$1290+1110=2400$ (人)\n\n$1290-1110=180$ (人)\n\n参加调查的学生共有 2400 人, 其中男生与女生相差 180 人。\n\n【点睛】此题考查的目的是理解掌握统计表的特点及作用, 并且能够根据统计表提供的信息, 解决有关的实际问题。", "solution": "null", "level": "三年级", "question": "下面是育英学校小学部和中学部学生喜欢的电视节目情况。(统计时每人只能选一种节目。)\n\n| 人数项 | 小学 | | 中学 | | 合计 |\n| :---: | :---: | :---: | :---: | :---: | :---: |\n| 目 | 男生 | 女生 | 男生 | 女生 | |\n| 新闻节目 | 120 | 80 | 160 | 90 | |\n| 体育节目 | 150 | 90 | 200 | 160 | |\n| 电视剧 | 170 | 120 | 100 | 150 | |\n| 卡通节目 | 200 | 180 | 190 | 240 | |\n| 总计 | | | | | |\n\n请把上表统计填完整,从调查统计表中可以看出:\n\n(1) 喜欢 ( ) 的人最多, 喜欢 ( ) 的人最少。\n\n(2)小学女生中喜欢()的人最多, 中学男生中喜欢( )的人最少。\n\n(3)参加调查的学生共有( )人,其中男生与女生相差()人。", "options": [], "subject": "计数", "analysis": "解$450 ; 600 ; 540 ; 810$\n\n$640 ; 470 ; 650 ; 640 ; 2400$\n\n(1) 卡通节目;新闻节目\n\n(2) 卡通节目; 电视剧\n\n(3) $2400 ; 180$\n\n【分析】(1) 依次求出各个栏目的总人数以及各个小学、中学的男、女生人数, 再填入统计表中。 (2) 比较喜欢各个栏目的总人数大小解答。\n\n(3) 比较小学女生喜欢各个栏目的人数大小解答。比较中学男生喜欢各个栏目的人数大小解答。\n\n(4) 先分别求出喜欢各个栏目的男生人数以及女生人数。再将男生人数加上女生人数, 求出总人数。用男生人数减去女生人数, 求出男生与女生相差的人数。\n\n【详解】\n\n| 人数 项 | 小学 | | 中学 | | 合计 |\n| :---: | :---: | :---: | :---: | :---: | :---: |\n| 栏目 | 男生 | 女生 | 男生 | 女生 | |\n| 新闻节目 | 120 | 80 | 160 | 90 | 450 |\n| 体育节目 | 150 | 90 | 200 | 160 | 600 |\n| 电视剧 | 170 | 120 | 100 | 150 | 540 |\n| 卡通节目 | 200 | 180 | 190 | 240 | 810 |\n| 总计 | 640 | 470 | 650 | 640 | 2400 |\n\n(1) $810>600>540>450$\n\n喜欢卡通节目的人最多,喜欢新闻节目的人最少。\n\n(2) $180>120>90>80$, 则小学女生中喜欢卡通节目的人最多。\n\n$200>190>160>100$, 则中学男生中喜欢电视剧的人最少。\n\n(3) $640+650=1290$ (人)\n\n$470+640=1110$ (人)\n\n$1290+1110=2400$ (人)\n\n$1290-1110=180$ (人)\n\n参加调查的学生共有 2400 人, 其中男生与女生相差 180 人。\n\n【点睛】此题考查的目的是理解掌握统计表的特点及作用, 并且能够根据统计表提供的信息, 解决有关的实际问题。"} {"id": "9115", "image": [], "answer": "解(1) 1\n\n(2) 18\n\n【分析】(1) 用体重在 $26-30$ 千克的二年级人数减去体重在 26-30千克的一年级人数解答。\n\n(2) 用一年级体重在 16-20千克的人数减去一年级体重在 $21-25$ 千克的人数解答。\n\n【详解】 (1) $7-6=1$ (千克) , 则体重在 $26-30$ 千克的一年级比二年级少 1 人。\n\n(2) $24-6=18$ (人), 则一年级体重在 $16-20$ 千克的比体重在 $21-25$ 千克的多 18 人。\n\n【点睛】此题考查的目的是理解掌握统计表的特点及作用, 并且能够根据统计表提供的信息, 解决有关的实际问题。", "solution": "null", "level": "三年级", "question": "下面是三年级某班全体学生在一、二年级时体重测试情况统计表。\n\n| 人数体重 | | | | | |\n| :---: | :---: | :---: | :---: | :---: | :---: |\n| 年级 | 15 岁及 15
岁以下 | $16-20$ | $21-25$ | $26-30$ | 30及 31
以上 |\n| 一年级 | 12 | 24 | 6 | 6 | 2 |\n| 二年级 | 1 | 9 | 27 | 7 | 5 |\n\n(1)体重在 26-30千克的一年级比二年级少( )。\n\n(2)一年级体重在 16-20千克的比体重在 $21-25$ 千克的多 $\\quad$ )。", "options": [], "subject": "计数", "analysis": "解(1) 1\n\n(2) 18\n\n【分析】(1) 用体重在 $26-30$ 千克的二年级人数减去体重在 26-30千克的一年级人数解答。\n\n(2) 用一年级体重在 16-20千克的人数减去一年级体重在 $21-25$ 千克的人数解答。\n\n【详解】 (1) $7-6=1$ (千克) , 则体重在 $26-30$ 千克的一年级比二年级少 1 人。\n\n(2) $24-6=18$ (人), 则一年级体重在 $16-20$ 千克的比体重在 $21-25$ 千克的多 18 人。\n\n【点睛】此题考查的目的是理解掌握统计表的特点及作用, 并且能够根据统计表提供的信息, 解决有关的实际问题。"} {"id": "9116", "image": [], "answer": "解(1) $49 \\quad 57$\n\n(2) $39 \\quad 30$\n\n$(3)$ 三 $(3)$\n\n(4) 见详解\n\n【分析】 (1) 将一 (3) 班各种龂齿数的人数相加求和。将三 (3) 班各种龉齿数的人数相加求和。 (2) 将一 (3) 班龄齿数为 1 颗、 2 颗、 3 颗以及 3 颗以上的人数相加求和。三 (3) 班龂齿数为 0 颗的人数就是牙齿健康的人数。\n\n(3) 比较两个班中各种龃齿数的人数大小解答。\n\n(4) 围绕爱护牙齿方面提出合理意见即可。\n\n【详解】 (1) $10+20+10+5+4=49$ (人)\n\n$30+14+8+4+1=57($ 人)\n\n一(3)班参加调查的一共有 49 人,三(3)班参加调查的一共有 57 人。\n\n(2) $20+10+5+4=39$ (人), 则一(3)班存在龊齿的有 39 人,三(3)班牙齿健康的有 30 人。\n\n(3) $10<30,20>14,10>8,5>4,4>1$\n\n则从表中分析, 三(3)班学生的牙齿比较健康。\n\n(4)你想对这样学生说: 要爱护牙齿, 少吃甜食, 保证每天两次刷牙, 每次刷牙不少于 1 分钟。\n\n【点睛】此题考查的目的是理解掌握统计表的特点及作用, 并且能够根据统计表提供的信息, 解决有关的实际问题。", "solution": "null", "level": "三年级", "question": "实验小学一 (3) 班和三 (3) 班学生牙齿情况调查结果如下表:\n\n| 人数 龂齿数 | 0颗 | 1颗 | 2颗 | 3颗 | 3颗以上 |\n| :---: | :---: | :---: | :---: | :---: | :---: |\n| 班级 | | | | | |\n| - (3) 班 | 10 | 20 | 10 | 5 | 4 |\n| 三(3) 班 | 30 | 14 | 8 | 4 | 1 |\n\n龄齿俗称虫牙、蛀牙, 是细菌性疾病, 请根据上面的牙齿情况调查统计表回答问题。\n\n(1)-(3)班参加调查的一共有( )人, 三(3)班参加调查的一共有( )人;\n\n(2)一(3)班存在龄齿的有 ( )人, 三 (3) 班牙齿健康的有 ( )人;\n\n(3)从表中分析, ( )班学生的牙齿比较健康。\n\n(4)据了解,低年级学生喜欢吃甜食,而且家长不督促就不刷牙,你想对这样学生说:( )。", "options": [], "subject": "计数", "analysis": "解(1) $49 \\quad 57$\n\n(2) $39 \\quad 30$\n\n$(3)$ 三 $(3)$\n\n(4) 见详解\n\n【分析】 (1) 将一 (3) 班各种龂齿数的人数相加求和。将三 (3) 班各种龉齿数的人数相加求和。 (2) 将一 (3) 班龄齿数为 1 颗、 2 颗、 3 颗以及 3 颗以上的人数相加求和。三 (3) 班龂齿数为 0 颗的人数就是牙齿健康的人数。\n\n(3) 比较两个班中各种龃齿数的人数大小解答。\n\n(4) 围绕爱护牙齿方面提出合理意见即可。\n\n【详解】 (1) $10+20+10+5+4=49$ (人)\n\n$30+14+8+4+1=57($ 人)\n\n一(3)班参加调查的一共有 49 人,三(3)班参加调查的一共有 57 人。\n\n(2) $20+10+5+4=39$ (人), 则一(3)班存在龊齿的有 39 人,三(3)班牙齿健康的有 30 人。\n\n(3) $10<30,20>14,10>8,5>4,4>1$\n\n则从表中分析, 三(3)班学生的牙齿比较健康。\n\n(4)你想对这样学生说: 要爱护牙齿, 少吃甜食, 保证每天两次刷牙, 每次刷牙不少于 1 分钟。\n\n【点睛】此题考查的目的是理解掌握统计表的特点及作用, 并且能够根据统计表提供的信息, 解决有关的实际问题。"} {"id": "9118", "image": [], "answer": "解(1) 六 一\n\n$(2) 12$\n\n(3) 182 六\n\n【分析】(1)观察统计表并比较表中数据可知, 六年级的男生人数最多, 一年级的女生人数最少。\n\n(2)观察统计表可知, 三年级的男生比女生人数少(104-92)人。\n\n(3)分别求出各班的总人数, 再比较即可。\n\n【详解】 (1) $115>107>92>88>84>78$\n\n$104>96>94>86>75>73$\n\n六年级的男生人数最多, 一年级的女生人数最少。\n\n(2) $104-92=12($ 人)\n\n所以, 三年级的男生比女生人数少 12 人。\n\n(3) $78+73=151$ (人)\n\n$88+94=182($ 人)\n\n$92+104=196($ 人 $)$\n\n$84+75=159($ 人 $)$\n\n$107+86=193($ 人 $)$\n\n$115+96=211($ 人)\n\n$211>196>193>182>159>151$\n\n二年级一共有 182 人, 六年级的人数最多。\n\n【点睛】熟练掌握从统计表的数据中获取信息的方法, 是解答此题的关键。", "solution": "null", "level": "三年级", "question": "根据下面统计图, 回答下列问题。\n\n| 人数 | 一年级 | 二年级 | 三年级 | 四年级 | 五年级 | 六年级 |\n| :---: | :---: | :---: | :---: | :---: | :---: | :---: |\n| 男生 | 78 | 88 | 92 | 84 | 107 | 115 |\n| 女生 | 73 | 94 | 104 | 75 | 86 | 96 |\n\n$(1)(\\quad)$ 年级的男生人数最多, $(\\quad)$ 年级的女生人数最少。\n\n(2)三年级的男生比女生人数少 $(\\quad)$ 人。\n\n(3)二年级一共有 $(\\quad)$ 人, $(\\quad$ 年级的人数最多。", "options": [], "subject": "计数", "analysis": "解(1) 六 一\n\n$(2) 12$\n\n(3) 182 六\n\n【分析】(1)观察统计表并比较表中数据可知, 六年级的男生人数最多, 一年级的女生人数最少。\n\n(2)观察统计表可知, 三年级的男生比女生人数少(104-92)人。\n\n(3)分别求出各班的总人数, 再比较即可。\n\n【详解】 (1) $115>107>92>88>84>78$\n\n$104>96>94>86>75>73$\n\n六年级的男生人数最多, 一年级的女生人数最少。\n\n(2) $104-92=12($ 人)\n\n所以, 三年级的男生比女生人数少 12 人。\n\n(3) $78+73=151$ (人)\n\n$88+94=182($ 人)\n\n$92+104=196($ 人 $)$\n\n$84+75=159($ 人 $)$\n\n$107+86=193($ 人 $)$\n\n$115+96=211($ 人)\n\n$211>196>193>182>159>151$\n\n二年级一共有 182 人, 六年级的人数最多。\n\n【点睛】熟练掌握从统计表的数据中获取信息的方法, 是解答此题的关键。"} {"id": "9119", "image": [], "answer": "解(1)图见详解\n\n(2) 5\n\n(3) 女\n\n【分析】(1)分别数出各个等级的男生人数以及女生人数, 再填入统计表中。\n\n(2) 将等级为优秀的男生人数加上女生人数解答。\n\n(3)根据统计表, 比较各个等级的男生人数与女生人数大小解答。\n\n【详解】(1)由分析得:\n\n| 次数等级 | 优秀 | 良好 | 及格 | 不及格 |\n| :---: | :---: | :---: | :---: | :---: |\n| 性别 | | | | |\n| 男生 | 2 | 1 | 2 | 1 |\n| 女生 | 3 | 2 | 1 | 0 |\n\n(2) $2+3=5$ (名)\n\n这个组一共有 5 名学生达到优秀。\n\n(3) $2<3,1<2,2>1,1>0$\n\n则等级为优秀和良好, 女生人数比男生人数多。等级为及格和不及格, 女生人数比男生人数少。综合分析,这个组女生的跳绳成绩更好。\n\n【点睛】本题考查复式统计表的数据分析和整理。根据统计表找出需要的数据, 列式计算即可。", "solution": "null", "level": "三年级", "question": "小刚在体育课上记录了本小组 12 名同学 1 分钟跳绳的次数。\n\n| 姓名 | 性
别 | 跳绳/
次 | 姓名 | 性
别 | 跳绳/
次 | 姓名 | 性
别 | 跳绳/
次 | 姓名 | 性
别 | 跳绳/
次 |\n| :---: | :---: | :---: | :---: | :---: | :---: | :---: | :---: | :---: | :---: | :---: | :---: |\n| 刘阳 | 男 | 120 | 孙小
敏 | 女 | 110 | 李军 | 男 | 90 | 吴为
冬 | 男 | 92 |\n| 张再
手 | 女 | 132 | 魏芳 | 女 | 117 | 赵继
业 | 男 | 118 | 钱亮 | 男 | 25 |\n| 徐小 | 男 | 105 | 王露 | 女 | 130 | 张放 | 女 | 76 | 田云 | 女 | 130 |\n\n\n| 明 | | | | | | | | | $飞$ | | |\n| :--- | :--- | :--- | :--- | :--- | :--- | :--- | :--- | :--- | :--- | :--- | :--- |\n\n《国家学生体质健康测试标准》对三年级学生 1 分钟跳绳的成绩规定如下:\n\n| 次数 等级 | 优秀 | 良好 | 及格 | 不及格 |\n| :---: | :---: | :---: | :---: | :---: |\n| 性别 | | | | |\n| 男生 | 116 以上 | $104-115$ | $34-103$ | 34以下 |\n| 女生 | 125 以上 | $109-124$ | $39-108$ | 39 以下 |\n\n(1)请你根据上面的标准帮小刚把他们组的跳绳成绩整理在下表中。\n\n| 次数 等级 | 优秀 | 良好 | 及格 | 不及格 |\n| :---: | :---: | :---: | :---: | :---: |\n| 性别 | | | | |\n| 男生 | | | | |\n| 女生 | | | | |\n\n(2)这个组一共有()名学生达到优秀。\n\n(3)男生和女生比较, 这个组()生的跳绳成绩更好。", "options": [], "subject": "计数", "analysis": "解(1)图见详解\n\n(2) 5\n\n(3) 女\n\n【分析】(1)分别数出各个等级的男生人数以及女生人数, 再填入统计表中。\n\n(2) 将等级为优秀的男生人数加上女生人数解答。\n\n(3)根据统计表, 比较各个等级的男生人数与女生人数大小解答。\n\n【详解】(1)由分析得:\n\n| 次数等级 | 优秀 | 良好 | 及格 | 不及格 |\n| :---: | :---: | :---: | :---: | :---: |\n| 性别 | | | | |\n| 男生 | 2 | 1 | 2 | 1 |\n| 女生 | 3 | 2 | 1 | 0 |\n\n(2) $2+3=5$ (名)\n\n这个组一共有 5 名学生达到优秀。\n\n(3) $2<3,1<2,2>1,1>0$\n\n则等级为优秀和良好, 女生人数比男生人数多。等级为及格和不及格, 女生人数比男生人数少。综合分析,这个组女生的跳绳成绩更好。\n\n【点睛】本题考查复式统计表的数据分析和整理。根据统计表找出需要的数据, 列式计算即可。"} {"id": "9195", "image": ["1514.jpg"], "answer": "解$\\quad 10 \\quad 4$\n\n【分析】每个小格的面积为 1 平方厘米, 所以每个小格的边长为 1 厘米, 上面图形的周长与长为 3 厘米、宽为 2 厘米的长方形的周长相等, 1 个小格的面积乘小格的个数即等于该图形的面积; 据此即可解答。\n\n【洋解】每个小格的面积为 1 平方厘米, 所以每个小格的边长为 1 厘米。\n\n$$\n(3+2) \\times 2\n$$\n\n$=5 \\times 2$\n\n$=10$ (厘米)\n\n$1 \\times 4=4$ (平方厘米)\n\n如图每个小格为 1 平方厘米, 拼起来的图形周长是 10 厘米, 面积是 4 平方厘米。\n\n【点睛】本题主要考查学生对周长和面积的认识, 把求组合图形的周长转化为求长方形的周长是解答本题的关键。", "solution": "null", "level": "三年级", "question": "如图每个小格为 1 平方厘米, 拼起来的图形周长是 (\n\n) 厘米,面积是 (\n\n) 平方厘米。\n\n", "options": [], "subject": "度量几何学", "analysis": "解$\\quad 10 \\quad 4$\n\n【分析】每个小格的面积为 1 平方厘米, 所以每个小格的边长为 1 厘米, 上面图形的周长与长为 3 厘米、宽为 2 厘米的长方形的周长相等, 1 个小格的面积乘小格的个数即等于该图形的面积; 据此即可解答。\n\n【洋解】每个小格的面积为 1 平方厘米, 所以每个小格的边长为 1 厘米。\n\n$$\n(3+2) \\times 2\n$$\n\n$=5 \\times 2$\n\n$=10$ (厘米)\n\n$1 \\times 4=4$ (平方厘米)\n\n如图每个小格为 1 平方厘米, 拼起来的图形周长是 10 厘米, 面积是 4 平方厘米。\n\n【点睛】本题主要考查学生对周长和面积的认识, 把求组合图形的周长转化为求长方形的周长是解答本题的关键。"} {"id": "9196", "image": ["1515.jpg"], "answer": "解$72 \\quad 1728$\n\n【分析】花坛的宽乘 3 等于花坛的长, 然后花坛的长乘宽即等于花坛的面积。\n\n【详解】 $24 \\times 3=72$ (米)\n$72 \\times 24=1728($ 平方米 $)$\n\n花坛的长为 72 米, 这个花坛的面积是 1728 平方米。\n\n【点睛】本题主要考查学生对长方形面积公式的掌握, 先计算出花坛的长是解答本题的关键。", "solution": "null", "level": "三年级", "question": "光明小学在修建一个长方形花坛, 宽为 24 米, 长是宽的 3 倍, 长为\n\n) 米,这个花坛的面积是 ( )平方米。\n\n", "options": [], "subject": "度量几何学", "analysis": "解$72 \\quad 1728$\n\n【分析】花坛的宽乘 3 等于花坛的长, 然后花坛的长乘宽即等于花坛的面积。\n\n【详解】 $24 \\times 3=72$ (米)\n$72 \\times 24=1728($ 平方米 $)$\n\n花坛的长为 72 米, 这个花坛的面积是 1728 平方米。\n\n【点睛】本题主要考查学生对长方形面积公式的掌握, 先计算出花坛的长是解答本题的关键。"} {"id": "9197", "image": [], "answer": "解$7800 \\quad 78$\n\n【分析】长方形的面积 $=$ 长 $\\times$ 宽, 把数据代入计算出写字台的面积, 1 平方分米 $=100$ 平方厘米, 把平方厘米换算成平方分米,据此即可解答。\n\n【详解】 $130 \\times 60=7800$ (平方厘米) $=78$ 平方分米\n\n它的面积是 7800 平方厘米,合 78 平方分米。\n\n【点睛】本题主要考查学生对长方形面积公式及面积单位换算知识的掌握。", "solution": "null", "level": "三年级", "question": "一个长方形写字台长 130 厘米, 宽 60 厘米。它的面积是 (\n\n) 平方厘米,合 (\n\n)平方分米。", "options": [], "subject": "度量几何学", "analysis": "解$7800 \\quad 78$\n\n【分析】长方形的面积 $=$ 长 $\\times$ 宽, 把数据代入计算出写字台的面积, 1 平方分米 $=100$ 平方厘米, 把平方厘米换算成平方分米,据此即可解答。\n\n【详解】 $130 \\times 60=7800$ (平方厘米) $=78$ 平方分米\n\n它的面积是 7800 平方厘米,合 78 平方分米。\n\n【点睛】本题主要考查学生对长方形面积公式及面积单位换算知识的掌握。"} {"id": "9205", "image": ["1518.jpg"], "answer": "解320 平方米\n\n【分析】根据长方形的面积 $=$ 长 $\\times$ 宽, 已知长方形的面积和宽, 用除法即可得到长方形的长, 然后乘扩建后长方形的宽即可得解。\n\n【详解】 $160 \\div 8 \\times 16$\n\n$=20 \\times 16$\n\n$=320$ (平方米)\n\n答:扩建后的绿地一共是 320 平方米。\n\n【点睛】清楚长方形的面积公式是解答此题的关键。", "solution": "null", "level": "三年级", "question": "一个长方形绿地面积是 160 平方米, 宽 8 米, 扩建后的绿地一共是多少平方米?\n\n", "options": [], "subject": "度量几何学", "analysis": "解320 平方米\n\n【分析】根据长方形的面积 $=$ 长 $\\times$ 宽, 已知长方形的面积和宽, 用除法即可得到长方形的长, 然后乘扩建后长方形的宽即可得解。\n\n【详解】 $160 \\div 8 \\times 16$\n\n$=20 \\times 16$\n\n$=320$ (平方米)\n\n答:扩建后的绿地一共是 320 平方米。\n\n【点睛】清楚长方形的面积公式是解答此题的关键。"} {"id": "9207", "image": [], "answer": "解2000 平方厘米, 20 平方分米\n\n【分析】根据长方形的面积 $=$ 长 $\\times$ 宽, 求出油画作品的面积。平方厘米和平方分米之间的进率是 100 ,据此将油画作品的面积换算成平方分米。\n\n【详解】 $80 \\times 25=2000$ (平方厘米)\n\n2000 平方厘米 $=20$ 平方分米\n\n答: 这幅油画作品的面积是 2000 平方厘米, 合 20 平方分米。\n\n【点睛】本题考查长方形面积公式的应用以及面积单位的换算, 关键是熟记公式。", "solution": "null", "level": "三年级", "question": "一幅长方形油画作品长 80 厘米, 宽 25 厘米, 这幅油画作品的面积是多少平方厘米? 合多少平方分米?", "options": [], "subject": "度量几何学", "analysis": "解2000 平方厘米, 20 平方分米\n\n【分析】根据长方形的面积 $=$ 长 $\\times$ 宽, 求出油画作品的面积。平方厘米和平方分米之间的进率是 100 ,据此将油画作品的面积换算成平方分米。\n\n【详解】 $80 \\times 25=2000$ (平方厘米)\n\n2000 平方厘米 $=20$ 平方分米\n\n答: 这幅油画作品的面积是 2000 平方厘米, 合 20 平方分米。\n\n【点睛】本题考查长方形面积公式的应用以及面积单位的换算, 关键是熟记公式。"} {"id": "9215", "image": [], "answer": "解平方厘米\\#\\# $\\mathrm{cm}^{2}$ 厘米\\#\\# $\\mathrm{cm} \\quad$ 平方米 $\\# \\mathrm{~m}^{2} \\quad$ 米\\#\\# $\\mathrm{m}$\n\n【分析】 1 平方厘米约为拇指甲的大小,因此计量电话手表表面的面积以“平方厘米”为单位。小学生双手张开, 手掌之间的距离大约是 100 厘米, 根据对长度单位的认识, 以及结合实际数据可知,计量莹莹的身高以“厘米”为单位。\n\n1 平方米大约有一大块正方形地砖那么大,因此计量卧室的面积以“平方米”为单位。小学生双手张开, 手掌之间的距离大约是 1 米, 因此根据实际数据可知, 计量单人床的长度以“米”为单位。\n\n【详解】根据分析,填空如下:\n\n电话手表表面的面积约 9 平方厘米; 莹莹的身高是 125 厘米; 小军的卧室占地面积约 12 平方米; 单人床的长是 2 米。\n\n【点睛】此题考查的是结合数据选择合适的单位名称, 熟练掌握对长度单位、面积单位的认识是解答此题的关键。", "solution": "null", "level": "三年级", "question": "电话手表表面的面积约 9(\n\n) ;莹莹的身高是 125(\n\n);小军的卧室占地面积约\n\n12( ); 单人床的长是 $2(\\quad)$ 。", "options": [], "subject": "度量几何学", "analysis": "解平方厘米\\#\\# $\\mathrm{cm}^{2}$ 厘米\\#\\# $\\mathrm{cm} \\quad$ 平方米 $\\# \\mathrm{~m}^{2} \\quad$ 米\\#\\# $\\mathrm{m}$\n\n【分析】 1 平方厘米约为拇指甲的大小,因此计量电话手表表面的面积以“平方厘米”为单位。小学生双手张开, 手掌之间的距离大约是 100 厘米, 根据对长度单位的认识, 以及结合实际数据可知,计量莹莹的身高以“厘米”为单位。\n\n1 平方米大约有一大块正方形地砖那么大,因此计量卧室的面积以“平方米”为单位。小学生双手张开, 手掌之间的距离大约是 1 米, 因此根据实际数据可知, 计量单人床的长度以“米”为单位。\n\n【详解】根据分析,填空如下:\n\n电话手表表面的面积约 9 平方厘米; 莹莹的身高是 125 厘米; 小军的卧室占地面积约 12 平方米; 单人床的长是 2 米。\n\n【点睛】此题考查的是结合数据选择合适的单位名称, 熟练掌握对长度单位、面积单位的认识是解答此题的关键。"} {"id": "9216", "image": [], "answer": "解$\\quad 100 \\quad 40$\n\n【分析】正方形的面积 $=$ 边长 $\\times$ 边长, 正方形的周长 $=$ 边长 $\\times 4$ 。根据正方形的面积和正方形的周长计算公式,即可解答。\n【详解】 $10 \\times 10=100$ (平方米)\n\n$10 \\times 4=40$ (米)\n\n一个正方形的边长是 10 米, 它的面积是 100 平方米, 周长是 40 米。\n\n【点睛】本题主要考查正方形的周长公式和面积公式, 属于基础知识, 要熟练掌握。", "solution": "null", "level": "三年级", "question": "一个正方形的边长是 10 米, 它的面积是(\n\n) 平方米, 周长是 (\n\n)米。", "options": [], "subject": "度量几何学", "analysis": "解$\\quad 100 \\quad 40$\n\n【分析】正方形的面积 $=$ 边长 $\\times$ 边长, 正方形的周长 $=$ 边长 $\\times 4$ 。根据正方形的面积和正方形的周长计算公式,即可解答。\n【详解】 $10 \\times 10=100$ (平方米)\n\n$10 \\times 4=40$ (米)\n\n一个正方形的边长是 10 米, 它的面积是 100 平方米, 周长是 40 米。\n\n【点睛】本题主要考查正方形的周长公式和面积公式, 属于基础知识, 要熟练掌握。"} {"id": "9241", "image": [], "answer": "解厘米\\#\\# $\\mathrm{cm}$ 厘米\\#\\# 450 平方厘米\\#\\# $450 \\mathrm{~cm}^{2}$\n\n【分析】根据生活经验以及对长度单位和数据大小的认识, 结合实际情况可知: 一本笔记本的封面的长 25 厘米, 宽 18 厘米, 再根据“长方形面积 $=$ 长×宽”, 求出这个笔记本封面的面积即可。\n\n【详解】根据分析可知,\n$25 \\times 18=450$ (平方厘米)\n\n一本笔记本的封面的长 25 厘米, 宽 18 厘米, 面积是 450 平方厘米。\n\n【点睛】此题考查根据情景选择合适的计量单位, 要注意联系生活实际、计量单位和数据的大小, 灵活的选择。", "solution": "null", "level": "三年级", "question": "一本笔记本的封面的长 $25(\\quad)$, 宽 $18(\\quad)$, 面积是 ( )。", "options": [], "subject": "度量几何学", "analysis": "解厘米\\#\\# $\\mathrm{cm}$ 厘米\\#\\# 450 平方厘米\\#\\# $450 \\mathrm{~cm}^{2}$\n\n【分析】根据生活经验以及对长度单位和数据大小的认识, 结合实际情况可知: 一本笔记本的封面的长 25 厘米, 宽 18 厘米, 再根据“长方形面积 $=$ 长×宽”, 求出这个笔记本封面的面积即可。\n\n【详解】根据分析可知,\n$25 \\times 18=450$ (平方厘米)\n\n一本笔记本的封面的长 25 厘米, 宽 18 厘米, 面积是 450 平方厘米。\n\n【点睛】此题考查根据情景选择合适的计量单位, 要注意联系生活实际、计量单位和数据的大小, 灵活的选择。"} {"id": "9242", "image": [], "answer": "解25 平方厘米\\#\\# $25 \\mathrm{~cm}^{2} \\quad 15$ 平方厘米\\#\\# $15 \\mathrm{~cm}^{2}$\n\n【分析】在长方形中剪下一个面积最大的正方形, 正方形的边长是 5 厘米, 正方形的面积 $=$ 边长 $\\times$ 边长; 长方形面积 $=$ 长 $\\times$ 宽, 剩下的纸的面积 $=$ 长方形面积一正方形面积; 据此解答。\n\n【详解】根据分析:\n\n(1) $5 \\times 5=25$ (平方厘米), 则这个正方形的面积是 25 平方厘米;\n\n(2) $8 \\times 5=40$ (平方厘米) , $40-25=15$ (平方厘米),则剩下的纸的面积是 15 平方厘米。\n\n【点睛】 注意从长方形纸上剪下一个面积最大的正方形, 则这个正方形的边长等于长方形的宽, 再根据正方形的面积公式和长方形的面积解答。", "solution": "null", "level": "三年级", "question": "从一张长 8 厘米、宽 5 厘米的长方形纸上剪下一个面积最大的正方形, 这个正方形的面积是\n\n( ), 剩下的纸的面积是 ( )。", "options": [], "subject": "度量几何学", "analysis": "解25 平方厘米\\#\\# $25 \\mathrm{~cm}^{2} \\quad 15$ 平方厘米\\#\\# $15 \\mathrm{~cm}^{2}$\n\n【分析】在长方形中剪下一个面积最大的正方形, 正方形的边长是 5 厘米, 正方形的面积 $=$ 边长 $\\times$ 边长; 长方形面积 $=$ 长 $\\times$ 宽, 剩下的纸的面积 $=$ 长方形面积一正方形面积; 据此解答。\n\n【详解】根据分析:\n\n(1) $5 \\times 5=25$ (平方厘米), 则这个正方形的面积是 25 平方厘米;\n\n(2) $8 \\times 5=40$ (平方厘米) , $40-25=15$ (平方厘米),则剩下的纸的面积是 15 平方厘米。\n\n【点睛】 注意从长方形纸上剪下一个面积最大的正方形, 则这个正方形的边长等于长方形的宽, 再根据正方形的面积公式和长方形的面积解答。"} {"id": "9243", "image": ["1526.jpg"], "answer": "解36\n\n【分析】根据题图可知, 图案中完整的方格有 32 个, 不完整的方格有 8 个, 把不完整格按半格计算,则图案的面积有 $(32+8 \\div 2) \\mathrm{cm}^{2}$ 。\n\n【详解】 $32+8 \\div 2$\n\n$=32+4$\n\n$=36\\left(\\mathrm{~cm}^{2}\\right)$\n\n则方格纸中图案的面积是 $36 \\mathrm{~cm}^{2}$ 。\n\n【点睛】本题考查不规则图形的面积, 用数格子的方法进行估算, 关键是明确把不完整格按半格计算。", "solution": "null", "level": "三年级", "question": "下图方格纸中图案的面积是 $(\\quad) \\mathrm{cm}^{2}$ 。\n\n", "options": [], "subject": "度量几何学", "analysis": "解36\n\n【分析】根据题图可知, 图案中完整的方格有 32 个, 不完整的方格有 8 个, 把不完整格按半格计算,则图案的面积有 $(32+8 \\div 2) \\mathrm{cm}^{2}$ 。\n\n【详解】 $32+8 \\div 2$\n\n$=32+4$\n\n$=36\\left(\\mathrm{~cm}^{2}\\right)$\n\n则方格纸中图案的面积是 $36 \\mathrm{~cm}^{2}$ 。\n\n【点睛】本题考查不规则图形的面积, 用数格子的方法进行估算, 关键是明确把不完整格按半格计算。"} {"id": "9244", "image": [], "answer": "解240\n\n【分析】长方形的面积 $\\mathrm{S}=\\mathrm{ab}$, 根据因数与积的变化规律, 长扩大到原来的 3 倍, 宽扩大到原来的 2 倍, 长方形的面积就扩大到原来的 $3 \\times 2$ 倍, 据此解答。\n\n【详解】 $40 \\times(3 \\times 2)$\n\n$=40 \\times 6$\n\n$=240($ 平方厘米 $)$\n\n所以它现在的面积是 240 平方厘米。\n\n【点睛】本题主要考查了长方形面积公式及积的变化规律的灵活应用。", "solution": "null", "level": "三年级", "question": "一个长方形的面积是 40 平方厘米, 如果长扩大到原来的 3 倍, 宽扩大到原来的 2 倍, 它现在的面积是 $(\\quad)$ 平方厘米。", "options": [], "subject": "度量几何学", "analysis": "解240\n\n【分析】长方形的面积 $\\mathrm{S}=\\mathrm{ab}$, 根据因数与积的变化规律, 长扩大到原来的 3 倍, 宽扩大到原来的 2 倍, 长方形的面积就扩大到原来的 $3 \\times 2$ 倍, 据此解答。\n\n【详解】 $40 \\times(3 \\times 2)$\n\n$=40 \\times 6$\n\n$=240($ 平方厘米 $)$\n\n所以它现在的面积是 240 平方厘米。\n\n【点睛】本题主要考查了长方形面积公式及积的变化规律的灵活应用。"} {"id": "9253", "image": ["1528.jpg", "1529.jpg", "1529.jpg"], "answer": "解图见详解; 面积相等的长方形和正方形的周长不一定相等, 且长方形的周长大于正方形的周长\n\n【分析】根据“每个口代表 1 平方厘米”可知, 口的边长是 1 厘米; 根据“长方形面积=长×宽、正方形面积 $=$ 边长 $\\times$ 边长\", 又: $16=16 \\times 1=8 \\times 2=4 \\times 4$, 据此可以画一个长 8 厘米、宽 2 厘米的长方形, 一个边长 4 厘米的正方形即可; 再根据“长方形周长 $=($ 长 + 宽 $) \\times 2 、$ 正方形周长 $=$ 边长 $\\times 4$ ”, 分别求出所画长方形、正方形的周长, 再比较即可。\n\n【详解】 $16=16 \\times 1=8 \\times 2=4 \\times 4$\n\n画一个长 8 厘米、宽 2 厘米的长方形, 一个边长 4 厘米的正方形, 如下:\n\n\n\n(长方形的画法不唯一)\n\n$(8+2) \\times 2$\n\n$=10 \\times 2$\n\n$=20$ (厘米)\n$4 \\times 4=16$ (厘米)\n\n$20>16$\n\n我发现:面积相等的长方形和正方形的周长不一定相等, 且长方形的周长大于正方形的周长。\n\n【点睛】熟记长方形、正方形面积计算公式、长方形、正方形周长计算公式, 是解答此题的关键。", "solution": "null", "level": "三年级", "question": "分别画出面积为 16 平方厘米的长方形和正方形, 然后比较一下它们的周长, 你发现了什么? (每个口代表 1 平方厘米)\n\n\n\n我发现", "options": [], "subject": "度量几何学", "analysis": "解图见详解; 面积相等的长方形和正方形的周长不一定相等, 且长方形的周长大于正方形的周长\n\n【分析】根据“每个口代表 1 平方厘米”可知, 口的边长是 1 厘米; 根据“长方形面积=长×宽、正方形面积 $=$ 边长 $\\times$ 边长\", 又: $16=16 \\times 1=8 \\times 2=4 \\times 4$, 据此可以画一个长 8 厘米、宽 2 厘米的长方形, 一个边长 4 厘米的正方形即可; 再根据“长方形周长 $=($ 长 + 宽 $) \\times 2 、$ 正方形周长 $=$ 边长 $\\times 4$ ”, 分别求出所画长方形、正方形的周长, 再比较即可。\n\n【详解】 $16=16 \\times 1=8 \\times 2=4 \\times 4$\n\n画一个长 8 厘米、宽 2 厘米的长方形, 一个边长 4 厘米的正方形, 如下:\n\n\n\n(长方形的画法不唯一)\n\n$(8+2) \\times 2$\n\n$=10 \\times 2$\n\n$=20$ (厘米)\n$4 \\times 4=16$ (厘米)\n\n$20>16$\n\n我发现:面积相等的长方形和正方形的周长不一定相等, 且长方形的周长大于正方形的周长。\n\n【点睛】熟记长方形、正方形面积计算公式、长方形、正方形周长计算公式, 是解答此题的关键。"} {"id": "9254", "image": [], "answer": "解108 平方分米\n\n【分析】根据“长方形面积=长 $\\times$ 宽”,计算出这幅汴绣的面积。\n\n【详解】 $18 \\times 6=108$ (平方分米)\n\n答:这幅汴绣的面积是 108 平方分米。\n\n【点睛】熟记长方形面积计算公式, 是解答此题的关键。", "solution": "null", "level": "三年级", "question": "油绣, 素有“国里宝”之称。王阿姨绣了一幅长 18 分米、宽 6 分米的长方形油绣。这幅油绣的面积是多少平方分米?", "options": [], "subject": "度量几何学", "analysis": "解108 平方分米\n\n【分析】根据“长方形面积=长 $\\times$ 宽”,计算出这幅汴绣的面积。\n\n【详解】 $18 \\times 6=108$ (平方分米)\n\n答:这幅汴绣的面积是 108 平方分米。\n\n【点睛】熟记长方形面积计算公式, 是解答此题的关键。"} {"id": "9255", "image": ["1530.jpg"], "answer": "解(1) 264 平方米\n\n(2) 72 平方米; 7200 平方分米\n\n【分析】(1) 根据长方形的面积 $=$ 长 $\\times$ 宽, 将长方形花坛的长和宽代入到长方形面积公式中, 即可算出这个花坛的面积是多少平方米。\n\n(2) 花坛的四周有一条 1 米宽的小路, 则花坛的长两端各多出了 1 米, 花坛的宽两端各多出了 1 米,根据长方形的面积 $=$ 长 $\\times$ 宽, 先算出小路和花坛的总面积是多少, 再减去花坛的面积, 即可得到小路的面积是多少平方米,再根据 1 平方米 $=100$ 平方分米,进行单位换算即可。\n\n【详解】 (1) $22 \\times 12=264$ (平方米)\n\n答:花坛的面积是 264 平方米。\n\n(2) $22+2=24($ 米)\n\n$12+2=14($ 米 $)$\n\n$24 \\times 14=336$ (平方米)\n\n$336-264=72$ (平方米)\n\n72 平方米 $=7200$ 平方分米\n\n答: 小路的面积是 72 平方米, 合 7200 平方分米。\n\n【点睛】本题主要考查长方形面积公式的灵活运用, 以及面积单位的换算, 属于基础知识, 要熟练掌握。", "solution": "null", "level": "三年级", "question": "如图, 长方形花坛长 22 米、宽 12 米, 它的四周有一条 1 米宽的小路。\n\n\n\n(1) 花坛的面积是多少平方米?\n\n(2)小路的面积是多少平方米?合多少平方分米?", "options": [], "subject": "度量几何学", "analysis": "解(1) 264 平方米\n\n(2) 72 平方米; 7200 平方分米\n\n【分析】(1) 根据长方形的面积 $=$ 长 $\\times$ 宽, 将长方形花坛的长和宽代入到长方形面积公式中, 即可算出这个花坛的面积是多少平方米。\n\n(2) 花坛的四周有一条 1 米宽的小路, 则花坛的长两端各多出了 1 米, 花坛的宽两端各多出了 1 米,根据长方形的面积 $=$ 长 $\\times$ 宽, 先算出小路和花坛的总面积是多少, 再减去花坛的面积, 即可得到小路的面积是多少平方米,再根据 1 平方米 $=100$ 平方分米,进行单位换算即可。\n\n【详解】 (1) $22 \\times 12=264$ (平方米)\n\n答:花坛的面积是 264 平方米。\n\n(2) $22+2=24($ 米)\n\n$12+2=14($ 米 $)$\n\n$24 \\times 14=336$ (平方米)\n\n$336-264=72$ (平方米)\n\n72 平方米 $=7200$ 平方分米\n\n答: 小路的面积是 72 平方米, 合 7200 平方分米。\n\n【点睛】本题主要考查长方形面积公式的灵活运用, 以及面积单位的换算, 属于基础知识, 要熟练掌握。"} {"id": "9256", "image": [], "answer": "解300 平方分米; 3 平方米\n\n【分析】根据长方形的面积 $=$ 长 $\\times$ 宽, 用这个长方形花圃的长乘宽, 即可算出这个长方形的花圃面积是多少平方分米, 再根据 100 平方分米 $=1$ 平方米, 进行单位换算即可。据此解答。\n\n【详解】 $30 \\times 10=300$ (平方分米)\n\n300 平方分米 $=3$ 平方米\n答: 这个花围的面积是 300 平方分米, 合 3 平方米。\n\n【点睛】本题主要考查长方形面积计算公式和面积单位间的换算, 属于基础知识, 要熟练掌握。", "solution": "null", "level": "三年级", "question": "一个长方形花圃, 长 30 分米、宽 10 分米, 这个花圃的面积是多少平方分米, 合多少平方米?", "options": [], "subject": "度量几何学", "analysis": "解300 平方分米; 3 平方米\n\n【分析】根据长方形的面积 $=$ 长 $\\times$ 宽, 用这个长方形花圃的长乘宽, 即可算出这个长方形的花圃面积是多少平方分米, 再根据 100 平方分米 $=1$ 平方米, 进行单位换算即可。据此解答。\n\n【详解】 $30 \\times 10=300$ (平方分米)\n\n300 平方分米 $=3$ 平方米\n答: 这个花围的面积是 300 平方分米, 合 3 平方米。\n\n【点睛】本题主要考查长方形面积计算公式和面积单位间的换算, 属于基础知识, 要熟练掌握。"} {"id": "9257", "image": [], "answer": "解(1) 100 平方米\n\n(2) 154 平方米\n\n【分析】(1) 长方形的长=面积:宽, 依此计算出长方形绿地原来的长, 再用原来的宽减 2 米, 计算出现在长方形绿地的宽, 再根据“长方形的面积 $=$ 长×宽”计算出减少后的面积即可。\n\n(2) 先用长方形绿地原来的长加 2 米, 从而计算出长方形绿地现在的长, 再根据“长方形的面积 $=$ 长 $\\times$ 宽”计算出增加后的面积即可。\n\n【详解】 (1) $140 \\div 7=20$ (米)\n\n$7-2=5$ (米)\n\n$20 \\times 5=100$ (平方米)\n\n答: 减少后的面积是 100 平方米。\n\n(2) $20+2=22$ (米)\n\n$22 \\times 7=154$ (平方米)\n\n答: 增加后的面积是 154 平方米。\n\n【点睛】此题考查的是长方形面积的计算,先计算出长方形绿地原来的长是解答此题的关键。", "solution": "null", "level": "三年级", "question": "有一块长方形绿地 (如下图) :\n\n7米\n\n140 平方米\n\n(1) 如果它的长不变, 宽减少 2 米, 减少后的面积是多少平方米?\n\n(2)如果它的宽不变, 长增加 2 米, 增加后的面积是多少平方米?", "options": [], "subject": "度量几何学", "analysis": "解(1) 100 平方米\n\n(2) 154 平方米\n\n【分析】(1) 长方形的长=面积:宽, 依此计算出长方形绿地原来的长, 再用原来的宽减 2 米, 计算出现在长方形绿地的宽, 再根据“长方形的面积 $=$ 长×宽”计算出减少后的面积即可。\n\n(2) 先用长方形绿地原来的长加 2 米, 从而计算出长方形绿地现在的长, 再根据“长方形的面积 $=$ 长 $\\times$ 宽”计算出增加后的面积即可。\n\n【详解】 (1) $140 \\div 7=20$ (米)\n\n$7-2=5$ (米)\n\n$20 \\times 5=100$ (平方米)\n\n答: 减少后的面积是 100 平方米。\n\n(2) $20+2=22$ (米)\n\n$22 \\times 7=154$ (平方米)\n\n答: 增加后的面积是 154 平方米。\n\n【点睛】此题考查的是长方形面积的计算,先计算出长方形绿地原来的长是解答此题的关键。"} {"id": "9263", "image": [], "answer": "解$\\quad 5000 \\quad 3 \\quad 72 \\quad 10000$\n\n【分析】 1 平方米 $=100$ 平方分米 $=10000$ 平方厘米、 1 平方分米 $=100$ 平方厘米, 据此进行单位换算即可。\n\n【详解】根据分析可知,\n\n50 平方分米 $=5000$ 平方厘米\n\n300 平方厘米 $=3$ 平方分米\n\n7200 平方分米 $=72$ 平方米\n\n1 平方米 $=10000$ 平方厘米\n\n【点睛】熟记各个面积单位之间的进率, 是解答此题的关键。", "solution": "null", "level": "三年级", "question": "在括号里填上合适的数。\n\n50 平方分米 $=(\\quad)$ 平方厘米 $\\quad 300$ 平方厘米 $=(\\quad) \\quad$ 平方分米\n\n7200 平方分米 $=(\\quad)$ 平方米 $\\quad 1$ 米 $=(\\quad)$ 平方厘米", "options": [], "subject": "度量几何学", "analysis": "解$\\quad 5000 \\quad 3 \\quad 72 \\quad 10000$\n\n【分析】 1 平方米 $=100$ 平方分米 $=10000$ 平方厘米、 1 平方分米 $=100$ 平方厘米, 据此进行单位换算即可。\n\n【详解】根据分析可知,\n\n50 平方分米 $=5000$ 平方厘米\n\n300 平方厘米 $=3$ 平方分米\n\n7200 平方分米 $=72$ 平方米\n\n1 平方米 $=10000$ 平方厘米\n\n【点睛】熟记各个面积单位之间的进率, 是解答此题的关键。"} {"id": "9264", "image": [], "answer": "解(1) 1\n\n【洋解】 (1) 边长是 1 分米的正方形, 面积是 1 平方分米。生活中, 经常能够接触到 1 平方分米大小的物品, 如粉笔盒正面的面积大约是 1 平方分米,家里的肥㿝盒大约是 1 平方分米那么大。\n\n(2) 边长是 1 米的正方形, 面积是 1 平方米。教师的讲桌桌面大约是 1 平方米那么大。\n\n(3)边长是 1 厘米的正方形, 面积是 1 平方厘米。大拇指的指甲盖、骰子的每个面、电脑键盘上的按键等的大小都大约是 1 平方厘米。", "solution": "null", "level": "三年级", "question": "想一想,填一填。\n\n(1)边长是 $(\\quad)$ 分米的正方形,面积是 1 平方分米。\n\n(2)边长是 ( ) 米的正方形, 面积是 1 平方米。\n\n(3)边长是 $(\\quad)$ 厘米的正方形, 面积是 1 平方厘米。", "options": [], "subject": "度量几何学", "analysis": "解(1) 1\n\n【洋解】 (1) 边长是 1 分米的正方形, 面积是 1 平方分米。生活中, 经常能够接触到 1 平方分米大小的物品, 如粉笔盒正面的面积大约是 1 平方分米,家里的肥㿝盒大约是 1 平方分米那么大。\n\n(2) 边长是 1 米的正方形, 面积是 1 平方米。教师的讲桌桌面大约是 1 平方米那么大。\n\n(3)边长是 1 厘米的正方形, 面积是 1 平方厘米。大拇指的指甲盖、骰子的每个面、电脑键盘上的按键等的大小都大约是 1 平方厘米。"} {"id": "9266", "image": ["1534.jpg"], "answer": "解24\n\n【分析】大三角形的面积与长方形的面积相等, 长方形的长乘宽等于长方形的面积, 据此即可解答。\n【详解】 $8 \\times 3=24$ (平方厘米)\n\n【点睛】明确大三角形的面积与长方形的面积相等是解答本题的关键。", "solution": "null", "level": "三年级", "question": "如图, 一个长方形, 长 8 厘米, 宽 3 厘米。将这个长方形剪成两个三角形, 再拼成一个大三角形。这个大三角形的面积是 $(\\quad)$ 平方厘米。\n", "options": [], "subject": "度量几何学", "analysis": "解24\n\n【分析】大三角形的面积与长方形的面积相等, 长方形的长乘宽等于长方形的面积, 据此即可解答。\n【详解】 $8 \\times 3=24$ (平方厘米)\n\n【点睛】明确大三角形的面积与长方形的面积相等是解答本题的关键。"} {"id": "9267", "image": ["1535.jpg"], "answer": "解10\n\n【分析】边长 $1 \\mathrm{~cm}$ 的正方形, 面积是 $1 \\mathrm{~cm}^{2}$, 因此图中有几个小正方形, 这个图形的面积就是几 $\\mathrm{cm}^{2}$,依此填空。\n\n【详解】图中有 10 个小正方形, 即这个图形的面积是 $10 \\mathrm{~cm}^{2}$ 。\n\n【点睛】熟练掌握对面积的认识是解答此题的关键。", "solution": "null", "level": "三年级", "question": "图中每个小正方形的边长都是 $1 \\mathrm{~cm}$, 这个图形的面积是 ( ) $\\mathrm{cm}^{2}$ 。\n\n", "options": [], "subject": "度量几何学", "analysis": "解10\n\n【分析】边长 $1 \\mathrm{~cm}$ 的正方形, 面积是 $1 \\mathrm{~cm}^{2}$, 因此图中有几个小正方形, 这个图形的面积就是几 $\\mathrm{cm}^{2}$,依此填空。\n\n【详解】图中有 10 个小正方形, 即这个图形的面积是 $10 \\mathrm{~cm}^{2}$ 。\n\n【点睛】熟练掌握对面积的认识是解答此题的关键。"} {"id": "9269", "image": [], "answer": "解$600 \\quad 4$\n\n【分析】 1 平方米 $=100$ 平方分米, 6 平方米里面有几个 1 平方米, 就是几百平方分米; 100 平方厘米 $=1$ 平方分米, 400 平方厘米里面有几个 100 平方厘米, 就是几平方分米。\n\n【详解】 6 平方米里面有 6 个 1 平方米, 也就是 6 个 100 平方分米, 因此 6 平方米 $=600$ 平方分米; 400 平方厘米里面有 4 个 100 平方厘米, 也就是 4 个 1 平方分米, 即 400 平方厘米 $=4$ 平方分米。\n\n【点睛】此题考查的是面积单位之间的换算,熟记面积单位之间的进率是解答本题的关键。", "solution": "null", "level": "三年级", "question": "6 平方米 $=($ ) 平方分米 400 平方厘米 $=($ ) 平方分米", "options": [], "subject": "度量几何学", "analysis": "解$600 \\quad 4$\n\n【分析】 1 平方米 $=100$ 平方分米, 6 平方米里面有几个 1 平方米, 就是几百平方分米; 100 平方厘米 $=1$ 平方分米, 400 平方厘米里面有几个 100 平方厘米, 就是几平方分米。\n\n【详解】 6 平方米里面有 6 个 1 平方米, 也就是 6 个 100 平方分米, 因此 6 平方米 $=600$ 平方分米; 400 平方厘米里面有 4 个 100 平方厘米, 也就是 4 个 1 平方分米, 即 400 平方厘米 $=4$ 平方分米。\n\n【点睛】此题考查的是面积单位之间的换算,熟记面积单位之间的进率是解答本题的关键。"} {"id": "9270", "image": [], "answer": "解$\\quad 324 \\quad 50$\n\n【分析】一张长 25 厘米, 宽 18 厘米的长方形纸, 从中剪下一个最大的正方形, 这个正方形的边长应等于长方形的宽, 是 18 厘米, 根据正方形的面积 $=$ 边长 $\\times$ 边长, 代入数据计算即可求出面积; 剩下部分的长是 18 厘米, 宽是 $(25-18)$ 厘米, 根据长方形的周长 $=($ 长+宽 $) \\times 2$, 把数据代入公式解答。\n\n【详解】 $18 \\times 18=324$ (平方厘米)\n\n$25-18=7$ (厘米)\n\n$(18+7) \\times 2$\n\n$=25 \\times 2$\n\n$=50$ (厘米)\n\n剪去的正方形纸的面积是 324 平方厘米。留下长方形纸的周长是 50 厘米。\n【点睛】此题主要根据正方形的面积和长方形的周长计算方法解决问题。", "solution": "null", "level": "三年级", "question": "一张长 25 厘米, 宽 18 厘米的长方形纸, 从中剪下一个最大的正方形, 剪去的正方形纸的面积是 $(\\quad)$ 平方厘米。留下长方形纸的周长是 $(\\quad)$ 厘米。", "options": [], "subject": "度量几何学", "analysis": "解$\\quad 324 \\quad 50$\n\n【分析】一张长 25 厘米, 宽 18 厘米的长方形纸, 从中剪下一个最大的正方形, 这个正方形的边长应等于长方形的宽, 是 18 厘米, 根据正方形的面积 $=$ 边长 $\\times$ 边长, 代入数据计算即可求出面积; 剩下部分的长是 18 厘米, 宽是 $(25-18)$ 厘米, 根据长方形的周长 $=($ 长+宽 $) \\times 2$, 把数据代入公式解答。\n\n【详解】 $18 \\times 18=324$ (平方厘米)\n\n$25-18=7$ (厘米)\n\n$(18+7) \\times 2$\n\n$=25 \\times 2$\n\n$=50$ (厘米)\n\n剪去的正方形纸的面积是 324 平方厘米。留下长方形纸的周长是 50 厘米。\n【点睛】此题主要根据正方形的面积和长方形的周长计算方法解决问题。"} {"id": "9279", "image": ["1539.jpg"], "answer": "解5 平方米\n\n【分析】根据题意, 正方形的边长是 5 分米, 用乘法计算出卫生间的地面的长是 ( $5 \\times 5$ ) 分米, 用乘法计算出卫生间的宽是 $(4 \\times 5)$ 分米铺成的, 最后根据长方形面积 $=$ 长 $\\times$ 宽, 代入数据即可求出卫生间地面的面积, 据此解答。\n\n【详解】 $(5 \\times 5) \\times(5 \\times 4)$\n\n$=25 \\times 20$\n\n$=500($ 平方分米 $)$\n\n500 平方分米 $=5$ 平方米\n\n答: 这个卫生间地面的面积是 5 平方米。\n\n【点睛】本题考查长方形的面积以及平方分米和平方米之间的进率, 熟记公式以及基本单位间的进率是解答本题的关键。", "solution": "null", "level": "三年级", "question": "下图是用边长 5 分米的正方形地砖铺的卫生间地面, 你知道这个卫生间地面的面积是多少平方米吗?\n\n", "options": [], "subject": "度量几何学", "analysis": "解5 平方米\n\n【分析】根据题意, 正方形的边长是 5 分米, 用乘法计算出卫生间的地面的长是 ( $5 \\times 5$ ) 分米, 用乘法计算出卫生间的宽是 $(4 \\times 5)$ 分米铺成的, 最后根据长方形面积 $=$ 长 $\\times$ 宽, 代入数据即可求出卫生间地面的面积, 据此解答。\n\n【详解】 $(5 \\times 5) \\times(5 \\times 4)$\n\n$=25 \\times 20$\n\n$=500($ 平方分米 $)$\n\n500 平方分米 $=5$ 平方米\n\n答: 这个卫生间地面的面积是 5 平方米。\n\n【点睛】本题考查长方形的面积以及平方分米和平方米之间的进率, 熟记公式以及基本单位间的进率是解答本题的关键。"} {"id": "9280", "image": [], "answer": "解2400 块\n\n【分析】根据长方形的面积 $=$ 长 $\\times$ 宽, 求出文化墙的面积。平方米和平方分米之间的进率是 100 , 据此将文化墙的面积换算成平方分米,再用文化墙的面积除以一块瓷砖的面积,求出需要瓷砖数量。\n\n【详解】 $8 \\times 6=48$ (平方米)\n\n48 平方米 $=4800$ 平方分米\n\n$4800 \\div 2=2400$ (块)\n\n答: 一共要用 2400 块彩色瓷砖。\n\n【点睛】本题考查长方形面积公式的应用以及面积单位的换算, 关键是熟记公式, 求出文化墙的面积。", "solution": "null", "level": "三年级", "question": "一面文化墙长 6 米, 宽 8 米, 用面积是 2 平方分米的彩色瓷砖在文化墙上贴出, 一共要用多少块彩色瓷砖?", "options": [], "subject": "度量几何学", "analysis": "解2400 块\n\n【分析】根据长方形的面积 $=$ 长 $\\times$ 宽, 求出文化墙的面积。平方米和平方分米之间的进率是 100 , 据此将文化墙的面积换算成平方分米,再用文化墙的面积除以一块瓷砖的面积,求出需要瓷砖数量。\n\n【详解】 $8 \\times 6=48$ (平方米)\n\n48 平方米 $=4800$ 平方分米\n\n$4800 \\div 2=2400$ (块)\n\n答: 一共要用 2400 块彩色瓷砖。\n\n【点睛】本题考查长方形面积公式的应用以及面积单位的换算, 关键是熟记公式, 求出文化墙的面积。"} {"id": "9281", "image": ["1540.jpg"], "answer": "解21 平方厘米\n\n【分析】每个小方格的面积是 1 平方厘米, 则每个小方格的边长是 1 厘米。先数出空白部分的长与宽分别是几个小方格的边长和, 进而求出空白部分的长与宽。根据长方形的面积 $=$ 长 $\\times$ 宽解答。\n\n【详解】根据题图可知, 空白部分的长是 7 厘米, 宽是 3 厘米。\n\n$7 \\times 3=21($ 平方厘米 $)$\n\n答:空白部分的面积是 21 平方厘米。\n\n【点睛】本题考查长方形面积公式的应用, 关键是求出空白部分的长与宽。", "solution": "null", "level": "三年级", "question": "如图, 由小方格围成的空白部分的面积是多少?(每个小方格的面积是 1 平方厘米)在下面的用文字叙述(或式子)把你的想法表示出来。\n\n", "options": [], "subject": "度量几何学", "analysis": "解21 平方厘米\n\n【分析】每个小方格的面积是 1 平方厘米, 则每个小方格的边长是 1 厘米。先数出空白部分的长与宽分别是几个小方格的边长和, 进而求出空白部分的长与宽。根据长方形的面积 $=$ 长 $\\times$ 宽解答。\n\n【详解】根据题图可知, 空白部分的长是 7 厘米, 宽是 3 厘米。\n\n$7 \\times 3=21($ 平方厘米 $)$\n\n答:空白部分的面积是 21 平方厘米。\n\n【点睛】本题考查长方形面积公式的应用, 关键是求出空白部分的长与宽。"} {"id": "9282", "image": [], "answer": "解(1) 7200 平方米\n\n(2) 360 米\n\n【分析】(1)根据“长方形面积 $=$ 长 $\\times$ 宽”可知, 即可求出这个花坛的占地面积。\n\n(2) 根据“长方形周长 $=($ 长十宽 $) \\times 2$ ”可知, 即可求出围栏的长度。\n\n【详解】 (1) $120 \\times 60=7200$ (平方米)\n\n答:这个花坛的占地面积是 7200 平方米。\n\n(2) $(120+60) \\times 2$\n\n$=180 \\times 2$\n\n$=360($ 米 $)$\n\n答: 围栏的长度是 360 米。\n\n【点睛】熟记长方形面积、周长计算公式, 是解答此题的关键。", "solution": "null", "level": "三年级", "question": "一个长方形的花坛, 长 120 米, 宽 60 米。\n\n(1) 求这个花坛的占地面积。\n\n(2) 在花坛的四周围一圈围栏, 求围栏的长度。", "options": [], "subject": "度量几何学", "analysis": "解(1) 7200 平方米\n\n(2) 360 米\n\n【分析】(1)根据“长方形面积 $=$ 长 $\\times$ 宽”可知, 即可求出这个花坛的占地面积。\n\n(2) 根据“长方形周长 $=($ 长十宽 $) \\times 2$ ”可知, 即可求出围栏的长度。\n\n【详解】 (1) $120 \\times 60=7200$ (平方米)\n\n答:这个花坛的占地面积是 7200 平方米。\n\n(2) $(120+60) \\times 2$\n\n$=180 \\times 2$\n\n$=360($ 米 $)$\n\n答: 围栏的长度是 360 米。\n\n【点睛】熟记长方形面积、周长计算公式, 是解答此题的关键。"} {"id": "9209", "image": ["1519.jpg", "1520.jpg", "1520.jpg"], "answer": "解见详解\n\n【分析】根据长方形的周长 $=($ 长十宽 $) \\times 2$, 周长为 12 厘米的长方形, 长宽和是 6 厘米, 可以是长 4 厘米宽 2 厘米, 也可以是长 5 厘米宽 1 厘米。再根据长方形的面积 $=$ 长 $\\times$ 宽解答。\n\n\n\n$4 \\times 2=8$ (平方厘米)\n\n$5 \\times 1=5$ (平方厘米)\n\n则两个长方形的面积分别是 8 平方厘米和 5 平方厘米。\n\n【点睛】本题考查长方形周长和面积公式的应用, 关键是熟记公式。", "solution": "null", "level": "三年级", "question": "在格子图中画出两个周长都是 12 厘米但面积不同的长方形, 并写出它们的面积大小。(1 个小格子的边长是 1 厘米。)\n\n", "options": [], "subject": "画法几何学", "analysis": "解见详解\n\n【分析】根据长方形的周长 $=($ 长十宽 $) \\times 2$, 周长为 12 厘米的长方形, 长宽和是 6 厘米, 可以是长 4 厘米宽 2 厘米, 也可以是长 5 厘米宽 1 厘米。再根据长方形的面积 $=$ 长 $\\times$ 宽解答。\n\n\n\n$4 \\times 2=8$ (平方厘米)\n\n$5 \\times 1=5$ (平方厘米)\n\n则两个长方形的面积分别是 8 平方厘米和 5 平方厘米。\n\n【点睛】本题考查长方形周长和面积公式的应用, 关键是熟记公式。"} {"id": "8147", "image": ["1266.jpg", "1267.jpg"], "answer": "详解(1) 6\n (2) 7", "solution": "null", "level": "三年级", "question": "1)下列图形中有 \\$ \\qquad \\$个三角形。(3 分)\n\n\n\n(2)下列图中有 \\$ \\qquad \\$个四边形。(3 分)\n\n", "options": [], "subject": "画法几何学", "analysis": "详解(1) 6\n (2) 7"} {"id": "9214", "image": ["1522.jpg", "1523.jpg", "1523.jpg"], "answer": "解32\n\n【分析】用数格子估计不规则图形面积, 分别数出整格数和不完整格数, 根据整格数和所有格数确定面积大小的范围, 把不完整格按半格计算加上整格数, 估算出面积。\n\n【详解】 如下图所示:\n\n\n\n红线圈出部分为整格数: 28 个\n\n不完整格数已标示序号:8 个\n\n利用割补法:\n\n序号 1 和 8 可以拼成完整 1 格; 序号 2 和 7 可以拼成完整 1 格序号 3 和 6 可以拼成完整 1 格; 序号 4 和 5 可以拼成完整 1 格不完整格能拼成 4 个整格\n\n$28+4=32($ 个 $)$\n\n$32 \\times 1=32 \\mathrm{~cm}^{2}$\n\n【点睛】本题考查通过网格图估算不规则图形面积(含曲线), 数出这个图形的所占的格子数, 是解决本题的关键。", "solution": "null", "level": "三年级", "question": "下图中每个小方格面积是 $1 \\mathrm{~cm}^{2}$, 图中阴影部分面积是 $\\quad \\mathrm{cm}^{2}$ 。\n\n", "options": [], "subject": "组合几何学", "analysis": "解32\n\n【分析】用数格子估计不规则图形面积, 分别数出整格数和不完整格数, 根据整格数和所有格数确定面积大小的范围, 把不完整格按半格计算加上整格数, 估算出面积。\n\n【详解】 如下图所示:\n\n\n\n红线圈出部分为整格数: 28 个\n\n不完整格数已标示序号:8 个\n\n利用割补法:\n\n序号 1 和 8 可以拼成完整 1 格; 序号 2 和 7 可以拼成完整 1 格序号 3 和 6 可以拼成完整 1 格; 序号 4 和 5 可以拼成完整 1 格不完整格能拼成 4 个整格\n\n$28+4=32($ 个 $)$\n\n$32 \\times 1=32 \\mathrm{~cm}^{2}$\n\n【点睛】本题考查通过网格图估算不规则图形面积(含曲线), 数出这个图形的所占的格子数, 是解决本题的关键。"} {"id": "9283", "image": [], "answer": "解3600 平方厘米; 36 平方分米\n\n【分析】根据“正方形周长 $=$ 边长 $\\times 4$ 、正方形面积 $=$ 边长 $\\times$ 边长”, 先求出地砖的边长, 再求出地砖的面积, 最后进行单位换算即可。\n\n【详解】 $240 \\div 4=60$ (厘米)\n\n$60 \\times 60=3600$ (平方厘米)\n\n3600 平方厘米 $=36$ 平方分米\n\n答: 它的面积是 3600 平方厘米, 合 36 平方分米。\n\n【点睛】熟记正方形周长、面积计算公式, 是解答此题的关键。", "solution": "null", "level": "三年级", "question": "有一块正方形的地砖, 周长是 240 厘米, 它的面积是多少平方厘米? 合多少平方分米?", "options": [], "subject": "立体几何学", "analysis": "解3600 平方厘米; 36 平方分米\n\n【分析】根据“正方形周长 $=$ 边长 $\\times 4$ 、正方形面积 $=$ 边长 $\\times$ 边长”, 先求出地砖的边长, 再求出地砖的面积, 最后进行单位换算即可。\n\n【详解】 $240 \\div 4=60$ (厘米)\n\n$60 \\times 60=3600$ (平方厘米)\n\n3600 平方厘米 $=36$ 平方分米\n\n答: 它的面积是 3600 平方厘米, 合 36 平方分米。\n\n【点睛】熟记正方形周长、面积计算公式, 是解答此题的关键。"} {"id": "8972", "image": ["1438.jpg"], "answer": "能运走。\n\n【分析】由图可知, 每箱货物的净重都是 25 千克, 所以可以用每箱的净重 $\\times$ 箱数的和, 先算出苹果,香梨和香蕉的总重量, 再和一吨去比较, 如果大于 1 吨, 则无法带走, 如果小于 1 吨, 则可以一次运走,据此解答。\n\n【详解】 $(9+8+7) \\times 25$\n\n$=600$ (千克)\n\n600 千克 $<1$ 吨\n答: 他能一次把这些苹果、香梨和香蕉都运走。\n\n【点睛】熟练掌捯两位数乘两位数的实际应用。", "solution": "null", "level": "三年级", "question": "李叔叔的货车能装 1 吨货物, 他能一次把这些苹果、香梨和香蕉都运走吗?\n", "options": [], "subject": "立体几何学", "analysis": "能运走。\n\n【分析】由图可知, 每箱货物的净重都是 25 千克, 所以可以用每箱的净重 $\\times$ 箱数的和, 先算出苹果,香梨和香蕉的总重量, 再和一吨去比较, 如果大于 1 吨, 则无法带走, 如果小于 1 吨, 则可以一次运走,据此解答。\n\n【详解】 $(9+8+7) \\times 25$\n\n$=600$ (千克)\n\n600 千克 $<1$ 吨\n答: 他能一次把这些苹果、香梨和香蕉都运走。\n\n【点睛】熟练掌捯两位数乘两位数的实际应用。"} {"id": "9367", "image": [], "answer": "(1)错误", "solution": "null", "level": "三年级", "question": "一个正方形的边长增加 3 分米, 面积就增加 9 平方分米。", "options": [], "subject": "解析几何", "analysis": "(1)错误"} {"id": "7500", "image": [], "answer": ":0固我每天早上吃一个约重 60 克的鸡蛋,喝一袋 250 克的牛奶,中午吃 3 个 100 克的慢头。晚上吃得更多,和两个小伙伴能把 1 啉的米饭吃完。\n\n30 克改为 30 千克\n\n60 千克改为 60 克\n\n250 千克改为 250 克\n\n100 千克改为 100 克\n\n1 吨改为 1 千克", "solution": "null", "level": "三年级", "question": "请你圈出下面文字中质量单位使用不合理的地方,并改正。\n\n妈妈, 我参加夏令营已经 9 天了, 最近我的身体更加健壮了, 体重已经达到了 30 克。我每天早上吃一个约重 60 千克的鸡蛋, 喝一袋 250 千克的牛奶, 中午吃 3 个 100 千克的馒头。晚上吃得更多, 和两个小伙伴能把 1 吨的米饭吃完。", "options": [], "subject": "逻辑题", "analysis": ":0固我每天早上吃一个约重 60 克的鸡蛋,喝一袋 250 克的牛奶,中午吃 3 个 100 克的慢头。晚上吃得更多,和两个小伙伴能把 1 啉的米饭吃完。\n\n30 克改为 30 千克\n\n60 千克改为 60 克\n\n250 千克改为 250 克\n\n100 千克改为 100 克\n\n1 吨改为 1 千克"} {"id": "7301", "image": [], "answer": "(1) 780 个\n\n(2) 120 个\n\n【详解】(1) $260+520=780$ (个)\n\n答:今天运出去篮球和足球一共有 780 个。\n\n(2) $640-520=120$ (个)\n\n答: 运出的足球比排球少 120 个。", "solution": "null", "level": "三年级", "question": "这是今天某工厂运出的各种球类的个数:\n\n| 种 类 | 篮球 | 足球 | 排球 |\n| :---: | :---: | :---: | :---: |\n| 数量(个) | 260 | 520 | 640 |\n\n(1)今天运出去篮球和足球一共有多少个?\n\n(2)运出的足球比排球少多少个?", "options": [], "subject": "代数", "analysis": "(1) 780 个\n\n(2) 120 个\n\n【详解】(1) $260+520=780$ (个)\n\n答:今天运出去篮球和足球一共有 780 个。\n\n(2) $640-520=120$ (个)\n\n答: 运出的足球比排球少 120 个。"} {"id": "7365", "image": ["1021.jpg", "1022.jpg", "1022.jpg"], "answer": "图见洋解; 700 米\n\n【分析】根据给出的图, 从小华家到体育场踢足球: (1)从家经过超市, 再到体育场; (2)从家经过学校,经过超市, 再到体育场; (3)从家经过学校, 经过书店, 再到超市, 最后到体育场; (4)从家经过学校,经过书店, 到体育场, 计算对比分析, 走(1)的这条路最近, 要求小华走多少米, 把两段路程相加即可,据此解答。\n\n【详解】作图如下:\n\n\n\n(1) 从家经过超市,再到体育场:\n\n$300+400=700$ (米)\n\n(2) 从家经过学校,经过超市,再到体育场\n\n$360+500+400$\n\n$=860+400$\n\n$=1260$ (米)\n\n(3)从家经过学校, 经过书店, 再到超市, 最后到体育场:\n\n$360+160+700+400$\n\n$=520+700+400$\n\n$=1220+400$\n\n$=1620$ (米)\n\n(4) 从家经过学校, 经过书店, 到体育场:\n\n$360+160+660$\n\n$=520+660$\n\n$=1180$ (米)\n\n1620 米>1260 米>1180 米>700 米\n答: 小华要走 700 米。\n\n【点睛】本题考查了路线图以及整数加法的实际应用, 熟练掌握并灵活运用。", "solution": "null", "level": "三年级", "question": "星期天, 小华要到体育场踢足球, 怎么走最近? 把最近的路线用蓝色的钢笔描出来。并算一算,小华要走多少米?\n\n", "options": [], "subject": "图论", "analysis": "图见洋解; 700 米\n\n【分析】根据给出的图, 从小华家到体育场踢足球: (1)从家经过超市, 再到体育场; (2)从家经过学校,经过超市, 再到体育场; (3)从家经过学校, 经过书店, 再到超市, 最后到体育场; (4)从家经过学校,经过书店, 到体育场, 计算对比分析, 走(1)的这条路最近, 要求小华走多少米, 把两段路程相加即可,据此解答。\n\n【详解】作图如下:\n\n\n\n(1) 从家经过超市,再到体育场:\n\n$300+400=700$ (米)\n\n(2) 从家经过学校,经过超市,再到体育场\n\n$360+500+400$\n\n$=860+400$\n\n$=1260$ (米)\n\n(3)从家经过学校, 经过书店, 再到超市, 最后到体育场:\n\n$360+160+700+400$\n\n$=520+700+400$\n\n$=1220+400$\n\n$=1620$ (米)\n\n(4) 从家经过学校, 经过书店, 到体育场:\n\n$360+160+660$\n\n$=520+660$\n\n$=1180$ (米)\n\n1620 米>1260 米>1180 米>700 米\n答: 小华要走 700 米。\n\n【点睛】本题考查了路线图以及整数加法的实际应用, 熟练掌握并灵活运用。"} {"id": "23985", "image": [], "answer": "解$\\sqrt{ }$\n\n【分析】一个数(0 除外)的 最大因数和最小倍数都是这个数的本身。据此判断即可。\n\n【详解】由分析可知:\n$\\mathrm{m}$ 是非 0 自然数, $\\mathrm{m}$ 的最小倍数和最大因数都是 $\\mathrm{m}$ 。说法正确。\n\n故答案为: $\\sqrt{ }$\n\n【点睛】本题考查因数和倍数, 明确因数和倍数的定义是解题的关键。", "solution": "null", "level": "五年级", "question": "$\\mathrm{m}$ 是非 0 自然数, $\\mathrm{m}$ 的最小倍数和最大因数都是 $\\mathrm{m}$ 。( )", "options": [], "subject": "算术", "analysis": "解$\\sqrt{ }$\n\n【分析】一个数(0 除外)的 最大因数和最小倍数都是这个数的本身。据此判断即可。\n\n【详解】由分析可知:\n$\\mathrm{m}$ 是非 0 自然数, $\\mathrm{m}$ 的最小倍数和最大因数都是 $\\mathrm{m}$ 。说法正确。\n\n故答案为: $\\sqrt{ }$\n\n【点睛】本题考查因数和倍数, 明确因数和倍数的定义是解题的关键。"} {"id": "23986", "image": [], "answer": "解$\\sqrt{ }$\n\n【分析】奇数 + 奇数 $=$ 偶数, 偶数 + 偶数 $=$ 偶数, 偶数个奇数的和是偶数, 据此解题。\n\n【详解】1、23、19、33、51、77 都是奇数, 有 6 个奇数, 所以 $1+23+19+33+51+77$ 的和是偶数。故答案为: $\\sqrt{ }$\n\n【点睛】本题考查了奇数和偶数的运算性质, 掌握“奇数十奇数=偶数”是解题的关键。", "solution": "null", "level": "五年级", "question": "$1+23+19+33+51+77$ 的和是偶数。( )", "options": [], "subject": "算术", "analysis": "解$\\sqrt{ }$\n\n【分析】奇数 + 奇数 $=$ 偶数, 偶数 + 偶数 $=$ 偶数, 偶数个奇数的和是偶数, 据此解题。\n\n【详解】1、23、19、33、51、77 都是奇数, 有 6 个奇数, 所以 $1+23+19+33+51+77$ 的和是偶数。故答案为: $\\sqrt{ }$\n\n【点睛】本题考查了奇数和偶数的运算性质, 掌握“奇数十奇数=偶数”是解题的关键。"} {"id": "23987", "image": [], "answer": "解$\\sqrt{ }$\n\n【分析】因为 5 的倍数的特点是:个位上的数是 0 或 $5 ; 2$ 的倍数的特点是:个位上的数是 $0 、 2 、 4$ 、 $6 、 8$ 的数; 所以既是 2 的倍数又是 5 的倍数, 个位上一定是 0 。\n\n【详解】由分析可知, 既是 2 的倍数又是 5 的倍数的数, 个位上一定是 0 ; 原题说法正确。故答案为: $\\sqrt{ }$\n\n【点睛】本题考查了 2,5 倍数的特点, 要记住这些特点, 并灵活运用。", "solution": "null", "level": "五年级", "question": "既是 2 的倍数又是 5 的倍数的数, 它的个位上一定是 0 。()", "options": [], "subject": "算术", "analysis": "解$\\sqrt{ }$\n\n【分析】因为 5 的倍数的特点是:个位上的数是 0 或 $5 ; 2$ 的倍数的特点是:个位上的数是 $0 、 2 、 4$ 、 $6 、 8$ 的数; 所以既是 2 的倍数又是 5 的倍数, 个位上一定是 0 。\n\n【详解】由分析可知, 既是 2 的倍数又是 5 的倍数的数, 个位上一定是 0 ; 原题说法正确。故答案为: $\\sqrt{ }$\n\n【点睛】本题考查了 2,5 倍数的特点, 要记住这些特点, 并灵活运用。"} {"id": "23988", "image": [], "answer": "解$\\times$\n\n【分析】 2 的倍数特征:个位上是 $0 、 2 、 4 、 6 、 8$ 的数;5的倍数特征:个位上是 0 或 5 的数;2、 5 的倍数特征: 个位上是 0 的数; 3 的倍数特征: 各个数位上的数字相加, 和要能被 3 整除; 据此解答。\n\n【详解】 10 不是 3 的倍数, 同时是 $2 、 3 、 5$ 的倍数的最小两位数是 30 。\n\n原题说法错误。\n\n故答案为: $\\times$\n\n【点睛】掌握 $2 、 3 、 5$ 的倍数特征是解题的关键。", "solution": "null", "level": "五年级", "question": "同时是 $2 、 3 、 5$ 的倍数的最小两位数是 10 。()", "options": [], "subject": "算术", "analysis": "解$\\times$\n\n【分析】 2 的倍数特征:个位上是 $0 、 2 、 4 、 6 、 8$ 的数;5的倍数特征:个位上是 0 或 5 的数;2、 5 的倍数特征: 个位上是 0 的数; 3 的倍数特征: 各个数位上的数字相加, 和要能被 3 整除; 据此解答。\n\n【详解】 10 不是 3 的倍数, 同时是 $2 、 3 、 5$ 的倍数的最小两位数是 30 。\n\n原题说法错误。\n\n故答案为: $\\times$\n\n【点睛】掌握 $2 、 3 、 5$ 的倍数特征是解题的关键。"} {"id": "23989", "image": [], "answer": "解$\\times$\n\n【分析】若整数 $\\mathrm{a}$ 能够被 $\\mathrm{b}$ 整除, $\\mathrm{a}$ 叫做 $\\mathrm{b}$ 的倍数, $\\mathrm{b}$ 就叫做 $\\mathrm{a}$ 的因数, 因数与倍数是相互依存的,据此解答。\n\n【详解】因为 $5 \\times 0.8=4,0.8$ 是小数, 因此这三个数之间没有因数和倍数的关系。\n\n故答案为: $x$\n\n【点睛】此题考查了因数与倍数的意义, 注意研究因数与倍数必须在整数范围内。", "solution": "null", "level": "五年级", "question": "因为 $5 \\times 0.8=4$, 所以 5 和 0.8 是 4 的因数, 4 是 5 和 0.8 的倍数。( )", "options": [], "subject": "算术", "analysis": "解$\\times$\n\n【分析】若整数 $\\mathrm{a}$ 能够被 $\\mathrm{b}$ 整除, $\\mathrm{a}$ 叫做 $\\mathrm{b}$ 的倍数, $\\mathrm{b}$ 就叫做 $\\mathrm{a}$ 的因数, 因数与倍数是相互依存的,据此解答。\n\n【详解】因为 $5 \\times 0.8=4,0.8$ 是小数, 因此这三个数之间没有因数和倍数的关系。\n\n故答案为: $x$\n\n【点睛】此题考查了因数与倍数的意义, 注意研究因数与倍数必须在整数范围内。"} {"id": "23990", "image": [], "answer": "解5212019\n\n【分析】最小的质数是 2 , 最小的偶数是 0 , 最小的奇数是 1 , 一位数中 9 既是合数又是奇数。据此解题。\n\n【详解】结合质数、合数、奇数、偶数的概念可得:\n答: 东东家电话号码是 5212019。\n\n【点睛】熟悉 10 以内质数、合数、奇数、偶数的具体情况, 是解题关键。", "solution": "null", "level": "五年级", "question": "东东家电话号码前三位是 521 , 第四位是最小的质数, 第五位是最小的偶数, 第六位是最小的奇数, 末尾数字既是合数又是奇数, 东东家电话号码是多少?", "options": [], "subject": "算术", "analysis": "解5212019\n\n【分析】最小的质数是 2 , 最小的偶数是 0 , 最小的奇数是 1 , 一位数中 9 既是合数又是奇数。据此解题。\n\n【详解】结合质数、合数、奇数、偶数的概念可得:\n答: 东东家电话号码是 5212019。\n\n【点睛】熟悉 10 以内质数、合数、奇数、偶数的具体情况, 是解题关键。"} {"id": "23991", "image": [], "answer": "解15 平方分米\n\n【分析】根据长方形周长公式: 长方形周长 $=($ 长十宽 $) \\times 2$; 长十宽 $=$ 长方形周长 $\\div 2$, 求出长方形的长和宽的和; 再根据质数的意义: 一个自然数, 如果只有 1 和它本身两个因数, 这样的数叫做质数,把长和宽的和分解成两个质数的和, 再根据长方形面积公式: 长 $\\times$ 宽, 求出面积。\n\n【详解】 $16 \\div 2=8$ (分米)\n\n$8=5+3$\n\n$5 \\times 3=15$ (平方分米)\n\n答:这个长方形面积是 15 平方分米。\n\n【点睛】解答本题的关键是把长方形的长与宽的和分解成两个质数和, 进而求出面积。", "solution": "null", "level": "五年级", "question": "一个长方形的周长是 16 分米, 它的长和宽都是质数, 这个长方形的面积是多少平方分米?", "options": [], "subject": "算术", "analysis": "解15 平方分米\n\n【分析】根据长方形周长公式: 长方形周长 $=($ 长十宽 $) \\times 2$; 长十宽 $=$ 长方形周长 $\\div 2$, 求出长方形的长和宽的和; 再根据质数的意义: 一个自然数, 如果只有 1 和它本身两个因数, 这样的数叫做质数,把长和宽的和分解成两个质数的和, 再根据长方形面积公式: 长 $\\times$ 宽, 求出面积。\n\n【详解】 $16 \\div 2=8$ (分米)\n\n$8=5+3$\n\n$5 \\times 3=15$ (平方分米)\n\n答:这个长方形面积是 15 平方分米。\n\n【点睛】解答本题的关键是把长方形的长与宽的和分解成两个质数和, 进而求出面积。"} {"id": "23993", "image": [], "answer": "解3 米; 19 段\n\n【分析】将三根铁丝剪成每段相等的长度, 每段尽量长一些, 又不能有剩余, 每段铁丝的长度就是求 15、18 和 24 的最大公因数; 三根铁丝总长 $\\div 一$ 共能剪的段数=每段的长度。\n\n【详解】 15 的因数有 $1,3,5,15$;\n\n18 的因数有 $1,2,3,6,9,18$;\n\n24 的因数有 $1,2,3,4,6,8,12,24$;\n\n15、18、24 的最大公因数是 3 ;\n\n$(15+18+24) \\div 3$\n\n$=57 \\div 3$\n\n$=19$ (段)\n\n答: 每段长 3 米,一共能剪 19 段。\n\n【点睛】本题考查最大公因数知识点, 运用最大公因数知识解决实际问题。", "solution": "null", "level": "五年级", "question": "三根铁丝分别长 15 米、 18 米和 24 米, 现将它们剪成每段相等的长度, 每段尽量长一些, 又不能有剩余,每段长多少米?一共能剪多少段?", "options": [], "subject": "算术", "analysis": "解3 米; 19 段\n\n【分析】将三根铁丝剪成每段相等的长度, 每段尽量长一些, 又不能有剩余, 每段铁丝的长度就是求 15、18 和 24 的最大公因数; 三根铁丝总长 $\\div 一$ 共能剪的段数=每段的长度。\n\n【详解】 15 的因数有 $1,3,5,15$;\n\n18 的因数有 $1,2,3,6,9,18$;\n\n24 的因数有 $1,2,3,4,6,8,12,24$;\n\n15、18、24 的最大公因数是 3 ;\n\n$(15+18+24) \\div 3$\n\n$=57 \\div 3$\n\n$=19$ (段)\n\n答: 每段长 3 米,一共能剪 19 段。\n\n【点睛】本题考查最大公因数知识点, 运用最大公因数知识解决实际问题。"} {"id": "23995", "image": [], "answer": "解小刚说的对, 因为 705 能被 3 整除。\n\n【分析】门票每人 3 元, 那么花费的总钱数一定要能被 3 整除, 如果一个数各位上数字加起来的和是 3 的倍数, 那么这个数就是 3 的倍数, 据此解答即可。\n\n【详解】 $6+2+3=11,11$ 不能被 3 整除, 所以 623 元不符合题意;\n\n$5+9+8=22,22$ 不能被 3 整除, 所以 598 元不符合题意;\n\n$7+0+5=12,12$ 能被 3 整除, 所以 705 元符合题意。\n\n答: 小刚说的对, 因为 705 能被 3 整除。\n\n【点睛】本题主要考查 3 的倍数的特征: 一个数各位上数字加起来的和是 3 的倍数, 那么这个数就是 3 的倍数。", "solution": "null", "level": "五年级", "question": "光明小学五年级有 6 个班学生去公园, 门票每人 3 元,小明说: “一共是 623 元”。小红说: “一共是 598 元”。小刚说:“一共是 705 元”。老师笑着说:“他们三人只有一人算对了。”你认为谁算的对? 为什么?", "options": [], "subject": "算术", "analysis": "解小刚说的对, 因为 705 能被 3 整除。\n\n【分析】门票每人 3 元, 那么花费的总钱数一定要能被 3 整除, 如果一个数各位上数字加起来的和是 3 的倍数, 那么这个数就是 3 的倍数, 据此解答即可。\n\n【详解】 $6+2+3=11,11$ 不能被 3 整除, 所以 623 元不符合题意;\n\n$5+9+8=22,22$ 不能被 3 整除, 所以 598 元不符合题意;\n\n$7+0+5=12,12$ 能被 3 整除, 所以 705 元符合题意。\n\n答: 小刚说的对, 因为 705 能被 3 整除。\n\n【点睛】本题主要考查 3 的倍数的特征: 一个数各位上数字加起来的和是 3 的倍数, 那么这个数就是 3 的倍数。"} {"id": "23996", "image": [], "answer": "解73、75、77\n\n【分析】用“ $225 \\div 3=75$ ”, 求出中间的那个奇数, 又因为两个连续的奇数相差“ 2 ”, 进而分别求出另外 2 个奇数即可。\n\n【详解】 $225 \\div 3=75$\n\n$75-2=73$\n\n$75+2=77$\n\n答:这三个奇数分别是 $73 、 75 、 77$ 。\n\n【点睛】解答此题的关键是求出中间的那个奇数, 然后根据两个连续的奇数的特征来解答。", "solution": "null", "level": "五年级", "question": "三个连续奇数的和是 225 , 这三个奇数分别是多少?", "options": [], "subject": "算术", "analysis": "解73、75、77\n\n【分析】用“ $225 \\div 3=75$ ”, 求出中间的那个奇数, 又因为两个连续的奇数相差“ 2 ”, 进而分别求出另外 2 个奇数即可。\n\n【详解】 $225 \\div 3=75$\n\n$75-2=73$\n\n$75+2=77$\n\n答:这三个奇数分别是 $73 、 75 、 77$ 。\n\n【点睛】解答此题的关键是求出中间的那个奇数, 然后根据两个连续的奇数的特征来解答。"} {"id": "24030", "image": [], "answer": "解$\\times$", "solution": "null", "level": "五年级", "question": "2 既是最小的质数, 又是最小的偶数. ( )", "options": [], "subject": "算术", "analysis": "解$\\times$"} {"id": "24031", "image": [], "answer": "解$\\times$\n\n【分析】一个数最小的因数是 1 , 最大的因数是它本身, 一个数的因数的个数是有限的。\n\n【详解】 30 的因数有 $1 、 2 、 3 、 5 、 6 、 10 、 15 、 30$, 共 8 个。原题说法错误。\n\n故答案为: $x$", "solution": "null", "level": "五年级", "question": "30 的因数有无数个。()", "options": [], "subject": "算术", "analysis": "解$\\times$\n\n【分析】一个数最小的因数是 1 , 最大的因数是它本身, 一个数的因数的个数是有限的。\n\n【详解】 30 的因数有 $1 、 2 、 3 、 5 、 6 、 10 、 15 、 30$, 共 8 个。原题说法错误。\n\n故答案为: $x$"} {"id": "24032", "image": [], "answer": "解$\\times$\n\n【分析】一个数最小的因数是 1, 最大的因数是它本身。\n\n【详解】12 的因数还有 1 , 所以判断错误。", "solution": "null", "level": "五年级", "question": "$2 、 3 、 4 、 6 、 12$ 是 12 的全部因数。()", "options": [], "subject": "算术", "analysis": "解$\\times$\n\n【分析】一个数最小的因数是 1, 最大的因数是它本身。\n\n【详解】12 的因数还有 1 , 所以判断错误。"} {"id": "24034", "image": [], "answer": "解错误\n\n【分析】质数只有 1 和本身两个因数; 所以质数无论多么大,都只有两个因数; 12 比 15 小, 12 的因数有 $1 、 2 、 3 、 4 、 6 、 12$ 一共 6 个, 15 的因数有 $1 、 3 、 5 、 15$ 一共 4 个, 12 的因数比 15 的因数多.\n\n【详解】解: 一个数越大, 它的因数不一定越多; 一个数越小, 它的因数也不一定就越少. 故答案为错误.", "solution": "null", "level": "五年级", "question": "一个数越大, 它的因数的个数就越多; 一个数越小, 它的因数的个数就越小. ( )", "options": [], "subject": "算术", "analysis": "解错误\n\n【分析】质数只有 1 和本身两个因数; 所以质数无论多么大,都只有两个因数; 12 比 15 小, 12 的因数有 $1 、 2 、 3 、 4 、 6 、 12$ 一共 6 个, 15 的因数有 $1 、 3 、 5 、 15$ 一共 4 个, 12 的因数比 15 的因数多.\n\n【详解】解: 一个数越大, 它的因数不一定越多; 一个数越小, 它的因数也不一定就越少. 故答案为错误."} {"id": "24035", "image": [], "answer": "解$\\times$\n【分析】一个数的倍数, 最小的倍数是它本身, 没有最大的倍数。据此解答即可。\n\n【详解】由分析可知, $2 、 3 、 5$ 本身就是质数, 可是 2 是 2 的倍数, 3 是 3 的倍数, 5 是 5 的倍数,所以说质数一定不是 2、3、5 的倍数, 这种说法是错误的。\n\n故答案为: $x$\n\n【点睛】本题考查质数和倍数的定义, 明确它们的定义是解题的关键。", "solution": "null", "level": "五年级", "question": "质数一定不是 $2,3,5$ 的倍数。( )", "options": [], "subject": "算术", "analysis": "解$\\times$\n【分析】一个数的倍数, 最小的倍数是它本身, 没有最大的倍数。据此解答即可。\n\n【详解】由分析可知, $2 、 3 、 5$ 本身就是质数, 可是 2 是 2 的倍数, 3 是 3 的倍数, 5 是 5 的倍数,所以说质数一定不是 2、3、5 的倍数, 这种说法是错误的。\n\n故答案为: $x$\n\n【点睛】本题考查质数和倍数的定义, 明确它们的定义是解题的关键。"} {"id": "24036", "image": [], "answer": "解9775\n\n【详解】试题分析: 先找出较大数 23 的倍数(200 以内), 再用 200 减去这些倍数, 从中找出属于 17 的倍数, 即可确定这两个自然数, 算出乘积.\n\n解: 200 以内是 23 的倍数的数是: $23,46,69,92,115,138,161,184$ 共有八个.用 200 依次减去这八个数得 $177,154,131,108,85,62,39,16$,其中只有 85 是 17 的倍数.\n\n所以 $200=115+85$,\n\n$115 \\times 85=9775 ;$\n\n答: 这两个自然数的积是 9775 .\n\n点评: 此题主要考查找一个数的倍数的方法以及解决问题的能力.", "solution": "null", "level": "五年级", "question": "将 200 拆成两个自然数之和, 其中一个是 17 的倍数, 另一个是 23 的倍数, 那么这两个自然数的积是多少?", "options": [], "subject": "算术", "analysis": "解9775\n\n【详解】试题分析: 先找出较大数 23 的倍数(200 以内), 再用 200 减去这些倍数, 从中找出属于 17 的倍数, 即可确定这两个自然数, 算出乘积.\n\n解: 200 以内是 23 的倍数的数是: $23,46,69,92,115,138,161,184$ 共有八个.用 200 依次减去这八个数得 $177,154,131,108,85,62,39,16$,其中只有 85 是 17 的倍数.\n\n所以 $200=115+85$,\n\n$115 \\times 85=9775 ;$\n\n答: 这两个自然数的积是 9775 .\n\n点评: 此题主要考查找一个数的倍数的方法以及解决问题的能力."} {"id": "24037", "image": [], "answer": "解214\n\n【分析】除了 1 和它本身外没有别的因数的数为质数, 除了 1 和它本身外还有别的因数的数为合数,由此可知, 最小的质数为 2 , 最小的合数为 4 , 既不是质数也不是合数是 0 或 1 . 即这个数的百位为 2 ,十位为 0 或 1 . 再根据这个三位数减去最小的合数后, 既是 2 的倍数也是 3 和 5 的倍数, 由能被 2 整除数的特征, 能被 3 整除数的特征, 能被 5 整除数的特征可知, 个位数为 $0+4=4$. 再分析 $204-4=200$, $214-4=210$, 看 200 和 210 谁能被 3 整除即为所求。\n\n【详解】根据质数与合数的定义可知, 这个数的百位为 2 , 十位为 0 或 1 ;\n\n又根据能被 2、3、5 整除数的特征可知,\n\n这个数个位数为 $0+4=4$,\n\n则这个三位数是 204 或 214 ;\n\n由于 $204-4=200,200$ 不是 3 的倍数, 不符合题意, 舍去;\n\n$214-4=210,210$ 既是 2 的倍数也是 3 和 5 的倍数, 符合题意。\n\n故小明家的门牌号是 214 .\n\n【点睛】本题考查了合数与质数, 找一个数的倍数的方法。其中的知识点为: 合数与质数的定义; 2、 3 与 5 的整除特征。", "solution": "null", "level": "五年级", "question": "小明家的门牌号上是一个三位数, 它的最高位上是最小的质数, 十位上的数既不是质数也不是合数。如果这个三位数减去最小的合数后, 既是 2 的倍数也是 3 和 5 的倍数。小明家的门牌号是多少?", "options": [], "subject": "算术", "analysis": "解214\n\n【分析】除了 1 和它本身外没有别的因数的数为质数, 除了 1 和它本身外还有别的因数的数为合数,由此可知, 最小的质数为 2 , 最小的合数为 4 , 既不是质数也不是合数是 0 或 1 . 即这个数的百位为 2 ,十位为 0 或 1 . 再根据这个三位数减去最小的合数后, 既是 2 的倍数也是 3 和 5 的倍数, 由能被 2 整除数的特征, 能被 3 整除数的特征, 能被 5 整除数的特征可知, 个位数为 $0+4=4$. 再分析 $204-4=200$, $214-4=210$, 看 200 和 210 谁能被 3 整除即为所求。\n\n【详解】根据质数与合数的定义可知, 这个数的百位为 2 , 十位为 0 或 1 ;\n\n又根据能被 2、3、5 整除数的特征可知,\n\n这个数个位数为 $0+4=4$,\n\n则这个三位数是 204 或 214 ;\n\n由于 $204-4=200,200$ 不是 3 的倍数, 不符合题意, 舍去;\n\n$214-4=210,210$ 既是 2 的倍数也是 3 和 5 的倍数, 符合题意。\n\n故小明家的门牌号是 214 .\n\n【点睛】本题考查了合数与质数, 找一个数的倍数的方法。其中的知识点为: 合数与质数的定义; 2、 3 与 5 的整除特征。"} {"id": "24038", "image": [], "answer": "解102 人\n【分析】根据题干可得, 这列学生总人数是一个大于 100 的三位数, 且是 5 的倍数少 2 的数, 由此先求出大于 100 的 5 的倍数最小是 105 , 再减去 2 即可得出这批学生的人数。\n\n【详解】大于 100 的 5 的倍数最小是 105 ,\n\n$105-2=103$ (人) ,\n\n答: 这些学生最少有 102 人。\n\n【点睛】此题主要考查了求 5 的倍数的方法的灵活应用。", "solution": "null", "level": "五年级", "question": "有一百多名同学站队, 站成了 5 列, 却少了 2 人, 这些学生最少有多少人?", "options": [], "subject": "算术", "analysis": "解102 人\n【分析】根据题干可得, 这列学生总人数是一个大于 100 的三位数, 且是 5 的倍数少 2 的数, 由此先求出大于 100 的 5 的倍数最小是 105 , 再减去 2 即可得出这批学生的人数。\n\n【详解】大于 100 的 5 的倍数最小是 105 ,\n\n$105-2=103$ (人) ,\n\n答: 这些学生最少有 102 人。\n\n【点睛】此题主要考查了求 5 的倍数的方法的灵活应用。"} {"id": "24041", "image": [], "answer": "解9781692\n\n【分析】因为电话号码中的 7 个数字都是一位数, 结合一个数的因数、倍数以及质数、合数、偶数的定义, 即可分别判断出这个电话号码中从左到右的每一个数字。\n\n【详解】第一位: 有因数 $1 、 3 、 9$ 的数字是: $9,18 \\ldots$, 所以这个数字是 9 ;\n\n第二位: 是 7 的倍数, 7 的倍数有 $7,14 \\ldots$, 所以这个数字是 7 ;\n\n第三位: 一个数的最小倍数是它本身, 所以这个数字是 8 ;\n\n第四位: 1 既不是质数, 也不是合数, 所这个数字是 1 ;\n\n第五位: 是 $1 、 2$ 和 3 的倍数是 $6,12, \\ldots$ 所以这个数字是 6 ;\n\n第六位: 最大的一位数是 9 ;\n第七位:既是偶数又是质数的数字是 2 。\n\n这个电话号码是 9781692 。\n\n【点睛】解答此题的关键是要明确电话号码中每一位上的数字都是一位数, 且要熟记一些特殊的数字: 1 既不是质数也不是合数, 2 是质数也是偶数以及求一个数的倍数与因数的方法。", "solution": "null", "level": "五年级", "question": "请你写出王老师家的电话号码。\n\n第一位: 有因数 $1 、 3 、 9$; 第二位: 是 7 的倍数;\n\n第三位:最小倍数是 8 ; 第四位:不是质数,不是合数,又不是 0 ;\n\n第五位: 是 $1 、 2$ 和 3 的倍数; 第六位: 最大的一位数;\n\n第七位:是偶数又是质数。\n\n电话号码是(从左到右的顺序):", "options": [], "subject": "算术", "analysis": "解9781692\n\n【分析】因为电话号码中的 7 个数字都是一位数, 结合一个数的因数、倍数以及质数、合数、偶数的定义, 即可分别判断出这个电话号码中从左到右的每一个数字。\n\n【详解】第一位: 有因数 $1 、 3 、 9$ 的数字是: $9,18 \\ldots$, 所以这个数字是 9 ;\n\n第二位: 是 7 的倍数, 7 的倍数有 $7,14 \\ldots$, 所以这个数字是 7 ;\n\n第三位: 一个数的最小倍数是它本身, 所以这个数字是 8 ;\n\n第四位: 1 既不是质数, 也不是合数, 所这个数字是 1 ;\n\n第五位: 是 $1 、 2$ 和 3 的倍数是 $6,12, \\ldots$ 所以这个数字是 6 ;\n\n第六位: 最大的一位数是 9 ;\n第七位:既是偶数又是质数的数字是 2 。\n\n这个电话号码是 9781692 。\n\n【点睛】解答此题的关键是要明确电话号码中每一位上的数字都是一位数, 且要熟记一些特殊的数字: 1 既不是质数也不是合数, 2 是质数也是偶数以及求一个数的倍数与因数的方法。"} {"id": "24056", "image": [], "answer": "解 一个数的最小倍数是 8 , 那这个数一定是 8 , 说法正确。\n\n故答案为: $\\sqrt{ }$ 。", "solution": "null", "level": "五年级", "question": "(2022 秋・定州市期末)一个数的最小倍数是 8 , 那这个数一定是 8 。", "options": [], "subject": "算术", "analysis": "解 一个数的最小倍数是 8 , 那这个数一定是 8 , 说法正确。\n\n故答案为: $\\sqrt{ }$ 。"} {"id": "24058", "image": [], "answer": "解一个三位数 $24 \\square$, 它既是 2 的倍数又是 3 的倍数, $\\square$ 里可以填 0 和 6 两个数, 故原题说法正确。\n\n故答案为: $\\sqrt{ }$ 。", "solution": "null", "level": "五年级", "question": "(2022 秋・茂名期末)一个三位数 $24 \\square$, 它既是 2 的倍数又是 3 的倍数, $\\square$ 里可以有 2 种填法。", "options": [], "subject": "算术", "analysis": "解一个三位数 $24 \\square$, 它既是 2 的倍数又是 3 的倍数, $\\square$ 里可以填 0 和 6 两个数, 故原题说法正确。\n\n故答案为: $\\sqrt{ }$ 。"} {"id": "24059", "image": [], "answer": "$4 \\times 7=28,4$ 和 7 是 28 的因数, 28 是 4 和 7 的倍数。原题说法错误。故答案为: $\\times$ 。", "solution": "null", "level": "五年级", "question": "2022 秋 ・市中区期末) $4 \\times 7=28 , 4$ 是因数, 7 是因数, 28 是倍数。", "options": [], "subject": "算术", "analysis": "$4 \\times 7=28,4$ 和 7 是 28 的因数, 28 是 4 和 7 的倍数。原题说法错误。故答案为: $\\times$ 。"} {"id": "24062", "image": [], "answer": "解$45=1 \\times 45=3 \\times 15=5 \\times 9$\n\n$$\n\\begin{aligned}\n& 9 \\times 1=9 \\\\\n& 9 \\times 2=18 \\\\\n& 9 \\times 3=27 \\\\\n& 9 \\times 4=36 \\\\\n& 9 \\times 5=45\n\\end{aligned}\n$$\n\n所以, 既是 45 的因数, 又是 9 的倍数的有: 9、 45 。\n\n答: 这个计算器的价格可能是 9 元, 也可能是 45 元。", "solution": "null", "level": "五年级", "question": "(2022 春・固安县期中)一个计算器的价格既是 45 的因数, 又是 9 的倍数, 这个计算器的价格可能是多少元?", "options": [], "subject": "算术", "analysis": "解$45=1 \\times 45=3 \\times 15=5 \\times 9$\n\n$$\n\\begin{aligned}\n& 9 \\times 1=9 \\\\\n& 9 \\times 2=18 \\\\\n& 9 \\times 3=27 \\\\\n& 9 \\times 4=36 \\\\\n& 9 \\times 5=45\n\\end{aligned}\n$$\n\n所以, 既是 45 的因数, 又是 9 的倍数的有: 9、 45 。\n\n答: 这个计算器的价格可能是 9 元, 也可能是 45 元。"} {"id": "24063", "image": [], "answer": "解因为 365 不是 2 的倍数, 所以每 2 瓶装一提, 不能正好装完。\n\n因为 365 是 5 的倍数, 所以每 5 瓶装一箱, 能正好装完。", "solution": "null", "level": "五年级", "question": "(2021 秋・兴平市期末)便民超市新运进 365 瓶无菌消毒洗手液,如果每 2 瓶装一提,能正好装完吗?如果每 5 瓶装一箱, 能正好装完吗?为什么?", "options": [], "subject": "算术", "analysis": "解因为 365 不是 2 的倍数, 所以每 2 瓶装一提, 不能正好装完。\n\n因为 365 是 5 的倍数, 所以每 5 瓶装一箱, 能正好装完。"} {"id": "24064", "image": [], "answer": "解 (1) 牛奶的价格同时是 $2 、 3 、 5$ 的倍数, 所以个位是 0 , 十位的数字加 0 是 3 的倍数, 所以十位最小是 3 , 那么这个价格就是 30 元。\n\n答: 这批牛奶最低价格是 30 元。\n\n(2)最贵的牛奶价格是 $2 、 3 、 5$ 的倍数, 所以个位是 0 , 十位的数字加 0 是 3 的倍数, 这个数比 100 小, 所以十位的数尽可能的大, 十位为 9 时, $9+0=9$ 是 3 的倍数, 所以低于 100 元的最贵的牛奶是 90 元。\n\n答: 如果最贵的牛奶价格低于 100 元, 最贵 90 元。", "solution": "null", "level": "五年级", "question": "(2022 春・忻州月考)新源超市新进一批不同包装的牛奶,每箱牛奶价格既是 2 的倍数又是 3 的倍数, 还是 5 的倍数。\n\n(1)这批牛奶最低价格是多少?\n\n(2)如果最贵的牛奶价格低于 100 元,最贵多少元?", "options": [], "subject": "算术", "analysis": "解 (1) 牛奶的价格同时是 $2 、 3 、 5$ 的倍数, 所以个位是 0 , 十位的数字加 0 是 3 的倍数, 所以十位最小是 3 , 那么这个价格就是 30 元。\n\n答: 这批牛奶最低价格是 30 元。\n\n(2)最贵的牛奶价格是 $2 、 3 、 5$ 的倍数, 所以个位是 0 , 十位的数字加 0 是 3 的倍数, 这个数比 100 小, 所以十位的数尽可能的大, 十位为 9 时, $9+0=9$ 是 3 的倍数, 所以低于 100 元的最贵的牛奶是 90 元。\n\n答: 如果最贵的牛奶价格低于 100 元, 最贵 90 元。"} {"id": "24175", "image": [], "answer": "解$\\times$\n\n【分析】根据因数和倍数的意义, 在乘法算式 $\\mathrm{a} \\times \\mathrm{b}=\\mathrm{c}(\\mathrm{a} 、 \\mathrm{~b} 、 \\mathrm{c}$ 均为非 0 的自然数 $)$ 中, $\\mathrm{a} 、 \\mathrm{~b}$ 就是 $\\mathrm{c}$的因数, $\\mathrm{c}$ 就是 $\\mathrm{a} 、 \\mathrm{~b}$ 的倍数。据此解答。\n\n【详解】因为 $3 \\times 15=45$, 所以 $3 、 15$ 是 45 的因数, 45 是 $3 、 15$ 的倍数。但不能说 3 和 15 是因数, 45 是倍数。所以原题的说法是错误的。\n\n故答案为: $\\times$\n\n【点睛】此题主要考查因数和倍数的意义, 因数和倍数两个不同的概念是相互依存的, 不能单独存在。", "solution": "null", "level": "五年级", "question": "因为 $3 \\times 15=45$, 所以 3 和 15 是因数, 45 是倍数。 $($ )", "options": [], "subject": "算术", "analysis": "解$\\times$\n\n【分析】根据因数和倍数的意义, 在乘法算式 $\\mathrm{a} \\times \\mathrm{b}=\\mathrm{c}(\\mathrm{a} 、 \\mathrm{~b} 、 \\mathrm{c}$ 均为非 0 的自然数 $)$ 中, $\\mathrm{a} 、 \\mathrm{~b}$ 就是 $\\mathrm{c}$的因数, $\\mathrm{c}$ 就是 $\\mathrm{a} 、 \\mathrm{~b}$ 的倍数。据此解答。\n\n【详解】因为 $3 \\times 15=45$, 所以 $3 、 15$ 是 45 的因数, 45 是 $3 、 15$ 的倍数。但不能说 3 和 15 是因数, 45 是倍数。所以原题的说法是错误的。\n\n故答案为: $\\times$\n\n【点睛】此题主要考查因数和倍数的意义, 因数和倍数两个不同的概念是相互依存的, 不能单独存在。"} {"id": "24177", "image": [], "answer": "解$\\sqrt{ }$\n\n【分析】根据能被 $2 、 3 、 5$ 整除数的特征可知; 能被 2 整除的数个位上要首先满足是 $0 、 2 、 4 、 6 、 8$,然后分析能被 3 整除的数的特征, 即求出各个数位上的和, 分析是不是 3 的倍数, 能被 5 整除的数个\n位上是 0 和 5 , 据此分析由数字卡片 307 组成的所有三位数是否满足即可。\n\n【详解】因为 $3+7+5=15,15$ 是 3 的倍数;\n\n所以由数字卡片 3 3 7 任意摆出一个三位数, 这个三位数一定是 3 的倍数, 也就一定有因数 3 。故答案为: $\\sqrt{ }$\n\n【点睛】解答此题的关键是:根据能被 3 整除的数的特征, 进行解答。", "solution": "null", "level": "五年级", "question": "用数字卡片 3 活意摆一个三位数, 这个数一定有因数 3 。( ( )", "options": [], "subject": "算术", "analysis": "解$\\sqrt{ }$\n\n【分析】根据能被 $2 、 3 、 5$ 整除数的特征可知; 能被 2 整除的数个位上要首先满足是 $0 、 2 、 4 、 6 、 8$,然后分析能被 3 整除的数的特征, 即求出各个数位上的和, 分析是不是 3 的倍数, 能被 5 整除的数个\n位上是 0 和 5 , 据此分析由数字卡片 307 组成的所有三位数是否满足即可。\n\n【详解】因为 $3+7+5=15,15$ 是 3 的倍数;\n\n所以由数字卡片 3 3 7 任意摆出一个三位数, 这个三位数一定是 3 的倍数, 也就一定有因数 3 。故答案为: $\\sqrt{ }$\n\n【点睛】解答此题的关键是:根据能被 3 整除的数的特征, 进行解答。"} {"id": "24179", "image": [], "answer": "解$\\times$\n\n【分析】一个数, 如果只有 1 和它本身两个因数, 那么这样的数叫做质数; 一个数, 如果除了 1 和它本身还有别的因数, 那么这样的数叫做合数; 自然数中, 能被 2 整除的数是偶数, 不能被 2 整除的数是奇数; 据此解答。\n\n【详解】并不是所有的奇数都是合数, 如: 3 是奇数, 但同时 3 也是质数。所以原题说法错误。故答案为: $\\times$\n\n【点睛】本题主要考查学生对质数、合数、奇数、偶数的认识, 掌握它们的意义是解答题目的关键。", "solution": "null", "level": "五年级", "question": "所有的奇数都是合数。( )", "options": [], "subject": "算术", "analysis": "解$\\times$\n\n【分析】一个数, 如果只有 1 和它本身两个因数, 那么这样的数叫做质数; 一个数, 如果除了 1 和它本身还有别的因数, 那么这样的数叫做合数; 自然数中, 能被 2 整除的数是偶数, 不能被 2 整除的数是奇数; 据此解答。\n\n【详解】并不是所有的奇数都是合数, 如: 3 是奇数, 但同时 3 也是质数。所以原题说法错误。故答案为: $\\times$\n\n【点睛】本题主要考查学生对质数、合数、奇数、偶数的认识, 掌握它们的意义是解答题目的关键。"} {"id": "24180", "image": [], "answer": "解$\\times$\n\n【分析】根据偶数、奇数的意义: 是 2 的倍数的数叫做偶数; 一个数只有 1 和它本身两个因数, 这个数叫作质数, 质数除了 2 以外都是奇数, 根据奇数 + 偶数 $=$ 奇数, 奇数 + 奇数 $=$ 偶数, 解答判断即可。\n\n【详解】由于最小的质数为 2 ,\n\n偶数十奇数 $=$ 奇数,\n\n质数中除了 2 之外的所有质数都为奇数,\n\n2 加其它的任意一个质数的和都为奇数,\n\n所以, 两个质数的和都是偶数的说法是错误的。\n\n故答案为: $\\times$\n\n【点睛】此题主要考查奇数与偶数的认识、奇数和偶数的运算性质以及合数与质数的定义。", "solution": "null", "level": "五年级", "question": "任意两个质数的和都是偶数。( )", "options": [], "subject": "算术", "analysis": "解$\\times$\n\n【分析】根据偶数、奇数的意义: 是 2 的倍数的数叫做偶数; 一个数只有 1 和它本身两个因数, 这个数叫作质数, 质数除了 2 以外都是奇数, 根据奇数 + 偶数 $=$ 奇数, 奇数 + 奇数 $=$ 偶数, 解答判断即可。\n\n【详解】由于最小的质数为 2 ,\n\n偶数十奇数 $=$ 奇数,\n\n质数中除了 2 之外的所有质数都为奇数,\n\n2 加其它的任意一个质数的和都为奇数,\n\n所以, 两个质数的和都是偶数的说法是错误的。\n\n故答案为: $\\times$\n\n【点睛】此题主要考查奇数与偶数的认识、奇数和偶数的运算性质以及合数与质数的定义。"} {"id": "24181", "image": ["12449.jpg"], "answer": "解小芳; 726 是 3 的倍数。\n\n【分析】一瓶饮料 3 元钱, 总钱数应该是 3 的倍数, 3 的倍数的特征: 一个数各个数位上的数字的和是 3 的倍数, 这个数就是 3 的倍数, 据此分析。\n\n【详解】 $9+8+9=26$\n\n$6+3+7=16$\n\n$7+2+6=15$\n\n989 和 637 不是 3 的倍数, 726 是 3 的倍数。\n\n答: 小芳算的对, 因为小芳算的钱数是 3 的倍数。\n【点睛】关键是掌握 3 的倍数的特征。", "solution": "null", "level": "五年级", "question": "星期天, 实验小学组织两个年级的学生去参加公益活动, 每个年级都有 3 个班。休息时, 老师为每名同学各买了一瓶 3 元的饮料, 请大家帮忙算一算一共花了多少钱。\n\n\n\n老师说只有一个人算对了, 你认为谁算的对呢? 为什么?", "options": [], "subject": "算术", "analysis": "解小芳; 726 是 3 的倍数。\n\n【分析】一瓶饮料 3 元钱, 总钱数应该是 3 的倍数, 3 的倍数的特征: 一个数各个数位上的数字的和是 3 的倍数, 这个数就是 3 的倍数, 据此分析。\n\n【详解】 $9+8+9=26$\n\n$6+3+7=16$\n\n$7+2+6=15$\n\n989 和 637 不是 3 的倍数, 726 是 3 的倍数。\n\n答: 小芳算的对, 因为小芳算的钱数是 3 的倍数。\n【点睛】关键是掌握 3 的倍数的特征。"} {"id": "24184", "image": [], "answer": "解68 个\n\n【分析】 4 个 4 个的数正好数完, 说明这筐苹果的个数是 4 的倍数, 找出小于 70 的 4 的最大倍数是多少即可。\n\n【详解】 $70 \\div 4=17 \\cdots \\cdots \\cdot 2$\n\n$70-2=68$ (个)\n\n答: 这筐苹果最多有 68 个。\n\n【点睛】本题考查的是因数与倍数, 一个非零自然数的倍数有无限个, 其中最小的倍数是其自身。", "solution": "null", "level": "五年级", "question": "一筐苹果,不到 70 个,如果 4 个 4 个的数正好数完,这筐苹果最多有多少个?", "options": [], "subject": "算术", "analysis": "解68 个\n\n【分析】 4 个 4 个的数正好数完, 说明这筐苹果的个数是 4 的倍数, 找出小于 70 的 4 的最大倍数是多少即可。\n\n【详解】 $70 \\div 4=17 \\cdots \\cdots \\cdot 2$\n\n$70-2=68$ (个)\n\n答: 这筐苹果最多有 68 个。\n\n【点睛】本题考查的是因数与倍数, 一个非零自然数的倍数有无限个, 其中最小的倍数是其自身。"} {"id": "24185", "image": [], "answer": "解$\\mathrm{a}$ 是 17; b 是 2; c 是 11\n\n【分析】 221 是 $\\mathrm{a}$ 与 $(\\mathrm{b}+\\mathrm{c})$ 的积, 先将 221 写成两个数相乘的形式, 即 $221=13 \\times 17$, 因为 $\\mathrm{b}$ 与 $\\mathrm{c}$ 的和为奇数, 所以 $\\mathrm{b}$ 和 $\\mathrm{c}$ 一个为奇数一个为偶数, $\\mathrm{a} 、 \\mathrm{~b} 、 \\mathrm{c}$ 是不同的质数, 且 2 是唯一的偶质数, 则 $\\mathrm{b}$和 $\\mathrm{c}$ 中有一个数为 2 , 据此求出 $\\mathrm{a} 、 \\mathrm{~b} 、 \\mathrm{c}$ 的值。\n\n【详解】分析可知, $221=13 \\times 17, \\mathrm{~b}$ 和 c 中有一个数为 2 。\n\n当 $\\mathrm{a}=13$ 时, $17-2=15,15$ 不是质数, 所以不符合题意;\n\n当 $\\mathrm{a}=17$ 时, $13-2=11,11$ 是质数且 $\\mathrm{b}<\\mathrm{c}$ ,所以 $\\mathrm{a}$ 为 $17, \\mathrm{~b}$ 为 $2, \\mathrm{c}$ 为 11 。\n\n答: a 代表 $17, \\mathrm{~b}$ 代表 $2, \\mathrm{c}$ 代表 11 。\n【点睛】把 221 分解为两个数相乘的积, 再根据 b、c 之间的关系求出 $a 、 b 、 c$ 的值是解答题目的关键。", "solution": "null", "level": "五年级", "question": "如果 $a \\times(b+c)=221$, 且 $a 、 b 、 c$ 是不同的质数 $(\\mathrm{b}<\\mathrm{c})$ ,那么 $a 、 b 、 c$ 各代表多少?写出分析过程。", "options": [], "subject": "算术", "analysis": "解$\\mathrm{a}$ 是 17; b 是 2; c 是 11\n\n【分析】 221 是 $\\mathrm{a}$ 与 $(\\mathrm{b}+\\mathrm{c})$ 的积, 先将 221 写成两个数相乘的形式, 即 $221=13 \\times 17$, 因为 $\\mathrm{b}$ 与 $\\mathrm{c}$ 的和为奇数, 所以 $\\mathrm{b}$ 和 $\\mathrm{c}$ 一个为奇数一个为偶数, $\\mathrm{a} 、 \\mathrm{~b} 、 \\mathrm{c}$ 是不同的质数, 且 2 是唯一的偶质数, 则 $\\mathrm{b}$和 $\\mathrm{c}$ 中有一个数为 2 , 据此求出 $\\mathrm{a} 、 \\mathrm{~b} 、 \\mathrm{c}$ 的值。\n\n【详解】分析可知, $221=13 \\times 17, \\mathrm{~b}$ 和 c 中有一个数为 2 。\n\n当 $\\mathrm{a}=13$ 时, $17-2=15,15$ 不是质数, 所以不符合题意;\n\n当 $\\mathrm{a}=17$ 时, $13-2=11,11$ 是质数且 $\\mathrm{b}<\\mathrm{c}$ ,所以 $\\mathrm{a}$ 为 $17, \\mathrm{~b}$ 为 $2, \\mathrm{c}$ 为 11 。\n\n答: a 代表 $17, \\mathrm{~b}$ 代表 $2, \\mathrm{c}$ 代表 11 。\n【点睛】把 221 分解为两个数相乘的积, 再根据 b、c 之间的关系求出 $a 、 b 、 c$ 的值是解答题目的关键。"} {"id": "24485", "image": [], "answer": "解$\\times$\n\n【分析】棱长 $1 \\mathrm{~m}$ 的正方体, 体积是 $1 \\mathrm{~m}^{3}$, 棱长 $1 \\mathrm{dm}$ 的正方体, 体积是 $1 \\mathrm{dm}^{3}, 1 \\mathrm{dm}^{3}=1 \\mathrm{~L}$, 据此根据体积和容积单位的认识, 以及生活经验进行判断。\n\n【详解】 $1 \\mathrm{~m}^{3}$ 的水太多了, 一天, 小明渴极了, 一次喝了 $1 \\mathrm{~L}$ 的水还有可能, 所以原题说法错误。故答案为: $\\times$\n\n【点睛】关键是建立单位标准,可以利用身边熟悉的事物建立单位标准。", "solution": "null", "level": "五年级", "question": "一天,小明渴极了,一次喝了 $1 \\mathrm{~m}^{3}$ 的水。()", "options": [], "subject": "算术", "analysis": "解$\\times$\n\n【分析】棱长 $1 \\mathrm{~m}$ 的正方体, 体积是 $1 \\mathrm{~m}^{3}$, 棱长 $1 \\mathrm{dm}$ 的正方体, 体积是 $1 \\mathrm{dm}^{3}, 1 \\mathrm{dm}^{3}=1 \\mathrm{~L}$, 据此根据体积和容积单位的认识, 以及生活经验进行判断。\n\n【详解】 $1 \\mathrm{~m}^{3}$ 的水太多了, 一天, 小明渴极了, 一次喝了 $1 \\mathrm{~L}$ 的水还有可能, 所以原题说法错误。故答案为: $\\times$\n\n【点睛】关键是建立单位标准,可以利用身边熟悉的事物建立单位标准。"} {"id": "24198", "image": [], "answer": "解 $2,3,5 ; 4 ; 1,3,5 ; 2,4 ; 1,5$", "solution": "null", "level": "五年级", "question": "在 $1,2,3,4,5$ 中, 质数有 $\\qquad$ , 合数有 $\\qquad$ , 奇数有 $\\qquad$ , 偶数有 $\\qquad$ ; 其中 $\\qquad$是 5 的因数。", "options": [], "subject": "算术", "analysis": "解 $2,3,5 ; 4 ; 1,3,5 ; 2,4 ; 1,5$"} {"id": "24199", "image": [], "answer": " $96 ; 102$", "solution": "null", "level": "五年级", "question": "既是 2 的倍数, 又有因数 3 的最大两位数是 $\\qquad$ , 最小三位数是 $\\qquad$。", "options": [], "subject": "算术", "analysis": " $96 ; 102$"} {"id": "24200", "image": [], "answer": "解 $4 ; 97$", "solution": "null", "level": "五年级", "question": "186 至少再加上 $\\qquad$就是 5 的倍数; $29 \\square 0$ 是 3 的倍数, 末尾的两位数最大是。 $\\qquad$", "options": [], "subject": "算术", "analysis": "解 $4 ; 97$"} {"id": "24201", "image": [], "answer": "解 57129", "solution": "null", "level": "五年级", "question": "赵阿姨手机锁屏密码是由六位数 A5BCDE 组成的。其中 A 有因数 2 和 $3, \\mathrm{~B}$ 的最大因数是 7 , $\\mathrm{C}$ 既不是质数也不是合数, $\\mathrm{D}$ 是最小的质数, $\\mathrm{E}$ 既是奇数也是合数。锁屏密码是 $\\qquad$。", "options": [], "subject": "算术", "analysis": "解 57129"} {"id": "24202", "image": [], "answer": "解偶数; 奇数", "solution": "null", "level": "五年级", "question": "两个奇数的差是 $\\qquad$ , 奇数与偶数的和是 $\\qquad$。(选填“偶数”或“奇数”)", "options": [], "subject": "算术", "analysis": "解偶数; 奇数"} {"id": "24203", "image": [], "answer": "2", "solution": "null", "level": "五年级", "question": "一个两位数是 5 的倍数, 它的各个数位上数字的和是 9 的倍数, 这样的两位数一共有个。", "options": [], "subject": "算术", "analysis": "2"} {"id": "24205", "image": ["12451.jpg", "12452.jpg", "12453.jpg", "12454.jpg", "12455.jpg", "12455.jpg"], "answer": "", "solution": "null", "level": "五年级", "question": "把下面的数按要求填入圈内。\n(1) 92 11\n6\n2 的倍数\n28\n\n\n$58 \\quad 125$\n\n\n既是 2 的倍数,又是 5 的倍数\n\n\n\n(2)猜猜我是谁。\n\n我的最大因数和最小倍数都是 18 。\n\n\n\n我是 ( )\n\n质数\n\n合数", "options": [], "subject": "算术", "analysis": ""} {"id": "24206", "image": [], "answer": " $20 \\div 2=10$ (厘米)\n\n$7+3=10$\n$7 \\times 3=21$ (平方厘米)\n\n答: 这个长方形的面积是 21 平方厘米。", "solution": "null", "level": "五年级", "question": "一个长方形的周长为 20 厘米。已知这个长方形的长和宽都是以厘米为单位的不同的质数。这个长方形的面积是多少厘米?", "options": [], "subject": "算术", "analysis": " $20 \\div 2=10$ (厘米)\n\n$7+3=10$\n$7 \\times 3=21$ (平方厘米)\n\n答: 这个长方形的面积是 21 平方厘米。"} {"id": "24207", "image": [], "answer": "解 $1+3+4=8,8$ 不是 3 的倍数, 售货员阿姨说错了。\n\n答:售货员阿姨说错了,因为钱数应该是 3 的倍数。", "solution": "null", "level": "五年级", "question": "小朋友到文具店头日记本,日记本的单价已看不清楚,他买了 3 本日记本,售货员阿姨说应付 134 元,小红认为不对。你能解释这是为什么吗?", "options": [], "subject": "算术", "analysis": "解 $1+3+4=8,8$ 不是 3 的倍数, 售货员阿姨说错了。\n\n答:售货员阿姨说错了,因为钱数应该是 3 的倍数。"} {"id": "24209", "image": [], "answer": "解$2 \\times 3 \\times 5$\n\n$$\n\\begin{aligned}\n& =6 \\times 5 \\\\\n& =30 \\text { (人) } \\\\\n& 30 \\times 4=120 \\text { (人) }\n\\end{aligned}\n$$\n\n答:五年级至少有 120 人。", "solution": "null", "level": "五年级", "question": "荣怀国际小学五年级总人数共一百多人, 年级总人数恰好是 $2,3,5$ 的倍数, 五年级至少有多少人?", "options": [], "subject": "算术", "analysis": "解$2 \\times 3 \\times 5$\n\n$$\n\\begin{aligned}\n& =6 \\times 5 \\\\\n& =30 \\text { (人) } \\\\\n& 30 \\times 4=120 \\text { (人) }\n\\end{aligned}\n$$\n\n答:五年级至少有 120 人。"} {"id": "24210", "image": [], "answer": " (1) 10 以内最大的质数是 7; (2) 既是奇数又是合数的一位数是 9; (3)既是质数又是偶数是 2 ; (4) 既不是质数也不是合数的奇数是 1 ; (5) 最小的合数是 4 , 所以李老师车牌号的数字部分是: 79214。\n\n答: 李老师车牌号的数字部分是 79214。", "solution": "null", "level": "五年级", "question": "李老师的车牌号码从左往右的数字依次是:(1)10以内最大的质数;(2)既是奇数又是合数的数; (3) 既是质数又是偶数; (4) 既不是质数也不是合数的奇数; (5)最小的合数。你知道李老师车牌号的数字部分是多少吗?", "options": [], "subject": "算术", "analysis": " (1) 10 以内最大的质数是 7; (2) 既是奇数又是合数的一位数是 9; (3)既是质数又是偶数是 2 ; (4) 既不是质数也不是合数的奇数是 1 ; (5) 最小的合数是 4 , 所以李老师车牌号的数字部分是: 79214。\n\n答: 李老师车牌号的数字部分是 79214。"} {"id": "24226", "image": [], "answer": "解$\\sqrt{ }$\n\n【分析】 3 的倍数的特征是: 一个数各个数位上的数字之和是 3 的倍数, 则这个数就是 3 的倍数, 据此判断。\n\n【详解】因为 $0+2+5+8=15,15$ 是 3 的倍数, 所以用 $0 、 2 、 5 、 8$ 组成四位数一定是 3 的倍数, 原题说法正确。\n\n故答案为: $\\sqrt{ }$\n\n【点睛】掌握 3 的倍数的特征是解答的关键。", "solution": "null", "level": "五年级", "question": "用 $0 、 2 、 5 、 8$ 组成四位数, 组成的四位数一定是 3 的倍数。( )", "options": [], "subject": "算术", "analysis": "解$\\sqrt{ }$\n\n【分析】 3 的倍数的特征是: 一个数各个数位上的数字之和是 3 的倍数, 则这个数就是 3 的倍数, 据此判断。\n\n【详解】因为 $0+2+5+8=15,15$ 是 3 的倍数, 所以用 $0 、 2 、 5 、 8$ 组成四位数一定是 3 的倍数, 原题说法正确。\n\n故答案为: $\\sqrt{ }$\n\n【点睛】掌握 3 的倍数的特征是解答的关键。"} {"id": "24227", "image": [], "answer": "解$\\times$\n\n【分析】质数是指除了 1 和它本身的两个因数以外再没有其他的因数的数。能被 2 整除的数叫偶数。据此找出两个质数, 把它们加起来, 验证和是否都为偶数。\n\n【详解】根据分析得, 2 是质数, 3 是质数,\n$2+3=5$\n\n5 是奇数, 即 2 和 3 这两个质数的和是奇数, 所以原题的说法是错误的。\n\n故答案为: $\\times$\n\n【点睛】此题的解题关键是理解质数和偶数的定义, 通过举出反例, 得出结论。", "solution": "null", "level": "五年级", "question": "100 以内的任意两个质数的和都是偶数。( )", "options": [], "subject": "算术", "analysis": "解$\\times$\n\n【分析】质数是指除了 1 和它本身的两个因数以外再没有其他的因数的数。能被 2 整除的数叫偶数。据此找出两个质数, 把它们加起来, 验证和是否都为偶数。\n\n【详解】根据分析得, 2 是质数, 3 是质数,\n$2+3=5$\n\n5 是奇数, 即 2 和 3 这两个质数的和是奇数, 所以原题的说法是错误的。\n\n故答案为: $\\times$\n\n【点睛】此题的解题关键是理解质数和偶数的定义, 通过举出反例, 得出结论。"} {"id": "24228", "image": [], "answer": "解$\\times$\n\n【分析】质数只有 1 和它本身两个因数; 合数除了 1 和它本身以外, 还有其他因数; 1 只有它本身 1 个因数, 所以 1 既不是质数也不是合数; 据此判断。\n\n【详解】1 只有它本身 1 个因数, 所以原题说法错误。\n\n故答案为: $\\times$\n\n【点睛】本题的解题关键是理解掌握质数与合数以及因数的意义。", "solution": "null", "level": "五年级", "question": "任何一个非零的自然数至少都有两个因数。( )", "options": [], "subject": "算术", "analysis": "解$\\times$\n\n【分析】质数只有 1 和它本身两个因数; 合数除了 1 和它本身以外, 还有其他因数; 1 只有它本身 1 个因数, 所以 1 既不是质数也不是合数; 据此判断。\n\n【详解】1 只有它本身 1 个因数, 所以原题说法错误。\n\n故答案为: $\\times$\n\n【点睛】本题的解题关键是理解掌握质数与合数以及因数的意义。"} {"id": "24230", "image": [], "answer": "解$\\times$\n\n【分析】因数和倍数的概念是在非 0 自然数范围内的,据此判断。\n\n【详解】 7.2 是小数, 所以不能说 7.2 是 8 的倍数, 8 是 7.2 的因数。\n\n故答案为: $\\times$\n\n【点睛】本题考查了因数和倍数的认识。在非 0 自然数中, 如果 $a \\div b=c$, 那么 $a$ 是 $b$ 和 $c$ 的倍数, $b$和 $\\mathrm{c}$ 是 $\\mathrm{a}$ 的因数。", "solution": "null", "level": "五年级", "question": "7.2 是 8 的倍数, 8 是 7.2 的因数。( )", "options": [], "subject": "算术", "analysis": "解$\\times$\n\n【分析】因数和倍数的概念是在非 0 自然数范围内的,据此判断。\n\n【详解】 7.2 是小数, 所以不能说 7.2 是 8 的倍数, 8 是 7.2 的因数。\n\n故答案为: $\\times$\n\n【点睛】本题考查了因数和倍数的认识。在非 0 自然数中, 如果 $a \\div b=c$, 那么 $a$ 是 $b$ 和 $c$ 的倍数, $b$和 $\\mathrm{c}$ 是 $\\mathrm{a}$ 的因数。"} {"id": "24231", "image": [], "answer": "解$\\sqrt{ }$\n\n【分析】根据奇数、偶数的性质, 奇数 + 奇数 $=$ 偶数, 奇数 + 偶数 $=$ 奇数, 偶数 + 偶数 $=$ 偶数, 据此判断即可。\n\n【详解】根据奇数、偶数的性质可知, 奇数与偶数的和一定是奇数, 原题说法正确;\n\n故答案为: $\\sqrt{ }$\n\n【点睛】此题考查的目的是掌握奇数、偶数的性质。", "solution": "null", "level": "五年级", "question": "奇数与偶数的和一定是奇数。( )", "options": [], "subject": "算术", "analysis": "解$\\sqrt{ }$\n\n【分析】根据奇数、偶数的性质, 奇数 + 奇数 $=$ 偶数, 奇数 + 偶数 $=$ 奇数, 偶数 + 偶数 $=$ 偶数, 据此判断即可。\n\n【详解】根据奇数、偶数的性质可知, 奇数与偶数的和一定是奇数, 原题说法正确;\n\n故答案为: $\\sqrt{ }$\n\n【点睛】此题考查的目的是掌握奇数、偶数的性质。"} {"id": "24232", "image": [], "answer": "解3 个\n\n【分析】 5 的倍数特征:个位上的数字是 0 或 5 的数是 5 的倍数, 据此分析。\n\n【详解】 $20-17=3$ (个)\n\n答: 至少再来 3 个人才能正好分完。\n\n【点睛】关键是掌握 5 的倍数的特征。", "solution": "null", "level": "五年级", "question": "操场上一共有 17 个人, 5 个人分成一组, 至少再来几个人才能正好分完?", "options": [], "subject": "算术", "analysis": "解3 个\n\n【分析】 5 的倍数特征:个位上的数字是 0 或 5 的数是 5 的倍数, 据此分析。\n\n【详解】 $20-17=3$ (个)\n\n答: 至少再来 3 个人才能正好分完。\n\n【点睛】关键是掌握 5 的倍数的特征。"} {"id": "24233", "image": [], "answer": "解不对, 王老师付的钱数和花去的钱数均为偶数, 找回的钱数也应该为偶数。\n\n【分析】钢笔和笔记本的单价为偶数, 由“总价=单价 $\\times$ 数量”可知, 购买钢笔和笔记本花的钱数为偶数,总钱数为 100 元,找回的钱数=总钱数一花去的钱数,偶数与偶数的差为偶数,则找回的钱数应该为偶数, 据此解答。\n【详解】不对, 钢笔的支数和笔记本的本数均为自然数, 则买钢笔花的钱数为偶数, 买卡通笔记本花的钱数也为偶数, 找回的钱数 $=$ 总钱数 - 花去的钱数, 根据“偶数一偶数 $=$ 偶数”可知, 找回的钱数应该为偶数,营业员找给王老师 15 元, 15 为奇数,所以营业员找的钱数不对。\n\n【点睛】判断花去钱数的奇偶性并根据和差的奇偶性判断出找回钱数的奇偶性是解答题目的关键。", "solution": "null", "level": "五年级", "question": "王老师到文化用品超市买了一些钢笔和卡通笔记本。钢笔每支 8 元, 卡通笔记本每本 6 元, 她付给营业员 100 元,营业员找给王老师 15 元。请判断营业员找的钱对吗?并说明理由。", "options": [], "subject": "算术", "analysis": "解不对, 王老师付的钱数和花去的钱数均为偶数, 找回的钱数也应该为偶数。\n\n【分析】钢笔和笔记本的单价为偶数, 由“总价=单价 $\\times$ 数量”可知, 购买钢笔和笔记本花的钱数为偶数,总钱数为 100 元,找回的钱数=总钱数一花去的钱数,偶数与偶数的差为偶数,则找回的钱数应该为偶数, 据此解答。\n【详解】不对, 钢笔的支数和笔记本的本数均为自然数, 则买钢笔花的钱数为偶数, 买卡通笔记本花的钱数也为偶数, 找回的钱数 $=$ 总钱数 - 花去的钱数, 根据“偶数一偶数 $=$ 偶数”可知, 找回的钱数应该为偶数,营业员找给王老师 15 元, 15 为奇数,所以营业员找的钱数不对。\n\n【点睛】判断花去钱数的奇偶性并根据和差的奇偶性判断出找回钱数的奇偶性是解答题目的关键。"} {"id": "24234", "image": [], "answer": "解$11 、 13 、 15 、 17 、 19$\n\n【分析】相邻的奇数之间相差 2 , 用五个连续的奇数的和 $\\div 5$, 求出中间奇数, 进而推算出其它奇数。\n\n【详解】 $75 \\div 5=15$\n\n$15-2=13$\n\n$13-2=11$\n\n$15+2=17$\n\n$17+2=19$\n\n答: 这五个奇数分别是 $11 、 13 、 15 、 17 、 19$ 。\n\n【点睛】整数中, 是 2 的倍数的数叫偶数, 不是 2 的倍数的数叫奇数。", "solution": "null", "level": "五年级", "question": "五个连续的奇数的和是 75 , 这五个奇数分别是多少?", "options": [], "subject": "算术", "analysis": "解$11 、 13 、 15 、 17 、 19$\n\n【分析】相邻的奇数之间相差 2 , 用五个连续的奇数的和 $\\div 5$, 求出中间奇数, 进而推算出其它奇数。\n\n【详解】 $75 \\div 5=15$\n\n$15-2=13$\n\n$13-2=11$\n\n$15+2=17$\n\n$17+2=19$\n\n答: 这五个奇数分别是 $11 、 13 、 15 、 17 、 19$ 。\n\n【点睛】整数中, 是 2 的倍数的数叫偶数, 不是 2 的倍数的数叫奇数。"} {"id": "24236", "image": [], "answer": "解30 元\n\n【分析】 2 的倍数特征:个位是 $0 、 2 、 4 、 6 、 8$ 的数。 5 的倍数特征:个位是 0 或 5 的数。 $2 、 5$ 的倍数特征: 个位是 0 的数。先求出 40 元的一半是 20 元,那么花的钱数就在 $20 \\sim 40$ 元以内; 找出 20〜 40 内的 5 的倍数, 再在 5 的倍数中找 2 的倍数, 这样就可以知道王丽买书花的钱数。\n\n【详解】 $40 \\div 2=20$ (元)\n\n大于 20 且小于 40 内 5 的倍数有: $25 、 30 、 35$;\n\n其中是 2 的倍数的是: 30 。\n\n答: 王丽买书花了 30 元。\n\n【点睛】掌握 $2 、 5$ 的倍数特征是解题的关键。", "solution": "null", "level": "五年级", "question": "王丽带 40 元去买书, 她花的钱数既是 2 的倍数, 又是 5 的倍数, 而且花的钱数超过了带的钱数的一半。王丽买书花了多少钱?", "options": [], "subject": "算术", "analysis": "解30 元\n\n【分析】 2 的倍数特征:个位是 $0 、 2 、 4 、 6 、 8$ 的数。 5 的倍数特征:个位是 0 或 5 的数。 $2 、 5$ 的倍数特征: 个位是 0 的数。先求出 40 元的一半是 20 元,那么花的钱数就在 $20 \\sim 40$ 元以内; 找出 20〜 40 内的 5 的倍数, 再在 5 的倍数中找 2 的倍数, 这样就可以知道王丽买书花的钱数。\n\n【详解】 $40 \\div 2=20$ (元)\n\n大于 20 且小于 40 内 5 的倍数有: $25 、 30 、 35$;\n\n其中是 2 的倍数的是: 30 。\n\n答: 王丽买书花了 30 元。\n\n【点睛】掌握 $2 、 5$ 的倍数特征是解题的关键。"} {"id": "24237", "image": [], "answer": "解$28 、 30 、 32$\n【分析】整数中, 相邻的两个偶数相差 2 , 由此可设和为 90 的三个连续偶数中的最小的一个为 $\\mathrm{x}$, 则另两个分别为 $\\mathrm{x}+2, \\mathrm{x}+4$, 由此可列方程, 求出三个偶数。\n\n【详解】解:设最小的一个偶数为 $\\mathrm{x}$\n\n$x+x+2+x+4=90$\n\n$3 x=90-6$\n\n$3 x=84$\n\n$x=28$\n\n即三个连续偶数中, 最小的一个是 28 , 则另两个是 30,32 。\n\n答:他们的座位号各是 $28 、 30 、 32$ 。\n\n【点睛】本题考查了方程的应用, 了解整数中, 偶数的排列规律是完成本题的关键。", "solution": "null", "level": "五年级", "question": "刘明一家三口到体育场看球赛, 他们的座位号是三个连续的和为 90 的偶数, 其中有一个座位号是 5 的倍数, 他们的座位号各是多少?", "options": [], "subject": "算术", "analysis": "解$28 、 30 、 32$\n【分析】整数中, 相邻的两个偶数相差 2 , 由此可设和为 90 的三个连续偶数中的最小的一个为 $\\mathrm{x}$, 则另两个分别为 $\\mathrm{x}+2, \\mathrm{x}+4$, 由此可列方程, 求出三个偶数。\n\n【详解】解:设最小的一个偶数为 $\\mathrm{x}$\n\n$x+x+2+x+4=90$\n\n$3 x=90-6$\n\n$3 x=84$\n\n$x=28$\n\n即三个连续偶数中, 最小的一个是 28 , 则另两个是 30,32 。\n\n答:他们的座位号各是 $28 、 30 、 32$ 。\n\n【点睛】本题考查了方程的应用, 了解整数中, 偶数的排列规律是完成本题的关键。"} {"id": "24250", "image": [], "answer": "解$\\times$\n【分析】一个非 0 整数的最小因数是 1 , 最大因数是它本身, 一个非 0 整数的因数的个数是有限的。\n\n【详解】一个非 0 的整数的因数的个数是有限的。\n\n原题说法错误。\n\n故答案为: $\\times$\n\n【点睛】掌握因数的意义, 明确只在自然数(零除外)范围内研究倍数和因数。", "solution": "null", "level": "五年级", "question": "一个整数的因数的个数是有限的。()", "options": [], "subject": "算术", "analysis": "解$\\times$\n【分析】一个非 0 整数的最小因数是 1 , 最大因数是它本身, 一个非 0 整数的因数的个数是有限的。\n\n【详解】一个非 0 的整数的因数的个数是有限的。\n\n原题说法错误。\n\n故答案为: $\\times$\n\n【点睛】掌握因数的意义, 明确只在自然数(零除外)范围内研究倍数和因数。"} {"id": "24251", "image": [], "answer": "解$\\times$\n\n【分析】在整数除法中,如果商是整数而没有余数,那么被除数就是除数和商的倍数,除数和商就是被除数的因数。根据因数和倍数的意义解答即可。\n\n【详解】 5.4 是小数, 不是整数, 所以 5.4 是 3 的倍数的说法错误。\n\n故答案为: $\\times$\n\n【点睛】在研究因数和倍数时, 所说的数指的是不包括 0 的自然数。", "solution": "null", "level": "五年级", "question": "5.4 是 3 的倍数。( )", "options": [], "subject": "算术", "analysis": "解$\\times$\n\n【分析】在整数除法中,如果商是整数而没有余数,那么被除数就是除数和商的倍数,除数和商就是被除数的因数。根据因数和倍数的意义解答即可。\n\n【详解】 5.4 是小数, 不是整数, 所以 5.4 是 3 的倍数的说法错误。\n\n故答案为: $\\times$\n\n【点睛】在研究因数和倍数时, 所说的数指的是不包括 0 的自然数。"} {"id": "24252", "image": [], "answer": "解$\\times$\n\n【分析】 3 的倍数的特征: 一个数各位上的数的和是 3 的倍数, 这个数就是 3 的倍数。 3 的倍数个位上可以是 $0 \\sim 9$ 中的任意数。只看一个数的个位数字,不能判断这个数是不是 3 的倍数。\n\n【详解】根据 3 的倍数的特征可知: 个位上是 $3,6,9$ 的数不一定是 3 的倍数。比如 $13,16,19$ 等都不是 3 的倍数。所以原题说法错误。\n\n故答案为: $\\times$\n\n【点睛】明确 3 的倍数的特征是解决此题的关键。", "solution": "null", "level": "五年级", "question": "个位上是 $3 、 6 、 9$ 的数都是 3 的倍数。( )", "options": [], "subject": "算术", "analysis": "解$\\times$\n\n【分析】 3 的倍数的特征: 一个数各位上的数的和是 3 的倍数, 这个数就是 3 的倍数。 3 的倍数个位上可以是 $0 \\sim 9$ 中的任意数。只看一个数的个位数字,不能判断这个数是不是 3 的倍数。\n\n【详解】根据 3 的倍数的特征可知: 个位上是 $3,6,9$ 的数不一定是 3 的倍数。比如 $13,16,19$ 等都不是 3 的倍数。所以原题说法错误。\n\n故答案为: $\\times$\n\n【点睛】明确 3 的倍数的特征是解决此题的关键。"} {"id": "24253", "image": [], "answer": "解$\\times$\n\n【分析】一个数(0 除外)的因数除了 1 和它本身以外, 还有其它的因数, 这样的数就是合数; 一个数 (0 除外) 的因数只有 1 个和它本身两个因数, 这样的数就是质数; 一个数能被 2 整除, 这样的数就是偶数; 所以 2 的质数且是偶数, 据此判断即可。\n\n【详解】由分析可知:\n\n2 是质数且是偶数, 所以原题干说法错误。\n\n故答案为: $\\times$\n\n【点睛】本题考查质数、合数和偶数,明确它们的定义是解题的关键。", "solution": "null", "level": "五年级", "question": "2 是合数中唯一的偶数。( )", "options": [], "subject": "算术", "analysis": "解$\\times$\n\n【分析】一个数(0 除外)的因数除了 1 和它本身以外, 还有其它的因数, 这样的数就是合数; 一个数 (0 除外) 的因数只有 1 个和它本身两个因数, 这样的数就是质数; 一个数能被 2 整除, 这样的数就是偶数; 所以 2 的质数且是偶数, 据此判断即可。\n\n【详解】由分析可知:\n\n2 是质数且是偶数, 所以原题干说法错误。\n\n故答案为: $\\times$\n\n【点睛】本题考查质数、合数和偶数,明确它们的定义是解题的关键。"} {"id": "24254", "image": [], "answer": "解$\\sqrt{ }$\n\n【分析】一个数的因数和倍数的特征:\n\n一个数的倍数的个数是无限的, 最小的是它本身, 没有最大的倍数;\n\n一个数的因数的个数是有限的, 最小的因数是 1 , 最大的因数是它本身;\n\n一个数最大的因数=最小的倍数=这个数本身。据此解答。\n【详解】根据分析得, 一个非 0 自然数 $\\mathrm{b}$, 它的最大因数是 $\\mathrm{b}$, 它的最小倍数也是 $\\mathrm{b}$ 。原题说法正确。例如:\n\n13 的最大因数是 13 , 最小倍数也是 13 ;\n\n957 的最大因数是 957 , 最小倍数也是 957 。\n\n故答案为: $\\sqrt{ }$\n\n【点睛】此题应根据因数和倍数的意义进行解答。", "solution": "null", "level": "五年级", "question": "一个非 0 自然数 $b$, 它的最大因数是 $b$, 它的最小倍数也是 $b$ 。( )", "options": [], "subject": "算术", "analysis": "解$\\sqrt{ }$\n\n【分析】一个数的因数和倍数的特征:\n\n一个数的倍数的个数是无限的, 最小的是它本身, 没有最大的倍数;\n\n一个数的因数的个数是有限的, 最小的因数是 1 , 最大的因数是它本身;\n\n一个数最大的因数=最小的倍数=这个数本身。据此解答。\n【详解】根据分析得, 一个非 0 自然数 $\\mathrm{b}$, 它的最大因数是 $\\mathrm{b}$, 它的最小倍数也是 $\\mathrm{b}$ 。原题说法正确。例如:\n\n13 的最大因数是 13 , 最小倍数也是 13 ;\n\n957 的最大因数是 957 , 最小倍数也是 957 。\n\n故答案为: $\\sqrt{ }$\n\n【点睛】此题应根据因数和倍数的意义进行解答。"} {"id": "24256", "image": [], "answer": "解不对; 见详解\n\n【分析】找出 85 的因数, 判断 4 是否是 85 的因数, 如果不是, 就不能平均每人分到 4 本书正好分完。\n\n【详解】 85 的因数是: $1,5,17,85$;\n\n4 不是 85 的因数, 所以小芳的说法错误。\n\n答:小芳编的这道题不对, 因为 4 不是 85 的因数, 所以不可能平均每人分到 4 本书正好分完。\n\n【点睛】本题考查因数的实际应用, 掌握找一个数的因数的方法是解题的关键。", "solution": "null", "level": "五年级", "question": "课间时,小芳和小丽玩编题游戏。小芳指着教室后面的图书角编了一道题: “五(1)班的图书角有 85 本书,平均每人可以分到 4 本,正好分完。问:五(1)班有多少人?\"小芳编的这道题对吗?为什么?", "options": [], "subject": "算术", "analysis": "解不对; 见详解\n\n【分析】找出 85 的因数, 判断 4 是否是 85 的因数, 如果不是, 就不能平均每人分到 4 本书正好分完。\n\n【详解】 85 的因数是: $1,5,17,85$;\n\n4 不是 85 的因数, 所以小芳的说法错误。\n\n答:小芳编的这道题不对, 因为 4 不是 85 的因数, 所以不可能平均每人分到 4 本书正好分完。\n\n【点睛】本题考查因数的实际应用, 掌握找一个数的因数的方法是解题的关键。"} {"id": "24257", "image": [], "answer": "解不对\n\n【分析】不能被 2 整除的自然数叫奇数, 能被 2 整除的自然数叫偶数。 12 和 6 都是偶数, 30 元也是偶数,偶数与不是 0 的正整数相乘,都是偶数,偶数加偶数也是偶数,偶数减偶数也是偶数,所以营业员找 5 元是奇数,营业员找错了。\n\n【详解】两种钢笔的单价都是偶数, 因为偶数 $\\times$ 奇数 $=$ 偶数, 偶数 $\\times$ 偶数 $=$ 偶数, 偶数 + 偶数 $=$ 偶数,所以买钢笔一共所花的钱数一定是偶数,\n\n又因为 30 是偶数,偶数一偶数=偶数,即找回的钱一定是偶数。 5 是奇数,所以找回的钱不对。\n\n答:钱没有找对。\n\n【点睛】此题的解题关键是灵活运用奇数和偶数的运算性质求解。", "solution": "null", "level": "五年级", "question": "张明在文具店买了几支单价是 12 元和 6 元的钢笔, 付给营业员 30 元, 找回 5 元。请你判断: 钱找对了吗?", "options": [], "subject": "算术", "analysis": "解不对\n\n【分析】不能被 2 整除的自然数叫奇数, 能被 2 整除的自然数叫偶数。 12 和 6 都是偶数, 30 元也是偶数,偶数与不是 0 的正整数相乘,都是偶数,偶数加偶数也是偶数,偶数减偶数也是偶数,所以营业员找 5 元是奇数,营业员找错了。\n\n【详解】两种钢笔的单价都是偶数, 因为偶数 $\\times$ 奇数 $=$ 偶数, 偶数 $\\times$ 偶数 $=$ 偶数, 偶数 + 偶数 $=$ 偶数,所以买钢笔一共所花的钱数一定是偶数,\n\n又因为 30 是偶数,偶数一偶数=偶数,即找回的钱一定是偶数。 5 是奇数,所以找回的钱不对。\n\n答:钱没有找对。\n\n【点睛】此题的解题关键是灵活运用奇数和偶数的运算性质求解。"} {"id": "24259", "image": [], "answer": "解可能是 14 人、 21 人、 42 人\n\n【分析】根据找一个数的因数的个数的方法, 列举出 42 的因数有哪些, 进而依据题意求出可以分给小朋友的人数。\n\n【详解】 $42=1 \\times 42=2 \\times 21=3 \\times 14$\n\n42 的因数有: $1 、 2 、 3 、 14 、 21 、 42$\n\n大于 10 的有: $14 、 21 、 42$\n\n答:小朋友的人数可能是 14 人、 21 人、 42 人。\n\n【点睛】此题主要考查求一个数的因数的方法, 根据求一个数的因数的方法解决问题。", "solution": "null", "level": "五年级", "question": "幼儿园里有 10 个以上的小朋友, 王老师拿了 42 颗糖平均分给他们, 正好分完。小朋友的人数可能是多少?", "options": [], "subject": "算术", "analysis": "解可能是 14 人、 21 人、 42 人\n\n【分析】根据找一个数的因数的个数的方法, 列举出 42 的因数有哪些, 进而依据题意求出可以分给小朋友的人数。\n\n【详解】 $42=1 \\times 42=2 \\times 21=3 \\times 14$\n\n42 的因数有: $1 、 2 、 3 、 14 、 21 、 42$\n\n大于 10 的有: $14 、 21 、 42$\n\n答:小朋友的人数可能是 14 人、 21 人、 42 人。\n\n【点睛】此题主要考查求一个数的因数的方法, 根据求一个数的因数的方法解决问题。"} {"id": "24261", "image": [], "answer": "解30\n\n【分析】三个连续偶数, 它们的平均数是中间的偶数, 如果三个连续偶数的和是 84 , 用 $84 \\div 3$ 可求出中间的偶数, 然后用中间的偶数加 2 即可求出最大的偶数。\n\n【详解】中间的偶数: $84 \\div 3=28$\n\n最大的偶数: $28+2=30$\n\n答:最大的偶数是 30 。\n\n【点睛】解决此题关键是先求出中间的那个偶数的数值, 进而根据相邻两个偶数之间相差 2 得解。", "solution": "null", "level": "五年级", "question": "如果三个连续偶数的和是 84 , 最大的偶数是多少?", "options": [], "subject": "算术", "analysis": "解30\n\n【分析】三个连续偶数, 它们的平均数是中间的偶数, 如果三个连续偶数的和是 84 , 用 $84 \\div 3$ 可求出中间的偶数, 然后用中间的偶数加 2 即可求出最大的偶数。\n\n【详解】中间的偶数: $84 \\div 3=28$\n\n最大的偶数: $28+2=30$\n\n答:最大的偶数是 30 。\n\n【点睛】解决此题关键是先求出中间的那个偶数的数值, 进而根据相邻两个偶数之间相差 2 得解。"} {"id": "24538", "image": [], "answer": "解$\\sqrt{ }$\n\n【分析】容器所能容纳物体的体积叫做它们的容积, 据此判断。\n\n【详解】由分析可得:一个杯子只能装水 65 毫升, 我们说杯子的容积是 65 毫升, 原题说法正确;故答案为: $\\sqrt{ }$\n\n【点睛】掌握容积的意义是解题的关键。", "solution": "null", "level": "五年级", "question": "一个杯子只能装水 65 毫升, 我们说杯子的容积是 65 毫升。( )", "options": [], "subject": "算术", "analysis": "解$\\sqrt{ }$\n\n【分析】容器所能容纳物体的体积叫做它们的容积, 据此判断。\n\n【详解】由分析可得:一个杯子只能装水 65 毫升, 我们说杯子的容积是 65 毫升, 原题说法正确;故答案为: $\\sqrt{ }$\n\n【点睛】掌握容积的意义是解题的关键。"} {"id": "24277", "image": [], "answer": "解$\\times$\n\n【分析】质数是指除了 1 和它本身的两个因数以外再没有其他的因数。合数是指就除了 1 和它本身的两个因数以外还有其他的因数。能被 2 整除的数叫偶数。据此解答。\n\n【详解】2 是偶数, 但 2 是质数, 不是合数; 所以“所有的偶数(不包括 0 )一定是合数”这样的说法是错误的。\n\n故答案为: $\\times$\n\n【点睛】此题的解题关键是理解掌握质数、合数以及偶数的定义。", "solution": "null", "level": "五年级", "question": "所有的偶数(不包括 0 ) 一定是合数。( )", "options": [], "subject": "算术", "analysis": "解$\\times$\n\n【分析】质数是指除了 1 和它本身的两个因数以外再没有其他的因数。合数是指就除了 1 和它本身的两个因数以外还有其他的因数。能被 2 整除的数叫偶数。据此解答。\n\n【详解】2 是偶数, 但 2 是质数, 不是合数; 所以“所有的偶数(不包括 0 )一定是合数”这样的说法是错误的。\n\n故答案为: $\\times$\n\n【点睛】此题的解题关键是理解掌握质数、合数以及偶数的定义。"} {"id": "24278", "image": [], "answer": "解$\\times$\n\n【分析】根据因数和倍数的意义可知, 在乘法算式 $\\mathrm{a} \\times \\mathrm{b}=\\mathrm{c}$ ( $\\mathrm{a} 、 \\mathrm{~b} 、 \\mathrm{c}$ 均为非 0 的自然数)中, $\\mathrm{a} 、 \\mathrm{~b}$就是 $\\mathrm{c}$ 的因数, $\\mathrm{c}$ 就是 $\\mathrm{a} 、 \\mathrm{~b}$ 的倍数。据此解答。\n\n【详解】根据分析得, 虽然 $2 \\div 0.4=5$, 但我们不能说 2 是 0.4 的倍数, 0.4 是 2 的因数。因为对于因数和倍数而言, 只能适用于不为 0 的自然数。 0.4 是小数, 所以原题的说法是错误的。\n\n故答案为: $x$\n\n【点睛】此题的解题关键是充分理解因数和倍数的意义。", "solution": "null", "level": "五年级", "question": "因为 $2 \\div 0.4=5$, 所以 2 是 0.4 的倍数, 0.4 是 2 的因数。( )", "options": [], "subject": "算术", "analysis": "解$\\times$\n\n【分析】根据因数和倍数的意义可知, 在乘法算式 $\\mathrm{a} \\times \\mathrm{b}=\\mathrm{c}$ ( $\\mathrm{a} 、 \\mathrm{~b} 、 \\mathrm{c}$ 均为非 0 的自然数)中, $\\mathrm{a} 、 \\mathrm{~b}$就是 $\\mathrm{c}$ 的因数, $\\mathrm{c}$ 就是 $\\mathrm{a} 、 \\mathrm{~b}$ 的倍数。据此解答。\n\n【详解】根据分析得, 虽然 $2 \\div 0.4=5$, 但我们不能说 2 是 0.4 的倍数, 0.4 是 2 的因数。因为对于因数和倍数而言, 只能适用于不为 0 的自然数。 0.4 是小数, 所以原题的说法是错误的。\n\n故答案为: $x$\n\n【点睛】此题的解题关键是充分理解因数和倍数的意义。"} {"id": "24281", "image": [], "answer": "解$\\sqrt{ }$\n\n【分析】一个数(0 除外)的 最大因数和最小倍数都是这个数的本身。据此判断即可。\n\n【详解】由分析可知:\n\n$m$ 是非 0 自然数, $m$ 的最小倍数和最大因数都是 $m$ 。说法正确。\n\n故答案为: $\\sqrt{ }$\n\n【点睛】本题考查因数和倍数, 明确因数和倍数的定义是解题的关键。", "solution": "null", "level": "五年级", "question": "$\\mathrm{m}$ 是非 0 自然数, $\\mathrm{m}$ 的最小倍数和最大因数都是 $\\mathrm{m}$ 。( )", "options": [], "subject": "算术", "analysis": "解$\\sqrt{ }$\n\n【分析】一个数(0 除外)的 最大因数和最小倍数都是这个数的本身。据此判断即可。\n\n【详解】由分析可知:\n$\\mathrm{m}$ 是非 0 自然数, $\\mathrm{m}$ 的最小倍数和最大因数都是 $\\mathrm{m}$ 。说法正确。\n\n故答案为: $\\sqrt{ }$\n\n【点睛】本题考查因数和倍数, 明确因数和倍数的定义是解题的关键。"} {"id": "24282", "image": [], "answer": "解$\\sqrt{ }$\n\n【分析】奇数 + 奇数 $=$ 偶数, 偶数 + 偶数 $=$ 偶数, 偶数个奇数的和是偶数, 据此解题。\n\n【详解】1、23、19、33、51、77 都是奇数, 有 6 个奇数, 所以 $1+23+19+33+51+77$ 的和是偶数。故答案为: $\\sqrt{ }$\n\n【点睛】本题考查了奇数和偶数的运算性质, 掌握 “奇数 + 奇数 $=$ 偶数”是解题的关键。", "solution": "null", "level": "五年级", "question": "$1+23+19+33+51+77$ 的和是偶数。( )", "options": [], "subject": "算术", "analysis": "解$\\sqrt{ }$\n\n【分析】奇数 + 奇数 $=$ 偶数, 偶数 + 偶数 $=$ 偶数, 偶数个奇数的和是偶数, 据此解题。\n\n【详解】1、23、19、33、51、77 都是奇数, 有 6 个奇数, 所以 $1+23+19+33+51+77$ 的和是偶数。故答案为: $\\sqrt{ }$\n\n【点睛】本题考查了奇数和偶数的运算性质, 掌握“奇数十奇数=偶数”是解题的关键。"} {"id": "23994", "image": [], "answer": "解可以分 12 组, 每组 4 人; 可以分 8 组, 每组 6 人; 可以分 6 组, 每组 8 人; 可以分 4 组, 每组 12 人。\n\n【分析】根据找一个数因数的方法解答即可。\n\n【详解】 48 的因数有 $1 、 2 、 3 、 4 、 6 、 8 、 12 、 16 、 24 、 48$\n\n$48 \\div 4=12$ (组)\n\n$48 \\div 6=8$ (组)\n\n$48 \\div 8=6$ (组)\n\n$48 \\div 12=4$ (组)\n答: 可以分 12 组, 每组 4 人; 可以分 8 组, 每组 6 人; 可以分 6 组, 每组 8 人; 可以分 4 组, 每组 12 人。\n\n【点睛】本题关键在于找到 48 的因数, 然后根据题目进行取舍。", "solution": "null", "level": "五年级", "question": "学校开展体操比赛, 要把同学们分成人数相等的几个小组(每组至少 4 人, 最多 12 人)。五 (1)班有 48 人,可以怎样分组?", "options": [], "subject": "组合数学", "analysis": "解可以分 12 组, 每组 4 人; 可以分 8 组, 每组 6 人; 可以分 6 组, 每组 8 人; 可以分 4 组, 每组 12 人。\n\n【分析】根据找一个数因数的方法解答即可。\n\n【详解】 48 的因数有 $1 、 2 、 3 、 4 、 6 、 8 、 12 、 16 、 24 、 48$\n\n$48 \\div 4=12$ (组)\n\n$48 \\div 6=8$ (组)\n\n$48 \\div 8=6$ (组)\n\n$48 \\div 12=4$ (组)\n答: 可以分 12 组, 每组 4 人; 可以分 8 组, 每组 6 人; 可以分 6 组, 每组 8 人; 可以分 4 组, 每组 12 人。\n\n【点睛】本题关键在于找到 48 的因数, 然后根据题目进行取舍。"} {"id": "24014", "image": ["11897.jpg", "11898.jpg", "11898.jpg"], "answer": "解5 种\n\n\n\n\n\n5 种(11 (5)号位各一种)", "solution": "null", "level": "五年级", "question": "在下图中添上一个同样大的正方体, 使得从左面看到的图形不变。想一想, 一共有多少种不同的添法?(添上的正方体和原来的图形至少有一个面连在一起)\n\n", "options": [], "subject": "组合数学", "analysis": "解5 种\n\n\n\n\n\n5 种(11 (5)号位各一种)"} {"id": "24039", "image": [], "answer": "解选 3 块一盒规格的月饼盒正好装完. 因为 3 是 87 的因数\n\n【详解】试题分析: $2 、 3 、 5$ 中哪个是 87 的因数, 就选则哪个包装盒, 据此解答即可.\n\n解: $87 \\div 2=43.5$ (盒) ;\n\n$87 \\div 5=17.4$ (盒);\n\n$87 \\div 3=29$ (盒) ;\n\n所以选 3 块一盒规格的月饼盒正好装完.\n\n答: 选 3 块一盒规格的月饼盒正好装完. 因为 3 是 87 的因数.\n\n点评: 此题考查根据一个数的因数解决问题, 要灵活运用.", "solution": "null", "level": "五年级", "question": "月饼厂有三种包装盒, 规格分别为 3 块一盒, 5 块一盒和 2 块一盒. 现在有人订货 87 块月饼,应该选哪种规格的月饼盒正好装完? 为什么?", "options": [], "subject": "组合数学", "analysis": "解选 3 块一盒规格的月饼盒正好装完. 因为 3 是 87 的因数\n\n【详解】试题分析: $2 、 3 、 5$ 中哪个是 87 的因数, 就选则哪个包装盒, 据此解答即可.\n\n解: $87 \\div 2=43.5$ (盒) ;\n\n$87 \\div 5=17.4$ (盒);\n\n$87 \\div 3=29$ (盒) ;\n\n所以选 3 块一盒规格的月饼盒正好装完.\n\n答: 选 3 块一盒规格的月饼盒正好装完. 因为 3 是 87 的因数.\n\n点评: 此题考查根据一个数的因数解决问题, 要灵活运用."} {"id": "24040", "image": [], "answer": "解8 种\n\n【分析】求多少种不同的分法, 即求 60 的所有偶数的约数, 先根据找一个数的因数的方法进行列举,进而得出结论。\n\n【详解】 60 的约数中是偶数的有: $2 、 4 、 6 、 10 、 12 、 20 、 30 、 60$ 共 8 个数;\n\n所以可分成 $2 、 4 、 6 、 10 、 12 、 20 、 30 、 60$ 堆, 即有 8 种分法;\n\n答: 有 8 种不同的分法。\n\n【点睛】此题考查了找一个数因数的方法, 用到的知识点: 偶数的含义: 自然数中, 是 2 的倍数的数叫做偶数。", "solution": "null", "level": "五年级", "question": "把 60 个桃子分成偶数堆, 使得每堆的个数相等, 有多少种不同的分法?", "options": [], "subject": "组合数学", "analysis": "解8 种\n\n【分析】求多少种不同的分法, 即求 60 的所有偶数的约数, 先根据找一个数的因数的方法进行列举,进而得出结论。\n\n【详解】 60 的约数中是偶数的有: $2 、 4 、 6 、 10 、 12 、 20 、 30 、 60$ 共 8 个数;\n\n所以可分成 $2 、 4 、 6 、 10 、 12 、 20 、 30 、 60$ 堆, 即有 8 种分法;\n\n答: 有 8 种不同的分法。\n\n【点睛】此题考查了找一个数因数的方法, 用到的知识点: 偶数的含义: 自然数中, 是 2 的倍数的数叫做偶数。"} {"id": "24065", "image": [], "answer": "解(1)30 个数字中, 偶数有:\n\n$30 \\div 2=15$ (人)\n\n答: 参加跑步的有 15 人.\n\n(2) 30 以内 6 的倍数有: $6,12,18,24,30$,\n\n$30 \\div 3=10$ (个)\n\n$10-5=5$ (人)\n\n答: 参加跳绳的有 5 人.", "solution": "null", "level": "五年级", "question": "(2022 春・端州区月考) 体育课上, 30 名学生站成一行, 按老师口令从左到右报数: 1, 2, 3, 4, , , 30 .\n\n(1)老师先让所报的数是 2 的倍数的同学去跑步,参加跑步的有多少人?\n\n(2)余下学生中所报的数是 3 的倍数的同学进行跳绳训练, 参加跳绳的有多少人?", "options": [], "subject": "组合数学", "analysis": "解(1)30 个数字中, 偶数有:\n\n$30 \\div 2=15$ (人)\n\n答: 参加跑步的有 15 人.\n\n(2) 30 以内 6 的倍数有: $6,12,18,24,30$,\n\n$30 \\div 3=10$ (个)\n\n$10-5=5$ (人)\n\n答: 参加跳绳的有 5 人."} {"id": "24104", "image": ["12098.jpg", "12099.jpg", "12100.jpg"], "answer": "解16 块; 13 块\n\n【分析】由从正面看到的图形可得几何体底层有 2 列 4 层正方体, 由从侧面看到的图形可得几何体底层有 3 行正方体, 所以最多有 $(4+3 \\times 4)$ 个, 最少有 $(4+2 \\times 4+1)$, 据此解答。\n\n【详解】最多: $4+3 \\times 4$\n\n$=4+12$\n\n$=16$ (块)\n\n最少: $4+2 \\times 4+1$\n\n$=4+8+1$\n\n$=13$ (块)\n\n答:这堆木块最多有 16 块,最少有 13 块。\n\n【点睛】本题主要考查三视图, 正方体最多的个数为行数 $\\times$ 列数, 最少个数保证每行或每列有一个正方体即可。", "solution": "null", "level": "五年级", "question": "(2022 ・淮安区)如图:有一些大小相同的正方体木块堆成一堆,从上往下看是图(1),从前往后看是图(2),从左往右看是图(3),那么这堆木块最多有多少块?最少有多少块?\n\n\n\n图 (1)\n\n\n\n图 (2)\n\n\n\n图 (3)", "options": [], "subject": "组合数学", "analysis": "解16 块; 13 块\n\n【分析】由从正面看到的图形可得几何体底层有 2 列 4 层正方体, 由从侧面看到的图形可得几何体底层有 3 行正方体, 所以最多有 $(4+3 \\times 4)$ 个, 最少有 $(4+2 \\times 4+1)$, 据此解答。\n\n【详解】最多: $4+3 \\times 4$\n\n$=4+12$\n\n$=16$ (块)\n\n最少: $4+2 \\times 4+1$\n\n$=4+8+1$\n\n$=13$ (块)\n\n答:这堆木块最多有 16 块,最少有 13 块。\n\n【点睛】本题主要考查三视图, 正方体最多的个数为行数 $\\times$ 列数, 最少个数保证每行或每列有一个正方体即可。"} {"id": "24182", "image": [], "answer": "解48 人\n\n【分析】每组 6 人或者每组 8 人, 两种分法都刚好分完, 先找出 6 和 8 的最小公倍数, 然后根据最小公倍数的倍数找出符合题意的人数即可。\n\n【详解】 6 和 8 的最小公倍数是 24 。\n\n$24 \\times 2=48$ (人)\n\n这个班的学生最多可能有 48 人。\n\n【点睛】求得 6 和 8 的最小公倍数, 再根据题目要求找出符合已知条件的数值是解答本题的关键。", "solution": "null", "level": "五年级", "question": "五(1)班学生人数不超过 50 人,在分小组做游戏时,可以分为每组 6 人或者每组 8 人,两种分法都刚好分完。这个班的学生最多可能有多少人?", "options": [], "subject": "组合数学", "analysis": "解48 人\n\n【分析】每组 6 人或者每组 8 人, 两种分法都刚好分完, 先找出 6 和 8 的最小公倍数, 然后根据最小公倍数的倍数找出符合题意的人数即可。\n\n【详解】 6 和 8 的最小公倍数是 24 。\n\n$24 \\times 2=48$ (人)\n\n这个班的学生最多可能有 48 人。\n\n【点睛】求得 6 和 8 的最小公倍数, 再根据题目要求找出符合已知条件的数值是解答本题的关键。"} {"id": "24183", "image": [], "answer": "解5 瓶装\n\n【分析】只要包装瓶数是 460 的因数即可正好包装完, 据此用饮料瓶数分别包装瓶数, 能整除的即可。\n\n【详解】 $460 \\div 3,4+6+0=10,10$ 不是 3 的倍数, 所以 $460 \\div 3$ 不能整除, 3 瓶装就不能打包完; $460 \\div 5,460$ 个位上是 0 , 说明 460 是 5 的倍数, 所以 $460 \\div 5$ 能整除, 5 瓶装就正好能打包完; $460 \\div 8=57 \\ldots . .4$, 所以 8 瓶装就不能打包完。\n\n答: 5 瓶装正好能打包完。\n\n【点睛】此题的解题关键是利用 $3 、 5$ 的倍数的数的特征, 通过整数的含义, 找到正确的答案。", "solution": "null", "level": "五年级", "question": "饮料厂要把 460 瓶饮料打包, 下面哪种方式正好能打包完?\n\n3 瓶装 5 瓶装 8 瓶装", "options": [], "subject": "组合数学", "analysis": "解5 瓶装\n\n【分析】只要包装瓶数是 460 的因数即可正好包装完, 据此用饮料瓶数分别包装瓶数, 能整除的即可。\n\n【详解】 $460 \\div 3,4+6+0=10,10$ 不是 3 的倍数, 所以 $460 \\div 3$ 不能整除, 3 瓶装就不能打包完; $460 \\div 5,460$ 个位上是 0 , 说明 460 是 5 的倍数, 所以 $460 \\div 5$ 能整除, 5 瓶装就正好能打包完; $460 \\div 8=57 \\ldots . .4$, 所以 8 瓶装就不能打包完。\n\n答: 5 瓶装正好能打包完。\n\n【点睛】此题的解题关键是利用 $3 、 5$ 的倍数的数的特征, 通过整数的含义, 找到正确的答案。"} {"id": "24186", "image": ["12450.jpg"], "answer": "解$28 、 30 、 32$\n\n【分析】相邻两个偶数相差 2 , 把最小的座位号设为未知数, 表示出其它两个座位号, 再根据三个偶数的和为 90 列方程解答。\n\n【详解】解:设最小的座位号为 $\\mathrm{x}$, 中间的座位号为 $(\\mathrm{x}+2)$, 最大的座位号为 $(\\mathrm{x}+4)$ 。\n\n$x+(x+2)+(x+4)=90$\n\n$3 \\mathrm{x}+6=90$\n\n$3 \\mathrm{x}=90-6$\n\n$3 x=84$\n\n$\\mathrm{x}=84 \\div 3$\n\n$x=28$\n\n$28+2=30$\n\n$28+4=32$\n\n$30 \\div 5=6$, 则 30 是 5 的倍数。\n\n答: 他们的座位号分别是 $28 、 30 、 32$ 。\n\n【点睛】根据相邻偶数的差表示出其它两个偶数是解答题目的关键。", "solution": "null", "level": "五年级", "question": "寒假期间, 龙龙、北北、鸣鸣到电影院去看电影, 根据三人的对话, 你能判断他们的座位号是多少吗?\n", "options": [], "subject": "组合数学", "analysis": "解$28 、 30 、 32$\n\n【分析】相邻两个偶数相差 2 , 把最小的座位号设为未知数, 表示出其它两个座位号, 再根据三个偶数的和为 90 列方程解答。\n\n【详解】解:设最小的座位号为 $\\mathrm{x}$, 中间的座位号为 $(\\mathrm{x}+2)$, 最大的座位号为 $(\\mathrm{x}+4)$ 。\n\n$x+(x+2)+(x+4)=90$\n\n$3 \\mathrm{x}+6=90$\n\n$3 \\mathrm{x}=90-6$\n\n$3 x=84$\n\n$\\mathrm{x}=84 \\div 3$\n\n$x=28$\n\n$28+2=30$\n\n$28+4=32$\n\n$30 \\div 5=6$, 则 30 是 5 的倍数。\n\n答: 他们的座位号分别是 $28 、 30 、 32$ 。\n\n【点睛】根据相邻偶数的差表示出其它两个偶数是解答题目的关键。"} {"id": "24208", "image": [], "answer": "解 $42=1 \\times 42,42=2 \\times 21,42=3 \\times 14,42=6 \\times 7$;\n\n由于组数是大于 1 的数,\n\n所以分成 2 组, 每组 21 人, 不符合题意;\n\n分成 21 组, 每组 2 人, 符合题意;\n\n分成 3 组, 每组 14 人, 符合题意;\n\n分成 14 组, 每组 3 人,不符合题意;\n\n分成 6 组, 每组 7 人, 不符合题意;\n\n分成 7 组, 每组 6 人, 符合题意。\n\n答: (1)分成 21 组, 每组 2 人; (2)分成 3 组, 每组 14 人; (3)分成 7 组, 每组 6 人。", "solution": "null", "level": "五年级", "question": "42 名同学去参观水立方, 老师要把同学们平均分成若干小组, 而且每组人数都是偶数, 可以分成几组, 每组几人?(组数大于 1)(写出思考过程)", "options": [], "subject": "组合数学", "analysis": "解 $42=1 \\times 42,42=2 \\times 21,42=3 \\times 14,42=6 \\times 7$;\n\n由于组数是大于 1 的数,\n\n所以分成 2 组, 每组 21 人, 不符合题意;\n\n分成 21 组, 每组 2 人, 符合题意;\n\n分成 3 组, 每组 14 人, 符合题意;\n\n分成 14 组, 每组 3 人,不符合题意;\n\n分成 6 组, 每组 7 人, 不符合题意;\n\n分成 7 组, 每组 6 人, 符合题意。\n\n答: (1)分成 21 组, 每组 2 人; (2)分成 3 组, 每组 14 人; (3)分成 7 组, 每组 6 人。"} {"id": "24260", "image": [], "answer": "解奇数\n\n【分析】由奇数、偶数的运算性质可知, 奇数 + 奇数 $=$ 偶数, 偶数 + 偶数 $=$ 偶数, 奇数 + 偶数 $=$ 奇数,这 7 个奇数两两结合直到最后求出结果为奇数, 据此解答。\n\n【详解】奇数 + 奇数 + 奇数 + 奇数 + 奇数 + 奇数 + 奇数\n\n$=$ (奇数 + 奇数 $)+$ (奇数 + 奇数 $)+$ (奇数 + 奇数 $)+$ 奇数\n\n$=$ 偶数 + 偶数 + 偶数 + 奇数\n\n$=$ (偶数 + 偶数 $)+$ (偶数 + 奇数 $)$\n\n$=$ 偶数十奇数\n\n$=$ 奇数\n\n答:参加学校活动小组的总人数是奇数。\n\n【点睛】掌握奇数、偶数的运算性质是解答题目的关键。", "solution": "null", "level": "五年级", "question": "同学们在第二课堂分别参加了学校 7 个不同的活动小组, 每个活动小组都有奇数个同学, 参加学校活动小组的总人数是奇数还是偶数?", "options": [], "subject": "组合数学", "analysis": "解奇数\n\n【分析】由奇数、偶数的运算性质可知, 奇数 + 奇数 $=$ 偶数, 偶数 + 偶数 $=$ 偶数, 奇数 + 偶数 $=$ 奇数,这 7 个奇数两两结合直到最后求出结果为奇数, 据此解答。\n\n【详解】奇数 + 奇数 + 奇数 + 奇数 + 奇数 + 奇数 + 奇数\n\n$=$ (奇数 + 奇数 $)+$ (奇数 + 奇数 $)+$ (奇数 + 奇数 $)+$ 奇数\n\n$=$ 偶数 + 偶数 + 偶数 + 奇数\n\n$=$ (偶数 + 偶数 $)+$ (偶数 + 奇数 $)$\n\n$=$ 偶数十奇数\n\n$=$ 奇数\n\n答:参加学校活动小组的总人数是奇数。\n\n【点睛】掌握奇数、偶数的运算性质是解答题目的关键。"} {"id": "24279", "image": [], "answer": "解$\\times$\n\n【分析】 3 的倍数特征: 各个数位上的数字相加, 和要能被 3 整除; 据此解答。\n\n【详解】 $2+5+8=15$\n\n15 是 3 的倍数, 所以用 $2 、 5 、 8$ 这三个数字组成的三位数一定是 3 的倍数。原题说法错误。\n\n故答案为: $\\times$\n\n【点睛】掌握 3 的倍数特征是解题的关键。", "solution": "null", "level": "五年级", "question": "用 $2 、 5 、 8$ 这三个数字组成的三位数不一定是 3 的倍数。( )", "options": [], "subject": "组合数学", "analysis": "解$\\times$\n\n【分析】 3 的倍数特征: 各个数位上的数字相加, 和要能被 3 整除; 据此解答。\n\n【详解】 $2+5+8=15$\n\n15 是 3 的倍数, 所以用 $2 、 5 、 8$ 这三个数字组成的三位数一定是 3 的倍数。原题说法错误。\n\n故答案为: $\\times$\n\n【点睛】掌握 3 的倍数特征是解题的关键。"} {"id": "24008", "image": ["11873.jpg", "11874.jpg", "11873.jpg", "11874.jpg"], "answer": "解$\\times$\n\n【分析】若两个立体图形从前面、左面和上面看到的图形相同, 则这两个立体图形形状可能相同, 单从前面、左面看到的形状相同, 这两个立体图形形状不一定相同。\n\n【详解】如图所示:\n\n\n左、右两个立体图形从前面、左面看到的形状相同都是一个“田”字型, 如图所示:\n\n\n\n而左、右两个立体图形的形状却不同,所以原题的说法错误。\n\n故答案为: $\\times$\n【点睛】要确定一个物体的形状, 要通过这个物体的三视图才能确定, 而二视图、一视图都不能确定其形状。", "solution": "null", "level": "五年级", "question": "如果两个立体图形从前面和左面看到的图形相同, 那么这两个立体图形一定相同。( )", "options": [], "subject": "立体几何学", "analysis": "解$\\times$\n\n【分析】若两个立体图形从前面、左面和上面看到的图形相同, 则这两个立体图形形状可能相同, 单从前面、左面看到的形状相同, 这两个立体图形形状不一定相同。\n\n【详解】如图所示:\n\n\n左、右两个立体图形从前面、左面看到的形状相同都是一个“田”字型, 如图所示:\n\n\n\n而左、右两个立体图形的形状却不同,所以原题的说法错误。\n\n故答案为: $\\times$\n【点睛】要确定一个物体的形状, 要通过这个物体的三视图才能确定, 而二视图、一视图都不能确定其形状。"} {"id": "24009", "image": ["11875.jpg", "11876.jpg", "11877.jpg", "11878.jpg", "11879.jpg", "11877.jpg", "11878.jpg", "11879.jpg"], "answer": "解$\\sqrt{ }$\n\n【分析】分别将从正面、上面和右面看到的形状画出来, 再进行判断即可。\n\n【详解】从正面看:\n\n\n\n从上面看:\n\n\n\n从右面看:\n\n\n\n所以从正面、上面和右面看到的形状完全不同,原题说法正确;\n\n故答案为: $\\sqrt{ }$ 。\n\n【点睛】本题较易, 考查了观察物体的知识点。", "solution": "null", "level": "五年级", "question": "\n\n\n从正面、上面和右面看到的形状完全不同。\n", "options": [], "subject": "立体几何学", "analysis": "解$\\sqrt{ }$\n\n【分析】分别将从正面、上面和右面看到的形状画出来, 再进行判断即可。\n\n【详解】从正面看:\n\n\n\n从上面看:\n\n\n\n从右面看:\n\n\n\n所以从正面、上面和右面看到的形状完全不同,原题说法正确;\n\n故答案为: $\\sqrt{ }$ 。\n\n【点睛】本题较易, 考查了观察物体的知识点。"} {"id": "24010", "image": ["11880.jpg", "11881.jpg"], "answer": "解5 个, 7 个", "solution": "null", "level": "五年级", "question": "佳佳在桌子上用小正方体摆了一个几何体, 从上面看到的图形是\n\n\n\n,从左面看到\n\n", "options": [], "subject": "立体几何学", "analysis": "解5 个, 7 个"} {"id": "24011", "image": ["11882.jpg"], "answer": "解$11 \\mathrm{~cm}^{2} 3$ 号\n\n【详解】 $1 \\times 1 \\times(6+5)=11\\left(\\mathrm{~cm}^{2}\\right)$\n\n取走 3 号小正方体后,从上面和左面看到的形状不变。", "solution": "null", "level": "五年级", "question": "把 9 个棱长是 $1 \\mathrm{~cm}$ 的小正方体拼在一起(如下图),从正面看和从左面看,所看到的图形的面积之和是多少?取走几号小正方体后,从上面和左面看到的形状不变?\n\n", "options": [], "subject": "立体几何学", "analysis": "解$11 \\mathrm{~cm}^{2} 3$ 号\n\n【详解】 $1 \\times 1 \\times(6+5)=11\\left(\\mathrm{~cm}^{2}\\right)$\n\n取走 3 号小正方体后,从上面和左面看到的形状不变。"} {"id": "24013", "image": ["11895.jpg", "11896.jpg"], "answer": "解移动(1)号小正方体,摆在(3)号小正方体前面。", "solution": "null", "level": "五年级", "question": "一个几何体如下图所示,移动其中的一个小正方体,使移动后得到的几何体从上面看到的图形是\n\n\n\n,想一想该怎样移动。(摆放时每相邻两个小正方体有一个面重合)\n\n", "options": [], "subject": "立体几何学", "analysis": "解移动(1)号小正方体,摆在(3)号小正方体前面。"} {"id": "24015", "image": ["11899.jpg", "11900.jpg"], "answer": "解(1) 第 7 个正方体积木可以放在 2 号或 5 号位置。\n\n(2) 如果第 7 个正方体积木放在 2 号位置, 那么第 8 个正方体积木应放在 $4 、 5$ 或 6 号位置; 如果第 7 个正方体积木放在 5 号位置, 那么第 8 个正方体积木应放在 $1 、 2$ 或 3 号位置。\n\n【分析】(1)如果用 7 个正方体积木搭这个几何体,第 7 个正方体积木可以放在下层前排的中间或后排的中间位置。\n\n(2)根据(1)的结果, 如果第 7 个正方体放在 2 的位置, 则第 8 个正方体可以放在 $4 、 5 、 6$ 的任意一个的上面; 如果第 7 个正方体放在 5 的位置, 则第 8 个正方体可以放在 $1,2,3$ 的任意一个的上面。\n\n【详解】(1)第 7 个正方体积木可以放在 2 号或 5 号位置。\n\n(2) 如果第 7 个正方体积木放在 2 号位置, 那么第 8 个正方体积木应放在 $4 、 5$ 或 6 号位置; 如果第 7 个正方体积木放在 5 号位置, 那么第 8 个正方体积木应放在 $1 、 2$ 或 3 号位置。\n\n【点睛】本题考查从不同方向观察物体和几何图形, 是培养学生的观察能力。", "solution": "null", "level": "五年级", "question": "一个由正方体积木搭成的几何体, 从不同方向看到的图形如下。\n\n\n(1)如果用 7 个正方体积木搭这个几何体,第 7 个正方体积木可以放在哪个位置?(图中的序号是位置号)\n\n\n\n(2)如果再增加 1 个正方体积木,使从上面看到的图形不变,从左面看到的图形是\n\n| | |\n| :--- | :--- |\n| | |\n\n那么第 8 个正方体积木可以放在哪个位置?(图中的序号是位置号)\n\n| 1 | 2 | 3 |\n| :--- | :--- | :--- |\n| 4 | 5 | 6 |", "options": [], "subject": "立体几何学", "analysis": "解(1) 第 7 个正方体积木可以放在 2 号或 5 号位置。\n\n(2) 如果第 7 个正方体积木放在 2 号位置, 那么第 8 个正方体积木应放在 $4 、 5$ 或 6 号位置; 如果第 7 个正方体积木放在 5 号位置, 那么第 8 个正方体积木应放在 $1 、 2$ 或 3 号位置。\n\n【分析】(1)如果用 7 个正方体积木搭这个几何体,第 7 个正方体积木可以放在下层前排的中间或后排的中间位置。\n\n(2)根据(1)的结果, 如果第 7 个正方体放在 2 的位置, 则第 8 个正方体可以放在 $4 、 5 、 6$ 的任意一个的上面; 如果第 7 个正方体放在 5 的位置, 则第 8 个正方体可以放在 $1,2,3$ 的任意一个的上面。\n\n【详解】(1)第 7 个正方体积木可以放在 2 号或 5 号位置。\n\n(2) 如果第 7 个正方体积木放在 2 号位置, 那么第 8 个正方体积木应放在 $4 、 5$ 或 6 号位置; 如果第 7 个正方体积木放在 5 号位置, 那么第 8 个正方体积木应放在 $1 、 2$ 或 3 号位置。\n\n【点睛】本题考查从不同方向观察物体和几何图形, 是培养学生的观察能力。"} {"id": "24432", "image": [], "answer": "解一个正方体的棱长扩大到原来的 4 倍, 则表面积扩大 $4 \\times 4=16$ 倍, 体积扩大 $4 \\times 4$ $\\times 4=64$ 倍。\n\n故答案为: $\\times$ 。", "solution": "null", "level": "五年级", "question": "(2023 - 梁子湖区模拟)正方体的棱长扩大到原来的 4 倍, 表面积和体积就扩大到原来的 9 倍。", "options": [], "subject": "立体几何学", "analysis": "解一个正方体的棱长扩大到原来的 4 倍, 则表面积扩大 $4 \\times 4=16$ 倍, 体积扩大 $4 \\times 4$ $\\times 4=64$ 倍。\n\n故答案为: $\\times$ 。"} {"id": "24433", "image": [], "answer": "解设长方体的长为 3 分米、宽为 2 分米、高为 1 分米, 这时长方体棱长总和为 24 分米,体积为 $3 \\times 2 \\times 1=6$ (立方分米)。\n\n正方体棱长为 $24 \\div 12=2$ (分米), 体积为 $2 \\times 2 \\times 2=8$ (立方分米)。\n\n因为 8 立方分米 $>6$ 立方分米, 故棱长总和相等的长方体和正方体, 正方体的体积大。故答案为: $\\sqrt{ }$ 。", "solution": "null", "level": "五年级", "question": "(2022 春 ・㵛阳期末)棱长总和相等的正方体和长方体, 正方体的体积一定比长方体的体积大。", "options": [], "subject": "立体几何学", "analysis": "解设长方体的长为 3 分米、宽为 2 分米、高为 1 分米, 这时长方体棱长总和为 24 分米,体积为 $3 \\times 2 \\times 1=6$ (立方分米)。\n\n正方体棱长为 $24 \\div 12=2$ (分米), 体积为 $2 \\times 2 \\times 2=8$ (立方分米)。\n\n因为 8 立方分米 $>6$ 立方分米, 故棱长总和相等的长方体和正方体, 正方体的体积大。故答案为: $\\sqrt{ }$ 。"} {"id": "24434", "image": [], "answer": "解 $24 \\div 12=2$ (厘米)\n\n$2 \\times 2 \\times 2$\n\n$=4 \\times 2$\n\n$=8$ (立方厘米)\n\n所以这个正方体的体积是 8 立方厘米是正确的。\n\n故答案为: $\\sqrt{ }$ 。", "solution": "null", "level": "五年级", "question": "(2022・梁子湖区)一个正方体的棱长和是 24 厘米,那么这个正方体的体积是 8 立方厘米。", "options": [], "subject": "立体几何学", "analysis": "解 $24 \\div 12=2$ (厘米)\n\n$2 \\times 2 \\times 2$\n\n$=4 \\times 2$\n\n$=8$ (立方厘米)\n\n所以这个正方体的体积是 8 立方厘米是正确的。\n\n故答案为: $\\sqrt{ }$ 。"} {"id": "24436", "image": [], "answer": "解$1 \\times 1 \\times 5=5$ (平方米)\n\n所以, 做一个棱长为 1 米的无盖正方体铁箱, 至少需要铁皮 5 平方米。\n\n因此, 题干中的计算是错误的。\n\n故答案为: $\\times$ 。", "solution": "null", "level": "五年级", "question": "(2022 春・灵宝市期中)做一个棱长为 $1 m$ 的无盖正方体铁箱, 至少需要铁皮 $6 m^{2}$ 。", "options": [], "subject": "立体几何学", "analysis": "解$1 \\times 1 \\times 5=5$ (平方米)\n\n所以, 做一个棱长为 1 米的无盖正方体铁箱, 至少需要铁皮 5 平方米。\n\n因此, 题干中的计算是错误的。\n\n故答案为: $\\times$ 。"} {"id": "24437", "image": [], "answer": "解容积和体积的计算方法虽然相同,但物体的体积和容积的意义不同,物体的体积要大于容积,所以原题说法错误。\n\n故答案为: $\\times$ 。", "solution": "null", "level": "五年级", "question": "(2022 春・巧家县期中)容器的容积计算方法与体积计算方法相同, 容器的容积等于它的体积。", "options": [], "subject": "立体几何学", "analysis": "解容积和体积的计算方法虽然相同,但物体的体积和容积的意义不同,物体的体积要大于容积,所以原题说法错误。\n\n故答案为: $\\times$ 。"} {"id": "24441", "image": [], "answer": "解$0.065 \\times 6=0.39$ (立方米)\n\n答: 这根木料的体积是 0.39 立方米。", "solution": "null", "level": "五年级", "question": "(2022 春・汉寿县期中)一根长方体木料, 长 $6 m$, 横截面的面积是 $0.065 m^{2}$ 。这根木料的体积是多少?", "options": [], "subject": "立体几何学", "analysis": "解$0.065 \\times 6=0.39$ (立方米)\n\n答: 这根木料的体积是 0.39 立方米。"} {"id": "24442", "image": [], "answer": "解 $24 \\times 18 \\times(12-8)$\n\n$$\n\\begin{aligned}\n& =432 \\times 4 \\\\\n& =1728 \\text { (立方厘米) }\n\\end{aligned}\n$$\n\n答: 这个铁块的体积是 1728 立方厘米。", "solution": "null", "level": "五年级", "question": "(2022 春・沈丘县期末)一个长 $24 \\mathrm{~cm}$, 宽 $18 \\mathrm{~cm}$ 的长方体水槽中水深 $8 \\mathrm{~cm}$, 放入一个正方体铁块后,水深 $12 \\mathrm{~cm}$, 这个铁块的体积是多少?", "options": [], "subject": "立体几何学", "analysis": "解 $24 \\times 18 \\times(12-8)$\n\n$$\n\\begin{aligned}\n& =432 \\times 4 \\\\\n& =1728 \\text { (立方厘米) }\n\\end{aligned}\n$$\n\n答: 这个铁块的体积是 1728 立方厘米。"} {"id": "24081", "image": ["11978.jpg", "11979.jpg", "11980.jpg", "11981.jpg", "11982.jpg", "11983.jpg", "11980.jpg", "11981.jpg", "11982.jpg", "11983.jpg"], "answer": "解\n\n\n\n从左面看\n\n【分析】根据题目信息,可知这个几何体的形状是:\n\n\n\n据此即可知道该立方体从正面和左面看到的平面图形。\n\n\n\n【详解】\n\n\n\n从左面看\n\n【点睛】本题考查观察物体的知识, 需要根据平面图形发挥想象, 能够还原出立体图形, 考查学生的立体想象能力。", "solution": "null", "level": "五年级", "question": "如图是由几个同样的小正方体所组成的几何体从上面看到的图形, 小正方形种的数字表示在该位置的小正方体的个数, 在下列方格图中画出从正面和左面看到的图形。\n\n| 2 | 3 | |\n| :--- | :--- | :--- |\n| 1 | | 2 |\n\n\n\n从正面看\n\n\n\n从左面看", "options": [], "subject": "立体几何学", "analysis": "解\n\n\n\n从左面看\n\n【分析】根据题目信息,可知这个几何体的形状是:\n\n\n\n据此即可知道该立方体从正面和左面看到的平面图形。\n\n\n\n【详解】\n\n\n\n从左面看\n\n【点睛】本题考查观察物体的知识, 需要根据平面图形发挥想象, 能够还原出立体图形, 考查学生的立体想象能力。"} {"id": "24082", "image": ["11984.jpg", "11985.jpg", "11986.jpg", "11986.jpg"], "answer": "解\n \n\n(答案不唯一)", "solution": "null", "level": "五年级", "question": "由 5 个小正方体搭成的几何体,从正面看到的是图形(1),从左面看到的是图形(2),你知道从上面看到的是什么图形吗?(画出两种)\n\n\n\n(1)\n\n\n\n(2)", "options": [], "subject": "立体几何学", "analysis": "解\n \n\n(答案不唯一)"} {"id": "24083", "image": ["11987.jpg", "11987.jpg"], "answer": "解见详解\n\n【分析】此题答案不唯一, 只要把 5 个小正方体摆成一层, 且保证是三行三列即可符合题意, 据此只要画出符合题意的一种情况即可解答问题。\n\n【详解】搭法如下:\n\n\n\n(答案不唯一)\n\n【点睛】此题考查了从不同方向观察物体和几何体, 锻炼了学生的空间想象力和抽象思维能力。", "solution": "null", "level": "五年级", "question": "从正面和侧面看到的都是 $\\square$, 用 5 个正方体可以怎样搭?", "options": [], "subject": "立体几何学", "analysis": "解见详解\n\n【分析】此题答案不唯一, 只要把 5 个小正方体摆成一层, 且保证是三行三列即可符合题意, 据此只要画出符合题意的一种情况即可解答问题。\n\n【详解】搭法如下:\n\n\n\n(答案不唯一)\n\n【点睛】此题考查了从不同方向观察物体和几何体, 锻炼了学生的空间想象力和抽象思维能力。"} {"id": "24097", "image": ["12067.jpg", "12068.jpg", "12069.jpg", "12070.jpg", "12071.jpg", "12072.jpg", "12069.jpg", "12070.jpg", "12071.jpg", "12072.jpg"], "answer": "解$\\sqrt{ }$\n\n【分析】\n\n\n\n从正面看为\n\n\n\n,从上面看为\n\n\n\n, 据此解答即可。\n\n\n\n故答案为: $\\sqrt{ }$ 。\n\n【点睛】本题考查了空间思维能力, 观察哪个方位的平面图形就假设自己站在什么位置。", "solution": "null", "level": "五年级", "question": "(2022・南京)几何体\n\n\n从正面和上面看到的都是形状\n\n", "options": [], "subject": "立体几何学", "analysis": "解$\\sqrt{ }$\n\n【分析】\n\n\n\n从正面看为\n\n\n\n,从上面看为\n\n\n\n, 据此解答即可。\n\n\n\n故答案为: $\\sqrt{ }$ 。\n\n【点睛】本题考查了空间思维能力, 观察哪个方位的平面图形就假设自己站在什么位置。"} {"id": "24098", "image": ["12073.jpg", "12074.jpg", "12073.jpg", "12074.jpg"], "answer": "解$\\times$\n\n【分析】从不同角度方向观察物体,常常得到不同的结果,但也可能看到相同的形状,举例说明即可。\n\n【详解】如图\n\n\n,从正面和左面看到的形状都是\n\n\n\n所以原题说法错误。\n\n【点睛】关键是具有一定的空间想象能力,可以画一画示意图。", "solution": "null", "level": "五年级", "question": "2022・淮安区)同一个几何体从不同的方向看到的图形不可能相同。()", "options": [], "subject": "立体几何学", "analysis": "解$\\times$\n\n【分析】从不同角度方向观察物体,常常得到不同的结果,但也可能看到相同的形状,举例说明即可。\n\n【详解】如图\n\n\n,从正面和左面看到的形状都是\n\n\n\n所以原题说法错误。\n\n【点睛】关键是具有一定的空间想象能力,可以画一画示意图。"} {"id": "24099", "image": ["12075.jpg", "12076.jpg", "12077.jpg", "12076.jpg", "12077.jpg"], "answer": "解$\\sqrt{ }$\n\n【分析】观察图形可知, 从左面看到的图形是有 2 列, 左列有 4 个小正方形, 右列有 1 个小正方形,由此判断。\n\n【详解】根据分析可知, 观察立体图形\n\n\n\n, 从左面看到的图形是\n\n\n原题干正确。故答案为: $\\sqrt{ }$\n\n【点睛】本题考查从不同方向观察物体, 根据立体图形确定三视图。", "solution": "null", "level": "五年级", "question": "(2022・海门市)观察下面的立体图形,从左面看到的图形是\n", "options": [], "subject": "立体几何学", "analysis": "解$\\sqrt{ }$\n\n【分析】观察图形可知, 从左面看到的图形是有 2 列, 左列有 4 个小正方形, 右列有 1 个小正方形,由此判断。\n\n【详解】根据分析可知, 观察立体图形\n\n\n\n, 从左面看到的图形是\n\n\n原题干正确。故答案为: $\\sqrt{ }$\n\n【点睛】本题考查从不同方向观察物体, 根据立体图形确定三视图。"} {"id": "24100", "image": ["12078.jpg", "12079.jpg", "12080.jpg", "12081.jpg", "12080.jpg", "12081.jpg"], "answer": "解$\\times$\n\n【分析】从一个方向观察由小正方体拼成的立体图形, 无法确定小正方体的个数, 举例说明即可。\n\n【详解】从正面看到是\n\n\n的图形,如图\n\n\n,是由 4 个小正方体拼成的,所以原题说法错误。故答案为: $\\times$\n\n【点睛】画一画示意图是解决本题比较好的方法,或者具有一定的空间想象能力。", "solution": "null", "level": "五年级", "question": "(2022・南京)从正面看到是\n\n\n的图形,一定是由 3 个小正方体拼成的。\n\n", "options": [], "subject": "立体几何学", "analysis": "解$\\times$\n\n【分析】从一个方向观察由小正方体拼成的立体图形, 无法确定小正方体的个数, 举例说明即可。\n\n【详解】从正面看到是\n\n\n的图形,如图\n\n\n,是由 4 个小正方体拼成的,所以原题说法错误。故答案为: $\\times$\n\n【点睛】画一画示意图是解决本题比较好的方法,或者具有一定的空间想象能力。"} {"id": "24101", "image": ["12082.jpg", "12083.jpg", "12082.jpg", "12083.jpg"], "answer": "解$\\times$\n\n【分析】最少需要 3 个小正方体:把 3 个小正方体摆成一行,把左边一个向后平移一行,把右边一个向前平移一行, 即可满足条件, 据此即可解答问题。\n\n【详解】从正面、左面看到的形状都是\n\n\n的图形, 小正方体最少是 3 个, 如图:\n\n\n\n故答案为: $x$ 。\n\n【点睛】本题考查了学生的空间想象能力, 一定要能够根据三视图画出立体图形。", "solution": "null", "level": "五年级", "question": "2022・南京)从正面、左面看到的形状都是\n\n的立体图形, 至少需要 6 个小正方体才能摆成。( )", "options": [], "subject": "立体几何学", "analysis": "解$\\times$\n\n【分析】最少需要 3 个小正方体:把 3 个小正方体摆成一行,把左边一个向后平移一行,把右边一个向前平移一行, 即可满足条件, 据此即可解答问题。\n\n【详解】从正面、左面看到的形状都是\n\n\n的图形, 小正方体最少是 3 个, 如图:\n\n\n\n故答案为: $x$ 。\n\n【点睛】本题考查了学生的空间想象能力, 一定要能够根据三视图画出立体图形。"} {"id": "24105", "image": ["12101.jpg", "12102.jpg", "12103.jpg", "12104.jpg", "12105.jpg"], "answer": "解(1); (4)\n\n【详解】从正面看到的形状是图(1); 从左面看到的形状是图(4)", "solution": "null", "level": "五年级", "question": "(2022・南京)\n\n\n\n摆出这个立体图形, 从正面看到的形状是图几? 从左面看到的形状是图几?\n\n\n\n(1)\n\n\n\n(2)\n\n\n\n(3)\n\n\n\n(4)", "options": [], "subject": "立体几何学", "analysis": "解(1); (4)\n\n【详解】从正面看到的形状是图(1); 从左面看到的形状是图(4)"} {"id": "24106", "image": ["12106.jpg", "12107.jpg", "12108.jpg", "12107.jpg", "12108.jpg"], "answer": "解(1) 5 个\n\n(2) 8 个\n\n【分析】根据三视图拼组几何体即可\n\n【详解】(1)最少需要 5 个, 如下图:\n\n\n\n(2)最多需要 8 个,如下图:\n\n\n\n【点睛】本题主要考查从不同的方向观察几何体。", "solution": "null", "level": "五年级", "question": "(2022・海门市)一个立体图形, 从上面看到的是\n\n| | | | |\n| :--- | :--- | :--- | :--- |\n\n,从左面看到的是\n\n\n\n(1)摆这样的立体图形, 至少需要多少个小正方体?\n\n(2)摆这样的立体图形, 最多可以有多少个小正方体?", "options": [], "subject": "立体几何学", "analysis": "解(1) 5 个\n\n(2) 8 个\n\n【分析】根据三视图拼组几何体即可\n\n【详解】(1)最少需要 5 个, 如下图:\n\n\n\n(2)最多需要 8 个,如下图:\n\n\n\n【点睛】本题主要考查从不同的方向观察几何体。"} {"id": "24107", "image": ["12109.jpg", "12110.jpg", "12111.jpg", "12112.jpg"], "answer": "解小红", "solution": "null", "level": "五年级", "question": "(2022・南京)想一想, 搭一搭.\n\n从正面看\n\n\n\n从左面看\n\n\n\n从上面看\n\n\n三个小朋友根据上面看到的图形, 想象搭成的立体图形, 谁搭得对?\n\n", "options": [], "subject": "立体几何学", "analysis": "解小红"} {"id": "24108", "image": ["12113.jpg", "12114.jpg", "12115.jpg"], "answer": "解$5+1+2=8$ (个)", "solution": "null", "level": "五年级", "question": "(2022・南京)一个由若干个小正方体搭成的立体图形, 从三个不同的方向观察, 看到的形状如下图, 这里至少有多少个小方正方体才能搭成?\n\n\n\n从正面看\n\n\n\n从右面看\n\n\n\n从上面看", "options": [], "subject": "立体几何学", "analysis": "解$5+1+2=8$ (个)"} {"id": "24122", "image": [], "answer": "解$\\times$\n\n【分析】最多可以看到三个面: 长方形共有八个顶点, 从顶点的方向看, 每次最多看到三个面。据此判断。\n\n【详解】站在不同的位置观察一个长方体, 最多能看到长方体的 3 个面。原题说法错误。故答案为: $\\times$\n\n【点睛】此题考查了从不同方向观察物体和几何体, 锻炼了学生的空间想象力和抽象思维能力。", "solution": "null", "level": "五年级", "question": "从一个方向看一个长方体, 最多可以看到 4 个面。( )", "options": [], "subject": "立体几何学", "analysis": "解$\\times$\n\n【分析】最多可以看到三个面: 长方形共有八个顶点, 从顶点的方向看, 每次最多看到三个面。据此判断。\n\n【详解】站在不同的位置观察一个长方体, 最多能看到长方体的 3 个面。原题说法错误。故答案为: $\\times$\n\n【点睛】此题考查了从不同方向观察物体和几何体, 锻炼了学生的空间想象力和抽象思维能力。"} {"id": "24123", "image": ["12229.jpg", "12230.jpg", "12230.jpg"], "answer": "解$\\sqrt{ }$\n\n【分析】根据从上面看到的图形, 可知几何体的最底层有 3 个正方体; 然后根据从正面、左面看到的图形, 可知几何体有 2 层, 上面一层有 1 个正方体; 据此判断。\n\n【详解】如图:\n\n\n\n这个几何体是由 4 个同样大小的正方体组成的。\n\n原题说法正确。\n\n故答案为: $\\sqrt{ }$\n\n【点睛】本题考查根据三视图还原立体图形的能力, 培养学生的空间想象力。", "solution": "null", "level": "五年级", "question": "一个几何体,从正面、上面和左面看到都是\n\n\n, 则这个几何体是由 4 个同样大小的正方体\n组成的。( )", "options": [], "subject": "立体几何学", "analysis": "解$\\sqrt{ }$\n\n【分析】根据从上面看到的图形, 可知几何体的最底层有 3 个正方体; 然后根据从正面、左面看到的图形, 可知几何体有 2 层, 上面一层有 1 个正方体; 据此判断。\n\n【详解】如图:\n\n\n\n这个几何体是由 4 个同样大小的正方体组成的。\n\n原题说法正确。\n\n故答案为: $\\sqrt{ }$\n\n【点睛】本题考查根据三视图还原立体图形的能力, 培养学生的空间想象力。"} {"id": "24126", "image": ["12235.jpg", "12236.jpg", "12235.jpg", "12236.jpg"], "answer": "解$\\times$\n\n【分析】如图所示,\n\n\n\n这个立体图形从左面看到的形状是\n\n\n\n但是这个立体图形是由 6 个小正方体摆成的。\n\n【详解】一个立体图形从左面看到的形状是 | | |\n| :--- | :--- |\n| | |, ,这个立体图形不一定是由 4 个小正方体摆成的。故答案为: $\\times$\n\n【点睛】本题主要考查根据观察到的图形确定几何体。", "solution": "null", "level": "五年级", "question": "一个立体图形从左面看到的形状是\n\n| | |\n| :--- | :--- |\n| | |\n\n, 这个立体图形一定是由 4 个小正方体摆成的。(", "options": [], "subject": "立体几何学", "analysis": "解$\\times$\n\n【分析】如图所示,\n\n\n\n这个立体图形从左面看到的形状是\n\n\n\n但是这个立体图形是由 6 个小正方体摆成的。\n\n【详解】一个立体图形从左面看到的形状是 | | |\n| :--- | :--- |\n| | |, ,这个立体图形不一定是由 4 个小正方体摆成的。故答案为: $\\times$\n\n【点睛】本题主要考查根据观察到的图形确定几何体。"} {"id": "24128", "image": ["12242.jpg", "12243.jpg"], "answer": "解5 个; 8 个\n\n【解析】略", "solution": "null", "level": "五年级", "question": "一个几何体,从上面看到的图形是\n\n\n,从左面看到的图形是\n\n\n的几何体, 最少需要几个小正方体?最多呢?", "options": [], "subject": "立体几何学", "analysis": "解5 个; 8 个\n\n【解析】略"} {"id": "24129", "image": ["12244.jpg", "12245.jpg", "12245.jpg"], "answer": "解7 个\n\n【分析】从正面看、左面看可以判定有两列, 三行, 从正面看、左面看、上面看判定第一层有 4 个正方体, 第二层有 2 个正方体, 第三层有 1 个正方体, 由此得出答案即可。\n\n【详解】由分析可得下图:\n\n\n\n一层有 4 个正方体, 第二层有 2 个正方体, 第三层有 1 个小正方体。一共有 $4+2+1=7$ (个)。\n\n【点睛】本题关键是掌握从三个方向确定物体的方法。", "solution": "null", "level": "五年级", "question": "由若干个小正方体木块搭成的立体图形, 从三个方向所看到的图形如下。摆一摆, 看它是由多少个小正方体木块搭成的。\n\n\n从正面看 从左面看 从上面看", "options": [], "subject": "立体几何学", "analysis": "解7 个\n\n【分析】从正面看、左面看可以判定有两列, 三行, 从正面看、左面看、上面看判定第一层有 4 个正方体, 第二层有 2 个正方体, 第三层有 1 个正方体, 由此得出答案即可。\n\n【详解】由分析可得下图:\n\n\n\n一层有 4 个正方体, 第二层有 2 个正方体, 第三层有 1 个小正方体。一共有 $4+2+1=7$ (个)。\n\n【点睛】本题关键是掌握从三个方向确定物体的方法。"} {"id": "24130", "image": ["12246.jpg", "12247.jpg", "12248.jpg", "12249.jpg", "12250.jpg", "12251.jpg", "12249.jpg", "12250.jpg", "12251.jpg"], "answer": "解(1) 8\n\n(2)(3)图见详解\n\n【分析】(1) 观察上面的物体, 数一数正方体的数量即可解答。\n\n(2)观察这些图形, 并把从前面看到的图形画下来即可解题。\n\n(3)根据原物体从左面看到的图形是\n\n\n即可解答。\n\n【详解】(1)上面的物体是由 8 个小正方体搭成的。\n\n(2)作图如下:\n\n\n\n(3)作图如下:\n\n\n\n【点睛】本题考查了从不同的角度观察物体, 关键是要掌握从不同的角度观察物体的方法, 会分析从不同的角度观察到的图形的特点。", "solution": "null", "level": "五年级", "question": "添一添,画一画。\n\n\n\n(1)上面的物体是由()个小正方体搭成的。\n\n(2)如果给下面的物体添上一个正方体后,从前面看到的图形变成了\n\n\n这个正方体应该添在什么位置?请你在图上用 标出来。\n\n(3)如果给下面的物体添上一个正方体后,从左面看到的图形和原来相同,这个正方体应该添在什么位置?请你在图上用序号(1)、(2)、(3)、(4)表示出 4 种符合要求的情况。\n\n", "options": [], "subject": "立体几何学", "analysis": "解(1) 8\n\n(2)(3)图见详解\n\n【分析】(1) 观察上面的物体, 数一数正方体的数量即可解答。\n\n(2)观察这些图形, 并把从前面看到的图形画下来即可解题。\n\n(3)根据原物体从左面看到的图形是\n\n\n即可解答。\n\n【详解】(1)上面的物体是由 8 个小正方体搭成的。\n\n(2)作图如下:\n\n\n\n(3)作图如下:\n\n\n\n【点睛】本题考查了从不同的角度观察物体, 关键是要掌握从不同的角度观察物体的方法, 会分析从不同的角度观察到的图形的特点。"} {"id": "24131", "image": ["12252.jpg", "12253.jpg", "12254.jpg"], "answer": "解(1)第 7 个正方体积木可以放在 2 号或 5 号位置。\n\n(2)如果第 7 个正方体积木放在 2 号位置, 那么第 8 个正方体积木应放在 $4 、 5$ 或 6 号位置; 如果第 7 个正方体积木放在 5 号位置, 那么第 8 个正方体积木应放在 $1 、 2$ 或 3 号位置。\n\n【分析】(1)如果用 7 个正方体积木搭这个几何体, 第 7 个正方体积木可以放在下层前排的中间或后排的中间位置。\n\n(2)根据(1)的结果, 如果第 7 个正方体放在 2 的位置, 则第 8 个正方体可以放在 $4 、 5 、 6$ 的任意一个的上面; 如果第 7 个正方体放在 5 的位置, 则第 8 个正方体可以放在 $1 、 2 、 3$ 的任意一个的上面。\n\n【详解】(1)第 7 个正方体积木可以放在 2 号或 5 号位置。\n\n(2)如果第 7 个正方体积木放在 2 号位置, 那么第 8 个正方体积木应放在 $4 、 5$ 或 6 号位置; 如果第 7 个正方体积木放在 5 号位置, 那么第 8 个正方体积木应放在 1 、 2 或 3 号位置。\n\n【点睛】本题考查从不同方向观察物体和几何图形, 是培养学生的观察能力。", "solution": "null", "level": "五年级", "question": "一个由正方体积木搭成的几何体,从不同方向看到的图形如下。\n\n\n\n从上面看\n\n\n\n从正面看\n\n(1)如果用 7 个正方体积木搭这个几何体,第 7 个正方体积木可以放在哪个位置?(图中的序号是位置号)\n\n| 1 | 2 | 3 |\n| :--- | :--- | :--- |\n| 4 | 5 | 6 |\n\n(2)如果再增加 1 个正方体积木,使从上面看到的图形不变,从左面看到的图形是\n\n| | |\n| :--- | :--- |\n| | |\n\n那么第 8 个正方体积木可以放在哪个位置?(图中的序号是位置号)\n\n", "options": [], "subject": "立体几何学", "analysis": "解(1)第 7 个正方体积木可以放在 2 号或 5 号位置。\n\n(2)如果第 7 个正方体积木放在 2 号位置, 那么第 8 个正方体积木应放在 $4 、 5$ 或 6 号位置; 如果第 7 个正方体积木放在 5 号位置, 那么第 8 个正方体积木应放在 $1 、 2$ 或 3 号位置。\n\n【分析】(1)如果用 7 个正方体积木搭这个几何体, 第 7 个正方体积木可以放在下层前排的中间或后排的中间位置。\n\n(2)根据(1)的结果, 如果第 7 个正方体放在 2 的位置, 则第 8 个正方体可以放在 $4 、 5 、 6$ 的任意一个的上面; 如果第 7 个正方体放在 5 的位置, 则第 8 个正方体可以放在 $1 、 2 、 3$ 的任意一个的上面。\n\n【详解】(1)第 7 个正方体积木可以放在 2 号或 5 号位置。\n\n(2)如果第 7 个正方体积木放在 2 号位置, 那么第 8 个正方体积木应放在 $4 、 5$ 或 6 号位置; 如果第 7 个正方体积木放在 5 号位置, 那么第 8 个正方体积木应放在 1 、 2 或 3 号位置。\n\n【点睛】本题考查从不同方向观察物体和几何图形, 是培养学生的观察能力。"} {"id": "24132", "image": ["12255.jpg", "12256.jpg", "12257.jpg", "12258.jpg", "12259.jpg", "12260.jpg", "12261.jpg", "12262.jpg", "12262.jpg"], "answer": "解(1) (3)(5)(6) (1)(4) (3)\n(2)\n\n(答案不唯一)", "solution": "null", "level": "五年级", "question": "用 5 个同样大小的小正方体摆出了下面的几个几何体.\n\n\n\n(1)\n\n\n\n(2)\n\n\n\n(3)\n\n\n\n(4)\n\n\n\n(5)\n\n\n\n(6)\n\n(1)从正面看是 エ的有哪几个? 从左面看是的有哪几个 $\\square$ ? 从上面看是\n\n\n个?\n\n(2)如果从正面看到的和(2)一样,用 5 个小正方体摆一摆,还可以怎么摆?", "options": [], "subject": "立体几何学", "analysis": "解(1) (3)(5)(6) (1)(4) (3)\n(2)\n\n(答案不唯一)"} {"id": "24148", "image": [], "answer": "解$\\sqrt{ }$\n\n【分析】立方体一个顶点有三个面, 站在顶点处看到的面最多, 能看到三个面, 据此分析。\n\n【详解】站在同一位置, 最多能看到一个立方体的三个面, 说法正确。\n\n故答案为: $\\sqrt{ }$\n\n【点睛】从不同角度方向观察物体,常常得到不同的结果。", "solution": "null", "level": "五年级", "question": "站在同一位置, 最多能看到一个立方体的三个面。( )", "options": [], "subject": "立体几何学", "analysis": "解$\\sqrt{ }$\n\n【分析】立方体一个顶点有三个面, 站在顶点处看到的面最多, 能看到三个面, 据此分析。\n\n【详解】站在同一位置, 最多能看到一个立方体的三个面, 说法正确。\n\n故答案为: $\\sqrt{ }$\n\n【点睛】从不同角度方向观察物体,常常得到不同的结果。"} {"id": "24149", "image": ["12349.jpg", "12350.jpg", "12351.jpg", "12350.jpg", "12351.jpg"], "answer": "解$\\times$\n\n【分析】将从前面和右面看到的图形画出来,再进行判断即可。\n【详解】从前面看:\n\n\n\n从右面看:\n\n\n\n从前面和右面看到的图形不相同;\n\n故答案为: $\\times$\n\n【点睛】本题考查了空间思维能力, 画什么方位的平面图形就假设自己站在什么位置。", "solution": "null", "level": "五年级", "question": "\n\n\n\n从前面和右面看到的图形相同。( )", "options": [], "subject": "立体几何学", "analysis": "解$\\times$\n\n【分析】将从前面和右面看到的图形画出来,再进行判断即可。\n【详解】从前面看:\n\n\n\n从右面看:\n\n\n\n从前面和右面看到的图形不相同;\n\n故答案为: $\\times$\n\n【点睛】本题考查了空间思维能力, 画什么方位的平面图形就假设自己站在什么位置。"} {"id": "24150", "image": [], "answer": "解$\\sqrt{ }$\n\n【分析】根据从不同的角度观察物体的方法可知: 不同的物体分别从不同的角度观察, 看到的形状可能是相同的, 也可能是不同的, 由此进行解答。\n\n【详解】不同的物体分别从不同的角度观察, 看到的形状可能是相同的, 也可能是不同的, 原题说法正确。\n\n故答案为: $\\sqrt{ }$\n\n【点睛】本题考查了从不同的角度观察物体, 关键是要掌握从不同的角度观察物体的方法。", "solution": "null", "level": "五年级", "question": "不同的物体分别从不同的角度观察,看到的形状可能是相同的,也可能是不同的。", "options": [], "subject": "立体几何学", "analysis": "解$\\sqrt{ }$\n\n【分析】根据从不同的角度观察物体的方法可知: 不同的物体分别从不同的角度观察, 看到的形状可能是相同的, 也可能是不同的, 由此进行解答。\n\n【详解】不同的物体分别从不同的角度观察, 看到的形状可能是相同的, 也可能是不同的, 原题说法正确。\n\n故答案为: $\\sqrt{ }$\n\n【点睛】本题考查了从不同的角度观察物体, 关键是要掌握从不同的角度观察物体的方法。"} {"id": "24152", "image": ["12355.jpg", "12356.jpg", "12356.jpg"], "answer": "解$\\times$\n\n【分析】从物体的侧面看到一行 4 个正方形,而从正面只看到一个正方形。\n\n\n\n故答案: $\\times$\n\n【点睛】此题考查从不同方向观察物体, 要充分发挥空间想象力, 也可以动手摆一摆。", "solution": "null", "level": "五年级", "question": "从物体\n\n\n\n侧面和正面看到的形状相同。( )", "options": [], "subject": "立体几何学", "analysis": "解$\\times$\n\n【分析】从物体的侧面看到一行 4 个正方形,而从正面只看到一个正方形。\n\n\n\n故答案: $\\times$\n\n【点睛】此题考查从不同方向观察物体, 要充分发挥空间想象力, 也可以动手摆一摆。"} {"id": "24153", "image": [], "answer": "解$\\sqrt{ }$\n\n【分析】该几何体从正面看到的图形是 , 说明该几何体至少有一行, 这一行里有两层, 两层都是 2 个小正方体; 所以至少是需要 4 个小正方体; 据此解答。\n\n【详解】由分析可知: 要摆出从正面看到的图形是 的几何体, 至少需要 4 个小正方体, 所以原题说法正确。\n\n故答案为: $\\sqrt{ }$\n\n【点睛】本题考查了观察物体, 关键是要掌握从不同角度观察到的立体图形的特点, 分析出该立体图形的形状。", "solution": "null", "level": "五年级", "question": "要摆出从正面看到的图形是 $\\square$的几何体, 至少需要 4 个小正方体。", "options": [], "subject": "立体几何学", "analysis": "解$\\sqrt{ }$\n\n【分析】该几何体从正面看到的图形是 , 说明该几何体至少有一行, 这一行里有两层, 两层都是 2 个小正方体; 所以至少是需要 4 个小正方体; 据此解答。\n\n【详解】由分析可知: 要摆出从正面看到的图形是 的几何体, 至少需要 4 个小正方体, 所以原题说法正确。\n\n故答案为: $\\sqrt{ }$\n\n【点睛】本题考查了观察物体, 关键是要掌握从不同角度观察到的立体图形的特点, 分析出该立体图形的形状。"} {"id": "24155", "image": ["12359.jpg", "12360.jpg", "12361.jpg", "12362.jpg", "12363.jpg", "12364.jpg", "12365.jpg", "12363.jpg", "12364.jpg", "12365.jpg"], "answer": "解见详解\n\n【分析】观察图形可知, 从上面可以看到的图形有两排, 第一排有 2 个正方形靠左边, 第二排有 3 个正方形; 从正面看到的图形有两层, 第一层有 3 个正方形, 第二层有 2 个正方形靠最左边; 从左面看到的图形有两层, 第一层有 2 个正方形, 第二层有 1 个正方形靠最左边。据此作图即可。\n\n【详解】由分析可知, 如图所示:\n\n\n\n上面\n\n\n\n正面\n\n\n\n左面\n\n【点睛】本题考查从不同方向观察物体, 明确从各个方向看到图形的形状是解题的关键。", "solution": "null", "level": "五年级", "question": "下面立体图形从上面、正面和左面看到的形状分别是什么?画一画。\n\n\n\n\n\n上面\n\n\n\n正面\n\n\n\n左面", "options": [], "subject": "立体几何学", "analysis": "解见详解\n\n【分析】观察图形可知, 从上面可以看到的图形有两排, 第一排有 2 个正方形靠左边, 第二排有 3 个正方形; 从正面看到的图形有两层, 第一层有 3 个正方形, 第二层有 2 个正方形靠最左边; 从左面看到的图形有两层, 第一层有 2 个正方形, 第二层有 1 个正方形靠最左边。据此作图即可。\n\n【详解】由分析可知, 如图所示:\n\n\n\n上面\n\n\n\n正面\n\n\n\n左面\n\n【点睛】本题考查从不同方向观察物体, 明确从各个方向看到图形的形状是解题的关键。"} {"id": "24159", "image": ["12381.jpg", "12382.jpg", "12382.jpg"], "answer": "解见详解\n\n【分析】这个立方体图形从上面能看到 4 个正方形, 分两行, 上行 2 个, 下行 2 个, 右齐; 从右面能看到 5 个正方形, 分三行, 上行 1 个, 中行 2 个, 下行 2 个, 右齐; 据此作图。\n\n【详解】从右面看到的图形如图:\n\n\n\n【点晴】本题是考查作简单图形的三视图, 能正确辨认从正面、上面、左面(或右面)观察到的简单几何体的平面图形。", "solution": "null", "level": "五年级", "question": "左下图是用同样大小的立方体搭成的几何体,从上往下看到的形状,数字表示该位置立方体的个数, 请你把从右面看到的图形画在右图中。\n", "options": [], "subject": "立体几何学", "analysis": "解见详解\n\n【分析】这个立方体图形从上面能看到 4 个正方形, 分两行, 上行 2 个, 下行 2 个, 右齐; 从右面能看到 5 个正方形, 分三行, 上行 1 个, 中行 2 个, 下行 2 个, 右齐; 据此作图。\n\n【详解】从右面看到的图形如图:\n\n\n\n【点晴】本题是考查作简单图形的三视图, 能正确辨认从正面、上面、左面(或右面)观察到的简单几何体的平面图形。"} {"id": "24456", "image": [], "answer": "解$\\times$\n\n【分析】体积和容积既有联系也有区别, 它们的联系是计算方法相同, 它们的区别是计算体积要从外面测量有关数据(如长方体的长、宽和高), 计算容积是从容器的里面测量有关数据, 如果容纳的物\n体是液体就用容积单位升和毫升, 以此解答即可。\n\n【详解】计算冰箱的体积是从外面测量他长、宽、高; 计算冰箱的容积是从里面测量它的长、宽和高;因此电冰箱的容积就是电冰箱的体积, 这种说法是错误的。\n\n故答案为: $\\times$\n\n【点睛】此题主要考查体积和容积的意义以及它们之间的联系与区别。", "solution": "null", "level": "五年级", "question": "电冰箱的体积就是电冰箱的容积。", "options": [], "subject": "立体几何学", "analysis": "解$\\times$\n\n【分析】体积和容积既有联系也有区别, 它们的联系是计算方法相同, 它们的区别是计算体积要从外面测量有关数据(如长方体的长、宽和高), 计算容积是从容器的里面测量有关数据, 如果容纳的物\n体是液体就用容积单位升和毫升, 以此解答即可。\n\n【详解】计算冰箱的体积是从外面测量他长、宽、高; 计算冰箱的容积是从里面测量它的长、宽和高;因此电冰箱的容积就是电冰箱的体积, 这种说法是错误的。\n\n故答案为: $\\times$\n\n【点睛】此题主要考查体积和容积的意义以及它们之间的联系与区别。"} {"id": "24457", "image": [], "answer": "解$\\sqrt{ }$\n\n【分析】棱长 1 分米的正方体, 体积是 1 立方分米, 根据长方体体积 $=$ 长 $\\times$ 宽 $\\times$ 高, 求出长方体木块的体积, 就是切成棱长是 1 分米的小正方体木块的块数, 据此分析。\n\n【详解】 $5 \\times 4 \\times 2=40$ (块)\n\n把一个长 5 分米, 宽 4 分米, 高 2 分米的长方体木块切成棱长是 1 分米的小正方体木块, 可以切 40 块, 说法正确。\n\n故答案为: $\\sqrt{ }$\n\n【点睛】关键是掌握并灵活运用长方体体积公式, 理解体积单位的建立标准。", "solution": "null", "level": "五年级", "question": "把一个长 5 分米, 宽 4 分米, 高 2 分米的长方体木块切成棱长是 1 分米的小正方体木块, 可以切 40 块。( )", "options": [], "subject": "立体几何学", "analysis": "解$\\sqrt{ }$\n\n【分析】棱长 1 分米的正方体, 体积是 1 立方分米, 根据长方体体积 $=$ 长 $\\times$ 宽 $\\times$ 高, 求出长方体木块的体积, 就是切成棱长是 1 分米的小正方体木块的块数, 据此分析。\n\n【详解】 $5 \\times 4 \\times 2=40$ (块)\n\n把一个长 5 分米, 宽 4 分米, 高 2 分米的长方体木块切成棱长是 1 分米的小正方体木块, 可以切 40 块, 说法正确。\n\n故答案为: $\\sqrt{ }$\n\n【点睛】关键是掌握并灵活运用长方体体积公式, 理解体积单位的建立标准。"} {"id": "24458", "image": [], "answer": "解$\\times$\n\n【分析】物体所占空间的大小叫做体积, 体积需要从物体的外部测量; 箱子、油桶、仓库所能容纳物体的体积叫做容积, 容积需要从物体的内部测量; 据此解答。\n\n【详解】 $2 \\mathrm{~L}=2$ 立方分米\n\n水壸本身有厚度,能装 $2 \\mathrm{~L}$ 水的水壸,它的体积大于 2 立方分米。\n\n故答案为: $\\times$\n\n【点睛】本题主要考查学生对体积和容积的认识, 掌握两者的区别是解答题目的关键。", "solution": "null", "level": "五年级", "question": "能装 $2 \\mathrm{~L}$ 水的水壸, 它的体积就是 2 立方分米。( )", "options": [], "subject": "立体几何学", "analysis": "解$\\times$\n\n【分析】物体所占空间的大小叫做体积, 体积需要从物体的外部测量; 箱子、油桶、仓库所能容纳物体的体积叫做容积, 容积需要从物体的内部测量; 据此解答。\n\n【详解】 $2 \\mathrm{~L}=2$ 立方分米\n\n水壸本身有厚度,能装 $2 \\mathrm{~L}$ 水的水壸,它的体积大于 2 立方分米。\n\n故答案为: $\\times$\n\n【点睛】本题主要考查学生对体积和容积的认识, 掌握两者的区别是解答题目的关键。"} {"id": "24459", "image": [], "answer": "解$\\times$\n\n【分析】物体所有面的总面积叫做物体的表面积; 物体所占空间的大小叫做物体的体积。\n\n【详解】表面积与体积的意义不同, 是不同类量, 无法比较大小。\n\n棱长为 $6 \\mathrm{dm}$ 的正方体的体积和表面积不能比较大小。\n\n原题说法错误。\n\n故答案为: $\\times$\n\n【点睛】本题考查正方体的表面积、体积的意义,明确只有同类量才能比较大小。", "solution": "null", "level": "五年级", "question": "棱长为 $6 \\mathrm{dm}$ 的正方体的体积和表面积相等。( )", "options": [], "subject": "立体几何学", "analysis": "解$\\times$\n\n【分析】物体所有面的总面积叫做物体的表面积; 物体所占空间的大小叫做物体的体积。\n\n【详解】表面积与体积的意义不同, 是不同类量, 无法比较大小。\n\n棱长为 $6 \\mathrm{dm}$ 的正方体的体积和表面积不能比较大小。\n\n原题说法错误。\n\n故答案为: $\\times$\n\n【点睛】本题考查正方体的表面积、体积的意义,明确只有同类量才能比较大小。"} {"id": "24460", "image": [], "answer": "解$\\times$\n\n【分析】假设出原来长方体的长、宽、高,根据“长方体的表面积=(长 $\\times$ 宽 + 长 $\\times$ 高 + 宽 $\\times$ 高)×2”“长方体的体积 $=$ 长 $\\times$ 宽 $\\times$ 高\"表示出原来和现在长方体的表面积和体积, 最后用除法求出表面积和体积扩大的倍数, 据此解答。\n【详解】假设原来长方体的长为 $\\mathrm{a}$, 宽为 $\\mathrm{b}$, 高为 $\\mathrm{h}$, 则现在长方体的长为 $2 \\mathrm{a}$, 宽为 $2 \\mathrm{~b}$, 高为 $2 \\mathrm{~h}$ 。原来的表面积: $( a b+a h+b h ) \\times 2$\n\n$=2(\\mathrm{ab}+\\mathrm{ah}+\\mathrm{bh})$\n\n现在的表面积: $\\quad(2 \\mathrm{a} \\times 2 \\mathrm{~b}+2 \\mathrm{a} \\times 2 \\mathrm{~h}+2 \\mathrm{~b} \\times 2 \\mathrm{~h}) \\times 2$\n\n$=(4 \\mathrm{ab}+4 \\mathrm{ah}+4 \\mathrm{bh}) \\times 2$\n\n$=(a b+a h+b h) \\times 4 \\times 2$\n\n$=8(a b+a h+b h)$\n\n$8(a b+a h+b h) \\div 2(a b+a h+b h)=4$\n\n所以, 长方体的表面积扩大到原来的 4 倍。\n\n原来的体积: $a b h$\n\n现在的体积: $2 \\mathrm{a} \\times 2 \\mathrm{~b} \\times 2 \\mathrm{~h}$\n\n$=8 a b h$\n\n$8 \\mathrm{abh} \\div \\mathrm{abh}=8$\n\n所以, 长方体的体积扩大到原来的 8 倍。\n\n故答案为: $\\times$\n\n【点睛】长方体的长、宽、高同时扩大到原来的 $\\mathrm{a}$ 倍, 长方体的表面积扩大到原来的 $\\mathrm{a}^{2}$ 倍, 体积扩大到原来的 $\\mathrm{a}^{3}$ 倍。", "solution": "null", "level": "五年级", "question": "如果长方体的长、宽、高都扩大到原来的 2 倍, 则它的表面积和体积都扩大到原来的 8 倍。", "options": [], "subject": "立体几何学", "analysis": "解$\\times$\n\n【分析】假设出原来长方体的长、宽、高,根据“长方体的表面积=(长 $\\times$ 宽 + 长 $\\times$ 高 + 宽 $\\times$ 高)×2”“长方体的体积 $=$ 长 $\\times$ 宽 $\\times$ 高\"表示出原来和现在长方体的表面积和体积, 最后用除法求出表面积和体积扩大的倍数, 据此解答。\n【详解】假设原来长方体的长为 $\\mathrm{a}$, 宽为 $\\mathrm{b}$, 高为 $\\mathrm{h}$, 则现在长方体的长为 $2 \\mathrm{a}$, 宽为 $2 \\mathrm{~b}$, 高为 $2 \\mathrm{~h}$ 。原来的表面积: $( a b+a h+b h ) \\times 2$\n\n$=2(\\mathrm{ab}+\\mathrm{ah}+\\mathrm{bh})$\n\n现在的表面积: $\\quad(2 \\mathrm{a} \\times 2 \\mathrm{~b}+2 \\mathrm{a} \\times 2 \\mathrm{~h}+2 \\mathrm{~b} \\times 2 \\mathrm{~h}) \\times 2$\n\n$=(4 \\mathrm{ab}+4 \\mathrm{ah}+4 \\mathrm{bh}) \\times 2$\n\n$=(a b+a h+b h) \\times 4 \\times 2$\n\n$=8(a b+a h+b h)$\n\n$8(a b+a h+b h) \\div 2(a b+a h+b h)=4$\n\n所以, 长方体的表面积扩大到原来的 4 倍。\n\n原来的体积: $a b h$\n\n现在的体积: $2 \\mathrm{a} \\times 2 \\mathrm{~b} \\times 2 \\mathrm{~h}$\n\n$=8 a b h$\n\n$8 \\mathrm{abh} \\div \\mathrm{abh}=8$\n\n所以, 长方体的体积扩大到原来的 8 倍。\n\n故答案为: $\\times$\n\n【点睛】长方体的长、宽、高同时扩大到原来的 $\\mathrm{a}$ 倍, 长方体的表面积扩大到原来的 $\\mathrm{a}^{2}$ 倍, 体积扩大到原来的 $\\mathrm{a}^{3}$ 倍。"} {"id": "24465", "image": ["12494.jpg", "12495.jpg", "12495.jpg"], "answer": "解6 分米\n\n\n面面积,列式解答即可。\n\n【详解】 $150 \\div(50 \\div 2)$\n\n$=150 \\div 25$\n\n$=6$ (分米)\n\n答: 这根木料原来 6 分米长。\n\n【点睛】关键是掌握并灵活运用长方体体积公式。", "solution": "null", "level": "五年级", "question": "将一根体积为 150 立方分米的长方体木料垂直截开表面积增加了 50 平方分米, 那么这根木料原来多长?\n\n", "options": [], "subject": "立体几何学", "analysis": "解6 分米\n\n\n面面积,列式解答即可。\n\n【详解】 $150 \\div(50 \\div 2)$\n\n$=150 \\div 25$\n\n$=6$ (分米)\n\n答: 这根木料原来 6 分米长。\n\n【点睛】关键是掌握并灵活运用长方体体积公式。"} {"id": "24467", "image": [], "answer": "解不会\n\n【分析】根据正方体的体积公式: $\\mathrm{V}=\\mathrm{a}^{3}$, 据此求出铁块的体积; 再根据长方体的体积公式: $\\mathrm{V}=\\mathrm{abh}$,据此求出水的体积, 用铁块的体积加上水的体积, 再与玻璃缸的体积对比即可。\n\n【详解】 $3 \\times 3 \\times 3+8 \\times 6 \\times 2.8$\n\n$=27+134.4$\n\n$=161.4$ (立方分米)\n\n$8 \\times 6 \\times 4$\n\n$=48 \\times 4$\n\n$=192$ (立方分米)\n\n$161.4<192$\n\n答: 缸里的不会溢出。\n\n【点睛】本题考查正方体和长方体的体积, 熟记公式是解题的关键。", "solution": "null", "level": "五年级", "question": "一只长方体玻璃缸, 长 8 分米, 宽 6 分米, 高 4 分米, 水深 2.8 分米, 如果投入一块棱长为 3 分米的正方体铁块,水会溢出来吗?", "options": [], "subject": "立体几何学", "analysis": "解不会\n\n【分析】根据正方体的体积公式: $\\mathrm{V}=\\mathrm{a}^{3}$, 据此求出铁块的体积; 再根据长方体的体积公式: $\\mathrm{V}=\\mathrm{abh}$,据此求出水的体积, 用铁块的体积加上水的体积, 再与玻璃缸的体积对比即可。\n\n【详解】 $3 \\times 3 \\times 3+8 \\times 6 \\times 2.8$\n\n$=27+134.4$\n\n$=161.4$ (立方分米)\n\n$8 \\times 6 \\times 4$\n\n$=48 \\times 4$\n\n$=192$ (立方分米)\n\n$161.4<192$\n\n答: 缸里的不会溢出。\n\n【点睛】本题考查正方体和长方体的体积, 熟记公式是解题的关键。"} {"id": "24468", "image": [], "answer": "解$8000 \\mathrm{~cm}^{2}$\n\n【分析】木料截为 5 段, 实际只截了 4 次。每截 1 次比原来多 2 个截面, 截 4 次就多 8 个截面。表面积比原来增加了 $128 \\mathrm{~cm}^{2}$, 即这 8 个截面的面积和是 $128 \\mathrm{~cm}^{2}$, 故一个面的面积是 $128 \\div 8=16 \\mathrm{~cm}^{2}$ 。截面即底面, 木料体积 $=$ 底面积 $\\times$ 长。注意转换为统一的单位。\n\n【详解】 $5-1=4$ (次)\n\n$128 \\div(4 \\times 2)$\n\n$=128 \\div 8$\n\n$=16 \\mathrm{~cm}^{2}$\n\n$5 \\mathrm{~m}=500 \\mathrm{~cm}$\n\n$16 \\times 500=8000 \\mathrm{~cm}^{2}$\n\n答: 原来的长方体木料的体积是 $8000 \\mathrm{~cm}^{2}$ 。\n\n【点睛】本题主要考查长方体的体积的实际应用, 关键要理解截成 5 段实际只截了 4 次。", "solution": "null", "level": "五年级", "question": "把一根长 $5 \\mathrm{~m}$ 的长方体木料截为 5 段后, 表面积比原来增加了 $128 \\mathrm{~cm}^{2}$, 求原来的长方体木料的体积。", "options": [], "subject": "立体几何学", "analysis": "解$8000 \\mathrm{~cm}^{2}$\n\n【分析】木料截为 5 段, 实际只截了 4 次。每截 1 次比原来多 2 个截面, 截 4 次就多 8 个截面。表面积比原来增加了 $128 \\mathrm{~cm}^{2}$, 即这 8 个截面的面积和是 $128 \\mathrm{~cm}^{2}$, 故一个面的面积是 $128 \\div 8=16 \\mathrm{~cm}^{2}$ 。截面即底面, 木料体积 $=$ 底面积 $\\times$ 长。注意转换为统一的单位。\n\n【详解】 $5-1=4$ (次)\n\n$128 \\div(4 \\times 2)$\n\n$=128 \\div 8$\n\n$=16 \\mathrm{~cm}^{2}$\n\n$5 \\mathrm{~m}=500 \\mathrm{~cm}$\n\n$16 \\times 500=8000 \\mathrm{~cm}^{2}$\n\n答: 原来的长方体木料的体积是 $8000 \\mathrm{~cm}^{2}$ 。\n\n【点睛】本题主要考查长方体的体积的实际应用, 关键要理解截成 5 段实际只截了 4 次。"} {"id": "24469", "image": ["12497.jpg"], "answer": "解(1) 96 厘米;\n\n(2) 480 立方厘米\n\n【分析】相交于长方体一个顶点的三条棱分别是长方体的长、宽、高, 根据长方体的棱长和 $=($ 长 +宽十高) $\\times 4$, 长方体的体积 $=$ 长 $\\times$ 宽 $\\times$ 高, 分别列式计算即可。\n\n【详解】(1) $(10+8+6) \\times 4$\n\n$=24 \\times 4$\n\n$=96$ (厘米)\n\n答: 这个长方体的棱长和是 96 厘米。\n\n(2) $10 \\times 8 \\times 6=480$ (立方厘米)\n\n答: 这个长方体的体积是 480 立方厘米。", "solution": "null", "level": "五年级", "question": "图中只给了一长方体的三条棱的长度(如图)。\n\n(1)这个长方体的棱长和是多长?\n\n(2)这个长方体的体积是多大?\n\n", "options": [], "subject": "立体几何学", "analysis": "解(1) 96 厘米;\n\n(2) 480 立方厘米\n\n【分析】相交于长方体一个顶点的三条棱分别是长方体的长、宽、高, 根据长方体的棱长和 $=($ 长 +宽十高) $\\times 4$, 长方体的体积 $=$ 长 $\\times$ 宽 $\\times$ 高, 分别列式计算即可。\n\n【详解】(1) $(10+8+6) \\times 4$\n\n$=24 \\times 4$\n\n$=96$ (厘米)\n\n答: 这个长方体的棱长和是 96 厘米。\n\n(2) $10 \\times 8 \\times 6=480$ (立方厘米)\n\n答: 这个长方体的体积是 480 立方厘米。"} {"id": "24483", "image": [], "answer": "解$\\sqrt{ }$\n\n【分析】根据长方体和正方体的体积公式, 长方体的体积 $=$ 长 $\\times$ 宽 $\\times$ 高, 长 $\\times$ 宽 $=$ 长方体的底面积; 即长方体的体积 $=$ 底面积 $\\times$ 高, 正方体的体积 $=$ 棱长 $\\times$ 棱长 $\\times$ 棱长, 棱长 $\\times$ 棱长 $=$ 正方体的底面积; 正方体的体积 $=$ 底面积 $\\times$ 高, 由此解答。\n\n【详解】长方体的体积 $=$ 底面积 $\\times$ 高, 正方体的体积 $=$ 底面积 $\\times$ 高;\n\n因此正方体和长方体的体积都可以用底面积乘以高来进行计算, 这种说法是正确的。\n\n故答案为: $\\sqrt{ }$\n\n【点睛】本题主要考查长方体正方体的体积公式,熟练掌握长方体正方体的体积公式并灵活运用。", "solution": "null", "level": "五年级", "question": "长方体和正方体的体积都等于底面积乘高。( )", "options": [], "subject": "立体几何学", "analysis": "解$\\sqrt{ }$\n\n【分析】根据长方体和正方体的体积公式, 长方体的体积 $=$ 长 $\\times$ 宽 $\\times$ 高, 长 $\\times$ 宽 $=$ 长方体的底面积; 即长方体的体积 $=$ 底面积 $\\times$ 高, 正方体的体积 $=$ 棱长 $\\times$ 棱长 $\\times$ 棱长, 棱长 $\\times$ 棱长 $=$ 正方体的底面积; 正方体的体积 $=$ 底面积 $\\times$ 高, 由此解答。\n\n【详解】长方体的体积 $=$ 底面积 $\\times$ 高, 正方体的体积 $=$ 底面积 $\\times$ 高;\n\n因此正方体和长方体的体积都可以用底面积乘以高来进行计算, 这种说法是正确的。\n\n故答案为: $\\sqrt{ }$\n\n【点睛】本题主要考查长方体正方体的体积公式,熟练掌握长方体正方体的体积公式并灵活运用。"} {"id": "24484", "image": [], "answer": "解$\\times$\n\n【分析】根据长方体的体积公式: $\\mathrm{V}=\\mathrm{Sh}$, 一个长方体纸箱的体积是 1 立方米, 这个纸箱的底面积不一定是 1 平方米。据此判断。\n\n【详解】比如: 一个长方体纸箱的底面边长是 0.5 米, 高是 4 米, 纸箱的底面积是: $0.5 \\times 0.5=0.25$ (平方米), 体积是: $0.5 \\times 0.5 \\times 4=1$ (立方米)。\n\n所以一个长方体纸箱的体积是 1 立方米, 这个纸箱的底面积不一定是 1 平方米。\n\n因此题干中的结论是错误的。\n故答案为: $\\times$\n\n【点睛】此题主要考查长方体体积公式的灵活运用, 关键是熟记公式。", "solution": "null", "level": "五年级", "question": "一个纸箱体积是 $1 \\mathrm{~m}^{3}$, 它的占地面积一定是 $1 \\mathrm{~m}^{2}$ 。( )", "options": [], "subject": "立体几何学", "analysis": "解$\\times$\n\n【分析】根据长方体的体积公式: $\\mathrm{V}=\\mathrm{Sh}$, 一个长方体纸箱的体积是 1 立方米, 这个纸箱的底面积不一定是 1 平方米。据此判断。\n\n【详解】比如: 一个长方体纸箱的底面边长是 0.5 米, 高是 4 米, 纸箱的底面积是: $0.5 \\times 0.5=0.25$ (平方米), 体积是: $0.5 \\times 0.5 \\times 4=1$ (立方米)。\n\n所以一个长方体纸箱的体积是 1 立方米, 这个纸箱的底面积不一定是 1 平方米。\n\n因此题干中的结论是错误的。\n故答案为: $\\times$\n\n【点睛】此题主要考查长方体体积公式的灵活运用, 关键是熟记公式。"} {"id": "24486", "image": [], "answer": "解$\\times$\n\n【分析】立体图形的表面积是指组成它的所有面的面积和, 而其体积是指它所占空间的大小, 两者意义不同, 不能比较大小。\n\n【详解】棱长为 $6 \\mathrm{~m}$ 的正方体表面积和体积不是同类量, 无法比较大小。\n\n原题说法错误。\n\n故答案为: $\\times$\n\n【点睛】本题考查正方体表面积和体积的意义, 明确表面积和体积不能比较大小。", "solution": "null", "level": "五年级", "question": "棱长为 $6 \\mathrm{~m}$ 的正方体表面积和体积相等。()", "options": [], "subject": "立体几何学", "analysis": "解$\\times$\n\n【分析】立体图形的表面积是指组成它的所有面的面积和, 而其体积是指它所占空间的大小, 两者意义不同, 不能比较大小。\n\n【详解】棱长为 $6 \\mathrm{~m}$ 的正方体表面积和体积不是同类量, 无法比较大小。\n\n原题说法错误。\n\n故答案为: $\\times$\n\n【点睛】本题考查正方体表面积和体积的意义, 明确表面积和体积不能比较大小。"} {"id": "24487", "image": [], "answer": "解$\\times$\n\n【分析】把一个长方体切成两个小长方体, 需要锯 1 次, 每锯一次就会多出 2 个长方体的横截面, 因此, 这两个小长方体的表面积之和大于原来长方体的表面积。据此解答。\n\n【详解】由分析可知:\n\n把一个长方体切成两个小长方体, 这两个小长方体的表面积之和与原来长方体的表面积相比, 要比原来的大。\n\n故答案为: $\\times$\n\n【点睛】本题考查长方体的表面积, 明确切成两个小长方体表面积会增加两个横截面的面积是解题的关键。", "solution": "null", "level": "五年级", "question": "把一个长方体切开, 分成两个相同的长方体, 这两个长方体表面积之和与原来长方体的表面积相等。( $)$", "options": [], "subject": "立体几何学", "analysis": "解$\\times$\n\n【分析】把一个长方体切成两个小长方体, 需要锯 1 次, 每锯一次就会多出 2 个长方体的横截面, 因此, 这两个小长方体的表面积之和大于原来长方体的表面积。据此解答。\n\n【详解】由分析可知:\n\n把一个长方体切成两个小长方体, 这两个小长方体的表面积之和与原来长方体的表面积相比, 要比原来的大。\n\n故答案为: $\\times$\n\n【点睛】本题考查长方体的表面积, 明确切成两个小长方体表面积会增加两个横截面的面积是解题的关键。"} {"id": "24491", "image": [], "answer": "解27 升; 22.95 千克\n\n【分析】根据正方体的体积(容积)公式: $\\mathrm{V}=\\mathrm{a}^{3}$, 代入棱长的数据, 即可求出这个油箱的容积; 再根据 1 升 $=1000$ 立方厘米, 把油箱里能装油的体积换算单位后, 再乘每升汽油的质量, 即可求出这一箱汽油的质量。\n\n【详解】 $30 \\times 30 \\times 30=27000$ (立方厘米)\n\n27000 立方厘米 $=27$ 升\n\n$27 \\times 0.85=22.95$ (千克)\n\n答: 这个油箱的容积是 27 升, 这样一箱汽油重 22.95 千克。\n【点睛】此题的解题关键是掌握正方体的体积(容积)公式以及体积、容积单位之间的换算。", "solution": "null", "level": "五年级", "question": "一个正方体油箱, 从里面量棱长为 30 厘米, 这个油箱的容积是多少升? 如果每升汽油 0.85 千克,这样一箱汽油重多少千克?", "options": [], "subject": "立体几何学", "analysis": "解27 升; 22.95 千克\n\n【分析】根据正方体的体积(容积)公式: $\\mathrm{V}=\\mathrm{a}^{3}$, 代入棱长的数据, 即可求出这个油箱的容积; 再根据 1 升 $=1000$ 立方厘米, 把油箱里能装油的体积换算单位后, 再乘每升汽油的质量, 即可求出这一箱汽油的质量。\n\n【详解】 $30 \\times 30 \\times 30=27000$ (立方厘米)\n\n27000 立方厘米 $=27$ 升\n\n$27 \\times 0.85=22.95$ (千克)\n\n答: 这个油箱的容积是 27 升, 这样一箱汽油重 22.95 千克。\n【点睛】此题的解题关键是掌握正方体的体积(容积)公式以及体积、容积单位之间的换算。"} {"id": "24494", "image": [], "answer": "解286 立方厘米\n\n【分析】先根据“长方形的周长 $=($ 长 + 宽 $) \\times 2$ ”求出底面的长宽之和, 再找出符合题意的长和宽, 最后利用“长方体的体积 $=$ 长 $\\times$ 宽 $\\times$ 高”求出这个长方体的体积, 据此解答。\n\n【详解】 $30 \\div 2=15$ (厘米)\n\n长和宽的长度都是质数, 当长为 13 厘米, 宽为 2 厘米时, $2+13=15$ (厘米), 符合题意。\n$13 \\times 2 \\times 11$\n\n$=26 \\times 11$\n\n$=286$ (立方厘米)\n\n答: 这个长方体的体积是 286 立方厘米。\n\n【点睛】根据题意求出符合题意的长和宽,并掌握长方体的体积计算公式是解答题目的关键。", "solution": "null", "level": "五年级", "question": "一个长方体的底面是一个周长为 30 厘米的长方形, 高为 11 厘米, 如果长和宽的长度都是质数,那么这个长方体的体积是多少?", "options": [], "subject": "立体几何学", "analysis": "解286 立方厘米\n\n【分析】先根据“长方形的周长 $=($ 长 + 宽 $) \\times 2$ ”求出底面的长宽之和, 再找出符合题意的长和宽, 最后利用“长方体的体积 $=$ 长 $\\times$ 宽 $\\times$ 高”求出这个长方体的体积, 据此解答。\n\n【详解】 $30 \\div 2=15$ (厘米)\n\n长和宽的长度都是质数, 当长为 13 厘米, 宽为 2 厘米时, $2+13=15$ (厘米), 符合题意。\n$13 \\times 2 \\times 11$\n\n$=26 \\times 11$\n\n$=286$ (立方厘米)\n\n答: 这个长方体的体积是 286 立方厘米。\n\n【点睛】根据题意求出符合题意的长和宽,并掌握长方体的体积计算公式是解答题目的关键。"} {"id": "24495", "image": [], "answer": "解18.84 平方厘米\n\n【分析】首先根据圆柱的体积公式: $\\mathrm{V}=\\mathrm{Sh}$, 求出当把长方体铁块从水中取出后下降部分水的体积(长方体铁块的体积),再根据长方体体积 $\\mathrm{V}=\\mathrm{Sh}$ ,那么 $\\mathrm{S}=\\mathrm{V} \\div \\mathrm{h}$ ,把数据代入公式解答。\n\n【详解】 $3.14 \\times(12 \\div 2)^{2} \\times 0.5 \\div 3$\n\n$=3.14 \\times 36 \\times 0.5 \\div 3$\n\n$=56.52 \\div 3$\n\n$=18.84$ (平方厘米)\n\n答:这个圆锥形铅锤的底面积是 18.84 平方厘米。\n\n【点睛】此题主要考查圆柱、长方体体积公式的灵活运用, 关键是熟记公式。", "solution": "null", "level": "五年级", "question": "一个圆柱形玻璃容器, 从里面量得底面直径为 12 厘米, 里面盛有水, 水中完全浸没着一个高为 3 厘米的长方体铁块, 当把长方体铁块从水中取出后, 水面下降了 0.5 厘米。这个长方体铁块的底面积是多少平方厘米?", "options": [], "subject": "立体几何学", "analysis": "解18.84 平方厘米\n\n【分析】首先根据圆柱的体积公式: $\\mathrm{V}=\\mathrm{Sh}$, 求出当把长方体铁块从水中取出后下降部分水的体积(长方体铁块的体积),再根据长方体体积 $\\mathrm{V}=\\mathrm{Sh}$ ,那么 $\\mathrm{S}=\\mathrm{V} \\div \\mathrm{h}$ ,把数据代入公式解答。\n\n【详解】 $3.14 \\times(12 \\div 2)^{2} \\times 0.5 \\div 3$\n\n$=3.14 \\times 36 \\times 0.5 \\div 3$\n\n$=56.52 \\div 3$\n\n$=18.84$ (平方厘米)\n\n答:这个圆锥形铅锤的底面积是 18.84 平方厘米。\n\n【点睛】此题主要考查圆柱、长方体体积公式的灵活运用, 关键是熟记公式。"} {"id": "24496", "image": [], "answer": "解 2.4 平方米\n\n【分析】由于通风管没有底面, 所以只求它的侧面积即可, 长方体的侧面积 $=$ 底面周长 $\\times$ 高, 据此列式解答。\n\n【详解】12 分米 $=1.2$ 米\n\n$1.2 \\times 2=2.4$ (平方米)\n\n答:做这个通风管至少需要 2.4 平方米的铁皮。\n\n【点睛】解答有关长方体计算的实际问题,一定要搞清所求的是什么(体积、表面积还是几个面的面积),再进一步选择合理的计算方法进行计算解答问题。", "solution": "null", "level": "五年级", "question": "一个长方体铁皮通风管长 2 米, 管口是周长为 12 分米的正方形, 做这个通风管至少需要多少平方米的铁皮?", "options": [], "subject": "立体几何学", "analysis": "解 2.4 平方米\n\n【分析】由于通风管没有底面, 所以只求它的侧面积即可, 长方体的侧面积 $=$ 底面周长 $\\times$ 高, 据此列式解答。\n\n【详解】12 分米 $=1.2$ 米\n\n$1.2 \\times 2=2.4$ (平方米)\n\n答:做这个通风管至少需要 2.4 平方米的铁皮。\n\n【点睛】解答有关长方体计算的实际问题,一定要搞清所求的是什么(体积、表面积还是几个面的面积),再进一步选择合理的计算方法进行计算解答问题。"} {"id": "24258", "image": [], "answer": "解25.5 平方厘米或 45.5 平方厘米\n\n【分析】一个数只有 1 和它本身两个因数, 这个数叫做质数。先把 20 拆分成两个质数相加, 这两个质数就是直角三角形的底和高, 最后根据三角形的面积公式求出直角三角形的面积。\n\n【详解】 $20=3+17=7+13$\n\n所以这个直角三角形的两条直角边分别是 3 厘米和 17 厘米, 或者是 7 厘米和 13 厘米;\n\n$3 \\times 17 \\div 2$\n\n$=51 \\div 2$\n\n$=25.5$ (平方厘米)\n\n$7 \\times 13 \\div 2$\n\n$=91 \\div 2$\n\n$=45.5$ (平方厘米)\n\n答:这个直角三角形的面积是 25.5 平方厘米或 45.5 平方厘米。\n\n【点睛】本题考查了质数的认识以及三角形面积公式的灵活应用。", "solution": "null", "level": "五年级", "question": "有一个直角三角形, 两条直角边的长的数值是两个质数, 它们的和是 20 厘米, 这个直角三角形的面积是多少平方厘米?", "options": [], "subject": "立体几何学", "analysis": "解25.5 平方厘米或 45.5 平方厘米\n\n【分析】一个数只有 1 和它本身两个因数, 这个数叫做质数。先把 20 拆分成两个质数相加, 这两个质数就是直角三角形的底和高, 最后根据三角形的面积公式求出直角三角形的面积。\n\n【详解】 $20=3+17=7+13$\n\n所以这个直角三角形的两条直角边分别是 3 厘米和 17 厘米, 或者是 7 厘米和 13 厘米;\n\n$3 \\times 17 \\div 2$\n\n$=51 \\div 2$\n\n$=25.5$ (平方厘米)\n\n$7 \\times 13 \\div 2$\n\n$=91 \\div 2$\n\n$=45.5$ (平方厘米)\n\n答:这个直角三角形的面积是 25.5 平方厘米或 45.5 平方厘米。\n\n【点睛】本题考查了质数的认识以及三角形面积公式的灵活应用。"} {"id": "24512", "image": [], "answer": "解$\\times$\n\n【分析】周长相等的长方形和正方形, 正方形的面积比长方形的面积大,长方体和正方体的体积都可以按照底面积乘高来算, 高相同, 底面积越大, 体积越大。\n\n【详解】长方体和正方体的体积都能用 $\\mathrm{V}=\\mathrm{Sh}$ 计算, 因为它们的高相等, 而周长相等的正方形和长方形相比, 正方形的面积大于长方形的面积, 所以底面周长和高都相等的长方体与正方体相比, 正方体的体积大。所以原题的说法是错误的。\n\n故答案为: $\\times$\n\n【点睛】此题考查长方体和正方体体积的求法, 掌握周长相等的长方形和正方形, 正方形的面积比长方形的面积大也是解题的关键。", "solution": "null", "level": "五年级", "question": "一个长方体和正方体的底面周长和高都相等,那么它们的体积也相等。( )", "options": [], "subject": "立体几何学", "analysis": "解$\\times$\n\n【分析】周长相等的长方形和正方形, 正方形的面积比长方形的面积大,长方体和正方体的体积都可以按照底面积乘高来算, 高相同, 底面积越大, 体积越大。\n\n【详解】长方体和正方体的体积都能用 $\\mathrm{V}=\\mathrm{Sh}$ 计算, 因为它们的高相等, 而周长相等的正方形和长方形相比, 正方形的面积大于长方形的面积, 所以底面周长和高都相等的长方体与正方体相比, 正方体的体积大。所以原题的说法是错误的。\n\n故答案为: $\\times$\n\n【点睛】此题考查长方体和正方体体积的求法, 掌握周长相等的长方形和正方形, 正方形的面积比长方形的面积大也是解题的关键。"} {"id": "24513", "image": [], "answer": "解$\\sqrt{ }$\n\n【分析】根据容积的意义, 某容器所能容纳别的物体的体积叫做这个容器的容积。计量容积, 一般就\n用体积单位, 计算容器的容积一般是从容器的里面测量, 容器的体积是从外面测量。据此判断即可。\n\n【详解】由分析可知:\n\n计量容积一般使用体积单位, 但容器的容积不一定等于它的体积, 因为容器是有厚度的。所以原题干说法正确。\n\n故答案为:\n\n【点睛】本题考查容积和体积, 明确容积是从里面测量而体积是从外面测量是解题的关键。", "solution": "null", "level": "五年级", "question": "计量容积一般使用体积单位, 但容器的容积不一定等于它的体积。( )", "options": [], "subject": "立体几何学", "analysis": "解$\\sqrt{ }$\n\n【分析】根据容积的意义, 某容器所能容纳别的物体的体积叫做这个容器的容积。计量容积, 一般就\n用体积单位, 计算容器的容积一般是从容器的里面测量, 容器的体积是从外面测量。据此判断即可。\n\n【详解】由分析可知:\n\n计量容积一般使用体积单位, 但容器的容积不一定等于它的体积, 因为容器是有厚度的。所以原题干说法正确。\n\n故答案为:\n\n【点睛】本题考查容积和体积, 明确容积是从里面测量而体积是从外面测量是解题的关键。"} {"id": "24514", "image": ["12515.jpg", "12516.jpg", "12517.jpg", "12515.jpg", "12516.jpg", "12517.jpg"], "answer": "解$\\times$\n\n【分析】两个完全一样的正方体可以拼成一个长方体,两个正方体所占空间的大小等于长方体所占空间的大小, 则长方体的体积等于两个正方体的体积之和; 长方体的表面积比原来两个正方体的表面积之和减少 2 个正方形的面积, 则表面积比原来减少了, 据此解答。\n\n【详解】假设小正方体棱长为 1 厘米。\n\n\n\n1 厘米\n\n\n\n1 厘米\n\n\n\n2厘米\n1 厘米\n\n1 厘米\n\n体积: $1 \\times 1 \\times 1 \\times 2=2$ (立方厘米)\n\n$2 \\times 1 \\times 1=2$ (立方厘米)\n\n因为 2 立方厘米 $=2$ 立方厘米, 所以体积总和不变。\n\n表面积: $1 \\times 1 \\times 6 \\times 2$\n\n$=6 \\times 2$\n\n$=12$ (平方厘米)\n\n$(2 \\times 1+1 \\times 1+2 \\times 1) \\times 2$\n\n$=(2+1+2) \\times 2$\n\n$=5 \\times 2$\n\n$=10$ (平方厘米)\n\n因为 12 平方厘米 $>10$ 平方厘米,所以表面积比原来减少了。\n\n故答案为: $\\times$\n\n【点睛】掌握长方体、正方体的表面积和体积的计算公式是解答题目的关键。", "solution": "null", "level": "五年级", "question": "将两个相同的正方体拼在一起, 体积总和不变, 表面积总和也不变。( )", "options": [], "subject": "立体几何学", "analysis": "解$\\times$\n\n【分析】两个完全一样的正方体可以拼成一个长方体,两个正方体所占空间的大小等于长方体所占空间的大小, 则长方体的体积等于两个正方体的体积之和; 长方体的表面积比原来两个正方体的表面积之和减少 2 个正方形的面积, 则表面积比原来减少了, 据此解答。\n\n【详解】假设小正方体棱长为 1 厘米。\n\n\n\n1 厘米\n\n\n\n1 厘米\n\n\n\n2厘米\n1 厘米\n\n1 厘米\n\n体积: $1 \\times 1 \\times 1 \\times 2=2$ (立方厘米)\n\n$2 \\times 1 \\times 1=2$ (立方厘米)\n\n因为 2 立方厘米 $=2$ 立方厘米, 所以体积总和不变。\n\n表面积: $1 \\times 1 \\times 6 \\times 2$\n\n$=6 \\times 2$\n\n$=12$ (平方厘米)\n\n$(2 \\times 1+1 \\times 1+2 \\times 1) \\times 2$\n\n$=(2+1+2) \\times 2$\n\n$=5 \\times 2$\n\n$=10$ (平方厘米)\n\n因为 12 平方厘米 $>10$ 平方厘米,所以表面积比原来减少了。\n\n故答案为: $\\times$\n\n【点睛】掌握长方体、正方体的表面积和体积的计算公式是解答题目的关键。"} {"id": "24516", "image": [], "answer": "解$\\sqrt{ }$\n\n【分析】根据正方体的总棱长 $=$ 棱长 $\\times 12$, 据此可求出正方体的 1 条棱长, 再根据正方体的体积 $=$ 棱长 $\\times$ 棱长 $\\times$ 棱长, 据此可求出正方体的体积。\n【详解】 $12 \\div 12=1(\\mathrm{~cm})$\n\n$1 \\times 1 \\times 1=1\\left(\\mathrm{~cm}^{3}\\right)$\n\n所以原题干说法正确。\n\n故答案为: $\\sqrt{ }$\n\n【点睛】本题考查正方体的体积, 熟记公式是解题的关键。", "solution": "null", "level": "五年级", "question": "一个正方体的棱长之和是 $12 \\mathrm{~cm}$, 体积是 $1 \\mathrm{~cm}^{3}$ 。( )", "options": [], "subject": "立体几何学", "analysis": "解$\\sqrt{ }$\n\n【分析】根据正方体的总棱长 $=$ 棱长 $\\times 12$, 据此可求出正方体的 1 条棱长, 再根据正方体的体积 $=$ 棱长 $\\times$ 棱长 $\\times$ 棱长, 据此可求出正方体的体积。\n【详解】 $12 \\div 12=1(\\mathrm{~cm})$\n\n$1 \\times 1 \\times 1=1\\left(\\mathrm{~cm}^{3}\\right)$\n\n所以原题干说法正确。\n\n故答案为: $\\sqrt{ }$\n\n【点睛】本题考查正方体的体积, 熟记公式是解题的关键。"} {"id": "24517", "image": [], "answer": "解$\\sqrt{ }$\n\n【分析】根据等积类应用题的基本关系式: 变形前的体积=变形后的体积,所以这个面团无论变成什么形状,它的体积都不会改变。据此解答。\n\n【详解】根据分析得, 把一块面团揉成长条后再压扁, 面团的体积不变。这种说法是正确的。\n\n故答案为: $\\sqrt{ }$\n\n【点睛】此题的解题关键是通过转化的数学思想, 理解体积的等积变形。", "solution": "null", "level": "五年级", "question": "把一块面团揉成长条后再压扁, 面团的体积不变。( )", "options": [], "subject": "立体几何学", "analysis": "解$\\sqrt{ }$\n\n【分析】根据等积类应用题的基本关系式: 变形前的体积=变形后的体积,所以这个面团无论变成什么形状,它的体积都不会改变。据此解答。\n\n【详解】根据分析得, 把一块面团揉成长条后再压扁, 面团的体积不变。这种说法是正确的。\n\n故答案为: $\\sqrt{ }$\n\n【点睛】此题的解题关键是通过转化的数学思想, 理解体积的等积变形。"} {"id": "24520", "image": ["12520.jpg"], "answer": "解 128 平方厘米\n\n【分析】根据题意, 把一个长方体木块锯成两块后, 表面积增加 2 个截面的面积, 截面是一个边长为 8 厘米的正方形,根据正方形的面积公式求出其中一个截面的面积,再乘 2 即可求出木块的表面积增加的面积。\n\n【详解】 $8 \\times 8 \\times 2=128$ (平方厘米)\n\n答:木块的表面积增加 128 平方厘米。\n\n【点睛】掌握长方体切割的特点, 明确增加的表面积是哪些面的面积, 以此为突破口, 利用公式列式计算。", "solution": "null", "level": "五年级", "question": "如图, 木块从中间锯成两块后,木块的表面积增加多少平方厘米?\n\n", "options": [], "subject": "立体几何学", "analysis": "解 128 平方厘米\n\n【分析】根据题意, 把一个长方体木块锯成两块后, 表面积增加 2 个截面的面积, 截面是一个边长为 8 厘米的正方形,根据正方形的面积公式求出其中一个截面的面积,再乘 2 即可求出木块的表面积增加的面积。\n\n【详解】 $8 \\times 8 \\times 2=128$ (平方厘米)\n\n答:木块的表面积增加 128 平方厘米。\n\n【点睛】掌握长方体切割的特点, 明确增加的表面积是哪些面的面积, 以此为突破口, 利用公式列式计算。"} {"id": "24522", "image": ["12521.jpg"], "answer": "解18000 立方分米\n\n【分析】将古城墙分成两个长方体, 古城墙的体积 $=$ 两个长方体体积之和, 长方体体积 $=$ 长 $\\times$ 宽 $\\times$ 高,据此列式解答。\n\n【详解】 $3 \\times 1 \\times 2+4 \\times 3 \\times 1$\n\n$=6+12$\n\n$=18$ (立方米)\n\n$=18000$ (立方分米)\n\n答:这段古城墙的体积是 18000 立方分米。\n\n【点睛】关键是掌握并灵活运用长方体体积公式。", "solution": "null", "level": "五年级", "question": "永年区有一段古城墙(如图所示), 这段古城墙的体积是多少立方分米?\n\n", "options": [], "subject": "立体几何学", "analysis": "解18000 立方分米\n\n【分析】将古城墙分成两个长方体, 古城墙的体积 $=$ 两个长方体体积之和, 长方体体积 $=$ 长 $\\times$ 宽 $\\times$ 高,据此列式解答。\n\n【详解】 $3 \\times 1 \\times 2+4 \\times 3 \\times 1$\n\n$=6+12$\n\n$=18$ (立方米)\n\n$=18000$ (立方分米)\n\n答:这段古城墙的体积是 18000 立方分米。\n\n【点睛】关键是掌握并灵活运用长方体体积公式。"} {"id": "24523", "image": [], "answer": "解286 平方厘米\n\n【分析】根据长方体的棱长总和 $=($ 长十宽十高) $\\times 4$ 可知, 长方体的高 $=$ 棱长总和 $\\div 4-$ 长一宽, 求\n出长方体的高; 再根据长方体的表面积 $=($ 长 $\\times$ 宽十长 $\\times$ 高十宽 $\\times$ 高 $) \\times 2$, 代入数据计算, 即可求出这个长方体的表面积。\n\n【详解】高:\n\n$84 \\div 4-9-7$\n\n$=21-9-7$\n\n$=5$ (厘米)\n\n表面积:\n\n$(9 \\times 7+9 \\times 5+7 \\times 5) \\times 2$\n\n$=(63+45+35) \\times 2$\n\n$=143 \\times 2$\n\n$=286$ (平方厘米)\n\n答: 它的表面积是 286 平方厘米。\n\n【点睛】本题考查长方体的棱长总和、表面积公式的灵活运用, 利用长方体的棱长总和公式, 求出长方体的高是解题的关键。", "solution": "null", "level": "五年级", "question": "一个长方体的棱长和是 84 厘米, 它的长是 9 厘米, 宽 7 厘米, 它的表面积是多少平方厘米?", "options": [], "subject": "立体几何学", "analysis": "解286 平方厘米\n\n【分析】根据长方体的棱长总和 $=($ 长十宽十高) $\\times 4$ 可知, 长方体的高 $=$ 棱长总和 $\\div 4-$ 长一宽, 求\n出长方体的高; 再根据长方体的表面积 $=($ 长 $\\times$ 宽十长 $\\times$ 高十宽 $\\times$ 高 $) \\times 2$, 代入数据计算, 即可求出这个长方体的表面积。\n\n【详解】高:\n\n$84 \\div 4-9-7$\n\n$=21-9-7$\n\n$=5$ (厘米)\n\n表面积:\n\n$(9 \\times 7+9 \\times 5+7 \\times 5) \\times 2$\n\n$=(63+45+35) \\times 2$\n\n$=143 \\times 2$\n\n$=286$ (平方厘米)\n\n答: 它的表面积是 286 平方厘米。\n\n【点睛】本题考查长方体的棱长总和、表面积公式的灵活运用, 利用长方体的棱长总和公式, 求出长方体的高是解题的关键。"} {"id": "24524", "image": [], "answer": "解36 立方分米\n\n【分析】根据不规则物体的体积 $=$ 容器的底面积 $\\times$ 水面上升的高度, 据此进行计算即可。\n\n【详解】 $6 \\times 4 \\times(4.5-3)$\n\n$=24 \\times 1.5$\n\n$=36$ (立方分米)\n\n答:这个石块的体积是 36 立方分米。\n\n【点睛】本题考查求不规则物体的体积, 明确求不规则物体的体积的计算方法是解题的关键。", "solution": "null", "level": "五年级", "question": "有一个长方体玻璃缸, 从里面量长 6 分米、宽 4 分米、高 5 分米, 里面注入了一些水, 水深 3 分米。如果把一个石块完全浸没在水中, 这时水深 4.5 分米。这个石块的体积是多少立方分米?", "options": [], "subject": "立体几何学", "analysis": "解36 立方分米\n\n【分析】根据不规则物体的体积 $=$ 容器的底面积 $\\times$ 水面上升的高度, 据此进行计算即可。\n\n【详解】 $6 \\times 4 \\times(4.5-3)$\n\n$=24 \\times 1.5$\n\n$=36$ (立方分米)\n\n答:这个石块的体积是 36 立方分米。\n\n【点睛】本题考查求不规则物体的体积, 明确求不规则物体的体积的计算方法是解题的关键。"} {"id": "24537", "image": [], "answer": "解$\\sqrt{ }$\n\n【分析】体积是指物体所占空间的大小; 物体的表面或封闭图形的大小, 叫做它们的面积, 据此分析。\n\n【详解】体积和面积是两个不同的概念, 无法进行比较, 原题说法正确。\n\n故答案为: $\\sqrt{ }$\n\n【点睛】关键是理解体积和面积的含义, 理解它们之间的区别。", "solution": "null", "level": "五年级", "question": "“体积单位比面积单位大”这种说法是错误的。()", "options": [], "subject": "立体几何学", "analysis": "解$\\sqrt{ }$\n\n【分析】体积是指物体所占空间的大小; 物体的表面或封闭图形的大小, 叫做它们的面积, 据此分析。\n\n【详解】体积和面积是两个不同的概念, 无法进行比较, 原题说法正确。\n\n故答案为: $\\sqrt{ }$\n\n【点睛】关键是理解体积和面积的含义, 理解它们之间的区别。"} {"id": "24539", "image": ["12535.jpg", "12536.jpg", "12536.jpg"], "answer": "解$\\sqrt{ }$\n\n【分析】根据正方体展开图的 11 种特征, 此图属于正方体展开图的“ $1-4-1$ ”型, 折成正方体后, 左边的“好”与“学”相对, 右边的“好”与“习”相对, “向”与“上”相对。\n\n【详解】如图:\n\n\n\n是一个正方体的展开图, 与“上”字相对面上的字是“向”。\n\n原题说法正确。\n\n故答案为: $\\sqrt{ }$\n\n【点睛】根据正方体的特征:相对的面不相邻,同时结合空间想象力,找到展开图每个面的相对面是解题的关键。", "solution": "null", "level": "五年级", "question": "如图是一个正方体的展开图,与“上”字相对面上的字是“向”。()\n\n", "options": [], "subject": "立体几何学", "analysis": "解$\\sqrt{ }$\n\n【分析】根据正方体展开图的 11 种特征, 此图属于正方体展开图的“ $1-4-1$ ”型, 折成正方体后, 左边的“好”与“学”相对, 右边的“好”与“习”相对, “向”与“上”相对。\n\n【详解】如图:\n\n\n\n是一个正方体的展开图, 与“上”字相对面上的字是“向”。\n\n原题说法正确。\n\n故答案为: $\\sqrt{ }$\n\n【点睛】根据正方体的特征:相对的面不相邻,同时结合空间想象力,找到展开图每个面的相对面是解题的关键。"} {"id": "24540", "image": [], "answer": "解$\\times$\n\n【分析】一个长方体切成两个体积相等的正方体后, 两个正方体的表面积之和比原来长方体的表面积增加正方体两个面的面积, 据此判断。\n\n【详解】一个长方体切成两个体积相等的正方体后, 每个正方体的表面积不是原来长方体表面积的 $\\frac{1}{2}$,原题说法错误。\n\n故答案为: $\\times$\n\n【点睛】本题考查立体图形的切拼, 明确一个长方体切成两个正方体后表面积的变化是解题的关键。", "solution": "null", "level": "五年级", "question": "一个长方体切成两个体积相等的正方体后, 每个正方体的表面积是原来长方体表面积的 $\\frac{1}{2}$ 。", "options": [], "subject": "立体几何学", "analysis": "解$\\times$\n\n【分析】一个长方体切成两个体积相等的正方体后, 两个正方体的表面积之和比原来长方体的表面积增加正方体两个面的面积, 据此判断。\n\n【详解】一个长方体切成两个体积相等的正方体后, 每个正方体的表面积不是原来长方体表面积的 $\\frac{1}{2}$,原题说法错误。\n\n故答案为: $\\times$\n\n【点睛】本题考查立体图形的切拼, 明确一个长方体切成两个正方体后表面积的变化是解题的关键。"} {"id": "24541", "image": [], "answer": "解$\\sqrt{ }$\n\n【分析】用除法分别求出外包装盒的长、宽、高里各有几个橡皮的长、宽、高, 再利用长方体的体积公式 $\\mathrm{V}=\\mathrm{abh}$ 把各边可以放的个数相乘, 即可求出外包装盒最多能放橡皮的个数。\n\n【详解】 $20 \\div 4=5$ (个)\n\n$12 \\div 3=4$ (个)\n\n$3 \\div 1=3$ (个)\n\n最多能放:\n\n$5 \\times 4 \\times 3$\n\n$=20 \\times 3$\n\n$=60$ (个)\n\n这个橡皮的外包装盒最多能放 60 个橡皮, 原题说法正确。\n\n故答案为: $\\sqrt{ }$\n\n【点睛】本题考查“包含”除法的意义及应用, 灵活运用长方体的体积计算公式是解题的关键。", "solution": "null", "level": "五年级", "question": "一块橡皮长为 $4 \\mathrm{~cm}$, 宽为 $3 \\mathrm{~cm}$, 高为 $1 \\mathrm{~cm}$ 。现有一个橡皮的外包装盒, 从里面量长为 $20 \\mathrm{~cm}$, 宽为 $12 \\mathrm{~cm}$, 高为 $3 \\mathrm{~cm}$ 。这个橡皮的外包装盒最多能放 60 个橡皮。( )", "options": [], "subject": "立体几何学", "analysis": "解$\\sqrt{ }$\n\n【分析】用除法分别求出外包装盒的长、宽、高里各有几个橡皮的长、宽、高, 再利用长方体的体积公式 $\\mathrm{V}=\\mathrm{abh}$ 把各边可以放的个数相乘, 即可求出外包装盒最多能放橡皮的个数。\n\n【详解】 $20 \\div 4=5$ (个)\n\n$12 \\div 3=4$ (个)\n\n$3 \\div 1=3$ (个)\n\n最多能放:\n\n$5 \\times 4 \\times 3$\n\n$=20 \\times 3$\n\n$=60$ (个)\n\n这个橡皮的外包装盒最多能放 60 个橡皮, 原题说法正确。\n\n故答案为: $\\sqrt{ }$\n\n【点睛】本题考查“包含”除法的意义及应用, 灵活运用长方体的体积计算公式是解题的关键。"} {"id": "24546", "image": [], "answer": "解40 立方分米\n【分析】根据长方体的特征可知, 底面的长和宽都为 4 分米, 高为 2.5 分米, 根据长方体的体积公式: $\\mathrm{V}=\\mathrm{abh}$, 把数据代入即可求出这个礼品盒的体积。\n\n【详解】 $4 \\times 4 \\times 2.5=40$ (立方分米)\n\n答: 这个礼品盒的体积是 40 立方分米。\n\n【点睛】此题的解题关键是熟练运用长方体的体积公式求解。", "solution": "null", "level": "五年级", "question": "一底面为正方形的礼品盒, 底面边长 4 分米, 高 2.5 分米。这个礼品盒的体积是多少立方分米?", "options": [], "subject": "立体几何学", "analysis": "解40 立方分米\n【分析】根据长方体的特征可知, 底面的长和宽都为 4 分米, 高为 2.5 分米, 根据长方体的体积公式: $\\mathrm{V}=\\mathrm{abh}$, 把数据代入即可求出这个礼品盒的体积。\n\n【详解】 $4 \\times 4 \\times 2.5=40$ (立方分米)\n\n答: 这个礼品盒的体积是 40 立方分米。\n\n【点睛】此题的解题关键是熟练运用长方体的体积公式求解。"} {"id": "24547", "image": ["12540.jpg", "12541.jpg"], "answer": "解10 厘米\n\n【分析】根据题干分析可得, 可设两个容器的水深相同为 $\\mathrm{x}$ 厘米, 根据长方体的体积公式可知, 甲容器中的水的体积是: $50 \\times 40 \\times \\mathrm{x}$ 立方厘米;乙容器中的水的体积是 $40 \\times 25 \\times \\mathrm{x}$ 立方厘米, 根据两个容器内水的体积之和等于乙容器中高为 30 厘米时的水的体积, 即可列出方程, 求出 $\\mathrm{x}$ 的值即可解答问题。\n\n【详解】解:设两个容器的水深相同为 $\\mathrm{x}$ 厘米, 根据题意可得方程:\n\n$50 \\times 40 \\times x+40 \\times 25 \\times x=40 \\times 25 \\times 30$\n\n$2000 x+1000 x=30000$\n\n$3000 x=30000$\n\n$x=30000 \\div 3000$\n\n$\\mathrm{x}=10$\n\n答: 这时两个容器水深是 10 厘米。\n\n【点睛】此题考查了长方体的体积公式的计算应用,抓住水的体积不变列出方程解决问题。", "solution": "null", "level": "五年级", "question": "如图所示, 甲是空的长方体容器, 乙长方体容器中水深 30 厘米, 要将容器乙中的水倒一部分给\n容器甲, 使两个长方体容器中水的高度相同, 这时水深多少厘米?\n\n\n\n甲\n\n\n\n乙", "options": [], "subject": "立体几何学", "analysis": "解10 厘米\n\n【分析】根据题干分析可得, 可设两个容器的水深相同为 $\\mathrm{x}$ 厘米, 根据长方体的体积公式可知, 甲容器中的水的体积是: $50 \\times 40 \\times \\mathrm{x}$ 立方厘米;乙容器中的水的体积是 $40 \\times 25 \\times \\mathrm{x}$ 立方厘米, 根据两个容器内水的体积之和等于乙容器中高为 30 厘米时的水的体积, 即可列出方程, 求出 $\\mathrm{x}$ 的值即可解答问题。\n\n【详解】解:设两个容器的水深相同为 $\\mathrm{x}$ 厘米, 根据题意可得方程:\n\n$50 \\times 40 \\times x+40 \\times 25 \\times x=40 \\times 25 \\times 30$\n\n$2000 x+1000 x=30000$\n\n$3000 x=30000$\n\n$x=30000 \\div 3000$\n\n$\\mathrm{x}=10$\n\n答: 这时两个容器水深是 10 厘米。\n\n【点睛】此题考查了长方体的体积公式的计算应用,抓住水的体积不变列出方程解决问题。"} {"id": "24548", "image": [], "answer": "解5600 千克\n\n【分析】根据正方体的容积公式: $\\mathrm{V}=\\mathrm{a}^{3}$, 据此求出油箱的容积, 然后用油箱的容积乘每升汽油的重量即可。\n\n【详解】 $2 \\times 2 \\times 2=8$ (立方米) $=8000$ (升)\n\n$8000 \\times 0.7=5600$ (千克)\n\n答:这箱汽油一共重 5600 千克。\n\n【点睛】本题考查正方体的容积, 熟记公式是解题的关键。", "solution": "null", "level": "五年级", "question": "给一个棱长为 2 米的正方体油箱装满汽油(油箱壁厚度不计), 如果每升汽油重 0.7 千克, 这箱汽油一共重多少千克?", "options": [], "subject": "立体几何学", "analysis": "解5600 千克\n\n【分析】根据正方体的容积公式: $\\mathrm{V}=\\mathrm{a}^{3}$, 据此求出油箱的容积, 然后用油箱的容积乘每升汽油的重量即可。\n\n【详解】 $2 \\times 2 \\times 2=8$ (立方米) $=8000$ (升)\n\n$8000 \\times 0.7=5600$ (千克)\n\n答:这箱汽油一共重 5600 千克。\n\n【点睛】本题考查正方体的容积, 熟记公式是解题的关键。"} {"id": "24549", "image": ["12542.jpg"], "answer": "解不会\n\n【分析】利用长方体的体积公式“ $V=a b h$ ”分别求出杯中无水部分的体积和冰糖的体积,最后比较大小,据此解答。\n\n【详解】 $5 \\times 3 \\times(8-7)$\n\n$=5 \\times 3 \\times 1$\n\n$=15$ (立方厘米)\n\n$3 \\times 3 \\times 1=9$ (立方厘米)\n因为 15 立方厘米 $>9$ 立方厘米, 所以水不会溢出来。\n\n答: 水不会溢出来。\n\n【点睛】掌握长方体的体积计算公式是解答题目的关键。", "solution": "null", "level": "五年级", "question": "壮壮给奶奶倒了一杯水(如图),还想放入一块长 3 厘米、宽 3 厘米、高 1 厘米的冰糖。(杯壁厚度忽略不计)\n\n", "options": [], "subject": "立体几何学", "analysis": "解不会\n\n【分析】利用长方体的体积公式“ $V=a b h$ ”分别求出杯中无水部分的体积和冰糖的体积,最后比较大小,据此解答。\n\n【详解】 $5 \\times 3 \\times(8-7)$\n\n$=5 \\times 3 \\times 1$\n\n$=15$ (立方厘米)\n\n$3 \\times 3 \\times 1=9$ (立方厘米)\n因为 15 立方厘米 $>9$ 立方厘米, 所以水不会溢出来。\n\n答: 水不会溢出来。\n\n【点睛】掌握长方体的体积计算公式是解答题目的关键。"} {"id": "24055", "image": [], "answer": "解 所有的质数不一定都是奇数, 所有的奇数不一定都是质数, 如: 2 是质数, 但不是\n奇数; 1 是奇数, 但不是质数。\n\n原题说法错误。\n\n故答案为: $\\times$ 。", "solution": "null", "level": "五年级", "question": "(2022 秋 ・市中区期末)所有的质数都是奇数, 所有的奇数都是质数。", "options": [], "subject": "逻辑题", "analysis": "解 所有的质数不一定都是奇数, 所有的奇数不一定都是质数, 如: 2 是质数, 但不是\n奇数; 1 是奇数, 但不是质数。\n\n原题说法错误。\n\n故答案为: $\\times$ 。"} {"id": "24057", "image": [], "answer": "解由分析可知: 一个数可以既是合数又是质数说法错误。\n\n故答案为: $\\times$ 。", "solution": "null", "level": "五年级", "question": "(2022 秋・茂名期末)一个数可以既是合数又是质数。", "options": [], "subject": "代数", "analysis": "解由分析可知: 一个数可以既是合数又是质数说法错误。\n\n故答案为: $\\times$ 。"} {"id": "24619", "image": [], "answer": "解$\\times$\n\n【分析】自然数是从 0 开始的整数, 比较因数的个数, 举两个例子推翻即可。\n\n【详解】例如 12 的因数有:1、2、3、4、6、12。13 的因数有:1、13。\n\n因为 $13>12$, 但 12 的因数比 13 多。\n\n故答案为: $\\times$\n\n【点睛】本题考查因数的定义, 根据定义举例子推翻原题即可。判断题有时可以根据定义或法则直接判断对错,也可以找到符合条件的例子,推翻原题。", "solution": "null", "level": "五年级", "question": "一个自然数越大, 它的因数个数就越多。()", "options": [], "subject": "代数", "analysis": "解$\\times$\n\n【分析】自然数是从 0 开始的整数, 比较因数的个数, 举两个例子推翻即可。\n\n【详解】例如 12 的因数有:1、2、3、4、6、12。13 的因数有:1、13。\n\n因为 $13>12$, 但 12 的因数比 13 多。\n\n故答案为: $\\times$\n\n【点睛】本题考查因数的定义, 根据定义举例子推翻原题即可。判断题有时可以根据定义或法则直接判断对错,也可以找到符合条件的例子,推翻原题。"} {"id": "25223", "image": [], "answer": "解$\\times$\n\n【分析】根据 1 立方米 $=1000$ 立方分米, 1 立方分米 $=1$ 升, 1 升 $=1000$ 毫升, 可知 1 立方米 $=1000000$毫升,据此判断。\n\n【详解】1立方米 $=1000000$ 毫升\n\n毫升和立方米之间的进率是 1000000 。\n\n原题说法错误。\n\n故答案为: $\\times$\n\n【点睛】熟练掌握体积和容积的单位换算是解题的关键。", "solution": "null", "level": "五年级", "question": "毫升和立方米之间的进率是 10000。()", "options": [], "subject": "代数", "analysis": "解$\\times$\n\n【分析】根据 1 立方米 $=1000$ 立方分米, 1 立方分米 $=1$ 升, 1 升 $=1000$ 毫升, 可知 1 立方米 $=1000000$毫升,据此判断。\n\n【详解】1立方米 $=1000000$ 毫升\n\n毫升和立方米之间的进率是 1000000 。\n\n原题说法错误。\n\n故答案为: $\\times$\n\n【点睛】熟练掌握体积和容积的单位换算是解题的关键。"} {"id": "24061", "image": [], "answer": "解 由分析可得:\n\n$32=1 \\times 32$, 即每行 1 人, 排 32 行, 不符合题意; 或者每行 32 人, 排 1 行, 不符合题意。\n\n$32=2 \\times 16$, 即每行 2 人, 排 16 行; 或每行 16 人,排 2 行;\n\n$32=4 \\times 8$, 即每行 4 人, 排 8 行; 或每行 8 人, 排 4 行;\n\n答: 可以排 2 行、 4 行、 8 行、 16 行。共有 4 种排法。", "solution": "null", "level": "五年级", "question": "(2021 秋・略阳县期末)五(1)班有 32 名同学参加广播操比赛,要使每行人数都相等,可以排几行? 共有几种排法?(每行或每列不少于 2 人)", "options": [], "subject": "计数", "analysis": "解 由分析可得:\n\n$32=1 \\times 32$, 即每行 1 人, 排 32 行, 不符合题意; 或者每行 32 人, 排 1 行, 不符合题意。\n\n$32=2 \\times 16$, 即每行 2 人, 排 16 行; 或每行 16 人,排 2 行;\n\n$32=4 \\times 8$, 即每行 4 人, 排 8 行; 或每行 8 人, 排 4 行;\n\n答: 可以排 2 行、 4 行、 8 行、 16 行。共有 4 种排法。"} {"id": "24235", "image": [], "answer": "解可以分为 $3 、 4 、 6$ 人为一组。\n\n【分析】先找出 48 的所有因数, 只要大于 2 小于 8 的因数都可以作为每组人数, 列乘法算式找因数,按照从小到大的顺序,一组一组地写出所有积是这个数的乘法算式,乘法算式中的两个因数就是这个数的因数。\n\n【详解】 $48=1 \\times 48=2 \\times 24=3 \\times 16=4 \\times 12=6 \\times 8$\n\n48 的因数有 $1 、 2 、 3 、 4 、 6 、 8 、 12 、 16 、 24 、 48$, 比 2 大比 8 小的有 $3 、 4 、 6$ 。\n\n答: 可以分为 $3 、 4 、 6$ 人为一组。\n\n【点睛】一个数的因数的个数是有限的, 最小的因数是 1 , 最大的因数是它本身。", "solution": "null", "level": "五年级", "question": "五(9)班有学生 48 人,实验课上,老师要把他们平均分组,每组多于 2 人,少于 8 人,可以怎样分组?", "options": [], "subject": "计数", "analysis": "解可以分为 $3 、 4 、 6$ 人为一组。\n\n【分析】先找出 48 的所有因数, 只要大于 2 小于 8 的因数都可以作为每组人数, 列乘法算式找因数,按照从小到大的顺序,一组一组地写出所有积是这个数的乘法算式,乘法算式中的两个因数就是这个数的因数。\n\n【详解】 $48=1 \\times 48=2 \\times 24=3 \\times 16=4 \\times 12=6 \\times 8$\n\n48 的因数有 $1 、 2 、 3 、 4 、 6 、 8 、 12 、 16 、 24 、 48$, 比 2 大比 8 小的有 $3 、 4 、 6$ 。\n\n答: 可以分为 $3 、 4 、 6$ 人为一组。\n\n【点睛】一个数的因数的个数是有限的, 最小的因数是 1 , 最大的因数是它本身。"} {"id": "24310", "image": [], "answer": "解见详解\n\n【分析】一瓶饮料是 3 元, 那么最后的花销一定是 3 的倍数, 3 的倍数特征为各个数位上的数字相加起来的和是 3 的倍数。\n\n【详解】我认为小英猜得对。因为 1 瓶饮料 3 元, 老师花的钱数应是 3 的倍数。一个数各个数位上的数字相加的和是 3 的倍数, 这个数就是 3 的倍数, 小英: $1+2+9=12$, 小东: $9+7=16$, 小芳: $1+4+3=8$ 。\n只有小英猜的钱数是 3 的倍数。\n\n答: 因为小英说的钱数是 3 的倍数,所以是小英说的对。\n\n【点睛】此题考查 3 的倍数的应用, 熟练掌握 3 的倍数特征是解题的关键。", "solution": "null", "level": "五年级", "question": "五(1)班的同学去春游, 小刚说: “老师为我们每位同学买了 1 瓶 3 元的饮料, 请大家猜一猜,一共花了多少钱?”英说: \"129 元。”小东说: “97 元。”小芳说:“143 元。”小明说:“你们中只有一个人猜对了! ”你认为准猜得对呢? 为什么?", "options": [], "subject": "计数", "analysis": "解见详解\n\n【分析】一瓶饮料是 3 元, 那么最后的花销一定是 3 的倍数, 3 的倍数特征为各个数位上的数字相加起来的和是 3 的倍数。\n\n【详解】我认为小英猜得对。因为 1 瓶饮料 3 元, 老师花的钱数应是 3 的倍数。一个数各个数位上的数字相加的和是 3 的倍数, 这个数就是 3 的倍数, 小英: $1+2+9=12$, 小东: $9+7=16$, 小芳: $1+4+3=8$ 。\n只有小英猜的钱数是 3 的倍数。\n\n答: 因为小英说的钱数是 3 的倍数,所以是小英说的对。\n\n【点睛】此题考查 3 的倍数的应用, 熟练掌握 3 的倍数特征是解题的关键。"} {"id": "24593", "image": [], "answer": "解把 1 升的水倒入空碗中结果溢出, 说明碗的容量比 1 升小, 所以原题说法错误。故答案为: $\\times$ 。", "solution": "null", "level": "五年级", "question": "(2022 秋・定州市期末)把 1 升的水倒入空碗中结果溢出, 说明碗的容量比 1 升大。", "options": [], "subject": "计数", "analysis": "解把 1 升的水倒入空碗中结果溢出, 说明碗的容量比 1 升小, 所以原题说法错误。故答案为: $\\times$ 。"} {"id": "24707", "image": [], "answer": "解 25 -奇数 $=$ 偶数;\n\n$25-1=24$,\n\n24-偶数=偶数。\n\n答: 有 25 个小伙伴要分成甲、乙两个组去植树, 如果甲队人数为奇数, 那么乙队人数为偶数;如果有 1 人请假未到, 这时甲队人数为偶数, 那么乙队人数为偶数。", "solution": "null", "level": "五年级", "question": "“植树节”到了,有 25 个小伙伴要分成甲、乙两个组去植树,如果甲队人数为奇数,那么乙队人数为奇数还是偶数?如果有 1 人请假未到,这时甲队人数为偶数,那么乙队人数呢?", "options": [], "subject": "计数", "analysis": "解 25 -奇数 $=$ 偶数;\n\n$25-1=24$,\n\n24-偶数=偶数。\n\n答: 有 25 个小伙伴要分成甲、乙两个组去植树, 如果甲队人数为奇数, 那么乙队人数为偶数;如果有 1 人请假未到, 这时甲队人数为偶数, 那么乙队人数为偶数。"} {"id": "25387", "image": [], "answer": "解 2.4 平方米 $=0.024$ 平方米\n\n$0.024 \\times 3 \\times 500$\n\n$=0.072 \\times 500$\n\n$=36$ (立方米)\n\n36 立方米 $=36$ 方\n\n$800 \\times 36=28800$ (元)\n\n答: 这些木料一共 36 方, 要花 28800 元。", "solution": "null", "level": "五年级", "question": "(2022 春・定南县期中)家具厂订购 500 根方木,每根方木横截面的面积是 $2.4 \\mathrm{dm}^{2}$, 长是 $3 \\mathrm{~m}$ 。这批木料一共有多少方 $\\left(\\mathrm{m}^{3}\\right)$ ?如果收购价是 800 元/方,这批木料要花多少钱?", "options": [], "subject": "计数", "analysis": "解 2.4 平方米 $=0.024$ 平方米\n\n$0.024 \\times 3 \\times 500$\n\n$=0.072 \\times 500$\n\n$=36$ (立方米)\n\n36 立方米 $=36$ 方\n\n$800 \\times 36=28800$ (元)\n\n答: 这些木料一共 36 方, 要花 28800 元。"} {"id": "24079", "image": ["11972.jpg", "11973.jpg", "11973.jpg"], "answer": "解见详解\n\n【分析】从正面看到的图形是下排三个正方形,上面靠左边一个正方形; 从上面看到的图形是下面三个正方形, 上面靠右一个正方形; 从左面看到的是下排两个正方形, 上面靠右一个正方形。\n\n【详解】如图:\n\n\n\n正面\n\n上面\n\n左面\n\n【点睛】此题主要考查学生对三视图的理解与实际应用。", "solution": "null", "level": "五年级", "question": "分别画出下图从正面、上面、左面看到的形状。\n\n\n正面\n\n上面\n\n左面", "options": [], "subject": "画法几何学", "analysis": "解见详解\n\n【分析】从正面看到的图形是下排三个正方形,上面靠左边一个正方形; 从上面看到的图形是下面三个正方形, 上面靠右一个正方形; 从左面看到的是下排两个正方形, 上面靠右一个正方形。\n\n【详解】如图:\n\n\n\n正面\n\n上面\n\n左面\n\n【点睛】此题主要考查学生对三视图的理解与实际应用。"} {"id": "24080", "image": ["11974.jpg", "11975.jpg", "11976.jpg", "11977.jpg", "11975.jpg", "11976.jpg", "11977.jpg"], "answer": "\n\n【分析】观察图形可知, 从正面看到的是 3 层: 下层 3 个正方形, 中层 1 个正方形居中, 上层 1 个正方形居中; 从上面看到的是 2 层: 下层 2 个正方形, 上层 2 个正方形靠右边; 从左面看到的图形是 3 层: 下层 2 个正方形, 中层 2 个正方形, 上层 1 个正方形靠左边。\n\n【详解】根据分析画图如下:\n\n\n\n【点睛】本题主要考查立体图形三视图的画法。\n\n", "solution": "null", "level": "五年级", "question": "把下面的几何体从正面、上面、左面观察到的图形在方格纸上画出来。\n\n", "options": [], "subject": "画法几何学", "analysis": "\n\n【分析】观察图形可知, 从正面看到的是 3 层: 下层 3 个正方形, 中层 1 个正方形居中, 上层 1 个正方形居中; 从上面看到的是 2 层: 下层 2 个正方形, 上层 2 个正方形靠右边; 从左面看到的图形是 3 层: 下层 2 个正方形, 中层 2 个正方形, 上层 1 个正方形靠左边。\n\n【详解】根据分析画图如下:\n\n\n\n【点睛】本题主要考查立体图形三视图的画法。\n\n"} {"id": "24085", "image": ["11991.jpg", "11992.jpg", "11993.jpg", "11993.jpg"], "answer": "\n\n【详解】整个图形有 7 个正方体组成, 从正面看到的是下层 3 个正方形, 中层和上层中间各有一个正方形; 从左面看到的是下层 3 个正方形, 上层和中层靠左各有一个正方形.", "solution": "null", "level": "五年级", "question": "请画出这个几何体从正面和左面所看到的图形.\n\n\n\n我搭的积木从上面着兄这个形状。\n\n\n上面的数字表示在迻个位䏕上所用的小正:方体的个数。", "options": [], "subject": "画法几何学", "analysis": "\n\n【详解】整个图形有 7 个正方体组成, 从正面看到的是下层 3 个正方形, 中层和上层中间各有一个正方形; 从左面看到的是下层 3 个正方形, 上层和中层靠左各有一个正方形."} {"id": "24102", "image": ["12084.jpg", "12085.jpg", "12085.jpg"], "answer": "解4 种\n【详解】\n\n\n\n4 种(1) (4)号位各一种)", "solution": "null", "level": "五年级", "question": "(2022・海门市)在下图中添上一个同样大的正方体,使得从上面看到的图形不变。想一想,一共有多少种不同的添法?(添上的正方体和原来的图形至少有一个面连在一起)\n\n", "options": [], "subject": "组合几何学", "analysis": "解4 种\n【详解】\n\n\n\n4 种(1) (4)号位各一种)"} {"id": "24124", "image": [], "answer": "解$\\times$\n\n【分析】首先将 4 个正方体摆成一排, 再将最后 1 个正方体放在已摆出的立体图形上方, 一共有 4 种不同的放法。据此判断。\n【详解】用 5 个同样大的正方体摆出的物体,从上面看是 $\\square$ 又 , 一共有 4 种不同的摆法。所以判断错误。\n\n【点睛】本题考查了观察物体,有一定空间观念是解题的关键。", "solution": "null", "level": "五年级", "question": "用 5 个同样大的正方体摆出的物体, 从上面看是\n\n| | | | |\n| :--- | :--- | :--- | :--- |\n\n一共有 3 种不同的摆法。(", "options": [], "subject": "组合几何学", "analysis": "解$\\times$\n\n【分析】首先将 4 个正方体摆成一排, 再将最后 1 个正方体放在已摆出的立体图形上方, 一共有 4 种不同的放法。据此判断。\n【详解】用 5 个同样大的正方体摆出的物体,从上面看是 $\\square$ 又 , 一共有 4 种不同的摆法。所以判断错误。\n\n【点睛】本题考查了观察物体,有一定空间观念是解题的关键。"} {"id": "24464", "image": [], "answer": "解992 元\n\n【分析】粉刷面积 $=$ 长 $\\times$ 宽 + 长 $\\times$ 高 $\\times 2+$ 宽 $\\times$ 高 $\\times 2$ 一门窗面积, 粉刷面积 $\\times$ 每平方米费用 $=$ 总费用, 据此列式解答。\n\n【详解】 $5 \\times 3.5+5 \\times 3 \\times 2+3.5 \\times 3 \\times 2-6.5$\n\n$=17.5+30+21-6.5$\n\n$=62$ (平方米)\n\n$62 \\times 16=992$ (元)\n\n答: 粉刷这个房间需要花费 992 元。\n\n【点睛】关键是掌握并灵活运用长方体表面积公式。", "solution": "null", "level": "五年级", "question": "小宇要粉刷房间的四壁和屋顶。房间的长是 5 米, 宽是 3.5 米, 高是 3 米, 门窗面积是 6.5 平方米。如果每平方米需要花 16 元涂料费, 粉刷这个房间需要花费多少元?", "options": [], "subject": "度量几何学", "analysis": "解992 元\n\n【分析】粉刷面积 $=$ 长 $\\times$ 宽 + 长 $\\times$ 高 $\\times 2+$ 宽 $\\times$ 高 $\\times 2$ 一门窗面积, 粉刷面积 $\\times$ 每平方米费用 $=$ 总费用, 据此列式解答。\n\n【详解】 $5 \\times 3.5+5 \\times 3 \\times 2+3.5 \\times 3 \\times 2-6.5$\n\n$=17.5+30+21-6.5$\n\n$=62$ (平方米)\n\n$62 \\times 16=992$ (元)\n\n答: 粉刷这个房间需要花费 992 元。\n\n【点睛】关键是掌握并灵活运用长方体表面积公式。"} {"id": "24466", "image": ["12496.jpg"], "answer": "解330 立方厘米\n\n【分析】根据长方体的容积公式: $\\mathrm{V}=\\mathrm{abh}$, 据此求出长方体容器的底面积; 再根据求不规则物体的体积的方法, 苹果的体积等于上升的水的体积加上溢出的水的体积。据此进行计算即可。\n\n【详解】 $600 \\div 8=75$ (平方厘米)\n$75 \\times(12-8)+30$\n\n$=75 \\times 4+30$\n\n$=300+30$\n\n$=330$ (毫升)\n\n$=330$ (立方厘米)\n\n答: 苹果的体积是 330 立方厘米。\n\n【点睛】本题考查长方体的体积, 熟记公式是解题的关键。", "solution": "null", "level": "五年级", "question": "妈妈买回来一些苹果,小明准备做实验测量其中一个苹果的体积。他先将 600 毫升水倒入长方体容器里, 量得水深是 8 厘米, 然后他将苹果浸入水中, 发现有水溢出来, 量得溢出的水是 30 毫升。请你算一算这个苹果的体积。\n", "options": [], "subject": "度量几何学", "analysis": "解330 立方厘米\n\n【分析】根据长方体的容积公式: $\\mathrm{V}=\\mathrm{abh}$, 据此求出长方体容器的底面积; 再根据求不规则物体的体积的方法, 苹果的体积等于上升的水的体积加上溢出的水的体积。据此进行计算即可。\n\n【详解】 $600 \\div 8=75$ (平方厘米)\n$75 \\times(12-8)+30$\n\n$=75 \\times 4+30$\n\n$=300+30$\n\n$=330$ (毫升)\n\n$=330$ (立方厘米)\n\n答: 苹果的体积是 330 立方厘米。\n\n【点睛】本题考查长方体的体积, 熟记公式是解题的关键。"} {"id": "24492", "image": [], "answer": "解850 元\n\n【分析】只粉刷教室内的四壁及顶面, 相当于求长方体 4 个侧面和 1 个底面共 5 个面的面积, 根据长方体的表面积公式: $\\mathrm{S}=\\mathrm{a} \\times \\mathrm{b}+\\mathrm{a} \\times \\mathrm{h} \\times 2+\\mathrm{b} \\times \\mathrm{h} \\times 2$, 求出四壁及顶面的面积, 再减去门窗的面积, 即是应粉刷的面积, 再乘每平方米需要的涂料费, 即可求出粉刷这个教室共要多少钱。\n\n【详解】 $(10 \\times 6+10 \\times 4 \\times 2+6 \\times 4 \\times 2-18) \\times 5$\n\n$=(60+80+48-18) \\times 5$\n\n$=170 \\times 5$\n\n$=850$ (元)\n\n答: 粉刷这个教室共要 850 元。\n\n【点睛】解答有关长方体表面积计算的实际问题, 一定要搞清所求的是哪几个面的面积, 再进一步选择合理的计算方法进行计算解答问题。", "solution": "null", "level": "五年级", "question": "粉刷一个教室内的四壁及顶面, 教室长 10 米、宽 6 米、高 4 米, 除去门窗 18 平方米, 如果每平方米需要花 5 元涂料费, 粉刷这个教室共要多少钱?", "options": [], "subject": "度量几何学", "analysis": "解850 元\n\n【分析】只粉刷教室内的四壁及顶面, 相当于求长方体 4 个侧面和 1 个底面共 5 个面的面积, 根据长方体的表面积公式: $\\mathrm{S}=\\mathrm{a} \\times \\mathrm{b}+\\mathrm{a} \\times \\mathrm{h} \\times 2+\\mathrm{b} \\times \\mathrm{h} \\times 2$, 求出四壁及顶面的面积, 再减去门窗的面积, 即是应粉刷的面积, 再乘每平方米需要的涂料费, 即可求出粉刷这个教室共要多少钱。\n\n【详解】 $(10 \\times 6+10 \\times 4 \\times 2+6 \\times 4 \\times 2-18) \\times 5$\n\n$=(60+80+48-18) \\times 5$\n\n$=170 \\times 5$\n\n$=850$ (元)\n\n答: 粉刷这个教室共要 850 元。\n\n【点睛】解答有关长方体表面积计算的实际问题, 一定要搞清所求的是哪几个面的面积, 再进一步选择合理的计算方法进行计算解答问题。"} {"id": "24521", "image": [], "answer": "解 1425 平方厘米\n\n【分析】求准备多少平方厘米的商标纸, 就是求这个长方体 5 个面的面积, 缺少上面, 由此根据长方体的表面积的公式: $S=\\mathrm{a} \\times \\mathrm{b}+\\mathrm{a} \\times \\mathrm{h} \\times 2+\\mathrm{b} \\times \\mathrm{h} \\times 2$, 代入数据即可得解。\n\n【详解】 $15 \\times 15+15 \\times 20 \\times 2+15 \\times 20 \\times 2$\n\n$=225+600+600$\n\n$=1425$ (平方厘米)\n\n答: 至少需要准备 1425 平方厘米的商标纸。\n\n【点睛】解答有关长方体表面积计算的实际问题, 一定要搞清所求的是几个面的面积, 再进一步选择合理的计算方法进行计算解答问题。", "solution": "null", "level": "五年级", "question": "一个无盖的长方体食品盒, 长 15 厘米, 宽 15 厘米, 高 20 厘米, 如果在它的外面全部贴上商标纸, 至少需要准备多少平方厘米的商标纸?", "options": [], "subject": "度量几何学", "analysis": "解 1425 平方厘米\n\n【分析】求准备多少平方厘米的商标纸, 就是求这个长方体 5 个面的面积, 缺少上面, 由此根据长方体的表面积的公式: $S=\\mathrm{a} \\times \\mathrm{b}+\\mathrm{a} \\times \\mathrm{h} \\times 2+\\mathrm{b} \\times \\mathrm{h} \\times 2$, 代入数据即可得解。\n\n【详解】 $15 \\times 15+15 \\times 20 \\times 2+15 \\times 20 \\times 2$\n\n$=225+600+600$\n\n$=1425$ (平方厘米)\n\n答: 至少需要准备 1425 平方厘米的商标纸。\n\n【点睛】解答有关长方体表面积计算的实际问题, 一定要搞清所求的是几个面的面积, 再进一步选择合理的计算方法进行计算解答问题。"} {"id": "24601", "image": [], "answer": "解 $50 \\times 30+50 \\times 2.5 \\times 2+30 \\times 2.5 \\times 2$\n\n$=1500+250+150$\n\n$=1900$ (平方米)\n\n答:一共需要贴 1900 平方米的瓷砖。", "solution": "null", "level": "五年级", "question": "(2022 秋・宁乡市期末)一个游泳池长 $50 m$, 宽 $30 m$, 深 $2.5 m$, 要在它的四周和底面贴上瓷砖,一共需要贴多少平方米的瓷砖?", "options": [], "subject": "度量几何学", "analysis": "解 $50 \\times 30+50 \\times 2.5 \\times 2+30 \\times 2.5 \\times 2$\n\n$=1500+250+150$\n\n$=1900$ (平方米)\n\n答:一共需要贴 1900 平方米的瓷砖。"} {"id": "24646", "image": [], "answer": "解$\\times$\n\n【分析】物体所占空间的大小叫做物体的体积, 计算冰箱的体积需要从外部测量长、宽、高, 容器所能容纳物体的体积叫做它们的容积, 计算冰箱的容积需要从内部测量长、宽、高, 据此解答。\n\n【详解】分析可知, 从外部测量冰箱的长、宽、高比从内部测量冰箱的长、宽、高多一个冰箱的厚度,所以冰箱的体积一定大于冰箱的容积。\n\n故答案为: $\\times$\n\n【点睛】本题主要考查体积和容积大小的比较, 不忽略物体的厚度时,物体的体积一定大于它的容积。", "solution": "null", "level": "五年级", "question": "小明说, 他家的冰箱体积和容积一样大。( )", "options": [], "subject": "度量几何学", "analysis": "解$\\times$\n\n【分析】物体所占空间的大小叫做物体的体积, 计算冰箱的体积需要从外部测量长、宽、高, 容器所能容纳物体的体积叫做它们的容积, 计算冰箱的容积需要从内部测量长、宽、高, 据此解答。\n\n【详解】分析可知, 从外部测量冰箱的长、宽、高比从内部测量冰箱的长、宽、高多一个冰箱的厚度,所以冰箱的体积一定大于冰箱的容积。\n\n故答案为: $\\times$\n\n【点睛】本题主要考查体积和容积大小的比较, 不忽略物体的厚度时,物体的体积一定大于它的容积。"} {"id": "24653", "image": [], "answer": "解13.6 米\n\n【分析】先统一单位, 2.2 米 $=220$ 厘米, 然后根据长方体的棱长和 $=($ 长+宽 + 高 $) \\times 4$, 用 $(220+$ $40+80 \times 4$ 即可求出这个柜台需要角铁的长度, 再转化为米单位。\n\n【详解】 2.2 米 $=220$ 厘米\n\n$(220+40+80) \\times 4$\n\n$=340 \\times 4$\n\n$=1360$ (厘米)\n\n1360 厘米 $=13.6$ 米\n\n答: 这个柜台需要 13.6 米角铁。\n\n【点睛】本题考查了长方体棱长和公式的灵活应用。", "solution": "null", "level": "五年级", "question": "小卖部要做一个长 2.2 米, 宽 40 厘米, 高 80 厘米的玻璃柜台, 现要在柜台各边都安上角铁, 这个柜台需要多少米角铁?", "options": [], "subject": "度量几何学", "analysis": "解13.6 米\n\n【分析】先统一单位, 2.2 米 $=220$ 厘米, 然后根据长方体的棱长和 $=($ 长+宽 + 高 $) \\times 4$, 用 $(220+$ $40+80 \times 4$ 即可求出这个柜台需要角铁的长度, 再转化为米单位。\n\n【详解】 2.2 米 $=220$ 厘米\n\n$(220+40+80) \\times 4$\n\n$=340 \\times 4$\n\n$=1360$ (厘米)\n\n1360 厘米 $=13.6$ 米\n\n答: 这个柜台需要 13.6 米角铁。\n\n【点睛】本题考查了长方体棱长和公式的灵活应用。"} {"id": "24654", "image": [], "answer": "解4 平方米; 20.4 平方米; 8 立方米\n\n【分析】根据题意, 占地面积是水池的底面积, 底面积 $=$ 长 $\\times$ 宽; 抹水泥的面积 $=$ 长 $\\times$ 宽 + 长 $\\times$ 高 $\\times 2+$宽 $\\times$ 高 $\\times 2$; 求装水多少也就是求长方体水池的体积, 根据公式: 长方体的体积 $=$ 长 $\\times$ 宽 $\\times$ 高, 代入数据\n计算出结果即可; 据此解答。\n\n【详解】 $2.5 \\times 1.6=4$ (平方米)\n\n$2.5 \\times 1.6+2.5 \\times 2 \\times 2+1.6 \\times 2 \\times 2$\n\n$=4+10+6.4$\n\n$=20.4$ (平方米)\n\n$2.5 \\times 1.6 \\times 2$\n\n$=4 \\times 2$\n\n$=8$ (立方米)\n\n答: 这个蓄水池的占地面积是 4 平方米, 抹水泥的面积是 20.4 平方米,蓄水池最多能装 8 立方米水。\n\n【点睛】解答有关长方体计算的实际问题, 一定要搞清所求的是什么, 再进一步选择合理的计算方法进行计算解答问题。", "solution": "null", "level": "五年级", "question": "张大爷为了蓄水浇灌果园特意在果园旁边砌了一个长 2.5 米、宽 1.6 米、深 2 米的蓄水池。这个蓄水池的占地面积是多少?如果在蓄水池的底面与四壁抹上水泥, 抹水泥的面积是多少?蓄水池最多能装多少水?", "options": [], "subject": "度量几何学", "analysis": "解4 平方米; 20.4 平方米; 8 立方米\n\n【分析】根据题意, 占地面积是水池的底面积, 底面积 $=$ 长 $\\times$ 宽; 抹水泥的面积 $=$ 长 $\\times$ 宽 + 长 $\\times$ 高 $\\times 2+$宽 $\\times$ 高 $\\times 2$; 求装水多少也就是求长方体水池的体积, 根据公式: 长方体的体积 $=$ 长 $\\times$ 宽 $\\times$ 高, 代入数据\n计算出结果即可; 据此解答。\n\n【详解】 $2.5 \\times 1.6=4$ (平方米)\n\n$2.5 \\times 1.6+2.5 \\times 2 \\times 2+1.6 \\times 2 \\times 2$\n\n$=4+10+6.4$\n\n$=20.4$ (平方米)\n\n$2.5 \\times 1.6 \\times 2$\n\n$=4 \\times 2$\n\n$=8$ (立方米)\n\n答: 这个蓄水池的占地面积是 4 平方米, 抹水泥的面积是 20.4 平方米,蓄水池最多能装 8 立方米水。\n\n【点睛】解答有关长方体计算的实际问题, 一定要搞清所求的是什么, 再进一步选择合理的计算方法进行计算解答问题。"} {"id": "24672", "image": [], "answer": "解$\\times$\n\n【详解】略", "solution": "null", "level": "五年级", "question": "$2 \\mathrm{~m}^{3}$ 和 $200 \\mathrm{dm}^{3}$ 都一样大. ( )", "options": [], "subject": "度量几何学", "analysis": "解$\\times$\n\n【详解】略"} {"id": "24675", "image": [], "answer": "解$\\sqrt{ }$\n\n【分析】计量水、油等液体的体积, 常用容积单位“升”和“毫升”作单位, 用字母表示为: $\\mathrm{L}$ 和 $\\mathrm{mL}$,据此解答即可。\n\n【详解】计量水、油等液体的体积, 常用容积单位, 原题说法正确;\n\n故答案为: V。\n\n【点睛】本题考查了计量液体体积的方法, 一定掌握基础知识。", "solution": "null", "level": "五年级", "question": "计量水、油等液体的体积, 常用容积单位。()", "options": [], "subject": "度量几何学", "analysis": "解$\\sqrt{ }$\n\n【分析】计量水、油等液体的体积, 常用容积单位“升”和“毫升”作单位, 用字母表示为: $\\mathrm{L}$ 和 $\\mathrm{mL}$,据此解答即可。\n\n【详解】计量水、油等液体的体积, 常用容积单位, 原题说法正确;\n\n故答案为: V。\n\n【点睛】本题考查了计量液体体积的方法, 一定掌握基础知识。"} {"id": "24681", "image": ["12583.jpg"], "answer": "解不能\n\n【分析】折成的长方体水槽的长、宽、高分别为 $(16.5-2 \\times 2)$ 分米、 $(8-2 \\times 2)$ 分米、 2 分米, 又因长方体的容积 $=$ 长 $\\times$ 宽 $\\times$ 高, 将数据代入公式即可求出这个水槽的容积, 与 110 升比较即可。\n\n【详解】 $(16.5-2 \\times 2) \\times(8-2 \\times 2) \\times 2$\n$=(16.5-4) \\times(8-4) \\times 2$\n\n$=12.5 \\times 4 \\times 2$\n\n$=100$ (立方分米)\n\n$=100$ (升)\n\n100 升 $<110$ 升\n\n答:这个水槽不能装下 110 升的水。\n\n【点睛】此题主要考查长方体容积的计算方法, 关键是求出长方体的长、宽、高。", "solution": "null", "level": "五年级", "question": "一块长 16.5 分米, 宽 8 分米的铁皮, 切除 4 个边长 2 分米的正方形角做一个无盖的水槽, 能装下 110 升的水吗?\n\n", "options": [], "subject": "度量几何学", "analysis": "解不能\n\n【分析】折成的长方体水槽的长、宽、高分别为 $(16.5-2 \\times 2)$ 分米、 $(8-2 \\times 2)$ 分米、 2 分米, 又因长方体的容积 $=$ 长 $\\times$ 宽 $\\times$ 高, 将数据代入公式即可求出这个水槽的容积, 与 110 升比较即可。\n\n【详解】 $(16.5-2 \\times 2) \\times(8-2 \\times 2) \\times 2$\n$=(16.5-4) \\times(8-4) \\times 2$\n\n$=12.5 \\times 4 \\times 2$\n\n$=100$ (立方分米)\n\n$=100$ (升)\n\n100 升 $<110$ 升\n\n答:这个水槽不能装下 110 升的水。\n\n【点睛】此题主要考查长方体容积的计算方法, 关键是求出长方体的长、宽、高。"} {"id": "24721", "image": [], "answer": "解1.5; 克; $3600 ; \\mathrm{ml}$", "solution": "null", "level": "五年级", "question": "在横线上添上合适的数或单位。\n$1500 \\mathrm{~mL}=$ $\\qquad$ $\\mathrm{L}$ 一个鸡蛋的重量约 50\n$3.6 \\mathrm{dm}^{2}=$ $\\qquad$ $\\mathrm{cm}^{2}$\n一个水杯的容积约 300", "options": [], "subject": "度量几何学", "analysis": "解1.5; 克; $3600 ; \\mathrm{ml}$"} {"id": "25070", "image": [], "answer": "解35 平方米\n\n【分析】一个数只有 1 和它本身两个因数, 这个数叫做质数。根据长方形的周长 $=($ 长十宽 $) \\times 2$, 用 $24 \\div 2$ 即可求出菜地的长与宽的和, 根据质数的定义, 可将 12 拆分成 $5+7$, 据此根据长方形的面积公式求解即可。\n\n【详解】 $24 \\div 2=12$ (米)\n\n$12=5+7$\n\n$5 \\times 7=35$ (平方米)\n\n答:这块菜地的面积是 35 平方米。\n\n【点睛】本题主要考查了质数的意义、长方形的面积公式和周长公式的灵活应用。", "solution": "null", "level": "五年级", "question": "王爷爷有块长方形的菜地, 周长 24 米, 它的长和宽都是质数, 这块菜地的面积是多少平方米?", "options": [], "subject": "解析几何", "analysis": "解35 平方米\n\n【分析】一个数只有 1 和它本身两个因数, 这个数叫做质数。根据长方形的周长 $=($ 长 + 宽 $) \\times 2$, 用\n$24 \\div 2$ 即可求出菜地的长与宽的和, 根据质数的定义, 可将 12 拆分成 $5+7$, 据此根据长方形的面积公式求解即可。\n\n【详解】 $24 \\div 2=12$ (米)\n\n$12=5+7$\n\n$5 \\times 7=35$ (平方米)\n\n答:这块菜地的面积是 35 平方米。\n\n【点睛】本题主要考查了质数的意义、长方形的面积公式和周长公式的灵活应用。"} {"id": "11750", "image": ["2494.jpg", "2495.jpg"], "answer": "(1) $1 ; 2 ; 5 ; 4$(2)(3) 超市", "solution": "null", "level": "五年级", "question": "按要求画一画, 填一填。\n\n\n\n(1) 用数对表示明明家和书店的位置。明明家 \\$ \\qquad \\$ , \\$ \\qquad \\$ ),书店 \\$ \\qquad \\$ \\$ \\qquad \\$ )\n\n(2)幼儿园的位置用数对表示是 $(3,7)$ 请在图中标出来。\n\n(3)公园往南走 $300 \\mathrm{~m}$ ,再往西走 $400 \\mathrm{~m}$ 就到了 \\$ \\qquad \\$ .", "options": [], "subject": "解析几何", "analysis": ""} {"id": "11751", "image": ["2496.jpg", "2497.jpg"], "answer": "答案:(1) A $(2,2), \\mathrm{B}(6,4), \\mathrm{C}(10,1)$;(2) 图见详解。【分析】(1)根据用数对表示点的位置的方法, 第一个数表示列数, 第二个数表示行数, 即可用数对表示出点 A、B、C 的位置。(2) 同理, 即可在图中描出 $\\mathrm{D} 、 \\mathrm{E}$ 两点, 再顺次连结 $\\mathrm{B}, \\mathrm{D}, \\mathrm{E}, \\mathrm{B}$, 进而看出是什么图形。【详解】 (1) 用数对表示 A、B、C 的位置, A $(2,2), B(6,4), C(10,1) 。$(2) 在图中标出点 D $(6,1), \\mathrm{E}(10,4)$, 再顺次连结 B, D, E, B 并围成图形(如下图):顺次连接 $B 、 D 、 C 、 E 、 B$ 围成的是长方形。【点睛】此题主要是考查数对与位置, 在无特殊说明的情况下, 数对中第一个数表示列, 第二个数表示行。", "solution": "null", "level": "五年级", "question": "(1)用数对表示 $A 、 B 、 C$ 的位置。\n\n(2) 在图中标出点 $\\mathrm{D}(6,1), \\mathrm{E}(10,4)$, 再顺次连接 B、D、C、E、B, 围成的是什么图形?\n\n", "options": [], "subject": "解析几何", "analysis": ""} {"id": "12447", "image": ["2651.jpg"], "answer": "答案:(1) 0\n\n(2) $90^{\\circ}$\n\n(3) $\\mathrm{D}$\n\n(4) $270^{\\circ}$", "solution": "null", "level": "五年级", "question": "![](本地图片-4280/.jpg)\n\n(1) 图形 $\\mathrm{B}$ 可以看作图形 $\\mathrm{A}$ 绕点 ( ) 顺时针方向旋转 $90^{\\circ}$ 得到的。\n\n(2) 图形 $\\mathrm{C}$ 可以看作图形 $\\mathrm{B}$ 绕点 0 顺时针方向旋转 ( ) 得到的。\n\n(3) 图形 $B$ 绕点 0 顺时针旋转 $180^{\\circ}$ 到图形 (\n\n", "options": [], "subject": "变换几何", "analysis": "答案:(1) 0\n\n(2) $90^{\\circ}$\n\n(3) $\\mathrm{D}$\n\n(4) $270^{\\circ}$"} {"id": "12449", "image": ["2653.jpg"], "answer": "答案: 略", "solution": "null", "level": "五年级", "question": "(10 分) (1) 画 出三角形 $\\mathrm{ABC}$ 绕 点 $\\mathrm{C}$ 逆时针旋转 $90^{\\circ}$ 后的图形。\n\n(2) 画出长方形 MNDP 绕点 D 顺时针旋转 $180^{\\circ}$ 后的图形\n\n", "options": [], "subject": "变换几何", "analysis": "答案: 略"}